ഇസ്ലാം

/ഇസ്ലാം
വിമർശനം: അലക്സാണ്ട്രിയയിലെ പൗരാണിക പുസ്തകശാലയും ഗ്രന്ഥശേഖരങ്ങളും നശിപ്പിച്ചു എന്നത്, ഇസ്‌ലാം ശാസ്ത്ര പഠനത്തിനും വിജ്ഞാന അഭിവൃദ്ധിക്കും എതിരാണ് എന്നതിന് തെളിവല്ലെ ? മറുപടി: വിജ്ഞാന പ്രചാരണത്തിലും ശാസ്ത്രവിദ്യഭ്യാസ അഭ്യുന്നതിയിലും ഖലീഫ ഉമർ (റ) വഹിച്ച നിസ്തുലമായ സേവനവും ഉൽസുകതയും തെളിവു സഹിതം ചർച്ച ചെയ്യവെ, അബ്ബാസ് മഹ്മൂദ് അൽ അക്കാദ് തന്റെ ‘അബ്‌ക്വരിയ്യതു ഉമർ’ എന്ന ഗ്രന്ഥത്തിൽ (പേജ്: 190- 197) ഈ ആരോപണത്തിന് സവിസ്തരം മറുപടി നൽകുന്നുണ്ട്. മറുപടിയുടെ രത്നചുരുക്കം ഇവിടെ ചേർക്കാം: യുദ്ധ വിജയാവസരത്തിൽ അലക്സാണ്ട്രിയയിലെ മഹാഗ്രന്ഥാലയത്തെ സംബന്ധിച്ച വിവരം സേനാധിപനായ അംറിബ്നു ആസ് (റ), ഖലീഫ ഉമറിന്(റ) എത്തിച്ചപ്പോൾ ഖലീഫ ഇപ്രകാരം കൽപ്പന പുറപ്പെടുവിച്ചു: “താങ്കൾ പറഞ്ഞ ഗ്രന്ഥങ്ങളിലെ ഉള്ളടക്കം ക്വുർആനിനോട് യോജിച്ചതാണെങ്കിൽ, പിന്നെ ആ ഗ്രന്ഥങ്ങളുടെ ആവശ്യമില്ലല്ലൊ. ഇനി ഗ്രന്ഥങ്ങളിലെ ഉള്ളടക്കം ക്വുർആനിനോട് എതിരാണെങ്കിൽ അതിൽ നിന്ന് നാം ധന്യരാണ്. അവ നശിപ്പിച്ച് കളയുക.” അങ്ങനെ പട്ടണത്തിലെ നാലായിരം കായലുകളിൽ മുക്കി നശിപ്പിക്കാൻ ശ്രമിച്ചെങ്കിലും പ്രസ്തുത ഉദ്യമം പൂർത്തീകരിക്കാൻ ആറു മാസത്തോളം എടുത്തു; പുസ്തകങ്ങളുടെ ആധിക്യം കാരണം.! ഈ ഗുരുതരമായ ആരോപണത്തിൽ നിന്നും ഖലീഫ ഉമറിനെ(റ) നിരപരാധിയായി പ്രഖ്യാപിച്ച ചരിത്രകാന്മാരുടെ നിരയിൽ ഭൂരിഭാഗവും യൂറോപ്യരും ഒറിയന്റലിസ്റ്റുകളുമായ പണ്ഡിതന്മാരാണ്. ഇംഗ്ലിഷ് ചരിത്രകാരനായ എഡ്വർഡ് ഗിബ്ബൻ തന്റെ The History Of The Decline And Fall Of The Roman Empire എന്ന ഗ്രന്ഥത്തിൽ എഴുതി: “എന്നെ സംബന്ധിച്ചിടത്തോളം ഈ സംഭവവും അതുമായി ബന്ധപ്പെട്ട എല്ലാ ആരോപണങ്ങളെയും നിഷേധിക്കാനാണ് എനിക്ക് അതിയായി തോന്നുന്നത്. കാരണം ഈ കഥ വളരെയധികം വിചിത്രമാണ്. ഈ കഥ എഴുതി പിടിച്ച ചരിത്രകാരന്മാർ തന്നെ അവയെ സംബന്ധിച്ച് ചകിതരാവണം എന്ന ഉദ്ദേശത്തോടെ തന്നെയാണ് അത് എഴുതിയത്. സംഭവം നടന്ന് 600 നൂറ്റാണ്ട് കഴിഞ്ഞ് ഒരു അപരിചിതനായ വിദേശി ആ സംഭവത്തെ സംബന്ധിച്ച് എഴുതുക എന്നത് തന്നെ സംഭവത്തിന്റെ വിശ്വസനീയതയെ ചോദ്യം ചെയ്യുന്നുണ്ട്. ഈജുപ്തുകാരായ -അതും ക്രിസ്ത്യാനികളായ- ആ കാലഘട്ടത്തിലെ രണ്ട് ചരിത്രകാരന്മാർ ഈ സംഭവത്തെ സംബന്ധിച്ച് മൗനം ദീക്ഷിച്ചു എന്നതും ശ്രദ്ധേയമാണ്. അവയിൽ ഏറ്റവും പൗരാണികൻ Patriarch Eutychius ആണ്. അലക്സാണ്ട്രിയൻ വിജയത്തെ സംബന്ധിച്ച് അദ്ദേഹം വിശദമായി എഴുതിയിട്ടുണ്ട്. എന്നിട്ടും ഇത്തരമൊരു സംഭവത്തെ സംബന്ധിച്ച യാതൊരു പരാമർശവുമില്ല. യുദ്ധങ്ങളിൽ ജൂത ക്രിസ്ത്യാനികളിൽ നിന്ന് യുദ്ധാർജിത സ്വത്തായി ലഭിക്കുന്ന മത ഗ്രന്ഥങ്ങൾ കത്തിച്ചു കളയുന്നത് നിഷിദ്ധമാണെന്നും, ചരിത്രകാരന്മാർ, കവികൾ, വൈദ്യന്മാർ, തത്ത്വചിന്തകർ തുടങ്ങിയവർ എഴുതിയ തനി ഭൗതികമായ ഗ്രന്ഥങ്ങൾ വിശ്വാസികൾക്ക് ഉപകാരപ്രദമായ രൂപത്തിൽ ഉപയോഗപ്പെടുത്തലാണ് വേണ്ടതെന്നും മതവിധി നൽകിയ മുസ്‌ലിം കർമ്മശാസ്ത്ര പണ്ഡിതന്മാരിൽ ശരിയായ വീക്ഷണം വെച്ചുപുലർത്തുന്നവരുടെ അടുക്കൽ ഉമറിലേക്ക് ചേർക്കപ്പെടുന്ന ഈ രൂക്ഷമായ വിധിന്യായം അങ്ങേയറ്റം നിന്ദ്യമായതാണ്. മുഹമ്മദ് (നബിക്ക്) ശേഷം വന്ന ആദ്യ ഖലീഫമാരിലേക്ക് ചേർക്കപ്പെടുന്ന ഇത്തരം ധ്വംസന കഥകൾ ഇതിനേക്കാൾ ജുഗുപ്സാവഹമാണ്. എങ്കിലും ഇത്തരം നിവേദനങ്ങൾ സ്വഹീഹ് (വിശ്വസനീയതിൽ സ്വീകാര്യം) ആയിരുന്നെങ്കിൽ കത്തിക്കാൻ തുനിഞ്ഞ പ്രസ്‌താവിതമായ ഗ്രന്ഥങ്ങൾ അവയുടെ അൽപത കാരണം പെട്ടെന്ന് പെട്ടെന്നു തന്നെ കത്തി തീരുമായിരുന്നു! എന്നു മാത്രമല്ല അവയുടെ ഭയാനകത സീസറിന്റെ കൈകളാൽ ലൈബ്രറികൾ കത്തിയെരിയപ്പെട്ടതിനോളം എത്തില്ല. വിഗ്രഹാരാധകരുടെ കാലഘട്ടത്തിന്റെ ശേഷിപ്പുകൾ ഉൻമൂലനാശം വരുത്താനുള്ള വൈവിധ്യമായ മാധ്യമങ്ങളെ സംബന്ധിച്ച് പൗരാണിക ക്രിസ്ത്യൻ പട നയിച്ച ഗൂഢാലോചനയുടെ വർഗീയ പ്രാകൃതങ്ങളിലേക്കും ആ നിവേദനങ്ങളുടെ ഉള്ളടക്കങ്ങളൊന്നും എത്തില്ല…” അലക്സാണ്ട്രിയയിലും ഈജിപ്തിലുമുള്ള അറബ് ദ്വിഗ് വിജയങ്ങളെ സംബന്ധിച്ച ചരിത്ര ഗവേഷണത്തിൽ വലിയ പങ്കുള്ള ഇംഗ്ലീഷ് ചരിത്രകാരനായ ഡോ. ആൽഫ്രഡ് ബട്ട്ലർ ഖലീഫ ഉമറിലേക്ക് ചേർക്കപ്പെടുന്ന ഈ കഥയുടെ രത്നചുരുക്കം രേഖപ്പെടുത്തിയ ശേഷം പല കാരണങ്ങൾ നിരത്തി അതിനെ ഖണ്ഡിക്കുന്നുണ്ട്. ലൈബ്രറികളുടെ കാര്യത്തിൽ അംറിബ്നു ആസുമായി സംസാരിച്ചു എന്ന് പറയപ്പെടുന്ന ഹനാ ഫിലിപ്പോത്തോസ് അറബികൾ ഈജിപ്ത് വിജയിച്ചടക്കുന്ന സമയത്ത് ജീവിച്ചിരുന്നില്ല. മാത്രമല്ല, ഏഴാം നൂറ്റാണ്ടിലെ ഭൂരിഭാഗം ഗ്രന്ഥങ്ങളും നേർത്ത തോൽ പാളികളിലാണ് എഴുതപ്പെട്ടിരുന്നത്. അവ കത്തിക്കാൻ കഴിയാത്തവയാണ്. ഇനി അങ്ങനെ അല്ലെങ്കിൽ കത്തിക്കാനായി ഖലീഫ കൽപ്പന പുറപ്പെടുവിച്ചിരുന്നെങ്കിൽ തൽക്ഷണം ആ കൽപ്പന നടപ്പാക്കപ്പെടുമായിരുന്നു. പ്രയാസങ്ങൾ സഹിച്ച് കായലുകളിലേക്ക് താങ്ങി കൊണ്ടുപോകുമായിരുന്നില്ല. ഇനി കൊണ്ടുപോയിരുന്നുവെങ്കിൽ വളരെയേറെ വിലക്കുറവിൽ മറ്റുള്ളവർക്ക് അത് വാങ്ങാമായിരുന്നു… അലക്സാണ്ട്രിയൻ വിജയത്തിന് അഞ്ചര നൂറ്റാണ്ടുകൾക്ക് ശേഷം മാത്രമാണ് ഈ കഥ രേഖപ്പെടുത്തപ്പെടുന്നത് എന്നതു തന്നെ കഥക്ക് യാതൊരു കഴമ്പുമില്ല എന്നത് സംശയലേശമന്യെ തെളിയിക്കുന്നു. മാത്രമല്ല എഴുതപ്പെട്ടതാവട്ടെ അവലംബനീയമായ സ്രോതസ്സുകളൊ നിവേദന പരമ്പരകളൊ ഇല്ലാതെയുമാണ് എന്നെല്ലാം ഡോ. ആൽഫ്രഡ് ബട്ട്ലർ കഥയെ നിരൂപണം ചെയ്യുന്നു. ഒറിയന്റലിസ്റ്റായ കാസനോവ ഈ നിവേദനത്തെ കെട്ടുകഥ എന്നാണ് വിളിക്കുന്നത്. വിജയം നടന്ന് ആറ് നൂറ്റാണ്ടു കഴിഞ്ഞാണ് ഈ കഥ ഉടലെടുക്കുന്നത് എന്ന് അദ്ദേഹം പറയുന്നു. മുമ്പു സൂചിപ്പിച്ച കാരണങ്ങളെല്ലാം നിരത്തി സംഭവത്തെ നിഷേധിച്ചതിനു പുറമെ ഒരു കാരണം കൂടി അദ്ദേഹം ഉദ്ധരിക്കുന്നു: “… ഇപ്പറഞ്ഞ കാരണങ്ങൾക്കെല്ലാം പുറമെ കൂടുതൽ ശക്തമായ ഒരു കാരണം കൂടി ഈ കഥയുടെ വ്യർത്ഥതയെ സൂചിപ്പിക്കുന്നതായിട്ടുണ്ട്. യഹ്‌യ അന്നഹ്‌വിയിൽ നിന്നുമുള്ള ഒരു ഉദ്ധരണി, പത്താം നൂറ്റാണ്ടിന്റെ അന്ത്യദശകളിൽ ഇബ്നു നദീം തന്റെ അൽ ഫഹ്റസത്തിൽ ഉദ്ധരിക്കുന്നുണ്ട്. അതായത് യഹ്‌യ അന്നഹ്‌വി ഈജിപ്ത് വിജയിച്ചടക്കുന്ന സന്ദർഭത്തിൽ ജീവിച്ചിരുന്ന വ്യക്തിയാണ്. അംറിബ്നു ആസുമായി വളരെ അടുപ്പമുണ്ടായിരുന്നു അദ്ദേഹത്തിന്. അലക്സാണ്ട്രിയൻ ലൈബ്രറിയെ സംബന്ധിച്ച് ഒന്നും തന്നെ അദ്ദേഹം പ്രസ്‌താവിച്ചിട്ടേയില്ല. അപ്പോൾ ലൈബ്രറി ധ്വംസനം ഇബ്നുൽ കഫ്ത്വിയുടെ ഭാവനയുടെ വ്യുൽപ്പന്നം മാത്രമാണ്. അദ്ദേഹത്തിന്റെ കാലഘട്ടത്തിൽ പ്രചാരത്തിലുണ്ടായിരുന്ന കെട്ടുകഥകളിൽ ഭാവനകളും ചേർത്തുണ്ടാക്കിയ കള്ളകഥ.” ഒറിയന്റലിസ്റ്റ് കാസനോവ തുടർന്നെഴുതി: “ഇബ്നു ഖൽദൂൻ ഇപ്രകാരം പറയുകയുണ്ടായി: അറബികൾ പേർഷ്യ പിടിച്ചടക്കിയപ്പോൾ പേർഷ്യക്കാരുടെ “ശാസ്ത്ര”ങ്ങളടങ്ങുന്ന (അഭിചാര ക്രിയകളും ജ്യോതിഷവും Astrology അടങ്ങുന്ന ) ഗ്രന്ഥങ്ങൾ എന്തു ചെയ്യണമെന്ന് സഅ്ദിബ്നു അബീ വക്വാസ്, ഉമറിന് കത്തെഴുതി ചോദിച്ചു. അവ പുഴയിലെറിയാൻ അദ്ദേഹം കൽപ്പിച്ചു.* ഈ കഥ പേർഷ്യയിൽ നിന്നും കാലാന്തരങ്ങളിൽ അലക്സാണ്ട്രിയയിൽ എത്തി. പലരുടേയും പല ഭാവനകളും ആ കഥയിൽ കലർന്നു മറിഞ്ഞു. പല സർവ്വവിജ്ഞാനകോശങ്ങളിൽ കഥ പല മാറ്റങ്ങളോടെ രേഖപ്പെടുത്തപ്പെട്ടു. ഈജിപ്ത് പിടിച്ചടക്കിയപ്പോൾ അറബികൾ അലക്സാണ്ട്രിയൻ ലൈബ്രറി കത്തിച്ചുവെന്ന് സബ്റഞ്ചലിൽ നിന്ന് ഉദ്ധരിക്കപ്പെട്ടു; ഖലീഫ മുതവക്കിൽ അത് പുതുക്കി പണിതു. 868 ൽ തുർക്കികൾ ഈജിപ്ത് പിടിച്ചടക്കിയപ്പോൾ അഹ്മദിബ്നു ത്വുലൂൻ ഗ്രന്ഥാലയത്തിന് തീ വെച്ചു. എന്നാൽ അഹ്മദിബ്നു ത്വൂലൂൻ ഈജിപ്ത് വിജയിച്ചടക്കിയിട്ടില്ല. അന്നത്തെ ഖലീഫ അദ്ദേഹത്തെ ബഗ്ദാദിനു മേൽ ഗവർണറാക്കുക മാത്രമാണ് ഉണ്ടായത്. അപ്പോൾ ആരോപിത സംഭവവുമായി തുർക്കികൾക്ക് യാതൊരു ബന്ധവുമില്ല… 1877 ൽ കൗണ്ട് ദെ ലണ്ട്ബർഗ് ഇപ്രകാരമെഴുതി: അലക്സാണ്ട്രിയയിലെ ലൈബ്രറി കത്തിച്ചത് നെപ്പോളിയൻ ഒന്നാമനാണെന്ന് ഇംഗ്ലീഷ് പടയാളികൾ ആരോപിക്കുകയുണ്ടായി… (ഖലീഫ ഉമറിലേക്ക് ചേർക്കപ്പെടുന്ന) ഈ കെട്ടുകഥ ക്രിസ്താബ്ദം പതിമൂന്നാം നൂറ്റാണ്ടിൽ മാത്രം ഉടലെടുത്തതാണ്… ഇബ്നുൽ ക്വഫ്ത്വിയാണ് ആദ്യമായി ഈ കെട്ടുകഥ പടച്ചുണ്ടാക്കുന്നത്…” (അബ്‌ക്വരിയ്യതു ഉമർ: പേജ്: 190-197) ക്രിസ്താബ്ദം 641 ൽ (ഹിജ്റ 20) നടന്ന ഈജിപ്ത് വിജയമൊ ക്രിസ്താബ്ദം 642 ൽ നടന്ന പേർഷ്യൻ വിജയമൊ ആയി ബന്ധപ്പെട്ട അതി ഗൗരവകരമായ ഒരു “സംഭവം”, 1172 ൽ ജനിച്ച ജമാലുദ്ദീൻ അൽ ക്വഫത്വിയും, 1162 ൽ ജനിച്ച അബ്ദുല്ലത്വീഫ് അൽ ബഗ്ദാദി, 1226 ൽ ജനിച്ച അബുൽ ഫറജ് ഇബ്നു ഹാറൂൻ അൽ മിൽത്വി എന്നിവരൊക്കെയാണ് ചരിത്രത്തിൽ ആദ്യമായി രേഖപ്പെടുത്തുന്നത് എന്നതും ആ കാലഘട്ടത്തിലേക്ക് ചേർക്കപ്പെടുന്ന ഒരു നിവേദക പരമ്പരയൊ പൗരാണിക സ്രോതസ്സൊ ഇല്ലെന്നതും അബ്ബാസ് മഹ്മൂദ് അൽ അക്കാദ് തന്റെ ഗ്രന്ഥത്തിൽ നിരൂപണം നടത്തുന്നുണ്ട്. ഈ നൂറ്റാണ്ടുകളത്രയും കടന്നുപോയിട്ടും ആധികാരികനായ ഒരു അമുസ്‌ലിം ചരിത്രകാരന്മാർ പോലും അറബികളുടെ ദ്വിഗ് വിജയങ്ങളെ സംബന്ധിച്ച് സ്മരിക്കവെ അത്തരമൊരു സംഭവം തങ്ങളുടെ ഗ്രന്ഥത്തിൽ രേഖപ്പെടുത്തിയിട്ടെ ഇല്ല എന്നതിലേക്കും അദ്ദേഹം വിരൽ ചൂണ്ടുന്നുണ്ട്. ചുരുക്കത്തിൽ, പേർഷ്യക്കാരുടെ ആഭിചാര ക്രിയകളും ജ്യോതിഷവും അടങ്ങുന്ന ഗ്രന്ഥങ്ങൾ പുഴയിൽ നിക്ഷേപിക്കാനൊ ഈജിപ്തിലെ അലക്സാണ്ട്രിയൻ ലൈബ്രറി കത്തിക്കാനൊ ഖലീഫ ഉമർ (റ) കൽപ്പന പുറപ്പെടുവിച്ചതായി വിശ്വസനീയമൊ ആധികാരികമൊ ആയ ഒരു ചരിത്ര രേഖയുമില്ല. …………………………………………………. * 1. ഒരു നിവേദക പരമ്പരയൊ പൗരാണിക സ്രോതസ്സൊ ഇല്ലാതെ തന്റെ കാലഘട്ടത്തിൽ പ്രചാരത്തിലുള്ള ഒരു കഥ -നിരൂപണത്തിനൊ ഗവേഷണത്തിനൊ തുനിയാതെ – ഒഴുക്കൻ മട്ടിൽ ഉദ്ധരിക്കുക മാത്രമാണ് ഇബ്നു ഖൽദൂൻ, മുക്വദ്ദിമയിൽ ചെയ്തിട്ടുള്ളത്. 2. പേർഷ്യക്കാരുടെ “ശാസ്ത്രങ്ങൾ” ഉമർ പുഴയിൽ തള്ളാൻ പറഞ്ഞുവെന്ന് കേട്ട ഉടനെ ഇസ്‌ലാം Science ന് എതിരാണെന്ന് പ്രഖ്യാപിക്കാൻ ഒരു പഴുതു കണ്ടെത്തിയ ആഹ്ളാദ ഭേരി മുഴക്കുകയാണ് നാസ്തികർ. എന്നാൽ പേർഷ്യക്കാരുടെ “ശാസ്ത്രങ്ങൾ” എന്നതുകൊണ്ട് എന്താണ് ഉദ്ദേശിച്ചതെന്ന് മുക്വദ്ദിമയിൽ തന്നെ ഇബ്നു ഖൽദൂൻ വിശദീകരിക്കുന്നത് കാണാത്തതായി ഭാവിക്കുകയാണ് നാസ്തികർ ചെയ്യുന്നത്. ആഭിചാര ക്രിയകളും ക്ഷുദ്രവിദ്യകളും ജ്യോതിഷവും (Astrology) രാശിയും ത്വൽസമാത്തുമൊക്കെയായിരുന്നു ഈ ശാസ്ത്രങ്ങൾ എന്ന് പ്രത്യേകം രേഖപ്പെടുത്തപ്പെട്ടിട്ടുണ്ട്. (മുക്വദ്ദിമതു ഇബ്നു ഖൽദൂൻ: 1/479) അപ്പോൾ, മുക്വദ്ദിമയിൽ ഉദ്ധരിക്കപ്പെട്ട നിവേദനത്തിന് വല്ല അടിത്തറയും ഉണ്ടെന്ന് വാദത്തിന് വേണ്ടി മാത്രം സമ്മതിച്ചുവെന്ന് കരുതുക. എങ്കിൽ പോലും ഖലീഫ ഉമർ (റ) വിജ്ഞാന പ്രചരണത്തിനൊ ശാസ്ത്ര ശാക്തീകരണത്തിനൊ എതിരാണെന്ന് അവയൊന്നും തന്നെ സൂചിപ്പിക്കുന്നില്ല.

വിമർശനം: ഇസ്‌ലാമിലെ പ്രധാനപ്പെട്ട പുണ്യകർമങ്ങളായ ബാങ്ക്, ഇമാമത്ത് എന്നിവ സ്ത്രീകൾക്ക് തീർത്തും നിഷിദ്ധമാക്കുകയും പുരുഷന്മാർക്ക് യഥേഷ്‌ടം അനുവദിച്ചു കൊടുക്കുകയും ചെയ്തതിലൂടെ പുരുഷാധിപത്യത്തിനു കൊടിപിടിക്കുകയാണ് നബി ചെയ്തത്. സ്വന്തം മതത്തിലെ വലിയ പുണ്യകർമങ്ങൾ പോലും സ്ത്രീക്കു നിരുപാധികം നിഷിദ്ധമാക്കിയ ഒരാളെ ദൈവദൂതനായി കണക്കാക്കുന്നതു തന്നെ സ്ത്രീവിരുദ്ധമാണ്.

ഉത്തരം: ഇസ്‌ലാമിലെ പ്രധാനപ്പെട്ട പുണ്യകർമങ്ങളായ ബാങ്കും ഇമാമത്തും (നമസ്കാരത്തിനു നേതൃത്വം നൽകൽ) നിർവഹിക്കുന്നത് സ്ത്രീകൾക്കു നിഷിദ്ധമാക്കപ്പെട്ട കാര്യങ്ങളാണെന്ന അറിവില്ലായ്മയിൽ നിന്നാണ് ഈ വിമർശനം ജന്മമെടുത്തിരിക്കുന്നത്. ഇസ്‌ലാംവിമർശനങ്ങളിൽ പലതും ഉടലെടുക്കുന്നത് തികഞ്ഞ അറിവില്ലായ്മയിൽ നിന്നാണെന്ന് ഇത്തരം വിമർശനങ്ങളെ അവലോകനം ചെയ്താൽ ഏതൊരാൾക്കും നിഷ്പ്രയാസം മനസ്സിലാക്കാവുന്ന വസ്തുതയാണ്. വാസ്തവത്തിൽ ബാങ്കും ഇമാമത്തും നബി(സ്വ) സ്ത്രീകൾക്ക് വിലക്കുകയോ വിരോധിക്കുകയോ ചെയ്തിട്ടുണ്ടോ?. ഇല്ല എന്നതാണ് യാഥാർഥ്യം. വിശുദ്ധ ഖുർആനോ പ്രവാചക വചനങ്ങളോ ബാങ്കോ, ഇമാമത്തോ നിർവഹിക്കുന്നതിൽ നിന്നും സ്ത്രീകളെ വിലക്കുന്നില്ല. പിന്നെ എവിടെ നിന്നാണാവോ വിമർശകർ ഇത്തരം വിവരക്കേടുകൾ ഗവേഷണം ചെയ്തെടുക്കുന്നതെന്ന് മസ്സിലാകുന്നില്ല. മാത്രമല്ല സ്ത്രീകൾക്ക് ബാങ്കില്ല എന്ന നിലപാടിനെ കുറ്റകരമായ കാര്യമായാണ് സ്വഹാബികൾ മനസ്സിലാക്കിയിരുന്നത്. വസ്തുതകൾ നമുക്കൊന്നു വിശകലനം ചെയ്യാം.

سئل ابن عمر هل على النساء أذان؟ فغضب، وقال: أنا أنهى عن ذكر الله؟

സ്ത്രീകൾക്ക് ബാങ്ക് ഉണ്ടോ ? എന്ന് ഇബ്നു ഉമർ (റ) ചോദിക്കപ്പെട്ടു. അപ്പോൾ അദ്ദേഹം കോപിഷ്ടനായി പറഞ്ഞു: (സ്ത്രീകൾക്ക് ബാങ്കില്ല എന്ന് പറഞ്ഞ്) അല്ലാഹുവെ സ്മരിക്കുന്നതിൽ നിന്ന് ഞാൻ അവരെ തടയണൊ ?!. (മുസ്വന്നഫ് ഇബ്നു അബീ ശൈബ: 1/223). സ്ത്രീകൾക്ക് ബാങ്കില്ല എന്ന നിലപാടിനെ, അല്ലാഹുവെ സ്മരിക്കുന്നതിൽ നിന്നും അവരെ തടയുന്ന നിലപാടായാണ് ഇബ്നു ഉമർ (റ) കണ്ടതെന്നു ഈ നിവേദനം വ്യക്തമാക്കുന്നുണ്ട്. മാത്രമല്ല പ്രവാചക പത്നി ആഇശ (റ) തന്നെ ബാങ്കും ഇക്കാമത്തും ഇമാമത്തും നിർവഹിച്ചതായും ഹദീസ് ഗ്രന്ഥങ്ങളിൽ നിവേദനം ചെയ്യപ്പെട്ടിട്ടുണ്ട്.

وعن عائشة أنها كانت تؤذن وتقيم، وتؤم النساء، وتقف وسطهن

ആഇശയിൽ (റ) നിന്നും: അവർ ബാങ്കുവിളിക്കുകയും ഇകാമത്ത് കൊടുക്കുകയും സ്ത്രീകൾക്ക് ഇമാമായി (നേതാവായി) – അവരുടെ നടുവിൽ- നിൽക്കുകയും ചെയ്യുമായിരുന്നു. (സുനനുൽ കുബ്റാ: 1/408, 3:131, മുസ്തദ്‌റക്: ഹാകിം: 1/203-204, മുസ്വന്നഫ് ഇബ്നു അബീ ശൈബ: 1/223, 2/89).

മറ്റു ചില നിവേദനങ്ങൾ കൂടി നമുക്കു പരശോധിക്കാം.

حديث رائطة الحنفية أن عائشة أمت نسوة في المكتوبة فأمتهن بينهن وسطا.

നിർബന്ധ നമസ്കാരങ്ങളിൽ ആഇശ (റ) സ്ത്രീകൾക്ക് ഇമാമായി (നേതൃത്വം നൽകുന്നവരായി) നിന്നു. അവരുടെ നടുവിൽ നിന്നു കൊണ്ടാണ് ആഇശ (റ) നേതൃത്വം നൽകിയിരുന്നത്. (മുസ്വന്നഫ് അബ്ദുർറസാക് : 3/141, ദാറകുത്നി: 1/404, ബൈഹകി: 3/131) സ്ത്രീകൾക്കു വേണ്ടി സ്ത്രീ ഇമാം നിക്കുമ്പോൾ നടുവിലാണ് നിൽക്കേണ്ടതെന്നും, ബാങ്കും ഇക്കാമത്തും നിർവഹിക്കാമെന്നും ഈ നിവേദനങ്ങൾ വ്യക്തമാക്കുന്നുണ്ട്. ആഇശ(റ) മാത്രമല്ല മറ്റു നബിപത്നിമാരും സ്ത്രീകൾക്കു വേണ്ടിയുള്ള നമസ്ക്കാരങ്ങൾക്ക് നേതൃത്വം നൽകിയതായും ഇസ്‌ലാമിക പ്രമാണങ്ങളിൽ കാണാവുന്നതാണ്. അത്തരമൊരു റിപ്പോർട്ട് കാണുക.

رواية حجيرة بنت حصين قالت: ” أمتنا أم سلمة في صلاة العصر قامت بيننا “. رواه عبد الرزاق أيضا، وابن أبي شيبة (2 / 88)،

ഹജീറ ബിൻത് ഹുസ്വൈൻ (റ) പറഞ്ഞു: അസർ നമസ്കാരത്തിന് (പ്രവാചക പത്നി) ഉമ്മു സലമ (റ) ഞങ്ങൾക്ക് ഇമാമായി (നേതൃത്വം നൽകുന്നവരായി) നിന്നു. അവർ ഞങ്ങളുടെ നടുവിൽ നിന്നു കൊണ്ടാണ് നേതൃത്വം നൽകിയത്. (മുസ്വന്നഫ് അബ്ദുർറസാക്, മുസ്വന്നഫ് ഇബ്നു അബീ ശൈബ: 2:88)

باب إِمَامَةِ النِّسَاءِ അഥവാ സ്ത്രീകളുടെ ഇമാമത്ത് (നേതൃത്വം) എന്ന അധ്യായങ്ങൾ ഇസ്‌ലാമിക ഗ്രന്ഥങ്ങളിൽ സ്ഥാനം പിടിച്ചിരിക്കെ, ഇസ്‌ലാം സ്ത്രീകളെ ബാങ്കും ഇമാമത്തും നിർവഹിക്കുന്നതിൽ നിന്നും തടഞ്ഞു എന്ന കല്ലുവെച്ച കള്ളം ഒരു പെൺപക്ഷ ഗവേഷണ പഠനമായി അവതരിപ്പിക്കാൻ അസാമാന്യ തൊലിക്കട്ടി തന്നെ വേണം!. വൈജ്ഞാനികസത്യസന്ധതക്കു കുഴിവെട്ടിയവരല്ലാതെ ഈ കാലഘട്ടത്തിൽ ഇസ്‌ലാം വിമർശനം നടത്തുന്നില്ലെന്നു പറയുന്നത് വെറുതെയല്ലെന്ന് ഇത്തരം വിമർശനങ്ങൾ ഓർമപെടുത്തുന്നുണ്ട്.

നബി പത്നിമാരല്ലാത്ത സ്വഹാബി വനിതകളും സ്ത്രീകൾക്കു വേണ്ടി ഇമാമത്ത് നിർവഹിച്ചിരുന്നു എന്നതും ഹദീസ് ഗ്രന്ഥങ്ങളിൽ ധാരാളം കാണാവുന്നതാണ്. അത്തരം ഒരു റിപ്പോർട്ട് കൂടി നമുക്കു പഠനവിധേയമാക്കാം.

عَنْ أُمِّ وَرَقَةَ بِنْتِ عَبْدِ اللَّهِ بْنِ نَوْفَلٍ الْأَنْصَارِيَّةِ أَنَّ النَّبِيَّ صَلَّى اللَّهُ عَلَيْهِ وَسَلَّمَ … وَكَانَتْ قَدْ قَرَأَتْ الْقُرْآنَ فَاسْتَأْذَنَتْ النَّبِيَّ صَلَّى اللَّهُ عَلَيْهِ وَسَلَّمَ أَنْ تَتَّخِذَ فِي دَارِهَا مُؤَذِّنًا فَأَذِنَ لَهَا قَالَ وَكَانَتْ قَدْ دَبَّرَتْ غُلَامًا لَهَا وَجَارِيَةً فَقَامَا إِلَيْهَا بِاللَّيْلِ فَغَمَّاهَا بِقَطِيفَةٍ لَهَا حَتَّى مَاتَتْ وَذَهَبَا فَأَصْبَحَ عُمَرُ فَقَامَ فِي النَّاسِ فَقَالَ مَنْ كَانَ عِنْدَهُ مِنْ هَذَيْنِ عِلْمٌ أَوْ مَنْ رَآهُمَا فَلْيَجِئْ بِهِمَا فَأَمَرَ بِهِمَا فَصُلِبَا فَكَانَا أَوَّلَ مَصْلُوبٍ بِالْمَدِينَةِ

ഉമ്മു വറക്ക ബിൻത് അബ്ദുല്ലാഹിബ്നു നൗഫൽ അൽ അൻസ്വാരിയ്യ (റ) യിൽ നിന്ന്: ….അവർ ക്വുർആൻ മനപാഠമാക്കിയ ഒരു സ്ത്രീയായിരുന്നു. തന്റെ ഭവനത്തിൽ ബാങ്കുകൊടുക്കുന്ന ഒരാളെ ഏർപ്പാടു ചെയ്യാൻ അവർ നബിയോട് അനുവാദം ചോദിച്ചു. അദ്ദേഹം അവർക്ക് അനുവാദം നൽകി. അവർക്ക് ഭൃത്യരായ ഒരു പയ്യനും പെൺകുട്ടിയും ഉണ്ടായിരുന്നു. അവരെ രണ്ടു പേരെയും കൂട്ടി അവർ രാത്രി നമസ്ക്കരിക്കുമായിരുന്നു…(സുനനു അബൂദാവൂദ്: 591).

വീട്ടിലുള്ള പുരുഷന്മാർക്കുവരെ നമസ്കാരത്തിനു സ്ത്രീകൾ നേതൃത്വം നൽകിയ സംഭവങ്ങൾ ഹദീസ് ഗ്രന്ഥങ്ങൾ പരമ്പരകളോടെ നമുക്കു പറഞ്ഞുതരുന്നുണ്ട്. അത്തരം ഒരു നിവേദനം നമുക്കു കാണാം.

عَنْ أُمِّ وَرَقَةَ بِنْتِ عَبْدِ اللَّهِ بْنِ الْحَارِثِ بِهَذَا الْحَدِيثِ وَالْأَوَّلُ أَتَمُّ قَالَ *وَكَانَ رَسُولُ اللَّهِ صَلَّى اللَّهُ عَلَيْهِ وَسَلَّمَ يَزُورُهَا فِي بَيْتِهَا وَجَعَلَ لَهَا مُؤَذِّنًا يُؤَذِّنُ لَهَا وَأَمَرَهَا أَنْ تَؤُمَّ أَهْلَ دَارِهَا قَالَ عَبْدُ الرَّحْمَنِ فَأَنَا رَأَيْتُ مُؤَذِّنَهَا شَيْخًا كَبِيرًا

പ്രവാചകൻ (സ) ഉമ്മു വറകയുടെ വീട് സന്ദർശിക്കാറുണ്ടായിരുന്നു. അദ്ദേഹം അവർക്ക്, ബാങ്ക് വിളിക്കാൻ ആളെ ഏർപ്പാടു ചെയ്തു കൊടുത്തിരുന്നു. തന്റെ വീട്ടിലുള്ളവർക്ക് നേതൃത്വം നൽകി നമസ്ക്കരിക്കാൻ അവരോട് പ്രവാചകൻ (സ) കൽപ്പിക്കുകയും ചെയ്തു. അബ്ദുർറഹ്മാൻ പറഞ്ഞു: അവരുടെ ബാങ്കുവിളിക്കാരനായ പടു വൃദ്ധനെ ഞാൻ കാണുകയുണ്ടായി. (സുനനു അബൂദാവൂദ്: 592).

“അവരുടെ ബാങ്കുവിളിക്കാരനായ പടു വൃദ്ധനെ ഞാൻ കാണുകയുണ്ടായി.” എന്ന നിവേദനത്തിലെ വാചകങ്ങൾ, വീട്ടിലെ പുരുഷന്മാർക്കു വരെ ഇമാം നിൽക്കുന്നതിന് സ്ത്രീകളെ ഇസ്‌ലാം അനുവദിക്കുന്നുണ്ടെന്നു വ്യക്തമാക്കുന്നതാണ്.

ഇമാം സ്വൻആനി പറഞ്ഞു:

والحديث دليل علـى صحة إمامة المرأة أهل دارها، وإن كان فيهم الرجال، فإنه كان لها مـؤذن وكان شيخًا كما في الرواية والظاهر أنها كانت تؤمه وغلامها وجاريتهـا

വീട്ടിലുള്ളവർക്ക് സ്ത്രീ ഇമാമായി (നേതൃത്വം നൽകി) നമസകരിക്കൽ സ്വീകാര്യമാണെന്നതിന് ഹദീസ് തെളിവാണ്; വീട്ടുകാരിൽ പുരുഷൻ ഉൾപ്പെടുമെങ്കിലും. അവർക്ക് (ഉമ്മു വറക്കക്ക്) ഒരു ബാങ്കുവിളിക്കാരനുണ്ടായിരുന്നു. അയാൾ വൃദ്ധനായ ഒരു പുരുഷനായിരുന്നു എന്നാണ് നിവേദനത്തിൽ വന്നിരിക്കുന്നത്. അവർ ഭൃത്യരായ ആൺകുട്ടിക്കും പെൺകുട്ടിക്കും ഇമാമായി നമസ്ക്കരിച്ചു എന്ന് നിവേദനത്തിന്റെ പ്രത്യക്ഷാർത്ഥം സൂചിപ്പിക്കുന്നു. (സുബുലുസ്സലാം: 2:48)

ശംസുൽ ഹക്ക് അസീമാബാദി പറഞ്ഞു:

ثبت من هذا الحديث أن إمامة النساء وجماعتهن صحيحة ثابتة من أمر رسول الله صلى الله عليه وسلم ، وقد أمت النساء عائشة رضي الله عنها وأم سلمة رضي الله عنها في الفرض والتراويح قال الحافظ في تلخيص الحبير : حديث عائشة أنها أمت نساء فقامت وسطهن رواه عبد الرزاق… وظهر من هذه الأحاديث أن المرأة إذا تؤم النساء تقوم وسطهن معهن ولا تقدمهن . قال في السبل : والحديث دليل على صحة إمامة المرأة أهل دارها وإن كان فيهم الرجل فإنه كان لها مؤذنا وكان شيخا كما في الرواية ، والظاهر أنها كانت تؤمه وغلامها وجاريتها ، وذهب إلى صحة ذلك أبو ثور والمزني والطبري…

സ്ത്രീകളുടെ ഇമാമത്തും ജമാഅത്തും സ്വീകാര്യമാണെന്നും പ്രവാചകന്റെ (സ) കൽപ്പനയാൽ അത് സ്ഥാപിതമാണെന്നുമാണ് ഹദീസിൽ നിന്നും സ്ഥിരപ്പെടുന്നത്. ആഇശയും(റ) ഉമ്മു സലമയും(റ) സ്ത്രീകൾക്ക് നിർബന്ധ നമസ്ക്കാരത്തിലും തറാവീഹിലും നേതൃത്വം നൽകിയിട്ടുണ്ട്.

സ്ത്രീകൾ ഇമാം നിൽക്കുകയാണെങ്കിൽ നടുവിലാണ് നിൽക്കേണ്ടത് എന്നും മുന്നിൽ നീങ്ങി നിൽക്കുകയല്ല വേണ്ടത് എന്നും ഹദീസുകളിൽ നിന്നും വ്യക്തമാവുന്നു. പുരുഷന്മാർ ഉൾപ്പെടെ വീട്ടിലുള്ളവർക്ക് സ്ത്രീ ഇമാമായി (നേതൃത്വം നൽകി) നമസകരിക്കൽ സ്വീകാര്യമാണെന്നതിന് ഹദീസ് തെളിവാണ് എന്ന് സുബുലുസ്സലാമിൽ പറയപ്പെട്ടിരിക്കുന്നു… അബു സൗർ, മുസ്നി, ത്വബ്‌രി തുടങ്ങിയവർ ഈ അഭിപ്രായത്തെ ശരിവെക്കുന്നു… (ഔനുൽ മഅ്ബൂദ്: 2:225)

കാര്യങ്ങൾ വളരെ ലളിതമാണ്, വ്യക്തമാണ്, സ്ത്രീകൾക്ക് ഇസ്‌ലാം ബാങ്കും ഇമാമത്തും വിരോധിചിട്ടില്ല, വിലക്കിയിട്ടില്ല. സ്ത്രീകൾക്ക് സ്ത്രീ ഇമാം നിൽക്കുന്നത് മാത്രമല്ല, വീട്ടിലെ പുരുഷന്മാർക്ക് പോലും സ്ത്രീക്ക് ഇമാം നിൽക്കാൻ ഇസ്‌ലാം അനുവാദം നൽകിയിട്ടുണ്ട്. ഇസ്‌ലാമിക പ്രമാണങ്ങൾ ഇത്രമേൽ ചർച്ച ചെയ്ത ഒരു വിഷയത്തിൽ പോലും, അതെല്ലാം മറച്ചുവെച്ചും ദുർവ്യാഖ്യാനിച്ചും ഇത്തരം കളവുകൾ പ്രചരിപ്പിക്കുവാൻ ഇസ്‌ലാംവിമർശകർക്ക് ഒരു തരിമ്പ് പോലും ലജ്ജയില്ലാതായത് അവരുടെ ചർമസൗഭാഗ്യത്തിന്റെ അപാരതയല്ലാതെ മറ്റെന്താണ് കുറിക്കുന്നത്.

സാമൂഹ്യബാധ്യതയുടെ ഭാരം ഇസ്‌ലാം സ്ത്രീയുടെ ചുമലിൽ കെട്ടിവെച്ചില്ല.

എന്നാൽ സാമൂഹ്യബാധ്യത എന്ന നിലയിൽ പുരുഷന്മാരടക്കമുള്ള മൊത്തം സമൂഹത്തിനുവേണ്ടി ബാങ്കും ഇമാമത്തും നിർവഹിക്കേണ്ട ബാധ്യതയിൽ നിന്നും ഇസ്‌ലാം സ്ത്രീയെ മാറ്റി നിർത്തിയിട്ടുണ്ട്. ആ രംഗത്ത് ഇസ്‌ലാം ബാധ്യതയുടെ ഭാരം പുരുഷനെയാണ് ഏൽപ്പിച്ചിരിക്കുന്നത്. അതു പക്ഷെ സ്ത്രീകൾക്ക് ഏതെങ്കിലും തരത്തിലുള്ള ആത്മീയ അർഹതകുറവുണ്ടെന്നു കണകാക്കികൊണ്ടുള്ള ഒരു നിലപാടല്ല. മറിച്ച് പുരുഷ പ്രകൃതിയാണ് ആ ബാധ്യതയുടെ ഭാരം ഏറ്റെടുക്കാൻ ഏറ്റവും അനുയോജ്യമെന്നതു കൊണ്ടാണ്. ഇസ്‌ലാമിന്റെ ഈ വിഷയകമായുള്ള സമീപനത്തെ യുക്തിപരമായി സമീപിക്കുന്ന ഏതൊരാൾക്കും അതിലെ ന്യായം ഉൾകൊള്ളാൻ സാധിക്കുന്നതാണ്. നൈതികതയുടെ ഒരു പ്രശ്നവും ആ രംഗത്ത് കണ്ടെത്താൻ നിഷ്പക്ഷമായ ഒരു വിശകലനത്തിനും സാധ്യമല്ല തന്നെ. ഓരോ കാര്യങ്ങളും നമുക്ക് യുക്തിപരമായി വിശകലം ചെയ്യാം.

1, സ്വഫ്ഫിന്റെ ഘടനയും പെണ്ണിമാമത്തും:

ജമാഅത്ത് നമസ്കാരത്തിലെ (സംഘ നമസ്കാരം) സ്വഫ്‌ഫിന്റെ (അണി) ഘടനക്ക് ഇസ്‌ലാം പുണ്യം നിശ്ചയിച്ച ഒരു രീതിയുണ്ട്. സ്വഫ്ഫിലെ ആദ്യ ഭാഗമാണ് പുരുഷന്മാർക്ക് ഏറ്റവും പുണ്യകരം. അതിനു ശേഷം ഉള്ള സ്വഫ്ഫുകൾ പിറകിലേക്ക് പോകുന്തോറും പുണ്യം കുറഞ്ഞു വരുന്നു. സ്ത്രീകൾക്കാവട്ടെ അണിയുടെ അവസാനത്തിൽ നിന്നാണ് പുണ്യം തുടങ്ങുന്നത്. അത് മുന്നിലേക്ക് പോകുന്തോറും പുണ്യം കുറഞ്ഞു വരുന്നു. ഇതാണ് സ്വഫ്ഫിൽ ഇസ്‌ലാം നിശ്ചയിച്ച പുണ്യത്തിന്റെ ഘടന. അതു പ്രകാരം സ്ത്രീ, പുരുഷന്മാർ അടക്കമുള്ള സംഘനമസ്കാരത്തിന് നേതൃത്വം നൽകുമ്പോൾ സ്വാഭാവികമായും അവളുടെ തൊട്ട് പിറകിൽ നിൽക്കേണ്ടത് പുരുഷന്മാരാണ്. നമസ്കാരമെന്നത് വിവിധ ചലനങ്ങൾ ഉൾകൊള്ളുന്ന ഒരു പ്രാർത്ഥനയാണ്. സ്ത്രീ നിന്നും വളഞ്ഞും ഇരുന്നും ഇളകിയും കുമ്പിട്ടുകിടന്നും നമസ്കാരത്തിന് നേതൃത്വം നൽകുമ്പോൾ അവൾക്കു പിറകിൽ നിൽക്കുന്ന പുരുഷന്മാർക്ക് അതു ജൈവികമായ ചില പ്രശ്നങ്ങൾ സൃഷിച്ചേക്കാം. കാരണം അത്തരം സന്ദർഭങ്ങളിൽ സ്ത്രീയുടെ നിമ്‌നോന്നതികള്‍ വെളിവാക്കപ്പെടാനുള്ള സാധ്യത കൂടുതലാണ്. പിറകിൽ നിൽക്കുന്ന പുരുഷന്മാരിൽ ചിലർക്കെങ്കിലും അതുമൂലം ആത്മസാന്നിധ്യം നഷ്ട്ടപെടാൻ ഇടയുണ്ട്. മാത്രമല്ല സ്ത്രീകൾക്കും അത്തരം ഒരു സന്ദർഭത്തിൽ മനസാന്നിധ്യത്തോടെ നമസ്കാരത്തിനു നേതൃത്വം നൽകാൻ ബുദ്ധിമുട്ടുണ്ടാകുമെന്നത് ആർക്കും നനസ്സിലാക്കാവുന്ന വസ്തുതയാണ്. സ്ത്രീ-പുരുഷ മനഃശാസ്ത്രവും ലൈംഗിക ശാസ്ത്രവും പഠിച്ച ഏതൊരാൾക്കും ഇവിടെ യുക്തിപരമായി സമർപ്പിക്കാവുന്ന നിർദേശം, സാമൂഹ്യ ബാധ്യതയുള്ള സംഘനമസ്കാരത്തിനു സ്ത്രീയേക്കാൾ നല്ലത് പുരുഷനെ തിരഞ്ഞെടുക്കുന്നതാണെന്നായിരിക്കും. കേവലം യുക്തിപരമായി ചിന്തിച്ചാൽ പോലും സാമൂഹ്യ ബാധ്യതയായ സംഘനമസ്കാരത്തിന് നേതൃത്വം കൊടുക്കാൻ ഏറ്റവും അനുയോജ്യമായ പ്രകൃതിഘടന സ്ത്രീയേക്കാൾ പുരുഷനിലാണ് കാണാനാകുന്നത്. അപ്പോൾ മതം ആ നിലപാട് സ്വീകരിക്കുമ്പോൾ അതിലൊരു പെൺവിരുദ്ധതയും കാണേണ്ടതില്ലെന്നർത്ഥം.

2, സ്ത്രീയുടെ സ്വകാര്യ സമയങ്ങളും ഇമാമത്തും:

സ്ത്രീകളെ സംബന്ധിച്ച് അവരുടെ സ്വകാര്യ സമയങ്ങളിൽ പെട്ടതാണ് ആർത്തവ സമയം. അതു മറ്റുള്ളവർ അറിയുന്നത് പൊതുവേ സ്ത്രീകൾ ഇഷ്ട്ടപെടാറില്ല. അതെല്ലാം ജനമധ്യത്തിൽ കൊട്ടിഘോഷിക്കുന്നതിൽ അഭിരമിക്കുന്ന പിടിവിട്ട പരിഷ്കരണവാദികളായ സ്ത്രീകളെ ഒഴിച്ചു നിർത്തിയാൽ, ആത്മാഭിമാനമുള്ള പല സ്ത്രീകളും അത്തരം സമയങ്ങൾ സ്വകാര്യമാക്കി വെക്കാനാണ് ഇഷ്ട്ടപ്പെടുക. ഒരു സ്ത്രീക്ക് സാമൂഹ്യബാധ്യത എന്ന നിലയിൽ പുരുഷന്മാരടക്കമുള്ള മൊത്തം സമൂഹത്തിനുവേണ്ടി ബാങ്കും ഇമാമത്തും നിർവഹിക്കേണ്ട ഉത്തരവാദിത്തം ഏൽപ്പിക്കപ്പെട്ടാൽ ഫലത്തിൽ സംഭവിക്കുന്നത് അവളുടെ അത്തരം സ്വകാര്യ സമയങ്ങളെ പരസ്യമാക്കാൻ നിർബന്ധിക്കലായിരിക്കും. കാരണം ആർത്തവ സമയങ്ങളിൽ സ്ത്രീക്ക് നമസ്കാരമടക്കമുള്ള പല ആരാധനകളും നിർവഹിക്കുവാൻ പാടില്ലാത്തതാണ്. അപ്പോൾ മാസത്തിൽ ചുരുങ്ങിയത് ഏഴ് ദിവസങ്ങളെങ്കിലും അവൾക്ക് സമൂഹത്തിന് ഇമാം നിൽക്കാൻ സാധ്യമല്ലാതെ വരും. നിരന്തരമായി മാസത്തിലെ പ്രത്യേക ദിവസങ്ങളിൽ ഇമാം ഇല്ലാതെ വരുമ്പോൾ സമൂഹം തിരിച്ചറിയും ഈ ദിനങ്ങളാണ് നമ്മുടെ ഇമാമിന്റെ ആർത്തവ ദിനങ്ങളെന്ന്. മാത്രമല്ല ആർത്തവചക്രം കൃത്യമല്ലെങ്കിൽ ഇമാമിന്റെ ആർത്തവ പ്രശ്നവും സമൂഹമറിയും. ഇനി ദീർഘിച്ചതോ ചുരുങ്ങിയതോ ആയ ആർത്തവചക്രമുള്ള സ്ത്രീയാണ് ഇമാം എങ്കിൽ അതും പൊതുസമൂഹത്തിന്റെ ശ്രദ്ധയിൽ പെടും. ഇതെല്ലാം തന്നെ അഭിമാനബോധമുള്ള സ്ത്രീകൾക്ക് വലിയ പ്രയാസം സൃഷ്ടിക്കുന്ന കാര്യങ്ങളാണെന്ന് ആർക്കും പറഞ്ഞുകൊടുക്കേണ്ടതില്ലല്ലോ. ഇത്തരം സ്വകാര്യ സമയങ്ങൾ സമൂഹത്തിൽ കൊട്ടിഘോഷിക്കുന്നതിൽ ഹരം കൊള്ളുന്ന പരിഷ്കാരിണികൾ ഇതിനെല്ലാം അപവാദമാണെങ്കിലും, മതത്തിനു പക്ഷെ മാന്യമഹിളകളെ കണ്ടല്ലേ ഒരു നിയമം ആവിഷ്കരിക്കാനാവുക. പരിഷ്കാരിണികൾ തൽക്കാലം ക്ഷമിക്കുകയല്ലാതെ മറ്റു പോംവഴികളില്ല. ഇതെല്ലാം മതം സ്വീകരിച്ച നിലപാടുകളിൽ ന്യായങ്ങൾ കണ്ടെത്താനുള്ള കേവല യുക്തിചിന്താപരിശ്രമം എന്നതിനപ്പുറം, ഇതാണ് മതത്തിനു പറയാനുള്ള ന്യായവാദങ്ങൾ എന്നു വെറുതെ തെറ്റിദ്ധരിച്ചിടരുത് എന്ന് ഇസ്‌ലാംവിമർശകരെ പ്രത്യേകം ഓർമപ്പെടുത്തുന്നു.

3, സ്ത്രീയുടെ അബലതകളെ പരിഗണിക്കാത്ത നിലപാടാണ് പെണ്ണിമാമത്ത്:

പുരുഷനെ അപേക്ഷിച്ച് സ്ത്രീകൾക്ക് പ്രകൃതിപരമായി തന്നെ ധാരാളം അബലതകൾ ഉണ്ടെന്നത് ഒരു വസ്തുതയാണ്. ഗർഭധാരണം, പ്രസവം, പ്രസവാനന്തരമുള്ള ദീർഘകാലയളവിലെ പ്രശ്നങ്ങൾ, മുലയൂട്ടൽ, ആർത്തവം, ആർത്തവ വിരാമം തുടങ്ങിയ പ്രശ്നങ്ങളും കൂടാതെ ഈ കാലയളവിൽ സ്ത്രീ അനുഭവിക്കുന്ന ശാരീരികവും മാനസീകവും ഹോർമോൺ സംബന്ധവുമായ മറ്റൊരുപാട് പ്രശ്നങ്ങളും വേറെയുമുണ്ട്. അത്തരം അവസ്ഥകളിൽ, സാമൂഹ്യബാധ്യത എന്ന നിലയിൽ പുരുഷന്മാരടക്കമുള്ള മൊത്തം സമൂഹത്തിനുവേണ്ടി ബാങ്കും ഇമാമത്തും നിർവഹിക്കേണ്ട ഉത്തരവാദിത്തം സ്ത്രീയെ ഏൽപ്പിക്കുന്നത് അവളോട് ചെയ്യുന്ന വലിയ ക്രൂരതയായിരിക്കും. കാരണം ഒരു ദിവസം പുലരി മുതൽ രാത്രി വരെ സമയ ബന്ധിതമായി നിർവഹിക്കാൻ ബാധ്യതപെട്ട ഒരു ഉത്തരവാദിത്തം ഈ പ്രകൃതിപരമായ പ്രശ്നങ്ങൾ ഒന്നുമില്ലാത്ത പുരുഷൻ ഉണ്ടായിരിക്കെ സ്ത്രീയെ പിടിച്ച് അതേൽപ്പിക്കുന്നത് കടുത്ത അപരാധമായിരിക്കുമെന്നു യുക്തിക്കു സ്വീകാര്യമായ ഒരു നിലപാടാണല്ലോ. ഒരുപാട് പ്രകൃതിപരമായ അബലതകൾ ഉള്ള ഒരു വിഭാഗവും അത്തരം പ്രശ്നങ്ങൾ ഒന്നുമില്ലാത്ത മറ്റൊരു വിഭാഗവും മുന്നിലിരിക്കെ ഇത്തരം സാമൂഹിക ഉത്തരവാദിത്തം ഏതു വിഭാഗത്തെയാണ് ഏൽപ്പിക്കേണ്ടത്?. യുക്തിപരമായി ചിന്തിക്കുന്ന ഏതൊരാൾക്കും പറയാനുള്ള മറുപടിയെന്താണോ അതു മാത്രമല്ലേ ഇസ്‌ലാം ഇവിടെ സ്വീകരിച്ചിരിക്കുന്ന നിലപാട്. അപ്പോൾ സ്ത്രീക്ക് ബാങ്കും ഇമാമത്തും ഇസ്‌ലാം വിലക്കി എന്ന വിമർശനം പ്രമാണവിരുദ്ധവും, സാമൂഹ്യബാധ്യതയായ ബാങ്കും ഇമാമത്തും ഇസ്‌ലാം സ്ത്രീകളെ ഏൽപ്പിച്ചില്ല എന്ന വിമർശനം യുക്തിവിരുദ്ധവുമാകുന്നു. എങ്ങനെ നോക്കിയാലും ഇസ്‌ലാംവിമർശനമെന്ന പരിപ്പ് ബുദ്ധിയുള്ളവർക്കിടയിൽ വേവില്ലെന്ന് മനസ്സിലാക്കി ആ കലം അടുപ്പിൽ നിന്നും ഇറക്കിവെക്കുന്നതാണ് ഇസ്‌ലാംവിമർശകർക്കെല്ലാം നല്ലത്. കലത്തിന് തുള വീഴുമെന്നല്ലാതെ പരിപ്പ് വേവില്ല കൂട്ടരേ. കാരണം ഇതു ദൈവിക മതമാണ്.

വിമർശനം:

In Islam, sex with animals and sex with dead bodies is halal.

Book: Sunan Abu Dawood (Ifa), Chapter: 33 / Provision of Punishment, Hadith Number: 4406

4406. Narrated from Ahmad Ibn Yunus (R): Ibn Abbas (R). He said: There is no punishment for having intercourse with an animal.

There are two opinions if a woman inserts the penis of an animal (into her vagina), and if she inserts a separate penis; The most accurate is that the woman’s genitals should be washed.

MuslimSahih Muslim – Book of Menstruation – hadith # 525 – Commentary

Comment: The sex or separate male genitalia of any animal of Momina can be inserted into its female genitalia.

According to some Sunni Islamic scholars, sexual intercourse with an animal does not invalidate fasting or Hajj.

(വിമർശകരുടെ തന്നെ (ഗൂഗിൾ) പരിഭാഷ:)

ഇസ്‌ലാമിൽ മൃഗങ്ങളുമായുള്ള ലൈംഗികതയും മൃതദേഹവുമായുള്ള ലൈംഗികതയും ഹലാലാണ്.

പുസ്തകം: സുനൻ അബു ദാവൂദ് അധ്യായം: 33 / ശിക്ഷയുടെ വ്യവസ്ഥ, ഹദീസ് നമ്പർ: 4406 4406.

അഹ്മദ് ഇബ്നു യൂനുസ് (റ): ഇബ്നു അബ്ബാസ് (റ) ൽ നിന്ന് നിവേദനം. അദ്ദേഹം പറഞ്ഞു: മൃഗവുമായി ഇണചേരുന്നതിന് ശിക്ഷയില്ല. ഒരു സ്ത്രീ ഒരു മൃഗത്തിന്റെ ലിംഗം (അവളുടെ യോനിയിൽ) പ്രവേശിപ്പിക്കുകയാണെങ്കിൽ, അവൾ ഒരു ച്ഛേദിക്കപ്പെട്ട ലിംഗം പ്രവേശിപ്പിക്കുകയൊ ആണെങ്കിൽ അക്കാര്യത്തിൽ രണ്ട് അഭിപ്രായങ്ങളുണ്ട്; സ്ത്രീയുടെ ജനനേന്ദ്രിയങ്ങൾ കഴുകണം എന്നതാണ് ഏറ്റവും കൃത്യമായത്.

സഹിഹ് മുസ്ലിം – ഹദീസ് # 525 – വ്യാഖ്യാനം അഭിപ്രായം: ഏതെങ്കിലും മൃഗത്തിന്റെ ലിംഗഭേദം അല്ലെങ്കിൽ പ്രത്യേക പുരുഷ ജനനേന്ദ്രിയം അതിന്റെ സ്ത്രീ ജനനേന്ദ്രിയത്തിൽ ചേർക്കാവുന്നതാണ്.

ചില സുന്നി ഇസ്ലാമിക പണ്ഡിതരുടെ അഭിപ്രായത്തിൽ, മൃഗവുമായുള്ള ലൈംഗികബന്ധം നോമ്പിനെയോ ഹജ്ജിനെയോ അസാധുവാക്കില്ല.

മറുപടി:

1. ഇസ്‌ലാമിലെ പ്രമാണങ്ങൾ ക്വുർആനും സ്വഹീഹായ ഹദീസുകളുമാണ്. അല്ലാതെ പണ്ഡിതാഭിപ്രായങ്ങളല്ല. പണ്ഡിത വ്യാഖ്യാനങ്ങൾ പ്രമാണങ്ങളെ മനസ്സിലാക്കി തരുന്ന ഗൈഡൻസുകളാണ്. സുനനു അബൂദാവൂദ് അടക്കം ഹദീസ് ഗ്രന്ഥങ്ങളിൽ ഹദീസുകൾക്ക് പുറമെ ഫിക്ഹ് (കർമ്മശാസ്ത്രമുണ്ട്), ക്വുർആൻ വ്യാഖ്യാനമുണ്ട്, പണ്ഡിതാഭിപ്രായങ്ങളുണ്ട്, ഭാഷാ ചർച്ചകൾ… എന്നിങ്ങനെ പലതുമുണ്ട്. അങ്ങനെ പല വിവരങ്ങളും ഉൾകൊള്ളുന്ന ഒരു ഗ്രന്ഥമാണത്. അതിലെ ഹദീസുകളാണ് മുസ്‌ലിംകൾ പ്രമാണമായി കാണുന്നത്.

( https://www.snehasamvadam.org/ഇസ്‌ലാമിലെ-രണ്ട്-അടിസ്ഥാ/ )

പ്രവാചകന്റെ(സ) വാക്കുകളുടേയും പ്രവർത്തനങ്ങളുടേയും ക്രോഡീകരണമാണ് ഹദീസ്. (നുസ്ഹത്തുന്നദ്ർ: 1:36, മുഖദ്ദിമ ഫീ ഉസൂലുൽ ഹദീസ്: 1:33, അൽഫദ്‌ലുൽ മുബീൻ അലാ അക്ദി ജൗഹരി സ്സമീൻ: 61)

ഇവിടെ, ഇസ്‌ലാം മൃഗരതിക്ക് അനുവാദം നൽകുന്നു എന്നതിന് തെളിവായി വിമർശകർ എടുത്തുദ്ധരിച്ചിരിക്കുന്നത് ക്വുർആനൊ, സ്വഹീഹായ ഹദീസുകളോ അല്ല. പണ്ഡിതാഭിപ്രായങ്ങളാണ്. ഇത്തരം പണ്ഡിതാഭിപ്രായങ്ങൾ ഇസ്‌ലാമിൽ പ്രമാണങ്ങളല്ല. അവ ഫിക്ഹ് (കർമ്മശാസ്ത്രം) ആകുന്നു. ഫിക്ഹ് എന്നത് ഒരു പഠനശാഖയാണ് എന്ന് ഏവർക്കുമറിയാം. അതിൽ പ്രമാണങ്ങളോട് യോജിക്കുന്നവ മുസ്‌ലിംകൾ സ്വീകരിക്കുകയും പ്രമാണങ്ങളോട് യോജിക്കാത്തവ തിരസ്കരിക്കുകയും ചെയ്യും. ഹദീസുകൾക്ക് പുറമെ, ഇത്തരത്തിലുള്ള ഫിക്‌ഹും ഹദീസ് ഗ്രന്ഥങ്ങളിലും അവക്ക് പണ്ഡിതന്മാർ രചിച്ച വിശദീകരണങ്ങളിലുമുണ്ട്. ആ ഫിക്ഹുകൾ മുസ്‌ലിംകളുടെ അടുക്കലൊ ഇസ്‌ലാമിലോ അലംഘനീയമായ പ്രമാണങ്ങൾ അല്ല. (ഉലൂമുൽ ഹദീസ്: ഇബ്നു സ്വലാഹ്: 22, 23)

2. ഇസ്‌ലാമിക പ്രമാണങ്ങൾ മൃഗരതിയെ എങ്ങനെ കാണുന്നു എന്ന് തിരിച്ചറിയാൻ ക്വുർആനിലൂടെയും സ്വഹീഹായ ഹദീസുകളിലൂടെയും ഒന്ന് കണ്ണോടിച്ചാൽ പോരെ ?! ഫിക്ഹിന്റെ അന്തരാളങ്ങളിൽ ഊളിയിടണമെന്നുണ്ടോ ?!!

ക്വുർആൻ പറയുന്നു: “തങ്ങളുടെ ലൈംഗികവിശുദ്ധി കാത്തുസൂക്ഷിക്കുന്നവരുമാണ്. തങ്ങളുടെ ഭാര്യമാരുമായോ, തങ്ങളുടെ അധീനത്തിലുള്ള അടിമസ്ത്രീകളുമായോ ഉള്ള ബന്ധം ഒഴികെ. അപ്പോള്‍ അവര്‍ ആക്ഷേപാര്‍ഹരല്ല. എന്നാല്‍ അതിന്നപ്പുറം ആരെങ്കിലും ആഗ്രഹിക്കുന്ന പക്ഷം അവര്‍ തന്നെയാണ് അതിക്രമകാരികള്‍.” (ക്വുർആൻ: 23: 5-7)

ക്വുർആനിൽ ഇത്ര വ്യക്തമായി പറഞ്ഞിരിക്കെ പിന്നെ മൃഗരതിയെ ഇസ്‌ലാമിന്റെ പേരിൽ അവതരിപ്പിക്കുന്നതിൽ എന്ത് അർത്ഥമാണുള്ളത് ?!

നബി (സ) പറഞ്ഞു: لعن اللهُ مَن وقع على بهيمةٍ لعن اللهُ من عمِل عملَ قومِ لوطٍ “മൃഗങ്ങളുമായി രതിയിൽ ഏർപ്പെട്ടവനെ ദൈവം ശപിച്ചിരിക്കുന്നു. ലൂത്തിന്റെ ജനതയുടെ പ്രവർത്തനത്തിൽ (സ്വവർഗരതി) ഏർപ്പെട്ടവനെ അല്ലാഹു ശപിച്ചിരിക്കുന്നു.” (മുസ്നദ് അഹ്മദ്: 2816, ഇബ്നുഹിബ്ബാൻ: 4417, ഹാകിം: 8052 )

ملعونٌ مَنْ وقعَ على بهيمَةٍ ، ملعونٌ مَنْ عمِلَ بعمَلِ قومِ لوطٍ

“മൃഗങ്ങളുമായി രതിയിൽ ഏർപ്പെട്ടവൻ ശപിക്കപ്പെട്ടിരിക്കുന്നു. ലൂത്തിന്റെ ജനതയുടെ പ്രവർത്തനത്തിൽ (സ്വവർഗരതി) ഏർപ്പെട്ടവൻ ശപിക്കപ്പെട്ടിരിക്കുന്നു.” (മുസ്നദു അഹ്മദ്: 1875, ഇബ്നുഹിബ്ബാൻ: 4417, ത്വബ്റാനി: 11546 )

3. പ്രവാചകാനുചരന്മാരുടെ കാലഘട്ടത്തിലെ അവസാന സന്ധിയിൽ ഇസ്‌ലാമിന്റെ വളർച്ച ദ്രുതഗതിയിലാവുകയും ലക്ഷോപലക്ഷങ്ങൾ ഇസ്‌ലാമിലേക്ക് കടന്നുവരികയും ചെയ്യാൻ തുടങ്ങിയപ്പോൾ ദുർബല വിശ്വാസികളും കപട വിശ്വാസികളും ഇസ്‌ലാമിക സമൂഹത്തിൽ സ്ഥാനം പിടിച്ചു. വളർച്ച പ്രാപിച്ച മറ്റേത് സമുദായങ്ങളിലേതുമെന്ന പോലെ മുസ്‌ലിംകൾക്കിടയിലും ജീർണതകളും അധാർമികതകളും വർധിച്ചു. മദ്യം, വ്യഭിചാരം, കളവ്, മോഷണം, കൊള്ള, കൊല തുടങ്ങിയ സാമൂഹിക- വൈയക്തിക തിന്മകൾ എല്ലാ സമൂഹങ്ങളിലും ഇടതടവില്ലാതെ നടക്കുന്നുണ്ട്. മുസ്‌ലിം സമൂഹത്തിൽ ദുർബല വിശ്വാസികളും കപട വിശ്വാസികളും സ്ഥാനം പിടിച്ചതോടെ അവരിലും ഇത്തരം ദൂഷ്യങ്ങൾ പെരുകുക സ്വഭാവികം മാത്രം. അതുകൊണ്ട് തന്നെ ഈ സാമൂഹിക ഭൂമികയെ പരിഗണിച്ച് കർമ്മശാസ്ത്ര ചർച്ചകളെ വാർത്തെടുക്കാൻ കർമശാസ്ത്ര പണ്ഡിതർ നിർബന്ധിതരായി. അല്ലാതെ സാമൂഹിക യഥാർത്ഥ്യങ്ങളെ അവഗണിച്ച് എങ്ങനെ ഫിക്ഹ് (കർമ്മശാസ്ത്ര) ചർച്ച ചെയ്യും? അധാർമിക പ്രവണതകളെയും ആ പ്രവണതകളുടെ അനന്തരഫലമായുണ്ടാകുന്ന സാമൂഹിക- കുടുംബ- വൈയക്തിക പ്രശ്നങ്ങളെയുമെല്ലാം കർമ്മശാസ്ത്ര പണ്ഡിതർ ചർച്ച ചെയ്തിട്ടുണ്ട്. അതിൽ ഒരു വിഷയവും അറപ്പു കൊണ്ട് മാറ്റി വെച്ചിട്ടില്ല. ഒരു കൈപ്പുറ്റ യാഥാർത്ഥ്യത്തെയും അവഗണിച്ച് ഫിക്ഹിനെ (കർമ്മശാസ്ത്ര) വാർത്തെടുക്കാൻ ശ്രമിച്ചിട്ടുമില്ല, ശ്രമിക്കാനും പാടില്ല. സ്വാഭാവികമായും നിഷിദ്ധമായ രതി വൈകൃതങ്ങൾ സമൂഹത്തിൽ നടക്കാനുള്ള സാധ്യത മുന്നിൽ കണ്ട് അത്തരം പ്രശ്നങ്ങളിലെ കർമ്മശാസ്ത്ര വിധികളെ പറ്റി ചർച്ച ചെയ്തിട്ടുണ്ട്. അതിനർത്ഥം അത്തരം അവിഹിതങ്ങളെയൊ രതി വൈകൃതങ്ങളെയൊ ഇസ്‌ലാമിക കർമ്മശാസ്ത്രമോ, കർമ്മശാസ്ത്ര ചർച്ചകളുടെ അവലംബമായ ഇസ്‌ലാമിക പ്രമാണങ്ങളൊ അംഗീകരിക്കുന്നു എന്നല്ല. ഒരു രാജ്യത്ത് കൊലപാതകവും കലാപവും ബലാത്സംഗവുമെല്ലാം നടന്നാലുള്ള അനന്തര നടപടികളെ സംബന്ധിച്ച നിയമങ്ങൾ ഉണ്ടെന്നതും ഭരണഘടന അത് ചർച്ച ചെയ്യുന്നുണ്ട് എന്നതും ആ രാജ്യവും ഭരണഘടനയും ഈ ദ്രോഹങ്ങൾക്ക് അംഗീകാരം നൽകുന്നു എന്നതിന് തെളിവാണോ ? അല്ലല്ലൊ. ഈ വസ്തുത മനസ്സിലാക്കിയതിന് ശേഷമാവണം ഹദീസ് ഗ്രന്ഥങ്ങളിലെ കർമ്മശാസ്ത്ര ചർച്ചയെ നാം സമീപിക്കേണ്ടത്.

“In Islam, sex with animals and sex with dead bodies is halal… ” “ഇസ്‌ലാമിൽ മൃഗങ്ങളുമായുള്ള ലൈംഗികതയും മൃതദേഹവുമായുള്ള ലൈംഗികതയും ഹലാലാണ്…” എന്ന പെരും നുണ പറഞ്ഞു കൊണ്ടാണ് വിമർശകർ പ്രചരിപ്പിക്കുന്ന ഈ വിമർശനം ആരംഭിക്കുന്നത്. തുടർന്ന് അതിന് തെളിവായി ഉദ്ധരിക്കുന്നത് ഇതാണ്:

Book: Sunan Abu Dawood (Ifa), Chapter: 33 / Provision of Punishment, Hadith Number: 4406

4406. Narrated from Ahmad Ibn Yunus (R): Ibn Abbas (R). He said: There is no punishment for having intercourse with an animal.

“പുസ്തകം: സുനൻ അബു ദാവൂദ് അധ്യായം: 33 / ശിക്ഷയുടെ വ്യവസ്ഥ, ഹദീസ് നമ്പർ: 4406 4406.

അഹ്‌മദ്‌ ഇബ്നു യൂനുസ് (റ): ഇബ്നു അബ്ബാസ് (റ) ൽ നിന്ന് നിവേദനം. അദ്ദേഹം പറഞ്ഞു: മൃഗവുമായി ഇണചേരുന്നതിന് ശിക്ഷയില്ല.”

സുനനു അബൂദാവൂദിൽ നിന്ന് ദുർവ്യാഖ്യാനിക്കാൻ സാധ്യത കാണുന്ന ഒരു വാചകമങ്ങെടുത്ത് ചേർത്തു! അതിന്റെ മുന്നും പിന്നുമൊന്നുമില്ല.!! അധ്യായം മുഴുവനായും ഇവിടെ ഉദ്ധരിച്ചിട്ടാവാം നമ്മുടെ ചർച്ച:

30 – باب فيمن أتى بهيمةً

4464 – حدَّثنا عبدُ الله بنُ محمَّد النُّفيليُّ، حدَّثنا عبدُ العزيز بنُ محمدٍ، حدَّثني عَمرُو بنُ أبي عَمرو، عن عِكرمة

عن ابنِ عباس، قال: قال رسولُ الله – صلَّى الله عليه وسلم -: “مَن أتى بهيمةً، فاقتلوهُ واقتلوها مَعَهم” قال: قُلتُ له: ما شأنُ البهيمةِ؟ قال: ما أُراه قال ذلك إلا أنه كَرِهَ أن يُؤكَلَ لحمُها، وقد عُمِلَ بها ذلك العملُ…

4465 – حدَّثنا أحمدُ بنُ يونسَ، أن شريكاً وأبا الأحوصِ وأبا بكر بنَ عياشٍ حدَّثوهم، عن عاصِمٍ، عن أبي رزينٍ

عن ابنِ عباس، قال: ليس على الذي يأتي البهيمة حدٌّ.

قال أبو داود: وكذا قال عطاءٌ، وقال الحكم: أرى أن يُجلَدَ ولا يُبْلَغَ به الحدّ، وقال الحسنُ: هو بمنزلةِ الزَّاني

“അധ്യായം: മൃഗവുമായി രതിയിൽ ഏർപ്പെട്ടവന് എന്ത് ശിക്ഷാ നടപടിയാണ് സ്വീകരിക്കേണ്ടത്.” എന്ന അധ്യായത്തിന്റെ നാമം തന്നെ വായിച്ചാൽ കാര്യം സ്പഷ്ടമാണ്. മൃഗരതി അനുവദനീയമാണൊ അല്ലെ എന്നല്ല അധ്യായം. മൃഗരതി എന്ന വൻപാപം ഒരാൾ ചെയ്താൽ അയാളുടെ മേൽ ഒരു ഇസ്‌ലാമിക ഭരണകൂടം ഭൗതികമായ ശിക്ഷാ നടപടികൾ എന്തെങ്കിലും നടപ്പാക്കണൊ വേണ്ടെ എന്നത് മാത്രമാണ് ചർച്ച. മൃഗരതി എന്ന വൻപാപത്താൽ പരലോകത്ത് നൽകപ്പെടുന്ന ദൈവികമായ ശിക്ഷയൊ, ഭൗതികമായ പ്രായശ്ചിത്തമൊ ‘തഅ്സീറൊ’ മാത്രമാണ് അയാളുടെ മേൽ ഉള്ളു, വധശിക്ഷയൊ സമാനമായ ഭൗതിക ശിക്ഷാ നടപടികൾ ഇല്ല എന്ന് ചില പണ്ഡിതന്മാർ അഭിപ്രായപ്പെട്ടിട്ടുണ്ട്. നബിയുടെ (സ) കാലഘട്ടത്തിൽ മൃഗരതിയിൽ ആരും ഏർപ്പെടാത്തതിനാൽ ശിക്ഷാ നടപടികൾ ഒന്നും നടപ്പാക്കിയതായി ഇസ്‌ലാമിക പ്രമാണങ്ങളിൽ രേഖപ്പെടുത്തപ്പെട്ടിട്ടില്ല എന്നതാണ് ഈ ഒറ്റപ്പെട്ട അഭിപ്രായത്തിന് കാരണം. ഹദ്ദ് അഥവാ ശിക്ഷക്ക് പകരം കഫ്ഫാറത്ത് അഥവാ പ്രായശ്ചിത്തം ആണ് വിധിയെന്ന് അവർ വാദിക്കുന്നു.

മൃഗരതിക്ക് ഭൗതിക ശിക്ഷാ നടപടികൾ ഒന്നുമില്ല എന്ന ഇബ്നു അബ്ബാസിന്റെ അഭിപ്രായം സുനനു അബൂദാവൂദിൽ നിന്നും കോട്ടിമാട്ടിയ വിമർശകർ ആ വാചകത്തിന്റെ തൊട്ടു മുകളിലും താഴെയുമുള്ള വാചകങ്ങൾ കട്ടു മറച്ചു.

مَن أتى بهيمةً، فاقتلوهُ واقتلوها مَعَهم

“മൃഗരതിയിൽ ഏർപ്പെടുന്നവന് വധശിക്ഷയുണ്ട്” എന്ന ഇബ്നു അബ്ബാസിന്റെ(റ) തന്നെ അഭിപ്രായം വിമർശകർ മുക്കി.

രണ്ടാമതായി വിമർശകർ മുക്കിയ വാചകമിതാണ്: وقال الحكم: أرى أن يُجلَدَ ولا يُبْلَغَ به الحدّ، وقال الحسنُ: هو بمنزلةِ الزَّاني

“ഹകം പറഞ്ഞു: മൃഗരതിയിൽ ഏർപ്പെട്ടവന് മർദ്ദന ശിക്ഷ നടപ്പാക്കണം. അങ്ങേയറ്റം ശിഷക്ക് വിധേയനാക്കണം. ഹസൻ പറഞ്ഞു: അവൻ വ്യഭിചാരിക്ക് സമാനമായതിനാൽ വ്യഭിചാരത്തിനുള്ള ശിക്ഷ അവനിൽ നടപ്പാക്കണം.”

ഇതൊന്നും കാണാത്ത മട്ടിൽ ഭൗതിക ശിക്ഷാവിധി ഉണ്ടെന്നും ഇല്ലെന്നും ഒരു പണ്ഡിതൻ തന്നെ അഭിപ്രായപ്പെട്ട രണ്ട് അഭിപ്രായത്തിൽ ശിക്ഷാവിധി ഇല്ലെന്ന വാചകം മാത്രമെടുത്തു വെച്ച്, ഫ്രെയ്‌മിട്ടു. ചർച്ച ശിക്ഷാവിധി ഇല്ല എന്ന ആശയത്തിൽ നിന്നും അനുവദനീയം എന്ന ആശയത്തിലേക്ക് തള്ളി മാറ്റി… എങ്ങനെയെങ്കിലും തങ്ങളുടെ മനസ്സുകളിലെ ലൈംഗിക വൈകൃതങ്ങൾ മറ്റുള്ളവരിൽ പ്രതിഫലിപ്പിച്ച് കാണിക്കുന്ന ഈ മ്ലേച്ഛത എത്ര ഭീകരം !!

ഇസ്‌ലാമിൽ മൃഗരതി അനുവദനീയമാണ് എന്ന് കെട്ടിച്ചമക്കാൻ വിമർശകർ ഉദ്ധരിക്കുന്ന മറ്റൊരു വാചകമിതാണ്:

There are two opinions if a woman inserts the penis of an animal (into her vagina), and if she inserts a separate penis; The most accurate is that the woman’s genitals should be washed.

MuslimSahih Muslim – Book of Menstruation – hadith # 525 – Commentary

According to some Sunni Islamic scholars, sexual intercourse with an animal does not invalidate fasting or Hajj.

“ഒരു സ്ത്രീ ഒരു മൃഗത്തിന്റെ ലിംഗം (അവളുടെ യോനിയിൽ) പ്രവേശിപ്പിക്കുകയാണെങ്കിൽ, അവൾ ഒരു ച്ഛേദിക്കപ്പെട്ട ലിംഗം പ്രവേശിപ്പിക്കുകയൊ ആണെങ്കിൽ അക്കാര്യത്തിൽ രണ്ട് അഭിപ്രായങ്ങളുണ്ട്; സ്ത്രീയുടെ ജനനേന്ദ്രിയങ്ങൾ കഴുകണം എന്നതാണ് ഏറ്റവും കൃത്യമായത്.

സഹിഹ് മുസ്ലിം – ഹദീസ് # 525 – വ്യാഖ്യാനം അഭിപ്രായം: ഏതെങ്കിലും മൃഗത്തിന്റെ ലിംഗഭേദം അല്ലെങ്കിൽ പ്രത്യേക പുരുഷ ജനനേന്ദ്രിയം അതിന്റെ സ്ത്രീ ജനനേന്ദ്രിയത്തിൽ ചേർക്കാവുന്നതാണ്.

ചില സുന്നി ഇസ്ലാമിക പണ്ഡിതരുടെ അഭിപ്രായത്തിൽ, മൃഗവുമായുള്ള ലൈംഗികബന്ധം നോമ്പിനെയോ ഹജ്ജിനെയോ അസാധുവാക്കില്ല.”

മുകളിൽ ഉദ്ധരിക്കപ്പെട്ടത് ക്വുർആനൊ നബിയുടെ ഹദീസൊ അല്ല. അറബിയിൽ എഴുതപ്പെടുന്നതെല്ലാം ഇസ്‌ലാമാണെന്ന തെറ്റിദ്ധാരണ നാം മാറ്റി വെക്കുക.

ക്രിസ്താബ്ദം 1277 ൽ അഥവാ ഹിജ്റാബ്ദം 676 ൽ നിര്യാതനായ ഇമാം നവവി എന്ന ശാഫിഈ കർമ്മശാസ്ത്ര പണ്ഡിതന്റെ വാചകമാണ് മുകളിലെ ആദ്യത്തെ പാരഗ്രാഫ്. പ്രവാചക വിയോഗത്തിന് ശേഷം ആറ് നൂറ്റാണ്ട് കഴിഞ്ഞ് വന്ന ഒരു പണ്ഡിതൻ, ഹദീസിന്റെ വ്യാഖ്യാനത്തിൽ കുറിച്ചിട്ട തന്റെ കർമ്മശാസ്ത്ര സംബന്ധമായ അഭിപ്രായങ്ങങ്ങളെ ഇസ്‌ലാമിന്റെ പ്രമാണമൊ വിശുദ്ധ ഗ്രന്ഥമൊ ആയി അവതരിപ്പിക്കുന്നതു തന്നെ അബദ്ധമാണ്.

മാത്രമല്ല, ഇമാം നവവി കുറിച്ചിട്ട ചർച്ചയും ശ്രദ്ധ അർഹിക്കുന്നുണ്ട്. മൃഗരതി എന്ന വൻപാപം അനുവദനീയമൊ നിഷിദ്ധമൊ എന്നതല്ല അദ്ദേഹവും ചർച്ച ചെയ്യുന്നത്!! അത്തരമൊരു മ്ലേച്ഛത ഒരാൾ ചെയ്താൽ അതിനെ തുടർന്നു വരുന്ന മറ്റു കർമ്മശാസ്ത്ര വിധികളെ പറ്റി മാത്രമാണ് ഇവിടെയും ചർച്ച.

മൃഗരതിയെ സംബന്ധിച്ച ധാർമ്മിക വിധി എന്താണെന്ന് വളരെ വ്യക്തമായി ഇമാം നവവി തന്നെ രേഖപ്പെടുത്തുന്നത് കാണുക:

ﻭﻳﺤﺮﻡ اﺗﻴﺎﻥ اﻟﺒﻬﻴﻤﺔ ﻟﻘﻮﻟﻪ ﻋﺰ ﻭﺟﻞ (ﻭاﻟﺬﻳﻦ ﻫﻢ ﻟﻔﺮﻭﺟﻬﻢ ﺣﺎﻓﻈﻮﻥ اﻻ ﻋﻠﻰ ﺃﺯﻭاﺟﻬﻢ ﺃﻭ ﻣﺎ ﻣﻠﻜﺖ ﺃﻳﻤﺎﻧﻬﻢ ﻓﺈﻧﻬﻢ ﻏﻴﺮ ﻣﻠﻮﻣﻴﻦ) ﻓﺈﻥ ﺃﺗﻰ اﻟﺒﻬﻴﻤﺔ ﻭﻫﻮ ﻣﻤﻦ ﻳﺠﺐ ﻋﻠﻴﻪ ﺣﺪ اﻟﺰﻧﺎ ﻓﻔﻴﻪ ﺛﻼﺛﺔ ﺃﻗﻮاﻝ…

“മൃഗങ്ങളുമായുള്ള രതി നിഷിദ്ധമാണ്. കാരണം, അല്ലാഹു ഇപ്രകാരം പറഞ്ഞിരിക്കുന്നു: ‘തങ്ങളുടെ ഗുഹ്യാവയവങ്ങളെ കാത്തുസൂക്ഷിക്കുന്നവരുമത്രെ അവര്‍. തങ്ങളുടെ ഭാര്യമാരുമായോ, തങ്ങളുടെ അധീനത്തിലുള്ള അടിമസ്ത്രീകളുമായോ ഉള്ള ബന്ധം ഒഴികെ. അപ്പോള്‍ അവര്‍ ആക്ഷേപാര്‍ഹരല്ല. എന്നാല്‍ അതിന്നപ്പുറം ആരെങ്കിലും ആഗ്രഹിക്കുന്ന പക്ഷം അവര്‍ തന്നെയാണ് അതിക്രമകാരികള്‍.’ (ക്വുർആൻ: 23:5-7). ഇനി ആരെങ്കിലും മൃഗങ്ങളെ സമീപിച്ചാൽ അയാളിൽ നടപ്പാക്കേണ്ട ശിക്ഷാവിധിയിൽ മൂന്ന് അഭിപ്രായങ്ങൾ പണ്ഡിതന്മാർക്കിടയിലുണ്ട്…” (അൽ മജ്മൂഉ ശർഹുൽ മുഹദ്ദബ്: 20:30)

തുടർന്ന് ഇമാം നവവി മൂന്ന് അഭിപ്രായങ്ങൾ വിശദീകരിച്ചു.

(ﺃﺣﺪﻫﺎ) ﺃﻧﻪ ﻳﺠﺐ ﻋﻠﻴﻪ اﻟﻘﺘﻞ

അതിൽ പ്രഥമവും പ്രധാനവുമായ അഭിപ്രായം അയാൾക്ക് വധശിക്ഷ നൽകപ്പെടണം എന്നതാണ്.

രണ്ടാമത്തെ അഭിപ്രായം മൃഗരതിക്കുള്ള ശിക്ഷ വ്യഭിചാരത്തിനുള്ള ശിക്ഷ പോലെയാണ് എന്നാണ്. വിവാഹിതൻ വ്യഭിചരിച്ചാൽ വധശിക്ഷയും അവിവാഹിതനാണെങ്കിൽ മർദ്ദനവുമാണ് ശിക്ഷ. മൂന്നാമത്തെ അഭിപ്രായം അയാൾക്ക് ഭൗതികമായ ശിക്ഷാവിധികളൊന്നും ഭരണാധികാരി നടപ്പാക്കേണ്ടതില്ല എന്നതാണ്. (അൽ മജ്‌മൂഉ ശർഹുൽ മുഹദ്ദബ്: 20:30)

അത്തരമൊരു രതി വൈകൃതത്തിൽ ഏർപ്പെട്ടാൽ പിന്നീടു മറ്റു കർമ്മങ്ങൾ നിർവ്വഹിക്കാൻ അംഗശുദ്ധിയും കുളിയും നിർബന്ധമാണൊ എന്നതാണ് ഒരു ചർച്ച.

‏وَلَوْ اسْتَدْخَلَت الْمَرْأَة ذَكَرَ بَهِيمَة وَجَبَ عَلَيْهَا الْغُسْل , وَلَوْ اسْتَدْخَلَت ذَكَرًا مَقْطُوعًا فَوَجْهَانِ أَصَحّهمَا يَجِب عَلَيْهَا الْغُسْل

“ഒരു സ്ത്രീ, മൃഗത്തിന്റെ ലിംഗം തന്റെ ഗുഹ്യാവയവത്തിൽ പ്രവേശിപ്പിച്ചാൽ കുളിച്ച് ശുദ്ധി വരുത്തൽ അവൾക്ക് നിർബന്ധമാണ്. ഒരു സ്ത്രീ, ച്ഛേദിക്കപ്പെട്ട ഒരു ലിംഗം തന്റെ ഗുഹ്യാവയവത്തിൽ പ്രവേശിപ്പിച്ചാലുള്ള വിധിയിൽ രണ്ടഭിപ്രായമുണ്ട്. കുളിച്ച് ശുദ്ധി വരുത്തൽ നിർബന്ധമാണ് എന്നതാണ് ശരിയായ അഭിപ്രായം.” ഇതാണ് ഇമാം നവവിയുടെ വാചകം. ഭൗതികവാദി അത് ഇംഗ്ലീഷിലേക്ക് മാറ്റിയപ്പോൾ പരിഭാഷയിൽ വേണ്ടുവോളം വെള്ളം ചേർത്തു എന്ന് സാന്ദർഭികമായി സൂചിപ്പിക്കട്ടെ. ഭൗതികവാദിയുടെ പരിഭാഷ ശ്രദ്ധിക്കുക: There are two opinions if a woman inserts the penis of an animal (into her vagina), and if she inserts a separate penis; The most accurate is that the woman’s genitals should be washed.

അത്തരമൊരു രതി വൈകൃതത്തിൽ ഏർപ്പെട്ടാൽ മഹാപാപിയായി പരിണമിക്കുമെങ്കിലും ആ വൻപാപം മൂലം വ്രതമോ ഹജ്ജ് കർമ്മമോ അസാധുവായി പോവുമോ എന്ന ചർച്ചയും ചില കർമ്മശാസ്ത്ര പണ്ഡിതർ നടത്തിയിട്ടുണ്ടാവാം. അതാണ് ഈ വാചകത്തിന്റെ പൊരുൾ: “According to some Sunni Islamic scholars, sexual intercourse with an animal does not invalidate fasting or Hajj.” മൃഗരതിയുടെ ധാർമ്മിക വിധിയല്ല ഇവിടെയും അന്വേഷിക്കപ്പെടുന്നത്.

പൈശാചിക സ്വാധീനത്താൽ ഒരു ദുർബല നിമിഷത്തിൽ മൃഗ രതിയിൽ ഒരാൾ ഏർപ്പെട്ടു എന്ന് കരുതുക. ചിലപ്പോൾ, മൃഗരതി ലൈംഗിക അവകാശത്തിന്റെ ഭാഗമായി കാണുന്ന ഒരു എക്സ് മുസ്‌ലിമിന്റെയൊ ഫ്രീതിങ്കേഴ്സിന്റെയൊ അവകാശ വാദങ്ങളിൽ സ്വാധീനിക്കപ്പെട്ട് കൊണ്ട് അങ്ങനെയൊന്ന് ഒരാളിൽ നിന്ന് സംഭവിച്ചു എന്ന് കരുതുക. ശേഷം, അയാൾ, “മൃഗങ്ങളുമായി രതിയിൽ ഏർപ്പെട്ടവനെ ദൈവം ശപിച്ചിരിക്കുന്നു…”(മുസ്നദ് അഹ്മദ്: 2816) എന്ന നബിയുടെ (സ) ഹദീസിൽ നിന്ന് ആ പാപത്തിന്റെ ഭയാനകത മനസ്സിലാക്കുകയും ചെയ്തു. ഇത്തരമൊരു സാഹചര്യത്തിൽ തുടർന്ന് അയാൾ പാലിക്കേണ്ട ശുദ്ധി, ആരാധനാ കർമ്മങ്ങൾ എന്നിങ്ങനെയുള്ള പശ്ചാത്താപ-ആത്മീയ അനുഷ്ഠാനങ്ങളെ സംബന്ധിച്ച വിധിവിലക്കുകൾ കർമ്മശാസ്ത്രം ചർച്ച ചെയ്യേണ്ടതില്ലെ ?! അത്തരം തുടർന്നടപടികൾ ചർച്ച ചെയ്താൽ അതിനർത്ഥം ആ വൻപാപത്തെ ആ കർമ്മശാസ്ത്ര പണ്ഡിതർ അനുകൂലിക്കുന്നു എന്നാണെന്ന് വർഗീയ തിമിരം ബാധിച്ചവരല്ലാതെ വാദിക്കുകയില്ല.

ചുരുക്കത്തിൽ, മൃഗരതി നന്മയാണോ തിന്മയാണോ എന്നതല്ല ഇവിടെയൊന്നും ചർച്ച. ആ തിന്മ ഒരാൾ ചെയ്താൽ മറ്റു കർമ്മങ്ങളെ അത് എപ്രകാരം ബാധിക്കും എന്നാണ് ചർച്ച. അങ്ങനെ സംഭവിച്ചാൽ തുടർന്നുള്ള വിധികളെ സംബന്ധിച്ച ചില പണ്ഡിതരുടെ ചർച്ചകൾ “ഇസ്‌ലാമിന്റെ വിശുദ്ധ പ്രമാണങ്ങൾ” എന്ന വ്യാജേന അവതരിപ്പിക്കൽ ഭൗതികവാദികളുടെ സ്ഥിരം ഗവേഷക സംസ്കാരമാണ്.

Comment: The sex or separate male genitalia of any animal of Momina can be inserted into its female genitalia.

മൃഗരതിയുടെ ധാർമ്മിക വിധി ചർച്ച ചെയ്തു കൊണ്ട് ഇമാം നവവിയൊ മറ്റൊരു മുസ്‌ലിം പണ്ഡിതനൊ ഇത്തരമൊരു അഭിപ്രായം കുറിച്ചിട്ടിട്ടില്ല. ഇത്തരമൊരു വാചകം ഭൗതീകവാദികൾക്ക് കിട്ടിയത് “ഇമാം” ഡോകിൻസിൽ നിന്നൊ “ഇമാം” സാം ഹാരിസിൽ നിന്നൊ ഒക്കെയാവാനാണ് സാധ്യത.!!

വൈവാഹിക ബലാത്സംഗത്തെ (Marital rape) ഇസ്‌ലാം പ്രോത്സാഹിപ്പിക്കുന്നു എന്നതാണ് പുതിയ കണ്ടെത്തൽ !! പുരുഷന്മാരുടെ ഇംഗിതത്തിനു സ്ത്രീകൾ വഴങ്ങണമെന്നാണ് ഹദീസുകൾ പഠിപ്പിക്കുന്നത്, ലൈംഗിക വിഷയത്തിൽ പുരുഷർക്ക് ഏകപക്ഷീയമായ സർവാധികാരമാണ് ഇസ്‌ലാം നൽകുന്നത് എന്നുമൊക്കെ ആരോപണങ്ങളായി പ്രചരിക്കപ്പെടുന്നത്. ആരോപണങ്ങൾക്കായി അവലംബിക്കുന്ന ഹദീസ് ഇതാണ്:

إِذَا دَعَا الرَّجُلُ امْرَأَتَهُ إلى فِرَاشِهِ فأبَتْ فَبَاتَ غَضْبَانَ عَلَيْهَا لَعَنَتْهَا المَلَائِكَةُ حتَّى تُصْبِحَ.

“കിടപ്പറയിലേക്ക് ക്ഷണിച്ചിട്ട് അവിടേക്ക് വരാൻ വിസമ്മതിച്ച ഒരു സ്ത്രീ… തന്നിമിത്തം ഭർത്താവ് രാത്രി മുഴുവൻ അവളോട് കോപിച്ചവളായി കഴിച്ചു കൂട്ടുകയും ചെയ്യുന്നു. അങ്ങനെയുള്ള സ്ത്രീയെ മലക്കുകൾ പുലരുന്നതു വരെ ‘ശപിക്കുന്നതാണ്’…”(1) (സ്വഹീഹു മുസ്‌ലിം: 1436, സ്വഹീഹുൽ ബുഖാരി: 5193, 3237)

മറുപടി:

1. യഥാർത്ഥത്തിൽ, ഇസ്‌ലാമിൽ വൈവാഹിക ബലാത്സംഗം (Marital rape) ഇല്ല എന്നതിനുള്ള തെളിവാണ് ഈ ഹദീസ്. കാരണം, തനിക്ക് ലൈംഗിക ബന്ധം നിഷേധിച്ച ഭാര്യയോട് ‘കോപിച്ച് രാത്രി കഴിച്ചു കൂട്ടുക’ മാത്രമാണ് ഒരു വിശ്വാസിയായ ഭർത്താവ് ചെയ്യുന്നത് എന്ന് ഹദീസിൽ തന്നെ വ്യക്തമാക്കപ്പെട്ടിരിക്കുന്നു എന്നത് വിമർശകർ ബോധപൂർവ്വം അവഗണിച്ചിരിക്കുകയാണ്.

فَبَاتَ غَضْبَانَ عَلَيْهَا “തന്നിമിത്തം ഭർത്താവ് രാത്രി മുഴുവൻ അവളോട് കോപിച്ചവളായി കഴിച്ചു കൂട്ടുകയും ചെയ്തു…” എന്നാണല്ലൊ ഹദീസിൽ പറഞ്ഞിരിക്കുന്നത്. അതല്ലാതെ ഭാര്യക്കെതിരെ വാചകം കൊണ്ട് പോലും ദ്രോഹിക്കുന്നതായൊ ഒരു സൂചനയും ഹദീസിൽ ഇല്ല. വാചികമായൊ കായികമായൊ ഭർത്താവ് ഭാര്യയെ ലൈംഗിക ബന്ധത്തിന് നിർബന്ധിക്കുന്നതായ സൂചനയും ഹദീസിൽ ഇല്ല. പിന്നെ എങ്ങനെയാണിത് വൈവാഹിക ബലാത്സംഗമാകുന്നത് എന്ന് മാത്രം വിമർശകരോട് ചോദിക്കരുത്!

2. വെറുപ്പോടെയൊ നിർബന്ധിതമായൊ അടിച്ചേൽപ്പിക്കുന്ന രതി ഇസ്‌ലാം വ്യക്തമായി വിലക്കിയിട്ടുണ്ട്:

ﻗَﺎﻝَ ﺭَﺳُﻮﻝُ اﻟﻠَّﻪِ ﺻَﻠَّﻰ اﻟﻠﻪُ ﻋَﻠَﻴْﻪِ ﻭَﺳَﻠَّﻢَ: ﻻَ ﺗَﺤْﻤِﻠُﻮا اﻟﻨِّﺴَﺎءَ ﻋَﻠَﻰ ﻣَﺎ ﻳَﻜْﺮَﻫْﻦَ

“സ്ത്രീകളെ അവർ വെറുക്കുന്നത് ചെയ്യാൻ നിങ്ങൾ നിർബന്ധിക്കരുത്.” (മുസ്വന്നഫ് അബ്ദുർ റസാഖ്: 10320) എന്ന് മുഹമ്മദ് നബി (സ) പഠിപ്പിച്ചിട്ടുണ്ട്.

എന്നാൽ, ന്യായമായ കാരണങ്ങളൊ തടസ്സങ്ങളൊ ഇല്ലാതെ ലൈംഗികമായി ആസക്തനായ തന്റെ ഇണ ലൈംഗികതക്കായും അനുരാഗ സാന്നിധ്യത്തിനായും സ്നേഹപൂർവ്വം ക്ഷണിക്കുമ്പോൾ -തനിക്ക് ലൈംഗിക തൃഷ്ണ ഇപ്പോൾ അനുഭവപ്പെടുന്നില്ല എന്ന കാരണത്താൽ മാത്രം- ഒരു സ്ത്രീ ലൈംഗികത ഭർത്താവിന് നിഷേധിക്കരുത് എന്ന് മാത്രമാണ് ഹദീസിൽ സൂചിപ്പിക്കപ്പെടുന്നത്.

വെറുപ്പോടെയൊ നിർബന്ധിതമായൊ വിധേയമാക്കപ്പെടുന്ന രതിയും ലൈംഗിക തൃഷ്ണ ഇല്ലാതെ ഏർപ്പെടുന്ന രതിയും തമ്മിൽ വലിയ വ്യത്യാസമുണ്ട് എന്ന് തന്നെയാണ് ലൈംഗിക ശാസ്ത്രജ്ഞർ അഭിപ്രായപ്പെടുന്നത്.

ലൈംഗിക തൃഷ്ണയോ (Sexual appetite), ഭാവനിലയൊ (mood) ഇല്ലാതെയുള്ള രതിക്ക് സ്ത്രീകൾ തയ്യാറാവുക എന്നത് സ്ത്രീകളുടെ ലൈംഗിക പ്രകൃതിയെ സംബന്ധിച്ചിടത്തോളം തീർത്തും സ്വാഭാവികമായ ഒരു കാര്യമാണ്. സ്ത്രീ പുരുഷ ലൈംഗികാവയങ്ങളുടെ വ്യത്യാസങ്ങൾ തന്നെ ഇത് പ്രത്യക്ഷമായി തെളിയിക്കുന്നുണ്ട്. ലൈംഗിക തൃഷ്ണ ഇല്ലാതെ തന്നെ ലൈംഗികതക്ക് സജ്ജമായ രൂപത്തിലാണ് സ്ത്രീകളുടെ ലൈംഗിക സംഭോഗവുമായി ബന്ധപ്പെട്ട അവയവങ്ങൾ. പുരുഷന്റേതാകട്ടെ ലൈംഗിക സംഭോഗവുമായി ബന്ധപ്പെട്ട അവയവമായ ലിംഗത്തിന്റെ ഉദ്ധാരണം (erection) ലൈംഗികാഭിലാഷത്തിലൂടെയാണ് സാധാരണയായി സാധ്യമാവുക. ഈ വ്യതിരിക്തത തന്നെ ലൈംഗിതയും ലൈംഗിക തൃഷ്ണയും തമ്മിലുള്ള ബന്ധത്തിൽ സ്ത്രീയും പുരുഷനും വ്യത്യസ്തരാണ് എന്ന് പ്രത്യക്ഷമായി സൂചിപ്പിക്കുന്നുണ്ട്.

Sexual desire is typically higher in men than in women, with testosterone (T) thought to account for this difference as well as within-sex variation in desire in both women and men. ( https://pubmed.ncbi.nlm.nih.gov/22552705/ )

ടെസ്റ്റോസ്റ്റിറോണിന്റെ (ടി) വ്യത്യാസവും വ്യതിയാനവും കാരണം ലൈംഗികാഭിലാഷം സാധാരണയായി സ്ത്രീകളേക്കാൾ പുരുഷന്മാരിൽ കൂടുതലാണ്.

സ്ത്രീകളേക്കാൾ ലൈംഗിക തൃഷ്ണ (sexual appetite) പുരുഷന്മാരിൽ ശക്തമായി നിലനിൽക്കുന്നു എന്നതിനാൽ തന്നെ സ്ത്രീകളേക്കാൾ ലൈംഗിക ബന്ധങ്ങൾക്കായി പുരുഷൻ ആഗ്രഹിക്കുക എന്നത് സ്വാഭാവികമാണ്.

ഇതിന്റെ ഫലമായി, ദമ്പതികൾ തമ്മിൽ ലൈംഗിക ചോദനകളിൽ ചേർച്ചയില്ലാതിരിക്കുക എന്ന അവസ്ഥ ധാരാളമായും സംഭവിക്കുന്നു എന്നതാണ് മറ്റൊരു പ്രധാനപ്പെട്ട വസ്തുത.

Mismatched sex drives, or sex drive discrepancy (SDD), is the most common of these situations….

പൊരുത്തമില്ലാത്ത ലൈംഗിക ചോദനകൾ, അല്ലെങ്കിൽ സെക്‌സ് ഡ്രൈവ് ഡിസ്‌ക്രീപൻസി (SDD) ദമ്പതികൾക്കിടയിൽ വളരെ സാധാരണമാണ്…. ( https://www.medicalnewstoday.com/articles/mismatched-sex-drives#management-techniques )

Research shows that up to 80% of couples regularly experience situations where one partner wants to have sex and the other doesn’t (Day, Muise, Joel, & Impett, 2015). While sex therapists would tell you that low sexual desire is the most common sexual problem, desire discrepancy is considered more distressing due to its dampening down the romance in a relationship (Mark, 2015).

ഒരു പങ്കാളി ലൈംഗികബന്ധത്തിൽ ഏർപ്പെടാൻ ആഗ്രഹിക്കുന്നതും മറ്റെയാൾക്ക് ആഗ്രഹമില്ലാതിരിക്കുന്നതുമായ സാഹചര്യങ്ങൾ 80% ദമ്പതികൾക്കും പതിവായി അനുഭവപ്പെടുന്നതായി ഗവേഷണങ്ങൾ കാണിക്കുന്നു (Day, Muise, Joel, & Impett, 2015). ലൈംഗികാഭിലാഷം കുറയുന്നതാണ് ലൈംഗിക പ്രശ്‌നങ്ങളുടെ ഏറ്റവും സാധാരണമായ സ്രോതസ് എന്ന് സെക്‌സ് തെറാപ്പിസ്റ്റുകൾ നിങ്ങളോട് പറയുമെങ്കിലും, ഒരു ദാമ്പത്യത്തിലെ ബന്ധത്തിലെ പ്രണയം, കുറയുന്നതിന് കാരണമായി നിലകൊള്ളുന്നു എന്നതിനാൽ തന്നെ ലൈംഗികാഭിലാഷത്തിലെ പൊരുത്തക്കേട് ഏറ്റവും ഗുരുതരമായ ദാമ്പത്യ പ്രശ്നമായി കണക്കാക്കപ്പെടുന്നു. (Mark, 2015).

( https://www.google.com/amp/s/www.psychologytoday.com/us/blog/intersections/201912/sexual-desire-discrepancy-why-it-s-big-deal-couples%3famp#ip=1 )

പുരുഷ ലൈംഗികതയുടെ മറ്റൊരു വ്യതിരിക്തതയാണ് യാദൃശ്ചികമായ അല്ലെങ്കിൽ പെട്ടെന്ന് ഉത്ഭവിക്കുന്ന ലൈംഗികാഭിലാഷം.

Spontaneous sexual desire: As the name implies, this form of desire happens randomly, with or without stimulation. This desire supports the linear view of sexuality that begins with desire, followed by excitement, finally leading to orgasm.

Nagoski states that around 70% of men have this type of sexual desire while only about 10–20% of women do….

യാദൃശ്ചികമായ ലൈംഗികാഭിലാഷം:

ഉത്തേജനത്തോടുകൂടിയോ അല്ലാതെയോ ലൈംഗികേച്ഛ ക്രമരഹിതമായി ഉത്ഭവിക്കുന്ന പ്രകൃതം. ഈ പ്രകൃതി, ലൈംഗിക ദാഹത്തിലൂടെ തുടങ്ങി, ഉത്തേജനത്തിലൂടെ കടന്നുപോയി, ഒടുവിൽ രതിമൂർച്ഛയിലേക്ക് അവസാനിക്കുന്ന ലൈംഗികതയുടെ രേഖീയ വീക്ഷണത്തെ പ്രതിനിധീകരിക്കുന്നു.

ഏകദേശം 70% പുരുഷന്മാർക്കും ഇത്തരത്തിലുള്ള ലൈംഗികാഭിലാഷം ഉണ്ടെന്ന് നാഗോസ്കി പ്രസ്താവിക്കുന്നു, എന്നാൽ 10-20% സ്ത്രീകൾക്ക് മാത്രമേ ഇത് ഉള്ളൂ….

( https://www.medicalnewstoday.com/articles/mismatched-sex-drives#management-techniques )

While most men are “preheated,” most women warm up to sex slowly. Learning the reasons for this difference can help men become better lovers.

Most men over 50 can remember experiencing libido as a strong drive — akin, almost, to hunger: They felt horny and went after sex. (Indeed, some still do.) But recent research shows that women experience libido as an urge far less compelling than that. In a landmark study published in the Journal of the American Medical Association in 1999, for example, University of Chicago sociologist Edward O. Laumann revealed his findings that 30 percent of women have low or no libido. (This sexual desire difference is one of the most frequent causes for women and couples to seek sex therapy.)

But there was also a silver lining: If these women have sex and enjoy it, they eventually experience desire.

മിക്ക പുരുഷന്മാരും ലൈംഗിക കേളികൾക്ക് മുമ്പ് തന്നെ ലൈംഗിക ഔൽസുക്യം (preheated) വെച്ചുപുലർത്തുമ്പോൾ, മിക്ക സ്ത്രീകളും ലൈംഗികതയിലേക്ക് ഊഷ്മളത കൈവരിക്കുന്നത് സാവധാനത്തിലാണ്. ഈ വ്യത്യാസത്തിന്റെ കാരണങ്ങൾ പഠിക്കുന്നത് പുരുഷന്മാരെ മികച്ച പ്രണയികളാകാൻ സഹായിക്കും.

50 വയസ്സിനു മുകളിലുള്ള മിക്ക പുരുഷന്മാർക്കും ലിബിഡോ ഒരു ശക്തമായ പ്രേരണയായി മനസ്സിലാക്കാം- ഏതാണ്ട്, വിശപ്പിന് സമാനമാണത്: പുരുഷൻമാർക്ക് ലൈംഗിക തൃഷ്ണ തോന്നുന്നു, അവർ ലൈംഗിക ബന്ധത്തെ പിന്തുടരുന്നു. എന്നാൽ സമീപകാല ഗവേഷണങ്ങൾ കാണിക്കുന്നത് സ്ത്രീകൾ ലിബിഡോ (കാമചോദന) അനുഭവിക്കുന്നത് അതിനേക്കാൾ വളരെ കുറഞ്ഞ ഒരു പ്രേരണയായാണ്. ഉദാഹരണത്തിന്, 1999-ൽ അമേരിക്കൻ മെഡിക്കൽ അസോസിയേഷൻ ജേണലിൽ പ്രസിദ്ധീകരിച്ച ഒരു പ്രധാന പഠനത്തിൽ, ചിക്കാഗോ സർവകലാശാലയിലെ സാമൂഹ്യശാസ്ത്രജ്ഞനായ എഡ്വേർഡ് ഒ. ലൗമാൻ തന്റെ കണ്ടെത്തലുകളിൽ വെളിപ്പെടുത്തിയത് 30 ശതമാനം സ്ത്രീകൾക്ക് ലിബിഡോ (കാമചോദന) കുറവോ ഇല്ലെന്നോ ആണ്. (ഈ ലൈംഗിക ചോദനയുടെ വ്യത്യാസം സ്ത്രീകളും ദമ്പതികളും സെക്‌സ് തെറാപ്പി തേടുന്നതിനുള്ള ഏറ്റവും സാധാരണമായ കാരണങ്ങളിലൊന്നാണ്.)

എന്നാൽ ഈ പ്രവണതക്കു പിന്നിൽ ഒരു ഗുണകരമായ സവിശേഷതയും ഉണ്ട്: കാമചോദന കുറഞ്ഞതോ/ഇല്ലാത്തതോ ആയ ഈ സ്ത്രീകൾ ലൈംഗിക ബന്ധത്തിൽ ഏർപ്പെടുകയും അത് ആസ്വദിക്കുകയും ചെയ്താൽ, അവർ ലൈംഗികത ആഗ്രഹിക്കുന്നവരായി സാവകാശം പരിണമിക്കുന്നു.

( https://www.aarp.org/home-family/sex-intimacy/info-2014/sexual-desire-and-gender-castleman.html )

In 2000 and 2002, a University of British Columbia psychiatrist named Rosemary Basson interviewed hundreds of women who reported feeling “erotically neutral” at the start of sex. Only when they started making love — and enjoyed it — did they warm up and feel actual desire.

In the years since then, other researchers have corroborated her findings. In the largest study, a survey of 3,687 women, Portuguese sex researcher Ana Carvalheira found that women who said that sex preceded desire outnumbered those who reported desire first by a margin of 2 to 1. Today, sex therapists increasingly accept Basson’s view that for many (if not most) women, desire is not the cause of sex, but its result.

But if women don’t feel lust before sex, why do they make love? According to Basson, it’s not primarily to have orgasms, but rather to share intimacy. This means men become intimate to gain sex, whereas women have sex to gain intimacy.

2000-ലും 2002-ലും, റോസ്മേരി ബാസൺ എന്ന ബ്രിട്ടീഷ് കൊളംബിയയിലെ ഒരു സൈക്യാട്രിസ്റ്റ്, ലൈംഗിക ബന്ധത്തിന്റെ പ്രാരംഭ ഘട്ടം “ലൈംഗികമായ നിശ്ചലത” (erotically neutral) അനുഭവിച്ചതായി റിപ്പോർട്ട് ചെയ്ത നൂറുകണക്കിന് സ്ത്രീകളെ അഭിമുഖം നടത്തി. ലൈംഗിക കേളികൾ (പുരുഷ പ്രേരിതമായി) തുടങ്ങുകയും, അത് ആസ്വദിക്കുക്കുകയും ചെയ്യുമ്പോൾ അവർക്ക് ചൂടുപിടിക്കുകയും അതിനെ പിന്തുടർന്ന് യഥാർത്ഥ ലൈംഗിക തൃഷ്ണ അനുഭവപ്പെടുകയും ചെയ്യുകയാണുണ്ടായത്.

തുടർന്നുള്ള വർഷങ്ങളിൽ, മറ്റ് ഗവേഷകരും ഈ കണ്ടെത്തലുകൾ സ്ഥിരീകരിച്ചു. ഏറ്റവും വലിയ പഠനങ്ങളിൽ ഒന്നിൽ, 3,687 സ്ത്രീകളിൽ നടത്തിയ ഒരു സർവേയിൽ, ലൈംഗികതയ്ക്ക് മുമ്പുള്ള, ലൈംഗിക തൃഷ്ണ ഉണ്ടെന്ന് പറയുന്ന സ്ത്രീകളേക്കാൾ, ലൈംഗികബന്ധം ആരംഭിച്ചതിന് ശേഷമാണ് ലൈംഗിക തൃഷ്ണ അനുഭവിച്ചത് എന്ന് റിപ്പോർട്ട് ചെയ്തവരാണ് അധികവും. ഒന്നിന് രണ്ട് എന്ന മാർജിനിൽ വരെ കൂടുതലാണ് ആദ്യ വിഭാഗത്തേക്കാൾ രണ്ടാമത്തെ വിഭാഗം എന്നാണ് പോർച്ചുഗീസ് ലൈംഗിക ഗവേഷകയായ അന കാർവൽഹീറ കണ്ടെത്തിയത്. ഇന്ന്, സെക്‌സ് തെറാപ്പിസ്റ്റുകൾ കൂടുതലായി ബാസന്റെ വീക്ഷണത്തെ അംഗീകരിക്കുന്നു. സ്ത്രീകളിൽ, ലൈംഗിക തൃഷ്ണയൊ ആഗ്രഹമോ അല്ല ലൈംഗികതയുടെ കാരണം, മറിച്ച് ലൈംഗികതയുടെ ഫലമാണ് ലൈംഗിക തൃഷ്ണ.

എന്നാൽ ലൈംഗികബന്ധത്തിന് മുമ്പ് സ്ത്രീകൾക്ക് കാമവികാരമില്ലെങ്കിൽ, അവർ എന്തിനാണ് ലൈംഗികതയിൽ ഏർപ്പെടുന്നത്? ബാസന്റെ അഭിപ്രായത്തിൽ, അതിന്റെ പ്രാഥമിക കാരണം രതിമൂർച്ഛ (orgasm) നേടുകയല്ല, മറിച്ച് അടുപ്പം പങ്കിടുക (share intimacy) എന്നതാണ്. ഇതിനർത്ഥം പുരുഷന്മാർ ലൈംഗിക ബന്ധത്തിൽ ഏർപ്പെടാൻ വേണ്ടി അടുപ്പമുള്ളവരാകുമ്പോൾ, സ്ത്രീകൾ അടുപ്പം നേടുന്നതിനായി ലൈംഗിക ബന്ധത്തിൽ ഏർപ്പെടുന്നു എന്നാണ്.

( https://www.aarp.org/home-family/sex-intimacy/info-2014/sexual-desire-and-gender-castleman.html )

ഈ പഠനങ്ങളിൽ നിന്ന് മനസ്സിലാക്കാവുന്ന ശ്രദ്ധേയമായ രണ്ട് വിവരങ്ങൾ ഇവയാണ്:

1. ലൈംഗിക തൃഷ്ണയുടെ കാര്യത്തിൽ സ്ത്രീയും പുരുഷനും വളരെ വലിയ അന്തരം പുലർത്തുന്നു. പുരുഷൻ ധാരാളമായി ലൈംഗിക ബന്ധങ്ങൾക്കായി ആഗ്രഹിക്കുമ്പോൾ സ്ത്രീകൾ വളരെ കുറവൊ ഇല്ലായ്മയൊ അനുഭവിക്കുന്നു.

2. പുരുഷ ലൈംഗികതയുടെ രേഖ തൃഷ്ണയിലൂടെ തുടങ്ങി, രതിമൂർച്ചയിൽ അവസാനിക്കുന്നു. സ്ത്രീയുടെ ലൈംഗികത, തൃഷ്ണയിലൂടെ അല്ല -ഭൂരിഭാഗവും- ആരംഭിക്കുന്നത്. ലൈംഗികതയിൽ ഏർപ്പെടുന്നതിന്റെ ഫലമായാണ് ലൈംഗികതൃഷ്ണ ഉടലെടുക്കുന്നത്.

ഈ പ്രകൃതിയെ നിഷേധിച്ചു കൊണ്ട് സ്ത്രീ പുരുഷ സമത്വ സിദ്ധാന്തം വിളമ്പുന്ന സമൂഹങ്ങളിൽ പൊരുത്തമില്ലാത്ത ലൈംഗിക ചോദനകളാൽ (sex drive discrepancy: SDD) ഉണ്ടാവുന്ന ദാമ്പത്യ പിണക്കങ്ങളും പിരിമുറുക്കങ്ങളും പിരിയലുകളും അൽപ്പമൊന്നുമല്ല.

Every 42 seconds, there is one divorce in America, That equates to 86 divorces per hour, 2,046 divorces per day, 14,364 divorces per week, and 746,971 divorces per year.

ഓരോ 42 സെക്കൻഡിലും അമേരിക്കയിൽ ഒരു വിവാഹമോചനം നടക്കുന്നു, മണിക്കൂറിൽ 86 വിവാഹമോചനങ്ങൾ, പ്രതിദിനം 2,046 വിവാഹമോചനങ്ങൾ, ആഴ്ചയിൽ 14,364 വിവാഹമോചനങ്ങൾ, പ്രതിവർഷം 746,971 വിവാഹമോചനങ്ങൾ. ( https://www.wf-lawyers.com/divorce-statistics-and-facts/ )

When couples report high sexual satisfaction, this contributes 15%-20% to their overall relationship satisfaction. But couples’ low sexual satisfaction accounts for 50% to a whopping 70% of their total relationship satisfaction, or lack thereof (Girard & Wooley, 2017).

ദമ്പതികൾ ഉയർന്ന ലൈംഗിക സംതൃപ്തി റിപ്പോർട്ട് ചെയ്യുമ്പോൾ, ഇത് അവരുടെ മൊത്തത്തിലുള്ള ബന്ധത്തിന്റെ സംതൃപ്തിക്ക് 15%-20% സംഭാവന ചെയ്യുന്നു. എന്നാൽ ദമ്പതികളുടെ കുറഞ്ഞ ലൈംഗിക സംതൃപ്തി അവരുടെ മൊത്തം ബന്ധത്തിന്റെ അസംതൃപ്തിയുടെ 50% മുതൽ 70% വരെയോ, അല്ലെങ്കിൽ അതിന്റെ സംതൃപ്തിയുടെ അഭാവമൊ സൂചിപ്പിക്കുന്നു. (Girard & Wooley, 2017).

(www.psychologytoday.com/us/blog/intersections/201912)

ചില ദമ്പതികളാകട്ടെ ലൈംഗിക ശൂന്യമായ ദാമ്പത്യത്തിലേക്ക് പൂർണമായും തള്ളപ്പെടുന്നു (!!) : There are varying definitions of a sexless marriage or sexless relationship: no sex in the past year, no sex in the past six months or sex 10 or fewer times a year. According to one study, approximately 15 percent of married couples are sexless: Spouses haven’t had sex with each other in the past six months to one year.

( https://www.google.com/amp/s/www.nytimes.com/2018/03/10/style/sexless-relationships-men-low-libido.amp.html )

ലൈംഗികാഭിലാഷം കുറയുന്നതാണ് ലൈംഗിക പ്രശ്‌നങ്ങളുടെ ഏറ്റവും സാധാരണമായ സ്രോതസ് എന്നതിന് പുറമെ ഒരു ദാമ്പത്യത്തിലെ ബന്ധത്തിലെ പ്രണയം, കുറയുന്നതിന് കാരണമായി നിലകൊള്ളുന്നു എന്നതിനാൽ തന്നെ ലൈംഗികാഭിലാഷത്തിലെ പൊരുത്തക്കേട് ഏറ്റവും ഗുരുതരമായ ദാമ്പത്യ പ്രശ്നമായി കണക്കാക്കപ്പെടുന്നു. (Mark, 2015).

(www.psychologytoday.com/us/blog/intersections/201912)

അഥവാ പ്രണയശൂന്യമായ, നരകതുല്യമായ ദാമ്പത്യ ജീവിതമാണ്, ലൈംഗിക ശൂന്യമായ ദാമ്പത്യത്തിന്റെ മറുപുറം.

ഇത്തരം തീക്ഷ്ണമായ പരിണിതഫലങ്ങൾ ഒഴിവാക്കാൻ പല പരിഹാര മാർഗങ്ങൾ ലൈംഗിക ശാസ്ത്രവും മനശ്ശാസ്ത്രവും പഠന വിധേയമാക്കിയിട്ടുണ്ട്. അവയിൽ ഏറ്റവും പ്രഥമവും പ്രധാനവുമായ മാർഗം ഇണകൾ/ദമ്പതികളും പരസ്പരം അനുരഞ്ജന രതിക്ക് തയ്യാറാവുക എന്നതു തന്നെയാണ്.

Make compromises

Sometimes, some people are just not as sexual as their partners. A person with low libido can meet their partner halfway by still engaging in sex despite having a low sex drive.

A 2015 study found that partners with high communal strength or those who are motivated to care about and be more responsive to their partners reported enhanced sexual and relationship satisfaction.

However, couples willing to compromise are not restricted to sex. They can also consider alternatives.

In a 2020 studyTrusted Source on couples’ strategies for dealing with differences in sexual desire, masturbation is the most common strategy reported by participants.

Other alternatives to penetrative sex include oral sex, manual stimulation, and using sex toys on each other. Couples can also engage in activities that may trigger desire, such as watching intimate movies together.

പരസ്പര വിട്ടുവീഴ്ചകൾ ചെയ്യുക:

ചിലപ്പോൾ, ചിലർ അവരുടെ പങ്കാളികളെപ്പോലെ ലൈംഗിക അഭിലാഷമൊ ചോദനയൊ പുലർത്തുന്നവരാകില്ല. കുറവാണെങ്കിലും ലൈംഗികതയിൽ ഏർപ്പെടുന്നതിലൂടെ ലിബിഡോ കുറവുള്ള ഒരാൾക്ക് പങ്കാളിയുമായി പാതിവഴിയിൽ സന്ധിക്കാൻ സാധിക്കുന്നതാണ്.

ഉയർന്ന സാമൂഹ്യബദ്ധതയുള്ള, അല്ലെങ്കിൽ സ്വന്തം പങ്കാളികളോട് കൂടുതൽ പരിപാലന ക്ഷമതയും പ്രതികരണാത്മകതയും പുലർത്തുന്ന വ്യക്തികളിൽ മെച്ചപ്പെട്ട ലൈംഗിക ജീവിതവും, ബന്ധങ്ങളിൽ കൂടുതൽ സംതൃപ്തിയും റിപ്പോർട്ട് ചെയ്തതായി 2015 ലെ ഒരു പഠനം കണ്ടെത്തി.

( https://www.medicalnewstoday.com/articles/mismatched-sex-drives#management-techniques )

മനശാസ്ത്രജ്ഞർ മുന്നോട്ടു വച്ച ഈ പരിഹാരമാർഗം ആത്മീയമായ മാനത്തോടെ അവതരിപ്പിക്കുകയല്ലെ ഹദീസ് ചെയ്തത് ?! ഈ സ്ത്രീ വിരുദ്ധ – പുരുഷാധിപത്യ ആരോപണങ്ങൾക്കൊന്നും ഒരു കഴമ്പുമില്ലെന്നും സ്ത്രീ പുരുഷ ലൈംഗിതയുടേയും ലൈംഗിക തൃഷ്ണയുടേയും പ്രകൃതി വ്യതിരിക്തതയും അവ തമ്മിൽ ഇണക്കേണ്ടത് എങ്ങനെയാണെന്നും 1400 വർഷങ്ങൾക്ക് മുമ്പ് തിരിച്ചറിയുകയും പരിഹാരമാർഗ്ഗവും ഒപ്പം അതിന് വിശ്വാസിയായ സ്ത്രീക്കുള്ള ആത്മീയമായ പ്രേരണയും പ്രോത്സാഹനവും നൽകി മാർഗദർശനം അവതരിപ്പിച്ചു എന്നുള്ളതാണോ ഹദീസിലെ അപരാധം ?!

3. ലൈംഗിക തൃഷ്ണ എന്നത് സ്ത്രീകളുടെ കാര്യത്തിൽ ലൈംഗിക ബന്ധത്തിന്റെ അനന്തരഫലമാണെന്ന് പഠനങ്ങൾ സൂചിപ്പിച്ചുവല്ലൊ. സ്ത്രീ ലൈംഗികതയിൽ ഏർപ്പെടുന്നതിലൂടെയാണ് സാധാരണയായി ലൈംഗിക തൃഷ്ണ നേടിയെടുക്കുക. അത് ദമ്പതികൾക്കിടയിലെ ലൈംഗിക സംതൃപ്തിയിലേക്കും തുടർന്ന് ദാമ്പത്യ ജീവിതത്തിലെ മൊത്തം സംതൃപ്തിയിലേക്കും നയിക്കുന്നു. ഇവയെല്ലാം പരസ്പര ബന്ധിതമായ ദാമ്പത്യ ബിന്ദുക്കളാണ്. അപ്പോൾ ന്യായമായ കാരണങ്ങളൊന്നും കൂടാതെ, ലൈംഗിക തൃഷ്ണയൊ താൽപര്യമൊ അനുഭവപ്പെടുന്നില്ല എന്ന കാരണത്താൽ മാത്രം ഇണക്ക് രതി നിഷേധിക്കുന്ന ഒരു സ്ത്രീ പ്രണയ പൂർണമായ ഒരു ദാമ്പത്യ ജീവിതമാണ് സ്വയം നിരാകരിക്കുന്നത്.

ഹദീസിൽ മലക്കുകളുടെ എതിർപ്പിന് വിധേയയാവുന്ന സ്ത്രീ ഭർത്താവിന് ഈ രതി നിഷേധിക്കുക മാത്രമല്ല ചെയ്യുന്നത്. സ്ത്രീ പുരുഷ ലൈംഗിക തൃഷ്ണയെ സംബന്ധിച്ച അവളുടെ അജ്ഞതയൊ അവജ്ഞയൊ പ്രണയനിർഭരമായ ഒരു ദാമ്പത്യ ജീവിതത്തെ അസാധ്യമാക്കുന്നു. തന്റെ സാന്നിധ്യം പോലും നിഷേധിച്ചു കൊണ്ട് വിരിപ്പിലേക്ക് പോലും വരാതെ, പരിസരത്തു പോലും നിൽക്കാനുള്ള സഹിഷ്ണുത കാണിക്കാത്ത ഒരു സ്ത്രീയെ സംബന്ധിച്ചാണ് ഹദീസ് സംസാരിക്കുന്നത്:

إِذَا بَاتَتِ المَرْأَةُ، هَاجِرَةً فِرَاشَ زَوْجِهَا “ഒരു സ്ത്രീ, തന്റെ ഇണയുടെ വിരിപ്പ് ബഹിഷ്ക്കരിച്ച് രാത്രി മുഴുവൻ കഴിച്ചു കൂട്ടിയാൽ…” (സ്വഹീഹു മുസ്‌ലിം: 1436) എന്ന് ഹദീസിന്റെ പദങ്ങളിൽ വന്നിരിക്കുന്നു.

കേവലം ലൈംഗികവേഴ്‌ച (sexual intercourse) മാത്രമല്ല അവൾ ഭർത്താവിന് നിഷേധിക്കുന്നത് എന്ന് വ്യക്തം. ലൈംഗിക ചോദനയിലെ പൊരുത്തക്കേടിന് പരിഹാരമായി മനശ്ശാസ്ത്രജ്ഞർ നിർദേശിച്ച – ലൈംഗികവേഴ്‌ചക്ക് പകരമായ – (alternatives) ലൈംഗിക കേളികൾക്കും പ്രണയലീലകൾക്കും ഒന്നും ഹദീസിൽ പറയപ്പെട്ട സ്ത്രീ ഒരുക്കമല്ല എന്നർത്ഥം. ഇവിടെ പുരുഷൻ അവന്റെ പ്രകൃതിയനുസ്സരിച്ച് നിസ്സഹായനായി പരിണമിക്കുകയും ചെയ്യുന്ന ദുരവസ്ഥ പല ദാമ്പത്യങ്ങളിലും നിലനിൽക്കുന്നു. ഈ സവിശേഷ സാഹചര്യത്തെ പരിഗണിച്ചു കൊണ്ടുള്ള മാർഗ്ഗനിർദ്ദേശമാണ് ഹദീസിന്റെ ഉള്ളടക്കം.

3.സ്ത്രീകളിലാണ് ഈ രതി നിഷേധ പ്രവണത കൂടുതൽ എന്നതിനാലും പുരുഷന്മാരാണ് ലൈംഗിക തൃഷ്ണയുടെ കാര്യത്തിൽ പ്രകൃതിപരമായി മുന്തി നിൽക്കുന്നത് എന്നതിനാലുമാണ് സ്ത്രീകളെ ശക്തമായി പ്രോത്സാഹിപ്പിച്ചു കൊണ്ടുള്ള പ്രതിബാധന ശൈലി ഹദീസ് വന്നിരിക്കുന്നത്.

ﻭَﻓِﻴﻪِ ﺃَﻥَّ ﺻَﺒْﺮَ اﻟﺮﺟﻞ ﻋَﻠَﻰ ﺗَﺮْﻙِ اﻟْﺠِﻤَﺎﻉِ ﺃَﺿْﻌَﻒُ ﻣِﻦْ ﺻَﺒْﺮِ اﻟْﻤَﺮْﺃَﺓِ ﻗَﺎﻝَ ﻭَﻓِﻴﻪِ ﺃَﻥَّ ﺃَﻗْﻮَﻯ اﻟﺘَّﺸْﻮِﻳﺸَﺎﺕِ ﻋَﻠَﻰ اﻟﺮَّﺟُﻞِ ﺩَاﻋِﻴَﺔُ اﻟﻨِّﻜَﺎﺡِ ﻭَﻟِﺬَﻟِﻚَ ﺣَﺾَّ اﻟﺸَّﺎﺭِﻉُ اﻟﻨِّﺴَﺎءَ ﻋَﻠَﻰ ﻣُﺴَﺎﻋَﺪَﺓِ اﻟﺮِّﺟَﺎﻝِ ﻓِﻲ ﺫَﻟِﻚَ

“സ്ത്രീകളേക്കാൾ ലൈംഗിക ബന്ധം ഉപേക്ഷിക്കുന്ന കാര്യത്തിൽ ദുർബലർ പുരുഷന്മാരാണ്. ആസക്തിയാകട്ടെ സ്ത്രീകളേക്കാൾ പുരുഷന്മാർക്കാണ് പ്രബലം. അപ്പോൾ അക്കാര്യത്തിൽ പുരുഷന്മാരെ സഹായിക്കാനും അവരോട് കരുണ ചെയ്യാനും സ്ത്രീകൾക്ക് പ്രോത്സാഹനം നൽകപ്പെട്ടു.” (ഫത്ഹുൽ ബാരി: 9:294)

എന്നിരുന്നാലും രതി നിഷേധ പ്രവണത പുരുഷന്മാരിലും ഇസ്‌ലാം അനുവദിക്കുന്നില്ല എന്നതാണ് ശ്രദ്ധേയമായ മറ്റൊരു വസ്തുത.

പ്രവാചകൻ ചില ദിവസങ്ങൾ ഉറക്കമിളച്ച് നമസ്ക്കരിക്കുകയും ചില ദിവസങ്ങൾ ഭാര്യമാരുമൊത്ത് ഉറങ്ങുകയും ചെയ്യുമായിരുന്നു എന്ന് അറിഞ്ഞ ചില അനുചരർ വിവാഹം ചെയ്യില്ലെന്നും രാത്രി മുഴുവൻ ആരാധനാനിമഗ്നരായി കഴിയുമെന്നും ശപഥം ചെയ്യുകയുണ്ടായി. ഇതറിഞ്ഞ പ്രവാചകൻ (സ) കോപിഷ്ടനാവുകയും ഇത്തരം ബ്രഹ്മചര്യ പ്രവണതകളെയും വിരക്തിയെയും നിശിതമായി വിമർശിക്കുകയുണ്ടായി:

فمَن رغِب عن سنَّتي فليس منِّي

“എന്റെ ചര്യയെ ആരാണോ വെറുക്കുന്നത് അവൻ എന്നിൽ പെട്ടവനല്ല.” (സ്വഹീഹുൽ ബുഖാരി: 5063)

പുരുഷന് രതി നിഷേധിച്ച സ്ത്രീക്കെതിരെ മലക്കുകൾ പ്രാർത്ഥിക്കുമെന്നാണ് ഹദീസിൽ സൂചിപ്പിക്കപ്പെട്ടതെങ്കിൽ, സ്ത്രീക്ക് രതി നിഷേധിക്കുന്ന പുരുഷന്മാർ “എന്നിൽ പെട്ടവനല്ല” എന്ന്, കൂടുതൽ ഗുരുതരമായ പദപ്രയോഗമാണ് ഹദീസ് നടത്തിയിരിക്കുന്നത് എന്നത് ഈ ‘സ്ത്രീ പക്ഷ വായകർ’ ആരും ശ്രദ്ധിച്ചിട്ടില്ല !!

ﺃَﻥَّ ﺳَﻠْﻤَﺎﻥَ اﻟْﻔَﺎﺭِﺳِﻲَّ ﻗَﺎﻝَ ﻷَِﺑِﻲ اﻟﺪَّﺭْﺩَاءِ ﺇﻥْ ﻟِﺠَﺴَﺪِﻙَ ﻋَﻠَﻴْﻚَ ﺣَﻘًّﺎ، ﻭَﺇِﻥَّ ﻷَِﻫْﻠِﻚَ ﻋَﻠَﻴْﻚ ﺣَﻘًّﺎ، ﺃَﻋْﻂِ ﻛُﻞَّ ﺫِﻱ ﺣَﻖٍّ ﺣَﻘَّﻪُ: ﺻُﻢْ، ﻭَﺃَﻓْﻄِﺮْ، ﻭَﻗُﻢْ، ﻭَﻧَﻢْ، ﻭَﺃْﺕِ ﺃَﻫْﻠَﻚَ ﻓَﺄَﺧْﺒَﺮَ ﺃَﺑُﻮ اﻟﺪَّﺭْﺩَاءِ ﺑِﺬَﻟِﻚَ ﺭَﺳُﻮﻝَ اﻟﻠَّﻪِ – ﺻَﻠَّﻰ اﻟﻠَّﻪُ ﻋَﻠَﻴْﻪِ ﻭَﺳَﻠَّﻢَ – ﻓَﻘَﺎﻝَ ﻟَﻪُ ﺭَﺳُﻮﻝُ اﻟﻠَّﻪِ – ﺻَﻠَّﻰ اﻟﻠَّﻪُ ﻋَﻠَﻴْﻪِ ﻭَﺳَﻠَّﻢَ – ﻣِﺜْﻞَ ﻗَﻮْﻝِ ﺳَﻠْﻤَﺎﻥَ» .

തന്റെ ഭാര്യയിലും ലൈംഗികതയിലും വിരക്തനായി ആരാധനകളിൽ മാത്രം മുഴുകിയ അബൂദർദാഇനോട് സൽമാനുൽ ഫാരിസി ഇപ്രകാരം ഉപദേശിക്കുകയുണ്ടായി: “തീർച്ചയായും നിന്റെ ശരീരത്തിന് നിന്റെ മേൽ അവകാശമുണ്ട്, തീർച്ചയായും നിന്റെ ഭാര്യക്ക് നിന്റെ മേൽ അവകാശമുണ്ട്. ഓരോ അവകാശിക്കും അവരുടെ അവകാശങ്ങൾ വകവെച്ച് നൽകുക. അതുകൊണ്ട് നീ നോമ്പ് നോൽക്കുകയും ചില ദിവസങ്ങൾ നോമ്പ് നോൽക്കാതിരിക്കുകയും ചെയ്യുക. നീ രാത്രി നമസ്ക്കരിക്കുക. (ചില ദിവസങ്ങൾ ഭാര്യയോടൊന്നിച്ച്) ഉറങ്ങുകയും ചെയ്യുക.” ഇത് കേട്ട അബൂദർദാഅ് അല്ലാഹുവിന്റെ ദൂതനരികിൽ ചെന്ന് സൽമാനുൽ ഫാരിസിയുടെ ഉപദേശത്തെ സംബന്ധിച്ച് അറിയിച്ചു. അപ്പോൾ അല്ലാഹുവിന്റെ ദൂതനും അതു തന്നെ അബൂദർദാഇനെ ഉപദേശിച്ചു. (അൽ മുഹല്ല: 9:175)

4. “തന്നിമ്മിത്തം ഭർത്താവ് രാത്രി മുഴുവൻ അവളോട് കോപിച്ചവളായി കഴിച്ചു കൂട്ടുകയും ചെയ്തു…” എന്ന വാചകത്തിന്റെ വെളിച്ചത്തിൽ നിന്നും അല്ലാതെയും കാരണങ്ങളൊന്നും കൂടാതെ ഭാര്യമാർ ലൈംഗികത നിഷേധിക്കുന്നതാണ് ഇവിടെ ഉദ്ദേശം എന്ന് ഹദീസ് വിശാരദർ സകലരും വ്യക്തമാക്കിയിട്ടുണ്ട്.” ( ഫത്ഹുൽ ബാരി: 9:294, ഔനുൽ മഅ്ബൂദ്: ഇബ്നുൽ കയ്യിം: 6:126, ഫൈദുൽ കദീർ: 1:344 )

കാരണം, ന്യായമായ കാരണങ്ങളോടെയാണ് ലൈംഗിക ബന്ധത്തിൽ നിന്ന് ഭാര്യ വിട്ടു നിൽക്കുന്നതെങ്കിൽ “തന്നിമ്മിത്തം ഭർത്താവ് രാത്രി മുഴുവൻ അവളോട് കോപിച്ചവളായി കഴിച്ചു കൂട്ടുക…” യില്ലല്ലൊ.

ന്യായമായ കാരണങ്ങളൊന്നും കൂടാതെ ലൈംഗിക ബന്ധത്തിൽ നിന്ന് വിട്ടു നിൽക്കുന്ന സാഹചര്യത്തിലാണ് സ്ത്രീ വിമർശന വിധേയയാവുന്നത് എന്ന് ഈ ഹദീസിന്റെ സൂചനകളുടെ വെളിച്ചത്തിൽ ഇമാം ദഹബി, റാഫിഈ, നവവി, ഇബ്നുർ രിഫ്അ, ഹൈതമി എന്നിങ്ങനെ ഒട്ടനവധി ഹദീസ് പണ്ഡിതർ വ്യക്തമാക്കുന്നുമുണ്ട്… (മൗസൂഅതുൽ ഫിക്ഹിയ്യ അൽ കുവൈതിയ്യ: 38:44)

(ﻓﻠﻢ ﺗﺄﺗﻪ) ﻧ ﻏﻴﺮ ﻋﺬﺭ ﺑﻬﺎ ﺗﺎﺭﻛﺔ (ﻓﺮاﺵ ﺯﻭﺟﻬﺎ) ﺑﻐﻴﺮ ﻣﺎﻧﻊ ﻣﻦ ﻣﺮﺽ ﺃﻭ اﻣﺘﻨﺎﻉ ﻟﺘﺴﻠﻢ ﺻﺪاﻕ ﺣﺎﻝ ﻋﻘﺪﺕ ﻋﻠﻴﻪ

“രോഗം, ഭർത്താവിന്റെ കൃത്യവിലോപം തുടങ്ങി മതപരമായ മറ്റു കാരണങ്ങൾ ഒന്നും കൂടാതെ രതി നിഷേധിക്കുമ്പോഴാണ് സ്ത്രീ കുറ്റക്കാരിയായി മാറുന്നത്” എന്നും അവർ വ്യക്തമാക്കി. (ദലീലുൽ ഫാലിഹീൻ ലിത്വുറുക്വി റിയാദുസ്വാലിഹീൻ: 3:110)

കുറിപ്പുകൾ

1. ഹദീസിലെ (لَعَنَتْهَا المَلَائِكَةُ) എന്ന വാചകം ശ്രദ്ധ അർഹിക്കുന്നുണ്ട്. “മലക്കുകൾ അവളെ ശപിക്കും” എന്ന് പരിഭാഷ ചെയ്യപ്പെടാറുണ്ടെങ്കിലും അറബി ഭാഷയിൽ ‘ലഅനത്ത്’ (لعنة) എന്ന പദത്തിന്റെ യഥാർത്ഥ അർത്ഥമെന്താണെന്ന് നാം മനസ്സിലാക്കണം. പരിഭാഷാർത്ഥം (ശാപം) പൂർണമായും ആ പദത്തിനെ ഉൾക്കൊള്ളുന്നില്ല. ‘ലഅനത്ത്’ (لعنة) എന്ന പദത്തിന്റെ വിവക്ഷ “ദൈവ കാരുണ്യത്തിൽ നിന്ന് അകലുക” (الطرد من رحمة الله) എന്നാണ്. (അനീസുൽ ഫുകഹാ: 162, കശ്ശാഫ്: 2:577, അൽകുല്ലിയാത്ത്: 1278)

അപ്പോൾ “മലക്കുകൾ അവളെ ‘ലഅനത്ത്’ (لعنة) ചെയ്യും” എന്നാൽ “ദൈവ കാരുണ്യത്തിൽ നിന്ന് അവൾ അകലട്ടെ” എന്ന് മലക്കുകൾ പ്രാർത്ഥിക്കും എന്നർത്ഥം. തനിക്ക് ആസക്തിയൊ ഭാവ നിലയൊ ഇല്ലാതെ ലൈംഗിക ബന്ധത്തിന് ഒരുക്കമല്ലെന്ന് ശഠിക്കുകയും ഭർത്താവിന്റെ പുരുഷ പ്രകൃതി പരിഗണിക്കാതിരിക്കുകയും യാതൊരു നിലയിലും അയാളോട് ‘കരുണ’ കാണിക്കാതിരിക്കുകയും ചെയ്യുന്ന ഒരു സ്ത്രീക്ക് ദൈവവും ‘കരുണ’ കാണിക്കരുതേ എന്ന് പ്രാർത്ഥിക്കുകയാണ് ഇവിടെ മലക്കുകൾ ചെയ്യുന്നത്. ഇതിൽ എവിടെയാണ് ഗാർഹീക പീഢനവും ദാമ്പത്യ ബലാൽസംഘവും ?!

പ്രാർത്ഥനയൊക്കെ പീഢനത്തിന്റെ ഒരു രൂപമായി നാസ്തികർ അവകാശപ്പെടുന്നത് വലിയ കോമഡിയാണ്. അതും, പ്രാർത്ഥിക്കുന്നതാവട്ടെ ഭർത്താവല്ല, മറിച്ച് ആത്മീയ ജീവികളായ മലക്കുകളാണ്. ആത്മാവിലും അഭൗതികതകളിലും വിശ്വാസമില്ലെങ്കിലും മലക്കുകളുടെ ന്യായമായ ഈ പ്രാർത്ഥന നാസ്തികർക്ക് പീഢനമുറയാണ്.

വിമർശനം:

അടിമ സ്ത്രീകളെ വിവസ്ത്രരാക്കാനും ഇഷ്ടാനുസാരം ശരീരാവയവങ്ങൾ സ്പർശിക്കുവാനും ഇസ്‌ലാം അനുവദിച്ചുവെന്ന് ഹദീസുകളിൽ ഇല്ലേ ?

മറുപടി:

അടിമ സ്ത്രീകളെ വേശ്യാവൃത്തിക്കും അശ്ലീലതകൾക്കും വിധേയരാക്കിയിരുന്ന ഒരു കാലഘട്ടത്തിൽ ലൈംഗിക ശുദ്ധി കാത്തുസൂക്ഷിക്കാനുള്ള അവരുടെ അവകാശത്തിനായി ഘോരമായി ശബ്ദിക്കുകയും നിയമങ്ങൾ ആവിഷ്കരിക്കുകയും ചെയ്ത മതമാണ് പരിശുദ്ധ ഇസ്‌ലാം.

“നിങ്ങളുടെ അടിമസ്ത്രീകള്‍ ചാരിത്രശുദ്ധിയോടെ ജീവിക്കാന്‍ അഗ്രഹിക്കുന്നുണ്ടെങ്കില്‍ ഐഹികജീവിതത്തിന്‍റെ വിഭവം ആഗ്രഹിച്ചു കൊണ്ട് നിങ്ങള്‍ അവരെ വേശ്യാവൃത്തിക്ക് നിര്‍ബന്ധിക്കരുത്‌…” (ഖുർആൻ: 24:33)

അന്യ സ്ത്രീകളെ ഇഷ്ടാനുസാരം വിവസ്ത്രരാക്കാനും ശരീരാവയവങ്ങൾ സ്പർശിക്കുവാനും പോയിട്ട് ഒന്ന് തൊടുന്നത് പോലും ഇസ്‌ലാം വിലക്കിയിട്ടുണ്ട്.

لَأنْ يُطعَنَ في رأسِ رجلٍ بِمِخْيَطٍ من حديدٍ خيرٌ من أن يمَسَّ امرأةً لا تَحِلُّ له

“നിന്റെ ശിരസ്സിൽ ഒരു ഇരുമ്പിന്റെ ആണികൊണ്ട് കുത്തി തറക്കുന്നതാണ് നിനക്ക് അനുവദനീയമല്ലാത്ത ഒരു സ്ത്രീയെ സ്പർശിക്കുന്നതിനേക്കാൾ നിനക്ക് നല്ലത്.” (ത്വബ്റാനി: 487, മുസ്നദു റുയാനി: 1283) എന്നാണ് മുഹമ്മദ് നബി (സ) മുസ്‌ലിംകളെ പഠിപ്പിച്ചത്.

ﻗَﺎﻝَ ﺭَﺳُﻮﻝُ اﻟﻠَّﻪِ ﺻَﻠَّﻰ اﻟﻠﻪُ ﻋَﻠَﻴْﻪِ ﻭَﺳَﻠَّﻢَ: ﻻَ ﺗَﺤْﻤِﻠُﻮا اﻟﻨِّﺴَﺎءَ ﻋَﻠَﻰ ﻣَﺎ ﻳَﻜْﺮَﻫْﻦَ

“സ്ത്രീകളെ അവർ വെറുക്കുന്നത് ചെയ്യാൻ നിങ്ങൾ നിർബന്ധിക്കരുത്.” (മുസ്വന്നഫ് അബ്ദുർ റസാഖ്: 10320) എന്നും മുഹമ്മദ് നബി (സ) പഠിപ്പിച്ചിട്ടുണ്ട്.

ഇക്കാര്യങ്ങളിലെല്ലാം സ്വതന്ത്ര സ്ത്രീകളും അടിമ സ്ത്രീകളും സമമാണ്.

“സ്വതന്ത്ര സ്ത്രീകളുടെ കാര്യത്തിൽ നിഷിദ്ധമായവ അടിമ സ്ത്രീകളുടെ വിഷയത്തിലും നിഷിദ്ധമാണ്” എന്നാണ് പ്രവാചകാനുചരന്മാർ (റ) മനസ്സിലാക്കിയിട്ടുള്ളതും. (അൽ ഉമ്മ്: ഇമാം ശാഫിഈ: 5:3)

കാര്യങ്ങൾ ഇങ്ങനെയൊക്കെ ആണെങ്കിലും, ഇസ്‌ലാമിലെ പ്രമാണങ്ങളായ ക്വുർആനും സ്വഹീഹായ ഹദീസുകളും മറച്ചു പിടിച്ച് പകരം കുറേ കള്ള കഥകളും ദുർബല നിവേദനങ്ങളും സോഷ്യൽ മീഡിയകളിലൂടെ പ്രചരിപ്പിക്കുന്ന ഉദ്യമത്തിൽ വിരാജിക്കുകയാണ് ഇസ്‌ലാമോഫോബിയക്കാർ.

അടിമ സ്ത്രീകളെ വിവസ്ത്രരാക്കാനും ഇഷ്ടാനുസാരം ശരീരാവയവങ്ങൾ സ്പർശിക്കുവാനും ഇസ്‌ലാം അനുവദിച്ചുവെന്ന് വരുത്തി തീർക്കാൻ ഉദ്ധരിക്കുന്ന നിവേദനങ്ങളാകട്ടെ സാങ്കേതികമായി ഹദീസുകൾ പോലുമല്ല !! ‘അറബി കിതാബു’കളിൽ ഉള്ളതെല്ലാം ഇസ്‌ലാമാണെന്നും മുസ്‌ലിംകൾക്കിടയിൽ അറബിയിലെ വരികൾക്കെല്ലാം ആത്മീയ പ്രാധാന്യമുണ്ട് എന്നും വിശ്വസിക്കുന്ന നിലയിലേക്ക് കൂപ്പ് കുത്തി പോയോ ഇസ്‌ലാം – നാസ്തികത സംവാദങ്ങൾ?!! ഗവേഷണാത്മകതയുടെയും വൈജ്ഞാനിക ധർമ്മത്തിന്റെയും ഒരു കണിക പോലും നിങ്ങളുടെ ഇസ്‌ലാം അവലോകനത്തിൽ ബാക്കി ഇല്ലാതായി പോയോ?

പ്രതിപക്ഷ മര്യാദ അന്യംനിന്നു പോയിട്ടില്ലാത്തവർക്ക് വേണ്ടി ഇസ്‌ലാമിന്റെ രണ്ട് അടിസ്ഥാന തത്ത്വങ്ങൾ അടങ്ങുന്ന ഒരു ലേഖനം ഇവിടെ ചേർത്തു വെക്കട്ടെ:

വിമർശകർ പ്രചരിപ്പിച്ചു കൊണ്ടിരിക്കുന്ന നിവേദനങ്ങളുടെ റെഫറൻസും നിവേദക പരമ്പരകളും ആദ്യം ചർച്ചാവിധേയമാക്കാം:

പരമ്പര: 1 മുസ്വന്നഫ് അബ്ദുർറസാക്: 13208

13208 – عبد الرزاق عن بن جريج قال أخبرنى من أصدق عمن سمع عليا يسأل عن الأمة تباع أينظر إلى ساقها وعجزها وإلى بطنها قال لا بأس بذلك لا حرمة لها إنما وقفت لنساومها

അടിമ സ്ത്രീയെ വാങ്ങുമ്പോൾ അവളുടെ തണ്ടങ്കാലും അരയും നോക്കാം എന്ന് അലി (റ) അനുവാദം നൽകിയതായ നിവേദനം…

പരമ്പരയിലെ ഇബ്നു ജുറൈജ് ഉദ്ധരിക്കുന്നത് ഒരു ‘മജ്ഹൂൽ’ (അജ്ഞാതൻ) ൽ നിന്നാണ് ആ ‘മജ്ഹൂൽ’ ഉദ്ധരിക്കുന്നത് മറ്റൊരു ‘മജ്ഹൂൽ’ (അജ്ഞാതൻ) ൽ നിന്നും.!!! പരമ്പര വളരെ ദുർബലം.

പരമ്പര: 2 മുസ്വന്നഫ് അബ്ദുർറസാക്: 13206

13206 – عبد الرزاق عن بن جريج عن رجل عن بن المسيب أنه قال يحل له أن ينظر إلى كل شيء فيها ما عدا فرجها

താബിഈ പണ്ഡിതനായ (ഒരു പണ്ഡിതന്റെ അഭിപ്രായം !) സഈദിബ്നുൽ മുസ്വയ്യിബിലേക്ക് ചേർക്കപ്പെടുന്ന മറ്റൊരു നിവേദനം…

പരമ്പരയിൽ ഇബ്നു ജുറൈജ് ഉദ്ധരിക്കുന്നത് ഒരു ‘മജ്ഹൂൽ’ (അജ്ഞാതൻ) ൽ നിന്നാണ്. പരമ്പര അങ്ങേയറ്റം ദുർബലം.

പരമ്പര: 3 മുസ്വന്നഫ് അബ്ദുർറസാക്: 13207

13207 – عبد الرزاق عن الثوري عن جابر عن الشعبي قال إذا كان الرجل يبتاع الأمة فإنه ينظر إلى كلها إلا الفرج

താബിഈ പണ്ഡിതനായ (ഒരു പണ്ഡിതന്റെ അഭിപ്രായം !) ശുഅ്ബിയിലേക്ക് ചേർക്കപ്പെട്ട അഭിപ്രായം…

പരമ്പരയിലെ ജാബിർ അൽ ജഅ്ദി ശീഈയും നുണയനായി അരോപിക്കപ്പെട്ട വ്യക്തിയുമാണെന്ന് ഒട്ടനവധി ഹദീസ് വിശാരദർ വ്യക്തമാക്കിയിട്ടുണ്ട്. (തഹ്ദീബുൽ കമാൽ: മിസ്സി: 4:468)

പരമ്പര: 4 മുസ്വന്നഫ് ഇബ്നു അബീ ശൈബ: 2662 (29)

نا علي بن مسهر عن عبيد الله عن نافع عن ابن عمرأنه كان إذا أراد أن يشتري الجارية وضع يده على أليتيها أو بين فخذها وربما كشف عن ساقيها

ഇബ്നു ഉമറിലേക്ക് ചേർത്തി ഉദ്ധരിക്കപ്പെട്ട സമാനമായ ഒരു നിവേദനം…

പരമ്പരയിൽ ‘അലിയ്യുബ്നു മുസ്ഹിർ’ എന്ന റാവി അന്ധത ബാധിച്ചതിനു ശേഷം ധാരാളം ഒറ്റപ്പെട്ട, അസ്വീകാര്യമായ നിവേദനങ്ങൾ ഉദ്ധരിക്കുമായിരുന്നു എന്ന് വിമർശിക്കപ്പെട്ടിട്ടുണ്ട്. (തക്’രീബു തഹ്ദീബ്: 1:703)

പരമ്പര: 5 മുസ്വന്നഫ് അബ്ദുർറസാക്: 13200

13200 – عبد الرزاق عن عبد الله بن عمر عن نافع عن ابن عمر، ومعمر عن أيوب عن نافع عن ابن عمر، كان إذا أراد أن يشتري جارية، فراضاهم على ثمن، وضع يده على عجزها، وينظر إلى ساقيها، وقبلها، يعني بطنها

മൂന്ന് പരമ്പരകൾ ഈ നിവേദനത്തിൽ സമന്വയിപ്പിക്കപ്പെട്ടിട്ടുണ്ട്.

ഒന്നാമത്തെ പരമ്പര: عَنْ عَبْدِ اللَّهِ بْنِ عُمَرَ ، عَنْ نَافِعٍ ، عَنِ ابْنِ عُمَرَ

ഈ പരമ്പരയിൽ സ്മരിക്കപ്പെടുന്ന ഇബ്നു ഉമർ, അബ്ദുല്ലാഹിബ്നു ഉമർ എന്ന പ്രവാചക അനുചരനല്ല. അബ്ദുല്ലാഹിബ്നു ഉമർ ബിൻ ഹഫ്സ് എന്ന പിൻ തലമുറക്കാരനായ ഒരു വ്യക്തിയാണ്. (പ്രവാചകാനുചരനായ ഇബ്നു ഉമറിലേക്ക് ചേർത്തു കൊണ്ട് ഇത്തരമൊരു ആരോപണം ഉന്നയിക്കപ്പെട്ടിട്ടുണ്ട്. ആ പരമ്പരയെ സംബന്ധിച്ച ചർച്ച തുടർന്ന് വരുന്നുണ്ട് എന്ന് സാന്ദർഭികമായി സൂചിപ്പിക്കട്ടെ). ഇദ്ദേഹം ദുർബലനാണെന്നും ഇദ്ദേഹത്തിൽ നിന്ന് ഹദീസ് ഉദ്ധരിച്ചു കൂട എന്നും ഇബ്നുൽ മദീനി, യഹ്‌യൽ കത്വാൻ തുടങ്ങി പണ്ഡിതർ സൂചിപ്പിച്ചിട്ടുണ്ട്. (സിയറു അഅ്ലാമിന്നുബലാഅ്: 7: 340)

രണ്ടാമത്തെ പരമ്പര: وَمَعْمَرٍ ، عَنْ أَيُّوبَ ، عَنْ نَافِعٍ ، عَنِ ابْنِ عُمَرَ

പരമ്പരയിലെ മഅ്മർ ഇറാഖുകാരിൽ നിന്ന് ഉദ്ധരിക്കുന്നത് ‘ദഈഫ്’ ദുർബലമാണെന്ന് ഹദീസ് നിരൂപണ ശാസ്ത്ര പണ്ഡിതർ വ്യക്തമാക്കിയിട്ടുണ്ട്. ( ശർഹു ഇലലു തുർമുദി: ഇബ്നു റജബ്: 2: 774)

മഅ്മർ ‘ഇറാഖു’കാരനായ ‘അയ്യൂബ് അസ്സഖ്തിയാനി’യിൽ നിന്ന് (സിയറു അഅ്ലാമിന്നുബലാഅ്: 6:16) ഉദ്ധരിക്കുന്നതായാണ് പരമ്പര. അതിനാൽ തന്നെ ദുർബലവും.

മൂന്നാമത്തെ നിവേദനം: عَنْ مَعْمَرٍ ، عَنِ الزُّهْرِيِّ ، عَنْ سَالِمٍ ، عَنِ ابْنِ عُمَرَ مِثْلَهُ

ഈ പരമ്പരയിൽ രണ്ട് ന്യൂനതകൾ ഉണ്ട്. ഒന്നാമതായി പരമ്പരയുടെ ഉള്ളടക്കം എന്താണെന്ന് കൃത്യമായി ഉദ്ധരിക്കപ്പെട്ടിട്ടില്ല. രണ്ടാമതായി, പരമ്പരയിലെ റാവിയായ ഇമാം സുഹ്‌രി, നേരിട്ട് കേട്ടു എന്ന് വ്യക്തമായി സൂചിപ്പിക്കുന്ന പദം ഉപയോഗിക്കാത്തതിനാൽ സനദ് ‘മുദല്ലസ്’ ആവാൻ സാധ്യത നിലനിൽക്കുന്നു. (ത്വബക്കാത്തുൽ മുദല്ലിസീൻ: ഇബ്നു ഹജർ: 45). അതിനാൽ തന്നെ പരമ്പര ദുർബലമായി (ദഈഫ്) തീരുന്നു.

പരമ്പര: 6 മുസ്വന്നഫ് അബ്ദുർറസാക് : 13205 13205 – عبد الرزاق عن بن جريج عن نافع أن بن عمر كان يكشف عن ظهرها وبطنها وساقها ويضع يده على عجزها

പരമ്പരയിലെ ‘ഇബ്നു ജുറൈജ്’ നേരിട്ട് കേട്ടു എന്ന് വ്യക്തമായി സൂചിപ്പിക്കുന്ന പദം ഉപയോഗിക്കാത്തതിനാൽ പരമ്പര ദഈഫ് (ദുർബലം) ആകുന്നു.

‘ഇബ്നു ജുറൈജ്’ നേരിട്ട് കേട്ടു എന്ന് വ്യക്തമായി സൂചിപ്പിക്കാത്ത നിവേദനങ്ങൾ അങ്ങേയറ്റം ദുർബലവും വ്യർത്ഥവുമാണെന്ന് ഹദീസ് നിദാന ശാസ്ത്രം സൂചിപ്പിക്കുന്നു. (സിയറു അഅ്ലാമിന്നുബലാഅ്: 6: 328, തഹ്ദീബു തഹ്ദീബ്: 6/405 )

പരമ്പര: 7 മുസ്വന്നഫ് അബ്ദുർ റസാക് : 13198 13198 -عبد الرزاق عن بن جريج عن عطاء قال قلت له الرجل يشتري الأمة أينظر إلى ساقيها وقد حاضت أو إلى بطنها قال نعم قال عطاء كان بن عمر يضع يده بين ثدييها وينظر إلى بطنها وينظر إلى ساقيها أو يأمر به

അടിമ സ്ത്രീയെ വാങ്ങുമ്പോൾ അവളുടെ തണ്ടങ്കാലും അരയും നോക്കാം എന്ന്, ഇബ്നു ജുറൈജ് അത്വാഅ് എന്ന താബീഈ പണ്ഡിതനിലേക്ക് ചേർത്തു കൊണ്ട് ഉദ്ധരിക്കപ്പെടുന്ന നിവേദനം…

ഇബ്നു ജുറൈജ് എന്ന റാവി അത്വാഅ് ൽ നിന്ന് ഉദ്ധരിക്കുന്ന നിവേദനങ്ങൾ അങ്ങേയറ്റം ദുർബലമാണ് എന്ന് കാര്യകാരണ സഹിതം ഹദീസ് വിശാരദർ വ്യക്തമാക്കിയിട്ടുണ്ട്. (തഹ്ദീബു തഹ്ദീബ്: 6/406)

പരമ്പര: 8 മുസ്വന്നഫ് അബ്ദുർ റസാക് : 13199 13199 – أخبرنا عبد الرزاق قال أخبرنا بن جريج قال أخبرني عمرو أو أبو الزبير عن بن عمر أنه وجد تجارا مجتمعين على أمة فكشف عن بعض ساقها ووضع يده على بطنها

പരമ്പരയിലെ ഇബ്നു ജുറൈജ്, തന്നോട് നിവേദനം ഉദ്ധരിച്ചു കേൾപ്പിച്ചത് “അംറ് (അംറിബ്നു ദീനാർ) അല്ലെങ്കിൽ അബു സ്സുബൈർ ആകുന്നു” എന്നാണ്. അഥവാ ആരിൽ നിന്നാണ് ഈ നിവേദനം ഉദ്ധരിക്കുന്നതെന്ന് വ്യക്തമായി പ്രസ്ഥാവിക്കുന്നില്ല എന്നർത്ഥം. മാത്രമല്ല അബു സ്സുബൈർ എന്ന റാവിയെ സംബന്ധിച്ച് ഒരുപാട് ഭിന്നാഭിപ്രായങ്ങൾ നിലനിൽക്കുന്നു. പല പണ്ഡിതരും അദ്ദേഹം ദുർബലനാണ് എന്ന് സൂചിപ്പിച്ചിട്ടുണ്ട്. (തഹ്ദീബുത്തഹ്ദീബ്: 9: 391)

അബു സ്സുബൈർ നേരിട്ട് കേട്ടു എന്ന് വ്യക്തമായി സൂചിപ്പിക്കാത്ത നിവേദനങ്ങൾ അങ്ങേയറ്റം ദുർബലമാണെന്നും ഹദീസ് നിദാന ശാസ്ത്രം സൂചിപ്പിക്കുന്നുണ്ട്. (ത്വബകാതുൽ മുദല്ലിസീൻ: 13)

പരമ്പര: 9 സുനനുൽ കുബ്റാ: ബൈഹക്വി: 10513

أنَّ ابنَ عمرَ كان يضعُ يدَهُ بيْنَ ثَديَيها ( يعنى الجاريةَ ) وعلى عُجُزِها من فوقِ الثيابِ ويكَشفُ عن ساقِها

ഇബ്നു ഉമർ (റ) തന്റെ കൈ അടിമ പെൺകുട്ടിയുടെ മാറിടത്തിനിടയിലും അരയിലും വസ്ത്രത്തിന് മുകളിലൂടെ കൈ വെച്ച് നോക്കുമായിരുന്നു, അവളുടെ തണ്ടം കാലും വെളിവാക്കി നോക്കുകയും ചെയ്യുമായിരുന്നു. (സുനനുൽ കുബ്റാ: ബൈഹക്വി: 10513)

أخبرنا أبو الحسين بن بشران العدل ببغداد ، أنا إسماعيل بن محمد الصفار، ثنا الحسن بن علي بن عفان، ثنا ابن نمير عن عبيد الله بن عمر عن نافع عن ابن عمر

പരമ്പരയിൽ ‘ഉബൈദുല്ലാഹിബ്നു ഉമർ’ എന്ന ‘റാവി’യിൽ (നിവേദകൻ) നിന്ന് ഉദ്ധരിക്കുന്നത് ‘ഇബ്നു നുമൈർ’ എന്ന ‘റാവി’യാണ് (നിവേദകൻ). ‘ഇബ്നു നുമൈർ’ എന്ന പേര് മാത്രം ഉദ്ധരിക്കപ്പെടുമ്പോൾ സാധാരണ ഗതിയിൽ ഹദീസ് പണ്ഡിതർ ഉദ്ദേശിക്കാറുള്ളത് ‘മുഹമ്മദ് ഇബ്നു അബ്ദുല്ലാഹിബ്നു നുമൈർ’ ആണ്.

ഉദാഹരണത്തിന് ഇമാം ബൈഹക്വി തന്നെ ‘അയ്യാമുത്തശ്‌രീകി’നെ സംബന്ധിച്ച ഹദീസിനെ സംബന്ധിച്ച് വിവരിക്കവെ ഇപ്രകാരം എഴുതുകയുണ്ടായി: رواه مسلم في الصحيح عن ابن نمير. “ഈ ഹദീസ് ഇമാം മുസ്‌ലിം തന്റെ സ്വഹീഹിൽ ‘ഇബ്നു നുമൈറി’ൽ നിന്ന് ഉദ്ധരിക്കുന്നുണ്ട്… ” (മഅ്’രിഫതു സ്സുനനു വൽ ആസാർ: ബൈഹക്വി: 3: 439. നമ്പർ: 2599)

സ്വഹീഹു മുസ്‌ലിമിൽ ഈ ഹദീസ് ഉദ്ധരിച്ച ‘ഇബ്നു നുമൈർ’, ‘മുഹമ്മദിബ്നു അബ്ദുല്ലാഹിബ്നു നുമൈർ’ ആണ്.

1141 وحدثنا سريج بن يونس حدثنا هشيم أخبرنا خالد عن أبي المليح عن نبيشة الهذلي قال قال رسول الله صلى الله عليه وسلم أيام التشريق أيام أكل وشرب حدثنا محمد بن عبد الله بن نمير حدثنا إسمعيل يعني ابن علية عن خالد الحذاء حدثني أبو قلابة عن أبي المليح عن نبيشة قال خالد فلقيت أبا المليح فسألته فحدثني به فذكر عن النبي صلى الله عليه وسلم بمثل حديث هشيم وزاد فيه وذكر لله

(സ്വഹീഹു മുസ്‌ലിം: 1141)

ولد سنة نيف وستين ومائة قال البخاري : مات في شعبان أو رمضان سنة أربع وثلاثين ومائتين

മുഹമ്മദിബ്നു അബ്ദുല്ലാഹിബ്നു നുമൈറിന്റെ ജനനം ഹിജ്റാബ്ദം 161 ൽ ആണ്. അദ്ദേഹം മരണമടയുന്നത് ഹിജ്റാബ്ദം 234 ൽ ആണ്. (സിയറു അഅ്ലാമിന്നുബലാഅ്: 11: 456)

قال الهيثم بن عدي : مات سنة سبع وأربعين ومائة ‘ഉബൈദുല്ലാഹിബ്നു ഉമർ’ മരണമടയുന്നത് 167 ലാണ്. (സിയറു അഅ്ലാമിന്നുബലാഅ്: 6: 305 )

ഈ രണ്ട് ‘റാവി’മാർ തമ്മിൽ പരസ്പരം കണ്ടുമുട്ടിയിട്ടില്ല എന്നതിനാൽ തന്നെ പരമ്പര ‘മുറിഞ്ഞ’താണ് (മുൻകത്വിഅ്).

അതേസമയം, പരമ്പരയിൽ ‘ഇബ്നു നുമൈർ’ എന്നതുകൊണ്ട് ഉദ്ദേശിക്കപ്പെട്ടിരിക്കുന്നത് ‘അബ്ദുല്ലാഹിബ്നു നുമൈർ’ എന്ന റാവിയാണ് എന്ന് അഭിപ്രായപ്പെട്ട പണ്ഡിതർ പരമ്പര ‘മുൻകത്വിഅ്’ അല്ല, പരമ്പര സ്വഹീഹാണ് എന്ന് നിരീക്ഷിക്കുകയുണ്ടായി. (ഇർവാഉൽ ഗലീൽ: അൽബാനി: 6: 201)

ഈ പരമ്പരയെ സംബന്ധിച്ച വീക്ഷണ വ്യത്യാസത്തിൽ നിന്നും ചുരുങ്ങിയ പക്ഷം ഈ പരമ്പരയും വിമർശന വിധേയമാണെന്ന് മനസ്സിലാക്കാം.

ഈ പരമ്പരകളിലൂടെ എല്ലാം ഒരു ഓട്ട പ്രദക്ഷിണം നടത്തിയാൽ താഴെ പറയുന്ന കാര്യങ്ങൾ മനസ്സിലാക്കാം:

1. ഇവയൊന്നും തന്നെ ഹദീസുകൾ അല്ല. പ്രവാചകന്റെ (സ) വാക്കോ പ്രവർത്തനമൊ ഉൾക്കൊള്ളുന്നതല്ല ഇവയൊന്നും തന്നെ. ഇസ്‌ലാമിലെ പ്രമാണങ്ങൾ ക്വുർആനും സ്വഹീഹായ ഹദീസുകളുമാണ്.

2. ചില പണ്ഡിതരുടെ അഭിപ്രായങ്ങളാണ് പല നിവേദനങ്ങളും. പ്രവാചകാനുചരന്മാരുടെ തന്നെ അഭിപ്രായങ്ങൾ ഇസ്‌ലാം മതത്തിൽ സ്വമേധയാ പ്രമാണങ്ങളൊ തെളിവുകളൊ അല്ല.

3. എല്ലാ നിവേദനങ്ങളുടെ പരമ്പരകളും ദഈഫ് (ദുർബലം) ആകുന്നു. സ്വഹീഹാണെന്ന് ചില പണ്ഡിതന്മാർ അഭിപ്രായപ്പെട്ടിട്ടുള്ളത് പരമ്പര: 9 മാത്രമാണ്. ആ പരമ്പരയുടെ സ്വീകാര്യതയിൽ തന്നെ അഭിപ്രായാന്തരം നിലനിൽക്കുന്നു.

പരമ്പര: 9 തന്നെ കൂലങ്കഷമായ ഒരു വിശകലനത്തിനെടുത്താൽ വിമർശകർ വർണ്ണിക്കുന്നതു പോലെ അശ്ലീലതയൊന്നും ഉള്ളടക്കത്തിൽ ഇല്ലെന്ന് മനസ്സിലാക്കാം.

ഒന്നാമതായി, അടിമ സ്ത്രീയെ വിവസ്ത്രയാക്കുന്നതായൊ, നഗ്നത വെളിവാക്കി നിരീക്ഷിക്കുന്നതായൊ, ലൈംഗിക അവയവങ്ങൾ സ്പർശിക്കുന്നതായൊ ഒന്നും പരമ്പര 9ൽ ഇല്ല. “വസ്ത്രത്തിന് മുകളിലൂടെ”യാണ് (من فوقِ الثيابِ) ഇബ്നു ഉമർ സമീപിക്കുന്നത്, “മാറിടത്തിനിടയിൽ” (بيْنَ ثَديَيها) ആണ് കൈ വെക്കുന്നത്, മാറിടത്തിൽ അല്ല, “അരക്ക് മുകളിൽ” (على عُجُزِها) വസ്ത്രം പിടിച്ചു കൊണ്ടുമാണ് പരിശോധന. ആ അടിമ സ്ത്രീ പ്രായപൂർത്തിയും വളർച്ചയും എത്തിയ സ്ത്രീ തന്നെയാണൊ എന്ന് പരിശോധിക്കുവാൻ മാത്രമായിരുന്നു ആ പ്രവർത്തനം. അതാകട്ടെ അദ്ദേഹത്തിന്റെ വ്യക്തിപരമായ ഒരു നിലപാട് മാത്രമാണ്. ഇസ്‌ലാമിൽ അതിന് ഒരു പ്രാമാണികതയും ഇല്ല. എല്ലാത്തിനുമുപരി അവിതർക്കിതവും സ്വഹീഹുമായ നിവേദക പരമ്പരയിലൂടെ അങ്ങനെ ഒരു പ്രവർത്തനം അദ്ദേഹത്തിൽ നിന്ന് സംഭവിച്ചിട്ടുണ്ട് എന്ന് ഖണ്ഡിതമായി ഉദ്ധരിക്കപ്പെട്ടിട്ടില്ല. ‘സ്വഹീഹാ’യ പരമ്പരകൾ മാത്രമാണ് മുസ്‌ലിംകൾ സ്വീകരിക്കുക. സ്വഹീഹായ ‘ഹദീസുകൾ’ (പ്രവാചക പാഠങ്ങൾ) ആകുന്നു ഇസ്‌ലാമിലെ പ്രമാണം.

വിമർശനം:

അവിശ്വാസികളെ ചങ്ങലകളിൽ തളച്ച്, നിർബന്ധിച്ച് ഇസ്‌ലാം സ്വീകരിപ്പിക്കുന്നവരാണ് “ഉത്തമ ജനത” എന്ന് ഹദീസുകളിൽ വന്നിരിക്കുന്നു !! ഇസ്‌ലാം നിർബന്ധിത മതപരിവർത്തനത്തെ പ്രോത്സാഹിപ്പിക്കുന്നു എന്നല്ലെ ഇത് തെളിയിക്കുന്നത്?

മറുപടി:

1. നിർബന്ധിത ആദർശപരിവർത്തനത്തെ നിശിതമായി വിമർശിച്ച മതമാണ് ഇസ്‌ലാം. ആദർശ സ്വാതന്ത്ര്യത്തിന് അടിവരയിട്ടു കൊണ്ടുള്ള ക്വുർആൻ വചനങ്ങൾ അനവധിയാണ്:

“മതകാര്യത്തില്‍ ഒരുവിധ ബലപ്രയോഗവുമില്ല. ‎നന്മതിന്മകളുടെ വഴികള്‍ വ്യക്തമായും ‎വേര്‍തിരിഞ്ഞുകഴിഞ്ഞിരിക്കുന്നു…” (ക്വുർആൻ: 2:256)

“നിന്റെ നാഥന്‍ ഇച്ഛിച്ചിരുന്നെങ്കില്‍ ഭൂമിയിലുള്ളവരൊക്കെയും സത്യവിശ്വാസം സ്വീകരിക്കുമായിരുന്നു. എന്നിരിക്കെ ജനങ്ങള്‍ വിശ്വാസികളാകാന്‍ നീ അവരെ നിർബന്ധിക്കുകയോ? യാതൊരാള്‍ക്കും അല്ലാഹുവിന്‍റെ അനുമതിപ്രകാരമല്ലാതെ വിശ്വസിക്കാന്‍ കഴിയുന്നതല്ല. ചിന്തിച്ചു മനസ്സിലാക്കാത്തവര്‍ക്ക് അല്ലാഹു നികൃഷ്ടത വരുത്തിവെക്കുന്നതാണ്‌.”

“പറയുക: സത്യം നിങ്ങളുടെ രക്ഷിതാവിങ്കല്‍ നിന്നുള്ളതാകുന്നു. അതിനാല്‍ ഇഷ്ടമുള്ളവര്‍ വിശ്വസിക്കട്ടെ.” (ക്വുർആൻ: 18:29)

യുദ്ധങ്ങളിൽ ബന്ദികളാക്കപ്പെടുന്നവർക്ക് നിരുപാധിക മത സ്വാതന്ത്ര്യം ഇസ്‌ലാം എക്കാലത്തും വകവെച്ചു കൊടുത്തിട്ടുണ്ട്. ഖൈബറിൽ ബന്ദിയാക്കപ്പെട്ടിരുന്ന സ്വഫിയ്യയോട് പ്രവാചകൻ (സ്വ) ഇപ്രകാരം പറയുകയുണ്ടായി:

.ﺇﻥْ ﺃَﻗَﻤْﺖ ﻋﻠﻰ ﺩﻳﻨﻚ ﻟَﻢْ ﺃُﻛْﺮِﻫْﻚ، ﻭَﺇِﻥْ اﺧْﺘَﺮْﺕ اﻟﻠﻪَ ﻭَﺭَﺳُﻮﻟَﻪُ ﻓَﻬُﻮَ ﺧَﻴْﺮٌ ﻟَﻚ

“നിന്റെ പഴയ മതത്തില്‍ തന്നെ നില്‍ക്കാനാണ് തീരുമാനമെങ്കില്‍ അതുപേക്ഷിക്കാന്‍ ഞാന്‍ നിന്നെ നിർബന്ധിക്കുകയില്ല.” (അൽ മഗാസി: വാഖിദി: 2/675)

2. അവിശ്വാസികളെ ചങ്ങലകളിൽ തളച്ച്, നിർബന്ധിച്ച് ഇസ്‌ലാം സ്വീകരിപ്പിക്കുന്നവരാണ് “ഉത്തമ ജനത” എന്ന് ഒരു സ്വഹീഹായ ഹദീസിലും വന്നിട്ടില്ല. വിമർശകർ ഉന്നയിക്കുന്ന ആശയത്തോട് പുലബന്ധം പോലുമില്ലാത്ത ഒരു ഹദീസിനെ അപനിർമ്മിച്ചുകൊണ്ട് കെട്ടിയുണ്ടാക്കിയ ഒരു വികൃതമായ ആരോപണം മാത്രമാണ് അത്.

വിമർശന വിധേയമായ ഹദീസ് ഇപ്രകാരമാണ്:

.عن أبى هريرة رضى الله عنه عن النبى صلى الله عليه وسلم قال: (عَجِبَ اللَّهُ مِنْ قَوْمٍ يَدْخُلُونَ الجَنَّةَ فِي السَّلاَسِلِ)

അബൂഹുറൈറ (റ) യിൽ നിന്ന്, അദ്ദേഹം പ്രവാചകനിൽ (സ്വ) നിന്നും ഉദ്ദരിക്കുന്നു. അദ്ദേഹം (സ്വ) പറഞ്ഞു: ചങ്ങലകെട്ടുകളിലായി സ്വർഗത്തിൽ പ്രവേശിക്കുന്ന ഒരു പറ്റം ജനങ്ങളെ സംബന്ധിച്ച് അല്ലാഹു തൃപ്തി പ്രകടിപ്പിക്കും. (സ്വഹീഹുൽ ബുഖാരി: 3010)

സത്യനിഷേധികളുടെ അടുക്കൽ ബന്ദികളായി ജീവിക്കുകയും അതെ അവസ്ഥയിൽ മരണപ്പെടുകയൊ, കൊല്ലപ്പെടുകയൊ ചെയ്യുന്ന മുസ്‌ലിംകളെ സംബന്ധിച്ചാണ് ഹദീസ് പരാമർശിക്കുന്നത്. അത്തരം മുസ്‌ലിംകൾ അവർ മരണപ്പെട്ട ബന്ധനാവസ്ഥയിൽ തന്നെ പുനർജീവിപ്പിക്കപ്പെടാൻ കാരണം അവരുടെ ത്യാഗത്തിനും രക്തസാക്ഷിത്വത്തിനും ആദരവ് എന്ന നിലക്കാണ്. ഇത്രയും സുവ്യക്തമായ ഒരു ആശയത്തെ എത്ര വിദൂരമായ അർത്ഥതലത്തിലേക്കാണ് വിമർശകർ ദുർവ്യാഖ്യാനിച്ച് വലിച്ചു നീട്ടിയത് എന്ന് ശ്രദ്ധിക്കുക?!

വിമർശകർ വളച്ചൊടിച്ച് ദുർവ്യാഖ്യാനിക്കാറുള്ള, സമാനമായ മറ്റൊരു നിവേദനം ഇപ്രകാരമാണ്:

عن أبي هريرة في قوله تعالى: (كنتم خير أمة أخرجت للناس ) قال : ( خير الناس للناس ، يأتون بهم في السلاسل في أعناقهم حتى يدخلوا في الإسلام.

അല്ലാഹു പറഞ്ഞു: “മനുഷ്യവംശത്തിനു വേണ്ടി രംഗത്ത് കൊണ്ടുവരപ്പെട്ട ഉത്തമസമുദായമാകുന്നു നിങ്ങള്‍…” (ക്വുർആൻ: 3:110) എന്ന ക്വുർആൻ വചനത്തെ വ്യാഖ്യാനിച്ചു കൊണ്ട് അബൂഹുറൈറ (റ) പറഞ്ഞു: ജനങ്ങൾക്കായുള്ള ജനങ്ങളിൽ നിന്നുള്ള ഏറ്റവും ഉത്തമർ. കഴുത്തുകളിൽ ചങ്ങലകളണിയപ്പെട്ട നിലക്ക് അവരെ കൊണ്ട് വരും; അവർ ഇസ്‌ലാം സ്വീകരിക്കും വരെ.

ഈ രണ്ടാമത്തെ നിവേദനം അബൂ ഹുറൈറയുടെ (റ) വാചകമാണ്, മുഹമ്മദ് നബിയുടെ (സ) ഹദീസ് അല്ല എന്ന് സാന്ദർഭികമായി സൂചിപ്പിക്കട്ടെ. ഈ നിവേദനത്തിന്റെയും ഉള്ളടക്കം മുമ്പത്തെ ഹദീസിന്റെ ആശയം തന്നെയാണ് എന്നതിൽ സംശയമില്ല. അവിശ്വാസികളെ ചങ്ങലകളിൽ തളക്കാനൊ നിർബന്ധിതമായി മതപരിവർത്തനത്തിന് വിധേയമാക്കാനൊ രണ്ട് നിവേദനങ്ങളിലും ഒരു സൂചനയും ഇല്ല.

ഈ നിവേദനങ്ങൾക്ക് മുസ്‌ലിം പണ്ഡിതന്മാർ നൽകിയ രണ്ട് വ്യാഖ്യാനങ്ങൾ ശ്രദ്ധിച്ചാൽ തന്നെ ഈ വസ്തുത സുതരാം വ്യക്തമാവുന്നതാണ്.

വ്യാഖ്യാനം: ഒന്ന്

وقال غيره يحتمل أن يكون المراد المسلمين المأسورين عند أهل الكفر يموتون على ذلك أو يقتلون فيحشرون كذلك وعبر عن الحشر بدخول الجنة لثبوت دخولهم عقبة والله أعلم സത്യനിഷേധികളുടെ അടുക്കൽ ബന്ദികളായി ജീവിക്കുകയും അതെ അവസ്ഥയിൽ മരണപ്പെടുകയൊ കൊല്ലപ്പെടുകയൊ ചെയ്യുന്ന മുസ്‌ലിംകളാണ് ഹദീസിന്റെ താൽപര്യമെന്ന് പല പണ്ഡിതന്മാരും ഹദീസിനെ വ്യാഖ്യാനിക്കുന്നു. അത്തരം മുസ്‌ലിംകൾ അവർ മരണപ്പെട്ട ബന്ധനാവസ്ഥയിൽ തന്നെ പുനർജീവിപ്പിക്കപ്പെടുകയും ചെയ്യുമെന്നർത്ഥം. (ഫത്ഹുൽ ബാരി: 6:101)

വ്യാഖ്യാനം: രണ്ട്

قال ابن الجوزي معناه انهم أسروا وقيدوا فلما عرفوا صحة الاسلام دخلوا طوعا فدخلوا الجنة فكان الاكراه على الأسر والتقييد هو السبب الأول وكانه أطلق على الاكراه التسلسل ولما كان هو السبب في دخول الجنة أقام المسبب مقام السبب

ഇബ്നുൽ ജൗസി പറഞ്ഞു: ഹദീസിന്റെ ഉദ്ദ്യേശമിതാണ്: അവർ യുദ്ധങ്ങളിൽ പിടിക്കപ്പെടുകയും ബന്ദികളാക്കപ്പെടുകയും ചെയ്ത (അമുസ്‌ലിംകളായ) വരാണ്. എന്നാൽ പിന്നീട് ഇസ്‌ലാമിന്റെ സത്യത അവർ സ്വമേധയാ തിരിച്ചറിയുകയും സ്വേച്‌ഛയാൽ ഇസ്‌ലാം സ്വീകരിക്കുകയും ചെയ്തു. അവർ ഇസ്‌ലാം സ്വീകരിച്ചത് നിർബന്ധിതമായിട്ടല്ലെങ്കിലും ഇസ്‌ലാമിനെ (അവർ അറിയാനും ആശ്ലേഷിക്കാനും) വഴിവെച്ചത് ബന്ധനമാണ് എന്നതിനാൽ ബന്ധനത്തെ സ്വർഗപ്രവേശനത്തിന്റെ (യും ഇസ്‌ലാം ആശ്ലേഷണത്തിന്റെയും) കാരണമായി ആലങ്കാരികമായി പ്രയോഗിക്കപ്പെട്ടിരിക്കുകയാണ് ഇവിടെ. (ഫത്ഹുൽ ബാരി: 6:101)

പ്രവാചകാനുചരൻ അബൂ അസീസ് ഇബ്നു ഉമൈറിന്റെ (റ) ജീവിതാനുഭവം ഈ വ്യാഖ്യാനത്തിന് ഒരു ഉദാഹരണമാണ്. അദ്ദേഹം (റ) പറഞ്ഞു: ബദ്ർ യുദ്ധാനന്തരം (ശത്രു പക്ഷത്ത് നിന്ന്) ബന്ദികളാക്കപ്പെട്ടവരിൽ ഞാനും ഉണ്ടായിരുന്നു. (അദ്ദേഹം പിന്നീടാണ് ഇസ്‌ലാം സ്വീകരിക്കുന്നത്) അപ്പോൾ അല്ലാഹുവിന്റെ ദൂതർ മുസ്‌ലിംകളോട് കൽപ്പിച്ചു: “ബന്ദികളോട് നന്മ ചെയ്യാൻ ഞാൻ നിങ്ങളോട് അനുശാസനം നല്‍കുന്നു”. ഞാൻ അൻസ്വാരികളുടെ ഒരു കൂട്ടത്തിലാണ് (ബന്ധനസ്ഥനായ നിലക്ക്) ഉണ്ടായിരുന്നത്. രാവിലേയും വൈകുന്നേരവും അവരുടെ അടുക്കൽ ഭക്ഷണം കൊണ്ടു വരപ്പെടുമ്പോഴെല്ലാം -ബന്ദികളോട് നന്മ ചെയ്യാനുള്ള പ്രവാചകന്റെ അനുശാസനം പരിഗണിച്ച് – അവർ ഈത്തപഴം ഭക്ഷിക്കുകയും എനിക്ക് റൊട്ടി നൽകുകയും ചെയ്യുമായിരുന്നു. (ത്വബ്റാനി: മുഅ്ജമു സ്വഗീർ: 409, അൽ ഖബീർ: 977, മജ്മഉ സവാഇദ്: 10007)

(ബദർ യുദ്ധം നടത്തിയതും തുടക്കം കുറിച്ചതും സത്യനിഷേധികളാണ് എന്ന് സാന്ദർഭികമായി സൂചിപ്പിക്കട്ടെ. യുദ്ധത്തിൽ സത്യനിഷേധികൾ പരാചയപ്പെടുകയും ബന്ദികളാക്കപ്പെടുകയും ചെയ്തപ്പോൾ കൂട്ടത്തിൽ പ്രവാചകാനുചരൻ ഇബ്നു ഉമൈറും (റ) ഉണ്ടായിരുന്നു).

ഇസ്‌ലാമിന്റെ നന്മകൾ അടുത്തറിയാനും മുസ്‌ലിംകളിലെ മൂല്യങ്ങൾ അനുഭവിച്ചറിയാനും ഇസ്‌ലാം മതത്തിൽ ആകൃഷ്ടനാവാനും തുടർന്ന് സ്വേച്‌ഛ പ്രകാരം മതത്തെ സ്വീകരിക്കാനും അദ്ദേഹത്തിന് നിമിത്തമായത് ബദർ യുദ്ധാനന്തരമുണ്ടായ ബന്ധനമാണ്. ഇങ്ങനെ ഇസ്‌ലാമാശ്ലേഷിച്ച, അബൂ അസീസ് ഇബ്നു ഉമൈറിനെ (റ) പോലെയുള്ളവരെ സംബന്ധിച്ചാണ് ഹദീസിലെ ഉദ്ദേശ്യം എന്നതാണ് രണ്ടാമത്തെ വ്യാഖ്യാനം.

വിമർശനം: നിർബന്ധിത വസ്ത്രാക്ഷേപം വഴി അടിമ സ്ത്രീകൾക്ക് വസ്ത്ര സ്വാതന്ത്ര്യം ഖലീഫ ഉമർ നിഷേധിക്കുകയുണ്ടായി. അദ്ദേഹത്തിന്റെ കാലഘട്ടത്തിൽ മാറ് മറക്കാൻ പോലും അവർക്ക് സാധിച്ചിരുന്നില്ല. മാന്യമായ വസ്ത്രം ധരിക്കുന്ന അടിമ സ്ത്രീകളെ ഉമർ വടി കൊണ്ട് അടിക്കുമായിരുന്നു. മറുപടി:

ഒരു കുന്നിക്കുരുവോളം വരുന്ന അർദ്ധ സത്യത്തിനു പുറത്ത്, കളവുകൾക്കു മേൽ കളവ് കയറ്റി വെച്ച് കളവിൻ കൊട്ടാരം കെട്ടിപടുക്കുകയാണ് ഇവിടെ ഇസ്‌ലാം വിമർശകർ ചെയ്തിരിക്കുന്നത്. ചേരുവയായി വ്യാജ നിവേദനങ്ങളും ദുർവ്യാഖ്യാനങ്ങളും എമ്പാടും ഉണ്ട്. വിമർശകർ ഉദ്ധരിച്ച ഓരോ ആരോപണങ്ങൾക്കും അക്കമിട്ടുള്ള മറുപടിയിലേക്ക് കടക്കാം

1. അടിമസ്ത്രീകളെ നിർബന്ധിതമായി വസ്ത്രാക്ഷേപം നടത്താനോ വസ്ത്രസ്വാതന്ത്ര്യം നിഷേധിക്കാനോ ഇസ്‌ലാം നിഷ്‌കർശിച്ചുവോ ?

ഒരിക്കലുമില്ല. അന്യപുരുഷരുടെ ശല്യത്തിൽ നിന്നും ലൈംഗിക ഉപദ്രവത്തിൽ നിന്നും സ്ത്രീക്ക് ഇസ്‌ലാം സമ്മാനിച്ച രക്ഷാകവചമാണ് ഹിജാബ്‌. സ്ത്രീയെ അടിച്ചമർത്താനോ ബുദ്ധിമുട്ടിക്കാനോ അല്ല ഹിജാബിലൂടെ ഇസ്‌ലാം ശ്രമിച്ചത് എന്നതിന് ഏറ്റവും നല്ല തെളിവാണ് അടിമ സ്ത്രീകൾക്ക് തങ്ങളുടെ ശരീരാവയങ്ങൾ മറക്കുന്ന കാര്യത്തിൽ നൽകപ്പെട്ട ഇളവ്. അടിമസ്ത്രീകൾക്ക് സ്വതന്ത്ര സ്ത്രീകളിൽ നിന്നും വസ്ത്രത്തിന്റെ കാര്യത്തിൽ ചില ഇളവുകൾ നല്കപ്പെട്ടത് അവർക്ക് അവരുടെ ജീവിതവും ഉദ്യോഗവും എല്ലാം എളുപ്പമാക്കാൻ വേണ്ടി ആയിരുന്നു എന്നത് ഇസ്‌ലാമിന്റെ കാരുണ്യത്തെയാണ്, കാർക്കശ്യത്തെയല്ല തെളിയിക്കുന്നത്. ഉടമസ്ഥർക്ക് വേണ്ടി വീട്ടുജോലികളും വേലകളും ചെയ്യുകയായിരുന്നു അടിമസ്ത്രീകളുടെ പതിവ് എന്നതിനാൽ സ്വതന്ത്ര സ്ത്രീകളെ പോലെ വളരെ കണിശമായി വസ്ത്ര ധാരണത്തിൽ ശ്രദ്ധ ചെലുത്തുക അവരെ സംബന്ധിച്ചിടത്തോളം ശ്രമകരമായിരിക്കുമല്ലോ, ഈ ക്ലേശത്തെ അവരിൽ നിന്നും നീക്കുവാനാണ് ശ്രമം. വൃദ്ധരായ സ്ത്രീകൾക്കും ഹിജാബിന്റെ വിഷയത്തിൽ ഇത്തരം ഇളവുകൾ നൽകപ്പെട്ടതായി ഖുർആനിൽ ദർശിക്കാം.

وَالْقَوَاعِدُ مِنَ النِّسَاءِ اللَّاتِي لَا يَرْجُونَ نِكَاحًا فَلَيْسَ عَلَيْهِنَّ جُنَاحٌ أَنْ يَضَعْنَ ثِيَابَهُنَّ غَيْرَ مُتَبَرِّجَاتٍ بِزِينَةٍ وَأَنْ يَسْتَعْفِفْنَ خَيْرٌ لَهُنَّ

“വിവാഹ ജീവിതം പ്രതീക്ഷിക്കാത്ത വൃദ്ധകളെ സംബന്ധിച്ചിടത്തോളം സൗന്ദര്യം പ്രദര്‍ശിപ്പിക്കാത്തവരായിക്കൊണ്ട് തങ്ങളുടെ മേല്‍വസ്ത്രങ്ങള്‍ മാറ്റി വെക്കുന്നതില്‍ അവര്‍ക്ക് കുറ്റമില്ല. അവര്‍ മാന്യത കാത്തുസൂക്ഷിക്കുന്നതാണ് അവര്‍ക്ക് കൂടുതല്‍ നല്ലത്‌. അല്ലാഹു എല്ലാം കേള്‍ക്കുന്നവനും എല്ലാം അറിയുന്നവനുമാകുന്നു.” (ക്വുർആൻ: 24:60)

വൃദ്ധകൾക്കും അടിമസ്ത്രീകൾക്കും അവരുടെ ജീവിത ആശ്വാസത്തെ പരിഗണിച്ചു കൊണ്ട് വസ്ത്രധാരണ വിഷയത്തിൽ ഇളവുകൾ അനുവദിച്ചു എന്നത് ഇസ്‌ലാം സ്ത്രീകൾക്ക് സമാധാനവും സുരക്ഷയുമായി നിലകൊണ്ട ആദർശമാണെന്നതിനുള്ള അനിഷേധ്യമായ തെളിവാണ്.

ഉമറിൽ നിന്നും ഉദ്ധരിക്കപ്പെട്ട സംഭവങ്ങളും മറിച്ചൊന്നല്ല സ്ഥാപിക്കുന്നത്.

ﻭاﻵﺛﺎﺭ ﻋَﻦْ ﻋُﻤَﺮَ ﺑْﻦِ اﻟْﺨَﻄَّﺎﺏِ ﺭَﺿِﻲَ اﻟﻠﻪُ ﻋَﻨْﻪُ ﻓِﻲ ﺫَﻟِﻚَ ﺻَﺤِﻴﺤَﺔٌ، ﻭَﺇِﻧَّﻬَﺎ ﺗَﺪُﻝُّ ﻋَﻠَﻰ ﺃَﻥَّ ﺭَﺃْﺳَﻬَﺎ ﻭَﺭَﻗَﺒَﺘَﻬَﺎ ﻭَﻣَﺎ ﻳَﻈْﻬَﺮُ ﻣِﻨْﻬَﺎ ﻓِﻲ ﺣَﺎﻝِ اﻟْﻤِﺤْﻨَﺔِ ﻟَﻴْﺲَ ﺑِﻌَﻮْﺭَﺓٍ

ഉമർ ബിൻ ഖത്താബിൽ നിന്നും ഈ വിഷയത്തിൽ ഉദ്ധരിക്കപ്പെട്ടിട്ടുള്ള നിവേദനങ്ങൾ സ്വഹീഹ് ആകുന്നു. തങ്ങളുടെ തല, പിരടി എന്നിങ്ങനെ ജോലി ചെയ്യുന്ന അവസ്ഥയിൽ വല്ല ശരീരഭാഗങ്ങൾ വെളിവാവുകയാണെങ്കിൽ അത് അടിമ സ്ത്രീകൾക്ക് ഔറത്ത് (നഗ്നത) ആയി പരിഗണിക്കപ്പെടില്ല എന്ന് ഈ നിവേദനങ്ങൾ സൂചിപ്പിക്കുന്നു. (സുനനുൽ കുബ്റാ: ബൈഹകി: 2:360)

എങ്കിൽ പിന്നെ ഇളവുകൾ സ്വീകരിക്കാൻ ഉമർ അടിമസ്ത്രീകളെ എന്തിനു നിർബന്ധിച്ചു? എന്നതാണ് അടുത്ത സംശയം.

ഉത്തരം സരളമാണ്. തന്റെ ദീർഘദൃഷ്ടിയുടെ അടിസ്‌ഥാനത്തിൽ അദ്ദേഹം സ്വീകരിച്ച വ്യക്തിപരമായ ഒരു നയം മാത്രമായിരുന്നു അത്. അടിമസ്ത്രീകളെല്ലാം സ്വതന്ത്ര സ്ത്രീകളെ പോലെ വസ്ത്രം ധരിക്കാൻ ആരംഭിച്ചാൽ അവർ തമ്മിലുള്ള കാഴ്ചയിലുള്ള വ്യതിരിക്തത ഇല്ലാതാവുകയും കാലാന്തരങ്ങളിൽ ആ വസ്ത്രധാരണ രീതിയിൽ അടിമസ്ത്രീകളുടെ മേലും കാർക്കശ്യവും നിർബന്ധിത സ്വഭാവവും വന്നുചേർന്നേക്കാം. ഇത് അടിമസ്ത്രീകളെ തന്നെ അവരുടെ ജീവിതസന്ധാരണ മാർഗവും ശൈലിയും പരിഗണിക്കുമ്പോൾ അങ്ങേയറ്റം ആയാസകരമായ സ്ഥിതിവിശേഷത്തിലേക്ക് കൊണ്ടെത്തിക്കുകയും ചെയ്യും. ഈ അവസ്ഥയെ മുൻകൂട്ടി കണ്ട് വ്യക്തിപരമായ ഒരു നയത്തിലൂടെ നിർഭാടനം ചെയ്യാനാണ് തന്റെ നിർബന്ധ കല്പനയിലൂടെ ഉമർ (റ) ശ്രമിച്ചത്. ഈ നയമാകട്ടെ പ്രവാചകനോ(സ) അദ്ദേഹത്തിന് ശേഷം വന്ന ഖലീഫ അബൂബക്കറോ(സ) ഇസ്‌ലാമിക പ്രമാണങ്ങളോ മതത്തിന്റെ ഭാഗമായി ആവിഷ്കരിച്ച നിയമമല്ല എന്നു പ്രത്യേകം മനസ്സിലാക്കണം. പൊതു നന്മയെയും ആസന്നമായ ഭാവിയെയും പരിഗണിച്ച് കൊണ്ട് ഒരു ഭരണാധികാരി എന്ന നിലയിൽ ഉമർ(റ) ഇതു പോലെ പല നയങ്ങളും സ്വീകരിച്ചിട്ടുണ്ട്. അവയെല്ലാം ഇസ്‌ലാമിന്റെ അധ്യാപനങ്ങളായി അദ്ദേഹം പോലും മനസ്സിലാക്കിയിട്ടില്ല. ഉദാഹരണത്തിന് മുത്അതായി ഹജ്ജ് ചെയ്യുന്ന വിഷയം തന്നെ എടുക്കാം. ഹജ്ജും ഉംറയും ഒരുമിച്ചു നിർവഹിക്കുന്ന ഈ രീതിയിൽ പ്രവാചകനും അനുചരന്മാരും ധാരാളമായി ഹജ്ജ് ചെയ്തിട്ടുള്ളതാണ്. അഥവാ ഇസ്‌ലാമിക ആചാര വൃത്തത്തിന്റെ ഉള്ളിൽ ആയിട്ടു പോലും ഹജ്ജും ഉംറയും ചേർത്തു നിർവഹിക്കുന്നത് ഖലീഫ ഉമർ തടയുകയുണ്ടായി. ഹജ്ജിന്റെ സീസണിൽ തീർഥാടകരെ കൊണ്ട് മുഖരിതമാകുന്ന കഅ്ബാലയം മറ്റു മാസങ്ങളിൽ ഒഴിഞ്ഞു കിടക്കുന്ന അവസ്ഥ ഖലീഫ ഉമർ(റ) ശ്രദ്ധിക്കുകയുണ്ടായി. ഹജ്ജും ഉംറയും ചെയ്തു കഴിഞ്ഞത് കൊണ്ട് ഉംറക്കായി മറ്റൊരു വരവ് വരേണ്ടതില്ല എന്നതായിരുന്നു ഇതിനു കാരണം എന്ന് അദ്ദേഹം മനസ്സിലാക്കി. ഹജ്ജിന്റെ മാസം കഴിഞ്ഞിട്ടേ ഉംറ ചെയ്യാവൂ എന്ന നിർബന്ധനിയമത്തിലൂടെ ഈ പ്രശ്നം പരിഹരിക്കാമെന്ന് സ്വാഭാവികമായും അദ്ദേഹം പ്രതീക്ഷിച്ചു. ഇത്തരമൊരു നയപരമായ നിലപാട് മാത്രമാണ് അടിമസ്ത്രീകളുടെ വസ്ത്രത്തിലും ഉമർ(റ) സ്വീകരിച്ചതായി നാം ദർശിക്കുന്നത്. ശക്തമായ വ്യക്തിത്വത്തിനുടമയായത് കൊണ്ട് തന്നെ ഉമറിന്റെ(റ) ഇത്തരം നയനിലപാടുകളിൽ അല്പം കാർക്കശ്യവും കാഠിന്യവും നിഴലിച്ചിരുന്നു എന്നത് വാസ്തവമായിരിക്കാം. എന്നാൽ അതൊന്നും അദ്ദേഹത്തിന്റെ പ്രജകൾക്ക് അക്രമമൊ അനീതിയൊ ആയി അനുഭവപ്പെട്ടിട്ടില്ല എന്ന് മാത്രമല്ല നീതിയുടെ പര്യായമായാണ് അദ്ദേഹം സർവ്വരാലും വാഴ്ത്തപ്പെട്ടിരുന്നത്.

2. വസ്ത്രാക്ഷേപം നടത്തി, മാറു മറക്കാനുള്ള അവകാശം പോലും നിഷേധിച്ചു എന്നൊക്കെയുള്ള ആരോപണങ്ങൾ ആണ് അടുത്തത്.

അവയെല്ലാം ആനനുണകളാണ്. തലയും മുഖവും കൈകളും മറക്കരുത് എന്നു മാത്രമായിരുന്നു ഉമറിന്റെ(റ) നിർദേശം എന്ന് വ്യക്തമായി വിമർശന വിധേയമായ നിവേദനങ്ങളിൽ തന്നെ രേഖപ്പെടുത്തപ്പെട്ടിട്ടുണ്ട്.

فَيَظْهَرُ مِنْ الْأَمَةِ : رَأْسُهَا ، وَيَدَاهَا ، وَوَجْهُهَا

“അടിമ സ്ത്രീ തന്റെ തലയും ഇരു കൈകളും മുഖവും വെളിവാക്കി കൊള്ളട്ടെ…” (മജ്മൂഉൽ ഫതാവാ: 15: 372 ) എന്നാണ് ഉമർ (റ) നിർദ്ദേശിച്ചത്.

മാറിടം വെളിവാക്കാൻ കല്പിക്കപ്പെട്ടതായി സൂചിപ്പിക്കുന്ന ഒരു നിവേദനവും കുഴിയാനക്കുഴിയിൽ പോലും തപ്പിയാൽ കണ്ടു കിട്ടില്ല. ആകെക്കൂടി ദുർവ്യാഖ്യാനിക്കാവുന്നത് താഴെ പറയുന്ന നിവേദനമാണ്.

.بْنِ مَالِكٍ قَالَ: “كُنَّ إِمَاءُ عُمَرَ رَضِيَ اللهُ عَنْهُ يَخْدِمْنَنَا كَاشِفَاتٍ عَنْ شُعُورِهِنَّ تَضْرِبُ ثُدِيّهُنَّ”

അനസ് (റ) പറയുന്നു: ഉമറിന്റെ(റ) (കാലഘട്ടത്തിലെ) സ്ത്രീകൾ ഞങ്ങൾക്ക് വീട്ടുജോലികൾ ചെയ്തു തന്നിരുന്നു; അവരുടെ മുടി അവരുടെ മാറിടങ്ങളിൽ ഉലയ്‌ക്കുവോളം മുടി അഴിച്ചിട്ട നിലയിൽ തന്നെ (അവർ ജോലികളിൽ വ്യാപൃതരായിരുന്നു). (സുനനുൽ ബൈഹകി: 3222, ഇർവാഉൽ ഗലീൽ: 6:204)

كَاشِفَاتٍ عَنْ شُعُورِهِنَّ تَضْرِبُ ثُدِيّهُنَّ

“അവരുടെ മുടി അവരുടെ മാറിടങ്ങളിൽ ഉലയ്‌ക്കുവോളം മുടി അഴിച്ചിട്ട നിലയിൽ….” എന്ന നിവേദനത്തിലെ ഭാഗം വളച്ചൊടിച്ചും ദ്വയാർത്ഥം വരാവുന്ന അതി ദുർബലമായ ചില നിവേദനങ്ങൾ കൂട്ടുപിടിച്ചും അടിമ സ്ത്രീകൾ “അവരുടെ മാറിടങ്ങൾ വെളിവാക്കി” കൊണ്ടാണ് ജോലി ചെയ്തിരുന്നത് എന്ന് ദുർവ്യാഖ്യാനിക്കുകയാണ് ഇസ്‌ലാം / ഹദീസ് വിരോധികളുടെ കുതന്ത്രം.

.عَنْ أَنَسِ بْنِ مَالِكٍ قَالَ: ” كُنَّ جَوَارِي عُمَرَ يَخْدُمْنَنَا كَاشِفَاتِ الرُّءُوسِ، تَضْطَرِبُ ثُدِيُّهُنَّ بَادِيَةً خِدَامُهُنَّ”

“അവരുടെ മുടി അവരുടെ മാറിടങ്ങളിൽ ഉലയ്‌ക്കുമായിരുന്നു…” എന്ന ശരിയായ പദങ്ങൾക്ക് പകരം, “അവരുടെ മാറിടങ്ങൾ ഉലയ്‌ക്കുമായിരുന്നു…” എന്ന് അനസ് (റ) പറഞ്ഞതായി തെറ്റായി ഉദ്ധരിക്കപ്പെട്ട നിവേദനമാകട്ടെ (തഫ്സീറു യഹ്‌യബ്നു സലാം: 1:441) അങ്ങേയറ്റം ദുർബലമായ നിവേദക പരമ്പരയിലൂടെയാണ് വന്നിരിക്കുന്നത്. പരമ്പരയിലെ ‘നസ്വർ ബിൻ ത്വരീഫ്’ എന്ന നിവേദകനെ സംബന്ധിച്ച ഹദീസ് നിദാനശാസ്ത്ര പണ്ഡിതന്മാരുടെ അഭിപ്രായം ഇതാണ്: യഹ്‌യ പറഞ്ഞു: കള്ള ഹദീസുകൾ ഉണ്ടാക്കുന്നതിൽ പ്രസിദ്ധനാണ് അയാൾ. ഫല്ലാസ് പറഞ്ഞു: നുണയന്മാരുടെ കൂട്ടത്തിൽ പെട്ടവനെന്നതിൽ ഏകാഭിപ്രായമുള്ള നിവേദകരിൽ ഒരാളാണ് ‘നസ്വർ ബിൻ ത്വരീഫ്’. (ലിസാനുൽ മീസാൻ: 6:153)

3. അടിമസ്ത്രീയെ ഖലീഫ ഉമർ തന്റെ വടി കൊണ്ട് മർദ്ദിക്കുമായിരുന്നു എന്നതാണ് അടുത്ത ആരോപണം. ഇതും വാസ്തവവിരുദ്ധവും ദുർവ്യാഖ്യാനവുമാണ്. തലയിൽ നിന്നും വസ്ത്രം നീക്കാൻ അമാന്തം കാണിച്ച ചില അടിമസ്ത്രീകളുടെ തലയിൽ നിന്ന് വടി കൊണ്ട് അടിച്ചു വസ്ത്രം തട്ടിയിടുക മാത്രമാണ് അദ്ദേഹം ചെയ്തത്. ഒരു അന്യ സ്ത്രീയെ നേരിട്ട് സ്പർശിക്കാൻ പാടില്ല എന്നതാണ് ഇതിനു കാരണം.

رأى عمرُ أمةً عليْها جِلبابٌ فقال : عَتَقْتِ ؟ قالتْ : لا ، قال ضَعِيهِ عن رَأْسِكِ ، إِنَّما الجِلْبابُ على الحَرَائِرِ ، فَتَلَكَّأَتْ فقامَ إليها بِالدُّرَّةِ ، فضربَ رأسَها حتى ألقَتْهُ

“തലയിൽ നിന്ന് മറ നീക്കാൻ ഒരു അടിമസ്ത്രീ അമാന്തം കാണിച്ചപ്പോൾ തന്റെ വടി കൊണ്ട് തലയിലെ തട്ടം അടിച്ച് നിലത്തിട്ടു” എന്ന് ചില നിവേദനങ്ങളിൽ പ്രത്യേകം സൂചിപ്പിക്കപ്പെട്ടിട്ടുണ്ട്. (മുസ്വന്നഫ് ഇബ്നു അബീ ശൈബ: 6295)

ഇവിടെയും അക്കാലഘട്ടത്തിലെ ലൈംഗിക ശുദ്ധിയും സ്ത്രീകളോട് പുലർത്തിയിരുന്ന മാന്യതയും മാത്രമാണ് ദോഷൈകദൃക്കുകൾ അല്ലാത്തവർക്ക് കാണാൻ സാധിക്കുക. ഇത്രയും സരളവും സ്വാഭാവികവുമായ ഒരു സംഭവത്തെ വർണം ചേർത്തു പൊലിപ്പിച്ചു കാണിച്ചു വിവാദവൽക്കരിക്കാനുള്ള ശ്രമം എത്ര ജുഗുപ്സാവഹം.

താഴ്ന്ന ജാതിക്കാർക്കും അവർണ വിഭാഗത്തിൽ പെട്ട സ്ത്രീകൾക്കും മാറു മറക്കാനുള്ള അവകാശം നിഷേധിക്കുകയും മുലക്കരം ഏർപ്പെടുത്തുകയും ഒക്കെ ചെയ്തിരുന്ന സവർണ ഫാഷിസ്റ്റ് രതി വൈകൃതങ്ങളുടെ ആലയിലേക്ക് ഇസ്‌ലാമിനെയും വലിച്ചു കെട്ടാനുള്ള ശ്രമമാണ് ഖലീഫ ഉമറിനെ സംബന്ധിച്ച ഈ ആക്ഷേപ പ്രചാരണത്തിന്റെ ലക്ഷ്യം. എന്നാൽ ആ ആലയിലേക്ക് എത്തിക്കാൻ ഈ നുരുമ്പിച്ച കയറിന് നീളം പോര.

ഇസ്‌ലാം സ്ത്രീകളെ ആദരിച്ച മതമാണെന്നത് മുസ്‌ലീംകളുടെ കേവലം അവകാശവാദം മാത്രമാണ്. കാരണം പ്രവാചകന്‍ തന്നെ പല ഘട്ടങ്ങളിലും സ്ത്രീകളെ വളരെ മോശമായ രൂപത്തില്‍ ആക്ഷേപിച്ചു സംസാരിച്ചത് ഹദീസ് ഗ്രന്ഥങ്ങളില്‍ തന്നെ രേഖപ്പെടുത്തപ്പെട്ടിരിക്കുന്നു. സ്ത്രീയും നായയും കഴുതയും നമസ്‌ക്കാരത്തെ മുറിച്ചുകളയുമെന്ന നബി പാഠം തന്നെ തികഞ്ഞ സ്ത്രീവിരുദ്ധത നിറഞ്ഞു നില്‍ക്കുന്ന പരാമര്‍ശമാണ്. നായയോടും കഴുതയോടും പെണ്ണിനെ ഉപമിക്കുക വഴി തികഞ്ഞ സ്ത്രീവിരുദ്ധ പരാമര്‍ശമാണ് പ്രവാചകന്‍ നടത്തിയിരിക്കുന്നത്. പിന്നെ എങ്ങനെയാണ് ഇസ്‌ലാം സ്ത്രീകളെ ആദരിച്ച മതമാണെന്ന് മുസ്‌ലീംകള്‍ക്ക് വാദിക്കാനാവുന്നത് ?

മറുപടി:

അധമവിചാരികള്‍ക്ക് സത്യം എപ്പോഴും അപ്രാപ്യമായിരിക്കുമെന്ന നിരീക്ഷണം എത്രയോ വസ്തുതാപരമാണെന്ന് ഈ വിമര്‍ശനം വിളിച്ചറിയിക്കുന്നുണ്ട്. ഒറ്റ വായനയില്‍ തന്നെ സാമാന്യബുദ്ധിക്കു ഗ്രാഹ്യമായ ഒരു ആശയത്തെ എത്രമാത്രം സാഹസപ്പെട്ടാണ് ഇസ്‌ലാംവിരോധികളിവിടെ വികൃതവല്‍ക്കരിച്ചിരിക്കുന്നത്. വിമര്‍ശകരുടെ “വക്രീകര ശാസ്ത്ര സാഹസം” എത്രമാത്രം ബാലിശമാണെന്ന് മനസ്സിലാക്കാനായി നമുക്കൊരു സര്‍ക്കാര്‍ മാര്‍ഗനിര്‍ദ്ദേശ പത്രിക ഉദാഹരണമായെടുക്കാം. കോവിഡ് 19 വ്യാപനം തടയുന്നതിന്റെ ഭാഗമായി കേന്ദ്ര-സംസ്ഥാന സര്‍ക്കാറുകള്‍ പുറത്തിറക്കിയ പ്രസ്തുത മാര്‍ഗനിര്‍ദ്ദേശ പത്രികയില്‍ പരാമര്‍ശിച്ച പ്രത്യേക സംരക്ഷണം നല്‍കേണ്ടവരുടെ പട്ടികയില്‍ ഇപ്രകാരം കാണാം: “കുട്ടികള്‍, അംഗപരിമിതര്‍, മാനസിക രോഗികള്‍, സ്ത്രീകള്‍…” ഈ പത്രിക ചൂണ്ടികാട്ടി ഇസ്‌ലാംവിരോധികളാരെങ്കിലും പറഞ്ഞേക്കുമോ, കേന്ദ്ര-സംസ്ഥാന സര്‍ക്കാറുകള്‍ സ്ത്രീകളെയും മാനസിക രോഗികളെയും ഒരേ തരത്തിലാണ് കാണുന്നത്. അതുകൊണ്ടാണ് അവരെ ഒരേ പട്ടികയില്‍ തന്നെ ചേര്‍ത്തതെന്ന്!. വാസ്തവത്തില്‍ ഇസ്‌ലാംവിരോധികളുടെ ഹദീസ് വിമര്‍ശനവും ഈ വിഢിത്തത്തിനപ്പുറം ഒന്നുമല്ലെന്ന് വിമര്‍ശന വിധേയമായ ഹദീസിന്റെ യാഥാര്‍ഥ്യം കൃത്യമായി പരിശോധിച്ചാല്‍ തന്നെ ബോദ്ധ്യമാകുന്നതാണ്. അതിനാല്‍ നമുക്ക് ഇസ്‌ലാംവൈരികള്‍ വിമര്‍ശനവിധേയമാക്കിയ ഹദീസും അതിന്റെ താല്‍പര്യവും മനസ്സിലാക്കാം.

“അബൂഹുറയ്‌റ (റ) പറയുന്നു: റസൂല്‍ (സ്വ) പറഞ്ഞു: സ്ത്രീയും കഴുതയും നായയും നമസ്‌ക്കാരം മുറിക്കും. ഒട്ടകക്കട്ടിലിന്റെ പിന്നിലെ വടിപോലുള്ള ഒന്ന് (ഉണ്ടായാല്‍) അത് തടുക്കാവുന്നതാണ്.” (മുസ്‌ലിം 790, അഹ്‌മദ്, ഇബ്‌നു അബീശൈബ, ഇബ്‌നു ഹിബ്ബാന്‍) ഈ ഹദീസ് വായിച്ചപ്പോള്‍ സ്ത്രീയെ മൃഗങ്ങളോട് ഉപമിച്ചതായി ഇസ്‌ലാംവിരോധികള്‍ക്കു തോന്നി എന്നതല്ലാതെ മറ്റൊരു പ്രശ്നവുമിവിടെയില്ല. ഇസ്‌ലാംവിരോധികള്‍ക്കു തോന്നുന്ന അധമവിചാരങ്ങളും വികാരങ്ങളും കൊണ്ടുതള്ളാനുള്ള പുറമ്പോക്കല്ല ഇസ്‌ലാമെന്ന് അവര്‍ തിരിച്ചറിഞ്ഞാല്‍ തീരാവുന്ന പ്രശ്‌നമേ ഇവിടെയുള്ളൂ. ഈ ഹദീസിനെ പറ്റി പരാമര്‍ശിക്കവെ ഇമാം ക്വുര്‍ത്തുബിയുടെ(റ) വാചകങ്ങള്‍ “അല്‍ മുഫ്ഹിം ലിമാ അശ്കല മിന്‍ തല്‍ഖീസ്വി സ്വഹീഹ് മുസ്‌ലിമി”ല്‍ ഇപ്രകാരം രേഖപ്പെടുത്തിയിരിക്കുന്നു: “കാരണം തീര്‍ച്ചയായും സ്ത്രീ (സാന്നിദ്ധ്യം പുരുഷമനസ്സുകളില്‍) പ്രലോഭനമുണ്ടാക്കും, കഴുതയാകട്ടെ ഒച്ചയുണ്ടാക്കുകയും നായ ഭയം സൃഷ്ടിക്കുകയും ചെയ്യും. അപ്പോള്‍ (നമസ്‌ക്കരിക്കുന്ന) ചിന്താശേഷിയുള്ള വ്യക്തിയില്‍ (അവയുടെ സാന്നിദ്ധ്യം) ആശയക്കുഴപ്പം സൃഷ്ടിക്കുകയും അവന്റെ നമസ്‌ക്കാരം മുറിഞ്ഞു പോകുമാറ് കുഴപ്പമുണ്ടാക്കുകയും ചെയ്യും. അപ്പോള്‍ നമസ്‌ക്കാരത്തെ (ഏകാഗ്രത) മുറിച്ചു കളയുന്നതിലേക്ക് കാര്യങ്ങള്‍ എത്തുന്നതുകൊണ്ടാണ് (അവയെ) നമസ്‌ക്കാരം മുറിച്ചുകളയുന്ന കാര്യങ്ങളിലുള്‍പ്പെടുത്തിയത്.” (അല്‍ മുഫ്ഹിം ലിമാ അശ്കല മിന്‍ തല്‍ഖീസ്വി സ്വഹീഹ് മുസ്‌ലിം: 2/ 109)

“നമസ്ക്കാരത്തെ മുറിക്കും” എന്നാൽ നമസ്ക്കാരത്തിലെ ഏകാഗ്രത, പൂർണ്ണത മുറിക്കുകയും കുറക്കുകയും ചെയ്യുമെന്നാണ് വിവക്ഷയെന്ന് ഒട്ടുമിക്ക പണ്ഡിതരും വ്യക്തമാക്കിയിട്ടുണ്ട്.

وقال مالك وأبو حنيفة والشافعي رضي الله عنهم وجمهور من السلف والخلف: لا تبطل الصلاة بمرور شيء من هؤلاء ولا من غيرهم، وتأول هؤلاء هذا الحديث على أن المراد بالقطع نقص الصلاة لشغل القلب بهذه الأشياء وليس المراد إبطالها

“ഇമാം മാലിക്, അബൂ ഹനീഫ, ശാഫിഈ തുടങ്ങി പൂർവ്വീകരും ശേഷക്കാരുമായ ഭൂരിഭാഗം പണ്ഡിതന്മാരും പറയുന്നത്: ഈ വിഭാഗക്കാരുൾപ്പെടെ ഒന്നും തന്നെ നമസ്ക്കാരത്തെ നിഷ്ഫലമാക്കും എന്ന അർത്ഥത്തിൽ ‘മുറിക്കും’ എന്നല്ല. നമസ്ക്കാരത്തെ മുറിക്കും എന്നാൽ, ഇത്തരം കാര്യങ്ങളിൽ മനസ്സ് വ്യാപൃതമാവുക വഴി ഏകാഗ്രത മുറിയുകയും നമസ്ക്കാരത്തിൽ ന്യൂനത സംഭവിക്കും എന്നുമാണ്.” (തുഹ്ഫതുൽ അഹ്‌വദി: 1:371)

നമസ്കാരത്തെ മുറിക്കും എന്നാൽ നമസ്ക്കാരത്തിന്റെ ഭയ ഭക്തിക്ക് ഭംഗം വരുത്തും എന്നാണ്. അല്ലാതെ നമസ്ക്കാരത്തിൽ നിന്ന് പുറത്താവുമെന്നല്ല. (ഫത്ഹുൽ ബാരി: 2:273)

ഇമാം അഹ്‌മദ് പറഞ്ഞു: നമസ്ക്കരിക്കുന്നതിനിടയിൽ തന്റെ തൊട്ട് മുമ്പിൽ ഒരു നായയൊ ഒരു സ്ത്രീയൊ നിൽക്കുമ്പോൾ മനസ്സിൽ നമസ്കാരമല്ലാത്ത മറ്റു ചിന്തകൾ കടന്നുവന്നേക്കാം…. (ഫത്ഹുൽ ബാരി: 2:273)

സുവ്യക്തമാണ് കാര്യങ്ങള്‍, വിമര്‍ശന വിധേയമായ ഹദീസ് ഒരിക്കലും സ്ത്രീയെ മൃഗങ്ങളോട് ഉപമിക്കുകയല്ല ചെയ്തിരിക്കുന്നത്, മറിച്ച് ഒരു പുരുഷന്റെ നമസ്‌ക്കാരത്തില്‍ ഭംഗമുണ്ടാക്കുന്ന കാര്യങ്ങളെ പറ്റി പറയുമ്പോള്‍ ആ കൂട്ടത്തില്‍ സ്ത്രീയേയും പരാമര്‍ശിച്ചെന്ന് മാത്രം. നമസ്‌ക്കരിക്കുന്ന മനുഷ്യനില്‍, നായ ഭയം ജനിപ്പിച്ചും കഴുത ഒച്ചവെച്ചും നമസ്‌ക്കാരത്തിലെ ശ്രദ്ധമുറിച്ചു കളയാന്‍ സാധ്യതയുള്ളതു പോലെ സ്ത്രീ സാന്നിദ്ധ്യം പുരുഷ ഹൃദയങ്ങളില്‍ പ്രലോഭനങ്ങള്‍ സൃഷ്ടിക്കാനും തന്നിമിത്തം നമസ്‌ക്കാരത്തിലെ ശ്രദ്ധ മുറിയുവാനും ഇടയാക്കുമെന്ന് പഠിപ്പിക്കുക മാത്രമേ ഹദീസ് ചെയ്യുന്നുള്ളൂ. നായ, കഴുത, സ്ത്രീ എന്നിവയുടെ സാന്നിദ്ധ്യം നിരുപാധികം നമസ്‌കാരം മുറിച്ചു കളയുമെന്ന് ഹദീസ് പഠിപ്പിക്കുന്നില്ല.

അപ്പോള്‍ അസ്തിത്വത്തിലെ നീചത്വം കൊണ്ടാണ് ഇവയെല്ലാം നമസ്‌കാരം മുറിച്ചുകളയുന്നതെന്ന് ഹദീസ് ഒരിക്കലും പഠിപ്പിക്കുന്നില്ല മറിച്ച് അവരുടെ സാന്നിദ്ധ്യം സൃഷ്ടിച്ചേക്കാവുന്ന ശ്രദ്ധതെറ്റിക്കലാണ് ഹദീസിന്റെ പ്രതിപാദനം. അങ്ങനെ വരുമ്പോള്‍ വാസ്തവത്തില്‍ ഇവിടെ സ്ത്രീ നിന്ദിക്കപ്പെടുകയാണോ ചെയ്യുന്നത്?! പരോക്ഷമായ സ്തുതിയാണ് ഹദീസിൽ നിന്ന് സ്ത്രീത്വത്തിന് ലഭിക്കുന്നത്. നമസ്‌കാര സമയത്തു പോലും പുരുഷ ഹൃദയങ്ങളെ ആകര്‍ഷിക്കുവാനും സ്വാധീനിക്കുവാനും പ്രലോഭിപ്പിക്കുവാനും സ്ത്രീ സൗന്ദര്യത്തിനു സാധിക്കുമെന്ന് പറയുന്നത് സ്ത്രീകളെ അവമതിക്കലാണെന്ന് തെറ്റുദ്ധരിക്കാന്‍ മാത്രം സാധുക്കളല്ല ഇസ്‌ലാംവിരോധികളെന്ന് ആര്‍ക്കും പ്രത്യേകം പറഞ്ഞു തരേണ്ടതില്ലല്ലോ?. ഇത്രയേ ഉള്ളൂ, ഇസ്‌ലാംവിരോധികള്‍ ഹദീസിനുമേല്‍ അടയിരുന്നു വിരിയിച്ച അധമവിചാരങ്ങളുടെ അവസ്ഥ. ഹദീസുകളല്ല; ഇസ്‌ലാംവിരോധികളുടെ അന്തഃരംഗമാണ് ഇരുട്ട് പരത്തുന്നതെന്നര്‍ഥം. ഹദീസുകളെടുത്തുവെച്ച് തങ്ങളുടെ വികൃതമായ താല്‍പര്യങ്ങള്‍ക്കനുസൃതമായി അതിനെ വ്യാഖ്യാനിക്കുകയും എന്നിട്ട് അത് പ്രവാചകനും ഇസ്‌ലാമിനും മേല്‍ ആരോപിച്ച് പുളകം കൊള്ളുകയും ചെയ്യുക എന്നത് ഇസ്‌ലാംവിരോധികളുടെ സ്ഥിരം പതിവാണ്. വൈജ്ഞാനിക സത്യസന്ധതയില്ലാത്തവര്‍ക്ക് എന്തുമാകാമല്ലോ. പക്ഷെ ഇത്തരം അൽപജ്ഞാനികളുടെ “ശര്‍ഹ്” കേട്ടപ്പോഴേക്കും ചിന്താശേഷിയില്‍ എട്ടുകാലിവല കെട്ടിയ “മഹാ പിള്ള”മാരുടെ കാര്യമാണ് കഷ്ടം.! നബി(സ)യില്‍ നിന്നും നിവേദനം ചെയ്യപ്പെട്ട ഹദീസുകള്‍ ഇസ്‌ലാമിക പണ്ഡിത ലോകം എപ്രകാരമാണ് ഗ്രഹിച്ചതും ഉള്‍കൊണ്ടതുമെന്ന അന്വേഷണത്തില്‍ നിന്നു വേണം സത്യസന്ധമായ ഹദീസ് വിമര്‍ശനം തുടങ്ങേണ്ടതെന്ന വൈജ്ഞാനിക മര്യാദ പോലും ഇസ്‌ലാംവിരോധികള്‍ക്കില്ലെന്ന വസ്തുതക്ക് ഒന്നു കൂടി അടിവരയിടുകയാണിവിടെ. എന്തിനാണ് തങ്ങള്‍ ഹദീസുകളെ വിമര്‍ശിക്കുന്നതെന്ന ബോധം പോലും ഇസ്‌ലാംവിമര്‍ശകര്‍ക്കു നഷ്ടപ്പെട്ടിരിക്കുന്നു. ഒരു ഹദീസ് വായിച്ചപ്പോള്‍ ഇസ്‌ലാംവിരോധികള്‍ക്കു തോന്നിയ അധമവിചാരത്തെ അടിസ്ഥാനപ്പെടുത്തിയല്ല ഹദീസുകള്‍ മനസ്സിലാക്കപ്പെടേണ്ടത്; മറിച്ച് ഇസ്‌ലാമിക പണ്ഡിത ലോകം അതിനെ എപ്രകാരമാണ് ഗ്രഹിച്ചതും വ്യാഖ്യാനിച്ചതും ഉള്‍കൊണ്ടതുമെന്നതിന്റെ അടിസ്ഥാനത്തിലായിരിക്കണം. കാരണം അതാണല്ലോ നബിപാഠങ്ങള്‍ ഏതു തരം സ്വാധീനമാണ് സമൂഹത്തിലുണ്ടാക്കിയത് എന്ന് തിരിച്ചറിയാനുള്ള പ്രത്യക്ഷ മാര്‍ഗം. തെറ്റായ യാതൊരു സ്വാധീനവും സമൂഹത്തില്‍ സൃഷ്ടിച്ചതായി അവകാശപ്പെടാന്‍ സാധ്യമല്ലാത്ത ഒരു ഹദീസ്, ഇസ്‌ലാംവിരോധവായനകള്‍ക്കു തോന്നിയ അധമവിചാരങ്ങളെ അടിസ്ഥാനപ്പെടുത്തി വിമര്‍ശന വിധേയമാക്കുക എന്നത് വൈജ്ഞാനിക സത്യസന്ധതക്ക് ഒരിക്കലും ചേര്‍ന്ന സമീപനമല്ല.

ഹദീസുകള്‍ പെണ്ണിനെ നായയോടും കഴുതയോടും ഉപമിച്ചു എന്ന ഇസ്‌ലാംവിരോധികളുടെ വിമര്‍ശനം പാടേ നുള്ളി കളഞ്ഞത് പ്രവാചക പത്‌നിയായ ആഇശ (റ) തന്നെയാണ്. അതും പ്രവാചകനുമായുള്ള തന്റെ സ്വന്തം അനുഭവത്തെ മുന്‍ നിര്‍ത്തികൊണ്ട്. പ്രസ്തുത സംഭവം ഹദീസ് ഗ്രന്ഥങ്ങളില്‍ നിന്നു തന്നെ നമുക്കു വായിക്കാം.

“ആഇശ(റ)യുടെ അടുക്കല്‍വച്ച്, നമസ്‌ക്കാരത്തെ മുറിക്കുന്നവയാണ് നായ, കഴുത, സ്ത്രീ എന്നിവയെന്ന് പറയപ്പെടുകയുണ്ടായി. അന്നേരം അവര്‍ ചോദിച്ചു: കഴുതകളോടും നായ്ക്കളോടുമാണോ നിങ്ങള്‍ ഞങ്ങളെ സാദൃശ്യപ്പെടുത്തിയിരിക്കുന്നത്? അല്ലാഹുവാണ്, നബി(സ്വ)ക്കും ക്വിബ്‌ലക്കുമിടയിലെ കട്ടിലില്‍ ഞാന്‍ കിടക്കവെ, അവിടുന്ന് നമസ്‌കരിക്കുന്നത് ഞാന്‍ കണ്ടിട്ടുണ്ട്. അന്നേരം എനിക്ക് (വിസര്‍ജനത്തിന്) ആവശ്യം ഉണ്ടാകും. അപ്പോള്‍ എഴുന്നേറ്റിരുന്ന് നബിക്ക് പ്രയാസം വരുത്താന്‍ ഞാന്‍ ഇഷ്ടപ്പെടാത്തതിനാല്‍ കട്ടിലിന്റെ കാലുകളുടെ ഭാഗത്തുകൂടെ ഞാന്‍ ഊര്‍ന്നിറങ്ങും.” (ബുഖാരി 484, മുസ്‌ലിം)

നമസ്‌കരിക്കുന്നവനില്‍ പ്രലോഭനമുണര്‍ത്തി അതിനു ഭംഗം വരുത്താവുന്ന ഒരു കാര്യമായി സ്ത്രീയെ പരാമര്‍ശിച്ചു എന്നതല്ലാതെ, പ്രവാചകന്‍ ഒരിക്കലും സ്ത്രീയെ നായയോടും കഴുതയോടും ഉപമിച്ചിട്ടില്ല. അഥവാ, അസ്തിത്വത്തിലെ നീചത്വം കൊണ്ടാണ് സ്ത്രീ സാന്നിധ്യംമൂലം നമസ്‌കാരം മുറിഞ്ഞുപോകുന്നതെന്ന് പ്രവാചകൻ (സ) പഠിപ്പിച്ചിട്ടില്ലെന്നർത്ഥം. മറിച്ച് അവരുടെ സാന്നിദ്ധ്യം സൃഷ്ടിച്ചേക്കാവുന്ന ശ്രദ്ധതെറ്റിക്കലാണ് പ്രവാചകൻ (സ) ഉദ്ദേശിച്ചത്. അതു വ്യക്തമാക്കി കൊടുക്കാനാണ്, തന്റെ സാന്നിധ്യം തൊട്ടുമുമ്പിൽ ഉണ്ടായിരിക്കെയാണ് പ്രവാചകൻ (സ) പലപ്പോഴും നമസ്ക്കാരം നിർവ്വഹിച്ചിരുന്നതെന്നു ആഇശ (റ) വ്യക്തമാക്കിയതും.

സ്ത്രീ സാന്നിദ്ധ്യം നിരുപാധികം നമസ്‌കാരം മുറിക്കുന്ന കാര്യമായിരുന്നെങ്കില്‍, നബിയുടെ മുമ്പില്‍ സ്ത്രീയായ ആഇശ (റ) പല തവണ നിവര്‍ന്നു കിടന്നിട്ടും ഒരിക്കല്‍ പോലും അതിന്റെ പേരില്‍ തന്റെ നമസ്‌കാരം മുറിച്ചെന്നു പറഞ്ഞ് നബി(സ്വ) അവരെ ആക്ഷേപിച്ചിട്ടില്ല. മാത്രമല്ല നബിയുടെ മുമ്പില്‍ അവരെങ്ങനെ കിടന്നുവോ അതേ രൂപത്തില്‍ തന്നെ അവരെ തുടരാന്‍ അനുവദിച്ചുകൊണ്ടു തന്നെ തന്റെ നമസ്‌കാരം പൂര്‍ത്തീകരിക്കുകയാണ് നബി (സ്വ) ചെയ്തതെന്ന ആഇശയുടെ അനുഭവ സാക്ഷ്യം ഇസ്‌ലാംവിരോധികളുടെ വിമര്‍ശനത്തില്‍ യാതൊരു കഴമ്പുമില്ലെന്ന് വ്യക്തമാക്കുന്നതാണ്. എങ്കില്‍ പിന്നെ അനാവശ്യമായ ഇത്തരം ഉണക്ക വേവലാതികള്‍ മാറ്റിവെച്ച് കാര്യപ്പെട്ട വല്ല ഏര്‍പ്പാടിനും സമയം കണ്ടെത്തുന്നതല്ലേ ഇസ്‌ലാംവിരോധികള്‍ക്കു നല്ലത്. പക്ഷെ, അങ്ങനെയൊക്കെ നേരെ ചൊവ്വായ അര്‍ഥതലങ്ങളൊന്നും ആരും ഹദീസുകള്‍ക്കു മനസ്സിലാക്കികൂടാ എന്നത് ഇസ്‌ലാംവിരോധികളുടെ നിതാന്ത നികൃഷ്ഠ നിര്‍ബന്ധബുദ്ധിയാണ്. അതില്‍ സംഖി, കൃസംഖി, എമു, ഫെമിനി, യുക്തിവാദി… തുടങ്ങിയ ഒരു ജനുസ്സിലും പെട്ട ഇസ്‌ലാംവിരോധികളാരും വ്യത്യസ്തരല്ല എന്നതാണ് വസ്തുത.

വിമർശനം:

“സ്ത്രീകൾക്ക് എഴുത്ത് പഠിപ്പിക്കരുത്” എന്ന് മുഹമ്മദ് നബി തന്റെ അനുചരന്മാരോട് കൽപ്പിച്ചതായി ഹദീസുകളിൽ വന്നിട്ടില്ലേ ?

മറുപടി:

പ്രവാചകന്റെ(സ) കൽപ്പനകളും അധ്യാപനങ്ങളും വിദ്യാഭ്യാസത്തേയും എഴുത്തും വായനയേയും പ്രോത്സാഹിപ്പിക്കുന്നവയാണ്. വിമർശകർ ഉദ്ധരിക്കുന്ന ഈ കള്ള ഹദീസ് പ്രവാചകന്റെ(സ) വാചകമല്ല, അദ്ദേഹത്തിന്റെ പേരിൽ കെട്ടിച്ചമക്കപ്പെട്ടതാണ്. ഈ വസ്തുത സമൂഹത്തെ ബോധ്യപ്പെടുത്താൻ ഉൽപതിഷ്ണുക്കളായ മുസ്‌ലിം പണ്ഡിതന്മാർ, പുരോഗമ വിരോധികളായ ഒരു പറ്റം പൗരോഹിത്യവൃന്ദത്തോട് ആദർശസമരത്തിൽ ഏർപ്പെട്ടിരുന്ന ഒരു കാലഘട്ടമുണ്ടായിരുന്നു.

പതിറ്റാണ്ടുകൾക്ക് മുമ്പ് കേരളത്തിലെ മുസ്‌ലിം സമൂഹം -പ്രത്യേകിച്ച് മുസ്‌ലിം സ്ത്രീകൾ- വിദ്യാഭ്യാസ രംഗത്ത് പിന്നോക്കാവസ്ഥയിലായിരുന്ന ആ കാലഘട്ടത്തിൽ നവോത്ഥാനത്തിന്റെ അമരക്കാരായ കേരളത്തിലെ മുസ്‌ലിം പണ്ഡിതർ നടത്തിയ ഇസ്‌ലാമിക പ്രബോധനങ്ങളുടെ സദ്‌ഫലമാണ് മുസ്‌ലിം കൈരളിയുടെ ശോഭനമായ വിദ്യാഭ്യാസ രംഗം. ഇന്ന് കേരളത്തിൽ, ഓരോ വർഷവും പത്താം ക്ലാസ് റിസൽട്ട് പ്രഖ്യാപിക്കപ്പെടുമ്പോഴും ഏറ്റവും കൂടുതൽ ഏ പ്ലസ്സുകാരുള്ള ജില്ല മലപ്പുറമായി മാറിയിരിക്കുന്നു; ആ ഏ പ്ലസ്സുകാരിൽ കൂടുതൽ പേരും പെൺകുട്ടികളും!! അല്ലാഹുവിന് സ്തുതി.

പണ്ട് സാമൂഹികോദ്ധാരണങ്ങളിൽ നിന്നും സമുദായത്തെ പിന്നോട്ട് വലിച്ചിരുന്ന പൗരോഹിത്യം തന്നെ ‘ഇസ്‌ലാമിന്റെ യഥാർത്ഥ അധ്യാപനങ്ങളെ ഉൾകൊണ്ട്’, തങ്ങൾ പേറി നടന്നിരുന്ന പല കള്ള വൃത്താന്തങ്ങളും അന്ധവിശ്വാസ അനാചാരങ്ങളും ചരിത്രത്തിന്റെ ചവറ്റുകൊട്ടയിലേക്ക് സ്വയം ചുരുട്ടി വലിച്ചെറിയുകയാണുണ്ടായത്. അവർ പോലും കൈയ്യൊഴിഞ്ഞ ഈ വ്യാജ വാറോലകൾ തപ്പി തിരഞ്ഞ് തിരിച്ച് കൊണ്ടുവരാനുള്ള ബദ്ധപ്പാടിലാണ് ആധുനിക ഭൗതീകവാദികൾ !! ഇസ്‌ലാമിനെ വിമർശിക്കാൻ എന്തെങ്കിലുമൊക്കെ വേണ്ടേ ?! ഇത്തരം ചപ്പുചവറുകളും ഭൗതികവാദികൾക്ക് നിലനിൽപ്പിന് അനിവാര്യമായി തീർന്നിരിക്കുന്നു എന്നതാണ് ദുരവസ്ഥ !!

ഈ വിദ്യാഭ്യാസ നവോദ്ധാനത്തിൽ വല്ല പങ്കും ഭൗതീക വാദികൾക്ക് അവകാശപ്പെടാനുണ്ടോ ?! ക്വുർആൻ വാഖ്യങ്ങളും ഹദീസുകളും സ്‌തുതിഘോഷിച്ച് ഈ സാക്ഷരതാ സമരത്തിന് നേതൃത്വം വഹിച്ചത് വക്കം മൗലവി, കെ.എം മൗലവി, ചാലിലകത്ത് കുഞ്ഞഹമ്മദ് ഹാജി, അബുസ്സബാഹ് മൗലവി, എം.സി.സി സഹോദരങ്ങൾ, ഹലീമാബീവി, പി.കെ സുബൈദ തുടങ്ങിയ മുസ്‌ലിം പണ്ഡിതന്മാരും പണ്ഡിതകളും തന്നെയാണ് എന്നിരിക്കെ ചില പിന്നാക്ക സംഘടനകളുടെ കാലഹരണം സംഭവിച്ച പ്രമേയങ്ങളും കള്ള ഹദീസുകളുമൊക്കെ കുത്തിപ്പൊക്കാൻ ഭൗതീകവാദികൾക്ക് എന്ത് ധാർമിക അവകാശമാണുള്ളത് ?!

‘സ്ത്രീകൾക്ക് എഴുത്ത് പഠിപ്പിക്കരുത്, അവർക്ക് തുന്നലും സൂറത്തുന്നൂറും പഠിപ്പിക്കുക…’ എന്നൊക്കെ പ്രവാചകൻ (സ) കൽപ്പിച്ചതായി പറയപ്പെടുന്ന വ്യാജ നിവേദനം പരിശോധന വിധേയമാക്കാം.

1. ആദ്യമായി ഈ നിവേദനങ്ങളുടെ നിവേദക പരമ്പരകൾ (സനദ്) പരിശോദിക്കുക:

* ത്വബ്റാനി തന്റെ മുഅ്ജമിൽ ഉദ്ധരിച്ച നിവേദക പരമ്പര ഇപ്രകാരമാണ്:

حدثنا محمد بن عبدالله الحضرمي قال ثنا محمد بن ابراهيم الشامي قال ناشعيب بن اسحاق عن هشام بن عروة عن ابيه عن عائشة قالت…

നിവേദക പരമ്പരയിൽ ‘മുഹമ്മദ് ഇബ്നു ഇബ്റാഹീം അശ്ശാമി’ എന്ന ‘റാവി’യുണ്ട്. ഇയാൾ കളവു പറയുകയും വ്യാജ നിവേദനങ്ങൾ നിർമ്മിക്കുകയും ചെയ്യുന്ന വ്യക്തിയാണ്.

هذا الحديث لا يصح… قال أبو حاتم بن حبان: كان محمد بن إبراهيم الشامي يضع الحديث على الشاميين لا يحل الرواية عنه إلا عند الاعتبار. روى أحاديث لا أصول لها من كلام رسول الله صلى الله عليه وسلم لا يحل الاحتجاج به.

ഇബ്നുൽ ജൗസി പറഞ്ഞു: “ഈ ഹദീസ് സ്വഹീഹ് അല്ല… ഇബ്നു ഹിബ്ബാൻ പറഞ്ഞു: ‘മുഹമ്മദ് ഇബ്നു ഇബ്റാഹീം അശ്ശാമി’ ശാമുകാരുടെ പേരിൽ ഹദീസുകൾ വ്യാജമായി നിർമ്മിക്കുമായിരുന്നു. അയാളിൽ നിന്ന് ഹദീസ് ഉദ്ധരിക്കൽ അനുവദനീയമല്ല. അല്ലാഹുവിന്റെ ദൂതന്റെ(സ) സംസാരങ്ങളിൽ പെടാത്ത, അവയുടെ യാതൊരു അടിത്തറയുമില്ലാത്ത ഹദീസുകൾ അയാൾ പടച്ചുണ്ടാക്കുമായിരുന്നു. അവ പ്രമാണമായി സ്വീകരിക്കൽ അനുവദനീയമല്ല.” (അൽ മൗദൂആത്ത്: 2/269)

ഇബ്നു അദിയ്യ് പറഞ്ഞു: വിശ്വസ്ഥരായ നിവേദകർക്കെതിരായി, അങ്ങേയറ്റം ദുർബലമായ നിവേദനങ്ങൾ ഉദ്ധരിക്കുന്ന വ്യക്തിയാണയാൾ. അയാളുടെ ഭൂരിഭാഗം ഹദീസുകളും പ്രാമാണികമായ ഹദീസുകൾക്ക് വിരുദ്ധമാണ്. ദാറകുത്നി പറഞ്ഞു: അയാൾ നുണയനാണ്. ഹാകിം, നികാശ് എന്നിവർ പറഞ്ഞു: അയാൾ ധാരാളം കള്ള ഹദീസുകൾ ഉദ്ധരിച്ചിട്ടുണ്ട്. (തഹ്ദീബു തഹ്ദീബ്: 9:13)

* ഹാകിം തന്റെ മുസ്തദ്റകിൽ ഉദ്ധരിച്ച, അതേ നിവേദനത്തിന്റെ മറ്റൊരു നിവേദക പരമ്പര ഇപ്രകാരമാണ്:

حدثنا أبوعلي الحافظ انبأ محمد بن سليمان حدثنا عبدالوهاب بن الضحاك حدثنا شعيب بن اسحاق عن هشام بن عروة عن أبيه عن عائشة رضي الله عنها قالت…

ഈ നിവേദകപരമ്പരയിലെ ‘അബ്ദുൽ വഹാബ് ഇബ്നു ദഹ്ഹാക് ഇബ്നു അബ്ബാൻ അൽ ഉർദി’ നുണയനാണെന്ന് അബൂഹാതിമും, ഇബ്നു ഹജർ അൽഅസ്കലാനിയും വ്യക്തമാക്കുന്നുണ്ട്. (തക്‌രീബു തഹ്‌ദീബ്: 1:626)

ശീഈ സൈറ്റുകളിൽ ഓടുന്ന ഈ നിവേദനത്തിന് വ്യക്തമായ നിവേദക പരമ്പരകൾ പോലുമില്ല എന്നത് സാന്ദർഭികമായി സൂചിപ്പിക്കട്ടെ. നുണയന്മാരിൽ നിന്നും വ്യാജഹദീസ് നിർമ്മാതാക്കളിൽ നിന്നും ഉദ്ധരിക്കുന്നതിന് പുറമെ നിവേദക പരമ്പരയിലെ റാവിമാരെ വ്യക്തമാക്കാതെ “ആരൊക്കെയോ ഉദ്ധരിച്ചു” എന്ന് പറഞ്ഞ് പ്രവാചകന്റെ മേൽ കെട്ടിയുണ്ടാക്കിയ ഇത്തരം കള്ള ഉദ്ധരണികളൊന്നും ഇസ്‌ലാമിൽ പ്രമാണമല്ല.

2. ഇത്തരം വ്യാജ ഉദ്ധരണികൾ ഇസ്‌ലാമിലെ പ്രമാണങ്ങളായ ക്വുർആനിനും സ്വഹീഹായ ഹദീസുകൾക്കും എതിരാണ് എന്നതാണ് മറ്റൊരു വസ്തുത. ഇസ്‌ലാമിന്റേയും മുഹമ്മദ് നബിയുടേയും(സ) സ്ഥിരപ്പെട്ട എല്ലാ അധ്യാപനങ്ങൾക്കും ഈ വ്യാജ വൃത്താന്തം എതിരാണ്.

അറിവിനും വിദ്യാ സമ്പാധനത്തിനും അളവില്ലാത്ത പ്രാധാന്യം നൽകിയ മതമാണ് ഇസ്‌ലാം. പ്രവാചകന് (സ) ആദ്യമായി അവതരിപ്പിക്കപ്പെട്ട ദിവ്യബോധനത്തിലെ രണ്ട് വരി ഇപ്രകാരമായിരുന്നു:

“നീ വായിക്കുക നിന്‍റെ രക്ഷിതാവ് ഏറ്റവും വലിയ ഔദാര്യവാനാകുന്നു. പേന കൊണ്ട് പഠിപ്പിച്ചവന്‍…” (ക്വുർആൻ: 96:3,4)

‘ക്വലം’ അഥവാ പേന എന്ന പേരിൽ ഒരു അധ്യായം തന്നെ ക്വുർആനിലുണ്ട്. ആ അധ്യായം ആരംഭിക്കുന്നത് തന്നെ ഇപ്രകാരമാണ്: “നൂന്‍. പേനയും അവര്‍ എഴുതിവെക്കുന്നതും സാക്ഷി…” (ക്വുർആൻ: 68:1)

മനുഷ്യർക്ക് ദൈവം നൽകിയ ഏറ്റവും വലിയ അനുഗ്രഹവും അവന്റെ ഏറ്റവും വലിയ ദൃഷ്ടാന്തവുമായാണ് പേനയെ ക്വുർആൻ കാണുന്നത്. അതുകൊണ്ടെല്ലാം തന്നെ എഴുത്ത് പഠിക്കുന്നതിനെ പ്രവാചകൻ (സ) അങ്ങേയറ്റം പ്രോത്സാഹിപ്പിച്ചു.

“അൻസ്വാരി സ്ത്രീകൾ എത്ര നല്ലവരാണ്. മതപാണ്ഡിത്യം നേടുന്ന കാര്യത്തിൽ ലജ്ജ അവരെ തടയുന്നില്ല” (സ്വഹീഹു മുസ്‌ലിം: 332) എന്ന് ശ്ലാഘിച്ചു കൊണ്ട്, അറിവിന്റേയും പഠനത്തിന്റേയും വിഷയത്തിൽ അന്തർമുഖരായ സ്ത്രീ ജനങ്ങളെ മുന്നോട്ടാനയിക്കുകയാണ് പ്രവാചകൻ (സ) ചെയ്തത്. സ്ത്രീ അറിവു നേടുന്നതിൽ തടസ്സമായി ഒന്നും തന്നെ നില നിൽക്കരുത് എന്ന വിപ്ലവ പാഠം ലോകത്തിന് മുമ്പിൽ വിളംബരം ചെയ്ത മുഹമ്മദ് നബി (സ) അറിവിന്റെ സുപ്രധാന സ്രോതസ്സായ വായനയും മാധ്യമമായ എഴുത്തും അവർക്കു തടഞ്ഞു എന്ന് വിഡ്ഢികളല്ലാതെ ആരാണ് വിശ്വസിക്കുക.?! അടിമ സ്ത്രീകളുടെ വിദ്യാഭ്യാസത്തെ പോലും പ്രവാചകൻ (സ) പ്രചോദിപ്പിക്കുകയാണുണ്ടായത്: “തന്റെ കീഴിലുള്ള അടിമ സ്‌ത്രീയെ സംസ്കാര സമ്പന്നയാക്കുകയും അവൾക്ക്‌ ഏറ്റവും നന്നായി വിദ്യാഭ്യാസം നൽകുകയും പിന്നീട്‌ അവളെ മോചിപ്പിച്ച്‌ സ്വയം വിവാഹം കഴിക്കുകയും ചെയ്തവനും ഇരട്ടി പ്രതിഫലമുണ്ട്‌” (ബുഖാരി: 5083).

ബദ്ർ യുദ്ധാനന്തരം ബന്ധികളാക്കപ്പെട്ട പടയാളികൾക്ക് മോചനദ്രവ്യമായി നിശ്ചയിക്കപ്പെട്ടിരുന്നത് നാൽപത് ഊക്കിയ വെള്ളി, അഥവാ നാലായിരം വെള്ളി ദിർഹമുകളായിരുന്നു മോചനദ്രവ്യം. അത് നൽകാൻ കഴിയാത്തവർക്ക് പത്ത് മുസ്‌ലിം കുട്ടികൾക്ക് എഴുത്ത് പഠിപ്പിക്കൽ മോചനദ്രവ്യമായി പ്രവാചകൻ (സ) നിശ്ചയിച്ചു.

ഇബ്നു അബ്ബാസ് (റ) പറഞ്ഞു: ബദ്ർ യുദ്ധാനന്തരം ബന്ധികളാക്കപ്പെട്ടവരിൽ ചിലർക്കും മോചനദ്രവ്യം നൽകാൻ ഒന്നുമുണ്ടായിരുന്നില്ല. അൻസ്വാരികളുടെ കുട്ടികൾക്ക് ‘എഴുത്ത് പഠിപ്പിക്കൽ’ മോചന മാർഗമായി അല്ലാഹുവിന്റെ ദൂതൻ അവർക്ക് നിശ്ചയിച്ച് കൊടുത്തു. (മുസ്നദ് അഹ്മദ്: 2217)

പുരുഷന്മാർക്ക് പുറമെ, അക്കാലഘട്ടത്തിലെ ധാരാളം സ്ത്രീകൾക്കും എഴുത്തും വായനയും അറിയാമായിരുന്നുവെന്ന് ചരിത്രകാരനും ഭൂമിശാസ്‌ത്രഗ്രന്ഥകാരനുമായ ബലാദുരി വ്യക്തമാക്കുന്നുണ്ട്. (ഫുതൂഹുൽ ബുൽദാൻ: 581)

ഹഫ്‌സ ബിൻത്ത് ഉമർ, ശിഫാഅ് ബിൻത്ത് അബ്ദുശ്ശംസ്, ഉമ്മു കുൽസൂം ബിൻത്ത് ഉക്ബ, ആഇശ ബിൻത്ത് സഅ്ദ്, കരീമ ബിൻത്ത് മിക്‌ദാദ് എന്നിവർ ഉദാഹരണം. (ഫുതൂഹുൽ ബുൽദാൻ: 581, അൽ ഇസ്തീആബ്: 4: 1811,1869,1953, ത്വബകാത്തു ഇബ്നു സഅ്ദ്: 8: 467, തക്‌രീബു തഹ്ദീബ്: 2: 857)

എഴുത്തും വായനയുമറിയാവുന്ന സ്ത്രീകളെ കൊണ്ട് തന്റെ പത്നിമാർക്കും എഴുത്തും വായനയും പഠിപ്പിക്കാനും പ്രവാചകൻ (സ) ശ്രമിച്ചിരുന്നു. ശിഫാഅ് ബിൻത്ത് അബ്ദുശ്ശംസ് പ്രവാചക പത്നി ഹഫ്‌സക്ക് എഴുത്ത് പഠിപ്പിക്കുന്ന സന്ദർഭത്തിൽ വീട്ടിൽ കയറി വന്ന പ്രവാചകൻ (സ), അധ്യാപികയോട് “താങ്കൾ അവൾക്ക് (ഹഫ്‌സ) എഴുത്തു പഠിപ്പിച്ചതു പോലെ രോഗ പ്രാർത്ഥനയും പഠിപ്പിച്ചു കൂടേ?” എന്ന് ചോദിച്ചതായി ഹദീസുകളിൽ കാണാം. (സ്വഹീഹു അബൂദാവൂദ്: 3887)

ഇസ്‌ലാമിക ചരിത്രത്തെ സംബന്ധിച്ച് അൽപ്പമെങ്കിലും പരിജ്ഞാനമുള്ള ആരും തന്നെ ഇസ്‌ലാം, സ്ത്രീകൾ കയ്യെഴുത്ത് പരിശീലിക്കുന്നതിനെ എതിർത്തുവെന്ന് പറയാൻ യാതൊരു സാധ്യതയുമില്ല. കാരണം ഇസ്‌ലാമിക ചരിത്രത്തിലുടനീളം പുരുഷന്മാരേക്കാൾ സ്ത്രീകൾ അതിവിദൂരം മുന്നിട്ടു നിന്ന പ്രത്യേകം ചില മേഖലകളിലൊന്നായിരുന്നു ‘കയ്യെഴുത്ത്’. ചില ഉദാഹരണങ്ങൾ കുറിക്കാം:

* അച്ചടി യന്ത്രമോ ആധുനിക സാങ്കേതിക ഉപകരണങ്ങളോ നിലവിലില്ലായിരുന്ന മധ്യ കാലഘട്ടത്തിൽ ഗ്രന്ഥങ്ങളുടെ പ്രതികൾ കൈപ്പടം കൊണ്ട് പകർത്തി എഴുതുന്ന തൊഴിൽ ചെയ്തിരുന്നവരെ ‘വർറാക്കു’കൾ (الوراقون) എന്നാണ് അറിയപ്പെട്ടിരുന്നത്. പൗരാണിക കാലഘട്ടത്തിലെ അച്ചടിശാലകളായിരുന്നു അവർ. സ്ത്രീകൾ ചേതോഹരമായ കൈയ്യക്ഷരത്തിന് ഉടമകളായത് കൊണ്ട് തന്നെ അവർ തന്നെയായിരുന്നു ഈ മേഖലയിൽ പ്രധാനികൾ.

* കോർഡോവയിൽ നൂറ്റി എഴുപത് സ്ത്രീകൾ കൂഫി ലിപിയിൽ മുസ്ഹഫുകൾ ഇറക്കിയിരുന്നുവെന്ന് സ്പെയിനിലെ ചരിത്രകാരനായ അബുൽഫിയാദ് രേഖപ്പെടുത്തുന്നുണ്ട്.

* കവയത്രി ത്വലൈത്വലിയ (ഹി. 540) ഇക്കൂട്ടത്തിൽ പെടുന്നു.

* ബാഗ്‌ദാദിലെ ഏറ്റവും വലിയ അച്ചടിശാലയിൽ പ്രൂഫ് റീഡിംഗിലും പകർപ്പിലും പങ്കെടുത്തിരുന്ന ജാരിയ്യ തൗഫീക് സൗദാഇനെ സംബന്ധിച്ച് സാഹിത്യകാരൻ അബുൽ അലാഅ് മഅരി സ്മരിക്കുന്നുണ്ട്.

* ആറാം നൂറ്റാണ്ടുകാരിയായ പണ്ഡിത മർയം ബിൻത് അബ്ദുൽ കാദിർ, അബൂ നസ്ർ അൽജൗഹരിയുടെ ‘കാമൂസു സ്വിഹാഹ്’ എന്ന ഗ്രന്ഥം പകർത്തിയെഴുതിയിട്ടുണ്ട്, അതിന്റെ പ്രതി ബാഗ്‌ദാദിലെ ഹൈദർഖാന ലൈബ്രറിയിൽ സൂക്ഷിക്കപ്പെട്ടിട്ടുമുണ്ട്. ഗ്രന്ഥത്തിന്റെ അവസാനത്തിൽ അവർ ഇപ്രകാരം രേഖപ്പെടുത്തിയതായി കാണാം : “ഈ പ്രതിയിൽ വല്ല സ്ഖലിതങ്ങളും ശ്രദ്ധയിൽ പെട്ടാൽ വായനക്കാർ എന്നോട് പൊറുക്കുമെന്ന് ഞാൻ ആശിക്കുന്നു. കാരണം, എന്റെ വലതു കൈ കൊണ്ട് ഞാൻ ഗ്രന്ഥം പകർത്തുമ്പോൾ ഇടതു കൈ കൊണ്ട് എന്റെ കുഞ്ഞിനെ ഞാൻ തൊട്ടിലിൽ ആട്ടുകയും ചെയ്യുമായിരുന്നു.”

* ഹിജ്റ 374ൽ മരണപ്പെട്ട ലുബ്ന അൽ ഖുർത്വബിയ, ഇസ്‌ലാമിക് സ്പെയിനിലെ കവിയത്രി, കൈയെഴുത്തുശാസ്‌ത്രത്തിലും (calligraphy) ലിപിന്യാസത്തിലും (penmanship) അഗ്രേസരയായിരുന്നു. സ്പെയിനിലെ ഉമവി ഖലീഫയായിരുന്ന അൽ ഹകം അൽ മുസ്തൻസിർബില്ലായുടെ കൊട്ടാര എഴുത്തുകാരിയായിരുന്നു. ബഹുമുഖപ്രതിഭയായിരുന്ന ഇവർ ശാസ്‌ത്രചിന്തകയും ഗണിതജ്ഞയും വ്യാകരണപണ്ഡിതയുമെല്ലാമായിരുന്നു. ഒരു അടിമ പെൺകുട്ടിയായിട്ടാണ് ജനിച്ചതെങ്കിലും ഖലീഫയുടെ സെക്രട്ടറിയായിത്തീർന്നു. ഖുർത്വുബയിലെ സഹ്റാഅ നഗരത്തിലെ പ്രശസ്തമായ ലൈബ്രറിയുടെ നിർമ്മാണത്തിലും നടത്തിപ്പിലും വിജ്ഞാനകുതുകിയായ ഇവർ നിസ്തുലമായ പങ്ക് വഹിച്ചു. എണ്ണമറ്റ കൈയ്യെഴുത്തു പ്രതികൾ ലൈബ്രറിക്കായി വിവർത്തനം ചെയ്തു.

* ഹിജറാബ്ദം 440 ൽ മരണപ്പെട്ട, പണ്ഡിതയും എഴുത്തുകാരിയുമായ ‘ബിൻത്തുൽ അക്റഅ് ‘ജനങ്ങളെ മനോഹരമായ കൈപ്പടയിൽ എഴുതാൻ പഠിപ്പിച്ചിരുന്നു. അന്നത്തെ ഖലീഫയുടെ എഴുത്തുകുത്തുകൾ കൈകാര്യം ചെയ്യാൻ ഔദ്യോഗികമായി നിയോഗിതയായ ഇവരുടെ എഴുത്ത് അക്കാലഘട്ടത്തിലെ മികച്ച കയ്യെഴുത്തിന് മാതൃകയായി അവലംബിക്കപ്പെട്ടു എന്ന് ചരിത്ര ഗ്രന്ഥമായ സിയറു അഅ്ലാമിന്നുബലാഅ് വർണിക്കുന്നു.

* ഉമ്മു മുഹമ്മദ് ശുഹ്ദ ബിൻത്ത് അഹ്മദ്. ഹിജ്റാബ്ദം 484 ഇറാഖിലെ ബാഗ്‌ദാദിൽ ജനനം. ഹദീസ് പണ്ഡിത; ഇമാം മാലികിൻ്റെ മുവത്വയിലെ ഹദീഥുകൾ മുഴുവനായും ചെറുപ്രായത്തിലെ മനപാഠമാക്കി. സാഹിത്യകാരി, പ്രാസംഗിക, ‘കൈയെഴുത്ത് കലാ പ്രതിഭ’ തുടങ്ങി സകല കലാ വല്ലഭയായ ശുഹ്ദ ‘സ്ത്രീകൾക്ക് അഭിമാനഹേതുവായി’രുന്നതിനാൽ ‘ഫഖ്റുന്നിസാഅ്’ എന്ന് വിളിക്കപ്പെട്ടു. ഫഖ്റുന്നിസയുടെ കീഴിൽ ഹദീഥ് പഠിക്കുക എന്നത് ഇസ്‌ലാമിക ലോകത്തെ നാനാ ദിക്കുകളിൽ നിന്നുമുള്ള ഹദീസ് പണ്ഡിതർ അനുഗ്രഹവും അഭിമാനവുമായി പരിഗണിച്ചു. ഇബ്നു അസാകിർ, സംആനി, ഇബ്നുൽ ജൗസി തുടങ്ങിയ എണ്ണമറ്റ വിശ്വപ്രസിദ്ധ പണ്ഡിതർ ഫഖ്റുന്നിസയിൽ നിന്നും ഹദീസ് പഠിച്ചവരാണ്.

* ഹിജ്റാബ്ദം 460 ൽ മരണപ്പെട്ട ഹദീസ് പണ്ഡിതയും പ്രഭാഷകയുമായ ആഇശ ബിൻത്ത് ഹസനിബ്നു ഇബ്റാഹീം അൽ ഇസ്ബഹാനിയ്യയുടെ പാണ്ഡിത്യത്തെ സംബന്ധിച്ച് ഇമാം ദഹബി, സിയറിൽ വാനോളം വാഴ്ത്തുന്നുണ്ട്. ഇബ്നു മന്ദയുടെ ആമാലി എന്ന ഗ്രന്ഥം അദ്ദേഹത്തിൽ നിന്ന് അവർ നേരിട്ട് കേട്ടെഴുതിയ കയ്യെഴുത്തു പ്രതിയായിരുന്നു ഗ്രന്ഥത്തിന്റെ പ്രചാരണത്തിന്റെ പ്രധാന അവലംബം.

ഇമാം ദഹബിയുടെ സിയറു അഅ്ലാമിന്നുബലാഅ്, ഇമാം ഇബ്നു ഹജറിന്റെ അൽ ഇസ്വാബ, ഇമാം മിസ്സിയുടെ തഹ്ദീബുൽ കമാൽ, ഇമാം ഇബ്നു കസീറിന്റെ അൽ ബിദായത്തു വന്നിഹായ, ഉമർ രിദായുടെ അഅ്ലാമുന്നിസാഅ് തുടങ്ങിയ ഗ്രന്ഥങ്ങളിൽ പ്രവാചക കാലഘട്ടം മുതൽ മധ്യകാലഘട്ടം വരെയുള്ള എഴുത്തുകാരികളായ ആയിരക്കണക്കിന് മുസ്‌ലിം പണ്ഡിതകളുടെ ജീവചരിത്രം നമുക്ക് വായിക്കാൻ സാധിക്കും.

ഹിജറാബ്ദം 384 ൽ ജനിച്ച ലോക പ്രസിദ്ധ കർമ്മശാസ്ത്ര പണ്ഡിതനായ ഇബ്നു ഹസം പൗരാണിക ഇസ്‌ലാമിക സമൂഹത്തിലെ പണ്ഡിതകളെ സംബന്ധിച്ച് വിശദീകരിക്കവെ അവരുടെ ‘കയ്യെഴുത്തി’നെ സംബന്ധിച്ച് പ്രത്യേകം പരാമർശിക്കുന്നുണ്ട്.

“സ്ത്രീകളുടെ മടിത്തട്ടിലാണ് ഞാൻ വളർന്നത്, അവരുടെ മുമ്പിലാണ് ഞാൻ പിച്ചവെച്ചതും. വലുതായതിന് ശേഷമാണ് പുരുഷന്മാരുമായി ഞാൻ സഹവസിക്കാൻ ആരംഭിച്ചത്… (അതിന് മുമ്പേ) സ്ത്രീകളാണ് എന്നെ കുർആൻ പഠിപ്പിച്ചത്. ധാരാളം കവിതകൾ പാടാനും ‘നല്ല കൈപ്പടയിൽ എഴുതാനും’ എന്നെ പരിശീലിപ്പിച്ചതും അവർ തന്നെയായിരുന്നു…”

വിമർശനം:

പ്രവാചക ശിഷ്യൻ സുബൈർ (റ), തന്റെ ഭാര്യയെ സ്ഥിരമായി തല്ലാറുണ്ടായിരുന്നു.

മറുപടി:

സംഭവം സ്ഥിരപ്പെട്ട പരമ്പരയോടെ ഉദ്ധരിക്കപ്പെട്ടിട്ടില്ല. നിവേദനങ്ങളുടെ പരമ്പരകൾ ദുർബലമാണ്.

സുബൈർ (റ) തന്റെ ഭാര്യയായ അസ്മാഅ് ബിൻത്ത് അബൂബക്കറിനെ സ്ഥിരമായി തല്ലാറുണ്ടായിരുന്നു എന്ന് ചില നിവേദനങ്ങൾ പല ഗ്രന്ഥങ്ങളിലും വന്നിട്ടുണ്ടെങ്കിലും എല്ലാ നിവേദനങ്ങളും വളരെ ദുർബലമാണ്.

1. ‘അസ്മാഅ് (റ) തന്റെ ഭർത്താവ് സുബൈറിനെ (റ) സംബന്ധിച്ച് തന്റെ പിതാവിനോട് പരാധിപ്പെട്ടു’ എന്ന് സൂചിപ്പിക്കുന്ന ചില നിവേദനങ്ങൾ ഇബ്നു സഅ്ദ്, ‘ത്വബകാത്തുൽ കുബ്റാ’ യിലും (8:251), ഇബ്നു അസാകിർ, ‘താരീഖു മദീനത്തു ദിമശ്ക്കി'(69:15)ലും അബ്ദുർറസാക് തന്റെ ‘മുസ്വന്നഫി’ലും (20599) ഉദ്ധരിക്കുന്നുണ്ട്. ‘ഇക്‌രിമ’യിൽ നിന്നാണ് സംഭവം നിവേദനം ചെയ്യപ്പെടുന്നത്. എന്നാൽ, ഇക്‌രിമ ഈ സംഭവത്തിന് സാക്ഷിയല്ല. അദ്ദേഹം അബൂബക്കറിനെ (റ) കണ്ടുമുട്ടിയിട്ടില്ല, അസ്മാഇൽ നിന്ന് നേരിട്ട് കേട്ടതാണെന്ന് വ്യക്തമാക്കിയിട്ടുമില്ല. അതിനാൽ നിവേദനം മുർസൽ (المرسل) അഥവാ പരമ്പര മുറിഞ്ഞത് ആകുന്നു. (സിൽസിലത്തു സ്വഹീഹ: 3:276)

ورجاله ثقات لكن عكرمة لم يدرك القصة ولم يتبين لي هل سمع من أسماء أم لا فيتحتمل أنه أخذه منها… (أرشيف منتدى الألوكة – مسألة ضرب الزوجات وهل أثر الزبير رضي الله عنه صحيحا – المكتبة الشاملة الحديثة )

ഇനി ഇബ്നു സഅ്ദ്, ഇബ്നു അസാകിർ,അബ്ദുർറസാക് തുടങ്ങിയവർ ഉദ്ധരിച്ച ഈ നിവേദനങ്ങൾ സ്വഹീഹ് (സ്വീകാര്യതയുടെ മാനദണ്ഡങ്ങൾ പൂർത്തിയായവ) ആണെന്ന് വന്നാൽ തന്നെ സുബൈർ (റ) തന്റെ ഭാര്യയായ അസ്മാഇനെ തല്ലുമായിരുന്നെന്ന് ഈ നിവേദനങ്ങളിൽ പ്രസ്ഥാവിക്കുന്നേ ഇല്ല.! അസ്മാഅ്, തന്റെ പിതാവ് അബൂബക്കറിന്റെ അടുത്തു വന്ന്, ഭർത്താവ് സുബൈർ തന്നോട് “പാരുഷ്യം കാണിക്കുന്നു എന്ന് പരാധിപ്പെട്ടു” (فشكت شدة الزبير عليها) എന്നാണ് ഈ നിവേദനങ്ങളിൽ പ്രസ്ഥാവിക്കപ്പെട്ടിട്ടുള്ളത്. സുബൈർ (റ) ലൗകിക വിരക്തനായതിനാൽ, സമ്പത്ത് ചെലവഴിക്കുന്നതിലുള്ള അദ്ദേഹത്തിന്റെ കാർക്കശ്യമാണ് ഇവിടെ അസ്മാഅ് പരാധിപ്പെടുന്ന ‘പാരുഷ്യം’ എന്ന് പല പണ്ഡിതന്മാരും വ്യക്തമാക്കിയിട്ടുണ്ട്.

ഭർത്താക്കന്മാരെ സംബന്ധിച്ച് നവ വധുക്കൾ പരാധി പറയുക സാധാരണ കാര്യമാണ്. തന്നെ സ്നേഹിക്കുന്നില്ല, തന്നോട് മോശമായി പെരുമാറുന്നു എന്നൊക്കെ മാതാപിതാക്കളോട് പരാതി പറയുന്നത് എല്ലാം ശരിയാവണമെന്നില്ലല്ലൊ. അവ കേവല ആത്മനിഷ്ടമായ തോന്നലുകൾ ആവാം… നിസ്സാരമായ കാര്യങ്ങളെ പർവ്വതീകരിക്കലുമാവാം… എന്നൊക്കെ പിതാവ് അബൂബക്കർ മകളുടെ പിണക്കത്തെ പറ്റി മനസ്സിലാക്കിയുള്ളു. അതുകൊണ്ട് ദാമ്പത്യ ജീവിതത്തിലെ സംയമനത്തേയും ക്ഷമയേയും പറ്റി മകളെ ഉപദേശിച്ചു. ഭർത്താവായ സുബൈറിനെ ‘സദ്‌വൃത്തനായ ഭർത്താവ്’ (زوج صالح) എന്ന് അബൂബകർ (റ) വിശേഷിപ്പിക്കുകയും ചെയ്തു എന്ന് അതേ നിവേദനത്തിൽ കാണാം.

വളരെ തരള ഹൃദയനായിരുന്നു അബൂബക്കർ (റ). അതുകൊണ്ടു തന്നെ, വളരെ അലിവോടും വാൽസല്യത്തോടു കൂടെയുമാണ് അദ്ദേഹം തന്റെ പെൺമക്കളെ വളർത്തിയത്. അത്ര ലാളനയിലും വാൽസല്യത്തിലും വളർന്ന പെൺകുട്ടികൾക്ക് വിവാഹാനന്തരം ചെറിയ ജീവിത മാറ്റങ്ങൾ തന്നെ അസഹ്യമായി അനുഭവപ്പെടുക സ്വഭാവികമാണ്. അബൂബക്കറിന്റെ(റ) മറ്റൊരു മകളായ ആഇശക്ക്(റ) ഈ മാറ്റം അനുഭവപ്പെടാതിരുന്നത് പിതാവ് അബൂബക്കറിനേക്കാൾ സ്നേഹവും കരുണയും വാൽസല്യവും ഭർത്താവായ പ്രവാചകനിൽ (സ) നിന്ന് ലഭിച്ചതു കൊണ്ടാകാം. പ്രവാചകനെ പോലെ അലിവിന്റെ ഖനിയാവാനും ഭാര്യമാരെ പൂർണമായും തൃപ്തിപ്പെടുത്താനും എല്ലാ ഭർത്താക്കന്മാർക്കും കഴിഞ്ഞെന്നു വരില്ലല്ലൊ. അത്രയെ സുബൈറിന്റെ ‘തെറ്റായി’ ഈ നിവേദത്തിൽ നിന്ന് മനസ്സിലാക്കാനുള്ളു.

അതേ സമയം സുബൈർ മകളെ തല്ലുന്നതായി പരാധിപ്പെട്ടിരുന്നെങ്കിൽ അബൂബക്കർ (റ) മകളെ ദാമ്പത്യ ജീവിതത്തിലെ സംയമനത്തേയും ക്ഷമയേയും പറ്റി ഉപദേശിക്കുകയല്ല, ശക്തമായി തന്നെ പ്രതികരിക്കുകയിരുന്നു ചെയ്യുക. തന്നെ തല്ലുന്നതായി മകൾ പരാധിപ്പെട്ടതായി സ്വഹീഹായ ഒരു നിവേദനത്തിലും കാണുന്നില്ല.

2. സുബൈർ (റ) ഭാര്യയായ അസ്മാഇനെ (റ) “തല്ലിയിരുന്നു” എന്നോ “തല്ലി” എന്നോ സൂചിപ്പിക്കുന്ന നിവേദനങ്ങൾ താഴെ പറയുന്നവയാണ്:

a) ത്വബ്റാനി തന്റെ ‘മുഅ്ജമുൽ കബീറി’ൽ (234) ഉദ്ധരിച്ച കഥയുടെ പരമ്പര:

حَدَّثَنَا أَحْمَدُ بن زَيْدِ بن هَارُونَ، حَدَّثَنَا إِبْرَاهِيمُ بن الْمُنْذِرِ الْحِزَامِيُّ، حَدَّثَنَا عَبْدُ اللَّهِ بن مُحَمَّدِ بن يَحْيَى بن عُرْوَةَ، عَنْ هِشَامِ بن عُرْوَةَ، قَالَ:…

പരമ്പരയിലെ അബ്ദുല്ലാഹിബ്നു മുഹമ്മദിബ്നു യഹ്‌യ എന്ന നിവേദകൻ ദുർബലനാണെന്ന് ഇമാം ഹൈസമി ‘മജ്‌മഉസ്സവാഇദ്’ ൽ വ്യക്തമാക്കിയിരിക്കുന്നു.

وفيه عبد الله بن محمد بن يحيى بن عروة وهو ضعيف

മാലികിൽ നിന്ന് ഇബ്നുൽ അറബി, കുർതുബി എന്നിവർ സുബൈറിന്റെ(റ) ‘അടിയെ’ സൂചിപ്പിക്കുന്ന സംഭവം ഉദ്ധരിച്ചിട്ടുണ്ട്. നിവേദനത്തെ സംബന്ധിച്ച് ‘ഉപോൽബലകമായ നിവേദകസാക്ഷ്യങ്ങളില്ലാത്ത ഒറ്റപ്പെട്ട നിവേദനം’ എന്ന് കാദി അബൂബക്കറും, സംഭവം തന്റെ ഗ്രന്ഥത്തിൽ എടുത്തുദ്ധരിച്ച ഇബ്നുൽ അറബി തന്നെയും വ്യക്തമാക്കിയിട്ടുണ്ട്. കൂടാതെ നിവേദനം ഉദ്ധരിച്ച മാലിക് കഥക്ക് സാക്ഷിയല്ല എന്നതു കൂടി ചരിത്രത്തിൽ നിന്ന് വ്യക്തമാണ്. കാരണം മാലിക് (ജനനം: ഹിജ്റ: 93) അസ്മാഅ്നെ (മരണം: ഹിജ്റ: 73) ജീവിതത്തിൽ ഒരിക്കലും കണ്ടുമുട്ടിയിട്ടില്ല. അതിനാൽ തന്നെ പരമ്പര മുറിഞ്ഞതും ദുർബലവുമാണ് എന്ന് വ്യക്തം.

3. “അല്ലാഹുവിന്റെ ദൂതൻ (സ) തന്റെ കൈ കൊണ്ട് ഒരു സ്ത്രീയെ പോലും ഒരിക്കലും അടിച്ചിട്ടില്ലെന്ന്…” (സ്വഹീഹു മുസ്‌ലിം: 2328) പത്നി ആഇശ (റ) പറയുന്നു. “അല്ലാഹുവിന്റെ അടിയാത്തികളെ നിങ്ങൾ അടിക്കരുത്…” (لَا تَضْرِبُوا إِمَاءَ اللَّهِ) എന്നാണ് പ്രവാചകൻ (സ) തന്റെ ശിഷ്യന്മാരെ ഉപദേശിച്ചത്. (അബൂദാവൂദ്: 2146) “നാണമില്ലേ നിങ്ങള്‍ക്ക്? അടിമയെ അടിക്കുന്നതുപോലെ സ്വന്തം ഭാര്യയെ അടിക്കാന്‍; പിന്നെ അവളോടൊത്ത് ശയിക്കാനും” (സ്വഹീഹുൽ ബുഖാരി: 4908, മുസ്വന്നഫ് അബ്ദുർറസാക്ക്) എന്ന് ഭാര്യമാരെ അടിക്കുന്ന ചിലരുടെ നിലപാടിനെ പ്രവാചകൻ (സ) ശക്തമായി അപലപിച്ചു.

അതുകൊണ്ടു തന്നെ, പ്രവാചകന്റെ(സ) മാതൃകയെ കണിശമായി പിന്തുടരുന്ന, അദ്ദേഹത്തിന്റെ കൽപനയെ സമ്പൂർണമായും ശിരസാവഹിക്കുന്ന പ്രവാചകാനുചരന്മാർ ഒരിക്കലും തങ്ങളുടെ ഭാര്യമാരെ ഗാർഹിക പീഢനങ്ങൾക്ക് വിധേയമാക്കില്ലെന്നത് തീർച്ചയാണ്. വിശിഷ്യാ പ്രവാചകന്റെ(സ) സന്ധത സഹചാരിയായ സുബൈർ (റ) തന്റെ ഭാര്യയെ സ്ഥിരമായി തല്ലുകയെന്നത് അസംഭവ്യമാണ്.

4. അതിഗുരുതരമായ സ്വഭാവദൂഷ്യങ്ങൾ ഭാര്യമാരിൽ നിന്ന് പ്രകടമാകുന്ന സന്ദർഭത്തിൽ ഭാര്യമാരെ ഉപദേശിച്ച് മര്യാദ പഠിപ്പിക്കാനും, അതുകൊണ്ടും കാര്യമില്ലെന്ന് കണ്ടാൽ കിടപ്പറയിൽ വിട്ടു കിടന്ന് പിണക്കം പ്രകടിപ്പിക്കാനും, അതുകൊണ്ടും ഫലമില്ലെങ്കിൽ ‘തല്ലാനും’ ക്വുർആൻ (സൂറത്തു ന്നിസാഅ്: 34) അനുവാദം നൽകുന്നുണ്ടെങ്കിലും ആ തല്ല് ശാരീരികമായി വേദനപ്പിക്കുന്നതാവരുത്, ആ തല്ലിലൂടെ തന്റെ ശക്തമായ പ്രതിഷേധമറിയിക്കാനുള്ള ഒരു അടയാളം മാത്രമാകണമെന്നാണ് പ്രവാചകൻ (സ) വ്യഖ്യാനിച്ചത്. “വേദനിപ്പിക്കാത്ത അടി” (ضربا غير مبرح) എന്നു തന്നെ പ്രവാചകൻ (സ) ആ അടിയെ വിശേഷിപ്പിക്കുകയും ചെയ്തു. (സ്വഹീഹു മുസലിം: 2138)

ഇബ്നു കസീർ പറഞ്ഞു: “അവളെ അടിക്കുക എന്ന് ക്വുർആൻ പറഞ്ഞത് ഉപദേശം കൊണ്ടോ പിണക്കം കൊണ്ടോ പരിഹാരമില്ലാത്ത സന്ദർഭത്തിലാണ്. അതു തന്നെ ശക്തമല്ലാത്ത അടിയാണ് അനുവദിച്ചത്… ഹസനുൽ ബസ്വരി പറഞ്ഞു: “പാടൊ അടയാളമൊ പതിയാത്ത അടിയാണിത്.” കർമ്മശാസ്ത്ര പണ്ഡിതർ പറഞ്ഞു: അവയവങ്ങൾക്ക് മുറിവോ വേദനയോ, അടയാളമോ ഒന്നും ഉണ്ടാകാത്ത അടിയാണ് ഉദ്ദേശ്യം.” (തഫ്സീറു ഇബ്നുകസീർ: 2:293)

ഇമാം കുർത്തുബി പറഞ്ഞു: “ഉപദേശം കൊണ്ടും പിണക്കം കൊണ്ടുമാണ് പ്രശ്നപരിഹാരത്തിനുള്ള ശ്രമം ആരംഭിക്കേണ്ടത്. എന്നിട്ടും പരിഹാരമില്ലാത്ത സന്ദർഭത്തിലാണ് അടി… ആയത്തിൽ പറയുന്ന അടി മര്യാദയുടെ അടിയാണ്. വേദനാജനകമായ അടിയല്ല. ആ അടി മൂലം അസ്ഥി ഒടിയുകയോ അവയവത്തിന് മുറിവോ ചതവോ വരുകയോ തുടങ്ങിയവയൊന്നും സംഭവിക്കരുത്. ആ അടി കൊണ്ടുള്ള ഉദ്ദേശം ദാമ്പത്ത്യത്തിന്റെ നന്മ മാത്രമാകണം.” (ജാമിഉൽ അഹ്‌കാം: 5:172)

ഇമാം നവവി (റ) പറഞ്ഞു: ” ‘വേദനിപ്പിക്കാത്ത അടി’ (ضربا غير مبرح) എന്നു പ്രവാചകൻ (സ) പറഞ്ഞതിലെ ഉദ്ദേശം ശക്തമോ ശാരീരിക പ്രയാസമോ ഉണ്ടാക്കാത്ത അടിയാണ്. ‘അൽ ബർഹ്’ (البَرْح) എന്നാൽ ‘ശാരീരിക പ്രയാസം’ (المشقة) എന്നാണ് അർത്ഥം. അൽ മുബർരിഹ് (المبرِّح) എന്നാൽ ശാരീരിക പ്രയാസം സൃഷ്ടിക്കുന്നത്. അത്തരം അടിയാകരുത് അത് എന്നാണ് പ്രവാചകൻ (സ) ഹദീസിൽ പ്രസ്ഥാവിച്ചത്.” (ശർഹു മുസ്‌ലിം: 8:184)

ഇനി, ഇത്തരം സന്ദർഭങ്ങളിൽ, മൃദുലമായ അടി അനുവദിച്ചപ്പോഴും “ഭാര്യയുടെ മുഖത്ത് അടിക്കുകയോ അപമാനിക്കുകയോ ചെയ്യരുത്” എന്നും “ജനങ്ങൾക്കിടയിൽ പിണക്കം പ്രകടിപ്പിക്കുകയോ അവമതിക്കുകയോ ചെയ്യരുത്” എന്നും പ്രവാചകൻ (സ) പ്രത്യേകം ഉപദേശിച്ചതു കൂട (സ്വഹീഹു അബൂദാവൂദ്: 2143 ) ചേർത്തു വായിക്കുക.

” …അയാൾ അയാളുടെ പ്രിയതമയായ ക്ലാര പെറ്റാച്ചിയേയും കൊണ്ട്, സ്വിസ് അതിർത്തി കടന്നു ശത്രുക്കളിൽ നിന്ന് രക്ഷപ്പെടാനായി ഒളിച്ചോടി. സ്വിസ് അതിർത്തി കടന്നാൽ ഒളിവിൽ ജീവിക്കാം എന്ന് ആ ഇണകൾ സ്വപ്നം കണ്ടു. ശത്രുക്കൾ തിരിച്ചറിയാതിരിക്കാൻ അയാൾ വേഷപ്രച്ഛന്നനായി. പക്ഷെ പടയാളികൾ അയാളെ തിരിച്ചറിഞ്ഞപ്പോൾ ഒരു കളപ്പുരയിൽ അവർ രണ്ടു പേരും അഭയം തേടി. പക്ഷെ പടയാളികൾ അവരെ വെറുതെ വിട്ടില്ല. അവർ അവരെ കളപ്പുരയിൽ നിന്ന് നിർബന്ധിച്ച് പുറത്തു കൊണ്ട് വന്നു. വില്ല ബെൽ‌മോണ്ടിന്റെ പ്രവേശന കവാടത്തിനടുത്തുള്ള ഒരു ഇഷ്ടിക മതിലിനു നേരെ നിൽക്കാൻ ഇണകളോട് ആവശ്യപ്പെട്ടു. അവിടെ വെച്ച് പടയാളികൾ അവർക്കു നേരെ നിറയൊഴിച്ചു. വെടിയുതിർക്കുമ്പോൾ അയാൾ കേണപേക്ഷിച്ചു “അരുതേ! അരുതേ!” അതായിരുന്നു അയാളുടെ അവസാന വാക്കുകൾ. ശേഷം അയാളുടെ കൂട്ടാളികളിൽ നിന്ന് പിടിക്കപ്പെട്ട പതിനഞ്ചു പേരെയും അവർ വലിച്ചിഴച്ച് കൊണ്ട് വന്ന് അതേ സ്ഥലത്ത് വെച്ച് വെടി വെച്ച് കൊന്നു…” രണ്ടാം ലോക മഹായുദ്ധത്തിലെ ഒരു രംഗമാണിത്. ഈ രംഗം മാത്രം വായിക്കുന്നവർക്ക് കൊല്ലപ്പെട്ട വ്യക്തിയോടും അയാളുടെ ഇണയോടും അയാളുടെ കൂട്ടാളികളോടും സഹതാപം തോന്നിയേക്കാം. ഈ ദാരുണമായ കൂട്ടക്കൊലയെ സംബന്ധിച്ച വാർത്തയുടെ തലക്കെട്ടായി ‘ന്യൂയോർക്ക് ടൈംസ്’ (the New York Times) ദിനപത്രത്തിൽ വന്നത് “നികൃഷ്‌ടമായ ഒരു ജീവിതത്തിന് ലഭിച്ച ഒരു ഉചിതമായ അന്ത്യം” (A fitting end to a wretched life) എന്നായിരുന്നു എന്നു കൂടി പറഞ്ഞാൽ ആ പത്രത്തോട് തന്നെ നമുക്ക് ഭീതിയും വെറുപ്പും ജനിച്ചേക്കും.

പക്ഷെ ഈ കൊല്ലപ്പെട്ടവർ ഫാഷിസത്തിന്റെ പിതാവും ഏകാധിപതിയുമായ മുസോളനിയും കിങ്കരന്മാരായ ഫാഷിസ്റ്റുകളുമാണ് എന്ന് തിരിച്ചറിയുമ്പോൾ നമ്മളുടെ വൈകാരികത എതിർ ദിശയിലേക്കാണ് പ്രവഹിക്കുക. രണ്ടാം ലോകമഹായുദ്ധത്തിൽ 400,000 ത്തിലധികം പേരുടേയും ഇറ്റാലിയൻ എത്യോപ്യ ആക്രമണത്തിൽ 30,000 പേരുടേയും കൊലപാതകത്തിന് അധ്യക്ഷത വഹിച്ച മുസ്സോളിനിയുടെ ദാരുണാന്ത്യമാണ് മുകളിൽ നാം വായിച്ചത്. !

ചില യുദ്ധങ്ങളെ പറ്റി ക്വുർആനിലും ഹദീസുകളിലും വന്നിട്ടുള്ള ചില വിശേഷണങ്ങളും, യുദ്ധരംഗങ്ങളും, സാഹചര്യത്തിൽ നിന്നും ചരിത്ര പശ്ചാത്തലത്തിൽ നിന്നും അടർത്തിയെടുത്ത് അവതരിപ്പിക്കുന്ന ഇസ്‌ലാം വിമർശകരുടെ സ്ഥിരം വേല ഇതിനോട് സമാനമാണ്. പ്രസ്തുത യുദ്ധ പ്രസ്ഥാവനകളുടേയും രംഗങ്ങളുടേയും പിന്നാമ്പുറവും കാരണങ്ങളും ‘ബാക്ക് സ്റ്റോറിയു’മൊക്കെ അറിയാത്തവരിൽ അതുപയോഗിച്ച് ഇസ്‌ലാം ഭീതിയും വെറുപ്പും സൃഷ്ടിക്കുകയാണ് ഈ കുബുദ്ധികളുടെ ലക്ഷ്യം. ഒരു ദീർഘകാല യുദ്ധങ്ങളുടെ വിവരണങ്ങളിലെ ഒരു അടര് മാത്രം ചുവപ്പ് പൂശി പ്രദർശിപ്പിച്ച് നായകന്മാരെ വില്ലന്മാരായി ചിത്രീകരിക്കുകയും വില്ലന്മാരെ നായകന്മാരായി ദുവ്യാഖ്യാനിക്കുകയും ചെയ്യുന്ന സ്ഥിരം കുതന്ത്രം !!

അതിനായി സാധാരണ ഗതിയിൽ ദുർവ്യാഖ്യാതാക്കൾ ഉപയോഗിക്കാറുള്ള ഹദീസുകളിലെ ഒരു യുദ്ധരംഗം ഇപ്രകാരമാണ്:

لَا تَقُومُ السَّاعَةُ حَتَّى يُقَاتِلَ الْمُسْلِمُونَ الْيَهُودَ ، فَيَقْتُلُهُمُ الْمُسْلِمُونَ حَتَّى يَخْتَبِئَ الْيَهُودِيُّ مِنْ وَرَاءِ الْحَجَرِ وَالشَّجَرِ، فَيَقُولُ الْحَجَرُ أَوِ الشَّجَرُ: يَا مُسْلِمُ يَا عَبْدَ اللهِ هَذَا يَهُودِيٌّ خَلْفِي ، فَتَعَالَ فَاقْتُلْهُ ، إِلَّا الْغَرْقَدَ، فَإِنَّهُ مِنْ شَجَرِ الْيَهُودِ.

“അന്ത്യദിനം സംഭവിക്കുകയില്ല; മുസ്‌ലിംകൾ ജൂതന്മാരോട് യുദ്ധം ചെയ്യുന്നത് വരെ. അങ്ങനെ ആ യുദ്ധത്തിൽ മരത്തിന്റേയും കല്ലിന്റേയും പിറകിൽ ജൂതൻ ഒളിച്ചിരിക്കുവോളം മുസ്‌ലിംകൾ ജൂതന്മാരെ വധിക്കും. അപ്പോൾ ആ മരം അല്ലെങ്കിൽ കല്ല് വിളിച്ചു പറയും: അല്ലയോ മുസ്‌ലിം, അല്ലാഹുവിന്റെ ദാസാ, ഇതാ എന്റെ പിറകിൽ ഒരു ജൂതൻ. അപ്പോൾ ആ മുസ്‌ലിം വന്ന് അയാളെ വധിക്കും. ഗിർകദ് വൃക്ഷമൊഴികെ; അത് ജൂതരുടെ വൃക്ഷമാണ്.” (സ്വഹീഹു മുസ്‌ലിം: 2922)

ഈ രംഗം മാത്രമാണ് വിമർശകർ ഉദ്ധരിക്കുക. ഈ യുദ്ധം ഏതാണെന്നോ യുദ്ധത്തിന്റെ പശ്ചാത്തലം എന്താണെന്നോ ആരാണ് വൃക്ഷത്തിനു പിന്നിൽ ഒളിച്ചിരിക്കുന്ന ജൂതനെ കൊല്ലുന്ന ആ ‘മുസ്‌ലിം’ എന്നോ, മരം ആ മുസ്‌ലിമിനോട് എങ്ങനെയാണ് സംസാരിക്കുക എന്നോ വിമർശകർ വിശദീകരിക്കില്ല.!! കാരണം അതൊക്കെ വിവരിച്ചാൽ വിമർശനം താനെ അസാധുവാകും. അതുകൊണ്ട് തന്നെ ആ യുദ്ധത്തിലെ ഒരു സീൻ മാത്രം അവതരിപ്പിച്ച് ഭീതി പടർത്തുകയാണ് ദുർവ്യാഖ്യാതാക്കളുടെ ലക്ഷ്യം.

അന്ത്യദിനത്തോട് അടുത്തായി നടക്കുന്ന ഒരു യുദ്ധത്തിലെ (ഒരു സ്വാതന്ത്ര്യ സമരത്തിലെ) രംഗമെടുത്ത് ജൂതന്മാരോട് യുദ്ധം ചെയ്യലും അവരെ വധിക്കലും ഇസ്‌ലാമിലെ ഒരു നിയമമോ പാഠമോ ആയി ദുർവ്യാഖ്യാനിക്കുകയാണ് വിമർശകർ. എന്നാൽ സമാധാന ചിത്തരായി ജീവിക്കുന്ന സാധാരണ അമുസ്‌ലിംകളോട് എന്ത് നിലപാടാണ് സ്വീകരിക്കേണ്ടത് എന്നതുമായി ബന്ധപ്പെട്ട ഇസ്‌ലാമിക നിയമവും പ്രവാചക പാഠവും മറ്റു ഹദീസുകളിൽ വ്യക്തമായി വായിക്കാൻ സാധിക്കും:

അല്ലാഹു പറഞ്ഞു: “മതകാര്യത്തില്‍ നിങ്ങളോട് യുദ്ധം ചെയ്യാതിരിക്കുകയും, നിങ്ങളുടെ വീടുകളില്‍ നിന്ന് നിങ്ങളെ പുറത്താക്കാതിരിക്കുകയും ചെയ്യുന്നവരെ സംബന്ധിച്ചിടത്തോളം നിങ്ങളവര്‍ക്ക് നന്‍മ ചെയ്യുന്നതും നിങ്ങളവരോട് നീതി കാണിക്കുന്നതും അല്ലാഹു നിങ്ങളോട് നിരോധിക്കുന്നില്ല. തീര്‍ച്ചയായും അല്ലാഹു നീതി പാലിക്കുന്നവരെ ഇഷ്ടപ്പെടുന്നു. മതകാര്യത്തില്‍ നിങ്ങളോട് യുദ്ധം ചെയ്യുകയും നിങ്ങളുടെ വീടുകളില്‍ നിന്ന് നിങ്ങളെ പുറത്താക്കുകയും നിങ്ങളെ പുറത്താക്കുന്നതില്‍ പരസ്പരം സഹകരിക്കുകയും ചെയ്തവരെ സംബന്ധിച്ചു മാത്രമാണ് -അവരോട് മൈത്രികാണിക്കുന്നത് – അല്ലാഹു നിരോധിക്കുന്നത്‌. വല്ലവരും അവരോട് മൈത്രീ ബന്ധം പുലര്‍ത്തുന്ന പക്ഷം അവര്‍ തന്നെയാകുന്നു അക്രമകാരികള്‍.” (ക്വുർആൻ 60: 8, 9)

പ്രവാചകൻ മുഹമ്മദ് (സ) പറഞ്ഞു: مَنْ قَتَلَ مُعَاهَدًا لَمْ يَرَحْ رَائِحَةَ الْجَنَّةِ، وَإِنَّ رِيحَهَا لَيُوجَد مِنْ مَسِيرَةِ أَرْبَعِينَ عَامًا

“സമാധാന സന്ധിയിലുള്ള ഒരു അമുസ്‌ലിമിനെ ആരെങ്കിലും കൊന്നാൽ അവന് സ്വർഗത്തിന്റെ സുഗന്ധം പോലും ലഭിക്കില്ല.” (സ്വഹീഹുൽ ബുഖാരി: ഹദീസ് നമ്പർ: 3166)

عَنْ صَفْوَانَ بْنَ سُلَيْمٍ عَنْ رَسُولِ اللَّهِ صلى الله عليه وسلم قَالَ ‏ “‏ أَلاَ مَنْ ظَلَمَ مُعَاهِدًا أَوِ انْتَقَصَهُ أَوْ كَلَّفَهُ فَوْقَ طَاقَتِهِ أَوْ أَخَذَ مِنْهُ شَيْئًا بِغَيْرِ طِيبِ نَفْسٍ فَأَنَا حَجِيجُهُ يَوْمَ الْقِيَامَةِ

സ്വഫ്‌വാനു ബ്‌നു സുലൈമില്‍(റ) നിന്ന് നിവേദനം: പ്രവാചകൻ (സ) പറഞ്ഞു: ‘അറിയണം, ആരെങ്കിലും സമാധാന സന്ധിയിലുള്ള അമുസ്‌ലിമിനെ ഉപദ്രവിക്കുകയോ, അവന് കിട്ടേണ്ട അവകാശങ്ങളില്‍ കുറവ് വരുത്തുകയോ, സാധ്യമാകുന്നതിലുപരി വഹിക്കാന്‍ അവനെ നിര്‍ബന്ധിക്കുകയോ, മനപ്പൊരുത്തമില്ലാതെ അവനില്‍ നിന്നും വല്ലതും കവര്‍ന്നെടുക്കുകയോ ചെയ്താൽ ഉയിര്‍ത്തെഴുന്നേല്‍പ്പ് നാളില്‍ ഞാന്‍ അവനുമായി (ആ അമുസ്‌ലിമിന്റെ അവകാശങ്ങളുടെ കാര്യത്തിൽ) തര്‍ക്കത്തിലേര്‍പെടും. (അബൂദാവൂദ്: 3052)

ഇതാണ് സമാധാന ചിത്തരായി ജീവിക്കുന്ന സാധാരണ അമുസ്‌ലിംകളോട് സ്വീകരിക്കേണ്ട നിലപാടായി ഇസ്‌ലാം പഠിപ്പിച്ച നിയമവും ശാസനയും. എത്ര തവണ വ്യക്തമാക്കിയാലും ഈ ആയത്തുകളും ഹദീസുകളും വിമർശകർ കാണാത്ത മട്ടാണ് !!

ഹദീസിൽ പ്രസ്ഥാവിക്കപ്പെട്ടിട്ടുള്ള യുദ്ധരംഗമാകട്ടെ ഒരു നിയമമോ അനുശാസയോ അല്ല. മറിച്ച് അന്ത്യദിനത്തിൽ നടക്കാനിരിക്കുന്ന ഒരു മഹായുദ്ധത്തിൽ നിന്ന് അല്ലെങ്കിൽ ഒരു സ്വതന്ത്ര്യ സമരത്തിൽ നിന്നുള്ള ഒരു രംഗത്തിന്റെ വിവരണമാണ്.

എപ്പോഴാണ് ഈ യുദ്ധം നടക്കുക ?

فالمراد بقتال اليهود : وقوع ذلك إذا خرج الدجال ونزل عيسى

“ജൂതന്മാരോടുള്ള യുദ്ധം നടക്കുക (അന്ത്യ ദിനത്തോടടുത്ത്) ദജ്ജാൽ പുറപ്പെടുകയും ഈസാ നബി (യേശു) ഇറങ്ങുകയും ചെയ്യുന്ന സന്ദർഭത്തിലാണ്.” (ഫത്ഹുൽ ബാരി: 6: 610)

المراد بقوله (تقاتلون اليهود) إذا نزل عيسى ، فإن المسلمين معه ، واليهود مع الدجال.

ഇബ്നുൽ മുലക്കിൻ പറഞ്ഞു: “മുസ്‌ലിംകൾ ജൂതന്മാരോട് യുദ്ധം ചെയ്യും എന്ന് ഹദീസിൽ ഉദ്ദേശിച്ചിരിക്കുന്നത് പ്രവാചകൻ ഈസാ (ഭൂമിയിൽ) ഇറങ്ങുന്ന സന്ദർഭത്തിലാണ്. മുസ്‌ലിംകൾ പ്രവാചകൻ ഈസായോടൊപ്പവും ജൂതന്മാർ ദജ്ജാലിനോടൊപ്പവുമായിരിക്കും.” (അത്തൗദീഹു ലി ശർഹിൽ ജാമിഅ്: 17:663)

ഇക്കാര്യം ഒട്ടനവധി പണ്ഡിതന്മാർ ഹദീസിന്റെ വ്യാഖ്യാനത്തിൽ വിശദീകരിച്ചിട്ടുണ്ട് എന്ന് മാത്രമല്ല ഇമാം തുർമുദി, ചർച്ചാ വിഷയകമായ ഹദീസ് (സുനനു തുർമുദി: 2236) ഉദ്ധരിച്ചിരിക്കുന്നതു തന്നെ “ദജ്ജാലിന്റെ അടയാളങ്ങളായി ഹദീസിൽ വന്ന കാര്യങ്ങൾ” (باب ما جاء في علامة الدجال) എന്ന അധ്യായത്തിലാണ്.

ഇക്കാര്യം ഹദീസിലും അർത്ഥ ശങ്കക്കിടയില്ലാത്ത വിധം സൂചിപ്പിക്കപ്പെട്ടിട്ടുണ്ട്:

വിവാദ വിഷയകമായ ഹദീസ് ആരംഭിക്കുന്നതു തന്നെ “അന്ത്യദിനം സംഭവിക്കുകയില്ല; മുസ്‌ലിമുകൾ ജൂതന്മാരോട് യുദ്ധം ചെയ്യുന്നത് വരെ….” എന്ന് പറഞ്ഞു കൊണ്ടാണ്. (സ്വഹീഹു മുസ്‌ലിം: 2922)

മറ്റൊരു ഹദീസ് കാണുക:

قَالَ فَحِينَ يَرَى الْكَذَّابُ يَنْمَاثُ كَمَا يَنْمَاثُ الْمِلْحُ فِي الْمَاءِ فَيَمْشِي إِلَيْهِ فَيَقْتُلُهُ حَتَّى إِنَّ الشَّجَرَةَ وَالْحَجَرَ يُنَادِي يَا رُوحَ اللَّهِ هَذَا يَهُودِيٌّ فَلا يَتْرُكُ مِمَّنْ كَانَ يَتْبَعُهُ أَحَدًا إِلا قَتَلَهُ

“പ്രവാചകൻ ഈസായെ(അ) ആ പെരും നുണയൻ (ദജ്ജാൽ) കാണുമ്പോൾ ഉപ്പ്, വെള്ളത്തിൽ അലിയുന്നതു പോലെ അവൻ അലിയും. അപ്പോൾ അദ്ദേഹം (ഈസാ നബി (അ) അവന്റെ അടുത്തേക്ക് നടന്നു ചെന്ന് അവനെ വധിക്കും. എത്രത്തോളമെന്നാൽ പാറയും മരവും വിളിച്ചു പറയും: (അല്ലയോ മുസ്‌ലിം,) അല്ലാഹുവിന്റെ അടുക്കലുള്ള ആത്മാവിനാൽ സൃഷ്ടിക്കപ്പെട്ടവനേ, എന്റെ (പിറകിൽ) അതാ ഒരു ജൂതൻ. അങ്ങനെ ദജ്ജാലിനെ പിൻപറ്റിയവരിൽ പെട്ട ഒരാളെയും വധിക്കാതെ അദ്ദേഹം വിടില്ല.” (മുസ്നദു അഹ്മദ്: 14426)

ഹദീസ് പ്രവചിക്കുന്ന ‘സ്വാതന്ത്ര്യ സമര’ത്തെ സംബന്ധിച്ച പല കാര്യങ്ങളും ഈ ഹദീസിൽ നിന്നും മനസ്സിലാക്കാം:

1. ജൂതന്മാരോടുള്ള ഈ യുദ്ധം ഒരു മതനിയമമായിട്ടല്ല ഹദീസിൽ വന്നിട്ടുള്ളത്. അന്ത്യദിനത്തോടടുത്ത് ഭാവിയിൽ സംഭവിക്കാനിരിക്കുന്ന ഒരു യുദ്ധത്തെ സംബന്ധിച്ച് വിവരണം മാത്രമാണിത്. ദജ്ജാലെന്ന ഏകാധിപതിയുടെ നേതൃത്വത്തിലുള്ള ശത്രു പക്ഷത്തോട് ഈസാ നബിയുടെ(അ) നേതൃത്വത്തിൽ മുസ്‌ലിംകൾ സ്വാതന്ത്ര്യത്തിനായി നടത്തുന്ന ഒരു പോരാട്ടമാണ് ഹദീസുകളുടെ ഉള്ളടക്കം.

2. “അങ്ങനെ ദജ്ജാലിനെ പിൻപറ്റിയവരിൽ പെട്ട ഒരാളെയും വധിക്കാതെ അദ്ദേഹം വിടില്ല.” എന്ന വാചകത്തിൽ നിന്നും ദജ്ജാലെന്ന ഏകാധിപതിയുടെ ആരാധകരും ഭടൻമാരുമായ ജൂതന്മാരെയാണ് മുസ്‌ലിംകൾ വധിക്കുക എന്ന് മനസ്സിലാക്കാം. സമാധാന ചിത്തരായ സാധാരണ അമുസ്‌ലിംകളെയല്ല.

3. കല്ലും മരവും “അല്ലയോ മുസ്‌ലിമേ, എന്റെ പിറകിൽ ഒരു ജൂതനുണ്ട്” എന്ന് വിളിച്ചു പറയുന്ന ആ ‘വിളിക്കപ്പെടുന്ന മുസ്‌ലിം’ പ്രവാചകനായ ഈസായാണ് (അ) എന്നാണ് ഹദീസ് സൂചിപ്പിക്കുന്നത്. ഈ അത്ഭുത പ്രതിഭാസം ഈസായുടെ (അ) ‘മുഅ്ജിസത്തെ’ന്ന (അല്ലാഹുവിൽ നിന്നുള്ള അമാനുഷിക ദൃഷ്ടാന്തം) നിലയിൽ നടക്കുന്നതാകുവാനാണ് സാധ്യത.

ആരാണ് യുദ്ധം ആരംഭിക്കുന്നത്?

മുസ്‌ലിംകൾ ജൂതരുമായി യുദ്ധം നടത്തുമെന്ന് പല ഹദീസുകളിലും പൊതുവായി പ്രസ്ഥാവിക്കപ്പെട്ടതിൽ നിന്നും മുസ്‌ലിംകൾ അങ്ങോട്ടാണ് യുദ്ധം ആരംഭിക്കുക എന്ന ഒരു തെറ്റുദ്ധാരണയുണ്ടാകാം. എന്നാൽ യുദ്ധത്തിന് ആരംഭം കുറിക്കുന്നത് ദജ്ജാലിന്റെ ആർമിയാണെന്ന് വ്യക്തമായി സൂചിപ്പിക്കുന്ന ചില ഹദീസുകളും സ്വഹീഹുൽ ബുഖാരിയിൽ തന്നെ നമുക്ക് കാണാവുന്നതാണ്:

تُقَاتِلُكُمُ اليَهُودُ فَتُسَلَّطُونَ عَلَيْهِمْ ، ثُمَّ يَقُولُ الحَجَرُ : يَا مُسْلِمُ هَذَا يَهُودِيٌّ وَرَائِي، فَاقْتُلْهُ.

“ജുതന്മാർ നിങ്ങളോട് യുദ്ധം ചെയ്യും അപ്പോൾ അവർക്കുമേൽ അല്ലാഹു നിങ്ങൾക്ക് വിജയം നൽകും. ശേഷം പാറക്കല്ല് പറയും: മുസ്‌ലിമേ, ഇതാ എന്റെ പിറകിൽ ഒരു ജൂതൻ. അവനെ വധിക്കൂ.” (സ്വഹീഹുൽ ബുഖാരി: 3593, സ്വഹീഹു മുസ്‌ലിം: 2921)

ആരാണ് മസീഹുദ്ദജ്ജാൽ ?

അന്ത്യനാളിനോടടുത്ത് പ്രത്യക്ഷനാകുമെന്ന് പല മത ഗ്രന്ഥങ്ങളും പ്രവചിച്ച, അന്തിക്രിസ്‌തു (antichrist) എന്ന പേരിൽ അറിയപ്പെടുന്ന ഒരു ഭാവി വ്യക്തിത്വമാണ് ‘മസീഹു ദ്ദജ്ജാൽ’. ലോകത്ത് ജീവിച്ചിരുന്നതും ജീവിച്ചിരിക്കുന്നവരുമായ ഏകാധിപതികളേക്കാൾ ഏറ്റവും കിരാതനും ക്രൂരനുമായ ഏകാധിപതിയായിരിക്കും (dictator) മസീഹു ദ്ദജ്ജാൽ എന്നാണ് ഹദീസുകൾ പഠിപ്പിക്കുന്നത്. മനുഷ്യരെ ഭൗതീകവും ആത്മീയവുമായ പാരതന്ത്ര്യത്തിലേക്ക് വലിച്ചിഴക്കുന്ന ഈ ദജ്ജാലിന്റെ ഉപദ്രവങ്ങളിൽ നിന്ന് രക്ഷ തേടാൻ പ്രവാചകൻ (സ) തന്റെ അനുചരന്മാരോട് പ്രത്യേകം ഉപദേശിക്കുക കൂടി ചെയ്തതായി കാണാം. (സ്വഹീഹു മുസ്‌ലിം: 924)

ദജ്ജാലിനാൽ ഭാവിയിൽ, ലോകത്ത് വിതക്കപ്പെടാനിരിക്കുന്ന കുഴപ്പങ്ങളും ഛിദ്രതകളും സമാനതകൾ ഇല്ലാത്തതായിരിക്കും.

مَا بَيْنَ خَلْقِ آدَمَ إِلَى أَنْ تَقُومَ السَّاعَةُ فِتْنَةٌ أَكْبَرُ مِنْ فِتْنَةِ الدَّجَّالِ .

പ്രവാചകൻ (സ) പറഞ്ഞു: “(ആദ്യ മനുഷ്യൻ) ആദമിനെ സൃഷ്ടിച്ചതു മുതൽ അന്ത്യദിനം സംഭവിക്കുന്നു വരെ ദജ്ജാലിന്റെ കുഴപ്പത്തേക്കാൾ വലിയ ഒരു ആപത്തും ഇല്ലതന്നെ.” (മുസ്നദു അഹ്മദ്: 15831)

ലോക രാജ്യങ്ങൾ മുഴുവൻ വെട്ടിപ്പിടിച്ച് തന്റെ അധികാരത്തിനും ആജ്ഞാപനത്തിനും കീഴിലാക്കുന്ന ലോകം കണ്ട ഏറ്റവും വലിയ ഏകാധിപതിയായിരിക്കും ‘മസീഹു ദ്ദജ്ജാൽ’.

ليس من بلد إلا سيطؤه الدجال إلا مكة والمدينة ليس له من نقابها نقب إلا عليه الملائكة صافين يحرسونها

ദജ്ജാൽ കാലു കുത്താത്ത ഒരു നാടും അവശേഷിക്കില്ല; മക്കയും മദീനയും ഒഴികെ. ആ രണ്ട് നാടുകളെയും സംരക്ഷിച്ചു കൊണ്ട് മലക്കുകൾ വലയം ചെയ്യുന്നുണ്ടാകും. (സ്വഹീഹുൽ ബുഖാരി: 1881, സ്വഹീഹു മുസ്‌ലിം: 2943)

നാടുകളിൽ കാലുകുത്തുക എന്നതുകൊണ്ടുദ്ദേശം പടയോട്ടത്തിലൂടെ അധികാരത്തിന് കീഴിലാക്കുക എന്നാണ്.

فلا يبقى له موضع إلا ويأخذه غير مكة والمدينة

“മക്കയും മദീനയും തുടങ്ങിയ സ്ഥലങ്ങളല്ലാതെ ഒരു സ്ഥലവും അവൻ പിടിച്ചടക്കാത്തതായി അവശേഷിക്കില്ല” എന്ന് ചില നിവേദനങ്ങളിൽ കാണാം. (ഉംദത്തുൽ കാരി: 10:244)

“എല്ലാ ജല തടത്തിലും അവന്റെ അധികാരമെത്തും” (يبلغ سلطانه كل منهل) എന്ന് മറ്റു ചില നിവേദനങ്ങളിലും വന്നിരിക്കുന്നു. (മുസ്നദു അഹ്മദ്: 23139)

ദജ്ജാലിന്റെ സാമ്രാജ്യത്വ അധിനിവേശത്തിനു കീഴിൽ മനുഷ്യ ജീവിതങ്ങൾ ഞെരിഞ്ഞമരും. അങ്ങനെ ലോകം മുഴുവൻ അക്രമവും അനീതിയും അടിച്ചമർത്തലുകളും നിറയുന്ന ഘട്ടത്തിൽ മഹ്ദി എന്ന സ്ഥാനപേരിലുള്ള ഒരു ഭരണാധികാരിയുടേയും പ്രവാചകനായ ഈസാ(അ)യുടേയും നേതൃത്വത്തിൽ അന്ത്യദിനത്തോടടുത്ത് നടക്കുന്ന സ്വാതന്ത്ര്യ സമരമാണ് ഹദീസുകളുടെ ഇതിവൃത്തം.

أبشركم بالمهدي يبعث على اختلاف من الناس وزلازل فيملأ الأرض قسطاً وعدلاً كما ملئت جوراً وظلماً

“ലോകം മുഴുവൻ അക്രമങ്ങളും സ്വേച്ഛാധിപത്യവും കൊണ്ട് നിറഞ്ഞ സന്ദർഭത്തിൽ ലോകത്തെ നീതിയും ന്യായവും കൊണ്ട് മഹ്ദി നിറക്കുമെന്ന്” ഹദീസിൽ പ്രസ്ഥാവിക്കുന്നുണ്ട്. (മുസ്നദു അഹ്മദ്: 11344, അൽ അഹ്കാമുശറഇയ്യ അൽ കുബ്റാ: അബ്ദുൽ ഹക്ക് അൽ ഇശ്ബീലി: 4/532, മജ്മഉസ്സവാഇദ്: 7/316)

അത്ഭുത സിദ്ധികൾ പലതും പ്രദർശിപ്പിച്ച് ജനങ്ങളെ വശീകരിക്കുന്നതിന് പുറമെ, ദജ്ജാലെന്ന ഏകശസനാധികാരി ആളുകളെ ചതിയിലൂടെ അവരെ അഗ്നിയിലേക്ക് നയിക്കുമെന്ന സൂചനകൾ ഹദീസുകളിൽ കാണാം.

‘അയാളുടെ കൂടെ രണ്ട് ജല തടങ്ങളുണ്ടാകും. ഒന്ന് പ്രത്യക്ഷ ദൃഷ്ട്യാ വെള്ളമായിരിക്കും. മറ്റൊന്ന് ദൃഷ്ട്യാൽ അഗ്നിയായിരിക്കും. എന്നാൽ വെള്ളമെന്ന് തോന്നിപ്പിക്കപ്പെടുന്ന തടങ്ങൾ യഥാർത്ഥത്തിൽ അഗ്നിയായിരിക്കും എന്നും’ ഹദീസുകൾ സൂചിപ്പിക്കുന്നുണ്ട്. (സ്വഹീഹു മുസ്‌ലിം: 5223)

അത്ഭുത പ്രവർത്തനങ്ങൾ പ്രദർശിപ്പിച്ച് ദിവ്യത്വം വാദിക്കുക കൂടി ചെയ്യും. അധികാരത്തിലൂടെ തന്റെ ദിവ്യത്വം ജനങ്ങളിൽ അടിച്ചേൽപ്പിക്കുക എന്നത് ദജ്ജാലിന്റെ പദ്ധതികളിൽ ഒന്നായിരിക്കും.

“ദജ്ജാൽ ജനങ്ങളോട്‌ പറയും: ഈ വ്യക്തിയെ (ഒരു മുസ്‌ലിം) ഞാൻ കൊല്ലുകയും പിന്നീട് ജീവിപ്പിക്കുകയും ചെയ്താൽ (ഞാൻ നിങ്ങളുടെ ദൈവമാണെന്നതിൽ) നിങ്ങൾ സംശയിക്കുമോ? ജനങ്ങൾ പറയും: ഇല്ല. അപ്പോൾ ദജ്ജാൽ അയാളെ കൊല്ലുകയും ജീവിപ്പിക്കുകയും ചെയ്യും. അപ്പോൾ ജീവൻ തിരിച്ചു കിട്ടിയ വ്യക്തി പറയും: “അല്ലാഹുവാണേ, മുമ്പൊരിക്കലും നിന്നെക്കുറിച്ച്‌ എനിക്കില്ലാതിരുന്ന വ്യക്തമായ തിരിച്ചറിവ് ഇന്നെനിക്ക് ലഭിച്ചു.” (അഥവാ നബി പറഞ്ഞ ദജ്ജാൽ നീ തന്നെയാണു എന്ന് എനിക്കിപ്പോൾ ഉറപ്പായി എന്നയാൾ ആണയിടും. കാരണം ഇപ്രകാരം ഒരു സംഭവമുണ്ടാകുമെന്ന പ്രവാചകന്റെ ഹദീസിന് ജീവിക്കുന്ന തെളിവായി അയാൾ മാറുമല്ലൊ) അപ്പോൾ ദജ്ജാൽ അയാളെ വീണ്ടും കൊല്ലാൻ ശ്രമിക്കും. പക്ഷെ അതിന് ദജ്ജാലിനെ അല്ലാഹു അനുവദിക്കില്ല.” (സ്വഹീഹുൽ ബുഖാരി: 6599)

തന്റെ ദിവ്യത്വത്തിൽ ജനങ്ങളെ നിർബന്ധിച്ച് വിശ്വസിപ്പിക്കാനായി അധികാരവും മായാജാലവുമൊക്കെ ദജ്ജാൽ ഉപയോഗിക്കും. എന്നിട്ടും വിശ്വസിക്കാത്തവർക്ക് വധശിക്ഷയാണ് ദജ്ജാലിന്റെ ഭരണകൂടം വിധിക്കുക. വിശ്വാസ സ്വാതന്ത്ര്യം നിശ്ശേഷം നിഷേധിക്കപ്പെടുന്ന ഈ സാമ്രാജ്യത്വ സ്വേച്ഛാധിപത്യത്തോടാണ് മുസ്‌ലിംകൾ പടക്കിറങ്ങുന്നത് എന്നർത്ഥം.

സ്വന്തം മതത്തിന്റെ ശാസനകളും പാഠങ്ങളും വലിച്ചെറിഞ്ഞ ഒരു കൂട്ടം ജൂതന്മാർ ഈ ഏകാധിപതിയിൽ ആകൃഷ്ടരാവുകയും അയാളുടെ ആജ്ഞാധികാരവും ദിവ്യത്വവും സ്വീകരിക്കുകയും ചെയ്യും. “എഴുപതിനായിരത്തോളം ജൂതന്മാർ ദജ്ജാലിന്റെ അനുയായികളായി മാറും” എന്ന് ഹദീസുകളിൽ കാണാം. (മുസ്നദു അഹ്മദ്: 12865)

ഈ സ്വേച്ഛാധിപതിയുടെ കിങ്കരന്മാരും പടയാളികളുമായ ജൂതന്മാരോടാണ് മുസ്‌ലിംകൾ അന്ത്യ നാളിനോടടുത്ത് യുദ്ധം ചെയ്യുന്നത്. അല്ലാതെ സമാധാനത്തോടെ ജീവിക്കുന്ന അമുസ്‌ലീംകളോടല്ല.

عن أنس بن مالك رضي الله عنه : أن رسول الله صلى الله عليه وسلم قال : ” يَتْبَعُ الدَّجَّالَ مِنْ يَهُودِ أَصْبَهَانَ سَبْعُونَ أَلْفًا عَلَيْهِمْ الطَّيَالِسَةُ ” والطيالسة : كساء غليظ مخطط . وفي رواية للإمام أحمد : ” سَبْعُونَ أَلْفًا مِنْ الْيَهُودِ عَلَيْهِمْ التِّيجَانُ”

എഴുപതിനായിരത്തോളം വരുന്ന ദജ്ജാലിന്റെ അനുയായികളായ ജൂതന്മാർ ‘ത്വയാലിസ’ (الطَّيَالِسَةُ) എന്ന ഒരു തരം പരുപരുത്ത ദൃഢമായ വസ്ത്രം ധരിക്കുന്നവരായിരിക്കും എന്ന് ഹദീസുകളിൽ കാണാം. (സ്വഹീഹു മുസ്‌ലിം: 5237)

“അവർ മകുടം അണിഞ്ഞവരായിരിക്കും” എന്ന് മറ്റു ചില ഹദീസുകളിലും പ്രസ്ഥാവിച്ചിട്ടുണ്ട്. (മുസ്നദു അഹ്മദ്: 12865)

ഏകരൂപമായ ഈ വസ്ത്രം അഥവാ യൂനിഫോമും ഹെൽമെറ്റും സൂചിപ്പിക്കുന്നത് അവർ ആർമി ഭടന്മാരാണ് എന്നാണ്. അല്ലാതെ സാധാരണ പൗരന്മാരല്ല.

(فَلا يَتْرُكُ مِمَّنْ كَانَ يَتْبَعُهُ أَحَدًا إِلا قَتَلَهُ)

“അങ്ങനെ ദജ്ജാലിനെ പിൻപറ്റിയവരിൽ പെട്ട ഒരാളെയും വധിക്കാതെ അദ്ദേഹം വിടില്ല.” (മുസ്നദു അഹ്മദ്: 14426 )

ഹദീസിൽ പ്രസ്ഥാവിക്കപ്പെട്ടിട്ടുള്ളതിൽ നിന്നും ദജ്ജാലിന്റെ ഭടന്മാർ അല്ലാത്ത അമുസ്‌ലിംകളോടല്ല യുദ്ധം എന്ന് വ്യക്തമാണ്.

ഈ വസ്തുതകളും പശ്ചാത്തലവുമെല്ലാം മറച്ചുവെച്ച് കൊണ്ട് യുദ്ധത്തിലെ ഒരു രംഗം മാത്രം അടർത്തിയെടുത്ത് അവതരിപ്പിക്കുന്ന സെലക്റ്റീവ് നെറേഷനും (Selective narrative) സെലക്റ്റീവ് എമ്പതിയും (selective empathy) എത്രമാത്രം വഞ്ചനാത്മകമാണ്!!

വിമർശനം:

(സോഷ്യൽ മീഡിയകളിൽ വ്യാപകമായി പ്രചരിക്കുന്ന വിമർശനം വിമർശകരുടെ അശ്ലീലചുവയുള്ള വരികൾ – തെറികൾ ഒഴിവാക്കി – ഇവിടെ എടുത്തു ചേർക്കുന്നു:)

“ഇസ്‌ലാമിക പ്രമാണം അനുസരിച്ചു ഒരു പുരുഷന് സ്വന്തം ഭാര്യയുടെ അനുജത്തിയെയും ലൈംഗികമായി ബന്ധപ്പെടാം. ആൺ കുട്ടികളുമായി പ്രകൃതി വിരുദ്ധ പീഡനം നടത്താം. അമ്മായിയമ്മയെ ഭോഗിക്കാം…നിങ്ങൾ ഈ വായിക്കുന്നത് ഇസ്‌ലാമിന്റെ രണ്ടാം പ്രമാണം ആയ സ്വഹീഹ് ബുഖാരിയുടെ ഹദീസിൽ നിന്നാണ്. ഓരോ മുസ്‌ലിമും നിർബന്ധമായും അംഗീകരിക്കുകയും അനുസരിക്കുകയും ചെയ്യേണ്ട ഇസ്‌ലാമിക പ്രമാണം. സ്വഹീഹ് ബുഖാരിയെ തള്ളിപ്പറയുന്നവൻ മുസ്‌ലിമല്ല – എന്നാണു മതപണ്ഡിതരുടെ ഫത്‌വ.”

മറുപടി:

സ്വഹീഹുൽ ബുഖാരിയിലെ വിമർശന വിധേയമായ കർമ്മശാസ്ത്ര ചർച്ച നേരിട്ട് കാണുക:

“ഇക്‌രിമ പറഞ്ഞു: ഇബ്നു അബ്ബാസിൽ നിന്ന്: ഒരാൾ തന്റെ ഭാര്യയുടെ സഹോദരിയെ ‘വ്യഭിചരിച്ചാൽ’ ഭാര്യയുമായുള്ള വിവാഹ ബന്ധം നിഷിദ്ധമായി മാറുന്നില്ല.

യഹ്‌യൽ കിന്ദിയിൽ നിന്ന് ഉദ്ധരിക്കപ്പെടുന്നു, അദ്ദേഹം ശുഅ്ബിയിൽ നിന്നും അബൂജഅ്ഫറിൽ നിന്നു ഉദ്ധരിക്കുന്നു: ഒരാൾ ഒരു കുട്ടിയുമായി (ലൈംഗിക) വിനോദത്തിൽ ഏർപ്പെടുകയും അവനുമായി ബന്ധപ്പെടുകയും ചെയ്താൽ അയാൾക്ക് കുട്ടിയുടെ മാതാവിനെ ഒരിക്കലും വിവാഹം ‘ചെയ്യാവതല്ല’…

ഇക്‌രിമ പറഞ്ഞു: ഇബ്നു അബ്ബാസിൽ നിന്ന്: ഒരാൾ തന്റെ ഭാര്യയുടെ മാതാവിനെ ‘വ്യഭിചരിച്ചാൽ’ ഭാര്യയുമായുള്ള വിവാഹ ബന്ധം നിഷിദ്ധമായി മാറുന്നില്ല.” (സ്വഹീഹുൽ ബുഖാരി)

1. ഇസ്‌ലാമിലെ പ്രമാണങ്ങൾ കുർആനും സ്വഹീഹായ ഹദീസുകളുമാണ്. അല്ലാതെ കുർആനും സ്വഹീഹുൽ ബുഖാരിയുമല്ല. സ്വഹീഹുൽ ബുഖാരി ഹദീസുകൾ ശേഖരിക്കപ്പെട്ട ഒരു ഗ്രന്ഥം മാത്രമാണ്. ആ ഗ്രന്ഥത്തിൽ ഹദീസുകളുണ്ട്, ഫിക്ഹ് (കർമ്മശാസ്ത്രമുണ്ട്), കുർആൻ വ്യാഖ്യാനമുണ്ട്, ഭാഷാ ചർച്ചകളുണ്ട്… അങ്ങനെ പല അറിവുകളും ഉൾകൊള്ളുന്ന ഒരു ഗ്രന്ഥമാണത്. അതിലെ ഹദീസുകളാണ് മുസലിംകൾ പ്രമാണമായി കാണുന്നത്. ആ ഹദീസുകളെ തള്ളി പറയുന്നവനാണ് ഇസ്‌ലാമിന് പുറത്ത്, അല്ലാതെ സ്വഹീഹുൽ ബുഖാരി എന്ന ഗ്രന്ഥത്തിലെ ഏത് കുറിപ്പുകളേയും തള്ളിപ്പറയുന്നവനല്ല. ബുഖാരിയിൽ നിന്ന് എന്ന് മാത്രമല്ല, വേറെ ഏത് ഹദീസ് ഗ്രന്ഥത്തിലേയും സ്വഹീഹായ ഹദീസുകളെ നിഷേധിക്കുന്നവൻ ഇസ്‌ലാമിൽ നിന്ന് പുറത്താണ്. ഇക്കാര്യത്തിൽ സ്വഹീഹുൽ ബുഖാരിക്ക് മാത്രം ഒരു പ്രത്യേകതയും ഇല്ല.

2. പ്രവാചകന്റെ(സ) വാക്കുകളുടേയും പ്രവർത്തനങ്ങളുടേയും ക്രോഡീകരണമാണ് ഹദീസ്. (നുസ്ഹത്തുന്നദ്ർ: 1:36 ,മുഖദ്ദിമ ഫീ ഉസൂലുൽ ഹദീസ്: 1:33, അൽഫദ്‌ലുൽ മുബീൻ അലാ അക്ദി ജൗഹരി സ്സമീൻ: 61)

ഇവിടെ, ഇസ്‌ലാം അവിഹിത ബന്ധങ്ങൾക്ക് അനുവാദം നൽകുന്നു എന്നതിന് തെളിവായി വിമർശകർ സ്വഹീഹുൽ ബുഖാരിയിൽ നിന്ന് എടുത്തുദ്ധരിച്ചിരിക്കുന്നത് ഇബ്നു അബ്ബാസ്, ശുഅ്ബി, അബൂ ജഅ്ഫർ എന്നീ പണ്ഡിതന്മാരുടെ വാക്കുകളാണ്. ഇത്തരം പണ്ഡിതാഭിപ്രായങ്ങൾ ഇസ്‌ലാമിൽ പ്രമാണങ്ങളല്ല. അവ ഫിക്ഹ് (കർമ്മശാസ്ത്രം) ആകുന്നു. ഫിക്ഹ് എന്നത് ഒരു പഠനശാഖയാണ് എന്ന് ഏവർക്കുമറിയാം. അതിൽ പ്രമാണങ്ങളോട് യോജിക്കുന്നവ മുസ്‌ലിംകൾ സ്വീകരിക്കുകയും പ്രമാണങ്ങളോട് യോജിക്കാത്തവ തിരസ്കരിക്കുകയും ചെയ്യും. ഇത്തരത്തിലുള്ള ഫിക്‌ഹും – ഹദീസുകൾക്ക് പുറമെ – സ്വഹീഹുൽ ബുഖാരിയിലുണ്ട്. ആ ഫിക്ഹുകൾ മുസ്‌ലിംകളുടെ അടുക്കലൊ ഇസ്‌ലാമിലൊ അലംഘനീയമായ പ്രമാണങ്ങൾ അല്ല. (ഉലൂമുൽ ഹദീസ്: ഇബ്നു സ്വലാഹ്: 22,23)

3. ഇസ്‌ലാമിക പ്രമാണങ്ങൾ അവിഹിത ബന്ധങ്ങളെ എങ്ങനെ കാണുന്നു എന്ന് തിരിച്ചറിയാൻ കുർആനിലൂടെയും സ്വഹീഹായ ഹദീസുകളിലൂടെയും ഒന്ന് കണ്ണോടിച്ചാൽ പോരെ ?! ഫിക്ഹിന്റെ അന്തരാളങ്ങളിൽ ഊളയിടണമെന്നുണ്ടോ ?!!

വ്യഭിചാരത്തെ വൻപാപങ്ങളിൽ ഒന്നായാണ് ഇസ്‌ലാം പഠിപ്പിച്ചത്.

“നിങ്ങള്‍ വ്യഭിചാരത്തെ സമീപിച്ചു പോകരുത്‌. തീര്‍ച്ചയായും അത് ഒരു നീചവൃത്തിയും ദുഷിച്ച മാര്‍ഗവുമാകുന്നു.” (കുർആൻ: 17:32)

അവിവാഹിതർ വ്യഭിചരിച്ചാൽ അവർക്ക് ഒരു ഇസ്‌ലാമിക രാഷ്ട്രത്തിൽ ലഭിക്കുന്ന ശിക്ഷയെ പറ്റി കുർആൻ പറയുന്നു:

“വ്യഭിചരിക്കുന്ന സ്ത്രീ പുരുഷന്‍മാരില്‍ ഓരോരുത്തരെയും നിങ്ങള്‍ നൂറ് അടി അടിക്കുക. നിങ്ങള്‍ അല്ലാഹുവിലും അന്ത്യദിനത്തിലും വിശ്വസിക്കുന്നവരാണെങ്കില്‍ അല്ലാഹുവിന്‍റെ മതനിയമത്തില്‍ (അത് നടപ്പാക്കുന്ന വിഷയത്തില്‍) അവരോടുള്ള ദയയൊന്നും നിങ്ങളെ ബാധിക്കാതിരിക്കട്ടെ.” (കുർആൻ: 24:2)

വിവാഹിതരായവർ വ്യഭിചരിച്ചാൽ, അവിഹിത ബന്ധങ്ങളിൽ ഏർപ്പെട്ടാൽ അവർക്ക് ഒരു ഇസ്‌ലാമിക രാഷ്ട്രത്തിൽ ലഭിക്കുന്ന ശിക്ഷയെപറ്റി പ്രവാചകൻ (സ) പഠിപ്പിച്ചത് ഇപ്രകാരമാണ്: “എന്നിൽ നിന്ന് നിങ്ങൾ പഠിക്കു… എന്നിൽ നിന്ന് നിങ്ങൾ പഠിക്കു… വിവാഹിതർ പരസ്പരം വ്യഭിചരിച്ചാൽ നൂറ് അടിയും കല്ലെറിഞ്ഞ് വധശിക്ഷ നടപ്പാക്കലുമാണ് (വിധി).” (സ്വഹീഹു മുസ്‌ലിം: 1690)

ഉമർ (റ) പറയുന്നു: (വിവാഹിതനായ വ്യഭിചാരിക്ക് ശിക്ഷയായി) അല്ലാഹുവിന്റെ ദൂതൻ (സ) കല്ലെറിഞ്ഞ് വധശിക്ഷ നടപ്പാക്കുകയാണുണ്ടായത്. അദ്ദേഹത്തിന്റെ വിയോഗാനന്തരം ഞങ്ങളും വിവാഹിതനായ വ്യഭിചാരിക്ക് കല്ലെറിഞ്ഞ് വധശിക്ഷ നടപ്പാക്കി… വിവാഹിതരായ സ്ത്രീകളോ പുരുഷന്മാരോ വ്യഭിചരിച്ചുവെന്ന് തെളിവ് സ്ഥിരപ്പെട്ടാൽ കല്ലെറിഞ്ഞുള്ള വധശിക്ഷയാണുള്ളത്… (സ്വഹീഹുൽ ബുഖാരി: 6442,സ്വഹീഹുൽ മുസ്‌ലിം: 169)

വ്യഭിചരിക്കുക പോയിട്ട് ഒരു അന്യ സ്ത്രീയെ വികാരത്തോടെ സ്പർശിക്കുന്നതും അവരുമായി ഒരു മുറിയിൽ തനിച്ചാകുന്നതും പോലും ഇസ്‌ലാമിൽ നിഷിദ്ധമാണ്.

പ്രവാചകൻ (സ) പറഞ്ഞു: “നിന്റെ തലയിൽ ഒരു ഇരുമ്പാണി കൊണ്ട് കുത്തി തറക്കുന്നതാണ് അനുവദനീയയല്ലാത്ത ഒരു സ്ത്രീയെ സ്പർശിക്കുന്നതിനേക്കാൾ നിനക്ക് നല്ലത്.” പ്രവാചകൻ (സ) പറഞ്ഞിട്ടുണ്ട്. (അൽ മുഅ്ജമുൽ കബീർ: ത്വബ്റാനി: 20:211)

പ്രവാചകൻ (സ) പറഞ്ഞു: ഒരു പുരുഷനും ഒരു സ്ത്രീയുമായി തനിച്ചാവുകയെ അരുത്… (സ്വഹീഹുൽ ബുഖാരി: 2844, സ്വഹീഹു മുസ്‌ലിം: 1341)

പ്രവാചകൻ (സ) പറഞ്ഞു: “സ്ത്രീകളുടെ അടുക്കൽ (അവർ തനിച്ചായിരിക്കെ) ചെല്ലുന്നത് നിങ്ങൾ സൂക്ഷിക്കുക. “അപ്പോൾ അൻസ്വാരികളിൽ നിന്ന് ഒരാൾ ചോദിച്ചു: അല്ലാഹുവിന്റെ ദൂതരേ, ഒരാളുടെ ഭാര്യയുടെ അടുക്കൽ (അവർ തനിച്ചായിരിക്കെ) ഭർതൃ സഹോദരൻ പ്രവേശിക്കുന്നതോ ? ‘അതും നാശമാണെന്ന്’ പ്രഖ്യാപിച്ച് അവിഹിത ബന്ധങ്ങൾക്കുള്ള പഴുതുകളെല്ലാം പ്രവാചകൻ (സ) നിഷിദ്ധമാക്കി. (സ്വഹീഹുൽ ബുഖാരി: 4934)

ഇവയെല്ലാം സ്വഹീഹുൽ ബുഖാരിയിൽ തന്നെ ഹദീസുകളായി നിലനിൽക്കുന്നുവെന്നതും വിമർശകർ ബോധപൂർവ്വം അവ അവഗണിച്ചു എന്നതും പ്രത്യേകം ശ്രദ്ധ അർഹിക്കുന്ന കാര്യമാണ്.

ഭാര്യാ സഹോദരിയെ വ്യഭിചരിക്കുക പോയി വിവാഹം ചെയ്യൽ പോലും ഇസ്‌ലാമിൽ നിഷിദ്ധമാണ്.

തന്റെ സഹോദരിയെയും കൂടി വിവാഹം ചെയ്തു കൂടെ, എങ്കിൽ ഞാൻ നിലകൊള്ളുന്ന നന്മയിൽ എന്റെ സഹോദരിക്കും പങ്കാളിയാകാമായിരുന്നു എന്ന് ആഗ്രഹം പറഞ്ഞ ഭാര്യ ഉമ്മു ഹബീബയോട് പ്രവാചകൻ (സ) പറഞ്ഞു: അതെനിക്ക് അനുവദനീയമല്ല. (സ്വഹീഹുൽ ബുഖാരി: 5101അബൂദാവൂദ്: 2056)

ഇസ്‌ലാം ആശ്ലേഷണത്തിന് മുമ്പ് രണ്ട് സഹോദരിമാരെ വിവാഹം ചെയ്ത ഒരാൾ എന്തു ചെയ്യണമെന്ന് പ്രവാചകനോട് ആരാഞ്ഞപ്പോൾ ഒരാളെ വിവാഹ മോചനം ചെയ്യാൻ പ്രവാചകൻ (സ) നിർദേശം നൽകി. (അബൂദാവൂദ്: 2243, തുർമുദി: 1129)

ഇനി, സ്വവർഗരതിയുടെ കാര്യമെടുക്കാം. ഒരു മനുഷ്യന് പ്രവർത്തിക്കാൻ കഴിയുന്നതിൽ വെച്ച് ഏറ്റവും വലിയ പാതകമായി സ്വവർഗരതിയെ പഠിപ്പിച്ച വ്യക്തിയാണ് പ്രവാചകൻ (സ). സ്വവർഗരതിയിൽ കൂട്ടായി നിർലജ്ജം അഭിരമിച്ച ലൂത്ത് നബിയുടെ ജനതയെ അല്ലാഹു ശിക്ഷിച്ച ചരിത്രം പ്രവാചകൻ (സ) ഈ ലോകത്തെ പഠിപ്പിച്ചു: “അങ്ങനെ സൂര്യോദയത്തോടെ ആ ഘോരശബ്ദം അവരെ പിടികൂടി.അങ്ങനെ ആ രാജ്യത്തെ നാം തലകീഴായി മറിക്കുകയും, ചുട്ടുപഴുത്ത ഇഷ്ടികക്കല്ലുകള്‍ അവരുടെ മേല്‍ നാം വര്‍ഷിക്കുകയും ചെയ്തു.” (കുർആൻ 15: 73,74)

പ്രവാചകൻ (സ) പറഞ്ഞു: ആരെങ്കിലും ലൂത്ത് നബിയുടെ ജനതയുടെ പ്രവർത്തനം (സ്വവർഗരതി) പ്രവർത്തിച്ചാൽ ആ പ്രവർത്തനത്തിൽ ഏർപ്പെട്ടവന്റെയും അതിന് വിധേയനായി നിന്നു കൊടുത്തവന്റെയും മേൽ വധശിക്ഷ നടപ്പാക്കണം. (തുർമുദി: 1456, അബൂദാവൂദ്: 4462, ഇബ്നുമാജ: 2561)

പ്രവാചകൻ (സ) പറഞ്ഞു: ലൂത്ത് നബിയുടെ ജനതയുടെ പ്രവർത്തനം (സ്വവർഗരതി) പ്രവർത്തിച്ചവനെ അല്ലാഹു ശപിച്ചിരിക്കുന്നു. പ്രവാചകൻ (സ) മൂന്ന് തവണ ഇത് ആവർത്തിച്ചു പറഞ്ഞു. (മുസ്നദു അഹ്മദ്: 2915)

ചുരുക്കത്തിൽ അവിഹിത ബന്ധങ്ങൾ, വ്യഭിചാരം, സ്വവർഗരതി തുടങ്ങിയവ ഇസ്‌ലാമിൽ നിഷിദ്ധമാണെന്ന് ഇസ്‌ലാമിക പ്രമാണങ്ങൾ തന്നെ വ്യക്തമായി പ്രസ്ഥാവിക്കുന്നു. ഇവയെല്ലാം സ്വഹീഹുൽ ബുഖാരിയിൽ തന്നെ ഹദീസുകളായി നിലനിൽക്കുകയും ചെയ്യുന്നു.

3. പ്രവാചകാനുചരന്മാരുടെ കാലഘട്ടത്തിലെ അവസാന സന്ധിയിൽ ഇസ്‌ലാമിന്റെ വളർച്ച ദ്രുതഗതിയിലാവുകയും ലക്ഷോപലക്ഷങ്ങൾ ഇസ്‌ലാമിലേക്ക് കടന്നുവരികയും ചെയ്യാൻ തുടങ്ങിയപ്പോൾ ദുർബല വിശ്വാസികളും കപട വിശ്വാസികളും ഇസ്‌ലാമിക സമൂഹത്തിൽ സ്ഥാനം പിടിച്ചു. വളർച്ച പ്രാപിച്ച മറ്റേത് സമുദായങ്ങളിലേതുമെന്ന പോലെ മുസ്‌ലിംകൾക്കിടയിലും ജീർണതകളും അധാർമികതകളും വർദ്ധിച്ചു. മദ്യം, വ്യഭിചാരം, കളവ്, മോഷണം, കൊള്ള, കൊല തുടങ്ങിയ സാമൂഹിക- വൈയക്തിക തിന്മകൾ എല്ലാ സമൂഹങ്ങളിലും ഇടതടവില്ലാതെ നടക്കുന്നുണ്ട്. മുസ്‌ലിം സമൂഹത്തിൽ ദുർബല വിശ്വാസികളും കപട വിശ്വാസികളും സ്ഥാനം പിടിച്ചതോടെ അവരിലും ഇത്തരം ദൂഷ്യങ്ങൾ പെരുകുക സ്വഭാവികം മാത്രം. അതുകൊണ്ട് തന്നെ ഈ സാമൂഹിക ഭൂമികയെ പരിഗണിച്ച് കർമ്മശാസ്ത്ര ചർച്ചകളെ വാർത്തെടുക്കാൻ കർമശാസ്ത്ര പണ്ഡിതർ നിർബന്ധിതരായി. അല്ലാതെ സമൂഹിക യഥാർത്ഥ്യങ്ങളെ അവഗണിച്ച് എങ്ങനെ ഫിക്ഹ് (കർമ്മശാസ്ത്ര) ചർച്ച ചെയ്യും? അധാർമിക പ്രവണതകളെയും ആ പ്രവണതകളുടെ അനന്തരഫലമായുണ്ടാകുന്ന സാമൂഹിക- കുടുംബ- വൈയക്തിക പ്രശ്നങ്ങളെയെല്ലാം കർമ്മശാസ്ത്ര പണ്ഡിതർ ചർച്ച ചെയ്തിട്ടുണ്ട്. അതിൽ ഒരു വിഷയവും അറപ്പു കൊണ്ട് മാറ്റി വെച്ചിട്ടില്ല. ഒരു കൈപ്പുറ്റ യാഥാർത്ഥ്യത്തെയും അവഗണിച്ച് ഫിക്ഹിനെ (കർമ്മശാസ്ത്ര) വാർത്തെടുക്കാൻ ശ്രമിച്ചിട്ടുമില്ല, ശ്രമിക്കാനും പാടില്ല. സ്വാഭാവികമായും അവിഹിതങ്ങൾ നടക്കുകയും അവ മൂലം കുടുംബ – സാമൂഹിക പ്രശ്നങ്ങൾ ഉടലെടുക്കുകയും ചെയ്തപ്പോൾ അത്തരം പ്രശ്നങ്ങളിലെ കർമ്മശാസ്ത്ര വിധികളെ പറ്റി ചർച്ച ചെയ്തിട്ടുണ്ട്. അതിനർത്ഥം അത്തരം അവിഹിതങ്ങളെ ഇസ്‌ലാമിക കർമ്മശാസ്ത്രമോ, കർമ്മശാസ്ത്ര ചർച്ചകളുടെ ആധാരമായ ഇസ്‌ലാമിക പ്രമാണങ്ങളൊ അംഗീകരിക്കുന്നു എന്നല്ല. ഒരു രാജ്യത്ത് കൊലപാതകവും കലാപവും ബലാത്സംഗവുമെല്ലാം നടന്നാലുള്ള അനന്തര നടപടികളെ സംബന്ധിച്ച നിയമങ്ങൾ ഉണ്ടെന്നതും ഭരണഘടന അത് ചർച്ച ചെയ്യുന്നുണ്ട് എന്നതും ആ രാജ്യവും ഭരണഘടനയും ഈ ദ്രോഹങ്ങൾക്ക് അംഗീകാരം നൽകുന്നു എന്നതിന് തെളിവാണോ ? അല്ലല്ലൊ. ഈ വസ്തുത മനസ്സിലാക്കിയതിന് ശേഷമാവണം സ്വഹീഹുൽ ബുഖാരിയിലെ കർമ്മശാസ്ത്ര ചർച്ചയെ നാം സമീപിക്കേണ്ടത്.

ബലാൽസംഗങ്ങളുടേയും അവിഹിത ബന്ധങ്ങളുടേയും അനന്തര നടപടികൾ ചർച്ച ചെയ്യുകയാണ് ബുഖാരി ഇവിടെ ചെയ്യുന്നത്. അല്ലാതെ അവിഹിതങ്ങളെ ഒരു വരിയിൽ പോലും അംഗീകരിക്കുന്നില്ല. അവിഹിത ബന്ധങ്ങൾ നിഷിദ്ധമാണെന്നും അവ ഇഹ-പര ലോകങ്ങളിലെ ശിക്ഷക്ക് അർഹമാണെന്നും ഹദീസുകളിലൂടെ ബുഖാരി തന്നെ വ്യക്തമാക്കിയതും നാം കണ്ടു. എന്നാൽ ഇതൊന്നും വില വെക്കാത്ത ഒരു പുരുഷൻ അവിഹിത ബന്ധങ്ങളിൽ ഏർപ്പെട്ടാൽ തുടർന്ന് ബന്ധങ്ങളിൽ എന്തൊക്കെ സംഭവിക്കും എന്ന് മാത്രമാണ് ചർച്ച. അത് അവിഹിതങ്ങൾ നടത്താൻ തെളിവാണെന്ന് ദുർവ്യാഖ്യാനിച്ച് അവതരിപ്പിക്കുന്നതിന് ബുഖാരി എന്ത് പിഴച്ചു ?

ഒരു പുരുഷൻ ഭാര്യയുടെ സഹോദരിയുമായും അല്ലെങ്കിൽ ഭാര്യയുടെ മാതാവുമായും അവിഹിത ബന്ധത്തിൽ ഏർപ്പെട്ടാൽ ഭാര്യയുമായുള്ള വിവാഹ ബന്ധം നിഷിദ്ധമാകുമോ അഥവാ വിവാഹ ബന്ധം വേർപ്പെടുമോ എന്നത് പ്രസക്തമായ ഒരു ചോദ്യമല്ലെ ? അവിഹിത വീരന്റെ പക്ഷത്ത് നിന്ന് ചിന്തിക്കാതെ ഇരയായ ഭാര്യയുടെയും മക്കളുടേയും പക്ഷത്ത് നിന്ന് ചിന്തിക്കുക. ഒരു വിശ്വാസിയായ മുസ്‌ലിം സ്ത്രീ, മുസ്‌ലിം നാമധാരിയായ ഒരു എക്സ് മുസ്‌ലിമിനെ (അയാളുടെ ഭൗതീക വാദത്തെ പറ്റി അറിയാതെ) വിവാഹം ചെയ്തുവെന്ന് കരുതുക; ആ ബന്ധത്തിൽ മക്കളുമുണ്ടായി. ഭൗതീക വാദിയായ ഇയാൾക്ക് അവിഹിത ബന്ധങ്ങൾ ലൈംഗിക സ്വാതന്ത്ര്യത്തിന്റെ ഭാഗമാണല്ലൊ. അതുകൊണ്ട് തന്നെ അയാൾ ഭാര്യയുടെ സഹോദരിയെ വ്യഭിചരിച്ചു. അതോടെ അയാൾക്ക് ഭാര്യയുമായുള്ള വിവാഹ ബന്ധം നിഷിദ്ധമാവുമോ? വിവാഹ ബന്ധം വേർപ്പെടുമോ? എന്നത് ചർച്ച ചെയ്യേണ്ടതില്ലെ?! ഇല്ലെങ്കിൽ ഇരയായ ഭാര്യയുടെയും മക്കളുടേയും അവകാശങ്ങളുടെ കാര്യമോ? അവിഹിത ബന്ധത്തിൽ അഥവാ വ്യഭിചാരത്തിൽ ഏർപ്പെട്ട ഭർത്താവിന് ഇസ്‌ലാമിക ഭരണകൂടം വധശിക്ഷ നടപ്പിലാക്കും. പക്ഷെ അയാൾ വ്യഭിചരിച്ചു എന്നതിനാൽ അയാളുടെ ഭാര്യ അയാൾക്ക് ഭാര്യ അല്ലാതായി എന്ന് പറഞ്ഞാൽ ഭാര്യക്കും കുട്ടികൾക്കും അയാളുടെ സ്വത്തിൽ നിന്ന് കിട്ടേണ്ട അനന്തരാവകാശങ്ങളും മറ്റു സാമൂഹിക-സാമ്പത്തിക അവകാശങ്ങളും നിഷേധിക്കപ്പെടില്ലെ? പുരുഷൻ അവിഹിതത്തിൽ ഏർപ്പെട്ടു എന്നത് കൊണ്ട് ഭാര്യ എന്ത് പിഴച്ചു ?! അപ്പോൾ, ഭാര്യ സഹോദരിയുമായി അല്ലെങ്കിൽ ഭാര്യാമാതാവുമായി പുരുഷൻ അവിഹിതത്തിൽ ഏർപ്പെട്ടാൽ അവളുമായുള്ള വിവാഹ ബന്ധം നിഷിദ്ധമായി എന്ന് പറഞ്ഞാൽ അത് ഇരക്ക് അനുകൂലമായാണോ പ്രതികൂലമായാണോ ഭവിക്കുക എന്നത് ചർച്ച ചെയ്യേണ്ടതില്ലെ?!! “അയ്യേ അവിഹിതം ചർച്ച ചെയ്യുന്നു” എന്ന് പറഞ്ഞ് മാറി നിൽക്കുന്നതാണോ ധാർമികത!?

ഇനി, “ഒരാൾ ഒരു കുട്ടിയുമായി (ലൈംഗിക) വിനോദത്തിൽ ഏർപ്പെടുകയും അവനുമായി ബന്ധപ്പെടുകയും ചെയ്താൽ അയാൾക്ക് കുട്ടിയുടെ മാതാവിനെ ഒരിക്കലും വിവാഹം ‘ചെയ്യാവതല്ല’… ” എന്ന ശുഅ്ബി, അബൂജഅ്ഫർ എന്നീ പണ്ഡിതന്മാരുടെ കർമ്മശാസ്ത്ര (ഹദീസ് അല്ലെന്ന് പ്രത്യേകം ഓർക്കുക) അഭിപ്രായം എടുക്കുക.

ഒന്ന്, കുട്ടിയുമായുള്ള ലൈംഗിക ബന്ധം അനുവദനീയമാണെന്ന സൂചന പോലും അതിലില്ല. എന്ന് മാത്രമല്ല സ്വവർഗരതി നിഷിദ്ധമാണെന്നും അവ ഇഹ-പര ലോകങ്ങളിലെ ശിക്ഷക്ക് അർഹമാണെന്നും ഹദീസുകളിലൂടെ ബുഖാരി തന്നെ വ്യക്തമാക്കിയതും നാം കണ്ടു.

രണ്ട്, കുട്ടിയുടെ മാതാവിനെ ഒരിക്കലും വിവാഹം ‘ചെയ്യാവതല്ല’ എന്നാണ് ശുഅ്ബി, അബൂജഅ്ഫർ എന്നീ പണ്ഡിതന്മാർ പറഞ്ഞത്; അല്ലാതെ വിവാഹം ചെയ്യാം എന്നല്ല. അങ്ങനെ ഒരു വൻ പാപമായ നിഷിദ്ധ ബന്ധം സംഭവിക്കുകയും അയാൾ കുട്ടിയെ ലൈംഗിക പീഢനത്തിന് ഇരയാക്കിയെന്ന് കോടതിയിൽ തെളിവൊന്നും ഹാജരാക്കാൻ കഴിയാതിരിക്കുകയും ചെയ്താൽ അയാൾക്ക് മാതാവിനെ വിവാഹം ചെയ്യാനുള്ള അവകാശമെ നൽകരുത് എന്നെ പറഞ്ഞിട്ടുള്ളു. ലൈംഗിക പീഢനത്തിന് ഇരയാക്കിയെന്നതിന് തെളിവ് ഹാജരാക്കപ്പെട്ടാൽ അയാൾക്ക് ശിക്ഷയും വിധിക്കണം. ഈ അതി പ്രസക്തമായ ചർച്ചകളെ ദുർവ്യാഖ്യാനിച്ച് ഇസ്‌ലാം വൻപാപങ്ങളിൽ ഒന്നായി കാണുന്ന, വധശിക്ഷക്ക് അർഹമായി പഠിപ്പിച്ച സ്വവർഗരതി ഇസ്‌ലാം തന്നെ അംഗീകരിക്കുന്നു എന്ന് ചിത്രീകരിക്കുന്ന ഭോഷത്തത്തെ പുച്ഛത്തോടെ വലിച്ചെറിയുക.

ഇത്തരം കയ്പുറ്റ, സമാനമായ വേറേയും വിഷയങ്ങളിലും കർമശാസ്ത്ര ചർച്ചകൾ മുസ്‌ലിം പണ്ഡിതരുടെ ഗ്രന്ഥങ്ങളിൽ നടന്നിട്ടുണ്ട്.

ഒന്ന്, ഭാര്യാ ഭർത്താക്കന്മാരിൽ ഒരാൾ ഇസ്‌ലാം മതം ഉപേക്ഷിച്ച് ‘മുർത്തദ്ദാ’യാൽ തുടർന്നുള്ള അനന്തരഫലങ്ങൾ കർമ്മശാസ്ത്ര പണ്ഡിതർ ചർച്ച ചെയ്തിട്ടുണ്ട്. അതിനർത്ഥം മുർത്തദ്ദ് (മത പരിത്യാഗി) ആവൽ ഇസ്‌ലാം അനുവദിച്ചു എന്നാണോ ?!!

ഇബ്നു കുദാമ പറഞ്ഞു: ഭാര്യാ ഭർത്താക്കന്മാരിൽ ഒരാൾ മുർത്തദ്ദായാൽ – അത് പരസ്പര ലൈംഗീകബന്ധത്തിന് മുമ്പാണെങ്കിൽ വിവാഹ ബന്ധം വേർപ്പെടും… ദാവൂദിൽ നിന്ന് ഉദ്ധരിക്കപ്പെട്ടിരിക്കുന്നത് മത പരിത്യാഗം മൂലം വിവാഹ ബന്ധം വേർപ്പെടില്ല; അത് അവശേഷിക്കുമെന്നാണ്… (മുഗ്നി: 7:133)

പരസ്പര ലൈംഗീകബന്ധത്തിന് ശേഷമാണെങ്കിലും ഭാര്യാ ഭർത്താക്കന്മാരിൽ ഒരാൾ മുർത്തദ്ദായാൽ വിവാഹ ബന്ധം വേർപ്പെടും എന്നാണ് ഹനഫി മാലികി മദ്ഹബിലെ അഭിപ്രായം… (അൽ മൗസൂഅത്തുൽ ഫിക്ഹിയ്യ:22/198, തുഹ്‌ഫത്തുൽ മുഹ്താജ്: 7:328 )

ഈ ചർച്ചകളൊക്കെ തെളിയിക്കുന്നത് ദമ്പതിമാരിൽ ഒരാൾക്ക് മുർത്തദ്ദ് ആവൽ ഇസ്‌ലാം അനുവദിക്കുന്നു എന്നാണോ ?!! (ഇതിൽ ഏത് അഭിപ്രായമാണ് ഇസ്‌ലാമിക പ്രമാണങ്ങളുടെ പിൻബലമുള്ളത് എന്നത് മറ്റൊരു വിഷയമാണ്. അത് ഇവിടെ ചർച്ച ചെയ്യുന്നത് ഉചിതമല്ലല്ലൊ)

രണ്ട്, ഭാര്യാ ഭർത്താക്കന്മാരിൽ ഒരാൾ മറ്റൊരാളെ കൊലപ്പെടുത്തിയാൽ മക്കൾക്ക് പ്രതിക്രിയയിൽ അഭിപ്രായം പ്രകടിപ്പിക്കാനുള്ള അവകാശമുണ്ട് എന്ന് തുടങ്ങി… കൊലയെ തുടർന്നുള്ള അനന്തര നടപടികൾ കർമ്മശാസ്ത്ര പണ്ഡിതർ ചർച്ച ചെയ്തിട്ടുണ്ട്. (മുഗ്നി: 8/287, കശ്ശാഫുൽ കനാഅ്:5:529, അൽ മൗസൂഅത്തുൽ ഫിക്ഹിയ്യ അൽകുവൈത്തിയ്യ: 21:56)

അതിനർത്ഥം ദമ്പതികളിൽ ഒരാൾക്ക് മറ്റൊരാളെ കൊലപ്പെടുത്താൻ ഇസ്‌ലാം അനുവാദം നൽകുന്നു എന്നാണോ ?!!

ശവഭോഗത്തെ ഇസ്‌ലാം എങ്ങനെ കാണുന്നു?

ഇല്ല. ഒരു നിലക്കും അനുവദിക്കുന്നില്ല. ഇസ്‌ലാമിലെ ഭോഗാനുവാദങ്ങളെ കുറിച്ച് മുകളിൽ കുറിച്ചത് മനസ്സിലാക്കിയല്ലോ. അനുവാദങ്ങൾക്കപ്പുറത്തെത്തിയ നിരോധിത കാര്യമാണെങ്കിലും ശവഭോഗത്തിനുള്ള ശിക്ഷയെക്കുറിച്ച് ഖുർആൻ ഹദീസിൽ പ്രത്യേകം പരാമർശങ്ങളില്ല. അക്കാലത്ത് അങ്ങനെയൊരു സംഭവം മനുഷ്യർക്കിടയിൽ കാണപ്പെട്ടിരുന്നില്ല എന്നാണതിനർത്ഥം. മനുഷ്യരിൽ നടമാടിയിരുന്ന അസാന്മാർഗ്ഗികതകളെയാണ് ഇസ്‌ലാം അഡ്രസ് ചെയ്തത്. ചിലതിനു ഖണ്ഡിതമായ ശിക്ഷ പ്രഖ്യാപിച്ചു. അവയിലടങ്ങിയ പൊതു തത്ത്വങ്ങളുടെ അടിസ്ഥാനത്തിൽ, അസാന്മാർഗ്ഗികതകളുടെ പുതിയ അവതാരങ്ങളെ നേരിടുവാൻ ജ്ഞാന -അധികാര നേതൃത്വങ്ങൾക്ക് അനുവാദം നൽകിയിരിക്കുകയാണ്. തദടിസ്ഥാനത്തിൽ, ശവഭോഗത്തിനുള്ള ശിക്ഷ തീരുമാനിക്കേണ്ടത്, അതാതുകാലത്തെ പരിഗണനകൾക്കും ധാർമ്മിക സാംസ്കാരിക മാനങ്ങൾക്കും വിധേയമായാണ്. ‘ശാരീരികശിക്ഷ’ (ഹദ്ദ്) പ്രഖ്യാപിക്കാത്ത അസാന്മാർഗിക കൃത്യങ്ങൾക്ക് നൽകേണ്ട ശിക്ഷയെക്കുറിച്ച് പ്രമുഖ ഖുർആൻ വ്യാഖ്യാതാവും ശാഫിഈ കർമ്മ ധാരയുടെ വക്താവുമായ അല്ലാമാ ഖത്വീബ് ശർബീനി രേഖപ്പെടുത്തുന്നു: “എന്നാൽ, സ്ത്രീകൾ തമ്മിലുള്ള ഭോഗം, ശവ ഭോഗം, സ്വയം ഭോഗം എന്നിവയ്ക്ക് (വ്യഭിചാരം, സാഡോമി എന്നിവയുടെ ശിക്ഷപോലെ) പ്രഖ്യാപിത ശിക്ഷകളൊന്നും മതനിയമാക്കപ്പെട്ടിട്ടില്ല. അവയ്ക്ക് ‘തഅസീർ’ മാത്രമേ നിയമത്തിലുള്ളൂ”. وأما السحاق من النساء وإتيان المرأة الميتة والاستمناء باليد فلا يشرع فيه شيء من ذلك إلا التعزير (سراج المنير ظ الخطيب الشربيني .

ഇസ്‌ലാമിക ധാർമ്മിക കാഴ്ചപ്പാടിൽ, തഅസീർ അർഹിക്കുന്ന അസാന്മാർഗ്ഗികതയാണ് ശവഭോഗം. അത് അവസാനിപ്പിക്കാൻ താക്കീത് മുതൽ ശാരീരിക ശിക്ഷ വരെ ആകാം.

മൃഗഭോഗത്തിനുള്ള ശിക്ഷയെന്താണ് ഇസ്‌ലാമിൽ?

മൃഗഭോഗം നിഷിദ്ധമാണെന്നു മുസ്‌ലിം സമുദായത്തിലെ എല്ലാ ചിന്താധാരകളും ഏകസ്വരത്തിൽ പ്രഖ്യാപിച്ചിരിക്കുന്നു. (أَجْمَعَتِ الْأُمَّةُ عَلَى حُرْمَةِ إِتْيَانِ الْبَهَائِمِ തഫ്‌സീർ റാസി, തഫ്‌സീർ ഖത്തീബ് ശിർബീനി). അനുവാദത്തെ അതിക്രമിക്കുന്ന അസാന്മാർഗ്ഗികതയിൽ പെട്ട കാര്യമാണ് മൃഗഭോഗമെന്ന്, 23 / 7 വ്യാഖ്യാനിച്ചുകൊണ്ട് അല്ലാമാ സആലബി രേഖപ്പെടുത്തുന്നു: { هُمُ ٱلْعَادُونَ } يقتضي تحريمَ الزِّنا والاستمناءِ ومواقعةِ البهائم، وكُلُّ ذلك داخل في قوله: { وَرَآءَ ذَٰلِكَ } . ഖുർആനിൽ ഇത് സംബന്ധമായ പരാമർശങ്ങൾ കാണുന്നില്ലെങ്കിലും ഹദീസുകളിൽ വന്നിട്ടുള്ള ഗൗരവതരമായ വിലക്കുകളാണ് മൃഗഭോഗ നിരോധനത്തിന് നിദാനം. “മൃഗത്തെ ഭോഗിക്കുന്നവൻ ശപിക്കപ്പെട്ടവനാകുന്നു.” ملعون من وقع على بهيمة (ഇബ്‌നു അബ്ബാസ്/ അഹ്മദ്), “നാല് കൂട്ടർ, അല്ലാഹുവിന്റെ കോപത്തിൽ പുലരുന്നു; അവന്റെ കോപത്തിൽ അസ്തമിക്കുന്നു: സ്ത്രീ പ്രകൃതം നടിക്കുന്ന പുരുഷൻമാർ, പുരുഷ പ്രകൃതം നടിക്കുന്ന സ്ത്രീകൾ, മൃഗഭോഗികൾ, പുരുഷന്മാരെ ഭോഗിക്കുന്ന പുരുഷന്മാർ എന്നിവരാകുന്നു അവർ” أربعة يصبحون في غضب الله ويمسون في سخط الله قلت : من هم يا رسول الله ؟ قال : المتشبهون من الرجال بالنساء والمتشبهات من النساء بالرجال والذي يأتي البهيمة والذي يأتي الرجال (ത്വബ്റാനി) തുടങ്ങിയ ഹദീസുകളാകുന്നു മുഖ്യം. ഹദീസ് കൃതികളിലും ധർമ്മ ശിക്ഷണ കൃതികളിലും ‘മൃഗത്തെ പരിണയിക്കുന്നവൻ’ എന്ന അധ്യായത്തിൽ ഇതെല്ലാം രേഖപ്പെടുത്തപ്പെട്ടിട്ടുണ്ട്.

മൃഗ ഭോഗികൾക്കുള്ള ശിക്ഷ ഖുർആനിൽ വ്യക്തമാക്കുന്നില്ല. നബിയുടെ കാലത്ത് ഇത്തരം സംഭവം റിപ്പോർട്ട് ചെയ്യപ്പെട്ടിട്ടില്ല. അതിനാൽ മൃഗഭോഗിക്കുള്ള ശിക്ഷ ഖണ്ഡിതമല്ല. എന്നാൽ, നബി ശിഷ്യന്മാരിൽ നിന്നും ലഭിച്ചിട്ടുള്ള അഭിപ്രായങ്ങൾ വ്യത്യസ്തമായതിനാൽ, മൃഗഭോഗിക്കുള്ള ശിക്ഷയിൽ കർമ്മ ശാസ്ത്രം ഭിന്ന വീക്ഷണത്തിലാണ്. പ്രമാണങ്ങളെ വിശകലനം ചെയ്തുകൊണ്ടിരിക്കെ, ഇമാം ശാഫിഈ പലസമയങ്ങളിലായി മൂന്നു നിലപാടുകൾ പ്രകടിപ്പിച്ചിട്ടുണ്ട്. മൃഗഭോഗിക്ക് വ്യഭിചാരിയുടെ ശിക്ഷ നൽകണം എന്നതായിരുന്നു ഒരു നിലപാട്. അതായത്, വിവാഹിതനായ മൃഗഭോഗിയെ എറിഞ്ഞുകൊല്ലുകയും അവിവാഹിതനാണെങ്കിൽ നൂറടി നൽകി നാടുകടത്തുകയും വേണം. വ്യഭിചാരത്തോടു സമീകരിച്ചായിരുന്നു ഈ നിലപാട്. താബിഈ പ്രമുഖൻ ഹസൻ അൽ ബസ്വരിയുടെ നിലപാട് ഇതായിരുന്നു. മറ്റൊരിക്കൽ കൂടുതൽ രൂക്ഷമായ നിലപാടായിരുന്നു ഇമാം ശാഫിഈ എടുത്തത്: വിവാഹിതനായാലും അല്ലെങ്കിലും മൃഗഭോഗിയെ വധിക്കണം. ‘മൃഗത്തെ ഭോഗിച്ചവനെ വധിച്ചുകളയുക’ എന്ന നബിവചനം ഇബ്‌നു അബ്ബാസ് ഉദ്ധരിച്ചത് കണ്ടപ്പോഴായിരുന്നു ഈ നിലപാടിലെത്തിയത്. എന്നാൽ, പ്രസ്തുത ഹദീസിന്റെ നിവേദക പരമ്പരയിൽ ശങ്ക ജനിക്കുകയും, മറ്റു കർമ്മശാസ്ത്ര ധാരകളുടെ വക്താക്കളുടെ വീക്ഷണങ്ങൾ വിശകലനം ചെയ്യുകയും ചെയ്തപ്പോൾ, ഖണ്ഡിതമായ ശിക്ഷ പ്രഖ്യാപിക്കാതെ ‘തഅസീർ’ നടപ്പിലാക്കേണ്ട അസാന്മാർഗ്ഗികതയായി മൃഗഭോഗത്തെ പ്രഖ്യാപിക്കുകയായിരുന്നു. വ്യത്യസ്ത കർമ്മ ശാസ്ത്ര ധാരകളുടെ വക്താക്കളായ ഇമാം അബൂ ഹനീഫ, ഇമാം മാലിക്, ഇമാം സൗരി, ഇമാം അഹ്മദ് തുടങ്ങിയവർ ഇതേ അഭിപ്രായമാണ്‌ പ്രകടിപ്പിച്ചത്. സാധാരണ മനുഷ്യ വികാരം പ്രേരിപ്പിക്കുന്ന തിന്മകളെ സമൂലം ഇല്ലാതാക്കുവാനാണ് ശരീഅത്ത് ഹദ്ദ് ശിക്ഷ വിധിച്ചിട്ടുള്ളത്. എന്നാൽ, മൃഗഭോഗം സാധാരണ മനുഷ്യരുടെ ശാരീരിക തൃഷ്ണ അല്ലെന്നതുകൊണ്ടാണ് അതിനു കൃത്യമായ ഹദ്ദ് ഇല്ലാതിരുന്നത് എന്നും ഇമാമുകൾ വ്യക്തമാക്കുന്നു. (ഇമാം റാസി / തഫ്‌സീർ ). أَجْمَعَتِ الْأُمَّةُ عَلَى حُرْمَةِ إِتْيَانِ الْبَهَائِمِ. وَلِلشَّافِعِيِّ رَحِمَهُ اللَّه فِي عُقُوبَتِهِ أَقْوَالٌ: أَحَدُهَا يَجِبُ بِهِ حَدُّ الزِّنَا فَيُرْجَمُ الْمُحْصَنُ وَيُجْلَدُ غَيْرُ الْمُحْصَنِ وَيُغَرَّبُ وَالثَّانِي: أَنَّهُ يُقْتَلُ مُحْصَنًا كَانَ أَوْ غَيْرَ مُحْصَنٍ. لِمَا رُوِيَ عَنِ ابْنِ عَبَّاسٍ رَضِيَ اللَّه عَنْهُمَا قَالَ قَالَ رَسُولُ اللَّه صَلَّى اللَّه عَلَيْهِ وَسَلَّمَ: «مَنْ أَتَى بَهِيمَةً فَاقْتُلُوهُ وَاقْتُلُوهَا مَعَهُ» فَقِيلَ لِابْنِ عَبَّاسٍ: مَا شَأْنُ الْبَهِيمَةِ؟ فَقَالَ: مَا أَرَاهُ قَالَ ذَلِكَ إِلَّا أَنَّهُ كَرِهَ أَنْ يُؤْكَلَ لَحْمُهَا، وَقَدْ عُمِلَ بِهَا ذَلِكَ الْعَمَلُ. وَالْقَوْلُ الثَّالِثُ: وَهُوَ الْأَصَحُّ وَهُوَ قَوْلُ أَبِي حَنِيفَةَ وَمَالِكٍ وَالثَّوْرِيِّ وَأَحْمَدَ رَحِمَهُمُ اللَّه: أَنَّ عليه التعزير لِأَنَّ الْحَدَّ شُرِعَ لِلزَّجْرِ عَمَّا تَمِيلُ النَّفْسُ إِلَيْهِ، وَهَذَا الْفِعْلُ لَا تَمِيلُ النَّفْسُ إِلَيْهِ، .وَضَعَّفُوا حَدِيثَ ابْنِ عَبَّاسٍ رَضِيَ اللَّه عَنْهُمَا لِضَعْفِ إِسْنَادِهِ وَإِنْ ثَبَتَ فَهُوَ مُعَارَضٌ بِمَا رُوِيَ أَنَّهُ عَلَيْهِ السَّلَامُ نَهَى عَنْ ذَبْحِ الْحَيَوَانِ إِلَّا لِأَكْلِهِ

ചുരുക്കത്തിൽ, മുഹമ്മദ് നബിയുടെ കാലത്തും സ്വഹാബികളുടെ കാലത്തും മൃഗഭോഗ സംഭവം ഉണ്ടാവുകയോ അതിനുള്ള ശിക്ഷ നടപ്പാക്കുകയോ ചെയ്ത കാര്യം റിപ്പോർട്ട് ചെയ്യപ്പെടാത്തതിനാൽ, ശിക്ഷയുടെ കാര്യത്തിൽ ഭിന്ന വീക്ഷണം നിലനിൽക്കുന്നു. വധ ശിക്ഷ വേണമെന്ന് ഒരു വിഭാഗം. എന്ത് ശിക്ഷ വേണമെന്ന് കോടതിക്ക് തീരുമാനിക്കാമെന്ന് മറ്റൊരു വിഭാഗം.

وَقَدْ اخْتَلَفَ أَهْلُ الْعِلْمِ فِيمَنْ وَقَعَ عَلَى بَهِيمَةٍ، فَأَخْرَجَ الْبَيْهَقِيُّ عَنْ جَابِرِ بْنِ زَيْدٍ أَنَّهُ قَالَ: مَنْ أَتَى الْبَهِيمَةَ أُقِيمَ عَلَيْهِ الْحَدُّ. وَأَخْرَجَ أَيْضًا عَنْ الْحَسَنِ بْنِ عَلِيٍّ – رَضِيَ اللَّهُ عَنْهُمَا – أَنَّهُ قَالَ: إنْ كَانَ مُحْصَنًا رُجِمَ وَرُوِيَ أَيْضًا عَنْ الْحَسَنِ الْبَصْرِيِّ أَنَّهُ قَالَ: هُوَ بِمَنْزِلَةِ الزَّانِي، قَالَ الْحَاكِمُ: أَرَى أَنْ يُجْلَدَ وَلَا يُبْلَغَ بِهِ الْحَدُّ، وَهُوَ مُجْمَعٌ عَلَى تَحْرِيمِ إتْيَانِ الْبَهِيمَةِ، كَمَا حَكَى ذَلِكَ صَاحِبُ الْبَحْرِ. (نيل الاوطار )

ഇസ്‌ലാമിക ദൃഷ്ടിയിൽ തെറ്റായതും കൃത്യമായ ശിക്ഷ പ്രഖ്യാപിക്കാത്തതുമായ കാര്യങ്ങൾ തടയാൻ ‘ഉത്തരവാദപ്പെട്ട’വർക്ക് നൽകുന്ന ശിക്ഷാ അനുവാദമാണ് തഅസീർ. ഇതിന്റെ പരമാവധി നിലവിൽ നിയമമായിട്ടുള്ള വധം, അടി, നാടുകടത്തൽ തുടങ്ങിയ ഹദ്ദ് പരിധിയാണ്. പ്രസ്തുത അസാന്മാർഗ്ഗികത/ അധാർമികത അവസാനിപ്പിക്കാൻ കേവല താക്കീത് മതിയെങ്കിൽ അതാണ് ചുരുങ്ങിയ രൂപം. ഹദ്ദ് പരിധിയിൽ വരുന്ന ശിക്ഷ നടപ്പിലാക്കാൻ സ്ഥലത്തെ ഖലീഫ/അമീറിന് മാത്രമേ അധികാരമുള്ളൂ. ലളിത ശിക്ഷകൾ രക്ഷാധികാരിക്കും ഗുരുനാഥനും ചെയ്യാവുന്നതാണ്. ചുരുക്കത്തിൽ, ഇസ്‌ലാമിക ധാർമ്മിക കാഴ്ചപ്പാടിൽ, തഅസീർ അർഹിക്കുന്ന അസാന്മാർഗ്ഗികതയാണ് മൃഗഭോഗം.

നാല് സാക്ഷികൾ കൊണ്ട് സംഭവം സ്ഥിരീകരിക്കപ്പെടണം അല്ലെങ്കിൽ സ്വയം കുറ്റസമ്മതം നടത്തണം എന്നത് ഹദ്ദും തഅസീറും നടപ്പിലാക്കാൻ നിബന്ധനയുണ്ടെന്നു ശാഫിഈ കർമ്മ ശാസ്ത്രകാരനായ അല്ലാമാ ഇബ്‌നു ഹജർ അൽഹൈതമി രേഖപ്പെടുത്തുന്നു. (وَيُشْتَرَطُ لِلزِّنَا) وَاللِّوَاطِ وَإِتْيَانِ الْبَهِيمَةِ وَوَطْءِ الْمَيِّتَةِ (أَرْبَعَةُ رِجَالٍ) بِالنِّسْبَةِ لِلْحَدِّ أَوْ التَّعْزِيرِ لِقَوْلِهِ تَعَالَى {ثُمَّ لَمْ يَأْتُوا بِأَرْبَعَةِ شُهَدَاءَ} [النور: 4]

ഇസ്‌ലാമിലെ ലൈംഗികാനുവാദങ്ങൾ, വിലക്കുകൾ ചുരുക്കിപ്പറയാമോ?

പറയാം:

വിവാഹം ചെയ്ത ഇണകളെയും ‘വലതുകൈ ഉടമപ്പെടുത്തിയ’ ദാസികളെയും മാത്രമേ ഭോഗിക്കാൻ ഇസ്‌ലാം അനുവദിക്കുന്നുള്ളൂ. ഇവരുടെ പോലും ഗുദം ഭോഗിക്കുന്നത് ഇസ്‌ലാം കർശനമായി നിരോധിച്ചിരിക്കുന്നു. അതുപോലെ, ഇവരുടെ ആർത്തവ, പ്രസവാനന്തര നാളുകളിൽ ഭോഗിക്കുന്നതും നിരോധിച്ചിരിക്കുന്നു. സ്വയം ഭോഗം, അന്യരെ ഉപയോഗപ്പെടുത്തിയുള്ള മൈഥുനം, മൃഗഭോഗം, ശവഭോഗം, പരസ്ത്രീ/പരപുരുഷ ഭോഗം, സമയ ബന്ധിത കരാർ ഭോഗം (മുത്അ), സ്വവർഗ്ഗഭോഗം, കൂട്ടഭോഗം എന്നിവയെല്ലാം നിഷിദ്ധമാകുന്നു. ഇവയിൽ ചിലത് മറ്റു ചിലതിനേക്കാൾ ഗൗരവമേറിയ നിഷിദ്ധങ്ങളാണ്. ഇസ്‌ലാം സുരക്ഷിതമായിരിക്കണമെന്നാഗ്രഹിക്കുന്ന വിവിധ കാര്യങ്ങൾ പരിഗണിച്ച് ഇവയുടെ ശിക്ഷകൾക്കും വ്യത്യാസമുണ്ട്. ഇവയോടുള്ള ഖുർആൻ ഹദീസുകളുടെ ഗൗരവ സമീപനവും മൗനവും ശിക്ഷയുടെ ഗൗരവലാഘവങ്ങളെ ബാധിക്കുന്നു.

വിശുദ്ധ ഖുർആനിലെ സൂറ 23 സൂക്തം 5 -7 ലൈംഗിക ഭോഗത്തിനുള്ള അനുവാദം വ്യക്തമാക്കുന്നു: “തങ്ങളുടെ ഗുഹ്യ അവയവങ്ങളെ ദോഷമുക്തമായി കാത്തുസൂക്ഷിക്കുന്നവരുമത്രെ അവർ(വിജയികളായ സത്യവിശ്വാസികൾ). തങ്ങളുടെ ഇണകളുമായോ, വലതുകരം അധീനപ്പെടുത്തിയവരുമായോ ഉള്ള ലൈംഗിക ബന്ധത്തിനൊഴികെ. ഇവരെ ഭോഗിക്കുന്നത് ആക്ഷേപാർഹമല്ല. എന്നാൽ അതിനപ്പുറം ലൈംഗിക സുഖം തേടുന്നവർ, അവർ തന്നെയാണ് അതിക്രമകാരികൾ”. وَالَّذِينَ هُمْ لِفُرُوجِهِمْ حَافِظُونَ*إِلَّا عَلَى أَزْوَاجِهِمْ أوْ مَا مَلَكَتْ أَيْمَانُهُمْ فَإِنَّهُمْ غَيْرُ مَلُومِينَ*فَمَنِ ابْتَغَى وَرَاءَ ذَلِكَ فَأُولَئِكَ هُمُ الْعَادُونَ﴾[المؤمنون: 5-7]

സൂക്തത്തിൽ നൽകിയിട്ടുള്ള അനുവാദത്തിൽ, ഇണകളുടെയും ദാസികളുടെയും ഗുദം ഉൾപ്പെടില്ലെന്ന്‌, പ്രഥമ ഖുർആൻ അധ്യാപകനും വ്യാഖ്യാതാവുമായ മുഹമ്മദ് നബി വ്യക്തമാക്കിയിരിക്കുന്നു. ആർത്തവ സമയത്ത്, അവരെ പ്രയാസപ്പെടുത്തലായതിനാൽ, ലിംഗഭോഗം ഒഴിവാക്കണമെന്ന് വിശുദ്ധ ഖുർആൻ അൽബഖറ 222 ഉദ്‌ബോധനം ചെയ്തു; സഹശയനമോ മറ്റു ശരീര ഭാഗങ്ങളിലെ ആസ്വാദനമോ ഒഴിവാക്കേണ്ടതില്ലെന്ന് തിരുനബി വ്യക്തമാക്കി; ലിംഗാസ്വാദനത്തിലേക്ക് വഴുതിപ്പോകാതിരിക്കാൻ തുടമുതൽ പൊക്കിൾ വരെ ആസ്വാദനം ഒഴിവാക്കാൻ കർമ്മ ശാസ്ത്ര പണ്ഡിതർ കരുതൽ വിലക്ക് പ്രഖ്യാപിച്ചു.

വ്യഭിചാരത്തെ നിഷിദ്ധമായി പ്രഖ്യാപിക്കുന്നതും അതിനുള്ള ഭൗതിക ശിക്ഷാ നടപടികൾ വിവരിക്കുന്നതുമായ നിരവധി സൂക്തങ്ങൾ ഖുർആനിലും, അവയുടെ വിശദ വിവരങ്ങളും നടപടിക്രമങ്ങളും ഹദീസുകളിലും വന്നിട്ടുണ്ട്. വംശശുദ്ധിയും കുടുംബഭദ്രതയും കാത്തുസൂക്ഷിക്കുകയാണ് വ്യഭിചാര നിരോധനത്തിലൂടെ. പുരുഷന്മാർ തമ്മിലുള്ള ഭോഗത്തെ നിരോധിക്കുകയും, മാനവ ചരിത്രത്തിൽ ഈ സംസ്കാരശൂന്യത ഇദംപ്രഥമമായി പരസ്യമായി കാണിച്ച സദൂം പ്രദേശത്തുകാരുടെ ദുരന്തസമാപനചരിത്രം വിവരിക്കുകയും ചെയ്യുന്ന നിരവധി സൂക്തങ്ങൾ വിശുദ്ധ ഖുർആനിൽ കാണാം. സംസ്കാര ശൂന്യത, വൃത്തിഹീനത, ലക്ഷ്യം തെറ്റിയുള്ള ദുരുപയോഗവും ദുർവിനിയോഗവും തുടങ്ങിയ മാനങ്ങളാണ് ഇവിടെ പരിഗണിക്കപ്പെട്ടിട്ടുള്ളത്.

സ്ത്രീകളെ അധികാരം ഏല്‍പ്പിച്ച ഒരു ജനതയും വിജയിക്കില്ലെന്ന് പഠിപ്പിച്ചത് മുഹമ്മദ് നബിയാണ്. അതുകൊണ്ടുതന്നെ പ്രവാചകാനുചരന്മാര്‍ സ്ത്രീകളുടെ അധികാരം അംഗീകരിക്കാന്‍ തയ്യാറായിരുന്നില്ല. ജമല്‍ യുദ്ധം നയിച്ച ആഇശ പ്രവാചകപത്‌നിയായിരുന്നിട്ടുപോലും, ഒരു സ്ത്രീ ആയതിനാല്‍ അവരുടെ നേതൃത്വത്തിനോട് പ്രവാചക ശിഷ്യന്‍ അബൂബക്‌റത് മുഖം തിരിച്ചു നിന്നത്,’സ്ത്രീയെ അധികാരം ഏല്‍പ്പിച്ച ജനത വിജയിക്കില്ലെന്ന’ പ്രവാചകോപദേശം മുഖവിലക്കെടുത്തതിനാലായിരുന്നു എന്നത് അദ്ദേഹം തന്നെ പറഞ്ഞതായി ഹദീഥ് ഗ്രന്ഥങ്ങള്‍ രേഖപ്പെടുത്തിയിട്ടുണ്ട്. പ്രത്യക്ഷമായ പെണ്‍വിരുദ്ധതയാണ് പ്രവാചകന്‍ പഠിപ്പിച്ചിരുന്നതെന്ന് ഇത് വ്യക്തമാക്കുന്നുണ്ടല്ലോ ?

ഒരു ഹദീഥ് ഉദ്ദരിക്കുകയും എന്നിട്ട് തങ്ങള്‍ക്ക് എന്താണോ തോന്നുന്നത് അപ്രകാരം അതിനെ വിലയിരുത്തുകയും വിമര്‍ശനവിധേയമാക്കുകയും ചെയ്യുക എന്നത് ഇസ്‌ലാംവിദ്വേഷികളുടെ സ്ഥിരം പതിവാണ്. ആ ഹദീഥിനെ പ്രമാണമായി കാണുന്ന മുസ്‌ലിം സമൂഹം അതിനെ എപ്രകാരമാണ് വിലയിരുത്തിയതും സ്വീകരിച്ചതുമെന്നും ഹദീഥിന്റെ ആധികാരികവും പ്രാമാണികവുമായ വ്യഖ്യാനം എങ്ങനെയാണെന്നും ഇസ്‌ലാമിക പ്രമാണങ്ങളുടെ ആകത്തുക ഈ വിഷയത്തെ എങ്ങനെ വീക്ഷിക്കുന്നുവെന്നുമുള്ള വളരെ പ്രധാനപ്പെട്ട ഘടകങ്ങളെ അവഗണിക്കുക എന്നത് നിരൂപണ രംഗത്തെ സത്യസന്ധതയില്ലായ്മയെയാണ് അടിവരയിടുന്നത്. സ്ത്രീ ഭരണാധികാരിയാവുക അല്ലെങ്കില്‍ അധികാരത്തിന്റെ ഏതെങ്കിലും ഒരു പങ്ക് സ്ത്രീയെ ഏല്‍പ്പിക്കുക എന്നത് ഇസ്‌ലാമിക പ്രമാണങ്ങള്‍ക്ക് വിരുദ്ധമാണോ? അല്ല എന്നതാണ് വസ്തുത. കാരണം, തന്റെ സമൂഹത്തെ ഏറ്റവും നല്ല നിലയില്‍ നയിച്ച ഒരു സ്ത്രീയുടെ ചരിത്രം വിശുദ്ധ ക്വുര്‍ആന്‍ തന്നെ മുസ്‌ലിം സമൂഹത്തെ പഠിപ്പിച്ചിട്ടുണ്ട്; സൂറത്തു’ന്നംലി’ല്‍ ക്വുര്‍ആന്‍ വിവരിച്ച ‘സബഇ’ലെ രാജ്ഞി ബല്‍ഖീസിന്റെ ചരിത്രം. നല്ല യുക്തിയോടും വിവേകത്തോടും കൂടിയാണ് അവര്‍ ഭരണം നടത്തിയിരുന്നത്. തന്റെ തീരുമാന ശക്തിയും വിവേചനാധികാരവും ഉപയോഗിച്ച് സര്‍വനാശത്തില്‍ എത്തിപ്പെടുമായിരുന്ന ഒരു യുദ്ധത്തില്‍ നിന്നും അവര്‍ സ്വന്തം ജനതയെയും രാഷ്ട്രത്തെയും രക്ഷിച്ച ചരിത്രം വിശുദ്ധ ക്വുര്‍ആന്‍ പരാമര്‍ശിച്ചിട്ടുണ്ട്. നയതന്ത്രപരമായ പക്വതയുള്ള ഒരു ഭരണാധികാരിയായാണ് ബല്‍ഖീസ് രാജ്ഞിയെ ക്വുര്‍ആന്‍ അടയാളപ്പെടുത്തിയത്. തന്റെ അധികാര പരിധിയില്‍ കീഴൊതുങ്ങുവാനുള്ള സര്‍വാധിപതിയായ സുലൈമാന്റെ(അ) ആഹ്വാനദൂത് എത്തിയപ്പോള്‍ അവര്‍ തന്റെ കൂടിയാലോചനാ സദസ്സ് വിളിച്ചുകൂട്ടി. പ്രസ്തുത സംഭവം ക്വുര്‍ആന്‍ ഇപ്രകാരം വിവരിച്ചു:

”അവള്‍ പറഞ്ഞു: ഹേ, പ്രമുഖന്മാരേ, എനിക്കിതാ മാന്യമായ ഒരെഴുത്ത് നല്‍കപ്പെട്ടിരിക്കുന്നു” ”അത് സുലൈമാന്റെ പക്കല്‍ നിന്നുള്ളതാണ്. ആ കത്ത് ഇപ്രകാരമത്രെ: ‘പരമകാരുണികനും കരുണാനിധിയുമായ അല്ലാഹുവിന്റെ നാമത്തില്‍’ ”എനിക്കെതിരില്‍ നിങ്ങള്‍ അഹങ്കാരം കാണിക്കാതിരിക്കുകയും, കീഴൊതുങ്ങിയവരായിക്കൊണ്ട് നിങ്ങള്‍ എന്റെ അടുത്ത് വരികയും ചെയ്യുക” ”അവള്‍ പറഞ്ഞു: ഹേ, പ്രമുഖന്മാരേ, എന്റെ ഈ കാര്യത്തില്‍ നിങ്ങള്‍ എനിക്ക് നിര്‍ദ്ദേശം നല്‍കുക. നിങ്ങള്‍ എന്റെ അടുക്കല്‍ സന്നിഹിതരായിട്ടല്ലാതെ യാതൊരു കാര്യവും ഖണ്ഡിതമായി തീരുമാനിക്കുന്നവളല്ല ഞാന്‍” ”അവര്‍ പറഞ്ഞു: ‘നാം ശക്തിയുള്ളവരും ഉഗ്രമായ സമരവീര്യമുള്ളവരുമാണ്. അധികാരം അങ്ങയ്ക്കാണല്ലോ, അതിനാല്‍ എന്താണ് കല്‍പിച്ചരുളേണ്ടതെന്ന് ആലോചിച്ചു നോക്കുക” ”അവള്‍ പറഞ്ഞു: തീര്‍ച്ചയായും രാജാക്കന്മാര്‍ ഒരു നാട്ടില്‍ കടന്നാല്‍ അവര്‍ അവിടെ നാശമുണ്ടാക്കുകയും, അവിടത്തുകാരിലെ പ്രതാപികളെ നിന്ദ്യരാക്കുകയും ചെയ്യുന്നതാണ്. അപ്രകാരമാകുന്നു അവര്‍ ചെയ്തുകൊണ്ടിരിക്കുന്നത്” ”ഞാന്‍ അവര്‍ക്ക് ഒരു പാരിതോഷികം കൊടുത്തയച്ചിട്ട് എന്തൊരു വിവരവും കൊണ്ടാണ് ദൂതന്മാര്‍ മടങ്ങി വരുന്നതെന്ന് നോക്കാന്‍ പോകുകയാണ്.” (ക്വുര്‍ആന്‍ 27: 29-35)

സുലൈമാന്‍ എന്ന സര്‍വാധിപതിക്കെതിരെ ഒരു യുദ്ധമുണ്ടായാല്‍ തന്റെ സമൂഹത്തിന്റെ സര്‍വനാശമായിരിക്കും പരിണിതഫലം എന്നു ദീര്‍ഘദര്‍ശനം ചെയ്ത അവള്‍, തനിക്കു യുദ്ധ പിന്തുണ പ്രഖ്യാപിച്ച പ്രമുഖന്മാരുടെ തീരുമാനത്തെ അവഗണിച്ചുകൊണ്ട് യുദ്ധം ഒവിവാക്കുക എന്ന വിവേക പൂര്‍ണ്ണമായ നിലപാട് സ്വീകരിക്കുകയും സ്വന്തം രാഷ്ട്രത്തെയും ജനങ്ങളെയും നാശത്തില്‍ നിന്നും രക്ഷിക്കുകയുമാണുണ്ടായത്. അവസാനം അവര്‍ സുലൈമാന്റെ(അ) അരികിലെത്തി സത്യവിശ്വാസം സ്വീകരിക്കുകയും ചെയ്തതായി ക്വുര്‍ആന്‍ പഠിപ്പിക്കുന്നുണ്ട്. (27:44)

ഭരണാധികാരിയായ ഒരു സ്ത്രീയുടെ വിവേകപൂര്‍ണ്ണമായ ഭരണത്തെ എടുത്തു പരാമര്‍ശിച്ച ക്വുര്‍ആന്‍, ഒരിക്കലും സ്ത്രീ ഭരണാധികാരം കൈയ്യാളാന്‍ പാടില്ലെന്ന് പറഞ്ഞില്ല. മാത്രമല്ല ബല്‍ഖീസ് രാജ്ഞിയുടെ സത്യവിശ്വാസപൂര്‍വകാലത്തെ തെറ്റുകളെ പരാമര്‍ശിച്ച ക്വുര്‍ആന്‍ (27: 24) ഭരണാധികാരം അവര്‍ കയ്യാളിയത് ഒരു തെറ്റായി സൂചിപ്പിച്ചുമില്ല.

എങ്കില്‍ പിന്നെ ‘തങ്ങളുടെ ഭരണനേതൃത്വം സ്ത്രീയെ ഏല്‍പ്പിച്ച ജനത വിജയിക്കുകയില്ല’ എന്ന ഹദീഥിനെ എങ്ങനെയാണ് മനസ്സിലാക്കേണ്ടത്?

ഒന്ന്, പ്രവാചക കാലഘട്ടത്തില്‍ നിലനിന്നിരുന്ന വമ്പിച്ച രണ്ട് സാമ്രാജ്യത്വ ശക്തികളായിരുന്നു റോമും പേര്‍ഷ്യയും. സാമ്രാജ്യത്വ വികസനത്തിനായി പരസ്പരം ശക്തമായ പോരാട്ടം ഇരു സാമ്രാജ്യത്വങ്ങളും നടത്തിയിരുന്ന ഘട്ടത്തിലാണ് പേര്‍ഷ്യന്‍ ചക്രവര്‍ത്തി കിസ്റാ മരണപ്പെടുന്നത്. തുടര്‍ന്ന് പേര്‍ഷ്യന്‍ സാമ്രാജ്യത്വത്തിന്റെ അധികാരത്തിനുവേണ്ടി രാജകുടുംബങ്ങള്‍ക്കിടയില്‍ ശക്തമായ അധികാര വടംവലി നടക്കുകയുണ്ടായി. രാജകുടുംബങ്ങളിലെ പലരും കൊലചെയ്യപ്പെട്ടു. അവസാനം ചക്രവര്‍ത്തിയുടെ മകളെ സമൂഹം ഭരണനേതൃത്വം ഏല്‍പ്പിച്ചു. ഈ സന്ദര്‍ഭത്തിലാണ് നബി (സ) ‘തങ്ങളുടെ ഭരണനേതൃത്വം സ്ത്രീയെ ഏല്‍പ്പിച്ച ജനത വിജയിക്കുകയില്ല’ (ബുഖാരി) എന്നു പറഞ്ഞത്. പ്രവാചകന്‍ (സ) ഇതു പറഞ്ഞ് ഏറെ താമസിയാതെ തന്നെ പേര്‍ഷ്യന്‍ സാമ്രാജ്യം തകരുകയാണുണ്ടായത്. അതുകൊണ്ടുതന്നെ ഇത് പേര്‍ഷ്യന്‍ സാമ്രാജ്യത്വത്തെ സംബന്ധിച്ച ഒരു പ്രവചനമാണോ – അഥവാ ഹദീഥ് പശ്ചാത്തലത്തോട് മാത്രം ബന്ധപ്പെട്ടതാണോ – അതല്ല ഹദീഥിലെ ആശയത്തെ സാമാന്യവല്‍ക്കരിക്കാമോ എന്ന കാര്യത്തില്‍ പണ്ഡിതന്മാര്‍ക്കിടയില്‍ അഭിപ്രായാന്തരമുണ്ട്.

മുഹമ്മദ് അമാനി മൗലവി എഴുതുന്നു: ”പേര്‍ഷ്യക്കാരുടെ ഭരണം ഒരു രാജ്ഞിയുടെ കൈയ്യിലായിരുന്ന ഘട്ടത്തിലാണ് നബി (സ) ഇതു പറഞ്ഞത്. ഈ ഹദീഥില്‍ സ്ത്രീനേതൃത്വത്തെ ആക്ഷേപിച്ചിരിക്കുന്നത് പേര്‍ഷ്യക്കാരെ മാത്രം ബാധിക്കുന്ന ഒരു സംഗതിയാണോ, അതോ സ്ത്രീനേതൃത്വത്തെ തത്ത്വത്തില്‍ എതിര്‍ത്തിരിക്കുകയാണോ എന്ന കാര്യത്തില്‍ ഭിന്നാഭിപ്രായങ്ങളുണ്ട്. ഏതായാലും പേര്‍ഷ്യക്കാരുടെ കാര്യത്തില്‍ നബി (സ) പറഞ്ഞത് താമസിയാതെ തന്നെ പുലര്‍ന്നുവെന്നത് ഒരു ചരിത്ര യാഥാര്‍ത്ഥ്യമാണ്” (ഇസ്‌ലാമിക ജീവിതം, പേജ്: 163,164)

ശറഫുദ്ദീന്‍ ത്വീബി പറഞ്ഞു: ”പേര്‍ഷ്യക്കാര്‍ രാഷ്ട്രീയ വിജയം കൈവരിക്കില്ല എന്ന് ഉറപ്പ് നല്‍കിക്കൊണ്ടുള്ള പ്രവചനമാണിത്. അറബികള്‍ക്ക് (അവരുടെ മേല്‍) കൈവരുന്ന വിജയത്തെ സംബന്ധിച്ച സൂചന ഈ പ്രസ്താവനയില്‍ ഉണ്ട്. അപ്പോള്‍ ഈ പ്രസ്താവന മുഅ്ജിസത്ത് (പ്രവാചകത്വത്തിനുള്ള അമാനുഷികമായ തെളിവ്) ആകുന്നു” (ഫൈദുല്‍ കദീര്‍: 5/303)

‘ഹദ്യുല്‍ ഇസ്‌ലാം’ എന്ന ഗ്രന്ഥത്തില്‍ ഡോ. യൂസുഫുല്‍ ഖറദാവി പറഞ്ഞു: ”ഈ ഹദീഥിന്റെ ആശയം സാമാന്യവല്‍ക്കരിക്കാമോ അതോ അതിന്റെ പശ്ചാത്തലത്തോട് മാത്രം ബന്ധപ്പെട്ടതാണോ? പേര്‍ഷ്യക്കാര്‍ വിജയിക്കാന്‍ പോകുന്നില്ല എന്ന് തിരുമേനി പറഞ്ഞത്, സമുദായത്തില്‍ അവളേക്കാള്‍ ആയിരം മടങ്ങ് പ്രാപ്തരും ശ്രേഷ്ഠരുമായ ആളുകളുണ്ടായിരുന്നിട്ടും ചക്രവര്‍ത്തിയുടെ മകളെ പാരമ്പര്യമായി വാഴിച്ചു എന്ന അര്‍ത്ഥത്തിലാണ്. അത്തരക്കാര്‍ വിജയിക്കുകയില്ല.”

ഈജിപ്റ്റിലെ പണ്ഡിത സഭയുടെ ഫത്‌വ ബോര്‍ഡ് ‘ദാറുല്‍ ഇഫ്താഉല്‍ മിസ്രിയ്യ’ പറയുന്നു: ”കിസ്റാ ചക്രവര്‍ത്തിക്ക് പ്രവാചകന്‍ (സ) കത്തയച്ചപ്പോള്‍ അയാള്‍ അത് പിച്ചിചീന്തുകയാണ് ചെയ്തത്. അയാള്‍ തന്റെ കത്ത് പിച്ചിചീന്തിയതുപോലെ അയാളുടെ അധികാരവും ചിന്നഭിന്നമാക്കപ്പെടാന്‍ പ്രവാചകന്‍ (സ) പ്രാര്‍ത്ഥിച്ചു. അപ്പോള്‍ പ്രവാചകന്റെ പ്രാര്‍ത്ഥനക്കുത്തരമെന്നോണം അല്ലാഹു അയാളുടെ മകനെ അയാള്‍ക്കെതിരെ തിരിച്ചു. മകന്‍ പിതാവിനെ വധിച്ചു. തുടര്‍ന്ന് സഹോദരങ്ങളേയും. അങ്ങനെ പ്രവാചകന്‍ പ്രാര്‍ത്ഥിച്ചതുപോലെ ഭരണകൂടം ചിന്നഭിന്നമാക്കപ്പെട്ടു. ഒരു സ്ത്രീയെ ചക്രവര്‍ത്തി ആക്കുന്നതിലേക്ക് കാര്യം ചെന്നെത്തി. അത് ഭരണകൂടത്തിന്റെ ഉന്മൂല നാശത്തിന് കാരണമാവുകയും ചെയ്തു. രാജകുടുംബത്തിലെ പുരുഷന്മാര്‍ അധികാര വടംവലിയില്‍ മുഴുകിയതിനാലും പരസ്പരം കൊന്നൊടുക്കി കൊണ്ടിരുന്നതിനാലും ഒരു സ്ത്രീയെ ചക്രവര്‍ത്തിയാക്കി എന്നറിഞ്ഞപ്പോള്‍ അത് അവരുടെ ഭരണകൂടത്തിന്റെ ഉന്മൂലനാശത്തിന്റെ അടയാളമാണെന്ന് പ്രവാചകന്‍ (സ) അറിയിച്ചു. ലോകത്ത് ഏത് സ്ത്രീയും വിജയം കൈവരിക്കില്ല എന്നല്ല പ്രവാചന്‍ അറിയിച്ചത്. പ്രത്യേക വ്യക്തികളെ സംബന്ധിച്ച പ്രസ്താവന പൊതുവല്‍ക്കരിക്കാന്‍ പാടില്ല എന്നത് ഇസ്‌ലാമിലെ അടിസ്ഥാന തത്ത്വങ്ങളെ സംബന്ധിച്ച വിജ്ഞാന ശാഖയിലെ ഒരു അംഗീകൃത തത്ത്വമാണ്. ഇമാം ശാഫിഈ പറഞ്ഞതായി ഉദ്ധരിക്കപ്പെടുന്നു.”പ്രത്യേക സാഹചര്യങ്ങളുമായി ബന്ധപ്പെട്ട വിഷയങ്ങളില്‍ (പ്രത്യേകവും പൊതുവുമായ ഉദ്ദേശതലങ്ങളില്‍) ഏതും ഉദ്ദേശിക്കപ്പെട്ടതായിരിക്കാന്‍ സാധ്യത നിലനില്‍ക്കുന്നുവെങ്കില്‍ അതുകൊണ്ട് തെളിവു പിടിക്കാന്‍ കഴിയില്ല.” ചര്‍ച്ചാ വിഷയകമായ ഹദീഥ് ഒരു വ്യക്തിയുടെ വിഷയത്തില്‍ ആയതുകൊണ്ട് തന്നെ അതിനെ പൊതുവായ തെളിവായി സ്വീകരിക്കല്‍ ശരിയാവുകയില്ല; അത് പൊതുവായ വിധിയാണെന്ന് സൂചിപ്പിക്കുന്ന മറ്റു തെളിവുകള്‍ ഉണ്ടായാല്‍ ഒഴികെ.” (htthp://www.fatawa.com/view/15196)

ഒരു പ്രത്യേക വ്യക്തിയുടെ വിഷയത്തിലൊ പ്രത്യേക അവസ്ഥയിലൊ പ്രമാണങ്ങലില്‍ വന്ന വിധിയെ പൊതുവായി എല്ലാവര്‍ക്കുമുള്ള തെളിവായി സ്വീകരിക്കല്‍ അനുവദനീയമല്ല; അത് പൊതുവായ വിധിയാണെന്ന് സൂചിപ്പിക്കുന്ന മറ്റു തെളിവുകള്‍ ഉണ്ടായാല്‍ ഒഴികെ. ‘വകാഇഉല്‍ അഅ്യാന്‍’ (وقا ئع الأعيان) എന്നാണ് ഇത്തരം കേസുകളുടെ ഇസ്‌ലാമിക കര്‍മ്മശാസ്ത്രത്തിലെ സാങ്കേതിക നാമം. ‘വകാഇഉല്‍ അഅ്യാന്‍’ ആയ വിഷയങ്ങളെ വ്യവച്ഛേദിക്കുന്നതില്‍ – ചില അഭിപ്രായ വ്യത്യാസങ്ങള്‍ നിലനില്‍ക്കുന്നതോടൊപ്പം തന്നെ, കര്‍മ്മശാസ്ത്രവിധി നിര്‍ധാരണത്തിന്റെ അടിസ്ഥാന തത്ത്വങ്ങളില്‍ സര്‍വാംഗീകൃതമായ ഒരു തത്ത്വമാണിത്. (അല്‍ കവാഇദുല്‍ ഹിസ് നി: 3/78, വകാഇഉല്‍ അഅ്‌യാന്‍ ഫില്‍ ഇബാദത്ത്: ദിറാസ ഫിക്ഹിയ്യ: 1/4, കള്വായ അല്‍ അഅ്‌യാന്‍: ദിറാസ ഉസൂലിയ തത്ബീകിയ്യ : 16)

ഈ വിഷയകമായി ജോര്‍ദാനിലെ ഫത്‌വ സമിധി നടത്തിയ പഠനവും ഇവിടെ ഉദ്ധരിക്കുന്നത് പ്രസക്തമായിരിക്കും: ”നബി (സ) അബൂദർറിനോട്, അദ്ദേഹം അധികാരം ചോദിച്ച സന്ദര്‍ഭത്തില്‍ ഇപ്രകാരം പറയുകയുണ്ടായി ‘അബൂദറെ തീര്‍ച്ചയായും താങ്കള്‍ ഒരു ദുര്‍ബല പ്രകൃതിയിലുള്ള മനുഷ്യനാണ്. അധികാരം എന്നു പറയുന്നതാകട്ടെ അമാനത്താണ്. അത് പരലോകത്ത് നിന്ദ്യതയും ഖേദവുമായി പരിണമിക്കുന്നതാണ്; അതിന്റെ അവകാശങ്ങള്‍ പൂര്‍ണമായും വകവെച്ചു കൊടുത്തു കൊണ്ട് അതിനെ സ്വീകരിച്ച ആളുകള്‍ ഒഴികെ, ഒരാളുടെ മേല്‍ ആ അധികാരം കിട്ടികഴിഞ്ഞാല്‍ എന്തൊക്കെ ബാധ്യതകളുണ്ടോ അതു നിര്‍വഹിക്കുകയും ചെയ്തവനൊഴികെ’ (മുസ്‌ലിം). ഈ സമുദായത്തില്‍ നിന്ന് സ്ത്രീയാകട്ടെ പുരുഷനാകട്ടെ പ്രവാചകന്‍ (സ) പറഞ്ഞ നിബന്ധനകളും ഘടകങ്ങളും ആരിലാണോ പൂര്‍ത്തിയാക്കപ്പെടുന്നത് എന്നുള്ളതില്‍ വ്യത്യാസമില്ല. കാരണം പ്രവാചകന്‍ (സ) ഇങ്ങനെയും പറഞ്ഞിട്ടുണ്ട്. ‘സ്ത്രീകള്‍ പുരുഷന്മാരുടെ കൂടെപിറപ്പുകളാണ്’ (അബൂദാവൂദ്). സ്ത്രീകളുമായും കുടുംബവുമായുമൊക്കെ ബന്ധപ്പെട്ട വ്യവസ്ഥകളുടെയും നിയമങ്ങളുടെയും കാര്യത്തില്‍ സമത്വവും നേരെചൊവ്വെയുള്ള നിലപാടുകളും ഭരണകൂടത്തിനു മുമ്പില്‍ എത്തണമെങ്കില്‍ യുക്തിദീക്ഷിതമായി ചിന്തിക്കുന്ന സ്ത്രീയുടെ അഭിപ്രായത്തിന് വളരെ അധികം പ്രാധാന്യമുണ്ട്. ‘തങ്ങളുടെ ഭരണനേതൃത്വം ഒരു സ്ത്രീയെ ഏല്പിച്ച ജനത വിജയിക്കുകയില്ല’ (ബുഖാരി) എന്ന ഹദീഥില്‍ അത് ഉള്‍പ്പെടുകയില്ല. കാരണം ഈ ഹദീഥ് അതിന്റെ പ്രത്യേകമായ സാഹചര്യവും അതുപോലെ തന്നെ സന്ദര്‍ഭത്തില്‍ നിന്നും മാത്രമേ മനസ്സിലാക്കാന്‍ പാടുള്ളൂ. പൊതുകാര്യങ്ങളിലായാല്‍ പോലും വളരെ പ്രധാനപ്പെട്ട കാര്യങ്ങള്‍ തന്നെ വിശ്വാസികളുടെ മാതാക്കളായ പ്രവാചക പത്നിമാരോട് പ്രവാചകന്‍ കൂടിയാലോചന നടത്താറുണ്ടെന്നത് സ്ഥാപിതമായ കാര്യമാണ്.” (ലജനത്തുല്‍ ഇഫ്ത്വാഅ് അല്‍ ഉര്‍ദുനിയ)

‘തങ്ങളുടെ ഭരണനേതൃത്വം ഒരു സ്ത്രീയെ ഏല്‍പ്പിച്ച ജനത വിജയിക്കുകയില്ല’ എന്ന ഹദീഥ് സാമാന്യവല്‍ക്കരിക്കേണ്ടതില്ലെന്നും അത് പ്രത്യേകമായ സാഹചര്യവും സന്ദര്‍ഭവും കൊണ്ട് വിശദീകരിക്കപ്പെടേണ്ടതാണെന്ന വീക്ഷണം തന്നെയാണ് യു എ ഇയിലേയും കുവൈത്തിലെയും പണ്ഡിത സഭകളുടെയും നിലപാട് (https:www.aljarida.com/articles/146360661264961300/). യൂറോപ്യന്‍ രാജ്യങ്ങളിലെ ഇസ്‌ലാമിക സംഘടനകളുടെ യൂണിയനു കീഴില്‍ നിലനില്‍ക്കുന്ന ഫത്‌വ ബോര്‍ഡിന്റെയും (അല്‍ മജ്‌ലിസുല്‍ ഔറുബി ലില്‍ ഇഫ്താഇ വല്‍ ബുഹൂസ്) പഠനങ്ങളും സമാനമാണ്. (tthps://bit.ly/3fHBRwD)

രണ്ട്, കൂര്‍മബുദ്ധി, പ്രായപൂര്‍ത്തി, (അടിമയല്ലാത്ത) സ്വതന്ത്ര്യനാവുക, പുരുഷനാവുക, ധീരനും ശൂരനുമാവുക, കല്‍പ്പനാധികാരശേഷി, കാഴ്ച, കേള്‍വി, സംസാരം എന്നിങ്ങനെയുള്ള അവയവങ്ങളുടെ ശേഷി, ജനസമ്മതി, തന്റേയും ഗോത്രത്തിന്റേയും ഔന്നത്യത്തെ സംബന്ധിച്ച ജനങ്ങളുടെ അംഗീകാരം, പ്രജകളുടെ അനുസരണ സമ്മതി, പ്രജകളുടെ പിന്തുണയും വിശ്വാസവും എന്നിങ്ങനെ പൗരാണിക കാലഘട്ടത്തിലെ ജനങ്ങള്‍ ഒരു ഭരണാധികാരിയില്‍ സമ്മേളിക്കേണ്ട നിബന്ധനകളായി കരുതിപ്പോന്നിരുന്ന പല ഗുണവിശേഷങ്ങളുമുണ്ടായിരുന്നു. ജാതി-മത-രാഷ്ട്ര-ഭാഷാ വ്യത്യാസമില്ലാത്ത അന്നത്തെ ജനങ്ങള്‍ ഈ നിബന്ധനകളെ ഭരണസാരഥ്യത്തിന്റേയും നേതൃത്വത്തിന്റെയും നിര്‍ബന്ധ ഘടകങ്ങളായി മനസ്സിലാക്കി പോന്നു. അതുകൊണ്ടുതന്നെ ഈ നിബന്ധനകള്‍ ഉള്‍ക്കൊള്ളുന്ന ഭരണാധികാരികള്‍ അനുസരിക്കപ്പെട്ടു. ഭരണത്തിന്റെ വിജയം ഭരണീയരുടെ അനുസരണ മനോഭാവവും സമ്മതിയുമാണല്ലോ. ഈ നിബന്ധനകള്‍ പൂര്‍ത്തിയാക്കപ്പെടാതെ വരുമ്പോള്‍ ഭരണീയരില്‍ നിന്ന് അനുസരണക്കേടുകളും, മാനസീകവും പ്രാവര്‍ത്തികവുമായ വിയോജിപ്പുകളും സംഭവിക്കും. പ്രതികരിക്കാന്‍ തീരെ ശേഷിയില്ലാത്ത പ്രജകളാണെങ്കില്‍ വെറുപ്പോടെ ഭരണാധികാരിയെ അംഗീകരിച്ചതായി നടിക്കുന്നു. ഇത്തരം ഒരു ഭരണ നേതൃത്വത്തിന് അസന്തുഷ്ട സമൂഹത്തില്‍ അല്‍പ്പകാലം മാത്രമേ നിലനില്‍പ്പ് ഉണ്ടാകൂ എന്നത് ആ കാലഘട്ടത്തിലെ ചരിത്ര വസ്തുതയാണ്. ഉടനെ ഭരണനേതൃത്വത്തിനെതിരെ പ്രതിലോമ വിപ്ലവങ്ങളും, അധികാര വടംവലിയും ഉടലെടുക്കുക തന്നെ ചെയ്യും. പേര്‍ഷ്യക്കാര്‍ ഒരു സ്ത്രീയെ പരമാധികാരിയായി അവരോധിച്ചു എന്നറിഞ്ഞപ്പോള്‍ ‘ഒരു സ്ത്രീയെ തങ്ങളുടെ ഭരണനേതൃത്വം ഏല്‍പ്പിച്ച ഒരു ജനത വിജയിക്കില്ല’ എന്ന് പ്രവാചകന്‍ (സ) പറഞ്ഞത് ഇക്കാരണത്താലാണ് എന്ന് ശാഹ് വലിയുല്ലാഹി ദ്ദഹ്ലവി തന്റെ ‘ഹുജ്ജത്തുല്ലാഹില്‍ ബാലിഗ’: 2/131’യില്‍ വ്യക്തമാക്കുന്നുണ്ട്.

അതായത് ഒരു സ്ത്രീ പരമാധികാരം കൈയ്യാളല്‍ ആത്മീയമായ പരാജയമാണെന്നോ, മതപരമായി നിഷിദ്ധമാണെന്നോ ഒന്നും പ്രവാചകന്‍ (സ) ഉദ്ദേശിച്ചിട്ടില്ല. മറിച്ച് ആ കാലഘട്ടത്തിലെ രാഷ്ട്രീയമാണ് പ്രവാചകന്‍ (സ) ചര്‍ച്ച ചെയ്യുന്നത്. പ്രവാചകന്റെ(സ) ഈ രാഷ്ട്രീയ പ്രവചനം പുലരുകയും ചെയ്തു. പേര്‍ഷ്യക്കാരുടെ ഈ വനിതാ ഭരണാധികാരിക്കെതിരെ അവരുടെ ഉള്ളില്‍ നിന്ന് തന്നെ പ്രതിവിപ്ലവങ്ങള്‍ ഉടലെടുക്കുകയും സാമ്രാജ്യം ക്ഷയിക്കുകയും നേതൃത്വം ദുര്‍ബലമാവുകയും ചെയ്തു. മുസ്‌ലിംകള്‍ പിന്നീട് പേര്‍ഷ്യന്‍ സാമ്രാജ്യത്തിനു മേല്‍ അധികാരം കൈയ്യാളുകയും ചെയ്തു.

”ഭരണാധികാരികള്‍ കുറൈശികളില്‍ നിന്നാവണം” (മുസ്‌നദ് അഹ്‌മദ്: 3/129) എന്ന് പ്രവാചകന്‍ (സ) പറഞ്ഞതും ഇതേ രാഷ്ട്രീയ കാരണത്താലാണ്. അറബികളെ ഭരിക്കാന്‍ കുറൈശി ഗോത്രത്തിനേ കഴിയൂ എന്നതാണ് അന്നത്തെ അറേബ്യയുടെ രാഷ്ട്രീയം. അത് മതപരമായ ഒരു നിര്‍ബന്ധ ബാധ്യതയായിരുന്നെങ്കില്‍ അക്കാലം തൊട്ട് ഇക്കാലം വരെ ഇസ്‌ലാമിക ലോകത്ത് കുറൈശികളല്ലാത്ത അനേകായിരം ഭരണാധികാരികള്‍ ഉണ്ടാകുമായിരുന്നോ?! അന്ന് അറബികള്‍ക്കിടയില്‍ ഭാഷകൊണ്ടും അംഗശക്തികൊണ്ടും ധനംകൊണ്ടും മതനിഷ്ഠകൊണ്ടും ഭക്തി, ധീരത, സംസ്‌കാരം, പൈതൃകം എന്നിവ കൊണ്ടെല്ലാം പ്രബലര്‍ കുറൈശികള്‍ ആയിരുന്നു. അപ്പോള്‍ അന്നത്തെ അറബികളെ ഭരിക്കാന്‍ മറ്റൊരു ഗോത്രക്കാരെ തിരഞ്ഞെടുക്കപ്പെട്ടാല്‍ ജനസമ്മതിയും അനുസരണവും ഭരണനേതൃത്വത്തില്‍ കേന്ദ്രീകരിക്കപ്പെടില്ല. നേതൃത്വത്തെ വെല്ലുവിളിക്കുന്നവര്‍ ഉയര്‍ന്നുവരാനും ഭരണം സ്തംഭിക്കാനുമാണ് സാധ്യത. ഈ രാഷ്ട്രീയമാണ് ഹദീഥിലെ ചര്‍ച്ച.

മൂന്ന്, ഉപര്യുക്ത ഹദീഥ് കേവലം അതിന്റെ പ്രത്യേകമായ പശ്ചാതലത്തില്‍ നിന്നുകൊണ്ടാണ് മനസ്സിലാക്കേണ്ടത് എന്നു നിഷ്‌കര്‍ഷിച്ച പണ്ഡിത വീക്ഷണങ്ങളെയാണ് നാം ഇതുവരെ വിലയിരുത്തിയതെങ്കില്‍ ഇനി നമുക്കു പരിശോധിക്കുവാനുള്ളത്, പ്രസ്തുത ഹദീഥിലെ ആശയത്തെ സാമാന്യവല്‍ക്കരിച്ചുകൊണ്ട് മനസ്സിലാക്കണമെന്നു നിഷ്‌കര്‍ഷിച്ചാല്‍ തന്നെ ഹദീഥ് സ്ത്രീ വിരുദ്ധമാകുന്നില്ല എന്ന വസ്തുതയെയാണ്. ‘തങ്ങളുടെ ഭരണനേതൃത്വം ഒരു സ്ത്രീയെ ഏല്‍പ്പിച്ച ജനത വിജയിക്കുകയില്ല’ എന്ന ഹദീഥ് പേര്‍ഷ്യക്കാരുടെ കാര്യത്തില്‍ പറയപ്പെട്ടതാണെങ്കിലും അതിലെ ആശയം സാമാന്യവല്‍ക്കരിച്ചു മനസ്സിലാക്കേണ്ടതുണ്ട് എന്നു നിഷ്‌കര്‍ഷിച്ചാല്‍ തന്നെ ഹദീഥില്‍ നിരുത്സാഹപ്പെടുത്തപ്പെട്ട പെണ്ണധികാരം ‘പരമാധികാര’ത്തിന്റെ വിഷയത്തില്‍ മാത്രമാണ്. അഥവാ ‘സര്‍വ്വാധിപത്യം’ ഒരു പെണ്ണില്‍ അര്‍പ്പിതമാകുന്നതിനെയാണ് ഹദീഥ് പ്രശ്നവല്‍കരിച്ചിരിക്കുന്നതെന്നര്‍ഥം.

ഇമാം ഇബ്‌നു ഹസം പറഞ്ഞു: ‘സ്ത്രീ ന്യായാധിപത്യം ഏറ്റെടുക്കല്‍ അനുവദനീയമാണ്. അതാണ് ഇമാം അബൂഹനീഫയുടേയും അഭിപ്രായം. ഖലീഫ ഉമര്‍ ബിന്‍ ഖത്താബ് തന്റെ ഗോത്രത്തില്‍പ്പെട്ട ശിഫാഅ് എന്ന സ്ത്രീയെ മാര്‍ക്കറ്റിലെ (നിയമപാലനവുമായി ബന്ധപ്പെട്ട ഉത്തരവാദിത്തം) ഏല്‍പ്പിച്ചതായി നിവേദനം ചെയ്യപ്പെട്ടിട്ടുണ്ടല്ലോ. ‘തങ്ങളുടെ കാര്യം സ്ത്രീയെ ഏല്‍പ്പിച്ച ഒരു സമൂഹം വിജയിക്കില്ല’ എന്നു പ്രവാചകന്‍ (സ) പറഞ്ഞിട്ടുണ്ടല്ലോ എന്ന് ആരെങ്കിലും ചോദിച്ചാല്‍ നാം ഇപ്രകാരം മറുപടി നല്‍കും: പ്രവാചകന്‍ (സ) അതുകൊണ്ടുദ്ദേശിച്ചത് ഖിലാഫത്ത് ഏല്‍പ്പിക്കുന്നതിനെ സംബന്ധിച്ചാണ്. കാരണം, ഭര്‍ത്താവിന്റെ ധനം, ഭവനം, സ്വകുടുംബം തുടങ്ങി മറ്റു പല അധികാര അവകാശങ്ങളും പ്രവാചകന്‍ (സ) സ്ത്രീക്കു വക വെച്ചുകൊടുത്തിട്ടുണ്ട് എന്നതാണ് അതിനുള്ള തെളിവ്. സ്ത്രീക്ക് വക്കീലയും (കാര്യസ്ഥത, പ്രതിനിധി) വസ്വിയ്യയും (ഒരാളുടെ മരണാനന്തരം അയാളുടെ കാര്യങ്ങളുടെ കൈകാര്യകര്‍ത്താവായി അയാള്‍ തന്നെ – ജീവിത കാലത്ത് നിശ്ചയിക്കുന്ന പ്രതിനിധി) ആകല്‍ അനുവദനീയമാണെന്ന് മാലികി മദ്ഹബുകാര്‍ അഭിപ്രായപ്പെടുന്നു. പരമാധികാരം അല്ലാത്ത (അതിനു താഴെ വരുന്ന) പല കാര്യങ്ങളുടേയും അധികാരം സ്ത്രീ ഏറ്റെടുക്കുന്നത് ഇസ്‌ലാമിക പ്രമാണങ്ങള്‍ തടയുന്നില്ല…” അല്ലാഹു നിങ്ങളോടിതാ കല്‍പ്പിക്കുന്നു: നിങ്ങളെ വിശ്വസിച്ചേല്‍പ്പിച്ച വസ്തുക്കള്‍ അവയുടെ അവകാശികളെ തിരിച്ചേല്‍പ്പിക്കുക. ജനങ്ങള്‍ക്കിടയില്‍ തീര്‍പ്പ് കല്‍പ്പിക്കുകയാണെങ്കില്‍ നീതിപൂര്‍വ്വം വിധി നടത്തുക” (ക്വുര്‍ആന്‍ 4:58) ക്വുര്‍ആനിലെ ഈ വചനം അഭിമുഖീകരിക്കുന്നത് പുരുഷനേയും സ്ത്രീയേയും സ്വതന്ത്രനേയും അടിമയേയും ഉള്‍കൊള്ളുന്ന നിലക്ക് പൊതുവായിട്ടാണ്. മതം മൊത്തത്തില്‍ സ്ത്രീക്കും പുരുഷനും ഒന്നാണ്; സ്ത്രീയേയും പുരുഷനേയും അവരുടെ (പ്രകൃതി സഹജമായ വ്യത്യാസങ്ങളെ പരിഗണിച്ച്) വ്യത്യാസമുള്ള വിധികള്‍ വല്ലതും പ്രമാണങ്ങളില്‍ വന്നിട്ടുള്ള ചില വിഷയങ്ങളൊഴിച്ച്” (അല്‍ മുഹല്ലാ: 9/429)

ഇമാം ത്വബ്‌രി പറഞ്ഞു: ”സ്ത്രീയെ ഏതു കാര്യത്തിലും, നിരുപാധികം വിധി കര്‍തൃത്വം/ന്യായാധിപത്യം ഏല്‍പ്പിക്കാന്‍ അനുവദനീയമാണ്” (ബിദായത്തുല്‍ മുജ്തഹിദ്: 3/445)

ഇമാം ഹസുനല്‍ ബസ്വരി (ജനനം: ഹിജ്റ 21), മാലികീ മദ്ഹബിലെ ഇമാം ഇബ്നുല്‍ ക്വാസിം (ജനനം: ഹ്ജ്റ 132) എന്നിവരും സ്ത്രീകളെ വിധി കര്‍തൃത്വം/ന്യായാധിപത്യം ഏല്‍പ്പിക്കാന്‍ അനുവദനീയമാണ് എന്ന അഭിപ്രായപ്പെടുന്നു (മവാഹിബുല്‍ ജലീല്‍: ഹത്താബ് അര്‍റുഅ്യാനി: 8:65)

ഇമാം ഇബ്നു ജരീര്‍ (ജനനം: ഹിജ്റ 224) സ്ത്രീകള്‍ക്ക് നിരുപാധികം ന്യായാധിപത്യം നല്‍കപ്പെടുമെന്ന് അഭിപ്രായപ്പെടുന്നു. സ്ത്രീകളുടെ ഫത്‌വകള്‍ (മത വിധികള്‍) ഏതു വിഷയത്തിലും സ്വീകരിക്കപ്പെടുമെന്നതാണ് അവരുടെ ന്യായാധിപത്യവും നിരുപാധികം സ്വീകരിക്കാന്‍ അദ്ദേഹം ന്യായമായി കാണുന്നത് (അല്‍ മുഗ്‌നി 9:39, അദബുല്‍ ക്വാളി: 1:625, റൗളത്തുല്‍ ക്വുള്വാത്ത്: 1:53, ഫത്ഹുല്‍ കദീര്‍: 5:485, ഫത്ഹുല്‍ബാരി: 13/147. ഉദ്ദരണം: അല്‍ മൗസൂഅത്തുല്‍ ഫിക്ഹിയ്യ: 33:294)

”യഹ്‌യബ്നു അബീ സുലൈം പറഞ്ഞു: പ്രവാചകാനുചരയായ (സ്വഹാബി വനിത) സംറാഅ് ബിന്‍ത്ത് നുഹൈക് പരുക്കന്‍ മുഖമക്കനയും പടച്ചട്ടയും ധരിച്ച്, കയ്യില്‍ ചാട്ടവാറേന്തി ജനങ്ങളില്‍ മര്യാദ നടപ്പാക്കിയും നന്മ കല്‍പ്പിച്ചും തിന്മ വിരോധിച്ചും നടക്കുന്നത് ഞാന്‍ കാണുകയുണ്ടായി” (അല്‍ മുഅ്ജമുല്‍ കബീര്‍: ത്വബ്റാനി: 24/311, നമ്പര്‍: 785, നിവേദക പരമ്പരയിലെ എല്ലാവരും വിശ്വസ്തരാണെന്ന് ഇമാം ഹൈഥമി പറയുന്നു (9/264). ശൈഖ് അല്‍ബാനി പരമ്പര നല്ലതെന്ന് അഭിപ്രായപ്പെട്ടിരിക്കുന്നു (24/311:785). (ഉമര്‍ (റ) ശിഫാഅ് ബിന്‍ത് അബ്ദില്ല എന്ന സ്ത്രീയെ അങ്ങാടിയിലെ നിയമപാലന ചുമതല ഏല്‍പ്പിച്ചിരുന്നു എന്ന നിവേദനത്തില്‍ ദുര്‍ബലത കാണുന്നവര്‍ക്ക് സംറാഅ് ബിന്‍ത്ത് നുഹൈകിന്നെ പറ്റിയുള്ള പ്രബലമായ ഈ നിവേദനം സ്വീകരിക്കാവുന്നതാണ്)

‘തങ്ങളുടെ ഭരണനേതൃത്വം ഒരു സ്ത്രീയെ ഏല്‍പ്പിച്ച ഒരു ജനത വിജിയിക്കില്ല’ എന്ന ഹദീഥിനെ പ്രത്യേക പശ്ചാത്തലത്തില്‍ ബന്ധിപ്പിക്കരുതെന്നും സാമാന്യര്‍ത്ഥത്തില്‍ തന്നെ അതിനെ മനസ്സിലാക്കിയാലും അതുകൊണ്ടുദ്ദേശിക്കപ്പെടുന്നത്; സ്ത്രീക്ക് – പൗരാണിക രാഷ്ട്ര വ്യവസ്ഥയില്‍ നില നിന്നിരുന്ന സ്വഭാവത്തിലുള്ള – സര്‍വ്വാധിപത്യം നല്‍കുന്നതിനെപ്പറ്റിയാണ്. അല്ലാതെ അധികാരത്തില്‍ നിന്നും ഒരു പങ്കും സ്ത്രീക്ക് നല്‍കരുതെന്ന നിലപാടല്ല. അതുകൊണ്ട് തന്നെ ആധുനിക ജനാധിപത്യ വ്യവസ്ഥകളുടെ കീഴില്‍ സ്ത്രീയെ മന്ത്രിസ്ഥാനം, അഡ്മിനിസ്ട്രേഷന്‍, ജനപ്രതിനിധി തുടങ്ങിയ അധികാരങ്ങള്‍ ഏല്‍പ്പിക്കുന്നത് ഹദീഥിന്റെ പ്രതലം പറ്റികൊണ്ട് എതിര്‍ക്കപ്പെടേണ്ട കാര്യമല്ല. കാരണം ആധുനിക ജനാധിപത്യ വ്യവസ്ഥകളുടെ കീഴില്‍ സ്ത്രീയെ മന്ത്രിസ്ഥാനം, അഡ്മിനിസ്ട്രേഷന്‍, ജനപ്രതിനിധി തുടങ്ങിയ അധികാരങ്ങള്‍ ഏല്‍പ്പിക്കുമ്പോള്‍ ഭരണഭാരം പൂര്‍ണ്ണമായും സ്ത്രീയുടെ തലയില്‍ ചുമത്തുന്നില്ല. മറിച്ച് കൂട്ടുത്തരവാദിത്വത്തിലെ ഒരു പങ്കു മാത്രമാണ് അവിടെ സ്ത്രീക്ക് വഹിക്കേണ്ടി വരുന്നത്.

ബ്രിട്ടനിലെ താച്ചറുടേയും ഇന്ത്യയിലെ ഇന്ദിരാഗാന്ധിയുടേയും അധിനിവേശ ഫലസ്തീനിലെ ഗോള്‍ഡാമീറിന്റേയും ഭരണരീതി സമൂഹത്തില്‍ ഒരു സ്ത്രീയുടെമാത്രം ഭരണമല്ല. മറിച്ച് ഭരിക്കുന്ന ഗ്രൂപ്പുകളുടേയും വ്യവസ്ഥയുടേയും ഭരണമാണ്. തലപ്പത്ത് സ്ത്രീയാണെന്ന് മാത്രം. ഭരണനിര്‍വ്വഹണം നടത്തുന്നത് കൂട്ടുത്തരവാദിത്വമുള്ള മന്ത്രിസഭകളാണ്. അവിടെ സ്ത്രീ സര്‍വ്വ ഭാരവും ഉത്തരവാദിത്തവും മുഴുവന്‍ പേറേണ്ടിവരുന്ന ഭരണാധികാരിയല്ല. പാര്‍ട്ടിയിലേയോ വ്യവസ്ഥയിലേയോ ഭൂരിപക്ഷത്തിന്റെ പിന്തുണയും ആനുകൂല്യം സഹായവും അവള്‍ക്ക് ലഭ്യമാകുന്നുണ്ട്. വിമര്‍ശനവിധേയമായ ഹദീഥിനെ സാമാന്യ അർത്ഥത്തില്‍ തന്നെ എടുക്കണമെന്ന നിലപാട് സ്വീകരിച്ച പണ്ഡിതന്മാരില്‍ പലരും ഇതിനപ്പുറത്തേക്ക് ഹദീഥിനെ വ്യാഖ്യാനിച്ചിട്ടില്ല. ഈ നിലപാടാകട്ടെ ആധുനിക ജനാധിപത്യ വ്യവസ്ഥകളുടെ നിലപാടുകള്‍ക്ക് വിരുദ്ധവുമല്ല. കാരണം സര്‍വ്വാധിപത്യമെന്നത് ജനാധിപത്യവിരുദ്ധമായ ഒരു നിലപാടാണല്ലൊ. ഒരു ജനാധിപത്യവ്യവസ്ഥയുടെ ഏതു പ്രതലത്തില്‍ നിന്നുകൊണ്ടും, പ്രസ്തുത ഹദീഥിനെ പ്രശ്നവല്‍ക്കരിച്ച് പെണ്‍വിരുദ്ധതയുടെ ആലയില്‍ കൊണ്ടുപോയി കെട്ടാന്‍ സാധ്യമല്ല; വിശിഷ്യ ജനാധിപത്യവാദികള്‍ക്ക്.

‘തങ്ങളുടെ ഭരണനേതൃത്വം ഒരു സ്ത്രീയെ ഏല്‍പ്പിച്ച ഒരു ജനത വിജയിക്കില്ല’ എന്ന ഹദീഥിന് നല്‍കപ്പെട്ട രണ്ട് രൂപത്തിലുള്ള വിശദീകരണങ്ങളില്‍ ഏതു സ്വീകരിച്ചാലും ശരി (പ്രത്യേക പശ്ചാത്തലത്തില്‍ ബന്ധിതമാണ് ഹദീഥ്; അല്ല അതിനെ സാമാന്യ അര്‍ത്ഥത്തില്‍ എടുക്കണം) സ്ത്രീയുടെ അധികാരത്തിലുള്ള എല്ലാ വിധ പങ്കിനേയും ഹദീഥ് എതിര്‍ക്കുന്നില്ല എന്നത് സ്പഷ്ടമാണ്.

പൗരാണിക രാഷ്ട്ര – രാഷ്ട്രീയ വ്യവസ്ഥയില്‍ നിലനിന്നിരുന്ന – സാധാരണ ഗതിയില്‍ പുരുഷ സമൂഹത്താല്‍ കയ്യാളപ്പെട്ടിരുന്ന – പരമാധികാരത്തിന്റെ സ്വഭാവവും പ്രകൃതിയും ചരിത്രത്തിലൂടെ കണ്ണോടിക്കുമ്പോള്‍, എന്തുകൊണ്ട് സ്ത്രീത്വത്തിന് അത് യോജിക്കില്ലെന്നുള്ള അഭിപ്രായം രൂപീകൃതമായി എന്ന് മനസ്സിലാക്കാന്‍ എളുപ്പമായിരിക്കും. അതിനായി അക്കാലഘട്ടത്തിലെ പേര്‍ഷ്യന്‍ റോമന്‍ സാമ്രാജ്യങ്ങളിലെ ‘പരമാധികാരവും’ അതിനു കീഴില്‍ നടമാടിയിരുന്ന രക്താഭിഷിക്തമായ സാമൂഹിക-രാഷ്ട്രീയ അന്തരീക്ഷവും തന്നെ പരിശോധനക്കെടക്കാം:

ഇസ്‌ലാമിന്റെ ആവിര്‍ഭാവത്തിനു മുമ്പ് തന്നെ പേര്‍ഷ്യന്‍ റോമന്‍ സാമ്രാജ്യങ്ങള്‍ക്കിടയിലും പാഴ്‌സികള്‍, ക്രിസ്ത്യാനികള്‍, ജൂതന്മാര്‍ എന്നിവര്‍ക്കിടയിലും നടമാടിയിരുന്ന ഭരണ വടം വലികളും അധികാരമുപയോഗിച്ച് നടപ്പാക്കിയിരുന്ന മത ധ്വംസനങ്ങളും അന്നത്തെ ലോക വ്യവസ്ഥയിലെ അസഹിഷ്ണുതയുടെ ആഴം വ്യക്തമാക്കി തരുന്നുണ്ട്. ലബ്‌നാനിലെ പ്രസിദ്ധ സാഹിത്യകാരനും ചരിത്രകാരനുമായ ജോര്‍ജി സൈദാന്റെ ‘താരീഖുത്തമദ്ദുനുല്‍ ഇസ്‌ലാമി’ (تاريخ التمدن الإسلامي) എന്ന വിശ്രുതമായ ഗ്രന്ഥത്തില്‍ നിന്ന് ചില ചരിത്ര സാക്ഷ്യങ്ങള്‍ ചുവടെ ഉദ്ധരിക്കാം. (ജോര്‍ജി സൈദാന്‍ മുസ്‌ലിമായിരുന്നില്ല, ക്രിസ്ത്യാനിയായിരുന്നു എന്ന് സാന്ദര്‍ഭികമായി സൂചിപ്പിക്കട്ടെ.)

പേര്‍ഷ്യയും റോമും തമ്മിലുളള ശത്രുത പൗരാണികമാണ്. ഒരു പക്ഷെ ബി.സി അഞ്ചാം നൂറ്റാണ്ടിനുമപ്പുറം അതിന്റെ വേരുകള്‍ എത്തി നില്‍ക്കുന്നുണ്ടാകാം. ലോകത്തെ അടക്കി ഭരിക്കാനുള്ള ഇരു സാമ്രാജ്യങ്ങളുടെയും അത്യാഗ്രഹമായിരുന്നു ഈ ശത്രുതയുടെ അടിത്തറ. നൂറ്റാണ്ടുകളോളം നീണ്ടു നിന്ന ഈ അധികാര വടം വലി ഇസ്‌ലാമിന്റെ ആവിര്‍ഭാവ കാലഘട്ടത്തിലും തുടര്‍ക്കഥയായിരുന്നു. പേര്‍ഷ്യന്‍ സാമ്രാജ്യത്തിന്റെ ആധിപത്യം ഇസ്ര അനൂഷര്‍വാന്‍ ചക്രവര്‍ത്തിയില്‍ എത്തിച്ചേര്‍ന്നപ്പോള്‍ റോമന്‍ സാമ്രാജ്യത്തെ അല്‍പാല്‍പ്പമായി പിടിച്ചടക്കാന്‍ അദ്ദേഹം സൈന്യ വ്യൂഹത്തെ വിന്യസിച്ചു. സിറിയ പിടിച്ചടക്കുകയും അന്താഖിയ ചുട്ടു നശിപ്പിക്കുകയും ഏഷ്യാ മൈനര്‍ കൊള്ളയടിക്കുകയും ചെയ്തു. അന്നത്തെ റോമന്‍ ചക്രവര്‍ത്തിയായിരുന്ന ജസ്റ്റീനിയന്‍ ചക്രവര്‍ത്തിയും വിട്ടുവീഴ്ചയ്ക്ക് തയ്യാറല്ലായിരുന്നു. ക്രിസ്താബ്ദം 541 മുതല്‍ 561 വരെ ഇരുപതു വര്‍ഷം ഇരു രാഷ്ട്രങ്ങളും യുദ്ധത്തില്‍ മുഴുകി.

പര്‍വേസ് ചക്രവര്‍ത്തിയുടെ കാലഘട്ടത്തില്‍ തന്റെ സുഹൃത്ത് മോറിസിന്റെ കൊലപാതകത്തിന് പ്രതികാരമെന്ന പേരില്‍ റോമന്‍ സാമ്രാജ്യത്തെ പര്‍വേസ് ചക്രവര്‍ത്തി ആക്രമിക്കുകയുണ്ടായി. ക്രിസ്താബ്ദം 614 ല്‍ സിറിയ പിടിച്ചടക്കി… ജൂതന്മാരുടെ അകമഴിഞ്ഞ സഹായത്താല്‍ ബൈസാന്റിയന്‍ പടയെ പര്‍വേസ് ചക്രവര്‍ത്തി ഒന്നൊന്നായി കീഴടക്കി. ഈജിപ്ത്, അന്താഖിയ, ദമാസ്‌ക്കസ്, ബൈത്തുല്‍ മുഖദസ് തുടങ്ങിയവ പിടിച്ചടക്കി. ജറുസലേമിലെ ബൈത്തുല്‍ മുഖദസ് കൊള്ളയടിക്കാനും ക്രിസ്ത്യന്‍ പള്ളികളും പുണ്യപുരുഷന്മാരുടെ കല്ലറകള്‍ തീയിടാനും അവിടെയുള്ള വിലമതിക്കാനാകാത്ത സ്വത്തുക്കള്‍ പിടിച്ചു പറിക്കാനും തന്റെ സൈന്യത്തിന് പര്‍വേസ് ചക്രവര്‍ത്തി അനുവാദം നല്‍കി. സിറിയ വരെ ഈ കൊലയും കൊള്ളയും തുടര്‍ന്നു. 90000 ക്രിസ്ത്യാനികളെ സൈന്യം കൊന്നൊടുക്കി… ഇതെല്ലാം കണ്ടിട്ടും കാണാത്ത മട്ടില്‍ ഭീരുവായി (അന്നത്തെ) ഹെറാക്ലിയസ് ചക്രവര്‍ത്തി കൊട്ടാരത്തില്‍ തന്നെ ഇരുന്നു; ക്രിസ്താബ്ദം 632 ല്‍ ഏഷ്യാ മൈനറില്‍ വെച്ച് കൊല്ലപ്പെടുന്നത് വരെ… (ഈ വര്‍ഷമാണ് ഇങ്ങു അറേബ്യയില്‍, മുസ്‌ലിംകള്‍ മക്കയില്‍ നിന്ന് പലായനം ചെയ്യുന്നത്) (താരീഖുത്തമദ്ദുനുല്‍ ഇസ്‌ലാമി: ജോര്‍ജി സൈദാന്‍: 1: 43-48) ക്രിസ്ത്യന്‍ റോമും ജൂതന്മാരും തമ്മിലുള്ള അതിര്‍ത്തി യുദ്ധങ്ങളും കലാപങ്ങളും അക്കാലഘട്ടത്തിന്റെ പ്രധാന ഇതിവൃത്തം ആയിരുന്നു.

പ്രൊഫ. പി. എസ്. വേലായുധന്‍ എഴുതുന്നു: ”കോണ്‍സ്റ്റന്റൈന്‍ ചക്രവര്‍ത്തി ചരമം പ്രാപിച്ചപ്പോള്‍ രണ്ടു പുത്രന്മാരും ചക്രവര്‍ത്തിമാരായി. അവരുടെ ചാര്‍ച്ചക്കാരനായ ജൂലിയന്‍ ചക്രവര്‍ത്തിയായി ഏ.ഡി. 360 മുതല്‍ 363 വരെ ഭരിച്ച അദ്ദേഹം ഒരു ക്രൈസ്തവ വിരോധിയായിരുന്നു. അദ്ദേഹത്തില്‍ നിന്ന് ക്രിസ്ത്യാനികള്‍ക്ക് പീഢനം സഹിക്കേണ്ടി വന്നു. അദ്ദേഹത്തിനു ശേഷം വന്ന ചക്രവര്‍ത്തിമാരില്‍ ഏറ്റവും പ്രമാണി തിയോഡോഷ്യസ് ഒന്നാമനായിരുന്നു. അദ്ദേഹം എ.ഡി. 378 മുതല്‍ 395 വരെ ഭരിച്ചു. അദ്ദേഹത്തിന്റെ ഭരണകാലത്താണ് ക്രിസ്തുമതം ഔദ്യോഗികമായി അംഗീകരിക്കപ്പെട്ടത്. തന്റെ സമസ്ത പ്രജകളും ക്രിസ്തുമതം സ്വീകരിക്കണമെന്നുള്ള രാജകീയശാസനം എ.ഡി. 380 ല്‍ അദ്ദേഹം പുറപ്പെടുവിച്ചു. പിന്നീട് ക്രൈസ്തവേതര്‍ക്ക് മതാനുഷ്ഠാനത്തിനുള്ള സര്‍വാവകാശങ്ങളും നിഷേധിച്ചുകൊണ്ടും അവര്‍ക്ക് ഗവണ്‍മെന്റുദ്യോഗങ്ങളില്‍ പ്രവേശനം നിരസിച്ചു കൊണ്ടും ഉത്തരവു പുറപ്പെടുവിച്ചു. അങ്ങനെ അദ്ദേഹം ക്രിസ്തുമതത്തെ രാഷ്ട്ര മതമായി സ്വീകരിച്ചു.” (ലോക ചരിത്രം: ഒന്നാം ഭാഗം: പ്രൊഫ. പി. എസ്. വേലായുധന്‍: പേജ്: 174: കേരള ഭാഷാ ഇന്‍സ്റ്റിറ്റ്യൂട്ട്)

ഹെറാക്ലിയസിന്റെ കാലഘട്ടത്തില്‍ അന്‍താഖിയായില്‍ നടന്നിരുന്ന കൂട്ടക്കുരുതികള്‍ ഭയാനകമായിരുന്നു. ജൂതന്മാര്‍ വിപ്ലവത്തിലൂടെ ക്രിസ്ത്യാനികളെ വധിക്കുകയും ശരീരം കഷ്ണങ്ങള്‍ ആക്കുകയും അതി നിഷ്ഠൂരമായ വൈകൃത-ക്രൂരതകള്‍ ഒരു വശത്ത് നടപ്പാക്കിയപ്പോള്‍ മറുവശത്ത് ഹെറാക്ലിയസ് ചക്രവര്‍ത്തി (ഹെറാക്ലിയസ് എന്നത് റോമന്‍ ചക്രവര്‍ത്തിമാരുടെ സ്ഥാനപേരാണ്) എണ്ണമറ്റ ജൂതന്മാരെ കൊന്നൊടുക്കിക്കൊണ്ടിരുന്നു. നാട്ടില്‍ കാണപ്പെടുന്ന ജൂതന്മാരെയെല്ലാം വധിക്കുക എന്ന ഉത്തരവിറക്കുകയും ചെയ്തു. ഫലസ്തീനിലും പരിസര പ്രദേശങ്ങളിലും ഇത്തരം സംഘട്ടനങ്ങള്‍ പതിവായിരുന്നു… അന്നത്തെ റോമന്‍ ചക്രവര്‍ത്തിയുടെ സഹോദരന്‍ ആയിരുന്ന തിയഡോര്‍ ഫലസ്തീനിലെ ഭരണ കാര്യങ്ങള്‍ക്ക് നിയോഗിതനായപ്പോള്‍ നാട്ടിലെ സര്‍വ്വ ജൂതന്മാരെയും തിരഞ്ഞു പിടിച്ച് വധിക്കാന്‍ തുടങ്ങി. ജൂതന്മാര്‍ ആകട്ടെ തങ്ങളുടെ സഹായികളായ പേര്‍ഷ്യക്കാരില്‍ നിന്നും 80000 ക്രിസ്ത്യന്‍ ബന്ദികളെ വിലയ്ക്ക് വാങ്ങി അറുത്തു കൊന്നു… (താരീഖുത്തമദ്ദുനുല്‍ ഇസ്‌ലാമി: ജോര്‍ജി സൈദാന്‍: 1: 43-48)

ഈ വിധ്വംസനമയമായ രാഷ്ട്രവ്യവസ്ഥകളിലും ക്രൂരമായ രാഷ്ട്രീയവടംവലികളിലും സ്ത്രീയെ തള്ളിയിടുന്നതിനെ ‘സ്ത്രീ സ്വാതന്ത്ര്യ’മെന്നോ ‘സ്ത്രീ വിമോചന’മെന്നോ പറയാന്‍ വിവേകമതികളായ ആര്‍ക്കെങ്കിലും സാധിക്കുമോ ? സ്‌നേഹം, കാരുണ്യം, ആര്‍ദ്രത എന്നീ മനുഷ്യ മൂല്യങ്ങളുടെ പുനരുദ്ധാരകരും വാഹകരുമായി മാനവ ചരിത്രത്തിലുടനീളം നിലകൊണ്ടവരും ഇന്നും നിലകൊള്ളുന്നവരുമാണ് സ്ത്രീകള്‍. ലോകം സുബുദ്ധിയും സമാധാനവും കൈവിട്ട് ഊഷരമായി പരിണമിച്ച സന്ധികളില്‍, മനുഷ്യ മനസ്സുകളിലെ വൈകാരിക മണ്ഡലത്തില്‍ സഹാനുഭൂതിയുടേയും ആര്‍ദ്രതയുടേയും വിത്തുകളായി വര്‍ത്തിച്ച സ്ത്രീത്വത്തിന്റെ മനോഹാരിതയും സ്വത്വസൗന്ദര്യവും തിരിച്ചറിയാത്തവര്‍ക്കേ അവളെ ചോര കൊണ്ട് അലങ്കരിക്കാന്‍ തോന്നൂ.

വിഷയത്തിലേക്ക് തിരിച്ചു വരാം. സ്ത്രീയുടെ അധികാരത്തിലുള്ള എല്ലാ വിധ പങ്കിനേയും ഹദീഥ് എതിര്‍ക്കുന്നില്ല, ഏറിവന്നാല്‍ പൗരാണിക രാഷ്ട്ര വ്യവസ്ഥയിലെ അവളുടെ സര്‍വ്വാധിപത്യത്തെ മാത്രമാണ് അത് ചോദ്യം ചെയ്യുന്നത് എന്ന് സൂചിപ്പിച്ചുവല്ലൊ. അതുകൊണ്ടു തന്നെയാണ് ചരിത്രത്തിന്റെ വ്യത്യസ്ഥ ദിക്കുകളില്‍, ഇസ്‌ലാമിക പ്രവിശ്യകളില്‍, വൈവിധ്യമാര്‍ന്ന ചിന്താധാരകളില്‍ സ്ത്രീ ഭരണങ്ങളുടെ എണ്ണിയാല്‍ തീരാത്തത്ര ചരിത്രങ്ങള്‍ രേഖപ്പെടുത്തപ്പെട്ടത്. സ്ത്രീക്ക് അധികാരത്തില്‍ നിന്നും ഒരു പങ്കും നല്‍കാന്‍ ഇസ്‌ലാം അനുവദിച്ചിട്ടില്ലായിരുന്നെങ്കില്‍ അത്തരം ചരിത്രങ്ങള്‍ മുസ്‌ലിം നാടുകളില്‍ നിന്നും നമുക്ക് ഒരിക്കലും വായിച്ചെടുക്കാനാവില്ല. ഈജിപ്തിലെ ഫത്‌വ ബോര്‍ഡ്, ദാറുല്‍ ഇഫ്താഉല്‍ മിസ്‌രിയ്യയുടെ ഫത്‌വയില്‍ നിന്നും ആ ചരിത്ര നിമിഷങ്ങളെ പറ്റിയുമുള്ള സ്മൃതികള്‍ നമുക്കു വായിക്കുക. ”വ്യത്യസ്ഥങ്ങളായ കാലഘട്ടത്തില്‍ ഇസ്‌ലാമിക രാജ്യങ്ങളുടെ വ്യത്യസ്ഥ ഭാഗങ്ങളില്‍ സ്ത്രീകള്‍ ഭരിച്ചിട്ടുണ്ട്. അവര്‍ വ്യത്യസ്ഥങ്ങളായ നാമദേയത്തിലാണ് അറിയപ്പെട്ടത്. സുല്‍ത്താന, മലിക, ഹുര്‍റ, ഖാത്തൂന്‍ എന്നിങ്ങനെയുള്ള പേരുകളിലാണ് അവര്‍ അറിയപ്പെട്ടിരുന്നത്. ചരിത്രത്തിന്റെ വ്യത്യസ്ഥ ദിക്കുകളില്‍ ഇസ്‌ലാമിക രാജ്യങ്ങളില്‍ പല നാടുകളും അമ്പതിലേറെ സ്തീകള്‍ ഭരിച്ചതായി ഇസ്‌ലാമിക ചരിത്രം നമുക്ക് വ്യക്തമാക്കിതരുന്നുണ്ട്. ഇബ്‌നു കസീറിന്റെ ‘അല്‍ബിദായഃ വന്നിഹായഃ’യിലും ഇബ്നുല്‍ ജൗസിയുടെ ‘അല്‍മുന്‍തള്വിം’ എന്നു പറയുന്ന ഗ്രന്ഥത്തിലും സുമല്‍ അല്‍ കഹ്റുമാന എന്നു പറയുന്ന ഒരു സ്ത്രീ ക്വാളിയായി (ജഡ്ജിയായി) പ്രവര്‍ത്തിച്ചതായി ചരിത്രം നമുക്ക് വിവരിച്ചുതരുന്നുണ്ട്. അവരുടെ സദസ്സില്‍ ജഡ്ജിമാരും കര്‍മ്മശാസ്ത്രപണ്ഡിതന്മാരും മഹാന്മാരുമെല്ലാം ഹാജരായിരുന്നു. ഹിജ്റ 317ലാണ് അവര്‍ മരണപ്പെടുന്നത്. ഭരണം നടത്തിയിരുന്ന സ്ത്രീകളില്‍ ചിലര്‍ ക്രിമിനല്‍ കേസുകളില്‍ വിധി പറയുന്നവരായും ഉണ്ടായിരുന്നു. സുല്‍ത്താന തുര്‍ക്കാന്‍ ഖാത്തൂന്‍ ഒരുദാഹരണമാണ്. അവരുടെ അടുക്കല്‍ ക്രിമിനല്‍ കേസുകള്‍ വന്നിട്ടുണ്ടെങ്കില്‍ അവര്‍ നീതിപൂര്‍വവും നന്മയോടും കൂടി വിധി നല്‍കുമായിരുന്നു. ജിഹാദിലും യുദ്ധങ്ങളിലുമെല്ലാം സ്ത്രീകള്‍ പങ്കാളികളാകുന്നതിനെ നബി (സ) അംഗീകരിച്ചിട്ടുണ്ട് എന്നതാണ് മറ്റൊരു കാര്യം. സ്ത്രീകള്‍ പ്രവാചകന്റെ ഒപ്പം യുദ്ധം ചെയ്തിട്ടുണ്ട്. ഉമ്മുസുലൈം, ഉമ്മുഹറാം, ബിന്‍ത് മില്‍ഹാന്‍, ഉമ്മുല്‍ ഹാരിസ അല്‍ അന്‍സാരി, റബീഅ ബിന്‍ത് മുഅവ്വദ്ബ്നു അഫ്റാഅ്, ഉമ്മുസിനാന്‍ അല്‍ അസ്‌ലമിയ്യ, ഹംന ബിന്‍ത് ജഹ്ശ്, ഉമ്മു സിയാദ് അല്‍ അശ്ജഈയ്യ പോലെയുള്ള സ്ത്രീകള്‍ അതിനുദാഹരണമാണ്. അതുപോലെതന്നെ ഇസ്‌ലാമിക ചരിത്രത്തിന്റെ വിവിധ ഘട്ടങ്ങളില്‍ ആയിരകണക്കിന് നിപുണകളായ പണ്ഡിതകളും, അതുപോലെതന്നെ ഇസ്‌ലാമികവും അറബിയുമായും ബന്ധപ്പെട്ട വിജ്ഞാനശാസ്ത്രങ്ങളിലുമെല്ലാം കഴിവുറ്റ പ്രതിഭകളും ഉണ്ടായിട്ടുണ്ട് ‘അല്‍ ഇസ്വാബഃ ഫീതമീസിസ്സ്വഹാബ’ എന്ന ഇബ്നു ഹജര്‍ അല്‍ അസ്‌ക്വലാനിയുടെ ഗ്രന്ഥത്തില്‍, അദ്ദേഹം ഇത്തരത്തില്‍ ഉള്ള ആയിരത്തി അഞ്ഞൂറ്റി നാല്‍പത്തിമൂന്ന് സ്ത്രീകളുടെ ചരിത്രം ഉള്‍പ്പെടുത്തിയിട്ടുണ്ട്. അവരുടെ കൂട്ടത്തില്‍ കര്‍മ്മശാസ്ത്ര പണ്ഡിതകളും, ഹദീഥ് പണ്ഡിതകളും, സാഹിത്യകാരികളും, വിധിയും നിയമനിര്‍മ്മാണവും നീതിപാലനവുമായും ബന്ധപ്പെട്ട സ്ഥാനങ്ങള്‍ വഹിച്ച സ്ത്രീകളും, ഇസ്‌ലാമിക ഫിഖ്ഹില്‍ ‘അല്‍ ഹസബ’ എന്നറിയപ്പെട്ടിരുന്ന പ്രത്യേക സ്ഥാനങ്ങള്‍ സ്ത്രീകള്‍ വഹിച്ചതായും ഒരുപാട് നിവേദനങ്ങള്‍ ഉദ്ദരിക്കപ്പെട്ടിട്ടുണ്ട്. ഇതെല്ലാം ഹിജ്റ ഒന്നാം നൂറ്റാണ്ടിലായിരുന്നു എന്നത് പ്രത്യേകം ഓര്‍ക്കണം. (ഫത്‌വ ബോര്‍ഡ് ദാറുല്‍ ഇഫ്താഉല്‍ മിസ്രിയ്യ, ഈജിപ്ത് (tthps://bit.ly/38GZKjD)

മൂന്നാം നൂറ്റാണ്ടുകാരിയായ സിത്തുല്‍ മുല്‍ക്, അഞ്ചാം നൂറ്റാണ്ടിലെ സ്വന്‍ആഅ് ഭരിച്ച അസ്മാഅ്, നാലാം നൂറ്റാണ്ടില്‍ യമനില്‍ ജനിച്ച അര്‍വ്വ ബിന്‍ത് അഹ്‌മദ്, സ്പെയിനിലെ സൈനബ് നഫ്സാവിയ്യ, സുല്‍ത്താന റദ്വിയ, ഏഴാം നൂറ്റാണ്ടില്‍ ഈജിപ്ത് ഭരിച്ച ശജറുദ്ദുര്‍റ്, സ്പെയിനിലെ ആഇശ ഹുര്‍റ, സിത്തുല്‍ അറബ്, സിത്തുല്‍ അജം, സിത്തുല്‍ വുസറാഅ് അത്തന്നൂഖിയ്യ, ശരീഫ ഫാത്തിമിയ്യ, ഗാലിയ്യ വഹ്ഹാബിയ്യ, ഖാത്തൂന്‍ ഖത്ലഅ് താര്‍കാന്‍, ഖാത്തൂന്‍ ബാദ്ശാഹ്, ഗസാല ശബീബ, സുല്‍ത്താന ഖദീജ, അബിശ് ഖാത്തൂന്‍, ദൗലത്ത് ഖാത്തൂന്‍, തുര്‍ഖാന്‍ ഖാത്തൂന്‍ തുടങ്ങി വ്യത്യസ്ത മദ്ഹഹബ്‌കാരും, വിഭാഗക്കാരുമായ സ്ത്രീകള്‍ ഭരണ- സൈനിക സാരഥ്യം വഹിച്ചിട്ടുണ്ടെന്നതിന് ചരിത്രം സാക്ഷിയാണ്. (tthps://www.dtoosr.org/699052)

ഭരണം, വിധി, നിയമനിര്‍മ്മാണം, നീതിപാലനം, സൈനികം, കര്‍മ്മശാസ്ത്രം, ഹദീഫ്, സാഹിത്യം, ഭാഷാശാസ്ത്രം, വിദ്യഭ്യാസ പ്രവര്‍ത്തനങ്ങള്‍, വൈദ്യം തുടങ്ങി വിശാലമായ പല മേഖലകളിലും സ്ത്രീ നേതാക്കള്‍ ഇസ്‌ലാമിക ചരിത്രത്തില്‍ സുലഭമായിരുന്നു. ‘മുഅ്ജമു അഅ്ലാമി നിസാഅ്’ അഥവാ ‘സ്ത്രീ നേതാക്കളെ പറ്റിയുള്ള നിഖണ്ഡു’ എന്ന ഒരു ഗ്രന്ഥം മുഹമ്മദ് തന്നൂഖിയുടേതായി കാണാം. ചരിത്രത്തിന്റെ വ്യത്യസ്ത ദിക്കുകളില്‍, ഇസ്‌ലാമിക രാജ്യങ്ങളില്‍ മേല്‍പറയപ്പെട്ട സ്തുത്യാര്‍ഹമായ മേഖലകളില്‍ വിരാജിച്ച നിപുണകളായ മുസ്‌ലിം സ്ത്രീ രത്നങ്ങളുടെ ജീവചരിത്രശേഖരമാണ് പ്രസ്തുത ഗ്രന്ഥം. പതിനായിരത്തിലതികം സ്ത്രീകളെപറ്റി ആ ഗ്രന്ഥം ലോകത്തോട് സംസാരിക്കുന്നുണ്ട്. അത്തരത്തിലുള്ള ഒട്ടനവധി ഗ്രന്ഥസമുച്ചയങ്ങള്‍ തന്നെ ഇസ്‌ലാമിക ലോകത്ത് ഇന്നും സുലഭമാണ്.

ഉര്‍വത്തിബ്നു സുബൈര്‍ (റ) പറഞ്ഞു: ”ഖുര്‍ആന്‍, അനന്തരാവകാശ നിയമങ്ങള്‍, ഹറാം ഹലാലുകള്‍ (അഥവാ കര്‍മ്മശാസ്ത്രം), കവിത, അറബികളുടെ നാട്ടറിവുകള്‍, കുടുംബപരമ്പരകള്‍ എന്ന് തുടങ്ങി ഒരു വിഷയത്തിലും ആഈശയേക്കാള്‍ അറിവുള്ള ഒരാളെയും ജനങ്ങളില്‍ ഞാന്‍ കണ്ടിട്ടില്ല.” (തദ്കിറത്തുല്‍ ഹുഫ്ഫാദ്, ദഹബി: 1/25) മസ്റൂക് പറഞ്ഞു: ”എന്റെ ആത്മാവ് ആരുടെ കൈയ്യിലാണോ അവന്‍ തന്നെ സത്യം. മുഹമ്മദ് നബി (സ)യുടെ അനുചരന്മാരില്‍ മഹാപണ്ഡിതരായ തലമുതിര്‍ന്നവര്‍ ആഇശ(റ)യോട് അനന്തരാവകാശ നിയമങ്ങളെ സംബന്ധിച്ച് ചോദിച്ച് പഠിക്കുന്നത് ഞാന്‍ കണ്ടിട്ടുണ്ട്.” (മുസ്വന്നഫ് ഇബ്നു അബീശൈബ: 30387) അലി (റ) പറഞ്ഞു: ”ആരെയൊക്കെ കൊണ്ടാണ് ഞാന്‍ പരീക്ഷിക്കപ്പെട്ടിരിക്കുന്നത് എന്ന് നിനക്കറിയാമോ? ജനങ്ങളില്‍ ഏറ്റവും, ജനങ്ങളാല്‍ അനുസരിക്കപ്പെടുന്ന ആഇശയെകൊണ്ടും, ജനങ്ങളില്‍ ഏറ്റവും ശക്തനായ സുബൈറിനെകൊണ്ടും, ജനങ്ങളില്‍ ഏറ്റവും ബുദ്ധിശാലിയായ ത്വല്‍ഹയെകൊണ്ടും, ജനങ്ങളില്‍ ഏറ്റവുംശുദ്ധനായ യഅ്ലബ്നു ഉമയ്യയെകൊണ്ടുമാണ് ഞാന്‍ പരീക്ഷിക്കപ്പെട്ടിരിക്കുന്നത്”. (ഫത്ഹുല്‍ ബാരി: 13/57)

ആഇശ എന്ന സ്ത്രീയുടെ മതപരമായ വിധികര്‍തൃത്വം മുസ്‌ലിം സമൂഹം അംഗീകരിച്ചിരുന്നതിന്റെ ചരിത്ര സാക്ഷ്യങ്ങളാണിത്. ‘ജനങ്ങളില്‍ ഏറ്റവും, ജനങ്ങളാല്‍ അനുസരിക്കപ്പെട്ടിരുന്ന വ്യക്തി’ എന്നാണ് അലി (റ) ആഇശയെപറ്റി പരാമര്‍ശിക്കുന്നത്. ജമല്‍ യുദ്ധത്തില്‍ തനിക്കെതിരായി ആഇശ നേതൃത്വം നല്‍കിയ കാര്യത്തെ അനുസ്മരിക്കുമ്പോള്‍ പോലും, അവരുടെ നേതൃപാടവത്തേയും ജനസമ്മതിയേയും അംഗീകരിച്ചുകൊണ്ടാണ് അലി(റ) സംസാരിക്കുന്നത്. സ്ത്രീ രാഷ്ട്രീയ നേതൃത്വം കൈയ്യാളുന്നത് ഇസ്‌ലാമില്‍ നിഷിദ്ധമായിരുന്നെങ്കില്‍ അദ്ദേഹം പ്രതിയോഗിക്കെതിരെ ആദ്യമായി ഉന്നയിക്കുമായിരുന്നു ഏറ്റവും പ്രധാനപ്പെട്ട വിമര്‍ശനം അതാകുമായിരുന്നില്ലേ?! ഒരു സ്ത്രീ ആയിരുന്നിട്ടും ആഇശയുടെ മതപരമായ വിധികര്‍തൃത്വം അംഗീകരിക്കാന്‍ ഇസ്‌ലാം ജനങ്ങള്‍ക്ക് തടസ്സം നിന്നിട്ടില്ലെങ്കില്‍ ഭൗതിക കാര്യങ്ങളില്‍പെട്ട വിധികര്‍തൃത്വം (ഭരണം) അംഗീകരിക്കാന്‍ അത് തടസ്സമാകുന്നതെങ്ങനെ?!

ജമല്‍ യുദ്ധത്തില്‍ ഒട്ടകത്തിന്റെ സംഘക്കാരുടെ (ആഇശയുടെ) കൂടെ ചേര്‍ന്ന് യുദ്ധം ചെയ്യാന്‍ ഞാനാഗ്രഹിച്ചപ്പോള്‍ പ്രവാചകനില്‍ നിന്ന് കേട്ട ഒരു വചനം എനിക്ക് ഏറെ ഉപകാരപ്പെട്ടു. പേര്‍ഷ്യക്കാര്‍ കിസ്റയുടെ മകളെ അധികാരമേല്‍പിച്ച വിവരം പ്രവാചകന് ലഭിച്ചപ്പോള്‍ അവിടുന്ന് പറഞ്ഞു: ‘സ്ത്രീയെ തങ്ങളുടെ ഭരണനേതൃത്വം ഏല്‍പ്പിച്ച ഒരു ജനത വിജയിക്കുകയില്ല’ (ബുഖാരി) എന്ന അബൂബക്റത്തി(റ)ന്റെ നിലപാടിനെ നാം അപ്പോള്‍ എങ്ങനെയാണ് മനസ്സിലാക്കേണ്ടത്.?

അബൂബക്റത് (റ) സ്വഹാബിയാണ് എന്നതുകൊണ്ട് അദ്ദേഹത്തിന്റെ ഗ്രാഹ്യതയില്‍ അബദ്ധം സംഭവിക്കില്ലെന്ന് പറയാനാവില്ല. ‘ഒരു സ്ത്രീയെ തങ്ങളുടെ ഭരണനേതൃത്വം ഏല്‍പ്പിച്ച ഒരു ജനത വിജയിക്കില്ല’ എന്ന ഹദീഥാണ് ജമല്‍ യുദ്ധത്തില്‍ ആഇശയുടെ പക്ഷം ചേരാതിരിക്കാന്‍ തന്നെ പ്രേരിപ്പിച്ച ഘടകമെന്ന അദ്ദേഹത്തിന്റെ വാചകം ധാരണപിശക് മാത്രമാണ്. കാരണം അതേ ഹദീഥ് കേട്ട അനേകം സ്വഹാബിമാര്‍ ആഇശ(റ)യുടെ പക്ഷം ചേര്‍ന്നിരുന്നു എന്നു നാം മനസ്സിലാക്കണം. സ്വര്‍ഗ്ഗം കൊണ്ട് സുവിശേഷം അറിയിക്കപ്പെട്ട സുബൈര്‍ (റ), ത്വല്‍ഹ (റ) തുടങ്ങിയവരും അബ്ദുല്ലാബിബ്നു ആമിര്‍ (റ), യഅ്ലബ്നു മനിയ്യ (റ), മുഹമ്മദിബ്നു ത്വല്‍ഹ (റ) തുടങ്ങിയ സ്വഹാബികളും ആഇശയുടെ നേതൃത്വം അംഗീകരിച്ചവരായിരുന്നു. അവരാരും തന്നെ, അബൂബക്റത്ത് (റ) ഹദീഥിനെ ഗ്രഹിച്ചതുപോലെ മനസ്സിലാക്കിയിട്ടുണ്ടായിരുന്നില്ല. ഉണ്ടായിരുന്നെങ്കില്‍ അവരും ആഇശയില്‍ നിന്നും വിട്ടുനില്‍ക്കുമായിരുന്നു. അതുകൊണ്ടാണ് അബൂബക്റത്തിന് സംഭവിച്ച ധാരണപിശക് മാത്രമാണിതെന്ന് പറഞ്ഞത്.

മാത്രമല്ല തന്റെ ഭാര്യമാരില്‍ ഒരാള്‍ ഭാവിയില്‍ ഈ യുദ്ധത്തിനു (ജമല്‍) നേതൃത്വം നല്‍കുമെന്ന് പ്രവാചകന്‍ (സ) തന്നെ അറിയിച്ചിരുന്ന കാര്യമായിരുന്നു. പ്രസ്തുത നിവേദനം നാം കാണുക. ”രാത്രിയില്‍ ബനൂ ആമിറുകാരുടെ നദീജല സ്രോതസ്സിനടുത്തെത്തിയപ്പോള്‍ നായ്ക്കള്‍ കുരക്കുന്നതായി ആഇശ (റ) കേട്ടു. ഇതേതാണ് ഈ ജലസ്രോതസ്സെന്ന് അവര്‍ ചോദിച്ചു. ഹൗഅബ് ജലതടമാണെന്ന് പറയപ്പെട്ടു. അപ്പോള്‍ അവര്‍ പറഞ്ഞു: മടങ്ങുവാനല്ലാതെ മറ്റൊന്നും ഞാന്‍ ഉദ്ദേശിക്കുന്നില്ല. തീര്‍ച്ചയായും അല്ലാഹുവിന്റെ തിരുദൂതന്‍ പറയുന്നത് ഞാന്‍ കേട്ടിട്ടുണ്ട്. ഹൗഅബിലെ നായ്ക്കള്‍ നിങ്ങളില്‍ (ഭാര്യമാരില്‍) ഒരാളുടെ മേല്‍ കുരക്കുന്നതിനെപറ്റി എങ്ങനെയുണ്ടെന്ന് തോന്നുന്നു. അപ്പോള്‍ സുബൈര്‍ (റ) പറഞ്ഞു: മടങ്ങുകയോ നിങ്ങള്‍ നിമിത്തം അല്ലാഹു ജനങ്ങള്‍ക്കിടയില്‍ രഞ്ചിപ്പുണ്ടാക്കിയാലോ? (അത്തരം ഒരവസരം നഷ്ടപ്പെടുത്തരുതെന്നര്‍ത്ഥം) (മുസ്‌നദ് അഹ്‌മദ്: 24654, ഇബ്‌നുഅബീശൈബ: 7/536, സില്‍സിലത്തു സ്വഹീഹ: 474, ഇബ്‌നു ഹിബ്ബാന്‍: 6732, ഹാകിം: 4613)

തന്റെ ഭാര്യമാരില്‍ ഒരാള്‍ തനിക്കുശേഷം അത്തരം ഒരു യുദ്ധത്തിന് നേതൃത്വം നല്‍കുമെന്ന് പ്രവചിച്ച നബി (സ), ആ ഘട്ടത്തില്‍ ഹൗഅബിലെ നായ്ക്കള്‍ ആര്‍ക്കുമേലായിരിക്കുമോ കുരക്കുന്നത് അവരായിരിക്കും അതെന്ന് അടയാളവും പറഞ്ഞു കൊടുത്തു. ഭാവിയില്‍ മുസ്‌ലിംങ്ങള്‍ക്കിടയില്‍ രൂപപ്പെടുന്ന ഒരു വലിയ കുഴപ്പത്തിന്റെ മുന്നോടിയായുള്ള സൂചനയായി അത് തന്റെ ഭാര്യമാരെ പഠിപ്പിച്ച പ്രവാചകന്‍, ആ ഘട്ടത്തില്‍ പോലും സ്ത്രീ നേതൃത്വത്തെ പഴിച്ചില്ലെന്നത് നാം പ്രത്യേകം മനസ്സിലാക്കുക. അഥവാ തെറ്റായ ഒരു പടപ്പുറപ്പാടായി ജമല്‍ യുദ്ധത്തെപറ്റി പ്രവാചകന്‍ (സ) പരാമര്‍ശിച്ചത്; അതിനു നേതൃത്വം നല്‍കുന്നത് സ്ത്രീയായതുകൊണ്ടല്ല. മറിച്ച് അത് മുസ്‌ലിംകള്‍ക്കിടയില്‍ വലിയ കുഴപ്പങ്ങള്‍ക്ക് നിമിത്തമാകും എന്നതുകൊണ്ടായിരുന്നു. സ്ത്രീ നേതൃത്വം ഇസ്‌ലാം വെച്ചുപൊറുപ്പിക്കാത്ത ഒന്നായിരുന്നെങ്കില്‍ ഈ സന്ദര്‍ഭത്തില്‍ പ്രവാചകന്‍ അത് പ്രത്യേകം പരാമര്‍ശിക്കുമായിരുന്നു. അപ്പോള്‍ ഭാവിയില്‍ സംഭവിക്കാനിരിക്കുന്ന ജമല്‍ യുദ്ധത്തെയും അതിന് ഒരു സ്ത്രീയായിരിക്കും നേതൃത്വം നല്‍കുകയെന്നും അത് തന്റെ ഭാര്യമാരില്‍ ഒരാളായിരിക്കും എന്നുമെല്ലാം മനസ്സിലാക്കിയ പ്രവാചകന്‍ അതിനെപറ്റി തന്റെ ഭാര്യമാര്‍ക്ക് മുന്നറിയിപ്പ് നല്‍കുമ്പോള്‍ സ്ത്രീ നേതൃത്വത്തെ പ്രശ്നവല്‍കരിക്കാതെ മുസ്‌ലിം സമൂഹത്തിനിടയില്‍ സംഭവിക്കുന്ന ഭിന്നതയേയും കുഴപ്പത്തേയും പറ്റി മാത്രം പ്രശ്നവല്‍കരിച്ചു സംസാരിച്ചു എന്നതില്‍ നിന്നു തന്നെ അബൂബക്റത്തിന്റെ (റ) നിലപാട് ധാരണപിശകില്‍നിന്ന് രൂപപ്പെട്ട ഒന്നാണെന്ന് വ്യക്തം. ചുരുക്കത്തില്‍, ‘സ്ത്രീയെ തങ്ങളുടെ ഭരണനേതൃത്വം ഏല്‍പ്പിച്ച ഒരു ജനത വിജയിക്കുകയില്ല’ എന്ന ഹദീഥ് സ്ത്രീയുടെ അധികാരത്തിലുള്ള എല്ലാ വിധ പങ്കിനേയും എതിര്‍ക്കുന്നില്ല; ഏറിവന്നാല്‍, പൗരാണിക രാഷ്ട്ര വ്യവസ്ഥയിലെ അവളുടെ സര്‍വ്വാധിപത്യത്തെ മാത്രമാണ് അത് ചോദ്യം ചെയ്യുന്നത്. അതുകൊണ്ടുതന്നെ പ്രസ്തുത ഹദീഥും പൊക്കിപ്പിടിച്ച് പൊഴിക്കുന്ന മുതലകണ്ണീരുകൊണ്ടൊന്നും കാര്യമായ പ്രയോജനം ഉണ്ടാകില്ലെന്ന് തിരിച്ചറിഞ്ഞ് ഒന്ന് മാറ്റിപിടിക്കുന്നതാണ് ഇസ്‌ലാംവിമര്‍ശകര്‍ക്ക് നല്ലത്. ഉഡായിപ്പുകള്‍കൊണ്ടൊന്നും വൈജ്ഞാനിക രംഗത്ത് മേല്‍വിലാസമുണ്ടാക്കാനാവില്ലെന്ന് ഇവരൊക്കെ ഇനി എന്നാണാവോ തിരിച്ചറിയുക.

നരകവാസികളില്‍ അധികവും സ്ത്രീകളാണെന്ന് പ്രഖ്യാപിക്കുക വഴി സ്ത്രീ സമൂഹത്തെ മൊത്തം അടച്ചാക്ഷേപിച്ച ഒരു വ്യക്തിയെ എങ്ങനെയാണ് ആത്മാഭിമാനമുളള സ്ത്രീകള്‍ ദൈവദൂതനായി അംഗീകരിക്കുക?

ഊഷരമായ ഹൃദയങ്ങളിലാണ് ഇസ്‌ലാംവിമര്‍ശനങ്ങളുടെ നിര്‍മ്മിതി നടക്കുന്നതെന്ന നിരീക്ഷണത്തിന് അടിവരയിടുന്ന ആരോപണമാണ് ഇതെന്നു പറയാതിരിക്കാന്‍ നിര്‍വാഹമില്ല. സ്ത്രീ സമൂഹത്തോട് ഏറെ ഗുണകാംക്ഷാനിര്‍ഭരമായി നല്‍കിയ ഒരു ഉപദേശത്തെപ്പോലും പെണ്‍വിരുദ്ധതയുടെ ആലയില്‍ കൊണ്ട്‌കെട്ടിയത് അക്ഷന്തവ്യമായ ഒരപരാധം തന്നെയാണ്. ആരോപണ വിധേയമായ നബിവചനം നമുക്കു പരിശോധിക്കുക: ”അബൂ സഊദുല്‍ ഖുദ്‌രി (റ) നിവേദനം: ഒരിക്കല്‍ തിരുമേനി (സ) വലിയ പെരുന്നാള്‍ ദിവസം നമസ്‌ക്കാര മൈതാനത്തേക്ക് പുറപ്പെട്ടു. (പൊതു ഉപദേശത്തിനുശേഷം) നബി (സ) സ്ത്രീകളുടെ അടുക്കലേക്ക് ചെന്നു. അവിടുന്ന് അരുളി: സ്ത്രീ സമൂഹമേ! നിങ്ങള്‍ ദാനധര്‍മ്മങ്ങള്‍ ചെയ്യുക. നരകവാസികളില്‍ അധികവും സ്ത്രീകളെയാണ് ഞാന്‍ കണ്ടിരിക്കുന്നത്. അപ്പോള്‍ സ്ത്രീകള്‍ ചോദിച്ചു: അല്ലാഹുവിന്റെ ദൂതരേ! എന്തുകൊണ്ടാണത്? നബി (സ) പ്രത്യുത്തരം നല്‍കി: അവര്‍ ശാപം വര്‍ദ്ധിപ്പിക്കും. ഭര്‍ത്താവിന്റെ നന്മയെ നിഷേധിക്കുകയും ചെയ്യും.” (ബുഖാരി)

മറ്റൊരു നിവേദനം ഇപ്രകാരമാണ്. ”ഞാന്‍ നരകത്തിലേക്ക് നോക്കി. അതില്‍ സ്ത്രീകളെയാണ് അധികമായി ഞാന്‍ കണ്ടത്. ഭര്‍ത്താവിനോട് അവര്‍ നന്ദികേട് കാണിക്കുന്നു. കാലം മുഴുവന്‍ നീ അവള്‍ക്ക് നന്മ ചെയ്തശേഷം നിന്നില്‍ നിന്ന് ഒരു ന്യൂനത കണ്ടാല്‍ അവള്‍ പറയും: നിങ്ങളില്‍ നിന്ന് ഇന്നോളം ഒരു നന്മയും എനിക്ക് കിട്ടിയിട്ടില്ല.”

പ്രസ്തുത നിവേദനങ്ങളില്‍ എവിടെയാണ് പെണ്‍വിരുദ്ധത കുടികൊള്ളുന്നത്. സ്ത്രീ പ്രകൃതിയില്‍ അതികമായി വന്നുപോകാറുള്ള ചില വീഴ്ച്ചകള്‍ സ്ത്രീ സമൂഹത്തെ ഉണര്‍ത്തുകയും നരക പ്രവേശനത്തിനു ഇടയാകാന്‍ സാധ്യതയുള്ള അത്തരം വീഴ്ച്ചകളെപറ്റി സ്ത്രീകളെ ബോധവല്‍കരിക്കുകയും അത്തരം തെറ്റുകള്‍ സംഭവിച്ചുപോകുന്നതിനുള്ള പ്രായശ്ചിത്തമായി ദാനധര്‍മ്മങ്ങള്‍ അധികരിപ്പിക്കുവാനും അതിലൂടെ മരണാനന്തര ജീവിതത്തില്‍ നരകമോചനം നേടിയെടുക്കാനുമുള്ള ഒരു ഉപദേശത്തെ പെണ്‍വിരുദ്ധതയുടെ പറ്റു പുസ്തകത്തില്‍ കുറിച്ചിടുന്നവരെ പറ്റി നാം എന്തുപറയണം!. ആത്മാഭിമാനമുള്ള സ്ത്രീകളെല്ലാം തന്നെ പ്രസ്തുത പ്രവാചകോപദേശത്തെ, സ്ത്രീ സമൂഹത്തോട് ഏറെ ഗുണകാംക്ഷ പുലര്‍ത്തിയ ഒരു മഹത്‌വ്യക്തിയുടെ സന്മാര്‍ഗദര്‍ശനമായാണ് വിലയിരുത്തുക.

ഹദീഥുകളില്‍ പരാമര്‍ശിച്ച രണ്ട് തിന്മകള്‍ സ്ത്രീകള്‍ക്കുമേല്‍ ആരോപിച്ചതിലെ ന്യായമാണ് ഒന്നാമതായി വിമര്‍ശകരെ അസ്വസ്ഥമാക്കുന്ന ഘടകം. ആ അസ്വസ്ഥത വിശ്വാസികള്‍ക്കിടയില്‍ വിതറുവാനാണ് അവര്‍ പരിശ്രമിച്ചുകൊണ്ടിരിക്കുന്നത്. ശപിക്കലും, ജീവിത പങ്കാളിയുടെ നന്മയെ നിഷേധിക്കലും ആണിലും പെണ്ണിലും കാണപ്പെടുന്ന തിന്മകളായിരിക്കെ, അത് സ്ത്രീകള്‍ക്കുമേല്‍ മാത്രമായി ചുമത്തിയതിലെ സ്ത്രീവിരുദ്ധത ഉല്‍ഖനനം ചെയ്യുകയാണവര്‍. ഹദീഥുകളുടെ പശ്ചാത്തലം പരിശോദിച്ചാല്‍ തീരാവുന്ന ഒരു പ്രശ്‌നം മാത്രമാണിത്. വലിയെ പെരുന്നാള്‍ ദിവസം നമസ്‌ക്കാര മൈതാനിയില്‍ വെച്ചാണ് പ്രസ്തുത സംഭവം നടക്കുന്നത്. അവിടെ ആദ്യം നബി (സ) ഒരു പൊതു ഉപദേശം (ഖുതുബ) നടത്തി. അതിനുശേഷം സ്ത്രീകളുടെ അടുത്തുചെന്ന് അവരെ മാത്രമായി ഉപദേശിക്കുകയാണുണ്ടായത്. സ്ത്രീ സമൂഹത്തെ അവമതിക്കലായിരുന്നു പ്രസ്തുത ഉപദേശം വഴി നബി (സ) ലക്ഷ്യം വെച്ചിരുന്നതെങ്കില്‍ അത് പൊതു ഉപദേശവേളയില്‍ തന്നെ ആകാമായിരുന്നു. കാരണം അപ്പോഴാണല്ലോ അത് സമൂഹമദ്ധ്യത്തില്‍ പരസ്യമാവുക. എന്നാല്‍, സ്ത്രീകളെ പ്രത്യേകം ബോധവല്‍ക്കരിക്കേണ്ട വിഷയം, തന്റെ പൊതു ഉപദേശത്തില്‍ (ഖുതുബ) ഉള്‍പ്പെടുത്താതെ സ്ത്രീകളുടെ മാത്രമായ സദസ്സില്‍ ചെന്ന് അവരെ പ്രത്യേകം ഉണര്‍ത്തുകയാണ് നബി (സ) ചെയ്തത്. വിമര്‍ശകര്‍ ആരോപിക്കുംവിധം സ്ത്രീകളെ അവമതിക്കുകയോ, പുരുഷ വര്‍ഗ ശ്രേഷ്ഠത പ്രഖ്യാപിക്കുകയോ ആണ് നബി(സ)യുടെ ഉദ്ദേശമെങ്കില്‍ അത് പ്രവചാകന്റെ പൊതു ഉപദേശവേളയിലാകുമായിരുന്നു. സ്ത്രീയിലും പുരുഷനിലും കാണപ്പെടുന്ന തിന്മകളെപ്പറ്റി പറയുമ്പോള്‍ എന്തുകൊണ്ടാണ് സ്ത്രീകള്‍ക്കുമേല്‍ അത് പ്രത്യേകം പരാമര്‍ശിക്കുന്നത്? തിന്മകള്‍ ചിലപ്പോള്‍ അങ്ങനെയാണ്. സ്ത്രീയിലും പുരുഷനിലും ഒരുപോലെ കാണപ്പെടുന്ന തിന്മകളുണ്ട്; അവയെ പറ്റി പരാമര്‍ശിക്കുമ്പോള്‍ അത് രണ്ട് കൂട്ടരിലും ഒരുപോലെ ചേര്‍ത്ത് പറയും. പുരുഷനില്‍ കാണാമെങ്കിലും സ്ത്രീയില്‍ അതിനേക്കാള്‍ അധികം കാണപ്പെടുന്ന തിന്മകളുണ്ട്; അവയെപ്പറ്റി പരാമര്‍ശിക്കുമ്പോള്‍ അതില്‍ സ്ത്രീകളെ പ്രത്യേകമാക്കി പറയും. ഉപര്യുക്ത ഹദീഥുകളില്‍ പരാമര്‍ശിച്ച തിന്മകള്‍ അത്തരത്തില്‍പ്പെട്ടതാണ്. സ്ത്രീ പ്രകൃതിയില്‍ അത് പുരുഷപ്രകൃതിയേക്കാളും ഏറെ കാണപ്പെടുന്നു. ഇനി സ്ത്രീകളില്‍ കാണപ്പെടുമെങ്കിലും പുരുഷന്മാരില്‍ ധാരാളമായി കാണുന്ന തിന്മകളുണ്ട്; അവയെപ്പറ്റി പരാമര്‍ശിക്കുമ്പോള്‍ അവിടെ പുരുഷനെ പ്രത്യേകമാക്കി പറയും. ഹദീഥുകളില്‍ വന്ന അത്തരം തിന്മകളെ പറ്റിയുള്ള പരാമര്‍ശങ്ങളില്‍ നിന്നും ഏതാനും ഉദാഹരണങ്ങള്‍ കാണുക:

1, ”അബൂഹുറൈറ(റ)യില്‍ നിന്നും നിവേദനം: അല്ലാഹുവിന്റെ ദൂതര്‍ (സ) പറഞ്ഞു: രണ്ടു വിഭാഗക്കാര്‍ നരകാവകാശികളാണ്; അവരെ ഞാനിതുവരെ കണ്ടുമുട്ടിയിട്ടില്ല: (ഒന്നാമത്തെ വിഭാഗം) ചില പുരുഷന്മാരാണ്. അവരോടൊപ്പം പശുവിന്റെ വാലുപോലെയുള്ള ചാട്ടവാറുകള്‍ ഉണ്ടാകും; അതുപയോഗിച്ച് അവര്‍ ജനങ്ങളെ അടിക്കും.” (അല്‍ മുഅ്ജമുല്‍ അവ്‌സത്: 5854, ശുഅ്ബുല്‍ ഈമാന്‍: 4972) ”അവന്‍ അല്ലാഹുവിന്റെ അമര്‍ഷത്തിലായികൊണ്ട് രാവിലെ പുറപ്പെടുകയും അവന്റെ കോപത്തിന് പാത്രീയരായി വൈകുന്നേരം മടങ്ങിവരികയും ചെയ്യും.” (മുസ്‌നദ് അഹ്‌മദ്: 22150)

അക്രമവാസനയാണ് ഇവിടെ ദൈവകോപത്തിനു വിധേയമായ തിന്മയായി പഠിപ്പിക്കുന്നത്. അക്രമവാസന സ്ത്രീകളിലുണ്ടെങ്കിലും പുരുഷപ്രകൃതിയിലാണ് അത് ധാരാളമായി കാണാറുള്ളത്. അതുകൊണ്ട് പ്രസ്തുത തിന്മയെ പുരുഷനിലേക്ക് ചേര്‍ത്ത് പറഞ്ഞെന്നുമാത്രം.

2, ”അല്ലാഹുവിന്റെ അടുക്കല്‍ ഏറ്റവും ഗുരുതരമായ മഹാപാപങ്ങളില്‍ പെട്ടതാണ്, (ഒന്ന്) ഒരാള്‍ ഒരു സ്ത്രീയെ വിവാഹം ചെയ്യുകയും എന്നിട്ട് അവളില്‍ നിന്നുള്ള ആവശ്യം പൂര്‍ത്തീകരിച്ചാല്‍ അവളെ വിവാഹമോചനം ചെയ്യുകയും അവളുടെ വിവാഹ മൂല്യവുമായി കടന്നുകളയുകയും ചെയ്യുക എന്നുള്ളതും, (രണ്ട്) ഒരാളെ പണിക്ക് വെച്ചിട്ട് (പണി പൂര്‍ത്തിയാക്കാതെ) കൂലിയും കൊണ്ട് പോവുക എന്നതും, (മൂന്ന്) അനാവശ്യമായി മൃഗങ്ങളെ കൊല്ലുക എന്നുള്ളതും.” (അല്‍ മുസ്തദ്‌റക്: ഹാകിം: 2/182)

വിശ്വാസ വഞ്ചനയും, തട്ടിപ്പും, അക്രമവുമാണ് ഉപര്യുക്ത ഹദീഥുകളിലെ പരമാര്‍ശ വിഷയം. സ്ത്രീകളിലും ഇത്തരം പ്രവണതകള്‍ കാണുമെങ്കിലും പുരുഷന്മാരിലാണ് ഇത് വ്യാപകമായി കാണുന്നത്. അതിനാല്‍ പ്രസ്തുത പാപങ്ങള്‍ പുരുഷന്മാരിലേക്ക് പ്രത്യേകം ചേര്‍ത്തു പറഞ്ഞു. റജുലുൻ (പുരുഷൻ), രിജാല്‍ (പുരുഷന്മാർ) എന്ന പദങ്ങളാണ് പ്രസ്തുത പാപങ്ങളുടെ ഉടമകളെ കുറിക്കാന്‍ ഹദീഥ് ഉപയോഗിച്ചിരിക്കുന്നത്. ഇത് ആണ്‍വിരുദ്ധതയുടെ ഭാഗമായി വിമര്‍ശകന്മാര്‍ അടയാളപ്പെടുത്തുമോ?! വേണമെങ്കില്‍ ഇസ്‌ലാമിക് ഫെമിനിസ്റ്റുകള്‍ക്ക് ഹദീഥ് ദാനം ചെയ്യുകയുമാകാം!!

3, ”അബൂസഈദി(റ)ല്‍ നിന്ന് നിവേദനം: നബി (സ) പറഞ്ഞിരിക്കുന്നു. പരസ്പരം അടുത്തിടപഴകിക്കഴിഞ്ഞ ശേഷം ഭാര്യയുടെ രഹസ്യങ്ങള്‍ പ്രചരിപ്പിക്കുന്ന പുരുഷനത്രെ പുനരുത്ഥാന നാളില്‍ അല്ലാഹുവിങ്കല്‍ ഏറ്റം മോശമായ സ്ഥാനമുള്ളവന്‍” (മുസ്‌ലിം).

കിടപ്പറ രഹസ്യങ്ങള്‍ മറ്റുള്ളവരോട് പങ്ക്‌വെച്ച് രസിക്കുന്ന ആണിനും പെണ്ണിനും ഹദീഥ് ഒരുപോലെ ബാധകമാണ്. രണ്ടുകൂട്ടരിലും പ്രസ്തുത തിന്മകള്‍ കാണാറുണ്ട്. എന്നാല്‍ സ്ത്രീകളെ അപേക്ഷിച്ച് പുരുഷന്മാരിലാണ് ആ പ്രവണത ധാരാളമായി കാണാറുള്ളത്. അതിനാല്‍ പ്രസ്തുത തിന്മയെ പുരുഷന്മാരിലേക്ക് പ്രത്യേകമാക്കി സൂചിപ്പിച്ചു.

ഇനി അവശേഷിക്കുന്ന പ്രശ്‌നം നരകവാസികളില്‍ അധികവും സ്ത്രീകളാണെന്ന ഹദീഥിലെ പരാമര്‍ശമാണ്. നബിപാഠങ്ങളില്‍ നിന്നും പെണ്‍വിരുദ്ധത ചിക്കിചികയുന്നവര്‍ ജന്മം കൊള്ളുന്നതിനും എത്രയോ നൂറ്റാണ്ടുകള്‍ക്കു മുമ്പുതന്നെ ഇസ്‌ലാമിക പണ്ഡിതന്മാര്‍ പ്രസ്തുത വിഷയം മുടിനാരിഴ കീറി പരിശോദിച്ചിരിക്കുന്നു. പ്രസ്തുത പരിശോദനകള്‍ മറുപടി നല്‍കാത്ത ഒരു പ്രശ്‌നംപോലും പെണ്‍പക്ഷവാദികള്‍ക്ക് ഇന്നും ചോദിക്കാനില്ലെന്നതാണ് അതിശയകരമായ സംഗതി. പക്ഷെ അതൊന്നും കാണാനോ പഠിക്കാനോ തയ്യാറാവാതെ ഒരു ഹദീഥ് കിട്ടുമ്പോഴേക്കും അതില്‍നിന്നും പെണ്‍വിരുദ്ധത എങ്ങിനെ കടഞ്ഞെടുക്കാമെന്നതാണ് ഇസ്‌ലാംവിമര്‍ശകരുടെ ഇഷ്ടവിനോദം. അതുകൊണ്ടുതന്നെ പ്രസ്തുത ഹദീഥുമായി ബന്ധപ്പെട്ട് നൂറ്റാണ്ടുകള്‍ക്കുമുമ്പുതന്നെ ഇസ്‌ലാമിക ലോകത്ത് നടന്ന മുടിനാരിഴ കീറിയ പരിശോദനകളില്‍ നിന്നും ചര്‍ച്ചകളില്‍ നിന്നും ഒരു ശകലം ഇസ്‌ലാംവിരോധികള്‍ക്കും മുര്‍തദ്ദാവാന്‍ മുട്ടി നില്‍ക്കുന്നവര്‍ക്കും മുമ്പില്‍ ഒരുപോലെ സമര്‍പ്പിക്കുകയാണ്.

സ്വര്‍ഗവാസികളിലും അധികം സ്ത്രീകള്‍ തന്നെ!

ഇബ്‌നു സിരീന്‍ (റ) പറഞ്ഞു: ഒരിക്കല്‍, സ്വര്‍ഗത്തില്‍ കൂടുതല്‍ ആരാണെന്ന കാര്യത്തില്‍ സ്ത്രീകളും പുരുഷന്മാരും തമ്മില്‍ തര്‍ക്കിക്കുകയുണ്ടായി. അങ്ങനെ അവര്‍ (അക്കാര്യത്തെ സംബന്ധിച്ച് നബി(സ)യില്‍ നിന്നുള്ള അറിവിനായി) അബൂഹുറൈറ(റ)യോട് ചോദിച്ചു: ”അപ്പോള്‍ അദ്ദേഹം പറഞ്ഞു. അബുല്‍ കാസിം (നബി (സ)) ഇപ്രകാരം പറഞ്ഞിട്ടില്ലേ, സ്വര്‍ഗ്ഗത്തില്‍ പ്രവേശിക്കുന്ന ആദ്യ സംഘം പൗര്‍ണമി രാത്രിയിലെ പൂര്‍ണ ചന്ദ്രനെ പോലെയായിരിക്കും. അവര്‍ക്കു ശേഷമുള്ളവര്‍ ആകാശത്തെ ഏറ്റവും തിളങ്ങുന്ന നക്ഷത്രത്തോളം പ്രകാശ പൂരിതരായിരിക്കും. അവരില്‍ ഒരോരുത്തര്‍ക്കും (സ്ത്രീകളില്‍ നിന്നുള്ള) രണ്ട് ഇണകള്‍ വീതം ഉണ്ടായിരിക്കും.” (സ്വഹീഹു മുസ്‌ലിം: 2834)

ഇമാം ഇബ്‌നു ഹജര്‍ അല്‍ അസ്‌ക്വലാനി (റ) പറഞ്ഞു: ”അവരില്‍ ഓരോരുത്തര്‍ക്കും രണ്ട് ഇണകള്‍ വീതം ഉണ്ടായിരിക്കും അഥവാ ഇഹലോകത്തെ സ്ത്രീകളില്‍ നിന്ന് രണ്ടുപേര്‍ ഉണ്ടായിരിക്കും.” (ഫത്ഹുല്‍ ബാരി: 6:325)

ഇമാം നവവി (റ) പറയുന്നു: ”ഇമാം ക്വാദി ഇയാദ് (റ) പറഞ്ഞു: സ്വര്‍ഗത്തില്‍ സ്ത്രീകളാണ് കൂടുതല്‍ എന്നാണ് ഹദീഥിന്റെ പ്രത്യക്ഷം സൂചിപ്പിക്കുന്നത്.” (സ്വഹീഹു മുസ്‌ലിം ബി ശര്‍ഹിന്നവവി: 9/142)

ഇബ്‌നു തിമിയ്യ (റ) പറഞ്ഞു: ”…കാരണം സ്ത്രീകളാണ് സ്വര്‍ഗത്തില്‍ പുരുഷന്മാരേക്കാള്‍ അധികം.” (മജ്‌മൂഉല്‍ ഫതാവാ: 6:432)

സ്വര്‍ഗത്തില്‍ സ്ത്രീകള്‍ തന്നെയാണ് കൂടുതലെന്ന് വേറെയും ഹദീഥുകള്‍ സൂചന നല്‍കുന്നുണ്ട്. ”നബി (സ) പറഞ്ഞു: സ്വര്‍ഗക്കാരെ സംബന്ധിച്ച് ഞാന്‍ നിങ്ങള്‍ക്ക് വിവരം നല്‍കട്ടെയോ? ”അതെ” എന്ന് പ്രവാചകാനുചരന്മാര്‍ പറഞ്ഞപ്പോള്‍ നബി (സ) വിശദീകരിച്ചു: എല്ലാ ദുര്‍ബലരും അടിച്ചമര്‍ത്തപ്പെട്ടവരുമാണവര്‍.” (സ്വഹീഹുല്‍ ബുഖാരി: 4967)

”അബൂ ഹുറൈറ(റ)ല്‍ നിന്നും നിവേദനം: നബി (സ) പറഞ്ഞു: അല്ലാഹുവാണെ സാക്ഷി; അനാഥ, സ്ത്രീ എന്നീ രണ്ട് ദുര്‍ബല വിഭാഗങ്ങളുടെ അവകാശങ്ങളെ (നിറവേറ്റുന്നതില്‍ പരാജയപ്പെടുന്നതിലുള്ള പാപത്തെ) സംബന്ധിച്ച് ഞാന്‍ നിങ്ങള്‍ക്ക് ശക്തമായ താക്കീത് നല്‍കുന്നു.” (മുസ്‌നദ് അഹ്‌മദ്: 2/439, റിയാളുസ്സ്വാലിഹീന്‍: 146)

എങ്കില്‍ നരകത്തില്‍ സ്ത്രീകളാണ് കൂടുതലെന്നു പറയുന്ന ഹദീഥുകളെ നാം എങ്ങനെയാണ് മനസ്സിലാക്കേണ്ടത്? അതും ഇസ്‌ലാമിക പണ്ഡതന്മാര്‍ ചര്‍വിതചര്‍വണം ചര്‍ച്ചചെയ്തിട്ടുണ്ട്. അവയില്‍ നിന്നും ഏതാനും ഭാഗങ്ങള്‍ നമുക്കും കാണാം.

ഇമാം നവവി (റ) ശര്‍ഹു മുസ്‌ലിമില്‍ പറയുന്നു: ”ഇമാം ക്വാദി ഇയാദ് പറഞ്ഞു: (സ്വര്‍ഗത്തിലെന്ന പോലെ) …നരകത്തിലും സ്ത്രീകളാണ് കൂടുതല്‍ എന്ന് മറ്റു ഹദീഥുകളിലും കാണാം. സ്ത്രീകളാണ് ആദം സന്തതികളില്‍ അധികവും എന്ന് ഇതില്‍ നിന്നും മനസ്സിലാക്കാം.” (ശര്‍ഹു മുസ്‌ലിം: 9/142)

ഇബ്‌നു തിമിയ്യ:(റ) പറഞ്ഞു: ”…അത് എന്തുകൊണ്ടെന്നാല്‍ പുരുഷന്മാരേക്കാള്‍ സ്ത്രീകളാണ് കൂടുതല്‍. നരകത്തിലും അപ്രകാരം തന്നെ. അപ്പോള്‍ സൃഷ്ടികളില്‍ കൂടുതലും സ്ത്രീകളാണ് എന്ന് വരുന്നു.” (മജ്‌മൂഉല്‍ ഫതാവാ: 6:432)

ഹാഫിള് അല്‍ ഇറാക്വി (റ) പറഞ്ഞു: ”…എന്നാല്‍ സ്ത്രീകള്‍ നരകത്തില്‍ കൂടുതല്‍ ഉള്ളതിനാല്‍ സ്വര്‍ഗത്തില്‍ കുറവാവല്‍ അനിവാര്യമല്ല. പ്രത്യുത സ്ത്രീകളാണ് (മനുഷ്യരില്‍) എണ്ണത്തില്‍ കൂടുതല്‍ എന്നതിനാല്‍ രണ്ടിടത്തും അവരാണ് അധികം.” (ത്വര്‍ഹുത്തസ്‌രീബ്: 8:270)

‘നരകവാസികളില്‍ അധികവും സ്ത്രീകളാണ്’ എന്ന ഹദീഥിന് മറ്റു ചില വ്യാഖ്യാനങ്ങള്‍ കൂടി പണ്ഡിതന്മാര്‍ നല്‍കിയിട്ടുണ്ട്. അവയില്‍ നിന്നും പ്രസക്തമായവ കാണാം.

ഇബ്‌നു ഹജര്‍ അല്‍ അസ്‌ക്വലാനി (റ) പറഞ്ഞു: ”ആദ്യത്തില്‍ നരകത്തില്‍ കൂടുതല്‍ സ്ത്രീകള്‍ ആയിരിക്കുകയും; അഥവാ, നരകത്തില്‍ നിന്ന് പാപങ്ങള്‍ക്കുള്ള ശിക്ഷ കഴിഞ്ഞതിന് ശേഷം ശുപാര്‍ശ വഴി ദോഷികള്‍ സ്വര്‍ഗത്തിലെത്തി ചേരുന്നതിന് മുമ്പാണ് ഈ അവസ്ഥ എന്ന് അനുമാനിക്കപ്പെടുന്നു. (ശേഷം സ്വര്‍ഗത്തില്‍ കൂടുതല്‍ സ്ത്രീകളും നരകത്തില്‍ കുറവും ആയിത്തീരും)” (ഫത്ഹുല്‍ ബാരി: 7:267)

ഇനിയും തീരാത്ത പെണ്‍പക്ഷ വിമര്‍ശനങ്ങള്‍

‘സ്വര്‍ഗവാസികളില്‍ എണ്ണത്തില്‍ ഏറ്റവും കുറഞ്ഞവര്‍ സ്ത്രീകളാണ്’ എന്ന് സ്വഹീഹ് മുസ്‌ലിമില്‍ (2738) കാണാം. ഈ ഹദീഥ് മേല്‍ പറയപ്പെട്ട പണ്ഡിതാഭിപ്രായങ്ങളെ മുഴുവന്‍ ഖണ്ഡിക്കുന്നതാണല്ലോ? എന്ന മറ്റൊരു വിമര്‍ശനം ചിലപ്പോള്‍ ഇസ്‌ലാംവിമര്‍ശകര്‍ ഉന്നയിച്ചേകാം. പ്രസ്തുത വിമര്‍ശനവും കൂടി നമുക്കു പഠനവിധേയമാക്കാം.

‘സ്വര്‍ഗവാസികളില്‍ എണ്ണത്തില്‍ ഏറ്റവും കുറഞ്ഞവര്‍ സ്ത്രീകളാണ്’ എന്ന ഹദീഥും ഇസ്‌ലാമിക പണ്ഡിതന്മാര്‍ വ്യക്തമായ നിലയില്‍ തന്നെ പഠനവിധേയമാക്കിയിട്ടുണ്ട്. ആ പഠനങ്ങളില്‍ നിന്നും പ്രസക്തമായ ഏതാനും ചില ചര്‍ച്ചകള്‍ കൂടി നമുക്കു കാണാം.

ഇമാം ഹാഫിള് അല്‍ ഇറാക്വി (റ) പറഞ്ഞു:”നരകത്തില്‍ കൂടുതല്‍ സ്ത്രീകളാണ് എങ്കില്‍ സ്വര്‍ഗവാസികളില്‍ കുറവ് സ്ത്രീകളായിരിക്കുമല്ലോ എന്ന് തെറ്റിദ്ധരിച്ച്, ഹദീഥിന്റെ ആശയം നിവേദനം ചെയ്തപ്പോള്‍ നിവേദകന് തെറ്റ് പറ്റിയതാകാം. എന്നാല്‍ സ്ത്രീകള്‍ നരകത്തില്‍ കൂടുതല്‍ ഉള്ളതിനാല്‍ സ്വര്‍ഗത്തില്‍ കുറവാവല്‍ അനിവാര്യമല്ല. പ്രത്യുത സ്ത്രീകളാണ് (മനുഷ്യരില്‍) എണ്ണത്തില്‍ കൂടുതല്‍ എന്നതിനാല്‍ രണ്ടിടത്തും അവരാണ് അധികം.” (ത്വര്‍ഹുത്തസ്‌രീബ്: 8:270)

ഇമാം മുനാവി (റ) പറഞ്ഞു: ”സ്വര്‍ഗവാസികളില്‍ കുറവ് സ്ത്രീകളായിരിക്കും” എന്നത് കൊണ്ടുദ്ദേശം, നരകത്തില്‍നിന്ന് ദോഷികള്‍ സ്വര്‍ഗത്തിലെത്തി ചേരുന്നതിന് മുമ്പാണ്. അപ്പോള്‍ ഇഹലോകത്തെ സ്ത്രീകള്‍ സ്വര്‍ഗത്തില്‍ പുരുഷന്മാരേക്കാള്‍ കുറവാണെന്ന് ഹദീഥ് തെളിയിക്കുന്നില്ല.” (ഫൈളുല്‍ ഖദീര്‍: 2/543)

ഇമാം ഇബ്‌നു ഹജര്‍ അല്‍ അസ്‌ക്വലാനി (റ) പറഞ്ഞു: ”നരകത്തില്‍ കൂടുതല്‍ സ്ത്രീകളാണ് എങ്കില്‍ സ്വര്‍ഗവാസികളില്‍ കുറവ് സ്ത്രീകളായിരിക്കുമല്ലോ എന്ന് തെറ്റിദ്ധരിച്ച്, ഹദീഥിന്റെ ആശയം നിവേദനം ചെയ്തപ്പോള്‍ നിവേദകന് തെറ്റ് പറ്റിയതാകാനാണ് സാധ്യത.” (ഫത്ഹുല്‍ ബാരി: 7:267)

‘സ്വര്‍ഗവാസികളില്‍ എണ്ണത്തില്‍ ഏറ്റവും കുറഞ്ഞവര്‍ സ്ത്രീകളാണ്’ എന്ന ഹദീഥിനെ പറ്റി ചര്‍ച്ചചെയ്യവെ അത് നിവേദകനില്‍ സംഭവിച്ച അബദ്ധമായിരിക്കാം എന്ന് പൂര്‍വ്വികരും ആധുനികരുമായ പണ്ഡിതന്മാരില്‍ പലരും ചൂണ്ടി കാണിക്കുവാനുള്ള ന്യായമെന്താണ്?

‘സ്വര്‍ഗവാസികളില്‍ കുറവ് സ്ത്രീകളായിരിക്കു’മെന്ന ഹദീഥ് ഇംറാനുബ്‌നു ഹുസൈനില്‍ നിന്ന് ഉദ്ദരിച്ചിരിക്കുന്നത് അബുത്തയ്യാഹ് എന്ന നിവേദകനാണ്. പ്രവാചകശിഷ്യന്‍(സ്വഹാബി) ഇംറാനുബ്‌നു ഹുസൈന്റെ ഈ ഹദീഥ് ഉദ്ദരിച്ച അബുത്തയ്യാഹ് എന്ന നിവേദകനല്ലാതെ മറ്റുനിവേദകരാരും ഇംറാനുബ്‌നു ഹുസൈനില്‍ നിന്ന് ഇപ്രകാരം ഉദ്ദരിച്ചിട്ടില്ല. (ഉദാഹരണത്തിന് ഇംറാനുബ്‌നു ഹുസൈനില്‍ നിന്നും അബു റജാഅ് ഉദ്ദരിച്ച ഹദീഥ്) മറിച്ച്, അവരെല്ലാം ഉദ്ദരിച്ചിരിക്കുന്നത് ‘നരകവാസികളില്‍ അധികവും സ്ത്രീകളാണ്’ എന്നു മാത്രമാണ്. (സ്വഹീഹുല്‍ ബുഖാരി: 3241, മുസ്‌നദു ഇബ്‌നു ജഅ്ദ്: 3049, മുസ്‌നദുഅഹ്‌മദ്: 19852, മുഅ്ജമുല്‍ കബീര്‍ ത്വബ്‌റാനി: 275) എന്നാല്‍ അബുത്തയ്യാഹ് മാത്രമാണ് ഇംറാനുബ്‌നു ഹുസൈനില്‍ നിന്നും ‘സ്വര്‍ഗവാസികളില്‍ ഏറ്റവും കുറവ് സ്ത്രീകളായിരിക്കും’ എന്ന ഭാഗം നിവേദനം ചെയ്തത്. ‘നരകത്തില്‍ കൂടുതല്‍ സ്ത്രീകളാണ്’ എന്ന ഹദീഥ് കേട്ടപ്പോള്‍ സ്വാഭാവികമായും ‘സ്വര്‍ഗത്തില്‍ കുറവ് സ്ത്രീകളാണെ’ന്ന് അദ്ദേഹം തെറ്റിദ്ധരിച്ചതാണെന്നർത്ഥം. അല്ലെങ്കില്‍ മറ്റു നിവേദകന്മാരും ഇംറാനുബ്‌നു ഹുസൈനില്‍ നിന്ന് അപ്രകാരം തന്നെ റിപ്പോര്‍ട്ട് ചെയ്യുമായിരുന്നു. അവരുടെ റിപ്പോര്‍ട്ടുകളിലൊന്നും തന്നെ ‘നരകത്തില്‍ കൂടുതല്‍ സ്ത്രീകളാണ്’ എന്ന വാചകമല്ലാതെ ‘സ്വര്‍ഗവാസികളില്‍ ഏറ്റവും കുറവ് സ്ത്രീകളായിരിക്കും’ എന്ന പരാമര്‍ശമില്ല. തീര്‍ച്ചയായും ഇത് നിവേദകനായ അബുത്തയ്യാഹിന് സംഭവിച്ച അബദ്ധമാണ്. മാത്രമല്ല സ്വഹീഹ് മുസ്‌ലിമില്‍ ഇംറാനുബ്‌നുഹുസൈനില്‍ നിന്ന് അബുത്തയ്യാഹ് വഴി നിവേദനം ചെയ്യപ്പെട്ട റിപ്പോര്‍ട്ടിലല്ലാതെ, മറ്റു സ്വഹാബികളായ ഇബ്‌നു അബ്ബാസ് (സ്വഹീഹു മുസ്‌ലിം: 5049), ഉസാമത്തിബ്‌നു സൈദ് (സ്വഹീഹു മുസ്‌ലിം: 5048) തുടങ്ങിയവര്‍ വഴിക്കുള്ള നിവേദനങ്ങളിലും ‘നരകത്തില്‍ കൂടുതല്‍ സ്ത്രീകളാണ്’ എന്നതല്ലാതെ ‘സ്വര്‍ഗവാസികളില്‍ കുറവ് സ്ത്രീകളായിരിക്കും’ എന്ന് റിപ്പോര്‍ട്ട് ചെയ്യപ്പെട്ടിട്ടില്ല. ഇതെല്ലാം തന്നെ, ‘സ്വര്‍ഗവാസികളില്‍ ഏറ്റവും കുറവ് സ്ത്രീകളായിരിക്കും’ എന്ന നിവേദനം നിവേദനകന് സംഭവിച്ച അബന്ധമാണെന്നുള്ള പണ്ഡിത വീക്ഷണങ്ങളെ എമ്പാടും സാധൂകരിക്കുന്നതാണെന്നതര്‍ത്ഥം.

ഒന്ന്; അപ്പോള്‍ ‘സ്വര്‍ഗവാസികളില്‍ എണ്ണത്തില്‍ ഏറ്റവും കുറഞ്ഞവര്‍ സ്ത്രീകളാണ്’ എന്ന ഹദീഥ് ഇംറാനുബ്‌നു ഹുസൈനില്‍(റ) നിന്ന് അബുത്തയ്യാഹ് വഴി ഉദ്ദരിക്കപ്പെട്ട നിവേദനത്തില്‍ മാത്രമാണുള്ളത്. ഇംറാനുബ്‌നു ഹുസൈനില്‍(റ) നിന്നു തന്നെ മറ്റു നിവേദകന്മാരാല്‍ ഉദ്ദരിച്ച നിവേദനങ്ങളിലൊന്നും തന്നെ പ്രസ്തുത പരാമര്‍ശമില്ല.

രണ്ട്; ഇംറാബ്‌നു ഹുസൈനല്ലാത്ത മറ്റു സ്വഹാബികളില്‍ നിന്നും ഉദ്ധരിക്കപ്പെട്ട (സ്വഹീഹു മുസ്‌ലിമില്‍ തന്നെ) നിവേദനങ്ങളിലും ‘സ്വര്‍ഗത്തില്‍ കുറവ് സ്ത്രീകളായിരിക്കും’ എന്ന പരാമര്‍ശമില്ല.

മൂന്ന്; സ്വര്‍ഗവാസികളില്‍ സ്ത്രീകളായിരിക്കും അധികമെന്ന അബൂഹുറൈറ(റ)യില്‍ നിന്നും വ്യക്തമായി സൂചിപ്പിക്കുന്ന സ്വഹീഹു മുസ്‌ലിമിലെ തന്നെ (2834) ഹദീഥിന് എതിരാണ് ‘സ്വര്‍ഗത്തില്‍ കുറവ് സ്ത്രീകളാണ്’ എന്ന അബുത്തയ്യാഹ് വഴി നിവേദനം ചെയ്യപ്പെട്ട ഹദീഥ്.

ഈ ചര്‍ച്ചകളെല്ലാം തന്നെ ഇസ്‌ലാമിക ലോകത്ത് നടന്നത് നൂറ്റാണ്ടുകള്‍ക്കു മുമ്പാണെന്നത് നാം പ്രത്യേകം ശ്രദ്ധിക്കുക. ആധുനിക ഇസ്‌ലാമോഫോബിക്കുകളോ, ഫെമിനിസ്റ്റുകളോ, ഇസ്‌ലാംവിമര്‍ശകരോ, നവനാസ്തികരോ ഒന്നും തന്നെ ജന്മംകൊള്ളുകയോ; ഹദീഥുകളില്‍ നിന്നും പെണ്‍വിരുദ്ധത ഉല്‍ഖനനം ചെയ്‌തെടുക്കപ്പെടുന്ന പ്രവണതക്ക് അടയിരിക്കപ്പെടുകയോ ചെയ്തിട്ടില്ലാത്ത ഒരു കാലത്താണ് ഈ ചര്‍ച്ചകള്‍ നടന്നതെന്ന് നാം പ്രത്യേകം മനസ്സിരുത്തി പഠിക്കുക. കാരണം പുതിയ കാലത്താണ് ഇസ്‌ലാമിക പ്രബോധകന്മാര്‍ ഇത്തരം വിശദീകരണങ്ങള്‍ ഹദീഥുകള്‍ക്കു നല്‍കുന്നതെങ്കില്‍ ‘വ്യഖ്യാന ഫാക്റ്ററികള്‍’ എന്നാക്ഷേപിക്കപ്പെടുമായിരുന്നു. എന്നാല്‍ ഹദീഥുകള്‍ക്കുമേല്‍ ഇത്തരം വിമര്‍ശനങ്ങളോ, ചോദ്യം ചെയ്യലുകളോ നടന്നിട്ടില്ലാത്ത ഒരു കാലത്ത് മുസ്‌ലിം പണ്ഡിതന്മാര്‍ ഇപ്രകാരം ചര്‍ച്ച ചെയ്തതിനു പിന്നില്‍ ഒരൊറ്റ കാരണമേ സല്‍ബുദ്ധിയുള്ളവര്‍ക്ക് കണ്ടെത്താനുള്ളത്; ഹദീഥുകള്‍ പെണ്‍വിരുദ്ധമല്ലെന്ന് അവര്‍ എന്നേ മനസ്സിലാക്കിയിരുന്നു എന്നു മാത്രം.

സ്ത്രീകളെ തികഞ്ഞ അശ്ലീലതകള്‍ക്കു പ്രേരിപ്പിക്കുന്ന പല ഉപദേശങ്ങളും ഹദീഥ് ഗ്രന്ഥങ്ങളില്‍, നബിക്കു ലഭിച്ച ദൈവിക വെളിപാടുകളായി ഇന്നും രേഖപ്പെടുത്തപ്പെട്ടു കിടക്കുന്നതു കാണാം. സുഹൈലിന്റെ മകള്‍ സഹ്‌ല എന്ന സ്ത്രീയോട് സാലിം എന്ന പരപുരുഷനെ മുലയൂട്ടി മകനായി സ്വീകരിക്കാന്‍ ആവശ്യപ്പെട്ട ഹദീഫുകള്‍ സ്വഹീഹു മുസ്‌ലിമില്‍ വരെ കാണാം. ഈ പ്രവാചകോപദേശം സ്വീകരിക്കാന്‍ മാന്യതയുള്ള മുസ്‌ലിം സ്ത്രീകള്‍ക്കു സാധിക്കുമോ? മാത്രമല്ല പ്രവാചക പത്‌നി ആഇശ ഇത്തരത്തില്‍ പരപുരുഷന്മാരെ മുലയൂട്ടാറുണ്ടായിരുന്നുവെന്നും പലപ്പോഴും ആഇശയുടെ അരികില്‍ പരപുരുഷന്മാരെ കാണുക വഴി നബിക്കുതന്നെ അതില്‍ അനിഷ്ടമുണ്ടാവുകയും ചെയ്തിരുന്നു എന്ന് സ്വഹീഹ് മുസ്‌ലിമിലെ 1455-ാം ഹദീഥായി രേഖപ്പെടുത്തിയിട്ടുണ്ട്. സ്വയം ചെയ്തിരുന്നു എന്നു മാത്രമല്ല തന്റെ സഹോദര-സഹോദരി പുത്രിമാരോട് അപ്രകാരം ചെയ്യാന്‍ ആഇശ ഉപദേശിക്കുമായിരുന്നു എന്നു കൂടി ഹദീഥ് ഗ്രന്ഥങ്ങളില്‍ രേഖപ്പെടുത്തപ്പെട്ടിട്ടുണ്ട്. മാത്രമല്ല, കല്ലെറിയലിന്റെ വചനവും പത്ത് പ്രാവശ്യമാണ് മുതിര്‍ന്ന പുരുഷന്മാരുടെ മുലകുടിയെന്ന വചനവും (ആദ്യകാലത്ത് ക്വുര്‍ആനില്‍) അവതരിപ്പിക്കപ്പെട്ടിരുന്നെന്നും അതെഴുതിയ രേഖ എന്റെ തലയിണക്കടിയിലുണ്ടായിരുന്നെന്നും ദൈവദൂതന്‍ മരണപ്പെട്ടപ്പോള്‍, ഞങ്ങളെല്ലാം അതുമായി ബന്ധപ്പെട്ട തിരക്കുകള്‍ക്കടിയിലായിരുന്ന സന്ദര്‍ഭത്തില്‍ ഒരു ആട് അകത്ത് കടന്ന് അത് തിന്നുകളഞ്ഞു എന്നും ആഇശയില്‍ നിന്ന് സുനനു ഇബ്‌നുമാജയും മുസ്‌നദ് അഹ്‌മദും ഉദ്ദരിച്ചിട്ടുണ്ട്. സ്ത്രീകളെക്കൊണ്ട് പരപുരുഷന്മാര്‍ക്ക് മുലയൂട്ടുന്ന നടപടിയാണോ ഇസ്‌ലാം പഠിപ്പിച്ച ധാര്‍മ്മികത?

അസത്യങ്ങളും അര്‍ദ്ധസത്യങ്ങളും കോര്‍ത്തിണക്കി നെയ്‌തെടുത്ത ഒരു വ്യാജപ്രചരണമാണിത്. തെറിവിളികള്‍ക്കും പരമതനിന്ദക്കും ‘വിമര്‍ശനം’ എന്നുപേരിട്ട് വൈജ്ഞാനിക രംഗത്ത് കൃത്രിമ മേല്‍വിലാസമുണ്ടാക്കി വെറുപ്പുകച്ചവടം നടത്താനുള്ള കുടില വ്യഗ്രത മാത്രമാണ് പ്രസ്തുത ആരോപണങ്ങള്‍ക്കു പിന്നിലുള്ളത്. ‘ദൈവസ്‌നേഹം’ നെറ്റിയിലൊട്ടിച്ച് പരമത വിദ്വേഷം ഹൃദയത്തില്‍ ഒളിപ്പിച്ചുവെച്ച ചില മിഷണറി നുണ ഫാക്ടറികള്‍ നിര്‍മ്മിച്ച ഈ നബിനിന്ദാ പ്രചരണം, യുക്തിവാദികളും ഫെമിനിസ്റ്റുകളും ഫാഷിസ്റ്റുകളുമെല്ലാം മൊത്തമായും ചില്ലറയായും വിപണനം ചെയ്തുകൊണ്ടിരിക്കുകയാണ്. അതുകൊണ്ടുതന്നെ പ്രസ്തുത ആരോപണം ഇഴയടര്‍ത്തി പരിശോധിക്കേണ്ടിയിരിക്കുന്നു.

അറബികള്‍ക്കിടയില്‍ ബന്ധം സ്ഥാപിതമാകാനുള്ള കാരണങ്ങള്‍ നാലെണ്ണമായിരുന്നു; പ്രസവം, വിവാഹം, മുലകുടി, ദത്തെടുക്കല്‍. ഇതില്‍ ദത്തെടുക്കല്‍, ബന്ധം സ്ഥാപിതമാകാനുള്ള കാരണങ്ങളില്‍ നിന്നും ഇസ്‌ലാം ഒഴിവാക്കിയപ്പോഴാണ് സാലിമിന്റെ(റ) വിഷയത്തിലുള്ള പ്രശ്‌നം ഉണ്ടാകുന്നത്. അഥവാ അബൂഹുദൈഫഃ(റ)യുടെയും സഹ്‌ല(റ)യുടെയും ദത്തുപുത്രനായിരുന്നു സാലിം (റ). ഇസ്‌ലാം ദത്തുപുത്ര സമ്പ്രദായത്തിനു അറുതിവരുത്തിയപ്പോള്‍ ഇന്നലെവരെ തന്റെ മകനായി ജീവിച്ച സാലിം, സഹ്‌ല എന്ന വളര്‍ത്തു മാതാവിന് അന്യപുരുഷനായി മാറി. ഇതില്‍ തീവ്ര ദുഃഖമറിയിച്ചുകൊണ്ട് അവര്‍ പ്രവാചകനെ(സ) സമീപിക്കുകയുണ്ടായി. പ്രസ്തുത ചരിത്ര നിമിഷങ്ങള്‍ ഹദീഥ് ഗ്രന്ഥങ്ങളില്‍ നിന്നും നമുക്കു വായിച്ചെടുക്കാം. ആഇശ (റ) നിവേദനം: സുഹൈലിന്റെ മകള്‍ സഹ്‌ല ഒരിക്കല്‍ നബി(സ)യുടെ അരികില്‍ വന്നുപറഞ്ഞു. ‘അല്ലാഹുവിന്റെ ദൂതരേ, സാലിം എന്റെ അടുത്ത് പ്രവേശിക്കുന്നതില്‍ അബൂഹുദൈഫഃയുടെ (സഹ്‌ലയുടെ ഭര്‍ത്താവാണ് അബൂഹുദൈഫഃ) മുഖത്ത് വെറുപ്പുള്ളതായി തോന്നുന്നു. അപ്പോള്‍ നബി (സ) പറഞ്ഞു: ‘അയാള്‍ക്കു നീ മുലപ്പാല്‍ കൊടുക്കുക’. അവള്‍ ചോദിച്ചു ‘അയാള്‍ വലിയ മനുഷ്യനാണല്ലോ എങ്ങനെ ഞാന്‍ മുലപ്പാല്‍ കൊടുക്കും?!’. അപ്പോള്‍ നബി (സ) പുഞ്ചിരിച്ചുകൊണ്ട് പറഞ്ഞു: ‘അയാള്‍ വലിയ മനുഷ്യനാണെന്നു എനിക്കറിയാം. (സ്വഹീഹു മുസ്‌ലിം 1453)

മുസ്‌ലിമിന്റെ തന്നെ മറ്റൊരു നിവേദനം ഇപ്രകാരമാണ്. ”അബൂഹുദൈഫഃ(റ)യുടെ അടിമ സാലിം (റ), അബൂഹുദൈഫഃയുടെയും കുടുംബത്തിന്റെയും കൂടെ അവരുടെ വീട്ടിലായിരുന്നു താമസം. സുഹൈലിന്റെ മകള്‍ നബി(സ)യുടെ അടുത്ത് വന്നു പറഞ്ഞു: ‘തീര്‍ച്ചയായും സാലിമിന്നു സാധാരണ പുരുഷന്മാരാകുന്ന നിലയില്‍ ബുദ്ധിയും പ്രായവും തികഞ്ഞിട്ടുണ്ട്. അവന്‍ ഞങ്ങളുടെ അടുത്ത് കടന്നുവരാറുണ്ട്. അബൂഹുദൈഫഃയുടെ മനസ്സില്‍ അതില്‍ വെറുപ്പുള്ളതായി എനിക്ക് തോന്നുന്നു’. അപ്പോള്‍ നബി (സ) പറഞ്ഞു: ‘നീ അയാള്‍ക്ക് മുലപ്പാല്‍ കൊടുക്കുക. എന്നാല്‍ അയാള്‍ അടുത്ത ബന്ധുവായിത്തീരും. അബൂഹുദൈഫഃയുടെ മനസ്സില്‍ വെറുപ്പും ഇല്ലാതാകും’. അവള്‍ വീണ്ടും നബി(സ)യുടെ അരികെ വന്നു പറഞ്ഞു: ‘ഞാന്‍ അയാള്‍ക്ക് മുലപ്പാല്‍ കൊടുത്തു. അങ്ങനെ അബൂഹുദൈഫഃയുടെ മനസ്സില്‍ വെറുപ്പ് ഇല്ലാതാകുകയും ചെയ്തു’. (സ്വഹീഹു മുസ്‌ലിം)

ഈ ഹദീഥുകള്‍ മനസ്സിലാക്കിത്തരുന്ന കാര്യങ്ങളെ ഇങ്ങനെ സംഗ്രഹിക്കാം

1) അബൂഹുദൈഫഃ(റ)യുടെ മോചിത അടിമയായ സാലിമി(റ)നെ നന്നെ ചെറുപ്പത്തില്‍ തന്നെ അബൂഹുദൈഫ-സഹ്‌ല ദമ്പതിമാര്‍ ദത്തുപുത്രനായി സ്വീകരിക്കുകയും സ്വപുത്രന്റെ സ്ഥാനം നല്‍കി വളര്‍ത്തുകയും ചെയ്തു. 2) ഇസ്‌ലാം ദത്തുപുത്രസമ്പ്രദായം അവസാനിപ്പിച്ചതോടെ, മതപരമായി സാലിം(റ) ആ കുടുംബത്തിന് മകനല്ലാതായിത്തീരുകയും, വളര്‍ത്തു മാതാവാണെങ്കിലും സഹ്‌ല(റ)യെ സംബന്ധിച്ച് സാലിമി(റ)നുമേല്‍ അന്യപുരുഷന്റെ വിധി ബാധകമാവുകയും ചെയ്തു. 3) വളര്‍ത്തുപുത്രനാണെങ്കിലും മതപരമായി അന്യപുരുഷന്റെ വിധി ബാധകമായ ഒരാള്‍, തന്റെ ഭാര്യയെ സന്ദര്‍ശിക്കുന്നതും അവരുടെ അടുക്കല്‍ പ്രവേശിക്കുന്നതും ഭര്‍ത്താവായ അബൂഹുദൈഫഃ(റ)യില്‍ വെറുപ്പുളവാക്കി. 4) മകന്റെ സ്ഥാനം നല്‍കി താന്‍ വളര്‍ത്തിയ ഒരാളെ ഒരു നിമിഷം കൊണ്ട് അന്യനെപ്പോലെ ഒഴിവാക്കുവാന്‍ സഹ്‌ല(റ)യുടെ മാതൃഹൃദയം അവരെ അനുവദിച്ചില്ല. അതിനാല്‍ തങ്ങളുടെ കാര്യത്തില്‍ ഒരു പോംവഴി തേടിക്കൊണ്ട് നീറുന്ന ഹൃദയവുമായി അവള്‍ പ്രവാചക(സ)ന്റെ അരികിലെത്തി. 5) അവരുടെ പ്രശ്‌നത്തിനു പരിഹാരമായി ‘സാലിമിന് മുലപ്പാല്‍ നല്‍കാനും അങ്ങനെ സ്വപുത്രനെ പോലെ സ്വീകരിക്കുവാനും’ പ്രവാചകന്‍ (സ) അവര്‍ക്ക് ഇളവുനല്‍കി. അതോടെ അബൂഹുദൈഫഃ(റ)യുടെ മനസ്സില്‍നിന്നും ആ വെറുപ്പ് ഇല്ലാതാവുകയും ചെയ്തു.

സാലിം എന്ന മുതിര്‍ന്ന ആണ്‍കുട്ടിക്ക് മുലപ്പാല്‍ നല്‍കി മകനായി സ്വീകരിക്കുവാന്‍ സഹ്‌ലയോട് നിര്‍ദ്ദേശിച്ചതാണ്, പ്രവാചകനെ പെണ്‍വിരുദ്ധനും സ്ത്രീകളെ അശ്ലീലതകള്‍ക്കു പ്രേരിപ്പിച്ച വ്യക്തിത്വമായും മുദ്ര കുത്താന്‍ വിമര്‍ശകരെ പ്രേരിപ്പിക്കുന്ന ഘടകം. വസ്തുതകള്‍ പലതും മറച്ചുപിടിച്ചുകൊണ്ടാണ് ഈ വിമര്‍ശനം നെയ്‌തെടുത്തിരിക്കുന്നത്. ‘മുലപ്പാല്‍ നല്‍കുക’ എന്നു പറഞ്ഞതിനെ ‘പാല്‍ കുടിക്കാന്‍ സ്തനം നല്‍കുക’ എന്ന്, അശ്ലീല ഭാവനകള്‍ കൊണ്ട് ലൈംഗിക ഛായം പൂശുകയാണ് വാസ്തവത്തില്‍ വിമര്‍ശകന്മാര്‍ ചെയ്തിരിക്കുന്നത്. ‘ബാബു രിദ്വാഅത്തില്‍ കബീരി’ അഥവാ വലിയവരുടെ രിദ്വാഅഃ (മുലപ്പാല്‍ കുടിക്കല്‍) എന്ന അദ്ധ്യായങ്ങള്‍ക്കു കീഴിലാണ് പ്രസ്തുത ഹദീഥുകല്‍ ഉദ്ദരിക്കപ്പെട്ടിരിക്കുന്നത്. രിദ്വാഅഃ (الرضاعة) എന്ന പദം സ്ത്രീയുടെ മുലപ്പാല്‍ സ്തനത്തില്‍ നിന്ന് നേരിട്ട് ഈമ്പിക്കുടിക്കുന്ന രീതിക്കു മാത്രമാണോ അറബി ഭാഷയില്‍ ഉപയോഗിക്കുന്നത്? അല്ല എന്നതാണ് വസ്തുത. പക്ഷേ ആ വസ്തുതകളൊന്നും കാണാന്‍ വിമര്‍ശകര്‍ തയ്യാറല്ല. തയ്യാറായാല്‍ ഈ അശ്ലീല ഭാവനകള്‍ക്കൊന്നും നിലനില്‍പ്പുണ്ടാവില്ലെന്ന് അവര്‍ക്ക് നല്ലപോലെ അറിയാം.

എന്താണ് രിദ്വാഅഃ? (الرضاعة)

ഒരു സ്ത്രീയുടെ മുലപ്പാല്‍ മറ്റൊരാളുടെ അല്ലെങ്കില്‍ കുഞ്ഞിന്റെ വയറ്റില്‍ എത്തുക എന്നതിനെയാണ് ഇര്‍ദ്വാഅ് (الارضاع), രിദ്വാഅഃ (الرضاعة) എന്നൊക്കെ പറയുന്നത്. അതാകട്ടെ പല രീതിയിലും നടക്കാം. സ്ത്രീയുടെ മുലപ്പാല്‍ സ്തനത്തില്‍ നിന്ന് നേരിട്ട് ഈമ്പിക്കുടിക്കുന്ന രീതി അഥവാ ‘മസ്വ്’ (المص), സ്തനം സ്പര്‍ശിക്കാതെ കുട്ടിയുടെ വായിലേക്ക് പാല്‍ ചുരന്ന് ഒഴുക്കുന്ന രീതി അഥവാ ‘സ്വബ്ബ്’ (الصب), സ്തനം സ്പര്‍ശിക്കാതെ സ്ത്രീയുടെ മുലപ്പാല്‍ പിഴിഞ്ഞെടുത്ത് തളികയിലാക്കി കുട്ടിയുടെ തൊണ്ടയിലേക്ക് ചൊരിയുന്ന രീതി അഥവാ ‘വുജൂര്‍’ (الوجور), സ്തനം സ്പര്‍ശിക്കാതെ സ്ത്രീയുടെ മുലപ്പാല്‍ പിഴിഞ്ഞെടുത്ത് തളികയിലാക്കി കുട്ടിയുടെ നാസദ്വാരത്തില്‍ ഒഴിക്കുന്ന രീതി അഥവാ ‘സുഊത്വ്’ (السعوط), സ്തനം സ്പര്‍ശിക്കാതെ സ്ത്രീയുടെ മുലപ്പാല്‍ പൈപ്പിലൂടെയോ, സിറിഞ്ചിലൂടെയോ കുട്ടിക്കു നല്‍കുന്ന രീതി അഥവാ ‘ഹിക്‌നത്ത്’ (الحقنة). ഈ ഏത് രീതിയിലും മുലപ്പാല്‍ നല്‍കുന്നതിനും ഇസ്‌ലാമിലെ സാങ്കേതിക പദവും കര്‍മ്മശാസ്ത സംജ്ഞ്യയുമായ ‘രിദ്വാഅഃ’ എന്ന് പറയുമെന്ന് പൗരാണികരായ അറബി ഭാഷാ പണ്ഡിതരും സര്‍വ്വ മദ്ഹബുകളിലും പെട്ട കര്‍മ്മശാസ്ത്ര വിശാരദന്മാരും വ്യക്തമാക്കിയിട്ടുണ്ട്. (മുഖ്തസറു ഖലീല്‍: 162, മവാഹിബുല്‍ ജലീല്‍: 4/178, ശര്‍ഹുല്‍ കബീര്‍: 2/502, മന്‍ഹുല്‍ ജലീല്‍: 4/371, അല്‍ ഫവാകിഹുദ്ദവാനി: 2/54, മുഗ്നി അല്‍ മുഹ്താജ്: 3/414, ഫത്ഹുല്‍ വഹാബ്: 2/194, അസ്സിറാജുല്‍ വഹ്ഹാജ്: 460, അല്‍ മുബ്ദിഅ് 8/160, ശര്‍ഹു മുന്‍തഹല്‍ ഇറാദത്ത്: 3/213, കശ്ശാഫുല്‍ കനാഅ്- 5: 442, മതാലിബു ഉലിന്നുഹ- 5: 596, അല്‍ ബഹ്‌റു റാഇഖ്- 3: 238, മജ്മഉല്‍ അന്‍ഹുര്‍ : 1/551) (ഉദ്ദരണം: ഡോ. രിയാദ് മിശ്അലിന്റെ ലേഖനത്തില്‍ നിന്ന് Midad Al-Adab Refereed Quarterly journal : Vol.1 : Issue 1 : Article 10, ഇറാക് സര്‍വ്വകലാശാല)

പൗരാണിക ഭാഷാ പണ്ഡിതനായ ഇബ്‌നു മന്‍ളൂര്‍ (ജനനം: ഹിജ്‌റ 630) തന്റെ അറബി നിഘണ്ഡുവായ ‘ലിസാനുല്‍ അറബി’ല്‍ (2/92) പറഞ്ഞു: ”സ്തനത്തില്‍ നിന്ന് നേരിട്ട് മുലയൂട്ടുന്നത് വിവാഹബന്ധം നിഷിദ്ധമാക്കുന്നത് പോലെ തന്നെ സ്ത്രീയുടെ സ്തനത്തില്‍ നിന്ന് നേരിട്ടല്ലാതെ പാല്‍ ചുരന്നെടുത്ത് കുട്ടിയെ കുടിപ്പിക്കുന്നതും (രിദ്വാഅഃ ആയതിനാല്‍) വിവാഹബന്ധം നിഷിദ്ധമാക്കുന്നതാണ്. സ്തനത്തില്‍ നിന്ന് വേറിട്ടതിനാല്‍ അത് രിദ്വാഅഃ ആകാതിരിക്കുന്നില്ല.”

അപ്പോള്‍ ഭാഷാപരമായി പോലും രിദ്വാഅഃ എന്നു പറഞ്ഞാല്‍, സ്ത്രീയുടെ മുലപ്പാല്‍ സ്തനത്തില്‍ നിന്ന് നേരിട്ട് ഈമ്പി കുടിക്കുന്ന രീതിയാകണം എന്നില്ല. പ്രവാചകന്‍ (സ) സഹ്‌ലയോട് പറഞ്ഞതാകട്ടെ ‘അവന് മുലപ്പാല്‍ നല്‍കുക’ അര്‍ദ്വഈഹി (ارضعيه) എന്നാണ്. അല്ലാതെ നിന്റെ സ്തനം അവന് കുടിക്കാന്‍ കൊടുക്കുക എന്നല്ല. സഹ്‌ല-സാലിം സംഭവം കേവലം ഇസ്‌ലാംവിരോധികളുടെ മ്ലേഛമായ ഭാവനാവിലാസങ്ങള്‍ക്കപ്പുറം, യാതൊരു ആധികാരികതയുമില്ലാത്ത ദുരാരോപണം മാത്രമാണെന്ന് വ്യക്തമാക്കുന്ന തെളിവുകളാണ്, പൂര്‍വ്വകാലം മുതല്‍ മുസ്‌ലിം പണ്ഡിതന്മാര്‍ പ്രസ്തുത ഹദീഥുകള്‍ക്കു നല്‍കിയ വ്യാഖ്യാനം. അതിലൂടെ നമുക്കൊന്ന് കണ്ണോടിക്കാം.

ഇമാം ഇബ്‌നു കുതൈബ (ജനനം: ഹിജ്‌റ: 213) പറഞ്ഞു: ”പ്രവാചകന്‍ (സ) സഹ്‌ലയോട് പറഞ്ഞത് ‘അവന് മുലപ്പാല്‍ നല്‍കുക'(ارضعيه) എന്നാണ്. നിന്റെ സ്തനം അവന് കുടിക്കാന്‍ കൊടുക്കുക എന്ന് (പ്രവാചകന്‍) ഊദ്ദേശിച്ചിട്ടേ ഇല്ല. നിന്റെ പാല്‍ ചുരന്നെടുത്ത് തളികയിലാക്കി കുറച്ച് അവനെ കുടിപ്പിക്കുക എന്നേ അദ്ദേഹം ഉദ്ദേശിച്ചിട്ടുള്ളൂ. അതല്ലാതെ സ്തനത്തില്‍ നിന്ന് നേരിട്ട് കുടിക്കല്‍ അനുവദനീയമല്ല. കാരണം സഹ്‌ലയുടെ സ്തനത്തിലേക്ക് നോക്കുന്നത് തന്നെ സാലിമിന് നിഷിദ്ധമാണ്. പിന്നെയെങ്ങനെ നിഷിദ്ധമായ ഒരു കാര്യം പ്രവാചകന്‍ അദ്ദേഹത്തിന് അനുവദിച്ച് കൊടുക്കും!” (തഅ്‌വീലു മുക്തലിഫില്‍ ഹദീഥ്: 1/437) ഇതേ വ്യാഖ്യാനം തന്നെ ഇമാം ക്വാള്വീ ഇയാള്വും (ജനനം: ഹിജ്‌റ: 476) ആവര്‍ത്തിക്കുന്നതു കാണാം (ശര്‍ഹുന്നവവി അലാ സ്വഹീഹു മുസ്‌ലിം : 10/30,31)

ഇമാം മാവര്‍ദി (ജനനം : ഹിജ്‌റ : 364) പറയുന്നു: ”സാലിമിന്റെ വിഷയത്തില്‍ സ്തനത്തില്‍ നിന്ന് മുലപ്പാല്‍ ഊട്ടിയെന്ന പ്രസ്താവന ഒന്നും തന്നെയില്ല എന്നത് സുവിദിതമാണ്. അതുകൊണ്ട് തന്നെ പ്രവാചകന്‍ ഉദ്ദേശിച്ചത് ‘വുജൂര്‍’ (الوجور സ്തനം സ്പര്‍ശിക്കാതെ സ്ത്രീയുടെ മുലപ്പാല്‍ പിഴിഞ്ഞെടുത്ത് തളികയിലാക്കി കുട്ടിയുടെ തൊണ്ടയില്‍ ചൊരിയുന്ന രീതി) ആണെന്ന് സ്ഥാപിതമാകുന്നു” (അല്‍ ഹാവി അല്‍അല്‍ കബീര്‍ : 11/372)

ഇബ്‌നു അബ്ദുല്‍ ബിര്‍റ് (ജനനം: ഹിജ്‌റ: 368) പറഞ്ഞു: ”മുതിര്‍ന്നവരുടെ മുലകുടി എന്നാല്‍ പാല്‍ പാത്രത്തില്‍ ചുരന്ന് അതില്‍ നിന്ന് കുടിപ്പിക്കലാണ്. എന്നാല്‍ കുട്ടികള്‍ക്ക് നല്‍കുന്നതുപോലെ സ്തനത്തില്‍ നിന്ന് നേരിട്ട് കുടിപ്പിക്കല്‍ മുതിര്‍ന്നവരുടെ കാര്യത്തില്‍ നിഷിദ്ധമാണ് എന്ന് പണ്ഡിതസംഘം വ്യക്തമാക്കിയിട്ടുണ്ട്” (അത്തംഹീദ്: 8/257, അല്‍ ഇസ്തിദ്കാര്‍: 6/255)

ആധുനിക കാലഘട്ടത്തിലെ മുസ്‌ലിം പണ്ഡിതന്മാര്‍ ‘മുലയൂട്ടല്‍’ വിവാദവുമായി ബന്ധപ്പെട്ട ആരോപണങ്ങള്‍ക്ക് മറുപടി പറയുമ്പോള്‍ അതെല്ലാം ‘വ്യാഖ്യാന ഫാക്ടറികള്‍’ എന്നു പറഞ്ഞ് ഒഴിഞ്ഞുമാറുന്ന ഇസ്‌ലാംവിരോധികള്‍ക്ക്, നൂറ്റാണ്ടുകള്‍ക്കു മുമ്പേ പൂര്‍വ്വകാല മുസ്‌ലിം പണ്ഡിതന്മാര്‍ പ്രസ്തുത വിഷയത്തില്‍ നല്‍കിയ ഈ മറുപടികളെപ്പറ്റി എന്തു പറയാനുണ്ട്?!. മിഷണറി നുണഫാക്ടറികളില്‍ ‘മുലയൂട്ടല്‍’ വിവാദം നിര്‍മ്മിച്ചെടുക്കപ്പെടുന്നതിനും എത്രയോ നൂറ്റാണ്ടുകള്‍ക്ക് മുമ്പാണ് ഇമാം ഇബ്‌നു കുതൈബയും, ഇമാം ക്വാള്വീ ഇയാള്വും, ഇമാം മാവര്‍ദിയും, ഇബ്‌നു അബ്ദുല്‍ ബിര്‍റുമെക്കെ ഈ വിവാദങ്ങള്‍ക്ക് മറുപടി പറഞ്ഞതെന്ന വസ്തുതയെങ്കിലും ഇസ്‌ലാം വിരോധം കൊണ്ട് അന്ധത ബാധിച്ചവരുടെ കണ്ണുകള്‍ തുറപ്പിച്ചിരുന്നെങ്കില്‍!! പണ്ഡിത വ്യാഖ്യാനങ്ങള്‍ മാത്രമല്ല ചരിത്ര രേഖകളും പ്രസ്തുത ‘മുലപ്പാല്‍ നല്‍കല്‍’ അപ്രകാരമായിരുന്നെന്ന് വ്യക്തമാക്കുന്നുണ്ട്. ‘ഒരു പാത്രത്തില്‍ മുലപ്പാല്‍ ചുരന്നൊഴിച്ച് എല്ലാ ദിവസവും സാലിം കുടിച്ചു കൊണ്ടിരുന്നുവെന്നും അതിനു ശേഷമാണ് സഹ്‌ലയുടെ വീട്ടില്‍ പ്രവേശിച്ചത്’ എന്നും ചില ചരിത്ര നിവേദനങ്ങള്‍ കാണാം. (ത്വബകാത്തു ഇബ്‌നു സഅ്ദ്: 8/271, ശര്‍ഹു സര്‍ക്കാനി : 3/316)

അശ്ലീല ഭാവനകള്‍ കൊണ്ട് പ്രവാചക ജീവിതത്തിനുമേല്‍ ലൈംഗിക ഛായം പൂശുവാന്‍ വെമ്പല്‍ കൊള്ളുന്ന ഇസ്‌ലാംവിരോധികള്‍ നബി(സ)യില്‍ നിന്നും നിവേദനം ചെയ്യപ്പെട്ട ഈ ഹദീഥെങ്കിലും ഒന്നു പരിഗണിച്ചിരുന്നെങ്കില്‍ അവരുടെ വായകൊണ്ട് ഇത്ര ഗുരുതരമായ അപവാദം പറയാന്‍ ഒന്നു മടിക്കുമായിരുന്നു. നബി(സ) പറഞ്ഞു: ”നിന്റെ തലയില്‍ ഇരുമ്പിന്റെ സൂചികൊണ്ട് കുത്തി തറക്കുന്നതാണ് അനുവദനീയമല്ലാത്ത ഒരു സ്ത്രീയെ സ്പര്‍ശിക്കുന്നതിനേക്കാള്‍ നിനക്ക് നല്ലത്” (അല്‍ മുഅ്ജമുല്‍ കബീര്‍: ത്വബ്‌റാനി: 20:211) അന്യസ്ത്രീകളെ സ്പര്‍ശിക്കുന്നതിനേക്കാള്‍ നല്ലത് തലയില്‍ ഇരുമ്പുസൂചി കുത്തി തറക്കുന്നതാണെന്നു പഠിപ്പിച്ച ഒരു വിശുദ്ധ വ്യക്തിത്വത്തിനുമേല്‍, പരപുരുഷന്മാരെ മുലയൂട്ടാന്‍ സ്ത്രീകളെ പ്രേരിപ്പിച്ചെന്ന അപവാദം പറഞ്ഞു പ്രചരിപ്പിച്ചവര്‍ ദൈവത്തിന്റെ ‘കുഞ്ഞാടാ’വാന്‍ എന്തുകൊണ്ടും യോഗ്യരാണ്?!.

വിമര്‍ശകരുടെ കാപട്യം

വിമര്‍ശനവിധേയമായ ഹദീഥുകളുമായി ബന്ധപ്പെട്ട് മ്ലേച്ഛഭാവനകളുടെ മേച്ചില്‍പ്പുറങ്ങള്‍ തേടിയിറങ്ങിയ ഇസ്‌ലാംവിരോധികള്‍ ബോധപൂര്‍വ്വം മറച്ചുപിടിച്ച ചില വസ്തുതകള്‍ കൂടി നാം തിരിച്ചറിയേണ്ടിയിരിക്കുന്നു. 1) ഹദീഥുകളില്‍ പരാമര്‍ശിക്കപ്പെട്ട ഈ ഇളവ് കേവലം സാലിമിന്റെ വിഷയത്തില്‍ മാത്രം പരിമിതമാണ്. അതൊരു പൊതുവിധിയല്ലെന്നര്‍ത്ഥം. അതിനാല്‍ സഹ്‌ലയല്ലാത്ത ഒരു മുസ്‌ലിം സ്ത്രീക്കും ആ ഇളവ് ബാധകമല്ല. ‘മുഹമ്മദ് നബി(സ)യുടെ ഈ അദ്ധ്യാപനം പിന്‍പറ്റാന്‍ മാന്യതയുള്ള മുസ്‌ലിം സ്ത്രീകള്‍ക്കു സാധിക്കുമോ’ എന്നു ചോദിക്കുന്നവരുടെ ആശങ്കകള്‍ അടിസ്ഥാന രഹിതമാണ്. സ്വഹീഹു മുസ്‌ലിമില്‍ തന്നെ ആ ഇളവ് സാലിമിന് മാത്രമുള്ളതാണെന്ന് വ്യക്തമാക്കുന്ന നിവേദം കാണാവുന്നതാണ്. പ്രവാചക പത്‌നി ഉമ്മുസലമ (റ) പറയാറുണ്ടായിരുന്നു: ”ആ നിലക്ക് മുലകുടി ബന്ധത്തിലുള്ള (രണ്ടു വയസ്സിനുള്ളില്‍ മുലകുടി ബന്ധം സ്ഥാപിതമായിട്ടില്ലാത്ത) ഒരാളും നബി (സ) യുടെ ഭാര്യമാരുടെ അരികെ പ്രവേശിക്കുന്നതിനെ അവര്‍ വിസമ്മതിച്ചിരുന്നു. അവര്‍ ആഇശ(റ)യോട് പറഞ്ഞു: ‘ഇത് സാലിമിന്നു മാത്രമായി റസൂല്‍ (സ) പറഞ്ഞ ഒരു ഇളവാകുന്നു. അതിനാല്‍ ആ നിലക്ക് മുലകുടി ബന്ധമുള്ള ഒരാളും നമ്മുടെ അടുത്ത് പ്രവേശിക്കേണ്ടതില്ല. നമ്മളെ കാണുകയും ചെയ്യേണ്ട.” (സ്വഹീഹു മുസ്‌ലിം: 1454, മുസ്‌നദു അഹ്‌മദ്: 26660, സുനനു അബൂദാവൂദ്: 2061)

2) തന്റെ ഭാര്യയുടെ അടുത്ത് വളര്‍ത്തുപുത്രന്‍ സാലിം പ്രവേശിക്കുന്നത്- ദത്തുപുത്ര സമ്പ്രദായം അവസാനിപ്പിക്കപ്പെട്ടതോടെ- മതപരമായി അനുവദനീയമല്ലാതായി. അതുകൊണ്ടാണ് അബൂഹുദൈഫഃക്ക് സാലിമിന്റെ പ്രവേശനം നീരസമുള്ളതായി മാറിയത്. ആ നീരസം മാറാന്‍ വേണ്ടിയാണ് മുലപ്പാല്‍ നല്‍കി മാതൃബന്ധം സ്ഥാപിക്കുവാന്‍ പ്രവാചകന്‍ (സ) ഉപദേശിച്ചത്. അതേസമയം സ്തനങ്ങളില്‍ നിന്ന് നേരിട്ട് കുടിക്കാനായിരുന്നു നബി(സ)യുടെ ഉപദേശമെങ്കില്‍ അത് അബൂഹുദൈഫഃയുടെ നീരസവും മനഃപ്രയാസവും അധികരിപ്പിക്കുകയാണുണ്ടാവുക. ഈ വസ്തുത വിമര്‍ശകര്‍ ബോധപൂര്‍വ്വം മറച്ചുപിടിക്കുകയാണ്. പ്രവേശനം തന്നെ നീരസവും മനഃപ്രയാസവും സൃഷ്ടിക്കുന്ന ഒരാളെ കൊണ്ട് തന്നെ, സ്തനങ്ങളില്‍ നിന്ന് നേരിട്ട് മുലപ്പാല്‍ കുടിക്കുവാന്‍ കല്‍പ്പിക്കില്ലെന്ന് ഏതു വിഡ്ഢിക്കാണ് മനസ്സിലാകാതിരിക്കുക?!

‘മുലയൂട്ടല്‍’ ആഇശ(റ)യുടെ പേരിലെ പച്ചക്കള്ളങ്ങള്‍

അശ്ലീല ഭാവനകള്‍ കൊണ്ട് ഹദീഥുകളെ ദുഷിപ്പിച്ചവതരിപ്പിക്കുക മാത്രമല്ല, പച്ചകള്ളങ്ങള്‍ പോലും ഇസ്‌ലാമിനെതിരെ പടച്ചുണ്ടാക്കാന്‍ വരെ മടിയില്ലാതായിരിക്കുന്നു ഇസ്‌ലാംവിമര്‍ശകര്‍ക്ക്. ലജ്ജാവഹവും നിന്ദാപരവുമായ ഇത്തരം ഏര്‍പ്പാടുകളെ ‘സുവിശേഷ വേല’ എന്നുപേരിട്ടു വിളിക്കുന്നത് അല്പം കടന്ന കയ്യാണ്. ‘മുലയൂട്ടല്‍’ വിവാദവുമായി ബന്ധപ്പെട്ട് പ്രവാചക പത്‌നി ആഇശ(റ)ക്കെതിരെ മിഷണറി നുണഫാക്ടറികള്‍ ഉല്‍പ്പാദിപ്പിച്ച അത്തരം രണ്ടു കളവുകള്‍ നാം കാണുക.

‘പ്രവാചക പത്‌നി ആഇശ (റ) അത്തരത്തില്‍ പരപുരുഷന്മാരെ മുലയൂട്ടാറുണ്ടായിരുന്നുവെന്നും പലപ്പോഴും ആഇശയുടെ അരികില്‍ പരപുരുഷന്മാരെ കാണുക വഴി നബി(സ)ക്കുതന്നെ അതില്‍ അനിഷ്ടമുണ്ടാവുകയും ചെയ്തിരുന്നു എന്ന് സ്വഹീഹു മുസ്‌ലിമിലെ 1455-ാം ഹദീഥായി രേഖപ്പെടുത്തിയിട്ടുണ്ട്’. ഇതാണ് സോഷ്യല്‍ മീഡിയ വഴി ചില മിഷണറി നുണ ഫാക്ടറികള്‍ നിരന്തരം പ്രചരിപ്പിച്ചതും, യുക്തിവാദികളും ഫെമിനിസ്റ്റുകളും ഫാഷിസ്റ്റുകളുമടങ്ങുന്ന ഇസ്‌ലാംവിരോധികള്‍ ഏറ്റെടുത്തതുമായ ഒന്നാമത്തെ കളവ്. സ്വഹീഹു മുസ്‌ലിമിലെ ആ നിവേദനം നാം ആദ്യം കാണുക. ”മസ്‌റൂഖ് (റ) നിവേദനം. ആഇശ (റ) പറഞ്ഞതായി അദ്ദേഹം പറഞ്ഞു: റസൂല്‍ (സ) എന്റെ അടുത്ത് കടന്നുവന്നു. എന്റെ അരികെ ഒരാള്‍ ഇരിക്കുന്നുണ്ടായിരുന്നു? അത് നബി(സ)ക്ക് അതിയായ പ്രയാസമുണ്ടാക്കി. നബി(സ)യുടെ മുഖത്ത് ഞാന്‍ കോപം കണ്ടു. അപ്പോള്‍ ഞാന്‍ ചോദിച്ചു: ‘അല്ലാഹുവിന്റെ പ്രവാചകരെ, നിശ്ചയം ഇദ്ദേഹം മുലകുടി ബന്ധത്തില്‍ എന്റെ സഹോദരനാകുന്നു’. അപ്പോള്‍ നബി (സ) പറഞ്ഞു: നിങ്ങള്‍ മുലകുടി ബന്ധത്തിലുള്ള സഹോദരന്മാരെപ്പറ്റി ശരിക്കും നോക്കണം. വിശപ്പടങ്ങുന്ന നിലക്ക് മുല കുടിച്ചാല്‍ മാത്രമെ മുലകുടി ബന്ധം ഉണ്ടാവുകയുള്ളൂ”. (സ്വഹീഹു മുസ്‌ലിം: 1455) ഈ സംഭവത്തെ ദുര്‍വ്യാഖ്യാനിച്ചുകൊണ്ടാണ് ആഇശ (റ) പരപുരുഷന്മാരെ മുലയൂട്ടിയിരുന്നെന്ന് പച്ചകള്ളം ഇവര്‍ പ്രചരിപ്പിച്ചത്. ഹദീഥുകള്‍ നേര്‍ക്കുനേരെ ഒരാവര്‍ത്തി വായിച്ചാല്‍ തന്നെ അതൊരു ദുരാരോപണം മാത്രമാണെന്ന് മനസ്സിലാക്കാന്‍ കഴിയും. ‘അല്ലാഹുവിന്റെ പ്രവാചകരേ, നിശ്ചയം ഇദ്ദേഹം മുലകുടി ബന്ധത്തില്‍ എന്റെ സഹോദരനാകുന്നു’ എന്ന ആഇശ(റ)യുടെ വാക്കുകള്‍ സ്പഷ്ടമായും മനസ്സിലാക്കി തരുന്നത്; ആഇശ(റ)യും അവരുടെ സമീപത്തിരുന്ന വ്യക്തിയും കുഞ്ഞായിരുന്നപ്പോള്‍ ഒരേ സ്ത്രീയുടെ മുലപ്പാല്‍ കുടിച്ചിട്ടുണ്ട് എന്നതാണ്. അല്ലാതെ ആഇശ (റ) അദ്ദേഹത്തെ ‘മുലയൂട്ടി’ എന്നല്ല. ഒന്നുകില്‍ ആഇശ (റ) കൈകുഞ്ഞായിരിക്കുമ്പോള്‍ അദ്ദേഹത്തിന്റെ മാതാവിന്റെ മുലപ്പാല്‍ കുടിച്ചിട്ടുണ്ടാകും. അല്ലെങ്കില്‍ അദ്ദേഹം ശൈശവ ഘട്ടത്തില്‍ ആഇശ(റ)യുടെ മാതാവിന്റെ പാല്‍കുടിച്ചതുമാകാം. ഇതുരണ്ടുമല്ലെങ്കില്‍ അവര്‍ രണ്ടുപേരും മറ്റേതോ ഒരു സ്ത്രീയുടെ പാല്‍കുടിച്ചിട്ടുണ്ടാകാം. അതുകൊണ്ടാണ് ആഇശ(റ)ക്കും അദ്ദേഹത്തിനുമിടയില്‍ സഹോദര ബന്ധം സ്ഥാപിതമായത്. ഈ സംഭവത്തെ ദുര്‍വ്യാഖ്യാനിച്ചുകൊണ്ടാണ് ഈ ദുരാരോപണം ഇസ്‌ലാംവിരോധികള്‍ പ്രചരിപ്പിക്കുന്നത്. ആഇശ (റ) പ്രസവിക്കാത്ത ഒരു സ്ത്രീയായിരുന്നു. അതുകൊണ്ടുതന്നെ അവര്‍ക്ക് ഒരു കുഞ്ഞിനും മുലയൂട്ടാന്‍ സാധ്യവുമല്ല. ഈ വസ്തുതകളെല്ലാം അറിഞ്ഞിട്ടും വിമര്‍ശകര്‍ ഇത്തരം കള്ളപ്രചരണം നടത്തിക്കൊണ്ടിരിക്കുന്നത്, ഇസ്‌ലാംവിരോധം അവരുടെ മനഃസാക്ഷിയെപോലും കൊന്നുകളഞ്ഞിരിക്കുന്നു എന്നതിന്റെ സൂചനയാണ്. എങ്കില്‍പിന്നെ പ്രവാചകന്‍ കോപിച്ചതെന്തിനാണ്? അതിന്റെ ഉത്തരവും ഹദീഥിലുണ്ട്. ‘നിങ്ങള്‍ മുലകുടി ബന്ധത്തിലുള്ള സഹോദരന്മാരെപ്പറ്റി ശരിക്കും നോക്കണം. വിശപ്പടങ്ങുന്ന നിലക്ക് മുലകുടിച്ചാല്‍ മാത്രമേ മുലകുടി ബന്ധം ഉണ്ടാവുകയുള്ളു’ എന്ന നബി(സ)യുടെ വിശദീകരണം അതിന്റെ മറുപടിയാണ്. മുലപ്പാല്‍ കുടിക്കുന്നതിലൂടെ വിശപ്പടങ്ങുന്ന പ്രായത്തിലുള്ള മുലകുടിയെ മാത്രമേ ഇസ്‌ലാം – വിവാഹബന്ധം നിഷിദ്ധമായ- സഹോദരബന്ധമായി പരിഗണിക്കുകയുള്ളൂ. അത് രണ്ടു വയസ്സിനുള്ളില്‍ അഞ്ച് തവണയെങ്കിലും നടന്നിരിക്കണം. മുലകുടിയുമായി ബന്ധപ്പെട്ട ഈ നിബന്ധനകള്‍ എല്ലാം യോജിക്കുന്നതാണോ നിങ്ങള്‍ക്കിടയിലുള്ള സഹോദര ബന്ധങ്ങള്‍ എന്ന് നിങ്ങള്‍ ഉറപ്പു വരുത്തണമെന്ന കാര്‍ക്കശ്യമാണ് പ്രവാചക(സ)ന്റെ കോപത്തിനു പിന്നിലെ കാരണം. അല്ലാതെ ആഇശ(റ)യുടെ സമീപത്ത് അവരുടെ മുലകുടി ബന്ധത്തിലെ സഹോദരനെ കണ്ടപ്പോള്‍ എന്തെങ്കിലും സംശയം ഹൃദയത്തില്‍ വന്നുപോയതുകൊണ്ടല്ല. കാരണം ആഇശ (റ) അത്തരം ബന്ധുക്കള്‍ (മുലകുടിയിലൂടെയുള്ള ബന്ധുക്കള്‍) അവരെ സന്ദര്‍ശിക്കാന്‍ അനുവാദം തേടുമ്പോള്‍ വളരെ സൂക്ഷ്മതയോടെ മാത്രമേ അനുവാദം നല്‍കാറുള്ളൂ എന്ന് നബി(സ)ക്ക് അറിയാമായിരുന്നു. ഒരു സംഭവം നാം കാണുക: ”ആഇശ (റ) പറഞ്ഞു: മുലകുടി ബന്ധത്തിലൂടെയുള്ള എന്റെ പിതാമഹന്‍ എന്റെ അടുക്കല്‍ (വീട്ടില്‍)കയറാന്‍ സമ്മതം ചോദിച്ചു. അപ്പോള്‍ അല്ലാഹുവിന്റെ ദൂതരുടെ സമ്മതം കിട്ടാത്തതിനാല്‍ ഞാന്‍ വിസ്സമ്മതിച്ചു. അപ്പോള്‍ അല്ലാഹുവിന്റെ ദൂതന്‍ (സ) പറഞ്ഞു: നിന്റെ പിതാമഹനെ നീ അടുത്ത് പ്രവേശിപ്പിച്ചുകൊള്ളുക…” (സ്വഹീഹു മുസ്‌ലിം: 1445)

വിവാഹബന്ധം നിഷിദ്ധമായവര്‍ മാത്രമേ തന്റെ വീട്ടില്‍ പ്രവേശിക്കാവൂ എന്ന കാര്യത്തില്‍ അതീവ ശ്രദ്ധ പുലര്‍ത്തിയിരുന്ന ഒരു വ്യക്തിത്വമായിരുന്നു ആഇശ (റ). അല്ലാത്ത പുരഷന്മാരെ മറക്കു പിറകില്‍ നിന്നുകൊണ്ടല്ലാതെ അവര്‍ അഭിസംബോധന ചെയ്തിട്ടില്ല. അത്തരം ഒരു വ്യക്തിയെ പറ്റി ഇത്രമാത്രം ദുഷിച്ച കള്ളകഥകള്‍ പ്രചരിപ്പിക്കുവാന്‍ മിഷണറിമാര്‍ക്കല്ലാതെ മറ്റാര്‍ക്കു കഴിയും. ഇതിനെയെല്ലാം നാം ‘സുവിശേഷ വേല’ എന്നുതന്നെ വിളിക്കണോ?!. ദുഷിച്ച ഭാവനകള്‍കൊണ്ട് ഹദീഥുകളില്‍ നിന്നും അശ്ലീലതകള്‍ മെനഞ്ഞെടുത്ത ‘സുവിശേഷ വേല’യാണ് നാം മുകളില്‍ കണ്ടതെങ്കില്‍, വ്യാജതൂലികകള്‍ കൊണ്ട് ഹദീഥുകളില്‍ നേര്‍ക്കുനേരെ കൃത്രിമത്വം നടത്തുന്ന ‘സുവിശേഷ വേല’യാണ് ഇനി കാണാനുള്ളത്. വിഷയം ഒരു ദുര്‍ബലമായ ഹദീഥാണ്. മുസ്‌ലിംകള്‍ പ്രമാണമായി കാണാത്ത ഒരു വാറോല. പക്ഷെ അതില്‍പ്പോലും പച്ചയായ കാപട്യം കാണിക്കുവാന്‍ മിഷണറികള്‍ക്ക് മടിയില്ലെന്നതാണ് അതിശയകരമായ സംഗതി.

‘കല്ലെറിയലിന്റെ വചനവും പത്ത് പ്രാവശ്യമാണ് മുതിര്‍ന്ന പുരുഷന്മാര്‍ക്ക് മുലകുടിയെന്ന വചനവും (ആദ്യകാലത്ത് കുര്‍ആനില്‍) അവതരിപ്പിക്കപ്പെട്ടിരുന്നുവെന്നും, അതെഴുതിയ രേഖ തന്റെ തലയിണക്കടയില്‍ സൂക്ഷിച്ചുവെച്ചിരുന്നെന്നും പ്രവാചകന്‍ മരണപ്പെട്ടപ്പോള്‍, ഞങ്ങളെല്ലാം അതുമായി ബന്ധപ്പെട്ട തിരക്കുകള്‍ക്കിടയിലായിരുന്ന സന്ദര്‍ഭത്തില്‍ ഒരു ആട് അകത്ത് കടന്ന് അത് തിന്നുകളഞ്ഞെന്നും ആഇശയില്‍ നിന്നും സുനനു ഇബ്‌നു മാജയും മുസ്‌നദു അഹ്‌മദും നിവേദനം ചെയ്തിട്ടുണ്ടെന്നതാണ് വ്യാജമായ ആരോപണം. ആയതിനാല്‍ ദുര്‍ബലമായ ആ ഹദീഥ് നമുക്ക് ആദ്യം പരിശോധിക്കാം.

"ആഇശ (റ) പറഞ്ഞു: കല്ലെറിയലിന്റെ വചനവും പത്ത് പ്രാവശ്യമാണ് മുലകുടിയെന്ന വചനവും അവതരിക്കപ്പെട്ടിരുന്നു. അതെഴുതിയ രേഖ എന്റെ തലയിണക്കടിയിലുണ്ടായിരുന്നു. ദൈവദൂതന്‍ മരണപ്പെട്ടപ്പോള്‍, ഞങ്ങളെല്ലാം അതുമായി ബന്ധപ്പെട്ട തിരക്കുകള്‍ക്കിടയിലായിരുന്ന സന്ദര്‍ഭത്തില്‍ ഒരു ആട് അകത്ത് കടന്ന് അത് തിന്നു കളഞ്ഞു” (സുനനു ഇബ്‌നു മാജ: 1944, മുസ്‌നദു അഹ്‌മദ്: 43/343) ഹദീഥ് ദുര്‍ബലമാണ്. അതുകൊണ്ടുതന്നെ മുസ്‌ലിംകള്‍ക്ക് അത് പ്രമാണവുമല്ല. ഈ നിവേദനത്തില്‍ മുഹമ്മദ്ബ്‌നു ഇസ്ഹാഖ് എന്ന ഒരു നിവേദകനുണ്ട്. അദ്ദേഹത്തെ തെളിവിനു കൊള്ളില്ലെന്ന് ഹദീഥ് നിവേദനം ചെയ്ത അഹ്‌മദ് തന്നെ വ്യക്തമാക്കിയിട്ടുണ്ട്. അയാള്‍ ദുര്‍ബലനാണെന്ന് യഹ്‌യബിന്‍ മഈനും, ശക്തനല്ലെന്ന് നസാഇയും പറഞ്ഞിട്ടുണ്ട്. (തഹ്ദീബ് അത്തഹ്ദീബ് 9/45)

പക്ഷെ നമ്മുടെ പ്രശ്‌നം അതല്ല. ദുര്‍ബലമായ ആ നിവേദനത്തില്‍ പോലും പച്ചയായ കൃത്രിമത്വം നടത്തിയിരിക്കുകയാണ് ചില മിഷണറി നുണ ഫാക്ടറികള്‍. ‘കല്ലെറിയലിന്റെ വചനവും പത്ത് പ്രാവശ്യമാണ് മുലകുടിയെന്ന വചനവും അവതരിക്കപ്പെട്ടിരുന്നു’ എന്ന ഭാഗത്ത് മിഷണറിമാര്‍ എഴുതി പിടിപ്പിച്ചത് ‘കല്ലെറിയലിന്റെ വചനവും പത്ത് പ്രാവശ്യമാണ് മുതിര്‍ന്ന പുരുഷന്മാര്‍ക്ക് മുലകുടിയെന്ന വചനവും അവതരിപ്പിക്കപ്പെട്ടിരുന്നു എന്നാണ്. ‘മുതിര്‍ന്ന പുരുഷന്മാര്‍’ എന്നത് ‘സുവിശേഷകന്മാരു’ടെ കള്ളകോലാണ്. അവര്‍ കൃത്രിമമായി എഴുതിച്ചേര്‍ത്തു പ്രചരിപ്പിച്ചതാണെന്നര്‍ത്ഥം. എത്രമാത്രം ലജ്ജാവഹവും നിന്ദാപരവുമാണ് ഇവരുടെ പ്രവര്‍ത്തനങ്ങള്‍. ഇത്തരം ചവറുകളാണ്, ‘സയന്‍സ് ടെമ്പര്‍’ ഉണ്ടാക്കാന്‍ കൂട്ടയോട്ടം നടത്തുന്ന യുക്തിവാദികളും സ്ത്രീ ശാക്തീകരണം പത്യമാക്കിയ ഫെമിനിസ്റ്റുകളും സനാതനധര്‍മ്മം സ്ഥാപിക്കാന്‍ നെട്ടോട്ടമോടുന്ന ഫാഷിസ്റ്റുകളും കൊണ്ടുനടക്കുന്നതെന്നോര്‍ക്കുമ്പോള്‍ സഹതാപം തോന്നുന്നു. ‘ദൈവം സ്‌നേഹ’മാണെന്ന് നാഴികക്ക് നാല്‍പ്പത് വട്ടവും പാട്ടുപാടുന്നവര്‍ക്കെങ്കിലും, അവര്‍ പാടിയതിലും പറഞ്ഞതിലും തെല്ല് ആത്മാര്‍ത്ഥത ഉണ്ടായിരുന്നെങ്കില്‍ എത്ര നന്നായിരുന്നു.

‘മുലയൂട്ടല്‍’ ആഇശ(റ)യുടെ നിലപാടെന്തായിരുന്നു

വിവാഹബന്ധം നിഷിദ്ധമാക്കപ്പെടുന്ന ബന്ധങ്ങള്‍ സ്ഥാപിക്കാനായി ആഇശ (റ) തന്റെ സഹോദരി പുത്രിമാരോടും സഹോദര പുത്രിമാരോടും, മുതിര്‍ന്ന പുരുഷന്മാര്‍ക്ക് മുലപ്പാല്‍ നല്‍കുവാന്‍ ഉപദേശിച്ചിരുന്നു എന്നതാണ് വിഷയ സംബന്ധിയായ അടുത്ത വിമര്‍ശനം. വാസ്തവത്തില്‍ ആഇശ (റ) അപ്രകാരം ആരോടെങ്കിലും ഉപദേശിക്കുകയോ നിര്‍ദ്ദേശിക്കുകയോ ചെയ്തിരുന്നോ? പ്രസ്തുത വിഷയം ചര്‍ച്ച ചെയ്യും മുന്‍പ് ഇസ്‌ലാമില്‍ മുലകുടി ബന്ധം എപ്രകാരമാണ് സ്ഥാപിതമാകുന്നതെന്ന് നാം മനസ്സിലാക്കേണ്ടിയിരിക്കുന്നു. എങ്കില്‍ മാത്രമേ ആഇശ(റ)യുടെ നിലപാട് എന്തായിരുന്നു എന്ന് നമുക്ക് വേര്‍തിരിച്ചു മനസ്സിലാക്കാനാകൂ. രണ്ടു വയസ്സിനുള്ളില്‍, വിശപ്പ് അടങ്ങുംവിധം അഞ്ചു തവണയെങ്കിലും മുലപ്പാല്‍ നല്‍കിയാല്‍ മാത്രമാണ്, വിവാഹം നിഷിദ്ധമാക്കപ്പെടുന്ന ബന്ധം സ്ഥാപിതമാവുകയുള്ളൂ എന്നതാണ് ഈ വിഷയകമായി ഇസ്‌ലാമിക നിയമം. അഥവാ കുഞ്ഞുങ്ങള്‍ക്ക് മുലയൂട്ടുന്നതിലൂടെ മാത്രമേ മുലകുടി ബന്ധം സ്ഥാപിതമാകൂ എന്നതാണ് ഇസ്‌ലാമിക വിധി.

”മാതാക്കള്‍ തങ്ങളുടെ സന്താനങ്ങള്‍ക്ക് പൂര്‍ണ്ണമായ രണ്ടുകൊല്ലം മുല കൊടുക്കേണ്ടതാണ്. (കുട്ടിയുടെ) മുലകുടി പൂര്‍ണ്ണമാക്കണം എന്ന് ഉദ്ദേശിക്കുന്നവര്‍ക്കത്രെ ഇത്”. (ക്വുര്‍ആന്‍: 2:233) ”അവന്റെ മുലകുടി നിര്‍ത്തുന്നതാകട്ടെ രണ്ടുവര്‍ഷം കൊണ്ടുമാണ്” (ക്വുര്‍ആന്‍: 31:14) ”അവന്റെ ഗര്‍ഭകാലവും മുലകുടി നിര്‍ത്തലും കൂടി മുപ്പത് മാസക്കാലമാകുന്നു” (ക്വുര്‍ആന്‍: 46:15) ”വയറിനെ പുഷ്ടിപ്പിക്കുന്നതില്‍ അല്ലാതെ രിദ്വാഅ് (പാലൂട്ടല്‍) ഇല്ല” (ഇബ്‌നുമാജ: 1946, ഇബ്‌നുഹിബ്ബാന്‍: 4225) ”അസ്ഥിയെ ശക്തിപ്പെടുത്തുകയും മാംസം പുഷ്ടിപ്പെടുത്തുകയും ചെയ്യുന്നതല്ലാത്തത് രിദ്വാഅ് അല്ല” (സുനനുല്‍ ക്വുബ്‌റാ, ബൈഹകി: 15653) ”രണ്ടു വയസ്സിനുള്ളില്‍ ഊട്ടുന്നതല്ലാത്ത രിദ്വാഅ് ഇല്ല.” (സുനനുദ്ദാറക്വുത്‌നി : 4364) ”വിശപ്പടങ്ങുന്ന കുട്ടി മാത്രമാണ് (ബന്ധം സ്ഥാപിക്കുന്ന) രിദ്വാഅ് (മുലകുടി).” (സ്വഹീഹു മുസ്‌ലിം:32, അബൂദാവൂദ് : 12/8 നസാഈ : 26/51, ഇബ്‌നുമാജ : 9/37, അഹ്‌മദ്: 6/93) മുലപ്പാല്‍ കുടിക്കുന്നതിലൂടെ വിശപ്പടങ്ങുക കുഞ്ഞുങ്ങള്‍ക്കാണ്. അതുകൊണ്ടുതന്നെ മുലകുടിബന്ധം സ്ഥാപിതമാവുക കുഞ്ഞായിരിക്കുമ്പോള്‍ മുലപ്പാല്‍ കുടിക്കുന്നതിലൂടെ മാത്രമാണ് എന്ന് ഹദീഥിനെ വ്യാഖ്യാനിച്ച പണ്ഡിതന്മാരെല്ലാം വ്യക്തമാക്കുന്നുണ്ട്. (ശര്‍ഹു മുസ്‌ലിം: 1/434, ഫത്ഹുല്‍ ബാരി: 1/100, ഉംദത്തുല്‍ കാരി: 20/97, ഇര്‍ശാദുസ്സരി: 8/33, ഫൈദുല്‍ കദീര്‍: 3/360, മിര്‍ക്കാത്തുല്‍ മഫാത്തീഹ്: 3168, ശര്‍ഹു സര്‍ക്കാനി അലാല്‍ മുവത്വഃ: 3/374, ഹാശിയത്തു സിന്ദി അലാ സുനനി ഈബ്‌നുമാജ: 1/600, ഔനുല്‍ മഅ്ബൂദ്: 6/47) പ്രവാചകാനുചരന്മാരായ ഉമ്മര്‍, അലി, അബ്ദുല്ലാഹിബ്‌നു മസ്ഊദ്, അബ്ദുല്ലാഹിബ്‌നു അബ്ബാസ്, ഇബ്‌നു ഉമ്മര്‍, അബൂഹുറൈറ, ജാബിര്‍, പ്രവാചക പത്‌നിമാര്‍ തുടങ്ങി ഹസനുല്‍ ബസ്വരി, സുഹ്‌രി, ഇക്‌രിമ, ഔസാഈ, ശുഅ്ബി, കത്താദ, ഇബ്‌നു ശബ്‌റമ, ഇബ്‌നു അബീ ലൈല, ഹസനിബ്‌നു സ്വാലിഹ്, ഇബ്‌നു അബി ദിഅ്ബ്, സൗരി, ഇസ്ഹാക്, അബൂസൗര്‍, അബൂ ഉബൈദ്, ത്വബ്‌രി, ലൈസ് എന്നിങ്ങനെ പൗരാണിക കര്‍മ്മശാസ്ത്ര പണ്ഡിതരായ സര്‍വ്വരുടേയും അഭിപ്രായവും ഇതുതന്നെയാണ്. (മുഖ്തസറു ഇഖ്തിലാഫുല്‍ ഉലമ: 2/314, അല്‍ഹാവി അല്‍കബീര്‍: മാവര്‍ദി 11/366, മുഹല്ലാ: 10/17, മുഗ്നി: 11/319, സാദുല്‍ മആദ്: 5/577, തഫ്‌സീറു ഇബ്‌നു കസീര്‍ : 1/284, ഉംദത്തുല്‍ കാരി : 20/85) ഹനഫീ, മാലികീ, ശാഫിഈ, ഹമ്പലി കര്‍മ്മശാസ്ത്ര മദ്ഹബുകളുടെ പണ്ഡിതന്മാരുടെ അഭിപ്രായവും തഥൈവ. (അല്‍ മബ്‌സൂത്: 5/135, അല്‍ മുദവ്വനത്തുല്‍ കുബ്‌റാ: 5/407, അല്‍ ഉമ്മ്: 5/428, മുഗ്‌നി: 11/319)

അപ്പോള്‍ ഇസ്‌ലാമിന്റെ വിധി മുതിര്‍ന്നവര്‍ക്ക് മുലകുടി ഇല്ല എന്നതാണ്. എന്നാല്‍ സാലിമിന്റെ വിഷയത്തില്‍ ഉണ്ടായത് തീര്‍ത്തും അവരുടെ വിഷയത്തില്‍ മാത്രം ബാധകമായ ഒരു ഇളവു മാത്രമായിരുന്നു. അതും മുലപ്പാല്‍ സ്തനങ്ങളില്‍ നിന്നും നേരിട്ടു നല്‍കാതെ, പാല്‍ പിഴിഞ്ഞ് തളികയിലാക്കി കൊടുക്കുക മാത്രമാണുണ്ടായതെന്ന് നാം മുമ്പ് സൂചിപ്പിച്ചിട്ടുണ്ട്. എന്നാല്‍ ‘സാലിമിന്റെ വിഷയത്തിലെ പ്രവാചക നടപടി തെളിവ് പിടിച്ചുകൊണ്ട് ആഇശ (റ) തന്റെ വീട്ടില്‍ പ്രവേശിക്കാന്‍ താല്‍പര്യപ്പെടുന്ന മുതിര്‍ന്നവര്‍ക്ക് മുലപ്പാല്‍ നല്‍കാനായി തന്റെ സഹോദര പുത്രിമാരോടും, സഹോദരി പുത്രിമാരോടും നിര്‍ദ്ദേശിക്കാറുണ്ടായിരുന്നു എന്നും, അപ്രകാരം അവര്‍ (സഹോദര- സഹോദരി പുത്രിമാര്‍) അഞ്ചുതവണ മുലപ്പാല്‍ നല്‍കി മുലകുടിബന്ധം സ്ഥാപിതമായതിന് ശേഷമേ ആഇശ (റ) അവരെ തന്റെ വീട്ടില്‍ പ്രവേശിപ്പിക്കാറുള്ളൂ’ (സുനനു അബീദാവൂദ്: 2061) എന്ന നിവേദനത്തെ ദുര്‍വ്യാഖ്യാനിച്ചു കൊണ്ടാണ് ഇസ്‌ലാമില്‍ മുതിര്‍ന്ന പുരുഷന്മാര്‍ക്കും മുലകുടിയുണ്ടെന്ന വ്യാജപ്രചരണം ഇസ്‌ലാംവിരോധികള്‍ അഴിച്ചുവിട്ടു കൊണ്ടിരിക്കുന്നത്. എന്നാല്‍ ഇവിടെ നാം പ്രത്യേകം മനസ്സിലാക്കേണ്ടുന്ന ചില കാര്യങ്ങളെ ഇപ്രകാരം സംഗ്രഹിക്കാം:

ഒന്ന്: ആഇശ (റ) ആര്‍ക്കും മുലപ്പാല്‍ – സ്തനങ്ങളില്‍ നിന്ന് നേരിട്ടോ അല്ലാതെയോ കൊടുത്തിട്ടില്ല. ഒരിക്കലും പ്രസവിച്ചിട്ടില്ലാത്ത അവര്‍ക്ക് എങ്ങനെ മുലപ്പാലുണ്ടാകും? അതുകൊണ്ടാണ് മുലപ്പാല്‍ നല്‍കാനായി ആഇശ (റ) തന്റെ സഹോദരി പുത്രിമാരോടും സഹോദര പുത്രിമാരോടും നിര്‍ദ്ദേശിച്ചത്. ആഇശ (റ) മുതിര്‍ന്ന പുരുഷന്മാരെ മുലയൂട്ടിയെന്ന മിഷണറി പ്രചരണം കല്ലുവെച്ച കളവാണെന്ന് വ്യക്തം.

രണ്ട്: സാലിമിന് സഹ്‌ല മുലപ്പാല്‍ നല്‍കിയത് സ്തനങ്ങളില്‍ നിന്ന് നേരിട്ടല്ലെന്നും, ഒരു തളികയില്‍ പാല്‍ പിഴിഞ്ഞ് കുടിക്കാന്‍ കൊടുക്കുന്ന രീതി – ‘വുജൂര്‍’ (الوجور) – യാണ് അവലംബിച്ചിരുന്നതെന്നും നാം മുമ്പ് വ്യക്തമായ തെളിവുകളുടെ അടിസ്ഥാനത്തില്‍ സ്ഥാപിച്ചതാണ്. അതെ രീതിയില്‍ (‘വുജൂര്‍’) തന്നെ മുലപ്പാല്‍ കൊടുക്കാനാണ് ആഇശ (റ) തന്റെ സഹോദര പുത്രിമാരോടും, സഹോദരി പുത്രിമാരോടും നിര്‍ദ്ദേശിച്ചതെന്ന വസ്തുത മറച്ചുവെച്ചുകൊണ്ട്, ആഇശ(റ)യുടെ നിര്‍ദ്ദേശം – (സഹോദര, സഹോദരി പുത്രിമാരോട്) നേരിട്ട് സ്തനങ്ങളില്‍ നിന്നും മുലയൂട്ടുവാനായിരുന്നു എന്ന പച്ചക്കള്ളം പ്രചരിപ്പിക്കുകയാണ് ഇസ്‌ലാംവിമര്‍ശകര്‍ ചെയ്യുന്നത്. സാലിമിന്റെ വിഷയത്തിലെ പ്രവാചക നടപടി തെളിവ് പിടിച്ചുകൊണ്ട്, മുലകുടി പ്രായം (രണ്ടു വയസ്സ്) പിന്നിട്ട ഒരാള്‍ക്ക് മുലപ്പാല്‍ നല്‍കുക വഴി വിവാഹം നിഷിദ്ധമാക്കപ്പെടുന്ന ബന്ധം സ്ഥാപിക്കാമെന്ന് വിലയിരുത്തിയ ആഇശ (റ), പക്ഷെ ആ വിഷയത്തില്‍ പ്രവാചകന്‍ നിര്‍ദ്ദേശിച്ച മുലപ്പാല്‍ നല്കുന്ന രീതി (വുജൂര്‍) പരിഗണിച്ചില്ലെന്നത് കേവലം വിമര്‍ശകരുടെ ലൈംഗിക ഭാവന മാത്രമാണ്.

മൂന്ന്: ഇസ്‌ലാമില്‍ ബന്ധം സ്ഥാപിതമാക്കുന്നത് മൂന്ന് മാര്‍ഗങ്ങളിലൂടെയാണ്. പ്രസവം, വിവാഹം, മുലകുടി. ഈ ബന്ധങ്ങളില്‍ പെടാത്ത, വിവാഹബന്ധം നിഷിദ്ധമാക്കപ്പെട്ടുവെന്ന് സ്ഥിരപ്പെടാത്ത ഒരു അന്യപുരുഷനേയും തന്റെ വീട്ടില്‍ പ്രവേശിപ്പിക്കില്ലെന്ന ദൃഢനിശ്ചയം ആഇശ(റ)യുടെ കണിശമായ പാതിവ്രത്യത്തേയും പരിശുദ്ധിയേയുമാണ് തെളിയിക്കുന്നത്. അത്തരം മഹത്തരമായ ഒരു നിലപാടി നെയാണ്, തങ്ങളുടെ ദുഷിച്ച ലൈംഗിക ഭാവനയിലൂടെ അശ്ലീലതയുടെ ചായം പൂശാന്‍ മിഷണറി നുണ ഫാക്ടറികള്‍ ഒരുമ്പെട്ടതെന്നത് എത്രമാത്രം നെറികെട്ട ‘സുവിശേഷ വേല’യായിപ്പോയി.

നാല്: ‘തന്റെ വീട്ടില്‍ പ്രവേശിക്കാന്‍ താല്‍പര്യപ്പെടുന്ന മുതിര്‍ന്നവര്‍ക്ക് മുലപ്പാല്‍ നല്‍കാനായി അവര്‍ കല്‍പ്പിക്കുമായിരുന്നു’ എന്ന ഹദീഥില്‍ പരാമര്‍ശിക്കപ്പെട്ട ‘മുതിര്‍ന്നവര്‍’ (الكبير) എന്ന പദപ്രയോഗം യഥാര്‍ത്ഥത്തില്‍ മുതിര്‍ന്ന പുരുഷന്മാരെയല്ല ഉദ്ദേശിക്കുന്നത്. മറിച്ച്, മുലകുടിപ്രായം കഴിഞ്ഞ വലിയ കുട്ടികളെയാണ്. ഇത് ചില ഹദീഥുകളില്‍ വ്യക്തമായി തന്നെ പരാമര്‍ശിച്ചിട്ടുണ്ട്.

ഉമ്മുസലമ (റ) ആഇശ(റ)യോട് പറഞ്ഞു ”നിങ്ങളുടെ അടുക്കല്‍ മുലകുടി പ്രായം കഴിഞ്ഞ കുട്ടി – الغلام الايفع – പ്രവേശിക്കുന്നുണ്ടല്ലോ. (അവര്‍ക്ക് മുലപ്പാല്‍ നല്‍കിയാല്‍ മുലകുടിബന്ധം സ്ഥാപിതമാകുമെന്ന് വിശ്വസിക്കാത്തതുകൊണ്ട്) അവര്‍ എന്റെ അടുക്കല്‍ പ്രവേശിക്കുന്നത് ഞാനിഷ്ടപ്പെടുന്നില്ല. അപ്പോള്‍ അവര്‍ക്ക് മുലപ്പാല്‍ നല്‍കുക വഴി മുലകുടിബന്ധം സ്ഥാപിതമാകുമെന്നതിന് തെളിവായി ആഇശ (റ) സാലിമിന്റെ സംഭവം ഉദ്ധരിക്കുകയും ചെയ്തു…” (സ്വഹീഹു മുസ്‌ലിം : 1453, മുസ്‌നദു അഹ്‌മദ്: 25415) ഹദീഥില്‍ പരാമര്‍ശിക്കപ്പെട്ട, അല്‍ ഗുലാം അല്‍ അയ്ഫഅ് (الغلام الايفع) എന്നതുകൊണ്ട് വിവക്ഷിക്കുന്നത് പ്രായപൂര്‍ത്തിയാകാത്ത ആണ്‍കുട്ടിയെയാണെന്ന് ഇമാം നവവി (റ) ഹദീഥിനെ വ്യാഖ്യാനിക്കവെ വ്യക്തമാക്കുന്നുണ്ട്. (ശര്‍ഹു മുസ്‌ലിം: 10/33)

തന്റെ വീട്ടില്‍ പ്രവേശിക്കാന്‍ താല്‍പര്യപ്പെടുന്ന ‘പുരുഷന്മാര്‍ക്ക്’ മുലപ്പാല്‍ നല്‍കി മുലകുടിയിലെ ബന്ധുവാക്കാന്‍ ആഇശ (റ) ശ്രദ്ധിച്ചിരുന്നു എന്ന് സൂചിപ്പിക്കുന്ന ഹദീഥുകളിലെ ‘പുരുഷന്മാര്‍’ (الرجال) എന്നതുകൊണ്ട് ഉദ്ദേശിക്കുന്നതും മുലകുടി പ്രായം കഴിഞ്ഞ, എന്നാല്‍ പ്രായപൂര്‍ത്തിയായിട്ടില്ലാത്ത ആണ്‍കുട്ടികളെയാണ്. അല്ലാതെ വലിയ പുരുഷന്മാരെയല്ല. പുല്ലിംഗത്തെ (Male Gender) സൂചിപ്പിക്കാനായി രിജാല്‍ (الرجال) എന്ന് അറബിയില്‍ ഉപയോഗിക്കാറുണ്ട്. എന്ന് മാത്രമല്ല, ശാരീരികമായി പുരുഷനായി മാറിയിട്ടില്ലാത്ത ആണ്‍കുട്ടിയെ അവന്റെ ലിംഗഭേദം (Gender) പുല്ലിംഗമായതിനാല്‍ അവന്‍ ഭാവിയില്‍ പുരുഷനായി മാറും എന്നത് പരിഗണിച്ച് റജുല്‍ (الرجل) ‘പുരുഷന്‍’ എന്ന് അറബിഭാഷാ നിയമപ്രകാരം വിളിക്കാവുന്നതാണ്. ഇതിനെ അറബി അലങ്കാര ശാസ്ത്രത്തില്‍ (Rhetoric) ‘ഭാവിയില്‍ ആയി മാറുന്നതിനെ പരിഗണിച്ചുകൊണ്ടുള്ള പ്രയോഗം’ (اعتبار ما يكون) എന്നാണ് പറയുക. (അല്‍ മജാസുല്‍ മുര്‍സല്‍ ഫീ ലിസാനില്‍ അറബ്: അഹ്‌മദ് ഹിന്ദാവി അബ്ദുല്‍ ഗഫ്ഫാര്‍, കിത്താബു മിന്‍ഹാജുല്‍ വാള്വിനി ലില്‍ ബലാഗ: 3/300, ഗായത്തുല്‍ മുസൂല്‍ ഫീ ശര്‍ഹി ലുബ്ബുല്‍ ഉസൂല്‍: 1/51)

ഇത് കേവലം ഭാഷ ശാസ്ത്ര നിയമപ്രകാരമുള്ള ഒരു സമര്‍ത്ഥനമല്ല; മറിച്ച് ഹദീഥുകള്‍ വ്യക്തമായി സൂചിപ്പിച്ച ഒരു വിഷയം ഭാഷ പ്രയോഗങ്ങള്‍ക്കുപോലും അന്യമല്ലെന്ന ബോധ്യപ്പെടുത്തലാണ്. ആഇശ (റ) ഉദ്ദേശിച്ച ‘പുരുഷന്മാര്‍’ (الرجال) മുലകുടി പ്രായം പിന്നിട്ട, എന്നാല്‍ പ്രായപൂര്‍ത്തിയായിട്ടില്ലാത്ത ആണ്‍കുട്ടികളാണെന്ന് വ്യക്തമാക്കുന്ന മറ്റു ചരിത്ര നിവേദനങ്ങളും നമുക്ക് കാണാം.

(a) ”സാലിമിബ്‌നു അബ്ദുല്ലാഹിബ്‌നു ഉമര്‍ (സഹ്‌ല മുലപ്പാല്‍ നല്‍കിയ സാലിം അല്ല ഇത്) പറയുന്നു: ഞാന്‍ കുഞ്ഞായിരിക്കെ ആഇശ (റ) എന്നെ അവരുടെ സഹോദരി ഉമ്മുകുല്‍സും ബിന്‍ത് അബൂബക്കറിന്റെ അടുത്തേക്ക് അയച്ചു. എന്നിട്ടവരോട് ഇങ്ങനെ പറഞ്ഞു: ‘ഇവന് പത്തു തവണ മുലപ്പാല്‍ ഊട്ടുക എങ്കില്‍ ഇവന് എന്റെ അടുത്ത് (വീട്ടില്‍) പ്രവേശിക്കാമല്ലോ.’ സാലിം പറയുന്നു: ഉമ്മുകുല്‍സും എനിക്ക് മൂന്നു തവണ മുലപ്പാല്‍ തന്നു. പിന്നീടെനിക്ക് രോഗമായി. അതിനാല്‍ എനിക്ക് മൂന്നു തവണ മാത്രമേ മുലപ്പാല്‍ തരാന്‍ സാധിച്ചുള്ളൂ. ഉമ്മുകുല്‍സൂം എനിക്ക് പത്തു തവണ മുലപ്പാല്‍ തരാതിരുന്നത് കാരണത്താല്‍ (വലുതായപ്പോള്‍) ഞാന്‍ ആഇശയുടെ അടുത്ത് പ്രവേശിക്കില്ലായിരുന്നു” (മുവത്വഅ്: 2/603)

ചെറുപ്രായത്തില്‍ ഉമ്മുകുല്‍സൂമിന്റെ മുലപ്പാല്‍, ബന്ധം സ്ഥാപിതമാകുന്ന നിലയ്ക്ക് പൂര്‍ണമായും കുടിക്കാതിരിക്കുന്നത് മൂലം ആഇശ(റ)യുമായി മുലകുടിയിലെ ബന്ധം സ്ഥാപിതമായില്ല. അതു കാരണത്താല്‍ ആഇശ (റ) തന്നെ വലുതായപ്പോള്‍ അവരുടെ വീട്ടില്‍ പ്രവേശിപ്പിക്കില്ലായിരുന്നു എന്ന് ചുരുക്കം. വലിയ പുരുഷന്മാര്‍ മുലപ്പാല്‍ കുടിച്ചാല്‍ മുലകുടി ബന്ധം സ്ഥാപിതമാകും എന്നതായിരുന്നു ആഇശ(റ)യുടെ വാദമെങ്കില്‍ സാലിമിബ്‌നു അബ്ദുല്ലാഹിബ്‌നു ഉമ്മറിനെ വലിയ പുരുഷനായതിനുശേഷവും, സഹോദരിയുടെ മുലപ്പാല്‍ നല്‍കിയതിനുശേഷം തന്റെ വീട്ടില്‍ പ്രവേശിപ്പിക്കാമായിരുന്നില്ലേ? അപ്പോള്‍ കാര്യം വളരെ വ്യക്തമാണ്. മുലപ്പാല്‍ ഊട്ടപ്പെടുന്ന ‘രിജാല്‍’ (പുരുഷന്മാര്‍) എന്നതുകൊണ്ട് ആഇശ (റ) ഉദ്ദേശിച്ചത്, മുലകുടി പ്രായം പിന്നിട്ട, എന്നാല്‍ പ്രായപൂര്‍ത്തിയെത്തിയിട്ടില്ലാത്ത ആണ്‍കുട്ടികളെ മാത്രമാണ്.

(b) ”കുട്ടികള്‍ക്ക് (الصبيان) അവര്‍ പുരുഷന്മാരായാല്‍ തന്റെ അടുത്ത് പ്രവേശിക്കുന്നതിന് തടസ്സമാകാതിരിക്കാന്‍ – ചെറുപ്രായത്തില്‍ മുലപ്പാല്‍ ഊട്ടുവാന്‍ ആഇശ (റ) തന്റെ സഹോദരന്‍ അബ്ദുര്‍റഹ്‌മനിബ്‌നു അബൂബക്കറിനോട് കല്‍പ്പിക്കുമായിരുന്നു”. (ബദാഇഉ സ്വനാഇഅ്: 4/6, ശര്‍ഹു മുഖ്തസറു ത്വഹാവി: 3/325) ഇവിടെ ആഇശ (റ) ഉദ്ദേശിച്ച പ്രായം വളരെ വ്യക്തമാണ്. കുട്ടികള്‍ പുരുഷന്മാരായാല്‍ തന്റെ അടുത്ത് പ്രവേശിക്കുന്നതിന് തടസ്സമാകാതിരിക്കുവാന്‍- ചെറുപ്രായത്തില്‍ തന്നെ മുലപ്പാല്‍ ഊട്ടുവാനാണ് അവര്‍ നിര്‍ദേശിക്കുന്നത്. ഇവിടെയും, ‘പുരുഷന്മാര്‍’ എന്നതുകൊണ്ട് മുതിര്‍ന്ന പുരുഷന്മാരെയല്ല, പ്രായപൂര്‍ത്തിയെത്താത്ത എന്നാല്‍ മുലകുടി പ്രായം കഴിഞ്ഞ ആണ്‍കുട്ടികളെയാണ് അവര്‍ ഉദ്ദേശിച്ചതെന്ന് വ്യക്തം. ഹദീഥില്‍ ‘രിജാല്‍’ (പുരുഷന്മാര്‍) എന്നത് ലിംഗഭേദത്തെ കുറിക്കാനാണ്, പ്രായത്തെ കുറിക്കാനല്ല ആഇശ (റ) ഉപയോഗിച്ചതെന്ന് പൗരാണികവും പ്രാമാണികവുമായ അറബി ഡിക്ഷണറികളിലെല്ലാം വ്യക്തമാക്കപ്പെട്ടിട്ടുണ്ട്. (താജുല്‍ ഉറൂസ്: 29/34, അല്‍ കാമൂസുല്‍ മുഹീത്: 1/1297, ലിസാനുല്‍ അറബ്: 11/265, അല്‍ കാമില്‍ ഫില്ലുഗത്തി വല്‍ അദബ്: 1/100)

(c) വലിയ പുരുഷന്മാര്‍ക്ക് മുലപ്പാല്‍ നല്‍കുക വഴി ബന്ധം സ്ഥാപിതമാകും എന്നതല്ല, മറിച്ച് മുലകുടി പ്രായമായ രണ്ടു വയസ്സു കഴിഞ്ഞ മുതിര്‍ന്ന ആണ്‍കുട്ടികള്‍ക്ക് മുലപ്പാല്‍ നല്‍കുക വഴി മുലകുടിബന്ധം സ്ഥാപിതമാകും എന്ന് മാത്രമാണ് ആഇശയുടെ അഭിപ്രായം എന്നതിനാലാണ് അതിനെതിരെ, ആഇശ (റ)യുടെ അഭിപ്രായം തിരുത്തിക്കൊണ്ട് എതിരഭിപ്രായം പ്രകടിപ്പിച്ച ഉമ്മുസലമ (റ) ‘തൊട്ടിലില്‍ വെച്ച് തന്നെ മുലകുടിച്ചാലെ’ (حتي يرضع في المهد) മുലകുടി ബന്ധം സ്ഥാപിതമാകൂ എന്ന പദപ്രയോഗം തന്നെ ഉപയോഗിച്ചത്. (മുസ്‌നദു അഹ്‌മദ്: 6/270, മുസ്‌നദു ഇബ്‌നു ഉവാന: 3/122, സുനനു അബൂദാവൂദ്: 2/223) അതുകൊണ്ടു തന്നെ ഈ വിഷയത്തില്‍ ആഇശയുടെ ഒറ്റപ്പെട്ട അഭിപ്രായം, വലിയ പുരുഷന്മാര്‍ക്ക് മുലപ്പാല്‍ നല്‍കുക വഴി ബന്ധം സ്ഥാപിതമാകും എന്നല്ല മറിച്ച്, മുലകുടി പ്രായമായ രണ്ട് വയസ്സ് കഴിഞ്ഞ ആണ്‍കുട്ടികള്‍ക്കും മുലപ്പാല്‍ നല്‍കുക വഴി മുലകുടി ബന്ധം സ്ഥാപിതമാകും എന്നു മാത്രമാണ്. അതും തീര്‍ത്തും അവരുടെ ഒറ്റപ്പെട്ട ഒരഭിപ്രായം മാത്രമാണത്. അതുകൊണ്ടാണ് മറ്റു പ്രവാചകപത്‌നിമാരെല്ലാം അവരെ ആ വിഷയത്തില്‍ എതിര്‍ത്തതും തിരുത്തിയതും.

അതിനാല്‍ ആഇശ(റ)യുടെ ഈ ഒറ്റപ്പെട്ട അഭിപ്രായത്തെ ദുര്‍വ്യാഖ്യാനിച്ച് അശ്ലീലതകള്‍ വിതറുവാനുള്ള മിഷണറി നുണ ഫാക്ടറികളുടെ ഉദ്യമം വളരെ തരംതാഴ്ന്ന പ്രവര്‍ത്തനമായിപ്പോയി. ചെറിയ ആണ്‍കുട്ടികള്‍ക്കു മുലയൂട്ടുന്നതിലൂടെ, അതും സ്തനങ്ങളില്‍ നിന്ന് നേരിട്ടല്ലാതെ തളികയില്‍ പിഴിഞ്ഞ് കുടിപ്പിക്കപ്പെടുന്നതിലൂടെ (വുജൂര്‍) ബന്ധുക്കളായാല്‍ അവര്‍ക്ക് പ്രായപൂര്‍ത്തിയെത്തിയാലും അവരുടെ സാന്നിധ്യം അന്യപുരുഷ സാന്നിധ്യമാകില്ലല്ലോ എന്ന് കണക്കുകൂട്ടിയാണ് ഇത്തരമൊരു നയം ആഇശ (റ) സ്വീകരിച്ചത്.

‘മുലയൂട്ടല്‍’ വിവാദം ഇനിയും അവസാനിക്കാത്ത ‘സുവിശേഷ വേലകള്‍’

രണ്ടു വയസ്സിനുള്ളില്‍ വിശപ്പടങ്ങും വിധം അഞ്ചു തവണയെങ്കിലും മുലയൂട്ടിയാല്‍ മാത്രമേ, വിവാഹം നിഷിദ്ധമാക്കപ്പെടുന്ന ബന്ധം സ്ഥാപിതമാകൂ എന്ന കണിശമായ കര്‍മശാസ്ത്ര വിധി നിഷ്‌കര്‍ഷിച്ച ഒരു മതത്തെ അപകീര്‍ത്തിപ്പെടുത്താനും വൈകൃതവും പ്രാകൃതവുമായ ഒരു ധര്‍മ്മശാസ്ത്രമായി അതിനെ ചിത്രീകരിക്കുവാനും വേണ്ടി മിഷണറി നുണ ഫാക്ടറികള്‍ കൈവെച്ചത് ‘സഹ്‌ല-സാലിം മുലപ്പാലൂട്ടല്‍’ സംഭവമാണ്. സാലിമിന്റെ വിഷയത്തില്‍ മാത്രം, അദ്ദേഹത്തിന്റെ മാതാവ് സഹ്‌ലയുടെ തീവ്ര ദുഃഖത്തിന് ഒരു പോംവഴിയായി – ഇളവ് നല്‍കപ്പെട്ട ഒരു കാര്യമാണ് അതെന്ന് വ്യക്തമായിട്ടും ആ സംഭവത്തെ ഇത്രമേല്‍ മ്ലേച്ഛമായവതരിപ്പിച്ചു എന്നതു മാത്രമല്ല ‘മുലയൂട്ടല്‍’ വിവാദവുമായി ബന്ധപ്പെട്ട് ‘കുഞ്ഞാടുകള്‍’ ചെയ്ത ക്രൂരത. ഒരു കുഞ്ഞിനും ഒരിക്കലും മുലയൂട്ടാന്‍ സാധ്യമല്ലാത്ത, ഒരിക്കലും പ്രസവിച്ചിട്ടില്ലാത്ത പ്രവാചക പത്‌നി ആഇശ (റ) അത്തരത്തില്‍ ‘മുലയൂട്ടല്‍’ നടത്തിയിരുന്നു എന്ന പച്ചക്കള്ളം പടച്ചുണ്ടാക്കാന്‍ ഇസ്‌ലാമിന്റെ അടിസ്ഥാന പ്രമാണങ്ങളില്‍ ഒന്നായ ഹദീഥുകളില്‍ കൈവെച്ചു. എന്നിട്ടും അരിശം തീരാതെ ആഇശയുടെ പ്രസ്തുത വിഷയകമായ ഒറ്റപ്പെട്ട അഭിപ്രായത്തെ ഉയര്‍ത്തിക്കാട്ടി വീണ്ടും അശ്ലീലതകള്‍ക്ക് പഴുതു തപ്പി നടന്നു. അതും അവരുടെ അഭിപ്രായത്തെ വസ്തുതാവിരുദ്ധമായ നിലയില്‍ ദുര്‍വ്യാഖ്യാനിച്ചു കൊണ്ട്. ഇനിയും തീര്‍ന്നിട്ടില്ല ഇവരുടെ മഹത്തായ ‘സുവിശേഷ വേല’കള്‍. തന്റെ ഒറ്റപ്പെട്ട അഭിപ്രായത്തില്‍ നിന്നും അഥവാ മുലകുടി പ്രായം (രണ്ടു വയസ്സ്) കഴിഞ്ഞതും എന്നാല്‍ പ്രായപൂര്‍ത്തിയെത്തിയിട്ടില്ലാത്തതുമായ ആണ്‍കുട്ടികള്‍ക്ക് മുലപ്പാല്‍ തളികയില്‍ പിഴിഞ്ഞു കുടിപ്പിച്ചാല്‍ ‘ബന്ധം’ സ്ഥാപിതമാകും എന്ന അഭിപ്രായത്തില്‍ നിന്നും പിന്നീട് ആഇശ (റ) മടങ്ങിയിട്ടുണ്ട് എന്ന വസ്തുത ഈ ‘കുഞ്ഞാടുകള്‍’ മറച്ചുവെച്ചിട്ടുണ്ടെന്നതും കൂടി നാം അറിയണം. അപ്പോള്‍ നമുക്ക് മനസ്സിലാകും മിഷണറി പക്ഷം വിമര്‍ശനമാണോ അല്ല വിരോധമാണോ പ്രകടിപ്പിക്കുന്നതെന്ന്.

ഇമാം കാസാനി പറഞ്ഞു: ”എന്നാല്‍ ആഇശ (ഈ വിഷയത്തിലുള്ള) നയം അവര്‍ തിരുത്തുകയുണ്ടായി എന്ന് സൂചിപ്പിക്കുന്ന നിവേദനങ്ങള്‍ ഉദ്ദരിക്കപ്പെട്ടിട്ടുണ്ട്. അവര്‍ ഇപ്രകാരം പറഞ്ഞതായി നിവേദനം ചെയ്യപ്പെട്ടിരിക്കുന്നു: ‘രക്തവും മാംസവും മുളപ്പിക്കുന്നതല്ലാത്ത (കൈകുഞ്ഞായിരിക്കുമ്പോളല്ലാത്ത) മുലകുടി വിവാഹബന്ധം നിഷിദ്ധമാക്കില്ല (മുലകുടി ബന്ധം സ്ഥാപിതമാകില്ല)” (ബദാഇഉസ്വനാഇ: 4/6) ‘തൊട്ടിലില്‍ വെച്ചുതന്നെ മുല കുടിച്ചാലെ’ മുലകുടി ബന്ധം സ്ഥാപിതമാകൂ എന്നും (മുസ്‌നദു അഹമ്ദ്: 6/270, മുസ്‌നദു ഇബ്‌നു ഉവാന: 3/122, സുനനു അബൂദാവൂദ്: 2/223) ‘അല്ലാഹുവാണേ, സാലിമിന്റെ വിഷയത്തില്‍ മാത്രമായി (ഖാസ്) പ്രവാചകന്‍ അനുവദിച്ചു കൊടുത്ത ഇളവായിരുന്നു അത് (ആ ഇളവ് എല്ലാവര്‍ക്കും ബാധകമല്ല)’ (സ്വഹീഹു മുസ്‌ലിം: 1454, മുസ്‌നദു അഹ്‌മദ്: 26660, സുനനു അബൂദാവൂദ്: 2061) എന്നുമുള്ള പ്രവാചക പത്‌നി ഉമ്മുസലമ(റ)യുടെ തിരുത്ത് ആഇശ (റ) സ്വീകരിച്ചു എന്നും, തന്റെ ഒറ്റപ്പെട്ട പൂര്‍വ്വ അഭിപ്രായത്തില്‍ നിന്നും അവര്‍ മടങ്ങിയെന്നും ഹദീഥ് ഗ്രന്ഥങ്ങളില്‍ തന്നെ വ്യക്തമാക്കിയിരിക്കെ അതെല്ലാം മറച്ചുവച്ചുകൊണ്ട് ഇത്തരം കുത്സിത പ്രവര്‍ത്തനങ്ങള്‍ നടത്താന്‍ ‘ദൈവത്തിന്റെ കുഞ്ഞാടുകള്‍’ക്കെങ്ങിനെ കഴിഞ്ഞു. മിഷണറി നുണ ഫാക്ടറികള്‍ നിര്‍മ്മിച്ച ഈ നുണകഥയും പൊക്കിപ്പിടിച്ച് ഇപ്പോഴും യുക്തിവാദികളും ഫെമിനിസ്റ്റുകളും ഫാഷിസ്റ്റുകളുമടങ്ങുന്ന ഇസ്‌ലാംവിമര്‍ശകര്‍ ചോദിക്കുന്നു ‘പ്രവാചകന്റെ ഈ കല്പന നടപ്പിലാക്കുവാന്‍ ഈ കാലഘട്ടത്തിലെ മുസ്‌ലിം സ്ത്രീകള്‍ തയ്യാറാകുമോ? തങ്ങളുടെ ഭാര്യമാരെ കൊണ്ട് ഈ പ്രവാചക നിര്‍ദ്ദേശം നടപ്പില്‍ വരുത്തുവാന്‍ മുസ്‌ലിം പുരുഷന്മാര്‍ ഒരുക്കമാണോ?’ എന്ന്. പാവം! സാധുക്കള്‍ വല്ലതും അറിയുന്നുണ്ടോ ഇതു മിഷണറി ‘സുവിശേഷ വേല’ മാത്രമാണെന്ന കഥ.!!!

മുഹമ്മദ് നബി സ്ത്രീ പീഢകനായിരുന്നു എന്ന് വ്യക്തമാക്കുന്ന നിവേദനങ്ങള്‍ സ്വഹീഹുല്‍ ബുഖാരിയില്‍ തന്നെ കാണാം. ‘ശൗത്ത്’ എന്നു വിളിക്കപ്പെട്ടിരുന്ന ഒരു തോട്ടത്തില്‍ വെച്ച് ‘ജൗന്‍’ ഗോത്രത്തിലെ ഉമൈമ:ബിന്‍ത് ശറാഹീല്‍ എന്ന സ്ത്രീയെ പ്രവാചകന്‍ കടന്നുപിടിക്കാന്‍ ശ്രമിക്കുകയും അവര്‍ ശക്തമായി പ്രതിരോധിക്കുകയും ചെയ്ത സംഭവം ഹദീഥ് ഗ്രന്ഥങ്ങള്‍ തന്നെ റിപ്പോര്‍ട്ട് ചെയ്തിട്ടുണ്ട്. സ്ത്രീകളെ കടന്നുപിടിക്കാന്‍ പോലും മടിയില്ലാതിരുന്ന ഒരു വ്യക്തിയെ എങ്ങനെയാണ് മാനവികതയുടെ പ്രവാചകനായി വിലയിരുത്തുക?

ഇമാം ബുഖാരി തന്റെ സ്വഹീഹില്‍ വ്യത്യസ്ത പരമ്പരകളിലൂടെ ഉദ്ധരിച്ച ഒരു സംഭവത്തിന്റെ ഏതാനും ഭാഗങ്ങള്‍ അടര്‍ത്തിയെടുത്ത് പച്ചക്ക് ദുര്‍വ്യാഖാനിച്ചിരിക്കുകയാണിവിടെ. ഉമൈമ: ബിന്‍ത് ശറാഹീലുമായി ബന്ധപ്പെട്ട് ഇമാം ബുഖാരി അദ്ദേഹത്തിന്റെ സ്വഹീഹില്‍ ഉദ്ധരിച്ച നിവേദനങ്ങള്‍ വായിച്ചാല്‍ സംഭവം പ്രവാചക ജീവിതത്തിലെ ഒരു പുഴുകുത്തല്ലെന്നും മറിച്ച് പ്രവാചകന്റെ മഹാമനസ്‌കതയും സഹിഷ്ണുതയും കാരുണ്യവും അനുവാചകര്‍ക്ക് ബോധ്യപ്പെടുക മാത്രമാണുണ്ടാവുക എന്ന ഉത്തമബോധ്യമുള്ളതുകൊണ്ടു തന്നെ ഹദീഥുകളില്‍ നിന്നും തങ്ങള്‍ക്ക് കൈവെക്കാനൊക്കുന്നതു മാത്രം തിരഞ്ഞുപിടിച്ചു ദുര്‍വ്യാഖ്യാനിച്ചു ദുഷിപ്പിക്കുക എന്ന തന്ത്രമാണ് ഇസ്‌ലാംവിമര്‍ശകര്‍ ചെയ്തുകൊണ്ടിരിക്കുന്നത്. അതിനാല്‍ പ്രസ്തുത സംഭവവുമായി ബന്ധപ്പെട്ട് ഇമാം ബുഖാരി അദ്ദേഹത്തിന്റെ സ്വഹീഹില്‍ ഉദ്ധരിച്ച ഏതാനും ചില നിവേദനങ്ങള്‍ താഴെ കൊടുക്കുകയാണ്. ഒരൊറ്റ വായനയിലൂടെ തന്നെ ഏതൊരാള്‍ക്കും എളുപ്പം ഗ്രഹിക്കാന്‍ സാധിക്കുന്ന ഒരു വിഷയത്തെ പോലും ഇത്തരത്തില്‍ ദുര്‍വ്യാഖ്യാനിക്കുവാന്‍ ഇസ്‌ലാം വിമര്‍ശകര്‍ക്ക് യാതൊരു ലജ്ജയുമില്ലെന്നത് അവരുടെ സംസ്‌കാരത്തിന് നേരെയുള്ള ചൂണ്ടുവിരലാണ്.

”ഹംസത്തിബ്‌നു അബീ ഉസൈദ് നിവേദനം: അബൂ ഉസൈദ് (റ) പറഞ്ഞു: ഒരിക്കല്‍ പ്രവാചകനോടൊപ്പം(സ) ഞങ്ങള്‍ ഒരു യാത്ര പുറപ്പെട്ടു. ‘ശൗത്ത്’ എന്ന് വിളിക്കപ്പെടുന്ന ഒരു തോട്ടത്തെ ലക്ഷ്യമാക്കി ഞങ്ങള്‍ നീങ്ങി. ഞങ്ങള്‍ രണ്ട് തോട്ടങ്ങള്‍ക്കിടയിലെത്തിയപ്പോള്‍, അവിടെ ഞങ്ങളിരുന്നു. ഞങ്ങളോട് അവിടെ ഇരിക്കാൻ പ്രവാചകൻ (സ) പറഞ്ഞു. എന്നിട്ട് അദ്ദേഹം ആ തോട്ടത്തിലേക്ക് പോയി. ജൗന്‍ ഗോത്രത്തിലെ സ്ത്രീയെ അവിടേക്ക് കൊണ്ടുവരപ്പെട്ടിരുന്നു. ഈത്തപ്പന കൊണ്ടുണ്ടാക്കിയ വീട്ടിലായിരുന്നു അവര്‍. ഉമൈമ: ബിന്‍ത് ശറാഹീല്‍ എന്നായിരുന്നു അവരുടെ നാമം. അവരോടൊപ്പം അവരുടെ മുലകുടി ബന്ധത്തിലെ പോറ്റുമ്മയും ഉണ്ടായിരുന്നു. അവരുടെ അടുത്തേക്ക് അല്ലാഹുവിന്റെ ദൂതൻ (സ) പ്രവേശിക്കുകയും ‘നീ നിന്നെ എനിക്ക് സമര്‍പ്പിക്കുക’ എന്ന് പറയുകയും ചെയ്തു. അപ്പോള്‍ അവര്‍ പറഞ്ഞു: ‘ഒരു രാജ്ഞി അവരെ ഏതെങ്കിലും ഒരു സാധാരണക്കാരന് സമര്‍പ്പിക്കുമോ?’ അവരുടെ മേല്‍ കൈവെച്ച് അവരെ ശാന്തയാക്കാനായി തന്റെ കൈകള്‍ അദ്ദേഹം നീട്ടി. അപ്പോള്‍ അവര്‍ പറഞ്ഞു: ‘നിങ്ങളില്‍ നിന്നും ഞാൻ അല്ലാഹുവില്‍ ശരണം തേടുന്നു’. അപ്പോൾ പ്രവാചകൻ (സ) പറഞ്ഞു: ‘ശരണം തേടുവാന്‍ ഏറ്റവും അർഹനായവനിലാണ് നീ ശരണം തേടിയിരിക്കുന്നത്. പിന്നീട് അദ്ദേഹം ഞങ്ങളുടെ അടുത്തേക്ക് വന്നുകൊണ്ട് പറഞ്ഞു. “അബൂ ഉസൈദ്, അവര്‍ക്ക് രണ്ട് റാസിഖിയ്യാ വസ്ത്രങ്ങള്‍ നല്‍കുകയും അവരുടെ കുടുംബത്തിലേക്ക് അവരെ തിരിച്ചെത്തിക്കുകയും ചെയ്യുക.” (ബുഖാരി: 5255)

”പ്രവാചകൻ (സ) ഉമൈമ: ബിന്‍ത് ശറാഹീലിനെ വിവാഹം ചെയ്തു. അവരെ അദ്ദേഹത്തിന്റെ അടുത്തേക്ക് പ്രവേശിക്കപ്പെട്ടപ്പോള്‍, അദ്ദേഹം തന്റെ കൈകൾ നീട്ടി സ്വീകരിച്ചു. അവര്‍ക്കത് ഇഷ്ടപെടാത്തത് പോലെ അവര്‍ പ്രതികരിച്ചു. അപ്പോള്‍ അവര്‍ക്ക് തിരികെ സ്വഗൃഹത്തിലേക്ക് പോകാന്‍ യാത്രാ സൗകര്യങ്ങള്‍ ചെയ്യാനും, രണ്ട് റാസിഖിയ്യാ വസ്ത്രങ്ങള്‍ സമ്മാനമായി നല്‍കാനും പ്രവാചകൻ (സ) അബൂ ഉസൈദിനോട് കല്പിച്ചു.” (ബുഖാരി: 5256)

”ഇമാം ഔസാഇ (റ) പറഞ്ഞു: ഞാന്‍ സുഹ്‌രിയോട് ചോദിച്ചു: ‘പ്രവാചകൻ(സ)യുടെ ഭാര്യമാരില്‍ ആരാണ് അദ്ദേഹത്തില്‍ നിന്നും ശരണം തേടിയത്?’ അദ്ദേഹം പറഞ്ഞു: ‘എന്നോട് ആഇശ(റ)യില്‍ നിന്നും ഇപ്രകാരം ഉര്‍വ അറിയിക്കുകയുണ്ടായി. ജൗന്‍ ഗോത്രക്കാരിയെ പ്രവാചകന്റെ(സ) അരികിലേക്ക് (അദ്ദേഹത്തിന്റെ പത്‌നിയായി) ആനയിക്കപ്പെടുകയും അദ്ദേഹം അവളുടെ അരികിലേക്ക് ചെല്ലുകയും ചെയ്തപ്പോള്‍ അവള്‍ പറഞ്ഞു: ‘ഞാന്‍ താങ്കളില്‍ നിന്നും അല്ലാഹുവിനോട് ശരണം തേടുന്നു.’ അപ്പോള്‍ അദ്ദേഹം പറഞ്ഞു: ‘അതിമഹത്വമുള്ളവനിലാണ് നീ ശരണം തേടിയിരിക്കുന്നത്. നീ നിന്റെ കുടുംബത്തിലേക്ക് മടങ്ങിക്കൊള്ളുക.” (ബുഖാരി: 5254)

ഉമൈമ: ബിന്‍ത് ശറാഹീലീനെ പ്രവാചകൻ (സ) വിവാഹം ചെയ്തിരുന്നു എന്ന വസ്തുത മറച്ചു പിടിച്ചു കൊണ്ട് പ്രസ്തുത സംഭവം വിമര്‍ശകര്‍ അവതരിപ്പിക്കാറ് എന്ന് മുകളിലെ മൂന്ന് ഹദീസുകളും ഒരുമിച്ചു വെച്ച് വായിക്കുമ്പോൾ സുതരാം വ്യക്തമാവുന്നു. പ്രവാചകനെ പെണ്ണു പിടുത്തക്കാരനായും പരസ്ത്രീകളെ കടന്നുപിടിക്കുന്ന വ്യക്തിയായും താറടിക്കുകയാണ് ഈ ദുര്‍വ്യാഖ്യാന കസര്‍ത്ത് നടത്തുന്നതിന് പിന്നിലെ ചേതോവികാരം. താന്‍ വിവാഹം ചെയ്ത സ്ത്രീക്ക് തന്നോടൊപ്പം ജീവിക്കുവാന്‍ താല്‍പര്യമില്ലെന്നറിഞ്ഞപ്പോള്‍, നിര്‍ബന്ധിച്ച് കൂടെ താമസിപ്പിക്കാതെ മാന്യമായി അവരെ സ്വഗൃഹത്തിലേക്ക് യാത്രയാക്കുകയും വേര്‍പിരിയും മുമ്പ് അവര്‍ക്ക് സമ്മാനങ്ങള്‍ നല്‍കുകയും ചെയ്ത മാതൃകാപരമായ ഒരു നപടിയെ എത്ര നികൃഷ്ടമായാണ് ഇസ്‌ലാംവിമര്‍ശകര്‍ അവതരിപ്പിച്ചിരിക്കുന്നത്.

ﻻَ ﺗَﺤْﻤِﻠُﻮا اﻟﻨِّﺴَﺎءَ ﻋَﻠَﻰ ﻣَﺎ ﻳَﻜْﺮَﻫْﻦَ

“സ്ത്രീകളെ അവർക്ക് വെറുക്കുന്നത് ചെയ്യാൻ നിങ്ങൾ നിർബന്ധിക്കരുത്.” (മുസ്വന്നഫ് അബ്ദുർ റസാഖ്: 10320) എന്ന് അനുചരന്മാരെ പഠിപ്പിക്കുക മാത്രമല്ല കാരുണ്യ മൂർത്തിയായ പ്രവാചകൻ (സ) ചെയ്തത്, പ്രത്യുത ഉമൈമയോട് അനുവർത്തിച്ച നിലപാടിലൂടെ തന്റെ ആദർശനിഷ്ട സ്വജീവിതത്തിൽ പ്രാവർത്തികമായി തെളിയിക്കുക കൂടി അദ്ദേഹം ചെയ്തു.

പ്രവാചകന്‍ (സ) ഉദ്ദേശിച്ചിരുന്നെങ്കില്‍ അവരെ നിര്‍ബന്ധപൂര്‍വ്വം കൂടെ താമസിപ്പിക്കുവാന്‍ യാതൊരു തടസ്സവുമുണ്ടായിരുന്നില്ല. കാരണം അവിടുന്ന് ഇസ്‌ലാമിക സാമ്രാജ്യത്തിന്റെ അധിപനായിരുന്നു. രാജാക്കന്മാരും ചക്രവര്‍ത്തിമാരും ഒരു പെണ്ണിനെ ആഗ്രഹിച്ചു കഴിഞ്ഞാല്‍ അവളുടെ താല്‍പര്യം അന്വേഷിക്കുന്ന പതിവില്ലെന്ന് എല്ലാവര്‍ക്കുമറിയാവുന്ന വസ്തുതയാണ്. അതിനെതിരെ ഒരു ശബ്ദവുമവിടെ ഉയരുകയില്ല. ഇവിടെ പ്രവാചകന്‍ (സ) മാതൃകയാവുകയാണ്. താന്‍ വിവാഹം ചെയ്ത ഒരു സ്ത്രീക്ക് തന്നോടൊപ്പം ജീവിക്കുവാന്‍ താല്‍പര്യമില്ലെന്നറിഞ്ഞ നിമിഷം അവളെ ആശ്വസിപ്പിക്കുകയും സമാധാനിപ്പിക്കുകയും നിര്‍ഭയത്വത്തോടെ സ്വഗൃഹത്തിലേക്ക് മടങ്ങാന്‍ അവസരമൊരുക്കുകയും ചെയ്യുന്ന പ്രവാചകന്‍, ഒരു രാഷ്ട്രത്തിന്റെ ചോദ്യം ചെയ്യപ്പെടാത്ത ഭരണാധിപനാണെന്ന വസ്തുതയും ചേര്‍ത്തു മനസ്സിലാക്കുമ്പോള്‍ എത്രമാത്രം ആദരവും താല്‍പര്യവുമാണ് ആ വ്യക്തിത്വത്തിനോട് തോന്നേണ്ടത്. ഇസ്‌ലാം വിമര്‍ശകര്‍ക്ക് പക്ഷെ അത്തരം ഊഷ്മളമായ ചിന്തയും വികാരവുമൊന്നും ഉണ്ടാവുകയില്ല. കാരണം അവരുടെ ഹൃദയം കടുത്തു പോയിരിക്കുന്നു. ഊഷരമായ ചിന്തയും വികാരവുമാണ് അവരെ നയിക്കുന്നത്. വെറുപ്പും വിദ്വേഷവും മാത്രമാണ് അവരെ ഭരിക്കുന്നത്.

സമാധാനിപ്പിക്കാനായി -അതും സ്വന്തം ഭാര്യയുടെ മേൽ- കൈ വെക്കാൻ തുനിഞ്ഞതിനെ ‘കേറിപ്പിടിക്കലാക്കി’ ചിത്രീകരിക്കാനുള്ള ‘അപാരമായ കഴിവ് ‘ അഗമ്യഗമനത്തേയും, ബലാൽസംഗത്തേയും ശവരതിയേയുമെല്ലാം പ്രണയിക്കുന്നവരുടെ ‘ഹൈപ്പർ സെക്ഷ്വാലിറ്റിയുടെ'(Hypersexuality)ഭാഗമാണ്. ഇത്തരക്കാർക്ക് ഒരു തെറാപ്പിസ്റ്റിനെ സംഘടിപ്പിച്ച് കൊടുക്കുന്നതിന് പകരം സ്റ്റേജും, പേജും നൽകി സമൂഹത്തിലേക്ക് അഴിച്ചു വിടുന്നതാണ് ഭൗതികവാദികൾ മത വിശ്വാസികളോട് ചെയ്യുന്ന ഏറ്റവും വലിയ ദ്രോഹം.

ചരിത്രനിമിഷങ്ങളെ നാം തിരിച്ചൊന്ന് വിഭാവനം ചെയ്തു നോക്കൂ; ഉമൈമ: ബിന്‍ത് ശറാഹീലിന്റെ മനോഗതങ്ങളിലൂടെ. എന്തായിരിക്കും അവരുടെ ഹൃദയത്തില്‍ നിറഞ്ഞു നിന്ന വികാരങ്ങളും വിചാരങ്ങളും. തന്നെ വിവാഹം ചെയ്ത ആളോട്, അദ്ധേഹത്തെ തനിക്ക് ഇഷ്ടമല്ലെന്ന് അറിയിച്ചപ്പോള്‍ അവള്‍ കണ്ടത് ശാന്തവും മാന്യവുമായ പ്രതികരണമാണ്. ഒരു നിമിഷം പോലും അവളെ അദ്ദേഹത്തിന്റെ താല്‍പര്യങ്ങള്‍ക്കു വിധേയമാക്കാന്‍ ശ്രമിക്കാതെ, സമ്മാനങ്ങള്‍ നല്‍കി സ്വന്തം ഗൃഹത്തിലേക്ക് അവള്‍ക്ക് യാത്രാസൗകര്യമൊരുക്കിയ ഇസ്‌ലാമിക രാഷ്ട്രനായകനെ അവളുടെ ഹൃദയം എത്രമാത്രം ആദരിച്ചിട്ടുണ്ടാകും. തന്റെ നാട്ടിൽ തിരിച്ചെത്തിയ ഉമൈമ പ്രവാചകനുമായുള്ള ദാമ്പത്യ ജീവിതം ഉപേക്ഷിച്ചതിൽ പിന്നീട് ഖേദിക്കുകയും, ‘തനിക്ക് അമളി പറ്റി പോയി’ എന്ന് പറയുകയും ചെയ്തതായി ചരിത്രത്തിൽ തന്നെ കാണാം. (ഫത്ഹുൽ ബാരി: 9:314)

പക്ഷേ ഇസ്‌ലാം വിമര്‍ശകരുടെ വരണ്ട ഹൃദയങ്ങള്‍ക്ക് അത്തരം വിഭാവനങ്ങള്‍ അന്യമാണ്. ഹൃദയങ്ങളില്‍ അവര്‍ സൂക്ഷിച്ചിവെച്ചിരിക്കുന്ന ഇസ്‌ലാമിനോടുള്ള വെറുപ്പും വിദ്വേഷവും അവരെ എത്രമാത്രം ഊഷരവും വൃത്തിഹീനവുമായ മനോഗതിക്കാരാക്കിയിരുന്നു.! കഷ്ടം.!!

സ്ത്രീകള്‍ ബുദ്ധിയും മതവും കുറഞ്ഞവരാണെന്ന് ഹദീസുകള്‍ പഠിപ്പിക്കുന്നുണ്ട്. നബിപാഠങ്ങളില്‍ സ്ത്രീവിരുദ്ധതയുണ്ടെന്ന വിമര്‍ശനത്തെ അതു സാധൂകരിക്കുന്നില്ലെ?.

ഇസ്‌ലാം സ്ത്രീവിരുദ്ധമാണെന്ന് സ്ഥാപിക്കുവാന്‍ വിമര്‍ശകര്‍ സാധാരണയായി ഉന്നയിക്കാറുള്ള ഒന്നാണ് സ്ത്രീ ബുദ്ധിയും മതവും കുറഞ്ഞവളാണെന്ന് ഹദീസ് പഠിപ്പിക്കുന്നു എന്ന ആരോപണം. സന്ദര്‍ഭങ്ങളില്‍ നിന്നും സാഹചര്യങ്ങളില്‍ നിന്നും കാര്യങ്ങളെ അടര്‍ത്തിയെടുത്ത് തങ്ങളുടെ വക ദുര്‍വ്യാഖ്യാനങ്ങള്‍ ചമച്ചു കൊണ്ടാണ് പ്രസ്തുത ഹദീസ് വിമര്‍ശകര്‍ അവതരിപ്പിക്കാറ്. അതിനാല്‍ വിമര്‍ശന വിധേയമായ ഹദീസ് നമുക്ക് ആദ്യം പരിശോധന വിധേയമാക്കാം.

ഇമാം ബുഖാരിയും ഇമാം മുസ്‌ലിമും തങ്ങളുടെ സ്വഹീഹുകളില്‍ ഉദ്ധരിച്ച, അല്‍പം ദീര്‍ഘമായ ഒരു ഹദീസിന്റെ അവസാന ഭാഗമാണിത്. അതിപ്രകാരം ഭാഷാന്തരം ചെയ്യാം: ”….ദൃഢചിത്തരായ പുരുഷന്‍മാരുടെ ഹൃദയങ്ങളെ ഇളക്കുവാന്‍ ബുദ്ധിയും മതവും കുറഞ്ഞ നിങ്ങളെക്കാള്‍ കഴിവുള്ളവരെ ഞാന്‍ വേറെ കണ്ടിട്ടില്ല. സ്ത്രീകള്‍ ചോദിച്ചു: തിരുദൂതരേ! ബുദ്ധിയിലും മതത്തിലും ഞങ്ങള്‍ക്കെന്താണ് കുറവ്? അവിടുന്ന് അരുളി: സ്ത്രീയുടെ സാക്ഷ്യം പുരുഷന്റെ പകുതി സാക്ഷ്യത്തിന് തുല്യമായല്ലെ പരിഗണിക്കപ്പെടുന്നത് ? അവര്‍ പറഞ്ഞു. അതെ. പ്രവാചകന്‍ (സ) അരുളി: അതാണ് അവരുടെ ബുദ്ധി കുറവ്.

ആര്‍ത്തവമുണ്ടായാല്‍ സ്ത്രീ നമസ്‌കാരവും നോമ്പും ഉപേക്ഷിക്കുന്നില്ലേ? അവര്‍ പറഞ്ഞു: അതെ. പ്രവാചകന്‍ (സ) പ്രതിവചിച്ചു: അതാണ് അവരുടെ മതത്തിന്റെ കുറവ്.” (സ്വഹീഹുല്‍ ബുഖാരി, കിതാബുല്‍ ഹൈള്വ്, 6/304)

സ്ത്രീകള്‍ ബുദ്ധികുറഞ്ഞവരാണ് എന്ന പരാമര്‍ശത്തിന്റെ ഉദ്ദേശം ഹദീസില്‍ തന്നെ വിശദീകരിക്കുന്നുണ്ട്. ‘സ്ത്രീയുടെ സാക്ഷ്യം പുരുഷന്റെ പകുതി സാക്ഷ്യത്തിന് തുല്യമായല്ലെ പരിഗണിക്കപ്പെടുന്നത്?’. മതം കുറവാണ് എന്നുപറഞ്ഞതിന്റെ അര്‍ത്ഥവും ഹദീസ് വ്യക്തമാക്കി. ‘ആര്‍ത്തവമുണ്ടായാല്‍ സ്ത്രീ നമസ്‌കാരവും നോമ്പും ഉപേക്ഷിക്കുന്നില്ലേ?’. ആദ്യം നമുക്ക് മതത്തിലെ ‘കുറവി’നെപ്പറ്റി പരിശോധിക്കാം. കാരണം അതേ പ്രശ്‌നം മാത്രമാണ് ബുദ്ധികുറവിനെപ്പറ്റിയുള്ള ഹദീസിലെ പരാമര്‍ശത്തിലുമുള്ളത്. അഥവാ രണ്ടു പ്രശ്‌നവും സമാനമാണെന്നര്‍ത്ഥം.

മതം കുറവാണ് എന്നു പറഞ്ഞതുകൊണ്ടര്‍ത്ഥമാക്കുന്നത്; സ്ത്രീകള്‍ ദൈവഭക്തിയില്‍ കുറവുള്ളവരാണെന്നോ, ആചാരാനുഷ്ഠാനങ്ങള്‍ ശ്രദ്ധിക്കാത്തവരാണെന്നോ, മതശാസനകള്‍ പാലിക്കുന്നതില്‍ വീഴ്ച വരുത്തുന്നവരാണെന്നോ ഒന്നുമല്ല. പുരുഷന്മാരേക്കാള്‍ സ്ത്രീകള്‍ക്ക് മത ബാധ്യതകളും നിയമങ്ങളും കുറവാണ് എന്നാണ് ‘മതം കുറഞ്ഞവര്‍’ എന്നതുകൊണ്ടുദ്ദേശം. അഥവാ മതനിയമവും ബാധ്യതയുമായി ബന്ധപ്പെട്ട്, സ്ത്രീകള്‍ക്ക് പുരുഷന്മാരേക്കാള്‍ മതത്തില്‍ ഇളവ് / കുറവ് അനുവദിക്കപ്പെട്ടിരിക്കുന്നു.

പ്രസവ- ആര്‍ത്തവ കാലഘട്ടങ്ങളില്‍, സ്ത്രീകളുടെ സൃഷ്ടിപരവും പ്രകൃതിപരവുമായ തടസ്സങ്ങളാല്‍ മതാനുഷ്ടാനത്തില്‍ വരുന്ന കമ്മിയാണ് ‘കുറവ്’ (നുക്‌സാന്‍ النقصان) എന്നതുകൊണ്ട് വിശേഷിപ്പിച്ചിരിക്കുന്നത്. സ്ത്രീയുടെ കുറ്റംകൊണ്ട് സംഭവിക്കുന്ന കാര്യങ്ങളല്ല അതൊന്നും. അതുകൊണ്ട് തന്നെ അതവളുടെ വീഴ്ചയുമല്ല. പ്രസ്തുത പ്രശ്നത്തില്‍ സ്ത്രീകളെ ഹദീഥ് കുറ്റപ്പെടുത്തുന്നുമില്ല. കാരണം ദൈവിക നിര്‍ദേശമനുസരിച്ചാണ് വാസ്തവത്തില്‍ സ്ത്രീകള്‍ ആര്‍ത്ത-പ്രസവാനന്തര കാലത്ത് നമസ്‌കാരവും നോമ്പും ഉപേക്ഷിക്കുന്നത്. എന്നു മാത്രമല്ല, ദൈവ കല്‍പ്പനയോടുള്ള അനുസരണത്തിന് അവള്‍ക്ക് അക്കാര്യങ്ങളില്‍ പ്രതിഫലവും ലഭിക്കും. എന്നിരുന്നാലും ഫലത്തില്‍ പുരുഷന്മാരേക്കാള്‍ കുറവ് ആരാധനകളാണല്ലോ സ്ത്രീക്കുള്ളത്. അതുകൊണ്ട്, പുരുഷന്മാരേക്കാള്‍ സ്ത്രീകള്‍ക്ക് മതം കുറവാണ് എന്ന് പറയുന്നതില്‍ ഭാഷാപരമായി തെറ്റില്ല.

ഇവിടെ, നുക്‌സാന്‍ (النقصان) എന്നത് reduction (‘കുറവ്’) എന്ന അര്‍ത്ഥത്തിലാണ് പ്രയോഗിക്കപ്പെട്ടിരിക്കുന്നത്, deficiency (അപര്യാപ്തത, പോരായ്മ) എന്ന അര്‍ത്ഥത്തിലല്ല.

സമാനപ്രശ്നം തന്നെയാണ് ബുദ്ധിയുടെ കാര്യത്തിലും ഉള്ളത്. അഥവാ സ്ത്രീ യുക്തിവൈഭവം ഇല്ലാത്തവളാണെന്നോ, ചിന്താശേഷി കുറഞ്ഞവളാണെന്നോ, ഗ്രാഹ്യശേഷി നഷ്ടപ്പെട്ടവളാണെന്നോ ഒന്നുമല്ല ഹദീഥ് പഠിപ്പിക്കുന്നത്. മറിച്ച്, സാമ്പ്രദായികവും പ്രകൃതിപരവുമായ കാരണങ്ങളാല്‍ സ്ത്രീക്ക് പുരുഷനെ പോലെ ബൗദ്ധിക ക്ഷമത പുലര്‍ത്താന്‍ കഴിയാത്ത ചില സാഹചര്യങ്ങളും മേഖലകളുമുണ്ട് എന്നു മാത്രമാണ് ഉദ്ദേശിക്കപ്പെട്ടത്. ‘സ്ത്രീയുടെ സാക്ഷ്യം പുരുഷന്റെ പകുതി സാക്ഷ്യത്തിന് തുല്യമായല്ലെ പരിഗണിക്കപ്പെടുന്നത് ?’ എന്നതാണ് ‘ബുദ്ധിയുടെ കുറവ്’ എന്നതുകൊണ്ട് ഉദ്ദേശിച്ചത് എന്ന് ഹദീസില്‍ തന്നെ വിശദീകരിക്കപ്പെടുന്നുണ്ട്.

സ്ത്രീയുടെ എല്ലാ സാക്ഷ്യത്തിനും പുരുഷന്റെ പകുതി സാക്ഷ്യത്തിന്റെ സ്ഥാനമാണോ ഉള്ളത്. തീര്‍ച്ചയായും അല്ല. പിന്നെ ഏതു സാക്ഷ്യത്തിന്റെ കാര്യത്തിലാണ് നബി (സ) അങ്ങനെ പറഞ്ഞത്. അതറിയാന്‍ വിശുദ്ധ ക്വുര്‍ആനിലേക്ക് മടങ്ങണം; വിശുദ്ധ ക്വുര്‍ആനിലെ ഏറ്റവും വലിയ സൂക്തമായ ‘ആയത്തു ദൈനി’ലേക്ക്. അഥവാ കടമിടപാടുകള്‍ നടത്തുമ്പോള്‍ പാലിക്കേണ്ട മര്യാദ പഠിപ്പിക്കുന്ന സൂക്തത്തിലേക്ക്. അവിടെ വിശുദ്ധ ക്വുര്‍ആന്‍ പറഞ്ഞു: ”… ഇനി കടബാധ്യതയുള്ള ആള്‍ വിവേകമില്ലാത്തവനോ, കാര്യശേഷിയില്ലാത്തവനോ, (വാചകം) പറഞ്ഞുകൊടുക്കാന്‍ കഴിവില്ലാത്തവനോ ആണെങ്കില്‍ അയാളുടെ രക്ഷാധികാരി അയാള്‍ക്കുവേണ്ടി നീതിപൂര്‍വം (വാചകം) പറഞ്ഞുകൊടുക്കേണ്ടതാണ്. നിങ്ങളില്‍പെട്ട രണ്ട് പുരുഷന്‍മാരെ നിങ്ങള്‍ സാക്ഷി നിര്‍ത്തുകയും ചെയ്യുക. ഇനി ഇരുവരും പുരുഷന്‍മാരായില്ലെങ്കില്‍ നിങ്ങളിഷ്ടപ്പെടുന്ന സാക്ഷികളില്‍നിന്ന് ഒരു പുരുഷനും രണ്ടു സ്ത്രീകളും ആയാലും മതി. അവരില്‍ ഒരുവള്‍ക്ക് തെറ്റുപറ്റിയാല്‍ മറ്റവള്‍ അവളെ ഓര്‍മിപ്പിക്കാന്‍ വേണ്ടി.” (വി. ക്വുര്‍ആന്‍ 2:282)

കടമിടപാട് രംഗത്തെ സാക്ഷ്യത്തെപറ്റിയാണ് വിശുദ്ധ ക്വുര്‍ആനും വിമര്‍ശനവിധേയമായ ഹദീഥും പരാമര്‍ശിക്കുന്നത്. ഇവിടെയാണ് ഒരു പുരുഷന്റെ സാക്ഷ്യത്തിനുപകരം രണ്ടു സ്ത്രീകളുടെ സാക്ഷ്യം നിര്‍ദേശിക്കപ്പെട്ടിരിക്കുന്നത്. എന്തുകൊണ്ടാണ് അത്തരമൊരു നിര്‍ദേശം മുന്നോട്ടുവെക്കുന്നതെന്ന് ക്വുര്‍ആന്‍ വ്യക്തമാക്കി. ‘അവരില്‍ ഒരുവള്‍ക്ക് തെറ്റുപറ്റിയാല്‍ മറ്റവള്‍ അവളെ ഓര്‍മിപ്പിക്കാന്‍ വേണ്ടി.’ സ്ത്രീയുടെ സാക്ഷ്യത്തില്‍ തെറ്റുപറ്റാനുള്ള സാധ്യത ക്വുര്‍ആന്‍ എടുത്തുപറഞ്ഞത് സ്ത്രീവിരുദ്ധതയുടെ ഭാഗമായാണോ? ഒരിക്കലുമല്ല. സാമ്പ്രദായികവും പ്രകൃതിപരവുമായ ചിലകാരണങ്ങളാലാണ് അപ്രകാരം സൂചിപ്പിക്കപ്പെട്ടത്. കടമിടപാട് അഥവാ സാമ്പത്തികരംഗം, കച്ചവടം, ഗണിതം തുടങ്ങിയ മേഖലകളിലാണ് സ്ത്രീക്ക് മറവിയും തെറ്റും സംഭവിക്കാനുള്ള സാധ്യതയെ പരിഗണിച്ചതും, അതു പരിഹരിക്കാന്‍ രണ്ടു സ്ത്രീകളുടെ സാക്ഷ്യം ഒരു പുരുഷന്റെ സാക്ഷ്യത്തിന് തുല്യമായി കണക്കാക്കിയതും. അല്ലാതെ എല്ലാ മേഖലകളിലും സ്ത്രീക്ക് മറവിയും തെറ്റും സംഭവിക്കാമെന്നോ എല്ലാ വിഷയത്തിലും രണ്ടു സ്ത്രീകളുടെ സാക്ഷ്യം ഒരു പുരുഷന്റെ സാക്ഷ്യത്തിന് തുല്യമാണെന്നോ ഇസ്‌ലാം എവിടെയും പറഞ്ഞിട്ടില്ല. മറിച്ച് ഒട്ടനവധി വിഷയങ്ങളില്‍ സ്ത്രീയുടേയും പുരുഷന്റേയും സാക്ഷ്യം തുല്യമായി പരിഗണിച്ചിട്ടുമുണ്ട്. അപ്പോള്‍ സാമ്പത്തികരംഗം, ഗണിതം, വാണിജ്യം തുടങ്ങിയ മേഖലകളിലാണ് സ്ത്രീക്ക് പുരുഷന്മാരോളം ബൗദ്ധിക ക്ഷമത ഇല്ല എന്ന് ഖുര്‍ആനിലെ ‘ആയത്തു ദ്ദൈനും’, ചര്‍ച്ചാ വിധേയമായ ഹദീസും സൂചിപ്പിക്കുന്നത്. ഇതിന് ചില ന്യായമായ കാരണങ്ങളുണ്ട്:

ഒന്ന്, കച്ചവടം, സാമ്പത്തികം, ഗണിതം തുടങ്ങിയവ സാമ്പ്രദായികമായി പുരുഷന്മാരുടെ മേഖലകളാണ്; പ്രത്യേകിച്ച്, പ്രവാചകന്റെ (സ) ആഗമന കാലഘട്ടത്തിന് മുമ്പ്.

കുടുംബത്തിന്റെ ചെലവും സാമ്പത്തിക ബാധ്യതകളും പുരുഷന്റെ മേലാണ് ഇസ്‌ലാം നിര്‍ബന്ധമാക്കിയത്. സ്ത്രീക്ക് അവ ഐച്ഛികമാണ്. അതുകൊണ്ട് പ്രവാചകന്റെ(സ) ആഗമനത്തിന് ശേഷവും ഈ മേഖലകളില്‍ പുരുഷന്മാര്‍ തന്നെ വിഹരിച്ചു.

സാമ്പ്രദായികമായി, സ്ത്രീകള്‍ ബന്ധപ്പെടാത്ത മേഖലയായതു കൊണ്ട് തന്നെ തത് വിഷയങ്ങളില്‍ സ്ത്രീകള്‍ക്ക് മറവിയും തെറ്റും സംഭവിക്കാനുള്ള സാധ്യതയെ പരിഗണിക്കുകയും, അതു പരിഹരിക്കാന്‍ രണ്ടു സ്ത്രീകളുടെ സാക്ഷ്യം ഒരു പുരുഷന്റെ സാക്ഷ്യത്തിന് തുല്യമായി കണക്കാക്കുകയും ചെയ്തു. സ്വഭാവികമായും അന്നത്തെ സ്ത്രീകള്‍ ഇത്തരം മേഖലകളില്‍ പുരുഷന്മാരേക്കാള്‍ അറിവും ബൗദ്ധിക പാടവവും കുറഞ്ഞവരായിരുന്നു എന്ന സാമൂഹിക വസ്തുതയെ ഉദ്ധരിക്കുക മാത്രമാണ് പ്രവാചകന്‍ (സ) ചെയ്തത്. അല്ലാതെ അതേ അവസ്ഥ തുടരുമെന്നോ തുടരണമെന്നോ പ്രവാചകന്‍ (സ) അനുശാസിച്ചിട്ടില്ല. എന്ന് മാത്രമല്ല, സ്ത്രീ വിദ്യഭ്യാസത്തെ പ്രോത്സാഹിപ്പിച്ച വ്യക്തിത്വമാണ് അദ്ദേഹം. ‘ഒരാളുടെ അടുക്കല്‍ ഒരു അടിമ സ്ത്രീ ഉണ്ടാവുകയും അവളെ ഏറ്റവും നല്ല രീതിയില്‍ പഠിപ്പിക്കുകയും ഏറ്റവും നല്ല വിദ്യാഭ്യാസം നല്‍കുകയും ശേഷം മോചിപ്പിക്കുകയും വിവാഹം കഴിക്കുകയും ചെയ്താല്‍ ഇരട്ടി പ്രതിഫലമുണ്ട്’ എന്ന പ്രവാചക വചനം ഉദാഹരണം. (സ്വഹീഹുല്‍ ബുഖാരി: 3446)

അടിമ സ്ത്രീക്ക് പോലും വിദ്യാഭ്യാസം നല്‍കല്‍ പ്രതിഫലാര്‍ഹമായ പുണ്യകര്‍മ്മമാണെന്ന് പഠിപ്പിച്ച മതമാണ് ഇസ്‌ലാം.

ഹദീസിന്റെ പശ്ചാത്തലവും പ്രസക്തമാണ്. ഒരു പൊതുവേദിയിലല്ല പ്രവാചകന്‍ (സ) ഇതു സംസാരിക്കുന്നത്. സ്ത്രീകള്‍ മാത്രമുള്ള, സ്ത്രീകള്‍ക്ക് വേണ്ടി മാത്രം ഒരുക്കപ്പെട്ട ഒരു സദസ്സില്‍ സ്ത്രീകളെ ഗുണദോഷിക്കുകയാണ് പ്രവാചകന്‍ (സ).

‘അല്ലാഹുവിന്റെ ദൂതന്‍ (സ) പെരുന്നാള്‍ ദിവസം നമസ്‌ക്കാരസ്ഥലത്തേക്ക് പുറപ്പെട്ടു. (നമസ്‌ക്കാരത്തില്‍ നിന്ന്) പിരിഞ്ഞ ശേഷം അദ്ദേഹം ജനങ്ങള്‍ക്ക് (പൊതുവായി) ഉപദേശം നല്‍കി. അവരോട് ദാനധര്‍മ്മം ചെയ്യാനായി കല്‍പ്പിച്ചു. അദ്ദേഹം പറഞ്ഞു: ‘ജനങ്ങളേ, നിങ്ങള്‍ ദാനധര്‍മ്മം നിര്‍വ്വഹിക്കുക…’ എന്നിട്ട് അദ്ദേഹം സ്ത്രീകളുടെ അടുത്തു കൂടി കടന്നുവന്നു, എന്നിട്ട് (അവരെ ഉപദേശിച്ചു കൊണ്ട്) പറഞ്ഞു: ‘ഓ, സ്ത്രീ സമൂഹമേ, നിങ്ങള്‍ ദാനധര്‍മ്മം നിര്‍വ്വഹിക്കുക…’ (സ്വഹീഹുല്‍ ബുഖാരി: 1462)

തുടര്‍ന്ന് പ്രവാചകന്‍ (സ) സ്ത്രീകള്‍ക്ക് മാത്രം പ്രത്യേകമായി നല്‍കിയ ഉത്‌ബോധനങ്ങള്‍ക്കിടയിലാണ് ചര്‍ച്ചാ വിഷയകമായ വാചകം പറയുന്നത് എന്നോര്‍ക്കണം. ഇവിടെ, പുരുഷന്മാരടങ്ങുന്ന പൊതു സദസ്സില്‍ സ്ത്രീകളെ അവമതിക്കുകയോ, സ്ത്രീ പുരുഷ വര്‍ഗ ശ്രേഷ്ഠതകളേയും ശക്തി ക്ഷയങ്ങളേയും താരതമ്യ പഠനത്തിന് വിധേയമാക്കുകയോ, പുരുഷ മേല്‍കോയ്മക്കും വര്‍ഗ ശ്രേഷ്ഠതക്കും താത്ത്വികമായ അടിത്തറ അവതരിപ്പിക്കുകയോ ഒന്നുമല്ല പ്രവാചകന്‍ (സ) ഉദ്ദേശിച്ചത്. അതായിരുന്നു പ്രവാചകന്റെ ഉദ്ദേശമെങ്കില്‍, പെരുന്നാള്‍ ദിവസം ആദ്യമായി നടത്തിയ പൊതു പ്രഭാഷണത്തിലായിരുന്നു പ്രവാചകന്‍ (സ) ഈ പ്രസ്താവന നടത്തുമായിരുന്നത് എന്ന് ചിന്താശേഷിയുള്ള ഏവര്‍ക്കും എളുപ്പത്തില്‍ മനസ്സിലാക്കാവുന്നതേയുള്ളു. പ്രത്യുത, ഈ ‘വഅള്’ (الوعظ) അഥവാ ഉപദേശ പ്രസംഗത്തിന്റെ ഉദ്ദേശം സ്ത്രീകള്‍ക്ക് ആത്മ വിമര്‍ശനത്തിന്റേയും ആത്മ വിചിന്തനത്തിന്റേയും വാതിലുകള്‍ തുറന്നു കൊടുക്കുകമാത്രമായിരുന്നു.

ഇവിടെയും, നുക്‌സാന്‍ (النقصان) എന്നത് reduction (‘കുറവ്’) എന്ന അര്‍ത്ഥത്തിലാണ് പ്രയോഗിക്കപ്പെട്ടിരിക്കുന്നത്, deficiency (അപര്യാപ്തത, പോരായ്മ) എന്ന അര്‍ത്ഥത്തിലല്ല. കാരണം, ബുദ്ധി (intelligence) എന്നത് ജന്മസിദ്ധമായ ബുദ്ധി (inborn intelligence), ആര്‍ജ്ജിത ബുദ്ധി (acquired intelligence) എന്നിങ്ങനെ രണ്ടായി തിരിക്കാം. ഇവിടെ ഹദീസിലെ സംബോധനം ആര്‍ജ്ജിത ബുദ്ധിയെ സംബന്ധിച്ചാണ്. അതിന്റെ കുറവ് ചൂണ്ടികാണിക്കുന്നതില്‍, സ്ത്രീകളുടെ ബുദ്ധി ജൈവശാസ്ത്രപരമായി അപര്യാപ്തമാണെന്ന സൂചനയില്ല.

അപ്പോള്‍, ചില മേഖലകളില്‍ ബുദ്ധി വൈഭവത്തില്‍ പുരുഷന്മാര്‍ക്ക് പിന്നിലാണ് സ്ത്രീകളെന്ന് അവരെ ബോധവല്‍ക്കരിക്കുകയും അത്തരം ന്യൂനതകള്‍ പരിഹരിക്കാതെ, പകരം സ്‌ത്രൈണതയെ പുരുഷന്മാരെ വശീകരിക്കുന്നതില്‍ കേന്ദ്രീകരിക്കുകയും ചെയ്യുന്നത് ആത്മനിന്ദയും സ്വയം നശീകരണവുമാണെന്നും അവരെ പഠിപ്പിക്കുകയാണ് പ്രവാചകന്‍ (സ).

രണ്ട്, സാമ്പത്തികം (Economics, ഗണിതം (Mathematics, യന്ത്രശാസ്ത്രം (Engineering) തുടങ്ങിയ പല വിഷയങ്ങളിലും പ്രകൃതിസഹജമായ കാരണങ്ങളാല്‍ തന്നെ സ്ത്രീകളേക്കാള്‍ പുരുഷന്മാര്‍ ബൗദ്ധിക ക്ഷമതയും മികവും പുലര്‍ത്തുന്നുവെന്ന് സൂചിപ്പിക്കുന്ന അക്കാദമിക ഗവേഷണങ്ങളും ശാസ്ത്രപഠനങ്ങളും തന്നെ സുലഭമാണ്. അതിനെതിരായും ശാസ്ത്രീയ നിരൂപണങ്ങളും പഠനങ്ങളും നിലനില്‍ക്കുന്നുണ്ട് എന്നത് വിസ്മരിക്കുന്നില്ല. പക്ഷെ അവ തമ്മില്‍ മാറ്റുരക്കുക എന്നത് വിശാലമായ മറ്റൊരു മേഖലയാണ്. അതിന് ഇവിടെ മുതിരുന്നില്ല. മറിച്ച്, പല മേഖലകളിലും പുരുഷന്മാര്‍ സ്ത്രീകളേക്കാള്‍ ബുദ്ധിചാതുര്യം പ്രകടിപ്പിക്കുന്നു എന്ന് സൂചിപ്പിക്കുന്ന അധുനിക ഗവേഷണങ്ങളും പഠനങ്ങളും തന്നെ നിലനില്‍ക്കെ, പൗരാണിക സ്ത്രീ സമൂഹത്തിന്റെ സാമൂഹിക പരിതസ്ഥിതി തുറന്നു കാണിച്ച്, സ്ത്രീകള്‍ക്ക് ആത്മ വിമര്‍ശനത്തിന് ചിന്താ വേദിയൊരുക്കിയ പ്രവാചകന്‍ (സ) എങ്ങനെ സ്ത്രീ വിരുദ്ധനാകും?!

മൂന്ന്, സ്ത്രീകള്‍ക്ക് ചില പ്രത്യേക സന്ദര്‍ഭങ്ങളില്‍ യുക്തിവിചാരശേഷിയില്‍ (logical reasoning) കുറവ് സംഭവിക്കുവാന്‍ സാധ്യതയുണ്ടെന്നത് ഇന്നു പല ശാസ്ത്ര പഠനങ്ങളും വ്യക്തമാക്കിയിട്ടുള്ള കാര്യമാണ്. ആര്‍ത്തവത്തിന് തൊട്ടുമുമ്പുള്ള ദിവസങ്ങളിലെ മനഃസംഘര്‍ഷം, ഗര്‍ഭധാരണത്തിന്റെ ആദ്യനാളുകളിലെ മാനസിക പ്രശ്നങ്ങള്‍, ആര്‍ത്തവ വിരാമത്തോടനുബന്ധിച്ചുള്ള മാനസിക പ്രയാസങ്ങള്‍, പ്രസവകാലത്തെ പ്രശ്നങ്ങള്‍, ഗര്‍ഭഛിദ്രമുണ്ടാക്കുന്ന മാനസികാഘാതം ഇങ്ങനെ ഒട്ടനവധി പ്രശ്നങ്ങള്‍ സ്ത്രീകള്‍ മാത്രം നേരിടുന്നവയാണ്. ഈ സാഹചര്യങ്ങളില്‍ ശാരീരിക പ്രയാസങ്ങള്‍ക്കുപുറമെ മനോമാന്ദ്യം (slow mindedness, ഏകാഗ്രതയില്ലായ്മ, ഓര്‍മക്കുറവ് എന്നിങ്ങനെ ഒട്ടനവധി മാനസിക പ്രശ്നങ്ങള്‍ക്ക് പല സാഹചര്യങ്ങളിലും സ്ത്രീകള്‍ വിധേയരാകുന്നുണ്ടെന്നത് ശാസ്ത്രീയ ഗവേഷണങ്ങള്‍ വ്യക്തമാക്കുന്നുണ്ട്.

മാത്രമല്ല സ്ത്രീകളുടെ ബുദ്ധി വൈകാരിക പ്രധാനവും പുരുഷന്റേത് വൈചാരിക പ്രധാനവുമാണെന്നും പുതിയകാല ശാസ്ത്രഗവേഷണങ്ങള്‍ അടിവരയിടുന്നുണ്ട് (www.telegraph.co.uk). സാക്ഷ്യപ്രഖ്യാപന വേളയില്‍ വൈകാരികാന്തരീക്ഷം സംജാതമായാല്‍ അതും സാക്ഷ്യത്തെ പ്രതികൂലമായി ബാധിക്കാന്‍ സാധ്യതയുള്ള ഒരു മേഖലയാണ്. പ്രകൃതിപരമായി തന്നെ സ്ത്രീയില്‍ ഈ അബലതകള്‍ ഉള്ളതുകൊണ്ടാകാം ‘അവരില്‍ ഒരുവള്‍ക്ക് തെറ്റുപറ്റിയാല്‍ മറ്റവള്‍ അവളെ ഓര്‍മിപ്പിക്കാന്‍ വേണ്ടി’ എന്നു ക്വുര്‍ആന്‍ പ്രത്യേകം എടുത്തുപറയുന്നത്. ഇത് സ്ത്രീയെ അവമതിക്കലല്ല; പ്രകൃതിപരമായി അവളുടെ അബലതകളെ ഉള്‍ക്കൊള്ളലാണ്.

എന്നാല്‍ സാക്ഷ്യത്തിന്റെ കാര്യത്തില്‍ എല്ലാ രംഗത്തും ഈ സമീപനം ഇസ്‌ലാം പഠിപ്പിക്കുന്നുണ്ടോ? ഇല്ല എന്നുമാത്രമല്ല പല വിഷയങ്ങളും ഏകസ്ത്രീ സാക്ഷ്യം സ്വഹാബിമാരും താബിഉകളും സ്വീകരിച്ചിട്ടുണ്ട്.

പ്രവാചക ശിഷ്യന്‍ അബൂ മൂസാ (റ) പറഞ്ഞു: ‘ഏതൊരു ഹദീഥിനെ സംബന്ധിച്ചും ഞങ്ങള്‍, പ്രവാചകാനുചരന്‍മാര്‍ക്കിടയില്‍ എന്ത് പ്രശ്നവും സംശയവും ഉദിച്ചാലും, അതിനെപ്പറ്റി ആഇശ(റ)യോട് ഞങ്ങള്‍ ആരാഞ്ഞാല്‍ അവരുടെ അടുക്കല്‍ അതിനെ സംബന്ധിച്ച് അറിവ് ഉണ്ടാകാതിരുന്നിട്ടില്ല; ഒരിക്കലും.” (സുനനു തിര്‍മിദി: 3883).

”മസ് റൂക് പറഞ്ഞു: എന്റെ ആത്മാവ് ആരുടെ കയ്യിലാണോ അവന്‍ തന്നെ സത്യം. മുഹമ്മദ് നബി(സ)യുടെ അനുചരന്‍മാരില്‍ മഹാപണ്ഡിതരായ തലമുതിര്‍ന്നവര്‍ ആഇശ(റ)യോട് അനന്തരാവകാശ നിയമങ്ങളെ സംബന്ധിച്ച് ചോദിച്ചു പഠിക്കുന്നത് ഞാന്‍ കണ്ടിട്ടുണ്ട്.” (മുസ്വന്നഫ് ഇബ്നു അബീ ശൈബ: 30387).

ഇമാം ദഹബി (റ) പറഞ്ഞു: ”സ്ത്രീകളായ ഹദീഥ് നിവേദകരില്‍ ആരെയും ഹദീഥ് പണ്ഡിതന്‍മാര്‍ കളവ് ആരോപിച്ചതായോ, നിവേദനത്തിന് പറ്റാത്തവരായി കണ്ട് ഉപേക്ഷിച്ചവരായോ ഞാന്‍ അറിഞ്ഞിട്ടില്ല.” (മീസാനുല്‍ ഇഅ്തിദാല്‍: 4/604).

സ്ത്രീയെ പൊതുവായി ബുദ്ധിയില്ലാത്തവളായും ഏക സാക്ഷ്യത്തിന് കൊള്ളാത്തവളായും ഇസ്‌ലാം പരിഗണിച്ചിരുന്നുവെങ്കില്‍ ആഇശ(റ)യുടേയും മറ്റു ഹദീസ് നിവേദകരായ സ്ത്രീ രത്‌നങ്ങളുടേയും വാതില്‍ക്കല്‍ ജ്ഞാനത്തിനായി പണ്ഡിത പുരുഷ വൃന്ദം തടിച്ചു കൂടുമായിരുന്നില്ലല്ലോ.

ഇമാം ശൗക്കാനി പറഞ്ഞു : "സ്ത്രീയാണെന്ന കാരണത്താൽ, ഒരു സ്ത്രീ പറഞ്ഞുതന്ന വിവരങ്ങൾ (ഹദീഥ്) പണ്ഡിതന്മാർ തള്ളിയതായി ഒരൊറ്റ പണ്ഡിതന്മാരിൽ നിന്നും ഉദ്ധരിക്കപ്പെട്ടിട്ടില്ല. എത്രയെത്ര ഹദീസുകളാണ് ഒരൊറ്റ സ്വഹാബിവനിതയിൽ നിന്നും കേട്ടു എന്നതിനാൽ മുസ്‌ലിം സമൂഹം മൊത്തം അതു സ്വീകരിച്ചതായിട്ടുള്ളത്. ഹദീസ് ശാസ്ത്രത്തെപ്പറ്റി അല്പമെങ്കിലും വിവരമുള്ളവരാരും ഇത് നിഷേധിക്കുകയില്ല.'' (നൈലുൽ ഔത്വാർ 6/360)

പല വിഷയങ്ങളിലും ഏക സ്ത്രീ സാക്ഷ്യം ഇസ്‌ലാം പരിഗണിച്ചിട്ടുണ്ട്. എന്നാല്‍ കടമിടപാട് രംഗത്ത് മാത്രമാണ് രണ്ടു സ്ത്രീ സാക്ഷ്യത്തെ ഒരു പുരുഷ സാക്ഷ്യത്തിനു പകരമായി പരിഗണിച്ചിരിക്കുന്നത്. അത് നാം മുമ്പ് സൂചിപ്പിച്ചതുപോലെ അവളുടെ പ്രകൃതിപരമായ അബലതകള്‍ മൂലം വല്ല മറവിയോ അബദ്ധമോ സംഭവിക്കാനുള്ള സാധ്യതയെ പരിഗണിച്ചുകൊണ്ടുള്ള നിലപാട് മാത്രമാണ്. സ്ത്രീയുടെ ബുദ്ധിയെയോ ഓര്‍മയെയോ പ്രതികൂലമായി സ്വാധീനിക്കാനിടയുള്ള ഇത്തരം പ്രകൃതിപരമായ പ്രശ്‌നങ്ങളോ, വൈകാരികപ്രധാനമായ ചിന്താശേഷിയോ ഭയപ്പെടേണ്ട സാഹചര്യമില്ലെങ്കില്‍ പ്രസ്തുത മേഖലയിലും ഏകസ്ത്രീ സാക്ഷ്യം പരിഗണിക്കപ്പെടണമെന്നത് പണ്ഡിതന്‍മാരില്‍ പലരും വ്യക്തമാക്കിയിട്ടുള്ള കാര്യമാണ്. ശൈഖുല്‍ ഇസ്‌ലാം ഇബ്നു തീമിയ്യ (റ) പറഞ്ഞു: ”സ്ത്രീകളുടെ സാക്ഷ്യങ്ങളില്‍ പൊതുവെ തെറ്റ് ഭയപ്പെടാത്ത എന്തെങ്കിലുമുണ്ടെങ്കില്‍, അതില്‍ അവരെ പുരുഷന്റെ പകുതിയായി കണക്കാക്കില്ല. അവരുടെ സാക്ഷ്യം സമ്പൂര്‍ണ സാക്ഷ്യമായാണ് പരിഗണിക്കപ്പെടുക.” (അത്തുറുകുല്‍ ഹുകുമിയ്യ: 1/128).

”സത്യസന്ധത, വിശ്വസ്തത, ഭക്തി എന്നിവയില്‍ സ്ത്രീ പുരുഷന് തുല്യമാണ്. അവള്‍ മറക്കുമെന്നോ അല്ലെങ്കില്‍ തെറ്റിദ്ധരിക്കുമെന്നോ ഭയപ്പെടുന്ന വിഷയങ്ങളില്‍ ഒഴികെ, അത്തരം വിഷയങ്ങളില്‍ അവളുടെ ഒറ്റക്കുള്ള സാക്ഷ്യം തന്നെപ്പോലെയുള്ള മറ്റൊരാളുടെ സാക്ഷ്യംകൊണ്ട് ശക്തിപ്പെടുന്നു. അപ്പോള്‍ അവരുടെ രണ്ടുപേരുടെയും ഐക്യകണ്‌ഠേനയുള്ള സാക്ഷ്യം ഒരു പുരുഷന്റെ സാക്ഷ്യത്തെക്കാളും ശക്തിയുള്ളതായി പരിണമിക്കുന്നു.” (അത്തുറുകുല്‍ ഹുകുമിയ്യ: 1/136).

സ്ത്രീ ബുദ്ധിയും മതവും കുറഞ്ഞവളാണെന്ന ഹദീഥിന്റെ താല്‍പര്യം വളരെ വ്യക്തമാണ്. കച്ചവടം, സാമ്പത്തികം തുടങ്ങി സാമ്പ്രദായികമായി, സ്ത്രീകള്‍ ബന്ധപ്പെടാത്ത മേഖലകളില്‍ സ്ത്രീകള്‍ പുരുഷന്മാരേക്കാള്‍ അറിവും ബൗദ്ധിക പാടവവും കുറഞ്ഞവരായിരുന്നു എന്ന സാമൂഹിക വസ്തുതയെ ഉദ്ധരിക്കുക മാത്രമാണ് പ്രവാചകന്‍ (സ) ചെയ്തത്. അല്ലാതെ പൊതു സദസ്സില്‍ സ്ത്രീകളെ അവമതിക്കുകയോ, സ്ത്രീ പുരുഷ വര്‍ഗ ശ്രേഷ്ഠതകളേയും വൃദ്ധിക്ഷയങ്ങളേയും താരതമ്യ പഠനത്തിന് വിധേയമാക്കുകയോ, പുരുഷ മേല്‍കോയ്മക്കും വര്‍ഗ ശ്രേഷ്ഠതക്കും താത്ത്വികമായി അടിത്തറ സ്ഥാപിക്കുകയോ ഒന്നുമല്ല പ്രവാചകന്‍ (സ) ചെയ്തത്. സ്ത്രീകള്‍ക്ക് ആത്മ വിമര്‍ശനത്തിന്റേയും ആത്മ വിചിന്തനത്തിന്റേയും വാതിലുകള്‍ തുറന്നു കൊടുക്കുകമാത്രമായിരുന്നു. അതിനാല്‍, നബി പാഠങ്ങളില്‍നിന്നും സ്ത്രീവിരുദ്ധത ചിക്കിചികയുന്നവര്‍ക്ക് പ്രസ്തുത ഹദീഥില്‍ നിന്ന് ഒന്നും തരപ്പെടില്ലെന്നര്‍ത്ഥം.

സ്ത്രീയും സുഗന്ധദ്രവ്യവും

നിഷേധിക്കാനാവാത്ത വിധം പ്രകടമാണ് ഇസ്‌ലാമിന്റെ സ്ത്രീവിരുദ്ധ നിലപാടുകള്‍. പെണ്ണവകാശങ്ങള്‍ക്കു നേരെ എന്നും കണ്ണടച്ചു നില്‍ക്കാനാണ് ഇസ്‌ലാമിനിഷ്ടം. പെണ്‍ഹിതങ്ങള്‍ക്കു നേരെ പുറം തിരിഞ്ഞു നില്‍ക്കുന്ന ഇസ്‌ലാമിക സമീപനങ്ങള്‍ക്ക് ഉത്തമ ഉദാഹരണമാണ് സുഗന്ധം ഉപയോഗിക്കുന്നതില്‍ നിന്നും സ്ത്രീയെ വിലക്കിയ അതിന്റെ സമീപനം. സ്ത്രീക്ക് സുഗന്ധം വിരോധിക്കുക മാത്രമല്ല, അതു ഉപയോഗിക്കുന്ന പെണ്ണിനെ വ്യഭിചാരിണിയായി പ്രഖ്യാപിക്കുക കൂടി ചെയ്യുന്നുണ്ട് ഇസ്‌ലാം. സുഗന്ധമുപയോഗിക്കുക എന്ന മനുഷ്യസഹജമായ ഒരു താല്‍പര്യത്തെ പുരുഷന്മാരുടെ കാര്യത്തില്‍ ഏറെ പരിഗണിച്ച ഇസ്‌ലാം, സ്ത്രീയുടെ കാര്യം എത്തുമ്പോള്‍ നിഷേധഭാവം സ്വീകരിക്കുന്നു. മാനവികതയുടെ മതമാണ് ഇസ്‌ലാം എന്ന ജല്‍പ്പനം എത്രമാത്രം സത്യവിരുദ്ധമാണെന്ന് സുഗന്ധത്തിന്റെ കാര്യത്തിലുള്ള ഈ ഇരട്ടത്താപ്പ് വ്യക്തമാക്കുന്നുണ്ട്.

ഇസ്‌ലാംവിമര്‍ശകര്‍ പ്രത്യേകിച്ച് ഫെമിനിസ്റ്റുകളും യുക്തിവാദികളും സര്‍വ്വസാധാരണയായി ഉദ്ധരിക്കാറുള്ള ഹദീസുകളാണ് സ്ത്രീയെ സുഗന്ധം ഉപയോഗിക്കുന്നതില്‍ നിന്നും കര്‍ശനമായി വിലക്കുന്ന നബിവചനങ്ങള്‍. സുഗന്ധം ഉപയോഗിക്കുന്ന കാര്യത്തില്‍ പുരുഷന്മാരെ പ്രോത്സാഹിപ്പിക്കുകയും സ്ത്രീകളെ വിലക്കുകയും ചെയ്യുന്ന ഹദീസുകള്‍ തീര്‍ത്തും മാനവിക വിരുദ്ധമായ നിലപാടാണെന്നതാണ് വിമര്‍ശകരുടെ ആരോപണം. വാസ്തവത്തില്‍ പ്രസ്തുത വിഷയത്തില്‍ ഇസ്‌ലാമിന്റെ സമീപനമെന്താണ്? വിമര്‍ശകര്‍ ഉന്നയിക്കുന്ന വിധം മാനവിക വിരുദ്ധമായ സമീപനമാണൊ ഇസ്‌ലാം ഇവിടെ സ്വീകരിച്ചിരിക്കുന്നത്? സ്ത്രീയെ സുഗന്ധം ഉപയോഗിക്കുന്നതില്‍ നിന്നും വിലക്കുക വഴി എന്ത് യുക്തിയാണ് ഇസ്‌ലാം മുന്നോട്ടുവെക്കുന്നത്? ചര്‍ച്ച ചെയ്യപ്പെടേണ്ട ചിന്തകള്‍ തന്നെയാണിത്. അതിനാല്‍ പ്രസ്തുത രംഗത്ത് ഇസ്‌ലാമിക സമീപനവും യുക്തിയും വിശദമായി തന്നെ നമുക്ക് ചര്‍ച്ച ചെയ്യാം.

”ഏതൊരു സ്ത്രീ സുഗന്ധം ഉപയോഗിക്കുകയും എന്നിട്ട് (പുരുഷ) സമൂഹത്തിനരികിലൂടെ അവളുടെ സുഗന്ധം അവര്‍ക്ക് ലഭിക്കാനായി നടന്നു പോവുകയും ചെയ്താല്‍ അവള്‍ വ്യഭിചാരിണിയാണ്. (അവളെ ലൈംഗിക തൃഷ്ണയോടെ നോക്കുന്ന) എല്ലാ കണ്ണുകളും വ്യഭിചരിക്കുന്നതാണ്.” മുസ്‌നദ് അഹ്മദ് (19726), സ്വഹീഹു ഇബ്‌നു ഹിബ്ബാന്‍ (4422), സ്വഹീഹു ഇബ്‌നു ഖുസൈമ (1681), സ്വഹീഹുല്‍ ജാമിഅ് (2701) തുടങ്ങിയ ഹദീസ് ഗ്രന്ഥങ്ങളും അല്ലാത്തവയും പ്രസ്തുത ഹദീസ് വിവിധരൂപങ്ങളില്‍ നിവേദനം ചെയ്തിട്ടുണ്ട്. പ്രസ്തുത ഹദീസ് പരിശോധന വിധേയമാക്കുമ്പോള്‍ ബോധ്യപ്പെടുന്ന വസ്തുതകളെ ഇപ്രകാരം സംഗ്രഹിക്കാം.

1, ഹദീസില്‍ വ്യഭിചാരവുമായി ചേര്‍ത്തു പരാമര്‍ശിച്ചത് സ്ത്രീയെ മാത്രമല്ല അവളെ ലൈംഗിക തൃഷ്ണയോടെ നോക്കുന്ന പുരുഷന്മാരെയും ചേര്‍ത്തുകൊണ്ടാണ്.

2, കേവലം സുഗന്ധം ഉപയോഗിച്ചതിനാലല്ല സ്ത്രീയെ ഹദീസ് വിമര്‍ശിക്കുന്നത് പ്രത്യുത അവളത് ഉപയോഗിച്ചത് പരപുരുഷന്മാരെ വശീകരിക്കാനാണ്.

3, പരപുരുഷന്മാരെ ആകര്‍ഷിക്കുവാനും വശീകരിക്കുവാനും വേണ്ടി ചില സ്ത്രീകള്‍ സുഗന്ധം ഉപയോഗിക്കുന്ന സമ്പ്രദായം ജാഹിലിയ്യാ കാലഘട്ടത്തില്‍ (ഇസ്‌ലാം പൂര്‍വ്വ കാലഘട്ടം) പതിവായിരുന്നു. അതിനെ പറ്റിയാണ് ഹദീസ് പരാമര്‍ശിക്കുന്നത്.

4, പരപുരുഷന്മാരെ ആകര്‍ഷിക്കുകയും വശീകരിക്കുകയും ചെയ്യുക എന്ന ഉദ്ദേശപൂര്‍വ്വം സുഗന്ധദ്രവ്യങ്ങള്‍ ഉപയോഗിക്കുന്ന സമ്പ്രദായത്തെ കേവലം സാധാരണ സുഗന്ധം ഉപയോഗിക്കുന്ന രീതിയുമായി താരതമ്യം ചെയ്യുന്നത് അടിസ്ഥാനരഹിതമാണ്.

5, പുരുഷനെ ആകര്‍ഷിക്കുവാനും വശീകരിക്കുവാന്‍ ഉപയോഗിക്കുന്ന വസ്തുക്കളില്‍ പ്രഥമസ്ഥാനത്ത് നില്‍ക്കുന്ന ഒന്നാണ് സുഗന്ധ ദ്രവ്യങ്ങള്‍. ഇന്നും പല സുഗന്ധ ദ്രവ്യങ്ങളുടേയും മേന്മയായി അതു പുരുഷന്മാരെ വശീകരിക്കാന്‍ ശേഷിയുള്ളതാണ് എന്ന് പല പെര്‍ഫ്യൂം കമ്പനികളും പരസ്യം ചെയ്യുന്നത് അതുകൊണ്ടാണ്. www.best-selling-perfume-for-women.com തങ്ങളുടെ 20 പെര്‍ഫ്യൂമുകളെ പരിചയപ്പെടുത്തിയത് 20 best sexy women perfume to seduce a man in 2020 എന്നാണ്.

6, വ്യഭിചാരത്തിന്റെ സകല കവാടങ്ങളും കൊട്ടിയടക്കാന്‍ കര്‍ശനമായ ഉപാധികള്‍ സ്വീകരിക്കുന്ന ഒരു മതത്തിന് വശീകരണോദ്ദേശത്തോടെ സ്ത്രീകള്‍ സുഗന്ധദ്രവ്യങ്ങള്‍ ഉപയോഗിക്കുന്നതിനെ ശക്തമായി വിമര്‍ശിക്കാതിരിക്കുവാന്‍ സാധ്യമല്ല. ഫെമിനിസ്റ്റുകള്‍ക്കും യുക്തിവാദികള്‍ക്കും അതിലെ ധാര്‍മികതയും യുക്തിയും ബോധ്യപ്പെട്ടില്ലെങ്കില്‍ പോലും ഇസ്‌ലാമിന്റെ നിലപാട് കണിശമാണ്. അത്തരം സ്ത്രീകള്‍ വ്യഭിചാരിണികള്‍ മാത്രമാണെന്നല്ല ഇസ്‌ലാം പഠിപ്പിക്കുന്നത്; മറിച്ച് അവള്‍ക്ക് സ്വര്‍ഗ്ഗത്തിന്റെ പരിമളം പോലും നിഷേധിക്കപ്പെടുമെന്നുകൂടി ഇസ്‌ലാം പഠിപ്പിക്കുന്നുണ്ട്.

അബൂഹുറൈറ (റ) നിന്നും ഇമാം ബുഖാരിയും മുസ്‌ലിമും ഉദ്ധരിച്ച ഹദീസില്‍ അത്തരം സ്ത്രീകളെ പറ്റി വ്യക്തമായി പരാമര്‍ശിക്കുന്നുണ്ട്. ”രണ്ടു വിഭാഗം ആളുകള്‍ നരകവാസികളാണ്. അവരെ ഞാന്‍ കണ്ടിട്ടില്ല. ഒരു വിഭാഗം; അവരുടെ കയ്യില്‍ പശുവിന്റെ വാലുകള്‍ പോലുള്ള ചമ്മട്ടികള്‍ ഉണ്ട്, അവകൊണ്ട് അവര്‍ ജനങ്ങളെ അടിക്കുന്നു. (രണ്ടാമത്തെ വിഭാഗം) വസ്ത്രം ധരിച്ച; എന്നാല്‍ നഗ്നതയുടുത്ത (മറ്റുള്ളവരെ തങ്ങളിലേക്ക്) ചായ്ക്കുന്ന (മറ്റുള്ളവരിലേക്ക്) ചായുന്ന സ്ത്രീകളാണ്. അവരുടെ തലകള്‍ ഒട്ടകങ്ങളുടെ പൂഞ്ഞപോലെയാകുന്നു. ഇങ്ങനെയുള്ളവര്‍ സ്വര്‍ഗത്തില്‍ പ്രവേശിക്കുകയില്ല. അതിന്റെ പരിമളം പോലും അവര്‍ക്ക് ആസ്വദിക്കുവാന്‍ സാധ്യമല്ല. സ്വര്‍ഗ്ഗത്തിലെ പരിമളം ഇത്രയിത്ര വഴിദൂരം വരെ എത്തുന്നതാണ്”.

കേവലം സുഗന്ധം ഉപയോഗിച്ചു എന്നതുകൊണ്ടല്ല ഹദീസുകള്‍ സ്ത്രീയെ ആക്ഷേപിക്കുന്നത്. മറിച്ച് പുരുഷന്മാരെ വശീകരിക്കുക എന്ന ഉദ്ദേശത്തോടെ അത് ഉപയോഗിക്കുന്നു എന്നതാണ് അവിടെ ആക്ഷേപാര്‍ഹമായ സംഗതി. ഇനി ഒരു സ്ത്രീ അത്തരം ദുരുദ്ദേശങ്ങളൊന്നുമില്ലാതെ സുഗന്ധം ഉപയോഗിക്കുന്നതിന്റെ വിധി എന്താണ്?

ഇമാം നസാഈ, ഇമാം തുര്‍മുദി, ഇമാം ബസ്സാര്‍ തുടങ്ങിയ ഹദീസ് പണ്ഡിതന്മാര്‍ അവരുടെ ഹദീസ് ഗ്രന്ഥങ്ങളില്‍ ഉദ്ധരിച്ച നിവേദനങ്ങള്‍ പ്രസ്തുത രംഗത്തെ ഇസ്‌ലാമിക നിലപാട് വ്യക്തമാക്കുന്നുണ്ട്.

”പുരുഷന്മാരുടെ സുഗന്ധം എന്നത് നിറം നേരിയതും വാസന കൂടിയതുമാണ്, സ്ത്രീകളുടെ സുഗന്ധം എന്നത് നിറം കൂടിയതും വാസന നേരിയതുമാണ്” (ബസ്സാര്‍: 6886)

”പുരുഷന്മാരുടെ സുഗന്ധത്തില്‍ ഉത്തമമായത് നിറം നേരിയതും സുഗന്ധം പ്രകടമായതുമാണ്, സ്ത്രീകളുടെ സുഗന്ധത്തില്‍ ഉത്തമമായത് നിറം പ്രകടമായതും സുഗന്ധം നേരിയതുമാണ്”. (തിര്‍മ്മിതി: 5/107, ഹദീസ് 2788).

സമാനമായ ഹദീസ് ഇമാം നസാഇയും നിവേദനം ചെയ്യുന്നുണ്ട്. പ്രസിദ്ധ ഹദീസ് പണ്ഡിതന്‍ ശൈഖ് നാസ്വിറുദ്ദീനുല്‍ അല്‍ബാനി ഹദീസ് സ്വഹീഹാണെന്ന് വിധിക്കുകയും ചെയ്തിട്ടുണ്ട്. ഇതില്‍ നിന്നും സ്ത്രീകള്‍ക്ക് സുഗന്ധം ഉപയോഗിക്കാമെന്നും കടുത്ത വാസനയുള്ളവ ഒഴിവാക്കുകയാണ് ശരിയായ നടപടിയെന്നും വ്യക്തമാകുന്നു. അല്ലാഹു അഅ്‌ലം.

മനുഷ്യരെല്ലാം ശുദ്ധപ്രകൃതിയിലാണ് ജനിക്കുന്നത് എന്നാണ് ഇസ്‌ലാം പഠിപ്പിക്കുന്നത്. മാതാപിതാക്കളുടെ ജീനുകൾ ലഭിക്കുക വഴി ജനനത്തോട് കൂടിത്തന്നെ അയാളുടെ സ്വഭാവത്തിന്റെ ബഹുഭൂരിപക്ഷവും നിർണയിക്കപ്പെട്ടിട്ടുണ്ടാവുമെന്നാണ് ശാസ്ത്രം പറയുന്നത്. പന്ത്രണ്ട് വയസ്സുവരെയുള്ള Primary Socialization അഥവാ Parenting വഴി ഒരാളുടെ സ്വഭാവത്തിന്റെ മുഴുവൻ രൂപരേഖയും (blue print) വളരെ Concrete ആയിത്തന്നെ രൂപപ്പെടുന്നു. പ്രതികൂലമായ കാലാവസ്ഥയിൽ ജനിക്കുകയും വളരുകയും ചെയ്തവർക്ക് നന്മയിലേക്കുള്ള പാത ദുർഘടമല്ലേ? അവർ സ്വാഭാവികമായും തിന്മയിൽ മുഴുകാനല്ലേ സാധ്യത? ഇങ്ങനെയുള്ളവരെ ശിക്ഷിക്കുന്നതിന്റെ ഇസ്‌ലാമിക യുക്തിയെന്താണ്?

അബിൻ

ശുദ്ധപ്രകൃതി(ഫിത്വറ)യോട് കൂടിയാണ് കുഞ്ഞുങ്ങളെല്ലാം ജനിക്കുന്നതെന്നും അവരെ വ്യത്യസ്ത മതക്കാരാക്കിത്തീർക്കുന്നത് അവരുടെ മാതാപിതാക്കളാണെന്നും മുഹമ്മദ് നബി (സ) പറഞ്ഞതായി അബൂഹുറൈറ(റ)യിൽ നിന്ന് ഇമാമുമാരായ ബുഖാരിയും മുസ്‌ലിമും തങ്ങളുടെ സ്വഹീഹുകളിൽ നിവേദനം ചെയ്തിട്ടുണ്ട്. പാപികളായാണ് മനുഷ്യരെല്ലാം ജനിക്കുന്നതെന്ന ക്രൈസ്തവവിശ്വാസത്തെ നിഷേധിക്കുന്നതാണ് ഈ ഹദീഥ്. ആരും പാപികളായല്ല ജനിക്കുന്നത്, പ്രത്യുത ശുദ്ധപ്രകൃതിയോടെയാണ് എന്നും ജനിച്ച ശേഷമാണ് അവർ വ്യസ്ത്യസ്ത വിഭാഗങ്ങളിൽ പെടുന്നവരെന്നും അതിന് പ്രധാന കാരണം മാതാപിതാക്കളുടെ ഇടപെടലുകളാണെന്നും വ്യക്തമാക്കുകയാണ് ഈ ഹദീഥിലൂടെ പ്രവാചകൻ (സ) ചെയ്യുന്നത്.

മാതാപിതാക്കളുടെ എല്ലാ ദുസ്വഭാവങ്ങളും ജീനുകളിലൂടെ മക്കൾക്ക് പകർന്നു ലഭിക്കുന്നുണ്ട് എന്നാണ് ആധുനിക ശാസ്ത്രം പറയുന്നത് എന്നാണ് ചോദ്യകർത്താവ് തെറ്റിദ്ധരിച്ചിരിക്കുന്നത് എന്ന് തോന്നുന്നു. എന്നാൽ വസ്തുതയതല്ല. മനുഷ്യന്റെ പെരുമാറ്റങ്ങളിലുള്ള ജീനുകളുടെ സ്വാധീനത്തെ കുറിച്ച പഠനമാണ് പെരുമാറ്റ ജനിതകം. ബോധപൂര്‍വ്വമല്ലാതെയോ സ്വാഭാവികമായോ ബോധോദയത്താലോ ഉണ്ടാവുന്ന നിരീക്ഷണയോഗ്യമായ പ്രവൃത്തികളോ വികാരങ്ങളോ താല്‍പര്യങ്ങളോ ആണ് 'പെരുമാറ്റം' (behaviour) കൊണ്ടുള്ള വിവക്ഷ. ജീനുകള്‍ എങ്ങനെയാണ് സാഹചര്യങ്ങള്‍ക്കകത്ത് പ്രവര്‍ത്തിച്ചുകൊണ്ട് പെരുമാറ്റത്തെ സ്വാധീനിക്കുന്നതെന്ന പഠനമാണ് പെരുമാറ്റജനിതകത്തിനടുത്ത് നടക്കുന്നത്. പെരുമാറ്റത്തെ സ്വാധീനിക്കുന്ന ജനിതക ഘടകങ്ങളേയും സാഹചര്യങ്ങളേയും ഒരേപോലെ പരിഗണിക്കുന്ന ശാസ്ത്രശാഖയാണിത്. ജീനുകളോ സാഹചര്യങ്ങളോ ഏതെങ്കിലൊന്ന് സര്‍വ്വശക്തമാണെന്നോ അതു മാത്രമാണ് പെരുമാറ്റത്തെ സ്വാധീനിക്കുന്നത് എന്നോ ഉള്ള ധാരണയല്ല പെരുമാറ്റ ജനിതക ശാസ്ത്രജ്ഞന്മാര്‍ക്കുള്ളത്; ഓരോന്നോ രണ്ടും കൂടിയോ പെരുമാറ്റത്തെ സ്വാധീനിക്കാവുന്നതാണ് എന്നാണ്‌ പെരുമാറ്റജനിതകം പറയുന്നത്‌. ജീനുകള്‍ കൂടാതെ പെരുമാറ്റത്തെ സാഹചര്യങ്ങള്‍ കൂടി സ്വാധീനിക്കുന്നുവെന്ന് പറയുമ്പോള്‍ എന്താണ് സാഹചര്യം കൊണ്ട് വിവക്ഷിക്കുന്നതെന്ന ചോദ്യമുയരാവുന്നതാണ്. ജനിതകമല്ലാത്ത എല്ലാ സ്വാധീനങ്ങളേയും സാഹചര്യം (environment) എന്നാണ് വിളിക്കുന്നത്. അത് ബാക്ടീരിയ, വൈറസ്, മരുന്നുകള്‍, പോഷകങ്ങള്‍ തുടങ്ങിയ ആന്തരികമായി പ്രവര്‍ത്തിക്കുന്ന വസ്തുക്കളാവാം; അതല്ലെങ്കില്‍ രക്ഷാകര്‍ത്തൃത്വം, കുടുംബജീവിതം, സമപ്രായക്കാര്‍, മാധ്യമങ്ങള്‍, പ്രകൃതിദുരന്തങ്ങള്‍, കാലാവസ്ഥാമാറ്റങ്ങള്‍, രോഗം, യുദ്ധം എന്നീ ബാഹ്യമായി സ്വാധീനിക്കുന്ന കാര്യങ്ങളുമാകാം. മൊത്തത്തില്‍ മനുഷ്യപെരുമാറ്റങ്ങളെ എങ്ങനെ ജീനുകളും എല്ലാതരം ബാഹ്യവും ആന്തരികവുമായ സാഹചര്യങ്ങളും കൂടി സ്വാധീനിക്കുന്നുവെന്ന് പഠിക്കുകയാണ് പെരുമാറ്റജനിതകം ചെയ്യുന്നതെന്ന് പറയാം.

മനുഷ്യനെ സംബന്ധിച്ചിടത്തോളം ജീനുകളാല്‍ മാത്രം നിശ്ചയിക്കപ്പെടുന്ന ചില അടിസ്ഥാനസ്വഭാവങ്ങളുണ്ട്. കണ്ണിന്റെ നിറം ഉദാഹരണം. എന്നാല്‍ പെരുമാറ്റങ്ങളില്‍ മിക്കതും ജീനുകളാല്‍ സ്ഥാപിക്കപ്പെട്ടതും സാഹചര്യങ്ങളാല്‍ തീരുമാനിക്കപ്പെടുന്നതുമാണ്. നല്ല പേശീബലവും ഉയര്‍ന്ന ശ്വാസകോശക്ഷമതയും ഒരാള്‍ നല്ല കായികതാരമായിത്തീരുന്നതിന് കാരണം അയാളുടെ ജീനുകളാണെന്ന് വേണമെങ്കില്‍ പറയാവുന്നതാണ്. എന്നാല്‍ അയാള്‍ ഒരു ഓട്ടക്കാരനോ, നീന്തല്‍ക്കാരനോ കളിക്കാരനോ ആവുന്നതില്‍ പ്രധാനമായും സാഹചര്യങ്ങളുടെ സ്വാധീനമാണുള്ളത്. അയാളുടെ മാതാപിതാക്കള്‍ ദരിദ്രരും കുട്ടികളെ സ്‌കൂളിലയക്കാന്‍ കഴിവില്ലാത്തവരുമാണെങ്കില്‍ അയാള്‍ക്ക് ആവശ്യമായ പരിശീലനം കിട്ടാതെ പോവുകയും അതുവഴി ഒരു കിലോമീറ്റര്‍പോലും ഓടാന്‍ കഴിയാത്തവനായി അയാള്‍ മാറുകയും ചെയ്‌തേക്കാനും സാധ്യതയുണ്ട്. പ്രകൃതിയും പരിസ്ഥിതിയും കൂടിച്ചേര്‍ന്ന് (Nature and Nurture) വ്യക്തിയുടെ സ്വഭാവങ്ങളും പെരുമാറ്റങ്ങളും നിര്‍ണയിക്കുന്നതെന്നാണ് പെരുമാറ്റജനിതകം പൊതുവായി പഠിപ്പിക്കുന്നത്.

പ്രസിദ്ധ പെരുമാറ്റ മനഃശാസ്ത്രജ്ഞനായ സി. ബേക്കര്‍ എഴുതുന്നത് കാണുക: 'ഉയരം, തൂക്കം, രക്തസമ്മര്‍ദ്ദം, ദഹനശേഷി തുടങ്ങിയ ഒരു വിധം ഭൗതിക സവിശേഷതകളും അവസ്ഥകളുമെല്ലാം ജീനുകളില്‍ നിന്ന് ഉല്‍ഭവിക്കുകയും സാഹചര്യങ്ങളുടെ പശ്ചാത്തലങ്ങള്‍ക്കനുസരിച്ച് പ്രവൃത്തിപഥത്തില്‍ വ്യതിരിക്തത പുലര്‍ത്തുകയും ചെയ്യുന്നവയാണ്. ഇതുതന്നെയാണ് എല്ലാ സങ്കീര്‍ണമായ മാനസിക-സാമൂഹികപെരുമാറ്റങ്ങളുടേയും അവസ്ഥ. വ്യത്യസ്ത ജീനുകള്‍ വ്യത്യസ്ത സാഹചര്യങ്ങളുടെ സ്വാധീനവുമായി കൂട്ടിമുട്ടുമ്പോഴാണ് ഒരോ പെരുമാറ്റങ്ങളും പുറത്തുവരുന്നത്... നിര്‍ഭാഗ്യവശാല്‍ പലരും ഇതില്‍ നിന്ന് വ്യത്യസ്തമായ ധാരണയുമായി കഴിയുന്നവരാണ്. ഒരോ സ്വഭാവസവിശേഷതകളേയും ഒരോ ജീനുകള്‍ നിയന്ത്രിക്കുന്നുവെന്നാണ് അവരുടെ ധാരണ. ഒരു ജീവിയുടെ വളര്‍ച്ചയെ പൂര്‍ണമായും നിശ്ചയിക്കുന്നത് ജീനുകളാണെന്ന ഈ വിശ്വാസത്തിനാണ് ജനിതക നിര്‍ണയവാദം (genetic determinism) എന്നുപറയുന്നത്. ജനിതക നിര്‍ണയവാദം തെറ്റായ ഒരു ധാരണയാണ്. ശാസ്ത്രീയ ഗവേഷണങ്ങളെ ശരിയായി മനസ്സിലാക്കാത്തതില്‍ നിന്നാണ് ഈ ധാരണയുണ്ടാവുന്നത്... എതെങ്കിലുമൊരു ജീനിനേയും സങ്കീര്‍ണമായ ഏതെങ്കിലുമൊരു മനുഷ്യ സ്വഭാവത്തേയും തമ്മില്‍ നേര്‍ക്കുനേരെ ബന്ധിപ്പിക്കാന്‍ കഴിയുന്ന യാതൊരു വിധശാസ്ത്രീയ ഗവേഷണഫലവും ഇതേവരെ ഉണ്ടായിട്ടില്ലെന്നതാണ് വാസ്തവം. നിരവധി ജീനുകള്‍ നിരവധി സാഹചര്യങ്ങളുടെ സ്വാധീനത്തിനു നടുവില്‍ പ്രവര്‍ത്തനനിരതമാവുമ്പോഴാണ് ഏതെങ്കിലുമൊരു പെരുമാറ്റമുണ്ടാവുന്നത്.''(C. Baker: Behavioral Genetics: An Introduction to How Genes and Environments in treat through Development to Shape Difference in Mood, Personality and Intelligence (2004) Page 17,18)

ഇതേപോലെത്തന്നെയാണ് പാരന്റിംഗിന്റെയും അവസ്ഥ. പന്ത്രണ്ട് വയസ്സുവരെയുള്ള ഒരാളുടെ അനുഭവങ്ങളും അയാളുടെ ആശയവിനിമയങ്ങളുമാണ് ഒരാളുടെ അടിസ്ഥാനവ്യക്തിത്വം രൂപീകരിക്കുന്നത് എന്ന് പാരന്റിംഗ് വിദഗ്ദർ പറയാറുണ്ടെന്നത് ശരിയാണ്. അക്കാര്യം തന്നെയാണ് ആദ്യം പറഞ്ഞ ഹദീഥിൽ പ്രവാചകൻ (സ) സൂചിപ്പിച്ചിരിക്കുന്നത്. ചെറുപ്പത്തിലുള്ള മാതാപിതാക്കളുടെ പെരുമാറ്റങ്ങളും മാതൃകയും മറ്റുള്ളവരുമായി നടത്തുന്ന ആശയവിനിമയങ്ങളുമാണ് ഒരാളുടെ അടിസ്ഥാനവ്യക്തിത്വം തീരുമാനിക്കുന്നത് എന്ന് പറഞ്ഞാൽ അതിനർത്ഥം അയാളുടെ വ്യക്തിത്വത്തിൽ പിന്നെ മാറ്റമൊന്നും കഴിയുകയില്ല എന്നല്ല. ബോധപൂർവ്വമല്ലാതെയുള്ള വ്യക്തിത്വരൂപീകരണമാണ് കൗമാരപ്രായത്തിലെത്തുമ്പോൾ പൂർത്തിയാവുന്നത്. ബോധപൂർവമായ വ്യക്തിത്വമാറ്റം നടക്കുന്നത് യഥാർത്ഥത്തിൽ കൗമാരത്തിനു ശേഷമാണ്. ബോധപൂർവ്വമായ വ്യക്തിത്വമാറ്റത്തിനാവശ്യം ബോധ്യവും സന്നദ്ധതയുമാണ്. മതബോധനങ്ങളും ധാർമ്മികനിയമങ്ങളുമെല്ലാം വ്യക്തിയിൽ സ്വാധീനം ചെലുത്തുന്നത് ഇക്കാലത്താണ്. അത് വഴിയുള്ള ബോധപൂവ്വമായ വ്യക്തിത്വമാറ്റമാണ് ഇസ്‌ലാം മനുഷ്യരോട് ആവശ്യപ്പെടുന്നത്.

കുഞ്ഞുങ്ങളുടെ വ്യക്തിത്വരൂപീകരണത്തിൽ പങ്കാളിത്തമുള്ളവരായതുകൊണ്ട് തന്നെ അവരെ നന്മയുൾക്കൊള്ളുന്നവരായി വളർത്തുവാൻ ഇസ്‌ലാം മാതാപിതാക്കളെ അനുശാസിക്കുന്നുണ്ട്. അത് അവരുടെ ഉത്തരവാദിത്തമാണ്. പ്രസ്തുത ബാധ്യത യഥാരൂപത്തിൽ നിർവ്വഹിച്ചില്ലെങ്കിൽ അവർ ചോദ്യം ചെയ്യപ്പെടും. എന്നാൽ മാതാപിതാക്കൾ എങ്ങനെ വളർത്തിയവരാണെങ്കിലും സ്വയം ബോധത്തോടെ ശരിയും തെറ്റും തെരെഞ്ഞെടുക്കുവാൻ വ്യക്തികൾക്ക് കഴിയും. പ്രസ്തുത തെരെഞ്ഞെടുപ്പിനാണ് അവർ ദൈവികമായ മാർഗ്ഗദർശനങ്ങൾ ഉപയോഗപ്പെടുത്തേണ്ടത്. ജനിതകമായി ലഭിച്ച സവിഷേതകളുടെ പേരിലോ മാതാപിതാക്കളുണ്ടാക്കിയ വ്യക്തിത്വത്തിന്റെ പേരിലോ അല്ല ഒരാളുടെ രക്ഷയും ശിക്ഷയുമൊന്നും തീരുമാനിക്കപ്പെടുന്നത്. അയാൾ സ്വയം തെരെഞ്ഞെടുത്ത മാർഗത്തിന്റെ അടിസ്ഥാനത്തിലാണ്. ഭൗതികമായ കോടതികളും ശിക്ഷ വിധിക്കുന്നത് കുറ്റവാളിയുടെ സ്വന്തമായ തെരെഞ്ഞെടുപ്പിന്റെ അടിസ്ഥാനത്തിലാണല്ലോ.

സ്വർഗത്തിൽ പുരുഷന്മാർക്ക് ഹൂറികൾ എന്ന ഇണകളുണ്ടെന്ന് പറയുന്ന ക്വുർആൻ എന്ത് കൊണ്ടാണ് സ്ത്രീകൾക്കുള്ള ഇണകളെക്കുറിച്ച് യാതൊന്നും പറയാതിരിക്കുന്നത് ?

ഷെമീർ

പുരുഷനായിരുന്നാലും സ്ത്രീ ആയിരുന്നാലും അവർ സത്യവിശ്വാസം ഉൾക്കൊള്ളുകയും സന്മാർഗനിഷ്ഠരാവുകയും ചെയ്‌താൽ അവർക്ക് സ്വർഗപ്രവേശമുണ്ടെന്നും തങ്ങളുടെ ചെയ്തികൾക്കെല്ലാം തക്കതായ പ്രതിഫലം അവർക്ക് ലഭിക്കുമെന്നും ക്വുർആൻ വ്യക്തമാക്കുന്നുണ്ട്. "(അല്ലാഹുവിന്‌) കീഴ്പെടുന്നവരായ പുരുഷന്‍മാര്‍, സ്ത്രീകള്‍, വിശ്വാസികളായ പുരുഷന്‍മാര്‍, സ്ത്രീകള്‍, ഭക്തിയുള്ളവരായ പുരുഷന്‍മാര്‍, സ്ത്രീകള്‍, സത്യസന്ധരായ പുരുഷന്‍മാര്‍, സ്ത്രീകള്‍, ക്ഷമാശീലരായ പുരുഷന്‍മാര്‍, സ്ത്രീകള്‍, വിനീതരായ പുരുഷന്‍മാര്‍, സ്ത്രീകള്‍, ദാനം ചെയ്യുന്നവരായ പുരുഷന്‍മാര്‍, സ്ത്രീകള്‍, വ്രതമനുഷ്ഠിക്കുന്നവരായ പുരുഷന്‍മാര്‍, സ്ത്രീകള്‍, തങ്ങളുടെ ഗുഹ്യാവയവങ്ങള്‍ കാത്തുസൂക്ഷിക്കുന്നവരായ പുരുഷന്‍മാര്‍, സ്ത്രീകള്‍, ധാരാളമായി അല്ലാഹുവെ ഓര്‍മിക്കുന്നവരായ പുരുഷന്‍മാര്‍, സ്ത്രീകള്‍ - ഇവര്‍ക്ക്‌ തീര്‍ച്ചയായും അല്ലാഹു പാപമോചനവും മഹത്തായ പ്രതിഫലവും ഒരുക്കിവെച്ചിരിക്കുന്നു" (33: 35)

സ്വർഗത്തിൽ പ്രവേശിക്കുന്ന സത്യവിശ്വാസികൾക്കിടയിൽ ലിംഗത്തിന്റെ വെളിച്ചത്തിൽ എന്തെങ്കിലും തരത്തിലുള്ള അനീതിയുണ്ടാവുകയില്ലെന്ന വസ്തുതയും ക്വുർആൻ പഠിപ്പിക്കുന്നുണ്ട്. "ആണാകട്ടെ പെണ്ണാകട്ടെ, ആര്‍ സത്യവിശ്വാസിയായിക്കൊണ്ട്‌ സല്‍പ്രവൃത്തികള്‍ ചെയ്യുന്നുവോ അവര്‍ സ്വര്‍ഗത്തില്‍ പ്രവേശിക്കുന്നതാണ്‌. അവരോട്‌ ഒരു തരിമ്പും അനീതി കാണിക്കപ്പെടുന്നതല്ല." (4:124)

സദ്‌വൃത്തരായ ഇണകളും സന്താനങ്ങളുമെല്ലാം ഇണകളും സന്താനങ്ങളുമായിത്തന്നെ സ്വർഗത്തിൽ ഒരുമിച്ചുകൂടുമെന്നാണ് ക്വുർആൻ മനസ്സിലാക്കിത്തരുന്നത്. "ഞങ്ങളുടെ രക്ഷിതാവേ, അവര്‍ക്ക്‌ നീ വാഗ്ദാനം ചെയ്തിട്ടുള്ള സ്ഥിരവാസത്തിനുള്ള സ്വര്‍ഗങ്ങളില്‍ അവരെയും അവരുടെ മാതാപിതാക്കളെയും, ഭാര്യമാര്‍, സന്തതികള്‍ എന്നിവരില്‍ നിന്നു സദ്‌വൃത്തരായിട്ടുള്ളവരെയും നീ പ്രവേശിപ്പിക്കേണമേ. തീര്‍ച്ചയായും നീ തന്നെയാകുന്നു പ്രതാപിയും യുക്തിമാനും." (40:8)

"ഏതൊരു കൂട്ടര്‍ വിശ്വസിക്കുകയും അവരുടെ സന്താനങ്ങള്‍ വിശ്വാസത്തില്‍ അവരെ പിന്തുടരുകയും ചെയ്തിരിക്കുന്നുവോ അവരുടെ സന്താനങ്ങളെ നാം അവരോടൊപ്പം ചേര്‍ക്കുന്നതാണ്‌. അവരുടെ കര്‍മ്മഫലത്തില്‍ നിന്ന്‌ യാതൊന്നും നാം അവര്‍ക്കു കുറവു വരുത്തുകയുമില്ല. ഏതൊരു മനുഷ്യനും താന്‍ സമ്പാദിച്ച്‌ വെച്ചതിന്‌ (സ്വന്തം കര്‍മ്മങ്ങള്‍ക്ക്‌) പണയം വെക്കപ്പെട്ടവനാകുന്നു." (52: 21)

"അതായത്‌, സ്ഥിരവാസത്തിനുള്ള സ്വര്‍ഗത്തോപ്പുകള്‍. അവരും, അവരുടെ പിതാക്കളില്‍ നിന്നും, ഇണകളില്‍ നിന്നും സന്തതികളില്‍ നിന്നും സദ്‌വൃത്തരായിട്ടുള്ളവരും അതില്‍ പ്രവേശിക്കുന്നതാണ്‌. മലക്കുകള്‍ എല്ലാ വാതിലിലൂടെയും അവരുടെ അടുക്കല്‍ കടന്നുവന്നിട്ട്‌ പറയും:

നിങ്ങള്‍ ക്ഷമ കൈക്കൊണ്ടതിനാല്‍ നിങ്ങള്‍ക്ക്‌ സമാധാനം! അപ്പോള്‍ ലോകത്തിന്‍റെ പര്യവസാനം എത്ര നല്ലത്‌!" (13: 23, 24)

സ്വർഗത്തിൽ പ്രവേശിക്കുന്ന പുരുഷനും സ്ത്രീക്കുമെല്ലാം അവർ ആഗ്രഹിക്കുന്നതെല്ലാം അവിടെയുണ്ടാവുമെന്ന് ക്വുർആൻ അർത്ഥശങ്കക്കിടയില്ലാതെ വ്യക്തമാക്കുന്നുണ്ട്. "എന്‍റെ ദാസന്‍മാരേ, ഇന്ന്‌ നിങ്ങള്‍ക്ക്‌ യാതൊരു ഭയവുമില്ല. നിങ്ങള്‍ ദുഃഖിക്കേണ്ടതുമില്ല.

നമ്മുടെ ദൃഷ്ടാന്തങ്ങളില്‍ വിശ്വസിക്കുകയും കീഴ്പെട്ടു ജീവിക്കുന്നവരായിരിക്കുകയും ചെയ്തവരത്രെ(നിങ്ങള്‍)

നിങ്ങളും നിങ്ങളുടെ ഇണകളും സന്തോഷഭരിതരായികൊണ്ട്‌ സ്വര്‍ഗത്തില്‍ പ്രവേശിച്ചു കൊള്ളുക.

സ്വര്‍ണത്തിന്‍റെ തളികകളും പാനപാത്രങ്ങളും അവര്‍ക്ക്‌ ചുറ്റും കൊണ്ടു നടക്കപ്പെടും. മനസ്സുകള്‍ കൊതിക്കുന്നതും കണ്ണുകള്‍ക്ക്‌ ആനന്ദകരവുമായ കാര്യങ്ങള്‍ അവിടെ ഉണ്ടായിരിക്കും. നിങ്ങള്‍ അവിടെ നിത്യവാസികളായിരിക്കുകയും ചെയ്യും."(43: 68- 71)

സ്ത്രീകൾക്കും പുരുഷന്മാർക്കുമെല്ലാം അവർ ആഗ്രഹിക്കുന്നതെല്ലാം സ്വർഗത്തിൽ ലഭിക്കുമെന്ന് വ്യക്തമാക്കുന്ന ക്വുർആൻ പക്ഷെ, ധർമ്മനിഷ്ഠ പാലിക്കുന്ന പുരുഷന്മാർക്ക് ഇണകളായി ലഭിക്കുന്ന സ്വർഗസ്ത്രീകളെക്കുറിച്ച് പ്രത്യേകമായി എടുത്ത് പറയുന്നുണ്ട്. "തീര്‍ച്ചയായും ധര്‍മ്മനിഷ്ഠപാലിക്കുന്നവര്‍ സ്വര്‍ഗത്തോപ്പുകളിലും സുഖാനുഗ്രഹങ്ങളിലുമായിരിക്കും. തങ്ങളുടെ രക്ഷിതാവ്‌ അവര്‍ക്കു നല്‍കിയതില്‍ ആനന്ദം കൊള്ളുന്നവരായിട്ട്‌. ജ്വലിക്കുന്ന നരകത്തിലെ ശിക്ഷയില്‍ നിന്ന്‌ അവരുടെ രക്ഷിതാവ്‌ അവരെ കാത്തുരക്ഷിക്കുകയും ചെയ്യും. നിങ്ങള്‍ പ്രവര്‍ത്തിച്ചിരുന്നതിന്‍റെ ഫലമായി നിങ്ങള്‍ സുഖമായി തിന്നുകയും കുടിക്കുകയും ചെയ്തു കൊള്ളുക. വരിവരിയായ്‌ ഇട്ട കട്ടിലുകളില്‍ ചാരിയിരിക്കുന്നവരായിരിക്കും അവര്‍. വിടര്‍ന്ന കണ്ണുകളുള്ള വെളുത്ത തരുണികളെ നാം അവര്‍ക്ക്‌ ഇണചേര്‍ത്തു കൊടുക്കുകയും ചെയ്യും." (52: 17 - 20)

"സൂക്ഷ്മത പാലിച്ചവര്‍ തീര്‍ച്ചയായും നിര്‍ഭയമായ വാസസ്ഥലത്താകുന്നു; തോട്ടങ്ങള്‍ക്കും അരുവികള്‍ക്കുമിടയില്‍; നേര്‍ത്ത പട്ടുതുണിയും കട്ടിയുള്ള പട്ടുതുണിയും അവര്‍ ധരിക്കും. അവര്‍ അന്യോന്യം അഭിമുഖമായിട്ടായിരിക്കും ഇരിക്കുന്നത്‌. അങ്ങനെയാകുന്നു (അവരുടെ അവസ്ഥ.) വിശാലമായ നേത്രങ്ങളുള്ള വെളുത്ത സ്ത്രീകളെ അവര്‍ക്ക്‌ ഇണകളായി നല്‍കുകയും ചെയ്യും. സുരക്ഷിതത്വ ബോധത്തോട്‌ കൂടി എല്ലാവിധ പഴങ്ങളും അവര്‍ അവിടെ വെച്ച്‌ ആവശ്യപ്പെട്ടുകൊണ്ടിരിക്കും. (44: 51- 55)

"അവയില്‍ സുന്ദരികളായ ഉത്തമ തരുണികളുണ്ട്‌. അപ്പോള്‍ നിങ്ങള്‍ ഇരു വിഭാഗത്തിന്‍റെയും രക്ഷിതാവിന്‍റെ അനുഗ്രഹങ്ങളില്‍ ഏതിനെയാണ്‌ നിങ്ങള്‍ നിഷേധിക്കുന്നത്‌?; കൂടാരങ്ങളില്‍ ഒതുക്കി നിര്‍ത്തപ്പെട്ട വെളുത്ത തരുണികള്‍!; അപ്പോള്‍ നിങ്ങള്‍ ഇരു വിഭാഗത്തിന്‍റെയും രക്ഷിതാവിന്‍റെ അനുഗ്രഹങ്ങളില്‍ ഏതിനെയാണ്‌ നിങ്ങള്‍ നിഷേധിക്കുന്നത്‌? അവര്‍ക്ക്‌ മുമ്പ്‌ മനുഷ്യനോ ജിന്നോ അവരെ സ്പര്‍ശിച്ചിട്ടില്ല." (55: 70- 74)

എന്തുകൊണ്ട് സ്വർഗസ്ത്രീകളെക്കുറിച്ച് മാത്രം ക്വുർആൻ പ്രതിപാദിക്കുന്നുവെന്ന ചോദ്യത്തിനുത്തരം സ്ത്രീസൗന്ദര്യം വലിയൊരു പരീക്ഷണമാവുന്നത് പുരുഷന് മാത്രമാണെന്നും പ്രസ്തുത പരീക്ഷണത്തിൽ വിജയിക്കുന്നവർക്കുള്ള പ്രത്യേകമായ സമ്മാനമാണ് അതെന്നുമാണ്. സ്ത്രീയുടെ ശാരീരികസൗന്ദര്യവും അർദ്ധനഗ്നതയും നഗ്നതയുമെല്ലാം പുരുഷനിൽ കാമാവേശമുണ്ടാക്കുന്നതാണ്. പെണ്ണിനെ അവൻ ഇഷ്ടപ്പെടുന്ന രൂപത്തിൽ കാണുമ്പോൾ തന്നെ അവന്റെ ശരീരം ഉണരുന്നു. ലൈംഗികമായി അവൻ ഉത്തേജിതനായിത്തീരുന്നു; അവന്റെ ശരീരത്തിൽ രതിബന്ധത്തിന് പ്രേരിപ്പിക്കുന്ന ഹോർമോണുകൾ ഉത്പാദിപ്പിക്കപ്പെടുന്നു. അവന്റെ ലിംഗത്തിൽ സ്നേഹദ്രവമെന്ന് വിളിക്കുന്ന കൗപേഴ്സ് സ്രവമുണ്ടാവുന്നു. അതുകൊണ്ട് തന്നെ, ആസ്വദിക്കുന്ന രൂപത്തിൽ അന്യസ്ത്രീകളെ നോക്കരുതെന്ന് ഇസ്‌ലാം പഠിപ്പിക്കുന്നു; തനിക്ക് ആസ്വദിക്കുവാൻ അർഹതയില്ലാത്തവരുടെ നഗ്നതയോ അർധനഗ്നതയോ കാണരുതെന്ന് ഇസ്‌ലാം പുരുഷനോട് കല്പിക്കുന്നു. നഗ്നതയും അർദ്ധനഗ്നതയും സ്ത്രീസൗന്ദര്യത്തിന്റെ പ്രദർശനവുമെല്ലാം വ്യാപകമായ സമൂഹത്തിൽ പുറത്തിറങ്ങുന്ന പുരുഷനെ സംബന്ധിച്ചിടത്തോളം ഈ വിലക്കുകൾ പാലിക്കുക വളരെ പ്രയാസകരമാണ്. സ്ത്രീസൗന്ദര്യത്തിനു നേരെ കണ്ണുകൾ താഴ്ത്തണമെന്ന ക്വുർആനിക നിർദേശം പാലിക്കുക എളുപ്പമല്ല. പരീക്ഷണങ്ങൾ സഹിച്ച് ഈ ദൈവികനിർദേശം പാലിക്കുന്ന പുരുഷന് ലഭിക്കുന്ന സമ്മാനമാണ് ക്വുർആനിൽ പ്രതിപാദിക്കുന്ന സ്വർഗസ്ത്രീകൾ. അവരെക്കുറിച്ച പ്രതിപാദനങ്ങൾ അന്യസ്ത്രീകളുടെ സൗന്ദര്യം ആസ്വദിക്കുന്നതിൽ നിന്ന് പുരുഷന്മാരെ പൂർണമായും പിന്തിരിപ്പിക്കുന്ന രീതിയിലുള്ളവയാണ്. വിശുദ്ധമായ ദാമ്പത്യജീവിതത്തിൽ മാത്രം തങ്ങളുടെ ലൈംഗികാസ്വാദനം ഒതുക്കുവാൻ അത് പുരുഷന്മാരെ പ്രേരിപ്പിക്കുന്നു.

പുരുഷന്റെ സൗന്ദര്യം സ്ത്രീയെയും ആകർഷിക്കുമെങ്കിലും അത് അവളിൽ രതിത്വരയുണ്ടാക്കുന്നില്ല. അവളിൽ അത്തരമൊരു ത്വരയുണ്ടാവണമെങ്കിൽ ആദ്യം പുരുഷൻ അവൾക്ക് ഇഷ്ടപ്പെട്ടവനാവണം. തനിക്ക് ഇഷ്ടപ്പെട്ട പുരുഷന്റെ ശൃംഗാരവും വർത്തമാനങ്ങളും സ്പർശവുമാണ് സ്ത്രീയിൽ രതിമോഹം ഉണർത്തുന്നത്. സ്ത്രീസൗന്ദര്യം ആസ്വദിക്കുമ്പോൾ പുരുഷനിലുണ്ടാവുന്ന തരത്തിലുള്ള ഹോർമോൺ ഉത്പാദനവും സ്രവങ്ങളുമെല്ലാം സ്ത്രീയിൽ ഉണ്ടാവുന്നത് തനിക്ക് ഇഷ്ടപ്പെട്ട പുരുഷന്റെ തലോടലേൽക്കുമ്പോഴാണ്. അങ്ങാടിയിൽ വ്യാപകമായ സൗന്ദര്യപ്രദർശനമോ നഗ്നതയുടെയും അർധനഗ്നതയുടെയും വിളയാട്ടങ്ങളോ സ്ത്രീക്ക് വലിയൊരു പരീക്ഷണമാവുന്നില്ല. തനിക്ക് ഇഷ്ടപ്പെട്ട പുരുഷന്റെ സൗന്ദര്യവും നഗ്നതയും മാത്രമേ അവളിൽ രതിത്വരയുണ്ടാക്കുന്നുള്ളൂ. പുരുഷസൗന്ദര്യം പെണ്ണിന് ഒരു പരീക്ഷണമേയല്ല എന്ന സാരം. അതുകൊണ്ടുതന്നെ ആ രംഗത്ത് അവൾക്ക് നൽകുന്ന പ്രത്യേകമായ സമ്മാനങ്ങളെക്കുറിച്ച് എടുത്ത് പറയേണ്ടതില്ല. എന്നാൽ അവൾക്ക് അവൾ ആഗ്രഹിക്കുന്ന തരത്തിലുള്ള എല്ലാ ആസ്വാദനങ്ങളും സ്വർഗ്ഗത്തിലുണ്ടാവുമെന്നും അവളോട് യാതൊരു വിധ അനീതിയുമുണ്ടാവുകയില്ലെന്നും പ്രത്യേകം പറയുകയും ചെയ്തിരിക്കുന്നു. സ്വർഗസ്ത്രീകളെക്കുറിച്ച പരാമർശങ്ങളിൽ നിന്ന് പുരുഷന്മാർക്ക് ഉണ്ടാവുന്നത് പോലെയുള്ള, അധാർമികകാഴ്ചകളിൽ നിന്ന് അകന്നു നിൽക്കാൻ വേണ്ടിയുള്ള പ്രചോദനം സ്വർഗപുരുഷന്മാരെക്കുറിച്ച പ്രതിപാദനം വഴി സ്ത്രീകളിൽ ഉണ്ടാക്കാൻ കഴിയില്ല. അതുകൊണ്ട് തന്നെയായിരിക്കണം ആണിനേയും പെണ്ണിനേയും കൃത്യമായി അറിയാവുന്ന അല്ലാഹു അത്തരം പരാമർശങ്ങൾ നടത്താതിരുന്നത്.

പുരുഷനെയും പെണ്ണിനേയും സന്മാർഗ്ഗനിഷ്ഠരാകുവാൻ പ്രചോദിപ്പിക്കുന്ന സ്വർഗ്ഗത്തെക്കുറിച്ച ചില വചനങ്ങളുടെ സാരം കാണുക: "അന്നേ ദിവസം യാതൊരാളോടും അനീതി ചെയ്യപ്പെടുകയില്ല. നിങ്ങള്‍ പ്രവര്‍ത്തിച്ചു കൊണ്ടിരുന്നതിനല്ലാതെ നിങ്ങള്‍ക്ക്‌ പ്രതിഫലം നല്‍കപ്പെടുകയുമില്ല. തീര്‍ച്ചയായും സ്വര്‍ഗവാസികള്‍ അന്ന്‌ ഓരോ ജോലിയിലായിക്കൊണ്ട്‌ സുഖമനുഭവിക്കുന്നവരായിരിക്കും.അവരും അവരുടെ ഇണകളും തണലുകളില്‍ അലംകൃതമായ കട്ടിലുകളില്‍ ചാരിയിരിക്കുന്നവരായിരിക്കും.അവര്‍ക്കവിടെ പഴവര്‍ഗങ്ങളുണ്ട്‌, അവര്‍ക്ക്‌ തങ്ങള്‍ ആവശ്യപ്പെടുന്നതല്ലാമുണ്ട്‌.സമാധാനം! അതായിരിക്കും കരുണാനിധിയായ രക്ഷിതാവിങ്കല്‍ നിന്ന്‌ അവര്‍ക്കുള്ള അഭിവാദ്യം. (36: 54- 58)

വിഷയവുമായി ബന്ധപ്പെട്ട വീഡിയോ

അംഗവൈകല്യമുള്ളവരെയും ബുദ്ധിമാന്ദ്യമുള്ളവരെയുമെല്ലാം സൃഷ്ടിച്ചതാരാണ്? ദൈവം മനഃപൂർവമാണ് അവരെ സൃഷ്ടിച്ചതെങ്കിൽ അത് ക്രൂരതയല്ലേ? അവരോട് മാത്രമായി ഇത്ര വലിയ ക്രൂരത ദൈവം കാണിക്കുന്നതെന്തുകൊണ്ട്?

അൻവർ പുനലൂർ

പ്രപഞ്ചത്തിലെ എല്ലാറ്റിനെയും സൃഷ്ടിച്ചവൻ അല്ലാഹുവാണ്. "പറയുക: അല്ലാഹുവത്രെ എല്ലാ വസ്തുക്കളുടെയും സ്രഷ്ടാവ്‌. അവന്‍ ഏകനും സര്‍വ്വാധിപതിയുമാകുന്നു" (ക്വുർആൻ 13: 16)

അല്ലാഹുവിന്റെ സൃഷ്ടികളെല്ലാം കുറ്റമറ്റതാണ് എന്നുകൂടി പഠിപ്പിക്കുന്നുണ്ട് ക്വുർആൻ. "എല്ലാകാര്യവും കുറ്റമറ്റതാക്കിത്തീര്‍ത്ത അല്ലാഹുവിന്റെ പ്രവര്‍ത്തനമത്രെ അത്"(27:88) എന്നും "താന്‍ സൃഷ്ടിച്ച എല്ലാ വസ്തുക്കളെയും വിശിഷ്ടമാക്കിയവനത്രെ അവന്‍" (32:7) എന്നും പറയുമ്പോൾ ക്വുർആൻ ഈ പാഠമാണ് നൽകുന്നത്. അല്ലാഹുവിന്റെ സൃഷ്ടികളിലൊന്നും ആത്യന്തികമായി വൈകല്യങ്ങളൊന്നുമുണ്ടാവില്ല എന്ന് തന്നെയാണ് ഇതിനർത്ഥം. നമ്മുടെ കാഴ്ചയിൽ വൈകല്യങ്ങളായി തോന്നുന്നവ പോലും അല്ലാഹുവിന്റെ ദൃഷ്ടിയിൽ വലിയ അനുഗ്രഹങ്ങളായിരിക്കുമെന്നാണ് ഇതിൽ നിന്ന് നാം ഉൾക്കൊള്ളേണ്ടത്.

അല്ലാഹു 'റഹ്‌മാൻ' അഥവാ പരമകാരുണികനാണ് എന്നാണ് ക്വുർആൻ പഠിപ്പിക്കുന്നത്. തീവ്രമായ കാരുണ്യം എല്ലായ്‌പ്പോഴും എല്ലാ സൃഷ്ടികളിലേക്കും ചൊരിഞ്ഞുകൊണ്ടിരിക്കുന്നവൻ എന്നാണ് 'റഹ്‌മാൻ' എന്ന ദൈവനാമം അർത്ഥമാക്കുന്നത്. സ്വന്തം അസ്തിത്വത്തിൽ തന്നെ കാരുണ്യത്തെ ഒരു ബാധ്യതയായി രേഖപ്പെടുത്തിയവനായാണ് ക്വുർആൻ അല്ലാഹുവിനെ പരിചയപ്പെടുത്തുന്നത്. (6: 12) അല്ലാഹുവിൽ നിന്നുണ്ടാവുന്നതെല്ലാം കാരുണ്യമാണ് എന്നർത്ഥം. നന്മയായും തിന്മയായും നമുക്ക് തോന്നുന്ന കാര്യങ്ങളിലെല്ലാം അല്ലാഹുവിന്റെ കാരുണ്യം അന്തർലീനമായിരിക്കും എന്ന പാഠമാണ് 'എന്‍റെ കാരുണ്യമാകട്ടെ സര്‍വ്വ വസ്തുക്കളെയും ഉള്‍കൊള്ളുന്നതായിരിക്കും' (ക്വുർആൻ 7: 156) എന്ന ദൈവവചനം വ്യക്തമാക്കുന്നത്.

യൂസുഫ്‍ നബിയുടെ(അ) ചരിത്രം വിവരിച്ചുകൊണ്ട് ക്വുർആൻ നമ്മെ പഠിപ്പിക്കുന്ന പ്രധാനപ്പെട്ട പാഠങ്ങളിലൊന്ന് ദുരിതങ്ങളായി നാം മനസ്സിലാക്കുന്ന സംഭവങ്ങൾക്കെല്ലാം പിന്നിൽ കാരുണ്യത്തിന്റെ വലിയ ദൈവപദ്ധതികളുണ്ടാവുമെന്നാണ്. കിണറ്റിൽ എറിയപ്പെട്ട യൂസുഫ് എന്ന ബാലന്റെ ദുരിതത്തെ കുറിച്ച് വേവലാതിപ്പെടുമ്പോൾ തന്നെ ആ ദുരിതമാണ് അദ്ദേഹത്തെ ഈജിപ്തിന്റെ ഭക്ഷ്യമന്ത്രിയുടെ പദവിയിലെത്തിച്ച സംഭവപരമ്പരയുടെ തുടക്കമെന്നുകൂടി നാം മനസ്സിലാക്കണം. ജീവിതത്തിൽ വന്നുഭവിച്ച ദുരിതങ്ങളും പ്രയാസങ്ങളുമെല്ലാം ഈ ജീവിതത്തിൽ തന്നെ നന്മയായിത്തീർന്ന നിരവധി കാര്യങ്ങൾ നമുക്ക് ഓരോരുത്തർക്കും പറയാനുണ്ടാവും.

അംഗവൈകല്യത്തെയും ബുദ്ധിമാന്ദ്യത്തെയുമെല്ലാം തങ്ങൾക്ക് വലിയ സാധ്യതകളായിത്തീർന്ന നിരവധി പേർ ചരിത്രത്തിലും വർത്തമാനത്തിലുമുണ്ട്. അന്ധയും ബധിരയും മൂകയുമായി ജനിച്ച ഹെലൻ കെല്ലർ നല്ലൊരു ഉദാഹരണമാണ് . തന്റെ വൈകല്യങ്ങളെ പഴിച്ച് ജീവിതം പാഴാക്കാതെ അവയെ അവസരങ്ങളായി കണ്ട് പ്രവർത്തിച്ചതിനാൽ അവർക്ക് പ്രശസ്തിയുടെ കൊടുമുടിയിലേക്ക് ഉയരാൻ കഴിഞ്ഞു. ഇന്നോളം ജീവിച്ചിരുന്നവരിൽ ഏറ്റവുമധികം മസ്തിഷ്കശേഷിയുള്ള മനുഷ്യനായി വാഴ്ത്തപ്പെടുന്ന ആൽബർട്ട് ഐൻസ്റ്റീൻ മൂന്നു വയസ്സ് വരെ പഠനവൈകല്യങ്ങൾ പ്രകടിപ്പിച്ചിരുന്ന ഒരാളായിരുന്നുവെന്ന വസ്തുത നാം മനസ്സിലാക്കണം. രണ്ട് എമ്മി അവാർഡുകളും ആറ് ഗോൾഡൻ ഗ്ലോബ് അവാർഡുകളും രണ്ട് സ്ക്രീൻ ആക്‌ടേഴ്‌സ് ഗിൽഡ് അവാർഡുകളും അഞ്ച് ഗ്രാമി അവാർഡുകളും നേടിയ ഹോളിവുഡിലെ പ്രശസ്ത താരം റോബിൻ വില്യംസ് ചെറുപ്പത്തിൽ AHDH എന്ന മാനസികവൈകല്യമുള്ളയാളായിരുന്നുവെന്ന് കൂടി നാം അറിയണം. വൈകല്യങ്ങളെ വൈകല്യങ്ങളായി കാണാതെ അവസരങ്ങളായി കണ്ടാൽ വലിയ സാധ്യതകളിലേക്ക് അവ വഴി തുറന്നേക്കുമെന്നാണ് ഇവരുടെയെല്ലാം ചരിത്രം നമ്മെ പഠിപ്പിക്കുന്നത്.

ഈ പ്രപഞ്ചത്തിലേക്കുള്ള അല്ലാഹുവിന്റെ കാരുണ്യത്തിന്റെ പ്രകാശനം ഒരാൾക്ക് ദുരിതമായും ദുരന്തമായുമെല്ലാം അനുഭവപ്പെടാവുന്നതാണ്. അയാൾക്ക് വന്നു ഭവിക്കുന്ന പ്രയാസങ്ങൾക്ക് പിന്നിലുള്ള അല്ലാഹുവിന്റെ കാരുണ്യം എന്താണെന്ന് ചിലപ്പോൾ ഇവിടെ വെച്ച് നമുക്ക് മനസ്സിലായിക്കൊള്ളണമെന്നില്ല. അതിന് അല്ലാഹുവിന്റെ കാരുണ്യത്തിന്റെ പൂർണപ്രകാശനം നടക്കുന്ന മരണാനന്തരജീവിതത്തിലേ കഴിയൂ. സ്വന്തം അസ്തിത്വത്തിന്റെ ബാധ്യതയായി നിശ്ചയിക്കപ്പെട്ട കാരുണ്യത്തിന്റെ 99 ശതമാനവും പ്രകടിപ്പിക്കപ്പെടുന്നത് അവിടെ വെച്ചാണ്. അംഗവൈകല്യത്തിന്റെയും ബുദ്ധിമാന്ദ്യത്തിന്റെയുമെല്ലാം പിന്നിലുണ്ടായിരുന്ന അല്ലാഹുവിന്റെ അനുഗ്രഹങ്ങളെന്തൊക്കെയാണെന്ന് അവിടെ വെച്ച് എല്ലാവർക്കും മനസ്സിലാവും. അവർക്കും മറ്റു സൃഷ്ടികൾക്കുമെല്ലാം വലിയ നന്മയായിരുന്നു അവർ അനുഭവിച്ച ദുരിതങ്ങൾ എന്ന് മനസ്സിലാവുക മാത്രമല്ല, തങ്ങൾ അനുഭവിച്ച പ്രയാസങ്ങൾക്കുള്ള പ്രതിഫലം കൂടി അവിടെനിന്ന് അവർക്ക് ലഭിക്കും. മരണാനന്തരജീവിതത്തെക്കൂടി പരിഗണിച്ചുകൊണ്ടുള്ള വിശാലമായ ക്യാൻവാസിന് മാത്രമേ മനുഷ്യർ ഇവിടെ അനുഭവിക്കുന്ന പ്രയാസങ്ങളെയെല്ലാം തൃപ്തികരമായി വിശദീകരിക്കാനാവൂ.

ഇഹലോകജീവിതം ഒരു പരീക്ഷണം മാത്രമാണെന്ന് മനസ്സിലാക്കുന്ന വിശ്വാസിയെ സംബന്ധിച്ചിടത്തോളം തനിക്കുണ്ടാവുന്ന ദുരിതങ്ങളെല്ലാം മരണാനന്തരജീവിതത്തിൽ അല്ലാഹുവിന്റെ കാരുണ്യത്തിന്റെ പൂർണമായ പ്രകാശനത്തിന് അർഹനാകുന്നതിനു വേണ്ടിയുള്ള അവസരങ്ങളാണ്. ബുദ്ധിവൈകല്യമായാലും അംഗവൈകല്യമായാലും അവയെ പഴിക്കാതെ അവയിലെ അനുഗ്രഹങ്ങൾ അറിഞ്ഞ് പ്രവർത്തിക്കുകയും അവയുടെ സാധ്യതകൾ ഉപയോഗിക്കുകയുമാണ് അവർ ചെയ്യുക. ദുരിതങ്ങൾ അനുഭവിക്കുന്നവർ മറ്റുള്ളവക്ക് തങ്ങൾക്ക് ലഭിച്ച അനുഗ്രഹങ്ങൾക്ക് കൃതജ്ഞത പ്രകടിപ്പിക്കാനുള്ള നിമിത്തങ്ങളുമായിത്തീരുന്നു. വൈകല്യങ്ങളനുഭവിക്കുന്നവർക്ക് മരണാനന്തരം ലഭിക്കാനിരിക്കുന്ന വലിയ അനുഗ്രഹങ്ങളെക്കുറിച്ച അറിവ് അവർക്ക് സമാധാനം നൽകുകയും ചെയ്യുന്നു. നൽകിയും നൽകാതെയും ഒരാൾ പരീക്ഷിക്കപ്പെടുമ്പോൾ അവയെല്ലാം അല്ലാഹുവിന്റെ കാരുണ്യത്തിന്റെ ഭാഗമാണെന്ന് മനസ്സിലാക്കി ക്ഷമിക്കുവാനും അതിന്ന് അവനിൽ നിന്ന് പ്രതിഫലം പ്രതീക്ഷിക്കാനും കഴിയുക വിശ്വാസിക്ക് മാത്രമാണ്. പരീക്ഷണങ്ങളുടെ കൊടുമുടിയിൽ പോലും ക്ഷമിച്ചുകൊണ്ട് അവയുടെ സാധ്യതകൾ പ്രയോജനപ്പെടുത്താൻ വിശ്വാസികൾല്ലാതെ ആർക്കാണ് കഴിയുക? അത്തരക്കാർക്ക് മതം നൽകുന്ന പ്രതീക്ഷ അപാരമാണ്. ക്വുർആൻ പറയുന്നു: "കുറച്ചൊക്കെ ഭയം, പട്ടിണി, ധനനഷ്ടം, ജീവ നഷ്ടം, വിഭവ നഷ്ടം എന്നിവ മുഖേന നിങ്ങളെ നാം പരീക്ഷിക്കുക തന്നെ ചെയ്യും. ( അത്തരം സന്ദര്‍ഭങ്ങളില്‍ ) ക്ഷമിക്കുന്നവര്‍ക്ക്‌ സന്തോഷവാര്‍ത്ത അറിയിക്കുക. തങ്ങള്‍ക്ക്‌ വല്ല ആപത്തും ബാധിച്ചാല്‍ അവര്‍ പറയുക 'ഞങ്ങള്‍ അല്ലാഹുവിന്‍റെ അധീനത്തിലുള്ളവരും അവങ്കലേക്ക്‌ തന്നെ മടങ്ങേണ്ടവരുമാണ്'‌ എന്നായിരിക്കും. അവര്‍ക്കത്രെ തങ്ങളുടെ രക്ഷിതാവിങ്കല്‍ നിന്ന്‌ അനുഗ്രഹങ്ങളും കാരുണ്യവും ലഭിക്കുന്നത്‌. അവരത്രെ സന്‍മാര്‍ഗം പ്രാപിച്ചവര്‍." (2:156)

നിരാശരാവുകയും കോപിക്കുന്നവരുമാണ് ദൈവങ്ങളെല്ലാം; ഇച്ഛാഭംഗത്താൽ വലയുന്നവർ! മാനുഷികമായ ഇത്തരം വികാരങ്ങളുള്ള ദൈവങ്ങളെ മനുഷ്യർ തന്നെ സൃഷ്ടിച്ചതല്ലേ?

ബിജുകുമാർ

അല്ല. മനുഷ്യരടക്കമുള്ള മുഴുവൻ സൃഷ്ടികളെയും പടച്ചു പരിപാലിക്കുന്നവനാണ് യഥാർത്ഥ ദൈവം. ദൈവങ്ങളെയാവട്ടെ, മനുഷ്യരുണ്ടാക്കിയതാണ്. മനുഷ്യരുണ്ടാക്കിയ ദൈവങ്ങളുടെ മേൽ മാനുഷികമായ പരിമിതികൾ ആരോപിക്കപ്പെട്ടിട്ടുണ്ട്. എന്നാൽ മനുഷ്യരെ സൃഷിടിച്ചവൻ മാനുഷികമായ പരിമിതികളിൽ നിന്നെല്ലാം മുക്തനാണ്. അല്ലാഹുവെ പരിചയപ്പെടുത്തുന്ന ഒരു ഖുർആൻ സൂക്തം കാണുക: "അല്ലാഹു; അവനല്ലാതെ ആരാധ്യനില്ല. എന്നെന്നും ജീവിച്ചിരിക്കുന്നവന്‍. എല്ലാം നിയന്ത്രിക്കുന്നവന്‍. മയക്കമോ ഉറക്കമോ അവനെ ബാധിക്കുകയില്ല. ‌ആകാശങ്ങളിലുള്ളതും ഭൂമികളിലുള്ളതുമെല്ലാം അവന്‍റെതാണ്. അവന്‍റെ അനുവാദപ്രകാരമല്ലാതെ അവന്‍റെയടുക്കല്‍ ശുപാര്‍ശ നടത്താനാരുണ്ട്‌? അവരുടെ മുമ്പിലുള്ളതും അവര്‍ക്ക്‌ പിന്നിലുള്ളതും അവന്‍ അറിയുന്നു. അവന്‍റെ അറിവില്‍ നിന്ന്‌ അവന്‍ ഇച്ഛിക്കുന്നതല്ലാതെ യാതൊന്നും അറിയാന്‍ അവർക്ക് കഴിയില്ല. ആകാശങ്ങളേയും ഭൂമിയെയും ഉള്‍കൊള്ളുന്നതാണ് അവന്‍റെ പാദപീഠം. അവയുടെ സംരക്ഷണം അവന്ന്‌ ഒട്ടും ഭാരമുള്ളതല്ല. അത്യുന്നതനും മഹാനുമാണവൻ." (2:255)

എല്ലാറ്റിനെയും സൃഷ്ടിക്കുകയും പരിപാലിക്കുകയും നിയന്ത്രിക്കുകയും ചെയ്യുന്നവന് എന്ത് പരിമിതിയുണ്ടാവാനാണ്? അതുകൊണ്ട് തന്നെ ഇച്ഛാഭംഗത്തിൽ വലയുകയും തന്റെ ചെയ്തികളിൽ ദുഃഖിക്കുകയും നിരാശനാവുകയും ചെയ്യുന്ന ദൈവത്തെ ഇസ്‌ലാമിന് പരിചയമില്ല. അങ്ങനെ ദൈവത്തെ പരിചയപ്പെടുത്തുന്ന ചില ഗ്രന്ഥങ്ങളുണ്ടെന്നത് ശരിയാണ്. വേദഗ്രന്ഥങ്ങളിൽ മാനുഷികമായ കരവിരുതുകൾ നടന്നതു കൊണ്ടുണ്ടായ അബദ്ധങ്ങളിൽ ചിലതാണിവ. മാനുഷികമായ യാതൊരു കൈ കടത്തലുകളും നടന്നിട്ടില്ലാത്ത ഖുർആനിൽ ഇത്തരം പരാമർശങ്ങളൊന്നും കാണാൻ കഴിയില്ല. താൻ ഉദ്ദേശിക്കുന്നത് വൈകല്യങ്ങളില്ലാതെ നടപ്പാക്കുന്നവനാണ് ഖുർആൻ പരിചയപ്പെടുത്തുന്ന അല്ലാഹു. " തീര്‍ച്ചയായും അല്ലാഹു താന്‍ ഉദ്ദേശിക്കുന്നത്‌ പ്രവര്‍ത്തിക്കുന്നു" (22: 14)

എന്നാൽ ദൈവം നിര്‍ഗുണനാണെന്ന വീക്ഷണം ഇസ്‌ലാം അംഗീകരിക്കുന്നില്ല. ഖുര്‍ആന്‍ പരിചയപ്പെടുത്തുന്ന അല്ലാഹു സഗുണസമ്പൂര്‍ണനാണ്. പക്ഷേ, ദൈവികഗുണങ്ങള്‍ മാനുഷിക ഗുണങ്ങളുമായി താരതമ്യം ചെയ്യാവതല്ല. മനുഷ്യന്‍ സൃഷ്ടിയായതുകൊണ്ടുതന്നെ അവന്റെ ഗുണങ്ങള്‍ പരിമിതങ്ങളും സ്ഥലകാലബന്ധനത്തിന്നധീനവുമാണ്. ഇതില്‍നിന്നു വ്യത്യസ്തമായി സ്ഥല-കാലങ്ങളെ സൃഷ്ടിച്ച അല്ലാഹുവിന്റെ ഗുണങ്ങള്‍ പരിമിതികള്‍ക്കതീതമാണ്. ''അവന്ന് തുല്യനായി ആരും തന്നെയില്ല''(112:4)യെന്നും ''അവന്ന് സദൃശ്യമായി യാതൊന്നുമില്ല''(42:11)യെന്നുമുള്ള ഖുര്‍ആനിക പരാമര്‍ശങ്ങള്‍ സ്രഷ്ടാവിന്റെ സത്തയെക്കുറിച്ചു മാത്രമല്ല, ഗുണങ്ങളെക്കുറിച്ചുകൂടിയുള്ളതാണ്. അല്ലാഹുവിന്റെ ഗുണങ്ങള്‍ക്കു തുല്യമായ ഗുണങ്ങള്‍ സൃഷ്ടികള്‍ക്കൊന്നിനുമുണ്ടാവില്ല.

അല്ലാഹു ഇഷ്ടപ്പെടുന്നതായും സ്നേഹിക്കുന്നതായും കാരുണ്യം കാണിക്കുന്നതായുമെല്ലാം പറയുന്നത് പോലെത്തന്നെ അവൻ കോപിക്കുന്നതായും വെറുക്കുന്നതായുമെല്ലാം ഖുർആൻ പരാമർശിക്കുന്നുണ്ട്. ഇവയൊന്നും തന്നെ മനുഷ്യരുടേതുപോലെയുള്ള വികാരങ്ങളല്ല. മനുഷ്യരുടെ കോപവുമായോ വെറുപ്പുമായോ ഇവയെ താരതമ്യം ചെയ്യാനാവില്ല. എന്നാൽ ഇവയൊന്നും അല്ലാഹുവിന്റെ ദുർഗുണങ്ങളായല്ല ഖുർആനും നബിവചനങ്ങളും പരിചയപ്പെടുത്തുന്നത്; കോപവും വെറുപ്പുമെല്ലാം മാനുഷികമായ തലത്തിൽ പോലും അനിവാര്യമായ വികാരങ്ങളായിത്തീരുന്ന അവസ്ഥയുണ്ടാവാറുണ്ട് എന്ന വാസ്തവം എല്ലാവരും അംഗീകരിക്കുന്നതാണ്. തിന്മകളെ വെറുക്കാനും ക്രൂരന്മാരോട് കോപിക്കാനും കഴിയുന്നവരാകണം നല്ല മനുഷ്യർ. അല്ലാഹു തിന്മകളെ വെറുക്കുന്നുവെന്നും പൈശാചികജീവിതം നയിച്ചവരോട് കോപിക്കുമെന്നുമെല്ലാം ഖുർആൻ പഠിപ്പിക്കുന്നുണ്ട്. അവന്റെ കാരുണ്യത്തിന്റെയും നീതിയുടെയും പ്രകാശനങ്ങളാണവ, ആരെങ്കിലും അവനിൽ ആരോപിച്ചവയല്ല.

മതം ഇല്ലെങ്കിലും മനുഷ്യർക്ക് മനുഷ്യരായി ജീവിച്ചു കൂടെ?

ഷമീം ബാദ്‌ഷാ

'മനുഷ്യരായി ജീവിക്കുക' എന്നത് കൊണ്ട് എന്താണ് ഉദ്ദേശിക്കുന്നത് എന്നതിന്റെ അടിസ്ഥാനത്തിലായിരിക്കും ഇതിന്റെ ഉത്തരം. മനുഷ്യരെങ്ങനെ മനുഷ്യരാകണം എന്ന് പഠിപ്പിക്കാനായി മനുഷ്യരെ പടച്ചവൻ പറഞ്ഞയച്ച പ്രവാചകന്മാർ ജീവിച്ചു കാണിച്ച് തന്ന ദൈവികമായ ജീവിതദർശനമാണ് മതം. മാനവികതയിലേക്ക് മനുഷ്യരെ നയിക്കുന്ന നന്മകൾ എന്തൊക്കെയാണെന്നും പൈശാചികതയിലേക്ക് മനുഷ്യരെ ആപതിപ്പിക്കുന്ന തിന്മകൾ എന്തൊക്കെയാണെന്നും പഠിപ്പിക്കുകയാണ് പ്രവാചകന്മാർ ചെയ്തത്. നന്മകളായി മനുഷ്യർ മനസ്സിലാക്കുന്ന കാര്യങ്ങളും തിന്മകളായി മനസ്സിലാക്കുന്ന കാര്യങ്ങളുമെല്ലാം വ്യവച്ഛേദിച്ച് പഠിപ്പിച്ചത് പ്രവാചകന്മാരാണ്, അതല്ലാതെ ആരും ഗവേഷണം ചെയ്ത് കണ്ടുപിടിച്ചതല്ല. പ്രവാചകന്മാരെ അംഗീകരിച്ചില്ലെങ്കിലും അവർ പഠിപ്പിച്ച നന്മ-തിന്മകൾ അംഗീകരിക്കുകയും അവയനുസരിച്ച് ജീവിതം മുന്നോട്ടു നയിക്കുകയുമാണെങ്കിൽ മാനവികതയുൾക്കൊണ്ട് ജീവിക്കാൻ ആർക്കും കഴിയും. പ്രവാചകന്മാരെ അംഗീകരിക്കുന്നവരാണെങ്കിലും അവർ പഠിപ്പിച്ച നന്മകൾ പുലർത്താതെയും തിന്മകളിൽ നിന്ന് അകന്നു നിൽക്കാതെയുമാണ് ഒരാൾ ജീവിക്കുന്നതെങ്കിൽ അയാളുടെ ജീവിതം പൈശാചികമായിരിക്കും. നന്മ-തിന്മകളുടെ വ്യവച്ഛേദനത്തിന് കൃത്യമായ ദൈവികവെളിപാടുകളുടെ അകമ്പടിയുണ്ടാവുമ്പോൾ എന്താണ് നന്മയെന്ന കാര്യത്തിൽ ആശയക്കുഴപ്പമുണ്ടാവുകയില്ല. നന്മകൾ ചെയ്യണമെന്ന് ആഗ്രഹിക്കുന്നവർക്ക്‌ തന്നെ എന്താണ് നന്മയെന്ന് തീരുമാനിക്കാൻ കഴിയാത്ത സാഹചര്യങ്ങളുണ്ടാകാറുണ്ട്. അത്തരം സാഹചര്യങ്ങളിൽ ദൈവികവെളിപാടുകൾ പ്രകാരമുള്ള നന്മ-തിന്മകളുടെ വ്യവച്ഛേദനം വഴി മതവിശ്വാസിക്ക് യഥാർത്ഥ മാനവികതയുടെ വക്താവാകാൻ കഴിയും. ദൈവപ്രീതി മാത്രം ലക്ഷ്യമാക്കി കർമങ്ങൾ ചെയ്യുമ്പോൾ മാത്രമേ സ്വാർത്ഥതയോ ലോകമാന്യമോ ഇല്ലാതെ നന്മകളാൽ ജീവിതത്തെ പുഷ്കലമാക്കാൻ കഴിയൂ. അതാണ് യാഥാർത്ഥത്തിലുള്ള മാനവികത.

വിഷയവുമായി ബന്ധപ്പെട്ട വീഡിയോ

ആരുടെയും ആശ്രയം ആവശ്യമില്ലാത്തവനാണ് അല്ലാഹുവെന്ന് ഖുർആൻ പറയുന്നു. എന്നാൽ മലക്കുകളെ അല്ലാഹു ആശ്രയിക്കുന്നുണ്ട് താനും. വഹ്‌യ്‌ എത്തിക്കാൻ മലക്ക് വേണം അല്ലാഹുവിന്; മറ്റു പല കാര്യങ്ങൾക്കും അല്ലാഹുവിന് മലക്കുകളുടെ സഹായം വേണം. പിന്നെയെങ്ങനെ അല്ലാഹു ആരെയും ആശ്രയിക്കുന്നില്ല എന്ന് പറയും?

അഫ്‌സൽ അബ്ദുൽ ലത്തീഫ്

അല്ലാഹുവിന്റെ പേരുകളിലൊന്നാണ് സ്വമദ്. ഏവര്‍ക്കും ആശ്രയമായിട്ടുള്ളവനും ഒന്നിന്റെയും ആശ്രയമാവശ്യമില്ലാത്തവനും എന്നാണ് സ്വമദ് എന്ന പദത്തിന്റെ വിവക്ഷ. മലക്കുകളും മനുഷ്യരുമെല്ലാം അല്ലാഹുവിന്റെ സൃഷ്ടികളാണ്. ഓരോ സൃഷ്ടികൾക്കും അല്ലാഹു നിശ്ചയിച്ച ചില ദൗത്യങ്ങളുണ്ട്. ആ ദൗത്യങ്ങൾ നിർവഹിക്കുകയാണ് ആ സൃഷ്ടികളുടെ ഉത്തരവാദിത്തം. തങ്ങളെ അല്ലാഹു ഏല്പിച്ച ദൗത്യങ്ങൾ നിർബന്ധിതമായി അനുസരിക്കുന്നവരാണ് മലക്കുകൾ. വ്യത്യസ്ത കാര്യങ്ങൾക്കായി അല്ലാഹു നിശ്ചയിച്ച അവന്റെ സൃഷ്ടികളാണവർ. അവരെ അല്ലാഹു ആശ്രയിക്കുകയല്ല ചെയ്യുന്നത്, അവർ അല്ലാഹുവെ ആശ്രയിക്കുകയാണ്. അവയ്ക്ക് അസ്തിത്വം നൽകിയതും ദൗത്യം നൽകിയതും അത് നിർവഹിക്കാനാവശ്യമായ കഴിവുകളും പാടവവുമെല്ലാം നൽകിയതും അല്ലാഹുവാണ്. അവയെല്ലാം നൽകിയത് പ്രസ്തുത ദൗത്യം നിർവഹിക്കുവാനുള്ള സംവിധാനം എന്ന നിലയ്ക്കാണ്. പിന്നെയെങ്ങനെയാണ് അല്ലാഹു അവരെ ആശ്രയിക്കുന്നുവെന്ന് പറയുക?! തേങ്ങയുണ്ടാവുന്നതിനു വേണ്ടി അല്ലാഹുവുണ്ടാക്കിയ സംവിധാനമായ തെങ്ങിനെ തേങ്ങയുണ്ടാക്കുവാൻ അല്ലാഹു ആശ്രയിക്കുന്നതെന്തിനാണ് എന്ന് ചോദിക്കുന്നത് പോലെ ബാലിശമാണ് മലക്കുകളെ അല്ലാഹു ആശ്രയിക്കുന്നതെന്തിനാണ് എന്ന ചോദ്യവും. പ്രവാചകന്മാർക്ക് വഹ്‌യ്‌ എത്തിക്കുന്നതിന് ചുമതലപ്പെടുത്തപ്പെട്ട മലക്കിനെ അതിന്നായി അല്ലാഹു തന്നെ സൃഷ്ടിച്ചതായിരിക്കെ അല്ലാഹു അതിനെ ആശ്രയിക്കുന്നുവെന്ന് കരുതുന്നത് തന്നെ വിവരക്കേടാണ്. ദൈവദൂതന്മാർക്ക് വഹ്‌യ് എത്തിക്കുന്നതിനായി അല്ലാഹു ഉണ്ടാക്കിയ സംവിധാനമാണ് മലക്ക്. തന്റെ ദൗത്യനിർവഹണത്തിനായി മലക്ക് അല്ലാഹുവിനെയാണ് ആശ്രയിക്കുന്നത് എന്നർത്ഥം.

എന്തുകൊണ്ടാണ് അല്ലാഹു സ്ത്രീകളെ പ്രവാചകന്മാരായി നിയോഗിക്കാതിരുന്നത്? സ്ത്രീകളെ ഇസ്‌ലാം അവഗണിക്കുന്നുവെന്നതിന് ഇത് തന്നെ നല്ലൊരു തെളിവല്ലേ?

അൻസാർ .N

അല്ലാഹുവിന്റെ ചെയ്തികളിലെല്ലാം ന്യായവും കാരുണ്യവുമുണ്ടാവും എന്ന് മനസ്സിലാക്കുന്നവനാണ് മുസ്‌ലിം. പ്രസ്തുത ന്യായങ്ങൾ ചിലപ്പോൾ നമുക്ക് മനസ്സിലായിക്കൊള്ളണമെന്നില്ല. മനസ്സിലായാലും ഇല്ലെങ്കിലും മാനവികതക്കോ നീതിക്കോ നിരക്കാത്ത യാതൊന്നും അല്ലാഹുവിൽ നിന്നുണ്ടാവുകയില്ലെന്ന സത്യം മുസ്‌ലിംകളെല്ലാം ഉൾക്കൊള്ളുന്നു.

'താങ്കൾക്കു മുമ്പ് പുരുഷന്മാരെയല്ലാതെ നാം ദൂതന്മാരായി നിയോഗിച്ചിട്ടില്ല'(21:7) എന്ന് മുഹമ്മദ് നബി(സ)യെ അഭിസംബോധന ചെയ്തു കൊണ്ട് ഖുർആനിൽ അല്ലാഹു പറയുന്നതിൽ നിന്ന് ദൂത് നൽകിക്കൊണ്ട് ഒരു സ്ത്രീയെയും അല്ലാഹു അയച്ചിട്ടില്ലെന്ന് വ്യക്തമാവുന്നുണ്ട്. എന്നാൽ ചില സ്ത്രീകൾക്ക് അല്ലാഹു ബോധനം നൽകിയതായി ഖുർആൻ പറയുന്നുണ്ട്. യേശുമാതാവായ മറിയവും മോശെയുടെ മാതാവും ഉദാഹരണം. ഇതിൽ നിന്ന് വഹ്‌യ്‌ ലഭിച്ചവർ എന്ന നിലയിൽ അവരെ പ്രവാചകന്മാരായി (നബി) കണക്കാക്കാമെന്നും, സത്യമതപ്രബോധനമെന്ന ദൈവദൂതന്മാരുടെ (റസൂൽ) ദൗത്യം സ്ത്രീകളെ ഏൽപിച്ചിട്ടില്ലെന്നാണ് ഖുർആൻ(21:7) വ്യക്തമാക്കുന്നതെന്നും അഭിപ്രായപ്പെട്ട പണ്ഡിതന്മാരുണ്ട്. അബുൽഹസൻ അശ്അരി, ഇമാം ഖുർത്തുബി, ഇമാം ഇബ്നു ഹസം (റ) എന്നിവർ ഈ അഭിപ്രായക്കാരാണ്. പ്രത്യേക സന്ദർഭത്തിൽ അല്ലാഹു ചില ബോധനങ്ങൾ നൽകിയെന്നല്ലാതെ മർയമിനും ആസ്യ(റ)ക്കുമൊന്നും പ്രവാചകത്വം നൽകിയെന്ന് പറയാൻ കഴിയില്ലെന്നാണ് ഭൂരിപക്ഷം പണ്ഡിതന്മാരുടെയും അഭിപ്രായം. സത്യം അല്ലാഹുവിന്നറിയാം. സത്യമതപ്രബോധനമെന്ന ദൈവദൂതന്മാരുടെ (റസൂൽ) ദൗത്യം സ്ത്രീകളെ ഏൽപിച്ചിട്ടില്ലെന്ന കാര്യത്തിൽ പണ്ഡിതന്മാരെല്ലാം ഒരേ അഭിപ്രായക്കാരാണ്.

എന്തുകൊണ്ടാണ് സ്ത്രീകളെ ദൈവദൂതന്മാരായി അല്ലാഹു തെരെഞ്ഞെടുക്കാതിരുന്നത് എന്ന് നമുക്കറിയില്ല; അത് അല്ലാഹുവിന്റെ യുക്തിയാണ്. നമുക്ക് മനസ്സിലാക്കാൻ കഴിയുന്ന ചിലവസ്തുതകൾ ഇങ്ങനെ സംക്ഷേപിക്കാം:

ഒന്ന്) പ്രവാചകത്വം വലിയൊരു ഉത്തരവാദിത്തമാണ്; ഏറെ ത്യാഗങ്ങളും പ്രയാസങ്ങളും ആവശ്യപ്പെടുന്ന ഒരു ഉത്തരവാദിത്തം. സ്ത്രീകൾക്ക് പ്രസ്തുത ഉത്തരവാദിത്തം നിർവഹിക്കുക ഏറെ പ്രയാസകരമായിരിക്കും. ബുദ്ധിമുട്ടുകൾ ഏറെ സഹിക്കേണ്ടി വരുന്ന ആ ഉത്തരവാദിതത്വം നിർവഹിക്കാൻ സ്ത്രീകളെ തെരെഞ്ഞെടുക്കാതിരിക്കുക വഴി അല്ലാഹു അവരോട് വലിയ കാരുണ്യമാണ് ചെയ്തിരിക്കുന്നത്.

രണ്ട്) സ്ത്രീശരീരം പൊതുവിൽ പുരുഷന്മാരെ ലൈംഗീകമായി പ്രചോദിപ്പിക്കുന്നതും ഉത്തേജിപ്പിക്കുന്നതുമാണ്. പുരുഷന്മാർക്കും സ്ത്രീകൾക്കുമെല്ലാം ദൈവികസന്ദേശം പകർന്നു നൽകുകയെന്ന ഭാരിച്ച ഉത്തരവാദിത്തം നിർവഹിക്കേണ്ടവർക്ക് തങ്ങളുടെ പ്രബോധിതരുടെ തൃഷ്ണയോടെയുള്ള പെരുമാറ്റം ഏറെ പ്രയാസങ്ങൾ സൃഷ്ടിക്കും.

മൂന്ന്) പ്രവാചത്വം പോലെയുള്ള ഭാരിച്ച ഉത്തരവാദിത്തങ്ങൾ നിർവഹിക്കുന്നവർക്ക് അവധാനതയോടെയും ബുദ്ധിപൂർവ്വകമായും പല തീരുമാനങ്ങളും സ്വയം എടുക്കേണ്ടി വരും. അതിന്ന് അപാരമായ യുക്തിബോധമാണാവശ്യം. വരുംവരായ്കകളെക്കുറിച്ച് ചിന്തിച്ച് പെട്ടെന്ന് എടുക്കേണ്ട തീരുമാനങ്ങളും അവയിലുണ്ടാവും. വികാരപ്രധാനമായ സ്ത്രൈണപ്രകൃതിക്ക് അതിന് കഴിഞ്ഞുകൊള്ളണമെന്നില്ല.

നാല്) മതപരമായ കർമങ്ങൾക്ക് നേതൃത്വം വഹിക്കേണ്ടവരാണ് പ്രവാചകന്മാർ. ആർത്തവവും പ്രസവവും പോലെയുള്ള സ്ത്രീഅവസ്ഥകളിൽ അതിന്ന് അവർക്ക് കഴിയില്ല. അവരിൽ പ്രവാചകത്വമെന്ന ബാധ്യത ഏൽപ്പിച്ചാൽ ഇത്തരം സാഹചര്യങ്ങളിൽ അവരുടെ സ്ത്രൈണപ്രകൃതിയോട് ചെയ്യുന്ന ക്രൂരതയായിത്തീരും.

അഞ്ച്) സ്ത്രീകളുടെ പ്രകൃതിപരമായ ചോദനയാണ് മാതൃത്വം. മക്കളെ വളർത്തുകയും അവർക്ക് ശാരീരികവും വൈകാരികവുമായ ആരോഗ്യം നൽകുകയും ചെയ്യുവാനാവശ്യമായ അനുകൂലനങ്ങളെല്ലാം പടക്കപ്പെട്ടിരിക്കുന്നത് സ്ത്രീശരീരത്തിലാണ്. പ്രവാചകത്വമേൽപ്പിക്കപ്പെട്ടാൽ സ്ത്രീകൾക്ക് തങ്ങളുടെ മക്കളോടും ഇണകളോടുമുള്ള ഉത്തരവാദിത്വങ്ങൾ നിർവഹിക്കാൻ അത് പ്രയാസകരമായിതത്തീരും.

പ്രവാചകത്വത്തിന് സ്ത്രീകളെ പരിഗണിക്കാത്ത അല്ലാഹുവിന്റെ നടപടി അവളോടുള്ള അവഗണനയല്ല, അവളുടെ പ്രകൃതിയോടുള്ള ആദരവും കാരുണ്യവുമാണ് സൂചിപ്പിക്കുന്നത് എന്ന വസ്തുതയാണ് ഇത് വ്യക്തമാക്കുന്നത്. ആത്മീയമായ ഔന്നത്യത്തിന്റെ കാര്യത്തിലാണെങ്കിൽ സ്ത്രീക്ക് പുരുഷനെപ്പോലെയോ അതിനേക്കാളധികമോ ഉയരാൻ കഴിയുമെന്ന് ഖുർആൻ തന്നെ പഠിപ്പിക്കുന്നുണ്ട്. സകല സത്യവിശ്വാസികൾക്കുമുള്ള മാതൃകകളായി ഖുർആൻ വരച്ചു കാണിച്ചത് രണ്ട് സ്ത്രീകളെയാണ്. (66:11,12). ഫറോവയുടെ പത്നിയും യേശുവിന്റെ മാതാവും പുരുഷ-സ്ത്രീ വ്യത്യാസമില്ലാതെ സകല സത്യവിശ്വാസികൾക്കുമുള്ള മാതൃകയാണെന്ന് ഖുർആൻ പറയുമ്പോൾ ആത്മീയമായി അവർക്ക് എത്രയും ഉയരാൻ കഴിമെന്നു തന്നെയാണ് അത് വ്യക്തമാക്കുന്നത്. സ്ത്രീകളിൽ നിന്ന് പ്രവാചകന്മാരെ തെരെഞ്ഞെടുക്കാതിരുന്നത് അവർക്ക് ആത്മീയമായ ഔന്നത്യം നിഷേധിച്ചതുകൊണ്ടല്ല, പ്രത്യുത അവർക്കത് പ്രയാസകരമാണ് എന്നതുകൊണ്ടായിരിക്കാം എന്നാണ് ഇതെല്ലാം വ്യക്തമാക്കുന്നത്.

അടിമത്തത്തിന്റെ കെടുതികള്‍ ഇല്ലായ്മ ചെയ്യുന്നതിനായി അഞ്ച് മാര്‍ഗങ്ങളിലൂടെ ഇസ്‌ലാം ശ്രമിച്ചതായി കാണാന്‍ കഴിയും.

  1. സാഹോദര്യം വളര്‍ത്തി

സര്‍വ മനുഷ്യരും ദൈവസൃഷ്ടികളും ഒരേ മാതാപിതാക്കളുടെ മക്കളുമാണെന്ന ബോധം വളര്‍ത്തിക്കൊണ്ട് അടിമയും ഉടമയുമെല്ലാം സഹോദരങ്ങളാണെന്ന ധാരണയുണ്ടാക്കുകയാണ് ഖുര്‍ആന്‍ ആദ്യമായി ചെയ്തത്. ''മനുഷ്യരേ, ഒരു പുരുഷനില്‍നിന്നും സ്ത്രീയില്‍നിന്നു മാണ് നിങ്ങളെ നാം പടച്ചിരിക്കുന്നത്, തീര്‍ച്ച. ഗോത്രങ്ങളും ജനപദങ്ങളുമായി നിങ്ങളെ തിരിച്ചിരിക്കുന്നത് പരസ്പരം തിരിച്ചറിയുന്നതി നായാണ്. അല്ലാഹുവിങ്കല്‍ നിങ്ങളിലെ ഭക്തനാണ് ഉത്തമന്‍'' (ഖുര്‍ആന്‍ 49:13).

ജന്മത്തിന്റെ പേരിലുള്ള സകലമാന സങ്കുചിതത്തങ്ങളുടെയും അടിവേരറുക്കുകയാണ് ഇവിടെ ഖുര്‍ആന്‍ ചെയ്തിരിക്കുന്നത്. നിറത്തി ന്റെയേകാ കുലത്തിന്റെയോ പണത്തിന്റെയോ അടിസ്ഥാനത്തിലല്ല; പ്രത്യുത, ഭക്തിയുടെ അടിസ്ഥാനത്തിലാണ് ശ്രേഷ്ഠത നിശ്ചയിക്കപ്പെ ടുന്നതെന്നാണ് പ്രവാചകന്‍ (സ) പഠിപ്പിച്ചത്. ''അറബിക്ക് അനറബിയേക്കാളുമോ അനറബിക്ക് അറബിയെക്കാളുമോ വെളുത്തവന് കറുത്തവനെക്കാളുമോ കറുത്തവന് വെളുത്തവനെക്കാളുമോ യാതൊരു ശ്രേഷ്ഠതയുമില്ല, ദൈവഭക്തിയുടെ അടിസ്ഥാനത്തിലല്ലാതെ'' (ത്വബ്‌രി).

അടിമകളെക്കുറിച്ച് പരാമര്‍ശിക്കുന്നിടത്ത് ''നിങ്ങള്‍ ചിലര്‍ ചിലരില്‍ നിന്നുണ്ടായവരാണല്ലോ'' (ഖുര്‍ആന്‍ 4:25) എന്ന് വിശുദ്ധഖുര്‍ആന്‍ പറയുന്നുണ്ട്. അടിമയും ഉടമയുമെല്ലാം സഹോദരന്മാരാണെന്നും സാഹചര്യങ്ങളാണ് ചിലരുടെമേല്‍ അടിമത്വം അടിച്ചേല്‍പിച്ചതെന്നു മുള്ള വസ്തുതകള്‍ വ്യക്തമാക്കുകയാണ് ഇവിടെ ഖുര്‍ആന്‍ ചെയ്യുന്നത്.

  1. അടിമയുടെ അവകാശങ്ങളെക്കുറിച്ച് ബോധ്യം വരുത്തി

അടിമ കേവലം ഒരു ഉപഭോഗവസ്തു മാത്രമായിരുന്നു, പൗരാണിക സമൂഹങ്ങളിലെല്ലാം. അവന് ബാധ്യതകള്‍ മാത്രമേ ഉണ്ടായിരുന്നു ള്ളൂ. ഉടമയുടെ സുഖസൗകര്യങ്ങള്‍ വര്‍ധിപ്പിക്കുന്നതിനു വേണ്ടി യത്‌നിക്കുകയായിരുന്നു അവന്റെ ബാധ്യത- അതില്‍ യാതൊരു വിട്ടുവീ ഴ്ചയുമുണ്ടായിരുന്നില്ല. ഉടമക്കുവേണ്ടി പണിയെടുക്കുന്നതിന് അടിമയുടെ ആരോഗ്യം നിലനിര്‍ത്തേണ്ടത് അനിവാര്യമായിരുന്നു. അതി നുവേണ്ടി മാത്രമായിരുന്നു അവന് ഭക്ഷണം നല്‍കിയിരുന്നത്. കാലികള്‍ക്കു നല്‍കുന്ന സൗകര്യംപോലും ഇല്ലാത്ത തൊഴുത്തുകളിലായി രുന്നു അവരെ താമസിപ്പിച്ചിരുന്നത്. അവര്‍ക്ക് നല്‍കിയിരുന്ന വസ്ത്രമാകട്ടെ, കേവലം നാണം മറക്കാന്‍പോലും അപര്യാപ്തമായ രീതി യിലുള്ളതായിരുന്നു. അതും വൃത്തികെട്ട തുണിക്കഷ്ണങ്ങള്‍!

ഇസ്‌ലാം ഈ അവസ്ഥക്ക് മാറ്റം വരുത്തി. അടിമ ഉടമയുടെ സഹോദരനാണെന്നും അവന് അവകാശങ്ങളുണ്ടെന്നും പഠിപ്പിച്ചു. പ്രവാച കന്‍ (സ) നിഷ്‌കര്‍ഷിച്ചു: ''നിങ്ങളുടെ സഹോദരങ്ങളും ബന്ധുക്കളുമാണവര്‍! തന്റെ കീഴിലുള്ള ഒരു സഹോദരന്  താന്‍ കഴിക്കുന്നതുപോ ലെയുള്ള ഭക്ഷണവും താന്‍ ധരിക്കുന്നതുപോലെയുള്ള വസ്ത്രവും നല്‍കിക്കൊള്ളട്ടെ. അവര്‍ക്ക് കഴിയാത്ത ജോലികളൊന്നും അവരെ ഏല്‍പിക്കരുത്. അവര്‍ക്ക് പ്രയാസകരമായ വല്ല പണികളും ഏല്‍പിക്കുകയാണെങ്കില്‍ നിങ്ങള്‍ അവരെ സഹായിക്കണം'' (ബുഖാരി, മുസ്‌ലിം).

അധ്വാനിക്കുകയെന്നതു മാത്രമായിരുന്നില്ല പൗരാണിക സമൂഹങ്ങളില്‍ അടിമയുടെ കര്‍ത്തവ്യം. യജമാനന്റെ ക്രൂരമായ വിനോദങ്ങള്‍ ഏറ്റുവാങ്ങുവാന്‍ കൂടി വിധിക്കപ്പെട്ടവനായിരുന്നു അവന്‍. അധ്വാനവേളകളില്‍ ക്രൂരമായ ചാട്ടവാറടികള്‍! യജമാനന്റെ ആസ്വാദനത്തി നുവേണ്ടി കൊല്ലുവാനും കൊല്ലപ്പെടുവാനും തയാറാവേണ്ട അവസ്ഥ! ഇത് മാറണമെന്ന് ഖുര്‍ആന്‍ കല്‍പിച്ചു. അടിമകളോട് നല്ല നിലയില്‍ പെരുമാറണമെന്ന് നിഷ്‌കര്‍ഷിച്ചു. ''ബന്ധുക്കളോടും അനാഥകളോടും പാവങ്ങളോടും കുടുംബബന്ധമുള്ള അയല്‍ക്കാരോടും അന്യരായ അയല്‍ക്കാരോടും സഹവാസിയോടും വഴിപോക്കനോടും നിങ്ങളുടെ വലതുകൈകള്‍ ഉടമപ്പെടുത്തിയ അടിമകളോടും നല്ല നിലയില്‍ വര്‍ത്തിക്കുക'' (ഖുര്‍ആന്‍ 4:36).

പ്രവാചകന്‍ (സ) വ്യക്തമായി പറഞ്ഞു: ''വല്ലവനും തന്റെ അടിമയെ വധിച്ചാല്‍ നാം അവനെയും വധിക്കും. വല്ലവനും തന്റെ അടിമയെ അംഗവിഛേദം ചെയ്താല്‍  നാം അവനെയും അംഗവിഛേദം ചെയ്യും. വല്ലവനും തന്റെ അടിമയെ ഷണ്ഡീകരിച്ചാല്‍ നാം അവനെയും ഷണ്ഡീകരിക്കും'' (മുസ്‌ലിം, അബൂദാവൂദ്).

യജമാനന് ഇഷ്ടമുള്ളതെല്ലാം ചെയ്യാവുന്ന 'ചരക്ക്' എന്ന അവസ്ഥയില്‍നിന്ന് അടിമ സ്വന്തമായ വ്യക്തിത്വവും അവകാശങ്ങളുമുള്ളവനാ യിത്തീരുകയായിരുന്നു. അടിമകളെ ഷണ്ഡീകരിക്കുകയെന്ന അതിനികൃഷ്ടമായ സമ്പ്രദായം നിലനിന്നിരുന്ന സമൂഹത്തിലാണ് അവരെ ഷണ്ഡീകരിച്ചാല്‍ അതു ചെയ്ത യജമാനനെ ഞാനും ഷണ്ഡീകരിക്കുമെന്ന് പ്രവാചകന്‍ (സ) അര്‍ഥശങ്കയില്ലാത്തവിധം വ്യക്തമാക്കിയത്. ലൈംഗിക വികാരം നശിപ്പിച്ചുകൊണ്ട് അടിമകളെക്കൊണ്ട് മൃഗതുല്യമായി അധ്വാനിപ്പിക്കുന്നതിനുവേണ്ടിയായിരുന്നു അവരെ ഷണ്ഡീക രിച്ചിരുന്നത്. ഇത് നിരോധിച്ച ഇസ്‌ലാം അടിമകള്‍ക്കും വികാരശമനത്തിനും മാര്‍ഗമുണ്ടാക്കണമെന്ന് പ്രത്യേകം നിഷ്‌കര്‍ഷിക്കുന്നുണ്ട്. ''നിങ്ങളിലുള്ള അവിവാഹിതരെയും നിങ്ങളുടെ അടിമകളില്‍ നിന്നും അടിമസ്ത്രീകളില്‍നിന്നും നല്ലവരായിട്ടുള്ളവരെയും നിങ്ങള്‍ വിവാ ഹബന്ധത്തിലേര്‍പ്പെടുത്തുക. അവര്‍ ദരിദ്രരാണെങ്കില്‍ അല്ലാഹു തന്റെ അനുഗ്രഹത്തില്‍നിന്ന് അവര്‍ക്ക് ഐശ്വര്യം നല്‍കുന്നതാണ്. അല്ലാഹു വിപുലമായ കഴിവുള്ളവനും സര്‍വജ്ഞനുമത്രേ'' (ഖുര്‍ആന്‍ 24:32).

അടിമസ്ത്രീകളെ വേശ്യാവൃത്തിക്ക് നിര്‍ബന്ധിക്കുന്ന സമ്പ്രദായത്തെ ഖര്‍ആന്‍ വിലക്കി. ''ചാരിത്ര്യശുദ്ധിയോടെ ജീവിക്കാനാഗ്രഹിക്കുന്ന നിങ്ങളുടെ അടിമസ്ത്രീകളെ ഐഹിക ജീവിതത്തിന്റെ വിഭവം ആഗ്രഹിച്ചുകൊണ്ട് നിങ്ങള്‍ വേശ്യാവൃത്തിക്ക് നിര്‍ബന്ധിക്കരുത്'' (ഖുര്‍ആന്‍ 24:33).

അടിമയുടെ അഭിമാനത്തിന് ക്ഷതം പറ്റുന്ന പരാമര്‍ശങ്ങള്‍ പോലും നടത്തരുതെന്നാണ് പ്രവാചകന്‍ (സ) പഠിപ്പിച്ചത്. ''അത് എന്റെ ദാസ ന്‍, ഇത് എന്റെ ദാസി എന്നിങ്ങനെ നിങ്ങള്‍ പറയരുത്'' എന്ന് അദ്ദേഹം പഠിപ്പിച്ചു. അടിമക്കും അഭിമാനമുണ്ടെന്നും അത് ക്ഷതപ്പെടു ത്താന്‍ ആര്‍ക്കും അവകാശമില്ലെന്നുമുള്ള വസ്തുതയാണെന്നും വ്യക്തമാക്കി. ഒരു അടിമയുമായി ശണ്ഠകൂടിയപ്പോള്‍  അയാളെ 'കറുത്ത പെണ്ണിന്റെ മോനേ' എന്നുവിളിച്ച തന്റെ ശിക്ഷ്യനായ അബൂദര്‍റി(റ)നെ പ്രവാചകന്‍ (സ) ഗുണദോഷിച്ചത് ഇങ്ങനെയായിരുന്നു. ''അബു ദര്‍റേ... അന്തരാളകാലത്തെ സംസ്‌കാരത്തില്‍ ചിലത് ഇനിയും താങ്കളില്‍ ബാക്കിയുണ്ട്''.

അടിമയ്ക്ക് നേതാവാകുവാന്‍ പോലും അവകാശമുണ്ടെന്നും അങ്ങനെ നേതാവായി നിയോഗിക്കപ്പെട്ടു കഴിഞ്ഞാല്‍ അയാളെ അനുസരി ക്കേണ്ടത് നിര്‍ബന്ധമാണെന്നും ഇസ്‌ലാം പഠിപ്പിക്കുന്നുണ്ട്്. ''നിങ്ങളുടെ നേതാവായി വരുന്നത് ഉണങ്ങിയ മുന്തിരിപോലെ തലയുള്ള ഒരു നീഗ്രോ അടിമയാണെങ്കിലും നിങ്ങള്‍ അയാളെ കേള്‍ക്കുകയും അനുസരിക്കുകയും വേണം''. അടിമയെ പിറകില്‍ നടത്തിക്കൊണ്ട് വാഹന ത്തില്‍ സഞ്ചരിക്കുകയായിരുന്ന ഒരാളോട് പ്രവാചക ശിക്ഷ്യനായ അബൂഹുറയ്‌റ (റ) പറഞ്ഞു. ''നിന്റെ പിറകില്‍ അവനെയും കയറ്റുക. നിന്റെ സഹോദരനാണവന്‍, നിന്‍േറതുപോലുള്ള ആത്മാവാണ് അവനുമുള്ളത്''.

അടിമക്കും ഉടമക്കും ഒരേ ആത്മാവാണുള്ളതെന്നും അവര്‍ തമ്മില്‍ സഹോദരങ്ങളാണെന്നും പഠിപ്പിച്ചുകൊണ്ട് അടിമ-ഉടമ ബന്ധത്തിന് ഒരു പുതിയ മാനം നല്‍കുകയാണ് ഇസ്‌ലാം ചെയ്തത്. അടിമ, ഉടമയുടെ അധീനത്തിലാണെന്നത് ശരിതന്നെ. എന്നാല്‍, അടിമയുടെ അവകാ ശങ്ങള്‍ വകവെച്ചുകൊടുക്കാന്‍ ഉടമ ബാധ്യസ്ഥനാണ്. ഭക്ഷണം, വസ്ത്രം, ലൈംഗികത തുടങ്ങിയ അടിമയുടെ ആവശ്യങ്ങള്‍ നിര്‍വഹിച്ചു കൊടുക്കേണ്ടത് അയാളുടെ ചുമതലയാണ്. അടിമയെ ഉപദ്രവിക്കാന്‍ പാടില്ല. അയാളെ പ്രയാസകരമായ ജോലികള്‍ ഏല്‍പിച്ച് ക്ലേശിപ്പി ക്കുവാനും പാടില്ല. ഇങ്ങനെ, ചരിത്രത്തിലാദ്യമായി അടിമയെ സ്വതന്ത്രന്റെ വിതാനത്തിലേക്കുയര്‍ത്തുകയെന്ന വിപ്ലവം സൃഷ്ടിക്കുക യാണ് ഇസ്‌ലാം ചെയ്തത്. ഇതുവഴി ഉടമയുടെയും അടിമയുടെയും മാനസികാവസ്ഥകള്‍ തമ്മിലുള്ള അന്തരം കുറക്കുവാന്‍ ഇസ്‌ലാമിന് സാധിച്ചു. തന്റെ ഇഷ്ടങ്ങളെല്ലാം പ്രയോഗിക്കാവുന്ന ഒരു ചരക്ക് മാത്രമാണ് അടിമയെന്ന വിചാരത്തില്‍നിന്ന് ഉടമയും, സഹിക്കുവാനും ക്ഷമിക്കുവാനും നിര്‍വഹിക്കുവാനും മാത്രം വിധിക്കപ്പെട്ടവനാണ് താനെന്ന വിചാരത്തില്‍നിന്ന് അടിമയും സ്വതന്ത്രരാവുകയായിരുന്നു ഈ വിപ്ലവത്തിന്റെ ഫലം.

  1. അടിമമോചനം ഒരു പുണ്യകര്‍മമായി പ്രഖ്യാപിച്ചു

അവകാശങ്ങളുള്ള ഒരു അസ്തിത്വമായി അടിമയെ പ്രഖ്യാപിക്കുക വഴി അടിമത്തത്തെ സാങ്കേതികമായി ഇല്ലാതാക്കുകയാണ് ഇസ്‌ലാം ചെയ്തത്. എന്നാല്‍, ഇതുകൊണ്ടും നിര്‍ത്താതെ ആ സമ്പ്രദായത്തെ പ്രായോഗികമായിത്തന്നെ നിര്‍മൂലനം ചെയ്യുവാന്‍  ആവശ്യമായ നടപ ടിയിലേക്ക്  ഇസ്‌ലാം തിരിയുകയുണ്ടായി. അടിമമോചനം ഒരു പുണ്യകര്‍മമായി പ്രഖ്യാപിക്കുകയായിരുന്നു അടിമ സമ്പ്രദായത്തെ പ്രാ യോഗികമായി ഇല്ലാതാക്കുവാന്‍ ഇസ്‌ലാം സ്വീകരിച്ച നടപടികളിലൊന്ന്. ''അവരുടെ ഭാരങ്ങളും അവരുടെ മേലുണ്ടായിരുന്ന വിലങ്ങു കളും അദ്ദേഹം ഇറക്കിവെക്കുകയും ചെയ്യുന്നു'' (7:157) എന്ന ഖുര്‍ആനിക പരാമര്‍ശത്തെ അന്വര്‍ഥമാക്കുന്നതായിരുന്നു അടിമമോചനത്തി ന്റെ വിഷയത്തില്‍ പ്രവാചകന്റെ (സ) നിലപാട്.

അടിമമോചനം അതിവിശിഷ്ടമായ ഒരു പുണ്യകര്‍മമാണെന്ന് വ്യക്തമാക്കുന്ന ഖുര്‍ആന്‍ സൂക്തം ഇങ്ങനെയാണ്. ''എന്നിട്ട് അവന്‍ ആ മലമ്പാത താണ്ടിക്കടന്നില്ല. ആ മലമ്പാതയെന്താണെന്ന് നിനക്കറിയാമോ? അടിമമോചനം. അല്ലെങ്കില്‍ പട്ടിണിയുടെ നാളില്‍ കുടുംബബന്ധ മുള്ള ഒരു അനാഥക്കോ കടുത്ത ദാരിദ്ര്യമുള്ള ഒരു സാധുവിനോ ഭക്ഷണം നല്‍കുക'' (90:12-16).

അടിമമോചനത്തിന്റെ കാര്യത്തില്‍ പ്രവാചകന്‍ (സ)തന്നെ മാതൃക കാണിച്ചുകൊണ്ടാണ് അനുചരന്മാരെ അതിനുവേണ്ടി പ്രേരിപ്പിച്ചത്. തന്റെ കൈവശമുണ്ടായിരുന്ന അടിമയെ അദ്ദേഹം മോചിപ്പിച്ചു. അദ്ദേഹത്തിന്റെ അനുചരന്മാര്‍ പ്രസ്തുത പാത പിന്തുടര്‍ന്നു. സഖാക്ക ളില്‍ പ്രമുഖനായിരുന്ന അബൂബക്കര്‍ (റ) സത്യനിഷേധികളില്‍നിന്ന് അടിമകളെ വിലയ്ക്കുവാങ്ങി മോചിപ്പിക്കുന്നതിനായി അളവറ്റ സമ്പ ത്ത് ചെലവഴിച്ചിരുന്നതായി കാണാനാവും.

അടിമമോചനത്തെ പ്രോല്‍സാഹിപ്പിക്കുന്ന ഒട്ടേറെ നബിവചനങ്ങള്‍ കാണാന്‍ കഴിയും: ''സത്യവിശ്വാസിയായ ഒരു അടിമയെ ആരെങ്കിലും മോചിപ്പിച്ചാല്‍ ആ അടിമയുടെ ഓരോ അവയവത്തിനും പകരം  അല്ലാഹു അവന്റെ അവയവത്തിന് നരകത്തില്‍നിന്ന് മോചനം നല്‍കു ന്നതാണ്. അഥവാ കയ്യിന് കയ്യും കാലിന് കാലും ഗുഹ്യാവയവത്തിന് ഗുഹ്യാവയവവും വരെ'' (ബുഖാരി, മുസ്‌ലിം).

ഒരിക്കല്‍ സഖാവായിരുന്ന അബൂദര്‍റ്‌(റ) നബി(സ)യോട് ചോദിച്ചു: 'അടിമമോചനത്തില്‍ ഏറ്റവും ശ്രേഷ്ഠമായത് ഏതാണ്? തിരുമേനി പ്രതിവചിച്ചു: 'യജമാനന് ഏറ്റവും വിലപ്പെട്ട  അടിമകളെ മോചിപ്പിക്കല്‍'.

അല്ലാഹുവിന്റെ പ്രതിഫലത്തിന് രണ്ടു തവണ അര്‍ഹരാവുന്നവരെ എണ്ണിപ്പറയവെ തിരുമേനി (സ) പറഞ്ഞു: ''തന്റെ കീഴിലുള്ള അടിമ സ്ത്രീയെ സംസ്‌കാര സമ്പന്നയാക്കുകയും അവള്‍ക്ക് ഏറ്റവും നന്നായി വിദ്യാഭ്യാസം നല്‍കുകയും പിന്നീട് അവളെ മോചിപ്പിച്ച് സ്വയം വിവാഹം കഴിക്കുകയും ചെയ്തവനും ഇരട്ടി പ്രതിഫലമുണ്ട്'' (ബുഖാരി, മുസ്‌ലിം).

പടച്ചതമ്പുരാനില്‍നിന്നുള്ള പ്രതിഫലം കാംക്ഷിച്ചുകൊണ്ട് സത്യവിശ്വാസികള്‍ പ്രവാചകന്റെ കാലത്തും ശേഷവും അടിമകളെ മോചിപ്പി ക്കുവാന്‍ തുടങ്ങി. ഇതുകൂടാതെ സകാത്തിന്റെ ധനം പോലും അടിമമോചനത്തിന് ചെലവഴിക്കുന്ന അവസ്ഥയുണ്ടായി. ഉമറുബ്‌നു അബ്ദില്‍ അസീസിന്റെ ഭരണകാലത്ത് സകാത്ത് സ്വീകരിക്കുവാന്‍ ഒരു ദരിദ്രന്‍ പോലുമില്ലാത്ത അവസ്ഥ സംജാതമായെന്നും അപ്പോള്‍ അടിമകളെ വിലക്കെടുത്ത് മോചിപ്പിക്കാനാണ് സകാത്ത് ഇനത്തിലുള്ള ധനം ചെലവഴിക്കപ്പെട്ടതെന്നും ചരിത്രത്തില്‍ കാണാന്‍ കഴിയും.

  1. പലതരം കുറ്റങ്ങള്‍ക്കുമുള്ള പ്രായശ്ചിത്തമായി അടിമമോചനം നിശ്ചയിക്കപ്പെട്ടു.

അടിമമോചനത്തെ ഒരു പുണ്യകര്‍മമായി അവതരിപ്പിച്ചുകൊണ്ട് സത്യവിശ്വാസികളെ അക്കാര്യത്തില്‍ പ്രോല്‍സാഹിപ്പിച്ചതോടൊപ്പം തന്നെ പലതരം കുറ്റകൃത്യങ്ങള്‍ക്കുള്ള പ്രായശ്ചിത്തമായി അടിമമോചനത്തെ ഇസ്‌ലാം നിശ്ചയിച്ചു. മനഃപൂര്‍വമല്ലാത്ത കൊലപാതകം, ഭാര്യയെ സമീപിക്കുകയില്ലെന്ന ശപഥത്തിന്റെ ലംഘനം തുടങ്ങിയ പാപങ്ങള്‍ക്കുള്ള പ്രായശ്ചിത്തം ഒരു അടിമയെ മോചിപ്പിക്കുകയാണ്. ദൈവിക പ്രതിഫലം കാംക്ഷിച്ചുകൊണ്ടുമാത്രം അടിമകളെ മോചിപ്പിക്കാന്‍ തയാറില്ലാത്തവരെ സംബന്ധിച്ചിടത്തോളം അത് നിര്‍ബന്ധമാ ക്കിത്തീര്‍ക്കുന്ന അവസ്ഥയാണ് തെറ്റുകള്‍ക്കുള്ള പ്രായശ്ചിത്തമായി അടിമകളെ മോചിപ്പിക്കണമെന്ന വിധി.

  1. മോചനമൂല്യത്തിനു പകരമായി സ്വാതന്ത്ര്യം നല്‍കുന്ന സംവിധാനമുണ്ടാക്കി.

മുകളില്‍ പറഞ്ഞ മാര്‍ഗങ്ങളിലൂടെയൊന്നും സ്വതന്ത്രനാകാന്‍ ഒരു അടിമക്ക് സാധിച്ചില്ലെന്നിരിക്കട്ടെ.  അപ്പോഴും അവന് മോചനം അസാ ധ്യമല്ല. സ്വയം മോചനമാഗ്രഹിക്കുന്ന ഏതൊരു അടിമക്കും മോചിതനാകുവാനുള്ള മാര്‍ഗം ഇസ്‌ലാം തുറന്നുകൊടുത്തിട്ടുണ്ട്. 'മുകാതബ' യെന്ന് സാങ്കേതികമായി വിളിക്കുന്ന മോചനപത്രത്തിലൂടെയാണ് ഇത് സാധ്യമാവുക. സ്വാതന്ത്ര്യമെന്ന അഭിലാഷം ഹൃദയത്തിനകത്ത് മൊട്ടിട്ടു കഴിഞ്ഞാല്‍ 'മുകാതബ'യിലൂടെ ഏതൊരു അടിമക്കും സ്വതന്ത്രനാകാവുന്നതാണ്. അടിമയും ഉടമയും യോജിച്ച് ഒരു മോചനമൂ ല്യവും അത് അടച്ചുതീര്‍ക്കേണ്ട സമയവും തീരുമാനിക്കുന്നു. ഈ മോചനമൂല്യം സമാഹരിക്കുന്നതിനുവേണ്ടി അടിമയ്ക്ക് പുറത്തുപോ യി ജോലി ചെയ്യാം. അങ്ങനെ ഗഡുക്കളായി അടിമ മോചനദ്രവ്യം അടച്ചുതീര്‍ക്കുന്നു. അത് അടച്ചു തീര്‍ക്കുന്നതോടെ അയാള്‍ സ്വതന്ത്ര നാവുന്നു.

സ്വാതന്ത്ര്യമെന്ന സ്വപ്‌നം പൂവണിയുന്നതിനായി ആ ആഗ്രഹം മനസ്സില്‍ മൊട്ടിട്ടുകഴിഞ്ഞ ഏതൊരു അടിമക്കും അവസരമുമുണ്ടാക്കി കൊടുക്കുകയാണ് ഇൗ സംവിധാനത്തിലൂടെ ഇസ്‌ലാം ചെയ്തിരിക്കുന്നത്. മോചനപത്രമെഴുതിയ ഒരു അടിമക്ക് നിശ്ചിത സമയത്തിനകം മോചനമൂല്യം അടച്ചുതീര്‍ക്കാന്‍ കഴിഞ്ഞില്ലെങ്കിലോ? അതിനുള്ള സംവിധാനവും ഇസ്‌ലാം നിര്‍ദേശിക്കുന്നുണ്ട്. സകാത്ത് ധനം ചെലവഴി ക്കപ്പെടേണ്ട എട്ടു വകുപ്പുകളിലൊന്ന് അടിമമോചനമാണ് (ഖുര്‍ആന്‍ 9:60). മുകാതബ പ്രകാരമുള്ള മോചനദ്രവ്യം കൊടുത്തുതീര്‍ക്കാന്‍ ഒരു അടിമക്ക് കഴിയാത്ത സാഹചര്യങ്ങളില്‍ അയാള്‍ക്ക് ബൈത്തുല്‍മാലിനെ (പൊതുഖജനാവ്) സമീപിക്കാം. അതില്‍നിന്ന് നിശ്ചിത സംഖ്യയടച്ച് അയാളെ മോചിപ്പിക്കേണ്ടത് അതു കൈകാര്യം ചെയ്യുന്നവരുടെ കടമയാണ്. പണക്കാരന്‍ നല്‍കുന്ന സ്വത്തില്‍ നിന്നുതന്നെ അടിമയെ മോചിപ്പിക്കുവാനുള്ള വക കണ്ടെത്തുകയാണ് ഇസ്‌ലാം ഇവിടെ ചെയ്തിരിക്കുന്നത്.

അടിമകളെ സ്വാതന്ത്ര്യമെന്താണെന്ന് പഠിപ്പിക്കുകയും പാരതന്ത്ര്യത്തില്‍നിന്ന് മോചിതരാകുവാന്‍  അവരെ സ്വയം സന്നദ്ധരാക്കുകയും ചെയ്തുകൊണ്ട് ചങ്ങലക്കെട്ടുകളില്‍നിന്ന് മുക്തമാക്കുകയെന്ന  പ്രായോഗികമായ നടപടിക്രമമാണ് ഇസ്‌ലാം അടിമത്തത്തിന്റെ കാര്യ ത്തില്‍ സ്വീകരിച്ചത്. അക്കാര്യത്തില്‍ ഇസ്‌ലാം സ്വീകരിച്ചതിനേക്കാള്‍ ഉത്തമമായ മാര്‍ഗമിതായിരുന്നുവെന്ന് ചൂണ്ടിക്കാണിക്കുവാന്‍ പറ്റിയ ഒരു മാര്‍ഗവും നിര്‍ദേശിക്കുവാന്‍ ആര്‍ക്കും കഴിയില്ലെന്നതാണ് വാസ്തവം. അത് യഥാര്‍ഥത്തില്‍ ഉള്‍ക്കൊള്ളണമെങ്കില്‍ അടിമ ത്തം ഒരു സ്ഥാപനമായി നിലനിന്നിരുന്ന സമൂഹത്തിന്റെ ഭൂമികയില്‍നിന്നുകൊണ്ട് പ്രശ്‌നത്തെ നോക്കിക്കാണണമെന്നുമാത്രം.

നമ്മുടെ സമൂഹത്തില്‍ ഇന്ന് നിലവിലില്ലാത്ത ഒരു സമ്പ്രദായമാണ് അടിമത്തം. ഇന്നത്തെ ചുറ്റുപാടുകളിലിരുന്നുകൊണ്ട് പ്രസ്തുത ഭൂതകാ ലപ്രതിഭാസത്തെ അപഗ്രഥിക്കുമ്പോള്‍ അതിന്റെ വേരുകളെയും അതു നിലനിന്നിരുന്ന സമൂഹങ്ങളില്‍ അതിനുണ്ടായിരുന്ന സ്വാധീനത്തെ യും കുറിച്ച് വ്യക്തമായി മനസ്സിലാക്കേണ്ടതുണ്ട്. വര്‍ത്തമാനത്തിന്റെ ഭൂമികയില്‍നിന്നുകൊണ്ട് ഭൂതകാലപ്രതിഭാസങ്ങളെ അപഗ്രഥിച്ച് കേവല നിഗമനത്തിലെത്തുവാന്‍ കഴിയില്ല. അടിമത്തമെന്നാല്‍ എന്താണെന്നും പുരാതന സമൂഹങ്ങളില്‍ അത് ചെലുത്തിയ സ്വാധീനമെന്താ യിരുന്നുവെന്നും മനസ്സിലാക്കുമ്പോഴേ അതിനെ ഇസ്‌ലാം സമീപിച്ച രീതിയുടെ മഹത്വം നമുക്ക് ബോധ്യമാകൂ.

ഒരു വ്യക്തി മറ്റൊരാളുടെ സമ്പൂര്‍ണമായ അധികാരത്തിന് വിധേയമായിത്തീരുന്ന സ്ഥിതിക്കാണ് അടിമത്തമെന്ന് പറയുന്നത്. ശരീരവും ജീവനും കുടുംബവും അങ്ങനെ തനിക്ക് എന്തൊക്കെ സ്വന്തമായുണ്ടോ അതെല്ലാം മറ്റൊരാള്‍ക്ക് അധീനമാക്കപ്പെട്ട രീതിയില്‍ ജീവിതം നയിക്കുന്നവനാണ് അടിമ. അവന്‍ ഉടമയുടെ ജംഗമസ്വത്താണ്. ഉടമ ഒരു വ്യക്തിയോ സമൂഹമോ രാഷ്ട്രമോ ആകാം. ആരായിരുന്നാലും അയാള്‍ക്ക് നല്‍കുന്ന അവകാശങ്ങള്‍ മാത്രം അനുഭവിച്ച്  ഉടമക്കുകീഴില്‍ ജീവിക്കാന്‍ വിധിക്കപ്പെട്ടവനാണ് അടിമ.

അടിമസമ്പ്രദായത്തിന്റെ ഉല്‍പത്തി എങ്ങനെയാണെന്നോ, അത് എവിടെ, എന്നാണ് തുടങ്ങിയതെന്നോ ഉറപ്പിച്ച് പറയാന്‍ പറ്റിയ രേഖക ളൊന്നും ഉപലബ്ധമല്ല. ക്രിസ്തുവിന് 20 നൂറ്റാണ്ടുകള്‍ക്കുമുമ്പുതന്നെ ഈ സമ്പ്രദായം ആരംഭിച്ചുകഴിഞ്ഞിരുന്നുവെന്ന് ഉറപ്പാണ്. ബി.സി 2050-നോടടുത്ത് നിലനിന്നിരുന്ന ഉര്‍നാമു (Ur Namu) നിയമസംഹിതയില്‍ അടിമകളെക്കുറിച്ച് പരാമര്‍ശിക്കപ്പെട്ടിട്ടുണ്ട്. യുദ്ധത്തില്‍ തടവു കാരായി പിടിക്കപ്പെടുന്നവര്‍ക്ക് സ്വാതന്ത്ര്യം നിഷേധിച്ചുകൊണ്ട് കൂലിയില്ലാതെ ജോലിചെയ്യിക്കുന്ന പതിവില്‍ നിന്നാവണം അടിമത്തം നിലവില്‍ വന്നതെന്നാണ് അനുമാനം. പുരാതന സുമേറിയന്‍ ഭാഷയില്‍ അടിമകളെ വിളിക്കാന്‍ ഉപയോഗിച്ചിരുന്ന പദങ്ങളാണ് ഈ അനു മാനത്തിന് നിദാനം. പുരുഷഅടിമക്ക് നിദാ-കൂര്‍ എന്നും സ്ത്രീ അടിമക്ക് മുനുസ്-കൂര്‍ എന്നുമായിരുന്നു പേര്‍. വിദേശിയായ പുരുഷന്‍, വിദേശിയായ സ്ത്രീ എന്നിങ്ങനെയാണ് യഥാക്രമം ഈ പദങ്ങളുടെ അര്‍ഥം. യുദ്ധത്തടവുകാരെ കൊണ്ടുവന്നിരുന്നത് വിദേശത്തുനിന്നായി രുന്നതിനാല്‍ അവരെ അടിമകളാക്കിയപ്പോള്‍ ഈ പേരുകള്‍ വിളിക്കപ്പെട്ടുവെന്നാണ് ഊഹിക്കപ്പെടുന്നത്.

ലോകത്ത് ഏകദേശം എല്ലാ പ്രദേശങ്ങളിലും ഒരു രൂപത്തിലല്ലെങ്കില്‍ മറ്റൊരു രൂപത്തില്‍ അടിമത്തം നിലനിന്നിരുന്നു. പുരാതന ഇസ്രാ യേല്‍ സമുദായത്തിന്റെ കഥ പറയുന്ന പഴയ നിയമ ബൈബിളില്‍ അടിമത്തത്തെക്കുറിച്ച് ഒരുപാട് പരാമര്‍ശങ്ങളുണ്ട്. പൗരാണിക പ്രവാ ചകനായിരുന്ന അബ്രഹാമിന്റെ കാലത്തുതന്നെ മനുഷ്യരെ വിലയ്ക്കു വാങ്ങുന്ന സമ്പ്രദായം നിലനിന്നിരുന്നുവെന്ന് കാണാന്‍ കഴിയും (ഉല്‍പത്തി 17:13, 14). യുദ്ധത്തടവുകാരെ അടിമകളാക്കണമെന്നാണ് ബൈബിളിന്റെ അനുശാസന (ആവര്‍ത്തനം 20:10, 11). അടിമയെ യഥേ ഷ്ടം മര്‍ദിക്കുവാന്‍ യജമാനന് സ്വാതന്ത്ര്യം നല്‍കുന്ന ബൈബിള്‍ പക്ഷേ, പ്രസ്തുത മര്‍ദനങ്ങള്‍ക്കിടയില്‍ അടിമ മരിക്കാനിടയാകരുതെന്ന് പ്രത്യേകം നിഷ്‌കര്‍ഷിക്കുന്നുണ്ട്. 'ഒരുവന്‍ തന്റെ ആണ്‍ അടിമയെയോ പെണ്‍അടിമയെയോ വടി കൊണ്ടടിക്കുകയും അയാളുടെ കൈയാ ല്‍ അടിമ മരിക്കുകയും ചെയ്താല്‍ അയാളെ ശിക്ഷിക്കണം. പക്ഷേ, അടിമ ഒന്നോ രണ്ടോ ദിവസം ജീവിച്ചാല്‍ അയാളെ ശിക്ഷിക്കരുത്. കാര ണം അടിമ അയാളുടെ സ്വത്താണ്' (പുറപ്പാട്21:20, 21) എന്നതായിരുന്നു ഇവ്വിഷയകമായി ഇസ്രായേല്‍ സമുദായത്തില്‍ നിലനിന്നിരുന്ന നിയമം.

യേശുക്രിസ്തുവിന്റെ കാലത്തും ശേഷവുമെല്ലാം അടിമസമ്പ്രദായം നിലനിന്നിരുന്നു. അടിമകളോടു സ്വീകരിക്കേണ്ട നിലപാടുകളെ സംബ ന്ധിച്ച ഉപദേശങ്ങളൊന്നും യേശുവിന്റെ വചനങ്ങളിലില്ല. 'കര്‍ത്താവിന്റെ വിളി ലഭിച്ചുകഴിഞ്ഞ അടിമകള്‍ ആത്മാര്‍ഥമായി യജമാന ന്മാരെ സേവിക്കണം' (എേഫ 6:5-9). 'അടിമകളേ, നിങ്ങളുടെ ലൗകിക യജമാനന്മാരെ എല്ലാ കാര്യങ്ങളിലും അനുസരിക്കുക; യജനമാനന്മാര്‍ കാണ്‍കെ, അവരെ പ്രീണിപ്പിക്കാന്‍ വേണ്ടി മാത്രമല്ല, ആത്മാര്‍ഥതയോടുകൂടി കര്‍ത്താവിനെ ഭയപ്പെട്ട് യജമാനന്മാരെ അനുസരിക്കുക' (കൊളോ 3:22) എന്നുമുള്ള പരാമര്‍ശങ്ങള്‍ ഒഴിച്ചാല്‍ അടിമത്തവുമായി ബന്ധപ്പെട്ട മറ്റു പ്രധാന നിര്‍ദേശങ്ങളൊന്നും പൗലോസിന്റെ ലേഖനങ്ങളിലും കാണാന്‍ കഴിയുന്നില്ല. ഗ്രീക്കോ-റോമന്‍ നാഗരികതയില്‍ നിലനിന്നിരുന്ന അതിക്രൂരമായ അടിമത്ത സമ്പ്രദായം അവയുടെ ക്രൈസ്തവവത്കരണത്തിനു ശേഷവും മാറ്റമൊന്നുമില്ലാതെ നിലനിന്നിരുന്നുവെന്ന് കാണാനാവും. അടിമവ്യവസ്ഥിതിയുടെ ക്രൂരവും നികൃഷ്ടവുമായ കഥകള്‍ ഏറെ പറയാനുള്ള റോമാസംസ്‌കാരത്തിന്റെ ഔദ്യോഗിക മതം ക്രൈസ്തവതയായിരുന്നുവെന്ന വസ്തുത പ്രത്യേകം പരാമര്‍ശമര്‍ഹിക്കുന്നു.

ഇന്ത്യയില്‍ അടിമത്തം നിലനിന്നത് മതത്തിന്റെ ഭാഗമായിക്കൊണ്ടാണ്. ൈവദികമതത്തിന്റെ അവിഭാജ്യഘടകമാണ് വര്‍ണാശ്രമ വ്യവ സ്ഥ. ശ്രുതികളില്‍ പ്രഥമഗണനീയമായി പരിഗണിക്കപ്പെടുന്ന വേദസംഹിതകളില്‍ ഒന്നാമതായി വ്യവഹരിക്കപ്പെടുന്ന ഋഗ്വേദത്തിലെ പുരു ഷസൂക്തത്തിലാണ് (10:90:12) ജാതിവ്യവസ്ഥയുടെ ബീജങ്ങള്‍ നമുക്ക് കാണാന്‍ കഴിയുന്നത്. 'പരമപുരുഷന്റെ ശിരസ്സില്‍നിന്ന് ബ്രാഹ്മണ നും കൈകളില്‍നിന്ന് വൈശ്യനും പാദങ്ങളില്‍ നിന്ന് ശൂദ്രനും സൃഷ്ടിക്കപ്പെട്ടുവെന്ന ഋഗ്വേദ പരാമര്‍ശത്തിന്റെ അടിസ്ഥാനത്തില്‍ നിലവി ല്‍വന്ന വര്‍ണാശ്രമ വ്യവസ്ഥ ബ്രാഹ്മണനെ ഉത്തമനും ശൂദ്രനെ അധമനുമായി കണക്കാക്കിയത് സ്വാഭാവികമായിരുന്നു. എല്ലാ ഹൈന്ദവഗ്ര ന്ഥങ്ങളും ചാതുര്‍വര്‍ണ്യ വ്യവസ്ഥയെ ന്യായീകരിക്കുകയാണ് ചെയ്തിട്ടുള്ളത്. ആധുനിക ഹൈന്ദവതയുടെ ശ്രുതിഗ്രന്ഥമായി അറിയപ്പെ ടുന്ന ഭഗവത്ഗീത 'ചാതുര്‍വര്‍ണ്യം മയാസൃഷ്ടം ഗുണ കര്‍മ വിഭാഗശഃ' (4:13) എന്നാണ് പറഞ്ഞിരിക്കുന്നത്. 'ഗുണകര്‍മങ്ങളുടെ വിഭാഗത്തി നനുസരിച്ച് നാലു വര്‍ണങ്ങളെ സൃഷ്ടിച്ചിരിക്കുന്നത് ഞാന്‍ തന്നെയാണ്’എന്നര്‍ഥം.

ദൈവത്തിന്റെ പാദങ്ങളില്‍നിന്ന് പടക്കപ്പെട്ടവര്‍ പാദസേവ ചെയ്യുവാനായി വിധിക്കപ്പെട്ടത് സ്വാഭാവികമായിരുന്നു. ദാസ്യവേലക്കു വേണ്ടി പ്രത്യേകമായി പടക്കപ്പെട്ടവരാണ് ശൂദ്രരെന്നായിരുന്നു വിശ്വാസം. മുജ്ജന്മപാപത്തിന്റെ ശിക്ഷയായാണ് അവര്‍ണനായി ജനിക്കേ ണ്ടിവന്നതെന്നും അടുത്ത ജന്മത്തിലെങ്കിലും പാപമോചനം ലഭിച്ച് സവര്‍ണനായി ജനിക്കണമെങ്കില്‍ ഈ ജീവിതം മുഴുവന്‍ സവര്‍ണരുടെ പാദസേവ ചെയ്തത് അവരെ സംതൃപ്തരാക്കുകയാണ് വേണ്ടതെന്നുമാണ് അവരെ മതഗ്രന്ഥങ്ങള്‍ പഠിപ്പിച്ചത്. അടിമകളായി ജനിക്കാന്‍ വിധിക്കപ്പെട്ട ചണ്ഡാളന്മാരെ പന്നികളോടും പട്ടികളോടുമൊപ്പമാണ് ഛന്ദോഗ്യോപനിഷത്ത് (5:10:7) പരിഗണിച്ചിരിക്കുന്നത്. അവരോടു ള്ള പെരുമാറ്റ രീതിയും ഈ മൃഗങ്ങളോടുള്ളതിനേക്കാള്‍ നീചവും നികൃഷ്ടവുമായിരുന്നുവെന്ന് മനുസ്മൃതിയും പരാശരസ്മൃതിയു മെല്ലാം വായിച്ചാല്‍ മനസ്സിലാകും.

ജന്മത്തിന്റെ പേരില്‍ അടിമത്തം വിധിക്കപ്പെടുന്ന സമ്പ്രദായത്തോടൊപ്പംതന്നെ ഇന്ത്യയില്‍ അടിമവ്യാപാരവും അതിന്റെ സകലവിധ ക്രൂരഭാവങ്ങളോടുംകൂടി നിലനിന്നിരുന്നതായി കാണാനാവും. തമിഴ്‌നാട്ടില്‍നിന്ന് ലഭിച്ച ശിലാലിഖിതങ്ങളില്‍നിന്ന് ചോളകാലത്തും ശേഷവും ക്ഷേത്രങ്ങളോട് ബന്ധപ്പെട്ടുകൊണ്ട് അടിമവ്യാപാരം നിലനിന്നിരുന്നുവെന്ന് മനസ്സിലാകുന്നുണ്ട്. മൈസൂരിലും ബീഹാറിലും കേരളത്തിലുമെല്ലാം അടിമവ്യാപാരം നിലനിന്നിരുന്നു. ഇംഗ്ലീഷ് ഈസ്റ്റ് ഇന്ത്യാ കമ്പനിയുടെ ആഗമനത്തോടെ ഇന്ത്യയില്‍ നിന്ന് ആളുകളെ പിടിച്ച് അടിമകളാക്കി വിദേശങ്ങളിലെത്തിക്കുന്ന സമ്പ്രദായവും നിലവില്‍ വന്നു. ട്രാന്‍ക്യൂബാറിലെ ഒരു ഇറ്റാലിയന്‍ പുരോഹിതന്‍, മധുരക്കാരനായ ഒരു ക്രിസ്ത്യാനിയുടെ ഭാര്യയെയും നാലു മക്കളെയും മുപ്പത് 'പഗോഡ'ക്ക് മനിലയിലേക്ക് പോകുന്ന ഒരു കപ്പലിലെ കപ്പിത്താന് വിറ്റതായി ചില ചരിത്രരേഖകളിലുണ്ട് (സര്‍വവിജ്ഞാനകോശം. വാല്യം1, പുറം 258). 1841-ലെ ഒരു സര്‍വേ പ്രകാരം അന്ന് ഇന്ത്യയില്‍ എണ്‍പത് ലക്ഷത്തിനും തൊണ്ണൂറു ലക്ഷത്തിനുമിടയില്‍ അടിമകളുണ്ടായിരുന്നു. മലബാറിലായിരുന്നു ഇന്ത്യയിലെ അടിമക ളുടെ നല്ലൊരു ശതമാനമുണ്ടായിരുന്നത്. അവിടത്തെ ആകെ ജനസംഖ്യയില്‍ 15 ശതമാനം അടിമകളായിരുന്നുവത്രേ! (Encyclopaedia Britanica Vol 27, page 289).

റോമാ സാമ്രാജ്യത്തില്‍ ക്രിസ്തുവിന് മുമ്പ് രണ്ടാം നൂറ്റാണ്ടു മുതല്‍ നീണ്ട ആറു ശതാബ്ദക്കാലം നിലനിന്ന അടിമവ്യവസ്ഥയായിരുന്നു ചരിത്രത്തിലെ ഏറ്റവും ക്രൂരമായ അടിമ സമ്പ്രദായമെന്നാണ് മനസ്സിലാവുന്നത്. ഏതെങ്കിലും രീതിയിലുള്ള യാതൊരു അവകാശവുമി ല്ലാത്ത വെറും കച്ചവടച്ചരക്കായിരുന്നു റോമാ സാമ്രാജ്യത്തിലെ അടിമ. ഉടമയെ രസിപ്പിക്കുന്നതിനുവേണ്ടി മറ്റൊരു അടിമയുമായി ദ്വന്ദയുദ്ധത്തിലേര്‍പ്പെട്ട് മരിച്ചു വീഴാന്‍ മാത്രം വിധിക്കപ്പെട്ടവനായിരുന്നു അവന്‍. അടിമകളുടെ ശരീരത്തില്‍നിന്ന് ദ്വന്ദയുദ്ധക്കളരിയില്‍ ഉറ്റിവീഴുന്ന രക്തത്തിന്റെ അളവ് വര്‍ധിക്കുമ്പോള്‍ യജമാനന്മാര്‍ 'ഹുറേ' വിളികളുമായി അവരെ പ്രോല്‍സാഹിപ്പിക്കുകയും ചാട്ടവാര്‍ ചുഴറ്റിക്കൊണ്ട് അവരെ ഭീതിപ്പെടുത്തുകയും ചെയ്യുകയായിരുന്നു പതിവ്! അടിമത്തത്തിന്റെ അതിക്രൂരമായ രൂപം!

കൊളംബസിന്റെ അമേരിക്ക കണ്ടുപിടിത്തമാണ് ആധുനിക ലോകത്ത് അടിമവ്യാപാരത്തെ ആഗോളവ്യാപകമാക്കിത്തീര്‍ത്തത്. നീഗ്രോ കള്‍ അടിമകളാക്കപ്പെടുകയും കമ്പോളങ്ങളില്‍ വെച്ച് കച്ചവടം ചെയ്യപ്പെടുകയുമുണ്ടായി. ഒരു സ്പാനിഷ് ബിഷപ്പായിരുന്ന ബാര്‍തലോച ദെ ലാസ്‌കാസാസ് ആയിരുന്നു അമേരിക്കന്‍ അടിമത്തൊഴില്‍ വ്യവസ്ഥക്ക് തുടക്കം കുറിച്ചത്. അടിമവ്യാപാരത്തിനായി മാത്രം രജിസ്റ്റര്‍ ചെയ്യപ്പെട്ട കമ്പനികള്‍ യൂറോപ്പിലുണ്ടായിരുന്നു. ആഫ്രിക്കന്‍ തീരപ്രദേശങ്ങളില്‍നിന്ന് അടിമകളെക്കൊണ്ടുവന്ന് അമേരിക്കയില്‍ വില്‍ക്കു കയായിരുന്നു ഈ കമ്പനികളുടെ വ്യാപാരം. പതിനേഴ് മുതല്‍ പത്തൊമ്പത് വരെ നൂറ്റാണ്ടുകള്‍ക്കിടയ്ക്ക് അമേരിക്കയില്‍ ഇങ്ങനെ ഒന്നര ക്കോടിയോളം അടിമകള്‍ ഇറക്കുമതി ചെയ്യപ്പെട്ടിട്ടുണ്ടെന്നാണ് കണക്ക്. മണിബന്ധത്തിലും കണങ്കാലിലും രണ്ട് അടിമകളെ വീതം കൂട്ടിക്കെട്ടിയായിരുന്നു കപ്പലില്‍ കൊണ്ടുപോയിരുന്നത്. അത്‌ലാന്റിക് സമുദ്രം തരണം ചെയ്യുന്നതിനിടക്ക് നല്ലൊരു ശതമാനം അടിമകള്‍ മരിച്ചുപോകുമായിരുന്നു. ഇങ്ങനെ മരണമടഞ്ഞവരുടെ എണ്ണമെത്രയെന്നതിന് യാതൊരു രേഖകളുമില്ല. അടിമയുടെ ജീവന് എന്തു വില?!

അടിമത്തത്തെക്കുറിച്ച് ചര്‍ച്ച ചെയ്യുന്നതിന്റെ മുന്നോടിയായാണ് പ്രസ്തുത സമ്പ്രദായത്തിന്റെ ഉല്‍പത്തിയെയും ചരിത്രത്തെയും കുറിച്ച് സംക്ഷിപ്തമായി പ്രതിപാദിച്ചത്. നൂറ്റാണ്ടുകളായി നിലനിന്നിരുന്ന ഒരു വ്യവസ്ഥയെ ഒരു പ്രഖ്യാപനത്തിലൂടെ തുടച്ചു നീക്കുക യെന്ന അപ്രായോഗികവും അശാസ്ത്രീയവുമായ നിലപാടിനുപകരം പ്രായോഗികമായി അടിമത്തം ഇല്ലാതാക്കുന്നതിനുവേണ്ടിയുള്ള നടപടികളായിരുന്നു അതു നിലനിന്നിരുന്ന സമൂഹങ്ങളില്‍ ആവശ്യമായിരുന്നത്. ഇതു മനസ്സിലാകാതെ അടിമത്തത്തെ ഇസ്‌ലാം സമീപിച്ച രീതിയുടെ മാനവികത മനസ്സിലാക്കാന്‍ കഴിയില്ല.

ഇസ്‌ലാമിക സമീപനത്തിന്റെ മാനവികത

അടിമത്തം ഒരു സ്ഥാപനമായി നിലനിന്നിരുന്ന സമൂഹത്തില്‍ അതിന്റെ കെടുതികള്‍ ഇല്ലാതെയാക്കാന്‍ എന്താണ് ചെയ്യാനാവുകയെന്ന് പഠിക്കുവാന്‍ അടിമയുടെയും ഉടമയുടെയും മനഃശാസ്ത്രമെന്താണെന്ന് മനസ്സിലാക്കേണ്ടതുണ്ട്. അടിമയുടെ മാനസികഘടനയും സ്വതന്ത്ര ന്റെ മാനസികഘടനയും തമ്മില്‍ വളരെയേറെ വ്യത്യാസങ്ങളുണ്ട്. നിരന്തരമായ അടിമജീവിതം അടിമയുടെ മനോനിലയെ രൂപപ്പെടുത്തു ന്നതില്‍ കാര്യമായ സ്വാധീനം ചെലുത്തുന്നുണ്ട്. തലമുറകളായി കൈമാറ്റം ചെയ്യപ്പെടുന്ന അടിമത്തനുകം ചുമലില്‍ വഹിക്കുന്നതുകൊണ്ട് അവന്റെ മനസ്സില്‍ അനുസരണത്തിന്റെയും കീഴ്‌വഴക്കത്തിന്റെയും ശീലങ്ങള്‍ ആഴത്തില്‍ വേരൂന്നിയിട്ടുണ്ടാവും. ചുമതലകള്‍ ഏറ്റെടുക്കുവാനോ ഉത്തരവാദിത്തങ്ങള്‍ വഹിക്കുവാനോ അവന് കഴിയില്ല. ഉടമയുടെ  കല്‍പന ശിരസാവഹിക്കാന്‍ അവന്റെ മനസ്സ് സദാ സന്നദ്ധമാണ്.  അയാളുടെ ഇച്ഛക്കനുസരിച്ച് കാര്യങ്ങളെല്ലാം നിര്‍വഹിക്കാന്‍ അടിമക്ക് നന്നായറിയാം. എന്നാല്‍, അനുസരിക്കാനും നടപ്പാക്കാനും മാത്രമാണ് അവനു സാധിക്കുക. ഉത്തരവാദിത്തമേറ്റെടുക്കുവാന്‍  അവന്റെ മനസ്സ് അശക്തമായിരിക്കും. ഭാരം താങ്ങു വാന്‍ അവന്റെ മനസ്സിന് കഴിയില്ല. ചുമതലകള്‍ ഏറ്റെടുക്കുന്നതില്‍നിന്ന് ഓടിയകലാനേ അവനു സാധിക്കൂ. എന്നാല്‍ യജമാനന്‍ എന്തു കല്‍പിച്ചാലും അതു ശിരസാവഹിക്കാന്‍ അവന്‍ സദാ സന്നദ്ധനുമായിരിക്കും.

അടിമയുടെയും ഉടമയുടെയും മാനസികാവസ്ഥകള്‍ രണ്ട് വിരുദ്ധധ്രുവങ്ങളില്‍ സ്ഥിതി ചെയ്യുന്നവയാണ്. ഒന്ന് അഹങ്കാരത്തിന്‍േറതാ ണെങ്കില്‍ മറ്റേത് അധമത്വത്തിന്‍േറതാണ്. വിരുദ്ധധ്രുവങ്ങളില്‍ സ്ഥിതി ചെയ്യുന്ന ഈ മാനസികാവസ്ഥകളെ ഒരേ വിതാനത്തിലേക്ക് കൊണ്ടുവരാതെ അടിമമോചനം യഥാര്‍ഥത്തിലുള്ള മോചനത്തിനുതകുകയില്ലെന്നതിന് ഏറ്റവും നല്ല ഉദാഹരണം അമേരിക്കയുടേതുത ന്നെയാണ്. എബ്രഹാം ലിങ്കന്റെ പ്രവര്‍ത്തനങ്ങള്‍ വഴി, ഒരൊറ്റ പ്രഖ്യാപനത്തിലൂടെ അമേരിക്കയിലെ അടിമകള്‍ക്ക് മോചനം ലഭിക്കുക യായിരുന്നു, ഒരു പ്രഭാതത്തില്‍! പക്ഷേ, എന്താണവിടെ സംഭവിച്ചത്? നിയമംമൂലം സ്വാതന്ത്ര്യം ലഭിച്ച അടിമകള്‍ക്ക് പക്ഷേ, സ്വാതന്ത്ര്യത്തി ന്റെ 'ഭാരം' താങ്ങുവാന്‍ കഴിഞ്ഞില്ല. എന്തു ചെയ്യണമെന്നറിയാതെ അവര്‍ ചുറ്റുപാടും നോക്കി. ആരും കല്‍പിക്കാനില്ലാത്തതുകൊണ്ട് അവര്‍ക്ക് ഒന്നും ചെയ്യുവാനായില്ല. അവര്‍ തിരിച്ച് യജമാനന്മാരുടെ അടുത്തുചെന്ന് തങ്ങളെ അടിമകളായിത്തന്നെ സ്വീകരിക്കണമെന്ന പേക്ഷിച്ചു. മാനസികമായി സ്വതന്ത്രരായി കഴിയാത്തവരെ ശാരീരികമായി സ്വതന്ത്രരാക്കുന്നത് വ്യര്‍ഥമാണെന്ന വസ്തുതയാണ് ഇവിടെ അനാവൃതമാവുന്നത്.

മനുഷ്യരുടെ ശരീരത്തെയും മനസ്സിനെയും പറ്റി ശരിക്കറിയാവുന്ന ദൈവത്തില്‍നിന്ന് അവതീര്‍ണമായ ഖുര്‍ആനും അതിന്റെ പ്രായോഗിക വിശദീകരണമായ മുഹമ്മദ് നബി (സ)യുടെ ജീവിതവും ഇക്കാര്യത്തില്‍ തികച്ചും പ്രായോഗികമായ നടപടിക്രമത്തിനാണ് രൂപം നല്‍കിയി ട്ടുള്ളത്. അറേബ്യന്‍ സമ്പദ്ഘടനയുടെ സ്തംഭങ്ങളിലൊന്നായിരുന്നു അടിമവ്യവസ്ഥിതി. ഒരു കേവല നിരോധത്തിലൂടെ പിഴുതെറിയു വാന്‍ സാധിക്കുന്നതിലും എത്രയോ ആഴത്തിലുള്ളവയായിരുന്നു അതിന്റെ വേരുകള്‍. ഇസ്‌ലാം പ്രചരിപ്പിക്കപ്പെട്ട പ്രദേശങ്ങളിലും അല്ലാത്തയിടങ്ങളിലുമെല്ലാം നിലനിന്നിരുന്ന ഒരു വ്യവസ്ഥയെന്ന നിലയ്ക്ക് അതു നിരോധിക്കുന്നത് പ്രായോഗികമായി പ്രയാസകരമായി രിക്കുമെന്നു മാത്രമല്ല, അത്തരമൊരു നടപടി ഗുണത്തേക്കാളധികം ദോഷമാണ് ചെയ്യുകയെന്നുള്ളതാണ് സത്യം. അതുകൊണ്ടുതന്നെ മനുഷ്യ സമൂഹത്തെക്കുറിച്ച് കൃത്യമായി അറിയാവുന്ന അല്ലാഹു അടിമത്തത്തെ പാടെ നിരോധിക്കുന്ന ഒരു നിയമം കൊണ്ടു വരികയല്ല. പ്രത്യുത, അതിന്റെ കെടുതികള്‍ ഇല്ലാതാക്കുവാനുള്ള പ്രായോഗികമായ നടപടികള്‍ സ്വീകരിക്കുകയാണ് ചെയ്തത്.

രണ്ടു വിരുദ്ധ തീവ്രമാനസിക നിലകളില്‍ സ്ഥിതി ചെയ്യുന്നവരെ ഒരേ വിതാനത്തിലേക്ക് കൊണ്ടുവരികയാണ് ഇസ്‌ലാം ആദ്യമായി ചെ യ്തത്. അടിമയെയും ഉടമയെയും  സംസ്‌കരിക്കുയെന്ന പദ്ധതിയാണ് ഖുര്‍ആന്‍ മുന്നോട്ടുവെച്ചത്. പിന്നെ, സ്വാതന്ത്ര്യം ദാനമായും അധ്വാ നിച്ചും നേടിയെടുക്കുവാനാവശ്യമായ വഴികളെല്ലാം തുറന്നുവെക്കുകയും ചെയ്തു. അടിമയെയും  ഉടമയെയും സമാനമായ മാനസിക നിലവാരത്തിലെത്തിച്ചുകൊണ്ട് സ്വാതന്ത്ര്യം നേടുവാനുള്ള വഴികള്‍ തുറക്കുകയും അതു ലഭിച്ചുകഴിഞ്ഞാല്‍ അതു സംരക്ഷിക്കുവാന്‍ അവനെ പ്രാപ്തനാക്കുകയുമാണ് ഖുര്‍ആന്‍ ചെയ്തത്. അതുമാത്രമായിരുന്നു അക്കാര്യത്തില്‍ പ്രായോഗികമായിരുന്നത്.

അടിമവ്യവസ്ഥിതിയിലെ തിന്‍മകള്‍ ഇല്ലാതാക്കുവാനും കെടുതികള്‍ കുറച്ചുകൊണ്ടുവരുവാനും ആവശ്യമായ പ്രായോഗികമായ നടപടി ക്രമങ്ങള്‍ സ്വീകരിച്ച ഇസ്‌ലാം പക്ഷേ, മദ്യമോ വ്യഭിചാരമോ നിരോധിച്ചതുപോലെ അടിമത്തത്തെ പാടെ നിരോധിച്ചുകൊണ്ടുള്ള ഉത്തര വുകളൊന്നും പുറപ്പെടുവിച്ചിട്ടില്ല. എന്തുകൊണ്ടാണിതെന്ന് ചോദിക്കുന്നവരുണ്ട്. ഇസ്‌ലാം ഏകപക്ഷീയമായി അടിമത്തം നിരോധിക്കുന്ന തുകൊണ്ട് ഗുണത്തിലേറെ ദോഷമാണുണ്ടാവുക. അടിമത്തത്തിന്റെ കെടുതികള്‍ അനുഭവിക്കേണ്ടി വരുന്നവരുടെ ഭൂമികയില്‍ നിന്നു കൊണ്ട് ചിന്തിച്ചാല്‍ ഇക്കാര്യം ബോധ്യപ്പെടും.

അടിമത്തത്തെ പാടെ നിരോധിക്കാത്ത ഇസ്‌ലാമിന്റെ നടപടി അതിന്റെ സര്‍വകാലികതയാണ് യഥാര്‍ത്ഥത്തില്‍ വ്യക്തമാക്കുന്നത്. മനുഷ്യ സമൂഹത്തിന്റെ ഗതിവിഗതികളെയും പരിണാമപ്രക്രിയയെയും കുറിച്ച് ശരിക്കറിയാവുന്ന ദൈവം തമ്പുരാനില്‍നിന്നുള്ളതാണ് ഖുര്‍ആ ന്‍ എന്ന വസ്തുതയാണ് ഈ വിഷയത്തിലെ അതിന്റെ നിലപാടില്‍നിന്നും നമുക്ക്  മനസ്സിലാക്കാന്‍ കഴിയുന്നത്. ഇസ്‌ലാം കാലാതിവര്‍ ത്തിയാണെന്നും അതിന്റെ നിര്‍ദേശങ്ങള്‍ എക്കാലത്തും പ്രായോഗികമാണെന്നുമുള്ള വസ്തുതകളാണ് അടിമത്തം പാടെ നിരോധിക്കാത്ത അതിന്റെ നടപടിയെക്കുറിച്ച് അവഗാഹമായി പഠിച്ചാല്‍ ബോധ്യപ്പെടുക.

അടിമത്ത വ്യവസ്ഥിതിയുടെ ആരംഭംതന്നെ യുദ്ധത്തടവുകാരില്‍നിന്നായിരുന്നുവല്ലോ. അടിമത്തത്തെ പാടെ നിരോധിച്ചുകൊണ്ട് ആധുനി ക രാഷ്ട്രങ്ങള്‍ നടത്തിയ പ്രഖ്യാപനങ്ങള്‍ക്കു മുമ്പ് യുദ്ധത്തടവുകാരെ അടിമകളാക്കി മാറ്റുന്ന സമ്പ്രദായമായിരുന്നു വ്യാപകമായി നിലനി ന്നിരുന്നത്. യുദ്ധത്തില്‍ ബന്ദികളായി പിടിക്കപ്പെടുന്നവരെ ഒന്നുകില്‍ കൊന്നുകളയുക, അല്ലെങ്കില്‍ അടിമകളാക്കുക. ഇതാണ് നടന്നിരുന്നത്. ഇവ മാത്രമായിരുന്നു പ്രായോഗികമായ മാര്‍ഗങ്ങള്‍. അതല്ലാതെ അവരെ തടവുകാരായി പാര്‍പ്പിക്കുവാനാവശ്യമായ സംവിധാനങ്ങളൊ ന്നും അന്നുണ്ടായിരുന്നില്ലല്ലോ.

യുദ്ധത്തില്‍ തടവുകാരായി പിടിക്കപ്പെടുന്നവരെ എന്തു ചെയ്യണം? ഇക്കാര്യത്തില്‍ ഖുര്‍ആന്‍ നല്‍കുന്ന നിര്‍ദേശമിങ്ങനെയാണ്: ''ആകയാ ല്‍ സത്യനിഷേധികളുമായി നിങ്ങള്‍ ഏറ്റുമുട്ടിയാല്‍ (നിങ്ങള്‍) പിരടികളില്‍ വെട്ടുക. അങ്ങനെ അവരെ നിങ്ങള്‍ അമര്‍ച്ച ചെയ്തുകഴിഞ്ഞാ ല്‍ നിങ്ങള്‍ അവരെ ശക്തിയായി ബന്ധിക്കുക. എന്നിട്ട് അതിനുശേഷം (അവരോട്) ദാക്ഷിണ്യം കാണിക്കുകയോ, അല്ലെങ്കില്‍ മോചനമൂല്യം വാങ്ങി വിട്ടയക്കുകയോ ചെയ്യുക. യുദ്ധം അതിന്റെ ഭാരങ്ങള്‍ ഇറക്കിവെക്കുന്നതുവരെയാണിത്. അതാണ് (യുദ്ധത്തിന്റെ) മുറ'' (47:4). ശത്രുക്കളെ യുദ്ധഭൂമിയില്‍ വെച്ച് വധിക്കുവാന്‍ അനുശാസിക്കുന്ന ഈ സൂക്തത്തില്‍ ബന്ധനസ്ഥരായവരെ പ്രതിഫലം വാങ്ങിയോ അല്ലാ തെയോ വിട്ടയക്കുവാനാണ് കല്‍പിച്ചിരിക്കുന്നത്. ഈ സൂക്തത്തിന്റെ വെളിച്ചത്തില്‍ പ്രവാചകാനുചരന്മാരില്‍ പ്രമുഖരെല്ലാം യുദ്ധത്തട വുകാരെ അനിവാര്യമായ അവസ്ഥകളിലല്ലാതെ വധിക്കാന്‍ പാടില്ലെന്ന് അഭിപ്രായപ്പെട്ടിട്ടുണ്ട്.

യുദ്ധത്തടവുകാരെ നാലു വിധത്തില്‍ കൈകാര്യം ചെയ്യുവാന്‍ പ്രവാചകന്‍ (സ) മാതൃക കാണിച്ചിട്ടുണ്ട്.

  1. വെറുതെ വിട്ടയക്കുക. അവരെ വിട്ടയക്കുന്നത് മുസ്‌ലിം സമൂഹത്തിന് ഹാനികരമല്ലെന്ന് ബോധ്യപ്പെടുന്ന അവസ്ഥയില്‍ യുദ്ധത്തടവു കാരെ വെറുതെ വിട്ടയക്കാവുന്നതാണ്.
  2. ശത്രുക്കള്‍ പിടിച്ചുവെച്ച മുസ്‌ലിം തടവുകാര്‍ക്കു പകരമായി അവരെ കൈമാറുക.
  3. പ്രതിഫലം വാങ്ങി തടവുകാരെ വിട്ടയക്കുക.
  4. അടിമകളാക്കി മാറ്റി മുസ്‌ലിം യോദ്ധാക്കള്‍ക്ക് ഭാഗിച്ച് നല്‍കുക.

പ്രവാചകന്‍ (സ) വിവിധ യുദ്ധങ്ങളില്‍ മുകളില്‍ പറഞ്ഞ വ്യത്യസ്ത മാര്‍ഗങ്ങള്‍ സ്വീകരിച്ചിരുന്നതായി കാണാം. ഇതില്‍ നാലാമത്തെ മാര്‍ഗമായ യുദ്ധത്തടവുകാരെ അടിമകളാക്കി മാറ്റുന്ന രീതി, മറ്റു മൂന്നു മാര്‍ഗങ്ങളും അപ്രായോഗികമായിത്തീരുന്ന അവസ്ഥകളിലാണ് സ്വീകരിച്ചിരുന്നത്. അടിമത്തം പൂര്‍ണമായി നിരോധിക്കപ്പെട്ടിരുന്നുവെങ്കില്‍ ഈ മാര്‍ഗം സ്വീകരിക്കുവാന്‍ മുസ്‌ലിം സമൂഹത്തിന് ഒരി ക്കലും സാധ്യമാകാത്ത അവസ്ഥ സംജാതമാകുമായിരുന്നു. അത്തരമൊരു അവസ്ഥ അടിമത്തം ഒരു സ്ഥാപനമായി നിലനിന്നിരുന്ന സാമൂ ഹിക സംവിധാനത്തില്‍ മുസ്‌ലിംകള്‍ക്ക് ഏറെ പ്രയാസങ്ങള്‍ സൃഷ്ടിക്കുമായിരുന്നു എന്നതാണ് വസ്തുത.

മുസ്‌ലിം സമൂഹവുമായി യുദ്ധം ചെയ്യുന്നവര്‍ അടിമത്തത്തെ ഒരു മാര്‍ഗമായി അംഗീകരിക്കുന്നവരും അടിമകളെ ലഭിക്കുക എന്നതുകൂടി ലക്ഷ്യമായിക്കണ്ട് യുദ്ധത്തില്‍ ഏര്‍പ്പെടുന്നവരുമായിരുന്നു. അവരുമായി യുദ്ധം ചെയ്യുമ്പോള്‍ മുസ്‌ലിംകളില്‍നിന്ന് അവര്‍ തടവുകാരാ യി പിടിക്കുന്നവരെ അവര്‍ അടിമകളാക്കി മാറ്റുകയോ വധിച്ചുകളയുകയോ ചെയ്യുമായിരുന്നു. അടിമത്തം നിരോധിക്കപ്പെട്ടിരുന്നുവെ ങ്കില്‍ മുസ്‌ലിംകള്‍ക്ക് അവരില്‍നിന്നുള്ള ബന്ദികളെ അടിമകളാക്കുവാന്‍ പറ്റുകയില്ല. ഇത് ശത്രുക്കള്‍ക്ക് മുസ്‌ലിം ബന്ദികളുടെ മേല്‍ കൂടു തല്‍ ക്രൂരത കാണിക്കുവാനുള്ള അവസരമുണ്ടാക്കുകയാണ് ചെയ്യുക. മുസ്‌ലിംകള്‍ക്കാണെങ്കില്‍ അവരില്‍നിന്ന് പിടിക്കപ്പെട്ടവര്‍ക്കു വേണ്ടി വില പേശുവാനായി ശത്രുക്കളില്‍നിന്ന് പിടിക്കപ്പെട്ട ബന്ദികളെ ഉപയോഗിക്കുവാനും കഴിയില്ല.

ഇസ്‌ലാമില്‍ അടിമത്തം നിരോധിക്കപ്പെട്ടാല്‍ അവരില്‍നിന്നുള്ളവരെ അടിമകളാക്കുവാനോ വധിക്കുവാനോ മുസ്‌ലിംകള്‍ക്ക് കഴിയുകയി ല്ലെന്ന് ശത്രുക്കള്‍ക്കറിയാം. അതുകൊണ്ടുതന്നെ അവരില്‍നിന്ന് പിടിക്കപ്പെട്ട ബന്ദികള്‍ക്ക് പകരമായി മുസ്‌ലിംകളില്‍നിന്ന് പിടിക്കപ്പെട്ട ബന്ദികളെ സ്വതന്ത്രരാക്കുകയെന്ന പരസ്പരധാരണക്ക് ശത്രുക്കള്‍ സന്നദ്ധരാവുകയില്ല.

മുസ്‌ലിംകള്‍ക്കാണെങ്കില്‍ ശത്രുക്കളില്‍നിന്നുള്ള ബന്ദികള്‍  തലവേദന മാത്രമായിത്തീരുകയും ചെയ്യും. അവര്‍ക്കുള്ള താമസസ്ഥലം ഉണ്ടാ ക്കുക മുസ്‌ലിം സമൂഹത്തിന്റെ ബാധ്യതതായിത്തീരും. ആയിരക്കണക്കിനാളുകള്‍ ബന്ദികളായി പിടിക്കപ്പെടുന്ന അവസരങ്ങളില്‍ അവ ര്‍ക്കെല്ലാം താമസിക്കാനാവശ്യമായ സൗകര്യങ്ങളുണ്ടാക്കുക ഏറെ ദുഷ്‌കരമായിത്തീരുമെന്ന് പറയേണ്ടതില്ലല്ലോ. അവര്‍ക്കുള്ള ഭക്ഷണ വും വസ്ത്രവുമെല്ലാം നല്‍കാന്‍ മുസ്‌ലിം സമൂഹം ബാധ്യസ്ഥമായിത്തീരും. അവര്‍ ഇവിടെ ഇസ്‌ലാമിക സമൂഹത്തിന്റെ സംരക്ഷണത്തി ല്‍ സുഖകരമായി ജീവിക്കുമ്പോള്‍ മുസ്‌ലിംകളില്‍നിന്ന് പിടിക്കപ്പെട്ട ബന്ദികള്‍ ഇസ്‌ലാമിന്റെ ശത്രുക്കളുടെ ക്രൂരതകള്‍ സഹിച്ച് അവര്‍ ഏല്‍പിക്കുന്ന കഠിനമായ ജോലികള്‍ ചെയ്തുകൊണ്ടിരിക്കുകയാവും. ഇത് ഒരിക്കലും നീതിയാവുകയില്ലല്ലോ. മുസ്‌ലിം സമൂഹത്തിന്റെ നാശത്തിനാണ് അതു നിമിത്തമാവുക. യുദ്ധം ഇസ്‌ലാമിക സമൂഹത്തെ നശിപ്പിക്കുകയല്ലാതെ മറ്റൊന്നും ചെയ്യാത്ത ഒരു അവസ്ഥയാണ് ഇതുവഴി സംജാതമാവുക. അതുകൊണ്ടുതന്നെ  ലോകം മുഴുവനായി അടിമത്തം നിരോധിക്കാത്ത അവസ്ഥയില്‍ ഇസ്‌ലാം ഏകപക്ഷീയമാ യി അടിമത്തം നിരോധിച്ചിരുന്നുവെങ്കില്‍ അത് ആത്മഹത്യാപരമാകുമായിരുന്നു. അതുകൊണ്ടുതന്നെയായിരിക്കണം സര്‍വകാലജ്ഞാനി യായ അല്ലാഹു അടിമത്തം നിരോധിക്കാതിരുന്നത്.

ലോകത്ത് അടിമത്തം ഒരു സ്ഥാപനമായി നിലനില്‍ക്കെ ഇസ്‌ലാം അതു നിരോധിക്കുന്നതുകൊണ്ട് പ്രായോഗിക തലത്തില്‍ ഗുണത്തേക്കാളേ റെ ദോഷമാണുണ്ടാവുകയെന്നുള്ളതാണ് വാസ്തവം. അടിമത്തം അനുവദിച്ചിരിക്കുന്ന ഇസ്‌ലാം അടിമയും ഉടമയുമെല്ലാം  സഹോദരങ്ങ ളാണെന്നും അടിമക്ക് അവകാശങ്ങളുണ്ടെന്നും പഠിപ്പിക്കുകയും അവനുമായി നല്ല നിലയില്‍ വര്‍ത്തിക്കണമെന്നും ക്രൂരമായി പെരുമാറ രുതെന്നും അപമാനിക്കരുതെന്നുമെല്ലാം നിഷ്‌കര്‍ഷിക്കുകയും ചെയ്യുന്നു. അതുകൊണ്ടുതന്നെ ഒരു മുസ്‌ലിമിന്റെ കീഴില്‍ ജീവിക്കുന്ന അടിമയെ സംബന്ധിച്ചിടത്തോളം അടിമത്തം അവന് ഒരു ഭാരമായിത്തീരുകയില്ല. അതോടൊപ്പംതന്നെ അവന്‍ സ്വതന്ത്രനാകുവാന്‍ ഏതു സമയത്തും സാധ്യതയുണ്ടുതാനും. സ്വാതന്ത്ര്യം വേണമെന്ന് സ്വയം തോന്നുമ്പോള്‍ അവന് സ്വാതന്ത്ര്യം നേടുവാന്‍ സാധിക്കുകയും ചെയ്യും.

എന്നാല്‍ ഇതേ അടിമ ഇത്തരം ധര്‍മങ്ങളിലൊന്നും വിശ്വാസമില്ലാത്ത ഒരു അമുസ്‌ലിമിന്റെ കീഴിലാണുള്ളതെങ്കിലോ? അയാള്‍ക്ക് അതി ക്രൂരമായ പെരുമാറ്റവും അതിനീചമായ അപമാനവുമാണ് ലഭിക്കുക. അയാളെ സംബന്ധിച്ചിടത്തോളം  അടിമത്തത്തില്‍നിന്നുള്ള മോച നം ഒരിക്കലും നടപ്പിലാകാത്ത സ്വപ്‌നം മാത്രമായിരിക്കും. ഒരു മുസ്‌ലിമിന്റെ കീഴിലുള്ള അടിമയായിരിക്കാനാണ് അതുകൊണ്ടുതന്നെ അടിമകള്‍ ഇഷ്ടപ്പെടുക. അവിടെ മാന്യമായ പെരുമാറ്റവും സഹാനുഭൂതിയോടുകൂടിയുള്ള സഹകരണവും കിട്ടുമല്ലോ. എന്നാല്‍, ഇസ്‌ ലാം അടിമത്തം നിരോധിച്ചിരുന്നെങ്കില്‍ ഒരിക്കലും അടിമക്ക് അത്തരമൊരു ജീവിതം നല്‍കുവാന്‍ ആരും സന്നദ്ധരാവുകയില്ല. മുസ്‌ലിമി നാണെങ്കില്‍ അടിമകളെ വെച്ചുകൊണ്ടിരിക്കാന്‍ പറ്റുകയുമില്ലല്ലോ.

അടിമത്തം നിലനില്‍ക്കുന്ന ഒരു സാമൂഹിക സംവിധാനത്തില്‍ ഇസ്‌ലാം മാത്രം അടിമത്തം നിരോധിക്കുന്നതുകൊണ്ട് കാര്യമായ ഗുണങ്ങ ളൊന്നുമില്ലെന്ന് മാത്രമല്ല അടിമയെ സംബന്ധിച്ചിടത്തോളം അത് കൂടുതല്‍ പ്രയാസങ്ങളുണ്ടാക്കുക മാത്രമേ ചെയ്യൂ. മുസ്‌ലിം സമൂഹത്തി ന്റെ നിലനില്‍പിനെത്തന്നെ ആ നിരോധം പ്രതികൂലമായി ബാധിക്കുകയും ചെയ്യും. അതുകൊണ്ടുതന്നെ അടിമകളുടെ മാനസികവും ശാരീ രികവുമായ മോചനത്തിനുവേണ്ടി ശ്രമിക്കുകയും അതിനാവശ്യമായ പ്രായോഗിക നിയമങ്ങള്‍ ആവിഷ്‌കരിക്കുകയുമാണ് ഇസ്‌ലാം ചെ യ്തത്. അതു മാത്രമാണ് അത്തരമൊരു സമൂഹത്തില്‍ കരണീയമായിട്ടുള്ളത്; പ്രായോഗികവും.

അടിമയുടെ രക്ഷിതാവ് ഉടമയാണ്, പുരുഷനായിരുന്നാലും സ്ത്രീയായിരുന്നാലുമെല്ലാം. പുരുഷനായ ഉടമയുടെ കീഴില്‍ കഴിയുന്ന അടിമ സ്ത്രീയുടെ കൈകാര്യകര്‍തൃത്വം ആ പുരുഷനിലാണ് നിക്ഷിപ്തമായിരിക്കുന്നത്.  അവളെ മറ്റൊരാള്‍ക്ക് വിവാഹം ചെയ്തുകൊടുക്കുക യാണെങ്കില്‍ അവനാണ് അത് നിര്‍വഹിക്കേണ്ടത്. അവളുടെ കാര്യങ്ങളെല്ലാം ശ്രദ്ധിക്കേണ്ടതും അവന്‍തന്നെ. അതുകൊണ്ടുതന്നെ അവളെ വിവാഹം ചെയ്യുകയെന്ന കര്‍മം നടക്കേണ്ടതില്ല. സ്ത്രീയുടെ രക്ഷിതാവും വരനും തമ്മില്‍ നടക്കുന്ന കരാറാണ് ഇസ്‌ലാമിലെ വിവാഹം. ഇവിടെ രണ്ടു പേരും ഉടമതന്നെയാണ്. അതുകൊണ്ടുതന്നെ വിവാഹച്ചടങ്ങ് അപ്രസക്തമാണ്.

പുരുഷന്റെ ഭാര്യമാരുടെ എണ്ണം പരമാവധി നാലായിരിക്കണമെന്നാണ് ഖുര്‍ആന്‍ നിഷ്‌കര്‍ഷിക്കുന്നത് (4:3). നാലു ഭാര്യമാരുള്ള ഒരാളുടെ കീഴില്‍ ജീവിക്കുന്ന ഒരു അടിമസ്ത്രീ ഉണ്ടെന്നിരിക്കട്ടെ, അയാള്‍ക്ക് അവളെ വിവാഹം ചെയ്യുവാന്‍ പറ്റുകയില്ല. അവളെ സ്വതന്ത്രയാക്കു വാന്‍ അയാള്‍ സന്നദ്ധനല്ലെങ്കില്‍ പാരതന്ത്ര്യത്തില്‍നിന്നുള്ള മോചനം അവള്‍ക്ക് ഒരു സ്വപ്‌നമായി അവശേഷിക്കും. അവളുടെ ലൈംഗിക ദാഹം ശമിപ്പിക്കുവാന്‍ വ്യഭിചാരത്തെ സമീപിക്കുവാന്‍ അവള്‍ നിര്‍ബന്ധിതയാവും. എന്നാല്‍, ഉടമക്ക് വിവാഹം കൂടാതെതന്നെ അവളു മായി ലൈംഗികബന്ധം പുലര്‍ത്താമെന്ന നിയമമുള്ളതിനാല്‍ ഈ പ്രശ്‌നങ്ങള്‍ക്കെല്ലാം പരിഹാരമാവും. ഉടമയുടെ കുഞ്ഞിനെ പ്രസവിക്കു കവഴി സ്വാതന്ത്ര്യത്തിലേക്ക് നടന്നുപോകാന്‍ അവള്‍ക്ക് സാധിക്കും. അവളുടെ ലൈംഗികതക്കുള്ള പരിഹാരമാകും. അടിമത്തം നിലനില്‍ ക്കുന്ന ഒരു സമൂഹത്തില്‍ വമ്പിച്ച മൂല്യത്തകര്‍ച്ചക്കാണ്, അവളുടെ ലൈംഗികദാഹം ശമിപ്പിക്കുവാനുള്ള സംവിധാനമുണ്ടാക്കിയില്ലെ ങ്കില്‍  അതു നിമിത്തമാവുക. അതോടൊപ്പംതന്നെ വ്യഭിചാരത്തിലൂടെയുണ്ടാകുന്ന കുഞ്ഞുങ്ങളുടെ പ്രശ്‌നവുമുണ്ട്.  അവരും സ്വാഭാവി കമായും അടിമകളായി മാറുകയാണ് ചെയ്യുക. അടിമത്തം ഒരിക്കലും അവസാനിപ്പിക്കാനാവാത്ത ഒരു സ്ഥാപനമായിത്തീരുകയാണ് ഇതിന്റെ ഫലം.

ഇസ്‌ലാമാകട്ടെ, അടിമസ്ത്രീകളുമായി ബന്ധപ്പെടുവാന്‍ ഉടമകളെ അനുവദിക്കുക വഴി അടിമത്തത്തെ ഒരു തലമുറയോടെ ഇല്ലാതാക്കുവാ നുള്ള സംവിധാനമാണുണ്ടാക്കുന്നത്. അതിന് 'വിവാഹം' ഒരു നിബന്ധനയായി വെക്കുകയാണെങ്കില്‍ ഇസ്‌ലാം ഉദ്ദേശിക്കുന്ന ഫലങ്ങളുണ്ടാ ക്കുവാന്‍ അതുമൂലം കഴിയുകയില്ല. അടിമസ്ത്രീയെ വിവാഹം ചെയ്യുവാന്‍ എല്ലാ ഉടമകളും തയാറാവുകയില്ല; നാല് ഭാര്യമാരുള്ളവര്‍ക്ക് അത് അസാധ്യവുമാണ്.

വിവാഹം നാലില്‍ പരിമിതപ്പെടുത്തണമെന്ന് നിഷ്‌കര്‍ഷിച്ച ഇസ്‌ലാം പക്ഷേ, കൈവശം വെക്കാവുന്ന അടിമസ്ത്രീകളുടെ എണ്ണത്തിന് യാതൊരു നിയന്ത്രണവുമേര്‍പ്പെടുത്തിയിട്ടില്ല. ഒരാള്‍ക്ക് എത്ര അടിമസ്ത്രീകളെയും കൈവശം വെച്ചുകൊണ്ടിരിക്കാം എന്നര്‍ഥം. എന്തു കൊണ്ടാണ് ഇസ്‌ലാം ഇത് അനുവദിച്ചത് എന്ന് പരിശോധിക്കപ്പെടേണ്ടതുണ്ട്.

അടിമകള്‍ യജമാനന്റെ കൈവശം എത്തിച്ചേരുന്നത് മൂന്നു മാര്‍ഗങ്ങളിലൂടെയാണ്. അനന്തരാവകാശമായി ലഭിക്കുക, സ്വയം വാങ്ങുക, യുദ്ധത്തില്‍ തടവുകാരായി പിടിക്കപ്പെടുക എന്നീ വഴികളിലൂടെ. ഇതില്‍ ഒരാള്‍ക്ക് സ്വന്തം ഇച്ഛ പ്രകാരം അടിമകളുടെ എണ്ണം പരിമിത പ്പെടുത്തുവാന്‍ കഴിയുക സ്വയം വാങ്ങുന്ന കാര്യത്തില്‍ മാത്രമാണ്. അനന്തരാവകാശമായി കിട്ടുകയോ യുദ്ധത്തില്‍ തടവുകാരായി പിടിക്കപ്പെട്ട് അടിമകളായിത്തീര്‍ന്ന് ഒരാളുടെ കൈവശം എത്തിച്ചേരുകയോ ചെയ്യുന്നവരുടെ എണ്ണം അയാള്‍ക്ക് നിയന്ത്രിക്കുവാനോ പരിമിതപ്പെടുത്തുവാനോ കഴിയില്ല. യുദ്ധത്തടവുകാരായി പിടിക്കപ്പെടുന്നവരെ അടിമകളാക്കുവാനാണ് ഭരണകൂടം തീരുമാനിക്കുന്ന തെങ്കില്‍ യുദ്ധത്തില്‍ പങ്കെടുത്തവര്‍ക്കിടയില്‍ അവരെ വീതിച്ചു നല്‍കുകയാണ് ചെയ്യുക. കുറെയേറെപ്പേരെ തടവുകാരായി പിടിക്കുക യാണെങ്കില്‍ ഓരോരുത്തരുടെയും കൈവശം എത്തിപ്പെടുന്ന അടിമകളുടെ എണ്ണവും കൂടും. ഹുനൈന്‍ യുദ്ധത്തില്‍ ആറായിരത്തോളം പേരെ തടവുകാരായി പിടിച്ചിരുന്നുവെന്ന് ചരിത്രത്തില്‍ കാണാം.

യുദ്ധത്തില്‍ പിടിക്കപ്പെടുന്നവരെ അനിവാര്യമായ സാഹചര്യങ്ങളില്‍ മാത്രമേ അടിമകളാക്കി മാറ്റിയിരുന്നുള്ളൂ. യുദ്ധത്തില്‍ ശത്രുക്കള്‍ പരാജയപ്പെട്ടാല്‍ അവരുടെ രാജ്യത്തെ സ്ത്രീകളെയോ കുട്ടികളെയോ തടവുകാരായി പിടിക്കുന്ന പതിവ് മുസ്‌ലിംകള്‍ക്കുണ്ടായിരുന്നില്ല. ഖലീഫമാരുടെ കാലത്ത് നടന്ന യുദ്ധങ്ങളില്‍  സിറിയ, ഫലസ്തീന്‍, ഇറാഖ്, ഈജിപ്ത് എന്നീ രാജ്യങ്ങളില്‍ ആരെയും അടിമകളാക്കി മാറ്റി യിരുന്നില്ലെന്ന് കാണാനാവും. യുദ്ധത്തില്‍ പുരുഷന്മാര്‍ വധിക്കപ്പെടുകയോ ബന്ധനസ്ഥരായി പിടിക്കപ്പെടുകയോ ചെയ്താല്‍ സ്ത്രീകളും കുട്ടികളും അനാഥരായിത്തീരുകയായിരിക്കും ഫലം. അവരെ യുദ്ധത്തില്‍ വധിക്കുവാന്‍ പാടില്ലെന്ന് ഇസ്‌ലാം നിഷ്‌കര്‍ഷിച്ചിട്ടുണ്ട്. പുരു ഷന്മാരോടൊപ്പം യുദ്ധത്തിന് പുറപ്പെടുന്ന സ്ത്രീകളും കുട്ടികളും തടവുകാരായി പിടിക്കപ്പെട്ടാല്‍തന്നെ മുസ്‌ലിം തടവുകാര്‍ക്ക് പകരമായി കൈമാറുകയായിരുന്നു പലപ്പോഴും ചെയ്തിരുന്നത്. ചില അവസരങ്ങളില്‍ അവരെ അടിമകളാക്കി മാറ്റുവാനും ഇസ്‌ലാം അനുവദിച്ചി രുന്നു. അടിമത്തം നിലനിന്നിരുന്ന ഒരു സമൂഹത്തിലായിരുന്നു ഈ അനുവാദമെന്നോര്‍ക്കണം.

ഏതായിരുന്നാലും ഈ വഴികളിലൂടെയെല്ലാം തങ്ങളുടെ കൈവശമെത്തിച്ചേരുന്ന അടിമകളെ പരിമിതപ്പെടുത്തുക അന്നത്തെ സാഹചര്യ ത്തില്‍ തികച്ചും പ്രയാസകരമായിരുന്നു. സ്ത്രീകളുടെ സ്ഥിതിയും അതുതന്നെ.  ഇങ്ങനെ കൈവശം എത്തിച്ചേരുന്ന സ്ത്രീകളെ എന്തുചെയ്യ ണമെന്നുള്ളതാണ് പ്രശ്‌നം. അവരെ മറ്റൊരാള്‍ക്ക് വിവാഹം ചെയ്തുകൊടുക്കാം. ഒരു സ്വതന്ത്രന്‍ അടിമയെ വിവാഹം ചെയ്യുവാനുള്ള സാധ്യത തുലോം വിരളമായിരുന്നുവെന്നോര്‍ക്കുക. അല്ലെങ്കില്‍ മറ്റൊരു അടിമക്കു വിവാഹം ചെയ്തുകൊടുക്കാം. രണ്ടാണെങ്കിലും അവ ള്‍ അയാളുടെ സ്വത്തായിരിക്കുന്നിടത്തോളം കാലം അവളിലുണ്ടാകുന്ന കുഞ്ഞുങ്ങളും ഇയാളുടെ സ്വത്തായിരിക്കും. അവരും അടിമകളാ യിത്തീരുമെന്നര്‍ഥം. അതല്ലെങ്കില്‍ നിരുപാധികം അവരെ സ്വതന്ത്രരാക്കി വിടാം. അത്തരത്തിലുള്ള സ്വാതന്ത്ര്യം അവരുടെ അനാഥത്വത്തി നാണ് വഴിവെക്കുക; ഗുരുതരമായ മൂല്യത്തകര്‍ച്ചക്കും. മറ്റൊരു മാര്‍ഗമാണ് അവളെ വിവാഹം കഴിക്കാതെതന്നെ, അവളുടെ മാനുഷിക മായ എല്ലാ അവകാശങ്ങളും അനുവദിച്ചുകൊണ്ട് ഉടമയോടൊപ്പം താമസിപ്പിക്കുകയെന്നത്. (അവളെ സ്വതന്ത്രയാക്കി വിവാഹം ചെയ്യു ക എല്ലാ ഉടമകളുടെ കാര്യത്തിലും പ്രായോഗികമായിരിക്കുകയില്ലല്ലോ).

അങ്ങനെ ജീവിക്കുന്ന അടിമകളുടെ എണ്ണം പരിമിതപ്പെടുത്തിയാല്‍ ആ പരിധിക്കു മുകളില്‍ വരുന്ന ഉടമയോടൊപ്പം കഴിയുന്ന അടിമ സ്ത്രീകളെ എന്തു ചെയ്യണമെന്ന പ്രശ്‌നമുത്ഭവിക്കും. അവര്‍ക്ക് സ്വാതന്ത്ര്യം പ്രാപിക്കുവാന്‍ മാര്‍ഗങ്ങളൊന്നുമുണ്ടാവുകയില്ല. അവരുടെ ലൈംഗികത അവഗണിക്കപ്പെടുകയോ ചൂഷണം ചെയ്യപ്പെടുകയോ ചെയ്യും. ഇത് വമ്പിച്ച ധാര്‍മികപ്രശ്‌നങ്ങള്‍ക്ക് നിമിത്തമാകും.

അടിമവ്യവസ്ഥിതി നിലനില്‍ക്കുന്ന സമൂഹത്തിന്റെ ഭൂമികയില്‍ നിന്നുകൊണ്ട് ഈ പ്രശ്‌നത്തെയും നോക്കിക്കണ്ടാല്‍ ഇക്കാര്യത്തില്‍ ഇസ്‌ലാം  നിശ്ചയിച്ച നിയമങ്ങളാണ് മാനവികവും പ്രാ യോഗികവുമെന്ന വസ്തുത വ്യക്തമാവും. പ്രസ്തുത സമൂഹത്തില്‍ ഒരാളുടെ കൈവശം എത്തിച്ചേരുന്ന പുരുഷ അടിമകളുടെ എണ്ണത്തി ന് പരിധി കല്‍പിക്കാന്‍ കഴിയില്ല. ഇതുതന്നെയാണ് സ്ത്രീ അടിമകളുടെയും അവസ്ഥ. അത്തരം നിയന്ത്രണങ്ങള്‍ പ്രസ്തുത സമൂഹത്തില്‍ അപ്രായോഗികമാണ്. അതുകൊണ്ടുതന്നെയാണ് ഇസ്‌ലാം അതിനു തുനിയാതിരുന്നത്.

അടിമസ്ത്രീ ഉടമയുടെ സ്വത്താണെന്ന് അംഗീകരിക്കുമ്പോള്‍ തന്നെ, അവളെ വ്യഭിചാരത്തിന് പ്രേരിപ്പിക്കാന്‍ ഉടമക്ക് അവകാശമില്ലെന്ന് ഖുര്‍ആന്‍ പഠിപ്പിക്കുന്നു(24:33). പുരുഷന്മാരായ അടിമകളെപ്പോലെ സ്ത്രീ അടിമകള്‍ക്കും അവകാശങ്ങളുണ്ട്. അവര്‍ക്ക് ഭക്ഷണം, പാര്‍ പ്പിടം, വസ്ത്രം എന്നിവയെല്ലാം നല്‍കേണ്ടത് യജമാനന്റെ കടമയാണ്. അവര്‍ക്ക് വിവാഹത്തിനുള്ള സൗകര്യം ചെയ്തുകൊടുക്കാനും ഖുര്‍ആന്‍ ഉടമയോട് ആവശ്യപ്പെടുന്നുണ്ട് (24:32).

അടിമസ്ത്രീകളുടെ ലൈംഗികമായ ആവശ്യങ്ങളുടെ പൂര്‍ത്തീകരണത്തിനാവശ്യമായ സംവിധാനങ്ങള്‍ ചെയ്യാന്‍ ഖുര്‍ആന്‍ ഉടമകളെ പ്രേരിപ്പിക്കുന്നുണ്ടെന്നര്‍ഥം.

എന്നാല്‍, വിവാഹിതയല്ലാത്ത ഒരു അടിമസ്ത്രീയുമായി ലൈംഗികബന്ധം സ്ഥാപിക്കുന്നതില്‍നിന്ന് ഉടമയെ ഇസ്‌ലാം വിലക്കുന്നില്ല. ഈ അനുവാദം ഉടമയില്‍ മാത്രം പരിമിതമാണ്. മറ്റൊരാള്‍ക്കും അവളെ ഉപയോഗിക്കുവാന്‍ അനുവാദമില്ല. യജമാനന്റെ അനുവാദത്തോടെ യാണെങ്കിലും ശരി!

യജമാനന്റെ കുഞ്ഞിനെ പ്രസവിക്കുന്നതോടുകൂടി അടിമസ്ത്രീക്ക് പുതിയ അവകാശങ്ങളുണ്ടാവുകയാണ്. അവളെ പിന്നെ വില്‍ക്കു വാന്‍ യജമാനന് അവകാശമില്ല. അവള്‍ പിന്നെ യജമാനന്റെ കുട്ടികളുടെ മാതാവാണ്. ആ കുട്ടികള്‍ക്ക് യജമാനന്റെ മറ്റു കുട്ടികളെപ്പോ ലെയുള്ള എല്ലാ അവകാശങ്ങളുമുണ്ടുതാനും. അടിമസ്ത്രീയിലുണ്ടാവുന്ന കുഞ്ഞുങ്ങള്‍ക്ക് പിതൃസ്വത്തില്‍ അവകാശങ്ങളൊന്നുമില്ലെന്ന യഹൂദനിലപാടുമായി ഇസ്‌ലാം വിയോജിക്കുന്നു. ആ കുട്ടികള്‍ എല്ലാ അര്‍ഥത്തിലും അയാളുടെ മക്കള്‍തന്നെയാണ്. യാതൊരു തരത്തി ലുള്ള ഉച്ചനീചത്വങ്ങളും അവരും മറ്റു മക്കളും തമ്മില്‍ ഉണ്ടാകുവാന്‍ പാടില്ല.

യജമാനന്റെ മരണത്തോടെ അയാളുടെ മക്കളുടെ ഉമ്മയായ അടിമസ്ത്രീ സ്വതന്ത്രയായിത്തീരുന്നു. പിന്നെ അവളെ സംരക്ഷിക്കുന്നത് അവ ളുടെ മക്കളാണ്. അവര്‍ക്കാണെങ്കില്‍ പിതൃസ്വത്തില്‍നിന്ന് മറ്റു മക്കളെപ്പോലെതന്നെയുള്ള അവകാശം ലഭിക്കുകയും ചെയ്യും.

ഒരു തലമുറയോടെ അടിമത്തത്തിന്റെ വേരറുക്കുന്നതിനുവേണ്ടിയുള്ള ഒരു സംവിധാനമാണ് ദാസിമാരെ ജീവിത പങ്കാളികളായി സ്വീകരി ക്കാനുള്ള അനുവാദം. അടിമത്തം പ്രായോഗികമായി ഇല്ലാതാക്കുന്നതിനുവേണ്ടിയുള്ള ഇസ്‌ലാമിന്റെ വ്യത്യസ്തമായ നടപടികളിലൊ ന്നായിരുന്നു അടിമസ്ത്രീകളുമായി ബന്ധപ്പെടുന്നതിനുവേണ്ടിയുള്ള അനുവാദമെന്നുള്ളതാണ് വാസ്തവം. ഇസ്‌ലാമിക ലോകത്ത് നടന്ന തും അതുതന്നെയാണ്.

അടിമസ്ത്രീകളുമായി ബന്ധപ്പെടുന്നതിന് ഇസ്‌ലാം നല്‍കിയ അനുവാദം വ്യഭിചാരത്തിനുള്ള അനുമതിയാ ണെന്ന് തെറ്റിദ്ധരിപ്പിക്കുന്നവ രുണ്ട്. അടിമസ്ത്രീയുമായുള്ള ലൈംഗികബന്ധത്തെ വ്യഭിചാരവുമായി താരതമ്യം ചെയ്യാനേ കഴിയില്ല. രണ്ടും രണ്ട് വിരുദ്ധ ധ്രൂവങ്ങളില്‍ നില്‍ക്കുന്നു. ഒന്ന് ഉടമയുടെ കീഴില്‍ അയാളുടെ സംരക്ഷണത്തില്‍ കഴിയുന്നവളുമായുള്ള ബന്ധമാണ്. ആ ബന്ധത്തില്‍നിന്നുണ്ടാകുന്ന ബാധ്യതകള്‍ ഏറ്റെടുക്കാന്‍ സന്നദ്ധനായിക്കൊണ്ടുള്ള ബന്ധം. ലൈംഗികതയ്ക്കപ്പുറമുള്ള അവളുടെ വ്യക്തിത്വം അംഗീകരിച്ചുകൊണ്ടുള്ള ബന്ധം. അവ ള്‍ക്ക് സ്വാതന്ത്ര്യത്തിലേക്കുള്ള വാതായനം തുറന്നുകൊടുക്കുന്ന ബന്ധം. രണ്ടാമത്തേതോ ഒരു ഉത്തരവാദിത്വവുമില്ലാത്ത വേശ്യാബന്ധം. വേശ്യ യഥാര്‍ഥത്തില്‍ അടിമയേക്കാള്‍ പതിതയാണ്. അവള്‍ ആത്മാവില്ലാത്ത ഒരു മൃഗം മാത്രം. പുരുഷന്റെ മാംസദാഹം തീര്‍ക്കുകയാണ് അവളുടെ കര്‍ത്തവ്യം. ആ ബന്ധത്തില്‍ സ്‌നേഹത്തിന്റെയും കാരുണ്യത്തിന്റെയും കണികപോലുമില്ല; ആത്മസംതൃ പ്തിയുടെ സ്പര്‍ശം ലേശം പോലുമില്ല. പണത്തിനുവേണ്ടി നടത്തുന്ന ഒരു കച്ചവടം മാത്രമാണത്. മാംസക്കച്ചവടം! അതില്‍നിന്നുള്ള ബാധ്യ തയേറ്റെടുക്കുവാന്‍ മാംസദാഹം തീര്‍ക്കുവാന്‍ വേണ്ടി വന്ന പുരുഷന്‍ സന്നദ്ധനല്ല. അവള്‍ക്ക് എന്തെങ്കിലുമൊരു അവകാശം അവ ന്റെ മേല്‍ ഇല്ല. അവന്റെ മാംസദാഹം തീര്‍ക്കാന്‍ വിധിക്കപ്പെട്ട ഒരു മൃഗം മാത്രമാണവള്‍. അവളുടെ ഓരോ ബന്ധവും അവളെ വേശ്യാവൃത്തി യുടെ മൃഗീയതയിലേക്ക് കൂടുതല്‍ ആഴത്തില്‍ ആപതിപ്പിക്കുകയാണ് ചെയ്യുന്നത്. അവള്‍ക്ക് സ്വാതന്ത്ര്യത്തെക്കുറിച്ച സ്വപ്‌നം പോലും അന്യമാണ്.  തൊലി ചുളിഞ്ഞ് ആര്‍ക്കും വേണ്ടാതായി മാറി രോഗിണിയാവുമ്പോള്‍ അനാഥത്വം പേറുവാന്‍ വിധിക്കപ്പെട്ടവള്‍!

അടിമക്കു സ്വാതന്ത്ര്യത്തിലേക്കുള്ള വാതായനം തുറന്നുകൊടുക്കുന്ന സമ്പ്രദായമെവിടെ? സ്ത്രീയെ പാരതന്ത്ര്യത്തില്‍നിന്ന് പാരതന്ത്ര്യത്തി ലേക്കു നയിക്കുന്ന ദുഷിച്ച വ്യവസ്ഥയെവിടെ? ഇവ രണ്ടും തമ്മില്‍ താരതമ്യം പോലും അസാധ്യമാണ്. രണ്ടും രണ്ടു വിരുദ്ധ ധ്രുവങ്ങളില്‍ നില്‍ക്കുന്ന സമ്പ്രദായങ്ങള്‍. ഒന്ന് മനുഷ്യത്വം അംഗീകരിക്കുന്നത്, മറ്റേത് മൃഗീയതയിലേക്ക് ആപതിപ്പിക്കുന്നത്.

അടിമസ്ത്രീയെ വിവാഹം ചെയ്യണമെന്നുള്ളവര്‍ക്ക് അങ്ങനെ ചെയ്യാനുള്ള അനുവാദം ഖുര്‍ആന്‍ നല്‍കുന്നുണ്ട് (4:27). ഇങ്ങനെയുള്ള വിവാഹം ഇരട്ടി പ്രതിഫലം നല്‍കുന്നതാണെന്നാണ് പ്രവാചകന്‍ r പഠിപ്പിച്ചിരിക്കുന്നത്. ''തന്റെ കീഴിലുള്ള അടിമസ്ത്രീയെ സംസ്‌കാര സമ്പന്നയാക്കുകയും അവള്‍ക്ക് ഏറ്റവും നന്നായി വിദ്യാഭ്യാസം  നല്‍കുകയും പിന്നീട് അവളെ മോചിപ്പിച്ച് സ്വയം വിവാഹം കഴിക്കുക യും ചെയ്തവനും ഇരട്ടി പ്രതിഫലമുണ്ട്'' (ബുഖാരി, മുസ്‌ലിം).

വിഷയവുമായി ബന്ധപ്പെട്ട വീഡിയോ

ക്വുർആനിലെ 2: 2:233 വചനത്തില്‍ മുലകുടി പ്രായം രണ്ടു വര്‍ഷമാണെന്നും 46:15 വചനത്തില്‍ ഗര്‍ഭകാലവും മുലകുടി പ്രായവും കൂടി മുപ്പതു മാസമാണെന്നും പറഞ്ഞതിനെ താരതമ്യം ചെയ്താൽ ഗർഭകാലം ആറ് മാസമാണ് എന്നാണ് വന്നു ചേരുക. ഇത് വ്യക്തമായ അബദ്ധമല്ലേ ?

സൂറത്തുല്‍ ബക്വറയിലെ 2:233-ാം വചനത്തിലും സൂറത്തു ലുഖ്മാനിലെ പതിനാലാം വചനത്തിലും മുലകുടി പ്രായം രണ്ടു വര്‍ഷമാണെന്ന് വ്യക്തമാക്കുന്നുണ്ട്. മുലകുടി പൂർത്തിയായ്ക്കാനുദ്ദേശിക്കുന്നവർ കുഞ്ഞുങ്ങൾക്ക് രണ്ട് വര്‍ഷമാണ് മുല കൊടുക്കേണ്ടതെന്ന് ഈ വചനങ്ങൾ വ്യക്തമാക്കുന്നു. സൂറത്തുല്‍ അഹ്ക്വാഫിലെ പതിനഞ്ചാം വചനത്തിൽ "അവന്റെ ഗര്‍ഭകാലവും മുലകുടിനിര്‍ത്തലും കൂടി മുപ്പത് മാസക്കാലമാകുന്നു" എന്ന് വ്യക്തമായി പരാമർശിച്ചിട്ടുണ്ട്. ഇതിൽ നിന്ന് ഗർഭകാലം ആറ് മാസമാണ് എന്നല്ലേ മനസ്സിലാവുകയെന്നാണ് വിമർശനം. അങ്ങനെത്തന്നെയാണ് മനസ്സിലാക്കേണ്ടത് എന്നാണ് അതിനുള്ള വിശദീകരണം. ഒൻപത് മാസം കഴിഞ്ഞാണ് സാധാരണഗതിയിൽ പ്രസവം നടക്കാറുള്ളതെന്ന് ആരും പറഞ്ഞു കൊടുക്കാതെതന്നെ എല്ലാവർക്കും അറിയാവുന്ന കാര്യമാണ്. ഇക്കാര്യം മുഹമ്മദ് നബിക്ക് (സ) അറിയുമായിരുന്നില്ല എന്ന കരുതുന്നത് ശുദ്ധ അസംബന്ധമാണ്.

ഈ വചനങ്ങളിൽ നിന്ന് നാം മനസിലാക്കേണ്ടത് കുറഞ്ഞ ഗർഭകാലം ആറു മാനസമാണ് എന്നാണ്. ഇങ്ങനെ മനസ്സിലാക്കിയവരായിരുന്നു ആദ്യകാല മുസ്ലിംകൾ. ഒരു സംഭവം നോക്കുക:

വിവാഹത്തിനുശേഷം ആറുമാസങ്ങള്‍ കഴിയുന്നയുടനെ പ്രസവിച്ച ഒരു സ്ത്രീയെക്കുറിച്ച ഒരു പരാതി ഖലീഫ ഉമറിന്റെ (റ) അടുത്തെത്തി. പ്രസവിക്കപ്പെട്ട കുഞ്ഞിന് ജീവനും ആരോഗ്യവുമുള്ളതിനാല്‍ വിവാഹപൂര്‍വരതിയിലൂടെയുണ്ടായതാവണം അവരുടെ ഗര്‍ഭധാരണമെന്നും അതിനാല്‍ അവര്‍ക്ക് വ്യഭിചാരത്തിനുള്ള ശിക്ഷ നല്‍കണമെന്നുമായിരുന്നു പരാതിക്കാരുടെ പക്ഷം. പ്രശ്‌നത്തിനു പരിഹാരം തേടി പ്രവാചകാനുചരന്‍മാരുമായി ഉമര്‍ (റ) കൂടിയാലോചന നടത്തി. അബ്ദുല്ലാഹിബ്‌നു അബ്ബാസാണ് (റ) പ്രസ്തുത പ്രസവത്തെ ക്വുര്‍ആനിന്റെ അടിസ്ഥാനത്തില്‍ ന്യായീകരിച്ചത്. സൂറത്തുല്‍ ബക്വറയിലെ 2:233-ാം വചനവും സൂറത്തുല്‍ അഹ്ക്വാഫിലെ പതിനഞ്ചാം വചനവും ഉദ്ധരിച്ചുകൊണ്ട് ഈ വചനങ്ങള്‍പ്രകാരം കുറഞ്ഞ ഗര്‍ഭകാലം ആറുമാസമാണെന്ന് സ്ഥാപിക്കുകയാണ് അദ്ദേഹം ചെയ്തത്. സൂറത്തുല്‍ ബക്വറയിലെ വചനത്തില്‍ മുലകുടി പ്രായം രണ്ടു വര്‍ഷമാണെന്നും സൂറത്തുല്‍ അഹ്ക്വാഫില്‍ ഗര്‍ഭകാലവും മുലകുടി പ്രായവും കൂടി മുപ്പതു ദിവസമാണെന്നും പറഞ്ഞതിനെ താരതമ്യം ചെയ്തുകൊണ്ടാണ് പ്രവാചകാനുചരന്‍മാരിലെ ക്വുര്‍ആന്‍ വ്യാഖ്യാതാവെന്ന് അറിയപ്പെട്ടിരുന്ന ഇബ്‌നു അബ്ബാസ് (റ) കുറഞ്ഞ ഗര്‍ഭകാലം ആറു മാസമാണെന്ന് സമര്‍ത്ഥിച്ചത്. ഭരണാധികാരിയായ ഉമര്‍ (റ) അടക്കമുള്ള സ്വഹാബിമാരെല്ലാം അത് അംഗീകരിക്കുകയും കുറ്റാരോപിതയെ വെറുതെ വിടാന്‍ ഖലീഫ കല്‍പിക്കുകയും ചെയ്തു.(ഇമാം അബ്ദുര്‍റസാഖ് തന്റെ മുസന്നഫിലും (7: 352) ഇമാം സുയൂത്തി തന്റെ ദുര്‍റുല്‍ മന്‍സൂറിലും (7: 442) നാഫിഉ ബിന്‍ ജുബൈറില്‍ നിന്ന് നിവേദനം ചെയ്തത്.)

ഉഥ്മാന്റെ (റ) ഭരണകാലത്തും സമാനമായ സംഭവമുണ്ടായതായി ഇമാം മാലിക് (റ) നിവേദനം ചെയ്യുന്നുണ്ട്. ആറാം മാസം കഴിഞ്ഞയുടനെ ആരോഗ്യമുള്ള കുഞ്ഞിനെ പ്രസവിച്ച സ്ത്രീക്ക് വ്യഭിചാരക്കുറ്റത്തിന് ശിക്ഷ വിധിച്ച ഖലീഫയെ തിരുത്തിയത് അലി(റ)യാണ്. നടേ പറഞ്ഞ ആയത്തുകള്‍ ഉദ്ധരിച്ചുകൊണ്ട് ഖലീഫയുടെ വിധിയെ വിമര്‍ശിച്ചത് ഉഥ്മാന്‍ (റ) അംഗീകരിക്കുകയും സ്ത്രീയെ വെറുതെ വിടുകയും ചെയ്തു.(ഇമാം മാലികിന്റെ മുവത്വ 41: 11)

കുറഞ്ഞ ഗര്‍ഭകാലമെത്രയാണെന്ന കാര്യത്തില്‍ പ്രവാചകാനുചരന്‍മാരുടെ കാലം മുതല്‍ മുസ്‌ലിം ലോകത്ത് കാര്യമായ തര്‍ക്കങ്ങളുണ്ടായിട്ടില്ല. നാലു കര്‍മശാസ്ത്ര സരണികളും കുറഞ്ഞ ഗര്‍ഭകാലം ആറുമാസമാണെന്ന് അംഗീകരിക്കുന്നു. പിതൃത്വവും ശിക്ഷാവിധികളുമായി ബന്ധപ്പെട്ട മദ്ഹബീ നിയമങ്ങളിലെല്ലാം ഈ അംഗീകാരത്തിന്റെ സ്വാധീനം കാണാനാവും. മുസ്‌ലിം ലോകത്ത് പതിനാലു നൂറ്റാണ്ടുകളായി അംഗീകരിക്കപ്പെട്ടുവരുന്ന കുറഞ്ഞ ഗര്‍ഭകാലം തന്നെയാണ് ശരിയെന്ന വസ്തുത അംഗീകരിക്കുകയാണ് ആധുനിക സാങ്കേതികവിദ്യകളുടെ സഹായത്തോടെ ശാസ്ത്രം ഇന്നു ചെയ്യുന്നത്. കുറഞ്ഞ ഗര്‍ഭകാലത്തെക്കുറിച്ച സംവാദങ്ങളും തര്‍ക്കങ്ങളും ഭ്രൂണശാസ്ത്രലോകത്ത് സജീവമാണെങ്കിലും നിയമപരമായി അംഗീകരിക്കാവുന്ന കുറഞ്ഞ ഗര്‍ഭകാലം ആറുമാസമാണെന്ന വസ്തുത ഇന്ന് എല്ലാവരും സമ്മതിക്കുന്നുണ്ട്.

ഗര്‍ഭാശയത്തിനുപുറത്ത് ഗര്‍ഭസ്ഥശിശുവിന് ജീവിക്കാനുള്ള കഴിവിനെയാണ് ശിശുജീവനസാമര്‍ത്ഥ്യം (Fetal Viability) എന്നുവിളിക്കുന്നത്. ഗര്‍ഭകാലത്തെ മൂന്നു ത്രൈമാസിക യൂണിറ്റുകളായാണ് (trimester) ഭ്രൂണശാസ്ത്രജ്ഞന്‍മാര്‍ പഠിക്കുന്നത്. ആദ്യത്തെ ത്രൈമാസികത്തിലാണ് ഭ്രൂണത്തില്‍ അടിസ്ഥാനപരമായ മാറ്റങ്ങളെല്ലാം ഉണ്ടാകുന്നത്. ഒന്നാം ത്രൈമാസത്തിനകത്ത് പ്രസവിക്കപ്പെട്ടാല്‍ ശിശുജീവനസാമര്‍ത്ഥ്യം പൂജ്യമായിരിക്കും. അഥവാ അങ്ങനെ പ്രസവിക്കപ്പെടുന്ന കുഞ്ഞ് ഒരു കാരണവശാലും ജീവിച്ചിരിക്കുകയില്ല. രണ്ടാം ത്രൈമാസികത്തില്‍ നടക്കുന്നത് പ്രധാനമായും അവയവങ്ങളുടെ വികാസമാണ്. രണ്ടാമത്തെ തൈമാസികം അവസാനിക്കുമ്പോള്‍ പ്രസവിക്കപ്പെടുന്ന കുഞ്ഞിന് നല്ല പരിചരണം നല്‍കിയാല്‍ അത് ജീവിക്കും. ഈ സമയത്തെ ശിശുജീവനസാമര്‍ത്ഥ്യം (Fetal Viability) 90 ശതമാനമാണ്. നല്ല പരിചരണം നല്‍കിയാല്‍ കുഞ്ഞിനെ രക്ഷിക്കുവാനും കാര്യമാത്രപ്രസക്തമായ വൈകല്യങ്ങളൊന്നുമില്ലാതെ നിലനിര്‍ത്തുവാനും കഴിയുന്ന പ്രായമാണിത് എന്നര്‍ത്ഥം.

ഗര്‍ഭസ്ഥ ശിശുവിന് ഇരുപത്തിരണ്ടാമത്തെ ആഴ്ച പ്രായമാകുന്നതുമുതല്‍ തന്നെ ശിശുജീവനസാമര്‍ത്ഥ്യത്തിന് നേരിയ സാധ്യതകളുണ്ടെന്നാണ് പുതിയ പഠനങ്ങള്‍ വ്യക്തമാക്കുന്നത്. ഇരുപത്തിമൂന്നാമത്തെ ആഴ്ച ഇത് പത്തുമുതല്‍ മുപ്പത്തിയഞ്ച് വരെ ശതമാനവും ഇരുപത്തിനാലാമത്തെ ആഴ്ച ഇത് നാല്‍പത് മുതല്‍ എഴുപത് വരെ ശതമാനവും ഇരുപത്തിയഞ്ചാമത്തെ ആഴ്ച ഇത് അമ്പത് മുതല്‍ എണ്‍പതു വരെ ശതമാനവും ഇരുപത്തിയാറാമത്തെ ആഴ്ച ഇത് എണ്‍പത് മുതല്‍ തൊണ്ണൂറുവരെ ശതമാനവും ഇരുപത്തിയേഴാമത്തെ ആഴ്ച മുതല്‍ ഇത് തൊണ്ണൂറ് ശതമാനത്തിനു മുകളിലുമാണ്. ആറു മാസങ്ങള്‍ക്ക് മുമ്പുള്ള ശിശുജീവനസാമര്‍ത്ഥ്യത്തിന്റെ ശതമാനക്കകണക്ക് ഉയരാനുള്ള കാരണം ചികിത്സാരംഗത്തും സാങ്കേതിക വിദ്യയാലുമുണ്ടായ പുരോഗതിയാണ്. ഈ പുരോഗതിയുണ്ടായിട്ട് ഏതാണ്ട് പതിറ്റാണ്ടുകളേ ആയിട്ടുള്ളൂ. 1973ലെ പ്രസിദ്ധമായ ഒരു ഗര്‍ഭഛിദ്ര കേസില്‍ പോലും അമേരിക്കന്‍ സുപ്രീം കോടതി വിധിച്ചത് ശിശുജീവനസാമര്‍ത്ഥ്യം ഇരുപത്തിയെട്ട് ആഴ്ചകളെങ്കിലും പൂര്‍ത്തിയായാലേ ഉണ്ടാവുകയുള്ളുവെന്നാണ് പൊതുവെ കരുതി വരാറുള്ളതെന്നാണ്. ഇരുപത്തിനാല് ആഴ്ചകളെങ്കിലും പൂര്‍ത്തിയായാലേ ശിശുവിന് ജീവനസാമര്‍ത്ഥ്യമുണ്ടാകൂവെന്നാണ് ഇന്ന് പൊതുവെ ചികിത്സാരംഗത്തുള്ളവര്‍ പറയാറുള്ളതെങ്കിലും അതിനേക്കാള്‍ മുമ്പ് പ്രസവിക്കപ്പെട്ടിട്ടും ജീവിച്ച റിക്കാര്‍ഡുകളുണ്ട്.

2006 ഒക്‌ടോബര്‍ 24ന് ഫ്‌ളോഡിറിയില്‍ ഇരുപത്തിരണ്ട് ആഴ്ചകള്‍ മാത്രം കഴിഞ്ഞ് ജനിച്ച അമില്ലിയ ടൈലറെന്ന പെണ്‍കുട്ടിയാണ് ഏറ്റവും കുറഞ്ഞ ഗര്‍ഭകാലം കഴിഞ്ഞ് ജീവനസാമര്‍ത്ഥ്യത്തോടെയിരിക്കുകയും പിന്നീട് വളര്‍ന്നു വലുതാവുകയും ചെയ്തയാളായി രേഖപ്പെടുത്തപ്പെട്ടിരിക്കുന്നത്. ശ്വാസകോശങ്ങള്‍ക്കും ദഹനവ്യവസ്ഥക്കും തലച്ചോറിനുമെല്ലാം നിരവധി തകരാറുകളുണ്ടായിരുന്നുവെങ്കിലും മികച്ച സാങ്കേതികവിദ്യകളുടെ സഹായത്താല്‍ ഒരു കൂട്ടം ഭിഷഗ്വരന്‍മാര്‍ ഭഗീരഥപ്രയത്‌നം നടത്തി കുട്ടിയെ ജീവനോടെ നിലനിര്‍ത്തുകയാണുണ്ടായത്. നീണ്ട നാലുമാസങ്ങളില്‍ ആശുപത്രിയിലെ ശിശു തീവ്രപരിചരണ വിഭാഗത്തില്‍ കിടത്തിയുള്ള നിരന്തരമായ പരിശ്രമങ്ങളുടെ ഫലമായാണ് അവരുടെ മാതാപിതാക്കള്‍ക്ക് ജീവനുള്ള കുഞ്ഞിനെ ലഭിച്ചത് എന്നര്‍ത്ഥം.

രണ്ടാമത്തെ ത്രൈമാസം കഴിയുമ്പോഴേക്ക് ഗര്‍ഭസ്ഥശിശുവില്‍ ഒരുവിധം എല്ലാ ബാഹ്യാവയവങ്ങളും ആന്തരാവയവങ്ങളും വളര്‍ന്നുവന്നിരിക്കുമെന്നതിനാല്‍ തന്നെ അതിനുശേഷം പ്രസവിക്കപ്പെടുന്ന കുഞ്ഞുങ്ങള്‍ ജീവിച്ചിരിക്കുവാനുള്ള സാധ്യത അഥവാ ശിശുജീവനസാമര്‍ത്ഥ്യം തൊണ്ണൂറു ശതമാനത്തിനു മുകളിലാണ്. ഗര്‍ഭാശയത്തില്‍വെച്ചു തന്നെ പൂര്‍ണ വളര്‍ച്ചയെത്തി പുറത്തുവരുന്ന കുഞ്ഞ് മാതൃശരീരത്തിനകത്ത് തന്റെ ആദ്യകോശമുണ്ടാകുന്നതു മുതല്‍ മുപ്പത്തിയൊന്‍പത് ആഴ്ചക്കാലമാണ് കഴിച്ചുകൂട്ടുന്നത്. പൂര്‍ണമായ ഗര്‍ഭകാലമാണിത്. ഇതിനുമുമ്പ് ഏതു സമയത്തും കുഞ്ഞ് പ്രസവിക്കപ്പെടാം. ഗര്‍ഭാശയത്തിനകത്തും പുറത്തും കുഞ്ഞിന് വളരാനാവശ്യമായ സംവിധാനങ്ങളെല്ലാം ചെയ്തുവെച്ചിരിക്കുന്നവനാണ് സ്രഷ്ടാവ്. മാതൃശരീരത്തില്‍ നിന്ന് പുറത്തുവരുന്ന ശിശുവിന് പിന്നെ മാതാവുമായുള്ള ജൈവികബന്ധം അതിന്റെ മുലകുടിയാണ്. മനുഷ്യശിശുവിന്റെ മുലകുടി പ്രായം രണ്ടു വര്‍ഷമാണെന്ന കാര്യത്തില്‍ ശാസ്ത്രവും ക്വുര്‍ആനും ഒരേ അഭിപ്രായമാണ് പുലര്‍ത്തുന്നത്.

പൂര്‍ണമായ മുലകുടി പ്രായം രണ്ടു വര്‍ഷമാണെന്നു പറയുമ്പോള്‍ അതിനുമുമ്പ് ഏതുസമയത്തും മാതാവിന്റെയോ കുഞ്ഞിന്റെയോ ആരോഗ്യപരമായ കാരണങ്ങളാല്‍ മുലകുടി നിന്നുപോകുവാനുള്ള സാധ്യത ക്വുര്‍ആന്‍ അംഗീകരിക്കുന്നു. മുലകുടിയോടു കൂടി ബന്ധപ്പെടുത്തിയാണ് കുറഞ്ഞ ഗര്‍ഭകാലത്തെക്കുറിച്ച് ക്വുര്‍ആന്‍ പറഞ്ഞിരിക്കുന്നതെന്ന വസ്തുത ശ്രദ്ധേയമാണ്. മുലകുടിയും ഗര്‍ഭകാലവും കൂടി മുപ്പത് മാസമാണെന്ന ക്വുര്‍ആനിക പരാമര്‍ശമാണ് ചുരുങ്ങിയ ഗര്‍ഭകാലം ആറുമാസമാണെന്ന നിഗമനത്തിലെത്താന്‍ പ്രവാചകാനുചരന്മാരെ സഹായിച്ചത്. കാര്യമായ സാങ്കേതിക സഹായങ്ങളൊന്നുമില്ലെങ്കില്‍ പോലും, ആറു മാസങ്ങള്‍ പൂര്‍ത്തിയാക്കി ഗര്‍ഭാശയത്തിനകത്തുനിന്ന് പുറത്തുവരുന്ന കുഞ്ഞിന് ജീവിക്കുവാന്‍ കഴിയുമെന്നാണ് പുതിയ പഠനങ്ങളും വ്യക്തമാക്കുന്നത്. ആറുമാസം പൂര്‍ത്തിയാക്കുന്നതോടെ ശിശുവിന്റെ ജീവനസാമര്‍ത്ഥ്യം തൊണ്ണൂറ് ശതമാനമാണെന്നാണല്ലോ പഠനങ്ങള്‍ കാണിക്കുന്നത്.

ആറു മാസങ്ങള്‍ പൂര്‍ത്തീകരിക്കുന്നതിന് ഏതാനും ദിവസങ്ങള്‍ക്ക് മുമ്പ് പ്രസവിക്കുന്ന കുഞ്ഞുങ്ങളുടെ ജീവന്‍ രക്ഷിക്കുവാന്‍ സാങ്കേതിക സഹായങ്ങളോടെ സാധിക്കുമെന്നതിനാലാണ് കഴിഞ്ഞ അരനൂറ്റാണ്ടിനിടെ ജീവനസാമര്‍ത്ഥ്യത്തില്‍ കാര്യമായ പുരോഗതിയുണ്ടായിട്ടുണ്ടെന്നു പറയുന്നത്. പ്രസ്തുത പുരോഗതിയുടെ ഫലമായി ഇരുപത്തിനാല് ആഴ്ചകളെങ്കിലും പൂര്‍ത്തിയാക്കിയ കുഞ്ഞുങ്ങളെ രക്ഷിക്കാന്‍ കഴിഞ്ഞേക്കുമെന്ന ഒരു ധാരണ ചികിത്സാരംഗത്തുണ്ടായിട്ടുണ്ട്. വിദഗ്ധരായ ചികിത്സകരുടെ മേല്‍നോട്ടത്തില്‍ ശക്തമായ സാങ്കേതിക സഹായത്തോടെയാണ് പ്രസ്തുത രക്ഷിക്കല്‍ ശ്രമം നടക്കുന്നത്. അങ്ങനെ രക്ഷപെടുന്ന കുഞ്ഞുങ്ങള്‍ വ്യത്യസ്തതരം വൈകല്യങ്ങള്‍ക്ക് വിധേയമായിരിക്കും. തലച്ചോറ് വേണ്ടത്ര വികസിച്ചിട്ടില്ലാത്തതിനാല്‍ ഓട്ടിസമടക്കമുള്ള വൈകല്യങ്ങളുണ്ടാവാനുള്ള സാധ്യതയേറെയാണ്. ശ്വാസകോശങ്ങളുടെയും കണ്ഠനാളികളുടെയും വളര്‍ച്ച പൂര്‍ത്തീകരിച്ചു കഴിഞ്ഞിട്ടില്ലാത്തതിനാല്‍ അത്തരം ശിശുക്കള്‍ക്ക് മുല കുടിക്കുവാന്‍ പലപ്പോഴും കഴിയാറില്ല. മാതൃമുലപ്പാല്‍ പിഴിഞ്ഞ് വായിലേക്ക് ഉറ്റിച്ചുകൊടുക്കുകയോ സമാന്തര പോഷകങ്ങള്‍ നല്‍കിയോ ആണ് ചികിത്സകന്‍മാര്‍ ഈ പ്രശ്‌നം പരിഹരിക്കാറുള്ളത്. ആറു മാസങ്ങള്‍ പൂര്‍ത്തിയാക്കുന്നതിന് ഏതാനും ദിവസങ്ങള്‍ക്ക് മുമ്പ് പ്രസവിക്കുന്ന കുഞ്ഞുങ്ങളുടെ ജീവന്‍ രക്ഷിക്കാന്‍ കഴിയുമെങ്കിലും മുലകുടിയടക്കമുള്ള പല ശൈശവക്രിയകളും ചെയ്യാന്‍ അവയ്ക്ക് കഴിയുകയില്ലെന്നര്‍ത്ഥം.

മാതാപിതാക്കളോടുള്ള ബാധ്യതകളെക്കുറിച്ചു പറയുമ്പോള്‍ മാതാവ് തനിക്കുവേണ്ടി സഹിച്ച ത്യാഗങ്ങളെപ്പറ്റി ഓര്‍മിപ്പിച്ചുകൊണ്ടാണ് ക്വുര്‍ആന്‍ മുലകുടി പ്രായവും ഗര്‍ഭകാലവും കൂടി മുപ്പത് മാസങ്ങളാണെന്ന് പരാമര്‍ശിക്കുന്നതെന്ന വസ്തുത ശ്രദ്ധേയമാണ്. ആറുമാസമെങ്കിലുമുള്ള പൊക്കിള്‍കൊടി ബന്ധവും രണ്ടു വര്‍ഷത്തെ മുലകുടി ബന്ധവുമാണ് മാതൃശരീരവുമായി കുഞ്ഞിനുള്ള ജൈവികബന്ധമെന്ന ക്വുര്‍ആന്‍ പരാമര്‍ശം വളര്‍ന്നു വലുതായ ശേഷമുള്ള മാതാപിതാക്കളോടുള്ള ബാധ്യതയെക്കുറിച്ച് ഓര്‍മപ്പെടുത്തുന്നതിനിടയിലാണ് കടന്നുവരുന്നത്. മുലകുടി പ്രായവും കുറഞ്ഞ ഗര്‍ഭകാലവും കൂടി മുപ്പത് മാസങ്ങളാണെന്ന ക്വുര്‍ആന്‍ പരാമര്‍ശം ശാസ്ത്രീയമായ കൃത്യത മാത്രമല്ല വൈകാരിക ബന്ധത്തിനുണ്ടാവേണ്ട ആഴവും വ്യക്തമാക്കുന്നതാണ്.

സൂര്യനെ വിളക്കായും ചന്ദ്രനെ പ്രകാശമായും വിശേഷിപ്പിച്ച ക്വുർആൻ സൂര്യൻ പ്രക്സശസ്രോതസ്സാണെന്നും ചന്ദ്രൻ അതിന്റെ പ്രകാശം പ്രതിഫലിപ്പിക്കുകയാണ് ചെയ്യുന്നതെന്നുമാണ് വ്യക്തമാക്കുന്നതെന്നും കൃത്യമായ പരാമർശങ്ങളാണ് ക്വുർആൻ നടത്തുന്നതെന്ന് ഇതിന്റെ അടിസ്ഥാനത്തിൽ വാദിക്കുന്നത് ശുദ്ധ തട്ടിപ്പാനിന്നും യുക്തിവാദികൾ പറയുന്നു. ഖുര്‍ആന്‍ 33:45,46ല്‍ മുഹമ്മദ് നബിയെ വിളക്കായും (സിറാജ്) 24:35ല്‍ അല്ലാഹുവിനെ പ്രകാശമായും (നൂര്‍) ഉപമിച്ചിട്ടുണ്ട്. മുഹമ്മദ് നബിയാണ് പ്രകാശ സ്രോതസ്സെന്നും അല്ലാഹു അദ്ദേഹത്തിന്റെ പ്രകാശം പ്രതിഫലിപ്പിക്കുകയാണെന്നുമല്ലേ നടേ പറഞ്ഞ വ്യാഖ്യാനം അംഗീകരിച്ചാല്‍ വന്നു ചേരുക. യുക്തിവാദികളുടെ വിമര്ശനത്തെപ്പറ്റി എന്ത് പറയുന്നു?

സൂര്യനെ വിളക്കായും ചന്ദ്രനെ പ്രകാശമായും ക്വുർആൻ വിശേഷിപ്പിച്ചിട്ടുണ്ട്. സൂര്യൻ പ്രക്സശസ്രോതസ്സാണെന്ന് അതിനെ വിശേഷിപ്പിച്ച സിറാജ് എന്ന പദം തന്നെ വ്യക്തമാക്കുന്നുണ്ട്. വിളക്ക് എന്നാണ് ആ പദത്തിന്റെ നേർക്ക് നേരെയുള്ള അർഥം. എന്നാൽ ചന്ദ്രനെ വിശേഷിപ്പിച്ച നൂർ എന്ന പദത്തിന് നേർക്ക് നേരെ പ്രതിഫലിക്കപ്പെട്ട പ്രകാശം എന്ന അർത്ഥമില്ല; പ്രകാശം എന്ന് മാത്രമാണ് അതിന്റെ അർത്ഥം . ചന്ദ്രൻ സൂര്യന്റെ പ്രകാശം പ്രതിഫലിപ്പിക്കുകയാണ് ചെയ്യുന്നതെന്ന് നമുക്കറിയാം. ഏത് തരം പ്രകാശമായാലും അതിന് നൂർ എന്ന് പറയും; പ്രതിഫലിക്കപ്പെട്ടതാവട്ടെ അല്ലാത്തതാകട്ടെ. സൂര്യനെക്കുറിച്ച് മാത്രമേ സിറാജ് എന്ന് ക്വുർആൻ പ്രയോഗിച്ചിട്ടുള്ളൂവെന്നും ചന്ദ്രനെക്കുറിച്ച് നൂർ എന്നും മുനീർ എന്നുമാണ് പ്രയോഗിച്ചതെന്നുമുള്ള വസ്തുതകൾ വ്യക്തമാക്കുന്നത് പ്രകാശസ്രോതസ്സാണ് സൂര്യനെന്ന വസ്തുത അറിയാവുന്നവനിൽ നിന്നാണ് അത് അവതരിപ്പിക്കപ്പെട്ടിരിക്കുന്നത് എന്നാണ്. ഇതാണ് ഇവ്വിഷയകമായി ഇസ്‌ലാമിക പ്രബോധകർ പറയാറുള്ളത്. രാത്രിയിൽ കാണുന്ന ചന്ദ്രനെയാണ് വിളക്ക് എന്ന അർത്ഥത്തിൽ സിറാജ് എന്ന് വിളിക്കാൻ ഒരു മരുഭൂനിവാസിക്ക് അനുയോജ്യമെങ്കിലും ഖുർആൻ ഒരിക്കലും ചന്ദ്രനെ അങ്ങനെ വിളിക്കുന്നില്ലെന്നതാണ് അതിലെ പദപ്രയോഗങ്ങളിലെ കൃത്യതയും സൂക്ഷ്മതയും അങ്ങനെ ദൈവികതയും വ്യക്തമാക്കുന്നത്.

ഖുര്‍ആന്‍ 33:45,46ല്‍ മുഹമ്മദ് നബിയെ വിളക്കായും (സിറാജ്) 24:35ല്‍ അല്ലാഹുവിനെ പ്രകാശമായും (നൂര്‍) ഉപമിച്ചിട്ടുണ്ടെന്നും മുഹമ്മദ് നബിയാണ് പ്രകാശ സ്രോതസ്സെന്നും അല്ലാഹു അദ്ദേഹത്തിന്റെ പ്രകാശം പ്രതിഫലിപ്പിക്കുകയാണെന്നുമല്ലേ നടേ പറഞ്ഞ വിശദീകരണം അംഗീകരിച്ചാല്‍ വന്നു ചേരുകയെന്നുമാണ് വിമര്‍ശകർ ചോദിക്കുന്നത്.

ഉദ്ധരിക്കപ്പെട്ട ഖുര്‍ആന്‍ വചനങ്ങള്‍ പരിശോധിക്കുക. ''നബിയേ, തീര്‍ച്ചയായും നിന്നെ നാം ഒരു സാക്ഷിയും സന്തോഷവാര്‍ത്ത അറിയിക്കുന്നവനും, താക്കീതുകാരനും ആയിക്കൊണ്ട് നിയോഗിച്ചിരിക്കുന്നു. അല്ലാഹുവിന്റെ ഉത്തരവനുസരിച്ച് അവങ്കലേക്ക് ക്ഷണിക്കുന്നവനും, പ്രകാശം നല്‍കുന്ന ഒരു വിളക്കും ആയിക്കൊണ്ട്.'' (33:45,46)

''അല്ലാഹു ആകാശങ്ങളുടെയും ഭൂമിയുടെയും പ്രകാശമാകുന്നു. അവന്റെ പ്രകാശത്തിന്റെ ഉപമയിതാ: (ചുമരില്‍ വിളക്ക് വെക്കാനുള്ള) ഒരു മാടം അതില്‍ ഒരു വിളക്ക്. വിളക്ക് ഒരു സ്ഫടികത്തിനകത്ത്. സ്ഫടികം ഒരു ജ്വലിക്കുന്ന നക്ഷത്രം പോലെയിരിക്കുന്നു. അനുഗൃഹീതമായ ഒരു വൃക്ഷത്തില്‍ നിന്നാണ് അതിന് (വിളക്കിന്) ഇന്ധനം നല്‍കപ്പെടുന്നത്. അതായത് കിഴക്ക് ഭാഗത്തുള്ളതോ പടിഞ്ഞാറ് ഭാഗത്തുള്ളതോ അല്ലാത്ത ഒലീവ് വൃക്ഷത്തില്‍ നിന്ന്. അതിന്റെ എണ്ണ തീ തട്ടിയില്ലെങ്കില്‍ പോലും പ്രകാശിക്കുമാറാകുന്നു. (അങ്ങനെ) പ്രകാശത്തിന്‍മേല്‍ പ്രകാശം. അല്ലാഹു തന്റെ പ്രകാശത്തിലേക്ക് താന്‍ ഉദ്ദേശിക്കുന്നവരെ നയിക്കുന്നു. അല്ലാഹു ജനങ്ങള്‍ക്ക് വേണ്ടി ഉപമകള്‍ വിവരിച്ചുകൊടുക്കുന്നു. അല്ലാഹു ഏത് കാര്യത്തെപ്പറ്റിയും അറിവുള്ളവനത്രെ.'' (24:35)

ഈ രണ്ട് വചനങ്ങളും രണ്ട് സ്വതന്ത്ര വചനങ്ങളാണ്; ഒന്ന് മറ്റേതിന്റെ ബാക്കിയോ വിശദീകരണമോ അല്ല. സൂറത്തു അഹ്‌സാബിലെ 45,46 വചനങ്ങള്‍ മുഹമ്മദ് നബി(സ)യുടെ സവിശേഷതകള്‍ വിവരിക്കുകയാണ് ചെയ്യുന്നത്. അദ്ദേഹം സാക്ഷിയും സന്തോഷവാര്‍ത്ത അറിയിക്കുന്നവനും താക്കീതുകാരനും അല്ലാഹുവിന്റെ ഉത്തരവനുസരിച്ച് അവങ്കലേക്ക് ക്ഷണിക്കുന്നവനുമാണ്; അതോടൊപ്പംതന്നെ അദ്ദേഹം ജനങ്ങള്‍ക്ക് പ്രകാശം നല്‍കുന്നവനും സ്വയം തന്നെ പ്രകാശിക്കുന്നവനമാണ്. അത് ക്പന്റാണ് അദ്ദേഹത്തെ സിറാജന്‍ മുനീറാ എന്ന് വിളിച്ചിരിക്കുന്നത്. ഇതൊരു ഉപമാലങ്കാരമാണ്. മുഹമ്മദ് നബി (സ) സ്വയം പ്രകാശിക്കുന്ന വിളക്കാണ് എന്ന കാര്യത്തില്‍ സംശയമൊന്നുമില്ല. അദ്ദേഹത്തിന്റെ കര്‍മ്മങ്ങളും നിര്‍ദ്ദേശങ്ങളും അനുവാദങ്ങളുമെല്ലാം അവസാനനാളുവരെയുള്ള മനുഷ്യര്‍ക്കെല്ലാം വെളിച്ചമായിത്തീരുന്നവയാണ്. മുഹമ്മദ് നബി (സ)യെന്ന വിളക്കില്‍ നിന്ന് പുറപ്പെടുന്ന വെളിച്ചമാണ് സുന്നത്ത്. ഇസ്‌ലാമിന്റെ രണ്ടാമത്തെ പ്രമാണമാണത്. മുഹമ്മദ് നബി (സ) സ്വയം വിളക്കായിത്തീര്‍ന്നതല്ല, പ്രത്യുത അല്ലാഹു അദ്ദേഹത്തെ വിളക്കാക്കിത്തീര്‍ത്തതാണ്. സ്വന്തം ജീവിതത്തിന്റെ പ്രകാശത്തിലൂടെ അവസാനനാളുവരെയുള്ള മുഴുവന്‍ മനുഷ്യര്‍ക്കും വഴികാട്ടിയായിത്തീരുവാനുള്ള വിളക്ക്. കെട്ടുപോയ വിളക്കല്ല അദ്ദേഹം; പ്രകാശം നല്‍കികൊണ്ടിരിക്കുന്ന സജീവമായ വിളക്കാണ്-സിറാജന്‍ മുനീറാ. എത്ര സുന്ദരമായ ഉപമാലങ്കാരം!

അല്ലാഹുവിനെ പരിചയപ്പെടുത്തുന്ന അതിസുന്ദരമായ ഖുര്‍ആന്‍ വചനങ്ങളിലൊന്നാണ് സൂറത്തുന്നൂറിലെ 35ാമത്തെ വചനം. ഇതും ഒരു ഉപമാലങ്കാരമാണ്. ആകാശഭൂമികളുടെ പ്രകാശമാണ് അല്ലാഹു. പ്രപഞ്ചത്തിന് മുഴുവന്‍ വെളിച്ചം നല്‍കുന്ന അവന്റെ പ്രകാശം മറ്റേതെങ്കിലും സ്രോതസ്സില്‍ നിന്ന് വരുന്നതല്ല. അവന്‍തന്നെയാണ് വിളക്കും വിളക്കുമാടവും അത് വെച്ചിരിക്കുന്ന സ്ഫടികക്കൂടുമെല്ലാം. പ്രകാശത്തിനു മേല്‍ പ്രകാശമാണവന്‍. അവന്റെ പ്രകാശത്തിലേക്ക് ആളുകളെ നയിക്കുന്നതും അവന്‍തന്നെ. ഇവിടെ അല്ലാഹുവിനെ കേവല പ്രകാശത്തോടല്ല ഉപമിച്ചിട്ടുള്ളതെന്ന കാര്യം പ്രത്യേകം ശ്രദ്ധേയമാണ്. അവന്‍തന്നെയാണ് വിളക്കും വിളക്കുമാടവും സ്ഫടികക്കൂടുമെല്ലാം എന്ന് വ്യക്തമാക്കുകയും അവന്റെ പ്രകാശത്തിലേക്ക് അവന്‍ തന്നെയാണ് ജനങ്ങളെ നയിക്കുന്നതെന്ന് പഠിപ്പിക്കുകയും ചെയ്യുന്നതാണ് ഈ വചനം. അല്ലാഹുവിനെ എത്ര സുന്ദരമായാണ് ഈ ഉപമയിലൂടെ ഖുര്‍ആന്‍ പരിചയപ്പെടുത്തിയിരിക്കുന്നത്!

സൂറത്തുല്‍ അഹ്‌സാബിലെ വചനം മുഹമ്മദ് നബി(സ)യെയും സൂറത്തുന്നൂറിലെ വചനം അല്ലാഹുവിനെയും സ്വതന്ത്രമായി ഉപമാലങ്കാരത്തിലൂടെ പരിചയപ്പെടുത്തുകയാണ് ചെയ്യുന്നത്. മുഹമ്മദ്‌നബി (സ) വിളക്കും അദ്ദേഹത്തില്‍നിന്നു പുറപ്പെടുന്ന പ്രകാശം അല്ലാഹുവുമാണെന്ന് ഈ വചനങ്ങള്‍ സൂചിപ്പിക്കുന്നുപോലുമില്ല. സ്വയം പ്രകാശിച്ചുകൊണ്ട് മനുഷ്യര്‍ക്ക് വെളിച്ചമാകുവാന്‍ അല്ലാഹു നിയോഗിച്ചതാണ് മുഹമ്മദ് നബി (സ)യെയെന്ന് ഒന്നാമത്തെ വചനവും പ്രപഞ്ചത്തിന്റെ വിളക്കും വെളിച്ചവുമാണ് അല്ലാഹുവെന്ന് രണ്ടാമത്തെ വചനവും വ്യക്തമാക്കുന്നു. സൂര്യനെ വിളക്കും ചന്ദ്രനെ പ്രകാശവുമായി പരിചയപ്പെടുത്തിയ വചനങ്ങളിലാകട്ടെ രണ്ടും ഒരേ വചനത്തില്‍തന്നെ പ്രതിപാദിക്കുകയും ഒന്ന് മറ്റേതിന് ഉപോല്‍ബലകമാണെന്ന് വ്യക്തമാക്കുകയും ചെയ്യുന്നുണ്ട്. സൂര്യനെ സിറാജും ചന്ദ്രനെ നൂറുമായി പരിചയപ്പെടുത്തിയതും അല്ലാഹുവിനെ നൂറും മിസ്വ്ബാഹുമായും മുഹമ്മദ് നബിയെ സിറാജന്‍ മുനീറയായും പരിചയപ്പെടുത്തിയതും തമ്മില്‍ താരതമ്യത്തിനുതന്നെ പറ്റാത്തത്ര വ്യത്യാസമുണ്ടെന്ന് സാരം.

 അബ്രഹാമും ഇശ്മയേലും ഏകദൈവാരാധക്കുവേണ്ടി സ്ഥാപിച്ച പ്രാര്‍ത്ഥനാമന്ദിരത്തോടനുബന്ധിച്ച് പില്‍ക്കാലത്ത് ബഹുദൈവവിശ്വാസപരമായ ചടങ്ങുകള്‍ രൂപം കൊണ്ടതാണെന്നും കഅ്ബയുടെ സാക്ഷാല്‍ ലക്ഷ്യം പ്രപഞ്ചനാഥനെ മാത്രം ആരാധിക്കുകയാണെന്നും പല അറബികള്‍ക്കും പ്രവാചകനിയോഗത്തിന്റെ കാലഘട്ടത്തില്‍പോലും അറിയാമായിരുന്നുവെന്നും അവരാണ് ഹനീഫുകള്‍ എന്നറിയപ്പെട്ടതെന്നും ഉള്ള മുസ്‌ലിം ചരിത്രകാരന്‍മാരുടെ വാദം അടിസ്ഥാനരഹിതമാണ്. ഹനീഫുകള്‍ എന്ന പേരില്‍ ജാഹിലിയ്യ അറബികള്‍ക്കിടയില്‍ ജീവിച്ചിരുന്നവരൊന്നും ഈ വിശ്വാസമുള്ളവരല്ലായിരുന്നുവെന്ന് അവരെക്കുറിച്ചുള്ള നിവേദനങ്ങളില്‍ നിന്ന് സ്പഷ്ടമാണ്. മക്കയുടെ അബ്രഹാമിക പാരമ്പര്യത്തെ നിഷേധിക്കുന്ന ഓറിയന്റലിസ്റ്റ്-മിഷനറി രചനകളില്‍ സര്‍വസാധാരണമായ ഈ വാദങ്ങള്‍ ഹനീഫിയ്യത്തിനെ സംബന്ധിച്ച മുസ്‌ലിം അവകാശവാദത്തെ പുനപരിശോധിക്കാന്‍ പ്രേരിപ്പിക്കുന്നവയല്ലേ?

ല്ല. മക്കയുടെ ഇബ്‌റാഹീമീ പാരമ്പര്യം മുഹമ്മദ് നബി (സ) പറഞ്ഞുണ്ടാക്കിയതാണ് എന്ന വിമര്‍ശനത്തെ എല്ലാ അര്‍ത്ഥത്തിലും കടപുഴക്കുന്നതാണ് പ്രവാചകനിയോഗത്തിനുമുമ്പേ അറേബ്യയിലുണ്ടായിരുന്ന ഹനീഫുകളുടെ സാന്നിധ്യം. അറബികള്‍ ഇബ്‌റാഹീമീ ഏകദൈവാരാധനയില്‍നിന്ന് വ്യതിചലിച്ചുപോയതായി മനസ്സിലാക്കുകയും ബഹുദൈവാരാധനാപരമായ അറബ് അനുഷ്ഠാനങ്ങളോട് വിരക്തി പ്രകടിപ്പിച്ച് ഇബ്‌റാഹീമീ മാര്‍ഗത്തിന്റെ വീണ്ടെടുപ്പ് സ്വന്തം ജീവിതത്തില്‍ ആഗ്രഹിക്കുകയും ചെയ്ത ന്യൂനപക്ഷമാണ് മക്കയിലും പരിസരപ്രദേശങ്ങളിലും ഹനീഫുകള്‍ എന്നു വിളിക്കപ്പെട്ടത് എന്ന് അവരെ സംബന്ധിച്ച നിവേദനങ്ങളെല്ലാം വ്യക്തമാക്കുന്നുണ്ട്. ഇബ്‌റാഹീമീ രക്തത്തോടൊപ്പം ആദര്‍ശവും കുറേയെങ്കിലും അറേബ്യയില്‍ മുഹമ്മദ് നബി(സ)യുടെ കാലം വരെ നിലനിന്നുവെന്ന് ഹനീഫുകളുടെ ചരിത്രം ബോധ്യപ്പെടുത്തുന്നു എന്നതിനാലാണ് ഹനീഫുകള്‍ ഇബ്‌റാഹീമീ നിലപാടുകളുടെ പുനരുജ്ജീവനത്തിന് പരിശ്രമിച്ചവരായിരുന്നില്ലെന്ന് ചില നിവേദനങ്ങളിലെ പരാമര്‍ശങ്ങളുടെ വെളിച്ചത്തില്‍ സ്ഥാപിച്ചെടുക്കാന്‍ ഓറിയന്റലിസ്റ്റുകള്‍ പരിശ്രമിച്ചു നോക്കിയിട്ടുള്ളത്.

വാസ്തവത്തില്‍, അറേബ്യയില്‍ നിലനിന്നിരുന്ന ഹനീഫിയ്യത്തിനെ സംബന്ധിച്ച ചരിത്രനിവേദനങ്ങളുടെ വിശകലനം ഇബ്‌റാഹീം നബി(അ)യുടെ ആദര്‍ശമനുസരിച്ച് ജീവിക്കാനുളള അദമ്യമായ ആഗ്രഹമാണ് ഹനീഫുകളെ വ്യതിരിക്തരാക്കിയത് എന്നുതന്നെയാണ് ബോധ്യപ്പെടുത്തുന്നത്. ഇബ്‌റാഹീമിലേക്ക് മടങ്ങുവാനുള്ള ത്വര മുഹമ്മദ് നബി (സ) പുതിയ ദേശപാരമ്പര്യം മെനഞ്ഞുണ്ടാക്കി മക്കക്കാരില്‍ കൃത്രിമമായി സന്നിവേശിപ്പിച്ചതാണെന്ന വിമര്‍ശക വീക്ഷണം പ്രസ്തുത നിവേദനങ്ങള്‍ക്കുമുന്നില്‍ ഒരിക്കലും നിലനില്‍ക്കുകയില്ല. തങ്ങള്‍ തുടര്‍ന്നുകൊണ്ടിരിക്കുന്ന വിശ്വാസ-കര്‍മ മാര്‍ഗം പൂര്‍ണമായും ഇബ്‌റാഹീമിന്റെയും ഇസ്മാഈലിന്റേതുമാണ് എന്ന് വലിയൊരു വിഭാഗം അറബികള്‍ തെറ്റിദ്ധരിച്ചപ്പോഴും അങ്ങനെയല്ലെന്നും അതില്‍ കലര്‍പ്പുകള്‍ വന്നിട്ടുണ്ടെന്നും ശരിയായി തിരിച്ചറിഞ്ഞ ഒറ്റപ്പെട്ട വ്യക്തികളായിരുന്നു ഹനീഫുകള്‍. അല്ലാഹുവിനു മാത്രം ആരാധനകള്‍ സമര്‍പ്പിക്കണമെന്നു വാദിച്ചിരുന്നതുകൊണ്ടാണ് അവര്‍ ഹനീഫുകള്‍ (ഋജുമാനസ്‌കര്‍-കലര്‍പ്പുകള്‍ അനുവദിക്കാത്തവര്‍) എന്നറിയപ്പെട്ടത്.

മക്കന്‍ മുഖ്യധാരയോട് കലഹിച്ച് ഇബ്‌റാഹീമീ സരണിയോട് വിഗ്രഹാരാധനയും അറബ് അനാചാരങ്ങളും ഒത്തുപോവുകയില്ലെന്ന് ഒറ്റയാനായി പ്രഖ്യാപിച്ച സയ്ദ്ബ്‌നു അംറുബ്‌നു നുഫയ്ല്‍ ആണ് ചരിത്രത്തിലെ ഏറ്റവും പ്രഖ്യാതനായ ഹനീഫ്. മുഹമ്മദ് നബി(സ)യുടെ സമകാലീനനായിരുന്നെങ്കിലും അദ്ദേഹത്തിന്റെ പ്രവാചകത്വത്തിനുമുമ്പ് മരണപ്പെട്ടുപോയ വ്യക്തിയായിരുന്നു സയ്ദ് എന്നാണ് മനസ്സിലാകുന്നത്. മക്കന്‍ വിഗ്രഹാരാധന ശരിയല്ലെന്ന് മനസ്സിലാക്കി ശരിയായ ദൈവികപാത തേടി സിറിയയിലേക്കടക്കം യാത്ര പോയ സയ്ദിന് ജൂത, ക്രൈസ്തവ പണ്ഡിതരടക്കം ഉപദേശിച്ചുകൊടുത്തത് ഇബ്‌റാഹീമീ ഹനീഫിയ്യത്തായിരുന്നുവെന്ന് സ്വഹീഹുല്‍ ബുഖാരിയിലെ തീര്‍ത്തും പ്രബലമായ നിവേദനത്തിലുണ്ട്. സിറിയയില്‍ നിന്നു മടങ്ങിയപ്പോള്‍ അദ്ദേഹം കൈകളുയര്‍ത്തി ”എന്റെ രക്ഷിതാവേ, ഞാന്‍ ഇബ്‌റാഹീമിന്റെ മതത്തിലാണെന്നതിന് നീ സാക്ഷ്യം വഹിച്ചുകൊള്ളുക” എന്ന് പ്രഖ്യാപിച്ചതായി ഇബ്‌നു ഉമര്‍ (റ) റിപ്പോര്‍ട്ട് ചെയ്തിട്ടുണ്ട് (ബുഖാരി). താന്‍ അല്ലാഹുവിനെ മാത്രമേ ആരാധിക്കുകയുള്ളൂവെന്നും അതാണ് ഇബ്‌റാഹീമിന്റെ ശരിയായ മതം എന്നും താനാണ് അതില്‍ നിലനില്‍ക്കുന്നതെന്നും ഇബ്‌റാഹീമിന്റെ പൈതൃകം അവകാശപ്പെടുമ്പോഴും മറ്റു മക്കക്കാര്‍ അദ്ദേഹത്തിന്റെ മതത്തില്‍ നിന്നും വ്യതിചലിച്ചുപോയിരിക്കുന്നുവെന്നും വ്യക്തമാക്കിക്കൊണ്ട് സയ്ദ്ബ്‌നു അംറ് കഅ്ബയുടെ ചാരത്തുനിന്ന് ”ഖുറയ്ശികളേ, സയ്ദിന്റെ ആത്മാവ് ആരുടെ കയ്യിലാണോ, അവനാണ് (അല്ലാഹു) സത്യം; നിങ്ങളിലൊരാളുമല്ല, മറിച്ച് ഞാനാണ് ഇബ്‌റാഹീമിന്റെ മതത്തിലുള്ളത്” (മാ അസ്ബ്ഹ മിന്‍കും അഹദുന്‍ അലാ ദീനി ഇബ്‌റാഹീമ ഗ്വയ്‌രീ) എന്നു പ്രഖ്യാപിച്ചത് ഇബ്‌നു ഇസ്ഹാക്വിന്റെ സീറയിലുണ്ട് (Guillaume, 99-100).

താന്‍ ഇബ്‌റാഹീമിന്റെ മാര്‍ഗം തെരഞ്ഞെടുക്കുന്നു എന്ന് സയ്ദ് പറയുന്നത് ആ മാര്‍ഗം മക്കക്കാര്‍ നേരത്തെ അവകാശപ്പെട്ടുകൊണ്ടിരുന്നതാണ് എന്ന അവബോധത്തോടെയാണ് എന്ന് അദ്ദേഹത്തിന്റെ വാക്കുകള്‍ സുതരാം വ്യക്തമാക്കുന്നുണ്ട്. ഹനീഫിയ്യാ ഏകദൈവാരാധനാനിഷ്ഠ സ്വീകരിച്ചതിന്റെ ഫലമായി, അദ്ദേഹം വിഗ്രഹാരാധനയും വിഗ്രഹങ്ങള്‍ക്ക് നിവേദിക്കപ്പെട്ട ഭക്ഷണം കഴിക്കുന്നതും പൂര്‍ണമായി ഉപേക്ഷിച്ചുവെന്നും ഇബ്‌നു ഇസ്ഹാക്വ് രേഖപ്പെടുത്തുന്നു (Ibid, p. 99). മദീനയില്‍ പ്രവാചകാഗമനത്തിനു മുമ്പുതന്നെ ഹനീഫ് ആയി ജീവിച്ചിരുന്ന ബനൂ അദിയ്യ ഗോത്രക്കാരന്‍ അബൂ ക്വയ്‌സ് ബിന്‍ അബൂ അനസിന്റെയും കഥ ഏതാണ്ട് സമാനം തന്നെയാണ്. വിഗ്രഹങ്ങളുപേക്ഷിക്കുകയും ജൂതനോ ക്രൈസ്തവനോ ആകുന്നതിനുപകരം ശുദ്ധമായ ഇബ്‌റാഹീമീ സരണി പുല്‍കുന്നുവെന്ന് പ്രഖ്യാപിക്കുകയും ചെയ്ത അദ്ദേഹം ”ഞാന്‍ ഇബ്‌റാഹീമിന്റെ നാഥനെയാണ് ആരാധിക്കുന്നത്” എന്ന് വിശദീകരിച്ച് പ്രാര്‍ത്ഥന നിര്‍വഹിക്കുവാന്‍ വേണ്ടി ഒരു ആരാധനാലയം പണിതതായി ഇബ്‌നു ഇസ്ഹാക്വില്‍ തന്നെയുണ്ട്. ഇദ്ദേഹം നബി (സ) മദീനയിലെത്തിയപ്പോള്‍ ഇസ്‌ലാം സ്വീകരിച്ചു (Ibid, pp. 236-9).

സയ്ദിനെയും അബൂക്വയ്‌സിനെയും സംബന്ധിച്ചുള്ള നിവേദനങ്ങള്‍ വ്യക്തമാക്കുന്ന ഒരു കാര്യം, ഇബ്‌റാഹീം നബി(അ)യും ഇസ്മാഈല്‍ നബി(അ)യും പഠിപ്പിച്ചിരുന്നത് ശുദ്ധ ഏകദൈവാരാധനയാണെന്നും എന്നാല്‍ അറബികള്‍ അവരുടെ മാര്‍ഗത്തില്‍നിന്ന് വ്യതിചലിച്ച് വിഗ്രഹാരാധനയിലും അധാര്‍മികതകളിലും എത്തിപ്പെട്ടു എന്നുമുള്ള അടിസ്ഥാന ബോധ്യങ്ങളാണ് അവര്‍ക്കുണ്ടായിരുന്നത് എന്നാണ്. ഇബ്‌റാഹീമീ സരണിക്ക് നിരക്കുന്നതല്ലെന്ന് തങ്ങള്‍ക്ക് ബോധ്യം വന്ന തിന്മകളില്‍ നിന്ന് വിട്ടുനില്‍ക്കുകയും സാധ്യമാകുന്ന തരത്തില്‍ ഏകദൈവാരാധന നിര്‍വഹിക്കുകയും ചെയ്ത് മനസ്സാക്ഷിയെ തൃപ്തിപ്പെടുത്താന്‍ ശ്രമിച്ചവരായിരുന്നു അവര്‍. അതല്ലാതെ, അല്ലാഹുവിനുവേണ്ടി നിര്‍വഹിക്കേണ്ടുന്ന ആരാധനകളുടെ വിശദമായ കര്‍മശാസ്ത്രത്തെക്കുറിച്ചോ അനുഷ്ഠിക്കേണ്ട സല്‍പ്രവര്‍ത്തനങ്ങളെക്കുറിച്ചോ ജീവിതവിശുദ്ധി നിലനിര്‍ത്താന്‍ ഉപേക്ഷിക്കേണ്ട തിന്മകളെക്കുറിച്ചോ കൃത്യവും സമഗ്രവുമായ ധാരണകളൊന്നും അവര്‍ക്കുണ്ടായിരുന്നില്ല. വഹ്‌യ് ലഭിക്കുന്ന ഒരു പ്രവാചകന്റെ അസാന്നിധ്യമായിരുന്നു ഈ പ്രതിസന്ധിക്കു കാരണം.

അബൂക്വയ്‌സും സയ്ദും ആര്‍ത്തവകാരികളുമായുള്ള ലൈംഗിക ബന്ധത്തില്‍നിന്ന് വിട്ടുനിന്നതും സയ്ദ് ശവവും രക്തവും ഭക്ഷിക്കുന്നത് ഒഴിവാക്കിയതും പെണ്‍മക്കളെ ജീവനോടെ കുഴിച്ചുമൂടുന്നതിനെ എതിര്‍ത്തതുമെല്ലാം ഇബ്‌നു ഇസ്ഹാക്വ് വിവരിക്കുന്നുണ്ട്. ഇവയെല്ലാം അവരുടെ അന്വേഷണങ്ങളില്‍ നിന്ന് അവരെത്തിപ്പെട്ട ധാര്‍മിക നിലപാടുകളായിരുന്നു. കുറേക്കൂടി നിഷ്‌കൃഷ്ടമായ മാര്‍ഗദര്‍ശനത്തിനുവേണ്ടി അവര്‍ ദാഹിച്ചിരുന്നുവെന്ന് സയ്ദ്ബ്‌നു അംറിന്റെ വാക്കുകള്‍ വ്യക്തമാക്കുന്നുണ്ട്. അദ്ദേഹം പറഞ്ഞു : ”എന്റെ രക്ഷിതാവേ, നിന്നെ ആരാധിക്കാനുള്ള കൂടുതല്‍ നല്ല മാര്‍ഗങ്ങള്‍ അറിയുമായിരുന്നുവെങ്കില്‍ ഞാനത് സ്വീകരിക്കുമായിരുന്നു; പക്ഷേ എന്തു ചെയ്യാം, എനിക്കതറിയില്ല!” തുടര്‍ന്ന് കഅ്ബക്കുനേരെ തിരിഞ്ഞ് അല്ലാഹുവിനു സുജൂദ് ചെയ്ത് അദ്ദേഹം വാക്കുകള്‍ ഇങ്ങനെ മുഴുമിപ്പിച്ചു: ”എന്റെ നാഥന്‍ ഇബ്‌റാഹീമിന്റെ നാഥനാണ്, എന്റെ മതം ഇബ്‌റാഹീമിന്റെ മതവുമാണ്.” (Ibid, p. 100; Dr. Mahdi  Rizqullah Ahmad, A Biography of the Prophet of Islam in the light of Original Sources  (Riyadh: Darussalam, 2005), p. 58).

മക്കയിലും മദീനയിലുമുണ്ടായിരുന്ന ഹനീഫുകളായി അറിയപ്പെട്ടിരുന്ന  ചില വ്യക്തികള്‍ നബി(സ)യുടെ പ്രവാചകത്വം അംഗീകരിക്കുവാന്‍ വിസമ്മതിക്കുകയും അദ്ദേഹത്തോട് ആശയപരമായ വിയോജിപ്പുകള്‍ രേഖപ്പെടുത്തുകയും ചെയ്തതായി പറയുന്ന ചില നിവേദനങ്ങളുടെ അടിസ്ഥാനത്തിലാണ് വിമര്‍ശകര്‍ ഹനീഫുകള്‍ ഇബ്‌റാഹീമീ മാര്‍ഗത്തിന്റെ പുനരുജ്ജീവനത്തിനു ശ്രമിച്ചവരല്ലായിരുന്നു എന്ന് സ്ഥാപിക്കുവാന്‍ ശ്രമിക്കുന്നത്. അവര്‍ക്കുള്ള മറുപടി സയ്ദിന്റെ വാക്കുകളില്‍ തന്നെയുണ്ട് എന്നുള്ളതാണ് വാസ്തവം. ഇബ്‌റാഹീമീ ഏകദൈവാരാധനയുടെ അടിസ്ഥാനമൂല്യങ്ങള്‍ ഉള്‍ക്കൊണ്ടിരുന്നവരെല്ലാം അറേബ്യയില്‍ ഹനീഫുകളായാണ് അറിയപ്പെട്ടിരുന്നത്. അവരില്‍ നബി(സ)യുടെ പ്രവാചകത്വം അംഗീകരിച്ചവരും നിഷേധിച്ചവരുമുണ്ടാകാം, പ്രവാചകന്‍ സ) പ്രബോധനം ചെയ്ത ധാര്‍മിക പദ്ധതിയുടെ വിശദാംശങ്ങളോട് യോജിച്ചവരും വിയോജിച്ചവരുമുണ്ടാകാം, ഏകദൈവാരാധന മനസ്സിലുള്‍ക്കൊണ്ടാല്‍ മതിയെന്നും നബി (സ) ചെയ്യുന്നതുപോലെ സമൂഹത്തില്‍ അത് വ്യാപകമായി പ്രചരിപ്പിക്കേണ്ടതില്ലെന്നും കരുതിയ ആദര്‍ശ പ്രതിബദ്ധത കുറഞ്ഞ വ്യക്തികളുമുണ്ടാകാം. ഹനീഫുകള്‍ ആരാണെന്ന് മനസ്സിലാക്കിയവര്‍ക്ക് ചരിത്രപരമായി ഇവയിലൊന്നും യാതൊരു അസാംഗത്യവും അനുഭവപ്പെടുകയില്ല.

ഏകശിലാത്മകമായ ഒരു സമൂഹമായിരുന്നില്ല ഹനീഫുകളുടേത്; മറിച്ച് ഇബ്‌റാഹീമീ ഏകദൈവാരാധനയെക്കുറിച്ച് നിശ്ചയവും വ്യക്തതയുമുണ്ടായിരുന്ന, എന്നാല്‍ അനുബന്ധങ്ങളില്‍ ആശയക്കുഴപ്പങ്ങളും അഭിപ്രായാന്തരങ്ങളുമുണ്ടായിരുന്ന ഒറ്റയും തെറ്റയുമായ വ്യക്തിത്വങ്ങളാണ്. അതുകൊണ്ടുതന്നെ വിമര്‍ശകര്‍ എടുത്തുദ്ധരിക്കുന്ന നിവേദനങ്ങള്‍ നിദാനശാസ്ത്രപരമായി ആധികാരികമാണെങ്കിലും അല്ലെങ്കിലും, അവരുടെ വാദം സ്ഥാപിക്കുവാന്‍ പര്യാപ്തമായവയല്ല എന്നതാണ് വാസ്തവം. ചില ഹനീഫുകള്‍ പ്രവാചകന്റെ കൂടെ നിന്നില്ല എന്നുമാത്രമാണ് പരാമൃഷ്ട നിവേദനങ്ങള്‍ വ്യക്തമാക്കുന്നത്. എന്നാല്‍ ഇബ്‌റാഹീം നബി(അ)യുടെ ആശയങ്ങളിലേക്ക് മടങ്ങിപ്പോകണമെന്ന കാര്യത്തില്‍ അവര്‍ക്ക് നബി(സ)യുമായി യാതൊരു അഭിപ്രായവ്യത്യാസവുമുണ്ടായിരുന്നില്ല  എന്ന് അതേ നിവേദനങ്ങള്‍ തന്നെ വ്യക്തമാക്കുന്നുണ്ട്. മക്കയില്‍ പൂര്‍വപിതാക്കളായ ഇബ്‌റാഹീമിെനയും ഇസ്മാഈലിനെയും കുറിച്ചുള്ള ബോധ്യവും അവരുടെ ചര്യകള്‍ മുറുകെപ്പിടിക്കണമെന്ന വികാരവും പ്രവാചകന്‍ (സ) പുതുതായി സൃഷ്ടിച്ചെടുത്തതല്ലെന്ന് എല്ലാ അര്‍ത്ഥത്തിലും സ്ഥാപിക്കുന്നവയാണ് ആ ഉദ്ധരണികള്‍. ഹനീഫുകള്‍ എന്നാല്‍ മുഹമ്മദീയ ഇസ്‌ലാമിന്റെ എല്ലാ ആശയങ്ങളും അംഗീകരിച്ചവരായിരുന്നു എന്ന് മുസ്‌ലിംകള്‍ വാദിക്കുന്നതായി തെറ്റിദ്ധരിപ്പിച്ചുകൊണ്ടാണ് വിമര്‍ശകര്‍ അവയുടെ വിശകലനം നിര്‍വഹിക്കുന്നത്. ഈ തെറ്റിദ്ധാരണയില്‍ നിന്ന് മോചിതരായാല്‍  വിമര്‍ശകര്‍ സ്വന്തം അടിസ്ഥാനങ്ങളെയാണ് തകര്‍ത്തുകളയുന്നത് എന്ന് ആര്‍ക്കും ബോധ്യമാകും. മദീനയിലെ ഔസ് ഗോത്രത്തിന്റെ നേതാക്കളിലൊരാളായിരുന്ന അബൂ ആമിര്‍ അംറുബ്‌നു സയ്ഫും ഔസ് ഗോത്രക്കാരന്‍ തന്നെയായിരുന്ന കവി അബുക്വയ്‌സ് ബിന്‍ അസ്‌ലതും ത്വാഇഫുകാരനായ ഉമയ്യയുമാണ് ഹനീഫുകളായി അറിയപ്പെട്ടിരുന്ന, ഇസ്‌ലാം സ്വീകരിച്ചിട്ടില്ലെന്ന് ചില നിവേദനങ്ങള്‍ പറയുന്ന വ്യക്തികള്‍. അബൂ ആമിറും നബി(സ)യും തമ്മില്‍ മദീനയില്‍വെച്ച് നേരില്‍ കണ്ടുമുട്ടിയപ്പോള്‍ അവര്‍ തമ്മില്‍ നടന്ന സംഭാഷണം ഇബ്‌നു ഇസ്ഹാക്വിന്റെ, വിമര്‍ശകര്‍ ആശ്രയിക്കുന്ന നിവേദനത്തില്‍ തന്നെയുണ്ട്. ഏതു മതവുമായാണ് പ്രവാചകന്‍ (സ) നിയോഗിക്കപ്പെട്ടതെന്ന അബൂ ആമിറിന്റെ ചോദ്യത്തിന് ‘ഹനീഫിയ്യ; ഇബ്‌റാഹീമിന്റെ മതം’ എന്ന് നബി (സ) മറുപടി പറഞ്ഞപ്പോള്‍ താനും ആ മതത്തില്‍ തന്നെയാണ് എന്നായിരുന്നു അബൂ ആമിറിന്റെ പ്രത്യുത്തരം. അബൂ ആമിര്‍ ഇബ്‌റാഹീമിന്റെ മതം ശരിയായി പിന്തുടരുന്നില്ലെന്ന് പറഞ്ഞ പ്രവാചകന്‍(സ)യോട് അദ്ദേഹം പറഞ്ഞത് ‘ഹനീഫിയ്യത്തില്‍ ഇല്ലാത്ത പലതും, മുഹമ്മദ്, താങ്കള്‍ അതില്‍ കൂട്ടിച്ചേര്‍ത്തിരിക്കുന്നു’ (ഇന്നക്ക അദ്ഖല്‍ത, യാ മുഹമ്മദ്, ഫില്‍ ഹനീഫിയ്യ മാ ലയ്‌സ മിന്‍ഹാ) എന്നായിരുന്നുവെന്നും ഇബ്‌റാഹീമിന്റെ മതത്തെ അതിന്റ ശുദ്ധതയില്‍ അപ്പടി പ്രബോധനം ചെയ്യുക മാത്രമാണ് താന്‍ ചെയ്യുന്നതെന്ന് നബി (സ) ഇതിനോട് പ്രതിവചിച്ചുവെന്നും ഇബ്‌നു ഇസ്ഹാക്വ് രേഖപ്പെടുത്തുന്നു. ഇബ്‌റാഹീമീ സരണി പിന്തുടരുന്നുവെന്ന് അവകാശപ്പെട്ടതിനാല്‍ തന്നെയാണ് അബൂ ആമിര്‍ ഹനീഫ് ആയി അറയപ്പെട്ടതെന്നും ഇബ്‌റാഹീമിന്റെ മാര്‍ഗത്തിന്റെ വിശദാംശങ്ങളില്‍ എന്തെല്ലാം വരുമെന്ന കാര്യത്തിലാണ് അദ്ദേഹത്തിന് പ്രവാചകനുമായി അഭിപ്രായവ്യത്യാസമുണ്ടായതെന്നും നിവേദനത്തില്‍ പരാമര്‍ശിക്കപ്പെട്ടിരിക്കുന്ന സംവാദത്തില്‍ നിന്ന് വ്യക്തമാണ്. അറേബ്യയില്‍ പ്രവാചകനുമുമ്പേ ഇബ്‌റാഹീമീ വികാരങ്ങള്‍ നിലനിന്നിരുന്നുവെന്നതിന്റെ സൂചകമാണ് ഫനീഫുകള്‍ എന്ന നിരീക്ഷണത്തെ ഈ നിവേദനം എങ്ങനെ തകര്‍ക്കുമെന്നാണ് വിമര്‍ശകര്‍ പറയുന്നത്? രണ്ടാമത്തെയാളായ അബൂ ക്വയ്‌സ് ബിന്‍ അസ്‌ലത്, ‘ഞാന്‍ ഇബ്‌റാഹീമിന്റെ മതം പിന്തുടരുന്നു; മരണം വരെ ഞാനതില്‍ നിന്ന് പിന്‍മാറുകയില്ല’ എന്ന് പ്രസ്താവിക്കുമായിരുന്നുവെന്ന് ഓറിയന്റലിസ്റ്റുകള്‍ ആശ്രയിക്കുന്ന ഇബ്‌നു സഅദിന്റെ നിവേദനത്തില്‍ തന്നെയുണ്ട്. ഹനീഫിയ്യത്തിന് ഏകദൈവാരാധനാ നിലപാടുകളുമായി ബന്ധമുണ്ടായിരുന്നില്ലെന്ന വിമര്‍ശകവാദത്തെ എല്ലാ അര്‍ത്ഥത്തിലും പൊളിച്ചുകളയുന്നതാണ് ഉമയ്യയെക്കുറിച്ചുള്ള നിവേദനങ്ങളെല്ലാം. അദ്ദേഹം വിഗ്രഹാരാധനയില്‍ നിന്ന് വിട്ടുനില്‍ക്കുകയും അതിന്റെ പേരില്‍ ജൂതനായിപ്പോലും വിചാരിക്കപ്പെടുകയും ചെയ്ത, ഇബ്‌റാഹീമിന്റെയും ഇസ്മാഈലിന്റെയും യഥാര്‍ത്ഥ പൈതൃകം ഹനീഫിയ്യത്താണെന്ന് സ്ഥാപിച്ചുകൊണ്ട് സമൃദ്ധമായി കവിതകളെഴുതിയ ആളായിരുന്നുവെന്നാണ് ചരിത്രഗ്രന്ഥങ്ങള്‍ പറയുന്നത്. അദ്ദേഹത്തിന്റെ ‘ഹനീഫീ’ കവിതള്‍ അറബ് വാമൊഴി പാരമ്പര്യത്തില്‍ സജീവമായി നിലനിന്നതുകൊണ്ടുതന്നെ, ജാഹിലിയ്യാ കാലഘട്ടം മുതല്‍ ഒന്‍പതാം നൂറ്റാണ്ടുവരെയുള്ള അറബിക്കവിതകളുടെ ബൃഹദ്‌ശേഖരം അരനൂറ്റാണ്ടു കാലത്തെ അധ്വാനംകൊണ്ട് അബുല്‍ ഫറജ് ഇസ് ഫഹാനി സി. ഇ പത്താം നൂറ്റാണ്ടില്‍ അഗാനി എന്ന പേരില്‍ പ്രസിദ്ധീകരിച്ചപ്പോള്‍ അതില്‍പോലും ഇടം പിടിച്ചിട്ടുണ്ട്. വിമര്‍ശകര്‍ക്കാവശ്യമുള്ളതൊന്നും ഉമയ്യയെ വിശകലനം ചെയ്തതുകൊണ്ട് ലഭിക്കുകയില്ലെന്ന് ഇതില്‍ നിന്നുതന്നെ വ്യക്തമാണ്. ”ഹനീഫിയ്യത്ത് സത്യമാണെന്നെനിക്കറിയാം; എന്നാല്‍ മുഹമ്മദിന്റെ കാര്യത്തിലാണ് എനിക്ക് തീര്‍ച്ചയില്ലാത്തത്” (വ അന അഅ്‌ലമു അന്നല്‍ ഹനീഫിയ്യ ഹക്വുന്‍ വലാകിന്നശ്ശക്ക്വ യുദാഖിലുനീ ഫീ മുഹമ്മദ്) എന്നാണ് ഉമയ്യ പ്രവാചകനുമായുള്ള തന്റെ അഭിപ്രായവ്യത്യാസത്തെക്കുറിച്ച് പറഞ്ഞത് (ഫത്ഹുല്‍ബാരി). മുഹമ്മദ് നബി(സ)യുടെ പ്രവാചകത്വം അംഗീകരിക്കാന്‍ വിമുഖത കാണിച്ചുവെന്നതുകൊണ്ട് ഉമയ്യ ഇബ്‌റാഹീമീ ആദര്‍ശ പിന്തുടര്‍ച്ച അവകാശപ്പെട്ടിരുന്നുവെന്ന വസ്തുത എങ്ങനെയാണ് ഇല്ലാതാവുക? അറബികള്‍ക്കിയടിലുണ്ടായിരുന്ന ഇബ്‌റാഹീമീ ബോധത്തെ അദ്ദേഹത്തെ സംബന്ധിച്ച നിവേദനങ്ങള്‍ സാധൂകരിക്കുകയല്ലാതെ എങ്ങനെയാണ് നിരാകരിക്കുക? ചുരുക്കത്തില്‍, ഹനീഫുകളെക്കുറിച്ചുള്ള ചരിത്ര/ഹനീഥ് നിവേദനങ്ങള്‍ അവര്‍ ഇബ്‌റാഹീമീ ആദര്‍ശ വ്യതിരിക്തത അവകാശപ്പെട്ടിരുന്നവരല്ല എന്ന് സൂചിപ്പിക്കുന്നുവെന്ന വിമര്‍ശകരുടെ വാദം ഒരു കഴമ്പുമില്ലാത്തതാണ്.

 മക്കന്‍ ബഹുദൈവാരാധക പാരമ്പര്യത്തില്‍ വികസിച്ചുവന്ന ഒരു ശിലാപൂജക ക്ഷേത്രമായിരുന്നു വാസ്തവത്തില്‍ കഅ്ബ. തെക്കനറേബ്യന്‍ വിഗ്രഹാരാധകരുടെ മുന്‍കയ്യില്‍ ശിലാപൂജക്കുവേണ്ടി സ്ഥാപിക്കപ്പെട്ട ക്ഷേത്രമാണ് കഅ്ബയെന്ന് ചരിത്രരേഖകളും കഅ്ബയില്‍ ഇപ്പോഴും സ്ഥിതിചെയ്യുന്ന ഹജറുല്‍ അസ്‌വദും വ്യക്തമാക്കുന്നുണ്ട്. ഒരു പ്രാകൃത ബഹുദൈവാരാധനാ ശേഷിപ്പിനെയാണ് പ്രവാചകന്‍ മുസ്‌ലിംകളുടെ ക്വിബ്‌ലയായി പ്രഖ്യാപിച്ചത് എന്ന് മനസ്സിലാക്കാനേ അവയുടെ വെളിച്ചത്തില്‍ ചരിത്രാന്വേഷകര്‍ക്ക് കഴിയൂ. കഅ്ബ ഇബ്‌റാഹീമും ഇസ്മാഈലും സ്ഥാപിച്ചതല്ലെന്ന് സമര്‍ഥിക്കാന്‍ ചില ഓറിയന്റലിസ്റ്റുകളും മിഷനറിമാരും ഉന്നയിച്ചിട്ടുള്ള ഈ വാദം ശരിയാണോ?

ല്ല. തെക്കേ അറേബ്യയിലെ വിഗ്രഹാരാധകര്‍ ശിലാപൂജക്കുവേണ്ടി സ്ഥാപിച്ച ആരാധനാലയമാണ് കഅ്ബയെന്ന് ‘ചരിത്രരേഖകള്‍’ വ്യക്തമാക്കുന്നുവെന്ന് അവകാശപ്പെടുന്ന ഇസ്‌ലാം വിമര്‍ശകരൊന്നും തന്നെ ഒരു ‘ചരിത്രരേഖ’യും നാളിതുവരെ ഇവ്വിഷയകമായി ഹാജരാക്കിയിട്ടില്ല എന്നതാണ് സത്യം. ദക്ഷിണ അറേബ്യ എന്ന ഭൂമിശാസ്ത്രപരമായ മേല്‍വിലാസം സ്ഥാപിക്കുന്നതുപോയിട്ട് ഏതെങ്കിലും തരത്തിലുള്ള വിഗ്രഹാരാധനക്കുവേണ്ടി ആരെങ്കിലും നിര്‍മിച്ചതാണ് കഅ്ബയെന്ന് സൂചിപ്പിക്കുകയെങ്കിലും ചെയ്യുന്ന ഒരു ചരിത്രരേഖയും ഇല്ല. വിഖ്യാതനായ ഗ്രീക്ക് ചരിത്രകാരന്‍ ഹെറഡോട്ടസ് ‘അലിലത്’ എന്നുപേരുള്ള ഒരു അറേബ്യന്‍ ദേവതയെക്കുറിച്ച് പരാമര്‍ശിക്കുന്നുവെന്നതാണ് കഅ്ബ വിഗ്രഹാരാധകര്‍ സ്ഥാപിച്ചതാണെന്ന് തെളിയിക്കാന്‍ പല ഇസ്‌ലാം വിമര്‍ശകരും എടുത്തുദ്ധരിച്ചിരിക്കുന്നത്. പ്രവാചകന്റെ കാലത്ത് ത്വാഇഫില്‍ പൂജിക്കപ്പെട്ടിരുന്ന ലാത്ത എന്ന വിഗ്രഹമാണ് ഹെറഡോട്ടസിന്റെ പരാമര്‍ശത്തിന് വിധേയമായതെന്ന് അവരില്‍ ചിലര്‍ വ്യാഖ്യാനിക്കുകയും ചെയ്തിട്ടുണ്ട്.

പുരാതനകാലം മുതല്‍ക്കുതന്നെ കഅ്ബ വിഗ്രഹാരാധകരുടെ ദേവാലയമായിരുന്നുവെന്ന് ഹെറഡോട്ടസിന്റെ എഴുത്തുകള്‍ തെളിയിക്കുന്നുവെന്നാണ് വിമര്‍ശകരുടെ വാദം. ഇവിടെ, ഒന്നാമതായി മക്കയെക്കുറിച്ചോ കഅ്ബയെക്കുറിച്ചോ ഹെറഡോട്ടസ് യാതൊരു പ്രസ്താവനയും നടത്തുന്നില്ല എന്നതാണ് സത്യം. ഹെറഡോട്ടസിന്റെ ‘അലിലത്ത്’ ‘ലാത്ത’യാണെങ്കില്‍ ത്വാഇഫിനെക്കുറിച്ചാണ് അദ്ദേഹത്തിന്റെ പരാമര്‍ശം എന്നാണ് വരിക. അങ്ങനെയാണെങ്കിലും അല്ലെങ്കിലും, അറേബ്യയുടെ ഒരു ഭാഗത്ത് ബി.സി. ഇ അഞ്ചാം നൂറ്റാണ്ടില്‍ ഒരു വിഗ്രഹം ദേവതയായി ആരാധിക്കപ്പെടുകയും പൂജിക്കപ്പെടുകയും ചെയ്തിരുന്നുവെന്ന് മാത്രമാണ് അദ്ദേഹത്തിന്റെ വിവരണം തെളിയിക്കുന്നത്. ഇതിന് മക്കയുമായും കഅ്ബയുമായും എന്തു ബന്ധമാണുള്ളത്? കഅ്ബ വിഗ്രഹാരാധനക്കുവേണ്ടി പടുത്തുയര്‍ത്തപ്പെട്ട ഭവനമാണെന്ന  വാദത്തിന് അതെങ്ങനെയാണ് തെളിവാകുക?

രണ്ടാമതായി, ഹെറഡോട്ടസ് ഇനി സാക്ഷാല്‍ കഅ്ബയെക്കുറിച്ചുതന്നെ ഇതേ കാര്യം എഴുതിയാലും അദ്ദേഹത്തിന്റെ കാലമായപ്പോഴേക്കും കഅ്ബക്കു ചുറ്റുമുള്ള ജനത ദേവതാപൂജ ആരംഭിച്ചുവെന്നല്ലാതെ കഅ്ബ സ്ഥാപിക്കപ്പെട്ടത് അന്നായിരുന്നുവെന്നും അതിനുവേണ്ടിയായിരുന്നുവെന്നും എങ്ങനെയാണ് സ്ഥാപിക്കപ്പെടുക? ഇബ്‌റാഹിം നബി(അ)യാണ് കഅ്ബ സ്ഥാപിച്ചതെന്ന പ്രവാചകാധ്യാപനത്തെ അതെങ്ങനെയാണ് തിരുത്തുക? ബൈബിള്‍ കാലഗണന പ്രകാരം ഹെറഡോട്ടസിന് ഒന്നര സഹസ്രാബ്ദത്തോളം മുമ്പ് ജീവിച്ച ഇബ്‌റാഹിം പ്രവാചകന്‍ സ്ഥാപിച്ച ഒരാരാധനാലയത്തിന്റെ പരിസരത്ത് ഹെറഡോട്ടസിന്റെ കാലമായപ്പോഴേക്കും വിശ്വാസവ്യതിചലനങ്ങളുടെ ഫലമായി വിഗ്രഹങ്ങള്‍ വന്നുചേരുന്നതില്‍ എന്ത് അസാംഗത്യമാണുള്ളത്? കഅ്ബയെ ഇബ്‌റാഹിം നബി (അ) പഠിപ്പിച്ച ഏകദൈവാരാധനാപരമായ മൂല്യങ്ങളില്‍ തന്നെ അറബികള്‍ ചരിത്രത്തിലുടനീളം നിലനിര്‍ത്തി എന്ന് മുഹമ്മദ് നബി(സ)യോ മുസ്‌ലിംകളോ അവകാശപ്പെട്ടിട്ടില്ലെന്നു മാത്രമല്ല, ചരിത്രത്തില്‍ സംഭവിച്ച അപഭ്രംശങ്ങള്‍ വഴി പരിശുദ്ധമായ ആ ദൈവഭവനം പില്‍ക്കാലത്ത് വിഗ്രഹങ്ങള്‍കൊണ്ട് മലിനമായിത്തീര്‍ന്നു എന്നാണ് ഇസ്‌ലാം പഠിപ്പിക്കുന്നത്. അത്തരം അപഭ്രംശങ്ങളില്‍ നിന്നു വിമോചിപ്പിച്ച് കഅ്ബയെ അതിന്റെ ആദിമ അബ്രഹാമിക വിശുദ്ധയില്‍ പുനഃസ്ഥാപിക്കുക എന്ന ദൗത്യമാണല്ലോ, മുഹമ്മദ് നബി (സ) ഏഴാം നൂറ്റാണ്ടില്‍ നിര്‍വഹിച്ചത്.

ബി. സി. ഇ ഒന്നാം നൂറ്റാണ്ടില്‍ ജീവിച്ച ഡിയോഡറസ് സിക്കുലസ് (Diodorus Siculus) എന്ന ഗ്രീക്ക് ചരിത്രകാരന്‍ അന്ന് അദ്ദേഹത്തിനറിയാമായിരുന്ന ഭൂപ്രദേശങ്ങളെക്കുറിച്ചെഴുതിയ ബിബഌയോത്തിക്ക ഹിസ്റ്റോറിക്ക (Bibliotheca Historica) എന്ന ഗ്രന്ഥത്തില്‍ മക്കയെ പരമാര്‍ശിച്ചുകൊണ്ട് ”അവിടെ അതിവിശുദ്ധവും എല്ലാ അറബികളും അങ്ങേയറ്റം ആദരിക്കുന്നതുമായ ഒരാരാധനാലയം നിലിവിലുണ്ട്” എന്നെഴുതിയിട്ടുള്ളതാണ് വിമര്‍ശകരുടെ മറ്റൊരു ‘തെളിവ്.’ ഗ്രീക്കുകാര്‍ക്കുപോലും പരിചിതമാകുംവിധമുള്ള പ്രശസ്തി ക്രിസ്തുയേശുവിനു മുമ്പുതന്നെ കഅ്ബ കൈവരിച്ചിരുന്നുവെന്ന് മാത്രമാണ് വാസ്തവത്തില്‍ ഡിയോഡറസിന്റെ ഗ്രന്ഥം തെളിയിക്കുന്നത്. കഅ്ബ ഇബ്‌റാഹിം നബി(അ) സ്ഥാപിച്ചതാണെന്ന യാഥാര്‍ത്ഥ്യത്തെ നിഷേധിക്കുന്ന വിദൂരമായ സൂചനകള്‍പോലും അദ്ദേഹത്തിന്റെ എഴുത്ത് ഉള്‍ക്കൊള്ളുന്നില്ല. കഅ്ബയില്‍ വിഗ്രഹാരാധന നടക്കുന്നതായിത്തന്നെ ഡിയോഡറസിന്റെ എഴുത്തുകള്‍ പറഞ്ഞാല്‍പോലും അദ്ദേഹത്തിനു രണ്ട് സഹസ്രാബ്ദങ്ങള്‍ക്കുമുമ്പ് ഇബ്‌റാഹിം നബി (അ) ഏകദൈവാരാധനക്കുവേണ്ടി സ്ഥാപിച്ചതായിരുന്നു പ്രസ്തുത ഭവനമെന്ന യാഥാര്‍ത്ഥ്യത്തെ അതൊരിക്കലും ബാധിക്കുകയില്ലെന്നതാണ് വാസ്തവം.

കഅ്ബ ഇബ്‌റാഹിം പ്രവാചകനും പുത്രന്‍ ഇസ്മാഈല്‍ പ്രവാചകനും ചേര്‍ന്ന് പ്രപഞ്ചനാഥനെ മാത്രം ആരാധിക്കുവാന്‍വേണ്ടി പടുത്തുയര്‍ത്തിയതാണെന്ന ഇസ്‌ലാമിക നിലപാടിനെ നിരാകരിക്കുകയോ ബഹുദൈവാരാധകര്‍ സ്ഥാപിച്ചതോ ബഹുദൈവാരാധനയ്ക്കുവേണ്ടി സ്ഥാപിക്കപ്പെട്ടതോ ആയ ദേവാലയമാണ് അത് എന്നു സൂചിപ്പിക്കുകയോ ചെയ്യുന്നില്ല വിമര്‍ശകരുടെ കൈവശമുള്ള ‘ചരിത്രരേഖ’കളൊന്നും തന്നെ എന്ന് നമുക്ക് വ്യക്തമായി. കഅ്ബയെ ശിലാപൂജയുമായി ബന്ധിപ്പിക്കുവാന്‍ ഇസ്‌ലാം വിമര്‍ശകര്‍ പിന്നീട് ആശ്രയിക്കുന്നത് ഹജറുല്‍ അസ്‌വദ് എന്ന, കഅ്ബയുടെ ചുമരില്‍ പരിരക്ഷിക്കപ്പെടുന്ന കറുത്ത കല്ലിനെയാണ്. വാസ്തവത്തില്‍, അങ്ങേയറ്റം പരിഹാസ്യമായ ഒരു വാദമാണിത്. ഹജറുല്‍ അസ്‌വദ് എന്ന അറബി വാക്കിനര്‍ത്ഥം കറുത്ത കല്ല് എന്നു മാത്രമാണ്. ആരാധനാലയങ്ങളുടെ ചുമരുകള്‍ നിര്‍മിക്കപ്പെടുക സ്വാഭാവികമായും കല്ലുപയോഗിച്ചു തന്നെയാണ്. കഅ്ബ പടുത്തുയര്‍ത്തുവാനാരംഭിച്ചപ്പോള്‍ മൂലശിലയായി ഇബ്‌റാഹിമും ഇസ്മാഈലും ഉപയോഗിച്ച കറുത്ത കല്ലാണ് ഹജറുല്‍ അസ്‌വദ് എന്ന പേരില്‍ വിശ്രുതമായിത്തീര്‍ന്നത്. ചരിത്രത്തിന്റെ വ്യത്യസ്ത സന്ദര്‍ഭങ്ങളില്‍ നടന്ന പുനര്‍നിര്‍മാണ വേളകളില്‍ ഈ മൂലശില എടുത്തുമാറ്റപ്പെടാതെ പരിരക്ഷിക്കപ്പെട്ടുവെന്നതാണ് അതിന്റെ ചരിത്രപരമായ പ്രാധാന്യം.

ആരാധനാലയങ്ങളുടെ ചുമരുകളില്‍ കല്ലുകളുണ്ടാകുന്നതിന്റെ പേരല്ല ശിലാപൂജ, മറിച്ച് ചുമരിലോ അല്ലാതെയോ ഉള്ള ഏതെങ്കിലും കല്ല് പൂജിക്കപ്പെടുന്നതിന്റെ പേരാണ്. അറബികള്‍ ഇബ്‌റാഹിമീ ഏകദൈവ വിശ്വാസത്തില്‍നിന്ന് പല രീതിയിലും വ്യതിചലിച്ചു പോവുകയും മുഹമ്മദ് നബി (സ)യുടെ കാലമായപ്പോഴേക്കും കടുത്ത വിഗ്രഹാരാധകരായി മാറുകയും ചെയ്തിരുന്നു. എന്നാല്‍ അവരുടെ പൂജാവസ്തുക്കളില്‍ ഒരിക്കല്‍പോലും ഹജറുല്‍ അസ്‌വദ് ഉള്‍പ്പെട്ടിരുന്നില്ലെന്നതാണ് വാസ്തവം. ചരിത്രത്തിന്റെ ഒരു സന്ദര്‍ഭത്തിലും ആരാധിക്കപ്പെട്ടിട്ടില്ലാത്ത, കഅ്ബയുടെ മൂലശിലയെന്ന നിലയില്‍ അപ്പടി നിലനിര്‍ത്തപ്പെടുക മാത്രം ചെയ്തിട്ടുള്ള ഒരു കല്ലാണത്. കഅ്ബയെ ശിലാപൂജയുടെ കേന്ദ്രമായി അവതരിപ്പിക്കാനാഗ്രഹമുള്ള മിഷനറിമാര്‍ക്കും ഓറിയന്റലിസ്റ്റുകള്‍ക്കുമൊന്നും ഹജറുല്‍ അസ്‌വദിനെ അറബികള്‍ പൂജിച്ചിരുന്നുവെന്ന് കാണിക്കുന്ന ഒരു ചരിത്രരേഖയും ഉദ്ധരിക്കാന്‍ കഴിയാത്തത് അതുകൊണ്ടാണ്. കഅ്ബക്കകത്തുപോലും വിഗ്രഹങ്ങള്‍ സ്ഥാപിക്കപ്പെട്ടപ്പോഴും അറബികള്‍ ഹജറുല്‍ അസ്‌വദിനെ ആരാധനാമൂര്‍ത്തിയായി സ്വീകരിച്ചില്ലെന്ന് പറയുമ്പോള്‍ ഹജറുല്‍ അസ്‌വദിനെ ചൂണ്ടിക്കാണിച്ച് ‘ശിലാപൂജ’ സമര്‍ത്ഥിക്കുവാന്‍ ശ്രമിക്കുന്നത് എത്രമേല്‍ ബാലിശമാണെന്ന് ആര്‍ക്കും വ്യക്തമാകും.

ഭക്തി സമര്‍പ്പിക്കപ്പെടുകയോ നിവേദ്യങ്ങള്‍ നേരപ്പെടുകയോ കാര്യകാരണ ബന്ധങ്ങള്‍ക്കതീതമായ ഉപകാരോപദ്രവങ്ങള്‍ പ്രതീക്ഷിക്കപ്പെടുകയോ ഭരമേല്‍പിക്കപ്പെടുകയോ സഹായമഭ്യര്‍ത്ഥിക്കപ്പെടുകയോ പ്രാര്‍ത്ഥിക്കപ്പെടുകയോ ഒന്നും ചെയ്തിട്ടില്ലാത്ത ഹജറുല്‍ അസ്‌വദ്, പ്രപഞ്ചനാഥനുള്ള ആരാധനയായി കഅ്ബയെ വലം വെക്കുന്നവര്‍ക്ക് (ത്വവാഫ്) അത് തുടങ്ങുവാനുള്ള സ്ഥലം അടയാളപ്പെടുത്തുന്ന നാട്ടക്കുറിയായാണ് ഇസ്‌ലാമിക കര്‍മശാസ്ത്രത്തില്‍ നിലനില്‍ക്കുന്നത്. ഇസ്‌ലാം പൂര്‍വ അറബികള്‍ക്കിടയിലും അതങ്ങനെ മാത്രമാണ് നിലനിന്നത്. ഹജറുല്‍ അസ്‌വദിനെ ആരാധിക്കുന്ന യാതൊരു കര്‍മവും ഹജ്ജോ ഉംറയോ ഉള്‍ക്കൊള്ളുന്നില്ല, ഉള്‍ക്കൊണ്ടിട്ടുമില്ല. ചരിത്രത്തില്‍ നിന്നോ കഅ്ബയുടെ നിര്‍മാണരീതിയില്‍ നിന്നോ കഅ്ബയ്ക്ക് ഇബ്‌റാഹിം നബി (അ)യുമായുള്ള പൊക്കിള്‍ക്കൊടി ബന്ധം നിഷേധിക്കാനാവശ്യമായ യാതൊരു തെളിവും ഹാജരാക്കാനാവില്ല എന്നു ചുരുക്കം.

ആരാധിക്കപ്പെടുന്ന കല്ലുകള്‍ ദൈവത്തിന്റെ പ്രവാചകന്‍മാര്‍ സ്ഥാപിക്കുകയില്ലെന്ന കാര്യത്തില്‍ യാതൊരു സംശയവുമില്ല. എന്നാല്‍ സാക്ഷാല്‍ ദൈവത്തെ ആരാധിക്കുവാനുള്ള സ്ഥലം അടയാളപ്പെടുത്താന്‍വേണ്ടി ദൈവനിര്‍ദേശപ്രകാരം കല്ലുകള്‍ വെക്കുക പ്രവാചകന്‍മാരുടെ സമ്പ്രദായം തന്നെയാണ്. കഅ്ബയുടെ കാര്യത്തില്‍ ഇബ്‌റാഹിം നബി(അ) ചെയ്തത് അതുമാത്രമാണ്. ആരാധനാലയത്തിന്റെ നിര്‍മാണം തുടങ്ങാന്‍വേണ്ടി ഉപയോഗിച്ച കല്ലിനെ ചൂണ്ടിക്കാണിച്ച് കഅ്ബ അബ്രഹാമികമല്ലെന്ന് ‘സമര്‍ത്ഥിക്കുന്നവര്‍’, ഈ രീതി പ്രവാചകന്‍മാര്‍ക്കുണ്ടായിരുന്നുവെന്ന് ബൈബിള്‍ തന്നെ വ്യക്തമാക്കുന്നുണ്ടെന്ന വസ്തുതയെ വളരെ സമര്‍ത്ഥമായി മറച്ചുവെക്കുകയാണ് ചെയ്യുന്നത്. അബ്രഹാമിന്റെ പുത്രന്‍ ഇസ്ഹാഖിന്റെ പുത്രനും ഇസ്രാഈല്‍ ഗോത്രങ്ങളുടെ പിതാവുമായ യാക്കോബ് പ്രവാചകന്റെ ജീവിതത്തില്‍ നിന്ന് ബൈബിള്‍ പഴയ നിയമം ഉദ്ധരിക്കുന്ന ഒരു സംഭവം നോക്കുക: ”യാക്കോബ് ബേര്‍ഷെബായില്‍ നിന്ന് ഹാരാനിലേക്ക് പുറപ്പെട്ടു. സൂര്യന്‍ അസ്തമിച്ചപ്പോള്‍ അവന്‍ വഴിക്ക് ഒരിടത്തു തങ്ങുകയും രാത്രി അവിടെ ചെലവഴിക്കുകയും ചെയ്തു. ഒരു കല്ലെടുത്ത് തലയ്ക്കുകീഴെ വെച്ച് അവന്‍ ഉറങ്ങാന്‍ കിടന്നു. അവന് ഒരു ദര്‍ശനം ഉണ്ടായി; ഭൂമിയില്‍ ഉറപ്പിച്ചിരുന്ന ഒരു ഗോവണി, അതിന്റെ അറ്റം ആകാശത്തു മുട്ടിയിരിക്കുന്നു. ദൈവദൂതന്‍മാര്‍ അതിലൂടെ കയറുകയും ഇറങ്ങുകയും ചെയ്തുകൊണ്ടിരുന്നു. ഗോവണിയുടെ മുകളില്‍ നിന്നുകൊണ്ട് കര്‍ത്താവ് അരുളി ചെയ്തു: ഞാന്‍ നിന്റെ പിതാവായ അബ്രഹാമിന്റെയും ഇസ്ഹാഖിന്റെയും ദൈവമായ കര്‍ത്താവാണ്. നീ കിടക്കുന്ന ഈ മണ്ണ് നിനക്കും നിന്റെ സന്തതികള്‍ക്കും ഞാന്‍ നല്‍കും. നിന്റെ സന്തതികള്‍ ഭൂമിയിലെ പൂഴിപോലെ എണ്ണമറ്റവരായിരിക്കും. കിഴക്കോട്ടും പടിഞ്ഞാറോട്ടും തെക്കോട്ടും വടക്കോട്ടും നിങ്ങള്‍ വ്യാപിക്കും. നിന്നിലൂടെയും നിന്റെ സന്തതികളിലൂടെയും ഭൂമിയിലെ ഗോത്രങ്ങളെല്ലാം അനുഗ്രഹിക്കപ്പെടും. ഇതാ, ഞാന്‍ നിന്നോടു കൂടെയുണ്ട്. നീ പോകുന്നിടത്തെല്ലാം ഞാന്‍ നിന്നെ കാത്തുരക്ഷിക്കും. നിന്നെ ഈ നാട്ടിലേക്ക് തിരികെ കൊണ്ടുവരും. നിന്നോട് പറഞ്ഞതൊക്കെ നിറവേറ്റുന്നതുവരെ ഞാന്‍ നിന്നെ കൈവിടുകയില്ല. അപ്പോള്‍ യാക്കോബ് ഉറക്കില്‍ നിന്നുണര്‍ന്നു. അവന്‍ പറഞ്ഞു: തീര്‍ച്ചയായും കര്‍ത്താവ് ഈ സ്ഥലത്തുണ്ട്. എന്നാല്‍, ഞാന്‍ അതറിഞ്ഞില്ല. ഭീതിപൂണ്ട് അവന്‍ പറഞ്ഞു; ഈ സ്ഥലം എത്ര ഭയാനകമാണ്! ഇത് ദൈവത്തിന്റെ ഭവനമല്ലാതെ മറ്റൊന്നുമല്ല. സ്വര്‍ഗത്തിന്റെ കവാടമാണിവിടം. യാക്കോബ് അതിരാവിലെ എഴുന്നേറ്റ് തലയ്ക്കുകീഴെ വെച്ചിരിക്കുന്ന കല്ലെടുത്ത് ഒരു തൂണായി കുത്തിനിര്‍ത്തി അതിന്‍മേല്‍ എണ്ണയൊഴിച്ചു. അവന്‍ ആ സ്ഥലത്തിന് ബഥേല്‍ എന്നു പേരിട്ടു. ലൂസ് എന്നായിരുന്നു ആ പട്ടണത്തിന്റെ ആദ്യത്തെ പേര്. അതുകഴിഞ്ഞ് യാക്കോബ് ഒരു പ്രതിജ്ഞ ചെയ്തു. ദൈവമായ കര്‍ത്താവ് എന്റെ കൂടെ ഉണ്ടായിരിക്കുകയും, ഈ യാത്രയില്‍ എന്നെ സംരക്ഷിക്കുകയും, എനിക്ക് ഉണ്ണാനും ഉടുക്കാനും തരികയും, എന്റെ പിതാവിന്റെ വീട്ടിലേക്ക് സമാധാനത്തോടെ ഞാന്‍ തിരിച്ചെത്തുകയും ചെയ്താല്‍ കര്‍ത്താവായിരിക്കും എന്റെ ദൈവം. തൂണായി കുത്തി നിര്‍ത്തിയിരിക്കുന്ന ഈ കല്ല് ദൈവത്തിന്റെ ഭവനമായിരിക്കും. അവിടുന്ന് എനിക്ക് തരുന്നതിന്റെയെല്ലാം പത്തിലൊന്ന് ഞാന്‍ അവിടുത്തേക്ക് സമര്‍പ്പിക്കുകയും ചെയ്യും.” (ഉല്‍പത്തി 28 : 10-22).

ദൈവം തന്റെ ജനതക്ക് നല്‍കിയ വാഗ്ദത്ത വിശുദ്ധഭൂമിയില്‍ ദൈവനിര്‍ദേശപ്രകാരം ആരാധനാലയമടയാളപ്പെടുത്താന്‍ കല്ലുനാട്ടിയ യാക്കോബ് ചെയ്തുവെന്ന്  ബൈബിള്‍ പറയുന്നതിലധികമൊന്നും അബ്രഹാമും ഇശ്മയേലും മക്ക എന്ന വാഗ്ദത്ത വിശുദ്ധ ഭൂമിയില്‍ ഹജറുല്‍ അസ്‌വദ് എന്ന മൂലശില കൊണ്ട് കഅ്ബയുടെ നിര്‍മാണമാരംഭിച്ചപ്പോള്‍ സംഭവിച്ചിട്ടില്ല എന്നതാണ് സത്യം. ഹജറുല്‍ അസ്‌വദ് ചൂണ്ടിക്കാണിച്ച് കഅ്ബയുടെ അബ്രഹാമിക പ്രവാചകപാരമ്പര്യം നിഷേധിക്കുന്നത് അതിനാല്‍ തന്നെ, എല്ലാ അര്‍ത്ഥത്തിലും അടിസ്ഥാനരഹിതമാണ്.

ഇസ്‌ലാമിക പാരമ്പര്യം പറയുന്നതുപോലെ അബ്രഹാമിനോട് ദൈവം ബലിയായി നല്‍കാന്‍ നിര്‍ദ്ദേശിച്ച് പരീക്ഷിച്ചത് ഇശ്മയേലിനെയല്ല, ഇസ്ഹാഖിനെയാണ്. ഇശ്മയേലിനു പകരമല്ല, ഇസ്ഹാഖിനു പകരമാണ് അബ്രഹാം ആടിനെ ബലിയറുത്തത്. അതിനാല്‍ ഹജ്ജിന്റെ ഭാഗമായുള്ള ഇസ്‌ലാമിക ബലി അബ്രഹാമികമല്ല, മറിച്ച് ബഹുദൈവാരാധനാപരമായ മക്കന്‍ ആചാരങ്ങളുടെ ശേഷിപ്പാണ്. മക്കയുടെ അബ്രഹാമിക പൈതൃകത്തെ നിരാകരിക്കാന്‍ ശ്രമിച്ചുകൊണ്ടുള്ള മിഷനറി വിശകലനങ്ങളില്‍ സര്‍വസാധാരണമായ ഈ വാദങ്ങള്‍ ശരിയാണോ?

ല്ല. ദൈവികനിര്‍ദ്ദേശപ്രകാരം ഇശ്മയേലിനെ ബലിയറുക്കാനൊരുങ്ങിയപ്പോഴല്ല മറിച്ച് ഇസ്ഹാഖിനെ ബലിയറുക്കാനൊരുങ്ങിയപ്പോഴാണ് ആടിനെ ബലിയായി നില്‍കാനുള്ള കല്‍പന അബ്രഹാമിന് ലഭിച്ചതെന്ന് ബൈബിള്‍ പറയുന്നുവെന്നും അതിനാല്‍ ഹജ്ജിന്റെ മൃഗബലി ബഹുദൈവാരാധനാപരമായ പശ്ചാത്തലങ്ങളില്‍ നിന്നുടലെടുത്തതാണെന്നുമാണ് മിഷനറിമാര്‍ വാദിക്കുറുള്ളത്.

ഒന്നാമതായി, ബൈബിള്‍ പറയുന്നുവെന്നതുകൊണ്ടു മാത്രം അബ്രഹാം ബലി നല്‍കാനൊരുങ്ങിയത് ഇസ്ഹാഖിനെയാണെന്ന് സ്ഥാപിക്കപ്പെടുകയില്ല. ആധികാരികമായ ഒരു ചരിത്രസ്രോതസ്സല്ലാത്തതുകൊണ്ടുതന്നെ, ബൈബിള്‍ വിവരണങ്ങളുടെ മാത്രം വെളിച്ചത്തില്‍ ഇശ്മയേലിനെ ബലി നല്‍കാനാണ് ദൈവനിര്‍ദേശമുണ്ടായത് എന്ന നിലപാടിനെ നിരാകരിക്കുന്നത് അര്‍ത്ഥശൂന്യമാണ്.

രണ്ടാമതായി, ഇസ്ഹാഖിനെയായിരുന്നോ ഇശ്മയേലിനെയായിരുന്നോ ദൈവം ബലിക്ക് നിര്‍ദ്ദേശിച്ചത് എന്ന തര്‍ക്കത്തിന്, ഹജ്ജിന്റെ ഭാഗമായ ബലികര്‍മം ബഹുദൈവാരാധനാപരമായ വേരുകളുള്ളതാണെന്ന് സ്ഥാപിക്കാനുള്ള മിഷനറി പരിശ്രമത്തില്‍ ഒരു പങ്കും വഹിക്കാനില്ല. തീര്‍ത്ഥാടകര്‍ സ്വന്തം സമ്പത്ത് ചെലവഴിച്ച് വാങ്ങിയറുത്ത മൃഗങ്ങളുടെ മാംസം പ്രപഞ്ചനാഥന്റെ തൃപ്തി കാംക്ഷിച്ച് പാവങ്ങള്‍ക്ക് വിതരണം ചെയ്യാന്‍ സന്നദ്ധരാകുകയാണ് ഹജ്ജ് ബലിയില്‍ സംഭവിക്കുന്നത്. പ്രപഞ്ചരക്ഷിതാവല്ലാത്ത മറ്റൊരു ശക്തിയോടുമുള്ള ഭക്തി ഇസ്‌ലാമിക ബലികര്‍മത്തില്‍ കടന്നുവരുന്നില്ല. പ്രപഞ്ചനാഥനായ അല്ലാഹുവിലേക്ക്, ബലിയറുക്കപ്പെടുന്ന മൃഗത്തിന്റെ മാംസമോ രക്തമോ അല്ല, മറിച്ച് അതിന് സന്നദ്ധനാകുന്ന വ്യക്തിയുടെ മനസ്സിന്റെ നന്മയാണ് എത്തിച്ചേരുന്നതെന്ന് ക്വുര്‍ആന്‍ വ്യക്തമാക്കുന്നുണ്ട്. ”അവയുടെ മാംസമോ രക്തമോ അല്ലാഹുവിങ്കല്‍ എത്തുന്നതേയില്ല. എന്നാല്‍ നിങ്ങളുടെ ധര്‍മ്മനിഷ്ഠയാണ് അവങ്കല്‍ എത്തുന്നത്.” (22 : 37).

ശുദ്ധ ഏകദൈവാരാധനയുടെ ഭാഗമായി, ദൈവത്തിന് ഭക്ഷണമാവശ്യമാണെന്ന യാതൊരു തെറ്റിദ്ധാരണയുമില്ലാതെ നിര്‍വഹിക്കപ്പെടുന്ന സേവനപ്രവര്‍ത്തനമാണ് ഹജ്ജ് ബലിയും അതോടനുബന്ധിച്ച മാംസവിതരണവുമെന്ന് ചുരുക്കം. ഇതില്‍ ബഹുദൈവാരാധനയുടെ എന്ത് അടരുകളുണ്ടെന്നാണ് മിഷനറിമാര്‍ പറയുന്നത്?

പല ബഹുദൈവാരാധക സമൂഹത്തിന്റെയും അനുഷ്ഠാനമുറകളില്‍ മൃഗബലിയുണ്ടെന്ന കാര്യത്തില്‍ സംശയമൊന്നുമില്ല. പ്രപഞ്ചനാഥനു പുറമെയുള്ള വിഗ്രഹങ്ങളുടെയും സാങ്കല്‍പിക അദൃശ്യശക്തികളുടെയും പൊരുത്തത്തിനുവേണ്ടിയും പലപ്പോഴും അവയെ ‘ഊട്ടാന്‍’ വേണ്ടിയും നിര്‍വഹിക്കപ്പെടുന്ന അന്ധവിശ്വാസ ജഡിലമായ അറവുകളാണവ. പ്രവാചകന്‍മാര്‍ ഏകദൈവവിശ്വാസത്തിന്റെ ഭാഗമായി പഠിപ്പിച്ച ശരിയായ ബലിരീതികളില്‍ നിന്ന് വ്യതിചലിച്ചുപോയാണ് ബഹുദൈവാരാധകര്‍ ഇത്തരം ദുരാചാരങ്ങളില്‍ എത്തിപ്പെട്ടതെന്നാണ് മതഗ്രന്ഥങ്ങളില്‍ നിന്നെല്ലാം മനസ്സിലാക്കാന്‍ കഴിയുന്നത്. മക്കയിലെ ബഹുദൈവാരാധക സംസ്‌കൃതിയിലും ഇത്തരത്തിലുള്ള പല ബലികളുമുണ്ടായിരുന്നു. എന്നാല്‍ ഹജ്ജിന്റെ ഭാഗമായി മിനയില്‍ വെച്ചു നടന്നിരുന്ന ബലി, ഇസ്‌ലാം പൂര്‍വകാലത്തുപോലും ഒരിക്കലും വിഗ്രഹങ്ങളുടെ പ്രസാദത്തിനുവേണ്ടി നിര്‍വഹിക്കപ്പെട്ടിരുന്നില്ല. അത് എപ്പോഴും അല്ലാഹുവിനെ മാത്രമാണ് ലക്ഷ്യമാക്കിയിരുന്നത്. പിന്നെയെങ്ങനെയാണ് ഹജ്ജ് ബലി ബഹുദൈവാരാധനയുടെ ശേഷിപ്പാണെന്ന് മിഷനറിമാര്‍ വാദിക്കുക? എന്ത് തെളിവാണ് ചരിത്രപരമായി അവര്‍ക്കീ വിഷയത്തില്‍ ഹാജരാക്കാനുള്ളത്? ഇബ്‌റാഹീമിനോട് ദൈവം സംസാരിച്ചത് ഏത് പുത്രന്റെ കാര്യമായിരുന്നാലും, ഇബ്‌റാഹീം പഠിപ്പിച്ച ഏകദൈവാരാധനാപരമായ മൃഗബലി ഒരു സമൂഹത്തില്‍ നിലനില്‍ക്കുന്നതില്‍ എന്ത് അസാംഗത്യമാണുള്ളത്?

പ്രപഞ്ചനാഥന്റെ പ്രീതി ഉദ്ദേശിച്ചുള്ള മൃഗബലി പ്രവാചകാധ്യാപനമാണെന്നും ബഹുദൈവാരാധനയുടെ ശേഷിപ്പല്ലെന്നുമാണ് ബൈബിളിന്റെ അസന്നിഗ്ധമായ വീക്ഷണം. ബൈബിളെഴുത്തുകാര്‍ യഹോവക്കുവേണ്ടിയുള്ള മൃഗബലിയുടെ വിശദമായ കര്‍മശാസ്ത്രം വിവരിച്ചിട്ടുള്ളത് പൂര്‍ണമായും പ്രവാചകാധ്യാപനങ്ങളുടെ വെളിച്ചത്തിലാണോ എന്ന കാര്യം സംശയാസ്പദമാണെങ്കിലും, ഏകദൈവാരാധനയുടെ ഭാഗമായി പ്രവാചകനിര്‍ദ്ദേശങ്ങള്‍ പ്രകാരംതന്നെ സെമിറ്റിക് ചരിത്രത്തിലുടനീളം മൃഗബലി നിലനിന്നുവെന്ന് അവ വ്യക്തമാക്കുന്നുണ്ടെന്ന കാര്യം ശ്രദ്ധേയമാണ്.

ഫലസ്ത്വീനിലെ വിശുദ്ധഗേഹം കേന്ദ്രീകരിച്ച് ജൂതന്‍മാര്‍ നടത്തിയിരുന്ന മതാനുഷ്ഠാനങ്ങളുടെ സുപ്രധാനമായ ഭാഗം തന്നെ മൃഗബലിയായിരുന്നു. സി.ഇ എഴുപതില്‍ റോമക്കാര്‍ ആരാധനാലയം തകര്‍ത്തപ്പോഴാണ് മൃഗബലി ജൂതന്‍മാര്‍ക്കിടയില്‍നിന്ന് ഇല്ലാതായിത്തുടങ്ങിയതെന്നാണ് ഗവേഷകരുടെയെല്ലാം അഭിപ്രായം. പഴയനിയമ പ്രവാചകന്‍മാരെല്ലാം പഠിപ്പിച്ച അനുഷ്ഠാന ചട്ടക്കൂടിനെ ദുര്‍ബലപ്പെടുത്തി പൗലോസ് സ്ഥാപിച്ച പുതിയ മതത്തിന്റെ വക്താക്കളായി മാറിയപ്പോഴാണ് ക്രൈസ്തവരില്‍ നിന്ന് ബൈബിളികമായ മൃഗബലി സമ്പ്രദായം അന്യം നിന്നുപോയത്. ബൈബിളില്‍ ഇപ്പോഴും വളരെ വ്യക്തമായി അനുശാസിക്കപ്പെടുന്ന, എന്നാല്‍ അതിന്റെ അനുയായികള്‍ യഥാവിധി പിന്തുടരാന്‍ സന്നദ്ധത കാണിക്കാത്ത പ്രവാചകപാരമ്പര്യമാണ് മൃഗബലിയെന്നര്‍ത്ഥം. ഇത് ഇസ്മാഈലിന്റെ സന്തതിപരമ്പരകളിലെങ്കിലും അവശേഷിക്കുന്നതില്‍ ബൈബിളിന്റെ വക്താക്കള്‍ വേവലാതിപ്പെടുന്നതെന്തിനാണ്?

ആദിമനുഷ്യനായ ആദാമിനു തന്നെ മൃഗബലിയുടെ മഹത്വത്തെക്കുറിച്ച് ദൈവം അറിവു നല്‍കിയിരുന്നുവെന്നാണ് ബൈബിളില്‍ നിന്നു മനസ്സിലാക്കാന്‍ കഴിയുന്നത്. അതുകൊണ്ടാണല്ലോ, ആദാമിന്റെ പുത്രനായ ആബേല്‍ ”തന്റെ ആട്ടിന്‍കുട്ടത്തിലെ കടിഞ്ഞൂല്‍ കുഞ്ഞുങ്ങളെയെടുത്ത് അവയുടെ കൊഴുപ്പുള്ള ഭാഗങ്ങള്‍ അവിടുത്തേക്ക് കാഴ്ചവെച്ചതും, ആബേലിലും അവന്റെ കാഴ്ചവസ്തുക്കളിലും അവിടുന്ന്” പ്രസാദിച്ചതും (ഉല്‍പത്തി 4 : 4). ഇവിടം മുതല്‍ പഴയനിയമത്തില്‍ മുഴുവന്‍ മൃഗബലി വിവരണങ്ങള്‍ നിറഞ്ഞുനില്‍ക്കുന്നതാണ് നമുക്ക് ബൈബിളില്‍ കാണാന്‍ കഴിയുന്നത്. ദൈവത്തില്‍ നിന്ന് ലഭിച്ച അനുഗ്രഹങ്ങള്‍ക്കുള്ള നന്ദിയായും നേര്‍ച്ചയുടെ ഭാഗമായും പെസഹ ആഘോഷത്തിനുവേണ്ടിയും തെറ്റുകള്‍ക്കുള്ള പ്രായശ്ചിത്തമായും അനുശോചനച്ചടങ്ങെന്ന നിലയിലും സ്വതതന്ത്രമായ പുണ്യകര്‍മമെന്ന നിലക്കുമെല്ലാം പുരോഹിതന്‍മാരും രാജാക്കന്‍മാരും സമൂഹവും വ്യക്തികളുമെല്ലാം നടത്തിയ മൃഗബലികളുടെ കഥകള്‍ കൊണ്ട് സമൃദ്ധമാണ് ബൈബിള്‍ പഴയനിയമം. ബലിമൃഗത്തെ അറുത്തശേഷം പുകയും മാംസഗന്ധവും ദൈവത്തിലേക്കയക്കാന്‍ വേണ്ടി മൃഗശരീരം കത്തിക്കുന്നതുമുതല്‍ ബലിമൃഗത്തിന്റെ രക്തം പുരോഹിത നേതൃത്വത്തില്‍ അള്‍ത്താരക്കുചുറ്റും തളിക്കുന്നതുവരെയുള്ള, ഇസ്‌ലാമിക കര്‍മശാസ്ത്രത്തിന് പരിചയമില്ലാത്ത രീതികളും ജൂതസമൂഹത്തില്‍ മൃഗബലിയുടെ ഭാഗമായി നിലനിന്നിരുന്നുവെന്നത് വ്യാപകമായി അറിയപ്പെടുന്ന ചരിത്രവസ്തുതയാണ്. ബൈബിളും ബൈബിളിന്റെ വെളിച്ചത്തില്‍ ജീവിച്ച സമൂഹവും മൃഗബലിയെ മനസ്സിലാക്കിയ രീതിയാണ് ഇതില്‍ നിന്നെല്ലാം വ്യക്തമാകുന്നത്. മൃഗബലി എന്നു കേള്‍ക്കുമ്പോഴേക്കും ബഹുദൈവാരാധനയെക്കുറിച്ച് ചിന്തിക്കുവാനാരംഭിക്കുന്ന ആധുനിക മിഷനറിമാര്‍, തങ്ങളുടെ സ്വന്തം മതഗ്രന്ഥത്തെ തന്നെയാണ് അപഹസിക്കുന്നത് എന്നതത്രെ സത്യം.

മൂന്നാമതായി പരിശോധിക്കുവാനുള്ളത്, ഇസ്ഹാഖിനെ ബലിയറുക്കുവാനാണ് ദൈവനിര്‍ദ്ദേശമുണ്ടായത് എന്നാണ് ബൈബിളെഴുത്തുകാര്‍ മനസ്സിലാക്കിയത് എന്ന അവകാശവാദത്തിന്റെ  വസ്തുതാപരതയാണ്. മക്കക്കടുത്തുള്ള മിനയില്‍വെച്ച് ഇബ്‌റാഹീം ദൈവനിര്‍ദ്ദേശപ്രകാരം ഇസ്മാഈലിനെ ബലിയറുക്കാനൊരുങ്ങിയെന്നും ഇസ്മാഈലിനെ ബലിയാക്കലായിരുന്നില്ല, മറിച്ച് ദൈവനിര്‍ദ്ദേശങ്ങള്‍ അപ്പടി സ്വീകരിക്കുവാനുള്ള ത്യാഗസന്നദ്ധത ഇബ്‌റാഹീമില്‍ നിന്ന് ആവശ്യപ്പെടുകയായിരുന്നു ഇവ്വിഷയകമായ ദൈവിക പദ്ധതി എന്നതിനാല്‍ ബലിയറുക്കാനുള്ള സന്നദ്ധത ഇബ്‌റാഹീം തെളിയിച്ചപ്പോള്‍ മകനെ ബലിയറുക്കേണ്ടതില്ലെന്നു പറഞ്ഞ് പകരം ബലിയറുക്കാനായി ദൈവം ഒരാടിനെ നല്‍കിയെന്നുമുള്ള അറബ് ഇസ്‌ലാമിക പാരമ്പര്യത്തെ നിഷേധിക്കാനാണ് ബൈബിളുപയോഗിച്ച് മിഷനറിമാര്‍ മെനക്കെടുന്നത്.

ഇസ്ഹാഖിനെ ബലി നല്‍കുവാനാണ് ദൈവം ആവശ്യപ്പെട്ടതെന്ന് ബൈബിള്‍ പറയുന്നുവെന്ന കാര്യം ശരിയാണ്. എന്നാല്‍ ഈ പരാമര്‍ശമുള്‍ക്കൊള്ളുന്ന ഖണ്ഡിക പൂര്‍ണമായി വായിച്ചാല്‍ ഇസ്ഹാഖ് എന്ന പദം അതില്‍ പിന്നീടാരോ എഴുതിച്ചേര്‍ത്തതാണെന്ന് പകല്‍പോലെ വ്യക്തമാണ്. ബൈബിള്‍ വചനങ്ങള്‍ നോക്കുക: ”പിന്നീടൊരിക്കല്‍ ദൈവം അബ്രഹാമിനെ പരീക്ഷിച്ചു. അബ്രഹാം, അവിടുന്ന് വിളിച്ചു. ഇതാ ഞാന്‍, അവന്‍ വിളികേട്ടു. നീ സ്‌നേഹിക്കുന്ന നിന്റെ ഏക മകന്‍ ഇസ്ഹാഖിനെയും കൂട്ടിക്കൊണ്ട് കോറിയ ദേശത്തേക്കു പോവുക. അവിടെ ഞാന്‍ കാണിച്ചുതരുന്ന മലമുകളില്‍ നീ അവനെ എനിക്ക് ഒരു ദഹനബലിയായി അര്‍പ്പിക്കണം…. ദൈവം പറഞ്ഞ സ്ഥലത്തെത്തിയപ്പോള്‍ അബ്രഹാം അവിടെ ഒരു ബലിപീഠം പണിതു. വിറക് അടുക്കിവെച്ചിട്ട് ഇസ്ഹാഖിനെ ബന്ധിച്ച് വിറകിനുമീതെ കിടത്തി. മകനെ ബലി കഴിക്കാന്‍ അബ്രഹാം കത്തി കയ്യിലെടുത്തു. തല്‍ക്ഷണം കര്‍ത്താവിന്റെ ദൂതന്‍ ആകാശത്തുനിന്ന് അബ്രഹാം അബ്രഹാം എന്നുവിളിച്ചു. ഇതാ ഞാന്‍, അവന്‍ വിളികേട്ടു. കുട്ടിയുടെ മേല്‍ കൈവെക്കരുത്. അവനെ ഒന്നും ചെയ്യരുത്. നീ ദൈവത്തെ ഭയപ്പെടുന്നുവെന്ന് എനിക്കിപ്പോള്‍ ഉറപ്പായി. കാരണം, നിന്റെ ഏകപുത്രനെ എനിക്കു തരാന്‍ നീ മടി കാണിച്ചില്ല…… കര്‍ത്താവിന്റെ ദൂതന്‍ ആകാശത്തുനിന്ന് വീണ്ടും അബ്രഹാമിനെ വിളിച്ചുപറഞ്ഞു: കര്‍ത്താവ് അരുളി ചെയ്യുന്നു, നീ നിന്റെ ഏകപുത്രനെപ്പോലും എനിക്കുതരാന്‍ മടിക്കായ്ക കൊണ്ട് ഞാന്‍ ശപഥം ചെയ്യുന്നു: ഞാന്‍ നിന്നെ സമൃദ്ധമായി അനുഗ്രഹിക്കും.” (ഉല്‍പത്തി 22 : 1-17).

 ദൈവം ആവശ്യപ്പെട്ടതും അബ്രഹാമിന്റെ ആത്മാര്‍പ്പണത്തെ പ്രശംസിച്ചതുമെല്ലാം ബൈബിള്‍പ്രകാരം അബ്രഹാമിന്റെ ‘ഏക പുത്രനെ’ പരാമര്‍ശിച്ചുകൊണ്ടാണ്. അബ്രഹാമിന് ആദ്യമുണ്ടായ പുത്രന്‍ ഇസ്മാഈല്‍ ആണെന്ന് ഏത് ബൈബിള്‍ വായനക്കാരനാണ് അറിയാത്തത്? ഇസ്ഹാഖ് ജനിക്കുന്നതിനുമുമ്പാണ് ഈ സംഭവങ്ങളെല്ലാമുണ്ടായതെന്ന് ‘ഏകപുത്രന്‍’ എന്ന പ്രയോഗത്തില്‍നിന്ന് സുതരാം വ്യക്തമാണ്. ഇസ്മാഈലിനെയാണ് ഇബ്‌റാഹീം ബലിയറുക്കാനായി കൊണ്ടുപോയതെന്ന് ബൈബിളെഴുത്തുകാര്‍ക്കു പോലും ബോധ്യമുണ്ടായിരുന്നുവെന്നര്‍ത്ഥം.

ഇസ്ഹാഖിനെയും ഇസ്ഹാഖ് ജീവിച്ച പ്രദേശത്തിന്റെ ഭൂമിശാസ്ത്രത്തെയും ഈ സംഭവവിവരണത്തിലേക്ക് പിന്നീടാരോ ചേര്‍ത്തുവെച്ചതാണെന്ന് മനസ്സിലാക്കാനേ നിഷ്പക്ഷരായ ബൈബിള്‍ പഠിതാക്കള്‍ക്ക് കഴിയൂ. ഇതിന് പ്രേരകമായി വര്‍ത്തിച്ചത് യഹൂദവംശീയതയാണെന്ന കാര്യത്തില്‍ സംശയമൊന്നുമില്ല. തങ്ങളുടെ വംശപിതാവായ യഅ്ക്വൂബിന്റെ പിതാവിനില്ലാത്ത മഹത്വം അറബികളുടെ പിതാവായ ഇസ്മാഈലിനുണ്ടായിക്കൂടെന്ന യഹൂദശാഠ്യത്തില്‍ നിന്നുണ്ടായ ഒരു  കൈക്രിയയെയാണ് മിഷനറിമാര്‍ ‘ചരിത്ര’മായി അവതരിപ്പിക്കുന്നത് എന്ന കാര്യം എന്തുമാത്രം സഹതാപാര്‍ഹമല്ല!

ഹാജറ അടിമസ്ത്രീയായിരുന്നുവെന്നും അവരിലുണ്ടായ പുത്രന്‍ അബ്രഹാമിന്റെ യഥാര്‍ത്ഥ പുത്രനല്ലെന്നും ചില മിഷനറിമാര്‍ വാദിച്ചുനോക്കാറുണ്ട്. മാനവവിരുദ്ധമെന്നതിനു പുറമെ, ബൈബിള്‍ വിരുദ്ധം കൂടിയാണ് ഈ നിലപാട്. അടിമസ്ത്രീയായിരുന്ന ഹാജറയെ ഭാര്യയായാണ് അബ്രഹാം സ്വീകരിച്ചതെന്ന് ബൈബിള്‍ വ്യക്തമാക്കുന്നുണ്ട്. ”കാനാന്‍ ദേശത്ത് പത്തു വര്‍ഷം താമസിച്ചു കഴഞ്ഞപ്പോള്‍ അവന്റെ ഭാര്യ സാറ ദാസിയായ ഈജിപ്തുകാരി ഹാഗറിനെ തന്റെ ഭര്‍ത്താവിന് ഭാര്യയായി നല്‍കി.” (ഉല്‍പത്തി 16 : 3).

ഇസ്മാഈലിനെ, വളര്‍ച്ചയുടെ ഓരോ ഘട്ടത്തിലും പുത്രനായിത്തന്നെയാണ് അബ്രഹാം പരിഗണിച്ചതെന്നാണ് ബൈബിളിന്റെയും പക്ഷം. സാറയുടെ അടിമസ്ത്രീയായിരുന്ന ഹാഗാറിനെ അബ്രഹാമിനും സാറക്കുമൊന്നും ഇഷ്ടമുണ്ടായിരുന്നില്ലെന്നും സാറയെയാണ് തന്റെ പ്രിയതമയായി അബ്രഹാം പരിഗണിച്ചിരുന്നതെന്നും വംശീയത തലക്കുപിടിച്ച ചില മിഷനറിമാര്‍ സമര്‍ത്ഥിക്കുവാന്‍ ശ്രമിക്കാറുണ്ട്. അവരുടെ സങ്കുചിതത്വത്തെ വാദത്തിനുവേണ്ടി അംഗീകരിച്ചുകൊടുത്താല്‍ തന്നെ ബൈബിള്‍ പ്രകാരം ഇസ്മാഈല്‍ മാത്രമേ അബ്രഹാമിന്റെ ആദ്യ ജാതനാകൂ എന്നുള്ളതാണ് വാസ്തവം. ആവര്‍ത്തന പുസ്തകം പറയുന്നത് കാണുക: ”ഒരാള്‍ക്ക് രണ്ട് ഭാര്യമാരുണ്ടായിരിക്കുകയും അവന്‍ ഒരുവളെ സ്‌നേഹിക്കുകയും മറ്റവളെ ദ്വേഷിക്കുകയും ഇരുവരിലും അവന് സന്താനങ്ങളുണ്ടാവുകയും ആദ്യജാതന്‍ ദ്വേഷിക്കുന്നവളില്‍ നിന്നുള്ളവനായിരിക്കുകയും ചെയ്താല്‍ അവന്‍ തന്റെ വസ്തുവകകള്‍ പുത്രന്‍മാര്‍ക്ക് ഭാഗിച്ചുകൊടുക്കുമ്പോള്‍ താന്‍ വെറുക്കുന്നവളുടെ മകനും ആദ്യജാതനുമായവനെ മാറ്റി നിര്‍ത്തിയിട്ട് പകരം താന്‍ സ്‌നേഹിക്കുന്നവളുടെ മകനെ ആദ്യജാതനായി കണക്കാക്കരുത്. അവന്‍ തന്റെ സകല സമ്പത്തുക്കളുടെയും രണ്ടോഹരി വെറുക്കുന്നവളുടെ മകനുകൊടുത്ത് അവനെ ആദ്യജാതനായി അംഗീകരിക്കണം. അവനാണ് തന്റെ പുരുഷത്വത്തിന്റെ ആദ്യഫലം. ആദ്യജാതന്റെ അവകാശം അവനുള്ളതാണ്.” (ആവര്‍ത്തനം 21 : 15-17).

ചുരുക്കത്തില്‍, ഇസ്മാഈലിനെയല്ല മറിച്ച് ഇസ്ഹാഖിനെയാണ് ബലി നല്‍കാന്‍ ദൈവം നിര്‍ദ്ദേശിച്ചത് എന്ന് ബൈബിള്‍ സ്ഥാപിക്കുന്നുവെന്നും  അതിനാല്‍ ഹജ്ജ് ബലി അബ്രഹാമികമല്ലെന്നുമുള്ള മിഷനറിവാദം എല്ലാ അര്‍ത്ഥത്തിലും അടിസ്ഥാനരഹിതമാണ്.

രു പാട് പ്രശ്നനങ്ങളുടെ പരിഹാരമായാണ് ഇസ്‌ലാം ബഹുഭാര്യത്വം അവതരിപ്പിക്കുന്നത്. അതിൽ വ്യയക്തിമായ പ്രയാസങ്ങളുണ്ട്; സാമൂഹികമായ പ്രശ്നങ്ങളുണ്ട്. ഈ   പ്രശ്‌നങ്ങള്‍ക്ക്  ഇസ്‌ലാമിന്റെ പക്കല്‍ കൃത്യമായ പരിഹാരമാര്‍ഗങ്ങളുണ്ട്. അതിലൊന്നാണ് ബഹുഭാര്യാത്വം. ഇസ്‌ലാമിനെ വിമര്‍ശിക്കുന്നവരുടെ പക്കലോ? പുരുഷലൈംഗീകതയുടെ സ്വാഭാവികതയായി ലൈംഗീകാശാസ്തജ്ഞന്മാർ പറയുന്ന ലൈംഗീകദാരിദ്ര്യം പോലെയുള്ള പ്രശ്നങ്ങൾക്ക് അവരുടെ പക്കലുള്ള പരിഹാരമെന്താണ്? യുദ്ധം മൂലം വിധവകളും അനാഥകളുമുണ്ടാകുന്ന സാഹചര്യത്തില്‍ അവരുടെ പ്രശ്‌നങ്ങള്‍ പരിഹരിക്കുവാന്‍ എന്തുപരിഹാരമാണ് ബഹുഭാര്യത്വ വിരോധികളുടെ പക്കലുള്ളത്?

സാംസ്കാരിക നായകന്മാരെന്നു പറയുന്നവരും മാധ്യമങ്ങളുമെല്ലാം ഈ രംഗത്ത് പലപ്പോഴും കാണിക്കുന്നത് ഇരട്ടമുഖമാണ്. മാതൃഭൂമി ആഴ്ചപ്പതിപ്പ് കുറെ കാലമായി നട്ത്തുന്ന ചർച്ചകൾ ശ്രദ്ധിക്കുക. മാസങ്ങളായുള്ള ചര്‍ച്ച കുടുംബമെന്ന സ്ഥാപനത്തെ തകര്‍ക്കുന്നതങ്ങനെയെന്നതാണ്; ലൈംഗികതയെ വിവാഹത്തില്‍ നിന്നെങ്ങനെ വേര്‍പെടുത്താം എന്നാണ് ചർച്ചകളിൽ പലതിന്റെയും കാതൽ. കുടുംബബാഹ്യമായ ലൈംഗികതക്ക് അംഗീകാരം നല്‍കണമെന്ന രൂപത്തില്‍ സമൂഹത്തിന്റെ മനസാക്ഷിയെ വളര്‍ത്തിക്കൊണ്ടുവരികയെന്ന ലക്ഷ്യത്തോടു കൂടി തന്നെ നിലനില്‍ക്കുന്ന പ്രസിദ്ധീകരണമാണ്  മാതൃഭൂമിയെന്ന് തോന്നിക്കുന്ന തരത്തിലാണ് അതില്‍ ലേഖനങ്ങള്‍ വന്നുകൊണ്ടിരിക്കുന്നത്. സ്ത്രീക്കും പുരുഷനും ലൈംഗിക പങ്കാളികള്‍ എത്രയുമാകാം എന്നാണ് പുരോഗമനക്കാർ എന്ന രൂപത്തിൽ പരിചയപ്പെടുത്തുന്നവരുടെ പക്ഷം. അങ്ങനെ പറയുന്ന ആനുകാലികം തന്നെ കിടപ്പറ പങ്കിടുന്നവരെ നിയമപരമായി ഭാര്യമാരാക്കുമ്പോള്‍ അതിനെതിരെ സംസാരിക്കുന്നത് എങ്ങനെ ന്യായീകരിക്കാനാകും? അത് രണ്ടും കൂടി എങ്ങനെയാണ് യോജിക്കുക.

യുക്തിവാദികളുടെ ആനുകാലികമായ യുക്തിവിചാരം മാസികയില്‍ രസകരമായ ഒരു വൈരുദ്ധ്യമുണ്ടായി. ഒരു ലക്കം യുക്തിവിചാരം എഡിറ്റോറിയലില്‍ ലൈംഗികതയെ വിവാഹത്തില്‍ നിന്ന് വേര്‍പെടുത്തേണ്ടതുണ്ടെന്ന് ശക്തമായി വാദിച്ചു. അഥവാ വിവാഹേതരമായ ബന്ധങ്ങള്‍ക്ക് അംഗീകാരം നല്‍കണമെന്നും അനുവദിക്കണമെന്നുമാണ് പത്രാധിപപക്ഷം. ഇതേ മാഗസിനിലെ അകത്തുള്ള ലേഖനം 'ഇസ്‌ലാമിലെ ബഹുഭാര്യത്വം അപരിഷ്‌കൃതം' എന്നു സ്ഥാപിക്കാന്‍ ഉദ്ദേശിച്ചുകൊണ്ടുള്ളതായിരുന്നു. എന്താണ് ഇതിനര്‍ഥം? എത്ര സ്ത്രീകളുമായും പുരുഷന് ബന്ധപ്പെടാം. പക്ഷെ, അവരെ നിയമപരമായി ഭാര്യമാരാക്കുന്നതാണ് തെറ്റ്. ഇതിന്റെ യുക്തിയും മാനവികതയും ധാര്‍മികതയും നമുക്ക് മനസ്സിലാകുന്നില്ല. ഇത് സ്ത്രീക്ക് അനുകൂലമാണോ? പ്രതികൂലമാണോ? ഫെമിനിസ്റ്റുകള്‍ മറുപടി പറയേണ്ടതാണ്. ഒരു പെണ്ണ് ഭാര്യയായിത്തീരുന്നതാണോ, അഭിസാരികയായിത്തീരുന്നതാണോ നല്ലത്? ഇസ്‌ലാമിന്റെ ഉത്തരം ഒന്നാമത്തേതാണ് നല്ലതെന്നാണ്. രണ്ടാമത്തേതാണ് ശരിയെന്ന് കരുതുന്നവര്‍ക്കത് സ്വീകരിക്കാം. ഇസ്‌ലാമിന് അവിടെ കൃത്യമായ കാഴ്ചപ്പാടുണ്ട്. ആ കാഴ്ചപ്പാടിന്റെ കാതല്‍ പെണ്ണ് ഒരര്‍ഥത്തിലും പീഡിപ്പിക്കപ്പെട്ടുകൂടാ എന്നു തന്നെയാണ്.

ഇസ്‌ലാം അനുവദിച്ച ബഹുഭാര്യത്വം നിലനില്‍ക്കുമ്പോള്‍ ഒരാളുടെ കീഴിലുള്ള ഭാര്യമാരെല്ലാം പുരുഷന്റെ നിയമപരമായ ഇണകളാണ്. അതിലുണ്ടാകുന്ന കുഞ്ഞുങ്ങള്‍ക്ക് നിയമപരമായ പരിരക്ഷ ലഭിക്കും. ആ ഭാര്യമാര്‍ക്കും  കുഞ്ഞുങ്ങള്‍ക്കും അനന്തരാവകാശസ്വത്ത് ലഭിക്കും. എല്ലാ ഭാര്യമാര്‍ക്കും ലൈംഗിക സംതൃപ്തി അനുഭവിക്കാനുള്ള അവസരമുണ്ട്; അവകാശവുമുണ്ട്. അവരോട് അനീതിയോടു കൂടി ഒരു കാരണവശാലും പെരുമാറിക്കൂടാ. പിന്നെയെങ്ങനെയാണ് അത് മാനവിക വിരുദ്ധമാവുക? ഇത് മാനവികവിരുദ്ധമാണെന്ന് വാദിക്കുന്നവര്‍ തന്നെയാണ് ലൈംഗികതയുടെ ഉദാരീകരണത്തിന് വേണ്ടിയും ലൈംഗികതൊഴിലാളികളുടെ അവകാശങ്ങള്‍ക്ക് വേണ്ടിയും കുടുംബബാഹ്യമായ ലൈംഗികതയുടെ അംഗീകാത്തിന് വേണ്ടിയും ശബ്ദമുയര്‍ത്തുന്നത്. ബഹുഭാര്യത്വം നിയമം മൂലം നിരോധിക്കണമെന്ന് വാദിക്കുന്നവര്‍ ധാര്‍മികതയുടെ ആരാച്ചാര്‍മാരാണെന്ന വസ്തുതയാണ് നമുക്കിവിടെ ബോധ്യമാകുന്നത്.

ബഹുഭാര്യത്വ വിരോധികളുടെ ആത്യന്തികലക്ഷ്യം ഇസ്‌ലാമികസമൂഹത്തില്‍ നിലനില്‍ക്കുന്ന ശക്തമായ പാരസ്പര്യത്തിലധിഷ്ഠിതമായ കുടുംബമെന്ന സ്ഥാപനം തകര്‍ക്കുകയാണ്. അത് തകര്‍ത്തുകഴിഞ്ഞാല്‍ മാത്രമേ മുസ്‌ലിംകളെ കൂടി തങ്ങള്‍ക്കാവശ്യമായ ലൈംഗിക ഉദാരീകരണത്തിന്റെ മാര്‍ഗത്തിലേക്ക് കൊണ്ടുവരാന്‍ കഴിയൂ. ഇതാണ് ഏറ്റവും അടിസ്ഥാനപരമായ ലക്ഷ്യം. വിവാഹവും കുടുംബവും തകര്‍ത്തു കഴിഞ്ഞാല്‍ ലൈംഗിക ഉദാരീകരണമാണ് പകരമായി വരുന്നത്. ഫ്രീസെക്‌സിനെക്കുറിച്ച വില്‍ഹം റീഹിന്റെ സങ്കല്‍പത്തിന്റെ പ്രയോഗവല്‍ക്കരണം. എന്നാല്‍ സ്വതന്ത്ര ലൈംഗികതക്കു മുമ്പില്‍ ഇസ്‌ലാമിക നിയമങ്ങള്‍ ശക്തമായ പ്രതിരോധനം തീര്‍ത്ത് നിലകൊള്ളുന്നു. ഒരു പുരുഷന് ഒന്നിലധികം സ്ത്രീകളുമായി ബന്ധപ്പെടേണ്ടി വരികയാണെങ്കില്‍, അതിനവന്റെ പ്രകൃതി അനിവാര്യമാക്കുന്നുവെങ്കില്‍ അവന് വിവാഹം കഴിക്കാനുള്ള അവസരമുണ്ട്. അങ്ങനെയുള്ളവരുടെ മുമ്പില്‍ വിവാഹത്തിന്റെ വാതില്‍ അടഞ്ഞുകഴിഞ്ഞാല്‍ പിന്നീടുള്ള മാര്‍ഗം സ്വതന്ത്രലൈംഗികതയുടേതാണ്.  ലൈംഗിക അരാജകത്വത്തിന് കേരളത്തെ മാര്‍ക്കറ്റാക്കുന്നതിന് വേണ്ടിയുള്ള ഗൂഢാലോചനയുടെ ഭാഗമാണിതെല്ലാം.

ബഹുഭാര്യത്വത്തെ എതിര്‍ക്കുമെന്ന് പുരോഗമനം പറയുന്ന അതേ ആളുകള്‍ തന്നെയാണ് ഇന്ത്യയില്‍  സ്വവര്‍ഗരതി നിയമം മൂലം അനുവദിക്കേണ്ടതുണ്ടെന്ന് പറയുന്നതും. സ്വവര്‍ഗരതി എന്നു പറഞ്ഞാലെന്താണ്? പുരുഷന്മാര്‍ക്ക് പുരുഷന്മാരുമായി ബന്ധപ്പെടാനുള്ള അവസരമാണത്; സ്ത്രീകള്‍ക്ക് സ്ത്രീകളുമായി ബന്ധപ്പെടാനുള്ള അവസരമാണ്. ഒരു പുരുഷന് എത്ര പുരുഷന്മാരുമായും ബന്ധപ്പെടാം; പക്ഷെ, അയാള്‍ ഒന്നിലധികം സ്ത്രീകളെ നിയമാനുസൃതമായ ഇണകളായി സ്വീകരിച്ചുകൂടാ. ഇതാണോ പുരോഗമനവാദം? ഇതില്‍ നിന്നെല്ലാം മനസ്സിലാകുന്നത് ലൈംഗിക ഉദാരീകരണത്തിന് മുസ്‌ലിംങ്ങളെയടക്കം സജ്ജമാക്കുക എന്നതാണ് ബഹുഭാര്യത്വവിരോധനത്തിന്റെ ലക്ഷ്യമെന്നാണ്. ഇസ്‌ലാം പഠിപ്പിക്കുന്ന ബഹുഭാര്യത്വമാകട്ടെ, എല്ലാ കാലത്തും നന്മകൾ മാത്രമേയുണ്ടാക്കൂ. അധാര്മികതകളില്ലാത്ത സമൂഹത്തിന്റെ സൃഷ്ടി ഉദ്ദേശിച്ച് കൊണ്ടുള്ള അനുവാദങ്ങളിലൊന്നാണത്.

വിഷയവുമായി ബന്ധപ്പെട്ട വീഡിയോ

ഖുര്‍ആനില്‍ തന്നെ ബഹുഭാര്യത്വത്തെ നിരുത്സാഹപ്പെടുത്തുന്ന, അത് ഒരിക്കലും സാധ്യമല്ലെന്ന് വ്യക്തമാക്കുന്ന വചനങ്ങളുണ്ടെന്നും അതുകൊണ്ട് തന്നെ ഖുര്‍ആന്‍ ഏകപത്‌നീവ്രതത്തെയാണ്; ബഹുഭാര്യത്വത്തെയല്ല  അംഗീകരിക്കുന്നതെന്നും ചിലര്‍ വാദിക്കാറുണ്ട്. അടിസ്ഥാനരഹിതമാണ് ഈ വാദം.  സൂറത്തുന്നിസാഇലെ 129-ാമത്തെ വചനത്തിന്റെയടിസ്ഥാനത്തിലാണ് ഈ വാദം ഉന്നയിക്കപ്പെടുന്നത്. പ്രസ്തുത വചനം യഥാര്‍ഥത്തില്‍ എന്താണ് വ്യക്തമാക്കുന്നതെന്ന് അത്  കൃത്യമായി പരിശോധിച്ചാല്‍ തന്നെ നമുക്ക് മനസ്സിലാകും.

''നിങ്ങള്‍ എത്രതന്നെ ആഗ്രഹിച്ചാലും ഭാര്യമാര്‍ക്കിടയില്‍ തുല്യനീതി പാലിക്കാന്‍ നിങ്ങള്‍ക്കൊരിക്കലും സാധിക്കുകയില്ല. അതിനാല്‍ നിങ്ങള്‍ (ഒരാളിലേക്ക്) പൂര്‍ണ്ണമായി തിരിഞ്ഞുകൊണ്ട് മറ്റവളെ കെട്ടിയിട്ടവളെപ്പോലെ വിട്ടേക്കരുത്. നിങ്ങള്‍ (പെരുമാറ്റം) നന്നാക്കിത്തീര്‍ക്കുകയും, സൂക്ഷ്മത പാലിക്കുകയും ചെയ്യുന്ന പക്ഷം അല്ലാഹു ഏറെ പൊറുക്കുന്നവനും കരുണാനിധിയുമാകുന്നു'' (ഖുര്‍ആന്‍ 4:129)

ഇങ്ങനെയാണ് പരിശുദ്ധഖുര്‍ആന്‍ പറയുന്നത്. തുല്യനീതി എന്നുള്ളത് കൊണ്ടുള്ള വിവക്ഷ എല്ലാ അര്‍ഥത്തിലുമുള്ള നീതി പാലിക്കലാണ്. ഒരാളുടെ കീഴില്‍ രണ്ടു ഭാര്യമാരുണ്ടാകുമ്പോള്‍ ഒരുവളോട് കൂടുതല്‍ ഇഷ്ടമുണ്ടാകാം. ഇഷ്ടം മാനസികമായ ഒരു പ്രതിഭാസമാണ്. ആ ഇഷ്ടം നിങ്ങളുടെ പെരുമാറ്റങ്ങളെ ഒരര്‍ഥത്തിലും സ്വാധീനിക്കരുതെന്നാണ് ഖുര്‍ആന്‍ പറയുന്നത്. ആയത്ത് പൂര്‍ണമായും വായിക്കാതെ അതിന്റെ  ആദ്യഭാഗം മാത്രം വായിക്കുന്നതിനാലാണ് കുഴപ്പമുണ്ടാകുന്നത്. നിങ്ങള്‍  നിങ്ങള്‍ ഒന്നിലധികം സ്ത്രീകളെ വിവാഹം കഴിച്ചു കൂടെന്നല്ല. മറിച്ച്, തുല്യനീതി പാലിക്കാന്‍ നിങ്ങള്‍ക്ക് സാധ്യമല്ലെന്നാണ് ഈ ഖുര്‍ആന്‍ വചനം വ്യക്തമാക്കുന്നത്. എല്ലാ അര്‍ഥത്തിലുമുള്ള നീതി  മനുഷ്യരെന്ന നിലക്ക് സാധ്യമല്ലാത്ത കാര്യമാണ്. നിങ്ങളുടെ മനസ്സ് എപ്പോഴും നിങ്ങളുടെ അധീനത്തില്‍ മാത്രമല്ല ഉണ്ടാവുക. സ്വാഭാവികമായും രണ്ടാളുകളുടെയും പ്രതികരണങ്ങളുടെ വ്യത്യാസമനുസരിച്ച് നിങ്ങളുടെ മനസ്സില്‍  ഇഷ്ടവും ഇഷ്ടക്കേടുകളും ഉണ്ടാകാം. അവരുടെ സ്വഭാവവും കൂടി ബന്ധപ്പെട്ടുകൊണ്ടാണിതുണ്ടാകുന്നത്. അങ്ങനെ ഒരാളോട് ഇഷ്ടമോ ഇഷ്ടക്കേടോ ഉണ്ടായാലും  നിങ്ങള്‍ ഒരാളെ കെട്ടിയിട്ടതുപോലെയാക്കരുത്. ആ പ്രയോഗം വളരെ കൃത്യമാണ്; ലൈംഗിക ബന്ധത്തിന്റെ കാര്യത്തില്‍ പോലും ഒരാളെ മാത്രം പരിഗണിച്ച്, മറ്റൊരാളെ പരിഗണിക്കാതിരിക്കുന്ന സ്ഥിതിയുണ്ടാവാന്‍ പാടില്ല. ഖുര്‍ആന്‍ പറയുന്നത് അതാണ്.

റസൂൽ (സ) യാണല്ലോ ഖുര്‍ആന്‍ വിശദീകരിച്ചുതരേണ്ടത്. നിങ്ങള്‍ക്കൊരിക്കലും നീതിപുലര്‍ത്താന്‍ സാധ്യമല്ലെന്ന് ഖുര്‍ആന്‍ പറയുമ്പോള്‍, അതുകൊണ്ട് നിങ്ങള്‍ ഒരിക്കലും വിവാഹം ചെയ്തുകൂടാ എന്നാണെന്ന് മനസ്സിലാക്കി ബഹുഭാര്യത്വത്തില്‍ നിന്ന് നബി (സ)ഒഴിഞ്ഞു നിന്നിട്ടില്ല. ഇതില്‍ നിന്ന് സഹാബികളാരും തന്നെ അങ്ങനെ മനസ്സിലാക്കിയിട്ടില്ല. പ്രവാചകനില്‍ നിന്ന് മതം പഠിച്ച ആദിമതലമുറക്കാരാരെങ്കിലും ഖുര്‍ആനില്‍ നിന്ന് ഭാര്യമാരോട് തുല്യനീതി പുലര്‍ത്താന്‍ നമുക്ക് ഒരിക്കലും സാധ്യമല്ല; അതുകൊണ്ട് ഖുര്‍ആന്‍ പറഞ്ഞു തരുന്നത് നമ്മള്‍ മറ്റൊരു വിവാഹം ചെയ്യാന്‍ പാടില്ല എന്നാണ് എന്ന് കരുതി ഏകപത്‌നീവ്രതത്തിലേര്‍പ്പെട്ടതായി കാണുന്നില്ല. ഖുര്‍ആനിലെ ആയത്ത് എന്ത് വിവക്ഷിക്കുന്നുവെന്നത് ആ ആയത്തില്‍ നിന്നു തന്നെ വ്യക്തമാണ്. റസൂലുള്ള (സ) യുടെ വിശദീകരണത്തില്‍ നിന്നും സഹാബിമാരുടെ പ്രയോഗവല്‍ക്കരണത്തില്‍ നിന്നുമത് സ്പഷ്ടമാണ്. അതില്‍ നിന്നും വ്യത്യസ്തമായി പരിശുദ്ധഖുര്‍ആനെ സ്വന്തം ഇഷ്ടമനുസരിച്ച് ദുര്‍വ്യാഖ്യാനം ചെയ്ത് ഖുര്‍ആന്‍ ഉദ്ദേശിക്കാത്ത ഒരര്‍ഥത്തിലേക്ക് ഈ വചനത്തെ വ്യാഖ്യാനിച്ച് കൊണ്ടുപോകുന്നത് ഏതായിരുന്നാലും ശരിയല്ല.

പുരുഷന്മാരെപ്പോലെ ലൈംഗികദാരിദ്ര്യം സ്ത്രീകൾക്കും ഉണ്ടായിക്കൂടെയെന്നും അത്തരം അവസരങ്ങളിൽ എന്തുകൊണ്ടാണ് ഇസ്‌ലാം അവർക്കും ഒന്നിലധികം ഇണകളെ സ്വീകരിക്കുവാനുള്ള അവകാശം നൽകാതിരിക്കുന്നത് എന്ന ചോദ്യമുന്നയിച്ച് ഇസ്‌ലാമിനെ പെൺപീഡനത്തിന്റെ പ്രതിക്കൂട്ടിൽ കയറ്റാൻ ശ്രമിക്കുന്നവരുണ്ട്.

ബഹുഭാര്യത്വമനുവദിച്ച ഇസ്‌ലാം എന്തുകൊണ്ടാണ് ബഹുഭര്‍തൃത്വമനുവദിക്കാത്തത് എന്നാണ് അവർ ഉന്നയിക്കുന്ന ചോദ്യം. ബഹുഭാര്യത്വം പല പ്രശ്‌നങ്ങള്‍ക്കുമുള്ള പരിഹാരമാണ്; ബഹുഭര്‍തൃത്വമാകട്ടെ ഒരു പ്രശ്‌നം മാത്രമാണ്. ഒന്നിനുമുള്ള പരിഹാരമല്ല എന്നാണ് അതിനുള്ള ഉത്തരം. 'ബഹുഭാര്യത്വം സ്വീകരിക്കുവാന്‍ പുരുഷനെ നിര്‍ബന്ധിക്കുന്ന സാഹചര്യങ്ങള്‍ക്ക് സമാനമായ സാഹചര്യങ്ങള്‍ സ്ത്രീകള്‍ക്കുണ്ടായാല്‍ അവര്‍ക്ക് എന്തു പരിഹാരമാണുള്ളത്?' എന്ന ചോദ്യമുയർത്തിക്കൊണ്ടാണ് ഇസ്‌ലാമിനെതിരെ ഇത്തരക്കാർ സംസാരിക്കുന്നത്. . പ്രസ്തുത പ്രശ്‌നങ്ങള്‍ പരിശോധിച്ചാല്‍ ഈ ചോദ്യത്തിന് ഉത്തരം കണ്ടെത്താന്‍ പ്രയാസമുണ്ടാവുകയില്ല.

ഒന്ന്: വൈയക്തികമായ പ്രശ്‌നങ്ങള്‍: സ്ത്രീയുടെ ലൈംഗിക സംതൃപ്തിക്കുവേണ്ടി ഒന്നിലധികം പുരുഷന്മാരെ ആവശ്യമായി വരുന്ന സന്ദര്‍ഭങ്ങള്‍ തീരെയില്ലെന്നുതന്നെ പറയാം. ആരോഗ്യവാനായ ഒരു പുരുഷന്‍തന്നെ സ്ത്രീക്ക് തന്റെ ലൈംഗിക ആവശ്യത്തിന് ധാരാളമാണ്. സ്ത്രീയുടെ ആര്‍ത്തവം, പ്രസവം തുടങ്ങിയ അവസ്ഥകളില്‍ ലൈംഗികാസക്തനായ പുരുഷന്‍ പ്രയാസപ്പെടുന്നതുപോലെ സ്ത്രീയുമായി ബന്ധത്തിന് തടസ്സം നില്‍ക്കുന്ന അവസ്ഥകളൊന്നും സാധാരണ നിലയില്‍ പുരുഷനില്ല. അതുകൊണ്ടുതന്നെ സ്ത്രീക്ക് ലൈംഗിക സംതൃപ്തിക്ക് വേണ്ടി ഒന്നിലധികം പുരുഷന്മാരെ ഭര്‍ത്താക്കന്മാരാക്കേണ്ട ആവശ്യം വരുന്നില്ല.

പുരുഷന്റെ ലൈംഗികശേഷിയില്ലായ്മ, വന്ധ്യത എന്നിവയാണ് മറ്റു മുഖ്യപ്രശ്‌നങ്ങള്‍. പുരുഷനില്‍ വന്ധ്യതക്കുള്ള കാരണങ്ങള്‍ ബീജരാഹിത്യം, ബീജങ്ങളുടെ ചലനശേഷിയില്ലായ്മ, ശുക്ലത്തിലെ ബീജങ്ങളുടെ എണ്ണത്തിലുള്ള കുറവ്, ഉല്‍പാദന ഗ്രന്ഥികളുടെ തകരാറുകള്‍ എന്നിവയാണ്. ഇവയൊന്നും സ്ഥിരമായ വന്ധ്യതക്കുള്ള കാരണമല്ല. എല്ലാം ഫലപ്രദമായ ചികില്‍സകൊണ്ട് മാറ്റാവുന്നതാണ്. പുരുഷന് ലൈംഗിക ശേഷിയില്ലെങ്കില്‍ സ്ത്രീക്ക് അയാളില്‍നിന്ന് വിവാഹമോചനം നേടാവുന്നതാണ്. ലൈംഗികശേഷിയില്ലാത്ത ഒരു പുരുഷനോടൊപ്പം ജീവിക്കുവാന്‍ ഇസ്‌ലാം സ്ത്രീയെ നിര്‍ബന്ധിക്കുന്നില്ല. അത്തരം അവസ്ഥയില്‍ വിവാഹമോചനം തന്നെയാണ് യുക്തമായ പരിഹാരം; ബഹുഭര്‍തൃത്വമല്ല.

രണ്ട്: സാമൂഹികമായ പ്രശ്‌നങ്ങള്‍: പുരുഷന്മാരുടെ എണ്ണം സ്ത്രീകളു ടേതിനേക്കാള്‍ കൂടുന്ന അവസ്ഥയില്‍ ബഹുഭര്‍തൃത്വമനുവദിച്ചുകൂടേയെന്ന് ചോദിക്കാവുന്നതാണ്. ഇത്തരമൊരവസ്ഥ സാധാരണഗതിയില്‍ സംജാതമാവുകയില്ല എന്നതാണ് അതിനുള്ള ഉത്തരം. സാധാരണ നടക്കുന്ന പ്രസവങ്ങളില്‍ പുരുഷന്മാരുടെ എണ്ണം വര്‍ധിക്കുവാനുള്ള സാധ്യത തീരെയില്ല. യുദ്ധങ്ങളിലോ മറ്റോ സ്്രതീകള്‍ കൂടുതലായി കൊല്ലപ്പെടുകയും സ്ത്രീകളേക്കാള്‍ അധികം പുരുഷന്മാര്‍ ഉണ്ടാവുകയും ചെയ്യുന്ന അവസ്ഥയും ഉണ്ടാവുകയില്ല. അപ്പോള്‍ സ്ത്രീ-പുരുഷ അനുപാതത്തില്‍ പുരുഷന്മാരുടെ എണ്ണം വര്‍ധിക്കുകയെന്നത് ഇല്ലാത്ത പ്രശ്‌നമാണ്. അതുകൊണ്ടുതന്നെ അതിനുള്ള പരിഹാരമായി ബഹുഭര്‍തൃത്വം നിര്‍ദേശിക്കുന്നത് വ്യര്‍ഥമാണ്.

ഇന്ത്യയെപ്പോലെയുള്ള രാജ്യങ്ങളില്‍ ഈ അടുത്ത കാലത്തെ ജനസംഖ്യാ കണക്കെടുപ്പില്‍ പുരുഷന്മാരുടെ എണ്ണമാണ് സ്ത്രീകളേക്കാള്‍ കൂടുതലെന്ന വസ്തുത ഈ വാദത്തിനെതിരില്‍ ചൂണ്ടിക്കാണിക്കാവുന്നതാണ്. അതിനുള്ള കാരണമെന്താണ്? സ്ത്രീ ഭ്രൂണഹത്യ. ഗര്‍ഭസ്ഥ ശിശുവിന്റെ ലിംഗനിര്‍ണയം നടത്തി പിറക്കാന്‍ പോകുന്നത് പെണ്‍കുഞ്ഞാണെങ്കില്‍  അതിനെ ഗര്‍ഭത്തില്‍വെച്ചുതന്നെ നശിപ്പിക്കുന്ന ക്രൂരമായ ഏര്‍പ്പാടിന്റെ പരിണിത ഫലമാണിത്. പെണ്‍കുഞ്ഞുങ്ങളെ കൊല്ലൂന്ന പ്രാകൃത സമ്പ്രദായത്തിന്റെ പുനരാഗമനഫലം. ഇത് ഖുര്‍ആന്‍ ശക്തിയായി വിമര്‍ശിച്ചിട്ടുള്ളതാണ് (16:59, 6:137, 17:31, 81:9). അതുകൊണ്ടുതന്നെ ഒരു ഇസ്‌ലാമിക സമൂഹത്തില്‍ പെണ്‍ഭ്രൂണഹത്യകളോ ആണ്‍ഭ്രൂണഹത്യകളോ ഉണ്ടാവില്ല. സ്വാഭാവികമായ പ്രസവം നടക്കുന്ന ഒരു സമൂഹത്തില്‍ സ്ത്രീകളുടെ എണ്ണമാണ് കൂടുതലുണ്ടാവുക. പ്രകൃതിയിലെ സംവിധാനം അങ്ങനെയുള്ളതാണ്.

ഇനി ഒരു രാജ്യത്ത് സ്ത്രീകളുടെ എണ്ണം പുരുഷന്മാരുടേതിനേക്കാള്‍ കുറവാണെങ്കില്‍തന്നെ അവിടെ ജീവിക്കുന്ന പുരുഷന്മാര്‍ക്ക് അയല്‍നാടുകളില്‍പോയി ഭാര്യമാരെ കണ്ടെത്താവുന്നതാണ്. പുറംനാടുകളില്‍ സഞ്ചരിക്കുവാനും അവിടെ ഇണകളെ കണ്ടെത്തുന്നതിനും സ്ത്രീകളേക്കാള്‍ സാധിക്കുക പുരുഷന്മാര്‍ക്കാണ്. അധികം വരുന്ന സ്ത്രീകളോട് പുറം നാടുകളില്‍നിന്ന് ഭര്‍ത്താക്കന്മാരെ കണ്ടെത്തുവാന്‍ പറയുന്നത് തീരെ പ്രായോഗികമല്ല. പുരുഷന്മാരുടെ സ്ഥിതി അതല്ല. തങ്ങള്‍ ജീവിക്കുന്ന പ്രദേശത്ത് സ്ത്രീകള്‍ കുറവാണെങ്കില്‍ അവര്‍ക്ക് അന്യനാടുകളില്‍നിന്ന്  ഇണകളെ കണ്ടെത്തുക അത്രതന്നെ പ്രയാസകരമാവുകയില്ല. സാധാരണഗതിയില്‍ ഇത്തരമൊരവസ്ഥ ഉണ്ടാവുകയില്ലെങ്കിലും അഥവാ ഉണ്ടായാല്‍ അതിനുള്ള പരിഹാരവുമുണ്ട് എന്നതാണ് വാസ്തവം. ബഹുഭര്‍തൃത്വം  അനിവാര്യമായിത്തീരുന്ന യാതൊരു സാഹചര്യങ്ങളുമില്ലാത്തതിനാലാണ് ഇസ്‌ലാം അത് അനുവദിക്കാത്തത് എന്നര്‍ഥം.

എല്ലാ അര്‍ഥത്തിലും പ്രകൃതിവിരുദ്ധവും സ്ത്രീവിരുദ്ധവുമാണ് ബഹുഭര്‍തൃത്വമെന്ന ആശയം. ലൈംഗീക ചോദനകയുടെ നരവംശശാസ്ത്രത്തെക്കുറിച്ച് ഗവേഷണം നടത്തിയ മാര്‍ഡോക്കിന്റെ പഠനം വ്യക്തമാക്കിയത് ബഹുഭർതൃത്വം നില നിന്നിരുന്ന സമൂഹങ്ങൾ വളരെ വിരളമായിരുന്നുവെന്നാണ്. 1170 സംസ്‌കാരങ്ങളെ പഠനവിധേയമാക്കിയപ്പോള്‍ അതിലൊരേയൊരു നാഗരികത മാത്രമാണ് ബഹുഭര്‍തൃത്ത്വം അംഗീകരിച്ചതായി കണ്ടത്. അതുതന്നെ പൂര്‍ണമായ അര്‍ഥത്തിലല്ലതാനും. ഇന്ത്യയില്‍ ബഹുഭര്‍തൃത്വം അനുവദിക്കപ്പെട്ടിരുന്നസമൂഹങ്ങള്‍ അതിനെ കയ്യൊഴിച്ചതും അത് പ്രകൃതി വിരുദ്ധമാണെന്ന് അനുഭവത്തില്‍ നിന്ന് പാഠം പഠിച്ചതുകൊണ്ട് തന്നെയായിരിക്കണം. ഇസ്‌ലാം ബഹുഭര്‍തൃത്ത്വം അംഗീകരിക്കാതിരിക്കാനുള്ള കാരണവും മറ്റൊന്നല്ല. ഇസ്‌ലാം പ്രകൃതിമതമാണ്; പ്രകൃതിവിരുദ്ധവും മാനവവിരുദ്ധവുമായ ഒരാശയവും ഇസ്‌ലാം പഠിപ്പിക്കുന്നില്ല.

ന്റെ മകളുടെ ഭര്‍ത്താവ് മറ്റൊരു വിവാഹത്തിന് ശ്രമിച്ചപ്പോള്‍ നബി (സ) തടഞ്ഞു എന്ന് സ്വഹീഹുൽ ബുഖാരിയിലും സ്വഹീഹ് മുസ്ലിമിലും നിവേദനം ചെയ്യപ്പെട്ട ഹദീഥുകളിലുണ്ട്. ഈ ചരിത്രസംഭവത്തില്‍ നിന്നും പ്രവാചകൻ (സ) ബഹുഭാര്യത്വം നിരുല്‍സാഹപ്പെടുത്തുകയാണ് ചെയ്തിട്ടുള്ളതെന്നും പ്രത്യേകിച്ച് ഭാര്യാപിതാക്കന്മാര്‍ക്ക് അങ്ങനെ തടയാന്‍ അവകാശവും അവസരവുമുണ്ടെന്നെല്ലാമുള്ള വാദം ചിലര്‍ ഉന്നയിക്കാറുണ്ട്. അടിസ്ഥാനരഹിതമാണ് ഈ വാദം.

എന്തുകൊണ്ട് നബി (സ) അലി (റ) േന മറ്റൊരു വിവാഹത്തില്‍ നിന്ന് തടഞ്ഞുവെന്നത് അതേ ഹദീഥിലൂടെ തന്നെ വ്യക്തമാക്കിയിട്ടുണ്ട്. അലി (റ)വിവാഹത്തിന് തുനിഞ്ഞതിനെയല്ല നബി (സ) തടഞ്ഞത്. മറിച്ച്, വിവാഹാലോചന നടന്നത് അന്നത്തെ മുശ്‌രിക്കുകളുടെ നേതാവായ അബൂജഹലിന്റെ മകളുമായിട്ടാണ്. റസൂൽ (സ) പറഞ്ഞു: "അല്ലാഹുവിന്റെ ദൂതന്റെ മകളും അല്ലാഹുവിന്റെ ശത്രുവിന്റെ മകളും ഒരേ പുരുഷന്റെ കീഴില്‍ യോജിക്കുകയില്ല." (ബുഖാരി, കിതാബുനിക്കാഹ്‌)

പ്രവാചകൻ (സ) യുടെ മകള്‍, ഇസ്‌ലാമിന്റെ ശത്രുവിന്റെ മകള്‍, രണ്ടുപേരും ഒരു ഭര്‍ത്താവിന്റെ ഭാര്യമാരായി വരുന്നത് ഭൂഷണമല്ലെന്നും അത് അനുവദിക്കേണ്ടതില്ലെന്നും റസൂൽ (സ) തീരുമാനിക്കുകയായിരുന്നു. അതല്ലാതെ താന്‍ തന്ന സ്വയം ബഹുഭാര്യത്വത്തിലേര്‍പ്പെടുകയും അനുയായികളെയൊന്നും അതില്‍ നിന്ന് തടയാതിരിക്കുകയും ചെയ്ത മുഹമ്മദ് നബി (സ) സ്വന്തം മകളുടെ കാര്യം വന്നപ്പോള്‍ മാത്രം സ്വാര്‍ഥത കാണിച്ചുവെന്ന് ആ ജീവിതത്തെക്കുറിച്ച് സത്യസന്ധമായി പഠിച്ചവരൊന്നും തന്നെ പറയുകയില്ല. പ്രവാചകൻ (സ) യുടെ നിര്‍ദേശങ്ങള്‍ നേരിട്ട് പഠിച്ചവരാണ് സഹാബിമാര്‍. ആ സഹാബിമാരാരെങ്കിലും ഈ തലം വെച്ചുകൊണ്ട് തങ്ങളുടെ മക്കളുടെ ഭര്‍ത്താക്കന്മാരെ അവരുടെ രണ്ടാം വിവാഹത്തില്‍ നിന്ന് തടഞ്ഞതായി നമുക്ക് കാണാന്‍ കഴിയുന്നില്ല. ഇതിൽ നിന്ന് വളരെ വ്യക്തമാണ് കാര്യം. സഹാബിമാരാരും തന്നെ അലി (റ) യെ പ്രവാചകന്‍ രണ്ടാം വിവാഹത്തില്‍ നിന്ന് തടഞ്ഞതില്‍ നിന്ന് അങ്ങനെ മനസ്സിലാക്കിയില്ലെന്നര്‍ഥം.

സഹാബികള്‍ മാത്രമല്ല; താബിഉകളും താബിഊതാബിഉകളും (പ്രവാചകനില്‍ നിന്നും മതം പഠിച്ച മൂന്ന് തലമുറകളും ഈ സംഭവത്തില്‍ നിന്ന് തങ്ങളുടെ മരുമക്കളെ രണ്ടാം വിവാഹത്തില്‍ നിന്ന് തടയാന്‍ തങ്ങള്‍ക്ക് അവകാശമുണ്ടെന്ന് മനസ്സിലാക്കിയിട്ടില്ല. പ്രവാചകനില്‍ നിന്ന് മതം പഠിച്ച ഒന്നാമത്തെ തലമുറ. അതിന്റെടുത്തത് രണ്ടാത്തേത്. അവരില്‍ നിന്ന് പഠിച്ചവര്‍ മൂന്നാമത്തേത്. ഈ മൂന്ന് തലമുറകളെക്കുറിച്ചാകട്ടെ, റസൂലുല്ലാഹി (സ) 'എന്റെ തലമുറയാണ് ഏറ്റവും നല്ല തലമുറയെന്നും അതിന് ശേഷം അതിന് ശേഷം വരുന്ന തലമുറയെന്നും അതിന് ശേഷം അതിന് ശേഷം വരുന്ന തലമുറയെന്നും' പ്രത്യേകമായ അംഗീകാരം നല്‍കിയിട്ടുണ്ട്. അതുകൊണ്ട് ഈ തലമുറകളൊന്നും പ്രവാചകൻ (സ) യുടെ നിര്‍ദേശത്തിന്റെ അടിസ്ഥാനത്തില്‍ വിവാഹത്തിന്റെ കാര്യത്തില്‍ തങ്ങളുടെ പെണ്‍മക്കളുടെ ഭര്‍ത്താക്കന്മാര്‍ മറ്റൊരു വിവാഹം ചെയ്യുന്നത് തടഞ്ഞതായി നമുക്ക് ചരിത്രത്തില്‍ കാണാന്‍ സാധ്യമല്ല. അതിനര്‍ഥം പ്രവാചകനില്‍ നിന്ന് മതം പഠിച്ചവര്‍ അങ്ങനെ മനസ്സിലാക്കിയിട്ടില്ല എന്നാണ്. എന്താണ് തടഞ്ഞതിന് കാരണമെന്ന് മുഹമ്മദ് നബി (സ) തന്നെ വ്യക്തമാക്കിയ സ്ഥിതിക്ക് അതിനെക്കുറിച്ച് പിന്നീടൊരു വിശദീകരണത്തിന്റെ ആവശ്യവുമില്ല.

സൃഷ്ടിപൂജയെ ശക്തമായി വെറുക്കുന്ന ആദര്‍ശമാണ് ഇസ്‌ലാം. അല്ലാഹുവല്ലാത്തവരെ ആരാധിക്കുന്ന സമ്പ്രദായവുമായി ഏതെങ്കിലും രൂപത്തിലുള്ള അടുപ്പം അനുവദിക്കാത്ത ആദര്‍ശം. പ്രവാചകനുമുന്നില്‍ നീ ഞങ്ങളുടെ ദൈവങ്ങളെയും ഞങ്ങള്‍ നിന്റെ ദൈവത്തെയും ആരാധിച്ചുകൊണ്ട് ഒരു ഒത്തുതീര്‍പ്പിലെത്താമെന്ന മൈത്രീനിര്‍ദ്ദേശം വെച്ച മക്കാമുശ്‌രിക്കുകളില്‍ ചിലരോട് മതത്തിന്റെ യാതൊരു വിട്ടു വീഴ്ചയുമില്ലെന്ന് തുറന്നുപ്രഖ്യാപിക്കാനാവശ്യപ്പെടുന്ന ക്വുര്‍ആനിലെ 109-ാം അധ്യായം സൃഷ്ടിപൂജയോടുള്ള ഇസ്‌ലാമിന്റെ വിരോധ വും ഒത്തുതീര്‍പ്പില്ലായ്മയും വ്യക്തമാക്കുന്നുണ്ട്. ''(നബിയേ), പറയുക: അവിശ്വാസികളേ, നിങ്ങള്‍ ആരാധിച്ചുവരുന്നതിനെ ഞാന്‍ ആരാ ധിക്കുന്നില്ല. ഞാന്‍ ആരാധിച്ചുവരുന്നതിനെ നിങ്ങളും ആരാധിക്കുന്നവരല്ല. നിങ്ങള്‍ ആരാധിച്ചുവന്നതിനെ ഞാന്‍ ആരാധിക്കാന്‍ പോകുന്ന വനുമല്ല. ഞാന്‍ ആരാധിച്ചു വരുന്നതിനെ നിങ്ങളും ആരാധിക്കാന്‍ പോകുന്നവരല്ല. നിങ്ങള്‍ക്ക് നിങ്ങളുടെ മതം. എനിക്ക് എന്റെ മതവും.'' (109:1-6)

സൃഷ്ടിപൂജയോടും അവിശ്വാസത്തോടുമുള്ള വെറുപ്പും വിരോധവും ശത്രുതയും വ്യക്തമാക്കുന്ന സാങ്കേതിക ശബ്ദമാണ് 'ബറാഅ്'. വൈയ ക്തികമോ കുടുംബപരമോ ഗോത്രപരമോ വര്‍ഗീയമോ ആയ വെറുപ്പും വിദ്വേഷവുമല്ല, പ്രത്യുത തെറ്റായ ആദര്‍ശങ്ങളോടും അതുള്‍ക്കൊ ള്ളുന്ന ജീവിതക്രമത്തോടുമുള്ള വെറുപ്പും വിരോധവുമാണ് അത് ദ്യോതിപ്പിക്കുന്നത്. സൃഷ്ടിപൂജയുടെ എല്ലാ തലങ്ങളോടും ഒരേസമയം പോരാടിയ മഹാപ്രവാചകനായി ക്വുര്‍ആന്‍ പരിചയപ്പെടുത്തുന്ന ഇബ്‌റാഹീം നബി (അ) തന്റെ പ്രബോധിതരുടെ തെറ്റായ വിശ്വാസങ്ങ ളോടും ജീവിതക്രമത്തോടും ശത്രുത പ്രഖ്യാപിച്ചിരുന്നതായി ക്വുര്‍ആന്‍ വ്യക്തമാക്കുന്നുണ്ട്. ''നിങ്ങള്‍ക്ക് ഇബ്രാഹീമിലും അദ്ദേഹത്തി ന്റെ കൂടെയുള്ളവരിലും ഉത്തമമായ ഒരു മാതൃക ഉണ്ടായിട്ടുണ്ട്. അവര്‍ തങ്ങളുടെ ജനതയോട് ഇപ്രകാരം പറഞ്ഞ സന്ദര്‍ഭം: നിങ്ങളുമാ യും അല്ലാഹുവിന് പുറമെ നിങ്ങള്‍ ആരാധിക്കുന്നവയുമായുള്ള ബന്ധത്തില്‍ നിന്നു തീര്‍ച്ചയായും ഞങ്ങള്‍ ഒഴിവായവരാകുന്നു. നിങ്ങ ളില്‍ ഞങ്ങള്‍ അവിശ്വസിച്ചിരിക്കുന്നു. നിങ്ങള്‍ അല്ലാഹുവില്‍ മാത്രം വിശ്വസിക്കുന്നത് വരെ എന്നെന്നേക്കുമായി ഞങ്ങളും നിങ്ങളും തമ്മില്‍ ശത്രുതയും വിദ്വേഷവും പ്രകടമാവുകയും ചെയ്തിരിക്കുന്നു. തീര്‍ച്ചയായും ഞാന്‍ താങ്കള്‍ക്ക് വേണ്ടി പാപമോചനം തേടാം, താങ്ക ള്‍ക്ക് വേണ്ടി അല്ലാഹുവിങ്കല്‍ നിന്ന് യാതൊന്നും എനിക്ക് അധീനപ്പെടുത്താനാവില്ല എന്ന് ഇബ്രാഹീം തന്റെ പിതാവിനോട് പറഞ്ഞ വാക്കൊഴികെ. (അവര്‍ ഇപ്രകാരം പ്രാര്‍ത്ഥിച്ചിരുന്നു:) ഞങ്ങളുടെ രക്ഷിതാവേ, നിന്റെ മേല്‍ ഞങ്ങള്‍ ഭരമേല്‍പിക്കുകയും, നിന്നിലേക്ക് ഞങ്ങള്‍ മടങ്ങുകയും ചെയ്തിരിക്കുന്നു. നിന്നിലേക്ക് തന്നെയാണ് തിരിച്ചുവരവ്.'' (60:4)

വൈയക്തികമായ വിരോധമോ ശത്രുതയോ അല്ല ഈ വചനത്തില്‍ വെളിപ്പെടുത്തപ്പെട്ട ഇബ്‌റാഹീമീവിരോധമെന്ന് 'നിങ്ങള്‍ അല്ലാഹുവി ല്‍ മാത്രം വിശ്വസിക്കുന്നതുവരെ'യുള്ളതാണ് അതെന്ന പരാമര്‍ശത്തില്‍ നിന്ന് സുതരാം വ്യക്തമാണ്. തെറ്റായ ആദര്‍ശങ്ങളോട് ശത്രുതയും വെറുപ്പും ഉള്ളതുകൊണ്ടാണ് പ്രവാചകന്‍മാര്‍ അവയില്‍നിന്ന് ജനങ്ങളെ രക്ഷപെടുത്താന്‍ ശ്രമിച്ചത്. പ്രസ്തുത പരിശ്രമമാണ് അവരെ ലോകം കണ്ട ഏറ്റവും വലിയ സാമൂഹ്യ സേവകരും മനുഷ്യസ്‌നേഹികളുമാക്കിത്തീര്‍ത്തത്. നിത്യനരകത്തില്‍നിന്ന് സഹജീവികളെ രക്ഷ പെടുത്തുവാന്‍ ശ്രമിക്കുകയെന്നതിനേക്കാള്‍ വലിയ മനുഷ്യസ്‌നേഹമെന്താണ്! താന്‍ നിര്‍വഹിക്കുന്ന ഈ സ്‌നേഹസേവനത്തെക്കുറിച്ച് അന്തിമപ്രവാചകന്‍ അതിസുന്ദരമായ ഒരു ഉപമയിലൂടെ നമ്മെ പഠിപ്പിക്കുന്നുണ്ട്. 'എന്റെയും ജനങ്ങളുടെയും ഉപമ തീ കത്തിച്ചുവെച്ച ഒരു മനുഷ്യന്റേതാണ്. തീയുടെ വെളിച്ചത്തില്‍ ആകൃഷ്ടരായെത്തിയ പാറ്റകളും പറവകളും ആ തീയില്‍ വെന്തെരിഞ്ഞുകൊണ്ടിരുന്നു. അവയെ ആട്ടിമാറ്റിക്കൊണ്ട് തീയില്‍ വീഴാതെ രക്ഷിക്കുവാന്‍ അയാള്‍ പരിശ്രമിച്ചുകൊണ്ടിരുന്നെങ്കിലും അയാളെ അനുസരിക്കാതെ അവ തീയില്‍ വീണുനശിച്ചുകൊണ്ടേയിരുന്നു. അതേപോലെ, നരകാഗ്നിയില്‍ കടക്കാതിരിക്കുവാന്‍ നിങ്ങളുടെ അരപ്പട്ട പിടിച്ച് ഞാന്‍ നരകത്തി ല്‍നിന്ന് നിങ്ങളെ തളളിമാറ്റിക്കൊണ്ടിരിക്കുന്നു; പക്ഷേ നിങ്ങള്‍ അതില്‍ പതിക്കണമെന്ന് നിര്‍ബന്ധം പുലര്‍ത്തിക്കൊണ്ടിരിക്കുകയാണ്.' (ബുഖാരി, മുസ്‌ലിം).

നരകത്തെയും നരകത്തിലെത്തിക്കുന്ന പാതകളെയും വെറുക്കുന്ന സത്യവിശ്വാസികള്‍ പ്രസ്തുത പാതകളില്‍ നിന്ന് തങ്ങളുടെ സഹജീവി കളെ രക്ഷിക്കുവാന്‍ വേണ്ടി നടത്തുന്ന സേവന പ്രവര്‍ത്തനമാണ് ഇസ്‌ലാമിക പ്രബോധനം. വെറുപ്പില്‍ നിന്നല്ല, സ്‌നേഹത്തില്‍ നിന്നാണ് പ്രസ്തുത പ്രവര്‍ത്തനമുണ്ടാകുന്നത്. രോഗത്തെ വെറുക്കുകയും രോഗിയെ സ്‌നേഹിക്കുകയും ചെയ്തുകൊണ്ട് ഭിഷഗ്വരന്‍ നടത്തുന്ന ചികിത്സ പോലെയാണത്. പ്രബോധിതരില്‍ നിന്ന് യാതൊരു പ്രതിഫലവും സ്വീകരിക്കാതെ അവരുടെ നന്മ മാത്രം ലക്ഷ്യമാക്കിയുള്ള പ്രവര്‍ ത്തനം! പ്രബോധനപ്രവര്‍ത്തനങ്ങള്‍ എത്രത്തോളം മാന്യവും സ്‌നേഹമസൃണവുമാകണമെന്ന് പ്രവാചകന്‍മാരുടെ പ്രബോധനചരിത്രം വിവരിച്ചുകൊണ്ട് ക്വുര്‍ആന്‍ പഠിപ്പിക്കുന്നുണ്ട്. ''ആദ് സമുദായം ദൈവദൂതന്‍മാരെ നിഷേധിച്ചു തള്ളി. അവരുടെ സഹോദരന്‍ ഹൂദ് അവരോട് പറഞ്ഞ സന്ദര്‍ഭം: നിങ്ങള്‍ സൂക്ഷ്മത പാലിക്കുന്നില്ലേ? തീര്‍ച്ചയായും ഞാന്‍ നിങ്ങള്‍ക്ക് വിശ്വസ്തനായ ഒരു ദൂതനാകുന്നു. അതിനാല്‍ നിങ്ങള്‍ അല്ലാഹുവെ സൂക്ഷിക്കുകയും, എന്നെ അനുസരിക്കുകയും ചെയ്യുവിന്‍.ഇതിന്റെ പേരില്‍ ഞാന്‍ നിങ്ങളോട് യാതൊ രു പ്രതിഫലവും ചോദിക്കുന്നില്ല. എനിക്കുള്ള പ്രതിഫലം ലോകരക്ഷിതാവിങ്കല്‍ നിന്ന് മാത്രമാകുന്നു.'' (26:123-127)

ഇക്കാര്യം തന്നെ സ്വാലിഹ് നബിയും (''ഥമൂദ് സമുദായം ദൈവദൂതന്‍മാരെ നിഷേധിച്ചു തള്ളി. അവരുടെ സഹോദരന്‍ സ്വാലിഹ് അവ രോട് പറഞ്ഞ സന്ദര്‍ഭം: നിങ്ങള്‍ സൂക്ഷ്മത പാലിക്കുന്നില്ലേ? തീര്‍ച്ചയായും ഞാന്‍ നിങ്ങള്‍ക്ക് വിശ്വസ്തനായ ഒരു ദൂതനാകുന്നു. അതിനാല്‍ നിങ്ങള്‍ അല്ലാഹുവെ സൂക്ഷിക്കുകയും, എന്നെ അനുസരിക്കുകയും ചെയ്യുവിന്‍. നിങ്ങളോട് ഞാന്‍ ഇതിന്റെ പേരില്‍ യാതൊരു പ്രതിഫ ലവും ചോദിക്കുന്നില്ല. എനിക്കുള്ള പ്രതിഫലം ലോകരക്ഷിതാവിങ്കല്‍ നിന്ന് മാത്രമാകുന്നു.'' (26:141-145)) ലൂത്വ് നബിയും (''ലൂത്വിന്റെ ജനത ദൈവദൂതന്‍മാരെ നിഷേധിച്ചു തള്ളി. അവരുടെ സഹോദരന്‍ ലൂത്വ് അവരോട് പറഞ്ഞ സന്ദര്‍ഭം: നിങ്ങള്‍ സൂക്ഷ്മത പാലിക്കുന്നില്ലേ? തീര്‍ച്ചയായും ഞാന്‍ നിങ്ങള്‍ക്ക് വിശ്വസ്തനായ ഒരു ദൂതനാകുന്നു. അതിനാല്‍ നിങ്ങള്‍ അല്ലാഹുവെ സൂക്ഷിക്കുകയും, എന്നെ അനുസരി ക്കുകയും ചെയ്യുവിന്‍. ഇതിന്റെ പേരില്‍ നിങ്ങളോട് ഞാന്‍ യാതൊരു പ്രതിഫലവും ചോദിക്കുന്നില്ല. എനിക്കുള്ള പ്രതിഫലം ലോകരക്ഷി താവിങ്കല്‍ നിന്ന് മാത്രമാകുന്നു.'' (26:160-164)) ശുഐബ് നബിയും (''ഐക്കത്തില്‍ (മരക്കൂട്ടങ്ങള്‍ക്കിടയില്‍) താമസിച്ചിരുന്നവരും ദൈവദൂത ന്‍മാരെ നിഷേധിച്ചുതള്ളി. അവരോട് ശുഐബ് പറഞ്ഞ സന്ദര്‍ഭം: നിങ്ങള്‍ സൂക്ഷ്മത പാലിക്കുന്നില്ലേ? തീര്‍ച്ചയായും ഞാന്‍ നിങ്ങള്‍ക്ക് വിശ്വസ്തനായ ഒരു ദൂതനാകുന്നു. അതിനാല്‍ നിങ്ങള്‍ അല്ലാഹുവെ സൂക്ഷിക്കുകയും, എന്നെ അനുസരിക്കുകയും ചെയ്യുവിന്‍. ഇതിന്റെ പേരില്‍ യാതൊരു പ്രതിഫലവും ഞാന്‍ നിങ്ങളോട് ചോദിക്കുന്നില്ല. എനിക്കുള്ള പ്രതിഫലം ലോകരക്ഷിതാവിങ്കല്‍ നിന്ന് മാത്രമാകുന്നു. നിങ്ങള്‍ അളവു പൂര്‍ത്തിയാക്കികൊടുക്കുക. നിങ്ങള്‍ (ജനങ്ങള്‍ക്ക്) നഷ്ടമുണ്ടാക്കുന്നവരുടെ കൂട്ടത്തിലാകരുത്. കൃത്രിമമില്ലാത്ത തുലാസ് കൊണ്ട് നിങ്ങള്‍ തൂക്കുക. ജനങ്ങള്‍ക്ക് അവരുടെ സാധനങ്ങളില്‍ നിങ്ങള്‍ കമ്മിവരുത്തരുത്. നാശകാരികളായിക്കൊണ്ട് നിങ്ങള്‍ ഭൂമിയില്‍ അതിക്രമം പ്രവര്‍ത്തിക്കരുത്. നിങ്ങളെയും പൂര്‍വ്വതലമുറകളെയും സൃഷ്ടിച്ചവനെ നിങ്ങള്‍ സൂക്ഷിക്കുകയും ചെയ്യുക.'' (26:176-184)) പറ ഞ്ഞിരുന്നതായി ക്വുര്‍ആന്‍ വ്യക്തമാക്കുന്നു. അത്യുന്നതനായ രക്ഷിതാവ് താന്‍ തന്നെയാണെന്ന് സ്വയം പ്രഖ്യാപിക്കുകയും (79:24) ഇസ്രയീ ല്യരിലെ ആണ്‍കുട്ടികളെയെല്ലാം കൊല്ലാന്‍ ഉത്തരവിടുകയും (2:49) പീഡനങ്ങളാല്‍ അവരെ പ്രയാസപ്പെടുത്തുകയും (7:141) ചെയ്ത ഫിര്‍ ഔനിനെ ദൈവമാര്‍ഗത്തിലേക്ക് ക്ഷണിക്കുവാന്‍ വേണ്ടിയുള്ള മൂസ(അ)യോടുള്ള ദൈവകല്‍പനയില്‍പ്പോലും സൗമ്യത വെടിയാതെ യായിരിക്കണം അതു നിര്‍വഹിക്കേണ്ടതെന്നു പറയുന്നതില്‍നിന്ന് എത്രത്തോളം സ്‌നേഹത്തോടെയായിരിക്കണം പ്രബോധകകൃത്യങ്ങള്‍ നിര്‍വഹിക്കേണ്ടതെന്ന് മനസ്സിലാകുന്നുണ്ട്. ''നിങ്ങള്‍ രണ്ടുപേരും ഫിര്‍ഔന്റെ അടുത്തേക്ക് പോകുക. തീര്‍ച്ചയായും അവന്‍ അതിക്രമകാ രിയായിരിക്കുന്നു. എന്നിട്ട് നിങ്ങള്‍ അവനോട് സൗമ്യമായ വാക്ക് പറയുക. അവന്‍ ഒരു വേള ചിന്തിച്ച് മനസ്സിലാക്കിയേക്കാം. അല്ലെങ്കില്‍ ഭയപ്പെട്ടുവെന്ന് വരാം.'' (20:43, 44)

യുക്തിദീക്ഷയോടെയും സദുപദേശങ്ങളിലൂടെയും സ്‌നേഹസംവാദങ്ങളിലൂടെയുമാകണം സത്യമത പ്രബോധനമെന്നും നല്ല വര്‍ത്തമാനം മാത്രമാണ് സത്യവിശ്വാസികള്‍ പറയേണ്ടതെന്നും പഠിപ്പിക്കുന്ന ക്വുര്‍ആന്‍ വെറുപ്പും വിദ്വേഷവും പ്രകടിപ്പിച്ചുകൊണ്ടല്ല സൗമ്യതയും സ്‌നേഹവും വെളിപ്പെടുത്തിക്കൊണ്ടാണ് ആദര്‍ശപ്രബോധനം നടത്തേണ്ടതെന്നു തന്നെയാണ് വ്യക്തമാക്കുന്നത്. ''യുക്തിദീക്ഷയോടു കൂടി യും, സദുപദേശം മുഖേനയും നിന്റെ രക്ഷിതാവിന്റെ മാര്‍ഗത്തിലേക്ക് നീ ക്ഷണിച്ച് കൊള്ളുക. ഏറ്റവും നല്ല രീതിയില്‍ അവരുമായി സംവാദം നടത്തുകയും ചെയ്യുക. തീര്‍ച്ചയായും നിന്റെ രക്ഷിതാവ് തന്റെ മാര്‍ഗം വിട്ട് പിഴച്ച് പോയവരെപ്പറ്റി നല്ലവണ്ണം അറിയുന്ന വനത്രെ. സന്‍മാര്‍ഗം പ്രാപിച്ചവരെപ്പറ്റിയും നല്ലവണ്ണം അറിയുന്നവനത്രെ.'' (16:125). ''നീ എന്റെ ദാസന്‍മാരോട് പറയുക; അവര്‍ പറയു ന്നത് ഏറ്റവും നല്ല വാക്കായിരിക്കണമെന്ന്. തീര്‍ച്ചയായും പിശാച് അവര്‍ക്കിടയില്‍ (കുഴപ്പം) ഇളക്കിവിടുന്നു. തീര്‍ച്ചയായും പിശാച് മനുഷ്യന് പ്രത്യക്ഷ ശത്രുവാകുന്നു.'' (17:53)

സൃഷ്ടിപൂജയോടും ബഹുദൈവാരാധനയോടും അവിശ്വാസത്തോടും വെറുപ്പുള്ളതോടൊപ്പം തന്നെ അവയുടെ നിരര്‍ത്ഥക ബോധ്യപ്പെടു ത്തിക്കൊണ്ടാകണം, അല്ലാതെ പരസ്പരം തെറി പറഞ്ഞുകൊണ്ടാകരുത് പ്രബോധനം നിര്‍വഹിക്കേണ്ടതെന്നു പഠിപ്പിക്കുന്ന ക്വുര്‍ആന്‍ ആ രംഗത്തുണ്ടാകേണ്ട മാന്യതയെന്തായിരിക്കണമെന്നാണ് വിശ്വാസികളെ തെര്യപ്പെടുത്തുന്നത്. അല്ലാഹുവല്ലാത്തവരൊന്നും ആരാധിക്ക പ്പെട്ടുകൂടായെന്നു സമര്‍ത്ഥിക്കുകയല്ലാതെ മറ്റുള്ളവരുടെ ആരാധ്യന്‍മാരെ തെറിപറയുന്ന പതനത്തിലേക്ക് പ്രബോധകര്‍ എത്തിപ്പെട്ടാല്‍ അത് പരസ്പരമുള്ള തെറിയഭിഷേകങ്ങള്‍ക്കുമാത്രമേ കാരണമാകൂവെന്നും അതല്ല മാന്യമായ ആദര്‍ശപ്രബോധനമാണ് അല്ലാഹു കാംക്ഷി ക്കുന്നതെന്നും വ്യക്തമാക്കുന്ന ക്വുര്‍ആന്‍ വചനം നോക്കുക. ''അല്ലാഹുവിനു പുറമെ അവര്‍ വിളിച്ച് പ്രാര്‍ത്ഥിക്കുന്നവരെ നിങ്ങള്‍ ശകാരി ക്കരുത്. അവര്‍ വിവരമില്ലാതെ അതിക്രമമായി അല്ലാഹുവെ ശകാരിക്കാന്‍ അത് കാരണമായേക്കും. അപ്രകാരം ഓരോ വിഭാഗത്തിനും അവരുടെ പ്രവര്‍ത്തനം നാം ഭംഗിയായി തോന്നിച്ചിരിക്കുന്നു. പിന്നീട് അവരുടെ രക്ഷിതാവിങ്കലേക്കാണ് അവരുടെ മടക്കം. അവര്‍ ചെ യ്തുകൊണ്ടിരുന്നതിനെപ്പറ്റിയെല്ലാം അപ്പോള്‍ അവന്‍ അവരെ അറിയിക്കുന്നതാണ്.'' (6:108)

മാന്യമായി ആദര്‍ശപ്രബോധനം നിര്‍വഹിച്ചുകൊണ്ട് ജീവിക്കുന്ന സത്യവിശ്വാസികളോട് ശത്രുതയും വിദ്വേഷവും വെച്ചുപുലര്‍ത്തു കയും സ്വന്തം വീടുകളില്‍ നിന്ന് അവരെ പുറത്താക്കാന്‍ ശ്രമിക്കുകയും അവരോട് യുദ്ധം ചെയ്യുകയും ദൈവിക മാര്‍ഗനിര്‍ദ്ദേശങ്ങളില്‍ നിന്ന് അവരെ തെറ്റിക്കുവാന്‍ തക്കം പാര്‍ത്തിരിക്കുകയും ചെയ്യുന്നവരോട് സത്യവിശ്വാസികളും ശത്രുത പ്രഖ്യാപിക്കണമെന്നും അവരു മായി സ്‌നേഹബന്ധം പാടില്ലെന്നും തന്നെയാണ് ഇസ്‌ലാമിന്റെ നിലപാട്. ഈ നിലപാട് പ്രഖ്യാപിക്കുന്നതാണ് സൂറത്തുല്‍ മുംതഹിനയി ലെ ഒന്നാമത്തെ വചനം. ''ഹേ; സത്യവിശ്വാസികളേ, എന്റെ ശത്രുവും നിങ്ങളുടെ ശത്രുവും ആയിട്ടുള്ളവരോട് സ്‌നേഹബന്ധം സ്ഥാപിച്ച് കൊണ്ട് നിങ്ങള്‍ അവരെ മിത്രങ്ങളാക്കി വെക്കരുത്. നിങ്ങള്‍ക്കു വന്നുകിട്ടിയിട്ടുള്ള സത്യത്തില്‍ അവര്‍ അവിശ്വസിച്ചിരിക്കുകയാണ്. നിങ്ങള്‍ നിങ്ങളുടെ രക്ഷിതാവായ അല്ലാഹുവില്‍ വിശ്വസിക്കുന്നതിനാല്‍ റസൂലിനെയും നിങ്ങളെയും അവര്‍ നാട്ടില്‍ നിന്നു പുറ ത്താക്കു ന്നു. എന്റെ മാര്‍ഗത്തില്‍ സമരം ചെയ്യുവാനും എന്റെ പ്രീതിതേടുവാനും നിങ്ങള്‍ പുറപ്പെട്ടിരിക്കുകയാണെങ്കില്‍ (നിങ്ങള്‍ അപ്രകാരം മൈത്രീ ബന്ധം സ്ഥാപിക്കരുത്). നിങ്ങള്‍ അവരുമായി രഹസ്യമായി സ്‌നേഹബന്ധം സ്ഥാപിക്കുന്നു. നിങ്ങള്‍ രഹസ്യമാക്കിയതും പരസ്യ മാക്കിയതും ഞാന്‍ നല്ലവണ്ണം അറിയുന്നവനാണ്. നിങ്ങളില്‍ നിന്ന് വല്ലവനും അപ്രകാരം പ്രവര്‍ത്തിക്കുന്ന പക്ഷം അവന്‍ നേര്‍മാര്‍ഗ ത്തില്‍ നിന്ന് പിഴച്ചു പോയിരിക്കുന്നു.''(60:1)

അല്ലാഹുവിനോടും ദൂതനോടും ശത്രുത പ്രഖ്യാപിക്കുകയും സത്യവിശ്വാസികളെ ജീവിക്കാനനുവദിക്കാതിരിക്കുകയും ചെയ്യുന്നവര്‍ അവര്‍ എത്രതന്നെ അടുത്ത ബന്ധമുള്ളവരാണെങ്കില്‍പോലും അവരോട് സത്യവിശ്വാസികള്‍ക്ക് ആത്മാര്‍ത്ഥമായ സ്‌നേഹബന്ധം കഴിയി ല്ലെന്ന വസ്തുത ക്വുര്‍ആന്‍ വ്യക്തമാക്കുന്നുണ്ട്. ''അല്ലാഹുവിലും അന്ത്യദിനത്തിലും വിശ്വസിക്കുന്ന ഒരു ജനത അല്ലാഹുവോടും അവ ന്റെ റസൂലിനോടും എതിര്‍ത്തു നില്‍ക്കുന്നവരുമായി സ്‌നേഹബന്ധം പുലര്‍ത്തുന്നത് നീ കണ്ടെത്തുകയില്ല. അവര്‍ (എതിര്‍പ്പുകാര്‍) അവ രുടെ പിതാക്കളോ, പുത്രന്‍മാരോ, സഹോദരന്‍മാരോ ബന്ധുക്കളോ ആയിരുന്നാല്‍ പോലും. അത്തരക്കാരുടെ ഹൃദയങ്ങളില്‍ അല്ലാഹു വിശ്വാസം രേഖപ്പെടുത്തുകയും അവന്റെ പക്കല്‍ നിന്നുള്ള ഒരു ആത്മചൈതന്യം കൊണ്ട് അവന്‍ അവര്‍ക്ക് പിന്‍ബലം നല്‍കുകയും ചെയ്തിരിക്കുന്നു. താഴ്ഭാഗത്തു കൂടി അരുവികള്‍ ഒഴുകുന്ന സ്വര്‍ഗത്തോപ്പുകളില്‍ അവന്‍ അവരെ പ്രവേശിപ്പിക്കുകയും ചെയ്യും. അവ രതില്‍ നിത്യവാസികളായിരിക്കും. അല്ലാഹു അവരെ പറ്റി തൃപ്തിപ്പെട്ടിരിക്കുന്നു. അവര്‍ അവനെ പറ്റിയും തൃപ്തിപ്പെട്ടിരിക്കുന്നു. അത്തരക്കാരാകുന്നു അല്ലാഹുവിന്റെ കക്ഷി. അറിയുക: തീര്‍ച്ചയായും  അല്ലാഹുവിന്റെ കക്ഷിയാകുന്നു വിജയം പ്രാപിക്കുന്നവര്‍.'' (58:22)

എന്നാല്‍ ഈ ശത്രുതയും വെറുപ്പമെല്ലാം ഇസ്‌ലാമിനോടും മുസ്‌ലിംകളോടും ശത്രുത പ്രഖ്യാപിക്കുകയും അവരെ പീഡിപ്പിക്കുകയും ചെയ്യുന്നവരോടുമാത്രമാണ്. അതല്ലാത്ത അമുസ്‌ലിംകളോട് മാന്യമായി വര്‍ത്തിക്കുകയും നീതിനിഷ്ഠമായി പെരുമാറുകയും നന്മയോടെ സഹവര്‍ത്തിക്കുകയും ചെയ്യണമെന്നു തന്നെയാണ് ഇസ്‌ലാം പഠിപ്പിക്കുന്നത്. ഇസ്‌ലാമിന്റെ ശത്രുക്കളോട് ശത്രുത പ്രഖ്യാപിക്കണമെന്നു പഠിപ്പിച്ചു തുടങ്ങുന്ന സൂറത്തുല്‍ മുംതഹനയില്‍ തന്നെ അത് എങ്ങനെയുള്ളവരോടാണെന്നും ശത്രുതയില്ലാത്ത മറ്റുള്ള അമുസ്‌ലിംകളോട് എന്തുനിലപാടാണ് സ്വീകരിക്കേണ്ടെതെന്നും കൂടി വ്യക്തമാക്കുന്നുണ്ട്. ''മതകാര്യത്തില്‍ നിങ്ങളോട് യുദ്ധം ചെയ്യാതിരിക്കുകയും, നിങ്ങ ളുടെ വീടുകളില്‍ നിന്ന് നിങ്ങളെ പുറത്താക്കാതിരിക്കുകയും ചെയ്യുന്നവരെ സംബന്ധിച്ചിടത്തോളം നിങ്ങളവര്‍ക്ക് നന്‍മ ചെയ്യുന്നതും നിങ്ങളവരോട് നീതി കാണിക്കുന്നതും അല്ലാഹു നിങ്ങളോട് നിരോധിക്കുന്നില്ല. തീര്‍ച്ചയായും അല്ലാഹു നീതി പാലിക്കുന്നവരെ ഇഷ്ടപ്പെ ടുന്നു.  മതകാര്യത്തില്‍ നിങ്ങളോട് യുദ്ധം ചെയ്യുകയും നിങ്ങളുടെ വീടുകളില്‍ നിന്ന് നിങ്ങളെ പുറത്താക്കുകയും നിങ്ങളെ പുറത്താക്കുന്ന തില്‍ പരസ്പരം സഹകരിക്കുകയും ചെയ്തവരെ സംബന്ധിച്ചുമാത്രമാണ് -അവരോട് മൈത്രികാണിക്കുന്നത് - അല്ലാഹു നിരോധിക്കു ന്നത്. വല്ലവരും അവരോട് മൈത്രീ ബന്ധം പുലര്‍ത്തുന്ന പക്ഷം അവര്‍ തന്നെയാകുന്നു അക്രമകാരികള്‍.'' (60:8,9)

ആദിമാതാപിതാക്കളില്‍ നിന്നുണ്ടായ മനുഷ്യരെല്ലാം അടിസ്ഥാനപരമായി സഹോദരങ്ങളാണെന്ന് പഠിപ്പിക്കുന്ന മതമാണ് ഇസ്‌ലാം. ജന്മ ത്തിന്റെ പേരിലുള്ള ഉച്ഛനീചത്വങ്ങളെല്ലാം അടിസ്ഥാനരഹിതമാകുന്നത് ഈ സാഹോദര്യത്തിന്റെ വെളിച്ചത്തിലാണെന്നും വിമലമായ വിശ്വാസവും വിശുദ്ധമായ ജീവിതവുമാണ് മനുഷ്യരെ ഔന്നത്യത്തിലേക്ക് നയിക്കുന്നതെന്നാണ് ഈ സാഹോദര്യം വ്യക്തമാക്കുന്നതെ ന്നുമാണ് ക്വുര്‍ആന്‍ പഠിപ്പിക്കുന്നത്. ''ഹേ; മനുഷ്യരേ, തീര്‍ച്ചയായും നിങ്ങളെ നാം ഒരു ആണില്‍ നിന്നും ഒരു പെണ്ണില്‍ നിന്നുമായി സൃഷ്ടി ച്ചിരിക്കുന്നു. നിങ്ങള്‍ അന്യോന്യം അറിയേണ്ടതിന് നിങ്ങളെ നാം വിവിധ സമുദായങ്ങളും ഗോത്രങ്ങളും ആക്കുകയും ചെയ്തിരിക്കുന്നു. തീര്‍ച്ചയായും അല്ലാഹുവിന്റെ അടുത്ത് നിങ്ങളില്‍ ഏറ്റവും ആദരണീയന്‍ നിങ്ങളില്‍ ഏറ്റവും ധര്‍മ്മനിഷ്ഠ പാലിക്കുന്നവനാകുന്നു. തീര്‍ ച്ചയായും അല്ലാഹു സര്‍വജ്ഞനും സൂക്ഷ്മജ്ഞാനിയുമാകുന്നു.'' (ക്വുര്‍ആന്‍ 49:13)

പ്രകൃതിപരവും പ്രാദേശികവുമായ സാഹോദര്യങ്ങളെല്ലാം ഇസ്‌ലാം അംഗീകരിക്കുന്നുണ്ട്. ആദമിന്റെ മക്കളായ രണ്ടുപേരെക്കുറിച്ച് പരാമര്‍ശിക്കുമ്പോള്‍ ക്വുര്‍ആന്‍ സല്‍കര്‍മകാരിയെയും ദുഷ്‌കര്‍മകാരിയെയും സഹോദരങ്ങളായിത്തന്നെയാണ് പരിചയപ്പെടുത്തുന്നത്. ലോകരക്ഷിതാവായ അല്ലാഹുവിനെ ഭയപ്പെട്ടുകൊണ്ട് ആയുധമെടുക്കാതിരുന്ന സദ്‌വൃത്തനെയും അല്ലാഹുവെ ധിക്കരിച്ച് മറ്റെയാളെ കൊലപ്പെടുത്തിയ ദുര്‍വൃത്തനെയും കുറിച്ച് പരാമര്‍ശിക്കുമ്പോള്‍ മൂന്നുതവണ ക്വുര്‍ആന്‍ അവരെക്കുറിച്ച് സഹോദരങ്ങളെന്ന് പരിചയ പ്പെടുത്തുന്നുണ്ട്. വിശ്വാസകര്‍മങ്ങളിലെ നന്മയും തിന്മയുമൊന്നും പ്രകൃതിപരമായ സാഹോദര്യത്തെ ബാധിക്കുകയില്ല. ''എന്നിട്ട് തന്റെ സഹോദരനെ കൊല്ലുവാന്‍ അവന്റെ മനസ്സ് അവന് പ്രേരണ നല്‍കി. അങ്ങനെ അവനെ കൊലപ്പെടുത്തി. അതിനാല്‍ അവന്‍ നഷ്ടക്കാരില്‍ പെട്ടവനായിത്തീര്‍ന്നു. അപ്പോള്‍ തന്റെ സഹോദരന്റെ മൃതദേഹം മറവു ചെയ്യേണ്ടത് എങ്ങനെയെന്ന് അവന് കാണിച്ചുകൊടുക്കുവാനാ യി നിലത്ത് മാന്തികുഴിയുണ്ടാക്കുന്ന ഒരു കാക്കയെ അല്ലാഹു അയച്ചു. അവന്‍ പറഞ്ഞു: എന്തൊരു കഷ്ടം! എന്റെ സഹോദരന്റെ മൃത ദേഹം മറവുചെയ്യുന്ന കാര്യത്തില്‍ ഈ കാക്കയെപ്പോലെ ആകാന്‍ പോലും എനിക്ക് കഴിയാതെ പോയല്ലോ. അങ്ങനെ അവന്‍ ഖേദക്കാരു ടെ കൂട്ടത്തിലായിത്തീര്‍ന്നു.''(5:30,31)

ജീവിക്കുന്ന പ്രദേശത്തുള്ളവരെ സഹോദരങ്ങളായി കാണുന്നതിനും വിശ്വാസമോ കര്‍മമോ തടസ്സമാകുന്നില്ലെന്നു തന്നെയാണ് ക്വുര്‍ആനി കാധ്യാപനങ്ങള്‍ നമ്മെ പഠിപ്പിക്കുന്നത്. ആദ് ജനതയിലേക്ക് അയക്കപ്പെട്ട ഹൂദ് നബി(അ)യെയും ഥമൂദ് ജനതയിലേക്ക് അയക്കപ്പെട്ട സ്വാലി ഹ് നബി(അ)യെയും ക്വുര്‍ആന്‍ പരിചയപ്പെടുത്തുന്നത് അയക്കപ്പെടുന്ന ജനതയുടെ സഹോദരങ്ങളായാണ്. സൃഷ്ടിപൂജയിലധിഷ്ഠിതമായ ജീവിതം നയിക്കുകയും പ്രവാചകന്‍മാരെ തള്ളിക്കളയുകയും ചെയ്ത ദൈവധിക്കാരികളായ ജനതയെക്കുറിച്ചാണ് മഹാന്‍മാരായ രണ്ടു പ്രവാചകന്‍മാരുടെ സഹോദരങ്ങളെന്ന് പരാമര്‍ശിച്ചിട്ടുള്ളതെന്ന വസ്തുത ശ്രദ്ധേയമാണ്. ആദര്‍ശവ്യത്യാസമോ ജീവിതക്രമത്തോടുള്ള വിരോധമോ ഒന്നും തന്നെ പ്രദേശവാസികള്‍ തമ്മിലുള്ള സാഹോദര്യത്തെ തിരസ്‌കരിക്കുന്നില്ലെന്ന് ഈ വചനങ്ങള്‍ വ്യക്തമാക്കുന്നുണ്ട്. അല്ലാഹുവിനെ ധിക്കരിക്കുകയും ദൈവദൂതന്‍മാരെ തള്ളിക്കളയുകയും അവരിലൂടെയുണ്ടായ ദൃഷ്ടാന്തങ്ങളെ കളവാക്കുകയും ചെയ്തി ട്ടുപോലും ആ ജനതകളെ പ്രവാചകന്‍മാരുടെ സഹോദരങ്ങളായി പരിചയപ്പെടുത്തുകവഴി മനുഷ്യരെ മൊത്തത്തിലും നാട്ടുകാരെ വിശേ ഷിച്ചും സഹോദരങ്ങളായി സംബോധന ചെയ്യുന്നതില്‍ തെറ്റൊന്നുമില്ലെന്ന വലിയ പാഠവും ഈ വചനങ്ങള്‍ പഠിപ്പിക്കുന്നുണ്ട്. ''ആദ് ജനത യിലേക്ക് അവരുടെ സഹോദരനായ ഹൂദിനെയും (നാം നിയോഗിക്കുകയുണ്ടായി). അദ്ദേഹം പറഞ്ഞു: എന്റെ ജനങ്ങളേ, നിങ്ങള്‍ അല്ലാ ഹുവിനെ ആരാധിക്കുക. നിങ്ങള്‍ക്ക് അവനല്ലാതെ യാതൊരു ദൈവവുമില്ല. നിങ്ങള്‍ കെട്ടിച്ചമച്ച് പറയുന്നവര്‍ മാത്രമാകുന്നു.'' (11:50) ഇതേ വാക്കുകളില്‍ തന്നെയാണ് സ്വാലിഹ് നബി ഥമൂദ് ഗോത്രത്തോട് നടത്തിയ സംഭാഷണവും ക്വൂര്‍ആന്‍ ഉദ്ധരിക്കുന്നത്. (11:61).

പ്രകൃതിപരവും പ്രദേശികവുമായ സാഹോദര്യങ്ങളെല്ലാം അംഗീകരിക്കുകയും അവയെ ആദരിക്കേണ്ട തലത്തില്‍ ആദരിക്കാന്‍ അനുവ ദിക്കുകയും ചെയ്തുകൊണ്ടുതന്നെ, അവയ്‌ക്കെല്ലാം മുകളിലുള്ളതാണ് ആദര്‍ശസാഹോദര്യമെന്ന വസ്തുതയാണ് ഇസ്‌ലാമിക പ്രമാണ ങ്ങള്‍ നമ്മെ പഠിപ്പിക്കുന്നത്. ആദിമനുഷ്യരുടെ മക്കളെന്ന നിലയ്ക്കുള്ള മനുഷ്യസാഹോദാര്യത്തെ പ്രതിപാദിക്കുന്ന സൂറത്തുല്‍ ഹുജുറാ ത്തിലെ പതിമൂന്നാം വചനത്തിലെ അല്ലാഹുവിങ്കല്‍ ആദരിക്കപ്പെടുന്നത് ധര്‍മനിഷ്ഠ പാലിക്കുന്നവരാണെന്ന പരാമര്‍ശത്തിന്റെ വെളിച്ച ത്തിലുള്ളതാണ് ഇസ്‌ലാമിക സാഹോദര്യമെന്നതാണ് വാസ്തവം. സ്രഷ്ടാവിങ്കല്‍ ആദരിക്കപ്പെടുന്നതിനുവേണ്ടി പരമാവധി പരിശ്രമിക്കു കയും മത്സരിക്കുകയും ചെയ്യുന്നവരുടെ കൂട്ടായ്മയാണല്ലോ സത്യവിശ്വാസികളുടെ സമൂഹം. അതിലെ അംഗങ്ങള്‍ക്ക് എല്ലാത്തരം ഭൗതി ക വിഭവങ്ങളെക്കാളും ബന്ധങ്ങളെക്കാളും വലുത് അല്ലാഹുവിന്റെ തൃപ്തിയും അതിനായി അവന്റെ ദൂതരിലൂടെ പഠിപ്പിക്കപ്പെട്ട മാര്‍ ഗദര്‍ശനവുമാണ്. അങ്ങനെ അല്ലാതിരിക്കുന്നത് നാശത്തിനു നിമിത്തമാകുമെന്ന് പഠിപ്പിക്കപ്പെട്ടവരാണവര്‍. ''(നബിയേ,) പറയുക: നിങ്ങളു ടെ പിതാക്കളും, നിങ്ങളുടെ പുത്രന്‍മാരും, നിങ്ങളുടെ സഹോദരങ്ങളും, നിങ്ങളുടെ ഇണകളും, നിങ്ങളുടെ ബന്ധുക്കളും, നിങ്ങള്‍ സമ്പാദി ച്ചുണ്ടാക്കിയ സ്വത്തുക്കളും, മാന്ദ്യം നേരിടുമെന്ന് നിങ്ങള്‍ ഭയപ്പെടുന്ന കച്ചവടവും, നിങ്ങള്‍ തൃപ്തിപ്പെടുന്ന പാര്‍പ്പിടങ്ങളും നിങ്ങള്‍ക്ക് അല്ലാഹുവെക്കാളും അവന്റെ ദൂതനെക്കാളും അവന്റെ മാര്‍ഗത്തിലുള്ള സമരത്തെക്കാളും പ്രിയപ്പെട്ടതായിരുന്നാല്‍ അല്ലാഹു അവ ന്റെ കല്‍പന കൊണ്ടുവരുന്നത് വരെ നിങ്ങള്‍ കാത്തിരിക്കുക. അല്ലാഹു ധിക്കാരികളായ ജനങ്ങളെ നേര്‍വഴിയിലാക്കുന്നതല്ല.'' (9:24)

മാതാപിതാക്കളെക്കാളും സന്താനങ്ങളെക്കാളും ഇണകളെക്കാളും സഹോദരീസഹോദരന്‍മാരെക്കാളും മറ്റ് ഭൗതികവിഭവങ്ങളെക്കാളു മെല്ലാം ഉപരിയായി അല്ലാഹുവിനെയും അവന്റെ ദൂതനെയും സ്‌നേഹിക്കുകയും അവന്റെ മാര്‍ഗത്തിലുള്ള ധര്‍മസമരത്തിലേര്‍പ്പെ ടുകയും ചെയ്യുന്നവര്‍ തമ്മിലുള്ള ആദര്‍ശ സാഹോദര്യം മറ്റെല്ലാവിധ സാഹോദര്യങ്ങളുടെയും മുകളിലായിരിക്കുമെന്നുറപ്പാണ്. പ്രസ്തു ത സാഹോദര്യത്തിന്റെ മഹത്വവും കെട്ടുറപ്പും വിളംബരം ചെയ്യുന്ന നിരവധി വചനങ്ങള്‍ ക്വുര്‍ആനിലും ഹദീഥുകളിലുമുണ്ട്. ''സത്യവി ശ്വാസികള്‍ (പരസ്പരം) സഹോദരങ്ങള്‍ തന്നെയാകുന്നു. അതിനാല്‍ നിങ്ങളുടെ രണ്ടു സഹോദരങ്ങള്‍ക്കിടയില്‍ നിങ്ങള്‍ രഞ്ജിപ്പുണ്ടാ ക്കുക. നിങ്ങള്‍ അല്ലാഹുവെ സൂക്ഷിക്കുകയും ചെയ്യുക. നിങ്ങള്‍ക്ക് കാരുണ്യം ലഭിച്ചേക്കാം.'' (49:10). ''നിങ്ങളൊന്നിച്ച് അല്ലാഹുവിന്റെ കയറില്‍ മുറുകെപിടിക്കുക. നിങ്ങള്‍ ഭിന്നിച്ച് പോകരുത്. നിങ്ങള്‍ അന്യോന്യം ശത്രുക്കളായിരുന്നപ്പോള്‍ നിങ്ങള്‍ക്ക് അല്ലാഹു ചെയ്ത അനുഗ്രഹം ഓര്‍ക്കുകയും ചെയ്യുക. അവന്‍ നിങ്ങളുടെ മനസ്സുകള്‍ തമ്മില്‍ കൂട്ടിയിണക്കി. അങ്ങനെ അവന്റെ അനുഗ്രഹത്താല്‍ നിങ്ങള്‍ സഹോദരങ്ങളായിത്തീര്‍ന്നു.'' (3:103) ''സത്യവിശ്വാസികളും സത്യവിശ്വാസിനികളും അന്യോന്യം മിത്രങ്ങളാകുന്നു. അവര്‍ സദാചാരം കല്‍പിക്കുകയും, ദുരാചാരത്തില്‍ നിന്ന് വിലക്കുകയും, നമസ്‌കാരം മുറപോലെ നിര്‍വഹിക്കുകയും, സകാത്ത് നല്‍കുകയും, അല്ലാഹു വെയും അവന്റെ ദൂതനെയും അനുസരിക്കുകയും ചെയ്യുന്നു. അത്തരക്കാരോട് അല്ലാഹു കരുണ കാണിക്കുന്നതാണ്. തീര്‍ച്ചയായും അല്ലാ ഹു പ്രതാപിയും യുക്തിമാനുമാണ്.'' (9:71). ''കാരുണ്യത്തിലും സ്‌നേഹത്തിലും അനുകമ്പയിലും വിശ്വാസികളെ ഉപമിക്കാവുന്നത് ഒരു ശരീരത്തോടാണ്. ഏതെങ്കിലും ഒരു അവയവത്തിന് വേദനിച്ചാല്‍ മുഴുവന്‍ ശരീരവും പനിച്ചുകൊണ്ടും ഉറക്കമിളച്ചുകൊണ്ടും അതിനോട് പ്രതികരിക്കുന്നു.'' (ബുഖാരി, മുസ്‌ലിം) ''മുസ്‌ലിംകളെല്ലാം ഒരു മനുഷ്യനെപ്പോലെയാണ്. കണ്ണിനുകേടുപറ്റിയാല്‍ ശരീരം മുഴുവന്‍ പ്രയാ സപ്പെടുന്നു. തലയ്ക്ക് കേടുണ്ടായാല്‍ ശരീരത്തിനു മുഴുവന്‍ ബുദ്ധിമുട്ട് അനുഭവപ്പെടുന്നു.'' (മുസ്‌ലിം). ''ഒരു വിശ്വാസി മറ്റൊരു വിശ്വാസി യുമായി ഒരു മതിലിലെ കല്ലുകളെപ്പോലെയാണ്. ഒരാള്‍ മറ്റൊരാളെ ശക്തിപ്പെടുത്തുന്നു.'' (മുസ്‌ലിം). ''മുസ്‌ലിമിനോട് വിദ്വേഷമുണ്ടാക രുത്; അസൂയയുണ്ടാകരുത്; പുറത്താക്കുകയോ എതിരെ പ്രവര്‍ത്തിക്കുകയോ ചെയ്യരുത്. അല്ലാഹുവിന്റെ ദാസാ, നിങ്ങള്‍ സഹോദരന്‍ മാരെപ്പോലെയാവുക. മൂന്നിലധികം ദിവസങ്ങള്‍ തന്റെ സഹോദരനുമായി പിണങ്ങി നില്‍ക്കുവാന്‍ മുസ്‌ലിമിന് അനുവാദ മില്ല.'' (മുസ്‌ ലിം). ''വിശ്വസിക്കാതെ നിങ്ങളാരും സ്വര്‍ഗത്തില്‍ പ്രവേശിക്കുകയില്ല; പരസ്പരം സ്‌നേഹിക്കാതെ നിങ്ങളാരും വിശ്വാസികളാവുകയില്ല. പരസ്പരമുള്ള സ്‌നേഹവര്‍ദ്ധനവിനാവശ്യമായ ഒരു കാര്യം നിങ്ങള്‍ക്ക് പറഞ്ഞുതരട്ടെയോ? നിങ്ങള്‍ക്കിടയില്‍ സലാം പ്രചരിപ്പിക്കുക. ''(മുസ്‌ലിം) ''സ്വന്തത്തിനായി നിങ്ങള്‍ ഇഷ്ടപ്പെടുന്നത് സ്വന്തം സഹോദരനുവേണ്ടിയും നിങ്ങള്‍ ഇഷ്ടപ്പെടാത്തിടത്തോളം നിങ്ങളാരും വിശ്വാ സികളാവുകയില്ല'' (ബുഖാരി, മുസ്‌ലിം)

ആദര്‍ശസാഹോദര്യത്തില്‍ നിന്ന് ഉയിര്‍കൊള്ളുന്ന, വിശ്വാസികള്‍ തമ്മിലുള്ള ഈ ആത്മബന്ധത്തിനാണ് സാങ്കേതികമായി 'വലാഅ്' എന്നു പറയുന്നത്. സാധാരണയുള്ള സ്‌നേഹബന്ധത്തിനും സൗഹൃദത്തിനും ഉപരിയായുള്ള ആത്മബന്ധമാണിത്; അല്ലാഹുവിന്റെ പ്രീതി മാത്രം കാംക്ഷിച്ചുകൊണ്ടുള്ള ആത്മബന്ധം. മക്കളോടും മാതാപിതാക്കളോടും ഇണകളോടും സഹോദരങ്ങളോടുമെല്ലാം ഉള്ള സ്‌നേഹബന്ധത്തി നുമുകളിലുള്ള ഹൃദയബന്ധം. സത്യവിശ്വാസത്തിന്റെ അടിത്തറയിലുള്ള ഈ ആദര്‍ശബന്ധമുണ്ടാകേണ്ടത് അല്ലാഹുവിനോടും ദൈവദൂ തനോടും സത്യവിശ്വാസികളോടുമാകണമെന്ന് ക്വുര്‍ആന്‍ പഠിപ്പിക്കുന്നുണ്ട്. ''അല്ലാഹുവും അവന്റെ ദൂതനും, താഴ്മയുള്ളവരായിക്കൊ ണ്ട് നമസ്‌കാരം മുറപോലെ നിര്‍വഹിക്കുകയും സകാത്ത് നല്‍കുകയും ചെയ്യുന്ന സത്യവിശ്വാസികളും മാത്രമാകുന്നു നിങ്ങളുടെ ഉറ്റമിത്ര ങ്ങള്‍. വല്ലവനും അല്ലാഹുവെയും അവന്റെ ദൂതനെയും, സത്യവിശ്വാസികളെയും മിത്രങ്ങളായി സ്വീകരിക്കുന്നുവെങ്കില്‍ തീര്‍ച്ചയായും അല്ലാഹുവിന്റെ കക്ഷി തന്നെയാണ് വിജയം നേടുന്നവര്‍.'' (5:55,56)

സത്യവിശ്വാസികള്‍ തമ്മിലുള്ള ഈ ആത്മബന്ധത്തിന്റെ അടിത്തറ തന്നെ അവര്‍ തമ്മില്‍ നടക്കുന്ന സംസ്‌കരണ പ്രക്രിയയാണെന്നാണ് ക്വു ര്‍ആനിക അധ്യാപനം. ക്വുര്‍ആനിന്റെയും സുന്നത്തിന്റെയും വെളിച്ചത്തില്‍ പരസ്പരം നന്മ കല്‍പിക്കുകയും തിന്മ വിരോധിക്കുകയും ചെയ്യുന്നതുകൊണ്ടുണ്ടാകുന്ന ഈ ആത്മബന്ധമുണ്ടാവുക വിശ്വാസികള്‍ തമ്മില്‍ മാത്രമായിരിക്കുമെന്ന് പ്രത്യേകം പറയേണ്ടതില്ല. ''സത്യ വിശ്വാസികളും സത്യവിശ്വാസിനികളും അന്യോന്യം മിത്രങ്ങളാകുന്നു. അവര്‍ സദാചാരം കല്‍പിക്കുകയും, ദുരാചാരത്തില്‍ നിന്ന് വില ക്കുകയും, നമസ്‌കാരം മുറപോലെ നിര്‍വഹിക്കുകയും, സകാത്ത് നല്‍കുകയും, അല്ലാഹുവെയും അവന്റെ ദൂതനെയും അനുസരിക്കു കയും ചെയ്യുന്നു. അത്തരക്കാരോട് അല്ലാഹു കരുണ കാണിക്കുന്നതാണ്. തീര്‍ച്ചയായും അല്ലാഹു പ്രതാപിയും യുക്തിമാനുമാണ്.''(9:71)

ആദര്‍ശത്തിന്റെ അടിസ്ഥാനത്തില്‍ മാത്രമുണ്ടാവേണ്ട 'വലാഅ്' എന്ന ആത്മബന്ധം വിശ്വാസികള്‍ തമ്മിലാണുണ്ടാകേണ്ടതെന്നും അവി ശ്വാസികളോട് അത്തരം ആത്മബന്ധം പുലര്‍ത്താന്‍ പാടില്ലെന്നും പഠിപ്പിക്കുന്ന നിരവധി വചനങ്ങള്‍ ക്വുര്‍ആനിലുണ്ട്. ''സത്യവിശ്വാ സികള്‍ സത്യവിശ്വാസികളെയല്ലാതെ സത്യനിഷേധികളെ മിത്രങ്ങളാക്കിവെക്കരുത്. -അങ്ങനെ വല്ലവനും ചെയ്യുന്ന പക്ഷം അല്ലാഹുവു മായി അവന് യാതൊരു ബന്ധവുമില്ല- നിങ്ങള്‍ അവരോട് കരുതലോടെ വര്‍ത്തിക്കുകയാണെങ്കിലല്ലാതെ. അല്ലാഹു അവനെപ്പറ്റി നിങ്ങള്‍ ക്ക് താക്കീത് നല്‍കുന്നു. അല്ലാഹുവിങ്കലേക്കത്രെ (നിങ്ങള്‍) തിരിച്ചുചെല്ലേണ്ടത്.'' (3:28). ''അവര്‍ അല്ലാഹുവിലും പ്രവാചകനിലും, അദ്ദേഹ ത്തിന് അവതരിപ്പിക്കപ്പെട്ടതിലും വിശ്വസിച്ചിരുന്നുവെങ്കില്‍ അവരെ (അവിശ്വാസികളെ) ഉറ്റമിത്രങ്ങളായി സ്വീകരിക്കുമായിരുന്നില്ല. പക്ഷെ, അവരില്‍ അധികപേരും ധിക്കാരികളാകുന്നു.'' (5:81). ''സത്യവിശ്വാസികളേ, യഹൂദരെയും ക്രൈസ്തവരേയും നിങ്ങള്‍ ഉറ്റമിത്രങ്ങ ളായി സ്വീകരിക്കരുത്. അവരാകട്ടെ, അന്യോന്യം ഉറ്റമിത്രങ്ങളാണ് താനും. നിങ്ങളില്‍ നിന്നാരെങ്കിലും അവരെ ഉറ്റമിത്രങ്ങളായി സ്വീകരി ക്കുന്ന പക്ഷം അവനും അവരില്‍ പെട്ടവന്‍ തന്നെയാണ്. അക്രമികളായ ആളുകളെ അല്ലാഹു നേര്‍വഴിയിലാക്കുകയില്ല; തീര്‍ച്ച.'' (5:51)

സ്വന്തത്തേക്കാളധികം മുഹമ്മദ് നബി(സ)യെ സ്‌നേഹിക്കുന്ന മുസ്‌ലിംകള്‍ക്ക് നബി(സ)യെന്ന പേരിട്ടുകൊണ്ട് വൃത്തികെട്ട ചിത്രങ്ങള്‍ വര ക്കപ്പെടുകയും ലോകത്തിന് പ്രകാശം നല്‍കുകയും നന്മ മാത്രം വിതറുകയും ചെയ്ത പ്രവാചകന്‍ ക്രൂരമായി അഭിശംസിക്കപ്പെടുകയും ചെയ്യുമ്പോള്‍ വേദനയുണ്ടാവുക സ്വാഭാവികമാണ്. സ്വന്തത്തെയല്ലാതെ മറ്റൊന്നിനെയും ആത്മാര്‍ത്ഥമായി സ്‌നേഹിക്കാന്‍ പഠിച്ചിട്ടില്ലാ ത്ത ആത്മരതിയുടെ സംസ്‌കാരം പേറുന്നവര്‍ക്ക് മുസ്‌ലിംകളുടെ നബിസ്‌നേഹത്തിന്റെ ആഴവും അര്‍ത്ഥവുമറിയാന്‍ കഴിയില്ല. ഒരാളു ടെ കാരിക്കേച്ചറുകളും കോമാളി വരകളും എന്തിനാണ് ഒരു ജനസമൂഹത്തെ വൈകാരികമായി സ്വാധീനിക്കുന്നതെന്ന് അവര്‍ക്ക് മനസ്സിലാ കാത്തത് ദൈവസ്‌നേഹത്തിന്റെയും അതില്‍ നിന്ന് ഉയിരെടുക്കുന്ന നബിസ്‌നേഹത്തിന്റെയും അവാച്യമായ അനുഭൂതി അവര്‍ക്ക് ആസ്വ ദിക്കാന്‍ കഴിയാത്തതുകൊണ്ടാണ്. വിശ്വാസം രൂഢമൂലമായ മനസ്സിനേ പ്രസ്തുത അനുഭൂതിയുടെ ആസ്വാദ്യത അനുഭവിക്കുവാന്‍ ഭാഗ്യം ലഭിക്കൂ. അവിശ്വാസത്തിന്റെ ആന്ധ്യവും ആസ്വാദനമെന്നാല്‍ ഇന്ദിയാസ്വാദനം മാത്രമാണെന്ന അധകൃത ബോധവും ബാധിച്ചവര്‍ പ്രസ്തു ത ഭാഗ്യമില്ലാതെ പോയവരാണ് എന്നതിനാല്‍ അവരോട് സഹതപിക്കാന്‍ മാത്രമെ വിശ്വാസികള്‍ക്കാവൂ. ലോകനാഥനോടുള്ള സ്‌നേഹ വും പ്രസ്തുത സ്‌നേഹം എങ്ങനെയെന്ന് പഠിപ്പിച്ച ദൂതനോടുള്ള പ്രേമവുമാണ് മുസ്‌ലിമിന്റെ വ്യക്തിത്വത്തെ പ്രകാശമാനമാക്കുന്നത്. മറ്റൊരാളെ സ്വന്തത്തെക്കാളേറെ സ്‌നേഹിക്കുന്നതെങ്ങനെയെന്ന് മനസ്സിലാക്കുവാന്‍ പോലും അവിശ്വാസവും ആത്മാരാധനയും മൂലം കടു ത്തുപോയ ഹൃദയത്തിന്റെ ഉടമകള്‍ക്ക് കഴിയില്ല. അതിന് ദൈവസമര്‍പ്പണത്തിന്റെ വെളിച്ചം ലഭിച്ച മനസ്സുണ്ടാകണം. അതില്ലാത്തവര്‍ ക്ക് നബിനിന്ദ മുസ്‌ലിം മനസ്സില്‍ സൃഷ്ടിക്കുന്ന വേദനയുടെ ആഴമളക്കാനാവില്ല. സ്വന്തം ഹൃദയത്തിനകത്തേക്ക് അമ്പ് തുളച്ചുകയറുമ്പോഴു ണ്ടാകുന്ന വേദനയേക്കാള്‍ അസഹ്യമാണ് മുസ്‌ലിമിന് നബിനിന്ദ കേള്‍ക്കുമ്പോഴുള്ള വേദന. അളന്നു തിട്ടപ്പെടുത്തുവാനോ അഭിനയിച്ചു കാണിച്ചുകൊടുക്കുവാനോ കഴിയുന്നതല്ല അത്. നബിസ്‌നേഹത്തിന്റെ ആസ്വാദ്യത അനുഭവിക്കുവാന്‍ ഭാഗ്യം ലഭിച്ചവര്‍ക്കേ പ്രസ്തുത വേദനയുടെ ആഴം മനസ്സിലാകൂ; ആത്മാരാധകരായ നബിനിന്ദകര്‍ക്ക് അത് മനസ്സിലാവില്ല.

നബിസ്‌നേഹത്തിന്റെ പ്രകടനത്തിന് ഹാവഭാവങ്ങളുടെ അകമ്പടിയുണ്ടാവണമെന്നില്ല. ചരിത്രത്തില്‍ ജീവിച്ചുകഴിഞ്ഞവരില്‍ നിന്നെല്ലാം നബി(സ) വ്യത്യസ്തമായതുപോലെ അദ്ദേഹത്തോടുള്ള സ്‌നേഹപ്രകടനവും വ്യത്യസ്തമാണ്. നബിസ്‌നേഹത്തിന്റെ പ്രയോഗവല്‍ക്കര ണമെന്നാല്‍ നബിജീവിതത്തിന്റെ അനുധാവനമാണെന്നാണ് അന്തിമ പ്രവാചകന്‍(സ) വിശ്വാസികളെ പഠിപ്പിച്ചത്. ജീവിതത്തിന്റെ ഏതേ ത് തുറകളിലുള്ളവര്‍ക്കാണെങ്കിലും മാതൃകയാക്കാന്‍ പറ്റിയതായി അവതരിപ്പിക്കാനാവുന്ന മറ്റൊരു വ്യക്തിത്വമില്ല; ഞങ്ങളുടെ പ്രവാച കനല്ലാതെയെന്ന വസ്തുത ആരുടെ മുന്നിലും ഉറക്കെപ്പറയാന്‍ കഴിയുന്ന ഒരേയൊരു ജനസമൂഹമാണ് മുസ്‌ലിംകള്‍. നബിജീവിതത്തിന്റെ മാതൃക വെളിച്ചം വീശാത്ത മേഖലകളൊന്നുമില്ല. സ്‌നേഹിക്കുന്നവരോടും വെറുക്കുന്നവരോടുമെല്ലാം എങ്ങനെ പെരുമാറണമെന്ന് പ്രവാ ചകന്‍ (സ) പഠിപ്പിച്ചിട്ടുണ്ട്. അനുയായികളെയും എതിരാളികളെയും എങ്ങനെ കൈകാര്യം ചെയ്യണമെന്നും സ്തുതിപാഠകന്‍മാരോടും നിന്ദകരോടുമുള്ള നിലപാടുകള്‍ എങ്ങനെയാകണമെന്നും നബി(സ) മാതൃക കാണിക്കാതെ വിട്ടുപോയിട്ടില്ല. ഏതാനും ചില അനുയായിക ളല്ലാതെ മക്കയിലുള്ള മറ്റുള്ളവരെല്ലാം പ്രവാചകന്റെ പ്രബോധനകാലത്തിന്റെ തുടക്കത്തില്‍ നബിനന്ദകരായിരുന്നു. പ്രസ്തുത നിന്ദക രില്‍ നിന്നാണ് പ്രവാചകാനുചരന്‍മാരെല്ലാം പരിണമിച്ചുണ്ടായത് എന്നുള്ള വസ്തുത നബിചരിത്രം നമ്മെ പഠിപ്പിക്കുന്നു. ഉമറും അബൂ സുഫ്‌യാനും ഖാലിദുബ്‌നു വലീദും സുഹൈലുബ്‌നു അംറും ഹിന്ദും തുടങ്ങി അറിയപ്പെട്ട നബിനിന്ദകരെയെല്ലാം ഇസ്‌ലാമിന്റെ കൊടി ക്കീഴില്‍ അണിനിരത്തി നബിസ്‌നേഹത്തിന്റെ നിതാന്ത മാതൃകകളാക്കിത്തീര്‍ത്ത ചരിത്രമാണ് പ്രവാചകന്റേത്. നിന്ദകരെ കൊന്നൊടു ക്കുകയല്ല, സ്വന്തം ജീവിതവിശുദ്ധിയിലൂടെ അനുയായികളാക്കിത്തീര്‍ക്കുകയാണ് വേണ്ടതെന്ന പാഠമാണ് നബി (സ) സ്വന്തം കര്‍മങ്ങളി ലൂടെ പഠിപ്പിച്ചത്. നബിനിന്ദകര്‍ക്കുള്ള പ്രവാചകകാലത്തെ മറുപടി നബിജീവിതത്തിന്റെ വെളിച്ചമായിരുന്നുവെങ്കില്‍ ഇന്നത്തെയും മറുപടി പ്രസ്തുത വെളിച്ചം തന്നെയാണ്. പതിനാല് നൂറ്റാണ്ടുകള്‍ക്ക് മുമ്പ് നബി(സ) പ്രസരിപ്പിച്ച വെളിച്ചം അതിന്റെ ഒളി ഒരല്‍പ്പംപോ ലും മങ്ങാതെ ഇന്നും ഇവിടെയുണ്ട്. പ്രസ്തുത വെളിച്ചത്തെ നബിനിന്ദകര്‍ക്കു നേരെ തിരിച്ചുവെക്കുകയെന്നതാണ് മുസ്‌ലിംകളുടെ ദൗത്യം. ലോകത്തെങ്ങുമുള്ള നബിനിന്ദകരെ കൊന്നൊടുക്കുവാന്‍ നബി (സ) പഠിപ്പിച്ചിട്ടില്ലാത്തതിനാല്‍ അങ്ങനെ ചെയ്യാനൊരുമ്പെടുന്നവര്‍ ഇക്കാ ര്യത്തില്‍ നബി(സ)യില്‍ നിന്ന് വെളിച്ചം സ്വീകരിക്കാത്തവരാണ്. ഇക്കാര്യത്തില്‍ അവരും നബിനിന്ദകരും ഒരേ പോലെയാണെന്ന് പറയു ന്നത് അതുകൊണ്ടാണ്.

വിശുദ്ധ ക്വുര്‍ആനിന്റെയും ഹദീഥുകളുടെയും വെളിച്ചത്തില്‍, ജിഹാദിനെക്കുറിച്ച് വര്‍ഗീകരിച്ചു വിശദീകരിച്ച പണ്ഡിതന്മാരില്‍ പ്രമുഖ നാണ് ഹിജ്‌റ എട്ടാം നൂറ്റാണ്ടില്‍ മരണപ്പെട്ട ഇമാം ഇബ്‌നുല്‍ ഖയ്യിം അല്‍ ജൗസിയ്യഃ. തന്റെ സാദുല്‍ മആദ് എന്ന ഗ്രന്ഥാത്തിന്റെ മൂന്നാം വാല്യത്തില്‍ ജിഹാദിനെക്കുറിച്ച് അദ്ദേഹം വിശദമായി ഉപന്യസിക്കുന്നുണ്ട്. സ്വന്തത്തോടുള്ളത് (ജിഹാദുന്നഫ്‌സ്), ചെകുത്താനോടുള്ളത് (ജിഹാദുശ്ശൈത്വാന്‍), സത്യനിഷേധികളോടും കപടവിശ്വാസികളോടുമുള്ളത് (ജിഹാദുല്‍ കുഫ്ഫാറു വല്‍ മുനാഫിഖീന്‍), അനീതിയുടെയും തിന്മകളുടെയും അനാചാരങ്ങളുടെയും ആളുകള്‍ക്കെതിരെയുള്ളത് (ജിഹാദു അര്‍ബാബുദ്ദ്വുല്‍മി വല്‍ ബിദ്ഇ വല്‍ മുന്‍കറാത്ത്) എന്നി ങ്ങനെ നാലായി ജിഹാദിനെ വര്‍ഗീകരിക്കാമെന്നാണ് അദ്ദേഹത്തിന്റെ പക്ഷം. സ്വന്തത്തോടുള്ള ജിഹാദില്‍ നിന്നാണ്  ഒരാള്‍ തന്റെ ഇസ്‌ ലാമിക ജീവിതം ആരംഭിക്കുന്നത്. സ്വന്തം താല്‍പര്യങ്ങളുടെയും ആഗ്രഹങ്ങളുടെയും തൃഷ്ണയുടെയും തടവറയില്‍ നിന്ന് സ്വയം മോചി തനായി ദൈവിക നിയമങ്ങള്‍ക്ക് വിധേയനാവലാണ് സ്വന്തത്തോടുള്ള ജിഹാദ്. ശരിയായ വിശ്വാസത്തെ നശിപ്പിക്കാനായി പിശാച് മനസ്സി ല്‍ ചെലുത്തുന്ന സംശയങ്ങളെ ദൂരീകരിക്കുകയും അതിനെതിരെ സജ്ജമാവുകയും ചെയ്യുന്നതും തെറ്റായ താല്‍പര്യങ്ങളും അനനുവദനീയ മായ തൃഷ്ണകളും വളര്‍ത്തി പ്രലോഭിപ്പിക്കുവാന്‍ വേണ്ടിയുള്ള പിശാചിന്റെ പരിശ്രമങ്ങളെ ചെറുക്കുകയും അവഗണിക്കുകയും ചെ യ്യുന്നതുമാണ് ജിഹാദുശ്ശൈത്വാനിന്റെ രൂപങ്ങള്‍. ഹൃദയംകൊണ്ടും നാവുകൊണ്ടും സമ്പത്തുകൊണ്ടും ശരീരംകൊണ്ടും നാലു രൂപത്തി ലാണ് ഒരാള്‍ ജിഹാദുല്‍ കുഫ്ഫാര്‍ വല്‍ മുനാഫിഖീന്‍ നിര്‍വഹിക്കുന്നത്. അനീതിയുടെയും അനാചാരങ്ങളുടെയും തിന്മകളുടെയും വക്താക്കള്‍ക്കെതിരെ നടത്തുന്ന ജിഹാദിന് മൂന്ന് രൂപങ്ങളാണുള്ളത്. കൈകൊണ്ടുള്ളത്, നാവുകൊണ്ടുള്ളത്, മനസ്സുകൊണ്ടുള്ളത് എന്നിവ യാണവ.

മുഹമ്മദ് നബി(സ) ആയുധമെടുക്കാനാരംഭിച്ചത് മദീനയിലെത്തി ഇസ്‌ലാമിക രാഷ്ട്രത്തിന്റെ നായകത്വമേറ്റെടുത്തതിന് ശേഷമായിരുന്നു വെന്ന വസ്തുത ഇസ്‌ലാമിന്റെ ശത്രുക്കളും മിത്രങ്ങളുമെല്ലാം സമ്മതിക്കുന്നതാണ്. എന്നാല്‍ ജിഹാദാരംഭിച്ചത് മദീനയില്‍ വെച്ചല്ല; പീഡ നങ്ങളുടെ മക്കാകാലത്ത് തന്നെയാണ്. ജിഹാദിനെക്കുറിച്ച ക്വുര്‍ആന്‍ നിര്‍ദേശങ്ങള്‍ അവതരിക്കുവാനാരംഭിച്ചത് മക്കയില്‍ വെച്ചാണ്. പീഡനങ്ങളും പ്രയാസങ്ങളും സഹിച്ച്  ഇസ്‌ലാമിക പ്രബോധനം നിര്‍വഹിച്ചുകൊണ്ടിരിക്കുന്നതിനിടയില്‍ അവതരിപ്പിക്കപ്പെട്ട ക്വുര്‍ആ ന്‍ സൂക്തങ്ങളില്‍ ജിഹാദിന് പ്രേരിപ്പിക്കുന്ന വചനങ്ങള്‍ കാണാം. മക്കയില്‍വെച്ച് അവതരിപ്പിക്കപ്പെട്ട സൂറത്തുല്‍ അന്‍കബൂത്തിലെ അവസാനത്തെ വചനത്തില്‍, 'നമ്മുടെ മാര്‍ഗത്തില്‍ ജിഹാദില്‍ ഏര്‍പെട്ടവരാരോ, അവരെ നമ്മുടെ വഴികളിലേക്ക് നാം നയിക്കുകതന്നെ ചെയ്യുന്നതാണ്.' എന്നാണു അല്ലാഹു പറയുന്നത്. മക്കയില്‍വെച്ച് ചെയ്യേണ്ട ജിഹാദ് എന്താണ്? മക്കയില്‍ വെച്ച് അവതരിപ്പിക്കപ്പെട്ട സൂറത്തുല്‍ ഫുര്‍ക്വാനിലെ അന്‍പത്തി രണ്ടാമത്തെ സൂക്തം ഇതിനു മറുപടി പറയുന്നുണ്ട്. സത്യനിഷേധികളോട് ക്വുര്‍ആന്‍ ഉപയോഗിച്ച് ജിഹാദ് ചെയ്യാനാണ് പ്രസ്തുത സൂക്തത്തില്‍ നബിയോട് ആവശ്യപ്പെട്ടിരിക്കുന്നത്. ആശയസമരമാകുന്ന പ്രസ്തുത ജിഹാദാണ് ഏറ്റവും വലിയ ജിഹാദെന്നും ഈ ക്വുര്‍ആന്‍ സൂക്തം വ്യക്തമാക്കുന്നു. ഈ ജിഹാദാണ് മുസ്‌ലിംകള്‍ എപ്പോഴും നടത്തേണ്ട പോരാട്ടം. ക്വുര്‍ആനി ന്റെയും നബിചര്യയുടെയും ആശയങ്ങളുപയോഗിച്ച് തിന്മകള്‍ക്കെതിരെ നടത്തുന്ന ആദര്‍ശപോരാട്ടമാണിത്; ആയുധങ്ങളുപയോഗി ച്ചുള്ള സമരമല്ല.

ഒരു ജീവിതദര്‍ശനമെന്ന നിലക്ക് ഇസ്‌ലാം അതിന്റെ അനുയായികളെ ആയുധമണിയാനനുവദിച്ചിട്ടില്ലെന്നല്ല ഇപ്പറഞ്ഞതിന്നര്‍ത്ഥം. ആയുധ മെടുക്കേണ്ട സന്ദര്‍ഭത്തില്‍ അതിനു മടിച്ച് നില്‍ക്കരുത് എന്നു തന്നെയാണ് ഇസ്‌ലാം മുസ്‌ലിംകളോട് പറഞ്ഞിരിക്കുന്നത്. സായുധസമരം വലിയ പുണ്യകര്‍മമായി പഠിപ്പിച്ച ആദര്‍ശം തന്നെയാണ് ഇസ്‌ലാം. പക്ഷെ അത് ചെയ്യേണ്ടത് ചെയ്യേണ്ട രൂപത്തില്‍ ചെയ്യേണ്ടവരായി രിക്കണമെന്നു കൂടി ഇസ്‌ലാം നിഷ്‌കര്‍ഷിച്ചിട്ടുണ്ട്. ആദര്‍ശത്തിന് വേണ്ടിയുള്ളതാണ് ഇസ്‌ലമിലെ യുദ്ധം. എതിരാളികളെയെല്ലാം ഉന്മൂല നം ചെയ്യുകയോ ലോകത്തിന്റെ മേല്‍ മുസ്‌ലിംകളുടെ രാഷ്ട്രീയാധിനിവേശം സ്ഥാപിക്കുകയോ ചെയ്യുന്നതിന് വേണ്ടിയുള്ളതല്ല അത്. ദൈവത്തമായ ജീവിതവ്യവസ്ഥയനുസരിച്ച് ജീവിക്കുവാനും അതു മറ്റുള്ളവര്‍ക്ക് പകര്‍ന്നു കൊടുക്കുവാനുമുള്ള സ്വാതന്ത്ര്യമുണ്ടാക്കു കയാണ് യുദ്ധത്തിന്റെ ലക്ഷ്യം. മുസ്‌ലിംകളായി ജീവിക്കാന്‍ അനുവദിക്കാതെ ജനിച്ചുവളര്‍ന്ന നാട്ടില്‍ നിന്ന് പലായനം ചെയ്യേണ്ടിവന്നവ ര്‍ക്ക്  തങ്ങളുടെ അവകാശങ്ങള്‍ വീണ്ടെടുക്കുന്നതിന് വേണ്ടി പേരാടുവാന്‍ അനുവദിച്ചുകൊണ്ടുള്ളതാണ് യുദ്ധത്തെക്കുറിച്ച് പരാമര്‍ശി ച്ചുകൊണ്ട് ആദ്യമായി അവതരിപ്പിക്കപ്പെട്ട ക്വുര്‍ആന്‍ വചനങ്ങള്‍.   വീടും സമ്പത്തുകളും ഉപേക്ഷിച്ചുകൊണ്ട് ആദര്‍ശമനുസരിച്ച് ജീവി ക്കുവാന്‍ വേണ്ടി മാത്രമായി പലായനം ചെയ്തവര്‍ക്ക് അവര്‍ പലായനം ചെയ്‌തെത്തി ജീവിക്കുന്ന നാട്ടില്‍ സൈ്വര്യമായിരിക്കുവാന്‍ അവസരം നല്‍കാതിരിക്കുന്നവര്‍ക്കെതിരെ യുദ്ധം ചെയ്യാന്‍ അനുവാദമുണ്ടെന്നും അതിനായി ആയുധമെടുത്തവരെ അല്ലാഹു സഹായി ക്കുമെന്നും അറിയിച്ചുകൊണ്ടുള്ള സൂറത്തുല്‍ ഹജ്ജിലെ 39, 40 വചനങ്ങളാണവ.

ആക്രമണത്തെ പ്രതിരോധിക്കുന്നതിനു വേണ്ടിയാണ് മുസ്‌ലിംകള്‍ യുദ്ധം ചെയ്യേണ്ടതെന്നു  വ്യക്തമാക്കുന്ന ഈ സൂക്തങ്ങള്‍ അത്തരം പ്രതി രോധങ്ങളുണ്ടായിട്ടില്ലെങ്കില്‍ മുസ്‌ലിംകള്‍ക്ക് മാത്രമല്ല മറ്റു മതവിശ്വാസികള്‍ക്കും സൈ്വര്യമായി ആരാധനകള്‍ നിര്‍വഹിച്ചുകൊണ്ട് ജീവിക്കുവാന്‍ കഴിയാത്ത സാഹചര്യമാണുണ്ടാവുകയെന്ന്  വ്യക്തമാക്കുന്നു. സന്യാസിമഠങ്ങളും ചര്‍ച്ചുകളും സിനഗോഗുകളും മസ്ജി ദുകളുമെല്ലാം തകര്‍ക്കപ്പെടുകയും വിശ്വാസികള്‍ക്കൊന്നും സ്വസ്ഥമായി ആരാധനകള്‍ നിര്‍വഹിക്കാനാവാത്ത അവസ്ഥ ഉണ്ടാവുകയുമാ യിരിക്കും മതപീഡകന്‍മാരെ സ്വതന്ത്രമായി അഴിഞ്ഞാടുവാന്‍ അനുവദിച്ചാലുണ്ടാവുന്ന ഫലങ്ങള്‍. മതത്തിന്റെ പേരിലുള്ള പീഡനം ഇല്ലാതെയാക്കുന്നതിന് വേണ്ടി മുസ്‌ലിംകള്‍ നടത്തുന്ന പോരാട്ടം മറ്റു മതവിശ്വാസികള്‍ക്കും സമാധാനപൂര്‍ണമായി ആരാധനകള്‍ നിര്‍വ ഹിക്കുവാനുള്ള സാഹചര്യമൊരുക്കുകയാണ് സ്വാഭാവികമായും ചെയ്യുകയെന്നര്‍ത്ഥം.

ഇസ്‌ലാമിക പ്രബോധനരംഗത്തെ തടസ്സങ്ങള്‍ നീക്കുകയാണ് സായുധജിഹാദിന്റെ പ്രഥമലക്ഷ്യം. ഫൈസലിന്റെ ഘാതകനെ കൊന്നവര്‍ മുസ്‌ലിംപേരുള്ളവരാണെങ്കില്‍, അവര്‍ യഥാര്‍ത്ഥത്തില്‍ ചെയ്തിരിക്കുന്നത് പ്രബോധനപ്രവര്‍ത്തനങ്ങളെയും പ്രബോധകരെയും പ്രതിക്കൂട്ടിലാക്കുകയും ആ രംഗത്ത് പ്രയാസങ്ങളുണ്ടാക്കുകയുമാണ്. ഇസ്‌ലാംഭീതി വളര്‍ത്തുവാന്‍ ശത്രുക്കള്‍ക്ക് ഒരു പുതിയ മരുന്ന് കൂടി നല്‍കുകയാണ് അവര്‍ ചെയ്തിരിക്കുന്നത്. എങ്ങനെയെല്ലാമാണ് അത് കത്തിപ്പടരാന്‍ പോകുന്നത് എന്ന് കേരളീയര്‍ കണ്ടറിയാനിരി ക്കുന്നതേയുള്ളൂ. പ്രബോധനരംഗത്തെ ഭീതിയോടെ നോക്കിക്കാണുന്ന അവസ്ഥ സൃഷ്ടിക്കുക മാത്രമാണ് ഈ പകപോക്കലുകൊണ്ടുണ്ടായി രിക്കുന്നത്. പ്രബോധിത സമൂഹത്തില്‍ ഇസ്‌ലാം വെറുപ്പും ഇസ്‌ലാംഭീതിയും സൃഷ്ടിക്കുന്നതിനു മാത്രം നിമിത്തമാകുന്ന ചെയ്തികള്‍ ആരില്‍ നിന്നുണ്ടായാലും അത് ജിഹാദിന്റെ ആത്മാവിനു വിരുദ്ധമാണ് എന്ന് പറയേണ്ടി വരും. ആരുടെയെങ്കിലും ആഹ്വാനത്തില്‍ ആകൃഷ്ടരായി നടത്തുന്ന ആത്മഹത്യാപരമായ സായുധപ്രവര്‍ത്തനത്തെ ജിഹാദായല്ല, ഫിത്‌നയായാണ് പണ്ഡിതന്മാര്‍ എണ്ണിയിരിക്കു ന്നത്. പൗരന്‍മാര്‍ക്കുമേല്‍ അധികാരമുള്ള ഭരണാധികാരിക്ക് കീഴില്‍ വ്യക്തമായ ലക്ഷ്യത്തോടെ നടത്തുന്ന സായുധസമരമാണ് ജിഹാദ് എന്നോര്‍ക്കുക.

സായുധസമരങ്ങള്‍ രണ്ടുരൂപത്തിലാണുള്ളത്. ഒന്ന് പ്രതിരോധമാണ്. ശത്രുക്കളില്‍ നിന്ന് ഇസ് ലാമിക സമൂഹത്തെ രക്ഷിക്കുന്നതിന് വേ ണ്ടി നടത്തുന്ന യുദ്ധമാണത്. നാടിനെ നശിപ്പിക്കാനും ഇസ്‌ലാമികാദര്‍ശത്തെ വേരോടെ പിഴുതെറിയാനും വേണ്ടി ഇസ്‌ലാമിക രാഷ്ട്രത്തെ ആക്രമിക്കാനായി വരുന്നവരെ പ്രതിരോധിച്ച് നാടിനെ സംരക്ഷിക്കലാണിത്. പ്രവാചകജീവിതത്തിലെ ബദറും ഉഹ്ദും ഖന്ദഖുമെല്ലാം ഇത്തരം പ്രതിരോധയുദ്ധങ്ങളായിരുന്നു. കര്‍മശാസ്ത്രജ്ഞന്മാര്‍ 'ജിഹാദുദഫ്അ' എന്ന് വിളിച്ചത് ഇത്തരം യുദ്ധങ്ങളെയാണ്. സഖ്യകക്ഷി കളെയുമായിവന്ന് മദീനയെ തകര്‍ക്കുകയെന്ന ലക്ഷ്യത്തോടെയുള്ള അഹ്‌സാബ് യുദ്ധം പരാജയപ്പെട്ട് ശത്രുക്കള്‍ പിന്തിരിഞ്ഞു പോയ പ്പോള്‍ പുതിയ യുദ്ധതന്ത്രം പ്രവാചകന്‍ പ്രഖ്യാപിച്ചു: 'ഇനി മുതല്‍ നാം അവരോട് യുദ്ധം ചെയ്യും; അവര്‍ നമ്മോട് യുദ്ധം ചെയ്യുകയില്ല' (ബുഖാരി). രാഷ്ട്രതന്ത്രജ്ഞനായ നേതാവിന്റെ മഹാപ്രഖ്യാപനം! ശത്രുക്കള്‍ മദീനാ രാജ്യത്തേക്ക് കടന്നുവന്ന് അവിടെയുള്ളവരെ കൊന്നൊടുക്കുകയും രാജ്യസമ്പത്ത് കൊള്ളയടിക്കുകയും നാടിനെ തകര്‍ക്കുകയും ചെയ്യുന്നതിന് സമ്മതിക്കാതെ അവരുടെ കേന്ദ്രങ്ങളില്‍ പോയി ആക്രമിക്കുകയും മദീനയുടെ സുരക്ഷിതത്വം ഉറപ്പുവരുത്തുകയും ചെയ്യുകയെന്ന തന്ത്രമാണ് ഇതിനുശേഷമുള്ള പ്രവാചക യുദ്ധ ങ്ങളിലെല്ലാം കാണാന്‍ കഴിയുന്നത്. പ്രത്യാക്രമണത്തിലൂടെ സ്വന്തം നാടിനെ പ്രതിരോധിക്കുകയെന്ന തന്ത്രം. പ്രത്യാക്രമണ സ്വഭാവമുള്ള ഇത്തരം യുദ്ധങ്ങളെയാണ് കര്‍മശാസ്ത്രജ്ഞന്മാര്‍ 'ജിഹാദുത്വലബ്' എന്ന് വിളിച്ചിരിക്കുന്നത്. രണ്ടും രാഷ്ട്രനേതൃത്വത്തിന്റെ കീഴില്‍ സൈനികര്‍ നടത്തുന്ന യുദ്ധം തന്നെയാണ്; കലാപമോ കുഴപ്പം സൃഷ്ടിക്കലോ അല്ല.

യുദ്ധത്തിലല്ലാതെ മുസ്‌ലിംകള്‍ക്ക് ആയുധമെടുക്കുവാന്‍ അനുവാദമുള്ളത് സ്വന്തം ജീവനും സ്വത്തും സംരക്ഷിക്കുന്നതിന് വേണ്ടി നേര്‍ക്കു നേരെ നടത്തുന്ന പോരാട്ടത്തില്‍ മാത്രമാണ്. 'സ്വന്തത്തെ പ്രതിരോധിക്കുവാന്‍ വേണ്ടി കൊല്ലപ്പെടുന്നവന്‍ രക്തസാക്ഷിയാണ്; സ്വന്തം കുടും ബത്തെ സംരക്ഷിക്കുന്നതിനു വേണ്ടി കൊല്ലപ്പെടുന്നവന്‍ രക്തസാക്ഷിയാണ്; സ്വന്തം സ്വത്ത് സംരക്ഷിക്കുന്നതിനായി കൊല്ലപ്പെടുന്നവന്‍ രക്തസാക്ഷിയാണ്' എന്ന് പ്രവാചകന്‍ പറഞ്ഞതായി അബൂദാവൂദും തിര്‍മിദിയും നസാഇയും സ്വീകാര്യയോഗ്യമായ പരമ്പരയോടെ നിവേദനം ചെയ്തിട്ടുണ്ട്. നേര്‍ക്കുനേരെയുള്ള ഏറ്റുമുട്ടലില്‍ കൊല്ലപ്പെടുന്നവരെക്കുറിച്ചുള്ളതാണീ പ്രവാചകവചനം; അല്ലാതെ പ്രതികാ രത്തിനുള്ള തെളിവല്ല. സ്വന്തത്തെയോ കുടുംബത്തെയോ ആക്രമിക്കാന്‍ വരികയോ സ്വത്ത് കവരാന്‍ ശ്രമിക്കുകയോ ചെയ്യുന്ന അക്രമിയെ സായുധമായിത്തന്നെ പ്രതിരോധിക്കാമെന്നും പ്രസ്തുത പ്രതിരോധത്തിന് ആരുടെയും സമ്മതമാവശ്യമില്ലെന്നും പ്രസ്തുത പ്രതിരോധത്തി നിടയില്‍ കൊല്ലപ്പെട്ടവന് രക്തസാക്ഷിയുടെ സ്ഥാനമാണുള്ളതെന്നുമാണ് പ്രവാചകന്‍ ഇവിടെ പഠിപ്പിക്കുന്നത്. ഇങ്ങനെയുള്ള പ്രതിരോധ ത്തെ ജിഹാദായല്ല പണ്ഡിതന്മാര്‍ എണ്ണിയിരിക്കുന്നത്; കേവല പ്രതിരോധമായാണ്. 'ദഫ്ഉ സ്വാഇല്‍' എന്നാണു ഇതിനെ വിളിക്കുക.

തന്നെയോ  കുടുംബത്തെയോ ആക്രമിക്കുകയോ സ്വത്ത് കവരുകയോ ചെയ്തവനെ പിടികൂടി ഇസ്‌ലാമിക ശിക്ഷകള്‍ നടപ്പാക്കാന്‍ മുസ്‌ലി മിന് അധികാരം നല്‍കുന്നതല്ല പ്രതിരോധിക്കാന്‍ അനുവദിക്കുന്ന പ്രവാചകവചനം; പ്രത്യുത നേര്‍ക്കുനേരെയുള്ള പ്രതിരോധത്തിന് മാത്രം അനുവദിക്കുന്നതാണ്.

പ്രതികാരനടപടികള്‍ സ്വീകരിക്കാന്‍ ഭരണാധികാരിക്ക് മാത്രമേ അവകാശമുള്ളൂ എന്നാണു ഇസ്‌ലാമിക പാഠം. വ്യക്തികളെയോ ആള്‍ ക്കൂട്ടങ്ങളെയോ പ്രതികാരനടപടികള്‍ക്ക് അനുവദിച്ചാല്‍ വ്യാപകമായ കുഴപ്പമാണുണ്ടാവുക. അങ്ങനെ നിയമം കയ്യിലെടുക്കാന്‍ ഇസ്‌ ലാം ആരെയും അനുവദിച്ചിട്ടില്ല. ഇസ്‌ലാമിക രാഷ്ട്രത്തിലാണെങ്കില്‍ ഇങ്ങനെ നിയമം കയ്യിലെടുക്കുന്നവരെ കുറ്റവാളികളായി കണക്കാ ക്കുകയും ശരീഅത്ത് അനുശാസിക്കുന്ന ശിക്ഷ നല്‍കുകയും ചെയ്യും. ഖലീഫ ഉമറിന്റെ മകന്‍ ഉബൈദുല്ലക്ക് ഇസ്‌ലാമിക രാഷ്ട്രം വിധിച്ച ശിക്ഷതന്നെ ഉദാഹരണം. അബൂലുഅ് ലുഅ് എന്നറിയപ്പെട്ടിരുന്ന പേര്‍ഷ്യന്‍ അടിമയായ ഫൈറൂസാണ് ഇരുതലമൂര്‍ച്ഛയുള്ള കത്തികൊ ണ്ട് ഉമറിനെ(റ) കുത്തിക്കൊന്നത്. രക്ഷപ്പെടാനുള്ള ശ്രമത്തിനിടയ് ക്ക് പന്ത്രണ്ട് മുസ്‌ലിംകള്‍ക്ക് അയാള്‍ പരിക്കേല്‍പ്പിക്കുകയും ചെയ്തു. ഇവരില്‍ ഒന്‍പതു പേര്‍ ഈ പരിക്കിനാല്‍ പിന്നീട് മരണപ്പെട്ടു. അബൂലുഅ്‌ലുഅ് സ്വയം തന്നെ കുത്തിച്ചാവുകയും ചെയ്തു. തന്റെ പിതാവ് കൊല്ലപ്പെട്ട വിവരമറിഞ്ഞ ഉബൈദുല്ലാഹ് (റ) ക്ഷുഭിതനാവുകയും തന്റെ പിതാവിന്റെ ഘാതകരോട് പ്രതികാരം ചെയ്യാന്‍ തീരുമാനിച്ച് പുറത്തിറങ്ങുകയും ചെയ്തു. അബൂലുഅ്‌ലുഅയുടെ വീട്ടിലെത്തി അയാളുടെ ഭാര്യയേയും മകളെയും അദ്ദേഹം കൊന്നു. വധഗൂഡാലോചനയില്‍ പങ്കുണ്ടെന്ന് സംശയിച്ച ക്രൈസ്തവനായിരുന്ന ജാഫ്‌നയെയും പാര്‍സിയായിരുന്ന ഹുര്‍മുസാനെയും ഉബൈദു ല്ലാഹ് (റ) വധിച്ചു. ഉഥ്മാന്‍ (റ)വിന് ഇസ്‌ലാമിക രാഷ്ട്രത്തിന്റെ അധികാരമേറ്റശേഷം ആദ്യമായി തീരുമാനിക്കേണ്ടി വന്ന കേസായിരുന്നു ഇത്. വിധി തീരുമാനിക്കുവാനായി അലി (റ), അംറ്ബ്‌നുല്‍ ആസ് (റ) എന്നിവരടങ്ങുന്ന പ്രമുഖ സ്വഹാബിമാരുടെ ഒരു കമ്മിറ്റിയെ ചുമത ലപ്പെടുത്തി. നിയമം കയ്യിലെടുത്ത ഉബൈദുല്ല(റ)യെ ദാക്ഷിണ്യമൊന്നുമില്ലാതെ വധശിക്ഷക്ക് വിധേയമാക്കണമെന്നായിരുന്നു അലി(റ) യുടെ അഭിപ്രായം. ഇന്നലെ  ഉമറിനെ നഷ്ടപ്പെട്ട കുടുംബത്തിന് ഇന്ന് തന്നെ മകനായ ഉബൈദുള്ളയെക്കൂടി നഷ്ടപ്പെടുന്നത് സഹിക്കാനാവു കയില്ലെന്നും അതിനാല്‍ അദ്ദേഹത്തെ രക്ഷപ്പെടുത്തുന്നതിന് പഴുതുകളെന്തെങ്കിലുമുണ്ടോയെന്ന് അന്വേഷിക്കണമെന്നുമായിരുന്നു അംറ്ബ്‌ നുല്‍ ആസ് (റ) അടക്കമുള്ള മറ്റുള്ളവരുടെ അഭിപ്രായം. ഉഥ്മാന്ന്‍ വിഷയം പഠിച്ചു. നിയമം കയ്യിലെടുത്ത ഉബൈദുല്ല കുറ്റവാളി തന്നെയാ ണ് എന്നും അദ്ദേഹം വധശിക്ഷ അര്‍ഹിക്കുന്നുവെന്നും എന്നാല്‍ മരണപ്പെട്ടവരുടെ അനന്തരാവകാശികള്‍ക്ക് വേണമെങ്കില്‍ നഷ്ടപരിഹാ രം വാങ്ങി അദ്ദേഹത്തിനു വധശിക്ഷയില്‍ നിന്ന് ഇളവ് നല്കാമെന്നുമായിരുന്നു ഖലീഫ ഉഥ്മാനിന്റെ വിധി. മരണപ്പെട്ടവര്‍ക്ക് അനന്തരാ വകാശികളൊന്നുമില്ലാത്തതിനാല്‍ രാഷ്ട്രത്തിനാണ് നഷ്ടപരിഹാരം സ്വീകരിക്കാനുള്ള അവകാശം. നാലു പേര്‍ വധിക്കപ്പെട്ടതിനാല്‍ ഓരോ രുത്തര്‍ക്കും ആയിരം ദീനാര്‍ (4.25 കിലോഗ്രാം സ്വര്‍ണം) വീതം, ആകെ നാലായിരം ദീനാര്‍ നഷ്ടപരിഹാരമായി നല്‍കണമെന്നായിരുന്നു വിധി. പത്തു വര്‍ഷക്കാലം ഇസ്‌ലാമിക രാഷ്രത്തിന്റെ അധിപനായിരുന്ന ഉമറിന്റെ മകന്റെ പക്കല്‍ നഷ്ടപരിഹാരമായി നല്‍കാന്‍ ഈ തുകയുണ്ടായിരുന്നില്ല. തന്റെ സ്വന്തം സ്വത്തില്‍ നിന്ന് ഈ തുക പൊതുഖജനാവിലേക്ക് അടച്ചാണ് ഉഥ്മാന്‍, ഉബൈദുള്ളയെ വധശിക്ഷയി ല്‍ നിന്ന് രക്ഷിച്ചത്. സ്വന്തം പിതാവിന്റെ ഘാതകരോട് പ്രതികാരം ചെയ്യാന്‍ ഇസ്‌ലാമിക രാഷ്ട്രം അദ്ദേഹത്തിന്റെ മകനെ അനുവദിക്കു ന്നില്ലെങ്കില്‍, നിയമം കയ്യിലെടുക്കാന്‍ ഇസ്‌ലാം ആരെയും ഒരവസരത്തിലും അനുവദിക്കുന്നില്ലെന്നാണ് അതിന്റെ അര്‍ഥം. ഇസ്‌ലാമിക രാഷ്ട്രത്തിലില്ലാത്ത ഒരു അവകാശം മുസ്‌ലിമിന് ഇസ്‌ലാമികേതരമായ ഒരു രാജ്യത്തുണ്ടാവുകയില്ലെന്നുറപ്പാണല്ലോ.

അല്ലാഹുവിനു സാക്ഷ്യം വഹിക്കുന്നവരെന്ന നിലയില്‍ നീതിക്കുവേണ്ടി നിലനില്‍ക്കണമെന്ന് സത്വവിശ്വാസികളെ വിളിച്ച് ആഹ്വാനം ചെയ്യുന്ന ക്വുര്‍ആന്‍ വചനം (4:135) പ്രസ്തുത നീതി സ്വന്തത്തിനോ സ്വന്തക്കാര്‍ക്കോ ബന്ധുക്കള്‍ക്കോ എതിരാണെങ്കില്‍ പോലും അതിനു വേണ്ടി കണിശമായിത്തന്നെ നിലകൊള്ളണമെന്ന് നിര്‍ദേശിച്ച ശേഷം 'നീതിപാലിക്കാതെ നിങ്ങള്‍ തന്നിഷ്ടങ്ങളെ പിന്‍പറ്റരുത്; നിങ്ങള്‍ വളച്ചൊടിക്കുകയോ ഒഴിഞ്ഞുമാറുകയോ ചെയ്യുന്നപക്ഷം, നിങ്ങള്‍ ചെയ്യുന്നതിനെക്കുറിച്ചെല്ലാം സൂക്ഷ്മമായി അറിയുന്നവനാകുന്നു അല്ലാഹു, തീര്‍ച്ച'യെന്ന് പറഞ്ഞുകൊണ്ടാണ് അവസാനിക്കുന്നതെന്ന കാര്യം ശ്രദ്ധേയമാണ്. പ്രതിക്രിയ നടപ്പാക്കുകയെന്ന നീതി നിര്‍വഹി ക്കുവാന്‍ അല്ലാഹു ഉത്തരവാദപ്പെടുത്തിയിരിക്കുന്നത് ഭരണാധികാരികളെയാണ്. അക്കാര്യത്തില്‍ ഭരണാധികാരിയുടെ നിര്‍ദേശം പാലി ക്കുക മാത്രമാണ് പൗരന്‍മാരുടെ ഉത്തരവാദിത്തം. ജീവിക്കുന്ന സ്വന്തം നാട്ടില്‍ നീതി ലഭിക്കാത്ത സാഹചര്യമുണ്ടായാല്‍ അവിടെ ആയുധ മെടുക്കാമെന്ന് പഠിപ്പിക്കുന്ന ഇസ്‌ലാമിക പ്രമാണങ്ങളൊന്നുമില്ല. ആയുധമെടുത്ത് നീതി നടപ്പാക്കാന്‍ ശ്രമിക്കുന്നവര്‍ക്ക്, അതിന് എന്തെന്തു ന്യായീകരണങ്ങളുണ്ടെങ്കിലും, പ്രസ്തുത നീതിനിര്‍വഹണത്തെ ഇസ്‌ലാം അനുവദിക്കുന്നില്ല. അവരുടെ ന്യായീകരണങ്ങളെല്ലാം ഉള്‍ക്കൊ ള്ളുന്നത് ആത്മഹത്യാപ്രത്യയശാസ്ത്രത്തിന്റെ യുക്തിബോധമല്ലാതെ മറ്റൊന്നുമല്ലെന്ന്  ന്യായീകരണവിദഗ്ധരെ കാലം പഠിപ്പിക്കും; പ്രഭാ കരനും ബിന്‍ ലാദനും തോറ്റിടത്ത് ജയിക്കാന്‍ പാര്‍ട്ടികളുടെ പേരോ അക്ഷരങ്ങളുടെ എണ്ണമോ നിരന്തരം മാറ്റിക്കൊണ്ടിരിന്നാല്‍ മതിയെന്ന വങ്കത്തത്തിന് ബലികൊടുക്കേണ്ടി വരിക സമുദായോദ്ദാരണത്തിനുപയോഗിക്കപ്പെടേണ്ട യുവത്വത്തിന്റെ ഊര്‍ജ്ജവും ജീവനുമാണല്ലോ എന്നാലോചിക്കുമ്പോഴാണ് സങ്കടം തോന്നുന്നത്.

വ്യക്തികളുടെ ലൈംഗികാവകാശങ്ങള്‍ സംരക്ഷിക്കപ്പെടേണ്ടതുണ്ടെന്ന് ശക്തമായി പഠിപ്പിച്ച ദര്‍ശനമാണ് ഇസ്‌ലാം. ഇണകളുടെ തെര ഞ്ഞെടുപ്പില്‍ പരസ്പരമുള്ള ഇഷ്ടത്തിന് പ്രധാനപ്പെട്ട പരിഗണ നല്‍കണമെന്നാണ് പ്രവാചക നിര്‍ദേശം. തന്റെ സംരക്ഷണയിലുള്ള അനാഥ പെണ്‍കുട്ടിയെ വിവാഹാലോചന നടത്തിയ രണ്ടുപേരില്‍ ധനികനെയാണ് തനിക്കിഷ്ടമെന്നും എന്നാല്‍ ദരിദ്രനോടാണ് അവള്‍ക്ക് താല്‍പര്യ മെന്നും അറിയിച്ചുകൊണ്ട് അവളെ ആര്‍ക്കു വിവാഹം ചെയ്തുകൊടുക്കണമെന്ന് അഭിപ്രായമാരാഞ്ഞയാളോട് 'പരസ്പരം ഇഷ്ടപ്പെട്ട വര്‍ക്ക് വിവാഹമല്ലാതെ മറ്റൊന്നും ഗുണകരമായി തെളിയിക്കപ്പെട്ടിട്ടില്ല'യെന്നതായിരുന്നു പ്രവാചകന്റെ (സ) മറുപടി(ഇബ്‌നു മാജ, ഹാക്വിം, അസ്വഹീഹ 624). ഒരാള്‍ വിവാഹമന്വേഷിച്ച് വന്നാല്‍ അയാളുടെ മതബോധത്തിലും സ്വഭാവചര്യകളിലും നിങ്ങള്‍ സംതൃപ്ത രാണെങ്കില്‍ അയാള്‍ക്ക് നിങ്ങള്‍ വിവാഹം ചെയ്തുകൊടുക്കണമെന്നും അങ്ങനെ ചെയ്തിട്ടില്ലെങ്കില്‍ വമ്പിച്ച കുഴപ്പങ്ങളും നാശനഷ്ടങ്ങളു മാണുണ്ടാവുകയെന്നും പഠിപ്പിച്ച മുഹമ്മദ് നബി (സ)       (തിര്‍മിദി, ഇബ്‌നു മാജ, അസ്വഹീഹ 1022), ഇഷ്ടപ്പെട്ടവര്‍ തമ്മില്‍ ഇണകളായി ത്തീരുവാനുള്ള അവസരമൊരുക്കണം എന്നുതന്നെയാണ് വിശ്വാസികളെ തെര്യപ്പെടുത്തുന്നത്. സ്വന്തം ഇണയുമായുള്ള സംസര്‍ഗത്തില്‍ താല്‍പര്യം കാണിക്കാതെ നമസ്‌കാരത്തിലും വ്രതാനുഷ്ഠാനത്തിലും നിമഗ്‌നനായി ആത്മീയോല്‍കര്‍ഷം നേടാമെന്ന് ധരിച്ച ഉഥ്മാനുബ്‌നു മദ്വ്ഊനി(റ)നോട് 'താങ്കള്‍ എന്റെ ചര്യയെ അവഗണിക്കുകയാണോ' എന്ന് ഗുണദോഷിക്കുകയും 'താങ്കള്‍ക്ക് താങ്കളുടെ ശരീരത്തോടും ഇണകളോടുമെല്ലാം ബാധ്യതകളുണ്ട്'(അഹ്മദ്, അബൂദാവൂദ്, ഇര്‍വാഉല്‍ ഗലീല്‍ 2015) എന്ന് പഠിപ്പിക്കുകയും ചെയ്ത പ്രവാചകന്‍ (സ) ഒരാളുടെ ലൈംഗികാവകാശങ്ങളെ ഹനിക്കാന്‍ സ്വന്തം ഇണക്കുപോലും അവകാശമില്ലെന്നാണ് പഠിപ്പിക്കുന്നത്. ലൈംഗിക ശേഷിയില്ലാ ത്ത പുരുഷനില്‍നിന്ന് വിവാഹ മോചനം നേടാന്‍ സ്ത്രീകളെ അനുവദിക്കുകയും(സ്വഹീഹുല്‍ ബുഖാരി) അങ്ങാടിയില്‍നിന്ന് ഏതെങ്കിലും സ്ത്രീകളെ കണ്ട് പ്രലോഭിതനായാല്‍ തന്റെ ഇണയ്ക്കടുത്തെത്തി തന്റെ തൃഷ്ണ തീര്‍ക്കണമെന്ന് ഉപദേശിക്കുകയും(സ്വഹീഹു മുസ്‌ലിം) അടുപ്പിനടുത്താണെങ്കില്‍ പോലും ഇണയുടെ ലൈംഗികതാല്‍പര്യങ്ങളെ അവഗണിക്കരുതെന്ന് പഠിപ്പിക്കുകയും(തിര്‍മിദി, അഹ്മദ്, അസ്വ ഹീഹ 1202) ചെയ്ത പ്രവാചകന്‍ (സ) സ്ത്രീക്കും പുരുഷനുമെല്ലാം ലൈംഗികാവകാശങ്ങളുണ്ടെന്ന് വ്യക്തമാക്കുകയും അവ നേടുന്നതില്‍ നിന്ന് തടയാന്‍ ആര്‍ക്കും അവകാശമില്ലെന്ന് തെര്യപ്പെടുത്തുകയുമാണ് ചെയ്യുന്നത്.

വ്യക്തികളുടെ ലൈംഗികാവകാശങ്ങളില്‍പെട്ടതാണ് തങ്ങള്‍ക്ക് ഇഷ്ടപ്പെട്ടവരെ മാത്രം ഇണയായി സ്വീകരിക്കാനുള്ള അവകാശം. ഈ രംഗ ത്ത് പ്രലോഭനങ്ങളൊന്നുമുണ്ടായിക്കൂടെന്നാണ് ഇസ്‌ലാമിന്റെ അധ്യാപനം. വിവാഹാന്വേഷണം നടത്തുന്നവര്‍ ഇണയെ കാണണമെന്നും (സ്വഹീഹു മുസ്‌ലിം) വിവാഹയോഗ്യയാണെന്ന് ബോധ്യപ്പെടുംവരെ കാഴ്ച തുടരാമെന്നും(അബൂദാവൂദ്, അഹ്മദ്, ഇര്‍വാഉല്‍ ഗലീല്‍ 1701) ഇണയെ കാണുന്നത് ഇണകള്‍ രണ്ടുപേര്‍ക്കുമിടയില്‍ സ്വരച്ചേര്‍ച്ചയുണ്ടാക്കുന്നതിന് ഉപകരിക്കുമെന്നുമാണ്(അഹ്മദ്, ഹാക്വിം, അസ്വ ഹീഹ 96) പ്രവാചകനിര്‍ദേശം.

പുരുഷന്‍ സ്ത്രീയുടെ പിതാവിനോടോ രക്ഷിതാക്കളോടോ ആണ് വിവാഹാലോചന നടത്തേണ്ടത് എന്നതിനാല്‍ അവളുടെ സമ്മതമില്ലാതെ അവളെ ആര്‍ക്കും വിവാഹം ചെയ്തുകൊടുക്കരുതെന്ന് പ്രവാചകന്‍ (സ) പിതാക്കളോട്/രക്ഷിതാക്കളോട് പ്രത്യേകം കല്‍പിച്ചതായി കാണാന്‍ കഴിയും. കന്യകയേയും വിധവയേയുമെല്ലാം അവേരാട് ചോദിച്ച ശേഷമായിരിക്കണം മറ്റൊരാള്‍ക്ക് വിവാഹം ചെയ്തുകൊടു ക്കേണ്ടതെന്നാണ് മുഹമ്മദ് നബി (സ) നിഷ്‌കര്‍ഷിച്ചിരിക്കുന്നത്(സ്വഹീഹുല്‍ ബുഖാരി, സ്വഹീഹു മുസ്‌ലിം). വിധവകള്‍ കാര്യപ്രാപ്തി യുള്ളവരും അനുഭവസമ്പത്തുള്ളവരുമായതിനാല്‍ അവരോട് വിവാഹകാര്യം കൂടിയാലോചിക്കണമെന്നും സ്വന്തം കാര്യത്തില്‍ അവര്‍ ക്ക് രക്ഷാകര്‍ത്താക്കളേക്കാള്‍ അവകാശമുണ്ടെന്നും കന്യകയോടും സമ്മതം ചോദിക്കണമെന്നും അവള്‍ മൗനം അവലംബിക്കുകയാണെ ങ്കില്‍ അത് സമ്മതമായി കണക്കാക്കണമെന്നുമെല്ലാം നിഷ്‌കര്‍ഷിച്ച പ്രവാചകന്‍ (സ) സ്വന്തം ഇണയെ തീരുമാനിക്കാനും തിരസ്‌കരിക്കാനു മുള്ള സ്ത്രീകളുടെ അവകാശത്തെക്കുറിച്ചാണ് സമൂഹത്തെ തെര്യപ്പെടുത്തിയിരിക്കുന്നത്(സ്വഹീഹു മുസ്‌ലിം). സ്വന്തം ഇഷ്ടപ്രകാരമ ല്ലാതെ, അറേബ്യന്‍ ഗോത്രവര്‍ഗരീതിയനുസരിച്ച് വിവാഹം ചെയ്യപ്പെട്ടവര്‍ക്ക് തങ്ങളുടെ ഇഷ്ടപ്രകാരം വിവാഹബന്ധം തുടരാനോ ബന്ധം അവസാനിപ്പിച്ച് വിവാഹമോചനം നേടാനോ ഉള്ള സ്വാതന്ത്ര്യം മുഹമ്മദ് നബി (സ) നല്‍കിയ ഒന്നിലധികം സംഭവങ്ങള്‍ ഹദീഥ് ഗ്രന്ഥങ്ങള്‍ നിവേദനം ചെയ്തിട്ടുണ്ട്(ഇബ്‌നുമാജ; സ്വഹീഹു ഇബ്‌നുമാജ 1520, സ്വഹീഹുല്‍ ബുഖാരി, അഹ്മദ്, ഇര്‍വാഉല്‍ ഗലീല്‍ 1835). തന്റെ ഇണ ആരായിരിക്കണമെന്ന് തീരുമാനിക്കാന്‍ പുരുഷനും പെണ്ണിനും അവകാശം നല്‍കിയ ഇസ്‌ലാം വളരെ പ്രധാനപ്പെട്ടതും ചരിത്രത്തിലുടനീളം അവഗണിക്കപ്പെട്ടുപോന്നതുമായ ഒരു ലൈംഗികാവകാശമാണ് വകവെച്ചുനല്‍കിയത്.

പ്പോഴാണ് വിവാഹം വേണ്ടതെന്ന് തീരുമാനിക്കാനുള്ള അവകാശം പുരുഷനും സ്ത്രീക്കുമാണെന്നാണ് ഇസ്‌ലാമിന്റെ അധ്യാപനം. വ്യക്തിപരമായ ലൈംഗികവിശുദ്ധി കാത്തുസൂക്ഷിക്കുകയും മതത്തിന്റെ പകുതി പൂര്‍ത്തിയാക്കുകയുമാണ് ഒരു മുസ്‌ലിമിനെ സംബന്ധി ച്ചിടത്തോളം തന്റെ വിവാഹ ലക്ഷ്യമെന്നതിനാല്‍ എപ്പോള്‍ വിവാഹിതരാകണമെന്ന് തീരുമാനിക്കാന്‍ ഓരോരുത്തര്‍ക്കും അവകാശ മുണ്ട്; വിവാഹിതരാകുന്നതുവരെ ലൈംഗിക വിശുദ്ധി കാത്തുസൂക്ഷിക്കാന്‍ വ്യക്തിപരമായി വിവാഹം നീട്ടിവെക്കുന്നവര്‍ ബാധ്യസ്ഥരാ ണെന്ന് മാത്രമേയുള്ളു. വൈക്തികമായ കാരണങ്ങളാല്‍ വിവാഹം നീട്ടിവെക്കേണ്ടി വരുന്നവര്‍ തങ്ങളുടെ കണ്ണും കാതും മനസ്സും വിമല മായി സൂക്ഷിക്കാന്‍ പോന്ന രീതിയില്‍ വ്രതമനുഷ്ഠിക്കണമെന്നാണ് പ്രവാചകന്‍ നിര്‍ദേശിച്ചിരിക്കുന്നത് എന്ന വസ്തുത നല്‍കുന്ന വെളി ച്ചം എന്തിന്, എപ്പോള്‍, വിവാഹമെന്ന കാര്യത്തില്‍ മുസ്‌ലിംകള്‍ക്ക് കൃത്യമായ ദിശാബോധം നല്‍കാന്‍ പര്യാപ്തമാണ്.

താന്‍ എപ്പോഴാണ് വിവാഹിതനാവേണ്ടതെന്ന് തീരുമാനിക്കാന്‍ പുരുഷന് അവകാശമുള്ളതുപോലെ, രക്ഷിതാവുമായി കൂടിയാലോചിച്ച ശേഷം തന്റെ വിവാഹപ്രായം തീരുമാനിക്കാന്‍ സ്ത്രീയ്ക്കും അനുവാദമുണ്ട്. തന്റെ മകളെ എപ്പോഴാണ് വിവാഹം ചെയ്ത് അയക്കേ ണ്ടതെന്ന് തീരുമാനിക്കുന്നത് പിതാവാണെങ്കിലും അവള്‍ക്ക് അതിന് സമ്മതമുണ്ടോയെന്ന് ആരായുകയും സമ്മതമില്ലെങ്കില്‍ അതില്‍നിന്ന് പിന്‍മാറുകയും ചെയ്യേണ്ടത് അയാളുടെ ബാധ്യതയാണ്. വിവാഹിതയാകുന്നത് വരെ സ്വന്തം പാതിവ്രത്യത്തിന് കളങ്കമുണ്ടാകുന്ന വാ ക്കോ നോട്ടമോ പ്രവര്‍ത്തനങ്ങളോ ഇല്ലാതെ സൂക്ഷിക്കാന്‍ അവളും അതുവരാതെ സംരക്ഷിക്കാന്‍ അയാളും ബാധ്യസ്ഥരാണെന്നു മാത്രമേ യുള്ളു. തന്റെ മകളുടെ ലൈംഗികവിശുദ്ധി കളങ്കപ്പെടുമെന്ന് ആശങ്കിക്കുന്ന ഒരാള്‍, അവളുടെ സമ്മതത്തോടെ അവളെ ഒരാള്‍ക്ക് വിവാ ഹം ചെയ്തുകൊടുക്കുന്നതിന് പ്രായം മാനദണ്ഡമാക്കേണ്ടതില്ലെന്നു തന്നെയാണ് ഇസ്‌ലാമിന്റെ പക്ഷം. ഇതില്‍ തീരുമാനമെടുക്കേണ്ടത് അവളും പിതാവും തന്നെയാണ്. ഈ സ്വാതന്ത്ര്യത്തില്‍ ഇടപെടുന്നത് ഒരാളുടെ ലൈംഗികാവകാശത്തിനു നേരെയുള്ള വെല്ലുവിളിയാ യാണ് കാണേണ്ടത് എന്നാണ് ഇസ്‌ലാമിക പ്രമാണങ്ങള്‍ പഠിപ്പിക്കുന്നത്. ഒരാള്‍ക്ക് അതിശക്തമായ ലൈംഗികാഭിനിവേശമുണ്ടാവുകയും ലൈംഗികാസ്വാദനം മൂലമുണ്ടാകുന്ന ബാധ്യതകള്‍ ഏറ്റെടുക്കാന്‍ അയാള്‍ സന്നദ്ധമാവുകയും ചെയ്തിട്ടും ലൈംഗികാസക്തിയുടെ പൂര്‍ ത്തീകരണത്തിനാവശ്യമായ നിയമപരവും വിശുദ്ധവും പ്രകൃതിപരവുമായ ഒരേയൊരു മാര്‍ഗം അയാളുടെ മുമ്പില്‍ കൊട്ടിയടക്കുന്നതി നേക്കാള്‍ വലിയ മനുഷ്യാവകാശ ലംഘനമെന്താണുള്ളത്?

പെണ്‍കുട്ടികളുടെ വിവാഹപ്രായത്തില്‍ യാതൊരു നിബന്ധനയും വെക്കാന്‍ ആര്‍ക്കും അവകാശമില്ലെന്നല്ല ഇപ്പറഞ്ഞതിനര്‍ഥം. തനിക്ക് വൈവാഹിക ജീവിതം നയിക്കാന്‍ മാനസികവും ശാരീരികവുമായ പ്രാപ്തിയുണ്ടാകുന്നതുവരെ വിവാഹം നീട്ടിവെക്കാന്‍ അവള്‍ക്കും പിതാവിനും അവകാശമുണ്ടെന്നു തന്നെയാണ് പ്രവാചകജീവിതത്തിലെ വ്യത്യസ്ത സംഭവങ്ങളില്‍നിന്ന് മനിലാവുന്നത്. പ്രവാചകപത്‌ നിമാര്‍ക്കിടയിലെ ഒരേയൊരു കന്യകയായിരുന്ന ആയിശ(റ)യുമായി പ്രവാചകന്‍(സ) തന്റെ വൈവാഹിക ജീവിതമാരംഭിക്കുന്നത് അവ ര്‍ക്ക് ഒമ്പതു വയസ്സ് പ്രായമുള്ളപ്പോഴായിരുന്നു(സ്വഹീഹുല്‍ ബുഖാരി). എന്നാല്‍, പ്രവാചകപുത്രിയായ ഫാത്വിമയും പ്രവാചകന്റെ പിതൃവ്യപുത്രനായ അലി(റ)യും തമ്മിലുള്ള വൈവാഹിക ജീവിതം ആരംഭിച്ചത് പതിനെട്ടാമത്തെ വയസ്സിലായിരുന്നു എന്ന് ചരിത്രകാ രന്‍മാര്‍ വ്യക്തമാക്കിയിട്ടുണ്ട്(താരീഖുത്ത്വബ്‌രി; കിത്താബുത്വബഖാത്, ഇബ്‌നു സഅദ് 8:11). അതിനുമുമ്പ് ഫാത്വിമ(റ)യെ വിവാഹമാ ലോചിച്ച അബൂബക്കര്‍ (റ), ഉമര്‍ (റ) എന്നീ അരുമ ശിഷ്യന്‍മാരോട് അവള്‍ ചെറുപ്പമാണെന്ന് പറഞ്ഞ് അവളെ വിവാഹം ചെയ്തുകൊടു ക്കാന്‍ സന്നദ്ധമാകാതിരിക്കുകയാണ് നബി (സ) ചെയ്തത്(നസാഈ: സഹീഹു നസാഈ 3221) ഇവയില്‍നിന്ന്, ഒരുവളുടെ പാകതയും പക്വ തയും വൈവാഹിക ജീവിതത്തിന്റെ അനിവാര്യതയും പരിഗണിച്ചുകൊണ്ട് അവരുടെ വിവാഹപ്രായം നിശ്ചയിക്കാനും നീട്ടിവെക്കാ നും അവള്‍ക്കും രക്ഷിതാക്കള്‍ക്കും അവകാശമുണ്ടെന്ന വസ്തുത വ്യക്തമാകുന്നുണ്ട്.

ഇന്നത്തെ ലോകക്രമമനുസരിച്ച്, വൈവാഹിക ജീവിതമാരംഭിക്കുന്നതിനുമുമ്പ് പുരുഷനും സ്ത്രീയും അവര്‍ക്കാവശ്യമായ ഭൗതികവും ആത്മീയവുമായ വിജ്ഞാനീയങ്ങള്‍ കരസ്ഥമാക്കാന്‍ ശ്രമിക്കുന്നതുതന്നെയാണ് അഭികാമ്യം. ഔദ്യോഗിക ജീവിതം ആഗ്രഹിക്കാത്ത സ്ത്രീ കള്‍ പോലും ആവശ്യമായ വിദ്യാഭ്യാസം നേടേണ്ടത് അവരുടെ കുടുംബജീവിതത്തിനും കുട്ടികളുടെ വൈജ്ഞാനിക പരിചരണത്തിനും സാമൂഹ്യബോധമുണ്ടാകുന്നതിനുമെല്ലാം അനിവാര്യമാണ്. വൈവാഹിക ജീവിതം ആരംഭിച്ചുകഴിഞ്ഞാല്‍, പലപ്പോഴും സ്ത്രീകള്‍ക്ക് വിദ്യാഭ്യാസം തുടരാനാവാത്ത സ്ഥിതിവിശേഷമാണുണ്ടാവുക. മാതൃത്വത്തിന്റെ ഉന്നതമായ ഉത്തരവാദിത്തമേറ്റെടുക്കുന്നവര്‍ക്ക് സ്വയം പഠനത്തേക്കാളധികം ഉമ്മയുടെ ബാധ്യതാനിര്‍വഹണത്തിന് പ്രാധാന്യം നല്‍കേണ്ടതായും ഭര്‍തൃപരിചരണത്തിനും കുടുംബ ഭരണത്തിനും തന്നെ തന്റെ സമയം തികയാതെ വരികയും ചെയ്യുന്ന അവസ്ഥയുണ്ടാകാവുന്നതാണ്. അതുകൊണ്ടുതന്നെ ഇന്നത്തെ സാഹചര്യത്തില്‍ പതിനെട്ടുവയസ്സിനുമുമ്പ് വിവാഹിതരാകേണ്ടതില്ലെന്ന് തീരുമാനിക്കുകയും അതുവരെ പഠനകാര്യങ്ങളില്‍ മാത്രം ശ്രദ്ധ കേന്ദ്രീകരിക്കുക യും ചെയ്യുന്നതുതന്നെയാണ് അഭികാമ്യമായ നടപടി. പക്ഷെ അത് തീരുമാനിക്കേണ്ടത് പെണ്‍കുട്ടിയും അവളുടെ രക്ഷിതാക്കളുമാണ്. അവരെ അതിന് പ്രേരിപ്പിക്കുന്നതിനുള്ള ബോധവല്‍കരണ നടപടികളും വിവാഹേതര ലൈംഗികാസ്വാദനത്തിലേക്ക് നയിക്കുന്ന സാമൂ ഹ്യസാഹചര്യങ്ങളുടെ നിര്‍മൂലനവുമാണ് ഇക്കാര്യത്തില്‍ സര്‍ക്കാരിന് ചെയ്യാനാവുന്ന നടപടികള്‍. ഇതൊന്നും ചെയ്യാതെ, ലൈംഗികോ ത്തേജനത്തിന് നിമിത്തമാകുന്ന സാഹചര്യങ്ങള്‍ സൃഷ്ടിക്കാന്‍ വിപണീവിദഗ്ധര്‍ക്ക് സകലവിധ സ്വാതന്ത്ര്യവും നല്‍കുകയും വിവാഹേ തര ലൈംഗികാസ്വാദനങ്ങള്‍ക്കുള്ള അവസരങ്ങളുണ്ടാക്കുകയും ചെയ്യുകയും ലൈംഗികാസ്വാദനത്തിനുള്ള പ്രകൃതിപരമായ മാര്‍ഗം സ്വീകരിക്കുന്നവര്‍ക്ക് മാത്രം ശിക്ഷ വിധിക്കുകയും ചെയ്യുന്നത് ക്രൂരതയും മനുഷ്യാവകാശ ലംഘനവുമാണ്. വ്യക്തികളുടെ അവകാശ ങ്ങള്‍ സംരക്ഷിക്കാന്‍ ബാധ്യതയുള്ള ജനാധിപത്യക്രമത്തില്‍നിന്ന് അത്തരമൊരു മനുഷ്യാവകാശ ലംഘനമുണ്ടാകുന്നത് നാണക്കേടാണ്.

ഇബ്രാഹിം നബി (അ) യുടെ കാലം മുതൽ വിശ്വാസികൾക്കിടയിൽ നിലനിന്നിരുന്ന ഒരു ആചാരമാണ് ലിംഗാഗ്രഛേദനം.മുഹമ്മദ് നബി (സ) അത് പഠിപ്പിക്കുകയും പ്രോത്സാഹിപ്പിക്കുകയും ചെയ്തതുകൊണ്ടാണ് വിശ്വാസികൾ അതു ചെയ്യുന്നത്.

അഗ്രചര്‍മഛേദനം നടത്തിയ പുരുഷന്മാര്‍ക്ക് സ്ത്രീ ലൈംഗികതയോട് താല്‍പര്യം കുറയുമെന്ന മാതൃഭൂമി ലേഖകന്റെ 'സാമാന്യ അറിവ്' വസ്തുതാ വിരുദ്ധമാണ്. സ്വവര്‍ഗപ്രേമികള്‍ സംഘടിപ്പിക്കാറുള്ള 'പൊങ്ങച്ചപ്രകടന' (pride parade) ങ്ങളില്‍ തങ്ങള്‍ മനുഷ്യാവകാശങ്ങ ള്‍ക്കു വേണ്ടിയാണ് നിലനില്‍ക്കുന്നതെന്ന് വരുത്തിത്തീര്‍ക്കാനായി ആണുങ്ങളില്‍ നടക്കുന്ന ലിംഗാഗ്രചര്‍മഛേദ (circumci-sion) ത്തിനെതി രെയുള്ള ബോധവല്‍ക്കരണവും പ്രദര്‍ശനങ്ങളുമുണ്ടാവാറുണ്ട്. അഗ്രചര്‍മഛേദം നടത്തിയ പുരുഷന്മാര്‍ക്ക് അത് നടത്താത്ത പുരുഷന്‍ മാരെക്കാള്‍ ലിംഗത്തിന് സംവേദനക്ഷമത കുറവായിരിക്കുമെന്ന് പ്രചരിപ്പിച്ചുകൊണ്ടുള്ള അവരുടെ ലഘുലേഖകളില്‍ നിന്നാവാം പള്ള ത്തിന് ഈ വിവരം ലഭിച്ചത്. ലിംഗാഗ്രചര്‍മഛേദത്തിനെതിരെയുള്ള സ്വവര്‍ഗകാമികളുടെ വാദങ്ങളെ സമര്‍ഥിച്ചുകൊണ്ട് ഏഴുപേര്‍ ചേര്‍ന്നെഴുതിയ ഒരു ലേഖനം ബ്രിട്ടീഷ് ജേണല്‍ ഓഫ് യൂറോളജി ഇന്റര്‍നാഷണലിന്റെ 2007 ഏപ്രില്‍ ലക്കത്തില്‍ പ്രസിദ്ധീകരിച്ചിരുന്നു.     (Morris L. Sorrells, James L. Snyder, Mark D. Reiss, Christopher Eden, Marilyn F. Milos, Norma Wilcox and Robert S. Van Howe: “Fine-touch pressure thres holds in the adult penis”, BJU International Volume 99, Issue 4, April 2007, pages 864-869.) പ്രസ്തുത ലേഖനത്തില്‍ സമര്‍ത്ഥിച്ചിരിക്കുന്ന കാര്യങ്ങള്‍ വാസ്തവവിരുദ്ധമാണെന്നും തങ്ങളുടെ വാദം സമര്‍ത്ഥിക്കാനായി പല സ്ഥിതിവിവരക്കണക്കുകളും വളച്ചൊടിക്കുകയോ അതിശയോ ക്തിപരമായി വ്യാഖ്യാനിക്കുകയോ ആണ് ലേഖകന്മാര്‍ ചെയ്തിട്ടുള്ളതെന്നും സമര്‍ഥിച്ചുകൊണ്ട് പ്രസ്തുത ജേണലില്‍ തന്നെ രണ്ടു മാസ ത്തിനുശേഷം വന്ന കുറിപ്പ് ഈ വാദം എത്രത്തോളം ബാലിശമാണെന്ന് വ്യക്തമാക്കുന്നുണ്ട്. ലിംഗാഗ്രചര്‍മഛേദനം ലൈംഗികാസ്വാദനത്തെ യോ ലൈംഗിക ത്വരയെയോ ഒരുവിധത്തിലും പ്രതികൂലമായി ബാധിക്കുകയില്ലെന്ന വസ്തുതയാണ് പ്രസ്തുത കുറിപ്പില്‍ സമര്‍ഥിക്കപ്പെട്ടി രിക്കുന്നത്.( Jake H. Waskett, Brian J. Morris: Fine-Touch Pressure Thresholds In The Adult Penis, BJU International, Volume 99, Issue 6, June 2007, pages 1551-1552.)

ഓസ്‌ട്രേലിയയില്‍, സിഡ്‌നി സര്‍വകലാശാലയിലെ മോളിക്യുലാര്‍ മെഡിക്കല്‍ സയന്‍സസ് പ്രഫസറും 250ഓളംവരുന്ന ഗവേഷണപ്രബന്ധ ങ്ങളുടെ കര്‍ത്താവും രണ്ട് അന്താരാഷ്ട്ര ഗവേഷണ പ്രസിദ്ധീകരണങ്ങളുടെ പത്രാധിപസമിതി അംഗവുമായ പ്രൊഫ. ബ്രിയാന്‍ മോറിസ് എഴുതിയ ലിംഗാഗ്രചര്‍മഛേദനത്തിന് അനുകൂലമായി എന്ന ഗ്രന്ഥത്തില്‍ എന്തുകൊണ്ടാണ് മതാഭിമുഖ്യമില്ലാത്തവര്‍ പോലും പരിഛേദ നയെ പ്രോത്സാഹിപ്പിക്കുന്നതെന്ന് വ്യക്തമാക്കുന്നുണ്ട്. മൂത്രാശയ രോഗങ്ങളെയും എയിഡ്‌സ് അടക്കമുള്ള ലൈംഗിക രോഗങ്ങളെയും ലിം ഗാര്‍ബുദത്തെയും പ്രോസ്‌റ്റേറ്റ് ക്യാന്‍സറിനെയുമെല്ലാം പ്രതിരോധിക്കുവാന്‍ മാത്രമല്ല, ലൈംഗികാസ്വാദനത്തെയും ശേഷിയേയും ഗുണപ രമായി സ്വാധീനിക്കുവാനും ലിംഗാഗ്രചര്‍മഛേദനം വഴി സാധിക്കുമെന്ന് ഈ പുസ്തകത്തില്‍ വസ്തുനിഷ്ഠമായി സമര്‍ഥിച്ചിട്ടുണ്ട്.( Brian Morris: In Favour of Circumcision, University of New South Wales Press, 1999.)

മ്ലേച്ഛവും വർജിക്കപ്പെടേണ്ടതുമായ കൊടിയ പാപമായാണ് ഇസ്‌ലാം സ്വവർഗാനുരാഗത്തെയും രതിയെയും കാണുന്നത്.സ്വവർഗ കാമിക ളായിരുന്ന സദൂം നിവാസികൾക്കിടയിലേക്ക് നിയോഗിക്കപ്പെട്ട പ്രവാചകനായ ലൂത്ത്(അ)അവിടുത്തുകാരുമായി നടത്തിയ സംഭാഷണങ്ങ ളിൽ നിന്ന് സ്വവർഗ രതിയെക്കുറിച്ചുള്ള ഇസ്‌ലാമിക വീക്ഷണം ആർക്കും മനസ്സിലാക്കാൻ കഴിയും.

'നിങ്ങള്‍ ലോകരില്‍ നിന്ന് ആണുങ്ങളുടെ അടുക്കല്‍ ചെല്ലുകയാണോ? നിങ്ങളുടെ രക്ഷിതാവ് നിങ്ങള്‍ക്ക് വേണ്ടി സൃഷ്ടിച്ചു തന്നിട്ടുള്ള നിങ്ങളുടെ ഇണകളെ വിട്ടുകളയുകയുമാണോ? അല്ല, നിങ്ങള്‍ അതിക്രമകാരികളായ ഒരു ജനത തന്നെ.'(1)

'നിങ്ങള്‍ കാമനിവൃത്തിക്കായി സ്ത്രീകളെ വിട്ട് പുരുഷന്‍മാരുടെ അടുക്കല്‍ ചെല്ലുകയാണോ? അല്ല. നിങ്ങള്‍ അവിവേകം കാണിക്കുന്ന ഒരു ജനതയാകുന്നു.'(2)

'സ്ത്രീകളെ വിട്ട് പുരുഷന്‍മാരുടെ അടുത്ത് തന്നെ നിങ്ങള്‍ കാമവികാരത്തോടെ ചെല്ലുന്നു. അല്ല, നിങ്ങള്‍ അതിരുവിട്ട് പ്രവര്‍ത്തിക്കുന്ന ഒരു ജനതയാകുന്നു.'(3)

'അദ്ദേഹം പറഞ്ഞു: എന്റെ രക്ഷിതാവേ, കുഴപ്പക്കാരായ ഈ ജനതക്കെതിരില്‍ എന്നെ നീ സഹായിക്കണമേ.'(4)

സ്വവര്‍ഗരതിക്കാരുടെ സമൂഹത്തെകുറിച്ച വ്യക്തമായ ചിത്രം നല്‍കുവാന്‍ പര്യാപ്തമാണ് ഈ വചനങ്ങള്‍. അതിക്രമകാരികളായ ജനത (ഖൗമുന്‍ ആദ്ദൂന്‍), അവിവേകം കാണിക്കുന്ന ജനത (ഖൗമുന്‍ തജ്ഹലൂന്‍), അതിരുവിട്ട് പ്രവര്‍ത്തിക്കുന്ന ജനത (ഖൗമുന്‍ മുസ്‌രിഫൂന്‍), കുഴപ്പക്കാരായ ജനത (ഖൗമില്‍ മുഫ്‌സിദീന്‍) എന്നിങ്ങനെയാണ് ഈ വചനങ്ങളില്‍ സ്വവര്‍ഗഭോഗികളായ ഭൂമിയിലെ ആദ്യസമുദായത്തെ വിളിച്ചിരിക്കുന്നത്. അവര്‍ ചെയ്തുകൊണ്ടിരിക്കുന്ന സ്വവര്‍ഗരതി അവിവേകവും അതിക്രമവും അതിരുവിട്ട പ്രവര്‍ത്തനവും കുഴപ്പവു മാണെന്ന വസ്തുതയാണ് ക്വുര്‍ആന്‍ ഈ വചനങ്ങളിലൂടെ പഠിപ്പിക്കുന്നത്. ലൂത്ത് നബിയുടെ സമുദായം ചെയ്തുകൊണ്ടിരുന്ന ലൈംഗിക വൈകൃതത്തെപ്പറ്റി പരാമര്‍ശിക്കുമ്പോഴും ക്വുര്‍ആന്‍ ശക്തമായ പ്രയോഗങ്ങളാണ് നടത്തുന്നത്.

'നാം അവരുടെ മേല്‍ ഒരു തരം മഴ വര്‍ഷിപ്പിക്കുകയും ചെയ്തു. അപ്പോള്‍ ആ കുറ്റവാളികളുടെ പര്യവസാനം എങ്ങനെയായിരുന്നുവെന്ന് നോക്കുക.'(5)

'ലൂത്തിനെയും (ദൂതനായി അയച്ചു). തന്റെ ജനതയോട് അദ്ദേഹം ഇപ്രകാരം പറഞ്ഞ സന്ദര്‍ഭം (ശ്രദ്ധേയമാകുന്നു): തീര്‍ച്ചയായും നിങ്ങള്‍ നീചകൃത്യമാണ് ചെയ്തുകൊണ്ടിരിക്കുന്നത്. നിങ്ങള്‍ക്കു മുമ്പ് ലോകരില്‍ ഒരാളും അതുചെയ്യുകയുണ്ടായിട്ടില്ല.'(6)

'നിന്റെ ജീവിതം തന്നെയാണെ സത്യം. തീര്‍ച്ചയായും അവര്‍ അവരുടെ ലഹരിയില്‍ വിഹരിക്കുകയായിരുന്നു.'(7)

സ്വവര്‍ഗരതിയിലേര്‍പ്പെടുന്നവര്‍ കുറ്റവാളികളാണെന്നും (മുജ്‌രിമീന്‍) അവര്‍ ചെയ്തുകൊണ്ടിരിക്കുന്നത് ലോകത്ത് അതുവരെ ഒരാളും ചെയ്തിട്ടില്ലാത്ത അതീവ നികൃഷ്ടമായ കാര്യങ്ങളാണെന്നും (ഫാഹിശത്ത്) അതിലേര്‍പ്പെട്ടവര്‍ ഒരു തരം ലഹരിയിലാണെന്നും (സക്‌റത്ത്) ഈ വചനങ്ങള്‍ വ്യക്തമാക്കുന്നു. ഒരു അധര്‍മത്തെ വിളിക്കാവുന്ന പദങ്ങളെല്ലാം ക്വുര്‍ആന്‍ സ്വവര്‍ഗരതിക്കെതിരെ പ്രയോഗിച്ചിട്ടുണ്ടെ ന്നാണ് ഇവ നമ്മെ പഠിപ്പിക്കുന്നത്. പ്രകൃതിവിരുദ്ധവും മനുഷ്യത്വരഹിതവും കാമത്തില്‍ മാത്രം കേന്ദ്രീകൃതവുമായ സ്വവര്‍ഗരതിയെ ഒരുനിലക്കും അംഗീകരിക്കാനാവില്ലെന്ന നിലപാടാണ് ഇസ്‌ലാമിന്റേത് എന്നര്‍ഥം.

മുഹമ്മദ് നബി (സ) ഇക്കാര്യം ഊന്നിപറയുന്നുണ്ട്. സ്വവര്‍ഗരതിയുടെ നികൃഷ്ടത വ്യക്തമാക്കുന്ന നിരവധി ഹദീഥുകളുണ്ട്. ജാബിര്‍ (റ) നിവേദനം, നബി (സ) പറഞ്ഞു: 'ലൂത്തിന്റെ സമുദായം ചെയ്ത തിന്മയാണ് എന്റെ ജനതയില്‍ ഞാന്‍ ഏറ്റവും കൂടുതല്‍ ഭയപ്പെടുന്നത്.'(8)

ഇബ്‌നു അബ്ബാസ് (റ) നിവേദനം: നബി (സ) പറഞ്ഞു: 'ലൂത്തിന്റെ സമുദായം ചെയ്ത തിന്മ ചെയ്യുന്നവരെ അല്ലാഹു ശപിക്കട്ടെ; മൃഗങ്ങളെ കാമനിവൃത്തിക്കുവേണ്ടി ഉപയോഗിക്കുന്നവരെയും അല്ലാഹു ശപിക്കട്ടെ' (നബി ഇത് മൂന്നു പ്രാവശ്യം ആവര്‍ത്തിച്ചു).(9)

ഇബ്‌നു അബ്ബാസ് (റ) നിവേദനം: നബി (സ) പറഞ്ഞു: 'ലൂത്തിന്റെ ജനം ചെയ്ത തിന്മ ചെയ്യുന്നവരെ നിങ്ങള്‍ കണ്ടാല്‍ അത് ചെയ്തവരേ യും അതിന് ഉപയോഗിച്ചവരേയും നിങ്ങള്‍ കൊന്നുകളയുക.'(10)

സ്വവര്‍ഗരതിക്കെതിരെയുള്ള നിയമങ്ങള്‍ പഠിപ്പിക്കുക മാത്രമല്ല, അതില്ലാതാക്കുവാനുള്ള ധാര്‍മികനിര്‍ദേശങ്ങള്‍ കൂടി നല്‍കുന്നുണ്ട് ഇസ്‌ ലാം. ചെറുപ്പത്തിലുള്ള മനോവ്യതിയാനമാണ് ചിലരെ സ്വവര്‍ഗരതിക്കാരാക്കി തീര്‍ക്കുന്നതെന്ന് അത്തരക്കാരുടെ ജീവിതാനുഭവവിവര ണങ്ങളില്‍ നിന്ന് വ്യക്തമാവുന്നുണ്ട്. ഇത്തരം വ്യതിയാനങ്ങള്‍ക്കുള്ള നിമിത്തമായിത്തീരുന്നത് പലപ്പോഴും ഇതര ലിംഗത്തിലുള്ളവരുടെ വസ്ത്രം ധരിക്കുന്ന ശീലമാണ്. വെറുതെ ഒരു തമാശക്കുവേണ്ടി മാതാപിതാക്കളാല്‍ തുടക്കം കുറിക്കപ്പെടുന്ന ഈ ദുഃശ്ശീലം പലപ്പോഴും മാരകമായ മനോവ്യതിയാനമായിത്തീരാറുണ്ട്. എതിര്‍ലിംഗത്തിലുള്ളവരുടെ വസ്ത്രം ധരിക്കുമ്പോള്‍ സംതൃപ്തിക്ക് അടിമപ്പെടുന്നവ രില്‍ (ransvestism) പലരും സ്വവര്‍ഗാനുരാഗികളായിത്തീരാറുണ്ട്. സ്ത്രീപുരുഷന്മാര്‍ എതിര്‍ലിംഗത്തിലുള്ളവരുടെ വസ്ത്രം ധരിക്കുന്നത് ഇസ്‌ലാം ശക്തമായി വിലക്കുന്നുണ്ട്. ലൈംഗികവൈകൃതങ്ങളിലേക്ക് കാലെടുത്തുവെക്കുന്നതിന്റെ പ്രാഥമികപടി അടച്ചുകളയുകയാണ് ഇസ്‌ലാം ഇതുവഴി ചെയ്തിരിക്കുന്നത്. ഇബ്‌നു അബ്ബാസ് (റ) നിവേദനം: 'സ്ത്രീവേഷം ധരിക്കുന്ന പുരുഷന്മാരേയും പുരുഷവേഷം ധരി ക്കുന്ന സ്ത്രീകളേയും നബി(സ) ശപിക്കുകയും അത്തരക്കാരെ വീട്ടില്‍ നിന്ന് പുറത്താക്കുവാന്‍ കല്‍പ്പിക്കുകയും ചെയ്തു.'(11)

ലൈംഗിക അരാജകത്വമാണ് സ്വാതന്ത്രമെന്നു കരുതുന്ന ഉദാരലൈംഗികതയുടെ വക്താക്കളാണ് മതങ്ങളും രാഷ്ട്രമീമാംസകളെയെല്ലാം സഹ സ്രാബ്ദങ്ങളായി കുറ്റകരമായ പാപമായി കണ്ടിരുന്ന സ്വവർഗാനുരാഗം പ്രകൃതി പരവും ജനിതകവുമാണെന്ന് വരുത്താൻ വേണ്ടി ഈയ ടുത്തായി പരിശ്രമിച്ചുകൊണ്ടിരിക്കുന്നത്.

കുറിപ്പുകൾ

  1. ക്വുര്‍ആന്‍ 26:165,166.
  2. ക്വുര്‍ആന്‍ 27:55.
  3. ക്വുര്‍ആന്‍ 7:81.
  4. ക്വുര്‍ആന്‍ 29:30.
  5. ക്വുര്‍ആന്‍ 7:84.
  6. ക്വുര്‍ആന്‍ 29:28.
  7. ക്വുര്‍ആന്‍ 15:72.
  8. തിര്‍മിദി, ഇബ്‌നുമാജ.
  9. തിര്‍മിദി, ഇബ്‌നുമാജ.
  10. തിര്‍മിദി, അബൂദാവൂദ്, ഇബ്‌നുമാജ.
  11. ബുഖാരി.

ഭൂമിയിലെ ജീവജാലങ്ങൾക്കെല്ലാം ആവശ്യമായ വിഭവങ്ങൾ ഭൂമിയിൽ സൃഷ്ടാവ് ഒരുക്കിവെച്ചിട്ടുണ്ടെന്നാണ് ഇസ്‌ലാമിക വിശ്വാസം. ജനപ്പെരുപ്പം നിയന്ത്രിച്ചുകൊണ്ടല്ല മാനവവിഭവശേഷിയുടെ ശരിയായ ആസൂത്രണം വഴിയാണ് മനുഷ്യർ പുരോഗമിക്കുകയെന്ന് കരുതു കയും അതിന്നാവശ്യമായ നിയമങ്ങൾ നിര്ദേശിക്കുകയുമാണ് ഇസ്‌ലാം ചെയ്യുന്നത്.

ജനസംഖ്യാ വര്‍ധനവ് വഴി പട്ടിണിയും ദാരിദ്യവും ക്ഷാമവും രോഗങ്ങളും അകാലമരണങ്ങളുമാണ് സംഭവിക്കാന്‍ പോകുന്നതെന്നും ജന പ്പെരുപ്പം നിയന്ത്രിച്ചിട്ടില്ലെങ്കില്‍ പ്രകൃതി ക്രൂരമായി തിരിച്ചടിക്കുമെന്നും സ്ഥാപിച്ചുകൊണ്ട് തോമസ് റോബര്‍ട്ട് മാല്‍ത്തൂസ് എന്ന കത്തോ ലിക്കാ പാതിരി 1798ല്‍ എഴുതിയ എന്‍ എസ്സെ ഓണ്‍ ദി പ്രിന്‍സിപ്പിള്‍ ഓഫ് പോപ്പുലേഷന്‍ ആണ് ജനസംഖ്യാ വര്‍ധനവിനെക്കുറിച്ച് ഭീതി ജനിപ്പിച്ചു കൊണ്ട് എഴുതപ്പെട്ട ആദ്യ കൃതി. അതിനുശേഷം നീണ്ട രണ്ടു നൂറ്റാണ്ടുകാലം മാര്‍ത്തൂസിനെ അനുകൂലിച്ചും പ്രതികൂലിച്ചും കൊണ്ടുള്ള പഠനങ്ങളുണ്ടായി. പത്തൊമ്പതാം നൂറ്റാണ്ടിലെ പ്രമുഖ അമേരിക്കന്‍ സാമ്പത്തിക ശാസ്ത്രജ്ഞനായ ഹെന്‍ട്രി ചാള്‍സ്‌കാരെ (1793-1879) മാല്‍ത്തൂസിനെ വിമര്‍ശിച്ച ആദ്യകാല പ്രമുഖര്‍ക്ക് മുന്നില്‍ നടക്കുമ്പോള്‍ അനുകൂലിച്ചവര്‍ക്ക് നേതൃത്വം നല്‍കുന്നത് പത്തൊ മ്പതാം നൂറ്റാണ്ടുകാരന്‍ (1806-1873) തന്നെയായ പ്രമുഖ ഉപയോഗക്ഷമതാവാദിയും യൂട്ടിലിറ്റേറിയനിസം എന്ന കൃതിയുടെ കര്‍ത്താവ് ജോണ്‍ സ്റ്റുവാര്‍ട്ട് മില്‍ ആണ്. സംതൃപ്തി പ്രദാനം ചെയ്യുന്ന കര്‍മങ്ങളെല്ലാം ശരിയാണെന്നും അതുണ്ടാക്കാത്തവയെല്ലാം തെറ്റാണെന്നുമു ള്ള ഉപയോഗക്ഷമതാവാദത്തിന്റെ വക്താവ് മാര്‍ത്തൂസിന് അനുകൂലമായതും, വസ്തുനിഷ്ഠമായ തെളിവുകളുടെ വെളിച്ചത്തില്‍ വ്യക്തിയുടെയും സമൂഹത്തിന്റെയും സാമ്പത്തിക സുസ്ഥിതിയെക്കുറിച്ച് പഠിക്കുന്ന സാമ്പത്തിക ശാസ്ത്രവിദഗ്ദന്‍ അദ്ദേഹത്തിനെതി രായതും സ്വാഭാവികമാണ്. അനുഭവങ്ങളും തെളിവുകളുമെല്ലാം മാര്‍ത്തൂസിന് എതിരാണ്; തങ്ങളുടെ സന്തോഷവും സുഖവുമെല്ലാം നഷ്ട പ്പെട്ടുപോയെന്ന് ആധിയുള്ളവര്‍ പക്ഷെ മാല്‍ത്തൂസിനോടൊപ്പം കൂടും. അതാണ് മാല്‍ത്തൂസിന്റെ കാലം മുതല്‍ ഇന്നുവരെയുള്ള അവ സ്ഥ. മാല്‍ത്തൂസിന്റെ മനസ്സുമായി കേരളത്തെ വന്ധ്യംകരിക്കാന്‍ ഒരുങ്ങിപ്പുറപ്പെട്ടവര്‍ യൂട്ടിലിറ്റേറിയന്‍ മനസ്സുള്ളവരാണെന്ന് വ്യക്തം. മാല്‍ത്തൂസിയന്‍ സിദ്ധാന്തത്തെക്കുറിച്ച് മാര്‍ക്‌സിസ്റ്റ് സൈദ്ധാന്തികനായ ഫ്രെഡറിക് എംഗല്‍സ് പറഞ്ഞതേ നിയോ മാല്‍ത്തൂസിയന്‍മാ രോടും നമുക്ക് പറയാനുള്ളൂ. 'നിന്റെ അയല്‍ക്കാരനെ നിന്നെപ്പോലെ സ്‌നേഹിക്കുകയെന്നും ലോകപൗരത്വമെന്നുമെല്ലാമുള്ള അതിസുന്ദര മായ ആശയങ്ങളെ തകര്‍ക്കാന്‍ പര്യാപ്തമായ നിരാശാജനകമായ വ്യവസ്ഥയെ പ്രതിനിധാനം ചെയ്യുന്ന, ഇന്നുവരെ ഉണ്ടായിട്ടുള്ളതില്‍ വെച്ച് ഏറ്റവും പ്രാകൃതവും കാടത്തം നിറഞ്ഞതുമായ സിദ്ധാന്തം' എന്നാണ് മാല്‍ത്തൂസിയന്‍ സിദ്ധാന്തത്തെ എംഗല്‍സ് തന്റെ ഔട്ട്‌ലൈ ന്‍സ് ഓഫ് എ ക്രിട്ടിക്ക് ഓഫ് പൊളിറ്റിക്കല്‍ എക്കോണമി എന്ന പ്രബന്ധത്തില്‍ വിശേഷിപ്പിച്ചിരിക്കുന്നത്. 'അര്‍ഹതയുള്ളതിന്റെ അതി ജീവനം' എന്ന സിദ്ധാന്തത്തെ ന്യായീകരിക്കാനായി ചാള്‍സ് ഡാര്‍വിനും നിയോഡാര്‍വിനിസ്റ്റായ ആല്‍ഫ്രഡ് റസ്സല്‍ വാലസും കൂട്ടുപിടി ച്ചതും 'രക്തശുദ്ധിയും പാരമ്പര്യമഹിമയുമുള്ള ആര്യന്‍മാരുടെ ആധിപത്യം' എന്ന ആശയത്തെ സാധൂകരിക്കാനായി അഡോള്‍ഫ് ഹിറ്റ്‌ ലര്‍ ഉപയോഗിച്ചതും മാല്‍ത്തൂസിന്റെ സിദ്ധാന്തത്തെയായിരുന്നുവെന്ന വസ്തുത 'കാടന്‍ സിദ്ധാന്ത'മെന്ന് എംഗല്‍സിന്റെ വിശേഷണത്തെ ന്യായീകരിക്കുന്നുണ്ട്.

മാല്‍ത്തൂസിയന്‍ കാഴ്ചപ്പാടുകള്‍ അബദ്ധമാണെന്ന വസ്തുത മനസ്സിലാക്കാന്‍ അദ്ദേഹത്തിനു ശേഷമുള്ള ലോകക്രമത്തെക്കുറിച്ച് പഠിച്ചാ ല്‍ മാത്രംമതി. 1798ല്‍ മാല്‍ത്തൂസ് തന്റെ പുസ്തമെഴുതുമ്പോള്‍ അന്നത്തെ ലോകജനസംഖ്യ 90 കോടിയോളമായിരുന്നു. രണ്ടു നൂറ്റാണ്ടുകള്‍ കൊണ്ട് അത് എഴുനൂറ് കോടിയോളമായി. മാല്‍ത്തൂസിന്റെ വീക്ഷണങ്ങള്‍ ശരിയായിരുന്നെങ്കില്‍ ഇന്ന് ലോകം തന്നെ നിലനില്‍ക്കുമായി രുന്നില്ല. പട്ടിണി, ക്ഷാമം, തൊഴിലില്ലായ്മ, രോഗങ്ങള്‍ എന്നിവയില്‍ മാനവരാശി തകര്‍ന്നുപോകുമായിരുന്നു. മാല്‍ത്തൂസ് പുസ്തകമെഴു തിയ രണ്ടു നൂറ്റാണ്ടിനു മുമ്പുള്ളതിനേക്കാള്‍ എട്ടിരട്ടി മനുഷ്യരുണ്ട് ഇന്ന് ഭൂമിയില്‍. ഈ രണ്ടു നൂറ്റാണ്ടുകള്‍ കൊണ്ട് ക്ഷാമമാണോ ക്ഷേമ മാണോ മനുഷ്യരാശിക്കുണ്ടായത്? ഉത്തരം ക്ഷേമമെന്നു തന്നെയാണ്. ജനസംഖ്യാ വര്‍ധനവ് ക്ഷാമത്തിലേക്കല്ല, ക്ഷേമത്തിലേക്കാണ് മനു ഷ്യരെ നയിക്കുകയെന്ന വസ്തുതയ്ക്ക് വേറെ തെളിവുകളൊന്നും വേണ്ട. എന്തുകൊണ്ടാണിങ്ങനെ സംഭവിച്ചത്? മാല്‍ത്തൂസിന്റെ കണ ക്കുകള്‍ തെറ്റായതു കൊണ്ടാണോ? അല്ല. അദ്ദേഹത്തിന്റെ കണക്കുകള്‍ ശരിയായിരുന്നു; പക്ഷെ, പ്രസ്തുത കണക്കുകളും നിഗമനങ്ങളുമെ ല്ലാം അദ്ദേഹം ജീവിച്ചിരുന്ന കാലഘട്ടത്തിന്റെ സാഹചര്യത്തിന്റെയും സാങ്കേതികവിദ്യയുടെയും അടിസ്ഥാനത്തിലുള്ളവയായിരുന്നു. അതിനുശേഷം വമ്പിച്ച വൈജ്ഞാനികമുന്നേറ്റങ്ങളും സാങ്കേതിക വിപ്ലവങ്ങളുമുണ്ടായി. പ്രസ്തുത വിപ്ലവങ്ങളുടെ അനന്തരഫലമായി ജനങ്ങളുടെ ജീവിതനിലവാരം മെച്ചപ്പെട്ടു. ഈ മുന്നേറ്റങ്ങളൊന്നും യാദൃച്ഛികമായി ഉണ്ടായതല്ല. ജനസംഖ്യാ വര്‍ധനവിന്റെ ഫലമായാണ് ശാസ്ത്രപുരോഗതിയും സാങ്കേതികമുന്നേറ്റങ്ങളുമെല്ലാം ഉണ്ടാകുന്നത്. മനുഷ്യരുടെ എണ്ണം കൂടുന്നതിനനുസരിച്ച് അവര്‍ക്ക് ജീവിക്കുവാ നുള്ള വക കണ്ടെത്തുവാനുള്ള അറിവും വിദ്യയും പടച്ചവന്‍ പ്രദാനം ചെയ്യും. ദാരിദ്ര്യഭയത്താല്‍ കുഞ്ഞുങ്ങളെ കൊന്നുകളഞ്ഞിരുന്ന അജ്ഞാനകാലത്തെ അറബികളോടായി ക്വുര്‍ആന്‍ പറഞ്ഞത് തന്നെയാണ് വിജ്ഞാനഭാരത്തിന്റെ അഹങ്കാരത്താല്‍ വരും തലമുറയെ വെട്ടിമിനുക്കാന്‍ തത്രപ്പെടുന്ന ആധുനികരെയും നമുക്ക് തെര്യപ്പെടുത്താനുള്ളത്. ''ദാരിദ്ര്യഭയത്താല്‍ നിങ്ങള്‍ നിങ്ങളുടെ കുഞ്ഞുങ്ങളെ കൊന്നുകളയരുത്. നാമാണ് അവര്‍ക്കും നിങ്ങള്‍ക്കും ഉപജീവനം നല്‍കുന്നത്. അവരെ കൊല്ലുന്നത് തീര്‍ച്ചയായും ഭീമമായ അപരാധമാ കുന്നു.''(17:31)

''തീര്‍ച്ചയായും അല്ലാഹു തന്നെയാണ് ഉപജീവനം നല്‍കുന്നവനും ശക്തനും പ്രബലനും.'' (51:58)

മാല്‍ത്തൂസിന്റെ പാത പിന്‍തുടര്‍ന്നു കൊണ്ട് 1968ല്‍ അമേരിക്കന്‍ ജീവശാസ്ത്രജ്ഞനായ പോള്‍ ആര്‍ എല്‍റിച്ചും ഭാര്യ ആന്‍ എച്ച് എല്‍ റിച്ചും കൂടി എഴുതിയ പോപ്പുലേഷന്‍ ബോംബ് എന്ന ഗ്രന്ഥം. കണക്കുകള്‍ ഉദ്ധരിച്ചു കൊണ്ട് നടത്തിയ പ്രവചനങ്ങളെല്ലാം മിഥ്യയായിരു ന്നുവെന്ന് കാലം തെളിയിക്കുകയുണ്ടായി. 1985 ആകുമ്പോഴേക്ക് ലോകമാകെ ഭക്ഷ്യക്ഷാമമുണ്ടാകുമെന്നും സമുദ്രങ്ങള്‍ ഇല്ലാതാകുമെന്നും പാശ്ചാത്യരാജ്യങ്ങളില്‍ പലതും മരുഭൂമിയായിത്തീരുമെന്നും ശരാശരി ആയുര്‍ദൈര്‍ഘ്യം 42 ആയി കുറയുമെന്നുമെല്ലാമായിരുന്നു പ്രവച നങ്ങള്‍! ഇതു നടത്തിയ 1968നേക്കാള്‍ 1985 ആയപ്പോഴേക്ക് മനുഷ്യരുടെ ക്ഷേമാവസ്ഥയില്‍ മെച്ചമുണ്ടാവുക മാത്രമാണുണ്ടായത്. എല്‍റിച്ചി ന്റെ പ്രവചനങ്ങളെപ്പോലെത്തന്നെയാണ് കേരളീയസമൂഹത്തിലെ ജനസംഖ്യാ വളര്‍ച്ചയെക്കുറിച്ച് കണക്കുകള്‍ നിരത്തി മലയാളികളെ ഭയപ്പെടുത്തുന്ന നിയമജ്ഞരുടെ രേഖകളുടെ സ്ഥിതിയുമെന്നതാണ് വാസ്തവം. അവരുടെ ഉപദേശം സ്വീകരിച്ച് കേരളീയ സമൂഹത്തില്‍ ജനസംഖ്യ കുറയ്ക്കാനുള്ള നടപടികളുമായാണ് സര്‍ക്കാര്‍ മുന്നോട്ടുപോകുന്നതെങ്കില്‍ നമ്മുടെ പ്രധാനപ്പെട്ട സമ്പത്തായ മാനവവിഭവ ശേഷിയുടെ കടയ്ക്കായിരിക്കും അവര്‍ കത്തിവെക്കുന്നത്. തൊഴിലില്ലായ്മക്കും അതുമൂലമുണ്ടാകുന്ന ക്ഷാമത്തിനും സാമ്പത്തിക പ്രതിസ ന്ധിക്കുമായിരിക്കും അത്തരം നടപടികള്‍ നിമിത്തമാവുക.

ജനസംഖ്യാ വര്‍ധനവ് ഒരു വലിയ പ്രശ്‌നമാണെന്ന് വാദിക്കുന്നവരുടെ ന്യായങ്ങളും അവയ്ക്കുള്ള പ്രതികരണങ്ങളുമാണ് താഴെ.

ഒന്ന്: ജനസംഖ്യാ വര്‍ധനവ് മൂലം ഭക്ഷ്യക്ഷാമമുണ്ടാവും

കണക്കുകള്‍ ഈ ന്യായം ശരിയാണെന്ന് തോന്നിപ്പിക്കുമെങ്കിലും വസ്തുത മറിച്ചാണ്. 1998ല്‍ പ്രസിദ്ധീകരിക്കപ്പെട്ട യു.എന്‍. വേള്‍ഡ് പോപ്പുലേഷന്‍ പ്രോസ്‌പെക്ടസ് നോക്കുക. 1830ലെ ജനസംഖ്യ 100 കോടിയും 1930ലേത് 200 കോടിയും 1960ലേത് 300 കോടിയും 1975ലേത് 400 കോടിയും 1987ലേത് 500 കോടിയും 1999ലേത് 600 കോടിയുമാണെന്ന് പ്രസ്തുത രേഖ വ്യക്തമാക്കുന്നു. അതേ വര്‍ഷത്തെ യു.എന്‍.ഡി. പി. ഹ്യൂമണ്‍ ഡവലപ്‌മെന്റ് റിപ്പോര്‍ട്ട് പരിശോധിക്കുക. 1950ല്‍ 252 കോടി ജനസംഖ്യയുണ്ടായിരുന്ന സമയത്ത് ഭൂമിയിലെ ഭക്ഷ്യധാന്യ ഉല്‍പാദനം 62.4 കോടി ടണ്‍ ആയിരുന്നുവെന്നും 1990ല്‍ 520 കോടിയായി ജനസംഖ്യ വര്‍ധിച്ചപ്പോള്‍ ഉല്‍പാദനം 180 കോടിയായിത്തീരുന്നു വെന്നും പ്രസ്തുത രേഖ വ്യക്തമാക്കുന്നു. ജനസംഖ്യ ഇരട്ടിയായപ്പോള്‍ ഭക്ഷ്യഉല്‍പാദനം മൂന്നിരട്ടിയായിത്തീര്‍ന്നുവെന്ന് സാരം. എല്ലാവ ര്‍ക്കും ആവശ്യമായ ഭക്ഷ്യധാന്യങ്ങള്‍ വിതരണം ചെയ്യാനുള്ള സംവിധാനമുണ്ടായിരുന്നുവെങ്കില്‍ 1950ല്‍ ഓരോരുത്തര്‍ക്കും ലഭിച്ച ധാന്യ ങ്ങളേക്കാള്‍ ഒന്നരയിരട്ടി ലഭിക്കുമായിരുന്നു ഇരട്ടി ജനസംഖ്യയായിത്തീര്‍ന്ന 1990ല്‍ എന്നാണിത് വ്യക്തമാക്കുന്നത്. ജനസംഖ്യാവര്‍ധനവു മൂലം ഭക്ഷ്യക്ഷാമമുണ്ടാകുമെന്ന് കണക്കാക്കുന്നവര്‍ മാനവവിഭവശേഷിയുടെ അനന്തമായ സാധ്യതകള്‍ വേണ്ട രൂപത്തില്‍ പരിഗണിക്കാത്തതു കൊണ്ടാണ് അവര്‍ക്ക് തെറ്റുപറ്റുന്നത്. ''നാമാണ് അവര്‍ക്കും നിങ്ങള്‍ക്കും ഉപജീവനം നല്‍കുന്നത്''(17:31) എന്ന ദൈവികവാഗ്ദാനം തന്നെയാണ് ശരി!

രണ്ട്: ജനസംഖ്യാ വര്‍ധനവുണ്ടാവുമ്പോള്‍ രോഗങ്ങള്‍ വര്‍ധിക്കുകയും ആയുര്‍ദൈര്‍ഘ്യം കുറയുകയും ചെയ്യും

ഇന്‍സ്റ്റിറ്റ്യൂട്ട് ഓഫ് അപ്ലൈഡ് മാന്‍പവര്‍ റിസര്‍ച്ച് ഹ്യൂമണ്‍ റിസോഴ്‌സസ് പ്രസിദ്ധീകരിച്ച ഇന്ത്യയിലെ ജനസംഖ്യാ വളര്‍ച്ചയെയും ആയു ര്‍ദൈര്‍ഘ്യത്തെയും കുറിച്ച കണക്കുകള്‍ പരിശോധിക്കുക. 1901ല്‍ ജനസംഖ്യ 23.8 കോടിയായിരുന്നപ്പോള്‍ ഇന്ത്യക്കാരുടെ ശരാശരി ആയു ര്‍ദൈര്‍ഘ്യം 24 വയസ്സായിരുന്നുവെങ്കില്‍ 1941ല്‍ ജനസംഖ്യ 31.9 കോടിയായപ്പോള്‍ ആയുര്‍ദൈര്‍ഘ്യം 31 വയസ്സും 1981ല്‍ 68.3 കോടിയായ പ്പോള്‍ 55 വയസ്സും 2004ല്‍ 102 കോടിയായപ്പോള്‍ 62 വയസ്സുമായിത്തീര്‍ന്നുവെന്നാണ് പ്രസ്തുത രേഖ വ്യക്തമാക്കുന്നത്. ജനസംഖ്യ വര്‍ധി ക്കുന്നതിനനുസരിച്ച് ജനങ്ങളുടെ ജീവിതനിലവാരം വര്‍ധിക്കുകയും അതുവഴി ആരോഗ്യം കൂടുകയും രോഗങ്ങള്‍ കുറയുകയും രോഗങ്ങ ള്‍ കാരണമുള്ള കുട്ടികളുടെയും യുവാക്കളുടെയും മരണനിരക്ക് കുറയുകയും ചെയ്യുന്നതുകൊണ്ടാണല്ലോ ശരാശരി ആയുര്‍ദൈര്‍ഘ്യം വര്‍ധിക്കുന്നത്. ജനസംഖ്യാവര്‍ധനവു വഴി രോഗങ്ങള്‍ കുറയുകയും ആയുര്‍ദൈര്‍ഘ്യം വര്‍ധിക്കുകയുമാണ് ചെയ്യുകയെന്നര്‍ഥം.

മൂന്ന്: ജനസംഖ്യാവര്‍ധനവുവഴി ജനസാന്ദ്രത വര്‍ധിക്കുകയും അതുവഴി പ്രതിശീര്‍ഷ വരുമാനം കുറയുകയും ചെയ്യും

ജനസംഖ്യാവര്‍ധനവുവഴി ജനസാന്ദ്രത വര്‍ധിക്കുമെന്നത് ശരിയാണ്. എന്നാല്‍ ജനസാന്ദ്രത വര്‍ധിക്കുമ്പോള്‍ പ്രതിശീര്‍ഷ വരുമാനം കുറയു കയല്ല കൂടുകയാണ് ചെയ്യുക. പോപ്പുലേഷന്‍ റഫറന്‍സ് ബ്യൂറോ’പ്രസിദ്ധീകരിച്ച 2002ലെ ലോക ജനസംഖ്യയെക്കുറിച്ച വിവരപ്പട്ടിക പരി ശോധിക്കുക. ജനസംഖ്യയും ജനസാന്ദ്രതയും കുറഞ്ഞ രാജ്യങ്ങളാണ് കോംഗോ, സോമാലിയ, മാലി, നൈജര്‍, ഡാബിയ എന്നിവ. 2002 ലെ കണക്കുകള്‍ പ്രകാരം ഇത് യഥാക്രമം 9, 12,9, 9, 13 എന്നിങ്ങനെയാണ്. അവിടുത്തെ പ്രതിശീര്‍ഷ വരുമാനമാകട്ടെ യഥാക്രമം 570, 600, 780, 740, 750 ഡോളറുകളാണ്. ജനസാന്ദ്രത ഏറ്റവുമധികമുള്ള മകാഒ, മൊണാകോ, സിംഗപ്പൂര്‍, ഹോംഗ്‌കോങ്ങ് എന്നിവയിലെ പ്രതിശീര്‍ഷ വരുമാനം വളരെ കൂടുതലാണ്. ഉദാഹരണത്തിന് ജനസാന്ദ്രത 6815 രേഖപ്പെടുത്തിയ സിംഗപ്പൂരിന്റെ പ്രതിശീര്‍ഷ വരുമാനം 24910 ഡോള റുകളാണ്. ജനസംഖ്യാവര്‍ധനവിനനുസരിച്ച് ഉല്‍പാദനക്ഷമത വര്‍ധിക്കുകയും അതുവഴി പ്രതിശീര്‍ഷവരുമാനം വര്‍ധിക്കുകയും ചെയ്യു മെന്ന വസ്തുതയാണ് ഇത് വ്യക്തമാക്കുന്നത്. 2002ലെ കണക്കുകള്‍ പ്രകാരം ജനസാന്ദ്രത 25806 ആയ മകാഒയില്‍ ഒരാള്‍ക്ക് ശരാശരി ഒരു സെന്റ് ഭൂമിയാണ് ലഭിക്കുന്നതെങ്കിലും ഒരു ചതുരശ്ര കിലോമീറ്ററില്‍ നിന്ന് ശരാശരി 46,94,19,355 ഡോളര്‍ വരുമാനം ലഭിക്കും. എന്നാല്‍ ജനസാന്ദ്രത രണ്ടായ മംഗോളിയയില്‍ ഒരാള്‍ക്ക് ശരാശരി 123.5 ഏക്കര്‍ ഭൂമി ലഭിക്കുന്നുണ്ടെങ്കിലും ഒരു ചതുരശ്ര കിലോമീറ്റലിലെ വരുമാ നം 2699 ഡോളര്‍ മാത്രമാണ്. ജനസാന്ദ്രത വര്‍ധിക്കുന്നതിനനുസരിച്ച് ഉല്‍പാദനക്ഷമതയും അതുവഴി പ്രതിശീര്‍ഷ വരുമാനവും വര്‍ധിക്കു മെന്നു തന്നെയാണ് ഇതിന്നര്‍ഥം. ജനസംഖ്യ കുറയ്ക്കുവാനല്ല വര്‍ധിപ്പിക്കുവാനാണ് ക്ഷേമരാഷ്ട്രം സൃഷ്ടിക്കുവാന്‍ ശ്രമിക്കുന്നവര്‍ പരിശ്ര മിക്കേണ്ടതെന്ന് വ്യക്തമാക്കുന്നതാണ് ഈ കണക്കുകള്‍.

നാല്: ജനസംഖ്യാവര്‍ധനവ് വഴി തൊഴിലില്ലായ്മ വര്‍ധിക്കും

2011ലെ കണക്കുകള്‍ പ്രകാരം ഏറ്റവും കൂടുതല്‍ ജനസാന്ദ്രതയുള്ള നാട് വലുപ്പത്തില്‍ വത്തിക്കാനിന് നേരെ മുകളില്‍ നില്‍ക്കുന്ന മൊണാ കോയാണെന്ന് ആ രാജ്യത്തിന്റെ ഔദ്യോഗിക വെബ്‌സൈറ്റ് വ്യക്തമാക്കുന്നുണ്ട്. 1.98 ചതുരശ്ര കിലോമീറ്റര്‍ വിസ്തീര്‍ണമുള്ള മൊണാ കോയിലെ ഇന്നത്തെ ആകെ ജനസംഖ്യ 35,986 ആണ്. പ്രതിശീര്‍ഷ ജി.ഡി.പി 1,51,630 ഡോളറും ആയുര്‍ദൈര്‍ഘ്യം 90 വയസ്സുമായ അവിടെ തൊഴിലില്ലായ്മയെന്ന പ്രശ്‌നം തന്നെയില്ല. ജനസാന്ദ്രതയില്‍ മൊണാകോക്ക് തൊട്ടുപിന്നില്‍ നില്‍ക്കുന്ന രാജ്യമാണ് സിംഗപ്പൂര്‍. 2011 ജൂണി ലെ കണക്കുകള്‍ പ്രകാരം 51.8 ലക്ഷം പേരാണ് 704 ചതുരശ്ര കിലോമീറ്റര്‍ വിസ്തീര്‍ണമുള്ള സിംഗപ്പൂരിലുള്ളത്. അവിടെയും തൊഴിലില്ലാ യ്മ വളരെ കുറവാണ്. ചൈനക്ക് കീഴിലുള്ള പ്രത്യേക ഭരണപ്രദേശമായ മകാഒയാണ് ജനസാന്ദ്രത ഏറ്റവുമധികമുള്ള പട്ടണം. പുതിയ കണക്കുകള്‍ പ്രകാരം അവിടെ ഒരു ചതുരശ്ര കിേലാമീറ്റര്‍ 18428 പേര്‍ താമസിക്കുന്നുണ്ട്. അവിടെയും തൊഴിലില്ലായ്മ തീരെയില്ലെന്നാണ് ഔദ്യോഗികരേഖകള്‍ വ്യക്തമാക്കുന്നത്.

സമ്പത്തുണ്ടാക്കുന്നത് മാനവവിഭവശേഷിയാണെന്നും ജനങ്ങളുടെ എണ്ണം കൂടുമ്പോഴാണ് പുരോഗതിയും ക്ഷേമവുമുണ്ടാകുന്നത് എന്നുമു ള്ള വസ്തുതകളാണ് നാം കണ്ടത്. ഇതു തിരിച്ചറിയുന്നവരാണ് ഒന്നാം ലോകത്തിലെ സാമ്പത്തികവിദഗ്ദരും രാഷ്ട്രനായകരുമെല്ലാം. ജനസം ഖ്യാ വര്‍ധനവ് ത്വരിതപ്പെടുത്തുവാന്‍ 5300 കോടി ഡോളര്‍ ചെലവഴിച്ചതായും ഇനിയും കൂടുതല്‍ ശക്തമായി ആ നയവുമായി മുന്നോട്ട്‌ പോകുമെന്നും റഷ്യന്‍ പ്രസിഡണ്ട് വ്‌ളാദ്മിര്‍ പുടിന്‍ പ്രഖ്യാപിച്ചത് ഇത്രയടുത്താണല്ലോ. അമേരിക്കയിലെ 'നാഷണല്‍ ഫാദര്‍ ഹുഡ് ഇനീഷ്യേറ്റീവ്' നല്‍കുന്ന 'ഫാദര്‍ ഹുഡ് അവാര്‍ഡ്' കൂടുതല്‍ കുട്ടികളുണ്ടാവുന്നതിനെ പ്രോല്‍സാഹിപ്പിക്കുക കൂടി ലക്ഷ്യമാക്കിക്കൊണ്ടു ള്ളതാണ്. ആരും തിരിഞ്ഞു നോക്കാനില്ലായിരുന്ന സിംഗപ്പൂര്‍ വളര്‍ന്നു വികസിച്ചതു തന്നെ അതിലെ ജനസംഖ്യ വര്‍ധിച്ചപ്പോഴായിരുന്നു. മൂന്നാം ലോകരാജ്യങ്ങളില്‍ ജനസംഖ്യാനിയന്ത്രണത്തിനു വേണ്ടി ന്യായങ്ങള്‍ നിരത്തുന്നവര്‍ ഒന്നാംകിട രാജ്യങ്ങളിലെ ജനസംഖ്യ വര്‍ധിപ്പി ക്കുവാനുള്ള ശ്രമത്തെക്കുറിച്ച് നിശബ്ദമാവുന്നതെന്തു കൊണ്ടാണ്? പിന്നാക്ക രാജ്യങ്ങളിലെ തങ്ങളുടെ അധിശത്വം തുടരുന്നതിനുവേണ്ടി യുള്ള തന്ത്രങ്ങളിലൊന്നാണ് സാമ്രാജ്യത്വത്തിന്റെ ജനപ്പെരുപ്പ പ്രചരണമെന്നാണ് മനസ്സിലാവുന്നത്. മാനവവിഭവശേഷിയില്‍ മുന്നില്‍ നില്‍ ക്കുന്നവരും ഉല്‍പാദനക്ഷമമായ ഭൂമി കൈവശമുള്ളവരുമായ മൂന്നാം ലോകരാഷ്ട്രങ്ങള്‍ ജനസംഖ്യാവര്‍ധനവു മൂലം ശാസ്ത്ര സാങ്കേതിക രംഗങ്ങളില്‍ കൂടി വളര്‍ന്നു മുന്നേറിയാല്‍ സാമ്രാജ്യത്വത്തിന്റെ നുകക്കീഴില്‍ നിന്ന് അവര്‍ സ്വതന്ത്രരാവുമോയെന്നു ആശങ്കയായിരിക്കാം ജനസംഖ്യാ വിസ്‌ഫോടനത്തെക്കുറിച്ച് മിഥ്യാഭീതി സൃഷ്ടിക്കുന്നതിന് പിന്നില്‍ പ്രവര്‍ത്തിക്കുന്നത്.

മാനവവിഭവശേഷിയെ സൃഷ്ടിപരമായി നോക്കിക്കാണുകയും അതിന്റെ ആസൂത്രണത്തിലൂടെ ജനോപകാരപ്രദമായ പദ്ധതികള്‍ ആവി ഷ്‌കരിക്കുകയും ചെയ്യാനാണ് സര്‍ക്കാരുകള്‍ പരിശ്രമിക്കേണ്ടത്. ജനകീയാസൂത്രണം അന്വര്‍ഥമാവുക അത്തരം പരിശ്രമങ്ങളുണ്ടാകു മ്പോഴാണ്. സര്‍ക്കാരുകളുടെ പ്രാഥമിക ധര്‍മമാണത്. അത് വിസ്മരിച്ചുകൊണ്ട് ജനസംഖ്യാവര്‍ധനയാണ് നാം നേരിടുന്ന പ്രധാന പ്രശ്‌നമെ ന്ന് വരുത്തിത്തീര്‍ക്കാന്‍ ശ്രമിക്കുന്നത് ക്ഷേമരാഷ്ട്രത്തിന്റെ നിര്‍മിതിക്ക് ഉതകുകയില്ലെന്നുറപ്പാണ്; വ്യക്തികളുടെ എണ്ണക്കൂടുതല്‍ സമൂഹ ത്തിന് നന്മ മാത്രമെയുണ്ടാക്കൂവെന്ന് തിരിച്ചറിയാന്‍ കഴിയണമെങ്കില്‍ സുഖാസ്വാദനങ്ങള്‍ മാത്രമാണ് ജീവിധധര്‍മമെന്ന കാഴ്ചപ്പാടില്‍ നിന്ന് സ്വതന്ത്രരാവാന്‍ കഴിയണം. മനുഷ്യരെ, അവര്‍ തന്റെ വിഭവത്തില്‍ പങ്കുപറ്റുമെന്നതിനാല്‍ ശത്രുക്കളായിക്കാണുന്ന ജീവിതവീക്ഷണ ത്തിന് സമൂഹത്തിന് സ്വസ്ഥത നല്‍കുന്ന ജീവിതക്രമം പ്രദാനം ചെയ്യാനാവില്ല. ജനസംഖ്യാ വിസ്‌ഫോടനത്തെപ്പറ്റി വേവലാതിപ്പെടുന്നവര്‍ തന്നെയാണ് കന്നുകാലികളുടെയും ഇറച്ചിക്കോഴികളുടെയും പ്രത്യുല്‍പാദനശേഷി വര്‍ധിപ്പിക്കുന്നതിനായുള്ള ജനിതകവിദ്യകളെപ്പറ്റി വാചാലരാവുന്നത്. ആടിനും പശുവിനും കോഴിക്കും നല്‍കാവുന്നതിലേറെ നമ്മുടെ ഭൂമിക്കും ആവാസവ്യവസ്ഥക്കും ആവശ്യമായ കാര്യ ങ്ങള്‍ നല്‍കാന്‍ കഴിയുന്നവനാണ് മനുഷ്യന്‍. അവനാണ് സമ്പത്തിന്റെ ഉല്‍പാദകന്‍. അവനെ കേവലമൊരു ഉപഭോഗി മാത്രമായി കാണു ന്നതുകൊണ്ടാണ് മനുഷ്യരുടെ എണ്ണം കുറക്കുന്നതു വഴിയാണ് പുരോഗതിയുണ്ടാവുകയെന്ന മിഥ്യാധാരണയുണ്ടാവുന്നത്. പരമാവധി പേര്‍ ജീവിച്ചിരിക്കുന്നത് ഭൂമിക്ക് ഗുണം മാത്രമെ ചെയ്യൂ. അതുകൊണ്ടാണ് ജനങ്ങളുടെ എണ്ണപെരുപ്പത്തെക്കുറിച്ച് ഭയപ്പെടേണ്ടതില്ലെന്ന് വിശുദ്ധക്വുര്‍ആനും കൂടുതല്‍ പ്രസവിക്കുന്നവളെ ഇണയായി സ്വീകരിക്കുവാന്‍ മുഹമ്മദ് നബി(സ)യും നിര്‍ദേശിച്ചത്. ദാരിദ്രത്തിന്റെ ഇല്ലാത്ത കണക്കുകള്‍ നിരത്തി സന്താനനിയന്ത്രണത്തിന്റെ യുക്തിയെപ്പറ്റി വാചാലരാവുന്നവരോട് ക്വുര്‍ആന്‍ പറഞ്ഞതുതന്നെയാണ് നമുക്ക് ആവര്‍ത്തിക്കുവാനുള്ളത്. ''പിശാച് ദാരിദ്ര്യത്തെപ്പറ്റി നിങ്ങളെ പേടിപ്പെടുത്തുകയും, നീചവൃത്തികള്‍ക്ക് നിങ്ങളെ പ്രേരിപ്പിക്കു കയും ചെയ്യുന്നു. അല്ലാഹുവാകട്ടെ അവന്റെ പക്കല്‍ നിന്നുള്ള മാപ്പും അനുഗ്രഹവും നിങ്ങള്‍ക്ക് വാഗ്ദാനം ചെയ്യുന്നു. അല്ലാഹു വിപു ലമായ കഴിവുകളുള്ളവനും (എല്ലാം) അറിയുന്നവനുമാകുന്നു.'' (2:268)

മുമ്പ് വേദക്കാരികളായിരുന്ന രണ്ട് സ്ത്രീകള്‍ പ്രവാചകന്റെ (സ)ജീവിതപങ്കാളികളായിരുന്നുവെ ന്നത് ശരിയാണ്. യഹൂദനായ ഹുയയ്യുബ്‌നു അക്തബിന്റെ മകള്‍ സഫിയ്യയാണ് ഒന്ന്. ഈജിപ്തിലെ കിബ്ത്തി നേതാവ് സമ്മാനിച്ച മാരിയത്തുല്‍ കിബ്ത്തിയ്യയെ ന്ന ക്രൈസ്തവ വനിതയാണ് മറ്റൊന്ന്. ഇവര്‍ രണ്ടുപേരും പ്രവാചക ജീവിതത്തിലേക്ക് കടന്നുവന്നത് മദീനാ കാലഘട്ടത്തിലാണ്. ബനൂനളീ ര്‍ ഗോത്ര ഉപരോധവുമായി ബന്ധപ്പെട്ട് ഗോത്രത്തലവനായ ഹുയയ്യും സഫിയ്യയുടെ ഭര്‍ത്താവും കൊല്ലപ്പെട്ടതിനെ തുടര്‍ന്നാണ് ഹിജ്‌റ ഏഴാം വര്‍ഷത്തില്‍ അവരെ മുഹമ്മദ് നബി (സ)വിവാഹം ചെയ്യുന്നത്. മാരിയത്തുല്‍ കിബ്ത്തിയ്യയും പ്രവാചക ജീവിതത്തിലേക്ക് കടന്നുവരുന്നത് ഹിജ്‌റ ഏ ഴാം വര്‍ഷത്തില്‍ തന്നെയാണ്. പൂര്‍വ്വ പ്രവാചകന്മാരുടെ ചരിത്രം വിവരിക്കുന്ന ഖുര്‍ആന്‍ സൂ ക്തങ്ങളില്‍ ബഹുഭൂരിഭാഗവും അവതരിപ്പിക്കപ്പെട്ടത് മക്കയിലാണെന്നിരിക്കെ മദീനാ ജീവിതത്തി ന്റെ ഏഴു വര്‍ഷങ്ങള്‍ക്കുശേഷം പ്രവാചക ജീവിതത്തിലേക്ക് കടന്നുവന്ന രണ്ട് വനിതകളെങ്ങനെ യാണ് പ്രസ്തുത ചരിത്രങ്ങളുടെ സ്രോതസ്സായിത്തീരുക?

കോപ്റ്റിക് ക്രൈസ്തവര്‍ക്കിടയില്‍ പ്രചാരത്തിലുണ്ടായിരുന്ന ശൈശവ സുവിശേഷ (Gospel of the Infancy) ത്തിലുള്ള കഥകളാണ് തൊട്ടിലില്‍വെച്ച് ഉണ്ണിയേശു സംസാരിച്ചതായും പ്രസവവേദനയുടെ സമയത്ത് ഈത്തപ്പന കുലുക്കി പഴം ലഭിച്ചതായുമുള്ള കഥകളെല്ലാമെന്നും ഇവ കോപ്റ്റിക് ക്രി സ്ത്യാനിയായിരുന്ന മാരിയത്തുല്‍ കിബ്ത്തിയ്യ പറഞ്ഞുകൊടുത്തതാണെന്നുമാണ് മറ്റൊരു വാദം. ഈ വാദവും അടിസ്ഥാന രഹിതമാണ്. തൊട്ടിലില്‍വെച്ച് യേശു സംസാരിച്ചതായി സൂചിപ്പിക്കുന്ന വാക്യമുള്‍ക്കൊള്ളുന്ന ഖുര്‍ആനിലെ പത്തൊന്‍പതാം അധ്യായം സൂറത്തുമര്‍യം മക്കയില്‍വെച്ച് അവതരിപ്പിക്കപ്പെട്ടതാണ്. ഹിജ്‌റ ഏഴാം നൂറ്റാണ്ടില്‍ മാത്രം പ്രവാചക ജീവിതത്തിലേക്ക് കടന്നു വന്ന മാരിയത്തുല്‍ കിബ്ത്തിയ്യ പറഞ്ഞുകൊടുത്ത കഥയുടെ അടിസ്ഥാനത്തില്‍ മക്കയില്‍ വെച്ച് എങ്ങനെയാണ് മുഹമ്മദ് നബി (സ)ഈ സംഭവങ്ങളെഴൂതുക? മുഹമ്മദ് നബി (സ)യുടെ കാലത്ത് കോപ്റ്റിക് ക്രൈസ്തവര്‍ക്കിടയില്‍ ശൈശവ സുവിശേഷം പ്രചാരത്തിലിരുന്നുവെന്ന് ഖണ്ഡിതമാ യി തെളിയിക്കാന്‍ ഈ വിമര്‍ശനമുന്നയിച്ചവര്‍ക്കൊന്നും കഴിഞ്ഞിട്ടില്ല. ഇനി കഴിഞ്ഞാല്‍തന്നെ യേശുവിന്റെ ശൈശവകാല സംഭവങ്ങളെക്കുറിച്ച ഖുര്‍ആനിക പരാമര്‍ശങ്ങള്‍ ശൈശവ സുവിശേ ഷത്തിന്റെ അടിസ്ഥാനത്തില്‍ എഴുതിയതാണെന്ന് വസ്തുനിഷ്ഠമായി സ്ഥാപിക്കുവാന്‍ ആര്‍ക്കുംതന്നെ സാധിക്കുകയില്ല.

പ്രഗല്‍ഭനായ ഒരു പ്രവാചക ശിഷ്യനായിരുന്നു സല്‍മാനുല്‍ ഫാരിസി (റ). മദീനയ്ക്ക് ചുറ്റും കിടങ്ങ് കുഴിച്ചുകൊണ്ട് മക്കക്കാരുടെ ആക്രമണത്തെ പ്രതിരോധിക്കാമെന്ന അദ്ദേഹത്തിന്റെ നിര്‍ദ്ദേശമാണ് ഖന്‍ദഖ് യുദ്ധത്തില്‍ മുസ്‌ലിംകളുടെ വിജയത്തിന് നിമിത്തമായ പല കാരണങ്ങളിലൊന്ന്. സല്‍മാനു ല്‍ ഫാരിസിയെക്കുറിച്ച് പറയുമ്പോള്‍ ഖന്‍ദഖ് യുദ്ധമാണ് ഇസ്‌ലാമിക ചരിത്രം പഠിച്ചവരുടെ മന സ്സില്‍ ആദ്യമായി ഓടിയെത്തുക.

അഗ്‌നി ആരാധനയിലധിഷ്ഠിതമായ സരതുഷ്ട്രമതത്തിലായിരുന്ന സല്‍മാന്‍ പിന്നീട് ക്രിസ്തുമതം സ്വീകരിച്ചു. സത്യാന്വേഷിയായിരുന്ന അദ്ദേഹത്തിന് ക്രിസ്തുമതത്തിന്റെ ആശയങ്ങള്‍ പൂര്‍ണ     സം തൃപ്തി നല്‍കാത്തതുകൊണ്ട് തന്റെ അന്വേഷണം തുടരുകയും അവസാനം ഇസ്‌ലാമിലെത്തി ച്ചേരുകയും ചെയ്തു. സല്‍മാനുല്‍ ഫാരിസി (റ) ഇസ്‌ലാം സ്വീകരിച്ചത് മദീനയില്‍വെച്ചാണ്. അതി നുശേഷമാണ് അദ്ദേഹം പ്രവാചകന്റെ (സ) സഹചാരിയായിത്തീര്‍ന്നത്.

ഖുര്‍ആനിന്റെ ഏകദേശം മൂന്നില്‍ രണ്ടുഭാഗവും അവതരിപ്പിക്കപ്പെട്ടത് മക്കയില്‍വെച്ചാണ്. പൂര്‍വ്വ പ്രവാചകന്മാരെക്കുറിച്ച പരാമര്‍ങ്ങളധികവും മക്കയില്‍ അവതരിപ്പിക്കപ്പെട്ട സൂക്തങ്ങളിലാണുള്ളത്. മദീനയില്‍ വെച്ച് പ്രവാചകന്റെ അനുചരനായിത്തീര്‍ന്ന സല്‍മാനുല്‍ ഫാരിസി പറഞ്ഞുകൊടുത്ത വിവരങ്ങളുടെ അടിസ്ഥാനത്തില്‍ എങ്ങനെയാണ് മക്കയില്‍വെച്ച് മുഹമ്മദ് നബി (സ) പൂര്‍വ്വ പ്രവാചകന്മാരുടെ ചരിത്രമെഴുതുക?

ഖുര്‍ആനിന് സമാന്തരമായ ഒരു രചനയുണ്ടാക്കുവാനുള്ള അതിന്റെ വെല്ലുവി ളിയും പ്രസ്തുത വെല്ലുവിളിക്ക് ഉത്തരം നല്‍കുന്നതില്‍ അറബി സാഹിത്യകാരന്മാര്‍ കാലാകാലങ്ങ ളായി പരാജയപ്പെടുകയാണ് ചെയ്യുന്നതെന്ന യാഥാര്‍ത്ഥ്യവും  ഖുര്‍ആനിന്റെ സാഹിത്യശൈലി അതുല്യവും അനുകരണാതീതവുമാണെന്ന വസ്തുത വ്യക്തമാക്കുന്നുണ്ട്. അറബിയല്ലാത്ത-പേര്‍ഷ്യ ക്കാരനായ ഒരാളെങ്ങനെയാണ് അതുല്യമായ ഒരു അറബി സാഹിത്യസൃഷ്ടിയുടെ സ്രോതസ്സായിത്തീ രുക?

ഇങ്ങനെ ഏത് കോണിലൂടെ നോക്കിയാലും സല്‍മാനുല്‍ ഫാരിസി (റ)യാണ് ഖുര്‍ആനിലെ ചരി ത്ര കഥനങ്ങളുടെ സ്രോതസ്സെന്ന വാദം പരിഗണന പോലുമര്‍ഹിക്കാത്ത ഒരു കേവല വാദം മാത്രമാണെന്ന വസ്തുത വ്യക്തമാവും.

ഹൂദ ക്രൈസ്തവരോടൊപ്പം ജീവിക്കുവാന്‍ അവസരം ലഭിച്ച മുഹമ്മദ് നബി (സ) അവര്‍ പറ ഞ്ഞിരുന്ന പ്രവാചകകഥകള്‍ കേട്ടിരിക്കാനിടയുണ്ടെന്നും പ്രസ്തുത കഥകളില്‍ സ്വന്തമായ ഭാവന കൂട്ടിക്കലര്‍ത്തി അദ്ദേഹം രൂപപ്പെടുത്തിയെടുത്തതാണ് ഖുര്‍ആനിലെ ചരിത്രകഥകളെന്നും വാദിക്കു ന്നവരുണ്ട്. ഈ വാദം തീരെ ദുര്‍ബ്ബലവും വ്യക്തമായ ചരിത്ര വസ്തുതകള്‍ക്ക് വിരുദ്ധവുമാണ്. താഴെ പറയുന്ന വസ്തുതകള്‍ ശ്രദ്ധിക്കുക:

(1) ജൂതന്മാരൊ ക്രൈസ്തവരോ ഒരു മതസമൂഹമെന്ന നിലയ്ക്ക് മക്കയില്‍ ഉണ്ടായിരുന്നതായി യാ തൊരു രേഖയുമില്ല; ഒരു തെളിവുമില്ല. മുഹമ്മദ് നബി (സ) യുടെ കാലത്തോ മുമ്പോ യഹൂദ മതക്കാ രോ ക്രൈസ്തവരോ മക്കയില്‍ മതസമൂഹങ്ങളായി നിലനിന്നിരുന്നില്ലെന്നാണ് ചരിത്രം വ്യക്തമാ ക്കുന്നത്.

-(1)-  മുഹമ്മദ് നബി (സ) ക്കുമുമ്പുതന്നെ അറേബ്യന്‍ ബഹുദൈവാരാധന വെറുത്ത ഏതാനും മക്ക ക്കാര്‍ സ്വന്തമായി അബ്രാഹാമീ മതത്തിന്റെ വേരുകള്‍ തേടുകയും ഏകദൈവാരാധകരായി നില നില്‍ക്കുകയും ചെയ്തിരുന്നതായി ചരിത്രഗ്രന്ഥങ്ങളില്‍ കാണാന്‍ കഴിയും. 'ഹനീഫുകള്‍' എന്ന് വിളിക്കപ്പെടുന്ന ഇവര്‍ നാല് പേരാണ്. വറഖത്തുബ്‌നു നൗഫല്‍, അബ്ദുല്ലാഹിബ്‌നു ജഹ്ശ്, ഉഥ്മാ നുബ്‌നു ഹുവാരിഥ്, സൈദുബ്‌നു അംറ് എന്നിവരാണവര്‍. തങ്ങളുടെ സമൂഹത്തില്‍ നിലനിന്ന വിഗ്രഹാരാധനയെ വെറുക്കുകയും അബ്രഹാമീ മാര്‍ഗത്തില്‍നിന്ന് സ്വസമൂഹം വഴിതെറ്റിയതില്‍ ദുഃഖിക്കുകയും യഥാര്‍ത്ഥ ദൈവിക മതത്തിന്റെ വേരുകള്‍ തേടിപ്പോവുകയും ചെയ്ത വരായി രുന്നു അവര്‍. അവരിലൊരാളായ വറഖത്തുബ്‌നു നൗഫല്‍ ഈ അന്വേഷണത്തിന്റെ ഫലമായാണ് ക്രിസ്തുമതം സ്വീകരിച്ചത്. ഇവരെല്ലാവരും ഇബ്രാഹീമിന്റെ മതമായ യഥാര്‍ത്ഥ ദൈവിക മതത്തി ന്റെ വേരുകള്‍ തേടി മക്കവിട്ട് വ്യത്യസ്ത നാടുകളില്‍ അലഞ്ഞുതിരിഞ്ഞതായി ചരിത്രഗ്രന്ഥങ്ങള്‍ വ്യക്തമാക്കുന്നു. യഹൂദരോ ക്രൈസ്തവരോ മക്കയില്‍ ഉണ്ടായിരുന്നെങ്കില്‍ ഏകദൈവ വിശ്വാസ ത്തിലധിഷ്ഠിതമായ ഇബ്രാഹീമീ മാര്‍ഗത്തിന്റെ വേരുകള്‍ തേടി അവര്‍ ഒരിക്കലും മക്ക വിടേണ്ടി വരികയില്ലായിരുന്നു.

(ii) യമനില്‍ അതിശക്തമായ ക്രൈസ്തവ ഭരണമായിരുന്നു മുഹമ്മദ് നബി (സ) യുടെ ജനനകാലത്ത് നിലനിന്നിരുന്നത്. ക്രൈസ്തവ ഭരണാധികാരിയായിരുന്ന അബ്‌റഹ മക്കക്കെതിരെ നയിച്ച വിപ്ലവം പ്രസിദ്ധമാണ്. 'ആനക്കലഹം' എന്നറിയപ്പെട്ട പ്രസ്തുത വിപ്ലവം നടന്ന വര്‍ഷമാണ് മുഹമ്മദ് നബി (സ) യുടെ ജനനം. മക്കയിലെ കഅ്ബാലയം പൊളിച്ചു കളയുകയും താന്‍ സന്‍ആയില്‍ നിര്‍മ്മിച്ച ഖുലൈസ് എന്ന ദേവാലയത്തിലേക്ക് ജനശ്രദ്ധയാകര്‍ഷിക്കുകയും ചെയ്യുകയെന്ന ലക്ഷ്യത്തോടെ നടത്തപ്പെട്ട ആനക്കലഹത്തെ അല്ലാഹു അമ്പേ പരാജയപ്പെടുത്തിയ കഥ ഖുര്‍ആനിലെ 105-ാം അധ്യാ യത്തില്‍ സംക്ഷിപ്തമായി വിവരിക്കുന്നുണ്ട്. അതിങ്ങനെയാണ്: ''ആനക്കാരെക്കൊണ്ട് നിന്റെ രക്ഷിതാവ് പ്രവര്‍ത്തിച്ചത് എങ്ങനെയെന്ന് നീ കണ്ടില്ലേ? അവരുടെ തന്ത്രം അവന്‍ പിഴവിലാക്കിയി ല്ലേ? ചുട്ടുപഴുപ്പിച്ച കളിമണ്‍ കല്ലുകള്‍ കൊണ്ട് അവരെ എറിയുന്ന കൂട്ടംകൂട്ടമായിക്കൊണ്ടുള്ള പക്ഷികളെ അവരുടെ നേര്‍ക്ക് അവന്‍ അയക്കുകയും ചെയ്തു. അങ്ങനെ അവന്‍ അവരെ തിന്നൊ ടുക്കപ്പെട്ട വൈക്കോല്‍ തുരുമ്പുപോലെയാക്കി'' (വി.ഖു. 10).

കഅ്ബാലയം തകര്‍ക്കുകയും മക്കക്കാരെ ക്രൈസ്തവവല്‍ക്കരിക്കുകയും ചെയ്യുകയെന്ന ലക്ഷ്യ ത്തോടെയാണ് അബ്‌റഹത്തിന്റെ ആനപ്പടയുടെ പുറപ്പാടുണ്ടായത്. മക്കയില്‍ ക്രൈസ്തവ സമൂഹ മുണ്ടായിരുന്നുവെങ്കില്‍ ഇത്തരമൊരു പടനീക്കമുണ്ടാകുമായിരുന്നില്ലെന്ന് വ്യക്തമാണ്.

(iii) മക്കയില്‍ ഇസ്‌ലാമിനുമുമ്പ് നിലനിന്നിരുന്ന രേഖകളിലോ കവിതകളിലോ ഒന്നുംതന്നെ ജൂതരെ യോ ക്രൈസ്തവരെയോ സംബന്ധിച്ച യാതൊരു പരാമര്‍ശവുമില്ല.

(2) ഖുര്‍ആനില്‍ ആകെ 114 അധ്യായങ്ങളാണുള്ളത്. ഇതില്‍ 27 എണ്ണം മദീനയില്‍ വെച്ചും 87 എണ്ണം മക്കയില്‍വെച്ചുമാണ് അവതരിപ്പിക്കപ്പെട്ടത്.

പ്രവാചക ചരിത്രങ്ങള്‍ വിശദീകരിക്കുന്ന സൂക്തങ്ങളിലധികവും അവതരിപ്പിക്കപ്പെട്ടത് മക്കയി ലാണ്. അവിടെയാകട്ടെ മുഹമ്മദ് നബി (സ) ക്ക് കണ്ടുമുട്ടാനോ സംസാരിക്കാനോ ആയി യഹൂദരോ ക്രൈസ്തവരോ തീരെയുണ്ടായിരുന്നുമില്ല. പിന്നെയെങ്ങനെയാണ് പൂര്‍വ്വ പ്രവാചകന്മാരുടെ ചരി ത്രങ്ങള്‍ ഖുര്‍ആനിലുണ്ടായത്? ഖുര്‍ആന്‍ പറയുന്നതാണ് ശരിയായ ഉത്തരം! ''നിങ്ങളുടെ കൂട്ടുകാ രന്‍ വഴിതെറ്റിയിട്ടില്ല, ദുര്‍മാര്‍ഗിയായിട്ടുമില്ല. അദ്ദേഹം തന്നിഷ്ടപ്രകാരം സംസാരിക്കുന്നുമില്ല. അത് അദ്ദേഹത്തിന് ദിവ്യസന്ദേശമായി നല്‍കപ്പെടുന്ന ഒരു ഉത്‌ബോധനം മാത്രമാകുന്നു'' (വി.ഖു.53:2-4).

(3) മുഹമ്മദ് നബി (സ) യുടെ ജീവിതത്തില്‍, മക്കയിലോ മദീന യിലോവെച്ച് ക്രൈസ്തവ സമൂഹ വുമായി സമ്പര്‍ക്കത്തിലാകേണ്ട സാഹചര്യങ്ങളൊന്നുമുണ്ടായതായി ചരിത്രത്തില്‍നിന്ന് മനസ്സിലാ ക്കാനാവുന്നില്ല. മദീനയില്‍വെച്ച് ജൂത സമൂഹങ്ങളുമായി സമ്പര്‍ക്കത്തിലായിരുന്നു പ്രവാചകനും അനുയായികളുമെന്നത് നേരാണ്. എന്നാല്‍ അവിടെയും ക്രൈസ്തവര്‍ ഒരു സമൂഹമായി നിലനില്‍ ക്കുന്നുണ്ടായിരുന്നില്ലെന്നാണ് മനസ്സിലാക്കാന്‍ കഴിയുന്നത്. ഇക്കാ ര്യം കാത്തോലിക്ക വിജ്ഞാനകോ ശം പോലും സമ്മതിക്കുന്നുണ്ട്. ''ഹിജാസിനെ (അറേബ്യന്‍ ഉപദ്വീപ്) ഒരിക്കലും ക്രിസ്തുമത സന്ദേശ പ്രചരണം സ്പര്‍ശിച്ചിട്ടില്ല. അതുകൊണ്ടുതന്നെ അവിടെ ക്രൈസ്തവസഭകളുടെ ശുചീകരണം പ്രതീക്ഷിക്കാവതല്ല; അതൊട്ടുകാണാനും കഴിയുന്നില്ല'' (The New Catholic Encyclopaedia Vol. I, Page 721-722)

പ്രഗത്ഭ ഗവേഷകനായ റിച്ചാര്‍ഡ് ബെല്ലിന്റെ നിരീക്ഷണവും ഇതുതന്നെ! ''ഹിജാസിലോ മക്കയുടെ യോ മദീനയുടെയെങ്കിലുമോ പ്രാന്തപ്രദേശങ്ങളിലോ ക്രൈസ്തവത നിലനിന്നിരുന്നുവെന്നതിന് യാ തൊരു തെളിവുമില്ല'' (Richard Bell: The Origin of Islam in Its Christian Environment Page 42).

ഖുര്‍ആനില്‍ യേശുവിനെയും മാതാവിനെയും അവരുടെ കുടുംബത്തെയുംകുറിച്ച് പലസ്ഥലങ്ങ ളിലും പരാമര്‍ശിച്ചിട്ടുണ്ട്. ബൈബിളില്‍പോലും പരാമര്‍ശിച്ചിട്ടില്ലാത്ത പല സംഭവങ്ങളും യേശു വിന്റെ ജീവിതവുമായി ബന്ധപ്പെടുത്തി ഖുര്‍ആന്‍ ഉദ്ധരിക്കുന്നുണ്ട്. ഈ കാര്യങ്ങള്‍ മുഹമ്മദ് നബി (സ) ക്ക് എവിടെനിന്നു കിട്ടി?  മറ്റ് പ്രവാചകന്മാരുമായി ബന്ധപ്പെട്ട പരാമര്‍ശങ്ങള്‍ യഹൂദന്മാരുമാ യുള്ള സഹവര്‍ത്തിത്വത്തിന്റെകാലത്ത് അവര്‍ പറഞ്ഞുകൊടുത്ത കഥകളുടെ വെളിച്ചത്തില്‍ എഴു തിയതാണെന്ന് വാദിക്കുന്നവര്‍ യേശുവിനെയും മാതാവിനെയുംക്കുറിച്ച ഖുര്‍ആനിക വിവരണങ്ങ ളുടെ സ്രോതസ്സെന്തായിരുന്നുവെന്ന്, മറ്റ് പ്രവാചകന്മാരുടെ ചരിത്രകഥകളുടെ കാര്യത്തില്‍ അവര്‍ സ്വീകരിച്ച അതേ മാനദണ്ഡമുപയോഗിച്ച്, വ്യക്തമാക്കുവാന്‍ ബാധ്യസ്ഥരാണ്. പക്ഷെ, അവര്‍ക്ക് അതിന് സാധ്യമല്ല. യഥാര്‍ത്ഥത്തില്‍ യേശുവിനെയും മാതാവിനെയും കുറിച്ച ഖുര്‍ആനിലുള്ള അറിവിന്റെ സ്രോതസ്സെന്തായിരുന്നുവെന്ന് ഖുര്‍ആന്‍തന്നെ സൂചിപ്പിക്കുന്നുണ്ട്. അതല്ലാതെ മറ്റൊ രു വിശദീകരണവും ഇക്കാര്യത്തില്‍ മനുഷ്യബുദ്ധിയെ സംതൃപ്തമാക്കുന്നതായി നിലവിലില്ല. ''(നബിയേ) നാം നിനക്ക് ബോധനം നല്‍കുന്ന അദൃശ്യവാര്‍ത്തകളില്‍ പെട്ടതാകുന്നു അവയൊക്കെ. അവരില്‍ ആരാണ് മര്‍യത്തിന്റെ സംരക്ഷണം ഏറ്റെടുക്കേണ്ടതെന്ന് തീരുമാനിക്കുവാനായി അവര്‍ തങ്ങളുടെ അമ്പുകള്‍ ഇട്ടുകൊണ്ട് നറുക്കെടുപ്പ് നടത്തിയിരുന്ന സമയത്ത് നീ അവരുടെ അടുത്തുണ്ടായിരുന്നില്ലല്ലോ. അവര്‍ തര്‍ക്കത്തിലേര്‍പ്പെട്ടുകൊണ്ടിരുന്നപ്പോഴും നീ അവരുടെ അടുത്തുണ്ടായിരുന്നില്ല'' (വി.ഖു. 3:44)

മുഹമ്മദ് (സ) നബിക്ക് വഹ്‌യ് കിട്ടിയശേഷം അദ്ദേഹത്തെ പത്‌നി ഖദീജ (റ) തന്റെ ബന്ധുവായ വറഖത്തുബ്‌നു നൗഫലിന്റെ അടുക്കലേക്ക് കൊണ്ടുപോയതായി പറയുന്ന സഹീഹുല്‍ ബുഖാരിയിലെ രണ്ട് ഹദീസുകളുടെ വെളിച്ചത്തിലാണ് ഇസ്‌ലാം വിമർശകന്മാര്‍ വറഖയാവാം മുഹമ്മദി(സ)ന് ബൈബിളിലെ വിവരങ്ങള്‍ പറഞ്ഞുകൊടുത്തതെന്ന് സമര്ത്ഥിക്കുന്നത്. പ്രസ്തുത ഹദീസുകള്‍ കാണുക:

ആയിശ പറയുന്നു: ''നബി തിരുമേനി(സ)ക്ക് തുടക്കത്തിൽ ലഭിച്ച ദൈവിക സന്ദേശങ്ങളുടെ ആരംഭം ഉറക്കത്തില്‍ ദൃശ്യമാകുന്ന നല്ല സ്വപ്‌നങ്ങളായിരുന്നു. അവിടുന്ന് കാണുന്ന എല്ലാ സ്വപ്‌നങ്ങളും പ്രഭാതോദയം പോലെ സ്പഷ്ടമായി പുലർന്നു കൊണ്ടേയിരുന്നു. പിന്നീട് തിരുമേനിക്ക് ഏകാന്ത വാസം പ്രിയങ്കരമായിത്തോന്നി. അങ്ങനെ ഏതാനും രാത്രികള്‍ ഹിറാഗുഹയില്‍ ഏകാന്തവാസം അനുഷ്ഠിച്ചു. ആ രാത്രികള്ക്കുള്ള ആഹാരപദാര്ത്ഥ ങ്ങളുമായി ഗുഹയിലേക്ക് പോകും. കുറെ രാത്രി ആരാധനയില്‍ മുഴുകി അവിടെ കഴിച്ചുകൂട്ടും. പിന്നെ ഖദീജാ (റ)യുടെ അടുക്കലേക്ക് തിരിച്ചുവരും. വീണ്ടും ആഹാരപദാര്ത്ഥ്ങ്ങള്‍ തയ്യാറാക്കി പുറപ്പെടും. ഹിറാ ഗുഹയില്വെിച്ച് തിരുമേനിക്ക് സത്യം വന്നുകിട്ടുന്നതുവരെ ഈ നില തുടർന്നു പോന്നു. അങ്ങനെ മലക്ക് തിരുമേനി (സ)യുടെ മുമ്പില്‍ പ്രത്യക്ഷപ്പെട്ടു. ''വായിക്കുക'' എന്ന് പറഞ്ഞു. നബി (സ) പ്രതിവചിച്ചു. എനിക്ക് വളരെ വിഷമം അനുഭവപ്പെട്ടു. അനന്തരം എന്നെ വിട്ട് വീണ്ടും ''വായിക്കുക'' എന്ന് കല്പിപച്ചു. വായിക്കാന്‍ അറിയില്ലെന്ന് ഞാന്‍ അപ്പോഴും മറുപടി നല്കില. മലക്ക് എന്നെ പിടിച്ച് ശക്തിയായി ആശ്ലേഷിച്ചു. എനിക്ക് വളരെ വിഷമം തോന്നി. പിന്നീട് എന്നെ വിട്ടശേഷം ''വായിക്കുക'' എന്ന് പറഞ്ഞു. എനിക്ക് വായന അറിയില്ലായെന്ന് പിന്നെയും ഞാന്‍ പറഞ്ഞ പ്പോള്‍ മൂന്നാമതും മലക്ക് എന്നെ പിടിച്ച് ശക്തിയോടെ ആശ്ലേഷിച്ചു. അനന്തരം എന്നെ വിട്ടിട്ട് പറഞ്ഞു: ''സ്രഷ്ടാവായ നിന്റെ രക്ഷിതാവിന്റെ നാമത്തില്‍ വായിക്കുക. മനുഷ്യനെ അവന്‍ ഭ്രൂണത്തില്നികന്ന് സൃഷ്ടിച്ചിരിക്കുന്നു. നീ വായിക്കുക, നിന്റെ രക്ഷിതാവ് അത്യുദാരനത്രെ'' ഉടനെ പിടക്കുന്ന ഹൃദയത്തോടെ ഈ സന്ദേശവുമായി തിരുമേനി (സ) മടങ്ങി. ഖുവൈലിദിന്റെ മകള്‍ ഖദീജയുടെ അടുക്കല്‍ കയറിച്ചെന്ന് പുതച്ചുതരിക, പുതച്ചുതരിക എന്ന് അവിടുന്ന് അഭ്യര്ത്ഥിച്ചു. അവര്‍ പുതച്ചുകൊടുത്തു. ആ ഭയം നിശ്ശേഷം നീങ്ങിയപ്പോള്‍ നടന്ന സംഭവങ്ങളെല്ലാം ഖദീജാബീവിയെ ധരിപ്പിച്ചു. തന്റെ ജീവന് എന്തെങ്കിലും ആപത്ത് സംഭവിക്കുമോ എന്ന് ഭയപ്പെടുന്നതായി അദ്ദേഹം അവരോട് പറഞ്ഞു. അപ്പോള്‍ ഖദീജ പറഞ്ഞു: ഇല്ല, അല്ലാഹുവാണെ സത്യം. അവന്‍ അങ്ങയെ ഒരിക്കലും അപമാനിക്കുകയില്ല. താങ്കള്‍ കുടുംബബന്ധം പുലര്ത്തുന്നു. പരാശ്രയരുടെ ഭാരം ചുമക്കുന്നു. അഗതികള്ക്ക് സ്വയം അധ്വാനിച്ച് സഹായം ചെയ്തുകൊടുക്കുന്നു. അതിഥികളെ സല്ക്കടരിക്കുന്നു. വിപല്ഘ നട്ടങ്ങളില്‍ ശരിയായ സഹായം നല്കുകന്നു. പിന്നീട് തിരുമേനി(സ)യെയും കൂട്ടി ഖദീജ (റ) തന്റെ പിതൃവ്യപുത്രനായ വറഖത്ത്ബ്‌നു നൗഫലിബ്‌നി അസദിബ്‌നി അബ്ദില്‍ ഉസ്സയുടെ അടുക്കലേക്ക് ചെന്നു. വറഖത്ത് അജ്ഞാനകാലത്ത് ക്രിസ്ത്യാനിയായവനും ഹിബ്രു ഭാഷയില്‍ എഴുതാന്‍ പഠിച്ചവനുമായിരുന്നു. തന്നിമിത്തം അദ്ദേഹം സുവിശേഷത്തില്നിതന്ന് ചില ഭാഗങ്ങള്‍ ഹിബ്രുവില്‍ എഴുതിയെടുക്കാറുണ്ടായിരുന്നു. അദ്ദേഹം വയോവൃദ്ധനായി കണ്കാഴ്ച തന്നെ നഷ്ടപ്പെട്ടു കഴിഞ്ഞിരുന്നു. ഖദീജ (റ) പറഞ്ഞു: ''പിതൃവ്യപുത്രാ താങ്കളുടെ സഹോദര പുത്രന്റെ വിശേഷങ്ങള്‍ ഒന്ന് ശ്രദ്ധിക്കുക''. വറഖത്ത് ചോദിച്ചു: ''എന്റെ സഹോദര പുത്രാ നീ എന്താണ് ദര്ശിച്ചത്?'' കണ്ട കാഴ്ചകളെല്ലാം തിരുമേനി (സ) വറഖത്തിനെ അറിയിച്ചു. വറഖത്ത് പറഞ്ഞു: ഇത് അല്ലാഹു മൂസാ(അ)യുടെ അടുക്കലേക്ക് അയച്ചിരുന്ന അതേ നന്മ യുടെ രഹസ്യ സന്ദേശവാഹകനാണ്. താങ്കള്‍ മതപ്രബോധനം ചെയ്യുന്ന സന്ദര്ഭദത്തി ല്‍ ഞാനൊരു യുവാവായിരുന്നെങ്കില്‍! താങ്കളെ സ്വദേശത്ത് നിന്ന് സ്വജനത ബഹിഷ്‌കരിക്കുന്ന ഘട്ടത്തില്‍ ഞാനൊരു യുവാവായിരുന്നുവെങ്കില്‍!!'' തിരുമേനി (സ) ചോദിച്ചു. അവർ എന്നെ ബഹിഷ്‌കരിക്കുകയോ? വറഖത്ത് പറഞ്ഞു. താങ്കള്‍ കൊണ്ടുവന്നതുപോലെയുള്ള സന്ദേശങ്ങളുമായി വന്ന ഒരു മനുഷ്യനും തന്റെ ജനതയുടെ ശത്രുതയ്ക്ക് പാത്രമാകാതിരുന്നിട്ടില്ല. താങ്കളുടെ പ്രവര്ത്തളനങ്ങള്‍ നടക്കുന്നദിവസം ഞാന്‍ ജീവിച്ചിരിപ്പുണ്ടെങ്കില്‍ സുശ ക്തമായ ഒരു സഹായം താങ്കള്ക്ക്് നല്കുുമായിരുന്നു. പക്ഷെ, പിന്നീട് അധികം കഴിഞ്ഞില്ല. വറഖത്ത് മരണമടഞ്ഞു. ദൈ വിക സന്ദേശങ്ങളുടെ അവതരണം നിലയ്ക്കുകയും ചെയ്തു'' (സഹീഹുല്‍ ബുഖാരി).

ഈ ഹദീസുകള്‍ സത്യസന്ധവും മുന്ധാണരണയില്ലാത്തതുമായ വായനയ്ക്ക് വിധേയമാക്കിയാല്ത നന്നെ വറഖത്തു ബ്‌നു നൗഫലില്നിതന്നാണ് പ്രവാചകന്‍ (സ) ചരിത്രകഥകള്‍ മനസ്സിലാക്കിയത് എന്ന വാദം അടിസ്ഥാനരഹിതമാ ണെന്ന് മനസ്സിലാവും. താഴെ പറയുന്ന വസ്തുതകള്‍ ശ്രദ്ധിക്കുക:

(1) മുഹമ്മദ് നബി (സ)ക്ക് പ്രവാചകത്വം ലഭിക്കുമ്പോള്‍ വറ ഖത്തുബ്‌നു നൗഫല്‍ വാര്ധരക്യംമൂലം കാഴ്ച നഷ്ടപ്പെട്ട വ്യക്്തിയായിരുന്നു. ഇതുകഴിഞ്ഞ് അല്പ‍കാലത്തിനകം അദ്ദേഹം മരണപ്പെട്ടിരിക്കണം. പൂർവ്വ പ്രവാചക ന്മാരെക്കുറിച്ച ഖുർആനിക പരാമര്ശങ്ങള്‍ അവതരിക്കുന്ന കാലത്ത് അദ്ദേഹം ജീവിച്ചിരുന്നുവെന്ന് കരുതാന്‍ വയ്യ. അദ്ദേഹം പറഞ്ഞുകൊടുത്ത് എഴുതിയതാകാം ഖുർആനിലെ പ്രവാചക കഥനങ്ങളെന്ന് കരുതുന്നത് അതുകൊണ്ടുതന്നെ യുക്തിസഹമല്ല.

(2) പൂർവ്വ പ്രവാചകന്മാരില്‍ ചിലരുടെ കഥകള്‍ അടങ്ങിയ ഖുർആൻ സൂക്തങ്ങള്‍ അവതരി പ്പിക്കപ്പെട്ടത് പ്രവാചകനും (സ) അനുചരന്മാരും തമ്മില്‍ സംഭാഷണം നടത്തുമ്പോഴുള്ള പ്രശ്‌ന ങ്ങള്ക്ക്ക പരിഹാരം നിര്ദ്ദേശിക്കുന്നതിനും വേദക്കാരായ യഹൂദ ക്രൈസ്തവരുമായി സംവദി ക്കുമ്പോള്‍ അവരുടെ ചോദ്യങ്ങള്ക്ക്ക ഉത്തരം നല്‍കുന്നതിനു മായിരുന്നു. ഈ സമയത്തൊന്നും വറഖത്തുബ്‌നു നൗഫല്‍ പ്രവാചകനോടൊപ്പമുണ്ടായിരുന്നില്ലല്ലോ. പിന്നെയെങ്ങനെയാണ് പ്രവാചകന്‍ (സ) പൂർവ്വ പ്രവാചകന്മാരുടെ ചരിത്രത്തില്നികന്ന് സൂക്ഷ്മവും കൃത്യവുമായി കാര്യങ്ങളുദ്ധരിക്കുക?

(3) മുഹമ്മദ് നബി (സ)ക്ക് വറഖത്തുബ്‌നു നൗഫല്‍ അദ്ദേഹത്തിന്റെ പ്രവാചകത്വ ലബ്ധിക്കുമുമ്പ് എന്തെങ്കിലും കാര്യങ്ങള്‍ പഠിപ്പിച്ചുകൊടുത്തിരുന്നെങ്കില്‍ ആ സമൂഹത്തിലെ ചിലര്ക്കെങ്കിലും അക്കാര്യം അറിയാമായിരുന്നി രിക്കണം. പ്രവാചകന്റെ അനുചരന്മാരിലോ ശത്രുക്കളിലോ പെട്ട സമകാലികരായ ആരുംതന്നെ വറഖത്ത് ബ്‌നു നൗഫല്‍ പഠിപ്പിച്ചുകൊടുത്ത കാര്യങ്ങളാണ് മുഹ മ്മദ് നബി (സ) ഖുർആനില്‍ ഉള്ക്കൊചള്ളിക്കുന്നത് എന്ന ആരോ പണമുന്നയിച്ചിരുന്നില്ല.

4) ജൂത-ക്രൈസ്തവ വേദങ്ങളില്‍ പാണ്ഡിത്യമുണ്ടായിരുന്ന വറഖത്തുബ്‌നു നൗഫല്‍ മുഹമ്മദ് നബിക്ക് പൂർവ്വ പ്രവാചകന്മാരുടെ കഥകള്‍ പഠിപ്പിച്ചുകൊടുത്തിരുന്നുവെങ്കില്‍ യഹൂദരും ക്രൈസ്ത വരും വികലമാക്കിയ പ്രവാചക കഥനങ്ങളായിരിക്കണം അദ്ദേഹം പറഞ്ഞുകൊടുത്തിരിക്കുക. പ്രസ്തുത വിശദീകരണങ്ങളില്‍ ബൈബിളില്‍ ഇന്ന് കാണപ്പെടുന്ന രീതിയിലുള്ള അശാസ്ത്രീയവും ചരിത്രവിരുദ്ധവും വൈരുദ്ധ്യങ്ങളാല്‍ നിബിഡവുമായ കഥാകഥനങ്ങളു മുണ്ടാവും. അദ്ദേഹം പറഞ്ഞുകൊടുത്തതിന്റെ അടിസ്ഥാനത്തില്‍ എഴുതിയതായിരുന്നു ഖുർആനെങ്കില്‍ അതിലും ഇത്തരം അബദ്ധങ്ങളുണ്ടാവേണ്ടതായിരുന്നു. എന്നാല്‍ ഖുർആനില്‍ ഇത്തരം അബദ്ധങ്ങളൊന്നുമില്ല.

5) പൂർവ്വ വേദങ്ങളില്‍ പണ്ഡിതനായിരുന്ന വറഖത്തുബ്‌നു നൗഫല്‍ മുഹമ്മദ് നബി (സ) ക്കുണ്ടായ ആദ്യ ദിവ്യ ബോധനത്തിന്റെ അനുഭവങ്ങള്‍ കേട്ടപ്പോള്‍ ഇത് ദൈവിക ബോധനത്തിന്റെ ആരംഭമാണെന്നും ''താങ്കളെ ജനം കയ്യൊഴിയുമ്പോള്‍ ഞാന്‍ ജീവിച്ചിരിക്കുന്നുവെങ്കില്‍ ഞാന്‍ താങ്കളെ ശക്തമായി പിന്തുണക്കു''മെന്നും പറഞ്ഞതായി നടേ ഉദ്ധരിച്ച ഹദീസുകള്‍ വ്യക്തമാക്കുന്നത്. തന്നില്‍ നിന്ന് കേട്ടുപഠിച്ച പ്രവാചകകഥകളുടെ അടിസ്ഥാനത്തില്‍ താനും പ്രവാചകനാണെന്ന് വരുത്തിത്തീർക്കാന്‍ മുഹമ്മദ് (സ) ശ്രമിക്കുകയാണെന്ന ഒരു ചെറിയ ശങ്കപോലും വറഖത്തുബ്‌നു നൗഫലിനു ണ്ടായില്ല. മുമ്പൊരിക്കലും മുഹമ്മദി(സ)ന് പൂര്വ്വലപ്രവാചകന്മാരുടെ കഥകള്‍ വറഖത്തുബ്‌നു നൗഫല്‍ പഠിപ്പിച്ചുകൊടുത്തിട്ടില്ലെന്ന സത്യം ഇതിലൂടെ സുതരാം വ്യക്തമാവുന്നുണ്ട്.

കനായ സ്രഷ്ടാവ് നിയോഗിച്ചയച്ച പ്രവാചകന്മാരെക്കുറിച്ച് ബൈബിളിലും ഖുര്‍ആനിലും വന്ന സമാനമായ ചരിത്രപരാമര്‍ശങ്ങളുടെ വെളിച്ചത്തില്‍ ബൈബിളില്‍നിന്ന് പകര്‍ത്തിയെഴുതിയതാണ് ഖുര്‍ആന്‍ എന്ന വാദം മിഷനറിമാരും ഓറിയന്റലിസ്റ്റുകളും ഭൗതികവാദികളുമെല്ലാമായ വിമര്‍ ശകര്‍ ഒരേസ്വരത്തില്‍ ഉന്നയിക്കാറുണ്ട്. ഈ വാദത്തില്‍ എത്രത്തോളം കഴമ്പുണ്ട്? താഴെ പറയുന്ന വസ്തുതകളുടെ വെളിച്ചത്തില്‍ ചിന്തിക്കുമ്പോള്‍ ഈ വാദം ശുദ്ധ അസംബ ന്ധമാണെന്ന് ബോധ്യ മാകും.

ഒന്ന്) മുഹമ്മദ് നബി (സ) നിരക്ഷരനായിരുന്നു. ബൈബിള്‍ പഴയനിയമവും പുതിയനിയമവും വായിച്ചു മനസ്സിലാക്കി അതില്‍നിന്ന് പകര്‍ത്തിയെഴുതുക അദ്ദേഹത്തിന് സ്വന്തമായി അസാധ്യ മായിരുന്നു. ശിഷ്യന്മാരില്‍ ആരുടെയെങ്കിലും സഹായത്തോടെ അദ്ദേഹം അത് നിര്‍വഹിച്ചുവെന്ന് കരുതാനും വയ്യ. അങ്ങനെ ചെയ്തിരുന്നുവെങ്കില്‍ ശിഷ്യന്മാരില്‍ ചിലര്‍ക്കെങ്കിലും അത് അറിയാന്‍ കഴിയേണ്ടതായിരുന്നു. അത് മുഖേന മുഹമ്മദ് നബി (സ)യുടെ വിശ്വാസ്യതയില്‍ അവര്‍ സംശയി ക്കുകയും അവര്‍ തമ്മിലുള്ള ബന്ധത്തിന് ഉലച്ചില്‍തട്ടുകയും ചെയ്യുമായിരുന്നു. മുഹമ്മദ് നബി (സ)യുടെ ശരീരത്തില്‍ ഒരു പോറലെങ്കിലുമേല്‍ക്കുന്നതിന് പകരം സ്വന്തം ജീവന്‍ ബലിയര്‍പ്പിക്കു വാന്‍ സന്നദ്ധരായവരായിരുന്നു പ്രവാചക ശിഷ്യന്മാര്‍ എന്നോര്‍ക്കുക. പ്രവാചകനില്‍ (സ) ഏതെ ങ്കിലുംതരത്തിലുള്ള അവിശ്വാസ്യതയുണ്ടായിരുന്നുവെങ്കില്‍ ഇങ്ങനെ ത്യാഗം ചെയ്യാന്‍ സന്ന ദ്ധരായ ഒരു അനുയായിവൃന്ദത്തെ വളര്‍ത്തിയെടുക്കുവാന്‍ അദ്ദേഹത്തിന് കഴിയുമായിരുന്നില്ലെന്ന് തീര്‍ച്ച യാണ്.

''ഇതിന് മുമ്പ് നീ വല്ല ഗ്രന്ഥവും പാരായണം ചെയ്യുകയോ, നിന്റെ വലതുകൈകൊണ്ട് അത് എഴുതു കയോ ചെയ്തിരുന്നില്ല. അങ്ങനെയാണെങ്കില്‍ ഈ സത്യനിഷേധികള്‍ക്ക് സംശയിക്കാമായിരുന്നു'' (വി.ഖു. 29:48).

രണ്ട്) മുഹമ്മദ് നബി (സ) യുടെ ജീവിതകാലത്ത് ബൈബിള്‍ പഴയനിയമമോ പുതിയനിയമമോ അറ ബിയിലേക്ക് പരിഭാഷപ്പെടുത്തപ്പെട്ടിട്ടുണ്ടായിരുന്നില്ല. അറബിയിലുള്ള പഴയനിയമവും പുതിയ നിയമവുമെല്ലാം ഉണ്ടായതുതന്നെ ഇസ്‌ലാമിന്റെ ദിഗ്‌വിജയങ്ങള്‍ക്ക് ശേഷമാണ്. പഴയ നിയമ രേഖകളെക്കുറിച്ച് സൂക്ഷ്മ പഠനം നടത്തിയ ഏണസ്റ്റ് വൂര്‍ഥ്‌വിന്‍ എഴുതുന്നത് കാണുക: ''ഇസ്‌ലാ മിന്റെ വ്യാപനത്തോടുകൂടി അറബിയുടെ ഉപയോഗം വ്യാപകമാവുകയും ഇസ്‌ലാമിക രാജ്യങ്ങ ളിലെ ജൂതന്മാരുടെയും ക്രിസ്ത്യാനികളുടെയും ദൈനംദിനജീവിതത്തിലെ ഭാഷയായി അറബി മാറു കയും ചെയ്തു. ബൈബിളിന്റെ അറബി പതിപ്പുകള്‍ അനിവാര്യമാക്കി ത്തീര്‍ത്ത ഈ സാഹചര്യ ത്തില്‍ സ്വതന്ത്രവും പ്രാഥമികമായ വ്യാഖ്യാന സംബന്ധിയുമായ നിരവധി പതിപ്പുകള്‍ പുറത്തു വന്നു''.(Ernst Wurthewein: The Text of The Old Testament Page 104).

ഒമ്പതാം നൂറ്റാണ്ടിന്റെ ആദ്യപകുതിയിലാണ് പഴയ നിയമബൈ ബിള്‍ അറബിയിലേക്ക് പരിഭാഷപ്പെടുത്തപ്പെട്ടതെന്നാണ് ലഭ്യമായ കയ്യെഴുത്ത് രേഖകള്‍ വ്യക്തമാക്കുന്നത് (Ibid Page 224-225).

ഏക ദേശം ഇക്കാലത്തുതന്നെയാവണം പുതിയ നിയ മവും അറബിയിലേക്ക് ഭാഷാന്തരം ചെയ്യപ്പെട്ടത്. പ്രഗത്ഭനായ സിഡ്‌നി എച്ച്. ഗ്രിഫിത്തിന്റെ വരികള്‍ കാണുക:''അറബിയിലുള്ള സുവിശേഷങ്ങളടങ്ങിയ ഏറ്റവും പുരാതനമായ കയ്യെഴുത്ത് രേഖ 'സിനായ് അറ ബി കയ്യെഴുത്ത് പ്രതി 72' (Sinai Arabic MS72) ആണ്. ജറുസലേം സഭയുടെ ഗ്രീക്ക് പ്രാര്‍ത്ഥനാ കലണ്ടറി ന്റെ കാലക്രമാടിസ്ഥാനത്തില്‍ അധ്യായങ്ങള്‍ രേഖപ്പെടുത്തി യ നാല് കാനോനിക സുവിശേഷങ്ങ ളും ഇതിലുണ്ട്. രേഖയുടെ അന്ത്യത്തിലെ കുറിപ്പ് വ്യക്തമാക്കുന്നത് ഈ കയ്യെഴുത്ത് രേഖ അറബി കലണ്ടര്‍ 284ല്‍ അഥവാ ക്രിസ്താബ്ദം 897ല്‍ റംലയിലെ സ്റ്റീഫന്‍ (Stephen of Ramlah) എഴുതിയതാണെന്ന് (Sidney H Griffith: The Gospel in Arabic: An Enquiry Into its Appearance In the First Abbasi Century Page 132) എന്നാല്‍ അപ്പോസ്തല പ്രവൃത്തികളും പൗലോസിന്റെ ലേഖനങ്ങളും കാതോലിക ലേഖനങ്ങ ളുമുള്‍ക്കൊള്ളുന്ന Sinai Arabic MS151 എന്ന കയ്യെഴുത്ത് രേഖ ഹിജ്‌റ253 ല്‍ അഥവാ ക്രിസ്താബ്ദം 867ല്‍ സുറിയാനിയില്‍നിന്ന് അറബിയിലേക്ക് ബിസ്ര്‍ബ്‌നുസിര്‍റി എന്നയാള്‍ വിവര്‍ത്തനം ചെയ്തതായി കാണുന്നുണ്ട്. ഇതില്‍ സുവിശേഷങ്ങളില്ലെന്ന കാര്യം പ്രത്യേകം ശ്രദ്ധേയമാണ്. (Ibid Page 131).

മുഹമ്മദ് നബി(ല)ക്ക് ശേഷം രണ്ട് നൂറ്റാണ്ടുകളെങ്കിലും കഴിഞ്ഞാണ് പുതിയനിയമവും പഴയനി യമവുമെല്ലാം അറബിയിലേക്ക് വിവര്‍ത്തനം ചെയ്യപ്പെട്ടത്. നിരക്ഷരനായിരുന്ന മുഹമ്മദ് നബി (സ) മറ്റാരില്‍നിന്നെങ്കിലും അറബിയിലുള്ള ബൈബിള്‍ വായിച്ചുകേട്ടശേഷം അതിലെ കഥകള്‍ ഉള്‍ക്കൊ ള്ളിച്ചുകൊണ്ട് എഴുതിയതാണ് ഖുര്‍ആന്‍ എന്ന വാദവും ഇവിടെ അപ്രസക്തമാവുകയാണ്. അറ ബിയില്‍ നിലവിലില്ലാത്ത ഒരു ഗ്രന്ഥം വായിച്ചുകേട്ടുവെന്ന് കരുതുന്നത് നിരര്‍ത്ഥകമാണെന്ന് പറ യേണ്ടതില്ലല്ലോ.

മൂന്ന്) പ്രവാചകന്മാരുടെ ചരിത്രം വിവരിക്കുന്നിടത്ത്  അധാര്‍മ്മികരും അസാന്മാര്‍ഗികരുമായി രുന്നു അവരെന്ന് വരുത്തിത്തീര്‍ക്കുന്ന തരത്തിലാണ് ബൈബിള്‍ അത് നിര്‍വ്വഹിച്ചിരിക്കുന്നത്. മദ്യ പിച്ച് നഗ്‌നനായ നോഹും ലഹരിമൂത്ത് സ്വപുത്രിമാരുമായി ശയിച്ച ലോത്തും ചതിയനായ യാ ക്കോബും വിഷയലമ്പടനായ ദാവീദും മദ്യം വിളമ്പിയ യേശുവുമെല്ലാം, ധര്‍മ്മത്തിലേക്ക് ജനങ്ങളെ നയിക്കാനായി നിയോഗിക്കപ്പെട്ടവരായിരുന്നു പ്രവാചകന്മാര്‍ എന്ന സങ്കല്‍പത്തിന് കടക വിരുദ്ധ മായ കഥകളാണെന്ന് പറയേണ്ടതില്ലല്ലോ. ഖുര്‍ആനിലെ ചരിത്രവിവരണത്തില്‍ ഇത്ത രം യാതൊരു കഥകളും കാണുന്നില്ല. ബൈബിളില്‍നിന്ന് മുഹമ്മദ് നബി (സ) പകര്‍ത്തിയെഴു തിയതായിരുന്നു ഈ കഥകളെങ്കില്‍  പ്രവാചകന്മാരില്‍ ബൈബിള്‍ ആരോപിച്ച അധാര്‍മ്മികതകളി ലേതെങ്കിലും ഖുര്‍ആ നിലും സ്ഥാനം പിടിക്കേണ്ടതായിരുന്നു. അങ്ങനെയില്ലെന്ന് മാത്രമല്ല, പ്രവാ ചകന്മാരെല്ലാം ഉന്നതരും വിശുദ്ധരുമായിരുന്നുവെന്ന വസ്തുത വ്യക്തമാക്കുന്നതാണ് ഖുര്‍ആനിലെ പ്രവാചക കഥനങ്ങളെ ല്ലാമെന്ന കാര്യം അത് ബൈബിളില്‍നിന്ന് പകര്‍ത്തിയെഴുതിയതാണെന്ന വാദത്തിന്റെ നട്ടെല്ലൊടി ക്കുന്നുണ്ട്.

നാല്) ചരിത്രത്തിന്റെ അളവുകോലുകള്‍ വെച്ചുനോക്കുമ്പോള്‍ വസ്തുനിഷ്ഠചരിത്രത്തിന് നിരക്കാ ത്ത നിരവധി പ്രസ്താവനകള്‍ ബൈബിള്‍ നടത്തുന്നുണ്ട്. ഇത് ബൈബിള്‍ പണ്ഡിതന്മാര്‍ തന്നെ അംഗീ കരിക്കുന്നതാണ്. ''ചരിത്രപരമായി കൃത്യമല്ലാത്ത ചില പ്രസ്താവനകളും ബൈബിളില്‍ കണ്ടെന്നു വരാം'' (ബൈബിള്‍ വിജ്ഞാനകോശം പുറം 12). ബൈബിളില്‍നിന്ന് പകര്‍ത്തിയെഴുതിക്കൊണ്ട് മുഹമ്മദ് നബി (സ)രചിച്ചതായിരുന്നു ഖുര്‍ആനെങ്കില്‍ അതില്‍ ബൈബിളിലേതുപോലെ ചരിത്രപ രമായി കൃത്യമല്ലാത്ത പ്രസ്താവനകള്‍ കാണപ്പെടേണ്ടതായിരുന്നു. എന്നാല്‍, അത്തരം യാതൊരു പ്രസ്താവനയും ഖുര്‍ആനിലില്ല.

അഞ്ച്) ആധുനിക ശാസ്ത്രത്തിന്റെ കാഴ്ചപ്പാടിലൂടെ നോക്കുമ്പോള്‍ ബൈബിളില്‍ നിരവധി അശാ സ്ത്രീയമായ പരാമര്‍ശങ്ങള്‍ കാണാനാവും. സൂര്യന്റെ സൃഷ്ടിക്ക് മുമ്പുതന്നെ രാപ്പകലുകളുണ്ടായ തായി വിവരിക്കുന്ന ഉല്‍പത്തി പുസ്തകം മുതലാരംഭിക്കുന്നു ബൈബിളിലെ ശാസ്ത്രവിരുദ്ധമായ പരാമര്‍ശങ്ങള്‍. രാപ്പകലുകളുണ്ടാവുന്നത് സൂര്യചന്ദ്രന്മാരുടെ ചലനം മൂലമാണെന്നും (യേശു 10:12,13), ഭൂമി ഇളകാതെ നിശ്ചലമായി നില്‍ക്കുകയാണെന്നും (സങ്കീ 104:5) മുയല്‍ അയവിറക്കുന്ന ജീവിയാണെന്നു (ആവ 14:7) മെല്ലാമുള്ള ബൈബിള്‍ പരാമര്‍ശങ്ങള്‍ അതിന്റെ അശാസ്ത്രീയതക്ക് ഉദാഹരണങ്ങളാണ്. ഈ പരാമര്‍ശങ്ങളെല്ലാം വരുന്നത് പ്രവാചകകഥനങ്ങള്‍ക്കിടയിലാണെന്ന കാര്യം പ്രത്യേകം ശ്രദ്ധേയമാണ്. ബൈബിളായിരുന്നു ഖുര്‍ആനിന്റെ രചനയ്ക്കുപയോഗിച്ചിരുന്ന സ്രോതസ്സെങ്കില്‍ ഈ അശാസ്ത്രീയമായ പരാമര്‍ശങ്ങളെല്ലാം ഖുര്‍ആനിലും സ്ഥാനം പിടിക്കുമായി രുന്നു. ഈ പരാമര്‍ശങ്ങള്‍ വസ്തുതകള്‍ക്ക് നിരക്കാത്തതാണെന്ന അറിവ് മുഹമ്മദ് നബി (സ)യുടെ കാലത്തുണ്ടായിരുന്നില്ലെന്നോര്‍ക്കുക. എന്നാല്‍ ഖുര്‍ആനില്‍ ഇത്തരം യാതൊരുവിധ പരാമര്‍ശങ്ങ ളുമില്ല. ഖുര്‍ആനിലെ ഒരൊറ്റ വചനമെങ്കിലും ഏതെങ്കിലും ശാസ്ത്രവസ്തുതകളുമായി വൈരുദ്ധ്യം പുലര്‍ത്തുന്നതായി തെളിയിക്കപ്പെട്ടിട്ടില്ല. ബൈബിളില്‍നിന്ന് പകര്‍ത്തിക്കൊണ്ട് മുഹമ്മദ് നബി (സ) രചിച്ച ഗ്രന്ഥമാണ് ഖുര്‍ആനെന്ന് വാദിക്കുകയാണെങ്കില്‍ തനിക്ക് ശേഷം നൂറ്റാണ്ടുകള്‍ കഴിഞ്ഞ് വരാനിരിക്കുന്ന ശാസ്ത്രമുന്നേറ്റങ്ങള്‍ കൂടി മുന്‍കൂട്ടി കാണാന്‍ കഴിയുകയും അതിന്റെ അടിസ്ഥാന ത്തില്‍ ബൈബിളിലുള്ള അശാസ്ത്രീയതകള്‍ അറിഞ്ഞ് അവയെല്ലാം അരിച്ചൊഴിവാക്കി സംശുദ്ധ മായ ചരിത്രം മാത്രം എടുത്തുദ്ധരിക്കുകയും ചെയ്ത അതിമാനുഷനാണ് അദ്ദേഹമെന്ന് പറയേണ്ടി വരും. സര്‍വ്വശക്തനായ സ്രഷ്ടാവിന്റെ വചനങ്ങളാണ് ഖുര്‍ആനിലുള്ളതെന്ന വസ്തുത നിഷേധിക്കു വാന്‍ തെളിവ് പരതുന്നവര്‍ മുഹമ്മദ് നബി (സ)യെ ദൈവമാക്കുന്ന പരിണാമഗുപ്തിയിലാണ് എത്തിച്ചേരുകയെന്നര്‍ത്ഥം.

ആറ്) ബൈബിളില്‍ പറയാത്ത ചില പ്രവാചകന്മാരുടെയും സമുദായങ്ങളുടെയും ചരിത്രം ഖുര്‍ ആന്‍ വിവരിക്കുന്നുണ്ട്. ആദ്, സമൂദ് ഗോത്രങ്ങളിലേക്ക് നിയോഗിക്കപ്പെട്ട ഹൂദ് നബിയുടെയും സാലിഹ് നബിയുടെയും ചരിത്രം ഉദാഹരണം. ബൈബിളിലെവിടെയും കാണാനാവാത്ത പ്രവാച കന്മാരാണിവര്‍. ബൈബിളില്‍നിന്ന് കോപ്പിയടിക്കുകയാണ് മുഹമ്മദ് നബി (സ)ചെയ്തതെങ്കില്‍ ഈ ചരിത്ര ങ്ങള്‍ അദ്ദേഹത്തിന് എവിടെനിന്നാണ് കിട്ടിയത്?

ഏഴ്) ബൈബിളില്‍ പരാമര്‍ശിക്കപ്പെട്ട പ്രവാചകന്മാരുടെ ചരിത്രം പറയുമ്പോള്‍തന്നെ ബൈബിളി ലൊരിടത്തും പരാമര്‍ശിക്കാത്ത നിരവധി സംഭവങ്ങള്‍ ഖുര്‍ആനില്‍ വിശദീകരിക്കുന്നുണ്ട്. നൂഹ് നബി (അ) യും അവിശ്വാസിയായ മകനും തമ്മില്‍ നടന്ന സംഭാഷണവും മകന്‍ പ്രളയത്തില്‍പെട്ട സംഭവവിവരണവും സൂറത്തു ഹൂദില്‍ (11:42-46) കാണാം. ഇങ്ങനെ യാതൊന്നും ബൈബിളിലെ വിടെയുമില്ല. ഇബ്രാഹീം നബിയും നംറൂദും തമ്മില്‍ നടന്ന സംവാദവും (ഖുര്‍ആന്‍ 2:258) പിതാവു മായി നടന്ന സംഭാഷണവും (ഖുര്‍ആന്‍ 6:74, 19:41-49, 43:26,27) മരണാനന്തര ജീവിതത്തിന്റെ സത്യത ബോധ്യപ്പെടുന്നതിനായി, പക്ഷികളെ കഷ്ണിച്ച് നാല് മലകളില്‍വെച്ചശേഷം അവയെ വിളിച്ചാല്‍ അവ ഓടിവരുന്നതാണെന്ന് അല്ലാഹു അദ്ദേഹത്തോട് പറഞ്ഞ സംഭവവും (2:260) തീയിലേക്ക് വലി ച്ചെറിയപ്പെടുകയും അതില്‍ നിന്ന് അദ്ദേഹം അത്ഭുതകരമായി രക്ഷപ്പെടുകയും ചെയ്ത ചരിത്ര വു(21:56-70)മൊന്നും ബൈബിളിലൊരിടത്തും കാണാന്‍ കഴിയില്ല. ദൈവിക കല്‍പന പ്രകാരം ഒരു പശുവിനെ അറുക്കാന്‍ മൂസാ (അ) ഇസ്രായീല്യരോട് നിര്‍ദേശിക്കുകയും, പശുവിന്റെ പ്രത്യേക തകള്‍ ചോദിച്ച് അതിന്റെ നിര്‍വ്വഹണം അവര്‍ പ്രയാസകരമാക്കുകയും ചെയ്ത സംഭവവും (ഖുര്‍ആന്‍ 2:67-71) കൊലപാതകക്കുറ്റം തെളിയിക്കാനായി പശുവിനെ അറുത്ത് അതിന്റെ ഒരുഭാ ഗംകൊണ്ട് അടിക്കുവാന്‍ കല്‍പിച്ച കഥനങ്ങളും (2:72, 73) മൂസാ നബി ((അ))യുടെ ജീവിതവുമായി ബന്ധപ്പെടുത്തി ബൈബിളിലൊരിടത്തും പ്രസ്താവിക്കുന്നില്ല. ഈസാ നബി (അ) യുടെ ജനനം മുതല്‍ തന്നെയുള്ള ബൈബിളില്‍ പറയാത്ത പല സംഭവങ്ങളും ഖുര്‍ആനില്‍ പരാമര്‍ശിക്കുന്നുണ്ട്. സകരി യ്യായുടെ സംരക്ഷണത്തില്‍ പ്രാര്‍ത്ഥനാസ്ഥലത്ത് താമസിച്ചുകൊണ്ടിരുന്ന മര്‍യമിന്റെ കുട്ടിക്കാ ലത്ത് അവര്‍ക്ക് അത്ഭുതകരമായി ഭക്ഷണസാധനങ്ങള്‍ ലഭിച്ച സംഭവം (ഖുര്‍ആന്‍ 3:37), മര്‍യമിന്റെ പ്രസവസമയത്ത് അവര്‍ക്ക് നല്‍കപ്പെട്ട പ്രത്യേക അനുഗ്രഹങ്ങളെക്കുറിച്ച വിവരണം (19:23-26), ഈസാ (അ) തൊട്ടിലില്‍വെച്ച് സംസാരിച്ച് തന്റെ നിയോഗം പ്രഖ്യാപിച്ചുകൊണ്ട് തന്റെ ആദ്യത്തെ അത്ഭുതം പ്രവര്‍ത്തിച്ച ചരിത്രം (19:29,30), കളിമണ്ണുകൊണ്ട് പക്ഷിയുടെ രൂപമുണ്ടാക്കി ഈസാ (അ) അതില്‍ ഊതിയപ്പോള്‍ അല്ലാഹുവിന്റെ അനുമതിപ്രകാരം അതൊരു പക്ഷിയായി രൂപാന്തരപ്പെട്ട സംഭവം (3:49) ഇതൊന്നുംതന്നെ ബൈബിളില്‍ ഒരിടത്തും പരാമര്‍ശിക്കുന്നുപോലുമില്ല.

ബൈബിളി ല്‍നിന്ന് മുഹമ്മദ് നബി (സ) പകര്‍ത്തിയെഴുതിക്കൊണ്ടാണ് ഖുര്‍ആന്‍ രചിച്ചതെങ്കില്‍ ബൈബിളി ലൊരിടത്തും പരാമര്‍ശിക്കാത്ത പ്രവാചകന്മാരുടെ ജീവിതവുമായി ബന്ധപ്പെട്ട കഥകള്‍ അദ്ദേഹ ത്തിന് എവിടെനിന്നുകിട്ടി? സത്യത്തില്‍ ഖുര്‍ആന്‍ ദൈവ വചനമായതുകൊണ്ടാണ് ബൈബിളിലെ വിടെയും സൂചിപ്പിക്കാത്ത സംഭവങ്ങള്‍പോലും അതില്‍ നമുക്ക് കാണാന്‍ കഴിയുന്നത്. മര്‍യത്തി ന്റെ ബാല്യകാല സംഭവങ്ങള്‍ വിവരിക്കവെ ഖുര്‍ആന്‍ പറഞ്ഞത് എത്ര ശരി! ''(നബിയേ) നാം നിനക്ക് ബോധനം നല്‍കുന്ന അദൃശ്യവാര്‍ത്തകളില്‍ പെട്ടതാകുന്നു അവയൊക്കെ. അവരില്‍ ആരാണ് മര്‍യത്തിന്റെ സംരക്ഷണം ഏറ്റെടുക്കേണ്ടതെന്ന് തീരുമാനിക്കുവാനായി അവര്‍ തങ്ങളുടെ അമ്പുകള്‍ ഇട്ടുകൊണ്ട് നറു ക്കെടുപ്പ് നടത്തിയിരുന്ന സമയത്ത് നീ അവരുടെ അടുത്തുണ്ടായിരുന്നി ല്ലല്ലോ. അവര്‍ തര്‍ക്കത്തിലേര്‍പ്പെട്ടുകൊണ്ടിരുന്നപ്പോഴും നീ അവരുടെ അടുത്തുണ്ടായിരുന്നില്ല'' (വി.ഖു. 3:44).

എട്ട്) ബൈബിളില്‍ പരാമര്‍ശിക്കപ്പെട്ട കഥകള്‍ പറയുമ്പോഴും ബൈബിളില്‍നിന്ന് വ്യത്യസ്തമായി കൃത്യതയും സൂക്ഷ്മതയും ഖുര്‍ആന്‍ കാത്തുസൂക്ഷിക്കുന്നത് കാണാം. ഉദാഹരണത്തിന് മോശ-സീനായ് പര്‍വതത്തിലേക്ക് പോയ അവസരത്തില്‍ ഇസ്രായീല്യര്‍ക്ക് അവരുടെ ആവശ്യപ്രകാരം സ്വര്‍ണംകൊണ്ട് കാളക്കുട്ടിയെ നിര്‍മിച്ച് ആരാധനക്കായി നല്‍കിയത് മോശയുടെ കൂട്ടാളിയും പ്രവാ ചകനുമായ അഹരോണായിരുന്നുവെന്നാണ് പുറപ്പാട് പുസ്തകം (32:1-6) പറയുന്നത്. ഖുര്‍ആനും ബൈബിളുമെല്ലാം പരിശുദ്ധ പ്രവാചകനായി പരിചയപ്പെടുത്തുന്ന ഹാറൂനി((അ))ല്‍ നിന്ന് വിഗ്ര ഹാരാധനക്ക് കൂട്ടുനില്‍ക്കുകയെന്ന മഹാപാപം സംഭവിക്കാനിടയില്ലെന്ന് ഏത് സാമാന്യ ബുദ്ധി ക്കും മനസ്സിലാവും. ഖുര്‍ആനും പ്രസ്തുത സംഭവം വിവരിക്കുന്നുണ്ട്. പക്ഷെ, സ്വര്‍ണപശുവിനെ യുണ്ടാക്കുകയും അതിനെ ആരാധിക്കുവാന്‍ ഇസ്രായീല്യരെ പ്രേരിപ്പിക്കുകയും ചെയ്തത് ഹാറൂ ന(അ)ല്ല; പ്രത്യുത ഇസ്രായീല്യരില്‍പെട്ട ഒരു കപടനായ സാമിരിയാണ് ഇത് ചെയ്തതെന്നും അതു മൂലം അയാള്‍ ദൈവകോപത്തിനും ശപിക്കപ്പെട്ട രോഗത്തിനും വിധേയനായെന്നുമാണ് ഖുര്‍ആന്‍ പഠിപ്പിക്കുന്നത് (20:85-95). ബൈബിളില്‍ പരാമര്‍ശിക്കപ്പെട്ട കഥകള്‍ വിവരിക്കുമ്പോഴും അതിലെ നെല്ലും പതിരും വേര്‍ തിരിച്ച് സത്യസന്ധവും സൂക്ഷ്മവുമായ രീതിയില്‍ അവ ജനസമക്ഷം വെക്കു ന്ന ഖുര്‍ആന്‍ ദൈവികമാണെന്ന് അതിന്റെ ഈ പ്രത്യേകത തന്നെ സുതരാം വ്യക്തമാക്കുന്നു.

ബ്രഹാമിന്റെ ചരിത്രം പറയുന്നിടത്ത് ബൈബിളിലും ഖുർആനിലും ഏകദേശം സമാനമായ കഥകളാണുള്ളതെങ്കിലും ഈ രണ്ട് ഗ്രന്തങ്ങളിലെയും പ്രതിപാദനരീതികൾ തമ്മിൽ സാരമായ ചില അന്തരങ്ങളുണ്ട്. ബൈബിളിലെ ചരിത്ര വിശദീകരണത്തിലുടനീളം ഇസ്രായേലീ വംശീയത നിഴലിക്കുന്നതായി കാണാന്‍ കഴിയും. .അബ്രഹാമിന്റെ ചരിത്രകഥനത്തിൽ ഇത് വ്യക്തമായി കാണാനാവും. അദ്ദേഹത്തിന്റെ പുത്രന്മാരായ യിശ്മായേലിനെയും ഇസഹാഖി നെയും കുറിച്ചു വിവരിക്കുന്ന ഉല്‍പത്തി പുസ്തകഭാഗങ്ങളില്‍ യഹൂദ വംശീയതയുടെയും അടിമ കളോടുള്ള അവരുടെ ക്രൂരമായ പെരുമാറ്റത്തിന്റെയും പ്രതിബിംബമാണ് കാണാന്‍ കഴിയുന്നത്.

അബ്രഹാമിന് തന്റെ ആദ്യഭാര്യയായ സാറായില്‍ സന്താന സൗഭാഗ്യമില്ലായിരുന്നതിനാല്‍ സാറാ തന്നെ തന്റെ അടിമയായിരുന്ന ഹാഗാറിനെ അദ്ദേഹത്തിന് ഭാര്യയായി നല്കുികയാണുണ്ടായത്. 'ഭാര്യയായ സാറായ് തന്റെ ഈജിപ്തുകാ രിയായ ദാസി ഹാഗാറിനെ ഭര്ത്താാവായ അബ്രാമിന്ന് ഒരു ഭാര്യയായി കൊടുത്തു. അബ്രാം ഹാഗാറിനെ പ്രാപിച്ചു. അവള്‍ ഗര്ഭിരണിയായി' (ഉല്പതത്തി16:3). അബ്രഹാമില്‍ നിന്ന് ഗര്ഭിണിയായ ഹാഗാര്‍ സാറായോട് അപമര്യാദയായി പെരുമാറിയപ്പോള്‍ ഹാഗാറിനെ ക്രൂരമായി മര്ദിാക്കുവാനും ഗര്ഭി്ണിയായ അവളെ വീട്ടില്‍ നിന്നും ഓടിക്കുവാനും സാറായെ അബ്രഹാം അനുവദിച്ചുവെന്നാണ് ഉല്പ ത്തി പുസ്തകത്തില്‍ നാം വായിക്കുന്നത്. 'അബ്രാം സാറായിയോട് പറഞ്ഞു:' നോക്കൂ, നിന്റെ ദാസി നിന്റെ കീഴില്‍ തന്നെ യാണ്. നിനക്ക് ഇഷ്ടമുള്ള വിധം നീ അവളോട് പെരുമാറിക്കൊള്ളുക! തുടര്ന്നു സാറായ് ഹാഗാറി നോട് ക്രൂരമായി പെരുമാറി. ഹാഗാര്‍ അവിടെ നിന്ന് ഓടിപ്പോയി' (ഉല്പരത്തി 16:6). തന്നില്‍ നിന്ന് ഗര്ഭിാണി യായ സ്ത്രീയെ അവര്‍ അടിമയായി എന്ന കാരണത്താല്‍ അടിച്ചോടി ക്കുവാന്‍ കൂട്ടുനി ല്ക്കുന്ന അബ്രഹാമിന്റെ ചിത്രം നിര്മിറക്കുക വഴി അടിമസ്ത്രീകളോട് ഏതു തരം ക്രൂരതയും പ്രവര്ത്തിാക്കുവാനുള്ള നിയമം നിര്മിനക്കുവാനാണ് യഹൂദ റബ്ബിമാര്‍ ശ്രമിച്ചിരിക്കുന്നത്.

തങ്ങളുടെ ക്രൂരമായ ജീവിതരീതിക്കനുസൃതമായി മഹാന്മാരുടെ ജീവിതത്തെ മാറ്റിയെഴുതിയ പ്പോഴുണ്ടായ വൈരുദ്ധ്യങ്ങള്‍ ചരിത്ര മെഴുത്തിലുടനീളം കാണന്‍ കഴിയും. വാര്ദ്ധ്ക്യകാലത്തു ണ്ടായ തന്റെ പുത്രന്‍ യിശ്മായേലിനെയും ഭാര്യ ഹാഗാറിനെയും മരുഭൂമിയില്‍ തനിച്ചാക്കുവാ നുള്ള ദൈവിക കല്പഭന നിറവേറ്റിയ മഹാനാണ് വിശുദ്ധ ഖുര്ആചന്‍ പരിചയപ്പെടുത്തുന്ന ഇബ്രാഹീം (അ). ഉല്പാത്തി പുസ്തകമാകട്ടെ ഈ സംഭവത്തെയും ഹാഗാര്‍ എന്ന അടിമസ്ത്രീ യെയും പുത്രനെയും പീഡിപ്പിക്കാനുള്ള സാറായുടെ ശ്രമത്തിന് അബ്രഹാം കൂട്ടുനിന്നതിന് ഉദാ ഹരണമായിട്ടാണ് ഉദ്ധരിക്കുന്നത്. ഇരുപത്തൊന്നാം അധ്യായം നോക്കുക: 'ശിശു വളര്ന്നു . ഇസഹാ ഖിന്റെ മുലകുടി മാറ്റിയ ദിവസം അബ്രഹാം വലിയൊരു വിരുന്നു നല്കി്. ഈജിപ്റ്റുകാരിയായ ഹാഗാറില്‍ അബ്രഹാമിന് ജനിച്ച പുത്രന്‍ തന്റെ പുത്രനായ ഇസഹാഖിനോടൊന്നിച്ചു കളിക്കുന്നത് സാറാ കണ്ടു. സാറാ അബ്രഹാമിനോടുപറഞ്ഞു: 'ഈ അടിമപ്പെണ്ണിനെയും പുത്രനെയും അടിച്ചു പുറത്താക്കുക. ഈ അടിമപ്പെണ്ണിന്റെ പുത്രന്‍ എന്റെ പുത്രനായ ഇസഹാഖിന്നൊപ്പം അവകാശി യായി ത്തീര്ന്നുണ കൂടാ'. യിശ്മായേലും തന്റെ പുത്രനാകയാല്‍ ഇക്കാര്യം അബ്രഹാമിന് ഏറെ അനിഷ്ടമായി. എന്നാല്‍ ദൈവം അബ്രഹാമി നോട് അരുള്‍ ചെയ്തു. 'അടിമപ്പെണ്ണിനേയും കുട്ടി യേയും പ്രതി നീ അനിഷ്ടം വിചാരിക്കേണ്ട. സാറാ പറയുന്നതു പോലെ പ്രവര്‍ത്തിക്കുക. ഇസ ഹാഖിലൂടെയായിരിക്കും നിന്റെ സന്തതിപരമ്പര അറിയപ്പെടുക. അടിമപ്പെണ്ണിന്റെ പുത്രനെയും ഞാന്‍ ഒരു ജനത യാക്കും. അയാളും നിന്റെ സന്തതിയാണല്ലോ'. അബ്രഹാം അതി രാവിലെ എഴു ന്നേറ്റ് അപ്പവും ഒരു തുരുത്തിവെള്ളവും ഹാഗാറിനെ ഏല്പിെച്ചു. കുട്ടിയെ തോളില്വെനച്ച് അവളെ പറഞ്ഞയച്ചു.അവര്‍ അവിടം വിട്ടു, ബേര്ശേ്ബ മരുഭൂമിയില്‍ അലഞ്ഞു നടന്നു' (ഉല്പ.ത്തി 21:8-14).

യഥാര്ത്ഥ ത്തില്‍ യിശ്മായേലിനെ മരുഭൂമിയില്‍ ഉപേക്ഷിച്ച സംഭവം നടക്കുന്നത് ഇസ്ഹാഖിന്റെ ജനനത്തിനു മുമ്പാണ്. ഇസ്ഹാഖിനുള്ള ഔന്നത്യത്തിന്റെയും അടിമസ്ത്രീയോടും മകനോടും ചെയ്യാവുന്ന ക്രൂരതകളുടെയും തെളിവായുദ്ധരിക്കുന്ന തിന്നു വേണ്ടിയാണ് പ്രസ്തുത സംഭവത്തെ ഇസ്ഹാഖിന്റെ ജനനത്തിന് ശേഷത്തേക്ക് വലിച്ചിഴച്ചത്. യിശ്മയേല്‍ വളരെ ചെറിയ ഒരു കുഞ്ഞായിരുന്ന കാലത്താണ് ഈ സംഭവം നടന്നതെന്ന് മുകളി ലുദ്ധരിച്ച ഉല്പനത്തി വചനത്തില്‍ നിന്നു വ്യക്തമാണ്. 'കുട്ടിയെ തോളില്‍ വെച്ച് അവളെ പറഞ്ഞയച്ചു' (21:14) വെന്ന് പറഞ്ഞതില്‍ നിന്ന് തോളത്ത് വെക്കാന്‍ മാത്രമേ യിശ്മായിലിന് അന്ന് പ്രായമു ണ്ടായിരുന്നുള്ളൂവെന്ന് മനസ്സിലാ കുന്നു.'തുരുത്തിയിലെ വെള്ളം തീര്ന്നലപ്പോള്‍ അവള്‍ കുട്ടിയെ ഒരു കുറ്റിക്കാട്ടില്‍ ഉപേക്ഷിച്ചു. അവള്‍ അവിടെ നിന്ന് കുറച്ചകലെ, അതായത് ഏകദേശം ഒരു വില്പാളടു ദൂരെ മാറി പുറം തിരിഞ്ഞിരുന്ന്. 'എനിക്ക് കുഞ്ഞിന്റെ മരണം കാണേണ്ട' എന്നു പറഞ്ഞു. അവള്‍ ദൂരെ മാറി പിന്തി രിഞ്ഞിരുന്നപ്പോള്‍ കുട്ടി ഉറക്കെ കരഞ്ഞു' (ഉല്പ‍ത്തി 21:15,16). ഈ വചനങ്ങളെല്ലാം വളരെ ചെറിയ ഒരു ശിശുവായിരിക്കുമ്പോഴാണ് യിശ്മായേല്‍ തന്റെ മാതാവിനോടൊപ്പം മരുഭൂമിയില്‍ ഉപേക്ഷി ക്കപ്പെട്ടതെന്ന് വ്യക്തമാക്കുന്നു.

'ഹാഗാര്‍ യിശ്മായിലിനെ പ്രസവിച്ചപ്പോള്‍ അബ്രഹാമിന് എണ്പെ ത്താറ് വയസ്സായിരുന്നു' (ഉല്പപത്തി 16:16). 'പുത്രനായ ഇസ്ഹാഖ് പിറന്നപ്പോള്‍ അബ്രഹാമിന്ന് നൂറ് വയസ്സായിരുന്നു' (ഉല്പപത്തി 21:5). ഇതില്‍ നിന്ന് ഇസ്ഹാഖ് ജനിക്കുമ്പോള്‍ യിശ്മായേലിന് പതിനാലു വയസ്സ് പ്രായമായിരു ന്നുവെന്ന് മനസ്സിലാക്കാം. ഇസ്ഹാഖിന്റെ മുലകുടി മാറിയ ദിവസമാണ് ഉല്പനത്തി പുസ്തകം പറയുന്നതുപോലെ യിശ്മായിലിനെയും മാതാവിനെയും മരുഭൂമിയില്‍ ഉപേക്ഷിച്ചതെങ്കില്‍ അന്ന് യിശ്മായീല്‍ പതിനാറു വയസ്സു പ്രായമുള്ളയാളായിരി ക്കണം. ഒരു പതിനാറു വയസ്സുകാരനെ മാതാവ് തോളില്‍ വെക്കുമോ? മരുഭൂമിയില്‍ ഉപേക്ഷിക്ക പ്പെട്ട യിശ്മായേലിന്റെ ചിത്രം ഉല്പ?ത്തിപുസ്തകം ഇരുപത്തി യൊന്നാം അധ്യായപ്രകാരം ഒരു പതിനാറുകാരന്‍േറതല്ലെന്നുറപ്പാണ്. ഈ വൈരുദ്ധ്യത്തിനുള്ള കാരണം യഹൂദ റബ്ബിമാരുടെ കൈക്രിയകളാണ്. സ്വന്തം ഇച്ഛയ്ക്കനുസരിച്ച് പ്രവാചകന്മാരുടെ ചരിത്രം മാറ്റിയെഴുതിയപ്പോള്‍ സ്വാഭാവികമായുണ്ടായ വൈരുദ്ധ്യങ്ങളെക്കുറിച്ച് റബ്ബിമാര്‍ അജ്ഞരായിരുന്നുവെന്നു വേണം മനസ്സിലാക്കുവാന്‍.

സ്വപുത്രനെ ബലിയറുക്കുവാനുള്ള ദൈവകല്പഞന നിറവേറ്റുവാന്‍ സന്നദ്ധനായ അബ്രഹാമിനെ ക്കുറിച്ച് വിവരിക്കുന്നിടത്തും ഈ വൈരുദ്ധ്യം പ്രകടമാവുന്നുണ്ട്. അവ ഇസ്രായേല്യരില്‍ മാത്രമേ ദൈവാനുഗ്രഹമുണ്ടായിട്ടുള്ളുവെന്ന് സ്ഥാപിക്കുന്നതിന് വേണ്ടി ബലികര്മ്മഹത്തെ മാറ്റിയെഴുതി യതുകൊണ്ടുണ്ടായതാണ്. വാര്ധ്ികക്യത്തില്‍ ഇബ്രാഹീമിന് ആദ്യമുണ്ടായ പുത്രനെ ബലിയറു ക്കുവാന്‍ കല്പി്ച്ചു കൊണ്ട് അദ്ദേഹത്തെ അല്ലാഹു പരീക്ഷിച്ചുവെന്നും ത്യാഗങ്ങളുടെ തീച്ചൂളയി ലൂടെ ജീവിച്ചു വളര്ന്ന് മഹാനായ ഇബ്രാഹീം പ്രസ്തുത പരീക്ഷണത്തില്‍ വിജയിച്ചുവെന്നുമാണ് ഖുര്ആഹന്‍ പഠിപ്പിക്കുന്നത്. ബലിയറുക്കുവാന്‍ ദൈവം കല്പിാച്ചത് അബ്രഹാമി ന്റെ ആദ്യത്തെ പുത്രനെത്തന്നെയായിരുന്നുവെന്നാണ് ഉല്പിത്തി പുസ്തകത്തില്‍ നിന്ന് മനസ്സിലാവുന്നത്. സ്വന്തം പുത്രനെ അറുക്കുവാന്‍ വേണ്ടി കൈകാലുകള്‍ കെട്ടി ബലിപീഠത്തിന് മുകളില്‍ കിട ത്തിക്കൊണ്ട് കത്തി എടുത്ത സമയത്ത് ആകാശത്തു നിന്ന് കര്ത്താ്വിന്റെ മാലാഖ പറഞ്ഞതായി ഉല്പ്ത്തി പുസ്തകം ഉദ്ധരിക്കുന്നത് നോക്കുക: 'കുട്ടിയുടെ മേല്‍ കൈവെക്കരുത്. അവനെ ഒന്നും ചെയ്യരുത്. നിന്റെ പുത്രനെ, നിന്റെ ഏകജാതനെ തന്നെ തരുവാന്‍ നീ വൈമനസ്യം കാണിക്കായ്കയാല്‍, നീ ദൈവത്തെ ഭയപ്പെടുന്നു എന്ന് ഞാന്‍ ഇപ്പോള്‍ അറിയുന്നു (ഉല്പനത്തി 22:12). 'നിന്റെ ഏകജാതനെ' (Your only son) എന്നാണ് മാലാഖ പറയുന്നത് എന്ന കാര്യം ശ്രദ്ധിക്കുക. ബലിയറുക്കുവാനായി കല്പിളക്കപ്പെട്ട കാലത്ത് അബ്രഹാമിന് ഒരൊറ്റപുത്രന്‍ മാത്രമേയുണ്ടായിരുന്നുള്ളുവെന്നാണല്ലോ ഇതില്‍ നിന്ന് മനസ്സിലാക്കേണ്ടത്. ആദ്യപുത്രനായ യിശ്മായേലിനെയല്ലാതെ ദ്വിതീയനായ ഇസ്ഹാ ഖിനെ ബലിയറുക്കാനാണ് കല്പുനയുണ്ടായതെങ്കില്‍ 'നിന്റെ ഏകജാതനെ' എന്നു മാലാഖ പറയു മായിരുന്നുവോ?

യഥാര്ത്ഥ ത്തില്‍ ബലിയറുക്കുവാനുള്ള ദൈവകല്പപനയില്‍ നിന്നു തന്നെ ഇക്കാര്യം വ്യക്തമായും മനസ്സിലാകുന്നുണ്ട്. ' ദൈവം കല്പിിച്ചു: 'നിന്റെ പുത്രനെ, നീ അത്യധികം സ്‌നേഹിക്കുന്ന ഏകജാ തനായ ഇസ്ഹാഖിനെ, കൂട്ടിക്കൊണ്ട് മോറിയാ ദേശത്തേക്കു പോകുക. അവിടെ ഞാന്‍ കല്പിക്കു ന്ന മലയില്‍ അവനെ എനിക്കു ഹോമിക്കുക' (ഉല്പ ത്തി 22:2).ഇവിടെ, ഏകജാതനായ ഇസ്ഹാഖി' നെ എന്നാണ് ദൈവകല്പ്നയിലുള്ളത്. ഇസഹാഖ് എങ്ങനെയാണ് ഏകജാതനാകുന്നത്? അദ്ദേഹം അബ്രഹാമിന്റെ ദ്വിതീയ പുത്രനാണല്ലോ. ഇവിടെ, ഈ കല്പേനയില്‍ 'ഇസ്ഹാഖിനെ'യെന്ന് യഹൂദ റബ്ബിമാര്‍ കൂട്ടിച്ചേര്ത്തിതാണെന്നാണ് മനസ്സിലാക്കാന്‍ കഴിയുന്നത്. അടിമസ്ത്രീയില്‍ ജനിച്ച മക്കള്‍ സ്വന്തം പുത്രന്മാരായി അറിയപ്പെടുന്നത് അപമാനമായി കരുതപ്പെട്ടിരുന്ന യഹൂദ പാരമ്പര്യ ത്തിന് അനു സൃതമായ രീതിയില്‍ അബ്രാഹാമിന്റെ ചരിത്രം വളച്ചൊടിക്കപ്പെട്ട പ്പോഴാണ് 'ഇസ്ഹാഖ്' ഏകജാതനായി മാറിയത്.

അടിമ സ്ത്രീയിലുണ്ടായ പുത്രനെ അബ്രഹാം മകനായി ത്തന്നെ പരിഗണിച്ചിരുന്നില്ല എന്നാണല്ലോ ഇതില്‍ നിന്ന് മനസ്സിലാക്കേണ്ടത്.എന്നാല്‍ ഉല്പത്തി പുസ്തകം തന്നെ നല്കുുന്ന അബ്രഹാമിന്റെ ചരിത്രവുമായി ഇത് വ്യക്തമായ വൈരുദ്ധ്യം പ്രകടി പ്പിക്കുന്നു. അബ്രഹാം യിശ്മായിലിനെ പുത്രനായിത്തന്നെയാണ് പരിഗണിച്ചിരു ന്നത് എന്നാണ് ഉല്പമത്തി പുസ്തകത്തില്‍ നിന്ന് മനസ്സിലാകുന്നത്. ജനനം മുതല്‍ (16:15,16) പരിഛേ ദനയിലും മറ്റു കര്മ്മണങ്ങളിലും (17:23) പ്രാര്ത്ഥ്നയിലും (17:20) അങ്ങനെ സകലവിധ കാര്യങ്ങ ളിലും യിശ്മാ യിലിനെ സ്വപുത്രനായിത്തന്നെയാണ് അബ്രഹാം പരിഗണിച്ചിരുന്നത് എന്നു തന്നെ യാണ് ഉല്പ ത്തിപുസ്തകത്തിലുള്ളത്. മാത്രവു മല്ല, ഇഷ്ടഭാര്യയില്‍ ജനിച്ച രണ്ടാമത്തെപുത്രന് അനിഷ്ടഭാര്യ യില്‍ ജനിച്ച ആദ്യപുത്രന് നല്കേചണ്ട അവകാശം നല്കുളന്നത് പഴയനി യമപ്രകാരം പാപമാണ്. ആവര്ത്തണന പുസ്തകത്തിലെ ദൈവകല്‍ പന നോക്കുക: 'ഒരാള്ക്ക് രണ്ടു ഭാര്യമാര്‍ ഉണ്ട് എന്നും അവരില്‍ ഒരുവളെ അയാള്‍ സ്‌നേഹിക്കുകയും മറ്റവളെ വെറുക്കുകയും ചെയ്യുന്നു വെന്നും കരു തുക. സ്‌നേഹിക്കുകയും വെറുക്കുകയും ചെയ്യുന്ന ഭാര്യമാരില്‍ അയാള്ക്ക് സന്താന ങ്ങള്‍ ജനിക്കു കയും ആദ്യജാതന്‍ അയാള്‍ വെറുക്കുന്ന ഭാര്യയില്‍ നിന്നു ജനിക്കയും ചെയ്താല്‍, തന്റെ സ്വത്തു ക്കള്‍ സന്താനങ്ങള്ക്ക്് അവകാശമായി ഭാഗിച്ചു കൊടുക്കുമ്പോള്‍, വെറുക്കു ന്നവളില്‍ നിന്ന് ജനിച്ച വനും ആദ്യജാതനുമായവനു പകരം സ്‌നേഹിക്കുന്നവളിലുണ്ടായ പുത്രനെ ആദ്യജാതനായി അയാ ള്‍ പരിഗണിക്കരുത്. അയാള്‍ തനിക്കുള്ള എല്ലാ സ്വത്തില്‍ നിന്നും ഇരട്ടി ഓഹരി നല്കിയ വെറുക്കു ന്ന ഭാര്യയുടെ പുത്രനെ ആദ്യജാതനായി അംഗീകരിക്കണം. അയാളുടെ വീര്യത്തിന്റ ആദ്യഫലം ആ പുത്രനാണല്ലോ. ആദ്യജാതനുള്ള അവകാശം ആ പുത്രനുതന്നെ' (ആവര്ത്തണനം 21:15-17).

ഹാഗാര്‍ അടിമസ്ത്രീയായിരുന്നതിനാല്‍ അബ്രാഹാമിനാല്‍ വെറു ക്കപ്പെട്ടവളായിരുന്നുവെന്ന യഹൂദ റബ്ബി മാരുടെ വാദം അംഗീകരിച്ചാ ല്തോന്നെ, മുകളില്‍ വിവരിച്ച ദൈവകല്പാനപ്രകാരം ആദ്യജാതനു ള്ള അവകാശത്തിന് അര്ഹ,ന്‍ യിശ്മായേല്‍ തന്നെയാണെന്ന് വ്യക്തമാണ്. ഇസ്ഹാഖിനെ അബ്ര ഹാമിന്റെ ഏകജാതനായി പരിചയപ്പെടുത്തുക വഴി ഉല്പഅത്തി 22:2 പ്രകാരം ദൈവംതന്നെ യിശ്മായേല്‍ അബ്രഹാമിന്റെ പുത്രനല്ലെന്ന് അംഗീകരിച്ചുവെന്ന് പറയേണ്ടിവരും.

ഈ വൈരുദ്ധ്യത്തിനുള്ള കാരണം തങ്ങളുടെ വംശീയ ദുരഭിമാ നത്തിന് മാറ്റുകൂട്ടുവാന്‍ വേണ്ടി പ്രവാചകചരിത്രത്തില്‍ യഹൂദ റബ്ബിമാര്‍ നടത്തിയ കൈക്രിയകളാണ്. അബ്രഹാമിന്റെ ദ്വിതീയ പുത്രനായ ഇസ്ഹാഖിന്റെ മകനായ യാക്കോബിന്റെ പുത്രന്മാരാണ് ഇസ്രായേല്യര്‍. ദൈവികമായ സകല അനുഗ്രഹങ്ങളും വര്ഷിുക്കപ്പെട്ടിരിക്കുന്നതും വര്ഷിമക്കപ്പെടാന്പോ്വുന്നതും ഇസ്രായീ ല്യര്ക്കി ടയില്‍ മാത്രമാണെന്നായിരുന്നു യഹൂദ റബ്ബിമാര്‍ പ്രചരിപ്പിച്ചിരുന്നത്. സ്വപുത്രനെ ബലിയ റുക്കുവാനുള്ള കല്പയനയനുസരിച്ചുകൊണ്ട് ദൈവാനുഗ്രഹത്തിന് പാത്രമായ അബ്രഹാമിന് ദൈ വം നല്കു്ന്ന വരദാനങ്ങളെക്കുറിച്ച് ഉല്പരത്തി പുസ്തകത്തിലുണ്ട്. അതിങ്ങനെയാണ് 'നീ ഇതു ചെയ്തിരിക്കയാല്‍, നിന്റെ പുത്രനെ, നിന്റെ ഏകജാതനെ തരാന്‍ നീ മടിക്കായ്കയാല്‍ എന്നാണെ, ഞാന്‍ ഇങ്ങനെ പ്രതിജ്ഞ ചെയ്യുന്നു- ഇതു കര്ത്തായവാണ് അരുള്‍ ചെയ്യുന്നത്: ഞാന്‍ നിന്നെ സമൃ ദ്ധമായി അനുഗ്രഹിക്കും. നിന്റെ സന്തതിയെ ഞാന്‍ ആകാശത്തിലെ നക്ഷത്രങ്ങളെപ്പോലെയും കട ല്ക്ക്രയിലെ മണല്ത്ത്രി കളെപ്പോലെയും അത്യധികം വര്ദ്ധിിപ്പിക്കും. നിന്റെ സന്തതികള്‍ ശത്രുക്ക ളുടെ പട്ടണവാതിലുകള്‍ കൈവശപ്പെടുത്തും. നീ എന്റെ വാക്ക് അനുസരിച്ചതിനാല്‍ നിന്റെ സന്ത തികളിലൂടെ ഭൂമിയിലെ എല്ലാ ജനതകളും അനുഗ്രഹിക്കപ്പെടും'(ഉല്പതത്തി 22:16-19). യിശ്മാ യേലാണ് ബലിയറുക്കപ്പെടാനായി കല്പിെക്കപ്പെട്ടതെങ്കില്‍ ഈ അനുഗ്രഹങ്ങള്‍ മുഴുവനുമുണ്ടാ വുക യിശ്മായേല്‍ സന്തതികളിലാണെന്നു വരും. യഹൂദന്മാരാകട്ടെ യിശ്മായീലിന്റെ അനുജസ ഹോദരനായ ഇസ്ഹാഖിന്റെ സന്തതി പരമ്പരകളിലാണ് ഉള്പ്പെജടുന്നത്. തങ്ങളിലല്ലാതെ ദൈവാ നുഗ്രഹമു ണ്ടാവുകയെന്നത് ഇസ്രാഈല്യര്ക്ക്പ ഒരു കാരണവശാലും അംഗീകരിക്കാന്‍ പറ്റാത്ത കാര്യമായിരുന്നു. അതുകൊണ്ടു കൂടിയാണ് യഹൂദ റബ്ബിമാര്‍ ഇസ്ഹാഖിനെ അബ്രഹാമിന്റെ ഏകജാതനാക്കി മാറ്റിയത്.

തങ്ങളുടെ വംശീയ ദുരഭിമാനത്തിന്റെ സംരക്ഷണത്തിനു വേണ്ടി എന്തും ചെയ്യാന്‍ മടിയില്ലാത്ത വരും അടിമസ്ത്രീയിലുണ്ടായ സ്വപുത്രന് പുത്രപദവി നല്കുാവാന്‍ വിസമ്മതിച്ചിരുന്നവരുമായി രുന്നു ഇസ്രാഈല്യര്‍ എന്നാണ് അബ്രഹാമിന്റെ ചരിത്രത്തില്‍ യഹൂദറബ്ബിമാര്‍ നടത്തിയ കൈക്രിയ കള്‍ കാണിക്കുന്നത്. വംശീയ ദുരഭിമാനത്തിന്റെ കാര്യത്തില്‍ തങ്ങളുടെ പൂര്വ്വി കന്മാരുടെ സ്വഭാ വം പൂര്ണ്ണെമായിത്തന്നെ ഉള്ക്കൊങള്ളുന്ന അഭിനവ ഇസ്രാഈല്യരുടെ സ്ഥിതിയും ഇതില്‍ നിന്ന് വലിയ വ്യത്യാസമുള്ളതല്ലല്ലോ.

ബൈബിളിൽ നിന്ന് തികച്ചും വ്യതിരിക്തമായ ഇബ്രാഹീമീവ്യക്തിത്വത്തെയാണ് ഖുർആനിൽ നാം കാണുന്നത്. ഇബ്‌റാഹീം നബിയെക്കുറിച്ച് ഖുർആൻ പറയുന്നത് അദ്ദേഹം സദ്‌വൃത്തനും ക്ഷമാശീലനും മാതൃകാപുരുഷനുമായിരുന്നുവെന്നാണ്. (11:75) ഏതെങ്കിലും രൂപത്തിലുള്ള സങ്കുചിതത്വങ്ങളില്ലാത്ത ആദര്ശധീരൻ! അല്ലാഹു തന്നെ 'സ്വന്തം ചങ്ങാതി'യെന്ന് വിലക്കാൻ മാത്ത്രം ദൈവസാമീപ്യത്തിന് അർഹനായ വ്യക്തിത്വം ..."സദ്‌വൃത്തനായിക്കൊണ്ട്‌ തന്‍റെ മുഖത്തെ അല്ലാഹുവിന്‌ കീഴ്പെടുത്തുകയും, നേര്‍മാര്‍ഗത്തിലുറച്ച്‌ നിന്ന്‌ കൊണ്ട്‌ ഇബ്രാഹീമിന്‍റെ മാര്‍ഗത്തെ പിന്തുടരുകയും ചെയ്തവനേക്കാള്‍ ഉത്തമ മതക്കാരന്‍ ആരുണ്ട്‌? അല്ലാഹു ഇബ്രാഹീമിനെ സുഹൃത്തായി സ്വീകരിച്ചിരിക്കുന്നു." (ക്വുർആൻ 4:175)

ക്വുർആനിൽ പ്രതിപാദിക്കപ്പെട്ടിരിക്കുന്ന ഇരുപത്തിയഞ്ച് പ്രവാചകന്മാരിൽ പലരെയും കുറിച്ച് ബൈബിളിൽ പരാമർശങ്ങളുണ്ടെന്നത് ശരിയാണ്.ക്വുർആനിലേതിനേക്കാൾ വിശദമായ ചരിത്ര കഥനവും ബൈബിളിലുണ്ട്.എന്നാൽ തങ്ങളുടെ താല്‍പര്യത്തിനനുസരിച്ച് പ്രവാചകന്മാരുടെ കഥനങ്ങ ള്‍ അവതരിപ്പിക്കുകയാണ് ബൈബിള്‍ പുസ്തകങ്ങളുടെ രചയിതാക്കള്‍ ചെയ്തത്. അതുകൊണ്ടു തന്നെ സമൂഹത്തില്‍ നിലനിന്നിരുന്ന സകല വൃത്തികേടുകളും പ്രവാചകന്മാരില്‍ ആരോപിക്കു വാന്‍ ബൈബിള്‍ കര്‍ത്താക്കള്‍ക്ക് യാതൊരു വൈമനസ്യവുമുണ്ടായിരുന്നില്ല. സമൂഹത്തിന്റെ തിന്മകളെ ന്യായീകരിക്കാനായി പ്രസ്തുത തിന്മകളെല്ലാം പ്രവാചകന്മാരില്‍ ആരോപിക്കുവാനുള്ള പ്രവണതയാണ് ബൈബിളില്‍ നാം കാണുന്നത്.

ധര്‍മനിഷ്ഠനും കുറ്റമറ്റ മനുഷ്യനുമായി ബൈബിള്‍ പരിചയപ്പെടുത്തുന്ന നോഹിനെ (ഉല്‍ 6:9, 10) തന്നെയാണ് ആദ്യമായി വീഞ്ഞുണ്ടാക്കിയവനായും കുടിച്ച് തുണിയുരിഞ്ഞുപോയിട്ട് മക്കള്‍ തുണിയുടുത്തുകൊടുക്കേണ്ട അവസ്ഥയോളമെത്തുന്ന തര ത്തില്‍ ലഹരി ബാധിച്ചവനായുമെല്ലാം ഉല്‍പത്തി പുസ്തകം (9:20-23) വരച്ചുകാണിക്കുന്നത്.

വിശു ദ്ധനും നീതിമാനുമെന്ന് ബെബിള്‍ പറയുന്ന (2 പത്രോസ് 2:78) ലോ ത്ത് മദ്യപിച്ച് മത്തനായി തന്റെ പെണ്‍മക്കളുമായി ശയിക്കുകയും അവര്‍ക്ക് സ്വന്തം പിതാവില്‍ മക്കളുണ്ടാവുകയും ചെയ്ത കഥ ഉല്‍പത്തി പുസ്തകം (19:31-36) വിവരിക്കുന്നുണ്ട്.

ചതിയനും വഞ്ചകനുമായിരുന്നു ഇസ്രായേലി ന്റെ പിതാവായ യാക്കോബെന്നാണ് ബൈബിള്‍ പറയുന്നത് (27:1-46). തന്റെ പടയാളിയുടെ ഭാര്യ യുമായി ബന്ധപ്പെടുകയും അവള്‍ ഗര്‍ഭിണിയായപ്പോള്‍ അതിന്റെ ഉത്തരവാദിത്തം അവളുടെ ഭര്‍ത്താവില്‍ കെട്ടിവെക്കാന്‍ ശ്രമിക്കുകയും പ്രസ്തുത ശ്രമം പരാജയപ്പെട്ടപ്പോള്‍ പടയാളിയെ ചതി ച്ചുകൊല്ലുകയും അങ്ങനെ അയാളുടെ ഭാര്യയെ സ്വന്തമാക്കുകയുമെല്ലാം ചെയ്ത വ്യക്തിയാണ് ബൈബിള്‍ പ്രകാരം ദാവീദ് (2 ശാമു 11:1-27)

.ദാ വീദിന്റെ പുത്രനും ലോകത്തിലെ ഏറ്റവും വലിയ ജ്ഞാനിയുമായി ബൈബിള്‍ പരിചയപ്പെടുത്തുന്ന സോളമന്‍ (1 രാജാ 10:23) വിവാഹബന്ധം വിലക്ക പ്പെട്ടവരുമായി ബന്ധപ്പെടുകയും അവരെ അഗാധമായി സ്‌നേഹിക്കുകയും (1 രാജാ 11:2) അവരുടെ സമ്മര്‍ദ്ദത്തിന് വഴങ്ങി അന്യദേവതമാരെ ആരാധിക്കുകയു (1 രാജാ 11:3-7)മെല്ലാം ചെയ്ത വ്യക്തി യാണ്.

ഇസ്രായേലിന്റെ രക്ഷകനായി വന്ന യേശുക്രിസ്തുവിനെയും മാതൃബഹുമാനമില്ലാത്തവ നായും (യോഹ 2:5, 19:26) സഹിഷ്ണുതയില്ലാത്തവനായും (മത്താ 12:34, 12:39, യോഹ 8:44) ക്ഷിപ്രകോപിയായും (യോഹ 2:13-17, മത്താ 21:19) ജനങ്ങള്‍ക്ക് മദ്യമുണ്ടാക്കിക്കൊടുത്ത് അവരെ ലഹരിപിടിപ്പിച്ചവനായും (യോഹ 2:1-11)മെല്ലാമാണ് ബൈബിള്‍ അവതരിപ്പിച്ചിരിക്കുന്നത്.

ഇതില്‍ നിന്ന് തികച്ചും വ്യതിരിക്തമാണ് ഖുര്‍ആനിലെ സംഭവവിവരണങ്ങള്‍. പ്രവാചകന്മാരെല്ലാം സദ്‌വൃ ത്തരും മാതൃകായോഗ്യരും വിശുദ്ധരുമായിരുന്നുവെന്നാണ് ഖുര്‍ആന്‍ പഠിപ്പിക്കുന്നത്. ''നാം വേദ വും വിജ്ഞാനവും പ്രവാചകത്വവും നല്‍കിയിട്ടുള്ളവരത്രെ അവര്‍. ഇനി ഇക്കൂട്ടര്‍ അവയൊക്കെ നിഷേധിക്കുകയാണെങ്കില്‍ അവയില്‍ അവിശ്വസിക്കുന്നവരല്ലാത്ത ഒരു ജനവിഭാഗത്തെ നാം അത് ഭരമേല്‍പിച്ചിട്ടുണ്ട്. അവരെയാണ് അല്ലാഹു നേര്‍വഴിയിലാക്കിയിട്ടുള്ളത്. അതിനാല്‍ അവരുടെ നേര്‍മാര്‍ഗത്തെ നീ പിന്തുടര്‍ന്ന് കൊള്ളുക. (നബിയേ,) പറയുക. ഇതിന്റെ പേരില്‍ യാതൊരു പ്രതി ഫലവും ഞാന്‍ നിങ്ങളോട് ആവശ്യപ്പെടുന്നില്ല. ഇത് ലോകര്‍ക്കുവേണ്ടിയുള്ള ഒരു ഉത്‌ബോധനമ ല്ലാതെ മറ്റൊന്നുമല്ല'' (വി.ഖു. 6:89, 90).

''അവരെ നാം നമ്മുടെ കല്‍പനപ്രകാരം മാര്‍ഗദര്‍ശനം നല്‍കുന്ന നേതാക്കളാക്കുകയും ചെയ്തു. നല്ലകാര്യങ്ങള്‍ ചെയ്യണമെന്നും നമസ്‌കാരം മുറപോലെ നിര്‍വഹിക്കണമെന്നും സക്കാത്ത് നല്‍കണ മെന്നും നാം അവര്‍ക്ക് ബോധനം നല്‍കുകയും ചെയ്തു. നമ്മെ ആയിരുന്നു അവര്‍ ആരാധിച്ചിരു ന്നത്'' (വി.ഖു. 21:73). ദുര്‍മാര്‍ഗമൊന്നുമില്ലാത്ത നോഹയെയും (7:61) സല്‍മാര്‍ഗനിഷ്ഠനായ ലൂത്തി നെയും (70:80-84) സദ്‌വൃത്തനായ യാക്കോബിനെയും (21:72) വിനയാന്വിതരും ദൈവിക മാര്‍ഗ ത്തില്‍ ഉറച്ച് നിന്നവരുമായ ദാവൂദിനെയും സുലൈമാനെയും (27:15, 38:30) പരിശുദ്ധ പ്രവാചകനും മഹാനും (3:45) മാതൃബഹുമാനമുള്ളവനു (19:32)മായ യേശുവിനെയുമാണ് ഖുര്‍ആന്‍ പരിചയപ്പെടുത്തുന്നത്.

പ്രപഞ്ച നാഥൻ ഏകനാണെന്നും അവൻ മാത്രമാണ് ആരാധ്യനെന്നുമുള്ള ആദർശങ്ങളിൽ ക്വുർ ആനും ബൈബിളും യോജിക്കുന്നു.എന്നാൽ ബൈബിൾ വരച്ചു കാണിക്കുന്നതിൽ നിന്ന് തികച്ചും വ്യത്യസ്തമായ ദൈവസങ്കല്പമാണ് ക്വുർആനിലുള്ളത്. വ്യത്യാസങ്ങളെ ഇങ്ങനെ സംക്ഷേപിക്കാം:

  1. സൃഷ്ടികര്‍ത്താവിനെക്കുറിച്ച് പരാമര്‍ശിക്കുമ്പോള്‍ അവന്റെ മഹത്വത്തിന് അനുഗുണവും ഔന്നത്യത്തിന് ഗ്ലാനി സംഭവിക്കാത്തതുമായ പരാമര്‍ശങ്ങള്‍ മാത്രമെ ഖുര്‍ആനിലുള്ളൂ. ബൈബി ളിലാകട്ടെ, യഹോവയുടെ മഹത്വം ഉല്‍ഘോഷിക്കുന്നുണ്ടെങ്കിലും ഇസ്രായീല്‍ വംശീയതയുടെ സ്വാധീനമുള്ള വചനങ്ങളെത്തുമ്പോള്‍ ഈ മഹത്വത്തിന് വിരുദ്ധമായ നിരവധി പരാമര്‍ശങ്ങള്‍ നടത്തുന്നതായി കാണുന്നുണ്ട്. മനുഷ്യനെ ദൈവത്തിന്റെ പ്രതിഛായയില്‍ അവന്ന് സദൃശനാ യാണ് സൃഷ്ടിച്ചിരിക്കുന്നതെന്ന് (ഉല്‍ 1:26) പറയുന്നത് മുതല്‍ക്കാരംഭിക്കുന്നു ദൈവമഹത്വത്തിന് വിരുദ്ധമായ പരാമര്‍ശ ങ്ങള്‍. വിശ്രമമാവശ്യമുള്ളവനായി ദൈവത്തെ ചിത്രീകരിക്കുന്നതോടെ (ഉല്‍ 2:2,3) ദൈവനിന്ദ അതിന്റെ പരമ കാഷ്ഠയിലെത്തുന്നു. ഏദെന്‍ തോട്ടത്തില്‍ ഒളിച്ച ആദാമി നെയും ഹവ്വയെയും അന്വേഷിച്ച് തിരഞ്ഞുനടക്കുന്ന ബൈബിളിലെ ദൈവം (ഉല്‍ 3:8-13) കിരാത വര്‍ഗങ്ങള്‍ക്കിടയിലെ പ്രാകൃത സങ്കല്‍പത്തേക്കാള്‍ താഴ്ന്ന ദൈവസ ങ്കല്‍പമാണ് ദ്യോതിപ്പി ക്കുന്നത്. താന്‍ ചെയ്തുപോയ കാര്യമാലോചിച്ച് ദുഃഖിക്കുകയും (ഉല്‍ 6:6) മുന്‍കൂട്ടി തീരുമാ നിച്ചുറച്ച കാര്യങ്ങളില്‍ നിന്ന് മനസ് മാറുകയും (പുറ 32:14) ചെയ്യുന്ന ഇസ്രായേലിന്റെ ദൈവം (പുറപ്പാട് 20:5) ഗോത്രപിതാവായ യാക്കോബുമായി മല്ലയുദ്ധം നടത്തി പരാജയപ്പെട്ട കഥകൂടി (ഉല്‍ 32:28) ബൈബിള്‍ പറയു മ്പോള്‍ ഇസ്രായേല്‍ വംശീയതയുടെ സ്വാധീനം എത്രത്തോളം അതി ന്റെ രചനയിലുണ്ടായിട്ടുണ്ടെന്ന് സുതരാം ബോധ്യമാകുന്നു. ഖുര്‍ആനാകട്ടെ, പ്രപ ഞ്ചത്തിന്റെ സ്രഷ്ടാവും നിയന്താവുമായി അല്ലാഹുവിനെ പരിചയപ്പെടുത്തുമ്പോഴൊന്നും തന്നെ അവന്റെ മഹത്വത്തിനോ വിശുദ്ധിക്കോ കോട്ടം തട്ടിക്കുന്ന യാതൊരു പരാമര്‍ശവും നടത്തുന്നില്ല. അറബിക ളുടെയോ ഖുറൈശികളുടെയോ മാത്രം ദൈവമല്ല ഖുര്‍ആനിലെ അല്ലാഹു, പ്രത്യുത സര്‍വ്വലോക രക്ഷിതാവാണ്. ഏതാനും ഖുര്‍ആന്‍ സൂക്തങ്ങള്‍ കാണുക: ''അല്ലാഹു-അവനല്ലാ തെ ദൈവമില്ല. എന്നെന്നും ജീവിച്ചിരിക്കുന്നവന്‍, എല്ലാം നിയന്ത്രിക്കുന്നവന്‍, മയക്കമോ ഉറക്കമോ അവനെ ബാധിക്കുകയില്ല. അവന്റേ താണ് ആകാശഭൂമികളില്‍ ഉള്ളതെല്ലാം'' (വി.ഖു. 2:55).

''അവന്‍ തന്നെയാണ് ആകാശങ്ങളിലും ഭൂമികളിലും സാക്ഷാല്‍ ദൈവം. നിങ്ങളുടെ രഹസ്യവും നിങ്ങളുടെ പരസ്യവും അവന്‍ അറി യുന്നു. നിങ്ങള്‍ നേടിയെടുക്കുന്നതും അവന്‍ അറിയുന്നു'' (വി.ഖു.6:3).

''അവന്റെ പക്കലാകുന്നു അദൃശ്യകാര്യത്തിന്റെ ഖജനാവുകള്‍. അവനല്ലാതെ അവ അറിയുക യില്ല. കരയിലും കടലിലുമുള്ളത് അവന്‍ അറിയുന്നു. അവന്‍ അറിയാതെ ഒരു ഇല പോലും അന ങ്ങുന്നില്ല. ഭൂമിയുടെ ഇരുട്ടുകള്‍ക്കുള്ളിലിരിക്കുന്ന ഒരു ധാന്യമണിയാക ട്ടെ പച്ചയോ ഉണങ്ങിയതോ ആയ ഏതൊരു വസ്തുവാകട്ടെ വ്യക് തമായ ഒരു രേഖയില്‍ എഴുതപ്പെട്ടതായിട്ടല്ലാതെ ഉണ്ടാകില്ല'' (വി.ഖു.6:59). ''ആകാശങ്ങളുടെയും ഭൂമിയുടെയും സ്രഷ്ടാവാകുന്നു (അവന്‍). നിങ്ങള്‍ക്കുവേണ്ടി നിങ്ങളുടെ വര്‍ഗത്തില്‍നിന്നുതന്നെ അവന്‍ ഇണകളെ ഉണ്ടാക്കിത്തന്നിരിക്കുന്നു. കന്നുകാലികളില്‍ നിന്നും ഇണകളെ (ഉണ്ടാക്കിയിരിക്കുന്നു) അതിലൂടെ നിങ്ങളെ അവന്‍ സൃഷ്ടിച്ച് വര്‍ദ്ധിപ്പിക്കുന്നു. അവന് തുല്യമായി യാതൊന്നുമില്ല. അവന്‍ എല്ലാം കേള്‍ക്കുന്നവനും കാണുന്നവനുമാകുന്നു'' (വി.ഖു. 42:11).

 ''(നബിയേ) പറയുക; കാര്യം അല്ലാഹു ഏകനാണ് എന്നതാകുന്നു. അല്ലാഹു ഏവര്‍ക്കും ആശ്രയമാ യിട്ടുള്ളവനാകുന്നു. അവന്‍ (ആര്‍ക്കും) ജന്മം നല്‍കിയിട്ടില്ല. (ആരുടെയും സന്തതിയായി) ജനിച്ചിട്ടു മില്ല. അവന് തുല്യനായി ആരുമില്ലതാനും (വി.ഖു.  112:1-4)

  1. ഖുര്‍ആനിലെയും ബൈബിളിലെയും ചരിത്രകഥനങ്ങള്‍ തമ്മിലുള്ള അന്തരത്തിന്റെ ആത്മാവ് സ്ഥിതിചെയ്യുന്നത് അവയിലെ ദൈവസങ്കല്‍പങ്ങള്‍ തമ്മിലുള്ള അടിസ്ഥാനപരമായ വ്യത്യാസ ങ്ങളിലാണ്. ആദമിന്റെ കഥ വിവരിക്കുന്നിടത്തുതന്നെ കാണുന്ന അന്തരങ്ങള്‍ ശ്രദ്ധിക്കുക.

(i)-  ആദമിനോടും ഇണയോടും തിന്നരുതെന്ന് കല്‍പിച്ച പഴം നന്മ തിന്മകളെക്കുറിച്ച അറിവിന്റെ വൃക്ഷത്തിന്‍േറതായിരുന്നുവെന്നാണ് ബൈബിള്‍ പറയുന്നത്. (ഉല്‍ 2:17), ബൈബിള്‍ പ്രകാരം അത് തിന്നുക വഴിയാണ് മനുഷ്യന് നന്മതിന്മകളെക്കുറിച്ച് അറിവുണ്ടായത് (ഉല്‍ 3:6, 7; 3:22) നന്മതിന്മകളെ വ്യവഛേദിച്ച് മനസ്സിലാക്കാന്‍ കഴിവില്ലാത്ത മനുഷ്യനോട് എങ്ങനെയാണ് വിലക്കപ്പെട്ട കനി തിന്ന രുതെന്ന് കല്‍പിക്കുക? വിധിവിലക്കുകള്‍ പ്രസക്തമാകുന്നത് നന്മതിന്മകളെക്കുറിച്ച അറിവുണ്ടാ വുന്നതോടെയാണല്ലോ. (നന്മ തിന്മകളെക്കുറിച്ച ജ്ഞാനത്തിന്റെ അഭാവത്തില്‍ മൃഗങ്ങളുടെ ലോക ത്ത് വിധിവിലക്കുകള്‍ അപ്രസക്തമാണ് എന്നോര്‍ക്കുക). ഖുര്‍ആനിലെവിടെയും വിലക്കപ്പെട്ട കനി യെക്കുറിച്ച് 'നന്മതിന്മകളെക്കുറിച്ച അറിവിന്റെ വൃക്ഷ'മെന്ന് പരിചയപ്പെടുത്തുന്നില്ല. നന്മതിന്മക ളെക്കുറിച്ച അറിവും നന്മ സ്വീകരിച്ച് ഉന്നതനാകുവാനും തിന്മകളിലൂടെ അധമനാകുവാനുമുള്ള സാധ്യതയും അവന്റെ സൃഷ്ടിയില്‍തന്നെ നിലീനമാണെന്നാണ് ഖുര്‍ആനിക പരാമര്‍ശങ്ങളില്‍നിന്ന് മനസ്സിലാകുന്നത്. നന്മ തിന്മകള്‍ വ്യവഛേദിച്ച് മനസ്സിലാക്കുവാനും തദടിസ്ഥാനത്തില്‍ വസ്തുക്ക ള്‍ക്ക് നാമകരണം ചെയുവാനുമുള്ള കഴിവ് നല്‍കപ്പെടുകയും അങ്ങനെ മാലാഖമാരേക്കാള്‍ ഉന്നത നാവുകയും ചെയ്ത മനുഷ്യനെയാണ് ഖുര്‍ആന്‍ വരച്ചുകാണിക്കുന്നത്. വിലക്കപ്പെട്ട കനിയും നന്മതിന്മകളെക്കുറിച്ച അറിവും തമ്മില്‍ യാതൊരു വിധത്തിലും ഖുര്‍ആന്‍ ബന്ധപ്പെടുത്തുന്നില്ല. ''ഞാനിതാ ഭൂമിയില്‍ ഒരു ഖലീഫയെ നിയോഗിക്കുവാന്‍ പോവുകയാണെന്ന് നിന്റെ നാഥന്‍ മല ക്കുളോട് പറഞ്ഞ സന്ദര്‍ഭം. അവര്‍ പറഞ്ഞു: അവിടെ കുഴപ്പം ഉണ്ടാക്കുകയും രക്തം ചിന്തുകയും ചെയ്യുന്നവനെയാണോ നീ നിയോഗിക്കുന്നത്? ഞങ്ങളാകട്ടെ നിന്റെ മഹത്വത്തെ പ്രകീര്‍ത്തിക്കു കയും നിന്റെ പരിശുദ്ധിയെ വാഴ്ത്തുകയും ചെയ്യുന്നവരല്ലോ. അവന്‍ (അല്ലാഹു) പറഞ്ഞു: നിങ്ങ ള്‍ക്കറിഞ്ഞുകൂടാത്തത് എനിക്കറിയാം. അവന്‍ (അല്ലാഹു) ആദമിനെ നാമങ്ങളെല്ലാം പഠിപ്പിച്ചു. പിന്നീട് ആ പേരിട്ടവയെ അവന്‍ മലക്കുകള്‍ക്ക് കാണിച്ചു. എന്നിട്ടവന്‍ ആജ്ഞാപിച്ചു: നിങ്ങള്‍ സത്യവാന്മാരാണെങ്കില്‍ ഇവയുടെ നാമങ്ങള്‍ എനിക്ക് പറഞ്ഞുതരൂ. അവര്‍ പറഞ്ഞു: നിനക്ക് സ്‌തോത്രം! നീ പഠിപ്പിച്ചുതന്നതല്ലാത്ത യാതൊരു അറിവും ഞങ്ങള്‍ക്കില്ല. നീ തന്നെയാണ് സര്‍വ്വജ്ഞ നും അഗാധജ്ഞാനിയും. അനന്തരം അവന്‍ (അല്ലാഹു) പറഞ്ഞു: ആദമേ ഇവര്‍ക്ക് അവയുടെ നാമ ങ്ങള്‍ പറഞ്ഞുകൊടുക്കൂ. അങ്ങനെ അവന്‍ (ആദം) അവര്‍ക്ക് ആ നാമ ങ്ങള്‍ പറഞ്ഞുകൊടുത്ത പ്പോള്‍ അവന്‍ (അല്ലാഹു) പറഞ്ഞു: ആകാശഭൂമികളിലെ അദൃശ്യകാര്യങ്ങളും നിങ്ങള്‍ വെളിപ്പെ ടുത്തുന്നതും ഒളിച്ചുവെക്കുന്നതുമെല്ലാം എനിക്ക് അറിയാമെന്ന് ഞാന്‍ നിങ്ങളോട് പറഞ്ഞിട്ടില്ലേ'' (വി.ഖു.2:30-33).

ii)- വിലക്കപ്പെട്ട കനി തിന്നരുതെന്ന ദൈവിക കല്‍പനയില്‍ പ്രസ്തുത കനിതിന്നാല്‍ നീ മരിക്കുമെന്ന് ദൈവം ആദാമിനോട് പറയുന്നതായാണ് ബൈബിള്‍ ഉദ്ധരിക്കുന്നത് (ഉല്‍ 2:17). ദൈവിക കല്‍പന ലംഘിക്കുവാന്‍ മനുഷ്യരെ പ്രേരിപ്പിച്ച സര്‍പ്പമാകട്ടെ ''നിങ്ങ ള്‍ മരിക്കുകയില്ല. അത് തിന്നാല്‍ നിങ്ങ ളുടെ കണ്ണുകള്‍ തുറക്കുമെ ന്നും നന്മതിന്മകളെ തിരിച്ചറിഞ്ഞ് നിങ്ങള്‍ ദൈവത്തെപോലെ ആയിത്തീ രുമെന്നും ദൈവത്തിനറിയാം'' എന്ന് പറഞ്ഞുകൊണ്ടാണ് അവരെ പ്രലോഭിപ്പിച്ചത് (ഉല്‍ 3:5). വില ക്കപ്പെട്ട കനി തിന്നപ്പോള്‍ ദൈവം ഭീഷണിപ്പെടുത്തിയതുപോലെ ആദിമനുഷ്യര്‍ മരിച്ചില്ല, പ്രത്യുത അ വര്‍ക്ക് നന്മതിന്മകളെക്കുറിച്ച് ജ്ഞാനമുണ്ടാവുകയാണ് ചെയ്തത് (ഉല്‍ 3:6,7, 3:22). ദൈവം കളവ് പറഞ്ഞ് ആദി മനുഷ്യരെ ഭീഷണിപ്പെടുത്തിയെന്നും യാഥാര്‍ത്ഥ്യം ബോധ്യപ്പെടുത്തിയത് സര്‍പ്പമാ ണെന്നുമാണ് ഈ കഥ ശരിയാണെങ്കില്‍ വന്നുചേരുക. ദൈവിക മഹത്വത്തിന് ഇടിവ് വരുത്തുന്ന ഇത്തരം കഥകളൊന്നുംതന്നെ ഖുര്‍ആനിലില്ല.

iii)- നന്മതിന്മകളെക്കുറിച്ച അറിവിന്റെ കനിതിന്ന മനുഷ്യനെ ഭയപ്പെടുകയും ജീവന്റെ കനികൂടി തിന്ന് മനുഷ്യന്‍ ദൈവത്തെപ്പോലെയാകാതിരിക്കുവാന്‍ മുന്‍കരുതലെടുക്കുകയും ചെയ്യുന്ന ദൈവ ത്തെയാണ് ഉല്‍പത്തി പുസ്തകത്തില്‍ നാം കാണുന്നത് (ഉല്‍ 3: 22-24). വിലക്കപ്പെട്ട കനി തിന്നുക വഴി എന്തെങ്കിലും തരത്തിലുള്ള ദൈവികാംശം മനുഷ്യനുണ്ടായതായി ഖുര്‍ആന്‍ പഠിപ്പിക്കുന്നില്ല. ദൈവ ത്തിന്റെ ഔന്നത്യത്തെയും സര്‍വ്വജ്ഞതയെയും ചോദ്യംചെ യ്യുന്ന ഇത്തരം കഥകളൊന്നും ഖുര്‍ആ നിലില്ല.

(iv) - വിലക്കപ്പെട്ട കനി തിന്നുവാന്‍ മനുഷ്യനെ പ്രേരിപ്പിച്ചത് സര്‍പ്പമാണെന്നാണ് ബൈബിള്‍ പറയു ന്നത് (ഉല്‍ 3:1-5, 3:13). ഇങ്ങ നെ ചെയ്യുക വഴി ദൈവശാപത്തിന് സര്‍പം വിധേയമായി എന്നും, പ്രസ്തുത ശാപത്തിന്റെ ഫലമായാണ് സര്‍പ്പം ഉരസ്സുകൊണ്ട് ഇഴ ഞ്ഞുനടക്കുന്നതെന്നും, മനുഷ്യനും സര്‍പ്പവും തമ്മിലുള്ള വിരോധത്തിന്റെ അടിസ്ഥാനകാരണമിതാണെന്നുമാണ് ബൈബിള്‍ പാഠം (ഉല്‍ 3:14-15). മനുഷ്യന്റെ ശത്രുവായ സാത്താനാണ് മനുഷ്യനെ വഴിതെറ്റിക്കുകയും വിലക്കപ്പെട്ട കനി തിന്നുവാന്‍ പ്രേരിപ്പിക്കുകയും ചെയ്തതെന്നാണ് ഖുര്‍ആന്‍ പറയുന്നത് (ഖുര്‍ആന്‍ 2:35, 36). ഖുര്‍ ആനിലെ ഇവ്വിഷയകമായ പരാമര്‍ശങ്ങളിലൊന്നും സര്‍പ്പം കടന്നുവരുന്നേയില്ല. ദൈവികശാ പത്തിന്റെ ഫലമായിട്ടാണ് സര്‍പം ഇഴ ഞ്ഞ് നടക്കുകയും മനുഷ്യരാല്‍ വെറുക്കപ്പെടുന്നവനാവു കയും ചെയ്തതെങ്കില്‍ പ്രസ്തുത ശാപത്തിന് മുമ്പുള്ള സര്‍പം ഏത് തരത്തിലുള്ളതായിരുന്നുവെന്ന ചോദ്യത്തിന് ഖുര്‍ആനിക വിശദീകരണങ്ങളുടെ വെളിച്ചത്തില്‍ യാതൊരു സാധുതയുമില്ല.

(v) വിലക്കപ്പെട്ട കനി തിന്നതുവഴി ദൈവം ശപിച്ചതിനാലാണ് സ്ത്രീക്ക് ഗര്‍ഭപീഢയും പ്രസവ വേദന യുമുണ്ടായത് എന്നാണ് ബൈബിള്‍ പഠിപ്പിക്കുന്നത് (ഉല്‍ 3:16). ആദിമാതാവിന്റെ തെറ്റിനുള്ള ശിക്ഷ യാണ് ഇന്നും മാതാക്കള്‍ അനുഭവിച്ചുകൊണ്ടിരിക്കുന്നത് എന്നര്‍ത്ഥം. ഗര്‍ഭധാരണത്തെയും പ്രസവ ത്തെയുമെല്ലാം ശിക്ഷയായിക്കാണുന്നതിന് പകരം ദൈവികാനുഗ്രഹമായാണ് ഖുര്‍ആന്‍ മനസിലാക്കിത്തരുന്നത് (ഖുര്‍ആന്‍ 29:8, 46:15, 31:14). വിലക്കപ്പെട്ട കനിയുമായി പ്രസവവേദനക്കോ ഗര്‍ഭപീഢ ക്കോ എന്തെങ്കിലും തരത്തില്‍ ബന്ധമുള്ളതായി ഖുര്‍ആന്‍ പഠിപ്പിക്കുന്നേയില്ല.

(vi) മനുഷ്യരുടെ അധ്വാനവും കൃഷിയുമെല്ലാം വിലക്കപ്പെട്ട കനി തിന്നതുമൂലമുണ്ടായ ദൈവിക ശാപത്തിന്റെ ഫലമായുണ്ടായവയായാണ് ബൈബിള്‍ പരിചയപ്പെടുത്തുന്നത് (ഉല്‍ 3:18, 19). ഖുര്‍ ആനിലാകട്ടെ അധ്വാനിക്കുവാനും സമ്പാദിക്കുവാനുമെല്ലാമുള്ള മനുഷ്യ കഴിവിനെ ദൈവികാനു ഗ്രഹമായാണ് (62:10) വിശേഷിപ്പിച്ചിരിക്കുന്നത്. മനുഷ്യാധ്വാനവും വിലക്കപ്പെട്ട കനിയുമായി ബന്ധപ്പെടുത്തുന്ന യാതൊരു ഖുര്‍ആനിക പരാമര്‍ശവുമില്ല.

(vii) വിലക്കപ്പെട്ട കനി തിന്ന ആദാമും ഹവ്വയും പശ്ചാത്തപി ക്കുകയോ ദൈവം അവര്‍ക്ക് പൊറു ത്ത് കൊടുക്കുകയോ ചെയ്ത തായുള്ള യാതൊരു പരാമര്‍ശവും ബൈബിളിലില്ല. ഖുര്‍ആനാകട്ടെ, തങ്ങളുടെ തെറ്റില്‍ പശ്ചാത്താപവിവശരായ ആദിമാതാപിതാക്കളു ടെ ക്ഷമായാചനയും കാരുണ്യ മൂര്‍ത്തിയായ ദൈവം തമ്പുരാന്റെ പൊറുത്തുകൊടുക്കലും പ്രാധാന്യത്തോട് കൂടിത്തന്നെ പരാമര്‍ ശി ക്കുന്നുണ്ട്. ''അവര്‍ രണ്ടുപേരും  പറഞ്ഞു. ഞങ്ങളുടെ രക്ഷിതാവേ! ഞങ്ങള്‍ ഞങ്ങളോടുതന്നെ അക്രമം ചെയ്തിരിക്കുന്നു. നീ ഞങ്ങള്‍ക്ക് പൊറുത്ത് തരികയും കരുണ കാണിക്കുകയും ചെയ്തി ല്ലെങ്കില്‍തീര്‍ ച്ചയായും ഞങ്ങള്‍ നഷ്ടം പറ്റിയവരുടെ കൂട്ടത്തിലായിരിക്കും'' (വി.ഖു.7:23). ''അനന്തരം ആദം തന്റെ രക്ഷിതാവിങ്കല്‍നിന്ന് ചില വചന ങ്ങള്‍ സ്വീകരിച്ചു. (ആ വചനങ്ങള്‍ മുഖേന പശ്ചാ ത്തപിച്ച) ആദമിന് അല്ലാഹു പാപമോചനം നല്‍കി. അവന്‍ പശ്ചാത്താപം ഏറെ സ്വീക രിക്കുന്നവനും കരുണാനിധിയുമത്രെ'' (വി.ഖു. 2:37,38)

ആദി പാപമെന്ന ആശയം ക്വുർആൻ നിരാകരിക്കുന്നു.ഒരാളും മറ്റൊരാളുടെ പാപഭാരം ഏറ്റെടു ക്കേണ്ടി വരികയില്ലെന്നാണ് ഇസ്‌ലാമിക വിശ്വാസം.

"വല്ലവനും നേര്‍മാര്‍ഗം സ്വീകരിക്കുന്ന പക്ഷം തന്‍റെ സ്വന്തം ഗുണത്തിനായി തന്നെയാണ്‌ അവന്‍ നേര്‍ മാര്‍ഗം സ്വീകരിക്കുന്നത്‌. വല്ലവനും വഴിപിഴച്ച്‌ പോകുന്ന പക്ഷം തനിക്ക്‌ ദോഷത്തിനായി തന്നെയാ ണ്‌ അവന്‍ വഴിപിഴച്ചു പോകുന്നത്‌. പാപഭാരം ചുമക്കുന്ന യാതൊരാളും മറ്റൊരാളുടെ പാപ ഭാരം  ചുമക്കുകയില്ല. ഒരു ദൂതനെ അയക്കുന്നത്‌ വരെ നാം ( ആരെയും ) ശിക്ഷിക്കുന്നതുമല്ല." (ക്വുർആൻ:17: 15)

ദൈവം സ്‌നേഹസമ്പന്നനാണ്. കരുണാനിധിയാണവന്‍. അതോ ടൊപ്പം നീതിമാനുമാണ്. അവന്‍ ക്രൂരനോ ദുഷ്ടനോ ആണെന്ന് ആര്‍ക്കും അഭിപ്രായമില്ല.ആരെങ്കിലുമൊരാള്‍ ചെയ്ത തെറ്റിന് അയാളുടെ മക്കളെയോ പേരമക്കളെയോ ശിക്ഷിക്കുന്നത് ക്രൂര തയാണെങ്കില്‍, ആ ക്രൂരത ചെയ്യുവാന്‍ ദൈവം കൂട്ടുനില്‍ക്കുമോ? ഇല്ലതന്നെ. അവന്‍ നീതിമാനും സ്‌നേഹസമ്പന്നനുമാണ്. മനുഷ്യരുടെ നീതിബോധവു മായി താരതമ്യം ചെയ്യുവാന്‍ പോലും പറ്റാ ത്തത്ര വലിയ നീതിമാന്‍!

മക്കളുടെ തെറ്റുകള്‍ക്ക് പിതാക്കളോ, പിതാക്കളുടെ തെറ്റുകള്‍ക്ക് മക്കളോ ശിക്ഷ അനുഭവിക്കേണ്ട തില്ലെന്ന യാഥാര്‍ഥ്യം ഒരുവിധം എല്ലാ വേദഗ്രന്ഥങ്ങളും വ്യക്തമാക്കുന്നുണ്ട്. മോശയുടെ ന്യായ പ്രമാണം പറയുന്നത് നോക്കുക. 'മക്കള്‍ക്ക് വേണ്ടി പിതാക്കന്മാരെ യോ പിതാക്കന്മാര്‍ക്കുവേണ്ടി മക്കളെയോ വധിക്കരുത്. പാപത്തി നുള്ള മരണശിക്ഷ അവനവന്‍തന്നെ അനുഭവിക്കണം.' (ആവര്‍ത്തനം 24:16.)

ഓരോരുത്തരും ചെയ്ത പാപത്തിന്റെ ശിക്ഷയാണ് മരണാ നന്തര ജീവിതത്തില്‍ അവരവര്‍ ഏറ്റു വാങ്ങുകയെന്നാണ് പ്രവാച കന്മാരെല്ലാം പഠിപ്പിച്ചത്. സുന്ദരമായ ഒരുദാഹരണത്തിലൂടെ ജറമിയാ പ്രവാചകന്‍ ഇക്കാര്യം വ്യക്തമാക്കുന്നത് നോക്കുക. 'പിതാക്ക ന്മാര്‍ പച്ചമുന്തിരിങ്ങ തിന്നു മക്കളു ടെ പല്ലു പുളിച്ചുവെന്ന് ആ നാളുക ളില്‍ അവര്‍ പറയുകയില്ല. ഓരോരുത്തനും അവനവന്റെ അ കൃത്യം നിമിത്തമാണ് മരിക്കുക. പച്ചമുന്തിരിങ്ങ തിന്നവന്റെ പല്ലേ പുളിക്കൂ'(ജറെമിയാ 31:19, 30.)

ഇതേ കാര്യംതന്നെ വളരെ വ്യക്തമായ ഭാഷയില്‍ എസെക്കി യേല്‍ പ്രവാചകനും പറയുന്നുണ്ട്. 'പുത്രന്‍ പിതാവിന്റെ തിന്മകള്‍ ക്കുവേണ്ടിയോ പിതാവ് പുത്രന്റെ തിന്മകള്‍ക്കുവേണ്ടിയോ ശിക്ഷി ക്കപ്പെടുകയില്ല. നീതിമാന്‍ തന്റെ നീതിയുടെ ഫലവും ദുഷ്ടന്‍ തന്റെ ദുഷ്ടതയുടെ ഫലവും അനുഭവിക്കും.'(എസെക്കിയേല്‍ 18:20.)

ഈ വസ്തുതതന്നെയാണ് യേശുക്രിസ്തുവും പറയുന്നത്. 'വിധി ക്കപ്പെടാതിരിക്കാന്‍ നിങ്ങളും വിധിക്കരുത്. നിങ്ങള്‍ വിധിക്കുന്ന വിധിയാല്‍തന്നെ നിങ്ങളും വിധിക്കപ്പെടും. നിങ്ങള്‍ അളക്കുന്ന അള വുകൊണ്ടുതന്നെ നിങ്ങള്‍ക്കും അളന്നുകിട്ടും.' (മത്തായി 7:1,2. )ഖുര്‍ആന്‍ ഇക്കാര്യത്തിലേക്ക് വെളിച്ചം വീശുന്നതിങ്ങനെയാണ്. 'അപ്പോള്‍, ആള്‍ ഒരു അണുവിന്റെ തൂക്കം നന്മ ചെയ്തിരുന്നു വോ അതവന്‍ കാണും. ആര്‍ ഒരു അണുവിന്റെ തൂക്കം തിന്മ ചെയ്തി രുന്നുവോ അതുമവന്‍ കാണും.' (ഖുര്‍ആന്‍ 99:7,8.)

മനുഷ്യരുടെ മനസ്സ് അംഗീകരിക്കുന്ന നീതിബോധത്തിന് തീര്‍ത്തും എതിരാണ്, ക്രൈസ്തവ ദര്‍ശന ത്തിലെ ആദിപാപ സിദ്ധാന്തമെന്നതാണ് വസ്തുത. ആദം ഏദന്‍തോട്ടത്തിലെ വിലക്കപ്പെട്ട കനി തിന്നു പാപിയായി മാറി. പ്രസ്തുത പാപം ആദമി ന്റെ പുത്ര പരമ്പരകളിലും നിലനില്‍ക്കുന്നു. പൗലോസിന്റെ ഭാഷയില്‍ പറഞ്ഞാല്‍ 'ഏക മനുഷ്യനാല്‍ പാപവും പാപം മൂലം മരണവും ലോക ത്തില്‍ പ്രവേശിച്ചു.' (റോമന്‍ 5:12.)

ആദിപാപ സിദ്ധാന്തത്തിലാണ് ക്രിസ്തുവിന്റെ കുരിശുമര ണമെന്ന വിശ്വാസം പടുത്തുയര്‍ത്ത പ്പെട്ടിരിക്കുന്നത്. ആദം ചെയ്ത പാപത്താല്‍ തെറ്റുകാരായി മാറിയ മാനവരാശിയെ പാപത്തിന്റെ പടുകുഴിയില്‍നിന്നു രക്ഷിക്കാനായുള്ള പ്രായശ്ചിത്തമെന്ന നില യ്ക്കാണ് ക്രിസ്തു സ്വന്തം രക്തം കുരിശിലൂടെ ചിന്തിയത് എന്നാ ണല്ലോ വിശ്വാസം. കുരിശില്‍ ചിന്തിയ ചോരകൊണ്ട് ആദമിനാല്‍ ലോകത്തിലേക്കു പ്രവേശിച്ച പാപത്തെ കഴുകിക്കളഞ്ഞുവെന്നും വിശ്വസിക്കപ്പെടുന്നു.

ആദിപാപത്തിന്റെ പരിഹാരാര്‍ഥം സ്വയം ബലിയാകുവാനായിട്ടാണ് ദൈവം ജഡത്തില്‍ അവതരി പ്പിച്ചതെന്നാണ് ക്രൈസ്തവ വിശ്വാസം. പാപത്തിന് പ്രായശ്ചിത്തം ചെയ്യാതിരിക്കാന്‍ ദൈവി കനീതി അനുവദിക്കാത്തതിലാണ് ത്രിയേകദൈവം ഈ നാടകം കളിച്ചതെന്നും വിശദീകരിക്കപ്പെടുന്നു. ആദി പാപത്തിന്റെ പരിഹാരാര്‍ഥം സ്വയം ബലിയാടാവുകയെന്ന നാടകം! ഇതൊരു ഏകാംഗനാടക മാണ്. പാപത്തിന് പ്രായശ്ചിത്തം ചെയ്യുകയെന്നത് ദൈവികനീതി! പ്രായശ്ചിത്തം ചെയ്യാന്‍വേണ്ടി മനുഷ്യരൂപത്തില്‍ അവതരിപ്പിച്ചത് ദൈവം! കുരിശിലേറുന്നത് ദൈവം! ചോര ചിന്തുന്നത് ദൈവം!! അത് മൂലം മനുഷ്യരാശിക്ക് പാപത്തില്‍നിന്നും വിടുതല്‍ നല്‍കുന്നതും ദൈവം!!!

ഈ വിശ്വാസപ്രകാരം ക്രിസ്തുവിന്റെ ആഗമനലക്ഷ്യംതന്നെ മാനവരാശിയെ സ്വന്തം ജീവരക്ത ത്താല്‍ പാപത്തില്‍നിന്ന് രക്ഷിക്കുകയെന്നതായിരുന്നു. പക്ഷേ, ക്രിസ്തുവിന്റെ കുരിശു മരണത്തി നുമുമ്പുള്ള പ്രവര്‍ത്തനങ്ങള്‍ ബൈബിളില്‍നിന്നും വായിക്കുന്ന നാം അത്ഭുതപ്പെട്ടുപോകും. അദ്ദേഹ ത്തിന് മരിക്കാന്‍ തീരെ ഇഷ്ടമില്ലായിരുന്നുവെന്നാണ് സുവിശേഷങ്ങളില്‍നിന്ന് മനസ്സിലാവുന്നത്. പിലാത്തോസിന്റെ പടയാളികളും യഹൂദപൗരോ ഹിത്യവും ചേര്‍ന്നൊരുക്കിയ കുരിശില്‍നിന്ന് രക്ഷപ്പെടണമെന്നാ യിരുന്നു അദ്ദേഹത്തിന്റെ അദമ്യമായ ആഗ്രഹം.

യഹൂദന്മാര്‍ യേശുവിനെ വധിക്കാന്‍ തീരുമാനിച്ചു. ഇതറിഞ്ഞ ക്രിസ്തു സ്വന്തം രക്തം കൊണ്ട് മാന വരാശിയുടെ പാപം കഴുകിക്ക ളയുകയെന്ന തന്റെ ആഗമനലക്ഷ്യം പൂര്‍ത്തീകരിക്കാനായി യഹൂ ദര്‍ക്കിടയില്‍ത്തന്നെ സധൈര്യം സഞ്ചരിച്ചുവെന്നല്ല ബൈബി ള്‍ പഠിപ്പിക്കുന്നത്; പ്രത്യുത, പരസ്യ പ്രബോധനമൊഴിവാക്കി ക്കൊണ്ട് ശിഷ്യന്മാര്‍ക്കിടയില്‍ മാത്രം ഒതുങ്ങിക്കൂടുകയാണ് അദ്ദേഹം ചെയ്തത്. യോഹന്നാന്‍ പറയുന്നു: 'അന്നുമുതല്‍ അവനെ വധിക്കാനവര്‍ ആലോചിച്ചു കൊണ്ടി രുന്നു. അതുകൊണ്ട് യേശു പിന്നീടൊരിക്കലും യഹൂദരുടെയിടയില്‍ സഞ്ചരിച്ചില്ല. അവന്‍ പോയി മരുഭൂമിക്കടുത്തുള്ള എഫ്രയീം പട്ടണത്തില്‍ ശിഷ്യരോ ടൊത്ത് വസിച്ചു.' (യോഹന്നാന്‍ 11.53, 54.)

യഹൂദപൗരോഹിത്യം ഗൂഢാലോചനകള്‍ തുടര്‍ന്നുകൊണ്ടി രുന്നു. സത്യത്തിന്റെ സന്ദേശവാ ഹകനായ മനുഷ്യപുത്രനെ വധിക്കണം. പൗരോഹിത്യത്തിന്റെ പടയാളികള്‍, തന്നെ പിടിക്കുന്ന തിനായി തക്കം പാര്‍ത്തിരിക്കുന്നുണ്ടെന്നറിഞ്ഞ ക്രിസ്തു കയ്യുംകെട്ടി നോക്കിനില്‍ക്കാനല്ല ശിഷ്യ രോട് ആഹ്വാനം ചെയ്യുന്നത്. ആയുധങ്ങളുമായി ഒരുങ്ങിനില്‍ക്കാനും ആവശ്യമെ ങ്കില്‍ ഒരു ഏറ്റു മുട്ടലിന് തയാറാവാനുമാണ്. അദ്ദേഹം പറഞ്ഞു: 'മടിശ്ശീലയുള്ളവന്‍ അതെടുക്കട്ടെ. അതുപോലെ തന്നെ ഭാണ്ഡവും, വാളില്ലാത്തവര്‍ സ്വന്തം കുപ്പായം വിറ്റ് വാള്‍ വാങ്ങട്ടെ'(ലൂക്കോസ്. 22:36. ) ശിഷ്യന്മാര്‍ പറഞ്ഞു: 'ഇതാ ഇവിടെ രണ്ടു വാളുണ്ട്. അവന്‍ പറഞ്ഞു മതി' (ലൂക്കോസ് 22:38.).

ഏതെങ്കലും അപകടത്തിലകപ്പെട്ടാല്‍ അതില്‍നിന്നും രക്ഷ പ്പെടുവാന്‍ സ്വയം ശ്രമിക്കുക മാത്രമല്ല വിശ്വാസികള്‍ ചെയ്യുക. സ്വന്തമായി ചെയ്യാനുള്ളതെല്ലാം ചെയ്തശേഷം സര്‍വശക്തനായ ദൈവ ത്തില്‍ സകലതും സമര്‍പ്പിക്കുന്നവനാണ് യഥാര്‍ഥ വിശ്വാസി. അവനില്‍ പ്രവര്‍ത്തനവും പ്രാര്‍ഥന യും സമന്വയിച്ചിരിക്കും. പ്രവര്‍ത്തനമില്ലാത്ത പ്രാര്‍ഥനയും പ്രാര്‍ഥനയില്ലാത്ത പ്രവര്‍ത്ത നവും വിശ്വാസികള്‍ക്ക് അന്യമാണ്. ഇതുതന്നെയാണ് ക്രിസ്തുവും ചെയ്തത്. തന്നെ വധിക്കാനുള്ള ഗൂഢാ ലോചന നടക്കുന്നുവെന്ന റിഞ്ഞപ്പോള്‍ അദ്ദേഹം ശിഷ്യന്മാരോട് വാളുമായി കരുതിയിരി ക്കാനാവ ശ്യപ്പെട്ടശേഷം സര്‍വശക്തനായ ദൈവത്തോട് ആത്മാര്‍ഥമായി പ്രാര്‍ഥിക്കുന്നത് നോക്കുക: 'അവന്‍ അല്‍പദൂരം മുന്നോട്ടു ചെന്ന് കമിഴ്ന്നുവീണു പ്രാര്‍ഥിച്ചു, എന്റെ പിതാവേ, സാധ്യമെങ്കില്‍ പാന പാത്രം എന്നില്‍നിന്നകന്ന് പോകട്ടെ, എങ്കിലും എന്റെ ഹിതം പോലെയല്ല, അവിടുത്തെ ഹിതം പോലെയാകട്ടെ'(മത്തായി 26:39. )

യേശു വളരെ ആത്മാര്‍ഥമായാണ് പ്രാര്‍ഥിച്ചത്. തീവ്രവേദനയില്‍ മുഴുകിക്കൊണ്ട് ക്രിസ്തു പ്രാര്‍ഥി ച്ചപ്പോള്‍ അദ്ദേഹത്തിന്റെ വിയര്‍ പ്പുതുള്ളികള്‍ പോലും രക്തത്തെപ്പോലെയായി എന്നാണ് ലൂക്കോ സ് എഴുതുന്നത്. 'അവന്‍ തീവ്രവേദനയില്‍ മുഴുകി കൂടുതല്‍ തീക്ഷ്ണമായി പ്രാര്‍ഥിച്ചു. അവന്റെ വിയര്‍പ്പ് രക്തത്തുള്ളികള്‍ പോലെ നിലത്തുവീണു.'(ലൂക്കോസ് 22:44.)

ഇനി നാം ചിന്തിക്കുക, ക്രൈസ്തവ വിശ്വാസപ്രകാരം ക്രിസ്തുവിന്റെ ആഗമനലക്ഷ്യംതന്നെ ആദി പാപത്തിന്റെ പരിഹാരാര്‍ഥം ബലിയാവുകയെന്നതാണ്. പക്ഷേ, ആ 'ബലി' ക്രിസ് തു സ്വയം ഇഷ്ട പ്പെട്ടിരുന്നില്ല. അദ്ദേഹം അതില്‍നിന്ന് രക്ഷ പ്പെടാനാണ് ശ്രമിച്ചത്. ക്രിസ്തുവിന്റെ കുരിശുമരണം നടന്നിട്ടി ല്ലായിരുന്നുവെങ്കില്‍ മാനവരാശി ഇന്നും പാപത്തില്‍തന്നെ യാകുമായിരുന്നു. അപ്പോള്‍ ആരാണ് പാപത്തിന്റെ സാഗര ത്തില്‍നിന്നും മനുഷ്യരെ രക്ഷിച്ചത്? ക്രിസ്തുവല്ല. കാരണം അദ്ദേഹ ത്തിന് ബലിയാകുന്നത് ഇഷ്ടമില്ലായിരുന്നു. യൂദാസും പിലാത്തോസും ഇവിടെ രക്ഷകന്മാരായി മാറു ന്നു. യൂദാസ് ഒറ്റുകൊടുത്തില്ലായിരുന്നുവെങ്കില്‍, പിലാത്തോസ് കുരിശുമരണം വിധിച്ചിട്ടില്ലായിരു ന്നുവെങ്കില്‍, ക്രിസ്തുവിന്റെ രക്തം ചിന്തപ്പെടു മായിരുന്നില്ല! മാനവരാശി പാപത്തിന്റെ പടുകു ഴിയില്‍നിന്നു കരകയറുമായിരുന്നില്ല! കുരിശുമരണം പാപമോചനവുമായി ബന്ധപ്പെടുത്തുന്ന ക്രൈസ്തവര്‍ അവരറിയാതെ ക്രിസ്തുവിനെ ശപിക്കപ്പെട്ടവനും യൂദാസിനെയും പിലാത്തോസി നെയും രക്ഷ കരുമായി അവരോധിക്കുകയാണ് ചെയ്യുന്നത്.

വിഷയവുമായി ബന്ധപ്പെട്ട വീഡിയോ

ദൈവപുത്ര വാദത്തെ ശക്തമായി നിരാകരിക്കുകയും വിമർശിക്കുകയും ചെയ്യുന്നുണ്ട് ക്വുർആൻ.

"ഉസൈര്‍  ദൈവപുത്രനാണെന്ന്‌ യഹൂദന്‍മാര്‍ പറഞ്ഞു. മസീഹ്‌ ദൈവപുത്രനാണെന്ന്‌ ക്രിസ്ത്യാനികളും പറഞ്ഞു. അതവരുടെ വായ കൊണ്ടുള്ള വാക്ക്‌ മാത്രമാണ്‌. മുമ്പ്‌ അവിശ്വസിച്ചവരുടെ വാക്കിനെ അവര്‍ അനുകരിക്കുകയാകുന്നു. അല്ലാഹു അവരെ ശപിച്ചിരിക്കുന്നു. എങ്ങനെയാണവര്‍ തെറ്റിക്കപ്പെടുന്നത്‌?" (ക്വുർആൻ:9 -30)

"വേദക്കാരേ, നിങ്ങള്‍ മതകാര്യത്തില്‍ അതിരുകവിയരുത്‌. അല്ലാഹുവിന്‍റെ പേരില്‍ വാസ്തവമ ല്ലാതെ നിങ്ങള്‍ പറയുകയും ചെയ്യരുത്‌. മര്‍യമിന്‍റെ മകനായ മസീഹ്‌ ഈസാ അല്ലാഹുവിന്‍റെ ദൂത നും, മര്‍യമിലേക്ക്‌ അവന്‍ ഇട്ടുകൊടുത്ത അവന്‍റെ വചനവും, അവങ്കല്‍ നിന്നുള്ള ഒരു ആത്മാവും മാത്രമാകുന്നു. അത്‌ കൊണ്ട്‌ നിങ്ങള്‍ അല്ലാഹുവിലും അവന്‍റെ ദൂതന്‍മാരിലും വിശ്വസിക്കുക. ത്രിത്വം എന്ന വാക്ക്‌ നിങ്ങള്‍ പറയരുത്‌. നിങ്ങളുടെ നന്‍മയ്ക്കായി നിങ്ങള്‍ ( ഇതില്‍ നിന്ന്‌ ) വിരമി ക്കുക. അല്ലാഹു ഏക ആരാധ്യന്‍ മാത്രമാകുന്നു. തനിക്ക്‌ ഒരു സന്താനമുണ്ടായിരിക്കുക എന്നതില്‍ നിന്ന്‌ അവനെത്രയോ പരിശുദ്ധനത്രെ. ആകാശങ്ങളിലുള്ളതും ഭൂമിയിലുള്ളതുമെല്ലാം അവന്‍റെതാ കുന്നു. കൈകാര്യകര്‍ത്താവായി അല്ലാഹു തന്നെ മതി." (ക്വുർആൻ:4 -171)

ക്വുർആൻ നിഷേധിക്കുന്ന ദൈവപുത്രവാദത്തെ ബൈബിളിലെ ക്രിസ്തുവചനങ്ങളും അംഗീകരിക്കുന്നില്ല എന്നതാണ് സത്യം. ദിവ്യത്വമുള്ളയാളാണ് താൻ എന്ന് ക്രിസ്തു അവകാശപ്പെട്ടതായി വ്യക്തമാക്കുന്ന ബൈബിൾ വചന ങ്ങളൊന്നുമില്ല. ദൈവപുത്രനെന്ന് സ്വയം വിശേഷിപ്പിക്കുകയും ശിഷ്യന്മാര്‍ അതുവിളിക്കുന്നതിനെ നിരുല്‍സാഹപ്പെടുത്താതി രിക്കുകയും ചെയ്തിട്ടുണ്ടെന്നതിനാല്‍ അദ്ദേഹത്തില്‍ ദിവ്യത്വമു ണ്ടെന്നാണ് വാദിക്കപ്പെടാറുള്ളത്. ബൈബിള്‍ 'ദൈവപുത്രന്‍' എന്ന് സംബോധന ചെയ്യുമ്പോള്‍ ദൈവത്താല്‍ പ്രത്യേകം നിയുക്തനായ മനുഷ്യനെന്ന് മാത്രമാണ്   വിവക്ഷ. 'ദൈവാ ത്മാവ് നടത്തുന്നവര്‍ ഏവരും ദൈവത്തിന്റെ മക്കള്‍ ആകു ന്നു'(റോമാ 8:14.)വെന്നാണ് പൗലോസ് എഴുതുന്നത്.

'ദൈവപുത്രന്‍' എന്ന വിശേഷണം ദൈവത്തിന്റെ പ്രത്യേക ക്കാരെ ഉദ്ദേശിച്ചുകൊണ്ടാണ് പുതിയ നിയമത്തിലും പഴയ നിയമ ത്തിലും പ്രയോഗിച്ചിരിക്കുന്നത്. ക്രിസ്തുതന്നെ ഇക്കാര്യം വ്യക്തമാ ക്കുന്നുണ്ട്. 'ദൈവത്തിന്റെ വചനം ലഭിച്ചവരെ അവന്‍ ദേവന്മാര്‍ എന്നു വിളിച്ചുവെങ്കില്‍, ഞാന്‍ ദേവപുത്രനാണ് എന്നു പറഞ്ഞതുകൊണ്ട് പിതാവ് അഭിഷേകം ചെയ്തു ലോകത്തിലേക്ക് അയച്ച എന്നെപ്പറ്റി 'നീ ദൈവദൂഷണം നടത്തേണ്ടുന്നുവെന്ന് നിങ്ങള്‍ പറയുന്നുവോ?' (യോഹന്നാന്‍ 10:35, 36.) ദൈവവചനം നല്‍കപ്പെട്ട ഇസ്രായീ ല്യരെ ദൈവം ദേവന്മാര്‍ എന്നു സംബോധനം ചെയ്തതുപോലെ യാണ് ലോകത്തിലേക്ക് മാര്‍ഗദര്‍ശിയായി അയക്കപ്പെട്ട പ്രവാ ചകനായ ക്രിസ്തുവിനെ 'ദൈവപു ത്രന്‍' എന്നു വിളിക്കുന്നതെ ന്നര്‍ഥം.

പഴയനിയമം മുതലേ 'ദൈവപുത്രന്‍' എന്ന സംബോധനാരീതി നിലനിന്നിരുന്നതായി കാണാന്‍ കഴിയും. യാക്കോബും സോളമനും എ്രഫയീയും ദാവീദുമെല്ലാം പഴയ നിയമത്തിന്റെ ഭാഷയില്‍ ദൈവ പുത്രന്മാരാണ്.

'കര്‍ത്താവ് പറയുന്നു: ഇസ്രായില്‍ എന്റെ പുത്രനാണ്. എന്റെ ആദ്യജാതന്‍.' (പുറപ്പാട് 4:22)

'ഞാന്‍ അവന് (സോളമന്) പിതാവും അവന്‍ എനിക്ക് പുത്രനു മായിരിക്കും.' (സാമുവല്‍ 7:14.)

'ഞാന്‍ ഇസായിലിനു പിതാവാണ്. എഫ്രയീം എന്റെ ആദ്യ ജാതനും.' (യിരെമ്യാവ് 31:9. 16. സങ്കീര്‍ത്തനങ്ങള്‍ 2:7. )

'നീ (ദാവീദ്) എന്റെ പുത്രനാണ്; ഇന്ന് ഞാന്‍ നിനക്ക് ജന്മം നല്‍കി.' (17. യോഹന്നാന്‍ 1:12. 18. മത്തായി 5.9.)

ക്രിസ് തുവില്‍ വിശ്വസിക്കുന്നവര്‍ മുഴുവന്‍ ദൈവപുത്രന്മാരാണെന്നാണ് പുതിയ നിയമം പറയുന്നത്. യോഹന്നാന്‍ എഴുതുന്നു. 'എന്നാല്‍, തന്റെ കൈകൊണ്ട് തന്റെ നാമത്തില്‍ വിശ്വസി ക്കുന്ന ഏവര്‍ക്കും ദൈവമക്കള്‍ ആകുവാനുള്ള അവകാശം അവന്‍ കൊടുത്തു'(യോഹന്നാന്‍ 1:12.)

ദൈവപുത്രന്മാരെന്ന് അറിയപ്പെടുന്നവര്‍ ആരാണെന്ന് മത്തായി വ്യക്തമാക്കുന്നുണ്ട്. 'സമാധാനമു ണ്ടാക്കുന്നവര്‍ ഭാഗ്യവാന്മാര്‍; അവ ര്‍ ദൈവപുത്രന്മാര്‍ എന്നു വിളിക്കപ്പെടും' (മത്തായി 5.9.)

പിതാവില്ലാതെ ജനിച്ചതിനാലാണ് ക്രിസ്തു  ദൈവപുത്രനാണെന്ന് പറയുന്നതെന്ന് വാദിക്കാറുണ്ട്. എങ്കില്‍, പിതാവും മാതാവു മില്ലാതെ ജനിച്ച ആദാമാണല്ലോ യേശുവിനേക്കാള്‍ യോഗ്യനായ ദൈവപുത്രന്‍! ബൈബിള്‍ ആദാമി നെ ദൈവപുത്രനാണെന്ന് (ലൂക്കോസ് 3.38. ) പരിചയപ്പെടു ത്തുന്നുമുണ്ട്.

മഹാപുരോഹിതനായ മെല്‍ക്കീസേദക്കിനെക്കുറിച്ച് ബൈബിള്‍ പരിചയപ്പെടുത്തുന്നത് ആദിയും അന്ത്യവുമില്ലാത്തവനായിട്ടാണ്. സലോമിന്റെ രാജാവും അത്യുന്നതനുമായ ദൈവത്തിന്റെ പുരോ ഹിതനുമായ മെല്‍ക്കീസേദക്കിനെക്കുറിച്ച് പൗലോസ് എഴുതുന്നത് നോക്കുക. 'അവന്ന് പിതാവോ മാതാവോ വംശപരമ്പരയോ ഇല്ല. അവന്റെ ദിവസങ്ങള്‍ക്ക് ആരംഭമോ ആയുസ്സിന് അവസാനമോ ഇല്ല.'(ഹെബ്രായര്‍ 7:3.)

ക്രിസ്തുവിനെ സംബന്ധിച്ച്  'ദൈവപുത്രന്‍' എന്ന സംബോധന, സുവിശേഷങ്ങളില്‍ തുലോം വിരളമാണ്. പ്രസ്തുത പ്രയോഗം കൂടുതലായി കാണപ്പെടുന്നത് പൗലോസിന്റെ ലേഖനങ്ങളിലാണ്. സുവിശേ ഷങ്ങളില്‍ യേശുവിനെ സംബന്ധിച്ച് 'മനുഷ്യപുത്രന്‍' എന്നു 63 പ്രാവശ്യം പ്രയോഗിക്കപ്പെട്ടിരി ക്കുന്നു. അദ്ദേഹത്തെ മനുഷ്യന്‍ എന്നുമാത്രം 73 പ്രാവശ്യം സംബോധന ചെയ്തിരിക്കുന്നു.

ദൈവത്തിന് പുത്രന്മാരില്ലെന്നും ക്രിസ്തു മഹാനായ ഒരു പ്രവാചകൻ മാത്രമാണെന്നുമുള്ള ഖുർആനിക പാഠത്തെ സത്യപ്പെടുത്തുന്നവയാണ് ബൈബിൾ വചനങ്ങൾ എന്ന് സാരം.

വിഷയവുമായി ബന്ധപ്പെട്ട വീഡിയോ
ശിർക്ക്‌ എന്ന മഹാപാപം ഒരിക്കലും പൊറുക്കപ്പെടുകയില്ലെന്ന് ഖുര്‍ആനിലെ ചില സൂക്തങ്ങളിൽ (4:48, 4:116) വ്യക്തമാക്കുന്നു. ഇതിനു വിരുദ്ധമായി ശിർക്ക്‌ ചെയ്തവർക്ക്‌ അല്ലാഹു പൊറുത്തു കൊടുക്കുന്നതായി (4:153,25:68-71) സൂക്തങ്ങളിൽ പറയുന്നുണ്ട്. ഇവയെങ്ങനെ പൊരുത്തപ്പെടും?

പാപങ്ങള്‍ പല തരമുണ്ട്. അതില്‍ ഏറ്റവും ഗുരുതരമായതാണ് ശിര്‍ക്ക് അഥവാ ബഹുദൈവാരാധന. സ്രഷ്ടാവും സംരക്ഷകനുമായ അല്ലാഹുവിന് മാത്രം അവകാശപ്പെട്ട ആരാധനകള്‍ സൃഷ്ടികള്‍ക്ക് സമര്‍പ്പിക്കുന്നതിനേക്കാള്‍ വലിയ പാപങ്ങളേതുമില്ല. ശിര്‍ക്ക് ചെയ്യുന്നവന് ദൈവിക കാരുണ്യത്തിന്റെ ഭവനമായ സ്വര്‍ഗം നിഷിദ്ധമാണെന്നാണ് ഖുര്‍ആന്‍ പഠിപ്പിക്കുന്നത്. അല്ലാഹുവോട് വല്ലവനും പങ്ക് ചേര്‍ക്കുന്ന പക്ഷം തീര്‍ച്ചയായും അല്ലാഹു അവന് സ്വര്‍ഗം നിഷിദ്ധമാക്കുന്നതാണ്. നരകം അവന്റെ വാസസ്ഥലമായിരിക്കുകയും ചെയ്യും.(5:72)

ശിര്‍ക്കില്‍ നിന്ന് പൂര്‍ണമായി മുക്തനാവുമ്പോഴാണ് ഒരാള്‍ സത്യവിശ്വാസിയായിത്തീരുന്നത്. ബഹുദൈവാരാധന ഉള്‍കൊള്ളുന്ന സകലമാന പ്രവര്‍ത്തനങ്ങളില്‍ നിന്നും പരിശുദ്ധി പ്രാപിച്ച്, അല്ലാഹുവല്ലാതെ ആരാധ്യനില്ലെന്ന് ഞാന്‍ സാക്ഷ്യം വഹിക്കുന്നുവെന്ന് പ്രഖ്യാപിച്ചുകൊണ്ടാണ് ഇസ്ലാമിലേക്ക് പ്രവേശിക്കുന്നത്. ഇങ്ങനെപ്രവേശിക്കുന്ന വ്യക്തിയുടെ മുന്‍കഴിഞ്ഞ പാപങ്ങളെല്ലാം –ബഹുദൈവാരാധനയും ദൈവനിഷേധവുമടക്കം– പൊറുക്കപ്പെടുന്നതാണ്. ബഹുദൈവാരാധകരായിരുന്ന ജനങ്ങളിലേക്കാണ് അല്ലാഹു പ്രവാചകന്‍മാരെ നിയോഗിച്ചത്. പ്രവാചകന്‍മാരുടെ ശിഷ്യന്‍മാരായിത്തീരുകയും ഏകദൈവവിശ്വാസത്തിന്റെ പ്രചാരണത്തിനുവേണ്ടി ജീവന്‍ വരെ ത്യജിക്കുവാന്‍ സന്നദ്ധരാവുകയും ചെയ്തവര്‍ മുമ്പ് ബഹുദൈവാരാധകരായിരുന്നു. ഇസ്ലാം സ്വീകരിച്ചതോടുകൂടി അവരുടെ പാപങ്ങളെല്ലാം പൊറുക്കപ്പെടുകയും ജനിച്ചുവീണ കുഞ്ഞിനെപ്പോലെ പരിശുദ്ധരായിത്തീരുകയും ചെയ്തു. നരകത്തിന്റെ പാതയായ ശിര്‍ക്കില്‍നിന്ന് രക്ഷപ്പെട്ട് സ്വര്‍ഗത്തിന്റെ മാര്‍ഗമായ ഏകദൈവ വിശ്വാസത്തില്‍ എത്തിച്ചേര്‍ന്നവരെ സംബന്ധച്ചിടത്തോളം അവരുടെ മുന്‍കാല പ്രവര്‍ത്തനങ്ങളെ കുറിച്ചുള്ള വിചാരണയില്ലെന്നും മുസ്‌ലിമായതിനുശേഷമുള്ള കര്‍മ്മങ്ങളെ സംബന്ധിച്ചു മാത്രമെ ചോദ്യം ചെയ്യപ്പെടുകയുള്ളൂവെന്നും വ്യക്തമാക്കുന്ന ഒട്ടനവധി ഹദീസുകളുണ്ട്. എത്രകൊടിയ ബഹുദൈവാരാധകനും തോന്നിവാസിയുമാണെങ്കിലും സത്യവിശ്വാസം സ്വീകരിക്കുന്നതോടെ അയാള്‍ പാപമുക്തി പ്രാപിക്കുന്നുവെന്നര്‍ഥം.

സ്രഷ്ടാവും സംരക്ഷകനുമായ അല്ലാഹുവിന്ന് സ്വന്തം ജീവിതത്തെ സമര്‍പ്പിച്ചവനാണ് മുസ്‌ലിം. ദൈവിക വിധി വിലക്കുകള്‍ പാലിക്കുന്നതാണ് മുസ്‌ലിമിന്റെ ജീവിതത്തെ വിമലീകരിക്കുകയും മാതൃകായോഗ്യമാക്കുകയും ചെയ്യുന്നത്. എങ്കിലും മനുഷ്യന്‍ എന്ന നിലക്ക് മുസ്ലിമിന്റെ ജീവിതത്തിലും തെറ്റുകള്‍ കടന്നുവരാം; കുറ്റങ്ങളുണ്ടാകാം. ഏതെങ്കിലുമൊരു ദുര്‍ബല നിമിഷത്തില്‍ വല്ല തെറ്റുകുറ്റങ്ങളിലും ഏതെങ്കിലും ഒരു മുസ്‌ലിം അകപ്പെട്ടുപോയാല്‍ അതിനുള്ള പരിഹാരനിര്‍ദ്ദേശങ്ങളും ഖുര്‍ആനിലും നബി വചനങ്ങളിലുമുണ്ട്. പശ്ചാത്താപമാണ് പാപത്തിനുള്ള പരിഹാരമായി നിര്‍ദ്ദേശിക്കപ്പെട്ടിട്ടുള്ളത്. ചെയ്തുപോയ തെറ്റില്‍ ആത്മാര്‍ഥമായി അനുതപിക്കുകയും ഇനിയത് ആവര്‍ത്തിക്കുകയില്ലെന്ന് പ്രതിജ്ഞ ചെയ്യുകയും, പൊറുത്തു തരുന്നതിനു വേണ്ടി അല്ലാഹുവിനോട് അകമുരുകി പ്രാര്‍ഥിക്കുകയും ചെയ്തുകൊണ്ടുള്ള പശ്ചാത്താപമാണ് പാപത്തിനുള്ള പരിഹാരം.

പശ്ചാത്താപം സ്വീകരിക്കാന്‍ അല്ലാഹു ബാധ്യതയേറ്റിരിക്കുന്നത് അറിവുകേട് നിമിത്തം തിന്‍മ ചെയ്യുകയും, എന്നിട്ട് താമസിയാതെ പശ്ചാത്തപിക്കുകയും ചെയ്യുന്നവര്‍ക്ക് മാത്രമാകുന്നു. അങ്ങനെയുള്ളവരുടെ പശ്ചാത്താപം അല്ലാഹു സ്വീകരിക്കുന്നതാണ്. അല്ലാഹു എല്ലാം അറിയുന്നവനും യുക്തിമാനുമാകുന്നു. പശ്ചാത്താപമെന്നത് തെറ്റുകള്‍ ചെയ്തു കൊണ്ടിരിക്കുകയും എന്നിട്ട് മരണം ആസന്നമാവുമ്പോള്‍ ഞാനിതാ പശ്ചാത്തപിച്ചിരിക്കുന്നു എന്ന് പറയുകയും ചെയ്യുന്നവര്‍ക്കുള്ളതല്ല.സത്യനിഷേധികളായി മരണമട യുന്നവര്‍ക്കുമുള്ളതല്ല. അങ്ങനെയുള്ളവര്‍ക്ക് വേദനയേറിയ ശിക്ഷയാണ് നാം ഒരുക്കിവെച്ചിട്ടുള്ളത്. (4:17,18).

ഏതുതരം പാപങ്ങളും പശ്ചാത്താപം വഴി അല്ലാഹു പൊറുത്തുതന്നേക്കുമെന്ന പ്രതീക്ഷ നല്‍കുന്നുണ്ട് ഖുര്‍ആന്‍. ഒരു മുസ്‌ലിമിന്റെ ജീവിതത്തില്‍ വന്നു ഭവിക്കുന്ന ചെറിയ ചെറിയ തെറ്റുകള്‍ അവന്‍ ചെയ്യുന്ന സല്‍കര്‍മ്മങ്ങള്‍ വഴി പൊറുത്തു കൊടുക്കുമെന്ന് ഹദീസുകളില്‍ വ്യക്തമാക്കപ്പെട്ടിട്ടുമുണ്ട്. പാപം പൊറുക്കുക എന്നത് അല്ലാഹുവിന്റെ ഔദാര്യമാണ്. എത്ര തന്നെ പാപപങ്കിലമായ ജീവിതം നയിച്ച വ്യക്തിയാണെങ്കിലും അവന്‍ ആത്മാര്‍ഥമായി പശ്ചാത്തപിച്ചു മടങ്ങിയാല്‍ അല്ലാഹു അവനില്‍ നിന്ന് വന്നുപോയ പാപങ്ങള്‍ പൊറുത്തു കൊടുക്കാമെന്ന് വാഗ്ദാനം ചെയ്തിട്ടുണ്ട്:

പറയുക: സ്വന്തം ആത്മാക്കളോട് അതിക്രമം പ്രവര്‍ത്തിച്ചുപോയ എന്റെ ദാസന്‍മാരേ, അല്ലാഹുവിന്റെ കാരുണ്യത്തെപ്പറ്റി നിങ്ങള്‍ നിരാശപ്പെടരുത്. തീര്‍ച്ചയായും അല്ലാഹു പാപങ്ങളെല്ലാം പൊറുക്കുന്നതാണ്. തീര്‍ച്ചയായും അവന്‍ തന്നെയാകുന്നു ഏറെപൊറുക്കുന്നവനും കരുണാനിധിയും (39:53).

എന്നാല്‍, ഒരു മുസ്‌ലിമിന്റെ ജീവിതത്തില്‍ ഒരിക്കലും വന്നു ഭവിക്കാന്‍ പാടില്ലാത്ത പാപമാണ് ശിര്‍ക്ക്. അല്ലാഹുവില്‍ പങ്ക് ചേര്‍ക്കുകയെന്ന പാപം അവന്റെ ജീവിതത്തിലുണ്ടാവുകയെന്ന് പറഞ്ഞാല്‍ അത് അവന്റെ വിശ്വാസത്തില്‍ നിന്നുള്ള വ്യതിചലനമാണ്. കേവല പശ്ചാത്താപം കൊണ്ടോ മറ്റു സല്‍കര്‍മ്മങ്ങള്‍ വഴിയോ പൊറുക്കപ്പെടുന്ന പാപമല്ല അത്. പ്രത്യുത, സകലമാന സല്‍കര്‍മ്മങ്ങളെയും വിഴുങ്ങിക്കളയുന്ന അത്യുഗ്ര പാപമാണത്. സത്യമത പ്രബോധനത്തിന് വേണ്ടി നിയോഗിക്കപ്പെടുകയും ആ മാര്‍ഗത്തില്‍ ഒട്ടനവധി ത്യാഗപരിശ്രമങ്ങള്‍ നടത്തുകയും ചെയ്ത പ്രവാചകന്‍മാരുടെ ജീവിതത്തിലെവിടെയെങ്കിലും ശിര്‍ക്ക് എന്ന മഹാ പാപം വന്നുപോയാല്‍ അവരുടെ കര്‍മ്മങ്ങളെല്ലാം നിഷ്ഫലമാവുകയും അവര്‍ നരകാവകാശികളില്‍ പെടുകയും, ചെയ്യുമെന്നാണ് ഖുര്‍ആന്‍പഠിപ്പിക്കുന്നത്.

നീ ശിര്‍ക്ക് ചെയ്യുന്ന പക്ഷം തീര്‍ച്ചയായും നിന്റെ കര്‍മം നിഷ്ഫലമായിപ്പോവുകയും തീര്‍ച്ചയായും നീ നഷ്ടകാരുടെ കൂട്ടത്തിലാവുകയും ചെയ്യും (39:65) എന്നാണ് അന്തിമ പ്രവാചകന്‌ പോലും ഖുര്‍ആന്‍ മുന്നറിയിപ്പ് നല്‍കുന്നത്.

ശിര്‍ക്ക് ഒരിക്കലും പൊറുക്കപ്പെടാത്ത പാപമാണെന്ന് ഖുര്‍ആന്‍ പലതവണ വിശ്വാസികളെ ഉല്‍ബോധിപ്പിക്കുന്നുണ്ട്: തീര്‍ച്ചയായും അല്ലാഹു അവനോട് പങ്ക് ചേര്‍ക്കപ്പെടുന്നത് പൊറുക്കുകയില്ല. അതിനുപുറമെയുള്ളത് അവന്‍ ഉദ്ദേശിക്കുന്നവര്‍ക്ക് അവന്‍ പൊറു ത്തുകൊടുക്കുകയും ചെയ്യും. ആര് അല്ലാഹുവിനോട് പങ്ക് ചേര്‍ക്കുന്നുവോ അവന്‍, തീര്‍ച്ചയായും വമ്പിച്ച കുറ്റം ചമച്ചുണ്ടാക്കിയിരിക്കുന്നു. (4:48)

ശിര്‍ക്ക് ഒരിക്കലും പൊറുക്കുകയില്ലെന്നു പറഞ്ഞതിനര്‍ത്ഥം കേവല പശ്ചാത്താപം കൊണ്ടോ സല്‍കര്‍മ്മങ്ങള്‍ വഴിയായി മാത്രമോ അത് പൊറുക്കപ്പെടുകയില്ലെന്നാണ്. ശിര്‍ക്ക് ചെയ്യുന്നതോടെ അത് ചെയ്യുന്നയാള്‍ സത്യവിശ്വാസത്തിന്റെ വൃത്തത്തില്‍ നിന്ന് പുറത്തു കടന്നു കഴിഞ്ഞു. സല്‍കര്‍മ്മങ്ങള്‍ വഴി തെറ്റുകള്‍ പൊറുക്കാമെന്നത് വിശ്വാസികളോടുള്ള വാഗ്ദാനമാണ്. വിശ്വാസ വൃത്തത്തില്‍ നിന്ന് പുറത്തു കടന്നവന് ഈ വാഗ്ദാനം ബാധകമല്ല. ശിര്‍ക്ക് ചെയ്യുന്നതോടെ അവന്റെ സല്‍കര്‍മ്മങ്ങളെല്ലാം നിഷ്ഫലമായിപ്പോയി. അറിവുകേടു കൊണ്ടോ അബദ്ധവശാലോ ഒരു വിശ്വാസിയുടെ പ്രവര്‍ത്തനങ്ങളില്‍ ശിര്‍ക്കു വന്നുപോയാല്‍ പിന്നെ അയാള്‍ക്ക് ഒരിക്കലും മോചനമില്ലെന്നല്ല ഇതിനര്‍ഥം. പിന്നെയോ? അയാള്‍ക്ക് ഇനി മോചനം വേണമെങ്കില്‍ വിശ്വാസത്തിലേക്ക് മടങ്ങണം. ഒരു അവിശ്വാസി എങ്ങനെയാണോ വിശ്വാസിയായിത്തീരുന്നത്, ആ രൂപത്തില്‍ ഏകദൈവാദര്‍ശത്തിന്റെ സാക്ഷ്യവചനങ്ങള്‍ മനസ്സില്‍ ഉള്‍കൊണ്ട് പ്രഖ്യാപിക്കണം. ബഹുദൈവാരാധനയുടെ ലാഞ്ഛനയെങ്കിലും ഉള്‍ക്കൊള്ളുന്ന വിശ്വാസാചാരങ്ങളില്‍ നിന്ന് പൂര്‍ണമായും ഖേദിച്ചുമടങ്ങുകയും വേണം.

ശിര്‍ക്ക്, കൊലപാതകം, വ്യഭിചാരം തുടങ്ങിയ പാപങ്ങള്‍ ചെയ്തവര്‍ക്ക് പാപമോചനത്തിനുള്ള മാര്‍ഗമെന്താണെന്നതിനെ കുറിച്ചാണ് സൂറത്തുല്‍ ഫുര്‍ഖാനിലെ വചനങ്ങളില്‍ (25:68-71) വ്യക്തമാക്കുന്നത്:

പക്ഷേ, ആരെങ്കിലും പശ്ചാത്തപിക്കുകയും വിശ്വസിക്കുകയും സല്‍കര്‍മം പ്രവര്‍ത്തിക്കുകയും ചെയ്യുന്നതായാല്‍, അങ്ങനെയുള്ളവരുടെ തിന്മകളെ അല്ലാഹു നന്മകളാക്കി മാറ്റുന്നതാണ്. അല്ലാഹു ഏറെപൊറുക്കുന്നവനും കരുണാനിധിയുമാകുന്നു. (25:70).

ശിര്‍ക്കു ചെയ്ത വ്യക്തികള്‍ പശ്ചാത്തപിക്കുന്നതോടൊപ്പം സുദൃഢവും കളങ്കലേശമില്ലാത്തതുമായ ഏകദൈവ വിശ്വാസത്തിലേക്ക് മടങ്ങുക കൂടി ചെയ്യണമെന്ന് ഈ വചനങ്ങളില്‍ നിന്ന് സുതരാം വ്യക്തമാണ്.

ഇക്കാര്യത്തിലുള്ള ഒരു സംഭവ വിവരണമാണ് സൂറത്തുന്നിസാഇലെ വചനത്തി (4:153)ലുള്ളത്. പശുക്കുട്ടിയുടെ വിഗ്രഹമുണ്ടാക്കി അതിനെ ആരാധിച്ചവര്‍ക്ക് അല്ലാഹു പൊറുത്തു കൊടുത്തത് അവര്‍ വിശ്വാസത്തിലേക്ക് മടങ്ങുകയും പശ്ചാത്തപിക്കുകയും ചെയ്തതുകൊണ്ടാണ്. ഇക്കാര്യം സൂറത്തു അഅ്‌റാഫിലെ 152,153 വചനങ്ങളില്‍ ഖുര്‍ആന്‍ വ്യക്തമാക്കുന്നുണ്ട്:

നിശ്ചയമായും പശുക്കുട്ടിയെ ഉണ്ടാക്കി (ആരാധന നടത്തിയ)വര്‍ക്കു തങ്ങളുടെ റബ്ബിങ്കല്‍ നിന്ന് കോപവും, ഐഹിക ജീവിതത്തില്‍ നിന്ദ്യതയും ബാധിക്കുന്നതാണ്. അപ്രകാരമത്രേ (വ്യാജം) കെട്ടിച്ചമക്കുന്നവര്‍ക്ക് നാം പ്രതിഫലം നല്‍കുന്നത്. തിന്മകള്‍ പ്രവര്‍ത്തിക്കുകയും പിന്നീട് അതിനുശേഷം പശ്ചാത്തപിക്കുകയും വിശ്വസിക്കുകയും ചെയ്തവരാകട്ടെ, നിശ്ചയമായും അതിനുശേഷം നിന്റെ റബ്ബ് (അവര്‍ക്ക്) പൊറുത്ത് കൊടുക്കുന്നവനും കരുണാനിധിയും തന്നെ.’

ഗോപൂജകന്‍മാരായിത്തീര്‍ന്ന ഇസ്‌റാഈല്‍ മക്കളില്‍ ഏകദൈവാദര്‍ശത്തിലേക്ക് തിരിച്ചുവന്ന് കളങ്കലേശമില്ലാത്ത വിശ്വാസം സ്വീകരിക്കുകയും, ചെയ്തുപോയ തെറ്റില്‍ ആത്മാര്‍ഥമായി പശ്ചാത്തപിക്കുകയും ചെയ്തവര്‍ക്ക് അല്ലാഹു പൊറുത്തു കൊടുത്ത കാര്യമാണ് ഈ സൂക്തങ്ങളില്‍ പ്രതിപാദ്യം.

ശിര്‍ക്ക് മഹാപാപമാണെന്നും അത് പൊറുക്കുകയില്ലെന്നും പറഞ്ഞതിന്റെ താല്‍പര്യം, അതു ചെയ്ത ശേഷം പശ്ചാത്തപിച്ച് സത്യവിശ്വാസത്തിലേക്ക് മടങ്ങാതെ മരണപ്പെട്ടവര്‍ക്ക് സ്വര്‍ഗം നിഷിദ്ധമാണെന്ന വസ്തുത വ്യക്തമാക്കുകയാണ്. വിശ്വാസിയായിരിക്കെ സംഭവിക്കുന്ന മറ്റു പാപങ്ങളെപ്പോലെ അല്ലാഹു പൊറുത്തു കൊടുക്കുമെന്ന പ്രതീക്ഷ ശിര്‍ക്കിന്റെ കാര്യത്തില്‍ അസ്ഥാനത്താണെന്നാണ് ഇതിന്നര്‍ഥം. അല്ലാതെ, ശിര്‍ക്ക് ചെയ്തുപോയവര്‍ക്ക് ഒരിക്കലും യാതൊരുവിധ മോചനവുമില്ലെന്ന് പഠിപ്പിക്കുകയല്ല, ശിര്‍ക്ക് പൊറുക്കപ്പെടാത്ത പാപമാണെന്ന് പറയുന്ന സൂക്തങ്ങള്‍ ചെയ്യുന്നത്. അതുകൊണ്ട് തന്നെ വിശ്വാസത്തിലേക്ക് മടങ്ങുകയും പശ്ചാത്തപിക്കുകയും ചെയ്തവര്‍ക്ക് അവര്‍ ചെയ്തുപോയ ശിര്‍ക്ക് പൊറുത്തു കൊടുക്കുമെന്ന് വ്യക്തമാക്കുന്ന സൂക്തങ്ങളുമായി ഇവ യാതൊരു വിധ വൈരുധ്യങ്ങളും വെച്ചുപുലര്‍ത്തുന്നില്ല. എന്നാല്‍ വിശ്വാസത്തില്‍ നിന്നും അവിശ്വാസത്തിലേക്കും വീണ്ടും വിശ്വാസത്തിലേക്കും പിന്നെയും അവിശ്വാസത്തിലേക്കും മാറി കൊണ്ടിരിക്കുന്ന പാപം അല്ലാഹു പൊറുക്കുകയേയില്ലെന്ന് 4:137 ല്‍ വ്യക്തമാക്കിയിട്ടുണ്ട്: ഒരിക്കല്‍ വിശ്വസിക്കുകയും പിന്നീട് അവിശ്വസിക്കുകയും വീണ്ടും വിശ്വസിച്ചിട്ട് പിന്നെയും അവിശ്വസിക്കുകയും അനന്തരം അവിശ്വാസം കൂടിക്കൂടിവരികയും ചെയ്തവരാരോ അവര്‍ക്ക് അല്ലാഹു പൊറുത്തു കൊടുക്കുകയേ ഇല്ല. അവരെ അവന്‍ നേര്‍വഴിയിലേക്ക് നയിക്കുന്നതുമല്ല.

ക്രിസ്തു ക്രൂശിക്കപ്പെട്ടിട്ടില്ലന്നാണ് ക്വുർആൻ വ്യക്തമാക്കുന്നത്.

"അല്ലാഹുവിന്‍റെ ദൂതനായ, മര്‍യമിന്‍റെ മകന്‍ മസീഹ്‌ ഈസായെ ഞങ്ങള്‍ കൊന്നിരിക്കുന്നു എന്നവര്‍ പറഞ്ഞതിനാലും ( അവര്‍ ശപിക്കപ്പെട്ടിരിക്കുന്നു. ) വാസ്തവത്തില്‍ അദ്ദേഹത്തെ അവര്‍ കൊലപ്പെടുത്തിയിട്ടുമില്ല, ക്രൂശിച്ചിട്ടുമില്ല. പക്ഷെ ( യാഥാര്‍ത്ഥ്യം ) അവര്‍ക്ക്‌ തിരിച്ചറിയാതാവുകയാണുണ്ടായത്‌. തീര്‍ച്ചയായും അദ്ദേഹത്തിന്‍റെ ( ഈസായുടെ ) കാര്യത്തില്‍ ഭിന്നിച്ചവര്‍ അതിനെപ്പറ്റി സംശയത്തില്‍ തന്നെയാകുന്നു. ഊഹാപോഹത്തെ പിന്തുടരുന്നതല്ലാതെ അവര്‍ക്ക്‌ അക്കാര്യത്തെപ്പറ്റി യാതൊരു അറിവുമില്ല. ഉറപ്പായും അദ്ദേഹത്തെ അവര്‍ കൊലപ്പെടുത്തിയിട്ടില്ല.എന്നാല്‍ അദ്ദേഹത്തെ അല്ലാഹു അവങ്കലേക്ക്‌ ഉയര്‍ത്തുകയത്രെ ചെയ്തത്‌. അല്ലാഹു പ്രതാപിയും യുക്തിമാനുമാകുന്നു." (ക്വുർആൻ 4:157,158)

'ക്രിസ്തു ക്രൂശിക്കപ്പെട്ടിട്ടില്ല' എന്ന് പറയുമ്പോള്‍ സ്വാഭാവികമായും ഉയര്‍ന്നുവരുന്നൊരു ചോദ്യ മുണ്ട്. നാല് സുവിശേഷ കര്‍ത്താക്കളും ക്രിസ്തുവിന്റെ കുരിശുമരണത്തെക്കുറിച്ച് പ്രതിപാ ദിക്കുന്നുണ്ട്. ദൃക്‌സാക്ഷികളായ നാലുപേരുടെ മൊഴി ഏതൊരു പ്രശ്‌നത്തിലും തീര്‍പ്പുകല്‍പിക്കാ നാവശ്യമായ തെളിവാണ്. സുവിശേഷകര്‍ത്താക്കളുടെ മൊഴിയുടെ അടിസ്ഥാനത്തില്‍ ക്രിസ്തു ക്രൂശിക്കപ്പെട്ടുവെന്ന നിഗമനത്തില്‍ത്തന്നെയാണ് ഏതൊരു ന്യായാധിപനും എത്തിച്ചേരുക. എങ്കില്‍ പിന്നെ കുരിശുമരണം നടന്നിട്ടില്ലെന്ന വാദത്തിന്റെ സാധുതയെന്താണ്?

ഈ വാദം തീര്‍ച്ചയായും പരിശോധിക്കപ്പെടേണ്ടതാണ്. ഒന്നാമതായി, സുവിശേഷകര്‍ത്താക്കള്‍ കുരിശുമരണത്തിന് ദൃക്‌സാക്ഷിക ളായിരുന്നുവോയെന്ന് പരിശോധിക്കപ്പെടണം. മത്തായിയുടെ സുവിശേഷം രചിച്ചത് അപ്പോസ്തലനായ മത്തായിയാണെന്ന് ഖണ്ഡിതമായി പറയാന്‍ ബൈബിള്‍ ചരിത്രത്തെക്കുറിച്ച് പഠിച്ചവരൊന്നും മിനക്കെടുന്നില്ല. ഇത് മത്തായിയുടെ പേരില്‍ മറ്റാരോ രചിച്ച താവാനാണ് സാധ്യതയെന്നാണ് വേദപുസ്തക നിഘണ്ടു പറയു ന്നത്. (റവ. എ.സി. ക്ലേയിറ്റന്‍: വേദ പുസ്തക  നിഘണ്ടു. പേജ്: 312. ) പത്രോസിന്റെ ദ്വിഭാഷിയായിരുന്ന മാര്‍ക്കോസ്, ക്രിസ്തുവിനെ കണ്ടിട്ടുപോലുമില്ല.'(Ibid പേജ് 322. ) പൗലോസിന്റെ ശിഷ്യനായ ലൂക്കോസിന്റെ സ്ഥിതിയും തഥൈവ.  യോഹന്നാന്‍ സുവിശേഷത്തിന്റെ കര്‍ത്താവിനെ സംബന്ധിച്ച് തര്‍ക്കങ്ങള്‍ പണ്ഡിത ര്‍ക്കിടയില്‍ ഇന്നും നിലനില്‍ക്കുന്നു.( Ibid പേജ് 430.) ഇതില്‍നിന്ന് കുരിശുമരണം റിപ്പോര്‍ട്ടു ചെയ്തു വെന്ന് പറയുന്ന സുവിശേഷകര്‍ത്താക്കളിലാരുംതന്നെ പ്രസ്തുത സംഭവത്തിനു ദൃക്‌സാക്ഷികളായി രുന്നില്ലെന്ന് സുതരാം വ്യക്തമാവുന്നു.

ഇനി നാം കോടതിയിലേക്ക് കടക്കുക. ഒരു സംഭവത്തിന് നാല് ദൃക്‌സാക്ഷികള്‍ കോടതിയില്‍ പ്രത്യക്ഷപ്പെട്ടുവെന്നിരിക്കട്ടെ, ഒരേ സംഭവത്തെക്കുറിച്ച് നാല് പേര്‍ പരസ്പരവിരുദ്ധമായ വിശദീകരണങ്ങള്‍ നല്‍കിയാല്‍ കോടതി എന്തുവിധിക്കും? ഈ നാലുപേരും തെളിവിന് കൊള്ളില്ലെന്നും മറ്റു തെളിവുകളില്ലെങ്കില്‍ പ്രസ്തുത സംഭവം നടന്നുവെന്നു പറയുക വയ്യെന്നുമായിരിക്കും കോടതിവിധി. ഇതേ അവസ്ഥയാണ് കുരിശുമരണത്തിനുമുള്ളത്. കുരിശുമരണവും അനന്തരസംഭവങ്ങളും വിശദീകരിക്കുന്നിടത്ത് നാലു സുവിശേഷങ്ങളും പരസ്പര വിരുദ്ധങ്ങളായ ഒട്ടനവധി പ്രസ്താവനകള്‍ നടത്തുന്നതായി കാണാന്‍ കഴിയും.

യേശുവിനെ ഒറ്റുകൊടുക്കുന്നത് മുതല്‍ ആരംഭിക്കുന്നു വൈരുധ്യങ്ങള്‍, പ്രസ്തുത സംഭവം മത്തായി വിവരിക്കുന്നത് ഇങ്ങനെയാണ്.

'അവന്‍ സംസാരിച്ചുകൊണ്ടിരിക്കുമ്പോള്‍തന്നെ പന്ത്രണ്ടു പേരില്‍ ഒരുവനായ യൂദാസ് അവിടെ യെത്തി. അവനോടുകൂടെ പ്രധാനപുരോഹിതന്മാരുടെയും ജനപ്രമാണികളുടെയും അടുക്കല്‍നിന്ന് വാളും വടികളുമായി ഒരു ജനക്കൂട്ടവും വന്നിരുന്നു. ഒറ്റുകാരന്‍ അവര്‍ക്ക് ഈ അടയാളം നല്‍കി യിരുന്നു. ഞാന്‍ ആരെ ചുംബിക്കുന്നുവോ, അവന്‍തന്നെ. അവനെ പിടിച്ചുകൊള്ളുക. അവന്‍ പെട്ടെന്ന് യേശുവിന്റെ അടുത്തുചെന്ന് 'ഗുരോ സ്വസ്തി' എന്നു പറഞ്ഞു അവനെ ചുംബിച്ചു. യേശു അവനോട് ചോദിച്ചു. സ്‌നേഹിതാ നീ എന്തിനാണ് വന്നത്? അപ്പോള്‍ അവര്‍ മുന്നോട്ടുവന്നു യേശു വിനെ പിടിച്ചു.'(മത്തായി 26: 47-50.)

ഈ സംഭവം ലൂക്കോസ് വിശദീകരിക്കുന്നത് നോക്കുക. 'അവന്‍ ഇത് പറഞ്ഞുകൊണ്ടിരിക്കുമ്പോള്‍ ഒരു ജനക്കൂട്ടം അവിടെവന്നു. പന്ത്രണ്ടു പേരില്‍ ഒരുവനായ യൂദാസാണ് അവരുടെ മുന്നില്‍ നടന്നി രുന്നത്. യേശുവിനെ ചുംബിക്കാന്‍ അവന്‍ മുന്നോട്ടുവന്നു. യേശു അവനോട് ചോദിച്ചു. യൂദാസെ നീ ചുംബനം കൊണ്ടോ മനുഷ്യ പുത്രനെ ഒറ്റുകൊടുക്കുന്നത്'?( ലൂക്കോസ് 22:47, 48.)

യേശുവിനെ ഒറ്റിക്കൊടുത്തു ബന്ധിച്ച സംഭവം യോഹന്നാന്‍ റിപ്പോര്‍ട്ടു ചെയ്യുന്നത് ഇതില്‍ നിന്നെല്ലാം വളരെ വ്യത്യാസമായി ക്കൊണ്ടാണ്. 'യൂദാസ് ഒരു ഗണം പടയാളികളെയും പുരോഹിത പ്രമുഖന്മാരുടെയും ഫരിസേയരുടെയും അടുക്കല്‍നിന്ന് സേവകരെയും കൂട്ടി പന്തങ്ങളും വിളക്കു കളും ആയുധങ്ങളുമായി അവിടെയെത്തി. തനിക്ക് സംഭവിക്കാനിരിക്കുന്നതെല്ലാം അറിഞ്ഞിരുന്ന യേശു മുന്നോട്ടുവന്ന് അവരോട് ചോദിച്ചു. നിങ്ങള്‍ ആരെയാണ് അന്വേഷിക്കുന്നത്? അവന്‍ പറഞ്ഞു: നസാറായക്കാരനായ യേശുവിനെ. യേശു പറഞ്ഞു: അത് ഞാനാണ്. അവനെ ഒറ്റിക്കൊ ടുത്ത യൂദാസും അവരോട് കൂടെ ഉണ്ടായിരുന്നു. ഞാനാണ് എന്ന് അവന്‍ പറഞ്ഞപ്പോള്‍ അവര്‍ പിന്‍വലിയുകയും നിലംപതിക്കുകയും ചെയ്തു. അവന്‍ വീണ്ടും ചോദിച്ചു. നിങ്ങള്‍ ആരെ അന്വേ ഷിക്കുന്നു. അവര്‍ പറഞ്ഞു. നസാറായക്കാരനായ യേശുവിനെ യേശു പ്രതിവചിച്ചു: ഞാനാണ് എന്നു നിങ്ങളോടു പറഞ്ഞുവല്ലോ. നിങ്ങള്‍ എന്നെ യാണ് അന്വേഷിക്കുന്നതെങ്കില്‍ ഇവര്‍ പൊയ്‌ക്കൊള്ളട്ടെ' (യോഹന്നാന്‍ 18: 3-8)

ഇതുപോലെ കുറേയധികം പരസ്പര വിരുദ്ധമായ പ്രസ്താവനകള്‍ നടത്തുന്നുണ്ട് സുവിശേഷങ്ങള്‍. യേശുവിനെ ബന്ധിച്ച ശേഷം പ്രധാന പുരോഹിതനായ കയ്യാഫാസിന്റെ അടുക്കലേക്കാണ് ആദ്യം കൊണ്ടുപോയതെന്ന് മത്തായിയും,( മത്തായി 26:57.) കയ്യാഫാസിന്റെ അമ്മായിഅപ്പനായ അന്നാ സിന്റെ അടുക്കലേക്കാണെന്ന് യോഹന്നാനും പറയുന്നു.( യോഹന്നാന്‍ 18:13. 87.  ) ഗോല്‍ഗോഥ മല യിലേക്ക് കുരിശ് ചുമന്നത് യേശു സ്വയമാണെന്ന് യോഹന്നാനും(യോഹന്നാന്‍ 19:17.)കാറേനേയക്കാ രനായ ശിമായോനാണെന്ന്(മത്തായി 27:32.) മത്തായിയും പ്രസ്താവിക്കുന്നു. ശേയുവിനോട് കൂടെ ക്രൂശിക്കപ്പെട്ട രണ്ട് കള്ളന്മാരില്‍ ഒരുത്തന്‍ മാത്രം അദ്ദേഹത്തെ തള്ളിപ്പറഞ്ഞുവെന്നും മറ്റവന്‍ അദ്ദേഹത്തില്‍ വിശ്വസിച്ചുവെന്നും ലൂക്കോസ്(ലൂക്കോസ് 23:42. ) പറയുമ്പോള്‍, രണ്ടു പേരും അവനെ പരിഹസിച്ചുവെന്നാണ് മത്തായി (മത്തായി 27:44. ) പറയുന്നത്. ഇങ്ങനെ കുറെയധികം വൈരുധ്യങ്ങളുണ്ട്.

യേശുവിന്റെ മരണത്തെക്കുറിച്ച് വ്യത്യസ്ത സുവിശേഷങ്ങള്‍ എങ്ങനെയാണ് വിവരിക്കുന്നതെ ന്നുകൂടി പരിശോധിക്കാം. 'ആറാം മണിക്കൂര്‍ മുതല്‍ ഒമ്പതാം മണിക്കൂര്‍ വരെ ഭൂമിയിലെങ്ങും അന്ധകാരം വ്യാപിച്ചു. ഏകദേശം ഒമ്പതാം മണിക്കൂറായപ്പോള്‍ യേശു ഉച്ചത്തില്‍ നിലവിളിച്ചു. 'ഏലി, ഏലി, ലമാ സബക്ഥാനീ' അതായത് എന്റെ ദൈവമേ, എന്റെ ദൈവമേ എന്തുകൊണ്ടു നീ എന്നെ ഉപേക്ഷിച്ചു! അടുത്തുനിന്നിരുന്നവരില്‍ ചിലര്‍ ഇതുകേട്ടു പറഞ്ഞു. അവന്‍ ഏലിയായെ വിളിക്കുന്നു. ഉടന്‍ അവരില്‍ ഒരാള്‍ ഓടിച്ചെന്ന് നീര്‍പ്പഞ്ഞിയെടുത്തു വിനാഗരിയില്‍ മുക്കി, ഒരു ഞാണിന്മേല്‍ ചുറ്റി അവനു കുടിക്കാന്‍ കൊടുത്തു. അപ്പോള്‍ മറ്റുള്ളവര്‍ പറഞ്ഞു:നില്‍ക്കൂഏലിയാ വന്ന് അവനെ രക്ഷിക്കുമോയെന്ന് കാണട്ടെ. യേശു ഉച്ചത്തില്‍ നിലവിളിച്ചുകൊണ്ട് ജീവന്‍ വെടിഞ്ഞു.'(മത്തായി 27:45-50. )

ഈ സംഭവം യോഹന്നാന്‍ വിശദീകരിക്കുന്നതിങ്ങനെയാണ്. 'അനന്തരം, എല്ലാം നിറവേറ്റിക്കഴിഞ്ഞു വെന്നറിഞ്ഞ് തിരുവെഴുത്ത് പൂര്‍ത്തിയാക്കാന്‍ വേണ്ടി യേശു പറഞ്ഞു: എനിക്കു ദാഹിക്കുന്നു. ഒരു പാത്രം നിറയെ വിനാഗരി അവിടെയുണ്ടായിരുന്നു. അവര്‍ വിനാഗരിയില്‍ കുതിര്‍ത്ത ഒരു നീര്‍ പ്പഞ്ഞി ഹിസോപ്പ് ചെടിയുടെ തണ്ടില്‍വെച്ച് അവന്റെ ചുണ്ടോടടുപ്പിച്ചു യേശു വിനാഗരി സ്വീക രിച്ചിട്ട് പറഞ്ഞു: എല്ലാം പൂര്‍ത്തിയായിരിക്കുന്നു. അവന്‍ തലചായ്ച്ച് ആത്മാവിനെ സമര്‍പ്പിച്ചു. '(യോഹന്നാന്‍ 19:28-30.)

ക്രൂശിതന്റെ അവസാനത്തെ വാക്കുകളെന്തായിരുന്നുവെന്നു പോലും ഖണ്ഡിതമായി, ഏകസ്വര ത്തില്‍ പറയാന്‍ പറ്റാത്ത സുവിശേഷകര്‍ത്താക്കളെ പ്രസ്തുത സംഭവത്തിന്റെ ദൃക്‌സാക്ഷി കളാ ക്കാന്‍ പറ്റുമോ? ഒരു പ്രാവശ്യമെങ്കിലും കോടതി വരാന്തയില്‍ പോയിട്ടുള്ളവരെല്ലാം ഉത്തരം പറയും. 'ഇല്ല' അവരുടെ പ്രസ്താവനകള്‍ പരസ്പരവിരുദ്ധമാണ്. അവരെയൊരിക്കലും സാക്ഷ്യ ത്തിന് കൊള്ളുകയില്ല.

ക്രിസ്തുവിന്റെ മരണത്തെക്കുറിച്ച് വ്യക്തമായ വിവരം നമുക്കെവിടെനിന്നുകിട്ടുമെന്ന് പരിശോ ധിക്കുമ്പോള്‍ നമുക്ക് മുന്നില്‍ പിന്നീട് വരുന്നത് ബര്‍ണബാസിന്റെ സുവിശേഷമാണ്. പുതിയ നിയ മത്തിലെവിടെയും കാണാത്ത ബര്‍ണബാസിന്റെ സുവിശേഷ ത്തെക്കുറിച്ച് കേള്‍ക്കുന്ന ക്രൈസ്ത വര്‍ ചിലപ്പോള്‍ അത്ഭുതപ്പെട്ടേക്കാം.

 ബൈബിള്‍ ഒരുവട്ടം വായിച്ചിട്ടുള്ളവര്‍ക്കൊന്നും ബര്‍ണബാസി നെപ്പറ്റി പറഞ്ഞുകൊടുക്കേണ്ട തായി വരികയില്ല. ബര്‍ണബാസാണ് പൗലോസിനെ അപ്പോസ്തലന്മാര്‍ക്കിടയിലേക്ക് കൊണ്ടു വന്നത്.( അപ്പോ. പ്രവൃ. 9:47.) 'ബര്‍ണാബാസ് സുവിശേഷ വേലകള്‍ ചെയ്തുകൊണ്ട് ചുറ്റി നടന്ന തായി അപ്പോസ്തല പ്രവൃത്തികളില്‍ പലപ്രാവശ്യം പറഞ്ഞി രിക്കുന്നു.

ക്രിസ്തുവിനുശേഷം ഒന്നാം നൂറ്റാണ്ടിലും, രണ്ടാം നൂറ്റാണ്ടിലും പ്രചാരത്തിലിരുന്ന സുവിശേഷങ്ങ ളിലൊന്നാണ് ബര്‍ണബാസിന്റെ സുവിശേഷം. ഈ സുവിശേഷം ക്രിസ്തുവിനെറ ദിവ്യത്വത്തെ നിരാകരിക്കുകയും ക്രിസ്തുവിന്റെ സുവിശേഷത്തിലെ സുപ്രധാന ഉപദേശമായ ഏകദൈവ വിശ്വാസത്തിന് ഊന്നല്‍ നല്‍കുകയും ചെയ്യുന്നു. പ്രാകൃത റോമന്‍ പുരാണങ്ങളില്‍നിന്ന് കടമെടുത്ത വിശ്വാസങ്ങളും ആചാരങ്ങളും ക്രിസ്ത്യാനികള്‍ക്കിടയില്‍ പ്രചരിപ്പിച്ചതിനെതിരെ ശക്തമായി നിലകൊണ്ട ഇറാനിയൂസ് (130þ200 CE)  അദ്ദേഹത്തിറെ കാഴ്ചപ്പാടുകളെ ന്യായീകരിക്കുന്നതിനു വേണ്ടി ബര്‍ണബാസിന്റെ സുവിശേഷത്തില്‍നിന്നും ഉദ്ധരിക്കുന്നുണ്ട്. ഇതില്‍നിന്ന് ഒന്ന്, രണ്ട് നൂറ്റാ ണ്ടുകളില്‍ പ്രസ്തുത സുവിശേഷത്തിനുണ്ടായിരുന്ന പ്രാധാന്യം വ്യക്തമാവുന്നുണ്ട്. അലക്‌സാ ണ്ട്രിയ ചര്‍ച്ചുകള്‍ (CHURCHES OF ALEXANDRIA) ക്രിസ്താബ്ദം 325 വരെ ബര്‍ണബാസിന്റെ സുവി ശേഷത്തെ കാനോനിക സുവിശേഷങ്ങളിലൊന്നായി അംഗീകരിച്ചിരുന്നുവെന്നതിന് തെളിവുകളുണ്ട്.

ക്രിസ്താബ്ദം 325-ല്‍ നടന്ന നിഖിയാ കൗണ്‍സിലിന്റെ തീരുമാനപ്രകാരം ഹിബ്രുവിലുള്ള സുവിശേ ഷങ്ങളുടെ എല്ലാ കൈയെഴുത്തു പ്രതികളും നശിപ്പിക്കപ്പെടുകയുണ്ടായി. ഹിബ്രു സുവിശേഷങ്ങള്‍ കൈവശം വെക്കുന്നവര്‍ക്ക് മരണശിക്ഷ വിധിച്ചുകൊണ്ടുള്ള കല്‍പന പുറപ്പെടുവിച്ചു. ഈ നിരോധ നാജ്ഞകളെയെല്ലാം അതിജീവിച്ചുകൊണ്ട് നിലനിന്ന സുവിശേഷമത്രേ ബര്‍ണബാസിന്റെ സുവി ശേഷം. വ്യക്തികളില്‍നിന്നും വ്യക്തികളിലേക്ക് കൈമാറ്റം ചെയ്യപ്പെട്ടുകൊണ്ട് 1738-ല്‍ വിയന്നയിലെ ഇംപീരിയല്‍ ലൈബ്രറിയില്‍ പ്രസ്തുത ഗ്രന്ഥം എത്തി. ഇന്ന് ഗ്രന്ഥത്തിന്റെ കൈയെഴുത്ത് കോപ്പി പ്രസ്തുത ലൈബ്രറിയിലാണുള്ളത്.

ക്രിസ്തുവല്ല, പ്രത്യുത ഒറ്റുകാരനായ യൂദാസാണ് മരത്തില്‍ തറച്ചുകൊല്ലപ്പെട്ടതെന്നാണ് ബര്‍ണ ബാസിന്റെ സുവിശേഷം പറയുന്നത്. യൂദാസാണത്രേ കുരിശില്‍ തൂങ്ങിക്കിടന്നുകൊണ്ട് ഇങ്ങനെ വിലപിച്ചത്. 'ദൈവമേ, നീ എന്തിനാണെന്നെ ഉപേക്ഷിച്ചത്, കുറ്റവാളി രക്ഷപ്പെടുന്നതും ഞാന്‍ അന്യാ യമായി മരിക്കുന്നതും കണ്ടുകൊണ്ട്? (THE GOSPEL OF BARNABAS: TRANSLATED BY LONSDALE AND LAURA RAGG, Chapter 217.)

വഞ്ചകനായ യൂദാസ് പിടിയിലായ സംഭവത്തെപ്പറ്റി ബര്‍ണബാസ് പറയുന്നതിങ്ങനെയാണ്. ''യേശു നിന്നിരുന്ന സ്ഥലത്തിനടുത്ത് യൂദാസും പട്ടാളക്കാരുമെത്തിയപ്പോള്‍ യേശു ജനങ്ങളുടെ ആരവം  കേട്ടുകൊണ്ട് വീട്ടില്‍നിന്നും പിന്‍വലിഞ്ഞു. പതിനൊന്ന് അപ്പോസ്തലന്മാരും അപ്പോള്‍ ഉറങ്ങുക യായിരുന്നു.

അപ്പോള്‍ ദൈവം, അവന്റെ സേവകന്‍ അപകടത്തിലാണെന്ന് മനസ്സിലാക്കിക്കൊണ്ട് അവന്റെ മന്ത്രിമാരായ ഗബ്രിയേല്‍, മിഖായേല്‍, റാഫേല്‍, യുറിയേല്‍ എന്നിവരോട് യേശുവിനെ ലോക ത്തില്‍നിന്ന് പുറത്തേക്കെടുക്കാന്‍ കല്‍പിച്ചു. തെക്കുഭാഗത്തേക്ക് തുറന്നിരിക്കുന്ന ജനവാതിലിലൂടെ യേശുവിനെ വിശുദ്ധ മാലാഖമാര്‍ പുറത്തേക്കെടുത്തു. അവര്‍ അദ്ദേഹത്തെ വഹിച്ചുകൊണ്ട് ദൈവാനുഗ്രഹം എന്നെന്നും നിലനില്‍ക്കുന്ന മാലാഖമാരുടെ സഹവാസത്തില്‍ മൂന്നാം ആകാശ ത്തില്‍ കൊണ്ടു ചെന്നുവെച്ചു.

യേശു എടുക്കപ്പെട്ട ഉടന്‍തന്നെ യൂദാസ് മറ്റുള്ളവര്‍ക്കു മുന്നില്‍ മുറിയിലേക്ക് എടുത്തുചാടി. എല്ലാ അപ്പോസ്തലന്മാരും ഉറങ്ങുകയായിരുന്നു. അപ്പോള്‍ അല്‍ഭുതകാരനായ ദൈവം അത്ഭുതം പ്രവര്‍ ത്തിച്ചു. യൂദാസിന്റെ സംസാരവും മുഖവും യേശുവിന്‍േറത് പോലെയായിത്തീര്‍ന്നു. ഞങ്ങ ളെല്ലാം അദ്ദേഹം യേശുവാണെന്ന് വിചാരിക്കുന്ന പരുവത്തിലായി മാറി. ഞങ്ങളെ ഉണര്‍ത്തി ക്കൊണ്ട് അവന്‍ ഗുരു എവിടെയാണെന്ന് തിരക്കി. അപ്പോള്‍ ഞങ്ങള്‍ അമ്പരന്നുകൊണ്ടു മറുപടി പറഞ്ഞു. 'കര്‍ത്താവേ, അങ്ങ് ഞങ്ങളുടെ ഗുരുവല്ലയോ, ഇപ്പോള്‍ ഞങ്ങളെയെല്ലാം അങ്ങു മറന്നുപോയോ? അവന്‍ പുഞ്ചിരിച്ചുകൊണ്ടു പറഞ്ഞു: ഞാന്‍ യൂദാസ് ഇസ്‌ക്കാരിയോസാണെന്ന് മനസ്സിലാക്കാത്ത നിങ്ങള്‍ ഇപ്പോള്‍ വിഡ്ഢികള്‍തന്നെ!'

ഇതുപറഞ്ഞുകൊണ്ടു നില്‍ക്കുമ്പോള്‍ പട്ടാളക്കാര്‍ പ്രവേശിച്ചു. എല്ലാ നിലക്കും യേശുവിനെപ്പോ ലെയായി മാറിയിരുന്ന യൂദാസിന്റെ മുകളില്‍ കൈവെച്ചു. ഞങ്ങള്‍ക്കു ചുറ്റും നിരന്നിരുന്ന പട്ടാള ക്കാര്‍ക്കിടയില്‍നിന്നും ഓടുമ്പോള്‍ യൂദാസ് പറയുന്നത് ഞങ്ങള്‍ക്ക് കേള്‍ക്കാമായിരുന്നു. ലിനെന്‍ തുണിയില്‍ പൊതി ഞ്ഞിരുന്ന യോഹന്നാന്‍ ഉണര്‍ന്നുകൊണ്ട് ഓടിയപ്പോള്‍ ഒരു പട്ടാളക്കാരന്‍ ലിനെന്‍തുണിയില്‍ കയറിപ്പിടിച്ചതിനാല്‍ അവന്റെ തുണിയഴിയുകയും നഗ്‌നനായി അവന്‍ ഓടി രക്ഷപ്പെടുകയും ചെയ്തു. യേശുവിന്റെ പ്രാര്‍ഥന ദൈവം ശ്രവിക്കുകയും പതിനൊന്ന് പേരും രക്ഷപ്പെടുകയും ചെയ്തു.

പട്ടാളക്കാര്‍ യൂദാസിനെ പിടിച്ചുബന്ധിച്ചത് അയാളെ അവഹേ ളിച്ചുകൊണ്ടായിരുന്നു. അയാള്‍ താന്‍ യേശുവല്ലെന്ന് നിഷേധിച്ചുകൊണ്ടിരുന്നു. പട്ടാളക്കാര്‍ അയാളെ കളിയാക്കിക്കൊണ്ട് പറഞ്ഞു 'സര്‍' പേടിക്കേണ്ട, താങ്കളെ ഇസ്രായീലിന്റെ രാജാവാക്കാനാണ് ഞങ്ങള്‍ വന്നിരിക്കുന്നത്. നിങ്ങള്‍ രാജാധികാരം നിഷേധിക്കുകയില്ലെന്ന് ഞങ്ങള്‍ക്കറിയാവുന്നതിനാലാണ് ഞങ്ങള്‍ നിങ്ങളെ ബന്ധിക്കു ന്നത്''. (Ibid Chapter 215, 217.)

യേശുവിനെ ചതിക്കാന്‍ ശ്രമിച്ച യൂദാസ് കുഴിച്ച കുഴിയില്‍ അദ്ദേഹം വീണുവെന്നും അയാളാണ് ക്രൂശിക്കപ്പെട്ടതെന്നും ബര്‍ണ ബാസിന്റെ സുവിശേഷം വ്യക്തമാക്കുന്നു. ഇവിടെ ക്രിസ്തുവിന്റെ പ്രവചനം പുലരുന്നു. 'മനുഷ്യപുത്രന്‍ എഴുതപ്പെട്ടതുപോലെ പോകുന്നു. പക്ഷേ, മനുഷ്യപുത്രനെ ഒറ്റിക്കൊടുക്കുന്നവനാരോ അവന് ദുരിതം. ജനിക്കാതിരുന്നെങ്കില്‍ അവന്നു നന്നായിരുന്നു. (മത്തായി 26:23, 20.) മനുഷ്യ പുത്രന്‍ ദൈവിക വിധിപ്രകാരം പോയി. ഒറ്റിക്കൊടുത്ത യൂദാസ് കുരിശില്‍ കിടന്നുപിടച്ചപ്പോള്‍ വിചാരിച്ചിരിക്കണം. 'ഞാന്‍ ജനിക്കാതിരുന്നെങ്കില്‍ എത്ര നന്നായിരുന്നു'വെന്ന്.

ഏതായിരുന്നാലും, ക്രിസ്തു ക്രൂശീകരിക്കപ്പെടുകയോ കൊല്ലപ്പെടുകയോ ചെയ്തിട്ടില്ലെന്ന ക്വുർആനികപ്രസ്താവനയെ വെല്ലാനാവശ്യമായ തെളിവുകളൊന്നും ബൈബിളോ മറ്റു ക്രൈസ്തവരചനകളോ നൽകുന്നില്ല.

 
വിഷയവുമായി ബന്ധപ്പെട്ട വീഡിയോ

മറ്റു പ്രവാചകന്മാരെപ്പോലെ ക്രിസ്തുവും നിരവധി അത്ഭുതങ്ങൾ ചെയ്തതായി ക്വുർആൻ സാക്ഷ്യപ്പെടുത്തുന്നു. ബൈബിളില്‍ പ്രതിപാദിച്ചിട്ടില്ലാത്ത ചില അത്ഭുതങ്ങളും ക്രിസ്തു ചെയ്തതായി ഖുര്‍ആന്‍ പഠിപ്പിക്കുന്നുണ്ട്. അതില്‍ ഒന്നാമത്തേതാണ് തൊട്ടിലില്‍ വെച്ചുള്ള അദ്ദേഹത്തിന്റെ സംസാരം. സ്വമാതാവിന്റെ മുന്നില്‍ വെച്ച് തൊട്ടിലില്‍ കിടക്കുന്ന ശിശു പ്രഖ്യാപിച്ചു. 'ഞാന്‍ ദൈവദാസനാ കുന്നു. അവന്‍ എനിക്ക് വേദഗ്രന്ഥം നല്‍കുകയും എന്നെയവന്‍ പ്രവാചകനാക്കുകും ചെയ്തി രിക്കുന്നു.' (ഖുര്‍ആന്‍ 19:30. ) ഈ അത്ഭുതം ബൈബിളിലൊരിടത്തും പറഞ്ഞിട്ടില്ല.

ക്രിസ്തു ചെയ്ത അത്ഭുതങ്ങൾ ഖുർആൻ അക്കമിട്ട് പ്രസ്താവിക്കുന്നുണ്ട്. യേശു പറഞ്ഞതായി ഖുര്‍ആന്‍ പഠിപ്പിക്കുന്നു. 'നിങ്ങളുടെ രക്ഷി താവിങ്കല്‍നിന്നുള്ള ദൃഷ്ടാന്തവും കൊണ്ടാണ് ഞാന്‍ നിങ്ങളുടെ അടുത്ത് വന്നിരിക്കുന്നത്. പക്ഷിയുടെ ആകൃതിയില്‍ ഒരു കളിമണ്‍ രൂപം നിങ്ങള്‍ക്കുവേണ്ടി ഞാന്‍ ഉണ്ടാക്കുകയും എന്നിട്ട് ഞാനതില്‍ ഊതുമ്പോള്‍ അല്ലാഹുവിന്റെ അനുവാദപ്രകാരം അതൊരു പക്ഷിയായിത്തീരുകയും ചെയ്യും. അല്ലാഹുവിന്റെ അനുവാദ പ്രകാരം ജന്മനാ കാഴ്ചയില്ലാത്തവനെയും പാണ്ഡുരോഗിയെയും ഞാന്‍ സുഖപ്പെടുത്തുകയും മരിച്ചവരെ ജീവിപ്പിക്കുകയും ചെയ്യും. നിങ്ങള്‍ ഭക്ഷിക്കുന്നതിനേക്കുറിച്ചും, നിങ്ങള്‍ നിങ്ങളുടെ വീടുകളില്‍ സൂക്ഷിച്ചുവെക്കുന്നതിനെപ്പറ്റിയും ഞാന്‍ നിങ്ങള്‍ക്ക് പറഞ്ഞറിയിച്ചു തരികയും ചെയ്യും. തീര്‍ച്ചയായും അതില്‍ നിങ്ങള്‍ക്ക് ദൃഷ്ടാന്തമുണ്ട്. നിങ്ങള്‍ വിശ്വസിക്കുന്നവരാണെങ്കില്‍.' (ഖുര്‍ആന്‍ 3:49.)

ഈ സൂക്തത്തില്‍ പറയുന്ന 'കളിമണ്‍ പക്ഷിക്കു ജീവന്‍ നല്‍കുക' 'രഹസ്യങ്ങള്‍ പറഞ്ഞുകൊടുക്കുക' തുടങ്ങിയ അത്ഭുതങ്ങള്‍ ക്രിസ്തു ചെയ്തതായി ബൈബിളിലെവിടെയും പ്രതിപാദിക്കുന്നില്ല. സുവിശേഷങ്ങള്‍ വിവരിക്കുന്നതിലുപരി മഹത്തായ അത്ഭുതങ്ങള്‍ ചെയ്ത വ്യക്തിയായി ക്രിസ്തുവിനെ അംഗീകരിക്കുന്ന ഗ്രന്ഥമാണ് ഖുര്‍ആന്‍.

ക്രിസ്തു കാണിച്ച അത്ഭുതങ്ങളുടെ വെളിച്ചത്തില്‍ അദ്ദേഹം ദൈവമാണെന്നും -ത്രിയേകത്വ ദൈവ ത്തിലെ ഒരു ആളത്വം-വാ ദിക്കപ്പെടാറുണ്ട്. അദ്ദേഹം അത്തരമൊരു അവകാശവാദം ഉന്നയിച്ച തായി സുവിശേഷങ്ങളോ മറ്റു ലേഖനങ്ങളോ ഒന്നും വ്യക്തമാക്കുന്നില്ല. വിനയത്തിന്റെ പര്യായ മായ മിശിഹ ഇങ്ങനെയാണ് പറഞ്ഞത്. 'സത്യം സത്യമായി ഞാന്‍ നിങ്ങളോട് പറയുന്നു. പിതാവ് ചെയ്തു കാണുന്നതല്ലാതെ പുത്രന് സ്വന്തം ഇഷ്ടമനുസരിച്ച് ഒന്നും പ്രവര്‍ത്തിക്കാന്‍ സാധിക്കുകയില്ല. ' (യോഹന്നാന്‍ 5:19.)

പിതാവ് മാത്രമാണ് സര്‍വശക്തനെന്നും 'പുത്ര'നായ ക്രിസ്തു അദ്ദേഹത്തിന്റെ സ്വന്തം കഴിവു കൊ ണ്ടല്ല, പ്രസ്തുത സര്‍വശക്തനായ ദൈവത്തിന്റെ അനുഗ്രഹം കൊണ്ട് മാത്രമാണ് അത്ഭുതങ്ങള്‍ കാണിച്ചതെന്നുമുള്ള സത്യമാണ് ഈ വചനത്തിലൂടെ പ്രകടമാക്കപ്പെടുന്നത്. ക്രിസ്തുവിനെ അയച്ച ദൈവത്തില്‍ വിശ്വസിക്കുകയും ക്രിസ്തുവിന്റെ വചനങ്ങള്‍ പ്രകാരം ജീവിക്കുകയും ചെയ്യാന്‍ ജനങ്ങളെ ഉപദേശിക്കുന്ന യേശുവിന്  താന്‍ ദൈവത്താല്‍ നിയോഗിതനായ പ്രവാചകനാണെന്ന് വ്യക്തമാക്കുന്നതിനുവേണ്ടി നല്‍കപ്പെട്ടവനായിരുന്നു ഈ അമാനുഷിക ദൃഷ്ടാന്തങ്ങള്‍. (യോഹന്നാന്‍ 5:23.)

ദൈവത്തിനു മാത്രം സാധിക്കുന്ന കാര്യങ്ങളില്‍ പലതും ക്രിസ്തു ചെയ്തിട്ടുണ്ട്. മരിച്ചവരെ പുനരു ജ്ജീവിപ്പിച്ചത് ഒരുദാഹരണം. ഇവ അദ്ദേഹത്തിന്റെ ദിവ്യത്വത്തെയാണ് സൂചിപ്പിക്കുന്നതെന്ന് പലപ്പോഴും വാദിക്കപ്പെടാറുണ്ട്. ഈ വാദത്തില്‍ യാതൊരു കഴമ്പുമില്ല. പ്രവാചകന്മാര്‍ക്ക് ദൈവം നല്‍കുന്ന ദൃഷ്ടാന്തങ്ങളെല്ലാംതന്നെ മനുഷ്യകഴിവിന്നതീതമായവയായിരിക്കും. ഒരു മനുഷ്യനും വടിയെ പാമ്പാക്കി മാറ്റുവാന്‍ കഴിയില്ല. മായാജാലക്കാരന്‍ വടി പാമ്പാണെന്ന് തോന്നിപ്പിക്കുക മാത്രമാണ് ചെയ്യുന്നത്. മോശെ പ്രവാചകനാകട്ടെ തന്റെ വടിയെ യഥാര്‍ഥത്തിലുള്ള ഒരു പാമ്പാക്കി മാറ്റുകയാണ് ചെയ്തത്. ഇതുപോലുള്ള അമാനുഷിക പ്രവര്‍ത്തനങ്ങള്‍ മാത്രമാണ് ക്രിസ്തുവും ചെയ്തത്.

ബൈബിള്‍ പഴയ നിയമം പരിശോധിച്ചാല്‍ ക്രിസ്തുവിന് തുല്യവും അതിനേക്കാളധികവും അത്ഭുത ങ്ങള്‍ കാണിച്ച പ്രവാചകന്മാരെക്കുറിച്ച് വിവരങ്ങള്‍ വായിക്കാനാവും. ഏലീശാ ദീര്‍ഘദര്‍ശി ശൂനേംകാരിയുടെ മരിച്ച കുട്ടിയെ ജീവിപ്പിച്ചതായും(. II  രാജാക്കന്മാര്‍ 4:32-37. )ഇരുപത് അപ്പം കൊണ്ട് നൂറുപേരെ തീറ്റിയതായും(II രാജാക്കന്മാര്‍ 4:42-44. ) സിറിയാ രാജാവി ന്റെ സേനാപതി നാമാന്റെ കുഷ്ഠരോഗം സുഖപ്പെടുത്തിയതായും (രാജാക്കന്മാര്‍ 5:1-14. )ആളുകള്‍ക്ക് അന്ധതയു ണ്ടാക്കുകയും ആ അന്ധത പിന്നീട് സുഖ പ്പെടുത്തിയതായും (II രാജാക്കന്മാര്‍ 6:18-21.) രണ്ടാം രാജാ ക്കന്മാരില്‍ കാണാന്‍ കഴിയും. ദീര്‍ഘദര്‍ശിയായ ഏലിയാവ് ഗൃഹനായിക യുടെ കുഞ്ഞിനു ജീവന്‍ കൊടുത്തതായി ഒന്നാം രാജാക്കന്മാരിലുമുണ്ട്.( I രാജാക്കന്മാര്‍ 17:17-22.)യെഹസ്‌കേല്‍ എന്ന പ്രവാ ചകന്‍ ശ്മശാനത്തിലെ അസ്ഥികളെ മാംസം കൊണ്ട് പൊതിയിപ്പി ക്കുകയും  ജീവനിടീക്കുകയും ചെയ്തതായി യെഹസ് കേല്‍ എന്ന പുസ്തകത്തിലുണ്ട്. (Iയെഹസ്‌ക്കേല്‍ 37: 6-10.)

മരിച്ചവരെ പുനരുജീജീവിപ്പിച്ചുവെന്ന കാരണത്താല്‍ ക്രിസ്തുവി നെ ദൈവമായി ഉയര്‍ത്തുകയാ ണെങ്കില്‍ ഏലീശായും ഏലിയാവും യെഹസ്‌കേലുമെല്ലാം ദൈവമായിമാറും. ത്രിയേക വിശ്വാസ ത്തില്‍ നിന്നും ക്രൈസ്തവര്‍ ബഹുദൈവ വിശ്വാസത്തിലേക്ക് പരിണമിക്കേണ്ടിവരും. അത്ഭുതങ്ങൾ ക്രിസ്തുവിന്റെ ദിവ്യത്വത്തെയല്ല, അദ്ദേഹത്തിന്റെ പ്രവാചകത്വത്തെയാണ് വെളിപ്പെടുത്തുന്നത്. ക്രിസ്തു പറഞ്ഞതായി ഖുർആൻ ഉദ്ധരിക്കുന്നു. 'നിങ്ങളുടെ രക്ഷിതാവില്‍നിന്നുള്ള ദൃഷ്ടാന്തവും ഞാന്‍ നിങ്ങള്‍ക്കു കൊണ്ടുവന്നിരിക്കുന്നു. അതിനാല്‍ നിങ്ങള്‍ അല്ലാഹുവിനെ സൂക്ഷിക്കുകയും എന്നെ അനുസരിക്കുകയും ചെയ്യുവീന്‍.'(ഖുര്‍ആന്‍ 3:50.)

 

വിശുദ്ധ ഖുര്‍ആന്‍, അവതരിപ്പിക്കപ്പെട്ട മുഴുവന്‍ വേദഗ്രന്ഥങ്ങളെ യും അംഗീകരിക്കുന്നു; ആദരിക്കുന്നു, അവയനുസരിച്ചുകൊണ്ട് ജീവിക്കേണ്ടത് അതാത് കാലഘട്ടങ്ങളിലെ ആളുകളുടെ കടമയായിരുന്നുവെന്ന് പ്രഖ്യാപിക്കുകയും ചെയ്യുന്നു. അത് ആദരിക്കേണ്ടത് മുസ്‌ലിമെന്നുള്ള നിലക്ക് - എല്ലാ പ്രവാചകന്മാരും പഠിപ്പിച്ച ആദര്‍ശവുമായി ജീവിക്കുന്നവനെന്നുള്ള നിലക്ക്- അങ്ങനെ ജീവിക്കേണ്ടത് ഒരു മുസ്‌ലിമിന്റെ കടമയാണെന്ന് പ്രഖ്യാപിക്കുകയും ചെയ്യുന്നു.

പരി ശുദ്ധ ഖുര്‍ആനിലെ മൂന്നാത്തെ അധ്യായം സൂറത്ത് ആലുഇംറാനിലെ 84-ാമത്തെ വചനം വിശ്വാ സികളോട് ആവശ്യപ്പെടുന്നു: ''പറയുക: അല്ലാഹുവിലും ഞങ്ങള്‍ക്ക് അവതരിപ്പിക്ക പ്പെട്ടതിലും ഇബ്രാഹീം, ഇസ്മാഈല്‍, ഇസ്ഹാഖ്, യഅ്ഖൂബ്, യഅ്ഖൂബ് സന്തതികള്‍ എന്നി വര്‍ക്ക് അവതരി പ്പിക്കപ്പെട്ടതിലും, മൂസാക്കും ഈസാക്കും മറ്റു പ്രവാചകന്മാര്‍ക്കും തങ്ങളുടെ രക്ഷിതാവിങ്കല്‍ നിന്ന് നല്‍കപ്പെട്ടതിലും ഞങ്ങള്‍ വിശ്വസിച്ചിരിക്കുന്നു. അവരില്‍ ആര്‍ക്കിടയിലും ഞങ്ങള്‍ വിവേചനം കല്‍പിക്കുന്നില്ല ഞങ്ങള്‍ അല്ലാഹുവിന് കീഴ്‌പെട്ടവരാകുന്നു''.

പ്രവാചക ന്മാരെല്ലാം ഒരേ പോലെ പടച്ചവനാല്‍ ആദരിക്കപ്പെടുന്നവരാണ് എന്നും അവരുടെ മാര്‍ഗ ദര്‍ശനം പൂര്‍ണമാ യിതന്നെ മനുഷ്യരെ നന്മയിലേക്ക് നയിക്കുന്നവയായിരു ന്നുവെന്നും വിശ്വസി ക്കുന്നരാണ് ഞങ്ങള്‍ എന്ന് പ്രഖ്യാപിക്കപ്പെടു വാനാണ് ഖുര്‍ആന്‍ ആവശ്യപ്പെടുന്നത്. ഇങ്ങനെ എല്ലായിടത്തേക്കും അവതരിപ്പിക്കപ്പെട്ട വേദങ്ങളില്‍ വിശ്വസിക്കേണ്ടത് ബാധ്യതയാണെന്ന് പ്രഖ്യാപിക്കുന്നതോടൊപ്പം പരിശുദ്ധഖുര്‍ആന്‍ നാല് വേദഗ്രന്ഥ ങ്ങളുടെ പേരെടുത്തു പറയുക കൂടി ചെയ്യുന്നുണ്ട്. അതില്‍ ഒന്ന് പരിശുദ്ധ ഖുര്‍ആന്‍ തന്നെയാണ്. അതോടൊപ്പം തന്നെ ഖുര്‍ആനിനു മുമ്പ് അവതരിപ്പിക്കപ്പെട്ട മൂന്നു വേദഗ്രന്ഥങ്ങളെ പേരെടുത്തുതന്നെ ഖുര്‍ആന്‍ സൂചിപ്പിക്കുന്നുണ്ട്. അതിലൊന്ന് മോശെ പ്രവാച കന് (മൂസാ നബി (അ) ക്ക്) അവതരിപ്പിക്കപ്പെട്ട തൗറാത്താണ്. പരി ശുദ്ധ ഖുര്‍ആനിലെ അഞ്ചാമത്തെ അധ്യായം സൂറത്തുല്‍ മാഇദയിലെ 44-ാത്തെ വചനത്തില്‍ ഇങ്ങനെ കാണാം: ''തീര്‍ച്ചയായും നാമാ കുന്നു മോശെ പ്രവാചകന് തൗറാത്ത് (തോറ) അവത രിപ്പിച്ചിട്ടുള്ളത്. അതില്‍ മാര്‍ഗദര്‍ശനവും പ്രകാശവും ഉണ്ട്''. അതേപോലെതന്നെ ഖുര്‍ആനിലെ 17-ാ മത്തെ അധ്യായം സൂറത്തുല്‍ ഇസ്‌റാഇലെ 55-ാ മത്തെ വചനത്തില്‍ 'ദാവൂദിന് സബൂര്‍ അവതരി പ്പിച്ചിട്ടുള്ളത് നാമാകുന്നു' എന്ന് കാണാന്‍ കഴിയും. കൂടാതെ യേശുക്രിസ്തുവിനെക്കുറിച്ച് പറയു മ്പോള്‍ ഖുര്‍ആനിലെ അഞ്ചാമത്തെ അധ്യാ യം സൂറത്തു ല്‍ മാഇദയിലെ 46-ാമത്തെ വചനത്തില്‍ അദ്ദേഹത്തിന് സുവിശേഷം -ഇഞ്ചീല്‍-അവത രിപ്പിച്ചുകൊടുക്കുകയും അതില്‍ മാര്‍ഗദര്‍ശനവും പ്രകാശവും നല്‍കുകയും ചെയ്ത വനാണ് പടച്ചവനെന്ന് പറ യുന്നത് കാണാം. ചുരുക്കത്തില്‍ പരിശുദ്ധ ഖുര്‍ആന്‍ മുമ്പ് അവതരിപ്പിക്ക പ്പെട്ട വേദഗ്രന്ഥങ്ങളെ അംഗീകരിക്കുന്നു; ആദരിക്കുന്നു. അവ അവതരിപ്പിച്ചത് പടച്ച വന്‍തന്നെയാണെന്ന് പ്രഖ്യാപിക്കുകയും ചെ യ്യുന്നു.

എന്നാല്‍ വേദഗ്രന്ഥങ്ങളുടെ അവതരണത്തിനുശേഷം  നൂറ്റാണ്ടുകള്‍ കഴിഞ്ഞ പ്പോള്‍ വേദഗ്രന്ഥങ്ങളുടെ പുറംചട്ടകളണിഞ്ഞുകൊണ്ട് മനുഷ്യരാല്‍ രചിക്കപ്പെട്ട ചില ഗ്രന്ഥങ്ങള്‍ വന്നു എന്നും ആ ഗ്രന്ഥങ്ങളാണ് മനുഷ്യരെ തിന്മയിലേക്കും തെറ്റുകളിലേക്കും അധര്‍മത്തിലേക്കും പൈശാചിക പ്രലോഭനങ്ങളിലേക്കുമെല്ലാം കൊണ്ടുപോയത് എന്നും പരിശുദ്ധ ഖുര്‍ആന്‍ വ്യക്തമാക്കുന്നു. ''സ്വന്തം കൈകള്‍ കൊണ്ട് ഗ്രന്ഥം എഴുതിയുണ്ടാ ക്കുകയും സ്വാര്‍ഥമായ താല്‍പര്യങ്ങള്‍ക്കുവേണ്ടിയും തുച്ഛമായ താല്‍പര്യങ്ങള്‍ക്കുവേണ്ടിയും അത് ദൈവികമാണെന്ന് പറയുകയും ചെയ്യുന്നവര്‍ക്കാകുന്നു നാശ''(''എന്നാല്‍ സ്വന്തം കൈകള്‍ കൊണ്ട് ഗ്രന്ഥം എഴുതിയുണ്ടാക്കുകയും എന്നിട്ട് അത് അല്ലാ ഹുവിങ്കല്‍ നിന്ന് ലഭിച്ചതാണെന്ന് പറയുകയും ചെയ്യുന്നവര്‍ക്കാകുന്നു നാശം.അത് മുഖേന വില കുറഞ്ഞ നേട്ടങ്ങള്‍ കരസ്ഥമാക്കാന്‍ വേണ്ടിയാകുന്നു (അരവിത് ചെയ്യുന്നത്) അവരുടെ കൈകള്‍ എഴുതിയ വകയിലും അവര്‍ സമ്പാദി ക്കുന്ന വകയിലും അവര്‍ക്ക് നാശം'' (ഖുര്‍ആന്‍ 2:79)

ദൈവിക വചനങ്ങളിലെ സത്യങ്ങ ളെ അസത്യങ്ങളുമായി കൂട്ടിക്കലര്‍ത്തിക്കൊണ്ട് അവതരി പ്പിക്കുകയാണ് വേദങ്ങളുടെ വക്താക്കളെന്ന് പറഞ്ഞ പുരോഹിതന്മാര്‍ ചെയ്തതെന്നും ക്വുർആൻ വ്യക്തമാക്കുന്നുണ്ട്. ''വേദക്കാരേ, നിങ്ങളെന്തിനാണ് സത്യത്തെ അസത്യവുമായി കൂട്ടിക്കലര്‍ത്തുകയും,അറിഞ്ഞുകൊണ്ട് സത്യം മറച്ചുവെക്കുകയും ചെയ്യുന്നത്?'' (ഖുര്‍ആന്‍ 3:71) വേദഗ്രന്ഥങ്ങളില്‍ പലപ്പോഴും മനുഷ്യരുടെ കൈകടത്തലു കള്‍ നടക്കുകയും മനുഷ്യര്‍ സ്വന്തം ഗ്രന്ഥങ്ങളെഴുതിയുണ്ടാക്കി അവ ദൈവികമാണെന്ന് അവകാശപ്പെടുകയും ചെയ്തിട്ടുണ്ട് എന്ന് ഖുര്‍ആന്‍ സൂചിപ്പിക്കുന്നു. മുമ്പ് അവതരിപ്പിച്ച വേദഗ്രൻഥങ്ങൾ മനുഷ്യരുടെ കൈകടത്തലുകൾക്ക് വിധേയമായിട്ടുണ്ടെങ്കിലും അവയിലെ ആശയങ്ങളുടെ സംരക്ഷണം ഖുർആൻ വഴി പടച്ചവൻ നിര്വഹിച്ചിട്ടുണ്ടെന്നും ഖുർആൻ വ്യക്തമാക്കുന്നു. ''(നബിയെ) നിനക്കിതാ സത്യപ്രകാരം വേദഗ്രന്ഥം അവതരിപ്പിച്ചു തന്നിരിക്കുന്നു. അതിന്റെ മുമ്പിലുള്ള വേദഗ്രന്ഥങ്ങളെ ശരിവെക്കുന്നതും, അവയെ കാത്തുരക്ഷിക്കുന്ന തുമത്രെ അത്.''(ഖുര്‍ആന്‍ 5:48)

എന്താണ് ബൈബിൾ എന്ന ചോദ്യത്തിന് ഇസ്‌ലാമിക പരിപ്രേക്ഷ്യത്തിലുള്ള മറുപടിയിതാണ്. , പ്രവാചകന്മാര്‍ക്ക് അവതരിപ്പിക്കപ്പെട്ടിട്ടുള്ള ദൈവവചനങ്ങളിൽ ചിലവ ബൈബിള്‍ പുസ്തകങ്ങളുടെ രചയിതാക്കളില്‍ പലരും ഉദ്ധരിച്ചിട്ടുണ്ട്. അതോടൊപ്പം മനുഷ്യരുടെ വചനങ്ങളും ബൈബിളിൽ ഉണ്ട്. ഏതാ ണ് ദൈവികവചനം, ഏതാണ് മാനുഷികവചനമെന്ന് മനസ്സിലാക്കാന്‍ സാധ്യമല്ലാത്ത അവസ്ഥയി ലാണ് ഇന്നത്തെ ബൈ ബിള്‍ സ്ഥിതി ചെയ്യുന്നത്.

പ്രവാചകന്മാർക്ക് അവതരിക്കപ്പെട്ടതായി ക്വുർആൻ പറയുന്ന വേദഗ്രന്ഥങ്ങളെക്കുറിച്ച് ബൈബിള്‍ സൂചനകള്‍ നല്‍കുകയും ആ വേദഗ്രന്ഥങ്ങള്‍ ഇന്ന ത്തെ ബൈബിളിലില്ലായെന്നുള്ള വസ്തുത വ്യക്തമായി മനസ്സിലാ ക്കിത്തരുകയും ചെയ്യുന്നുണ്ട്. മൂസാ നബി (അ)ക്ക് പടച്ചവന്‍ അവതരിപ്പിച്ച തോറയെക്കു റിച്ച് - തൗറാത്തിനെക്കുറിച്ച് - ആവർത്തിച്ചുള്ള പരാമർശങ്ങൾ ബൈബി ളിലുണ്ട്. ഉദാഹരണത്തിന്, ആവര്‍ത്തനപുസ്തകത്തില്‍ ഒന്നാമത്തെ അധ്യായത്തിലെ അഞ്ചാമത്തെ വചനത്തില്‍ നമുക്ക് കാണാം. 'മോശെ പ്രവാചകന്‍ ഈ നിയമം ജനങ്ങള്‍ക്ക് വിശദീകരി ക്കാന്‍ തുടങ്ങി'. മോശെ പ്രവാചകന്‍ വിശദീകരിച്ച നിയമമാണ് തോറ; ആ തോറയാകുന്നു ഖുര്‍ആന്‍ പറയുന്ന തൗറാത്ത്. ഇന്ന് ബൈബിളില്‍ കാണുന്ന ഉല്‍പത്തി മുതല്‍ ആവര്‍ത്തനം വരെയുമുള്ള പഞ്ച പുസ്തകങ്ങൾ തോറയല്ലെന്ന് വ്യക്തം. മോശെയുടെ ഒരുപാട് വചനങ്ങള്‍ അതിലുണ്ട്. എന്നാല്‍, മോശയുടെ മരണത്തിനുശേഷം പലരും എഴുതിയ പല വചനങ്ങളും അതിലുണ്ട്.

ആവര്‍ത്തന പുസ്തകത്തിന്റെ അവസാന ഭാഗം നോക്കുക: ''അങ്ങനെ യഹോവയുടെ ദാസനായ മോശെ യഹോവയുടെ വചനപ്രകാരം അവിടെ മോവാബ് ദേശത്തു വെച്ചു മരിച്ചു. അവന്‍ അവനെ മോവാബ് ദേശത്ത് ബേത്ത് -പെയോരിന്നെതിരെയുള്ള താഴ്‌വരില്‍ അടക്കി. എങ്കിലും ഇന്നുവരെയും അവന്റെ ശവക്കുഴിയു ടെ സ്ഥലം ആരും അറിയുന്നില്ല. മോശെ മരിക്കുമ്പോര്‍ അവന്നു നൂറ്റിഇരുപത് വയസ്സായിരുന്നു. അവന്റെ കണ്ണ് മങ്ങാതെയും അവന്റെ ദേഹബലം ക്ഷയിക്കാതെയും ഇരുന്നു.യിസ്രായേല്‍മക്കള്‍ മേശെയെക്കുറിച്ച് മോവാബ് സമഭൂമിയില്‍ മുപ്പത് ദിവസം കരഞ്ഞു കൊണ്ടിരിന്നു. അങ്ങനെ മേശെയെക്കുറിച്ച് കരഞ്ഞു വിലപിക്കുന്ന കാലം തികഞ്ഞു.ന്റെ മകനായ യോശുവായെ മോശെ കൈവച്ച് അനുഗ്രഹിച്ചിരിക്കുന്നതുകൊണ്ട്          അവന്‍ ജ്ഞാനാത്മപൂര്‍ണ്ണനായിത്തീര്‍ന്നു. യഹോവ മോശെയോടു കല്‍പ്പിച്ചതുപോലെ               യിസ്രായോല്‍ മക്കള്‍ അവനെ അനുസരിച്ചു. എന്നാല്‍ മിസ്രയീം ദേശത്തു ഫറനോനോടും അവന്റെ സകല ഭൃത്യന്മാരോടും അവന്റെ സര്‍വ്വ ദേശത്തോടും ചെയ്യുവാന്‍ യഹോവ മോശെയെ നിയോ ഗിച്ചയച്ച സകല അല്‍ഭുതങ്ങളും ഭുജവീയ്യവും എല്ലാ യിസ്രായേലും കാണ്‍കെ മോശെ പ്രവര്‍ത്തിച്ച ഭയങ്കര കാര്യമെക്കെയും വിചാരിച്ചാല്‍ യഹോവ അഭി മുഖമായി അറിഞ്ഞു മോശെയെപ്പോലെ ഒരു പ്രവാചകന്‍ യിസ്രായേലില്‍ പിന്നെ ഉണ്ടായിട്ടില്ല' (ആവര്‍ത്തന പുസ്തകം 34: 5-12)

മോശയുടെ മരണത്തെയും മരണത്തിനുശേഷമുള്ള കാര്യങ്ങളെയും കുറിച്ച് പറഞ്ഞുകൊണ്ട് . ആവര്‍ത്തനപുസ്തകം അവസാനിപ്പിക്കുന്നത് ''പിന്നീട് ഇസ്‌റായീലില്‍ ഇതു വരെ മോശെയെപ്പോലൊരു പ്രവാചകന്‍ ഉണ്ടായിട്ടില്ല'' എന്ന പ്രസ്താവനയോടെയാണ്. . ഇതിൽ നിന്ന് മോശെ പ്രവാചകന് അവതരിപ്പിക്കപ്പെടുകയും അദ്ദേഹം ജന ങ്ങള്‍ക്കു മുന്നില്‍ വിശദീകരിക്കുകയും ചെയ്ത നിയമപുസ്തകമല്ല ഇന്നത്തെ പഞ്ചപുസ്തകം എന്നും മോശെയ്ക്ക് നൂറ്റാണ്ടുകൾ കഴിഞ്ഞ് അതിൽ പലതും കടന്നുകൂടിയിട്ടുണ്ട് എന്നും നമുക്ക് മനസ്സിലാക്കാനാവും.

ദാവീദിന് ദൈവം  അവതരിപ്പിച്ചതായി ഖുര്‍ആന്‍ സൂചിപ്പിക്കുന്ന ഗ്രന്ഥമാണ് സബൂർ. സങ്കീര്‍ത്തന പുസ്തകങ്ങള്‍ ബൈബിളിലെ അതിസുന്ദരമായ പുസ്തക സഞ്ചയമാണ്. ഈ സങ്കീര്‍ത്തന പുസ്തകങ്ങളില്‍ ദാവീദിന്‍േറതെന്ന് തലവാചകമുള്ള ഏതാനും ചില സങ്കീര്‍ത്തന ങ്ങളാണുള്ളത്. അതല്ലാത്ത ചിലത് ആസാഫിന്റെയും ആസാഫിന്റെ പുത്ര ന്മാരുടെയും കോരഹി ന്റെയും കോരഹിന്റെ പുത്രന്മാരുടെയും അങ്ങനെ പലരുടേതുമായിക്കൊണ്ടാണ് ബൈബിളിലെ സങ്കീര്‍ത്തന സഞ്ചയങ്ങളുടെ തലവാചകങ്ങള്‍ തന്നെ സ്ഥിതി ചെയ്യുന്നത്. ദാവീദിന്‍േറതെന്ന് പറയ പ്പെടുന്ന 35-ഓളം സങ്കീര്‍ത്തനങ്ങളില്‍ തന്നെ ഏതെല്ലാം ദാവീദിന്‍േറതാണെന്ന് കൃത്യമായി പറയാന്‍ കഴിയില്ല എന്ന് റവ. എ.സി. ക്ലേയിറ്റനെപ്പോലെയുള്ള  ബൈബിള്‍ പണ്ഡിത ന്മാര്‍ സൂചിപ്പിക്കുകയും ചെയ്യുന്നു.

ഇനി യേശുക്രിസ്തുവിന്‍േറതായി അവതരിപ്പിക്കപ്പെട്ടു എന്ന് ഖു ര്‍ആന്‍ സൂചിപ്പിക്കുന്ന ഇന്‍ജീലിന്റെ -സുവിശേഷം- സ്ഥിതി എന്താണ്? . ഖുര്‍ആന്‍ പറയുന്നത് ഈസാ നബി (അ)ക്ക് പടച്ചതമ്പുരാന്‍ അവതരിപ്പിച്ച  വേദഗ്രന്ഥമാണ് ദൈവത്തിന്റെ സുവിശേഷം എന്നാണ്. ബൈ ബിളില്‍ നമുക്ക് കാണാന്‍ കഴിയും ''യോഹന്നാന്‍ ബന്ധനസ്ഥനായതിനുശേഷം യേശു ദൈവത്തിന്റെ സുവിശേഷം പ്രസംഗിച്ചുകൊണ്ട് ഗലീലിയിലേക്ക് വന്നു. അദ്ദേഹം പറഞ്ഞു: ''നിങ്ങള്‍ അനുതപിക്കുക. കാലം തികഞ്ഞിരിക്കുന്നു. ദൈവരാജ്യം സമീപസ്ഥമായിരിക്കുന്നു. നിങ്ങള്‍ സുവിശേഷത്തില്‍ വിശ്വസിക്കൂ. എന്നിലും വിശ്വസിക്കൂ' (മാര്‍ക്കോസ്1:14-15).

ഇവിടെ യേശുക്രിസ്തു വിശ്വസിക്കാന്‍ പറഞ്ഞ, യേശുക്രിസ്തു പ്രസംഗിച്ച ദൈവത്തിന്റെ സുവിശേഷം, അത് ഏതാണ്? അതാണ് ഖുർആൻ പറയുന്ന ഇന്‍ജീന്‍. അത് മത്തായിയുടേയോ  മാര്‍ക്കോസിന്‍േറയോ ലൂക്കോസിന്‍േറയോ യോഹന്നാന്റേയോ സുവിശേഷമല്ല. ദൈവം അവതരിപ്പിച്ച സുവിശേഷമാണത്. ആ സുവിശേഷത്തെക്കുറിച്ചാണ് യേശുക്രിസ്തു മര്‍ക്കോസിന്റെ സുവിശേഷത്തിന്റെ എട്ടാം അധ്യായത്തില്‍ 38-ാം വചനത്തില്‍ പറയുന്നത് ''ആരെങ്കിലും ഈ സുവിശേഷത്തിനു വേണ്ടി മരണപ്പെ ടുകയാണെങ്കില്‍ അവര്‍ക്ക് അവരുടെ ജീവന്‍ ലഭിക്കുന്നുവെന്നും  സകലവും നഷ്ടപ്പെടുത്തുവെങ്കില്‍ മുഴുവന്‍ അവര്‍ക്ക് ലഭിക്കുകയു മാണ് ചെയ്യുന്നത്'' എന്നും. 'എന്നാല്‍ സുവിശേഷം മുമ്പെ സകല ജാതികളോടും പ്രസംഗിക്കേണ്ടതാകുന്നു.''  (മര്‍ക്കോസ്13:1)

യേശുക്രിസ്തുവിന് പടച്ചവന്‍ അവതരിപ്പിച്ച സുവിശേഷമായ ഇന്ജീലിനെക്കുറിച്ച് ബൈബിള്‍ സൂചനകൾ നൽകുന്നുണ്ടെങ്കിലും ഇന്നത്തെ പുതിയ നിയമത്തിലെവിടെയും പ്രസ്തുത സുവിശേഷം ഉൾക്കൊള്ളുന്നില്ല. ഇന്ന് നില നിൽക്കുന്ന ബൈബിളിലെ പ്രവാചകന്മാരിലൂടെ അവതരിപ്പിക്കപ്പെട്ട മുഴുവൻ ദൈവ വചനങ്ങളെയും ഇസ്‌ലാം അംഗീകരിക്കുന്നു. അതോടൊപ്പം തന്നെ പിൽക്കാലത്ത് പലരും എഴുതിച്ചെർത്ത വചങ്ങൾ ഇസ്‌ലാം നിരാകരിക്കുകയും ചെയ്യുന്നു.

 

കനായ സ്രഷ്ടാവ് നിയോഗിച്ചയച്ച പ്രവാചകന്മാരെക്കുറിച്ച് ബൈബിളിലും ഖുര്‍ആനിലും വന്ന സമാനമായ ചരിത്രപരാമര്‍ശങ്ങളുടെ വെളിച്ചത്തില്‍ ബൈബിളില്‍നിന്ന് പകര്‍ത്തിയെഴുതിയതാണ് ഖുര്‍ആന്‍ എന്ന വാദം മിഷനറിമാരും ഓറിയന്റലിസ്റ്റുകളും ഭൗതികവാദികളുമെല്ലാമായ വിമര്‍ശകര്‍ ഒരേ സ്വരത്തില്‍ ഉന്നയിക്കാറുണ്ട്. ഈ വാദത്തില്‍ എത്രത്തോളം കഴമ്പുണ്ട്? താഴെ പറയുന്ന വസ്തുതകളുടെ വെളിച്ചത്തില്‍ ചിന്തിക്കുമ്പോള്‍ ഈ വാദം ശുദ്ധ അസംബന്ധമാണെന്ന് ബോധ്യമാകും.

ഒന്ന്) മുഹമ്മദ് നബി (സ) നിരക്ഷരനായിരുന്നു. ബൈബിള്‍ പഴയനിയമവും പുതിയനിയമവും വായിച്ചു മനസ്സിലാക്കി അതില്‍നിന്ന് പകര്‍ത്തിയെഴുതുക അദ്ദേഹത്തിന് സ്വന്തമായി അസാധ്യ മായിരുന്നു. ശിഷ്യന്മാരില്‍ ആരുടെയെങ്കിലും സഹായത്തോടെ അദ്ദേഹം അത് നിര്‍വഹിച്ചുവെന്ന് കരുതാനും വയ്യ. അങ്ങനെ ചെയ്തിരുന്നുവെങ്കില്‍ ശിഷ്യന്മാരില്‍ ചിലര്‍ക്കെങ്കിലും അത് അറിയാന്‍ കഴിയേണ്ടതായിരുന്നു. അത് മുഖേന മുഹമ്മദ് നബി (സ)യുടെ വിശ്വാസ്യതയില്‍ അവര്‍ സംശയിക്കു കയും അവര്‍ തമ്മിലുള്ള ബന്ധത്തിന് ഉലച്ചില്‍തട്ടുകയും ചെയ്യുമായിരുന്നു. മുഹമ്മദ് നബി (സ) യുടെ  ശരീരത്തില്‍ ഒരു പോറലെങ്കിലുമേല്‍ക്കുന്നതിന് പകരം സ്വന്തം ജീവന്‍ ബലിയര്‍പ്പിക്കുവാന്‍ സന്നദ്ധരായവരായിരുന്നു പ്രവാചക ശിഷ്യന്മാര്‍ എന്നോ ര്‍ക്കുക. പ്രവാചകനില്‍ (സ)ഏതെങ്കിലും തരത്തിലുള്ള അവിശ്വാസ്യ തയുണ്ടായിരുന്നുവെങ്കില്‍ ഇങ്ങനെ ത്യാഗം ചെയ്യാന്‍ സന്നദ്ധരായ ഒരു അനുയായിവൃന്ദത്തെ വളര്‍ത്തിയെടുക്കുവാന്‍ അദ്ദേഹത്തിന് കഴിയുമായിരുന്നില്ലെന്ന് തീര്‍ച്ച യാണ്.

''ഇതിന് മുമ്പ് നീ വല്ല ഗ്രന്ഥവും പാരായണം ചെയ്യുകയോ, നിന്റെ വലതുകൈകൊണ്ട് അത് എഴുതു കയോ ചെയ്തിരുന്നില്ല. അങ്ങനെയാണെങ്കില്‍ ഈ സത്യനിഷേധികള്‍ക്ക് സംശയിക്കാമായിരുന്നു'' (വി.ഖു. 29:48).

രണ്ട്) മുഹമ്മദ് നബി (സ)യുടെ ജീവിതകാലത്ത് ബൈബിള്‍ പഴയനിയമമോ പുതിയനിയമമോ അറബിയിലേക്ക് പരിഭാഷപ്പെടുത്തപ്പെട്ടിട്ടുണ്ടായിരുന്നില്ല. അറബിയിലുള്ള പഴയനിയമവും പുതിയനിയമവുമെല്ലാം ഉണ്ടായതുതന്നെ ഇസ്‌ലാമിന്റെ ദിഗ്‌വിജയങ്ങള്‍ക്ക് ശേഷമാണ്. പഴയ നിയമ രേഖകളെക്കുറിച്ച് സൂക്ഷ്മ പഠനം നടത്തിയ ഏണസ്റ്റ് വൂര്‍ഥ്‌വിന്‍ എഴുതുന്നത് കാണുക: ''ഇസ്‌ലാമിന്റെ വ്യാപനത്തോടുകൂടി അറബിയുടെ ഉപയോഗം വ്യാപകമാവുകയും ഇസ്‌ലാമിക രാജ്യങ്ങളിലെ ജൂതന്മാരുടെയും ക്രിസ്ത്യാനികളുടെയും ദൈനംദിനജീവിതത്തിലെ ഭാഷയായി അറബി മാറുകയും ചെയ്തു. ബൈബിളിന്റെ അറബി പതിപ്പുകള്‍ അനിവാര്യമാക്കി ത്തീര്‍ത്ത ഈ സാഹചര്യത്തില്‍ സ്വതന്ത്രവും പ്രാഥമികമായ വ്യാഖ്യാന സംബന്ധിയുമായ നിരവധി പതിപ്പുകള്‍ പുറത്തുവന്നു''.(Ernst Wurthewein: The Text of The Old Testament Page 104).

ഒമ്പതാം നൂറ്റാണ്ടിന്റെ ആദ്യപകുതിയിലാണ് പഴയ നിയമബൈബിള്‍ അറബിയിലേക്ക് പരിഭാഷപ്പെടുത്തപ്പെട്ടതെന്നാണ് ലഭ്യമായ കയ്യെഴുത്ത് രേഖകള്‍ വ്യക്തമാക്കുന്നത് (Ibid Page 224 -225). ഏകദേശം ഇക്കാലത്തുതന്നെയാവണം പുതിയ നിയമവും അറബിയിലേക്ക് ഭാഷാന്തരം ചെയ്യപ്പെട്ടത്. പ്രഗത്ഭനായ സിഡ്‌നി എച്ച്. ഗ്രിഫിത്തിന്റെ വരികള്‍ കാണുക:''അറബിയിലുള്ള സുവിശേഷങ്ങളടങ്ങിയ ഏറ്റവും പുരാതനമായ കയ്യെഴുത്ത് രേഖ 'സിനായ് അറബി കയ്യെഴുത്ത് പ്രതി 72' (Sinai Arabic MS72)ആണ്. ജറുസലേം സഭയുടെ ഗ്രീക്ക് പ്രാര്‍ത്ഥനാ കലണ്ടറിന്റെ കാലക്രമാടിസ്ഥാനത്തില്‍ അധ്യായങ്ങള്‍ രേഖപ്പെടുത്തിയ നാല് കാനോനിക സുവി ശേഷങ്ങളും ഇതിലുണ്ട്. രേഖയുടെ അന്ത്യത്തിലെ കുറിപ്പ് വ്യക്തമാക്കുന്നത് ഈ കയ്യെഴുത്ത് രേഖ അറബി കലണ്ടര്‍ 284ല്‍ അഥവാ ക്രിസ്താബ്ദം 897ല്‍ റംലയിലെ സ്റ്റീഫന്‍ (Stephen of Ramlah) എഴുതി യതാണെന്നാണ് (Sidney H Griffith: The Gospel in Arabic: An Enquiry Into its Appearance In the First Abbasi Century Page 132)

എന്നാല്‍ അപ്പോസ്തല പ്രവൃത്തികളും പൗലോസിന്റെ ലേഖനങ്ങളും കാതോലിക ലേഖനങ്ങളുമുള്‍ക്കൊള്ളുന്ന Sinai Arabic MS151 എന്ന കയ്യെഴുത്ത് രേഖ ഹിജ്‌റ253 ല്‍ അഥവാ ക്രിസ്താബ്ദം 867ല്‍ സുറിയാനിയില്‍ നിന്ന് അറബിയിലേക്ക് ബിസ്ര്‍ബ്‌നുസിര്‍റി എന്നയാള്‍ വിവര്‍ത്തനം ചെയ്തതായി കാണുന്നുണ്ട്. ഇതില്‍ സുവിശേഷങ്ങളില്ലെന്ന കാര്യം പ്രത്യേകം ശ്രദ്ധേയമാണ്. (Ibid Page 131).

മുഹമ്മദ് നബി (സ) ക്ക് ശേഷം രണ്ട് നൂറ്റാണ്ടുകളെങ്കിലും കഴി ഞ്ഞാണ് പുതിയനിയമവും പഴയ നിയമവുമെല്ലാം അറബിയിലേക്ക് വിവര്‍ത്തനം ചെയ്യപ്പെട്ടത്. നിരക്ഷരനായിരുന്ന മുഹമ്മദ് നബി (സ) മറ്റാരില്‍നിന്നെങ്കിലും അറബിയിലുള്ള ബൈബിള്‍ വായിച്ചുകേട്ടശേഷം അതിലെ കഥകള്‍ ഉള്‍ ക്കൊള്ളിച്ചുകൊണ്ട് എഴുതിയതാണ് ഖുര്‍ആന്‍ എന്ന വാദവും ഇവിടെ അപ്രസക്തമാവുകയാണ്. അറബിയില്‍ നിലവിലില്ലാത്ത ഒരു ഗ്രന്ഥം വായിച്ചുകേട്ടുവെന്ന് കരുതുന്നത് നിരര്‍ത്ഥകമാണെന്ന് പറയേണ്ടതില്ലല്ലോ.

മൂന്ന്) പ്രവാചകന്മാരുടെ ചരിത്രം വിവരിക്കുന്നിടത്ത് അധാര്‍മ്മികരും അസാന്മാര്‍ഗികരുമായി രുന്നു അവരെന്ന് വരുത്തിത്തീര്‍ക്കുന്ന തരത്തിലാണ് ബൈബിള്‍ അത് നിര്‍വ്വഹിച്ചിരിക്കുന്നത്. മദ്യപിച്ച് നഗ്‌നനായ നോഹും ലഹരി മൂത്ത് സ്വപുത്രിമാരുമായി ശയിച്ച ലോത്തും ചതിയനായ യാക്കോബും വിഷയലമ്പടനായ ദാവീദും മദ്യം വിളമ്പിയ യേശുവുമെല്ലാം, ധര്‍മ്മത്തിലേക്ക് ജനങ്ങളെ നയിക്കാനായി നിയോഗിക്കപ്പെട്ടവരായിരുന്നു പ്രവാചകന്മാര്‍ എന്ന സങ്കല്‍പത്തിന് കടകവിരുദ്ധമായ കഥകളാണെന്ന് പറയേണ്ടതില്ലല്ലോ. ഖുര്‍ആനിലെ ചരിത്രവിവരണത്തില്‍ ഇത്തരം യാതൊരു കഥകളും കാണുന്നില്ല. ബൈബിളില്‍നിന്ന് മുഹമ്മദ് നബി (സ) പകര്‍ത്തിയെഴു തിയതായിരുന്നു ഈ കഥകളെങ്കില്‍  പ്രവാചകന്മാരില്‍ ബൈബിള്‍ ആരോപിച്ച അധാര്‍മ്മികത കളിലേതെങ്കിലും ഖുര്‍ആനിലും സ്ഥാനം പിടിക്കേണ്ടതായിരുന്നു. അങ്ങനെയില്ലെന്ന് മാത്രമല്ല, പ്രവാചകന്മാരെല്ലാം ഉന്നതരും വിശുദ്ധരുമായിരുന്നുവെന്ന വസ്തുത വ്യക്തമാക്കുന്നതാണ് ഖുര്‍ ആനിലെ പ്രവാചക കഥനങ്ങളെല്ലാമെന്ന കാര്യം അത് ബൈബിളില്‍നിന്ന് പകര്‍ത്തിയെഴുതിയതാ ണെന്ന വാദത്തിന്റെ നട്ടെല്ലൊടിക്കുന്നുണ്ട്.

നാല്) ചരിത്രത്തിന്റെ അളവുകോലുകള്‍ വെച്ചുനോക്കുമ്പോള്‍ വസ്തുനിഷ്ഠചരിത്രത്തിന് നിര ക്കാത്ത നിരവധി പ്രസ്താവനകള്‍ ബൈബിള്‍ നടത്തുന്നുണ്ട്. ഇത് ബൈബിള്‍ പണ്ഡിതന്മാര്‍ തന്നെ അംഗീകരിക്കുന്നതാണ്. ''ചരിത്രപരമായി കൃത്യമല്ലാത്ത ചില പ്രസ്താവനകളും ബൈബിളില്‍ കണ്ടെന്നുവരാം'' (ബൈബിള്‍ വിജ്ഞാനകോശം പുറം 12). ബൈബിളില്‍നിന്ന് പകര്‍ത്തി യെഴുതി ക്കൊണ്ട് മുഹമ്മദ് നബി (സ)രചിച്ചതായിരുന്നു ഖുര്‍ആനെങ്കില്‍ അതില്‍ ബൈബിളിലേതു പോലെ ചരിത്രപരമായി കൃത്യമല്ലാത്ത പ്രസ്താവനകള്‍ കാണപ്പെടേണ്ടതായിരുന്നു. എന്നാല്‍, അത്തരം യാതൊരു പ്രസ്താവനയും ഖുര്‍ആനിലില്ല.

അഞ്ച്) ആധുനിക ശാസ്ത്രത്തിന്റെ കാഴ്ചപ്പാടിലൂടെ നോക്കുമ്പോള്‍ ബൈബിളില്‍ നിരവധി അശാസ്ത്രീയമായ പരാമര്‍ശങ്ങള്‍ കാണാനാവും. സൂര്യന്റെ സൃഷ്ടിക്ക് മുമ്പു തന്നെ രാപ്പകലുകളു ണ്ടായതായി വിവരിക്കുന്ന ഉല്‍പത്തി പുസ്തകം മുതലാരംഭിക്കുന്നു ബൈബിളിലെ ശാസ്ത്ര വിരു ദ്ധമായ പരാമര്‍ശങ്ങള്‍. രാപ്പകലുകളുണ്ടാവുന്നത് സൂര്യചന്ദ്രന്മാരുടെ ചലനം മൂലമാണെന്നും (യേശു 10:12,13), ഭൂമി ഇളകാതെ നിശ്ചലമായി നില്‍ക്കുകയാണെന്നും (സങ്കീ 104:5) മുയല്‍ അയവിറ ക്കുന്ന ജീവിയാണെന്നു (ആവ 14:7) മെല്ലാമുള്ള ബൈബിള്‍ പരാമര്‍ശങ്ങള്‍ അതിന്റെ അശാസ്ത്രീയ തക്ക് ഉദാഹരണങ്ങളാണ്. ഈ പരാമര്‍ശങ്ങളെല്ലാം വരുന്നത് പ്രവാചക കഥനങ്ങള്‍ക്കിടയിലാ ണെന്ന കാര്യം പ്രത്യേകം ശ്രദ്ധേയമാണ്. ബൈബിളായിരുന്നു ഖുര്‍ആനിന്റെ രചനയ്ക്കുപയോ ഗിച്ചിരുന്ന സ്രോതസ്സെങ്കില്‍ ഈ അശാസ്ത്രീയമായ പരാമര്‍ശങ്ങളെല്ലാം ഖുര്‍ആനിലും സ്ഥാനം പിടിക്കുമായിരുന്നു. ഈ പരാമര്‍ശങ്ങള്‍ വസ്തുതകള്‍ക്ക് നിരക്കാത്തതാണെന്ന അറിവ് മുഹമ്മദ് നബി (സ)യുടെ കാലത്തുണ്ടായിരുന്നില്ലെന്നോര്‍ക്കുക. എന്നാല്‍ ഖുര്‍ആനില്‍ ഇത്തരം യാതൊരുവിധ പരാമര്‍ശങ്ങളുമില്ല. ഖുര്‍ആനിലെ ഒരൊറ്റ വചനമെങ്കിലും ഏതെങ്കിലും ശാസ്ത്രവസ്തുത കളുമായി വൈരുദ്ധ്യം പുലര്‍ത്തുന്നതായി തെളിയിക്കപ്പെട്ടിട്ടില്ല. ബൈബിളില്‍നിന്ന് പകര്‍ത്തി ക്കൊണ്ട് മുഹമ്മദ് നബി (സ) രചിച്ച ഗ്രന്ഥമാണ് ഖുര്‍ആനെന്ന് വാദിക്കുകയാണെങ്കില്‍ തനിക്ക് ശേഷം നൂറ്റാണ്ടുകള്‍ കഴിഞ്ഞ് വരാനിരിക്കുന്ന ശാസ്ത്രമുന്നേറ്റങ്ങള്‍ കൂടി മുന്‍കൂട്ടി കാണാന്‍ കഴിയുകയും അതിന്റെ അടിസ്ഥാനത്തില്‍ ബൈബിളിലുള്ള അശാസ്ത്രീയതകള്‍ അറിഞ്ഞ് അവ യെല്ലാം അരിച്ചൊഴിവാക്കി സംശുദ്ധമായ ചരിത്രം മാത്രം എടുത്തുദ്ധരിക്കുകയും ചെയ്ത അതിമാനുഷനാണ് അദ്ദേഹമെന്ന് പറയേണ്ടിവരും. സര്‍വ്വശക്തനായ സ്രഷ്ടാവിന്റെ വചനങ്ങളാണ് ഖുര്‍ആനിലുള്ളതെന്ന വസ്തുത നിഷേധിക്കുവാന്‍ തെളിവ് പരതുന്നവര്‍ മുഹമ്മദ് നബി (സ)യെ ദൈവമാക്കുന്ന പരിണാമഗുപ്തിയിലാണ് എത്തിച്ചേരുകയെന്നര്‍ത്ഥം.

ആറ്) ബൈബിളില്‍ പറയാത്ത ചില പ്രവാചകന്മാരുടെയും സമുദായങ്ങളുടെയും ചരിത്രം ഖുര്‍ ആന്‍  വിവരിക്കുന്നുണ്ട്. ആദ്, സമൂ ദ് ഗോത്രങ്ങളിലേക്ക് നിയോഗിക്കപ്പെട്ട ഹൂദ് നബിയുടെയും സാലിഹ് നബിയുടെയും ചരിത്രം ഉദാഹരണം. ബൈബിളിലെവിടെയും കാണാനാവാത്ത പ്രവാ ചകന്മാരാണിവര്‍. ബൈബിളില്‍ നിന്ന് കോപ്പിയടിക്കുകയാണ് മുഹമ്മദ് നബി (സ)ചെയ്തതെങ്കില്‍ ഈ ചരിത്രങ്ങള്‍ അദ്ദേഹത്തിന് എവിടെനിന്നാണ് കിട്ടിയത്?

ഏഴ്) ബൈബിളില്‍ പരാമര്‍ശിക്കപ്പെട്ട പ്രവാചകന്മാരുടെ ചരിത്രം പറയുമ്പോള്‍തന്നെ ബൈബിളി ലൊരിടത്തും പരാമര്‍ശിക്കാത്ത നിരവധി സംഭവങ്ങള്‍ ഖുര്‍ആനില്‍ വിശദീകരിക്കുന്നുണ്ട്. നൂഹ് നബി(അ)യും അവിശ്വാസിയായ മകനും തമ്മില്‍ നടന്ന സംഭാഷണവും മകന്‍ പ്രളയത്തില്‍ പെട്ട സംഭവവിവരണവും സൂറത്തു ഹൂദില്‍(11: 42-46) കാണാം. ഇങ്ങനെ യാതൊന്നും ബൈബിളിലെവി ടെയുമില്ല. ഇബ്രാഹീം നബിയും നംറൂദും തമ്മില്‍ നടന്ന സംവാദവും (ഖുര്‍ആന്‍ 2:258) പിതാവു മായി നടന്ന സംഭാഷണവും (ഖുര്‍ആന്‍ 6:74, 19:41-49, 43:26,27) മരണാനന്തര ജീവിതത്തിന്റെ സത്യത ബോധ്യപ്പെടുന്നതിനായി, പക്ഷികളെ കഷ്ണിച്ച് നാല് മലകളില്‍വെച്ചശേഷം അവയെ വിളിച്ചാല്‍ അവ ഓടിവരുന്നതാണെന്ന് അല്ലാഹു അദ്ദേഹത്തോട് പറഞ്ഞ സംഭവവും (2:260) തീയിലേക്ക് വലിച്ചെറിയപ്പെടുകയും അതില്‍ നിന്ന് അദ്ദേഹം അത്ഭുതകരമായി രക്ഷപ്പെടുകയും ചെയ്ത ചരിത്രവു(21:56-70)മൊന്നും ബൈബിളിലൊരിടത്തും കാണാന്‍ കഴിയില്ല. ദൈവിക കല്‍പന പ്രകാരം ഒരു പശുവിനെ അറുക്കാന്‍ മൂസാ (അ) ഇസ്രായീല്യരോട് നിര്‍ദേശിക്കുകയും, പശുവിന്റെ പ്രത്യേകതകള്‍ ചോദിച്ച് അതിന്റെ നിര്‍വ്വഹണം അവര്‍ പ്രയാസകരമാക്കുകയും ചെയ്ത സംഭവവും (ഖുര്‍ആന്‍ 2:67-71) കൊലപാതകക്കുറ്റം തെളിയിക്കാനായി പശുവിനെ അറുത്ത് അതിന്റെ ഒരു ഭാഗംകൊണ്ട് അടിക്കുവാന്‍ കല്‍പിച്ച കഥനങ്ങളും (2:72, 73) മൂസാ നബി (അ)യുടെ ജീവിതവുമായി ബന്ധപ്പെടുത്തി ബൈബിളിലൊരിടത്തും പ്രസ്താവിക്കുന്നില്ല. ഈസാ നബി (അ)യുടെ ജനനം മുതല്‍ തന്നെയുള്ള ബൈബിളില്‍ പറയാത്ത പല സംഭവങ്ങളും ഖുര്‍ആനില്‍ പരാമര്‍ശിക്കുന്നുണ്ട്. സകരിയ്യായുടെ സംരക്ഷണത്തി ല്‍ പ്രാര്‍ത്ഥനാസ്ഥലത്ത് താമസിച്ചുകൊണ്ടി രുന്ന മര്‍യമിന്റെ കുട്ടിക്കാലത്ത് അവര്‍ക്ക് അത്ഭുതകരമായി ഭക്ഷണസാധനങ്ങള്‍ ലഭിച്ച സംഭവം (ഖുര്‍ആന്‍ 3:37), മര്‍യമിന്റെ പ്രസവസമയത്ത് അവര്‍ക്ക് നല്‍കപ്പെട്ട പ്രത്യേക അനുഗ്രഹങ്ങളെ ക്കുറിച്ച വിവരണം (19:23-26), ഈസാ (അ) തൊട്ടിലില്‍വെച്ച് സംസാരിച്ച് തന്റെ നിയോഗം പ്ര ഖ്യാപിച്ചു കൊണ്ട് തന്റെ ആദ്യത്തെ അത്ഭുതം പ്രവര്‍ത്തിച്ച ചരിത്രം (19:29,30), കളിമണ്ണുകൊണ്ട് പക്ഷിയുടെ രൂപമുണ്ടാക്കി ഈസാ (അ) അതില്‍ ഊതിയപ്പോള്‍ അല്ലാഹുവിന്റെ അനുമതിപ്രകാരം അതൊരു പക്ഷിയായി രൂപാന്തരപ്പെട്ട സംഭവം (3:49) ഇതൊന്നുംതന്നെ ബൈബിളില്‍ ഒരിടത്തും പരാമര്‍ശിക്കുന്നുപോലുമില്ല. ബൈബിളില്‍നിന്ന് മുഹമ്മദ് നബി (സ) പകര്‍ത്തിയെഴുതി ക്കൊണ്ടാണ് ഖുര്‍ആന്‍ രചിച്ചതെങ്കില്‍ ബൈബിളിലൊരിടത്തും പരാമര്‍ശിക്കാത്ത പ്രവാച കന്മാ രുടെ ജീവിതവുമായി ബന്ധപ്പെട്ട കഥകള്‍ അദ്ദേഹത്തിന് എവിടെനിന്നു കിട്ടി? മുഹമ്മദ് നബിയുടെ കാലത്ത് യഹൂദ ക്രൈസ്തവര്‍ക്കിടയില്‍ നിലനിന്നിരുന്ന കഥകളും ഐതിഹ്യങ്ങളുമാണ് ഈ പരാമര്‍ശങ്ങളുടെ സ്രോതസ്സ് എന്നു വാദിക്കപ്പെടാറുണ്ട്. ഈ വാദവും അടിസ്ഥാന രഹിതമാണ്. പ്രസ്തുത കഥകളു ടെയും പുരാണങ്ങളുടെയുമെല്ലാം ചരിത്രപരതയെ ചോദ്യം ചെയ്യുന്ന പഠനങ്ങളും ഗവേഷണങ്ങളും പില്‍ക്കാലത്ത് നടന്നിട്ടുണ്ട്. ഈ ഗവേഷണങ്ങളിലൊന്ന് പോലും ഒരു ഖുര്‍ആന്‍ കഥനത്തെയും വസ്തുനിഷ്ഠമായി വിമര്‍ശിക്കുന്നില്ലെന്ന കാര്യം പ്രത്യേകം ശ്രദ്ധേയമാണ്. സത്യത്തില്‍ ഖുര്‍ആന്‍ ദൈവവചനമായതുകൊണ്ടാണ് ബൈബിളിലെവിടെയും സൂചിപ്പിക്കാത്ത സംഭവങ്ങള്‍പോലും കൃത്യവും പ്രമാദരഹിതവുമായി അതില്‍ നമുക്ക് കാണാന്‍ കഴിയുന്നത്. മര്‍യത്തിന്റെ ബാല്യകാല സംഭവങ്ങള്‍ വിവരിക്കവെ ഖുര്‍ആന്‍ പറ ഞ്ഞത് എത്ര ശരി! ''(നബിയേ) നാം നിനക്ക് ബോധനം നല്‍കുന്ന അദൃശ്യവാര്‍ത്തകളില്‍ പെട്ടതാകുന്നു അവയൊക്കെ. അവരില്‍ ആരാണ് മര്‍യത്തിന്റെ സംരക്ഷണം ഏറ്റെടുക്കേണ്ടതെന്ന് തീരുമാനിക്കുവാനായി അവര്‍ തങ്ങളുടെ അമ്പുകള്‍ ഇട്ടുകൊണ്ട് നറുക്കെടുപ്പ് നടത്തിയിരുന്ന സമയത്ത് നീ അവരുടെ അടുത്തുണ്ടായിരുന്നില്ല ല്ലോ. അവര്‍ തര്‍ക്കത്തിലേര്‍പ്പെട്ടുകൊണ്ടിരുന്നപ്പോഴും നീ അവരുടെ അടുത്തുണ്ടായിരുന്നില്ല'' (വി.ഖു. 3:44).

എട്ട്) ബൈബിളില്‍ പരാമര്‍ശിക്കപ്പെട്ട കഥകള്‍ പറയുമ്പോഴും ബൈബിളില്‍നിന്ന് വ്യത്യസ്തമായി കൃത്യതയും സൂക്ഷ്മതയും ഖുര്‍ആന്‍ കാത്തുസൂക്ഷിക്കുന്നത് കാണാം. ഉദാഹരണത്തിന് മോശ- സീനായ് പര്‍വതത്തിലേക്ക് പോയ അവസരത്തില്‍ ഇസ്രായീല്യര്‍ക്ക് അവരുടെ ആവശ്യപ്രകാരം സ്വര്‍ണംകൊണ്ട് കാളക്കുട്ടിയെ നിര്‍മിച്ച് ആരാധനക്കായി നല്‍കിയത് മോശയുടെ കൂട്ടാളിയും പ്രവാ ചകനുമായ അഹരോണായിരുന്നുവെന്നാണ് പുറപ്പാട് പുസ്തകം (32:1-6) പറയുന്നത്. ഖുര്‍ആനും ബൈബിളുമെല്ലാം പരിശുദ്ധ പ്രവാചകനായി പരിചയപ്പെടുത്തുന്ന ഹാറൂനി(അ)ല്‍ നിന്ന് വിഗ്രഹാ രാധനക്ക് കൂട്ടുനില്‍ക്കുകയെന്ന മഹാപാപം സംഭവിക്കാനിടയില്ലെന്ന് ഏത് സാമാന്യ ബുദ്ധിക്കും മനസ്സിലാവും. ഖുര്‍ആനും പ്രസ്തുത സംഭവം വിവരിക്കുന്നുണ്ട്. പക്ഷെ, സ്വര്‍ണപശുവിനെയുണ്ടാ ക്കുകയും അതിനെ ആരാധിക്കുവാന്‍ ഇസ്രായീല്യരെ പ്രേരിപ്പിക്കുകയും ചെയ്തത് ഹാറൂന(അ)ല്ല; പ്രത്യുത ഇസ്രായീല്യരില്‍പെട്ട ഒരു കപടനായ സാമിരിയാണ് ഇത് ചെയ്തതെന്നും അതുമൂലം അയാള്‍  ദൈവകോപത്തിനും ശപിക്കപ്പെട്ട രോഗത്തിനും വിധേയനായെന്നുമാണ് ഖുര്‍ആന്‍ പഠിപ്പിക്കുന്നത് (20:85-95). ബൈബിളില്‍ പരാമര്‍ശിക്കപ്പെട്ട കഥകള്‍ വിവരിക്കുമ്പോഴും അതിലെ നെല്ലും പതിരും വേര്‍ തിരിച്ച് സത്യസന്ധവും സൂക്ഷ്മവുമായ രീതിയില്‍ അവ ജനസമക്ഷം വെക്കുന്ന ഖുര്‍ആന്‍ ദൈവികമാണെന്ന് അതിന്റെ ഈ പ്രത്യേകത തന്നെ സുതരാം വ്യക്തമാക്കുന്നു.

ഖുര്‍ആന്‍, ക്രിസ്തുവിന്റെ അത്ഭുതകരമായ ജനനത്തെ അംഗീകരിക്കുന്നതോടൊപ്പം തന്നെ അദ്ദേഹത്തില്‍ ദിവ്യത്വമാരോപിക്കുന്നതിനെ ശക്തിയായി വിമര്‍ശിക്കുന്നുണ്ട്. കന്യാമര്‍യമിന്റെ അടുക്കല്‍ പരിശുദ്ധാത്മാവ് മനുഷ്യരൂപത്തില്‍ പ്രത്യക്ഷപ്പെട്ട് ക്രിസ്തുവിന്റെ ജനനത്തെക്കുറിച്ച് സുവിശേഷം അറിയിച്ചതും ഈത്തപ്പനച്ചുവട്ടില്‍ നിന്നും പ്രസവം നടന്നതും തൊട്ടിലില്‍വെച്ചുതന്നെ കുഞ്ഞ് സംസാരിച്ചതുമെല്ലാം വിശുദ്ധ ഖുര്‍ആന്‍ വിശദമായി വിവരിക്കുന്നുണ്ട്. (19: 16-36, ആലുഇംറാന്‍ 42 – 50) ഇങ്ങനെയെല്ലാം വിവരിക്കുന്ന ഖുര്‍ആന്‍ തന്നെയാണ് 'അല്ലാഹു തന്നെയാണ്  മര്‍യമിന്റെ മകന്‍ മസീഹ്- എന്ന് പറയുന്നവര്‍ തീര്‍ച്ചയായും സത്യനിഷേധി കളായിരി ക്കുന്നു' (5: 19) വെന്നും 'മര്‍യമിന്റെ മകന്‍ മസീഹ് ഒരു ദൂതന്‍ അല്ലാതെ (മറ്റൊന്നും) അല്ല.' (5:78) എന്നും 'യഹൂദികള്‍ ഉസൈര്‍ അല്ലാഹുവിന്റെ പുത്രനാണെന്ന് പറയുന്നു; ക്രിസ്ത്യാനികള്‍ മസീഹ് അല്ലാഹുവിന്റെ പുത്രനാണെന്നും പറയുന്നു. അത് അവരുടെ വായകൊ ണ്ടുള്ള വാക്കത്രെ. മുമ്പ് അവിശ്വസിച്ചവരുടെ വാക്കിനോട് ഇവര്‍ സാമ്യം പുലര്‍ത്തിക്കൊണ്ടിരിക്കുന്നു. അല്ലാഹു അവരെ നശിപ്പിക്കട്ടെ! എങ്ങനെയാണവര്‍ തെറ്റിക്കപ്പെടുന്നത്' (9: 30)എന്നും പറയുന്നത് . ക്രിസ്തുവി ന്റെ   ജനനവും അത്ഭുത സംഭവങ്ങളുമെല്ലാം അംഗീകരിക്കുന്ന ഖുര്‍ആന്‍ അദ്ദേഹത്തില്‍ ദിവ്യത്വ ത്തിന്റെ ലാഞ്ഛനപോലും ദര്‍ശിക്കുന്നതിനെ എതിര്‍ക്കുന്നുണ്ട്.

മാതാവില്‍ മാത്രം, പിതാവില്ലാതെ ജനിച്ചതുകൊണ്ട് മാത്രം ക്രിസ്തു ദൈവമാണെന്ന് വാദി ക്കുകയാ ണെങ്കില്‍ പിതാവും മാതാവുമില്ലാതെ ജനിച്ച ആദാമാണ് ദൈവമാകാന്‍ ഏറ്റവും അര്‍ഹനെന്ന് ക്രൈസ്തവര്‍ സമ്മതിക്കേണ്ടിവരും. ആദാമിനെ ദൈവപുത്രനെന്ന് ബൈബിള്‍ പരിചയപ്പെടുത്തുന്ന തുകൊണ്ട് ദൈവിക ഏകത്വത്തിലെ മറ്റൊരു ആളത്വമാണ് ആദാം (ലൂക്കോസ് 3: 38)എന്നും സങ്കല്‍പി ക്കേണ്ടിവരും. അപ്പോള്‍ ത്രിയേകത്വം ചതുര്‍ ഏകത്വമായി (Tetranity) പരിണമിക്കേണ്ടിവരും.

അതുപോലെ, മഹാ പുരോഹിതനായ മെല്‍ക്കിസേദക്, ക്രിസ്തുവിനേക്കാളും പരിശുദ്ധാത്മാ വിനേക്കാളുമെല്ലാം ദൈവമാകാന്‍ അര്‍ഹനാണ്, പുതിയ നിയമത്തിന്റെ അഭിപ്രായത്തില്‍. സലേമിന്റെ രാജാവും, അത്യുന്നതനായ ദൈവത്തിന്റെ പുരോഹിതനുമെന്ന് പരിചയപ്പെടുത്ത പ്പെട്ട  മെല്‍ക്കീ സേദക്കിന് 'പിതാവോ മാതാവോ വംശപരമ്പരയോ ഇല്ല. അവന്റെ ദിവസങ്ങള്‍ക്ക് ആരംഭമോ ആയുസ്സിന് അവസാനമോ ഇല്ല. ദൈവ പുത്രനും സദൃശനായ അവന്‍ എന്നേക്കും പുരോഹിതനാണ്.'(എബ്രായര്‍ 7:3)

ആദിയും അന്ത്യവുമില്ലാത്ത, മാതാവും പിതാവുമില്ലാത്ത, മഹാ പുരോഹിതനായ 'മെല്‍ക്കീസോക്ക്', ദൈവപുത്രന് സദൃശ്യനാണെന്ന വ്യാഖ്യാനത്തില്‍ അഭയം തേടുകയാണ് ക്രൈസ്തവ സഭകള്‍. ദൈവപുത്രനെന്ന് വ്യവഹരിക്കപ്പെടുന്ന ക്രിസ്തുവിനുപോലും ആദിയും അന്ത്യവുമുണ്ടെന്നും മാതാവുണ്ടെന്നുമുള്ള വസ്തുതതയുടെ വെളിച്ചത്തില്‍ ക്രിസ്തുവെക്കാളും ദൈവമാകാന്‍ അര്‍ഹന്‍ 'മെല്‍ക്കി സേദക് തന്നെയാണ്. (?) ഇങ്ങനെ, കണക്കാക്കാന്‍ തുടങ്ങിയാല്‍ ക്രിസ്തുമതം ചതുര്‍കത്വത്തിലേക്കും പിന്നെ പഞ്ച ഏകത്വത്തിലേക്കും (Pentanity) പരിണമിക്കേണ്ടിവരുമെന്നര്‍ത്ഥം.

വിഷയവുമായി ബന്ധപ്പെട്ട വീഡിയോ

ഹിജ്‌റ നിർബന്ധമാണെന്ന് വ്യക്തമാക്കുന്ന ഖുർആൻ വചനങ്ങൾ മറുനാട്ടിൽ പോയി ജനിച്ച നാടിനെതിരെ പോരാടുവാനുള്ള പ്രചോദനം നൽകി ഭീകരത വളർത്തുന്നതാണെന്ന ആരോപണം പ്രസ്തുത വചങ്ങളുടെ അവതരണ പശ്ചാത്തലത്തെ കുറിച്ച് അറിയാത്തതുകൊണ്ടുണ്ടാവുന്നതാണ്.

''തീര്‍ച്ചയായും വിശ്വസിക്കുകയും, സ്വദേശം വെടിഞ്ഞ് പോകുകയും തങ്ങളുടെ സ്വത്തുക്കളും ശരീരങ്ങളും കൊണ്ട് അല്ലാഹുവിന്റെ മാര്‍ഗത്തില്‍ സമരത്തില്‍ ഏര്‍പെടുകയും ചെയ്തവരും, അവര്‍ക്ക് അഭയം നല്‍കുകയും സഹായിക്കുകയും ചെയ്തവരും അന്യോന്യം ഉറ്റമിത്രങ്ങളാകുന്നു. വിശ്വസിക്കുകയും എന്നാല്‍ സ്വദേശം വെടിഞ്ഞ് പോകാതിരിക്കുകയും ചെയ്തവരോട് അവര്‍ സ്വദേശം വെടിഞ്ഞ് പോരുന്നത് വരെ നിങ്ങള്‍ക്ക് യാതൊരു സംരക്ഷണ ബാധ്യതയുമില്ല. ഇനി മതകാര്യത്തില്‍ അവര്‍ നിങ്ങളുടെ സഹായം തേടുകയാണെങ്കില്‍ സഹായിക്കാന്‍ നിങ്ങള്‍ക്ക് ബാധ്യതയുണ്ട്. എന്നാല്‍ നിങ്ങളുമായി കരാറില്‍ ഏര്‍പെട്ടുകഴിയുന്ന ജനതയ്‌ക്കെതിരെ (നിങ്ങളവരെ സഹായിക്കാന്‍) പാടില്ല. അല്ലാഹു നിങ്ങള്‍ പ്രവര്‍ത്തിക്കുന്നതെല്ലാം കണ്ടറിയുന്നവനാകുന്നു.'' (8:72)

''വിശ്വസിക്കുകയും സ്വദേശം വെടിയുകയും തങ്ങളുടെ സ്വത്തുക്കളും ശരീരങ്ങളും കൊണ്ട് അല്ലാഹുവിന്റെ മാര്‍ഗത്തില്‍ സമരം നടത്തുകയും ചെയ്തവര്‍ അല്ലാഹുവിങ്കല്‍ ഏറ്റവും മഹത്തായ പദവിയുള്ളവരാണ്. അവര്‍ തന്നെയാണ് വിജയം പ്രാപിച്ചവര്‍. അവര്‍ക്ക് അവരുടെ രക്ഷിതാവ് അവന്റെ പക്കല്‍ നിന്നുള്ള കാരുണ്യത്തെയും പ്രീതിയെയും സ്വര്‍ഗത്തോപ്പുകളെയും പറ്റി സന്തോഷവാര്‍ത്ത അറിയിക്കുന്നു. അവര്‍ക്ക് അവിടെ ശാശ്വതമായ സുഖാനുഭവമാണുള്ളത്. അവരതില്‍ നിത്യവാസികളായിരിക്കും. തീര്‍ച്ചയായും അല്ലാഹുവിന്റെ അടുക്കലാണ് മഹത്തായ പ്രതിഫലമുള്ളത്.'' (9:20-22)

''(അവിശ്വാസികളുടെ ഇടയില്‍ തന്നെ ജീവിച്ചുകൊണ്ട്) സ്വന്തത്തോട് അന്യായം ചെയ്തവരെ മരിപ്പിക്കുമ്പോള്‍ മലക്കുകള്‍ അവരോട് ചോദിക്കും: നിങ്ങളെന്തൊരു നിലപാടിലായിരുന്നു? അവര്‍ പറയും: ഞങ്ങള്‍ നാട്ടില്‍ അടിച്ചൊതുക്കപ്പെട്ടവരായിരുന്നു. അവര്‍ (മലക്കുകള്‍) ചോദിക്കും: അല്ലാഹുവിന്റെ ഭൂമി വിശാലമായിരുന്നില്ലേ? നിങ്ങള്‍ക്ക് സ്വദേശം വിട്ട് അതില്‍ എവിടെയെങ്കിലും പോകാമായിരുന്നല്ലോ. എന്നാല്‍ അത്തരക്കാരുടെ വാസസ്ഥലം നരകമത്രെ. അതെത്ര ചീത്ത സങ്കേതം! എന്നാല്‍ യാതൊരു ഉപായവും സ്വീകരിക്കാന്‍ കഴിവില്ലാതെ, ഒരു രക്ഷാമാര്‍ഗവും കണ്ടെത്താനാകാതെ അടിച്ചൊതുക്കപ്പെട്ടവരായിക്കഴിയുന്ന പുരുഷന്‍മാരും സ്ത്രീകളും കുട്ടികളും ഇതില്‍ നിന്നൊഴിവാകുന്നു.'' (4:97,98)

മുഹമ്മദ് നബി (സ)  മദീനയിലെത്തുകയും അവിടെ ഒരു ഇസ്‌ലാമികസമൂഹം രൂപപ്പെടുകയും ചെയ്തതോടെ മക്കയിലുള്ള മുസ്‌ലിംകള്‍ക്കെല്ലാം ഹിജ്‌റ നിര്‍ബന്ധബാധ്യതയായിത്തീര്‍ന്നുവെന്ന് വ്യക്തമാക്കുന്നതാണ് ഈ വചനങ്ങൾ. പീഡനങ്ങള്‍ സഹിക്കേണ്ടിവരുമെന്ന് പേടിച്ച് ആദര്‍ശം പുറത്തുപറയാതെ ജീവിക്കേണ്ടുന്ന അവസ്ഥ വെടിഞ്ഞ് ഹിജ്‌റ ചെയ്ത് മദീനയിലെത്തുവാന്‍ വിശ്വാസികളെ ആഹ്വാനം ചെയ്യുന്നവയാണിവ. ഇസ്‌ലാമിക രാഷ്ട്രങ്ങളുണ്ടായതിനു ശേഷം പിന്നെ മക്കയില്‍ ജീവിക്കുന്നവര്‍ യഥാര്‍ഥത്തില്‍ സ്വീകരിച്ചിരിക്കുന്നത് ആത്മഹത്യാപരമായ നിലപാടാണെന്നും അവരുടെ ആദര്‍ശം സ്വാഭാവികമായും ചോര്‍ന്നുപോകാന്‍ സാധ്യതയുണ്ടെന്നും നബി (സ) വ്യക്തമാക്കിയിട്ടുണ്ട്. അതുകൊണ്ടാണ് ‘ബഹുദൈവാരാധകരോടൊപ്പം ജീവിക്കുന്ന മുസ്‌ലിംകളുടെ കാര്യത്തില്‍ എനിക്കൊരു ഉത്തരവാദിത്തവുമില്ല; 'അവരുടെ തീയില്‍ നിന്ന് നിങ്ങള്‍ വെളിച്ചമെടുക്കേണ്ടിവരരുത്'(സുനനു അബൂദാവൂദ്) എന്ന് പ്രവാചകൻ (സ) പഠിപ്പിച്ചത്. പ്രവാചകന്റെ നേതൃത്വത്തില്‍ ഇസ്‌ലാമികരാഷ്ട്രമുണ്ടാവുകയും അവിടെ സ്വതന്ത്രമായി മതമനുഷ്ഠിച്ച് ജീവിക്കുവാന്‍ കഴിയുന്ന അവസ്ഥയുണ്ടാവുകയും ചെയ്തശേഷം മുശ്‌രിക്കുകളോടൊപ്പം ജീവിച്ച് അവരുടെ പീഡനങ്ങള്‍ സഹിക്കേണ്ടതില്ലെന്നും അങ്ങനെ ചെയ്യുന്നപക്ഷം അവരുടെ ആശയങ്ങളില്‍ പലതും സ്വീകരിക്കേണ്ട അവസ്ഥയുണ്ടാകുമെന്നും അത്തരക്കാരുടെ കാര്യത്തില്‍ എനിക്ക് ഉത്തരവാദിത്തമൊന്നുമില്ലെന്നുമാണ് ഇവിടെ പ്രവാചകൻ (സ) നിഷ്‌കര്‍ഷിക്കുന്നത്.

ഇസ്‌ലാമികാദര്‍ശമനുസരിച്ച് ജീവിക്കുവാന്‍ സ്വാതന്ത്ര്യം നല്‍കപ്പെടുന്ന നാട്ടില്‍ നിന്ന് ഹിജ്‌റ പോവേണ്ടതില്ലെന്ന വസ്തുത ഹിജ്‌റ ആറാം വര്‍ഷം വരെ അബ്‌സീനിയയില്‍ കഴിയാന്‍ ജഅ്ഫറുബ്‌നു അബീത്വാലിബിനെയും  (റ)കൂടെയുള്ളവരെയും അനുവദിച്ച പ്രവാചകനടപടിയില്‍ നിന്ന് വ്യക്തമാവുന്നുണ്ട്. മദീനാരാഷ്ട്രത്തിന്റെ സൈനിക ശക്തി വര്‍ധിപ്പിക്കുവാനാണ് എല്ലാ മുസ്‌ലിംകളോടും ഹിജ്‌റ ചെയ്യാന്‍ മുഹമ്മദ് നബി (സ)കല്‍പിച്ചതെന്ന് വിമര്‍ശിക്കുന്നവര്‍ നബി (സ) ക്ക് അത്തരമൊരു ഉദ്ദേശമുണ്ടായിരുന്നെങ്കില്‍ തന്നെ അതിലെന്താണ് തെറ്റെന്ന് പറഞ്ഞുതരുവാന്‍ ബാധ്യസ്ഥരാണ്. മക്കയിലും മറ്റു സ്ഥലങ്ങളിലും പീഡനങ്ങളനുഭവിച്ചുകൊണ്ടിരിക്കുന്നവര്‍ മദീനയിലെത്തുകയും അതുവഴി ഇസ്‌ലാമികരാഷ്ട്രത്തിന്റെ സൈനികശക്തി വര്‍ധിക്കുകയും ചെയ്യണമെന്ന് ഒരു രാഷ്ട്രനേതാവെന്ന നിലയില്‍ മുഹമ്മദ് നബി (സ)ആഗ്രഹിച്ചുവെങ്കില്‍, ഏതു തരം നൈതികയുടെ അടിസ്ഥാനത്തിലാണ് അതില്‍ തെറ്റുകാണാന്‍ കഴിയുകയെന്ന് അവര്‍ വ്യക്തമാക്കേണ്ടത്.

അസ്വാതന്ത്ര്യത്തില്‍ നിന്ന് സ്വാതന്ത്ര്യത്തിലേക്കുള്ള പലായനമാണ് ഹിജ്‌റ. ഒരു രാജ്യം ഇസ്‌ലാമികമാണോ അല്ലേയെന്നതല്ല പ്രത്യുത മുസ്‌ലിമായി ജീവിക്കുവാന്‍ അനുവദിക്കുന്നതാണോ അല്ലേയെന്നതാണ് അവിടെ നിന്ന് മുസ്‌ലിംകള്‍ ഹിജ്‌റ പോകേണ്ടതുണ്ടോ ഇല്ലേയെന്ന് തീരുമാനിക്കുന്നത്. രാജ്യങ്ങളെ ദാറുല്‍ ഇസ്‌ലാം, ദാറുല്‍ ഹര്‍ബ്, ദാറുല്‍ അഹദ് എന്നിങ്ങനെ വര്‍ഗീകരിക്കുന്ന പില്‍ക്കാല രീതിയനുസരിച്ച് ദാറുല്‍ ഹര്‍ബില്‍ നിന്ന്- (മുസ്‌ലിംകളുമായി യുദ്ധത്തിലേര്‍പ്പെട്ടിരിക്കുന്ന അമുസ്‌ലിം രാഷ്ട്രം) ദാറുല്‍ ഇസ്‌ലാമിലേക്കാണ് (ഇസ്‌ലാമികരാഷ്ട്രം) ഹിജ്‌റ നടക്കേണ്ടത്. ദാറുല്‍ അഹദ് ഇസ്‌ലാമികരാഷ്ട്രവുമായി സന്ധിയിലേര്‍പ്പെട്ടിരിക്കുന്ന രാജ്യമാണ്; അവിടെ മുസ്‌ലിംകള്‍ക്ക് സ്വാതന്ത്ര്യത്തോടെ മതമനുഷ്ഠിച്ച് ജീവിക്കുവാന്‍ സാധിക്കുന്നതിനാല്‍ അത്തരം രാജ്യങ്ങളില്‍ നിന്ന് ഹിജ്‌റ ചെയ്യേണ്ടതില്ലെന്ന് വ്യക്തമാക്കുന്നതാണ് ഇസ്‌ലാമികരാഷ്ട്രമുണ്ടായി വര്‍ഷങ്ങള്‍ കഴിഞ്ഞിട്ടും അബ്‌സീനിയയില്‍ ജീവിക്കുവാന്‍ മുസ്‌ലിംകളെ അനുവദിച്ച പ്രവാചകന്റെ നടപടി. ജനാധിപത്യക്രമം നിലനില്‍ക്കുന്ന രാഷ്ട്രസംവിധാനങ്ങളില്‍ മതസ്വാതന്ത്ര്യം അനുവദിക്കപ്പെടുന്നുവെങ്കില്‍ അതിനെ വിളിക്കുവാന്‍ ഏറ്റവും അനുയോജ്യമായ പദം ‘ദാറുല്‍ അംന്’-സുരക്ഷിത നാട്- എന്നാണ്. മുസ്‌ലിമായി ജീവിക്കുവാന്‍ സ്വാതന്ത്ര്യം നല്‍കുകയും അത് പ്രബോധനം ചെയ്യാന്‍ അനുവദിക്കുകയും ചെയ്യുന്ന രാഷ്ട്രമാണ് ദാറുല്‍ അംന്. അത്തരം രാജ്യങ്ങളോട് കലാപത്തിലേര്‍പ്പെടുകയോ അവിടെ നിന്ന് പലായനം ചെയ്യുകയോ അല്ല; മാതൃകായോഗ്യരായി ജീവിക്കുകയും സഹജീവികള്‍ക്ക് സത്യസന്ദേശമെത്തിക്കുകയുമാണ് മുസ്‌ലിമിന്റെ കടമ.

'ഇനി ഹിജ്‌റയില്ല'(സ്വഹീഹുല്‍ ബുഖാരി) യെന്ന് മക്കാ വിജയ ദിവസം പ്രവാചകൻ (സ) പ്രഖ്യാപിച്ചതില്‍ നിന്നും 'മക്കാ വിജയത്തിനു ശേഷം ഹിജ്‌റയില്ല'(സ്വഹീഹുമുസ്‌ലിം) യെന്ന് അദ്ദേഹം വ്യക്തമാക്കിയതില്‍ നിന്നും സമാധാനത്തോടെ ജീവിക്കുവാന്‍ കഴിയുന്ന സാഹചര്യങ്ങള്‍ സംജാതമായിക്കഴിഞ്ഞാല്‍ പിന്നെ പലായനം വേണ്ടതില്ലെന്ന് മനസ്സിലാവുന്നുണ്ട്. മക്കയില്‍ മുസ്‌ലിമായി ജീവിക്കുവാനുള്ള സാഹചര്യങ്ങള്‍ ഒരുങ്ങിക്കഴിഞ്ഞ സ്ഥിതിക്ക് ഇനി മക്കയില്‍ നിന്ന് മദീനയിലേക്കുള്ള ഹിജ്‌റയുടെ ആവശ്യമില്ലെന്നാണ് ഇതു പഠിപ്പിക്കുന്നത്. എന്നാല്‍, മതത്തിന്റെ പേരിലുള്ള പീഡനങ്ങള്‍ നിലനില്‍ക്കുന്നിടത്തോളം ഹിജ്‌റ അനിവാര്യമായിത്തീരുമെന്നും പ്രവാചകൻ (സ)പഠിപ്പിച്ചിട്ടുണ്ട്. 'പശ്ചാത്താപം നിലയ്ക്കുന്നതുവരെ ഹിജ്‌റ അവസാനിക്കുകയില്ല. സൂര്യന്‍ പടിഞ്ഞാറ് നിന്ന് ഉദിച്ചുയരുന്നതുവരെ പശ്ചാത്താപം അവസാനിക്കുകയില്ല'(സുനനു അബൂദാവൂദ് ) എന്ന പ്രവാചകവചനം വ്യക്തമാക്കുന്നത് ഇക്കാര്യമാണ്. മതത്തിന്റെ പേരിലുള്ള പീഡനങ്ങള്‍ നിലനില്‍ക്കുന്നിടത്തോളം, അവസാനനാളു വരെ ഹിജ്‌റ പ്രസക്തമായിരിക്കുമെന്ന് സാരം.

ജനിച്ചു വളര്‍ന്ന നാടിനെക്കാളും പോറ്റി വളര്‍ത്തിയ മാതാപിതാക്കളേക്കാളും അധ്വാനിച്ചുണ്ടാക്കിയ സമ്പാദ്യത്തേക്കാളുമെല്ലാം മുസ്‌ലിംകള്‍ വിലമതിക്കുന്നത് ദൈവിക മാര്‍ഗദര്‍ശനപ്രകാരമുള്ള ജീവിതവും അതെപ്പറ്റി സഹജീവികള്‍ക്ക് പറഞ്ഞുകൊടുക്കുവാനുള്ള അവസരവുമാണെന്നതാണ് ഹിജ്‌റ നല്‍കുന്ന സന്ദേശം. കലാപവും കുഴപ്പവുമുണ്ടാക്കി ജീവിക്കുന്ന നാട്ടില്‍ അരാജകത്വം സൃഷ്ടിക്കുന്നതിനല്ല ആദര്‍ശജീവിതത്തിന് പറ്റുന്നിടത്തേക്ക് പലായനം ചെയ്യുവാനാണ് ഇസ്‌ലാം അനുശാസിക്കുന്നത്. അങ്ങനെ പലായനം ചെയ്യുന്നവര്‍ക്കാണ് അല്ലാഹുവിന്റെയടുക്കല്‍ ഉന്നതമായ പദവിയും സ്വര്‍ഗീയ സുഖസൗകര്യങ്ങളുമുണ്ടെന്ന് ക്വുര്‍ആന്‍ സന്തോഷവാര്‍ത്തയറിയിക്കുന്നത്.

''വിശ്വസിക്കുകയും സ്വദേശം വെടിയുകയും തങ്ങളുടെ സ്വത്തുക്കളും ശരീരങ്ങളും കൊണ്ട് അല്ലാഹുവിന്റെ മാര്‍ഗത്തില്‍ സമരം നടത്തുകയും ചെയ്തവര്‍ അല്ലാഹുവിങ്കല്‍ ഏറ്റവും മഹത്തായ പദവിയുള്ളവരാണ്. അവര്‍ തന്നെയാണ് വിജയം പ്രാപിച്ചവര്‍. അവര്‍ക്ക് അവരുടെ രക്ഷിതാവ് അവന്റെ പക്കല്‍ നിന്നുള്ള കാരുണ്യത്തെയും പ്രീതിയെയും സ്വര്‍ഗത്തോപ്പുകളെയും പറ്റി സന്തോഷവാര്‍ത്ത അറിയിക്കുന്നു. അവര്‍ക്ക് അവിടെ ശാശ്വതമായ സുഖാനുഭവമാണുള്ളത്. അവരതില്‍ നിത്യവാസികളായിരിക്കും. തീര്‍ച്ചയായും അല്ലാഹുവിന്റെ അടുക്കലാണ് മഹത്തായ പ്രതിഫലമുള്ളത്.'' (9:20-22)

മുഹമ്മദ് നബി (സ) പ്രവാചകത്വം വാദിച്ചത് തന്നെ അധികാരം ലക്ഷ്യമാക്കിയായിരുന്നു. കുറച്ച് അനുയായികളായപ്പോൾ തനിക്ക് അധികാരമുള്ള ഒരു നാട് അദ്ദേഹം അന്വേഷിക്കാൻ തുടങ്ങി. പ്രസ്തുത അന്വേഷണമാണ് അദ്ദേഹത്തെ മദീനയിലെത്തിക്കുകയും അവിടുത്തെ അധികാരിയായിത്ത്തീരുന്നതിന് കാരണമാവുകയും ചെയ്തത്. ഈ ആരോപണത്തെക്കുറിച്ച് എന്ത് പറയുന്നു?

ധികാരം മുന്നില്‍ കണ്ടുകൊണ്ട്, തന്റെ കീഴിലുള്ള ഒരു രാഷ്ട്ര നിര്‍മിതിക്കുവേണ്ടി മുഹമ്മദ് ല നടത്തിയ ശ്രമത്തിന്റെ പ്രാരംഭമായി ഒരു തലസ്ഥാന നഗരി അന്വേഷിച്ചതിന്റെ ഫലമായാണ് അദ്ദേഹം മദീനയിലെത്തിയതെന്ന് വിമര്‍ശിക്കുന്നവര്‍ മദീനാ ഹിജ്‌റയ്ക്കുമുമ്പ് നടന്ന സംഭവങ്ങളെല്ലാം കണ്ടില്ലെന്ന് നടിക്കുകയാണ് ചെയ്യുന്നത്. തനിക്ക് ഭരിക്കാന്‍ പറ്റുന്ന ഒരു പ്രദേശം തിരഞ്ഞ് കണ്ടെത്തുകയായിരുന്നില്ല, ഇസ്‌ലാമിക പ്രബോധനത്തിന് സ്വാതന്ത്ര്യമുള്ള ഒരിടത്ത് അഭയം ലഭിക്കുകയായായിരുന്നു നബി (സ)യുടെ ഉദ്ദേശമെന്ന് ദുഃഖവര്‍ഷമെന്ന് വിളിക്കപ്പെട്ട, അബൂത്വാലിബിന്റെയും ഖദീജ(റ) യുടെയും മരണം നടന്ന വര്‍ഷത്തിന് ശേഷമുള്ള നബിവര്‍ത്തനങ്ങള്‍ പരിശോധിക്കുന്ന ആര്‍ക്കും മനസ്സിലാകും. 'അബൂത്വാലിബ് ജീവിച്ചിരുന്നെങ്കില്‍ ഖുറൈശികള്‍ എന്നോട് ഇങ്ങനെയൊന്നും ചെയ്യുകയില്ലായിരുന്നു' വെന്ന് നബി (സ)യെക്കൊണ്ട് പറയിപ്പിക്കുന്ന രീതിയിലുള്ള ഖുറൈശീ ക്രൂരതകളാണ് മക്കയില്‍ നിന്ന് പോവുകയെന്ന തീരുമാനത്തിലേക്ക് നബി (സ) എത്തുന്നതിനുള്ള കാരണം.

തനിക്ക് സംരക്ഷണം തേടിക്കൊണ്ട് ഒന്നാമതായി നബി (സ) നടത്തിയ ത്വാഇഫിലേക്കുള്ള യാത്രയുടെ ലക്ഷ്യമെന്തായിരുന്നുവെന്ന് അവിടെ എത്തിയശേഷം അദ്ദേഹം എന്താണ് ആവശ്യപ്പെട്ടതെന്ന് പരിശോധിച്ചാല്‍ വ്യക്തമാകും. ഥഖീഫ് ഗോത്രത്തിലെ പ്രധാനികളായ, അംറുബ്‌നു ഉമൈറിന്റെ പുത്രന്‍മാരായ അബ്ദ് യഅ്‌ലില്‍, മസ്ഊദ്, ഹബീബ് എന്നിവരോട് ഇസ്‌ലാമിനെക്കുറിച്ച് പറഞ്ഞുകൊടുക്കുകയും അതിലേക്ക് ക്ഷണിക്കുകയും തന്നെ സംരക്ഷിക്കണമെന്ന് ആവശ്യപ്പെടുകയുമാണ് നബി (സ) ചെയ്തത്. ത്വാഇഫിന്റെ അധികാരം ചോദിക്കുകയോ അധികാരത്തില്‍ പങ്കുനല്‍കണമെന്ന് ആവശ്യപ്പെടുകയോ ചെയ്തതുകൊണ്ടാണ് ഥഖീഫ് ഗോത്രക്കാര്‍ നബി (സ)യെ അവമതിക്കുകയും അപമാനിക്കുകയും ചെയ്തതെന്ന് നബി (സ)യെക്കുറിച്ച് നബി വിരോധികള്‍ എഴുതിയ ചരിത്ര ഗ്രന്ഥങ്ങള്‍പോലും ആരോപിക്കുന്നില്ല.  ത്വാഇഫില്‍ നിന്ന് തുടങ്ങിയ സംരക്ഷണം ആവശ്യപ്പെട്ടുകൊണ്ടുള്ള നബി (സ)യുടെ യാത്രയാണ് യഥ്‌രിബില്‍ ചെന്ന് അവസാനിക്കുകയും അത് മദീനത്തുര്‍റസൂലായി മാറുന്നതിലേക്ക് നയിക്കുകയും ചെയ്തതെന്നുള്ളതാണ് യാഥാര്‍ഥ്യം.

ത്വാഇഫില്‍നിന്ന് മുത്വ്ഇം ബിന്‍ ഹാദിയുടെ സംരക്ഷണത്തില്‍ മക്കയിലേക്ക് തിരിച്ചെത്തിയതുമുതല്‍ മുഹമ്മദ് നബി (സ)യുടെ അന്വേഷണങ്ങള്‍ കേന്ദ്രീകരിച്ചിരുന്നത് തനിക്ക് അധികാരം ലഭിക്കുവാന്‍ എന്തെങ്കിലും മാര്‍ഗങ്ങളുണ്ടോയെന്നതിലായിരുന്നില്ലെന്നും പ്രത്യുത തനിക്കും ഇസ്‌ലാമിനും സംരക്ഷണം നല്‍കാന്‍ ആരെങ്കിലുമുണ്ടോയെന്നതിലായിരുന്നെന്നുമുള്ള വസ്തുത ഹിജ്‌റക്ക് പിന്നില്‍ അധികാര സ്ഥാപനം ആരോപിക്കുന്നവരുടെ കണ്ണ് തുറപ്പിക്കേണ്ടതാണ്. മക്കക്ക് പുറത്തുനിന്ന് ഹജ്ജിനും ഉംറക്കും വരുന്നവരോട് തന്നെ സംരക്ഷിക്കുവാന്‍ സന്നദ്ധരായ ആരെങ്കിലുമുണ്ടോയെന്ന് അന്വേഷിച്ച് മുഹമ്മദ് നബി (സ) ചുറ്റി നടന്നിരുന്നുവെന്ന് സ്വഹീഹായി നിവേദനം ചെയ്യപ്പെട്ടിട്ടുണ്ട്. 'എന്റെ നാഥന്റെ വചനങ്ങള്‍ ജനങ്ങള്‍ക്ക് എത്തിച്ചുകൊടുക്കുന്നതില്‍ നിന്ന് ഖുറൈശികള്‍ എന്നെ തടഞ്ഞിരിക്കുന്നു; തങ്ങളുടെ ജനതയിലേക്ക് എന്നെ കൊണ്ടുപോകുവാന്‍ ആരെങ്കിലും തയാറുണ്ടോ?'യെന്ന് ഹജ്ജിന് വരുന്നവരോടായി അറഫയില്‍വെച്ച് നബി പറഞ്ഞുകൊണ്ടിരുന്നതായി ജാബിറുബ്‌നു അബ്ദില്ല (റ) നിവേദനം ചെയ്യുന്നുണ്ട്.(അബൂദാവൂദ്, തിര്‍മിദി, ഇബ്‌നുമാജ) തന്നെക്കൊണ്ടാവുന്ന വിധത്തില്‍ ഓരോ തീര്‍ഥാടകരെയും നേരില്‍ കണ്ട് അവരുടെ പേര് വിളിച്ചുകൊണ്ട് പ്രവാചകൻ (സ) ഇങ്ങനെ പറയുമായിരുന്നുവത്രെ: 'ഞാന്‍ നിങ്ങളിലേക്കുള്ള അല്ലാഹുവിന്റെ ദൂതനാണ്. അവനെ ആരാധിക്കണമെന്നും അവനില്‍ ആരെയും പങ്കുചേര്‍ക്കരുതെന്നും അവന്‍ കല്‍പിച്ചിരിക്കുന്നു. അവനെയല്ലാതെ നിങ്ങള്‍ ആരാധിച്ചുകൊണ്ടിരിക്കുന്നവയെയെല്ലാം നിങ്ങള്‍ കയ്യൊഴിക്കുക; എന്നില്‍ വിശ്വസിക്കുകയും എന്നെ സംരക്ഷിക്കുകയും ചെയ്ത് എന്തിനാണോ ഞാന്‍ അയക്കപ്പെട്ടത് അത് ജനങ്ങള്‍ക്ക് സമര്‍പ്പിക്കുവാന്‍ എനിക്ക് അവസരം നല്‍കുക'(മുസ്‌നദ് ഇമാം അഹ്മദ് ). ഈ സംസാരങ്ങളിലൊന്നും തന്നെ അധികാര സംസ്ഥാപനത്തിന്റെയോ അതിനുള്ള ഒരുക്കത്തിന്റെയോ പ്രതീക്ഷയുടെയോ ലാഞ്ചനകളൊന്നും കാണാനാവുന്നില്ലെങ്കില്‍ പിന്നെ ഹിജ്‌റയുടെ ലക്ഷ്യം രാഷ്ട്രസംസ്ഥാപനമായിരുന്നുവെന്ന് കരുതുന്നതിന് എന്ത് ന്യായീകരണമാണുള്ളത്.

താന്‍ സഹായമഭ്യര്‍ഥിക്കുന്ന തീര്‍ഥാടകരുടെയെല്ലാം പിന്നില്‍ നടന്ന് അബൂലഹബ് പിന്തിരിപ്പിക്കുകയും അപവാദ പ്രചരണം നടത്തുകയും ചെയ്യുന്ന അവസ്ഥ സംജാതമായപ്പോള്‍ രഹസ്യമായി ഗോത്ര നേതാക്കളെ സന്ദര്‍ശിച്ച് സംരക്ഷണം അഭ്യര്‍ഥിക്കുന്ന പ്രവാചകനിലും (സ) നാം കാണുന്നത് ഒരു അധികാര കേന്ദ്രമന്വേഷിക്കുന്ന രാഷ്ട്രീയ നേതാവിനെയല്ല, പ്രത്യുത ആദര്‍ശ പ്രചരണ സ്വാതന്ത്ര്യത്തിന് ദാഹിക്കുന്ന ദൈവദൂതനെയാണ്. കിന്‍ക, കല്‍ബ്, ബനൂ ആമിര്‍, ബനൂ ഹനീഫ, മുഹാരിബ്, ഫുസാറ, ഗസ്സാന്‍, മുര്‍റ, ഹനീഫ, സുലൈം, അബ്‌സ്, ബനൂ നസ്വ്‌റ്, ബനൂ അല്‍ ബക്കാഅ്, അല്‍ ഹാരിഥ്, ഉദ്‌റ, ഹദാറമ തുടങ്ങിയ ഗോത്രങ്ങള്‍ക്കടുത്തെത്തി തന്നെയും തന്റെ ആദര്‍ശ പ്രചരണത്തെയും സംരക്ഷിക്കണമെന്ന് ആവശ്യപ്പെടുകയുമാണ് പ്രവാചകൻ (സ) ചെയ്തത്. അവിടെയൊന്നും തന്നെ അധികാര സ്ഥാപനത്തിന്റെ യാതൊരു പരാമര്‍ശവും കാണാന്‍ കഴിയില്ല. പ്രവാചകനോട്‌ (സ) വളരെ മോശമായി പെരുമാറിയ ഗോത്രങ്ങളിലൊന്ന് ബനൂ ആമിറുബ്‌നു സഅ്‌സാഅ് ആയിരുന്നു. തന്റെ സംരക്ഷണാഭ്യര്‍ഥനക്ക് മറുപടിയായി ബനൂ ആമിര്‍ ഗോത്രത്തിലെ ബൈഹറാബ്‌നു ഫിറാസ് 'താങ്കളുമായി ഞങ്ങള്‍ ഉടമ്പടി ചെയ്യുകയും എന്നിട്ട് അറബികള്‍ക്കുമേല്‍ അല്ലാഹു താങ്കളെ വിജയിപ്പിക്കുകയും ചെയ്താല്‍ താങ്കള്‍ക്കുശേഷം ഞങ്ങള്‍ക്ക് അധികാരം നല്‍കുമോ?'യെന്ന് ചോദിച്ചപ്പോള്‍ 'കാര്യം അല്ലാഹുവിന്റെ പക്കലാണ്; അവര്‍ ഉദ്ദേശിക്കുന്നയിടത്താണ് അവനത് വെക്കുക'യെന്നാണ് പ്രവാചകൻ (സ) മറുപടി പറഞ്ഞത്. 'നിനക്കുവേണ്ടി ഞങ്ങള്‍ ഞങ്ങളുടെ കഴുത്തുകള്‍ അറബികളുടെ വാളുകള്‍ക്ക് മുന്നില്‍ വെക്കുകയും അവസാനം വിജയിക്കുമ്പോള്‍ നേതൃത്വം മറ്റുള്ളവര്‍ക്കും അല്ലേ... നിന്റെ ഈ കാര്യം ഞങ്ങള്‍ക്കാവശ്യമില്ല'യെന്ന് പറഞ്ഞാണ് അവര്‍ അല്ലാഹുവിന്റെ ദൂതനെ തിരിച്ചയച്ചത്.

ഹജ്ജിന് വരുന്നവരെ ഇസ്‌ലാമിലേക്ക് ക്ഷണിക്കുന്ന സന്ദര്‍ഭത്തില്‍ സത്യദീനിന്റെ സൗരഭ്യം മനസ്സിലാക്കി അത് സ്വീകരിക്കുവാന്‍ ഭാഗ്യം ലഭിച്ചവരിലൂടെയാണ് ഇസ്‌ലാം ആദ്യമായി യഥ്‌രിബിലെത്തുന്നതും നബി (സ)യുടെ പലായനത്തിന്റെ വാതില്‍ തുറന്നതും. കവിയും ബുദ്ധിജീവിയും ഉന്നതകുലജാതനും ധൈര്യശാലിയുമെല്ലാമായതിനാല്‍ പൂര്‍ണനെന്ന് (അല്‍ കാമില്‍) വിളിക്കപ്പെട്ടിരുന്ന സുബൈദ് ബ്‌നു സാമിതും ഖസ്‌റജുകാര്‍ക്കെതിരെ കക്ഷിചേരുവാന്‍ ഖുറൈശികളോട് അഭ്യര്‍ഥിക്കാന്‍ വന്ന സംഘത്തിലെ യുവാവായിരുന്ന ഇയാസ് ബ്‌നു മുആദുമെല്ലാം പ്രവാചകന്റെ (സ) പ്രബോധനത്തില്‍ ആകൃഷ്ടരായി ഇസ്‌ലാം സ്വീകരിച്ച യഥ്‌രിബുകാരായിരുന്നു.

വ്യക്തിപരമായ പ്രബോധന പ്രവര്‍ത്തനങ്ങളിലൂടെ മെല്ലെ മെല്ലെ യഥ്‌രിബിലേക്ക് പ്രവേശിച്ചുകൊണ്ടിരുന്ന ഇസ്‌ലാമിന്റെ വെളിച്ചം പടര്‍ന്ന് പരന്ന് പ്രവാചകന്റെ (സ) പട്ടണമായിത്തീരുന്നതിലേക്ക് തിരിയുന്നതിന് കാരണമായിത്തീര്‍ന്ന ആദ്യ സംഭവം പ്രവാചക നിയോഗത്തിന്റെ പതിനൊന്നാം വര്‍ഷം നബിയുംല ഖസ്‌റജ് ഗോത്രക്കാരും തമ്മില്‍ നടന്ന ഒരു കൂടിക്കാഴ്ചയായിരുന്നു. ഖസ്‌റജുകാരുമായി നബി (സ) നടത്തിയ കൂടിക്കാഴ്ചയില്‍ ഇസ്‌ലാമിന്റെ അടിസ്ഥാന തത്ത്വങ്ങളെക്കുറിച്ച് പറഞ്ഞുകൊടുക്കുകയും താന്‍ ദൈവദൂതനാണെന്ന വസ്തുത വ്യക്തമാക്കുകയും ചെയ്തപ്പോള്‍ അവരോടൊപ്പം യഥ്‌രിബില്‍ വസിച്ചിരുന്ന ജൂതന്‍മാരില്‍നിന്ന് അന്തിമപ്രവാചകന്റെ ആഗമനമുണ്ടാകുമെന്ന് മനസ്സിലാക്കിയിരുന്ന അവര്‍ക്ക് അദ്ദേഹത്തില്‍ വിശ്വസിക്കാന്‍ പ്രയാസമുണ്ടായില്ല. അവരെല്ലാവരും മുസ്‌ലിംകളായിത്തീരുകയും അവരുടെ നാട്ടുകാര്‍ക്ക് ഇസ്‌ലാം എത്തിച്ചുകൊടുക്കാമെന്ന് സമ്മതിക്കുകയും ചെയ്തു. ആഭ്യന്തര യുദ്ധങ്ങളാല്‍ പരസ്പരം ശത്രുതയില്‍ വര്‍ത്തിക്കുന്ന തങ്ങളുടെ ജനത ഇസ്‌ലാമിലൂടെ ഒറ്റക്കെട്ടായിത്തീര്‍ന്നാല്‍ പ്രവാചകൻ (സ)നെക്കാള്‍ പ്രതാപവാനായി ആരുമില്ലാത്ത അവസ്ഥാവിശേഷമുണ്ടാകുമെന്ന് പ്രത്യാശ പുറപ്പെടുവിച്ചുകൊണ്ടാണ് ആ ഖസ്‌റജുകാര്‍ പിന്നീട് നബി (സ) യുടെ പട്ടണമായി-മദീനത്തുല്‍ റസൂല്‍ അറിയപ്പെട്ട യഥ്‌രിബിലേക്ക് യാത്രതിരിച്ചത്.

അധികാരകേന്ദ്രമന്വേഷിച്ചുള്ള പ്രയാണമല്ല, പ്രത്യുത ആദർശമനുസരിച്ച് ജീവിക്കുവാനും അത് പ്രബോധനം ചെയ്യുന്നതിനുമുള്ള സ്വാതന്ത്യവും തേടിയുള്ള യാത്രയാണ് യഥാർത്ഥത്തിൽ പ്രവാചകനെ മദീനയിലെത്തിച്ചതെന്ന വസ്തുതയാണ് ഇതെല്ലാം വ്യക്തമാക്കുന്നത്.

ല്ല. പ്രവാചകാനുചരന്മാരുടെ ഒന്നാമത്തെ ഹിജ്റ തന്നെ അമുസ്‌ലിം നാട്ടിലേക്കായിരുന്നു. മദീനയിലേക്കുള്ള ഹിജ്റയും അബ്സീനിയയിലേക്കുള്ള ഹിജ്റയും തമ്മിൽ എന്തെങ്കിലും വ്യത്യാസമുണ്ടെന്ന് പ്രവാചകൻ (സ) പഠിപ്പിച്ചിട്ടില്ല.

ആദര്‍ശമനുസരിച്ച് ജീവിക്കാനാവാത്ത സാഹചര്യമാണ് ജനിച്ചുവളര്‍ന്ന നാട്ടില്‍ നിലനില്‍ക്കുന്നതെങ്കില്‍ അതിന് സ്വാതന്ത്ര്യമുള്ള നാട്ടിലേക്ക്, അത് ആര് ഭരിക്കുന്നുവെന്നോ അവിടെ നിലനില്‍ക്കുന്ന വ്യവസ്ഥിതിയോ നോക്കാതെ പലായനം ചെയ്യുകയാണ് മുസ്‌ലിം ചെയ്യേണ്ടതെന്ന പാഠമാണ് ഹിജ്രകളെല്ലാം നൽകുന്നത്.. അബ്‌സീനിയ ക്രൈസ്തവ രാജ്യമാണെന്നതോ അവിടുത്തെ രാജാവ് ക്രിസ്തുമതക്കാരനാണെന്നതോ ഹിജ്‌റയില്‍നിന്ന് അനുചരന്മാരെ പിന്തിരിപ്പിക്കുവാനുള്ള കാരണമായി പ്രവാചകൻ (സ) കരുതിയില്ല. ത്രിത്വദൈവത്തില്‍ വിശ്വസിക്കുകയും യേശുക്രിസ്തുവിനെ ദൈവപുത്രനായി പരിഗണിച്ച് ആരാധിക്കുകയും ചെയ്തവര്‍തന്നെയായിരുന്നു അബ്‌സീനിയയിലെ ക്രിസ്ത്യാനികള്‍. വിശുദ്ധ ക്വുര്‍ആന്‍ ശക്തമായി വിമര്‍ശിക്കുകയും ശാസിക്കുകയും ചെയ്ത വിശ്വാസാചാരങ്ങള്‍ വെച്ചുപുലര്‍ത്തുന്നവരുടെ നാടാണെങ്കിലും സത്യമതമനുസരിച്ച് ജീവിക്കുവാനും അത് പ്രബോധനം ചെയ്യുന്നതിനുമുള്ള സ്വാതന്ത്ര്യം ലഭിക്കുകയാണെങ്കില്‍ അവിടെ സമാധാനപൂര്‍വം ജീവിക്കുകയാണ് മുസ്‌ലിംകള്‍ ചെയ്യേണ്ടതെന്ന പാഠവും അബ്‌സീനിയന്‍ ഹിജ്‌റ നല്‍കുന്നുണ്ട്.  ജീവിക്കുന്ന നാട്ടിലെ വ്യവസ്ഥിതിയേതാണെങ്കിലും അത് ഇസ്‌ലാം വിരുദ്ധമോ മതവിരുദ്ധമോ ആണെങ്കില്‍പോലും അവിടെ മതസ്വാതന്ത്ര്യമുണ്ടെങ്കില്‍ പൂര്‍ണ മുസ്‌ലിംകളും നല്ല പൗരന്‍മാരുമായി ജീവിക്കുവാന്‍ സത്യവിശ്വാസികള്‍ക്ക് കഴിയുമെന്ന് പാഠം.

മക്കയിലെ പീഡനങ്ങളില്‍നിന്ന് രക്ഷപ്പെട്ട്, സ്വസ്ഥതയോടെ മുസ്‌ലിംകളായി ജീവിക്കുവാനാഗ്രഹിച്ച് അബ്‌സീനിയയിലേക്ക് ഹിജ്‌റ പോയവരെ പ്രശംസിച്ചുകൊണ്ട് അവതരിപ്പിക്കപ്പെട്ട ക്വുര്‍ആന്‍ വചനങ്ങളും അവര്‍ക്ക് ഉയര്‍ന്ന പ്രതിഫലമുണ്ടെന്ന് അറിയിച്ചുകൊണ്ടുള്ള പ്രവാചക മൊഴികളും ആദര്‍ശ സ്വാതന്ത്ര്യത്തിനുവേണ്ടിയുള്ള ഹിജ്‌റ, പോകുന്ന പ്രദേശത്തിലെ വ്യവസ്ഥിതി പരിഗണിക്കാതെതന്നെ, മഹത്തരവും ഉജ്ജ്വലവുമാണെന്ന വസ്തുത വ്യക്തമാക്കുന്നവയാണ്. അബ്‌സീനിയയിലേക്ക് പലായനം ചെയ്തവരെക്കുറിച്ച് അവതരിപ്പിക്കപ്പെട്ടതാണെന്ന് വ്യാഖ്യാതാക്കളില്‍ ചിലര്‍ വ്യക്തമാക്കിയ ചില ക്വുര്‍ആന്‍ വചനങ്ങള്‍ നോക്കുക.

''അക്രമത്തിന് വിധേയരായതിന് ശേഷം അല്ലാഹുവിന്റെ മാര്‍ഗത്തില്‍ സ്വദേശം വെടിഞ്ഞ് പോയവരാരോ അവര്‍ക്ക് ഇഹലോകത്ത് നാം നല്ല താമസസൗകര്യം ഏര്‍പെടുത്തികൊടുക്കുകതന്നെ ചെയ്യും. എന്നാല്‍, പരലോകത്തെ പ്രതിഫലം തന്നെയാകുന്നു ഏറ്റവും മഹത്തായത്. അവര്‍ (അത്) അറിഞ്ഞിരുന്നുവെങ്കില്‍!'' (16:41)

''പറയുക: വിശ്വസിച്ചവരായ എന്റെ ദാസന്‍മാരേ, നിങ്ങള്‍ നിങ്ങളുടെ രക്ഷിതാവിനെ സൂക്ഷിക്കുക. ഈ ഐഹികജീവിതത്തില്‍ നന്‍മ പ്രവര്‍ത്തിച്ചവര്‍ക്കാണ് സല്‍ഫലമുള്ളത്. അല്ലാഹുവിന്റെ ഭൂമിയാകട്ടെ വിശാലമാകുന്നു. ക്ഷമാശീലര്‍ക്കു തന്നെയാകുന്നു തങ്ങളുടെ പ്രതിഫലം കണക്കുനോക്കാതെ നിറവേറ്റികൊടുക്കപ്പെടുന്നത്.'' (39:10)

അബ്‌സീനിയയിലേക്ക് ഹിജ്‌റ പോവുകയും പിന്നീട് അവിടെനിന്ന് മദീനയിലെത്തി പ്രവാചകനോടൊപ്പം ചേരുകയും ചെയ്തവര്‍ക്ക് രണ്ട് ഹിജ്‌റയുടെ പ്രതിഫലമുണ്ടെന്നാണ് ബുഖാരിയും മുസ്‌ലിമും ഉദ്ധരിച്ച ഒരു ദീര്‍ഘമായ ഹദീഥിൽ നബി (സ) വ്യക്തമാക്കിയിട്ടുള്ളത്.

അബ്‌സീനിയയിലെത്തി സമാധാന ജീവിതം നയിച്ചുവന്ന മുസ്‌ലിംകളില്‍ പലരും നബി (സ) മദീനയിലെത്തി അവിടെ ഇസ്‌ലാമിക രാഷ്ട്രം സ്ഥാപിച്ചുകഴിഞ്ഞ ശേഷവും അവിടെതന്നെ തങ്ങളുടെ ജീവിതം തുടര്‍ന്നുവെന്നും മദീനയിലേക്ക് വന്ന് ഇസ്‌ലാമിക രാഷ്ട്രത്തിന്റെ പ്രജയാകുവാന്‍ തിടുക്കം കാട്ടിയില്ലെന്നുമുള്ള വസ്തുതകള്‍ ശ്രദ്ധേയമാണ്. മദീനാ രാഷ്ട്രത്തിലേക്ക് അവരില്‍ ചിലര്‍ മാത്രമാണ് രാഷ്ട്ര നിര്‍മാണത്തിന്റെ ആദ്യഘട്ടത്തില്‍ മടങ്ങിയെത്തിയത്. മക്കയിലേക്ക് മടങ്ങിയെത്തിയത് മുപ്പത്തിമൂന്ന് പുരുഷന്‍മാരും എട്ട് സ്ത്രീകളുമായിരുന്നുവെന്നും അതില്‍ രണ്ടുപേര്‍ മക്കയില്‍വെച്ചുതന്നെ മരണപ്പെട്ടുവെന്നും ഏഴുപേരെ മക്കക്കാര്‍ ബന്ധനസ്ഥരാക്കിയെന്നും ഇരുപത്തിനാലുപേര്‍ ബദ്ര്‍ യുദ്ധത്തില്‍ പങ്കെടുത്തുവെന്നും ഇബ്‌നുസഅദ് രേഖപ്പെടുത്തുന്നുണ്ട്. അബ്സീനിയാഹിജ്‌റയിലെ നേതാവായിരുന്ന ജഅ്ഫര്‍ ബ്ന്‍ അബീത്വാലിബും ബാക്കിയുള്ളവരുമെല്ലാം മദീനയിലെത്തിയത് ഖൈബര്‍ ജയിച്ചടക്കിയതിന് ശേഷമാണെന്ന് ബുഖാരിയും മുസ്ലിമും നിവേദനം ചെയ്ത ഹദീഥിലുണ്ട്. അവര്‍ മദീനയിലെത്തിയപ്പോള്‍ പ്രവാചകൻ (സ) ഒട്ടകത്തിന്മേല്‍നിന്ന് ചാടിയിറങ്ങുകയും അദ്ദേഹത്തിന്റെ പിതൃവ്യപുത്രനായ ജഅ്ഫര്‍ ബിന്‍ അബീത്വാലിബിനെ ആശ്ലേഷിക്കുകയും ചെയ്തുകൊണ്ട് പറഞ്ഞതിങ്ങനെയാണ്: 'ഖൈബര്‍ വിജയമാണോ ജഅ്ഫറിന്റെ തിരിച്ചുവരവാണോ എന്നെ കൂടുതല്‍ സന്തോഷിപ്പിച്ചതെന്ന് എനിക്കറിയില്ല'.( ഇമാം ഇബ്‌നുല്‍ ഖയ്യിം അല്‍ ജൗസിയ്യയുടെ സാദുല്‍മആദില്‍ നിന്ന്)

മദീനാ രാഷ്ട്രത്തിന്റെ നിര്‍മാണം കഴിഞ്ഞ് നീണ്ട ഏഴുവര്‍ഷങ്ങള്‍ക്ക് ശേഷമാണ് ജഅ്ഫര്‍ ബ്ന്‍ അബീതാലിബിന്റെ നേതൃത്വത്തിലുള്ള മുസ്‌ലിം സംഘം അബ്‌സീനിയയിലേക്ക് മടങ്ങിയതെന്ന വസ്തുതയും അങ്ങനെ മടങ്ങിയെത്തിയപ്പോള്‍ പൂര്‍ണമനസ്സോടെ പ്രവാചകൻ (സ) അവരെ സ്വീകരിക്കുകയും അവര്‍ക്ക് രണ്ട് ഹിജ്‌റയുടെ പ്രതിഫലമുണ്ടെന്ന സന്തോഷവാര്‍ത്ത അറിയിക്കുകയുമാണ് ചെയ്തതെന്ന യാഥാര്‍ഥ്യവും പഠിപ്പിക്കുന്നത് ഇസ്‌ലാമികേതര രാഷ്ട്ര സംവിധാനത്തിലാണെങ്കിലും മുസ്‌ലിമായി ജീവിക്കുവാന് സ്വാതന്ത്ര്യവും അവസരവുമുണ്ടെങ്കില്‍ ജീവിക്കുന്നതിന് വിരോധമില്ലെന്നും അങ്ങനെ ജീവിക്കുന്നവരുടെ മതത്തെ പ്രസ്തുത ജീവിതം ഒരുതരത്തിലും പ്രതികൂലമായി ബാധിക്കുകയില്ലെന്നുംതന്നെയാണ്. അബ്‌സീനിയയില്‍വെച്ച് മരണപ്പെട്ട നിരവധി മുസ്‌ലിംകളുടെ പട്ടിക ഇബ്‌നു ഇസ്ഹാഖ് രേഖപ്പെടുത്തുന്നുണ്ട്.

ഇസ്‌ലാമിക രാഷ്ട്രമുണ്ടായിരിക്കെതന്നെ ഒരു ഇസ്‌ലാമികേതര രാഷ്ട്രത്തില്‍ ജീവിച്ച് മരണപ്പെട്ടതിനാല്‍ അവരുടെ മതത്തിന് എന്തെങ്കിലും ഗ്ലാനി സംഭവിച്ചുവെന്നോ അവരുടെ മതം അപൂര്‍ണമായിരുന്നുവെന്നോ പ്രവാചകൻ (സ) സൂചിപ്പിച്ചതായി രേഖകളൊന്നുംതന്നെയില്ല. പ്രവാചകന്റെ ഹിജ്‌റയെക്കുറിച്ചറിഞ്ഞപ്പോള്‍ യമനിലായിരുന്ന അബൂമൂസല്‍ അശ്അരിയും (റ)അമ്പതിലധികം പേരുമടങ്ങുന്ന സംഘം മദീനയിലെത്തണമെന്ന ആഗ്രഹത്തോടെ യാത്രതിരിക്കുകയും വഴിമധ്യെ അവര്‍ അബ്‌സീനിയയിലെത്തുകയും ജഅ്ഫര്‍ ബ്ന്‍ അബീത്വാലിബിനെ (റ)കണ്ടുമുട്ടുകയും അദ്ദേഹത്തിന്റെ നിര്‍ദേശപ്രകാരം അവര്‍ അവിടെ ഏറെക്കാലം തങ്ങുകയും അവര്‍ ഒന്നിച്ച് മദീനയിലേക്ക് യാത്രയാവുകയും ഖൈബര്‍ വിജയ സന്ദര്‍ഭത്തില്‍ പ്രവാചകനുമായി സന്ധിക്കുകയും ചെയ്തതായി വ്യക്തമാക്കുന്ന ഹദീഥ് സ്വഹീഹുല്‍ ബുഖാരിയിലുണ്ട്. ഇസ്‌ലാമിക രാഷ്ട്രമായ മദീനയിലേക്ക് പോകാതെ ഇസ്‌ലാമികേതര രാഷ്ട്രമായ അബ്‌സീനിയയില്‍ തങ്ങുവാനുള്ള കാരണമായി ജഅ്ഫര്‍ ബിന്‍ അബീത്വാലിബ് പറഞ്ഞത് ഇങ്ങനെയാണ്: 'ഇവിടേക്ക് ഞങ്ങളെ പറഞ്ഞയച്ചതും ഇവിടെ താമസിക്കുവാന്‍ ആവശ്യപ്പെട്ടതും അല്ലാഹുവിന്റെ ദൂതനാണ്; അതിനാല്‍ നിങ്ങളും ഞങ്ങളോടൊപ്പം താമസിക്കുക'.( സ്വഹീഹുല്‍ ബുഖാരി, കിതാബുല്‍മഗാസി)  ഇസ്‌ലാമിക രാഷ്ട്രത്തില്‍ മാത്രമെ മുസ്‌ലിമിന് പൂര്‍ണ മുസ്‌ലിമായി ജീവിക്കുവാനാകൂവെന്നതിനാല്‍ ലോകത്ത് എവിടെ ജീവിക്കുന്ന മുസ്‌ലിമും ആ പ്രദേശം ഇസ്‌ലാമിക രാഷ്ട്രമാക്കുവാനോ അതല്ലെങ്കില്‍ നിലവിലുള്ള ഏതെങ്കിലും ഇസ്‌ലാമിക രാഷ്ട്രത്തിന് കീഴിലാക്കുകയോ ശ്രമിച്ചുകൊണ്ടിരിക്കുമെന്നും അതുകൊണ്ടാണ് മുസ്‌ലിംകള്‍ കലാപകാരികളായിത്തീരുന്നതെന്നും വിമര്‍ശിക്കുന്നവര്‍ കാണാതെ പോകുന്ന ചരിത്ര സംഭവങ്ങളാണിവ. രാഷ്ട്രം ഇസ്‌ലാമികമാണോ അല്ലേയെന്നതല്ല, മുസ്‌ലിമായി ജീവിക്കുവാന്‍ അനുവദിക്കുന്നതാണോ അല്ലേയെന്നതാണ് ഒരു രാഷ്ട്ര സംവിധാനത്തോടുള്ള മുസ്‌ലിമിന്റെ സമീപനമെന്തായിരിക്കണമെന്ന് നിശ്ചയിക്കുന്ന സൂചികയെന്ന് ഈ സംഭവങ്ങള്‍ സുതരാം വ്യക്തമാക്കുന്നു.

ല്ല. മുസ്‌ലിമായി ജീവിക്കുവാൻ സ്വാതന്ത്ര്യം ലഭിക്കുന്നതിന് വേണ്ടിയുള്ള പലായനമാണത്. നബി ശിഷ്യന്മാരുടെ അബ്സീനിയ പലായനം ഇക്കാര്യം വ്യക്തമാക്കുന്നുണ്ട്.

പ്രവാചകത്വത്തിന്റെ ആറാം വര്‍ഷത്തിലെ റജബ് മാസത്തില്‍ പതിനൊന്ന് പുരുഷന്‍മാരും നാല് സ്ത്രീകളുമടങ്ങുന്ന മുസ്‌ലിംകളുടെ ആദ്യസംഘം അഭയാര്‍ഥികളായി അബ്‌സീനിയയിലേക്ക് പലായനം ചെയ്യുന്നത് മുതല്‍ ആരംഭിക്കുന്നു ഇസ്‌ലാമിലെ ഹിജ്‌റയുടെ ചരിത്രം. മക്കയിലെ പീഡനങ്ങളില്‍നിന്ന് രക്ഷപ്പെടുന്നതിനായാണ് അവര്‍ അബ്‌സീനിയയിലേക്ക് പോയത്. ഏറെ ക്രൂരമായി പീഡിപ്പിക്കപ്പെട്ടിരുന്ന അടിമകള്‍ക്ക് യജമാനന്‍മാരുടെ കണ്ണുവെട്ടിച്ച് ഓടിപ്പോവുക പ്രയാസകരമായതുകൊണ്ടാവാം ആദ്യത്തെ പലായന സംഘത്തിലുണ്ടായിരുന്നത് പ്രധാനമായും ഖുറൈശീ ഗോത്രത്തില്‍നിന്ന് ഇസ്‌ലാം സ്വീകരിച്ചവരും അവരുടെ ഭാര്യമാരുമായിരുന്നു. തന്റെ മകളായ റുഖിയ്യ (റ) യെയും ഭര്‍ത്താവായിരുന്ന ഉഥ്മാനു ബ്‌നു അഫ്ഫാനെ(റ)യുമാണ് പ്രവാചകൻ (സ) ആദ്യമായി പലായനത്തിന് പറഞ്ഞയച്ചത്. ആദര്‍ശജീവിതത്തിലേക്ക്, സ്വന്തമായതെല്ലാം ഉപേക്ഷിച്ചുകൊണ്ട് പലായനം ചെയ്യാന്‍ പ്രവാചകൻ (സ) മുന്നില്‍ നടത്തിയത് സ്വന്തം പുത്രിയെയും പുത്രി ഭര്‍ത്താവിനെയുമായിരുന്നു.

ഉഥ്മാനുബ്‌നു മദ്വ്ഊനിന്റെ(റ) നേതൃത്വത്തില്‍ അബ്‌സീനിയയിലേക്ക് പലായനം ചെയ്ത ആദ്യസംഘം ഏറെ താമസിയാതെ ആ വര്‍ഷം ശവ്വാലില്‍തന്നെ മക്കയിലേക്ക് തിരിച്ചുവന്നത് തങ്ങളുടെ നാട്ടില്‍തന്നെ മുസ്‌ലിമായി ജീവിക്കുവാനുള്ള സാഹചര്യമുണ്ടായിട്ടുണ്ടെന്ന് തെറ്റായി മനസ്സിലാക്കിയതിനാലായിരുന്നു. ഉമറി (റ) ന്റെയും ഹംസ (റ) യുടെയും ഇസ്‌ലാം ആശ്ലേഷണത്തെക്കുറിച്ചറിഞ്ഞ അവര്‍ അതുവഴി മക്കയില്‍ സ്വസ്ഥമായി ആദര്‍ശജീവിതം നയിക്കുവാനുള്ള സ്വാതന്ത്ര്യമുണ്ടായിട്ടുണ്ടെന്ന് തെറ്റിദ്ധരിക്കുകയായിരുന്നു. ദൈവികബോധന പ്രകാരം ജീവിക്കുകയും അത് പ്രബോധിപ്പിക്കുകയും ചെയ്യാനുള്ള സ്വാതന്ത്ര്യം ജന്മനാട്ടില്‍ ഒരുങ്ങിയിട്ടുണ്ടെന്ന് കരുതി മക്കയില്‍ തിരിച്ചെത്തിയ അവരെ സ്വാഗതം ചെയ്തത് മുസ്‌ലിംകള്‍ക്കെതിരെ ബഹുദൈവാരാധകര്‍ അഴിച്ചുവിട്ടുകൊണ്ടിരിക്കുന്ന ക്രൂരമായ പീഡനങ്ങളെക്കുറിച്ച വര്‍ധമാനമായ വാര്‍ത്തകളായിരുന്നു. തങ്ങള്‍ സത്യമെന്ന് മനസ്സിലാക്കിയ ആദര്‍ശമുള്‍ക്കൊണ്ട് ജീവിക്കുവാന്‍ സ്വദേശത്ത് സാഹചര്യമൊരുങ്ങിയിട്ടുണ്ടെന്ന് മനസ്സിലാക്കിയ ഉടനെ സന്തോഷത്തോടെ ജന്മനാട്ടിലേക്ക് തിരിച്ച അബ്‌സീനിയര്‍ മുഹാജിറുകളുടെ നടപടിയില്‍നിന്ന് അവരുടെ പലായനത്തിന്റെ ലക്ഷ്യമെന്തായിരുന്നുവെന്ന്  വ്യക്തമാവുന്നുണ്ട്. ആദര്‍ശ ജീവിതത്തിനുള്ള സ്വാതന്ത്ര്യം നേടിയെടുക്കുകയെന്നതില്‍ കവിഞ്ഞ ലക്ഷ്യങ്ങളൊന്നും തന്നെ അവര്‍ക്കുണ്ടായിരുന്നില്ലെന്ന് സ്പഷ്ടം. ഇതുതന്നെയായിരുന്നു പിന്നീട് നടന്ന പലായനങ്ങളുടെയും ലക്ഷ്യം.

തങ്ങൾ കരുതിയത് പോലെ മക്കയിൽ മതസ്വാതന്ത്ര്യമുള്ള അവസ്ഥ സംജാതമായിട്ടില്ലെന്ന് മനസ്സിലാക്കിയപ്പോഴാണ് അബ്‌സീനിയയില്‍നിന്ന് തിരിച്ചെത്തിയവര്‍ വീണ്ടും അവിടേക്ക് തന്നെ പലായനം ചെയ്തത്. എണ്‍പതിലധികം പുരുഷന്‍മാരും പത്തൊന്‍പത് സ്ത്രീകളുമടങ്ങുന്ന താരതമ്യേന വലിയ പലായനസംഘത്തെ ക്രൈസ്തവ ഭരണാധികാരിയായിരുന്ന നജ്ജാശിയുടെ നാട്ടിലേക്ക് നബി (സ) പറഞ്ഞയച്ചതും സ്രഷ്ടാവിനെ മാത്രം ആരാധിച്ച് ജീവിക്കുകയും അവന്റെ നിയമനിര്‍ദേശങ്ങള്‍ അനുസരിക്കുകയും ചെയ്യുന്നതിനുള്ള സ്വാതന്ത്ര്യത്തിനുവേണ്ടിയായിരുന്നു. അബ്‌സീനിയയിലേക്ക് പലായനം ചെയ്യാന്‍ പ്രചോദിപ്പിച്ചുകൊണ്ടുള്ള പ്രവാചക നിര്‍ദേശത്തില്‍നിന്ന്  ഇക്കാര്യം സുതരാം വ്യക്തമാകുന്നുണ്ട്. മക്കാ മുശ്‌രിക്കുകളുടെ അക്രമങ്ങളാല്‍ പൊറുതിമുട്ടിയിരുന്ന തന്റെ അനുചരന്‍മാരോട് പ്രവാചകൻ (സ) പറഞ്ഞതായി, അബ്‌സീനിയന്‍ പലായനത്തില്‍ പങ്കെടുത്ത, പിന്നീട് പ്രവാചക പത്‌നിയായിത്തീര്‍ന്ന ഉമ്മുസല്‍മ (റ)നിവേദനം ചെയ്യുന്നു: 'ആരെയും അക്രമിക്കുകയോ അടിച്ചമര്‍ത്തുകയോ ചെയ്യാത്ത ഒരു രാജാവാണ് അബ്‌സീനിയന്‍ പ്രദേശങ്ങള്‍ ഭരിക്കുന്നത്. അതിനാല്‍ നിങ്ങള്‍ അങ്ങോട്ട് പോവുകയും അദ്ദേഹത്തിന്റെ പ്രദേശങ്ങളില്‍ അഭയം തേടുകയും ചെയ്യുക; ഇപ്പോഴുള്ള അവസ്ഥയ്ക്ക് അല്ലാഹു ഒരു മാറ്റമുണ്ടാക്കുന്നതും നമുക്ക് മറ്റൊരു മാര്‍ഗമുണ്ടാവുന്നതുംവരെ'.( ഇബ്‌നു ഹിശാം സ്വീകാര്യമായ പരമ്പരയോടു (ഹസന്‍) നിവേദനം ചെയ്തത്)

മക്കയ്ക്ക് ചുറ്റും നിരവധി സ്ഥലങ്ങളുണ്ടായിട്ടും തന്റെ അനുചരന്‍മാരെ എന്തുകൊണ്ടാണ് പ്രവാചകൻ (സ) അവിടങ്ങളിലേക്കൊന്നും പറഞ്ഞയക്കാതെ അബ്‌സീനിയതന്നെ തെരഞ്ഞെടുത്തതെന്ന ചോദ്യത്തിനുള്ള ഉത്തരം ഉമ്മുസല്‍മ (റ)നിവേദനം ചെയ്ത ഹദീഥിലുണ്ട്. 'ആരെയും അക്രമിക്കുകയോ അടിച്ചമര്‍ത്തുകയോ ചെയ്യാത്ത ഒരു രാജാവാണ് അബ്‌സീനിയ ഭരിക്കുന്നത്'. അബ്‌സീനിയയിലേക്ക് കച്ചവടത്തിന് പോയ മക്കക്കാരില്‍നിന്ന് അവിടുത്തെ രാജാവിന്റെ നീതിനിഷ്ഠയെയും മാന്യതയെയുംകുറിച്ച് മനസ്സിലാക്കിയതില്‍നിന്നാവണം അദ്ദേഹം മുസ്‌ലിംകളെ സ്വീകരിക്കുകയും അവരുടെ മതമനുസരിച്ച് ജീവിക്കാന്‍ അനുവദിക്കുകയും ചെയ്യുമെന്ന് പ്രവാചകൻ (സ) കരുതിയത്. അദ്ദേഹത്തിന്റെ നിഗമനം തെറ്റിയിട്ടില്ലെന്ന് അബ്‌സീനിയയില്‍ മുസ്‌ലിംകള്‍ക്ക് ലഭിച്ച സ്വീകരണം തെളിയിക്കുകയും ചെയ്തു. 'ഞങ്ങള്‍ക്ക് അവിടെ ചെന്നപ്പോള്‍ നല്ല സ്വീകരണം ലഭിക്കുകയും നല്ല അയല്‍ക്കാരെ കിട്ടുകയും ചെയ്തു. ഞങ്ങളുടെ മതത്തിന്റെ കാര്യത്തില്‍ ശാന്തി അനുഭവിക്കുകയും പീഡനങ്ങളില്ലാത്ത സ്ഥിതി സംജാതമാവുകയും ചെയ്തു'വെന്ന പ്രവാചക പത്‌നി ഉമ്മുസല്‍മേയുടെ വാക്കുകള്‍ മുസ്‌ലിംകള്‍ക്ക് അബ്‌സീനിയയില്‍ ലഭിച്ച സ്വീകാര്യതയെ വെളിപ്പെടുത്തുന്നുണ്ട്.

പീഡനങ്ങളും പ്രയാസങ്ങളും അധികരിച്ചപ്പോള്‍ അവയില്‍നിന്ന് രക്ഷപ്പെടാന്‍ പ്രവാചകൻ (സ) നിര്‍ദേശിച്ചത് ജീവിക്കുന്ന നാട്ടില്‍ കലാപങ്ങളും കുഴപ്പങ്ങളുമുണ്ടാക്കി, അവിടെയുള്ളവരെ ഭീഷണിപ്പെടുത്തിക്കൊണ്ട് കാര്യം നേടുകയെന്ന മാര്‍ഗമല്ല, പ്രത്യുത ആദര്‍ശമനുസരിച്ച് ജീവിക്കുവാനാകുന്ന ഏതെങ്കിലുമൊരു സ്ഥലമുണ്ടെങ്കില്‍ അവിടെ പോയി ജീവിക്കുകയും ആശയപ്രബോധനം നടത്തുകയും ചെയ്യുകയാണെന്ന വസ്തുത വ്യക്തമാക്കുന്നതാണ് അബ്‌സീനിയയിലേക്ക് പലായനം ചെയ്യുവാനുള്ള നബിനിര്‍ദേശങ്ങള്‍. എണ്‍പതിലധികം പുരുഷന്‍മാരും അവര്‍ക്ക് നേതൃത്വം നല്‍കാനാവുന്ന ഉമറിനെയും (റ)ഹംസയെയും (റ)പോലെയുള്ള ശക്തരും ധൈര്യശാലികളുമായ പ്രമുഖരും ആദര്‍ശത്തിനായി എന്തുതരം ത്യാഗങ്ങളും സഹിക്കാനാകുമെന്ന് തെളിയിച്ച ബിലാലിനെ (റ) പോലെയുള്ള മഹത്തുക്കളും തനിക്ക് ചുറ്റുമുണ്ടായിരുന്നിട്ടും മക്കയില്‍ തന്നെയും അനുയായികളെയും പീഡിപ്പിക്കുന്നവര്‍ക്കെതിരെ ശക്തമായ ഒരു കലാപമഴിച്ചുവിട്ട് പീഡനത്തിന്റെ ചങ്ങലയറുക്കുവാനല്ല, പ്രത്യുത പീഡനങ്ങളില്ലാത്ത അയല്‍ ദേശത്തേക്ക് കുടിയേറുവാനാണ് പ്രവാചകൻ (സ) നിര്‍ദേശിച്ചതെന്ന വസ്തുത കലാപകാരിയായിരുന്നു മുഹമ്മദെന്നും (സ) മക്കയില്‍വെച്ച് അവസരമില്ലാത്തതുകൊണ്ടാണ് അദ്ദേഹം കലാപങ്ങളുടെ കെട്ടഴിക്കാതിരുന്നതെന്നും വിമര്‍ശിക്കുന്നവര്‍ക്ക് മറുപടി നല്‍കുന്നതാണ്.

കലാപമുണ്ടാക്കി കലുഷിതമായ സാമൂഹ്യാന്തരീക്ഷം സൃഷ്ടിക്കുകയും അതില്‍നിന്ന് തന്റെ താല്‍പര്യങ്ങള്‍ സംരക്ഷിക്കുകയുമല്ല, പ്രത്യുത സമാധാനത്തോടെ ദൈവികമതമനുസരിച്ച് ജീവിക്കുകയും അത് പ്രബോധിപ്പിക്കുകയും ചെയ്യാനാവശ്യമായ സാഹചര്യം സൃഷ്ടിക്കുകയുമായിരുന്നു പ്രവാചകന്റെ ലക്ഷ്യം. മക്കയിലെ നിലവിലുള്ള സാമൂഹ്യ സാഹചര്യത്തില്‍ ഗോത്രങ്ങളുടെ സംരക്ഷണത്തിന്റെ തണലില്‍ താനും അത് ലഭിച്ച സ്വഹാബികളും നിലയുറപ്പിക്കുകയും ആദര്‍ശജീവിതവും പ്രബോധനവും നിര്‍വഹിച്ചുകൊണ്ട് മുന്നോട്ട് പോവുകയും ചെയ്തുകൊണ്ടിരുന്നപ്പോള്‍തന്നെ, പ്രസ്തുത സംരക്ഷണം ലഭിക്കാത്തവരെ മതപീഡനമില്ലാത്ത അയല്‍നാട്ടിലേക്ക് പോകാന്‍ പ്രചോദിപ്പിക്കുകയും അവിടേക്ക് പറഞ്ഞയക്കുകയും ചെയ്ത പ്രവാചകനില്‍ (സ) ശാന്തമായി വിശുദ്ധജീവിതം നയിക്കുകയും അത് പ്രബോധിപ്പിക്കുകയും ചെയ്യാന്‍ അഭിവാഞ്ചയുള്ള ഒരു സാമൂഹ്യ പരിഷ്‌കര്‍ത്താവിനെയാണ് ആര്‍ക്കും കാണാനാവുക.

വിടവാങ്ങുക, ഉപേക്ഷിക്കുക, കയ്യൊഴിയുക, പരിത്യജിക്കുക, വേര്‍പെടുക, നിരാകരിക്കുക, സ്ഥലം വിടുക തുടങ്ങിയ അര്‍ഥങ്ങളില്‍ പ്രയോഗിക്കപ്പെടുന്ന ഹാ, ജീം, റാ എന്നീ അക്ഷരത്രയങ്ങളില്‍നിന്ന് നിഷ്പന്നമായ 'ഹിജ്‌റ'യെന്ന പദമാണ് പലായനം എന്ന് പരിഭാഷപ്പെടുത്തപ്പെടാറുള്ളത്. സ്വദേശം വെടിഞ്ഞുപോകുന്നതിനും തെറ്റുകള്‍ വെടിഞ്ഞ് വിശുദ്ധമാകുന്നതിനും പൈശാചിക പാതവെടിഞ്ഞ് സത്യമാര്‍ഗത്തിലെത്തിച്ചേരുന്നതിനുമെല്ലാം 'ഹിജ്‌റ'യെന്ന് പറയാവുന്നതാണ്. 'തെറ്റുകളും പാപങ്ങളും ഉപേക്ഷിക്കുന്നവനാണ് മുഹാജിര്‍' (പലായനം ചെയ്യുന്നവന്‍)( ഇമാമുമാര്‍ അഹ്മദ്, ഹാക്കിം, ത്വബ്‌റാനി എന്നിവര്‍ നിവേദനം ചെയ്ത സ്വഹീഹായ ഹദീഥ് ) എന്ന് പ്രവാചകൻ (സ) പറഞ്ഞത് ഈ അര്‍ഥങ്ങളെയെല്ലാം ദ്യോതിപ്പിച്ചുകൊണ്ടാണ്. പാപങ്ങള്‍ വെടിയുകയും നന്മകളില്‍ വ്യാപൃതരാവുകയും ചെയ്യുകയെന്ന ഹിജ്‌റ ചെയ്യേണ്ടവനാണ് ഓരോ മുസ്‌ലിമും. എലാ മുസ്ലിംകളും ആത്യന്തികമായി മുഹാജിറുകളാണെന്ന് സാരം.

പൈശാചികപാത വെടിഞ്ഞ് സത്യമാര്‍ഗത്തിലെത്തിച്ചേര്‍ന്ന ഒരാള്‍ക്ക് തന്റെ സത്യമെന്ന് തിരിച്ചറിഞ്ഞ ആദര്‍ശപ്രകാരം ജീവിക്കാൻ കഴിയാത്ത സാഹചര്യമുണ്ടാകുമ്പോഴാണ് അയാള്‍ സ്വന്തം നാട്ടില്‍ നിന്ന് പലായനം ചെയ്യുന്നത്. മാനസികമായ ഹിജ്‌റയില്‍നിന്നാണ് നാട് വെടിയേണ്ടിവരികയെന്ന ശാരീരികമായ ഹിജ്‌റയുണ്ടാവുന്നത്. പൈശാചിക പ്രലോഭനങ്ങളില്‍നിന്ന് രക്ഷപ്പെടുകയെന്ന മാനസികമായ ഹിജ്‌റ ചെയ്യാത്തവര്‍ ശാരീരിക ഹിജ്‌റ ചെയ്യുന്നതുകൊണ്ട് കാര്യമൊന്നുമില്ലെന്ന് പ്രവാചകൻ (സ) പഠിപ്പിച്ചിട്ടുണ്ട്. "പ്രവര്‍ത്തനങ്ങള്‍ ഉദ്ദേശ്യങ്ങള്‍ക്കനുസരിച്ചാണ് (പ്രതിഫലാര്‍ഹമാവുക) ഓരോ മനുഷ്യനും അവനിദ്ദേശിച്ചത് ലഭിക്കും. ഭൗതികനേട്ടത്തിനോ ഏതെങ്കിലും സ്ത്രീയെ വിവാഹം ചെയ്യാനോ ആണ് ആരുടെയെങ്കിലും ഹിജ്‌റയെങ്കില്‍ അവനതാണ് ലഭിക്കുക.( സ്വഹീഹുല്‍ ബുഖാരി, കിതാബുബദഉല്‍ വഹ്‌യ്; സ്വഹീഹുമുസ്‌ലിം, കിതാബുല്‍ഇമാറ)

സത്യമതത്തിന്റെ നിര്‍ദേശങ്ങളനുസരിച്ച് ജീവിക്കുന്നതിനും പ്രബോധനം ചെയ്യുന്നതിനും വേണ്ടി സ്വദേശം വെടിഞ്ഞുപോകുന്നത് വളരെ വലിയ ത്യാഗമാണ്. ജനിച്ചുവളര്‍ന്ന നാടും പിച്ചവെച്ചുവളര്‍ന്ന മണ്ണും താലോലിച്ച് വളര്‍ത്തിയ ബന്ധുമിത്രാദികളെയും അധ്വാനിച്ചുണ്ടാക്കിയ സമ്പാദ്യവുമെല്ലാം വെടിഞ്ഞ് ആദര്‍ശജീവിതത്തിനുള്ള സ്വാതന്ത്ര്യത്തിനുവേണ്ടി മാത്രമായുള്ള പലായനമെന്ന അല്ലാഹുവിന്റെ മാര്‍ഗത്തിലുള്ള ത്യാഗത്തിന് ഉന്നതമായ പ്രതിഫലമുണ്ടെന്ന് ക്വുര്‍ആന്‍ വ്യക്തമാക്കുന്നുണ്ട്.

''വിശ്വസിക്കുകയും സ്വദേശം വെടിയുകയും തങ്ങളുടെ സ്വത്തുക്കളും ശരീരങ്ങളും കൊണ്ട് അല്ലാഹുവിന്റെ മാര്‍ഗത്തില്‍ സമരം നടത്തുകയും ചെയ്തവര്‍ അല്ലാഹുവിങ്കല്‍ ഏറ്റവും മഹത്തായ പദവിയുള്ളവരാണ്. അവര്‍ തന്നെയാണ് വിജയം പ്രാപിച്ചവര്‍. അവര്‍ക്ക് അവരുടെ രക്ഷിതാവ് അവന്റെ പക്കല്‍ നിന്നുള്ള കാരുണ്യത്തെയും പ്രീതിയെയും സ്വര്‍ഗത്തോപ്പുകളെയും പറ്റി സന്തോഷവാര്‍ത്ത അറിയിക്കുന്നു. അവര്‍ക്ക് അവിടെ ശാശ്വതമായ സുഖാനുഭവമാണുള്ളത്. അവരതില്‍ നിത്യവാസികളായിരിക്കും. തീര്‍ച്ചയായും അല്ലാഹുവിന്റെ അടുക്കലാണ് മഹത്തായ പ്രതിഫലമുള്ളത്.'' (9:20-22)

''അല്ലാഹുവിന്റെ മാര്‍ഗത്തില്‍ വല്ലവനും സ്വദേശം വെടിഞ്ഞ് പോകുന്ന പക്ഷം ഭൂമിയില്‍ ധാരാളം അഭയസ്ഥാനങ്ങളും ജീവിതവിശാലതയും അവന്‍ കണ്ടെത്തുന്നതാണ്. വല്ലവനും തന്റെ വീട്ടില്‍ നിന്ന് - സ്വദേശം വെടിഞ്ഞ് കൊണ്ട് - അല്ലാഹുവിലേക്കും അവന്റെ ദൂതനിലേക്കും ഇറങ്ങി പുറപ്പെടുകയും, അനന്തരം (വഴി മധ്യേ) മരണമവനെ പിടികൂടുകയും ചെയ്യുന്ന പക്ഷം അവന്നുള്ള പ്രതിഫലം അല്ലാഹുവിങ്കല്‍ സ്ഥിരപ്പെട്ടു കഴിഞ്ഞു. അല്ലാഹു ഏറെ പൊറുക്കുന്നവനും കരുണാനിധിയുമാകുന്നു.'' (4:100)

സ്വര്‍ഗപ്രാപ്തിയെന്ന ലക്ഷ്യവും ആദര്‍ശജീവിതം നയിക്കുവാന്‍ അവസരമുണ്ടാകണമെന്ന ആഗ്രഹവും മാത്രമാണ് മുസ്‌ലിമിന്റെ പലായനത്തിന് പിന്നിലുള്ളത്. സ്വാര്‍ഥമായ താല്‍പര്യങ്ങള്‍ക്കോ ഭൗതികമായ ലക്ഷ്യങ്ങള്‍ക്കോ വേണ്ടിയുള്ള പലായനം ചിലപ്പോള്‍ പ്രസ്തുത ലക്ഷ്യങ്ങള്‍ നേടിയെടുക്കുവാന്‍ പ്രയോജനീഭവിക്കാമെങ്കിലും അത് അല്ലാഹുവിന്റെ മാര്‍ഗത്തിലുള്ള ഹിജ്‌റയാവുകയില്ലെന്നും ആത്യന്തികമായി അത് നന്മയ്ക്ക് നിമിത്തമാകുകയില്ലെന്നുമാണ് പ്രവാചകൻ (സ) പഠിപ്പിച്ചത്. ഭൗതികമായ ഒരു സാമ്രാജ്യ സ്ഥാപനമായിരുന്നു മുഹമ്മദ് നബി (സ) യുടെ ഹിജ്‌റക്കുപിന്നിലുള്ള ലക്ഷ്യമെന്ന് വിമര്‍ശിക്കുന്നവര്‍ നബി ദൗത്യത്തെക്കുറിച്ച് വിവരമില്ലാത്തവരാണ്. പ്രവാചകത്വം വാദിച്ചതുതന്നെ അറബികള്‍ക്കുമേലുള്ള അധീശത്വത്തിന് വേണ്ടിയായിരുന്നുവെന്ന് വാദിക്കുന്നവരാണ് മുഹമ്മദ് നബി (സ) യുടെ ഹിജ്‌റക്കുപിന്നിലും സാമ്രാജ്യസ്ഥാപനമെന്ന ലക്ഷ്യമുണ്ടായിരുന്നുവെന്ന് സമര്‍ഥിക്കാന്‍ ശ്രമിക്കുന്നത്. ദൈവികബോധനങ്ങള്‍ പ്രകാരമുള്ള ജീവിതത്തിനും അത് പ്രബോധനം ചെയ്യുന്നതിനുമുള്ള സ്വാതന്ത്ര്യമുണ്ടാവുകയെന്നതില്‍ കവിഞ്ഞ ലക്ഷ്യങ്ങളെന്തെങ്കിലും ഹിജ്‌റക്ക് പിന്നിലുണ്ടായിരുന്നുവെന്ന് സ്ഥാപിക്കുവാന്‍ വിമര്‍ശകരുടെ പക്കല്‍ തെളിവുകളൊന്നും തന്നെയില്ല. ഹിജ്‌റക്ക് മുമ്പുള്ള സംഭവങ്ങളെ സൂക്ഷ്മമായി അപഗ്രഥിക്കുകയും തലനാരിഴ കീറി പരിശോധിക്കുകയും ചെയ്താല്‍പോലും നബി (സ)യുടെ പലായനത്തിന് പിന്നില്‍ സ്വാര്‍ഥമായ വല്ല ലക്ഷ്യവുമുണ്ടായിരുന്നുവെന്ന് സ്ഥാപിക്കുവാനാവശ്യമായ തെളിവുകളൊന്നും ലഭിക്കുകയില്ലെന്നുറപ്പാണ്.

 

ഖുര്‍ആനിലെ പത്തൊന്‍പതാം അധ്യായമായ സൂറത്തുമറിയം തുടങ്ങുന്നതുതന്നെ സകരിയ്യാ (അ)  യുടെ വൃത്താന്തവുമായിക്കൊണ്ടാണ്. വാര്‍ധക്യകാലത്ത് വന്ധ്യയായ ഭാര്യയോടൊപ്പം ജീവിക്കുന്ന സകരിയ്യാ (അ) യുടെ ഒരു അനന്തരാവകാശിക്കുവേണ്ടിയുള്ള പ്രാര്‍ത്ഥനയും പ്രസ്തുത പ്രാര്‍ത്ഥനക്കുള്ള ഉത്തരമായി ഒരു ആണ്‍കുഞ്ഞുണ്ടായ കഥയുമെല്ലാം ഖുര്‍ആന്‍ വിവരിക്കുന്നുണ്ട്. സകരിയ്യായുടെ പ്രാര്‍ത്ഥനയ്ക്കുള്ള ഉത്തരമെന്നോണം സര്‍വ്വശക്തന്‍ അദ്ദേഹത്തിന് നല്‍കിയ വാഗ്ദാനം ഏഴാം വചനത്തില്‍ പറയുന്നുണ്ട്. അതിന്റെ സാരം ഇങ്ങനെയാണ്:

''ഹേ സക്കരിയാ, തീര്‍ച്ചയായും നിനക്ക് നാം ഒരു ആണ്‍കുട്ടിയെപറ്റി സന്തോഷവാര്‍ത്ത അറിയിക്കുന്നു. അവന്റെ പേര്‍ യഹ്‌യാ എന്നാകുന്നു. മുമ്പ് നാം ആരെയും അവന്റെ പേരുള്ളവരാക്കിയിട്ടില്ല'' (വി.ഖു.19:7)

ഈ വചനത്തില്‍ ''മുമ്പ് നാം ആരെയും അവന്റെ പേരുള്ളവരാക്കിയിട്ടില്ല''യെന്ന് യഹ്‌യാ (അ) യെക്കുറിച്ച് പറഞ്ഞത് ചരിത്രപരമായി അബദ്ധമാണെന്നാണ് ആരോപണം. 'യോഹന്നാന്‍ സ്‌നാപക'ന് അറബിയില്‍ പറയുന്ന പേരാണ് യഹ്‌യായെന്ന് പല ഖുര്‍ആന്‍ വ്യാഖ്യാതാക്കളും വ്യക്തമാക്കിയിട്ടുണ്ട്. ഇംഗ്ലീഷില്‍ പുറത്തിറങ്ങിയ ചില ഖുര്‍ആന്‍ പരിഭാഷാ ഗ്രന്ഥങ്ങളില്‍ യഹ്‌യായെന്നതിന് പകരമായി ജോണ്‍ (John) എന്നെഴുതുകയും മറ്റുചിലവയില്‍ യഹ്‌യായെന്നെഴുതി ജോണ്‍ എന്ന് ബ്രാക്കറ്റില്‍ ചേര്‍ക്കുകയും ചെയ്തിട്ടുണ്ട്. അപ്പോള്‍ യോഹന്നാന്‍ സ്‌നാപകനുമുമ്പ് യോഹന്നാന്‍ എന്നപേരുള്ളവരായി ആരുംതന്നെ ജീവിച്ചിരുന്നില്ല എന്നാണ് ഈ സൂക്തത്തില്‍ പറഞ്ഞതെന്ന് വരുന്നു. പഴയ നിയമത്തില്‍തന്നെ ഇരുപത്തിയേഴ് പ്രാവശ്യം യോഹന്നാന്‍ എന്ന നാമം പ്രയോഗിക്കപ്പെട്ടിട്ടുണ്ട്. പഴയ നിയമത്തിലെ യോഹന്നന്‍മാരൊന്നും തന്നെ അത്ര പ്രസിദ്ധരല്ലാത്തതിനാല്‍ മുഹമ്മദി(സ)ന് അവരെക്കുറിച്ച് അറിയുമായിരുന്നില്ല. അതുകൊണ്ട് വന്നുഭവിച്ച അബദ്ധമാണിത്': ഖുര്‍ആന്‍ വിമര്‍ശകരുടെ വാദം പോകുന്നത് ഇങ്ങനെയാണ്.

ഈ വിമര്‍ശനം പ്രധാനമായും ഒരു പദത്തെ ചുറ്റിപ്പറ്റിയുള്ളതാണ്. 'യഹ്‌യാ'യെന്നതാണ് ആ പദം. യോഹന്നാന്‍ സ്‌നാപകനെക്കുറിക്കാന്‍ ഖുര്‍ആനില്‍ പ്രയോഗിക്കപ്പെട്ട പദമാണത്.  പദോല്‍പത്തിയെക്കുറിച്ച് സൂക്ഷ്മമായി ഗവേഷണം നടത്താത്ത വ്യാഖ്യാതാക്കള്‍ യഹ്‌യായെന്ന പദത്തിന് പകരമായും തത്തുല്യമായും യോഹന്നാന്‍ എന്ന് പ്രയോഗിച്ചിട്ടുണ്ടെന്നത് നേരാണ്. ഖുര്‍ആനില്‍ പറഞ്ഞിരിക്കുന്നത് ''മുമ്പ് നാം ആരെയും അവന്റെ പേര് (യഹ്‌യാ) ഉള്ളവരാക്കിയിട്ടില്ല''യെന്നാണ്. ഖുര്‍ആനില്‍ മുമ്പ് ആര്‍ക്കുമുണ്ടായിരുന്നില്ലെന്ന് പറയുന്നത് 'യഹ്‌യാ'യെന്ന നാമമാണ്; യോഹന്നാന്‍ എന്ന പേരല്ല. യഹ്‌യ=യോഹന്നാന്‍ എന്ന് കരുതിയ വ്യാഖ്യാതാക്കളാണ് ഖുര്‍ആനില്‍ പറഞ്ഞത് യോഹന്നാന്‍ എന്നാണെന്ന് വരുത്തിത്തീര്‍ത്തത്. ബൈബിള്‍ പഴയ നിയമത്തില്‍ പലതവണ യോഹന്നാന്‍ എന്ന പേര് ഉപയോഗിക്കപ്പെട്ടിട്ടുണ്ടെന്നത് നേരാണ്. ''യഹ്‌യാ''യെന്നല്ല അവിടെയൊന്നും പ്രയോഗിച്ചിരിക്കുന്നത് എന്ന കാര്യം പ്രത്യേകം ശ്രദ്ധേയമാണ്. അറബി ബൈബിളില്‍നിന്നുള്ള ഏതാനും ഉദ്ധരണികള്‍ കാണുക:

                         ثَمانِيَةٌ وَعِشرُونَ رَجُلاً. 12 وَمِن بَنِي عَزْجَدَ يُوحَنانُ بْنُ هِقّاطانَ وَمَعَهُ مِئَةٌ وَعَشْرَةُ رِجالٍ.

                                                                                   (എസ്രാ 8:12)

                                              جاءَ رَجُلٌ مُرسَلاً مِنَ اللهِ اسْمُهُ يُوحَنّا.

                                                                                 (യോഹന്നാന്‍ 1:6)

''യഹ്‌യാ''യെന്ന അറബി പദത്തിന് തത്തുല്യമായ ഹിബ്രു പദമാണോ 'യോഹന്നാന്‍' എന്നാണ് നാം ആദ്യമായി പരിശോധിക്കേണ്ടത്. ഇവ്വിഷയകമായ പ്രാഥമിക പരിശോധനയ്ക്ക് നാം അറബി ബൈബിള്‍ പരിശോധിച്ചാല്‍ മതിയാവും. 1 രാജാക്കന്മാര്‍ 25:23, 1 ദിനവൃത്താന്തരം 3:15, 1 ദിനവൃത്താന്തം 3:24, എസ്രാ 8:12 തുടങ്ങിയ പഴയ നിയമ ഉദ്ധരണികളില്‍ യോഹന്നാനെക്കുറിച്ച് പറയുന്നുണ്ട്. ഇവിടെയെല്ലാം അറബി ബൈബിളില്‍ യൂഹന്നായെന്നാണ് പറഞ്ഞിരിക്കുന്നത് ''യോഹന്നാന്‍ സുവിശേഷം'' എന്ന തലക്കെട്ട് അറബി ബൈബിളില്‍ 'ബിശാറത്തു യൂഹന്നാ'(بشارت يوحنا) യെന്നാണ്. അറബി പുതിയ നിയമത്തില്‍ യോഹന്നാന്‍ സ്‌നാപകനെയും യേശു ശിഷ്യനായ യോഹന്നാനെയുമെല്ലാം 'യൂഹന്നാ'യെന്നുതന്നെയാണ് വിളിച്ചിരിക്കുന്നത്; എവിടെയും 'യഹ്‌യാ'യെന്ന് കാണുന്നില്ല. 'യോഹന്നാന്‍' എന്ന ഹിബ്രു ശബ്ദത്തിന് തത്തുല്യമായ അറബി പദമായിരുന്നു 'യഹ്‌യാ'യെങ്കില്‍ അറബി ബൈബിളില്‍ യോഹന്നാന്‍ എന്ന പദത്തിന് പകരമായി യഹ്‌യായെന്ന് പ്രയോഗിക്കുമായിരുന്നുവെന്ന് തീര്‍ച്ചയാണ്.

സത്യത്തില്‍, യഹ്‌യാ, യോഹന്നാന്‍ എന്നിവ തികച്ചും വ്യത്യസ്തങ്ങളായ രണ്ട് നാമങ്ങളാണ്. യോഹന്നാന്‍ എന്ന ഹിബ്രു പദത്തിനര്‍ത്ഥം 'യഹോവ കാരുണ്യം ചെയ്തിരിക്കുന്നു'' (Jehovah has graced) എന്നാണ്. രണ്ട് പദങ്ങള്‍ ചേര്‍ന്നുണ്ടായ ഒരു നാമമാണ് യോഹന്നാന്‍. യൂ+ഹന്നാന്‍. യഹോവയുടെ ചുരുക്കമായാണ് 'യൂ'യെന്ന് പ്രയോഗിച്ചിരിക്കുന്നത്. 'ഹന്നാന്‍ എന്ന ഹിബ്രു പദം 'ഹനാന്‍' എന്ന അരമായിക് മൂലത്തില്‍ നിന്നുണ്ടായതാണ്. 'അനുകമ്പ'യെന്നാണ് അര്‍ത്ഥം.'യഹോവ അനുകമ്പയുള്ളവനായിരിക്കുന്നു''എന്നോ ''യഹോവയുടെ അനുകമ്പ''എന്നോ ആണ് യോഹന്നാന്‍ എന്ന പദത്തിന്റെ മൂലാര്‍ത്ഥം. എന്നാല്‍ 'യഹ്‌യാ'യെന്ന അറബിപദമുണ്ടായിരിക്കുന്നത് 'ഹയാ' എന്ന മൂലത്തില്‍നിന്നാണ്. ഈ പദത്തിന് രണ്ട് അര്‍ത്ഥമുണ്ട്. ഒന്ന് 'അല്‍ഹയാത്തി'ല്‍നിന്ന് നിര്‍ധരിക്കപ്പെട്ട ത്. 'ജീവന്‍' എന്നര്‍ത്ഥം. മറ്റൊന്ന്  'അല്‍-ഹയാഇ'ല്‍ നിന്നുള്ളത്. 'നാണം' എന്ന് സാരം 'യഹ്‌യാ'യുടെ ഉല്‍പത്തി ഇവ രണ്ടില്‍ ഏതില്‍നിന്നായിരുന്നാലും യോഹന്നാന്‍ എന്ന ഹിബ്രു പദവുമായി യാതൊരു ബന്ധവും ഇതിനില്ലെന്ന് വ്യക്തമാണ്. രണ്ടിന്റെയും മൂലങ്ങള്‍ വ്യത്യസ്തങ്ങളാണ്; അര്‍ത്ഥങ്ങള്‍ തമ്മില്‍ യാതൊരു വിധ സാമ്യവുമില്ലതാനും.

യേശുവിന് തൊട്ടുമുമ്പ് വന്ന സകരിയ്യായുടെ പുത്രനെയാണ് ഖുര്‍ആന്‍ 'യഹ്‌യാ'യെന്ന് വിളിക്കുന്നത്. സകരിയ്യയുടെയും എലിസബത്തിന്റെയും പുത്രനാണ് ബൈബിളിലെ യോഹന്നാന്‍ സ്‌നാപകന്‍. എന്നാല്‍ 'യഹ്‌യാ'യെന്ന പദവും 'യോഹന്നാന്‍' എന്ന പദവും തമ്മില്‍ യാതൊരുവിധ ബന്ധവുമില്ല. രണ്ടും രണ്ട് മൂലങ്ങളില്‍നിന്നുണ്ടായവ; രണ്ട് അര്‍ത്ഥങ്ങളുള്‍ക്കൊള്ളുന്നവ. ഇതെങ്ങനെ സംഭവിച്ചു? ഒരാളുടെ തന്നെ രണ്ട് നാമങ്ങളാണോ യഹ്‌യായും യോഹന്നാനും? അതല്ല വിമര്‍ശകര്‍ ആരോപിക്കുന്നതുപോലെ മുഹമ്മദ് നബി (സ)ക്ക് പറ്റിയ ഒരു കൈപ്പിഴയാണോ ഇത്?

ഈ ചോദ്യങ്ങള്‍ക്ക് വസ്തുനിഷ്ഠമായ ഉത്തരം ലഭിക്കണമെങ്കില്‍ ഖുര്‍ആനും ബൈബിളുമല്ലാത്ത മറ്റുവല്ല രേഖകളും സ്‌നാപക യോഹന്നാനെക്കുറിച്ച് പറയുന്നതായി നിലനില്‍ക്കുന്നുണ്ടോയെന്ന് പരിശോധിക്കണം. അങ്ങനെ വല്ല രേഖകളുമുണ്ടെങ്കില്‍ അവ ഇക്കാര്യത്തില്‍ നല്‍കുന്ന അറിവ് ഏറെ പ്രധാനപ്പെട്ടതാണ്. യോഹന്നാനോ യഹ്‌യയോ എന്താണ് യഥാര്‍ത്ഥ നാമമെന്ന് കണ്ടുപിടിക്കുന്നതിനുവേണ്ടി മാത്രമല്ല പ്രസ്തുത രേഖകള്‍ പ്രയോജനപ്രദമാവുക. അതുവഴി ഏത് ഗ്രന്ഥമാണ് കൃത്യവും സൂക്ഷ്മവുമായ പരാമര്‍ശങ്ങള്‍ നടത്തുന്നതെന്ന് മനസ്സിലാക്കുവാനും ഏതിനാണ് അപ്രമാദിത്വമുള്ളതെന്ന് വ്യക്തമായി അറിയുവാനും കഴിയും.

യോഹന്നാന്‍ സ്‌നാപകനെക്കുറിച്ച്, അദ്ദേഹം ഒരു വിശുദ്ധ പുരുഷനായിരുന്നുവെന്ന് വിശ്വസിക്കുന്ന ക്രൈസ്തവരും മുസ്‌ലിംകളുമല്ലാത്ത മറ്റേതെങ്കിലും വിഭാഗങ്ങളുണ്ടോയെന്ന അന്വേഷണം പ്രസക്തമാണ്. അങ്ങനെയൊന്നുണ്ടെങ്കില്‍ ഇക്കാര്യത്തില്‍ ഖുര്‍ആനും ബൈബിളുമല്ലാത്ത ഒരു സ്രോതസ്സായി അവരുടെ ഗ്രന്ഥങ്ങളോ രേഖകളോ സ്വീകരിക്കുവാന്‍ പറ്റുമെന്ന കാര്യത്തില്‍ സംശയമൊന്നുമില്ല. പക്ഷെ, അങ്ങനെയെന്തെങ്കിലുമുണ്ടോ?

യോഹന്നാന്‍ സ്‌നാപകനെ പിന്തുടരുന്നവരാണ് തങ്ങളെന്നും അദ്ദേഹം അന്തിമ പ്രവാചകനായിരുന്നുവെന്നും അവകാശപ്പെടുന്ന ഒരു വിഭാഗം ഇറാഖിലും ഇറാനിലും ഇന്നുമുണ്ട്. പോര്‍ച്ചുഗീസ് ക്രിസ്ത്യന്‍ മിഷനറിമാര്‍ ഇവരെ വിളിച്ചത് 'യോഹന്നാന്‍ സ്‌നാപകന്റെ ക്രിസ്ത്യാനികള്‍' (Christians of John the Baptist) എന്നായിരുന്നു. ഏകദൈവാരാധകരായ ഇവരുടെ പ്രധാനപ്പെട്ട ഒരു ആചാരമാണ് ജ്ഞാനസ്‌നാനം (Baptism).തങ്ങളുടെ മതത്തെയും വര്‍ഗത്തെയും കുറിക്കുവാന്‍വേണ്ടി മന്‍ഡായി (Mandai) എന്നും മതവിശ്വാസികളെ സൂചിപ്പിക്കുവാന്‍ മാന്‍ഡിയന്മാര്‍ (Mandaens) എന്നുമാണ് അവര്‍ ഉപയോഗിക്കുന്നത്. ഇസ്‌ലാമിനോട് സമാനമായ ഒട്ടനവധി വിശ്വാസാചാരങ്ങള്‍ മാന്‍ഡിയന്‍മാര്‍ക്കുണ്ട്. ജ്ഞാനസ്‌നാനം, പ്രാര്‍ത്ഥനകള്‍, ഉപവാസം, ദാനം തുടങ്ങിയവയാണ് ഇവരുടെ അടിസ്ഥാനാചാരങ്ങള്‍. അരമായ ഭാഷയോട് സാദൃശ്യമുള്ളതും സെമിറ്റിക് മൂലത്തില്‍നിന്ന് നിര്‍ധരിക്കപ്പെട്ടതുമായ മാന്‍ഡിയാക് ഭാഷ (Mandiac language)യിലാണ് ഇവരുടെ മതഗ്രന്ഥങ്ങളെല്ലാം എഴുതപ്പെട്ടിരിക്കുന്നത്. ഗിന്‍സാ റാബ, ദ്രാഷ ഇദ് യഹ്‌യ, ആദാം ബോഗ്‌റ, ദി കിലെസ്ത, നിയാനി എന്നിവയാണ് ഇവരുടെ മതഗ്രന്ഥങ്ങള്‍.

 ബാഗ്ദാദിലെ കൗണ്‍സില്‍ ഓഫ് ജനറല്‍ അഫയേഴ്‌സിന് കീഴില്‍ പ്രവര്‍ത്തിക്കുന്ന മാന്‍ഡിയന്‍ റിസര്‍ച്ച് സെന്ററില്‍നിന്ന് ഈ മതവിഭാഗത്തെക്കുറിച്ച കൂടുതല്‍ അറിവ് ലഭിക്കും www. mandaean.com-au, www.mandaean.org   എന്നീ വെബ്‌സൈറ്റുകളില്‍നിന്ന് ഈ മതവിഭാഗത്തിന്റെ വിശ്വാസാചാരങ്ങളെക്കുറിച്ച് മനസ്സിലാക്കാനാവും.

മാന്‍ഡിയന്‍മാര്‍ തങ്ങളുടെ പ്രവാചകനും ഗുരുവുമായി സ്വീകരിച്ചിരിക്കുന്നത് യോഹന്നാന്‍ സ്‌നാപകനെയാണെന്ന് പറഞ്ഞുവല്ലോ. അവര്‍ അദ്ദേഹത്തെ വിളിക്കുന്നത് യഹ്‌യാ യൂഹന്നായെന്നാണ്. സ്‌നാപക യോഹന്നാന്‍േറതായി അവര്‍ വിശ്വസിക്കുന്ന ഉപദേശങ്ങളുടെയും അധ്യാപനങ്ങളുടെയും സമാഹാരമാണ് 'ദ്രാഷാ ഇദ് യഹ്‌യ'യെന്ന ഗ്രന്ഥം. 'യഹ്‌യായുടെ പുസ്തകം' എന്നര്‍ത്ഥം. അവരുടെ ഏറ്റവും പ്രധാനപ്പെട്ട ഗ്രന്ഥമായ ഗിന്‍സ റാബയിലെ നാനൂറ്റി പത്താം അധ്യായം തന്നെ' യഹ്‌യായുടെ പ്രാര്‍ത്ഥനകള്‍' എന്ന തലക്കെട്ടോടുകൂടിയതാണ്. ഇവയില്‍നിന്നെല്ലാംതന്നെ മാന്‍ഡിയന്‍മാര്‍ യോഹന്നാന്‍ സ്‌നാപകനെ വിളിക്കുന്നത് യഹ്‌യാ യൂഹന്നായെന്നായിരുന്നുവെന്ന് വ്യക്തമാകുന്നു. ഖുര്‍ആനില്‍ പറഞ്ഞതുപോലെ യോഹന്നാന്‍ സ്‌നാപകന് യഹ്‌യായെന്ന പേര് കൂടിയുണ്ടായിരുന്നുവെന്ന് ഇതില്‍നിന്ന് കൃത്യമായി മനസ്സിലാകുന്നുണ്ട്.

മാന്‍ഡായിക്കുകാരുടെ ഗ്രന്ഥങ്ങളില്‍നിന്നും ലേഖനങ്ങളില്‍നിന്നുമെല്ലാം യോഹന്നാന്‍ സ്‌നാപകന്റെ നാമം  യഹ്‌യാ യൂഹന്നായെന്നായിരുന്നുവെന്ന് മനസ്സിലാകുന്നുണ്ടെന്ന് പറഞ്ഞല്ലോ. എന്നാല്‍ ഖുര്‍ആനിലൊരിടത്തും അദ്ദേഹത്തെ യഹ്‌യാ യൂഹന്നായെന്ന് വിളിച്ചിട്ടില്ല. എന്തുണ്ടൊണിത്?

ഈ ചോദ്യത്തിന് ഉത്തരം ലഭിക്കണമെങ്കില്‍ എന്തുകൊണ്ടാണ് ഈ ഇരട്ട നാമം ഉപയോഗിക്കുന്നതെന്ന് കൃത്യമായി മനസ്സിലാക്കണം. മാന്‍ഡിയന്‍മാരെയും അവരുടെ വിശ്വാസസംഹിതകളെയും ആചാര രീതികളെയുംകുറിച്ച് വിശദമായി പഠിച്ചയാളാണ് ഇ.എസ്. ഡ്രോവര്‍. അവരുടെ 'ദി മാന്‍ഡിയന്‍സ് ഓഫ് ഇറാഖ് ആന്റ് ഇറാന്‍', 'ദി കാനോനിക്കല്‍ പ്രെയര്‍ ബുക്ക് ഓഫ് ദി മാന്‍ഡിയന്‍സ്' എന്നീ പുസ്തകങ്ങള്‍ ഇവ്വിഷയകമായ ആധികാരിക രേഖകളായി പരിഗണിക്കപ്പെടുന്നവയാണ്. അവരും ആര്‍. മാക്കൂച്ചും കൂടിച്ചേര്‍ന്ന് എഴുതിയ ഗ്രന്ഥമാണ് എ മാന്‍ഡായിക് ഡിക്ഷ്ണറി. (E.S. Drowoer: & R. Marcuch: A MANDAIC DICTIONARY 1963 OXFORD)  മാന്‍ഡിയന്മാരുടെ സാങ്കേതിക ശബ്ദങ്ങളും അവര്‍ അവയുപയോഗിച്ചിരുന്ന രീതിയുമെല്ലാം ഈ ശബ്ദകോശത്തിലുണ്ട്. പ്രസ്തുത ഡിക്ഷ്ണറിയുടെ 185-ാം പുറത്തില്‍ യഹ്‌യാ (iahia) യുടെയും 190-ാം പുറത്തില്‍ യോഹന്നാ (iuhana) യുടെയും സാരം നല്‍കിയിട്ടുണ്ട്. അവ നോക്കുക:

ഇവയില്‍നിന്നും മറ്റ് മാന്‍ഡിയന്‍ സാഹിത്യങ്ങളില്‍നിന്നുമായി അവരുടെ പേരുകളെക്കുറിച്ച് നമുക്ക് മനസ്സിലാകുന്ന ഒട്ടേറെ കാര്യങ്ങളുണ്ട്. എല്ലാ മാന്‍ഡിയന്മാര്‍ക്കും പൊതുവെ രണ്ട് പേരുകളുണ്ടായിരിക്കും. ഒന്നാമത്തെ പേര് മല്‍വാഷാ നാമമെന്നും (malwasha name) രണ്ടാമത്തെ പേര് ലഖബ് (lagab) എന്നുമാണ് അറിയപ്പെടുക. എന്തിനാണ് ഈ രണ്ട് പേരുകള്‍? ഇവ എന്താണ് അര്‍ത്ഥമാക്കുന്നത്?

ഇ.എസ്. ഡ്രോവര്‍ എഴുതുന്നു: ''രണ്ടാമത്തെ പേര് പൊതുവെ ഒരു മുഹമ്മദന്‍ നാമമായിരിക്കും. ഇതാണ് എല്ലാ സാധാരണ ആവശ്യങ്ങള്‍ക്കും ഉപയോഗിക്കപ്പെടാറുള്ളത്. ആദ്യത്തെ പേര് (malwasha) ആണ് അയാളുടെ യഥാര്‍ത്ഥ ആത്മീയ നാമം. മതപരവും മാന്ത്രികവുമായ സന്ദര്‍ഭങ്ങളിലെല്ലാം ഈ പേരാണ് ഉപയോഗിക്കുക'' (E.S. Drower: The Mandaeans of  Iraq and Iran (1962-Lieden) Page 81)

യോഹന്നാന്‍ സ്‌നാപകന്റെ മാല്‍വാഷാ നാമമാണ് യഹ്‌യ.യോഹന്നാന്‍ എന്നത് അദ്ദേഹത്തിന്റെ ലഖബും. ജനങ്ങള്‍ പൊതുവെ അദ്ദേഹത്തെ വിളിച്ചിരുന്നത് യോഹന്നാന്‍ എന്നായിരിക്കണം. കാരണം മാന്‍ഡിയന്‍മാര്‍ എല്ലാ സാധാരണ ആവശ്യങ്ങള്‍ക്കും പൊതുവായി ലഖബാണ് വിളിച്ചിരുന്നത്. എന്നാല്‍ അദ്ദേഹത്തിന്റെ യഥാര്‍ത്ഥ ആത്മീയ നാമം യഹ്‌യായെന്നായിരുന്നു. മതപരമായ സന്ദര്‍ഭങ്ങളുമായി ബന്ധപ്പെടുത്തി ഈ നാമമാണ് ഏറെ ഉപയോഗിക്കപ്പെട്ടിരിക്കുന്നത്. യഹ്‌യായുടെ പുസ്തകത്തിലെ മിക്ക അധ്യായങ്ങളും ആരംഭിക്കുന്നതുതന്നെ ''യഹ്‌യാ രാത്രികളില്‍ പ്രഖ്യാപിക്കുന്നു; യോഹന്നാ രാത്രിയുടെ സന്ധ്യകളിലും'' എന്ന് പറഞ്ഞുകൊണ്ടാണ്. ചുരുക്കത്തില്‍ യോഹന്നാന്‍ സ്‌നാപകന്റെ യഥാര്‍ത്ഥ ആത്മീയ നാമം 'യഹ്‌യാ'യെന്നായിരുന്നു; ജനങ്ങള്‍ ആ സമൂഹത്തില്‍ നിലനിന്നിരുന്ന സമ്പ്രദായപ്രകാരം അദ്ദേഹത്തിന്റെ ലഖബ് ആയ 'യോഹന്നാ' എന്ന പേരിലാണ് പൊതുവായി അദ്ദേഹത്തെ വിളിച്ചിരുന്നത് എന്ന് മാത്രമേയുള്ളൂ.

ഖുര്‍ആനില്‍ 'യഹ്‌യാ'യെന്ന് മാത്രമെ പ്രവാചക നാമമായി ഉപയോഗിക്കപ്പെട്ടിട്ടുള്ളൂവെന്ന് പറഞ്ഞുവല്ലോ. അതാണ്, അത് മാത്രമാണ് അദ്ദേഹത്തിന്റെ യഥാര്‍ത്ഥ നാമം എന്നുള്ളതുകൊണ്ടാണിത്. ദൈവിക കല്‍പന പ്രകാരം മാതാപിതാക്കള്‍ അദ്ദേഹത്തിന് നല്‍കിയ പേരായാണ് ഖുര്‍ആന്‍ 'യഹ്‌യാ'യെന്ന നാമത്തെ പരിചയപ്പെടുത്തുന്നത്. അതായിരുന്നു അദ്ദേഹത്തിന്റെ യഥാര്‍ത്ഥ ആത്മീയ നാമം എന്ന് മാന്‍ഡിയന്‍ സാഹിത്യങ്ങളില്‍നിന്ന് നമുക്ക് മനസ്സിലാകുന്നതോടെ ഖുര്‍ആനിന്റെ ദൈവികത ഒരിക്കല്‍കൂടി വ്യക്തമാവുകയാണ് ചെയ്യുന്നത്; ഒപ്പം, ബൈബിളില്‍നിന്ന് പകര്‍ത്തിയെഴുതിയതാണ് ഖുര്‍ആന്‍ എന്ന വാദത്തിന്റെ മൂലത്തില്‍തന്നെ ഈ വസ്തുതകള്‍ കഠാരകുത്തിക്കയറ്റുന്നു. ബൈബിളിലെവിടെയും പരാമര്‍ശിക്കപ്പെട്ടിട്ടില്ലാത്ത യോഹന്നാന്‍ സ്‌നാപകന്റെ യഥാര്‍ത്ഥ നാമമായ 'യഹ്‌യാ' ഖുര്‍ആനില്‍ വന്നത് യാദൃച്ഛികമാകാനിടയില്ലെന്ന് ഏതൊരു സാധാരണക്കാരന്നും മനസ്സിലാക്കാവുന്നതേയുള്ളൂ. സകരിയ്യാ (അ) യുടെ വാര്‍ധക്യകാലത്ത് വന്ധ്യയായ ഭാര്യയില്‍ പുത്രനെ പ്രദാനം ചെയ്യുകയും പുത്രന് 'യഹ്‌യാ' യെന്ന് പേരിടാന്‍ നിര്‍ദ്ദേശിക്കുകയും ചെയ്ത തമ്പുരാനില്‍നിന്ന് അവതീര്‍ണമായ ഗ്രന്ഥമായതിനാലാണ് ഖുര്‍ആനില്‍ ഇക്കാര്യത്തിലും കൃത്യവും സൂക്ഷ്മവുമായ പരാമര്‍ശങ്ങളുണ്ടായത് എന്ന് മാത്രമേ ചിന്തിക്കുവാന്‍ കഴിയുകയുള്ളൂ.

ബൈബിളില്‍ സ്‌നാപകനെ കുറിക്കുവാന്‍ യോഹന്നാന്‍ എന്ന് മാത്രമെ പ്രയോഗിക്കപ്പെട്ടിട്ടുള്ളൂവെന്നതും സ്വാഭാവികമാണ്. അദ്ദേഹത്തിന് ശേഷം പതിറ്റാണ്ടുകള്‍ ഏറെക്കഴിഞ്ഞ് രചിക്കപ്പെട്ട സുവിശേഷങ്ങളിലാണ് യോഹന്നാന്‍ സ്‌നാപകനെക്കുറിച്ച പരാമര്‍ശങ്ങളുള്ളത്. നടേ സൂചിപ്പിച്ചതുപോലെ തന്റെ കാലത്ത് അദ്ദേഹം ജനങ്ങളാല്‍ പൊതുവായി വിളിക്കപ്പെട്ടത് യോഹന്നായെന്നായിരിക്കണം. അതുകൊണ്ടുതന്നെ വാമൊഴിയായി പ്രചരിച്ച അദ്ദേഹത്തിന്റെ ജീവിതകഥനങ്ങളിലും ഉപദേശങ്ങളിലും യോഹന്നായെന്ന പേരായിരിക്കണം പ്രധാനമായും ഉപയോഗിക്കപ്പെട്ടത്. പതിറ്റാണ്ടുകള്‍ കഴിഞ്ഞ് രചിക്കപ്പെട്ട സുവിശേഷങ്ങളുടെ കര്‍ത്താക്കള്‍ യോഹന്നായെന്നാണ് അദ്ദേഹത്തിന്റെ നാമമെന്ന് കരുതിയത് സ്വാഭാവികം മാത്രം. എന്നാല്‍ ബൈബിള്‍ പുസ്തകങ്ങളുടെ കര്‍ത്താക്കള്‍ക്ക് തങ്ങളുടെ ഗ്രന്ഥരചനയില്‍ ദൈവനിവേശനമുണ്ടായിരുന്നുവെന്ന ക്രൈസ്തവ വിശ്വാസത്തിന് കോട്ടം തട്ടിക്കുന്ന പല തെളിവുകളിലൊന്നാണ് ഇതുമെന്ന വസ്തുത വിസ്മരിച്ചുകൂടാ. അങ്ങനെയൊരു ദൈവിക ഇടപെടലിന്റെ സ്വാധീനത്താല്‍ രചിക്കപ്പെട്ടതായിരുന്നു സുവിശേഷങ്ങളെങ്കില്‍ തീര്‍ച്ചയായും സ്‌നാപകന്റെ യഥാര്‍ത്ഥമായ ആത്മീയ നാമമായിരുന്നു അവയില്‍ പ്രതിപാദിക്കേണ്ടിയിരുന്നത്. എന്നാല്‍ ജനങ്ങള്‍ക്കിടയില്‍ പ്രചാരത്തിലിരുന്ന യോഹന്നാന്‍ എന്ന പേര് മാത്രമെ സുവിശേഷങ്ങള്‍ ഉപയോഗിക്കുന്നുള്ളൂ. തങ്ങള്‍ക്ക് വാമൊഴിയായി പകര്‍ന്നുകിട്ടിയതിനേക്കാള്‍ അധികമായ യാതൊരു അറിവും സുവിശേഷ കര്‍ത്താക്കള്‍ക്ക് ഉണ്ടായിരുന്നില്ലെന്ന വസ്തുതയാണല്ലോ ഇത് വെളിപ്പെടുത്തുന്നത്.

യഹ്‌യായുടെ ലഖബ് ആയ 'യോഹന്നാ'യെക്കുറിച്ച് എന്തെങ്കിലും അറിവ് ഖുര്‍ആന്‍ നല്‍കുന്നുണ്ടോയെന്ന് പരിശോധിക്കുമ്പോള്‍ അതിന്റെ ദൈവികത ഒന്നുകൂടി നമുക്ക് ബോധ്യപ്പെടുകയും സര്‍വ്വശക്തനായ സ്രഷ്ടാവിന് മുമ്പില്‍ നമ്രശിരസ്‌കരാവുന്നതിലേക്ക് നാം നയിക്കപ്പെടുകയും ചെയ്യുന്നു. 'യൂ', 'ഹന്നാന്‍' എന്നീ രണ്ട് വാക്കുകളുടെ സമ്മേളനത്തില്‍നിന്നാണ് യൂഹന്നായെന്ന പദമുണ്ടായിട്ടുള്ളതെന്നും 'ഹന്നാന്‍' എന്ന ഹിബ്രു പദം 'ഹനാന്‍' എന്ന അരമായ മൂലത്തില്‍നിന്നുണ്ടായതാണെന്നും 'അനുകമ്പ'യെന്നാണ് ഈ പദത്തിന് അര്‍ത്ഥമെന്നും നേരത്തെ സൂചിപ്പിച്ചത് ഓര്‍ക്കുക. 'ഹനാന്‍' എന്ന അറബി പദവും ഇതേ അര്‍ത്ഥമുള്‍ക്കൊള്ളുന്നതാണ്. അറബി-ഹിബ്രു-അരാമിക് തുടങ്ങിയ ഭാഷകളെല്ലാം ഒരേ സെമിറ്റിക് മൂലത്തില്‍നിന്നുണ്ടായവയാണല്ലോ.

ഖുര്‍ആനില്‍ ഒരു തവണമാത്രമെ 'ഹനാന്‍' എന്ന പദം ഉപയോഗിച്ചിട്ടുള്ളൂ; സൂറത്തുമര്‍യമിലെ പതിമൂന്നാം (19:13) സൂക്തത്തില്‍. ആ സൂക്തത്തിന്റെ മലയാളം ലിപ്യന്തരണം ഇങ്ങനെയാണ്:

വ ഹനാനന്‍ മിന്‍ ലദുന്നാ വ സകാത്തന്‍ വ കാന തഖിയ്യാ ''ഈ വചനത്തിന്റെ മലയാള പരിഭാഷ ''നമ്മുടെ പക്കല്‍നിന്നുള്ള അനുകമ്പയും പരിശുദ്ധിയും നല്‍കി; അദ്ദേഹം (യഹ്‌യാ) ധര്‍മ്മനിഷ്ഠയുള്ളവനായിരുന്നു'' (19:13)വെന്നാണ്.

ഈ വചനത്തില്‍ യഹ്‌യായെക്കുറിച്ച് 'നമ്മുടെ പക്കല്‍നിന്നുള്ള അനുകമ്പ' (ഹനാനന്‍ മിന്‍ ലദുന്ന)യെന്ന് പ്രയോഗിച്ചത് പ്രത്യേകം ശ്രദ്ധേയമാണ്. യഹ്‌യാ 'ദൈവത്തില്‍നിന്നുള്ള അനുകമ്പ'യാണെന്നര്‍ത്ഥം. യൂഹന്നയെന്ന പദത്തിനര്‍ത്ഥം 'ദൈവത്തില്‍നിന്നുള്ള അനുകമ്പ'യെന്നാണെന്ന് മുമ്പ് സൂചിപ്പിച്ചത് ഓര്‍ക്കുക. 'യോഹന്ന'യിലെ അതേ ഹനാന്‍ തന്നെയാണ് ഖുര്‍ആന്‍ ഇവിടെ പ്രയോഗിച്ചിരിക്കുന്നത്. യൂഹന്നയിലെ യൂ ഒഴിവാക്കി അതിന്റെ മൂലരൂപത്തിന് തത്തുല്യമായ 'ഹനാന്‍' എന്ന് പ്രയോഗിക്കുകയാണ് ഖുര്‍ആന്‍ ചെയ്തിരിക്കുന്നത്.

ഇവിടെ 'യൂ' ഒഴിവാക്കിയിരിക്കുന്നതും പ്രത്യേകം ശ്രദ്ധിക്കേണ്ടതാണ്. 'യഹോവ'യുടെ ചുരുക്കമായാണ് 'യൂ'യെന്ന് ഉപയോഗിച്ചിരിക്കുന്നതെന്ന് നേരത്തെ സൂചിപ്പിച്ചുവല്ലോ. അറബിയില്‍ ഏകദൈവത്തെക്കുറിക്കുവാന്‍ യഹോവയെന്ന് ഉപയോഗിക്കാറില്ല. അതുകൊണ്ടുതന്നെ യഹോവയുടെ ചുരുക്കപ്പേരായ 'യൂ'യെന്ന് ഖുര്‍ആനില്‍ പ്രയോഗിക്കുന്നത് സംഗതമല്ലല്ലോ. യൂഹന്നായെന്നത് യഹ്‌യായുടെ യഥാര്‍ത്ഥ നാമമല്ലെന്നും അദ്ദേഹത്തിന്റെ സ്വഭാവ സവിശേഷതകളുടെ അടിസ്ഥാനതില്‍ ജനം വിളിച്ചിരുന്ന പേരായിരുന്നുവെന്നും അത് അദ്ദേഹത്തിന്റെ സവിശേഷത മാത്രമാണ് വെളിപ്പെടുത്തുന്നതെന്നും ഓര്‍ക്കുക. അതുകൊണ്ടുതന്നെ 'ദൈവത്തില്‍നിന്നുള്ള അനുകമ്പ'യെന്ന അര്‍ത്ഥത്തിലുള്ള യൂഹന്നായെന്ന് അതേപോലെ അറബിയില്‍ പ്രയോഗിച്ചിരുന്നുവെങ്കില്‍ ആ പദം അര്‍ത്ഥരഹിതമാകുമായിരുന്നു. യൂഹന്നായെന്നത് അദ്ദേഹത്തിന്റെ പേരല്ലല്ലോ. എന്നാല്‍ 'യൂ' ഒഴിവാക്കിക്കൊണ്ട് 'ദൈവത്തില്‍നിന്നുള്ള ഹനാന്‍' എന്ന് കൃത്യമായി ഖുര്‍ആന്‍ പ്രയോഗിച്ചത് കാണുമ്പോള്‍ അതിന്റെ സൂക്ഷ്മതയും കൃത്യതയും നമുക്ക് വ്യക്തമായി മനസ്സിലാവുകയും തെറ്റുപറ്റാത്ത സ്രഷ്ടാവില്‍ നിന്നുള്ളതാണ് ഖുര്‍ആനെന്ന് സുതരാം ബോധ്യപ്പെടുകയും ചെയ്യുന്നു. ഖുര്‍ആനില്‍ ഒരേയൊരു സ്ഥലത്ത് മാത്രമെ ഹനാന്‍ എന്ന പദം ഉപയോഗിച്ചിട്ടുള്ളൂവെന്നും അത് യഹ്‌യായെക്കുറിച്ചാണെന്നതുംകൂടി ഇതോടൊപ്പം ചേര്‍ത്തുവായിക്കുമ്പോള്‍ ആര്‍ക്കാണ് അതിന്റെ ദൈവികത ബോധ്യപ്പെടാതിരിക്കുക?

ഇനി നാം ചോദ്യത്തിലേക്ക് തിരിച്ചുപോവുക. ഖുര്‍ആനിലെ സൂറത്തുമര്‍യം ഏഴാം വചനത്തില്‍ (19:7) പറയുന്നതെന്താണ്?

''ഹേ സക്കരിയാ, തീര്‍ച്ചയായും നിനക്ക് നാം ഒരു ആണ്‍കുട്ടി യെപറ്റി സന്തോഷവാര്‍ത്ത അറിയിക്കുന്നു. അവന്റെ പേര്‍ യഹ്‌യാ എന്നാകുന്നു. മുമ്പ് നാം ആരെയും അവന്റെ പേരുള്ളവരാക്കിയിട്ടില്ല'' (വി.ഖു.19:7)

ഈ വചനം രണ്ടുതരത്തില്‍ വ്യാഖ്യാനിക്കപ്പെട്ടിട്ടുണ്ട്.:

ഒന്ന്) ഇവിടെ ''ലം നജ്അല്‍ ലഹു മിന്‍ ഖബ്‌ലു സമിയ്യാ''യെന്ന വചനഭാഗത്തെയാണ് ''മുമ്പ് ആരെയും അവന്റെ പേരുള്ളവരാക്കിയിട്ടില്ല''യെന്ന് പരിഭാഷപ്പെടുത്തിയിരിക്കുന്നത്. 'സമിയ്യന്‍' എന്ന പദത്തെയാണ് പേരുള്ളവന്‍ എന്ന് ഭാഷാന്തരം ചെയ്തിരിക്കുന്നത്. ഇതിന് ''മിഥ്‌ലന്‍'' എന്നും ''ശബീഹന്‍'' എന്നുമെല്ലാം അര്‍ത്ഥമുണ്ട്. അദ്ദേഹത്തെ പോലെയുള്ളവന്‍ എന്നര്‍ത്ഥം. അപ്പോള്‍ ഈ വചനഭാഗത്തിന് ''മുമ്പ് ആരെയും അദ്ദേഹത്തെപ്പോലെയുള്ളവനാക്കിയിട്ടില്ല''എന്ന അര്‍ത്ഥംവരും. വൃദ്ധനായ പിതാവിന് വന്ധ്യയായ ഭാര്യയിലുണ്ടായ കുഞ്ഞാണ് യഹ്‌യ. ഇങ്ങനെയൊരു സംഭവം അദ്ദേഹത്തിന് മുമ്പുണ്ടായിട്ടില്ല. ഈ അര്‍ത്ഥത്തില്‍ യഹ്‌യായെപ്പോലെ ഒരാള്‍ അദ്ദേഹത്തിന് മുമ്പുണ്ടായിട്ടില്ലെന്നാണ് ഈ വചനത്തിന് ചില പണ്ഡിതന്മാര്‍ നല്‍കിയ വ്യാഖ്യാനം.

രണ്ട്) ഈ വചനഭാഗത്തിന്റെ നേര്‍ക്കുനേരെയുള്ള അര്‍ത്ഥം പരിഗണിച്ചുകൊണ്ട് 'യഹ്‌യാ'യെന്ന പേര് സ്‌നാപകനുമുമ്പ് മറ്റാര്‍ക്കുമുണ്ടായിട്ടില്ലെന്നാണ് മറ്റൊരു വിഭാഗം പണ്ഡിതന്മാരുടെ വ്യാഖ്യാനം.

രണ്ട് വ്യാഖ്യാനങ്ങള്‍ പ്രകാരം പരിശോധിച്ചാലും ഈ ഖുര്‍ആന്‍ സൂക്തത്തില്‍ യാതൊരു വിധ അബദ്ധവുമില്ലെന്നതാണ് വാസ്തവം. യഹ്‌യാ(അ)ക്കുമുമ്പ് വൃദ്ധനായ പിതാവിന് വന്ധ്യയായ മാതാവിലുണ്ടായ ഒരു കുഞ്ഞിന്റെ കഥ ബൈബിളോ ഖുര്‍ആനോ പരാമര്‍ശിക്കുന്നില്ല. രണ്ടാമത്തെ വ്യാഖ്യാനത്തില്‍ കടിച്ചുതൂങ്ങി ഖുര്‍ആനില്‍ അബദ്ധം ആരോപിക്കുവാന്‍ വേണ്ടി ശ്രമിക്കുന്നവരുടെ വിമര്‍ശനങ്ങള്‍ ഖുര്‍ആനിന്റെ പ്രോജ്ജ്വല പ്രകാശത്തിന് മുമ്പില്‍ കരിഞ്ഞുവീഴുന്നതാണ് നാം കണ്ടത്. യഹ്‌യായെന്ന ഒരു നാമം സ്‌നാപകനുമുമ്പ് ആര്‍ക്കെങ്കിലും നല്‍കപ്പെട്ടതായി സൂചിപ്പിക്കുന്ന രേഖകളൊന്നും തന്നെയില്ല. യഹ്‌യാ=യോഹന്നാന്‍ എന്ന സമവാക്യം ഖുര്‍ആനിന്‍േറതല്ല. അതുകൊണ്ടുതന്നെ അത് വിമര്‍ശനങ്ങള്‍ക്കുമുമ്പില്‍ തകരും. എന്നാല്‍ ഖുര്‍ആന്‍ മുന്നോട്ടുവെക്കുന്ന ആശയങ്ങളാകട്ടെ ഓരോ വിമര്‍ശനങ്ങളുന്നയിക്കപ്പെടുമ്പോഴും പൂര്‍വ്വാധികം പ്രോജ്ജ്വലമായി വിളങ്ങുകമാത്രേമയുള്ളൂ.

ബൈബിളിലെ ഉല്‍പത്തി പുസ്തകം മുപ്പത്തിയൊന്‍പതാം അധ്യായത്തില്‍ യാക്കോബിന്റെ പുത്രനായ യോസഫ് ഈജിപ്തിലെത്തിയതും ഫറോവയുടെ ഉദ്യോഗസ്ഥനായ പോത്തിഫറുടെ വീട്ടില്‍ കഴിഞ്ഞതും അവിടെവെച്ച് യജമാനന്റെ ഭാര്യ അദ്ദേഹത്തെ പ്രലോഭിപ്പിക്കാന്‍ ശ്രമിച്ചതും അതില്‍നിന്ന് അദ്ദേഹം രക്ഷപ്പെട്ടതുമായകാര്യങ്ങള്‍ വിശദമായിത്തന്നെ പ്രതിപാദിക്കുന്നുണ്ട്. അത് ഇങ്ങനെയാണ്. ”ഇതിനിടയില്‍ യിസ്മാഈല്യന്‍ യോസഫിനെ ഈജിപ്തിലേക്ക് കൊണ്ടുപോയി. അവിടെ ഫറോവയുടെ ഒരു ഉദ്യോഗസ്ഥനും അംഗരക്ഷകസേനയുടെ നായകനുമായ ഈജിപ്തുകാരന്‍ പോത്തിഫര്‍ അയാളെ വിലക്കുവാങ്ങി. കര്‍ത്താവ് യോസഫിന്റെ കൂടെയുണ്ടായിരുന്നതിനാല്‍ അയാള്‍ വിജയിയായി. യജമാനനായ ഈജിപ്തുകാരന്റെ വീട്ടില്‍ അയാള്‍ താമസിച്ചു. കര്‍ത്താവ് യോസഫിന്റെ കൂടെയുണ്ട് എന്നും അയാള്‍ ചെയ്യുന്നതെല്ലാം വിജയ പ്രദമാക്കുന്നതിന് സഹായിക്കുന്നുണ്ടെന്നും യജമാനന്‍ കണ്ടു. അതുകൊണ്ട് യജമാനന്‍ യോസഫില്‍ പ്രീതനായി. യോസഫ് അയാളെ സ്വീകരിച്ചു. പോത്തിഫര്‍ യോസഫിനെ തന്റെ ഗൃഹത്തിലെ മേല്‍നോട്ടക്കാരനായി നിയമിച്ചു. തനിക്കുള്ളതെല്ലാം അയാളുടെ ചുമതലയില്‍ ഏല്‍പിച്ചുകൊടുത്തു. അയാള്‍ യോസേഫിനെ തന്റെ ഗൃഹത്തിനും തനിക്കുള്ള എല്ലാറ്റിനും ചുമതലക്കാരനാക്കി. അപ്പോള്‍ മുതല്‍ യോസേഫ് നിമിത്തം കര്‍ത്താവ് ഈജിപ്തുകാരന്‍ പോത്തിഫറുടെ കുടുംബത്തെ അനുഗ്രഹിച്ചു. കര്‍ത്താവിന്റെ അനുഗ്രഹം വീട്ടിലും വയലിലും അയാള്‍ക്കുള്ള സകലതിന്മേലുമുണ്ടായി. അതുകൊണ്ട് പോത്തിഫര്‍ തനിക്കുള്ളതെല്ലാം യോസേഫിന്റെ ചുമതലയില്‍വിട്ടു. യോസേഫ് വീട്ടിലുണ്ടായിരുന്നതിനാല്‍ ഭക്ഷണത്തിനപ്പുറം മറ്റൊന്നിനും പോത്തിഫറിന് ശ്രദ്ധിക്കേണ്ടിവന്നില്ല. സുന്ദരനും സുമുഖനുമായിരുന്നു യോസേഫ്. കുറെകാലം കഴിഞ്ഞപ്പോള്‍ യജമാനനന്റെ ഭാര്യ യോസേഫില്‍ കണ്ണുവെച്ചു. ”എന്നോടൊപ്പം ശയിക്കുക” എന്ന് അവള്‍ പറഞ്ഞു. യോസേഫ് വിസമ്മതിച്ചു. യജമാനന്റെ ഭാര്യയോട് അയാള്‍ പറഞ്ഞു; ‘നോക്കൂ, ഞാന്‍ ഇവിടെയുള്ളതുകൊണ്ട് എന്റെ യജമാനന്‍ വീട്ടിലെ ഒരു കാര്യത്തെപ്പറ്റിയും ക്ലേശിക്കുന്നില്ല. തനിക്കുള്ളതെല്ലാം അദ്ദേഹംഎന്നെ ഏല്‍പിച്ചിരിക്കയാണ്. ഈ വീട്ടില്‍ അയാള്‍ എന്നേക്കാള്‍ വലിയവനല്ല. വീട്ടില്‍ എനിക്കൊന്നും അപ്രാപ്യമാക്കിയിട്ടില്ല; നിങ്ങളെയൊഴിച്ച്. കാരണം നിങ്ങള്‍ അദ്ദേഹത്തിന്റെ ഭാര്യയാണല്ലോ.അപ്പോള്‍ പിന്നെ ഞാന്‍ എങ്ങനെ ഈ വലിയ അധര്‍മ്മം പ്രവര്‍ത്തിക്കും; ദൈവത്തിന് എതിരായി പാപം ചെയ്യും? ”. ദിവസംതോറും യോേസഫിനോട് അവള്‍ ഇക്കാര്യം പറഞ്ഞിരുന്നെങ്കിലും അവളുടെ കൂടെ ശയിക്കാനോ അവളുടെയടുക്കല്‍ ഇരിക്കാനോ അയാള്‍ കൂട്ടാക്കിയില്ല. ഒരുദിവസം ജോലിചെയ്യുന്നതിനായി യോേസഫ് വീട്ടിനുള്ളിലേക്ക് കടന്നു. പുരുഷന്മാര്‍ ആരും വീട്ടിലില്ലായിരുന്നു. അയാളുടെ മേലങ്കിയില്‍ കയറിപ്പിടിച്ച് ‘എന്നോടൊപ്പം ശയിക്കുക‘ എന്ന് അവള്‍ ആവശ്യം ഉന്നയിച്ചു. മേലങ്കി അവളുടെ കയ്യില്‍ഉപേക്ഷിച്ച് അയാള്‍ ഓടി വീടിന് വെളിയിലേക്ക് പോയി. അയാള്‍ മേലങ്കിഉപേക്ഷിച്ച് വീടിന് പുറത്തേക്ക് ഓടിപ്പോയി എന്ന് കണ്ടപ്പോള്‍ അവള്‍ വീട്ടിലുള്ളവരെ വിളിച്ചുവരുത്തി ഇപ്രകാരം പറഞ്ഞു: ‘നോക്കൂ, നമ്മെ അപമാനിക്കാന്‍ അദ്ദേഹം ഒരു എബ്രായനെ നമ്മുടെയിടയില്‍ കൊണ്ടുവന്നിരിക്കുന്നു! എന്റെ കൂടെ ശയിക്കുന്നതിന് അയാള്‍ അകത്ത് കയറിവന്നു. ഞാന്‍ ഉച്ചത്തില്‍ നിലവിളിച്ചു. ഞാന്‍ ശബ്ദം ഉയര്‍ത്തി നിലവിളിക്കുന്നതുകേട്ട ഉടന്‍ അയാള്‍ മേലങ്കി ഉപേക്ഷിച്ച് ഓടി വീടിന് വെളിയിലിറങ്ങിപ്പോയി‘. യോസഫിന്റെ യജമാനന്‍ വീട്ടില്‍ വരുന്നതുവരെ അവള്‍ അയാളുടെ മേലങ്കി കൈവശംവെച്ചു. നേരത്തെ പറഞ്ഞ കഥ തന്നെ അവള്‍ അയാളോടും പറഞ്ഞു: ‘അങ്ങ് ഞങ്ങള്‍ക്ക് കൊണ്ടുവന്ന എബ്രായ ദാസന്‍ മാനം കെടുത്താന്‍ എന്റെ അടുക്കല്‍ വന്നു. ഞാന്‍ ശബ്ദംഉയര്‍ത്തി കരഞ്ഞയുടന്‍ അയാള്‍ മേലങ്കി എന്റെയടുത്ത് ഉപേക്ഷിച്ച് വീടിന് വെളിയിലേക്ക് ഓടിപ്പോയി‘. ‘അങ്ങയുടെ ദാസന്‍ ഈ വി ധത്തിലാണ് എന്നോട് പെരുമാറിയതെന്ന് ഭാര്യ പറയുന്നത് കേട്ടപ്പോള്‍ അദ്ദേഹം കോപംകൊണ്ട് ജ്വലിച്ചു. യജമാനന്‍ യോസേഫിനെ പിടിച്ച് രാജാവിന്റെ തടവുകാരെ അടച്ചിരുന്ന ജയിലില്‍ അടച്ചു. യോസേഫ് ജയിലിലായി. എന്നാല്‍, കര്‍ത്താവ് അയാളോട് കൂടെയായിരുന്ന് അചഞ്ചലമായ സ്‌നേഹം പ്രകടിപ്പിച്ചു. തന്മൂലം തടവറ സൂക്ഷിപ്പുകാരന് അയാളോട് പ്രീതി തോന്നി. അവിടെയുള്ള എല്ലാ തടവുകാരെയും തടവറ സൂക്ഷിപ്പുകാരന്‍ യോസേഫിന്റെ മേല്‍നോട്ടത്തില്‍ വിട്ടുകൊടുത്തു; അവിടെ നടന്നതിനെല്ലാം അയാളായിരുന്നു നടത്തിപ്പുകാരന്‍. കര്‍ത്താവ് യോസേഫിനോട് കൂടെയുണ്ടായിരുന്നതിനാല്‍ യോസേഫിനെ ഭരമേല്‍പിച്ച ഒരു കാര്യത്തിനും തടവറ സൂക്ഷിപ്പുകാരന്‍ ശ്രദ്ധിക്കേണ്ടിയിരുന്നില്ല. യോസേഫിന്റെ പ്രവര്‍ത്തികള്‍ എല്ലാം ദൈവം വിജയകരമാക്കി” (ഉല്‍പത്തി 39: 1-23). യൂസുഫ് നബി (عليهالسلام)യുടെ ചരിത്രം അല്‍പം വിശദമായിത്തന്നെ ഒരുകഥാകഥനത്തിന്റെ രീതിയില്‍ ഖുര്‍ആന്‍ പന്ത്രണ്ടാം അധ്യായമായ സൂറത്തുയൂസുഫില്‍ വിവരിക്കുന്നുണ്ട്. നടേ ഉദ്ധരിച്ച ബൈബിളില്‍ വചനങ്ങളില്‍ സൂചിപ്പിക്കപ്പെട്ട സംഭവങ്ങള്‍ വിവരിക്കുന്ന ഖുര്‍ആന്‍ സൂക്തങ്ങളുടെ സാരംകാണുക: ”ഈജിപ്തില്‍നിന്ന് അവനെ (യൂസുഫിനെ) വിലക്കെടുത്തയാള്‍ തന്റെ ഭാര്യയോട് പറഞ്ഞു: ഇവന് മാന്യമായ താമസസൗകര്യം നല്‍കുക. ഇവന്‍ നമുക്ക് പ്രയോജനപ്പെട്ടേക്കാം. അല്ലെങ്കില്‍ നമുക്ക് അവനെ മകനായി സ്വീകരിക്കാം. അപ്രകാരം യൂസുഫിന് നാം ആ ഭൂപ്രദേശത്ത് സൗകര്യമുണ്ടാക്കികൊടുത്തു. സ്വപ്‌നവാര്‍ത്തകളുടെ വ്യാഖ്യാനം അദ്ദേഹത്തിന് നാം അറിയിച്ചുകൊടുക്കുവാന്‍ വേണ്ടിയാണ് അത്. അല്ലാഹു തന്റെ കാര്യം ജയിച്ചടക്കുന്നവനത്രെ. പക്ഷെ, മനുഷ്യരില്‍ അധികപേരും അത് മനസ്സിലാക്കുന്നില്ല. അങ്ങനെ അദ്ദേഹം പൂര്‍ണ്ണ വളര്‍ച്ചയെത്തിയപ്പോള്‍ അദ്ദേഹത്തിന് നാം യുക്തിബോധവും അറിവും നല്‍കി. സുകൃതം ചെയ്യുന്നവര്‍ക്ക് അപ്രകാരം നാം പ്രതിഫലം നല്‍കുന്നു. അവന്‍ (യൂസുഫ്)ഏതൊരുവളുടെ വീട്ടിലാണോ അവള്‍ അവനെ വശീകരിക്കുവാന്‍ ശ്രമംനടത്തി. വാതിലുകള്‍ അടച്ചുപൂട്ടിയിട്ട് അവള്‍ പറഞ്ഞു. ഇങ്ങോട്ട് വാ. അവന്‍പറഞ്ഞു: അല്ലാഹുവില്‍ ശരണം! നിശ്ചയമായും അവനാണ് എന്റെ രക്ഷിതാവ്. അവന്‍ എന്റെ താമസം ക്ഷേമകരമാക്കിയിരിക്കുന്നു. തീര്‍ച്ചയായും അക്രമം പ്രവര്‍ത്തിക്കുന്നവന്‍ വിജയിക്കുകയില്ല. അവള്‍ക്ക് അവനില്‍ ആഗ്രഹം ജനിച്ചു. തന്റെ രക്ഷിതാവിന്റെ പ്രമാണം കണ്ടറിഞ്ഞില്ലായിരുന്നുവെങ്കില്‍ അവന് അവളിലും ആഗ്രഹം ജനിച്ചേനെ. അപ്രകാരം (സംഭവിച്ചത്) തിന്മയും നീചവൃത്തിയും അവനില്‍നിന്ന് നാം തിരിച്ചുവിടുന്നതിന് വേണ്ടിയത്രെ. തീര്‍ച്ചയായും അവന്‍ നമ്മുടെ നിഷ്‌കളങ്കരായ ദാസന്മാരില്‍ പെട്ടവനാകുന്നു. അവര്‍ രണ്ടുപേരും വാതില്‍ക്കലേക്ക് മത്‌സരിച്ചോടി. അവള്‍ പിന്നില്‍നിന്ന് അവന്റെ കുപ്പായം(പിടിച്ചു. അത്) കീറി. അവര്‍ ഇരുവരും വാതില്‍ക്കല്‍വെച്ച് അവളുടെ നാഥനെ (ഭര്‍ത്താവിനെ) കണ്ടുമുട്ടി. അവള്‍ പറഞ്ഞു. താങ്കളുടെ ഭാര്യയുടെ കാര്യത്തില്‍ ദുരുദ്ദേശം പുലര്‍ത്തിയവനുള്ള പ്രതിഫലം, അവന്‍ തടവിലാക്കപ്പെടുക എന്നതോ, വേദനയേറിയ മറ്റേതെങ്കിലും ശിക്ഷയോ തന്നെ ആയിരിക്കണം. യൂസുഫ് പറഞ്ഞു: അവളാണ് എന്നെ വശീകരിക്കുവാന്‍ശ്രമം നടത്തിയത്. അവളുടെ കുടുംബത്തില്‍പെട്ട ഒരു സാക്ഷി ഇപ്രകാരം സാക്ഷ്യപ്പെടുത്തി. അവന്റെ കുപ്പായം മുന്നില്‍ നിന്നാണ് കീറിയിട്ടുള്ളതെങ്കില്‍ അവള്‍ സത്യമാണ് പറഞ്ഞത്. അവനാകട്ടെ കളവ്പറയുന്നവരുടെ കൂട്ടത്തിലാണ്. എന്നാല്‍ അവന്റെ കുപ്പായം പിന്നില്‍നിന്നാണ് കീറിയിട്ടുള്ളതെങ്കില്‍ അവള്‍ കളവാണ് പറഞ്ഞത്. അവനാകട്ടെ സത്യം പറയുന്നവരുടെ കൂട്ടത്തിലാണ്. അങ്ങനെ അവന്റെ(യൂസുഫിന്റെ) കുപ്പായം പിന്നില്‍ നിന്നാണ് കീറിയിട്ടുള്ളത് എന്ന്കണ്ടപ്പോള്‍ അയാള്‍ (ഗൃഹനാഥന്‍ –തന്റെ ഭാര്യയോട്) പറഞ്ഞു. തീര്‍ച്ചയായും ഇത് നിങ്ങളുടെ (സ്ത്രീകളുടെ) തന്ത്രത്തില്‍ പെട്ടതാണ്.നിങ്ങളുടെ തന്ത്രം ഭയങ്കരംതന്നെ. യൂസുഫേ, നീ ഇത് അവഗണിച്ചേക്കുക. (പെണ്ണേ) നീ നിന്റെ പാപത്തിന് മാപ്പ് തേടുക. തീര്‍ച്ചയായും നീ പിഴച്ചവരുടെ കൂട്ടത്തിലാകുന്നു. നഗരത്തിലെ ചില സ്ത്രീകള്‍ പറഞ്ഞു. പ്രഭുവിന്റെ ഭാര്യ തന്റെ വേലക്കാരനെ വശീകരിക്കാന്‍ ശ്രമിക്കുന്നു. അവള്‍ അവനോട് പ്രേമബദ്ധയായി കഴിഞ്ഞിരിക്കുന്നു. തീര്‍ച്ചയായും അവള്‍ വ്യക്തമായ പിഴവില്‍ അകപ്പെട്ടതായി ഞങ്ങള്‍ കാണുന്നു. അങ്ങനെ ആസ്ത്രീകളുടെ കുസൃതിയെപ്പറ്റി അവള്‍ കേട്ടറിഞ്ഞപ്പോള്‍ അവരുടെ അടുത്തേക്ക് അവള്‍ ആളെ അയക്കുകയും, അവര്‍ക്ക് ചാരിയിരിക്കാവുന്ന ഇരിപ്പിടങ്ങള്‍ ഒരുക്കുകയും ചെയ്തു. അവരില്‍ ഓരോരുത്തര്‍ക്കും(പഴങ്ങള്‍ മുറിക്കാന്‍) അവള്‍ ഓരോ കത്തികൊടുത്തു. (യൂസഫിനോട്) അവള്‍പറഞ്ഞു. നീ അവരുടെ മുമ്പിലേക്ക് പുറപ്പെടുക. അങ്ങനെ അവനെ അവര്‍കണ്ടപ്പോള്‍ അവര്‍ക്ക് അവനെപ്പറ്റി വിസ്മയം തോന്നുകയും, അവരുടെ സ്വന്തം കൈകള്‍ തന്നെ അറുത്ത് പോവുകയും ചെയ്തു. അവര്‍ പറഞ്ഞു. അല്ലാഹു എത്ര പരിശുദ്ധന്‍. ഇത് ഒരു മനുഷ്യനല്ല, ആദരണീയനായ ഒരുമലക്ക് തന്നെയാണ്. അവള്‍ പറഞ്ഞു: എന്നാല്‍ ഏതൊരുവന്റെ കാര്യത്തില്‍ നിങ്ങളെന്നെ ആക്ഷേപിച്ചുവോ അവനാണ് ഇത്. തീര്‍ച്ചയായും അവനെ ഞാന്‍ വശീകരിക്കാന്‍ ശ്രമിച്ചിട്ടുണ്ട്. അപ്പോള്‍ അവന്‍ (സ്വയം കളങ്കപ്പെടുത്താതെ) കാത്തു സൂക്ഷിക്കുകയാണ് ചെയ്ത്. ഞാന്‍ അവനോട് കല്‍പിക്കുന്നപ്രകാരം അവന്‍ ചെയ്തില്ലെങ്കില്‍ തീര്‍ച്ചയായും അവന്‍ തടവിലാക്കപ്പെടുകയും നിന്ദ്യരുടെ കൂട്ടത്തിലായിരിക്കുകയും ചെയ്യും. അവന്‍ (യൂസുഫ്) പറഞ്ഞു. എന്റെ രക്ഷിതാവേ, ഇവര്‍ എന്നെ ഏതൊന്നിലേക്ക് ക്ഷണിക്കുന്നുവോ അതിനേക്കാളും എനിക്ക് കൂടുതല്‍ പ്രിയപ്പെട്ടത് ജയിലാകുന്നു. ഇവരുടെ കുതന്ത്രം എന്നെവിട്ട് നീ തിരിച്ചുകളയാത്ത പക്ഷം ഞാന്‍ അവരിലേക്ക് ചാഞ്ഞുപോയേക്കും. അങ്ങനെ ഞാന്‍ അവിവേകികളുടെ കൂട്ടത്തിലായിപ്പോവുകയും ചെയ്യും. അപ്പോള്‍ അവന്റെ പ്രാര്‍ത്ഥന തന്റെ രക്ഷിതാവ് സ്വീകരിക്കുകയും അവരുടെ കുതന്ത്രം അവനില്‍നിന്ന് അവന്‍ തട്ടിത്തിരിച്ച് കളയുകയുംചെയ്തു. തീര്‍ച്ചയായും അവന്‍ എല്ലാം കേള്‍ക്കുന്നവനും കാണുന്നവനുമത്രെ. പിന്നീട് തെളിവുകള്‍ കണ്ടറിഞ്ഞതിനുശേഷവും അവര്‍ക്ക് തോന്നി; അവനെ ഒരവധിവരെ തടവിലാക്കുകതന്നെവേണമെന്ന്” (വി.ഖു. 12:21-35). ഖുര്‍ആനും ബൈബിളും ഒരേ കഥയാണ് പരാമര്‍ശിക്കുന്നതെങ്കിലും അവയുടെ വിശദാംശങ്ങളില്‍ ഏറെ വ്യത്യാസങ്ങളുണ്ടെന്ന് ഇവ രണ്ടും വായിച്ചുനോക്കുന്ന ആര്‍ക്കും മനസ്സിലാക്കാവുന്നതേയുള്ളൂ. യൂസുഫ് കുറ്റക്കാരനാണെന്ന് യജമാനനും മറ്റും കരുതിയെന്ന രൂപത്തിലാണ് ബൈബിളിന്റെ വിവരണം. ഖുര്‍ആനിലാകട്ടെ യൂസുഫിന്റെ നിരപരാധിത്വം ബോധ്യപ്പെട്ടശേഷവും അദ്ദേഹത്തെ തടവിലാക്കുവാന്‍ യജമാനന്‍ തീരുമാനിച്ചതായാണ് പറഞ്ഞിരിക്കുന്നത്. മേലങ്കി ഊരി ഓടിയ യൂസുഫിനെയാണ് ബൈബിള്‍ പരിചയപ്പെടുത്തുന്നത്. യജമാന ഭാര്യ പിന്നില്‍നിന്ന് പിടിച്ചപ്പോള്‍ കുപ്പായം കീറിയതായാണ് ഖുര്‍ആനിലെ വിവരണം. ഇങ്ങനെ വിശദാംശങ്ങളില്‍ നിരവധി വ്യത്യാസങ്ങളുണ്ടെന്നതാണ് വാസ്തവം. യജമാനന്റെ പേരുമായി ബന്ധപ്പെട്ട് ഖുര്‍ആന്‍ നടത്തുന്ന പരാമര്‍ശങ്ങളുടെ വെളിച്ചത്തിലാണ് വിമര്‍ശകന്മാര്‍ ഖുര്‍ആനില്‍ അബദ്ധം ആരോപിക്കുന്നത്. ‘ബൈബിളില്‍ വ്യക്തമായിത്തന്നെ യജമാനന്റെ പേര് പറഞ്ഞിരിക്കുന്നു. പോത്തിഫര്‍. ഖുര്‍ആനിലാകട്ടെ യൂസുഫി(عليه السلام)ന്റെയജമാനനെക്കുറിച്ച് പരാമര്‍ശിക്കുമ്പോള്‍ അസീസ് എന്നാണ്പ്രയോഗിച്ചിരിക്കുന്നത്. പോത്തിഫര്‍ എന്ന നാമവുമായി ഒരു നിലക്കും യോജിക്കുന്ന പേരല്ല അസീസ്. മാത്രവുമല്ല, ഒരു ശുദ്ധ അറബി പദമാണ് ‘അസീസ്‘. യോസഫിന്റെ കാലത്തെ ഈജിപ്തില്‍ ജീവിച്ച ഒരു വ്യക്തിക്ക് ശുദ്ധമായ ഒരു അറബിപ്പേരുണ്ടായിരുന്നുവെന്ന് കരുതുന്നതുപോലും വിഡ്ഢിത്തമാണ്. യഹൂദ ക്രിസ്ത്യാനികളില്‍നിന്ന് യോസഫിന്റെ കഥകേട്ട് നാളുകള്‍ക്കുശേഷം മുഹമ്മദ് ഖുര്‍ആനില്‍ അത് എഴുതിച്ചേര്‍ത്തപ്പോള്‍ ഉണ്ടായ ഒരു അബദ്ധമാണിത്……..’ വിമര്‍ശകരുടെ ന്യായവാദങ്ങള്‍ ഇങ്ങനെപോകുന്നു. തങ്ങളുടെ വാദം സ്ഥാപിക്കുന്നതിനായി വിമര്‍ശകര്‍ ഉദ്ധരിക്കുന്ന ഖുര്‍ആന്‍ സൂക്തങ്ങളുടെ സാരമാണ് താഴെ: ”നഗരത്തിലെ ചില സ്ത്രീകള്‍ പറഞ്ഞു: പ്രഭു(അല്‍-അസീസ്)വിന്റെഭാര്യ തന്റെ വേലക്കാരനെ വശീകരിക്കാന്‍ ശ്രമിക്കുന്നു. അവള്‍ അവനോട് പ്രേമബദ്ധയായിക്കഴിഞ്ഞിരിക്കുന്നു. തീര്‍ച്ചയായും അവള്‍ വ്യക്തമായ പിഴവില്‍ അകപ്പെട്ടതായി ഞങ്ങള്‍ കാണുന്നു” (വി.ഖു: 12:30). ”(ആ സ്ത്രീകളെ) വിളിച്ചുവരുത്തിയിട്ട് അദ്ദേഹം (രാജാവ്) ചോദിച്ചു. യൂസുഫിനെ വശീകരിക്കുവാന്‍ നിങ്ങള്‍ ശ്രമം നടത്തിയപ്പോള്‍ നിങ്ങളുടെ സ്ഥിതി എന്തായിരുന്നു? അവര്‍ പറഞ്ഞു. അല്ലാഹു എത്ര പരിശുദ്ധന്‍. ഞങ്ങള്‍ യൂസുഫിനെപ്പറ്റി ദോഷകരമായ ഒന്നും മനസിലാക്കിയിട്ടില്ല. പ്രഭു(അല്‍-അസീസ്)വിന്റെ ഭാര്യ പറഞ്ഞു: ഇപ്പോള്‍ സത്യം വെളിപ്പെട്ടിരിക്കുന്നു. ഞാന്‍ അദ്ദേഹത്തെ വശീകരിക്കുവാന്‍ ശ്രമിക്കുകയാണുണ്ടായത്. തീര്‍ച്ചയായും അദ്ദേഹം സത്യവാന്മാരുടെ കൂട്ടത്തില്‍ തന്നെയാകുന്നു” വി.ഖു: 12:51). ഈ സൂക്തങ്ങളില്‍ പ്രഭുവെന്ന് പരിഭാഷപ്പെടുത്തിയിരിക്കുന്നത് ‘അല്‍അസീസ്‘ എന്ന അറബി പദത്തെയാണ്. ചില ഇംഗ്ലീഷ് പരിഭാഷകളില്‍ ഈപദം ഭാഷാന്തരം ചെയ്യാതെ അല്‍-അസീസ് എന്നുതന്നെ അതേപോലെ ഒരുനാമമെന്ന രൂപത്തില്‍ കൊടുത്തിട്ടുണ്ട്. അത് എടുത്തുകൊണ്ടാണ് പോത്തിഫറിന് പകരമായി അസീസ് എന്നാണ് ഖുര്‍ആനില്‍ പ്രയോഗിച്ചിരിക്കുന്നതെന്ന് വിമര്‍ശകര്‍ വാദിക്കുന്നത്. അസീസ് എന്ന അറബി നാമത്തിന്റെ അര്‍ത്ഥം ‘പ്രതാപി‘ യെന്നാണ്. അറബികള്‍ ഇത് ഒരു പേരായും പേരിന്റെ ഭാഗമായും ഉപയോഗിക്കാറുണ്ടെന്നത് ശരിയാണ്. അമുസ്‌ലിംകളും ഈ പേര് ഉപയോഗിക്കാറുണ്ട്. ഇറാഖി മന്ത്രിയായ താരീഖ് അസീസിന്റെ പേര് ഉദാഹരണം. അസീസിന്റെ മുമ്പില്‍ ‘അല്‍’ എന്ന അവ്യയം (definite article) ചേര്‍ത്ത്അല്‍-അസീസ് എന്നാണ് നടേ സൂചിപ്പിക്കപ്പെട്ട ഖുര്‍ആന്‍ സൂക്തങ്ങളില്‍ പ്രയോഗിച്ചിരിക്കുന്നത്. ഒരു പേര് എന്ന നിലയ്ക്ക് അറബിയില്‍ അല്‍-അസീസ് എന്നു പയോഗിക്കാറില്ല. ഇംഗ്ലീഷില്‍ the Lord എന്ന് പറയുന്നതിന് സമാനമായ ഒരു പ്രയോഗമാണിത്. അതുകൊണ്ടാണ് മലയാളത്തില്‍ ‘പ്രഭു‘വെന്ന് പരിഭാഷപ്പെടുത്തിയിരിക്കുന്നത്. വ്യക്തിയുടെ സ്ഥാനമാണ് അല്ലാതെ പേരല്ല അല്‍-അസീസ് എന്ന പ്രയോഗം ദ്യോതിപ്പിക്കുന്നത്. ബൈബിളില്‍ പോത്തിഫര്‍ എന്ന് വിളിച്ച വ്യക്തിയുടെ പേര് സൂചിപ്പിച്ചുകൊണ്ടുള്ള ഒരു പ്രയോഗമെന്ന നിലയ്ക്കല്ല, പ്രത്യുത അദ്ദേഹത്തിന്റെ സ്ഥാനം വ്യക്തമാക്കുന്ന ഒരു പദമെന്ന നിലയ്ക്കാണ് ഖുര്‍ആനില്‍ അല്‍-അസീസ് എന്ന് പറഞ്ഞിരിക്കുന്നത് എന്നര്‍ത്ഥം. പോത്തിഫറിനെക്കുറിച്ച് ബൈബിളില്‍ പറഞ്ഞിരിക്കുന്നത്” ഫറോവാന്റെ ഒരു ഉദ്യോഗസ്ഥനും അംഗരക്ഷക സേനയുടെ നായകനു” (ഉല്‍ 39:1)മെന്നാണല്ലോ. ഈ ഉന്നതസ്ഥാനം വ്യക്തമാക്കാനാണ് ഖുര്‍ആന്‍ ”അല്‍-അസീസ്” എന്ന് പ്രയോഗിച്ചതെന്നാണ് മനസ്സിലാവുന്നത്. ബൈബിളില്‍ പ്രയോഗിച്ച പോത്തിഫര്‍ എന്ന നാമം എന്തുകൊണ്ടാണ് ഖുര്‍ആനില്‍ ഉപയോഗിക്കാതിരുന്നത് എന്ന് ഖണ്ഡിതമായിപ്പറയാന്‍ നമുക്കാവില്ല. സര്‍വ്വശക്തന്റെ വചനങ്ങളുടെ സമാഹാരമാണ് ഖുര്‍ആന്‍. അതിലെ ഓരോ പദവും ഉപയോഗിച്ചതിനു പിന്നില്‍ എന്തെന്ത് യുക്തികളാണുള്ളതെന്ന് പറയാന്‍ പരിമിതമായ അറിവ് മാത്രം നല്‍കപ്പെട്ട നമുക്ക് കഴിയില്ല. ഇക്കാര്യത്തില്‍ സ്ഥിരപ്പെട്ട ഹദീസുകളോ പണ്ഡിതാഭിപ്രായങ്ങളോ ഉദ്ധരിക്കപ്പെട്ടതായി കാണാനും കഴിയുന്നില്ല. ‘പോത്തിഫര്‍’ എന്ന് ഖുര്‍ആനില്‍ പറയാതിരുന്നതിന്റെ കാരണം കൃത്യമായി സര്‍വ്വശക്തനേ അറിയൂ. എങ്കിലും, ‘പോത്തിഫര്‍’ എന്ന ഈജിപ്ഷ്യന്‍ നാമത്തിന്റെ അര്‍ത്ഥമെന്താണെന്ന് മനസ്സിലാക്കുമ്പോള്‍ ഖുര്‍ആനില്‍ പദങ്ങള്‍ പ്രയോഗിച്ചതിലുള്ള സൂക്ഷ്മത നമുക്ക് ഒരിക്കല്‍കൂടി ബോധ്യപ്പെടുമെന്നകാര്യം ഇവിടെ ചൂണ്ടിക്കാട്ടാമെന്ന് തോന്നുന്നു. പോത്തിഫര്‍ എന്ന ഈജിപ്ഷ്യന്‍ പദത്തിനര്‍ത്ഥം ‘റേ‘യാല്‍ നല്‍കപ്പെട്ടവന്‍” (he whom Re has given-Harpers Bible Dictionary P. 809) ഈജിപ്തുകാരുടെ സൂര്യദേവന്റെ പേരാണ്‘റേ‘ സൂര്യദേവന്റെ ദാനം‘ എന്ന് വേണമെങ്കില്‍ ”പോത്തിഫര്‍” എന്ന പേര് പരിഭാഷപ്പെടുത്താം. സൂര്യാരാധനയുമായി ബന്ധപ്പെട്ടതാണ് ഈ പേര്. ബഹുദൈവാരാധനയുമായി ബന്ധപ്പെടുന്ന നാമങ്ങളോ പ്രയോഗങ്ങളോ ഉപയോഗിക്കുവാന്‍ പാടില്ലെന്നാണ് ഇസ്‌ലാമിന്റെ അനുശാസന. ഇതുകൊണ്ടു കൂടിയായിരിക്കാം ഖുര്‍ആന്‍ ‘പോത്തിഫര്‍’ എന്നപേരുപയോഗിക്കാതെ അദ്ദേഹത്തിന്റെ സ്ഥാനത്തെ ദ്യോതിപ്പിക്കുന്ന ‘അല്‍-അസീസ് എന്നുമാത്രം പ്രയോഗിച്ചത്. ഖുര്‍ആന്‍ ‘പോത്തിഫര്‍’ എന്ന പേര് ഉപയോഗിക്കുകയാണെങ്കില്‍ ‘സൂര്യദേവന്റെ ദാന‘മാണ് അദ്ദേഹമെന്ന് അംഗീകരിക്കുന്നുവെന്നാണല്ലോ വന്നുചേരുക. ദൈവിക വചനങ്ങളുടെ സമാഹാരമാണ് ഖുര്‍ആന്‍ എന്ന വസ്തുത ഇവിടെയും വ്യക്തമായി വെളിപ്പെടുന്നു. ബൈബിളില്‍ നിന്ന് മുഹമ്മദ് നബി(ﷺ) പകര്‍ത്തിയെഴുതിയതായിരുന്നു യോസേഫിന്റെ കഥയെങ്കില്‍ സ്വാഭാവികമായും പോത്തിഫര്‍ എന്ന യജമാനനാമം ഖുര്‍ആനില്‍ സ്ഥാനം പിടിക്കേണ്ടതായിരുന്നു. പോത്തിഫര്‍ എന്നാല്‍ ”സൂര്യദേവന്റെ ദാനം”എന്നാണ് അര്‍ത്ഥമെന്ന് മുഹമ്മദ് നബിക്ക് അറിയാമായിരുന്നുവെന്നും അത്ബഹുദൈവ വിശ്വാസമുള്‍ക്കൊള്ളുന്നതായതിനാല്‍ ഖുര്‍ആനില്‍ നിന്ന്ഒഴിവാക്കിയെന്നും കരുതുന്നത് ശുദ്ധ ഭോഷ്‌ക്കാണെന്ന് പറയേണ്ടതില്ല. ഖുര്‍ആനിന്റെ അവതരണ കാലത്ത് ഈജിപ്ഷ്യന്‍ ഭാഷ അറിയാവുന്നവരായി ആരും തന്നെ അറേബ്യയിലുണ്ടായിരുന്നില്ലെന്ന വസ്തുത ഓര്‍ക്കുക. അതേപോലെതന്നെ പോത്തിഫര്‍ എന്ന നാമമൊഴിവാക്കിയത് യാദൃച്ഛികമാണെന്ന് കരുതാനും ന്യായമില്ല. ബൈബിളില്‍നിന്ന് കാര്യങ്ങള്‍ പകര്‍ത്തിയെഴുതുമ്പോള്‍ അതില്‍ പല തവണപ്രയോഗിക്കപ്പെട്ട ഒരു പേര് യാദൃച്ഛികമായി ഒഴിവായിപ്പോയെന്ന് കരുതുന്നതെങ്ങനെ? ഖുര്‍ആനിന്റെ ദൈവികതയ്ക്കുള്ള തെളിവാണിതെന്ന് പറയുന്നത് അതുകൊണ്ടാണ്. സര്‍വ്വജ്ഞനായ തമ്പുരാന്റെ വചനങ്ങളിലെവിടെയും യാതൊരു സ്ഖലിതവുമുണ്ടാവുകയില്ലെന്ന വസ്തുത ഇവിടെ നമുക്ക് ബോധ്യപ്പെടുന്നു. ഖുര്‍ആനിനെതിരെ ഉന്നയിക്കപ്പെട്ട എല്ലാ ആരോപണങ്ങളും ഖുര്‍ആനിന്റെ ദൈവികത വ്യക്തമാവുന്നിടത്തേക്കാണ് നമ്മെ നയിക്കുന്നതെന്നുള്ളത് എന്തുമാത്രം വിസ്മയകരമായിരിക്കുന്നു!
 

ഖുര്‍ആനിലെ ഇരുപത്തിയേഴാം അധ്യായമായ സൂറത്തുന്നംലിലെ 20മുതല്‍ 46വരെയുള്ള സൂക്തങ്ങളില്‍ സബഇലെ രാജ്ഞിയായിരുന്ന ബില്‍ഖീസിനെപ്പറ്റി സുലൈമാന്‍ നബി (അ)  മരംകൊത്തിപ്പക്ഷിയില്‍നിന്നും അറിഞ്ഞതും അവരെ ഇസ്‌ലാമിലേക്ക് ക്ഷണിച്ചുകൊണ്ട് എഴുത്തെഴുതിയതും അവര്‍ സുലൈമാന്‍ നബി (അ) യുടെ കൊട്ടാരം സന്ദര്‍ശിച്ചതുമെല്ലാം ഒരു കഥാകഥനത്തിന്റെ രൂപത്തില്‍തന്നെ അവതരിപ്പിക്കുന്നുണ്ട്. പ്രസ്തുത ഖുര്‍ആന്‍ സൂക്തങ്ങളുടെ സാരം കാണുക:

''അദ്ദേഹം പക്ഷികളെ പരിശോധിക്കു കയുണ്ടായി. എന്നിട്ട് അദ്ദേഹം പറഞ്ഞു: എന്തുപറ്റി? മരംകൊത്തിയെ കാണുന്നില്ലല്ലോ, അഥവാ അത് സ്ഥലംവിട്ടുപോയ കൂട്ടത്തിലാണോ? ഞാന്‍ അതിന് കഠിനശിക്ഷ നല്‍കുകയോ അല്ലെങ്കില്‍ അറുക്കുകയോ തന്നെ ചെയ്യും. അല്ലെങ്കില്‍ വ്യക്തമായ വല്ല ന്യായവും അത് എനിക്ക് ബോധിപ്പിച്ചുതരണം. എന്നാല്‍ അത് എത്തിച്ചേരാന്‍ അധികം താമസിച്ചില്ല. എന്നിട്ടത് പറഞ്ഞു: താങ്കള്‍ സൂക്ഷ്മമായി അറിഞ്ഞിട്ടില്ലാത്ത ഒരു കാര്യം ഞാന്‍ സൂക്ഷ്മമായി മനസിലാക്കിയിട്ടുണ്ട്. 'സബഇ'ല്‍ നിന്ന് യഥാര്‍ത്ഥമായ ഒരു വാര്‍ത്തയുംകൊണ്ടാണ് ഞാന്‍ വന്നിരിക്കുന്നത്. ഒരു സ്ത്രീ അവരെ ഭരിക്കുന്നതായി ഞാന്‍ കണ്ടെത്തുകയുണ്ടായി. എല്ലാ കാര്യങ്ങളില്‍നിന്നും അവള്‍ക്ക് നല്‍കപ്പെട്ടിട്ടുണ്ട്. അവള്‍ക്ക് ഗംഭീരമായ ഒരു സിംഹാസനവുമുണ്ട്. അവളും അവളുടെ ജനതയും അല്ലാഹുവിന് പുറമെ സൂര്യനെ പ്രണാമം ചെയ്യുന്നതായിട്ടാണ് ഞാന്‍ കണ്ടെത്തിയത്. പിശാച് അവര്‍ക്ക് തങ്ങളുടെ പ്രവര്‍ത്തനങ്ങള്‍ ഭംഗിയായി തോന്നിക്കുകയും അവരെ നേര്‍മാര്‍ഗത്തില്‍നിന്ന് തടയുകയും ചെയ്തിരിക്കുന്നു. അതിനാല്‍ അവര്‍ നേര്‍വഴിപ്രാപിക്കുന്നില്ല. ആകാശങ്ങളിലും ഭൂമിയിലും ഒളിഞ്ഞുകിടക്കുന്നത് പുറത്ത് കൊണ്ടുവരികയും നിങ്ങള്‍ രഹസ്യമാക്കുന്നതും പരസ്യമാക്കുന്നതും അറിയുകയും ചെയ്യുന്നവനായ അല്ലാഹുവിന് അവര്‍ പ്രണാമം ചെയ്യാതിരിക്കാന്‍വേണ്ടി (പിശാച് അങ്ങനെ ചെയ്യുന്നു). മഹത്തായ സിംഹാസനത്തിന്റെ നാഥനായ അല്ലാഹു അല്ലാതെ യാതൊരു ദൈവവുമില്ല. സുലൈമാന്‍ പറഞ്ഞു: നീ സത്യം പറയുന്നതാണോ അതല്ലാ നീ കള്ളം പറയുന്നവരുടെ കൂട്ടത്തിലായിരിക്കുന്നുവോ എന്ന് നാം നോക്കാം. നീ എന്റെ ഈ എഴുത്ത് കൊണ്ടുപോയി അവര്‍ക്കിട്ട് കൊടുക്കുക. പിന്നീട് നീ അവരില്‍നിന്ന് മാറിനിന്ന് അവ ര്‍ എന്ത് മറുപടി നല്‍കുന്നുവെന്ന് നോക്കുക. അവള്‍ പറഞ്ഞു: ഹേ, പ്രമുഖന്മാരേ, എനിക്ക് ഇതാ മാന്യമായ ഒരു എഴുത്ത് നല്‍കപ്പെട്ടിരിക്കുന്നു. അത് സുലൈമാന്റെ പക്കല്‍നിന്നുള്ളതാണ്. ആ കത്ത് ഇപ്രകാരമത്രെ: 'പരമകാരുണികനും കരുണാനിധിയുമായ അല്ലാഹുവിന്റെ നാമത്തില്‍. എനിക്കെതിരില്‍ നിങ്ങള്‍ അഹങ്കാരം കാണിക്കാതിരിക്കുകയും കീഴൊതുങ്ങിയവരായിക്കൊണ്ട് നിങ്ങള്‍ എന്റെ അടുത്ത് വരികയും ചെയ്യുക'. അവള്‍ പറഞ്ഞു: ഹേ പ്രമുഖന്മാരേ, എന്റെ ഈ കാര്യത്തില്‍ നിങ്ങള്‍ എനിക്ക് നിര്‍ദ്ദേശം നല്‍കുക. നിങ്ങള്‍ എന്റെ അടുത്ത് സന്നിഹിതരായിട്ടല്ലാതെ യാതൊരു കാര്യവും ഖണ്ഡിതമായി തീരുമാനിക്കുന്നവളല്ല ഞാന്‍. അവര്‍ പറഞ്ഞു: നാം ശക്തിയുള്ളവരും ഉഗ്രമായ സമരവീര്യമുള്ളവരുമാണ്. അധികാരം അങ്ങേക്കാണല്ലോ. അതിനാല്‍ എന്താണ് കല്‍പിച്ചരുളേണ്ടതെന്ന് ആലോചിച്ചുനോക്കുക. അവള്‍ പറഞ്ഞു: തീര്‍ച്ചയായും രാജാക്കന്മാര്‍ ഒരു നാട്ടില്‍ കടന്നാല്‍ അവര്‍ അവിടെ നാശമുണ്ടാക്കുകയും അവിടത്തുകാരിലെ പ്രതാപികളെ നിന്ദ്യന്മാരാക്കുകയും ചെയ്യുന്നവതാണ്. അപ്രകാരമാണ് അവര്‍ ചെയ്തു കൊണ്ടിരിക്കുന്നത്. ഞാന്‍ അവര്‍ക്ക് ഒരു പാരിതോഷികം കൊടുത്തയച്ചിട്ട് എന്തൊരു വിവരവും കൊണ്ടാണ് ദൂതന്മാര്‍ മടങ്ങിവരുന്നതെന്ന് നോക്കാന്‍ പോവുകയാണ്. അവന്‍ (ദൂതന്‍) സുലൈമാന്റെ അടുത്ത് ചെന്നപ്പോള്‍ അദ്ദേഹം പറഞ്ഞു: നിങ്ങള്‍ എന്നെ സമ്പത്ത് തന്ന് സഹായിക്കുകയാണോ?എന്നാല്‍ എനിക്ക് അല്ലാഹു നല്‍കിയിട്ടുള്ളതാണ് നിങ്ങള്‍ക്കവന്‍ നല്‍കിയിട്ടുള്ളതിനേക്കാള്‍ ഉത്തമം. പക്ഷെ, നിങ്ങള്‍ നിങ്ങളുടെ പാരിതോഷികം കൊണ്ട് സന്തോഷം കൊള്ളുകയാകുന്നു. നീ അവരുടെ അടുത്തേക്ക് തന്നെ മടങ്ങിച്ചെല്ലുക. തീര്‍ച്ചയായും അവര്‍ക്ക് നേരിടാന്‍ കഴിയാത്ത സൈന്യങ്ങളെയും കൊണ്ട് നാം അവരുടെ അടുത്ത് ചെല്ലുകയും നിന്ദ്യരും അപമാനിതരുമായ നിലയില്‍ അവരെ നാം അവിടെനിന്ന് പുറത്താക്കുകയും ചെയ്യുന്നതാണ്. അദ്ദേഹം (സുലൈമാന്‍) പറഞ്ഞു: ഹേ പ്രമുഖന്മാരേ, അവര്‍ കീഴൊതുങ്ങിക്കൊണ്ട് എന്റെ അടുക്കല്‍ വരുന്നതിന് മുമ്പായി നിങ്ങളില്‍ ആരാണ് അവളുടെ സിംഹാസനം എനിക്ക് കൊണ്ടുവന്ന് തരിക? ജിന്നുകളുടെ കൂട്ടത്തിലെ ഒരു മല്ലന്‍ പറഞ്ഞു: അങ്ങ് അങ്ങയുടെ ഈ സദസ്സില്‍നിന്ന് എഴുന്നേല്‍ക്കുന്നതിന് മുമ്പായി ഞാന്‍ അത് അങ്ങേക്ക് കൊണ്ടുവന്ന് തരാം. തീര്‍ച്ചയായും ഞാന്‍ അതിന് കഴിവുള്ളവനും വിശ്വസ്തനുമാകുന്നു. വേദത്തില്‍നിന്നുള്ള വിജ്ഞാനം കരസ്ഥമാക്കിയിട്ടുള്ള ആള്‍ പറഞ്ഞു: താങ്കളുടെ ദൃഷ്ടി താങ്കളിലേക്ക് തിരിച്ചുവരുന്നതിന് മുമ്പായി ഞാന്‍ അത് താങ്കള്‍ക്ക് കൊണ്ടുവന്ന് തരാം. അങ്ങനെ അത് (സിംഹാസനം) തന്റെ അടുക്കല്‍ സ്ഥിതിചെയ്യുന്നതായി കണ്ടപ്പോള്‍ അദ്ദേഹം പറഞ്ഞു: ഞാന്‍ നന്ദി കാണിക്കുമോ അതല്ല നന്ദികേട് കാണിക്കുമോ എന്ന് എന്നെ പരീക്ഷിക്കുവാനായി എന്റെ രക്ഷിതാവ് എനിക്ക് നല്‍കിയ അനുഗ്രഹത്തില്‍പെട്ടതാകുന്നു ഇത്. വല്ലവനും നന്ദികാണിക്കുന്നപക്ഷം തന്റെ ഗുണത്തിനായിട്ട് തന്നെയാകുന്നു അവന്‍ നന്ദി കാണിക്കുന്നത്. വല്ല വനും നന്ദികേട് കാണിക്കുന്നപക്ഷം തീര്‍ച്ചയായും എന്റെ രക്ഷിതാവ് പരാശ്രയമുക്തനും ഉല്‍കൃഷ്ടനുമാകുന്നു. അദ്ദേഹം (സുലൈമാന്‍) പറഞ്ഞു: നിങ്ങള്‍ അവളുടെ സിംഹാസനം അവള്‍ക്ക് തിരിച്ചറിയാത്ത വിധത്തില്‍ മാറ്റുക. അവള്‍ യാഥാര്‍ത്ഥ്യം മനസിലാക്കുമോ അതല്ല അവള്‍ യാഥാര്‍ത്ഥ്യം കണ്ടെത്താത്തവരുടെ കൂട്ടത്തിലായിരിക്കുമോ എന്ന് നമുക്ക് നോക്കാം. അങ്ങനെ അവള്‍ വന്നപ്പോള്‍ (അവളോട്) ചോദിക്കപ്പെട്ടു: താങ്കളുടെ സിംഹാസനം ഇതുപോലെയാണോ? അവള്‍ പറഞ്ഞു: ഇത് അത് തന്നെയാണെന്ന് തോന്നുന്നു. ഇതിന് മുമ്പുതന്നെ ഞങ്ങള്‍ക്ക് അറിവ് നല്‍കപ്പെട്ടിരുന്നു. ഞങ്ങള്‍ മുസ്‌ലിംകളാവുകയും ചെയ്തിരുന്നു. അല്ലാഹുവിന് പുറമെ അവള്‍ ആരാധിച്ചിരുന്നതില്‍നിന്ന് അദ്ദേഹം അവളെ തടയുകയും ചെയ്തു. തീര്‍ച്ചയായും അവള്‍ സത്യനിഷേധികളായ ജനതയില്‍പെട്ടവളായിരുന്നു. കൊട്ടാരത്തില്‍ പ്രവേശിച്ചുകൊള്ളുക എന്ന് അവളോട് പറയപ്പെട്ടു. എന്നാല്‍ അവള്‍ അത് കണ്ടപ്പോള്‍ അത് ഒരു ജലാശയമാണെന്ന് വിചാരിക്കുകയും തന്റെ കണങ്കാലുകളില്‍നിന്ന് വസ്ത്രം മേലോട്ട് നീക്കുകയും ചെയ്തു. സുലൈമാന്‍ പറഞ്ഞു: ഇത് സ്ഫടിക കഷണങ്ങള്‍ പാകിമിനുക്കിയ ഒരു കൊട്ടാരമാകുന്നു. അവള്‍ പറഞ്ഞു: എന്റെ രക്ഷിതാവേ, ഞാന്‍ എന്നോട് തന്നെ അന്യായം ചെയ്തിരിക്കുന്നു. ഞാനിതാ സുലൈമാനോടൊപ്പം ലോകരക്ഷിതാവായ അല്ലാഹുവിന് കീഴ്‌പ്പെട്ടിരിക്കുന്നു'' (വി.ഖു. 27:20-44)

സോളമന്റെകാലത്ത് ശേബായിലെ രാജ്ഞി അദ്ദേഹത്തിന്റെയടുക്കല്‍ ചെന്നതും അദ്ദേഹത്തിന്റെ ജ്ഞാനവും പ്രതാപവും കണ്ട് വളരെയേറെ പ്രശംസിച്ചതുമായി ബൈബിള്‍ പഴയ നിയമത്തിലുമുണ്ട്. (1 രാജാക്കന്മാര്‍ 10:1-13; 2 ദിനവൃത്താന്തം 9:1-12). എന്നാല്‍ ഖുര്‍ആനിലേതുപോലെയുള്ള വിശദമായ പരാമര്‍ശങ്ങള്‍ ബൈബിളിലെവിടെയുമില്ല. എന്നാല്‍ ചില യഹൂദ തര്‍ഗൂമുകളില്‍ ഈ കഥ ഏകദേശം ഖുര്‍ആനിലേതിന് തുല്യമായ രീതിയില്‍ വിവരിച്ചിട്ടുണ്ട്. തര്‍ഗുമുകളിലെ ഈ വിവരണങ്ങളില്‍നിന്ന് മുഹമ്മദ് നബി (സ) പകര്‍ത്തിയെഴുതിയതാണ് സുലൈമാന്‍-ബില്‍ഖീസ് രാജ്ഞി കഥയെന്നാണ് വിമര്‍ശകരുടെ വാദം.

'തര്‍ഗൂം' എന്ന അരമായ പദത്തിനര്‍ത്ഥം വിവര്‍ത്തനം അല്ലെങ്കില്‍ വ്യാഖ്യാനമെന്നാണ്. പഴയ നിയമഗ്രന്ഥങ്ങള്‍ക്ക് യഹൂദരുടെ ഇടയില്‍ പ്രചാരത്തിലായ അരമായ വിവര്‍ത്തനങ്ങളോ പരാവര്‍ത്തനങ്ങളോ ആണ് തര്‍ഗൂമുകള്‍ എന്ന് അറിയപ്പെടുന്നത്. പല തര്‍ഗൂമുകളും വിവര്‍ത്തനങ്ങള്‍ എന്നതിനേക്കാളുപരി വ്യാഖ്യാനങ്ങളാണ് എന്നാണ് എന്‍സൈക്ലോപീഡിയ ജൂദായിക്ക പറയുന്നത്. പഴയ നിയമത്തെക്കുറിച്ച് സൂക്ഷ്മമായി പഠിക്കുവാനും വിവിധ കാലങ്ങളില്‍ നിലനിന്നിരുന്ന വ്യാഖ്യാന രീതികള്‍ മനസ്സിലാക്കുവാനും തര്‍ഗൂമുകള്‍ പ്രയോജനപ്പെടുന്നു.

'എസ്‌തേറിന്റെ പുസ്തകത്തിനുള്ള രണ്ടാം തര്‍ഗൂം ആയ തര്‍ഗൂം ഷെനി' (Targum Sheni) യില്‍ സോളമനും ശേബാരാജ്ഞിയും തമ്മില്‍ നടന്ന കൂടിക്കാഴ്ചയെക്കുറിച്ച് വിവരിക്കുന്നുണ്ട്. ഈ വിവരണമാകട്ടെ, ഏകദേശം ഖുര്‍ആനിലേതിന് സമാനമാണ് താനും. ഇതില്‍നിന്ന് തര്‍ഗൂം ഷെനിയില്‍നിന്ന് മുഹമ്മദ് നബി (സ)പകര്‍ത്തിയെഴുതിയതാണ് സോളമന്‍-ബില്‍ഖീസ് കഥയെന്ന് പറയാനാകുമോ?

യഹൂദ പുരോഹിതന്മാര്‍ക്ക് മാത്രം പ്രാപ്യമായ താര്‍ഗൂമുകള്‍ പോലും സൂക്ഷ്മമായി പരിശോധിച്ച് അവയില്‍നിന്ന് പകര്‍ത്തിയെഴുതുവാന്‍ നിരക്ഷരനായ മുഹമ്മദ് നബി (സ) ക്ക് കഴിഞ്ഞുവെന്ന വാദംതന്നെ ശുദ്ധ അസംബന്ധമാണ്. അങ്ങനെ ചെയ്തിരുന്നുവെങ്കില്‍ അക്കാലത്തെ യഹൂദ പുരോഹിതന്മാര്‍ ആരെങ്കിലും പ്രസ്തുത ആരോപണം ഉന്നയിക്കേണ്ടതായിരുന്നു. എന്നാല്‍ ഈ ആരോപണ ത്തിന് ഒന്നര നൂറ്റാണ്ടിലേറെ പഴക്കമില്ലെന്നതാണ് വാസ്തവം.

ഉപലബ്ധമായ തര്‍ഗൂമുകളില്‍ ഏറ്റവും പഴക്കമുള്ളതിന്റെ കാലം ക്രിസ്താബ്ദം 700 നടുത്തായിരിക്കുമെന്നാണ് എന്‍സൈക്ലോപീഡിയ ജൂദായിക്ക പറയുന്നത് ("Targums" CD. Rom Edition) തര്‍ഗൂം ഷെനിയാകട്ടെ എട്ടാം നൂറ്റാണ്ടില്‍ രചിക്കപ്പെട്ടതാണെന്നാണ് പണ്ഡിതാഭിപ്രായം (Ibid)

മുഹമ്മദ് നബിക്ക് ശേഷം പതിറ്റാണ്ടുകള്‍ കഴിഞ്ഞാണ് തര്‍ഗൂം ഷെനി രചിക്കപ്പെട്ടതെന്നര്‍ത്ഥം.

മുഹമ്മദ് നബി (സ) ക്ക് ശേഷം രചിക്കപ്പെട്ട തര്‍ഗൂമില്‍നിന്ന് കോപ്പിയടിച്ചാണ് അദ്ദേഹം ഖുര്‍ആന്‍ രചിച്ചതെന്ന വാദം എന്തുമാത്രം ബാലിശമാണ്!

യഥാര്‍ത്ഥത്തില്‍, തര്‍ഗൂം രചയിതാക്കളാണ് ഖുര്‍ആനിലെ സോളമന്‍-ശേബാരാജ്ഞി കഥയില്‍നിന്ന് കടമെടുത്തത് എന്നാണ് മനസ്സിലാക്കാന്‍ കഴിയുന്നത്. ഖുര്‍ആനിനുശേഷം എഴുതപ്പെട്ട ഒരു കൃതിയില്‍ ഇത്തരമൊരു കടമെടുക്കല്‍ നടന്നിരിക്കാനുള്ള സാധ്യത ഒട്ടും നിഷേധിക്കാനാവില്ല. ഇക്കാര്യം എന്‍സൈക്ലോപീഡിയ ജൂതായിക്കതന്നെ സമ്മതിക്കുന്നുമുണ്ട്. ''ഇതിലെ ചില പ്രസക്ത ഭാഗങ്ങള്‍ ഖുര്‍ആനിലും കാണപ്പെടുന്നുണ്ട്. (27:20-40). ഇതില്‍നിന്ന് ഈ തര്‍ഗൂമിന്റെ രചയിതാവ് അറബി സ്രോതസ്സുകളെയും തന്റെ രചനക്ക് ഉപയോഗിച്ചിട്ടുണ്ടെന്നാണ് മനസിലാവുന്നത്'' (``Targum Sheni'', Encyclopaedia Judaica CD-Rom Edition)  യഹൂദ പുരാണങ്ങളെ അവലംബിച്ചുകൊണ്ട് മുഹമ്മദ് നബി (സ) മെനഞ്ഞുണ്ടാക്കിയ കഥയാണ് സുലൈമാന്‍-ബില്‍ഖീസ് കഥയെന്ന വിമര്‍ശകരുടെ വാദം തകരുക മാത്രമല്ല; പ്രത്യുത ഖുര്‍ആന്‍ ദൈവികമാണെന്ന വസ്തുത ഒരിക്കല്‍കൂടി ബോധ്യപ്പെടുക കൂടിയാണ് ഇവിടെ ചെയ്യുന്നത്. യഹൂദ തര്‍ഗൂമുകള്‍ രചിക്കപ്പെട്ടത് ഖുര്‍ആനിന് ശേഷമാണെന്ന വസ്തുത നാം മനസ്സിലാക്കി. ഖുര്‍ആനിലും തര്‍ഗൂമുകളിലുമൊഴിച്ച് മറ്റെവിടെയും ഈ കഥ വിശദാംശങ്ങളോടെ കാണുന്നുമില്ല. മുഹമ്മദ് നബിയാണ് ഖുര്‍ആന്‍ രചിച്ചതെങ്കില്‍ അദ്ദേഹത്തിന് ഈ കഥയെവിടെനിന്നുകിട്ടി? പൂര്‍വകാല ചരിത്രത്തെക്കുറിച്ച് ശരിക്കറിയാവുന്ന സര്‍വ്വശക്തന്റെ വചനങ്ങളാണ് ഖുര്‍ആനെന്ന വസ്തുതയാണ് ഇവിടെയും വ്യക്തമായി വെളിപ്പെടുന്നത്.

 

മൂസാ (അ) യും ഖിള്‌റും (അ)  തമ്മില്‍ നടന്ന സംഭാഷണങ്ങളും പിന്നീടുണ്ടായ സംഭവങ്ങളുമെല്ലാം സാമാന്യം വിശദമായിത്തന്നെ ഖുര്‍ആനിലെ സൂറത്തുല്‍ കഹ്ഫില്‍ (18:65-82) വിവരിക്കുന്നുണ്ട്. ഏലിജായും യോശുവ ബെന്‍ ലെവിയെന്ന റബ്ബിയും കൂടി നടത്തിയതായി യഹൂദ ഐതിഹ്യത്തില്‍ പറയുന്ന യാത്രയ്ക്കും സംഭവങ്ങ ള്‍ക്കും മൂസാ-ഖിള്ര്‍ സംഭവത്തെക്കുറിച്ച ഖുര്‍ആനിക വിശദീകരണങ്ങളുമായി സാമ്യമുണ്ടെന്നും അതുകൊണ്ട് യഹൂദ ഐതിഹ്യ ത്തില്‍നിന്ന് കടമെടുത്തുകൊണ്ട് മുഹമ്മദ് നബി (സ) കെട്ടിച്ചമച്ചുണ്ടാക്കിയ ഒരു കഥയാണിതെന്നുമാണ് വാദം. Jellinek, Betha-Midrasch, V, 1335ല്‍ ഈ ഐതിഹ്യം വിശദീകരിച്ചിട്ടുണ്ടെന്നും, Zunz, Gesmmelt Vortrage, X, 130ലാണ് ഇതും ഖുര്‍ആനിക കഥയും തമ്മില്‍ സാമ്യമുണ്ടെന്ന് ആദ്യമായി സൂചിപ്പിച്ചിട്ടുള്ളതെന്നും ഓറിയന്റലിസ്റ്റുകളുടെ രചനയായ Encyclopedia of Islam (Page 903 Under the title ``Al-Khidr'')ല്‍ പറയുന്നുണ്ട്. ചരിത്ര വസ്തുതകളും പ്രമാണങ്ങളും സൂക്ഷ്മ പരിശോധനയ്ക്ക് വിധേയമാക്കിയാല്‍ ഈ വാദം അടിസ്ഥാനരഹിതമാണെന്ന് ബോധ്യമാകും:

(1) യഹൂദന്മാര്‍ക്കിടയില്‍ മുഹമ്മദ് നബി (സ) യുടെ കാലത്ത് ഇത്തരം ഒരു ഐതിഹ്യം നിലനില്‍ക്കുകയും അതില്‍ ചില മാറ്റങ്ങള്‍ വരുത്തി മുഹമ്മദ് നബി (സ) ഖുര്‍ആനിലൂടെ അവതരിപ്പിക്കുകയും ചെയ്തിരുന്നുവെങ്കില്‍ അന്നുണ്ടായിരുന്ന യഹൂദന്മാര്‍ ഇക്കാര്യം എടുത്തുപറയുകയും നബി (സ) യെ വിമര്‍ശിക്കുകയും ചെയ്യുമായിരുന്നു. അങ്ങനെ യാതൊന്നും സംഭവിച്ചതായി രേഖപ്പെടുത്തപ്പെട്ടിട്ടില്ല.

(2) യഹൂദ മതത്തില്‍നിന്ന് ഇസ്‌ലാം സ്വീകരിച്ച ഒട്ടനവധി പ്രവാചക ശിഷ്യന്മാരുണ്ടായിരുന്നു. തങ്ങള്‍ കേട്ടുവളര്‍ന്ന ഒരു ഐതിഹ്യം ഏതാനും മാറ്റങ്ങളോടെ അവതരിപ്പിച്ച രീതിയാണ് മൂസാ-ഖിള്ര്‍ (അ)  സംഭവ വിവരണത്തിന്റെ കാര്യത്തില്‍ ഖുര്‍ആനിലുള്ളതെ ങ്കില്‍ അവര്‍ ഇക്കാര്യം സൂചിപ്പിക്കുകയും പ്രവാചകനുമായും മറ്റു ഹാബിമാരുമായും ഇത് ചര്‍ച്ച നടത്തുകയും ചെയ്യുമായിരുന്നു. അത്തരം യാതൊരു സംഭവവും രേഖപ്പെടുത്തപ്പെട്ടിട്ടില്ലാത്തതിനാല്‍തന്നെ അക്കാലത്തെ യഹൂദര്‍ക്കിടയില്‍ ഇത്തരമൊരു ഐതിഹ്യം പ്രചാരത്തിലില്ലായിരുന്നുവെന്ന് വ്യക്തമാകുന്നുണ്ട്.

(3) പതിനൊന്നാം നൂറ്റാണ്ടിന് മുമ്പ് നിലനിന്ന യാതൊരു യഹൂദ രേഖയിലും ഏലിജാ-യോശുവാ ഐതിഹ്യം പ്രതിപാദിക്കുന്നില്ല. (Brannon-M. Wheeler: ``The Jewish Origins of Quran IB: 65-82? Re examining Arent Jan Wensinck's Theory'': Journel of the American Oriental Society Vol 118, Page 115). മുഹമ്മദ് നബി (സ) യുടെ കാലത്ത് ഇത് പ്രചാരത്തിലിരുന്നെങ്കില്‍ ഇതിനേക്കാള്‍ പ്രാചീനമായ യഹൂദ രേഖകളില്‍ ഈ കഥ കാണേണ്ടതായിരുന്നു.

(4) ഖുര്‍ആനില്‍ പറഞ്ഞ മൂസാ-ഖിള്ര്‍ സംഭവവും യഹൂദ ഐ തിഹ്യങ്ങളും സൂക്ഷ്മ പഠനത്തിന് വിധേയമാക്കിയശേഷം ഓറിയന്റലിസ്റ്റായ ബ്രന്നോന്‍ എം. വീലര്‍ എത്തിച്ചേരുന്ന നിഗമനമിങ്ങനെയാണ്: ''ഈ പണ്ഡിതരോ (യൂദ ഐതിഹ്യത്തില്‍നിന്ന് കോപ്പിയടിച്ചതാണ് മൂസാ-ഖിള്ര്‍ കഥയെന്ന് വാദിക്കുന്നവര്‍) വെന്‍സില്‍ക്കോ Hibbur Yafeh me-ka-yeshuaയുടെ തലക്കെട്ടിന് കീഴില്‍ നല്‍കിയിട്ടുള്ള ഈ കഥ പതിനൊന്നാം നൂറ്റാണ്ടിലെ ഖൈറവാന്‍കാരനായ നി സ്സിം ബിന്‍ ഷഹിനിന്റെ പേരിലുള്ള ഒരു അറബി രചനയുടെ ഹിബ്രു പരാവര്‍ത്തനം മാത്രമാണെന്ന വസ്തുത ശ്രദ്ധിച്ചിട്ടേയില്ല....... ഇതിന്റെ അറബി ഒറിജിനല്‍ കണ്ടെത്തിയശേഷവും, പതിനൊന്നാം നൂറ്റാണ്ടിന് മുമ്പുള്ള ഒരു ഹിബ്രു സ്രോതസ്സും ഈ കഥയുള്‍ക്കൊള്ളുന്നില്ലെന്ന വസ്തുത പരിഗണിക്കാതെ ഇതിനെയാണ് ഖുര്‍ആന്‍ ആശ്രയിച്ചതെന്ന് പണ്ഡിതന്മാര്‍ ആവര്‍ത്തിച്ച് പറഞ്ഞുകൊണ്ടിരിക്കുകയാണ്....... ഏലിജയുടെയും യോശുവ ബിന്‍ലെവിയുടെയും കഥയെ ആശ്രയിച്ച് എഴുതപ്പെട്ടതല്ല ഖുര്‍ആന്‍ 18:65-82 എന്ന് ലഭ്യമായ തെളിവുകളെല്ലാം വ്യക്തമാക്കുന്നുണ്ട്. ഖുര്‍ആന്‍ 18:65-82ന്റെ വ്യാഖ്യാനങ്ങളെ, വിശേഷിച്ചും ഉബയ്യുബ്‌നു കഅ്ബിന്റെ കഥയെയും അതിന്റെ പില്‍ക്കാല വിശദീകരണങ്ങളെയും ആശ്രയിച്ചുകൊണ്ടാണോ ഇബ്‌നു ഷാഹിനിന്റെ കഥ രചിക്കപ്പെട്ടിട്ടുള്ളതെന്ന കാര്യം ഇപ്പോഴും ഒരു പ്രശ്‌നമായിത്തന്നെ അവശേഷിക്കുന്നു. ഖുര്‍ആനും ഉബയ്യുബ്‌നു കഅ്ബിന്റെ കഥയുടെ മൂലരൂപവുമായി താരതമ്യം ചെയ്യുമ്പോള്‍, ഇബ്‌നു ഷാഹിന്റെ രചന പുതിയതും ആദ്യകാല ഇസ്‌ലാമിക സ്രോതസ്സുകളുമായി യോജിക്കുന്ന നിരവധി കാര്യങ്ങള്‍ ഉള്‍ക്കൊള്ളുന്നതുമാണ്. ഏലീജയുടെയും യോ ശുവ-ബിന്‍ലെവിയുടെയും കഥയില്‍ ഖുര്‍ആന്‍ 18:65-82ലില്ലാത്തതും ഈ വചനങ്ങളുടെ വ്യാഖ്യാന ഗ്രന്ഥങ്ങളിലുള്ളതുമായ പല കാര്യങ്ങളുടെയും പ്രതിഫലനങ്ങളുണ്ട്. ഖിള്‌റിനുപകരം ഏലീജായെ ഉപയോഗിക്കുവാന്‍ ഇബ്‌നുഷാഹിനെ പ്രേരിപ്പിച്ചത് ഇസ്‌ലാമിക സ്രോതസ്സുകളില്‍ ഈ രണ്ട് വ്യക്തിത്വങ്ങളും തമ്മിലുള്ള അടുത്ത ബന്ധമായിരിക്കാമെന്ന് വ്യാഖ്യാനിക്കാവുന്നതാണ്''. (Ibid Page 155-171) ഖുര്‍ആനിക കഥയുടെ അടിസ്ഥാനത്തില്‍ പതിനൊന്നാം നൂറ്റാണ്ടിലെ ഏതോ യഹൂദന്റെ മനസ്സില്‍ രൂപംകൊണ്ട ഐതിഹ്യമെടുത്ത് പൊക്കി ഈ ഐതിഹ്യത്തിന്റെ അടിസ്ഥാനത്തില്‍ മുഹമ്മദ് നബി (സ) പടച്ചുണ്ടാക്കിയതാണ് മൂസാ-ഖിള്‌റ് സംഭവമെന്ന് വാദിക്കുന്നവര്‍ സ്വന്തം കണ്ണുപൊട്ടിച്ച് അന്ധനാകാന്‍ ശ്രമിക്കുന്നവനെപ്പോലെയാണെന്ന് പറയാതിരിക്കാന്‍ നിര്‍വ്വാഹമില്ല.

യേശുവിനെക്കുറിച്ച് സമകാലികരായിരുന്ന ചരിത്രകാരന്മാരൊന്നും രേഖപ്പെടുത്താത്തതിനാല്‍ അങ്ങനെയൊരു വ്യക്തി ജീവി ച്ചിരുന്നിട്ടേയില്ലയെന്ന് വാദിക്കുന്ന യുക്തിവാദികളുണ്ട്. ജോസിഫസ് എന്ന യൂദ ചരിത്രകാരന്‍ ക്രിസ്തുവിനെക്കുറിച്ച് പറഞ്ഞതായി ക്രിസ്തുമത പ്രചാരകന്മാര്‍ ഉദ്ധരിക്കാറുണ്ടെങ്കിലും പ്രസ്തുത പ്രസ്താവനകള്‍ ജോസിഫസിന്റെ ഗ്രന്ഥത്തില്‍ കൂട്ടിച്ചേര്ത്തസതാണെന്നാണ് വിമര്ശികര്‍ വാദിക്കുന്നത്. അത് എന്തായിരുന്നാലും, ക്രൈസ്തവരെ സംബന്ധിച്ചിടത്തോളം യേശുവിന്റെ ജനനം, പ്രബോധനങ്ങള്‍, മരണം, എന്നിവയെക്കുറിച്ച് അറിവു നല്കുനന്ന ഗ്രന്ഥങ്ങള്‍ നാലു സുവിശേഷങ്ങളാണ്. ഖുര്ആിനില്‍ യേശുവിനെക്കുറിച്ച് ഒരുപാട് കാര്യങ്ങള്‍ പറയുന്നുണ്ടെങ്കിലും ക്രൈസ്തവര്ക്ക്് അത് അസ്വീകാര്യമാണല്ലോ. അപ്പോള്‍ യേശുവിനെക്കുറിച്ച് ക്രൈസ്തവരുടെ അറിവ് സുവിശേഷങ്ങളെ ആശ്രയിച്ചാണിരിക്കുന്നത്. സുവിശേഷങ്ങള്‍ ദൈവനിവേശിതങ്ങളായതിനാല്‍ അവ നല്കുുന്ന വിവരങ്ങള്‍ നൂറു ശതമാനം സത്യസന്ധവും സ്വീകരിക്കാന്‍ കൊള്ളുന്നതുമാ ണെന്നായിരുന്നു പൊതുവായ ക്രൈസ്തവ കാഴ്ചപ്പാട്. ബൈബിള്‍ നല്കുാന്ന യേശുചിത്രം ചരിത്രപരവും വസ്തുനിഷ്ഠവുമാണെന്നായിരുന്നു വിശ്വാസം.

പുതിയ നിയമത്തെക്കുറിച്ച ഗവേഷണങ്ങള്‍ ഈ പരമ്പരാഗത ധാരണയ്ക്ക് ഇളക്കം തട്ടിച്ചിട്ടുണ്ട്. യേശുവിന്റെ ചരിത്രവുമായി ബന്ധപ്പെട്ട് സുവിശേഷകര്ത്തായക്കള്‍ ഉദ്ധരിക്കുന്ന സംഭവങ്ങളില്‍ പലതും ഭാവനാസൃഷ്ടിയാണെന്നാണ് ഗവേഷകര്‍ അഭിപ്രായപ്പെടുന്നത്. ക്രിസ്തുവിന്റെ ശിശുകഥയുമായി ബന്ധപ്പെട്ട് സുവിശേഷങ്ങള്‍ വിവരിക്കുന്ന കാര്യങ്ങള്‍ എത്രത്തോളം ചരിത്രപരമാണെന്ന് ലഭ്യമായ തെളിവുകളുടെ അടിസ്ഥാനത്തില്‍ പ്രഗത്ഭ ബൈബിള്‍ പണ്ഡിതനായ റെയ്മണ്ട്-ഇ-ബ്രൗണ്‍ തന്റെ 'മിശിഹായുടെ ജനനം' (The birth of Messiah) എന്ന ഗ്രന്ഥത്തില്‍ വിശദമായി പരിശോധിക്കുന്നുണ്ട്. ഫാദര്‍.ഫ്രാന്സി( കൊടിയന്‍, ഫാ.ജോസ് മാണപ്പറമ്പില്‍, ഫാ:വര്ഗീെസ് പെരേപ്പാടന്‍ തുടങ്ങിയവര്‍ ചേര്ന്ന് വിവര്ത്ത നം ചെയ്തിരിക്കുന്ന 401 പേജുകളുള്ള ഈ പുസ്തകത്തിന്റെ മലയാള പരിഭാഷ അലൂരിലെ ബിബ്‌ളിയ പബ്ലിക്കേഷന്സ്ന ആണ് പ്രസിദ്ധീകരിച്ചിരിക്കുന്നത്. ഇരുപതിലേറെ ഓണററി ഡോക്റ്ററേറ്റുകളുള്ള അമേരിക്കന്‍ ബൈബിള്‍ പണ്ഡിതനായ റെയ്മണ്ട്- ഇ-ബ്രൗണ്‍ വിശദമായ പഠനങ്ങള്ക്ക് ശേഷം എത്തിച്ചേരുന്ന നിഗമ നങ്ങള്‍ ഇങ്ങനെ സംക്ഷേപിക്കാം.

(1) മത്തായിയും ലൂക്കോസും നല്കുിന്ന യേശുവിന്റെ വംശാവലി കൃത്യമല്ല. വരാനിരിക്കുന്ന മിശിഹാ ദാവീദു വംശജനായിരിക്കുമെന്നായിരുന്നു യഹൂദവിശ്വാസം. യേശു മിശിഹയാണെങ്കില്‍ ദാവീദ് വംശജനാകണമല്ലോ. അതിനു വേണ്ടി ദാവീദിന്റെ വംശാവലിയില്‍ യേശുവിനെ തിരുകിക്കയറ്റി അവതരിപ്പിക്കുകയാണ് സുവിശേഷകന്മാര്‍ ചെയ്തിരിക്കുന്നത്. 'മത്തായിയും ലൂക്കായും തരുന്ന വംശാവലി കുടുംബത്തിന്‍േറതാണ് എന്ന നിഗമനത്തെ മുകളില്‍ കണ്ട പ്രശ്നങ്ങളുടെ വെളിച്ചത്തില്‍ മിക്ക ആധുനിക ചിന്തകരും തള്ളിക്കളയുന്നുണ്ട്. ജനകീയമായിരുന്ന ദാവീദ് രാജപരമ്പരയുടെ അന്ത്യത്തില്‍, യൗസേപ്പിന്റെയും യേശുവിന്റെയും പേരുകള്‍ കൂട്ടിച്ചേര്ത്തുതാണ് മത്തായി നല്കുിന്ന വംശാവലി; ലൂക്കോയുടെതാകട്ടെ യൗസേപ്പിന്റെ പൂര്വ്വി കരുടെ കുടുംബപട്ടികയും. ഇതാണ് ആധുനിക നിഗമനം'1

(2) ഇസ്രായീലിനെ ഫറോവയില്‍ നിന്നു രക്ഷിച്ച മോശയുടെ ചരിത്രത്തോട് സാമ്യപ്പെടുത്തിക്കൊണ്ടാണ് മത്തായി തന്റെ യേശുകഥ നിര്മിപക്കുന്നത്. അതിനാല്‍ യേശുവിന്റെ ജീവിതത്തില്‍ സംഭവിക്കാത്ത പലതും മത്തായിയുടെ യേശുകഥയിലുണ്ട്. 2

(3) പഴയനിയമത്തില്‍ പ്രവചിക്കപ്പെട്ട പ്രവാചകനാണ് യേശുവെന്ന് വരുത്തിത്തീര്ക്കു ന്നതിനു വേണ്ടിയാണ് മത്തായി ശ്രമിച്ചിരിക്കുന്നത്. അതിനാല്‍ പഴയനിയമ പ്രവചനങ്ങള്ക്ക്നുസൃതമായി യേശു കഥ മെനഞ്ഞെടുക്കുകയാണ് അദ്ദേഹം ചെയ്തിരിക്കുന്നത്.3

(4) യേശു ജനിച്ച കാലഘട്ടത്തിലെ മുഴുവന്‍ ജ്യേതിശാസ്ത്ര രേഖകളും പരിശോധിച്ചാലും മത്തായി 2:1-12ല്‍ പറയുന്ന രീതിയില്‍ യേശു ജനനത്തോടനുബന്ധിച്ച് ബെത്‌ലഹേമിന് മുകളില്‍ ഒരു നക്ഷത്രം പ്രത്യക്ഷപ്പെട്ടുവെന്ന വിവരണത്തിന് ഉപോല്ബകലകമായ ഒരു തെളിവുപോലും ലഭിക്കുകയില്ല. (ധൂമകേതുക്കള്‍, ഗ്രഹങ്ങളുടെ സംഗമം, നവജാതനക്ഷത്രങ്ങള്‍ തുടങ്ങിയവയെക്കുറിച്ച ജ്യോതിശാ സ്ത്ര രേഖകളെല്ലാം റെയ്മണ്ട് ബ്രൗണ്‍ പരിശോധിക്കുന്നുണ്ട്). ബെത്‌ലഹേമിനു മുകളില്‍ യേശുവിന്റെ ജനനത്തോടനുബന്ധിച്ച് ഒരു നക്ഷത്രം പ്രത്യക്ഷപ്പെടുകയും അതുകണ്ട് പൗരസ്ത്യ ദേശ ത്തു നിന്ന് ജ്ഞാനികള്‍ ഹെറോദോസ് രാജാവിനോട് യഹൂദ രാജാവിനെക്കുറിച്ച് അന്വേഷിക്കുകയും അങ്ങനെ യേശു ജനിച്ച സ്ഥലത്തെത്തിച്ചേരുകയും ചെയ്തുവെന്ന കഥ മത്തായി മെനഞ്ഞെടുത്ത താണ്. അത് ചരിത്രപരമല്ല.

ഈ കഥ ചരിത്രപരമല്ലെന്നതിന് മറ്റു പുതിയനിയമഭാഗങ്ങളും തെളിവു നല്കുുന്നുവെന്നാണ് റെയ്മണ്ട് ബ്രൗണ്‍ പറയുന്നത്. അദ്ദേഹം എഴുതുന്നു: 'മത്തായി രണ്ടാമധ്യായമനുസരിച്ച് ജ്ഞാനികള്‍ ഹേറോദോസിനെ സന്ദര്ശിാച്ചപ്പോഴാണ് അദ്ദേഹവും പ്രധാനപുരോഹിതന്മാരും പശീശന്മാരുമെല്ലാം യൂദരുടെ രാജാവിന്റെ ജനനത്തെപ്പറ്റി അറിഞ്ഞത്. ജറൂസലം മുഴുവന്‍ ആസംഭവത്തെ സംബന്ധിച്ച് അസ്വസ്ഥമാവുകയും ചെയ്തു. എന്നിട്ടുപോലും യേശു തന്റെ പരസ്യശുശ്രൂഷ ആരംഭിച്ചപ്പോള്‍ അവിടുത്തെപ്പറ്റി ആര്ക്കെലങ്കിലും കാര്യമായ അറിവോ പറയത്തക്ക പ്രതീക്ഷകളോ ഉണ്ടായിരുന്നതായി കാണുന്നില്ല. (മത്തായി 13:54-56) ഹെറോദോസിന്റെ മകന്‍ അന്തിപ്പാസിനാകട്ടെ യേശുവിനെപ്പറ്റി യാതൊരു വിവരവുണ്ടായിരുന്നില്ല. (ലൂക്കോസ് 8:7-9). ലൂക്കോസിന്റെ സുവിശേഷത്തില്‍ സ്‌നാപക യോഹന്നാന്റെ അമ്മയായ ഏലിശാ യേശുവിന്റെ അമ്മയായ മറിയത്തിന്റെ ബന്ധുവാണ്. തന്മൂലം അവരുടെ കുട്ടികള്‍ തമ്മിലും ബന്ധമുണ്ടല്ലോ. ഇതെല്ലാമായിട്ടും പരസ്യശുശ്രൂഷാകാലത്ത് യേശുവും സ്‌നാപകയോഹന്നാനും ബന്ധക്കാരാണെന്നതിനെപ്പറ്റി ഒരു സൂചനയും സുവിശേഷത്തിലില്ല. പോരെങ്കില്‍ യോഹന്നാന്‍1:33 ല്‍ സ്‌നാപകന്‍ ''ഞാന്‍ അവനെ അറിഞ്ഞിരുന്നില്ല'' എന്നു പറയുന്നുണ്ട്. ശൈശവ വിവരണങ്ങളുടെ ചരിത്രപരതയെപ്പറ്റി സംശയമുണര്ത്തുലന്ന സുവിശേഷ ഭാഗങ്ങളില്‍ ചിലതുമാത്രമാണിവ'.

(5) യേശു ജനിച്ചത് യഥാര്ത്ഥ ത്തില്‍ ബെത്‌ലഹേമിലല്ല. ദാവീദി ന്റെ പുത്രനായ മിശിഹ ബെത്‌ലഹേമിലാണ് ജനിക്കുകയെന്ന യഹൂദ പാരമ്പര്യത്തിനനുസൃതമായി യേശുകഥ മെനഞ്ഞെടുത്ത മത്തായിയും ലൂക്കോസും ബെത്‌ലഹേമിലാണ് യേശു ജനിച്ചതെന്ന് വരുത്തി ത്തീര്ക്കു്കയാണ് ചെയ്തിരിക്കുന്നത്.

'യഹൂദ ചിന്തയനുസരിച്ച് ദാവീദാത്മജനായ മിശിഹാ ദാവീദി ന്റെ പട്ടണമായ ബെത്‌ലഹേമില്‍ ജനിക്കണം. അതിനാല്‍ ഈശോ ജനിച്ചത് ബെത്‌ലഹേമിലാണ് എന്ന കഥ മെനഞ്ഞെടുത്തു'

'രണ്ടാമതായി, മറ്റു പുതിയ നിയമഭാഗങ്ങള്‍ ഈശോയുടെ ജനന സ്ഥലം ബേത്‌ലഹേമാണെന്നതില്‍ നിശ്ശബ്ദമാണെന്നു മാത്രമല്ല നസ്രത്തും ഗലീലിയും ആണ് ഇശോയുടെ സ്വന്തം പട്ടണമെന്നതിന് തെളിവുകളുമുണ്ട്. അവന്റെ ''സ്വന്തം പട്ടണം'' (his patris) ഈ പദം മത്തായിയും ലൂക്കായും ഉപയോഗിക്കുന്നത് ഈശോ ജനിച്ചത് ബത്‌ലഹേമിലാണെന്ന് അവര്‍ ഉറപ്പിച്ചച്ചു പറഞ്ഞിരിക്കുന്നതിനാല്‍, ഈശോ വളര്ന്നന സ്ഥലത്തെ ഉദ്ദേശിച്ചാണ് (മത്തായി 22-34, ലൂക്കോ 2:51). മാര്ക്കോ സില്‍ നിന്നാണ് (6:1,4) ഇവര്ക്ക് ഈ വിവരം ലഭിച്ചത് എന്നുള്ളതിനാലും മര്ക്കോതസിന് ബേത്‌ലഹേമിനെക്കുറിച്ച് ഒന്നും പറയാനുമില്ലാത്തതിനാലും അവന്റെ സ്വന്തം പട്ടണം നസ്‌റത്ത് അയിരിക്കണം. ഈശോ ജനിച്ചത് ഗലീലിയയിലായിരിക്കും.... നാലാ മത്തെ സുവിശേഷത്തില്‍ (1:46,7:41-42;52) ഈശൊയുടെ ഗാലിലേ യന്‍ ജനനത്തെക്കുറിച്ച് സൂചനകളുമുണ്ട്'.

(6) ലോകമാസകലമുള്ള ജനങ്ങളുടെ പേര് എഴുതിച്ചേര്ക്കനണം എന്ന് അഗസ്റ്റസ് സീസറിന്റെ കല്പഒനയനുസരിച്ച് യോസേഫും മറിയയും ബെത്‌ലഹേമിലേക്ക് പോയപ്പോഴാണ് യേശു ജനിച്ചതെ ന്നാണ് ലൂക്കോസിന്റെ സുവിശേഷം രണ്ടാം അധ്യായത്തില്‍ പറയുന്നത്. ഇത് ചരിത്രപരമായി ശരിയല്ല. അഗസ്റ്റസ് സീസറുടെ കാലത്ത് ലോകവ്യാപകമായ കണക്കെടുപ്പൊന്നും നടന്നിട്ടില്ല. ക്വരീ നിയസിന്റെ കാലത്ത് യൂദായിലുണ്ടായ കണക്കെടുപ്പ് (നസ്‌റോത്ത് അതില്പ്പെ ടുന്നില്ല) ഹെറോദോസ് മരിച്ച് പത്തുകൊല്ലങ്ങള്ക്കു് ശേഷമാണ് നടന്നത്. അപ്പോഴേക്കും യേശു ജനിച്ചിട്ടുണ്ടായിരിക്കണം. യേശുവിന്റെ ജനനത്തെ അതിന് കൊല്ലങ്ങള്ക്കുട മുമ്പു നടന്ന കണക്കെടുപ്പുമായി യോജിപ്പിക്കുകയാണ് ലൂക്കോസ് ചെയ്തിരിക്കുന്നത്. റെയ്മണ്ട് ബ്രൗണ്‍ എഴുതുന്നു: 'മഹാനായ ഹോരോദോസിന്റെ കാലത്ത് പാലസ്തീനായില്‍ ക്വിരീനിയൂസിന്റെ കീഴില്‍ റോമന്‍ സെന്സെസ് നടന്നു എന്നു വിശ്വസിക്കാന്‍ ഗൗരവതരമായ കാര ണങ്ങളില്ല. ലൂക്കോ 1ലെ വിവരം ശരിയായിരിക്കാം. മഹാനായ ഹെറോദോസിന്റെ കാലയളവില്‍ ഈശോ ജനിച്ചു. എന്നാല്‍ A.D 67 ല്‍ ക്വരീനിയൂസിന്റെ കീഴില്‍ യൂദായില്‍ (ഗലീലിയിലല്ല) നടന്ന സെന്സെസുമായി പൊരുത്തപ്പെടുത്തിയ ലൂക്കോയുടെ വിവരം അവിടെ A.D.30കളില്‍ (ഈശോയുടെ മരണത്തിന് ശേഷം) ഗമാലിയേല്‍ നടത്തുന്ന പ്രസംഗത്തില്‍ തെവുദാസിന്റെ (Thevudas) ഉയിര്ത്തെ ഴുന്നേല്പ്പി നെപ്പറ്റി പരാമര്ശിദക്കുന്നു. എന്നാലത് ഗമാലിയലിന്റെ പ്രസംഗത്തിനു 10 വര്ഷ‍ങ്ങള്ക്കുസ ശേഷവും നടന്നില്ല. തെവുദാസിനു ശേഷം ഗാലിലേയനായ യൂദാസിന്റെ (A.D 6-7) സെന്സംസും ഉയിര്ത്തെ ഴുന്നേല്പു്മായി ലൂക്കാ തെറ്റിദ്ധരിച്ചതാവാം.'

യേശുവിന്റെ ജനനവും ശൈശവകാലവുമായി ബന്ധപ്പെട്ടു കൊണ്ട് മത്തായിയും ലൂക്കോസും പറയുന്ന കാര്യങ്ങളൊന്നും ചരിത്രപരമല്ലെന്നും സുവിശേഷകന്മാരുടെ ഭാവനാസൃഷ്ടികളാണെന്നുമാണ് ആധുനിക ബൈബിള്‍ പണ്ഡിതന്മാരില്‍ പ്രമുഖനായ റെയ്മണ്ട്-ഇ-ബ്രൗണ്‍ തെളിവുകളുടെ അടിസ്ഥാനത്തില്‍ സ്ഥാപിക്കുന്നത്. സുവിശേഷകര്‍ തങ്ങളുടെ ഭാവനയില്‍ തോന്നിയ കാര്യങ്ങള്‍ യേശു കഥയോടു ബന്ധപ്പെടുത്തിക്കൊണ്ട് ചരിത്രമെന്ന നിലക്ക് അവതരിപ്പിച്ചെന്നും ജനങ്ങളെ തെറ്റിദ്ധരിപ്പിച്ചെന്നുമാണല്ലോ ഇതില്‍ നിന്നും മനസ്സിലാക്കേണ്ടത്. ശൈശവകാല വിവരണങ്ങളുടെ കാര്യത്തില്‍ മാത്രമാണോ ഇത് സംഭവിച്ചിരിക്കുന്നത്? അങ്ങനെ കരുതുകവയ്യ. യഹൂദന്മാര്‍ പ്രതീക്ഷിച്ചിരിക്കുന്ന മിശിഹാ യേശുവാണെന്ന് വരുത്തിത്തീര്ക്കു ന്നതിന് വേണ്ടി ശൈശവകാല വിവരണങ്ങളില്‍ കൃത്രിമത്വം കാണിച്ച സുവിശേഷകന്മാര്‍, യേശുവിന്റെ ജീവിതത്തേയും പ്രബോധനങ്ങളെയും മരണത്തെയുമെല്ലാം കുറിച്ച വിവരണങ്ങളില്‍ തങ്ങളുടെ ഇച്ഛക്കനുസരിച്ച് മിനുക്കു പണികള്‍ നടത്തിയിട്ടില്ല എന്നതിന് എന്താണുറപ്പ്? ദിവ്യാത്ഭുതങ്ങളായി സുവിശേഷങ്ങള്‍ വിവരിച്ചവ പോലും നൂറുശതമാനം ചരിത്രപരമായിക്കൊള്ളണമെന്നില്ലായെന്നാണ് റെയ്മണ്ട് ബ്രൗണിന്റെ അഭിപ്രായം. 'യേശുവിന്റെ ദിവ്യാത്ഭുതങ്ങളെപ്പറ്റിയുള്ള പാരമ്പര്യം ആധികാരികമാണെ ന്നിരിക്കിലും സുവിശേഷ ഗ്രന്ഥങ്ങളിലുള്ള ഓരോ ദിവ്യാത്ഭുതവും അക്ഷരാര്ത്ഥാത്തില്‍ ചരിത്രപരമായിക്കൊള്ളണമെന്നില്ല'

ഇതില്‍ നിന്ന് ഒരു കാര്യം സുതരാം വ്യക്തമാണ്. യേശുവിനെക്കുറിച്ച് സത്യസന്ധമായ വിവരങ്ങള്‍ ലഭിക്കുവാന്‍ സുവിശേഷങ്ങളെ മാത്രം ആശ്രയിച്ചിട്ട് ഫലമില്ല. അവയിലെ വിവരങ്ങള്‍ സത്യവും അസത്യവും കൂടിക്കലര്ന്ന് നിലയിലാണുള്ളത്. തങ്ങളുടെ ഭാവനക്കനുസൃതമായി ഓരോ സുവിശേഷകനും യേശുകഥ മെനഞ്ഞെടുക്കുകയാണ് ചെയ്തിരിക്കുന്നത്. അവയില്‍ നിന്ന് സത്യം ചികഞ്ഞെടുക്കുക അതീവ ദുഷ്‌കരമാണ്.

യേശുവിനെക്കുറിച്ച കൃത്യവും കളങ്കരഹിതവുമായ അറിവു നല്കുതന്നത് ദൈവീക ഗ്രന്ഥമായ ഖുർആനിൽ മാത്രമാണെന്ന വസ്തുതയാണ് ഇവിടെ വ്യക്തമാവുന്നത്.

വിഷയവുമായി ബന്ധപ്പെട്ട വീഡിയോ

ബഹുഭാര്യത്വം പ്രാകൃതമല്ല; മാനവികമാണ്, അത് ഇസ്‌ലാം അനുവദിച്ചതാണ്. പരിശുദ്ധഖുര്‍ആനും തിരുസുന്നത്തും വരച്ചുകാണിക്കുന്ന ധാര്‍മികജിവിതം നയിക്കാന്‍ സദാചാരത്തിലധിഷ്ഠിതമായ സമൂഹത്തില്‍ ചിലര്‍ക്കെങ്കിലും ബഹുഭാര്യത്വം അനിവാര്യമായിത്തീരും; പ്രകൃതിവിരുദ്ധമായ വിവാഹേതരരതിയെ അംഗീകരിക്കാത്തേടത്തോളം കാലം ഒരു സമൂഹത്തിലും ബഹുഭാര്യത്വത്തെ നിയമം മൂലം നിരോധിക്കാന്‍ സാധ്യമല്ല. ഇന്ത്യയിലെ നിലവിലുള്ള മുസ്‌ലിം വ്യക്തിനിയമം വ്യക്തിപരമായ വിഷയങ്ങളില്‍ (വിവാഹം, വിവാഹമോചനം, വഖ്ഫ്, അനന്തരാവകാശം) മുസ്‌ലിംകള്‍ക്ക് അവരുടെ പ്രമാണങ്ങളനുസരിച്ച് ജീവിക്കുവാനുള്ള അവകാശം നല്‍കുന്നുണ്ട്. ഈ അവകാശം നിലനിര്‍ത്തേണ്ടതുണ്ടോ എന്ന പ്രശ്‌നമുന്നയിച്ചുകൊണ്ടാണ് ബഹുഭാര്യത്വവുമായി ബന്ധപ്പെട്ട ചര്‍ച്ചകള്‍ ചൂടുപിടിക്കാറുള്ളത്. മുസ്‌ലിം വ്യക്തിനിയമത്തിന് പകരം ഏകസിവില്‍കോഡ് നടപ്പാക്കണമെന്ന വാദത്തിന് ഉപോല്‍ബലകമായാണ് ബഹുഭാര്യത്വം മൂലം കഷ്ടപ്പെടുന്ന മുസ്‌ലിം പെണ്ണുങ്ങളുടെ പ്രശ്‌നങ്ങള്‍ ചര്‍ച്ചക്ക് വരാറുള്ളത്.

ഇന്ത്യയിലെ പ്രബലന്യൂനപക്ഷമാണ് മുസ്‌ലിംകള്‍. അങ്ങനെ പ്രബല ന്യൂനപക്ഷമായ മുസ്‌ലിംകള്‍ക്ക് ഭരണഘടന നല്‍കുന്ന അടിസ്ഥാനപരമായ അവകാശമാണ് അവരുടെ വ്യക്തിനിയമമനുസരിച്ച് ജീവിക്കുവാനുള്ള അവകാശം. ഇത് ആരുടെയെങ്കിലും ഔദാര്യമായി കിട്ടിയതല്ല. മറിച്ച്, ഈ രാഷ്ട്രത്തിലെ പൗരന്മാരെന്ന നിലയ്ക്ക് ഈ രാജ്യത്തിന്റെ പുനര്‍നിര്‍മാണത്തിലെ പങ്കാളികളെന്ന നിലയ്ക്ക്, ഈ രാഷ്ട്രത്തിന്റെ ഭരണഘടന അനുവദിച്ചു തന്നിട്ടുള്ളതാണ്. ആ അനുവാദം എന്തെങ്കിലും ഒരു പ്രത്യേകസംഭവത്തിന്റെ അടിസ്ഥാനത്തില്‍ എടുത്തുകളയുവാന്‍ അനുവദിക്കേണമോ എന്നതാണ് വിഷയത്തിന്റെ മര്‍മം. സ്വാഭാവികമായും നിയമത്തിന് അതിന്റേതായ രീതിയുണ്ട്. നിയമത്തെ ദുര്‍വിനിയോഗിക്കുന്ന ആളുകളുണ്ടാകാം. ആ ആളുകളെ നിലയ്ക്ക് നിര്‍ത്തേണ്ടതുമുണ്ട്. പക്ഷെ, അത് മറ്റൊരാളുടെ അവകാശത്തെ ഹനിച്ചുകൊണ്ടായിക്കൂടാ. ഒരാള്‍ ബഹുഭാര്യത്വത്തിലേര്‍പ്പെടുകയും ആദ്യഭാര്യയെ പരിഗണിക്കാതെ അവള്‍ക്കാവശ്യമായ അവകാശങ്ങള്‍ നല്‍കാതിരിക്കുകയും ചെയ്യുന്നത് തെറ്റാണ്. അതിനെന്ത് പരിഹാരമുണ്ടെന്ന് ആലോചിക്കണം. അതിന് നിയമം മൂലമാണോ ബോധവല്‍ക്കരണം വഴിയാണോ, അതല്ല സമൂഹ്യമായ മറ്റു മാര്‍ഗങ്ങളുപയോഗിച്ചുകൊണ്ടാണോ പരിഹാരം കാണേണ്ടത് എന്ന് ആലോചിക്കേണ്ടതാണ്.

എന്നാല്‍ ഒറ്റപ്പെട്ട സംഭവങ്ങളെ ചൂണ്ടിക്കാണിച്ച് ധാര്‍മിക ജീവിതം നയിക്കുന്നതിന് വേണ്ടി അനിവാര്യമായ അവസരത്തില്‍ ബഹുഭാര്യത്വത്തിലേര്‍പ്പെടുനുള്ള ഒരാളുടെ അവകാശം നിഷേധിക്കുന്നതിന് മേല്‍നിയമം കാരണമായിക്കൂടാ. ഇന്ത്യന്‍ ഭരണഘടന അനുവദിച്ച മതമനുസരിച്ച് ജീവിക്കാനുള്ള അവകാശം ഒറ്റപ്പെട്ട സംഭവങ്ങളുടെ അടിസ്ഥാനത്തില്‍ നിഷേധിക്കാന്‍ കഴിയുമോ? അത് പാടില്ലെന്ന് വ്യക്തം. ബഹുഭാര്യത്വം നിരോധിക്കണമെന്നാണ് ഇസ്‌ലാം വിരുദ്ധര്‍ക്ക് പറയാനുള്ളത്. പക്ഷെ, ബഹുഭാര്യത്വം നിരോധിക്കണം, ബഹുഭാര്യത്വം പാടില്ല, ബഹുഭാര്യത്തിന് ഇന്നിന്ന പ്രശ്‌നങ്ങളുണ്ട് എന്നെല്ലാം പറയുന്നവരോട് പുരുഷന് ഒന്നിലധികം സ്ത്രീകളുമായി ബന്ധപ്പെടുന്ന കാര്യത്തിലും ഇത്തരം നിയമങ്ങള്‍ വേണമെന്ന് നിങ്ങള്‍ വാദിക്കുമോ എന്ന് ചോദിച്ചു നോക്കുക. ഉത്തരമില്ലാത്ത ഉരുണ്ടുകളി മാത്രമേ അവരില്‍ നിന്നുണ്ടാകൂ.

ഇന്ത്യയിലൊരു നിയമവും വ്യഭിചാരത്തിന് തടസ്സം നില്‍ക്കുന്നില്ല. അതിന് ആര്‍ക്കും യാതൊരു പ്രശ്‌നവുമില്ല. ഇന്ത്യയില്‍ ഒരു പുരുഷന് ഒന്നിലധികം ഭാര്യമാരുണ്ടാകുകയാണെങ്കില്‍ പ്രശ്‌നമാണ്; എന്നാല്‍ ഭാര്യയുണ്ടായിരിക്കെ അയാള്‍ മറ്റൊരു സ്ത്രീയുമായി ശാരീരികമായി ബന്ധപ്പെട്ടാലോ? ആ ബന്ധം തെളിയിച്ചാല്‍ വേണമെങ്കില്‍ ഭാര്യക്ക് വിവാഹമോചനത്തിന്നവകാശമുണ്ടെന്നതൊഴിച്ച് ആ ബന്ധം പാടില്ലാ എന്ന് വിധിക്കാന്‍, ആ ബന്ധത്തിലേര്‍പ്പെട്ടതുകൊണ്ട് എന്തെങ്കിലും ശിക്ഷ വിധിക്കാന്‍ ഇന്ത്യയില്‍ യാതൊരു മാര്‍ഗവുമില്ല. അതേ സമയം അത് നിയമപരമാകുമ്പോഴാണ്; അതായത് വിവാഹത്തിലൂടെയാകുകയാകുമ്പോഴാണ് പ്രശ്‌നം. ഒരാള്‍ക്ക് പത്തോ ഇരുപതോ സ്ത്രീകളുമായി ബന്ധപ്പെടാം. പക്ഷെ, രണ്ടാമതൊരു ഭാര്യയായാല്‍ കുഴപ്പമായി. പത്തോ ഇരുപതോ സ്ത്രീകളുമായി തന്റെ ഭര്‍ത്താവ് ബന്ധപ്പെടുമ്പോള്‍ ഭാര്യക്ക് അതിനെതിരെ കേസ് കൊടുക്കാന്‍ വല്ല വകുപ്പുമുണ്ടോ? വകുപ്പില്ല. അയാള്‍ക്കെന്തുമാകാം. ഭാര്യക്ക് വിവാഹമോചനമാവശ്യപ്പെടാം. അങ്ങനെ അവള്‍ക്കു കിട്ടുന്ന ജീവിതവിഭവങ്ങള്‍കൂടി ഇല്ലാതെയാകുന്ന അവസ്ഥ സംജതമാകാം. ഇതല്ലാതെ അതിനെതിരില്‍ നിയമപരമായി നടപടിയെടുക്കാന്‍ ഇന്ത്യയില്‍ എന്തെങ്കിലും സംവിധാനങ്ങളുണ്ടോ? നിയമങ്ങളുണ്ടോ? ഇല്ല. അതേസമയം ബന്ധം നിയമപരമായ വിവാഹത്തിലൂടെയാകുമ്പോള്‍ കുഴപ്പമായി. ഇതാണ് ജനാധിപത്യം പെണ്ണിന് നൽകുന്ന അംഗീകാരം !!!

ഇസ്‌ലാം അനുവദിച്ച ബഹുഭാര്യത്വത്തില്‍ പുരുഷന്‍ വിവാഹം ചെയ്യുന്ന സ്ത്രീകള്‍ക്കെല്ലാം നിയമപരമായ അവകാശങ്ങളുണ്ട്. അവള്‍ക്ക് അടിസ്ഥാനപരമായ അവകാശങ്ങള്‍ നല്‍കിയിട്ടില്ലെങ്കില്‍ കോടതിയില്‍ പോകാനുള്ള അധികാരമുണ്ട്. പെണ്ണിന്റെ ഏതൊരവകാശമാണെങ്കിലും അത് ജീവിത വിഭവങ്ങളുടെ കാര്യത്തിലും നീതിയോടുകൂടി വര്‍ത്തിക്കുക എന്നുള്ള കാര്യത്തിലുമെല്ലാം അവഗണിക്കപ്പെടുകയും പീഡിപ്പിക്കപ്പെടുകയും ചെയ്യുന്നുവെങ്കില്‍ അതിനെതിരെ കോടതിയെ സമീപിക്കാന്‍ ഇസ്‌ലാം അവള്‍ക്കനുവാദം നല്‍കുന്നുണ്ട്.

ഇസ്‌ലാം വിമര്‍ശകര്‍ പകരം നിര്‍ദേശിക്കുന്ന സ്വതന്ത്രരതിയില്‍ ഇത്തരം അവകാശങ്ങളൊന്നും അനുവദിക്കപ്പെടുന്നില്ല. അവളൊരു കേവലം സുഖഭോഗ വസ്തു മാത്രമാണവിടെ. സ്ത്രീത്വമാണ് അവിടെ അപമാനിക്കപ്പെടുന്നത്. അതിനെതിരെ ഒന്നും ഉരിയാടാന്‍ പെണ്‍വാദികള്‍ക്കൊന്നും തന്നെ നട്ടെല്ലുമില്ല. ഏകസിവില്‍കോഡ് വിവാദങ്ങള്‍ യഥാര്‍ഥത്തില്‍ നിയമത്തിനോടോ മുസ്‌ലിം സമുദായത്തിനോടോ ഉള്ള മമതയില്‍ നിന്നുണ്ടായതല്ല. സമൂഹത്തെ വളരെ മെല്ല ധാര്‍മികമായ ചട്ടക്കൂടുകളില്‍ നിന്നകറ്റി സ്വതന്ത്രരതിയുടെ പ്രദേശമാക്കി ഇന്ത്യയെ മാറ്റുവാന്‍ വേണ്ടിയുള്ള കരുതിക്കൂട്ടിയ ഗൂഢാലോചനകളുടെ ഭാഗമാണിത്.

ബഹുഭാര്യത്വം നിയന്ത്രിക്കുന്നതിനായി നിര്‍ദേശിക്കപ്പെട്ടിരിക്കുന്ന ബില്ലില്‍ ബഹുഭാര്യത്വത്തിലേര്‍പ്പടുകയാണെങ്കില്‍ ഉണ്ടാകേണ്ട കുറെ നിബന്ധനകളെക്കുറിച്ച് പറയുന്നുണ്ട്. പലപ്പോഴും വൈയക്തികമായ പ്രശ്‌നങ്ങളാണ് ബഹുഭാര്യത്വത്തിന് നിമിത്തമായി വരുന്നത്. അതിന് നിബന്ധനകള്‍ നിര്‍ദേശിക്കുമ്പോള്‍ മനുഷ്യന്റെ അടിസ്ഥാനപരമായ അവകാശമാണ് നിഷേധിക്കപ്പെടുന്നത്. ലൈംഗികത അടിസ്ഥാനപരമായ ചോദനയാണ്. ആ ചോദനയുടെ പൂര്‍ത്തീകരണമെന്നുള്ളത് അവന്റെ അടിസ്ഥാനാവശ്യമാണ്. അതിന് നിയതമായ മാര്‍ഗങ്ങളുണ്ടാകണം എന്നതാണ് ഇസ്‌ലാമികനിയമങ്ങളുടെ താല്‍പര്യം.

തന്റെ മതമനുസരിച്ച് ജീവിക്കുവാന്‍ ഓരോ ഇന്ത്യക്കാരനും അവകാശമുണ്ട്. മുസ്‌ലിമിനെസംബന്ധിച്ചേടത്തോളം മതമെന്നാല്‍ ജീവിതത്തെ പ്രകാശമാനമാക്കുന്ന ദൈവിക വിധിവിലക്കുകളുടെ പ്രയോഗമാണ്. അവ പ്രയോഗവല്‍ക്കരിക്കുവാന്‍ ഇന്ത്യന്‍ മുസ്‌ലിമിന് അവകാശമുണ്ട്. ആ അവകാശത്തെ നിഷേധിക്കാന്‍ ഒരു ജനാധിപത്യസമ്പ്രദായം സന്നദ്ധമാകുകയെന്നു പറഞ്ഞാല്‍ അതിന് ജനാധിപത്യസ്വഭാവം നഷ്ടപ്പെടുന്നുവെന്നതാണ് അര്‍ഥം. മറ്റുള്ള മതസമൂഹങ്ങള്‍ക്ക് ഇതേ പ്രശ്‌നങ്ങളില്ലേ എന്നു ചോദിക്കാവുന്നതാണ്. മറ്റുള്ള മതസമൂഹങ്ങളെ സംബന്ധിച്ചേടത്തോളം ഉദാരലൈംഗികതയുടെ തലത്തിലേക്ക് പോകുന്നതില്‍ നിന്ന് അവരെ തടയുന്ന ഇസ്‌ലാമിലേത് പോലെയുള്ള ശക്തമായ വിലക്കുകളില്ലായിരിക്കാം. എല്ലാവരും അങ്ങനെ ചെയ്യുന്നുണ്ടോ എന്നയാരിക്കും അടുത്ത ചോദ്യം. ഇല്ലായിരിക്കാം. പക്ഷെ, അവര്‍ക്ക് സ്വതന്ത്രലൈംഗികത ആസ്വദിക്കുവാന്‍ ഇന്ത്യയില്‍ അനുവാദമുണ്ട്. എന്നാല്‍ മുസ്‌ലിന് ഒരു കാരണവശാലും അതു പറ്റുകയില്ല. മുസ്‌ലിംകളെ മതേതരവല്‍ക്കരിച്ച് ഇവര്‍ പറയുന്ന ഉദാരലൈംഗികതയുടെ വക്താക്കളാക്കുകയാണ് ഈ ഉദ്യമങ്ങളുടെയെല്ലാം ആത്യന്തികലക്ഷ്യം.

ഉദാരലൈംഗികതയുടെ വക്താക്കള്‍ പറയുന്ന രൂപത്തിലുള്ള ധാര്‍മികത നടപ്പിലാക്കിയ പ്രദേശങ്ങളുടെ അവസ്ഥയെന്താണെന്ന് പരിശോധിക്കപ്പെടേണ്ടത് തന്നെയാണ്. ചൈനയാണെങ്കിലും റഷ്യയാണെങ്കിലും എല്ലാം ദീര്‍ഘകാലത്തെ മതേതരവല്‍ക്കരണത്തിലൂടെ എവിടെയെത്തി എന്നു നാം കണ്ടതാണ്. ഈ ഒരു അവസ്ഥയിലേക്ക് ഇന്ത്യയും പോയിക്കൊണ്ടിരിക്കുകയാണ്. പക്ഷെ, അങ്ങോട്ട് മുസ്‌ലിംകളെക്കൂടി കൂടെ കൊണ്ടുപോകേണമോ എന്നതാണ് പ്രശ്‌നം. ബുദ്ധിജീവികളെ സംബന്ധിച്ചേടത്തോളം മുസ്‌ലിം സ്ത്രീകളുടെ കണ്ണുനീരാണ് അവരെ ഏകസിവില്‍കോഡിന് വേണ്ടി വാദിപ്പിക്കുന്നതിന് പിന്നിലുള്ള പ്രശ്‌നമെന്നാണ് പറയുന്നത്. അവരുടെ സ്ത്രീസ്‌നേഹം ആത്മാര്‍ഥമാണെങ്കില്‍ മുസ്‌ലിം സമൂഹത്തിന്റെ മതവല്‍ക്കരണത്തിനാണ് അവര്‍ ശ്രമിക്കേണ്ടത്. മുസ്‌ലിം സ്ത്രീകളുടെ പ്രശ്‌നങ്ങള്‍ പരിഹരിക്കുവാന്‍ മുസ്‌ലിം സമൂഹത്തിന്റെ തന്നെ പൂര്‍ണമായ അര്‍ഥത്തിലുള്ള മതവല്‍ക്കരണമാണാവശ്യം, മതപരമായ ബോധവല്‍ക്കരണമാണാവശ്യം.

മുസ്‌ലിം സമൂഹത്തിലെ ഒരു പ്രശ്‌നമായിരുന്നു ത്വലാഖ്. പക്ഷെ, കേരളമുസ്‌ലിം സമൂഹത്തെ സംബന്ധിച്ചേടത്തോളം മതപരമായി കൂടുതല്‍ ബോധം നേടിയപ്പോള്‍ വിവാഹമോചനത്തിന്റെ തോത് സമുദായത്തില്‍ ഗണ്യമായി കുറഞ്ഞു. മതവല്‍ക്കരണമാണ് അവിടെ നടന്നത്. അത് നാം മനസ്സിലാക്കണം. മതനിഷേധത്തിനപ്പുറത്ത് മതപരമായ വിഷയങ്ങള്‍ കൃത്യമായി പഠിക്കുവാനും മതമനുസരിച്ചുള്ള ബോധനം നേടുവാനും സാധിക്കണം. അപ്പോള്‍ ഈ രംഗത്തെല്ലാമുള്ള ചൂഷണങ്ങളെ ഇല്ലാതാക്കാന്‍ കഴിയും.

മുസ്‌ലിം സമുദായത്തിലെ പ്രശ്‌നങ്ങളില്‍ കണ്ണുനീരുമായി നടക്കുന്ന ആളുകളെ സംബന്ധിച്ചേടത്തോളം യഥാര്‍ഥത്തിലുള്ള സ്ത്രീ പ്രശ്‌നത്തില്‍ അവര്‍ക്കൊന്നും ചെയ്യാനില്ല. സ്ത്രീധനത്തെപ്പോലെയുള്ള പ്രശ്‌നങ്ങളില്‍ അവരെന്തു ചെയ്തു? സ്ത്രീധനം നിയമം മൂലം നിരോധിക്കപ്പെട്ട നാടാണ് ഇന്ത്യ. ഏറ്റവും വലിയ സ്ത്രീവിരുദ്ധമായ, സ്ത്രീകളെ ഏറ്റവും കൂടുതല്‍ പീഡിപ്പിക്കുന്ന ഇക്കാര്യം പക്ഷെ, നിയമം മൂലം നിരോധിക്കപ്പെട്ട നാട്ടില്‍ നിര്‍ലോഭം നടന്നുകൊണ്ടിരിക്കുന്നു. അതില്‍ യാതൊരു കുഴപ്പവും ആരും കാണുന്നില്ല. അതിനെതിരെ എന്തെങ്കിലും ശക്തമായ കാല്‍വെയ്പ് നടത്തുന്നുണ്ടെങ്കില്‍ അത് ഈ രാജ്യത്തെ മതസംഘടനകളാണ്. അവരോടൊപ്പം നില്‍ക്കാന്‍ ഈ സ്ത്രീസ്‌നേഹികള്‍ സന്നദ്ധമാകുമോ?

നിയമം എന്തിനുവേണ്ടി എന്ന ചോദ്യത്തിന് നിയമമനുസരിച്ച് ജീവിക്കുന്നവനെ പ്രയാസപ്പെടുത്തുന്നതിന് വേണ്ടി എന്നാകരുത് ഉത്തരം. നിയമം, അതനുസരിച്ച് ജീവിക്കുന്നവന് നീതി ലഭിക്കുന്നതിന് വേണ്ടിയാണ്. പൗരന്മാരില്‍ പലരും നിയമമനുസരിച്ച് ജീവിക്കുന്നില്ല എന്നതാണ് നമ്മെ സംബന്ധിച്ചേടത്തോളമുള്ള പ്രശ്‌നം, അല്ലാതെ ബഹുഭാര്യത്വത്തിനുള്ള അനുവാദമല്ല. പൗരബോധമുള്ളവരാക്കി ഭാരതീയരെ മാറ്റിത്തീര്‍ക്കുന്നതിനും നിയമലംഘനത്തില്‍ നിന്ന് അവരെ അകറ്റി നിര്‍ത്തുന്നതിനും വേണ്ടിയുള്ള കൂട്ടായ പരിശ്രമമാണ് വേണ്ടത്. അതല്ലാതെ മതമനുസരിച്ച് ജീവിക്കുവാനുള്ള മൗലികാവകാശത്തിനെതിരെയുള്ള ചന്ദ്രഹാസമല്ല. ധാര്‍മികജീവിതം നയിക്കാന്‍ ആഗ്രഹിക്കുന്നവരെ പ്രയാസപ്പെടുത്താന്‍ വേണ്ടിയുള്ളതാകരുത് നിയമങ്ങള്‍; ഭാരതീയസമൂഹത്തെ ലൈംഗിക ഉദാരീകരണത്തിലേക്ക് നയിക്കുന്നതിന് വേണ്ടിയുള്ള പദ്ധതികളില്‍ നിന്ന് മാറി നില്‍ക്കുവാന്‍ മുസ്‌ലിംകളെ അനുവദിക്കണം. അത് അവരുടെ മതപരമായ അനിവാര്യതയാണ്; ഇന്ത്യന്‍ ഭരണഘടന നല്‍കുന്ന അവകാശവും.

വിഷയവുമായി ബന്ധപ്പെട്ട വീഡിയോ

തന്റെ ഭര്‍ത്താവ് മറ്റൊരു സ്ത്രീയെ വിവാഹം ചെയ്യുമ്പോഴുണ്ടാകുന്ന ആദ്യഭാര്യയുടെ വൈകാരിക പ്രയാസങ്ങള്‍ക്കുള്ള പരിഹാരമെന്ത് എന്ന ചോദ്യം പ്രസക്തമാണ്. ബഹുഭാര്യത്വത്തിലേര്‍പ്പെടുന്ന തന്റെ ഭര്‍ത്താവിനെ മറ്റൊരു സ്ത്രീയും കൂടി പങ്കുവെക്കുമെന്നുള്ള അറിവ് സ്ത്രീയെ സംബന്ധിച്ചേടത്തോളം വൈകാരികമായി പ്രയാസമുണ്ടാക്കുന്നത് തന്നെയാണ്. ഈ രംഗത്തുള്ള പെണ്ണിന്റെ സ്വാര്‍ഥത അവളുടെ അസ്തിത്വത്തിന്റെ ഭാഗം തന്നെയാണ്. നശീകരണാത്മകമായ തലത്തിലുള്ള സ്വാര്‍ഥതയല്ല ഇത്. പൊസസ്സീവ്‌നസ്സ് എന്നു പറയാം. തന്റേത് മാത്രമാകണമെന്ന വിചാരം എന്നര്‍ഥം.

തന്റേത് മാത്രമാക്കി നിലനിർത്താനാഗ്രഹിക്കുകയെന്ന പൊസസ്സീവ്‌നസ് നിലനില്‍ക്കേണ്ടത് സ്ത്രീയുടെ ലൈംഗികജീവിതത്തിനും കുടുംബജീവിതത്തിനും ആവശ്യം തന്നെയാണ്. ഇണക്ക് മുമ്പില്‍ അവള്‍ക്ക് സ്‌നേഹം ചൊരിയാന്‍ കഴിയണമെങ്കിലും നിസ്വാര്‍ഥമായി അവള്‍ക്ക് അവളുടെ കുഞ്ഞുങ്ങളെ വളര്‍ത്താന്‍ കഴിയണമെങ്കിലും ആ കുഞ്ഞുങ്ങളെ വളര്‍ത്തുന്ന രംഗത്ത് അവളുടെ പൂര്‍ണമായ ആ സമര്‍പ്പണത്തിന് വേണ്ടിയുല്ലൊം തന്നെ ഈ ഒരടിസ്ഥാനപരമായ സ്വാര്‍ഥതയുണ്ടാകണം.

വൈകാരികമായ ഈ സ്വാർത്ഥത നിലനില്‍ക്കുന്നതുകൊണ്ട് തന്നെ തന്റെ ഇണ പങ്കുവെക്കപ്പെടുന്നുവെന്ന അറിവ് സ്വാഭാവികമായും അവള്‍ക്ക് മനഃപ്രയാസമുണ്ടാക്കിയേക്കാം. ഒരു മുസ്‌ലിം സ്ത്രീയെസംബന്ധിച്ചേടത്തോളം അവള്‍ക്ക് മതം തുണയാകുന്നതവിടെയാണ്. മതത്തിന്റെ നിര്‍ദേശങ്ങള്‍ അവിടെയാണ് അവള്‍ക്ക് സമാധാനമേകേണ്ടത്. ജീവിതത്തിലുണ്ടാവുന്ന പല തരാം പരീക്ഷണങ്ങളെപ്പോലെ ഇതുമൊരു പരീക്ഷണമായി അവള്‍ക്ക് കാണാൻ കഴിയും. തനിക്കുണ്ടായ ഈ വൈകാരിക പരീക്ഷണത്തിൽ സഹനമവലംബിച്ച് പടച്ചവന്‍ പറഞ്ഞതനുസരിച്ച് അവള്‍ മുന്നോട്ട് പോകുമ്പോള്‍ തീര്‍ച്ചയായും തനിക്ക് ഇഹലോകത്തും പരലോകത്തും അതിനുള്ള പ്രതിഫലം നല്‍കപ്പെടുമെന്ന ചിന്ത അവൾക്ക് ആശ്വാസം നൽകും.

അനിവാര്യമായ സാഹചര്യത്തിൽ തന്റെ ഇണ മറ്റൊരു ഭാര്യയെ സ്വീകരിക്കുമ്പോൾ അയാളും ആ സമയത്ത് ക്ഷമ അവലംബിക്കുന്ന താനും പടച്ചവന്റെ കല്‍പന അനുസരിക്കുകയാണെന്ന ചിന്ത അവൾക്ക് സന്തോഷവും സമാധാനവും നൽകും. ജീവിതത്തില്‍ പ്രതിസന്ധികളുണ്ടാകാം. പ്രയാസങ്ങളുണ്ടാകാം. ആ സമയത്ത് ഒരു വിശ്വാസി ഏത് രൂപത്തില്‍ ക്ഷമിക്കേണമോ അതേ രൂപത്തില്‍ ഈ രംഗത്തും ക്ഷമിക്കുവാന്‍ വിശ്വാസിയായ അവള്‍ക്ക് കഴിയും.. ക്ഷമ പ്രതിഫലദായകമാണെന്ന പാഠം അവള്‍ക്ക് ആശ്വാസം നല്‍കും.

സ്ത്രീകളുടെ വൈകാരികപ്രയാസങ്ങൾക്കു പോലും മതത്തിന് അതിന്റേതായ കൃത്യമായ കാഴ്ചപ്പാടുണ്ട്. ഒന്നിലധികം ഇണകളുള്ള പുരുഷന്മാരുടെ ഭാര്യമാരായ സ്ത്രീകൾക്ക് സംതൃപ്തിയും ശാന്തിയുമുണ്ടാകുന്നതിനു വേണ്ടിയുള്ള നിര്‍ദേശങ്ങളുണ്ട്. നിയമത്തിന്റെ രീതിയില്‍ ഈ വൈകാരികപ്രയാസത്തെ പരിഹരിക്കാന്‍ കഴിയുമോ? കഴിയില്ല. നിയമം സംസാരിക്കുന്നത് നീതിയുമായി ബന്ധപ്പെട്ടാണ്. പെണ്ണിനെതിരില്‍ എന്തെങ്കിലും അനീതിയുണ്ടാകുന്നുവെങ്കില്‍ നിയമത്തിന് പരിഹരിക്കാന്‍ കഴിയും. പെണ്ണിനെതിരില്‍ ഒരക്രമം ഉണ്ടാകുന്നുവെങ്കില്‍ നിയമത്തിന് അവള്‍ക്കായി സംസാരിക്കാന്‍ കഴിയും. പെണ്ണിനെ ഏതെങ്കിലും രൂപത്തില്‍ അവഗണിക്കുന്നുവെങ്കില്‍ നിയമത്തിന് അതില്ലാതെയാക്കുവാന്‍ കഴിയും. എന്നാല്‍ ഇവിടെ പെണ്ണിന് മാനസികമായുണ്ടാകുന്ന പ്രയാസങ്ങള്‍ നിയമം മൂലം പരിഹരിക്കാന്‍ കഴിയുന്നതല്ല. അതൊരു വൈകാരികപ്രശ്‌നമാണ്. അതുകൊണ്ട് തന്നെ ധാര്‍മികമായ മതത്തിന്റെ നിര്‍ദേശങ്ങള്‍ അനുസരിച്ചുകൊണ്ട് അവിടെ പരിഹാരം നിര്‍ദേശിക്കുവാനേ സാധിക്കൂ. അതല്ലാതെ മറ്റു മാര്‍ഗങ്ങളില്ല.

വിശ്വാസിയായ മുസ്‌ലിം പുരുഷൻ അഥവാ ബഹുഭാര്യത്വത്തിലേര്‍പ്പെടുമ്പോള്‍ അതുവഴിയുണ്ടാകുന്ന ഇണയുടെ പ്രയാസങ്ങളില്‍ കൂടി താങ്ങായിത്തീരാന്‍ അയാള്‍ക്ക് കഴിയും. സ്‌നേഹത്തിന്റെ കവിഞ്ഞൊഴുക്ക് വഴി ഇത്തരം പ്രയാസങ്ങള്‍ കൂടി പരിഹരിക്കാന്‍ കഴിയും എന്നാണ് പ്രവാചകന്റെയും സഹാബിമാരുടെയും അതേ പോലെ പിന്‍കാലമുസ്‌ലിംകളുടെയുമെല്ലാം സംഭവങ്ങളും ജീവചരിത്രവും നമുക്ക് മനസ്സിലാക്കിത്തരുന്നത്.

മനുഷ്യപ്രകൃതിയുടെ മതമായ ഇസ്‌ലാം ബഹുഭാര്യത്വം പ്രകൃതിപരമായതിനാൽ അത് അനുവദിക്കുകയും ബഹുഭര്‍തൃത്വം പ്രകൃതിവിരുദ്ധമായതിനാൽ അത് നിരോധിക്കുകയും ചെയ്യുന്നുവെന്നാണ് എന്തുകൊണ്ട് ബഹുഭർതൃത്വം പാടില്ല എന്ന ചോദ്യത്തിനുള്ള മറുപടി.

ബഹുഭർതൃത്വം പ്രകൃതിവിരുദ്ധമാണെന്ന് പറയാനുള്ള കാരണമെന്താണ്? ചില പുരുഷന്മാര്‍ക്കെങ്കിലും ഒന്നിലധികം സ്ത്രീകളുമായി ബന്ധപ്പെടേണ്ട പ്രകൃതിപരവും അനിവാര്യവുമായ ചില കാരണങ്ങളുണ്ടാകും. എന്നാല്‍ സ്ത്രീപ്രകൃതി അങ്ങനെയുള്ളതല്ല. അവള്‍ക്ക് ഒന്നിലധികം പുരുഷന്മാര്‍ ആവശ്യമായി വരുന്ന അവസ്ഥകളൊന്നും തന്നെയില്ല. ഈ രംഗത്തെ ലൈംഗികശാസ്ത്രപഠനങ്ങള്‍ എടുത്തു പരിശോധിച്ചാൽ ഇക്കാര്യം ബോധ്യമാകും. ഇത്തരമൊരു അവസ്ഥ സ്ത്രീകള്‍ക്കുണ്ടാകുമെന്ന് ലൈംഗികശാസ്ത്രജ്ഞന്‍മാരൊന്നും പറയുന്നില്ല. ആരോഗ്യമുള്ള ഒരു പുരുഷനുള്ളിടത്തോളം ഒരു സ്ത്രീക്ക് മറ്റൊരു പുരുഷനെ അവളുടെ ലൈംഗികതക്ക് പ്രകൃതിപരമായ ആവശ്യമായി ഉപയോഗിക്കേണ്ടി വരുമെന്ന് വിലാസമുള്ള ഏതെങ്കിലൊരു ലൈംഗികശാസ്ത്രജ്ഞന്‍ വസ്തുനിഷ്ഠമായി തെളിയിക്കട്ടെ; അപ്പോള്‍ നമുക്ക് ഈ വിഷയം ചര്‍ച്ച ചെയ്യാവുന്നതാണ്. എന്നാല്‍ അതല്ല വസ്തുത.

സ്ത്രീക്ക് ബഹുഭര്‍തൃത്ത്വത്തിന് കഴിയില്ല. സ്ത്രീയുടെ മനസ്സും ശരീരവും ആ രൂപത്തിലുള്ളതാണ്. ലൈംഗികത്തൊഴിലാളികളില്ലേ എന്ന് മറുചോദ്യം ഉന്നയിക്കപ്പെടാവുന്നതാണ്. ലൈംഗികത്തൊഴിലാളികള്‍ അതിനെ തൊഴിലായി സ്വീകരിച്ചവരാണ്; അതിനായി അവരുടെ ശരീരവും മനസ്സും സമര്‍പ്പിക്കപ്പെട്ടിട്ടില്ല. രണ്ടോ മൂന്നോ നാലോ ഇണകളുണ്ടാകട്ടെ, പുരുഷന് അവര്‍ക്കോരോരുത്തര്‍ക്കും തന്റെ മനസ്സില്‍ സ്ഥാനമനുവദിക്കാനും സ്‌നേഹം കൊടുക്കാനും ആവശ്യമായ അവകാശങ്ങള്‍ കൊടുക്കാനും ലൈംഗികമായി അവരെയെല്ലാം തൃപ്തിപ്പെടുത്താനും കഴിയും.  എന്നാല്‍ സ്ത്രീക്കോ? ഒന്നിലധികം പുരുഷന്മാരെ തങ്ങളുടെ മനസ്സുകള്‍ക്കകത്തേക്ക് സന്നിവേശിപ്പിക്കാന്‍ കഴിയില്ല. അതു കൊണ്ട് തന്നെ ബഹുഭര്‍തൃത്ത്വം പ്രകൃതിവിരുദ്ധമാണ്.

പല പുരാതന സമൂഹങ്ങളിലും ബഹുഭര്‍തൃത്വം നിലനിന്നിരുന്നുവെന്നത് നേരാണ്. ടിബറ്റ്, സിലോണ്‍, സൈബീരിയ തുടങ്ങിയ സ്ഥലങ്ങളിലെല്ലാം ഒരു രൂപത്തിലല്ലെങ്കില്‍ മറ്റൊരു രൂപത്തില്‍ ബഹുഭര്‍തൃത്വം നിലനിന്നിരുന്നു. ഇന്ത്യയിലാകട്ടെ ഇതിഹാസകാലത്ത് ഈ സമ്പ്രദായം നിലനിന്നിരുന്നുവെന്ന് മനസ്സിലാവുന്നുണ്ട്. വേദകാലത്ത് ബഹുഭര്‍തൃത്വം നിലനിന്നിരുന്നതായി യാതൊരു സൂചനകളുമില്ല. ഐതരേയ ബ്രാഹ്മണത്തിലും തൈത്തിരീയ സംഹിതയിലും ബഹുഭാര്യത്വത്തെക്കുറിച്ച സൂചനകളുണ്ടെങ്കിലും ബഹുഭര്‍തൃത്വത്തെക്കുറിച്ച യാതൊരു പരാമര്‍ശവുമില്ലെന്ന വസ്തുത വേദകാലത്ത് ആ സമ്പ്രദായം നിലനിന്നിരുന്നില്ലെന്നാണ് വ്യക്തമാക്കുന്നത്. എന്നാല്‍ മഹാഭാരതത്തിലെ പാഞ്ചാലിയുടെ കഥയില്‍നിന്ന് ഇവിടെ ഇതിഹാസകാലമായപ്പോഴേക്ക് ബഹുഭര്‍തൃത്വ സമ്പ്രദായം നിലവില്‍ വന്നിരുന്നുവെന്ന് മനസ്സിലാക്കാവുന്നതാണ്. അടുത്തകാലം വരെയും കേരളത്തി ല്‍ ബഹുഭര്‍തൃത്വം നിലനിന്നിരുന്നു. കൊല്ലന്മാര്‍ക്കിടയിലും ആശാരിമാര്‍ക്കിടയിലും അനേകം സഹോദരന്മാര്‍ക്ക് ഒരു ഭാര്യയെന്ന സമ്പ്രദായമാണുണ്ടായിരുന്നത്. ഈഴവന്മാര്‍ക്കിടയിലും നായന്മാര്‍ക്കിടയിലുമെല്ലാം ഇതു നിലനിന്നിരുന്ന പ്രദേശങ്ങളുണ്ടായിരുന്നു. മലബാറിലും തിരുവിതാംകൂറിലും നായന്മാര്‍ക്കിടയില്‍ നിലനിന്നിരുന്ന സംബന്ധം പ്രസിദ്ധമാണല്ലോ. സുന്ദരികളായ സ്ത്രീകള്‍ക്ക് നാലും അഞ്ചും സംബന്ധക്കാരുണ്ടായിരുന്നുവത്രേ. ബഹുഭര്‍തൃത്വം അഭികാമ്യമായ ഒരു സാമൂഹ്യസ്ഥാപനമായിരുന്നെങ്കില്‍ ഈ സമൂഹങ്ങള്‍ അത് നിലനിര്‍ത്തേണ്ടതായിരുന്നു. എന്നാല്‍ അതുണ്ടായില്ല. ഈ സമൂഹങ്ങളിലെല്ലാം ഒരു പ്രത്യേകകാലത്ത് മാത്രമാണ് ബഹുഭര്‍തൃത്വം നിലനിന്നത്. കാലാന്തരത്തില്‍ പ്രസ്തുത സമൂഹങ്ങള്‍ തന്നെ ബഹുഭര്‍തൃത്വസമ്പ്രദായം ഉപേക്ഷിക്കുകയായിരുന്നു.

പുരുഷാധിപത്യത്തിലധിഷ്ഠിതമായ വ്യവസ്ഥിതി നിലനിര്‍ത്തുന്നതിന് വേണ്ടിയാണ് പെണ്‍കോയ്മയില്‍ അധിഷ്ഠിതമായ ബഹുഭര്‍തൃത്വസമ്പ്രദായം ഉപേക്ഷിക്കപ്പെട്ടത് എന്ന വാദത്തില്‍ കഴമ്പില്ല. രതിസുഖമനുഭവിക്കുന്ന പുരുഷന് ഉത്തരവാദിത്തങ്ങളില്‍ നിന്നെല്ലാം ഒഴിഞ്ഞു നില്‍ക്കാന്‍ സൗകര്യം നല്‍കുന്നതാണ് ബഹുഭര്‍തൃത്വവ്യവസ്ഥ. അതുകൊണ്ട് തന്നെ ആണ്‍കോയ്മ ബോധപൂര്‍വം ഉപേക്ഷിച്ചതാണ് അതെന്ന് കരുതാന്‍ ന്യായമില്ല. ഒരു സ്ഥാപനമെന്ന നിലക്ക് സമൂഹത്തിന്റെ സംതുലിതത്വം നിലനിര്‍ത്തുവാന്‍ സാധിക്കാത്തതിനാല്‍ സ്വാഭാവികമായി ബഹുഭര്‍തൃത്വം ഇല്ലാതാവുകയാണ് ചെയ്തത്. ജോര്‍ജ് പീറ്റര്‍ മര്‍ഡോക്കിന്റെ പഠനത്തിന് വിധേയമായ 1170 നാഗരികതകളില്‍ ഒരെണ്ണത്തില്‍ മാത്രമാണ് ബഹുഭര്‍തൃത്വം പേരിനെങ്കിലും നിലനിന്നിരുന്നത് എന്ന വസ്തുത അതൊരു സ്വീകാര്യമായ വ്യവസ്ഥിതിയായി ചരിത്രത്തിലൊന്നും ദീര്‍ഘകാലം നിലനിന്നിട്ടില്ലെന്ന വസ്തുത വ്യക്തമാക്കുന്നുണ്ട്.

സ്ത്രീവിരുദ്ധവുമാണ് യഥാര്‍ഥത്തില്‍ ബഹുഭര്‍തൃത്ത്വം. ബഹുഭാര്യത്വത്തില്‍ രണ്ടോ മൂന്നോ നാലോ ഇണകളുണ്ടാകട്ടെ നാല് പേരും പരിഗണിക്കപ്പെടുകയാണ്; എല്ലാ അര്‍ഥത്തിലും സംരക്ഷിക്കപ്പെടുകയാണ്. അവര്‍ക്കെല്ലാം ജീവസന്ധാരണത്തിന് ആവശ്യമായ സംവിധാനങ്ങളുണ്ടാക്കപ്പെടുകയും അവര്‍ക്കെല്ലാം ലൈംഗികമായ അവകാശങ്ങള്‍ നല്‍കപ്പെടുകയും ചെയ്യുന്നു. എന്നാല്‍ ബഹുഭര്‍തൃത്ത്വത്തിൽ അവളുടെ അവകാശങ്ങൾ ഹനിക്കപ്പെടുകയും അവൾ അവഗണിക്കപ്പെടുകയുമാണുണ്ടാവുക.

ഒന്നാമത്തെ പ്രശ്‌നം ലൈംഗികതയുമായി ബന്ധപ്പെട്ടത് തന്നെയാണ്. പെണ്ണിന് ലൈംഗികതയുടെ ഉണര്‍വിന് സ്വകാര്യതയും സ്പര്‍ശവുമാണ് ആവശ്യം. സ്വകാര്യതയില്ലാതെ അവള്‍ക്ക് സംതൃപ്തമായ ലൈംഗികബന്ധം സാധ്യമാകില്ല. ലൈംഗികമായ ഉണര്‍വുണ്ടാകുവാന്‍ അവള്‍ക്ക് സാഹചര്യമുണ്ടാകണം; സ്വകാര്യതയുണ്ടാകണം; ഇണയുടെ ശ്രമകാരമുണ്ടാവണം; ഒപ്പം തന്നെ അയാളുടെ സ്നേഹത്തോടെയുള്ള സ്പർശമനുഭവിക്കുവാനും കഴിയണം . സ്ത്രീ ലൈംഗികതയുടെ ഉദ്ദീപനാവയവങ്ങളിൽ പ്രധാനപ്പെട്ടതാണ് ത്വക്ക്. ചർമ്മത്തിലെ ഉദീപനകേന്ദ്രങ്ങളിൽ സ്നേഹത്തോടെ തലോടുമ്പോഴാണ് അവള്‍ക്ക് ആസക്തിയുണ്ടാകുന്നത്.

പെണ്ണിനെ സംബന്ധിച്ചിടത്തോളം അവളെ കൂടാതെ തന്റെ ഭര്‍ത്താവിന് മറ്റൊരു ഭാര്യയുണ്ടെങ്കിലും ലൈംഗികതയുടെ കാര്യത്തില്‍ ഒരു പ്രയാസമുണ്ടാകുന്നില്ല. അവന്റെ സാന്നിധ്യത്തിലും അവന്റെ സ്വകാര്യതയിലുമല്ലാതെ അവള്‍ക്ക് അവന്‍ ലൈംഗികമായി അനിവാര്യമായി വരുന്നൊരു സാഹചര്യമുണ്ടാകുന്നില്ല. എന്നാല്‍ പുരുഷനതല്ലതാനും. കാഴ്ചയാണ് പുരുഷന്റെ പ്രധാനമായും ആസക്തിയുള്ളവനാക്കിത്തുതീർക്കുന്നത്. അവന്റെ ഒന്നാമത്തെ ഉത്തേജനന്കേന്ദ്രം കണ്ണാണ്. കാഴ്ചമൂലം അവന്‍ പ്രലോഭിതനാവുകയും സ്നേഹസ്രവമാടക്കം ഉത്പാദിപ്പിക്കപ്പെടുകയും ചെയ്യും. അതുകൊണ്ടാണ് മുഹമ്മദ് നബി (സ) പറഞ്ഞത് 'നിങ്ങള്‍ പുറത്ത് ഏതെങ്കിലും പെണ്ണിനെ കണ്ട് ആഗ്രഹം തോന്നിപ്പോയാല്‍ നിങ്ങളുടെ വീടുകളിലെ ഇണകളിലേക്ക് പോകണം' എന്ന്. പുരുഷന്റെ പ്രകൃതിപരമായ സ്വാഭാവികതയാണത്. ആസക്തനായാല്‍ ഇണകളിലെത്തിച്ചേരാനുള്ള മാര്‍ഗം വേണം. താന്‍ ആഗ്രഹത്തോടുകൂടി ഇണയുടെ അടുത്തെത്തുമ്പോള്‍ അവളോടൊപ്പം മറ്റൊരു ഭര്‍ത്താവുണ്ടാവുന്നത് പ്രശ്‌നം സൃഷ്ടിക്കും. വലിയ കുഴപ്പത്തിന് അത് കാരണമാകും. ഇത്തരം കുഴപ്പങ്ങളില്‍ പുരുഷനെക്കാളധികം പ്രയാസപ്പെടേണ്ടി വരിക സ്ത്രീയായിരിക്കും.

ബഹുഭാര്യത്വം പോലെ ഒരു അവകാശമല്ല ബഹുഭര്‍തൃത്വം. ബഹുഭാര്യത്വത്തിലൂടെ സ്ത്രീ സംരക്ഷിക്കപ്പെടുകയും സാമൂഹികമായ ചില പ്രശ്‌നങ്ങള്‍ക്ക് പരിഹാരമുണ്ടാവുകയും ചെയ്യുമ്പോള്‍ ബഹുഭര്‍തൃത്വം മുഖേന സ്ത്രീയുടെയോ പുരുഷന്റെയോ ഒരു അവകാശവും നിറവേറ്റപ്പെടുകയോ സാമൂഹികമായ മറ്റെന്തെങ്കിലും പ്രശ്‌നങ്ങള്‍ പരിഹരിക്കപ്പെടുകയോ ചെയ്യുന്നില്ല. ബഹുഭര്‍തൃത്വം ഒന്നിനും ഒരു പരിഹാരമല്ല. മറിച്ച് ഒരുപാട് പ്രശ്‌നങ്ങള്‍ സൃഷ്ടിക്കുന്ന ഒരു പ്രശ്‌നം മാത്രമാണ്. കുടുംബജീവിതം തകരാറിലാവുകയും സാമൂഹിക ഭദ്രത തകരുകയുമാണ് ഇതിന്റെ ഫലം. വൈവാഹിക ജീവിതത്തിന്റെ അടിസ്ഥാനപരമായ ഉദ്ദേശ്യങ്ങളിലൊന്നുപോലും ബഹുഭര്‍തൃത്വം മുഖേന നിറവേറ്റപ്പെടുന്നില്ല. സ്ത്രീയുടെ നിലവാരം ഇടിയുകയും അവള്‍ അടിമയായി ആപതിക്കുകയും ചെയ്യുകയാണ് ബഹു ഭര്‍തൃത്വത്തിന്റെ പ്രായോഗിക പരിണതി.

ബഹുഭര്‍തൃത്വം സൃഷ്ടിക്കുന്ന പ്രശ്‌നങ്ങള്‍ നിരവധിയാണ്.

ഒന്ന്: ലൈംഗികബന്ധത്തിന്റെ കാര്യത്തില്‍ വിവിധ ഭര്‍ത്താക്കന്മാര്‍ക്കിടയില്‍ പെരുമാറ്റച്ചട്ടം ആവശ്യമായി വരുന്നു. മഹാഭാരതത്തില്‍ ബഹുഭര്‍തൃത്വം സ്വീകരിച്ച ദ്രൗപതിയുടെ ജീവിതത്തില്‍നിന്നുള്ള ഒരു സംഭവം ഇതിന് തെളിവാണ്. പഞ്ചപാണ്ഡവരില്‍ ഓരോരുത്തര്‍ക്കും രണ്ടര മാസക്കാലം വീതം പാഞ്ചാലി വീതിച്ചുനല്‍കിയിരുന്നുവത്രേ. ഒരാളോടൊപ്പം ശയനമുറിയിലിരിക്കുമ്പോള്‍ മറ്റുള്ളവരൊന്നും കടന്നുവരരുതെന്നായിരുന്നു അവര്‍ തമ്മിലുണ്ടായിരുന്ന കരാര്‍. ഒരിക്കല്‍ യുധിഷ്ഠിരനും പാഞ്ചാലിയുംകൂടി ശയനമുറിയിലായിരിക്കുമ്പോള്‍ അര്‍ജുനന്‍ അങ്ങോട്ട് കടന്നുചെന്നുകൊണ്ട് കരാര്‍ ലംഘിച്ചു. ഇതിനുള്ള പ്രായശ്ചിത്തമായി അര്‍ജുനന് പന്ത്രണ്ട് വര്‍ഷത്തെ വനവാസത്തിനു പോകേണ്ടിവന്നു എന്നാണ് കഥ. ലൈംഗികബന്ധത്തിന്റെ കാര്യത്തില്‍ പെരുമാറ്റച്ചട്ടമുണ്ടാക്കുന്നതിന്റെ അപ്രായോഗികത ഈ കഥയില്‍നിന്ന് സുതരാം വ്യക്തമാവുന്നുണ്ട്. വിവിധ ഭര്‍ത്താക്കന്മാര്‍ തമ്മിലുള്ള സംഘട്ടനത്തിനും പ്രശ്‌നങ്ങള്‍ക്കും അതു നിമിത്തമാകുന്നു.

രണ്ട്: ഗര്‍ഭധാരണത്തിനുശേഷമുണ്ടാകുന്ന പ്രശ്‌നങ്ങള്‍: ഒന്നിലധികം ഭര്‍ത്താക്കന്മാരുള്ള സ്ത്രീ ഗര്‍ഭിണിയായാല്‍ ആരാണ് പ്രസ്തുത ഗര്‍ഭത്തിന് ഉത്തരവാദിയെന്നു തീരുമാനിക്കാനാവില്ല. ഗര്‍ഭകാലത്ത് സ്ത്രീകള്‍ക്ക് ലഭിക്കേണ്ട ശുശ്രൂഷയെയും പരിചരണത്തെയും ഇത് പ്രതികൂലമായി ബാധിക്കും. ഗര്‍ഭിണിക്ക്  അസ്വസ്ഥതയും കഷ്ടതയും മാത്രമായിരിക്കും ഇത്തരമൊരു അവസ്ഥയിലുണ്ടാവുക. ഗര്‍ഭസ്ഥശിശു ആരുടേതാണെന്ന് അറിയാത്തതിനാല്‍ ആരുംതന്നെ ആത്മാര്‍ഥമായ ശുശ്രൂഷക്ക് തയാറാവുകയില്ല. സ്‌നേഹം മനസ്സിനകത്തുനിന്ന് സ്വമേധയാ നിര്‍ഗളിക്കുന്നതാണ്. യാന്ത്രികമായി നിര്‍മിച്ചെടുക്കാവുന്നതല്ല. ഗര്‍ഭിണികളുടെ ശുശ്രൂഷയും മറ്റു പരിചരണങ്ങളും സ്‌നേഹത്തില്‍നിന്ന് ഉയിര്‍കൊള്ളുന്നതാണ്; ആവണം. അല്ലാത്തപക്ഷം അത് യാന്ത്രികമായിരിക്കും. ഭര്‍ത്താവില്‍നിന്നും പരിചാരികയില്‍നിന്നും ലഭിക്കുന്ന ശുശ്രൂഷകള്‍ തമ്മില്‍ അത്തരം അവസ്ഥയില്‍ വ്യത്യാസമൊന്നുമുണ്ടാവുകയില്ല. ഗര്‍ഭിണി ആഗ്രഹിക്കുന്നത് അതല്ല. ഗര്‍ഭത്തിന്റെ ഉത്തരവാദിയില്‍നിന്നുള്ള സ്‌നേഹോഷ്മളമായ പരിചരണമാണ് അവള്‍ക്കാവശ്യം. അത് ആരാണെന്നറിയാത്തതിനാല്‍ അത്തരമൊരു പരിചരണം ലഭിക്കാതെ പോകുന്നു. അതുകൊണ്ടുതന്നെ ബഹുഭര്‍തൃത്വം സ്‌ത്രൈണ പ്രകൃതിയോടുതന്നെ ചെയ്യുന്ന അനീതിയായി ഭവിക്കും.

മൂന്ന്: കുട്ടികളുടെ പിതൃത്വത്തിന്റെ പ്രശ്‌നം: ബഹുഭര്‍തൃത്വത്തിലൂടെ ഉണ്ടാവുന്ന കുട്ടികളുടെ പിതാക്കള്‍ ആരൊക്കെയാണെന്ന് മനസ്സിലാക്കാന്‍ കഴിയില്ല. പിതാക്കളില്‍നിന്നു ലഭിക്കേണ്ട സ്‌നേഹം കുട്ടികള്‍ക്കു ലഭിക്കാതിരിക്കുന്നതിന് ഇതു കാരണമാകുന്നു. കുട്ടികളുടെ സംരക്ഷണം മാതാക്കളുടെ ബാധ്യതയായിത്തീരുന്നു. അത് അവര്‍ക്ക് പ്രശ്‌നങ്ങളുണ്ടാക്കുന്നു. രക്തപരിശോധനയിലൂടെയും 'ഡി.എന്‍.എ-വിരലടയാള' പരിശോധനയിലൂടെയും യഥാര്‍ഥ പിതാവിനെ കണ്ടുപിടിക്കാ'മല്ലോയെന്നു വേണമെങ്കില്‍ വാദിക്കാമെന്നത് ശരിയാണ്. പക്ഷേ, ഒരു കുഞ്ഞിന് പിതൃവാല്‍സല്യം ലഭിക്കണമെങ്കില്‍ ലാബോറട്ടറി റിസല്‍ട്ട് കാത്തിരിക്കണമെന്ന സാഹചര്യം എന്തുമാത്രം വലിയ അനീതിയല്ല! വൈദ്യപരിശോധനയിലൂടെ തെളിയിക്കപ്പെട്ട പിതൃത്വത്തിന് തന്റെ സന്താനങ്ങളോട് എത്രത്തോളം വൈകാരികമായ ബന്ധമുണ്ടാവുമെന്ന് ഊഹിച്ചാല്‍ മനസ്സിലാക്കാവുന്നതാണ്. ചുരുക്കത്തില്‍ പിതൃ-പുത്രബന്ധത്തിന് പ്രകൃതി നിശ്ചയിച്ച വൈകാരികഭാവങ്ങള്‍ക്ക് വിരുദ്ധമാണ് ബഹുഭര്‍തൃത്വമെന്ന സമ്പ്രദായം.

നാല്: അനന്തരാവകാശത്തിന്റെ പ്രശ്‌നം: പിതാവിനെ തിരിച്ചറിയാതിരിക്കുന്നതുമൂലം വന്നുഭവിക്കുന്ന ഒരു പ്രശ്‌നമാണിത്. ബഹുഭര്‍തൃത്വത്തിലൂടെ ജനിച്ച കുഞ്ഞിന് ഏത് ഭര്‍ത്താവിന്റെ സ്വത്താണ് നല്‍കുക? കുഞ്ഞുങ്ങള്‍ക്കെല്ലാം തുല്യമായി വീതിക്കാമെന്ന് കരുതാന്‍ കഴിയില്ല. ഒരു ഭര്‍ത്താവ് പണക്കാരനും മറ്റെയാള്‍ പാവപ്പെട്ടവനുമായിരിക്കാനുള്ള സാധ്യതയുണ്ടല്ലോ. അത്തരം അവസരങ്ങളില്‍ ഏതൊക്കെ മക്കള്‍ക്ക് ആരുടെയൊക്കെ സ്വത്താണ് വീതിക്കുക? ഏതെങ്കിലും ഒരു ഭര്‍ത്താവ് മരണപ്പെട്ടാല്‍ എല്ലാ മക്കള്‍ക്കും സ്വത്ത് നല്‍കണമോ? അതല്ല അയാളുടെ മക്കള്‍ക്ക് മാത്രം നല്‍കണമോ? ഇങ്ങനെ ഒട്ടനവധി പ്രശ്‌നങ്ങളുണ്ട്.

അഞ്ച്: വാര്‍ധക്യത്തിലെ സംരക്ഷണത്തിന്റെ പ്രശ്‌നം: ഒന്നിലധികം ഭര്‍ത്താക്കന്മാരുള്ള സ്ത്രീയെ ആരാണ് സംരക്ഷിക്കുക? വാര്‍ധക്യത്തില്‍ അവരുടെ തുണയ്ക്ക് ആരാണുണ്ടാവുക? അവളുടെ സംരക്ഷണ ഉത്തരവാദിത്തം ഭര്‍ത്താക്കന്മാര്‍ പങ്കിട്ടെടുത്തുവെന്ന് കരുതുക. അത്തരമൊരവസ്ഥയില്‍ ഈ സംരക്ഷണം തികച്ചും യാന്ത്രികമായിരിക്കും. സ്‌നേഹത്തില്‍നിന്നുണ്ടാവുന്ന സംരക്ഷണമല്ല അപ്പോള്‍ ലഭിക്കുക. സംരക്ഷണത്തിന്റെ കാര്യത്തില്‍ ഭര്‍ത്താക്കന്മാര്‍  തമ്മില്‍ കലഹമുണ്ടാവാനും അങ്ങനെ സ്ത്രീ അരക്ഷിതയായിത്തീരുവാനുള്ള സാധ്യതയുമുണ്ട്. സ്ത്രീയുടെ സംരക്ഷണത്തിനുവേണ്ടി ജഗന്നിയന്താവ് നിശ്ചയിച്ച സംവിധാനങ്ങളെ നിഷേധിക്കുന്നവര്‍ക്കു മാത്രമേ ബഹുഭര്‍തൃത്വം കരണീയമായി തോന്നൂ.

ആറ്: പുരുഷന്മാര്‍ തമ്മിലുള്ള കലഹം: ഭാര്യയെച്ചൊല്ലി ഭര്‍ത്താക്കന്മാര്‍ക്കിടയില്‍ കലഹമുണ്ടാകുവാന്‍ സാധ്യതയേറെയാണ്. ലൈംഗികബന്ധത്തിന്റെയും കുഞ്ഞിന്റെയും കുഞ്ഞിന്റെ പിതൃത്വത്തിന്റെയുമെല്ലാം  പേരില്‍ കലഹങ്ങളുണ്ടാവാം. ഈ കലഹങ്ങള്‍ കുടുംബസംവിധാനത്തിന്റെ തകര്‍ച്ചക്കും സ്ത്രീയുടെ നാശത്തിനും നിമിത്തമാകും.

ചുരുക്കത്തിൽ ബഹുഭാര്യത്വം പല പ്രശ്നങ്ങൾക്കുമുള്ള പരിഹാരമാണ്; ബഹുഭർതൃത്വമാകട്ടെ, നിരവധി പ്രശ്നങ്ങൾ സൃഷ്ടിക്കുന്ന ഒരു പ്രശ്നം മാത്രമാണ്. അത് കൊണ്ട് തന്നെയായിരിക്കണം മനുഷ്യരെക്കുറിച്ച് കൃത്യമായറിയാവുന്ന സ്ര്ഷ്ടാവ് ബഹുഭർതൃത്വം അനുവദിക്കാതിരുന്നത്.

വിഷയവുമായി ബന്ധപ്പെട്ട വീഡിയോ

ബഹുഭാര്യത്വം ഇസ്‌ലാം അനുവദിച്ചത് ഉഹ്ദ് യുദ്ധാനന്തരമുള്ള സാഹചര്യത്തിലാണ് എന്ന് പല ഖുര്‍ആന്‍ വ്യാഖ്യാതാക്കളും പറയുന്നുണ്ടെന്നും അതുകൊണ്ട് തന്നെ അത് അക്കാലഘട്ടത്തിലേക്ക് മാത്രമുള്ള ഒരു നിയമമാണെന്നും ചില ബുദ്ധിജീവികള്‍ ഈയിടെ എഴുതിക്കെണ്ടിരിക്കുന്നുണ്ട്. ഖുര്‍ആനിലെ നാലാമത്തെ അധ്യായം: സൂറ: നിസാഇലെ 3-ാം വചനത്തിന്റെ അവതരണപശ്ചാത്തലം ഉഹ്ദ് യുദ്ധത്തിന് ശേഷമുള്ള സാഹചര്യമാണെന്ന് പല പണ്ഡിതന്മാരും പറഞ്ഞിട്ടുണ്ടെന്നത് ശരിയാണ്.

ഇസ്‌ലാം ഒരു നിയമം നടപ്പാക്കുന്നത് പരിശുദ്ധഖുര്‍ആനിന്റെ പദപരമായ വ്യാഖ്യാനത്തിലധിഷ്ഠിതമായിക്കൊണ്ടല്ല. അല്ലാഹുവിന്റെ വചനങ്ങളായി അവതരിപ്പിക്കപ്പെട്ട  ഖുര്‍ആനിന്റെ പ്രായോഗിക വിശദീകരണം നല്‍കിയത് പ്രവാചകൻ (സ) ആണ്. പ്രവാചകനില്‍ നിന്ന് മതം നേരിട്ട് പഠിച്ച സഹാബികള്‍ അത് പ്രയോഗവല്‍ക്കരിച്ച് ലോകത്തിന് മാതൃകയായി. അപ്പോള്‍ ഖുര്‍ആനാകുന്ന പ്രമാണം പ്രവാചകൻ (സ) എങ്ങനെ വിശദീകരിച്ചു, അതനുസരിച്ച് സഹാബികള്‍ എങ്ങനെ ജീവിച്ചു, എന്നതാണ് ഇസ്‌ലാമിക കര്‍മശാസ്ത്രത്തിന്റെയും നിയമങ്ങളുടെയും അടിത്തറ. അടിസ്ഥാനസോത്രസ്സ് ഖുര്‍ആന്‍, അതിന്റെ പ്രയോഗവല്‍ക്കരണം പ്രവാചകചര്യ അഥവാ സുന്നത്ത്, ഇവയില്‍ നിന്ന് സഹാബി സമുച്ചയം മനസ്സിലാക്കിയതെന്ത് എന്നീ കാര്യങ്ങള്‍ മൊത്തത്തില്‍ പരിഗണിച്ചുകൊണ്ടാണ് ഇസ്‌ലാമിക നിയമങ്ങള്‍ നിര്‍ധരിക്കപ്പെടുന്നത്.

ഈ രീതിശാസ്ത്രപ്രകാരം ഖുര്‍ആനിലെ 4:3ന്റെ അവതരണശേഷം പ്രവാചകന്‍ എന്തു നിലപാട് സ്വീകരിച്ചു. പ്രവാചകനില്‍ നിന്ന് മതം പഠിച്ച സഹാബിമാര്‍ എന്തു നിലപാട് സ്വീകരിച്ചു, ഇതാണ് പരിശോധനാവിധേയമാക്കേണ്ടത്. അവ നമ്മെ അറിയിക്കുന്ന സത്യം,  ഈ ഖുര്‍ആനികവചനമിറങ്ങിയതോടു കൂടി നാലിലധികം ഭാര്യമാരുണ്ടായിരുന്ന ആളുകളോട് നാലില്‍ പരിമിതപ്പെടുത്തുവാന്‍ റസൂൽ (സ) നിഷ്‌കര്‍ഷിച്ചുവെന്നതാണ്. അതായത്, യുദ്ധത്തിന് ശേഷമുള്ള സാഹചര്യങ്ങളിലും യുദ്ധമില്ലാത്ത സാഹചര്യങ്ങളിലുമെല്ലാം ഒന്നിലധികം ഭാര്യമാരെ വേള്‍ക്കുന്നത് പ്രവാചകൻ (സ) നിരോധിച്ചില്ല; എതിര്‍ത്തില്ല. സഹാബിമാരില്‍ ഒരുവിധമെല്ലാവര്‍ക്കും ഒന്നിലധികം ഭാര്യമാരുണ്ടായിരുന്നു. അവരെല്ലാവരും യുദ്ധസാഹചര്യത്തിലൊന്നുമല്ല കല്യാണം കഴിച്ചത്. യുദ്ധസാഹചര്യത്തില്‍ വിവാഹിതരായിട്ടുണ്ട്; അതല്ലാത്തപ്പോഴും വിവാഹം കഴിച്ചിട്ടുണ്ട്. അപ്പോള്‍ ഇതില്‍ നിന്ന് ഒരു നിയമം എന്ന നിലക്ക് അടിസ്ഥാനസ്രോതസ്സ്, അതിന്റെ വിശദീകരണം, അതിന്റെ പ്രയോഗവല്‍ക്കരണം, ഇവ മൂന്നും കൂടി വെച്ചുനോക്കുമ്പോള്‍ മനസ്സിലാകുന്നത് ഖുര്‍ആന്‍ പറഞ്ഞ ബഹുഭാര്യത്വത്തിനുള്ള അനുവാദം യുദ്ധസാഹചര്യത്തില്‍ മാത്രമല്ലയെന്നാണ്.

ഇനി, ബഹുഭാര്യത്വമനുവദിച്ചത് യുദ്ധസാഹചര്യത്തില്‍ മാത്രമാണെന്ന് വിചാരിച്ചാല്‍ തന്നെ, ഇസ്‌ലാമിക നിയമങ്ങള്‍ ഇപ്പോള്‍ പ്രയോഗവല്‍ക്കരിക്കാന്‍ കഴിയുന്നതല്ലെന്ന് നമുക്കെങ്ങനെ പറയാന്‍ പറ്റും? എങ്ങനെ ഈ നിയമം വേണ്ട എന്നു പറയാന്‍ പറ്റും? യുദ്ധസാഹചര്യം എപ്പോഴും ഉണ്ടാകാമല്ലോ? യുദ്ധസാഹചര്യത്തില്‍ ഇറങ്ങിയതാണ്; അത്‌കൊണ്ട് ഇന്ത്യയില്‍ ബഹുഭാര്യത്വം നിരോധിക്കണമെന്നാണ് ചില ബുദ്ധിജീവികളുടെ വാദം. ഇന്ത്യയിലത് വിരോധിക്കപ്പെട്ടുവെന്ന് കരുതുക ബഹുഭാര്യത്വം അനുവദിച്ച യുദ്ധസാഹചര്യം ഇന്ന് ഉണ്ടായാല്‍ എന്തുചെയ്യും?  യുദ്ധത്തിന്റെ ഫലമെന്താണ്? അവിടെ അനാഥകളുടെ പ്രശ്‌നമുണ്ടാകും, യുദ്ധത്തില്‍ വിധവകളുണ്ടാകും. കാരണം, പുരുഷന്മാരാണ് യുദ്ധത്തില്‍ നേര്‍ക്കുനേരെ പങ്കെടുക്കുന്നവര്‍. ഇസ്‌ലാമികസമൂഹത്തില്‍ പുരുഷന്മാര്‍ തന്നെയാണ് ഭൂരിപക്ഷവും യുദ്ധത്തില്‍ പങ്കെടുക്കുന്നത്. അങ്ങനെ പങ്കെടുക്കുമ്പോള്‍ സ്വാഭാവികമായും സ്ത്രീകളുടെ വൈധവ്യം, കുട്ടികളുടെ അനാഥത്വം എന്നിവയുണ്ടാകും. ആ സമയത്ത് മാത്രമുള്ള ഒരു നിയമമാണെങ്കില്‍ പോലും ആ നിയമം ഇന്നും പ്രസക്തമാണ്. ഏതു സമയത്തും യുദ്ധമുണ്ടാകാറുണ്ട്. ഏത് സമയത്തും യുദ്ധം മൂലമുള്ള പ്രയാസങ്ങളുണ്ടാകാം. ഏത് സമയത്തും സ്ത്രീകളുടെ എണ്ണം കൂടുന്ന അവസ്ഥയുണ്ടാകാം. അതിനുള്ള സാഹചര്യങ്ങള്‍ ഇപ്പോഴും നിലനില്‍ക്കുന്നുണ്ട്. അങ്ങനെയാകുമ്പോള്‍ ഈ നിയമം അപ്രസക്തമാണ്; അപരിഷ്‌കൃതമാണ് എന്നു പറയുന്നതിൽ ന്യായമില്ല.

ഇസ്‌ലാം സ്ത്രീയെ ലൈംഗികജീവിയായി ആപതിപ്പിച്ചുവെന്നും അതുകൊണ്ടാണ് ഇസ്‌ലാം ബഹുവഭാര്യത്വം പോലെയുള്ള കാര്യങ്ങൾ അനുവദിച്ചന്നതെന്നുമാണ് ആരോപണം. അവളുടെ വ്യക്തിത്വത്തിന്റെ പ്രകാശനത്തിന് പറ്റുന്ന കാര്യങ്ങളൊന്നും ഇസ്‌ലാം അനുവദിച്ചിട്ടില്ലെന്നും ഇസ്‌ലാം ഈ രംഗത്ത് സ്ത്രീയെ തീരെ പരിഗണിച്ചിട്ടില്ലെന്നും ആരോപിക്കപ്പെടുന്നു.

സ്ത്രീയും പുരുഷനും ലൈംഗികജീവികളാണെന്ന് സമര്‍ഥിക്കാന്‍ വേണ്ടി തന്റെ ആയുസ്സ് മുഴുവനും ചെലവഴിച്ച ഫ്രോയിഡിനെ അംഗീകരിക്കുകയും ആദരിക്കുകയും, അദ്ദേഹത്തിന്റെ സിദ്ധാന്തങ്ങളാണ് ശരി എന്നു വാദിക്കുകയും ചെയ്യുന്നവര്‍ സ്ത്രീ ലൈംഗികതയുമായി ബന്ധപ്പെട്ട ചര്‍ച്ചയില്‍ മാത്രമെന്തുകൊണ്ടാണ് ഏകപക്ഷീയമായ നിലപാടെടുക്കുന്നത്? ലൈംഗികജീവിയെന്നതിന് ലൈംഗികത മാത്രം കഴിയുന്ന ഒരാള്‍ എന്ന അര്‍ഥം ഫ്രോയിഡ് കൊടുത്തിട്ടില്ല. സെക്‌സിനുള്ള ഒരുപകരണം മാത്രമാണ് സ്ത്രീയെന്ന് ഇസ്‌ലാമും പറഞ്ഞിട്ടില്ല. മനുഷ്യന്റെ നാഗരിക മുന്നേറ്റങ്ങള്‍ക്കെല്ലാമുള്ള അടിത്തറയായി വര്‍ത്തിച്ചത് ലൈംഗികതയെ പ്രത്യുല്‍പാദനത്തിനപ്പുറത്ത് തൃപ്തിക്കുവേണ്ടിയും സ്‌നേഹത്തിനുവേണ്ടിയുമുള്ള ഏര്‍പ്പാടാക്കി മനുഷ്യന്‍ പരിണമിപ്പിച്ചതാണ് എന്നാണ് ഫ്രോയിഡ് സ്ഥാപിക്കാന്‍ ശ്രമിക്കുന്നത്. അതിന്റെ ശരിതെറ്റുകള്‍ ഇപ്പോള്‍ നമ്മുടെ വിഷയമല്ല. പക്ഷെ ഒരു കാര്യം ഉറപ്പാണ്. മനുഷ്യന്റെ വൈയക്തികവും കുടുംബപരവും സാമൂഹികവുമായ സ്വാസ്ഥ്യത്തിന് ലൈംഗികത അനിവാര്യമാണ് എന്നുള്ള സത്യം. ഇത് ശാസ്ത്രീയമായി തെളിയിക്കപ്പെട്ടതാണ്.

മനുഷ്യരുടെ സ്വാസ്ഥ്യവും സമാധാനവും ലൈംഗിക സംതൃപ്തിയുമായി ബന്ധപ്പെട്ടു കിടക്കുന്നു. ലൈംഗിക സംതൃപ്തി മനുഷ്യന് അനിവാര്യമാണ്. ആ തൃഷ്ണ പൂര്‍ത്തീകരിക്കപ്പെടണം. പൂര്‍ത്തീകരിക്കപ്പെടേണ്ട ലൈംഗികതയില്‍ പുരുഷന് അവന്റെ പങ്കുണ്ട്, സ്ത്രീക്ക് അവളുടെ പങ്കുണ്ട്. സ്ത്രീയുടെ ലൈംഗികചോദനവും ലൈംഗികതലവും ലൈംഗികമായ ആസ്വാദനവും പുരുഷന്റേതില്‍ നിന്ന്് വ്യത്യസ്തമാണ്. പുരുഷന്റേത് സ്ത്രീയില്‍ നിന്നും വ്യത്യസ്തമാണ്. എന്നാല്‍ ഇവ തമ്മില്‍ ഒരു പാരസ്പര്യം നില നില്‍ക്കുന്നുമുണ്ട്. ഇത് വളരെ കൃത്യമായി മനസ്സിലാക്കാന്‍ ആള്‍ഫ്രഡ് കിന്‍സെയുടെ റിപ്പാര്‍ട്ടുകള്‍ പരിശോധിച്ചാല്‍ മതി. അതില്‍ Sexual behaviour in the human male എന്നും  Sexual behaviour in the human female എന്നും രണ്ടായി തന്നെ തരംതിരിക്കുന്നുണ്ട്. മനുഷ്യന്റെ സ്വാസ്ഥ്യത്തിനും അവന്റെ നിലനില്‍പ്പിനും അവന്റെ പുരോഗതിക്കും അവന്റെ അസ്തിത്വത്തിന് തന്നെയും അനിവാര്യമായ ലൈംഗികയുടെ അതിന്റെ തൃപ്തമായ പൂര്‍ത്തീകരണത്തില്‍ പുരുഷന് അവന്റെ പങ്കുണ്ട്, സ്ത്രീക്ക് അവളുടെ പങ്കുണ്ട്. ആ പങ്കു നിര്‍വഹിക്കുവാന്‍ പറ്റിയ രൂപത്തിലാണ് സ്ത്രീ സൃഷ്ടിക്കപ്പെട്ടിട്ടുള്ളത്; പുരുഷനും സൃഷ്ടിക്കപ്പെട്ടിട്ടുള്ളത് അവന്റെ പങ്ക് നിര്‍വഹിക്കുവാന്‍പറ്റിയ രൂപത്തില്‍ തന്നെ. സ്ത്രീക്ക് പ്രത്യുല്‍പാദനത്തില്‍ കൂടുതല്‍ പങ്കുണ്ട് എന്നതിനാല്‍ ആ പ്രത്യുല്‍പാദനവുമായി ബന്ധപ്പെട്ട് പ്രയാസങ്ങള്‍ സഹിക്കേണ്ടതും ബാധ്യതകളുള്ളതും  പ്രശ്‌നങ്ങളുള്ളതും അബലതകള്‍ കടന്നുവരുന്നതും സ്ത്രീക്കാണ്. അതിനാല്‍ സ്ത്രീയെ സംരക്ഷിക്കേണ്ടത് പുരുഷന്റെ ഉത്തരവാദിത്തമാണെന്ന് ഇസ്‌ലാം പറഞ്ഞുവെന്ന് മാത്രമേയുള്ളു.

വിവാഹം കുടുംബമെന്ന സ്ഥാപനത്തിലേക്കുള്ള പ്രവേശനമാണ്. ലൈംഗിക സംപൂര്‍ത്തീകരണം രണ്ടുപേര്‍ക്കും ആവശ്യമാണ്. രണ്ടുപേരുടെയും വ്യക്തിപരമായ സ്വാസ്ഥ്യത്തിന് അത് അനിവാര്യമാണ്;  കുടുംബത്തിലെ സമാധാനത്തിന് അനിവാര്യമാണ്; സാമൂഹികമായ നന്മകള്‍ക്കും അനിവാര്യമാണ്. ഫ്രോയിഡിന്റെ പഠനങ്ങളെടുത്ത് നോക്കിയാല്‍ രസകരമായ ചില കാര്യങ്ങള്‍ നമ്മളറിയും. സാമൂഹ്യദ്രോഹികളായി അറിയപ്പെടുന്ന ആളുകളുടെ മനോവിശ്ലേഷണത്തിലൂടെ അദ്ദേഹം എത്തിച്ചേര്‍ന്ന നിഗമനം ഇവയുടെയെല്ലാം അടിസ്ഥാനകാരണം അതൃപ്തമായ ലൈംഗികതയാണെന്നാണ്. അപ്പോള്‍ സമൂഹത്തിന്റെ സ്വാസ്ഥ്യത്തിന് സംതൃപ്ത ലൈംഗികത ആവശ്യമാണ്. അതിന് സ്ത്രീ ആവശ്യമാണ്; അവിടെ പുരുഷനും ആവശ്യമാണ്. അവിടെ രണ്ടു കൂട്ടര്‍ക്കും അവരുടേതായ റോളുകളുണ്ട്. പാരസ്പര്യത്തിന് അനിവാര്യമായ നിയമങ്ങള്‍ ആവശ്യമായി വരുന്നത് ഇവിടെയാണ്. പെണ്ണിന്റെയും ആണിന്റെയും ലൈംഗികതകൾ വ്യത്യസ്തമാണ്. ഇവയുടെ പാരസ്പര്യത്തെക്കുറിച്ച് പഠിക്കാന്‍ മാസ്‌റ്റേഴ്‌സിനും ജോണ്‍സണും സ്വന്തത്തെ തന്നെ സമര്‍പ്പിക്കേണ്ടി വന്നു. ഭാര്യഭര്‍ത്താന്മാരായ അവരുടെ ലൈംഗിക ചോദനയെക്കുറിച്ച്, അവരുടെ ലൈംഗികസ്വഭാവങ്ങളെക്കുറിച്ചുള്ള പഠനങ്ങളാണവര്‍ പുറത്ത് വിട്ടത്. സ്ത്രീയുടെ ലൈംഗികതയെക്കുറിച്ചോ, അവളുടെ ലൈംഗികമായ  അസ്തിത്വത്തെക്കുറിച്ചോ പൂര്‍ണമായി അവള്‍ക്ക് തന്നെ പൂർണമായും അറിയില്ലെന്നാണ് അവരുടെ പഠനങ്ങൾ വ്യക്തമാക്കിയത്. . പുരുഷന്റേതും അങ്ങനെത്തന്നെ. അവന്റെ സാധ്യതകളെന്തൊക്കെയെന്നും, അവളുടെ സാധ്യതകളെന്തൊക്കെയെന്നും കൃത്യവും വ്യക്തമായി അവര്‍ക്കറിയില്ല. ആ സാധ്യതകളെ മുഴുവനും പൂര്‍ണമായി പ്രകാശിപ്പിക്കാന്‍ കഴിയുന്ന രൂപത്തിലുള്ളതാകണം അവരുടെ പാരസ്പര്യവുമായി ബന്ധപ്പെടുന്ന നിയമങ്ങള്‍. പുരുഷന് വേണ്ടി സ്ത്രീ നിയമമുണ്ടാക്കിയാലും സ്ത്രീക്ക് വേണ്ടി പുരുഷന്‍ നിയമമുണ്ടാക്കിയാലും അവ മാനവികമാകുകയില്ല. രണ്ടിനും അപ്രായോഗികതയുണ്ടാകും. സ്ത്രീക്ക് വേണ്ടി സ്ത്രീ നിയമമുണ്ടാക്കിയാലും തഥൈവ. അതിന് പുരുഷപ്രകൃതിയെ ഉള്‍ക്കൊള്ളാനാവാതെ അത് ഒറ്റപ്പെട്ടുപോകും. പുരുഷന് വേണ്ടി പുരുഷന്‍ നിയമമുണ്ടാക്കിയാലും അങ്ങനെത്തന്നെയാണുണ്ടാവുക. ഇവിടെയാണ് രണ്ടുപേരെയും കൃത്യമായി അറിയാവുന്ന പടച്ചവന്റെ നിയമങ്ങള്‍ പ്രസക്തമാകുന്നത്. ആ നിയമത്തിന് മാത്രമെ സ്തീപുരുഷ പാരസ്പര്യത്തെ പ്രകാശമാനമാക്കാനാവൂ; രണ്ടു പേര്‍ക്കും പൂര്‍ണാര്‍ഥത്തിലുള്ള ആസ്വാദനം നല്‍കൂ; പടച്ചവന്റെ നിയമത്തില്‍ സ്ത്രീക്ക് കൊടുത്ത സ്ഥാനമല്ല പുരുഷന് കൊടുത്തിരിക്കുന്നത്. ആ സ്ഥാനങ്ങള്‍ക്ക് യാതൊരു കുഴപ്പവുമില്ല. അതിനെ കേവലം വിമര്‍ശനാത്മകമായി കണ്ട് പ്രാകൃതവല്‍ക്കരിക്കാന്‍ വേണ്ടി ശ്രമിക്കുമ്പോഴാണ് കുഴപ്പം.

ലൈംഗികതയുമായി ബന്ധപ്പെട്ട ഇസ്‌ലാമിക നിയമങ്ങളുടെ അടിത്തറ അവരുടെ പാരസ്പര്യത്തിലാണ് ഊന്നുന്നത്. ലൈംഗികത അനിവാര്യമാണ് രണ്ടു കൂട്ടര്‍ക്കും. ആ ലൈംഗികപൂര്‍ത്തീകരണത്തിന്റെ ഓരോ രംഗത്തും പെണ്ണ് സംരക്ഷിക്കപ്പെടണം. എന്തുകൊണ്ട്? പെണ്ണിന് മറ്റൊരു വലിയ ദൗത്യം ഏറ്റെടുക്കാനുണ്ട്; മാതാവാകുകയെന്ന ദൗത്യം. അതുകൊണ്ടാണ് വിവാഹത്തിന്റെ സമയത്ത് പെണ്ണിന് മഹര്‍ കൊടുക്കണമെന്ന് ഇസ്‌ലാം പ്രത്യേകമായി പറഞ്ഞത്. രണ്ടുകൂട്ടര്‍ക്കും ലൈംഗികത ആവശ്യമാണെങ്കില്‍  പിന്നെ  പെണ്ണിന് പുരുഷന്‍ ധനം കൊടുക്കണമെന്ന നിയമം എന്തുകൊണ്ടുണ്ടാകുന്നു; രണ്ട് പേര്‍ക്കും ആവശ്യമുള്ള വിവാഹത്തില്‍ പെണ്ണിന് ധനം ആവശ്യപ്പെടാനുള്ള അവകാശം എന്തുകൊണ്ടുണ്ടാകുന്നു? ലൈംഗികത വഴി ജൈവികമായ നഷ്ടങ്ങളുണ്ടാകുന്നത് പെണ്ണിനാണ്. അവളുടെ കൂടുതല്‍ സമര്‍പ്പണം ആവശ്യമുണ്ടവിടെ. കുടുംബമെന്ന സ്ഥാപനത്തിന്റെ നിലനില്‍പ്പിന്ന് പെണ്ണിന്റെ സമര്‍പ്പണം കൂടുതലാവശ്യമുണ്ട്. സ്ഥാപനത്തിന്റെ നിലനില്‍പിന്ന് അനിവാര്യമാണ് വിവാഹം; അത് സ്ത്രീ ശാക്തീകരണത്തിനെതിരല്ല.  മറിച്ച്, പെണ്ണ് വിചാരിച്ചാലേ ഗര്‍ഭിണിയാകാന്‍ കഴിയൂ. പെണ്ണ് വിചാരിച്ചാലേ പ്രസവിക്കാന്‍ കഴിയൂ. പെണ്ണിനേ മുലയൂട്ടാന്‍ കഴിയൂ. പെണ്ണിനേ കുട്ടികള്‍ക്ക് ചെറിയ പ്രായത്തിലുള്ള മാനസികവികാസത്തിനാവശ്യമായ ഗൃഹാന്തരീക്ഷം സൃഷ്ടിക്കാന്‍ കഴിയൂ. ഇതെല്ലാം പുരുഷനാവശ്യമാണ്; സ്ത്രീക്ക് ആവശ്യമാണ്; അടുത്ത തലമുറക്കാവശ്യമാണ്; സമൂഹത്തിനാവശ്യമാണ്; കുടുംബത്തിന്റെ സ്വാസ്ഥ്യത്തിന് ആവശ്യമായ മാതൃത്വമെന്ന കടമ അവള്‍ നിര്‍വഹിക്കുമ്പോള്‍ പുരുഷന്‍ അവള്‍ക്ക് താങ്ങാകണം. അവളെ ഒരു സാമ്പത്തിക ജീവിയായി കണ്ടുകൂടാ. സാമ്പത്തികസ്രോതസ്സ് അവളല്ല. അത് പുരുഷനായിത്തീരണം. അതാണ് ഇസ്‌ലാമിന്റെ കാഴ്ചപ്പാട്.

ലൈംഗികത പൂര്‍ത്തീകരിക്കപ്പെടേണ്ട  അനിവാര്യമായ വികാരണമാണെന്ന് തന്നെയാണ് ഇസ്‌ലാമിന്റെ വീക്ഷണം.  സ്ത്രീയെയും പുരുഷനെയും  ആ വികാരം നിലനില്‍ക്കുന്ന ജീവികളായി കാണണം. പ്രസ്തുത വികാരത്തിന്റെ പൂര്‍ത്തീകരണം വിവാഹത്തിലൂടെ നടക്കണം. കുടുംബജീവിതത്തിന്റെ ഓരോ രംഗത്തും അവരുടെ അവകാശങ്ങളും ഉത്തരവാദിത്തങ്ങളും കൃത്യമായി പഠിപ്പിക്കുന്നുണ്ട് ഇസ്‌ലാം. അതല്ലാതെ സ്ത്രീ വിരോധമോ പുരുഷ വിരോധമോ ഇസ്‌ലാമിക നിയമങ്ങളിലില്ല. ബഹുഭാര്യത്വവുമായി ബന്ധപ്പെട്ട നിയമങ്ങളും അങ്ങനെ തന്നെ. സ്ത്രീയുടെ ഏതെങ്കിലും ഒരവകാശം അതുവഴി നിഷേധിക്കപ്പെടുന്നില്ല. അതേസമയം ബഹുഭാര്യത്വം നിരോധിക്കുകയും ഒപ്പം തന്നെ ധാര്‍മികജീവിതം നിര്‍ബന്ധിക്കുകയും ചെയ്യുന്നത് പുരുഷന്റെ അടിസ്ഥാനപരമായ പ്രകൃതി ചിലയാളുകള്‍ക്കെങ്കിലും അടിച്ചമര്‍ത്തേണ്ടിവരുന്ന സാഹചര്യമുണ്ടാകുന്നു. ആ രംഗത്തെ പ്രഗല്‍ഭരായ ആളുകളുടെ പഠനമാണിത്. ഇവിടെ ഇസ്‌ലാമിക നിയമം പെണ്ണിനെ പരിഗണിക്കുന്നു. അവളുടെ അവസ്ഥകള്‍ പൂര്‍ണമായും മനസ്സിലാക്കുന്നു. അവളെ അടിച്ചമര്‍ത്തുവാനോ പ്രയാസപ്പെടുത്താനോ ഇസ്‌ലാം ഉദ്ദേശിക്കുന്നില്ല. ഇസ്‌ലാമിലെ നിയമങ്ങള്‍ അവളോട് യാതൊരുവിധ അനീതിയും ചെയ്യുന്നില്ല.

ലോകത്തെ മറ്റു സമൂഹങ്ങളിലേതുപോലെതന്നെ, പലപ്പോഴും മറ്റു സമൂഹങ്ങളെ കവച്ചുവെക്കുന്ന രീതിയില്‍ ബഹുഭാര്യത്വം അറേബ്യയില്‍ നിലവിലുണ്ടായിരുന്നു. അനിയന്ത്രിതമായ അനുവാദമായിരുന്നു ഇക്കാര്യത്തില്‍ അറബികള്‍ക്കിടയില്‍ നിലനിന്നിരുന്നത്. ഇതിനൊരു നിയന്ത്രണമുണ്ടാക്കുകയും നാലില്‍ പരിമിതിപ്പെടുത്തുകയുമാണ് ഖുര്‍ആന്‍ ചെയ്തത്. പല പ്രവാചകാനുചരന്മാര്‍ക്കും ഇസ്‌ലാം ആശ്ലേഷിക്കുന്നതിനു മുമ്പ് ഒരുപാട് ഭാര്യമാരുണ്ടായിരുന്നുവെന്ന വസ്തുത അറേബ്യന്‍ സമൂഹത്തില്‍ ഭാര്യമാരുടെ എണ്ണത്തിന് യാതൊരു പരിധിയുമുണ്ടായിരുന്നില്ലെന്ന് വ്യക്തമാക്കുന്നതാണ്. ഗീലാനുബ്‌നു സല്‍മത്തുസ്സക്കഫിക്ക് പത്തുഭാര്യമാരുണ്ടായിരുന്നു. അമീറത്തുല്‍ അസദിക്ക് എട്ട് ഭാര്യമാരും നൗഫലുബ്‌നു മുആവിയത്തുദ്ദയ്‌ലമിക്ക് അഞ്ച് ഭാര്യമാരുമുണ്ടായിരുന്നു. ഇസ്‌ലാം സ്വീകരിക്കുന്ന സമയത്ത് ഇഷ്ടമുള്ള നാലു ഭാര്യമാരെ നിലനിര്‍ത്തി ബാക്കിയുള്ളവരെ ഒഴിവാക്കാനാണ് പ്രവാചകന്‍ ല അവരോടാവശ്യപ്പെട്ടത്. കൈയും കണക്കുമില്ലാതെ എത്രയും ഭാര്യമാരെ വെച്ചുകൊണ്ടിരിക്കാമെന്ന അവസ്ഥ നിലനിന്നിരുന്ന സമൂഹത്തിലാണ് നീതി പുലര്‍ത്താനാവുമെങ്കില്‍ നാലു വരെ ആകാമെന്നും അതിനാവില്ലെങ്കില്‍ ഒന്നു മാത്രം മതിയെന്നുമുള്ള നിയമം കൊണ്ടുവന്നതെന്ന് സാരം.

അനിവാര്യമെന്നു തോന്നുന്നുവെങ്കില്‍ ഒന്നിലധികം ഭാര്യമാരെ സ്വീകരിക്കാന്‍ ഖുര്‍ആന്‍ അനുവാദം നല്‍കുന്നു. അവര്‍ക്കിടയില്‍ നീതി പാലിക്കണമെന്ന നിബന്ധനയോടെ.

ഒന്നിലധികം ഭാര്യമാരെ സ്വീകരിക്കുന്നതിനെ ആധുനികത വിലക്കുന്നു. നിയമാനുസൃതം ഒരു ഭാര്യ മാത്രമേ പാടുള്ളുവെന്ന് നിഷ്‌കര്‍ശിക്കുമ്പോള്‍തന്നെ കാള്‍ഗേളുകളുമായോ മറ്റോ ബന്ധം പുലര്‍ത്തുന്നതില്‍ അത് യാതൊരു തെറ്റും കാണുന്നില്ല.

ഏതാണ് സ്ത്രീകള്‍ക്ക് ഹിതകരമായ നിയമം?

വിവാഹേതര ബന്ധങ്ങള്‍, അതിന് എന്ത് പേരിട്ട് വിളിച്ചാലും ഇസ്‌ലാം അംഗീകരിക്കുന്നില്ല- വെറുക്കുന്നു എന്ന വസ്തുതയുടെ അടിത്തറയില്‍നിന്നുകൊണ്ടാണ് നാം ഈ പ്രശ്‌നത്തെ പരിശോധിക്കേണ്ടത്. ഇസ്‌ലാമികമായ ഭരണക്രമം നിലനില്‍ക്കുന്ന രാഷ്ട്രത്തിലാണെങ്കില്‍ വ്യഭിചരിച്ചവര്‍ക്ക്- നാല് ദൃക്‌സാക്ഷികളുടെ സാക്ഷ്യം കൊണ്ട് കുറ്റം തെളിഞ്ഞാല്‍- വിവാഹിതരല്ലെങ്കില്‍ നൂറ് അടിയും വിവാഹിതരെങ്കില്‍ മരണം വരെ കല്ലേറും ലഭിക്കും. വിവാഹത്തിന് പുറത്തുള്ള ലൈംഗിക ബന്ധത്തെ ഇസ്‌ലാം എന്തുമാത്രം വെറുക്കുന്നുവെന്ന് ഈ ശിക്ഷകള്‍ വ്യക്തമാക്കുന്നു. സമൂഹത്തിന്റെ അടിസ്ഥാന സ്ഥാപനമായ കുടുംബത്തിന്റെ തകര്‍ച്ചക്കും അതുവഴി ധാര്‍മികത്തകര്‍ച്ചക്കും വ്യഭിചാരം നിമിത്തമാവുമെന്നാണ് ഇസ്‌ലാമിന്റെ വീക്ഷണം. അതുകൊണ്ടുതന്നെ സദാചാര നിഷ്ഠമായ ഒരു സമൂഹത്തിന്റെ സൃഷ്ടിക്കുവേണ്ടി ശ്രമിക്കുന്ന ഒരു ദര്‍ശനത്തിന് അത് പൂര്‍ണമായി ഇല്ലാതാക്കുവാനാശ്യമായ നിയമങ്ങള്‍ ആവിഷ്‌കരിക്കേണ്ടി വരുന്നത് സ്വാഭാവികമാണ്. അതോടൊപ്പം മനുഷ്യപ്രകൃതിക്ക് ഇണങ്ങുന്ന നിയമങ്ങളില്‍ വികാരപൂര്‍ത്തീകരണമെന്ന ജൈവിക ആവശ്യം നിര്‍വഹിക്കുവാനുള്ള മാര്‍ഗങ്ങള്‍ ഉണ്ടാകുകയും വേണം. ഇവിടെയാണ് ഇസ്‌ലാം ബഹുഭാര്യത്വം അനുവദിച്ചതിലെ യുക്തി മനസ്സിലാക്കാനാവുന്നത്.

സദാചാരനിഷ്ഠമായ ഒരു സമൂഹത്തില്‍ ബഹുഭാര്യത്വം അനിവാര്യമാകുന്ന വൈയക്തികവും സാമൂഹികവുമായ അവസ്ഥകളുണ്ട്. വ്യക്തിപരമായ അവസ്ഥകളെ ഇങ്ങനെ സംക്ഷേപിക്കാം:

ഒന്ന്) പുരുഷന്റെ ലൈംഗികാസക്തി: ചില പുരുഷന്മാര്‍ക്കെങ്കിലും തങ്ങളുടെ ലൈംഗികാവശ്യങ്ങള്‍ക്ക് ഒരു സ്ത്രീ മതിയാകാതെ വരുന്ന സാഹചര്യങ്ങളുണ്ടെന്നത് നിഷേധിക്കാനാവാത്ത വസ്തുതയാണ്. സ്ത്രീയുടെ ആര്‍ത്തവകാലം, പ്രസവകാലം തുടങ്ങിയ കാലയളവുകളില്‍ ലൈംഗികബന്ധം അസാധ്യമാണല്ലോ. ഇത്തരം അവസ്ഥകളില്‍ ലൈംഗിക വികാരം നിയന്ത്രിക്കാന്‍ കഴിയാത്തവരുണ്ടാകാം. ബഹുഭാര്യത്വം അല്ലെങ്കില്‍ വ്യഭിചാരമാണ് അത്തരം ആളുകള്‍ക്ക് മുന്നിലുള്ള മാര്‍ഗം.

രണ്ട്) ഭാര്യയുടെ ലൈംഗികശേഷിയില്ലായ്മ: സ്ത്രീകളിലെ ലൈംഗികശേഷിക്കുറവ് ചിലപ്പോള്‍ ചികില്‍സിച്ചുമാറ്റാന്‍ കഴിയാത്തതാവാം. ഏതുതരം ശേഷിക്കുറവാണെങ്കിലും അതു നിലനില്‍ക്കുന്ന കാലയളവില്‍ പുരുഷന് വികാരശമനത്തിന് മാര്‍ഗം വേണമെന്നാണ് പ്രകൃതിയുടെ താല്‍പര്യം. ഒന്നുകില്‍ ബഹുഭാര്യത്വം അല്ലെങ്കില്‍ വ്യഭിചാരം. അതുമല്ലെങ്കില്‍ വിവാഹമോചനം. ഇങ്ങനെ മൂന്നു മാര്‍ഗങ്ങളുണ്ട് പുരുഷനു മുമ്പില്‍. വ്യഭിചാരം അധാര്‍മികമാണ്. വിവാഹമോചനം അനുവദനീയമെങ്കിലും കഴിയുന്നത്ര ഒഴിവാക്കാന്‍ ശ്രമിക്കേണ്ട കാര്യമാണ്. ഇത്തരം ഒരവസ്ഥയില്‍ ബഹുഭാര്യത്വമാണ് ഏറ്റവും കരണീയമായിട്ടുള്ളത്.

മൂന്ന്) ഭാര്യയുടെ വന്ധ്യത: ഭാര്യ വന്ധ്യയാണെങ്കില്‍ പുരുഷനു മുമ്പില്‍ മൂന്നു മാര്‍ഗങ്ങളുണ്ട്. ഒന്ന്. ജീവിതകാലം മുഴുവന്‍ കുട്ടികളില്ലാതെ ജീവിക്കുക. രണ്ട്. വന്ധ്യയായ സ്ത്രീയെ വിവാഹമോചനം ചെയ്തുകൊണ്ട് മറ്റൊരുത്തിയെ വേള്‍ക്കുക. മൂന്ന്. വന്ധ്യയായ സ്ത്രീയെ നിലനിര്‍ത്തിക്കൊണ്ടുതന്നെ മറ്റൊരുത്തിയെ വിവാഹം ചെയ്യുക.

ഒന്നാമത്തെ പരിഹാരം സ്വന്തത്തോടുചെയ്യുന്ന ക്രൂരതയാണ്. രണ്ടാമത്തേത് ഭാര്യയോടുള്ള ക്രൂരതയും: അവര്‍ ചെയ്ത തെറ്റുകൊണ്ടല്ല അവര്‍ വന്ധ്യയായിത്തീര്‍ന്നത്. മൂന്നാമത്തെ നിര്‍ദേശമാണ് മാനവികം. അതുവഴി ഭര്‍ത്താവിന്റെ കുഞ്ഞുങ്ങളെ തന്‍േറതെന്നവണ്ണം വളര്‍ത്തിക്കൊണ്ട് സായൂജ്യമണിയാന്‍ വന്ധ്യയായ സ്ത്രീക്കും അവസരം ലഭിക്കുന്നു. അങ്ങനെ മാതൃത്വത്തിന്റെ ദാഹം ശമിപ്പിക്കുവാന്‍ അവള്‍ക്കും സാധിക്കുന്നു.

നാല്) ഭാര്യയുടെ മാറാവ്യാധി: ചില രോഗങ്ങള്‍ ലൈംഗികബന്ധത്തെയും ഗര്‍ഭധാരണത്തെയും വിലക്കുന്നവയായുണ്ട്. അത്തരം രോഗങ്ങളുള്ള സ്ത്രീകളുടെ ഭര്‍ത്താക്കന്മാര്‍ എന്തുചെയ്യണം? മാറാരോഗം കാരണം ഗാര്‍ഹികജോലികള്‍ ചെയ്യാന്‍ പ്രയാസപ്പെടുന്നവരുമുണ്ടാകും. ഇവിടെയെല്ലാം പരിഹാരമായി നിര്‍ദേശിക്കപ്പെടുന്നത് വ്യഭിചാരമോ വിവാഹമോചനമോ ബഹുഭാര്യത്വമോ ആണ്. മാറാവ്യാധി പിടിപെട്ട സ്ത്രീയെ ഒഴിവാക്കുന്നതിലൂടെ അവളെ വഴിയാധാരമാക്കുകയാണ് ചെയ്യുന്നത്. ഇവിടെയും മാനവികമായ മാര്‍ഗം ബഹുഭാര്യത്വംതന്നെയാണ്.

മുകളില്‍ പറഞ്ഞ സാഹചര്യങ്ങളില്‍ വിവാഹമോചനമാണ് ചില മതഗ്രന്ഥങ്ങള്‍ ശുപാര്‍ശ ചെയ്യുന്നത്. മനുസ്മൃതിയുടെ കല്‍പന കാണുക:

വന്ധ്യാഷ്ട മേധി വേദ്യാബ്‌ദേ ദേശമേതുമൃതപ്രജാ

ഏകാ ദശേ സ്ത്രീ ജനനീ സത്യസ്ത്വപ്രിയ വാദിനീ

(മനുസ്മൃതി 9:81)

(വന്ധ്യയായ ഭാര്യയെ എട്ടു വര്‍ഷം കഴിഞ്ഞും, ചാപിള്ള പ്രസവിക്കുന്നവളെ പത്തുവര്‍ഷം കഴിഞ്ഞും, പെണ്ണുമാത്രം പ്രസവിക്കുന്നവളെ പതിനൊന്നുവര്‍ഷം കഴിഞ്ഞും, അപ്രിയം പറയുന്നവളെ ഉടനെയും ഉപേക്ഷിച്ച് വേറെ വിവാഹം ചെയ്യേണ്ടതാണ്. ഈ സ്ത്രീകള്‍ക്ക് സന്തോഷത്തിനായി യാതൊന്നും കൊടുക്കേണ്ടതില്ല).

യുക്തിവാദത്തിന്റെ പരിഹാരവും വിവാഹമോചനംതന്നെ. നിരീശ്വരത്വത്തിന്റെ ഏറ്റവും വലിയ തത്ത്വജ്ഞാനിയായി അറിയപ്പെടുന്ന ബര്‍ട്രന്‍ഡ് റസ്സല്‍ നിര്‍ദേശിക്കുന്ന പരിഹാരം കാണുക:

'സന്താനങ്ങളില്ലാത്ത വൈവാഹിക ജീവിതത്തില്‍, ഇരുകൂട്ടരും നന്നായി പെരുമാറുവാന്‍ പരമാവധി പരിശ്രമിക്കുന്നുണ്ടെന്നുവരികിലും, വിവാഹമോചനംതന്നെയാണ് ഏറ്റവും നല്ല പരിഹാരം' Marriage and Morals, Page:96)

ഇത്തരം അവസരങ്ങളില്‍ ബഹുഭാര്യത്വമോ വിവാഹമോചനമോ വ്യഭിചാരമോ ഏതാണ് ഒരു സ്ത്രീ ഭര്‍ത്താവില്‍നിന്ന് ആഗ്രഹിക്കുക? സന്മാര്‍ഗനിഷ്ഠയും സ്‌നേഹവതിയുമായ സ്ത്രീ തീര്‍ച്ചയായും കാംക്ഷിക്കുന്നത് ബഹുഭാര്യത്വമായിരിക്കും. ഇസ്‌ലാം ബഹുഭാര്യത്വം അനുവദിച്ചിരിക്കുന്നത് സ്ത്രീകളുടെകൂടി രക്ഷ കണക്കിലെടുത്തുകൊണ്ടാണ്. ഖുര്‍ആനിക ദര്‍ശനം മാനവികമാണെന്ന വസ്തുതയാണ് നമുക്കിവിടെ കാണാന്‍ കഴിയുന്നത്.

ബഹുഭാര്യത്വം സാമൂഹികമായ അനിവാര്യതയായിത്തീരുന്ന സന്ദര്‍ഭങ്ങളുമുണ്ട്. അവയെ ഇങ്ങനെ സംക്ഷേപിക്കാം:

ഒന്ന്) സ്ത്രീ-പുരുഷ അനുപാതത്തിലുണ്ടാവുന്ന വ്യത്യാസം: ഇതു രണ്ടു രൂപത്തില്‍ സംഭവിക്കാം; സ്വാഭാവികമായും യുദ്ധത്തിന്റെ ഫലമായും. ചരിത്രം പരിശോധിച്ചാല്‍ നമുക്ക് കാണാന്‍ കഴിയുന്ന ഒരു യാഥാര്‍ഥ്യമുണ്ട്. ഏതാണ്ട് എല്ലാ സമൂഹങ്ങളിലും എല്ലാ കാലഘട്ടങ്ങളിലും സ്ത്രീകളുടെ എണ്ണം പുരുഷന്മാരുടേതിനേക്കാള്‍ കൂടുതലായിരുന്നുവെന്ന വസ്തുതയാണത്. ആധുനിക ശാസ്ത്രം ഈ അവസ്ഥക്ക് വിശദീകരണം നല്‍കുന്നുണ്ട്. മനുഷ്യരുടെ ജനിതകനില പ്രകാരം ഗര്‍ഭസ്ഥശിശുക്കളുടെ ലിംഗവ്യത്യാസം ഏതാണ്ട് സമമായിരിക്കുമെങ്കിലും പെണ്‍ഭ്രൂണത്തിന് ആണ്‍ഭ്രൂണത്തെക്കാള്‍ രോഗപ്രതിരോധശേഷി കൂടുതലായതിനാല്‍ ജനിക്കുന്ന കുഞ്ഞുങ്ങളില്‍ കൂടുതല്‍ പെണ്‍കുട്ടികളായിരിക്കുമത്രേ. ആയിരം ആണ്‍കുട്ടികള്‍ക്ക് ഏകദേശം ആയിരത്തിപത്ത് പെണ്‍കുട്ടികള്‍ എന്ന നിരക്കിലായിരിക്കും ഈ വ്യത്യാസമെന്നാണ് മനസ്സിലാക്കാന്‍ കഴിയുന്നത്.

യുദ്ധത്തിന് ശേഷമുണ്ടാകുന്ന അവസ്ഥയാണ് മറ്റൊന്ന്. സ്വാഭാവികമായും യുദ്ധത്തില്‍ പങ്കെടുക്കുന്നത് പുരുഷന്മാരായിരിക്കും. അങ്ങനെ സ്ത്രീ- പുരുഷ അനുപാതത്തില്‍ വലിയ വ്യത്യാസമുണ്ടാവും. രണ്ടാം ലോകമഹായുദ്ധത്തില്‍ ജര്‍മനിയിലെ അമ്പത് ലക്ഷം പുരുഷന്മാരാണ് മരിച്ചുവീണത്. യുദ്ധത്തിന് മുമ്പ് അവിടത്തെ സ്ത്രീ-പുരുഷ അനുപാതം സമമായിരുന്നുവെങ്കില്‍ യുദ്ധശേഷം അമ്പത് ലക്ഷം സ്ത്രീകള്‍ അധികമായി ഭവിച്ചിട്ടുണ്ടാകുമെന്നര്‍ത്ഥം. ഭര്‍ത്താക്കന്മാരെ നല്‍കണമെന്നാവശ്യപ്പെട്ടുകൊണ്ട് ജപ്പാനിലെയും ജര്‍മനിയിലെയും സ്ത്രീകള്‍ പ്രകടനം നടത്തി. അവരുടെ  വീടുകള്‍ക്കു മുമ്പില്‍ 'ഒരു സായാഹ്‌ന അതിഥിയെ ആവശ്യമുണ്ട് (wanted an evening guest)  എന്ന ബോര്‍ഡുകള്‍ പ്രത്യക്ഷപ്പെട്ടു. ഇത് ജര്‍മനിയുടെ മാത്രം അവസ്ഥയല്ല. യുദ്ധം കഴിഞ്ഞാല്‍ ഏതു സമൂഹത്തിലുമുണ്ടാവുന്ന സ്വാഭാവികമായ സ്ഥിതിവിശേഷമാണ്.

സമൂഹത്തില്‍ സ്ത്രീകളുടെ എണ്ണം പുരുഷന്മാരുടേതിനേക്കാള്‍ വര്‍ധിക്കുമ്പോള്‍ മൂന്നു പ്രതിവിധികളാണ് സമൂഹത്തിന് സ്വീകരിക്കുവാന്‍ കഴിയുക.

  1. ഓരോ പുരുഷനും ഓരോ സ്ത്രീയെ മാത്രം വിവാഹം ചെയ്യുക. ബാക്കിയുള്ള സ്ത്രീകള്‍ ലൈംഗികതൃഷ്ണ ഒതുക്കിക്കൊണ്ട് ജീവിക്കുക.
  2. ഓരോ പുരുഷനും ഓരോ സ്ത്രീയെ വിവാഹം ചെയ്യുക. ബാക്കിയുള്ള സ്ത്രീകള്‍ വ്യഭിചാരത്തിലേര്‍പ്പെടുക.
  3. പ്രാപ്തരും ഭാര്യമാരോട് നീതിയില്‍ വര്‍ത്തിക്കുവാന്‍ കഴിയുമെന്ന് തോന്നുന്നവരുമായ പുരുഷന്മാര്‍ ഒന്നിലധികം ഭാര്യമാരെ സ്വീകരിക്കുക.

ഈ മൂന്നു പരിഹാരങ്ങളില്‍ ഏതാണ് മാനവികം? വിവാഹത്തിന് കഴിയാത്ത സ്ത്രീകളെ പരിഗണിക്കുമ്പോള്‍  ഒന്നാമത്തെ പ്രതിവിധി ക്രൂരവും പ്രകൃതിവിരുദ്ധവുമാണ്. രണ്ടാമത്തെ പ്രതിവിധിയാകട്ടെ ധാര്‍മിക വ്യവസ്ഥയെ തകര്‍ക്കുന്നതിലൂടെ സമൂഹത്തെ നശിപ്പിക്കാന്‍ പോന്നതാണ്. മൂന്നാമത്തെ പ്രതിവിധിതന്നെയാണ് സദാചാരനിഷ്ഠമായ സമൂഹത്തിന്റെ നിലനില്‍പ് കാംക്ഷിക്കുന്നവര്‍ തെരഞ്ഞെടുക്കുക. അതുകൊണ്ടാണല്ലോ 1948- ല്‍ മ്യൂണിക്കില്‍ സമ്മേളിച്ച ലോകയുവജനസംഘടന ജര്‍മനിയുടെ പ്രശ്‌നത്തിന് പരിഹാരമായി ബഹുഭാര്യത്വം നിര്‍ദേശിച്ചത്. ഇസ്‌ലാം നിര്‍ദേശിക്കുന്ന പരിഹാരവും ഇതുതന്നെ. ഒരു പരിഹാരം, ലോക യുവജനസംഘടന നിര്‍ദേശിക്കുമ്പോള്‍ മാനവികവും ഇസ്‌ലാം നിര്‍ദേശിക്കുമ്പോള്‍ അപരിഷ്‌കൃതവുമാകുന്നതെങ്ങനെയാണ്?

ഇത്തരമൊരു പ്രതിസന്ധിക്ക്, പരിശുദ്ധാത്മാവ് സകലസത്യത്തിലും വഴി നടത്തുന്നുവെന്ന് വിശ്വസിക്കുന്ന ക്രൈസ്തവസഭക്ക് നിര്‍ദേശിക്കുവാന്‍ കഴിയുന്ന പ്രതിവിധിയെന്താണ്? അധികം വരുന്ന സ്ത്രീകളെ എന്തു ചെയ്യണമെന്നാണ് അവര്‍ക്ക് പറയാനുള്ളത്? അവരെയെല്ലാം കര്‍ത്താവിന്റെ മണവാട്ടികളാണെന്ന മിഥ്യാബോധത്തില്‍ കുരുക്കി കന്യാസ്ത്രീകളാക്കാമെന്ന് സഭ കരുതുന്നുവോ? അതല്ല, ധാര്‍മികതയുടെ അതിരുകള്‍ അതിലംഘിച്ച് വേശ്യാവൃത്തിയിലേക്ക് നയിക്കപ്പെടുവാന്‍ സഭ അവര്‍ക്ക് കൂട്ടുനില്‍ക്കുമോ? സത്യത്തില്‍, ഏകഭാര്യാവ്രതമാണ് തങ്ങളുടെ മതത്തിന്റെ അനുശാസനയെന്ന് വീരവാദം മുഴക്കുന്നവരുടെ കൈയില്‍ സ്ത്രീകളുടെ എണ്ണം വര്‍ധിക്കുന്ന സാമൂഹികസാഹചര്യത്തില്‍ സ്വീകരിക്കേണ്ടത് എന്തു നിലപാടാണെന്നതിനെക്കുറിച്ച യാതൊരു ധാരണയുമില്ലെന്നതാണ് യാഥാര്‍ഥ്യം.

ബഹുഭാര്യത്വത്തെ അപരിഷ്‌കൃതമായി കാണുന്ന യുക്തിവാദികളുടെ കൈയില്‍ ഈ സാമൂഹിക സാഹചര്യത്തിനുള്ള പരിഹാരം വ്യഭിചാരമാണ്. ഗര്‍ഭനിരോധന മാര്‍ഗങ്ങള്‍ യഥേഷ്ടം ഉപയോഗിച്ചുകൊണ്ടുള്ള വ്യഭിചാരം! ബര്‍ട്രാന്‍ഡ് റസ്സല്‍ എഴുതുന്നു:

'മുകളില്‍ പറഞ്ഞ സാഹചര്യങ്ങളില്‍ സാമ്പത്തിക കാരണങ്ങളാല്‍ പുരുഷന്മാരില്‍ മിക്കവരും നേരത്തേ വിവാഹിതരാവുന്നത് അസാധ്യമായി കരുതുകയും അതേസമയം സ്ത്രീകളില്‍ കുറേപേര്‍ക്ക് വിവാഹിതരാവാന്‍തന്നെ കഴിയാതെ വരികയും ചെയ്യുന്ന സാഹചര്യങ്ങള്‍ നിലനില്‍ക്കുന്നിടത്തോളം കാലം സ്ത്രീ-പുരുഷന്മാര്‍ തമ്മിലുള്ള തുല്യാവകാശം സ്ത്രീകളുടെ ചാരിത്ര്യത്തെ സംബന്ധിച്ച പരമ്പരാഗത സങ്കല്‍പത്തില്‍ ഒരു മാറ്റം ആവശ്യപ്പെടുന്നു.  ലൈംഗികബന്ധം അനുവദിക്കപ്പെടുന്നുവെങ്കില്‍ (സത്യത്തില്‍ അത് നിലനില്‍ക്കുന്നുണ്ട്) സ്ത്രീകള്‍ക്കും അത് അനുവദിക്കപ്പെടണം. സ്ത്രീകള്‍ മിച്ചം വരുന്ന  നാടുകളില്‍ അവിവാഹിതരായി കഴിയുന്ന സ്ത്രീകളെയെല്ലാം ലൈംഗികാനുഭൂതിയില്‍നിന്ന് ഒഴിച്ചുനിര്‍ത്തുന്നത് വ്യക്തമായ അനീതിയാണ്. വനിതാ പ്രസ്ഥാനങ്ങളുടെ ആദ്യകാല വക്താക്കള്‍ക്ക് ഇക്കാര്യങ്ങള്‍ കാണാന്‍ കഴിഞ്ഞിരുന്നില്ലെങ്കിലും അവരുടെ ആധുനികരായ അനുയായികള്‍ ഇവ വ്യക്തമായി നോക്കിക്കാണുന്നുണ്ട്. ഈ അഭിപ്രായങ്ങളെ അനുകൂലിക്കാത്തവര്‍ സ്ത്രീ ലൈംഗികതയോട് നീതി ചെയ്യുന്നതിന് എതിരാണെന്ന് പറയേണ്ടിവരും (Marriage and morals, Page 59)

സ്വതന്ത്ര ലൈംഗികത അനുവദിക്കപ്പെടുന്ന സമൂഹത്തില്‍ സന്താനോല്‍പാദനം വിവാഹവൃത്തിയില്‍ മാത്രം ഒതുക്കണമെന്നും വിവാഹബാഹ്യമായ ലൈംഗികവേഴ്ചകളെല്ലാം ഗര്‍ഭനിരോധന മാര്‍ഗങ്ങളുപയോഗിച്ചുകൊണ്ടുള്ളതായിരിക്കണമെന്നും റസ്സല്‍ നിര്‍ദേശിക്കുന്നുണ്ട്.

സ്ത്രീ-പുരുഷ അനുപാതത്തില്‍ വ്യത്യാസമുണ്ടാവുന്ന സാമൂഹിക സാഹചര്യങ്ങൡ സാധിക്കുന്ന പുരുഷന്മാര്‍ ഒന്നിലധികം സ്ത്രീകളെ ഭാര്യമാരാക്കിവെച്ചുകൊണ്ട് അവര്‍ക്കിടയില്‍ നീതിയോടുകൂടി വര്‍ത്തിക്കണമെന്ന ഖുര്‍ആനിക നിര്‍ദേശമോ അധികം വരുന്ന സ്ത്രീകള്‍ വ്യഭിചാരത്തിലേര്‍പ്പെടണമെന്ന യുക്തിവാദ നിര്‍ദേശമോ ഏതാണ് മാനവികം? സ്ത്രീയോട് നീതി ചെയ്യുന്നത് ഏത് നിര്‍ദേശമാണ്?

അവിഹിതബന്ധം മൂലം സ്ത്രീ ക്രൂരമായി വഞ്ചിക്കപ്പെടുന്നുവെന്നതല്ലേ സത്യം! ഏതു നിമിഷവും അവളെ പുറംതള്ളാം. ഈ ബന്ധത്തില്‍ ജനിക്കുന്ന കുഞ്ഞിനെ-ഒരു ഗര്‍ഭ നിരോധനമാര്‍ഗവും നൂറുശതമാനം കുറ്റമറ്റതല്ലെന്നോര്‍ക്കുക- അതിന്റെ പിതാവിനോട് ചേര്‍ക്കാന്‍ പോലും അവള്‍ക്ക് അവകാശമില്ല. അവളുടെ മാംസളത കുറയുകയും തൊലി ചുളിയുകയും ചെയ്താല്‍ പിന്നെ അവളെ ആരും തിരിഞ്ഞുനോക്കുകയില്ല. അവകാശങ്ങളുള്ള അധികൃതമായ ഭാര്യ എന്ന പദവിയോ വേശ്യ എന്ന പേരോ ഏതാണ് അഭികാമ്യം? ഒന്നാമത്തെതായിരുന്നാലും നാലാമത്തെതായിരുന്നാലും അവകാശങ്ങളുള്ള അധികൃത ഭാര്യ എന്ന പദവിയും പെരുമാറ്റവും ലഭിക്കുവാന്‍ ഏതു സ്ത്രീക്കും അവകാശമുണ്ടെന്നാണ് ഇസ്‌ലാമിന്റെകാഴ്ചപ്പാട്. സപത്‌നിയായി ജീവിച്ച് തന്റെയും സന്താനങ്ങളുടെയും ചെലവുകള്‍ കണക്കുതീര്‍ത്തു വാങ്ങുകയും ഭര്‍ത്താവിന്റെ മരണശേഷം താനും കുട്ടികളും സ്വത്തില്‍  അവകാശികളുമായിത്തീരുകയും ചെയ്യുന്നതോ, ഒരു അവകാശവുമില്ലാതെ വേശ്യയായി ജീവിക്കുകയും അവസാനം നരകിച്ച് സമൂഹത്തിന് ഭാരമായിത്തീരുന്നതോ ഏതാണ് സ്ത്രീക്ക് അഭിമാനകരമായിട്ടുള്ളത്?

രണ്ട്) വിധവകളുടെയും അനാഥകളുടെയും സംരക്ഷണം: വിധവകളെയും അനാഥകളെയും  സംരക്ഷിക്കേണ്ടത് സമൂഹത്തിന്റെ ബാധ്യതയാണ്. ഈ ബാധ്യത യഥോചിതം നിര്‍വഹിക്കുന്നതിന് ബഹുഭാര്യത്വം ചിലപ്പോള്‍ അനിവാര്യമായിത്തീരുമെന്ന് കാണാനാവും. യുദ്ധങ്ങളിലും പൊതുജീവിതത്തിലെ അത്യാഹിതങ്ങളിലുമെല്ലാം കൂടുതല്‍ മരണപ്പെടുന്നത് പുരുഷന്മാരാണല്ലോ. അപ്പോള്‍ വിധവകളും അവരുടെ അനാഥരായ മക്കളും കൂടുതലായുണ്ടാവുകയും അവരുടെ സംരക്ഷണം സമൂഹത്തിന്റെ ബാധ്യതയായിത്തീരുകയും ചെയ്യുന്നു.

വിധവകളുടെ സംരക്ഷണമെന്നു പറയുമ്പോള്‍ കേവലം ഭക്ഷണസാമഗ്രികളോ പാര്‍പ്പിടമോ നല്‍കിയതുകൊണ്ട് അത് പൂര്‍ത്തിയാവുമെന്ന് പറയാന്‍ വയ്യ. പലപ്പോഴും വിധവകളായിത്തീരുന്നത് യുവതികളായിരിക്കും. അവര്‍ക്ക് ഒന്നോ രണ്ടോ കുട്ടികളുണ്ടെങ്കിലും മജ്ജയും മാംസവുമുള്ള മനുഷ്യരെന്ന നിലക്ക് ലൈംഗിക വികാരവുമുണ്ടാകും. ഭക്ഷണവും വസ്ത്രവും പാര്‍പ്പിടവും ലഭിക്കുന്നതുകൊണ്ട് ലൈംഗികതൃഷ്ണ ശമിപ്പിക്കപ്പെടുകയില്ലല്ലോ. അവരെ അങ്ങനെ വിടുന്നത് അസാന്മാര്‍ഗിക വൃത്തികളിലേക്ക്  ചായുന്നതിന് കാരണമാകും. സമൂഹത്തിന്റെ ധാര്‍മികതയെതന്നെ തകര്‍ക്കുന്ന നടപടിയാണത്. അപ്പോള്‍ അവര്‍ പുനര്‍വിവാഹം ചെയ്യപ്പെടണം. അതാണ് വിധവകളെ സംരക്ഷിക്കുന്നതിനുള്ള യഥാര്‍ഥ മാര്‍ഗം.

ആരാണ് വിധവകളെ സംരക്ഷിക്കുന്നതിന് സന്നദ്ധരാവുക? വിശേഷിച്ചും ഒന്നോ രണ്ടോ കുഞ്ഞുങ്ങളുള്ള വിധവകളാണെങ്കില്‍ പൊതുവേ പുരുഷന്മാര്‍ ആദ്യഭാര്യമാരായി വിധവകളെ സ്വീകരിക്കാന്‍ മടിക്കും. ഈ മടി പ്രകൃതിപരമായതിനാല്‍ അവര്‍ അക്കാര്യത്തില്‍ വിമര്‍ശിക്കപ്പെടുന്നത് നീതിയല്ല. ഇവിടെയാണ് ബഹുഭാര്യത്വം വിധവകളുടെ സംരക്ഷണത്തിനെത്തുന്നത്. ഒരു പുരുഷന്റെ രണ്ടാമത്തെയോ മൂന്നാമത്തെയോ ഭാര്യയായിത്തീരാന്‍ അവള്‍ സന്നദ്ധയാണെങ്കില്‍ മാനുഷികമായ എല്ലാ അവകാശങ്ങളും അനുഭവിക്കാന്‍ അവള്‍ക്ക് സാധിക്കും.

അനാഥകള്‍ക്കും അമ്മയുടെ രണ്ടാം വിവാഹം ആശ്വാസവും സംരക്ഷണവുമാണ് നല്‍കുക. അനാഥാലയങ്ങളില്‍ എന്തൊക്കെ സൗകര്യങ്ങളുണ്ടായാലും ഒരു കുടുംബത്തിന്റെ സാഹചര്യമുണ്ടാവുകയില്ലല്ലോ. ചെറുപ്പത്തില്‍തന്നെ അമ്മയില്‍നിന്നു പറിച്ചെടുക്കപ്പെട്ട് അനാഥാലയത്തില്‍ അയക്കപ്പെടുന്ന കുഞ്ഞുങ്ങളുടെ മനോനിലയെ അത് കുറച്ചൊന്നുമല്ല ബാധിക്കുക. അമ്മയുടെ മടിയില്‍ വളരേണ്ട കാലത്ത് അവര്‍ അവിടെതന്നെ വളരണം. ഒരു കുടുംബാന്തരീക്ഷത്തില്‍തന്നെ ജീവിക്കുവാന്‍ അവര്‍ക്ക് അവസരം ലഭിക്കണം. ഇതിനുള്ള അവസരമൊരുക്കാന്‍ വിധവയുടെ രണ്ടാം വിവാഹത്തിന് സാധിക്കുന്നു. ധാര്‍മികബോധവും മതനിഷ്ഠയുമുള്ളയാളാണ് അമ്മയുടെ പുതിയ ഭര്‍ത്താവെങ്കില്‍ പിതാവിന്‍േറതിന് തുല്യമായ പെരുമാറ്റവും സംതൃപ്തമായ കുടുംബാന്തരീക്ഷവും ആ അനാഥകള്‍ക്ക് ലഭിക്കുന്നു. അനാഥാലയത്തിലെ ജീവിതത്തെക്കാള്‍ എത്രയോ ഉത്തമമാണ് ഇതെന്നുള്ളതാണ് സത്യം.

വിധവകള്‍ക്ക് നിത്യദുഃഖമാണ് പല മതങ്ങളും നിഷ്‌കര്‍ഷിക്കുന്നത്. മനുസ്മൃതിയുടെ വിധി നോക്കുക.

ആ സീതാ മരണാല്‍ക്ഷാന്താ നിയതാ ബ്രഹ്മചാരിണീ

യോ ധര്‍മ്മ ഏക പത്‌നി നാം കാംക്ഷന്തി നമനുത്തമം (5:158)

(ഭര്‍ത്താവു മരിച്ചശേഷം സ്ത്രീ ജീവാവസാനം വരെ സഹനശീലയാ യും പരിശുദ്ധയായും ബ്രഹ്മധ്യാനമുള്ളവളായും മദ്യ-മാംസഭക്ഷണം ചെയ്യാത്തവളായും ഉല്‍കൃഷ്ടയായ പതിവ്രതയുടെ ധര്‍മത്തെ ആഗ്രഹിക്കുന്നവളായും ഇരിക്കേണ്ടതാകുന്നു)

ഇത്തരം നിയമങ്ങളില്‍നിന്നാണ് കാലക്രമേണ ഭര്‍ത്താവിന്റെ ചിതയില്‍ ഭാര്യയും മരിക്കണമെന്ന സതി സമ്പ്രദായം ഉടലെടുത്തത്. ഇസ്‌ലാമാകട്ടെ വിധവകളുടെ പ്രശ്‌നങ്ങളെ തൊട്ടറിയുകയും അതിനുള്ള പരിഹാരങ്ങള്‍ നിര്‍ദേശിക്കുകയും ചെയ്യുന്നു. ബഹുഭാര്യത്വം വഴി പരിഹരിക്കാന്‍ കഴിയുന്ന പ്രശ്‌നങ്ങള്‍ക്ക് നടുവില്‍നിന്ന് കണ്ണടച്ച് ഇരുട്ടാക്കുവാന്‍ അത് ആരോടും ആവശ്യപ്പെടുന്നില്ല. അത്തരം അവസരങ്ങളില്‍ ഒന്നിലധികം ഭാര്യമാരെ സ്വീകരിക്കുവാന്‍ അത് അനുവാദം നല്‍കുന്നു. വിധവകളുടെയും അനാഥകളുടെയും സംരക്ഷണത്തിനുതകുന്ന തികച്ചും മാനവികമായ ഒരു സംവിധാനമാണത്. അതുകൊണ്ടുതന്നെ ഇത്തരം ഒട്ടനവധി അവസരങ്ങളില്‍ ബഹുഭാര്യത്വം സ്ത്രീയുടെ സംരക്ഷണത്തിനെത്തുന്നതായാണ് നമുക്ക് കാണാന്‍ കഴിയുന്നത്.

ബഹുഭാര്യത്വമനുവദിക്കുന്നതിലൂടെ ഇസ്‌ലാം സ്ത്രീകളെ തരംതാഴ്ത്തിയെന്ന ആരോപണത്തില്‍ കഴമ്പില്ലെന്ന വസ്തുതയാണിവിടെ വ്യക്തമാവുന്നത്. സദാചാരനിഷ്ഠമായ ഒരു സാമൂഹിക സംവിധാനം കാംക്ഷിക്കുന്നവര്‍ക്കൊന്നുംതന്നെ ബഹുഭാര്യത്വത്തെ അപ്പടി അധിക്ഷേപിക്കുവാന്‍ കഴിയില്ല.

വിഷയവുമായി ബന്ധപ്പെട്ട വീഡിയോ

അല്ല. ബഹുഭാര്യത്വം ഇസ്‌ലാം കൊണ്ടുവന്ന ഒരു സമ്പ്രദായമേയല്ല. എല്ലാ സമൂഹങ്ങളിലും നാഗരികതകളിലും നിലനിന്നിരുന്നു, ബഹുഭാര്യത്വം. . പൗരാണിക സമുഹങ്ങളിലെല്ലാം ഒരു രൂപത്തിലല്ലെങ്കില്‍ മറ്റൊരു രൂപത്തില്‍ ബഹുഭാര്യത്വം നിലനിന്നിരുന്നുവെന്ന് വ്യക്തമാക്കുന്നതാണ് ഈ രംഗത്തെ ഗവേഷകനായ മര്‍ഡോക്കിന്റെ പഠനങ്ങൾ. പുരാതന സംസ്‌കാരങ്ങളില്‍ പൊതുവായി കാണപ്പെട്ടിരുന്ന ഒരു സമ്പ്രദായമാണത്. എന്‍സൈക്ലോപീഡിയ ബ്രിട്ടാണിക്ക എഴുതുന്നത് കാണുക: 'പൗരാണിക നാഗരികതയില്‍ അധിക സമൂഹങ്ങളിലും ബഹുഭാര്യത്വമോ വെപ്പാട്ടികളെ ഉപയോഗിക്കുന്ന സമ്പ്രദായമോ നിലനിന്നതായി കാണാന്‍ കഴിയും. നിയമാനുസൃതമായ ഭാര്യക്കുപുറമെ അനവധി സ്ത്രീകളെ വെച്ചുകൊണ്ടിരിക്കുന്ന സമ്പ്രദായം നിലനിന്നിരുന്ന ചൈനയില്‍ അത് സദാചാരത്തിനോ മാന്യതയ്‌ക്കോ വിരുദ്ധമായി കണക്കാക്കപ്പെട്ടിരുന്നില്ല. വെപ്പാട്ടിമാരെ ഉപയോഗിക്കുന്ന സമ്പ്രദായം ജപ്പാനില്‍ 1880 വരെ നിലനിന്നിരുന്നു. പുരാതന ഈജിപ്തില്‍ ബഹുഭാര്യത്വത്തിന് അനുമതിയുണ്ടായിരുന്നുവെങ്കിലും അത് സര്‍വസാധാരണമായിരുന്നില്ല. രാജാക്കന്മാര്‍ക്കിടയില്‍ അത് പതിവായിരുന്നു താനും'' (vol. xviii page 188)

റോമക്കാര്‍ക്കിടയില്‍ ഒരു പ്രത്യേക കാലഘട്ടത്തിലൊഴിച്ച് എല്ലാ പൗരാണിക സമൂഹങ്ങളിലും ബഹുഭാര്യത്വം സാര്‍വത്രികമായിരുന്നുവെന്നാണ് മനസ്സിലാക്കാന്‍ കഴിയുന്നത്. മധ്യാഫ്രിക്കയിലും ആസ്‌ട്രേലിയയിലുമുള്ള ചില സമൂഹങ്ങളില്‍ ധനികരായവര്‍ വിവാഹപ്രായമെത്തിയ പെണ്‍കുട്ടികളെ ഭാര്യമാരാക്കാന്‍ മല്‍സരിച്ചിരുന്നുവത്രേ. അവിടങ്ങളിലെ യുവാക്കള്‍ ഇക്കാരണത്താല്‍ വിവാഹം ചെയ്യാനാവാതെ പ്രയാസപ്പെട്ടിരുന്നുവെന്നും പിതാവിന്റെ മരണശേഷം അദ്ദേഹത്തിന്റെ പത്‌നിമാരെ വിവാഹം കഴിക്കുകയായിരുന്നു പലരും ചെയ്തിരുന്നതെന്നുമാണ് മനസ്സിലാക്കാന്‍ കഴിയുന്നത്. സിംബാബ്‌വേയിലെ മോണോമട്ടാവോ രാജാക്കന്മാര്‍ക്ക് മൂവായിരത്തോളം ഭാര്യമാരുണ്ടായിരുന്നുവത്രേ. സൈരേയിലെ ബകുബാ, ബകേത്തേ വര്‍ഗങ്ങളുടെ തലവന്മാര്‍ക്കായിരുന്നു ഏറ്റവും കൂടുതല്‍ ഭാര്യമാരുണ്ടായിരുന്നത് എന്നാണ് ഗിന്നസ് ബുക്കിന്റെ വിലയിരുത്തല്‍. അവര്‍ക്ക് നൂറുകണക്കിന് ഭാര്യമാരുണ്ടായിരുന്നുവത്രെ!

ബൈബിള്‍ പഴയനിയമത്തിലെ പല പ്രവാചകന്മാര്‍ക്കും ഒന്നിലധികം ഭാര്യമാരുണ്ടായിരുന്നു. യഹൂദ സമുദായത്തിന്റെ ആദര്‍ശപിതാവ് എന്നു വിശേഷിപ്പിക്കപ്പെടുന്ന അബ്രഹാമിന് സാറായ്, ഹാഗാര്‍ എന്നീ രണ്ടു ഭാര്യമാര്‍ ഉണ്ടായിരുന്നുവെന്ന് ഉല്‍പത്തി പുസ്തകം (16:1-3) വ്യക്തമാക്കുന്നു. സാറയുടെ മരണശേഷം അദ്ദേഹം കെതൂറയെന്നവളെയും വിവാഹം കഴിച്ചുവെന്നും ഇതുകൂടാതെ അനേകം ഉപഭാര്യമാരും അദ്ദേഹത്തിനുണ്ടായിരുന്നുവെന്നും ബൈബിളില്‍ കാണാം (ഉല്‍പത്തി 25:1-6). ഇസ്രായേല്‍ ഗോത്രത്തിന്റെ പിതാവായിരുന്ന യാക്കോബിന് ലേയാ (ഉല്‍പത്തി 29:21), ലാബാന്‍ (29:29), ബില്‍ഹാ (30:4), സില്‍വാ (30:9) എന്നീ നാലു ഭാര്യമാരുണ്ടായിരുന്നു. സങ്കീര്‍ത്തനകര്‍ത്താവായി അറിയപ്പെടുന്ന ദാവീദിനാവട്ടെ മീകല്‍ (1 ശാമുവേല്‍ 18:28), ബത്‌ശേബ (2 ശാമുവേല്‍ 11:27), അബീനോവം (2 ശാമുവേല്‍ 3:3) അബിഗായാല്‍, മാക്‌യ്, ഹഗ്ഗീതി, അബീതാല്‍, എഗ്ലായ്, (2 ശാമുവേല്‍ 3:4-5) തുടങ്ങി അനേകം ഭാര്യമാരുണ്ടായിരുന്നതായി കാണാന്‍ കഴിയും. അദ്ദേഹത്തിന്റെ പുത്രനും സുഭാഷിതങ്ങളുടെ കര്‍ത്താവുമായ സോളമനാകട്ടെ എഴുന്നൂറു ഭാര്യമാരും മുന്നൂറു ഉപഭാര്യമാരുമുണ്ടായിരുന്നുവത്രേ! (1 രാജാക്കന്മാര്‍ 11:3) പലരുടെയും മഹത്വമായി പഴയ നിയമം പറയുന്നത് തന്നെ 'അവര്‍ക്ക് അനേകം ഭാര്യമാരും പുത്രന്മാരുമുണ്ടായിരുന്നു'വെന്നാണ് (1 ദിനവൃത്താന്തം 7:3). പഴയ നിയമകാലത്ത് ബഹുഭാര്യത്വം സര്‍വസാധാരണമായിരുന്നുവെന്നാണല്ലോ ഇവ കാണിക്കുന്നത്.

യഹൂദമതത്തിന്റെ തുടര്‍ച്ചയായി വന്ന ക്രിസ്തുമതവും ബഹുഭാര്യത്വം നിഷിദ്ധമാണെന്ന് വിധിച്ചതായി ആദ്യകാല രേഖകളിലൊന്നും കാണുന്നില്ല. സുവിശേഷങ്ങളിലോ പ്രവൃത്തി പുസ്തകത്തിലോ വെളിപാടു പുസ്തകത്തിലോ അജപാലകലേഖനങ്ങളിലോ പൗലോസിന്റെ എഴുത്തുകളില്‍ പോലുമോ ബഹുഭാര്യത്വത്തെ നിരോധിക്കുന്ന ഒരു വചനം പോലും കാണാന്‍ കഴിയില്ല. എന്നാല്‍, പൗലോസിന്റെ ലേഖനങ്ങളില്‍ പൊതുവെ വിവാഹത്തെ തന്നെ പ്രോല്‍സാഹിപ്പിക്കാത്ത നിലപാടാണുള്ളത്. 'വിവാഹം കഴിക്കാതിരിക്കുന്നുവെങ്കില്‍ ഏറെ നല്ലത്' (1കൊരിന്ത്യര്‍ 7:38) എന്നു പഠിപ്പിച്ച പൗലോസിന്റെ അനുയായികള്‍ സന്യാസത്തിന് പ്രേരിപ്പിക്കുകയും അതു സാധ്യമല്ലാത്തവര്‍ ഒരൊറ്റ ഭാര്യയെ മാത്രം വേള്‍ക്കട്ടെയെന്ന തത്ത്വത്തിലെത്തിച്ചേരുകയുമാണുണ്ടായത്.

യേശുക്രിസ്തു അദ്ദേഹത്തിന്റെ വചനങ്ങളില്‍ എവിടെയെങ്കിലും ബഹുഭാര്യത്വത്തെ നിരോധിക്കുന്നതായി നമുക്ക് കാണാന്‍ കഴിയുന്നില്ല. അദ്ദേഹം വിവാഹിതനായതായി ബൈബിളില്‍ ഒരിടത്തുമില്ല. എന്നാല്‍ അന്ന് നിലനിന്നിരുന്ന ബഹുഭാര്യത്വമെന്ന സമ്പ്രദായത്തെ നിരോധിക്കുകയോ നിഷേധിക്കുകയോ വിമര്‍ശിക്കുകയോ എതിര്‍ക്കുകയോ ചെയ്തതായി പുതിയനിയമത്തിലെവിടെയും കാണാന്‍ കഴിയുന്നില്ല. എന്നാല്‍ ബൈബിളില്‍ തന്നെയുള്ള യേശുവിന്റെ ചില ഉപമകളില്‍ ബഹുഭാര്യത്വം കടന്നുവരുന്നുണ്ട്- ഉപമാലങ്കാരങ്ങളാല്‍ സമൃദ്ധമാണല്ലോ സുവിശേഷങ്ങള്‍. മത്തായിയുടെ സുവിശേഷത്തില്‍ പത്തു കന്യകമാരുടെ ഉപമയുണ്ട്. ഒരൊറ്റ മണവാളനെ കാത്തുനില്‍ക്കുന്ന പത്തുകന്യകമാര്‍. അവിടെ ഈ കന്യകമാരെക്കുറിച്ച് പറയുമ്പോള്‍ ബഹുഭാര്യത്വം ശരിയല്ലെന്ന സൂചനപോലും യേശു നല്‍കുന്നില്ല. മത്തായിയുടെ സുവിശേഷം നോക്കുക (മത്തായി 25:1-13) പൗലോസ് വിവാഹവിരുദ്ധനും സ്ത്രീ വിരുദ്ധനുമായിരുന്നു. എങ്കിലും അദ്ദേഹത്തിന്റെ ലേഖനങ്ങളിലും ബഹുഭാര്യത്വത്തെ നേര്‍ക്കുനേരെ വിമര്‍ശിക്കുന്നത് കാണാന്‍ സാധിക്കുന്നില്ല. മാത്രമല്ല, യേശുക്രിസ്തു വിവാഹിതനായിരുന്നും ഒന്നിലധികം സ്ത്രീകളുടെ ഭര്‍ത്താവായിരുന്നുവെന്നും ബൈബിളിന്റെ വെളിച്ചത്തില്‍ വാദിച്ചവരുണ്ടായിട്ടുണ്ട്. ഉയിര്‍ത്തെഴുന്നേല്‍പിനുശേഷം മഗ്ദലനമറിയം, സലോമി തുടങ്ങിയ സ്ത്രീകള്‍ക്കാണ് ക്രിസ്തു പ്രത്യക്ഷപ്പെട്ടതെന്നും തന്റെ അപ്പോസ്തലന്മാരെക്കാള്‍ അദ്ദേഹത്തിന് അടുപ്പമുണ്ടായിരുന്നത് ഈ സ്ത്രീകളോടായിരുന്നുവെന്നാണ് ഇതു കാണിക്കുന്നതെന്നും അവര്‍ യേശുവിന്റെ ഭാര്യമാരായിരിക്കാനാണ് സാധ്യതയെന്നുമാണ് അവര്‍ സമര്‍ഥിച്ചത്. ആദ്യകാലത്ത് ബഹുഭാര്യത്വം അനുവദനീയമാണെന്നുതന്നെയായിരുന്നു ക്രൈസ്തവ വീക്ഷണം. എന്‍സൈക്‌ളോപീഡിയ ബ്രിട്ടാണിക്കയില്‍ ഇക്കാര്യം സൂചിപ്പിച്ചിട്ടുണ്ട്. 'ബഹുഭാര്യത്വം മധ്യകാലത്ത് ക്രൈസ്തവസഭയില്‍ അംഗീകരിക്കപ്പെട്ടിരുന്നു. നിയമാനുസൃതമായി അത് നിലനിന്നിരുന്നു. പതിനാറാം നൂറ്റാണ്ടിന്റെ നടുവില്‍വരെ മതവും രാജ്യവും അനുവദിച്ചതിനാല്‍ നിയമാനുസൃതമായിത്തന്നെ പലയിടങ്ങളിലും അത് നിലനിന്നിരുന്നു. (vol xiv page:950)

ക്രൈസ്തവര്‍ക്കിടയില്‍ ബഹുഭാര്യത്വത്തിന് അനുകൂലവും പ്രതികൂലവുമായ വീക്ഷണങ്ങള്‍ എല്ലാകാലത്തും ഉണ്ടായിട്ടുണ്ട്. അവര്‍ക്കിടയില്‍ ഇന്ന് നിലനില്‍ക്കുന്ന വ്യത്യസ്തസംഘടനകള്‍, പാശ്ചാത്യര്‍ക്കിടയിലുള്ള സംഘടനകള്‍, ബഹുഭാര്യത്വമാണ് പാശ്ചാത്യസമൂഹമനുഭവിക്കുന്ന, പ്രത്യേകിച്ച്, ക്രൈസ്തവസമൂഹമനുഭവിക്കുന്ന ധാര്‍മികച്യുതിയില്‍ നിന്ന് അവരെ രക്ഷപ്പെടുത്തുവാനുള്ള ഒരേയൊരു മാര്‍ഗമെന്ന് വാദിച്ചുകൊണ്ട് രംഗത്തുവന്നിട്ടുണ്ട്. ഇക്കാര്യം ആര്‍ക്കും പരിശോധിക്കാന്‍ കഴിയും  www.christianpolygamy.comഎന്ന വെബ്അഡ്രസിലോ അല്ലെങ്കില്‍ christian polygamy.info എന്ന അഡ്രസിലോ അതല്ലെങ്കില്‍ www.lovenotforce.com- എന്ന വിലാസത്തിലോ സെര്‍ച്ചുചെയ്താല്‍  ഈ രംഗത്തെ പഠനങ്ങള്‍ കാണാന്‍ കഴിയും. അമേരിക്കയില്‍ ബഹുഭാര്യത്വം നിരോധിക്കപ്പെട്ടിട്ടുണ്ട്. നിയമപരമായി ഒന്നിലധികം ഭാര്യമാരെ വെക്കാന്‍ അവര്‍ക്ക് പാടില്ല. ക്രൈസ്തവര്‍ക്ക് ഒന്നിലധികം ഭാര്യമാരെ വെക്കുവാന്‍ അനുവാദം വേണമെന്ന് പറഞ്ഞു പോരാടിക്കൊണ്ടിരിക്കുകയാണ്  ഈ ക്രൈസ്തവ വിഭാഗങ്ങള്‍. ധാര്‍മികത നിലനിര്‍ത്താന്‍ ബഹുഭാര്യത്വം അനുവദിക്കണമെന്നാണ് അവരുടെ വാദം.

ക്രൈസ്തവര്‍ക്കിടയില്‍ നിന്ന് ഉണ്ടായിവന്ന മറ്റൊരു വിഭാഗമാണ് Fundamentalist Church of Jesus  Christ of Latterday Saints  അഥവാ മോര്‍മോണുകള്‍. ഇവര്‍ ബഹുഭാര്യത്വത്തെ അംഗീകരിക്കുകയും അനുവദിക്കുകയും ചെയ്തു. ആയിരത്തിയെണ്ണൂറുകള്‍ വരെ അവര്‍ക്കിടയില്‍ ബഹുഭാര്യത്വം സാര്‍വത്രികമായിരുന്നു. പ്രത്യേകിച്ച്, അമേരിക്കയിലെ Utah State ലുള്ള മോര്‍മോണുകള്‍ക്കിടയില്‍. ഇപ്പോള്‍ നിയമം മൂലം നിരോധിക്കപ്പെട്ടതുകൊണ്ട്, സ്വകാര്യമായി ബഹുഭാര്യത്വത്തിലേര്‍പ്പെടുകയാണിവര്‍ ചെയ്യുന്നത്. ഒന്നിലധികം ഭാര്യമാരുള്ള ഇരുപതിനായിരത്തോളം കുടുംബങ്ങള്‍ ഇന്ന് അവര്‍ക്കിടയിലുണ്ട്. ഇതെല്ലാം നമ്മെ പഠിപ്പിക്കുന്ന വസ്തുതകളോട് സൃഷ്ടിപരമായി പ്രതികരിക്കാന്‍ ബുദ്ധിജീവികള്‍ സന്നദ്ധമാകണം.

ഇന്ത്യയിലെ ഹൈന്ദവസമൂഹത്തിലാണെങ്കില്‍ ഭാര്യമാരുടെ എണ്ണം ഒരു മഹത്വമായി നിശ്ചയിക്കപ്പടുകയും നിര്‍ണയിക്കപ്പെടുകയും ചെയ്ത ഒരു കാലമുണ്ടായിരുന്നു. ഋഗ്വേദത്തിലെ പ്രധാന ദേവനായ ഇന്ദ്രന്  ഒന്നിലധികം ഭാര്യമാരുണ്ടായിരുന്നുവെന്നാണ് മനസ്സിലാവുന്നത്. ഇന്ദ്രപത്‌നിമാരില്‍ പ്രധാനിയായിരുന്ന ഇന്ദ്രാണിയുടേതായി ഒരു സൂക്തമുണ്ട് (ഋഗ്വേദം 10-ാം മണ്ഡലം 17-ാം സൂക്തം). പ്രസ്തുത സൂക്തത്തിലെ പ്രധാന പ്രതിപാദ്യം സപത്‌നീമര്‍ദനത്തിനുള്ള മന്ത്രമാണ്. സപത്‌നിയോട് രാജാവിനുള്ള പ്രേമം നശിപ്പിച്ച് തന്നിലേക്ക് ആകര്‍ഷിക്കാനുള്ള മന്ത്രമാണത്. ഇതില്‍നിന്ന് വേദകാലത്ത് ബഹുഭാര്യത്വം സാര്‍വത്രികമായിരുന്നുവെന്ന് മനസ്സിലാക്കാനാവും. ഇതിഹാസങ്ങളുടെ കാലമായപ്പോഴേക്കും ബഹുഭാര്യത്വം സമൂഹത്തിന്റെ പൂര്‍ണമായ അംഗീകാരത്തോടെ, വ്യാപകമായി നിലനിന്നിരുന്നുവെന്നാണ് മനസ്സിലാക്കാന്‍ കഴിയുന്നത്. ശ്രീകൃഷ്ണന്‍ ഇന്ത്യയില്‍ ഏറ്റവും അധികം ആരാധിക്കപ്പെട്ട ദേവനാണ്. അദ്ദേഹത്തിന്റെ ഗുണഗണങ്ങളില്‍ പ്രധാനപ്പെട്ടതായി പറയുന്നത് തന്നെ 16008 ഭാര്യമാരുണ്ടായിരുന്നു എന്നതാണ്. അത്് സാങ്കല്‍പികമാണോ അതല്ല; ഉപമാലങ്കാരമാണോ എന്നത് വേറെ പ്രശ്‌നം. രുഗ്മിണി,ജാംബവതി,സത്യഭാമ, കാളിന്തി, ചിത്രവന്ദ, സരസ്വതി,കൈകേയി, ലക്ഷ്മണ തുടങ്ങിയ എട്ടുപേരും നരകാസുരന്റെ പതിനാറായിരം പുത്രിമാരുമായിരുന്നു ശ്രീകൃഷ്ണഭാര്യമാര്‍ എന്നാണ് പുരാണങ്ങളിലുള്ളത്. ഒരു ഉപമാകഥ മാത്രമാണ് ശ്രീകൃഷ്ണ വിവാഹങ്ങള്‍ എന്ന വാദം ശരിയല്ലെന്ന് തന്നെയാണ് ഇത് കാണിക്കുന്നത്. കേവമൊരു കഥ മാത്രമാണിതെന്ന് വന്നാല്‍ പോലും ഇന്ത്യന്‍ മനസ്സില്‍ അദ്ദേഹത്തിന്റെ ഭാര്യമാരുടെ എണ്ണം ഒരു ബഹുമാനമായി, ഈ വിഗ്രഹവല്‍ക്കരണം എന്ന നിലക്ക് നിന്നിരുന്നുവെന്ന് ഇത് വ്യക്തമാക്കുന്നുണ്ട്.

ശ്രീരാമന്റെ പിതാവിന് കൗസല്യ, കൈകേയി, സുമിത്ര എന്നീ മൂന്ന് ഭാര്യമാരുണ്ടായിരുന്നതായി നമുക്ക് രാമായണത്തില്‍ കാണാന്‍ സാധിക്കും. പിന്നീട് മറ്റെല്ലാം നിയമങ്ങളെയും പോലെ ബഹുഭാര്യത്വവുമായി ബന്ധപ്പെട്ട നിയമങ്ങളും   വര്‍ണാശ്രമവ്യവസ്ഥയുടെ അടിസ്ഥാനത്തിലായിത്തീര്‍ന്നു. അപ്പോള്‍ ബ്രാഹ്മണന് ഭാര്യമാര്‍ കൂടുതലാകാം, ക്ഷത്രിയനത്ര പാടില്ല, വൈശ്യന് അത്ര പാടില്ല, ക്ഷൂദ്രന് സ്വന്തം ജാതിയില്‍ പെട്ട ഭാര്യമാരെ മാത്രമെ പറ്റൂ എന്നെല്ലാമുള്ള നിയമങ്ങള്‍ വന്നു. യാജ്ഞവല്‍ക്യസമൃതിയുടെ നിയമം കാണുക.

തിസ്വോവര്‍ണാനു പൂര്‍വ്യേണ ദ്വോ തഥൈകാ യഥാക്രമം

ബ്രാഹ്മണ ക്ഷത്രിയ വിശാം ഭാര്യാ സ്വാ ശൂദ്രജന്മനഃ

(യാജ്ഞവല്‍ക്യസ്മൃതി 1:57)

(വര്‍ണക്രമമനുസരിച്ച് ബ്രാഹ്മണന് മൂന്നും ക്ഷത്രിയന് രണ്ടും വൈശ്യന് ഒന്നും ഭാര്യമാരാകാം. ശൂദ്രന് സ്വജാതിയില്‍ നിന്നുമാത്രമേ വിവാഹം പാടുള്ളൂ)

ഇതെല്ലാമായിരുന്നിട്ടും ഭാര്യാസംസര്‍ഗത്തിനു പുറത്തുള്ള ബന്ധങ്ങള്‍, അറിഞ്ഞും അറിയാതേയും  ഭാരതീയര്‍ അംഗീകരിക്കുകയും ചെയ്തു. ഇതെല്ലാം നമുക്ക് മനുസ്മൃതിയായിരുന്നാലും യാജ്ഞവല്‍ക്യസ്മൃതിയിലായിരുന്നാലും പരാശരസ്മൃതിലായിരുന്നാലും കാണാന്‍ കഴിയുന്ന കാര്യങ്ങളാണ്. ഏകപത്‌നീവ്രതം നിലനില്‍ക്കുന്നുവെന്നവകാശപ്പെടുന്ന ആധുനിക സമൂഹങ്ങളിലും ഒന്നിലധികം സ്ത്രീകളുമായി ലൈംഗികവേഴ്ചയിലേര്‍പ്പെടുന്ന സമ്പ്രദായം സാര്‍വത്രികമാണെന്നതാണ് വസ്തുത. അതിന് പല വിധ ഓമനപ്പേരുകള്‍ നല്‍കുന്നുവെന്നു മാത്രമെയുള്ളൂ. 'പബ്ലിക് റിലേഷന്‍സി'ല്‍ ഏര്‍പ്പെട്ടിരിക്കുന്ന കാള്‍ഗേളുകളില്‍ പണക്കാരന്‍ ലൈംഗികദാഹം ശമിപ്പിക്കുമ്പോള്‍ വേശ്യാതെരുവുകളിലാണ് സാധാരണക്കാരന്‍ സമാധാനം കണ്ടെത്തുന്നത് എന്ന വ്യത്യാസമേയുള്ളൂ. പലതരം പേരുകളില്‍ വിളിക്കപ്പെടുന്ന അഭിസാരികകളെ ഒരു പ്രാവശ്യമെങ്കിലും സമീപിക്കാത്തവര്‍ ആധുനിക സമൂഹത്തില്‍ വളരെ വിരളമാണെന്നാണ് പഠനങ്ങള്‍ കാണിക്കുന്നത്. അതൊരു തെറ്റായി ആധുനിക സമൂഹം കാണുന്നേയില്ല. ഇവ കൂടാതെതന്നെ സമൂഹത്തിലെ ഉന്നതരില്‍ നടക്കുന്ന  ഭാര്യാവിക്രയം (wife swaping)-,- സംഘരതി (group sex or daisy chain),  തുടങ്ങിയ ലൈംഗിക വൈകൃതങ്ങളും വര്‍ധിച്ചുവരികയാണെന്നാണ് കണക്കുകള്‍ സൂചിപ്പിക്കുന്നത്. ബഹുഭാര്യത്വത്തിനെതിരെ ശക്തമായി സംസാരിക്കുന്നവരില്‍ പലരും ഇത്തരം ലൈംഗികബന്ധങ്ങളുടെ അടിമകളാണെന്നതാണ് വാസ്തവം.

ചരിത്രത്തില്‍ എല്ലാ കാലത്തും നിലനിന്ന ഒരു സ്ഥാപനമാണ്ബഹുഭാര്യത്വമെന്നതാണ് വാസ്തവം. ഈ വസ്തുതകള്‍ നല്‍കുന്ന പാഠത്തോട് മുഖം തിരിഞ്ഞുനില്‍ക്കുന്നതുകൊണ്ട് കാര്യമില്ല. അവ തെളിയിക്കുന്ന യാഥാര്‍ഥ്യത്തോട് ക്രിയാത്മകവും വസ്തുനിഷ്ഠവുമായി സംവദിക്കാന്‍ നമുക്കു കഴിയണം. അപ്പോള്‍ മനസ്സിലാവും, ഏകഭാര്യത്വം ചില വ്യക്തികളുടെയെങ്കിലും സ്വാഭാവികവും പ്രകൃതിപരവുമായ ദാഹം തീര്‍ക്കാന്‍ പര്യാപ്തമായ സമ്പ്രദായമല്ലെന്ന്. ഈ സത്യത്തിന് നേരെ കണ്ണടച്ചുകൊണ്ട് ബഹുഭാര്യത്വമെന്ന പ്രശ്‌നം ചര്‍ച്ച ചെയ്യുന്നത് വെറുതെയാണ്.

വിഷയവുമായി ബന്ധപ്പെട്ട വീഡിയോ

ലൈംഗികതയുമായി ബന്ധപ്പെട്ട വീക്ഷണങ്ങള്‍ രണ്ടാണ്. ഒന്ന്, ലൈംഗികതയുടെ സംപൂര്‍ത്തികരണത്തിന് വിവാഹമെന്ന സ്ഥാപനം അനിവാര്യമാണ് എന്ന വീക്ഷണം.  അതാവശ്യമില്ലെന്നുള്ള വീക്ഷണമാണ് രണ്ടാമത്തേത്. വിവാഹത്തിന്റെ ലക്ഷ്യം മനുഷ്യന്റെ അടിസ്ഥാനചോദനയായ ലൈംഗികതയുടെ സംപൂര്‍ത്തീകരണമാണ്. ലൈംഗികപൂര്‍ത്തീകരണത്തിന്റെ അനുബന്ധമായി വരുന്നതാണ് കുടുംബത്തിന്റെ നിലനില്‍പ്പ്, സന്താനങ്ങളുടെ വളര്‍ച്ച, കുട്ടികളുടെ മുലയൂട്ടല്‍ തുടങ്ങിയവ. ലൈംഗികത വിവാഹത്തിലൊതുങ്ങി നില്‍ക്കണമെന്ന ഒന്നാമത്തെ കാഴ്ചപ്പാടിനോടൊപ്പം നില്‍ക്കുന്നു ഇസ്‌ലാം. വിവാഹേതരലൈംഗികബന്ധങ്ങള്‍ ധാര്‍മികവിരുദ്ധമാണെന്നും പ്രകൃതിവിരുദ്ധമാണെന്നും മാനവവിരുദ്ധമാണെന്നും ഇസ്‌ലാം വിചാരിക്കുന്നു. ഇസ്‌ലാമിക നിയമങ്ങളുടെ മുഴുവന്‍ കാതലതാണ്.

വിവാഹത്തിൽ ഒതുങ്ങി നിൽക്കേണ്ടതല്ല ലൈംഗികതയെന്ന് കരുതുന്നവരുണ്ട്. . 1945ല്‍ വില്‍ഹം റീഹി (Wilham Reich)ന്റെ 'ലൈംഗിക വിപ്ലവം' (the sexual Revelution)  എന്ന പുസ്തകം പുറത്ത് വന്നു. ലൈംഗികരംഗത്തെ വിപ്ലവം! ആ പുസ്തകത്തിലൂടെ അദ്ദേഹം മുന്നോട്ട് വെച്ച ചില ആശയങ്ങളുണ്ട്. വിവാഹേതരലൈംഗികതയെ ധാര്‍മികമാക്കുന്നതിന് വേണ്ടിയാണ് അദ്ദേഹത്തിന്റെ പുസ്തകത്തില്‍ പരിശ്രമിച്ചിട്ടുള്ളത്. വിവാഹം ആവശ്യമില്ലാത്ത ഒരു സ്ഥാപനമാണെന്ന് തോന്നിപ്പിക്കുന്ന രൂപത്തിലുള്ള ചര്‍ച്ചകളാണ് അതിലിള്ളുത്. ഏറെ ചര്‍ച്ചചെയ്യപ്പെട്ട ഗ്രന്ഥമാണത്. അതിനുശേഷം വിവാഹ ബാഹ്യലൈംഗികതയെ വിഗ്രഹവല്‍ക്കരിക്കുന്നതിനുവേണ്ടി ഹ്യൂമണിസ്റ്റുകള്‍ വ്യാപകമായി പരിശ്രമിച്ചു. സിനിമയും നാടകങ്ങളും സാഹിത്യങ്ങളുമെല്ലാം ഈ ആവശ്യത്തെ മുന്‍നിര്‍ത്തിയുള്ളതായി. യൂറോപ്പിലും പാശ്ചാത്യരാജ്യങ്ങളിലും ഹ്യൂമണിസ്റ്റുകള്‍ തുടങ്ങിവെച്ച സ്വതന്ത്ര ലൈംഗികതയെന്ന ആശയം ലോകത്തെ വിവിധ ഭാഗങ്ങളിലേക്ക് ഇവയിലൂടെ കയറ്റി അയക്കപ്പെട്ടു. വിവാഹമെന്ന കാലഹരണപ്പെട്ട സ്ഥപനത്തിന്റെ ദൃംഷ്ടങ്ങളില്‍ നിന്ന് സമൂഹത്തെ രക്ഷിച്ചാല്‍ മാത്രമെ കപട സദാചാരത്തിന്റെ മുഖം മുടിക്കു പിന്നില്‍ മനുഷ്യന്‍ അനുഭവിക്കുന്ന ലൈംഗിക ദാരിദ്രത്തിന് പരിഹാരം കാണാനാകുവെന്ന സന്ദേശമാണ് ഇവ ലോകത്തിന് നല്‍കിയത്. കഴിഞ്ഞ അര നൂറ്റാണ്ടിലേറെ കാലമായി യൂറോപ്പിലെ ഹ്യൂമണിസ്റ്റുകള്‍ ഉന്നയിച്ചകൊണ്ടിരിക്കന്ന സ്വതന്ത്രലൈംഗികതയുടെ പ്രത്യയ ശാസ്ത്രത്തെ മഹത്വവല്‍കരിച്ചുകൊണ്ടിരിക്കുകയാണ് ഇന്ന് കേരളത്തിലെ ചില മുഖ്യധാരാ മാധ്യമങ്ങളും ആനുകാലികളും ചെയ്തുന്നത്. യൂറോപ്പ് ചവച്ചുതുപ്പിയതിന്റെ മഹത്വമുല്‍ഘോഷിക്കുവാനാണല്ലോ നമ്മുടെ വിധി!

വിവാഹേതരബന്ധങ്ങള്‍ കാലാകാലങ്ങളില്‍ നിലനിന്നുപോന്നിട്ടുണ്ട്. ചരിത്രത്തില്‍ നമുക്കത് കാണാന്‍ കഴിയും. പക്ഷെ, അതിനെ സമൂഹം നോക്കിക്കണ്ടത് ഒരു തിന്മയായിട്ടായിരുന്നു. അതിനെ ധാര്‍മികവല്‍ക്കരിക്കാനുള്ള ശ്രമങ്ങളും ധാരാളമായി നടന്നിട്ടുണ്ട്. വിവാേഹതരലൈംഗികതയുടെ ധാര്‍മികവല്‍ക്കരണം എന്നത്  തൊള്ളായിരത്തി നാല്‍പതുകള്‍ക്ക് ശേഷം ആധുനിക സമൂഹത്തിന്റെ ജനാധിപത്യച്ചുവയുടെയെല്ലാം പ്രതിഫലനമായി പൊതുവെ പരിചയപ്പെടുത്തപ്പെടുന്ന അവസ്ഥയുണ്ടായി; വിക്‌ടോറിയന്‍ സദാചാരസങ്കല്‍പങ്ങളെ വെല്ലുവിളിച്ചതെന്ന് കൊട്ടിഘോഷിക്കപ്പെടുന്ന ഈ 'ലൈംഗിക വിപ്ലവ'ത്തിന്റെ ആകെത്തുക വിവാഹേതരബന്ധങ്ങള്‍ അധാര്‍മികമല്ലെന്ന ആശയത്തിന്റെ പ്രയോഗവല്‍കരണമാണ്. അതിപ്പോഴെത്തിനില്‍ക്കുന്നത്, വിവാഹേതരബന്ധങ്ങള്‍ മാത്രമല്ല സ്വവര്‍ഗരതിയടക്കമുള്ള, ലൈംഗികവ്യതിയാനങ്ങളായി ഇതുവരെ കരുതിപ്പോന്ന കാര്യങ്ങളെല്ലാം, അനുവദിക്കപ്പെടേണ്ടതാണെന്നയിടത്താണ്. ഇണകള്‍ക്ക് തൃപ്തി നല്‍കുന്ന ലൈംഗികബന്ധങ്ങളെല്ല ശരിയാണ്; നന്മായാണ്. വിവാഹേതരബന്ധങ്ങളാകാം; സ്ത്രീയും സ്ത്രീയും തമ്മിലുള്ള ലെസ്ബിയന്‍ ലൈംഗികബന്ധങ്ങളാകാം. പുരുഷനും പുരുഷനും തമ്മിലുള്ള സഡോമിയാകാം; ഗ്രൂപ്പ് സെക്‌സിനെപ്പോലെയുള്ള ലൈംഗിക വൈകൃതങ്ങളാകാം- ഇതെല്ലാം അനുവദിക്കപ്പെടണമെന്നും ധാര്‍മികമായി പരിഗക്കപ്പെടണമെന്നുമാണ് വാദം. ഇസ്‌ലാം ഇതിനെതിര് നില്‍ക്കുന്നു. ഇസ്‌ലാം, ലൈംഗികതയുടെ പൂര്‍ത്തീകരണം നിയമപരമായി ഇണകളായിക്കഴിഞ്ഞവര്‍ തമ്മിലുള്ള ബന്ധത്തിലൂടെ മാത്രമേ ആകാന്‍ പാടുള്ളുവെന്ന് നിഷ്‌കര്‍ഷിക്കുന്നു. ഈയൊരു തലത്തില്‍ നിന്നുകൊണ്ടാണ് ഇസ്‌ലാം പറയുന്ന ബഹുഭാര്യത്വം പ്രകൃതിപരമാണോ അല്ലേ എന്ന പ്രശ്‌നം ചര്‍ച്ച ചെയ്യേണ്ടതുണ്ട്.

ചരിത്രത്തിലുടനീളം നമുക്ക് കാണാന്‍ കഴിയുന്ന ഒരു സത്യമുണ്ട്. ബഹുഭാര്യത്വം എക്കാലത്തും നിലനിന്നിരുന്നുവെന്നതാണത്. പ്രസിദ്ധ അമേരിക്കന്‍ ആന്ത്രോപോളജിസ്റ്റായ ജോര്‍ജ് പീറ്റര്‍ മര്‍ഡോക്, 1170 നാഗരികതകളെക്കുറിച്ച് പഠിച്ച്, ആ നാഗരികതകളില്‍ നിലനിന്ന വിവാഹബന്ധങ്ങളെക്കുറിച്ചുള്ള ഒരു ഗ്രാഫ് തയ്യാറാക്കിയിട്ടുണ്ട്.  അതില്‍ നമ്മള്‍ കാണുന്ന വസ്തുത, ബഹുഭൂരിപക്ഷം സമൂഹങ്ങളിലും നാഗരികതകളിലും ബഹുഭാര്യത്വം നിലനിന്നിരുന്നു എന്നതാണ്. മൊത്തം 1170 നാഗരികതകളില്‍ ഒരെണ്ണത്തില്‍ മാത്രമാണ് ബഹുഭര്‍തൃത്വം പേരിനെങ്കിലും നിലനിന്നിരുന്നതെന്നാണ് അദ്ദേഹം കണ്ടെത്തിയിട്ടുള്ളത്. ഏകഭാര്യത്വം നിലനിന്ന സമൂഹത്തേക്കാള്‍ എത്രയോ ഇരട്ടി സമൂഹങ്ങളില്‍ ബഹുഭാര്യത്വം നിലനിന്നിരുന്നു. ഇത് നമുക്ക് നല്‍കുന്ന ഒരറിവുണ്ട്. ആ അറിവിനോട് സൃഷ്ടിപരമായി പ്രതികരിക്കുവാന്‍ നമുക്ക് കഴിയണം. ഇസ്‌ലാം പറയുന്നുവെന്നതുകൊണ്ട് ആ അറിവിനെ പ്രാകൃതവല്‍ക്കരിക്കാന്‍ നാം ശ്രമിച്ചുകൂടാ. ബുദ്ധിജീവികള്‍ സത്യസന്ധരാണെങ്കില്‍ ഈ അറിവുമായി സത്യസന്ധമായി സംവദിക്കേണ്ടതുണ്ട്.POLYGYNY

Frequency of Marriage Types Across Cultures from the Standard Cross-Cultural Sample of pre-industrial societies (Murdock & White 1969)

ലൈംഗികതയെക്കുറിച്ച് ഒരുപാട് പഠനങ്ങള്‍ വന്നിട്ടുണ്ട്. ഈ പഠനങ്ങള്‍ ഒന്ന് വായിച്ചുനോക്കണം എന്നാണ് ഇസ്‌ലാമിന്റെ വിമര്‍ശകരോട് സൂചിപ്പിക്കാനുള്ളത്. ലൈംഗികപഠനരംഗത്ത് ഏറ്റവും അധികം അംഗീകരിക്കപ്പെടുന്ന മാസ്റ്റര്‍സ് ആന്റ് ജോണ്‍സന്റെ Human Sexual Responseഎന്ന കേസ് സ്റ്റഡികളാകട്ടെ, ഹാവ്‌ലോക്ക് എല്ലിസിന്റെ Studies in the psychology of sex എന്ന ഗവേഷണമാവട്ടെ, ആല്‍ഫ്രഡ് കിന്‍സെയുടെ  The Kinsey Report കളാകട്ടെ  പഠനവിധേയമാക്കിയാല്‍ നമുക്ക് ബോധ്യപ്പെടുന്നൊരു സത്യമുണ്ട്. പുരുഷന്മാരില്‍ ചിലര്‍ക്കെങ്കിലും അവരുടെ പ്രകൃതിയുമായി ബന്ധപ്പെട്ടു കിടക്കുന്നതാണ് ബഹുഭാര്യത്വം എന്നതാണത്. ഇതൊരു കേവലവാദമല്ല. ലോകത്തുടനീളം നിലനില്‍ക്കുന്ന വ്യഭിചാരത്തിന്റെ വ്യാപനം തന്നെ ഇക്കാര്യം വ്യക്തമാക്കുന്നു. കേരളത്തില്‍ തന്നെ നടന്ന പഠനങ്ങളില്‍ നിന്ന് മനസ്സിലാകുന്നത് 'സെക്‌സ് വര്‍ക്കേഴ്‌സി'ന്റെ അടുത്ത് പോകുന്ന ആളുകളില്‍ ബഹുഭൂരിപക്ഷവും വിവാഹിതരാണ് എന്നുള്ളതാണ്. അതു എന്തുകൊണ്ടെന്ന് പഠിക്കാന്‍ നാം സന്നദ്ധരാകണം. മാര്‍ക്‌സിന് ജെന്നി എന്ന ഭാര്യയെക്കൂടാതെ ഹെലന എന്ന വെപ്പാട്ടിയുമുണ്ടായിരുന്നു എന്നത് എല്ലാവര്‍ക്കും അറിയുന്ന കാര്യമാണ്. ഭൗതികവാദത്തിന്റെ എക്കാലത്തെയും വലിയ ധിഷണാശാലിയായി അറിയപ്പെടുന്ന റസ്സലിന് നാലു ഭാര്യമാരുണ്ടായിരുന്നു. ഇവര്‍ക്ക് പുറമെ മറ്റു പല സ്ത്രീകളുമായും അദ്ദേഹത്തിന് ബന്ധമുണ്ടായിരുന്നു; പുത്രന്റെ ഭാര്യയടക്കമുള്ളവരുമായിപ്പോലും അവിഹിതബന്ധമുണ്ടായിരുന്നു അദ്ദേഹത്തിന്. എന്ത് കൊണ്ടായിരുന്നു ഇതെന്ന് പഠിക്കുവാന്‍ റസ്സലിന്റെ അനുയായികളെന്ന് അഭിമാനിക്കുന്ന യുക്തിവാദികളെങ്കിലും സന്നദ്ധമാകണം.

പുരുഷലൈംഗികതയുമായി ബന്ധപ്പെട്ട് പറയുമ്പോള്‍ ബഹുഭാര്യത്വമെന്നത് പ്രകൃതിപരമാണ്; ചില പുരുഷന്മാര്‍ക്കെങ്കിലും അത് അനിവാര്യമാണ്; അതല്ലെങ്കില്‍ അധാര്‍മികതിയിലേക്ക് അവര്‍ പോകേണ്ടിവരും; വിവാഹേതരബന്ധങ്ങളില്‍ അവര്‍ അഭയം കണ്ടെത്തേണ്ടി വരുമെന്ന് ഈ വിഷയത്തെ ശാസ്ത്രീയമായി പഠിച്ചവരെല്ലാം വ്യക്തമാക്കിയിട്ടുള്ളതാണ്. ഇതില്‍ ഇനിയും സംശയമുള്ളവര്‍ മരീലാന്റിലെ കിന്‍സെ ഇന്‍സ്റ്റിറ്റ്യൂട്ടിന്റെ പഠനങ്ങളെടുത്ത് പരിശോധിച്ചാല്‍ മതിയാകും (www.kinseyinstitute.com)എന്ന വെബ് അഡ്രസില്‍ ഈ പഠനങ്ങള്‍ കാണാം) പ്രകൃതിപരമായ മനുഷ്യന്റെ ഈയൊരവസ്ഥയെ സൃഷ്ടിപരമായി നോക്കിക്കാണുകയും  കൃത്യമായ നിയമങ്ങളുടെ വെളിച്ചത്തില്‍ അതിനെ നിയന്ത്രിക്കുകയുമാണ്  ഇസ്‌ലാം ചെയ്യുന്നത്. മുഹമ്മദ് നബി(സ)കടന്നുവന്ന സമയത്ത് മക്കയില്‍ ഭാര്യമാരുടെ എണ്ണത്തിന് ഒരു പരിധിയുണ്ടായിരുന്നില്ല. അവരുടെ ലൈംഗികജീവിതം വളരെ കുത്തഴിഞ്ഞതായിരുന്നു. ഭാര്യമാര്‍ അവരുടെ ആഭിജാത്യത്തിന്റെ അടയാളമായിരുന്നു. ലൈംഗികത എന്നത് അവര്‍ക്ക് എപ്പോഴുമുള്ള ഒരേര്‍പ്പാടായിരുന്നു.  ഭാര്യമാരുടെ എണ്ണത്തിന് പരിധിയുണ്ടായിരുന്നില്ല. ഇസ്‌ലാം ഇതിന് നിയന്ത്രണമേര്‍പ്പെടുത്തി. നാലു വരെയേ ഭാര്യമാരാകാവൂ എന്ന് പഠിപ്പിച്ചു. ലൈംഗികസംപൂര്‍ത്തീകരണത്തിനും സംതൃപ്തിക്കും ഇസ്‌ലാം അനുവദിച്ചു. ഒപ്പം തന്നെ കൃത്യമായ കല്‍പന പുറപ്പെടുവിച്ചു; നിയമപരമായി ഇണകളായിത്തീര്‍ന്നവരുമായിട്ടല്ലാതെ ലൈംഗികബന്ധം പാടില്ല. അതോടൊപ്പം തന്നെ ഭാര്യമാര്‍ തമ്മില്‍ നീതിയോടു കൂടി പെരുമാറണമെന്ന് നിഷ്‌കര്‍ഷിക്കുകയും ചെയ്തു. അവരോട് അനീതി കാണിക്കാതിരിക്കുക. ഇസ്‌ലാം ബഹുഭാര്യത്വത്തെ നിയമമാക്കുന്നതിങ്ങനെയാണ്. ആ നിയമമാകട്ടെ, തികച്ചും പ്രകൃതിപരമാണ്; മാനവികമാണ്; സ്ത്രീ വിരുദ്ധമല്ല. അത് യഥാര്‍ഥത്തില്‍ എല്ലാ സമൂഹങ്ങളിലും നില നിന്നുപോന്നതാണ്. പഠനങ്ങള്‍ അത് പ്രകൃതിപരമാണെന്ന വസ്തുത വ്യക്തമാക്കുന്നുമുണ്ട്.

പുരുഷലൈംഗികതയുടെ സംപൂര്‍ത്തീകരണം ലക്ഷ്യമാക്കിക്കൊണ്ടാണ് ആധുനികസാമൂഹിക വ്യവസ്ഥതന്നെ ഡിസൈന്‍ ചെയ്യപ്പെട്ടിരിക്കുന്നത്. പരസ്യങ്ങള്‍ മുതല്‍ ഓഫീസ് ജീവനക്കാരികളുടെ വസ്ത്രങ്ങള്‍ വരെ ഈ ലക്ഷ്യം മുന്നില്‍ കണ്ടുകൊണ്ടു തന്നെയാണ് തയ്യാറാക്കപ്പെടുന്നത്. ''ഇഷ്ടമുള്ള ജോലി നേടാനും ഇഷ്ടമില്ലാത്ത ജോലി മാറാനു''മുള്ള മാതൃഭൂമി തൊഴില്‍ വാര്‍ത്താ പരസ്യം മാത്രം മതി ഇതിന്റെ ഉദാഹരണമായി. വിവാഹേതരലൈംഗികതയെ പാപമായി കാണാതിരിക്കുന്ന തലത്തിലേക്ക് മലയാളിമനസ്സിനെ നയിച്ചുകൊണ്ടുപോകുകയെന്ന ദൗത്യമേറ്റെടുത്തിരിക്കുന്നതും ഇസ്‌ലാമികനിയമങ്ങളെ വികൃതവല്‍ക്കരിച്ച് അവതരിപ്പിക്കുവാനായി കിട്ടാവുന്ന സകലരെയും ഉപയോഗപ്പെടുത്തി പരമാവധി പരിശ്രമിച്ചുകൊണ്ടിരിക്കുന്നതും മാതൃഭൂമി ആഴ്ചപ്പതിപ്പാണെന്നത് യാദൃച്ഛികതയൊന്നുമല്ല. ഇതു മൂന്നും യോജിക്കുന്നത് ഒരേയൊരു ബിന്ദുവിലാണ്. പുരുഷന്റെ അതിരില്ലാത്ത ലൈംഗികസുഖത്തിന് പറ്റിയ രൂപത്തിലുള്ള സാമൂഹികക്രമത്തിന്റെ സൃഷ്ടിയെന്ന ബിന്ദുവില്‍. പുരുഷന് എപ്പോഴും ഏത് സമയത്തും എങ്ങനെയും ലൈംഗികത ആസ്വദിക്കാവുന്ന പരുവത്തിലുള്ള സാമൂഹ്യസൃഷ്ടി. അവിടെ വിവാഹേതരബന്ധങ്ങള്‍ വിലക്കപ്പെടേണ്ട തെറ്റൊന്നുമല്ല; അത് വ്യത്യസ്തമായ പേരുകളില്‍ ന്യായീകരിക്കപ്പെടുകയാണ് ചെയ്യുന്നത്.

നിയമവിരുദ്ധമായ ലൈംഗികബന്ധങ്ങള്‍, അതിന് ഏത് പേരിട്ട് വിളിച്ചാലും ഇസ്‌ലാം അംഗീകരിക്കുന്നില്ല- വെറുക്കുന്നു. ഇസ്‌ലാമികമായ ഭരണക്രമം നിലനില്‍ക്കുന്ന രാഷ്ട്രത്തിലാണെങ്കില്‍ വ്യഭിചരിച്ചവര്‍ക്ക്-നാല് ദൃക്‌സാക്ഷികളുടെ സമ്മതം കൊണ്ട് കുറ്റം തെളിഞ്ഞാല്‍-വിവാഹിതരല്ലെങ്കില്‍ നൂറ് അടിയും വിവാഹിതരെങ്കില്‍ മരണം വരെ കല്ലേറും ലഭിക്കും. വ്യഭിചാരത്തെ ഇസ്‌ലാം എത്രമാത്രം വെറുക്കുന്നുവെന്ന് ഈ ശിക്ഷകള്‍ വ്യക്തമാക്കുന്നു. സമൂഹത്തിന്റെ അടിസ്ഥാനസ്ഥാപനമായ കുടുംബത്തിന്റെ തകര്‍ച്ചക്കും അതുവഴി ധാര്‍മികത്തകര്‍ച്ചക്കും വ്യഭിചാരം നിമിത്തമാവുമെന്നാണ് ഇസ്‌ലാമിന്റെ വീക്ഷണം. അതുകൊണ്ട് തന്നെ സദാചാരനിഷ്ഠമായ ഒരു സാമൂഹത്തിന്റെ സൃഷ്ടിക്കുവേണ്ടി ശ്രമിക്കുന്ന ഒരു ദര്‍ശനത്തിന് അത് പൂര്‍ണമായി ഇല്ലാതാക്കാനാവശ്യമായ നിയമങ്ങള്‍ ആവിഷ്‌കരിക്കേണ്ടി വരുന്നത് സാഭാവികമാണ്. അതോടൊപ്പം മനുഷ്യപ്രകൃതിക്ക് ഇണങ്ങുന്ന നിയമങ്ങളില്‍ വികാരപൂര്‍ത്തീകരണമെന്ന ജൈവികാവശ്യം നിര്‍വഹിക്കുവാനുള്ള മാര്‍ഗങ്ങള്‍ ഉണ്ടാകുകയും വേണം. ഇവിടെയാണ് ഇസ്‌ലാം ബഹുഭാര്യത്വം അനുവദിച്ചതിലെ യുക്തി മനസ്സിലാക്കാനാവുന്നത്.

ഇസ്‌ലാം ബഹുഭാര്യത്വം അനുവദിച്ചുവെന്ന് പറയുന്നതിനേക്കാള്‍ ശരിയാവുക, നിയമപരമായി ഇണകളായിത്തീന്നവര്‍ തമ്മിലുള്ളതല്ലാതെയുള്ള ലൈംഗികതയെ നിരോധിക്കുകയും പെണ്ണിന്റെ പൂര്‍ണ്ണമായ ഉത്തരവാദിത്തം ഏറ്റെടുക്കുവാന്‍ സന്നദ്ധതയുള്ളവരെന്ന് സ്വയം പ്രഖ്യാപിച്ച് അവളെ പരസ്യമായി സ്വീകരിക്കുന്നവരുമായി മാത്രം ബന്ധപ്പെടണമെന്ന് നിഷ്‌കര്‍ഷിക്കുകയും ചെയ്തു എന്ന് പറയുന്നതാവും. ഇക്കാര്യത്തില്‍ ഇസ്‌ലാമിനെ കുറ്റപ്പെടുത്തുന്നവര്‍ ഒന്നിലധികം പേരുമായി ബന്ധപ്പെടാനുള്ള വ്യക്തിയുടെ അവകാശം അംഗീകരിച്ചുകൊടുക്കണമെന്നും എന്നാല്‍ ഒന്നിലധികം വിവാഹം പാടില്ലെന്നു പറയുന്നവരുമാണ്. രണ്ടുപേര്‍ക്കും സമ്മതമാണങ്കില്‍ വിവാഹതരുമായോ അല്ലാത്തവരുമായോ ബന്ധപ്പെടുവാന്‍ എല്ലാവര്‍ക്കും സ്വാതന്ത്യമുണ്ടെന്നും പ്രലോഭനമോ പീഡനമോ കൂടാതെയാണ് പ്രസ്തുത ബന്ധം നടക്കുന്നതെങ്കില്‍ അതിലിടപെടുവാന്‍ നിയമത്തിന് കഴിയില്ലെന്നുമാണ് ആധുനിക ജനതാധിപത്യ രാഷ്ട്രങ്ങളുടെ ഭരണഘടനകളുടെയെല്ലാം നിലപാട്. ഇന്ത്യന്‍ ഭരണഘടനയും ഇതിന്നപപാദമല്ല. എത്രപേരുമായും ശാരീരികമായി ബന്ധപ്പെടാം, എന്നാല്‍ അവരെ നിയമാനുസൃതമായ ഇണയാക്കി തീര്‍ക്കുവാന്‍ പാടില്ലെന്ന നിലപാട് മാനവികവും ജനാധിപത്യവുപരവുമാണെന്ന് കൊട്ടിഘോഷിക്കുന്നവരാണ് ഒന്നിലധികം സ്ത്രീകളുമായി ശാരീരിക ബന്ധത്തിലേര്‍പ്പെടേണ്ട അനിവാര്യമായ സാഹചര്യങ്ങളുണ്ടെങ്കില്‍ അത് വിവാഹിത്തിലൂടെ മാത്രമെ പാടുള്ളുവെന്നും നാലിലധികം പേരെ വിവാഹം ചെയ്യാന്‍ പാടില്ലെന്നും നിഷ്‌കര്‍ഷിക്കുന്ന ഇസ്‌ലാമിനു നേരെ വാളോങ്ങുന്നതെന്ന വസ്തുത എത്രമാത്രം വലിയ വലിയ വിരോധാഭാസമല്ല! ഒന്നിലധികം സ്ത്രീകളെ ഇണകളാക്കേണ്ടിവരുന്നത് ചില പുരുഷന്‍മാര്‍ക്കെങ്കിലും പ്രകൃതിപരമാണെന്ന് വസ്തുതയുടെ വെളിച്ചത്തില്‍ അത് അനുവദിക്കുകുയം ഇണകളോട് നീതിപൂര്‍വ്വം പെരുമാറണമെന്ന് നിഷ്‌കര്‍ഷിക്കുകയും ചെയ്ത ഇസ്‌ലാമിന്റെ നിലാപടുതന്നെയാണ് ബഹുഭാര്യത്വവുമായി ബന്ധപ്പെട്ട ഏറ്റവും മാനവികമായ നിലപാടെന്ന വസ്തുത സത്യസന്ധമായി വിഷയത്തെ അപഗ്രഥിക്കുന്ന ആര്‍ക്കും ബോധ്യപ്പെടും.

പുരുഷാധിപത്യപരമാണ് ഇസ്‌ലാമികനിയമങ്ങളെന്നത് അടിസ്ഥാനരഹിതമായ ഒരു ആരോപണമാണ്. പുരുഷന്റെയും സ്ത്രീയുടെയും സ്രഷ്ടാവിനാണല്ലോ അവരുടെ പ്രകൃതിയെക്കുറിച്ച് നന്നായറിയുക. പടച്ചവൻ നിര്‍ദേശിക്കുന്ന ധാര്‍മിക വ്യവസ്ഥ ഒരിക്കലും തന്നെ ഒരു വിഭാഗത്തിന്റെ ആധിപത്യത്തിനും മറ്റേ വിഭാഗത്തിന്റെ അധഃസ്ഥിതത്വത്തിനും നിമിത്തമാവുകയില്ല. ഇസ്‌ലാമികനിയമങ്ങൾ സൂക്ഷ്മമായി അപഗ്രഥിച്ചാൽ അവയൊന്നും തന്നെ പുരുഷയാധിപത്യപരമോ പെണ്ണാധിപത്യപരമോ അല്ലെന്ന വസ്തുത മനസ്സിലാവും. പെണ്ണിന്റെ ശരീരത്തെ മാത്രം പരിഗണിക്കുന്ന ഭൗതികവാദത്തിന്റെ കണ്ണിലൂടെ നോക്കുന്നത് കൊണ്ടാണ് ചിലർക്ക് ഇസ്‌ലാം ആൺകോയ്മയിലധിഷ്ഠിതമാണെന്ന് തോന്നുന്നത്. പ്രശ്‌നം ഇസ്‌ലാമിക ധാര്‍മിക വ്യവസ്ഥയുടേതല്ല. മറിച്ച്, അതിനെ അളക്കാനുപയോഗിക്കുന്ന അളവുകോലിന്റേതാണ് എന്നർത്ഥം.

പുരുഷന്റെയും സ്ത്രീയുടെയും സഹകരണവും പാരസ്പര്യവുമാണ് കുടുംബമെന്ന സ്ഥാപനത്തിന്റെ നിലനില്‍പിന് ആധാരമെന്നാണ് ഇസ്‌ലാം പഠിപ്പിക്കുന്നത്. ധാര്‍മിക വ്യവസ്ഥ നിലനില്‍ക്കണമെങ്കില്‍ കുടുംബ മെന്ന സ്ഥാപനം കെട്ടുറപ്പോടുകൂടി നിലനില്‍ക്കണമെന്ന അടിത്തറയില്‍നിന്നുകൊണ്ടാണ് ഇസ്‌ലാം നിയമങ്ങളാവിഷ്‌കരിച്ചിരിക്കുന്നത്. കുടുംബംതന്നെ തകരേണ്ടതാണെന്ന തത്ത്വശാസ്ത്രത്തില്‍ വിശ്വസിക്കുന്നവര്‍ക്ക് ഇസ്‌ലാമിക നിയമങ്ങള്‍ അസ്വീകാര്യമായി അനുഭവപ്പെട്ടേക്കാം. എന്നാല്‍, ധാര്‍മികതയില്‍ അധിഷ്ഠിതമായ മനുഷ്യ സമൂഹത്തിന്റെ നിലനില്‍പിനെക്കുറിച്ച് ചിന്തിക്കുന്നവര്‍ക്കൊന്നും തന്നെ ഏതെങ്കിലുമൊരു ഇസ്‌ലാമിക നിയമം പുരുഷാധിപത്യത്തില്‍ അധിഷ്ഠിതമാണെന്ന് പറയാന്‍ കഴിയില്ല.

കുടുംബമെന്ന സ്ഥാപനത്തിന്റെ കെട്ടുറപ്പും ഭദ്രതയും ഉറപ്പുവരുത്തുന്നതിന് സ്ത്രീക്കും പുരുഷനും അവരുടേതായ പങ്കുവഹിക്കാനുണ്ടെന്നാണ് ഖുര്‍ആന്‍ പഠിപ്പിക്കുന്നത്. അവരുടെ അവകാശങ്ങളെയും ഉത്തരവാദിത്തങ്ങളെയും ബാധ്യതകളെയും കുറിച്ച ഇസ്‌ലാമിക നിയമങ്ങള്‍ ഈ അടിത്തറയില്‍നിന്നുകൊണ്ടുള്ളതാണ്. സ്ത്രീയെയും പുരുഷനെയും സംബന്ധിച്ച ഇസ്‌ലാമികവീക്ഷണത്തെ ഇങ്ങനെ സംഗ്രഹിക്കാം:

ഒന്ന്: സ്ത്രീയും പുരുഷനും ഒരേ ആത്മാവില്‍നിന്നുണ്ടായവരാണ്. ഒരു നാണയത്തിന്റെ രണ്ടു വശങ്ങള്‍പോലെയാണവര്‍. രണ്ടുപേരും സ്വതന്ത്രരാണെങ്കിലും ഇരുവരുടെയും പാരസ്പര്യമാണ് രണ്ടുപേര്‍ക്കും പൂര്‍ണത നല്‍കുന്നത്.

രണ്ട്: സ്ത്രീ പുരുഷനോ പുരുഷന്‍ സ്ത്രീയോ അല്ല. ഇരുവര്‍ക്കും തികച്ചും വ്യത്യസ്തവും അതേസമയം പരസ്പര പൂരകവുമായ അസ്തിത്വമാണുള്ളത്.

മൂന്ന്: സ്ത്രീക്കും പുരുഷനും അവകാശങ്ങളുണ്ട്. ഈ അവകാശങ്ങള്‍ നേടിയെടുക്കേണ്ടത് സംഘട്ടനത്തിലൂടെയല്ല, പാരസ്പര്യത്തിലൂടെയാണ്.

നാല്: രണ്ടു കൂട്ടര്‍ക്കും ബാധ്യതകളുണ്ട്. ഈ ബാധ്യതകള്‍ നിര്‍വഹിക്കുന്നതിലൂടെ മാത്രമേ വ്യഷ്ടിക്കും സമഷ്ടിക്കും നിലനില്‍ക്കാന്‍ കഴിയൂ.

അഞ്ച്: സ്ത്രീ പുരുഷധര്‍മം നിര്‍വഹിക്കുന്നതും പുരുഷന്‍ സ്ത്രീ ധര്‍മം നിര്‍വഹിക്കുന്നതും പ്രകൃതിയുടെ താല്‍പര്യത്തിനെതിരാണ്. ഓരോരുത്തരും അവരവരുടെ ധര്‍മങ്ങള്‍ നിര്‍വഹിക്കുകയാണ് വേണ്ടത്. ആറ്: ഓരോരുത്തരും അവരവരുടെ ധര്‍മം നിര്‍വഹിക്കുന്നതും അവകാശ ങ്ങള്‍ അനുഭവിക്കുന്നതും അപരന്റെ അവകാശങ്ങളെ ഹനിച്ചുകൊണ്ടായിക്കൂടാ.

പാരസ്പര്യത്തിന്റേതായ ഈ ഇസ്‌ലാമിക നിയമങ്ങളെല്ലാം തന്നെ പുരുഷനെയും സ്ത്രീയെയും തുല്യമായി പരിഗണിക്കുന്നവയാണ്. ഇവയിലൊന്നും തന്നെ ആൺകോയ്മയോ പെൺകോയ്മയോ ആർക്കും കാണാൻ കഴിയില്ല.

പുണ്യപുരുഷന്മാരെയും ആള്‍ദൈവങ്ങളെയുമെല്ലാം ആരാധിക്കുന്നവര്‍ പ്രസ്തുത ആരാധനമൂലം തങ്ങള്‍ക്ക് ഗുണം ലഭിക്കുന്നുണ്ടെന്ന് അവകാശപ്പെടാറുണ്ട്. ആത്യന്തികമായി മനുഷ്യര്‍ക്ക് ഗുണം ചെയ്യാന്‍ അല്ലാഹുവിന് മാത്രമേ കഴിയൂ. നാം പരസ്പരം ചെയ്യുന്ന ഉപകാരങ്ങളെല്ലാംതന്നെ അല്ലാഹു നമുക്ക് നല്‍കിയ അനുഗ്രഹത്തില്‍നിന്നുള്ള പരസ്പര സഹായം മാത്രമാണ്. അല്ലാഹു ഉപകാരം ചെയ്യാനുദ്ദേശിച്ചവനില്‍നിന്ന് അത് നീക്കിക്കളയുവാനോ ഉദ്ദേശിക്കാത്തവന് അത് നല്‍കുവാനോ ആര്‍ക്കും കഴിയില്ല. ഓരോരുത്തര്‍ക്കും എന്തെല്ലാമാണ് ഗുണകരമായി ഭവിക്കുകയെന്നും ദോഷകരമായി ത്തീരുകയെന്നുമെല്ലാം കൃത്യവും സൂക്ഷ്മവുമായി അറിയുന്നവനും അല്ലാഹു മാത്രമാണ്. അതുകൊണ്ടുതന്നെ ഉപകാരത്തിനും ഉപദ്രവങ്ങള്‍ നീങ്ങിപ്പോകുവാനുമെല്ലാം നാം പ്രാര്‍ഥിക്കേണ്ടത് അവനോട് മാത്രമാണ്. കാര്യകാരണബന്ധങ്ങള്‍ക്കതീതമായ രീതിയിലുള്ള ഉപകാരമോ ഉപദ്രവമോ ചെയ്യാന്‍ അവര്‍ക്കാര്‍ക്കുംതന്നെ കഴിയില്ല. ഖുര്‍ആന്‍ പറയുന്നത് കാണുക:

''അല്ലാഹുവിനു പുറമെ നിനക്ക് ഉപകാരം ചെയ്യാത്തതും, നിനക്ക് ഉപദ്രവം ചെയ്യാത്തതുമായ യാതൊന്നിനോടും നീ പ്രാര്‍ഥിക്കരുത്. നീ അപ്രകാരം ചെയ്യുന്നപക്ഷം തീര്‍ച്ചയായും നീ അക്രമികളുടെ  കൂട്ടത്തിലായിരിക്കും. നിനക്ക് അല്ലാഹു വല്ല ദോഷവും ഏല്‍പിക്കുന്നപക്ഷം അവനൊഴികെ അത് നീക്കം ചെയ്യാന്‍ ഒരാളുമില്ല.  അവന്‍ നിനക്ക് വല്ല ഗുണവും ഉദ്ദേശിക്കുന്നപക്ഷം അവന്റെ അനുഗ്രഹം  തട്ടിമാറ്റാന്‍ ഒരാളുമില്ല. തന്റെ ദാസന്മാരില്‍നിന്ന് താന്‍ ഇച്ഛിക്കുന്നവര്‍ക്ക് അത് (അനുഗ്രഹം) അവന്‍ അനുഭവിപ്പിക്കുന്നു. അവന്‍ ഏറെ പൊറുക്കുന്നവനും കരുണാനിധിയുമത്രെ'' (10:106,107).

''(നബിയേ), പറയുക: അല്ലാഹുവിനു  പുറമെ നിങ്ങള്‍ (ദൈവങ്ങളെന്ന്) വാദിച്ചുപോന്നവരെ നിങ്ങള്‍ വിളിച്ചുനോക്കു. നിങ്ങളില്‍നിന്ന് ഉപദ്രവം നീക്കുവാനോ (നിങ്ങളുടെ സ്ഥിതിക്ക്) മാറ്റം വരുത്തുവാനോ ഉള്ള കഴിവ് അവരുടെ അധീനത്തിലില്ല'' (17:56).

മരണപ്പെട്ടുപോയവരെ ആരാധിക്കുന്നവര്‍ അവര്‍ യഥാര്‍ഥത്തില്‍ മരിച്ചിട്ടില്ലെന്നും ജീവിച്ചിരിക്കുന്നവര്‍ സഹായിക്കുന്നതുപോലെ നമ്മെ സഹായിക്കാന്‍ അവര്‍ക്കു കഴിയുമെന്നും വാദിക്കാറുണ്ട്. മരണപ്പെട്ടവര്‍ മരണപ്പെട്ടവര്‍തന്നെയാണ്. ജീവിച്ചിരിക്കുന്നവര്‍ ചെയ്യുന്നതുപോലെയുള്ള പരസ്പര സഹായങ്ങള്‍ ചെയ്യാന്‍ അവര്‍ക്ക് കഴിയില്ല. അവര്‍ക്ക് സഹായിക്കാനാകുമെന്ന് കരുതുകയാണെങ്കില്‍ അവര്‍ ചെയ്യുന്നത് അഭൗതികമായ രീതിയില്‍, കാര്യകാരണബന്ധങ്ങള്‍ക്ക് അതീതമായ രീതിയിലുള്ള സഹായമായിരിക്കണം. അങ്ങനെ സഹായിക്കുവാന്‍ അല്ലാഹുവിന്നല്ലാതെ മറ്റാര്‍ക്കും കഴിയില്ല. മരണപ്പെട്ടവരാകട്ടെ, സ്വന്തം ഉയിര്‍ത്തെഴുന്നേല്‍പിന്റെ നാള്‍ എന്നാണെന്നു വരെ അറിയാനാവാത്തവരാണ്. പിന്നെയെങ്ങനെ പ്രാര്‍ഥിക്കുന്നവന്റെ മനസ്സറിയാന്‍ അവര്‍ക്ക് കഴിയും? ഖുര്‍ആനിന്റെ ഉല്‍ബോധനം ശ്രദ്ധിക്കുക:

''അല്ലാഹുവിന് പുറമെ അവര്‍ ആരെയൊക്കെ വിളിച്ചു പ്രാര്‍ഥിച്ചുകൊണ്ടിരിക്കുന്നുവോ അവര്‍ യാതൊന്നും സൃഷ്ടിക്കുന്നില്ല. അവരാകട്ടെ സൃഷ്ടിക്കപ്പെടുന്നവരുമാണ്. അവര്‍ (പ്രാര്‍ഥിക്കപ്പെടുന്നവര്‍) മരിച്ചവരാണ്. ജീവനുള്ളവരല്ല. ഏതു സമയത്താണ് അവര്‍ ഉയിര്‍ത്തെഴുന്നേല്‍പിക്കപ്പെടുക എന്ന് അവര്‍ അറിയുന്നുമില്ല''(16:20-21).

ഒരാളുടെ ഏറ്റവും വലിയ ബാധ്യത അവനെ സൃഷ്ടിച്ച് പരിപാലിക്കുന്ന തമ്പുരാനോടാണ്. സ്വന്തത്തോടും മറ്റുള്ളവരോടുമുള്ള ബാധ്യതകളുടെ നിര്‍വഹണം അല്ലാഹുവിനോടുള്ള ഉത്തരവാദിത്ത നിര്‍വഹണത്തിന് താഴെ മാത്രമെ വരുന്നുള്ളൂ. കാരണം, താനും മറ്റുള്ളവരുമെല്ലാം നിലവില്‍വന്നതും നിലനില്‍ക്കുന്നതുമെല്ലാം അവന്റെ അപാരമായ കാരുണ്യത്താലാണ്. തനിക്ക് എന്തെല്ലാമുണ്ടോ അതെല്ലാം അവന്‍ നല്‍കിയതാണ്. ഏത് സമയത്തും അവയെ പിന്‍വലിക്കുവാന്‍ കഴിയുന്നവനാണവന്‍. അവന്റെ അനുഗ്രഹങ്ങളാണ് താന്‍ അനുഭവിച്ചുകൊണ്ടിരിക്കുന്നത്. അതുകൊണ്ടുതന്നെ, അല്ലാഹുവിനോടുള്ള ബാധ്യതയാണ് ഒരാള്‍ക്ക് നിര്‍വഹിക്കുവാനുള്ള ഉത്തരവാദിത്തങ്ങളില്‍ ഒന്നാമത്തേത്. അത് നിര്‍വഹിക്കാതെ സ്വന്തത്തോടൊ മറ്റുള്ളവരോടോ ഉള്ള ഉത്തരവാദിത്തങ്ങള്‍ നിര്‍വഹിക്കുന്നതുകൊണ്ട് യാതൊരു കാര്യവുമില്ല.

എന്താണ് അല്ലാഹുവിനോട് നമുക്കുള്ള ഉത്തരവാദിത്തം? അവന്റെ അനുഗ്രഹങ്ങളനുഭവിച്ചുകൊണ്ട് ജീവിക്കുന്നവരാണ് നാം; തിരിച്ച് അല്ലാഹുവിനെ അനുഗ്രഹിക്കുവാന്‍ നമുക്ക് കഴിയുമോ? അവന്റെ കാരുണ്യത്താലാണ് നാം നിലനില്‍ക്കുന്നത്; തിരിച്ച് അല്ലാഹുവിന് കാരുണ്യം ചെയ്യാന്‍ നമുക്ക് കഴിയുമോ? അവനാണ് നമുക്കാവശ്യമായ ഭക്ഷണപാനീയങ്ങള്‍ ഒരുക്കിത്തന്നിരിക്കുന്നത്. തിരിച്ച് അവനെ ഭക്ഷിപ്പിക്കുവാനോ കുടിപ്പിക്കുവാനോ നമുക്ക് സാധിക്കുമോ? 'ഇല്ല'യെന്നാണല്ലോ ഇവയ്‌ക്കെല്ലാമുള്ള ഉത്തരം. എങ്കില്‍, പിന്നെയെങ്ങനെയാണ് അല്ലാഹുവിനോടുള്ള നമ്മുടെ ബാധ്യത നിര്‍വഹിക്കുകയെന്ന ചോദ്യം പ്രസക്തമാണ്. ഈ ചോദ്യത്തിന് അല്ലാഹുതന്നെ നല്‍കുന്ന ഉത്തരം ശ്രദ്ധിക്കുക:

''ജിന്നുകളേയും മനുഷ്യരേയും എന്നെ ആരാധിക്കുവാന്‍ വേണ്ടിയല്ലാതെ ഞാന്‍ സൃഷ്ടിച്ചിട്ടില്ല. ഞാന്‍ അവരില്‍നിന്ന് ഉപജീവനമൊന്നും ആഗ്രഹിക്കുന്നില്ല. അവര്‍ എനിക്ക് ഭക്ഷണം നല്‍കണമെന്നും ഞാന്‍ ആഗ്രഹിക്കുന്നില്ല. തീര്‍ച്ചയായും അല്ലാഹുതന്നെയാണ് ഉപജീവനം നല്‍കുന്നവനും ശക്തനും പ്രബലനും'' (51:56-58).

അല്ലാഹുവിനെ മാത്രം ആരാധിക്കുക. ഇതുമാത്രമാണ് നമുക്ക് ചെയ്യാനാവുന്ന കാര്യം. അല്ലാഹുവുമായി ബന്ധപ്പെട്ട വിഷയത്തില്‍ മനുഷ്യര്‍ക്ക് ചെയ്യാനാകുന്ന ഏക സംഗതി. അല്ലാഹു മാത്രമാണ് യഥാര്‍ഥത്തില്‍ ആരാധനകളെല്ലാം അര്‍ഹിക്കുന്നതെന്ന് നാം മനസ്സിലാക്കി. എന്നാല്‍, മനുഷ്യര്‍ക്ക് അല്ലാഹുവല്ലാത്തവരെ ആരാധിക്കുന്ന സമ്പ്രദായം സ്വീകരിക്കുവാന്‍ കഴിയും. അല്ലാഹുവുമായി ബന്ധപ്പെട്ട വിഷയത്തില്‍ മനുഷ്യര്‍ക്ക് സ്വാതന്ത്ര്യമുള്ള ഏക സംഗതിയാണത്. അല്ലാഹുവിന്റെ കാരുണ്യത്തെ തടയുകയോ മറ്റുള്ളവരെ പങ്കാളികളാക്കുകയോ ചെയ്യാന്‍ ആര്‍ക്കും കഴിയില്ല. അവന്റെ അധികാരാവകാശങ്ങളില്‍ കൈകടത്തുവാനോ പങ്കാളികളെവെക്കുവാനോ ഉള്ള കഴിവും മനുഷ്യര്‍ക്കില്ല. എന്നാല്‍ അവന്‍മാത്രം അര്‍ഹിക്കുന്ന ആരാധന മറ്റുള്ളവര്‍ക്ക് സമര്‍പ്പിക്കുവാന്‍ മനുഷ്യര്‍ക്ക് കഴിയും. ഇങ്ങനെ മറ്റുള്ളവരെ ആരാധിക്കുന്നത് മഹാപാപമാണ്. സകല പാപങ്ങളെക്കാളും വലിയ പാപം. കാരണം അല്ലാഹുവിനോടുള്ള ബാധ്യതയുടെ ലംഘനമാണത്. അവനോടുള്ള ഉത്തരവാദിത്തത്തിനുശേഷമേ മറ്റ് ഉത്തരവാദിത്തങ്ങളെല്ലാം വരുന്നുള്ളൂ. മറ്റുള്ള ബാധ്യതകള്‍ നിര്‍വഹിക്കാതിരിക്കുന്നത് പാപമാണെങ്കില്‍ അല്ലാഹുവിനോടുള്ള ബാധ്യത നിര്‍വഹിക്കാതിരിക്കുന്നത് മഹാപാപമാണ്.

ബഹുദൈവാരാധനയാണ് സകലവിധ തിന്മകളുടെയും മാതാവെന്നാണ് ഇസ്‌ലാമിക വീക്ഷണം. അതുകൊണ്ടുതന്നെ അല്ലാഹുവല്ലാത്തവരോടുള്ള ആരാധനയാണ് വിശുദ്ധ ഖുര്‍ആനില്‍ ഏറ്റവുമധികം  വിമര്‍ശിക്കപ്പെട്ടിരിക്കുന്നത്. ദൈവേതരന്മാരോട് പ്രാര്‍ഥിക്കുന്നതാണ് ഏറ്റവും വലിയ മഹാപാപമെന്നാണ് പ്രവാചകന്‍ പഠിപ്പിച്ചിരിക്കുന്നത്. ഒരിക്കലും പൊറുക്കപ്പെടാത്ത പാപമാണ് ദൈവത്തിന്റെ സത്തയിലോ ഗുണങ്ങളിലോ പ്രവര്‍ത്തനങ്ങളിലോ മറ്റുള്ളവരെ പങ്കുചേര്‍ക്കുകയെന്നത്. പ്രസ്തുത പാപം ചെയ്തവന്‍ ഇസ്‌ലാമില്‍ നിന്ന് പുറത്താണ്. വിശുദ്ധ ഖുര്‍ആന്‍ പറയുന്നതു കാണുക:

''തന്നോട് പങ്കുചേര്‍ക്കപ്പെടുക എന്നത് അല്ലാഹു പൊറുക്കപ്പെടുകയില്ല; തീര്‍ച്ച. അതൊഴിച്ചുള്ളത് അവനുദ്ദേശിക്കുന്നവര്‍ക്ക് അവന്‍ പൊറുത്തുകൊടുക്കുന്നതാണ്. ആര്‍ അല്ലാഹുവിനോട് പങ്കുചേര്‍ക്കുന്നുവോ അവന്‍ ബഹുദൂരം പിഴച്ചുപോയിരിക്കുന്നു'' (4:116).

ഇക്കാര്യം പ്രവാചകന്‍  (സ) വിശദീകരിച്ചു

''അബ്ദുല്ലാ (റ)പറയുന്നു:  ഞാന്‍ നബി  (സ)യോട് ചോദിച്ചു: ഏത് പാപമാണ് അല്ലാഹുവിന്റെ അടുക്കല്‍ ഏറ്റവും കഠിനമായത്. നബി  (സ) അരുളി: നിന്നെ സൃഷ്ടിച്ച നിന്റെ രക്ഷിതാവില്‍ നീ പങ്കാളിയെ സ്ഥാപിക്കലാണ്. ഞാന്‍ പറഞ്ഞു: അത് വളരെ ഗൗരവമേറിയതുതന്നെ. പിന്നെ ഏതാണ്: നബി  (സ)അരുളി: നിന്റെ കൂടെ ഭക്ഷണം കഴിച്ചുകളയുമെന്ന് ഭയന്ന് നിന്റെ സന്താനത്തെ വധിക്കലാണ്. പിന്നെ ഏതാണ്: ഞാന്‍ വീണ്ടും ചോദിച്ചു: അയല്‍ വാസിയുടെ ഭാര്യയെ നീ വ്യഭിചരിക്കലാണ്. നബി  (സ)പ്രത്യുത്തരം നല്‍കി'' (സ്വഹീഹുല്‍ ബുഖാരി, ഹദീഥ്: 6966).

അബ്ദുല്ലാഹിബ്‌നു മസ്ഊദ് (റ) നിവേദനം:''വിശ്വസിക്കുകയും, തങ്ങളുടെ വിശ്വാസത്തില്‍ അന്യായം കൂട്ടിക്കലര്‍ത്താതിരിക്കുകയും ചെയ്തവരാരോ അവര്‍ക്കാണ് നിര്‍ഭയത്വമുള്ളത്. അവര്‍തന്നെയാണ് നേര്‍മാര്‍ഗം പ്രാപിച്ചവര്‍'' (ഖുര്‍ആന്‍ 6:82) എന്ന ആയത്ത് അവതരിപ്പിക്കപ്പെട്ടപ്പോള്‍ ഞങ്ങള്‍ ചോദിച്ചു: 'പ്രവാചകരേ, ഞങ്ങളില്‍ സ്വന്തം ശരീരത്തോട് അക്രമം പ്രവര്‍ത്തിക്കാത്തവരാരാണ്?' അദ്ദേഹം പ്രതിവചിച്ചു: 'അത് നിങ്ങള്‍ പറഞ്ഞപോലെയല്ല. തങ്ങളുടെ വിശ്വാസത്തില്‍ അന്യായം കൂട്ടിക്കലര്‍ത്തുക'യെന്ന് പറഞ്ഞതിലെ അന്യായത്തിന്റെ വിവക്ഷ അല്ലാഹുവില്‍ പങ്കുചേര്‍ക്കലാണ്. ലുഖ്മാന്‍ തന്റെ മകനോടായി പറഞ്ഞത് നിങ്ങള്‍ കേട്ടിട്ടില്ലേ ''എന്റെ കുഞ്ഞ് മകനേ, നീ അല്ലാഹുവോട് പങ്കുചേര്‍ക്കരുത്. തീര്‍ച്ചയായും അങ്ങനെ പങ്കുചേര്‍ക്കുന്നത് വലിയ അക്രമംതന്നെയാകുന്നു'' (ഖുര്‍ ആന്‍: 31:13)(സ്വഹീഹുല്‍ ബുഖാരി: ഹദീഥ് 3110).

മനുഷ്യര്‍ക്ക് ആവശ്യമായതെല്ലാം ഇവിടെ ഒരുക്കി വെച്ചിരിക്കുന്നത് അല്ലാഹുവാണ്. അവനു പുറമെ ആരാധിക്കപ്പെടുന്നവര്‍ ആരായിരുന്നാലും അവരെ കൈവെടിയേണ്ടത് മുസ്‌ലിമിന്റെ ബാധ്യതയാണ്. ആരാധിക്കപ്പെടുന്നത് പുണ്യവാളനായാലും പിശാചായാലും സമമാണ്. രണ്ടുപേരും അതിന് അര്‍ഹരല്ല. പ്രാര്‍ഥനകള്‍ അല്ലാഹുവോട് മാത്രമാണ് നടത്തേണ്ടത്. സൃഷ്ടികളില്‍ ശ്രേഷ്ഠനായ മുഹമ്മദ് നബി (സ) യോടുപോലും പ്രാര്‍ഥനകള്‍ പാടില്ലെന്നാണ് ഇസ്‌ലാം പഠിപ്പിക്കുന്നത്. ഇബ്‌റാഹീം നബി  (സ) പറഞ്ഞതായി ഖുര്‍ആന്‍ ഉദ്ധരിക്കുന്ന വചനങ്ങള്‍ ഇവിടെ പ്രസക്തമാണ്.

''എന്നാല്‍ അവര്‍ (ആരാധിക്കപ്പെടുന്നവര്‍) എന്റെ ശത്രുകളാകുന്നു. ലോകരക്ഷിതാവ് ഒഴികെ. അതായത് എന്നെ സൃഷ്ടിച്ച് എനിക്ക് മാര്‍ഗദര്‍ശനം നല്‍കിക്കൊണ്ടിരിക്കുന്നവന്‍; എനിക്ക് ആഹാരവും കുടിനീരും നല്‍കുന്നവന്‍; എനിക്ക് രോഗം ബാധിച്ചാല്‍ അവനാണ് സുഖപ്പെടുത്തുന്നത്; എന്നെ മരിപ്പിക്കുകയും പിന്നെ ജീവിപ്പിക്കുകയും ചെയ്യുന്നവന്‍; പ്രതിഫലത്തിന്റെ നാളില്‍ ഏതൊരുത്തന്‍ എന്റെ തെറ്റുകള്‍ പൊറുത്തുതരുമെന്ന് ഞാന്‍ ആശിക്കുന്നുവോ അവന്‍'' (26:77-82).

പുണ്യവാന്മാര്‍ അനുഗ്രഹിക്കപ്പെടുകയും പാപികള്‍ ശിക്ഷിക്കപ്പെടുകയും ചെയ്യുന്ന മരണാനന്തരജീവിതത്തില്‍ ഒരിക്കലും പൊറുക്കപ്പെടാത്ത പാപം ചെയ്തുകൊണ്ട് മരണപ്പെട്ടവര്‍ക്ക് സ്വര്‍ഗം നിഷിദ്ധമാണ്; നരകത്തില്‍നിന്ന് അവര്‍ക്ക് മോചനമുണ്ടാവുകയില്ലെന്നാണ് ഖുര്‍ആന്‍ നല്‍കുന്ന പാഠം.

''മറ്‌യമിന്റെ മകന്‍ മസീഹ് തന്നെയാണ് അല്ലാഹു എന്ന് പറഞ്ഞവര്‍ തീര്‍ച്ചയായും അവിശ്വാസികളായിരിക്കുന്നു. എന്നാല്‍ മസീഹ് പറഞ്ഞത്; 'ഇസ്രാഈല്‍ സന്തതികളെ, എന്റെയും നിങ്ങളുടെയും രക്ഷിതാവായ അല്ലാഹുവെ നിങ്ങള്‍ ആരാധിക്കുവിന്‍. അല്ലാഹുവോട് വല്ലവനും പങ്കുചേര്‍ക്കുന്നപക്ഷം തീര്‍ച്ചയായും അല്ലാഹു അവന് സ്വര്‍ഗം നിഷിദ്ധമാക്കുന്നതാണ്. നരകം അവന്റെ വാസസ്ഥലമായിരിക്കുകയും ചെയ്യും. അക്രമികള്‍ക്ക് സഹായികളായി ആരുംതന്നെയില്ല' എന്നാണ്'' (5:72).

വിഷയവുമായി ബന്ധപ്പെട്ട വീഡിയോ

മനുഷ്യനെക്കാള്‍ ഉന്നതമായ മറ്റൊരു സൃഷ്ടിയും ഭൂമിയിലില്ലെന്ന് നമുക്കറിയാം. അവനെക്കാള്‍ ഉന്നതമായ ഒരുവന്‍ മാത്രമേയുള്ളൂ- അതാണ് സ്രഷ്ടാവ്. മനുഷ്യകഴിവില്‍ പെടാത്ത ഒട്ടനവധി കാര്യങ്ങള്‍ തനിക്കു ചുറ്റും തന്റെ സ്വന്തം ശരീരത്തിലും മനുഷ്യന് കാണാന്‍ കഴിയുന്നു. സ്വജീവന്‍ നിലനിര്‍ത്തുന്ന ഹൃദയമിടിപ്പോ രക്തചംക്രമണമോ മറ്റു ജീവികളില്‍നിന്ന് വ്യതിരിക്തമാക്കുന്ന ബൗദ്ധിക പ്രവര്‍ത്തനങ്ങളോ ഒന്നുംതന്നെ തന്റെ കഴിവിലോ നിയന്ത്രണ പരിധിയിലോ അല്ലെന്ന് അവന്‍ അറിയുന്നു.  ഭൂമിയില്‍ ജീവന്‍ നിലനിര്‍ത്താന്‍ പര്യാപ്തമായ രീതിയിലുള്ള സൂര്യന്റെ നിലനില്‍പോ ഭൂമിയുടെ പരിക്രമണമോ അന്തരീക്ഷത്തിന്റെ സംവിധാനമോ ഒന്നുംതന്നെ തന്റെ നിയന്ത്രണത്തിന്റെ വരുതിയിലല്ലെന്ന് അവന്‍ മനസ്സിലാക്കുന്നു. ഈ അറിവ് തന്നേക്കാള്‍ ശക്തിയുള്ള ഒന്നിനോട് സഹായമഭ്യര്‍ഥിക്കാന്‍ അവനെ പ്രേരിപ്പിക്കുന്നു. ഈ സഹായാഭ്യര്‍ഥനയാണ് പ്രാര്‍ഥന. ഭിഷഗ്വരനോട് രോഗി സഹായമഭ്യര്‍ഥിക്കുന്നതോ പണക്കാരനോട് പണിക്കാരന്‍ സാമ്പത്തിക സഹായം ചോദിക്കുന്നതോ പ്രാര്‍ഥനയുടെ പരിധിയില്‍ വരുന്നില്ല. അവ ലൗകികമായ കാര്യകാരണബന്ധങ്ങളുടെ അടിസ്ഥാനത്തിലുള്ള അര്‍ഥനയാകുന്നു. കാര്യകാരണബന്ധങ്ങള്‍ക്കതീതമായ രീതിയിലുള്ള സഹായാഭ്യര്‍ഥനയാണ് പ്രാര്‍ഥന. പ്രാര്‍ഥനയില്ലാത്ത ആരാധനകള്‍ വെറും ആചാരങ്ങള്‍ മാത്രമായി അവശേഷിക്കുന്നു.

സര്‍വലോകത്തിന്റെയും സ്രഷ്ടാവും സംരക്ഷകനും അല്ലാഹുവാണ്. മനുഷ്യരെ പടക്കുകയും പരിപാലിക്കുകയും ചെയ്യുന്നത് അവനാണ്. അവന്‍ പരമകാരുണികനും കരുണാനിധിയുമാണ്. നീതിമാനും പ്രതാപവാനുമാണ്. സര്‍വജ്ഞനും സര്‍വഗ്രാഹിയുമാണ്. സ്‌നേഹസമ്പന്നനും കൃപാനിധിയുമാണ്. അദൃശ്യകാര്യങ്ങള്‍ അറിയുന്നവനായ അല്ലാഹുവിനു മാത്രമേ മനുഷ്യമനസ്സിനകത്തുള്ളതെന്താണെന്ന് വ്യക്തമായി അറിയാന്‍ കഴിയൂ.

''നിശ്ചയം, അല്ലാഹു സര്‍വജ്ഞനും സൂക്ഷ്മജ്ഞനുമത്രേ'' (31:34).

മനുഷ്യകഴിവിന്നതീതമായ കാര്യങ്ങള്‍ അത്തരത്തിലുള്ള കഴിവുകളുണ്ടെന്നോ അത്തരം കഴിവുകള്‍ ഉള്ളവരില്‍ നിന്ന് നേടിത്തരാന്‍ കഴിയുമെന്നോ  കരുതുന്നവരോട് ചോദിക്കുന്നതിനാണല്ലോ പ്രാര്‍ഥനയെന്നു പറയുന്നത്. അഭൗതികമായ സഹായാഭ്യര്‍ഥനയാണ് പ്രാര്‍ഥനയെന്ന് പറയാം. പ്രാര്‍ഥനയുടെ ആത്മാവ് സ്ഥിതി ചെയ്യുന്നത് പ്രാര്‍ഥിക്കുന്നവന്റെ മനസ്സിനകത്താണ്. എത്ര സുന്ദരമായ പ്രാര്‍ഥ നാഗീതമായിരുന്നാലും അത് ഹൃദയത്തില്‍നിന്നു പുറത്തുവരുന്നതല്ലെങ്കില്‍  കേവലം വാചാടോപമായി പരിണമിക്കുന്നു. അതിന്ന് അധരങ്ങളുമായി മാത്രമേ ബന്ധം കാണൂ. പ്രാര്‍ഥിക്കുന്നവന്റെ ഹൃദയം പ്രാര്‍ഥിക്കപ്പെടുന്നവന്റെ സഹായം കാംക്ഷിക്കുമ്പോള്‍ മാത്രമാണ് പ്രാര്‍ഥനയെന്ന പദം അന്വര്‍ഥമാകുന്നത്. യഥാര്‍ഥത്തില്‍ പ്രാര്‍ഥനയാണ് എല്ലാവിധ ആരാധനകളുടെയും മജ്ജ. ആരാധനകളിലെ കര്‍മങ്ങള്‍ക്ക് ജീവനുണ്ടാകണമെങ്കില്‍ പ്രാര്‍ഥനയുണ്ടായിരിക്കണമെന്നര്‍ഥം. ഇതാണ് പ്രവാചകന്‍ പഠിപ്പിച്ചത്.

''നുഅ്മാനുബ്‌നു ബശീര്‍ (റ)നിവേദനം: നബി  (സ) പറഞ്ഞു പ്രാര്‍ഥന തന്നെയാകുന്നു ആരാധന. പിന്നീട് ഇങ്ങനെ ഓതി 'നിങ്ങളുടെ രക്ഷിതാവ് പറഞ്ഞിരിക്കുന്നു: നിങ്ങള്‍ എന്നോട് പ്രാര്‍ത്ഥിക്കൂ. ഞാന്‍

നിങ്ങള്‍ക്ക് ഉത്തരം നല്‍കാം. എന്നെ ആരാധിക്കാതെ അഹങ്കാരം നടിക്കുന്നവരാരോ അവര്‍ വഴിയെ നിന്ദ്യരായിക്കൊണ്ട് നരകത്തില്‍ പ്രവേശിക്കുന്നതാണ്; തീര്‍ച്ച' (40:60)'' (ജാമിഉത്തിര്‍മിദി: ഹദീഥ്: 4294).

പ്രാര്‍ഥനയുള്‍ക്കൊള്ളുന്ന കര്‍മങ്ങളെല്ലാം ആരാധനയാണ്. ദിവ്യനായി കരുതപ്പെടുന്ന ഒരാള്‍ അനുസരിക്കപ്പെടുന്നത് പ്രാര്‍ഥനാനിര്‍ഭരമായ മനസ്സോടുകൂടിയാണ്. അയാളില്‍നിന്ന് കാര്യകാരണ ബന്ധങ്ങള്‍ക്ക് അതീതമായ ഗുണമോ ദോഷമോ പ്രതീക്ഷിച്ചുകൊണ്ടാണ് പ്രസ്തുത അനുസരണം. അതുകൊണ്ടുതന്നെ ആ അനുസരണം ആരാധനയായിത്തീരുന്നു. ദൈവങ്ങള്‍ക്ക് സമര്‍പ്പിക്കപ്പെട്ട ഭക്ഷണം കഴിക്കുന്നത് പ്രാര്‍ഥനാനിര്‍ഭരമായ മനസ്സോടെയാണ്. അപ്പോള്‍ അത് കഴിക്കുന്നത് ആരാധനയാണ്. ശവകുടീരങ്ങളെ പ്രദക്ഷിണംവെക്കുന്നത് പ്രാര്‍ഥനയുള്‍ക്കൊള്ളുന്ന മനസ്സോടെയാവുമ്പോള്‍ ആ പ്രദക്ഷിണം ആരാധനയായിത്തീരുന്നു. ഇങ്ങനെ പ്രാര്‍ഥനയുള്‍ക്കൊള്ളുന്ന കര്‍മങ്ങളെല്ലാം ആരാധനയാണ്. ആരാധന അല്ലാഹുവിന് മാത്രമെ സമര്‍പ്പിക്കാവൂയെന്നാണ് ഇസ്‌ലാമിന്റെ അധ്യാപനം. പ്രാര്‍ഥനയുള്‍ക്കൊള്ളുന്ന ഒരു കര്‍മവും അല്ലാഹുവിന്നല്ലാതെ സമര്‍പ്പിച്ചുകൂടെന്നര്‍ഥം. പ്രാര്‍ഥനയാണ് ആരാധനയുടെ മജ്ജയെന്ന് വ്യക്തമാക്കുന്ന ഏതാനും ഖുര്‍ആന്‍ സൂക്തങ്ങള്‍ കാണുക:

''പള്ളികള്‍ അല്ലാഹുവിനുള്ളതാകുന്നു. അതിനാല്‍ നിങ്ങള്‍ അല്ലാഹുവോടൊപ്പം ആരെയും വിളിച്ച് പ്രാര്‍ഥിക്കരുത്'' (72:18).

''നിങ്ങളുടെ രക്ഷിതാവ് പറഞ്ഞിരിക്കുന്നു: നിങ്ങള്‍ എന്നോട് പ്രാര്‍ഥിക്കൂ. ഞാന്‍ നിങ്ങള്‍ക്ക് ഉത്തരം നല്‍കാം. എന്നെ ആരാധിക്കാതെ അഹങ്കാരം നടിക്കുന്നവരാരോ അവര്‍ വഴിയെ നിന്ദ്യരായിക്കൊണ്ട് നരകത്തില്‍ പ്രവേശിക്കുന്നതാണ്; തീര്‍ച്ച'' (40:60).

''അവനാകുന്നു എന്നെന്നും ജീവിച്ചിരിക്കുന്നവന്‍. അവനല്ലാതെ യാതൊരു ആരാധ്യനുമില്ല. അതിനാല്‍ കീഴ്‌വണക്കം അവന് നിഷ്‌ക്കളങ്കമാക്കിക്കൊണ്ട് നിങ്ങള്‍ അവനോട് പ്രാര്‍ഥിക്കുക. ലോകങ്ങളുടെ രക്ഷിതാവായ അല്ലാഹുവിന് സ്തുതി'' (40:65).

''വല്ലവനും അല്ലാഹുവോടൊപ്പം മറ്റുവല്ല ആരാധ്യരെയും വിളിച്ച് പ്രാര്‍ഥിക്കുന്നപക്ഷം -അതിന് അവന്റെ പക്കല്‍ യാതൊരു പ്രമാണവും ഇല്ലതന്നെ- അവന്റെ വിചാരണ അവന്റെ രക്ഷിതാവിന്റെ അടുക്കല്‍വെച്ചുതന്നെയായിരിക്കും. സത്യനിഷേധികള്‍ വിജയം പ്രാപിക്കുകയില്ല; തീര്‍ച്ച'' (23:117).

''അല്ലാഹുവിന് പുറമെ, ഉയര്‍ത്തെഴുന്നേല്‍പ്പിന്റെ നാളുവരെയും തനിക്ക് ഉത്തരം നല്‍കാത്തവരെ വിളിച്ച് പ്രാര്‍ഥിക്കുന്നവനെക്കാള്‍ വഴിപിഴച്ചവന്‍ ആരുണ്ട്? അവരാകട്ടെ ഇവരുടെ പ്രാര്‍ഥനയെപ്പറ്റി ബോധമില്ലാത്തവരാകുന്നു. മനുഷ്യരെല്ലാം ഒരുമിച്ച് കൂട്ടപ്പെടുന്ന സന്ദര്‍ഭത്തില്‍ അവര്‍ ഇവരുടെ ശത്രുക്കളായിരിക്കുകയും ചെയ്യും. ഇവര്‍ അവരെ ആരാധിച്ചിരുന്നതിനെ അവര്‍ നിഷേധിക്കുന്നവരായിത്തീരുകയും ചെയ്യും'' (46:5,6).

മുകളില്‍ സൂചിപ്പിച്ച ഖുര്‍ആന്‍ വാക്യങ്ങളില്‍ 'പ്രാര്‍ഥന'യും 'ആരാധന'യും തത്തുല്യ പദങ്ങള്‍ എന്ന നിലയ്ക്കാണ് പ്രയോഗിച്ചിരിക്കുന്നതെന്ന കാര്യം പ്രത്യേകം ശ്രദ്ധേയമാണ്. ഉദാഹരണത്തിന് അവസാനം ഉദ്ധരിച്ച സൂറത്തുല്‍ അഹ്ഖാഫിലെ അഞ്ചും ആറും വചനങ്ങള്‍ പരിശോധിക്കുക. അഞ്ചാമത്തെ വചനത്തില്‍ അല്ലാഹു വല്ലാത്തവരോടുള്ള പ്രാര്‍ഥനയെ ഏറ്റവും വലിയ വഴികേടായി ചിത്രീകരിച്ചിരിക്കുന്നു. ഇങ്ങനെ പ്രാര്‍ഥിക്കപ്പെട്ടവര്‍ ഉയിര്‍ത്തെഴുന്നേല്‍പിന്റെ നാളില്‍ അവരെ ആരാധിച്ചതിനെ നിഷേധിക്കുമെന്നാണ് അടുത്ത വചനത്തില്‍ പറഞ്ഞിരിക്കുന്നത്. പ്രാര്‍ഥനതന്നെയാണ് ആരാധനയെന്ന വസ്തുത ഈ സൂക്തങ്ങള്‍ സുതരാം വ്യക്തമാക്കുന്നു.

അപ്പോള്‍, മനുഷ്യര്‍ ആരാധനകള്‍ അര്‍പ്പിക്കേണ്ടത് ആര്‍ക്കാണ്? ആരോടാണവന്‍ പ്രാര്‍ഥിക്കേണ്ടത്? സകലമാന കഴിവുകളുടെയും ഉടമസ്ഥാവകാശം ആര്‍ക്കാണോ, രഹസ്യങ്ങളും പരസ്യങ്ങളുമറിയു ന്നവനാരാണോ, ഹൃദയത്തിനകത്തുള്ളത് വ്യക്തമായി അറിയുന്നവന്‍ ആരാണോ അവന്നാണ്; അവന്നു മാത്രമാണ് ആരാധനകളര്‍പ്പിക്കേണ്ടതെന്ന് സാമാന്യബുദ്ധി പറയുന്നു. ഈ സാമാന്യബുദ്ധിയുടെ വിധിതന്നെയാണ് ഇസ്‌ലാമിന്റെ അടിസ്ഥാന തത്ത്വം. സ്രഷ്ടാവും പരിപാലകനുമായ അല്ലാഹുവിന് മാത്രമേ ആരാധനകളര്‍പ്പിക്കാവൂ എന്നാണ് ഇസ്‌ലാം നിഷ്‌കര്‍ഷിക്കുന്നത്. പ്രാര്‍ഥനാവഴിപാടുകള്‍ അര്‍പ്പിക്കപ്പെടാന്‍ അര്‍ഹന്‍ അല്ലാഹു മാത്രമാണെന്ന് വിശ്വസിക്കുമ്പോള്‍ മാത്രമാണ് അല്ലാഹുവിലുള്ള വിശ്വാസം പൂര്‍ണമാകുന്നത്. സത്തയിലും സ്വഭാവങ്ങളിലും കഴിവുകളിലും അദ്വിതീയനായ ഒരു അസ്തിത്വത്തിലുള്ള വിശ്വാസം കൊണ്ട് ഇസ്‌ലാം വിവക്ഷിക്കുന്ന ഏകദൈവവിശ്വാസം (തൗഹീദ്) പൂര്‍ണമാകുന്നില്ല. ആ ഏകാസ്തിത്വത്തിനു മാത്രം ആരാധനകളര്‍പ്പിക്കപ്പെടുമ്പോഴാണ് അത് പൂര്‍ണ മാകുന്നത്. ഇസ്‌ലാമിക വീക്ഷണത്തില്‍, ഏകദൈവവിശ്വാസി (മുവഹ്ഹിദ്) എന്നു വിളിക്കപ്പെടണമെങ്കില്‍ അല്ലാഹു ഏകനാണെന്ന് വിശ്വസിച്ചാല്‍ മാത്രം പോരാ; അവന്നുമാത്രം അര്‍ഹതയുള്ള കാര്യങ്ങള്‍ അവന്നു മാത്രം അര്‍പ്പിക്കുകകൂടി വേണം. അല്ലാഹു മാത്രമാണ് ആരാധനകളര്‍ഹിക്കുന്നതെന്ന് പ്രഖ്യാപിച്ചു കൊണ്ടാണല്ലോ ഒരാള്‍ മുസ്‌ലിമാകുന്നത്. ആരാധനകളഖിലവും അല്ലാഹുവിന് മാത്രം അര്‍പ്പിക്കുന്നവനാണ് മുസ്‌ലിമെന്നര്‍ഥം.

അല്ലാഹുവിന്റെ സത്തയുടെ സവിശേഷതയാണ് കാരുണ്യമെന്നും അവന്‍തന്നെ കാരുണ്യത്തെ തന്റെമേല്‍  ഒരു ബാധ്യതയാക്കി  നിശ്ചയിക്കുകയും ചെയ്തിരിക്കുന്നുവെന്നുമാണ് ഖുർആൻ പഠിപ്പിക്കുന്നത്. . അവനില്‍ നിന്നുണ്ടാവുന്നതെല്ലാം കാരുണ്യമായിരിക്കുമെന്നർത്ഥം.. ദുഷ്ടതയോ ക്രൂരതയോ അവനില്‍ നിന്നുണ്ടാവുകയില്ല. ദിവ്യകാരുണ്യത്തിന്റെ ഭാഗമായി അവന്‍ സംവിധാനിച്ച വ്യവസ്ഥകള്‍ ചിലപ്പോള്‍ വ്യക്തികള്‍ക്ക് പ്രയാസകരമായി തോന്നിയേക്കാം. എന്നാല്‍ അവപോലും ആത്യന്തികമായി നന്മയും കാരുണ്യവുമായിരിക്കും.

നാം അനുഭവിക്കുന്ന വേദനയുടെ കാര്യമെടുക്കുക. പ്രതിസന്ധികളെ അതിജീവിക്കുന്നതിന് മനുഷ്യശരീരത്തില്‍ അല്ലാഹു സംവിധാനിച്ച അതിസൂക്ഷ്മമായ സംവിധാന ങ്ങളിലൊന്നാണ് വേദന. ദിവ്യകാരുണ്യത്തിന്റെ ഉത്തമ നിദര്‍ശനമാണത്. ശരീരത്തിന്റെ ഭാഗങ്ങളിലേതിലെങ്കിലും ഏതെങ്കിലും തരത്തിലുള്ള മുറിവോ പ്രയാസങ്ങളോ ഉണ്ടായാല്‍ അത് ഉടനെ അറിയുകയും ആവശ്യമായ രക്ഷാമാര്‍ഗങ്ങള്‍ കണ്ടെത്തുകയും ചെയ്യാനായി അല്ലാഹു നിശ്ചയിച്ച വാര്‍ത്താവിനിമയ സംവിധാനത്തിലെ പ്രധാനപ്പെട്ട ഒരു ഘടകമാണ് വേദന. മുറിവ് പറ്റിയിടത്ത് നന്നാ ക്കാനാവശ്യമായ സംവിധാനങ്ങള്‍ക്കുള്ള സന്ദേശങ്ങള്‍ കേന്ദ്രനാഡീവ്യൂഹത്തില്‍ നിന്ന് വരണം. അതിന് ഇന്ന സ്ഥലത്ത് മുറിവ് പറ്റിയിട്ടുണ്ടെന്ന് കേന്ദ്രനാഡീവ്യൂഹം അറിയണം. വേദനയെന്ന ദൂതന്‍ ഈ അറിയിപ്പാണ് നിര്‍വഹിക്കുന്നത്. പ്രയാസമനുഭവിക്കുന്ന സ്ഥലത്തെക്കുറിച്ച് ജാഗ്രതയില്‍ കഴിയുവാനും നിതാന്തശ്രദ്ധ  ചെലുത്തുവാനും മനുഷ്യനെ സജ്ജമാക്കുന്നതും വേദന തന്നെ! ഉറങ്ങുമ്പോഴും ഉണര്‍ന്നിരിക്കുമ്പോഴുമുള്ള നമ്മുടെ ശരീരത്തിന്റെ കാവല്‍ക്കാരനാണ് വേദനയെന്നര്‍ഥം. പരമകാരുണികനായ അല്ലാഹു നമ്മുടെ ശരീരത്തില്‍ സംവിധാനിച്ച പ്രതിരോധ സന്നാഹങ്ങളിലൊന്ന്. വേദന അനുഭവിക്കുന്നവന് അത് പ്രയാസകരമായി ഭവിക്കുന്നുവെന്നത് നേരാണ്. എന്നാല്‍ സൂക്ഷ്മമായി ചിന്തിച്ചാല്‍ വേദനയും പ്രയാസങ്ങളുമെല്ലാം ആത്യന്തികമായി ദിവ്യകാരുണ്യത്തെ തന്നെയാണ് വെളിപ്പെടുത്തുന്നതെന്ന് കാണാന്‍ കഴിയും.

രോഗങ്ങളുടെയും ദുരിതങ്ങളുടെയും അവസ്ഥയും ഇതില്‍ നിന്ന് ഭിന്നമല്ല. പ്രതിസന്ധികളെ അതിജീവിക്കുന്നതിന്നും വിശുദ്ധജീവിതം നയിക്കുന്നതിന്നുമായി മനുഷ്യശരീരത്തെയും മനസ്സിനെയും പര്യാപ്തമാക്കുന്ന സംവിധാനങ്ങളുടെ ഭാഗം തന്നെയാണ് നാം ദുരിതമെന്നും രോഗമെന്നും വിളിക്കുന്ന കാര്യങ്ങളെല്ലാമെന്ന് സൂക്ഷ്മമായി പരിശോധിച്ചാല്‍ കാണാന്‍ കഴിയും. അന്തരീക്ഷത്തിലെ ഊഷ്മവ്യതിയാനങ്ങളോട് പൊരുത്തപ്പെടുവാനുള്ള ശരീരത്തിലെ സംവിധാനങ്ങളും തെറ്റായ ആഹാരപാനീയങ്ങളുടെ ദുഷ്ഫലങ്ങളില്‍ നിന്ന് രക്ഷിക്കുവാനുള്ള സന്നാഹങ്ങളും രോഗാണുക്കളെയും മറ്റും പ്രതിരോധിക്കുവാനാവശ്യമായ നടപടികളുമാണ് രോഗമായി നമുക്കനുഭവപ്പെടുന്നത് എന്നതല്ലേ യാഥാര്‍ഥ്യം? ഈ യാഥാര്‍ഥ്യം മനസ്സിലാക്കുവാനും ഉള്‍ക്കൊള്ളാനും സന്നദ്ധമാവാതെ പ്രതിരോധ സംവിധാനങ്ങളായ രോഗങ്ങളോട് പ്രതികാരാത്മകമായി പ്രതികരിക്കുന്നതാണ് മാരകമായ രോഗങ്ങള്‍ക്ക് നിമിത്തമാവുന്നത് എന്നാണല്ലോ വൈദ്യശാസ്ത്ര രംഗത്തെ പുതിയ പഠനങ്ങള്‍ വ്യക്ത മാക്കുന്നത്. ഏത് പ്രതിസന്ധിയെയും തരണംചെയ്യാന്‍ നമ്മെ പര്യാപ്തമാക്കുവാനായുള്ള പരമകാരുണികന്റെ സംവിധാനങ്ങളാണ് നമുക്ക് പ്രയാസകരമായിത്തോന്നുന്ന കാര്യങ്ങള്‍പോലുമെന്ന വസ്തുതയാണ് ഇവിടെയെല്ലാം അനാവൃതമാവുന്നത്.

ദിവ്യകാരുണ്യത്തിന്റെ ഭാഗമായി നമ്മുടെ ശരീരത്തില്‍ സംവിധാനിച്ച ചില കാര്യങ്ങളെങ്കിലും വ്യക്തിപരമായ അനുഭവത്തില്‍ പ്രയാസകരമായി ഭവിക്കുന്നുവെന്ന വസ്തുത നാം മനസ്സിലാക്കി. ഇതേപോലെത്തന്നെയാണ് ഇവിടെയുള്ള കാര്യങ്ങളെല്ലാം എന്ന് ഉള്‍ക്കൊള്ളുവാന്‍ നമുക്ക് കഴിയണം.  ദുരിതവും  പ്രയാസവുമായി വ്യക്തികള്‍ക്കോ സമൂഹങ്ങള്‍ക്കോ തോന്നുന്ന കാര്യങ്ങള്‍ ആത്യന്തികമായി നന്മയും മനുഷ്യരാശിയെ മൊത്തമായെടുത്താല്‍ ദിവ്യ കാരുണ്യത്തിന്റെ പ്രകടനവുമാണെന്ന വസ്തുത ശരിയാംവണ്ണം ഉള്‍ക്കൊണ്ട ഒരാള്‍ക്ക് പിന്നെ നിരാശാബോധമുണ്ടാവുകയില്ല; അയാള്‍ നിഷേധാത്മക നിലപാടുകളെടുക്കുകയുമില്ല. പരമകാരുണികനില്‍ നിന്ന് ഉണ്ടാവുന്നതെല്ലാം അവന്റെ കാരുണ്യത്തിന്റെ പ്രകടനമാണെന്നും അത് തനിക്ക് തിന്മയായി ഭവിക്കുന്നത് ചിലപ്പോള്‍ തന്റെ വര്‍ത്തനങ്ങളിലുള്ള തകരാറുകൊണ്ടാണെന്നും മനസ്സിലാക്കുന്നവന്‍, തന്റെ പ്രവര്‍ത്തനങ്ങളിലുള്ള വീഴ്ചകളെപ്പറ്റി സദാ ബോധവാനായിരിക്കും; എന്തെങ്കിലും വീഴ്ച സംഭവിച്ചാല്‍ തന്നെ അത് ഉടനടി തിരുത്തുവാന്‍ അവന്‍ സന്നദ്ധനുമായിരിക്കും; എന്നിട്ടും താന്‍ പ്രയാസത്തില്‍ തന്നെയാണെങ്കില്‍ അത് ദിവ്യകാരുണ്യത്തിന്റെ ഭാഗമാണെന്നും തനിക്ക് ആത്യന്തികമായ നന്മയാണത് പ്രദാനം ചെയ്യുകയെന്നും മനസ്സിലാക്കി ക്ഷമിക്കുവാനും സഹിക്കുവാനും അവന്ന് സാധിക്കും. അല്ലാഹുവിനെ പരമകാരുണികനായി മനസ്സിലാക്കാത്തതാണ് നിരാശാബോധത്തിന്റെയും ആത്മഹത്യാ പ്രവണതയുടെയുമെല്ലാം അടിസ്ഥാന കാരണം, ദൈവിക കാരുണ്യത്തെക്കുറിച്ച് നിരാശരാകുന്നവര്‍ വഴിപിഴച്ചവര്‍ മാത്രമാണെന്നാണ് ഖുര്‍ആന്‍ പഠിപ്പിക്കുന്നത്.

''അദ്ദേഹം (ഇബ്രാഹീം) പറഞ്ഞു: തന്റെ രക്ഷിതാവിന്റെ കാരു ണ്യത്തെപ്പറ്റി ആരാണ് നിരാശപ്പെടുക? വഴിപിഴച്ചവരല്ലാതെ'' (15:56).

വിഷയവുമായി ബന്ധപ്പെട്ട വീഡിയോ

ഏകനും അദ്വിതീയനും സൃഷ്ടാവും സംരക്ഷകനുമാണ് ഖുർആൻ പരിചയപ്പെടുത്തുന്ന അല്ലാഹു. കാരുണ്യം തന്റെ മേൽ ബാധ്യതയാക്കിയിരിക്കുന്ന പരമ കാരുണികൻ. നന്മകളുടെയെല്ലാം പാരമ്യമായ അവൻ സഗുണസമ്പൂര്ണനാണ്. അല്ലാഹുവിന്റെ ഏകത്വവും പരാശ്രയമില്ലായ്മയും സമ്യക്കായും അതോടൊപ്പം സംക്ഷിപ്തമായും അവതരിപ്പിക്കുന്ന ഖുര്‍ആനിലെ ചെറിയൊരു അധ്യായമാണ് സൂറതുല്‍ ഇഖ്‌ലാസ്വ്. പ്രസ്തുത അധ്യായത്തിന്റെ സാരം ഇങ്ങനെയാണ്:

''പറയുക: കാര്യം അല്ലാഹു ഏകനാണ് എന്നതാകുന്നു. അല്ലാഹു പരാശ്രയം വേണ്ടാത്തവനും സകലര്‍ക്കും ആശ്രയമായിട്ടുള്ളവനു മാകുന്നു. അവന്‍ പിതാവോ സന്തതിയോ അല്ല. അവനു തുല്യമായി ഒന്നുംതന്നെയില്ല'' (112:1-4).

അല്ലാഹുവിനെക്കുറിച്ച ഇസ്‌ലാമിക വീക്ഷണത്തിന്റെ സമ്യക്കായ ഒരു വിവരണമാണിതെന്ന വസ്തുത ഖുര്‍ആന്‍ വ്യാഖ്യാതാക്കളെല്ലാം സൂചിപ്പിച്ചിട്ടുണ്ട്. അല്ലാഹുവിനെക്കുറിച്ച് ആദ്യമായി പറയുന്ന രണ്ടു വിശേഷണങ്ങള്‍ അവന്റെ സത്തയെ സംബന്ധിച്ച് നമുക്ക് മനസ്സിലാക്കാന്‍ കഴിയുന്നതിന്റെ പരമാവധി മനസ്സിലാക്കിത്തരുന്നുണ്ട്. ഒന്നാമതായി, അല്ലാഹു അഹദാണെന്ന കാര്യമാണത്. പരമമായ ഏകത്വമവകാശപ്പെടാന്‍ കഴിയുന്ന ഒരേയൊരു ഉണ്മ അല്ലാഹുവിന്‍േറതു മാത്രമാണ്. രണ്ടാമതായി അല്ലാഹു സ്വമദ് ആണെന്ന കാര്യമാണ്. ഒന്നിന്റെയും ആശ്രയം ആവശ്യമില്ലാത്തവനും എന്നാല്‍ അവന്റെ ആശ്രയം കൂടാതെ ഒന്നിനും നിലനില്‍ക്കാന്‍ സാധിക്കാത്തവനുമെന്നാണ് സ്വമദിന്റെ വിവക്ഷ. സൃഷ്ടികളുടെ നിലനില്‍പുതന്നെ പരസ്പരം ആശ്രയിച്ചുകൊണ്ടാണെന്ന് പറഞ്ഞുവല്ലോ. അല്ലാഹുവിന്നാകട്ടെ നിലനില്‍ക്കാന്‍  ആരുടെയും ആശ്രയം ആവശ്യമില്ല. അവന്റെ ആശ്രയം കൊണ്ടാണ് സൃഷ്ടികളെല്ലാം നിലനില്‍ക്കുന്നതുതന്നെ. അല്ലാഹുവിനെ സംബന്ധിച്ച 'സ്വമദ്' എന്ന വിശേഷണം ഒരിക്കല്‍ക്കൂടി അവനെക്കുറിച്ച് നമുക്ക് സൂക്ഷ്മമായി മനസ്സിലാക്കുവാന്‍ കഴിയില്ലെന്ന് വ്യക്തമാക്കുന്നു. ഒരു വസ്തുവെക്കുറിച്ച് നമുക്ക് നേരിട്ടു മനസ്സിലാക്കുവാന്‍ കഴിയില്ലെങ്കില്‍ അത് ആശ്രയിച്ചുനില്‍ക്കുന്ന മറ്റു വസ്തുക്കളെക്കുറിച്ച് പഠിച്ചുകൊണ്ട് അതിനെക്കുറിച്ച് കുറേയെല്ലാം മനസ്സിലാക്കാനാവും. അല്ലാഹു മറ്റൊന്നിനെയും ആശ്രയിച്ചല്ല നിലനില്‍ക്കുന്നതെന്നതിനാല്‍ ആ ഒരു മാര്‍ഗമുപയോഗിച്ചും അതിനെക്കുറിച്ച് മനസ്സിലാക്കുക അസാധ്യമാണ്. അവനെ അറിയുവാന്‍ പ്രമാദമുക്തമായ ദൈവിക പ്രമാണങ്ങളെ ആശ്രയിക്കുകയല്ലാതെ മറ്റു നിര്‍വാഹങ്ങളൊന്നുമില്ലെന്ന് സാരം.

'അവന്‍ പിതാവോ സന്തതിയോ അല്ല'യെന്ന ഖുര്‍ആനിക പരാമര്‍ശം അല്ലാഹുവിനെ സംബന്ധിച്ച് നിലനിന്നിരുന്ന രണ്ട് മൂഢവിശ്വാസങ്ങളെ തിരുത്തുകയാണ് ചെയ്യുന്നത്. ദൈവത്തിന് പുത്രന്മാരും പുത്രിമാരുമുണ്ടെന്ന വിശ്വാസമാണ് ഒന്നാമത്തേത്. ക്രിസ്ത്യാനികള്‍ യേശു ദൈവപുത്രനാണെന്ന് വാദിക്കുക വഴി യഥാര്‍ഥത്തില്‍ ദൈവികസത്തയുടെ ഏറ്റവും വലിയ സവിശേഷതയായ ഏകത്വത്തെ നിഷേധിക്കുകയാണ് ചെയ്യുന്നത്. ത്രിയേകവിശ്വാസം ദൈവിക ഏകത്വത്തിന്റെ വ്യക്തമായ നിഷേധമാണെന്ന കാര്യം എടുത്തു പറയേണ്ടതില്ല. സത്തയില്‍ ഒന്നായ മൂന്നു വ്യക്തികളെ സങ്കല്‍പി ക്കുന്നതിലേക്ക് ക്രൈസ്തവസഭ നയിക്കപ്പെട്ടതിന്റെ പ്രധാനപ്പെട്ട കാരണം ദൈവപുത്രസങ്കല്‍പമാണെന്ന് കാണാനാവും. പിതൃസ്വഭാവങ്ങള്‍ പുത്രനില്‍ പ്രതിഫലിക്കുമെന്ന സാമാന്യ തത്ത്വപ്രകാരം ദൈവപുത്രനായി വാഴ്ത്തപ്പെട്ട ക്രിസ്തു മെല്ലെ മെല്ലെ ദൈവികസത്തയിലേക്ക് സ്ഥാനാരോഹണം ചെയ്യിക്കപ്പെട്ടതായിട്ടാണ് ത്രിത്വത്തിന്റെ ചരിത്രത്തില്‍ നാം കാണുന്നത്. മാലാഖമാര്‍ ദൈവപുത്രികളായി സങ്കല്‍പിക്കപ്പെട്ടിരുന്ന സമൂഹങ്ങളിലെ സ്ഥിതിയും തഥൈവ.

'അവന്‍ സന്തതിയല്ല' അഥവാ ജനിച്ചവനല്ല എന്ന നിഷേധം മറ്റൊരു മൂഢധാരണകൂടി തിരുത്തുന്ന തിനുവേണ്ടിയാണ് പ്രയോഗിച്ചിരിക്കുന്നത്. അവതാര സങ്കല്‍പമാണത്. ദൈവം ഭൂമിയില്‍ മനുഷ്യരൂപത്തില്‍ ജനിച്ചുവളരുകയും ധര്‍മസംസ്ഥാപനം നടത്തുകയും ചെയ്യുമെന്ന വിശ്വാസം വിവിധ മതസമൂഹങ്ങള്‍ക്കിടയില്‍ നിലനിന്നിരുന്നു. സ്ഥലകാല സാതത്യത്തിന്നതീതനായ അല്ലാഹു ഒരു മാതാവിന്റെ ഉദരത്തില്‍ വളരുകയും പ്രസവിക്കപ്പെടുകയും ചെയ്യുകയെന്നത് സങ്കല്‍പിക്കാന്‍തന്നെ കഴിയില്ല. ജനനം, മരണം തുടങ്ങിയവയെല്ലാം സ്ഥലകാലത്തിനകത്തേക്ക് അല്ലാഹു നിര്‍ണയിച്ച പ്രതിഭാസങ്ങളത്രെ. അല്ലാഹുവില്‍ ജനനം ആരോപിക്കുന്നവര്‍ അവനെ സ്ഥലകാലത്തില്‍ പരിമിതപ്പെടുത്തുകയാണ് ചെയ്യുന്നത്. അതുകൊണ്ടുതന്നെ, പരമമായ ഏകത്വമെന്ന ദൈവികസത്തയുടെ സവിശേഷപ്രകൃതിക്ക് വിരുദ്ധമാണത്.

'അവന്നു തുല്യമായിട്ട് ആരും തന്നെയില്ല'യെന്ന സൂറത്തുല്‍ ഇഖ്‌ലാസ്വിലെ അവസാനത്തെ പരാമര്‍ശവും ദൈവികസത്തയെക്കുറിച്ച് മനുഷ്യന് അവന്റെ അപഗ്രഥന രീതിയുപയോഗിച്ച് സൂക്ഷ്മമായി അറിയാന്‍ കഴിയില്ലെന്ന് വ്യക്തമാക്കുന്നതുതന്നെ. വസ്തുക്കളെക്കുറിച്ച പഠനത്തിന് സാധാരണയായി ഉപയോഗിക്കുന്ന ഒരു മാര്‍ഗമാണ് അവക്ക് തുല്യമായ മറ്റു വസ്തുക്കളെക്കുറിച്ച് പഠിക്കുകയെന്നത്.

''അല്ലാഹുവിന് തുല്യനായി ആരുംതന്നെയില്ല'' (112:4) യെന്ന നിഷേധത്തിലൂടെ പ്രസ്തുത സാധ്യതയും തള്ളിക്കളയുകയാണ് ഖുര്‍ആന്‍ ചെയ്യുന്നത്.

''അവന് സദൃശമായി ഒന്നുംതന്നെയില്ല''(42:11)യെന്ന ഖുര്‍ആനിക പരാമര്‍ശവും സ്രഷ്ടാവിന്റെ സത്തയെ സൃഷ്ടികളുടേതുമായി താരതമ്യം ചെയ്ത് മനസ്സിലാക്കാനുള്ള സാധ്യതയെ പൂര്‍ണമായി നിഷേധിക്കുന്നു.

'അവന്നു തുല്യമായിട്ട് ആരും തന്നെയില്ല'യെന്ന അല്ലാഹുവിനെക്കുറിച്ച പരിശുദ്ധ ഖുർആനിലെ പരാമർശം (112:4) ദൈവികസത്തയെക്കുറിച്ച് മനുഷ്യന് അവന്റെ അപഗ്രഥന രീതിയുപയോഗിച്ച് സൂക്ഷ്മമായി അറിയാന്‍ കഴിയില്ലെന്ന് വ്യക്തമാക്കുന്നു. . വസ്തുക്കളെക്കുറിച്ച പഠനത്തിന് സാധാരണയായി ഉപയോഗിക്കുന്ന ഒരു മാര്‍ഗമാണ് അവക്ക് തുല്യമായ മറ്റു വസ്തുക്കളെക്കുറിച്ച് പഠിക്കുകയെന്നത്.''അല്ലാഹുവിന് തുല്യനായി ആരുംതന്നെയില്ല'' യെന്ന നിഷേധത്തിലൂടെ പ്രസ്തുത സാധ്യത തള്ളിക്കളയുകയാണ് ഖുര്‍ആന്‍ ചെയ്യുന്നത്.''അവന് സദൃശമായി ഒന്നുംതന്നെയില്ല''(42:11)യെന്ന ഖുര്‍ആനിക പരാമര്‍ശവും സ്രഷ്ടാവിന്റെ സത്തയെ സൃഷ്ടികളുടേതുമായി താരതമ്യം ചെയ്ത് മനസ്സിലാക്കാൻ കഴിയില്ലെന്ന് വ്യക്തമാക്കുന്നു.ദൈവികസത്ത മനുഷ്യവിശദീകരണങ്ങൾക്ക് വഴങ്ങുന്നതല്ലെന്ന വസ്തുതയാണ് ഇതെല്ലാം മനസ്സിലാക്കിത്തരുന്നത്.

ദൈവികസത്ത മാനുഷിക വിശദീകരണത്തിന്നതീതമാണെങ്കില്‍ പിന്നെയെങ്ങനെയാണ് മനുഷ്യര്‍ അല്ലാഹുവിനെക്കുറിച്ച് മനസ്സിലാക്കുക? അല്ലാഹുവിന്റെ അനുഗ്രഹങ്ങളെക്കുറിച്ച് ചിന്തിച്ചും അവന്റെ ഗുണവിശേഷങ്ങള്‍ മനസ്സിലാക്കിയും നാഥനെ അറിയുവാനാണ് വിശുദ്ധ ഖുര്‍ആനിന്റെ അനുശാസന. അല്ലാഹുവിന്റെ സവിശേഷതകളെ ദ്യോതിപ്പിക്കുന്ന നാമവിശേഷണങ്ങളിലൂടെയാണ് ഖുര്‍ആന്‍ അല്ലാഹുവിനെ പരിചയപ്പെടുത്തുന്നത് എന്നു പറയുന്നതാവും ശരി. ഈ നാമവിശേഷണങ്ങളിലൂടെ അല്ലാഹുവിനെക്കുറിച്ച് മനസ്സിലാക്കുവാന്‍, തന്റെ ബുദ്ധിയുടെ പരിധിക്കുള്ളില്‍നിന്നു കൊണ്ട് ഏതൊരാള്‍ക്കും സാധിക്കും. യഥാര്‍ഥത്തില്‍, 'അല്ലാഹുവിനെ അറിയുക'യെന്നാല്‍ അവന്റെ ഗുണങ്ങളെ അറിയുകയാണ്. വിശുദ്ധ ഖുര്‍ആന്‍ പരിചയപ്പെടുത്തുന്ന രൂപത്തില്‍ നാഥനെ അറിഞ്ഞ ഒരാള്‍ പിന്നെ 'അല്ലാഹുവിനല്ലാതെ ആരാധനകളര്‍പ്പിക്കുകയില്ല. എല്ലാ അര്‍ഥത്തിലും ആരാധനകളര്‍ഹിക്കുന്ന ഏക അസ്തിത്വം അല്ലാഹുവാണെന്ന് വ്യക്തമാക്കുന്നതാണ് നാമ-ഗുണ വിശേഷണങ്ങൾ എന്നർത്ഥം.

''അല്ലാഹുവിന് ഏറ്റവും വിശിഷ്ടമായ നാമങ്ങളുണ്ട്. അതിനാല്‍ ആ നാമങ്ങളില്‍ നിങ്ങള്‍ അവനെ വിളിച്ചുകൊള്ളുക. അവന്റെ പേരുകളില്‍ കൃത്രിമം കാണിക്കുന്നവരെ നിങ്ങള്‍ വിട്ടുകളയുക. അവര്‍ ചെയ്തുവരുന്നതിന്റെ ഫലം അവര്‍ക്ക് വഴിയെ നല്‍കപ്പെടും'' (7:180).

ദൈവം നിര്‍ഗുണനാണെന്ന വീക്ഷണം ഖുര്‍ആനികാധ്യാപനങ്ങള്‍ക്ക് വിരുദ്ധമാണ്. ഖുര്‍ആന്‍ പരിചയപ്പെടുത്തുന്ന അല്ലാഹു സഗുണസമ്പൂര്‍ണനാണ്. പക്ഷേ, ദൈവികഗുണങ്ങള്‍ മാനുഷികഗുണ ങ്ങളുമായി താരതമ്യം ചെയ്യാവതല്ല. മനുഷ്യന്‍ സൃഷ്ടിയായതുകൊണ്ടുതന്നെ അവന്റെ ഗുണങ്ങള്‍ പരിമിതങ്ങളും സ്ഥലകാലബന്ധനത്തിന്നധീനവുമാണ്. ഇതില്‍നിന്നു വ്യത്യസ്തമായി സ്ഥല-കാലങ്ങളെ സൃഷ്ടിച്ച അല്ലാഹുവിന്റെ ഗുണങ്ങള്‍ പരിമിതികള്‍ക്കതീതമാണ്.

''അവന്ന് തുല്യനായി ആരും തന്നെയില്ല''(112:4)യെന്നും ''അവന്ന് സദൃശ്യമായി യാതൊന്നുമില്ല'' (42:11)യെന്നുമുള്ള ഖുര്‍ആനിക പരാമര്‍ശങ്ങള്‍ സ്രഷ്ടാവിന്റെ സത്തയെക്കുറിച്ചു മാത്രമല്ല, ഗുണങ്ങളെക്കുറിച്ചുകൂടിയുള്ളതാണ്. അല്ലാഹുവിന്റെ ഗുണങ്ങള്‍ക്കു തുല്യമായ ഗുണങ്ങള്‍ സൃഷ്ടികള്‍ക്കൊന്നിനുമുണ്ടാവില്ല; തീര്‍ച്ച!

അല്ല. പ്രപഞ്ചത്തിന്റെ സ്രഷ്ടാവാണ് അല്ലാഹു. പരമാണു മുതല്‍ നക്ഷത്രജാലങ്ങള്‍ വരെയുള്ള ചെറുതും വലുതുമായ പ്രാപഞ്ചികവസ്തുക്കളുടെയെല്ലാം സ്രഷ്ടാവാണവന്‍. പ്രപഞ്ചത്തിലെ ഓരോ വസ്തുവിനും വസ്തുതക്കുമെല്ലാം കാരണങ്ങളന്വേഷിക്കുന്ന മനുഷ്യന്‍ അവസാനം കാരണങ്ങളാവശ്യമില്ലാത്ത ഒരു മഹാശക്തിയില്‍ എത്തിച്ചേരുകയാണ് ചെയ്യുന്നത്. അവനാണ് എല്ലാ അന്വേഷണങ്ങളുടെയും അന്ത്യം. സകലവിധ അന്വേഷണങ്ങളും തേട്ടങ്ങളും ഈ പരമമായ അന്ത്യത്തിലെത്തുമ്പോള്‍ അവസാനിക്കുന്നു. പ്രപഞ്ചത്തിലെ ഒരൊറ്റ പ്രതിഭാസവും കാരണമാവശ്യമില്ലാത്തതായി നമുക്ക് കാണാന്‍ കഴിയുന്നില്ല. ചെറുതും വലുതുമായ സകല വസ്തുക്കളും കാരണം തേടുന്നു. അപ്പോള്‍ കാരണങ്ങളാവശ്യമില്ലാത്ത പരാശക്തി പ്രപഞ്ചത്തിന് ഉപരിയായിരിക്കണം; പദാര്‍ഥാതീതനായിരിക്കണം. സകലമാന പ്രതിഭാസങ്ങളുടെയും സ്രഷ്ടാവാണ് അല്ലാഹു.  പ്രപഞ്ചത്തിന് ഉപരിയായ അല്ലാഹുവിന്റെ സത്ത പദാര്‍ഥാതീതമാണെന്ന് സാരം.

അല്ലാഹുവിന്റെ സൃഷ്ടിയാണ് സ്ഥല-കാല സാതത്യ (time-space continuum)ത്തിലധിഷ്ഠിതമായ നമ്മുടെ പ്രപഞ്ചം.  അല്ലാഹുവിന്റെ സത്ത സ്ഥല-കാല സാതത്യത്തിന് പുറത്തുള്ളതാണെന്ന് പറയുന്നത് അത് കൊണ്ടാണ്.  അത്തരമൊരു സത്തയെ സംബന്ധിച്ച് മനുഷ്യബുദ്ധിക്ക് സ്വന്തമായ നിഗമനങ്ങള്‍ നെയ്‌തെടുക്കുവാനോ മനസ്സിലാക്കുവാനോ കഴിയില്ല. അത് അവന്റെ വിവരങ്ങള്‍ക്ക് അതീതമാണ്. സ്ഥലകാല ബന്ധനത്തിലുള്ള മനുഷ്യ മസ്തിഷ്‌കത്തിന് സ്ഥലത്തിനും കാലത്തിനും അതീതനായ അല്ലാഹുവിനെക്കുറിച്ച ചിത്രങ്ങളുണ്ടാക്കാന്‍ കഴിയുകയില്ല.

'അവനെ ദൃഷ്ടികള്‍ കണ്ടെത്തുകയില്ല; അവനാകട്ടെ ദൃഷ്ടികളെ കണ്ടെത്തുകയും ചെയ്യുന്നു; അവന്‍ സൂക്ഷ്മജ്ഞാനിയും അഭിജ്ഞനുമാകുന്നു'(6:103)വെന്ന ഖുര്‍ആനിക വചനം വിരല്‍ ചൂണ്ടുന്നത് ഈ വസ്തുതയിലേക്കാണ്.

ദൈവിക സത്തയെക്കുറിച്ച് സ്വന്തമായ എന്തെങ്കിലും നിഗമനത്തിലെത്താന്‍ പര്യാപ്തമായ രീതിയിലല്ല മനുഷ്യബുദ്ധി സംവിധാനിക്കപ്പെട്ടിരിക്കുന്നതെന്ന് പറഞ്ഞുവല്ലോ. അല്ലാഹുവിന്റെ സത്തയെ സംബന്ധിച്ച് അവന്‍തന്നെ വെളിപ്പെടുത്തിയ കാര്യങ്ങള്‍ സ്വീകരിക്കുക മാത്രമേ നമുക്ക് നിര്‍വാഹമുള്ളൂ. വിശുദ്ധ ഖുര്‍ആനും തിരുനബിയുടെ മൊഴികളും അല്ലാഹുവിനെപ്പറ്റി പറഞ്ഞ കാര്യങ്ങള്‍ അപ്പടി സ്വീകരിക്കുകയും അവ പരിമിതമായ നമ്മുടെ മസ്തിഷ്‌കപരിധിക്കകത്ത് വിശദീകരിക്കുവാന്‍ അസാധ്യമായതാണെങ്കില്‍ 'അല്ലാഹുവിന്നറിയാം' എന്നു മാത്രം വിനീതമായി പറയുകയും ചെയ്യുന്ന രീതിയാണ് സച്ചരിതരായ പൂര്‍വപണ്ഡിതന്മാര്‍ സ്വീകരിച്ചുപോന്നിട്ടുള്ളത്; അതാണ് ശരിയും. ഇതില്‍നിന്ന് തെറ്റി, തങ്ങളുടെ ബുദ്ധിക്കും യുക്തിക്കുമനുസൃതമായ രീതിയില്‍ ദൈവികസത്തയെ വ്യാഖ്യാനിക്കുവാനും വിശദീകരിക്കുവാനും ശ്രമിച്ചവര്‍  തത്ത്വശാസ്ത്രത്തിന്റെ ഊരാക്കുടുക്കുകളില്‍പെട്ട് ചര്‍ച്ചകളും ചര്‍ച്ചോപ ചര്‍ച്ചകളുമായി ദൈവത്തെക്കുറിച്ച വിചിത്രവും വികലവുമായ ചിത്രങ്ങളുണ്ടാക്കുകയാണ് അവസാനം ചെയ്തിട്ടുള്ളത്. മുസ്‌ലിം സമൂഹത്തിന് പുറത്തും അകത്തുമുള്ളവര്‍ ഇത്തരം ചുഴികളിലകപ്പെട്ടിട്ടുണ്ട്. 'ദാത്ത്' എന്നതിന്റെ മലയാളപരിഭാഷയാണ് സത്ത. അല്ലാഹുവിനെ സംബന്ധിച്ച പരാമര്‍ശങ്ങളിലൊരിടത്തും 'ദാത്ത്' എന്ന് ഖുര്‍ആന്‍ പ്രയോഗിച്ചിട്ടില്ല. മറ്റു സ്ഥലങ്ങളില്‍ ഖുര്‍ആന്‍ പ്രയോഗിച്ച 'ദാത്തി'ന് സത്തയെന്ന വിവക്ഷയില്ലതാനും.

മനുഷ്യവിഭാവനക്കും ചിന്തയ്ക്കുമതീതമാണ് അല്ലാഹുവിന്റെ ദാത്ത് എന്നാണ് ഖുര്‍ആനും നബിവചനങ്ങളും പഠിപ്പിക്കുന്നത്. ഈ പ്രമാണങ്ങളെ പൂര്‍ണമായി സ്വീകരിച്ചുകൊണ്ട് അല്ലാഹുവിനെ സംബന്ധിച്ച് ഖുര്‍ആനില്‍ പറഞ്ഞ കാര്യങ്ങള്‍ അപ്പടി വിശ്വസിച്ചംഗീകരിക്കുക മാത്രമേ സൂക്ഷ്മാലുക്കള്‍ക്ക് നിര്‍വാഹമുള്ളൂ.

പ്രപഞ്ചത്തിന്റെ സ്രഷ്ടാവ് പ്രപഞ്ചാതീതനായിരിക്കുമെന്നത് സരളമായ യുക്തിയാണ്. ഒരു വസ്തുവിന്റെ സ്രഷ്ടാവ് അതിന്നു പുറത്തായിരിക്കുമെന്ന വസ്തുത മനസ്സിലാക്കുവാന്‍ തത്ത്വശാസ്ത്രത്തിന്റെ ഊരാക്കുടുക്കുകളൊന്നും ആവശ്യമില്ല. ഈ സരളമായ വസ്തുതയാണ് ഉപരിലോകത്തുള്ളവനായി അല്ലാഹുവിനെ പരിചയപ്പെടുത്തുന്ന ഖുര്‍ആന്‍ സൂക്തങ്ങള്‍ വ്യക്തമാക്കുന്നത്.

''ഉപരിയിലുള്ളവന്‍ നിങ്ങളെ ഭൂമിയില്‍ ആഴ്ത്തിക്കളയുന്നതിനെപ്പറ്റി നിങ്ങള്‍ നിര്‍ഭയരായിരിക്കുകയാണോ? അപ്പോള്‍ അത് (ഭൂമി) ഇളകിമറിഞ്ഞുകൊണ്ടിരിക്കും. അതല്ല ഉപരിയിലുള്ളവന്‍ നിങ്ങളുടെ നേരെ ഒരു ചരല്‍ വര്‍ഷം അയക്കുന്നതിനെപ്പറ്റി നിങ്ങള്‍ നിര്‍ഭയരായിരിക്കുകയാണോ? എന്റെ താക്കീത് എങ്ങനെയുണ്ടെന്ന് വഴിയേ നിങ്ങള്‍ അറിഞ്ഞുകൊള്ളും''(67:16,17).

വിഷയവുമായി ബന്ധപ്പെട്ട വീഡിയോ

പ്രപഞ്ചസ്രഷ്ടാവിന്റെ അസ്തിത്വത്തെക്കുറിച്ച് മനസ്സിലാക്കാൻ ബുദ്ധി മാത്രം മതി. ചെറുതും വലുതുമായ സകല പ്രാപഞ്ചിക പ്രതിഭാസങ്ങളും അവയ്ക്കു പിന്നിൽ ഒരു സ്രഷ്ടാവുണ്ടെന്ന സത്യം നമ്മോട് വിളിച്ച് പറയുന്നുണ്ട്. ചിന്തിക്കുന്ന ആർക്കും മനസ്സിലാക്കാവുന്ന സരളമായ ഒരു സത്യമാണ് ദൈവാസ്തിത്വമെന്ന് പറയുന്നത് അത് കൊണ്ടാണ്. എന്നാൽ സ്രഷ്ടാവ് എങ്ങനെയുളവനാണെന്നോ അവനോടുള്ള മനുഷ്യരുടെ ബന്ധമെങ്ങനെയുള്ളതാകണമെന്നോ മനസ്സിലാക്കാൻ കേവലബുദ്ധി മാത്രംമതിയാവുകയില്ല. പദാർത്ഥലോകത്തിന്റെ സ്രഷ്ടാവായ അല്ലാഹു പദാർഥാതീതനാകുമെന്നതു കൊണ്ട് തന്നെ അവനെക്കുറിച്ച് അറിയാൻ പ്രാപഞ്ചികപ്രതിഭാസങ്ങളെക്കുറിച്ച് മനസ്സിലാക്കാൻ ഉപയോഗിക്കുന്ന മാര്ഗങ്ങളൊന്നും പര്യാപ്തമാവുകയില്ല. അതിന് അവനിൽ നിന്നുള്ള അറിവ് തന്നെ നമുക്ക് ലഭിക്കണം.

അല്ലാഹുവിനെക്കുറിച്ചും ഇന്ദ്രിയങ്ങൾകൊണ്ടോ മസ്തിഷ്‌കം കൊണ്ടോ മനുഷ്യർക്ക് ചെന്നെത്താൻ കഴിയാത്ത മറ്റു മേഖലകളെക്കുറിച്ചുമുള്ള അറിവാണ് ആധ്യാത്മിക ജ്ഞാനം. ആത്യന്തികമായ ഈ അറിവിന്റെ സ്രോതസ്സായി ഇസ്‌ലാം അവതരിപ്പിക്കുന്നത് ഖുര്‍ആനിനെയും സുന്നത്തിനെയുമാണ്. ദൈവികമായ ഈ സ്രോതസ്സുകൾ പരീക്ഷണാത്മകമാണ്. ഖുര്‍ആനിനെ ഒരാള്‍ക്ക് പരിശോ ധിച്ച് അത് ദൈവികമാണെന്ന് ഉറപ്പുവരുത്താം. ഒരു സ്രോതസ്സിന്റെ ദൈവികത അളക്കുവാനായി ഉപയോഗിക്കാവുന്ന ഏതു മാനദണ്ഡം ഉപയോഗിച്ച് പരിശോധിച്ചാലും ഖുര്‍ആന്‍ ദൈവികമാണെന്ന വസ്തുത സുതരാം ബോധ്യമാകും. സുന്നത്തിന്റെ സ്ഥിതിയും ഇതുതന്നെയാണ്. മുഹമ്മദ് നബി (സ) പ്രവാചകനായിരുന്നോയെന്ന് ഏതൊരാള്‍ക്കും പരിശോധിച്ച് ഉറപ്പുവരുത്താന്‍ കഴിയും. അദ്ദേഹ ത്തിന്‍േറതായി ഉദ്ധരിക്കപ്പെടുന്ന വചനങ്ങളും ജീവിതചര്യയുമെല്ലാം അദ്ദേഹത്തില്‍നിന്നുള്ളതാണോയെന്ന് പരിശോധിച്ച് ഉറപ്പുവരുത്തുവാനും ശാസ്ത്രീയമായ മാര്‍ഗങ്ങളുണ്ട്. ഖുര്‍ആനും സുന്നത്തുമാകുന്ന ഇസ്‌ലാമിക പ്രമാണങ്ങള്‍ പരിശോധനാത്മകമാണെന്നര്‍ഥം. ഇവ പരിശോധിച്ച് ദൈവികമാണെന്ന് പൂര്‍ണബോധ്യം വരുത്തുകയും ശേഷം ആധ്യാത്മികജ്ഞാനത്തിന് ഇവയെ പ്രമാണങ്ങളായി സ്വീകരിക്കുകയും ചെയ്യാനാണ് ഇസ്‌ലാമിന്റെ അനുശാസന.

സ്രഷ്ടാവും നിയന്താവുമായ ഏകഉണ്‍മയെയാണ് ഇസ്‌ലാം അല്ലാഹു എന്ന് വിളിക്കുന്നത്. അല്ലാഹു അവന്റെ പ്രവര്‍ത്തനങ്ങളില്‍ അദ്വിതീയനാണെന്ന അറിവ് പ്രത്യക്ഷജ്ഞാനത്തിന്റെ പരിധിക്കുള്ളില്‍ വരുന്നതാണ്. എന്നാല്‍, അവന്റെ സത്തയുടെ സ്വഭാവമെന്തെന്ന് മനസ്സിലാക്കുന്നതെങ്ങനെ? പദാര്‍ഥലോകത്തിന്റെ സ്രഷ്ടാവായ അല്ലാഹു പദാര്‍ഥാതീതനാകണമെന്നത് സാമാന്യബുദ്ധിയുടെ വിധിയാണ്. പദാര്‍ഥാതീതമായ ഒരു സത്തയെ ഇന്ദ്രിയങ്ങള്‍ കൊണ്ട് അനുഭവിക്കുക അസാധ്യമാണ്.

ഇന്ദ്രിയങ്ങള്‍ നല്‍കുന്ന അറിവാണല്ലോ പ്രത്യക്ഷജ്ഞാനം. അതുമാത്രമാണ് മനുഷ്യന് നേടിയെടുക്കാന്‍ കഴിയുന്ന അറിവ്. ഇന്ദ്രിയങ്ങള്‍ നല്‍കുന്ന വിവരങ്ങള്‍ വെച്ച് ചിന്തിച്ച് സ്വന്തമായ നിഗമനങ്ങള്‍ നെയ്‌തെടുക്കുവാനുള്ള കഴിവാണ് മനുഷ്യനെ മൃഗങ്ങളില്‍ നിന്ന് വ്യതിരിക്തനും ഉല്‍കൃഷ്ടനുമാക്കുന്നത് ഇന്ദ്രിയക്ഷമതയുടെ കാര്യത്തില്‍ നാം മറ്റു ചില ജീവികളെക്കാളും പിന്നിലാണ്. നായയുടെ ഘ്രാണശക്തിയും പരുന്തിന്റെ കാഴ്ചശക്തിയും വവ്വാലിന്റെ ശ്രവണശേഷിയുമെല്ലാം മനുഷ്യരുടേതുമായി താരതമ്യം ചെ യ്യുമ്പോള്‍ വളരെ ഉയര്‍ന്നതാണ്. എന്നാല്‍, ഇവയ്‌ക്കൊന്നുംതന്നെ തങ്ങളുടെ ഇന്ദ്രിയങ്ങള്‍ നല്‍കുന്ന വിവരങ്ങളെ ജന്മനാലഭിച്ച കഴിവുകള്‍ക്കനുസരിച്ചല്ലാതെ ഉപയോഗിക്കുവാനോ ഉപഭോഗിക്കുവാനോ കഴിയില്ല. ഇവിടെയാണ് മനുഷ്യന്റെ ഉല്‍കൃഷ്ടത. അവന്റെ ചിന്താശേഷിയുടെ മഹത്വം. അവന് ഇന്ദ്രിയങ്ങള്‍ നല്‍കുന്ന നേരിട്ടുള്ള വിവരങ്ങളില്‍നിന്ന് കുറേയേറെ മുന്നിലേക്ക് കടന്ന് ചിന്തിക്കാന്‍ കഴിയും. പക്ഷേ, മനുഷ്യന്റെ ചിന്താശേഷിക്കും പരിമിതികളില്ലേ? ഉണ്ട്; തീര്‍ച്ചയായും ഉണ്ടാവണം. ഇന്ദ്രിയങ്ങള്‍ നല്‍കുന്ന അറിവുതന്നെയാണ് മനുഷ്യമസ്തിഷ്‌കത്തിന്റെ അസംസ്‌കൃത പദാര്‍ഥം. പദാര്‍ഥലോകമാണ് ഇന്ദ്രിയങ്ങളുടെ മേഖല. പദാര്‍ഥാതീതമായ ഒരു സത്തയെ സംബന്ധിച്ച് മനുഷ്യചിന്തക്ക് സ്വന്തമായി ഒരു തീരുമാനത്തിലോ നിഗമനത്തിലോ എത്തുക സാധ്യമല്ല. ഇതാണ് ചിന്തയുടെ പരിമിതി. അല്ലാഹുവിന്റെ സത്തയെ സംബന്ധിച്ച് ചിന്തിച്ച് സ്വന്തമായ ഒരു നിഗമനത്തിലെത്താന്‍ മനുഷ്യന് കഴിയില്ലെന്ന് പറയുന്നത് അതുകൊണ്ടാണ്.

അതുകൊണ്ടുതന്നെ ഇസ്‌ലാമിനെ സംബന്ധിച്ചിടത്തോളം അല്ലാഹുവെക്കുറിച്ച ഏതുതരം അറിവിന്റെയും സ്രോതസ്സ് ഖുര്‍ആനും സുന്നത്തുമാണ്. അവ ദൈവികമാണെന്ന് പൂര്‍ണബോധ്യമുള്ള തിനാല്‍ സ്രഷ്ടാവിനെ സംബന്ധിച്ച തെറ്റുപറ്റാത്ത അറിവ് നല്‍കുന്ന പ്രമാണങ്ങളായി അവയെയാണ് മുസ്‌ലിംകള്‍ സ്വീകരിക്കുന്നത്. അല്ലാഹുവിനെ സംബന്ധിച്ച് ഖുര്‍ആനും സുന്നത്തും പറഞ്ഞുതരു ന്നതെന്താണോ അത് അപ്പടി സ്വീകരിക്കുകയും അതിനെ മാനുഷിക തലത്തിലേക്ക് കൊണ്ടുവന്ന് വ്യാഖ്യാനങ്ങള്‍ക്ക് വിധേയമാക്കാതിരിക്കുകയും ചെയ്യുകയാണ് മുസ്‌ലിമിന്റെ രീതി. സര്‍വശക്തനെക്കു റിച്ച് അവന്‍ സ്വയം പറഞ്ഞതെന്തോ അതും അവന്റെ ബോധനപ്രകാരം സംസാരിക്കുകയും ജീവിക്കുകയും ചെയ്ത പ്രവാചകന്‍ വിശദീകരിച്ചതെന്തോ അതും അപ്പടി സ്വീകരിച്ചുകൊണ്ട് അവനെ അറിയുകയല്ലാതെ മറ്റൊരു മാര്‍ഗവും മനുഷ്യര്‍ക്കു മുന്നിലില്ല. അതല്ലാത്ത മാര്‍ഗങ്ങളെല്ലാം ഊഹാധിഷ്ഠിതമാണ്; അതുകൊണ്ടുതന്നെ തെറ്റുപറ്റാന്‍ ഏറെ സാധ്യതയുള്ളതുമാണ്.

അല്ല. പ്രപഞ്ചത്തിലെ വലുതും ചെറുതുമായ സകലവിധ പ്രതിഭാസങ്ങ ളുടെയും സൃഷ്ടിക്കും സ്ഥിതിക്കും സംഹാരത്തിനും കാരണഭൂതമായിരിക്കുന്ന ഏകാസ്തിത്വത്തിന് അറബിയില്‍ പറയുന്ന പേരാണ് അല്ലാഹു. സ്വന്തത്തെയും തന്റെ ചുറ്റുപാടുകളെയും കുറിച്ച് ഗൗരവതരമായി പഠിച്ചുമനസ്സിലാക്കുന്ന ഒരാള്‍ക്കും അവയ്ക്കു പിന്നിലുള്ള ഒരു മഹാചൈതന്യത്തിന്റെ അസ്തിത്വത്തെ നിഷേധിക്കുവാനാവില്ലെന്നതാണ് വസ്തുത. പരമാണുവിനുള്ളിലെ ആന്ദോളനങ്ങളുടെ മുതല്‍ താരാസമൂഹങ്ങളുടെ നിലനില്‍പിന്നാധാരമായ വ്യവസ്ഥകളുടെ വരെ കാരണം അല്ലാഹുവാണ്. എന്നാല്‍, അല്ലാഹു കേവലമായ ഒരു ശക്തിയോ അന്ധമായ ഒരു കാരണമോ ഉണ്മയില്ലാത്ത ഒരു ഊര്‍ജരൂപമോ മാത്രമാണെന്ന് ഖുര്‍ആന്‍ പഠിപ്പിക്കുന്നില്ല. ദൈവിക സത്ത മനുഷ്യന്റെ വിശദീകരണത്തിന്നതീതമാണ്. അല്ലാഹുതന്നെ അറിയിച്ചുതന്ന ദിവ്യവെളിപാടുകളിലൂടെ മാത്രമേ അവന്റെ സത്തയുടെ സ്വഭാവമെന്താണെന്ന് നമുക്ക് മനസ്സിലാക്കാനാവൂ. എങ്കിലും അവന്റെ ശക്തിയാല്‍ നിലനില്‍ക്കുന്ന പ്രാപഞ്ചികവസ്തുക്കള്‍ അവന്റെ അസ്തിത്വത്തെയും സ്വഭാവങ്ങളെയും ഒരു പരിധിവരെ വെളിപ്പെടുത്തുന്നുവെന്ന് പറയാം.

മലയാളത്തില്‍ 'ദൈവ'മെന്നും ഇംഗ്ലീഷില്‍ 'ഗോഡ്' എന്നും ഉറുദുവില്‍ 'ഖുദാ' എന്നും പറയുമ്പോഴുണ്ടാകുന്ന അര്‍ഥകല്‍പനയിലുള്ള അറബിപദമല്ല 'അല്ലാഹു'വെന്ന കാര്യം പ്രത്യേകം ശ്രദ്ധേയമാണ്. ദൈവം, ഗോഡ്, ഖുദാ തുടങ്ങിയ പദങ്ങള്‍ക്ക് ആരാധിക്കപ്പെടുന്നത് എന്നു മാത്രമേ അര്‍ഥമുള്ളൂ. ഇവയ്ക്ക് തത്തുല്യമായ അറബിപദമാണ് 'ഇലാഹ്'. ആരാധിക്കപ്പെടുന്ന ആരെയും 'ഇലാഹുകള്‍' എന്ന് വിളിക്കാം. ഇലാഹ് എന്ന പൊതുനാമത്തെ 'അല്‍' എന്ന അവ്യയം ചേര്‍ത്ത് വിശേഷവത്കരിച്ചതാണ് 'അല്ലാഹു' എന്ന പദമെന്നാണ് പദോല്‍പത്തിശാസ്ത്രജ്ഞന്മാരുടെ പക്ഷം. അപ്പോള്‍ അല്ലാഹു എന്ന പദത്തിനര്‍ഥം അല്‍ഇലാഹ് എന്നാണ്. 'ആരാധിക്കപ്പെടുവാന്‍ യഥാര്‍ഥത്തില്‍ അര്‍ഹതയുള്ളവന്‍' എന്നോ 'സാക്ഷാല്‍ ദൈവം' എന്നോ ഇതിന് അര്‍ഥം പറയാവുന്നതാണ്. 'അല്ലാഹു'വെന്ന പദത്തിന്റെ ഉല്‍പത്തിയെ സംബന്ധിച്ച് ഇതല്ലാത്ത അഭിപ്രായങ്ങളുമുണ്ട്. ഏതായിരുന്നാലും യഥാര്‍ഥ ദൈവത്തിനുള്ള സംജ്ഞാനാമമെന്ന നിലയ്ക്കാണ് ഇന്ന് അതുപയോഗിക്കപ്പെടുന്നത്. സത്യദൈവം എന്ന നിലയ്ക്കാണ് വിശുദ്ധ ഖുര്‍ആന്‍ 'അല്ലാഹു'വെന്ന പദത്തെ പരിചയപ്പെടുത്തുന്നത്.

സൂക്ഷ്മവും സ്ഥൂലവുമായ സകലമാന വസ്തുക്കളുടെയും സ്രഷ്ടാവും സംരക്ഷകനും നിയന്താവും സര്‍വശക്തനും സര്‍വജ്ഞനുമായവനാണ് അല്ലാഹു. ഇത് ഒരു കുലദൈവത്തിന്റെയോ ഗോത്രദൈവത്തിന്റെയോ പേരല്ല. എല്ലാ ദേശക്കാരുടെയും ഭാഷക്കാരുടെയും വര്‍ഗക്കാരുടെയും യഥാര്‍ഥമായ ദൈവത്തിന്റെ പേരാണ് അല്ലാഹു.നിരവധി ദൈവനാമങ്ങൾ ഖുർആനിലുണ്ട്. അത്യുന്നതനായ അവന്റെ ഗുണവിശേഷങ്ങളെ കുറിക്കുന്നവയാണ് ആ നാമങ്ങളെല്ലാം. അല്ലാഹുവെന്ന നാമത്തെ ഖുർആൻ പരിചയപ്പെടുത്തുന്നത് അവന്റെ യഥാർത്ഥ നാമമായിക്കൊണ്ടാണ്.

സ്രഷ്ടാവും പ്രപഞ്ചകര്‍ത്താവുമായ ദൈവത്തിന് വ്യത്യസ്തഭാഷകളില്‍ വ്യത്യസ്ത പേരുകളുപയോഗിച്ചതായി കാണാന്‍ കഴിയും. സംസ്‌കൃതത്തില്‍ 'പരബ്രഹ്മം' എന്നും 'പരമാത്മാവ്' എന്നുമാണ് സര്‍വശക്തനും സര്‍വലോകസ്രഷ്ടാവുമായ ദൈവത്തിന് പറയുന്ന പേരെന്ന് ഉപനിഷത്തുകളിലെ വിവരണങ്ങള്‍ വായിച്ചാല്‍ ബോധ്യമാവും. ബൈബിള്‍ പഴയ നിയമത്തില്‍ പല പേരുകളിലും സ്രഷ്ടാവായ ദൈവം അഭിസംബോധന ചെയ്യപ്പെട്ടിട്ടുണ്ട്. ഏല്‍, എലോഹിം, ഏല്‍ശദ്ദായി, ഏല്‍എല്യോന്‍, അഡോണായി, യാഹ്, യഹ്‌വെ (യഹോവ) തുടങ്ങിയ നാമങ്ങളെല്ലാം സര്‍വശക്തനായ ദൈവത്തിന് നല്‍കപ്പെട്ടിട്ടുണ്ട്. യേശുവിന്റെ ഉപദേശങ്ങളൊന്നും അദ്ദേഹത്തിന്റെ ഭാഷയായ അരമായഭാഷയില്‍ ലഭ്യമല്ലാത്തതിനാല്‍ അദ്ദേഹം എങ്ങനെയാണ് ദൈവത്തെ സംബോധന ചെയ്തതെ ന്ന് മനസ്സിലാക്കാന്‍ നിര്‍വാഹമില്ല. എങ്കിലും 'എലോഹി' എന്നായിരിക്കും അദ്ദേഹം ദൈവത്തെ സംബോധന ചെയ്തതെന്നാണ് ബൈബിള്‍ പണ്ഡിതന്മാരുടെ അഭിപ്രായം. ഇന്ന് നിലവിലുള്ള അറബി ബൈബിളുകളിലെല്ലാം അത്യുന്നതനായ ദൈവത്തെ വിളിച്ചിരിക്കുന്നതും അല്ലാഹു എന്നു തന്നെയാണ്.

സര്‍വലോകസ്രഷ്ടാവും സര്‍വശക്തനുമായ ദൈവത്തെ വിളിക്കുവാന്‍ ഖുര്‍ആന്‍ ഉപയോഗിച്ച പദമാണ് 'അല്ലാഹു'വെന്നും 'ആരാധിക്കാന്‍ യഥാര്‍ഥത്തില്‍ അര്‍ഹതയുള്ളവന്‍' എന്നാണ് ആ നാമത്തിന് അര്‍ഥമെന്നും പറഞ്ഞുവല്ലോ.'അല്ലാഹുവെന്ന പദത്തിന് തുല്യമായി മറ്റു ഭാഷകളിലൊന്നുംതന്നെ ഒറ്റവാക്കുകളില്ലെന്ന് പറയുന്ന പണ്ഡിതന്മാരുണ്ട്. അതെന്തായിരുന്നാലും, 'യഥാര്‍ഥത്തിലുള്ള ദൈവം' എന്ന അര്‍ഥത്തിലാണ്, ഏതെങ്കിലും ഒരു പ്രത്യേക വിഭാഗത്തിന്റെയോ ഗോത്രത്തിന്റെയോ ദൈവം എന്ന അര്‍ഥത്തിലല്ല ഖുര്‍ആന്‍ 'അല്ലാഹു'വെന്ന് പ്രയോഗിച്ചിരിക്കുന്നത്. മുസ്‌ലിംകളുടെയും ഹിന്ദുക്കളുടെയും ക്രിസ്ത്യാനികളുടെയും യഹൂദരുടെയും പാര്‍സികളുടെയും മറ്റെല്ലാ ജനവിഭാഗങ്ങളുടെയും കറുത്തവരുടെയും വെളുത്തവരുടെയും പാശ്ചാത്യരുടെയും പൗരസ്ത്യരുടെയും അവര്‍ണരുടെയും സവര്‍ണരുടെയും അഖിലചരാചരങ്ങളുടെയും എല്ലാം സ്രഷ്ടാവും സംരക്ഷകനും പരിപാലകനുമാണ് ഖുര്‍ആന്‍ പരിചയപ്പെടുത്തുന്ന അല്ലാഹു.

''അല്ലാഹു എല്ലാ വസ്തുക്കളുടെയും സ്രഷ്ടാവാകുന്നു. അവന്‍ എല്ലാ വസ്തുക്കളുടെ മേലും കൈകാര്യകര്‍ത്താവാകുന്നു'' (39:62).

''അവനാകുന്നു നിങ്ങളുടെ രക്ഷിതാവും എല്ലാ വസ്തുക്കളുടെ സൃഷ്ടികര്‍ത്താവുമായ അല്ലാഹു. അവനല്ലാതെ യാതൊരു ദൈവവുമില്ല. എന്നിരിക്കെ നിങ്ങള്‍ എങ്ങനെയാണ് (സന്മാര്‍ഗത്തില്‍ നിന്ന്) തെറ്റിക്കപ്പെടുന്നത്?'' (40:62).

''തീര്‍ച്ചയായും അല്ലാഹുതന്നെയാണ് എന്റെ രക്ഷിതാവും നിങ്ങളുടെ രക്ഷിതാവും. അതിനാല്‍ അവനെ നിങ്ങള്‍ ആരാധിക്കുക. ഇതാകുന്നു നേരായ പാത'' (43:64).

''ആകാശങ്ങളുടെയും ഭൂമിയുടെയും നിര്‍മാതാവാണവന്‍. അവന്ന് എങ്ങനെ ഒരു സന്താനമുണ്ടാകും? അവന്നൊരു കൂട്ടുകാരിയുമില്ലല്ലോ. എല്ലാ വസ്തുക്കളെയും അവന്‍ സൃഷ്ടിച്ചതാണ്. അവന്‍ എല്ലാ കാര്യത്തെപ്പറ്റിയും അറിയുന്നവനുമാണ്. അങ്ങനെയുള്ളവനാണ് നിങ്ങളുടെ രക്ഷിതാവായ അല്ലാഹു. അവനല്ലാതെ ഒരു ദൈവവുമില്ല. എല്ലാ വസ്തുക്കളുടെയും സ്രഷ്ടാവാണവന്‍. അതിനാല്‍ അവനെ നിങ്ങള്‍ ആരാധിക്കുക. അവന്‍ സകല കാര്യങ്ങളുടെയും കൈകാര്യക്കാരനാകുന്നു'' (6:101,102).

വിഷയവുമായി ബന്ധപ്പെട്ട വീഡിയോ

സൃഷ്ടിയാണ് സ്രഷ്ടാവിനുള്ള ഒന്നാമത്തെ തെളിവ്. പ്രപഞ്ചത്തിലെ പ്രതിഭാസങ്ങളെല്ലാം അവയ്ക്കു പിന്നില്‍ വ്യക്തമായ ആസൂത്രണം നടന്നിട്ടുണ്ടെന്ന് വ്യക്തമാക്കുന്നുണ്ട്. യാദൃച്ഛികമായി അവയൊന്നും ഉണ്ടാവുകയില്ലെന്ന് വ്യക്തം. സൃഷ്ടികള്‍ സ്രഷ്ടാവിന്റെ അസ്തിത്വവും വൈഭവവും വെളിപ്പെടുത്തുന്നുവെന്നതാണ് വസ്തുത. ജീവന്‍ നിലനില്‍ക്കുന്ന ഒരേയൊരു പ്രദേശമായ ഭൂമിയിലെ അനുകൂലനങ്ങളെക്കുറിച്ചുമാത്രം പഠിച്ചാല്‍മതി. പ്രപഞ്ചസൃഷ്ടിയിലെ ആസൂത്രണം എത്രമാത്രം സൂക്ഷ്മവും കൃത്യവുമാണെന്ന് മനസ്സിലാക്കുവാന്‍. നാലുകാര്യങ്ങളെങ്കിലും പൂര്‍ണമായുണ്ടാവുമ്പോള്‍ മാത്രമാണ് ഒരു പ്രപഞ്ചപ്രദേശത്തില്‍ ജീവന്‍ നിലനില്‍ക്കുക. ആവശ്യമായ താപം, ലഭ്യമായ ജലം, കൃത്യമായ വാതകമിശ്രിതം, നിലനില്‍ക്കാനാവുന്ന അന്തരീക്ഷം എന്നിവയാണവ. ജീവന്‍ നിലനിര്‍ത്തുന്ന രീതിയില്‍ ഇവ നാലും ഒത്തുവരുന്നത് ഭൂമിയില്‍ മാത്രമാണ്. ജീവനുവേണ്ടി ഭൂമിയെ പ്രത്യേകം സജ്ജമാക്കിയതാണെന്നര്‍ത്ഥം.

സൂര്യനില്‍നിന്ന് ഭൂമിയിലേക്കുള്ള 1496 ലക്ഷം കിലോമീറ്റര്‍ ദൂരം, ഭൂമിയില്‍ മാത്രം നിലനില്‍ക്കുന്ന ഹരിതഗേഹ പ്രഭാവം (Green house effect)എന്നിവയാണ് ഭൂമിയുടെ താപം -500 സെല്‍ഷ്യസിനും +500 സെല്‍ഷ്യസിനുമിടയില്‍ നിലനിര്‍ത്തുന്നത്. ജീവന്‍ നിലനില്‍ക്കാനാവശ്യമായ താപനിലയുള്ള പ്രദേശത്തെയാണ് ഗോള്‍ഡിലോക്‌സ് സ്ഥലം (goldilocks zone) അഥവാ വാസയോഗ്യസ്ഥലം (habitable zone) എന്നുപറയുന്നത്. ആവശ്യമായ കാര്‍ബണ്‍-നീരാവി സാന്നിധ്യമുള്ള അന്തരീക്ഷമുള്ളതുകൊണ്ടാണ് ഭൂമിയില്‍ ജീവന്‍ നിലനില്‍ക്കുന്നത്. പ്രസ്തുത സാന്നിധ്യത്തിന് നക്ഷത്രത്തില്‍നിന്ന് ഗ്രഹത്തിലേക്കുള്ള ദൂരം കൃത്യമാവുകയും ഒപ്പം തന്നെ അത് നിലനില്‍ക്കാനാവശ്യമായ ഗ്രഹസാഹചര്യമുണ്ടാവുകയും വേണം. ഭൂമിയും സൂര്യനും തമ്മില്‍ നിലനില്‍ക്കുന്ന ദൂരം കുറയുകയോ കൂടുകയോ ചെയ്താല്‍ ഇവിടെ ജീവനുണ്ടാവുകയില്ല. ഹരിതഗേഹപ്രഭാവമില്ലെങ്കില്‍ ജീവന്‍ നിലനില്‍ക്കുകയുമില്ല. ഭൂമിയിലെ ഹരിതഗേഹപ്രഭാവം നിലനിര്‍ത്തുന്നത് സസ്യങ്ങളാണ്. പ്രകാശസംശ്ലേഷണസമയത്ത് ഓക്‌സിജനും നീരാവിയും പുറത്തുവിടുന്നതുകൊണ്ടാണ് ഈ പ്രതിഭാസം നിലനില്‍ക്കുന്നത്. ആവശ്യമായ കാര്‍ബണ്‍-ജല അനുപാതം നിലനിന്നിട്ടില്ലെങ്കില്‍ ഭൂമി -73 ഡിഗ്രിയിലുള്ള തണുത്തുറച്ച ഗ്രഹമാകുമായിരുന്നു. ഈ അനുപാതം നിലനിര്‍ത്തുന്നത് സസ്യങ്ങളാണ്. സസ്യങ്ങള്‍ക്ക് നിലനില്‍ക്കാനാവശ്യമായ കാര്‍ബണ്‍ ഡയോക്‌സൈഡ് നിര്‍മിക്കുന്നത് ജന്തു പ്രവര്‍ത്തനങ്ങളാണ്. ഭൂമിയില്ലെങ്കില്‍ സസ്യങ്ങളില്ല! സസ്യങ്ങളില്ലെങ്കില്‍ ഭൂമിയില്ല! ജന്തുക്കളില്ലെങ്കില്‍ സസ്യങ്ങളില്ല! സസ്യങ്ങളും ഭൂമിയുമില്ലെങ്കില്‍ ജന്തുക്കളുമില്ല! എന്തൊരു പാരസ്പര്യം! ഈ പാരസ്പര്യം യാദൃച്ഛികമാവുമോ?

ജീവന്‍ നിലനില്‍ക്കണമെങ്കില്‍ ജലസാന്നിധ്യമുണ്ടാകണം. ഭൂമിയിലെ ജലസാന്നിധ്യം സ്ഥിരമാക്കുന്നതില്‍ ജന്തുക്കള്‍ക്കും സസ്യങ്ങള്‍ക്കും പങ്കുണ്ട്. ഭൗമാന്തരീക്ഷത്തില്‍ 78 ശതമാനം നൈട്രജനും 21 ശതമാനം ഒക്‌സിജനും 0.03 ശതമാനം കാര്‍ബണ്‍ഡയോക്‌സൈഡുമാണുള്ളത്. ഈ അനുപാതം നിലനിന്നാലേ ജീവനുണ്ടാവൂ. അത് നിലനിര്‍ത്തുന്നതില്‍ സസ്യങ്ങള്‍ക്കും ജീവികള്‍ക്കും പങ്കുണ്ട്. ജീവനും അതുനിലനില്‍ക്കുവാനുള്ള സംവിധാനങ്ങളുമെല്ലാം കൃത്യവും സൂക്ഷ്മവുമായി സംരക്ഷിക്കപ്പെടണമെങ്കില്‍ അതിനാവശ്യമായ അന്തരീക്ഷമുണ്ടാവണം. എണ്‍പതു മുതല്‍ നൂറു വരെ കിലോമീറ്റര്‍ കനത്തില്‍ ഭൂമിയെ ആവരണം ചെയ്യുന്ന വാതകങ്ങളുടെ മതിലാണ് അന്തരീക്ഷം. അപകടകരമായ പ്രപഞ്ചരശ്മികളില്‍ നിന്നും വസ്തുക്കളില്‍ നിന്നും ഭൂമിയെ രക്ഷപെടുത്താന്‍ കഴിയുന്ന അന്തരീക്ഷമുള്ളതുകൊണ്ടാണ് ഇവിടെ ജീവന്‍ നിലനില്‍ക്കുന്നത്. അന്തരീക്ഷത്തിലെ ഓക്‌സിജന്റെ അളവ് 21 ശതമാനത്തില്‍ കുറവാണെങ്കില്‍ ജന്തുക്കള്‍ക്ക് ജീവിക്കാനാവുകയില്ല. 25 ശതമാനത്തില്‍ കൂടിയാല്‍ സസ്യങ്ങളെല്ലാം കത്തി ചാമ്പലാവുകയും ചെയ്യും.

ഭൂമി, 23.5 ഡിഗ്രി ചരിഞ്ഞ് (സാങ്കല്‍പിക അച്ചുതണ്ടില്‍), സ്വയം ഭ്രമണം നടത്തുന്നതുകൊണ്ടാണ് നമ്മുടെ ഭൂഖണ്ഡങ്ങള്‍ തണുത്തുറച്ച് ഹിമഗ്രഹമാകാതിരിക്കുന്നത്. ഒരു മണിക്കൂറില്‍ ആയിരത്തി അറുന്നൂറ് കിലോമീറ്റര്‍ വേഗതയിലുള്ള സ്വംയഭ്രമണം വഴിയാണ് ഓരോ ഭൂഖണ്ഡങ്ങളിലും ആവശ്യമായ തോതിലുള്ള സൂര്യപ്രകാശത്തിന്റെ വിതരണം നടക്കുന്നത്. അത് അല്‍പം കുറയുകയോ കൂടുകയോ ചെയ്താല്‍ ജീവന്‍ സാധ്യമാവുമായിരുന്നില്ല. സൗരോപരിതലത്തിന്റെ താപമായ 5778 ഡിഗ്രിയാണ് ഭൂമിക്ക് ആവശ്യമായ ചൂട് നല്‍കുന്നത്. ഇത് കൂടുകയോ കുറയുകയോ ചെയ്താല്‍ ഭൂമിയില്‍ ജീവനുണ്ടാകുമായിരുന്നില്ല. ചന്ദ്രനും ഭൂമിയും തമ്മിലുള്ള ദൂരവും ചന്ദ്രപ്രതിഭാസങ്ങളുമെല്ലാം ജീവന്റെ സാധ്യതയെ സ്വാധീനിക്കുന്ന കാര്യങ്ങളാണ്. ചന്ദ്രന്‍ ഇന്നുള്ള 384400 കിലോമീറ്ററിനെക്കാള്‍ അടുത്തായിരുന്നു നിലനിന്നിരുന്നതെങ്കില്‍ സ്ഥിരമായ വേലിയേറ്റം കാരണം ഭൂമിയില്‍ ജീവന്‍ അസാധ്യമാകുമായിരുന്നു. ഭൂമിയുടെ മൂന്നില്‍രണ്ടു ഭാഗം സമുദ്രമായതും അവയുടെ ആഴവുമെല്ലാം ഇവിടുത്തെ ജീവന്‍ നിലനില്‍ക്കുന്നതിന് കാരണമായി ഭവിക്കുന്ന അനുകൂലനങ്ങളാണ്. ജീവന്‍ നിലനില്‍ക്കാന്‍ പാകത്തില്‍ ഭൂമിയെ ഒരുക്കിയവന്റെ ആസൂത്രണമല്ലാതെ മറ്റെന്താണ് ഇവിടെയെല്ലാം കാണാനാവുന്നത്! ക്വുര്‍ആന്‍ പറഞ്ഞതെത്ര ശരി!

''ഭൂമിയാകട്ടെ നാം അതിനെ വികസിപ്പിക്കുകയും, അതില്‍ ഉറച്ചുനില്‍ക്കുന്ന പര്‍വ്വതങ്ങള്‍ നാം സ്ഥാപിക്കുകയും കൗതുകമുള്ള എല്ലാ സസ്യവര്‍ഗങ്ങളും നാം അതില്‍ മുളപ്പിക്കുകയും ചെയ്തിരിക്കുന്നു. (സത്യത്തിലേക്ക്) മടങ്ങുന്ന ഏതൊരു ദാസനും കണ്ടുമനസ്സിലാക്കുവാനും അനുസ്മരിക്കുവാനും വേണ്ടി.'' (50:7,8)

സൃഷ്ടിപ്രപഞ്ചത്തിലെ സൂക്ഷ്മതയും ആസൂത്രണവും എല്ലാപ്രതിഭാസങ്ങള്‍ക്കും പിന്നിലുള്ള സര്‍ഗധനനായ സൃഷ്ടാവിനെ വെളിപ്പെടുത്തുന്നുണ്ട്. സ്വന്തത്തെയും ചുറ്റുപാടിനെയും കണ്ണും കാതും തുറന്നുവെച്ച് നിരീക്ഷിക്കുകയും അവയ്ക്ക് പിന്നിലുള്ള സര്‍ഗവൈഭവത്തെപ്പറ്റി ചിന്തിക്കുകയും ചെയ്യുന്നവര്‍ക്കെല്ലാം, അവര്‍ കളങ്കരഹിതമായ മനസ്സിന് ഉടമകളാണെങ്കില്‍ സര്‍വജ്ഞനായ പടച്ചവന്റെ അസ്തിത്വം ബോധ്യപ്പെടുക തന്നെ ചെയ്യും.

നിസ്തുലമായ സ്വന്തം ശരീരത്തെയും തനിക്ക് നല്‍കപ്പെട്ട അത്യത്ഭുതകരമായ സവിശേഷതകളെയും പറ്റി ചിന്തിച്ചാല്‍-

ജൈവലോകത്തെ ഉല്‍പാദകരായ സസ്യങ്ങളെയും ആഹാരനിര്‍മിതിക്കു വേണ്ടിയുള്ള സസ്യശരീരത്തിലെ അനുകൂലനങ്ങളെയുംകുറിച്ചു പഠിച്ചാല്‍-

കരയുടെ ഇരട്ടിവരുന്ന സമുദ്രത്തിലെ ജൈവവൈവിധ്യത്തെയും ആഴിയിലും ആഴത്തിലും ജീവിക്കുവാനായി അവയ്ക്കുനല്‍കിയ സവിശേഷതകളെയുംപറ്റി മനസ്സിലാക്കിയാല്‍-

വിമാനനിര്‍മാണത്തിന് മനുഷ്യരെ പഠിപ്പിച്ച പറവകളിലെ പറക്കാനുള്ള കഴിവുകളെയും അവ സംവിധാനിച്ച രീതിയെയും കുറിച്ച് അറിയാന്‍ ശ്രമിച്ചാല്‍-

തേനീച്ചയെപ്പോലെയുള്ള ചെറുപ്രാണികള്‍ക്കുപോലും നല്‍കപ്പെട്ട അനുകൂലനങ്ങളെയും പ്രകൃതിയില്‍ അവ നിര്‍വഹിക്കുന്ന ധര്‍മങ്ങളെയും അപഗ്രഥിച്ചാല്‍-

പരിസ്ഥിതി സന്തുലനത്തിനായി ഓരോ ജീവിയും നിര്‍വഹിക്കുന്ന ദൗത്യങ്ങളെയും അവയ്ക്കനുസൃതമായി അവയ്ക്ക് ലഭിച്ച ജന്മവാസനകളെയും പറ്റി ചിന്തിച്ചാല്‍-

ജൈവലോകത്ത് നിലനില്‍ക്കുന്ന പാരസ്പര്യത്തെയും പ്രസ്തുത പാരസ്പര്യം വഴി നിലനില്‍ക്കുന്ന പരിസ്ഥിതി സംതുലനത്തെയും വ്യക്തമായി അറിഞ്ഞാല്‍-

ജീവനുവേണ്ടി സൃഷ്ടിക്കപ്പെട്ട ഒരേയൊരു ഭൂമിയെയും അതിനുവേണ്ടി മാത്രമായി നല്‍കപ്പെട്ട സവിശേഷതകളെയും അപഗ്രഥിക്കാന്‍ സന്നദ്ധമായാല്‍-

അറിയുന്തോറും അറിയാത്തതാണ് കൂടുതലെന്ന് ബോധ്യപ്പെടുത്തുന്ന പ്രപഞ്ചത്തെയും മനുഷ്യബുദ്ധിക്ക് ചെന്നെത്താനാകാത്ത പ്രഹേളികയായി പ്രപഞ്ചം തുടരുമെന്ന അത്ഭുതം പ്രസ്താവിക്കുന്ന പ്രപഞ്ചപഠന സിദ്ധാന്തങ്ങളെയും മനസ്സിലാക്കാന്‍ ശ്രമിച്ചാല്‍-

ആര്‍ക്കാണ് ഈ മഹാസംവിധാനങ്ങള്‍ക്കെല്ലാം പിന്നിലുള്ള സര്‍വശക്തനായ സ്രഷ്ടാവിനെ നിഷേധിക്കാന്‍ കഴിയുക!?

വിഷയവുമായി ബന്ധപ്പെട്ട വീഡിയോ

ഒരു പോരാട്ടത്തിൽ മുസ്ലിമിന് പങ്കെടുക്കാൻ പറ്റുമോയെന്ന് തീരുമാനിക്കുന്നത് അത് എന്തിനു വേണ്ടി എങ്ങനെ ചെയ്യുന്നുവന്നതിന്റെ അടിസ്‌ഥാനത്തിലാണ്. ലക്ഷ്യവും മാർഗവുമെല്ലാം ഇസ്ലാമാമികമാവുമ്പോൾ മാത്രമേ എതൊരു പോരാട്ടവും ഇസ്‌ലാമികമാവൂ. ഇസ്‌ലാമിക സമൂഹത്തിന്റെ നേതൃത്വത്തിന് കീഴിൽ ഇസ്‌ലാമികമായ ലക്ഷ്യത്തിനു വേണ്ടി നടത്തുന്നതാണ് അല്ലാഹുവിന്റെ മാർഗത്തിലുള്ള ധര്മസമരം. രഹസ്യകേന്ദ്രങ്ങളിലിരുന്ന് ആരെങ്കിലും ആഹ്വാനം ചെയ്യുകയും രാഷ്ട്രീയമായ ലക്ഷ്യത്തോടുകൂടി നിയതമായ ക്രമങ്ങളൊന്നുമില്ലാതെ നടക്കുന്ന കലാപമല്ല അത്. അത്തരം കലാപങ്ങളിൽ പങ്കെടുക്കുന്നത് ഇസ്‌ലാമികമായി ന്യായീകരിക്കുവാൻ കഴിയില്ല. സമരാര്‍ജിത സ്വത്തിനോ ധൈര്യശാലിയെന്ന് അറിയപ്പെടുന്നതിനോ ലോകമാന്യത്തിനോ വേണ്ടി ചെയ്യപ്പെടുന്ന സായുധ സമരങ്ങളൊന്നും അല്ലാഹുവിന്റെ മാര്‍ഗത്തിലുള്ളവയല്ലെന്നും അല്ലാഹുവിന്റെ വചനം അത്യുന്നതമായിത്തീരുന്നതിനുവേണ്ടിയുള്ള പോരാട്ടം മാത്രമാണ് ദൈവമാര്‍ഗത്തില്‍ സ്വീകാര്യമായിത്തീരുകയെന്നും(സ്വഹീഹു മുസ്‌ലിം, കിതാബുല്‍ ജിഹാദ് വസ്‌സൈര്‍) വര്‍ഗീയതക്കോ വംശീയതക്കോ വേണ്ടി സമരം ചെയ്യുന്നവന്റെ മരണം ജാഹിലിയ്യത്തിന്റേതാണെന്നും(സ്വഹീഹു മുസ്‌ലിം, കിതാബുല്‍ ജിഹാദ് വസ്‌സൈര്‍) പഠിപ്പിക്കപ്പെട്ടവരാണ് മുസ്‌ലിംകള്‍. സ്വര്‍ഗം ആഗ്രഹിച്ചുകൊണ്ട് ആയുധമെടുക്കുന്നവര്‍ തങ്ങള്‍ അല്ലാഹുവിന്റെ മാര്‍ഗത്തില്‍ തന്നെയാണോയെന്ന് ശരിക്കും ആലോചിച്ച ശേഷമാകണം രണാങ്കണത്തില്‍ ഇറങ്ങേണ്ടത് എന്ന് പഠിപ്പിക്കുന്ന ഇസ്‌ലാമിക പ്രമാണങ്ങളില്‍നിന്ന് മതം പഠിക്കുന്നവര്‍ രാഷ്ട്രീയവും സാമുദായികവുമായ താല്‍പര്യങ്ങള്‍ സംരക്ഷിക്കുന്നതിനായി സംഘടിപ്പിക്കപ്പെടുന്ന ഭീകരവാദികളുടെ കൂട്ടായ്മകള്‍ക്കകത്ത് അംഗങ്ങളായിത്തീരുകയില്ലെന്നുറപ്പാണ്.

ആദര്‍ശത്തിനുവേണ്ടി നടത്തപ്പെടുന്ന യുദ്ധത്തില്‍ ഇസ്‌ലാമിനെതിരെ യുദ്ധം ചെയ്യുന്ന 'അവിശ്വാസിയും അയാളെ വധിക്കുന്ന വിശ്വാസിയും ഒരിക്കലും നരകത്തില്‍ ഒരുമിക്കുകയില്ല'(സ്വഹീഹു മുസ്‌ലിം, കിതാബുല്‍ ഇമാറ)യെന്ന് പഠിപ്പിച്ച പ്രവാചക(സ)നില്‍ലനിന്ന് മതം പഠിച്ചവര്‍ക്ക് സ്വര്‍ഗത്തിലെത്തിക്കുമെന്ന് ഉറപ്പുള്ള ലക്ഷ്യത്തിനുവേണ്ടിയല്ലാതെ ആയുധമെടുക്കുവാന്‍ കഴിയില്ല. ഭരണാധികാരികളുടെ നേതൃത്വത്തിലല്ലാതെയുള്ള സായുധ സമരങ്ങള്‍ അരാജകത്വവും കുഴപ്പങ്ങളും സൃഷ്ടിക്കുവാന്‍ മാത്രമാണ് നിമിത്തമാവുകയെന്നതുകൊണ്ടുതന്നെ അത്തരം മുന്നേറ്റങ്ങളില്‍നിന്ന് ഒഴിഞ്ഞുനില്‍ക്കണമെന്ന് പഠിപ്പിക്കപ്പെട്ടവര്‍ക്ക് എങ്ങനെയാണ് കലാപങ്ങളിലൂടെ രാഷ്ട്രീയലക്ഷ്യം നേടാന്‍ ശ്രമിക്കുന്ന ഭീകരവാദ കൂട്ടായ്മകളില്‍ ചേക്കേറാന്‍ കഴിയുക?

കുഴപ്പങ്ങളും കലാപങ്ങളുമുണ്ടാകുമ്പോള്‍ അവയില്‍നിന്ന് ഒഴിഞ്ഞുനില്‍ക്കണമെന്നാണ് വിശ്വാസി കല്‍പിക്കപ്പെട്ടിരിക്കുന്നത്.

''അബൂബക്‌റയില്‍നിന്ന്: നബി (സ) പറഞ്ഞു: 'കുഴപ്പങ്ങള്‍ ഉണ്ടാകും. അറിയുക, പിന്നെയും കുഴപ്പങ്ങള്‍ ഉണ്ടാകും. അറിയുക, കുഴപ്പങ്ങള്‍ വീണ്ടും സംഭവിക്കും. കലാപത്തില്‍ പങ്കെടുക്കാതെ വീട്ടിലിരിക്കുന്നവനാണ് അതില്‍ ഭാഗഭാക്കായി നടക്കുന്നവനേക്കാള്‍ ഉത്തമന്‍. കലാപത്തിലേക്ക് നടന്നുനീങ്ങുന്നവന്‍ അതിലേക്ക് ഓടിയടുക്കുന്നവനേക്കാള്‍ ഉത്തമനാണ്. കലാപം പൊട്ടിപ്പുറപ്പെട്ടാല്‍, അതില്‍ ഏര്‍പ്പെടാതെ ഒട്ടകമുള്ളവന്‍ ആ ഒട്ടകത്തിന്റെ കാര്യം ശ്രദ്ധിക്കട്ടെ. ആടുകളുള്ളവന്‍ അവയുടെ കാര്യം ശ്രദ്ധിക്കട്ടെ. ഭൂമിയുള്ളവന്‍ ഭൂമിയില്‍ പണിയെടുക്കട്ടെ' ഒരാള്‍ ചോദിച്ചു: 'ഒട്ടകമോ ആടോ ഭൂമിയോ ഇല്ലാത്തവനാണെങ്കിലോ?' അവിടുന്ന് പറഞ്ഞു: 'അവന്‍ തന്റെ വാള് കല്ലുപയോഗിച്ച് കുത്തിപ്പൊട്ടിക്കട്ടെ. എന്നിട്ട് കലാപത്തില്‍നിന്ന് രക്ഷപ്പെടാന്‍ കഴിയുമെങ്കില്‍ അവന്‍ രക്ഷപ്പെടട്ടെ. അല്ലാഹുവേ, ഞാന്‍ ഈ സന്ദേശം ജനങ്ങള്‍ക്ക് എത്തിച്ചില്ലയോ? അല്ലാഹുവേ, ഞാന്‍ എത്തിച്ചില്ലയോ? അല്ലാഹുവേ ഞാന്‍ എത്തിച്ചില്ലയോ?' അപ്പോള്‍ ഒരാള്‍ ചോദിച്ചു: 'പ്രവാചകരേ, നിര്‍ബന്ധിച്ച് എന്നെ കൊണ്ടുപോയി ഒരു ചേരിയില്‍ കക്ഷിചേര്‍ക്കുകയും അങ്ങനെ ഞാന്‍ വെട്ടേറ്റോ അമ്പേറ്റോ മരിക്കുകയും ചെയ്യുകയാണെങ്കിലോ?' അവിടുന്ന് പറഞ്ഞു: 'നിന്നെ വെട്ടിയവന്‍ നിന്റെയും അവന്റെയും പാപഭാരവുമായി നരകത്തിലേക്ക് പോകുന്നതാണ്.''(സ്വഹീഹു മുസ്‌ലിം, കിതാബുല്‍ ഇമാറ)

വിഷയവുമായി ബന്ധപ്പെട്ട വീഡിയോ

ഇസ്‌ലാമിക സമൂഹത്തിന്റെ നേതൃത്വത്തിന്റെ നിര്‍ദേശാനുസരണം കൃത്യവും നിയതവുമായ സൈനിക സന്നാഹങ്ങളോടെ നടക്കുന്നതാണ് ക്വുര്‍ആനും നബിവചനങ്ങളും നിഷ്‌കര്‍ഷിക്കുന്ന യുദ്ധം. രഹസ്യ കേന്ദ്രങ്ങളിലിരുന്ന് ആരെങ്കിലും ആഹ്വാനം ചെയ്യുകയും നിയതമായ ക്രമങ്ങളൊന്നുമില്ലാതെ നടപ്പാക്കപ്പെടുകയും ചെയ്യുന്ന കലാപമല്ല അത്. യുദ്ധം നിര്‍ബന്ധമാണെന്നും അതില്‍ പങ്കെടുത്താല്‍ വമ്പിച്ച പ്രതിഫലമുണ്ടെന്നും രക്തസാക്ഷികള്‍ക്ക് പാപമോചനവും സ്വര്‍ഗപ്രവേശവും ഉറപ്പാണെന്നുമെല്ലാം പഠിപ്പിക്കുന്ന ഇസ്‌ലാമിക പ്രമാണങ്ങള്‍ തന്നെ ആര്, എപ്പോള്‍, എങ്ങനെ യുദ്ധം ചെയ്യണമെന്നും കൂടി വ്യക്തമാക്കിയിട്ടുണ്ട്. നേതൃത്വത്തിന്റെ കീഴിലല്ലാതെയുള്ള സായുധ സമരങ്ങളൊന്നും തന്നെ പ്രവാചകൻ (സ)അനുവദിച്ചതായി കാണാന്‍ കഴിയില്ല. 'നിങ്ങള്‍ ആയുധമണിയാന്‍വേണ്ടി കല്‍പിക്കപ്പെട്ടാല്‍ അതിന്നായി മുന്നിട്ടിറങ്ങുക'(1)യെന്ന പ്രവാചക നിര്‍ദേശം വ്യക്തമാക്കുന്നത് നേതൃത്വത്തിന്റെ കല്‍പനയനുസരിക്കുകയാണ് പൗരന്‍മാര്‍ ചെയ്യേണ്ടതെന്നാണ്. പൗരന്‍മാര്‍ക്ക് കല്‍പന നല്‍കുവാന്‍ അധികാരമുള്ള നേതൃത്വമാണ് അവരോട് ആയുധമണിയുവാന്‍ പറയേണ്ടതെന്നും അങ്ങനെയുള്ള നേതൃത്വത്തിന്റെ നിര്‍ദേശമുണ്ടായാല്‍ ആയുധമണിയേണ്ടത് പൗരന്‍മാരുടെ ബാധ്യതയാണെന്നും ഇത് വ്യക്തമാക്കുന്നു. ഏതെങ്കിലുമൊരു ഭൂഭാഗത്ത് തങ്ങളുദ്ദേശിക്കുന്ന നിയമങ്ങള്‍ നടപ്പാക്കുവാനുള്ള അധികാരമോ തങ്ങളുടെ അനുയായികളുടെ പോലും മേല്‍ ശിക്ഷാവിധികള്‍ നടത്താനുള്ള നിയതമായ അവകാശമോ ഇല്ലാത്ത, സ്വന്തമായ മേല്‍വിലാസം പോലും രഹസ്യമായി വെക്കാന്‍ വിധിക്കപ്പെട്ട നേതൃത്വത്തിന് കീഴില്‍ നടക്കുന്ന കലാപമല്ല, പൗരന്‍മാരുെട മേല്‍ അവകാശവും അധികാരവുമുള്ള നേതൃത്വത്തിന് കീഴില്‍ നടക്കുന്ന, നിയമങ്ങള്‍ അനുസരിച്ചുകൊണ്ട് നടത്തുന്ന സായുധ സമരമാണ് ഇസ്‌ലാം അനുവദിക്കുകയും മഹത്ത്വവല്‍ക്കരിക്കുകയും ചെയ്തിട്ടുള്ളത്. നിയതമായ നേതൃത്വത്തിന് കീഴിലാണ് ഇസ്‌ലാം അനുവദിച്ച യുദ്ധമെന്ന് പ്രവാചകൻ (സ) വ്യക്തമാക്കിയിട്ടുണ്ട്.

''അബൂഹുറയ്‌റയില്‍നിന്ന്: നബി (സ) പറഞ്ഞു: ഇമാം ഒരു പരിച മാത്രമാകുന്നു. അദ്ദേഹത്തിന്റെ പിന്നില്‍നിന്ന് യുദ്ധം ചെയ്യപ്പെടുകയും അദ്ദേഹത്തെക്കൊണ്ട് സംരക്ഷിക്കപ്പെടുകയും ചെയ്യും. ഇനി, അദ്ദേഹം അല്ലാഹുവെ സൂക്ഷിക്കാന്‍ കല്‍പിക്കുകയും നീതിപൂര്‍വ്വം വര്‍ത്തിക്കുകയും ചെയ്താല്‍ അതിന് അദ്ദേഹത്തിന് പ്രതിഫലമുണ്ട്. ഇനി മറ്റ് വല്ലതിനുമാണ് അദ്ദേഹം കല്‍പിക്കുന്നതെങ്കില്‍ അതിന്റെ കുറ്റം അയാള്‍ക്കുണ്ട്.''(2)

''അബൂഹുറയ്‌റയില്‍നിന്ന്: അല്ലാഹുവിന്റെ ദൂതന്‍ പ്രസ്താവിക്കുന്നത് ഞാന്‍ കേട്ടു: 'നാം (ഇഹലോകത്തെ) അവസാനത്തെ ആളുകളും (പരലോകത്തെ) മുന്‍ഗാമികളുമായിരിക്കും. എന്നെ അനുസരിച്ചവന്‍ തീര്‍ച്ചയായും അല്ലാഹുവെ അനുസരിച്ചു. എന്നെ ധിക്കരിച്ചവന്‍ തീര്‍ച്ചയായും അല്ലാഹുവെ ധിക്കരിച്ചു. (സേനാ)നായകനെ അനുസരിച്ചവന്‍ തീര്‍ച്ചയായും എന്നെ അനുസരിച്ചു. സേനാനായകനെ ധിക്കരിച്ചവന്‍ തീര്‍ച്ചയായും എന്നെ ധിക്കരിച്ചു. നേതാവ് പരിചയാണ്. അദ്ദേഹത്തിന്റെ പിന്നില്‍ നിന്നാണ് യുദ്ധം ചെയ്യപ്പെടുന്നത്. അദ്ദേഹത്തിലൂടെയാണ് ശത്രുവില്‍നിന്ന് സുരക്ഷ നേടുന്നത്. അദ്ദേഹം അല്ലാഹുവോട് ഭക്തിപുലര്‍ത്താന്‍ കല്‍പിക്കുകയും നീതി പുലര്‍ത്തുകയുമാണെങ്കില്‍ തീര്‍ച്ചയായും അയാള്‍ക്ക് അതുവഴി പ്രതിഫലം ലഭിക്കും. അതല്ലാത്തതാണ് അയാള്‍ പറയുന്നതെങ്കില്‍ അയാള്‍ക്കായിരിക്കും അതിന്റെ പാപം.''(3)

അബുഹുറയ്റ(റ)ല്‍നിന്ന്: മഅ്കൂല്‍് (റ) നിവേദനം ചെയ്യുന്നു: അല്ലാഹുവിന്റെ ദൂതന്‍ പറഞ്ഞു: 'നല്ലയാളാണെങ്കിലും ചീത്തയാളാണെങ്കിലും ഭരണാധികാരിയോടൊപ്പം ജിഹാദ് ചെയ്യല്‍ നിങ്ങളുടെ ബാധ്യതയാണ്'.(4)

യുദ്ധത്തിന് ആഹ്വാനം ചെയ്യുകയും നേതൃത്വം വഹിക്കുകയും ചെയ്യേണ്ടത് മുസ്‌ലിം ഭരണാധികാരിയാണെന്ന് ക്വുര്‍ആനിന്റെയും നബിവചനങ്ങളുടെയും വെളിച്ചത്തില്‍ മതവിധികള്‍ നല്‍കിയ പൗരാണികരും ആധുനികരുമായ പണ്ഡിതന്മാരെല്ലാം വ്യക്തമാക്കിയിട്ടുള്ളതാണ്. ഹിജ്‌റ ആറാം നൂറ്റാണ്ടില്‍ ജീവിച്ച ഇമാം മവഫ്ഫവദ്ദീന്‍ ഇബ്‌നു ഖുദാമ (റഹ്) നല്‍കിയ ഫത്‌വ നോക്കുക: 'ഭരണാധികാരിയുടെ കൈകളാലും ഗവേഷണത്തിന്റെ (ഇജ്തിഹാദ്) അടിസ്ഥാനത്തിലുമാണ് ജിഹാദ് നടക്കേണ്ടത്. അത് എപ്പോഴാണ് നടക്കേണ്ടതെന്ന് ഭരണാധികാരി തീരുമാനിച്ചാല്‍ ജനങ്ങള്‍ അദ്ദേഹത്തെ അനുസരിക്കേണ്ടതാണ്'.(5)

ഹനഫീ പണ്ഡിതനായ ഇമാം സഫര്‍ അഹ്മദ് ഉഥ്മാനി (ഹിജ്‌റ 1310-1394) പറയുന്നത് ഇങ്ങനെയാണ്: 'മഅ്കൂലിന്റെ ഹദീഥില്‍നിന്ന് ഭരണാധികാരിയോടൊപ്പം ജിഹാദ് ചെയ്യല്‍ നിര്‍ബന്ധമാണെന്ന് വ്യക്തമാകുന്നുണ്ട്; അബൂബക്കര്‍്(റ) തന്റെ പിന്‍ഗാമിയായി ഉമറിനെ (റ) നിശ്ചയിച്ചതുപോലെ മുന്‍ ഭരണാധികാരിയാല്‍ നാമനിര്‍ദേശം ചെയ്യപ്പെടുന്ന ഭരണാധികാരിയാണെങ്കിലും പണ്ഡിതന്മാരും സമൂഹത്തിലെ വേണ്ടപ്പെട്ടവരും കൂടിയാലോചിച്ച് തീരുമാനമെടുക്കപ്പെടുന്ന ഭരണാധികാരിയാണെങ്കിലും അയാളോടൊപ്പം പ്രജകള്‍ ജിഹാദ് ചെയ്യേണ്ടതാണ്... അതിര്‍ത്തികള്‍ സംരക്ഷിക്കുവാനോ ജനങ്ങളുടെ അഭിമാനം സംരക്ഷിക്കുവാനോ രാഷ്ട്രീധികാരം ഉപയോഗിച്ച് സൈന്യത്തെ സംഘടിപ്പിക്കുവാനോ അധികാരവും ശക്തിയും ഉപയോഗിച്ച് മര്‍ദിതകര്‍ക്ക് നീതി നേടിക്കൊടുക്കുവാനോ കഴിയാത്ത ഒരാളെ പണ്ഡിതന്‍മാരും സമൂഹത്തിലെ വേണ്ടപ്പെട്ടവരും കൂടി നേതാവായി നിശ്ചയിച്ചാലും അയാള്‍ ഭരണാധികാരിയെന്നോ (അമീര്‍) നേതാവെന്നോ (ഇമാം) വിളിക്കപ്പെടാന്‍ അര്‍ഹനല്ലെന്നാണ് എന്റെ അഭിപ്രായം. ഭരണാധികാരത്തിന്റെയും നേതൃത്വത്തിന്റെയും അടിസ്ഥാനം അധികാരത്തിലും അവകാശത്തിലുമാണ് എന്നതിനാലും നേതാവെന്ന് വിളിക്കപ്പെടുന്നതുകൊണ്ടുമാത്രം അവ ൈകവരുകയില്ല എന്നതുകൊണ്ടുമാണ് ഇത്. അത്തരം ഒരാള്‍ക്ക് അനുസരണ പ്രതിജ്ഞ ചെയ്യുവാനോ അയാളെ അനുസരിക്കുവാനോ ജനങ്ങള്‍ ബാധ്യസ്ഥരല്ല. അയാളോടൊപ്പം ചേര്‍ന്ന് ജിഹാദ് ചെയ്യുവാനും പാടില്ല'.(6)

ഇന്ത്യന്‍ പണ്ഡിതനായ ഇമാം ഹമീദുദ്ദീന്‍ ഫറാഹിയുടെ (ക്രി. 1862-1930) വീക്ഷണമാണിത്: 'സ്വതന്ത്രമായ ഒരു നാട്ടിലേക്ക് പലായനം ചെയ്യാതെ, സ്വന്തം നാട്ടില്‍വെച്ച് ജിഹാദ് ചെയ്യാന്‍ പാടുള്ളതല്ല. ഇബ്‌റാഹീം നബി (സ)യുടെ ചരിത്രവും പലായനത്തെക്കുറിച്ച വചനങ്ങളും ഇതാണ് വ്യക്തമാക്കുന്നത്. പ്രവാചകൻ(സ)ന്റെ ജീവിതവും ഇതിനെയാണ് ബലപ്പെടുത്തുന്നത്. രാഷ്ട്രീയമായ അധികാരമില്ലാത്ത ഒരാളാണ് ജിഹാദ് നടത്തുന്നതെങ്കില്‍ അതുവഴി വ്യാപകമായ കുഴപ്പങ്ങളും അരാജകത്വവുമാണ് ഉണ്ടാകുകയെന്നതിനാലാണിത്'.(7)

പൗരാണിക പണ്ഡിതന്മാരുടെ വീക്ഷണങ്ങളെ പരാമര്‍ശിച്ചശേഷം ആധുനിക കര്‍മശാസ്ത്ര ഗ്രന്ഥകാരനായ സയ്യിദ് സാബിഖ് (ക്രി.1915-2000) ഈ വിഷയതതെ ക്രോഡീകരിക്കുന്നത് ഇങ്ങനെയാണ്: 'സാമൂഹിക ബാധ്യതകളില്‍ (ഫര്‍ദ് കിഫാഇ) ഭരണാധികാരിക്ക് നിര്‍ബന്ധമായ കാര്യങ്ങളുടെ ഉദാഹരണങ്ങളാണ് ജിഹാദും ശിക്ഷാവിധികളുടെ നിര്‍വഹണവും'.(8)

ആധുനിക പണ്ഡിതന്‍മാരില്‍ അഗ്രേസരനായ ശൈഖ് മുഹമ്മദ് ബിന്‍ സാലിഹ്ബ്‌നുല്‍ ഉഥൈമിന്‍ (ക്രി. 1925-2001) എഴുതുന്നു: 'ഏത് അവസരത്തിലാണെങ്കിലും ഭരണാധികാരിയുടെ നിര്‍ദേശമില്ലാതെ സൈന്യത്തിന് യുദ്ധത്തിന് പോകാന്‍ പാടുള്ളതല്ല. വ്യക്തികളല്ല, ഭരണാധികാരികളാണ് യുദ്ധം ചെയ്യുവാനും ജിഹാദിന് പുറപ്പെടുവാനും കല്‍പിക്കപ്പെട്ടിട്ടുള്ളത് എന്നതുകൊണ്ടാണിത്. തീരുമാനത്തിന് അധികാരമുള്ളവരെ അനുധാവനം ചെയ്യുകയാണ് വ്യക്തികളുടെ ഉത്തരവാദിത്തം. ഇമാമിന്റെ അനുവാദമില്ലാതെ ഒരാളും യുദ്ധത്തിന് ഇറങ്ങിപ്പുറപ്പെടാന്‍ പാടുള്ളതല്ല; പ്രതിരോധം ആവശ്യമുള്ളപ്പോഴല്ലാതെ. ശത്രുക്കള്‍ പൊടുന്നനെ ജനങ്ങെള അക്രമിക്കുകയും, അവര്‍ അത്തരം തിന്മകള്‍ ഭയപ്പെടുകയും ചെയ്യുന്നുവെങ്കില്‍, അവര്‍ക്ക് സ്വയം പ്രതിരോധിക്കാവുന്നതാണ്. അത്തരം അവസരങ്ങളില്‍ പ്രതിരോധത്തിന് വേണ്ടിയുള്ള പോരാട്ടം ഒരു വൈയക്തിക ബാധ്യതയായിത്തീരും എന്നതുകൊണ്ടാണിത്.'

ഭരണാധികാരികളുടെ ഉത്തരവാദിത്തമാണ് ജിഹാദ് സംബന്ധിയായ കാര്യങ്ങള്‍ എന്നതിനാലാണ് വ്യക്തികള്‍ക്ക് യുദ്ധം അനുവദനീയമല്ലാതിരിക്കുന്നതിന്റെ ഒന്നാമത്തെ കാരണം. ഭരണാധികാരിയുടെ അനുവാദമില്ലാതെ യുദ്ധത്തിന് പുറപ്പെടല്‍ അയാളുടെ അവകാശങ്ങളെ ലംഘിക്കുകയും പരിധി വിടലുമായിത്തീരുന്നു. ഭരണാധികാരിയുടെ സമ്മതമില്ലാതെ യുദ്ധംചെയ്യാന്‍ ജനങ്ങളെ അനുവദിച്ചാല്‍ അന് വ്യാപകമായ കുഴപ്പങ്ങളിലേക്കാണ് നയിക്കുക. കുതിരയെ സജ്ജമാക്കുവാനും യുദ്ധം ചെയ്യുവാനും ആഗ്രഹിക്കുന്നവരെയെല്ലാം അതിന് അനുവദിച്ചാല്‍ അതുമൂലമുണ്ടാകാവുന്ന വിപത്തുകള്‍ ചില്ലറയൊന്നുമല്ല. ശത്രുവിനെതിരെയെന്ന വ്യാജേന ഭരണാധികാരിക്കെതിരെ ആയുധമെടുക്കുവാനും മറ്റു ജനസമൂഹങ്ങള്‍ക്കെതിരെ കലാപങ്ങള്‍ അഴിച്ചുവിടുവാനുമായിരിക്കും ചിലര്‍ ശ്രമിക്കുക. 'സത്യവിശ്വാസികളായ രണ്ട് വിഭാഗം തമ്മില്‍ സംഘട്ടനത്തിലാവുകയാണെങ്കില്‍ അവര്‍ തമ്മില്‍ രഞ്ജിപ്പുണ്ടാക്കുക(ക്വുര്‍ആന്‍ 49:9)യെന്നാണ് അല്ലാഹുവിന്റെ കല്‍പന. ഈ മൂന്ന് കാരണങ്ങളാലും അതല്ലാത്ത മറ്റ് കാര്യങ്ങളാലും ഭരണാധികാരിയുടെ അനുവാദമില്ലാതെ യുദ്ധം ചെയ്യല്‍ അനുവദനീയമല്ല'.

സുഊദി അറേബ്യയിലെ ഉന്നതരായ പണ്ഡിതന്മാരുടെ സഭയായ 'അല്ലജ്‌നത്തുദ്ദാഇമലില്‍ ബുഹൂഥുല്‍ ഇല്‍മിയ്യ വല്‍ ഇഫ്താഅ്' ഇവ്വിഷയകമായി നല്‍കിയ ഫത്‌വ ഇങ്ങനെയാണ്: 'അല്ലാഹുവിന്റെ വചനത്തിന്റെ ഔന്നത്യത്തിനും ഇസ്‌ലാംമതത്തെ സംരക്ഷിക്കുന്നതിനും അത് എത്തിച്ചുകൊടുക്കുകയും പ്രചരിപ്പിക്കുകയും ചെയ്യുന്നതിനുമുള്ള സൗകര്യമൊരുക്കുന്നതിനും അതിന്റെ പവിത്രത കാത്തുസൂക്ഷിക്കുന്നതിനും വേണ്ടി ജിഹാദ് ചെയ്യല്‍ അത് ചെയ്യാന്‍ സാധിക്കുന്നവരുടെയെല്ലാം കടമയാണ്. അതിന്നായുള്ള സംഘത്തെ ഒരുക്കുകയും സംഘടിപ്പിക്കുകയും അയക്കുകയും ചെയ്യുന്നത് കൃത്യതയോടെയും അച്ചടക്കത്തോടെയുമാകാതിരുന്നാല്‍ അത് അരാജകത്വത്തിനും കുഴപ്പങ്ങള്‍ക്കും കാരണമാകുകയും ദൗര്‍ഭാഗ്യകരമായ പരിണിതിയില്‍ ചെന്നെത്തുന്ന അവസ്ഥയുണ്ടാവുകയുമാണ് ചെയ്യുക. അതിനാല്‍ ജിഹാദിന് തുടക്കം കുറിക്കുകയും സജ്ജീകരിക്കുകയും ചെയ്യേണ്ടത് മുസ്‌ലിം ഭരണാധികാരിയുടെ കര്‍ത്തവ്യമാണ്; അതിന് പ്രചോദിപ്പിക്കുകയാണ് പണ്ഡിതധര്‍മം. ഭരണാധികാരി ജിഹാദിനൊരുങ്ങുകയും മുസ്‌ലിംകളെ സംഘടിപ്പിക്കുകയും ചെയ്താല്‍ അയാളുടെ വിളിക്ക് ഉത്തരം നല്‍കേണ്ടത് അതിന് സാധിക്കുന്നവരുടെയെല്ലാം കടമയാണ്. അല്ലാഹുവിനെ കണ്ടുമുട്ടാന്‍ ആത്മാര്‍ഥമായി ആഗ്രഹിച്ചും സത്യത്തെ സഹായിക്കാനാകുമെന്ന് പ്രതീക്ഷിച്ചും ഇസ്‌ലാമിനെ സംരക്ഷിക്കണമെന്ന് ആഗ്രഹിച്ചും കൊണ്ടാണ് അത് ചെയ്യേണ്ടത്. ജിഹാദിനുവേണ്ടി വിളിക്കപ്പെട്ടാല്‍ കാരണങ്ങളൊന്നുമില്ലാതെ അതില്‍ നിന്നൊഴിഞ്ഞ് നില്‍ക്കുന്നത് കുറ്റകരമാണ്'.(9)

സുഊദി അറേബ്യയുടെ മതകാര്യവകുപ്പ് മന്ത്രിയും പ്രഗത്ഭ പണ്ഡിതനുമായ ശൈഖ് സ്വാലിഹ് ആലുശൈഖും ഇക്കാര്യം അസന്ദിഗ്ധമായി തുറന്ന് പ്രഖ്യാപിച്ചിട്ടുണ്ട്. 'മുസ്‌ലിം ഭരണാധികാരികള്‍ക്ക് മാത്രമെ ജിഹാദിനുവേണ്ടി ആഹ്വാനം ചെയ്യാന്‍ അവകാശമുള്ളൂ; പണ്ഡിതന്‍മാര്‍ക്കോ മറ്റ് ആര്‍ക്കെങ്കിലുമോ അതിനുള്ള അധികാരമില്ല. ഭരണാധികാരികളല്ലാത്തവര്‍ ജിഹാദിന് ആഹ്വാനം ചെയ്താല്‍ അത് വമ്പിച്ച കുഴപ്പങ്ങള്‍ക്കും അരാജകത്വത്തിനുമാണ് നിമിത്തമാവുക. ഭരണാധികാരിയുടെ സമ്മതമില്ലാതെ ആരെങ്കിലും ജിഹാദിന് ആഹ്വാനം ചെയ്താല്‍ അയാള്‍ ഇസ്‌ലാമിക തത്ത്വങ്ങള്‍ക്ക് എതിര് പ്രവര്‍ത്തിക്കുകയാണ് ചെയ്യുന്നത്. അമുസ്‌ലിംകളുമായി കരാറുകളും സന്ധികളും ഉണ്ടാക്കുക ഭരണാധികാരിയുടെ നിയമപരമായ അധികാര പരിധിയില്‍ വരുന്ന കാര്യങ്ങളാണ്. പ്രമാണങ്ങളും പണ്ഡിതന്മാരുടെ ഏകോപിച്ച അഭിപ്രായങ്ങളും വ്യക്തമാക്കുന്നത്, ഭരണാധികാരികളെ അനുസരിക്കല്‍ നിര്‍ബന്ധവും അവരോട് അനുസരണക്കേട് കാണിക്കുന്നത് ഗുരുതരമായ അപരാധവുമാണ് എന്നാണ്'.(10)

ഭരണാധികാരിയുടെ നേതൃത്വത്തില്‍ മുസ്‌ലിംകള്‍ അച്ചടക്കത്തോടെയും ആത്മാര്‍ഥതയോടെയും നടത്തുന്ന പടയോട്ടമാണ് ജിഹാദായിത്തീരുന്ന യുദ്ധം. ആര്‍ക്കാണ് നേതൃത്വമെന്ന് തിട്ടമില്ലാത്ത കലാപങ്ങളോ രാഷ്ട്രീയവും സാമുദായികവുമായ ലക്ഷ്യങ്ങള്‍ക്കുവേണ്ടി നടത്തപ്പെടുന്ന സായുധ സമരങ്ങളോ ജിഹാദിന്റെ ഗണത്തില്‍പെടുത്താനാവുന്നവയാണെന്ന് പ്രമാണങ്ങളൊന്നും പഠിപ്പിക്കുന്നില്ല. ഇസ്‌ലാമിനുവേണ്ടി നടത്തപ്പെടുന്ന യുദ്ധങ്ങളില്‍ മുസ്‌ലിം പടയാളിയുടെ ലക്ഷ്യം അല്ലാഹുവിന്റെ തൃപ്തിയും അവന്‍ തൃപ്തിപ്പെട്ടവര്‍ക്ക് നല്‍കുന്ന സ്വര്‍ഗവും മാത്രമായിരിക്കണം.

  1. സ്വഹീഹുല്‍ ബുഖാരി, കിതാബുല്‍ ജിഹാദ്; സ്വഹീഹു മുസ്‌ലിം, കിതാബുല്‍ ഹജ്ജ്
  2. സ്വഹീഹു മുസ്‌ലിം, കിതാബുല്‍ ഇമാറ
  3. സ്വഹീഹുല്‍ ബുഖാരി, കിതാബുല്‍ ജിഹാദ്
  4. സുനനു അബൂദാവൂദ്, കിതാബുല്‍ ജിഹാദ്; ഇത് ദ്വഈഫാണ്. ഇമാം അല്‍ബാനി: സുനനു അബൂദാവൂദ്, ഹദീഥ്: 2532
  5. ഇബ്‌നു ഖുദാമ അല്‍ മഖ്ദസീ: അല്‍ മുഗ്‌നി, അമ്മാന്‍, 2004, കിതാബുല്‍ ജിഹാദ്, വാല്യം 2, പുറം 2275
  6. Zafar Ahmad Uthmani: Ii'la'al-Sunan, 3rd ed., vol. 12, Karachi, 1415 AH, Pages15-16, Quoted by Dr. ShehzadSaleem: "No Jihad without the State"” (http://www.monthly-renaissance.com)
  7. Farahi: Majmu'ah Tafasiri Farahi, 1st ed., Lahore, 1991 Quoted by Al Mawrid: in "No Jihad without the State: View of the Jurists" (http://www.al-mawrid.org)
  8. സയ്യിദ് സാബിഖ്: ഫിഖ്ഹു സ്‌സുന്ന:, ബൈറൂത്, 1980, വാല്യം 3, പുറം 30
  9. ശൈഖ് മുഹമ്മദ് ബിന്‍ സ്വാലിഹ് ബിനുല്‍ ഉഥൈമീന്‍: അശ് ശറഹല്‍ മുംതിഅ്, 8/22 (http://archive.org/details/sharh_Mu-mti3)
  10. ഫതാവാ ലജ്‌നത്തുല്‍ ദാഇമ’(12/12)യില്‍ നിന്ന് ശൈഖ് മുഹമ്മദ് സ്വാലിഹ് അല്‍ മുനജ്ജിദ് ഉദ്ധരിച്ചത് Islam QA (http://islam-qa.info/en/ref/69746/jihad)
വിഷയവുമായി ബന്ധപ്പെട്ട വീഡിയോ

'എന്റെ സമുദായത്തില്‍ സത്യത്തിനുവേണ്ടി സമരം ചെയ്യുകയും അവരെ എതിര്ക്കുന്നവരോട് പോരാടി വിജയം വരിക്കുകയും ചെയ്യുന്ന ഒരു വിഭാഗം എന്നെന്നുമുണ്ടായിരിക്കും' എന്ന ഹദീഥ് ഭീകരവാദത്തിന് വളമായിത്തീരുന്നുവെന്നാണ് ആക്ഷേപം. ഹദീഥിന്റെ പൂർണരൂപം ഇങ്ങനെയാണ്: ഇംറാനുബ്‌നു ഹുസൈനിൽ നിന്ന്: അല്ലാഹുവിന്റെ ദൂതന്‍ പറഞ്ഞു: 'എന്റെ സമുദായത്തില്‍ സത്യത്തിനുവേണ്ടി സമരം ചെയ്യുകയും അവരെ എതിര്ക്കു ന്നവരോട് പോരാടി വിജയം വരിക്കുകയും ചെയ്യുന്ന ഒരു വിഭാഗം എന്നെന്നുമുണ്ടായിരിക്കും; അവരില്‍ അവസാനമുള്ളവര്‍ മസീഹുദ്ദജ്ജാലുമായി യുദ്ധം ചെയ്യുന്നതുവരെ'.(സുനനു അബൂദാവൂദ്, കിതാബുല്‍ ജിഹാദ്; മുസ്‌നദ് അഹ്മദ് (4/429); ഇത് സ്വഹീഹാണ്: ഇമാം അല്‍ബാനി: സുനനു അബൂദാവൂദ്, ഹദീഥ്: 2483)

യുദ്ധം നിലനില്ക്കു്ന്ന സാഹചര്യത്തിലുള്ള നിര്ദശങ്ങളാണ് ക്വുര്ആകനിലും ഹദീഥുകളിലുള്ള പോരാട്ടവുമായി ബന്ധപ്പെട്ട നിര്ദേആശങ്ങളെന്ന് പറയുമ്പോള്‍ പ്രവാചകന്റെയോ(സ) അനുചരന്മാ്രുടെയോ കാലത്ത് മാത്രം പ്രസക്തമാണ് ഈ നിര്ദേശങ്ങള്‍ എന്ന് മനസ്സിലാക്കിക്കൂടാത്തതാണ്. ഇസ്‌ലാമികമായ ലക്ഷ്യങ്ങള്ക്കുവേണ്ടി ഉണ്ടാകുന്ന യുദ്ധങ്ങള്‍, അത് എപ്പോള്‍ ഉണ്ടാകുന്നതാണെങ്കിലും ഈ നിര്ദേശങ്ങളെല്ലാം അവയ്ക്ക് ബാധകമാണ് . അത്തരം യുദ്ധങ്ങള്‍ അവസാനനാളുവരെ ഉണ്ടാകാൻ സാധ്യതയുണ്ടെന്നും അങ്ങനെ ഉണ്ടാകുമ്പോള്‍ അവിടെയെല്ലാം ദൈവിക നിര്ദേ്ശങ്ങള്ക്ക് വിധേയമായി രണാങ്കണത്തിലിറങ്ങുവാന്‍ വിശ്വാസി ബാധ്യസ്ഥനാണെന്നും വ്യക്തമാക്കുന്ന ഹദീഥാണിത്. അവസാനനാളുവരെ സത്യത്തിനുവേണ്ടി പോരാടുന്നവന്‍ ഉണ്ടാകുമെന്ന് പറയുമ്പോള്‍ എക്കാലത്തും യുദ്ധം നിലനില്ക്കുങമെന്നോ പോരാട്ടമില്ലാത്ത സ്ഥിതി ഒരിക്കലും ഉണ്ടാവുകയില്ലെന്നോ ആണ് പ്രവാചകൻ (സ) അര്ഥതമാക്കിയതെന്ന് മനസ്സിലാക്കിക്കൂടാത്തതാണ്. സത്യത്തിനുവേണ്ടി സമരസജ്ജരായ ഒരു വിഭാഗം എന്നെന്നുമുണ്ടാകുമെന്നും അനിവാര്യമായ സാഹചര്യങ്ങളില്‍ ആയുധമെടുത്ത് പോരാടുവാന്‍ അവര്ക്ക് യാതൊരുവിധ മടിയുമുണ്ടാവുകയില്ലെന്നുമാണ് പ്രവാചകൻ (സ) ഇവിടെ പഠിപ്പിക്കുന്നത്. ഇത് പറഞ്ഞ മുഹമ്മദ് നബി(സ)യുടെ കാലത്തുതന്നെ യുദ്ധമില്ലാത്ത കാലമുണ്ടായിരുന്നുവെന്ന് ചരിത്രം വ്യക്തമാക്കുന്നുണ്ട്. യുദ്ധമില്ലാതിരിക്കുവാന്‍ അവിശ്വാസികളുമായി കരാറുണ്ടാക്കിയ നബി (സ) എന്നെന്നും യുദ്ധം നിലനില്ക്കുറന്ന നാളുകളുണ്ടാകണം എന്നാണ് വിവക്ഷിച്ചതെന്ന് കരുതുവാന്‍ യാതൊരുവിധ ന്യായവുമില്ല. അനിവാര്യമെങ്കില്‍ ആയുധമെടുക്കുവാന്‍ സന്നദ്ധരായ ഒരു വിഭാഗം മുസ്‌ലിംകള്‍ എക്കാലത്തുമുണ്ടാകുമെന്നും അവരുമായി യുദ്ധം ചെയ്യുന്ന ശത്രുക്കളെ അവര്‍ പരാജയപ്പെടുത്തുമെന്നും മസീഹുദ്ദജ്ജാലുമായി നടക്കുന്ന യുദ്ധമായിരിക്കും ഈ ഗണത്തിലെ അവസാനത്തേത് എന്നുമുള്ള വസ്തുതകളാണ് ഇവിടെ പ്രവാചകൻ (സ) വ്യക്തമാക്കുന്നത്.

'യുദ്ധം ചെയ്യുന്നവര്‍ എന്റെ സമുദായത്തില്‍ എന്നെന്നുമുണ്ടായിരിക്കു'മെന്ന പ്രവാചക വചനമാണ് ലോകത്ത് നിലനില്ക്കുദന്ന ഭീകര പ്രവര്ത്തംനങ്ങള്ക്കെ ല്ലാം കാരണമായിത്തീരുന്നതെന്ന വാദം കഴമ്പില്ലാത്തതാണ്. ഈ പ്രവാചക വചനത്തിന്റെ വെളിച്ചത്തില്‍ ജീവിച്ച പ്രവാചകാനുചരന്മാ്രോ നബി (സ)യുടെ പ്രശംസകള്ക്കും ആശംസകള്ക്കു മെല്ലാം നിമിത്തമായ മൂന്ന് ആദിമ തലമുറകളോ ഒന്നുംതന്നെ ഇൗ ഹദീഥില്നിിന്ന് മുസ്‌ലിംകള്‍ എന്നെന്നും യുദ്ധം ചെയ്യുന്നവരായിരിക്കണമെന്ന് മനസ്സിലാക്കിയിട്ടില്ലെന്ന് അവരുടെ നടപടികള്‍ നമ്മെ തെര്യപ്പെടുത്തുന്നു. നടേ ഉദ്ധരിച്ച ഹദീഥ് വിശാരദരായ അബ്ദുല്ലാഹിബ്‌നു മുബാറക്കിന്റെ (റ) അഭിപ്രായത്തില്നി ന്ന് യുദ്ധം ചെയ്യാതിരിക്കുകയോ അതിന് ആഗ്രഹിക്കാതിരിക്കുകയോ ചെയ്യുന്നത് കാപട്യമാണെന്ന പ്രവാചക പ്രസ്താവന അദ്ദേഹം ജീവിച്ചിരുന്ന കാലത്തേക്ക് മാത്രം ബാധകമാണെന്ന് വ്യക്തമാകുന്നുണ്ട്. അതുകൊണ്ടുതന്നെ എന്നെന്നും യുദ്ധമുണ്ടാകുമെന്ന പ്രവാചക പ്രസ്താവനയില്നിയന്ന് പ്രചോദനമുള്ക്കൊണണ്ടാണ് ഭീകരവാദം നിലനില്ക്കുലന്നത് എന്ന വാദം ശുദ്ധ ഭോഷ്‌ക്കാണ്. യുദ്ധസന്നദ്ധരായ ഒരുവിഭാഗം എന്നെന്നും നിലനില്ക്കുംമെന്ന് മാത്രമാണ് പ്രസ്തുത പ്രവാചക വചനം വ്യക്തമാക്കുന്നത്; അവരോട് സമരം ചെയ്യുന്ന ശത്രുക്കള്ക്ക്ി അവരെ തോല്പിൊക്കാനാവുകയില്ലെന്ന പ്രവചനവും അത് ഉള്ക്കൊകള്ളുന്നുണ്ട്.

യുദ്ധം അനിവാര്യമായ സാഹചര്യങ്ങളില്‍ യുദ്ധത്തിന് ഒരുങ്ങേണ്ടത് ഓരോ മുസ്‌ലിമിന്റെയും കര്‍ത്തവ്യമാണെന്ന് വ്യക്തമാക്കുന്ന നിരവധി വാക്യങ്ങൾ ഖുർആനിലും ഹദീഥുകളിലുമുണ്ട്. . യുദ്ധരംഗത്തുനിന്ന് ഒഴിഞ്ഞുനില്‍ക്കുകയോ പിന്‍മാറുകയോ ചെയ്ത് ഭീരുത്വം പ്രകടിപ്പിക്കുന്നതിന് പകരം അല്ലാഹുവിന്റെ മാര്‍ഗത്തില്‍ പടപൊരുതുവാന്‍ സന്നദ്ധനാവുകയും ആ രംഗത്ത് ത്യാഗങ്ങള്‍ സഹിക്കുകയും ചെയ്യേണ്ടവനാണ് മുസ്‌ലിം എന്ന് തന്നെയാണ് പ്രസ്തുത വചനങ്ങൾ പഠിപ്പിക്കുന്നത്. . ആദര്‍ശ സംരക്ഷണത്തിനുവേണ്ടിയുള്ള യുദ്ധത്തില്‍ ത്യാഗങ്ങള്‍ സഹിക്കുകയും ആവശ്യമെങ്കില്‍ മരിക്കുവാന്‍ സന്നദ്ധനാവുകയും ചെയ്യുന്നതിന് പ്രചോദിപ്പിച്ചുകൊണ്ടുള്ള ക്വുര്‍ആന്‍ വചനങ്ങളും നബിനിര്‍ദേശങ്ങളും യുദ്ധങ്ങളും രക്തസാക്ഷ്യവും വഴി ലഭിക്കുവാനിരിക്കുന്ന നേട്ടങ്ങളെക്കുറിച്ച് വിശ്വാസികളെ തെര്യപ്പെടുത്തിക്കൊണ്ടാണ് ആ രംഗത്ത് മുന്നേറുവാന്‍ ആവേശം നല്‍കുന്നത്. ആദര്ശയുദ്ധത്തിനുള്ള പ്രചോദനമാണ്, കലാപത്തിനുള്ള ആഹ്വാനമല്ല അവയൊന്നും ഉൾക്കൊള്ളുന്നത്.

അനിവാര്യമായ യുദ്ധസാഹചര്യത്തില്‍ വിശ്വാസികളൊന്നും യുദ്ധരംഗത്തേക്കിറങ്ങുവാന്‍ അറച്ചുനില്‍ക്കരുതെന്ന് പഠിപ്പിക്കുകയും യുദ്ധത്തിന് പ്രചോദിപ്പിക്കുകയും ചെയ്തുകൊണ്ടുള്ള വചനങ്ങളെ സന്ദര്‍ഭത്തില്‍നിന്ന് അടര്‍ത്തിമാറ്റി വ്യാഖ്യാനിച്ചുകൊണ്ടാണ് ക്വുര്‍ആനും നബിവചനങ്ങളുമെല്ലാം വളര്‍ത്തുന്നത് യുദ്ധക്കൊതി പൂണ്ട സമൂഹത്തെയാണെന്ന് വിമര്‍ശകര്‍ സമര്‍ഥിക്കാന്‍ ശ്രമിക്കുന്നത്. ശത്രുക്കള്‍ യുദ്ധത്തിനൊരുങ്ങി വരുന്ന സാഹചര്യത്തില്‍ മുസ്‌ലിംകളെയും യുദ്ധ സജ്ജരാക്കുകയെന്ന ദൗത്യമാണ് പ്രസ്തുത വചനങ്ങള്‍ നിര്‍വഹിക്കുന്നതെന്ന് അവതരിക്കപ്പെട്ട സന്ദര്‍ങ്ങള്‍ക്കകത്തുനിന്നുകൊണ്ട് പ്രസ്തുത വചനങ്ങെള അപഗ്രഥിച്ചാല്‍ ആര്‍ക്കും മനസ്സിലാകും. ഇസ്‌ലാമിനെയും മുസ്‌ലിംകളെയും നശിപ്പിക്കുന്നതിനായി സര്‍വായുധ സജ്ജരായ ശത്രുക്കള്‍ ഒരുങ്ങിപ്പുറപ്പെടുമ്പോള്‍ മുസ്‌ലിംകളെ സമരസജ്ജരാക്കുകയും ശത്രുക്കളെ പ്രതിരോധിച്ച് ഇസ്‌ലാമിക സമൂഹത്തെ സംരക്ഷിക്കുവാന്‍ മുസ്‌ലിംകളെ പ്രചോദിപ്പിക്കുകയും ചെയ്യുന്നതിന് 'യുദ്ധക്കൊതി'യെന്ന് വിളിക്കാമെങ്കില്‍ മാത്രമെ ക്വുര്‍ആനും ഹദീഥുകളും വളര്‍ത്തുന്നത് യുദ്ധക്കൊതിപൂണ്ട സമൂഹത്തെയാണെന്ന് പറയാനാവൂ. യുദ്ധക്കൊതിയില്ലാത്തവര്‍ക്ക് ഭൂമിയില്‍ ജീവിക്കാനാവില്ലെന്നുകൂടി അനുബന്ധമായി പറയാന്‍ സന്നദ്ധരാണെങ്കില്‍ മാത്രമെ ഇസ്‌ലാമിക പ്രമാണങ്ങളെ തത്‌വിഷയകമായി പ്രതിക്കൂട്ടില്‍ കയറ്റാനാവൂ എന്നതാണ് വാസ്തവം.

യുദ്ധത്തിനുവേണ്ടി വിളിക്കപ്പെട്ടാല്‍ മുസ്‌ലിംകളെല്ലാം യുദ്ധസജ്ജരാകേണ്ടത് ഓരോരുത്തരുടെയും ബാധ്യതയാണെന്ന് ക്വുര്‍ആനും ഹദീഥുകളും വ്യക്തമാക്കുന്നുണ്ട്. യുദ്ധം അനിവാര്യമായ സാഹചര്യത്തില്‍ മുസ്‌ലിം ചെയ്യേണ്ട ഏറ്റവും പ്രധാനപ്പെട്ട ജിഹാദ് സ്വന്തം ജീവന്‍ ബലിയറുപ്പിക്കുവാന്‍ സന്നദ്ധനായി പടക്കളത്തിലിറങ്ങുകയാണ്. ദൈവിക വിധിവിലക്കുകള്‍ക്കനുസൃതമായി സ്വന്തത്തെ പരിവര്‍ത്തിപ്പിച്ചുകൊണ്ട് തുടങ്ങുന്ന ജിഹാദ്, ആദര്‍ശ പ്രബോധനത്തിന് നാവും പേനയും കരവും സമ്പത്തും സമയവുമെല്ലാം വിനിയോഗിച്ചുകൊണ്ട് തുടരുകയും അനിവാര്യമായ സാഹചര്യത്തില്‍ ആയുധമെടുത്ത് അടരാടിക്കൊണ്ട് പൂര്‍ണത പ്രാപിക്കുകയും ചെയ്യുന്നു. ത്യാഗ പരിശ്രമം എന്ന് അര്‍ഥംവരുന്ന ജിഹാദ് പൂര്‍ണമായി അന്വര്‍ഥമാക്കുന്നത് സ്വന്തം ജീവന്‍ ത്യജിച്ചുകൊണ്ട് അടര്‍ക്കളത്തിലിറങ്ങി ദൈവമാര്‍ഗത്തിലുള്ള പരിശ്രമത്തില്‍ ഏര്‍പ്പെടുമ്പോഴാണ്. ആദര്‍ശ സമരത്തിന്റെ പര്യായമെന്നോണം 'ജിഹാദ്' എന്ന് പ്രയോഗിക്കപ്പെടുന്ന സ്ഥിതിയുണ്ടായത് ജീവത്യാഗമാണ് ദൈവമാര്‍ഗത്തിലുള്ള ത്യാഗ പരിശ്രമങ്ങളില്‍ ഏറ്റവും കഠിനതരവും ഏറെ ക്ഷമയും സഹനവുമാവശ്യമുള്ളതുമെന്നതുകൊണ്ടായിരിക്കണം. ഹദീഥ് ഗ്രന്ഥങ്ങളിലും കര്‍മശാസ്ത്ര ഗ്രന്ഥങ്ങളിലുമെല്ലാം ജിഹാദിനെക്കുറിച്ച അധ്യായങ്ങളില്‍ പ്രധാനമായും ചര്‍ച്ച ചെയ്യുന്നത് സായുധ പോരാട്ടത്തെക്കുറിച്ചായതും അതുകൊണ്ടുതന്നെയായിരിക്കണം. 'ജിഹാദുകളില്‍ ഏറ്റവും മെച്ചപ്പെട്ടത് ഏതാണ്?' എന്ന അനുചരനായ അംറുബ്‌നു അബശിന്റെ ചോദ്യത്തിന് 'ഒരാളുടെ രക്തം ചിന്തുകയും കുതിരക്ക് പരിക്കുപറ്റുകയും ചെയ്യുന്നത്' എന്ന പ്രവാചകന്റെ(സ) ഉത്തരത്തില്‍നിന്നും(സുനനു ഇബ്‌നു മാജ, അബ്‌വാബുല്‍ ജിഹാദ്, ഇത് സ്വഹീഹാണ്: ഇമാം അല്‍ബാനി: സ്വഹീഹു ഇബ്‌നു മാജ, ഹദീഥ്: 2253) വ്യക്തമാവുന്നതും അനിവാര്യഘട്ടങ്ങളില്‍ ചെയ്യുന്ന സായുധ സമരം തന്നെയാണ് ജിഹാദുകളില്‍ ഉന്നതശീര്‍ഷമായത് എന്നുതന്നെയാണല്ലോ.

യുദ്ധത്തിന് പ്രേരിപ്പിക്കുകയും രക്തസാക്ഷ്യത്തിന്റെ മഹത്ത്വം ഉല്‍ഘോഷിക്കുകയും ചെയ്യുന്ന ക്വുര്‍ആന്‍ വചനങ്ങളും പ്രവാചക മൊഴികളുമൊന്നും യഥാര്‍ഥത്തില്‍ ഭീകരവാദത്തിന് കടന്നുവരാനുള്ള പഴുതുകളൊന്നും നല്‍കുന്നില്ലായെന്ന വസ്തുത തമസ്‌കരിച്ചുകൊണ്ടാണ് ഇസ്‌ലാമിക പ്രമാണങ്ങളില്‍നിന്നാണ് ഭീകരതയുടെ ബീജാവാപം നടക്കുന്നതെന്ന് വിമര്‍ശകര്‍ വാദിക്കുന്നത്. ആദര്‍ശത്തിന്റെ മാത്രം അടിസ്ഥാനത്തിലുള്ളതാകണം പോരാട്ടങ്ങള്‍ എന്ന് നിഷ്‌കര്‍ഷിക്കുന്ന ഇസ്‌ലാമിക പ്രമാണങ്ങള്‍ക്ക് എങ്ങനെയാണ് അന്ധമായ ദേശീയതയുടെയും വര്‍ഗീയതയുടെയും അടിസ്ഥാനത്തില്‍ നടക്കുന്ന ഭീകര പ്രവര്‍ത്തനങ്ങളെ അനുകൂലിക്കാനാവുക? 'വര്‍ഗീയതയെ പ്രതിനിധീകരിക്കുകയും അതിനുവേണ്ടി കോപിഷ്ഠനാവുകയും അതിന്റെ പേരില്‍ മരണം വരിക്കുകയും ചെയ്തവര്‍ ജാഹിലിയ്യാ മരണമാണ് പുല്‍കിയിരിക്കുന്നത്'(സ്വഹീഹു മുസ്‌ലിം, കിതാബുല്‍ ഇമാറ) എന്നും 'അല്ലാഹുവിന്റെ മാര്‍ഗത്തില്‍ പോരാടുമ്പോള്‍പോലും വല്ല ഭൗതികലക്ഷ്യവും മനസ്സിലുണ്ടെങ്കില്‍ അയാള്‍ക്ക് യാതൊരു പ്രതിഫലവുമില്ല'(സുനനു അബൂദാവൂദ്, കിതാബുല്‍ ജിഹാദ്, ഇത് സ്വഹീഹാണ്: ഇമാം അല്‍ബാനി: സുനനു അബൂദാവൂദ്, ഹദീഥ്: 2516) എന്നും പഠിപ്പിച്ച പ്രവാചക വചനങ്ങളുടെ വെളിച്ചത്തില്‍ എങ്ങനെയാണ് തികച്ചും രാഷ്ട്രീയവും വംശീയവും വര്‍ഗീയവുമായ ലക്ഷ്യങ്ങള്‍ക്കുവേണ്ടി സംഘടിപ്പിക്കപ്പെടുന്ന ഭീകര പ്രവര്‍ത്തനങ്ങള്‍ വളര്‍ന്നുവരിക? ഇസ്‌ലാമിക പ്രമാണങ്ങളുടെ ദുര്‍വ്യാഖ്യാനങ്ങള്‍ വഴിയല്ലാതെ, മുഹമ്മദ് നബി (സ) പഠിപ്പിച്ച മതദര്‍ശനത്തിന്റെ വെളിച്ചത്തില്‍ ഭീകരവാദത്തിന് വളരാന്‍ പഴുതുകളൊന്നും ഇല്ലെന്നതാണ് യാഥാര്‍ഥ്യം.

വർഗീയതക്കും വിഭാഗീയതക്കുമെതിരെ പോരാടിയയാളാണ് നബി(സ). അദ്ദേഹത്തിന്റെ ജീവിതത്തിലൊരിക്കലും വർഗീയതയുണ്ടാക്കുന്ന എന്തെങ്കിലും ചെയ്തതായി തെളിയിക്കാൻ ആർക്കും കഴിയില്ല. ആദര്‍ശത്തിനുവേണ്ടിയല്ലാത്ത പോരാട്ടങ്ങളൊന്നും അംഗീകരിച്ചുകൂടായെന്ന് മാത്രമല്ല, ഇസ്‌ലാമിനുവേണ്ടി നടക്കുന്ന പോരാട്ടത്തില്‍ പങ്കെടുക്കുമ്പോള്‍പോലും സത്യമതത്തിന്റെ ഔന്നത്യവല്‍ക്കരണമല്ലാത്ത മറ്റ് ലക്ഷ്യങ്ങളൊന്നും ഉണ്ടായിക്കൂടെന്നുകൂടിയാണ് മുഹമ്മദ് നബി (സ) പഠിപ്പിച്ചത്.

''അബൂമൂസല്‍ അശ്അരിയിൽ (റ) നിന്ന്: ഒരു ഗ്രാമീണ അറബി, നബി (സ)യുടെ അടുത്തുവന്ന് പറഞ്ഞു: 'ദൈവദൂതരേ, ഒരാള്‍ സമരാര്‍ജിത സ്വത്തിനുവേണ്ടി യുദ്ധം ചെയ്യുന്നു. മറ്റൊരാള്‍ കീര്‍ത്തിക്കുവേണ്ടി യുദ്ധം ചെയ്യുന്നു. വേറൊരാള്‍ തന്റെ സ്ഥാനവും ശൂരതയും ജനങ്ങളെ അറിയിക്കാന്‍ യുദ്ധം ചെയ്യുന്നു. ഇവരില്‍ ആരാണ് അല്ലാഹുവിന്റെ മാര്‍ഗത്തില്‍? പ്രവാചകൻ(സ)പ്രതിവചിച്ചു: 'അല്ലാഹുവിന്റെ വചനം അത്യുന്നതമായിത്തീരാന്‍ യുദ്ധം ചെയ്യുന്നവനാണ് അല്ലാഹുവിന്റെ മാര്‍ഗത്തില്‍ (യുദ്ധം ചെയ്യുന്നവന്‍).''(1)

അല്ലാഹുവിന്റെ മാര്‍ഗത്തില്‍ നടക്കുന്ന യുദ്ധങ്ങളില്‍ പങ്കെടുക്കുമ്പോള്‍പോലും താന്‍ നല്ലൊരു പടയാളിയാണെന്ന് അറിയപ്പെടണമെന്ന ആഗ്രഹമോ മറ്റെന്തെങ്കിലും തരത്തിലുള്ള സ്വാര്‍ഥമായ താല്‍പര്യങ്ങളോ ഉണ്ടായാല്‍ അതുവഴി നരകപ്രവേശമാണുണ്ടാവുകയെന്ന് ഗൗരവതരമായി പ്രവാചകൻ(സ)താക്കീത് ചെയ്തിട്ടുണ്ട്.

''അബൂഹുറയ്റ(റ)യില്‍നിന്ന്: അന്ത്യദിനത്തില്‍ ആദ്യമായി വിധി കല്‍പിക്കപ്പെടുക രക്തസാക്ഷിത്വം വരിച്ച ഒരാളുടെ കാര്യത്തിലാണ്. അയാളെ കൊണ്ടുവന്ന് അയാള്‍ക്ക് നല്‍കിയ അനുഗ്രഹങ്ങള്‍ അയാളെ ഓര്‍മിപ്പിക്കുകയും അയാള്‍ മനസ്സിലാക്കുകയും ചെയ്യും. അപ്പോള്‍ അല്ലാഹു ചോദിക്കും: 'അത് നീ എങ്ങനെ വിനിയോഗിച്ചു?' രക്തസാക്ഷിത്വം വരിക്കുവോളം നിന്റെ മാര്‍ഗത്തില്‍ ഞാന്‍ യുദ്ധം ചെയ്തുവെന്ന് അയാള്‍ പറയും. ഉടനെ അല്ലാഹു പറയും: 'നീ കളവ് പറയുകയാണ്. ധീരനെന്ന് പറയപ്പെടാന്‍ വേണ്ടിയാണ് നീ യുദ്ധം ചെയ്തത്. അത് പറയപ്പെടുകയും ചെയ്തിട്ടുണ്ടല്ലോ'. പിന്നീട് അയാള്‍ക്ക് ശിക്ഷ വിധിക്കുകയും മുഖം നിലത്തിട്ട് വലിച്ചിഴച്ച് നരകത്തില്‍ എറിയപ്പെടുകയും ചെയ്യും.(2)

''അബൂഹുറയ്റ(റ)ല്‍നിന്ന്: ഒരാള്‍വന്ന് ദൈവദൂതരോട് (സ) ചോദിച്ചു. 'അല്ലാഹുവിന്റെ ദൂതരേ, ഒരാള്‍ അല്ലാഹുവിന്റെ മാര്‍ഗത്തില്‍ ജിഹാദ് ചെയ്യുന്നതിനോടൊപ്പം ചില ഭൗതിക നേട്ടങ്ങളും ആഗ്രഹിക്കുന്നുവെങ്കില്‍ അയാളുടെ സ്ഥിതിയെന്താണ്?'  പ്രവാചകൻ(സ)മറുപടി പറഞ്ഞു: 'അയാള്‍ക്ക് യാതൊരുവിധ പ്രതിഫലവുമില്ല'. അയാള്‍ ഇത് മൂന്ന് പ്രാവശ്യം ആവര്‍ത്തിച്ച് ചോദിച്ചപ്പോഴും അല്ലാഹുവിന്റെ ദൂതൻ(സ)ഒരേ മറുപടിതന്നെ ആവര്‍ത്തിക്കുകയാണ് ചെയ്തത്.''(3)

''അബ്ദുല്ലാഹിബ്‌നു അംറില്‍്യനിന്ന്: ഞാന്‍ പ്രവാചകനോട് ആവശ്യപ്പെട്ട 'ജിഹാദിനെയും സൈനിക നടപടികളെയുംപറ്റി എനിക്ക് പറഞ്ഞുതന്നാലും'. അദ്ദേഹം പറഞ്ഞു: 'ഓ അബ്ദുല്ലാഹിബ്‌നു അംറ്! അല്ലാഹുവില്‍നിന്നുള്ള പ്രതിഫലം കാംക്ഷിച്ചുകൊണ്ടാണ് നീ സഹനത്തോെട പോരാടിയതെങ്കില്‍ അല്ലാഹുവില്‍നിന്നുള്ള നിന്റെ പ്രതിഫലവും പ്രദര്‍ശിപ്പിച്ചുകൊണ്ടായിരിക്കും നീ ഉയിര്‍ത്തെഴുന്നേല്‍പിക്കപ്പെടുക. ആളുകളെ കാണിക്കുവാനും ഭൗതിക വിഭവങ്ങള്‍ ആഗ്രഹിച്ചുംകൊണ്ടാണ് നീ അടരാടുന്നതെങ്കില്‍ അത് പ്രദര്‍ശിപ്പിച്ചുകൊണ്ടായിരിക്കും നീ ഉയര്‍ത്തെഴുന്നേല്‍പിക്കപ്പെടുക. അബ്ദുല്ലാഹിബ്‌നു അംറെ! എന്തൊരു ഉദ്ദേശത്തിനുവേണ്ടിയാണോ നീ പടപൊരുതിയതും കൊല്ലപ്പെട്ടതും. ആ അവസ്ഥയിലായിരിക്കും നീ ഉയിര്‍ത്തെഴുന്നേല്‍പിക്കപ്പെടുക.''(4)

ഗോത്രമഹിമക്കോ അന്ധമായ കക്ഷിമാല്‍സര്യത്തിനോ വേണ്ടിയുള്ള യുദ്ധങ്ങളെയും മുഹമ്മദ് നബി (സ) നിരോധിച്ചു. എന്തിനുവേണ്ടിയാണ് യുദ്ധമെന്ന് കൃത്യമായി മനസ്സിലാക്കാതെ തന്റെ കക്ഷിയാണ് ആയുധമെടുത്തിരിക്കുന്നത് എന്നതിനാല്‍ സായുധരാവുകയും രണഭൂമിയില്‍പോയി മരണപ്പെടുകയും  ചെയ്താല്‍ പ്രസ്തുത മരണം ഇസ്‌ലാമികമല്ലെന്നും അത് ജാഹിലിയ്യത്തിന്റെതാണെന്നുമാണ് നബി (സ) പഠിപ്പിച്ചത്.

''അബൂഹുറയ്റ(റ)യില്‍നിന്ന്: നബി (സ) പ്രസ്താവിച്ചു: 'അനുസരണം കൈവെടിയുകയും സംഘടനയെ വേര്‍പിരിയുകയും അങ്ങനെ ആ നിലയില്‍ മരിക്കുകയും ചെയ്തവന്റേത് ജാഹിലിയ്യാ മരണമാണ്. ഒരാള്‍ ഒരു പക്ഷത്തിനുവേണ്ടി ദേഷ്യപ്പെേട്ടാ ഒരു പക്ഷത്തേക്ക് ആളുകളെ ക്ഷണിച്ചുകൊണ്ടോ അല്ലെങ്കില്‍ ഒരു പക്ഷത്തെ സഹായിച്ചുകൊണ്ടോ അന്ധമായ കൊടിക്കുപിന്നില്‍ നിന്നുകൊണ്ട് യുദ്ധം ചെയ്യുകയും എന്നിട്ട് വധിക്കപ്പെടുകയും ചെയ്താല്‍ ജാഹിലിയ്യാ മരണമാണ് അയാള്‍ക്ക് സംഭവിച്ചിട്ടുള്ളത്. എന്റെ സമുദായത്തിലെ നല്ലവനെയും തെമ്മാടിയേയും വെട്ടിക്കൊന്ന് അവര്‍ക്കെതിരെ ഇറങ്ങിപ്പുറപ്പെടുകയും അവരിലെ സത്യവിശ്വാസിയുടെ കാര്യം സൂക്ഷിക്കാതിരിക്കുകയും കരാര്‍ ചെയ്തവനോട് ആ കരാര്‍ പാലിക്കാതിരിക്കുകയും ചെയ്തവന്‍ എന്നില്‍പ്പെട്ടവനല്ല; ഞാന്‍ അവനില്‍പ്പെട്ടവനുമല്ല.''(5)

'ജുന്‍ദബ്ബ്‌നു അബ്ദുല്ല (റ)യില്‍നിന്ന്: അല്ലാഹുവിന്റെ ദൂതന്‍ പറഞ്ഞു: വ്യക്തമല്ലാത്ത ഒരു കാരണത്തിനുവേണ്ടി അന്ധമായി ഒരാള്‍ യുദ്ധം ചെയ്യുകയും ഗോത്രവര്‍ഗീയതയെ പ്രതിനിധീകരിക്കുകയും അതിനുവേണ്ടി കോപിഷ്ടനാവുകയും മരണപ്പെടുകയും ചെയ്താല്‍ അയാള്‍ ജാഹിലിയ്യാ മരണമാണ് മരിച്ചിരിക്കുന്നത്'.(6)

അല്ലാഹുവിന്റെ മാര്‍ഗത്തിലുള്ള, അവന്റെ പ്രീതി മാത്രം ലക്ഷ്യമാക്കിക്കൊണ്ട് മുസ്‌ലിംകള്‍ നടത്തുന്ന യുദ്ധത്തില്‍ അവന്‍ ആഗ്രഹിക്കുകയും പ്രതിക്ഷീക്കുകയും ചെയ്യുന്നത് അല്ലാഹുവിന്റെ സഹായമാണ്.  അതുകൊണ്ടുതന്നെ അത്തരം യുദ്ധങ്ങളില്‍ അണിനിരക്കേണ്ടത് ആദര്‍ശ ശുദ്ധിയുള്ളവര്‍ മാത്രമാകണമെന്ന് പ്രവാചകൻ(സ)നിഷ്‌കര്‍ഷിച്ചതായി കാണാന്‍ കഴിയും. ബദ്‌റിലേക്കുള്ള യാത്രാമധ്യെ നടന്ന ഒരു സംഭവം നോക്കുക.

''നബിപത്‌നി ആഇശയിൽ (റ) നിന്ന്: നബി (സ) ബദ്‌റിന്റെ നേരെ പുറപ്പെട്ടു. ഹര്‍റതുല്‍വബറയിലെത്തിയപ്പോള്‍ ഒരാള്‍ നബി (സ)യെ കാണാനിടയായി. അയാളുടെ ധീരതയും സഹായമനസ്ഥിതിയും പ്രസിദ്ധമാണ്. അയാളെ കണ്ടപ്പോള്‍ നബി (സ)യുടെ അനുചരന്മാര്‍ സന്തോഷിച്ചു. അയാള്‍ നബി്യയുമായി സന്ധിച്ചപ്പോള്‍ പറഞ്ഞു: 'അങ്ങയെ പിന്തുടരാനും അങ്ങയോടൊപ്പം നേട്ടങ്ങള്‍ കൈവരിക്കാനുമാണ് ഞാന്‍ വന്നിട്ടുള്ളത്.' അപ്പോള്‍ അവിടുന്ന് ചോദിച്ചു: 'താങ്കള്‍ അല്ലാഹുവിലും അവന്റെ പ്രവാചകനിലും വിശ്വസിക്കുന്നുണ്ടോ?' അയാള്‍: 'ഇല്ല.' നബി: 'എങ്കില്‍ തിരിച്ചുപൊയ്‌ക്കൊള്ളുക. ഞാനൊരിക്കലും ബഹുദൈവ വിശ്വാസിയുടെ സഹായം തേടുക യില്ല.' അവര്‍ (ആഇശ) പറയുകയാണ്: അവിടുന്ന് വീണ്ടും മുന്നോട്ടുപോയി. അങ്ങനെ ഞങ്ങള്‍ അശ്ശജറ'യില്‍ എത്തിയപ്പോള്‍ നബി (സ)യെ ആ മനുഷ്യന്‍ വീണ്ടും കാണാനിടയായി. അപ്പോള്‍ അയാള്‍ നബി (സ)യോട് മുമ്പ്പറഞ്ഞത് ആവര്‍ത്തിച്ചു. നബി (സ)യും പഴയതുതന്നെ ആവര്‍ത്തിച്ചശേഷം പറഞ്ഞു: 'എങ്കില്‍ നിങ്ങള്‍ തിരിച്ചുപൊയ്‌ക്കൊള്ളുക. ഞാന്‍ ബഹുദൈവ വിശ്വാസിയുടെ സഹായം തേടുകയില്ല.' അവര്‍ (ആഇശ) പറയുന്നു: പിന്നീട് അയാള്‍ മടങ്ങിവന്നു. അങ്ങനെ മരുഭൂമിയില്‍വെച്ച് അയാള്‍ നബി (സ)യെ കണ്ടുമുട്ടി. അപ്പോഴും അയാളോട് നബി (സ)ആദ്യം ചോദിച്ചതുപോലെ ചോദിച്ചു. 'താങ്കള്‍ അല്ലാഹുവിലും അവന്റെ പ്രവാചകനിലും വിശ്വസിക്കുന്നുണ്ടോ?' അദ്ദേഹം പറഞ്ഞു: 'അതെ, വിശ്വസിക്കുന്നു.' അപ്പോള്‍ നബി (സ)പറഞ്ഞു: 'എങ്കില്‍ താങ്കള്‍ ഞങ്ങളുടെകൂടെ പുറപ്പെട്ടുകൊള്ളുക.''(7)

എത്രതന്നെ വീരശൂരനായ പോരാളിയാണെങ്കിലും അയാള്‍ മുസ്‌ലിമാണെന്ന് പ്രഖ്യാപിക്കാത്തിടത്തോളം ആദര്‍ശ സംരക്ഷണത്തിനുവേണ്ടിയുള്ള പടയോട്ടത്തില്‍ അയാളുടെ സഹായം ആവശ്യമില്ലെന്ന് വ്യക്തമാക്കുകയും ഇസ്‌ലാം സ്വീകരിച്ചതോെട അയാളെ പടയണിയില്‍ ചേര്‍ക്കുകയും ചെയ്ത പ്രവാചക നടപടിയില്‍നിന്ന് വര്‍ഗീയതയുടെ ലാഞ്ചനപോലുമില്ലാത്തതാണ് ഇസ്‌ലാം പഠിപ്പിക്കുന്ന യുദ്ധമെന്ന വസ്തുത സുതരാം വ്യക്തമാകുന്നുണ്ട്. ആദര്‍ശത്തിന്റെ പേരില്‍ രണ്ടുതവണ മാറ്റിനിര്‍ത്തപ്പെടുകയും 'താങ്കള്‍ ബഹുദൈവാരാധകനായതിനാല്‍ താങ്കളുടെ സഹായം ഞങ്ങള്‍ക്കാവശ്യമില്ലാ'യെന്ന് വ്യക്തമാക്കപ്പെടുകയും ചെയ്തയാള്‍ക്കുതന്നെ ഇസ്‌ലാം സ്വീകരിച്ചതോടെ മുസ്‌ലിം പടയണിയില്‍ സ്ഥാനം നല്‍കിയത് അയാളുടെ ആദര്‍ശമാറ്റംകൊണ്ട് മാത്രമാണെന്ന് ആര്‍ക്കും മനസ്സിലാകും. വംശീയതയുടെയും വര്‍ഗീയതയുടെയും പേരിലുള്ള യുദ്ധം ഇസ്‌ലാമിന് അന്യമാണെന്നാണ് ഇത് പഠിപ്പിക്കുന്നതെന്ന് വ്യക്തമാണ്.

  1. സ്വഹീഹു മുസ്‌ലിം, കിതാബുല്‍ ജിഹാദ് വസ്‌സിയാര്‍
  2. സ്വഹീഹു മുസ്‌ലിം, കിതാബുല്‍ ജിഹാദ് വസ്‌സിയാര്‍
  3. സുനനു അബൂദാവൂദ്, കിതാബുല്‍ ജിഹാദ്, ഇതിന്റെ നിവേദക പരമ്പര സ്വീകാര്യമാണെന്ന് (സ്വഹീഹ്) ഇമാം അല്‍ബാനി വ്യക്തമാക്കിയിട്ടുണ്ട്, സുനനു അബീദാവൂദ് ഹദീഥ്: 2516
  4. സുനനു അബൂദാവൂദ്, കിതാബുല്‍ ജിഹാദ്, ഇതിന്റെ നിവേദക പരമ്പര ഹസനാണെന്ന് ഇമാം ഹാക്കിം (2/85)വ്യക്തമാക്കിയിട്ടുണ്ട്, Nasirudheen Al Khattab: English Translation of Sunan Abu Dawud, Riyadh, 2008, Volume 3, Page 215
  5. സ്വഹീഹു മുസ്‌ലിം, കിതാബുല്‍ ഇമാറ
  6. സ്വഹീഹു മുസ്‌ലിം, കിതാബുല്‍ ഇമാറ
  7. സ്വഹീഹു മുസ്‌ലിം, കിതാബുല്‍ ജിഹാദ് വസ്‌സിയാര്‍
വിഷയവുമായി ബന്ധപ്പെട്ട വീഡിയോ

മദീനയിലെത്തി അവിടത്തെ ഭരണാധികാരിയായിത്ത്തീർന്നതോടെ നിരവധി യുദ്ധങ്ങൾ നയിക്കുകയും നാടുകൾ പിടിച്ചടക്കുയും ചെയ്ത മുഹമ്മദ് നബിയുടെ (സ) നടപടികൾ അദ്ദേഹം ഒരു അധികാരമോഹിയായിരുന്നുവെന്നല്ലേ വ്യക്തമാക്കുന്നത് ?

ദീനയിലെ ഭരണാധികാരിയായിത്ത്തീർന്ന ശേഷം നബി(സ) യുദ്ധം ചെയ്തിട്ടുണ്ടെന്നത് നേരാണ്. അയാൾ നാടുകളുടെ അധികാരം അദ്ദേഹത്തിന്റെ കൈകളിൽ അർപ്പിക്കപ്പെട്ടിട്ടുണ്ടെന്നതും ശരിയാണ്. എന്നാൽ അതിനെ അധികാഹാരമോഹത്തിന്റെ അളവുകോലുകളുപയോഗിച്ച് അനീതിയായിരിക്കും. അധീശത്വത്തിനോ അധികാര വിപുലീകരണത്തിനോ ആഡംബര ജീവിതത്തിനോ വേണ്ടി നടക്കേണ്ടവയാണ് യുദ്ധങ്ങളെന്ന രാഷ്ട്രമീമാംസയുടെ പരമ്പരാഗത പാഠങ്ങളില്‍ അഭിരമിക്കുന്നവർക്ക് മുഹമ്മദ് നബി (സ) ചെയ്ത യുദ്ധങ്ങളുടെ ആത്മാവെന്താണെന്ന് മനസ്സിലാവുകയില്ല. അധികാര വിപുലീകരണത്തിനുവേണ്ടി നടക്കുന്ന യുദ്ധങ്ങളെ അപഗ്രഥിക്കുവാന്‍ ഉപയോഗിക്കുന്ന അതേ മൂശയിലിട്ട് ആദര്ശ സംരക്ഷണത്തിനും ആദര്ശപമനുസരിച്ച് ജീവിക്കുവാനും വേണ്ടി നടത്തുന്ന യുദ്ധങ്ങളെ മനസ്സിലാക്കുവാന്‍ ശ്രമിച്ചതുകൊണ്ടാണ് മുഹമ്മദ് നബി(സ)യെ യുദ്ധക്കൊതിയനായി കാണാന്‍ അവർ ധൃഷ്ടരാവുന്നത്. ചരിത്രത്തിന് പരിചയമുള്ള അധികാര പ്രമത്തതയുടെ യുദ്ധങ്ങളെവിടെ, ആദര്ശമത്തിനുവേണ്ടി മുഹമ്മദ് നബി (സ) നയിച്ച യുദ്ധങ്ങളെവിടെ? താരതമ്യത്തിനുപോലും പറ്റാത്തത്രയും വ്യത്യസ്തങ്ങളാണ് ഇവ രണ്ടുമെന്നതാണ് വാസ്തവം.

നബി (സ) നയിച്ച യുദ്ധങ്ങളെ നബിജീവിതത്തിന്റെ മറ്റു വശങ്ങളെയുംകൂടി പരിഗണിച്ചുകൊണ്ട് അപഗ്രഥിക്കുവാന്‍ സന്നദ്ധമായാല്‍ തങ്ങളുടെ വാദങ്ങളെല്ലാം ശുദ്ധ അസംബന്ധമാണെന്ന് നബി (സ) യിെല യുദ്ധക്കൊതിയനെ ഗവേഷണം ചെയ്‌തെടുക്കുവാന്‍ ശ്രമിക്കുന്നവര്ക്കുവതന്നെ ബോധ്യമാകും. അധികാരപ്രമത്തതയാണ് പ്രവാചക പോരാട്ടങ്ങള്ക്ക് പിന്നിലെന്ന് പറയുന്നവര്ക്ക്െ പ്രത്യുത പ്രമത്തതയുടെ പ്രചോദനമെന്തായിരിക്കുമെന്ന് വിശദീകരിക്കേണ്ട ബാധ്യതയുണ്ട്. ഒരാള്ക്ക് അധികാരമെന്തിനാണ് എന്ന ചോദ്യത്തിന് ഇവരുടെ കയ്യിലുള്ള ഉത്തരം ആഡംബര ജീവിതത്തിന് എന്നാണ്. ആഡംബര ജീവിതത്തിനാണ് അധികാരമെങ്കില്‍, അതിനോടുള്ള പ്രതിപത്തിയുള്ളയാള്‍ തീര്ച്ചെയായും ആഡംബര പ്രിയനായിരിക്കണം. ഓരോ യുദ്ധവും കഴിയുകയും തന്റെ അധികാരപരിധി വര്ധിപക്കുകയും ചെയ്യുന്നതിനനുസരിച്ച് അയാളുടെ ആഡംബര ജീവിതത്തിന്റെ പൊലിമയും വര്ധിതച്ചുകൊണ്ടിരുന്നിരിക്കണം. അങ്ങനെ ആഡംബരങ്ങളില്‍ ആറാടിക്കൊണ്ട് താനും തന്റെ കുടുംബവും ജീവിക്കുന്നതിനിടയിലായിരിക്കണം അയാള്‍ ഇൗ ലോകത്തുനിന്ന് വിടപറഞ്ഞിരിക്കുക. അധികാര വിപുലീകരണത്തിനായി യുദ്ധം ചെയ്ത ലോകത്തിന് പരിചയമുള്ളവരുടെയെല്ലാം അവസ്ഥയിതാണ്. ഈ വസ്തുതകളുടെ വെളിച്ചത്തില്‍ നബിജീവിതത്തെ മുന്ധാിരണകളില്ലാതെ പരിശോധിക്കുവാന്‍ ആത്മാര്ഥഥമായി സന്നദ്ധമാവുകയാണെങ്കില്‍ അധികാര ദുര മൂത്ത് ചെയ്തതല്ല പ്രവാചക പോരാട്ടങ്ങളെന്ന വസ്തുത വിമര്ശടകര്ക്ക് പോലും സുതരാം വ്യക്തമാകും.

അറേബ്യയുടെ അധികാരിയായിരിക്കെ മരണപ്പെട്ട മുഹമ്മദ് നബി (സ) യുടെ 'ആഡംബര' ജീവിതത്തെക്കുറിച്ച് വ്യക്തമാക്കുന്ന ഹദീഥുകൾ പരിശോധിച്ചാൽ മതി, അധികാരമോഹമായിരുന്നു അദ്ദേഹത്തിന്റെ യുദ്ധങ്ങൾക്ക് പിന്നിലെന്ന ആരോപണത്തിന്റെ നിജസ്ഥിതിയറിയുവാൻ.

മാസങ്ങളോളം പച്ചവെള്ളവും കാരക്കയും തിന്ന് ജീവിതം തള്ളിനീക്കുന്ന പ്രവാചകനും കുടുംബവും!

ഒരു ദിവസം രണ്ടുനേരം പോലും വയര്നിനറച്ച് ഭക്ഷണം കഴിക്കാത്ത രാഷ്ട്രനേതാവ്!

വിലകുറഞ്ഞ കാരക്കപോലും ഭക്ഷിക്കുവാനില്ലാതെ വിശപ്പ് സഹിക്കുന്ന ജനനായകന്‍!

ശരീരത്തില്‍ ചുവന്ന പാടുകളുണ്ടാക്കുമാറുള്ള പരുക്കന്‍ ഈത്തപ്പനപ്പായയില്‍ അന്തിയുറങ്ങുന്ന അന്തിമ പ്രവാചകന്‍!

ദ്രവിച്ച ചെരിപ്പുകളും കഷ്ണംവെച്ച കുപ്പായവുമണിഞ്ഞ് രാഷ്ട്രഭരണം നടത്തുന്ന ദൈവദൂതന്‍!

സ്വന്തം കുടുംബത്തിന്റെ പട്ടിണി മാറ്റാനായി തന്റെ പടയങ്കി ജൂതന്റെ പക്കല്‍ പണയംവെച്ച രീതിയില്‍ മരണപ്പെട്ട രാഷ്ട്രപതി!

പൊട്ടിപ്പോയിട്ട് വിളക്കിവെച്ച് ഉപയോഗിക്കുന്ന പാനപാത്രം ഉപയോഗിക്കുന്ന ജനനേതാവ്! അനന്തരാവകാശമായി തന്റെ കുടുംബത്തിന് ഒന്നും അവശേഷിപ്പിക്കാതെ ഇഹലോകവാസം വെടിഞ്ഞ അന്തിമ പ്രവാചകന്‍!

പൊതുമുതലില്നിാന്ന് ഒരു കാരക്ക തിന്നാനൊരുമ്പെട്ട പിഞ്ചുബാലനായ പൗത്രെന തടയുകയും ശാസിക്കുകയും ചെയ്യുന്ന മാര്ഗുദര്ശാകന്‍!

അധ്വാനിച്ച് പൊട്ടിയ കൈകളുമായി തനിക്ക് ഒരു ഭൃത്യനെ അനുവദിച്ച് തരണമെന്നപേക്ഷിച്ച സ്വന്തം മകളോട് അതിന് കഴിയില്ലെന്ന് നിസ്സങ്കോചം മറുപടി പറഞ്ഞ രാഷ്ട്ര നേതാവ്!

നിരവധി യുദ്ധങ്ങള്‍ ചെയ്യുകയും അധികാരമുറപ്പിക്കുകയും ചെയ്തതിനുശേഷമുള്ള നബിജീവിതത്തിന്റെ നഖചിത്രമാണിത്. ഇനി നബിവിമര്ശകര്‍ പറയട്ടെ, പ്രവാചകൻ (സ) യുദ്ധം ചെയ്തത് തന്റെ ആഡംബര ജീവിതത്തെ പൊലിപ്പിക്കുവാനായിരുന്നുവെന്ന്! തനിക്ക് ജീവിതസുഖങ്ങള്‍ നല്കുൊന്ന അധികാരത്തോടുള്ള മോഹംകൊണ്ടാണ് നബി (സ) യുദ്ധം ചെയ്തതെന്ന് അദ്ദേഹത്തിന്റെ ജീവതത്തെക്കുറിച്ച് സമഗ്രമായി പഠിച്ചാല്‍ കടുത്ത നബിവിരോധിക്കുപോലും പറയാനാവില്ല എന്നതാണ് വാസ്തവം. അരപ്പട്ടിണിയും ഉണക്കറൊട്ടിയും കാരക്കയും പച്ചവെള്ളവും കഴിച്ചുള്ള ജീവിതവും മകളോട് അടുക്കളയില്‍ അധ്വാനിക്കുവാനുപദേശിച്ചുകൊണ്ടുള്ള സമാശ്വസിപ്പിക്കലും പനയോലപ്പായയില്‍ കിടന്നുറങ്ങിയും കഷ്ണംവെച്ച പരുക്കല്‍ കുപ്പായമിട്ടുകൊണ്ടുള്ള ഭരണനിര്വ്ഹണവുമായിരുന്നു അന്നത്തെ ആഡംബര ജീവിതമെന്ന ഗവേഷണഫലം പുറത്തുവിടാന്‍ നബിവിരോധികളൊന്നുംതന്നെ ധൃഷ്ടരാവുകയില്ലെന്ന് നാം പ്രത്യാശിക്കുക!

ഗോത്രങ്ങളുടെമേല്‍ അധീശത്വം സ്ഥാപിക്കുന്നതിനും അവരെ കൊള്ളയടിച്ച് യുദ്ധാര്ജിത സ്വത്തുക്കള്‍ നേടിയെടുക്കുന്നതിനും അതുവഴി ആഡംബര ജീവിതം നയിക്കുന്നതിനുംവേണ്ടി നടത്തിയിരുന്ന അറേബ്യന്‍ ഗോത്രവര്ഗീ യുദ്ധങ്ങള്‍ അവസാനിപ്പിക്കുകയും ആദര്ശത്തിനുവേണ്ടി മാത്രമാണ് യുദ്ധം ചെയ്യേണ്ടതെന്ന പാഠം പഠിപ്പിക്കുകയുമെന്ന ദൗത്യമാണ് മുഹമ്മദ് നബി (സ) താന്‍ നിയോഗിക്കപ്പെട്ട സമൂഹത്തില്‍ നിര്വഹിച്ചത്. അതോടൊപ്പംതന്നെ, സാമ്രാജ്യത്വ മോഹങ്ങളെ സാക്ഷാല്ക്കരിക്കുന്നതിനായി നിരപരാധികളെ കൊന്നൊടുക്കുന്നത് ന്യായീകരണമര്ഹി്ക്കാത്ത കുറ്റകൃത്യമാണെന്നും ആദര്ശ്മനുസരിച്ച് ജീവിക്കുവാനുള്ള സ്വാതന്ത്ര്യത്തിനുവേണ്ടി, പ്രസ്തുത സ്വാതന്ത്ര്യം തടയപ്പെടുമ്പോഴും സ്വന്തം സുരക്ഷയെ തകര്ത്തു കൊണ്ട് നാടിനെ അടിമപ്പെടുത്തുവാനായി സായുധ മുന്നേറ്റങ്ങളുണ്ടാകുമ്പോള്‍ അതിനെ പ്രതിരോധിക്കുവാനും വേണ്ടിയാണ് യുദ്ധം ചെയ്യേണ്ടതെന്നും ലോകത്തുള്ള മുഴുവന്‍ മനുഷ്യരെയും പഠിപ്പിക്കുകകൂടി ചെയ്തു, മുഹമ്മദ്‌ല. ഗോത്രവൈര്യത്തിലും അക്രമണോല്സുപകതയിലും എതിര്‍ ഗോത്രങ്ങളെ കൊള്ളയടിച്ച് അവരുടെ സ്വത്തുപയോഗിക്കുകയും തരുണീമണികളെ വെപ്പാട്ടികളാക്കിത്തീര്ത്ത്െ അവരില്നികന്ന് രതിസുഖം നുകരുകയും ചെയ്യുന്നതില്‍ എത്രമാത്രം അഭിരമിക്കുന്നവരായിരുന്നു അറബികളെന്ന് കഅ്ബാലയത്തില്‍ തൂക്കിയിട്ടിരിക്കുന്ന (മുഅല്ലഖാത്ത്) ഇംറുല്‍ ഖൈസ് ബ്‌നു ഹുജ്‌റ്, ത്വറഫതുബ്‌നു അബ്ദ്, സുഹൈറുബ്‌നു അബീസുല്മ്, ലബീദുബ്‌നു റബീഅഃ, അന്ത,രബ്‌നു ശദാദ്, അംറുബ്‌നു കുല്ഥൂംമ, ഹാരിഥുബ്‌നു ഹില്ലിസ എന്നീ ജാഹിലിയ്യാ കവികളുടെ രചനകള്‍ വ്യക്തമാക്കുന്നുണ്ട്. അത്തരക്കാരെ ആദര്ശ്ത്തിനുവേണ്ടി സമരം ചെയ്യുന്നവരാക്കിത്തീര്ത്ത് യുദ്ധത്തെ മാനവവല്ക്ക്രിക്കേണ്ടത് എങ്ങനെയെന്ന് ലോകത്തെ പഠിപ്പിക്കുകയാണ് മുഹമ്മദ് നബി (സ) ചെയ്തത്.

മദീനാരാഷ്ട്രത്തിന്റെ സുരക്ഷയായിരുന്നു മുഹമ്മദ് നബി (സ) നയിച്ച യുദ്ധങ്ങളുടെ ലക്ഷ്യമെങ്കിൽ പിന്നെയെന്തിനാണ് അദ്ദേഹം ഇങ്ങോട്ട് യുദ്ധത്തിന് വരാത്ത അയൽനാടുകളെ ആക്രമിച്ചത്? ഇത്തരം ആക്രമണങ്ങൾ മുഹമ്മദ് നബിക്ക് ശേഷം വ്യാപകമായി നടന്നിട്ടുണ്ടെന്ന് ചരിത്രം പഠിപ്പിക്കുന്നുമൂണ്ടല്ലൊ ?

രാഷ്ട്രസുരക്ഷയ്ക്കും സമാധാനത്തിനും,  പ്രതിരോധത്തെ പോലെ, ചിലപ്പോൾ പ്രത്യാക്രമണവും അനിവാര്യമായിത്തീരും. മദീനാ നാടിനെ തകർക്കാൻ വന്നവരെ പ്രതിരോധിക്കുകയായിരുന്നു ഉഹ്ദിലും ഖൻദഖിലും പ്രവാചകൻ(സ) ചെയ്തത്. തങ്ങളുടെ നാട് തകർക്കാനായി വരുന്നവരെ സ്വദേശത്ത് വെച്ച് പ്രതിരോധിക്കുകയെന്ന തന്ത്രം തുടർന്നാൽ അത് നാടിന്റെയും നാട്ടുകാരുടെയും നാശത്തിലാണ് കലാശിക്കുകയെന്ന് പ്രവാചകനിലെ രാഷ്ട്രനേതാവ് മനസ്സിലാക്കി. പതിനായിരത്തോളം വരുന്ന സഖ്യകക്ഷീസൈന്യത്തിനെങ്ങാനും ദിവസങ്ങളെടുത്ത് മദീനാപ്രവാസികൾ നിർമിച്ച കിടങ്ങ് കടന്ന് മദീനയെ ആക്രമിക്കാൻ കഴിഞ്ഞിരുന്നെങ്കിൽ മദീനാനാട് തന്നെ ഇല്ലാതാകുമായിരുന്നു. അവിടെയുള്ള പുരുഷന്മാരെല്ലാം കൊല്ലപ്പെടുകയും സ്ത്രീകൾ വെപ്പാട്ടികളായിതത്തീർന്ന് അടിമച്ചന്തകളിൽ വില്‍ക്കപ്പെടുകയും ചെയ്യുമായിരുന്നു. അല്ലാഹുവിന്റെ അപാരമായ അനുഗ്രഹത്താലാണ് മദീന രക്ഷപ്പെട്ടത്. ഇനിയും ശത്രുക്കൾക്ക് മദീനയെ വളഞ്ഞ് അതിനെ നശിപ്പിക്കാൻ അവസരം കൊടുത്തുകൂടെന്ന് യുദ്ധതന്ത്രജ്ഞനായ ആ രാഷ്ട്രനേതാവ് തീരുമാനിച്ചു. ഖന്‍ദഖില്‍വെച്ച് സഖ്യകക്ഷികള്‍ നിരാശയോടെ തിരിച്ചുപോയതിനുശേഷമുള്ള പ്രവാചകന്റെ (സ) പ്രഖ്യാപനത്തിൽ കാണാൻ കഴിയുന്നത് നാടിനെ തകർക്കാൻ വരുന്നവരെ അവരുടെ നാടുകളിൽ പോയി പ്രത്യാക്രമിക്കുകയെന്ന രാഷ്ട്രതന്ത്രമാണ്. അദ്ദേഹം പറഞ്ഞു: "ഇനി മുതല്‍ നാം അവരോട് യുദ്ധം ചെയ്യും. അവര്‍ നമ്മോട് യുദ്ധത്തിനൊരുമ്പെടുകയില്ല. (സ്വഹീഹുല്‍ ബുഖാരി കിത്താബുല്‍ മഗാസി)

രാഷ്ട്രതന്ത്രജ്ഞനായ നേതാവിന്റെ പ്രഖ്യാപനം. ശത്രുക്കള്‍ മദീനാ രാജ്യത്തേക്ക് കടന്നുവന്ന് അവിടെയുള്ളവരെ കൊന്നൊടുക്കുകയും രാജ്യസമ്പത്ത് കൊള്ളയടിക്കുകയും നാടിനെ തകര്‍ക്കുകയും ചെയ്യുന്നതിന് സമ്മതിക്കാതെ അവരുടെ കേന്ദ്രങ്ങളില്‍ പോയി ആക്രമിക്കുകയും മദീനയുടെ സുരക്ഷിതത്വം ഉറപ്പുവരുത്തുകയും ചെയ്യുകയെന്ന തന്ത്രമാണ് ഇതിനുശേഷമുള്ള പ്രവാചക യുദ്ധങ്ങളിലെല്ലാം കാണാന്‍ കഴിയുന്നത്. പ്രത്യാക്രമണത്തിലൂടെ സ്വന്തം നാടിനെ പ്രതിരോധിക്കുകയെന്ന തന്ത്രം. അതിന്ന് ആദ്യമായി സ്വന്തം നാടിനകത്തെ ആന്തരികശത്രുക്കളെ മാതൃകാപരമായി ഉന്‍മൂലനം ചെയ്യേണ്ടതുണ്ടായിരുന്നു, പ്രവാചകന്. അനുസരണയുള്ള പ്രജകളായി ജീവിക്കാമെന്ന് ആവര്‍ത്തിച്ച് കരാര്‍ ചെയ്യുകയും യുദ്ധസന്ദര്‍ഭത്തില്‍ കൂറുമാറുകയും ചെയ്ത ബനൂഖുറൈദക്കാരെ ഉന്‍മൂലനം ചെയ്തുകൊണ്ട് ആന്തരികശത്രുക്കള്‍ക്കെതിരെ കര്‍ശനമായ നടപടികളാണ് സ്വീകരിക്കപ്പെടുകയെന്ന് കപടവിശ്വാസികളടക്കമുള്ള മദീന പൗരന്‍മാരെ ബോധ്യപ്പെടുത്തുകയാണ് പ്രവാചകൻ(സ) ഖന് ദഖ്‌ യുദ്ധത്തിന് ശേഷം ആദ്യമായി ചെയ്തത്. .

മദീനാരാഷ്ട്രത്തെ തകര്‍ക്കാന്‍ ഗൂഢാലോചനകള്‍ നടത്തുന്നവരോട് അവര്‍ ഇങ്ങോട്ട് ആക്രമിക്കുന്നതിന് മുമ്പ് അവരുടെ കേന്ദ്രങ്ങളില്‍ പോയി ആക്രമിക്കുകയെന്ന പ്രതിരോധ തന്ത്രം പ്രഖ്യാപിക്കപ്പെട്ടതിനു ശേഷവും ശാന്തിയും സമാധാനവും നിലനിര്‍ത്താന്‍ ഏതുവിധത്തിലുള്ള വിട്ടുവീഴ്ചയ്ക്കും പ്രവാചകന്‍ സന്നദ്ധനായിരുന്നുവെന്ന് വ്യക്തമാക്കുന്നതാണ് ഹുദൈബിയ സന്ധി. സത്യനിഷേധികള്‍ സമാധാനത്തിലേക്ക് ചായവ് കാണിച്ചാല്‍ പ്രവാചകനും(സ) സമാധാനത്തിന് സന്നദ്ധനാകണമെന്ന ക്വുര്‍ആന്‍ നിര്‍ദേശം പ്രയോഗവത്ക്കരിക്കപ്പെടുകയാണ് ഹുദൈബിയ സന്ധിയില്‍ ഉണ്ടായത്. ഹിജ്‌റ ആറാം വര്‍ഷത്തില്‍ ഉംറ നിര്‍വഹിക്കാനായി മക്കയിലേക്ക് പുറപ്പെട്ട പ്രവാചകനും ആയിരത്തിഅഞ്ഞൂറോളം വരുന്ന അനുചരന്‍മാരും വഴിയിലുള്ള ഹുദൈബിയയില്‍ വെച്ച് തടയപ്പെട്ടതിനോടനുബന്ധിച്ച് ഉണ്ടാക്കിയ കരാറാണ് ഹുദൈബിയ സന്ധി എന്നറിയപ്പെടുന്നത്. 'പത്തുവര്‍ഷം ഇരുവിഭാഗവും പരസ്പരം യുദ്ധം ചെയ്യാന്‍ പാടില്ല'യെന്ന സമാധാനക്കരാറിനുവേണ്ടി ഏകപക്ഷീയമായി ഖുറൈശികള്‍ക്ക് അനുകൂലമെന്ന് ഒറ്റനോട്ടത്തില്‍ തോന്നുന്ന, പ്രവാചകാനുചരന്‍മാരടക്കം പ്രതിഷേധിച്ച കരാര്‍ വ്യവസ്ഥകള്‍ അംഗീകരിക്കാന്‍ മുഹമ്മദ് നബി(സ) സന്നദ്ധനായി. ഉമറിെേന  പോലെയുള്ള പ്രവാചകാനുചരന്‍മാര്‍ പോലും ശക്തമായ പ്രതിഷേധമറിയിച്ച കരാറിനെക്കുറിച്ചാണ് 'തീര്‍ച്ചയായും നിനക്ക് പ്രത്യക്ഷമായ ഒരു വിജയം നല്‍കിയിരിക്കുന്നു'(48:1)വെന്ന് അല്ലാഹു പറഞ്ഞതെന്ന കാര്യം പ്രത്യേകം ശ്രദ്ധിക്കുക.

ഹുദൈബിയ സന്ധി സൃഷ്ടിച്ച സമാധാനകാലത്താണ് അയല്‍നാടുകളിലെ ഭരണാധികാരികള്‍ക്ക് അവരെ ഇസ്‌ലാമിലേക്ക് ക്ഷണിച്ചുകൊണ്ട് പ്രവാചകൻ(സ)  കത്തുകളയച്ചത്; അയല്‍പ്രദേശങ്ങളിലേക്ക് ആദര്‍ശപ്രബോധനത്തിനായി അനുചരന്‍മാരെ പറഞ്ഞയച്ചതും ഇക്കാലത്തുതന്നെ. മക്കാ മുശ്‌രിക്കുകളുമായി സമാധാന സന്ധിയുണ്ടാക്കിയശേഷം വഞ്ചകരായ ജൂതന്‍മാര്‍ക്കെതിരെ ശക്തമായ നടപടികള്‍ സ്വീകരിക്കുകയാണ് പ്രവാചകൻ(സ)ചെയ്തത്. മദീനാ രാഷ്ട്രത്തില്‍നിന്ന് പുറത്താക്കപ്പെട്ടതിനുശേഷം ഖൈബറില്‍ ഒരുമിച്ച് ചേര്‍ന്ന് മദീനയെയും ഇസ്‌ലാമിക സമൂഹത്തെയും തകര്‍ക്കാന്‍ ശ്രമിച്ചുകൊണ്ടിരുന്ന ജൂതന്‍മാര്‍ക്കെതിരെ 1600 പേര്‍ അടങ്ങിയ സൈന്യത്തോടെ നടത്തിയ പടയോട്ടമാണ്' അവരുടെ നാടുകളില്‍ പോയി നാം യുദ്ധം ചെയ്യും' എന്ന പ്രവാചക പ്രഖ്യാപനത്തിന്റെ ആദ്യത്തെ പ്രയോഗവത്ക്കരണം. ഖൈബറില്‍ വച്ചു നടന്ന ശക്തമായ യുദ്ധത്തില്‍ മുസ്‌ലിംകള്‍ വിജയിച്ചു. 93 ജൂതന്‍മാരും 15 മുസ്‌ലിംകളും മരണപ്പെട്ട യുദ്ധത്തോടെ ഖൈബര്‍ മുസ്‌ലിംകളുടെ അധീനതയില്‍ വന്നുവെങ്കിലും അവിടുത്തെ യഹൂദന്‍മാരുടെ ആവശ്യങ്ങള്‍ അംഗീകരിക്കാന്‍ നബി (സ) സന്നദ്ധനായി. ഫലഭൂയിഷ്ടമായ ഖൈബറിലെ ഭൂമി പരിപാലിക്കുകയും കൃഷി ചെയ്തു വരുമാനമുണ്ടാക്കുകയും ചെയ്യാന്‍ തങ്ങളെ അനുവദിക്കണമെന്ന യഹൂദ ആവശ്യം ഉത്പന്നത്തിന്റെ പകുതി മുസ്‌ലിംകളുമായി പങ്കുവയ്ക്കണമെന്ന വ്യവസ്ഥയില്‍ അംഗീകരിക്കപ്പെട്ടു. ഖൈബറില്‍നിന്ന് പാഠമുള്‍ക്കൊണ്ട ഫദക്വിലെയും തയ്മാഇലെയും ജൂതന്‍മാര്‍ നബി(സ)യുമായി സമാധാനക്കരാറിലേര്‍പ്പെട്ടപ്പോള്‍ വാദില്‍ ഖുറയിലെ ജൂതന്‍മാര്‍ അക്രമത്തിന്റെ മാര്‍ഗമാണ് കൈക്കൊണ്ടത്. അതുകൊണ്ടുതന്നെ അവര്‍ക്കെതിരെ സൈനിക നടപടിയുണ്ടായി; അവര്‍ പരാജയപ്പെട്ടപ്പോള്‍ സ്വന്തം ഭൂമിയില്‍ കൃഷി ചെയ്യാന്‍ തങ്ങളെ അനുവദിക്കണമെന്ന അവിടെയുള്ള ജൂതന്മാരുടെ ആവശ്യവും അംഗീകരിക്കുകയാണ് പ്രവാചകൻ(സ)ചെയ്തത്.

റോമിലെ കൈസര്‍, സിറിയയിലെ കിസ്‌റ, അബ്‌സീനിയയിലെ നജ്ജാശി, ഈജിപ്തിലെ മുഖൗഖിസ്, ദമസ്‌കസിലെ ഹാരിസ്, ബഹ്‌റൈനിലെ മുന്‍ദിര്‍, സിറിയന്‍ അതിര്‍ത്തിയിലെ ഗസ്സാന്‍, ബുസ്‌റായിലെ അമീര്‍, യമാമയിലെ ഹൗദ, ഒമാനിലെ അമീര്‍ എന്നിവര്‍ക്ക് പ്രവാചകൻ(സ)അയച്ച ഇസ്‌ലാമിലേക്ക് ക്ഷണിച്ചുകൊണ്ടുള്ള കത്തുകള്‍ക്ക് വ്യത്യസ്ത രീതിയിലുള്ള പ്രതികരണങ്ങളാണുണ്ടായത്. റോമാ ചക്രവര്‍ത്തിയുടെ കീഴില്‍ ഭരണം നടത്തിയിരുന്ന ഗസ്സാന്‍ രാജാവ് പ്രവാചകൻ(സ)ന്റെ കത്തിനെ അവഹേളിക്കുകയും അതുമായി ചെന്ന ദൂതനെ വധിക്കുകയും ചെയ്തുകൊണ്ട് ഇസ്‌ലാമിനോടും പ്രവാചകനു (സ)മെതിരെയുള്ള ശാത്രവം പരസ്യമായി പ്രഖ്യാപിച്ചപ്പോള്‍ അദ്ദേഹത്തിനെതിരെ സൈനിക നടപടിയെടുക്കുകയല്ലാതെ നിര്‍വാഹമില്ലെന്ന അവസ്ഥയുണ്ടായി. ഹിജ്‌റ എട്ടാം വര്‍ഷം പ്രവാചകൻ(സ) പറഞ്ഞയച്ച മുവ്വായിരം മുസ്‌ലിംകള്‍ റോമാക്കാരുമായി നടത്തിയ പോരാട്ടമാണ് മുഅ്ത്വ യുദ്ധം എന്നറിയപ്പെടുന്നത്. മൂന്ന് സേനാനായകരടക്കം പന്ത്രണ്ട് പേര്‍ രക്തസാക്ഷികളായ ഈ യുദ്ധത്തിന്റെ ഗതി മുസ്‌ലിംകള്‍ക്ക് എതിരായി തീര്‍ന്നതിനാല്‍ സൈനിക നേതൃത്വമേറ്റെടുത്ത ഖാലിദുബ്‌നു വലീദ്‌ േതന്ത്രപരമായി യുദ്ധരംഗത്തുനിന്ന് പിന്‍മാറുകയാണ് ചെയ്തത്.

ഹുദൈബിയ സന്ധിയിലെ വ്യവസ്ഥകള്‍ പാലിക്കുന്നതില്‍ വീഴ്ചവരുത്തിയ മുശ്‌രിക്കുകള്‍ക്കെതിരെ നടന്ന സൈനിക നടപടിയാണ് മക്കാ വിജയമായി പരിണമിച്ചത്. ഹിജ്‌റ എട്ടാം വര്‍ഷം റമദാനില്‍ പതിനായിരത്തോളം വരുന്ന മുസ്‌ലിംകളുടെ സൈന്യം മക്കിയിലേക്ക് മാര്‍ച്ച് ചെയ്തു. ചരിത്രത്തില്‍ തുല്യതയില്ലാത്ത ഒരു രക്തരഹിത വിപ്ലവത്തിന് മക്ക വിധേയമായി. ഖുറൈശി നേതാവായിരുന്ന അബൂസുഫ്‌യാനടക്കം പലരും മുസ്‌ലിംകളായി. പ്രവാചകനെ പീഡിപ്പിക്കുകയും ജന്‍മനാട്ടില്‍നിന്ന് പുറത്താക്കുകയും അനുചരന്‍മാരെ ക്രൂരമായി കൊലപ്പെടുത്തുകയും ചെയ്തവര്‍ ഉള്‍ക്കൊള്ളുന്ന മക്കക്കാര്‍ക്ക് പ്രവാചകൻ(സ)ന്‍ല നിരുപാധികം മാപ്പു നല്‍കി. കഅ്ബാലയത്തിനകത്ത് പില്‍ക്കാലത്ത് സ്ഥാപിക്കപ്പെട്ട മുന്നൂറ്റി അറുപത് ശിലാവിഗ്രഹങ്ങളും നീക്കം ചെയ്ത് ശുദ്ധീകരിക്കുകയും സ്രഷ്ടാവിനെ മാത്രം ആരാധിക്കാനായി ആദ്യമായി നിര്‍മ്മിക്കപ്പെട്ട ഗേഹത്തില്‍നിന്ന് സൃഷ്ടിപൂജയുടെ ചിഹ്നങ്ങളെ പുറത്താക്കുകയും ചെയ്തു.(15) മക്കാ മണലാരണ്യത്തില്‍ വെച്ച് ക്രൂരമായി പീഡിപ്പിക്കപ്പെട്ട അടിമയായിരുന്ന ബിലാലിനോട് ഈ മഹാവിജയത്തിന്റെ പ്രഖ്യാപനമായി കഅ്ബാലയത്തിന് മുകളില്‍ കയറി ബാങ്ക് വിളിക്കാന്‍ ആവശ്യപ്പെട്ടുകൊണ്ട് മക്കാവിജയത്തെ സകലവിധ സങ്കുചിതത്വങ്ങളുടെയും അടിവേരറുക്കുന്നതിനുള്ള പ്രഖ്യാപനമാക്കിത്തീര്‍ത്തു. മക്കക്കാര്‍ക്കെല്ലാം പൊതുമാപ്പ് നല്‍കിയെങ്കിലും വഞ്ചകരും കൊടുംക്രൂരതകള്‍ കാണിച്ചവരുമായി ഏതാനും പേര്‍ക്കെതിരെ നടപടികളെടുത്തുകൊണ്ട് ചില കുറ്റകൃത്യങ്ങളോടുള്ള ഇസ്‌ലാമിന്റെ നിലപാട് കര്‍ക്കശമാണെന്ന് പ്രവാചകൻ(സ) മക്കാ വിജയത്തോടനുബന്ധിച്ച് വ്യക്തമാക്കി.

മക്കാ വിജയത്തോടനുബന്ധിച്ച് ഖുറൈശികള്‍ കീഴടങ്ങിയെങ്കിലും ശക്തരായ തങ്ങളെ കീഴടക്കാനാവില്ലെന്ന് അഹങ്കരിക്കുകയും നബി(സ)ക്കെതിരില്‍ പടയൊരുക്കം നടത്തുകയും ചെയ്ത ഹവാസീന്‍ ഗോത്രത്തെയും ഥഖീഫ് ഗോത്രത്തെയും പ്രതിരോധിക്കുന്നതിനുവേണ്ടിയാണ് ഹുനൈനില്‍ തുടങ്ങുകയും ത്വാഇഫില്‍ അവസാനിക്കുകയും ചെയ്ത യുദ്ധം നടന്നത്. മാലിക്കുബ്‌നു ഔഫിന്റെ നേതൃത്വത്തില്‍ വ്യത്യസ്തങ്ങളായ ഗോത്രങ്ങള്‍ സംഘടിച്ച് മുസ്‌ലിംകള്‍ക്കെതിരെ അണിനിരക്കുന്നുവെന്നറിഞ്ഞപ്പോഴാണ് പന്ത്രണ്ടായിരം പേരടങ്ങുന്ന വമ്പിച്ച ഒരു സൈന്യവുമായി മക്കാ വിജയം കഴിഞ്ഞ് രണ്ട് ആഴ്ചകള്‍ക്കുശേഷം പ്രവാചകൻ(സ) ഹുനൈനിലേക്ക് പുറപ്പെട്ടത്. എണ്ണപ്പെരുപ്പം സൃഷ്ടിച്ച മതിപ്പും ഇത്ര വലിയ സൈന്യത്തെ ജയിക്കാന്‍ ആര്‍ക്കും സാധിക്കില്ലെന്ന ആത്മസംതൃപ്തിയും അല്ലാഹുവിന്റെ പരീക്ഷണത്തിന് കാരണമായിയെന്ന് ക്വുര്‍ആന്‍ വ്യക്തമാക്കുന്നു.( 9:25) ഇടുങ്ങിയ മലമ്പാതകളിലൂടെ ഹുനൈനിലേക്ക് നീങ്ങിയ മുസ്‌ലിം സൈന്യത്തിന്റെ നേരെ താഴ്‌വരകളിലും കുന്നുകളിലും ഒളിഞ്ഞിരുന്ന ഒളിപ്പോരാളികള്‍ നടത്തിയ അപ്രതീക്ഷിതമായ മിന്നലാക്രമണം മുസ്‌ലിംകളുടെ അണിയെ ഛിന്നഭിന്നമാക്കി. മുസ്‌ലിംകള്‍ ചിന്നിച്ചിതറി നാലുഭാഗത്തേക്കും ഓടി. അവര്‍ക്കിടയിലൂടെ നടന്ന്'ഞാന്‍ നബിയാണ്; കള്ളനല്ല'; അബ്ദുല്‍മുത്തലിബിന്റെ പുത്രനാണ്' എന്ന് പ്രഖ്യാപിച്ചുകൊണ്ട് മുഹമ്മദ് നബി(സ) യുദ്ധരംഗം വിട്ടോടുന്നവരെ തിരിച്ചുവിളിച്ചു. അതോടെ അല്ലാഹുവിന്റെ അനുഗ്രഹത്താല്‍ മുസ്‌ലിംകള്‍ക്ക് ധൈര്യം ലഭിക്കുകയും യുദ്ധരംഗത്ത് സജീവമാകുകയും ചെയ്തു. അതോടെ ശത്രുക്കള്‍ തോറ്റോടി. ചിലര്‍ ത്വാഇഫിലേക്കാണ് ഓടിപ്പോയത്. അവിടെയുള്ള കോട്ടകള്‍ക്കകത്ത് അഭയം തേടിയവര്‍ക്കെതിരെ ശ്ക്തമായ നടപടികളുമായി പ്രവാചകനും പ്രവാചകൻ(സ) അനുചരന്‍മാരും മുന്നേറിയെങ്കിലും അവരെ കോട്ടകള്‍ക്കകത്തുനിന്ന് പുറത്തുകൊണ്ടുവരാന്‍ സാധിച്ചില്ല. ഹുെനെന്‍ യുദ്ധത്തില്‍ പിടിച്ചുവെക്കപ്പെട്ടവരെ തങ്ങള്‍ക്കുതന്നെ തിരിച്ചുതരണമെന്ന് യുദ്ധത്തില്‍ പരാജയപ്പെട്ട ഹവാസിന്‍ ഗോത്രക്കാര്‍ പ്രവാചകനുടുത്ത് വന്ന് ആവശ്യപ്പെട്ടപ്പോള്‍ പ്രവാചകൻ(സ)അത് അംഗീകരിക്കുകയാണ് ചെയ്തത്.

അറേബ്യന്‍ ഉപഭൂഖണ്ഡത്തിലെ പ്രധാനപ്പെട്ട പ്രദേശങ്ങളെല്ലാം ഇസ്‌ലാമിന് കീഴിലാവുമെന്ന് മനസ്സിലായപ്പോള്‍, അത് പ്രതിരോധിക്കാനായി റോമാ സാമ്രാജ്യം സായുധ ശേഖരണവും സൈനിക വിന്യാസവും നടത്തിയപ്പോള്‍ അവര്‍ക്കെതിരെ നടന്ന തബൂക്ക് യുദ്ധവും നാടിനെയും ആദര്‍ശത്തെയും സംരക്ഷിക്കുന്നതിനുവേണ്ടി നടന്നതായിരുന്നു. കഠിനമായ ചൂടുകാലത്താണ് മുപ്പതിനായിരം മുസ്‌ലിംകളുള്‍ക്കൊള്ളുന്ന പ്രവാചകന്റെ സൈന്യം തബൂക്കിലേക്ക് പോയത്. നിശ്ചയദാര്‍ഢ്യത്തോടെ വരുന്ന മുസ്‌ലിം സൈന്യത്തിന്റെ ഗതി കണ്ട് ഭയന്ന റോമന്‍ സൈന്യം തബൂക്കില്‍നിന്ന് പിന്‍മാറിയതിനാല്‍ യുദ്ധം നടന്നില്ല. പ്രവാചകൻ(സ) ന്‍നേരിട്ട് പങ്കെടുത്ത അവസാനത്തെ സൈനിക നീക്കവും കാര്യമായ ആള്‍നാശമൊന്നുമുണ്ടാകാതെ വിജയത്തിന്റെ പാതക ഉയര്‍ത്തുകയാണുണ്ടായത്-എന്നര്‍ഥം.

ആദര്‍ശത്തിന്റെ അടിസ്ഥാനത്തില്‍ സ്ഥാപിക്കപ്പെട്ട ഒരു രാഷ്ട്രത്തിന്റെ നേതാവ് എന്ന നിലയ്ക്കുള്ള തന്റെ ഉത്തരവാദിത്ത നിര്‍വഹണമാണ് പ്രവാചകൻ(സ) നയിച്ച യുദ്ധങ്ങളിലെല്ലാം നമുക്ക് കാണാന്‍ കഴിയുന്നത്. പൗരന്‍മാരുടെ ജീവനും സ്വത്തും അഭിമാനവും മതവും സംരക്ഷിക്കുക, നാടിന്റെ സുരക്ഷ ഉറപ്പുവരുത്തുക, ആന്തരിക ശത്രുക്കളെ ഇല്ലായ്മ ചെയ്യുക, ഇസ്‌ലാമികാദര്‍ശമനുസരിച്ച് ജീവിക്കാന്‍ സ്വാതന്ത്ര്യമുള്ള അവസ്ഥ എല്ലായിടത്തും നിലനില്‍ക്കുന്നുണ്ട് എന്ന് ഉറപ്പുവരുത്തുക എന്നിങ്ങനെ ഒരു ഭരണാധികാരിയുടെ ഉത്തരവാദിത്തങ്ങളുടെ നിര്‍വഹണത്തിന് വേണ്ടിയാണ് പ്രവാചകൻ(സ) യുദ്ധം ചെയ്തത്. മക്കയിലും മറ്റ് പ്രദേശങ്ങളിലുമെല്ലാം പീഡനങ്ങളനുഭവിക്കുന്ന മുസ്‌ലിംകളുടെ മോചനവും അവര്‍ക്ക് സ്വതന്ത്രമായി ഇസ്‌ലാം അനുധാവനം ചെയ്യാന്‍ കഴിയുന്ന അവസ്ഥയുടെ സംസ്ഥാപനവും പ്രവാചക യുദ്ധങ്ങളുടെ ലക്ഷ്യങ്ങളിലൊന്നായിരുന്നു.

ല്ല. അധികാരമോഹമായിരുന്നില്ല അധികാരമേല്പിക്കപ്പെട്ട നാടിന്റെ സംരക്ഷണമെന്ന ഉത്തരവാദിത്തത്തിൻറെ നിർവഹണമായിരുന്നു ബദർ യുദ്ധത്തിന് കാരണമായിത്തതീർന്നത്.

മുഹമ്മദ് നബി(സ) നേതൃത്വം നല്‍കിയ ആദ്യത്തെ പോരാട്ടമാണ് ബദ്‌റ് യുദ്ധം. തികച്ചും ആകസ്മികമായി ഉണ്ടായ ഒരു യുദ്ധമായിരുന്നു അത്. മദീനയിലെ ഇസ്‌ലാമിക സമൂഹത്തിന്റെ ഭാഗമാകാന്‍ സമ്മതിക്കാതെ മക്കയില്‍ ക്രൂരമായി പിടിച്ചുവെക്കപ്പെട്ട വിശ്വാസികളുടെ മോചനം, പ്രവാചകനെയും അനുയായികളെയും നശിപ്പിക്കുകയും മദീനയില്‍ വളര്‍ന്നുകൊണ്ടിരിക്കുന്ന മുസ്‌ലിം സമൂഹത്തെ തകര്‍ക്കുകയും ചെയ്യുമെന്ന മക്കാമുശ്‌രിക്കുകളുടെ ഭീഷണിക്ക് അറുതിവരുത്തുക, മദീനക്ക് ചുറ്റുമുള്ള അറബ് ഗോത്രങ്ങളുടെ അക്രമങ്ങളില്‍നിന്ന് നാടിനെ രക്ഷിക്കുകയും അവരുമായി സമാധാനസന്ധിയുണ്ടാക്കുകയും ചെയ്യുക. ആദര്‍ശമനുസരിച്ച് ജീവിക്കുവാനുള്ള സ്വാതന്ത്ര്യത്തിനുവേണ്ടി നാടുവിട്ടപ്പോള്‍ മുസ്‌ലിംകള്‍ മക്കയില്‍ ഉപേക്ഷിച്ചുപോന്ന സമ്പത്ത് അതിന്റെ അവകാശികള്‍ക്ക് തിരിച്ച് നല്‍കുന്നതിനുവേണ്ടി പരിശ്രമിക്കുക തുടങ്ങിയ ലക്ഷ്യങ്ങള്‍ക്കുവേണ്ടി മുഹമ്മദ് നബി(സ)യുടെ നേരിട്ടുള്ള നേതൃത്വത്തിലും അല്ലാതെയും നടന്ന സൈനിക നീക്കങ്ങളിലൊന്നാണ് ബദ്‌റ് യുദ്ധമായി പരിണമിച്ചത്.

മക്കയില്‍ മുശ്‌രിക്കുകള്‍ നയിക്കുന്ന വ്യാപാര സംഘങ്ങളെ തടയുകയും നടേ പറഞ്ഞ ലക്ഷ്യങ്ങള്‍ നേടുകയും ചെയ്യുകയെന്ന ഉദ്ദേശത്തോടെ ഹിജ്‌റക്കുശേഷം ഏഴാം മാസം നടത്തിയ സൈഫില്‍ ബഹ്‌റ് മുതലുള്ള സൈനിക നീക്കങ്ങളെപോലെയുള്ള ഒരു നീക്കം മാത്രമായിരുന്നു നബി(സ)യും സ്വഹാബിമാരും ബദ്‌റിലേക്ക് നടത്തിയത്. അതുകൊണ്ടുതന്നെ മദീനയിലുണ്ടായിരുന്ന സ്വഹാബിമാരില്‍ പലരും ബദ്‌റിലേക്ക് പോവുകയോ യുദ്ധത്തില്‍ പങ്കെടുക്കുകയോ ചെയ്തിരുന്നില്ല.(1) മുന്നൂറ്റിപ്പത്തില്‍പരം പേര്‍ മുഹമ്മദ് നബി(സ)യോടൊപ്പം ബദ്‌റില്‍ പങ്കെടുത്തുവെന്ന് ബുഖാരിയും(2) അവരുടെ എണ്ണം എണ്ണൂറ്റിപത്തൊന്‍പതായിരുന്നുവെന്ന് മുസ്‌ലിമും(3) നിവേദനം ചെയ്തിട്ടുണ്ട്. യുദ്ധത്തിന് ഒരുങ്ങിപോയവരല്ലാത്തതിനാല്‍തന്നെ അവരുടെ പക്കല്‍ ആവശ്യത്തിനുള്ള ആയുധങ്ങളൊന്നും ഉണ്ടായിരുന്നില്ല. അവര്‍ക്ക് പോരാടാനുണ്ടായിരുന്നതാകട്ടെ സര്‍വ്വായുധ സന്നദ്ധരായി യുദ്ധത്തിന് ഒരുങ്ങിവന്ന(4) യുദ്ധസജ്ജരും നിപുണരുമായ ഖുറൈശികളോടായിരുന്നു. രണ്ട് കുതിരകളും എഴുപത് ഒട്ടകങ്ങളും മുന്നൂറിലധികം പേരുമടങ്ങുന്ന മുഹമ്മദ് നബി(സ)യുടെ നേതൃത്വത്തിലുള്ള മുസ്‌ലിം സൈന്യവും നൂറിലധികം കുതിരകളും നിരവധി ഒട്ടകങ്ങളും ആയിരത്തിമുന്നൂറോളം വരുന്ന സായുധസജ്ജരുമടങ്ങുന്ന അബൂജഹ്‌ലിന്റെ നേതൃത്വത്തിലുള്ള ഖുറൈശീ സൈന്യവും തമ്മിലാണ് ബദ്‌റില്‍ ഏറ്റുമുട്ടിയത്.(5) പക്ഷെ, ഇസ്‌ലാമിക ചരിത്രത്തിലെ പ്രഥമ യുദ്ധത്തില്‍ മുസ്‌ലിംകള്‍ വിജയിച്ചു. ബഹുദൈവാരാധകരിലെ നേതാക്കളായ അബൂജഹ്‌ലും ഉമയ്യത്തുബിനു ഖലഫുമടക്കമുള്ള എഴുപത് പേര്‍ മരണപ്പെടുകയും എഴുപത് പേരെ മുസ്‌ലിംകള്‍ ബന്ദികളായി പിടിക്കുകയും െചയ്തപ്പോള്‍(6) പതിനാല് മുസ്‌ലിംകളാണ് ബദ്‌റ് യുദ്ധത്തില്‍ രക്ഷസാക്ഷികളായത്.(7) ചെറിയൊരു സംഘം അല്ലാഹുവിന്റെ സഹായത്താല്‍ വലിയൊരു സംഘത്തെ ജയിച്ചടക്കി. ബദ്‌റിലുണ്ടായ അല്ലാഹുവിന്റെ സഹായത്തെക്കുറിച്ച് വ്യക്തമാക്കുന്ന ക്വുര്‍ആന്‍ സൂക്തങ്ങള്‍ കാണുക: ''നിങ്ങള്‍ ദുര്‍ബലരായിരിക്കെ ബദ്‌റില്‍ വെച്ച് അല്ലാഹു നിങ്ങളെ സഹായിച്ചിട്ടുണ്ട്. അതിനാല്‍ നിങ്ങള്‍ അല്ലാഹുവെ സൂക്ഷിക്കുക. നിങ്ങള്‍ നന്ദിയുള്ളവരായേക്കാം. നബി(സ)യേ നിങ്ങളുടെ രക്ഷിതാവ് മൂവായിരം മലക്കുകളെ ഇറക്കികൊണ്ട് നിങ്ങളെ സഹായിക്കുക എന്നത് നിങ്ങള്‍ക്ക് മതിയാവുകയില്ലേ എന്ന് നീ സത്യവിശ്വാസികളോട് പറഞ്ഞിരുന്ന സന്ദര്‍ഭം (ഓര്‍ക്കുക)'' (3:123,124)

ആദര്‍ശത്തിന്റെ സംരക്ഷണത്തിനായി അന്തിമ പ്രവാചകനും അനുയായികളും തോളോട് തോളുരുമ്മി പടവെട്ടിയപ്പോള്‍ അവിടെ അല്ലാഹുവിന്റെ സഹായമുണ്ടായി. അല്ലാഹുവിന്റെ തൃപ്തി മാത്രം കാംക്ഷിച്ചുകൊണ്ടാണ് മുസ്‌ലിംകള്‍ പടവെട്ടിയതെന്ന വസ്തുത വ്യക്തമാക്കുന്നതാണ് നടേ വിവരിച്ച സംഭവം. അധികാരവും യുദ്ധാര്‍ജ്ജിത സമ്പത്തും അങ്ങനെ ലഭിക്കുന്ന അടിമപ്പെണ്‍കൊടികളുമൊത്തുള്ള ശയനവുമായിരുന്നു മുസ്‌ലിംകളെ യുദ്ധത്തിന് പ്രചോദിപ്പിച്ചതെന്ന വിമര്‍ശനത്തിന്റെ നട്ടെല്ലൊടിക്കുന്ന ഇത്തരം നിരവധി സംഭവങ്ങള്‍ നിവേദനം ചെയ്യപ്പെട്ടിട്ടുണ്ട്. അല്ലാഹുവിന്റെ പ്രീതി ആഗ്രഹിച്ചുകൊണ്ടും അവന്റെ സഹായം പ്രതീക്ഷിച്ചുകൊണ്ടും വിശ്വാസികള്‍ പടപൊരുതിയപ്പോള്‍ അല്ലാഹു അവരെ സഹായിച്ചു. യുദ്ധത്തില്‍ മുസ്‌ലിംകള്‍ വിജയിച്ചത് അല്ലാഹുവിന്റെ സഹായം കൊണ്ടാണ്. കൂടാരത്തില്‍ പ്രാര്‍ഥനാനിമഗ്‌നനായിരുന്ന പ്രവാചകൻ(സ) ഇടയ്ക്ക് പുറത്തുവന്നുകൊണ്ട് അബൂബക്കറിനോട്‌ (റ)പറഞ്ഞു: 'സന്തോഷവാര്‍ത്തയുണ്ട് അബൂബക്ര്‍. അല്ലാഹുവിന്റെ സഹായം വന്നെത്തിയിരിക്കുന്നു. തലപ്പാവ് വെച്ചുകൊണ്ട് തന്റെ കുതിരയെ നയിച്ച് ഇതാ ജിബ്‌രീല്‍ എത്തിക്കഴിഞ്ഞു. അദ്ദേഹത്തിന്റെ കുതിരയുടെ പല്ലില്‍ പൊടി പിടിച്ചിട്ടുണ്ട്. അല്ലാഹു വാഗ്ദാനം ചെയ്ത സഹായം ഇതാ വന്നെത്തിയിരിക്കുന്നു'(8)

അധികാരമോഹവും അധീശത്വ സ്വപ്നങ്ങളുമാണ് മുഹമ്മദ് നബി(സ)യെ ആയുധമെടുക്കുവാന്‍ പ്രചോദിപ്പിച്ചത് എന്ന് പറയുന്നവര്‍ക്ക് ബദ്‌റില്‍വെച്ച് അദ്ദേഹം മറ്റെന്ത് ചെയ്യണമെന്നാണ് നിര്‍േദശിക്കുവാനുള്ളതെന്ന് അറിയുവാന്‍ കൗതുകമുണ്ട്. മുസ്‌ലിംകള്‍ക്കെതിരെ പടപൊരുതിക്കൊണ്ട് തങ്ങളുടെ യശസ്സ് അറബ് ലോകത്തെ അറിയിക്കണമെന്ന് അഹങ്കരിക്കുന്ന അബൂജഹ്‌ലിന്റെ നേതൃത്വത്തിലുള്ള ഒരു വന്‍സൈന്ന്യം അതിക്രമങ്ങള്‍ക്കൊരുങ്ങി ബദ്‌റില്‍ തമ്പടിച്ചപ്പോള്‍ അവര്‍ക്കെതിരെ സായുധസന്നാഹം നടത്താതിരിക്കണമെന്ന് പറയുവാന്‍ രാഷ്ട്രതന്ത്രത്തിന്റെ ബാലപാഠമെങ്കിലും അറിയുന്നവര്‍ സന്നദ്ധമാകുമോ? അത് നടന്നിട്ടില്ലായിരുന്നുവെങ്കില്‍ പ്രസ്തുത സൈന്യം മദീനയിലേക്ക് മാര്‍ച്ച് ചെയ്യുകയും മദീനാപട്ടണം തന്നെ നാമാവശേഷമാകുന്ന സ്ഥിതി ഉണ്ടാവുകയും ചെയ്യുമായിരുന്നുവെന്ന് സംഭവങ്ങളെ നിരീക്ഷിക്കുന്ന ആര്‍ക്കാണ് മനസിലാക്കാന്‍ കഴിയാത്തത്! യുദ്ധത്തിന് ഒരുങ്ങിവന്ന ശത്രുക്കളില്‍നിന്ന് നാടിനെയും നാട്ടുകാരെയും സംരക്ഷിക്കുകയും അങ്ങനെ സമാധാനപൂര്‍ണമായി ഇസ്‌ലാമിക ജീവിതം നയിക്കുവാനും അതിലേക്ക് ആളുകളെ ക്ഷണിക്കുവാനുമുള്ള സാഹചര്യങ്ങള്‍ ഉണ്ടാക്കുകയും ചെയ്യുവാന്‍ വേണ്ടിയാണ് ബദ്‌റ് യുദ്ധം നടത്തണമെന്ന വസ്തുത സത്യസന്ധമായി ചരിത്രപഠനം നിര്‍വഹിക്കുന്നവര്‍ക്കൊന്നും നിഷേധിക്കാനാവില്ല, തീര്‍ച്ച.

  1. സ്വഹീഹുല്‍ ബുഖാരി, കിത്താബുല്‍ മഗാസി
  2. സ്വഹീഹുല്‍ ബുഖാരി, കിത്താബുല്‍ മഗാസി
  3. സ്വഹീഹു മുസ്‌ലിം, കിത്താബുല്‍ ജിഹാദ്, വസ്സിയീര്‍
  4. സ്വഹീഹു മുസ്‌ലിം, കിത്താബുല്‍ ജിഹാദ്, വസ്സിയീര്‍
  5. ഇമാം ഇബ്‌നുകഥീര്‍ തന്റെ അല്‍ ബിദായയില്‍ (3/284-285) മുര്‍സലായി നിവേദനം ചെയ്തത്. Mahdi Rizqulla Ahmed: A Biography of the Prophet of Islam; Riyadh 2005, page 391
  6. സ്വഹീഹു മുസ്‌ലിം, കിത്താബുല്‍ ജിഹാദ് വസ്സിയീര്‍
  7. ഇമാം ഇബ്‌നു കഥീര്‍ അല്‍ ബിദായയില്‍ (3/230) മൂസബ്‌നു ഉഖ്ബയില്‍ (റ) നിന്ന് നിവേദനം ചെയ്തത് M.R. Ahmad Opt.cit page 414
  8. സ്വഹീഹു മുസ്‌ലിം, കിത്താബുല്‍ ജിഹാബു വസ്സയീര്‍
  9. ഇബ്‌നു കഥീര്‍ അല്‍ബിദായയില്‍ ഹസനായി നിവേദനം ചെയ്തത് (ഇമാം അല്‍ബാനി: ഫിഖ്ഹുസ്‌സീറക്ക് എഴുതിയ അടിക്കുറിപ്പുകള്‍ പുറം 243)

ഭരണാധികാരം ഒരു ഉത്തരവാദിത്തമാണെന്നും അതിന്റെ നിര്‍വഹണം എങ്ങനെയായിരുന്നുവെന്ന വിഷയത്തില്‍ ഭരണാധികാരികള്‍ അല്ലാഹുവിന്റെ മുമ്പില്‍ വിചാരണ നേരിടുമെന്നും പഠിപ്പിച്ച ആദർശമാണ് ഇസ്‌ലാം. സുഖിക്കുവാനുള്ള മാർഗ്ഗമല്ല, പ്രത്യുത അല്ലാഹുവിന്റെ മുന്നിൽ ഉത്തരം പറയേണ്ട വലിയ ഉത്തരവാദിത്തമാണ് മുസ്ലിംകളെ സംബന്ധിച്ചിടത്തോളം ഭരണാധികാരം. വലിയൊരു ഉത്തരവാദിത്തമാണ് അധികാരമെന്ന് പഠിപ്പിക്കപ്പെടുമ്പോള്‍ ഭരണനിര്‍വഹണരംഗത്തെ വ്യത്യസ്തങ്ങളായ സാഹചര്യങ്ങളില്‍ എങ്ങനെ പെരുമാറണമെന്നുകൂടി വ്യക്തമാക്കേണ്ട ചുമതല അങ്ങനെ പഠിപ്പിക്കുന്നവര്‍ക്കുണ്ട്. യുദ്ധം അനിവാര്യമാകുന്ന സാഹചര്യങ്ങളില്‍ അത് നടത്തേണ്ടതെങ്ങനെയെന്ന് അവസാനനാളു വരെയുള്ളവരെയെല്ലാം പഠിപ്പിക്കുകയെന്ന ദൗത്യം നിര്‍വഹിക്കേണ്ടയാളാണ് അന്തിമ പ്രവാചകനായ മുഹമ്മദ് നബി(സ).

തിന്മയെ കൈകൊണ്ട് തടുക്കാനാകുമെങ്കില്‍ അങ്ങനെതന്നെ ചെയ്യണമെന്ന് നിഷ്‌കര്‍ഷിച്ച പ്രവാചകന്‌(സ), ഒരു ഭരണാധികാരിയെന്ന നിലയില്‍ എങ്ങനെയാണ് തിന്മകളെ കൈകൊണ്ട് പ്രതിരോധിക്കേണ്ടത് എന്നുകൂടി പ്രായോഗികമായി പഠിപ്പിക്കേണ്ടതുണ്ട്. ഇസ്‌ലാമിക നിയമങ്ങള്‍ നടപ്പിലാക്കിക്കൊണ്ട് തന്റെ ഭരണപ്രദേശത്ത് നടക്കുന്ന തിന്മകളെ എങ്ങനെ കൈകൊണ്ട് തടുക്കാമെന്ന് കാണിച്ചുതന്നതോടൊപ്പംതന്നെ തന്റെ പൗരന്മാര്‍ക്കും നാടിനുമെതിരെയുള്ള അതിക്രമങ്ങളോട് എന്ത് നിലപാടാണ് സ്വീകരിക്കേണ്ടത് എന്നുകൂടി സൈനികനീക്കങ്ങളിലൂടെയും നേര്‍ക്കുനേരെയുള്ള പോരാട്ടങ്ങളിലൂടെയും അദ്ദേഹം  അനുയായികളെ തെര്യപ്പെടുത്തി. അങ്ങനെ, ജീവിതത്തിന്റെ എല്ലാ രംഗങ്ങളെയും മാനവവല്‍ക്കരിക്കേണ്ടത് എങ്ങനെയെന്ന് മുഹമ്മദ് നബി(സ) മനുഷ്യരെ പഠിപ്പിച്ചു. 'തീര്‍ച്ചയായും താങ്കള്‍ മഹത്തായ സ്വഭാവത്തിലാകുന്നു' (ക്വുര്‍ആന്‍ 68:4) എന്നും 'തീര്‍ച്ചയായും നിങ്ങള്‍ക്ക് അല്ലാഹുവിന്റെ ദൂതനില്‍ മഹത്തായ മാതൃകയുണ്ട്' (33:21) എന്നുമുള്ള അല്ലാഹുവിന്റെ വചനങ്ങളെ അന്വര്‍ഥമാക്കിക്കൊണ്ട് താനുള്‍ക്കൊള്ളുന്ന ആദര്‍ശത്തെയും പ്രസ്തുത ആദര്‍ശത്തിന്റെ അടിത്തറയില്‍ സ്ഥാപിക്കപ്പെട്ട രാഷ്ട്രസംവിധാനത്തെയും തകര്‍ക്കുവാനുദ്ദേശിച്ച് യുദ്ധത്തിന് വരുന്നവരോടും അതിന് കോപ്പുകൂട്ടുന്നവരോടും എങ്ങനെ പ്രതികരിക്കണമെന്ന വിഷയത്തിലുള്ള മഹത്തായ മാതൃകയാണ് താന്‍ നയിച്ച യുദ്ധങ്ങളിലൂടെയും സൈനിക നടപടികളിലൂടെയും പ്രവാചകന്‍ല മാനവരാശിക്ക് മുമ്പില്‍വെച്ചത്.

ദൈവിക മാര്‍ഗദര്‍ശനപ്രകാരം ജീവിച്ച് ശാന്തിയും മരണാനന്തരം ശാന്തിയുടെ ഭവനവും നേടിയെടുക്കുവാന്‍ ശ്രമിക്കുകയും അത് സഹജീവികള്‍ക്ക് പറഞ്ഞുകൊടുക്കുവാനായി ത്യാഗപരിശ്രമങ്ങളില്‍ ഏര്‍പ്പെടുകയും ചെയ്യുന്നവരെ ജീവിക്കുവാൻ അനുവദിക്കുകയില്ലെന്ന് നിശ്ചയിച്ചുറച്ച് അതിക്രമങ്ങളില്‍ ഏര്‍പ്പെടുന്നവരെ കൈകൊണ്ട് തടയേണ്ടതെങ്ങനെയെന്ന് പഠിപ്പിക്കുകവഴി ഒരു ഭരണാധികാരിയുടെ ദൗത്യനിര്‍വഹണം എങ്ങനെയെന്ന് മനസ്സിലാക്കിക്കൊടുക്കുകയായിരുന്നു അദ്ദേഹം. ശാന്തമായ ഭൗതികജീവിതം നയിച്ച് ശാന്തിയുടെ ശാശ്വത ഭവനത്തിലെത്തിച്ചേരുവാന്‍ ആത്മാര്‍ഥമായി പരിശ്രമിക്കുന്നവരുടെ മുമ്പില്‍ വിഘ്‌നങ്ങളുണ്ടായിക്കൂടായെന്ന് നിഷ്‌കര്‍ഷിക്കുകയും ഉണ്ടാകുന്ന വിഘ്‌നങ്ങള്‍ എങ്ങനെ നീക്കം ചെയ്യാമെന്ന് കാണിച്ചുകൊടുക്കുകയും ചെയ്യുകവഴി 'ലോകര്‍ക്ക് മുഴുവന്‍ കാരുണ്യമായിക്കൊണ്ടല്ലാതെ താങ്കളെ നാം നിയോഗിച്ചിട്ടില്ല' (21:107)യെന്ന അല്ലാഹുവിന്റെ പ്രഖ്യാപനം അന്വർത്ഥമാക്കുക കൂടിയാണ് മുഹമ്മദ് നബി(സ) ചെയ്തത്.

നിയമം നടപ്പാക്കുകയാണ് ഭരണാധികാരിയുടെ ധര്‍മം. പൗരന്മാര്‍ക്ക് അടിസ്ഥാനപരമായ അഞ്ച് അവകാശങ്ങളുണ്ടെന്നും അവ സംരക്ഷിക്കുവാനാവശ്യമായ നിയമങ്ങളാണ് ഇസ്‌ലാം മുന്നോട്ടുവെക്കുന്നതെന്നും ക്വുര്‍ആനിലും ഹദീഥുകളിലുമുള്ള മാര്‍ഗനിര്‍ദേശങ്ങളുടെ വെളിച്ചത്തില്‍ അവയില്‍നിന്ന് നിയമങ്ങള്‍ നിര്‍ധരിച്ച് വിശദീകരിച്ച കര്‍മശാസ്ത്ര പണ്ഡിതന്മാര്‍ വ്യക്തമാക്കിയിട്ടുണ്ട്. മതം, ജീവന്‍, മനസ്സ്, അഭിമാനം, സ്വത്ത് എന്നീ അഞ്ച് അടിസ്ഥാന കാര്യങ്ങളുടെ സംരക്ഷണമാണ് നിയമത്തിന്റെ ധര്‍മം (മഖാസിദശ്ശരീഅ:). മനുഷ്യനും അവന്റെ സ്രഷ്ടാവും തമ്മിലുള്ള ബന്ധമായിരിക്കണം അവന്റെ ജീവിത നിലപാടുകളെ മൊത്തത്തില്‍ നിര്‍ണയിക്കേണ്ടത് എന്നതുകൊണ്ടുതന്നെ ദൈവിക മതം അനുധാവനം ചെയ്യുവാനുള്ള അവന്റെ അവകാശവും സ്വാതന്ത്ര്യവുമാണ് നിയമംമൂലം സംരക്ഷിക്കപ്പെടേണ്ട ഒന്നാമത്തെ കാര്യമെന്ന് ഇസ്‌ലാം നിഷ്‌കര്‍ഷിക്കുന്നു. കൊല, പീഡനം, ഭീതി, പട്ടിണി എന്നിവയുടെ ഭീഷണിയില്ലാതെ ജീവിക്കുവാന്‍ ഓരോ പൗരനും കഴിയണമെന്നതാണ് രണ്ടാമത്തെ കാര്യം. ലഹരി ഉപയോഗിക്കുന്നതുമൂലമുണ്ടാകുന്ന മാനസിക സ്ഥിരതയില്ലായ്മ, കളവ്, പരദൂഷണം, തെറ്റായ ആരോപണങ്ങള്‍ തുടങ്ങിയവ മൂലമുണ്ടാകുന്ന മാനസികസംഘര്‍ഷം തുടങ്ങിയവ ഇല്ലാതെയാക്കുകയാണ് നിയമത്തിന്റെ മൂന്നാമത്തെ ധര്‍മം. അഭിമാനവും അസ്തിത്വവും സംരക്ഷിക്കപ്പെടും. ഒരു സാമൂഹ്യ സംവിധാനത്തിന്റെ നിലനില്‍പാണ് നിയമത്തിന്റെ നാലാമത്തെ ധര്‍മം. കൊള്ള, ചൂഷണം, തകര്‍ച്ച, വഞ്ചന എന്നിവയില്‍നിന്ന് മുക്തമായ ഒരു സാമ്പത്തികക്രമത്തിന്റെ നിലനില്‍പാണ് അഞ്ചാമത്തേത്. ഈ അഞ്ച് കാര്യങ്ങളും നിര്‍വഹിക്കുവാന്‍ പര്യാപ്തമായ, ക്വുര്‍ആനും പ്രവാചകചര്യയും വരച്ചുകാണിക്കുന്ന നിയമങ്ങള്‍ നടപ്പാക്കുകയാണ് ഭരണാധികാരിയുടെ ഉത്തരവാദിത്തം. പ്രമാണങ്ങളില്‍ നേര്‍ക്കുനേരെ പരാമര്‍ശിക്കപ്പെട്ടിട്ടുള്ള നിയമങ്ങള്‍ മനസ്സിലാക്കുവാനും ഇല്ലാത്തവ അവയുടെ വെളിച്ചത്തില്‍ നിര്‍ധരിച്ചെടുക്കുവാനുമുള്ള അറിവ് അയാള്‍ക്കില്ലെങ്കില്‍, ഇക്കാര്യത്തിന് പണ്ഡിതന്മാരുടെ സഹായത്തോടെയാണ് അയാള്‍ ഭരണം നിര്‍വഹിക്കേണ്ടത്.

പൗരന്മാരുടെ മതം, ജീവന്‍, മനസ്സ്, അഭിമാനം, സ്വത്ത് എന്നിവയുടെ സംരക്ഷണത്തിനാവശ്യമായ നിയമങ്ങള്‍ സ്വന്തം നാട്ടിലെ പൗരന്‍മാരുടെ മേല്‍ മാത്രമെ നടപ്പാക്കുവാന്‍ ഭരണാധികാരിക്ക് കഴിയുകയുള്ളൂ. ഇവയെ ചോദ്യം ചെയ്യുന്നത് മറ്റൊരു നാട്ടുകാരാണെങ്കില്‍ പ്രസ്തുത നാട്ടിലെ അധികാര സ്ഥാപനങ്ങളിലൂടെ മാത്രമെ പ്രശ്‌നം പരിഹരിക്കുവാന്‍ കഴിയുകയുള്ളൂ. ഇസ്‌ലാമിക സമൂഹത്തിലെ പൗരന്‍മാരുടെ മതവും ജീവനും മനസ്സമാധാനവും അഭിമാനവും സ്വത്തുമെല്ലാം അപകടപ്പെടുത്തുവാന്‍ അന്യനാട്ടില്‍ അവിടുത്തെ അധികാരസ്ഥാപനങ്ങളുടെ സമ്മതത്തോടെയോ നേതൃത്വത്തിലോ ശ്രമം നടക്കുന്നുണ്ടെന്ന് മനസ്സിലായാല്‍ അതിനെ പ്രതിരോധിച്ചുകൊണ്ട് പൗരന്‍മാരുടെ അടിസ്ഥാനാവകാശങ്ങള്‍ സംരക്ഷിക്കേണ്ടത് ഭരണാധികാരിയുടെ ഉത്തരവാദിത്തമാണ്. സമാധനപൂര്‍ണമായ മാര്‍ഗങ്ങള്‍ പരാജയപ്പെടുമ്പോള്‍ ഇക്കാര്യത്തിനുവേണ്ടി അനിവാര്യമെങ്കില്‍ ആയുധമെടുക്കണമെന്നുതന്നെയാണ് ഇസ്‌ലാം ഭരണാധികാരികളോട് അനുശാസിക്കുന്നത്. യുദ്ധം ജിഹാദായിത്തീരുന്നത് ഇത്തരം സാഹചര്യങ്ങളിലാണ്.

മുസ്‌ലിംകളുടെ മതത്തിനും ജീവനും മനസ്സമാധാനത്തിനും അഭിമാനത്തിനും സ്വത്തിനും വെല്ലുവിളി ഉയര്‍ത്തിക്കൊണ്ട് ശത്രുക്കള്‍ സമരസജ്ജരാകുമ്പോള്‍ അതിനെ പ്രതിരോധിച്ച് പൗരന്‍മാരുടെ അടിസ്ഥാന അവകാശങ്ങള്‍ സംരക്ഷിക്കുന്നതിനായി അവരുടെ ഭരണാധികാരിയാണ് യുദ്ധം ചെയ്യേണ്ടത്. നിയമനിര്‍വഹണത്തിലൂടെ ഭരിക്കുന്ന നാടിനകത്ത് അടിസ്ഥാനാവകാശങ്ങളുടെ സംരക്ഷണത്തിനാവശ്യമായ സംവിധാനമുണ്ടാക്കേണ്ട ഉത്തരവാദിത്തം ആരുടെ മേലാണോ ബാധ്യതയായിട്ടുള്ളത് അവരുടെമേല്‍തന്നെയാണ് അയല്‍നാട്ടില്‍നിന്ന് ഈ അവകാശങ്ങള്‍ ചോദ്യം ചെയ്യപ്പെടുമ്പോള്‍ അത് നീക്കുവാനാവശ്യമായ കാര്യങ്ങള്‍ നിര്‍വഹിക്കേണ്ട ഉത്തരവാദിത്തവുമുള്ളത്. അതിന് സമാധാനസന്ധിയാണ് ആവശ്യമെങ്കില്‍ അതും സായുധസമരം അനിവാര്യമാണെങ്കില്‍ അതും ചെയ്യേണ്ടത് ഭരണാധികാരിയാണ്. യുദ്ധമാണ് വേണ്ടതെന്ന് അയാള്‍ തീരുമാനിക്കുകയും സായുധ ജിഹാദിന് സന്നദ്ധമാകുവാന്‍ പൗരന്‍മാരോട് ആവശ്യപ്പെടുകയും ചെയ്താല്‍ അതിന് സന്നദ്ധമാവുകയാണ്  അതിന് സാധിക്കുന്നവരുടെ ബാധ്യത. ഭരണാധികാരി  നയിക്കുന്നതോ ആഹ്വാനം നടത്തുകയോ ചെയ്യുന്ന യുദ്ധത്തില്‍ അണിചേരുക മാത്രമാണ് പൗരന്‍മാര്‍ ചെയ്യുന്നത് എന്നര്‍ഥം.

സാമ്രാജ്യത്വ സംസ്ഥാപനത്തിനും അധീശത്വത്തിനും അങ്ങനെയുള്ള സുഖലോലുപമായ ജീവിതത്തിനും വേണ്ടിയുള്ള യുദ്ധമെന്ന സങ്കല്പത്തെ മാറ്റിയെഴുതിയ മുഹമ്മദ് നബി(സ), മാനവികതക്ക് ‌ വേണ്ടി എങ്ങനെ യുദ്ധം ചെയ്യാമെന്നാണ് ലോകത്തെ പഠിപ്പിച്ചത്. അടിച്ചമർത്തപ്പെട്ടവയുടെ വിമോചനത്തിനും ആദർശമനുസരിച്ച് ജീവിക്കുവാനുള്ള അവകാശത്തിനും വേണ്ടിയുള്ളതാണ് ഇസ്‌ലാം പഠിപ്പിക്കുന്ന യുദ്ധം. മനുഷ്യാവകാശങ്ങളുടെ സംസ്ഥാപനത്തിന് വേണ്ടി നടത്തുന്ന, ഇസ്‌ലാമിലെ യുദ്ധസങ്കൽപം പൂർണമായും മാനവികമാണെന്ന് പറയുന്നത് അത് കൊണ്ടാണ്.

മുസ്ലിംകളുടെ യുദ്ധവെറിയല്ല, ശത്രുക്കളുടെ ഇസ്ലാം വിരോധത്തിന്റെ ക്രൂരമുഖമാണ് കുരിശുയുദ്ധങ്ങളിലൂടെ ലോകം കണ്ടത്. ഇസ്‌ലാമിനെയും മുസ്ലിംകളെയും ഉന്മൂലനം ചെയ്യുകയെന്ന ലക്ഷ്യത്തോടോടെ ആയുധമെടുത്തവരെപ്പോലും മാനവികതയെന്തെന്ന് പഠിപ്പിക്കുവാൻ മുസ്ലിംകൾക്ക് ലഭിച്ച അവസരമായിരുന്നു കുരിശുയുദ്ധങ്ങൾ എന്ന് പറയുന്നതാവും ശരി. മതത്തിന്റെ പേരിൽ  ഏറ്റവും വലിയ നരനായാട്ട് നടന്നത് കുരിശു യുദ്ധങ്ങളോട് അനുബന്ധിച്ചായിരുന്നവെന്ന് സമ്മതിക്കുമ്പോൾ തന്നെ, അവയുടെ കാരണം തികച്ചും മതപരമായിരുന്നില്ലെന്ന വസ്തുത കൂടി മനസ്സിലാക്കേണ്ടതുണ്ട്. തകർന്നുകൊണ്ടിരുന്ന ബൈസന്റൈൻ സാമ്രാജ്യം മതത്തെ ഉപയോഗപ്പെടുത്തി തങ്ങളുടെ അധികാരം സംരക്ഷിക്കാൻ നടത്തിയ പരിശ്രമമായിരുന്നു കുരിശുയുദ്ധങ്ങളിൽ കലാശിച്ചത്. ബൈസന്റൈൻ സാമ്രാജ്യത്തെ സംരക്ഷിക്കേണ്ടത് തങ്ങളുടെ ഉതതരവാദിത്തമായിക്കരുത്തിയ കത്തോലിക്കാ സഭ മുസ്ലിംകൾക്കും മുസ്ലിം നാടുകൾക്കുമെതിരെ കൊലവിളി നടത്താൻ ക്രിസ്ത്യാനികളെ പ്രചോദിപ്പിക്കുകയായിരുന്നു.

ബൈസന്റൈന്‍ ചക്രവര്‍ത്തിയായിരുന്ന അലക്‌സിയോസ് ഒന്നാമന്‍, അന്നത്തെ മാര്‍പാപ്പയായിരുന്ന പോപ്പ് അര്‍ബന്‍ രണ്ടാമന് തന്നെ സഹായിക്കണമെന്ന് ആവശ്യപ്പെട്ട് എഴുതിയ കത്തില്‍ നിന്ന് തുടങ്ങുന്നു കുരിശുയുദ്ധങ്ങളുടെ ചരിത്രം. 1071 ആഗസ്ത് 26ന് നടന്ന മാന്‍സികെര്‍ട്ട് യുദ്ധത്തില്‍ സില്‍ജൂക്ക് തുര്‍ക്കികള്‍ ബൈസന്റയിന്‍ സൈന്യത്തെ തുരത്തുകയും അനത്തോളിയ, അര്‍മേനിയ തുടങ്ങിയ പ്രദേശങ്ങള്‍ പിടിച്ചെടുക്കുകയും ചെയ്തതിന്റെ പശ്ചാത്തലത്തിലുള്ളതായിരുന്നു പ്രസ്തുത കത്ത്. 1095 നവംബര്‍ 18 മുതല്‍ 28 വരെ ഫ്രാന്‍സിലെ ക്ലെര്‍മണ്ടില്‍ വെച്ചു നടന്ന സഭാധ്യക്ഷന്‍മാരുടെ സമ്മേളനത്തില്‍, 27ാം തിയതി പോപ്പ് തന്നെ സമ്മേളനത്തെ അഭിസംബോധന ചെയ്തുകൊണ്ട് മുസ്‌ലിംകള്‍ക്കെതിരെ നടത്തേണ്ട യുദ്ധത്തെക്കുറിച്ച് സംസാരിക്കുകയും സദസ്സിനെ ആവേശഭരിതരാക്കി പ്രചോദിപ്പിക്കുകയും ചെയ്തു. ക്രൈസ്തവരുടെ വിശുദ്ധ സ്ഥലങ്ങള്‍ ഭരിക്കുന്ന അറബികളും തുര്‍ക്കികളുമാകുന്ന മുസ്‌ലിംകള്‍ പ്രാകൃതരാണെന്നും അതുകൊണ്ടു തന്നെ അവരെ തുരത്തേണ്ടത് ലോകത്തെങ്ങുമുള്ള ക്രൈസ്തവരുടെ ഉത്തരവാദിത്തമാണെന്നും ഇത് ക്രിസ്തുവിന്റെ കല്‍പനയാണെന്നും പ്രാകൃതരായ മുസ്‌ലിംകള്‍ക്കെതിരെയുള്ള യുദ്ധത്തില്‍ മരണപ്പെടുന്നവര്‍ക്ക് ഉടനടി പാപമോചനം ലഭിക്കുമെന്നും പിശാചുക്കളെ (demons) ആരാധിക്കുന്നവരുടെ മേല്‍ സര്‍വശക്തനായ ദൈവെത്ത പൂജിക്കുന്നവരുടെ ആധിപത്യത്തിനു വേണ്ടിയുള്ള നമ്മുടെ യുദ്ധത്തില്‍ ദൈവസഹായമുണ്ടാകുമെന്നും സഹോദരങ്ങള്‍ക്കും ബന്ധുക്കള്‍ക്കുമെതിരെ യുദ്ധം ചെയ്തിരുന്നവരെല്ലാം അവ നിര്‍ത്തി കാടന്‍മാര്‍ക്കെതിരെ(barbarians)യുള്ള ഈ യുദ്ധത്തിനു വേണ്ടി ഐക്യപ്പെടണമെന്നും മാര്‍പാപ്പ ആഹ്വാനം ചെയ്തു.(1)

മാര്‍പാപ്പയുടെ പ്രസംഗത്തില്‍ നിന്ന് പ്രചോദനമുള്‍ക്കൊണ്ട് 1096 ആഗസ്ത് 15ന് ഫ്രാന്‍സില്‍ നിന്നും ജര്‍മനിയില്‍ നിന്നും ഇറ്റലിയില്‍ നിന്നുമായി സന്യാസിയായ പത്രോസിന്റെ (Peter the hermit) നേതൃത്വത്തില്‍ 35000ത്തോളം വരുന്ന വലിയൊരു ക്രൈസ്തവസംഘം കോണ്‍സ്റ്റന്റിനോപ്പിളിലേക്ക് മാര്‍ച്ച് ചെയ്തു. വഴിയില്‍ കണ്ടവരെയെല്ലാം കൊന്നൊടുക്കിയും സാധാരണക്കാരെ ക്രൂരമായി പീഡിപ്പിച്ചും കൊണ്ടാണ് കുരിശുപടയാളികള്‍ മുന്നോട്ടു നീങ്ങിയത്. മൈല്‍ഗ്രേഡില്‍ വെച്ച് ഹംഗറിക്കാരുമായി അവര്‍ ഏറ്റുമുട്ടി; സ്‌പ്രെയറിലും മോംസിലും മെയിന്‍സിലുമുള്ള ജൂതന്‍മാരെ കൂട്ടക്കൊല നടത്തി. 1092ലെ മലിക്ഷാ ഒന്നാമന്റെ മരണത്തിന് ശേഷം ശിഥിലമായ സില്‍ജൂക്ക് തുര്‍ക്കി സാമ്രാജ്യത്തിന്റെ പതിതാവസ്ഥയെ മുതലെടുത്ത് കുരിശ് യോദ്ധാക്കള്‍ മുസ്‌ലിം നാടുകളിലേക്ക് ഇരച്ചു കയറുകയും അവിടങ്ങളില്‍ ക്രൂരതകൊണ്ട് സംഹാരതാണ്ഡവമാടുകയും ചെയ്തു. ക്രിസ്താബ്ദം 638ല്‍ ഉമറുല്‍ ഫാറൂഖിന്റെ ഭരണകാലം മുതല്‍ മുസ്‌ലിംകള്‍ ഭരിച്ചിരുന്ന ജറൂസലേം അങ്ങനെ 1099 ജൂലൈ 15ന്, ഫാത്വമിയ്യാക്കളുടെ കൈകളില്‍ നിന്ന് കുരിശുയോദ്ധാക്കള്‍ പിടിച്ചടക്കി.(2)

ഒന്നാമത്തെ കുരിശുയുദ്ധത്തിനു മുതല്‍ കൂട്ടക്കൊലകളുടെയും ക്രൂരതകളുടെയും ചരിത്രമാണ് പറയാനുള്ളത്. പടയാളികളെ മാത്രമായിരുന്നില്ല കുരിശുയോദ്ധാക്കള്‍ കൊന്നൊടുക്കിയത്; പ്രത്യുത സാധാരണക്കാരെക്കൂടിയായിരുന്നു. കുരിശുയുദ്ധത്തിനു മുമ്പ് ജറൂസലേം നഗരത്തിന്റെ ജനസംഖ്യ എഴുപതിനായിരമായിരുന്നെങ്കില്‍ യുദ്ധം കാരണത്താല്‍ അത് മുപ്പതിനായിരമായി ചുരുങ്ങിയെന്ന് ചരിത്രകാരന്‍മാര്‍ രേഖപ്പെടുത്തുന്നുണ്ട്.(3) നഗരത്തിലെ പകുതിയിലധികം പേരെയും കുരിശുയോദ്ധാക്കള്‍ കൊന്നുവെന്നര്‍ഥം.

മസ്ജിദുല്‍ അഖ്‌സയുടെ ഉള്ളില്‍ അഭയം തേടിയവരെയൊ സ്ത്രീകളേയൊ കുട്ടികളെയൊ അവര്‍ വെറുതെ വിട്ടില്ല. കണ്ണില്‍ കണ്ട എല്ലാവരെയും അവര്‍ കൊന്നൊടുക്കി. സിനഗോഗുകള്‍ക്കകത്ത് ജൂതന്മാര്‍ ഒളിച്ചപ്പോള്‍ അവരെ അതിനുള്ളിലിട്ട് പൂട്ടി അവര്‍ സിനഗോഗുകള്‍ക്ക് തീ വെച്ചു. യുദ്ധത്തില്‍ ബന്ദികളായി പിടിച്ചവരെയെല്ലാം കുരിശുദ്ധോക്കള്‍ കൂട്ടക്കൊല ചെയ്തു. തലകളുടെയും കൈകളുടെയും കാലുകളുടെയും  കൂമ്പാരങ്ങളായിരുന്നു ജെറുസെലേം നഗരത്തിലെന്നും അവയ്ക്കിടയിലൂടെ ഒരു പരവതാനിയില്‍ എന്നതുപോലെ നടന്നുല്ലസിക്കുകയായിരുന്നു കുരിശുദ്ധോക്കളെന്നും ''നമുക്ക് വേണ്ടി കര്‍ത്താവ് ഈ ദിവസം ഒരുക്കിത്തന്നിരിക്കുന്നത് ആഹ്ലാദിക്കുവാനും സന്തോഷം പങ്കിടാനുമാണ്'' എന്നു പ്രഖ്യാപിച്ച കുരിശുയോദ്ധാവായ അഗ്വിലേഴ്‌സ്‌കാരന്‍ റെയ്മണ്ട് രേഖപ്പെടുത്തുന്നുണ്ട്.(4) കുരിശു യുദ്ധത്തെക്കുറിച്ച് എഴുതപ്പെട്ട ലത്തീന്‍ ദിനവൃത്താന്തമായ ജെസ്റ്റ ഫ്രാന്‍ കോറം (Gesta Fran corum- ഫ്രാങ്കകളുടെ ചെയ്തികള്‍)യിലെ വരികള്‍ കാണുക: ''നമ്മുടെയാളുകള്‍ സോളമന്റെ ദേവാലയത്തിനടുത്തുവെച്ചുപോലും ആളുകളെ വെറുതെ വിട്ടില്ല. അവരെയെല്ലാം കൂട്ടക്കൊല ചെയ്തു. നെരിയാണിവരെ മുങ്ങുന്ന രക്തപ്പുഴയിലൂടെ നീന്തിയാണ് അവരിതെല്ലാം ചെയ്തത്. പ്രാകൃതന്‍മാരെ കീഴടക്കിയതിനു ശേഷം നമ്മുടെ ആള്‍ക്കാര്‍ അവരിലെ സ്ത്രീകളെയും കുട്ടികളെയുമടക്കം എല്ലാവരെയും ബന്ദികളാക്കി. ഓരോരുത്തരും ഇഷ്ടപ്രകാരം ബന്ദികളെ കൊല്ലുകയോ അടിമകളാക്കിത്തീര്‍ക്കുകയോ ചെയ്തു''.(5)

സോളമന്റെ ദേവാലയത്തില്‍ വെച്ച് നടന്ന കൂട്ടക്കൊലയെപ്പറ്റി ചര്‍ട്രസുകാരനായ ഫുള്‍ച്ചര്‍ വിവരിക്കുന്നത് ഇങ്ങനെയാണ്: ''ഈ ദേവാലയത്തില്‍വെച്ച് മാത്രം പതിനായിരം പേര്‍ കൊല്ലപ്പെട്ടു. അപ്പോള്‍ നിങ്ങള്‍ ഞങ്ങളെ കണ്ടിരുന്നെങ്കില്‍ കൊല്ലപ്പെട്ടവരുടെ ചോരയാല്‍ ഞങ്ങളുടെയെല്ലാം കണങ്കാലുകള്‍ ചുവന്നത് നിങ്ങള്‍ക്ക് കാണാമായിരുന്നു. സ്ത്രീകളെയും കുട്ടികളെയുമടക്കം ഒരാളെയും ഞങ്ങള്‍ വെറുതെ വിട്ടില്ല''.(6)

ദേവാലയത്തിലും സോളമന്റെ വരാന്തയിലുംകൂടി അവരുടെ മുട്ടുവരെ നിറഞ്ഞ ചോരയിലൂടെ ചോരപൂരണ്ട കടിഞ്ഞാണ്‍ പിടിച്ച് ജനം സവാരി നടത്തി(7)യെന്ന കുരിശുയോദ്ധാവിന്റെ ദൃക്‌സാക്ഷ്യം സംഭവത്തിന്റെ ഭീകരത വ്യക്തമാക്കുന്നുണ്ട്. സിനഗോഗുകള്‍ക്കകത്ത് അഭയം പ്രാപിച്ച ജൂതന്‍മാരെ അതിനുള്ളില്‍ പൂട്ടിയിട്ട് അതിന് തീക്കൊളുത്തിയശേഷം അഗ്നിസ്ഫുലിംഗങ്ങള്‍ ഉയര്‍ന്നു പൊങ്ങിയപ്പോള്‍ അതിനുചുറ്റും ആനന്ദനൃത്തമാടിക്കൊണ്ട് കുരിശുയോദ്ധാക്കള്‍ പാടി. 'ക്രിസ്തുവേ... നിന്നെ ഞങ്ങള്‍ ആരാധിക്കുന്നു; നീയാണ് ഞങ്ങളുടെ വെളിച്ചം. നീയാണ് ഞങ്ങളുടെ ദിശ; നീയാണ് ഞങ്ങളുടെ സ്‌നേഹം.'(8)

ഒന്നാമത്തെ കുരിശുയുദ്ധത്തോടെ മുസ്‌ലിം ലോകത്തിെന്റ അന്നത്തെ ദൗര്‍ബല്യം മനസ്സിലാക്കിയ കുരിശു പടയാളികള്‍ അര്‍ബന്‍ രണ്ടാമന്റെ നിര്‍ദേശം പാലിച്ചുകൊണ്ട് മുസ്‌ലിം ലോകത്ത് നടത്തിയ ക്രൂരതകളുടെ ഭീതിപ്പെടുത്തുന്ന കഥകളാണ് ഓരോ കുരിശു യുദ്ധത്തിനും പറയാനുള്ളത്. നിയമങ്ങളോ നീതിയോ ധാര്‍മികതയോയില്ലാത്ത കാടന്‍ യുദ്ധങ്ങളാണ് കുരിശുയുദ്ധക്കാര്‍ നടത്തിയത്. ഇവയാല്‍ ലക്ഷക്കണക്കിന് മനുഷ്യര്‍ കൊല്ലപ്പെട്ടു. മരിച്ചവരുടെ ശവശരീരങ്ങളെ കുരിശു പടയാളികള്‍ വികൃതമാക്കി. എട്ടും പൊട്ടും തിരിച്ചറിയാത്ത പൈതങ്ങളെപ്പോലും കൊല്ലുന്നതില്‍ നിന്ന് അവരെ പിന്‍തിരിപ്പിക്കുവാന്‍ 'ഒരു മുഖത്തടിച്ചാല്‍ മറ്റേ മുഖവും കാണിച്ചു കൊടുക്കണം' എന്ന ബൈബിള്‍ പാഠത്തിെന്റ അടിസ്ഥാനത്തില്‍ ജനങ്ങളെ നയിക്കുന്ന വരെന്ന് അവകാശപ്പെടുന്ന സഭാനേതൃത്വം സന്നദ്ധമായില്ല. ക്രിസ്തുവിന്റെ പേരില്‍ ആരോപിക്കപ്പെട്ട, അദ്ദേഹത്തിന്റേതല്ലാത്ത ഉപദേശം അപ്രായോഗികവും അക്രമികളെ സഹായിക്കുന്നതുമാണെന്ന വസ്തുതക്ക് സഭാനേതൃത്വം തന്നെ സ്വയം തെളിവായിത്തീരുകയായിരുന്നു എന്നു പറയുന്നതാവും ശരി. അതല്ലായിരുന്നുവെങ്കില്‍ ഈ കുരിശുയുദ്ധങ്ങളുടെ പേരില്‍ ഏഴു നൂറ്റാണ്ടുകള്‍ക്കു ശേഷം ലോകത്തെ പരമോന്നത ക്രിസ്ത്യാനിക്ക് കുമ്പസരിക്കേണ്ടി വരികയില്ലായിരുന്നല്ലോ!

ആയുധങ്ങളെടുത്ത് ആളുകളെ കൂട്ടക്കൊല ചെയ്യുക മാത്രമാണ് യുദ്ധമെന്ന് പഠിച്ച കുരിശുയോദ്ധാക്കള്‍ക്ക് മാന്യമായി എങ്ങെന യുദ്ധം ചെയ്യണമെന്ന് പഠിപ്പിച്ചത് സുല്‍ത്താന്‍ സ്വലാഹുദ്ദീന്‍ അയ്യൂബിയായിരുന്നു. നീണ്ട രണ്ടു നൂറ്റാണ്ടു കാലത്തെ കുരിശുയോദ്ധാക്കളുടെ ദുര്‍ഭരണത്തില്‍ നിന്ന് മസ്ജിദുല്‍ അഖ്‌സയെയും ജറൂസലേമിനെയും മോചിപ്പിച്ചത് അദ്ദേഹത്തിന്റെ കീഴിലുള്ള അബ്ബാസിയ്യാ മുസ്‌ലിം പടയാളികളായിരുന്നു. 1187 ജൂലൈ 4 ന് ശനിയാഴ്ച നടന്ന ഹത്തീന്‍ യുദ്ധത്തില്‍ കുരിശു പടയാളികള്‍ പരാജയപ്പെടുകയും ജറൂസലേമിന്റെ അധികാരം സ്വലാഹുദ്ദീന്റെ കൈകളില്‍ വന്നു ചേരുകയും ചെയ്തു. കുരിശുയോദ്ധാക്കളുടെ നേതാക്കളായ ഗൈ രാജാവും സഹോദരങ്ങളുമടക്കം നിരവധിപേരെ ബന്ധികളാക്കി സ്വലാഹുദ്ദീനു മുന്നില്‍ ഹാജരാക്കപ്പെട്ടു. അവര്‍ക്കെല്ലാം തണുത്ത പാനീയം നല്‍കി സ്വീകരിക്കുവാനാണ് സ്വലാഹുദ്ദീന്‍ തന്റെ സേവകന്‍മാരോട് ആവശ്യപ്പെട്ടത്. സ്വലാഹുദ്ദീനെയും മുസ്‌ലിം പടയാളികളെയും വഞ്ചിച്ചതിന് രാജസഹോദരനായ റെയ്‌നാള്‍ഡും മറ്റു ചിലരും മാത്രമാണ് വധിക്കപ്പെട്ടത്. ഗൈ രാജാവടക്കമുള്ളവരെ ആദ്യം ജയിലിലടക്കുകയും പിന്നെ വെറുതെ വിടുകയുമാണ് ചെയ്തത്. കുരിശുയോദ്ധാക്കള്‍ ജറൂസലേമില്‍ നിന്ന് പുറത്താക്കിയ ജൂതന്‍മാരുടെ പിന്‍ഗാമികളെ വരുത്തി അവിടെ പുനരധിവസിപ്പിക്കുകയാണ് ഭരണമേറ്റെടുത്തയുടനെ സ്വലാഹുദ്ദീന്‍ ചെയ്തത്.(9) മനുഷ്യരക്തം നീന്തി രക്തദാഹം തീര്‍ക്കുകയും പൈതങ്ങളെയടക്കം കൊന്നൊടുക്കുകയും ചെയ്ത് ജറൂസലേം പിടിച്ചടക്കിയവര്‍ക്ക് അതു തിരിച്ചുപിടിച്ചശേഷം യുദ്ധനേതൃത്വത്തിലുണ്ടായിരുന്നവര്‍ക്കടക്കം മാപ്പുനല്‍കി വിട്ടയച്ചുകൊണ്ട് എന്തായിരിക്കണം യുദ്ധമര്യാദയെന്ന് പഠിപ്പിക്കുന്നതായിരുന്നു സ്വലാഹുദ്ദീന്റെ നടപടി.

സ്വലാഹുദ്ദീന്‍ അയ്യൂബി ജറൂസലേം പിടിച്ചടക്കിയതറിഞ്ഞ പോപ്പ് അര്‍ബന്‍ മൂന്നാമന്‍ മാര്‍പാപ്പ ഹൃദയാഘാതം മൂലം മരണെപ്പട്ടു.(10) ജറൂസലേം നഷ്ടപ്പെട്ടതറിഞ്ഞ ക്രിസ്ത്യന്‍ യൂറോപ്പ് ഞെട്ടി. പുതുതായി സ്ഥാനമേറ്റെടുത്ത പോപ്പ് ഗ്രിഗറി എട്ടാമന്‍ മാര്‍പാപ്പ ജറൂസലേമിന്റെ നഷ്ടം ക്രൈസ്തവരുടെ പാപങ്ങള്‍ക്കുള്ള ശിക്ഷയാണെന്നും അതു തിരിച്ചുപിടിച്ചാല്‍ മാത്രമെ പ്രസ്തുത പാപങ്ങള്‍ക്ക് പരിഹാരമാവുകയുള്ളുവെന്നും പ്രഖ്യാപിച്ചു. പരസ്പരം യുദ്ധത്തിലായിരുന്ന ഇംഗ്ലണ്ടിലെ ഹെന്‍ട്രി രണ്ടാമനും ഫ്രാന്‍സിലെ ഫിലിപ്പ് രണ്ടാമനും വൈര്യങ്ങള്‍ വെടിഞ്ഞ് ഐക്യപ്പെടാനും സ്വലാഹുദ്ദീനെതിരെ ഒരുമിച്ച് പോരാടി ജറൂസലേം വീണ്ടെടുക്കുവാനും തിരുമാനിച്ചു. സലാദിന്‍ നികുതി(Saladin tithe)എന്ന പേരില്‍ രണ്ടു നാട്ടിലുമുള്ള പൗരന്‍മാരില്‍ നിന്ന് ചുങ്കംപിരിച്ച് യുദ്ധത്തിനുള്ള സമ്പത്ത് സമാഹരിച്ചു. 1189 ജൂലൈ 6ന് ഇംഗ്ലണ്ടിലെ രാജാവായ ഹെന്‍ട്രി രണ്ടാമന്‍ മരണപ്പെട്ടതോടെ യുദ്ധനേതൃത്വം മകന്‍ സിംഹഹൃദയനായ റിച്ചാര്‍ഡ് (Richard the lion-hearted) എന്നറിയപ്പെട്ട റിച്ചാര്‍ഡ് ഒന്നാമന്റെ ചുമതലയായിത്തീര്‍ന്നു. റിച്ചാര്‍ഡ് ഒന്നാമന്റെയും ഫിലിപ്പ് രണ്ടാമന്റെയും നേതൃത്വത്തിലുള്ള കുരിശുയോദ്ധാക്കളുടെ സൈന്യത്തോട് സ്വലാഹുദ്ദീന്‍ ജയില്‍ മോചിതനാക്കിയ ഗൈരാജാവും ഒന്നിക്കുകയും ഒരു വലിയ സൈന്യം സ്വലാഹുദ്ദീനെ ആക്രമിക്കാനെത്തുകയും ചെയ്തു. വഴിയില്‍വെച്ചു നടന്ന അക്രമങ്ങളിലൂടെ സ്ത്രീകളും കുട്ടികളുമടക്കമുള്ള മുസ്‌ലിംകളെ കൊന്നുകൂട്ടിക്കൊണ്ടായിരുന്നു കുരിശ് സൈന്യം മുന്നേറിയത്. 1191 സെപ്റ്റംബര്‍ 7ന് ജറൂസലേം പിടിച്ചടക്കുകയെന്ന ലക്ഷ്യത്തോട സിംഹഹൃദയനായ റിച്ചാര്‍ഡിന്റെ നേതൃത്വത്തിലുള്ള സൈന്യം ഫലസ്തിനിലുള്ള അര്‍സൂഫില്‍ വെച്ച് സ്വലാഹുദ്ദീനോട് ഏറ്റുമുട്ടിയെങ്കിലും അദ്ദേഹത്തിന് ലക്ഷ്യംനേടാന്‍ കഴിഞ്ഞില്ല. നിരന്തമായി നടന്ന യുദ്ധങ്ങള്‍ക്ക് വിരാമമുണ്ടായത് 1192 സെപ്റ്റംബര്‍ 2ന് സ്വലാഹുദ്ദീനും റിച്ചാര്‍ഡും തമ്മിലുണ്ടാക്കിയ സന്ധിയോടു കൂടിയാണ്. ജറൂസലേം മുസ്‌ലിംകളുടെ നിയന്ത്രണത്തിലായിരിക്കുമെന്നും എന്നാല്‍ നിരായുധരായ ക്രൈസ്തവരെ അവിടെയുള്ള ക്രിസ്ത്യന്‍ പുണ്യസ്ഥലങ്ങള്‍ സന്ദര്‍ശിക്കാന്‍ അനുവദിക്കുമെന്നുമായിരുന്നു കരാര്‍.(11) പുണ്യസ്ഥലങ്ങള്‍ സന്ദര്‍ശിക്കുന്നതില്‍ നിന്ന് മുമ്പും സ്വലാഹുദ്ദീന്‍ ക്രൈസ്തവെര വിലക്കിയിട്ടില്ല എന്നതുകൊണ്ടുതന്നെ കരാര്‍ വഴി കുരിശുയോദ്ധാക്കള്‍ക്ക് ഒന്നും നേടാനായില്ലെങ്കിലും, 1192 ഒക്‌ടോബര്‍ 9ന് റിച്ചാര്‍ഡ് ജറൂസലേമില്‍ നിന്ന് മടങ്ങിയത് യുദ്ധരംഗത്ത് പാലിക്കപ്പെടേണ്ട മര്യാദകളെന്തൊക്കെയാണെന്ന് പഠിച്ചുകൊണ്ടായിരുന്നു. ശത്രുവിനെ സ്‌നേഹിക്കേണ്ടത് എങ്ങനെയാണെന്ന് സഭയില്‍ നിന്ന് പഠിക്കാത്ത അദ്ദേഹത്തിന് അത് പഠിപ്പിച്ചത് സ്വലാഹുദ്ദീന്‍ അയ്യൂബിയുടെ യുദ്ധരംഗത്തെ വര്‍ത്തനങ്ങളായിരുന്നു.

യുദ്ധരംഗത്തായിരിക്കുമ്പോള്‍ പോലും ശത്രുവിന്റെ മാനുഷികമായ ആവശ്യങ്ങള്‍ നിര്‍വഹിച്ചു കൊടുക്കുവാന്‍ സ്വലാഹുദ്ദീന്‍ അയ്യൂബി സന്നദ്ധമായി. റിച്ചാര്‍ഡിന് പനിപിടിച്ചപ്പോള്‍ തന്റെ വൈദ്യനെ അയച്ച് ചികില്‍സിക്കുകയും മരുന്ന് നല്‍കുകയും നല്ല പഴങ്ങളും ശുദ്ധമായ തണുത്ത ജലവും കൊടുത്തയക്കുകയും ചെയ്തത് സ്വലാഹുദ്ദീനായിരുന്നു. യുദ്ധത്തിനിടയില്‍ റിച്ചാര്‍ഡിന് കുതിര നഷ്ടപ്പെട്ടപ്പോള്‍ പകരം രണ്ടു കുതിരകളെ നല്‍കിയതും അദ്ദേഹം തന്നെ. അതുകൊണ്ടു തന്നെ കുരിശുയോദ്ധാക്കള്‍ക്കിടയില്‍ പോലും  സ്വലാഹുദ്ദീന്‍ നായകനായിത്തീര്‍ന്നു. തന്റെ സേഹാദരിയും സിസിലിയിലെ രാജ്ഞിയുമായ ജോനിനെ സ്വലാഹുദ്ദീന്റെ സഹോദരന് വിവാഹം ചെയ്തു കൊടുക്കുവാന്‍ സന്നദ്ധമാണെന്ന് അറിയിച്ചുകൊണ്ടാണ് അദ്ദേഹം ജറൂസലേം വിട്ടത്.(12) കാടനായി വന്നയാളെ മാന്യനായിത്തീര്‍ത്ത് തിരിച്ചയച്ചത് ശത്രുവിനെ സ്‌നേഹിക്കണമെന്ന പ്രവാചകന്‍മാരുടെ നിര്‍ദേശം യഥാവിധി പാലിക്കുവാന്‍ സ്വലാഹുദ്ദീന്‍ അയ്യൂബി തയ്യാറായതു കൊണ്ടായിരുന്നു.

ജീവിതത്തിന്റെ മറ്റു രംഗങ്ങളിലേതു പോലെത്തന്നെ യുദ്ധരംഗത്തും പ്രവാചകന്‍മാര്‍ പഠിപ്പിച്ച സമാധാന സന്ദര്‍ഭത്തിലും യുദ്ധരംഗത്തുമെല്ലാം പാലിക്കേണ്ട നിയമങ്ങളെന്തൊക്കെയാണെന്ന് അദ്ദേഹം പഠിപ്പിച്ചു തരികയും പ്രായോഗികമാക്കി കാണിച്ചുതരികയും ചെയ്തിട്ടുണ്ട്. പീഡനങ്ങള്‍ സഹിച്ച് പ്രബോധനം ചെയ്യുമ്പോയും ആദര്‍ശജീവിതത്തിന് നിവൃത്തിയില്ലാതെ പാലായനം ചെയ്യുമ്പോഴും വിവിത സമുദായങ്ങള്‍ ജീവിക്കുന്ന നാട് ഭരിക്കേണ്ടിവരുമ്പോഴും ഭരണാധികാരിയായി ശത്രുരാജ്യങ്ങളോട് യുദ്ധം ചെയ്യുമ്പോഴും യുദ്ധത്തില്‍ ജയിക്കുമ്പോയും ശത്രുരാജ്യങ്ങളോട് സന്ധിയുണ്ടാക്കുമ്പോഴും ശത്രുരാജ്യത്തിനുമേല്‍ അധിശത്വം ലഭിക്കുമ്പോഴുമെല്ലാം എങ്ങനെ പെരുമാറണമെന്ന് അദ്ദേഹം കൃത്യമായി പഠിപ്പിച്ചിട്ടുണ്ട് യുദ്ധരംഗവും സമാധാനരംഗവുമെല്ലാം മാനവവല്‍ക്കരിക്കുവാന്‍ ആ ജീവിതം തന്നെയാണ് മനുഷ്യര്‍ മാതൃകയാക്കേണ്ടത്. എല്ലാ രംഗത്തുമുള്ള സല്‍സ്വഭാവങ്ങളുടെ പൂര്‍ത്തീകരണത്തിനു വേണ്ടിയാണല്ലോ നബിനിയോഗമുണ്ടായിട്ടുള്ളത്. ശാന്തിക്കും സമാധാനത്തിനും വേണ്ടി പ്രവാചകന്മാരെല്ലാം പഠിപ്പിച്ച പാതയിലേക്കുള്ള തിരിച്ചുപോക്കാണ് വേണ്ടതെന്നും അതുവഴി മാത്രമെ ശത്രുതയുടെ മാറ്റുരയ്ക്കപ്പെടുന്ന രണഭൂമിയെപ്പോലും മാനവവല്‍ക്കരിക്കാന്‍ കഴിയുകയുള്ളൂവെന്നുമാണ് കുരിശുയുദ്ധഭൂമിയിൽ വെച്ച് മുസ്ലിംകൾ ശത്രുക്കളെ പഠിപ്പിച്ചത്.

  1. Medieval Source book: Urban II (1085 - 1099) Speech at Counsil of Clerm-ont, 1095 (www.fordham-edu/halsall/source/urban 2-5 verntml.
  2. Thomas Asbridge: The Crusades. The Authoritative History of the war for the Holy Land, Chicago, 2010.
  3. Michael D Hull: First Crusade: Siege of Jerusalem, June 1999, Military History Magazine (www..historynet.com)
  4. Ibid
  5. Gesta Francorum” (fodham.edu/halsall/source/gesta-ede-asp
  6. Fulcher of chartresa: The siege of the city of Jerusalem; Gesta francorum (fordham.edu)
  7. Benjamin 2 Kedar & Jonathan SC: In the Crusades, New York 2004, Vol 3, page 65
  8. David A Rausch: A Legacy of Hatred: Why Christians Must not forget the Holocaust, Chicago, 1984, Page 63.
  9. Crusades’ Encyclopaedia Brittanica, Online edition, brittanica.com
  10. Pope Urban III”, Encyclopaedia of Keywords, Keg 1(keywen.com)
  11. Thomas Madden: The New Concise History of the Crusades, Mary Land, 2006, Page 90-98
  12. James Reston Jr: Richard the Lionheart and Saladin in the third crusade, Harpswell, 2002 Page 358-378
ല്ല. ലോകം മുഴുവൻ ഇസ്‌ലാമീകരിക്കുക എന്ന ഒരു ലക്‌ഷ്യം പ്രവാചകൻ ഉണ്ടായിരുന്നതായി വ്യക്തമാക്കുന്ന ആയത്തുകളോ ഹദീഥുകളോ ഒന്നുമില്ല.

യുദ്ധത്തിലൂടെയോ നിര്‍ബന്ധമോ ബലാല്‍ക്കാരമോ ചെലുത്തിക്കൊണ്ടോ സ്വീകരിക്കപ്പെടേണ്ടതല്ല മതവിശ്വാസമെന്നും സത്യവും അസത്യവും വിവേചിച്ച് മനസ്സിലാക്കിക്കൊടുക്കുകയാണ് പ്രബോധകനെന്ന നിലയില്‍ പ്രവാചകന്റെ ബാധ്യതയെന്നും അദ്ദേഹത്തിന് സ്വന്തമായി ആരെയും സന്മാര്‍ഗത്തിലാക്കുവാന്‍ സാധ്യമല്ലെന്നും സത്യവിശ്വാസം സ്വീകരിക്കുവാന്‍ മനസ്സ് പാകപ്പെട്ടവരെ അതിലേക്ക് നയിക്കുന്നത് അല്ലാഹുവാണെന്നും വ്യക്തമാക്കുന്ന ഗ്രൻഥമാണ് ക്വുര്‍ആൻ. പ്രവാചകജീവിതത്തിലെപ്പോഴെങ്കിലും ആരെയെങ്കിലും നിർബന്ധിച്ച് ഇസ്‌ലാം സ്വീകരിപ്പിച്ചതായി വ്യക്തമാക്കുന്ന രേഖകളൊന്നും തന്നെയില്ല. പിന്നെയെങ്ങനെയാണ് യുദ്ധത്തിലൂടെ ലോകം മുഴുവന്‍ ഇസ്‌ലാം സ്വീകരിക്കുന്ന അവസ്ഥയുണ്ടാക്കുകയാണ് മുസ്‌ലിംകളുടെ ധര്‍മമെന്ന് പറയാനാവുക ?! ലോകത്തുള്ള മനുഷ്യരെല്ലാം സത്യസന്ദേശം സ്വീകരിക്കുന്ന സ്ഥിതി ഉണ്ടാവുകയില്ലെന്ന് വ്യക്തമാക്കുന്ന ക്വുര്‍ആനെങ്ങനെ അത്തരമൊരു സ്ഥിതി സംജാതമാക്കുന്നതിന് യുദ്ധത്തിലേര്‍പ്പെടണമെന്ന് അതിന്റെ അനുയായികളെ ആഹ്വാനം ചെയ്യാന്‍ കഴിയും? ഒരിക്കലും ഉണ്ടാവുകയില്ലെന്നുറപ്പുള്ള സാമൂഹ്യാവസ്ഥയുടെ സൃഷ്ടിയെ യുദ്ധലക്ഷ്യമായി നിര്‍ണയിക്കുന്ന അന്ധമായ ദര്‍ശനമല്ല ഇസ്‌ലാമെന്ന് അതിന്റെ അടിസ്ഥാനാദര്‍ശങ്ങളെങ്കിലും പഠിച്ചാല്‍ ആര്‍ക്കും മനസ്സിലാക്കാവുന്നതാണ്. ലോകം മുഴുവന്‍ ഇസ്‌ലാമീകരിക്കുകയാണ് ഇസ്‌ലാമിന്റെ യുദ്ധലക്ഷ്യമെന്ന ആരോപണം അടിസ്ഥാനരഹിതമാണെന്നര്‍ഥം.

മതവിശ്വാസത്തിന്റെ സ്വീകരണത്തിന്റെയും തിരസ്‌കരണത്തിന്റെയും മാനദണ്ഡമെന്തായിരിക്കണമെന്ന് വ്യക്തമായി പഠിപ്പിക്കുന്ന ഗ്രന്ഥമാണ് ക്വുര്‍ആന്‍. ഇവ്വിഷയമായി ഏറെ പ്രസിദ്ധമായ ക്വുര്‍ആന്‍ സൂക്തങ്ങളുടെ സാരം ഇങ്ങനെയാണ്: ''മതത്തിന്റെ കാര്യത്തില്‍ ബലപ്രയോഗമേ ഇല്ല. സന്‍മാര്‍ഗം ദുര്‍മാര്‍ഗത്തില്‍ നിന്ന് വ്യക്തമായി വേര്‍തിരിഞ്ഞ് കഴിഞ്ഞിരിക്കുന്നു. ആകയാല്‍ ഏതൊരാള്‍ ദുര്‍മൂര്‍ത്തികളെ അവിശ്വസിക്കുകയും അല്ലാഹുവില്‍ വിശ്വസിക്കുകയും ചെയ്യുന്നുവോ അവന്‍ പിടിച്ചിട്ടുള്ളത് ബലമുള്ള ഒരു കയറിലാകുന്നു. അത് പൊട്ടി പോകുകയേ ഇല്ല. അല്ലാഹു (എല്ലാം) കേള്‍ക്കുന്നവനും അറിയുന്നവനുമാകുന്നു. വിശ്വസിച്ചവരുടെ രക്ഷാധികാരിയാകുന്നു അല്ലാഹു. അവന്‍ അവരെ ഇരുട്ടുകളില്‍ നിന്ന് വെളിച്ചത്തിലേക്ക് കൊണ്ടു വരുന്നു. എന്നാല്‍ സത്യനിഷേധികളുടെ രക്ഷാധികാരികള്‍ ദുര്‍മൂര്‍ത്തികളാകുന്നു. വെളിച്ചത്തില്‍ നിന്ന് ഇരുട്ടുകളിലേക്കാണ് ആ ദുര്‍മൂര്‍ത്തികള്‍ അവെര നയിക്കുന്നത്. അവരത്രെ നരകാവകാശികള്‍. അവരതില്‍ നിത്യവാസികളാകുന്നു.'' (2:256,257)

ഈ വചനങ്ങളുടെ അവതരണ പശ്ചാത്തലംകൂടി പരിശോധിച്ചാല്‍ നിര്‍ബന്ധ മതപരിവര്‍ത്തനമെന്ന ആശയത്തോട് ഇസ്‌ലാം എങ്ങനെ പ്രതികരിക്കുന്നുവെന്ന് സുതരാം വ്യക്തമാവും. ഏതെങ്കിലും സ്ത്രീകള്‍ക്ക് മക്കളുണ്ടായിട്ടില്ലെങ്കില്‍ തങ്ങള്‍ക്ക് കുഞ്ഞുണ്ടായാല്‍ അവനെ യഹൂദനാക്കി വളര്‍ത്താമെന്ന് നേര്‍ച്ചയാക്കുന്ന സ്രമ്പദായമുണ്ടായിരുന്നു, യഥ്‌രിബ് വാസികള്‍ക്കിടയില്‍. ജൂതന്മാരായ ബനൂ നദ്വീര്‍ ഗോത്രക്കാരെ മദീനയില്‍ നിന്ന് നാടുകടത്തിയപ്പോള്‍ അവരോടൊപ്പം ഇങ്ങനെ ജൂതന്മാരായിത്തീര്‍ന്ന ചില അന്‍സ്വാരി സ്ത്രീകളുടെ കുഞ്ഞുങ്ങളുമുണ്ടായിരുന്നു. അവര്‍ പ്രവാചകനോട് പറഞ്ഞു: ദൈവദൂതരേ, അവര്‍ ഞങ്ങളുടെ മക്കളാണ്. അവരെ ഞങ്ങള്‍ ജൂതന്മാരോടൊപ്പം പറഞ്ഞയക്കുകയില്ല. ഈ സന്ദര്‍ഭത്തിലാണ് ഭമതത്തില്‍ ബലാല്‍ക്കാരമില്ല' എന്ന് തുടങ്ങുന്ന സൂക്തം അവതരിക്കപ്പെട്ടതെന്ന് ഇബ്‌നു അബ്ബാസില്‍ നിന്ന് സ്വീകാര്യമായ പരമ്പരയോടെ (സ്വഹീഹ്) സുനനു അബീദാവൂദില്‍ ഉദ്ധരിച്ചിട്ടുണ്ട്.(സുനനു അബൂദാവൂദ്, കിതാബുല്‍ ജിഹാദ്‌) സ്വേച്ഛപ്രകാരമല്ലാതെ ജൂതമതം സ്വീകരിക്കേണ്ടിവന്ന സ്വന്തം മക്കളെപ്പോലും നിര്‍ബന്ധപൂര്‍വം ഇസ്‌ലാം സ്വീകരിപ്പിക്കേണ്ടതില്ലെന്ന് അവരുടെ മാതാക്കളെ നിഷ്‌കര്‍ഷിക്കുന്ന ക്വുര്‍ആന്‍ നിര്‍ബന്ധ മതപരിവര്‍ത്തനമെന്ന ആശയത്തെ ഒരു തരത്തിലും അംഗീകരിക്കുകയില്ലെന്ന് ഉറപ്പാണ്. അതുകൊണ്ട് തന്നെ ഇസ്‌ലാം യുദ്ധംചെയ്യാന്‍ പറയുന്നത് ലോകത്തുള്ള മനുഷ്യരെയെല്ലാം മുസ്‌ലിംകളാക്കുന്നതിന് വേണ്ടിയാണെന്ന വിമര്‍ശനം ശുദ്ധഭോഷ്‌ക്കാണ്.

മതത്തിന്റെ പേരില്‍ മര്‍ദിക്കപ്പെടുകയും, ഏകനായ സ്രഷ്ടാവിനെ മാത്രം ആരാധിച്ചുകൊണ്ട് സ്വസ്ഥമായി ജീവിക്കുകയും ആ മാര്‍ഗം മറ്റുള്ളവര്‍ക്ക് പകര്‍ന്നുനല്‍കാനുള്ള സ്വാതന്ത്ര്യം നിേഷധിക്കപ്പെടുകയും ചെയ്യുമ്പോള്‍ മര്‍ദനങ്ങള്‍ക്ക് അറുതി വരുത്തുന്നതിനും സ്വാതന്ത്ര്യം സ്ഥാപിക്കുന്നതിനും വേണ്ടി നടത്തുന്നതാണ് ഇസ്‌ലാം അനുവദിച്ച യുദ്ധം. ഇത്തരം സാഹചര്യങ്ങളില്‍ സായുധമായി പ്രതികരിക്കാതിരുന്നാല്‍ പിന്നെ എങ്ങനെയാണ് സത്യമതത്തിന് നിലനില്‍പുണ്ടാവുക? യുദ്ധം അനുവദിച്ച ഇസ്‌ലാമിന്റെ നടപടി തെറ്റായിപ്പോയെന്ന് പറയുന്നവര്‍ക്ക് ഇത്തരം സാഹചര്യങ്ങളില്‍ എന്ത് പരിഹാര മാര്‍ഗമാണ് നിര്‍ദേശിക്കുവാനുള്ളത്? യുദ്ധം അനുവദിക്കുക മാത്രമല്ല, ആരോട് എപ്പോള്‍ എങ്ങനെയെല്ലാം യുദ്ധംചെയ്യണമെന്നു കൂടി കൃത്യവും വ്യക്തവുമായി ഇസ്‌ലാം മുസ്‌ലിംകളെ പഠിപ്പിച്ചിട്ടുണ്ട് . ദൈവിക മാര്‍ഗദര്‍ശനപ്രകാരം അവര്‍ മുന്നേറിയപ്പോള്‍ മതത്തിന്റെ പേരിലുള്ള മര്‍ദനങ്ങള്‍ക്ക് അറുതി വരുത്തുവാനും ആരെയും ഭയപ്പെടാതെ അല്ലാഹുവിനെ മാത്രം ആരാധിച്ചുകൊണ്ട് ജീവിക്കുവാന്‍ കഴിയുന്ന സ്ഥിതി ഉണ്ടാക്കുവാനും അല്ലാഹുവിന്റെ നാം ഉയര്‍ത്തിപ്പിടിച്ച് അഭിമാനത്തോടെ ജീവിക്കാനും അവര്‍ക്ക് കഴിഞ്ഞു. ഫിത്‌ന(പീഢനം) ഇല്ലാതെയാവുകയും മതം മുഴുവന്‍ അല്ലാഹുവിന് വേണ്ടി മാത്രമായിത്തീര്‍ക്കുകയുമായിരുന്നു അവരുടെ യുദ്ധലക്ഷ്യം. പ്രസ്തുത ലക്ഷ്യം നേടുന്നത് വരെ അവര്‍ യുദ്ധം ചെയ്തു. മുസ്‌ലിമായിപ്പോയെന്ന കാരണത്താല്‍ മാത്രം മനുഷ്യര്‍ പീഡിപ്പിക്കപ്പെട്ടിരുന്ന അവസ്ഥക്ക് വിരാമമായി. മുസ്‌ലിമായി ജീവിക്കുവാനും സത്യമതപ്രബോധനത്തിനുമുള്ള സ്വാതന്ത്ര്യമുണ്ടായി. പ്രസ്തുത സ്വാതന്ത്ര്യമുപയോഗിച്ച് ലോകത്തിന്റെ മുക്കുമൂലകളിലേക്ക് സത്യമതത്തിന്റെ സന്ദേശം പ്രസരിപ്പിക്കുവാന്‍ അവര്‍ പരിശ്രമിച്ചു. നമസ്‌ക്കാരത്തിനായി ക്ഷണിക്കുന്ന ബാങ്ക്‌വിളി മുഴങ്ങാത്ത ഒരു നിമിഷം പോലും ഭൂഗോളത്തിലില്ലാത്ത സ്ഥിതി സംജാതമായത് അങ്ങനെയാണ്.

ആദര്‍ശത്തിന് വേണ്ടിയുള്ളതാണ് ഇസ്‌ലാമിലെ യുദ്ധം. എതിരാളികളെയെല്ലാം ഉന്മൂലനം ചെയ്യുകയോ ലോകത്തിന്റെ മേല്‍ മുസ്‌ലിംകളുടെ രാഷ്ട്രീയാധിനിവേശം സ്ഥാപിക്കുകയോ ചെയ്യുന്നതിന് വേണ്ടിയുള്ളതല്ല അത്. ദൈവത്തമായ ജീവിതവ്യവസ്ഥയനുസരിച്ച് ജീവിക്കുവാനും അതു മറ്റുള്ളവര്‍ക്ക് പകര്‍ന്നുകൊടുക്കുവാനുമുള്ള സ്വാതന്ത്ര്യമുണ്ടാക്കുകയാണ് യുദ്ധത്തിന്റെ ലക്ഷ്യം. മുസ്‌ലിംകളായി ജീവിക്കാന്‍ അനുവദിക്കാതെ ജനിച്ചുവളര്‍ന്ന നാട്ടില്‍ നിന്ന് പലായനം ചെയ്യേണ്ടി വന്നവര്‍ക്ക് തങ്ങളുടെ അവകാശങ്ങള്‍ വീണ്ടെടുക്കുന്നതിന് വേണ്ടി പേരാടുവാന്‍ അനുവദിച്ചുകൊണ്ടുള്ളതാണ് യുദ്ധത്തെക്കുറിച്ച് പരാമര്‍ശിക്കുന്ന ആദ്യമായി അവതരിക്കപ്പെട്ട വചനങ്ങള്‍. സൂറത്തുല്‍ ഹജ്ജിലെ 39,40 സൂക്തങ്ങള്‍ അവതരിക്കപ്പെട്ടപ്പോഴാണ് യുദ്ധമുണ്ടാകുവാന്‍ പോകുന്നുണ്ടെന്ന് തനിക്ക് മനസ്സിലായത് എന്ന് അബൂബക്കർ (റ) പറഞ്ഞതായി  സ്വീകാര്യമായ പരമ്പരയോടെ ഉദ്ധരിക്കപ്പെട്ടിട്ടുണ്ട്. (സുനനുന്നസാഇയിലും (ഹദീഥ് 365) ജാമിഉത്തിര്‍മിദിയിലും (3171) മുസ്‌നദ് ഇമാം അഹ്മദിലും (1216) അല്‍മുസ്തദ്‌റക്ക് അല്‍ ഹാകിമിലും (266) ഉദ്ധരിച്ചിരിക്കുന്ന ഈ ഹദീഥ് സ്വഹീഹാണെന്ന് ഇമാം നസാഈ രേഖപ്പെടുത്തിയതായി ഇമാം നൈസാബൂരി തന്റെ അസ്ബാബുന്നുസൂലില്‍’പറഞ്ഞിട്ടുണ്ട്.(Aiman Saleh Sha’aban & Mohammed Ismail: Al-Imam Al-Nais aburi’s"Reasons of Revelation of the Holy Qur'an'', Kaulalampur, 2010, page457))

വീടും സമ്പത്തുകളും ഉപേക്ഷിച്ചുകൊണ്ട് ആദര്‍ശമനുസരിച്ച് ജീവിക്കുവാന്‍ വേണ്ടി മാത്രമായി പലായനം ചെയ്തവര്‍ക്ക് അവര്‍ പലായനം ചെയ്‌തെത്തി ജീവിക്കുന്ന നാട്ടില്‍ സൈ്വര്യമായിരിക്കുവാന്‍ അവസരം നല്‍കാതിരിക്കുന്നവര്‍ക്കെതിരെ യുദ്ധം ചെയ്യാന്‍ അനുവാദമുണ്ടെന്നും ആയുധമെടുക്കുന്നവരെ അല്ലാഹു സഹായിക്കുമെന്ന് അറിയിക്കുകയും ചെയ്തുകൊണ്ടുള്ള പ്രസ്തുത സൂക്തങ്ങളുടെ സാരം ഇങ്ങനെയാണ്: ''യുദ്ധത്തിന്ന് ഇരയാകുന്നവര്‍ക്ക്, അവര്‍ മര്‍ദിതരായതിനാല്‍ (തിരിച്ചടിക്കാന്‍) അനുവാദം നല്‍കപ്പെട്ടിരിക്കുന്നു. തീര്‍ച്ചയായും അല്ലാഹു അവരെ സഹായിക്കാന്‍ കഴിവുള്ളവന്‍ തന്നെയാകുന്നു. യാതൊരു ന്യായവും കൂടാതെ, ഞങ്ങളുടെ രക്ഷിതാവ് അല്ലാഹുവാണ് എന്ന് പറയുന്നതിന്റെ പേരില്‍ മാത്രം തങ്ങളുടെ ഭവനങ്ങളില്‍ നിന്ന് പുറത്താക്കപ്പെട്ടവരെ്രത അവര്‍. മനുഷ്യരില്‍ ചിലരെ മറ്റുചിലരെക്കൊണ്ട് അല്ലാഹു തടുക്കുന്നില്ലായിരുന്നുവെങ്കില്‍ സന്യാസിമഠങ്ങളും, ക്രിസ്തീയദേവാലയങ്ങളും, യഹൂദദേവാലയങ്ങളും, അല്ലാഹുവിന്റെ നാമം ധാരാളമായി പ്രകീര്‍ത്തിക്കപ്പെടുന്ന മുസ്‌ലിം പള്ളികളും തകര്‍ക്കപ്പെടുമായിരുന്നു. തന്നെ സഹായിക്കുന്നതാരോ അവനെ തീര്‍ച്ചയായും അല്ലാഹു സഹായിക്കും. തീര്‍ച്ചയായും അല്ലാഹു ശക്തനും പ്രതാപിയും തന്നെയാകുന്നു.'' (22:39,40)

മതമര്ദനം ഇല്ലാതെയാക്കുന്നതിനും ആക്രമത്തെ പ്രതിരോധിക്കുന്നതിനും വേണ്ടിയാണ് മുസ്‌ലിംകള്‍ യുദ്ധം ചെയ്യേണ്ടതെന്ന് വ്യക്തമാക്കുന്ന ഈ സൂക്തങ്ങള്‍ അത്തരം പ്രതിരോധങ്ങളുണ്ടായിട്ടില്ലെങ്കില്‍ മുസ്‌ലിംകള്‍ക്ക് മാത്രമല്ല മറ്റു മതവിശ്വാസികള്‍ക്കും സൈ്വര്യമായി ആരാധനകള്‍ നിര്‍വഹിച്ചുകൊണ്ട് ജീവിക്കുവാന്‍ കഴിയാത്ത സാഹചര്യമാണുണ്ടാവുകയെന്ന് വ്യക്തമാക്കുന്നു. സന്യാസി മഠങ്ങളും ചര്‍ച്ചുകളും സിനഗോഗുകളും മസ്ജിദുകളുമെല്ലാം തകര്‍ക്കപ്പെടുകയും വിശ്വാസികള്‍ക്കൊന്നും സ്വസ്ഥമായി ആരാധനകള്‍ നിര്‍വഹിക്കാനാവാത്ത അവസ്ഥ ഉണ്ടാവുകയുമായിരിക്കും മതപീഡകന്മാരെ സ്വതന്ത്രമായി അഴിഞ്ഞാടുവാന്‍ അനുവദിച്ചാലുണ്ടാവുന്ന ഫലങ്ങള്‍. മതത്തിന്റെ പേരിലുള്ള പീഡനം ഇല്ലാതെയാക്കുന്നതിന് വേണ്ടി മുസ്‌ലിംകള്‍ നടത്തുന്ന പോരാട്ടം മറ്റു മതവിശ്വാസികള്‍ക്കും സമാധാനപൂര്‍ണമായി ആരാധനകള്‍ നിര്‍വഹിക്കുവാനുള്ള സാഹചര്യമൊരുക്കുകയാണ് സ്വാഭാവികമായും ചെയ്യുകയെന്നര്‍ഥം.

സൂറത്തുല്‍ ബക്വറയിലെ 190 മുതല്‍ 193 വരെ സൂക്തങ്ങളും സൂറത്തുന്നിസാഇലെ 75-ാം സൂക്തവുമെല്ലാം മുസ്‌ലിംകള്‍ എന്തിനുവേണ്ടി, എങ്ങനെയെല്ലാം യുദ്ധം ചെയ്യണമെന്ന് വ്യക്തമാക്കുന്നവയാണ്. പ്രസ്തുത സൂക്തങ്ങളുടെ സാരം കാണുക:

''നിങ്ങളോട് യുദ്ധം ചെയ്യുന്നവരുമായി അല്ലാഹുവിന്റെ മാര്‍ഗത്തില്‍ നിങ്ങളും യുദ്ധം ചെയ്യുക. എന്നാല്‍ നിങ്ങള്‍ പരിധിവിട്ട് പ്രവര്‍ത്തിക്കരുത്. പരിധിവിട്ട് പ്രവര്‍ത്തിക്കുന്നവരെ അല്ലാഹു ഇഷ്ടപ്പെടുകയില്ല തന്നെ. അവരെ കണ്ടുമുട്ടുന്നേടത്ത് വെച്ച് നിങ്ങളവരെ കൊന്നുകളയുകയും, അവര്‍ നിങ്ങളെ പുറത്താക്കിയേടത്ത് നിന്ന് നിങ്ങള്‍ അവരെ പുറത്താക്കുകയും ചെയ്യുക. (കാരണം, അവര്‍ നടത്തുന്ന) മര്‍ദനം കൊലയേക്കാള്‍ നിഷ്ഠൂരമാകുന്നു. മസ്ജിദുല്‍ ഹറാമിന്നടുത്ത് വെച്ച് നിങ്ങള്‍ അവരോട് യുദ്ധം ചെയ്യരുത്; അവര്‍ നിങ്ങളോട് അവിടെ വെച്ച് യുദ്ധം ചെയ്യുന്നത് വരെ. ഇനി അവര്‍ നിങ്ങളോട് (അവിടെ വെച്ച്) യുദ്ധത്തില്‍ ഏര്‍പെടുകയാണെങ്കില്‍ അവരെ കൊന്നുകളയുക. അപ്രകാരമാണ് സത്യനിഷേധികള്‍ക്കുള്ള പ്രതിഫലം. ഇനി അവര്‍ (പശ്ചാത്തപിച്ച്, എതിര്‍പ്പില്‍ നിന്ന്) വിരമിക്കുകയാണെങ്കിലോ തീര്‍ച്ചയായും ഏറെ പൊറുക്കുന്നവനും കരുണാനിധിയുമാണ് അല്ലാഹു. മര്‍ദനം ഇല്ലാതാവുകയും, മതം അല്ലാഹുവിന് വേണ്ടിയാവുകയും ചെയ്യുന്നത് വരെ നിങ്ങളവരോട് യുദ്ധം നടത്തിക്കൊള്ളുക. എന്നാല്‍ അവര്‍ (യുദ്ധത്തില്‍ നിന്ന്) വിരമിക്കുകയാണെങ്കില്‍ (അവരിലെ) അക്രമികള്‍ക്കെതിരിലല്ലാതെ പിന്നീട് യാതൊരു കയ്യേറ്റവും പാടുള്ളതല്ല.'' (2:190-193)

''അല്ലാഹുവിന്റെ മാര്‍ഗത്തില്‍ നിങ്ങള്‍ക്കെന്തുകൊണ്ട് യുദ്ധം ചെയ്തു കൂടാ? ഞങ്ങളുടെ രക്ഷിതാവേ, അക്രമികളായ ആളുകള്‍ അധിവസിക്കുന്ന ഈ നാട്ടില്‍ നിന്ന് ഞങ്ങളെ നീ മോചിപ്പിക്കുകയും, നിന്റെ വകയായി ഒരു രക്ഷാധികാരിയെയും നിന്റെ വകയായി ഒരു സഹായിയെയും ഞങ്ങള്‍ക്ക് നീ നിശ്ചയിച്ച് തരികയും ചെയ്യേണമേ. എന്ന് പ്രാര്‍ഥിച്ച് കൊണ്ടിരിക്കുന്ന മര്‍ദിച്ചൊതുക്കപ്പെട്ട പുരുഷന്‍മാര്‍ക്കും സ്ത്രീകള്‍ക്കും കുട്ടികള്‍ക്കും വേണ്ടിയും (നിങ്ങള്‍ക്കെന്തുകൊണ്ട് യുദ്ധം ചെയ്തു കൂടാ?)'' (4:75)

ഈ സൂക്തങ്ങളില്‍ നിന്ന് നിര്‍ധരിക്കപ്പെടുന്ന യുദ്ധവിധികള്‍ താഴെ പറയുന്നവയാണ്:

1. മുസ്‌ലിംകളോട് യുദ്ധം ചെയ്യുന്നവരോട് അവര്‍ക്ക് സായുധമായി പ്രതിരോധിക്കാവുന്നതാണ്.

2. മുസ്‌ലിംകളെ അക്രമിക്കുകയും പീഡിപ്പിക്കുകയും അവരുടെ ഭവനങ്ങള്‍ കയ്യേറുകയും സമ്പത്തുകള്‍ പിടിച്ചെടുക്കുകയും അവകാശങ്ങള്‍ അനുവദിക്കാതിരിക്കുകയും ചെയ്യുന്നവര്‍ക്കെതിരില്‍ അവര്‍ക്ക് യുദ്ധം ചെയ്യാവുന്നതാണ്.

3. മതവിശ്വാസത്തിന്റെ പേരില്‍ മര്‍ദിക്കപ്പെടുകയും പീഡിക്കപ്പെടുകയും ശത്രുരാജ്യത്ത് മതവിശ്വാസം പുറത്തുപറയാതെ ജീവിക്കുവാന്‍ നിര്‍ബന്ധിക്കപ്പെടുകയും ചെയ്യുന്നവരെ മോചിപ്പിക്കുന്നതിന് വേണ്ടി മുസ്‌ലിംകള്‍ക്ക് അടരാടാവുന്നതാണ്.

4. യുദ്ധം ചെയ്യുന്നത് ശത്രുക്കളോട് പ്രതികാരം ചെയ്യുവാനോ പ്രശസ്തിക്കുവേണ്ടിയോ അല്ല, പ്രത്യുത അല്ലാഹുവിന്റെ മാര്‍ഗത്തില്‍ അവന്റെ മതമനുസരിച്ച് ജീവിക്കുവാനും അതു പ്രബോധനം ചെയ്യുന്നതിനുമുള്ള സ്വാതന്ത്ര്യം നേടിയെടുക്കാന്‍ വേണ്ടിയായിരിക്കണം.

5. യുദ്ധത്തില്‍ അതിക്രമങ്ങള്‍ ഉണ്ടാകുവാന്‍ പാടില്ല.

6. യുദ്ധത്തിലോ യുദ്ധസന്ദര്‍ഭത്തിലോ ശത്രുക്കളെ വകവരുത്തുവാന്‍ സന്ദര്‍ഭങ്ങളൊന്നും പാഴാക്കരുത്, അവരെ കണ്ടിടത്തുവെച്ച് കൊല്ലുക തന്നെ വേണം.

7. മര്‍ദനം ഇല്ലാതായാവുകയും മതം അല്ലാഹുവിന് വേണ്ടിയാവുകയും ചെയ്യുന്നത് വരെയാണ് മുസ്‌ലിംകള്‍ യുദ്ധം ചെയ്യേണ്ടത്.

8. അക്രമകാരികള്‍ കുഴപ്പമുണ്ടാക്കുന്നതില്‍ നിന്ന് വിരമിച്ചാല്‍ പിന്നീട് അവരില്‍പെട്ട അതിക്രമകാരികള്‍ക്കെതിരല്ലാതെ മറ്റുള്ളവര്‍ക്കെതിരില്‍ യാതൊരുവിധ നടപടികളുണ്ടാകുവാന്‍ പാടുള്ളതല്ല.

സാമ്രാജ്യസ്ഥാപനത്തിനോ എതിരാളികളെ കൊന്നൊടുക്കുന്നതിനോ അധീശത്വം സ്ഥാപിച്ച് ആസ്വാദ്യകരമായ ജീവിതം ആസ്വദിക്കുന്നതിനോ വേണ്ടിയല്ല സത്യമെന്ന് ബോധ്യമുള്ള ആദര്‍ശമനുസരിച്ച് ജീവിക്കുന്നതിനുള്ള സ്വാതന്ത്ര്യത്തിന് വേണ്ടിയാണ് അനിവാര്യമായ സാഹചര്യത്തില്‍ ആയുധമെടുക്കുവാന്‍ ഇസ്‌ലാം അനുവദിച്ചിട്ടുള്ളത്. തങ്ങളെയും തങ്ങളുള്‍ക്കൊള്ളുന്ന ആദര്‍ശത്തെയും നശിപ്പിക്കുവാനൊരുങ്ങി സായുധ സമരം നടത്തുന്നവരോട് ആവശ്യമാണെങ്കില്‍ സായുധരായിത്തന്നെ പ്രതികരിക്കണമെന്നുതന്നെയാണ് ഇസ്‌ലാമിന്റെ നിലപാട്. ആക്രമിക്കുകയും പീഡിപ്പിക്കുകയും ഭവനങ്ങള്‍ കയ്യേറുകയും സമ്പത്തുക്കള്‍ പിടിച്ചടക്കുകയും ചെയ്യുന്നവരെ പ്രതിരോധിക്കുകയും സത്യമതമനുസരിച്ച് ജീവിക്കാന്‍ അനുവദിക്കാത്ത സാഹചര്യങ്ങളില്‍ ജീവിക്കുന്നവരെ സ്വതന്ത്രരാക്കുകയും സംരക്ഷിക്കുകയും ചെയ്യുവാനുള്ള യുദ്ധത്തില്‍പോലും അതിരുകള്‍ ലംഘിച്ചുകൂടായെന്നും അതിക്രമങ്ങള്‍ കാണിച്ചുകൂടായെന്നുമാണ് ഇസ്‌ലാം അനുശാസിക്കുന്നത്. സ്വയം ജീവിക്കുകയും മറ്റുള്ളവരെ ജീവിക്കുവാന്‍ അനുവദിക്കുകയും ചെയ്യുകയെന്നാണ് ഇസ്‌ലാം എല്ലാവരോടും അനുശാസിക്കുന്നത്. സ്വാര്‍ഥമായ ജീവിതത്തിനുവേണ്ടി മറ്റുള്ളവരെ ജീവിക്കുവാനനുവദിക്കാത്ത സാഹചര്യങ്ങളുണ്ടാകുമ്പോള്‍ അത്തരക്കാര്‍ക്കെതിരെ നടത്തുന്നതാണ് ഇസ്‌ലാം അനുവദിച്ച സായുധ സമരം. അതിക്രമകാരികളെ പ്രതിരോധിക്കുവാനായുള്ള പ്രസ്തുത സമരം നടന്നിട്ടില്ലെങ്കില്‍ സമാധാനപൂര്‍ണമായി ജീവിക്കുവാനുള്ള സ്വാതന്ത്ര്യമാണ് ഹനിക്കപ്പെടുകയെന്ന് വ്യക്തമാക്കിക്കൊണ്ടാണ് ക്വുർആൻ സായുധസമരത്തിന് അനുമതി നൽകിയതെന്ന വസ്തുത ശ്രദ്ധേയമാണ് (22: 39, 40)

വിഷയവുമായി ബന്ധപ്പെട്ട വീഡിയോ

മനുഷ്യജീവന് ഏറെ പവിത്രതയുണ്ടെന്ന് പഠിപ്പിക്കുന്ന ആദര്‍ശമാണ് ഇസ്‌ലാം. ജീവിക്കുവാനുള്ള മനുഷ്യരുടെ അവകാശെത്ത അന്യായമായി ഹനിക്കുവാന്‍ ആര്‍ക്കും അവകാശമില്ലെന്നാണ് അത് പഠിപ്പിക്കുന്നത്. ആദ്യത്തെ കൊലപാതകത്തെക്കുറിച്ച് പരാമര്‍ശിച്ചുകൊണ്ട് ക്വുര്‍ആന്‍ പറഞ്ഞു:

''അക്കാരണത്താല്‍ ഇസ്രായീല്‍ സന്തതികള്‍ക്ക് നാം ഇപ്രകാരം വിധിനല്‍കുകയുണ്ടായി: മറ്റൊരാളെ കൊന്നതിന് പകരമായോ, ഭൂമിയില്‍ കുഴപ്പമുണ്ടാക്കിയതിന്റെ പേരിലോ അല്ലാതെ വല്ലവനും ഒരാളെ കൊലപ്പെടുത്തിയാല്‍, അത് മനുഷ്യരെ മുഴുവന്‍ കൊലപ്പെടുത്തിയതിന് തുല്യമാകുന്നു. ഒരാളുടെ ജീവന്‍ വല്ലവനും രക്ഷിച്ചാല്‍, അത് മനുഷ്യരുടെ മുഴുവന്‍ ജീവന്‍ രക്ഷിച്ചതിന് തുല്യമാകുന്നു.'' (5:32)

ഒരാളും അന്യായമായി വധിക്കപ്പെട്ടുകൂടാ എന്ന് ഇസ്രാഈല്യര്‍ക്ക് നല്‍കിയ ദൈവിക ബോധനം അവര്‍ക്ക് നല്‍കപ്പെട്ട പത്ത് കല്‍പനകളില്‍ രേഖപ്പെടുത്തിയിട്ടുണ്ട്. ബൈബിള്‍ പഴയനിയമത്തിലെ പുറപ്പാട് പുസ്തകത്തിലും(പുറപ്പാട് 20:1-17)ആവര്‍ത്തന പുസ്തകത്തിലും(ആവര്‍ത്തനം 5:6-22) ഉദ്ധരിച്ചിരിക്കുന്ന പത്ത് കല്‍പനകളില്‍ ആറാമത്തേത് 'നീ കൊല്ലരുത്’എന്നാണ്.  ആദമിന് ശേഷം വന്ന പ്രവാചകന്മാരിലൂടെയെല്ലാം അന്യായമായ കൊലക്കെതിരെയുള്ള ബോധവല്‍ക്കരണമുണ്ടായിട്ടുണ്ടെന്ന് നടേ ഉദ്ധരിച്ച ക്വുര്‍ആന്‍ സൂക്തം സൂചിപ്പിക്കുന്നുണ്ട്. മറ്റൊരാളെ മാത്രമല്ല, സ്വന്തത്തെ വധിക്കാന്‍ പോലും ആര്‍ക്കും അവകാശമില്ലെന്നാണ് ഇസ്‌ലാമിന്റെ അധ്യാപനം. ക്വുര്‍ആന്‍ പറയുന്നു: ''നിങ്ങള്‍ നിങ്ങളെത്തന്നെ കൊലപ്പെടുത്തുകയും ചെയ്യരുത്. തീര്‍ച്ചയായും അല്ലാഹു നിങ്ങളോട് കരുണയുള്ളവനാകുന്നു. ആരെങ്കിലും അതിക്രമമായും അന്യായമായും അങ്ങനെ ചെയ്യുന്ന പക്ഷം നാമവനെ നരകാഗ്‌നിയിലിട്ട് കരിക്കുന്നതാണ്. അത് അല്ലാഹുവിന് എളുപ്പമുള്ള കാര്യമാകുന്നു.'' (4:29,30) ആത്മഹത്യ ചെയ്യുന്നവര്‍ക്ക് സ്വര്‍ഗം നിഷിദ്ധമാണെന്ന് മുഹമ്മദ് നബി (സ) പഠിപ്പിച്ചു: ജുന്‍ദുബില്‍നിന്ന്: നബി പറഞ്ഞു: മുറിവേറ്റിരുന്ന ഒരാള്‍ അക്കാരണത്താല്‍ ആത്മഹത്യ ചെയ്തു. അപ്പോള്‍ അല്ലാഹു പറയുന്നു:ന്നുഅവന്റെ ജീവന്റെ കാര്യത്തില്‍ എന്നെ മുന്‍കടന്ന് പ്രവര്‍ത്തിച്ചിരിക്കുന്നു. അതിനാല്‍ ഞാനവന്ന് സ്വര്‍ഗം നിഷിദ്ധമാക്കി.(സ്വഹീഹുല്‍ ബുഖാരി, കിത്താബുല്‍ ജനാഇസ്) അന്യായമായി ഒരാളുടെ രക്തം ചിന്തുവാന്‍ വഴികളന്വേഷിച്ച് നടക്കുന്നവന്‍ അല്ലാഹുവിന്റെ കോപത്തിന് ഇരയാകുന്നയാളുകളില്‍ മുന്‍പന്തിയിലായിരിക്കും ഉണ്ടാവുകയെന്നാണ് മുഹമ്മദ് നബി(സ)പഠിപ്പിച്ചത്: ഇബ്‌നുഅബ്ബാസില്‍ നിന്ന്: നബില പറഞ്ഞു: ജനങ്ങളില്‍ അല്ലാഹുവിന് ഏറ്റവും കോപമുണ്ടാവുക മൂന്ന് വിഭാഗം ആളുകളാണ്: മസ്ജിദുല്‍ ഹറാമില്‍ വെച്ച് അക്രമം പ്രവര്‍ത്തിക്കുന്നവന്‍, ജാഹിലീ കാലഘട്ടത്തിലെ നടപടികള്‍ പാലിക്കാന്‍ ആഗ്രഹിക്കുന്നവന്‍, അന്യായമായി ഒരാളുടെ രക്തം ചിന്താന്‍ വഴി അനേഷിക്കുന്നവന്‍.(സ്വഹീഹുല്‍ ബുഖാരി, കിതാബു ദ്ദിയാത്ത്)

അന്ത്യനാളില്‍ ആദ്യമായി വിധി തീര്‍പുകല്‍പിക്കപ്പെടുക കൊലക്കുറ്റത്തിന്റെ കാര്യത്തിലായിരിക്കുമെന്നും നബി (സ) പഠിപ്പിച്ചു: അബ്ദുല്ലയില്‍ നിന്ന്: നബി (സ)  പറഞ്ഞു: പരലോകത്ത് ആദ്യമായി വിധി തീര്‍പ്പുണ്ടാക്കുക കൊലക്കുറ്റങ്ങള്‍ക്കാണ്.(സ്വഹീഹുല്‍ ബുഖാരി, കിതാബു ര്‍റിഖാഖ്)

ഇസ്‌ലാമിക സമൂഹത്തില്‍ ജീവിക്കുന്ന ഒരാള്‍ക്ക് മതം നല്‍കുന്ന വിശാലത അനുഭവിക്കാനാവുക നിരപരാധികളുടെ രക്തം ചിന്താത്ത കാലത്തോളം മാത്രമായിരിക്കുമെന്ന് നബില വ്യക്തമാക്കി: ഇബ്‌നുഉമറില്‍നിന്ന്: റസൂല്‍(സ)പറഞ്ഞു: ആദരണീയ (നിരപരാധിയുടെ) രക്തം ചീന്താതിരിക്കുന്നിടത്തോളം സത്യവിശ്വാസിക്ക് ദീനിന്റെ വിശാലത ലഭിച്ചുകൊണ്ടിരിക്കും.(സ്വഹീഹുല്‍ ബുഖാരി, കിതാബു ദ്ദിയാത്ത്)

സത്യവിശ്വാസിയുടെ കരങ്ങളില്‍ നിന്ന് നിരപരാധികള്‍ എപ്പോഴും സുരക്ഷിതരായിരിക്കുമെന്ന് ക്വുര്‍ആന്‍ വ്യക്തമാക്കുന്നുണ്ട്. സത്യവിശ്വാസികളുടെ സ്വഭാവ വിശേഷങ്ങളെക്കുറിച്ച് പരാമര്‍ശിക്കവെ ക്വുര്‍ആന്‍ പറഞ്ഞു: ''അല്ലാഹുവോടൊപ്പം വേറെയൊരു ദൈവത്തെയും വിളിച്ചു പ്രാര്‍ഥിക്കാത്തവരും, അല്ലാഹു പവിത്രമാക്കി വെച്ചിട്ടുള്ള ജീവനെ ന്യായമായ കാരണത്താലല്ലാതെ ഹനിച്ചു കളയാത്തവരും, വ്യഭിചരിക്കാത്തവരുമാകുന്നു അവര്‍. ആ കാര്യങ്ങള്‍ വല്ലവനും ചെയ്യുന്ന പക്ഷം അവന്‍ പാപഫലം കണ്ടെത്തുക തന്നെ ചെയ്യും.'' (25:68)

നിഷിദ്ധമാക്കപ്പെട്ട കാര്യങ്ങളെക്കുറിച്ച് എണ്ണിപ്പറഞ്ഞപ്പോഴും കൊലപാതകത്തെ പ്രത്യേകമായി ക്വുര്‍ആന്‍ എടുത്തുപറഞ്ഞതായി കാണാന്‍ കഴിയും: ''(നബിയേ) പറയുക: നിങ്ങള്‍ വരൂ! നിങ്ങളുടെ രക്ഷിതാവ് നിങ്ങളുടെ മേല്‍ നിഷിദ്ധമാക്കിയത് നിങ്ങള്‍ക്ക് ഞാന്‍ പറഞ്ഞ് കേള്‍പിക്കാം. അവനോട് യാതൊന്നിനെയും നിങ്ങള്‍ പങ്കുചേര്‍ക്കരുത്. മാതാപിതാക്കള്‍ക്ക് നന്‍മചെയ്യണം. ദാരിദ്ര്യം കാരണമായി സ്വന്തം മക്കളെ നിങ്ങള്‍ കൊന്നുകളയരുത്. നാമാണ് നിങ്ങള്‍ക്കും അവര്‍ക്കും ആഹാരം തരുന്നത്. പ്രത്യക്ഷവും പരോക്ഷവുമായ നീചവൃത്തികളെ നിങ്ങള്‍ സമീപിച്ച് പോകരുത്. അല്ലാഹു പരിപാവനമാക്കിയ ജീവനെ ന്യായപ്രകാരമല്ലാെത നിങ്ങള്‍ ഹനിച്ചുകളയരുത്. നിങ്ങള്‍ ചിന്തിച്ച് മനസ്സിലാക്കുവാന്‍ വേണ്ടി. അവന്‍ (അല്ലാഹു) നിങ്ങള്‍ക്ക് നല്‍കിയ ഉപേദശമാണത്.'' (6:151)

മാതാവിന്റെ ഗര്‍ഭാശയത്തില്‍ ഉരുവം പ്രാപിച്ചു കഴിഞ്ഞാല്‍ എല്ലാവര്‍ക്കും ജനിക്കുവാനും ജീവിക്കുവാനും അവകാശമുണ്ടെന്നതാണ് ഇസ്‌ലാമിന്റെ നിലപാട്. പട്ടിണി ഭയന്ന് മക്കളെ കൊല്ലുവാന്‍ ധൃഷ്ടരായിരുന്ന ജാഹിലിയ്യാ അറബികളുടെ സമ്പ്രദായെത്ത വിമര്‍ശിക്കുന്ന ക്വുര്‍ആന്‍ വചനങ്ങള്‍ ഇന്നു യോജിക്കുക ജനസംഖ്യാ വര്‍ധനവിനെക്കുറിച്ച ഭീതിപരത്തി ഭ്രൂണഹത്യക്ക് പ്രേരിപ്പിക്കുന്നവരുടെ ചെയ്തികള്‍ക്കാണ്. ക്വുര്‍ആന്‍ നിഷ്‌കര്‍ഷിക്കുന്നത് കാണുക: ''ദാരിദ്ര്യഭയത്താല്‍ നിങ്ങള്‍ നിങ്ങളുടെ കുഞ്ഞുങ്ങളെ കൊന്നുകളയരുത്. നാമാണ് അവര്‍ക്കും നിങ്ങള്‍ക്കും ഉപജീവനം നല്‍കുന്നത്. അവരെ കൊല്ലുന്നത് തീര്‍ച്ചയായും ഭീമമായ അപരാധമാകുന്നു.'' (17:31) 'അല്ലാഹു പവിത്രമാക്കിയ ജീവന്‍ ഹനിക്കപ്പെടരുത്'എന്ന ക്വുര്‍ആനിക നിര്‍ദേശത്തോടൊപ്പം തന്നെ ന്യായമായ കാരണങ്ങളാലല്ലാതെ'യെന്നുകൂടി പറയുന്നുണ്ടെന്ന വസ്തുത ശ്രദ്ധേയമാണ്. ഒരു കാരണവശാലും ജീവന്‍ ഹനിക്കപ്പെട്ടുകൂടാ എന്ന അപ്രായോഗികമായ അഹിംസാ നിയമമല്ല ഇസ്‌ലാം പഠിപ്പിക്കുന്നത്. ന്യായമായ കാരണങ്ങളാല്‍ ചിലപ്പോള്‍ ജീവന്‍ ഹനിക്കേണ്ടിവരുമെന്ന വസ്തുത ഇസ്‌ലാം അംഗീകരിക്കുന്നു. ജീവന്‍ ഹിംസിക്കപ്പെടേണ്ട അനിവാര്യമായ സാഹചര്യങ്ങളെന്തൊക്കെയാണെന്ന് കുര്‍ആന്‍ തന്നെ വ്യക്തമാക്കുകയും ചെയ്യുന്നു. നിരപരാധിയുടെ കൊലപാതകം മനുഷ്യരാശിയെ മുഴുവന്‍ കൊല്ലുന്നതിന് സമാനമാണെന്ന് പ്രഖ്യാപിക്കുന്ന ക്വുര്‍ആന്‍ സൂക്തം തന്നെ ഇക്കാര്യം കൂടി പരാമര്‍ശിക്കുന്നുണ്ട്. (5:32) 'ഒരാളെ കൊന്നതിന് പകരമായോ ഭൂമിയില്‍ കുഴപ്പമുണ്ടാക്കുന്നതിന് പകരമായോ ആണ് ഒരാള്‍ വധിക്കപ്പെടേണ്ടത്' എന്ന് ഈ സൂക്തം വ്യക്തമാക്കുന്നു. കൊലപാതകം, കുഴപ്പം എന്നിവ കൊലക്കുറ്റം അര്‍ഹിക്കുന്ന തെറ്റുകളാണെന്നര്‍ഥം. ഈ തെറ്റുകള്‍ ചെയ്തയാളുടെ ജീവനെടുക്കുന്നത് ന്യായവും അതല്ലാത്തവ അന്യായവുമായ കൊലപാതകങ്ങളാണെന്ന് ഇതില്‍ നിന്ന് സുതരാം വ്യക്തമാണ്. തെറ്റുകള്‍ക്കുള്ള ശിക്ഷ വിധിക്കേണ്ടത് ന്യായാധിപനാണ്. കൊലപാതകത്തിനും കുഴപ്പത്തിനുമുള്ള ശിക്ഷയും വിധിക്കേണ്ടത് അയാള്‍ തന്നെ. ഭരണാധികാരിയുടെയോ ന്യായാധിപന്റെയൊ നിര്‍ദേശപ്രകാരമുള്ളതല്ലാതെയുള്ള കൊലപാതകങ്ങളെല്ലാം അന്യായമാണെന്നര്‍ഥം

. അബദ്ധത്തില്‍ സംഭവിച്ചുപോകുന്നതാണെങ്കില്‍ പോലും കൊലപാതകം അന്യായമായി തന്നെ മനസ്സിലാക്കുകയും അതിനുള്ള പ്രായശ്ചിത്തം ചെയ്യുകയും വേണമെന്ന് ഇസ്‌ലാമിക പാഠത്തില്‍ നിന്ന് ഇത് സുതരാം വ്യക്തമാകുന്നുണ്ട്. ക്വുര്‍ആന്‍ പഠിപ്പിക്കുന്നത് കാണുക: ''യാതൊരു വിശ്വാസിക്കും മറ്റൊരു വിശ്വാസിയെ കൊല്ലാന്‍ പാടുള്ളതല്ല; അബദ്ധത്തില്‍ വന്നുപോകുന്നതല്ലാതെ. എന്നാല്‍ വല്ലവനും ഒരു വിശ്വാസിയെ അബദ്ധത്തില്‍ കൊന്നുപോയാല്‍ (പ്രായശ്ചിത്തമായി) ഒരു വിശ്വാസിയായ അടിമയെ മോചിപ്പിക്കുകയും, അവന്റെ (കൊല്ലപ്പെട്ടവന്റെ) അവകാശികള്‍ക്ക് നഷ്ടപരിഹാരം നല്‍കുകയുമാണ് വേണ്ടത്. അവര്‍ (ആ അവകാശികള്‍) അത് ഉദാരമായി വിട്ടുതന്നെങ്കിലൊഴികെ. ഇനി അവന്‍ (കൊല്ലപ്പെട്ടവന്‍) നിങ്ങളോട് ശത്രുതയുള്ള ജനവിഭാഗത്തില്‍ പെട്ടവനാണ്; അവനാണെങ്കില്‍ സത്യവിശ്വാസിയുമാണ് എങ്കില്‍ സത്യവിശ്വാസിയായ ഒരു അടിമയെ മോചിപ്പിക്കുക മാത്രമാണ് വേണ്ടത്. ഇനി അവന്‍ (കൊല്ലപ്പെട്ടവന്‍) നിങ്ങളുമായി സഖ്യത്തിലിരിക്കുന്ന ഒരു ജനവിഭാഗത്തില്‍ പെട്ടവനാണെങ്കില്‍ അവന്റെ അവകാശികള്‍ക്ക് നഷ്ടപരിഹാരം നല്‍കുകയും വിശ്വാസിയായ ഒരു അടിമയെ മോചിപ്പിക്കുകയും ചെയ്യേണ്ടതാണ്. വല്ലവന്നും അത് സാധിച്ച് കിട്ടിയില്ലെങ്കില്‍ തുടര്‍ച്ചയായി രണ്ടുമാസം നോമ്പനുഷ്ഠിക്കേണ്ടതാണ്. അല്ലാഹു നിശ്ചയിച്ച പശ്ചാത്താപ(മാര്‍ഗ)മാണത്. അല്ലാഹു എല്ലാം അറിയുന്നവനും യുക്തിമാനുമാകുന്നു.'' (4:92)

കൊലപാതകത്തിനും കുഴപ്പത്തിനുമുള്ള ശിക്ഷയായി നല്‍കുന്ന വധശിക്ഷയാകട്ടെ സമൂഹത്തിന്റെ സുസ്ഥിതിക്കുവേണ്ടി നടപ്പാക്കപ്പെടേണ്ട പ്രതിക്രിയാ നടപടിയാണെന്നാണ് ഇസ്‌ലാമിന്റെ പക്ഷം. അത് നടപ്പാക്കപ്പെടാത്തപക്ഷം സമൂഹത്തില്‍ അന്യായമായ കൊലപാതകങ്ങള്‍ പെരുകും. കൊലയാളിയും കുഴപ്പമുണ്ടാക്കി സമൂഹത്തില്‍ കലാപങ്ങള്‍ വിതയ്ക്കുന്നവരും മാതൃകാപരമായി ശിക്ഷിക്കപ്പെടുമ്പോഴാണ് സ്വന്തം ജീവന്‍ ഹനിക്കപ്പെടുമോയെന്ന ഭയപ്പാടില്ലാതെ സമാധാനത്തോടെ പൗരന്മാര്‍ക്കെല്ലാം ജീവിക്കാന്‍ കഴിയുക. വധശിക്ഷ നിഷ്‌കര്‍ഷിക്കുന്ന ക്വുര്‍ആന്‍ സൂക്തം ഇക്കാര്യം സുതരാം വ്യക്തമാക്കുന്നുണ്ട്. ''സത്യവിശ്വാസികളേ, കൊലചെയ്യപ്പെടുന്നവരുടെ കാര്യത്തില്‍ തുല്യശിക്ഷ നടപ്പാക്കുക എന്നത് നിങ്ങള്‍ക്ക് നിയമമാക്കപ്പെട്ടിരിക്കുന്നു. സ്വതന്ത്രനു പകരം സ്വതന്ത്രനും, അടിമയ്ക്കു പകരം അടിമയും, സ്ത്രീക്കു പകരം സ്ത്രീയും (കൊല്ലപ്പെടേണ്ടതാണ്.) ഇനി അവന്ന് (കൊലയാളിക്ക്) തന്റെ സഹോദരന്റെ പക്ഷത്ത് നിന്ന് വല്ല ഇളവും ലഭിക്കുകയാണെങ്കില്‍ അവന്‍ മര്യാദ പാലിക്കുകയും, നല്ല നിലയില്‍ (നഷ്ടപരിഹാരം) കൊടുത്തു വീട്ടുകയും ചെയ്യേണ്ടതാകുന്നു. നിങ്ങളുടെ രക്ഷിതാവിങ്കല്‍ നിന്നുള്ള ഒരു വിട്ടുവീഴ്ചയും കാരുണ്യവുമാകുന്നു അത്. ഇനി അതിനു ശേഷവും ആരെങ്കിലും അതിക്രമം പ്രവര്‍ത്തിക്കുകയാണെങ്കില്‍ അവന് വേദനയേറിയ ശിക്ഷയുണ്ടായിരിക്കും. ബുദ്ധിമാന്‍മാരേ, (അങ്ങനെ) തുല്യശിക്ഷ നല്‍കുന്നതിലാണ് നിങ്ങളുടെ ജീവിതത്തിന്റെ നിലനില്‍പ്. നിങ്ങള്‍ സൂക്ഷ്മത പാലിക്കുന്നതിനു വേണ്ടിയത്രെ (ഈ നിയമനിര്‍േദശങ്ങള്‍).'' (2:178, 179)

കുറ്റവാളി മാത്രമാണ് ശിക്ഷിക്കപ്പെടേണ്ടതെന്നും ഒരാളുടെ അടിമയെ വധിച്ചാല്‍ തിരിച്ച് കൊലയാളിയുടെ അടിമയെ വധിക്കുന്നത് പോലെയുള്ള പ്രാകൃത സമ്പ്രദായങ്ങള്‍ അനീതിയാണെന്നും അവ വര്‍ജിക്കേണ്ടവയാണെന്നും കൊലയാളിയെ ശിക്ഷിക്കണമോ അതല്ല നഷ്ടപരിഹാരം സ്വീകരിച്ച് വെറുതെ വിടണമോ നഷ്ടപരിഹാരത്തില്‍ ഇളവ് ചെയ്യണമോ എന്നെല്ലാം തീരുമാനിക്കുവാനുള്ള ആത്യന്തികമായ അധികാരം കൊല്ലപ്പെട്ടവന്റെ അവകാശികള്‍ക്കാണെന്നും വ്യക്തമാക്കുന്ന ഈ ക്വുര്‍ആന്‍ സൂക്തങ്ങളിലെ പരാമര്‍ശങ്ങള്‍ അവസാനിപ്പിക്കുന്നത് ഭബുദ്ധിമാന്‍മാരേ, തുല്യശിക്ഷ നല്‍കുന്നതിലാണ് നിങ്ങളുടെ ജീവിതത്തിന്റെ നിലനില്‍പ്’ എന്ന് പറഞ്ഞുകൊണ്ടാണെന്ന വസ്തുത ശ്രദ്ധേയമാണ്. കുറ്റവാളികള്‍ ശിക്ഷിക്കപ്പെടാതിരിക്കുകയും സമൂഹത്തില്‍ അവര്‍ക്ക് സൈ്വര്യവിഹാരം നടത്തുവാന്‍ അവസരമുണ്ടാവുകയും ചെയ്താല്‍ സമാധാനപൂര്‍ണമായി ജീവിതം സാധ്യമാകാത്ത സ്ഥിതിയാണ് സംജാതമാവുകയെന്നും അതുകൊണ്ട് തന്നെ പ്രതിക്രിയ െചയ്യേണ്ടത് സമൂഹത്തിന്റെ ആവശ്യമാണെന്നും വ്യക്തമാക്കുകയാണ് ഈ സൂക്തങ്ങള്‍ ചെയ്യുന്നത്. നാട്ടില്‍ കലാപങ്ങളും കുഴപ്പങ്ങളുമുണ്ടാക്കുകയും സൈ്വര്യ ജീവിതത്തെ താറുമാറാക്കുകയും ചെയ്യുന്നവര്‍ക്കും നിഷ്‌കൃഷ്ടമായ ശിക്ഷകള്‍ തന്നെ നല്‍കണമെന്നാണ് ക്വുര്‍ആനിന്റെ അനുശാസന: ''അല്ലാഹുവോടും അവെന്റ ദൂതനോടും പോരാടുകയും, ഭൂമിയില്‍ കുഴപ്പമുണ്ടാക്കാന്‍ ശ്രമിക്കുകയും ചെയ്യുന്നവര്‍ക്കുള്ള പ്രതിഫലം അവര്‍ കൊന്നൊടുക്കപ്പെടുകയോ, ക്രൂശിക്കപ്പെടുകയോ, അവരുടെ കൈകളും കാലുകളും എതിര്‍വശങ്ങളില്‍ നിന്നായി മുറിച്ചുകളയപ്പെടുകയോ, നാടുകടത്തപ്പെടുകയോ ചെയ്യുക മാത്രമാകുന്നു. അതവര്‍ക്ക് ഇഹലോകത്തുള്ള അപമാനമാകുന്നു. പരലോകത്ത് അവര്‍ക്ക് കനത്ത ശിക്ഷയുമുണ്ടായിരിക്കും. എന്നാല്‍, അവര്‍ക്കെതിരില്‍ നടപടിയെടുക്കാന്‍ നിങ്ങള്‍ക്ക് കഴിയുന്നതിന്റെ മുമ്പായി പശ്ചാത്തപിച്ച് മടങ്ങിയവര്‍ ഇതില്‍ നിന്നൊഴിവാണ്. അല്ലാഹു ഏറെ പൊറുക്കുന്നവനും കരുണാനിധിയുമാണെന്ന് നിങ്ങള്‍ മനസ്സിലാക്കുക.'' (5:33,34)

കൊലപാതകങ്ങളും കുഴപ്പങ്ങളുമില്ലാത്ത ഒരു സമൂഹത്തിന്റെ സൃഷ്ടിയാണ് ഇസ്‌ലാം വിഭാവനം ചെയ്യുന്നത്. കൊലയാളിയെയും കുഴപ്പക്കാരെയും കലാപകാരികളെയും സൈ്വര്യമായ സാമൂഹ്യ ജീവിതത്തെ തകര്‍ക്കുന്നവരെയും മാതൃകാപരമായി ശിക്ഷിക്കണമെന്ന് പറയുമ്പോള്‍ അതുവഴി ഇത്തരം പ്രശ്‌നങ്ങളൊന്നുമില്ലാത്ത സമാധാനപൂര്‍ണമായ സാമൂഹ്യാന്തരീക്ഷം സൃഷ്ടിക്കണമെന്ന ലക്ഷ്യമാണ് ഇസ്‌ലാമിനുള്ളത്. ആദർശവിരോധികളെയെല്ലാം കൊന്നൊടുക്കണമെന്നല്ല, മുസ്ലിംകളുമായി യുദ്ധത്തിന് വരുന്നവർക്കെതിരെ സായുധമായി പോരാടണമെന്നാണ് ഇസ്‌ലാമിന്റെ കല്പന. ഭരണാധികാരിയുടെ കല്പനപ്രകാരമുള്ള വധശിക്ഷയും യുദ്ധത്തിൽ ശത്രുവിനെതിയുള്ള സായുധമുന്നേറ്റവുമൊഴിച്ചുള്ള സന്ദര്ഭങ്ങളിലൊന്നും ആർക്കെതിരെയും ആയുധമെടുക്കുവാൻ ഇസ്‌ലാം വിശ്വാസികളെ അനുവദിച്ചിട്ടില്ല. ഇസ്‌ലാമിനെതിരില്‍ യുദ്ധം ചെയ്യുകയും നാട്ടില്‍ കുഴപ്പങ്ങളുണ്ടാക്കുകയും ചെയ്തവർ പോലും പിടിക്കപ്പെടുന്നതിന് മുമ്പ്  പശ്ചാത്തപിക്കുകയും തങ്ങളുടെ നിലപാട് നന്നാക്കിത്തീര്‍ക്കുകയും ചെയ്താല്‍ അവര്‍ ശിക്ഷയില്‍ നിന്ന് ഒഴിവാണെന്ന് പ്രത്യേകം എടുത്തുപറഞ്ഞിരിക്കുന്നത് അതുകൊണ്ടാണ്. ശിക്ഷിക്കുകയല്ല, സമാധാനപൂര്‍ണമായ സാമൂഹ്യാന്തരീക്ഷം സൃഷ്ടിക്കുകയാണ് ഇസ്‌ലാമിക നിയമങ്ങളുടെ ലക്ഷ്യം. കുഴപ്പങ്ങളും കലാപകങ്ങളും കൊലപാതകങ്ങളും അധാര്‍മികതകളും ഇല്ലാത്ത എല്ലാവര്‍ക്കും ശാന്തിയോടെ ജീവിക്കാനാവുന്ന ഒരു സമൂഹത്തിന്റെ സൃഷ്ടി.

പീഡനങ്ങളുടെ മക്കാനാളുകളില്‍ പ്രവാചകന്‍(സ)സായുധമായി പ്രതികരിക്കാതിരുന്നത് ശക്തിയില്ലാത്തതിനാലായിരുന്നുവെന്നും മദീനയിലെത്തി താന്‍ ശക്തനാണെന്ന് തോന്നിയതു മുതല്‍ക്കാണ് സായുധപ്രതികരണങ്ങളും അതിക്രമങ്ങളുമാരംഭിച്ചതെന്നും വിമര്‍ശിക്കുന്നവര്‍ രണ്ടുസ്ഥലത്തെയും നബി(സ)യുടെ ഉത്തരവാദിത്തങ്ങളിലുള്ള വ്യത്യാസത്തെക്കുറിച്ച് നിശ്ശബ്ദരാവുകയാണ് ചെയ്യുന്നത്. രഹസ്യമായി ഇസ്‌ലാമികപ്രബോധനം നടന്നിരുന്ന ആദ്യകാലത്ത് മുസ്‌ലിംകള്‍ തീരെ ദുര്‍ബലരായിരുന്നുവെന്നും തിരിച്ചടിക്കുന്നതിനെക്കുറിച്ച് ചിന്തിക്കുവാന്‍ പോലും കഴിയാത്തവരായിരുന്നുവെന്നും പറയുന്നത് വാസ്തവമാണ്.എന്നാൽ മക്കാകാലം മുഴുവൻ തിരിച്ചടിയെക്കുറിച്ച് ആലോചിക്കാൻ പോലും കഴിയാത്തവിധം മുസ്ലിംകൾ ദുര്ബലരായിരുന്നുവെന്ന് പറയുന്നത് ശരിയല്ല. തിരിച്ചടിയെപ്പറ്റി പ്രവാചകാനുചരന്മാർ പോലും ചിന്തിച്ചിരുന്ന സന്ദർഭങ്ങൾ മക്കാകാലഘട്ടത്തിലുണ്ടായിരുന്നു എന്നതാണ് നേര്.

ധീരവീരശൂര പരാക്രമികളായി അറിയപ്പെട്ടിരുന്ന ഹംസ(റ)യു  ഉമറിന്റെയും(റ) ഇസ്‌ലാം സ്വീകരണത്തോടെത്തന്നെ തങ്ങളുടെ ആദർശം പരസ്യമായി പ്രഖ്യാപിക്കുവാനുള്ള ധൈര്യം മുസ്ലിംകൾക്കുണ്ടാകാൻ തുടങ്ങിയിരുന്നു. കഅ്ബാലയത്തിനു മുമ്പില്‍ വെച്ച് പരസ്യമായി നമസ്കരിക്കുവാനും  ഇസ്‌ലാമിനെക്കുറിച്ച് ഉറക്കെ ചര്‍ച്ച ചെയ്യുവാനും സത്യമതത്തിലേക്ക് പൊതുസ്ഥലങ്ങളില്‍ വെച്ച് ആളുളെ ക്ഷണിക്കുവാനുമെല്ലാമുള്ള ധൈര്യം അതിന്നുശേഷം മുസ്‌ലിംകള്‍ പ്രകടിപ്പിക്കാൻ തുടങ്ങി. . മക്കയില്‍ തങ്ങളെ പ്രയാസപ്പെടുത്തിക്കൊണ്ടിരിക്കുന്നവര്‍ക്കെതിരെ ഒരു കലാപം അഴിച്ചുവിടുവാന്‍ പ്രവാചകന്‍(സ)കല്‍പിച്ചാല്‍ അത് ശിരസ്സാവഹിക്കുവാന്‍ അവര്‍ക്ക് കഴിയുമായിരുന്നു. ഉമറിനെ(റ)പ്പോലുള്ളവരുടെ നേതൃത്വത്തില്‍ അത്തരമൊരു കലാപമുണ്ടായാല്‍ അതുമൂലം മക്കാമുശ്‌രിക്കുകള്‍ക്ക് വമ്പിച്ച നാശനഷ്ടങ്ങളുണ്ടാക്കുവാനും അത് നിമിത്തമാകുമായിരുന്നു. എന്നാല്‍ തങ്ങള്‍ ജീവിക്കുന്ന നാട്ടില്‍ കുഴപ്പുമുണ്ടാക്കുവാനല്ല, പ്രത്യുത നിലനില്‍ക്കുന്ന സാമൂഹ്യസാഹചര്യങ്ങളുടെ സാധ്യതകളുപയോഗിച്ച് തങ്ങള്‍ അനുഭവിച്ചുകൊണ്ടിരിക്കുന്ന പീഡനങ്ങള്‍ക്ക് അറുതിവരുത്തുവാനും തങ്ങള്‍ക്ക് മുസ്‌ലിംകളായി നിലനില്‍ക്കുവാനുമുള്ള സാഹചര്യങ്ങളൊരുക്കുവാനാണ് പ്രവാചകന്‍(സ)പരിശ്രമിച്ചത്. കലാപങ്ങള്‍ സമൂഹത്തിന്റെ സുസ്ഥിതിയെ ദോഷകരമായി ബാധിക്കുമെന്നതുകൊണ്ടാവാം, അത്തരമൊരു മുന്നേറ്റത്തിന് പ്രവാചകന്‍(സ)  പ്രചോദനം നല്‍കാതിരുന്നത്.

സായുധസന്നാഹങ്ങളും സംഘട്ടനങ്ങളുമെല്ലാം നന്നായി അറിയുവന്നവരായിരുന്നു മുസ്‌ലിംകളായിത്തീര്‍ന്ന മക്കയിലുള്ളവര്‍. ഗോത്രാഭിമാനത്തിനുവേണ്ടി തലമുറകള്‍ നീണ്ട യുദ്ധം ചെയ്തു പരിചയമുള്ള അവര്‍ക്ക് ശത്രുവിനെതിരെ സായുധസമരം നടത്തുന്നതിനുള്ള നിര്‍ദേശം മാത്രം മതിയായിരുന്നു, തങ്ങളെ പീഡിപ്പിക്കുന്നവര്‍ക്കെതിരെ ആഞ്ഞടിക്കുവാന്‍. ഉമര്‍(റ)ഇസ്‌ലാം സ്വീകരിച്ചതോടെ മുശ്‌രിക്കുകള്‍ പോലും തിരിച്ചടി പ്രതീക്ഷിച്ചിരുന്നുവെന്ന് അവരുടെ പ്രതികരണങ്ങളില്‍ നിന്ന് വ്യക്തമാകുന്നുണ്ടെന്ന വസ്തുത ഹദീഥ് ഗ്രൻഥങ്ങളിൽ നിന്ന് മനസ്സിലാകുന്നുണ്ട്. സ്വഹാബിമാരില്‍ പലരും അത്തരമൊരു തിരിച്ചടി ആഗ്രഹിക്കുകയും അതാണ് തങ്ങള്‍ക്ക് അഭിമാനകരമായ അസ്തിത്വം പ്രദാനം ചെയ്യുകയെന്ന് കരുതുകയും ചെയ്തുവെങ്കിലും പ്രവാചകന്‍(സ)അതിന് അനുവദിച്ചില്ല. ഒരു ഹദീഥ് നോക്കുക: ഇബ്‌നു അബ്ബാസില്‍ നിന്ന്: മക്കയില്‍ വെച്ച് അബ്ദുര്‍റഹ്മാനു ബ്‌നു ഔഫും അദ്‌ദേഹത്തിന്റെ സഖാക്കളും കൂടി പ്രവാചകന്റെ(സ)യടുക്കല്‍ ചെന്നു ചോദിച്ചു: 'ദൈവദൂതരേ, ഞങ്ങള്‍ ബഹുദൈവാരാധകരായിരുന്നപ്പോള്‍ ഞങ്ങള്‍ ആത്മാഭിമാനമുള്ളവരും ആദരിക്കപ്പെട്ടവരുമായിരുന്നു; വിശ്വാസികളായേതാടെ ഞങ്ങള്‍ അധഃസ്ഥിതരും അടിച്ചമര്‍ത്തപ്പെടുന്നവരുമായിത്തീര്‍ന്നു.' പ്രവാചകന്‍(സ)പറഞ്ഞു: 'ഞാന്‍ ക്ഷമിക്കുവാനാണ് കല്‍പിക്കപ്പെട്ടരിക്കുന്നത്; അതിനാല്‍ നിങ്ങള്‍ സായുധ സമരം നടത്തരുത്.'(സുനനുന്നസാഈ, കിതാബുല്‍ ജിഹാദ്, മുഹമ്മദ് ബിന്‍ അലി ബിന്‍ അല്‍ ഹസനില്‍ നിന്ന് ഇമാം ത്വബ്‌രി തന്റെ തഫ്‌സീറില്‍ (5/108) ഉദ്ധരിച്ച ഈ ഹദീഥ് സ്വഹീഹാണെന്ന്ഇമാം ഹാകിം (2/66) വ്യക്തമാക്കിയിട്ടുണ്ട്. Hafiz Abu Tahir Zubair Ali Za'i (Ed& Ref) English Translation of Sunan An-Nasai, Riyadh, 2008, Page 16)

ആളുകളുടെ എണ്ണക്കുറവാണ് പ്രവാചകനെ(സ) മക്കയില്‍വെച്ച് സായുധപ്രതികരണങ്ങളില്‍ നിന്ന് തടഞ്ഞുനിര്‍ത്തിയതെന്ന വിമര്‍ശനവും അടിസ്ഥാനരഹിതമാണ്. എണ്ണക്കുറവ് മുസ്‌ലിംകളെ സായുധ സമരത്തില്‍ നിന്ന് തടഞ്ഞുനിര്‍ത്തുകയില്ലെന്ന് ബദ്‌റിന്റെ ചരിത്രം പഠിച്ചാല്‍ ആര്‍ക്കും ബോധ്യപ്പെടും. മുസ്‌ലിം സൈന്യത്തിന്റെ എണ്ണം ശത്രുക്കളുടേതിന്റെ മൂന്നിലൊന്ന് മാത്രമാണന്ന വസ്തുതയോ അവരുടെയത്ര ആയുധങ്ങളോ വാഹനങ്ങളോ തങ്ങളുടെ പക്കലില്ലെന്ന ബോധമോ മുസ്‌ലിംകളെ ബദ്‌റില്‍ നിന്ന് പിന്തിരിപ്പിച്ചില്ലെങ്കില്‍ മക്കയില്‍ വെച്ച് മാത്രം തങ്ങള്‍ എണ്ണത്തില്‍ കുറവാണെന്ന ബോധം അവരെ സായുധ പ്രതികരണത്തില്‍ നിന്ന് തടഞ്ഞുവെന്ന് കരുതുന്നതില്‍ ന്യായമില്ല. അല്ലാഹുവിന്റെ അനുഗ്രഹമുണ്ടെങ്കില്‍ ചെറിയ സംഘങ്ങള്‍ക്ക് വലിയ സംഘങ്ങളെ തോല്‍പിക്കാനാകുമെന്ന വസ്തുത ക്വുര്‍ആന്‍ വ്യക്തമാക്കുന്നുമുണ്ട്. ''എത്രയെത്ര ചെറിയ സംഘങ്ങളാണ് അല്ലാഹുവിന്റെ അനുമതിയോടെ വലിയ സംഘങ്ങളെ കീഴ്‌പെടുത്തിയിട്ടുള്ളത്! അല്ലാഹു ക്ഷമിക്കുന്നവരുടെ കൂടെയാകുന്നു.'' (2:249)

വിശ്വാസികള്‍ക്ക് തങ്ങളെക്കാള്‍ പത്തിരട്ടി വരുന്നവരെ നേരിടാനാകുമെന്നാണ് ക്വുര്‍ആന്‍ നല്‍കുന്ന ഉല്‍ബോധനം. ''നബിയേ, നീ വിശ്വാസികളെ യുദ്ധത്തിന് പ്രോത്‌സാഹിപ്പിക്കുക. നിങ്ങളുടെ കൂട്ടത്തില്‍ ക്ഷമാശീലരായ ഇരുപത് പേരുണ്ടായിരുന്നാല്‍ ഇരുനൂറ് പേരെ അവര്‍ക്ക് ജയിച്ചടക്കാവുന്നതാണ്. നിങ്ങളുടെ കൂട്ടത്തില്‍ നൂറ് പേരുണ്ടായിരുന്നാല്‍ സത്യനിഷേധികളില്‍ നിന്ന് ആയിരം പേരെ അവര്‍ക്ക് ജയിച്ചടക്കാവുന്നതാണ്. അവര്‍ കാര്യം ഗ്രഹിക്കാത്ത ഒരു ജനവിഭാഗമാണ് എന്നതുകൊണ്ടത്രെ അത്.'' (8:65).

അനുയായികളുടെ എണ്ണക്കുറവോ മുശ്‌രിക്കുകളെ തങ്ങള്‍ക്ക് തോല്‍പിക്കാന്‍ കഴികയില്ലെന്ന തിരിച്ചറിവോ അനുചരന്‍മാര്‍ യുദ്ധസന്നദ്ധരല്ലാത്തതോ അല്ല മക്കയിലെ പീഡകര്‍ക്കെതിരെ ഒരു സായുധ കലാപത്തിന് ആഹ്വാനം ചെയ്യുന്നതില്‍ നിന്ന് മുഹമ്മദ് നബി(സ)യെ തടഞ്ഞു നിര്‍ത്തിയതെന്ന് ഇതില്‍ നിന്ന് വ്യക്തമാണ്. ജനിച്ചു വളര്‍ന്ന നാട്ടില്‍ കലാപമുണ്ടാക്കുകയോ നാട്ടിലെ സമാധാനാന്തരീക്ഷം തകര്‍ത്ത് തന്റെ ആധിപത്യം സ്ഥാപിക്കുകയോ തങ്ങളെ എതിര്‍ത്തവരെയെല്ലാം സായുധമായി സംഹരിക്കുകയോ ആയിരുന്നില്ല അദ്‌ദേഹത്തിന്റെ ലക്ഷ്യം. പ്രത്യുത സത്യമതത്തിലേക്ക് ജനങ്ങളെ ക്ഷണിക്കുകയും അങ്ങനെ അവര്‍ക്ക് സമാധാന സംതൃപ്തമായ ഇഹലോക ജീവിതവും ശാന്തസുന്ദരമായ മരണാനന്തര ജീവിതവും നേടിയെടുക്കുവാനുള്ള മാര്‍ഗമൊരുക്കുകയുമായിരുന്നു അദ്‌ദേഹം ചെയ്തത്. തന്നെ പീഡിപ്പിക്കുന്നവര്‍ക്കെതിരെ കലാപമുണ്ടാക്കാതെയും തന്നെ നശിപ്പിക്കാന്‍ ശ്രമിച്ചവരെ നശിപ്പിക്കുവാന്‍ ശ്രമിക്കാതെയും തന്നിലേര്‍പിക്കപ്പെട്ട ദൗത്യം നിര്‍വഹിക്കുകയായിരുന്നു മക്കയില്‍ വെച്ച് പ്രവാചകന്‍(സ) ചെയ്തത്. ആളോ അര്‍ഥമോ ഇല്ലാത്തതുകൊണ്ടല്ല ജീവിക്കുന്ന നാട്ടില്‍ കുഴപ്പങ്ങളുണ്ടാക്കുവാന്‍ ദൈവകല്‍പനയില്ലാത്തതിനാലാണ് മക്കയില്‍വെച്ച് സായുധ കലാപത്തിന് അദ്ദേഹം മുതിരാതിരുന്നത്. ത്യാഗങ്ങള്‍ സഹിച്ചും പീഡനങ്ങളില്ലാതെയാക്കുവാന്‍ പ്രാര്‍ഥിച്ചും നിയതമായ മാര്‍ഗങ്ങളിലൂടെ പ്രവര്‍ത്തിച്ചും കൊണ്ട് തന്നിലര്‍പിക്കപ്പെട്ട ആദര്‍ശ പ്രചരണമെന്ന ദൗത്യം നിര്‍വഹിക്കുന്നതില്‍ മാത്രമാണ് മക്കയില്‍ വെച്ച് മുഹമ്മദ് നബി(സ) ബദ്ധശ്രദ്ധനായത്. അതായിരുന്നു മക്കയിലെ ജിഹാദ്; ക്വുര്‍ആന്‍ പറഞ്ഞ, ക്വുര്‍ആനുപയോഗിച്ചുകൊണ്ടുള്ള ഏറ്റവും വലിയ ജിഹാദ്.

മുസ്ലിമിന്റെ ജീവിതത്തിൽ നിന്ന് ഒഴിച്ചുനിർത്താനാത്ത കാര്യമാണ് ജിഹാദ് എന്നാണ് ഖുർആനും നബിവചനങ്ങളും വ്യക്തമാക്കുന്നത്. നബിജീവിതത്തിന്റെ തുടക്കം മുതൽ തന്നെ ജിഹാദ് നിറഞ്ഞുനിന്നിരുന്നുവെന്നതാണ് സത്യം. ജിഹാദിനെക്കുറിച്ച ക്വുര്‍ആന്‍ നിര്‍ദേശങ്ങള്‍ അവതരിക്കുവാനാരംഭിച്ചത് മക്കയില്‍വെച്ചാണ്. പീഡനങ്ങളും പ്രയാസങ്ങളും സഹിച്ച് ഇസ്‌ലാമിക പ്രബോധനം നിര്‍വഹിച്ചുകൊണ്ടിരിക്കുന്നതിനിടയില്‍ അവതരിപ്പിക്കപ്പെട്ട ക്വുര്‍ആന്‍ സൂക്തങ്ങളില്‍ ജിഹാദിന് പ്രേരിപ്പിക്കുന്ന വചനങ്ങള്‍ കാണാം. മക്കയില്‍വെച്ച് അവതരിപ്പിക്കപ്പെട്ട സൂറത്തുല്‍ അന്‍കബൂത്തിലെ അവസാനത്തെ വചനം കാണുക: ''നമ്മുടെ മാര്‍ഗത്തില്‍ ജിഹാദില്‍ ഏര്‍പെട്ടവരാരോ, അവരെ നമ്മുടെ വഴികളിലേക്ക് നാം നയിക്കുക തന്നെ ചെയ്യുന്നതാണ്. തീര്‍ച്ചയായും അല്ലാഹു സദ്‌വൃത്തരോടൊപ്പമാകുന്നു'' (29:69).

മക്കയില്‍വെച്ച് ചെയ്യേണ്ട ജിഹാദ് എന്താണ്?

പൂര്‍ണമായും മക്കയില്‍വെച്ച് അവതരിച്ചതെന്ന് വ്യാഖ്യാതാക്കള്‍ വ്യക്തമാക്കിയ സൂറത്തുല്‍ ഫുര്‍ഖാനിലെ 52-ാമത്തെ സൂക്തം ഈ ചോദ്യത്തിനുള്ള കൃത്യമായ ഉത്തരം നല്‍കുന്നുണ്ട്: ''അതിനാല്‍ സത്യനിഷേധികളെ നീ അനുസരിച്ചു പോകരുത്. ഇത് (ക്വുര്‍ആന്‍) കൊണ്ട് നീ അവരോട് വലിയൊരു ജിഹാദ് നടത്തിക്കൊള്ളുക'' (25:52)

ക്വുര്‍ആന്‍ ഉപയോഗിച്ച് ശത്രുക്കളോട് മഹത്തായ ജിഹാദ് നടത്തുക! ഏതാണീ ജിഹാദ്? ക്വുര്‍ആനുപയോഗിച്ചു കൊണ്ടുള്ള ജിഹാദ് ആശയസമരമാണെന്ന് വ്യക്തം. മുഹമ്മദ് നബി(സ)യുടെ നീണ്ട പതിമൂന്ന് വര്‍ഷത്തെ ജിഹാദിനുള്ള ആയുധം ക്വുര്‍ആനായിരുന്നു. അവതരിപ്പിക്കപ്പെടുന്ന മുറയ്ക്ക് ക്വുര്‍ആന്‍ വചനങ്ങളുപയോഗിച്ച് സമൂഹത്തില്‍ നിലനിന്നിരുന്ന തിന്‍മകള്‍ക്കും അധര്‍മങ്ങള്‍ക്കുമെതിരെ അദ്ദേഹം ജിഹാദ് നടത്തുകയായിരുന്നു. ഈ ജിഹാദാണ് മക്കയിലെ പലരുടെയും മനസ്സുകളില്‍ പരിവര്‍ത്തനത്തിന്റെ കൊടുങ്കാറ്റ് സൃഷ്ടിച്ചത്. ബഹുദൈവാരാധനയും തന്നിഷ്ടപ്രകാരമുള്ള ജീവിതവും കാരണം ശിലാഹൃദയരായിത്തീര്‍ന്നവരുടെ മനസ്സുകളില്‍ നിന്ന് ഏകദൈവാരാധനയും ദൈവിക വെളിപാടുപ്രകാരമുള്ള ജീവിതവും സൃഷ്ടിക്കുന്ന ആര്‍ദ്രതയുടെ തെളിനീരുകള്‍ അരിച്ചിറങ്ങുകയും അത് നദികളായിത്തീര്‍ന്ന് അറേബ്യയിലും പ്രാന്തപ്രദേശങ്ങളിലുമുള്ളവര്‍ക്ക് ദൈവിക ദര്‍ശനത്തിന്റെ തീര്‍ത്ഥമായിത്തീരുകയും ചെയ്തത് ക്വുര്‍ആന്‍ ഉപയോഗിച്ചുള്ള ജിഹാദ് കൊണ്ടായിരുന്നു. എത്രയെത്ര ശിലാഹൃദയരെയാണ് ക്വുര്‍ആന്‍ നിര്‍മലമാനസന്‍മാരാക്കിത്തീര്‍ത്തത്! പ്രവാചകശിഷ്യരില്‍ പ്രമുഖനായിത്തീര്‍ന്ന ഉമറുബ്‌നുല്‍ ഖത്ത്വാബിന്റെ പരിവര്‍ത്തന ചരിത്രം ഉദാഹരണമാണ്. നബിയോടുള്ള ശത്രുത കാരണം അദ്ദേഹത്തെ വധിച്ചുകളയാനായി വാളുമേന്തി പുറപ്പെട്ട ഉമറാണ് (റ) സഹോദരിയുടെ ക്വുർആൻ പാരായണം കേട്ട് അതിൽ ആകൃഷ്ടനായി നബിയുടെ (സ) ശക്തനായ അനുയായിയായിത്ത്തീർന്നത്. ക്വുര്‍ആന്‍ വചനങ്ങളുടെ വശ്യതയില്‍ പെട്ട് പ്രവാചകവിരോധികളായ മക്കാമുശ്‌രിക്കുകള്‍ പോലും അതിന്റെ കല്‍പന സ്വീകരിക്കുന്ന സ്ഥിതിക്ക് മക്കാരാജ്യം സാക്ഷിയായിട്ടുണ്ട്. നബില ക്വുര്‍ആനിലെ 53ാം അധ്യായമായ സൂറത്തുന്നജ്മ് പാരായണം ചെയ്തപ്പോള്‍ സ്രഷ്ടാവിന് സാഷ്ടാംഗം നമിക്കുവാനുള്ള അതിലെ കല്‍പനക്ക് വിധേയമായി അദ്ദേഹവും അനുചരന്‍മാരും സാഷ്ടാംഗം ചെയ്തപ്പോള്‍ അതോടൊപ്പം പാരായണം കേട്ടുനിന്ന മുശ്‌രിക്കുകയും സാഷ്ടാംഗം ചെയ്തതായി പ്രബലമായ ഹദീഥുകളില്‍ കാണാതാവും. പരിശുദ്ധ ക്വുര്‍ആനിന് ജനഹൃദയങ്ങളില്‍ മാറ്റമുണ്ടാക്കുവാനുള്ള അപാരമായ ശേഷിയുണ്ടെന്ന് മക്കാമുശ്‌രിക്കുകള്‍ക്ക് അറിയാമായിരുന്നു. അതുകൊണ്ടു തന്നെ തങ്ങളുടെ ഭാര്യമാരും മക്കളുമൊന്നും ക്വുര്‍ആന്‍ പാരായണം കേള്‍ക്കരുതെന്ന് അവര്‍ ആഗ്രഹിക്കുകയും അതിന്നായുള്ള കരുതല്‍ നടപടികളെടുക്കുകയും ചെയ്തു. സത്യസന്ധരും സദ്‌വൃത്തരുമായ മുസ്‌ലിംകള്‍ മക്കയില്‍ തന്നെയുണ്ടാകണമെന്നാണ്, അവരെ പീഡിപ്പിച്ചുകൊണ്ടിരുന്നപ്പോള്‍ പോലും മക്കാമുശ്‌രിക്കുകള്‍ ആഗ്രഹിച്ചതെങ്കിലും അവരുടെ ക്വുര്‍ആന്‍ പാരായണം ജനഹൃദയങ്ങളെ മാറ്റിമറിക്കുമെന്ന് അവര്‍ ഭയപ്പെട്ടു. മുസ്‌ലിംകള്‍ക്ക് സംരക്ഷണം നല്‍കിയവരോട് അവര്‍ ഉറക്കെ ക്വുര്‍ആന്‍ പാരായണം ചെയ്യരുതെന്നും അതുകേട്ട് തങ്ങളുടെ നാട്ടുകാരും കുടുംബക്കാരും അതിന്റെ ആശയങ്ങളിലേക്ക് ആകൃഷ്ടരാകാന്‍ ഇടയാകരുതെന്നും അവര്‍ നിബന്ധനവെക്കുവാന്‍ കാരണമതായിരുന്നു

ക്വുര്‍ആനിന്റെ സ്വാധീനവലയത്തില്‍ പെടാത്തവരായി മക്കയില്‍ ആരും തന്നെയുണ്ടായിരുന്നില്ല. തങ്ങള്‍ക്ക് ലഭിച്ചുകൊണ്ടിരിക്കുന്ന സൗകര്യങ്ങള്‍ ഇല്ലാതെയാവുകയും സ്ഥാനമാനങ്ങള്‍ നഷ്ടപ്പെടുകയും ചെയ്യുമോയെന്ന ഭയമായിരുന്നു സത്യമതം സ്വീകരിക്കുന്നതില്‍ നിന്ന് അവരെ തടഞ്ഞു നിര്‍ത്തിയത്. ഖുറൈശി നേതാവും കവിയും സാഹിത്യകാരനുമായിരുന്ന വലീദുബ്‌നു മുഗീറ തന്നെ ഉദാഹരണം. ക്വുര്‍ആനില്‍ ആകൃഷ്ടനായ അദ്ദേഹം, പക്ഷേ, തനിക്ക് ലഭിച്ചുകൊണ്ടിരിക്കുന്ന സ്ഥാനമാനങ്ങള്‍ നഷ്ടപ്പെടുമോയെന്ന ഭയം കാരണം അതിനെ നിഷേധിക്കുവാനും തള്ളിപ്പറയുവാനും ധൃഷ്ടനായി. സൂറത്തുല്‍ മുദ്ദഥിറിലെ പതിനൊന്ന് മുതല്‍ മുപ്പതു വരെയുള്ള വചനങ്ങളുടെ അവതരണ പശ്ചാത്തലത്തെക്കുറിച്ച് വിവരിക്കുന്ന വ്യാഖ്യാതാക്കള്‍ ഈ സംഭവം ഉദ്ധരിക്കുന്നുണ്ട്. അതിന്റെ സംക്ഷിപ്തം ഇങ്ങനെയാണ്: ഖുറൈശികള്‍ക്കിടയിലെ ബനൂമഖ്‌സൂം ഗോത്രത്തിലെ ധനാഢ്യനും നേതാവും സാഹിത്യകാരനുമായിരുന്ന വലീദുബ്‌നു മുഗീറ 'ഖുറൈശികളുടെ സുഗന്ധച്ചെടി' (റൈഹാനത്തു ഖുറൈശ്) എന്ന പേരില്‍ പ്രസിദ്ധനായിരുന്നു. അദ്ദേഹം ഒരിക്കല്‍ മുഹമ്മദ് നബിയെ സമീപിച്ചു. പ്രവാചകന്‍ല വലീദിന് ക്വുര്‍ആനിലെ 41ാം അധ്യായമായ 'ഹാമിം സജദഃ' പാരായണം ചെയ്തു കേള്‍പിച്ചു. ക്വുര്‍ആന്‍ വചനങ്ങളില്‍ ആകൃഷ്ടനായ അദ്ദേഹം അതിനെ പുകഴ്ത്തിക്കൊണ്ടുള്ള പ്രസ്താവന നടത്തി. ഈ വിവരമറിഞ്ഞ, നബിലയുടെ കഠിന ശത്രുവായിരുന്ന അബൂജഹല്‍ അദ്ദേഹത്തെ പരിഹസിക്കുകയും മുഹമ്മദ് നബി(സ)യെയും ക്വുര്‍ആനിനെയും വിമര്‍ശിച്ചു പറഞ്ഞിട്ടില്ലെങ്കില്‍ ജനങ്ങളെല്ലാം അദ്ദേഹത്തിന് എതിരാകുമെന്ന് ബോധ്യപ്പെടുത്തുകയും ചെയ്തു. അപ്പോള്‍ അദ്ദേഹം പറഞ്ഞു: 'അവനെപ്പറ്റി ഞാനെന്തു പറയാനാണ്? അല്ലാഹുവാണ! എന്നെക്കാള്‍ കവിതയും പദ്യവും പാട്ടും ജിന്നുകളുടെ കാവ്യവുമറിയുന്നവരായി നിങ്ങളില്‍ ഒരാളും തന്നെയില്ല. അല്ലാഹുവാണ! അവന്‍ പറയുന്ന വചനത്തിനൊരു മാധുര്യമുണ്ട്. അതിനു താഴെയുള്ളതെല്ലാം അതു ചവിട്ടിത്താഴ്ത്തുക തന്നെ ചെയ്യും. അത് ഉന്നതമായിത്തീരും. അതിനുപരിയായി ഒന്നും നിലകൊള്ളുകയില്ല'. മുഹമ്മദ് നബി(സ)യെയും ക്വുര്‍ആനിനെയും നിന്ദിച്ചുകൊണ്ട് എന്തെങ്കിലും പറയണമെന്ന അബൂജഹ്‌ലിന്റെ നിര്‍ബന്ധത്തിന് വഴങ്ങി അവസാനം 'ക്വുര്‍ആന്‍ ഒരു മാരണമാണെന്ന്' പറഞ്ഞ് രക്ഷപ്പെടുകയാണ് വലീദ് ചെയ്തത്. ക്വുര്‍ആനിന്റെ വശ്യതയിലും ആശയ ഗാംഭീര്യത്തിലും ആകൃഷ്ടരായി ഇസ്‌ലാമില്‍ എത്തിച്ചേര്‍ന്ന നിരവധിപേരെക്കുറിച്ച് ചരിത്രം നമുക്ക് പറഞ്ഞുതരുന്നുണ്ട്. ക്വുര്‍ആനിന് മറുപടിയെഴുതാന്‍ വേണ്ടിയുള്ള ശ്രമത്തിനിടയില്‍ ഒരു മുസ്‌ലിം ബാലന്‍ സൂറത്തുല്‍ ഹൂദിലെ 44-ാമത്തെ വചനം പാരായണം ചെയ്യുന്നത് കേട്ട് മനഃപരിവര്‍ത്തനമുണ്ടാവുകയും ''അല്ലാഹുവാണെ! ക്വുര്‍ആനിന് മറുപടിയെഴുതുക അസാധ്യമാണെന്നും അത് മനുഷ്യവചനമല്ലെന്നും ഞാന്‍ സാക്ഷ്യം വഹിക്കുന്നു''വെന്ന് പ്രഖ്യാപിക്കുകയും ചെയ്ത ജാഹിലിയ്യാകാലത്തെ അറിയപ്പെടുന്ന എഴുത്തുകാരനായിരുന്ന അബ്ദുല്ലാഹിബ്‌നു മുഖാഫ്; പ്രവാചകനില്‍നിന്ന് സൂറതുത്ത്വൂറിലെ ഏഴ്, എട്ട് വചനങ്ങള്‍ ശ്രവിച്ച് മനഃപരിവര്‍ത്തനമുണ്ടായി ഇസ്‌ലാമിലെത്തിയ ജുബൈറുബ്‌നു മുത്അം; മദീനയില്‍ പ്രവാചക പ്രതിനിധിയായെത്തിയ മുസ്അബുബ്‌നു ഉമൈറിനോട് യഥ്‌രിബുകാരായ മൃദുലഹൃദയരെ വഴിതെറ്റിക്കുന്നവിധത്തില്‍ ക്വുര്‍ആന്‍ പാരായണം ചെയ്യരുതെന്ന് ആവശ്യപ്പെട്ടെത്തിയപ്പോള്‍ മുസ്അബിന്റെ അഭ്യര്‍ഥനയെ മാനിച്ച് അല്‍പനേരം ക്വുര്‍ആന്‍ പാരായണം ശ്രവിച്ചതോടെ അതില്‍ ആകൃഷ്ടനായി ഇസ്‌ലാം സ്വീകരിച്ച ഉസൈദുബ്‌നു ഹുദൈര്‍; ഉസൈദിന്റെ നിര്‍ദേശപ്രകാരം മുസ്അബില്‍നിന്ന് ക്വുര്‍ആന്‍ ശ്രവിച്ചതോടെ അതിന്റെ അനുയായിയായിത്തീര്‍ന്ന ഔസ് ഗോത്രനേതാവ് സഅദ്ബ്‌നു മുആദ്... ഇങ്ങനെ ക്വുര്‍ആന്‍ പാരായണം കേട്ടതുകൊണ്ടുമാത്രം ഇസ്‌ലാമിലെത്തിച്ചേര്‍ന്ന നിരവധിയാളുകളെക്കുറിച്ച് ചരിത്രം നമുക്ക് പറഞ്ഞുതരുന്നുണ്ട്. മുസ്അബുബ്‌നു ഉമൈറിന്റെ േക്വുര്‍ആന്‍ പാരായണം വഴിയാണ്, വാളുകൊണ്ടല്ല മദീനയെ ഇസ്‌ലാം കീഴടക്കിയതെന്ന വസ്തുത ചരിത്രകാരന്‍മാരെല്ലാം അംഗീകരിക്കുന്നതാണ് .

പ്രവാചകന്റെ കാലത്ത് മാത്രമല്ല, അതിന് ശേഷം ഇന്നുവരെയും ക്വുര്‍ആന്‍ ജനമനസ്സുകളില്‍ മാറ്റത്തിന്റെ കൊടുങ്കാറ്റ് വിതയ്ക്കുകയെന്ന ദൗത്യം നിര്‍വഹിച്ചുകൊണ്ടേയിരിക്കുന്നുണ്ട്. ആംഗ്ലിക്കന്‍ പാതിരിയുടെ മകനായി ജനിച്ച് ക്വുര്‍ആന്‍ പഠനത്തിലൂടെ ഇസ്‌ലാമിലെത്തിച്ചേരുകയും പിന്നീട് പ്രസിദ്ധമായ ഇംഗ്ലീഷ് ക്വുര്‍ആന്‍ വിവര്‍ത്തനം രചിക്കുകയും ചെയ്ത ബ്രിട്ടീഷ് നോവലിസ്റ്റ് മുഹമ്മദ് മാര്‍മഡ്യൂക് പിക്താള്‍; യഹൂദ റബ്ബിമാരുടെ പാരമ്പര്യമുള്‍ക്കൊള്ളുന്ന കുടുംബത്തില്‍ ജനിക്കുകയും ക്വുര്‍ആന്‍ പഠനത്തിലൂടെ ഇസ്‌ലാമിലെത്തിച്ചേര്‍ന്ന് മുഹമ്മദ് അസദ് എന്ന പേര് സ്വീകരിക്കുകയും സ്വന്തമായി ഇംഗ്ലീഷിലുള്ള ക്വുര്‍ആന്‍ വിവരണം രചിക്കുകയും ചെയ്ത ഓസ്ട്രിയന്‍ സഞ്ചാരിയും യഹൂദ പണ്ഡിതനുമായിരുന്നു ലിയോ പോള്‍ഡ് വെയ്സ്സ്. ക്വുര്‍ആനിന്റെ വെളിച്ചത്തിലൂടെ ഇസ്‌ലാമില്‍ എത്തുകയും അങ്ങനെ ക്വുര്‍ആനിന്റെ വെളിച്ചം ലോകത്തിന് എത്തിച്ചുകൊടുക്കുവാന്‍ മുന്നില്‍ നടക്കുകയും ചെയ്ത രണ്ട് മഹാരഥന്‍മാര്‍-പാശ്ചാത്യന്‍ സംഗീത ലോകത്തിന്റെ ഉന്നത വിതാനത്തില്‍ വിരാജിക്കവെ ക്വുര്‍ആനിന്റെ സന്ദേശം മനസ്സിലാക്കുവാന്‍ അവസരമുണ്ടാവുകയും യൂസുഫ് ഇസ്‌ലാം എന്ന പേര് സ്വീകരിച്ച് ഇസ്‌ലാമിക പ്രബോധന രംഗത്തേക്ക് കടന്നുവരികയും ചെയ്ത കാറ്റ് സ്റ്റീവന്‍സ്; പന്ത്രണ്ടാം വയസ്സില്‍ അഭിനയരംഗത്തെത്തുകയും നിരവധി ഹോളിവുഡ് സിനിമകളില്‍ അഭിനയിക്കുകയും തന്റെ അഭിനയ ജീവിതത്തിനിടയ്ക്ക് ക്വുര്‍ആനിനെ പരിചയപ്പെട്ട് അതിന്റെ പഠനത്തിലൂടെ ഇസ്‌ലാമിലെത്തുകയും ഇപ്പോള്‍ മറിയം ഫ്രാന്‍കോയിസ് എന്ന പേര് സ്വീകരിച്ച് ക്വുര്‍ആനിന്റെ ആശയ പ്രചാരണത്തിനായി പരിശ്രമിക്കുകയും ചെയ്യുന്ന എമിനി ഫ്രാന്‍കോയ്‌സ് സിര്‍റ; ബൈബിള്‍ പഠനത്തിനിടയില്‍ അവിചാരിതമായി ക്വുര്‍ആന്‍ പരിഭാഷ വായിക്കുവാന്‍ അവസരമുണ്ടാവുകയും അത് ക്വുര്‍ആനിന്റെ പ്രചാരകനാക്കിത്തീര്‍ക്കുന്നതിലേക്ക് നയിക്കുകയും ചെയ്ത ക്രിസ്ത്യന്‍ മിഷനറി പ്രവര്‍ത്തകന്‍ ജോഷുവ ഇവാന്‍സ്, സ്വിറ്റ്‌സര്‍ലാന്റില്‍ മുസ്‌ലിംകള്‍ക്ക് പള്ളി നിര്‍മിക്കുവാന്‍ അവസരം നല്‍കരുതെന്ന് വാദിക്കുകയും അതിനായി ജനഹിതമുണ്ടാക്കിയെടുക്കാന്‍ പരിശ്രമിക്കുകയും ചെയ്ത സ്വിസ് പീപ്പിള്‍സ് പാര്‍ട്ടിയുടെ മുന്നില്‍നിന്ന് പ്രവര്‍ത്തിക്കുന്നതിനിടെ ക്വുര്‍ആനികാശയങ്ങളെപ്പറ്റി പഠിക്കുവാന്‍ അവസരമുണ്ടാകുകയും ഇസ്‌ലാം സ്വീകരിക്കുന്നതിലേക്ക് അത് നയിക്കുകയും ചെയ്ത ഡാനിയല്‍ സ്ട്രീക്ക്-അന്ധകാര നിബിഡമായ മനസ്സുകളില്‍ ക്വുര്‍ആന്‍ സൃഷ്ടിച്ചുകൊണ്ടിരിക്കുന്ന മാറ്റത്തിന് പാശ്ചാത്യനാടുകളില്‍ നിന്നും ഇന്നും ജീവിച്ചിരിക്കുന്ന ഉദാഹരണങ്ങളാണിവ. നാടോടികളും അധര്‍മങ്ങളില്‍ നിമഗ്‌നരുമായിരുന്ന അറബികളെ ലോകത്തിന് വഴി കാണിക്കുവാന്‍ പര്യാപ്തരാക്കുകയെന്ന അത്ഭുതമാണ് ക്വുര്‍ആന്‍ ഉപയോഗിച്ചുകൊണ്ടുള്ള ജിഹാദ് വഴി പ്രവാചകന്‍(സ) സൃഷ്ടിച്ചത്. അന്ധവിശ്വാസങ്ങളില്‍നിന്നും അനാചാരങ്ങളില്‍നിന്നും ആധുനികത സൃഷ്ടിക്കുന്ന പുതിയ അടിമത്തങ്ങളില്‍നിന്നും മനുഷ്യരെ രക്ഷപ്പെടുത്തുകയും വിമലീകൃതമായ വിശുദ്ധ ജീവിതംവഴി കുടുംബത്തിലും സമൂഹത്തിലുമെല്ലാം പരിവര്‍ത്തനങ്ങള്‍ ഉണ്ടാക്കുവാന്‍ കഴിയുന്നവരാക്കി അവരെ മാറ്റിയെടുക്കുകയും ചെയ്തുകൊണ്ട് ക്വുര്‍ആനിന്റെ ജിഹാദ് ഇന്നും തുടര്‍ന്നുകൊണ്ടിരിക്കുന്നു. ആയുധങ്ങളല്ല ക്വുര്‍ആനികാശയങ്ങളാണ് അന്നും ഇന്നും ജനഹൃദയങ്ങളെ മാറ്റുവാനുള്ള നിമിത്തങ്ങളായിത്തീരുന്നത്. ജനഹൃദയങ്ങളെ മാറ്റിമറിക്കുവാനാകുന്ന ക്വുര്‍ആന്‍കൊണ്ടുള്ള ജിഹാദാണ് ഏറ്റവും വലിയ ജിഹാദ് എന്നാണല്ലോ ക്വുര്‍ആന്‍തന്നെ പരിചയപ്പെടുത്തുന്നത്.

സ്വന്തം തൃഷ്ണകളോട് ചെയ്യുന്ന പോരാട്ടമാണ് ഏറ്റവും വലിയ ജിഹാദെന്നും ജീവത്യാഗമാവശ്യപ്പെടുന്ന സായുധാജിഹാദിന് പോവുന്നവർ ആദ്യം ചെയ്യേണ്ടത് സ്വന്തത്തോടുള്ള ജിഹാദാണെന്നും പ്രവാചകവചങ്ങളുടെ അടിസ്ഥാനത്തിൽ പണ്ഡിതന്മാർ വ്യക്തമാക്കിയിട്ടുണ്ട്.  ജിഹാദിനെക്കുറിച്ച് വര്‍ഗീകരിച്ചു വിശദീകരിച്ച പണ്ഡിതന്‍മാരില്‍ പ്രമുഖനാണ് ഹിജ്‌റ എട്ടാം നൂറ്റാണ്ടില്‍ മരണപ്പെട്ട ഇമാം ഇബ്‌നുല്‍ ഖയ്യിം അല്‍ ജൗസിയ്യഃ (ക്രി. 1292-1350). സ്വന്തത്തോടുള്ളത് (ജിഹാദുന്നഫ്‌സ്), ചെകുത്താനോടുള്ളത് (ജിഹാദുശ്‌ശൈത്വാന്‍), സത്യനിഷേധികളോടും കപട വിശ്വാസികളോടുമുള്ളത് (ജിഹാദുല്‍ കുഫ്ഫാറു വല്‍ മുനാഫിഖീന്‍), അനീതിയുടെയും തിന്‍മകളുടെയും അനാചാരങ്ങളുടെയും ആളുകള്‍ക്കെതിരെയുള്ളത് (ജിഹാദു അര്‍ബാബുദ്ദ്വുല്‍മി വല്‍ ബിദ്ഇ വല്‍ മുന്‍കറാത്ത്) എന്നിങ്ങനെ നാലായി ജിഹാദിനെ വര്‍ഗീകരിക്കാമെന്നാണ് അദ്ദേഹത്തിന്റെ പക്ഷം.

സ്വന്തത്തോടുള്ള ജിഹാദില്‍ നിന്നാണ് ഒരാള്‍ തന്റെ ഇസ്‌ലാമിക ജീവിതം ആരംഭിക്കുന്നത്. സ്വന്തം താല്‍പര്യങ്ങളുടെയും ആഗ്രഹങ്ങളുടെയും തൃഷ്ണയുടെയും തടവറയില്‍ നിന്ന് സ്വയം മോചിതനായി ദൈവിക നിയമങ്ങള്‍ക്ക് വിധേയനാവലാണ് സ്വന്തത്തോടുള്ള ജിഹാദ്. നാലു രൂപത്തിലാണ് ഒരാള്‍ സ്വന്തത്തോട് പോരാടുന്നത്. ദൈവിക മതത്തിന്റെ യഥാര്‍ഥമായ മാര്‍ഗദര്‍ശനത്തെക്കുറിച്ച കൃത്യമായ പഠനം, അറിവു ലഭിച്ച കാര്യങ്ങളുടെ പ്രയോഗവല്‍ക്കരണം, താന്‍ അറിഞ്ഞ കാര്യങ്ങള്‍ അത് അറിയാത്തവര്‍ക്ക് പഠിപ്പിച്ചുകൊടുക്കുക, ദൈവിക മാര്‍ഗത്തിലേക്കുള്ള ക്ഷണത്തിന്റെ പാതയില്‍ ത്യാഗങ്ങള്‍ സഹിക്കേണ്ടി വരുമ്പോള്‍ ക്ഷമിക്കുക എന്നിവയാണ് ജിഹാദുന്നഫ്‌സിന്റെ നാല് രൂപങ്ങള്‍.

ജിഹാദുശ്‌ശൈത്വാനിന് രണ്ടു രൂപങ്ങളാണുള്ളത്. ശരിയായ വിശ്വാസത്തെ നശിപ്പിക്കാനായി പിശാച് മനസ്സില്‍ ചെലുത്തുന്ന സംശയങ്ങളെ ദൂരീകരിക്കുകയും അതിനെതിരെ സജ്ജമാവുകയും ചെയ്യുന്നതും തെറ്റായ താല്‍പര്യങ്ങളും അനനുവദനീയമായ തൃഷ്ണകളും വളര്‍ത്തി പ്രലോഭിപ്പിക്കുവാന്‍ വേണ്ടിയുള്ള പിശാചിന്റെ പരിശ്രമങ്ങളെ ചെറുക്കുകയും അവഗണിക്കുകയും ചെയ്യുന്നതുമാണ് ജിഹാദുശ്‌ശൈത്വാനിന്റെ രൂപങ്ങള്‍. നാല് രൂപത്തിലാണ് ഒരാള്‍ ജിഹാദുല്‍ കുഫ്ഫാര്‍ വല്‍ മുനാഫിഖീന്‍ നിര്‍വഹിക്കുന്നത്. ഹൃദയം കൊണ്ടും നാവുകൊണ്ടും സമ്പത്തുകൊണ്ടും ശരീരംകൊണ്ടും നടത്തുന്നവയാണ് അവ.

അനീതിയുടെയും അനാചാരങ്ങളുടെയും തിന്‍മകളുടെയും വക്താക്കള്‍ക്കെതിരെ നടത്തുന്ന ജിഹാദിന് മൂന്ന് രൂപങ്ങളാണുള്ളത്. കൈകൊണ്ടുള്ളത്, നാവുകൊണ്ടുള്ളത്, മനസ്സുകൊണ്ടുള്ളത് എന്നിവയാണവ.(1)

ഇബ്‌നുല്‍ ഖയ്യിം വര്‍ഗീകരിച്ച നാലുതരം ജിഹാദുകളും മക്കയില്‍ വെച്ചുതന്നെ പ്രവാചകനും(സ) അനുചരന്‍മാരും പ്രവാര്‍ത്തികമാക്കിയിരുന്നുവെന്ന് കാണാനാവും. മൂന്നാമത്തെ തരമായി എണ്ണിയ സത്യനിഷേധികളോടും കപടവിശ്വാസികളോടുമുള്ള ജിഹാദില്‍, കപടവിശ്വാസികളോടുള്ള ജിഹാദ് മാത്രമാണ് മദീനയിലെത്തിയശേഷം മാത്രമായി നിര്‍വഹിക്കപ്പെട്ടത്. മക്കയിലെ പീഡിപ്പിക്കപ്പെട്ട നാളുകളില്‍ പ്രവാചകാനുചരന്‍മാരോടൊപ്പം മുനാഫിഖുകള്‍ ഉണ്ടായിരുന്നില്ല എന്നതുകൊണ്ടായിരുന്നു അത്. സ്വന്തത്തോടും ചെകുത്താനോടും സത്യനിഷേധികളോടും തിന്‍മയുടെ വാഹകരോടുമുള്ള ജിഹാദുകള്‍ മക്കയില്‍ വെച്ചുതന്നെ നിര്‍വഹിക്കുവാന്‍ തുടങ്ങിയിരുന്നു. മക്കയിലെ വ്യത്യസ്ത സാഹചര്യങ്ങളില്‍ വ്യത്യസ്ത രൂപഭാവങ്ങളായാണ് പ്രസ്തുത ജിഹാദുകള്‍ നിര്‍വഹിക്കപ്പെട്ടത് എന്നു മാത്രമെയുള്ളു.

എല്ലാ ജിഹാദുകളുടെയും അടിസ്ഥാനം ജിഹാദുന്നഫ്‌സാണെന്ന് പണ്ഡിതന്‍മാര്‍ വ്യക്തമാക്കിയിട്ടുണ്ട്. 'സത്യനിഷേധികള്‍ക്കും കപടവിശ്വാസികള്‍ക്കുമെതിരെയുള്ള ജിഹാദിന്റെ അടിത്തറയുണ്ടാകുന്നത് സ്വന്തത്തിനും സ്വന്തം ഇച്ഛകള്‍ക്കുമെതിരെയുള്ള ജിഹാദില്‍ നിന്നാണ്; സ്വന്തത്തിനും സ്വന്തം ഇച്ഛകള്‍ക്കെതിരെയുമുള്ള ജിഹാദ് ചെയ്യാത്തവര്‍ക്ക് സത്യനിഷേധികള്‍ക്കും കപടവിശ്വാസികള്‍ക്കുമെതിരെയുള്ള ജിഹാദ് ചെയ്യുവാനോ അതിന്നായി പുറപ്പെട്ടിറങ്ങുവാനോ സാധ്യമല്ല'യെന്ന് ശൈഖുല്‍ ഇസ്‌ലാം ഇബ്‌നുതൈമിയ്യഃ പറഞ്ഞതായി ഇബ്‌നുല്‍ ഖയ്യിം ഉദ്ധരിക്കുന്നുണ്ട്.(2)

ഇസ്‌ലാമികമായ ആദര്‍ശങ്ങള്‍ക്കനുസരിച്ച് സ്വന്തം ജീവിതത്തെ ചിട്ടപ്പെടുത്തുകയും സത്യമെന്ന് ഉറപ്പുള്ള ആദര്‍ശത്തെ മറ്റുള്ളവര്‍ക്ക് എത്തിച്ചുകൊടുക്കുവാനായി പ്രയത്‌നിക്കുകയും അതിന്റെ മാര്‍ഗത്തില്‍ ത്യാഗങ്ങള്‍ സഹിക്കുകയും ചെയ്യുകയെന്ന ജിഹാദുന്നഫ്‌സ് ചെയ്യാത്തവര്‍ക്ക് എങ്ങനെയാണ് പിശാചിനെ തോല്‍പിക്കാനും അവിശ്വാസികളോടും കപടവിശ്വാസികളോടും ആദര്‍ശ സമരം നടത്തുവാനും അനിവാര്യമായ അവസരത്തില്‍ ആയുധമെടുത്ത് അടരാടുവാനും കഴിയുക? എല്ലാ ജിഹാദുകളുടെയും അടിത്തറ ജിഹാദുന്നഫ്‌സ് ആണെന്നതിനാല്‍ അതിനെ 'ജിഹാദുല്‍ അക്ബര്‍' എന്ന് ചില പണ്ഡിതന്‍മാര്‍ വിളിച്ചിട്ടുണ്ട്.(3)

സ്വന്തം ഇച്ഛകള്‍ക്കും തൃഷ്ണകള്‍ക്കുമെതിരെയുള്ള പോരാട്ടമാണ് അടര്‍ക്കളത്തിലിറങ്ങിക്കൊണ്ടുള്ള യുദ്ധത്തെക്കാള്‍ മഹത്തരമെന്ന് സ്ഥാപിക്കുവാനായി നബി(സ)യില്‍ നിന്ന് ഉദ്ധരിക്കപ്പെടുന്ന ഒരു സംഭവവിവരണം അടിസ്ഥാനരഹിതമാണെന്നും നിവേദക പരമ്പര ദുര്‍ബലമാണെന്നും ഹദീഥ് പണ്ഡിതന്‍മാര്‍ ചൂണ്ടിക്കാട്ടിയിട്ടുണ്ട്. അനുചരന്‍മാര്‍ ഒരു യുദ്ധം കഴിഞ്ഞു മടങ്ങിവന്നപ്പോള്‍ 'നല്ലൊരു വരവാണ് നിങ്ങള്‍ വന്നത്; ചെറിയൊരു ജിഹാദില്‍ നിന്ന് വലിയൊരു ജിഹാദിലേക്ക്-സ്വന്തം ഇച്ഛകള്‍ക്കെതിരെ ദൈവദാസന്‍മാര്‍ നടത്തുന്ന പോരാട്ടം-ആണ് നിങ്ങള്‍ വന്നിരിക്കുന്നത്' എന്ന് പ്രവാചകന്‍(സ) പറഞ്ഞതായി ജാബിറുബ്‌നു അബ്ദുല്ല(റ)നിവേദനം ചെയ്തതായി 'താരീഖുല്‍ ബാഗ്ദാദി'യില്‍ രേഖപ്പെടുത്തിയ ഹദീഥാണത്. ഈ പദങ്ങള്‍ യഥാര്‍ഥത്തില്‍ താബിഇയായ(4) ഇബ്‌റാഹിമുബ്‌നു അബ്‌ലഹിന്റെതാണെന്ന് പ്രമുഖ ഹദീഥ് നിദാനശാസ്ത്രജ്ഞനായ അന്നസാഈ രേഖപ്പെടുത്തിയിട്ടുണ്ടെന്നും ഹദീഥ് വ്യാഖ്യാതാക്കളില്‍ പ്രസിദ്ധനായ ഇമാം ഇബ്‌നു ഹജറുല്‍ അസ്ഖലാനി രേഖപ്പെടുത്തുന്നുണ്ട്.(5) ഇമാം സുയൂത്വി തന്റെ 'ജാമിഉസ്‌സഗീറി'ലും ഇമാം ദഹബി തന്റെ 'അല്‍മീസാനിലും' ഇമാം ഇബ്‌നു തൈമിയ്യ തന്റെ 'അല്‍ഫുര്‍ഖാനു ബൈനല്‍ ഔലിയാ ഉര്‍റഹ്മാനി വ ഔലിയാഉശ്ശൈത്വാനി'ലും ഇൗ ഹദീഥ് ദുര്‍ബലമാണെന്ന് വ്യക്തമാക്കിയിട്ടുമുണ്ട്.(6)

സ്വന്തം ഇച്ഛക്കെതിരെ ദൈവദാസന്‍മാര്‍ നടത്തുന്ന പോരാട്ടമാണ് വലിയ ജിഹാദെന്ന് പ്രസ്താവിക്കുന്ന ഹദീഥിന്റെ പരമ്പര ദുര്‍ബലമായതിനാല്‍ അത് പ്രവാചകന്‍ പറഞ്ഞതാണെന്ന് ഉറപ്പിക്കുവാന്‍ കഴിയാത്തതുകൊണ്ടുതന്നെ മതവിധികള്‍ നിര്‍ധരിക്കുന്നതിന് അത് ഉപയോഗിക്കുവാന്‍ തീരെ പാടില്ലാത്തതാണെങ്കിലും, സ്വന്തത്തോടുള്ള പോരാട്ടത്തെ 'ജിഹാദുല്‍ അക്ബര്‍' എന്നും പടക്കളത്തിലിറങ്ങി നടത്തുന്ന യുദ്ധത്തെ 'ജിഹാദുല്‍ അസ്ഗര്‍' എന്നും വിളിക്കുന്ന സമ്പ്രദായം നബി(സ)ക്കു ശേഷമുള്ള രണ്ടാം തലമുറ മുതല്‍ക്കെങ്കിലും നിലനിന്നിരുന്നുവെന്ന യാഥാര്‍ഥ്യം വ്യക്തമാക്കുന്നതാണ് ഈ ഹദീഥ് എന്ന വസ്തുത വിസ്മരിച്ചു കൂടാ. ആയുധങ്ങളെടുത്ത് അടരാടാന്‍ അവസരം ലഭിച്ചവരും അത് ലഭിക്കാത്തവരുമെല്ലാം ചെയ്യേണ്ട മഹത്തായ ജിഹാദാണ് സ്വന്തം ഇച്ഛയോടുള്ള പോരാട്ടമെന്നായിരുന്നു പ്രവാചകാനുചരന്‍മാരില്‍ നിന്ന് മതം നേര്‍ക്കുനേരെ അനുഭവിച്ചറിഞ്ഞവര്‍ മനസ്സിലാക്കിയിരുന്നത് എന്നാണല്ലോ ഇത് വ്യക്തമാക്കുന്നത്.

സ്വന്തം ഇച്ഛയോട് നടത്തേണ്ട പോരാട്ടത്തിന്റെ പ്രാധാന്യം വ്യക്തമാക്കുന്ന പ്രബലമായ ഹദീഥുകള്‍ അതിനെ ജിഹാദുല്‍ അക്ബര്‍ എന്നു വിളിക്കുന്നതിനെ സാധൂകരിക്കുന്നുണ്ടെന്ന് അഭിപ്രായപ്പെടുന്നവരുണ്ട്. താന്‍ നിര്‍വഹിച്ച ഹജ്ജ് കര്‍മത്തോടനുബന്ധിച്ച് പ്രവാചകന്‍(സ)നിര്‍വഹിച്ച വിടവാങ്ങല്‍ പ്രസംഗത്തിന്റെ, ഇമാം അഹ്മദും ത്വബ്‌റാനിയും ഇബ്‌നുമാജയും ഹാക്കിമുമെല്ലാം നിവേദനം ചെയ്ത സമ്പൂര്‍ണ രൂപത്തില്‍ 'അല്ലാഹുവിനെ അനുസരിക്കുന്ന വിഷയത്തില്‍ സ്വന്തത്തോടു ജിഹാദു ചെയ്യുന്നവനാണ് മുജാഹിദ്' എന്ന് നബി(സ) പ്രസ്താവിച്ചതായി രേഖപ്പെടുത്തിയത് സ്വഹീഹാണെന്ന് ഇമാം ഇബ്‌നു ഹിബ്ബാനും ഇമാം ഹാകിമും(7) ഇമാം അല്‍ബാനിയും(8) സാക്ഷ്യപ്പെടുത്തിയിട്ടുണ്ട്. അല്ലാഹുവിനെ അനുസരിക്കുന്നതില്‍ നിന്ന് സ്വന്തത്തെ തടയുന്ന തൃഷ്ണകളോട് പോരാടി അവനോടുള്ള അനുസരണത്തെ ദൃഢീകരിച്ച് ഉറപ്പിക്കുന്നവനാണ് യഥാര്‍ഥ മുജാഹിദെന്ന് പഠിപ്പിക്കുന്ന ഈ ഹദീഥ് ജിഹാദുന്നഫ്‌സിന്റെ പ്രാധാന്യം വ്യക്തമാക്കുന്നുണ്ട്.

അല്ലാഹുവെക്കുറിച്ച സ്മരണകളും അവന്റെ പ്രീതി നേടിയെടുക്കണമെന്ന അഭിലാഷവുമാണ് മുസ്‌ലിമിനെ സ്വന്തത്തോടും മറ്റുള്ളവരോടുമെല്ലാം ജിഹാദ് ചെയ്യുവാന്‍ പ്രേരിപ്പിക്കുന്ന ഘടകം. വ്യക്തിയുടെ വിമലീകരണത്തിന്റെ അടിത്തറയാണ് ദൈവസ്മരണ. ഒരു മുസ്‌ലിം ചെയ്യുന്ന മറ്റേത് കര്‍മത്തെക്കാളും മഹത്തരമായതാണ് അല്ലാഹുവിനെക്കുറിച്ച സ്മരണയെന്ന് അഹ്മദും തിര്‍മിദിയും ഇബ്‌നുമാജയും ഹാകിമുമെല്ലാം നിവേദനം ചെയ്ത സ്വഹീഹാണെന്ന് ഇമാം അല്‍ബാനി വ്യക്തമാക്കിയ(9) ഒരു ഹദീഥ് പഠിപ്പിക്കുണ്ട്. ഹദീഥിന്റെ സാരം ഇങ്ങനെയാണ്: അബിദ്ദര്‍ദാഇല്‍ േനിന്ന്: പ്രവാചകന്‍(സ) പറഞ്ഞു: നിങ്ങളുടെ രാജാധിരാജന് ഏറ്റവും ഇഷ്ടപ്പെട്ട, നിങ്ങളുടെ പദവികളെ ഉയര്‍ത്തുന്ന, സ്വര്‍ണവും വെള്ളിയും ദാനം ചെയ്യുന്നതിനെക്കാള്‍ ഉത്തമമായ, യുദ്ധത്തില്‍ നിങ്ങളുടെ ശത്രുവിനെ കണ്ടുമുട്ടുകയും അയാളുടെ തലകൊയ്യുകയും നിങ്ങളുടെ തലകൊയ്യപ്പെടുകയും ചെയ്യുന്നതിനെക്കാള്‍ മഹത്തരമായ, അത്യുത്തമമായ കര്‍മത്തെക്കുറിച്ച് ഞാന്‍ നിങ്ങള്‍ക്ക് പറഞ്ഞുതരട്ടയോ? അതാണ് 'അല്ലാഹുവെക്കുറിച്ച സ്മരണ' (ദിക്‌റുല്ലാ).(10)

തന്റെ നാല്‍പതാമത്തെ വയസ്സില്‍ പ്രവാചകത്വം ലഭിച്ചത് മുതല്‍ അറുപത്തി മൂന്നാമത്തെ വയസ്സില്‍ ഏല്‍പിക്കപ്പെട്ട ദൗത്യം പൂര്‍ത്തിയാക്കി ഈ ലോകത്തുനിന്ന് വിട പറയുന്നതു വരെയുള്ള നബി(സ)യുടെ ജീവിതം മുഴുവന്‍ ജിഹാദ് നിറഞ്ഞുനില്‍ക്കുകയായിരുന്നു. അതില്‍ സ്വന്തത്തോടുള്ള പോരാട്ടവും പിശാചിനോടുള്ള പോരാട്ടവും സത്യനിഷേധികള്‍ക്കും കപടവിശ്വാസികള്‍ക്കുമെതിരെയുള്ള പോരാട്ടവും അനീതിയുടെയും അധര്‍മത്തിന്റെയും അക്രമത്തിന്റെയും വക്താക്കള്‍ക്കെതിരെയുള്ള പോരാട്ടവുമുണ്ടായിരുന്നു. നബി(സ)യെ പിന്‍തുടര്‍ന്ന് സത്യമതത്തിന്റെ അനുയായികളായിത്തീര്‍ന്നവരും ഈ പോരാട്ടത്തില്‍ അദ്ദേഹത്തോടൊപ്പം ചേര്‍ന്നു. ഇസ്‌ലാം സ്വീകരിച്ചതു മുതല്‍ ജീവിതാവസാനം വരെ അവര്‍ പോരാട്ടത്തിലായിരുന്നു. മക്കയില്‍ വെച്ചു തന്നെ മരണപ്പെട്ടവരും മദീനയിലെത്തിയ ശേഷം ഏറെനാള്‍ ജീവിക്കുവാന്‍ അവസരം ലഭിച്ചവരുമെല്ലാമുണ്ട് അവര്‍ക്കിടയില്‍. അവരിലൊരാളും തന്നെ ജിഹാദ് ചെയ്യാതിരുന്നിട്ടില്ല. ആദര്‍ശപ്രബോധനത്തിന്റെ ആദ്യകാലത്ത് നബി(സ)ക്ക് താങ്ങും തണലുമായി നിലനിന്ന പ്രവാചക പത്‌നി ഖദീജയും(റ)സത്യത്തിന്റെ ശത്രുക്കളുടെ ക്രൂരമായ പീഡനങ്ങള്‍ക്കു മുമ്പില്‍ ക്ഷമ അവലംബിച്ച് അവസാനം മരണത്തിന് കീഴ്‌പെട്ട യാസിറും(റ) സുമയ്യ(റ)യെയുമല്ലാം മക്കയില്‍ വെച്ച് മരണപ്പെട്ട മുജാഹിദുകളായിരുന്നു. ജീവിതത്തില്‍ ഒരിക്കലും ആയുധമേന്തുവാന്‍ അവസരം ലഭിക്കാത്ത മുജാഹിദുകള്‍. സ്വന്തത്തിനും പിശാചിനും ശത്രുക്കള്‍ക്കുമെതിരെ ഏറ്റവും വലിയ ജിഹാദ് ചെയ്തുകൊണ്ടാണ് അവരെല്ലാം മരണപ്പെട്ടത്. ക്വുര്‍ആന്‍ ഉപയോഗിച്ചു കൊണ്ടുള്ള പോരാട്ടമാണ് ഏറ്റവും വലിയ ജിഹാദ് എന്നാണല്ലോ ക്വുര്‍ആന്‍ അവരെ പഠിപ്പിച്ചത്; ത്യാഗങ്ങള്‍ സഹിച്ചുകൊണ്ടുള്ള ഇസ്‌ലാമിക ജീവിതവും ആദര്‍ശ പ്രബോധനവുമായിരുന്നു അവര്‍ നയിച്ച ജിഹാദ്.

പ്രബോധകജീവിതത്തിന്റെ ആദ്യത്തെ നാളുകള്‍ മുതല്‍ യാതൊരു വിട്ടുവീഴ്ചയുമില്ലാതെ പ്രവാചകനും(സ) അനുയായികളും നയിച്ച ജിഹാദായിരുന്നു പ്രതിബന്ധങ്ങളെ തൃണവല്‍ഗണിച്ചു കൊണ്ടുള്ള ആദര്‍ശജീവിതവും ക്ഷമയും സഹനവും അവലംബിച്ചുകൊണ്ടുള്ള സത്യമത സന്ദേശപ്രചരണവും. മറ്റെന്ത് ത്യജിക്കേണ്ടി വന്നാലും ഈ ജിഹാദിന്റെ നിര്‍വഹണത്തില്‍ യാതൊരുവിധ വിട്ടുവീഴ്ചക്കും പ്രവാചകന്‍(സ) ഒരുക്കമല്ലായിരുന്നു. അതുകൊണ്ടാണ് സത്യസന്ധനെന്ന് വിളിച്ച് ആദരിച്ചിരുന്ന നാട്ടുകാരുടെ മുഴുവന്‍ വെറുപ്പിന് അദ്ദേഹം പാത്രമായത്; അല്ലലില്ലാതെയുള്ള ജീവിതത്തില്‍ നിന്ന് പട്ടിണിയിലേക്കും പീഡനങ്ങളിലേക്കും അദ്ദേഹം എടുത്തെറിയപ്പെട്ടത്; തനിക്ക് പ്രിയപ്പെട്ടവരും തന്നെ ഇഷ്ടപ്പെട്ടവരുമായിരുന്ന ബന്ധുക്കളുടെയും സ്വന്തക്കാരുടെയും വെര വിരോധത്തിന് അദ്ദേഹം പാത്രമായത്; തന്റെ അനുയായികൡല്‍ പലരും ക്രൂരമായ പീഡനങ്ങള്‍ക്ക് വിധേയമാകുന്നതും അവസാനം കൊല്ലപ്പെടുന്നതുമെല്ലാം നിസ്സഹായനായി അദ്ദേഹത്തിന് നോക്കി നില്‍ക്കേണ്ടി വന്നത്; ജനിച്ചു വളര്‍ന്ന നാടും വീടുമുപേക്ഷിച്ച് മറ്റൊരു ഗത്യന്തരവുമില്ലാത്തതു കൊണ്ട് പലായനം ചെേയ്യണ്ട ഗതിയുണ്ടായതും അതുകൊണ്ടു തന്നെ. വിട്ടുവീഴ്ചയില്ലാത്ത ആദര്‍ശജീവിതത്തിന് സ്വന്തത്തെ സജ്ജമാക്കുകയും സത്യത്തിന്റെ സല്‍പാന്ഥാവിലേക്ക് സഹജീവികളെ ആനയിക്കുകയും ചെയ്യുകയെന്ന ജിഹാദാണ് ഒരു വിട്ടുവീഴ്ചയുമില്ലാതെ, മുസ്‌ലിമിന്റെ ജീവിതത്തില്‍ എപ്പോഴുമുണ്ടാകേണ്ട ജിഹാദെന്ന് പഠിപ്പിക്കുകയായിരുന്നു, തിരുനബി(സ) തന്റെ ജീവിതത്തിലുടനീളം ചെയ്തത്.

  1. ഇമാം ഇബ്‌നുല്‍ ഖയ്യിം അല്‍ ജൗസിയ്യ: സാദുല്‍ മആദ്, ബൈറൂത്ത്, 2000, വാല്യം3, പുറം 9,10
  2. ഇമാം ഇബ്‌നുല്‍ ഖയ്യിം അല്‍ ജൗസിയ്യ: റൗദത്തുല്‍ മുഹിബ്ബീന്‍ വ നുസ്ഹത്തുല്‍ മുഷ്തക്കീന്‍. ബൈറൂത്ത്, 2000, പുറം 208
  3. അബൂബക്ര്‍ അല്‍ ഖാത്തിബ് അല്‍ ബാഗ്ദാദി: താരീഖുല്‍ ബഗ്ദാദി, 13/493 (http://theunjustmedia.com)
  4. പ്രവാചകാനുചരന്മാരായ സ്വഹാബികളില്‍ നിന്ന് മതം പഠിച്ച രണ്ടാം തലമുറയിലുള്ളവരെയാണ് താബിഉകള്‍ എന്ന് വിളിക്കുന്നത്
  5. ഇബ്‌നു ഹജറുല്‍ അസ്ഖലാനിയുടെ 'തസ്ദീദ് അല്‍ ഖൗസില്‍'’നിന്ന് ശൈഖ് ഹിഷാം കബ്ബാനി ഉദ്ധരിച്ചത്. (http://www.sunnah.org/tasawwuf/jihad004.html)
  6. Greater and "Lesser' Jihad?(http://the unjustmedia.com)
  7. സ്വഹീഹു ഇബ്‌നു ഹിബ്ബാന്‍ 11:203(http://www.sunnah.org/tasawwuf/jihad0 04.html)
  8. ഇമാം അല്‍ബാനി: സില്‍സിലത്തുല്‍ അഹാദീഥു സ്‌സ്വഹീഹ, വാല്യം 2, ഹദീഥ് 549 (http://www.alalbany.net)
  9. ഇമാം അല്‍ബാനി: സ്വഹീഹ് ഇബ്‌നുമാജ, ഹദീഥ് 3057 (http://www.alal-bany.net)
  10. ഇബ്‌നു മാജ, കിതാബുല്‍ ആദാബ്, ബാബു ഫദ്‌ലുദ്ദിക്ര്‍

റെ തെറ്റിദ്ധരിക്കപ്പെട്ട പദമാണ് ജിഹാദ്. വിവിധ ദേശങ്ങളില്‍ നടക്കുന്ന വിമോചന പോരാട്ടങ്ങളെക്കുറിച്ച പരാമര്‍ശങ്ങളിലും മാനവികതക്കു തന്നെ ഭീഷണിയായിത്തീരുന്ന ഭീകരവാദത്തെപ്പറ്റിയുള്ള അപഗ്രഥനങ്ങളിലും ഇസ്‌ലാമിനെയും മുഹമ്മദ് നബി(സ)യെയും പ്രതിചേര്‍ത്തുകൊണ്ടുള്ള പഠനങ്ങളിലുമെല്ലാം നിറഞ്ഞുനില്‍ക്കുന്ന പദമാണത്. ജിഹാദെന്ന ശബ്ദം കേള്‍ക്കുമ്പോഴേക്ക് ഒഴുകുന്ന രക്തവും കരയുന്ന വ്രണിതരും തകരുന്ന കെട്ടിടങ്ങളും നിരയായി കിടക്കുന്ന ശവശരീരങ്ങളും സ്‌ഫോടനത്തില്‍ നിന്നുയരുന്ന അഗ്‌നിസ്ഫുലിംഗങ്ങളും തോക്കും വാളും കഠാരയും മുഖംമൂടിയണിഞ്ഞ ആയുധധാരികളുമാണ് സാധാരണക്കാരന്റെ മനസ്സിലേക്ക് കടന്നു വരുന്നത്.ജിഹാദ് എന്ന പദവുമായി യാതൊരു ബന്ധവുമില്ലാത്ത ഈ പരികല്പനയുടെ സ്വാധീനം നമ്മുടെ മാധ്യമങ്ങളിലും പൊതുവ്യവഹാരങ്ങളിലുമെല്ലാം കാണാൻ കഴിയും.

കോടിക്കണക്കിന് കോപ്പികള്‍ വിറ്റഴിയുകയും മേശപ്പുറത്തെ എളുപ്പ റഫറന്‍സിന് ഏറ്റവുമധികം ഉപയോഗിക്കപ്പെടുകയും ചെയ്യുന്ന ഇംഗ്ലീഷ് നിഘണ്ടുവായ 'റീഡേഴ്‌സ് ഡൈജസ്റ്റ് ഓക്‌സ്‌ഫോര്‍ഡ് കംപ്ലീറ്റ് വേള്‍ഡ് ഫൈന്‍ഡര്‍' ജിഹാദിനെ നിര്‍വചിക്കുന്നത് 'അവിശ്വാസികള്‍ക്കെതിരെ മുസ്‌ലിംകള്‍ നടത്തുന്ന വിശുദ്ധയുദ്ധം'(1) എന്നാണ്. വിശദമായ ഇംഗ്ലീഷ് പദപഠനത്തിന് ഉപയോഗിക്കപ്പെടുന്ന 'വെബ്‌സ്റ്റേഴ്‌സ് എന്‍സൈക്ലോപീഡിക് ഡിക്ഷണറിയില്‍ 'മുസ്‌ലിംകള്‍ പരിശുദ്ധ ഉത്തരവാദിത്തമായി പരിഗണിക്കുന്ന വിശുദ്ധയുദ്ധം' എന്നും 'ഒരു ആശയത്തിനോ തത്ത്വത്തിനോ വേണ്ടി നടത്തുന്ന ഉജ്വലമായ, പലപ്പോഴും കയ്‌പേറിയ കുരിശു യുദ്ധം' എന്നുമാണ് ജിഹാദിന് നല്‍കിയിരിക്കുന്ന രണ്ട് നിര്‍വചനങ്ങള്‍.(2) 'മുഹമ്മദീയര്‍ നടത്തുന്ന പോര്; മതപ്രചാരണാര്‍ത്ഥം ഇസ്ലാം നിയമം അനുശാസിച്ചിട്ടുള്ള വിശുദ്ധയുദ്ധം' എന്നാണല്ലോ മലയാള ഭാഷാകാരന്‍മാരില്‍ പ്രമുഖനായ ശ്രീകണ്‌ഠേശ്വരം പത്മനാഭപിള്ള തന്റെ പ്രസിദ്ധമായ 'ശബ്ദ താരാവലി'യില്‍ ജിഹാദിനെ നിര്‍വചിച്ചത്.(3) പ്രൊഫ: ഇ.പി. നാരായണ ഭട്ടതിരിയുടെ അസ്സീസി മലയാളം നിഘണ്ടുവിലാകട്ടെ, 'മുസ്‌ലിംകളുടെ വിശുദ്ധയുദ്ധം; മതത്തിനു വേണ്ടിയുള്ള യുദ്ധം; കുരിശു യുദ്ധം; മിന്നലാക്രമണം പോലെ പെട്ടെന്നുള്ള ആക്രമണം' എന്നീ നാല് അര്‍ഥങ്ങള്‍ നല്‍കിയിട്ടുണ്ട്, ജിഹാദിന്.(4) നിഘണ്ടുകള്‍ വ്യാപകമായി നിര്‍മിക്കുവാനാരംഭിച്ച പതിനാറാം നൂറ്റാണ്ടിന്റെ തുടക്കം മുതലെങ്കിലും തന്നെ ജിഹാദെന്നാല്‍ കുരിശുയുദ്ധത്തിന് സമാനമായ ഇസ്‌ലാമിക സംജ്ഞയാണെന്ന ധാരണ നിലനിന്നിരുന്നുവെന്ന് ഇവ വ്യക്തമാക്കുന്നു. പദപരമായി നോക്കിയാൽ ജിഹാദ് എന്ന പദത്തിന് യുദ്ധവുമായി ബന്ധമൊന്നുമില്ല. ഹര്‍ബ്, വഇയ്യ്, ഇദാഅ്, ഖിസ്വാമ് എന്നിവയാണ് യുദ്ധത്തിനുപയോഗിക്കുന്ന അറബിപദങ്ങള്‍.(5) വിശുദ്ധ യുദ്ധത്തിന് 'അല്‍ ഹര്‍ബുല്‍ മുഖദ്ദസ' എന്നാണ് അറബിയില്‍ പറയുക. ഇങ്ങനെയൊരു സാങ്കേതിക ശബ്ദം ഇസ്‌ലാമിക ശബ്ദതാരാവലിയില്‍ കാണാന്‍ കഴിയില്ല. ജിഹാദിന് ഇങ്ങനെയൊരു അര്‍ഥം അംഗീകരിക്കപ്പെട്ട അറബി നിഘണ്ടുക്കളിലൊന്നും നല്‍കിയിട്ടുമില്ല. 'ജാഹദയുടെ നാമരൂപമായ ജിഹാദ് യഥാവിധി സൂചിപ്പിക്കുന്നത് ഒരാളുടെ പരമാവധി കഴിവും ശക്തിയും പരിശ്രമങ്ങളും അധ്വാനങ്ങളുമെല്ലാം അനിഷ്ടകരമായ എന്തിനെങ്കിലുമെതിരെ പോരാടുവാന്‍ വിനിയോഗിക്കുകയെന്നാണ്. ഇത് മൂന്നുതരമുണ്ട്. പ്രത്യക്ഷ ശത്രുവിനോടുള്ളത്, പിശാചിനോടുള്ളത്, സ്വന്തത്തോടുള്ളത്. ക്വുര്‍ആനിലെ 22:77 ലെ ഈ പ്രയോഗത്തില്‍ ഇവ മൂന്നും ഉള്‍പ്പെടുന്നു'െവന്നാണ് പ്രസിദ്ധമായ ലെയിന്‍സ് അറബി ഇംഗ്ലീഷ് ലെക്‌സിക്കണ്‍ പറയുന്നത്.(6) ഇതിനു സമാനമായ അര്‍ഥം തന്നെയാണ് ക്ലാസിക്കല്‍ അറബി നിഘണ്ടുക്കളും ജിഹാദിന് നല്‍കിയിട്ടുള്ളത്.(7)

ആദ്യകാലത്തോ പില്‍ക്കാലത്തോ ഉള്ള ക്വുര്‍ആന്‍ വ്യാഖ്യാതാക്കളാരും തന്നെ ജിഹാദിനെക്കുറിച്ച ചര്‍ച്ചയില്‍ അത് 'അല്‍ഹര്‍ബുല്‍ മുഖദ്ദസ'യാണെന്ന് സമര്‍ഥിച്ചതായി കാണാനാവില്ല. പ്രമുഖ ബ്രിട്ടീഷ് ഓറിയന്റലിസ്റ്റായ ആര്‍തര്‍ ജോണ്‍ ആര്‍ബറി തന്റെ ക്വുര്‍ആന്‍ പരിഭാഷയിലുടനീളം 'ജിഹാദി'നെ പരിഭാഷപ്പെടുത്തിയിരിക്കുന്നത്"struggle'എന്നാണ്, holywar എന്നല്ല എന്ന കാര്യവും ശ്രദ്ധേയമാണ്.(8) പോരാട്ടത്തിനും യുദ്ധത്തിനുമെല്ലാംstruggleഎന്നു പറയുമെങ്കിലും, എന്തെങ്കിലുമൊരു കാര്യം നേടിയെടുക്കുന്നതിനു വേണ്ടിയുള്ള തീവ്രമായ യജ്ഞത്തിനാണല്ലോ പൊതുവായി ആ ആംഗലേയ ശബ്ദം പ്രയോഗിക്കപ്പെടാറുള്ളത്; ഏതായിരുന്നാലും,struggle എന്ന ഇംഗ്‌ളീഷ് പദത്തിന് വിശുദ്ധയുദ്ധമെന്ന് അര്‍ഥമില്ലെന്ന് വ്യക്തമാണ്. ഇതു തന്നെയാണ് ജിഹാദ് എന്ന അറബി പദത്തിന്റെയും അവസ്ഥ. 'തീവ്രമായ പരിശ്രമമെന്നാണ്' പ്രസ്തുത പദത്തിന്റെ വിവക്ഷ. നന്‍മക്കു വേണ്ടിയുള്ള പോരാട്ടം തിന്‍മയെ വിപാടനം ചെയ്യുന്നതിനായി സ്വന്തം ജീവന്‍ നല്‍കിക്കൊണ്ട് നടത്തുന്ന തീവ്രയജ്ഞമായതിനാല്‍ അത് ജിഹാദാണ്. ജിഹാദ് എന്നാല്‍ പോരാട്ടമല്ല; എന്നാല്‍ പോരാട്ടം ജിഹാദായിത്തീരുമെന്നര്‍ഥം.

ജിഹാദായിത്തീരുന്ന പോരാട്ടത്തെക്കുറിച്ചു പോലും മുസ്‌ലിംകള്‍ 'വിശുദ്ധയുദ്ധം' എന്നു വിളിക്കാറില്ല. സ്വതവേ അശുദ്ധമായ രക്തച്ചൊരിച്ചിലുകളെ മാര്‍പാപ്പയാല്‍ മാമോദീസ മുക്കി വിശുദ്ധമാക്കുകയാണ് മധ്യകാല ക്രൈസ്തവ നേതൃത്വം ചെയ്തത്. അധീശത്വത്തിനുവേണ്ടി നടത്തുന്ന അതിക്രമങ്ങളുടെ സമാഹാരമായി നിലനില്‍ക്കുന്ന സാമൂഹ്യരൂപമായ യുദ്ധത്തെ അത് സഭയുടെ താല്‍പര്യസംരക്ഷണത്തിനാണ് എന്ന ഒരേയൊരു കാരണത്താല്‍ മാമോദീസ മുക്കി വിശുദ്ധമാക്കിയപ്പോള്‍ അതിന്റെ പേരില്‍ നടന്നത് നിരപരാധികളും നിഷ്‌കളങ്കരുമായ മനുഷ്യരുടെ കൂട്ടക്കുരുതികളായിരുന്നു.

പോരാട്ടം ജിഹാദായിത്തീരുന്ന സന്ദര്‍ഭത്തെക്കുറിച്ച് വിശദീകരിക്കുന്ന ഇസ്‌ലാമിക പ്രമാണങ്ങള്‍ അത് എന്തിന്, എങ്ങനെയാണ് നടത്തേണ്ടതെന്നു കൂടി കൃത്യമായി പഠിപ്പിക്കുന്നുണ്ട്.വിശ്വാസആരാധനാപ്രബോധന സ്വാതന്ത്ര്യങ്ങള്‍ നേടിയെടുക്കുന്നതിനായി നടക്കുന്ന പോരാട്ടങ്ങളുടെ ലക്ഷ്യവും മാര്‍ഗവും ദൈവിക മാര്‍ഗദര്‍ശന പ്രകാരമുള്ളതായിത്തീരുമ്പോള്‍ മാത്രമാണ് അത് ജിഹാദായിത്തീരുക. സ്വതവെ അശുദ്ധമായ യുദ്ധത്തെ പരിശുദ്ധാത്മ സാമീപ്യത്താലുള്ള മാര്‍പാപ്പ പ്രഖ്യാപനം വഴി വിശുദ്ധമാക്കി, അതിന്റെ പേരില്‍ അണികളെ ആയുധമണിയിച്ച് എന്തും ചെയ്യാനുള്ള അനുമതിയും കൊടുത്ത് തുറന്ന് വിടുന്ന ഏര്‍പ്പാടല്ല അത,് പ്രത്യുത അനിവാര്യമായ സാഹചര്യത്തില്‍ നടത്തുന്ന, നിയതമായ മാര്‍ഗവും കൃത്യമായ ലക്ഷ്യവുമുള്ള പോരാട്ടമാണത്; അശുദ്ധമായ ഒന്നിനെ വിശുദ്ധവല്‍ക്കരിച്ചതല്ല ജിഹാദ്; അത് എല്ലായ്‌പ്പോഴും വിശുദ്ധമാണ്; വിശുദ്ധമായതേ ജിഹാദാവൂ എന്നർത്ഥം. 'ജിഹാദ്' എന്ന നാമരൂപമോ അതിന്റെ ക്രിയാരൂപങ്ങളോ ആകെ 41 തവണയാണ് ക്വുര്‍ആനില്‍ പ്രയോഗിക്കപ്പെട്ടിരിക്കുന്നത്. നാമരൂപത്തില്‍ ജിഹാദ് എന്നു പ്രയോഗിച്ചിരിക്കുന്നത് നാലു തവണയാണ്; സൂറത്തുത്തൗബയിലെ 24ാം വചനത്തിലും സൂറത്തുല്‍ ഹജ്ജിലെ 78ാം വചനത്തിലും സൂറത്തുല്‍ ഫുര്‍ഖാനിലെ 52ാം വചനത്തിലും സൂറത്തുല്‍ മുംതഹനയിലെ ഒന്നാം വചനത്തിലുമാണ് അവ. മറ്റു സൂക്തങ്ങളില്‍, ജാഹദൂ, ജാഹിദൂ, യുജാഹിദു, ജാഹദ, യുജാഹിദൂന, തുജാഹിദൂന, മുജാഹിദൂന തുടങ്ങിയ വ്യത്യസ്ത ജാഹദാ രൂപങ്ങള്‍ പ്രയോഗിക്കുന്നുണ്ട് ക്വുര്‍ആന്‍. 'യുദ്ധംചെയ്യുക'യെന്ന കേവലമായ അര്‍ഥത്തിനപ്പുറം, യുദ്ധമടക്കമുള്ള ത്യാഗപരിശ്രമങ്ങളെക്കുറിക്കുവാനാണ് പ്രയോഗങ്ങള്‍ നടത്തുന്നതെന്ന് വചനങ്ങള്‍ പരിശോധിച്ചാല്‍ വ്യക്തമാവും.

യുദ്ധത്തെ മാത്രം സൂചിപ്പിക്കുന്നതിന് ക്വുര്‍ആനില്‍ പ്രയോഗിക്കുന്നത് 'ഖിതാല്‍' എന്ന പദമോ അതിന്റെ ക്രിയാരൂപങ്ങളോ ആണ്. പോരാട്ടം എന്ന് അര്‍ഥം വരുന്ന 'ഖിതാല്‍' എന്ന അറബിപദമാണ്, ധനസമ്പാദനം ലക്ഷ്യമാക്കി അറബികള്‍ ചെയ്തിരുന്ന യുദ്ധത്തെക്കുറിക്കുവാന്‍ അറബികള്‍ പ്രയോഗിച്ചിരുന്ന 'ഹര്‍ബിനും' ഗോത്രവൈരം തീര്‍ക്കാനുള്ള ഏറ്റുമുട്ടലുകള്‍ക്ക് അവര്‍ പറഞ്ഞിരുന്ന 'വഇയ്യിനും' പകരം ക്വുര്‍ആന്‍ പ്രയോഗിച്ചിരിക്കുന്നത് എന്ന കാര്യം പ്രത്യേകം പ്രസ്താവ്യമാണ്. ഖിതാലും ജിഹാദും പര്യായപദങ്ങളല്ല. ജിഹാദിന്റെ അര്‍ഥമായി 'ഖിതാല്‍' എന്ന് ഭാഷാകാരന്‍മാരൊന്നും രേഖപ്പെടുത്തിയിട്ടുമില്ല. ഖിതാല്‍ മാത്രമല്ല ജിഹാദ്; എന്നാല്‍ ഖിതാലും ജിഹാദായിത്തീരും എന്നാണ് ജിഹാദ്, ഖിതാല്‍ എന്നീ പ്രയോഗങ്ങള്‍ നടത്തിയ ക്വുര്‍ആന്‍ വചനങ്ങളും ഹദീഥുകളും പരിശോധിച്ചാല്‍ നമുക്ക് മനസ്സിലാവുക. മുസ്‌ലിംകളായ തങ്ങളുടെ സന്താനങ്ങളെ ബഹുദൈവാരാധനക്ക് നിര്‍ബന്ധിക്കുന്ന സത്യനിഷേധികളായ മാതാപിതാക്കളുടെ ചെയ്തിയെ ക്വുര്‍ആന്‍ വിശേഷിപ്പിച്ചിരിക്കുന്നത് 'ജിഹാദ്' എന്നാണ്: (29:8) ബഹുദൈവാരാധനക്ക് നിര്‍ബന്ധപൂര്‍വം ആവശ്യപ്പെടുന്നതിനാണ് ഈ വചനത്തില്‍ ജിഹാദ് എന്ന് പ്രയോഗിച്ചിരിക്കുന്നത് എന്ന് വ്യക്തമാണ്.

ജിഹാദിനെക്കുറിച്ച് പരാമര്‍ശിക്കുന്ന ക്വുര്‍ആന്‍ സൂക്തങ്ങളില്‍ ചിലതില്‍ സ്വശരീരംകൊണ്ടും സമ്പത്തുകൊണ്ടും ജിഹാദ് ചെയ്യാന്‍ കല്‍പിക്കുന്നതായി കാണാന്‍ കഴിയും: ''അല്ലാഹുവിലും അവന്റെ ദൂതനിലും വിശ്വസിക്കുകയും പിന്നീട് സംശയിക്കാതിരിക്കുകയും, തങ്ങളുടെ സ്വത്തുക്കളും ശരീരങ്ങളും കൊണ്ട് അല്ലാഹുവിന്റെ മാര്‍ഗത്തില്‍ സമരം നടത്തുകയും ചെയ്തവരാരോ അവര്‍ മാത്രമാകുന്നു സത്യവിശ്വാസികള്‍. അവര്‍ തന്നെയാകുന്നു സത്യവാന്‍മാര്‍.'' (49:15) രണഭൂമിയിലെ പോരാട്ടം അടക്കമുള്ള ത്യാഗപരിശ്രമങ്ങളെല്ലാം ശരീരംകൊണ്ടുള്ള ജിഹാദാണ്. നന്മയുടെ മാര്‍ഗത്തില്‍ സ്വത്തുക്കള്‍ ചെലവഴിക്കുകയെന്ന സാമ്പത്തിക ത്യാഗപരിശ്രമമാണ് ഇവിടെ സമ്പത്തുകൊണ്ടുള്ള ജിഹാദുകൊണ്ട് വിവക്ഷിക്കപ്പെട്ടിരിക്കുന്നത് എന്ന് വ്യക്തമാണ്.

ആദര്‍ശ ശത്രുക്കളോട് ക്വുര്‍ആന്‍ ഉപയോഗിച്ച് ജിഹാദ് ചെയ്യുവാനാണ് അല്ലാഹുവിന്റെ കല്‍പന: ''അതിനാല്‍ സത്യനിഷേധികളെ നീ അനുസരിച്ചു പോകരുത്. ഇത് (ഖുര്‍ആന്‍) കൊണ്ട് നീ അവരോട് വലിയൊരു സമരം നടത്തിക്കൊള്ളുക.'' (25:52) പീഡനങ്ങളും പ്രയാസങ്ങളും സഹിച്ച് മക്കയില്‍ ആദര്‍ശപ്രബോധനം നിര്‍വഹിക്കുന്നതിനിടയില്‍ അവതരിപ്പിക്കപ്പെട്ട ഈ സൂക്തത്തില്‍ ക്വുര്‍ആന്‍ ഉപയോഗിച്ചുകൊണ്ടാണ് ഏറ്റവും വലിയ ജിഹാദ് ചെയ്യേണ്ടതെന്നാണ് കല്‍പിക്കപ്പെട്ടിരിക്കുന്നതെന്ന വസ്തുത ശ്രദ്ധേയമാണ്. ക്വുര്‍ആനികാദര്‍ശങ്ങളെ സത്യനിഷേധികള്‍ക്ക് മുമ്പില്‍ സമര്‍പ്പിക്കുന്നതിനെയും അതുപയോഗിച്ച് ആദര്‍ശസമരം നടത്തുന്നതിനെയുംകുറിച്ചാണ് ഇവിടെ ക്വുര്‍ആന്‍ കൊണ്ടുള്ള ജിഹാദ് എന്ന് പരിചയപ്പെടുത്തിയിരിക്കുന്നത് എന്ന് വ്യക്തം.

'അല്ലാഹുവിന്റെ മാര്‍ഗത്തില്‍ ജിഹാദ് ചെയ്യേണ്ട മുറപ്രകാരം നിങ്ങള്‍ ജിഹാദ് ചെയ്യുക' (22:78) എന്ന് കല്‍പിക്കുന്ന ക്വുര്‍ആന്‍ വചനം 'നിങ്ങളുടെ പിതാവായ ഇബ്‌റാഹീമിന്റെ മാര്‍ഗമാണത്' എന്നാണ് അനുബന്ധമായി പറഞ്ഞിരിക്കുന്നത്. ത്യാഗനിര്‍ഭരമായ ആദര്‍ശ സമരമായിരുന്നു ഇബ്‌റാഹീം നബി(അ)യുടെ ജീവിത്തില്‍ നിറഞ്ഞുനിന്നിരുന്നത് എന്ന വസ്തുത ക്വുര്‍ആനിലൂടെ കണ്ണോടിക്കുന്ന ആര്‍ക്കും മനസ്സിലാകും. ഇബ്‌റാഹീം നബി(അ)നടത്തിയ സായുധ സമരങ്ങളെക്കുറിച്ച പരാമര്‍ശങ്ങളൊന്നുംതന്നെ ക്വുര്‍ആനിലില്ല. സ്രഷ്ടാവ് മാത്രമാണ് ആരാധനകള്‍ അര്‍ഹിക്കുന്നതെന്ന തത്ത്വം പ്രബോധനം ചെയ്യുകയും പ്രസ്തുത മാര്‍ഗത്തില്‍ ത്യാഗങ്ങള്‍ അനുഷ്ഠിക്കുകയും ചെയ്തതാണ് ഇബ്‌റാഹീം നബിൗയുടെ ജീവിതത്തില്‍ നിറഞ്ഞുനിന്നിരുന്ന ജിഹാദ്. വിശ്വാസികള്‍ക്ക് എന്നും മാതൃകയായ ജിഹാദാണത്; അവരുടെ ജീവിതത്തില്‍നിന്ന് ഒരിക്കലും മാറ്റിനിര്‍ത്താനാവാത്ത ജിഹാദ്.

1. "Jihad: a holy war undertaken by Muslims against unbelievers', Sarah Tulloch (Ed): Reader's Digest Oxford Complete Word finder, Milan, 1994, Page 819

2. "Jihad:a holy war undertaken as a sacred duty by Muslims; any vigoro-us, often bitter crusade for an idea or principle.'”Webster's Encyclopaedic Unabridged Dictionary of the English Language, New York, 1989, Page 767

3. ശബ്ദതാരാവലി, 9ാം പതിപ്പ്, ശ്രീ കണ്‌ഠേശ്വരം ജി. പത്മനാഭപ്പിള്ള, പ്രസാധകര്‍: സാഹിത്യ പ്രവര്‍ത്തക സഹകരണ സംഘം ലിമിറ്റഡ്, നാഷണല്‍ ബുക്ക്സ്റ്റാള്‍ കോട്ടയം, പേജ്: 871

4. പ്രൊഫ: ഇ.പി. നാരായണ ഭട്ടതിരി: അസ്സീസി മലയാളം നിഘണ്ടു, ചങ്ങനാശേരി, 1997,പുറം 328

5. حرب، وغي، عداء، خصام

6. "Jihad, infinitive noun of jaahada, properly signifies the using or exerting of one's utm-ost power, eÅorts, endeavours or ability, in contending with an object of disappro-bation; and this is of three kinds, namely, a visible enemy, the devil, and one's self; all of which are included in the term as used in the Quran 22:77.'' (Edward William Lane: An Arabic-English Lexicon, Beirut, 1968, Page 473)

7. ലിസാനുല്‍ അറബ് 4/107, താജുല്‍ അറൂസ് 2/329

8. A. J. Arberry: The Koran Interpreted: A Translation, Touchstone, 1996, Look 9:24, 22:78, 25:52, 60:1 etc.

 
 

സ്‌ലാമിക നിയമങ്ങളെല്ലാം മാനവികവും പ്രായോഗികവുമാണ്. സിവിൽ നിയമങ്ങളാണെങ്കിലും ക്രിമിനൽ നിയമങ്ങളാണെങ്കിലും ഇസ്‌ലാമിക നിയമങ്ങളിൽ മാനവവിരുദ്ധമായ യാതൊന്നും കാണാൻ കഴിയില്ല.സിവിൽ നിയമങ്ങൾ വ്യക്തി അനുസരിക്കേണ്ടതാണ്. ഏത് സമൂഹത്തിൽ ജീവിക്കുന്നവയാണെങ്കിലും മുസ്ലിം ആ നിയമങ്ങൾ അനുസരിച്ചാണ് ജീവിക്കേണ്ടത്. ക്രിമിനൽ നിയമങ്ങൾ രാഷ്ട്രം നടപ്പാക്കേണ്ടവയാണ്. ഇസ്‌ലാമിക രാഷ്ട്രത്തിലെ ഭരണാധികാരിക്ക് മാത്രമേ പ്രസ്തുത നിയമങ്ങൾ നടപ്പാക്കേണ്ട ഉത്തരവാദിത്തമുള്ളൂ. പ്രജകൾ പ്രസ്തുത നിയമങ്ങൾക്ക് വിധേയരായി ജീവിക്കേണ്ടവരാണ്.

 ഭരണാധികാരി തന്റെ പ്രജകൾക്ക് മേൽ നടപ്പാക്കേണ്ട ശിക്ഷാവിധികളെക്കുറിച്ച് പരാമര്‍ശിക്കുമ്പോള്‍ ഒരുകാര്യം വ്യക്തമാക്കേണ്ടതുണ്ട്. കുറ്റവാളികളെ ശിക്ഷിക്കുകയല്ല, പ്രത്യുത, കുറ്റകൃത്യ ങ്ങളില്ലാത്ത ഒരു സമൂഹത്തെ സൃഷ്ടിക്കുകയാണ് ഇസ്‌ലാമിന്റെ ലക്ഷ്യമെന്ന വസ്തുതയാണത്. കൊലപാതകിയെ കൊല്ലുകയോ മോഷ്ടാവിന്റെ കൈ വെട്ടുകയോ വ്യഭിചാരികളെ അടിക്കുകയോ ഒന്നുമല്ല, കൊലയും കൊള്ളയും അധാര്‍മികവൃത്തികളുമില്ലാത്ത ഒരു സമൂഹത്തെ സൃഷ്ടിക്കുകയാണ് ഇസ്‌ലാമിന്റെ ലക്ഷ്യം. ഇതിന് കര്‍ശനമായ ശിക്ഷാനിയമങ്ങള്‍ ആവശ്യമാണെന്നാണ് ഇസ്‌ലാമിന്റെ പക്ഷം.

വ്യക്തിസ്വാതന്ത്ര്യത്തിന്റെയും പരിഷ്‌കാരത്തിന്റെയും പേരില്‍ കുത്തഴിഞ്ഞ ജീവിതം അനുവദിക്കുന്ന 'പരിഷ്‌കൃത'നാടുകളില്‍ കുറ്റകൃത്യങ്ങള്‍ വര്‍ധിച്ചുവരുന്നതിന്റെ കാരണം അവിടത്തെ സമൂഹത്തിന്റെ ധാര്‍മികാടിത്തറയുടെ പിഴവും ശക്തമായ ശിക്ഷാനിയമങ്ങളുടെ അഭാവവുമാണെന്നാണ് സാമൂഹ്യ ശാസ്ത്രജ്ഞരുടെ പക്ഷം. കുറ്റകൃത്യങ്ങളില്‍ നിന്ന് മുക്തമായ ഒരു സമൂഹത്തിന്റെ സൃഷ്ടിക്ക് ശക്തമായ ശിക്ഷാനിയമങ്ങള്‍ ആവശ്യമാണെന്ന ഇസ്‌ലാമിന്റെ കാഴ്ചപ്പാട് നൂറ് ശതമാനവും മാനവികമാണെന്ന് സാരം. ഇസ്‌ലാം ശിക്ഷാവിധികള്‍ നടപ്പാക്കാന്‍ ആഹ്വാനം ചെയ്യുന്നത് അവയ്ക്ക് യാതൊരു പഴുതുമില്ലാത്ത ഒരു സമൂഹത്തിന്റെ സൃഷ്ടി സാധിച്ച ശേഷമാണെന്ന വസ്തുത പ്രത്യേകം പരാമര്‍ശിക്കപ്പെടേ ണ്ടതാണ്. കുറ്റകൃത്യങ്ങള്‍ക്ക് പ്രേരണ നല്‍കുന്ന പരിതഃസ്ഥിതികള്‍ ഇല്ലാതാക്കിയതിന്നു ശേഷവും കുറ്റകൃത്യങ്ങളിലേര്‍പ്പെടുന്നവര്‍ ക്കാണ് കഠിനമായ ശിക്ഷ വിധിക്കുവാന്‍ ഇസ്‌ലാം കല്‍പിക്കുന്നത്. ഇസ്‌ലാം അനുശാസിക്കുന്ന രൂപത്തിലുള്ള സകാത്ത് സമ്പ്രദായം നടപ്പാക്കുകവഴി സാധാരണക്കാരനില്‍ നിന്ന് പട്ടിണിയും ദാരിദ്ര്യവും തുടച്ചുമാറ്റിയ ശേഷവും ജനങ്ങളുടെ സൈ്വരജീവിതത്തിന് ഭീഷണിയുയര്‍ത്തുന്ന മോഷ്ടാക്കള്‍ നിലനില്‍ക്കുന്നുവെങ്കില്‍ അവരുടെ കരംഛേദിച്ചുകൊണ്ട് സമൂഹത്തിന് മാതൃക കാണിച്ചു കൊടുക്കണമെന്നാണ് ഇസ്‌ലാം പഠിപ്പിക്കുന്നത്. ശക്തമായ ശിക്ഷാ സമ്പ്രദായത്തിന്റെ അഭാവത്തില്‍ കുറ്റകൃത്യങ്ങൾ പെരുകുമെന്ന ഇസ്‌ലാമിന്‍െര്‍ കാഴ്ചപ്പാട് നൂറു ശതമാനവും ശരിയാണെന്ന് സമകാലീനസംഭവങ്ങള്‍ വ്യക്തമാക്കുന്നു.

തൊഴിലില്ലായ്മയും ദാരിദ്ര്യവും നിലനില്‍ക്കുന്ന ഒരു സമൂഹത്തില്‍, കുറ്റവാളികളെ വളര്‍ ത്താനാവശ്യമായ എല്ലാ സാഹചര്യങ്ങളുമുള്ളപ്പോള്‍, മോഷ്ടാവിന്റെ കൈ ഛേദിച്ചുകളയാന്‍ ഇസ്‌ലാം വിധിക്കുന്നില്ല. രാജ്യത്ത്ക്ഷാമം പടര്‍ന്നുപിടിച്ചിരുന്ന സമയത്ത് ഒരു മോഷ്ടാവിനെ ഖലീഫാ ഉമറിന്റെ(റ)സന്നിധിയില്‍ ഹാജരാക്കിയപ്പോള്‍ അയാള്‍ പ ട്ടിണിമൂലം മോഷണത്തിന് നിര്‍ബന്ധിതനായിരിക്കാമെന്ന കാരണ ത്താല്‍ അയാളെ വെറുതെ വിട്ടതായുള്ള ചരിത്രത്തില്‍ നിന്ന് ഇക്കാര്യം വളരെ വ്യക്തമായി മനസ്സിലാക്കാവുന്നതാണ്. ഇതു തന്നെയാണ് എല്ലാ ശിക്ഷാവിധികളിലുമുള്ള ഇസ്‌ലാമിന്റെ നിലപാട്.

വ്യഭിചാരത്തിന് വശംവദമാകത്തക്കരീതിയിലുള്ള ലൈംഗി കാഭാസങ്ങളെല്ലാം ഉന്മൂലനം ചെയ്ത ശേഷമാണ് വ്യഭിചാരത്തിനുള്ള ശിക്ഷ നടപ്പാക്കാന്‍ ഇസ്‌ലാം കല്‍പിക്കുന്നത്. മോഡലുകളും കാള്‍ഗേളുകളും സെക്‌സ് ബോംബുകളുമില്ലാത്ത, കാബറെ മുതല്‍ മോഹിനിയാട്ടം വരെയുള്ള നൃത്തങ്ങളിലൂടെ ലൈംഗികാഭാസങ്ങള്‍ക്ക് പ്രേരണയുണ്ടാക്കാത്ത, സ്വന്തം ശരീരവും സൗന്ദര്യവും വില്‍പനച്ചരക്കല്ലെന്ന് വിശ്വസിക്കുന്ന കുടുംബിനികള്‍ മാത്രമുള്ള, ഒരു വിവാഹംകൊണ്ട് ലൈംഗികദാഹം ശമിപ്പിക്കാന്‍ കഴിയാതെ വരുന്ന പുരുഷന്ന് നാലുവരെ ഇണകളെ സ്വീകരിക്കാന്‍ സ്വാതന്ത്ര്യം നല്‍കുന്ന ഒരു സമൂഹത്തില്‍ നാലുപേര്‍ കാണ്‍കെ വ്യഭിചാരത്തിലേര്‍പ്പെടുന്നവര്‍ ശക്തമായ ശിക്ഷാനടപടികള്‍ അര്‍ഹിക്കുന്നുവെ ന്നാണ് ഇസ്‌ലാമിന്റെ വിധി.

ശിക്ഷാസമ്പ്രദായങ്ങളിലും അതല്ലാത്ത നിയമങ്ങളിലുമെല്ലാം ഇസ്‌ലാമിക രാഷ്ട്രത്തിലെ എല്ലാ പൗരന്മാരും തുല്യരായാണ് പരി ഗണിക്കപ്പെടുന്നത്. സമ്പന്നനും ദരിദ്രനുമിടയിലോ ഭരണാധികാരിക്കും സാധാരണ പൗരന്നുമിടയിലോ യാതൊരുരീതിയിലുള്ള വിവേചനവും നീതിനിര്‍വഹണത്തിന്റെ കാര്യത്തില്‍ ഉണ്ടാവാന്‍ പാടില്ലെന്നാണ് ഇസ്‌ലാമിന്റെ അധ്യാപനം. 'സത്യവിശ്വാസികളേ, നിങ്ങള്‍ അല്ലാഹുവിനുവേണ്ടി സാക്ഷ്യം വഹിക്കുന്നവരെന്ന നിലയയില്‍ കണിശമായി നീതി നിലനിര്‍ത്തുന്നവരായിരിക്കണം. അത് നിങ്ങള്‍ക്ക്തന്നെയോ, നിങ്ങളുടെ മാതാപിതാക്കള്‍, അടുത്ത ബ ന്ധുക്കള്‍ എന്നിവര്‍ക്കോ പ്രതികൂലമായിത്തീര്‍ന്നാലും ശരി'(ക്വുർആൻ4:135) എന്നാണ് ക്വുര്‍ആന്‍ പഠിപ്പിക്കുന്നത്. കുറ്റകൃത്യങ്ങൾ ഇല്ലാതെയാക്കി സമൂഹത്തിൽ സമാധാനത്തോടെ ജീവിക്കുവാനാവശ്യമായ സംവിധാനങ്ങളുണ്ടാക്കുകയെന്ന ലക്ഷ്യത്തോടെയുള്ള ഇസ്‌ലാമിക ശിക്ഷാവിധികളെല്ലാം മാനവികമാണെന്ന് അല്പം ചിന്തിച്ചാൽ ആർക്കും ബോധ്യമാകും.

അല്ല. ഒരാൾ തൻറെ ഇച്ഛയനുസരിച്ച് ഭരിക്കുന്നതാണ് ഏകഛത്രാധിപത്യമെങ്കിൽ ഇസ്‌ലാം അത്തരമൊരു വ്യവസ്ഥയെ ഒരു തരത്തിലും അംഗീകരിക്കുന്നില്ല. ഇസ്‌ലാമികരാഷ്ട്രത്തിലെ അടിസ്ഥാന ധാര്മിക നിയമങ്ങൾ തീരുമാനിക്കുന്നത് ഏതെങ്കിലുമൊരാളോ ഒരു കൂട്ടം ആളുകളോ അല്ല; രാജാധിരാജനായ അല്ലാഹുവാണ്. അവന്റെ നിയമങ്ങൾ മാറ്റിമറിക്കുവാൻ ആർക്കും അവകാശമില്ല. ക്വുർആനും സുന്നത്തുമനുസരിച്ച് കാര്യനിര്വഹണം നടത്തുക മാത്രമാണ് ഇസ്‌ലാമികരാഷ്ട്രത്തിലെ ഭരണാധികാരി ചെയ്യുന്നത്. രാഷ്ട്രനിർമ്മാണപരമായ കാര്യങ്ങൾ തീരുമാനിക്കേണ്ടത് അയാളുടെ ചുമതലയാണ്. അവ പോലും എകാധിപത്യപരമായി തീരുമാനിക്കണമെന്നല്ല പ്രവാചകൻ (സ) പഠിപ്പിച്ചത്. അവ തീരുമാനിക്കേണ്ടത് കൂടിയാലോചനയിലൂടെയാണ്.

ഇസ്‌ലാമിക രാഷ്ട്രത്തിന്റെ നായകനാണ് ഖലീഫ. വിശുദ്ധ ക്വുര്‍ആനിലും പ്രവാചകചര്യയിലും വ്യക്തമാ ക്കപ്പെട്ട അടിസ്ഥാനതത്ത്വങ്ങള്‍ക്കനുസൃതമായി രാഷ്ട്രത്തെ നയി ക്കുകയാണ് ഖലീഫയുടെ ഉത്തരവാദിത്തം. ക്വുര്‍ആനിലും സുന്ന ത്തിലും വ്യക്തമായി വിവരിക്കപ്പെടാത്ത ഒട്ടനവധി പ്രശ്‌നങ്ങള്‍ ഒരു രാഷ്ട്രത്തിന് കൈകാര്യം ചെയ്യേണ്ടതായി വരുമല്ലോ. രാഷ്ട്രീയപ്രശ്‌നങ്ങളില്‍ ഖണ്ഡിതമായ നിയമങ്ങളെക്കാളധികം പാലിക്കപ്പെടേണ്ട തത്ത്വങ്ങളാണ് വിശുദ്ധക്വുര്‍ആന്‍ പരാമര്‍ശിക്കുന്നത്. ഈ തത്ത്വങ്ങളുടെ വെളിച്ചത്തില്‍, ഇസ്‌ലാമിന്റെ ആത്മാവിനിണങ്ങുന്ന വിധത്തില്‍ നിയമനിര്‍മാണം നടത്തേണ്ടത് ഖലീഫയുടെ ബാധ്യത യാണ്. ഈ നിയമങ്ങള്‍ നിര്‍മിക്കേണ്ടത് ഖലീഫയുടെ തന്നിഷ്ടപ്ര കാരമല്ല; പ്രത്യുത കൂടിയാലോചനയിലൂടെയാണ്. കൂടിയാലോചനാ സമിതിയെയാണ് ഇസ്‌ലാമികരാഷ്ട്രത്തില്‍ 'ശൂറാ' എന്നു വിളി ക്കുന്നത്.

പ്രവാചകന്‍പോലും രാഷ്ട്രീയമായ പ്രശ്‌നങ്ങള്‍ കൂടിയാ ലോചനയിലൂടെയായിരുന്നു തീരുമാനിച്ചിരുന്നതെന്ന് കാണാന്‍ കഴിയും. 'കാര്യങ്ങളില്‍ നീ അവരോട് കൂടിയാലോചിക്കുകയും ചെയ്യുക'(ക്വുർആൻ 3:159) എന്നാണ് പ്രവാചകനോടുള്ള ദൈവികകല്‍പന. അബൂഹുറയ്‌റ(റ)പറയുന്നു: 'പ്രവാചകനെപ്പോലെ സ്വന്തം അനുയായികളുമായി ഇത്രയേറെ കൂടിയാലോചന ചെയ്യുന്ന മറ്റൊരു വ്യക്തിയെ യും ഞാന്‍ കണ്ടിട്ടില്ല'.(ബുഖാരി) അനുയായികളുമായി കൂടിയാലോചിക്കുകയും അവരുടെ അഭിപ്രായങ്ങള്‍ മാനിക്കുകയും ചെയ്ത ആളായിരുന്നു പ്രവാചകന്‍(സ). ബദർ യുദ്ധവേളയില്‍ ആദ്യം തെരഞ്ഞെടുത്ത താവളം മാറ്റിയതും ഉഹ്ദ്‌യുദ്ധത്തിന് പുറപ്പെട്ടതും ഖന്‍ദഖ് യുദ്ധസമയത്ത് ഗത്ഫാന്‍ ഗോത്രവുമായി സന്ധിചെയ്യാനുള്ള തന്റെ തീരുമാനം മാറ്റിയതു മെല്ലാം അനുയായികളുമായി കൂടിയാലോചിച്ചശേഷമായിരുന്നു. ചുരുക്കത്തിൽ, ഏകഛത്രാധിപത്യത്തിന്റെ ലാഞ്ഛന പോലുമില്ലാത്തതാണ് ഇസ്‌ലാമിക രാഷ്ട്രം.

ന്ന് വ്യവഹിക്കപ്പെടുന്ന രീതിയിലുള്ള ജനാധിപത്യത്തെക്കുറിച്ച് നാം ചിന്തിക്കാനാരംഭിച്ചതു തന്നെ പതിനെട്ടാം നൂറ്റാണ്ടിനു ശേഷമാണ്. അതിനു മുൻപ് നിലനിന്നിരുന്ന വ്യത്യസ്തങ്ങളായ രാഷ്ട്രമീമാംസകളുണ്ട്. ഗോത്രാധിപത്യവും രാജാധിപത്യവുമെല്ലാം അവയിൽ ചിലതാണ്.സമൂഹത്തിന്റെ വളർച്ചയ്ക്കനുസരിച്ച് രൂപപ്പെട്ടു വന്നവയാണ് ഈ മീമാംസകളെല്ലാം. ഒരാൾ ഭരണാധികാരിയാവുന്നതെങ്ങനെയെന്ന് തീരുമാനിക്കുന്നതിന് അടിസ്ഥാനത്തിലാണ് അത് ജനാധിപത്യമാണോ രാജ്യാധിപത്യമാണോ എന്നെല്ലാം പറയുന്നത്. തെരഞ്ഞെടുക്കപ്പെടുന്നത് എങ്ങനെയെന്നതിലധികം പ്രധാനം തെരെഞ്ഞെടുക്കപ്പെടുന്നവർ എങ്ങനെ ഭരിക്കണം എന്ന വിഷയമാണെന്ന് പൗരന്മാരെ സംബന്ധിച്ചിടത്തോളം ഏറ്റവും പ്രധാനം. ജനാധിപത്യരീതിയിൽ തെരെഞ്ഞെടുക്കപ്പെട്ടവരെക്കൊണ്ട് പൊരുതി മുട്ടുന്ന സമൂഹങ്ങളെക്കുറിച്ച വാർത്തകൾ ദിനംപ്രതി വായിക്കുന്നവരാണ് നാം. നന്മ നിറഞ്ഞ നല്ല രാജാക്കന്മാരെ നൂറ്റാണ്ടുകൾ കഴിഞ്ഞും ആദരവോടെ സ്മരിക്കുന്ന സമൂഹങ്ങളുമുണ്ട്. ഭരണാധികാരി നന്മയുള്ളവരാവുകയെന്നതാണ് എങ്ങനെ തെരെഞ്ഞെടുക്കപ്പെടുന്നുവെന്നതിനേക്കാൾ പ്രധാനമെന്നർത്ഥം.

ഭരണാധികാരിയെ എങ്ങനെ തെരെഞ്ഞെടുക്കണമെന്ന വിഷയം സാമൂഹികവളർച്ചയ്ക്കനുസരിച്ച് തീരുമാനിക്കാൻ സ്വാതന്ത്ര്യം നൽകിയ മതമാണ് ഇസ്‌ലാം. പ്രവാചകന് ശേഷമുള്ള നാല് ഖലീഫമാരെയും തെരെഞ്ഞെടുത്ത രീതികൾ വ്യത്യസ്തമായത് ആ രംഗത്ത് നിഷ്‌കൃഷ്ടമായ നിയമങ്ങൾ പഠിക്കാത്തത് കൊണ്ടാണ്. പ്രവാചകന്‍(സ)തന്റെ പിന്‍ഗാമിയെ നിശ്ചയിക്കാതെയാണ് ദേഹവിയോഗം ചെയ്തത്. സർക്കാർ രൂപീകരണത്തിന്റെ നിശ്ചിതരൂപം പ്രവാചകന്‍(സ)കാണിച്ചു തന്നിട്ടില്ല. എന്നാല്‍, രാഷ്ട്രത്തിന്റെ മൗലികസ്വഭാവങ്ങളും സവിശേഷതകളുമെന്തെല്ലാമായിരിക്കണമെന്ന് കര്‍മപഥത്തിലൂടെ അദ്ദേഹം കാണിച്ചുതന്നിട്ടുണ്ട്. പ്രസ് തുത സവിശേഷതകളിലൂന്നി സന്ദര്‍ഭത്തിനൊത്ത ഗവണ്‍മെന്റ് രൂപീകരണരീതി സ്വീകരിക്കുവാനുള്ള വിപുലമായ സാധ്യതക്ക് അംഗീകാരം നല്‍കുകയാണ് പ്രവാചകന്‍ ചെയ്തത്. രൂപീകരണത്തിന്റെ രീതിയേക്കാളധികം പ്രധാനം ഗവണ്‍മെന്റ് എങ്ങനെ നീതിനിര്‍വഹിക്കണമെന്നതും പൗരന്മാരും ഗവണ്‍മെ ന്റും തമ്മില്‍ നിലനില്‍ക്കേണ്ട ബന്ധമെന്തായിരിക്കണമെന്നത മാണ്. ഇക്കാര്യങ്ങളിലേക്കാണ് പ്രവാചകന്‍(സ)തന്റെ ജീവിതത്തിലൂടെ പ്രധാനമായി വെളിച്ചം പകര്‍ന്നിരിക്കുന്നത്.

ഇസ് ലാമികഭരണം തന്റെ കുടുംബത്തിന്റെ അവകാശമായി മാറ്റണമെന്ന് പ്രാവാകന്ന് (സ) ഉദ്ദേശമുണ്ടായിരുന്നെങ്കിൽ അദ്ദേഹം അത് ചെയ്യുമായിരുന്നു. ഭരണാധികാരിയെ തെരെഞ്ഞെടുക്കേണ്ടതെങ്ങനെയെന്ന കാര്യം ജനങ്ങളുടെ തീരുമാനത്തിന് വിട്ടുകൊടുക്കുകയാണ് തിരുനബി(സ) ചെയ്തത്. പ്രവാചക വിയോഗത്തിനുശേഷം ഗവണ്‍മെന്റ് രൂപീകരണത്തിന്റെ പ്രശ്‌നം അനുചരന്മാര്‍ കൈകാര്യം ചെയ്തത് അദ്ദേഹം പഠിപ്പിച്ച മൗലികതത്ത്വങ്ങളുടെ അടിസ് ഥാനത്തില്‍ നിന്നുകൊണ്ടായിരുന്നു. മുഹമ്മദ് നബി(സ)യുടെ മരണശേഷം മൃതദേഹം സംസ്‌കരിക്കുന്നതിന് മുമ്പുതന്നെ അനുചരന്മാര്‍ സഖീഫത്തു ബനീസഈദില്‍ ഒത്തുകൂടി. ചര്‍ച്ചകള്‍ക്കു ശേഷം പ്രവാച കാനുചരന്മാരില്‍ പ്രമുഖനായ അബൂബക്കര്‍(റ)എഴുന്നേറ്റുനിന്ന് ഉമര്‍, അബൂ ഉബൈദത്തുല്‍ ജര്‍റാഹ് എന്നിവരുടെ പേരു നിര്‍ദേ ശിച്ചുകൊണ്ട് അവരിലൊരാളെ തെരഞ്ഞെടുക്കുവാനാവശ്യപ്പെട്ടു. ഉടനെ ഉമര്‍(റ)'താങ്കളെക്കാള്‍ അതിന്നര്‍ഹനായി മറ്റാരുമില്ല' എന്നു പറഞ്ഞ് അബൂബക്കറിനോട് കൈനീട്ടുവാന്‍ ആവശ്യപ്പെടുകയും അപ്പോള്‍ തന്നെ അദ്ദേഹത്തിന്റെ കരംഗ്രഹിച്ചുകൊണ്ട് അനുസരണ പ്രതിജ്ഞ (ബൈഅത്ത്) ചെയ്യുകയും ചെയ്തു. അതിന്നുശേ ഷം എല്ലാവരും അതിന്ന് അംഗീകാരം നല്‍കുകയും അബൂബക്കറി(റ)നെ ബൈഅത്ത് ചെയ്യുകയും ചെയ്തു.

രണ്ടാം ഖലീഫയായ ഉമറി(റ)ന നിയമിച്ചത് അബൂബക്കര്‍(റ)ആയിരുന്നു. അദ്ദേഹം തന്റെ പിന്‍ഗാമിയായി ഒരാളെ തെരഞ്ഞെ ടുക്കുവാന്‍ ജനങ്ങളോട് ആവശ്യപ്പെട്ടു. അപ്പോള്‍ പ്രസ്തുത തെര ഞ്ഞെടുപ്പിനുള്ള മുഴുവന്‍ അധികാരവും ജനങ്ങള്‍ അദ്ദേഹത്തിന് തന്നെ നല്‍കി. ഒന്നാം ഖലീഫ അബൂബക്കര്‍(റ)തന്റെ പിന്‍ഗാമി ആരാകണമെന്ന വിഷയത്തെക്കുറിച്ച് അബ്ദുറഹ്മാനുബ്‌നു ഔഫ്, ഉസ്മാനുബ്‌നു അഫ്ഫാന്‍, സഈദുബ്‌നുസൈദ്, ഉസൈദുബ്‌നു ഹുസൈനെ(റ) തുടങ്ങിയ പ്രഗല്‍ഭരുമായി ആലോചിച്ച ശേഷം പ്രസ്തുത സ്ഥാനത്തിന് അര്‍ഹന്‍ ഉമര്‍(റ)തന്നെയാണെന്ന് തീരുമാന ത്തിലെത്തി. ഖലീഫയായി നിയമിക്കപ്പെടയുടന്‍ ഉമർ (റ) ചെയ്ത പൊതുപ്രസംഗത്തില്‍ അദ്ദേഹം പറഞ്ഞു: 'നിങ്ങളുടെമേലുള്ള ഈ അധികാരസ്ഥാനം ഞാനൊരിക്കലും ആഗ്രഹിച്ചിരുന്നതല്ല. പരേതന്റെ മനസ്സില്‍ അല്ലാഹു തോന്നിച്ച ഒരു കാര്യമാണത്. എന്നില്‍ ഭരമേല്‍പിക്കപ്പെട്ട ഈ 'അനാമത്ത്' അനര്‍ഹരായ ആരെയെങ്കിലും ഏല്‍പിക്കണമെന്ന് ഞാന്‍ വിചാരിക്കുന്നില്ല. എന്നാല്‍, മുസ്‌ലിം കളുടെ അന്തസ്സ് ഉയര്‍ത്തുവാന്‍ കഴിയുമെന്ന് പ്രതീക്ഷിക്കുന്ന ആര്‍ക്കും ഈ സ്ഥാനം ഏല്‍പിക്കുവാന്‍ ഞാനൊരുക്കമാണ്. അവരാണ് അതിന്ന് കൂടുതല്‍ അര്‍ഹര്‍.' ഇത് കേട്ട ജനങ്ങള്‍ ഒന്നടങ്കം തങ്ങളി ഷ്ടപ്പെടുന്നത് ഉമറിനെത്തന്നെയാണെന്ന് പ്രഖ്യാപിക്കുകയാണു ണ്ടായത്.

ഒരു ഘാതകന്റെ കുത്തേറ്റ് മരണശയ്യയില്‍ കിടക്കുന്ന ഉമറി(റ)നോട് തന്റെ പിന്‍ഗാമിയെ നിശ്ചയിക്കുവാന്‍ വേണ്ടി മുസ്‌ലിംകള്‍ ആവശ്യപ്പെട്ടു. അദ്ദേഹം ഉസ്മാനുബ്‌നു അഫ്ഫാന്‍, അലിയ്യുബ്‌നു അബീത്വാലിബ്, അബ്ദുര്‍റഹ്മാനുബ്‌നു ഔഫ്, സുബൈറുബ്‌നു അവ്വാം, ത്വല്‍ഹത്തുബ്‌നു ഉബൈദില്ലാഹ്, സഅ്ദുബ്‌നു അബീവ ഖാസ്(റ) എന്നിവരുള്‍ക്കൊള്ളുന്ന ഒരു ആറംഗ സമിതി രൂപീകരിച്ച് അവരില്‍ നിന്ന് ഒരാളെ ഖലീഫയായി തെരഞ്ഞെടുക്കുവാന്‍ പ്രസ്തുത സമിതിയെത്തന്നെ ചുമതലപ്പെടുത്തി. തന്റെ മകനായ അബ്ദുല്ലാഹ്ബ്‌നു ഉമ(റ)ന കൂടിയാലോചനയില്‍ സഹായിക്കുവാനായി സമിതിയിലേക്ക് ക്ഷണിക്കണമെന്ന് നിര്‍ദേശിച്ചതോടൊപ്പം തന്നെ അദ്ദേഹത്തെ തെരഞ്ഞെടുക്കരുതെന്നും ഉമര്‍(റ)നിഷ്‌കര്‍ഷിച്ചിരുന്നു. സമിതി സമ്മേളിച്ചപ്പോള്‍ തന്നെ അബ്ദുറഹ്മാനുബ്‌നു ഔഫ്‌(റ)സ്ഥാനാര്‍ഥിത്വത്തില്‍ നിന്ന് പിന്‍വാങ്ങി. അപ്പോള്‍ ജനങ്ങളുമായി സമ്പര്‍ക്കത്തിലേര്‍പെട്ട് അവര്‍ ആഗ്രഹിക്കുന്ന വ്യക്തിയുടെ പേര് നിര്‍ദേശിക്കനായി സമിതി അദ്ദേഹത്തെ ചുമതലപ്പെടുത്തി. ജനങ്ങളുമായുള്ള സമ്പര്‍ക്കത്തില്‍ നിന്ന് അവര്‍ ഇഷ്ടപ്പെടുന്നത് ഉസ്മാനെയും അലിയെയുമാണെന്നും ഭൂരിപക്ഷം ഉസ്മാനോടൊപ്പമാണെന്നും അദ്ദേഹം മനസ്സിലാക്കി. മുസ്‌ലിംകളെ മുഴുവന്‍ അതി ന്നുശേഷം അദ്ദേഹം മദീനയിലെ മസ്ജിദുന്നബവിയില്‍ വിളിച്ചു കൂട്ടി. 'താങ്കളെ അധികാരമേല്‍പിച്ചാല്‍ അല്ലാഹുവിന്റെ ഗ്രന്ഥവും തിരുനബിയുടെ ചര്യയും പൂര്‍വികരായ രണ്ടു ഖലീഫമാരുടെ ചര്യയും അനുസരിച്ച് ഭരണം നടത്തുമോ?' അലിയെ അഭിസംബോധന ചെയ്തുകൊണ്ട് അബറുഹ്മാനിബ്‌നു ഔഫ് ചോദിച്ചു. 'കഴിവനുസരിച്ച് നടത്തും' എന്നാണ് അലി(റ)മറുപടി നല്‍കിയത്. ഇതേ ചോദ്യം ഉസ്മാ(റ)നോട് ചോദിച്ചപ്പോള്‍ 'അതെ'യെന്ന് അനുബന്ധമൊന്നും കൂടാതെ അദ്ദേഹം മറുപടി നല്‍കി. ഉടനെത്തന്നെ അബ്ദുറഹ്മാനുബ്‌നുഔഫ്‌(റ)ഉസ്മാന്റെ(റ) കരംഗ്രഹിച്ചുകൊണ്ട് അനുസരണ പ്രതിജ്ഞ ചെയ്തു. തുടര്‍ന്ന് മറ്റു മുസ്‌ലിംകളെല്ലാം അദ്ദേഹത്തിന് ബൈഅത്ത് ചെയ്തു.

ഉസ്മാന്‍(റ)വധിക്കപ്പെട്ടപ്പോള്‍ സൈന്യത്തലവന്മാര്‍ വന്ന് അലി(റ)യോട് അധികാരമേറ്റെടുക്കാനാവശ്യപ്പെട്ടു. അലി(റ)അധികാരമേ റ്റെടുക്കുവാന്‍ വിസമ്മതിക്കുകയാണുണ്ടായത്. മദീനയിലെ തലമുതിര്‍ന്ന നേതാക്കള്‍ ചെന്ന് കണ്ട് അധികാരമേറ്റെടുക്കാന്‍ ആവശ്യപ്പെട്ടപ്പോള്‍ പള്ളിയില്‍വെച്ച് പൊതുബൈഅത്ത് നടക്കുകയാണെങ്കില്‍ താന്‍ അധികാരമേറ്റെടുക്കാമെന്ന് അദ്ദേഹം സമ്മതിച്ചു. അങ്ങനെ മദീനയിലെ മസ്ജിദുന്നബവിയില്‍ മുസ്‌ലിം ബഹുജനങ്ങള്‍ സമ്മേളിച്ചു ബൈഅത്ത് ചെയ്ത ശേഷമാണ് അലി(റ)അധികാരമേറ്റെടുത്തത്.

സച്ചരിതരായിരുന്ന നാലു ഖലീഫമാരുടെ തെരഞ്ഞെടുപ്പ് രീതികള്‍ വ്യത്യസ്തമായിരുന്നുവെങ്കിലും അവയിലെല്ലാം പൊതുജനാഭിപ്രായത്തിന് അര്‍ഹമായ സ്ഥാനം നല്‍കപ്പെട്ടിരുന്നുവെന്ന് മുകളില്‍ വിവരിച്ച ചരിത്രത്തില്‍ നിന്ന് വ്യക്തമാണ്. ഇസ്‌ലാമിന്റെ അടിസ്ഥാനതത്ത്വങ്ങളില്‍ നിന്ന് വ്യതിചലിക്കാതെ സമയാനുസൃതമായ തെരഞ്ഞെടുപ്പുരീതി സ്വീകരിക്കുവാന്‍ ഇസ്‌ലാം സ്വാതന്ത്ര്യം നല്‍കിയിരിക്കുന്നുവെന്ന് സാരം. ഏത് തരത്തിൽ തെരെഞ്ഞെടുക്കപ്പെട്ടവരാണെങ്കിലും അവർ ഖുർആനും സുന്നത്തുമനുസരിച്ച് സത്യസന്ധവും നീതിനിഷ്ഠവുമായി ഭരണം നടത്തണമെന്നാണ് ഇസ്‌ലാമിന്റെ അനുശാസന.

രണ്ട് രീതിയിലാണ് ഒരു രാജ്യം ഇസ്‌ലാമികമായിത്തീരുന്നത്. അവിടെ ജീവിക്കുന്നവരിലെ മനഃപരിവർത്തനത്തിലൂടെയാണ് ഒന്നാമത്തേത്. മദീനാരാജ്യമുണ്ടായത് അങ്ങനെയാണ്. ഇസ്‌ലാമികരാഷ്ട്രത്തിന്റെ വികാസത്തിന്റെ ഭാഗമായി അയാൾ നാടുകൾ ഇസ്‌ലാമികമായിത്തത്തീരുന്നതാണ് രണ്ടാമത്തേത്. പ്രവാചകാനുചരന്മാരുടെ കാലത്ത് പലരാജ്യങ്ങളും ഇസ്‌ലാമികസാമ്രാജ്യത്തിന്റെ ഭാഗമായത് ഇങ്ങനെയാണ്. രണ്ടായിരുന്നാലും അന്ന് നിലവിലുള്ള രാഷ്ട്രനൈതികത അംഗീകരിച്ചവയാണവ. അട്ടിമറിയിലൂടെയോ തീവ്ര വാദപ്രവര്‍ത്തനങ്ങളിലൂടെയോ നാടിനെ ഇസ്‌ലാമികമാക്കുവാൻ കല്പിക്കുന്ന പ്രവാചകവചനങ്ങളൊന്നും തന്നെയില്ല. അട്ടിമറിയിലൂടെ ഇസ്‌ലാ മികരാഷ്ട്രം സ്ഥാപിക്കണമെന്ന് പ്രവാചകന്(സ) വിചാരമുണ്ടായി രുന്നുവെങ്കില്‍ അദ്ദേഹത്തിന്റെ പ്രവാചക ജീവിതത്തിന്റെ ആദ്യ ഘട്ടത്തില്‍ തന്നെ അത് സാധ്യമാകുമായിരുന്നു. അദ്ദേഹത്തിനടുത്ത് ക്വുറൈശീ പ്രതിനിധിയായ ഉത്ബത്തുബ്‌നു റബീഅ വന്ന് തന്റെ പ്രബോധനപ്രവര്‍ത്തനങ്ങള്‍ നിറുത്തിവെച്ചാല്‍ പണമോ പ്രതാപ മോ തരുണികളോ അധികാരമോ എന്താണ് ആവശ്യമെന്ന് വെച്ചാല്‍ അത് നല്‍കാമെന്ന് പറഞ്ഞപ്പോള്‍ ഒരു ചെറിയ അട്ടിമറി നടത്തിയാ ല്‍ മതിയായിരുന്നു. പ്രവാചകനിയോഗത്തിന്റെ ലക്ഷ്യം പക്ഷേ, അട്ടിമറിയിലൂടെ ഇസ്‌ലാമികരാഷ്ട്രം നിര്‍മിക്കുകയായിരുന്നില്ല. പ്ര ത്യുത, ജനങ്ങളെ സംസ്‌കരിക്കുകയും സംസ്‌കൃതമായ ഒരു സമൂഹ ത്തെ സൃഷ്ടിച്ച് ലോകത്തിന് മാതൃകയാവുകയുമായിരുന്നു. ഒരു ഇസ്‌ലാമികരാഷ്ട്രം നിര്‍മിച്ച് അതിന്റെ സാരഥിയാവുകയല്ല പ്രവാചകന്‍ ചെയ്തത്; മറിച്ച് ഒരു രാഷ്ട്രത്തിലെത്തിച്ചേരുവാനാ വശ്യമായ സാമൂഹ്യവിപ്ലവത്തിന് പാതയൊരുക്കുകയായിരുന്നു.

ഒരു രാഷ്ട്രത്തിലെ നിയമങ്ങളും അവിടുത്തെ പൗരന്മാരും തമ്മി ലുള്ള ബന്ധം രാഷ്ട്രമീമാംസയിലെ പ്രധാനപ്പെട്ട വിഷയങ്ങളിലൊ ന്നാണ്. ജനാധിപത്യ രാഷ്ട്രത്തില്‍ നിയമനിര്‍മാണം നടത്തുന്നത് ജനപ്രതിനിധി സഭയാണെങ്കിലും പൗരന് തന്നില്‍ അടിച്ചേല്‍പിക്കപ്പെടുന്നവയായിട്ടാണ് നിയമങ്ങള്‍ അനുഭവപ്പെടുന്നതെന്നാണ് സാമൂഹികശാസ്ത്രജ്ഞന്മാരുടെ പക്ഷം. ഉപഭോഗസംസ്‌കാരം നിലനില്‍ക്കുന്ന സമൂഹത്തില്‍ ഈ വികാരം അല്‍പം തീഷ്ണമാ യിരിക്കും. തന്റെ സുഖിക്കുവാനുള്ള അവകാശത്തിന്മേല്‍ രാഷ്ട്രം അടിച്ചേല്‍പിക്കുന്ന വിലക്കുകളായിട്ടാണ് നിയമങ്ങള്‍ അത്തരം സമൂഹങ്ങളിലെ പൗരന്ന് അനുഭവപ്പെടുക. അതുകൊണ്ടുതന്നെ നിയമത്തിന്റെ കുരുക്കുകളില്‍ നിന്ന് രക്ഷപ്പെടാന്‍ അവന്‍ പരമാ വധി ശ്രമിക്കും. അഴിമതിയും വഞ്ചനയും ജനാധിപത്യത്തിന്റെ ഉ പോല്‍പന്നമായി മാറുന്നത് അതുകൊണ്ടാണ്. മനംമാറ്റത്തിലാണ്

ഇസ്‌ലാമികരാഷ്ട്ര ത്തിലാകട്ടെ, താന്‍ അനുസരിക്കുന്നത് ദൈവികനിയമങ്ങളാണെന്ന ധാരണയുള്ളതിനാല്‍ നിയമലംഘനങ്ങള്‍ കുറവായിരിക്കും. തന്റെ ഭൗതികവും പാരത്രികവുമായ മോക്ഷത്തിന് അനിവാര്യമായ നിയമ ങ്ങളാണ് താന്‍ അനുസരിക്കുന്നതെന്ന ധാരണ പൗരനെ രാഷ്ട്ര ത്തോടു കൂറുള്ളവനാക്കിത്തീര്‍ക്കും. സര്‍വോപരിയായി നിയമപാല കരുടെ കണ്ണുവെട്ടിച്ചാലും നിയമദാതാവായ പടച്ചതമ്പുരാന്റെ കണ്ണു വെട്ടിക്കാന്‍ സാധ്യമല്ലെന്ന വിശ്വാസം അവനെ ഇസ്‌ലാമിക രാഷ് ട്രത്തിലെ നിയമങ്ങള്‍ പരസ്യമായും രഹസ്യമായും അനുസരി ക്കുന്നവനാക്കിത്തീര്‍ക്കും. അതാണ് ഒരു ദുര്‍ബലനിമിഷത്തില്‍ വ്യഭിചരിച്ചുപോയ ഇസ്‌ലാമിക രാഷ്ട്രത്തിലെ പൗരനെ സർക്കാരിന് മുമ്പില്‍ പ്രത്യക്ഷപ്പെട്ട് താന്‍ ചെയ്ത തെറ്റ് ഏറ്റുപറഞ്ഞ് ശിക്ഷ ശിരസാവഹിക്കുന്നവനാക്കിത്തീര്‍ത്തത്. രഹസ്യമായുള്ള നിയമ ലംഘനത്തില്‍ നിന്നുപോലും രാഷ്ട്രത്തിലെ പൗരന്മാരെ തടയുവാന്‍ തക്കവണ്ണമുള്ള രാഷ്ട്രസംവിധാനം സൃഷ്ടിക്കുവാന്‍ ഇസ്‌ലാ മിന്നല്ലാതെ മറ്റേത് തത്ത്വശാസ്ത്രത്തിനാണ് കഴിഞ്ഞിട്ടുള്ളത്? ഭരണമാറ്റം വഴി ഉണ്ടാകാനാവുന്നതല്ല ഈ പരിവർത്തനം; മനംമാറ്റത്തിലാണ് ഇസ്‌ലാം ഒന്നാമതായി ശ്രദ്ധിക്കുന്നത് എന്ന് പറയുന്നത് അത് കൊണ്ടാണ്.

രാഷ്ട്രത്തീയത്തെക്കുറിച്ച ഇസ്‌ലാമിന്റെ വീക്ഷണം അതിന്റെ സമൂഹസങ്കല്‍പത്തിന്റെ ഭാഗമാണ്. രാഷ്ട്രമില്ലെങ്കില്‍ ഇസ്‌ലാമിന്ന് തന്നെ നിലനില്‍പില്ലെന്ന വാദം അടിസ്ഥാനരഹിതമാണ്. അതേ പോലെത്തന്നെയാണ് രാഷ്ട്രീയത്തില്‍ ഇസ്‌ലാമികനിയമങ്ങള്‍ ക്കൊന്നും പ്രസക്തിയില്ലെന്ന വാദവും. ഇതു രണ്ടും രണ്ട് ആത്യന്തി കതകളാണ്. ഇവ്വിഷയകമായ ഇസ്‌ലാമിക വീക്ഷണമാകട്ടെ ഈ ആത്യന്തതകള്‍ക്ക് മധ്യെയാണ് താനും.

ഒരു ഇസ്‌ലാമികേതര രാഷ്ട്രത്തില്‍ ജീവിക്കുന്ന മുസ്‌ലിമിനെ സംബന്ധിച്ചിടത്തോളം അവന്ന് പാലിക്കുവാന്‍ കഴിയുന്ന രാഷ്ട്രീ യ നിയമങ്ങള്‍ ഇസ്‌ലാം പഠിപ്പിക്കുന്നുണ്ട്. രാഷ്ട്രം ഇസ്‌ലാമിനു വിരുദ്ധമാകാത്തിടത്തോളം തന്റെ രാജ്യത്തിന്റെ പുരോഗതിക്കു വേണ്ടി പ്രവര്‍ത്തിക്കുന്നതോ രാജ്യകാര്യങ്ങളില്‍ പങ്കുവഹിക്കുന്ന തോ മുസ്‌ലിമിന് നിഷിദ്ധമല്ല. മഹാനായ യൂസുഫ് നബിൗയുടെ ചരിത്രം വിവരിക്കുന്നിടത്ത് ക്വുര്‍ആന്‍ ഇക്കാര്യത്തിലേക്കാണ് വെളി ച്ചം വീശുന്നത്. ഇസ്‌ലാംമതം സ്വീകരിച്ചിട്ടില്ലാതിരുന്ന ഒരു രാജാവി നു കീഴില്‍ മന്ത്രിപദം വഹിച്ച ആ മഹാനായ പ്രവാചകന്‍ നീതിപാ ലിക്കുകയും രാഷ്ട്രസേവനം നിര്‍വഹിക്കുകയും ചെയ്തുവെന്ന് ക്വുര്‍ആനിലെ ചരിത്രകഥനം വ്യക്തമാക്കുന്നു. ഇസ്‌ലാമിനു വിരു ദ്ധമല്ലാത്ത അടിസ്ഥാനാദര്‍ശങ്ങളിന്‍മേല്‍ സ്ഥാപിതമായ ഒരു രാ ഷ്ട്രത്തിന്റെ കുഞ്ചികസ്ഥാനങ്ങള്‍ വഹിക്കുമ്പോള്‍ മുസ്‌ലിംകള്‍ എങ്ങനെയായിരിക്കണമെന്ന് പ്രസ്തുത കഥനത്തില്‍ നിന്ന് മനസ്സി ലാക്കാന്‍ കഴിയും.

മുഹമ്മദ് നബിയുടെ മക്കാ കാലഘട്ടത്തിലെ നടപടികളും ഇസ്‌ലാമികമല്ലാത്ത ഭരണസംവിധാനങ്ങൾക്ക് കീഴിൽ എങ്ങനെ മുസ്ലിംകൾ വർത്തിക്കണമെന്ന് പഠിപ്പിക്കുന്നുണ്ട്. നാടിനോട് കലാപമുണ്ടാക്കാതെ, നിലനിൽക്കുന്ന സാഹചര്യങ്ങളെ പരമാവധി പ്രയോജനപ്പെടുത്തിക്കൊണ്ട് മതജീവിതം നയിക്കുകയും മതപ്രബോധനത്തിലേർപ്പടുകയുമാണ് നബി (സ) ചെയ്തത്. മുസ്ലിംകൾ അല്ലാത്തവരുടെ സഹായം തെറ്റുകയും നിലവിലുള്ള വ്യവസ്ഥയുടെ സാധ്യതകൾ ഉപയോഗപ്പെടുത്തുകയും ചെയ്ത പ്രവാചകന്റെ മക്കാജീവിതനടപടികൾ ഇസ്‌ലാമികേതരമായ രാഷ്ടസംവിധാനങ്ങളോട് എത്രത്തോളം മുസ്ലിംകൾക്ക് സഹകരിക്കാനാവുമെന്ന് വ്യക്തമാക്കുന്നുണ്ട്.

മത-ധാര്‍മികമൂല്യങ്ങളില്‍ നിന്ന് വ്യതിചലിക്കാതെ, ഒരു ഇസ്‌ലാ മികേതര രാഷ്ട്രത്തിലെ രാഷ്ട്രീയ പ്രവര്‍ത്തനങ്ങളില്‍ പങ്കാളിയാ വുന്നതില്‍ നിന്ന് സത്യവിശ്വാസികളെ തടയുന്നതിന്ന് വിശുദ്ധ ക്വുര്‍ആനിന്റെയോ തിരുനബിചര്യയുടെയോ യാതൊരു പിന്‍ബലവു മില്ല. ഒരു മുസ്‌ലിം ജീവിത്തില്‍ എല്ലാ രംഗങ്ങളിലും ഇസ്‌ലാമികമാ യ വിധിവിലക്കുകള്‍ പാലിക്കാന്‍ ബാധ്യസ്ഥനായതുപോലെ രാഷ് ട്രീയത്തിലും പ്രസ്തുത വിധിവിലക്കുകള്‍ പാലിക്കുവാന്‍ ബാധ്യസ് ഥനാണ്. രാഷ്ട്രീയത്തില്‍ മതധാര്‍മികമൂല്യങ്ങള്‍ക്ക് യാതൊരു വിലയുമില്ലെന്ന വീക്ഷണം ഇസ്‌ലാമിന്ന് അന്യമാണ്. 'ദൈവത്തിനു ള്ളത് ദൈവത്തിനും സീസര്‍ക്കുള്ളത് സീസര്‍ക്കും' എന്ന ചര്‍ച്ചി ന്റെ വിഭജനരീതിയും ഇസ്‌ലാം അംഗീകരിക്കുന്നില്ല. ഭരണരംഗ ത്തുള്ളവര്‍ ദൈവികമായ വിധിവിലക്കുകള്‍ പാലിക്കണമെന്ന് ശക് തമായി നിഷ്‌കര്‍ഷിക്കുന്ന മതമാണ് ഇസ്‌ലാം. അഴിമതിയും സ്വജനപക്ഷപാതിത്വവും വഞ്ചനയും ചൂഷണവുമൊന്നും രാഷ്ട്രീയ പ്രവര്‍ത്തനത്തിന്റെ ഭാഗമായി ഒരു മുസ്‌ലിം സ്വീകരിക്കുവാന്‍ പാടി ല്ലാത്തതാണ്.

പ്രവാചകാനുചരന്‍ സഅദ്ബ്‌നു അബീവക്വാസ് ഒരു അമുസ്ലിമിനെ ഒട്ടകത്തിന്റെ താടിയെല്ലുകൊണ്ട് തലക്കടിച്ചുകൊന്ന സംഭവം മക്കയിൽ വെച്ചും മുസ്ലിംകൾ ആയുധമെടുത്തുവെന്നും അവർ കലാപകാരികളായിരുന്നുവെന്നുമല്ലേ വ്യക്തമാക്കുന്നത് ?

അദ്ബ്‌നു അബീവക്വാസ് മക്കയില്‍വെച്ച് ബഹുദൈവാരാധകനായ ഒരാളെ ഒട്ടകത്തിന്റെ താടിയെല്ലുകൊണ്ട് തലക്കടിച്ചുകൊന്ന സംഭവം മുസ്ലിംകൾ എന്നും കലാപകാരികളാണെന്നാണ് വ്യക്തമാക്കുന്നതെന്നും മക്കയിലെ സംഘര്‍ഷങ്ങളുടെ സ്വഭാവം എന്തെന്ന് മനസ്സിലാക്കാൻ ഈ സംഭവം പര്യാപ്തമാണെന്നും വാദിച്ചുകൊണ്ട് മക്കയിലെ മുസ്‌ലിംകള്‍ കലാപശ്രമങ്ങള്‍ നടത്തി എന്നു സമര്‍ത്ഥിക്കുവാന്‍ വേണ്ടി മിഷനറിമാരും ഓറിയന്റലിസ്റ്റുകളും ശ്രമിക്കാറുണ്ട്

സ്വന്തം നാടായ മക്കയില്‍ ശരിയെന്നു വിശ്വസിക്കുന്ന ആദര്‍ശം പ്രബോധനം ചെയ്തതിന്റെ പേരില്‍ തനിക്കും അനുയായികള്‍ക്കും പതിമൂന്നു വര്‍ഷം കടുത്ത മര്‍ദ്ദനങ്ങളേല്‍ക്കേണ്ടി വന്നിട്ടും മക്കയുടെ രാഷ്ട്രീയ നിയന്ത്രണം കൈക്കലാക്കാനോ എതിരാളികളെ വകവരുത്തുവാനോ വേണ്ടിയുള്ള സായുധ ഓപ്പറേഷനുകള്‍ക്കായുള്ള ഒരു ഗറില്ലാ പദ്ധതിയും ഒരിക്കല്‍ പോലും ആസൂത്രണം ചെയ്യാതെ ഗോത്രാധിപത്യ മക്കയില്‍ ക്ഷമാപൂര്‍വം നിയമവാഴ്ചക്കുള്ളില്‍ മാതൃകാപരമായി നിലനില്‍ക്കുകയാണ് നബി (സ) ചെയ്തതെന്ന യാഥാര്‍ത്ഥ്യം ഇസ്ലാംവിമര്‍ശകരുടെ പുസ്തങ്ങള്‍ തന്നെ വ്യക്തമാക്കുന്നുണ്ട് എന്നതാണ് യാഥാര്‍ത്ഥ്യം. പതിമൂന്നു വര്‍ഷക്കാലം ചരിത്രത്തിന്റെ വെള്ളിവെളിച്ചത്തില്‍. സംഭവബഹുലമായി നിലനിന്ന മക്കയിലെ നബിജീവിതത്തില്‍ നിന്ന് കലാപം കുഴിച്ചെടുക്കാനുള്ള സിദ്ധവൈഭവമുള്ള ഗവേഷകബുദ്ധികളുടെ അധ്വാനങ്ങള്‍ കേവലം ഒരു ‘ഒട്ടകയെല്ലില്‍’ തടഞ്ഞുനില്‍ക്കുന്നതുതന്നെ നബി(സ)യില്‍ ഗറില്ലാ പോരാളികള്‍ക്ക് പൂര്‍വമാതൃക കണ്ടെത്താനുള്ള എല്ലാ പരിശ്രമങ്ങളും അടിസ്ഥാനരഹിതമാണെന്നാണ് വ്യക്തമാക്കുന്നത്.

മതസ്വാതന്ത്ര്യവും പൗരാവകാശങ്ങളും നിഷേധിക്കപ്പെട്ട് പീഡനപര്‍വത്തില്‍ കിടന്നുപിടഞ്ഞ് പ്രക്ഷുബ്ധമായ ഒരു ജനസമൂഹമാണ് നബിനേതൃത്വത്തില്‍ സായുധമായ പ്രതികരണങ്ങള്‍ക്കും പ്രതികാരങ്ങള്‍ക്കുമുള്ള ഒരു ശ്രമവും നടത്താതെ മക്കയില്‍ ഒന്നര പതിറ്റാണ്ടിനടുത്ത് ശാന്തജീവിതം നയിച്ചത് എന്ന ചരിത്രവസ്തുത, ഇവ്വിഷയകമായ മുഴുവന്‍ വിമര്‍ശകഭാവനകളെയും കടപുഴക്കിയെറിയാന്‍ പോന്നതാണ്. സഹജമായ പ്രതികരണശേഷിയില്‍ നിന്നുടലെടുത്തേക്കാവുന്ന കലാപത്തിന്റെ ആശയങ്ങളൊന്നും അവരില്‍നിന്ന് നിര്‍ഗളിക്കാതിരുന്നത് പ്രവാചകന്‍ (സ) ബോധപൂര്‍വം അതിനെതിരായ ഉല്‍ബോധനങ്ങള്‍ നല്‍കിയതുകൊണ്ടാണെന്ന കാര്യം വ്യക്തമാണ്. പീഡനങ്ങളും പ്രയാസങ്ങളുമെല്ലാമുണ്ടായപ്പോള്‍ പോലും കലാപം പരിഹാരമല്ലെന്ന് മതപരമായിത്തന്നെ വിശ്വസിക്കുകയും പഠിപ്പിക്കുകയും ചെയ്ത ഒരു സമാധാനവാദിയെ ഭീകരവാദിയുടെ കുപ്പായമണിയിക്കുവാന്‍ നുരുമ്പിച്ച ഒരൊട്ടകയെല്ല് മതിയാകുമെന്നു കരുതുന്നവര്‍ വിഡ്ഡികളുടെ സ്വര്‍ഗത്തിലാണെന്ന് പറയാതിരിക്കാന്‍ യാതൊരു നിര്‍വാഹവുമില്ല തന്നെ.

വേണമെങ്കില്‍ പ്രതികാരസംഘങ്ങള്‍ക്ക് നിഷ്പ്രയാസം നേതൃത്വം കൊടുക്കുവാന്‍ കഴിയുമായിരുന്ന ഹംസയും ഉമറും കൂടെവന്നിട്ടും കലാപത്തിന്റെ വഴി ചെറുതായിപ്പോലും പുണരാതിരുന്ന നബിമാതൃക, അച്ചടക്കമുള്ള പൗരജീവിതം അദ്ദേഹത്തിന് ശക്തി കൈവരുന്നതിനനുസരിച്ച് കയ്യൊഴിക്കാനുള്ള ഒരു ‘സൗകര്യ’മായിരുന്നില്ല. മറിച്ച് കര്‍ക്കശമായ ഒരു മൂല്യവും നിലപാടുമായിരുന്നുവെന്ന് അസന്നിഗ്ധമായി വ്യക്തമാക്കുന്നുണ്ട്. വധഭീഷണികള്‍ ശക്തിപ്രാപിച്ച് പ്രവാചകനെ വിട്ടുകൊടുക്കാന്‍ സന്നദ്ധമാകാത്തതിന്റെ പേരില്‍ അദ്ദേഹത്തിന്റെ കുടുംബത്തിന് മൂന്നുവര്‍ഷത്തോളം പച്ചിലകള്‍ മാത്രം ഭക്ഷിച്ച് സാമൂഹികമായ ഭ്രഷ്ട് അനുഭവിക്കേണ്ടി വന്നപ്പോഴും അബൂത്വാലിബിന്റെ മരണത്തെത്തുടര്‍ന്ന് ഭീഷണികള്‍ക്ക് ഊക്ക് വര്‍ദ്ധിച്ചപ്പോഴും പ്രതികാരത്തിനുള്ള യാതൊരു രഹസ്യ/പരസ്യ നീക്കവുമില്ലാതെ ശാന്തനായി നിലനില്‍ക്കുകയാണ് നബി (സ) ചെയ്തത്. നബി(സ)യെ കലാപകാരിയാക്കി അവതരിപ്പിക്കാനാഗ്രഹമുള്ളവര്‍ മഷിയിട്ടു തിരഞ്ഞിട്ടും അവിടുത്തെ ജീവിതത്തില്‍ നിന്ന് കലാപത്തിന്റെ ചെറിയൊരു പൊട്ടുപോലും അവര്‍ക്ക് കണ്ടെടുക്കാന്‍ കഴിയാത്തതും അതുകൊണ്ടുതന്നെയാണ്.

കലാപം നബിചര്യയാണെന്ന ആരോപണത്തെ ചരിത്രം പൂര്‍ണമായി കരിച്ചുകളയുന്നുവെന്ന് മനസ്സിലാക്കിയ മിഷനറിമാര്‍, അവസാനത്തെ അവലംബം എന്ന നിലയിലാണ് സഅ്ദ്ബ്‌നു അബീ വക്വാസിനെക്കുറിച്ചുള്ള പരാമൃഷ്ട കഥയുദ്ധരിക്കുന്നത്. ഇബ്‌നു ഇസ്ഹാക്വ് തന്റെ സീറതുറസൂലില്ലയില്‍ സഅദ്ബ്‌നു അബീവക്വാസ് മക്കയില്‍വെച്ച് ഒരു അവിശ്വാസിയെ ഒട്ടകത്തിന്റെ താടിയെല്ലുകൊണ്ട് പ്രഹരിച്ചതായി പറയുന്നുണ്ട്. സത്യവും അസത്യവുമെല്ലാം കൂടിക്കുഴഞ്ഞു കിടക്കുന്ന ഒരു ചരിത്രവിവരണ ഗ്രന്ഥമായതുകൊണ്ടു തന്നെ, ഇബ്‌നു ഇസ്ഹാക്വിന്റെ സീറയില്‍ ഉള്ളതുകൊണ്ടുമാത്രം ഒരു സംഭവം വസ്തുതാപരമാകണമെന്നില്ല. സംഭവം ഉദ്ധരിക്കപ്പെട്ടിരിക്കുന്ന നിവേദക പരമ്പരയെ നിദാനശാസ്ത്രനിയമങ്ങള്‍ വെച്ചപഗ്രഥിച്ചാല്‍ മാത്രമേ മിഷനറിമാര്‍ ചൂണ്ടിക്കാണിക്കുന്ന കഥയുടെ ചരിത്രപരത നിര്‍ണയിക്കാനാകൂ. അതെന്തായിരുന്നാലും, കഥയിലേക്കുവരാം. കഥ ആധികാരികമാണോ അല്ലേ എന്ന ചര്‍ച്ചയ്ക്കപ്പുറത്ത് ആധികാരികമാണെങ്കില്‍തന്നെ മിഷനറിമാരുടെ വാദത്തിനനുകൂലമായ യാതൊന്നും പ്രസ്തുത കഥയിലില്ലെന്ന് സീറതു റസൂലില്ല വായിക്കുന്ന ആര്‍ക്കും ബോധ്യമാകും.

മക്കയിലെ അവിശ്വാസികളില്‍ പലരും പ്രവാചകനെയും ശിഷ്യന്‍മാരെയും കഠിനമായി എതിര്‍ക്കുകയും തരം കിട്ടിയാല്‍ മര്‍ദ്ദിക്കുകയും ചെയ്തിരുന്നതുകൊണ്ടുതന്നെ ജനങ്ങള്‍ അധികം ശ്രദ്ധിക്കാത്ത താഴ്‌വരകളില്‍ പോയാണ് നബിശിഷ്യന്‍മാരില്‍ പലരും മതം അനുശാസിക്കുന്ന പ്രാര്‍ത്ഥനകള്‍ നിര്‍വഹിച്ചിരുന്നത് എന്നു പറഞ്ഞുകൊണ്ടാണ് ഇബ്‌നു ഇസ്ഹാക്വ് ഈ സംഭവത്തിന്റെ വിവരണം ആരംഭിക്കുന്നത്. ഇങ്ങനെ ഒരു താഴ്‌വരയില്‍ സഅദ്ബ്‌നു അബീ വക്വാസ് (റ) ഉള്‍പ്പെടെയുള്ള ഒരു സംഘം പ്രവാചകാനുചരന്‍മാര്‍ പ്രാര്‍ത്ഥന നിര്‍വഹിച്ചുകൊണ്ടിരിക്കെ അതുവഴി കടന്നുവന്ന ബഹുദൈവാരാധകരുടെ ഒരു സംഘം തികഞ്ഞ ധാര്‍ഷ്ട്യത്തോടുകൂടി വളരെ പരുഷമായി പ്രാര്‍ത്ഥന തടസ്സപ്പെടുത്തുകയും വിശ്വാസികളെ ആക്ഷേപിക്കുകയും ചെയ്തുവെന്നും അത് വാക്കുതര്‍ക്കവും സംഘര്‍ഷവും ഏറ്റമുട്ടലും ഉണ്ടാക്കിയെന്നുമാണ് ഇബ്‌നു ഇസ്ഹാക്വ് പറയുന്നത്. ശാന്തമായി പ്രാര്‍ത്ഥന നിര്‍വഹിച്ചുകൊണ്ടിരിക്കുന്നവരെ കയ്യേറാന്‍ വന്ന സംഘവുമായുണ്ടായ ഉന്തിലും തള്ളിലും കയ്യില്‍ കിട്ടിയ ഒരൊട്ടകയെല്ല് സഅദ്ബ്‌നു അബീ വക്വാസ് ഒരു ശത്രുവിനുനേരെ പ്രയോഗിച്ചുവെന്നും അത് അയാളുടെ രക്തം ചിന്തിയെന്നുമാണ് നിവേദനം. (Alfred, Guillaume. The Life of Muhammad – A Translation of Ibn Ishaq’s Sirat Rasul Allah).

ഇബ്‌നു ഇസ്ഹാക്വിന്റെ ഈ വിവരണത്തെ സ്വീകരിച്ചിട്ടുള്ള ചില പണ്ഡിതന്‍മാര്‍ ഈ സംഭവത്തിനുശേഷം ഇത്തരം സംഘര്‍ഷങ്ങള്‍ ആവര്‍ത്തിക്കാതിരിക്കുവാന്‍ വേണ്ടിയാണ് വിദൂരവും വിജനവുമായ താഴ്‌വരകളുപേക്ഷിച്ച് കഅ്ബയ്ക്ക് തൊട്ടരുകില്‍ തന്നെയായിരുന്ന അര്‍ക്വമിന്റെ വീട്ടില്‍ (ദാറുല്‍ അര്‍ക്വം) ആളുകള്‍ ശ്രദ്ധിക്കാത്തവിധം പ്രാര്‍ത്ഥനക്കും പഠനത്തിനും ഒത്തുചേരാന്‍ നബി(സ)യും അനുയായികളും തീരുമാനിച്ചതെന്ന് അഭിപ്രായപ്പെട്ടിട്ടുണ്ട്. (അലി ബ്ന്‍ ബുര്‍ഹാനുദ്ദീന്‍ അല്‍ഹലബി, അസ്സീറത്തുല്‍ ഹലബിയ്യ, 1/456). ഇത് ശരിയാണെങ്കില്‍, എന്താണ് താഴ്‌വരാ സംഭവത്തിലുള്ളത്? നാം പരിശോധിക്കുക. പ്രകോപനങ്ങള്‍ ഒഴിവാക്കാനും മര്‍ദ്ദനത്തില്‍ നിന്ന് രക്ഷനേടാനും വേണ്ടി നഗരമധ്യത്തില്‍ നിന്നുമാറി ആളുകളില്ലാത്ത ഒരു താഴ്‌വരയില്‍ പോയി ശാന്തമായി പ്രാര്‍ത്ഥന നിര്‍വഹിക്കുന്ന വിശ്വാസികളുടെ ഒരു സംഘത്തെ അതുവഴി കടന്നുപോയ അവരുടെ തന്നെ നാട്ടുകാരായ ചിലര്‍ ഒരു കാരണവുമില്ലാതെ കടന്നാക്രമിക്കുന്നു. കയ്യില്‍ കിട്ടിയ വസ്തുക്കളുപയോഗിച്ച് കയ്യേറ്റക്കാരെ തുരത്താന്‍ വിശ്വാസികള്‍ ശ്രമിക്കുന്നു. അതിനിടക്ക് സ്വാഭാവികമായി സംഭവിച്ചതാണ് സഅദ്ബ്‌നു അബീ വക്വാസിന്റെ പ്രഹരം.

ശാന്തമായി നാം കടന്നുപോകുമ്പോള്‍ നമ്മെ കടന്നാക്രമിക്കുവാന്‍ വരുന്ന ഒരാളെ തുരത്താന്‍വേണ്ടി സാധ്യമായ എല്ലാ മാര്‍ഗങ്ങളും ഉപയോഗിക്കുന്നതിന്റെ പേരാണോ ‘കലാപം’? ആണെന്ന് വിവരമുള്ള ഒരാളും പറയില്ല. താഴ്‌വരയില്‍ വിശ്വാസികള്‍ പോയത് ആയുധപരിശീലനം നടത്താനോ അട്ടിമറി പ്രവര്‍ത്തനങ്ങള്‍ ആസൂത്രണം ചെയ്യാനോ സായുധ പ്രതികരണങ്ങള്‍ക്ക് പദ്ധതി തയ്യാറാക്കാനോ ആയിരുന്നുവെങ്കില്‍ അത് കലാപത്തിനുള്ള ശ്രമമായിരുന്നുവെന്ന് പറയാം. അങ്ങനെയാണെന്ന് മിഷനറിമാര്‍ക്കുപോലും വാദമില്ല. സ്വയം പ്രതിരോധത്തിന് ഒട്ടകത്തിന്റെ താടിയെല്ലു വരെയെടുക്കേണ്ടി വന്നതില്‍ നിന്ന് യാതൊരുവിധ ആയുധസന്നാഹങ്ങളും സംഘത്തിന്റെ കയ്യിലുണ്ടായിരുന്നില്ലെന്ന് വ്യക്തമാകുന്നുണ്ട്. ഏതെങ്കിലും മക്കക്കാരനെ പ്രതികാരമെന്ന നിലയിലോ അല്ലാതെയോ തിരഞ്ഞുചെന്നോ കണ്ടിടത്തുവെച്ചോ ആക്രമിച്ചതായിരുന്നുവെങ്കിലും അതിനെ മിഷനറിമാര്‍ സ്ഥാപിക്കാന്‍ ശ്രമിക്കുന്ന വകുപ്പുകളില്‍ ഉള്‍പ്പെടുത്താം. എന്നാല്‍ പരാമര്‍ശിക്കപ്പെടുന്ന കഥയിലുള്ളത് ഇതൊന്നും തന്നെയല്ല. ഇങ്ങോട്ടുവന്ന് ആക്രമിച്ച ഒരാളെ അതുണ്ടാക്കിയ ഏറ്റുമുട്ടലിനിടെ സ്വാഭാവികമായി തിരിച്ചടിച്ചതിനെക്കുറിച്ചാണ് നിവേദനം. ഏതു കോടതിയും പ്രകൃതിപരമായ പ്രതിരോധ നടപടിയെന്ന നിലയില്‍ നിയമവിധേയമായി അംഗീകരിക്കുന്ന ആത്മരക്ഷാ പ്രതികരണം മാത്രമാണത്. അതിനെ ‘കലാപ’മെന്നു പറയാന്‍ മിഷനറിമാര്‍ക്കു മാത്രമേ കഴിയൂ!

പ്രവാചകന്റെ (സ) പതിമൂന്നുവര്‍ഷത്തെ മക്കാജീവിതം പണിപ്പെട്ടു തിരഞ്ഞിട്ട് മിഷനറിമാര്‍ക്കു കിട്ടിയ ‘എല്ലിന്‍ കഷ്ണം’ തീര്‍ത്തും ഉപയോഗശൂന്യമാണെന്ന് സംഭവത്തിന്റെ പശ്ചാത്തലം അസന്നിഗ്ധമായി വ്യക്തമാക്കുന്നുണ്ടെന്നു ചുരുക്കം. പ്രവാചകന്റെ (സ) അസാന്നിധ്യത്തില്‍ അദ്ദേഹത്തിന്റെ നിര്‍ദ്ദേശപ്രകാരമല്ലാതെ തികച്ചും അപ്രതീക്ഷിതമായി ഒരുപറ്റം അനുയായികള്‍ അകപ്പെട്ടുപോയ സംഘര്‍ഷത്തില്‍ മനുഷ്യസഹജമായുണ്ടായ ഒരു പ്രഹരത്തെ നബി(സ)യുടെ കലാപമനസ്സായി വിവര്‍ത്തനം ചെയ്യുന്നര്‍ വഞ്ചിക്കുന്നത് അവരവരുടെ മനസാക്ഷിയെത്തന്നെയാണ്. സംഭവമറിഞ്ഞപ്പോള്‍ അത്തരം സംഭവങ്ങള്‍ ആവര്‍ത്തിക്കാതിരിക്കുവാനുള്ള ഇടപെടലാണ്, അല്ലാതെ വ്യാപിപ്പിക്കുവാനുള്ള പരിശ്രമമല്ല നബി (സ) നടത്തിയത് എന്ന് ദാറുല്‍ അര്‍ക്വമിനെ പുതിയ പാഠശാലയും പ്രാര്‍ത്ഥനാശാലയുമായി നിശ്ചയിച്ചതില്‍ നിന്ന് സുതരാം മനസ്സിലാകുന്നുണ്ട്. പ്രവാചകന് കലാപമനസ്സുണ്ടെന്ന് കാണിക്കുവാനുള്ള ഒരു തെളിവും സഅദ്ബ്‌നു അബീ വക്വാസിന്റെ താഴ്‌വരാ പ്രഹരത്തില്‍ -കഥ ആധികാരികമാണെങ്കില്‍ പോലും- ഇല്ലെന്നു സാരം.

റ്റേതൊരു രാഷ്ട്രത്തെയും പോലെ ഇസ്‌ലാമിക രാഷ്ട്രത്തിലും വ്യത്യസ്ത ഉത്തരവാദിത്തങ്ങളുള്ള പൗരന്മാരുണ്ടായിരിക്കും. പട്ടാളക്കാരന്റെ ഉത്തരവാദിത്തവും സർക്കാർ ജീവനക്കാരന്റെ ഉത്തരവാദിത്തവും സാധാരണ തൊഴിലാളിയുടെ ഉത്തരവാദിത്തവും ജനാധിപത്യരാഷ്ട്രത്തിൽ പോലും വ്യത്യസ്തമാണല്ലോ.ഇസ്‌ലാമികരാഷ്ട്രത്തിലെ സാമൂഹികനിയമങ്ങളും അന്തരീക്ഷവുമെല്ലാം ഇസ്‌ലാമികമായിരിക്കും. അതുകൊണ്ട് തന്നെ മുസ്ലിംകൾക്ക് ഉത്തരവാദിത്തം കൂടുതലുണ്ടായിരിക്കും. സൈനികസേവനത്തതിന് സദാസന്നദ്ധരായിരിക്കണം ഓരോ മുസ്ലിമും. എന്നാൽ മുസ്ലിംകളല്ലാത്തവർക്ക് ആ ഉത്തരവാദിത്തമില്ല. മുസ്ലിംകളിലെ പണക്കാരിൽ നിന്ന് നിന്ന് രാഷ്ട്രം സകാത്ത് പിരിച്ചെടുക്കുകയും പാവങ്ങൾക്ക് വിതരണം നടത്തുകയും ചെയ്യും. ഈ സകാത്തിൽ നിന്ന് നാടിന്റെ സംരക്ഷണത്തിനുള്ള യുദ്ധത്തിന് വേണ്ടിയും രാഷ്ട്രം ചിലവഴിക്കും. എന്നാൽ അമുസ്ലിംകൾ സകാത്ത് നൽകേണ്ടതില്ല. എന്നാൽ അവർ സംരക്ഷണ നികുതിയായ ജിസ് യ നൽകുവാൻ കടപ്പെട്ടവരാണ്. രാഷ്ട്രസംരക്ഷണത്തിനു വേണ്ടിയുള്ള യുദ്ധങ്ങളിൽ പങ്കെടുക്കാത്ത അവരുടെ ജീവനും സ്വത്തിനും നൽകുന്ന സംരക്ഷണത്തിന് പകരമായുള്ള നികുതി മാത്രമാണിത്.

എന്നാൽ, ശിക്ഷാസമ്പ്രദായങ്ങളിലും അതല്ലാത്ത നിയമങ്ങളിലുമെല്ലാം ഇസ്‌ലാമിക രാഷ്ട്രത്തിലെ എല്ലാ പൗരന്മാരും തുല്യരായാണ് പരി ഗണിക്കപ്പെടുന്നത്. സമ്പന്നനും ദരിദ്രനുമിടയിലോ ഭരണാധികാരി ക്കും സാധാരണ പൗരന്നുമിടയിലോ യാതൊരുരീതിയിലുള്ള വി വേചനവും നീതിനിര്‍വഹണത്തിന്റെ കാര്യത്തില്‍ ഉണ്ടാവാന്‍ പാടില്ലെന്നാണ് ഇസ്‌ലാമിന്റെ അധ്യാപനം. 'സത്യവിശ്വാസികളേ, നിങ്ങള്‍ അല്ലാഹുവിനുവേണ്ടി സാക്ഷ്യം വഹിക്കുന്നവരെന്ന നില യയില്‍ കണിശമായി നീതി നിലനിര്‍ത്തുന്നവരായിരിക്കണം. അത് നിങ്ങള്‍ക്ക്തന്നെയോ, നിങ്ങളുടെ മാതാപിതാക്കള്‍, അടുത്ത ബ ന്ധുക്കള്‍ എന്നിവര്‍ക്കോ പ്രതികൂലമായിത്തീര്‍ന്നാലും ശരി' എ ന്നാണ് ക്വുര്‍ആന്‍ പഠിപ്പിക്കുന്നത്.

ഒരിക്കല്‍ കുലീന കുടുംബത്തില്‍പ്പെട്ട ഒരു സ്ത്രീ മോഷണക്കു റ്റത്തിന് പിടിക്കപ്പെട്ടു. പ്രവാചക(സ)ന്റെ അടുത്ത അനുയായികളിലൊ രാളായ ഉസാമഃ (റ ) അവരുടെ കാര്യത്തില്‍ ശുപാര്‍ശ ചെയ്യുവാനായി പ്രവാചകനെ സമീപിച്ചു. അപ്പോള്‍ പ്രവാചകന്‍ (സ)പറഞ്ഞ വാക്കു കള്‍ ശ്രദ്ധേയമാണ്: 'നിങ്ങള്‍ക്ക് മുമ്പുള്ള സമുദായങ്ങള്‍ നശിപ്പിക്ക പ്പെട്ടത് ഇത്തരം പ്രവര്‍ത്തനങ്ങള്‍ മൂലമാണ്. സമ്പന്നര്‍ കുറ്റംചെയ് താല്‍ വെറുതെ വിടുകയും ദരിദ്രര്‍ കുറ്റംചെയ്താല്‍ ശിക്ഷിക്കുകയു മായിരുന്നു അവരുടെ സമ്പ്രദായം. അല്ലാഹുവാണെ സത്യം. ഈ പ്രവൃത്തി ചെയ്തത് മുഹമ്മദിന്റെ മകള്‍ ഫാത്തിമ തന്നെയായാ ലും അവളുടെ കരം ഞാന്‍ ഛേദിച്ചുകളയുന്നതാണ്'. നീതിയുടെ മുമ്പില്‍ ഭരണാധിപനും ഭരണീയനും സമമാണ് എന്നതാണ് ഇസ്‌ലാമിക രാഷ്ട്രസങ്കല്‍പത്തിന്റെ അടിസ്ഥാന സ്തംഭങ്ങളിലൊന്ന്.

ഇക്കാര്യത്തിലേക്ക് വെളിച്ചംവീശുന്ന ഒട്ടന വധി സംഭവങ്ങള്‍ ഇസ്‌ലാമിക ചരിത്രത്തില്‍ കാണാനാവും. ഒരു സംഭവമിതാ:ഉമറി(റ )ന്റെ ഭരണകാലത്ത് ഈജിപ്തിലെ ഗവര്‍ണറായിരുന്ന അംറുബ്‌നുല്‍ ആസി(റ )ന്റെ പുത്രന്‍ മുഹമ്മദുബ്‌നു അംറ് കോപ്റ്റിക് വിഭാഗത്തില്‍പ്പെട്ട ഒരു സാധാരണക്കാരനെ തന്റെ കുതി രയുടെ മുമ്പില്‍ സ്വന്തം കുതിരയെ ഓടിച്ചതിന്റെ പേരില്‍ അടിച്ചു. 'ഞാന്‍ മാന്യന്മാരുടെ പുത്രനാണ്' എന്നു പറഞ്ഞുകൊണ്ടായിരുന്നു പ്രഹരം. ഈ പ്രശ്‌നം ഖലീഫയുടെ അടുക്കലെത്തി. ഗവര്‍ണറെയും പുത്രനെയും മദീനയിലേക്ക് വിളിപ്പിച്ചു കൊണ്ട് ഉമര്‍(റ )ചോദിച്ചു: 'എന്നുമുതല്‍ക്കാണ് നിങ്ങള്‍ ജനങ്ങളെ അടിമകളാക്കിത്തുടങ്ങിയ ത്? അവരുടെ മാതാക്കള്‍ അവരെ പ്രസവിച്ചത് സ്വതന്ത്രരായിട്ടല്ലേ'. തുടര്‍ന്ന് മര്‍ദിതനായ ആ കോപ്റ്റിക് വംശജന്റെ കയ്യില്‍ ചാട്ടവാറു നല്‍കിക്കൊണ്ട് പ്രതികാരം ചെയ്യുവാന്‍ ഉമര്‍ കല്‍പിച്ചു. നിയമ ത്തിനു മുന്‍പില്‍ എല്ലാവരും സമന്മാരാണെന്ന തത്ത്വം ഇത്രത്തോ ളം പ്രായോഗികമാക്കിയ ഇസ്‌ലാമിക സമൂഹമല്ലാത്ത മറ്റൊരു സമൂ ഹത്തെ ലോകചരിത്രത്തില്‍ കാണുക സാധ്യമല്ല.

ഇസ്‌ലാമികരാഷ്ട്രത്തില്‍ ജീവിക്കുന്ന അമുസ്‌ലിം പൗരന്മാരോ ടും നീതിനിര്‍വഹണത്തിന്റെ കാര്യത്തില്‍ വിവേചനമൊന്നും കാണിക്കാന്‍ പാടില്ലെന്നാണ് ഇസ്‌ലാം പഠിപ്പിക്കുന്നത്. വിശുദ്ധക്വുര്‍ആന്‍ നിഷ്‌കര്‍ഷിക്കുന്നത് കാണുക: 'സത്യവിശ്വാസികളേ, നിങ്ങ ള്‍ അല്ലാഹുവിനുവേണ്ടി നിലകൊള്ളുന്നവരും നീതിക്ക് സാക്ഷ്യം വഹിക്കുന്നവരുമായിരിക്കുക. ഒരു ജനതയോടുള്ള അമര്‍ഷം നീതി പാലിക്കാതിരിക്കാന്‍ നിങ്ങള്‍ക്ക് പ്രേരകമാകരുത്. നിങ്ങള്‍ നീതി പാലിക്കുക. അതാണ് ധര്‍മനിഷ്ഠയോട് ഏറ്റവും അടുത്തത.

ഈ ക്വുര്‍ആനിക കല്‍പനയെ പൂര്‍ണമായി ഉള്‍ക്കൊണ്ടുകൊണ്ടായിരു ന്നു പ്രവാചകനും സച്ചരിതരായ ഖലീഫമാരും ഭരണം നടത്തിയത്. ക്രൈസ്തവനായ കോപ്റ്റിക് വംശജനും മുസ്‌ലിമായ ഗവര്‍ണ റുടെ പുത്രനും തമ്മിലുണ്ടായ പ്രശ്‌നത്തില്‍ മുസ്‌ലിം അനീതിചെയ് തുവെന്ന് മനസ്സിലാക്കിയ ഖലീഫാ ഉമറിെേന്റ കാലത്തുതന്നെ നടന്ന മറ്റൊരു സംഭവമിതാ: ഗവര്‍ണറായിരുന്ന അംറുബ്‌നുല്‍ ആസ്വ്‌(റ ) ഒരു ക്രൈസ്തവസ്ത്രീയുടെ വീട് നിര്‍ബന്ധപൂര്‍വം പള്ളി യോട് ചേര്‍ക്കുകയുണ്ടായി. സ്ത്രീ ഇതേക്കുറിച്ച് ഖലീഫയോട് പ രാതി പറഞ്ഞു: ഖലീഫാ ഉമര്‍ ഗവര്‍ണറെ വിളിച്ച് വിശദീകരണ മാവശ്യപ്പെട്ടു. അദ്ദേഹം പറഞ്ഞു: 'മുസ്‌ലിംകളുടെ അംഗസംഖ്യ വര്‍ധിച്ചതു കാരണം പള്ളിയില്‍ സ്ഥലമില്ലാതെ വിഷമംനേരിട്ടു. പള്ളി വിശാലമാക്കേണ്ടതിന്നായി അതിന്റെ അടുത്തുള്ള ക്രൈസ് തവസ്ത്രീയുടെ ഭവനം ആവശ്യപ്പെടുകയും അതിന്ന് ന്യായമായ വില നല്‍കാമെന്ന് ഉറപ്പുനല്‍കുകയും ചെയ്തു. അവര്‍ വിസമ്മതി ച്ചതുകാരണം വീടുപൊളിച്ച് പള്ളിയോട് ചേര്‍ക്കാന്‍ നിര്‍ബന്ധിത മാവുകയാണുണ്ടായത്. അതിന്റെ വില ആ സ്ത്രീക്ക് ആവശ്യമുള്ള ഏതുസമയത്തും സ്വീകരിക്കുവാന്‍ സാധിക്കുമാറ് പൊതുഖജനാ വില്‍ നിക്ഷേപിക്കുകയും ചെയ്തിട്ടുണ്ട്.' സാധാരണഗതിയില്‍ ന്യാ യമെന്നു പറയാവുന്ന ഗവര്‍ണറുടെ നടപടിയെ ശരിവെക്കാന്‍ ഉമര്‍ തയാറായില്ല. പള്ളിയുടെ പ്രസ്തുത ഭാഗം പൊളിച്ചുമാറ്റി പകരം ആ ക്രൈസ്തവസ്ത്രീയുടെ വീട് യഥാസ്ഥാനത്ത് നിര്‍മിച്ചു കൊടു ക്കുവാന്‍ കല്‍പിക്കുകയാണ് നീതിമാനായ ആ ഭരണാധികാരി ചെ യ്തത്.

നാലാം ഖലീഫയായിരുന്ന അലി(റ )ടെ കാലത്ത് നടന്ന ഒരു സംഭവം നോക്കുക: ഖലീഫ തന്റെ പടയങ്കി ഒരു ജൂതന്‍ മോഷ്ടിച്ചു വെന്ന് പറഞ്ഞ് കോടതിയെ സമീപിച്ചു. ന്യായാധിപനായ ശുറൈഹ് ജൂതന്റെയും ഖലീഫയുടെയും അവകാശവാദങ്ങള്‍ പരിശോധിച്ചു. തന്റെ കൈവശമുള്ള പടയങ്കി തന്‍േറതു തന്നെയാണെന്നു ജൂതന്‍ വാദിച്ചപ്പോള്‍ അതു തന്‍േറതാണെന്നും ജൂതന്‍ മോഷ്ടിച്ചതാണെ ന്നും ഖലീഫ അലി മൊഴി നല്‍കി. തന്റെ വാദം തെളിയിക്കാനായി ഖലീഫ ഹാജരാക്കിയ സാക്ഷികള്‍ മകന്‍ ഹസനും ഭൃത്യന്‍ ഖമറുമായിരുന്നു. ഇവര്‍ രണ്ടുപേരും ഖലീഫയുടെ സ്വന്തക്കാരായ തിനാല്‍ സാക്ഷ്യം സ്വീകാര്യമല്ലെന്നും മറ്റു തെളിവുകള്‍ ഇല്ലാത്തതി നാല്‍ പടയങ്കി ജൂതനില്‍നിന്നു വാങ്ങുവാന്‍ നിയമം അനുവദിക്കു ന്നില്ലെന്നും ന്യായാധിപന്‍ വിധിച്ചു. ന്യായധിപന്റെയടുത്തുനിന്ന് പടയങ്കിവാങ്ങി നടന്നുനീങ്ങിയ ജൂതന്‍ അല്‍പദൂരം മുന്നോട്ടുപോ യി തിരിച്ചുവന്നുകൊണ്ട് പറഞ്ഞു: 'നിശ്ചയമായും ഇത് പ്രവാചക ന്മാരുടെ നിയമം തന്നെയെന്ന് ഞാന്‍ സാക്ഷ്യം വഹിക്കുന്നു. മതി യായ തെളിവില്ലാത്തതിനാല്‍ കോടതി അദ്ദേഹത്തിനെതിരെ വിധി പ്രസ്താവിക്കുന്നു. ഇതു വളരെ മഹത്തരം തന്നെ. അതിനാല്‍ അ ല്ലാഹുവല്ലാതെ ആരാധ്യനില്ലെന്നും മുഹമ്മദ്(സ) അല്ലാഹുവിന്റെ ദൂത നാണെന്നും ഞാന്‍ സാക്ഷ്യം വഹിക്കുന്നു'. തുടര്‍ന്ന് ഖലീഫയുടെ നേരെ തിരിഞ്ഞ് അദ്ദേഹം ഇങ്ങനെ പറഞ്ഞു: 'ഈ പടയങ്കി താങ്ക ളുടേത് തന്നെയാണ്. നിങ്ങള്‍ സ്വിഫ്ഫീനിലേക്ക് പോകുമ്പോള്‍ പിന്നിലുണ്ടായിരുന്ന ഞാന്‍ താങ്കളുടെ ചാരനിറത്തിലുള്ള ഒട്ടക ത്തിന്റെ പുറത്തുനിന്ന് അത് തട്ടിയെടുത്തതാണ്'. ഖലീഫ ചിരിച്ചു കൊണ്ട് പ്രതിവചിച്ചു. 'ഞാനത് താങ്കള്‍ക്ക് തന്നെ ദാനമായി തന്നി രിക്കുന്നു'.

ഇസ്‌ലാമിക രാഷ്ട്രത്തില്‍ ജീവിക്കുന്ന അമുസ്‌ലിം പൗരന്മാരുടെ വിശ്വാസസ്വാതന്ത്ര്യമോ ആരാധനാസ്വാതന്ത്ര്യമോ ഹനിക്കുവാന്‍ ഖലീഫമാര്‍ക്കവകാശമില്ല. അമുസ്‌ലിം പൗരന്മാരോട് അനീതി കാണിക്കുന്നത് അക്ഷന്തവ്യമായ അപരാധമാണെന്നാണ് പ്രവാചകന്‍പഠിപ്പിച്ചിരിക്കുന്നത്. അദ്ദേഹം പറഞ്ഞു: 'സൂക്ഷിച്ചുകൊ ള്ളുക. അമുസ്‌ലിം പൗരന്മാരെ ആരെങ്കിലും അടിച്ചമര്‍ത്തുകയോ അവരുടെമേല്‍ കഴിവിന്നതീതമായി നികുതിഭാരം കെട്ടിയേല്‍പി ക്കുകയോ അവരോട് ക്രൂരമായി പെരുമാറുകയോ അവരുടെ അവ കാശങ്ങള്‍ വെട്ടിക്കുറക്കുകയോ ചെയ്യുകയാണെങ്കില്‍ അന്ത്യനാളി ല്‍ അവന്നെതിരില്‍ ഞാന്‍ സ്വയംതന്നെ പരാതി ബോധിപ്പിക്കുന്ന താണ്'. ചരിത്രപണ്ഡിതനായ സര്‍ തോമസ് ആര്‍നോള്‍ഡ് എഴുതുന്നത് കാണുക: 'മുഹമ്മദ് തന്നെ പല അറബ് ക്രൈസ്തവ ഗോത്രങ്ങളുമാ യും സന്ധിയിലേര്‍പെട്ടിരുന്നു. അവര്‍ക്ക് അദ്ദേഹം സംരക്ഷണവും സ്വന്തം മതമനുസരിച്ച് ജീവിക്കുവാനുള്ള സ്വാതന്ത്ര്യവും ഉറപ്പുവരു ത്തി. അവരുടെ പുരോഹിതര്‍ക്കുണ്ടായിരുന്ന സവിശേഷാധികാ രങ്ങള്‍ പഴയതുപോലെ നിലനിര്‍ത്തി'

പ്രവാചകന്റെ പാത പിന്‍തുടര്‍ന്ന അദ്ദേഹത്തിന്റെ ഖലീഫമാ രും അമുസ്‌ലിം പൗരന്മാരുടെ വിശ്വാസങ്ങളും ആരാധനാസമ്പ്രദായ ങ്ങളും തുടരാന്‍ അനുവദിച്ചിരുന്നതായും അവരുടെ ജീവനും സ്വ ത്തിനും പരിപൂര്‍ണ സംരക്ഷണം ഉറപ്പുവരുത്തിയിരുന്നതായും കാണാന്‍ കഴിയും. ഖലീഫ ഉമര്‍ േഈലിയാ നിവാസികള്‍ക്ക് നല്‍കിയ രക്ഷാകരാ റിലെ വ്യവസ്ഥകള്‍ കാണുക: 'ദൈവദാസനും വിശ്വാസികളുടെ നേതാവുമായ ഉമര്‍ ഈലിയാവാസികള്‍ക്ക് നല്‍കുന്ന സംരക്ഷണം. എല്ലാവരുടെയും ജീവന്നും സ്വത്തിനും ആരാധനാലയങ്ങള്‍ക്കും കുരിശുകള്‍ക്കും മതസംബന്ധിയായ എല്ലാറ്റിനും സംരക്ഷണം ഉറ പ്പുനല്‍കുന്നു. ആരുടെയും ചര്‍ച്ചുകള്‍ വാസസ്ഥലമാക്കുകയോ ന ശിപ്പിക്കുകയോ ചെയ്യരുത്. അവയോട് ചേര്‍ന്നു നില്‍ക്കുന്ന വസ്തു വഹകളോ കുരിശുകളോ പിടിച്ചെടുക്കരുത്. ആരെയും ഉപദ്രവിക്ക രുത്'.

ഇങ്ങനെ, ഇസ്‌ലാമിക രാഷ്ട്രത്തിലെ അമുസ്ലിം പൗരന്മാർക്ക് യാതൊരു വിധ അനീതിയും അനുഭവിക്കേണ്ടി വരാത്ത രീതിയിലുള്ളതാണ് ഇസ്‌ലാമിക നിയമങ്ങൾ എന്ന കാണാൻ കഴിയും.

മുഹമ്മദ് നബി(സ)യുടെ പ്രവാചകത്വത്തിന്റെ ആരംഭത്തെക്കുറിച്ച് മുസ്‌ലിംകള്‍ പറയുന്ന കഥ, വഹ്‌യിനെക്കുറിച്ച അദ്ദേഹത്തിന്റെ അവകാശവാദങ്ങള്‍ അടിസ്ഥാനരഹിതമാണെന്നാണ് വ്യക്തമാക്കുന്നുണ്ട്. ദൈവികമായ പ്രചോദനമുണ്ടാകുമ്പോള്‍ ഉണ്ടാകേണ്ട സമാധാനത്തിനും സന്തോഷത്തിനും പകരം ഭയവിഹ്വലനായി വീട്ടില്‍ തിരിച്ചെത്തുന്ന മുഹമ്മദിനെയാണ് വെളിപാട് തുടങ്ങിയെന്ന് പറയപ്പെടുന്ന ദിവസം ചരിത്രത്തില്‍ നാം കാണുന്നത്. ദൈവദൂതനായ ജിബ്‌രീല്‍ തന്നെയാണ് പ്രവാചകനടുക്കല്‍ വന്നിരുന്നതെങ്കില്‍ ഇതാകുമായിരുന്നില്ല അനുഭവം. അതിനാല്‍ മുഹമ്മദ് നബിയ്ക്ക് നാല്‍പതാം വയസ്സിലുണ്ടായിയെന്ന് പറയപ്പെടുന്ന വെളിപാട് ദൈവത്തില്‍ നിന്നല്ലെന്ന കാര്യം സ്പഷ്ടമാണ്. പ്രവാചകത്വാരംഭവുമായി ബന്ധപ്പെട്ട നിവേദനങ്ങളെ വിശകലനം ചെയ്തുകൊണ്ട് നബി (സ) അനുഭവിച്ച വെളിപാട് ദിവ്യമല്ലെന്ന് വരുത്തിത്തീര്‍ക്കാന്‍ വേണ്ടി നബിവിമര്‍ശകര്‍ ഉന്നയിക്കുന്ന പരാമൃഷ്ടവാദങ്ങള്‍ ന്യായമല്ലേ?

ല്ല. അന്തിമപ്രവാചകന് ദിവ്യവെളിപാടുകള്‍ കിട്ടിത്തുടങ്ങിയ സന്ദര്‍ഭത്തെക്കുറിച്ചുള്ള ചരിത്രരേഖകളൊന്നും പ്രസ്തുതവെളിപാടുകളുടെ ദൈവികതയെ സംശയാസ്പദമാക്കുന്ന യാതൊരു പരാമര്‍ശവും ഉള്‍ക്കൊള്ളുന്നില്ല. പ്രവാചകന്റെ നാല്‍പതാം വയസ്സിലുണ്ടായ പ്രവാചകത്വലബ്ധിയെയും വഹ്‌യിന്റെ ആരംഭത്തെയും കുറിച്ചുള്ള കുറ്റമറ്റ നിവേദകപരമ്പരയിലൂടെയുള്ള വിവരണമുള്ളത് സ്വഹീഹുല്‍ ബുഖാരിയിലാണ്. പ്രവാചകപത്‌നി ആഇശ (റ), തന്റെ സഹോദരീപുത്രന്‍ ഉര്‍വക്ക് ആദ്യവെളിപാടിന്റെ സമയത്തെ പ്രവാചകാനുഭവങ്ങള്‍ പ്രവാചകന്റെ തന്നെ ആത്മകഥനാപരമായ വാചകങ്ങളുദ്ധരിച്ചുകൊണ്ട് സ്വന്തം വാക്കുകളില്‍ വിശദീകരിച്ചുകൊടുത്തതാണ് ബുഖാരി തന്റെ സ്വഹീഹിന്റെ തുടക്കത്തില്‍ ഹദീഥായി രേഖപ്പെടുത്തിയിട്ടുള്ളത് (കിതാബുല്‍ വഹ്‌യ്). പ്രവാചകത്വത്തിന്റെ ആരംഭത്തെക്കുറിച്ച് പരാമൃഷ്ട ഹദീഥ് നല്‍കുന്നത് താഴെക്കൊടുത്തിരിക്കുന്ന വിവരങ്ങളാണ്.

1. മുഹമ്മദ് നബി(സ)ക്ക് പകല്‍വെളിച്ചം പോലെ സത്യമായിപ്പുലരുന്ന സ്വപ്‌നദര്‍ശനങ്ങള്‍ നിരന്തരമായി ഉണ്ടാകാന്‍ തുടങ്ങി. ഇതായിരുന്നു പ്രവാചകനുമായുള്ള ദൈവിക ആശയവിനിമയത്തിന്റെ ആരംഭം.

2. ഉറക്കത്തില്‍ താന്‍ കാണുന്ന സ്വപ്‌നങ്ങളുടെ പുലര്‍ച്ച പ്രവാചകനെ ചിന്താകുലനാക്കുകയും അദ്ദേഹം മക്കയിലെ ഒരു പര്‍വതത്തിനുമുകളിലുള്ള ഹിറാഗുഹയില്‍ ഏകാന്തനായി പ്രാര്‍ത്ഥനകളില്‍ മുഴുകിയിരിക്കുന്ന ശീലം ആരംഭിക്കുകയും ചെയ്തു. രണ്ടോ മൂന്നോ ദിവസങ്ങള്‍ക്കാവശ്യമായ ജീവിതവിഭവങ്ങളുമായി ഗുഹയില്‍ തങ്ങുകയും ശേഷം വീട്ടിലേക്ക് മടങ്ങി വീണ്ടും വിഭവശേഖരണം നടത്തി ഗുഹയിലേക്കുതന്നെ തിരിച്ചുപോവുകയുമായിരുന്നു പതിവ്.

3. ഇങ്ങനെ പ്രവാചകന്‍ ഗുഹയിലിരിക്കുന്ന ഒരു ദിവസമാണ് ക്വുര്‍ആന്‍ അവതരണത്തിന് സമാരംഭം കുറിച്ചുകൊണ്ട് ഒരു മലക്ക് പ്രവാചകനരികില്‍ പ്രത്യക്ഷപ്പെട്ടത്.

4. മലക്ക് പ്രവാചകനോട് ‘ഇക്വ്‌റഅ്’ (വായിക്കുക/ഓതുക) എന്ന് കല്‍പിച്ചു. ‘ഞാന്‍ വായിക്കാനറിയുന്നവനല്ല’ (മാ അന ബി ക്വാരിഅ്) എന്നായിരുന്നു നബി(സ)യുടെ മറുപടി. അപ്പോള്‍ മലക്ക് പ്രവാചകന് താങ്ങാന്‍ കഴിയുന്നതിലുമപ്പുറത്തുള്ള ശക്തിയോടെ അദ്ദേഹത്തെ പിടിച്ചുഞെരുക്കുകയും ശേഷം സ്വതന്ത്രനാക്കുകയും ചെയ്ത് ‘ഇക്വ്‌റഅ്’ എന്ന കല്‍പന ആവര്‍ത്തിച്ചു. വീണ്ടും പ്രവാചകന്‍ പഴയ മറുപടി തന്നെ പറഞ്ഞു. മലക്ക് വീണ്ടും പ്രവാചകനെ ഞെരുക്കുകയും സ്വതന്ത്രനാക്കുകയും കല്‍പന ആവര്‍ത്തിക്കുകയും ചെയ്തു. മൂന്നാം തവണയും ഇതേഘട്ടങ്ങള്‍ കടന്നപ്പോള്‍ മലക്ക് പരിശുദ്ധ ക്വുര്‍ആനില്‍ 96-ാം അധ്യായം സൂറത്തുല്‍ അലക്വിലെ ആദ്യത്തെ അഞ്ച് വചനങ്ങള്‍ പ്രവാചകനെ ഓതിക്കേള്‍പിച്ചു. ഇതായിരുന്നു ക്വുര്‍ആന്‍ അവതരണത്തിന്റെയും പ്രവാചകത്വത്തിന്റെയും തുടക്കം. പ്രസ്തുത വചനങ്ങളുടെ സാരം ഇപ്രകാരമാണ്: ”സൃഷ്ടിച്ചവനായ നിന്റെ രക്ഷിതാവിന്റെ നാമത്തില്‍ വായിക്കുക. മനുഷ്യനെ അവന്‍ ഭ്രൂണത്തില്‍ നിന്ന് സൃഷ്ടിച്ചിരിക്കുന്നു. നീ വായിക്കുക: നിന്റെ രക്ഷിതാവ് പേനകൊണ്ട് പഠിപ്പിച്ചവനായ ഏറ്റവും വലിയ ഔദാര്യവാനാകുന്നു. മനുഷ്യന് അറിയാത്തത് അവന്‍ പഠിപ്പിച്ചിരിക്കുന്നു.”

5. ഗുഹയില്‍ ഏകാന്തനായിരിക്കെ മലക്കുമായുണ്ടായ മുഖാമുഖവും തത്സസമയത്തെ അനുഭവങ്ങളും പ്രവാചകനെ ഭയവിഹ്വലനാക്കി. പരിഭ്രാന്തനായി അതിവേഗത്തില്‍ മിടിച്ചുകൊണ്ടിരിക്കുന്ന ഹൃദയവുമായി വീട്ടിലേക്ക് മടങ്ങിയെത്തിയ അദ്ദേഹം പത്‌നി ഖദീജ ബിന്‍ത് ഖുവയ്‌ലിദിനോട് (റ) തന്നെ പുതപ്പിക്കുവാന്‍ ആവശ്യപ്പെട്ടു. വിറ നിന്ന് സാധാരണ നില കൈവരിക്കുവോളം പ്രവാചകന്‍ (സ) പുതച്ചുമൂടി നിന്നു. 6. അനന്തരം പുതപ്പില്‍നിന്ന് പുറത്തുവന്ന പ്രവാചകന്‍ ഉണ്ടായ സംഭവങ്ങള്‍ വിവരിക്കുകയും തനിക്ക് ഭയം അനുഭവപ്പെടുന്നുവെന്ന് പറയുകയും ചെയ്തു. അപ്പോള്‍ ഖദീജ (റ) ‘ഇല്ല, അല്ലാഹുവാണ് സത്യം, അല്ലാഹു അങ്ങയെ നിന്ദിക്കുകയില്ല. കാരണം അങ്ങ് ബന്ധുക്കളുടെ കാര്യം ശ്രദ്ധിക്കുന്നു, ദുര്‍ബലരുടെ ഭാരങ്ങള്‍ ചുമക്കുന്നു, ദരിദ്രര്‍ക്കും അശരണര്‍ക്കും സംരക്ഷണമേകുന്നു, അതിഥികളെ ആദരിക്കുന്നു, പ്രയാസപ്പെടുന്നവരെ സഹായിക്കുന്നു’ എന്നുപറഞ്ഞുകൊണ്ട് പ്രവാചകനെ ആശ്വസിപ്പിച്ചു. 7. പ്രവാചകനെ ആശ്വസിപ്പിച്ചശേഷം ഖദീജ അദ്ദേഹത്തെയും കൂട്ടി തന്റെ ബന്ധുവും ക്രൈസ്തവ-ജൂതഗ്രന്ഥങ്ങളില്‍ പ്രാവിണ്യമുണ്ടായിരുന്ന പണ്ഡിതനുമായിരുന്ന വറക്വത്ബ്‌നു നൗഫലിനടുത്തേക്കുപോയി. വറക്വ കാഴ്ചശക്തിയടക്കം നഷ്ടപ്പെട്ട് അങ്ങേയറ്റം വൃദ്ധനായിത്തീര്‍ന്നിരുന്നു. പ്രവാചകനുണ്ടായ അനുഭവങ്ങള്‍ കേട്ട വറക്വ, മോശെ (മൂസ) പ്രവാചകന്റെയടുക്കലേക്ക് ദിവ്യവെളിപാടുമായി വന്ന മലക്ക് ജിബ്‌രീല്‍ തന്നെയാണ് ഹിറാ ഗുഹയില്‍ വന്നതെന്ന് പറയുകയും പ്രവാചകന്‍ തനിക്ക് ലഭിച്ചുതുടങ്ങിയ ദിവ്യവെളിപാടുകള്‍ പ്രബോധനം ചെയ്യാന്‍ തുടങ്ങുമ്പോള്‍ പൂര്‍വപ്രവാചകന്‍മാരെപ്പോലെ കടുത്ത എതിര്‍പ്പുകള്‍ നേരിടുമെന്നും മക്കയില്‍നിന്ന് പുറത്താക്കപ്പെടുമെന്നും അന്ന് ഒരു യുവാവായി നാട്ടിലുണ്ടാകണമെന്നും പ്രവാചകനെ പിന്തുണക്കാന്‍ കഴിയണമെന്നും താന്‍ ആഗ്രഹിച്ചുപോകുന്നുവെന്നും പറഞ്ഞു. പക്ഷേ ഈ കൂടിക്കാഴ്ച കഴിഞ്ഞ് അധികമാകും മുമ്പെ അദ്ദേഹം മരണപ്പെട്ടുപോയി.

പരിശുദ്ധ ക്വുര്‍ആനിന്റെ അവതരണസമയത്ത് പ്രവാചകനുണ്ടായ പരാമൃഷ്ടാനുഭവങ്ങളില്‍ അസംഗതമായിട്ടെന്താണുള്ളതെന്ന് വ്യക്തമാക്കേണ്ടത് വിമര്‍ശകര്‍ തന്നെയാണ്. ഭൗതികവാദികളായ നബിവിമര്‍ശകര്‍ക്ക് വഹ്‌യ് എന്ന ആശയത്തെ തന്നെ ഉള്‍ക്കൊള്ളാന്‍ കഴിയാതിരിക്കുക സ്വാഭാവികമാണ്. പ്രപഞ്ചത്തിന് ഒരു സ്രഷ്ടാവ് തന്നെയില്ലെന്ന് വിശ്വസിക്കുന്നവര്‍ക്ക് ആ സ്രഷ്ടാവ് മനുഷ്യര്‍ക്കായുളള ധാര്‍മികപദ്ധതി വെളിപാടുകള്‍ വഴി പ്രവാചകന്‍മാര്‍ക്കെത്തിച്ചു കൊടുക്കുന്നുവെന്ന ആശയത്തെ ഒരിക്കലും അംഗീകരിക്കാനാകില്ല. അവരുടെ നിഷേധത്തിന് വഹ്‌യിന്റെ സമയത്തെ പ്രവാചകാനുഭവങ്ങളെ വിശകലനം ചെയ്തുകൊണ്ട് മറുപടി പറയുന്നത് അര്‍ത്ഥശൂന്യമാണ്. ദൈവാസ്തിത്വത്തെക്കുറിച്ചും ദൈവിക മാര്‍ഗദര്‍ശനത്തിന്റെ അനിവാര്യതയെക്കുറിച്ചുമെല്ലാമുളള സംവാദങ്ങളാണ് അവരുമായി നടക്കേണ്ടത്. പടച്ചവനുണ്ടെന്നും വഹ്‌യ് എന്നൊരു സംവിധാനമുണ്ടെന്നും അംഗീകരിക്കുന്നവര്‍ക്ക് മാത്രമാണ് വഹ്‌യിന്റെ പ്രവാചകാനുഭവങ്ങളെ അപഗ്രഥിക്കേണ്ട ആവശ്യമുണ്ടാകുന്നത്.

മിഷനറിമാര്‍ തീര്‍ച്ചയായും രണ്ടാമതുപറഞ്ഞ ഗണത്തില്‍ വരുന്നവരാണ്. പ്രവാചകനുണ്ടായത് ദൈവിക വെളിപാടല്ലെന്ന് വരുത്തിത്തീര്‍ക്കാന്‍ ഹദീഥുകളിലുള്ള ഹിറാ വിവരണങ്ങളെ ഓറിയന്റലിസ്റ്റ് വ്യാഖാനങ്ങളെ ഉപജീവിച്ചുകൊണ്ട് അവര്‍ വിമര്‍ശനാത്മകമായി വിശകലനം ചെയ്തതിന് കയ്യും കണക്കുമില്ല. ആ വിമര്‍ശനങ്ങളെല്ലാം, സ്വാഭാവികമായും വഹ്‌യിന്റെ സമയത്ത് പ്രവാചകന്‍മാര്‍ക്കുണ്ടാകുന്ന അനുഭവങ്ങളെന്തൊക്കെയാണെന്ന് ബൈബിളുപയോഗിച്ച് വിശദീകരിച്ചശേഷം പ്രവാചകനുണ്ടായത് അത്തരത്തിലുള്ള അനുഭവങ്ങളൊന്നുമല്ലെന്ന് പറയാനാണ് അടിസ്ഥാനപരമായി ശ്രമിക്കുന്നത്. വഹ്‌യ് പ്രവാചകനില്‍ ഭൗതികമായി ഏതുതരം പ്രതിഫലനങ്ങളാണ് സൃഷ്ടിക്കുക എന്നതിനെ സംബന്ധിച്ച തെറ്റുപറ്റാത്ത ധാരണകള്‍ നാം സ്വരൂപിക്കേണ്ടത് ബൈബിളില്‍ നിന്നാണ് എന്ന നിലപാടാണ് ഈ വര്‍ത്തമാനത്തിന്റെ അടിത്തറ.

ബൈബിള്‍ നൂറുശതമാനവും ദൈവികമാണെന്നും അതില്‍ മനുഷ്യ നിഗമനങ്ങളിടം പിടിച്ചിട്ടില്ലെന്നും എഴുതപ്പെട്ട അതേ രീതിയില്‍ തന്നെ ബൈബിള്‍ പുസ്തകങ്ങള്‍ ഇപ്പോഴും നിലനില്‍ക്കുന്നുവെന്നും മിഷനറിമാര്‍ തെളിയിക്കുമ്പോള്‍ മാത്രമേ ബൈബിളുപയോഗിച്ച് മുഹമ്മദ് നബി(സ)യുടെ വഹ്‌യനുഭവങ്ങളെ നിരൂപണം ചെയ്യുന്നതിന് സാധൂകരണമുണ്ടാവുകയുള്ളൂ. ബൈബിളില്‍ വഹ്‌യിനെക്കുറിച്ച് ഏഴുതപ്പെട്ടിരിക്കുന്നതെന്നല്ലാം സത്യമാണെന്ന് വിശ്വസിക്കുന്നതില്‍ -ബൈബിളിന്റെ ദൈവികത തെളിയിക്കപ്പെടാത്തിടത്തോളം കാലം- യാതൊരു യുക്തിയുമില്ല. ഇതുപോലെത്തന്നെ പ്രധാനമാണ് എല്ലാ പ്രവാചകന്‍മാരുടെയും വെളിപാടനുഭവങ്ങള്‍ ഒരുപോലെയായിരിക്കണമെന്ന് ശഠിക്കുന്നത് അര്‍ത്ഥശൂന്യമാണെന്നതും. ദൈവം അവന്റെ പ്രവാചകന്‍മാരോട് സംസാരിക്കാന്‍ വ്യത്യസ്ത മാര്‍ഗങ്ങള്‍ സ്വീകരിക്കുകയും ആശയവിനിമയത്തിന് അവരെ വ്യത്യസ്ത അനുഭവങ്ങളിലൂടെ കൂട്ടിക്കൊണ്ടുപോവുകയും ചെയ്യുന്നതില്‍ എന്ത് അസാംഗത്യമാണുള്ളത്?

ചില പ്രവാചകന്‍മാരുടെ വെളിപാട് സ്വീകരണസമയത്തെ അനുഭവങ്ങള്‍ തന്നെ എല്ലാ പ്രവാചകന്‍മാര്‍ക്കും അതുപോലെ ആവര്‍ത്തിക്കുമെന്ന് ദൈവം പറയാത്തിടത്തോളം കാലം അത്തരം താരതമ്യങ്ങള്‍ തന്നെ അപ്രസക്തമാണെന്നതാണ് വസ്തുത. നബി(സ)യുടെ വ്യക്തിനിഷ്ഠമായ വെളിപാടനുഭവങ്ങളെ ബൈബിളിന്റെ മാത്രം അടിസ്ഥാനത്തില്‍ തള്ളിക്കളയാന്‍ ശ്രമിക്കുന്നത് തീര്‍ത്തും ബാലിശമായ ഒരു മിഷനറി സങ്കുചിതത്വമാണെന്ന് ചുരുക്കം.

ഇനി, ഹിറാ അനുഭവങ്ങളെ ബൈബിളുപയോഗിച്ച് വിശകലനം ചെയ്താല്‍ അവയുടെ ദൈവികതയെ നിഷേധിക്കേണ്ടി വരുമോ എന്ന് നാം പരിശോധിക്കുക. ബൈബിളിനെ അന്ധമായി പിന്‍പറ്റുന്നവര്‍ക്കുമാത്രം ബാധകമായ ഒരു ചര്‍ച്ചയാണ് ഇത്. എങ്കിലും നാം പരിശോധനയ്ക്ക് മുതിരുക. പ്രവാചകന് സത്യമായിപ്പുലരുന്ന സ്വപ്‌നദര്‍ശനങ്ങള്‍ ഉണ്ടായിത്തുടങ്ങിയെന്നും അതായിരുന്നു അല്ലാഹു അദ്ദേഹവുമായി സംവദിക്കാനാരംഭിച്ച രീതി എന്നുമാണ് ഹദീഥ് ഒന്നാമതായി പറയുന്നത്. ബൈബിളികമായ വീക്ഷണത്തില്‍ ഇതിന് എന്ത് കുഴപ്പമാണുള്ളത്? നേരായിപ്പുലരുന്ന സ്വപ്‌നങ്ങള്‍ ഉറക്കില്‍ പ്രവാചകന്‍മാരെ കാണിക്കുന്നത് ദൈവത്തിന്റെ രീതിയാണെന്നു തന്നെയല്ലേ ബൈബിളും പറയുന്നത്? ജോസഫും (ഉല്‍പത്തി 37: 5-8), ജേക്കബും (ഉല്‍പത്തി 28: 12-14), അബ്രഹാമും (ഉല്‍പത്തി 15: 1), സോളമനും (1 രാജാക്കന്‍മാര്‍ 8: 5) എല്ലാം ദൈവത്താല്‍ സത്യസ്വപ്‌നങ്ങള്‍ കാണിക്കപ്പെട്ടതിനെക്കുറിച്ചുള്ള ബൈബിള്‍ വചനങ്ങള്‍ ഉണ്ടെന്നിരിക്കെ മുഹമ്മദ് നബി(സ)ക്കുണ്ടായ സമാനമായ അനുഭവത്തെ ബൈബിളികമായ ഏത് പ്രതലമുപയോഗിച്ചാണ് മിഷനറിമാര്‍ക്ക് തള്ളിക്കളയേണ്ടി വരുന്നത്?

താന്‍ കാണാന്‍ തുടങ്ങിയ, അക്ഷരംപ്രതി പുലര്‍ന്നുകൊണ്ടിരിക്കുന്ന സ്വപ്‌നങ്ങള്‍ പ്രവാചകനെ (സ) അത്യധികം ചിന്താകുലനാക്കുകയും ഒറ്റയ്ക്കിരിക്കുവാന്‍ അദ്ദേഹം ഇഷ്ടപ്പെടുന്ന സ്ഥിതിയുണ്ടാക്കുകയും ചെയ്തതാണ് ഹിറയില്‍ പോയി പ്രാര്‍ത്ഥനകളില്‍ ശാന്തി കണ്ടെത്തുവാന്‍ ശ്രമിക്കുന്ന ഒരു സന്ദര്‍ഭം പ്രവാചകജീവിതത്തിലേക്ക് കൊണ്ടുവന്നത് എന്ന ചരിത്രവസ്തുത, തിരുനബി(സ)ക്ക് കാണിക്കപ്പെട്ട സ്വപ്‌നങ്ങള്‍ക്കുപിന്നില്‍ ജിബ്‌രീല്‍ മലക്കുമായി മുഖാമുഖം കാണാനുള്ള ഇടത്തിലേക്ക് അദ്ദേഹത്തെ നയിച്ചുകൊണ്ടുപോവുക എന്ന ദൈവിക പദ്ധതിയുണ്ടായിരുന്നുവെന്ന് സുതരാം വ്യക്തമാക്കുന്നുണ്ട്. അന്തിമവേദഗ്രന്ഥം അവതരിപ്പിച്ചു തുടങ്ങാനുള്ള സാഹചര്യം പ്രവാചകജീവിതത്തില്‍ സൃഷ്ടിക്കുവാനും പ്രവാചകത്വത്തിന്റെ മൂര്‍ത്തമായ ആരംഭം കുറിക്കുവാനും വേണ്ടി പടച്ചവന്‍ മുഹമ്മദ് നബി(സ)യുമായുള്ള സംവേദനത്തിന്റെ ആമുഖം സ്വപ്‌നങ്ങള്‍ വഴി നിര്‍വഹിച്ചുതുടങ്ങിയതില്‍ ശുദ്ധമായ ബൈബിള്‍ കണ്ണിലൂടെ നോക്കിയാല്‍ പോലും അസാധാരണമായി യാതൊന്നുമില്ലെന്ന് നാം കണ്ടു.

ഇനി ഹിറയിലേക്ക് വരാം. ഹിറാ ഗുഹയില്‍ വന്ന് മലക്ക് പ്രവാചകനെ ക്വുര്‍ആന്‍ വചനങ്ങള്‍ കേള്‍പിച്ച അനുഭവത്തെയാണ് ഓറിയന്റലിസ്റ്റുകളും മിഷനറിമാരും ഏറ്റവും പ്രധാനമായി ആക്രമിക്കാറുള്ളത്. മനഃശാസ്ത്രപരമായി പ്രസ്തുത സന്ദര്‍ഭത്തെ ഉപജീവിച്ചുകൊണ്ടുള്ള വിമര്‍ശനകാഠിന്യം തികച്ചും സ്വാഭാവികമാണ് എന്നുപറയുന്നതായിരിക്കും ശരി. കാരണം വെളിപാട് എന്നു പറയുമ്പോള്‍ അതിനെ കേവലം ഒരു ആന്തരിക പ്രചോദനമായി മാത്രം ചുരുക്കിയെഴുതാനാഗ്രഹിക്കുന്നവരാണ് മിഷനറിമാരെല്ലാം തന്നെ. തങ്ങള്‍ വേദഗ്രന്ഥമായി അവതരിപ്പിക്കുന്ന പഴയ-പുതിയ നിയമ പുസ്തകങ്ങളൊന്നും ദൈവമോ, ദൈവദൂതനോ ഏതെങ്കിലും പ്രവാചകനെ ഓതിക്കേള്‍പ്പിച്ചവയല്ലെന്നും പലരും പലകാലങ്ങളില്‍ സ്വന്തമായി എഴുതിയുണ്ടാക്കിയവയാണെന്നും ബോധ്യമുള്ളവര്‍ക്ക്, ദൈവം മനസ്സില്‍ സൃഷ്ടിക്കുന്ന ‘തോന്നലു’കള്‍ക്കനുസൃതമായി മനുഷ്യര്‍ സ്വന്തം വാചകങ്ങളില്‍ എഴുതിയുണ്ടാക്കുന്നവയാണ് വേദഗ്രന്ഥങ്ങള്‍ എന്ന് പറയേണ്ടി വരുന്നതില്‍ യാതൊരു അത്ഭുതത്തിനും വകയില്ല. ‘ദൈവനിവേശിതവും പരിശുദ്ധാത്മ പ്രചോദിതവുമാണ്, വാചകങ്ങള്‍ ദൈവികമല്ലെങ്കിലും വിശുദ്ധ ബൈബിള്‍’ എന്ന് സമാധാനിക്കുന്നവര്‍ക്കെങ്ങനെയാണ് തങ്ങള്‍ ശത്രുവായി കാണുന്ന മുഹമ്മദ് നബി(സ)ക്ക് ക്വുര്‍ആനിന്റെ പാഠം (text) ദൈവത്തിന്റെ മലക്ക് മൂര്‍ത്തമായി പ്രത്യക്ഷപ്പെട്ട് ഓതിക്കേള്‍പ്പിച്ചുതുടങ്ങി എന്ന വൃത്താന്തം അസ്വസ്ഥതയുണ്ടാക്കാതിരിക്കുക?

ക്വുര്‍ആനിലെ വചനങ്ങള്‍ നൂറുശതമാനം ദൈവികമാണെന്നും അവ അപ്പടി പ്രവാചകന് അവതരിപ്പിക്കപ്പെടുകയാണുണ്ടായതെന്നും വരുമ്പോള്‍ ബൈബിള്‍ ക്വുര്‍ആനുമായി താരതമ്യം പോലുമര്‍ഹിക്കുന്നില്ലെന്ന് തിരിച്ചറിയുന്നവരുടെ ആര്‍ക്കും പ്രവചിക്കാവുന്ന അസഹിഷ്ണുത മാത്രമാണ് ഹിറയില്‍ ചുറ്റിത്തിരിയുന്ന വിമര്‍ശകത്തൂലികകളുടെ മഷിയും ഊര്‍ജ്ജവുമെന്ന കാര്യം സ്പഷ്ടമാണ്. സത്യം സ്വാര്‍ത്ഥതാല്‍പര്യങ്ങള്‍ക്കെതിരായി വരുമ്പോള്‍ അസഹിഷ്ണുത കാണിക്കലല്ല മറിച്ച് അതിനോടുള്ള ശാത്രവം അവസാനിപ്പിക്കലാണ് രക്ഷയുടെ മാര്‍ഗമെന്ന് ഇവര്‍ തിരിച്ചറിഞ്ഞിരുന്നെങ്കില്‍ എന്ന് പ്രാര്‍ത്ഥിക്കുവാന്‍ മാത്രമേ നമുക്ക് കഴിയൂ. വെളിപാടിനെ ഒരു അന്തപ്രചോദനത്തിന്റെ മാത്രം തലമുള്ള മാനസികാനുഭൂതിയില്‍ ഒതുക്കി നിര്‍ത്തുകയല്ല അന്തിമപ്രവാചകന്റെ കാര്യത്തില്‍ പ്രപഞ്ചനാഥന്‍ ചെയ്തത്; മറിച്ച് അതിന്റെ ആദ്യ സന്ദര്‍ഭത്തില്‍ തന്നെ തികച്ചും മൂര്‍ത്തമായി തന്റെ മലക്കിനെ പ്രവാചകനുമുന്നില്‍ കൊണ്ടുചെന്നു നിര്‍ത്തുകയും ശാരീരികമായിത്തന്നെ പ്രവാചകനെ അതിശക്തമായി മലക്കിനെക്കൊണ്ട് സ്പര്‍ശിപ്പിക്കുകയും ശേഷം സ്ഫുടമായ വാചകങ്ങളിലുള്ള സംഭാഷണത്തിന് അവസരമൊരുക്കയും ചെയ്യുകയാണ്. അകത്തുനിന്ന് നിര്‍ഗളിക്കുന്ന യാതൊന്നുമല്ല, മറിച്ച് പുറത്തുനിന്ന് പ്രവാചകനിലേക്ക് പ്രവഹിക്കുന്നതാണ് വഹ്‌യ് എന്ന് ഇതിനേക്കാള്‍ വ്യക്തമായി ഭൗതിക പരിതസ്ഥിതികളുപയോഗിച്ച് പ്രതിഫലിപ്പിക്കാനാവുകയില്ല തന്നെ. വെളിപാടു സ്വീകരണത്തിന്റെ ഏറ്റവും ഉയര്‍ന്നതും സുവ്യക്തവും തീവ്രവുമായ വൈയക്തികാനുഭവങ്ങളാണ് പ്രവാചകശ്രേഷ്ഠനുണ്ടായതെന്ന് സാരം. നബിവിദ്വേഷത്താല്‍ ഉന്മാദം ബാധിച്ച തലച്ചോറുകള്‍ക്ക് പടച്ചവന്‍ പ്രവാചകനു നല്‍കിയ ഈ തുല്യതയില്ലാത്ത ആദരവിനെ തമസ്‌കരിക്കേണ്ടി വരുന്നതിന്റെ രാഷ്ട്രീയം യാതൊരു വിശദീകരണവുമില്ലാതെ തന്നെ വ്യക്തമാണ്.

ഹിറയില്‍വെച്ച് പ്രവാചകനുണ്ടായ അനുഭവങ്ങളെ നാം പരിഗണിക്കുക. പ്രവാചകത്വമോ വെളിപാടുകളോ ദീര്‍ഘകാലത്തേക്ക് ഉണ്ടായിട്ടില്ലാത്ത മക്കയിലെ അറബ് സമൂഹത്തില്‍ ജനിച്ചുവളര്‍ന്ന, വേദപുസ്തകങ്ങളുമായി യാതൊരു പരിചയവുമില്ലാത്ത സാധാരണക്കാരനായ ഒരു മനുഷ്യന്‍ ആള്‍പാര്‍പ്പില്ലാത്ത ഒരു കൂറ്റന്‍ പര്‍വതത്തിന്റെ ചെങ്കുത്തായ ചെരുവിലുള്ള ഹിറ എന്ന ചെറിയ ഗുഹയില്‍ ഒറ്റയ്ക്കിരിക്കുമ്പോള്‍ ഒരു ദിവസം പൊടുന്നനെ അല്ലാഹുവിന്റെ ദിവ്യസന്ദേശവുമായി അതിശക്തനായ ജിബ്‌രീല്‍ എന്ന മലക്ക് പ്രത്യക്ഷപ്പെടുകയും എഴുത്തോ വായനയോ അറിയാത്ത പ്രവാചകനോട് വായിക്കുവാനാജ്ഞാപിക്കുകയും വായിക്കാനറിയില്ലെന്നു പറഞ്ഞപ്പോള്‍ ആരും ഉലഞ്ഞുപോകുംവിധം ചേര്‍ത്തുപിടിച്ചമര്‍ത്തുകയും ഇത് രണ്ടുതവണ കൂടിയാവര്‍ത്തിച്ചശേഷം ഗാംഭീര്യം തുളുമ്പുന്ന അഞ്ച് ദിവ്യവചനങ്ങള്‍ ഓതികേള്‍പിക്കുകയും അപ്രത്യക്ഷനാവുകയും ചെയ്യുന്നു. പ്രവാചകന്‍ ഇത്തരമൊരനുഭവം ജീവിതത്തിലൊരിക്കലും പ്രതീക്ഷിച്ചിട്ടുപോലുമില്ലെന്ന കാര്യം കൂടി നമ്മളോര്‍ക്കണം. പ്രവാചകത്വത്തിന്റെയും വെളിപാടുകളുടെയും രീതിശാസ്ത്രവുമായുള്ള തികഞ്ഞ അപരിചിതത്വം, താന്‍ ഒരു പ്രവാചനായി നിയോഗിക്കപ്പെടുമെന്നോ തന്റെയടുക്കലേക്ക് മലക്ക് വരും എന്നോ ഉള്ള യാതൊരു വിചാരവുമില്ലാത്ത ഒരാളുടെ തയ്യാറെയുപ്പില്ലായ്മ. അതിഭീകരമായ ഏകാന്തത. ആരും ഭയന്നുപോകുംവിധമുള്ള ആശ്ലേഷം. ലോകര്‍ക്കു മുഴുവന്‍ ദിവ്യസന്ദേശമെത്തിക്കാനുള്ള ദൗത്യം ജഗന്നിയന്താവിനാല്‍ ഏല്‍പിക്കപ്പെടുന്നത് സൃഷ്ടിക്കുന്ന മനോവിസ്‌ഫോടനം. ദൈവത്തിന്റെ വചനങ്ങള്‍ അവന്റെ ദൂതനില്‍നിന്ന് നേരിട്ടുകേള്‍ക്കുന്നത് സൃഷ്ടിക്കുന്ന ശാരീരിക വ്യതിയാനങ്ങള്‍. ആരാണ് ഭയന്നുപോകാതിരിക്കുക? ആരാണ് വിറച്ചുപോകാതിരിക്കുക?

വിശുദ്ധ ക്വുര്‍ആനിലെ വചനങ്ങള്‍, അവയുടെ ആശയങ്ങളും പദങ്ങളും ഒരുപോലെ ദൈവത്തില്‍നിന്ന് നിര്‍ഗളിച്ചവയായതിനാല്‍, ശുദ്ധ മനസ്സുള്ള ആരിലും പ്രകമ്പനങ്ങള്‍ സൃഷ്ടിക്കുക തന്നെ ചെയ്യും. ക്വുര്‍ആന്‍ വചനങ്ങളുടെ കേവലമായ ശ്രുതി തന്നെ അതിന്റെ ദൈവികതയെക്കുറിച്ച ബോധമുള്ളവരുടെ മനസ്സില്‍ ആന്ദോളനങ്ങളുളവാക്കാന്‍ പോന്നതാണ്. ആശയങ്ങളറിഞ്ഞു കേള്‍ക്കുന്നവരില്‍ അത് ചെലുത്തുന്ന സ്വാധീനം അതിനേക്കാള്‍ ശക്തമാണ്. മനുഷ്യന്റെ നിസ്സാരതയും ദൈവത്തിന്റെ മഹത്വവും ദൈവതൃപ്തിക്കായുള്ള അധ്വാനങ്ങളില്‍ മനുഷ്യന്‍ വരുത്തുന്ന അലംഭാവത്തിന്റെ ഗൗരവവും ബോധ്യപ്പെടുത്തി ജീവിതത്തെ പൂര്‍ണമായി സംസ്‌കരിക്കുവാന്‍ ശേഷിയുള്ള അതിശക്തമായ ദൈവഭയം കേള്‍വിക്കാരനില്‍ നിറയ്ക്കുകയാണ് ഓരോ ക്വുര്‍ആന്‍ വചനവും ചെയ്യുന്നത്. സത്യവിശ്വാസികളില്‍ ക്വുര്‍ആന്‍ ശ്രവണം ഉണ്ടാക്കുന്ന ഫലത്തെക്കുറിച്ച് ക്വുര്‍ആന്‍ തന്നെ പറയുന്നതിപ്രകരമാണ്: ”അല്ലാഹുവെപ്പറ്റി പറയപ്പെട്ടാല്‍ ഹൃദയങ്ങള്‍ പേടിച്ചുനടുങ്ങുകയും അവന്റെ വചനങ്ങള്‍ വായിച്ചുകേള്‍പിക്കപ്പെട്ടാല്‍ വിശ്വാസം വര്‍ധിക്കുകയും തങ്ങളുടെ രക്ഷിതാവിന്റെ മേല്‍ ഭരമേല്‍പിക്കുകയും ചെയ്യുന്നവര്‍ മാത്രമാണ് സത്യവിശ്വാസികള്‍.” (ക്വുര്‍ആന്‍ 8: 2).

ഈ ഫലം സൃഷ്ടിക്കുന്ന ക്വുര്‍ആന്‍ വചനങ്ങള്‍, ദൈവത്തിന്റെ ഘനഗംഭീരമായ സംസാരം, ആകാശത്തുനിന്ന് ഭൂമിയിലേക്കിറങ്ങിവന്ന അതിശക്തനായ ഒരു മലക്കില്‍നിന്ന് മനുഷ്യരുടെ കൂട്ടത്തില്‍വെച്ച് ആദ്യമായി കേള്‍ക്കുകയാണ് മുഹമ്മദ് നബി (സ). ക്വുര്‍ആന്‍ അവതരണം പ്രവാചകനെ മാനസികമായും ശാരീരികമായും വിറപ്പിച്ച അനുഭവമായിത്തീരാതിരിക്കുക പിന്നെയെങ്ങനെയാണ്? ക്വുര്‍ആന്‍ അവതരണപ്രക്രിയയുടെ ഭാരത്തെക്കുറിച്ച് സുന്ദരമായ ഒരുപമയിലൂടെ ക്വുര്‍ആന്‍ തന്നെ വര്‍ണിക്കുന്നുണ്ട്: ”ഈ ക്വുര്‍ആനിനെ നാം (അല്ലാഹു) ഒരു പര്‍വതത്തിനുമുകളില്‍ അവതരിപ്പിച്ചിരുന്നെങ്കില്‍ അത് (പര്‍വതം) വിനീതമാകുന്നതും അല്ലാഹുവെപ്പറ്റിയുള്ള ഭയത്താല്‍ പൊട്ടിപ്പിളരുന്നതും നിനക്ക് കാണാമായിരുന്നു. ആ ഉദാഹരണങ്ങള്‍ നാം ജനങ്ങള്‍ക്കുേവണ്ടി വിവരിക്കുന്നു. അവര്‍ ചിന്തിക്കുവാന്‍വേണ്ടി.” (59: 21) വഹ്‌യ് പ്രവാചകനില്‍ സൃഷ്ടിച്ച വിഹ്വലത, തികച്ചും സ്വാഭാവികമായിരുന്നുവെന്നര്‍ത്ഥം. വഹ്‌യ് ലഭിച്ച സ്ഥലവും രീതിയും മുതല്‍ അതിന്റെ ഉള്ളടക്കവും ധ്വനികളും വരെ ആ വിഹ്വലതയില്‍ പ്രതിഫലിക്കുന്നുണ്ട്; ഇത്ര വലിയ പ്രകമ്പനങ്ങള്‍ സൃഷ്ടിക്കുന്ന ശാരീരിക-മാനസികാനുഭവങ്ങള്‍ സമ്മാനിക്കുന്ന ദൈവികബോധനപ്രക്രിയയെ താങ്ങാനുള്ള കരുത്ത് തനിക്കുണ്ടോ എന്ന ആവലാതി മുതല്‍ അന്തിമപ്രവാചകന്‍ എന്ന അതിഭയങ്കരമായ ഉത്തരവാദിത്തം ശിരസ്സാവഹിക്കാനുള്ള വലുപ്പം തനിക്കുണ്ടോ എന്ന ഭയപ്പാടുവരെ ആ വിറയലില്‍ പ്രതിധ്വനിക്കുന്നുണ്ട്.

പ്രവാചകന്‍ ഒരു മനുഷ്യനാണ് എന്ന വസ്തുത മാത്രമാണ് ഹിറാ സംഭവം തെളിയിക്കുന്നത്; നിഷ്‌കളങ്കനും ആത്മാര്‍ത്ഥതയുള്ളവനുമായ മനുഷ്യന്‍. പ്രവാചകത്വം ഏല്‍പിക്കുവാന്‍വേണ്ടി അതേ സാഹചര്യത്തിലുള്ള ഏതു മനുഷ്യനെ അതേ അനുഭവങ്ങളിലൂടെ കൊണ്ടുപോയാലും ഇതേ കാര്യങ്ങള്‍ തന്നെയാണ് സംഭവിക്കുക. മുഹമ്മദ് നബി (സ) മലക്കാണെന്നോ ദൈവമാണെന്നോ അല്ല മുസ്‌ലിംകള്‍ വിശ്വസിക്കുന്നത്; മനുഷ്യനാണെന്നു തന്നെയാണ്. ആ മനുഷ്യന് ദിവ്യബോധനങ്ങള്‍ ലഭിച്ചു എന്നാണ് മുസ്‌ലിംകള്‍ പറയുന്നത്. അതിനെ ഖണ്ഡിക്കുവാനുതകുന്ന യാതൊന്നും പ്രവാചകന്റെ ഹിറാ അനുഭവങ്ങളിലില്ല തന്നെ. ക്വുര്‍ആന്‍ പറയട്ടെ: ”( നബിയേ,) പറയുക: ഞാന്‍ നിങ്ങളെപ്പോലെയുള്ള ഒരു മനുഷ്യന്‍ മാത്രമാകുന്നു. നിങ്ങളുടെ ദൈവം ഏകദൈവം മാത്രമാണെന്ന് എനിക്ക് ബോധനം നല്‍കപ്പെടുന്നു. അതിനാല്‍ വല്ലവനും തന്റെ രക്ഷിതാവുമായി കണ്ടുമുട്ടണമെന്ന് ആഗ്രഹിക്കുന്നുവെങ്കില്‍ അവന്‍ സല്‍കര്‍മ്മം പ്രവര്‍ത്തിക്കുകയും, തന്റെ രക്ഷിതാവിനുള്ള ആരാധനയില്‍ യാതൊന്നിനെയും പങ്കുചേര്‍ക്കാതിരിക്കുകയും ചെയ്തുകൊള്ളട്ടെ.” (18: 110)

ജിബ്‌രീലിന്റെ ആഗമനം പ്രവാചകനില്‍ നിറച്ച ഭയവും അത് സൃഷ്ടിച്ച ശാരീരിക പ്രതിഫലനങ്ങളും ബൈബിള്‍ വിവരിക്കുന്ന പ്രവാചകാനുഭവങ്ങളുമായി ചേര്‍ന്നുപോകുന്നതല്ല എന്ന മിഷനറി വാദത്തിന്റെ (അങ്ങനെ ചേര്‍ന്നുപോകുക ഒരാളുടെ പ്രവാചകത്വം തെളിയിക്കുവാന്‍ ഒരു നിലയ്ക്കും ആവശ്യമല്ലെന്ന് നാം സൂചിപ്പിച്ചുകഴിഞ്ഞു) വസ്തുനിഷ്ഠതയാണ് ഇനി പരിശോധിക്കപ്പെടേണ്ടത്. ജിബ്‌രീല്‍ (ഗബ്രിയേല്‍) മലക്കുകളില്‍ ശക്തികൊണ്ടും ഗാംഭീര്യംകൊണ്ടും വേറിട്ടുനില്‍ക്കുന്നയാളാണെന്ന സങ്കല്‍പം ബൈബിള്‍ പ്രദാനം ചെയ്യുന്നുണ്ട്. പ്രശസ്തമായ ബൈബിള്‍ ഓണ്‍ലൈന്‍ വിജ്ഞാനകോശം www.newadvent.org പറയട്ടെ: ”ഗബ്രിയേല്‍, പേര് സൂചിപ്പിക്കുന്നതുപോലെ ദൈവത്തിന്റെ ശക്തിയുടെ ദൂതനാണ്. ഗബ്രിയേലിനെ പരാമര്‍ശിക്കുന്ന ബൈബിള്‍ വചനങ്ങളില്‍ മഹത്വത്തെയും ശക്തിയെയും അധികാരത്തെയും ബലത്തെയുമെല്ലാം സൂചിപ്പിക്കുന്ന പദങ്ങള്‍ നിരന്തരമായി ആവര്‍ത്തിക്കുന്നത് ശ്രദ്ധേയമാണ്… ജൂതന്‍മാര്‍ ഗബ്രിയേലിന്റെ ഈ വിശേഷണങ്ങളെ ആഴത്തില്‍ ഉള്‍ക്കൊണ്ടിട്ടുള്ളതുപോലെയാണ് തോന്നുന്നത്… സൊദോമിന്റെ നാശം ഗബ്രിയേലിന്റെ കൈകളിലൂടെയാണുണ്ടായതെന്ന് അവര്‍ മനസ്സിലാക്കുന്നു.” അതിശക്തനും ഗംഭീരഭാവമുള്ളയാളും ഒരു പ്രദേശത്തെയൊന്നടങ്കം നശിപ്പിക്കുവാന്‍ ദൈവം നിയോഗിച്ച ബലവാനുമെല്ലാമായാണ് ബൈബിള്‍ പഴയനിയമം ജിബ്‌രീലിനെ അവതരിപ്പിക്കുന്നതെന്ന് സാരം. ഹിറാ ഗുഹയില്‍ പ്രവാചകനുമുന്നില്‍ ജിബ്‌രീല്‍ പ്രത്യക്ഷപ്പെട്ടതിനെത്തുടര്‍ന്നുണ്ടായ സംഭവങ്ങളെ ന്യായീകരിക്കുക മാത്രമല്ലേ ഈ ബൈബിള്‍ വര്‍ണനകള്‍ ചെയ്യുന്നത്?

ഇനി ജിബ്‌രീല്‍ പ്രവാചകന്‍മാര്‍ക്ക് പ്രത്യക്ഷപ്പെട്ടതിനെ സംബന്ധിച്ച ബൈബിള്‍ വിവരണങ്ങളിലേക്കു വരാം. ഗബ്രിയേല്‍ ദൂതനുമായി ആശയവിനിമയം നടത്തിയ ഏറ്റവും ശ്രദ്ധേയനായ പഴയനിയമ കഥാപാത്രം ദാനിയേല്‍ ആണ്. എന്തായിരുന്നു ദാനിയേലിന്റെ അനുഭവം? ഭാവിയെക്കുറിച്ചുള്ള ഭീഷണമായ സുചനകളുള്‍ക്കൊള്ളുന്നുവെന്ന് തോന്നിപ്പിക്കുന്ന വിചിത്രദൃശ്യങ്ങളുള്ള ഒരു ദര്‍ശനം ദാനിയേലിനുണ്ടാകുന്നു. ദര്‍ശനത്തിന്റെ ആഘാതത്തില്‍ നില്‍ക്കവെ ഗബ്രിയേല്‍ മനുഷ്യരൂപത്തില്‍ ദാനിയേലിനു പ്രത്യക്ഷപ്പെട്ടതിനെക്കുറിച്ച് ബൈബിള്‍ പറയട്ടെ: ”ദാനിയേലായ ഞാന്‍ ഈ ദര്‍ശനം ഗ്രഹിക്കാന്‍ ശ്രമിച്ചുകൊണ്ടിരിക്കെ, ഇതാ എന്റെ മുമ്പില്‍ മനുഷ്യരൂപമുള്ള ഒരുവന്‍ നില്‍ക്കുന്നു. ഉലായ് തീരങ്ങളില്‍നിന്ന് ഒരുവന്‍ വിളിച്ചുപറയുന്നത് ഞാന്‍ കേട്ടു; ഗബ്രിയേല്‍, ദര്‍ശനം ഇവനെ ഗ്രഹിപ്പിക്കുക. ഞാന്‍ നിന്നിടത്തേക്ക് അവന്‍ (ഗബ്രിയേല്‍) വന്നു. അവന്‍ വന്നപ്പോള്‍ ഞാന്‍ ഭയവിഹ്വലനായി സാഷ്ടാംഗം വീണു… അവന്‍ എന്നോട് സംസാരിച്ചുകൊണ്ടിരിക്കുമ്പോള്‍ ഞാന്‍ മൂര്‍ഛിച്ചുവീണു. എന്നാല്‍ അവന്‍ എന്നെ തൊട്ട് എഴുന്നേല്‍പിച്ചു നിര്‍ത്തി.” (ദാനിയേല്‍ 8: 15-18)

മറ്റൊരിക്കല്‍ ഗബ്രിയേല്‍ ടൈഗ്രീസ് തീരത്തുവെച്ച് ദാനിയേലിനു പ്രത്യക്ഷപ്പെട്ടതിനെക്കുറിച്ച് പത്താം അധ്യായത്തിലുണ്ട്. അതിപ്രാകാരം: ”ഞാന്‍ കണ്ണുയര്‍ത്തി നോക്കിയപ്പോള്‍ ചണവസ്ത്രവും ഊഫാസിലെ സ്വര്‍ണം കൊണ്ടുള്ള അരപ്പട്ടയും ധരിച്ച ഒരുവനെ കണ്ടു. അവന്റെ ശരീരം ഗോമേദകം പോലെയും മുഖം മിന്നല്‍ പോലെയും കണ്ണുകള്‍ ജ്വലിക്കുന്ന ഒരു പന്തം പോലെയും ആയിരുന്നു. അവന്റെ കൈകാലുകള്‍ മിനുക്കിയ ഓടിന്റെ ഭംഗിയുള്ളവയും സ്വരം ജനക്കൂട്ടത്തിന്റെ ഇരമ്പല്‍ പോലെയും ആയിരുന്നു. ദാനിയേലായ ഞാന്‍ മാത്രം ഈ ദര്‍ശനം കണ്ടു; എന്നോടൊപ്പമുണ്ടായിരുന്നവര്‍ അതു കണ്ടില്ല. മഹാഭീതി പിടിപെട്ട് അവര്‍ ഓടിയൊളിച്ചു. അങ്ങനെ തനിച്ചായ ഞാന്‍ ഈ മഹാദര്‍ശനം കണ്ടു; എന്റെ ശക്തി ചോര്‍ന്നുപോയി. എന്റെ മുഖം തിരിച്ചറിയാന്‍ വയ്യാത്തവിധം മാറിപ്പോയി. എന്റെ ശക്തിയറ്റു. അപ്പോള്‍ ഞാന്‍ അവന്റെ സ്വരം കേട്ടു, അവന്റെ സ്വരം ശ്രവിച്ച ഞാന്‍ പ്രജ്ഞയറ്റ് നിലം പതിച്ചു. എന്നാല്‍, ഒരു കരം എന്നെ സ്പര്‍ശിച്ചു. അവന്‍ എന്നെ എഴുന്നേല്‍പിച്ചു. വിറയലോടെയാണെങ്കിലും മുട്ടും കയ്യും ഊന്നി ഞാന്‍ നിന്നു. അവന്‍ എന്നോട് പറഞ്ഞു: ഏറ്റവും പ്രിയങ്കരനായ ദാനിയേലേ, എഴുന്നേല്‍ക്കുക. ഞാന്‍ നിന്നോട് പറയുന്ന വാക്കുകള്‍ ശ്രദ്ധിച്ചുകേള്‍ക്കുക. എന്നെ നിന്റെയടുത്തേക്ക് അയച്ചിരിക്കുകയാണ്. അവന്‍ ഇതുപറഞ്ഞപ്പോള്‍ ഞാന്‍ വിറയലോടെ നിവര്‍ന്നുനിന്നു.” (ദാനിയേല്‍ 10: 5 – 11)

ഹിറയിലുണ്ടായ വെളിപാടിന്റെ ദൈവികതയെ നിഷേധിക്കാന്‍ ബൈബിളുപയോഗിച്ച് കഴിയില്ലെന്നും പ്രവാചകാനുഭവങ്ങളെ സാധൂകരിക്കുക മാത്രമാണ് ബൈബിള്‍ ഈ വിഷയത്തില്‍ ചെയ്യുന്നത് എന്നുമുള്ള കേവല വസ്തുതകളാണ് ഇവിടെ അനാവൃതമാകുന്നത്. ഗബ്രിയേലുമായുള്ള മനുഷ്യമുഖാമുഖത്തെക്കുറിച്ച് ബൈബിള്‍ പുതിയ നിയമവും ഇതേദിശയിലുള്ള സൂചനകള്‍ നല്‍കുന്നുണ്ട്. യോഹന്നാന്‍ സ്‌നാപകന്റെ പിതാവ് സെഖര്യാവിന്റെ അനുഭവങ്ങള്‍ ശ്രദ്ധിക്കുക: ”അപ്പോള്‍, കര്‍ത്താവിന്റെ ദൂതന്‍ ധൂപപീഠത്തിന്റെ വലതുവശത്ത് നില്‍ക്കുന്നതായി അവന് പ്രത്യക്ഷപ്പെട്ടു. അവനെക്കണ്ട് സഖറിയാ അസ്വസ്ഥനാവുകയും ഭയപ്പെടുകയും ചെയ്തു.” (ലൂക്കോസ് 1: 11, 12). പ്രവാചകന്‍ (സ) ഭയവിഹ്വലനായത് ചൂണ്ടിക്കാണിച്ച് അദ്ദേഹത്തിനുണ്ടായത് വഹ്‌യല്ലെന്നും വന്നത് മലക്കല്ലെന്നുമെല്ലാം സമര്‍ത്ഥിക്കുവാന്‍ പാടുപെടുന്ന മിഷനറിമാര്‍ തങ്ങളുടെ സ്വന്തം ബൈബിളിന്റെയടക്കം കഴുത്തിലാണ് ഈ അഭ്യാസപ്രകടനങ്ങള്‍ക്കിടയില്‍ കത്തിവെക്കുന്നതെന്നര്‍ത്ഥം.

ബൈബിള്‍ ശരിയായ രീതിയില്‍ വായിച്ചവര്‍ക്കൊന്നും മുഹമ്മദ് നബി(സ)യ്ക്ക് ഹിറാ ഗുഹയില്‍ വെച്ചുണ്ടായ വെളിപാടിനെ സാക്ഷീകരിക്കുവാനല്ലാതെ നിരാകരിക്കാന്‍ കഴിയില്ലെന്ന വസ്തുതയാണ് ഹദീഥിന്റെ അവസാന ഭാഗത്തുള്ള വറക്വത്ബ്‌നു നൗഫലിന്റെ വാചകങ്ങള്‍ തെളിയിക്കുന്നത്. ജൂത-ക്രിസ്തു ദര്‍ശനങ്ങളോട് ആഭിമുഖ്യവും അവരുടെ ഗ്രന്ഥങ്ങളില്‍ അഗാധപരിജ്ഞാനവുമുണ്ടായിരുന്ന വറക്വ, നബി(സ)യുടെ വെളിപാടനുഭവങ്ങള്‍ കേട്ടപ്പോള്‍ അത് ജിബ്‌രീല്‍ തന്നെയാണെന്ന് സാക്ഷ്യപ്പെടുത്തുകയും നബി(സ)ക്ക് പിന്തുണ പ്രഖ്യാപിക്കുകയുമാണ് ചെയ്തതെന്ന യാഥാര്‍ത്ഥ്യം മിഷനറിമാരുടെ കണ്ണുതുറപ്പിക്കേണ്ടതുണ്ട്. അതെ, ഖദീജ (റ) ആണയിട്ടു പറഞ്ഞതുപോലെ ബന്ധുക്കള്‍ക്കും ദുര്‍ബലര്‍ക്കും ദരിദ്രര്‍ക്കും അശരണര്‍ക്കും അതിഥികള്‍ക്കും പ്രയാസപ്പെടുന്നവര്‍ക്കും തണല്‍മരമായി നിന്നിരുന്ന മുഹമ്മദ് (സ) എന്ന നന്മകളുടെ ഉടല്‍ രൂപത്തെ പ്രവാചകനായി നിയോഗിക്കുവാന്‍ ജിബ്‌രീല്‍ എന്ന മലക്കു തന്നെയാണ് ഒന്നര സഹസ്രാബ്ദത്തോളം മുമ്പ് ഹിറാ ഗുഹയില്‍ പ്രത്യക്ഷപ്പെട്ടത്; മുഹമ്മദ് നബി (സ) ജിബ്‌രീലിനോട് സ്വയം സാക്ഷ്യപ്പെടുത്തിയതുപോലെ എഴുത്തോ വായനയോ അറിയാതിരുന്ന, വേദപുസ്തക പരിജ്ഞാനം അശേഷമില്ലാതിരുന്ന ആ സാധാരണക്കാരന്റെ നാവില്‍ നിന്ന് ലോകം വിസ്മയത്തോടുകൂടി ക്വുര്‍ആന്‍ കേട്ടത് ജിബ്‌രീല്‍ അദ്ദേഹത്തിനത് പഠിപ്പിച്ചുകൊടുത്തതുകൊണ്ടാണ്. ഹിറയില്‍ നിന്ന് പ്രസരിച്ച വെളിച്ചത്തിന് ബൈബിള്‍ മറയാകുമെന്ന് കരുതുന്നവര്‍ വിഡ്ഢികളുടെ സ്വര്‍ഗത്തിലാണെന്നു മാത്രമാണ് നമുക്ക് പറയാനുള്ളത്.

വിശുദ്ധ ക്വുര്‍ആന്‍ പ്രപഞ്ചനാഥന്‍ പ്രവാചകന് അവതരിപ്പിച്ചുകൊടുത്തതാണെന്ന് അതിന്റെ ഉള്ളടക്കം തെളിയിക്കുന്നുവെന്ന മുസ്‌ലിംകളുടെ വാദം അടിസ്ഥാനരഹിതമാണ്. മുഹമ്മദ് നബി (സ) തനിക്ക് പരിചയമുണ്ടായിരുന്ന ജൂതക്രൈസ്തവരില്‍ നിന്ന് നേടിയെടുത്ത മതവിജ്ഞാനീയങ്ങള്‍ ക്വുര്‍ആന്‍ വചനങ്ങളാക്കി പുനരാവിഷ്‌കരിക്കുകയാണ് യഥാര്‍ത്ഥത്തില്‍ ചെയ്തത്. പ്രവാചകപൂര്‍വ കാലഘട്ടത്തില്‍ ബൈസന്റൈന്‍ സിറിയന്‍ പ്രവിശ്യകളിലൂടെ അദ്ദേഹം നടത്തിയ യാത്രകളില്‍ കണ്ടുമുട്ടിയ ക്രൈസ്തവ പണ്ഡിതന്‍മാരുടെ ശിഷ്യത്വമാണ് പ്രവാചകനെ ക്വുര്‍ആനിന്റെ 'രചന'ക്ക് പ്രാപ്തനാക്കിയതെന്ന് മനസ്സിലാക്കുവാന്‍ കഴിയും. ഓറിയന്റലിസ്റ്റുകളും മിഷനറിമാരും മുഹമ്മദ് നബി(സ)യുടെ പ്രവാചകത്വത്തെ നിഷേധിക്കുവാന്‍ വേണ്ടി ഉന്നയിക്കുന്ന ഈ വാദങ്ങള്‍ സത്യസന്ധമാണോ?

ല്ല. ഓറിയന്റലിസ്റ്റുകള്‍ കൊണ്ടുനടക്കുന്ന 'സിറിയന്‍ കഥകള്‍'ക്കൊന്നും -അവയുടെ ചരിത്രപരത വേറെ പരിശോധിക്കപ്പെടേണ്ടതാണ്- മുഹമ്മദ് നബി(സ)യുടെ പ്രവാചകത്വത്തെയോ ക്വുര്‍ആന്റെ ദൈവികതയെയോ ഒരു നിലക്കും ചോദ്യം ചെയ്യാനുളള കെല്‍പില്ലെന്നതാണ് വാസ്തവം. ക്വുര്‍ആനിലെ വചനങ്ങള്‍ പ്രപഞ്ചനാഥന്റേതു മാത്രമാണെന്ന് അവയുടെ ഉള്ളടക്കവും ശൈലിയും സുതരാം വ്യക്തമാക്കുന്നുണ്ട്. ആയിരം സിറിയന്‍ യാത്രകളും പതിനായിരം ക്രൈസ്തവപണ്ഡിതന്‍മാരുടെ ശിഷ്യത്വവുമുണ്ടായാലും ക്വുര്‍ആനിനെപ്പോലൊരു രചന നിര്‍വഹിക്കുവാന്‍ ഒരാള്‍ക്കും സാധ്യമല്ലെന്നതാണ് വാസ്തവം.

ബൈബിള്‍ വിജ്ഞാനീയങ്ങളിലുള്ള അവഗാഹമാണല്ലോ, ക്രൈസ്തവ സമ്പര്‍ക്കങ്ങള്‍ മുഖേന നേടിയെടുക്കുവാന്‍ കഴിയുമെന്ന് വിമര്‍ശകര്‍ കരുതുന്ന ക്വുര്‍ആന്‍ രചനക്കാവശ്യമായ 'ആയുധം'. ബൈബിളില്‍ പ്രവാചകന്‍മാരെയും വേദഗ്രന്ഥങ്ങളെയും മാനവചരിത്രത്തെയും സംബന്ധിച്ച് രേഖപ്പെടുത്തിയിട്ടുള്ള 'വിവരങ്ങള്‍' പ്രമാദമുക്തമാണെന്ന് ഓറിയന്റലിസ്റ്റുകള്‍ക്ക് വാദമുണ്ടോ? അബദ്ധങ്ങള്‍കൊണ്ട് 'സമൃദ്ധ'മായ പ്രസ്തുത 'വിവര'ങ്ങളായിരുന്നു ക്വുര്‍ആനിന്റെ അവവലംബമെങ്കില്‍ ബൈബിളിനെപ്പോലെത്തന്നെ ക്വുര്‍ആനും തെറ്റുകളുടെ ഒരു ഘോഷയാത്രയായിത്തീരുമായിരുന്നുവെന്നതാണ് വാസ്തവം. എന്നാല്‍ ഒരൊറ്റ അബദ്ധംപോലും വരുത്താതെയാണ് സെമിറ്റിക് പ്രവാചകന്‍മാരെക്കുറിച്ചും വേദഗ്രന്ഥങ്ങളെക്കുറിച്ചും ഇസ്രയേല്‍ വംശത്തിന്റെ നാള്‍വഴിയെക്കുറിച്ചുമെല്ലാം പരിശുദ്ധ ക്വുര്‍ആന്‍ സംസാരിക്കുന്നത്. ജൂത-ക്രൈസ്തവ പുരോഹിതന്‍മാരും ബൈബിളെഴുത്തുകാരും ചരിത്രാഖ്യാനത്തില്‍ വരുത്തിയ സ്ഖലിതങ്ങളൊന്നുപോലും പരിശുദ്ധ ക്വുര്‍ആനില്‍ കടന്നുവരുന്നില്ലെന്ന യാഥാര്‍ത്ഥ്യം തന്നെ, ക്വുര്‍ആനിക ഉളളടക്കത്തിന് ക്രൈസ്തവസ്രോതസ്സുകളെ സങ്കല്‍പിക്കുന്നത് എന്തുമാത്രം വലിയ അസംബന്ധമാണെന്ന് വ്യക്തമാക്കുന്നുണ്ട്.

ക്രൈസ്തവമോ ക്രൈസ്തവേതരമോ ആയ ഒരു വൈജ്ഞാനിക പാരമ്പര്യത്തിനും പരിശുദ്ധ ക്വുര്‍ആനിന്റെ ഉള്ളടക്കത്തിനുള്ള വിശദീകരണമായിത്തീരാന്‍ കഴിയില്ല. കാരണം മനുഷ്യകര്‍തൃത്വമുള്ള പരാമൃഷ്ട വിജ്ഞാനശേഖരങ്ങളിലെല്ലാം തന്നെ മനുഷ്യസഹജമായ അബദ്ധങ്ങളുടെ നിറസാന്നിദ്ധ്യമുണ്ട്; കാലഘട്ടത്തിന്റെയും പ്രദേശത്തിന്റെയും ബുദ്ധിശക്തിയുടെയും ഓര്‍മശേഷിയുടെയും സത്യസന്ധതയുടെയും പരിമിതകള്‍ക്കൊണ്ടുവന്ന സ്വാഭാവികമായ അബദ്ധങ്ങള്‍. എന്നാല്‍ വിശുദ്ധ ക്വുര്‍ആനില്‍ സെമിറ്റിക് പാരമ്പര്യത്തെക്കുറിച്ച് എന്നല്ല, ആറായിരത്തില്‍പരം വചനങ്ങളിലായി പരന്നുകിടക്കുന്ന പരശ്ശതം വിഷയങ്ങളെക്കുറിച്ചുള്ള പരാമര്‍ശങ്ങളും ഒരു ചെറിയ സ്ഖലിതം പോലും പേറുന്നില്ലെന്ന സത്യം ക്വുര്‍ആന്‍ മനുഷ്യരചനയല്ലെന്നും മനുഷ്യരുടെയൊന്നും സഹകരണം അത്തരമൊരു ഗ്രന്ഥത്തിന്റെ രചനക്ക് ഉപകാരപ്പെടുകയില്ലെന്നും അസന്നിഗ്ധമായി വ്യക്തമാക്കുന്നുണ്ട്. സര്‍വജ്ഞനും സൂക്ഷ്മജ്ഞനുമായ അല്ലാഹുവിനു മാത്രമേ പരിശുദ്ധ ക്വുര്‍ആന്‍ അവതരിപ്പിക്കുവാന്‍ കഴിയൂ എന്നാണ് അതിലെ വചനങ്ങളുടെയെല്ലാം കണിശമായ കൃത്യത നമ്മെ ബോധ്യപ്പെടുത്തുന്നത്.

ക്വുര്‍ആന്‍ ദൈവികമല്ലെന്ന് വാദിക്കുന്നവരെ ക്വുര്‍ആനില്‍ മനുഷ്യസഹജമായ അബദ്ധങ്ങള്‍ ചൂണ്ടിക്കാണിച്ച് തങ്ങളുടെ വാദം തെളിയിക്കുവാന്‍ ക്വുര്‍ആന്‍ തന്നെ വെല്ലുവിളിച്ചിട്ടുണ്ടെന്ന കാര്യം ശ്രദ്ധേയമാണ്: ''അവര്‍ ക്വുര്‍ആനിനെക്കുറിച്ചാലോചിക്കുന്നില്ലേ; അത് അല്ലാഹു അല്ലാത്തവരുടെ പക്കല്‍ നിന്നായിരുന്നുവെങ്കില്‍ അവര്‍ക്കതില്‍ ധാരാളം അബദ്ധങ്ങള്‍ കണ്ടെത്തുവാന്‍ കഴിയുമായിരുന്നല്ലോ!'' (ക്വുര്‍ആന്‍ 4 : 82).

ഒന്നര സഹസ്രാബ്ദത്തോളമായി ലോകത്ത് അജയ്യമായി നിലനില്‍ക്കുന്ന ഈ വെല്ലുവിളിയെ ഫലപ്രദമായി നേരിടാന്‍ കഴിയാത്തതുകൊണ്ടാണ് ഓറിയന്റലിസ്റ്റുകളും മിഷനറിമാരും 'സിറിയയില്‍ പോയി' മനസ്സമാധാനം കണ്ടെത്താന്‍ ശ്രമിക്കുന്നത്. ഇതുപോലെത്തന്നെയാണ് ക്വുര്‍ആനിന്റെ അനാദൃശമായ പ്രതിപാദന സൗകുമാര്യവും. അറേബ്യയിലോ സിറിയയിലോ റോമിലോ പേര്‍ഷ്യയിലോ യമനിലോ ഭാരതത്തിലോ ഗ്രീസിലോ എല്ലമായി അന്ന് ജീവിച്ചിരുന്ന സാഹിത്യസാമ്രാട്ടുകള്‍ക്കൊന്നും വികൃതാനുകരണങ്ങള്‍ക്കുപോലും ശ്രമിച്ചുനോക്കാന്‍ കഴിയാതിരുന്ന, ഇരുപത്തിമൂന്ന് വര്‍ഷക്കാലം കൊണ്ട് വലിയൊരു സമൂഹത്തെ അപ്പാടെ അനുയായികളായി നേടിയെടുത്ത തികച്ചും മൗലികമായ പരിശുദ്ധ ക്വുര്‍ആനിന്റെ ശൈലി സിറിയയില്‍ നിന്ന് മുഹമ്മദ് നബി(സ)ക്ക് 'വീണുകിട്ടി'യതാണെന്നു പറയാന്‍ അന്ധതയുടെ മൂര്‍ധന്യതയിലുള്ളവര്‍ക്കു മാത്രമേ കഴിയൂ! മഹാപണ്ഡിതന്‍മാരും സാഹിത്യകാരന്‍മാരും മതപുരോഹിതന്‍മാരും പകച്ചുനിന്നുപോയ ക്വുര്‍ആന്‍ വചനങ്ങള്‍ ദിവ്യമെന്ന് സത്യസന്ധര്‍ക്കൊക്കെയും ബോധ്യമാകുംവിധം നിരക്ഷരനായ ഒരു മനുഷ്യന്റെ നാവില്‍നിന്ന് വശ്യമനോഹരമായി നിര്‍ഗളിച്ചതിന് ഏതെങ്കിലും 'യാത്രകള്‍' വിശദീകരണമാകുമെന്ന് കരുതുന്നവര്‍ സഹതാപം പോലുമര്‍ഹിക്കുന്നില്ലെന്നതാണ് വാസ്തവം.

യാത്രകള്‍ പലതവണ ചെയ്തവരും മഹാപണ്ഡിതന്‍മാരുടെ ശിഷ്യത്വം സ്വീകരിച്ചവരും അനേകമായിരമുണ്ടായിട്ടുണ്ടല്ലോ ലോകത്ത്. അവര്‍ക്കാര്‍ക്കും ക്വുര്‍ആനിനെപ്പോലൊരു രചന സാധ്യമാകാത്തതെന്തുകൊണ്ടാണെന്ന് വിമര്‍ശകര്‍ വിശദീകരിക്കുമോ? ക്വുര്‍ആനിലൂടെ പ്രപഞ്ചനാഥന്‍ തന്നെ സംസാരിക്കട്ടെ: ''നമ്മുടെ ദാസന് നാം അവതരിപ്പിച്ചുകൊടുത്തതിനെ (വിശുദ്ധ ക്വുര്‍ആനിനെ)പറ്റി ന്റത് പോലുള്ള ഒരു അദ്ധ്യായമെങ്കിലും നിങ്ങള്‍ കൊണ്ടുവരിക. അല്ലാഹുവിന് പുറമെ നിങ്ങള്‍ക്കുള്ള സഹായികളേയും വിളിച്ചുകൊള്ളുക. നിങ്ങള്‍ സത്യവാന്‍മാരാണെങ്കില്‍ (അതാണല്ലോ വേണ്ടത്). നിങ്ങള്‍ക്കത് ചെയ്യാന്‍ കഴിഞ്ഞില്ലെങ്കില്‍ നിങ്ങള്‍ക്കത് ഒരിക്കലും ചെയ്യാന്‍ കഴിയുകയുമില്ല. മനുഷ്യരും കല്ലുകളും ഇന്ധനമായി കത്തിക്കപ്പെടുന്ന നരകാഗ്‌നിയെ നിങ്ങള്‍ കാത്തുസൂക്ഷിച്ചുകൊള്ളുക. സത്യനിഷേധികള്‍ക്കുവേണ്ടി ഒരുക്കിവെക്കപ്പെട്ടതാകുന്നു അത്.'' (ക്വുര്‍ആന്‍ 2 : 23-24)

പരിശുദ്ധ ക്വുര്‍ആനിന് ക്രൈസ്തവസ്രോതസ്സുകള്‍ ആരോപിക്കുവാനുള്ള അടിസ്ഥാനന്യായമായി ഈ ദിശയില്‍ സംസാരിച്ചിട്ടുള്ള വിമര്‍ശകരെല്ലാം ചൂണ്ടിക്കാണിച്ചിട്ടുള്ളത് ബൈബിള്‍ ചരിത്രത്തില്‍ നിന്നുള്ള ചില ഭാഗങ്ങള്‍ പരിശുദ്ധ ക്വുര്‍ആനിലും കടന്നുവരുന്നുവെന്നതാണ്. ദിവ്യവെളിപാടുകളെയും പ്രവാചകന്‍മാരെയും വേദഗ്രന്ഥങ്ങളെയും കുറിച്ച് സാമാന്യധാരണയെങ്കിലുമുള്ള മുഴുവനാളുകള്‍ക്കും ഈ പ്രതിരോധം അങ്ങേയറ്റം ദുര്‍ബലവും പരിഹാസ്യവുമാണെന്ന് വളരെയെളുപ്പത്തില്‍ മനസ്സിലാകും. ആദം മുതല്‍ യേശു വരെയുള്ള പൂര്‍വപ്രവാചകന്‍മാരുടെ പിന്‍ഗാമിയായി, അവരെയെല്ലാം നിയോഗിച്ച പടച്ചതമ്പുരാനില്‍ നിന്നുള്ള ദിവ്യവെളിപാടുകള്‍ പരിശുദ്ധ ക്വുര്‍ആനിന്റെ രൂപത്തില്‍ ഏറ്റുവാങ്ങിക്കൊണ്ടാണ് മുഹമ്മദ് നബി (സ) എന്ന അന്തിമ പ്രവാചകന്‍ ലോകത്ത് നിയുക്തനായത്. ഒരേ ദൈവത്തില്‍ നിന്ന് വ്യത്യസ്ത പ്രവാചകന്‍മാര്‍ക്ക് ലഭിച്ച ഉപദേശങ്ങളില്‍ സമാനതകള്‍ കാണുന്നതില്‍ എന്താണത്ഭുതം? അന്തിമ പ്രവാചകന് ലഭിച്ച വേദഗ്രന്ഥത്തില്‍ പൂര്‍വപ്രവാചകന്‍മാരെയും പൂര്‍വവേദങ്ങളെയും കുറിച്ച വിവരണങ്ങള്‍ കടന്നുവരുന്നതില്‍ വിചിത്രമായി എന്തുണ്ടെന്നാണ് മിഷനറിമാര്‍ കരുതുന്നത്? ബൈബിള്‍ പ്രവാചകന്‍മാരെക്കുറിച്ച് പൂര്‍ണമായ മൗനം പാലിച്ചുകൊണ്ടോ അവരെ നിഷേധിച്ചുകൊണ്ടോ ആണ് ക്വുര്‍ആന്‍ അവതരിച്ചിരുന്നത് എങ്കിലല്ലേ മിഷനറിമാര്‍ ക്വുര്‍ആനിന്റെ ദൈവികതയെ സംശയിക്കേണ്ടത്?

ബൈബിള്‍ പ്രവാചകന്‍മാരെ സംബന്ധിച്ച സത്യസന്ധമായ വിവരണങ്ങള്‍ നല്‍കുകയും അവരെ സംബന്ധിച്ച് ക്രൈസ്തവസമൂഹത്തില്‍ പ്രചരിച്ചിരുന്ന വ്യാജവാര്‍ത്തകള്‍ നിഷേധിക്കുകയും ചെയ്തുകൊണ്ട് പൂര്‍വപ്രവാചകന്‍മാരെ സംബന്ധിച്ച ആശയക്കുഴപ്പങ്ങള്‍ക്ക് ദൈവികമായ വിരാമം കുറിക്കുകയാണ് പരിശുദ്ധ ക്വുര്‍ആന്‍ അതിന്റെ പ്രവാചകകഥനങ്ങളിലൂടെ ചെയ്യുന്നത്. പ്രസ്തുതകഥനങ്ങളില്‍ അബദ്ധങ്ങളുണ്ടെന്ന് സ്ഥാപിച്ചുകൊണ്ടു മാത്രമേ ക്വുര്‍ആനിന്റെ ദൈവികതയെ നിരാകരിക്കുവാന്‍ കഴിയൂ. അവയുടെ സാന്നിദ്ധ്യം, ക്വുര്‍ആനിന്റെ ദൈവികതയെ അരക്കിട്ടുറപ്പിക്കുക മാത്രമാണ് ചെയ്യുക എന്നതത്രെ യാഥാര്‍ത്ഥ്യം. പൂര്‍വ വേദങ്ങളുമായുള്ള ബന്ധം വിശദീകരിക്കവെ പരിശുദ്ധ ക്വുര്‍ആന്‍ തന്നെ അത് ഈ രംഗത്ത് നിര്‍വഹിക്കുന്ന ദൗത്യത്തെ കൃത്യമായി നിര്‍വചിക്കുന്നുണ്ട്: ''(നബിയേ,) നിനക്കിതാ സത്യപ്രകാരം വേദഗ്രന്ഥം അവതരിപ്പിച്ച് തന്നിരിക്കുന്നു. അതിന്റെ മുമ്പുള്ള വേദഗ്രന്ഥങ്ങളെ ശരിവെക്കുന്നതും അവയെ കാത്തുരക്ഷിക്കുന്നതുമത്രെ അത്. അതിനാല്‍ നീ അവര്‍ക്കിടയില്‍ നാം അവതരിപ്പിച്ച് തന്നതനുസരിച്ച് വിധികല്‍പിക്കുക. നിനക്ക് വന്നുകിട്ടിയ സത്യത്തെ വിട്ട് നീ അവരുടെ തന്നിഷ്ടങ്ങളെ പിന്‍പറ്റിപോകരുത്. നിങ്ങളില്‍ ഓരോ വിഭാഗത്തിനും ഓരോ നിയമക്രമവും കര്‍മ്മമാര്‍ഗവും നാം നിശ്ചയിച്ച് തന്നിരിക്കുന്നു. അല്ലാഹു ഉദ്ദേശിച്ചിരുന്നെങ്കില്‍ നിങ്ങളെ അവന്‍ ഒരൊറ്റ സമുദായമാക്കുമായിരുന്നു. പക്ഷെ നിങ്ങള്‍ക്കവന്‍ നല്‍കിയിട്ടുള്ളതില്‍ നിങ്ങളെ പരീക്ഷിക്കുവാന്‍ (അവന്‍ ഉദ്ദേശിക്കുന്നു). അതിനാല്‍ നല്ല കാര്യങ്ങളിലേക്ക് നിങ്ങള്‍ മത്സരിച്ച് മുന്നേറുക. അല്ലാഹുവിങ്കലേക്കത്രെ നിങ്ങളുടെയെല്ലാം മടക്കം. നിങ്ങള്‍ ഭിന്നിച്ചിരുന്ന വിഷയങ്ങളെപ്പറ്റി അപ്പോഴവന്‍ നിങ്ങള്‍ക്ക് അറിയിച്ച് തരുന്നതാണ്.'' (5 : 48)

യഥാര്‍ത്ഥത്തില്‍ ഒരു പ്രവാചകന്‍ തനിക്കുമുമ്പുവന്ന പ്രവാചകന്‍മാരുടെ കഥ പറയുകയും അവരെ ഉദ്ധരിക്കുകയും അവരെ സംബന്ധിച്ച തെറ്റിദ്ധാരണകളെ തിരുത്തുകയുമെല്ലാം ചെയ്യുന്നത് തികച്ചും സ്വാഭാവികമാണെന്നും അദ്ദേഹത്തിന് ലഭിക്കുന്ന ദിവ്യവെളിപാടുകളുടെ പ്രധാനപ്പെട്ടൊരു ഭാഗം ഇത്തരം കാര്യങ്ങളായിരിക്കുമെന്നും ബൈബിള്‍ പണ്ഡിതന്‍മാര്‍ക്ക് ആരും പറഞ്ഞുകൊടുക്കേണ്ടതില്ല. അത്തരം ഒരു നൈരന്തര്യത്തിന്റെ വിശദമായ വിവരണമാണ് ബൈബിളിന്റെ മുഖ്യപ്രമേയങ്ങളിലൊന്നുതന്നെ. ഇസ്രയേല്‍ തറവാട്ടിലേക്ക് കടന്നുവന്ന ഓരോ പ്രവാചകനും തനിക്കുമുമ്പുള്ള പ്രവാചകന്‍മാരെയും പുസ്തകങ്ങളെയും പരാമര്‍ശിച്ചത് അവരൊന്നും പ്രവാചകന്‍മാരല്ലെന്നും പൂര്‍വിക പ്രവാചകന്‍മാരുടെ പ്രബോധനങ്ങള്‍ പഠിച്ചുമനസ്സിലാക്കി വ്യാജ പ്രവാചകത്വം അവകാശപ്പെട്ടവരാണെന്നുമുള്ളതിന്റെ തെളിവാണെന്ന് വിവേകമുള്ള ആരെങ്കിലും പറയുമോ? യേശുവിന്റെ സംസാരങ്ങള്‍ ദിവ്യപ്രചോദിതമല്ലെന്നും പഴയനിയമ പാഠത്തില്‍ നിന്ന് അദ്ദേഹം സ്വന്തമായി രൂപീകരിച്ചതാണെന്നും വാദിക്കുന്നവരോട് മിഷനറിമാരുടെ നിലപാടെന്തായിരിക്കും? ആ വാദത്തില്‍ നിന്ന് ഒരു നിലക്കും ഭിന്നമല്ല പരിശുദ്ധ ക്വുര്‍ആനിലെ സെമിറ്റിക് പ്രവാചകചരിത്രത്തെ മാത്രം ആധാരമാക്കിക്കൊണ്ടുള്ള മുഹമ്മദ് നബി(സ)യുടെ പ്രവാചകത്വനിഷേധമെന്നു മനസ്സിലാക്കുവാന്‍ സെമിനാരി വിദ്യാഭ്യാസത്തിന്റെ പോലും ആവശ്യമില്ലെന്നതല്ലേ സത്യം?

ബൈബിള്‍ പ്രവാചകന്‍മാരില്‍ മിക്കവരും ജീവിച്ചത് പ്രവാചകന്‍മാര്‍ നിരന്തരമായി കടന്നുവന്ന ഇസ്രാഈല്യര്‍ക്കു മധ്യത്തിലാണ്. മുമ്പുകടന്നുവന്ന പ്രവാചകന്‍മാരുടെ ഉപദേശങ്ങളും പുസ്തകങ്ങളും അവരെ സംബന്ധിച്ച പാരമ്പര്യങ്ങളുമെല്ലാം സമൂഹത്തിലെ ഏതാണ്ടെല്ലാവര്‍ക്കും ചിരപരിചിതമായിരുന്ന സാഹചര്യങ്ങളിലേക്കാണ് ഓരോ പുതിയ ബൈബിള്‍ പ്രവാചകനും കടന്നുവരുന്നത്. 'വിവരങ്ങള്‍ കട്ടതാണെന്ന്' ആരോപിക്കേണ്ടവര്‍ക്ക് ആവശ്യമായ എല്ലാ 'പശ്ചാത്തല സൗകര്യ'ങ്ങളും അവരുടെ ജിവിതങ്ങള്‍ നല്‍കുന്നുണ്ടെന്ന് ചുരുക്കം. എന്നാല്‍ മുഹമ്മദ് നബി(സ)യുടെ ജീവിതപരിസരം ഇതില്‍നിന്നും തീര്‍ത്തുംവിഭിന്നമാണ്. ഇശ്മയേലിനുശേഷം പ്രവാചകന്‍മാരുടെ നിയോഗമൊന്നുമുണ്ടായിട്ടില്ലാത്ത, വേദഗ്രന്ഥങ്ങളൊന്നും കയ്യിലില്ലാതിരുന്ന, തികഞ്ഞ വിഗ്രഹാരാധകരും അധാര്‍മികരുമായി ജീവിച്ച മക്കന്‍ അറബികള്‍ക്കിടയില്‍ ജനിച്ചുജീവിച്ച മുഹമ്മദ് നബി(സ)യാണ് പൂര്‍വപ്രവാചകന്‍മാരെ സംബന്ധിച്ച വര്‍ത്തമാനങ്ങള്‍, അതും ബൈബിള്‍ വരുത്തിയ അബദ്ധങ്ങളില്‍ നിന്നുപോലും മുക്തമായി തന്റെ പ്രബോധിത സമൂഹത്തിനുമുന്നില്‍ വെക്കുന്നത്.

പ്രവാചകന്‍ സമ്പൂര്‍ണ നിരക്ഷരനായിരുന്നുവെന്നും മതപരമോ ഭൗതികമോ ആയിട്ടുള്ള യാതൊരു വിദ്യാഭ്യാസവും അദ്ദേഹം നേടിയിട്ടില്ലെന്നുമുള്ള വസ്തുതകള്‍ നാം ഇതിനോട് ചേര്‍ത്തുവായിക്കണം. പ്രവാചകന്റെ കിറുകൃത്യമായ പൂര്‍വപ്രവാചകാപഥനങ്ങള്‍ ദിവ്യവെളിപാടുകളുടെ വെളിച്ചത്തിലുള്ളതായിരിക്കുവാന്‍ മാത്രമേ തരമുള്ളുവെന്ന് ഈ സാഹചര്യത്തെളിവുകള്‍ മുഴുവന്‍ വ്യക്തമാക്കുന്നുണ്ട്. പ്രപഞ്ചനാഥനില്‍ നിന്നുള്ള വെളിപാടുകള്‍ ഇല്ലായിരുന്നെങ്കില്‍ ഇത്തരം വിഷയങ്ങളില്‍ പ്രവാചകന്‍ (സ) തികഞ്ഞ അജ്ഞനായിത്തന്നെ തുടരുമായിരുന്നുവെന്ന് ക്വുര്‍ആന്‍ തന്നെ എടുത്തുപറയുവാനുള്ള കാരണങ്ങള്‍ ഇതെല്ലമായിരിക്കാം. ക്വുര്‍ആന്‍ പറയുന്നത് കാണുക : ''അലിഫ്-ലാം-റാ. സ്പഷ്ടമായ വേദഗ്രന്ഥത്തിലെ വചനങ്ങളാകുന്നു അവ. നിങ്ങള്‍ ഗ്രഹിക്കുന്നതിന് വേണ്ടി അത് അറബിഭാഷയില്‍ വായിക്കപ്പെടുന്ന ഒരു പ്രമാണമായി അവതരിപ്പിച്ചിരിക്കുന്നു. നിനക്ക് ഈ ഖുര്‍ആന്‍ ബോധനം നല്‍കിയത് വഴി ഏറ്റവും നല്ല ചരിത്രവിവരണമാണ് നാം നിനക്ക് നല്‍കിക്കൊണ്ടിരിക്കുന്നത്. തീര്‍ച്ചയായും ഇതിനുമുമ്പ് നീ അതിനെപ്പറ്റി ബോധമില്ലാത്തവനായിരുന്നു.'' (12 : 1-3)

ക്വുര്‍ആനിലെ പൂര്‍വപ്രവാചക വിവരണങ്ങള്‍ ഏതു ദൈവിക ഗ്രന്ഥത്തിന്റെയും സ്വാഭാവികത മാത്രമാണെന്നും സെമിറ്റിക് സാമൂഹിക പശ്ചാത്തലത്തിലല്ലാതെ വളര്‍ന്നുവന്ന മുഹമ്മദ് നബി(സ)ക്ക് പൂര്‍വപ്രവാചകന്‍മാരെ സംബന്ധിച്ച കൃത്യമായ അറിവുകള്‍ പ്രപഞ്ചനാഥനില്‍ നിന്നാണ് ലഭിച്ചതെന്നും സെമിറ്റിക് സമ്പര്‍ക്കങ്ങള്‍ കൊണ്ടുപോലും സ്വരൂപിക്കാനാവാത്തവിധം അന്യൂനമായ സെമിറ്റിക് പ്രവാചക കഥനമാണ് പരിശുദ്ധ ക്വുര്‍ആന്‍ നടത്തുന്നതെന്നും നാം മനസ്സിലാക്കി. ഇനി ഓറിയന്റലിസ്റ്റുകളുടെയും മിഷനറിമാരുടെയും 'സിറിയന്‍ സിദ്ധാന്ത'ത്തിലേക്കുവരാം. ഒരു സിറിയന്‍ കഥക്കും വഴങ്ങാത്തവിധം സുഭദ്രമാണ് ക്വുര്‍ആനിലെ ചരിത്രാഖ്യാനത്തിന്റെ കെട്ടുറപ്പും കൃത്യതയുമെന്നതുകൊണ്ടുതന്നെ, ഒരു വൈജ്ഞാനികാന്വേഷണം എന്ന നിലക്കുമാത്രമാണ് അവയെക്കുറിച്ചുള്ള അപഗ്രഥനം പ്രസക്തമാകുന്നത്. മുഹമ്മദ് നബി (സ) ജീവിച്ച മക്കയില്‍ ക്രൈസ്തവ സമൂഹത്തിന്റെയോ ജൂത സമൂഹത്തിന്റെയോ സാന്നിദ്ധ്യമുണ്ടായിട്ടില്ലെന്ന കാര്യം ചരിത്രകാരന്‍മാര്‍ക്കിടയില്‍ സുവിദിതമാണ്. ഹിജാസിന്റെ തെക്ക് യമനിലും വടക്ക് ശാമിലും (ഇന്നത്തെ സിറിയന്‍, ഫലസ്ത്വീന്‍ പ്രവിശ്യകള്‍) ചെങ്കടലിനക്കരെ ആഫ്രിക്കന്‍ ഉപഭൂഖണ്ഡത്തില്‍ എത്യോപ്യയിലുമാണ് പ്രവാചകകാലഘട്ടത്തില്‍ പ്രധാനമായും ക്രൈസ്തവരുടെ സാമൂഹികസാന്നിദ്ധ്യമുണ്ടായിരുന്നത്. ഇതില്‍ സിറയയില്‍ നിന്നുള്ള ചില ക്രൈസ്തവ പണ്ഡിതന്‍മാര്‍ പ്രവാചകന് ബൈബിള്‍ കഥകളില്‍ 'ട്യൂഷന്‍' നല്‍കിയതായാണ് ഓറിയന്റലിസ്റ്റുകളും മിഷനറിമാരും വാദിക്കുന്നത്. ചരിത്രപരമായ ഒരു രേഖയുമില്ലാത്ത തികഞ്ഞ ഒരു അപസര്‍പ്പകകഥയാണ് ഇതെന്ന വസ്തുത അവര്‍ സമര്‍ത്ഥമായി മറച്ചുവെക്കുകയും ചെയ്യും!

ഹിജാസില്‍ വന്ന് മതപ്രബോധനത്തിനായി തമ്പടിച്ച ചില ക്രൈസ്തവ പണ്ഡിതന്‍മാരുടെ ശിഷ്യത്വം സ്വീകരിച്ച് പ്രവാചകന്‍ കാലം കഴിച്ചതായുള്ള കള്ളക്കഥകളെഴുതിവെച്ച മധ്യകാല ക്രൈസ്തവപുരോഹിതന്‍മാര്‍ മുതല്‍ പ്രവാചകന്‍ നുബുവ്വത്തിന് മുമ്പ് ബൈസന്റൈന്‍ റോമിന്റെ സിറിയന്‍ പ്രവിശ്യകളില്‍ കാലങ്ങളോളം ആത്മീയാന്വേഷണത്തിനായി അലഞ്ഞുതിരിഞ്ഞതായും മധ്യധാരണാഴിയുടെ തീരം വരെ ചെന്നെത്തിയതായും ചിലപ്പോഴൊക്ക മധ്യധാരണാഴിയില്‍ കപ്പല്‍ യാത്ര വരെ നടത്തിയതായുമുള്ള വന്യമായ ഭാവനകളെ 'ചരിത്ര'മായി പ്രതിഷ്ഠിക്കുവാന്‍ ശ്രമിച്ച കൊളോണിയല്‍കാല ഓറിയന്റലിസ്റ്റുകള്‍ വരെ ഈ അപസര്‍പ്പക കഥാകാരന്‍മാരുടെ നീണ്ടനിരയിലുണ്ട്! മുഹമ്മദ് നബി (സ) കള്ള പ്രവാചകനാണെന്ന തങ്ങളുടെ വാദത്തെ പരിശുദ്ധ ക്വുര്‍ആനിന്റെ ചരിത്രാഖ്യാനത്തിന്റെ കൃത്യത കടപുഴക്കുന്നതായി ബോധ്യപ്പെട്ട ഓറിയന്റലിസ്റ്റ്-മിഷനറി കൂട്ടുകെട്ട്, പ്രസ്തുത കൃത്യതക്ക് വിശദീകരണം നല്‍കുവാന്‍ വേണ്ടി നടത്തിയ മസ്തിഷ്‌ക വ്യായാമങ്ങളുടെ സന്തതികളായിപ്പിറന്ന പെരുംകള്ളങ്ങള്‍ മാത്രമാണ് പ്രവാചകജീവിതത്തിലെ ഈ വ്യാജ സിറിയന്‍ അധ്യായങ്ങള്‍ മുഴുവനുമെന്നതാണ് വസ്തുത.

മുഹമ്മദ് നബി(സ)യുടെ ജീവിതത്തില്‍ തങ്ങളുടെ വകയായി എഴുതിച്ചേര്‍ത്ത ഈ സംഭവങ്ങള്‍ക്ക് ആധാരമായ ചരിത്രരേഖകളേതൊക്കെയാണെന്ന് വ്യക്തമാക്കുവാന്‍ ഓറിയന്റലിസ്റ്റുകളൊന്നും സന്നദ്ധമായിട്ടില്ല. അക്കാദമിക, വൈജ്ഞാനികാന്വേഷണത്തിന്റെ കിരീടം അഭിമാനപൂര്‍വം തലയിലണിയുന്നവര്‍ ചരിത്രപരത ലവലേശവുമില്ലാത്ത കല്‍പിത കഥകളില്‍ അഭിരമിക്കുന്നുണ്ടെങ്കില്‍, ക്വുര്‍ആനിന്റെ ദൈവികത നിഷേധിക്കുവാന്‍ അവരെന്തു കടുംകയ്യും ചെയ്യും എന്ന് മനസ്സിലാക്കാനേ നമുക്ക് കഴിയൂ. സത്യസന്ധമായ മാര്‍ഗങ്ങളിലൂടെ ക്വുര്‍ആനിന്റെ ശോഭ കെടുത്താനാകില്ലെന്ന് തിരിച്ചറിഞ്ഞവര്‍ കല്ലുവെച്ച നുണകള്‍ ബോധപൂര്‍വം പറഞ്ഞ് അതിനെ തമസ്‌കരിക്കുവാന്‍ ശ്രമിക്കുമ്പോള്‍ ക്വുര്‍ആനിന്റെ അജയ്യതക്കും ഔജല്യത്തിനുമാണ് അടിവരയിടപ്പെടുന്നത് എന്നതാണ് സത്യം. ഒരാളെക്കുറിച്ച്, അദ്ദേഹത്തെ തോല്‍പിക്കുവാന്‍ വേണ്ടി, ഭാവനയില്‍ വരുന്ന എന്ത് കള്ളക്കഥയും പടച്ചുണ്ടാക്കുന്നതിന്റെ പേരാണ് ചരിത്രമെഴുത്തെന്ന് കരുതുന്നവരുടെ വ്യവഹാരങ്ങള്‍ അക്കാദമിക പഠനങ്ങളുടെ ഗതി നിശ്ചയിക്കുന്ന വൈചിത്ര്യത്തിനുമുന്നില്‍ പരിശുദ്ധ ക്വുര്‍ആന്‍ പറഞ്ഞതുമാത്രമാണ് നമുക്കും പറയാനുള്ളത്. ''അവരുടെ വായ്‌കൊണ്ട് അല്ലാഹുവിന്റെ പ്രകാശം കെടുത്തിക്കളയാമെന്ന് അവര്‍ ആഗ്രഹിക്കുന്നു. അല്ലാഹുവാകട്ടെ,തന്റെ പ്രകാശം പൂര്‍ണ്ണമാക്കാതെ സമ്മതിക്കുകയില്ല. സത്യനിഷേധികള്‍ക്ക് അത് അനിഷ്ടകരമായാലും. അവനാണ് സന്മാര്‍ഗവും സത്യമതവുമായി തന്റെ ദൂതനെ അയച്ചവന്‍. എല്ലാ മതത്തെയും അത് അതിജയിക്കുന്നതാക്കാന്‍ വേണ്ടി. ബഹുദൈവവിശ്വാസികള്‍ക്ക് അത് അനിഷ്ടകരമായാലും.'' (ക്വുര്‍ആന്‍ 9 : 32-33)

മുഹമ്മദ് നബി(സ)ക്ക് തന്റെ ജീവിതത്തില്‍ രണ്ട് സിറിയന്‍ യാത്രകള്‍ നടത്തിയതായി മാത്രമാണ് നിവേദനങ്ങളുള്ളത്. അതിലൊന്ന്, ഒന്‍പത് വയസ്സിലും പന്ത്രണ്ട് വയസ്സിനുമിടക്കെപ്പോഴോ അബൂത്വാലിബിന്റെ കൂടെ ഒരു കച്ചവടയാത്രയില്‍ മുഹമ്മദ് (സ) സിറിയയിലെ ബുസ്വ്‌റ വരെ പോയി എന്നു പറയുന്ന നിവേദനങ്ങളാണ്. ഇബ്‌നു ഇസ്ഹാഖ് അദ്ദേഹത്തിന്റെ സീറത്തു റസൂലില്ലയില്‍ ഈ കഥ പറയുന്നുണ്ട്. എന്നാല്‍ യാതൊരു നിവേദകപരമ്പരയും ഉദ്ധരിക്കാതെയാണ് അദ്ദേഹം കഥ എടുത്തുചേര്‍ത്തിട്ടുള്ളത് എന്നതിനാല്‍ തന്നെ നിവേദനങ്ങളുടെ നിദാനശാസ്ത്രമനുസരിച്ച് അദ്ദേഹത്തിന്റെ വര്‍ത്തമാനത്തെ ആധികാരികമായി പരിഗണിക്കുവാന്‍ യാതൊരു നിര്‍വാഹവുമില്ല. ഇമാം തുര്‍മുദി തന്റെ ജാമിഇലും ഇമാം ഹാകിം തന്റെ മുസ്തദ്‌റകിലും ഉദ്ധരിച്ചിട്ടുള്ള ഹദീഥുകളാണ് ഈ കഥക്കാധാരമായി പിന്നെയുള്ളത്. രണ്ട് ഹദീഥുകളുടെയും നിവേദകപരമ്പരകള്‍ അനേകം ദൗര്‍ബല്യങ്ങള്‍ ചൂണ്ടിക്കാണിക്കപ്പെട്ടിട്ടുള്ളതും അവ കാരണമായിത്തന്നെ പണ്ഡിതന്‍മാരാല്‍ സംശയാസ്പദമായി പരിഗണിക്കപ്പെട്ടിട്ടുള്ളവയുമാണ്. എന്നാല്‍ തുര്‍മുദി രേഖപ്പെടുത്തിയ ഹദീഥ് ചില പണ്ഡിതന്‍മാരെങ്കിലും ഹസന്‍ ആയോ സ്വഹീഹ് ആയോ എണ്ണിയിട്ടുണ്ടെന്ന കാര്യം ശരിയാണ്. ഹദീഥില്‍ പറയുന്ന സിറിയന്‍ യാത്രക്ക് സാക്ഷിയായിട്ടില്ലാത്ത അബൂമൂസല്‍ അശ്അരി (റ) എവിടെ നിന്നാണ് തനിക്കീ വിവരം കിട്ടിയത് എന്നുപറയാതെ പരാമൃഷ്ട കഥ പറയുന്നതാണ് ഹദീഥിലുള്ളത്. ഹദീഥിന്റെ നിവേദകപരമ്പരയെ വിമര്‍ശിച്ച പണ്ഡിതന്‍മാരുടെ അഭിപ്രായങ്ങളെ തള്ളിക്കളഞ്ഞാല്‍ പോലും, അബൂമൂസല്‍ അശ്അരി ഇവ്വിഷയകമായി കേട്ടകാര്യം എന്നുമാത്രമേ ഹദീഥിലെ കഥയെക്കുറിച്ച് പരമാവധി പറയാന്‍ പറ്റൂവെന്നര്‍ത്ഥം. അത്തരമൊരു 'കേള്‍വി' മാത്രമായതുകൊണ്ടുതന്നെ, ഒരുപാടു പൊരുത്തക്കേടുകളും അസംഭവ്യതകളും ഹദീഥിലെ കഥാകഥനത്തിലുള്ളതായി നിരൂപകര്‍ ചൂണ്ടിക്കാണിച്ചിട്ടുണ്ട്. ഈ സംശയാസ്പദമായ കേള്‍വി മാത്രമാണ് പ്രവാചകനെക്കുറിച്ച് പറയപ്പെടുന്ന ഒന്നാം സിറിയന്‍ യാത്രയെക്കുറിച്ച് ആകെക്കൂടി ചരിത്രത്തിലുള്ളത്.

ഇനി, എന്താണ് അബൂമൂസല്‍ അശ്അരി കേട്ടിട്ടുള്ളതെന്നുകൂടി നാം പരിശോധിക്കുക. അബൂത്വാലിബും സംഘവും സാധാരണയായി മക്കയില്‍ നിന്നുള്ള കച്ചവടസംഘങ്ങള്‍ കടന്നുപോകാറുള്ള ഒരു ക്രൈസ്തവമഠത്തിനുമുന്നിലെത്തിയപ്പോള്‍ അസാധാരണമാംവിധം അവിടുത്തെ മുഖ്യപുരോഹിതന്‍ പുറത്തേക്കിറങ്ങിവന്നുവെന്നും ബാലനായ മുഹമ്മദ് നബി(സ)യില്‍ ഭാവി പ്രവാചകന്റെ അടയാളങ്ങള്‍ ദര്‍ശിച്ചുവെന്നും അബൂത്വാലിബിനോട് ഈ കുട്ടിയെ റോമിലേക്ക് കൂട്ടുന്നത് അപകടമായിരിക്കുമെന്ന് പറയുകയും തിരിച്ചയക്കാന്‍ നിര്‍ബന്ധിക്കുകയും ചെയ്തുവെന്നും മുഹമ്മദ് നബി (സ) കച്ചവടസംഘത്തില്‍ നിന്ന് വേര്‍പിരിഞ്ഞ് അബൂത്വാലിബ് കൂടെ പറഞ്ഞയച്ച രണ്ട് പേരോടൊപ്പം അവിടെനിന്ന് മക്കയിലേക്ക് മടങ്ങിയെന്നും മാത്രമാണ് ഹദീഥിലുള്ളത്. തിരെ ചെറിയ പ്രായത്തില്‍ വളരെ കുറഞ്ഞൊരു സമയം ഒരു പുരോഹിതനെ കണ്ടുവെന്ന് മാത്രമാണ് ഹദീഥ് -അതില്‍ പറഞ്ഞ കാര്യങ്ങള്‍ ശരിയാണെങ്കില്‍- ആകെക്കൂടി നല്‍കുന്ന വിവരം. പ്രവാചകന് ക്രൈസ്തവപുരോഹിതന്‍മാര്‍ മതാധ്യാപനങ്ങള്‍ നല്‍കി എന്ന് ഹദീഥില്‍ എവിടെയാണുള്ളത്? സമൂഹത്തില്‍ ജീവിച്ച ഒരു വ്യക്തി എന്ന നിലക്ക് പ്രവാചകന്‍ പലരെയും കാണാനിടവന്ന നിലയില്‍ ഒരു ക്രൈസ്തവപുരോഹിതനെയും കണ്ടുമുട്ടി എന്ന, പ്രവാചകജീവിതത്തെ സംബന്ധിച്ച് ഒരു നിലക്കും പ്രസക്തമല്ലാത്ത ഒരറിവു മാത്രമാണ് ഹദീഥ് പങ്കുവെക്കുന്നതെന്ന് ചരുക്കും.

അതോടൊപ്പം, ഒരു ക്രൈസ്തവപുരോഹിതന്‍ ബൈബളില്‍ നിന്ന് ലഭിച്ച സൂചനകളുടെ വെളിച്ചത്തില്‍ മുഹമ്മദിനെ (സ) ചെറു്രപായത്തില്‍ തന്നെ ഭാവിപ്രവാചകനായി തിരിച്ചറിഞ്ഞു എന്ന അറിവുകൂടി ഹദീഥ് അവശേഷിപ്പിക്കുന്നു. വാസ്തവത്തില്‍, പ്രവാചകജീവിതവുമായി ബന്ധപ്പെട്ട് പരാമൃഷ്ട ഹദീഥിലുള്ള ഏറ്റവും പ്രസക്തമായ അറിവ് അതാണ്. മുഹമ്മദ് നബി (സ) ക്രൈസ്തവ പുരോഹിതനെ ഗുരുവായി സ്വീകരിച്ചുവെന്നല്ല, പ്രത്യുത മുഹമ്മദ് നബി(സ)യെ ലോകഗുരുവായി പുരോഹിതന്‍ അംഗീകരിച്ചുവെന്നാണ് ഹദീഥിലുള്ളത്. പ്രവാചകബാല്യത്തില്‍ നടന്നുവെന്ന് പറയപ്പെടുന്ന സിറിയന്‍ യാത്രയെ സംബന്ധിച്ച എല്ലാ നിവേദനങ്ങളും ഇക്കാര്യം ഐകകണ്‌ഠേന പറയുന്നുണ്ട്. ആ നിവേദനങ്ങള്‍ സ്വീകാര്യമാണെന്നണ് ഓറിയന്റലിസ്റ്റുകളുടെ പക്ഷമെങ്കില്‍, അവര്‍ പറയാനുദ്ദേശിക്കുന്നതിന് നേര്‍വിപരീതമായ കഥയാണ് അവ ഉല്‍പാദിപ്പിക്കുക എന്നുചുരുക്കം. സിറിയയില്‍വെച്ച് കണ്ടുമുട്ടിയ ക്രൈസ്തവ പാതിരിയുടെ പാത പിന്തുടര്‍ന്ന് പ്രവാചകനെ അംഗീകരിക്കുവാന്‍ മിഷനറിമാര്‍ സന്മനസ്സ് കാണിക്കുമോ? ബൈബിളില്‍ നബിയെക്കുറിച്ച പ്രവചനങ്ങളുണ്ടെന്ന വസ്തുതയെ അവര്‍ ഉള്‍ക്കൊള്ളുമോ ?

ഖദീജ(റ)യുമായുള്ള വിവാഹത്തിന് തൊട്ടുമുമ്പ് അവരുടെ മയ്‌സറ എന്ന ഭൃത്യന്റെ കൂടെ ഖദീജയുടെ കച്ചവടവസ്തുക്കളുമായി മുഹമ്മദ് നബി (സ) ശാമിലേക്ക് ഒരു യാത്രപോയതാണ് ചരിത്രഗ്രന്ഥങ്ങളിലുള്ള രണ്ടാമത്തെ സംഭവം. ഈ സംഭവം കുറേക്കൂടി പ്രശസ്തവും ചരിത്രകാരന്‍മാര്‍ അംഗീകരിക്കുന്നതുമാണ്; വിശദാംശങ്ങളെക്കുറിച്ച് അഭിപ്രായവ്യത്യാസങ്ങളുണ്ടെങ്കിലും. എന്നാല്‍ എല്ലാ അഭിപ്രായവ്യത്യാസങ്ങളെയും തള്ളിക്കളഞ്ഞ് മുഴുവന്‍ ചരിത്രകാരന്‍മാരുടെയും നിവേദനങ്ങളെ ആധികാരികമായി നാം സ്വീകരിച്ചാലും ആ യാത്രയില്‍ ഇവ്വിഷയകമായുണ്ടായത് ഒരു ക്രൈസ്തവപുരോഹിതനും ഒരു ജൂതവ്യാപാരിയും ഭാവി പ്രവാചകന്റെ അടയാളങ്ങള്‍ നബി(സ)യില്‍ ദര്‍ശിച്ചുവെന്നതു മാത്രമാണ്. ഒന്നാം സിറിയന്‍ യാത്രയെക്കുറിച്ചു പറഞ്ഞതുപോലെ, രണ്ടാം സിറിയന്‍ യാത്രയിലും ഓറിയന്റലിസ്റ്റുകള്‍ക്കാവശ്യമുള്ള യാതൊന്നും ഉണ്ടായതായി നിവേദനങ്ങള്‍ വ്യക്തമാക്കുന്നില്ലെന്ന് ചുരുക്കം. ചരിത്രത്തില്‍ നിന്ന് എത്ര വിദൂരമായാണ് ഓറിയന്റലിസ്റ്റുകളുടെയും മിഷനറിമാരുടെയും നബിവിദ്വേഷത്തില്‍ നിന്നുല്‍ഭൂതമാകുന്ന കാല്‍പനിക ഭാവനകള്‍ മേഞ്ഞു നടക്കുന്നതെന്നാലോചിച്ചുനോക്കൂ!

മുഹമ്മദ് നബി(സ)ക്ക് ബൈബിള്‍ പ്രവാചകന്‍മാരെക്കുറിച്ച് സിറിയന്‍ യാത്രകളില്‍ നിന്നുലഭിച്ച അറിവുകളാണ് ക്വുര്‍ആനിലുള്ളതെന്ന വിമര്‍ശക വിശദീകരണത്തിന്റെ പരിഹാസ്യത മനസ്സിലാക്കാന്‍ ക്വുര്‍ആനിലും ഹദീഥുകളിലും ചരിത്രപുസ്തകങ്ങളിലും സവിസ്തരം രേഖപ്പെടുത്തപ്പെട്ടിട്ടുള്ള, പ്രവാചകനും (സ) മക്കയിലെ ബഹുദൈവാരാധാകരും തമ്മില്‍ നടന്നിട്ടുള്ള സംവാദങ്ങളുടെ വിശദാംശങ്ങള്‍ മാത്രം പരിശോധിച്ചാല്‍ മതി എന്നതാണ് വാസ്തവം. രണ്ട് സിറിയന്‍ യാത്രകളില്‍ നിന്ന് സമാഹരിച്ചതാണ് ക്വുര്‍ആനിലെ പല വിവരങ്ങളുമെന്ന ആരോപണം, ക്വുര്‍ആനിനെതിരെ നൂറുകണക്കിന് വിമര്‍ശനങ്ങളുന്നയിച്ചിട്ടും ഒരു മക്കന്‍ ബഹുദൈവാരാധകനും ഒരിക്കല്‍പോലും ഉന്നയിച്ചിട്ടില്ലെന്ന യാഥാര്‍ത്ഥ്യം, 'സിറിയന്‍ സമ്പര്‍ക്കം' പ്രവാചകന്റെ വിയോഗത്തിനുശേഷം ആരുടെെയാക്കെയോ ഭാവനയില്‍ ഉരുത്തിരിഞ്ഞ 'ഫിക്ഷന്‍' മാത്രമാണെന്ന് സുതരാം വ്യക്തമാക്കുന്നില്ലേ? കളവുകള്‍ കൊണ്ട് എത്രകാലം ലോകെത്ത വഞ്ചിക്കാമെന്നാണ് 'ഭാവനാസമ്പന്നരായ' ഓറിയന്റലിസ്റ്റ് 'കാല്‍പനികര്‍' കണക്കുകൂട്ടുന്നത്?

ന്യമതസ്ഥരുടെ ആരാധനാമൂര്‍ത്തികളെ നശിപ്പിക്കുവാന്‍ ആവശ്യപ്പെടുന്ന സൂക്തങ്ങളൊന്നുംതന്നെ ഖുര്‍ആനിലില്ല. മാത്രവുമല്ല. അമുസ്‌ലിംകള്‍ ആരാധിക്കുന്ന വസ്തുക്കളെ അവഹേളിക്കരുതെന്നാണ് ഖുര്‍ആനിന്റെ അനുശാസന: ”അല്ലാഹുവിന് പുറമെ അവര്‍ വിളിച്ചു പ്രാര്‍ഥിക്കുന്നവരെ നിങ്ങള്‍ ശകാരിക്കരുത്” (16:108).

ഏകദൈവാരാധനയിലധിഷ്ഠിതമായ ഇസ്‌ലാം സൃഷ്ടിപൂജയെ വെറുക്കുന്നുവെന്നത് നേരാണ്. അതുകൊണ്ടുതന്നെ സൃഷ്ടിപൂജയുടെ നിരര്‍ഥകതയെ വെളിപ്പെടുത്തുന്ന ഒട്ടനവധി സൂക്തങ്ങള്‍ ഖുര്‍ആനിലുണ്ട്. ഇവയെല്ലാം മനുഷ്യബുദ്ധിയെ തൊട്ടുണര്‍ത്തുന്നവയാണ്. സൃഷ്ടിപൂജയുടെ അധമത്വത്തില്‍നിന്ന് മാനവ സമുദായത്തെ മോചിപ്പിക്കുകയാണ് ഇസ്‌ലാമിന്റെ ലക്ഷ്യം. ആരാധനാ മൂര്‍ത്തികളെ നശിപ്പിച്ചതുകൊണ്ട് മനുഷ്യരാശി സൃഷ്ടിപൂജയില്‍നിന്ന് കരകയറുമെന്ന മൂഢധാരണയൊന്നും ഇസ്‌ലാമിനില്ല. . അതുകൊണ്ടുതന്നെ ഇസ്‌ലാം നശിപ്പിക്കുവാന്‍ ശ്രമിക്കുന്നത് കല്ലുകളാലും മറ്റും നിര്‍മിക്കപ്പെട്ട ആരാധനാമൂര്‍ത്തികളെയല്ല, മനുഷ്യ മനസ്സുകളില്‍ കൊത്തിവെക്കപ്പെട്ട വിഗ്രഹങ്ങളെയാണ്. ഈ വിഗ്രഹ ധ്വംസനത്തിന് ശക്തി പ്രയോഗിക്കുകയല്ല, യുക്തിയെ പ്രവര്‍ത്തനക്ഷമമാക്കുകയാണ് വേണ്ടതെന്ന് അല്ലാഹുവിന് നന്നായി അറിയാം. അതുകൊണ്ടുതന്നെ ഇസ്‌ലാം ചെയ്യുന്നത് വിഗ്രഹാരാധനക്കെതിരെ ജനങ്ങളെ ബോധവത്കരിക്കുകയാണ്.

നജ്‌റാനിലെ ക്രൈസ്തവരുമായി പ്രവാചകനുണ്ടാക്കിയ കരാറില്‍നിന്ന് ഇക്കാര്യം സുതരാം വ്യക്തമാണ്. അതില്‍ നമുക്കിങ്ങനെ വായിക്കാം: ‘നജ്‌റാനിലെ ക്രൈസ്തവര്‍ക്കും അവരോടൊപ്പം ജീവിക്കുന്നവര്‍ക്കും അവരുടെ ജീവന്‍, മതം, ഭൂമി, ധനം എന്നിവക്കും അവരില്‍ സന്നിഹിതരായവര്‍ക്കും അല്ലാത്തവര്‍ക്കും അവരുടെ നിവേദക സംഘങ്ങള്‍ക്കും കുരിശ്, ക്രൈസ്തവ ദേവാലയം തുടങ്ങിയ മത ചിഹ്‌നങ്ങള്‍ക്കും അല്ലാഹുവിന്റെ അഭയവും അവന്റെ ദൂതന്‍ മുഹമ്മദിന്റെ സംരക്ഷണ ബാധ്യതയുമുണ്ട്. ഇവയുടെ നിലവിലുള്ള അവസ്ഥയില്‍ യാതൊരു മാറ്റവും വരുത്തുന്നതല്ല.  അവരുടെ പുരോഹിതനോ സന്യാസിയോ പരിപാലകനോ തല്‍സഥാനത്തുനിന്ന് നീക്കം ചെയ്യപ്പെടുകയോ അവരുടെ ഏതെങ്കിലും അവകാശങ്ങള്‍ ഹനിക്കപ്പെടുകയോ ഏതെങ്കിലും മതചിഹ്‌നങ്ങള്‍ മാറ്റപ്പെടുകയോ ഇല്ല‘.

പ്രവാചകനുശേഷം ഖലീഫമാരും അവര്‍ക്കുശേഷം വന്ന മുസ്‌ലിം ഭരണാധികാരികളുമെല്ലാം അന്യമതസ്ഥര്‍ക്ക് ആരാധനാ സ്വാതന്ത്ര്യമനുവദിച്ചിരുന്നതായി അക്കാലത്തെ രേഖകളില്‍നിന്ന് വ്യക്തമായി മനസ്സിലാക്കാന്‍ കഴിയും.

എന്നാല്‍ ഒരു രാഷ്ട്രം പൂര്‍ണമായി ഇസ്‌ലാമികമായിത്തീരുകയും വിഗ്രഹാരാധകരായി ആരും തന്നെ ഇല്ലാത്ത അവസ്ഥ സംജാതമാകുകയും ചെയ്ത സാഹചര്യത്തിൽ ദുഷിച്ച ബഹുദൈവാരാധനയിലേക്ക് മനുഷ്യരെ തിരിച്ചുവിടുന്ന വിഗ്രഹങ്ങളെയും ശവകുടീരങ്ങളെയും നശിപ്പിക്കാന്‍ പ്രവാചകന്‍ (ﷺ)കല്‍പിച്ചതായി കാണുവാന്‍ കഴിയും. രാഷ്ട്രം ഇസ്‌ലാമികവും ജനങ്ങൾ ഏകദൈവാരാധകരും ആയതിനു ശേഷമുള്ള നടപടികളിലൊന്നാണിത്. ഒരു ബഹുമതസമൂഹത്തില്‍ മുസ്‌ലിം സ്വീകരിക്കേണ്ട നടപടിയല്ലെന്നര്‍ത്ഥം.

 അമുസ്‌ലിംകളെ നിര്‍ബന്ധിച്ച് മതപരിവര്‍ത്തനം ചെയ്യിക്കണമെന്ന് അനുശാസിക്കുന്ന ഒരു വചനം പോലും ഖുര്‍ആനിലില്ല. നിര്‍ബന്ധ മതപരിവര്‍ത്തനം എന്ന ആശയത്തോടുതന്നെ ഖുര്‍ആന്‍ യോജിക്കുന്നില്ല.

ഇസ്‌ലാം എന്നാല്‍ സമര്‍പ്പണം, സമാധാനം എന്നിങ്ങനെയാണര്‍ഥം. സര്‍വശക്തന് സ്വന്തം ജീവിതത്തെ സമര്‍പ്പിക്കുന്നതുവഴി ഒരാള്‍ നേടിയെടുക്കുന്ന സമാധാനമാണ് ഇസ്‌ലാം എന്ന് പറയാം. സ്രഷ്ടാവിന് സ്വന്തത്തെ സമര്‍പ്പിച്ചവനാണ് മുസ്‌ലിം. ഒരാള്‍ മുസ്‌ലിമാവുകയെന്നാല്‍ ദൈവിക മാര്‍ഗനിര്‍ദേശങ്ങള്‍ക്കനുസരിച് ജീവിതത്തെ പരിവര്‍ത്തിപ്പിക്കുകയെന്നാണര്‍ഥം. ഈ പരിവര്‍ത്തനത്തിന്റെ മുളപൊട്ടേണ്ടത് മനസ്സിലാണ്. മനുഷ്യമനസ്സുകളില്‍ മാറ്റമുണ്ടാകാതെ മൗലികമായ യാതൊരു പരിവര്‍ത്തനവും സാധ്യമല്ലെന്നതാണ് ഖുര്‍ആനിന്റെ വീക്ഷണം. അതുകൊണ്ടുത ന്നെ നിര്‍ബന്ധിച്ച് ഒരാളെയും മതത്തില്‍ കൂട്ടുന്നതിനോട് അത് യോജിക്കുന്നില്ല. സത്യവിശ്വാസത്തിലേക്ക്  കടന്നുവരുന്നതിനായി സ്വന്തം സമുദായത്തെ ഉല്‍ബോധിപ്പിക്കുന്നതിനുവേണ്ടി കഠിനാധ്വാനം ചെയ്ത പ്രവാചകന് (ﷺ) സത്യനിഷേധികളുടെ നിലപാടില്‍ മാറ്റമൊന്നുമില്ലെന്ന് മനസ്സിലായപ്പോള്‍ ഉണ്ടായ മനോവ്യഥയെ ചോദ്യം ചെയ്തുകൊണ്ട്ഖുര്‍ആന്‍ പറയുന്നത് കാണുക: ”നിന്റെ രക്ഷിതാവ് ഉദ്ദേശിച്ചിരുന്നുവെങ്കില്‍ ഭൂമിയിലുള്ളവരെല്ലാം ഒന്നിച്ച് വിശ്വസിക്കുമായിരുന്നു. എന്നിരിക്കെ ജനങ്ങള്‍ സത്യവിശ്വാസികളാകുവാന്‍ നീ അവരെ നിര്‍ബന്ധിക്കുകയോ? (10:99).

സത്യമതപ്രബോധനത്തിനായി നിയുക്തരായ പ്രവാചകന്മാരില്‍ നിക്ഷിപ്തമായിരുന്ന ബാധ്യത മതപ്രചാരണം മാത്രമായിരുന്നുവെന്നും നിര്‍ബന്ധിച്ച് മതം മാറ്റുകയായിരുന്നില്ലെന്നുമുള്ള വസ്തുത ഖുര്‍ആന്‍ വ്യക്തമാക്കുന്നുണ്ട്: ”എന്നാല്‍ ദൈവദൂതന്മാരുടെ മേല്‍ സ്പഷ്ടമായ പ്രബോധനമല്ലതെ വല്ല ബാധ്യതയുമുണ്ടോ?” (16:36).

”ഇനി അവര്‍ തിരിഞ്ഞുകളയുകയാണെങ്കില്‍ (നബിയേ) നിന്നെ നാം അവരുടെ മേല്‍ കാവല്‍ക്കാരനായി അയച്ചിട്ടില്ല. നിന്റെ മേല്‍ പ്രബോധനബാധ്യത മാത്രമേയുള്ളൂ‘ (വി.ഖു 42:48).

സത്യമതത്തിലേക്ക് ജനങ്ങളെ ക്ഷണിക്കുകയല്ലാതെ അവരെ നിര്‍ബന്ധിച്ച് മാറ്റുന്നതിനുവേണ്ടി പ്രവാചകന്‍ പരിശ്രമിക്കേണ്ടതില്ലെന്ന് ഖുര്‍ആന്‍ അദ്ദേഹത്തോട് ആവര്‍ത്തിച്ചു പറയുന്നുണ്ട്”. പറയുക: സത്യം നിങ്ങളുടെ രക്ഷിതാവിങ്കല്‍നിന്നുള്ളതാവുന്നു. അതിനാല്‍ ഇഷ്ടമുള്ളവര്‍ വിശ്വസിക്കട്ടെ” (18:29).

”അതിനാല്‍ (നബിയേ) നീ ഉല്‍ബോധിപ്പിക്കുക. നീ ഒരു ഉല്‍ബോധകന്‍ മാത്രമാകുന്നു. നീ അവരുടെ മേല്‍ അധികാരം ചെലുത്തേണ്ടവനല്ല” (88:21,22)

ചുരുക്കത്തില്‍ പ്രവാചകന്മാരെല്ലാം സത്യമതപ്രബോധകര്‍ മാത്രമായിരുന്നു. അന്തിമ പ്രവാചകനും തഥൈവ. ജനങ്ങളുടെ മുമ്പില്‍ സത്യമേതെന്ന് തുറന്നു കാണിക്കേണ്ട ഉത്തരവാദിത്തം  മാത്രമേ അദ്ദേഹത്തിനുണ്ടായിരുന്നുള്ളൂ. അന്തിമ പ്രവാചകനിലൂടെ പൂര്‍ത്തീകരിക്കപ്പെട്ട സത്യമതത്തിന്റെ പ്രചാരണം ഉത്തരവാദിത്തമായി ഏല്‍പിക്കപ്പെട്ട സത്യവിശ്വാസികളുടെ ബാധ്യതയും ഇതുമാത്രമാണ്. അസത്യത്തില്‍നിന്ന് സത്യത്തെ വേര്‍തിരിച്ച് മനസ്സിലാക്കിക്കൊടുക്കുകയെന്ന ബാധ്യത മാത്രം. മതത്തില്‍ നിര്‍ബന്ധിച്ച് ആളെ ചേര്‍ക്കുന്നതിന് ഖുര്‍ആന്‍ ആരോടും ആവശ്യപ്പെടുന്നില്ലെന്നു മാത്രമല്ല, നിര്‍ബന്ധ മതപരിവര്‍ത്തനം ശരിയല്ലെന്ന നിലപാട് അത് പ്രഖ്യാപിക്കുകയും ചെയ്യുന്നു.

 ”മതത്തിന്റെ കാര്യത്തില്‍ ബലപ്രയോഗമേയില്ല. സന്മാര്‍ഗം ദുര്‍മാര്‍ഗത്തില്‍നിന്ന് വ്യക്തമായി വേര്‍തിരിഞ്ഞുകഴിഞ്ഞിരിക്കുന്നു” (2:256).

തന്നെ അല്ലാഹു ഒറ്റരാത്രി കൊണ്ട് മക്കയിലെ മസ്ജിദുല്‍ ഹറാമില്‍നിന്ന് ഫലസ്ത്വീനിലെ മസ്ജിദുല്‍ അക്വ്‌സയിലേക്ക് അതിവേഗ നിശാപ്രയാണം (ഇസ്രാഅ്) ചെയ്യിച്ചുവെന്ന മുഹമ്മദ് നബിയുടെ അവകാശവാദം വ്യാജമാണെന്ന് അതുസംബന്ധമായി അദ്ദേഹം നല്‍കിയ വിവരണങ്ങളിലെ കാലാനുചിതത്വത്തില്‍ നിന്നും വ്യക്തമാകുന്നുണ്ട്. പ്രവാചകന്റെ കാലഘട്ടത്തില്‍ മസ്ജിദുല്‍ അക്വ്‌സ നിലവിലുണ്ടായിരുന്നില്ല എന്നതാണ് യാഥാര്‍ത്ഥ്യം. ഖലീഫ ഉമറിന്റെ  കാലഘട്ടത്തിലാണ് പ്രസ്തുത ആരാധനാലയം നിര്‍മിക്കപ്പെടുന്നത്.’ ഇസ്രാഅ് കെട്ടുകഥയാണെന്നു സ്ഥാപിക്കുവാന്‍ നബിവിമര്‍ശകര്‍ ഉന്നയിക്കുന്ന ഈ ആരോപണത്തിന് എന്തെങ്കിലും അടിസ്ഥാനമുണ്ടോ

 പ്രബോധന പ്രവര്‍ത്തനങ്ങള്‍ക്ക് താങ്ങും തണലുമായി നിന്നിരുന്ന പിതൃവ്യന്‍ അബൂത്വാലിബും പത്‌നി ഖദീജ(റ)യും പ്രവാചകത്വത്തിന്റെ പത്താം വര്‍ഷം മരണപ്പെട്ടതിനെ തുടര്‍ന്ന് മുഹമ്മദ് നബി (സ) അങ്ങേയറ്റം ഖിന്നനും സാമൂഹികമായി അരക്ഷിതനുമായിത്തീര്‍ന്നതും മക്കക്കാരുടെ കഠിനശാത്രവത്തില്‍ നിന്ന് രക്ഷയാഗ്രഹിച്ച് അദ്ദേഹം ത്വാഇഫിലെ ഥക്വീഫ് ഗോത്രക്കാര്‍ക്കടുത്തേക്ക് അഭയവും പിന്തുണയുമഭ്യര്‍ത്ഥിച്ച് ചെന്നതും അവര്‍ പ്രവാചകനെ അതിനീചമായ രീതിയില്‍ കല്ലെറിഞ്ഞോടിച്ചു മക്കയിലേക്കു തന്നെ തിരിച്ചയച്ചതും ഇസ്‌ലാമിക ചരിത്രത്തിലെ പ്രസിദ്ധമായ അധ്യായങ്ങളാണ്. ഈ സാഹചര്യത്തില്‍ നബി(സ)ക്ക് അളവറ്റ സാന്ത്വനവും ആത്മവിശ്വാസവും പകരുകയും മാനവരില്‍ മഹോന്നതനും പ്രവാചകന്‍മാരുടെ നേതാവുമായി അദ്ദേഹത്തെ ആദരിക്കുകയും ചെയ്തുകൊണ്ട് അല്ലാഹു നടത്തിയ അത്യത്ഭുകരമായ ഇടപെടലായിരുന്നു ഇസ്രാഉം മിഅ്‌റാജും.

ഒരു രാത്രിയില്‍ മക്കയില്‍ നിദ്രയിലേക്കു ചായുകയായിരുന്ന നബി(സ)യെ അതിവേഗം ജറൂസലേമിലേക്ക് സഞ്ചരിപ്പിച്ചു കൊണ്ടുപോവുകയും (ഇസ്‌റാഅ്) അവിടെനിന്ന് ആകാശലോകത്തേക്കുയര്‍ത്തി അവിടുത്തെ കാഴ്ചകള്‍ കാണിക്കുകയും (മിഅ്‌റാജ്) ചെയ്തശേഷം തിരിച്ചു ജറൂസലേമിലേക്കും ആ രാത്രി പുലരുന്നതിനു മുമ്പുതന്നെ മക്കയിലേക്കുമെത്തിക്കുകയും ചെയ്ത ദിവ്യദൃഷ്ടാന്തം നബിജീവിതത്തില്‍ പ്രവാചകത്വത്തെ സാക്ഷീകരിച്ചുകൊണ്ടുണ്ടായ ഏറ്റവും വലിയ അടയാളങ്ങളില്‍ (മുഅ്ജിസത്ത്) ഒന്നായിരുന്നു

. പ്രവാചകന്റെ (സ) ഇസ്രാഅ് അനുഭവത്തെ പരാമര്‍ശിച്ചുകൊണ്ട് അവതരിപ്പിക്കപ്പെട്ട ക്വുര്‍ആന്‍ വചനത്തിലും നബി (സ) നല്‍കിയ വിവരണങ്ങളിലും പ്രമാദമായ ഒരു കാലാനുചിതത്വം മുഴച്ചു നില്‍ക്കുന്നുവെന്നും അത് ഇസ്രാഅ് ഒരു കെട്ടുകഥയാണെന്ന് വ്യക്തമാക്കുന്നുവെന്നുമാണ് പല മിഷനറിമാരും ഓറിയന്റലിസ്റ്റുകളും സ്ഥാപിക്കാന്‍ ശ്രമിച്ചിട്ടുളളത്. ഇസ്രാഇന്റെ സമയത്ത് ഫലസ്ത്വീനില്‍ മസ്ജിദുല്‍ അക്വ്‌സ ഇല്ല എന്നിരിക്കെ എങ്ങനെയാണ് അങ്ങോട്ടു നിശാസഞ്ചാരം നടന്നു എന്ന അവകാശവാദം വിശ്വസനീയമാവുക എന്ന ചോദ്യമാണ് അവര്‍ ഉന്നയിച്ചിരിക്കുന്നത്. എന്താണ് യാഥാര്‍ത്ഥ്യമെന്ന് നമുക്ക് പരിശോധിക്കാം.

ഇസ്രാഅ് സംബന്ധിയായുള്ള ക്വുര്‍ആന്‍ വചനത്തിന്റെ സാരം ഇങ്ങനെയാണ്. ”തന്റെ ദാസനെ (നബിയെ) ഒരു രാത്രിയില്‍ മസ്ജിദുല്‍ ഹറാമില്‍ നിന്ന് മസ്ജിദുല്‍ അക്വ്‌സയിലേക്ക് -അതിന്റെ പരിസരം നാം അനുഗൃഹീതമാക്കിയിരിക്കുന്നു- നിശായാത്ര ചെയ്യിച്ചവന്‍ എത്രയോ പരിശുദ്ധന്‍! നമ്മുടെ ദൃഷ്ടാന്തങ്ങളില്‍ ചിലത് അദ്ദേഹത്തിന് നാം കാണിച്ചുകൊടുക്കാന്‍ വേണ്ടിയത്രെ അത്. തീര്‍ച്ചയായും അവന്‍ (അല്ലാഹു) എല്ലാം കേള്‍ക്കുന്നവനും കാണുന്നവനുമത്രെ.”(1)

സംഭവത്തിന്റെ വിശദാംശങ്ങള്‍ പ്രബലമായ ഹദീഥുകളിലുണ്ട്. നബി (സ) മക്കയില്‍ ഖുറയ്ശികള്‍ പുനര്‍നിര്‍മിച്ച കഅ്ബയുടെ ചാരത്തുള്ള ഹിജ്‌റില്‍ ആയിരിക്കെയാണ് ഇസ്രാഅ് ആരംഭിച്ചതെന്നു അബ്ബാസുബ്‌നു മാലിക്കും(2) (റ), നബി (സ) ഉണര്‍വ്വിനും ഉറക്കിനുമിടയിലുള്ള ഒരവസ്ഥിയിലായിരിക്കെ ജിബ്‌രീല്‍ കടന്നുവന്ന്, കഴുതയെക്കാള്‍ വലതും കോവര്‍ കഴുതയെക്കാള്‍ ചെറുതുമായിരുന്ന, വെള്ള നിറത്തിലുള്ള ബുറാക്വ് എന്ന സവിശേഷ മൃഗത്തിന്റെ പുറത്തിരുത്തി കാഴ്ചയെത്തുന്നേടത്ത് കാലെത്തുന്ന അതിവേഗതയിലാണ് ഇസ്രാഇന് കൊണ്ടുപോയതെന്ന് അനസ്ബ്‌നു മാലിക്കും(3) (റ) പ്രവാചകനെ ഉദ്ധരിച്ചു പറഞ്ഞത് ആധികാരികമായി നിവേദനം ചെയ്യപ്പെട്ടിരിക്കുന്നു.

ഇങ്ങനെ പുറപ്പെട്ട യാത്ര ‘ബയ്തുല്‍ മക്വ്ദിസില്‍’ എത്തിയെന്നും അവിടെ ബുറാക്വിനെ തളച്ചശേഷം താന്‍ രണ്ടു റക്അത്ത് നമസ്‌കരിച്ചുവെന്നും നബി (സ) പറഞ്ഞതായി അനസ്ബ്‌നു മാലിക്കില്‍ നിന്നു തന്നെയുള്ള മറ്റൊരു നിവേദനത്തിലുണ്ട്.(4) ഇസ്രാഈല്‍ സമുദായത്തിന്റെ ജീവിതസിരാകേന്ദ്രമായിരുന്ന ജറൂസലേമിലുള്ള ബയ്തുല്‍ മക്വ്ദിസില്‍ വെച്ച് പൂര്‍വപ്രവാചകന്‍മാരായ അബ്രഹാം, മോശെ, യേശു എന്നിവരുമായുള്ള ആത്മീയ സമാഗമത്തിന് അല്ലാഹു തനിക്ക് അവസരമൊരുക്കിയെന്നും താന്‍ അവരുടെ സംഘപ്രാര്‍ത്ഥനയുടെ നേതാവായി നിശ്ചയിക്കപ്പെട്ടുവെന്നും നബി (സ) വ്യക്തമാക്കിയതായി അബൂ ഹുറയ്‌റ(റ)യില്‍ നിന്ന് ഉദ്ധരിക്കപ്പെടുന്ന മറ്റൊരു ഹദീഥില്‍(5) നിന്നു മനസ്സിലാക്കാം.

ഇസ്രാഅ് കഴിഞ്ഞ് മക്കയില്‍ തിരിച്ചെത്തി നബി (സ) കഅ്ബക്കരികില്‍ നില്‍ക്കുമ്പോള്‍ രാപ്രയാണത്തെക്കുറിച്ചു കേട്ട മക്കക്കാര്‍ ചോദ്യങ്ങളുമായി വളഞ്ഞുവെന്നും കഥനം സത്യമാണെന്നു തെളിയിക്കാന്‍ ബെയ്തുല്‍ മക്വ്ദിസിന്റെ ചില വിശദാംശങ്ങള്‍ -അല്‍പനേരം മാത്രം അവിടെ ചെലവഴിച്ചൊരാള്‍ക്ക് നല്‍കുവാനാകാത്തത്ര സൂക്ഷ്മമായവ- പറയാന്‍ ആവശ്യപ്പെട്ടുവെന്നും അപ്പോള്‍ അല്ലാഹു നബി(സ)ക്ക് മുന്നില്‍ ബയ്ത്തുല്‍ മക്വ്ദിസ് പ്രദര്‍ശിപ്പിച്ചുവെന്നും അതുനോക്കി അദ്ദേഹം അവരുടെ ചോദ്യങ്ങള്‍ക്കെല്ലാം കൃത്യമായ മറുപടി നല്‍കിയെന്നും അതേ ഹദീഥില്‍ തന്നെ സുതരാം വ്യക്തമാക്കപ്പെട്ടിട്ടുണ്ട്.

മക്കയിലെ കഅ്ബക്കരികില്‍ നിന്ന് ഫലസ്ത്വീനിലെ മസ്ജിദുല്‍ അക്വ്‌സ എന്നോ ബെയ്തുല്‍ മക്വ്ദിസ് എന്നോ പറയപ്പെടുന്ന സ്ഥലത്തേക്കാണ് ഇസ്രാഅ് നടന്നതെന്നാണ് നബി (സ) വിശദീകരിച്ചതെന്ന കാര്യം മേല്‍ പരാമര്‍ശിച്ച ക്വുര്‍ആന്‍ വാക്യത്തില്‍ നിന്നും ഹദീഥുകളില്‍ നിന്നും സ്പഷ്ടമാണ്. ആരാധനകള്‍ നടക്കുന്ന ഇടത്തിനാണ് ഇവിടെ യഥാക്രമം ‘മസ്ജിദ്’ എന്നോ ‘ബയ്ത്ത്’ എന്നോ പ്രയോഗിക്കപ്പെട്ടിരിക്കുന്നത് എന്ന കാര്യം വ്യക്തമാണ്. ‘മസ്ജിദ്’ എന്നാല്‍ ഭാഷാപരമായി ‘സാഷ്ടാംഗസ്ഥാനം’ എന്നും ‘ബയ്ത്ത്’ എന്നാല്‍ ഭവനം എന്നുമാണ് അര്‍ത്ഥം. ‘അക്വ്‌സാ’ എന്നാല്‍ വിദൂരമായത്/അങ്ങേയറ്റത്തുള്ളത് എന്നും ‘മക്വ്ദിസ്’ എന്നാല്‍ വിശുദ്ധമായത് എന്നുമാണ് ഭാഷാ വിവക്ഷ. ദൈവാരാധനക്കുവേണ്ടി സ്ഥാപിച്ചിട്ടുള്ളതില്‍വെച്ച് മക്കയില്‍ നിന്നേറ്റവും വിദൂരമായ ഒരു സാഷ്ടാംഗസ്ഥാനം/ആരാധനാ കേന്ദ്രം ജറൂസലേമില്‍ സ്ഥിതി ചെയ്യുന്നുവെന്നും കഅ്ബയില്‍ നിന്ന് അങ്ങോട്ടാണ് നബി(സ)യുടെ രാസഞ്ചാരം നടന്നതെന്നും ചുരുക്കം. ഏതാണ് ഈ ആരാധന കേന്ദ്രം? അത് നിര്‍മിച്ചത് ഖലീഫ ഉമറിന്റെ ഭരണകാലത്താണോ? നമുക്ക് അന്വേഷിക്കുക!

ഇസ്രാഇനെക്കുറിച്ചുള്ള ക്വുര്‍ആന്‍ വചനം നബി (സ) പാരായണം ചെയ്തു കേള്‍പ്പിക്കുന്നത് ഹിജാസിലെ തന്റെ സമകാലീനരെയാണ്. അവരില്‍ വിശ്വാസികള്‍ക്കു പുറമെ അവിശ്വാസികളായ ബഹുദൈവാരാധകരുമുണ്ട്. ഇസ്രാഇനെക്കുറിച്ച് നബി (സ) സ്വന്തം ഭാഷയില്‍ വിവരിക്കുന്നതും അവരോടു തന്നെയാണ്. ഇസ്രാഇനെക്കുറിച്ചുള്ള പ്രവാചകന്റെ (സ) കഥനം സ്വീകരിക്കുവാന്‍ സന്നദ്ധമാകാതെ അതിനെ കെട്ടുകഥയാക്കി മുദ്രകുത്തുകയും നബിനിഷേധം തുടരുകയുമാണ് അവിശ്വാസികളായ അദ്ദേഹത്തിന്റെ സമകാലീനര്‍ സ്വാഭാവികമായും ചെയ്തത്.

ഇസ്രാഇനെയും മിഅ്‌റാജിനെയും കുറിച്ചുള്ള പ്രവാചകാഖ്യാനത്തെ അപ്പടി തല്‍ക്ഷണം സ്വീകരിക്കുവാന്‍ സന്നദ്ധനായതിന്റെ പേരിലാണ് പ്രവാചകന്റെ ആത്മമിത്രം അബൂബക്‌റിന് ‘സ്വിദ്ദീക്വ്’ (വിശ്വസ്തത പുലര്‍ത്തുന്നയാള്‍) എന്ന അപരാഭിധാനം ലഭിച്ചത് എന്നു പറയുമ്പോള്‍, എത്ര വലിയ അവിശ്വസനീയതയോടെയാണ് ഇവ്വിഷയകമായ നബിവചനങ്ങളെ മക്ക എതിരേറ്റതെന്നു വ്യക്തമാകുന്നുണ്ട്. ‘ഇതും വിശ്വസിച്ചോ’ എന്ന മട്ടില്‍ അവിശ്വാസം മുറ്റുന്ന കണ്ണുകളുമായി തന്നെ സമീപിച്ച് ”അദ്ദേഹം രാത്രിയില്‍ ജറൂസലേമില്‍ പോയി പുലരുന്നതിനുമുമ്പ് തിരിച്ചെത്തിയെന്ന് പറയുന്നതിനെ താങ്കള്‍ക്കംഗീകരിക്കാനാകുന്നതെങ്ങനെ” എന്നു ചോദിച്ചവരോട് അബൂബക്ര്‍ പറഞ്ഞ ഈ മറുപടിയാണ് അദ്ദേഹത്തെ ‘സ്വിദ്ദീക്വ്’ ആക്കിയതെന്നാണ് ചരിത്രം പറയുന്നത്: ”അതെ! നിശ്ചയമായും ഞാന്‍ അദ്ദേഹം പറയുന്ന അതിനെക്കാള്‍ അത്ഭുതകരമായ കാര്യങ്ങളില്‍ വിശ്വസിക്കുന്നുണ്ടല്ലോ. രാവിലെയോ ദിനാന്ത്യത്തിലോ അദ്ദേഹത്തിന് ആകാശത്തുനിന്ന് വൃത്താന്തങ്ങളെത്തുന്നുവെന്ന് ഞാന്‍ അംഗീകരിക്കുന്നില്ലേ?”(6)

ദൈവിക വെളിപാടുകള്‍ കൊണ്ടനുഗ്രഹിക്കപ്പെട്ട പ്രവാചകന്‍ പ്രപഞ്ചനാഥന്‍ സംവിധാനിച്ച ഒരു അമാനുഷിക സഞ്ചാരത്തിന് തെരഞ്ഞെടുക്കപ്പെടുന്നതില്‍ യാതൊരു അസാംഗത്യവുമില്ലെന്ന, യുക്തിഭദ്രമായ ബോധ്യമാണ് അബൂബക്‌റിനെ ഇവിടെ നയിക്കുന്നത്. ഇസ്രാഅ് നടന്നുവെന്നംഗീകരിക്കുവാന്‍ വിസമ്മതിച്ച ഒരു വലിയ ആള്‍കൂട്ടം മക്കയിലുണ്ടായിരുന്നുവെന്നും അവരുടെ ‘പ്രശ്‌നവല്‍കരണങ്ങള്‍’ പ്രവാചകത്വം അംഗീകരിക്കുന്നുണ്ടോ ഇല്ലേ എന്നതാണ് അടിസ്ഥാന ചോദ്യമെന്നും അതംഗീകരിക്കുന്നവര്‍ക്ക് ഇസ്രാഅ് വാര്‍ത്ത അംഗീകരിക്കാതിരിക്കാനാവില്ലെന്നുമുള്ള അബൂബക്‌റിനെ പോലുള്ളവരുടെ സമര്‍ത്ഥനങ്ങള്‍ക്കുമുന്നില്‍ ബുദ്ധിപരമായി പരാജയപ്പെട്ടുവെന്നുമാണ് ഈ സംഭാഷണങ്ങള്‍ സൂചിപ്പിക്കുന്നത്.

ഒരു രാത്രികൊണ്ട് ഇത്രയധികം ദൂരം താണ്ടാനാകില്ല എന്നതായിരുന്നു അവിശ്വാസികളുടെ ‘പോയിന്റ്’ എന്നു സാരം. സാധാരണ നിയമങ്ങളെ മറികടന്നുകൊണ്ടുള്ള സവിശേഷമായ ഒരു ദൈവിക ഇടപെടലാണ് ‘ഇസ്രാഅ്’ എന്നതുകൊണ്ടു തന്നെ ആ പോയിന്റിന് സംവാദങ്ങളില്‍ പിടിച്ചുനില്‍ക്കാനാകുമായിരുന്നില്ല. എന്നാല്‍ പ്രവാചകകഥനത്തില്‍ ‘കാലാനുചിതത്വം’ ഉണ്ടായിരുന്നെങ്കില്‍ ഇസ്രാഅ് വൃത്താന്തം കള്ളമാണെന്നു തെളിയിക്കുവാന്‍ അത് തീര്‍ത്തും ഉപയുക്തമാകുമായിരുന്നു. ഫലസ്ത്വീന്‍ ഉള്‍ക്കൊള്ളുന്ന അന്നത്തെ ‘ശാം’ പ്രവിശ്യയിലേക്ക് നിരന്തരമായി കച്ചവടയാത്ര പോയിരുന്നവരായിരുന്നു മക്കയിലെ അറബികള്‍; ഫലസ്ത്വീന്‍ അതുകൊണ്ടു തന്നെ അവര്‍ക്ക് ചിരപരിചിതവുമായിരുന്നു. ഫലസ്ത്വീനില്‍ ഒരു മസ്ജിദുല്‍ അക്വ്‌സ/ബെയ്തുല്‍  മക്വ്ദിസ് ഇല്ലായിരുന്നുവെങ്കില്‍ നബബവിവരണങ്ങളെ ‘പൊളിക്കുവാന്‍’ അവര്‍ക്ക് വളരെ എളുപ്പമായിരുന്നു. എന്നിട്ടെന്തുകൊണ്ട് അവരിലൊരാള്‍ പോലും അക്കാര്യമുന്നയിച്ചില്ല?

ഫലസ്ത്വീനിന് മതപരവും വംശീയവുമായ വിശുദ്ധി കല്‍പിക്കുന്ന ജൂതന്‍മാര്‍ സമൃദ്ധമായിരുന്ന മദീനയിലേക്കാണ് നബി(സ)യും ശിഷ്യന്‍മാരും ഇസ്രാഅ് നടന്ന് ഏതാനും കാലത്തിനകം ഹിജ്‌റ പോകുന്നത്. ഇസ്രാഅ്-മിഅ്‌റാജ് കഥകള്‍ ജൂതന്‍മാരുമായി സ്വാഭാവികമായും മുസ്‌ലിംകളുടെ വിശ്വാസത്തിന്റെ ഭാഗമായി വിനിമയം ചെയ്യപ്പെട്ടു. എന്നിട്ടും ഒരു ജൂതന്‍പോലും ഒരിക്കല്‍ പോലും എന്തുകൊണ്ട് ഏത് ബയ്ത്തുല്‍ മക്വ്ദിസ്/ഏതു മസ്ജിദുല്‍ അക്വ്‌സ എന്നു ചോദിച്ച് പ്രവാചകനരികിലേക്കു വന്നില്ല?

ബെയ്തുല്‍ മക്വ്ദിസ്/മസ്ജിദുല്‍ അക്വ്‌സ എന്ന പേരിലുള്ള ഒരാരാധനാസ്ഥാനം പ്രവാചകകാലത്തുതന്നെ ഫലസ്ത്വീനിലുണ്ടായിരുന്നുവെന്നും ഹിജാസിലെ ജൂതന്‍മാരും ബഹുദൈവാരാധകരുമടക്കം അതിനെക്കുറിച്ചറിവുള്ളവരുമായിരുന്നുവെന്നുമാണ് ഇത് വ്യക്തമാക്കുന്നത്. പ്രവാചകന്റെ ഇസ്രാഅ് വിവരണത്തെ ചോദ്യം ചെയ്യാന്‍ കഅ്ബക്കരികില്‍ അദ്ദേഹത്തിനു ചുറ്റുംകൂടിയ മക്കന്‍ ബഹുദൈവാരാധകര്‍ ബെയ്തുല്‍ മക്വ്ദിസിന്റെ വിശദാംശങ്ങള്‍ പറഞ്ഞ് കഥയുടെ ആധികാരികത തെളിയിക്കുവാന്‍ അദ്ദേഹത്തെ വെല്ലുവിളിച്ചുവെന്നു പറഞ്ഞാല്‍ അതിനര്‍ത്ഥം ഫലസ്ത്വീന്‍ യാത്ര നടത്തിയവരും ബയ്ത്തുല്‍ മക്വ്ദിസ് നേരിട്ടുകണ്ടവരുമായ അനേകര്‍ അക്കൂട്ടത്തിലുണ്ടായിരുന്നുവെന്നാണല്ലോ!

യഥാര്‍ത്ഥത്തില്‍, വിമര്‍ശകരില്‍ ചിലരെങ്കിലും ധരിച്ചുവശായിട്ടുള്ളതുപോലെ ഇസ്രാഅ് വിവരണത്തില്‍ മാത്രം നബി (സ) പരാമര്‍ശിച്ച ഒന്നല്ല ഫലസ്ത്വീനിലെ വിശുദ്ധ മസ്ജിദ്. അദ്ദേഹത്തിന്റെ വര്‍ത്തമാനങ്ങളിലും അനുചരന്‍മാരുടെ ജീവിതവ്യവഹാരങ്ങളിലും നിരന്തരം കടന്നുവന്നിരുന്ന ഒരു റഫറന്‍സ് പോയിന്റ് ആയിരുന്നു അത്. മിഅ്‌റാജിന്റെ അവസരത്തിലാണ് ദിനേന അഞ്ചുനേരം നിര്‍ദ്ദിഷ്ടരീതിയില്‍ നമസ്‌കാരം നിര്‍വഹിക്കേണ്ടത് എല്ലാ മുസ്‌ലിംകള്‍ക്കും നിര്‍ബന്ധമാക്കിെക്കാണ്ടുള്ള ദൈവിക കല്‍പന നബി(സ)ക്ക് ലഭിക്കുന്നത്.(7) ഈ നമസ്‌കാരം ബയ്ത്തുല്‍ മക്വ്ദിസിന്റെ ദിശയിലേക്കു തിരിഞ്ഞുകൊണ്ട് നിര്‍വഹിക്കാനായിരുന്നു ദൈവകല്‍പന. തദടിസ്ഥാനത്തില്‍ മദീനാ ജീവിതത്തിന്റെ ആദ്യമാസങ്ങളില്‍ ബയ്ത്തുല്‍ മക്വ്ദിസിലേക്കു തിരിഞ്ഞാണ് മുസ്‌ലിംകള്‍ പ്രാര്‍ത്ഥിച്ചുകൊണ്ടിരുന്നത്. പിന്നീടാണ് മക്കയിലെ കഅ്ബയുടെ ദിശയിലേക്കു തിരിയുവാന്‍ നിര്‍ദ്ദേശിച്ചുകൊണ്ടുള്ള ക്വുര്‍ആന്‍ വചനം(8) അവതരിപ്പിക്കപ്പെടുന്നതും മുസ്‌ലിംകളുടെ ക്വിബ്‌ല ജറൂസലേമില്‍നിന്ന് മക്കയിലേക്കു മാറുന്നതും.

നബി (സ) മദീനയിലെത്തിയതിനുശേഷം പതിനാറോ പതിനേഴോ മാസങ്ങള്‍ നമസ്‌കാരം ബയ്ത്തുല്‍ മക്വ്ദിസിനു അഭിമുഖമായിട്ടായിരുന്നുവെന്നും ഒരു അസ്വ്ര്‍ നമസ്‌കാരമാണ് കഅ്ബയിലേക്കു തിരിഞ്ഞു ആദ്യമായി നബി(സ)യുടെ നേതൃത്വത്തില്‍ സംഘമായി നിര്‍വഹിക്കപ്പെട്ടതെന്നും പുതിയ ക്വിബ്‌ലക്കഭിമുഖമായി പ്രവാചകന്റെ (സ) കൂടെ ഈ നമസ്‌കാരത്തില്‍ പങ്കെടുത്ത ഒരാള്‍ നമസ്‌കാരം കഴിഞ്ഞു പുറത്തിറങ്ങി മദീനയില്‍ തന്നെയുള്ള മറ്റൊരു പള്ളിക്കരികിലെത്തിയപ്പോള്‍ അവിടെ ക്വിബ്‌ലമാറ്റ വിവരമറിയാതെ ജനങ്ങള്‍ ബയ്ത്തുല്‍ മക്വ്ദിസിന്റെ ദിശയില്‍ തന്നെ നമസ്‌കരിക്കുന്നതു കണ്ടുവെന്നും അപ്പോള്‍ അദ്ദേഹം ഉച്ചത്തില്‍ ക്വിബ്‌ലമാറ്റം വിളംബരം ചെയ്തുവെന്നും അതുകേട്ട ജനങ്ങള്‍ നമസ്‌കരിച്ചുകൊണ്ടിരിക്കെ തന്നെ പുതിയ ക്വിബ്‌ലയിലേക്ക് കൂട്ടമായി തിരിഞ്ഞുവെന്നും പ്രവാചകാനുചരനായ ബറാഉബ്‌നു ആസ്വിബ് (റ) വിശദീകരിച്ചത് ബുഖാരി രേഖപ്പെടുത്തിയിട്ടുണ്ട്.(9) ഇത് നല്‍കുന്ന അറിവെന്താണ്? ജറുസലേമിലെ ബയ്ത്തുല്‍ മക്വ്ദിസിലേക്ക് തിരഞ്ഞുകൊണ്ടാണ് മുസ്‌ലിംകളുടെ നിര്‍ബന്ധ പ്രാര്‍ത്ഥനകള്‍ എന്നകാര്യം പരസ്യമായിരുന്നുവെന്നും മദീനയിലെ പള്ളികള്‍ നിര്‍മിക്കപ്പെട്ടിരുന്നത് ബയ്ത്തുല്‍ മക്വ്ദിസിനുഭിമുഖമായി നമസ്‌കാരം നിര്‍വഹിക്കാനാകുംവിധമായിരുന്നുവെന്നും തന്നെയല്ലേ?

ജറുസലേമിനെക്കുറിച്ച് യഹൂദബൈബിളിന്റെയും തല്‍മൂദിന്റെയുമെല്ലാം വെളിച്ചത്തില്‍ കിറുകൃത്യമായ അറിവുണ്ടായിരുന്ന ജൂതന്‍മാര്‍ക്കു നടുവിലാണിതെല്ലാമെന്നോര്‍ക്കണം. നമസ്‌കാരത്തിനു നേതൃത്വം നല്‍കുന്നയാള്‍ക്ക് (ഇമാം) നില്‍ക്കാന്‍ ആദ്യം ബയ്ത്തുല്‍ മക്വ്ദിസിനഭിമുഖമായി നിര്‍മിക്കപ്പെട്ടിരുന്ന സ്ഥലം (മിഹ്‌റാബ്) കഅ്ബ ക്വിബ്‌ലയായതിനുശേഷവും അതേപടി സംരക്ഷിക്കപ്പെട്ട മദീനയിലെ പള്ളികളിലൊന്നാണ് ‘മസ്ജിദുല്‍ ക്വിബ്‌ലതയ്‌നി’ (രണ്ട് കിബ്‌ലകളുള്ള മസ്ജിദ്) എന്നപേരില്‍ മുസ്‌ലിം ലോകത്ത് കഴിഞ്ഞ പതിനാലു നൂറ്റാണ്ടുകാലവും പ്രശസ്തമായിരുന്നു. ഇന്നും മദീനയിലുള്ള ഈ പള്ളിയില്‍നിന്നും അടുത്തകാലത്തുമാത്രമാണ് ഇത് നീക്കം ചെയ്തത്. ഇത് ബനൂ സലമ ഗോത്രക്കാരുടെ പള്ളിയായിരുന്നുവെന്നും പ്രവാചകന്‍ അവിടെയിരിക്കുമ്പോഴാണ് ക്വിബ്‌ല മാറ്റത്തിനുള്ള സന്ദേശം ലഭിച്ച് കഅ്ബയിലേക്കു തിരിഞ്ഞ് നമസ്‌കരിച്ചതെന്നും അതുകൊണ്ടാണ് ഈ പള്ളി സവിശേഷമായ പേരിലും രീതിയിലും നിലനിന്നതെന്നും പറയുന്ന ഒരു പാരമ്പര്യം ഇബ്‌നു സഅദ് രേഖപ്പെടുത്തിയിട്ടുണ്ട്.(10) പ്രസ്തുത പാരമ്പര്യത്തിന്റെ ആധികാരികത എന്തു തന്നെയായിരുന്നാലും, ബയ്ത്തുല്‍ മക്വ്ദിസ് നമസ്‌കാരദിശയായിരുന്ന ഭൂതകാലം മുസ്‌ലിം സമൂഹത്തില്‍ കാലങ്ങള്‍ക്കുശേഷവും എത്ര സജീവമായി ചര്‍ച്ച ചെയ്യപ്പെട്ടുവെന്ന് അത് ബോധ്യപ്പെടുത്തുന്നുണ്ട്.

ഉമറിന്റെ ഭരണകാലത്തുമാത്രം നിലവില്‍ വന്ന ഒരു കേന്ദ്രത്തിലേക്കു തിരിഞ്ഞ് ഇതെല്ലാം എങ്ങനെ സംഭവിച്ചുവെന്നാണ് വിമര്‍ശകര്‍ മനസ്സിലാക്കുന്നത്? ‘ഇല്ലാത്ത’ ഒരിടത്തേക്കു തിരിഞ്ഞ് നമസ്‌കരിക്കുന്നതിലെ ‘അനൗചത്യം’ ചൂണ്ടിക്കാട്ടി മുസ്‌ലിംകളെ പരിഹസിക്കുവാന്‍ ഒരു യഹൂദന്‍ പോലും അക്കാലത്ത് രംഗത്തുവരാതിരുന്നത് എന്തുകൊണ്ടാണ്? ജറുസലേമിലെ ബയ്ത്തുല്‍ മക്വ്ദിസ് അന്നുമുണ്ടായിരുന്നുവെന്നതും അതിനെക്കുറിച്ച് അവര്‍ക്കെല്ലാം അറിവുണ്ടായിരുന്നുവെന്നതുമാണ് അതിന്റെ കാരണം. അതുകൊണ്ടാണ് പ്രവാചകാനുചരനായ ബറാഉബ്‌നു ആസ്വിബ് പറഞ്ഞത്: ”യഹൂദന്‍മാരും വേദക്കാരും പ്രവാചകന്‍ നമസ്‌കാരത്തില്‍ ബയ്ത്തുല്‍ മക്വ്ദിസിലേക്കു തിരിയുന്നതില്‍ സന്തുഷ്ടരായിരുന്നു. എന്നാല്‍ നമസ്‌കാരദിശ കഅ്ബയിലേക്ക് അദ്ദേഹം മാറ്റിയപ്പോള്‍ അവര്‍ അതില്‍ അദ്ദേഹത്തെ തള്ളിപ്പറഞ്ഞു.”(11)

യഹൂദന്‍മാര്‍ ബയ്ത്തുല്‍ മക്വ്ദിസ് എന്നുകേട്ട് അത്ഭുതപ്പെട്ടില്ലെന്നു മാത്രമല്ല, മുസ്‌ലിംകള്‍ അതിനെ തങ്ങളുടെ ക്വിബ്‌ലയായി സ്വീകരിച്ചതില്‍ ആവേശഭരിതരാവുക കൂടി ചെയ്തുവെന്നാണ് പരാമൃഷ്ട നിവേദനം തെളിയിക്കുന്നത്. വാസ്തവത്തില്‍, ക്വിബ്‌ല മാറ്റം അറിയിച്ചുകൊണ്ടുള്ള ക്വുര്‍ആന്‍ സൂക്തങ്ങളില്‍ തന്നെ അത് യഹുദരില്‍ സൃഷ്ടിച്ച ഭാവഭേദങ്ങളെ സംബന്ധിച്ച സൂചനകളുണ്ട്. ”അപ്രകാരം നാം നിങ്ങളെ ഒരു ഉത്തമ സമുദായമാക്കിയിരിക്കുന്നു. നിങ്ങള്‍ ജനങ്ങള്‍ക്ക് സാക്ഷികളായിരിക്കുവാനും റസൂല്‍ നിങ്ങള്‍ക്ക് സാക്ഷിയായിരിക്കുവാനും വേണ്ടി. റസൂലിനെ പിന്‍പറ്റുന്നതാരൊക്കെയെന്നും, പിന്‍മാറിക്കളയുന്നതാരൊക്കെയെന്നും തിരിച്ചറിയുവാന്‍ വേണ്ടി മാത്രമായിരുന്നു നീ ഇതുവരെ തിരിഞ്ഞു നിന്നിരുന്ന ഭാഗത്തെ നാം ക്വിബ്‌ലയായി നിശ്ചയിച്ചത്. അല്ലാഹു നേര്‍വഴിയിലാക്കിയവരൊഴിച്ച് മറ്റെല്ലാവര്‍ക്കും അത് ( ക്വിബ്‌ല മാറ്റം ) ഒരു വലിയ പ്രശ്‌നമായിത്തീര്‍ന്നിരിക്കുന്നു. അല്ലാഹു നിങ്ങളുടെ വിശ്വാസത്തെ പാഴാക്കിക്കളയുന്നതല്ല. തീര്‍ച്ചയായും അല്ലാഹു മനുഷ്യരോട് അത്യധികം ദയയുള്ളവനും കരുണാനിധിയുമാകുന്നു. (നബിയേ,) നിന്റെ മുഖം ആകാശത്തേക്ക് തിരിഞ്ഞുകൊണ്ടിരിക്കുന്നത് നാം കാണുന്നുണ്ട്. അതിനാല്‍ നിനക്ക് ഇഷ്ടമാകുന്ന ഒരു ക്വിബ്‌ലയിലേക്ക് നിന്നെ നാം തിരിക്കുകയാണ്. ഇനി മേല്‍ നീ നിന്റെ മുഖം മസ്ജിദുല്‍ ഹറാമിന്റെ നേര്‍ക്ക് തിരിക്കുക. നിങ്ങള്‍ എവിടെയായിരുന്നാലും അതിന്റെ നേര്‍ക്കാണ് നിങ്ങള്‍ മുഖം തിരിക്കേണ്ടത്. വേദം നല്‍കപ്പെട്ടവര്‍ക്ക് ഇത് തങ്ങളുടെ രക്ഷിതാവിങ്കല്‍ നിന്നുള്ള സത്യമാണെന്ന് നന്നായി അറിയാം. അവര്‍ പ്രവര്‍ത്തിക്കുന്നതിനെപ്പറ്റിയൊന്നും അല്ലാഹു അശ്രദ്ധനല്ല. വേദം നല്‍കപ്പെട്ടവരുടെ അടുക്കല്‍ നീ എല്ലാവിധ ദൃഷ്ടാന്തവും കൊണ്ട് ചെന്നാലും അവര്‍ നിന്റെ ക്വിബ്‌ലയെ പിന്തുടരുന്നതല്ല. അവരുടെ ക്വിബ്‌ലയെ നീയും പിന്തുടരുന്നതല്ല. അവരില്‍ ഒരു വിഭാഗം മറ്റൊരു വിഭാഗത്തിന്റെ ക്വിബ്‌ലയെ പിന്തുടരുകയുമില്ല. നിനക്ക് ശരിയായ അറിവ് വന്നുകിട്ടിയ ശേഷം നീയെങ്ങാനും അവരുടെ ഇച്ഛകളെ പിന്‍പറ്റിയാല്‍ നീയും അതിക്രമകാരികളുടെ കൂട്ടത്തില്‍ തന്നെയായിരിക്കും.” (2: 143-145)

ഇസ്രഈല്യര്‍ പൂര്‍വിക കാലം മുതല്‍ക്കുതന്നെ വിശുദ്ധമായി ആദരിച്ചിരുന്നതും അറബികളടക്കമുള്ള കച്ചവടയാത്രികര്‍ക്ക് സുപരിചിതമായിരുന്നതും മുഹമ്മദ് നബി(സ)യും അനുയായികളും വിശുദ്ധമായി അംഗീകരിച്ചിരുന്നതുമായ ഫലസ്ത്വീനിലെ ഒരു ആരാധനാ സ്ഥാനം ബയ്ത്തുല്‍ മക്വ്ദിസ് എന്നോ മസ്ജിദുല്‍ അക്വ്‌സ എന്നോ ഉള്ള പേരില്‍ അറേബ്യയിലുടനീളം അറിയപ്പെട്ടിരുന്നുവെന്നും തന്റെ നിശായാത്ര അവിടെയെത്തിയതിനെക്കുറിച്ചാണ് നബി(സ) സംസാരിച്ചതെന്നുമാണ് ഇവയില്‍ നിന്നെല്ലാം അനിഷേധ്യമാംവിധം ബോധ്യപ്പെടുന്നത്. ഏതായിരുന്നു ആ ആരാധാനാസ്ഥാനം? ഇസ്രാഈലി പൂര്‍വപ്രവാചകന്‍മാര്‍ക്ക് നേതൃത്വം നല്‍കി നബി(സ)ക്ക് അവിടെവെച്ച് നമസ്‌കരിക്കുവാനവസരമുണ്ടായതിലെ പ്രതീകാത്മകത സൂചിപ്പിക്കുന്നതുപോലെ ഇസ്രാഈല്‍ സമൂഹത്തില്‍ അവരുടെ പ്രവാചകന്‍മാരാല്‍ പരിപാലിക്കപ്പെട്ടിരുന്ന ആരാധനാകേന്ദ്രമായിരുന്നു ബയ്ത്തുല്‍ മക്വ്ദിസ്. ബൈബിളും ഇസ്‌ലാമിക പ്രമാണങ്ങളും പരിശോധിച്ചാല്‍ ഇക്കാര്യം മനസ്സിലാകും

. മാനവചരിത്രത്തില്‍ പ്രപഞ്ചനാഥനെ ആരാധിക്കുന്നതിനുവേണ്ടി സംവിധാനിക്കപ്പെട്ട രണ്ടാമത്തെ കേന്ദ്രം ബയ്ത്തുല്‍ മക്വ്ദിസ് ആയിരുന്നുവെന്നാണ് മുഹമ്മദ് നബി (സ) പഠിപ്പിച്ചിട്ടുള്ളത്, ഒന്നാമത്തേത് മക്കയിലെ കഅ്ബയും. അബൂ ദര്‍റ് (റ) നബി(സ)യോട് ചോദിച്ചു. ”അല്ലാഹുവിന്റെ ദൂതരെ, ഏതുപള്ളിയാണ് ഭൂമുഖത്ത് ആദ്യമായി നിര്‍മിക്കപ്പെട്ടത്?” അദ്ദേഹം പറഞ്ഞു, ”മസ്ജിദുല്‍ ഹറാം.” അബൂ ദര്‍റ് (റ) വീണ്ടും ചോദിച്ചു. ”പിന്നെയേതാണ് നിര്‍മിക്കപ്പെട്ടത്?” അദ്ദേഹം പ്രതിവചിച്ചു,  (ജറുസലേമിലെ) മസ്ജിദുല്‍ അക്വ്‌സാ.”  അബൂ ദര്‍റ് (റ) വീണ്ടും ചോദിച്ചു. ”അവ രണ്ടിന്റെയും നിര്‍മാണങ്ങള്‍ക്കിടയിലെ കാലവിളംബമെത്രയായിരുന്നു?” അദ്ദേഹം പറഞ്ഞു, ”നാല്‍പ്പത് വര്‍ഷം.”(12)

ഭൂമുഖത്ത് ആദ്യമായി നിര്‍മിക്കപ്പെട്ട ദൈവാരാധനാകേന്ദ്രമായ കഅ്ബയില്‍ നിന്ന് രണ്ടാമതായി നിര്‍മിക്കപ്പെട്ട ബയ്ത്തുല്‍ മക്വ്ദിസിലേക്കാണ് പ്രവാചകന്റെ (സ) നിശാപ്രയാണം നടന്നതെന്ന് സാരം. കഅ്ബയാണ് ചരിത്രത്തില്‍ ഒന്നാമതായി നിര്‍മിക്കപ്പെട്ട ദൈവാരാധനാ ഭവനമെന്ന് ക്വുര്‍ആന്‍ തന്നെ പ്രസ്താവിക്കുന്നുണ്ട്.(13) ആദിമനുഷ്യനായ ആദമിന്റെ കാലത്തുതന്നെ ഇവ രണ്ടും നിര്‍മിക്കപ്പെട്ടുവെന്നാണ് പണ്ഡിതന്‍മാര്‍ പൊതുവില്‍ മനസ്സിലാക്കുന്നത്. കഅ്ബയുടെ സ്ഥാനത്ത് പിന്നീടുള്ള പുനര്‍നിര്‍മാണങ്ങളും അതിന്റെ പരിപാലനവുമെല്ലാം ഇബ്‌റാഹീം, ഇസ്മാഈല്‍ പ്രവാചകന്‍മാരുടെയും അവരുടെ സന്തതിപരമ്പരകളായ അറബികളുടെയും കൈകളിലൂടെ നടന്നപ്പോള്‍ ബയ്ത്തുല്‍ മക്വ്ദിസിന്റേത് അതിനുചുറ്റും അധിവസിക്കുന്ന ഇസ്രാഈല്യരിലൂടെ നടന്നുവെന്നു മാത്രമേയുള്ളൂ. ഇസ്രാഈല്യര്‍ക്കുവേണ്ടി ബയ്ത്തുല്‍ മക്വ്ദിസിന്റെ പുനര്‍നിര്‍മാണം പൂര്‍ത്തിയാക്കിയത് അവരുടെ രാജാവും പ്രവാചകനും ആയിരുന്ന സുലൈമാന്‍ (അ) ആയിരുന്നുവെന്ന് നബി(സ) പറഞ്ഞതായി അബ്ദുല്ലാഹിബ്‌നു അംറില്‍ നിന്നുള്ള ഒരു നിവേദനത്തിലുണ്ട്.(14)

സോളമന്‍ നിര്‍മിച്ച ആരാധനാലയം എന്ന നിലയില്‍ തന്നെയാണ് യഹൂദര്‍ ബയ്ത്തുല്‍ മക്വ്ദിസിനെ മനസ്സിലാക്കിയത്. ജൂത-ക്രൈസ്തവ സാഹിത്യങ്ങളിലെ ‘The Temple’ സോളമന്‍ രാജാവിനാല്‍ നിര്‍മിക്കപ്പെട്ട ബയ്ത്തുല്‍ മക്വ്ദിസ് ആണ്. അത് നിര്‍മിക്കപ്പെട്ട ഉയര്‍ന്ന പ്രദേശമാണ് ‘Temple Mount’ എന്ന പേരില്‍ യഹൂദ ക്രൈസ്തവ പാരമ്പര്യത്തില്‍ അറിയപ്പെടുന്നത്. ജറുസലേമില്‍ നേരത്തെതന്നെ വിശുദ്ധ സ്ഥാനമായി മനസ്സിലാക്കപ്പെട്ടിരുന്ന Temple Mountല്‍ സോളമന്‍ രാജാവ് പണിതീര്‍ത്ത ദേവാലയമാണ് ബയ്ത്തുല്‍ മക്വ്ദിസ് എന്ന് യഹൂദപാരമ്പര്യവും ബൈബിള്‍ പഴയനിയമവും തെളിയിക്കുന്നുണ്ട്. ഉല്‍പത്തി പുസ്തകത്തിലെ ‘മോറിയാ ദേശത്തെ മലരുകള്‍'(15) ആണ് സോളമന്റെ ദേവാലയത്തിന്റെ ആസ്ഥാനമെന്ന നിലയില്‍ Temple Mount ആയി മാറിയത്. മോറിയാ പ്രദേശം ‘സൃഷ്ടിയുടെ ആരംഭം’ മുതല്‍ക്കുതന്നെ വിശുദ്ധമായി പ്രഖ്യാപിക്കപ്പെട്ടതായിരുന്നുവെന്നാണ് യഹൂദവിശ്വാസം.(16) ആദമിനെ സൃഷ്ടിക്കാനുതകുന്ന മണ്ണ് കര്‍ത്താവ് ഒരുക്കൂട്ടിയതിവിടെയാണെന്നു വിശ്വസിക്കുന്ന യഹൂദന്‍മാരുണ്ട്.(17) മോറിയാ പ്രദേശത്തിന്റെ വിശുദ്ധിയെക്കുറിച്ച ഈ ധാരണ ബൈബിളെഴുത്തുകാരെ ആഴത്തില്‍ സ്വാധീനിച്ചുവെന്ന് കാണാനാകും. അബ്രഹാമിനോട് കര്‍ത്താവ് പുത്രന്‍ ഇസ്ഹാക്വിനെ മോറിയായിലേക്കു കൊണ്ടുപോയി അവിടെവെച്ച് ബലിയറുക്കാനാവശ്യപ്പെട്ടുവെന്നാണ് ഉല്‍പത്തി പക്ഷം.(18)

മാനവചരിത്രത്തിന്റെ നന്നേ തുടക്കം മുതല്‍ക്കുതന്നെ വിശുദ്ധമായി നിശ്ചയിക്കപ്പെട്ട സ്ഥാനമായി ബയ്ത്തുല്‍ മക്വ്ദിസിന്റെ ഭൂമിയെ ഇസ്‌ലാമിക പാരമ്പര്യത്തെപ്പോലെത്തന്നെ യഹൂദപാരമ്പര്യവും മനസ്സിലാക്കുന്നുവെന്നു ചുരുക്കം. പ്രസ്തുത ഭൂമിയില്‍ പിതാവായ ദാവീദ് എല്‍പിച്ചതുപ്രകാരം സോളമന്‍ അതിഗംഭീരമായ ദേവാലയം ഏറ്റവും മികച്ച പണിത്തരങ്ങളുടെയും പണിക്കാരെയും വെച്ചു നിര്‍മിക്കുകയും അതിന്റെ കേന്ദ്രസ്ഥാനത്ത് മോശെക്ക് കര്‍ത്താവ് കല്‍പനകള്‍ എഴുതി നല്‍കിയ പലകകള്‍ സൂക്ഷിച്ചിരുന്ന വിശുദ്ധ പ്രമാണപെട്ടകം (Art of the covenant)സ്ഥാപിക്കുകയും മൃഗബലിയുള്‍പ്പെടെയുള്ള ഇസ്രാഈലി അനുഷ്ഠാനകര്‍മങ്ങളുടെയും പ്രാര്‍ത്ഥനകളുടെയും സിരാകേന്ദ്രമായി ദേവാലയം പരിലസിക്കുകയും ചെയ്തതിനെക്കുറിച്ചുള്ള വിശദമായ വര്‍ണനകള്‍ ബൈബിളെഴുത്തുകാര്‍ നടത്തിയിട്ടുണ്ട്.(19) ഈ ദേവാലയത്തെ യഹൂദന്‍മാര്‍ വിളിച്ചത് ഹീബ്രുവില്‍ ‘ബെയ്ത് ഹാമിക്ദാശ്’ (Beit Hamikdash) എന്നായിരുന്നു;(20) അറബികളുടെ ബയ്ത്ത് അല്‍ മക്വ്ദിസ് തന്നെ.

ഏതാണ് പരിശുദ്ധ ക്വുര്‍ആനും ഹദീഥുകളും നിശാപ്രയാണം ചെന്നെത്തിയെന്നു പറയുന്ന ഫലസ്ത്വീനിയന്‍ ആരാധനാസ്ഥാനമെന്ന് ഇത്രയും വിശദീകരിച്ചതില്‍ നിന്ന് സുതരാം വ്യക്തമാണ്. സോളമന്റെ ദേവാലയം മുഹമ്മദ് നബി(സ)യുടെ കാലത്ത് നിലവിലുണ്ടായിരുന്നില്ല എന്നുപറഞ്ഞുകൊണ്ടാണ് വിമര്‍ശകര്‍ ഈ വസ്തുതകളെ മറികടക്കാന്‍ നോക്കാറുള്ളത്. സോളമന്‍ നിര്‍മിച്ച കെട്ടിടം ഫലസ്ത്വീനില്‍ നിലവിലുണ്ടെന്ന് ക്വുര്‍ആനോ മുഹമ്മദ് നബി(സ)യോ എവിടെയും പറഞ്ഞിട്ടില്ല. പിന്നെ ഈ വിമര്‍ശനത്തിന് എന്ത് പ്രസക്തിയാണുള്ളത്?

സോളമന്‍ നിര്‍മിച്ച ദേവാലയം BCE 586ല്‍ ബാബിലോണിയക്കാര്‍ തകര്‍ത്തതും പിന്നീട് ഏഴു പതിറ്റാണ്ടോളം കഴിഞ്ഞ് യഹൂദര്‍ തല്‍സ്ഥാനത്ത് മറ്റൊരു കെട്ടിടം പണിതതും (second temple) ഹെറോദ് രാജാവ് അതിന് മോടി കൂട്ടിയതും CE 70ല്‍ യഹൂദകലാപകാരികളെ അമര്‍ച്ച ചെയ്യാന്‍ വന്ന റോമന്‍ സൈന്യം അതു തകര്‍ത്തുകളഞ്ഞതും(21) ചരിത്രത്തില്‍ പരക്കെ അറിയപ്പെട്ടതാണ്. First Temple എന്നും Second Temple എന്നും ബൈബിള്‍ പണ്ഡിതന്മാര്‍ വിളിക്കുന്ന കെട്ടിടങ്ങള്‍ Temple Mountല്‍ സംരക്ഷിക്കപ്പെട്ടുകിടക്കുന്നില്ലെന്നും അവ തകര്‍ക്കപ്പെട്ടുവെന്നും മുഹമ്മദ് നബി(സ)യുടെ സമകാലീനരായ മദീനാ യഹൂദന്മാര്‍ക്കും ശാമിലേക്ക് നിരന്തരമായി കച്ചവടയാത്ര പോയിരുന്ന മക്കന്‍ ബഹുദൈവാരാധകര്‍ക്കും അറിയാതിരിക്കുവാന്‍ ചരിത്രപരമായി യാതൊരു നിര്‍വാഹവുമില്ല. എന്നിട്ടും അവരൊന്നും നിശായാത്രാ വിവരണങ്ങളോടോ ബയ്ത്തുല്‍ മക്വ്ദിസിനെ ക്വിബ്‌ലയാക്കാനുള്ള തീരുമാനത്തോടോ ഇക്കാര്യം പറഞ്ഞ് ഏറ്റുമുട്ടാന്‍ വരാതിരുന്നത് നബി(സ) സംസാരിച്ചത് സോളമനോ പില്‍ക്കാല ഇസ്രാഈല്യരോ നിര്‍മിച്ച ഏതെങ്കിലും കെട്ടിടങ്ങളെ കുറിച്ചല്ല, മറിച്ച് ബയ്ത്തുല്‍ മക്വ്ദിസിനെക്കുറിച്ചും മസ്ജിദുല്‍ അക്വ്‌സായെക്കുറിച്ചും ആണ് എന്നതുകൊണ്ടാണ്.

മനുഷ്യചരിത്രത്തിന്റെ ആദ്യ നാളുകള്‍ തൊട്ട് പ്രപഞ്ചനാഥന്റെ നിര്‍ദേശപ്രകാരം അവനെ ആരാധിക്കുവാനായി ഉപയോഗിക്കപ്പെട്ട ഒരു ഭൂമിശാസ്ത്ര ഉണ്‍മയാണ് ഇസ്‌ലാമിക പരിപ്രേക്ഷ്യത്തില്‍ അത്. അവിടെ കെട്ടിടങ്ങള്‍ വിവിധ കാലഘട്ടങ്ങളില്‍ വന്നിട്ടും പോയിട്ടുമുണ്ട്, പക്ഷെ അവയുടെ ചുമരുകള്‍ തകര്‍ന്നാലും നിശ്ചിത അതിര്‍ത്തികള്‍ക്കുള്ളില്‍ അത് മസ്ജിദ് തന്നെയാണ്. ആരാധനകള്‍ നിര്‍വഹിക്കാന്‍ നിര്‍ദേശിക്കപ്പെട്ട സ്ഥലം എന്നു മാത്രമേ സാങ്കേതികമായി മസ്ജിദിന് അര്‍ത്ഥമുള്ളൂ, അത് ചുമരുകളുള്ള കെട്ടിടത്തിനകത്താകണമെന്ന് യാതൊരു നിര്‍ബന്ധവുമില്ല. മുന്‍പ്രവാചകന്മാരില്‍ നിന്ന് വ്യത്യസ്തമായി ഭൂമിയിലെവിടെയും വെച്ച് നിര്‍ബന്ധ നമസ്‌കാരം നിര്‍വഹിക്കാന്‍ തന്റെ അനുയായികള്‍ക്ക് അല്ലാഹു അനുമതി നല്‍കിയിരിക്കുന്നു   എന്ന് പഠിപ്പിച്ചുകൊണ്ട് മുഹമ്മദ് നബി (സ) പറഞ്ഞതിപ്രകാരമാണ്. ”ഭൂമി എനിക്ക് വിശുദ്ധവും ശുദ്ധിയുള്ളതും മസ്ജിദും ആക്കിത്തന്നിരിക്കുന്നു; അതിനാല്‍ പ്രാര്‍ത്ഥനാ സമയങ്ങള്‍ ആഗതമാകുമ്പോള്‍ നിങ്ങള്‍ എവിടെയായിരുന്നാലും അവിടെവെച്ച് നമസ്‌കരിക്കുക.”(22)

ആരാധനാസ്ഥാനങ്ങളാണ് മസ്ജിദുകള്‍. ചുമരുകള്‍ അതിനെ പരിപാലിക്കുന്ന മനുഷ്യരുടെ സംഭാവനയാണ്; അതാരെങ്കിലും കെട്ടിയാലും കെട്ടിയില്ലെങ്കിലും കെട്ടിയവ നിലനിന്നാലും ഇല്ലെങ്കിലും മസ്ജിദായി പ്രഖ്യാപിക്കപ്പെടുകയും നീക്കിവെക്കുകയും ചെയ്ത നിലങ്ങള്‍ മസ്ജിദായി തന്നെ നിലനില്‍ക്കും- ആരാധനകള്‍ നിര്‍വഹിക്കാവുന്ന ഒരു തുണ്ട് ഭൂമി എന്ന അര്‍ത്ഥത്തില്‍. ഭൂമി മുഴുവന്‍ ഇനി മുതല്‍ മസ്ജിദ് ആണെന്ന് മുഹമ്മദ് നബി (സ) പറയുമ്പോള്‍ അദ്ദേഹത്തിന്റെ കാലം മുതല്‍ ഭൂമിയിലെവിടെവെച്ചും നമസ്‌കരിക്കാമെന്ന പുതിയ നിയമം പടച്ചവന്‍ അവതരിപ്പിച്ചിരിക്കുന്നുവെന്നാണ് അതിനര്‍ത്ഥം; അല്ലാതെ ഭൂമിക്ക് ചുമരുകളുണ്ടെന്നല്ല.

ഇതുതന്നെയാണ് ‘ബയ്ത്’ എന്ന പ്രയോഗത്തിന്റെ കാര്യവും. മസ്ജിദ് എന്നു പറയുമ്പോഴുള്ള വിവക്ഷ മാത്രമേ അതിനുമുള്ളുവെന്ന് ക്വുര്‍ആന്‍ പരിശോധിച്ചാല്‍ തന്നെ മനസ്സിലാകും. ഇബ്‌റാഹീം നബി(അ)യും പുത്രന്‍ ഇസ്മാഈല്‍ നബി(അ)യും ചേര്‍ന്നാണ് മക്കയിലെ കഅ്ബ പടുത്തുയര്‍ത്തിയതെന്ന് ക്വുര്‍ആന്‍ ഖണ്ഡിതമായി പ്രസ്താവിക്കുന്നുണ്ട്: ”ഇബ്രാഹീമും ഇസ്മാഈലും കൂടി ആ ഭവനത്തിന്റെ (കഅ്ബയുടെ) അടിത്തറ കെട്ടി ഉയര്‍ത്തിക്കൊണ്ടിരുന്ന സന്ദര്‍ഭവും (അനുസ്മരിക്കുക). (അവര്‍ ഇപ്രകാരം പ്രാര്‍ത്ഥിച്ചിരുന്നു:) ഞങ്ങളുടെ രക്ഷിതാവേ, ഞങ്ങളില്‍ നിന്ന് നീയിത് സ്വീകരിക്കേണമേ. തീര്‍ച്ചയായും നീ എല്ലാം കേള്‍ക്കുന്നവനും അറിയുന്നവനുമാകുന്നു.”(23) ‘ബയ്ത്’ എന്നാണ്, മറ്റുപല ക്വുര്‍ആന്‍ വചനങ്ങളിലുമെന്നപോലെ ഇവിടെയും, കഅ്ബയെ സൂചിപ്പിക്കാന്‍ പ്രയോഗിക്കപ്പെട്ടിരിക്കുന്നത്. ഇബ്‌റാഹീം നബി (അ) ഭാര്യ ഹാജറിനെയും കൈക്കുഞ്ഞായ ഇസ്മാഈലിനെയും (അ) ആള്‍താമസമില്ലാത്ത മക്കയില്‍ കൊണ്ടുവന്നു താമസിപ്പിച്ച് സ്വദേശത്തേക്കു മടങ്ങിയും മക്കയില്‍ സംസം ഉറവയുണ്ടായതും അതിനുചുറ്റും ജനവാസമാരംഭിച്ചതും ഇസ്മാഈല്‍ (അ) വളര്‍ന്നുവലുതായതും അപ്പോള്‍ ഇബ്‌റാഹീം (അ) മടങ്ങിവന്നു മകനെയും കൂട്ടി കഅ്ബ നിര്‍മിച്ചതുമെല്ലാം ക്വുര്‍ആനിലും ഹദീഥുകളിലും വിശദമായി വിവരിക്കപ്പെട്ടിട്ടുണ്ട്. ആദം നബി(അ)യുടെ കാലത്ത് ആരാധനലായമുണ്ടായിരുന്ന സ്ഥാനം കണ്ടെത്തി ഇബ്‌റാഹീമും ഇസ്മാഈലും കഅ്ബ നിര്‍മിച്ചുവെന്നര്‍ത്ഥം. ഇബ്‌റാഹീം ഹാജറയെ മക്കയില്‍ കൊണ്ടുവന്നാക്കുമ്പോള്‍ അവിടെ കഅ്ബ എന്ന ഒരു നിര്‍മിതിയില്ല. എന്നാല്‍ ആ സമയത്തെ ഇബ്‌റാഹീമിന്റെ പ്രാര്‍ത്ഥന ക്വുര്‍ആന്‍ വിവരിക്കുന്നതിങ്ങനെയാണ്. ”ഞങ്ങളുടെ രക്ഷിതാവേ, എന്റെ സന്തതികളില്‍ നിന്ന് (ചിലരെ) കൃഷിയൊന്നും ഇല്ലാത്ത ഒരു താഴ്‌വരയില്‍, നിന്റെ പവിത്രമായ ഭവനത്തിന്റെ (കഅ്ബയുടെ) അടുത്ത് ഞാനിതാ താമസിപ്പിച്ചിരിക്കുന്നു. ഞങ്ങളുടെ രക്ഷിതാവേ, അവര്‍ നമസ്‌കാരം മുറപ്രകാരം നിര്‍വഹിക്കുവാന്‍ വേണ്ടിയാണ് (അങ്ങനെ ചെയ്തത്). അതിനാല്‍ മനുഷ്യരില്‍ ചിലരുടെ മനസ്സുകളെ നീ അവരോട് ചായ്‌വുള്ളതാക്കുകയും, അവര്‍ക്ക് കായ്കനികളില്‍ നിന്ന് നീ ഉപജീവനം നല്‍കുകയും ചെയ്യേണമേ. അവര്‍ നന്ദി കാണിച്ചെന്നു വരാം.”(24)

ഇവിടെ നിന്റെ പവിത്രമായ ‘ബയ്ത്തി’നടുത്തുള്ള താഴ്‌വരയില്‍ അവരെ പാര്‍പ്പിച്ചിരിക്കുന്നുവെന്നാണ് ഇബ്‌റാഹീം പടച്ചവനോടു പറയുന്നത്. അദ്ദേഹം കഅ്ബ നിര്‍മിക്കുന്നതിനു മുമ്പാണിതെന്നോര്‍ക്കണം. അപ്പോള്‍, അല്ലാഹുവിനെ ആരാധിക്കാന്‍ വേണ്ടി തെരഞ്ഞെടുക്കപ്പെട്ട ഒരു സ്ഥലം അവിെടയുണ്ടായിരുന്നു, എടുപ്പുകളൊക്കെ ഇല്ലാതായെങ്കിലും ‘ബയ്ത്’ ബയ്തല്ലാതെയായില്ല. ഇതുതന്നെയാണ് ഇസ്‌ലാമിക വീക്ഷണത്തില്‍ ബയ്ത്തുല്‍ മക്വ്ദിസിന്റെയും കാര്യം. സോളമന്റെ എടുപ്പുകള്‍ വന്നതോ പോയതോ അതിന്റെ ബയ്ത്/മസ്ജിദ് എന്ന സ്ഥാനത്തെ ഒരു നിലക്കും ബാധിക്കുന്നില്ല. അതിന്റെ അതിര്‍ത്തികള്‍ക്കുള്ളിലെ സ്ഥലം വിശുദ്ധമായ ഒരു ആരാധനാകേന്ദ്രം അഥവാ ബയ്ത്/മസ്ജിദ് ആകുന്നു; അങ്ങോട്ടു പോകുവാനുള്ള അനര്‍ഘ സൗഭാഗ്യം കൊണ്ടാണ് അന്ത്യപ്രവാചകനായ മുഹമ്മദ് നബി (സ) ഇസ്രാഇന്റെ രാത്രിയില്‍ അനുഗ്രഹിക്കപ്പെട്ടത്.

ഒന്നും രണ്ടും ടെമ്പിളുകള്‍ തകര്‍ക്കപ്പെട്ടതിനുശേഷം ‘Temple Mount’ യഹൂദസമൂഹത്തില്‍ വിഭാവനം ചെയ്യപ്പെട്ടതെങ്ങനെയാണെന്നുകൂടി പരിശോധിച്ചാല്‍ ഇവ്വിഷയകമായുള്ള വിമര്‍ശനങ്ങളുടെ അര്‍ത്ഥശൂന്യത പൂര്‍ണമായി ബോധ്യമാകും. റോമന്‍ ആക്രമണത്തില്‍ രണ്ടാം ദേവാലയം തകര്‍ക്കപ്പെട്ടതിനുശേഷം അവിടെ വേറെ ഏര്‍പ്പാടുകള്‍ക്കൊന്നും ആരും മുതിര്‍ന്നതായി കാണുന്നില്ല. ദേവാലയത്തിന് പവിത്രത കല്‍പിച്ചിരുന്ന യഹൂദന്‍മാര്‍ക്ക് ഫലസ്ത്വീന്‍ നഷ്ടപ്പെട്ടതു തന്നെയായിരുന്നു അതിന്റെ അടിസ്ഥാന കാരണം. ജറുസലേം പിന്നീട് റോമക്കാരുടെയും ബൈസന്റൈന്‍ സാമ്രാജ്യത്തിന്റെയും നിയന്ത്രണത്തിലേക്കാണ് പോയത്. ദേവാലയം നിലനിന്നിരുന്ന സ്ഥലത്തെ ബോധപൂര്‍വം അവഗണിച്ച് അത് പാഴ്ഭൂമിയാക്കാനായിരുന്നു അത് തച്ചുതകര്‍ത്ത റോമക്കാരുടെ ശ്രമം. അവരുടെ അനന്തരാവകാശികളായി വന്ന ബൈസന്റൈന്‍ ഭരണാധികാരികള്‍ ക്രിസ്ത്യാനികളായിരുന്നുവെങ്കിലും പഴയ നിയമപാഠങ്ങളെ പിരിഞ്ഞ് റോമന്‍ പൈതൃകത്തെ പുണര്‍ന്നിരുന്ന അവരും ഈ അവഗണന തുടര്‍ന്നു.

വിശുദ്ധ ദേവാലയ ഭൂമിയില്‍ പ്രാചീനമായ എടുപ്പുകളുടെ അവശിഷ്ടങ്ങള്‍ അനാഥമായിക്കിടന്ന ഈ കാലഘട്ടത്തിലാണ് മുഹമ്മദ് നബി(സ)യുടെ ഇസ്രാഅ് സംഭവിക്കുന്നത്; ക്രൈസ്തവര്‍ ആ സ്ഥലത്തിന്റെ വിശുദ്ധി പോലും മറന്നുതുടങ്ങിയിരുന്ന സന്ദര്‍ഭത്തില്‍! എന്നാല്‍ യഹൂദര്‍ അങ്ങനെയായിരുന്നില്ല. ദേവാലയഭൂമിയെക്കുറിച്ച വികാരസാന്ദ്രമായ ഓര്‍മകള്‍ പേറിയാണ് ഫലസ്ത്വീനില്‍നിന്ന് പുറത്താക്കപ്പെട്ട യഹൂദരുടെ തലമുറകള്‍ ലോകത്തിന്റെ വ്യത്യസ്ത ഭാഗങ്ങളില്‍ ജീവിച്ചത്. എന്തായിരുന്നു രണ്ടാം ദേവാലയം തകര്‍ക്കപ്പെട്ടതിനുശേഷം റോമന്‍-ബൈസന്റൈന്‍ അതിക്രമകാരികള്‍ ചപ്പുചവറുകള്‍ നിറച്ചവഹേളിച്ച മോറിയ കുന്നിനെക്കുറിച്ച് അവരുടെ വിശ്വാസം? ഇസ്‌ലാമിക സങ്കല്‍പം പോലെത്തന്നെ അവര്‍ക്കും അത് വിശുദ്ധസ്ഥാനം തന്നെയായി നിലനിന്നുവെന്നത് മാത്രമാണുത്തരം.

പതിനൊന്നാം നൂറ്റാണ്ടില്‍ ജീവിച്ച ലോകപ്രശസ്തനായ യഹൂദ റബ്ബി മയ്‌മോണിഡസിന്റെ (Maimonides) വാക്കുകളില്‍ പറഞ്ഞാല്‍ ”ബെയ്ത് ഹാമിക്ദാശ് ദൈവത്തിന്റെ അനശ്വര ഭവനമാണ്; ദേവാലയം തകര്‍ക്കപ്പെട്ടെങ്കിലും മോറിയാ കുന്നില്‍നിന്നും ‘ദൈവസാന്നിധ്യം’ പോയിട്ടില്ല; ‘മിശിഹ’ വരുമ്പോള്‍ അദ്ദേഹം അത് പുനര്‍നിര്‍മിക്കും.”(25) മുഹമ്മദ് നബി(സ)യുടെ മരണം കഴിഞ്ഞ് നൂറ്റാണ്ടുകള്‍ പിന്നിട്ടിട്ടും യഹൂദപരികല്‍പന അങ്ങനെതന്നെ നിലനിന്നുവെന്നണിതിനര്‍ത്ഥം. അവിടെച്ചെന്ന് ഇസ്രാഈലി പ്രവാചകന്‍മാരുടെ കൂടെ നമസ്‌കരിക്കുവാന്‍ നബി(സ)ക്ക് അല്ലാഹു അവസരമുണ്ടാക്കിക്കൊടുത്തതില്‍ പിന്നെയെന്ത് അസാംഗത്യമാണുള്ളത്?

ദേവാലയഭൂമി നഷ്ടപ്പെട്ട് പുരോഹിതാസ്ഥാനമില്ലാതായതോടെയാണ് സാധാരണക്കാര്‍ക്ക് മതം പഠിക്കാന്‍ യഹൂദ റബ്ബിമാര്‍ മിശ്‌നയും (Mishnah), ഗമറയുമെല്ലാം (Gemara) രചിച്ചുതുടങ്ങുന്ന സാഹചര്യമുണ്ടായത്. CE ആറാം നൂറ്റാണ്ടുവരെയുള്ള ഇക്കപം പണ്ഡിതരചനകളുടെ സമാഹാരമാണ് തല്‍മൂദ് (Talmud) എന്നറിയപ്പെടുന്നത്. തല്‍മൂദില്‍നിന്നുള്ള ഒരു ഭാഗം ഇപ്രകാരമാണ്: ”വിദേശത്ത് പ്രാര്‍ത്ഥനയില്‍ നില്‍ക്കുന്നവന്‍ ഹൃദയം കേന്ദ്രീകരിക്കേണ്ടത് ഇസ്രയേല്‍ രാജ്യത്തിലേക്കാണ്; കാരണം, ”അവര്‍ അങ്ങ് അവരുടെ പിതാക്കന്‍മാര്‍ക്ക് ദാനം ചെയ്ത ദേശത്തേക്കുനോക്കി അങ്ങയോടു പ്രാര്‍ത്ഥിച്ചാല്‍” (1 രാജാക്കന്‍മാര്‍ 8: 48) എന്നു പറഞ്ഞിരിക്കുന്നു. ഇസ്രായേല്‍ ദേശത്തുള്ളയാള്‍ പ്രാര്‍ത്ഥനയില്‍ ജറുസലേമിലേക്കാണ് ഹൃദയം കേന്ദ്രീകരിക്കേണ്ടത്; ‘അങ്ങ് തെരഞ്ഞെടുത്തിരിക്കുന്ന ഈ നഗരത്തിന് അഭിമുഖമായി നിന്ന് പ്രാര്‍ത്ഥിച്ചാല്‍’ (1 രാജാക്കന്‍മാര്‍ 8: 44) എന്നുമുണ്ട്. ഒരാള്‍ ജറുസലേമിലാണെങ്കില്‍ അയാര്‍ ഹൃദയം കേന്ദ്രീകരിക്കേണ്ടത് ബെയ്ത് ഹാമിക്ദാശിമിലേക്കാണ്. -‘അവര്‍ ഈ ഭവനത്തിലേക്ക് പ്രാര്‍ത്ഥിക്കണം’ (2 ദിനവൃത്താന്തം 6: 32) എന്നാണ് പറഞ്ഞിരിക്കുന്നത്.”(26)

എന്താണിതിനര്‍ത്ഥം? അതെ, ചുമരുകള്‍ താഴെ വീണാല്‍ ഇല്ലാതാകുന്നതായിരുന്നില്ല മതബോധമുള്ള ഇസ്രാഈല്യന് ബയ്തുല്‍ മക്വ്ദിസ്. കെട്ടിയുയര്‍ത്തിയ സൗധം നിലം പൊത്തിയെങ്കിലും അതിനെ ആരാധനാകേന്ദ്രമായി തന്നെ അവന്‍ കണ്ടു. വിശുദ്ധി അതുപോലെ നിലനില്‍ക്കുന്നതായി മനസ്സിലാക്കി, ലോകത്തിന്റെ പല ഭാഗങ്ങളിലേക്ക് റോമന്‍ പീഡനം കാരണമായി ചിതറേണ്ടി വന്നപ്പോഴും അവിടേക്കു തിരിഞ്ഞു തന്നെ പ്രാര്‍ത്ഥിച്ചു. ബയ്തുല്‍ മക്വ്ദിസ് എന്നുതന്നെ പലപ്പോഴും ആ മണ്ണിനെ വിളിച്ചു. ദേവാലയത്തിന്റെ കെട്ട് തകര്‍ത്തെന്നഹങ്കരിച്ചവര്‍ക്ക് ദേവാലയഭൂമിയും തങ്ങള്‍ക്ക് ദേവാലയമാണെന്ന് യഹൂദവൈകാരികത കാണിച്ചുകൊടുത്തു. അതേ അര്‍ത്ഥത്തില്‍ തന്നെയാണ് നബി(സ)യും ബയ്തുല്‍ മക്വ്ദിസില്‍ പോയെന്നു പറഞ്ഞത്; ക്വുര്‍ആന്‍ മസ്ജിദുല്‍ അക്വ്‌സ എന്ന വിശേഷണം നടത്തിയത്.

സുലയ്മാന്‍ നബി (അ) നിര്‍മിച്ച കെട്ടിടം തകര്‍ന്നതുകൊണ്ടോ ക്രൈസ്തവര്‍ പാഴ്ഭൂമിയായി ഉപേക്ഷിച്ചതുകൊണ്ടോ ബയ്തുല്‍ മക്വ്ദിസ് അതല്ലാതാവുകയില്ല എന്നതുകൊണ്ടു തന്നെ, തീര്‍ത്ഥയാത്ര പോകാവുന്ന വിശുദ്ധ കേന്ദ്രങ്ങളിലൊന്നായാണ് മുഹമ്മദ് നബി (സ) അനുചരന്‍മാര്‍ക്ക് മസ്ജിദുല്‍ അക്വ്‌സയെ പരിചയപ്പെടുത്തിക്കൊടുത്തത്. അദ്ദേഹം പറഞ്ഞു: ”മൂന്ന് മസ്ജിദുകള്‍ സന്ദര്‍ശിക്കാനല്ലാതെ നിങ്ങള്‍ തീര്‍ത്ഥയാത്ര പോകരുത്. ഈ എന്റെ മസ്ജിദ് (മദീനയിലെ മസ്ജിദുന്നബവി), (മക്കയിലെ) മസ്ജിദുല്‍ ഹറാം, മസ്ജിദുല്‍ അക്വ്‌സ.”(27)

മുസ്‌ലിംകളുടെ തീര്‍ത്ഥാടനകേന്ദ്രമായി നിശ്ചയിക്കപ്പെട്ടതുകൊണ്ടു തന്നെ രണ്ടാം ഖലീഫ ഉമറിന്റെ (റ) കാലത്ത് ഫലസ്ത്വീന്‍ ബൈസന്റൈന്‍കാരില്‍നിന്ന് മുസ്‌ലിം സൈന്യത്തിന്റെ നിയന്ത്രണത്തിലേക്കു വന്നപ്പോള്‍ ഉമര്‍ (റ) അവിടം സന്ദര്‍ശിക്കുകയും വൃത്തിയാക്കുകയും നമസ്‌കാര സൗകര്യമുളള ഒരു കെട്ടിടം നിര്‍മിക്കുകയും ചെയ്തു. ആ കെട്ടിടമല്ല മസ്ജിദുല്‍ അക്വ്‌സ, പ്രത്യുത മസ്ജിദുല്‍ അക്വ്‌സയായി ക്വുര്‍ആന്‍ വിശേഷിപ്പിച്ച സ്ഥലത്ത് ഒരു ഭാഗത്തായി നിര്‍മിക്കപ്പെട്ട ഒരു സൗകര്യവും സംവിധാനവും മാത്രമായിരുന്നു പ്രസ്തുത കെട്ടിടം.(28) അതിനു പിന്നീടുണ്ടായ രൂപാന്തരങ്ങളോ വികാസങ്ങളോ ചുരുക്കങ്ങളോ ഒന്നുമല്ല മസ്ജിദുല്‍ അക്വ്‌സയുടെ അകവും പുറവും നിര്‍ണയിക്കുന്നത്. കഅ്ബക്ക് ചുറ്റും നമസ്‌കരിക്കുന്നതിനായി ഉമവികളോ അബ്ബാസികളോ തുര്‍ക്കികളോ സുഊദികളോ പണികഴിപ്പിച്ച കെട്ടിടങ്ങളല്ല  മറിച്ച് ആ സ്ഥലമാണ് മസ്ജിദുല്‍ ഹറാം എന്നതുപോലെത്തന്നെ. ഇതുമനസ്സിലാക്കാതെയാണ് വിമര്‍ശകര്‍ മസ്ജിദുല്‍ അക്വ്‌സയും ഉമര്‍(റ) നിര്‍മിച്ച പള്ളിയും ഒന്നാണെന്ന് തെറ്റിദ്ധരിക്കുന്നത്.

ശ്രദ്ധിക്കുക: ഇസ്രാഅ് വിവരണങ്ങളില്‍ വിശ്വസിച്ച, ബയ്ത്തുല്‍ മക്വ്ദിസിലേക്ക് തിരിഞ്ഞ് ഒന്നര വര്‍ഷത്തോളം നബി(സ)ക്കൊപ്പം നമസ്‌കരിച്ച, മസ്ജിദുല്‍ അക്വ്‌സ തീര്‍ത്ഥാടന കേന്ദ്രമായി പ്രവാചകനാല്‍ പഠിപ്പിക്കപ്പെട്ട ഉമര്‍ (റ) ഫലസ്ത്വീനില്‍ എത്തിയപ്പോള്‍ മസ്ജിദുല്‍ അക്വ്‌സ എന്ന പേരില്‍ കറതീര്‍ന്ന ഒരു കെട്ടിടം കാണാതെ അത്ഭുത പരതന്ത്രനായിട്ടില്ല. ആരാധനകള്‍ക്ക് സൗകര്യപ്പെടുംവിധമുള്ള കെട്ടിടങ്ങളൊന്നും മസ്ജിദുല്‍ അക്വ്‌സയുടെ സ്ഥാനത്ത് നിലവിലില്ലെന്നു അറിയാവുന്നതുകൊണ്ട് സന്ദര്‍ശനാവസരത്തില്‍ തികച്ചും സ്വാഭാവികമായി പെരുമാറുകയും അവിടെയൊരു പള്ളി നിര്‍മിക്കാന്‍ മുന്‍കയ്യെടുക്കുകയുമാണ് അദ്ദേഹം ചെയ്തത്.

അതെ, ഒന്നാം ടെമ്പിളോ, രണ്ടാം ടെമ്പിളോ ഫലസ്ത്വീനില്‍ നില്‍ക്കുന്നതായി പ്രവാചകനോ അനുചരന്‍മാരോ ഒന്നും മനസ്സിലാക്കിയിരുന്നില്ല; അവയെ അല്ല അവരാരും മസ്ജിദുല്‍ അക്വ്‌സ എന്നോ ബയ്ത്തുല്‍ മക്വ്ദിസ് എന്നോ വിളിച്ചത്. അതിനാല്‍ ഇസ്രാഅ് വിവരണത്തില്‍ കാലാനുചിതത്വമുെണ്ടന്ന ആരോപണത്തില്‍ യാതൊരു കഴമ്പുമില്ല

കുറിപ്പുകള്‍

  1. ക്വുര്‍ആന്‍ 17: 1.
  2. ബുഖാരി, സ്വഹീഹ്/കിതാബു മനാകിബില്‍ അന്‍സ്വാര്‍.
  3. മുസ്‌ലിം, സ്വഹീഹ്/കിതാബുല്‍ ഈമാന്‍.
  4. മുസ്‌ലിം, സ്വഹീഹ്/കിതാബുല്‍ ഈമാന്‍.
  5. മുസ്‌ലിം, സ്വഹീഹ്/കിതാബുല്‍ ഈമാന്‍.
  6. ഹാകിം, മുസ്തദ്‌റക് അലസ്സ്വഹീഹയ്ന്‍. see Ali Muhammad As-Sallabee, The Noble life of the Prophet (Riyadh. Darussalam, 2005), vol. 1, pp 552-3.
  7. ബുഖാരി, സ്വഹീഹ്/കിതാബു ബദീല്‍ ഹല്‍ക്വ്.
  8. ക്വുര്‍ആന്‍ 2: 143-145.
  9. ബുഖാരി, സ്വഹീഹ്/കിതാബുല്‍ ഈമാന്‍.
  10. S.Moinul Haq, Ibn sa’d’s Al Tabaqat Al Kabir (New Delhi. kitab bhavan, 2009), vol. 1, 284.
  11. ബുഖാരി, സ്വഹീഹ്/കിതാബുല്‍ ഈമാന്‍.
  12. ബുഖാരി, സ്വഹീഹ്/കിതാബുല്‍ അമ്പിയാഅ്.
  13. ക്വുര്‍ആന്‍ 3: 96.
  14. അഹ്മദ് അന്നസാഇ, സുനനുസ്സുഗ്‌റ/കിതാബുല്‍ മസാജിദ്; യസീദ് ഇബ്‌നു മാജ, സുനന്‍/ കിതാബു ഇക്വാമതിസ്സ്വലാതി വസ്സുന്നതു ഫീഹാ.
  15. ബൈബിള്‍; ഉല്‍പത്തി 22: 2
  16. Rabbi Menachan M Sohneerson, ‘G-d’s chosen House-www,chabadozy
  17. see Jochua Hammes, ‘what is beneadh the temple mount?’, www.smithsonianmag.com
  18. ഉല്‍പത്തി 22: 2.ബൈബിള്‍, 1 ദിനവൃത്താന്തം അധ്യായങ്ങള്‍ 28, 29. 2 ദിനവൃത്താന്തം അധ്യായങ്ങള്‍ 1 – 7.
  19. www.chabad.org.op.cit
  20. see Temple Mount, the Temple Jerusalem during first Temple, second Temple of in www.jeuishviofnallibray.org
  21. ബുഖാരി, സ്വഹീഹ്/കിതാബുല്‍ മസാജിദി വ മവാദിഇ സ്സ്വലാത്ത്.
  22. ക്വുര്‍ആന്‍ 2: 127.
  23. ക്വുര്‍ആന്‍ 14: 37.
  24. See www.chabad.org.op.cit
  25. The William Davidson Talmud, berakhot Daf 30 a.

പ്രവാചകത്വപൂര്‍വകാലത്ത് പ്രവാചകന് അടുത്ത പരിചയമുണ്ടായിരുന്ന സയ്ദ്ബ്‌നു ഹാരിഥയില്‍ നിന്നും വറക്വത്ബ്‌നു നൗഫലില്‍ നിന്നുമാണ് ജൂതമതത്തെയും ക്രിസ്തുമതത്തെയും സംബന്ധിച്ച അറിവുകള്‍ പ്രവാചകന്‍ ശേഖരിച്ചത്. വേദക്കാരെ സംബന്ധിച്ചും അവരുടെ പ്രവാചകന്‍മാരെ സംബന്ധിച്ചുമുള്ള ക്വുര്‍ആനിക ദര്‍ശനം ഈ അറിവുകളെയാണ് പ്രതിഫലിപ്പിക്കുന്നത്. മക്കയില്‍ പ്രവാചകന്റെ ജീവിതപരിസരത്തുതന്നെ എപ്പോഴുമുണ്ടായിരുന്ന സയ്ദിന്റെയും വറക്വയുടെയും ക്രൈസ്തവപശ്ചാത്തലം ക്വുര്‍ആനിന്റെ മാനുഷിക സ്രോതസ്സുകളിലേക്ക് കൃത്യമായി വെളിച്ചം വീശുന്നുണ്ട്. ‘സിറിയന്‍ യാത്ര’കളെപ്പോലെത്തന്നെ ഓറിയന്റലിസ്റ്റുകളും മിഷനറിമാരും ക്വുര്‍ആനിന്റെ ദൈവികതയെ നിരാകരിക്കുവാന്‍ വേണ്ടി അവതരിപ്പിക്കുന്ന പരാമൃഷ്ട സയ്ദ്-വറക്വ സിദ്ധാന്തങ്ങളും കേവലം ഒരു മിത്താണോ?

തെ. മുഹമ്മദ് നബി (സ) തന്റെ ജീവിതത്തില്‍ കണ്ടുമുട്ടിയ മനുഷ്യരുടെ പട്ടിക അദ്ദേഹത്തെ സംബന്ധിച്ച ജീവചരിത്രപരമായ അറിവ് വര്‍ധിപ്പിക്കുമെന്നല്ലാതെ ക്വുര്‍ആനിക വചനങ്ങളുടെ ദൈവികതയെ നിഷേധിക്കുവാന്‍ ഒരു നിലക്കും ഉപകാരപ്പെടുകയില്ലെന്ന് ഓറിയന്റലിസ്റ്റുകള്‍ നിര്‍മിച്ച സിറിയന്‍ യാത്രാ കഥകള്‍ ചര്‍ച്ചയ്‌ക്കെടുത്തപ്പോള്‍ നാം വിശദമായി വ്യക്തമാക്കിയതാണ്. പരിശുദ്ധ ക്വുര്‍ആനിന്റെ ഉള്ളടക്കവും ശൈലിയും മനുഷ്യനിഷ്പന്നമാവുക അസാധ്യമാണ് എന്നതിനാല്‍ സയ്ദില്‍ നിന്നും വറക്വയില്‍ നിന്നും ക്രിസ്തുമതജ്ഞാനം പ്രവാചകന് കിട്ടിയിരുന്നു എന്ന വിമര്‍ശകസിദ്ധാന്തത്തെ അപ്പടി സ്വീകരിച്ചാല്‍ പോലും അത് ക്വുര്‍ആനിലെ പൂര്‍വപ്രവാചക കഥനങ്ങള്‍ക്കോ ജൂത-ക്രിസ്തുമതാപഗ്രഥനങ്ങള്‍ക്കോ ഉള്ള വിശദീകരണമാവുകയില്ല. പ്രവാചകന്‍ ഒരു മനുഷ്യന്‍ എന്ന നിലയില്‍ തന്റെ ചുറ്റുപാടുമുള്ളവരില്‍ നിന്ന് എന്തെല്ലാം പഠിച്ചു എന്നതല്ലാതെ, മറിച്ച് വെളിപാടുകളായി അദ്ദേഹം ലോകത്തിനുമുന്നിലവരതിപ്പിച്ച വചനങ്ങളുടെ സ്രോതസ്സ് ദൈവമല്ലാതെ മറ്റാരാണ് എന്നതാണ് മര്‍മപ്രധാനമായ വിഷയമെന്ന കാര്യം പോലും ഇത്തരം ‘കണ്ടെത്തലുകള്‍’ക്കായുള്ള ‘ഗവേഷണ’ബഹളങ്ങള്‍ക്കിടയില്‍ നബിവിമര്‍ശകര്‍ മറന്നുപോവുകയാണ്.

പ്രവാചകന്‍ പ്രകൃതിയില്‍ നിന്നോ സഹജീവികളില്‍ നിന്നോ നേടിയെടുത്ത വിവരങ്ങളൊന്നുമല്ല ക്വുര്‍ആന്‍ വചനങ്ങളായി അദ്ദേഹം ലോകത്തെ കേള്‍പിച്ചത് എന്ന് ക്വുര്‍ആനിന്റെ ഉള്ളടക്കവും ശൈലിയും എത്ര നിശിതമായി വിശകലനം ചെയ്താലും ആര്‍ക്കും ബോധ്യപ്പെടും. പ്രസ്തുത വിശകലനത്തിനാണ് നബിവിമര്‍ശകര്‍ ഒന്നാമതായി സന്നദ്ധമാകേണ്ടത്. ക്വുര്‍ആനിന്റെ രചന പ്രവാചകനിലോ ചുറ്റുമുള്ളവരിലോ ആരോപിക്കുവാന്‍ ചരിത്രം ചികയുന്നത് വ്യര്‍ത്ഥമാണെന്ന് ആ വിശകലനം അവരെ ബോധ്യപ്പെടുത്താതിരിക്കില്ല. ഒരു സയ്ദിനെയോ വറക്വയെയോ പരിചയമുണ്ടായാല്‍ നടക്കുന്നതായിരുന്നു ക്വുര്‍ആനിന്റെ രചനയെങ്കില്‍ അത്തരത്തിലുള്ള പലരെയും പരിചയമുണ്ടായിരുന്ന, എത്രപേരെ വേണമെങ്കിലും സംഘടിപ്പിക്കാമായിരുന്ന അറേബ്യന്‍ സാഹിത്യസാമ്രാട്ടുകള്‍ക്ക് ക്വുര്‍ആനിനെപ്പോലൊരു ഗ്രന്ഥം കൊണ്ടുവരാനുള്ള പ്രവാചകന്റെ വെല്ലുവിളിയെ വളരെയളുപ്പത്തില്‍ മറികടക്കാന്‍ കഴിയുമായിരുന്നുവെന്നതാണ് വസ്തുത. അങ്ങനെയല്ല പക്ഷേ സംഭവിച്ചതെന്ന് ചരിത്രത്തില്‍ നിന്ന് വ്യക്തമാണല്ലോ.

മക്കയില്‍ ക്രൈസ്തവരുടെ സാമൂഹ്യസാന്നിധ്യമുണ്ടായിട്ടില്ല എന്നും സിറിയന്‍ യാത്രകളില്‍ നിന്നാണ് പ്രവാചകന്‍ ജൂത-ക്രൈസ്തവ മതങ്ങളെക്കുറിച്ചുള്ള അറിവുകള്‍ ക്വുര്‍ആനില്‍ ഉള്‍ചേര്‍ത്തതെന്ന സിദ്ധാന്തം അത്യന്തം പരിഹാസ്യമാണെന്നും ബോധ്യമുള്ളതുകൊണ്ടാണ് മക്കയില്‍ തന്നെ പ്രവാചകന് ‘വിവരദായകരെ’ സങ്കല്‍പിക്കുവാന്‍ കഴിയുന്നതിലുള്ള ആവേശത്തോടെ ചില നബിവിമര്‍ശകര്‍ സയ്ദിന്റെയും വറക്വയുടെയും കഥകള്‍ അവതരിപ്പിക്കുന്നത്. യഥാര്‍ത്ഥത്തില്‍ ക്വുര്‍ആനിന്റെ രചനക്ക് ഒരു നിലയ്ക്കും വിശദീകരണമാകാന്‍ കഴിയാത്ത, ഒരു മനുഷ്യന്‍ എന്നനിലയില്‍ പലരില്‍ നിന്നും പലതും മനസ്സിലാക്കിയ  കൂട്ടത്തില്‍ ഈ രണ്ടുപേരില്‍ നിന്നും പ്രവാചകന്‍ ജൂത-ക്രൈസ്തവ വിശ്വാസങ്ങളെക്കുറിച്ച് ചിലതെല്ലാം മനസ്സിലാക്കിയിട്ടുണ്ടാകാം എന്ന ജീവചരിത്രപരമായ അറിവുമാത്രമാണ് ഈ ഗവേഷണം എത്ര കടന്നാലും സ്ഥാപിക്കുക എന്ന് നാം സൂചിപ്പിച്ചു. എന്നാല്‍ ആ ‘അറിവു’പോലും സയ്ദിനെയും വറക്വയെയും സംബന്ധിച്ച് പ്രവാചകനുമായി ബന്ധപ്പെടുത്തി ചരിത്രം നല്‍കുന്നില്ല എന്നതാണ് വാസ്തവം.

ആരാണ് സയ്ദ്ബ്‌നു ഹാരിഥ (റ)? ഇസ്‌ലാമിക ചരിത്രത്തിലെ സുപ്രസിദ്ധനായ  പ്രവാചകാനുചരന്‍മാരിലൊരാള്‍. അദ്ദേഹം പ്രവാചകന് എന്തെങ്കിലും ക്രിസ്തുമതാറിവുകള്‍ പകര്‍ന്നു നല്‍കിയതായി വ്യക്തമാക്കുന്ന ഒരൊറ്റ ചരിത്രരേഖയുമില്ല. ഇസ്‌ലാം പൂര്‍വകാലത്ത് ജൂതനോ ക്രിസ്ത്യാനിയോ ആയി ജീവിച്ച വ്യക്തിയുമല്ല അദ്ദേഹം. അദ്ദേഹത്തിന്റെ മാതാപിതാക്കളും ജൂതന്‍മാരോ ക്രിസ്ത്യാനികളോ അല്ല. നജ്ദിലെ കല്‍ബ് ഗോത്രത്തില്‍ യമനീ വേരുകളുള്ള ഒരു അറബ് കുടുംബത്തില്‍ ഒരു സാധാരണ അറബിയായിട്ടാണ് സയ്ദ് ജനിച്ചതും വളര്‍ന്നതും. പിതാവ് ഹാരിഥയുടെയും മാതാവ് സുഅ്ദ ബിന്‍ത് ഥഅ്‌ലബയുടെയും കുടുംബങ്ങള്‍ തമ്മിലുണ്ടായ ചില തര്‍ക്കങ്ങളെ തുടര്‍ന്ന് ഉമ്മയുടെ കയ്യില്‍നിന്ന് ചില അക്രമികള്‍ കേവലം എട്ടുവയസ്സിനോടടുത്ത് പ്രായമുള്ള സമയത്ത് തട്ടിയെടുത്ത് മക്കയ്ക്കടുത്തുള്ള ഒരടിമച്ചന്തയില്‍ കൊണ്ടുവന്ന് വിറ്റതാണ് സയ്ദിനെ. പിന്നീടുള്ള സയ്ദിന്റെ ജീവിതം മക്കയിലാണ്. പില്‍ക്കാലത്ത് പ്രവാചകപത്‌നിയായിത്തീര്‍ന്ന ഖദീജ ബിന്‍ത് ഖുവയ്‌ലിദിനുവേണ്ടി അവരുടെ സഹോദരീപൂത്രന്‍ ഹക്കീം ആണ് സയ്ദിനെ അടിമച്ചന്തയില്‍ നിന്നുവാങ്ങിയത്. സ്വാഭാവികമായും ഖദീജയുടെ ഭൃത്യനായി അവരുടെ വീട്ടിലാണ് സയ്ദിന്റെ അവിടുന്നങ്ങോട്ടുള്ള ജീവിതം.

പ്രവാചകന്‍ അദ്ദേഹത്തിന്റെ ഇരുപത്തിയഞ്ചാം വയസ്സില്‍ ഖദീജയെ വിവാഹം കഴിച്ച് അവര്‍ ഒരുമിച്ച് താമസമാക്കിയപ്പോള്‍ സയ്ദിനെ ഖദീജ പ്രവാചകന് സമ്മാനമായി നല്‍കി. പ്രവാചകന്റെ അധീനതയിലുള്ള അടിമയായാണ് സയ്ദ് പിന്നെ മക്കയില്‍ ജീവിക്കുന്നത്. ഇതാണ് സയ്ദ്ബ്‌നു ഹാരിഥ; ഇതാണ് അദ്ദേഹത്തിന് പ്രവാചകനുമായുള്ള ബന്ധം. സയ്ദിന്റെ വിപുലമായ ഗോത്രബന്ധങ്ങളില്‍ എവിടെയൊക്കെയോ ചിലര്‍ ഏതൊക്കെയോ കാലത്ത് ക്രിസ്തുമതം സ്വീകരിച്ചവരുണ്ടായിരുന്നുവെന്നും അവരില്‍നിന്ന് സയ്ദ് ധാരാളം ക്രിസ്തുമതജ്ഞാനങ്ങള്‍ എട്ടുവയസ്സിനകം സമ്പാദിച്ചുവെന്നും വര്‍ഷങ്ങള്‍ക്കുശേഷം അദ്ദേഹം അവ മുഹമ്മദിന് പകര്‍ന്നുകൊടുത്തുവെന്നുമാണ് ചില ഓറിയന്റലിസ്റ്റുകള്‍ സമര്‍ത്ഥിച്ചിട്ടുള്ളത്. ഓറിയന്റലിസ്റ്റുകള്‍ ക്രമത്തില്‍ പറയുന്ന ഈ മൂന്ന് വാദങ്ങളില്‍ ആദ്യത്തേതിനുമാത്രമാണ് അവര്‍ വസ്തുതാപരമായ തെളിവുകള്‍ ഹാജരാക്കാന്‍ ശ്രമിക്കുന്നത്. മറ്റേത് രണ്ടും യാതൊരു ചരിത്രാടിത്തറയുമില്ലാത്ത വന്യഭാവനകളാണ്. സങ്കല്‍പത്തിനും ഭാവനയ്ക്കുമാണെങ്കില്‍ പോലും ചില അടിസ്ഥാന ധാര്‍മികതകള്‍ ആവശ്യമില്ലേ? തീരെ ചെറിയ കുട്ടിയായിരിക്കുമ്പോള്‍ ബന്ദിയാക്കി അടിമച്ചന്തയില്‍ വില്‍ക്കപ്പെട്ട ഒരാള്‍ ആയുളുടെ അടിമജീവിതത്തിനുമുമ്പുള്ള അതിഹൃസ്വമായ ബാല്യത്തില്‍ അതിവിദൂരമായ ഗോത്രബന്ധങ്ങളിലെവിടെയോ ഉണ്ടായിരുന്ന ക്രിസ്തുമതത്തെക്കുറിച്ച് ‘നേടിയെടുക്കുക’യും പ്രായപൂര്‍ത്തിയാവുകയും ജീവിതസാഹചര്യങ്ങള്‍ അടിമുടി മാറുകയും ചെയ്തിട്ടും ‘അക്ഷരംപ്രതി ഓര്‍ത്തുവെക്കുക’യും ചെയ്ത വിവരങ്ങളാണ് ക്വുര്‍ആനിനെപ്പോലൊരു ഗ്രന്ഥത്തിന്റെ ഉള്ളടക്കം നിര്‍ണയിച്ചതെന്ന് സങ്കല്‍പിക്കുന്നവര്‍ സ്വയം പരിഹാസ്യരാവുകയല്ലാതെ മറ്റെന്താണ് ചെയ്യുന്നത്? സയ്ദ്ബ്‌നു ഹാരിഥ(റ)യെ ക്വുര്‍ആനിലെ ക്രിസ്തുമത പ്രതിപാദനങ്ങളുടെ സ്രോതസ്സായി പ്രതിഷ്ഠിക്കുവാനുള്ള വിമര്‍ശകപരിശ്രമം മൂര്‍ത്തമായ തെളിവുകളുടെ അഭാവം അവരെ എത്രത്തോളം ഹതാശരാക്കുന്നുവെന്ന് മാത്രമാണ് വ്യക്തമാക്കുന്നതെന്ന് ചുരുക്കം.

സയ്ദ്ബ്‌നു ഹാരിഥ(റ)യെ സംബന്ധിച്ച് ഉപലബധമായ എല്ലാ വിവരങ്ങളും പ്രവാചകന്‍ അദ്ദേഹത്തിന്റെ ശിഷ്യത്വം സ്വീകരിച്ചതായല്ല, മറിച്ച് പ്രവാചകവ്യക്തിത്വത്തില്‍ അഗാധമായി ആകര്‍ഷിക്കപ്പെട്ട് പ്രവാചകന്റെ ഉത്തമശഷ്യനായി സയ്ദ് മാറിയതിനെക്കുറിച്ചാണ് അസന്നിഗ്ധമായി സൂചിപ്പിക്കുന്നത്. പ്രവാചകനില്‍നിന്ന് പഠിക്കുവാന്‍ ധൃതിപ്പെട്ട സയ്ദ് ആണ്, പ്രവാചകനെ പഠിപ്പിച്ച സയ്ദ് അല്ല ചരിത്രത്തിലുള്ളത്. പ്രവാചകനെ കണ്ടുമുട്ടിയതുമുതല്‍ക്കുതന്നെ നബി വ്യക്തിത്വത്തിന്റെ വിശുദ്ധിയെയും സമഗ്രതയെയും കുറിച്ച് ആഴത്തില്‍ ബോധ്യം വരികയും ആ ബോധ്യം ജീവിതാന്ത്യം വരെ കെടാതെ കാക്കുകയും പ്രവാചകനിലുള്ള അടിയുറച്ച വിശ്വാസം കാരണമായി അദ്ദേഹത്തിനുവേണ്ടി എന്തും ചെയ്യാന്‍ സദാ സന്നദ്ധനായി നില്‍ക്കുകയും ചെയ്ത നിഷ്‌കളങ്കനായ പ്രവാചകാനുചരനാണ് ചരിത്രത്തിലെ സയ്ദ്ബ്‌നു ഹാരിഥ. പ്രവാചകന്റെ വെളിപാടുകളുടെ അവലംബം തന്നില്‍ നിന്ന് ലഭിച്ച വിവരങ്ങളാണെന്ന് ബോധ്യമുള്ള ഒരാളുടെ യാതൊരു ലക്ഷണവും ചരിത്രത്തില്‍ വിശദമായി രേഖപ്പെടുത്തിയിട്ടുള്ള സയ്ദിന്റെ ജീവിതത്തില്‍ നാം കാണുന്നില്ല.

പരസ്പരം നിഷ്‌കളങ്കത ബോധ്യപ്പെട്ടുള്ള പ്രാവചകന്റെയും സയ്ദിന്റെയും ഹൃദയഹാരിയായ ആത്മബന്ധം മക്കയില്‍ മുഴുവന്‍ പ്രശസ്തമായിരുന്നു. ‘ഹിബ്ബുറസൂലില്ലാഹ്’ (അല്ലാഹുവിന്റെ ദൂതന്റെ പ്രിയപ്പെട്ടവന്‍) എന്നാണ് സയ്ദ് (റ) ചരിത്രത്തില്‍ അറിയപ്പെടുന്നതുതന്നെ. ഭൃത്യനായി പ്രവാചകന്റെ കൂടെ ജീവിതമാരംഭിച്ച കാലം മുതല്‍ക്കുതന്നെ പ്രവാചകന്റെ വ്യക്തിവിശുദ്ധിയെക്കുറിച്ച് ബോധ്യം വന്നതുകൊണ്ടാണ് സയ്ദിന് മനസ്സിന്റെ ആഴങ്ങളില്‍നിന്ന് അദ്ദേഹത്തോടുള്ള സ്‌നേഹവും ആദരവും നിര്‍ഗളിച്ചത്; അതുതന്നെയാണ് അവര്‍ തമ്മിലുള്ള ഗാഢസൗഹൃദത്തിന്റെ അടിത്തറ കെട്ടിയതും. പ്രവാചകത്വപൂര്‍വകാലത്ത്, ചെറുപ്പത്തില്‍ തങ്ങളില്‍ നിന്ന് വേര്‍പെട്ടുപോയ സയ്ദ് മക്കയില്‍ മുഹമ്മദിന്റെ കൂടെയുണ്ടെന്ന് മനസ്സിലാക്കി അദ്ദേഹത്തെ തിരിച്ചുകൊണ്ടുപോകാന്‍ വന്ന പിതാവടക്കമുള്ള വീട്ടുകാരെ, നബി(സ)യെ പിരിയാന്‍ തനിക്കാവില്ലെന്നും തന്നെക്കുറിച്ച് യാതൊരാകുലതയുമാവശ്യമില്ലെന്നും അത്രയും കറകളഞ്ഞ ഒരു രക്ഷിതാവിനെയാണ് തനിക്ക് ലഭിച്ചിരിക്കുന്നതെന്നും വ്യക്തമാക്കി തിരിച്ചയച്ച സയ്ദിന്റെ (റ) ചരിത്രപ്രസിദ്ധമായ നടപടി, നബി വ്യക്തിത്വത്തെക്കുറിച്ച് വര്‍ഷങ്ങളുടെ സഹവാസം വഴി അദ്ദേഹത്തില്‍ അടിയുറച്ചുവന്ന ബോധ്യമാണ് കാണിക്കുന്നത്. തന്നെക്കാള്‍ പത്തുവയസ്സോളം പ്രായം കുറവുള്ള സയ്ദിനെ ഒരനിയനെയോ മകനെയോ പോലെ സ്‌നേഹിക്കുകയും സംരക്ഷിക്കുകയും ചെയ്ത പ്രവാചകത്വപൂര്‍വ നബിജീവിതത്തില്‍ കൃത്രിമത്വത്തിന്റെയോ തട്ടിപ്പിന്റെയോ ചെറിയ ലാഞ്ചനപോലും സയ്ദിനനുഭവപ്പെട്ടില്ല എന്നതിന്റെ രേഖയാണീ സംഭവം. പ്രവാചകന് വെളിപാട് ലഭിച്ചുതുടങ്ങുകയും അദ്ദേഹം തന്റെ പ്രവാചകത്വം പ്രഖ്യാപിക്കുകയും ചെയ്തപ്പോള്‍ ഉടനടി അദ്ദേഹത്തില്‍ വിശ്വസിച്ച ഒരാള്‍ സയ്ദ് ആയിരുന്നുവെന്ന വസ്തുതയെ ഇതിനോട് ചേര്‍ത്തുവായിക്കുക. ഓറിയന്റലിസ്റ്റുകള്‍ സങ്കല്‍പിക്കുന്ന ഛായയില്‍ വരക്കുക അസാധ്യമാണ് സയ്ദിന്റെ (റ) പ്രവാചകനുമായുള്ള ബന്ധമെന്ന കാര്യം ഇതില്‍ നിന്നെല്ലാം സുതരാം വ്യക്തമാണ്. പിന്നീട് മരിക്കുവോളം തിരുനബി(സ)യുടെ വിശ്വസ്തനായ സന്തതസഹചാരിയാണ് സയ്ദ് (റ).

മക്കയില്‍ പീഡനങ്ങള്‍ കനത്തപ്പോള്‍ ത്വാഇഫിലേക്ക് അഭയം തേടിപ്പോകുമ്പോഴും അവിടെ നിന്ന് കല്ലേറുകൊണ്ട് മടങ്ങിവരുമ്പോഴും പ്രവാചകന്റെ നിഴലായി കൂടെയുണ്ട് സയ്ദ്ബ്‌നു ഹാരിഥ. സയ്ദിനോടുള്ള ഇഷ്ടം കാരണം പ്രവാചകത്വപൂര്‍വകാലത്ത് നബി (സ) അദ്ദേഹത്തെ തന്റെ ദത്തുപുത്രനായി പ്രഖ്യാപിച്ച കാര്യം പ്രസിദ്ധമാണ്. പില്‍ക്കാലത്ത് മദീന കാലഘട്ടത്തില്‍ ക്വുര്‍ആനില്‍ ദത്തുപുത്ര സമ്പ്രദായം നിരോധിക്കുകയും സയ്ദ് (റ) വിവാഹമോചനം ചെയ്ത് സയ്‌നബ്(റ)നെ പ്രവാചകന്‍  വിവാഹം  ചെയ്തുകൊണ്ട് നിരോധനത്തിന്റെ  സാമൂഹ്യമായ അംഗീകാരം നേടിയെടുക്കണമെന്ന് കല്‍പ്പിക്കുകയും ചെയ്യുന്ന വചനങ്ങള്‍ സയ്ദിന്റെ പേരടക്കമുദ്ധരിച്ചുകൊണ്ട് അവതരിപ്പിക്കപ്പെടുകയും  അത് അറേബ്യയിലൊന്നടങ്കം പ്രവാചകന്റെ ശത്രുക്കള്‍ വന്‍വിവാദമാക്കുകയും ചെയ്തപ്പോഴും സയ്ദ് (റ) നബി(സ)യുടെ കൂടെ അടിയുറച്ചു നിന്നുവെന്ന വസ്തുത, ഓറിയന്റലിസ്റ്റ് ഭാവനകളെ മുഴുവന്‍ കശക്കിയെറിയുന്നെണ്ടെന്നതാണ് യാഥാര്‍ത്ഥ്യം. മദീനയില്‍ ഇസ്‌ലാമിക രാഷ്ട്രത്തിന്റെ സംരക്ഷണാര്‍ത്ഥം നബി (സ) നിയോഗിച്ചയച്ച അനേകം  സൈനിക ദൗത്യ സംഘങ്ങളുടെ നായകസ്ഥാനത്തുണ്ടായിരുന്നയാള്‍  കൂടിയാണ് സയ്ദ്(റ). പ്രതിബദ്ധതയില്‍ നബി(സ)ക്ക്  യാതൊരു  സംശയവുമില്ലാതിരുന്ന, പ്രവാചകനിലും വെളിപാടുകളിലുമുള്ള അടിയുറച്ച വിശ്വാസം കാരണം ഇസ്‌ലാമിന് വേണ്ടി ജീവത്യാഗം ചെയ്യാന്‍ പോലും സദാസന്നദ്ധനായിരുന്ന കളങ്കരഹിതനായ ഭക്തമുസ്‌ലിം ആയിരുന്നു സയ്ദ് (റ) എന്നര്‍ത്ഥം.

ഹിജ്‌റ എട്ടാം വര്‍ഷം പ്രസിദ്ധമായ മുഅ്താ യുദ്ധത്തില്‍ രണ്ടു ലക്ഷത്തോളം വരുന്ന ക്രൈസ്തവ ബയ്‌സന്റയ്ന്‍ പടയാളികളോട് മൂവായിരത്തോളം മാത്രം വരുന്ന മുസ്‌ലിം സേനയെവെച്ച് സേനാനായകസ്ഥാനത്തിരുന്ന് പൊരുതവെ രക്തസാക്ഷിത്വം വരിച്ചുകൊണ്ടാണ് സയ്ദിന്റെ മരണമെന്ന സത്യമെങ്കിലും പരിഗണിച്ചിരുന്നുവെങ്കില്‍ സയ്ദിനെക്കുറിച്ച്  ഇത്തരമൊരു കഥ വിമര്‍ശകര്‍ക്ക് മെനയാന്‍ കഴിയുമായിരുന്നില്ലെന്നതല്ലേ യാഥാര്‍ത്ഥ്യം?

ഇനി വറക്വത്ബ്‌നു നൗഫലിന്റെ കാര്യത്തിലേക്കു വരാം. പ്രവാചക പത്‌നി ഖദീജ(റ)യുടെ പിതൃപുത്രനായിരുന്നു ഖുറയ്ശ് ഗോത്രക്കാരനും മക്കകാരനും തന്നെയായിരുന്നു വറക്വത്ബ്‌നു നൗഫല്‍. വറക്വ, ജാഹിലിയ്യാ അറബികളുടെ വിഗ്രഹാരാധനയോട് വിരക്തി പ്രകടിപ്പിച്ച് ഏകദൈവാരാധനയുള്‍ക്കൊള്ളുകയും അതിന് ഇബ്‌റാഹീമി പൈതൃകത്തിന്റെ പിന്തുണ അവകാശപ്പെടുകയും  ചെയ്തിരുന്ന മക്കന്‍ ഹനീഫുകളില്‍ ഒരാളായിരുന്നുവെന്ന് പറയുന്ന നിവേദനങ്ങളുണ്ട്. മറ്റു ഹനീഫുകളില്‍നിന്ന് വ്യത്യസ്തമായി ജൂത-ക്രൈസ്തവ മതഗ്രന്ഥങ്ങള്‍ പഠിക്കുകയും അവയില്‍ നിന്ന് ഏകദൈവാരാധനയുടെ വഴി നിര്‍ധരിച്ചെടുക്കാന്‍ ശ്രമിക്കുകയും ചെയ്ത വ്യക്തിയായിരുന്നു വറക്വ എന്നു സൂചിപ്പിക്കുന്ന നിവേദനങ്ങളുമുണ്ട്. അദ്ദേഹം വേദക്കാരുടെ  മതം പിന്തുടര്‍ന്നിരുന്നുവെന്ന് തന്നെ ചില ചരിത്ര പണ്ഡിതന്‍മാര്‍ക്കഭിപ്രായമുണ്ട്. വറക്വക്ക് ഹിബ്രു ഭാഷയില്‍ പ്രാവീണ്യമുണ്ടായിരുന്നുവെന്നും വേദക്കാരുടെ മതഗ്രന്ഥങ്ങള്‍ അദ്ദേഹം ഹിബ്രുവില്‍ പകര്‍ത്തിയെഴുതിരുന്നുവെന്നും  ആയിശ(റ)യില്‍ നിന്നുദ്ധരിക്കപ്പെടുന്ന സ്വഹീഹുല്‍ ബുഖാരിയിലെ ഫദീഥിലുണ്ട്. (കിതാബുല്‍ വഹ്‌യ്).

ഇദ്ദേഹത്തില്‍ നിന്ന് നബി(സ)ക്ക് എന്ത് വിവരങ്ങള്‍ കിട്ടിയെന്നാണ് വിമര്‍ശകര്‍ക്ക് സ്ഥാപിക്കാനാവുക? ബൈബിള്‍ പഴയ-പുതിയ നിയമഗ്രന്ഥങ്ങളിലെ വിവരങ്ങള്‍ കിട്ടിയെന്നാണ് പരമാവധി പോയാല്‍ തെളിയിക്കാന്‍ കഴിയുക. എന്നാല്‍ ബൈബിള്‍ പരിജ്ഞാനം കുര്‍ആനിന്റെ രചനയ്ക്ക്  ഒരു നിലയ്ക്കും  വിശദീകരണമാവുകയില്ല എന്ന് സിറിയന്‍ സമ്പര്‍ക്ക ചര്‍ച്ചയെ വിശകലനം ചെയ്തപ്പോള്‍ നാം വ്യക്തമാക്കിയതാണ്.  ഇനി, ഒരു വൈജ്ഞാനികാന്വേഷണം എന്ന നിലയില്‍  നബി (സ) വറക്വയില്‍ നിന്ന് ബൈബിള്‍ വിജ്ഞാനീയങ്ങള്‍ ആര്‍ജ്ജിച്ചതിന് ചരിത്രത്തില്‍ രേഖയുണ്ടോ എന്ന് നാം പരിശോധിക്കുക. ഇല്ല! ബൈബിളികമോ അല്ലാത്തതോ ആയ ഒരു ജൂത/ക്രിസ്തുമത പാഠവും  വറക്വ നബി(സ)ക്ക്  പറഞ്ഞുകൊടുത്തതായി ഒരു തെളിവും ചരിത്രത്തിലില്ല. കേവലം ഭാവനകള്‍ മാത്രമാണ് നബിവിമര്‍ശകരുടെ ‘വറക്വ തിസീസുകളില്‍’ മുഴുവനുമുള്ളത്. നബി(സ)യും വറക്വത്ബ്‌നു നൗഫലും തമ്മില്‍  നടന്നതായി തെളിവുള്ള സമ്പര്‍ക്കം പ്രവാചകത്വലബ്ധിക്കുശേഷമുള്ളതാണ്.

വാര്‍ധക്യവും അന്ധതയുംകൊണ്ട്  വറക്വ വിശ്രമജീവിതം നയിക്കുന്ന സമയത്താണ് പ്രവാചകന്റെ മുന്നില്‍  ഹിറാഗുഹയില്‍ വെച്ച് ജിബ്‌രില്‍ എന്ന മലക്ക് പ്രത്യക്ഷപ്പെടുന്നതും ക്വുര്‍ആന്‍ അവതരണത്തിന്  നാന്ദി കുറിക്കുന്നതും എന്ന് വറക്വയെ പരാമര്‍ശിക്കുന്ന സ്വഹീഹുല്‍ ബുഖാരിയിലെ പരാമൃഷ്ട നിവേദനത്തിലുണ്ട്. ഖജീദ (റ) നബി(സ)യെയും കൂട്ടി വറക്വയുടെയടുക്കല്‍ പോകുന്നതും പ്രവാചകന്‍ തനിക്കുണ്ടായ അനുഭവങ്ങള്‍ വറക്വയെ കേള്‍പ്പിക്കുന്നതുമാണ് കഥാ സന്ദര്‍ഭം. വറക്വയുടെ പ്രതികരണം ഹഥീസ് രേഖപ്പെടുത്തുന്നത് ഇപ്രകാരമാണ്: ”ഇത് മൂസായുടെ (മോശയുടെ) അടുത്തേയ്ക്ക് അല്ലാഹു പറഞ്ഞയച്ച ‘നാമൂസ്’ (ജിബ്‌രീല്‍ മലക്ക്) ആണ്. ഞാന്‍ ഇപ്പോള്‍ എന്റെ യൗവ്വനത്തിലായിരുന്നെങ്കില്‍ എത്ര  നന്നായിരുന്നേനെ! താങ്കളുടെ ജനത താങ്കളെ നാട്ടില്‍നിന്ന് പുറത്താക്കുമ്പോള്‍ ഞാന്‍ ജീവനോടെ ഉണ്ടായിരുന്നെങ്കില്‍!” തന്റെ ജനത തന്നെ പുറത്താക്കുകയോ എന്ന് നബി (സ) വറക്വയോട് ചോദിച്ചു. അപ്പോള്‍  അദ്ദേഹം പറഞ്ഞു: ”അതെ, താങ്കള്‍ക്ക് വന്നു കിട്ടിയതുപോലുള്ള സന്ദേശങ്ങള്‍ പ്രബോധനം ചെയ്ത ആരും ശത്രുത സമ്പാദിക്കാതിരുന്നിട്ടില്ല. താങ്കളുടെ  ദിവസത്തിന് സാക്ഷിയാകാന്‍  എനിക്കായുസ്സുണ്ടെങ്കില്‍  എന്റെ കഴിവിന്റെ പരമാവധി ഞാന്‍ താങ്കളെ സഹായിക്കും.” എന്നാല്‍ അധികം കഴിയുന്നതിനുമുമ്പേ വറക്വ മരണപ്പെടുകയാണ് ചെയ്തതെന്ന് ഹഥീസ് രേഖപ്പെടുത്തുന്നു. (സ്വഹീഹുല്‍ ബുഖാരി/കിതാബുല്‍ വഹ്‌യ്)

വറക്വയും നബി(സ)യും തമ്മില്‍ നടന്നതായി ചരിത്രത്തില്‍ ആധികാരികമായ രേഖയുള്ള സംഭാഷണം ഇതാണ്. ഇതില്‍ നിന്ന് വ്യക്തമാകുന്നതെന്താണ് ?

1.    നബി(സ)ക്കുണ്ടായ ദിവ്യവെളിപാടിനെ വറക്വ ഒരിക്കലും സംശയിച്ചില്ല.

2.    വെളിപാടിനെ സംബന്ധിച്ച് ബൈബിള്‍  പഠനത്തില്‍ നിന്ന് ലഭിച്ച ധാരണകള്‍, മുഹമ്മദിനുണ്ടായത് ബൈബിളില്‍ പരാമര്‍ശിക്കപ്പെട്ട പൂര്‍വ്വ പ്രവാചകന്മാര്‍ക്കുണ്ടായ ദൈവികാനുഭവം തന്നെയാണെന്ന് മനസ്സിലാക്കാന്‍  വറക്വയെ സഹായിച്ചു.

3.    വരാനിരിക്കുന്ന പ്രവാചകനെക്കുറിച്ച ബൈബിള്‍ പ്രവചനങ്ങള്‍, വിശേഷിച്ചും മോശയെപ്പോലുള്ള പ്രവാചകനെ സംബന്ധിച്ച സൂചനകള്‍, നബി(സ)യില്‍ പൂര്‍ത്തീകരിക്കപ്പടുന്നതായി  വറക്വ മനസ്സിലാക്കി. ക്വുര്‍ആനിന്റെ രചനയ്ക്ക് വറക്വ നബി(സ)യെ സഹായിച്ചു എന്ന വാദം പൊള്ളയാണെന്ന് ഒന്നാമത്തെ വസ്തുത സുതരാം വ്യക്തമാക്കുന്നു. രണ്ടും മൂന്നും വസ്തുതകളാകട്ടെ, നബി(സ)യുടെ സമകാലികനായിരുന്ന ഒരു ബൈബിള്‍  പണ്ഡിതന്‍  നബി(സ)യുടെ വ്യക്തിത്വവും അദ്ദേഹത്തിനുണ്ടായ വെളിപാടനുഭവവും ശ്രദ്ധിച്ച് അദ്ദേഹം  പ്രവാചകനാണെന്ന് സാക്ഷീകരിച്ചുവെന്ന് തെളിയിക്കുന്നു. ബൈബിളിന്റെ അനുയായികള്‍ എന്നവകാശപ്പെടുന്നവര്‍ വറക്വയില്‍ നിന്ന് വ്യത്യസ്തമായി ഇന്ന് പ്രവാചകനെ നിന്ദിക്കുന്നത് അന്ധമായ  കുടിപ്പക കൊണ്ടാണെന്നും ബൈബിള്‍ വചനങ്ങളൊന്നും തന്നെ മുഹമ്മദ്  നബി(സ)യുടെ പ്രവാചകത്വത്തെ സത്യപ്പെടുത്തുകയല്ലാതെ  നിരാകരിക്കുന്നില്ലെന്നുമാണ് വറക്വയുടെ ചരിത്രം ആത്യന്തികമായി തെളിയിക്കുന്നതെന്ന് ചുരുക്കം. നബിവിമര്‍ശകര്‍ ഉദ്ദേശിക്കുന്നതിന് നേര്‍വിപരീതമായ കാര്യങ്ങള്‍ മാത്രമാണ് ആ ചരിത്രത്തില്‍ നിന്ന് നിര്‍ധരിച്ചെടുക്കാനാവുക. അവരാഗ്രിക്കുന്ന യാതൊന്നും അതിലില്ല തന്നെ!

സയ്ദ്ബ്‌നു ഹാരിഥ(റ)യും വറക്വത്ബ്‌നു നൗഫലും മക്കകാര്‍ക്ക് സുപരിചിതരായിരുന്ന വ്യക്തിത്വങ്ങളാണ്. നബി(സ)യുടെ പ്രവാചകത്വാരംഭത്തില്‍ തന്നെ അതിനെ സാക്ഷീകരിക്കുകയും പിന്തുണ പ്രഖ്യപിക്കുകയും ചെയ്ത വറക്വയാണ് നബി (സ) ഓതിക്കേള്‍പ്പിക്കുന്ന ഖുര്‍ആന്‍ വചനങ്ങളുടെ ആശയസ്രോതസ്സെന്ന് വാദിക്കാന്‍ മക്കയില്‍ ക്വുര്‍ആനെതിരെ ആരോപണങ്ങളുന്നയിക്കുവാന്‍  സകല സാധ്യതകളും തിരഞ്ഞുനടന്ന പ്രഗല്‍ഭരായ പ്രവാചകശത്രുക്കളൊന്നും ഒരിക്കലും സന്നദ്ധമാകാതിരുന്നതെന്തുകൊണ്ടാണ്? നബി(സ)യുടെ യൗവ്വനം  മുതല്‍ അദ്ദേഹത്തിന്റെ  സന്തത സഹചാരിയായിരുന്ന, മദീനയില്‍ ജൂതന്‍മാര്‍ക്കിടയില്‍ വരെ പ്രവാചകന്റെ കൂടെ എട്ടുവര്‍ഷം ജീവിച്ച, പ്രവാചകവിയോഗത്തിന് കേവലം  രണ്ടു വര്‍ഷം മുമ്പുമാത്രം ഈ ലോകത്തോട് വിട പറഞ്ഞ സയ്ദ്ബ്ന്‍ ഹാരിഥ(റ)യെ ചൂണ്ടി മക്കയിലെയോ മദീനിയിലെയോ ബഹുദൈവാരാധകരോ മദീനയിലെ ജൂതന്‍മാരോ ഒന്നും ഒരിക്കലും അദ്ദേഹമാണ് നബി(സ)യുടെ നാവില്‍  നിന്ന് കേള്‍ക്കുന്ന ചില ക്വുര്‍ആന്‍  വചനങ്ങളുടെ  വൈജ്ഞാനിക സ്രോതസ്സെന്ന് ആരോപിക്കാതിരുന്നതെന്തുകൊണ്ടാണ്? ഓറിയന്റലിസ്റ്റുകളും മിഷനറിമാരും കണ്ടെടുക്കുന്ന പുതിയ ‘സാധ്യതകള്‍’ ചരിത്രത്തിന്റെ ലിറ്റ്മസ് ടെസ്റ്റിനെ അതിജീവിക്കുവാനുള്ള ദുര്‍ബലമായ ശേഷിപോലും ഇല്ലാത്തവയായിത്തീരുന്നതില്‍ സഹതപിക്കുക നാം!

അബ്രഹാം പ്രവാചകന്‍ ഭാര്യ ഹാജറിനെയും പുത്രന്‍ ഇസ്മാഈലിനെയും മക്കയില്‍ കൊണ്ടുചെന്നാക്കിയെന്നും ഇസ്മാഈലിനെയും കൂട്ടി അവിടെ കഅ്ബ സ്ഥാപിച്ചുവെന്നും അതിനു ചുറ്റുമാണ് മക്കന്‍ നാഗരികത വളര്‍ന്നുവന്നതെന്നുമുള്ള മുഹമ്മദ് നബി(സ)യുടെ വിശദീകരണങ്ങള്‍ അടിസ്ഥാനരഹിതമാണെന്ന് ബൈബിള്‍ തെളിയിക്കുന്നുവെന്ന മിഷനറിമാരുടെ വാദത്തില്‍ എന്തെങ്കിലും കഴമ്പുണ്ടോ? ഹാജറും ഇസ്മാഈലും ഇബ്രാഹീമിനാല്‍ ഉപേക്ഷിക്കപ്പെട്ടത് മക്കയിലല്ലെന്ന് ബൈബിള്‍ വചനങ്ങളില്‍ നിന്ന് വ്യക്തമാകുന്നില്ലേ?

ല്ല. അബ്രഹാം ഹാജറിനെയും ഇസ്മാഈലിനെയും കൊണ്ടുപോയി പാര്‍പ്പിച്ചത് മക്കയിലല്ലെന്ന് ബൈബിള്‍ വചനങ്ങളില്‍ നിന്ന് മനസ്സിലാകുന്നുണ്ടെന്നും അതിനാല്‍ ഇബ്‌റാഹീമും ഇസ്മാഈലും ചേര്‍ന്നാണ് കഅ്ബ നിര്‍മിച്ചതെന്ന പ്രവാചകന്‍(സ)യുടെ അധ്യാപനം അടിസ്ഥാനരഹിതമാണെന്നും സമര്‍ത്ഥിക്കുവാനാണ് മിഷനറിമാര്‍ ശ്രമിച്ചുവരാറുള്ളത്. കഅ്ബയുടെ അബ്രഹാമിക പശ്ചാത്തലം മുഹമ്മദ് നബി(സ)ക്ക് നൂറ്റാണ്ടുകള്‍ക്കു മുമ്പു തന്നെ അറബികള്‍ക്കും അവരെ പരിചയമുണ്ടായിരുന്നവര്‍ക്കും ബോധ്യമുണ്ടായിരുന്നതാണെന്നും കഅ്ബ ഏകദൈവാരാധനക്കുവേണ്ടി അബ്രഹാം സ്ഥാപിച്ചതാണെന്ന വസ്തുതയെ നിരാകരിക്കുന്ന യാതൊരു ചരിത്രരേഖയുമില്ലെന്നും നാം നേരത്തെ കണ്ടുകഴിഞ്ഞു. മുഹമ്മദ് നബി (സ) പുതുതായി അവതരിപ്പിച്ച ഒരു വാദത്തെയല്ല, മറിച്ച് അറബികള്‍ക്കും ചരിത്രത്തിനും തര്‍ക്കമില്ലാത്ത ഒരു യാഥാര്‍ത്ഥ്യത്തെയാണ് മിഷനറിമാര്‍ ബൈബിളുപയോഗിച്ച് അട്ടിമറിക്കാന്‍ ശ്രമിക്കുന്നത് എന്നാണിതിനര്‍ത്ഥം. കഅ്ബ ഇബ്‌റാഹീമും ഇസ്മാഈലും ചേര്‍ന്ന് സ്ഥാപിച്ചതല്ലെന്ന് പറയാന്‍ മിഷനറിമാരുടെ കയ്യില്‍ ബൈബിളല്ലാത്ത യാതൊരു പ്രണാണവുമില്ല. അതുകൊണ്ടുതന്നെ, ബൈബിള്‍ പ്രമാദങ്ങളില്‍ നിന്ന് പൂര്‍ണമായും മുക്തമായ ചരിത്രസ്രോതസ്സാണെങ്കില്‍ മാത്രമേ ഈ വാദത്തിന് എന്തെങ്കിലും പ്രസക്തിയുണ്ട് എന്നു പറയാനാകൂ.

ക്വുര്‍ആന്‍ പ്രപഞ്ചനാഥനായ അല്ലാഹു മുഹമ്മദ് നബി(സ)ക്ക് അവതരിപ്പിച്ചുകൊടുത്ത വേദഗ്രന്ഥമാണെന്നാണ് ക്വുര്‍ആനും മുഹമ്മദ് നബി(സ)യും പഠിപ്പിച്ചിട്ടുള്ളത്; മുസ്‌ലിം ലോകം നാളിതുവരെയായി അങ്ങനെയാണ് മനസ്സിലാക്കി വന്നിട്ടുള്ളതും. ബൈബിള്‍ ക്രൈസ്തവരുടെ വേദഗ്രന്ഥമാണെന്നാണ് പറയപ്പെടാറുള്ളതെങ്കിലും അത് പ്രപഞ്ചരക്ഷിതാവിന്റെ വചനങ്ങളുടെ സമാഹാരമാണെന്ന് ബൈബിളെഴുത്തുകാര്‍ക്കോ ക്രൈസ്തവലോകത്തിനുപോലുമോ അവകാശവാദമില്ല.

പ്രവാചകന്‍മാരുടെയും ഇസ്രയേല്‍ സമൂഹത്തിന്റെയും ചരിത്രം പല കാലങ്ങളിലായി മനുഷ്യര്‍ എഴുതിവെച്ചതിന്റെ സമാഹാരമാണ് ബൈബിള്‍ പഴയ നിയമം. ആ ചരിത്രം പറഞ്ഞുപോകുമ്പോള്‍ പ്രവാചകവചനങ്ങളായി അവരുടെ ഓര്‍മയിലുള്ള പലതും ബൈബിളില്‍ കടന്നുവരുന്നുണ്ടെന്ന് മാത്രമേയുള്ളൂ. അതാത് പ്രവാചകന്‍മാരുടെ മരണം കഴിഞ്ഞ് കാലങ്ങള്‍ പിന്നിട്ടതിനുശേഷമാണ് ബൈബിള്‍ പുസ്തകങ്ങള്‍ പലതും രചിക്കപ്പെട്ടത് എന്നതുകൊണ്ടുതന്നെ, ബൈബിളില്‍ നമുക്കിന്ന് വായിക്കാന്‍ കഴിയുന്ന പ്രവാചകചരിത്രത്തിലും അതിന്റെ ഭാഗമായി ഉദ്ധരിക്കപ്പെട്ടിരിക്കുന്ന പ്രവാചക വചനങ്ങളിലും സ്ഖലിതങ്ങളുണ്ടാകാനുള്ള സാധ്യത വളരെയധികമാണ്. ഇങ്ങനെ സ്ഖലിത സാധ്യതകളോടുകൂടി എഴുതപ്പെട്ടു എന്നതിനുപുറമെ, എഴുതപ്പെട്ടതിനുശേഷം പില്‍ക്കാലക്കാരുടെ തിരുത്തലുകള്‍ക്ക് നിരന്തരമായി വിധേയമായി എന്നത് ബൈബിള്‍ വിവരണങ്ങളുടെ ആധികാരികതയെ പിന്നെയും സംശയാസ്പദമാക്കുന്നു. സത്യസന്ധരായ ബൈബിള്‍ പണ്ഡിതന്‍മാര്‍ക്കിടയില്‍ അഭിപ്രായവ്യത്യാസങ്ങളില്ലാത്ത വസ്തുതകളാണിവയെല്ലാം. ബൈബിള്‍ പറയുന്നു എന്നതുകൊണ്ടുമാത്രം ഒരു കാര്യം ശരിയോ തെറ്റോ ആകണമെന്നില്ല എന്നു തന്നെയാണ് ഇതിന്റെയര്‍ത്ഥം.

ബൈബിള്‍ പറയുന്ന ചരിത്രത്തില്‍ അനേകം അബദ്ധങ്ങളുണ്ടെന്ന് ഇതിനകം ചരിത്രഗവേഷകര്‍ തെളിയിച്ചു കഴിഞ്ഞിട്ടുണ്ട്. ഇബ്‌റാഹീം ഹാജറിനെയും ഇസ്മാഈലിനെയും കൊണ്ടുപോയി പാര്‍പ്പിച്ചത് മക്കയിലല്ലെന്ന് ബൈബിള്‍ പറയുന്നതുകൊണ്ടു മാത്രം അത് മക്കയിലല്ല എന്നു വരികയില്ലെന്ന് ചുരുക്കം. ‘ഞങ്ങളുടെ ബൈബിള്‍ പറയുന്നു; അതിനാല്‍ ലോകം അത് അംഗീകരിക്കണം’ എന്നു പറയുന്ന ഭക്തിയുടെയും വിശ്വാസത്തിന്റെയും ഭാഷ മിഷനറിമാരുടെ മനസ്സമാധാനത്തിനു മാത്രമേ ഉപകരിക്കൂ. ബൈബിള്‍ കുറ്റമറ്റ ചരിത്രസ്രോതസ്സാണെന്നും അതില്‍ യാതൊരുവിധ അബദ്ധവുമില്ലെന്നും തെളിയിക്കുവാന്‍ മിഷനറിമാര്‍ സന്നദ്ധമാകാത്തിടത്തോളം കാലം സത്യാന്വേഷികള്‍ അതിന് യാതൊരു വിലയും കല്‍പിക്കുകയില്ല.

ഇബ്‌റാഹീം നബി(അ)യുടെയും അദ്ദേഹത്തിന്റെ കുടുംബത്തിന്റെയും മക്കാ ബന്ധം നിഷേധിക്കുവാന്‍ മിഷനറിമാര്‍ ആശ്രയിക്കുന്നത് പഴയ നിയമത്തിലെ പഞ്ചപുസ്തകങ്ങളെയാണ്; കുറേക്കൂടി കൃത്യമായി പറഞ്ഞാല്‍ അവയില്‍ ആദ്യത്തേതായ ഉല്‍പത്തി പുസ്തകത്തെ. ഉല്‍പത്തി, പുറപ്പാട്, സംഖ്യ, ലേവ്യ, ആവര്‍ത്തനം എന്നിവയടങ്ങുന്ന പഞ്ചഗ്രന്ഥി തോറയാണെന്ന് സാമാന്യമായി പലരും പറഞ്ഞുപോകാറുണ്ടെങ്കിലും മോശെ പ്രവാചകനവതരിപ്പിക്കപ്പെട്ട തോറ അതേപടി സംരക്ഷിക്കപ്പെട്ടതല്ല, മറിച്ച് അതിലേക്ക് പല കാലങ്ങളിലായി പലതും പുരോഹിതന്‍മാര്‍ സ്വധാരണകള്‍ക്കനുസരിച്ച് എഴുതിച്ചേര്‍ത്തുണ്ടാക്കിയതാണ് പഞ്ചപുസ്തകങ്ങളുടെ ഉള്ളടക്കമെന്ന് ബൈബിള്‍ പണ്ഡിതന്‍മാര്‍ തന്നെ വ്യക്തമാക്കിയിട്ടുണ്ട്. ആവര്‍ത്തന പുസ്തകത്തിലെ മുപ്പത്തിനാലാം അധ്യായത്തില്‍ മോശെയുടെ മരണത്തെക്കുറിച്ചുള്ള പ്രതിപാദനങ്ങള്‍ പോലുമുണ്ട്. മോശെക്ക് കര്‍ത്താവ് അവതരിപ്പിച്ചുകൊടുത്ത വചനങ്ങള്‍ അപ്പടി പരിരക്ഷിക്കപ്പെടുന്നതല്ല പഞ്ചഗ്രന്ഥങ്ങളെന്ന് ഇതില്‍ നിന്നുതന്നെ വ്യക്തമാണ്. മോശെ പ്രവാചകനുശേഷവും പഞ്ചപുസ്തകങ്ങളില്‍ പലതും എഴുതിച്ചേര്‍ക്കപ്പെട്ടുവെന്ന് ചുരുക്കം.

അബ്രഹാമിനുശേഷം നൂറ്റാണ്ടുകള്‍ കഴിഞ്ഞ് ജീവിച്ച മോശെയുടെ മരണശേഷവും പുരോഹിതന്‍മാര്‍ വചനങ്ങള്‍ എഴുതിച്ചേര്‍ത്തിട്ടുള്ള ഒരു പുസ്തകത്തിലെ പരാമര്‍ശങ്ങള്‍ മാത്രം വെച്ച് അബ്രഹാം ഹാഗറിനെ കൊണ്ടുചെന്നാക്കിയത് മക്കയിലല്ല എന്നു തീരുമാനിക്കാന്‍ കഴിയുക എങ്ങനെയാണ്? പരിശുദ്ധ ക്വുര്‍ആന്‍ പറഞ്ഞതെത്ര ശരിയാണ്! ”വേദക്കാരേ, ഇബ്‌റാഹീമിന്റെ കാര്യത്തില്‍ നിങ്ങളെന്തിനാണ് തര്‍ക്കിക്കുന്നത്? തൗറാത്തും ഇന്‍ജീലും അവതരിപ്പിക്കപ്പെട്ടത് അദ്ദേഹത്തിനു ശേഷം മാത്രമാണല്ലോ. നിങ്ങളെന്താണ് ചിന്തിക്കാത്തത്?  ഹേ; കൂട്ടരേ, നിങ്ങള്‍ക്ക് അറിവുള്ള കാര്യത്തെപ്പറ്റി നിങ്ങള്‍ തര്‍ക്കിച്ചു. ഇനി നിങ്ങള്‍ക്ക് അറിവില്ലാത്ത വിഷയത്തില്‍ നിങ്ങളെന്തിന്ന് തര്‍ക്കിക്കുന്നു? അല്ലാഹു അറിയുന്നു, നിങ്ങള്‍ അറിയുന്നില്ല. ഇബ്രാഹീം യഹൂദനോ ക്രിസ്ത്യനോ ആയിരുന്നില്ല. എന്നാല്‍ അദ്ദേഹം ഹനീഫും മുസ്‌ലിമും ആയിരുന്നു. അദ്ദേഹം ബഹുദൈവാരാധകരില്‍ പെട്ടവനായിരുന്നിട്ടുമില്ല. തീര്‍ച്ചയായും ജനങ്ങളില്‍ ഇബ്രാഹീമിനോട് കൂടുതല്‍ അടുപ്പമുള്ളവര്‍ അദ്ദേഹത്തെ പിന്തുടര്‍ന്നവരും, ഈ പ്രവാചകനും, (അദ്ദേഹത്തില്‍) വിശ്വസിച്ചവരുമാകുന്നു. അല്ലാഹു സത്യവിശ്വാസികളുടെ രക്ഷാധികാരിയാകുന്നു.” (ക്വുര്‍ആന്‍ 3 : 65-68)

ഹാജറിന്റെയും ഇസ്മാഈലിന്റെയും പാലായനമുണ്ടായത് മക്കയിലേക്കല്ലെന്ന് ബൈബിള്‍ പുസ്തകങ്ങളുടെ മാത്രം അടിസ്ഥാനത്തില്‍ വാദിക്കുന്നത് അര്‍ത്ഥശൂന്യമാണെന്ന് നമുക്ക് വ്യക്തമായി. ഇബ്‌റാഹീമിന്റെ ജീവിതത്തിലെ അറേബ്യന്‍ അധ്യായങ്ങള്‍ ബൈബിള്‍ നിരാകരിക്കുന്നുണ്ടോ എന്നാണ് ഇനി പരിശോധിക്കാനുള്ളത്. ഇശ്മയേലിന്റെ പുത്രനായി ബൈബിള്‍ പരിചയപ്പെടുത്തുന്ന കേദാറിന്റെ (ഉല്‍പത്തി 25 : 13, ദിനവൃത്താന്തം 1 : 29) സന്തതിപരമ്പരകള്‍ അറേബ്യയിലാണ് നിവസിക്കുന്നതെന്ന് ബൈബിളെഴുത്തുകാര്‍ തന്നെ സൂചിപ്പിക്കുന്നുണ്ടെന്നതാണ് വാസ്തവം. യെശയ്യാ പ്രവാചകന്റെ പുസ്തകത്തില്‍ ഇപ്പോള്‍ ‘അറേബ്യയെക്കുറിച്ചുള്ള സന്ദേശം’ എന്ന തലക്കെട്ടോടുകൂടി പ്രസിദ്ധീകരിക്കപ്പെട്ടുവരുന്ന വചനങ്ങള്‍ വായിക്കുക: ”അറേബ്യയെക്കുറിച്ചുളള അരുള്‍പാട്: ദദാന്യരായ സാര്‍ഥവാഹകരേ, നിങ്ങള്‍ അറേബ്യയിലെ കുറ്റിക്കാട്ടില്‍ വസിക്കും. തേമാന്യരേ, നിങ്ങള്‍ ദാഹിക്കുന്നവര്‍ക്ക് ജലം നല്‍കുവിന്‍, പാലായനം ചെയ്യുന്നവര്‍ക്ക് അപ്പം കൊടുക്കുവിന്‍. എന്തെന്നാല്‍, അവര്‍ ഊരിയ വാളില്‍ നിന്നും കുലച്ച വില്ലില്‍ നിന്നും യുദ്ധത്തിന്റെ നടുവില്‍ നിന്നും രക്ഷപെട്ട് ഓടുന്നവരാണ്. കര്‍ത്താവ് എന്നോട് അരുളിചെയ്തു: കൂലിക്കാരന്‍ കണക്കാക്കുന്നതുപോലെ, കണിശം ഒരു വര്‍ഷത്തിനുള്ളില്‍ കേദാറിന്റെ സര്‍വ മഹത്വവും നശിക്കും. കേദാറിന്റെ വില്ലാളിവീരന്‍മാരില്‍ ചുരുക്കം ചിലര്‍മാത്രം അവശേഷിക്കും. ഇസ്രായേലിന്റെ ദൈവമായ കര്‍ത്താവാണ് അരുളിചെയ്തിരിക്കുന്നത്.” (എശയ്യ 21 : 13-17).

ഫലസ്ത്വീനു പുറത്തുള്ള അറേബ്യന്‍ മരുപ്രദേശങ്ങളില്‍ വ്യാപിച്ചുകിടക്കുന്നവരായാണ് ഇസ്മാഈലിന്റെ സന്തതിപരമ്പരകളെ ബൈബിളെഴുത്തുകാര്‍ മനസ്സിലാക്കിയിരുന്നതെന്ന് ഉല്‍പത്തി പുസ്തകം പൂര്‍ണമായി പരിശോധിച്ചാല്‍ തന്നെ മനസ്സിലാകും. ബൈബിള്‍ പരാമര്‍ശിക്കുന്ന ഇസ്മാഈല്‍ സന്തതികളില്‍പ്പെട്ടവരാണ് മക്കയില്‍ നിവസിക്കുന്നതെന്ന് അവരെ പരിചയമുണ്ടായിരുന്ന ജൂതന്‍മാര്‍ക്കും ക്രൈസ്തവര്‍ക്കും ബോധ്യമുണ്ടായിരുന്നതുകൊണ്ടാണ് മദീനയിലെ ജൂതന്‍മാര്‍ മുതല്‍ പ്രാചീന റോമില്‍ ജീവിച്ച ജോസിഫസിനും സോസിമേമസിനും വരെ മക്കയുടെ അബ്രഹാമിക പൈതൃകത്തെ സംബന്ധിച്ച് സംശയങ്ങളുണ്ടാകാതിരുന്നത്. മിഷനറിമാര്‍ നൂറുശതമാനം ആധികാരികമെന്ന് വിശ്വസിക്കുന്ന ബൈബിള്‍ വിവരണങ്ങള്‍പോലും അറേബ്യയുടെ ഇസ്മാഈലി വേരുകളിലേക്ക് വിരല്‍ ചൂണ്ടുന്നുണ്ടെന്ന് സാരം.

‘ഇശ്മയേല്‍ പാറാനിലെ മരുഭൂമിയില്‍ പാര്‍ത്തു’ എന്ന ബൈബിള്‍ വചനമാണ് (ഉല്‍പത്തി 21 : 20) ഇസ്മാഈലും ഹാജറും പാര്‍പ്പുറപ്പിച്ചത് മക്കയിലല്ലെന്ന് ബൈബിള്‍ സൂചിപ്പിക്കുന്നുവെന്ന് പറയാനായി മിഷനറിമാര്‍ ഉപയോഗിക്കാറുള്ളത്. ‘പാറാന്‍’ എന്ന പ്രയോഗം ബൈബിളില്‍ പലയിടങ്ങളിലായി കാണാന്‍ കഴിയും. അവിടെയെല്ലാം ബൈബിള്‍ ഉദ്ദേശിച്ചത് ഇന്ന് നമുക്ക് പരിചയമുള്ള ഏത് പ്രദേശത്തെയാണ് എന്ന് കൃത്യമായി നിര്‍ണയിക്കുവാന്‍ യാതൊരു നിര്‍വാഹവുമില്ലെന്നതാണ് വാസ്തവം. ഇശ്മയേല്‍ പാര്‍ത്ത പാറാന്‍ ഏതാണെന്ന കാര്യത്തിലും ബൈബിള്‍ പണ്ഡിതന്‍മാര്‍ക്കിടയില്‍ അനേകം അഭിപ്രായവ്യത്യാസങ്ങളുണ്ട്. ‘പാറാന്‍’ എന്ന, ഉദ്ദേശ്യം എന്താണെന്ന് വ്യക്തമല്ലാത്ത, അനേകം അര്‍ത്ഥസാധ്യതകള്‍ അവശേഷിപ്പിക്കുന്ന അവ്യക്തമായ പ്രയോഗമാണ് ഇസ്മാഈല്‍ വളര്‍ന്ന പ്രദേശത്തെക്കുറിച്ച് ബൈബിള്‍ നടത്തുന്നതെന്നര്‍ത്ഥം. പാറാന്‍ കൊണ്ട് വിവക്ഷിക്കപ്പെട്ടത് ഇന്ന പ്രദേശമാണെന്ന് തറപ്പിച്ചു പറയാന്‍ നിര്‍വാഹങ്ങളൊന്നുമില്ലെന്നിരിക്കെ, ബൈബിളെഴുത്തുകാരന്‍ ഉദ്ദേശിച്ചത് മക്കയല്ലെന്ന് മിഷനറിമാര്‍ക്ക് തീര്‍ച്ചപ്പെടുത്താന്‍ കഴിയുന്നതെങ്ങനെയാണ്? പാറാന്‍ ഒരു മരുഭൂമിയാണെന്ന അധികവിവരം മാത്രമാണ് പരാമൃഷ്ട ബൈബിള്‍ വചനത്തില്‍ നിന്നു ലഭിക്കുന്നത്. ആ വിവരമാകട്ടെ, പാറാന്‍ മക്കയാകാനുള്ള സാധ്യതയെ ശക്തിപ്പെടുത്തുക മാത്രമാണ് ചെയ്യുന്നത്.

ഫലസ്ത്വീനിലെ ‘ബിഅ്ര്‍ ശബ’യാണ് പാറാന്‍ കൊണ്ട് ഉദ്ദേശിക്കപ്പെട്ടതെന്ന് ബൈബിള്‍ വചനങ്ങള്‍ തന്നെ വ്യക്തമാക്കുന്നതായി ചില മിഷനറിമാര്‍ വാദിച്ചുനോക്കാറുണ്ട്. പാറാന്‍ മരുഭൂമിയെക്കുറിച്ച് പറയുന്നതിനു തൊട്ടുമുമ്പ് ബൈബിള്‍ ഹാജറിന്റെ ബിഅ്ര്‍ ശബ അനുഭവങ്ങളെക്കുറിച്ച് പറയുന്നുവെന്നതാണ് അവരുടെ ന്യായം. വചനങ്ങള്‍ ഇപ്രകാരമാണ്: ”അബ്രഹാം അതിരാവിലെ എഴുന്നേറ്റ് കുറേ അപ്പവും ഒരു തുകല്‍ സഞ്ചിയില്‍ വെള്ളവുമെടുത്ത് ഹാഗറിന്റെ തോളില്‍ വെച്ചുകൊടുത്തു. മകനെയും ഏല്‍പിച്ചിട്ട് അവളെ പറഞ്ഞയച്ചു. അവള്‍ അവിടെ നിന്നുപോയി. ബിഅ്ര്‍ ശബ മരുപ്രദേശത്ത് അലഞ്ഞുനടന്നു. തുകല്‍ സഞ്ചിയിലെ വെള്ളം തീര്‍ന്നപ്പോള്‍ അവള്‍ കുട്ടിയെ ഒരു കുറ്റിക്കാട്ടില്‍ കിടത്തി. കുഞ്ഞ് മരിക്കുന്നത് എനിക്കു കാണാന്‍ വയ്യ എന്നുപറഞ്ഞ് അവള്‍ കുറേ അകലെ, ഒരു അമ്പെയ്ത്ത് ദൂരെച്ചെന്ന് എതിര്‍വശത്തേക്ക് തിരിഞ്ഞിരുന്നു. കുട്ടി ഉച്ചത്തില്‍ കരയാന്‍ തുടങ്ങി. കുട്ടിയുടെ കരച്ചില്‍ ദൈവം കേട്ടു. സ്വര്‍ഗത്തില്‍ നിന്ന് ദൈവത്തിന്റെ ദൂതന്‍ അവളെ വിളിച്ചുപറഞ്ഞു: ഹാഗാര്‍, നീ വിഷമിക്കേണ്ടാ; ഭയപ്പെടുകയും വേണ്ട. കുട്ടിയുടെ കരച്ചില്‍ ദൈവം കേട്ടിരിക്കുന്നു. എഴുന്നേറ്റ് കുട്ടിയെ കയ്യിലെടുക്കുക. അവനില്‍ നിന്ന് ഞാന്‍ വലിയൊരു ജനതയെ പുറപ്പെടുവിക്കും. ദൈവം അവളുടെ കണ്ണുതുറന്നു. അവള്‍ ഒരു കിണര്‍ കണ്ടു. അവള്‍ ചെന്ന് തുകല്‍ സഞ്ചി നിറച്ച്, കുട്ടിക്ക് കുടിക്കാന്‍ കൊടുത്തു. ദൈവം ആ കുട്ടിയോടു കൂടെയുണ്ടായിരുന്നു. അവന്‍ മരുഭൂമിയില്‍ പാര്‍ത്തു. അവന്‍ വളര്‍ന്ന് സമര്‍ത്ഥനായൊരു വില്ലാളിയായിത്തീര്‍ന്നു. അവര്‍ പാറാനിലെ മരുഭൂമിയില്‍ പാര്‍ത്തു.” (ഉല്‍പത്തി 21 : 14-21).

ബിഅ്ര്‍ ശബയാണ് പാറാന്‍ എന്ന വാദം ഈ ബൈബിള്‍ ഖണ്ഡികയില്‍ നിന്ന് നിര്‍ധരിച്ചെടുക്കുന്നത് അബദ്ധമാണ് എന്നാണ് സൂഷ്മ വായനയില്‍ ബോധ്യപ്പെടുന്നത്. ഈ വാചകങ്ങള്‍ക്ക് തൊട്ടുമുകളിലുള്ള ബൈബിള്‍ വചനങ്ങള്‍ കൂടി പരിശോധിക്കുക: ”ഈജിപ്തുകാരിയായ ഹാഗാറില്‍ അബ്രഹാത്തിന് ജനിച്ച മകന്‍, തന്റെ മകനായ ഇസ്ഹാഖിനോടുകൂടെ കളിക്കുന്നത് സാറ കണ്ടു. അവള്‍ അബ്രഹാത്തോടു പറഞ്ഞു: ആ അടിമപ്പെണ്ണിനെയും അവളുടെ മകനെയും ഇറക്കിവിടുക. അവളുടെ മകന്‍ എന്റെ മകന്‍ ഇസ്ഹാഖിനോടൊപ്പം അവകാശിയാകാന്‍ പാടില്ല. തന്മൂലം മകനെയോര്‍ത്ത് അബ്രഹാം വളരെ അസ്വസ്ഥനായി. എന്നാല്‍, ദൈവം അബ്രഹാത്തിനോട് അരുളിചെയ്തു: കുട്ടിയെക്കുറിച്ചും നിന്റെ അടിമപ്പെണ്ണിനെക്കുറിച്ചും നീ ക്ലേശിക്കേണ്ട. സാറാ പറയുന്നതുപോലെ നീ ചെയ്യുക. കാരണം ഇസ്ഹാഖിലൂടെയാണ് നിന്റെ സന്തതികള്‍ അറിയപ്പെടുക. അടിമപ്പെണ്ണില്‍ ജനിച്ച മകനെയും ഞാനൊരു ജനതയാക്കും. അവനും നിന്റെ മകനാണല്ലോ.” (ഉല്‍പത്തി 21 : 9-13).

ഇശ്മയേലിനോടും ഹാജറിനോടും സാറയ്ക്ക് അസഹിഷ്ണുത തോന്നിയെന്നും തന്റെയും ഇസ്ഹാഖിന്റെയും ജീവിതപരിസരങ്ങളില്‍ നിന്ന് ഹാജറിനെയും ഇശ്മയേലിനെയും പുറത്താക്കാന്‍ സാറ അബ്രഹാമിനോട്  ആവശ്യപ്പെട്ടുവെന്നും ഇതില്‍ അസ്വസ്ഥനായ അബ്രഹാമിനെ, സാറയുടെ ആവശ്യം രണ്ട് വ്യത്യസ്ത ഭൂപ്രദേശങ്ങളില്‍ അബ്രഹാമിന്റെ രണ്ട് സന്തതിശാഖകള്‍ മഹാജനസഞ്ചയങ്ങളായി മാറുക എന്ന ദൈവിക പദ്ധതിയുടെ നിവൃത്തിയായി മാറുമെന്ന് പറഞ്ഞ് കര്‍ത്താവ് ആശ്വസിപ്പിച്ചുവെന്നും തദടിസ്ഥാനത്തില്‍ ഇസ്ഹാഖും സാറയും താമസിച്ചിരുന്ന നാട്ടില്‍ നിന്ന് വിദൂരവും സുരക്ഷിതവുമായ മറ്റൊരിടത്തേക്ക് അബ്രഹാം ഹാഗറിനെയും ഇശ്മയേലിനെയും പറഞ്ഞുവിട്ടുവെന്നും ആണ് ഉപര്യുക്ത ബൈബിള്‍ വചനങ്ങള്‍ വിശദീകരിക്കുന്നത്. ബൈബിള്‍പ്രകാരം ഹാഗറിന്റെയും ഇശ്മയേലിന്റെയും പാലായനത്തിന്റെ ലക്ഷ്യം തന്നെ സാറയുടെയും ഇസ്ഹാഖിന്റെയും ചുറ്റുവട്ടങ്ങളില്‍ നിന്ന് രക്ഷപെടലായിരുന്നുവെന്ന് സാരം.

ബിഅ്ര്‍ ശബ ഇസ്ഹാഖിന്റെയും സാറയുടെയും സ്വന്തം നാടാണെന്ന് ബൈബിളില്‍ നിന്ന് വ്യക്തമാണ്.  ബിഅ്ര്‍ ശബ ഉള്‍ക്കൊള്ളുന്ന ഫലസ്ത്വീന്‍ പ്രവിശ്യകളിലാണ് സാറയും ഇസ്ഹാഖും ഇസ്ഹാഖിന്റെ സന്തതിപരമ്പരകളും നിലനിന്നതെന്ന് ബൈബിളെഴുത്തുകാര്‍ സൂചിപ്പിക്കുന്നുണ്ട് (ഉല്‍പത്തി 25). ഇതിനര്‍ത്ഥമെന്താണ്? ബിഅ്ര്‍ ശബയില്‍ തന്നെ ഹാജറും ഇശ്മയേലും പാര്‍പ്പുറപ്പിച്ചാല്‍ അബ്രഹാം എന്തിനുവേണ്ടിയാണോ അവരെ വീട്ടില്‍ നിന്ന് പറഞ്ഞയച്ചത്, ആ ലക്ഷ്യം നിറവേറുകയില്ല എന്നല്ലേ? വാസ്തവത്തില്‍, ഫലസ്ത്വീനില്‍ നിന്ന് പാലായനം ആരംഭിക്കുന്ന ഹാജറും ഇശ്മയേലും  ബിഅ്ര്‍ ശബയില്‍ അലഞ്ഞുതിരിയുക എന്നത് വളരെ സ്വാഭാവികമാണ്; കാരണം അവര്‍ അതുവരെ ജീവിച്ച പ്രദേശമാണത്.

തോല്‍ സഞ്ചിയും കൈക്കുഞ്ഞുമായി  ബിഅ്ര്‍ ശബ മരുഭൂമിയിലൂടെ തന്നെയായിരിക്കണം ഹാജര്‍ പ്രയാണമാരംഭിച്ചിട്ടുണ്ടാവുക. എന്നാല്‍ വെള്ളം തീര്‍ന്നതും കിണറുകള്‍ കാണാതെ ഹാജറ അസ്വസ്ഥമായതും മലക്ക് പ്രത്യക്ഷപ്പെട്ടതുമെല്ലാം അവിടെ വെച്ചാണെന്ന് ബൈബിളെഴുത്തുകാര്‍ ഉദ്ദേശിച്ചിരിക്കണമെന്നില്ല. ഒന്നാമതായി, ബിഅ്ര്‍ ശബ അവര്‍ക്ക് ചിരപരിചിതമാണ്. രണ്ടാമതായി, ബിഅ്ര്‍ ശബ വെള്ളക്കിണറുകള്‍കൊണ്ട് സമൃദ്ധമാണ്. ആ കിണറുകളില്‍ പലതും ബൈബിള്‍പ്രകാരം ഹാഗാറിന്റെ ഭര്‍ത്താവായ അബ്രഹാം പണി കഴിപ്പിച്ചതുമാണ്. ബിഅ്ര്‍ എന്ന വാക്കിനുതന്നെ അര്‍ത്ഥം കിണര്‍ എന്നാണ്. ബിഅ്ര്‍ ശബ എന്നാല്‍ ഏഴ് കിണറുകള്‍ (അറബിയില്‍ ബിഅ്ര്‍ അസ്സബ്അ്) എന്നോ കരാര്‍ പ്രകാരമുള്ള കണറുകള്‍ എന്നോ ആണ്.  ജലസംഭരണികള്‍ സമൃദ്ധമായിരുന്നതുകൊണ്ടാണ് ബിഅ്ര്‍ ശബ  ആ പേരില്‍ അറിയപ്പെടാന്‍ ഇടയായത് (www.bible-archeology.info/bible-city-beershaba.html).

തനിക്ക് ചിരപരിചിതമായിരുന്ന, ജലസ്രോതസ്സുകളുടെ പേരില്‍ അറിയപ്പെട്ടിരുന്ന ഒരു പ്രദേശത്ത് ഹാജറ കുഞ്ഞിനെ മരണവക്കില്‍ കിടത്തി വെള്ളത്തിനുവേണ്ടി കരയുക എന്നത് മനസ്സിലാക്കുവാന്‍ പ്രയാസമുള്ള കാര്യമാണ്. ബൈബിളെഴുത്തുകാരന്‍ ഉദ്ദേശിച്ചിരിക്കാന്‍ സാധ്യത ഇതാണ്: ബിഅ്ര്‍ ശബയില്‍ നിന്നാണ് ഹാജര്‍ പ്രയാണമാരംഭിച്ചത്. ആ പ്രയാണം അവരെ അപരിചിതവും ജലശൂന്യവുമായ മറ്റൊരു മരുപ്രദേശത്തെത്തിച്ചു. ഉല്‍പത്തി പുസ്തകത്തിലെ 21-ാം അധ്യായത്തിലെ പതിനാലാം വചനം പലായനത്തിന്റെ തുടക്കത്തില്‍ ബിഅ്ര്‍ ശബയില്‍ സംഭവിച്ച കാര്യത്തിലേക്കും 15 മുതല്‍ 21 വരെയുള്ള വചനങ്ങള്‍ മറ്റൊരു മരുഭൂമിയില്‍ നടന്ന സംഭവങ്ങളിലേക്കും വിരല്‍ ചൂണ്ടുന്നതാകാനാണ് സാധ്യതയുള്ളതെന്ന് ചുരുക്കം. ആ മരുഭൂമിയെയാണ് വചനങ്ങളുടെ കര്‍ത്താവ് പാറാന്‍ എന്നു വിളിക്കുന്നത്.

കുഞ്ഞിന്റെ കരച്ചില്‍കേട്ട് ദൈവം സവിശേഷദാനമായി സൃഷ്ടിച്ച വെള്ളക്കിണറാണ് പാറാന്റെ അടയാളമായി ഉല്‍പത്തി പുസ്തകം പറയുന്നതെന്ന് നാം കണ്ടു. ഹാജറിന്റെ മനോവ്യഥക്കും ഇശ്മയേലിന്റെ നിലവിളിക്കും പരിഹാരമായി ദിവ്യകാരുണ്യത്തില്‍ നിന്ന് ഉറവയെടുത്ത സംസം കിണര്‍ നിലനില്‍ക്കുന്ന മക്കയല്ലാതെ മറ്റേതാണീ പ്രദേശം? മറ്റൊരു നാട്ടിലും ഇത്തരമൊരു ജലപ്രവാഹമുണ്ടായതായി ബൈബിളില്‍ എവിടെയും പറയുന്നില്ല. ഹാഗാര്‍ പാലായനം ചെയ്തത് മക്കയിലേക്കല്ലെന്ന് ബൈബിളുപയോഗിച്ച് സമര്‍ത്ഥിക്കുവാനാകില്ലെന്ന് ചുരുക്കം. ബൈബിളിനെ കണ്ണുമടച്ച് വിശ്വസിക്കുന്നവര്‍ക്കുമാത്രമേ ബൈബിള്‍ വിവരണങ്ങളുടെ മാത്രം വെളിച്ചത്തിലുള്ള ചരിത്രാഖ്യാനത്തെ അംഗീകരിക്കാന്‍ കഴിയൂ എന്ന് നാം നേരത്തെ പറഞ്ഞു. എന്നാല്‍ ബൈബിളിനുപോലും ഇല്ലാത്ത വാദങ്ങള്‍ ബൈബിളിനുമേല്‍ കെട്ടിവെച്ച് ഇശ്മയേലിനെ മക്കയില്‍ നിന്ന് ‘തള്ളിപ്പുറത്താക്കാന്‍’ ശ്രമിക്കുന്ന മിഷനറിമാര്‍ എന്തു വൈജ്ഞാനിക സത്യസന്ധതയാണ് പുലര്‍ത്തുന്നത്?

തന്റെ പ്രവാചകത്വത്തിന് സെമിറ്റിക് അംഗീകാരം ലഭിക്കാന്‍വേണ്ടി പ്രവാചകന്‍ പുതുതായി ചമച്ചുണ്ടാക്കിയ സിദ്ധാന്തമാണ് മക്കയുടെ അബ്രഹാമിക പൈതൃകം. അബ്രഹാം ഹാഗാറിനെയും ഇശ്മയേലിനെയും മക്കയില്‍ കൊണ്ടുചെന്നാക്കിയെന്നും അവിടെ കഅ്ബ നിര്‍മിച്ചുവെന്നും ഇശ്മയേലിന്റെ സന്തതിപരമ്പരകള്‍ പ്രസ്തുത താഴ്‌വരയില്‍ നിലനിന്നുവെന്നുമെല്ലാം പ്രവാചകന്‍ അറബികളെ പറഞ്ഞുപഠിപ്പിക്കുകയായിരുന്നു. മദീനയിലെ ജൂതന്‍മാരുടെ അനുഭാവം നേടിയെടുക്കാനും കഅ്ബയുടെ നിയന്ത്രണം കൈക്കലാക്കാനും വേണ്ടി ഇസ്‌ലാമിന്റെ അവസാന കാലഘട്ടത്തില്‍ കഅ്ബ അബ്രഹാം സ്ഥാപിച്ചതാണെന്നും തന്റെ ഗോത്രം അബ്രഹാമിന്റെ പരമ്പരയാണെന്നും കഅ്ബയെ അതിന്റെ അബ്രഹാമിക വിശുദ്ധിയിലേക്ക് തിരിച്ചുകൊണ്ടുപോകാനാണ് താന്‍ ശ്രമിക്കുന്നതെന്നും മുഹമ്മദ് നബി (സ) അവകാശപ്പെടുകയാണ് ചെയ്തത് എന്ന് സമര്‍ഥിക്കാന്‍ ശ്രമിക്കുന്ന ചില ഓറിയന്റലിസ്റ്റ്/മിഷനറി പഠനങ്ങളുടെ പരാമൃഷ്ട വാദങ്ങളില്‍ എന്തെങ്കിലും സത്യമുണ്ടോ?

ല്ല. ഓറിയന്റലിസ്റ്റുകളും മിഷനറിമാരും മക്കയുടെയും അതുവഴി മുഹമ്മദ് നബി(സ)യുടെയും അബ്രഹാമിക പൈതൃകത്തെ നിഷേധിക്കുവാന്‍വേണ്ടി പല രീതിയില്‍ ഉന്നയിച്ചിട്ടുള്ള വാദങ്ങളാണ് നടേ പറഞ്ഞവയെല്ലാം. മുഹമ്മദ് നബി(സ)യെയും അറബികളെയും വിശുദ്ധമായ അബ്രഹാമിക ശൃംഖലയില്‍ കണ്ണിചേര്‍ത്തു പറയാന്‍ ഇസ്രാഈലി വംശീയ ദുരഭിമാനമാണ് ഇവരില്‍ പലര്‍ക്കും തടസ്സമാകുന്നതെന്ന് അവരുടെ രചനകള്‍ പരിശോധിച്ചാല്‍ മനസ്സിലാക്കാന്‍ കഴിയും.

നബിചരിത്രത്തിന്റെ മുഴുവന്‍ സ്രോതസ്സുകള്‍ പ്രകാരവും സ്ഥിരപ്പെട്ടിട്ടുള്ള കാര്യമാണ് വാസ്തവത്തില്‍ ഇസ്മാഈല്‍ നബി (അ)യുടെ പുത്രന്‍ അദ്‌നാന്റെ പുത്രപരമ്പരയില്‍ ഖുറയ്ശ് ഗോത്രത്തില്‍ ബനൂഹാശിം കുടുംബത്തിലാണ് പ്രവാചകന്റെ ജനനമുണ്ടായത് എന്നത്. കുടുംബപരമ്പരാ പഠനം വ്യവസ്ഥാപിതമായിത്തന്നെ നിലനിന്നിരുന്ന അറബ് സമൂഹത്തില്‍ ഖുറയ്ശികളുടെ അദ്‌നാനീ പാരമ്പര്യത്തെക്കുറിച്ച് യാതൊരു സംശയങ്ങളുമില്ലാതിരുന്നതുകൊണ്ടാണ് പ്രവാചകചരിത്രത്തിന്റെ സ്രോതസ്സുകള്‍ ഇക്കാര്യത്തില്‍ ഏകോപിക്കുന്നതും അബ്രഹാമും ഇസ്മാഈലും ചേര്‍ന്നുനിര്‍മിച്ച കഅ്ബയുടെ പരിപാലനം മുഴുവന്‍ മക്കക്കാരുടെയും അംഗീകാരത്തോടുകൂടി ഖുറയ്ശികള്‍ നിര്‍വഹിച്ചുപോന്നുവെന്ന വസ്തുത ഐകകണ്‌ഠ്യേന ചൂണ്ടിക്കാണിക്കുന്നതും. പ്രവാചക ജീവചരിത്രകാരന്‍മാര്‍ക്കിടയില്‍ അഭിപ്രായവ്യത്യാസങ്ങളില്ലാതെ സ്ഥിരപ്പെടുവാന്‍ മാത്രം പ്രബലമായ ഒരു ചരിത്രവസ്തുതയെ, പ്രവാചകനെ പ്രവാചകനായി അംഗീകരിക്കുവാന്‍ അഹങ്കാരം സമ്മതിക്കാത്തതുകൊണ്ടു മാത്രമാണ് നബിവിമര്‍ശകര്‍ നിഷേധിക്കാന്‍ ശ്രമിക്കുന്നത്.

ഇബ്രാഹിം നബി (അ) മാതാവ് ഹാജറിനെയും പുത്രന്‍ ഇസ്മാഈലിനെയും മക്കയില്‍ കൊണ്ടുചെന്നാക്കിയെന്നും അവിടെ ദൈവാനുഗ്രഹമായി സംസം ജലം ഉറവയെടുത്തെന്നും ജുര്‍ഹൂം എന്ന അറബിഗോത്രം ജലസാന്നധ്യം കാരണം അവിടെ തമ്പടിച്ചെന്നും അവരുമായുള്ള വൈവാഹികബന്ധത്തിലൂടെ ഇസ്മാഈല്‍ നബി (അ) യുടെ സന്തതികള്‍ മക്കാ താഴ്‌വരയില്‍ നിലവില്‍ വന്നുവെന്നും ഇബ്‌റാഹിം നബി (അ) മക്കയിലേക്കുവന്ന് ഇസ്മാഈലിനെയും കൂട്ടി കഅ്ബ നിര്‍മിച്ചുവെന്നും മുഹമ്മദ് നബി (സ) പറയാതെ തന്നെ അറബികള്‍ നിര്‍വിവാദം അംഗീകരിച്ചു വന്നിരുന്നതാണ്. ചിരപുരാതന കാലം മുതല്‍ തലമുറകളിലൂടെ അവര്‍ കൈമാറി വന്ന ദേശചരിത്രമാണത്.

മുഹമ്മദ് നബി (സ) സമൂഹത്തിനുമുന്നില്‍ അവതരിപ്പിച്ച ദൈവസന്ദേശങ്ങള്‍ ഒട്ടുമിക്കതും ആശയപരമായ കാരണങ്ങളാല്‍ അദ്ദേഹത്തിന്റെ പ്രബോധിത സമൂഹത്തിന്റെ കടുത്ത എതിര്‍പ്പുകള്‍ക്ക് വിധേയമായിട്ടുണ്ട്. ക്വുര്‍ആനിലും ഹദീഥുകളിലും ചരിത്രഗ്രന്ഥങ്ങളിലും അവ വിശദമായി രേഖപ്പെടുത്തിയിട്ടുമുണ്ട്. എന്നാല്‍ കഅ്ബയുടെയും മക്കയുടെയും ഇബ്‌റാഹിമീ-ഇസ്മാഈലീ പാരമ്പര്യത്തെ സംബന്ധിച്ച് നബി(സ) പറഞ്ഞ കാര്യങ്ങളെയൊന്നും ഒരു സമകാലീനനും ചോദ്യം ചെയ്തിട്ടില്ലെന്നതാണ് യാഥാര്‍ത്ഥ്യം. കാരണം, അവര്‍ക്ക് നേരത്തെ തന്നെ ബോധ്യമുണ്ടായിരുന്ന കാര്യങ്ങളായിരുന്നു അവയൊക്കെയും. പ്രവാചകകാലഘട്ടത്തില്‍ മക്കയിലും പരിസരപ്രദേശങ്ങളിലും ജീവിച്ചിരുന്ന ജനങ്ങള്‍ക്ക് വിശ്വാസി-അവിശ്വാസി ഭേദമില്ലാതെ യോജിപ്പുണ്ടായിരുന്ന ചരിത്രമാണിത്. നബി(സ)യുടെ സമകാലീനരായിരുന്ന അറബ് ജൂതരോ ക്രൈസ്തവരോ ഒന്നും ഇത്തരം വിമര്‍ശനങ്ങള്‍ ഉന്നയിച്ചിട്ടില്ലെന്ന കാര്യം ശ്രദ്ധേയമാണ്. തങ്ങളുടെ ആദര്‍ശപിതാവായ അബ്രഹാമിന് മക്കയുമായുള്ള ബന്ധത്തെ അവരാരും നിരാകരിച്ചില്ലെന്നുവേണം മനസ്സിലാക്കാന്‍. പ്രവാചകനില്‍ നിന്ന് ഇക്കാര്യങ്ങള്‍ നേരിട്ടുകേട്ട, ബൈബിളിന്റെ കുറേക്കൂടി പുരാതനമായ പ്രതികളെ ഉപജീവിച്ചിരുന്ന ഏഴാം നൂറ്റാണ്ടിലെ അറബ് ജൂത-ക്രിസ്ത്യാനികള്‍ക്ക് തോന്നിയിട്ടില്ലാത്ത മനപ്രയാസമാണ് അവരുടെ പിന്‍മുറക്കാര്‍ ഇത്തരം വിഷയങ്ങളില്‍ പ്രകടമാക്കുന്നത്.

അറേബ്യയില്‍ ചര്‍ച്ചക്കുപോലും സാധ്യതകളില്ലാത്തവിധം സ്പഷ്ടമായിരുന്ന ഒരു യാഥാര്‍ത്ഥ്യമാണ് കഅ്ബയുടെ ഇബ്‌റാഹിമീ പൈകൃകം എന്ന് ഇതെല്ലാം വ്യക്തമാക്കുന്നുണ്ട്. ഒരു നാടിന്റെ ചരിത്രം ഒരു സുപ്രഭാതത്തില്‍ യാതൊരു ഒച്ചപ്പാടുകളുമില്ലാതെ പ്രവാചകന്‍ മാറ്റി പ്രഖ്യാപിച്ചു എന്ന് സ്ഥാപിക്കാന്‍ ശ്രമിക്കുന്നത് എന്തുമാത്രം വലിയ അസംബന്ധമല്ല!

ഖുറയ്ശികളുടെയും കഅ്ബയുടെയും അബ്രഹാമിക പൈതൃകത്തെ സംബന്ധിച്ച് അറബികള്‍ക്കോ അവരെ പരിചയമുണ്ടായിരുന്നവര്‍ക്കോ സംശയങ്ങളൊന്നുമുണ്ടായിരുന്നില്ല എന്നതുകൊണ്ടുതന്നെ, ഖുറയ്ശികളെയും കഅ്ബയെയും സംബന്ധിച്ച പ്രവാചകപൂര്‍വ അറബ് സംഭാഷണങ്ങളിലെല്ലാം ഇബ്‌റാഹീമി-ഇസ്മാഈലി പാരമ്പര്യത്തെക്കുറിച്ച പരാമര്‍ശങ്ങള്‍ സര്‍വസാധാരണമായിരുന്നുവെന്നതാണ് യാഥാര്‍ത്ഥ്യം. ഖുറയ്ശികളുടെ ഇസ്‌ലാം പൂര്‍വചരിത്രം ക്രോഡീകരിക്കുവാന്‍ ശ്രമിച്ച ചരിത്രകാരന്‍മാരുടെയെല്ലാം രചനകളില്‍, അതുകൊണ്ടുതന്നെ, ഇത്തരത്തിലുള്ള അനേകം നിവേദനങ്ങള്‍ കണ്ടെത്താന്‍ കഴിയും. പ്രവാചകന്റെ പിതാമഹനായിരുന്ന അബ്ദുല്‍ മുത്വലിബ് ”ഞങ്ങള്‍ അല്ലാഹുവിന്റെ നാട്ടിലെ അല്ലാഹുവിന്റെ ജനതയാണ്; അത് എല്ലായ്‌പ്പോഴും (അല്ലാഹുവുമായുള്ള) ഇബ്‌റാഹിമീന്റെ ഉടമ്പടി പ്രകാരമായിരുന്നു” (നഹ്‌നു അഹ്‌ലുല്ലാഹി ഫീ ബലദതിഹി, ലം യസല്‍ ദാക അലാ അഹ്ദി ഇബ്‌റാഹീം) എന്ന് പറഞ്ഞിരുന്നത് യഅ്ക്വൂബി തന്റെ താരിഖില്‍ ഉദ്ധരിക്കുന്നുണ്ട് (1/253).

പ്രവാചകന്റെ പിതൃവ്യനും ഗുണകാംക്ഷിയും എന്നാല്‍ അമുസ്‌ലിമും ആയിരുന്ന അബൂത്വാലിബ്, താന്‍ മുഹമ്മദിന്റെ (സ) പ്രവാചകത്വം അംഗീകരിക്കുന്നില്ലെങ്കിലും അദ്ദേഹത്തെ ശത്രുക്കളുടെ പീഡനത്തിന് വിട്ടുകൊടുക്കുകയില്ലെന്ന് മക്കളുടെ മുഴുവന്‍ പൈതൃകചിഹ്നങ്ങളെയും പിടിച്ചാണയിട്ട് ആലപിച്ച കവിതയില്‍ ”ഇബ്‌റാഹിമീന്റെ പാദമുദ്ര ഇപ്പോഴും ശുദ്ധമായി കിടക്കുന്ന ശില; അതിലെ രണ്ട് പാദമുദ്രകളും ചെരുപ്പില്ലാതെ നഗ്നമാണ്” എന്ന് മക്വാമു ഇബ്‌റാഹീമിനെക്കുറിച്ച് പ്രസ്താവിച്ചതായി ഇബ്‌നു ഇസ്ഹാക്വ്  നിവേദനം ചെയ്യുന്നുണ്ട്. (A. Guillaume, The Life of Muhammed: A Translation of Ibn Ishaq’s Sirat Rasul Allah (Oxford University Press, 2007), p. 123).

മുദാര്‍ ഗോത്രക്കാര്‍ ”ഇസ്മാഈല്‍ നമുക്കായി അവശേഷിപ്പിച്ച മതത്തിന്റെ കടമകള്‍ നമുക്കുവേണ്ടി നിര്‍വഹിച്ചുവന്നത് ഖുറയ്ശികളാണ്” എന്ന് പറയാറുണ്ടായിരുന്നുവെന്ന് മുഹമ്മദ് ഇബ്‌നു ഹബീബ് തന്റെ മുഹബ്ബറില്‍ (264) രേഖപ്പെടുത്തുന്നുണ്ട്.  ഇസ്മാഈലിന്റെ നേര്‍പൈതൃകം കാരണം അറബികള്‍ ഖുറയ്ശികള്‍ക്കു കല്‍പിച്ചുനല്‍കിയിരുന്ന ആദരവിന്റെ രേഖ കൂടിയാണ് മുദാര്‍ ഗോത്രക്കാരില്‍ നിന്നുള്ള പാരമൃഷ്ട ഉദ്ധരണി. ഖുറയ്ശികള്‍ അറബികള്‍ക്കിടയില്‍ അറിയപ്പെട്ടിരുന്നത് ‘സ്വരീഹു വുല്‍ദി ഇസ്മാഈല്‍’ (ഇസ്മാഈലിന്റെ സുവ്യക്തമായ/ഋജുവായ പരമ്പര) എന്നായിരുന്നുവെന്ന് ഇബ്‌നു ഇസ്ഹാക്വ് രേഖപ്പെടുത്തിയത് (4/205) ഇവിടെ ചേര്‍ത്തുവായിക്കുന്നത് പ്രസക്തമാണ്.

ഇസ്മാഈല്‍ നബി(അ)യുടെ പിന്‍മുറക്കാര്‍ എന്ന നിലയിലും കഅ്ബയുടെ കൈകാര്യകര്‍ത്താക്കള്‍ എന്ന നിലയിലുമാണ് ഖുറയ്ശികള്‍ മക്കയില്‍ സവിശേഷമായ സാമൂഹികാംഗീകാരങ്ങള്‍ ആസ്വദിച്ചിരുന്നത്. ഖുറയ്ശികളും ഥക്വീഫ ് ഗോത്രക്കാരും തമ്മില്‍ നടന്ന ഒരു സംഭാഷണത്തില്‍ കഅ്ബാ പരിപാലനത്തില്‍ ഥക്വീഫ് ഗോത്രക്കാരെ പങ്കാളിയാക്കാന്‍ തങ്ങള്‍ സന്നദ്ധമാണെന്നും അതിനുപകരമായി ഥക്വീഫുകാരുടെ നിയന്ത്രണത്തിലുള്ള പ്രവിശ്യകളിലെ സാമ്പത്തിക ഇടപാടുകളില്‍ തങ്ങള്‍ക്കവസരം നല്‍കണമെന്നും ഖുറയ്ശി പ്രതിനിധികള്‍ പറഞ്ഞപ്പോള്‍ ഥക്വീഫ് ഗോത്രനേതാക്കള്‍ നല്‍കിയ മറുപടി ഇബ്‌നു ഹബീബ് നിവേദനം ചെയ്യുന്നതിപ്രകാരമാണ്: ”നിങ്ങളെയെങ്ങനെയാണ് ഞങ്ങള്‍ ഞങ്ങളുടെ പിതാക്കള്‍ ആയുധങ്ങളില്ലാതെ വെറുംകൈകള്‍കൊണ്ട് പാറ വെട്ടിത്തുരന്ന് പാര്‍പ്പുറപ്പിച്ച ഭൂമിയുടെ അവകാശത്തില്‍ പങ്കുകാരാക്കുക? (പകരം നിങ്ങള്‍ ഞങ്ങളുമായി പങ്കുവെക്കാമെന്ന് പറയുന്ന) വിശുദ്ധ ഗേഹം നിങ്ങള്‍ സ്വയം  ഉണ്ടാക്കിയതല്ല. അത് സ്ഥാപിച്ചത് ഇബ്‌റാഹീം ആയിരുന്നു.” (കയ്ഫ നുശ്‌രികുകും ഫീ വാദിന്‍ നസലഹു അബൂനാ വ ഹറഫഹു ബിയദയ്ഹി ഫിസ്സ്വഖ്‌രി ലം യഖ്ഫിര്‍ഹു ബില്‍ ഹദീദ്, വ അന്‍തും ലം തജ്അലുല്‍ ഹറമ, ഇന്നമാ ജഅലഹു ഇബ്‌റാഹീം/ കിതാബുല്‍ മുനമ്മക്വ് ഫീ അഖ്ബാരി ഖുറയ്ശ്, 280. വിര്‍ജീനിയ യൂനിവേഴ്‌സിറ്റി പ്രസ് 1964ല്‍ മുഹമ്മദ് ഇബ്‌നു ഹബീബിന്റെ ഈ ഗ്രന്ഥം പുനഃപ്രസിദ്ധീകരിച്ചിട്ടുണ്ട്).

പ്രവാചകന്റെ മദീനാ കാലഘട്ടത്തില്‍ ഇസ്‌ലാം ആശ്ലേഷിച്ച മദീനയിലെ പ്രഗല്‍ഭനായ ജൂതപണ്ഡിതന്‍ അബ്ദുല്ലാഹിബ്‌നു സലാം ജൂതനായിരിക്കെ മദീനയിലെ ജൂതനേതാക്കളോട് കഅ്ബയെ ഉദ്ദേശിച്ചുകൊണ്ട് ”നമ്മുടെ പിതാവ് ഇബ്‌റാഹീമിന്റെ പള്ളി സന്ദര്‍ശിക്കുവാന്‍ ഞാന്‍ ആഗ്രഹിക്കുന്നു”വെന്ന് പറഞ്ഞതായി വ്യക്തമാക്കുന്ന നിവേദനങ്ങളും (ജലാലുദ്ദീന്‍ സുയൂത്വി, അദ്ദുര്‍റുല്‍ മന്‍ഥൂര്‍ ഫിത്തഫ്‌സീരി ബില്‍ മഅ്ഥൂര്‍, 6/410) അറേബ്യന്‍ സമൂഹത്തില്‍ ഇക്കാര്യങ്ങള്‍ക്കുണ്ടായിരുന്ന സര്‍വസമ്മതിയാണ് സൂചിപ്പിക്കുന്നത്. പ്രവാചകകാലഘട്ടത്തിലോ അതിനുമുമ്പോ, ഖുറയ്ശികളുടെയും കഅ്ബയുടെയും ഇബ്‌റാഹീമി-ഇസ്മാഈലി വേരുകള്‍ ഒരു തര്‍ക്കവിഷയമേ ആയിരുന്നില്ലെന്നും സര്‍വാംഗീകൃതമായ ഒരു ചരിത്രവസ്തുതയില്‍ നിന്ന് പാഠം പഠിച്ച് വിശ്വാസപരമായ വിശുദ്ധി വീണ്ടെടുക്കാന്‍ അറബികളെ ആഹ്വാനം ചെയ്യുക മാത്രമാണ് മുഹമ്മദ് നബി (സ) ചെയ്തത് എന്നുമാണ് ഇതെല്ലാം വ്യക്തമാക്കുന്നത്.

മുഹമ്മദ് നബി(സ)യുടെ കാലത്തെ മക്കന്‍ അറബികള്‍ തങ്ങളില്‍ പലരുടെയും പൂര്‍വപിതാക്കളായും കഅ്ബയുടെ സ്ഥാപകരായും ഇബ്‌റാഹിം(അ)യെയും, ഇസ്മാഈല്‍(അ)യെയും അറിയുകയും ആദരിക്കുകയും ചെയ്തുവന്നവരായിരുന്നുവെന്ന് മുഹമ്മദ് നബി (സ) മക്ക കീഴടക്കുമ്പോഴുള്ള കഅ്ബാലയത്തിന്റെ അവസ്ഥ തന്നെ വ്യക്തമാക്കുന്നുണ്ട്. കഅ്ബാലയത്തിനകത്ത് ഇബ്‌റാഹിമിന്റെയും ഇസ്മാഈലിന്റെയും ചിത്രങ്ങള്‍ കൊത്തിവെച്ചിട്ടുണ്ടായിരുന്നു ബഹുദൈവാരാധക അറബികള്‍. മക്ക, മദീന കേന്ദ്രമായുള്ള ഇസ്‌ലാമിക രാഷ്ട്രത്തിന്റെ ഭാഗമായി ഹിജ്‌റ എട്ടാം വര്‍ഷം മാറിയപ്പോള്‍ രാഷ്ട്രത്തലവന്‍ എന്ന നിലയില്‍ പ്രവാചകന്‍ (സ) പ്രസ്തുത ചിത്രങ്ങള്‍ മായ്ച്ചുകളയാന്‍ കല്‍പിക്കുകയാണ് ചെയ്തത്.

അമ്പുകളുപയോഗിച്ചുള്ള അന്ധവിശ്വാസജഡിലമായ ഭാഗ്യപരീക്ഷണത്തില്‍ ഇബ്‌റാഹീമും ഇസ്മാഈലും ഏര്‍പ്പെട്ടിരിക്കുന്നതായാണ് കഅ്ബക്കകത്ത് ചിത്രീകരിക്കപ്പെട്ടിരുന്നത്.  അറബികള്‍ ഇബ്‌റാഹീമിനെയും ഇസ്മാഈലിനെയും കഅ്ബയുമായുള്ള ബന്ധത്തിന്റെ പേരിലും രക്തബന്ധത്തിന്റെ പേരിലും ആദരിച്ചപ്പോഴും അവര്‍ പഠിപ്പിച്ച ശുദ്ധ ഏകദൈവാരാധനയില്‍ നിന്ന് ബഹുദൂരം അകന്നുപോവുകയും അവരെത്തന്നെ ബഹുദൈവാരാധനാപരമായ പശ്ചാത്തലങ്ങളില്‍ കൊണ്ടുപോയി പ്രതിഷ്ഠിക്കുകയും ചെയ്യുകയാണുണ്ടായതെന്ന പ്രവാചകന്റെ(സ) വിശദീകരണത്തെ കൃത്യമായി സാധൂകരിക്കുന്ന പുരാവസ്തു രേഖ കൂടിയായിരുന്നു വാസ്തവത്തില്‍ പ്രസ്തുത ചിത്രങ്ങള്‍. ഇത്തരം ചിത്രങ്ങള്‍ പ്രതിഷ്ഠിക്കപ്പെട്ടാല്‍ ആരാധനാലയത്തില്‍ പ്രപഞ്ചനാഥന്റെ അനുഗ്രഹവുമായി കടന്നുവരുന്ന മലക്കുകളുടെ സാന്നിധ്യം ഇല്ലാതാവുകയാണ് ചെയ്യുകയെന്നും ഇബ്‌റാഹിം, ഇസ്മാഈല്‍ പ്രവാചകന്‍മാരുമായി യാതൊരു ബന്ധവുമില്ലാത്ത ബഹുദൈവാരാധനാപരമായ ഭാഗ്യപരീക്ഷണാനുഷ്ഠാനത്തെയാണ് അവരുടെ പേരില്‍ ചിത്രകാരന്‍മാര്‍ ആരോപിച്ചിരിക്കുന്നതെന്നും അവ സ്ഥാപിച്ചവര്‍ പ്രപഞ്ചരക്ഷിതാവായ അല്ലാഹുവിന്റെ ശാപത്തിനര്‍ഹരാണെന്നും കഅ്ബക്കകത്തു കയറി പരാമൃഷ്ട ചിത്രങ്ങള്‍ കാണാനിടയായപ്പോള്‍ നബി(സ) പ്രതിവചിച്ചതായി ഇബ്‌നു അബ്ബാസില്‍ നിന്നുള്ള തീര്‍ത്തും ആധികാരികമായ നിവേദനങ്ങളിലുണ്ട് (ബുഖാരി). അമ്പുകളുപയോദിച്ച് ഭാഗ്യം പരീക്ഷിക്കുന്ന ജാഹിലിയ്യാ അറബ് സമ്പ്രദായത്തിനുപോലും ഇസ്മാഈലിനെക്കുറിച്ചുള്ള അവരുടെ ഓര്‍മകളുമായി ബന്ധമുണ്ടായിരുന്നുവെന്നതാണ് വാസ്തവം. ഇസ്മാഈല്‍ നബി (അ) പ്രഗല്‍ഭനായ ഒരു വില്ലാളിയായി മക്കയില്‍ വളര്‍ന്നുവന്നുവെന്ന അറിവില്‍നിന്നാണ് പില്‍ക്കാലത്ത് അവരുടെ അന്ധവിശ്വാസപരമായ ചടങ്ങുകളില്‍ അമ്പും വില്ലും വന്ന് നിറഞ്ഞതും ഇസ്മാഈലിന്റെ(അ) ചിത്രത്തിനും ഹുബുലിന്റെ വിഗ്രഹത്തിനുമെല്ലാം അമ്പുകളുടെ അകമ്പടിയുണ്ടായിത്തീര്‍ന്നതും.

ഇസ്മാഈല്‍ മരുഭൂമിയില്‍ ദൈവസംരക്ഷണത്തില്‍ വളര്‍ന്നുവന്നതിനെക്കുറിച്ച് പരാമര്‍ശിക്കവെ ബൈബിള്‍ തന്നെ ഇക്കാര്യത്തിലേക്ക് സൂചന നല്‍കുന്നുണ്ട്. ”ദൈവം ആ കുട്ടി (ഇശ്മയേല്‍)യോട് കൂടിയുണ്ടായിരുന്നു. അവന്‍ മരുഭൂമിയില്‍ പാര്‍ത്തു. അവന്‍ വളര്‍ന്നു സമര്‍ത്ഥനായൊരു വില്ലാളിയായിത്തീര്‍ന്നു” (ഉല്‍പത്തി 21 : 20). ഇക്കാര്യത്തെ ശരിവെക്കുന്ന പ്രസ്താവന മുഹമ്മദ് നബി(സ)യും നടത്തിയിട്ടുണ്ട്. അമ്പെയ്ത്തു മത്സരം നടത്തിക്കൊണ്ടിരുന്ന ഒരു അറബ് ഗോത്രത്തെക്കണ്ടപ്പോള്‍ പ്രവാചകന്‍ (സ) അവരോട്, ”ഇസ്മാഈല്‍ സന്തതികളേ, നിങ്ങള്‍ അമ്പെയ്ത്ത് പരിശീലിക്കുക; കാരണം നിങ്ങളുടെ പിതാവ് പ്രഗല്‍ഭനായ ഒരു വില്ലാളിയായിരുന്നു” എന്ന് പറയുകയുണ്ടായി (ബുഖാരി).

ഇസ്മാഈല്‍ നബി (അ) പഠിപ്പിച്ച ചില അനുഷ്ഠാനങ്ങള്‍ തികഞ്ഞ ബഹുദൈവാരാധനയിലേക്ക് കൂപ്പുകുത്തിയപ്പോഴും മക്കക്കാര്‍ക്കിടയില്‍ പ്രവാചകകാലഘട്ടം വരെ അതേപടി നിലനിന്നുവെന്നതും ഇതുപോലെത്തന്നെ ശ്രദ്ധേയമാണ്. ഇവയില്‍ ഏറ്റവും പ്രധാനപ്പെട്ട ഒന്നായിരുന്നു ചേലാകര്‍മം. ലിംഗ പരിഛേദനയ്ക്കുള്ള കല്‍പന ദൈവം പുറപ്പെടുവിച്ചത് അബ്രഹാമിന്റെ കാലത്താണ് എന്നാണ് ബൈബിളില്‍ നിന്നു മനസ്സിലാകുന്നത്. അബ്രഹാമിന് തൊണ്ണൂറും ഇശ്മയേലിന് പതിമൂന്നും വയസ്സുള്ളപ്പോള്‍ ലിംഗപരിഛേദനയ്ക്കുള്ള കല്‍പന വന്നുവെന്നും അവര്‍ രണ്ടുപേരും അപ്പോള്‍ പരിഛേദനയറ്റുവെന്നും സകലരെയും എട്ടാം ദിവസം ചേലാകര്‍മം ചെയ്യണമെന്ന നിയമം പ്രഖ്യാപിച്ചുവെന്നും ഉല്‍പത്തി പുസ്തകം വിശദീകരിക്കുന്നു (അധ്യായം 17). ഇസ്മാഈലിന്റെ പൈതൃകമെന്ന നിലയ്ക്കുതന്നെ അറബികള്‍ കുട്ടികളെ ചേലാകര്‍മം ചെയ്യുന്ന പതിവ് നിലനിര്‍ത്തിപ്പോന്നു. മുഹമ്മദ് നബി(സ)യുടെ ആഗമനകാലത്തെ ജാഹിലിയ്യാ അറബികള്‍ക്കിടയില്‍ പോലും ലിംഗാഗ്ര ഛേദനം സാര്‍വത്രികമായിരുന്നു. അവരതിലേക്ക് ബഹുദൈവാരാധനയുടെ ഘടകങ്ങള്‍ ചേര്‍ത്തിരുന്നുവെന്ന് മാത്രമേയുള്ളൂ. കഅ്ബാലയത്തിനകത്ത് തങ്ങള്‍ സ്ഥാപിച്ച ഹുബുല്‍ ദേവന്റെ വിഗ്രഹത്തിന്റെ മുന്നില്‍ പോയി അമ്പുകളുപയോഗിച്ച് ഭാഗ്യപരീക്ഷണം നടത്തിയാണ് ചേലാകര്‍മത്തിന്റെ വിശദാംശങ്ങള്‍ ജാഹിലിയ്യാ അറബികള്‍ നിശ്ചയിച്ചിരുന്നത് എന്ന് ഇബ്‌നു ഇസ്ഹാക്വ് രേഖപ്പെടുത്തിയിട്ടുണ്ട്.

ചേലാകര്‍മം ഇസ്മാഈലി പാരമ്പര്യത്തിന്റെ ഭാഗമായി അറബികള്‍ക്കിടയില്‍ നിലനിന്നുപോന്നതിനെക്കുറിച്ച് ഇസ്‌ലാമിക ചരിത്രസ്രോതസ്സുകള്‍ മാത്രമല്ല സംസാരിക്കുന്നത്. സി. ഇ ഒന്നാം നൂറ്റാണ്ടില്‍ റോമാ സാമ്രാജ്യത്തില്‍ ജീവിച്ച ജൂതപണ്ഡിതനും ചരിത്രകാരനുമായ ജോസിഫസ് ഫ്‌ളേവിയസ് (Josephus Flavius) അദ്ദേഹത്തിന്റെ പ്രശസ്തമായ Antiquities of the Jews എന്ന ഗ്രന്ഥത്തില്‍ അറബികള്‍ ”പതിമൂന്ന് വയസ്സിനുശേഷം ചേലാകര്‍മം നടത്തുന്നു; ഇശ്മയേല്‍, ആ ജനതയുടെ സ്ഥാപകന്‍, അബ്രഹാമിന്റെ വെപ്പാട്ടിയില്‍ പിറന്ന മകന്‍, ആ വയസ്സിലാണ് പരിഛേദനയേറ്റത്” എന്നു വ്യക്തമായി എഴുതുന്നുണ്ട്. മുഹമ്മദ് നബി(സ)ക്ക് ആറു നൂറ്റാണ്ടോളം മുമ്പും അറബികളുടെ ചേലാകര്‍മം പ്രശസ്തമായിരുന്നുവെന്നും ഇസ്മാഈലിന്റെ സന്തതിപരമ്പരകള്‍ പുലര്‍ത്തിപ്പോരുന്ന സ്വാഭാവികത എന്ന നിലയിലാണ് ജൂതപണ്ഡിതന്‍മാര്‍ പോലും ഈ പ്രതിഭാസത്തെ മനസ്സിലാക്കിയിരുന്നത് എന്നും ജോസിഫസിന്റെ പ്രസ്താവന തെളിയിക്കുന്നുണ്ട്. അറബികളുടെ ഇസ്മാഈലി വേരുകള്‍ മുഹമ്മദ് നബി(സ)യുടെയും മുസ്‌ലിംകളുടെയും പുതിയ ‘കണ്ടുപിടുത്ത’മാണെന്ന മിഷനറി വാദം കൂടിയാണ് ജോസിഫസിന്റെ വരികള്‍ക്കുമുന്നില്‍ ദയനീയമായി തകര്‍ന്നുപോകുന്നത്.

ബഹുദൈവാരാധനയുടെ ചിഹ്നങ്ങള്‍കൊണ്ട് നിറഞ്ഞ മക്കയുടെ സാമൂഹ്യപരിസരം ചികഞ്ഞ് ഉള്ളിലേക്കുപോയാല്‍ ഇബ്‌റാഹിം നബി(അ)യുടെയും ഇസ്മാഈല്‍ നബി(അ)യുടെയും പ്രബോധനങ്ങളുടെ ശേഷിപ്പുകള്‍ ഉറങ്ങിക്കിടക്കുന്നത് കണ്ടെത്താന്‍ കഴിയുമെന്ന, ദിവ്യബോധനങ്ങളുടെ വെളിച്ചത്തിലുള്ള മുഹമ്മദ് നബി(സ)യുടെ നിലപാടിനെയാണ് ഈ പശ്ചാതലത്തെളിവുകളെല്ലാം സാധൂകരിക്കുന്നത്. ദിവ്യബോധനങ്ങളുടെ വെളിച്ചത്തില്‍, ഹജ്ജിലെയും ഉംറയിലെയും ഇബ്‌റാഹീമി ഘടകങ്ങളെ അവയുടെ ആദിമവിശുദ്ധിയില്‍ പുനഃസ്ഥാപിക്കുകയും പില്‍ക്കാലത്ത് വന്നുചേര്‍ന്ന ബഹുദൈവാരാധനാപരമായ അനുഷ്ഠാനങ്ങളെയും അന്ധവിശ്വാസങ്ങളെയും തൂത്തുകളയുകയുമാണ് പ്രവാചകന്‍(സ) ചെയ്തത്. പില്‍ക്കാല വ്യതിയാനങ്ങള്‍ വഴി ദുഷിച്ചപോയ അബ്രഹാമിക പാരമ്പര്യമാണ് മക്കയില്‍ നിലനില്‍ക്കുന്നതെന്നും അതിനെ ശുദ്ധീകരിച്ചെടുക്കുകയാണ് ഇബ്‌റാഹീം നബി(അ)യോട് യഥാര്‍ത്ഥത്തില്‍ സ്‌നേഹമുള്ളവര്‍ ചെയ്യേണ്ടതെന്നും വസ്തുനിഷ്ഠമായി കാര്യങ്ങളെ നോക്കിക്കാണുന്നവര്‍ക്കെല്ലാം മനസ്സിലാകുമായിരുന്നു.

അതുകൊണ്ടാണ്, പ്രവാചകനുമുമ്പ് സി. ഇ അഞ്ചാം നൂറ്റാണ്ടില്‍ ഗസ്സയില്‍ ജീവിച്ച സോസമേമസ് (Sozomemes) എന്ന ക്രൈസ്തവ പണ്ഡിതന്‍ അദ്ദേഹത്തിന്റെ രചനയില്‍ ഈസ്മാഈലീ പരമ്പരയിലുള്ള അറബികളെക്കുറിച്ചും ബഹുദൈവാരാധകരായ അയല്‍സമൂഹങ്ങളുമായുള്ള സമ്പര്‍ക്കം അവരുടെ വിശ്വാസത്തെ മലിനമാക്കുന്നതിനെക്കുറിച്ചും തന്റെ രചനകളില്‍ രേഖപ്പെടുത്തുകയും മോശെക്കുമുമ്പ് ഇസ്രാഈല്യര്‍ ജീവിച്ചിരുന്ന നിയമങ്ങളനുസരിച്ചു തന്നെയാണ് വിശ്വാസവ്യതിചലനം സംഭവിക്കുന്നതുവരെ അറബികളും ജീവിച്ചിരുന്നത് എന്നും അവരില്‍ ചിലരെങ്കിലും ഇപ്പോഴും ആ പൈതൃകം മുറുകെപ്പിടിക്കുന്നുണ്ടെന്നും ഇബ്‌റാഹീമീ നിയമങ്ങളെ വീണ്ടെടുക്കാന്‍ ഇസ്രാഈല്യരുമായുള്ള ആശയവിനിമയങ്ങള്‍ അവര്‍ക്ക് സഹായകരമാകുന്നുണ്ടെന്നും നിരീക്ഷിക്കുകയും ചെയ്തത്. ക്രൈസ്തവ സഭാപിതാക്കളുടെ പ്രാചീന ഗ്രീക്ക് എഴുത്തുകള്‍ ക്രോഡീകരിച്ച് 1857-66 കാലഘട്ടത്തില്‍ പാരീസില്‍ നിന്ന് ജെ.പി മിഗ്‌നെയുടെ നേതൃത്വത്തില്‍ 161 വോള്യങ്ങളില്‍ പ്രസിദ്ധീകരിക്കപ്പെട്ട Patrologia Graeciaയില്‍ ഈ പരാമര്‍ശങ്ങളെയുള്‍കൊള്ളുന്ന Sozomen histori ecclesiastia എടുത്തുചേര്‍ത്തിട്ടുണ്ട്. (www.patristica.net എന്ന വെബ്‌സൈറ്റില്‍ ഈ രേഖകള്‍ ലഭ്യമാണ്).

മക്കയുടെ ഇബ്‌റാഹീമീ പൈതൃകം പ്രവാചകന്‍ പറഞ്ഞുണ്ടാക്കിയതാണെന്ന് പറയുന്ന ഓറിയന്റലിസ്റ്റുകളും മിഷനറിമാരും പ്രവാചകനുമുമ്പ് ജീവിച്ച ജൂത-ക്രൈസ്തവ പണ്ഡിതന്മാര്‍ മക്കയുടെ അബ്രഹാമിക പൈതൃകത്തെ സാധൂകരിച്ച് സംസാരിച്ചതിനെക്കുറിച്ച് പാലിക്കുന്ന മൗനം, കാപട്യത്തില്‍ നിന്ന് മാത്രമാണ് ഉയിര്‍കൊള്ളുന്നത്.

ക്രൈസ്തവ വിമര്‍ശകരാണ് മുഹമ്മദ് നബിക്ക് ലഭിച്ച വെളിപാടുകള്‍ പിശാചില്‍നിന്നാണെന്ന ആരോപണം ഉന്നയിക്കുന്നത്. മുഹമ്മദി(സ)ന് ലഭിച്ച വെളിപാടുകള്‍ പിശാചുബാധയുടെ ഫലമായുണ്ടായതാണെന്ന് വരുത്തിത്തീര്‍ക്കാനാണ് സി.ഡി. ഫാണ്ടര്‍, ക്ലേയ്ര്‍ ടിസ്ഡാല്‍, ജോഷ്മാക്ഡവല്‍, ജോണ്‍ജില്‍ ക്രിസ്റ്റ്, ജി. നെഹ്ല്‍സ് തുടങ്ങിയ ക്രൈസ്തവ ഗ്രന്ഥകാരന്മാരെല്ലാം ശ്രമിച്ചിരിക്കുന്നത്. യേശുക്രിസ്തുവിന്റെ ക്രൂശീകര ണത്തെയും അതുമൂലമുള്ള പാപപരിഹാരത്തെയും നിഷേധിച്ചു കൊണ്ട് മനുഷ്യരാശിയെ പാപത്തിന്റെ ഗര്‍ത്തത്തില്‍തന്നെ തളച്ചിടുവാനുള്ള പി ശാചിന്റെ പരിശ്രമമാണ് ഖുര്‍ആനിന്റെ രചനക്കു പിന്നിലുള്ളതെന്ന് അവര്‍ വാദിക്കുന്നു. മനുഷ്യശരീരത്തില്‍ പിശാച് കയറിക്കൂടുമോ? പിശാചുബാധ കൊണ്ട് ഒരാള്‍ക്ക് രോഗങ്ങളുണ്ടാവുമോ? പിശാചുബാധിച്ച ഒരാള്‍ക്ക് വെളിപാടുണ്ടാവുമോ? തുടങ്ങിയ ചര്‍ച്ചകള്‍ ഇവിടെ അപ്രസക്തമാണ്. ബൈ ബിള്‍ പ്രകാരം പിശാചുബാധിച്ച ഒരാളില്‍ കാണപ്പെടുന്ന അസുഖങ്ങള്‍ എന്തെല്ലാമാണെന്ന് പരിശോധിക്കുക.

1. ബുദ്ധിഭ്രമത്താല്‍ അലറി വിളിക്കല്‍ (മാര്‍ക്കോസ് 1:24, ലൂക്കോസ് 9:39, യോഹന്നാന്‍ 10:20)

2. സ്വയം നശീകരണ പ്രവണത (മത്തായി 55:9, 18: 17, 15:32, മര്‍ക്കോസ് 5: 13, ലൂക്കോസ്, 8:33)

3. നഗ്‌നമായി നടക്കുന്നതിനുള്ള പ്രവണത (ലൂക്കോസ് 8:2, 8:35)

4. പിശാചിനാല്‍ തള്ളയിടപ്പെടുക (മത്തായി 17:15, മര്‍ക്കോസ് 1:26,9:18, 9:20,9:26)

5. മൂകത (മര്‍ക്കോസ് 9:25, 9:32, 12:22, ലൂക്കോസ് 11:14)

6. ബധിരത (മര്‍ക്കോസ് 9: 25)

7. അന്ധത (മത്തായി 12:22)

8. മറ്റാരും കാണാത്തത് കാണുകയും അറിയുകയും ചെയ്യുക (മര്‍ക്കോസ് 1:24, ലൂക്കോസ് 4:3, മത്തായി 8:29)

പിശാചുബാധിതനില്‍ കാണപ്പെടുന്നതെന്ന് ബൈബിള്‍ ഉദ്‌ഘോഷിക്കു ന്ന ലക്ഷണങ്ങളൊന്നും മുഹമ്മദി(സ)ല്‍ ഉണ്ടായിരുന്നതായി നമുക്ക് കാ ണാന്‍ കഴിയുന്നില്ല. ദൈവിക വെളിപാടുകള്‍ ലഭിക്കുമ്പോള്‍ അവ ഒരു മണിനാദം പോലെ തനിക്ക് അനുഭവപ്പെടാറുണ്ടെന്നും അതാണ് ഏറ്റവും പ്രയാസകരമായ വെളിപാടു രീതിയെന്നും മുഹമ്മദ്(സ) പറഞ്ഞതാണ് അദ്ദേഹത്തെ പിശാചുബാധിച്ചിരുന്നുവെന്നും പൈശാചിക വെളിപാടുകളാണ് ഖുര്‍ആനെന്നും വാദിക്കുന്നവരുടെ ഒരു തെളിവ്. വെളിപാട് ലഭിച്ചു കൊ ണ്ടിരുന്ന അതിശൈത്യമുള്ള ഒരു ദിവസം പ്രവാചകന്റെ നെറ്റിയില്‍ വിയര്‍പ്പുതുള്ളിയുണ്ടായിരുന്നതായി ഞാന്‍ കണ്ടുവെന്ന പ്രവാചകപത്‌നി ആഇശ(റ) യുടെ നിവേദനമാണ് മറ്റൊരു തെളിവ്. ഇവിടെ പ്രസക്തമായ ഒരു ചോദ്യമുണ്ട്. പിശാചുബാധിതന് ചെവിയില്‍ മണിയടിക്കുന്നതുപോലെ തോന്നുമെന്നോ അവന്റെ നെറ്റിത്തടം അതിശൈത്യമാണെങ്കിലും വിയര്‍പ്പുതുള്ളികളാല്‍ നിറയുമെന്നോ ബൈബിളില്‍ എവിടെയെങ്കിലുമുണ്ടോ? ഇല്ലെങ്കില്‍, പ്രവാചക(സ)നില്‍ പിശാചുബാധ ആരോപിക്കുവാന്‍ ബൈബിളിന്റെ അനുയായികള്‍ക്ക് എന്തടിസ്ഥാനമാണുള്ളത്?

പ്രവാചകന് ലഭിച്ച ദൈവിക സന്ദേശങ്ങള്‍ പിശാചുബാധയുടെ ഉല്‍പ ന്നങ്ങളാണെന്ന് പറയുന്നവര്‍ തങ്ങളുടെതന്നെ വിശുദ്ധന്മാരാണ് പിശാചു ബാധയേറ്റവരെന്ന് പറയാന്‍ നിര്‍ബന്ധിതരാവുമെന്നതാണ് വാസ്തവം.

യേശുവിന്റെ ജീവിതകാലമത്രയും അദ്ദേഹത്തെയും അദ്ദേഹം പഠിപ്പിച്ച ആശയങ്ങളെയും നശിപ്പിക്കുവാന്‍ വേണ്ടി അഹോരാത്രം പരിശ്രമിക്കുക യും (അപ്പോസ്തല പ്രവൃത്തികള്‍ 9:1, 26:10, 8:1) അദ്ദേഹത്തിനുശേഷം ക്രിസ്തു തനിക്ക് വെളിപ്പെട്ടിട്ടുണ്ടെന്ന് അവകാശവാദമുന്നയിക്കുകയും ചെയ്തയാളാണ് 'വിശുദ്ധ പൗലോസ്'. അദ്ദേഹത്തിന് ക്രിസ്തുദര്‍ശനം ലഭിച്ച രീതിയെക്കുറിച്ച് ബൈബിള്‍ വിവരിക്കുന്നത് കാണുക: ''പിന്നെ അയാള്‍ യാത്ര പുറപ്പെട്ട് ഡമാസ്‌കസിനെ സമീപിച്ചപ്പോള്‍, പെട്ടെന്ന് ആകാശത്തുനിന്ന് ഒരു പ്രകാശം അയാളുടെ ചുറ്റും മിന്നലൊളി പരത്തി. സാവൂള്‍ നിലം പതിച്ചു. 'സാവൂള്‍, സാവൂള്‍ നീ എന്നെ പീഡിപ്പിക്കുന്നത് എന്തിന്? എന്ന് തന്നോട് ചോദിക്കുന്ന ഒരു സ്വരം കേള്‍ക്കയായി. അപ്പോള്‍ അയാള്‍ ചോദിച്ചു: 'പ്രഭോ നീ ആരാണ്? അവന്‍ പറഞ്ഞു: നീ പീഡിപ്പിക്കുന്ന യേശുവാണ് ഞാന്‍. എഴുന്നേറ്റ് നഗരത്തില്‍ ചെല്ലുക. നീ ചെയ്യേണ്ടത് എന്തെന്ന് അവിടെ വെച്ച് നിനക്ക് അറിവ് കിട്ടും'. 'അയാളോടൊപ്പം യാത്ര ചെയ്തിരുന്ന ആളുകള്‍ സ്വരം കേട്ടെങ്കിലും ആരെയും കാണായ്കയാല്‍ വിസ്മയ സ്തബ്ധരായി നിന്നുപോയി. വീണുകിടന്നിടത്തുനിന്ന് സാവൂള്‍ എഴുന്നേറ്റു. കണ്ണുതുറന്നിട്ടും അയാള്‍ക്ക് ഒന്നും കാണാന്‍ കഴിഞ്ഞില്ല. അതിനാല്‍ അവര്‍ അയാളെ കൈക്കുപിടിച്ച് ഡമാസ്‌കസിലേക്കു കൊണ്ടുപോയി. മൂന്നു ദിവസത്തേക്ക് അയാള്‍ക്ക് കാഴ്ചയില്ലായിരുന്നു; അയാള്‍ തിന്നുകയോ കുടിക്കുകയോ ചെയ്തതുമില്ല'' (അപ്പോസ്തല പ്രവൃത്തികള്‍ 9:3-9)

നിലംപതിക്കുന്നതും കൂടെയുള്ളവര്‍ കാണാത്തത് കാണുന്നതും കേള്‍ ക്കാത്തത് കേള്‍ക്കുന്നതും കണ്ണു കാണാതാവുന്നതുമെല്ലാം പിശാചുബാധ യുടെ ലക്ഷണങ്ങളായി സുവിശേഷങ്ങളില്‍ സൂചിപ്പിക്കപ്പെട്ടിട്ടുള്ളത് നാം കണ്ടു. ക്രിസ്തുവിനെ താന്‍ കണ്ടുവെന്ന് പൗലോസ് അവകാശപ്പെട്ട സംഭവത്തില്‍ ഇതെല്ലാം അദ്ദേഹം അനുഭവിക്കുന്നുമുണ്ട്. പൗലോസിന് പിശാചുബാധയാണ് ഉണ്ടായതെന്ന് വാദിച്ചാല്‍ അത് അംഗീകരിക്കാന്‍ ക്രൈസ്തവ സമൂഹം സന്നദ്ധമാവുമോ? മുഹമ്മദി(സ)ന് പിശാചുബാധയായിരുന്നുവെന്ന് സമര്‍ഥിക്കുവാന്‍ ബൈബിളില്‍നിന്ന് ഒരു തെളിവെങ്കിലുമുദ്ധരിക്കാ ന്‍ ക്രൈസ്തവ വിമര്‍ശകര്‍ക്ക് കഴിയില്ല. അതേസമയം, നിലവിലുള്ള ക്രിസ്തുമതത്തിന്റെ സ്ഥാപകനായ പൗലോസിന് പിശാചുബാധയാണ് അനുഭ വപ്പെട്ടതെന്ന് ബൈബിള്‍ ഉപയോഗിച്ചു കൊണ്ട് സ്ഥാപിക്കാന്‍ ഒരാള്‍ക്ക് കഴിയും. അപ്പോള്‍ ആര്‍ക്കാണ് പിശാചുബാധ? ഇനി, മുഹമ്മദ് നബി(സ)ക്ക് പിശാച് ബാധിച്ചതുകൊണ്ടാണ് ഖുര്‍ആന്‍ എഴുതിയുണ്ടാക്കിയതെന്ന ക്രൈസ്തവവാദത്തിന്റെ ആണിക്കല്ല് പരിശോധിക്കുക. യേശുക്രിസ്തുവിന്റെ കുരിശുമരണത്തിലൂടെയുള്ള പാപപരിഹാരം എന്ന ആശയത്തെ വിമര്‍ശിക്കുന്നതുമൂലമാണല്ലോ ഖുര്‍ആന്‍ പിശാചിന്റെ സൃഷ്ടിയാണെന്ന് വാദിക്കുന്നത്.

എന്നാല്‍, യാഥാര്‍ഥ്യമെന്താണ്? യേശുക്രിസ്തു പരിശുദ്ധനായിരുന്നുവെന്ന് മുസ്‌ലിംകളും ക്രൈസ്തവരും വിശ്വസിക്കുന്നു. അദ്ദേഹം സര്‍വശക് തനാല്‍ നിയുക്തനായ വ്യക്തിയാണെന്ന് ഇരുകൂട്ടരും സമ്മതിക്കുന്നു. അദ്ദേഹത്തിന് പിശാചുബാധയുണ്ടായിട്ടില്ലെന്ന് ഇരുകക്ഷികളും പറയുന്നു. എങ്കില്‍, മുഹമ്മദ് നബി(സ)ക്കോ പൗലോസിനോ ആര്‍ക്കാണ് പിശാചില്‍നിന്ന് വെളിപാടുണ്ടായതെന്ന് പരിശോധിക്കാന്‍ നമുക്കെന്തുകൊണ്ട് യേശുക്രിസ്തുവിന്റെ ഉപദേശങ്ങളുമായി അവരുടെ ഉപദേശങ്ങളെ താരതമ്യം ചെയ്തുകൂടാ? പിശാചില്‍നിന്ന് വെളിപാടുണ്ടായ വ്യക്തി യേശുവിന്റെ ശത്രുവായിരിക്കുമല്ലോ. ഒരു ദൈവദൂതന്റെ ശത്രു അയാള്‍ പ്രബോധനം ചെയ്യുന്ന ആശയങ്ങളുടെ ശത്രുവായിരിക്കും എന്നോര്‍ക്കുക.

യേശു പറഞ്ഞു: നിയമത്തെ (തോറ)യോ പ്രവാചകന്മാരെയോ റദ്ദാക്കാനല്ല ഞാന്‍ വന്നത്' (മത്തായി 5:17). ഖുര്‍ആന്‍ പറയുന്നു: 'തീര്‍ച്ചയായും നാം തന്നെയാണ് തൗറാത്ത് അവ തരിപ്പിച്ചിരിക്കുന്നത്, അതില്‍ മാര്‍ഗദര്‍ശനവും പ്രകാശവുമുണ്ട്' (5:44).

'മര്‍യമിന്റെ മകന്‍ ഈസ പറഞ്ഞ സന്ദര്‍ഭം: ഇസ്രായേല്‍ സന്തതികളേ, എനിക്കുമുമ്പുള്ള തൗറാത്തിനെ സത്യപ്പെടുത്തുന്നവനായിക്കൊണ്ടും എനിക്ക് ശേഷം വരുന്ന അഹ്മദ് എന്നു പേരുള്ളൊരു ദൂതനെപ്പറ്റി സന്തോ ഷവാര്‍ത്ത അറിയിക്കുന്നവനായിക്കൊണ്ടും നിങ്ങളിലേക്ക് അല്ലാഹുവിന്റെ ദൂതനായി നിയോഗിക്കപ്പെട്ടവനാകുന്നു ഞാന്‍' (61:6).

പൗലോസ് എഴുതി: 'നിയമാനുഷ്ഠാനങ്ങളെ (തോറ) ആശ്രയിക്കുന്നവ രെല്ലാം ശാപഗ്രസ്തരാണ് (ഗലാത്തിയക്കാര്‍ 3:10).

'ക്രിസ്തു നിയമത്തിന്റെ ശാപത്തില്‍നിന്നു നമ്മെ മോചിപ്പിച്ചിരിക്കു ന്നു' (ഗലാത്തിയക്കാര്‍ 3:13). 'അവന്‍ (യേശു) തന്റെ ശരീരത്തില്‍, നിയമത്തെ അതിന്റെ കല്‍പന കളോടും അനുശാസനങ്ങളോടുംകൂടി റദ്ദാക്കി' (എഫേസോസുകാര്‍2:15)

ഞാന്‍ നിയമത്തെ റദ്ദാക്കാനല്ല വന്നതെന്ന് യേശു, ഖുര്‍ആനും അതുതന്നെ പറയുന്നു. പൗലോസാകട്ടെ യേശു നിയമത്തില്‍നിന്ന് ലോകത്തെ രക്ഷിക്കാനാണ് വന്നത് എന്നു സമര്‍ഥിക്കുന്നു. ആര്‍ക്കാണ് പിശാചിന്റെ വെളിപാട്?

യേശുക്രിസ്തു താന്‍ ദൈവമാണെന്ന് പഠിപ്പിച്ചില്ല (മര്‍ക്കോസ് 12:29, മത്തായി 4:10) ഇക്കാര്യം ഖുര്‍ആന്‍ അര്‍ഥശങ്കക്കിടയില്ലാത്തവണ്ണം വ്യക്ത മാക്കുന്നു (3:51), എന്നാല്‍ പൗലോസ് പറഞ്ഞതാകട്ടെ 'പ്രകൃത്യാതന്നെ ദൈവമായിരുന്നിട്ടും ദൈവത്തോടു തനിക്കുള്ള തുല്യതയെ, മുറുകെപ്പിടിച്ചുകൊണ്ടിരിക്കേണ്ട ഒരു കാര്യമായി അവന്‍ പരിഗണിച്ചില്ല? (ഫിലിപ്പിയര്‍ 2:6). 'അവന്‍ അദൃശ്യനായ ദൈവത്തിന്റെ പ്രതിരൂപമാണ്; സര്‍വസൃഷ്ടികളിലും ആദ്യജാതന്‍' (കൊളോസിയക്കാര്‍ 1:15) എന്നിങ്ങനെയാണ്. യേശുക്രിസ്തുവിന് സ്വയം താന്‍ ദൈവമാണെന്ന വെളിപാട് ലഭിച്ചിട്ടില്ല. അങ്ങനെ ലഭിച്ചിരുന്നുവെങ്കില്‍ അദ്ദേഹം അത് പറയുമായിരുന്നു. എന്നാല്‍, പൗലോസിന് യേശു ദൈവമായിരുന്നുവെന്ന് വെളിപാട് കിട്ടി. പ്രസ്തുത വെളിപാട് എവിടെനിന്നായിരിക്കണം?

അബ്രഹാമിനോട് ദൈവം ചെയ്ത ഉടമ്പടിയായിട്ടാണ് പരിച്‌ഛേദനാകര്‍മത്തെ ബൈബിള്‍ പരിചയപ്പെടുത്തുന്നത്. 'നീയും നിനക്കു ശേഷം തലമുറയായി നിന്റെ സന്തതികളും പാലിക്കേണ്ട ഉടമ്പടി'യെന്നു പറഞ്ഞുകൊണ്ടാണ് അബ്രഹാമിനോട് കര്‍ത്താവ് പരിച്‌ഛേദന ചെയ്യുന്നതിനുള്ള കല്‍പന നല്‍കുന്നത് (ഉല്‍പത്തി 17:9-14) കര്‍ത്താവ് മോശയോടു പറഞ്ഞ തായി ബൈബിള്‍ ഉദ്ധരിക്കുന്നു: 'എട്ടാം ദിവസം ശിശുവിന്റെ പരിച്‌ഛേ ദനം നടത്തണം (ലേവിയര്‍ 12:3) ഈ ദൈവിക കല്‍പന യേശുവും അനുസ രിച്ചിരുന്നു. 'എട്ടു ദിവസം പൂര്‍ത്തിയായപ്പോള്‍ ശിശുവിന് പരിച്‌ഛേദനം നടത്തി' (ലൂക്കോസ് 2:21). പരിച്‌ഛേദനം ചെയ്യേണ്ടതില്ലെന്ന് യേശു ആരോടും പറഞ്ഞില്ല. കാരണം അദ്ദേഹത്തിന് അത്തരത്തിലുള്ള ഒരു ബോധനം ലഭിച്ചിരുന്നില്ല. എന്നാല്‍ പൗലോസ് പറയുന്നത് കാണുക: 'പരിച്‌ഛേദനം സ്വീകരിക്കുന്നുവെങ്കില്‍ നിങ്ങള്‍ക്ക് ക്രിസ്തുവിനെക്കൊണ്ട് നേട്ടമില്ല' (ഗലാത്തിയക്കാര്‍ 5:2). ഈ വെളിപാട് പൗലോസിന് എവിടെനിന്ന് കിട്ടി? ദൈവത്തില്‍ നിന്നാകാന്‍ വഴിയില്ല. പിന്നെയോ?

പിശാചില്‍നിന്നാണ് മുഹമ്മദി(ല)ന് വെളിപാടുണ്ടായത് എന്നുപറയാ നുള്ള പ്രധാനപ്പെട്ട കാരണം കുരിശുമരണത്തെയും പാപപരിഹാരബലി യെയും ഖുര്‍ആന്‍ നിഷേധിക്കുന്നുവെന്നതാണല്ലോ. യേശുവിനെയും മാതാവിനെയും പുകഴ്ത്തുകയും ആദരിക്കുകയും ചെയ്യുന്ന ഒട്ടനവധി സൂക്ത ങ്ങള്‍ ഖുര്‍ആനിലുണ്ട്. ഖുര്‍ആനില്‍ പേരു പരാമര്‍ശിക്കപ്പെട്ട ഏക വനിത മര്‍യമാണെന്നോര്‍ക്കുക. യേശു ചെയ്തതായി ബൈബിളില്‍ പറയാത്ത കളിമണ്‍പക്ഷികളില്‍ ഊതി അവയ്ക്ക് ജീവനിടുക തുടങ്ങിയ അത്ഭുതങ്ങ ളെക്കുറിച്ച് ഖുര്‍ആന്‍ പ്രതിപാദിക്കുന്നുമുണ്ട് (3:49). തൊട്ടിലില്‍ വെച്ച് ഉണ്ണിയേശു സംസാരിച്ചതായുള്ള ഖുര്‍ആനിക പരാമര്‍ശം (19:30) ബൈബി ളിലൊരിടത്തും കാണുവാന്‍ സാധ്യമല്ല. യേശുവിന്റെ വിശുദ്ധ വ്യക്തിത്വത്തില്‍ കളങ്കമുണ്ടാക്കുന്ന യാതൊന്നും ഖുര്‍ആനിലില്ല. യോഹന്നാന്റെ സുവിശേഷ പ്രകാരം ക്രിസ്തുവിന്റെ ആദ്യത്തെ അത്ഭുതം കാനായിലെ കല്യാണവിരുന്നില്‍ വെച്ച് മദ്യം നിര്‍മിച്ചു നല്‍കിയതാണെന്ന കാര്യം പ്രത്യേകം പ്രസ്താവ്യമാണ് (യോഹന്നാന്‍ 2:1-11). ഖുര്‍ആനില്‍ ഇത്തരം യാതൊരു പരാമര്‍ശവുമില്ല.

'മരത്തില്‍ തൂക്കിക്കൊല്ലപ്പെടുന്നവന്‍ ദൈവത്താല്‍ ശപിക്കപ്പെട്ടവനാണ്' (ആവര്‍ത്തനം 21:23)എന്നാണ് ബൈബിളിന്റെ സിദ്ധാന്തം. കുരിശില്‍ തറക്കുക വഴി യേശുവിനെ ശപിക്കപ്പെട്ടവനായി മുദ്രയടിക്കുകയാണ് തങ്ങ ള്‍ ചെയ്തതെന്നാണ് യഹൂദര്‍ കരുതിയത്. പൗലോസ് പറയുന്നതും മറ്റൊന്നല്ല. 'മരത്തില്‍ തൂക്കപ്പെടുന്നവരെല്ലാം ശപിക്കപ്പെട്ടവര്‍ എന്ന് എഴുതിയിരിക്കുന്നതുപോലെ ക്രിസ്തു നമുക്കുവേണ്ടി ശാപമായിത്തീരുന്നു' (ഗലാത്യര്‍ 3:13). അപ്പോള്‍ ക്രൂശീകരണം യേശുവിനെ ശപിക്കപ്പെട്ടവനാക്കുകയാണ് ചെയ്യുന്നത്. ലോകത്തിനുവേണ്ടി യേശു ശാപമായിത്തീര്‍ന്നുവെന്ന വാദം ഖുര്‍ആന്‍ അംഗീകരിക്കുന്നില്ല. ശാപത്തിന്റെ മരക്കുരിശില്‍നിന്ന് തന്നെ രക്ഷിക്കേണമേയെന്ന ക്രിസ്തുവിന്റെ പ്രാര്‍ഥന (മത്തായി 26:39) ദൈവം കേട്ടില്ലെന്നു കരുതുന്നത് ദൈവിക കാരുണ്യത്തിന്റെ നിഷേധമല്ലാതെ മറ്റെന്താണ്? ശപിക്കപ്പെട്ട മരക്കുരിശില്‍നിന്ന് പടച്ചതമ്പുരാന്‍ യേശുവിനെ രക്ഷിച്ചുകൊണ്ട് യഹൂദന്മാരുടെ ഗൂഢാലോചനയെ തകര്‍ക്കുകയാണ് ചെയ്തത് എന്നാണ് ഖുര്‍ആന്‍ പഠിപ്പിക്കുന്നത് (4:157,158).

മരക്കുരിശില്‍ ക്രൂശിക്കുക വഴി യേശുവിനെ ശപിക്കപ്പെട്ടവനാക്കിയെന്ന് യഹൂദന്മാര്‍. മരക്കുരിശില്‍ മരിച്ച് യേശു ശപിക്കപ്പെട്ടവനായിത്തീര്‍ന്നുവെന്ന് പൗ ലോസ്. മരക്കുരിശില്‍നിന്ന് പരിശുദ്ധനായ യേശുവിനെ ദൈവം രക്ഷിച്ചുവെന്ന് ഖുര്‍ആന്‍.

ഏതാണ് പിശാചിന്റെ വെളിപാട്? യേശുവിനെ മഹത്വപ്പെടുത്തുന്നതോ അതല്ല ശാപഗ്രസ്തനാക്കുന്നതോ? ചുരുക്കത്തില്‍, ഖുര്‍ആന്‍ പൈശാചിക വെളിപാടാണെന്ന് സമര്‍ഥിക്കു വാന്‍ വേണ്ടി തെളിവുകള്‍ പരതുന്നവര്‍ കുഴിക്കുന്ന കുഴികളില്‍ തങ്ങള്‍തന്നെയാണ് വീഴുന്നത് എന്നുള്ളതാണ് യാഥാര്‍ഥ്യം.

യുക്തിവാദികളായ വിമര്‍ശകന്മാര്‍ പ്രധാനമായും ഉന്നയിക്കുന്ന ആരോ പണമാണ് മുഹമ്മദ്(സ) നബിക്ക് ഉന്മാദരോഗ (Schizophrenia) മായിരുന്നുവെന്നത്. ദൈവത്തിന്റെ അസ്തിത്വം അംഗീകരിക്കാത്തവരെ സംബന്ധിച്ചിടത്തോളം അവര്‍ക്ക് വെളിപാടുകളുടെ സത്യതയെക്കുറിച്ച് എത്രതന്നെ പറഞ്ഞാലും ഉള്‍ക്കൊള്ളാന്‍ കഴിയില്ല. അതുകൊണ്ടുതന്നെ നിരീശ്വരവാദികളോടുള്ള ചര്‍ച്ച തുടങ്ങേണ്ടത് ദൈവാസ്തിത്വത്തെക്കുറിച്ച് സംസാരിച്ചുകൊണ്ടാണ്. പടച്ചതമ്പുരാന്റെ അസ്തിത്വംതന്നെ അംഗീകരിക്കാത്തവരെ അവനില്‍നിന്നുള്ള വെളിപാടുകള്‍ സത്യസന്ധമാണെന്ന് സമ്മതിപ്പിക്കുന്ന തെങ്ങനെ?

ചോദ്യത്തിന്റെ രണ്ടാം ഭാഗമാണ് ആദ്യമായി ചര്‍ച്ച ചെയ്യപ്പെടേണ്ടത്. സമകാലികരാല്‍ മുഹമ്മദ് (സ) ഭ്രാന്തനെന്ന് വിളിക്കപ്പെട്ടിരുന്നുവോ? ഉണ്ടെ ങ്കില്‍ ഭ്രാന്തിന്റെ എന്തെല്ലാം ലക്ഷണങ്ങളുടെ അടിസ്ഥാനത്തിലാണ് അവര്‍ ഈ ആരോപണം ഉന്നയിച്ചത്?

നാല്‍പതു വയസ്സുവരെ സത്യസന്ധനും സര്‍വരാലും അംഗീകരിക്ക പ്പെട്ട വ്യക്തിത്വത്തിന്റെ ഉടമയുമായിരുന്നു മുഹമ്മദ് (സ) . സുദീര്‍ഘമായ ഈ കാലഘട്ടത്തിനിടയ്ക്ക് ആരെങ്കിലും അദ്ദേഹത്തില്‍ ഏതെങ്കിലും തരത്തിലുള്ള മാനസിക വിഭ്രാന്തി ആരോപിച്ചിട്ടില്ല.

പ്രവാചകത്വത്തിനുശേഷം അദ്ദേഹം ഭ്രാന്തനെന്ന് ആരോപിക്കപ്പെട്ടിരുന്നുവെന്നത് ശരിയാണ്. ഭ്രാന്ത നെന്ന് മാത്രമല്ല മുഹമ്മദ്(സ) അധിക്ഷേപിക്കപ്പെട്ടത്; ജ്യോല്‍സ്യന്‍, മാരണ ക്കാരന്‍, മാരണം ബാധിച്ചവന്‍, കവി എന്നിങ്ങനെയുള്ള അധിക്ഷേപങ്ങളെ ല്ലാം അദ്ദേഹത്തിനുനേരെ ഉന്നയിക്കപ്പെട്ടിരുന്നു. അദ്ദേഹത്തിന്റെ വ്യക്തിത്വത്തിലോ മാനസിക സംതുലനത്തിലോ വല്ല വ്യത്യാസവും പ്രകടമായതുകൊണ്ടാണോ അവര്‍ അങ്ങനെ അധിക്ഷേപിച്ചത്? ആണെന്ന് അവരാരുംതന്നെ വാദിച്ചിട്ടില്ല. അവരുടെ പ്രശ്‌നം ഖുര്‍ആനും അതുള്‍ക്കൊള്ളുന്ന ആശയങ്ങളുമായിരുന്നു. തങ്ങളുടെ പാരമ്പര്യ വിശ്വാസങ്ങള്‍ക്കെതിരെയാണ് മുഹമ്മദ്(സ) സംസാരിക്കുന്നത്. അദ്ദേഹം ദൈവികമാണെന്ന് പറഞ്ഞുകൊണ്ട് ഓതിക്കേള്‍പ്പിക്കുന്ന ഖുര്‍ആനിലേക്ക് ജനങ്ങള്‍ ആകൃഷ്ടരാവുക യും ചെയ്യുന്നു. മുഹമ്മദി(സ)നെ സ്വഭാവഹത്യ നടത്താതെ ജനങ്ങളെ അദ്ദേഹത്തില്‍നിന്ന് അകറ്റാന്‍ മറ്റു മാര്‍ഗങ്ങളൊന്നുമില്ലെന്ന് കണ്ട പാരമ്പ ര്യമതത്തിന്റെ കാവല്‍ക്കാര്‍ ബോധപൂര്‍വം കെട്ടിച്ചമച്ച സ്വഭാവഹത്യയായിരുന്നു ഇവയെല്ലാം.

മുഹമ്മദ്(സ) പ്രവാചകത്വം പരസ്യമായി പ്രഖ്യാപിച്ചകാലം. ഹജ്ജ് മാസം ആസന്നമായി. അറേബ്യയുടെ വിവിധ ഭാഗങ്ങളില്‍നിന്നും ഹജ്ജിനു വരുന്നവരോട് മുഹമ്മദ് (സ) മതപ്രബോധനം നടത്തുമെന്നും ഖുര്‍ആനിന്റെ വശ്യതയില്‍ അവര്‍ ആകൃഷ്ടരാവുമെന്നും മക്കയിലെ പ്രമാണിമാര്‍ ഭയന്നു. അവര്‍ യോഗം ചേര്‍ന്നു. ഹജ്ജിന് എത്തിച്ചേരുന്നവരോട് ആദ്യമേതന്നെ മുഹമ്മദി(സ)നെതിരെ പ്രചാരവേലകള്‍ നടത്താന്‍ തീരുമാനിച്ചു. മുഹമ്മദി(സ)നെ എങ്ങനെ വിശേഷിപ്പിക്കണം, എന്നതായി പിന്നീടുള്ള ചര്‍ച്ച. പലരും പല രൂപത്തില്‍ പറയുന്നത് തങ്ങളുടെ വിശ്വാസ്യത തകര്‍ക്കും. എല്ലാവര്‍ക്കും ഒരേ രൂപത്തില്‍ പറയാന്‍ പറ്റുന്ന ആരോപണമെന്ത്? ചിലര്‍ പറഞ്ഞു: ''നമുക്ക് മുഹമ്മദ് ഒരു ജ്യോല്‍സ്യനാണെന്ന് പറയാം''. പൗരപ്രമുഖനായ വലീദുബ്‌നുമുഗീറ പറഞ്ഞു: ''പറ്റില്ല, അല്ലാഹുവാണ് സത്യം അവ ന്‍ ജ്യോല്‍സ്യനല്ല. ജ്യോല്‍സ്യന്മാരെ നാം കണ്ടിട്ടുണ്ട്. മുഹമ്മദിന്റെ വാക്കുകള്‍ ജ്യോല്‍സ്യന്മാരുടെ പ്രവചനങ്ങളല്ല''. മറ്റു ചിലര്‍ പറഞ്ഞു: ''നമുക്ക് അവന്‍ ഭ്രാന്തനാണെന്ന് പറയാം''. വലീദ് പറഞ്ഞു: ''അവന്‍ ഭ്രാന്തനല്ല. ഭ്രാന്തന്മാരെ നാം കണ്ടിട്ടുണ്ട്. അവരുടെ ഭ്രാന്തമായ സംസാരങ്ങളോ ഗോഷ്ഠികളോ പിശാചുബാധയോ ഒന്നും അവനില്ല''. അവര്‍ പറഞ്ഞു: ''എങ്കില്‍ അവന്‍ കവിയാണെന്ന് പറയാം''. വലീദ് പ്രതിവചിച്ചു: ''അവന്‍ കവിയല്ല. കവിതയുടെ എല്ലാ ഇനങ്ങളും നമുക്കറിയാം. അവന്‍ പറയുന്ന ത് കവിതയല്ല''. ജനം പറഞ്ഞു: ''എങ്കില്‍ അവന്‍ മാരണക്കാരനാണെന്ന് പറയാം' വലീദ് പ്രതികരിച്ചു: ''അവന്‍ മാരണക്കാരനുമല്ല. മാരണക്കാരെ നമുക്കറിയാം. അവരുടെ കെട്ടുകളോ, ഊത്തുകളോ ഒന്നും അവന്‍ പ്രയോഗിക്കുന്നില്ല''. അവര്‍ ചോദിച്ചു: ''പിന്നെ എന്താണ് നിങ്ങളുടെ നിര്‍ദേശം?'' അദ്ദേഹം പറഞ്ഞു: ''തീര്‍ച്ചയായും അവന്റെ വചനങ്ങളില്‍ മാധുര്യമുണ്ട്. അതിന്റെ മൂലം വിസ്തൃതവും ശാഖകള്‍ ഫലസമൃദ്ധവുമാണ്. നിങ്ങള്‍ അവനെപ്പറ്റി എന്തു പറഞ്ഞാലും അതു നിരര്‍ഥകമാണെന്നു തെളിയും. പിതാവിനും മക്കള്‍ക്കുമിടയിലും ഭാര്യക്കും ഭര്‍ത്താവിനുമിടയിലും ജ്യേഷ്ഠനും അനുജനുമിടയിലും പിളര്‍പ്പുണ്ടാക്കുവാന്‍ വേണ്ടി വന്ന ജാലവിദ്യക്കാരനാണ് അവനെന്ന് പറയുന്നതാണ് നല്ലത്!'' ജനം ഇതംഗീകരിച്ചു. അവര്‍ പ്രചാര ണം തുടങ്ങി.

ഈ സംഭവം മനസ്സിലാക്കിത്തരുന്ന വസ്തുതയെന്താണ്? പ്രവാചകപ്ര ബോധനങ്ങളില്‍ നിന്ന് ജനങ്ങളെ പിന്തിരിപ്പിക്കുവാന്‍ വേണ്ടി ശത്രുക്കള്‍ മെനഞ്ഞെടുത്ത പലതരം ദുഷ്പ്രചാരണങ്ങളിലൊന്നു മാത്രമാണ് അദ്ദേ ഹം ഭ്രാന്തനാണെന്ന ആരോപണം. ഈ പ്രചാരണം നടത്തിയിരുന്നവര്‍ക്കു തന്നെ അതില്‍ വിശ്വാസമുണ്ടായിരുന്നില്ല. അതുകൊണ്ടുതന്നെ, അവരുടെ പ്രചാരണത്തെ ഒരു തെളിവായി സ്വീകരിക്കുന്നത് അബദ്ധമാണ്.

പ്രവാചകന്‍ ജീവിച്ചത് പതിനാലു നൂറ്റാണ്ടുകള്‍ക്കുമുമ്പാണ്. അദ്ദേഹ ത്തിന് ഉന്മാദരോഗമുണ്ടായിരുന്നുവോയെന്ന് വസ്തുനിഷ്ഠമായി പരിശോധിക്കുവാന്‍ ഇപ്പോള്‍ അദ്ദേഹം നമ്മുടെ മുന്നില്‍ ജീവിച്ചിരിക്കാത്തതിനാല്‍ ഇന്ന് നമുക്ക് കഴിയില്ല. അദ്ദേഹത്തിനുണ്ടായ വെളിപാടുകളും സ്വപ്‌നദര്‍ശ നങ്ങളുമാണ് പ്രവാചകന്‍ ഉന്മാദരോഗിയായിരുന്നുവെന്ന് വാദിക്കുന്നവര്‍ക്കുള്ള തെളിവ്. വെളിപാടുകള്‍ സ്വീകരിക്കുമ്പോള്‍ പ്രവാചകനില്‍ കാണ പ്പെട്ട ഭാവവ്യത്യാസങ്ങളെയും വഹ്‌യ് എങ്ങനെയാണെന്നുള്ള പ്രവാചക ന്റെ വിവരങ്ങളെയും വിശദീകരിക്കുന്ന ഹദീഥുകളുടെ വെളിച്ചത്തിലാണ് വിമര്‍ശകന്മാര്‍ ഈ വാദമുന്നയിക്കുന്നത്. ഉന്മാദരോഗത്തിന്റെ ലക്ഷണ ങ്ങള്‍ പ്രവാചകനില്‍ കാണപ്പെട്ടിരുന്നുവോയെന്ന് വസ്തുനിഷ്ഠമായി പരി ശോധിച്ചാല്‍ ഈ വാദത്തില്‍ യാതൊരു കഴമ്പുമില്ലെന്ന് സുതരാം വ്യക് തമാവും.

ഒന്ന്: ഉന്മാദരോഗികളുടെ സ്വഭാവം നിരന്തരം മാറിക്കൊണ്ടിരിക്കും. മറ്റുള്ളവരോടുള്ള പെരുമാറ്റത്തിലും സംസാരത്തിലുമെല്ലാം ഈ വൈരുധ്യം പ്രകടമായിരിക്കും.

മുഹമ്മദി(സ)ന്റെ ജീവിതവും സംസാരങ്ങളും പരിശോധിക്കുക.

യാതൊരു രീതിയിലുള്ള സ്വഭാവ വൈരുദ്ധ്യങ്ങളും അദ്ദേഹത്തില്‍ നമുക്ക് കാണാന്‍ കഴിയില്ല. മാറിക്കൊണ്ടിരിക്കുന്ന പെരുമാറ്റ രീതികളുടെയും പൂര്‍വാപരബന്ധമില്ലാത്ത സംസാരത്തിന്റെയും ഉടമസ്ഥനായിരുന്നു മുഹമ്മദ് നബി(ല)യെങ്കില്‍ അദ്ദേഹത്തിന് പരശ്ശതം അനുയായികളുണ്ടായതെങ്ങ നെ? സാധാരണയായി നാം മനസ്സിലാക്കുന്ന 'ദിവ്യന്‍'മാരുടെ അനുയായികളെപ്പോലെയായിരുന്നില്ല മുഹമ്മദി(സ)ന്റെ അനുചരന്മാര്‍. അദ്ദേഹത്തിന്റെ ഉപദേശങ്ങള്‍ പ്രാവര്‍ത്തികമാക്കുന്നതിനുവേണ്ടി മല്‍സരിക്കുകയായിരുന്നു അവര്‍. ഒരു ഉന്മാദരോഗിയുടെ വാക്കുകള്‍ അനുസരിക്കുവാന്‍ വേണ്ടി ജനസഹസ്രങ്ങള്‍ മല്‍സരിച്ചുവെന്ന് പറഞ്ഞാല്‍ അത് വിശ്വസിക്കാനാവുമോ?

രണ്ട്: ഉന്മാദരോഗികളുടെ പ്രതികരണങ്ങള്‍ വൈരുധ്യാത്മകമായിരിക്കും. സന്തോഷവേളയില്‍ പൊട്ടിക്കരയുകയും സന്താപവേളയില്‍ പൊട്ടിച്ചിരിക്കുകയും ചെയ്യും. വെറുതെ ചിരിക്കുകയും കരയുകയും ചെയ്യുന്ന സ്വഭാവവും കണ്ടുവരാറുണ്ട്.

മുഹമ്മദ് നബി(സ) യുടെ പ്രതികരണങ്ങള്‍ സമചിത്തതയോടുകൂടിയുള്ളതായിരുന്നു. ഒരു സംഭവം: പ്രവാചകന്‍(സ)ഒരു മരത്തണലില്‍ വിശ്രമിക്കുകയാണ്. പെട്ടെന്ന് ഊരിപ്പിടിച്ച വാളുമായി മുന്നില്‍ ഒരു കാട്ടാളന്‍ പ്രത്യക്ഷപ്പെ ട്ടു. അയാള്‍ ചോദിച്ചു: ''എന്നില്‍നിന്ന് നിന്നെ ഇപ്പോള്‍ ആര് രക്ഷിക്കും?'' പ്രവാചകന്‍ അക്ഷോഭ്യനായി മറുപടി പറഞ്ഞു: 'അല്ലാഹു'. ഈ മറുപടിയു ടെ ദൃഢത കേട്ട് കാട്ടാളന്റെ കൈയില്‍നിന്ന് വാള്‍ വീണുപോയി. (ബുഖാരി, മുസ്‌ലിം)

ഒരു ഉന്മാദരോഗിയില്‍നിന്ന് ദൃഢചിത്തതയോടുകൂടിയുള്ള ഇത്തരം പെരുമാറ്റങ്ങള്‍ പ്രതീക്ഷിക്കുവാന്‍ കഴിയുമോ?

മൂന്ന്: ഉന്മാദരോഗികള്‍ അന്തര്‍മുഖരായിരിക്കും. പുറമെയുള്ള ലോക ത്ത് നടക്കുന്ന സംഭവങ്ങളിലൊന്നും അവര്‍ക്ക് യാതൊരു താല്‍പര്യവും കാണുകയില്ല.

മുഹമ്മദ് നബി(സ)അന്തര്‍മുഖനായിരുന്നില്ല. തന്റെ ചുറ്റുപാടും നടക്കുന്ന സംഭവങ്ങള്‍ അതീവ താല്‍പര്യത്തോടെ നിരീക്ഷിക്കുകയും തന്റെ പങ്ക് ആവശ്യമെങ്കില്‍ നിര്‍വഹിക്കുകയും ചെയ്തിരുന്ന വ്യക്തിയാണദ്ദേ ഹം. ജനങ്ങള്‍ക്ക് ധാര്‍മിക നിര്‍ദേശങ്ങള്‍ നല്‍കുക മാത്രമല്ല, അവര്‍ക്ക് മാതൃകയായി ജീവിച്ച് കാണിച്ചുകൊടുക്കുകകൂടി ചെയ്ത വ്യക്തിയായിരുന്നു അദ്ദേഹം.

ലാമാര്‍ട്ടിന്‍ എഴുതി: 'തത്ത്വജ്ഞാനി, പ്രസംഗകന്‍, ദൈവദൂതന്‍, നിയമ നിര്‍മാതാവ്, പോരാളി, ആശയങ്ങളുടെ ജേതാവ്, അബദ്ധ സങ്കല്‍പങ്ങളില്‍നിന്ന് മുക്തമായ ആചാര വിശേഷങ്ങളുടെയും യുക്തിബന്ധുരമായ വിശ്വാസപ്രമാണങ്ങളുടെയും പുനഃസ്ഥാപകന്‍, ഇരുപത് ഭൗതിക സാമ്രാജ്യങ്ങളുടെ സ്ഥാപകന്‍ -അതായിരുന്നു മുഹമ്മദ്. മനുഷ്യത്വത്തിന്റെ എല്ലാ മാനദണ്ഡങ്ങളും വെച്ച് പരിഗണിക്കുമ്പോള്‍ നാം വ്യക്തമായും ചോദിച്ചേക്കാം. മുഹമ്മദിനേക്കാള്‍ മഹാനായ മറ്റു വല്ല മനുഷ്യനുമുണ്ടോ?''(Historie De La turquie., Vol, 2 Page 277)

അന്തര്‍മുഖനായ ഒരു ഉന്മാദരോഗിയെക്കുറിച്ച വിലയിരുത്തലാണോ ഇത്?

നാല്: ഉന്മാദരോഗികള്‍ക്ക് നിര്‍ണിതമായ എന്തെങ്കിലും ലക്ഷ്യത്തിനുവേണ്ടി വ്യവസ്ഥാപിതമായി പ്രവര്‍ത്തിക്കാന്‍ കഴിയില്ല. കാര്യമായി യാതൊന്നും ചെയ്യാനാവാത്ത ഇവര്‍ ശാരീരികമായും മാനസികമായും തളര്‍ന്നവരായിരിക്കും.

മുഹമ്മദ് നബി(സ) ജനങ്ങളെ സത്യമാര്‍ഗത്തിലേക്ക് നയിക്കുന്നതിനുവേണ്ടി അയക്കപ്പെട്ട ദൈവദൂതന്മാരില്‍ അന്തിമനായിരുന്നു. തന്നിലേല്‍പിക്കപ്പെട്ട ഉത്തരവാദിത്തം രണ്ടു ദശാബ്ദത്തിലധികം ഭംഗിയായി നിര്‍വഹിക്കാന്‍ അദ്ദേഹത്തിന് കഴിഞ്ഞു. ചിട്ടയോടുകൂടിയുള്ള പ്രബോധന പ്രവര്‍ ത്തനങ്ങള്‍ വഴി ജനസഹസ്രങ്ങളെ ദൈവികമതത്തിലേക്ക് ആകര്‍ഷിക്കുവാന്‍ മുഹമ്മദി(സ)ന് സാധിച്ചു. സാംസ്‌കാരിക രംഗത്ത് വട്ടപ്പൂജ്യത്തിലായിരുന്ന ഒരു ജനവിഭാഗത്തെ ലോകത്തിന് മുഴുവന്‍ മാതൃകയാക്കി പരിവര്‍ത്തിപ്പിക്കുവാന്‍ വേണ്ടിവന്നത് കേവലം ഇരുപത്തിമൂന്ന് വര്‍ഷങ്ങള്‍ മാത്രം. ലോകചരിത്രത്തെ ഏറ്റവുമധികം സ്വാധീനിച്ച വ്യക്തിയാണ് മുഹമ്മദ്(സ) എന്ന് ചരിത്രത്തെ നിഷ്പക്ഷമായി നോക്കിക്കണ്ടവരെല്ലാം അഭിപ്രായപ്പെട്ടിട്ടുണ്ട്.

ഇതെല്ലാം ഒരു ഉന്മാദരോഗിക്ക് കഴിയുന്നതാണെന്ന് പ്രസ്തുത രോഗ ത്തെക്കുറിച്ച് അല്‍പമെങ്കിലും അറിയുന്നവരാരെങ്കിലും സമ്മതിക്കുമോ?

അഞ്ച്: ഉന്മാദരോഗി അശരീരികള്‍ കേള്‍ക്കുകയും(Auditory Hallucination) മിഥ്യാഭ്രമത്തിലായിരിക്കുകയും (Delusion)  മായാദൃശ്യങ്ങള്‍ കാണുക യും(Hallucination) ചെയ്യും. ഈ അശരീരികളും മായാദൃശ്യങ്ങളും യാഥാര്‍ഥ്യവുമായി യാതൊരു ബന്ധവുമുള്ളതായിരിക്കില്ല.

മുഹമ്മദ് നബി(സ)ക്കുണ്ടായ വെളിപാടുകളും ദര്‍ശനങ്ങളും ഈ ഗണത്തില്‍ പെടുത്തിക്കൊണ്ടാണ് വിമര്‍ശകര്‍ അദ്ദേഹത്തില്‍ ഉന്മാദരോഗം ആരോപിക്കുന്നത്. ഉന്മാദരോഗത്തിന്റെ മറ്റു ലക്ഷണങ്ങളൊന്നും നബി(സ)യില്‍ ഉണ്ടായിരുന്നില്ലെന്ന് നാം മനസ്സിലാക്കി. അപ്പോള്‍ ഈ വെളിപാടുകളുടെ മാത്രം വെളിച്ചത്തില്‍ അദ്ദേഹം ഉന്മാദരോഗിയാണെന്ന് പറയുന്ന തെങ്ങനെ? ഉന്മാദരോഗിക്കുണ്ടാവുന്ന 'വെളിപാടു'കള്‍ അയാളുടെ രോഗത്തിന്റെ ലക്ഷണമാണ്. ഈ വെളിപാടുകള്‍ അയാളുടെ വൈയക്തിക മേഖലകളുമായി മാത്രം ബന്ധപ്പെട്ടതായിരിക്കും. എന്നാല്‍, മുഹമ്മദി(സ)നുണ്ടായ വെളിപാടുകളോ? ആ വെളിപാടുകള്‍ ഒരു ഉത്തമ സമൂഹത്തെ പടിപടിയായി വാര്‍ത്തെടുക്കുകയായിരുന്നു. ആദ്യം ദൈവബോധവും പര ലോകചിന്തയും ജനങ്ങളില്‍ വളര്‍ത്തി. ഘട്ടം ഘട്ടമായി സമൂഹത്തെ മുച്ചൂടും ബാധിച്ചിരുന്ന എല്ലാ തിന്മകളുടെയും അടിവേരറുത്തു. അങ്ങനെ ഒരു മാതൃകാ സമൂഹത്തിന്റെ സൃഷ്ടിക്ക് നിമിത്തമാകുവാന്‍ മുഹമ്മദി(സ)ന് ലഭിച്ച വെളിപാടുകള്‍ക്ക് കഴിഞ്ഞു. അത് സൃഷ്ടിച്ച വിപ്ലവം മഹത്തരമാണ്. ചരിത്രകാലത്ത് അതിനു തുല്യമായ മറ്റൊരു വിപ്ലവം നടന്നിട്ടില്ല.

ഉന്മാദരോഗി കേള്‍ക്കുന്ന അശരീരികള്‍ക്ക് ഒരു മാതൃകാ സമൂഹത്തിന്റെ സൃഷ്ടിക്കോ നിസ്തുലമായ ഒരു വിപ്ലവത്തിനോ നിമിത്തമാകുവാന്‍ കഴിയുമോ?

മുഹമ്മദി(സ)ന് ഉന്മാദരോഗമായിരുന്നുവെന്നും അദ്ദേഹം ശ്രവിച്ച അശ രീരികളാണ് ഖുര്‍ആനിലുള്ളതെന്നുമുള്ള വാദം പരിഗണനപോലും അര്‍ഹിക്കാത്ത ആരോപണം മാത്രമാണെന്നാണ് ഇതില്‍നിന്നെല്ലാം വ്യക്തമാകുന്നത്.

നങ്ങളെ ധാര്‍മികതയിലേക്ക് നയിക്കുന്ന ഗ്രന്ഥമാണ് ഖുര്‍ആന്‍. മദ്യ ത്തിലും മദിരാക്ഷിയിലും യുദ്ധങ്ങളിലും സായൂജ്യമടഞ്ഞിരുന്ന ഒരു സമൂഹ ത്തെ കേവലം 23 വര്‍ഷക്കാലം കൊണ്ട് ധാര്‍മികതയുടെ പ്രയോക്താക്കളും പ്രചാരകരുമാക്കിയ ഗ്രന്ഥമെന്ന ഖ്യാതി ഖുര്‍ആനിനു മാത്രം അവകാശപ്പെ ട്ടതാണ്. എന്നാല്‍ ധാര്‍മിക നവോത്ഥാനത്തിനുവേണ്ടി മുഹമ്മദ്(സ) രചി ച്ചുകൊണ്ട് ദൈവത്തില്‍ ആരോപിച്ച ഗ്രന്ഥമാണ് ഖുര്‍ആന്‍ എന്ന വാദ ഗതി അടിസ്ഥാന രഹിതമാണെന്ന് അത് ഒരാവര്‍ത്തി വായിക്കുന്ന ഏവര്‍ക്കും ബോധ്യമാവും. താഴെപ്പറയുന്ന വസ്തുതകള്‍ ശ്രദ്ധിക്കുക.

ഒന്ന്: സത്യസന്ധനായിരുന്നു മുഹമ്മദ് (സ) എന്ന കാര്യത്തില്‍ പക്ഷാ ന്തരമില്ല. അത്തരമൊരാള്‍ ധാര്‍മിക നവോത്ഥാനത്തിനുവേണ്ടി ദൈവത്തിന്റെ പേരില്‍ ഒരു പച്ചക്കള്ളം പറഞ്ഞുവെന്നു കരുതുന്നത് യുക്തി സഹമ ല്ല. ധാര്‍മിക നവോത്ഥാനത്തിനുവേണ്ടി ആത്മാര്‍ഥമായി പരിശ്രമിക്കുന്ന ഒരു വ്യക്തി അക്കാര്യത്തിനുവേണ്ടി സ്വന്തമായി ഒരു വലിയ അധര്‍മം ചെയ്യുകയെന്നത് അവിശ്വസനീയമാണ്. ദൈവത്തിന്റെ പേരില്‍ കളവ് പറയുന്നതിനേക്കാള്‍ വലിയ പാപമെന്താണ്?

രണ്ട്: പടച്ചവന്റെ പേരില്‍ കളവു പറയുകയും സ്വയം കൃതരചന കള്‍ ദൈവത്തിന്‍േറതാണെന്ന് വാദിക്കുകയും ചെയ്യുന്നവനാണ് ഏറ്റവും വലിയ അക്രമിയെന്നാണ് ഖുര്‍ആന്‍ പറയുന്നത്. ''അല്ലാഹുവിന്റെ പേരില്‍ കള്ളം കെട്ടിച്ചമയ്ക്കുകയോ, തനിക്ക് യാതൊരു ബോധനവും നല്‍കപ്പെടാതെ 'എനിക്ക് ബോധനം ലഭിച്ചിരിക്കുന്നു' എന്ന് പറയുകയോ ചെയ്തവനേ ക്കാളും അല്ലാഹു അവതരിപ്പിച്ചതുപോലെയുള്ളത് ഞാനും അവതരിപ്പിക്കാമെന്ന് പറഞ്ഞവനേക്കാളും വലിയ അക്രമി ആരുണ്ട്?''(6:93). ഖുര്‍ആന്‍ മുഹമ്മദി(സ)ന്റെ രചനയാണെങ്കില്‍ ഈ സൂക്തത്തില്‍ പറഞ്ഞ 'ഏറ്റവും വലിയ അക്രമി' അദ്ദേഹം തന്നെയായിരിക്കുമല്ലോ. തന്നെത്തന്നെ 'ഏറ്റവും വലിയ അക്രമി'യെന്ന് വിളിക്കുവാനും അതു രേഖപ്പെടുത്തുവാനും അദ്ദേ ഹം തയാറാകുമായിരുന്നുവോ?

മൂന്ന്: സ്വയംകൃത രചനകള്‍ നടത്തി അത് ദൈവത്തില്‍ ആരോപിക്കു ന്നവരെ ഖുര്‍ആന്‍ ശപിക്കുന്നുണ്ട്. ''എന്നാല്‍ സ്വന്തം കൈകള്‍ കൊണ്ട് ഗ്രന്ഥം എഴുതിയുണ്ടാക്കുകയും എന്നിട്ട് അത് അല്ലാഹുവില്‍നിന്ന് ലഭിച്ച താണെന്ന് പറയുകയും ചെയ്യുന്നവര്‍ക്ക് നാശം!''(2:79) ഖുര്‍ആന്‍ മുഹ മ്മദി(സ)ന്റെ സൃഷ്ടിയാണെങ്കില്‍ ഈ ശാപം അദ്ദേഹത്തിനുകൂടി ബാധക മാണല്ലോ. സ്വന്തമായി ഒരു രചന നിര്‍വഹിക്കുക. ആ രചനയില്‍ സ്വന്ത ത്തെത്തന്നെ ശപിക്കുക. ഇത് വിശ്വസനീയമാണോ?

നാല്: ഖുര്‍ആന്‍ ഒന്നിച്ച് അവതരിപ്പിക്കപ്പെട്ട ഗ്രന്ഥമല്ല. നീണ്ട ഇരുപ ത്തിമൂന്ന് വര്‍ഷങ്ങള്‍ക്കിടയില്‍ വ്യത്യസ്ത സാഹചര്യങ്ങളിലാണ് ഖുര്‍ആ ന്‍ സൂക്തങ്ങള്‍ അവതരിപ്പിക്കപ്പെട്ടത്. ഓരോ വിഷയങ്ങളിലും ജനങ്ങളുടെ ചോദ്യങ്ങള്‍ക്കുള്ള മറുപടിയായിട്ടാണ് ചില സന്ദര്‍ഭങ്ങളില്‍ ഖുര്‍ആന്‍ സൂക്തങ്ങള്‍ അവതരിപ്പിച്ചിരിക്കുന്നത്. ഖുര്‍ആനില്‍ പതിനഞ്ചോളം സ്ഥ ലങ്ങളില്‍ 'അവര്‍ നിന്നോട്...നെക്കുറിച്ചു ചോദിക്കുന്നു. പറയുക: ...' എന്ന ശൈലിയിലുള്ള സൂക്തങ്ങളുണ്ട്. ഓരോ വിഷയങ്ങളിലും പ്രവാചകനോട് അവര്‍ ചോദിച്ച സമയത്ത് അദ്ദേഹത്തിന് ഉത്തരം നല്‍കാന്‍ സാധിച്ചില്ലെ ന്നും പിന്നീട് ഖുര്‍ആന്‍ വാക്യം അവതരിപ്പിച്ചതിനുശേഷം മാത്രമാണ് അത് സാധിച്ചതെന്നുമാണല്ലോ ഇതില്‍നിന്ന് മനസ്സിലാവുന്നത്. ധാര്‍മിക നവോത്ഥാനം ലക്ഷ്യമാക്കിക്കൊണ്ടുള്ള പ്രവാചക(സ)ന്റെ രചനയായിരുന്നു ഖുര്‍ആനെങ്കില്‍ ജനം ചോദിച്ചപ്പോള്‍ ഉടന്‍തന്നെ അദ്ദേഹത്തിന് മറുപടി പറയാന്‍ കഴിയുമായിരുന്നു.

ഉദാഹരണത്തിന്, മദ്യത്തില്‍നിന്നും ചൂതാട്ടത്തില്‍നിന്നും ജനങ്ങളെ രക്ഷിക്കണമെന്നായിരുന്നു പ്രവാചകന്റെ ഉദ്ദശ്യമെങ്കില്‍ അവയെക്കുറിച്ച് ചോദിച്ച ഉടന്‍തന്നെ അവ പാപമാണ് എന്ന് അ ദ്ദേഹം മറുപടി പറയുമായിരുന്നു. എന്നാല്‍, അദ്ദേഹം ചെയ്തത് അതല്ല; സ്വയം മറുപടി പറയാതെ ദൈവിക വെളിപാട് പ്രതീക്ഷിക്കുകയായിരുന്നു. ദൈവവചനങ്ങള്‍ വെളിപ്പെട്ടതിനുശേഷമാണ് ഈ തിന്മകള്‍ക്കെതിരെയുള്ള നടപടികള്‍ അദ്ദേഹം സ്വീകരിച്ചത്.

അഞ്ച്: മുഹമ്മദ് നബി(സ)യെ തിരുത്തുന്ന ചില ഖുര്‍ആന്‍ സൂക്തങ്ങളുണ്ട്. ഖുറൈശി പ്രമുഖരുമായി സംസാരിച്ചുകൊണ്ടിരിക്കുമ്പോള്‍ കടന്നുവന്ന അന്ധനായ അബ്ദുല്ലാഹിബ്‌നുഉമ്മിമക്തൂമിനെ പ്രസന്നതയോടെ സ്വീകരിക്കാതിരുന്ന പ്രവാചക(സ)ന്റെ നടപടിയെ തിരുത്തിയ ഖുര്‍ആന്‍ സൂക്തങ്ങള്‍ (80:1-10) സുവിദിതമാണ്.

മറ്റൊരു സംഭവം: മുസ്‌ലിംകള്‍ക്ക് ഏറെ നാശനഷ്ടങ്ങള്‍ വിതച്ച ഉഹ്ദ് യുദ്ധത്തില്‍ പ്രവാചകന്റെ ശരീരത്തിലും ഒരുപാട് മുറിവുകള്‍ ഉണ്ടായി. യുദ്ധശേഷം അദ്ദേഹം അവിശ്വാസികളി ല്‍ ചിലരെ ശപിക്കുകയും 'അവരുടെ പ്രവാചകനെ മുറിപ്പെടുത്തിയ സമൂഹ മെങ്ങനെയാണ് നന്നാവുക?' എന്ന് ആത്മഗതം നടത്തുകയും ചെയ്തു. ഉടന്‍ ഖുര്‍ആന്‍ സൂക്തമവതരിച്ചു; പ്രവാചക(സ)നെ തിരുത്തിക്കൊണ്ട്. ''(നബിയേ), കാര്യത്തിന്റെ തീരുമാനത്തില്‍ നിനക്ക് യാതൊരവകാശവുമില്ല. അവന്‍ (അല്ലാഹു) ഒന്നുകില്‍ അവരുടെ പശ്ചാത്താപം സ്വീകരിച്ചേ ക്കാം. അല്ലെങ്കില്‍ അവരെ അവന്‍ ശിക്ഷിച്ചേക്കാം. തീര്‍ച്ചയായും അവര്‍ അക്രമികളാകുന്നു''(3:128)(തിര്‍മിദി,ഇബ്‌നുമാജ).

ഇതൊന്നും പ്രവാചകനി ല്‍ ബോധപൂര്‍വ്വം വന്ന തെറ്റുകളല്ല. താന്‍ സ്വീകരിച്ച നിലപാടുകളിലുണ്ടാ യ അബദ്ധം മാത്രം. എന്നിട്ടും അവ തിരുത്തുന്ന വചനങ്ങള്‍ ഖുര്‍ആനി ലുണ്ടായി. ജനങ്ങളെ ധര്‍മനിഷ്ഠരാക്കുവാന്‍ വേണ്ടി പ്രവാചകന്‍(സ) പടച്ച ഗ്രന്ഥമായിരുന്നു ഖുര്‍ആനെങ്കില്‍ അദ്ദേഹത്തിന്റെ നടപടികളെ വിമര്‍ശിക്കുന്ന സൂക്തങ്ങള്‍ ഖുര്‍ആനിലുണ്ടാവുമായിരുന്നുവോ?

അറബികളെ ഐക്യപ്പെടുത്തുകയും പുരോഗതിയിലേക്ക് നയിക്കുകയു മായിരുന്നു ഖുര്‍ആനിന്റെ പിന്നിലുള്ള ലക്ഷ്യമെങ്കില്‍ അതിലെ പ്രതിപാദന ങ്ങളില്‍നിന്ന് ഇക്കാര്യം വ്യക്തമാവുമായിരുന്നു. എന്നാല്‍, ഖുര്‍ആന്‍ ഒരാവര്‍ത്തി വായിച്ച ഒരാള്‍ക്ക് അതില്‍ അറബി ദേശീയതയുടെ ഉയിര്‍ത്തെഴുന്നേല്‍പ്പ് ഒരു വിഷയമായി വരുന്നേയില്ലെന്ന വസ്തുത വ്യക്തമാവുന്നതാണ്. അറബികളുടെ നവോത്ഥാനമായിരുന്നു ഖുര്‍ആന്‍ രചനക്കുപിന്നിലുള്ള ഉദ്ദേശ്യമെന്ന വാദം താഴെ പറയുന്ന വസ്തുതകള്‍ക്കുമുന്നില്‍ അടിസ്ഥാന രഹിതമായിത്തീരുന്നു.

ഒന്ന്: അറബികളുടെ നവോത്ഥാനത്തെയോ ഐക്യത്തെയോ പ്രോല്‍സാഹിപ്പിക്കുന്ന രീതിയിലുള്ള ഒരു വചനംപോലും ഖുര്‍ആനിലില്ല.

രണ്ട്: ദേശീയമായ അതിര്‍വരമ്പുകളില്ലാത്ത ആദര്‍ശസമൂഹമെന്ന സങ്ക ല്‍പമാണ് ഖുര്‍ആന്‍ മുന്നോട്ടുവെക്കുന്നത്. 'ഉമ്മത്ത്' എന്ന സാങ്കേതിക സംജ്ഞയാല്‍ വിശേഷിപ്പിക്കപ്പെട്ടിരിക്കുന്ന ആദര്‍ശസമൂഹത്തില്‍ സത്യ വിശ്വാസം സ്വീകരിച്ച ഏവരും ദേശീയതയുടെയോ പ്രാദേശികത്വത്തിന്റെ യോ വര്‍ഗത്തിന്റെയോ ജാതീയതയുടെയോ അതിര്‍വരമ്പുകളില്ലാതെ അംഗങ്ങളാണ്. അറബിദേശീയതയെന്ന സങ്കല്‍പംതന്നെ ഖുര്‍ആനിന് അന്യമാണ്.

മൂന്ന്: അറബികളുടെ നവോത്ഥാനമായിരുന്നു മുഹമ്മദി(സ)ന്റെ ല ക്ഷ്യമെങ്കില്‍ അധികാരം വാഗ്ദാനം ചെയ്യപ്പെട്ടപ്പോള്‍ അത് സ്വീകരിക്കുക യും ശക്തിയും പാടവവുമുപയോഗിച്ച് അവരെ ഐക്യപ്പെടുത്തുകയും പുരോഗതിയിലേക്ക് നയിക്കുകയും ചെയ്യാന്‍ അദ്ദേഹം ശ്രമിക്കുമായിരുന്നു. എന്നാല്‍, അതുണ്ടായില്ല. അധികാരം സ്വീകരിച്ചുകൊണ്ട് നവോത്ഥാനത്തിന് ശ്രമിക്കുന്നതിന് പകരം അത് നിരസിക്കുകയാണ് അദ്ദേഹം ചെയ്തത്.

നാല്: അധികാരം ലഭിച്ചതിനുശേഷവും അദ്ദേഹം അറബികളുടെ ഏതെ ങ്കിലും തരത്തിലുള്ള ഔന്നത്യത്തിനുവേണ്ടി വാദിച്ചിട്ടില്ല. തന്റെ അന്തിമ പ്രസംഗത്തില്‍ അദ്ദേഹം അര്‍ഥശങ്കക്കിടയില്ലാത്ത വണ്ണം പ്രഖ്യാപിച്ചു: ''അറബിക്ക് അനറബിയേക്കാളോ അനറബിക്ക് അറബിയേക്കാളോ യാതൊരു ശ്രേഷ്ഠതയുമില്ല; ധര്‍മനിഷ്ഠയുടെ പേരിലല്ലാതെ''.(അഹ്മദ്)

ഇത് അറ ബ് ദേശീയതയുടെ നവോത്ഥാനത്തിനുവേണ്ടി ശ്രമിച്ച ഒരു വ്യക്തിയുടെ വാക്കുകളാകുമോ?

അഞ്ച്: സത്യവിശ്വാസികള്‍ക്ക് മാതൃകയായി ഖുര്‍ആനില്‍ പരാമര്‍ശിക്ക പ്പെട്ടിട്ടുള്ളത് രണ്ടു വനിതകളാണ്. ഒന്ന്, ഫറോവയുടെ പത്‌നി. രണ്ട്, യേശുവിന്റെ മാതാവ് (66:11,12). രണ്ടു പേരും അറബികളല്ല. അറബ് ദേശീയതക്കുവേണ്ടി ഗ്രന്ഥമെഴുതിയ വ്യക്തി ലോകത്തിന് മാതൃകയായി എടുത്തുകാണിക്കുന്നത് അറബികളുടെ എതിരാളികളെയാകുമോ? മര്‍യമിനെ ക്കുറിച്ച് ഖുര്‍ആന്‍ പറയുന്നതിങ്ങനെയാണ്: ''മലക്കുകള്‍ ഇപ്രകാരം പറ ഞ്ഞ സന്ദര്‍ഭം: മര്‍യമേ, തീര്‍ച്ചയായും അല്ലാഹു നിന്നെ പ്രത്യേകം തെര ഞ്ഞെടുക്കുകയും നിനക്ക് പരിശുദ്ധി നല്‍കുകയും ലോകത്തുള്ള സ്ത്രീകളില്‍ വെച്ച് ഉല്‍കൃഷ്ടയായി നിന്നെ തെരഞ്ഞെടുക്കുകയും ചെയ്തിരിക്കുന്നു''(3:42).

ബൈബിളിലൊരിടത്തും ഇത്ര ബഹുമാനത്തോടുകൂടി മര്‍യമിനെക്കുറിച്ച് പരാമര്‍ശിക്കപ്പെട്ടിട്ടില്ല എന്നുകൂടി ഓര്‍ക്കുക. ലോക വനിതകളില്‍ ഉല്‍കൃഷ്ടയായി ഖുര്‍ആന്‍ എടുത്തുകാണിക്കുന്നത് മുഹമ്മദി(ല)ന്റെ മാതാവിനെയോ ഭാര്യയെയോ മറ്റേതെങ്കിലും അറബ് സ്ത്രീയെയോ അല്ല; ഇസ്രായേല്‍ വനിതയായ മര്‍യമിനെയാണ്. അറബ് ദേശീയതയുടെ വക്താവില്‍നിന്ന് ഇത്തരമൊരു പരാമര്‍ശം പ്രതീക്ഷിക്കുവാന്‍ പറ്റുമോ?

ആറ്: അറബ് ദേശീയതയുടെ നവോത്ഥാനത്തിനുവേണ്ടി പണിയെടുക്കുന്ന ഒരു വ്യക്തി അറബികളുടെ അഹംബോധത്തെ ഉദ്ദീപിക്കുവാനായിരിക്കും തന്റെ രചനയില്‍ ശ്രമിക്കുക. അറബികളുടെ ശ്രേഷ്ഠതയെക്കുറിച്ചുമാത്രമായിരിക്കും അയാള്‍ സംസാരിക്കുക. എന്നാല്‍ ഖുര്‍ആന്‍ ഇസ്രായേല്യര്‍ക്ക് നല്‍കിയ ശ്രേഷ്ഠതയെക്കുറിച്ചും സംസാരിക്കുന്നുണ്ട്. ''ഇസ്രായേല്‍ സന്തതികളേ, നിങ്ങള്‍ക്ക് ഞാന്‍ ചെയ്തുതന്നിട്ടുള്ള അനുഗ്രഹവും മറ്റു ജനവിഭാഗങ്ങളേക്കാള്‍ നിങ്ങള്‍ക്ക് ഞാന്‍ ശ്രേഷ്ഠത നല്‍കിയതും നിങ്ങളോര്‍ക്കുക''(2:47).

ധികാരമോഹമെന്നാല്‍ എന്താണ്? രാജ്യത്തിന്റെ അധികാരം കൈ ക്കലാക്കി സുഖസമൃദ്ധമായ ജീവിതം നയിക്കാനുള്ള ആഗ്രഹം. പതിമൂന്ന് വര്‍ഷത്തെ കഷ്ടപ്പാടുകള്‍ക്കും പീഡനങ്ങള്‍ക്കും ശേഷം പലായനം ചെയ്തു മദീനയിലെത്തിയ പ്രവാചകന് അധികാരം ലഭിച്ചുവെന്നത് നേരാണ്. എന്നാല്‍, അദ്ദേഹത്തിന് അധികാരം സുഖലോലുപതയ്ക്കുള്ള മാര്‍ഗമായി രുന്നില്ല. ഭരണാധികാരിയായിരിക്കുമ്പോഴും ഈത്തപ്പനപ്പായയില്‍ അന്തിയുറങ്ങുകയും വസ്ത്രങ്ങള്‍ സ്വയം അലക്കുകയും പാദരക്ഷകള്‍ തുന്നുകയും ആടിനെ കറക്കുകയും ചെയ്യുന്ന മനുഷ്യനെ അധികാരമോഹിയെന്നു വിളിക്കാന്‍ ആര്‍ക്കാണ് സാധിക്കുക? അധികാരത്തിന്റെ പേരില്‍ ജനങ്ങളാല്‍ ആദരിക്കപ്പെടുകയും അവരില്‍ നിന്ന് ഉയര്‍ന്നുനില്‍ക്കുകയും ചെയ്യാന്‍ ആഗ്രഹിക്കുന്നവരാണ് അധികാരം മോഹിക്കുക. പ്രവാചക(സ)നാവട്ടെ ജനങ്ങളെ സേവിച്ച് ജനങ്ങളോടൊപ്പം ജീവിച്ചയാളായിരുന്നു. തന്നെ ബഹുമാനിച്ചുകൊണ്ട് ആളുകള്‍ എഴുന്നേറ്റു നില്‍ക്കുന്നതുപോലും അവിടുന്ന് ഇഷ്ടപ്പെട്ടിരുന്നില്ല.

അദ്ദേഹം ഉപദേശിച്ചു: ''ക്രിസ്ത്യാനികള്‍ മര്‍യമിന്റെ പുത്രനായ യേശുവിനെ പുകഴ്ത്തിയതുപോലെ എന്നെ നിങ്ങള്‍ പുകഴ്ത്തരുത്'' (ബുഖാരി, മുസ്‌ലിം). ഇതെല്ലാംതന്നെ മുഹമ്മദ് (സ) ഒരു അധികാര മോഹിയായിരുന്നില്ലെന്ന് വ്യക്തമാക്കുന്നു.

മാത്രവുമല്ല, തന്റെ പ്രബോധന പ്രവര്‍ത്തനങ്ങള്‍ നിര്‍ത്തിവെക്കുകയാ ണെങ്കില്‍, മക്കയിലെ പ്രയാസപൂര്‍ണമായ ആദ്യനാളുകളില്‍തന്നെ അധികാരം നല്‍കാമെന്ന് അദ്ദേഹം വാഗ്ദാനം ചെയ്യപ്പെട്ടിരുന്നു. സമൂഹത്തിലെ നേതാക്കന്മാരെല്ലാംകൂടി ഒരു ദിവസം മുഹമ്മദി(സ)ന്റെ അടുത്തുചെന്ന് അദ്ദേഹത്തെ വശീകരിക്കാനായി ആവത് ശ്രമിച്ചു നോക്കി. പക്ഷെ, നിരാശ മാത്രമായിരുന്നു ഫലം. മക്കയിലെ പ്രബോധന പ്രവര്‍ത്തനങ്ങളുടെ ആദ്യനാളുകളിലായിരുന്നു ഈ സംഭവം. ഖുര്‍ആന്‍ രചിച്ചുകൊണ്ട് താന്‍ ദൈവദൂതനാണെന്ന് വരുത്തിത്തീര്‍ത്ത് അധികാരം കൈക്കലാക്കുകയായിരുന്നു പ്രവാചക(ല)ന്റെ ലക്ഷ്യമെങ്കില്‍ പ്രയാസങ്ങള്‍ ഏറെയൊന്നും സഹിക്കാതെ അധികാരം തന്റെ കാല്‍ക്കീഴില്‍ വന്ന സമയത്ത് അദ്ദേഹം അത് സ്വീകരിക്കുവാന്‍ വൈമനസ്യം കാണിച്ചതെന്തിനാണ്? മുഹമ്മദ്അ(സ)ധികാരം കാംക്ഷിച്ചിരുന്നില്ലെന്ന് ഇതില്‍നിന്ന് സുതരാം വ്യക്തമാണ്. ഖുര്‍ആന്‍ കൊണ്ടുവന്നതിനു പിന്നില്‍ അധികാരമോഹമായിരുന്നില്ലെന്ന് സാരം.

നാഥനായി വളര്‍ന്ന മുഹമ്മദ്(സ) ചെറുപ്പത്തില്‍ ഒരുപാട് കഷ്ടപ്പാടുകള്‍ അനുഭവിച്ചിരിക്കാം. എന്നാല്‍, തന്റെ 25-ാം വയസ്സില്‍ നാല്‍പതുകാരിയായ കച്ചവടക്കാരി ഖദീജ( (റ)യെ വിവാഹം ചെയ്തതിനുശേഷം അദ്ദേ ഹത്തിന്റെ ജീവിതനിലവാരം സ്വാഭാവികമായും മെച്ചപ്പെട്ടതായി മാറിയിരിക്കണം. അത്യാവശ്യം നല്ല സാമ്പത്തിക ശേഷിയുണ്ടായിരുന്ന ഖദീജ( (റ)യുടെ ഭര്‍ത്താവായിരുന്ന അദ്ദേഹം സാമ്പത്തിക ക്ലേശങ്ങള്‍ അനുഭവിച്ചിരിക്കാനുള്ള സാധ്യത വിരളമാണ്.

ഖദീജയുമായുള്ള മുഹമ്മദി(സ) ന്റെ വിവാഹം നടന്നത് പ്രവാചകത്വം ലഭിക്കുന്നതിന് 15 വര്‍ഷങ്ങള്‍ക്കുമുമ്പാണ്. പതി നഞ്ച് വര്‍ഷം സാമ്പത്തികക്ലേശം കൂടാതെ ജീവിച്ചതിനുശേഷമാണ് താന്‍ പ്രവാചകനാണെന്നും ഖുര്‍ആന്‍ ദൈവവചനമാണെന്നുമുള്ള അവകാശവാദങ്ങളുമായി മുഹമ്മദ്(സ) രംഗപ്രവേശം ചെയ്യുന്നതെന്നര്‍ഥം. ഖുര്‍ആന്‍ ദൈവികമാണെന്ന് വാദിക്കുക വഴി ഭൗതികലാഭമാണ് അദ്ദേഹം ഇച്ഛിച്ചതെങ്കില്‍ ഈ വാദം ഉന്നയിച്ചതിനുശേഷം അദ്ദേഹത്തിന്റെ സാമ്പത്തിക സ്ഥിതി മെച്ചപ്പെട്ടിരിക്കണമല്ലോ.

എന്നാല്‍, എന്തായിരുന്നു സ്ഥിതി?

പ്രവാചകപത്‌നി ആഇശ(റ) പറയുന്നു: ''ഞങ്ങളുടെ വീട്ടില്‍ ഒന്നും പാചകം ചെയ്യാനില്ലാത്തതിനാല്‍ അടുപ്പു പുകയാതെ ഒന്നോ രണ്ടോ മാസ ങ്ങള്‍ കഴിഞ്ഞുപോകാറുണ്ടായിരുന്നു. ഈത്തപ്പഴവും വെള്ളവുമായിരുന്നു ഞങ്ങളുടെ ഉപജീവനം. ചിലപ്പോള്‍ മദീനത്തുകാര്‍ കൊണ്ടുവന്ന ആട്ടി ന്‍പാലും ഈത്തപ്പഴത്തോടു കൂടെയുണ്ടാവും''. (ബുഖാരി, മുസ്‌ലിം)

ആഇശ(റ) ഒരാളോട് പഴയകാര്യങ്ങള്‍ പറഞ്ഞുകൊണ്ടിരിക്കുകയാണ്. മദീനയിലേക്കുള്ള പലായനത്തിനുശേഷം പ്രവാചകനും കുടുംബവും സ ഹിച്ച പ്രയാസങ്ങളാണ് പ്രതിപാദ്യം. ഒരു രാത്രി തപ്പിത്തടഞ്ഞുകൊണ്ട് വീട്ടുജോലികള്‍ ചെയ്തകാര്യം അവര്‍ പറഞ്ഞു. അയാള്‍ ചോദിച്ചു: ''വിള ക്കില്ലായിരുന്നുവോ? അവര്‍ പ്രതിവചിച്ചു: ''വിളക്കു കത്തിക്കാനുള്ള എണ്ണ ഞങ്ങളുടെ പക്കലുണ്ടായിരുന്നുവെങ്കില്‍ വിശപ്പ് മാറ്റാന്‍ അത് കുടിക്കുമാ യിരുന്നു; കത്തിക്കുന്നതിനു പകരം''. (അഹ്മദ്, ത്വബ്‌റാനി)

ഇത് പ്രവാചകന്റെ ആദ്യകാലത്തെ മാത്രം അവസ്ഥയല്ല. മുഹമ്മദ്(സ) ശക്തമായ ഒരു സാമ്രാജ്യത്തിന്റെ ഭരണാധികാരിയായിരുന്നപ്പോഴും അദ്ദേ ഹത്തിന്റെ അവസ്ഥ ഇതില്‍നിന്ന് ഒട്ടും മെച്ചമായിരുന്നില്ല. ഇസ്‌ലാമിക സാമ്രാജ്യത്തിന്റെ അധിപന്റെ അന്തപുരത്തെക്കുറിച്ച് അദ്ദേഹത്തിന്റെ സഹചാരിയായിരുന്ന ഉമര്‍ (റ)തന്നെ പറയട്ടെ: ''പ്രവാചകന്റെ മുറിയില്‍ ഊറക്കിട്ട മൂന്ന് തോല്‍കഷ്ണങ്ങളും ഒരു മൂലയില്‍ അല്‍പം ബാര്‍ലിയുമല്ലാതെ മറ്റൊന്നുംതന്നെ ഞാന്‍ കണ്ടില്ല. ഞാന്‍ കരഞ്ഞുപോയി. പ്രവാചകന്‍ ചോദിച്ചു: 'എന്തിനാണ് താങ്കള്‍ കരയുന്നത്?' ഞാന്‍ പറഞ്ഞു: 'അല്ലാഹുവിന്റെ ദൂതരേ! ഞാനെങ്ങനെ കരയാതിരിക്കും? താങ്കളുടെ ശരീരത്തില്‍ ഈത്തപ്പനയോലകളുടെ പാട് ഞാന്‍ കാണുന്നു. ഈ മുറിയില്‍ എന്തെല്ലാമുണ്ടെന്നും ഞാനറിയുന്നു. അല്ലാഹുവിന്റെ ദൂതരേ! സമൃദ്ധമായ വിഭവങ്ങള്‍ക്കുവേണ്ടി അല്ലാഹുവിനോട് പ്രാര്‍ ഥിച്ചാലും. അവിശ്വാസികളും അല്ലാഹുവില്‍ പങ്കുചേര്‍ക്കുന്നവരുമായ പേര്‍ഷ്യക്കാരുടെയും റോമാക്കാരുടെയും രാജാക്കന്മാര്‍-സീസറും കൈസറുമെല്ലാം-അരുവികള്‍ ഒഴുകുന്ന തോട്ടങ്ങളില്‍ വസിക്കുമ്പോള്‍ അല്ലാഹുവിന്റെ തെരഞ്ഞെടുക്കപ്പെട്ട പ്രവാചകന്‍ ജീവിക്കുന്നത് ദാരുണമായ പട്ടിണിയില്‍!' എന്റെ ഈ സംസാരം കേട്ടപ്പോള്‍ തലയിണയില്‍ വിശ്രമിക്കുകയായിരുന്ന പ്രവാചകന്‍ എഴുന്നേറ്റിരുന്നു. എന്നിട്ടു പറഞ്ഞു: 'ഉമര്‍! താങ്കള്‍ ഈ വിഷയത്തില്‍ ഇനിയും സംശയാലുവാണോ? ഭൗതിക ജീവിതത്തിലെ സുഖസൗകര്യങ്ങളേക്കാള്‍ നല്ലത് മരണാനന്തര ജീവിതത്തിലെ സുഖസൗ കര്യങ്ങളാണ്. അവിശ്വാസികള്‍ അവരുടെ നന്മയുടെ വിഹിതം ഈ ജീവിതത്തില്‍ ആസ്വദിക്കുന്നു. നമ്മുടേതാകട്ടെ, മരണാനന്തര ജീവിതത്തിലേക്കുവേണ്ടി ബാക്കിവെച്ചിരിക്കുകയാണ്'. ഞാന്‍ അദ്ദേഹത്തോട് അപേക്ഷിച്ചു: 'ദൈവദൂതരെ! എനിക്കുവേണ്ടി മാപ്പിനപേക്ഷിച്ചാലും. എനിക്കു തെറ്റിപ്പോയി''.

ഖുര്‍ആന്‍ ഭൗതിക ലാഭങ്ങള്‍ക്കുവേണ്ടി പടച്ചുണ്ടാക്കിയ മുഹമ്മദി(സ) ന്റെ കൃതിയാണെന്ന വാദമാണിവിടെ തകരുന്നത്. ആകെ സ്വത്തായി ബാക്കിയുണ്ടായിരുന്ന ഏഴു ദീനാര്‍ മരണത്തിനുമുമ്പ് ദാനം ചെയ്യുകയും യഹൂദന് തന്റെ പടച്ചട്ട പണയം വെച്ചുകൊണ്ട് മരണപ്പെടുകയും ചെയ്ത മനുഷ്യന്‍ ധനമോഹിയായിരുന്നുവെന്ന് പറയുന്നത് അടിസ്ഥാനരഹിതമാണ്.

ഖുര്‍ആനിന്റെ രചനക്കുപിന്നില്‍ ധനമോഹമായിരുന്നുവെന്ന വാദം അടിസ്ഥാന രഹിതമാണെന്ന് ദി ന്യു കാത്തോലിക് എന്‍സൈക്ലോപീഡിയ പോലും സമ്മതിച്ചിട്ടുണ്ട്. ''മുഹമ്മദി(സ) ന്റെ മതവിപ്ലവത്തിനു പിന്നില്‍ ധനമോഹമായിരുന്നുവെന്ന ഒരു ധാരണ സൃഷ്ടിക്കപ്പെട്ടിട്ടുണ്ട്. വ്യക്തമാ യി അറിയപ്പെടുന്ന വസ്തുതകള്‍ ഈ ധാരണക്കെതിരാണ്'' (The New Catholic Encyclopedia Vol IX, Page 1001).

മുഹമ്മദ് നബി(സ) ജീവിച്ചത് ചരിത്രത്തിന്റെ വെളിച്ചത്തിലാണ്. അദ്ദേ ഹത്തിലൂടെയാണ് ലോകം ഖുര്‍ആന്‍ ശ്രവിച്ചത്. അതുകൊണ്ടുതന്നെ ഖുര്‍ആനിന്റെ ദൈവികത അംഗീകരിക്കാത്തവരെ സംബന്ധിച്ചിടത്തോളം അവര്‍ക്ക് പറയാനുള്ളത് ഇത് മുഹമ്മദി(സ)ന്റെ രചനയാണെന്നാണ്. ഈ വാദം വിശദമായി ചര്‍ച്ച ചെയ്യേണ്ടതാണ്. ചര്‍ച്ചയുടെ ആമുഖമായി നാം മനസ്സിലാക്കേണ്ട ചില വസ്തുതകളുണ്ട്. അവയുടെ അടിത്തറയില്‍നിന്നുകൊണ്ടായിരിക്കണം മുഹമ്മദ് നബി(സ)യില്‍ ഖുര്‍ആനിന്റെ കര്‍തൃത്വം ആരോപിക്കുന്നത്.

ഒന്ന്: നാല്‍പതു വയസ്സുവരെ അറബികള്‍ക്കിടയില്‍ സുസമ്മതനായ വ്യക്തിയായിരുന്നു മുഹമ്മദ്(സ). ഖുര്‍ആന്‍ ദൈവികമാണെന്നും അതിലെ വിധിവിലക്കുകള്‍ അനുസരിക്കേണ്ടതുണ്ടെന്നും പ്രബോധനം ചെയ്തതുകൊണ്ടാണ് അദ്ദേഹം വെറുക്കപ്പെട്ടവനായത്; ബഹിഷ്‌കരിക്കപ്പെട്ടത്; ജനിച്ച് വളര്‍ന്ന നാട്ടില്‍ നിന്ന് പലായനം ചെയ്യേണ്ടിവന്നത്.

രണ്ട്: സത്യസന്ധനായിരുന്നു മുഹമ്മദ്(സ)എന്ന കാര്യത്തില്‍ അദ്ദേഹ ത്തിന്റെ കഠിന ശത്രുക്കള്‍ക്കുപോലും അഭിപ്രായവ്യത്യാസമുണ്ടായിരുന്നി ല്ല. നാല്‍പതു വയസ്സുവരെ സത്യസന്ധനായി ജീവിച്ച അദ്ദേഹം ഒരു ദിവ സം പടച്ചതമ്പുരാന്റെ പേരില്‍ ഒരു പച്ചക്കള്ളം പറഞ്ഞുവെന്നും അത് പ്രചരിപ്പിക്കുന്നതിനുവേണ്ടി സ്വന്തം ജീവന്‍ തൃണവത്ഗണിച്ചുവെന്നും വിശ്വസിക്കുക പ്രയാസമാണ്.

മൂന്ന്: സാഹിത്യകാരന്മാര്‍ക്ക് അറേബ്യയില്‍ ഉന്നതമായ സ്ഥാനം നല്‍ കപ്പെട്ടിരുന്നു. ഖുര്‍ആന്‍ അത്യുന്നതമായ ഒരു സാഹിത്യ സൃഷ്ടിയാണെന്ന കാര്യത്തില്‍ ആര്‍ക്കും ഭിന്നാഭിപ്രായമുണ്ടായിരുന്നില്ല. അത് തന്‍േറതാണ് എന്ന് അവകാശപ്പെട്ടിരുന്നുവെങ്കില്‍ അദ്ദേഹത്തിന് അറബികള്‍ക്കിടയില്‍ ഉന്നതമായ സ്ഥാനമാനങ്ങള്‍ ലഭിക്കുമായിരുന്നു.

നാല്: മുഹമ്മദി(സ)ന്റെ ചില നടപടികളെ വിമര്‍ശിക്കുന്ന വാക്യങ്ങള്‍ ഖുര്‍ആനിലുണ്ട്. അഞ്ച്: മുഹമ്മദി(സ)നെ ശക്തമായി താക്കീത് ചെയ്യുന്ന വചനങ്ങളും ഖുര്‍ആനിലുണ്ട്. ഈ വസ്തുതകള്‍ മുന്നില്‍ വെച്ചുകൊണ്ടാണ് ഖുര്‍ആന്‍ മുഹമ്മദി(സ)ന്റെ സൃഷ്ടിയാണ് എന്ന വാദത്തിലെ ശരിയും തെറ്റും പരിശോധിക്കേ ണ്ടത്.

സാഹിത്യമൂല്യമുള്ള ഒരു സൃഷ്ടി നടത്തി അത് ദൈവത്തിന്റെ പേരില്‍ ആരോപിച്ചതാണെങ്കില്‍ അതിനു പിന്നില്‍ സ്വാര്‍ഥമായ വല്ല ലക്ഷ്യങ്ങളുമുണ്ടാവണമല്ലോ. അതെന്തായിരുന്നുവെന്നാണ് വിമര്‍ശകര്‍ ആദ്യം വ്യക്തമാക്കേണ്ടത്. അതിന്റെ അടിസ്ഥാനത്തിലാണ് ഈ വാദത്തിന്റെ സത്യത പരിശോധിക്കപ്പെടേണ്ടത്.

വിശാലമായ ഈ പ്രപഞ്ചത്തില്‍ മനുഷ്യന്‍ എത്രയോ നിസ്സാരന്‍. പ്രപഞ്ച സ്രഷ്ടാവിന് നിസ്സാരനായ മനുഷ്യന്റെ ''നീ വലിയവനാണ്'' (അല്ലാഹു അക്ബര്‍) എന്ന സര്‍ട്ടിഫിക്കറ്റ് ആവശ്യമുണ്ടോ? അല്ലാഹു ഒരു തന്നെപ്പൊക്കിയാണെന്നല്ലേ ഇതിനര്‍ഥം? മനുഷ്യന്‍ പുകഴ്ത്തിയില്ലെങ്കിലും ദൈവം വലിയവന്‍ തന്നെയല്ലേ?

  • 'അല്ലാഹു അക്ബര്‍' എന്നതിന് അല്ലാഹുവാണ് ഏറ്റവും വലിയവന്‍ എന്നാണര്‍ഥം. 'നീ വലിയവനാണ്' എന്നല്ല. അത്യന്തം സൂ ക്ഷ്മവും അതീവസ്ഥൂലവുമായ ഭൗതിക വസ്തുക്കളൊന്നും ഒരു മഹാസംവിധായകനില്ലാതെ നിലവില്‍വരുക സാദ്ധ്യമല്ലെന്നാണ് സത്യസന്ധതയും പക്വതയുമുള്ള ബുദ്ധിജീവികള്‍ ഉറപ്പിച്ചുപറയുന്നത്. മനുഷ്യശരീരത്തിലെ ശതകോടിക്കണക്കിലുള്ള ഡി.എന്‍.എ. തന്മാത്രകളില്‍ ഓരോന്നിലും രാസാക്ഷരങ്ങള്‍കൊണ്ട് കുറിച്ചിട്ടുള്ള വിവരശേഖരം ഒരു കംപ്യൂട്ടറില്‍ ഉള്‍ക്കൊള്ളിക്കാന്‍ കഴിയാത്ത ത്ര ഭീമമാണെന്നത്രെ ജിനോമിക്‌സ് ഗവേഷകരുടെ കണ്ടെത്തല്‍. ഒരു മില്ലിമീറ്ററിന്റെ മില്യനിലൊന്ന് മാത്രം വലിപ്പമുള്ള അതിസൂക്ഷ് മവസ്തുവില്‍ അപാരമായ ഈ വിവരശേഖരം തനിയെ വന്നുചേ ര്‍ന്നു എന്ന് പറയുന്നത് തികഞ്ഞ അസംബന്ധമാകുന്നു. എന്നാല്‍ വിശുദ്ധഖുര്‍ആനില്‍ സര്‍വജ്ഞനായ അല്ലാഹു പറയുന്നത് നോക്കുക:

''അല്ലാഹുവിന്റെതാകുന്നു ആകാശങ്ങളിലും ഭൂമിയിലുമുള്ള തെല്ലാം. തീര്‍ച്ചയായും അല്ലാഹു പരാശ്രയമുക്തനും സ്തുത്യര്‍ഹനുമാകുന്നു. ഭൂമിയിലുള്ള വൃക്ഷമെല്ലാം പേനയായിരിക്കുകയും സമുദ്രം മഷിയാവുകയും അതിന് പുറമെ ഏഴ് സമുദ്രങ്ങള്‍ അതിനെ പോഷിപ്പിക്കുകയും ചെയ്താലും അല്ലാഹുവിന്റെ വചനങ്ങള്‍ എഴുതിത്തീരുകയില്ല. തീര്‍ച്ചയായും അല്ലാഹു പ്രതാപിയും യുക്തിമാനുമാകുന്നു'' (വി.ഖു. 31:26,27).

മനുഷ്യന്‍ തന്റെ പരിമിതികളും സര്‍വശക്തനായ സ്രഷ്ടാവിന്റെ അപാരമായ കഴിവുകളും അറിഞ്ഞ് അംഗീകരിക്കുമ്പോഴാണ് മനുഷ്യജീവിതം ശരിയായ ദിശയില്‍ നീങ്ങുക. ദൈവത്തിന്റെ മുമ്പില്‍ വിനയവും താഴ്മയും പ്രകടിപ്പിക്കാത്ത മനുഷ്യരില്‍ അധിക പേരും അഹങ്കാരികളും മുഷ്‌കന്മാരും നികൃഷ്ടരുമാവുകയാണ് പതിവ്. സ്വഭാവം നിഷ്‌കളങ്കമായിരിക്കണമെന്നും വാഗ്‌വിചാരകര്‍മങ്ങള്‍ അന്യൂനമായിരിക്കണമെന്നും നിഷ്‌കര്‍ഷപുലര്‍ത്താന്‍ മനുഷ്യരെ പ്രേരിപ്പിക്കുന്ന ഏറ്റവും പ്രധാന ഘടകം അത്യുന്നതനായ ദൈവത്തോടുള്ള വിധേയത്വമാകുന്നു. സദാ ദൈവത്തെ സ്മരിക്കു കയും പ്രകീര്‍ത്തിക്കുകയും ചെയ്യുന്ന മനുഷ്യരിലാണ് നന്മയോടു ള്ള പ്രതിബദ്ധത മങ്ങാതെ നിലനില്‍ക്കുന്നത്. ''നീ ഭൂമിയില്‍ അഹന്തയോടെ നടക്കരുത്. തീര്‍ച്ചയായും നിനക്ക് ഭൂമിയെ പിളര്‍ക്കാനൊന്നും കഴിയില്ല. ഉയരത്തില്‍ നിനക്ക് പര്‍വ്വ തങ്ങള്‍ക്കൊപ്പമെത്താനും ആവില്ല; തീര്‍ച്ച'' (വി.ഖു. 17:37). പ്രപഞ്ച നാഥന് യഥാര്‍ഥത്തിലുള്ളതാണ് അളവില്ലാത്ത അറിവും കഴിവും. അവനെ സംബന്ധിച്ചിടത്തോളം ഒരു പൊക്കലിന്റെ ആവശ്യമില്ല. എന്നാല്‍ മനുഷ്യന് സ്വന്തമെന്ന് പറയാന്‍ ഒന്നുമില്ല. അവന്‍ അനുഭവിക്കുന്നതെല്ലാം ദൈവത്തിന്റെ ദാനമാണ്. അത് തുറന്ന് സമ്മതിച്ചുകൊണ്ട് വിനീതമായ ജീവിതം നയിക്കുന്നതിലൂ ടെയാണ് മനുഷ്യന് മഹത്വം കൈവരുന്നത്. ഈ മഹത്വം കരഗതമാക്കുന്നതിന് അവനെ സഹായിക്കുന്ന ഘടകങ്ങളത്രെ പ്രാര്‍ഥനകളും കീര്‍ത്തനങ്ങളും.

വിഷയവുമായി ബന്ധപ്പെട്ട വീഡിയോ

പ്രപഞ്ചവും നിലനില്‍പും എങ്ങനെ യാദൃച്ഛികമായുണ്ടാവും എന്ന് ചോദിക്കുന്നതും സ്രഷ്ടാവായ ദൈവം എങ്ങനെ കാരണമില്ലാതെയുണ്ടാവും എന്ന് ചോദിക്കുന്നതും ഒരുപോലെയല്ലേ? 

  • പ്രപഞ്ചം നിലനില്‍ക്കുന്നു എന്നത് മാത്രമല്ല, ഓരോ പ്രാപഞ്ചിക പ്രതിഭാസവും വ്യവസ്ഥാപിതമാണ് എന്നതും അനിഷേധ്യസത്യമാകുന്നു. അത്യന്തം സൂക്ഷ്മമായ ഒരു പരമാണുവിനകത്ത് ന്യൂട്രോണുകളും പ്രോട്ടോണുകളും ഇലക്‌ട്രോണുകളും വിന്യസിക്കപ്പെട്ടിട്ടുള്ളത് തികച്ചും വ്യവസ്ഥാപിതമായിട്ടാണ്. ആര്‍ക്കും എണ്ണിതിട്ടപ്പെടുത്താന്‍ കഴിയാത്തത്ര ഹൈഡ്രജന്‍ പരമാണുക്കള്‍ ഈ പ്രപഞ്ചത്തിലുണ്ട്. ഇവയുടെയെല്ലാം സൂക്ഷ്മഘടന തികച്ചും സമാനമാകുന്നു. നക്ഷത്രങ്ങളും ഗ്രഹങ്ങളും ഉപഗ്രഹങ്ങളും അടങ്ങിയ സ്ഥൂല പ്രപഞ്ചവും കണിശമായ വ്യവസ്ഥകള്‍ക്ക് വിധേയമായിത്തന്നെയാണ് വര്‍ത്തിക്കുന്നത്. ഭൂമിയും സൂര്യനും തമ്മിലുള്ള നിശ്ചിത അകലവും, സാങ്കല്‍പിക അച്ചുതണ്ടില്‍ ഭൂമിയുടെ ഭ്രമണവും സൂര്യന് ചുറ്റുമുള്ള ഭൂമിയുടെ പരിക്രമണവും എല്ലാം കണിശമായ വ്യവസ്ഥപ്രകാരം നിലനില്‍ക്കുന്നതുകൊണ്ടാണ് ഇവി ടെ ജീവസസ്യജാലങ്ങളുടെ നിലനില്‍പ് സാധ്യമാകുന്നത്.

ഓരോ വിത്തില്‍നിന്നും മുളച്ചുവളരുന്നത് വേര്, കാണ്ഡം, ഇല, പൂവ്, കായ് എന്നിവയുടെ കാര്യത്തില്‍ അതീവ സൂക്ഷ്മമായ സവിശേഷതകളുള്ള സസ്യമാണ്. വര്‍ണങ്ങളിലോ ഗന്ധങ്ങളിലോ രുചികളിലോ യാദൃച്ഛികമായി യാതൊരു മാറ്റവും സംഭവിക്കുന്നില്ല. ഓരോ ജീവവര്‍ഗത്തിന്റെയും പ്രത്യുല്‍പാദനകോശങ്ങള്‍ സംയോജിക്കുമ്പോള്‍ ലക്ഷണമൊത്ത പുതിയ ജീവതലമുറ പിറക്കുന്നു. പുരുഷന്റെ ബീജവും സ്ത്രീയുടെ അണ്ഡവും ചേര്‍ന്ന് ഭ്രൂണം രൂപംകൊണ്ടതിനുശേഷം അത്യന്തം സൂക്ഷ്മമായ വ്യവസ്ഥകള്‍ക്ക് വിധേയമായിക്കൊണ്ടുള്ള കോശവിഭജന പ്രക്രിയകളിലൂടെ മാസ ങ്ങള്‍ക്കുള്ളില്‍ അവയവത്തികവുള്ള കുഞ്ഞായി വളരുന്ന പ്രതിഭാസം തികച്ചും ആസൂത്രിതമായും വ്യവസ്ഥാപിതമായുമാണ് സംഭവിക്കുന്നത്. യുഗാന്തരങ്ങളായി സഹസ്രകോടിക്കണക്കില്‍ മാതാക്കളുടെ ഗര്‍ഭാശയങ്ങളില്‍ ഈ വ്യവസ്ഥാപിത പ്രതിഭാസം അരങ്ങേറിക്കൊണ്ടേയിരിക്കുന്നു. സാധാരണഗതിയില്‍ യാതൊരു താളപ്പിഴയുംകൂടാതെ ഇത് തുടരുന്നു. ആകസ്മികമായി ഈ വ്യവസ്ഥയില്‍ നിസ്സാര മാറ്റങ്ങള്‍ സംഭവിച്ചാല്‍പോലും മനുഷ്യര്‍ വിഷമിച്ചുപോകുമായിരുന്നു. ഉദാഹരണമായി കണ്‍പോളകളിലെ രോമങ്ങള്‍ തലമുടിപോലെ നിരന്തരം വളരാന്‍ തുടങ്ങിയാല്‍ എത്രത്തോളം വിഷമമാകുമായിരുന്നു എന്നാലോചിച്ച് നോക്കുക.

ചരിത്രകാലത്തിനിടയില്‍ ഒരിക്കല്‍പോലും യാദൃച്ഛികമായി തെങ്ങ് വാഴക്കുലയോ വാഴ തേങ്ങാക്കുലയോ ഉല്‍പാദിപ്പിച്ചിട്ടില്ല. ഇനി അങ്ങനെയൊരു ആകസ്മിക സംഭവം ഉണ്ടാകുമെന്ന് ആരും പ്രതീക്ഷിക്കുകയുമില്ല. യാദൃച്ഛികമായി സ്ത്രീയുടെ ശരീരത്തില്‍ തന്നെ പുരുഷ ബീജവും കൂടി ഉല്‍പാദിപ്പിക്കപ്പെടുകയും അണ്ഡ ബീജസങ്കലനം നടന്ന് അവള്‍ ഗര്‍ഭിണിയാവുകയും ചെയ്യുമെന്നും ആരും പ്രതീക്ഷിക്കുകയില്ല. ആകസ്മികമായി ഒരു ഡി.എന്‍.എ. തന്മാത്രയുടെ ഘടനയില്‍ മൗലികമായ മാറ്റമുണ്ടാകുമെന്ന് പോലും കരുതാന്‍ യാതൊരു ന്യായവും കാണുന്നില്ല. യാഥാര്‍ഥ്യം ഇതായിരിക്കെ അനാദിയില്‍നിന്ന് അനന്തതയിലേക്ക് അനുസ്യൂതം പ്രവഹി ക്കുന്ന ഏതോ ആകസ്മികതയായി സൂക്ഷ്മവും സ്ഥൂലവുമായ പ്രാപഞ്ചിക വ്യവസ്ഥയെ വിലയിരുത്തത് തനി അസംബന്ധമാകുന്നു. ആകസ്മികത എന്നാല്‍ വാഹനാപകടം പോലെ അവിചാരിത മായ സ്ഥലത്തും അപ്രതീക്ഷിതമായ സമയത്തും നടക്കുന്ന കാര്യമാണ്. അത് ഒരിക്കലും അനുസ്യൂതമായി പ്രവഹിക്കുകയില്ല. പടച്ചവനെ മാറ്റിനിര്‍ത്താന്‍ വേണ്ടി യുക്തിയുടെ പേരില്‍ ഉന്നയിക്കുന്ന വാദമുഖങ്ങള്‍ മരമണ്ടന്‍ സ്‌റ്റൈലിലുളളതാകുന്നത് ഒട്ടും ഭൂഷ ണമല്ല.

ഇരുമ്പയിരിന്റെ ഒരു വലിയ കൂമ്പാരം ഒരു സ്ഥലത്ത് കിടന്നിട്ട് ആകസ്മിക സംഭവങ്ങളുടെ എത്ര പ്രവാഹങ്ങള്‍ അതിന്റെ മീതെ കടന്നുപോയാലും പതിനായിരം കോടി കൊല്ലങ്ങള്‍ക്ക് ശേഷം പോ ലും അത് തനിയെ ട്രാക്ടറുകളോ തീവണ്ടി എഞ്ചിനുകളോ ആവു കയില്ല എന്ന കാര്യം ഉറപ്പാണ്. വന്‍തോതിലുള്ള ബൗദ്ധിക യത്‌നം കൊണ്ട്മാത്രമെ ഒരു യന്ത്രം രൂപപ്പെടുകയുള്ളൂവെങ്കില്‍ വിസ്മ യങ്ങളില്‍ വിസ്മയമായ മനുഷ്യാസ്തിത്വം സര്‍വജ്ഞനും സര്‍വ ശക്തനുമായ ഒരു സംവിധായകനില്ലാതെ രൂപംകൊള്ളുകയില്ല എന്ന കാര്യം നിഷേധിക്കാനാകാത്ത സത്യംതന്നെയാകുന്നു.

ജിനോമിക്‌സ് പഠനങ്ങള്‍ തെളിയിക്കുന്നത് മനുഷ്യശരീരത്തിലെ കോടിക്കണക്കില്‍ ഡി.എന്‍.എ. തന്മാത്രകളില്‍ ഓരോന്നിലുംരേഖ പ്പെടുത്തിയ വിവരശേഖരം പകര്‍ത്തിവെക്കാന്‍ അനേകം കംപ്യൂട്ടറുകള്‍വേണ്ടിവരുമെന്നാണ്. ഒരു മില്ലി മീറ്ററിന്റെ മില്യനില്‍ ഒരുഭാഗം മാത്രം വലിപ്പമുള്ള ജൈവ ഘടകത്തിലാണ് നാല് രാസപദാര്‍ഥ ങ്ങള്‍ അക്ഷരങ്ങളാക്കിക്കൊണ്ട് ഇത്ര ഭീമമായ വിജ്ഞാനശേഖരം രേഖപ്പെടുത്തിവെച്ചിരിക്കുന്നത്. ഇതൊക്കെ ആകസ്മികമാണെന്ന് പറയുന്നതും സര്‍വജ്ഞനായ മഹാശക്തന്‍ സംവിധാനിച്ചതാണെ ന്ന് പറയുന്നതും ഒരുപോലെയാണെന്ന്, അഥവാ തെറ്റാകാനും ശരി യാകാനും തുല്യസാധ്യതയുള്ളതാണെന്ന് തോന്നുന്നതാണ് മനുഷ്യബുദ്ധിക്ക് സംഭവിക്കാവുന്ന ഏറ്റവും കടുത്ത അപചയം.

സല്‍ബുദ്ധിയുള്ള ഓരോ മനുഷ്യനോടും വിശുദ്ധ ഖുര്‍ആന്‍ ചോ ദിക്കുന്നു: ''ഹേ; മനുഷ്യാ, ഉദാരനായ നിന്റെ രക്ഷിതാവിന്റെ കാര്യത്തില്‍ നിന്നെ വഞ്ചിതനാക്കിയ കാര്യം എന്താണ്? നിന്നെ സൃഷ്ടിക്കുകയും നിന്നെ സംവിധാനിക്കുകയും നിന്നെ ശരിയായ അവസ്ഥയിലാക്കുകയും താനുദ്ദേശിച്ച രൂപത്തില്‍ നിന്നെ സംഘടിപ്പിക്കു കയും ചെയ്തവനത്രെ അവന്‍'' (വി.ഖു. 82:6-8). ''ഭൂമിയിലുള്ള വൃക്ഷമെല്ലാം പേനയായിരിക്കുകയും സമുദ്രം മഷി യാവുകയും അതിന് പുറമെ ഏഴ് സമുദ്രങ്ങള്‍ അതിനെ പോഷിപ്പിക്കുകയും ചെയ്താലും അല്ലാഹുവിന്റെ വചനങ്ങള്‍ എഴുതിത്തീരുകയില്ല. തീര്‍ച്ചയായും അല്ലാഹു പ്രതാപിയും യുക്തിമാനുമാകുന്നു'' (വി.ഖു. 31:27).

ആശാരിയെ നിര്‍മിച്ചത് ആരാണെന്ന ചോദ്യത്തിന് മറുപടി നല്‍കാന്‍ കഴിയില്ല എന്ന് കരുതി ആരും മേശ നിര്‍മിച്ചത് ആശാരിയാ ണെന്ന് പറയാന്‍ മടിക്കാറില്ല. അതുപോലെതന്നെ ദൈവത്തെ ആര് സൃഷ്ടിച്ചുവെന്ന് വല്ലവരും ചോദിച്ചേക്കുമെന്ന് ആശങ്കിച്ച് പ്രപഞ്ചമാ കെ സൃഷ്ടിച്ചത് ദൈവമാണെന്ന് പറയാനും മടിക്കേണ്ടതില്ല. പ്രപഞ്ചമാകെ സൃഷ്ടിച്ചവന്‍ ആരാലും സൃഷ്ടിക്കപ്പെട്ടവനാകാതിരിക്കുക എന്നത് ബൗദ്ധികമായ അനിവാര്യതയാകുന്നു. എല്ലാ പ്രാപഞ്ചികപ്രതിഭാസങ്ങൾക്കും തുടക്കവും ഒടുക്കവുമുണ്ടെന്ന് ശാസ്ത്രം സിദ്ധാന്തിക്കുന്നു. അത് കൊണ്ട് തന്നെ അവയെല്ലാം സൃഷ്ടിക്കപെട്ടതാണ്. തുടക്കക്കാരനില്ലാതെ ഒന്നിനും തുടങ്ങാനാവില്ല. അല്ലാഹുവാകട്ടെ തുടക്കവും ഒടുക്കവുമില്ലാത്തവനാണ്. അത് കൊണ്ട് തന്നെ അവനെ സൃഷ്ടിക്കേണ്ടതില്ല. എന്നെന്നുമുള്ളവൻ സൃഷ്ടിക്കപ്പെടണമെന്ന് കരുതുന്നതാണ് യുക്തിവിരുദ്ധം; അവന് സ്രഷ്ടാവ് വേണ്ടെന്ന് പറയുന്നതല്ല.

വിഷയവുമായി ബന്ധപ്പെട്ട വീഡിയോ

സമൂഹത്തിലെ സമ്പന്നരുടെ സമ്പത്ത് എല്ലാവര്‍ക്കും തുല്യമായി നല്‍കുക എന്ന കമ്യൂണിസ്റ്റ് നയമല്ലേ ഇസ്‌ലാമിലെ ദാന-ധര്‍മ-സകാത്ത് സമ്പ്രാദയത്തെക്കാള്‍ വിശാലമായതും അനുകരിക്കാന്‍ എളുപ്പമുള്ളതും!

സര്‍വലോകരക്ഷിതാവിലുള്ള വിശ്വാസത്താല്‍ പ്രചോദിതനായി മനുഷ്യന്‍ ജീവിത വ്യവഹാരങ്ങളാകെ ക്രമീകരിക്കാനാണ് ഇസ്‌ലാം ആവശ്യപ്പെടുന്നത്. വാഗ്‌വിചാര കര്‍മങ്ങളാകെ ജഗന്നിയന്താവിന് സമര്‍പ്പിക്കുന്നതിന്റെ ഭാഗമായി ധനസമ്പാദനവും ധനവിനിമയവും അവന്റെ ഹിതത്തിന് വിധേയമാക്കുകയാണ് സത്യവിശ്വാസി ചെയ്യുന്നത്. അവന്റെ പ്രീതിയും പ്രതിഫലവുമാണ് അതിലൂടെ വിശ്വാസി കാംക്ഷിക്കുന്നത്. ഭൗതിക ജീവിതത്തെമാത്രം മുന്‍നിര്‍ത്തി രണ്ടു ചിന്തകന്മാര്‍ ആവിഷ്‌കരിച്ച രാഷ്ട്രീയ പ്രത്യയശാസ്ത്രമാണ് കമ്യൂണിസം. അതില്‍ ദൈവവിശ്വാസത്തിനോ പരലോകത്തെ സംബന്ധിച്ച പ്രതീക്ഷയ്‌ക്കോ യാതൊരു സ്ഥാനവുമില്ല.

സമ്പന്നരുടെ സമ്പത്ത് എല്ലാവര്‍ക്കും തുല്യമായി വീതിക്കുക എന്നതല്ല കമ്യൂണിസ്റ്റ് രാഷ്ട്രങ്ങള്‍ ചെയ്തത്. സാമ്പത്തിക സ്രോതസ്സുകളും ഉല്‍പാദനോപാധികളും പൂര്‍ണമായി രാഷ്ട്രത്തിന്റെ ഉടമസ്ഥതയിലാക്കി മാറ്റുകയും അധികാരം കയ്യാളുന്ന സമഗ്രാധിപതികള്‍ തങ്ങളുടെ ഹിതമനുസരിച്ച് അതൊക്കെ കൈകാര്യം ചെയ്യുകയും ചെയ്യുന്ന സമ്പ്രദായങ്ങളാണ് കമ്യൂണിസ്റ്റ് രാഷ്ട്രങ്ങളില്‍ നിലനിന്നു പോന്നത്. പാര്‍ട്ടിയിലെയും ഭരണകൂടത്തിലെയും ഉന്നതന്മാര്‍ക്കും വ്യവസായശാലകളിലെയും കൃഷിയിടങ്ങളിലെയും സാധാരണ തൊഴിലാളികള്‍ക്കും ലഭിക്കുന്ന വേതനവും ജീവിതസൗകര്യങ്ങളും തമ്മില്‍ അവിടങ്ങളില്‍ എക്കാലത്തും വലിയ അന്തരമുണ്ടായിരുന്നു. ഉന്നത ശാസ്ത്രജ്ഞര്‍ക്കും ട്രാക്ടര്‍ ഓടിക്കുന്നവര്‍ക്കും തുല്യമായ ശമ്പളവും ജീവിത സൗകര്യങ്ങളും നല്‍കുന്ന സമ്പ്രദായം കമ്യൂണിസ്റ്റ് രാഷ്ട്രങ്ങളിലൊന്നും ഇതഃപര്യന്തംനടപ്പാക്കിയിട്ടില്ല.

ചൈനയില്‍ മാവോയുടെ നേതൃത്വത്തില്‍ സാംസാകാരിക വിപ്‌ളവം അരങ്ങേറിയപ്പോള്‍ ഉയര്‍ന്ന തസ്തികകളിലുള്ളവരെക്കൊണ്ട് താഴേക്കിട ജോലികള്‍ ചെയ്യിക്കാന്‍ തുടങ്ങിയെങ്കിലും പ്രായോഗികമായി അത് തികഞ്ഞ പരാജയമായിരുന്നു. ഇപ്പോള്‍ ചൈനയുടെ സാമ്പത്തികനയം മുതലാളിത്തത്തില്‍ നിന്ന് ഏറെയൊന്നും വ്യത്യസ്തമല്ലാതായിട്ടുണ്ട്. ഷാങ്ഹായിയും ബെയ്ജിംഗും പോലുള്ള ചൈനീസ് നഗരങ്ങളിലെ വന്‍കിട വ്യാപാരികളുടെയും വ്യവസായികളുടെയും ലാഭത്തില്‍നിന്ന് പാവപ്പെട്ടവര്‍ക്ക് വല്ലതും ലഭിക്കണമെങ്കില്‍ അവര്‍ ദൈവത്തെയോര്‍ത്ത് എന്തെങ്കിലും ദാനം ചെയ്യുകതന്നെ വേണ്ടിവരും.

അല്ലാഹു ത്രികാലജ്ഞാനിയാണ്. സര്‍വജ്ഞനാണ്. അവനോട് ഏതു സമയത്തും നമുക്ക് പ്രാര്‍ഥിക്കാം. പിന്നെ എന്തിനാണ് പ്രത്യേക സമയം നിര്‍ണയിക്കപ്പെട്ട അഞ്ചുനേരത്തെ നിര്‍ബന്ധ നമസ്‌കാരങ്ങള്‍? സുജൂദ്, റുകൂഅ് പോലുള്ള പ്രത്യേകരൂപങ്ങള്‍ എന്തിനാണ് പ്രാര്‍ഥനക്ക്? നമുക്ക് ഇഷ്ടമുള്ള രൂപത്തില്‍ പ്രാര്‍ഥിച്ചാല്‍ പോരെ?

സത്യവിശ്വാസികള്‍ക്ക് അല്ലാഹുവോട് ഏത് സമയത്തും പ്രാര്‍ഥിക്കാം. ന്യായമായ ഏത് കാര്യത്തിനുവേണ്ടിയും പ്രാര്‍ഥിക്കാം. നമസ്‌കാരം എന്ന ആരാധനാകര്‍മത്തിന് മാത്രമാണ് സമയം നിര്‍ണ യിക്കപ്പെട്ടിട്ടുള്ളത്. ഖുര്‍ആന്‍പാരായണവും പ്രാര്‍ഥനയും പ്രകീര്‍ ത്തനവും നിര്‍ത്തവും ഇരുത്തവും കുമ്പിടലും സാഷ്ടാംഗവും എല്ലാം അടങ്ങിയതാണ് നമസ്‌കാരം. ദിനരാത്രങ്ങള്‍ക്കിടയില്‍ നിശ്ചിത ഇടവേളകളില്‍ ഈ ആരാധനാകര്‍മം നിര്‍വഹിക്കാന്‍ വിശ്വാസികളോട് അല്ലാഹു കല്‍പിച്ചിട്ടുള്ളത് അവന്റെ അനുഗ്രഹ ങ്ങളോട് അവര്‍ കൃതജ്ഞരായിരിക്കുന്നതിനും അവന്റെ വിധി വിലക്കുകളെ സംബന്ധിച്ച ബോധം അവരുടെ മനസ്സില്‍ സദാ സജീവമായിരിക്കുന്നതിനും വേണ്ടിയാകുന്നു. ഭൂഗോളത്തിന്റെ വിവിധ ഭാഗങ്ങളിലുള്ള മനുഷ്യര്‍ ഓരോനമസ്‌കാരവും വ്യത്യസ്ത സമയങ്ങളിലായിരിക്കും നിര്‍വഹിക്കുന്നത്. അതിനാല്‍ ഭൂമിയില്‍ നമസ്‌കാരം നിര്‍വഹിക്കപ്പെടാത്ത ഒരു സമയവും ഉണ്ടാകാനിടയില്ല.

ബഹുദൈവത്വത്തിന്റെയോ അന്ധവിശ്വാസങ്ങളുടെയോ യാതെരു ലാഞ്ചനയുമില്ല എന്ന് പറയപ്പെടുന്ന ഇസ്‌ലാമിലെ സുപ്രധാന കര്‍മമായ ഹജ്ജ് വേളയില്‍ 'ഹജറുല്‍ അസ്‌വദ്''എന്ന കല്ലിനെ ആദരവോടും ആവേശത്തോടും മുത്തുന്നതെന്തിനാണ്? കല്ലിനെയും മുള്ളിനെയും ആരാധിക്കരുത്, അവയോടൊന്നും ഒരുതരത്തിലുമുള്ള ആദരവും ബഹുമാനവും പാടില്ലായെന്ന് പഠിപ്പിച്ച നബി(സ)തന്നെ ഈ കര്‍മം നടത്തിയത് എന്തോ ഒരു ബഹുമാനം ആ കല്ലിനോട് ഉള്ളതുകൊണ്ടല്ലേ? നിര്‍ജീവമായ ആ കല്ലിനോട് ഒരു ആരാധനാ മനോഭാവമുള്ളത് കൊണ്ടല്ലേ? ഇന്ന് ജനങ്ങള്‍ ഈ കല്ലിനോട് കാണിക്കുന്ന ആവേശം അതല്ലേ സൂചിപ്പിക്കുന്നത്?

പ്രപഞ്ചനാഥനെ മാത്രം ആരാധിക്കാന്‍വേണ്ടി ഭൂമുഖത്ത് ആദ്യ മായി സ്ഥാപിക്കപ്പെട്ട ആരാധനാലയം എന്നതാണ് പരിശുദ്ധ കഅ്ബയുടെ മഹത്വമെന്ന് വിശുദ്ധ ഖുര്‍ആനില്‍ (3:96) വ്യക്തമാക്കിയിട്ടുണ്ട്. കഅ്ബയെ ലക്ഷ്യമാക്കി ഏകദൈവ വിശ്വാസികളായ തീര്‍ ത്ഥാടകര്‍ പോകുന്നത് ആവേശത്തോടെ തന്നെയാണ്. പക്ഷേ, അ വര്‍ ആരാധിക്കുന്നത് ആ ചതുരക്കെട്ടിടത്തെയല്ല. ലോകരക്ഷിതാവിനെ മാത്രം ആരാധിക്കാനുള്ള സങ്കേതമെന്ന നിലയില്‍ ആഭവനത്തോട് അവര്‍ക്കുള്ളത് ആദരവാണ്, ആരാധനയല്ല.

കഅ്ബയുടെ തെക്കുകിഴക്കെ മൂലയില്‍നിന്നാണ് പ്രദക്ഷിണം തുടങ്ങേണ്ടത്. ആ മൂലയില്‍ സ്ഥാപിച്ചിട്ടുള്ള ഒരു അടയാള കല്ലാണ് ഹജറുല്‍ അസ്‌വദ്. ആകല്ലില്‍ ചുംബിച്ചുകൊണ്ടോ തൊട്ട്മുത്തിക്കൊണ്ടോ അതിനുനേരെ കൈകൊണ്ട് ആംഗ്യം കാണിച്ചു കൊണ്ടോ പ്രദക്ഷിണം തുടങ്ങാം. എന്തായാലും തീര്‍ഥാടകര്‍ ആ കല്ലിന് ആരാധനയര്‍പ്പിക്കുന്നി ല്ല. ആ കല്ലിനോ മറ്റേതെങ്കിലും കല്ലിനോ കെട്ടിടത്തിനോ അഭൗതികമായ എന്തെങ്കിലും കഴിവുണ്ടെന്ന് വിശ്വസിക്കുന്നില്ല. ആ കല്ലില്‍നിന്ന് ഉപകാരം പ്രതീക്ഷിക്കുകയോ ഉപദ്രവം ഭയപ്പെടുകയോ ചെയ്യുന്നില്ല. ആ കല്ലിന്റെയും അത് ഉള്‍ക്കൊള്ളുന്ന കഅ്ബയുടെയും നാഥനായ അല്ലാഹുവെ മാത്രമാണ് അവര്‍ ആരാധിക്കുന്നത്. ആ കല്ലിനെ ആദരിക്കല്‍ ഹജ്ജിന്റെയോ ഉംറയുെടയോ നിര്‍ബന്ധിത കര്‍മ്മങ്ങളില്‍ പെട്ടതല്ല എന്ന കാര്യവും പ്രസ്താവ്യമാകുന്നു. അതിന്റെ നേരെനിന്നുകൊണ്ട് ത്വവാഫ് (പ്രദക്ഷിണം) തുടങ്ങുക മാത്രമാണ് നിര്‍ബന്ധമായിട്ടുള്ളത്.

നിലത്ത് നെറ്റിയും മൂക്കും വെച്ചുകൊണ്ടാണ് സത്യവിശ്വാസികള്‍ സാഷ്ടാംഗം ചെയ്യേണ്ടത്. എന്നാല്‍ മറ്റു ചിലര്‍ പറയുന്ന ഭൂമി പൂജ യുമായി ഇതിന് യാതൊരു ബന്ധവുമിെല്ലന്നും ഭൂമിയുടെ സ്രഷ്ടാവിന്റെ മുമ്പില്‍ പരമമായ വണക്കം പ്രകടിപ്പിക്കുക മാത്രമാണ് ഇ തുകൊണ്ട് ഉദ്ദേശിക്കപ്പെട്ടതെന്നും സത്യവിശ്വാസികള്‍ക്കെല്ലാം അറിയം. അതുപോലെ തന്നെയാണ് ഹജറുല്‍ അസ്‌വദിനെ ആദരിച്ചുകൊണ്ട് അല്ലാഹുവിന് മാത്രമുള്ള ആരാധനയായ ത്വവാഫ് നിര്‍വഹിക്കുന്ന ഏകദൈവ വിശ്വാസിയുടെ അവസ്ഥയും. സഫാ മര്‍വാ കുന്നുകളോടും മക്കയിലെ മറ്റ് പുണ്യസ്ഥലങ്ങളോടും സത്യവിശ്വാസികള്‍ക്കുള്ള മനോഭാവവും ഇതുപോലെതന്നെ.

ലോകത്തിലുള്ള എല്ലാ ദൈവവിശ്വാസികളും ഒരു വിധത്തില്‍ അല്ലെങ്കില്‍ മറ്റൊരുവിധത്തില്‍ വിഗ്രഹാരാധകന്മാരാണ്. അതായത് ഹൈന്ദവ വിശ്വാസികള്‍ വിവിധ ക്ഷേത്രങ്ങളില്‍ വ്യത്യസ്ത രൂപത്തിലുള്ള വിഗ്ര ഹങ്ങള്‍ പ്രതീകമാക്കി ഏകദൈവത്തെ ആരാധിക്കുന്നു. ക്രിസ്ത്യാനികള്‍ യേശുവിനെ വിവിധ ചര്‍ച്ചുകളില്‍ പ്രതീകമാക്കി ദൈവത്തോട് പ്രാര്‍ഥിക്കുന്നു. മുസ്‌ലിംകള്‍ മുഴുവന്‍ 'കഅ്ബ' എന്ന പ്രതീകത്തിന് നേരെ തിരിഞ്ഞ് ഏകദൈവത്തെ ആരാധിക്കുന്നു. ഇതെല്ലാം ഒത്തുചേരുന്നത് എല്ലാ മതങ്ങളും വിഗ്രഹാരാധനയില്‍ (ദൈവിക പ്രതീകാരാധനയില്‍) അധിഷ്ഠിതമാണ് എന്നതിലല്ലേ?

വിഗ്രഹങ്ങളെ പ്രതീകമാക്കി അവയെത്തന്നെയാണ് ഹൈന്ദവര്‍ ആരാധിക്കുന്നതും പ്രാര്‍ഥിക്കുന്നതും. ഓരോ ആവശ്യത്തിനും വി വിധ ദൈവങ്ങള്‍ക്ക് നേര്‍ച്ചകളര്‍പ്പിക്കുകയും അവരോട് പ്രാര്‍ഥിക്കുകയും ചെയ്യുന്നത് ഏകദൈവാരാധനയാണെന്ന് പറയുന്നതില്‍ യാ തൊരര്‍ഥവുമില്ല. ക്രൈസ്തവര്‍ ഏകദൈവാരാധനക്ക് യേശുവെ പ്രതീകമാക്കുകയല്ല; ഒരു ത്രിയേക ദൈവസങ്കല്‍പമുണ്ടാക്കി ദൈവ പുത്രന്‍ എന്ന പേരില്‍ യേശുവെ ആരാധിക്കുകയും അദ്ദേഹത്തോട് പ്രാര്‍ഥിക്കുകയുമാണ് ചെയ്യുന്നത്. ഏകദൈവമായ യഹോവയോട് മാത്രം പ്രാര്‍ഥിക്കുന്നവര്‍ ക്രൈസ്തവര്‍ക്കിടയില്‍ വളരെ വിരളമാ കുന്നു. എന്നാല്‍ കഅ്ബഃ ഏകദൈവത്തിന്റെ പ്രതീകമോ വിഗ്രഹമോ അല്ല. കഅ്ബയെ മുസ്‌ലിംകള്‍ ആരാധിക്കുകയോ കഅ്ബയോട് പ്രാര്‍ഥിക്കുകയോ ചെയ്യുന്നില്ല. ഏകദൈവത്തെ മാത്രം ആരാധി ക്കാന്‍വേണ്ടി, യഹൂദരുടെയും ക്രൈസ്തവരുടെയും മുസ്‌ലിംകളുടെയും പൂര്‍വികാചാര്യനായ പ്രവാചക ശ്രേഷ്ഠന്‍ ഇബ്രാഹീം(അ)അഥവാ അബ്രഹാം സ്ഥാപിച്ച ആരാധനാലയമാണ് കഅ്ബഃ. അദ്ദേഹം അവിടെനിന്ന് ഏകദൈവത്തോട് എങ്ങനെ പ്രാര്‍ഥിച്ചുവോ അതുപോലെ അങ്ങോട്ട് തിരിഞ്ഞുനിന്ന് ഏകദൈവത്തോട് മാത്രം പ്രാര്‍ഥിക്കുകയാണ് മുസ്‌ലിംകള്‍ ചെയ്യുന്നത്. വിഗ്രഹാരാധനയുടെ യാതൊരുസ്പര്‍ശവും യഥാര്‍ഥ മുസ്‌ലിംകളുടെ ആരാധനയിലോ പ്രാര്‍ഥനയിലോ ഇല്ല.

വിഷയവുമായി ബന്ധപ്പെട്ട വീഡിയോ

ഏറ്റവും നല്ല രൂപത്തിലാണ് മനുഷ്യനെ സൃഷ്ടിച്ചതെന്ന് ഖുര്‍ആന്‍ സൂചിപ്പിക്കുന്നു. (95:4) പിന്നെയെന്തിനാണ് കുറ്റമറ്റ ശരീരമുള്ള മനുഷ്യരോട്(പുരുഷന്മാരോട്)ചേലാകര്‍മം ചെയ്യാന്‍ കല്‍പിച്ചത്? ഇതിന് ഉത്തരമായി ഇത് കൊണ്ടുള്ള നേട്ടം നമുക്ക് അണിനിരത്താന്‍ സാധിക്കുമെങ്കിലും അതിന് ഒരു മറുചോദ്യമുണ്ടാകുന്നു. ചേലാകര്‍മംകൊണ്ട് ഇത്രയും ഉപകാരങ്ങള്‍ ഉണ്ടെങ്കില്‍ പിന്നെ എന്തുകൊണ്ടാണ് ചേലാകര്‍മം ചെയ്യാത്ത അവസ്ഥയില്‍ ദൈവം മനുഷ്യനെ സൃഷ്ടിച്ചത്? ദൈവം കുറ്റമറ്റവനും അത്യുന്നതനുമാണ് എന്നതിന് ഇത് ഒരു അപവാദമല്ലേ? അതല്ല, മനുഷ്യന്‍ ഏറ്റവും നല്ല ഘടനയിലല്ലേ സൃഷ്ടിക്കപ്പെട്ടത്?

'ജന്തുജാലങ്ങളില്‍ നിന്ന് വ്യത്യസ്തമായി ശരീരത്തിന്റെ ബാഹ്യഘടനയില്‍ ചില്ലറ മാറ്റങ്ങള്‍ വരുത്താനുള്ള സ്വാതന്ത്ര്യവും കൂടിയുള്ള നിലയിലാണ് ദൈവം മനുഷ്യനെ സൃഷ്ടിച്ചിട്ടുള്ളത്. തലമുടി പൂര്‍ണമായി മുണ്ഡനം ചെയ്യുകയോ, നിശ്ചിത വലിപ്പത്തില്‍ വെട്ടിനിര്‍ത്തുകയോ, ഒന്നും ചെയ്യാതെ ജടപിടിച്ച നിലയില്‍ വിടുകയോ, ചീകിയൊതുക്കുകയോ ചെയ്യാന്‍ അവന് സ്വാതന്ത്ര്യമുണ്ട്. ഒരു പരിധിയില്‍ കവിയാത്തവിധം വെട്ടി ചീകിയൊതുക്കി വെക്കുന്നതാണ് ഇസ്‌ലാം പ്രോത്‌സാഹിപ്പിക്കുന്ന രീതി. മുണ്ഡനം ഒരു ഫാഷനായി സ്വീകരിച്ചവരും ചിലകാലങ്ങളില്‍ ചില നാടുകളില്‍ ഉണ്ടാകാറുണ്ട്. തലയില്‍ ഒട്ടും മുടിവേണ്ടെന്നോ, നിശ്ചിത നീളത്തില്‍മാത്രം വള രുന്ന നിലയില്‍ (ശരീരത്തിന്റെ മറ്റ് പല ഭാഗങ്ങളിലും അങ്ങനെയാണല്ലോ) മതിയെന്നോ ദൈവം തീരുമാനിച്ചിരുന്നെങ്കില്‍ പല അഭിരുചിക്കാര്‍ക്കും അത് അസൗകര്യമായി ഭവിക്കും. നഖത്തിന്റെ കാര്യവും ഏറെ വ്യത്യസ്തമല്ല. നഖം ഒട്ടും മുറിക്കാതെ പലവിധത്തി ല്‍ നീട്ടിവളര്‍ത്തുന്നവര്‍ക്കും വിവിധ ആകൃതികളില്‍ വെട്ടി അല ങ്കരിച്ച് നടക്കുന്നവര്‍ക്കും വിരല്‍ത്തുമ്പിനൊപ്പം മുറിച്ച് ഒഴിവാക്കുന്നവര്‍ക്കും ഒരുപോലെ സൗകര്യപ്രദമായ നിലയിലാണ് ദൈവം സംവിധാനിച്ചിരിക്കുന്നത്.

വ്യത്യസ്ത സാദ്ധ്യതകളോടെ മുടിയും നഖവും സൃഷ്ടിച്ചത് ദൈവത്തിന്റെ കഴിവിന്റെ തെളിവാണെങ്കില്‍, മുറിച്ചു നീക്കുകയോ നിലനിര്‍ത്തുകയോ ചെയ്യാനുള്ള സാധ്യതയോടെ അഗ്രചര്‍മം സൃ ഷ്ടിച്ചത് എങ്ങനെയാണ് കഴിവുകേടോ അപൂര്‍ണതയോ ആകുന്നത്? പുരുഷ ശരീരത്തിലെ ഏറ്റവും മൃദുലവും 'സെന്‍സിറ്റീവും' ആയ ഭാഗമാണ് ലൈംഗികാവയവത്തിന്റെ അഗ്രഭാഗം. വസ്ത്രം ധരിക്കാത്ത അവസ്ഥയില്‍ പരിക്കേല്‍ക്കാന്‍ കൂടുതല്‍ സാധ്യതയുള്ള ആ ഭാഗത്തിന് ഒരു പ്രത്യേക ചര്‍മാവരണം സൃഷ്ടികര്‍ത്താവ് നല്‍കിയത് അവന്റെ കരുതലും കാരുണ്യവും മൂലമാണ്. എന്നാല്‍ വസ്ത്രം ധരിക്കുകയും ശ്രദ്ധയോടെ ജീവിക്കുകയും ചെയ്യുന്നയാളുടെ അവയവത്തിന് അഗ്രചര്‍മം മുഖേനയുള്ള സംരക്ഷണം ആവശ്യമായിവരുന്നില്ല. പ്രായപൂര്‍ത്തിയാകുന്നതോടെ ആ ചര്‍മത്തിന്റെ ഉള്‍ഭാഗം എപ്പോഴും മാലിന്യമുക്തമായി സൂക്ഷിക്കുക പ്രയാസമായിരിക്കും. പലതരം ശരീരദ്രാവകങ്ങളുടെ അംശങ്ങള്‍ ചെറിയ അളവിലെങ്കിലും ആഭാഗത്ത് അവശേഷിക്കുന്നത് അവയവത്തിന്റെ ആരോഗ്യത്തെ പ്രതികൂലമായി ബാധിക്കുമെന്നത് ഇന്ന് ആരോഗ്യശാസ്ത്രജ്ഞന്മാര്‍ക്കിടയില്‍ തര്‍ക്കമില്ലാത്ത വിഷയമാകുന്നു. ദൈവിക മതം ചേലാകര്‍മം അനുശാസിച്ചതിന്റെ പ്രസക്തി നമുക്ക് ഇവിടെ വ്യക്തമാകുന്നു.

ശരീരഘടനയില്‍ വിവിധ സാധ്യതകള്‍ ഉള്‍പ്പെടുത്തുകയും ചിന്തിക്കുന്ന മനുഷ്യര്‍ക്ക് അവയുമായി ബന്ധപ്പെട്ട ഏറ്റവും ഉത്തമമായ സമീപനം കൈക്കൊള്ളാന്‍ മാര്‍ഗദര്‍ശനം നല്‍കുകയും ചെയ്ത ദൈവം എല്ലാം അറിയുകയും എല്ലാം പരിഗണിക്കുകയും ചെയ്യുന്നുവെന്നത്രെ ഇതില്‍നിന്നൊക്കെ ഗ്രഹിക്കാവുന്നത്. ശരീരത്തിലെവിടെയും ബാഹ്യചര്‍മത്തിന് മീതെ മറ്റൊരു ചര്‍മമില്ലാതിരിക്കെ പ്രത്യുല്‍പാദന അവയവത്തിന്റെ അഗ്രഭാഗത്ത് മാത്രം ഒരു ചര്‍മ്മാവരണം നല്‍കപ്പെട്ടത് ഘടനാപരമായ ന്യൂനതയ്ക്കല്ല സൂക്ഷ്മതക്കാണ് തെളിവാകുന്നത്.

ഇസ്‌ലാമിനെക്കുറിച്ച് പഠിക്കാന്‍ ആഗ്രഹിക്കുന്ന ഒരു വ്യക്തിയാണ് ഞാന്‍. പക്ഷേ, ഇസ്‌ലാമിലെ ചില ആചാരരീതികള്‍ വളരെ ക്രൂരമായി തോന്നുന്നു. ഒരു മുസ്‌ലിമിന് ജീവിതത്തില്‍ ഒരിക്കല്‍ (കഴിവുണ്ടെങ്കില്‍) നിര്‍ബന്ധമായും ചെയ്യേണ്ടുന്ന ഏറ്റവും വലിയ പുണ്യകര്‍മമാണ് ഹജ്ജ്. എന്നാല്‍ ഇത്രയും വലിയ ഈ പുണ്യകര്‍മത്തില്‍ ഒരു മിണ്ടാപ്രാണിയെ കൊല്ലുന്നതും പുണ്യമാകുമോ? ഈ പ്രാവശ്യം ചുരുങ്ങിയത് 25 ലക്ഷം തീര്‍ഥാടകര്‍ ഹജ്ജ് ചെയ്യാനെത്തിയെന്നറിഞ്ഞു. എങ്കില്‍ ഒരാള്‍ ഒരാട് വീതമാണെങ്കില്‍പോലും 25 ലക്ഷം ആടുകള്‍ അറുക്കപ്പെട്ടിട്ടുണ്ടാകും. മനുഷ്യര്‍ക്ക് പുണ്യം കിട്ടാന്‍വേണ്ടി ആ മിണ്ടാപ്രാണികളെ എന്തിന് നമ്മള്‍ കൊല്ലണം? അതൊരു വലിയ പാപമാകില്ലേ? എന്റെ ഒരു മുസ്‌ലിം കൂട്ടുകാരനോട് ഇതേപ്പറ്റി ചോദിച്ചപ്പോള്‍ ദൈവത്തിന്റെ കല്‍പനയാണ് ഇതെന്നാണ് പറഞ്ഞത്. ഇബ്‌റാഹീം (അബ്രഹാം) പ്രവാചകനോട് മകനെ ബലി നല്‍കാന്‍ കല്‍പിച്ചുവെന്നും പ്രവാചകന്‍ അത് അനുസരിച്ച് മകനെ അറുക്കുവാന്‍ പോകുമ്പോള്‍ ദൈവം അദ്ദേഹത്തില്‍ സന്തുഷ്ടനായി മകന് പകരം ഒരാടിനെ ബലി നല്‍കുവാന്‍ പറഞ്ഞു എന്നും മറ്റും എന്നോട് പറഞ്ഞു. ഞാന്‍ പ്രതിനിധാനം ചെയ്യുന്ന മതത്തില്‍ ഒരുപാട് ബലികര്‍മങ്ങള്‍ ഉണ്ടായിരുന്നു. കാലാനുസൃതമായി അവയിലെല്ലാം മാറ്റങ്ങള്‍ വരുത്തി. ഇപ്പോഴും മാറ്റങ്ങള്‍ നടന്നുകൊണ്ടിരിക്കുന്നു. അതുപോലെ ഇതും മാറ്റിക്കൂടേ?

ചോദ്യകര്‍ത്താവിന്റെ ദൃഷ്ടിയില്‍ എന്താണ് ക്രൂരത? ഒരു ജീവി മറ്റൊരു ജീവിയെ തിന്നുന്നത് ക്രൂരതയാണോ? ദൈവിക വ്യവസ്ഥയില്‍ (അവിശ്വാസികളുടെ വീക്ഷണ പ്രകാരം പ്രകൃതി വ്യവസ്ഥയില്‍) ഇത് സാര്‍വത്രികമായി കാണപ്പെടുന്നുണ്ട്. ആയിരക്കണക്കിന് ജീവജാതികളുടെ മുഖ്യാഹാരം തന്നെ ഇതര ജന്തുക്കളാണ്. എന്നാല്‍ ഇങ്ങനെ ഇരയാകുന്ന ജന്തുക്കള്‍ക്ക് അതുമൂലം വംശനാശം സംഭവിക്കുന്നില്ല. കാരണം, അവ ഭീമമായ അളവില്‍ ഇവിടെ ജനിച്ച് വളരുന്നുണ്ട്. ജന്തുലോകത്തെ ഈ പ്രതിഭാസംക്രൂരതയാണെന്ന് തോന്നുന്നവര്‍ ഉണ്ടാകാം. പക്ഷേ, അവര്‍ക്ക് പ്രകൃതിയില്‍ വ്യാപകമായി നിലനില്‍ക്കുന്ന ഇരതേടല്‍ വ്യവസ്ഥക്ക് മാറ്റം വരുത്താനാവില്ലല്ലോ.

മനുഷ്യന്‍ മാംസാഹാരം കഴിക്കുന്നതാണ് ക്രൂരതയായി വീക്ഷിക്കുന്നതെങ്കില്‍ നമുക്ക് പറയാനുള്ളത് മാംസഭോജനം ജന്തു പ്രകൃതിയോട് യോജിക്കുന്നതും ലോകരക്ഷിതാവ് അനുവദിച്ചതുമാണ് എന്നാകുന്നു. ''കാലികളെയും അല്ലാഹു സൃഷ്ടിച്ചിരിക്കുന്നു. നിങ്ങള്‍ക്ക് അവയില്‍ തണുപ്പകറ്റാനുള്ളതും (കമ്പിളി) മറ്റ് പ്രയോജനങ്ങളുമുണ്ട്. അവയില്‍നിന്ന് തന്നെ നിങ്ങള്‍ ഭക്ഷിക്കുകയും ചെയ്യുന്നു'' (വി.ഖു. 16:5). ''അല്ലാഹുവാകുന്നു നിങ്ങള്‍ക്കുവേണ്ടി കന്നുകാലികളെ സൃഷ്ടിച്ച് തന്നവന്‍. അവയില്‍ ചിലതിനെ നിങ്ങള്‍ വാഹനമായി ഉപയോഗിക്കുന്നതിനുവേണ്ടി. അവയില്‍ ചിലതിനെ നിങ്ങള്‍ ഭക്ഷിക്കുകയും ചെയ്യുന്നു'' (40:79).

മത്‌സ്യം ഉള്‍പ്പെടെ യാതൊരു ജീവിയെയും മനുഷ്യന്‍ ഭക്ഷിക്കരുതെന്ന് നിഷ്‌കര്‍ഷിച്ചാല്‍ ജനകോടികളുടെ ഉപജീവനം തന്നെ മുടങ്ങിപ്പോകും. മാംസഭോജികള്‍ക്കുവേണ്ടി ആടുമാടുകളെ വളര്‍ത്തുന്നവരും മത്‌സ്യബന്ധനം കൊണ്ട് ജീവിക്കുന്നവരുമൊക്കെ കഷ്ടത്തിലാകും. മനുഷ്യരോടുള്ള ഈ ക്രൂരതയെ അപേക്ഷിച്ച് നോക്കുമ്പോള്‍ ഭക്ഷണത്തിനുവേണ്ടിയുള്ള ജന്തുഹത്യ ഏറെ ഗൗരവമുള്ള വിഷയമല്ല. ആടുകളെ അറുത്ത് ഭക്ഷിക്കുന്നത് ക്രൂരതയായി കാണുന്നില്ലെങ്കില്‍ അവയെ ദൈവത്തിന് ബലിയര്‍പ്പിക്കുന്നത് ക്രൂരതയാണെന്ന് പറയാന്‍ പ്രത്യേക ന്യായമൊന്നുമില്ല. ദൈവത്തിന് വേണ്ടി വെറുതെ ആടുകളെ കൊന്ന് തള്ളുകയല്ല; ബലിമൃഗത്തിന്റെ മാംസം സ്വയം ഭക്ഷിക്കുകയും പാവങ്ങള്‍ക്ക് ഭക്ഷിക്കാന്‍ കൊടുക്കുകയുമാണ് ചെയ്യുന്നത്. ഇത് സംബന്ധിച്ച് വിശുദ്ധ ഖുര്‍ആനില്‍ പറഞ്ഞിട്ടുള്ളത് ഇപ്രകാരമാകുന്നു: ''അവയുടെ മേല്‍ അല്ലാഹുവിന്റെ നാമം ഉച്ചരിച്ചുകൊണ്ട് നിങ്ങള്‍ ബലിയര്‍പ്പിക്കുക. അങ്ങനെ അവ പാര്‍ശ്വങ്ങളില്‍ വീണുകഴിഞ്ഞാല്‍ അവയില്‍നിന്നെടുത്ത് നിങ്ങള്‍ ഭക്ഷിക്കുകയും, (യാചിക്കാതെ) സംതൃപ്തിയടയുന്നവനും ആവശ്യപ്പെട്ടുവരുന്നവനും നിങ്ങള്‍ ഭക്ഷിക്കാന്‍ കൊടുക്കുകയും ചെയ്യുക, നിങ്ങള്‍ നന്ദി കാണിക്കാന്‍ വേണ്ടി അവയെ നിങ്ങള്‍ക്ക് അപ്രകാരം നാം കീഴ്‌പ്പെടുത്തിത്തന്നിരിക്കുന്നു.'' (22:36)

.വിശുദ്ധ ഖുര്‍ആന്‍ അവതരിക്കുന്ന കാലത്ത് അറേബ്യയിലെ ഒരു പ്രധാന വരുമാനമാര്‍ഗമായിരുന്നു കാലിവളര്‍ത്തല്‍. ഇന്നും ലോകത്തിന്റെ നാനാഭാഗങ്ങളില്‍ കോടിക്കണക്കിനാളുകള്‍ കാലിവളര്‍ത്തല്‍ ജീവിതോപാധിയാക്കുന്നുണ്ട്. അല്ലാഹുവിന്റെ അനുഗ്രഹങ്ങളിലൊന്നാണത്. ദാനത്തിലൂടെയോ ബലിയിലൂടെയോ ആണ് അവര്‍ അതിന് അല്ലാഹുവോട് കൃതജ്ഞത പ്രകടിപ്പിക്കേണ്ടത്. ഭുജിക്കാനും ബലിയര്‍പ്പിക്കാനും ജനങ്ങള്‍ ആടുമാടുകളെ ധാരാളമായി വാങ്ങുന്നതിനാലാണ് അവയെ വളര്‍ത്തുന്നത് ആദായകരമായിത്തീരുന്നത്. ആടുമാടുകളെ അറുക്കുന്നത് ക്രൂരതയും അധര്‍മവുമാണെന്ന് വിധി കല്‍പിക്കുകയോ നിയമംമൂലം നിരോധിക്കുകയോ ചെയ്താല്‍ അവയെ വളര്‍ത്താന്‍ ആരും താല്‍പര്യം കാണിക്കുകയില്ല. പാല്‍ ഉല്‍പാദിപ്പിക്കുക എന്ന ലക്ഷ്യത്തിന് വേണ്ടി മാത്രം കാലികളെ വളര്‍ത്തുന്നത് ആദായകരമല്ലാകാവുകയും ക്രമേണ അവയുടെ വംശനാശത്തിന് ഇടയാവുകയും ചെയ്യുന്നപക്ഷം അതായിരിക്കും അവയോടും മാംസ്യം ലഭിക്കേണ്ട മനുഷ്യരോടും കാണിക്കുന്ന ഏറ്റവും വലിയ ക്രൂരത.

അറുക്കപ്പെടുന്ന ജന്തുവിന് പരമാവധി കുറഞ്ഞ വേദന മാത്രം അനുഭവിക്കേണ്ടിവരുന്ന വിധത്തില്‍ നല്ല മൂര്‍ച്ചയുള്ള കത്തികൊണ്ട് വേഗത്തില്‍ അറുക്കണമെന്നാണ് പ്രവാചകന്‍ പഠിപ്പിച്ചിട്ടുള്ളത്. കഴുത്തിലെ ധമനി അറ്റുപോകുന്നതോടെ ശരീരവും മസ്തിഷ്‌കവും തമ്മിലുള്ള ബന്ധം വിഛേദിക്കപ്പെടുന്നതിനാല്‍ വേദന അവസാനിക്കുകയും രക്തം ഒഴുകിപ്പോകുന്നതോടെ മാംസം ശുദ്ധമായിത്തീരുകയും ചെയ്യുന്നു. കഴുത്തിലെ ധമനി അറ്റുപോയതിന് ശേഷം ജന്തു പിടയുന്നത് കടുത്ത വേദന അനുഭവിക്കുന്നതിന്റെ ലക്ഷണമല്ല. പേശികളില്‍ അവശേഷിക്കുന്ന ജൈവോര്‍ജത്തിന്റെ പ്രഭാവം മാത്രമാകുന്നു അത്. ചുരുക്കത്തില്‍, മനുഷ്യരോടും ജന്തുജാലങ്ങളോടും നീതിപുലര്‍ത്തുന്ന നിയമ നിര്‍ദേശങ്ങള്‍ മാത്രമെ പ്രപഞ്ചനാഥന്‍ അന്തിമ വേദഗ്രന്ഥത്തിലൂടെ അന്തിമ പ്രവാചകന്‍ മുഖേന അവതരിപ്പിച്ചിട്ടുള്ളൂ.

വ്രതാനുഷ്ഠാനം പല നിലയ്ക്കും നല്ലതുതന്നെ. എന്നാല്‍ ഒരുമാസം നീണ്ടുനില്‍ക്കുന്ന വ്രതാനുഷ്ഠാനം മനുഷ്യര്‍ക്ക് വിഷമകരമല്ലേ? ഇങ്ങനെ ബുദ്ധിമുട്ടിക്കുന്ന ദൈവം കാരുണ്യവാനാണോ?

പലനിലയ്ക്കും നല്ല കാര്യമാണ് വ്രതമെന്ന് പറയുന്നതും ഒരുമാസം വ്രതം അനുഷ്ഠിക്കണമെന്ന് ആവശ്യപ്പെടുന്നത് ബുദ്ധിമുട്ടിക്കലാണെന്ന് പറയുന്നതും തമ്മില്‍ വൈരുധ്യമുണ്ട്. പലനിലയ്ക്കും നല്ലതായ ഒരു പുണ്യകര്‍മം കൂടുതല്‍ ദിവസം ചെയ്യുമ്പോള്‍ കൂടുതല്‍ നന്മ സ്വായത്തമാക്കാന്‍ അവസരം ലഭിക്കുകയാണല്ലോ ചെയ്യുന്നത്. അതെങ്ങനെയാണ് ബുദ്ധിമുട്ടിക്കലാവുക?

ഇസ്‌ലാമികമായ വിശ്വാസം അല്ലാഹു അഥവാ ഏകദൈവം പരമകാരുണികനായ ലോകരക്ഷിതാവാണെന്നത്രെ. ഒരു നല്ല രക്ഷിതാവിന് മക്കളെക്കൊണ്ട് അവര്‍ക്ക് പ്രത്യക്ഷത്തില്‍ ബുദ്ധിമുട്ടാണെന്ന് തോന്നുന്ന പല കാര്യങ്ങളും-അധ്യയനം, തൊഴില്‍ പരിശീലനം തുടങ്ങിയവ-പതിവായി ചെയ്യിക്കാതിരിക്കാന്‍ പറ്റില്ല എന്ന കാര്യം എല്ലാവര്‍ക്കും മനസ്സിലാക്കാവുന്നതാണ്. മടിയനോ സുഖലോലുപനോ ആയ കുട്ടിക്ക് തന്റെ മാതാപിതാക്കള്‍ സ്ഥിരമായി തന്നെ ബുദ്ധിമുട്ടിക്കുകയാണെന്ന് തോന്നിയേക്കും. എന്നാല്‍ തന്നെ അത്യധികം സ്‌നേഹിക്കുന്ന മാതാപിതാക്കള്‍ ആവശ്യപ്പെടുന്ന കാര്യം അല്‍പം ബുദ്ധിമുട്ടുള്ളതാണെങ്കിലും അത് തനിക്ക് സ്ഥായിയായ ഗുണമുണ്ടാക്കുന്നതായിരിക്കും എന്ന് ആര്‍ജവമുള്ള കുട്ടി മനസ്സിലാക്കും.

മക്കളുടെ ഇഹലോകത്തിലെ നന്മയെ സംബന്ധിച്ച് മാത്രമെ മാതാപിതാക്കള്‍ക്ക് അറിയുകയുള്ളൂ. എന്നാല്‍ ലോകരക്ഷിതാവായ അല്ലാഹുവിന് അനശ്വരമായ പരലോകത്തെ നന്മയെ സംബന്ധിച്ചും വ്യക്തമായി അറിയാം. അതിനാല്‍ മനുഷ്യര്‍ക്ക് രണ്ട് ലോകത്തും ഗുണം ചെയ്യുന്ന മാര്‍ഗനിര്‍ദേശങ്ങളാണ് അല്ലാഹു നല്‍കുന്നത്. നിസ്സാരമായ ബുദ്ധിമുട്ടിന്റെ പേരില്‍ അവന്റെ മാര്‍ഗദര്‍ശനത്തിന്റെ മൂല്യം മനസ്സിലാക്കാതിരിക്കുന്നത് വലിയ നഷ്ടത്തിന് ഇടവരുത്തിയേക്കാം. പരിമിതമായ ജ്ഞാനത്തിന്റെ അടിസ്ഥാനത്തില്‍ ബുദ്ധിമുട്ടാണെന്ന് തോന്നുന്ന കാര്യം ദീര്‍ഘകാലാടിസ്ഥാനത്തില്‍ ഗുണകരമായിത്തീരുന്ന അനുഭവങ്ങള്‍ പലര്‍ക്കും ഉണ്ടാകാറുണ്ട്. പ്രഥമദൃഷ്ട്യാ ഇഷ്ടപ്പെടുന്ന കാര്യങ്ങള്‍ ദീര്‍ഘകാലാടിസ്ഥാനത്തില്‍ ദോഷകരമായി പരിണമിക്കുന്ന സംഭവങ്ങളും അപൂര്‍വമല്ല. ഈ കാര്യം വിശുദ്ധ ഖുര്‍ആന്‍ ചൂണ്ടിക്കാണിച്ചിട്ടുണ്ട്. അതിനാല്‍ സര്‍വജ്ഞനും ത്രികാലജ്ഞാനിയുമായ അല്ലാഹുവിന്റെ മാര്‍ഗദര്‍ശനം അന്യൂനവും പൂര്‍ണ നന്മയിലേക്ക് നയിക്കുന്നതുമായിരിക്കും എന്ന് മനസ്സിലാക്കുന്നതാണ് ശരിയായ യുക്തി.

മനുഷ്യന്റെ സൃഷ്ടിപ്പില്‍ ഒരു ന്യൂനതയും ചൂണ്ടിക്കാണിക്കാനില്ലെങ്കിലും മുതലാളിത്തവും ഭൂപ്രഭുത്വവും രാജാധിപത്യവും അടിമവ്യാപാരവും വെപ്പാട്ടി സമ്പ്രദായവും നിരോധിക്കാത്ത ഇസ്‌ലാമിക ത്വശാസ്ത്രം, ന്യൂനതകളുള്ളതും കാലഹരണപ്പെട്ടതുമായ ഗോത്രനിയമങ്ങളാന്നും പറയുന്നതില്‍ വസ്തുതയില്ലേ?

ദൈവം മനുഷ്യനെ സൃഷ്ടിച്ചത് തികച്ചും അന്യൂനമായിട്ടാണെന്ന വസ്തുത ശക്തിയുക്തം സമര്‍ത്ഥിക്കുന്ന ഏക വേദഗ്രന്ഥമാണ് വിശുദ്ധ ഖുര്‍ആന്‍. മുഹമ്മദ് നബി(ല)ക്ക് ഏറ്റവും ആദ്യമായി ദൈവത്തിങ്കല്‍നിന്ന് ജിബ്രീല്‍  എന്ന മാലാഖ മുഖേന കേള്‍പിക്കപ്പെട്ട അഞ്ച് വചനങ്ങളില്‍ മനുഷ്യനെ ഭ്രൂണത്തില്‍നിന്ന് സൃഷ്ടിച്ച് വളര്‍ത്തി എഴുത്തുകാരനും ജ്ഞാനിയുമാക്കി തീര്‍ക്കുന്ന രക്ഷിതാവിന്റെ നാമത്തില്‍ വായിക്കണം എന്നാണ് ആഹ്വാനം ചെയ്യുന്നത്. മുലപ്പാലിന്റെയും മസ്തിഷ്‌ക കോശങ്ങളുടെയും ഘടനതന്നെ മനുഷ്യന്‍ നാവിലൂടെയും വിരല്‍തുമ്പിലൂടെയും ആശയ പ്രകാശനം നടത്തുകയും ബൗദ്ധികമായ കഴിവുകള്‍ വികസിപ്പിക്കുകയും ചെയ്യുന്നതിന് ഉപയുക്തമായ രീതിയില്‍ പ്രത്യേകമായി സംവിധാനിക്കപ്പെട്ടതാണെന്ന് ആധുനിക ശാസ്ത്രം തെളിയിച്ചുകഴിഞ്ഞിട്ടുണ്ട്.

മനുഷ്യന്‍ ഏത് രാഷ്ട്രീയവ്യവസ്ഥിതിയില്‍ ജീവിച്ചാലും അവനെ സംബന്ധിച്ചേടത്തോളം ഏറ്റവും മൗലികമായിട്ടുള്ളത് അവന്റെ ഘടനാപരമായ സവിശേഷതകളാകുന്നു. ഘടനാപരമായ സാധ്യതകളുടെ സദ്‌വിനിയോഗമോ ദുരുപയോഗമോ ആണ് അവന്റെ ഭാഗധേയത്തെ സംബന്ധിച്ചേടത്തോളം നിര്‍ണായകമാകുന്നത്. ഇസ്‌ലാം എന്ന പദത്തിന്റെ അര്‍ത്ഥം ജീവിതം അല്ലാഹുവിന് സ്വയം സമര്‍പ്പിക്കുക എന്നാണ്. വിശ്വാസത്താല്‍ പ്രചോദിതരായിട്ടാണ് വ്യക്തികള്‍ ഇസ്‌ലാമികമായ അനുഷ്ഠാനങ്ങളും ധര്‍മങ്ങളും നിര്‍വഹിക്കേണ്ടത്. ഇസ്‌ലാമിക ധര്‍മത്തിന്റെ കാതലായ ഭാഗം നീതിയും സദ്ഭാവവും ഔദാര്യവുമാണ്. വിശുദ്ധ ഖുര്‍ആന്‍ പറയുന്നത് നോക്കുക: ''തീര്‍ച്ചയായും അല്ലാഹു കല്‍പിക്കുന്നത് നീതി പാലിക്കാനും കുടുംബബന്ധമുള്ളവര്‍ക്ക് സഹായം നല്‍കാനുമാണ്. അവന്‍ വിലക്കുന്നത് നീചവൃത്തിയും ദുരാചാരവും അതിക്രമവുമാകുന്നു. നിങ്ങള്‍ ചിന്തിച്ചു ഗ്രഹിക്കാന്‍വേണ്ടി അവന്‍ നിങ്ങള്‍ക്ക് ഉപദേശം നല്‍കുന്നു'' (16:90). ഈ സൂക്തമനുസരിച്ച് നീതിക്ക് വിരുദ്ധമായതും അതിക്രമപരമായതും നീചവും ദുഷ്ടവുമായ എല്ലാ നടപടികളും ആചാരങ്ങളും ഒരു മുസ്‌ലിമിന് നിഷിദ്ധമാകുന്നു. ഇതില്‍ കാലഹരണപ്പെട്ട യാതൊരു തത്വവും ഉള്‍പ്പെട്ടിട്ടില്ല. ഇസ്‌ലാമിന്റെ സാമ്പത്തിക ധര്‍മവും നീതിയില്‍ അധിഷ്ഠിതമത്രെ. ചൂഷണവും കൃത്രിമവും കബളിപ്പിക്കലും അടങ്ങിയ എല്ലാ സാമ്പത്തിക ഇടപാടുകളും ഇസ്‌ലാം നിഷിദ്ധമായി വിധിച്ചിരിക്കുന്നു. മുതലാളിത്തത്തിന്റെ ആധാരശിലയായ പലിശയെ ഇസ്‌ലാം കര്‍ശനമായി നിരോധിച്ചിരിക്കുന്നു. ധനം സമ്പര്‍ക്കിടയില്‍ മാത്രം കറങ്ങുന്നതായിരിക്കരുതെന്ന് ഇസ്‌ലാം നിഷ്‌കര്‍ഷിക്കുന്നു. (വി.ഖു. 59:7 നോക്കുക).

വിശുദ്ധ ഖുര്‍ആനിലും നബിവചനങ്ങളിലും നിര്‍ദേശിക്കപ്പെട്ട ധനവിനിമയ നിയമങ്ങളിലെല്ലാം ഈ നിഷ്‌കര്‍ഷത തെളിഞ്ഞുകാണാം. പൊതുധനവും സ്വകാര്യസ്വത്തിന്റെ നിശ്ചിത വിഹിതവും ദരിദ്രര്‍, അഗതികള്‍, അനാഥകള്‍, കടബാധിതര്‍ തുടങ്ങിയ അത്യാവശ്യക്കാര്‍ക്കിടയില്‍ വിതരണം ചെയ്യണമെന്ന് ഖുര്‍ആന്‍ സൂക്തങ്ങളിലും നബിവചനങ്ങളിലും അനുശാസിച്ചിട്ടുണ്ട്. നിര്‍ദിഷ്ട മാര്‍ഗങ്ങളില്‍ ചെലവഴിക്കാതെ ധനം സമാഹരിച്ചുവെക്കുന്നവര്‍ കടുത്ത ശിക്ഷ നേരിടേണ്ടിവരുമെന്ന് വിശുദ്ധ ഖുര്‍ആനില്‍ താക്കീത് നല്‍കിയിട്ടുമുണ്ട്. ''സ്വര്‍ണവും വെള്ളിയും നിക്ഷേപമാക്കിവെക്കുകയും അല്ലാഹുവിന്റെ മാര്‍ഗത്തില്‍ അത് ചെലവഴിക്കാതിരിക്കുകയും ചെയ്യുന്നവരാരോ അവര്‍ക്ക് വേദനയേറിയ ശിക്ഷയെപ്പറ്റി നീ 'സന്തോഷവാര്‍ത്ത' അറിയിക്കുക. നരകാഗ്‌നിയില്‍ അവ ചുട്ടുപഴുപ്പിക്കുകയും എന്നിട്ട് അതുകൊണ്ട് അവരുടെ നെറ്റികളിലും പാര്‍ശ്വങ്ങളിലും മുതുകുകളിലും ചൂടുവെക്കുകയും ചെയ്യുന്ന ദിവസം (അവരോട് പറയപ്പെടും:) നിങ്ങള്‍ നിങ്ങള്‍ക്കുവേണ്ടിതന്നെ നിക്ഷേപിച്ചുവെച്ചതാണിത്. അതിനാല്‍ നിങ്ങള്‍ നിക്ഷേപിച്ചുവെച്ചിരുന്നത് നിങ്ങള്‍ ആസ്വദിച്ചുകൊള്ളുക'' (വി.ഖു. 9:34,35).

സ്വകാര്യ സ്വത്തവകാശം പൂര്‍ണമായി നിരോധിക്കുക എന്ന അന്യായവും അപ്രായോഗികവുമായ നിലപാട് ഇസ്‌ലാം സ്വീകരിച്ചിട്ടില്ല എന്നത് ശരിയാണ്. എന്നാല്‍ മുതലാളിത്തത്തിന്റെ ദുഷ്പ്രവണതകളെയെല്ലാം വിലക്കിക്കൊണ്ട് സന്തുലിതമായൊരു സാമ്പത്തിക സമീപനമാണ് ഇസ്‌ലാം സ്വീകരിച്ചിട്ടുള്ളത്. സ്വകാര്യ സ്വത്തവകാശം നിരോധിച്ചിരുന്ന ചീന ഇപ്പോള്‍ ആ നിലപാട് അപ്രായോഗികമാണെന്ന് ബോധ്യപ്പെട്ട് ഉപേഭൂമിയുടെ മേല്‍ കുത്തകാവകാശം സ്ഥാപിച്ച് സ്വന്തമായി കൃഷി ചെയ്യാതെ കൃഷിഭൂമിയെ ഒരു ചൂഷണോപാധിയാക്കിത്തീര്‍ക്കാന്‍ ഇസ്‌ലാം ആരെയും അനുവദിക്കുന്നില്ല. ഒന്നുകില്‍ സ്വന്തമായി കൃഷി ചെയ്യുകയോ അതല്ലെങ്കില്‍ മറ്റുള്ളവര്‍ക്ക് കൃഷി ചെയ്യാന്‍ ഭൂമി വിട്ടുകൊടുക്കുകയോ ചെയ്യണമെന്നാണ് നബി (സ) കല്‍പിച്ചത്. കാര്‍ഷികോല്‍പന്നങ്ങളുടെ മേല്‍ പത്ത് ശതമാനം സകാത്ത് ചുമത്തിയ ഇസ്‌ലാം ഈ രംഗത്ത് സാമൂഹിക നീതി ഉറപ്പ് വരുത്തിയിരിക്കുന്നു.

ഭരണത്തലവന്റെ പേര് രാജാവെന്നോ ഖലീഫയെന്നോ പ്രസഡന്റെന്നോ പ്രധാനമന്ത്രിയെന്നോ ആകുന്നതല്ല മൗലികമായ വിഷയം. ഭരണത്തില്‍ ഏകാധിപത്യവും അനീതിയും അക്രമവും അഴിമതിയും ഉണ്ടാകാതിരിക്കുകയും ജനാഭിലാഷം മാനിക്കുകയും ചെയ്യുക എന്നതാണ് പ്രധാനം. മദീനയിലെ സത്യവിശ്വാസികളുടെമേല്‍ ഭരണാധികാരമുണ്ടായിരുന്ന നബിതിരുമേനി ഇത്തരം സ്ഥാനപ്പേരുകളൊന്നും സ്വീകരിക്കുകയുണ്ടായില്ല. അദ്ദേഹം സ്വയം വിശേഷിപ്പിച്ചത് അല്ലാഹുവിന്റെ ദാസനായ ദൂതന്‍ എ ന്നായിരുന്നു. അദ്ദേഹത്തിന്റെ ശേഷം ഇസ്‌ലാമിക രാഷ്ട്രത്തിന്റെ ഭരണാധികാരിയായി നിയോഗിക്കപ്പെട്ട അബൂബക്കര്‍(്യ), ഖലീഫത്തുല്‍ റസൂല്‍ (റസൂലിന്റെ പിന്‍ഗാമി) എന്ന പേരിലാണ് അറിയപ്പെട്ടത്. അദ്ദേഹത്തിന്റെ പിന്‍ഗാമി ഉമര്‍ (്യ) 'അമീറുല്‍ മുഅ്മിനീന്‍' (വിശ്വാസികളുടെ നേതാവ്) എന്നാണ് വിളിക്കപ്പെട്ടത്. ഭരണാധികാരിയുടെ സ്ഥാനപ്പേര് എന്തായിരുന്നാലും ജനങ്ങളുമായി കൂടിയാലോചിച്ച് നീതിപൂര്‍വം ഭരണം നടത്തണമെന്ന് വിശുദ്ധ ഖുര്‍ആനിലൂടെ അല്ലാഹു കല്‍പിച്ചു. (5:42, 42:38 എന്നീ സൂക്തങ്ങള്‍ നോക്കുക).

സ്വതന്ത്രമായ സമൂഹത്തില്‍നിന്ന് ബലം പ്രയോഗിച്ച് ആരെയെങ്കിലും അടിമയാക്കി മാറ്റാന്‍ ഇസ്‌ലാം ആര്‍ക്കും അനുവാദം നല്‍കിയിട്ടില്ല. മുസ്‌ലിം യുദ്ധത്തടവുകാരെ ശത്രുക്കള്‍ അടിമകളാക്കി മാറ്റിയിരുന്ന സാഹചര്യത്തില്‍ തത്തുല്യ നടപടി മുസ്‌ലിംകളും സ്വീകരിക്കേണ്ടിവരികയാണുണ്ടായത്. ഇസ്‌ലാം ഇതൊരു സ്ഥായിയായ നിയമമാക്കിയിട്ടില്ല. യുദ്ധത്തടവുകാരുടെ കാര്യത്തില്‍ വിശുദ്ധ ഖുര്‍ആനിലുള്ള നിര്‍ദേശം (47:4) അവരെ നിരുപാധികമോ മോചനദ്രവ്യം വാങ്ങിയോ വിട്ടയക്കണമെന്നാണ്. സ്ത്രീകളായ യുദ്ധത്തടവുകാര്‍ ലൈംഗികമായ അതിക്രമത്തിനും ചൂഷണത്തിനും ഇരയാകുന്നതായാണ് ആധുനിക രാഷ്ട്രങ്ങളില്‍ നിന്നുപോലും റിപ്പോര്‍ട്ട് ചെയ്യപ്പെടുന്നത്. അവര്‍ക്ക് കുടുംബജീവിതത്തിന്റെ സ്വാസ്ഥ്യം നഷ്ടപ്പെടുകയും മക്കളുണ്ടായാല്‍ അവര്‍ക്ക് പിതൃത്വമോ നിയമാനുസൃതമായ അവകാശങ്ങളോ ലഭിക്കാതെ പോവുകയും ചെയ്യുന്നു. ലൈംഗിക അരാജകത്വത്തെ ശക്തിയായി എതിര്‍ക്കുന്ന ഇസ്‌ലാം അടിമത്വം അനുഭവിക്കേണ്ടിവരുന്ന യുദ്ധത്തടവുകാരിക്കും ഉത്തരവാദിത്ത പൂര്‍വകമായ ലൈംഗിക ജീവിതത്തിന് അവസരം നല്‍കിയിരിക്കുന്നു. യജമാനന്‍ ജീവിതപങ്കാളിയായി സ്വീകരിക്കുന്ന അടിമസ്ത്രീക്ക് ഒരു കുഞ്ഞ് പിറക്കുന്നതോടെ അവള്‍ സ്വതന്ത്ര സ്ത്രീയുടേത് പോലുള്ള സ്ഥാനത്തിന് അവകാശിയാവുകയും ആ ബന്ധത്തില്‍ ജനിച്ച കുട്ടിക്ക് കുടുംബത്തില്‍ തുല്യസ്ഥാനവും മറ്റുമക്കള്‍ക്കുള്ളപോലെ അനന്തരാവകാശവും ലഭിക്കുകയും ചെയ്യുമെന്നാണ് ഇസ്‌ലാമിക നിയമം. മറ്റ് സമൂഹങ്ങളിലെ വെപ്പാട്ടി സമ്പ്രദായത്തില്‍നിന്ന് ഏറെ വ്യത്യസ്തമാകുന്നു അടിമസ്ത്രീയെയും മക്കളെയും സ്വതന്ത്ര പൗരന്മാരാക്കി പരിവര്‍ത്തിപ്പിക്കുന്ന ഇസ്‌ലാമിക നിയമം. യുദ്ധത്തടവുകാരെ അടിമകളാക്കേണ്ട സാഹചര്യം ഒഴിവായാല്‍ ഈ പ്രശ്‌നങ്ങളൊക്കെ തനിയെ ഇല്ലാതായിത്തീരുന്നതാണ്.

വിശുദ്ധ ഖുര്‍ആനിലെ അഭിസംബോധനകള്‍ ഏറെയും മനുഷ്യരേ എന്നോ വിശ്വാസികളെ എന്നോ ആണ്. ഏതെങ്കിലുമൊരു ഗോത്രക്കാരെ മാത്രം അഭിസംബോധന ചെയ്യുന്നതോ ഗോത്ര ദുരഭിമാനം വളര്‍ത്തുന്നതോ ആയ യാതൊരു വചനവും ഖുര്‍ആനിലില്ല. ഗോത്രവംശ വിഭാഗീയതകള്‍ക്ക് അതീതമായി വിശ്വമാനവികത ഉയര്‍ത്തിക്കാണിക്കുന്ന ഒരു ഖുര്‍ആന്‍ സൂക്തം നോക്കുക: ''ഹേ, മനുഷ്യരേ, തീര്‍ച്ചയായും നിങ്ങളെ നാം ഒരാണില്‍നിന്നും ഒരു പെണ്ണില്‍നിന്നുമായി സൃഷ്ടിച്ചിരിക്കുന്നു. നിങ്ങള്‍ അന്യോന്യം അറിയേണ്ടതിന് നിങ്ങളെ നാം വിവിധ സമുദായങ്ങളും ഗോത്രങ്ങളും ആക്കുകയും ചെയ്തിരിക്കുന്നു. തീര്‍ച്ചയായും അല്ലാഹുവിന്റെയടുത്ത് നിങ്ങളില്‍ ഏറ്റവും ആദരണീയന്‍ നിങ്ങളില്‍ ഏറ്റവും ധര്‍മ്മനിഷ്ഠപാലിക്കുന്നവനാകുന്നു. തീര്‍ച്ചയായും അല്ലാഹു സര്‍വ്വജ്ഞനും സൂക്ഷ്മജ്ഞാനിയുമാകുന്നു'' (49:13).

യാതൊരു ഗോത്രത്തിനും മറ്റൊരു ഗോത്രത്തേക്കാള്‍ യാതൊരു മഹത്വവുമില്ലെന്ന് ഉറപ്പിച്ച് പറയുന്ന ഇസ്‌ലാമിന്റെ നിയമങ്ങളെ കേവലം ഗോത്ര നിയമങ്ങളാക്കി ചുരുക്കുന്നത് സങ്കുചിത വീക്ഷണക്കാര്‍ക്ക് മാത്രമെ ഭൂഷണമാകൂ.

എല്ലാ മനുഷ്യരെയും സൃഷ്ടിച്ച് അവരുടെ വിധികള്‍ നിശ്ചയിച്ച് നമ്മെ ഭൂമിയിലേക്ക് അയച്ച അല്ലാഹു തന്നെ ചിലരെ നല്ലവരും മറ്റു ചിലരെ ചീത്തവരുമാക്കിയെന്നും അല്ലാഹു തീരുമാനിച്ചവര്‍ മാത്രമെ നന്നാകൂവെന്നും പഠിപ്പിക്കുന്ന ഇസ്‌ലാമിലെ വിധിവിശ്വാസമനുസരിച്ച് പിന്നെ മനുഷ്യർക്ക് തെരെഞ്ഞെടുക്കുവാൻ എന്ത് സാധ്യതയാണുള്ളത്?

അല്ലാഹു സര്‍വജ്ഞനും സര്‍വശക്തനുമാണെന്ന് വിശുദ്ധ ഖുര്‍ആനിലെ അനേകം സൂക്തങ്ങള്‍ നമുക്ക് മനസ്സിലാക്കിത്തരുന്നു. അല്ലാഹുവിന്റെ അറിവ് ഒരിക്കലും തെറ്റാവുകയില്ല. അവന്റെ യാ തൊരു തീരുമാനവും നടപ്പാക്കാതെപോവുകയില്ല. പ്രപഞ്ചത്തില്‍ ചെറുതും വലുതമായ ഏത് കാര്യം സംഭവിക്കുന്നതും അവന്‍ നിശ്ചയിച്ചതനുസരിച്ചാണ്. ഇതില്‍ മാറ്റംവരുത്താന്‍ സൃഷ്ടികളില്‍ ആര്‍ക്കും സാധ്യമല്ല. ''ഭൂമിയിലോ നിങ്ങളുടെ ദേഹങ്ങളില്‍ തന്നെയോ യാതൊരാപത്തും ബാ ധിക്കുകയുണ്ടായിട്ടില്ല; അതിനെ നാം ഉണ്ടാക്കുന്നതിനു മുമ്പ് തന്നെ ഒരു രേഖയില്‍ ഉള്‍പെട്ടുകഴിഞ്ഞതായിട്ടല്ലാതെ. തീ ര്‍ച്ചയായും അത് അല്ലാഹുവെ സംബന്ധിച്ചേടത്തോളം എളുപ്പമുള്ളതാകുന്നു'' (വി.ഖുര്‍ആന്‍ 57:22).

അല്ലാഹുവിന്റെ തീരുമാനമാണ് പരമവും നിര്‍ണായകവുമെന്ന് വ്യക്തമാക്കുന്ന ഖുര്‍ആനില്‍ തന്നെ മനുഷ്യരുടെ അഭിപ്രായ-പ്രവര്‍ത്തന സ്വാതന്ത്ര്യത്തെ സംബന്ധിച്ച് അല്ലാഹു വ്യക്തമായി പ്രതിപാദിച്ചി ട്ടുണ്ട്. ''പറയുക. സത്യം നിങ്ങളുടെ രക്ഷിതാവിങ്കല്‍ നിന്നുള്ളതാകുന്നു. അതിനാല്‍ ഇഷ്ടമുള്ളവര്‍ വിശ്വസിക്കട്ടെ. ഇഷ്ടമുള്ളവര്‍ അവിശ്വസിക്കട്ടെ. അക്രമികള്‍ക്ക് നാം നരകാഗ്നി ഒരുക്കി വെച്ചിട്ടുണ്ട്'' (18:29).

അല്ലാഹുവിന്റെ കാലാതീതമായ അറിവോ തീരുമാനമോ, ശരിയും തെറ്റും തെരഞ്ഞെടുക്കാനും സ്വന്തം ഹിതമനുസരിച്ച് പ്രവര്‍ത്തിക്കാനുമുള്ള മനുഷ്യന്റെ സ്വാതന്ത്ര്യത്തിന് തടസ്സമാവുകയില്ല എന്നത്രെ ഇതില്‍നിന്ന് വ്യക്തമാകുന്നത്. നന്മ തെരഞ്ഞെടുക്കാനും സ്വീകരിക്കാനുമുള്ള സ്വാതന്ത്ര്യവും സൗകര്യവും ഉണ്ടായിട്ടും അന്യായമായി തിന്മയിലേക്ക് നീങ്ങിയതിനാലാണ് മനുഷ്യര്‍ നരകശിക്ഷ അനുഭവിക്കേണ്ടിവരുന്നതെന്നും ഈ സൂക്തം സൂചിപ്പിക്കുന്നു.

ഞങ്ങള്‍ ബഹുദൈവാരാധകരായത് അങ്ങനെ അല്ലാഹു മുന്‍കൂട്ടി നിശ്ചയിച്ചതുകൊണ്ടാണ് എന്ന് വാദിക്കുന്നവരെ വിശുദ്ധ ഖുര്‍ആനില്‍ അല്ലാഹു ഖണ്ഡിക്കുകയാണ് ചെയ്തിട്ടുള്ളത്. ''ആ ബഹു ദൈവാരാധകര്‍ പറഞ്ഞേക്കും; ഞങ്ങളോ ഞങ്ങളുടെ പിതാക്കളോ (അല്ലാഹുവോട് ഇതര ദൈവങ്ങളെ) പങ്കുചേര്‍ക്കുമായിരുന്നില്ല; ഞങ്ങള്‍ യാതൊന്നും നിഷിദ്ധമാക്കുമായിരുന്നുമില്ല എന്ന്. ഇതേപ്രകാരം അവരുടെ മുന്‍ഗാമികളും, നമ്മുടെ ശിക്ഷ ആസ്വദിക്കുന്നതുവരെ നിഷേധിച്ചുകളയു കയുണ്ടായി. പറയുക: നിങ്ങളുടെ പക്കല്‍ വല്ല വിവരവുമുണ്ടോ? ഉണ്ടെങ്കില്‍ ഞങ്ങ ള്‍ക്ക് നിങ്ങള്‍ അതൊന്ന് വെളിപ്പെടുത്തിത്തരൂ. ഊഹത്തെമാത്രമാണ് നിങ്ങള്‍ പി ന്തുടരുന്നത്. നിങ്ങള്‍ അനുമാനിക്കുക മാത്രമാണ് ചെയ്യുന്നത്'' (6:148). ഈ വിഷയം 16:35 സൂക്തത്തിലും പരാമര്‍ശിച്ചിട്ടുണ്ട്.

നമ്മെ സംബന്ധിച്ച് അല്ലാഹു ഉദ്ദേശിച്ചത് ഇന്നവിധത്തിലാണെന്ന് അറിയാന്‍ യാതൊരു മാര്‍ഗവുമില്ല. അത് സംബന്ധിച്ച നമ്മുടെ ഊഹവും അനുമാനവും ശരിയാകണമെന്നില്ല. എന്നാല്‍ സത്യം പറയാനും അസത്യം പറയാനുമുള്ള സ്വാതന്ത്ര്യം നാം ഇവിടെ അനുഭവിക്കുന്നു എന്നത് അനിഷേധ്യ സത്യമാണ്. അതിന്റെ അടിസ്ഥാനത്തില്‍ സ്രഷ്ടാവും മാര്‍ഗദര്‍ശകനുമായ അല്ലാഹുവോടുള്ള ബാധ്യത നിറവേറ്റിയാല്‍ യാതൊരു ആശങ്കയ്ക്കും സ്ഥാനമില്ല. സര്‍വജ്ഞനായ അല്ലാഹുവിന്റെ അറിവും തീരുമാനവും നമുക്ക് പൂര്‍ണമായി ഉള്‍കൊള്ളാന്‍ കഴിയണമെന്ന് ശഠിക്കുന്നത് ശരിയല്ല. നമ്മുടെ പരിമിതികള്‍ അറിഞ്ഞും അംഗീകരിച്ചും കൊണ്ട് നമ്മുടെ മുമ്പിലുള്ള സാധ്യതകള്‍ കണ്ടെത്തുകയാണ് ശരിയായ മാര്‍ഗം.

വിഷയവുമായി ബന്ധപ്പെട്ട വീഡിയോ

സ്വന്തം അറിവിന്റെയും യുക്തിയുടെയും ശാസ്ത്രത്തിന്റെയും വെളിച്ചത്തില്‍ ചിന്തിച്ചാൽ തന്നെ സ്രഷ്ടാവുണ്ടെന്നും സൃഷ്ടിപൂജയുടെ നിരര്‍ഥകതയും ബോധ്യപ്പെടുമെന്നിരിക്കെ തമ്മിൽ തല്ലാൻ വേണ്ടി മാത്രം എന്തിനാണ് മനുഷ്യന് മതങ്ങളും ജാതികളും? ദൈവത്തോട് പ്രാര്‍ഥിക്കാന്‍ മതത്തിന്റെ ആവശ്യമുണ്ടോ? വിശ്വാസങ്ങളും ആചാരങ്ങളുമെല്ലാം മനുഷ്യസൃഷ്ടിയല്ലേ?

ലോകരക്ഷിതാവ് കാലാകാലങ്ങളിൽ നിയോഗിച്ച പ്രവാചകന്മാരും അവരുടെ യഥാര്‍ഥ ശിഷ്യന്മാരും സ്വീകരിച്ച വിശ്വാസമാണ് ശരിയെന്നാണ് മുസ്‌ലിംകള്‍ കരുതുന്നത്. പ്രവാചകന്മാരാരും ഈ വിശ്വാസം ജനങ്ങളുടെമേല്‍ അടിച്ചേല്‍പിച്ചിട്ടില്ല. വിശ്വസിക്കാത്തവരെ തല്ലാനും കൊല്ലാനും മുതിര്‍ന്നിട്ടുമില്ല. അതിനാല്‍ ഈ വിശ്വാസം കലഹത്തിന് നിമിത്തമാവുകയില്ല . അല്ലാഹു പ്രവാചകന്മാര്‍ക്ക് അവതരിപ്പിച്ചുകൊടുത്ത വേദഗ്രന്ഥങ്ങളെല്ലാം അവയുടെ സാക്ഷാല്‍ രൂപത്തില്‍ സത്യമാണെന്നാണ് മുസ്‌ലിംകള്‍ വിശ്വസിക്കുന്നത്. ഒട്ടും സങ്കുചിതത്വമില്ലാത്ത വിശ്വാസമാണിത്. ഒരു പ്രവാചകനില്‍ വിശ്വസിക്കുകയും മറ്റൊരു പ്രവാചകനെ തള്ളിക്കളയുകയും ചെയ്യുന്നവന്‍ യഥാര്‍ഥ മുസ്‌ലിമല്ല. അതുപോലെതന്നെയാണ് ഒരു വേദഗ്രന്ഥത്തില്‍ വിശ്വസിക്കുകയും മറ്റൊരു വേദഗ്രന്ഥത്തെ തള്ളിക്കളയുകയും ചെയ്യുന്നവന്റെ സ്ഥിതിയും.

മനുഷ്യന്റെ അറിവിനും യുക്തിക്കും ശാസ്ത്രത്തിനുമെല്ലാം അതാതിന്റേതായ മൂല്യമുണ്ട്. എന്നാല്‍ സര്‍വജ്ഞനായ ലോകരക്ഷിതാവിന്റെ അറിവ്‌പോലെ ഒട്ടും തെറ്റുപറ്റാത്ത അവസ്ഥ മനുഷ്യരുടെയൊന്നും അറിവിനില്ല. ഇറാഖിലെ അമേരിക്കന്‍ അധിനിവേശം തികച്ചും ന്യായമാണെന്ന് കരുതുന്നവരും തികച്ചും അന്യായവും തെറ്റുമാണെന്ന് കരുതുന്നവരുമുണ്ട്; അറിവും യുക്തിയും ശാസ്ത്രബോധവുമെല്ലാമുള്ള ആളുകളുടെ കൂട്ടത്തില്‍. വിവാഹം പവിത്രവും മൗലികവുമാണെന്ന് കരുതുന്നവരും, അത് കാലഹരണപ്പെട്ട ഒരു പഴഞ്ചന്‍ ഏര്‍പ്പാടാണെന്ന് കരുതുന്നവരും ഏറെയുണ്ട് വലിയ ബുദ്ധിമാന്മാരുടെ കൂട്ടത്തില്‍. വിവാഹത്തിന് പുറത്തുള്ള സ്വതന്ത്രരതിയെയും പ്രകൃതിവിരുദ്ധ രതിയെയും മ്ലേഛമായിഗണിക്കുന്നവരും അതിനെയൊക്കെ ന്യായീകരിക്കുന്നവരും ശാസ്ത്രബോധമുള്ളവരുടെ കൂട്ടത്തില്‍തന്നെയുണ്ട്. ഈ കാരണത്താല്‍ സത്യവും അസത്യവും ആത്യന്തികമായി നിര്‍ണയിക്കുന്നതിന്, പരിമിതികള്‍ പലതുമുള്ള മനുഷ്യന്റെ ജ്ഞാനത്തെയും യുക്തിയെയും ശാസ്ത്രത്തെയും അവലംബിക്കുന്നത് അന്യൂനമായ മാര്‍ഗമല്ലെന്ന് വ്യക്തമാകുന്നു.

ജാതിയെയും മതത്തെയും ഇണചേര്‍ത്തുകൊണ്ടാണ് പല വിമര്‍ശകരും സംസാരിക്കാറുള്ളത്. ഇത് ശരിയല്ല. മതങ്ങൾ മനുഷ്യനിര്‍മിതമാണ്.  മതങ്ങളില്‍ സാക്ഷാല്‍ ലോകരക്ഷിതാവായ ദൈവം തന്റെ ദൂതന്മാര്‍ മുഖേനയും വേദഗ്രന്ഥങ്ങളിലൂടെയും അറിയിച്ച കാര്യങ്ങളും പുരോഹിതന്മാരും മറ്റും സ്വന്തം വകയായി കൂട്ടിച്ചേര്‍ത്ത അധികപ്പറ്റുകളുമുണ്ട്. മതവും ജാതിയും ഒന്നല്ല. ജാതിചിന്തകൊണ്ട് മനുഷ്യര്‍ക്ക് എക്കാലത്തും ദോഷം മാത്രമെ ഉണ്ടായിട്ടുള്ളൂ. ഉയര്‍ന്ന ജാതിക്കാര്‍ അഹങ്കാരികളും അതിക്രമകാരികളുമായിത്തീരുകയും അതവര്‍ക്ക് പലവിധത്തില്‍ ദോഷകരമായിത്തീരുകയും ചെയ്തിട്ടുണ്ട്. താഴ്ന്ന ജാതിക്കാര്‍ക്കൊക്കെ മിക്കപ്പോഴും അവഹേളനവും പീഡനവും ഏറ്റുവാങ്ങേണ്ടിവരികയാണുണ്ടായിട്ടുള്ളത്.ജാതി കൊണ്ട് മാനവരാശിക്ക് നാശം മാത്രമേയുണ്ടായിട്ടുള്ളൂ; മതം അങ്ങനെയല്ല.  ദൈവദൂതന്മാര്‍ പഠിപ്പിച്ച സാക്ഷാല്‍ മതം മാനവരാശിക്ക് എപ്പോഴും അളവറ്റ നന്മ നേടിക്കൊടുത്തിട്ടുണ്ട്.

'മനുഷ്യമതം' അബദ്ധങ്ങളില്‍നിന്ന് മുക്തമാവുകയില്ല. സ്ഥലകാല പരിമിതികള്‍ക്കുള്ളില്‍ കഴിയുന്ന മനുഷ്യന്‍ എത്ര വലിയ പ്രതിഭാശാലിയാണെങ്കിലും അവന്‍ സ്വയം ആവിഷ്‌കരിക്കുന്ന മതം പ്രമാദമുക്തമാവുകയില്ല. മറ്റുള്ളവര്‍ക്കെല്ലാം അതിനോട് യോജിക്കാന്‍ കഴിയുകയുമില്ല. ലോകം കണ്ട ഏറ്റവും വലിയ ബുദ്ധിമാന്മാരില്‍ പലരുടെയും സ്വഭാവ സമീപനങ്ങളിലെ വൈകല്യങ്ങള്‍ ചരിത്രകാരന്മാരും സാമൂഹ്യശാസ്ത്രജ്ഞരും ചൂണ്ടിക്കാണിച്ചിട്ടുണ്ട്. ആയതിനാല്‍ ത്രികാലജ്ഞാനിയായ, മനുഷ്യജീവിതത്തിന്റെ എല്ലാ തലങ്ങളെയും സംബന്ധിച്ച് സമ്പൂര്‍ണ ജ്ഞാനമുള്ള ലോകരക്ഷിതാവിന്റെ മാര്‍ഗദര്‍ശനം പിന്തുടരുന്നത് അനുപേക്ഷ്യമാകുന്നു. സാക്ഷാല്‍ ലോകരക്ഷിതാവ് പഠിപ്പിച്ച വിശ്വാസങ്ങളും ആചാരങ്ങളുമുണ്ട്. അതൊക്കെ ഗുണകരവും മോക്ഷദായകവുമാണ്. പുരോഹിതന്മാരും മറ്റും കെട്ടിച്ചമച്ച അന്ധവിശ്വാസങ്ങളും അനാചാരങ്ങളും ഇതില്‍നിന്ന് വ്യത്യസ്തമാണ്. അവയെ അനുധാവനം ചെയ്യുന്നവര്‍ ദുര്‍മാര്‍ഗത്തിലാണ് ചെന്നെത്തുക.

വിഷയവുമായി ബന്ധപ്പെട്ട വീഡിയോ
ഇസ്‌ലാമിമിന്റേത് കുറ്റമറ്റ തത്ത്വശാസ്ത്രമാണെങ്കില്‍ എന്തുകൊണ്ട് മുസ്‌ലിംകള്‍ അത് പൂര്‍ണമായും പ്രയോഗവത്കരിക്കുന്നതില്‍ പരാജയപ്പെടുന്നു? ഇസ്‌ലാമിന്റെ സിദ്ധാന്തങ്ങളില്‍ പലതും അപ്രായോഗികമായതുകൊണ്ടല്ലേ മുസ്‌ലിംകള്‍ അവയെ അവഗണിക്കുന്നത്?
  • ഭക്ഷ്യവസ്തുക്കളിലും ഉപഭോഗ സാധനങ്ങളിലും മരുന്നുകളില്‍പോലും വ്യാപകമായി മായം ചേര്‍ക്കപ്പെടുന്നുണ്ടല്ലോ. എല്ലാ സമുദായങ്ങളില്‍പെട്ടവരും മതവിശ്വാസമില്ലാത്തവരും മായക്കാരുടെ കൂട്ടത്തിലുണ്ട്. ഭക്ഷണത്തിലെ മായം നിമിത്തം ജനലക്ഷങ്ങള്‍ രോഗികളായിത്തീരുന്നു. മരുന്നിലെ മായം നിമിത്തം മരിച്ചവരും നിത്യരോഗികളായവരും ഏറെയുണ്ട്. ഇതിന്റെ അടിസ്ഥാനത്തില്‍ മായം നിഷിദ്ധമാണെന്ന തത്ത്വം അപ്രായോഗികമാണെന്നും തള്ളിക്കളയേണ്ടതാണെന്നും ചിന്താശീലമുള്ള ആരും പറയാനിടയില്ല. മുസ്‌ലിം സമുദായത്തിലെ ഇസ്‌ലാമിക പ്രതിബദ്ധതയില്ലാത്ത ആളുകള്‍ അന്ധവിശ്വാസങ്ങളെ അനുധാവനം ചെയ്യുകയോ അധാര്‍മിക വൃത്തികളില്‍ ഏര്‍പ്പെടുകയോ ചെയ്യുന്നത് ഇസ്‌ലാമിക തത്ത്വങ്ങള്‍ അപ്രായോഗികമായതുകൊണ്ടല്ല; ഉല്‍കൃഷ്ടമായ ഒരു ജീവിതത്തിന്റെ മൗലികതയും മഹനീയതും അവര്‍ മനസ്സിലാക്കാത്തതുകൊണ്ടാണ്. പ്രപഞ്ചനാഥനെ മാത്രം ആരാധിക്കുകയും അവനോട് മാത്രം പ്രാര്‍ഥിക്കുകയും ചെയ്യുക എന്നതാണ് ഇസ്‌ലാമിന്റെ സുപ്രധാന തത്ത്വം. മാനവരാശിയുടെ ആരംഭം മുതല്‍ ഇന്നേവരെ അനേകകോടി വിശ്വാസികള്‍ ഈ തത്ത്വം മുറുകെ പിടിച്ചിട്ടുണ്ട്. പ്രത്യക്ഷവും പരോക്ഷവുമായ ദുര്‍വൃത്തികള്‍ വര്‍ജിക്കുകയും സദ്ഗുണങ്ങളും സദ്പ്രവൃത്തികളും ശീലിക്കുകയും ചെയ്യുക എന്നതാണ് മറ്റൊരു പ്രധാന തത്ത്വം. ഇതും മുറുകെ പിടിക്കുന്ന ജനകോടികളുണ്ട്. യാതൊരു തെറ്റും വീഴ്ചയും സംഭവിക്കരുതെന്ന് ഇസ്‌ലാം ശഠിക്കുന്നില്ല. ബോധപൂര്‍വം ചെയ്യുന്ന കുറ്റകൃത്യങ്ങള്‍ ഒഴികെയുള്ള മനുഷ്യസഹചമായ തെറ്റുകുറ്റങ്ങള്‍ അല്ലാഹു പൊറുക്കുമെന്നാണ് ഇസ്‌ലാം പഠിപ്പിക്കുന്നത്. മനുഷ്യനെ സംബന്ധിച്ച് സൂക്ഷ്മമായി അറിയുന്ന കരുണാവാരിധിയായ ജഗന്നിയന്താവ് മനുഷ്യജീവിതം എല്ലാവിധത്തിലും വിജയകരമായിത്തീരേണ്ടതിന് വേണ്ടി നല്‍കിയ മാര്‍ഗദര്‍ശനം അപ്രായോഗികമാകുന്ന പ്രശ്‌നമേയില്ല.
ഹിന്ദുക്കള്‍ ഇബ്‌റാഹീം നബിയുടെ സമൂഹത്തിന്റെ പിന്‍ഗാമികളാണെന്നതിന് പല തെളിവുകളും ചില ചരിത്രകാരന്മാരും പണ്ഡിതന്മാരും ചൂണ്ടിക്കാണിക്കുന്നുണ്ട്. ബ്രാഹ്മണര്‍ എന്ന പദമുണ്ടായത് ഇബ്‌റാഹീം എന്ന പദത്തില്‍ നിന്നാണ്, കഅ്ബയുടെ ഘടനയിലും പ്രാചീന ഹിന്ദു ദേവാലയങ്ങളുടെ ഘടനയിലുമുള്ള സാമ്യത, ഇബ്‌റാഹീം നബിയുടെ പിതാവിന്റെ തൊഴില്‍ വിഗ്രഹ നിര്‍മാണവും പരിപാലനവുമായിരുന്നു; ബ്രാഹ്മണരും പാരമ്പര്യമായി അതേ തൊഴില്‍ ചെയ്യുന്നു, ഇബ്‌റാഹീം നബിയെ തീക്കുണ്ഠത്തിലിട്ടു; അമ്പലങ്ങളിലും തീക്കുണ്ഠമുണ്ടാക്കി അതില്‍ ചാടുന്ന ആചാരമുണ്ട്, കഅ്ബയിലെയും ക്ഷേത്രങ്ങളിലെയും പ്രദക്ഷിണങ്ങളിലെ സാമ്യത, ബ്രാഹ്മണരുടെ പൂണൂല്‍ ധാരണവും ഹജ്ജ് ചെയ്യുന്ന മുസ്‌ലിംകളുടെ ഇഹ്‌റാം കെട്ടലും തുടങ്ങിയ പലതുമാണ് അതിനുള്ള തെളിവുകൾ . ഈ കാര്യങ്ങള്‍ പരിഗണിച്ചുകൊണ്ടുള്ള പ്രബോധന പ്രവര്‍ത്തനങ്ങള്‍ ഹിന്ദുക്കളുമായുള്ള സൗഹൃദം വര്‍ധിപ്പിക്കാന്‍ വഴിയൊരുക്കില്ലേ? ! ഹിന്ദുക്കള്‍ ഇബ്‌റാഹീം നബി(അ)യുടെ സമൂഹത്തിന്റെ പിന്‍ഗാമികളാണെന്ന് പിലര്‍ ചൂണ്ടിക്കാണിച്ചിട്ടുണ്ടെങ്കിലും ചരിത്രപരമായി സാധൂകരിക്കുവാന്‍ തക്ക തെളിവുകളുടെ അഭാവംകൊണ്ട് പ്രസ്തുത നിരീക്ഷണങ്ങളെ നരവംശ ഗവേഷകരാരും തന്നെ ഗൗരവതരമായി എടുത്തതായി ശ്രദ്ധയില്‍ പെട്ടിട്ടില്ല. ശബ്ദ സാദൃശ്യത്തിന്റെ മാത്രം വെളിച്ചത്തില്‍ ബ്രാഹ്മണ ശബ്ദം ഇബ്‌റാഹീം എന്ന പദത്തില്‍നിന്ന് നിഷ്പന്നമായതാണെന്ന് പറയാന്‍ കഴിയില്ല. ബ്രാഹ്മണ ശബ്ദത്തില്‍നിന്നാണ് ബ്രാഹ്മണന്‍ ഉണ്ടായിരിക്കുന്നത്. ബ്രഹ്മാവ് സ്രഷ്ടാവാണ്. ബ്രഹ്മാവിന്റെ മുഖത്തില്‍നിന്നുണ്ടായവന്‍ ബ്രാഹ്മണന്‍. സ്രഷ്ടാവിന്റെ ഭൂമിയിലെ പ്രതിനിധിയാണവന്‍. ഇങ്ങനെയാണ് ബ്രാഹ്മണ ശബ്ദത്തിന്റെ വിശദീകരണം പോകുന്നത്. ഇബ്‌റാഹീം നബി(അ)യുടെ പിതാവിന്റെ തൊഴില്‍ വിഗ്രഹ നിര്‍മാണമായിരുന്നുവെന്നത് നേര്. അതിന്നെതിരെ പോരാടിയ ആളാണ് ഇബ്‌റാഹീം നബി(അ). പ്രദക്ഷിണം, ദേവാലയ ഘടന തുടങ്ങിയ കാര്യങ്ങളില്‍ ഒരുവിധം എല്ലാ മതസമൂഹങ്ങളിലും ഒരുതരം ഏകീഭാവം കാണാനാവും. മതങ്ങളെല്ലാം പരിണമിച്ചത് ദൈവികമതത്തില്‍നിന്നായതുകൊണ്ടാണിത്. ഇബ്‌റാഹീം നബി(അ)യെ എറിഞ്ഞ തീക്കുണഠം നശീകരണത്തിന്റേതാണ്. അമ്പലങ്ങളിലെ തീക്കുണ്ഠം ആരാധനയുടേതാണ്. തീക്കുണ്ഠത്തില്‍ ചാടുന്ന സമ്പ്രദായമല്ല; തീക്കനലിലൂടെ നടക്കുന്ന സമ്പ്രദായമാണ് ചില ഹിന്ദു സമൂഹങ്ങളിലുള്ളത്. പാര്‍സി മതത്തിന്റെ അഗ്‌നിപൂജാ സമ്പ്രദായത്തില്‍നിന്ന് കടം കൊണ്ടതാണ് ഇത്തരം ആചാരങ്ങളെന്നാണ് പ്രാമാണികരായ നരവംശ ശാസ്ത്രജ്ഞന്മാരില്‍ പലരുടെയും പക്ഷം.അസത്യങ്ങളുടെയോ അര്‍ധ സത്യങ്ങളുടെയോ അകമ്പടിയോടുകൂടിയല്ല സത്യമതപ്രബോധനം നിര്‍വഹിക്കേണ്ടത്. സത്യമെന്ന് നൂറുശതമാനം ബോധ്യമുള്ളള കാര്യങ്ങളാണ് പ്രബോധകന്മാര്‍ അവതരിപ്പിക്കേണ്ടത്. മറ്റു മതക്കാരുമായി സാഹോദര്യമുണ്ടാക്കുന്നതിന് നമുക്ക് ഉറപ്പില്ലാത്ത കാര്യങ്ങള്‍ പറഞ്ഞ് പ്രചരിപ്പിക്കുക വഴി കഴിയുമെന്ന് ഈ ലേഖകന്‍ വിശ്വസിക്കുന്നില്ല. ഇസ്‌ലാം പഠിപ്പിച്ച രീതിയില്‍ സത്യമത പ്രബോധനം നിര്‍വഹിക്കുകയും ഒപ്പംതന്നെ ഇസ്‌ലാമികമായ സ്വഭാവ മര്യാദകള്‍ പാലിച്ചൂകൊണ്ട് മറ്റുള്ളവരുമായി സൗഹൃദവും സ്‌നേഹബന്ധവും സ്ഥാപിക്കുവാന്‍ കഴിയുമെന്നാണ് അനുഭവങ്ങള്‍ നമ്മെ പഠിപ്പിക്കുന്നത്.

ക്രൂരവും രക്തരൂക്ഷിതവുമാണല്ലോ യുദ്ധങ്ങളെല്ലാം. മാനവികതയുടെ മതമാണെന്ന് അവകാശപ്പെടുന്ന ഇസ്‌ലാം യുദ്ധം അനുവദിച്ചത് അതിന്റെ ഈ അവകാശവാദം പൊള്ളയാണെന്നല്ലേ വ്യക്തമാക്കുന്നത്?

യുദ്ധം എക്കാലത്തുമുണ്ടായിരുന്നു. മാനവ സംസ്‌കാരത്തിന്റെ ആദ്യകാലം മുതല്‍ അത് നിലനിന്നിരുന്നു. യുദ്ധം നിലനിന്നിരുന്ന സമൂഹങ്ങളിലേക്കാണ് ദൈവിക മാര്‍ഗദര്‍ശനങ്ങള്‍ക്കനുസരിച്ച് പ്രസ്തുത സമൂഹങ്ങളിലെ ജനങ്ങളെ നയിക്കുവാനായി പ്രവാചകന്‍മാര്‍ നിയോഗിക്കപ്പെട്ടത്. മനുഷ്യരെ നേരായ പാതയിലൂടെ നയിക്കാന്‍ ശ്രമിച്ച ദൈവദൂതന്‍മാര്‍ യുദ്ധരംഗത്തേക്കും ആവശ്യമായ മാര്‍ഗനിര്‍ദ്ദേശങ്ങള്‍ നല്‍കിയിരിക്കണം. അക്രമത്തിന്റെയും അധര്‍മത്തിന്റെയും കറ പുരളാതെ യുദ്ധരംഗത്തെ നിലനിര്‍ത്താനാവശ്യമായ, അവര്‍ നല്‍കിയ നിര്‍ദ്ദേശങ്ങള്‍ പ്രകാരം നടന്ന യുദ്ധങ്ങള്‍ വഴി നന്മ പുലരുകയും തിന്മ തകരുകയും ചെയ്തിരിക്കണം. സ്വന്തം നാടുകളില്‍ നിന്ന് പുറത്താക്കപ്പെട്ട ഇസ്രാഈല്യര്‍ യുദ്ധം ചെയ്തു വിജയിക്കുകയും അങ്ങനെ അത് ദാവൂദിന്റെ രാജാധിപത്യത്തിലേക്ക് നയിക്കുകയും ചെയ്ത സംഭവം ക്വുര്‍ആന്‍ വിശദീകരിക്കുന്നുണ്ട്. (2:246)

സ്വന്തം താമസസ്ഥലത്ത് നിന്നും സന്തതികള്‍ക്കിടയില്‍ നിന്നും പുറത്താക്കപ്പെട്ട ഇസ്രാഈല്യർ യുദ്ധം ചെയ്തപ്പോൾ അവര്‍ക്ക് ദൈവിക സഹായം ഉണ്ടായി എന്നും പ്രാര്‍ത്ഥനയോടു കൂടി അവര്‍ യുദ്ധത്തിനിറങ്ങിയപ്പോൾ അഹങ്കാരിയായ ജാലൂത്തിനെ വധിക്കാന്‍ കഴിഞ്ഞുവെന്നും ഖുർആൻ പറയുന്നുണ്ട്. (2:250,251) താലൂത്തിന്റെ നേതൃത്വത്തില്‍ ജാലൂത്തിനെതിരെ നടന്ന യുദ്ധത്തെയും അതില്‍വെച്ച് ദാവീദ് ജാലൂത്തിനെ വധിച്ചതിനെയും അങ്ങനെ അദ്ദേഹം ഇസ്രാഈല്യരുടെ രാജാവായിത്തീര്‍ന്നതിനെയും പറ്റി പരാമര്‍ശിച്ചുകൊണ്ട് ഈ വിഷയം അല്ലാഹു അവസാനിപ്പിക്കുന്നത് ‘മനുഷ്യരില്‍ ചിലരെ ചിലര്‍മ മുഖേന അല്ലാഹു തടുക്കുന്നില്ലായിരുന്നുവെങ്കില്‍ ഭൂലോകം കുഴപ്പത്തിലാകുമായിരുന്നു. പക്ഷേ, അല്ലാഹു ലോകരോട് വളരെ ഉദാരനത്രെ’ എന്ന് പ്രസ്താവിച്ചുകൊണ്ടാണെന്ന കാര്യം പ്രത്യേകം ശ്രദ്ധേയമാണ്. മുഹമ്മദ് നബി(സ)ക്ക് യുദ്ധാനുമതി നല്‍കിക്കൊണ്ടുള്ള ആദ്യ കല്‍പനയിലും ‘മനുഷ്യരില്‍ ചിലരെ മറ്റു മനുഷ്യരെക്കൊണ്ട് തടുക്കുന്നില്ലായിരുന്നുവെങ്കില്‍ സന്യാസിമഠങ്ങളും, ക്രിസ്തീയദേവാലയങ്ങളും, യഹൂദദേവാലയങ്ങളും, അല്ലാഹുവിന്റെ നാമം ധാരാളമായി പ്രകീര്‍ത്തിക്കപ്പെടുന്ന മുസ്‌ലിം പള്ളികളും തകര്‍ക്കപ്പെടുമായിരുന്നു’ (22:40) എന്ന് പറയുന്നതായി കാണാം.

പ്രവാചകന്‍മാരിലൂടെ പഠിപ്പിക്കപ്പെട്ട യുദ്ധങ്ങളുടെ ലക്ഷ്യം മനുഷ്യരില്‍ ചിലരെ മറ്റു ചിലരെക്കൊണ്ട് തടഞ്ഞുനിര്‍ത്തിക്കൊണ്ട് അവര്‍ നടത്തുന്ന മര്‍ദ്ദനങ്ങള്‍ക്കും പീഡനങ്ങള്‍ക്കും അറുതിവരുത്തി ഭൂമിയെ കുഴപ്പത്തില്‍നിന്ന് സംരക്ഷിക്കുവാനും ആരാധനാ സ്വാതന്ത്ര്യവും പ്രബോധന സ്വാതന്ത്ര്യവും സ്ഥാപിച്ചെടുക്കുകയുമായിരുന്നുവെന്നാണ് ഈ വചനങ്ങള്‍ വ്യക്തമാക്കുന്നത്. യുദ്ധം ചെയ്ത പ്രവാചകന്‍മാരെല്ലാം ഈയൊരു ലക്ഷ്യത്തിനുവേണ്ടി തന്നെയായിരിക്കണം ആയുധമെടുത്തത്. പ്രവാചകന്‍മാര്‍ അതിക്രമകാരികളോ പീഡകരോ ആയിരുന്നില്ല, മനുഷ്യരുടെ വിമോചകരായിരുന്നു. അവര്‍ ചെയ്ത യുദ്ധങ്ങളെല്ലാം, അതുകൊണ്ട് തന്നെ മനുഷ്യരെ പീഡനങ്ങളില്‍ നിന്നും മോചിപ്പിക്കാന്‍ വേണ്ടിയുള്ളതായിരുന്നു. മുഹമ്മദ് നബി (സ) നയിച്ച യുദ്ധങ്ങളെടുത്ത് പരിശോധിച്ചാൽ നമുക്ക് കാണാനാവുക മനുഷ്യവിമോചനമെന്ന ഈ ഉന്നത ലക്ഷ്യമായിരുന്നു; ആളുകളെ കൂട്ടക്കൊല ചെയ്യാനോ സാമ്രാജ്യം സ്ഥാപിക്കുവാനോ വേണ്ടിയുള്ളതായിരുന്നില്ല പ്രസ്തുത യുദ്ധങ്ങളൊന്നുമെന്ന വസ്തുത സത്യസന്ധമായി ചരിത്രം പരിശോധിക്കുന്ന ആർക്കും ബോധ്യപ്പെടും.

പ്രവാചകനിൽ നിന്ന് മതം പഠിച്ച ശിഷ്യന്മാരും പിൽക്കാല മുസ്ലിംകളുമെല്ലാം യുദ്ധം ചെയ്തത് വലിയ മാനവികലക്ഷ്യങ്ങളോട് കൂടിയായിരുന്നു. അതുകൊണ്ടാണ് ഭരണാധികാരികളുടെ പീഡനങ്ങളാൽ പൊരുതി മുട്ടിയിരുന്ന ജനങ്ങൾ മുസ്ലിം ഭരണാധികാരികളെ തങ്ങളുടെ നാടുകളിലേക്ക് ക്ഷണിച്ചു വരുത്തിയ സംഭവങ്ങളുണ്ടായത്. ഇങ്ങനെ പോയ മുസ്ലിം പടയാളികളാണ് മാന്യവും മാനവികവുമായി എങ്ങനെ യുദ്ധം ചെയാമെന്ന് ലോകത്തെ പഠിപ്പിച്ചത്.

കുരിശുയുദ്ധങ്ങളുടെ കാര്യമെടുക്കുക. മുസ്‌ലിം ലോകത്ത് നടത്തിയ ക്രൂരതകളുടെ ഭീതിപപെടുത്തുന്ന കഥകളാണ് ഓരോ കുരിശു യുദ്ധത്തിനും പറയാനുളളത്. നിയമങ്ങളോ നീതിയോ ധാര്‍മികതയോ ഇല്ലാത്ത കാടന്‍ യുദ്ധങ്ങളാണ് കുരിശുയുദ്ധക്കാര്‍ നടത്തിയത്. ഇവയിൽ ലക്ഷക്കണക്കിന് മനുഷ്യര്‍ കൊല്ലപ്പെട്ടു. മരിച്ചവരുടെ ശവശരീരങ്ങളെ കുരിശു പടയാളികള്‍ വികൃതമാക്കി. എട്ടും പൊട്ടും തിരിച്ചറിയാത്ത പൈതങ്ങളെപ്പോലും കൊല്ലുന്നതില്‍ നിന്ന് അവരെ പിന്‍തിരിപ്പിക്കുവാന്‍ 'ഒരു മുഖത്തടിച്ചാല്‍ മറ്റേ മുഖവും കാണിച്ചു കൊടുക്കണം' എന്ന ബൈബിള്‍ പാഠത്തിന്റെ അടിസ്ഥാനത്തില്‍ ജനങ്ങളെ നയിക്കുന്നവരെന്ന് അവകാശപ്പെടുന്ന സഭാനേതൃത്വം സന്നദ്ധമായില്ല. ക്രിസ്തുവിന്റെ പേരില്‍ ആരോപിക്കപ്പെട്ട, അദ്ദേഹത്തിന്റെതല്ലാത്ത ഉപദേശം അപ്രായോഗികവും അക്രമികളെ സഹായിക്കുന്നതുമാണെന്ന വസ്തുതയ്ക്ക് സഭാനേതൃത്വം തന്നെ സ്വയം തെളിവായിത്തീരുകയായിരുന്നു എന്ന് പറയുന്നതാവും ശരി. അതല്ലായിരുന്നുവെങ്കില്‍ ഈ കുരിശുയുദ്ധങ്ങളുടെപേരില്‍ ഏഴു നൂറ്റാണ്ടുകള്‍ക്ക് ശേഷം ലോകത്തെ പരമോന്നത ക്രിസ്ത്യാനിക്ക് കുമ്പസരിക്കേണ്ടി വരികയില്ലായിരുന്നല്ലോ!

ആയുധങ്ങളെടുത്ത് ആളുകളെ കൂട്ടക്കൊല ചെയ്യുക മാത്രമാണ് യുദ്ധമെന്ന് പഠിച്ച കുരിശുയോദ്ധാക്കള്‍ക്ക് മാന്യമായി എങ്ങനെ യുദ്ധം ചെയ്യണമെന്ന് പഠിപ്പിച്ചത് സുല്‍ത്താന്‍ സ്വലാഹുദ്ദീന്‍ അയ്യൂബിയായിരുന്നു. നീണ്ട രണ്ട് നൂറ്റാണ്ടു കാലത്തെ കുരിശു യോദ്ധാക്കളുടെ ദുര്‍ഭരണത്തില്‍ നിന്ന് മസ്ജിദുല്‍ അക്വ്‌സയെയും ജറുസലേമിനേയും മോചിപ്പിച്ചത് അദ്ദേഹത്തിന്റെ കീഴിലുളള അബ്ബാസിയ്യാ മുസ്‌ലിം പടയാളികളായിരുന്നു. 1187 ജൂലൈ നാല് ശനിയാഴ്ച നടന്ന ഹത്തീന്‍ യുദ്ധത്തില്‍ കുരിശു പടയാളികള്‍ പരാജയപ്പെടുകയും ജറുസലേമിന്റെ അധികാരം സ്വലാഹുദ്ദീന്റെ കൈകളില്‍ വന്നു ചേരുകയും ചെയ്തു. കുരിശുയോദ്ധാക്കളുടെ നേതാക്കളായ ഗൈ രാജാവും സോഹദരങ്ങളുമടക്കം നിരവധി പേരെ ബന്ധികളാക്കി സ്വലാഹുദ്ദീനുമുന്നില്‍ ഹാജരാക്കപ്പെട്ടു. അവര്‍ക്കെല്ലാം തണുത്ത പാനീയം നല്‍കി സ്വീകരിക്കുവാനാണ് സ്വലാഹുദ്ദീന്‍ തന്റെ സേവകന്മാരോട് ആവശ്യപ്പെട്ടത്. സ്വലാഹുദ്ദീനെയും മുസ്‌ലിം പടയാളികളെയും വഞ്ചിച്ചതിന് രാജസഹോദരനായ റെയ്‌നാള്‍ഡും മറ്റു ചിലരും മാത്രമാണ് വധിക്കപ്പെട്ടത്. ഗൈ രാജാവടക്കമുള്ളവരെ ആദ്യം ജയിലിലടയ്ക്കുകയും പിന്നെ വെറുതെ വിടുകയുമാണ് ചെയ്തത്. കുരിശുയോദ്ധാക്കള്‍ ജറുസലേമില്‍ നിന്ന് പുറത്താക്കിയ ജൂതന്മാരുടെ പിന്‍ഗാമികളെ വരുത്തി അവിടെ പുനരധിവസിപ്പിക്കുകയാണ് ഭരണമേറ്റെടുത്തയുടനെ സ്വലാഹുദ്ദീന്‍ ചെയ്തത്. മനുഷ്യ രക്തം നീന്തി രക്തദാഹം തീര്‍ക്കുകയും പൈതങ്ങളെയടക്കം കൊന്നൊടുക്കുകയും ചെയ്ത് ജെറുസലേം പിടിച്ചടക്കിയവര്‍ക്ക് അത് തിരിച്ചു പിടിച്ചശേഷം യുദ്ധനേതൃത്വത്തിലുണ്ടായിരുന്നവര്‍ക്കടക്കം മാപ്പ് നല്‍കി വിട്ടയച്ചുകൊണ്ട് എന്തായിരിക്കണം യുദ്ധമര്യാദ എന്ന് പഠിപ്പിക്കുന്നതായിരുന്നു സ്വലാഹുദ്ദീന്റെ നടപടി.

സ്വലാഹുദ്ദീന്‍ അയ്യൂബി ജറുസലേം പിടിച്ചടക്കിയതറിഞ്ഞ പോപ്പ് അര്‍ബന്‍ മൂന്നാമന്‍ മാര്‍പാപ്പ ഹൃദയാഘാതം മൂലം മരണപ്പെട്ടു. ജറുസലേം നഷ്ടപ്പെട്ടതറിഞ്ഞ ക്രിസ്ത്യന്‍ യൂറോപ്പ് ഞെട്ടി. പുതുതായി സ്ഥാനമേറ്റെടുത്ത പോപ്പ് ഗ്രിഗറി എട്ടാമന്‍ മാര്‍പാപ്പ ജറുസലേമിന്റെ നഷ്ടം ക്രൈസ്തവരുടെ പാപങ്ങള്‍ക്കുള്ള ശിക്ഷയാണെന്നും അത് തിരിച്ചു പിടിച്ചാല്‍ മാത്രമെ പ്രസ്തുത പാപങ്ങള്‍ക്ക് പരിഹാരമാവുകയുള്ളൂ എന്നും പ്രഖ്യാപിച്ചു. പരസ്പരം യുദ്ധത്തിലായിരുന്ന ഇംഗ്ലണ്ടിലെ ഹെന്‍ട്രി രണ്ടാമനും ഫ്രാന്‍സിലെ ഫിലിപ്പ് രണ്ടാമനും വൈര്യങ്ങള്‍ വെടിഞ്ഞ് ഐക്യപ്പെടാനും സ്വലാഹുദ്ദീനെതിരെ ഒരുമിച്ചു പോരാടി ജറുസലേം വീണ്ടെടുക്കുവാനും തീരുമാനിച്ചു. സലാദിന്‍ നികുതി (Saladin tithe) എന്ന പേരില്‍ രണ്ടു നാട്ടിലുമുള്ള പൗരന്മാരില്‍ നിന്ന് ചുങ്കം പിരിച്ച് യുദ്ധത്തിനുള്ള സമ്പത്ത് സമാഹരിച്ചു. 1189 ജൂലൈ ആറിന് ഇംഗ്ലണ്ടിലെ രാജാവായ ഹെന്‍ട്രി രണ്ടാമന്‍ മരണപ്പെട്ടതോടെ യുദ്ധനേതൃത്വം മകന്‍ സിംഹഹൃദയനായ റിച്ചാര്‍ഡ് (Richard the lion hearted) എന്നറിയപ്പെട്ട റിച്ചാര്‍ഡ് ഒന്നാമന്റെ ചുമതലയായിത്തീര്‍ന്നു. റിച്ചാര്‍ഡ് ഒന്നാമന്റെയും ഫിലിപ്പ് രണ്ടാമന്റെയും നേതൃത്വത്തിലുള്ള കുരിശുയോദ്ധാക്കളുടെ സൈന്യത്തോട് സ്വലാഹുദ്ദീന്‍ ജയില്‍ മോചിതനാക്കിയ ഗൈ രാജാവും ഒന്നിക്കുകയും ഒരു വലിയ സൈന്യം സ്വലാഹുദ്ദീനെ അക്രമിക്കാനെത്തുകയും ചെയ്തു. വഴിയില്‍ വച്ച് നടന്ന അക്രമങ്ങളിലൂടെ സ്ത്രീകളും കുട്ടികളും അടക്കമുള്ള മുസ്‌ലിംകളെ കൊന്നുകൂട്ടിക്കൊണ്ടായിരുന്നു കുരിശ് സൈന്യം മുന്നേറിയത്. 1191 സെപ്റ്റംബര്‍ ഏഴിന് ജെറുസലേം പിടിച്ചടക്കുക എന്ന ലക്ഷ്യത്തോടെ സിംഹഹൃദയനായ റിച്ചാര്‍ഡിന്റെ നേതൃത്വത്തിലുള്ള സൈന്യം ഫലസ്തീനിലുള്ള അല്‍സൂഫില്‍ വച്ച് സ്വലാഹുദ്ദീനോട് ഏറ്റുമുട്ടിയെങ്കിലും അദ്ദേഹത്തിന് ലക്ഷ്യം നേടാന്‍ കഴിഞ്ഞില്ല. നിരന്തരമായുണ്ടായ യുദ്ധങ്ങള്‍ക്ക് വിരാമമുണ്ടായത് 1192 സെപ്റ്റംബര്‍ രണ്ടിന് സ്വലാഹുദ്ദീനും റിച്ചാര്‍ഡും തമ്മിലുണ്ടാക്കിയ സന്ധിയോടുകൂടിയാണ്. ജറുസലേം മുസ്‌ലിംകളുടെ നിയന്ത്രണത്തിലായിരിക്കുമെന്നും എന്നാല്‍ നിരായുധരായ ക്രൈസ്തവരെ അവിടെയുള്ള ക്രിസ്ത്യന്‍ പുണ്യസ്ഥലങ്ങള്‍ സന്ദര്‍ശിക്കാന്‍ അനുവദിക്കുമെന്നും ആയിരുന്നു കരാര്‍. പുണ്യസ്ഥലങ്ങള്‍ സന്ദര്‍ശിക്കുന്നതില്‍ നിന്ന് മുമ്പും സ്വലാഹുദ്ദീന്‍ ക്രൈസ്തവരെ വിലക്കിയിട്ടില്ല. എന്നതുകൊണ്ടു തന്നെ കരാര്‍ വഴി കുരിശുയോദ്ധാക്കള്‍ക്ക് ഒന്നും നേടാനായില്ലെങ്കിലും, 1192 ഒക്‌ടോബര്‍ ഒന്‍പതിന് റിച്ചാര്‍ഡ് ജറുസലേമില്‍ നിന്ന് മടങ്ങിയത് യുദ്ധരംഗത്ത് പാലിക്കപ്പെടേണ്ട മര്യാദകളെന്തൊക്കെയാണെന്ന് പഠിച്ചുകൊണ്ടായിരുന്നു. ശത്രുവിനെ സ്‌നേഹിക്കേണ്ടത് എങ്ങനെയാണെന്ന് സഭയില്‍ നിന്ന് പഠിക്കാത്ത അദ്ദേഹത്തിന് അത് പഠിപ്പിച്ചത് സ്വലാഹുദ്ദീന്‍ അയ്യൂബിയുടെ യുദ്ധരംഗത്തെ വര്‍ത്തനങ്ങളായിരുന്നു.

യുദ്ധരംഗത്തായിരിക്കുമ്പോള്‍ പോലും ശത്രുവിന്റെ മാനുഷികമായ ആവശ്യങ്ങള്‍ നിര്‍വ്വഹിച്ചു കൊടുക്കുവാന്‍ സ്വലാഹുദ്ദീന്‍ അയ്യൂബി സന്നദ്ധമായി. റിച്ചാര്‍ഡിന് പനി പിടിച്ചപ്പോള്‍ തന്റെ വൈദ്യനെ അയച്ച് ചികില്‍സിക്കുകയും മരുന്ന നല്‍കുകയും നല്ല നല്ല പഴങ്ങളും ശുദ്ധമായ തണുത്ത ജലവും കൊടുത്തയക്കുകയും ചെയ്തത് സ്വലാഹുദ്ദീനായിരുന്നു. യുദ്ധത്തിനിടയില്‍ റിച്ചാര്‍ഡിന് കുതിര നഷ്ടപ്പെട്ടപ്പോള്‍ പകരം രണ്ടു കുതിരകളെ നല്‍കിയതും അദ്ദേഹം തന്നെ. അതുകൊണ്ട് തന്നെ കുരിശുയോദ്ധാക്കള്‍ക്കിടയില്‍ പോലും സ്വലാഹുദ്ദീന്‍ നായകനായിത്തീര്‍ന്നു. തന്റെ സഹോദരിയും സിസിലിയിലെ രാജ്ഞിയുമായ ജോനിനെ സ്വലാഹുദ്ദീന്റെ സഹോദരന് വിവാഹം ചെയ്തു കൊടുക്കുവാന്‍ സന്നദ്ധമാണന്ന് അറിയിച്ചുകൊണ്ടാണ് അദ്ദേഹം ജറുസലേം വിട്ടത്. കാടനായി വന്നയാളെ മാന്യനാക്കിത്തീര്‍ത്ത് തിരിച്ചയച്ചത് ശത്രുവിനെ സ്‌നേഹിക്കണമെന്ന പ്രവാചകന്‍മാരുടെ നിര്‍ദ്ദേശം യഥാവിധി പാലിക്കുവാന്‍ സ്വലാഹുദ്ദീന്‍ അയ്യൂബി തയ്യാറായതു കൊണ്ടായിരുന്നു.

യുദ്ധഭൂമികളില്‍ പൈശാചികമായ രക്തദാഹവുമായി ഉറഞ്ഞു തുള്ളുകയാണ് മതധര്‍മ്മമെന്ന് പഠിപ്പിക്കപ്പെട്ട ക്രൈസ്തവ പടയാളികള്‍ക്ക് മനുഷ്യപക്ഷത്തുനിന്ന് യുദ്ധം ചെയ്യേണ്ടതെങ്ങനെയെന്ന് കാണിച്ചുകൊടുക്കാന്‍ ഒരു സ്വലാഹുദ്ദീന്‍ വേണ്ടി വന്നത് തീരെ യാദൃഛികമായിരുന്നില്ല. യുദ്ധ ധാര്‍മികത അജ്ഞാതമായിരുന്ന ക്രൈസ്തവ ലോകത്തിന് ആര്‍ദ്രതയെ മത പ്രചോദിതമായി രണവീറിലേക്ക് ചേര്‍ത്തുപിടിക്കേണ്ടതുണ്ടെന്ന് മനസ്സിലാക്കിക്കൊടുക്കാന്‍ മുഹമ്മദ് നബി(സ)യുടെ അനുയായികള്‍ക്കല്ലാതെ കഴിയുമായിരുന്നില്ല, ഇസ്‌ലാമിക ഭീകരതയുടെ മീഡിയാ വിശകലനങ്ങള്‍ അന്ധമാക്കിയ നമ്മുടെ പൊതുബോധം അതുള്‍ക്കൊള്ളാന്‍ ഇന്ന് പ്രയാസപ്പെടുമെങ്കിലും. ജീവിതത്തിന്റെ മറ്റു രംഗങ്ങളിലേതു പോലെത്തന്നെ സമാധാന സന്ദര്‍ഭത്തിലും യുദ്ധരംഗത്തുമെല്ലാം പാലിക്കേണ്ട നിയമങ്ങളെന്തൊക്കെയാണെന്ന് മുഹമ്മദ് നബി(സ) പഠിപ്പിച്ചു തരികയും പ്രായോഗികമാക്കി കാണിച്ചുതരുകയും ചെയ്തിട്ടുണ്ട്. പീഡനങ്ങള്‍ സഹിച്ച് പ്രബോധനം ചെയ്യുമ്പോഴും ആദര്‍ശജീവിതത്തിന് നിവൃത്തിയില്ലാതെ പാലായനം ചെയ്യുമ്പോഴും വിവിധ സമുദായങ്ങള്‍ ജീവിക്കുന്ന നാട് ഭരിക്കേണ്ടി വരുമ്പോഴും ഭരണാധികാരിയായി ശത്രുരാജ്യങ്ങളോട് യുദ്ധം ചെയ്യുമ്പോഴും യുദ്ധത്തില്‍ ജയിക്കുമ്പോഴും ശത്രുരാജ്യങ്ങളോട് സന്ധിയുണ്ടാക്കുമ്പോഴും ശത്രുരാജ്യത്തിനുമേല്‍ അധീശത്വം ലഭിക്കുമ്പോഴുമെല്ലാം എങ്ങനെ പെരുമാറണമെന്ന് അദ്ദേഹം കൃത്യമായി പഠിപ്പിച്ചിട്ടുണ്ട്. യുദ്ധരംഗത്തും സമാധാനരംഗത്തുമെല്ലാം മാനവവല്‍ക്കരിക്കുവാന്‍ ആ ജീവിതം തന്നെയാണ് മനുഷ്യര്‍ മാതൃകയാക്കേണ്ടത്. എല്ലാ രംഗത്തുമുള്ള സല്‍സ്വഭാവങ്ങളുടെ പൂര്‍ത്തീകരണത്തിനു വേണ്ടിയാണല്ലോ നബി നിയോഗമുണ്ടായിട്ടുള്ളത്.

മുസ്ലിംകളെല്ലാത്തവരെ മിത്രങ്ങളാക്കരുതെന്ന് സൂചിപ്പിക്കുന്ന ഖുർആൻ വചനങ്ങൾ എങ്ങനെയുള്ളവരോടാണ് അത്തരം ബന്ധവിച്ഛേദമുണ്ടാവേണ്ടത് എന്ന് കൂടി പഠിപ്പിക്കുന്നുണ്ട്. ഇസ്‌ലാമിനും മുസ്ലിംകൾക്കുമെതിരെ ശത്രുത പുലർത്തുകയും തരാം കിട്ടിയാൽ അവരെയും അവരുടെ ആദർശത്തെയും തകർക്കാൻ ശ്രമിക്കുകയും ചെയ്യുന്ന അമുസ്ലിംകളോടാണ് ശത്രുതയുണ്ടാവേണ്ടത് എന്നാണ് ഇസ്‌ലാമികനിർദേശം. അതല്ലാത്തവരോട് മൈത്രീഭാവം പുലർത്തുന്നതോ സ്നേഹത്തിൽ വർത്തിക്കുന്നതോ ഇസ്‌ലാം വിരോധിച്ചിട്ടില്ല.

”മത കാര്യത്തില്‍ നിങ്ങളോട് യുദ്ധം ചെയ്യാതിരിക്കുകയും, നിങ്ങളുടെ വീടുകളില്‍നിന്ന് നിങ്ങളെ പുറത്താക്കാതിരിക്കുകയു ചെയ്യുന്നവരെ സംബന്ധിച്ചിടത്തോളം നിങ്ങള്‍ അവര്‍ക്ക് നന്മ ചെയ്യുന്നതില്‍ നിന്നും നിങ്ങളവരോട് നീതി കാണിക്കുന്നതില്‍നിന്നും അല്ലാഹു നിങ്ങളെ വിലക്കുന്നില്ല. തീര്‍ച്ചയായും അല്ലാഹു നീതി പാലിക്കുന്നവരെ ഇഷ്ടപ്പെടുന്നു” (ഖുര്‍ ആന്‍ 60:8).

”മതകാര്യത്തില്‍ നിങ്ങളോട് യുദ്ധം ചെയ്യുകയും നിങ്ങളുടെ വീടുകളില്‍നിന്ന് നിങ്ങളെ പുറത്താക്കുകയും നിങ്ങളെ പുറത്താക്കുന്നതില്‍ പരസ്പരം സഹകരിക്കുകയും ചെയ്തവരെ സംബന്ധിച്ചുമാത്രമാണ് അവരോട് മൈത്രി കാണിക്കുന്നത് അല്ലാഹു നിരോധിക്കുന്നത്. വല്ലവരും അവരോട് മൈത്രീബന്ധം പുലര്‍ത്തുന്നപക്ഷം അവര്‍ തന്നെയാകുന്നു അക്രമകാരികള്‍” (60:9).

അമുസ്‌ലിംകളെ മിത്രങ്ങളാക്കരുത് എന്ന് അനുശാസിക്കുന്നത് ഇസ്‌ലാമിനെയും മുസ്‌ലിംകളെയും നശിപ്പിക്കുന്നതിനുവേണ്ടി തന്ത്രങ്ങള്‍ ആവിഷ്‌കരിച്ച് നടപ്പാക്കുന്നവരെ ഉദ്ദേശിച്ചുകൊണ്ടാണെന്ന വസ്തുത, ഈ സൂക്തത്തില്‍നിന്ന് സുതരാം വ്യക്തമാണ്. സാധാരണക്കാരായ അമുസ്‌ലിംകളുമായി സ്‌നേഹബന്ധം സ്ഥാപിക്കുന്നതുകൊണ്ട് അത് മതത്തിന് ഹാ നികരമാകാത്തിടത്തോളം എന്തെങ്കിലും കുഴപ്പമുണ്ടെന്ന് ഖുര്‍ആന്‍ പരാമര്‍ശിക്കുന്നില്ല.

സത്യനിഷേധികളെ ഉറ്റ മിത്രങ്ങളാക്കുവാന്‍ പാടില്ലെന്ന വ്യക്തമായ വിലക്ക് ഖുര്‍ആനിലുണ്ട്. ഏതാനും സൂക്തങ്ങള്‍ കാണുക:

”സത്യവിശ്വാസികള്‍ സത്യവിശ്വാസികളെയല്ലാതെ സത്യനിഷേധികളെ മിത്രങ്ങളാക്കി വെക്കരുത്. അങ്ങനെ വല്ലവനും ചെയ്യുന്നപക്ഷം അല്ലാഹുവുമായി അവന് യാതൊരു ബന്ധവുമില്ല; നിങ്ങള്‍ അവരോട് കരുതലോടെ വര്‍ത്തിക്കുകയാണെങ്കിലല്ലാതെ. അല്ലാഹു അവനെപ്പറ്റി നിങ്ങള്‍ക്ക് താക്കീത് നല്‍കുന്നു. അല്ലാഹുവിങ്കലേക്കത്രേ (നിങ്ങള്‍) തിരിച്ചു ചെല്ലേണ്ടത്” (3:28).

”സത്യവിശ്വാസികളേ, യഹൂദരെയും ക്രൈസ്തവരെയും നിങ്ങള്‍ ഉറ്റ മിത്രങ്ങളായി സ്വീകരിക്കരുത്. അവരാകട്ടെ അന്യോന്യം ഉറ്റമിത്രങ്ങളാണുതാനും. നിങ്ങളില്‍നിന്നാരെങ്കിലും അവരെ ഉറ്റമിത്രങ്ങളായി സ്വീകരിക്കുന്നപക്ഷം അവനും അവരില്‍പെട്ടവന്‍തന്നെയാണ്. അക്രമികളായ ആളുകളെ അല്ലാഹു നേര്‍വഴിയിലാക്കുകയില്ല, തീര്‍ച്ച” (വി.ഖു. 5:51).

”അല്ലാഹുവും അവന്റെ ദൂതനും താഴ്മയുള്ളവരായിക്കൊണ്ട് നമസ്‌കാരം മുറപോലെ നിര്‍വഹിക്കുകയും സകാത്ത് നല്‍കുകയും ചെയ്യുന്ന സത്യവിശ്വാസികളും മാത്രമാകുന്നു നിങ്ങളുടെ ഉറ്റമിത്രങ്ങള്‍” (5:55).

”വല്ലവനും അല്ലാഹുവെയും അവന്റെ ദൂതനെയും സത്യവിശ്വാസികളെയും മിത്രങ്ങളായി സ്വീകരിക്കുന്നുവെങ്കില്‍ തീര്‍ച്ചയായും അല്ലാഹുവിന്റെ കക്ഷിതന്നെയാണ് വിജയം നേടുന്നവര്‍” (5:56).

”സത്യവിശ്വാസികളേ, നിങ്ങള്‍ക്ക് മുമ്പ് വേദഗ്രന്ഥം നല്‍കപ്പെട്ടവരില്‍നിന്ന് നിങ്ങളുടെ മതത്തെ തമാശയും വിനോദവിഷയവുമാക്കിത്തീര്‍ത്തവരെയും സത്യനിഷേധികളെയും നിങ്ങള്‍ ഉറ്റമിത്രങ്ങളായി സ്വീകരിക്കരുത്. നിങ്ങള്‍ സത്യവിശ്വാസികളാണെങ്കില്‍ അല്ലാഹുവെ സൂക്ഷിക്കുവിന്‍” (5:57).

”നിങ്ങളെ നമസ്‌കാരത്തിനായി വിളിച്ചാല്‍, അവരതിനെ ഒരു തമാശയും വിനോദവിഷയവുമാക്കിത്തീര്‍ക്കുന്നു. അവര്‍ ചിന്തിച്ചു മനസ്സിലാക്കാത്ത ഒരു ജനവിഭാഗമായതുകൊണ്ടത്രെ അത്” (5:58).

ഏതു തരത്തിലുള്ള അമുസ്‌ലിംകളോടാണ് മുസ്‌ലിംകള്‍ മൈത്രീബന്ധം സ്ഥാപിക്കാന്‍ പാടില്ലാത്തത് എന്ന് ഖുര്‍ആന്‍തന്നെ വിശദീകരിക്കുന്നു.

”ഹേ, സത്യവിശ്വാസികളേ, എന്റെ ശത്രുവും നിങ്ങളുടെ ശത്രുവും ആയിട്ടുള്ളവരോട് സ്‌നേഹബന്ധം സ്ഥാപിച്ചുകൊണ്ട് നിങ്ങള്‍ അവരെ മിത്രങ്ങളാക്കി വെക്കരുത്. നിങ്ങള്‍ക്ക് വന്നുകിട്ടിയിട്ടുള്ള സത്യത്തില്‍ അവര്‍ അവിശ്വസിച്ചിരിക്കയാണ്. നിങ്ങള്‍ നിങ്ങളുടെ രക്ഷിതാവായ അല്ലാഹുവില്‍ വിശ്വസിക്കുന്നതിനാല്‍ റസൂലിനെയും നിങ്ങളെയും അവര്‍ നാട്ടില്‍ നിന്ന് പുറത്താക്കുന്നു. എന്റെ മാര്‍ഗത്തില്‍ സമരം ചെയ്യുവാനും എന്റെ പ്രീതി തേടുവാനും  നിങ്ങള്‍ പുറപ്പെട്ടിരിക്കയാണെങ്കില്‍ (നിങ്ങള്‍ അപ്രകാരം മൈത്രീബന്ധം സ്ഥാപിക്കരുത്). നിങ്ങള്‍ അവരുമായി രഹസ്യമായി സ്‌നേഹബന്ധം സ്ഥാപിക്കുന്നു. നിങ്ങള്‍ രഹസ്യമാക്കിയതും പരസ്യമാക്കിയതും ഞാന്‍ നല്ലവണ്ണം അറിയുന്നവനാണ്. നിങ്ങളില്‍നിന്ന് വല്ലവനും അപ്രകാരം പ്രവര്‍ത്തിക്കുന്നപക്ഷം അവന്‍ നേര്‍മാര്‍ഗത്തില്‍ നിന്ന് പിഴച്ചുപോയിരിക്കുന്നു”(60:1).

”അവര്‍ നിങ്ങളെ കണ്ടുമുട്ടുന്നപക്ഷം അവര്‍ നിങ്ങള്‍ക്ക് ശത്രുക്കളായിരിക്കും. നിങ്ങളുടെ നേര്‍ക്ക് ദുഷ്ടതയും കൊണ്ട് അവരുടെ കൈകളും നാവുകളും അവര്‍ നീട്ടുകയും  നിങ്ങള്‍ അവിശ്വസിച്ചിരുന്നെങ്കില്‍ എന്ന് അവര്‍ ആഗ്രഹിക്കുകയും ചെയ്യും” (60:2).

ഇസ്‌ലാമിനെയും മുസ്‌ലിംകളെയും ശത്രുതയോടെ വീക്ഷിക്കുകയും നശിപ്പിക്കുവാനായി ഗൂഢതന്ത്രങ്ങള്‍ മെനയുകയും അല്ലാഹുവില്‍ വിശ്വസിച്ചതിനാല്‍ നാട്ടില്‍നിന്ന് മുസ്‌ലിംകളെ പുറത്താക്കുകയും ചെയ്യുന്ന അമുസ്‌ലിംകളുമായി, അവര്‍ വേദക്കാരാകട്ടെ അല്ലാത്തവരാകട്ടെ, മൈത്രീബന്ധം സ്ഥാപിക്കുന്നതിനെയാണ് ഖുര്‍ആന്‍ വിലക്കുന്നത്. ഇത്തരം സത്യനിഷേധികളുമായി, ദൈവമാര്‍ഗത്തില്‍ സമരം ചെയ്യുന്ന ഒരു മുസ്‌ലിമിനും മൈത്രീബന്ധം സ്ഥാപിക്കുവാനാവുകയില്ലെന്നുറപ്പാണ്. എന്നാല്‍, ചില കപടവിശ്വാസികള്‍ രഹസ്യമായി ഇസ്‌ലാമിന്റെ ശത്രുക്കളുമായി ചങ്ങാത്തത്തിലാവുകയും മുസ്‌ലിംകളോട് ഞങ്ങള്‍ നിങ്ങളോടൊപ്പമാണ് എന്ന് നുണ പറയുകയും ചെയ്തിരുന്നു. അവരെക്കുറിച്ചാണ്, ”നിങ്ങളില്‍നിന്നാരെങ്കിലും അവരെ ഉറ്റമിത്രങ്ങളായി സ്വീകരിക്കുന്ന പക്ഷം അവനും അവരില്‍പെട്ടവന്‍തന്നെയാണ്” (5:51) എന്ന് ഖുര്‍ആന്‍ പറഞ്ഞിട്ടുള്ളത്.

അന്യമതസ്ഥരുടെ ആരാധനാമൂര്‍ത്തികളെ നശിപ്പിക്കുവാന്‍ ആവശ്യപ്പെടുന്ന സൂക്തങ്ങളൊന്നുംതന്നെ ഖുര്‍ആനിലില്ല. മാത്രവുമല്ല. അമുസ്‌ലിംകള്‍ ആരാധിക്കുന്ന വസ്തുക്കളെ അവഹേളിക്കരുതെന്നാണ് ഖുര്‍ആനിന്റെ അനുശാസന: ”അല്ലാഹുവിന് പുറമെ അവര്‍ വിളിച്ചു പ്രാര്‍ഥിക്കുന്നവരെ നിങ്ങള്‍ ശകാരിക്കരുത്” (16:108). ഏകദൈവാരാധനയിലധിഷ്ഠിതമായ ഇസ്‌ലാം സൃഷ്ടിപൂജയെ വെറുക്കുന്നുവെന്നത് നേരാണ്. അതുകൊണ്ടുതന്നെ സൃഷ്ടിപൂജയുടെ നിരര്‍ഥകതയെ വെളിപ്പെടുത്തുന്ന ഒട്ടനവധി സൂക്തങ്ങള്‍ ഖുര്‍ആനിലുണ്ട്. ഇവയെല്ലാം മനുഷ്യബുദ്ധിയെ തൊട്ടുണര്‍ത്തുന്നവയാണ്. സൃഷ്ടിപൂജയുടെ അധമത്വത്തില്‍നിന്ന് മാനവ സമുദായത്തെ മോചിപ്പിക്കുകയാണ് ഖുര്‍ആനിന്റെ ലക്ഷ്യം. ആരാധനാ മൂര്‍ത്തികളെ നശിപ്പിച്ചതുകൊണ്ട് മനുഷ്യരാശി സൃഷ്ടിപൂജയില്‍നിന്ന് കരകയറുമെന്ന മൂഢധാരണയൊന്നും ഖുര്‍ആനിനില്ല. അതുകൊണ്ടുതന്നെ ഖുര്‍ആന്‍ നശിപ്പിക്കുവാന്‍ ശ്രമിക്കുന്നത് കല്ലുകളാലും മറ്റും നിര്‍മിക്കപ്പെട്ട ആരാധനാമൂര്‍ത്തികളെയല്ല, മനുഷ്യ മനസ്സുകളില്‍ കൊത്തിവെക്കപ്പെട്ട വിഗ്രഹങ്ങളെയാണ്. ഈ വിഗ്രഹ ധ്വംസനത്തിന് ശക്തി പ്രയോഗിക്കുകയല്ല, യുക്തിയെ പ്രവര്‍ത്തനക്ഷമമാക്കുകയാണ് വേണ്ടതെന്ന് അല്ലാഹുവിന് നന്നായി അറിയാം. അതുകൊണ്ടുതന്നെ ഖുര്‍ആന്‍ ചെയ്യുന്നത് വിഗ്രഹാരാധനക്കെതിരെ ജനങ്ങളെ ബോധവത്കരിക്കുകയാണ്. നജ്‌റാനിലെ ക്രൈസ്തവരുമായി പ്രവാചകനുണ്ടാക്കിയ കരാറില്‍നിന്ന് ഇക്കാര്യം സുതരാം വ്യക്തമാണ്. അതില്‍ നമുക്കിങ്ങനെ വായിക്കാം: ‘നജ്‌റാനിലെ ക്രൈസ്തവര്‍ക്കും അവരോടൊപ്പം ജീവിക്കുന്നവര്‍ക്കും അവരുടെ ജീവന്‍, മതം, ഭൂമി, ധനം എന്നിവക്കും അവരില്‍ സന്നിഹിതരായവര്‍ക്കും അല്ലാത്തവര്‍ക്കും അവരുടെ നിവേദക സംഘങ്ങള്‍ക്കും കുരിശ്, ക്രൈസ്തവ ദേവാലയം തുടങ്ങിയ മത ചിഹ്‌നങ്ങള്‍ക്കും അല്ലാഹുവിന്റെ അഭയവും അവന്റെ ദൂതന്‍ മുഹമ്മദിന്റെ സംരക്ഷണ ബാധ്യതയുമുണ്ട്. ഇവയുടെ നിലവിലുള്ള അവസ്ഥയില്‍ യാതൊരു മാറ്റവും വരുത്തുന്നതല്ല.  അവരുടെ പുരോഹിതനോ സന്യാസിയോ പരിപാലകനോ തല്‍സഥാനത്തുനിന്ന് നീക്കം ചെയ്യപ്പെടുകയോ അവരുടെ ഏതെങ്കിലും അവകാശങ്ങള്‍ ഹനിക്കപ്പെടുകയോ ഏതെങ്കിലും മതചിഹ്‌നങ്ങള്‍ മാറ്റപ്പെടുകയോ ഇല്ല‘. പ്രവാചകനുശേഷം ഖലീഫമാരും അവര്‍ക്കുശേഷം വന്ന മുസ്‌ലിം ഭരണാധികാരികളുമെല്ലാം അന്യമതസ്ഥര്‍ക്ക് ആരാധനാ സ്വാതന്ത്ര്യമനുവദിച്ചിരുന്നതായി അക്കാലത്തെ രേഖകളില്‍നിന്ന് വ്യക്തമായി മനസ്സിലാക്കാന്‍ കഴിയും. എന്നാല്‍ ഒരു രാഷ്ട്രം പൂര്‍ണമായി ഇസ്‌ലാമികമായിത്തീരുകയും വിഗ്രഹാരാധകരായി ആരും തന്നെ അവശേഷിക്കാത്ത അവസ്ഥ സംജാതമാകുകയും ചെയ്യുന്ന സാഹചര്യങ്ങളില്‍ ദുഷിച്ച ബഹുദൈവാരാധനയിലേക്ക് മനുഷ്യരെ തിരിച്ചുവിടുന്ന വിഗ്രഹങ്ങളെയും ശവകുടീരങ്ങളെയും നശിപ്പിക്കാന്‍ പ്രവാചകന്‍ (ﷺ)കല്‍പിച്ചതായി കാണുവാന്‍ കഴിയും. ജനങ്ങളെല്ലാം ഏകദൈവാരാധകരായി മാറിയതിനുശേഷമുളള ഒരു നടപടിയാണിത്. ഒരു ബഹുമതസമൂഹത്തില്‍ മുസ്‌ലിം സ്വീകരിക്കേണ്ട നടപടിയല്ലെന്നര്‍ത്ഥം.

 അമുസ്‌ലിംകളെ നിര്‍ബന്ധിച്ച് മതപരിവര്‍ത്തനം ചെയ്യിക്കണമെന്ന് അനുശാസിക്കുന്ന ഒരു വചനം പോലും ഖുര്‍ആനിലില്ല. നിര്‍ബന്ധ മതപരിവര്‍ത്തനം എന്ന ആശയത്തോടുതന്നെ ഖുര്‍ആന്‍ യോജിക്കുന്നില്ല.

ഇസ്‌ലാം എന്നാല്‍ സമര്‍പ്പണം, സമാധാനം എന്നിങ്ങനെയാണര്‍ഥം. സര്‍വശക്തന് സ്വന്തം ജീവിതത്തെ സമര്‍പ്പിക്കുന്നതുവഴി ഒരാള്‍ നേടിയെടുക്കുന്ന സമാധാനമാണ് ഇസ്‌ലാം എന്ന് പറയാം. ദൈവം തമ്പുരാന് സ്വന്തത്തെ സമര്‍പ്പിച്ചവനാണ് മുസ്‌ലിം. ഒരാള്‍ മുസ്‌ലിമാവുകയെന്നാല്‍ ദൈവിക മാര്‍ഗനിര്‍ദേശങ്ങള്‍ക്കനുസരിച് ജീവിതത്തെ പരിവര്‍ത്തിപ്പിക്കുകയെന്നാണര്‍ഥം. ഈ പരിവര്‍ത്തനത്തിന്റെ മുളപൊട്ടേണ്ടത് മനസ്സിലാണ്. മനുഷ്യമനസ്സുകളില്‍ മാറ്റമുണ്ടാകാതെ മൗലികമായ യാതൊരു പരിവര്‍ത്തനവും സാധ്യമല്ലെന്നതാണ് ഖുര്‍ആനിന്റെ വീക്ഷണം. അതുകൊണ്ടുത ന്നെ നിര്‍ബന്ധിച്ച് ഒരാളെയും മതത്തില്‍ കൂട്ടുന്നതിനോട് അത് യോജിക്കുന്നില്ല. സത്യവിശ്വാസത്തിലേക്ക്  കടന്നുവരുന്നതിനായി സ്വന്തം സമുദായത്തെ ഉല്‍ബോധിപ്പിക്കുന്നതിനുവേണ്ടി കഠിനാധ്വാനം ചെയ്ത പ്രവാചകന് (ﷺ) സത്യനിഷേധികളുടെ നിലപാടില്‍ മാറ്റമൊന്നുമില്ലെന്ന് മനസ്സിലായപ്പോള്‍ ഉണ്ടായ മനോവ്യഥയെ ചോദ്യം ചെയ്തുകൊണ്ട്ഖുര്‍ആന്‍ പറയുന്നത് കാണുക: ”നിന്റെ രക്ഷിതാവ് ഉദ്ദേശിച്ചിരുന്നുവെങ്കില്‍ ഭൂമിയിലുള്ളവരെല്ലാം ഒന്നിച്ച് വിശ്വസിക്കുമായിരുന്നു. എന്നിരിക്കെ ജനങ്ങള്‍ സത്യവിശ്വാസികളാകുവാന്‍ നീ അവരെ നിര്‍ബന്ധിക്കുകയോ? (10:99).

സത്യമതപ്രബോധനത്തിനായി നിയുക്തരായ പ്രവാചകന്മാരില്‍ നിക്ഷിപ്തമായിരുന്ന ബാധ്യത മതപ്രചാരണം മാത്രമായിരുന്നുവെന്നും നിര്‍ബന്ധിച്ച് മതം മാറ്റുകയായിരുന്നില്ലെന്നുമുള്ള വസ്തുത ഖുര്‍ആന്‍ വ്യക്തമാക്കുന്നുണ്ട്: ”എന്നാല്‍ ദൈവദൂതന്മാരുടെ മേല്‍ സ്പഷ്ടമായ പ്രബോധനമല്ലതെ വല്ല ബാധ്യതയുമുണ്ടോ?” (16:36).

”ഇനി അവര്‍ തിരിഞ്ഞുകളയുകയാണെങ്കില്‍ (നബിയേ) നിന്നെ നാം അവരുടെ മേല്‍ കാവല്‍ക്കാരനായി അയച്ചിട്ടില്ല. നിന്റെ മേല്‍ പ്രബോധനബാധ്യത മാത്രമേയുള്ളൂ‘ (വി.ഖു 42:48).

സത്യമതത്തിലേക്ക് ജനങ്ങളെ ക്ഷണിക്കുകയല്ലാതെ അവരെ നിര്‍ബന്ധിച്ച് മാറ്റുന്നതിനുവേണ്ടി പ്രവാചകന്‍ പരിശ്രമിക്കേണ്ടതില്ലെന്ന് ഖുര്‍ആന്‍ അദ്ദേഹത്തോട് ആവര്‍ത്തിച്ചു പറയുന്നുണ്ട്”. പറയുക: സത്യം നിങ്ങളുടെ രക്ഷിതാവിങ്കല്‍നിന്നുള്ളതാവുന്നു. അതിനാല്‍ ഇഷ്ടമുള്ളവര്‍ വിശ്വസിക്കട്ടെ” (18:29).

”അതിനാല്‍ (നബിയേ) നീ ഉല്‍ബോധിപ്പിക്കുക. നീ ഒരു ഉല്‍ബോധകന്‍ മാത്രമാകുന്നു. നീ അവരുടെ മേല്‍ അധികാരം ചെലുത്തേണ്ടവനല്ല” (88:21,22)

ചുരുക്കത്തില്‍ പ്രവാചകന്മാരെല്ലാം സത്യമതപ്രബോധകര്‍ മാത്രമായിരുന്നു. അന്തിമ പ്രവാചകനും തഥൈവ. ജനങ്ങളുടെ മുമ്പില്‍ സത്യമേതെന്ന് തുറന്നു കാണിക്കേണ്ട ഉത്തരവാദിത്തം  മാത്രമേ അദ്ദേഹത്തിനുണ്ടായിരുന്നുള്ളൂ. അന്തിമ പ്രവാചകനിലൂടെ പൂര്‍ത്തീകരിക്കപ്പെട്ട സത്യമതത്തിന്റെ പ്രചാരണം ഉത്തരവാദിത്തമായി ഏല്‍പിക്കപ്പെട്ട സത്യവിശ്വാസികളുടെ ബാധ്യതയും ഇതുമാത്രമാണ്. അസത്യത്തില്‍നിന്ന് സത്യത്തെ വേര്‍തിരിച്ച് മനസ്സിലാക്കിക്കൊടുക്കുകയെന്ന ബാധ്യത മാത്രം. മതത്തില്‍ നിര്‍ബന്ധിച്ച് ആളെ ചേര്‍ക്കുന്നതിന് ഖുര്‍ആന്‍ ആരോടും ആവശ്യപ്പെടുന്നില്ലെന്നു മാത്രമല്ല, നിര്‍ബന്ധ മതപരിവര്‍ത്തനം ശരിയല്ലെന്ന നിലപാട് അത് പ്രഖ്യാപിക്കുകയും ചെയ്യുന്നു. ”മതത്തിന്റെ കാര്യത്തില്‍ ബലപ്രയോഗമേയില്ല. സന്മാര്‍ഗം ദുര്‍മാര്‍ഗത്തില്‍നിന്ന് വ്യക്തമായി വേര്‍തിരിഞ്ഞുകഴിഞ്ഞിരിക്കുന്നു” (2:256).

റച്ചുവെക്കുന്നവന്‍ എന്നാണ് കാഫിര്‍ എന്ന പദത്തിന്റെ ഭാഷാര്‍ഥം. വിത്ത് മണ്ണിനടിയില്‍ മറച്ചുവെക്കുന്നവനായതിനാല്‍ കര്‍ഷകനെ കാഫിര്‍ എന്നു വിളിക്കും. ലഭിച്ച നേട്ടങ്ങള്‍ മറച്ചുവെക്കുന്നവനെ കാഫിര്‍ എന്നു വിളിക്കുന്ന രീതി പൗരാണിക അറേബ്യയില്‍തന്നെ നിലവിലുണ്ടായിരുന്നു. നന്ദികെട്ടവന്‍ എന്ന അര്‍ഥത്തിലും കാഫിര്‍ എന്ന് പ്രയോഗിക്കപ്പെട്ടതായി കാണുവാന്‍ കഴിയും.

സത്യനിഷേധി, നന്ദികേട് കാണിക്കുന്നവന്‍, അവിശ്വസിക്കുന്നവന്‍എന്നീ അര്‍ഥങ്ങളിലാണ് ഖുര്‍ആന്‍ കാഫിര്‍ എന്നു പ്രയോഗിക്കുന്നത്. പുലഭ്യം പറയുന്ന രീതിയിലല്ല, പ്രത്യുത ഉദ്ദേശിക്കപ്പെടുന്നവരുടെ സ്വഭാവം വിശദീകരിക്കുന്ന രീതിയിലാണ് ഖുര്‍ആന്‍ ഈ പദം പ്രയോഗിച്ചിരിക്കുന്നത്. കാഫിര്‍ എന്ന ഏകവചനപ്രയോഗവും കാഫിറൂന്‍, കുഫ്ഫാര്‍ എന്നീ ബഹുവചനപ്രയോഗങ്ങളും ഖുര്‍ആനില്‍ പലതവണ ആവര്‍ത്തിക്കപ്പെട്ടിട്ടുണ്ട്. അവിടെയെല്ലാം തന്നെ പ്രതിപാദിക്കപ്പെട്ടവരുടെ സ്വഭാവങ്ങള്‍ വ്യക്തമാക്കുകയാണ് ഖുര്‍ആന്‍ ചെയ്യുന്നത്. ഏതാനും ഖുര്‍ആന്‍ വചനങ്ങള്‍ കാണുക:

”അല്ലാഹുവിലും അവന്റെ ദൂതന്മാരിലും അവിശ്വസിക്കുകയും (വിശ്വാസ കാര്യത്തില്‍) അല്ലാഹുവിനും അവന്റെ ദൂതന്മാര്‍ക്കുമിടയില്‍ വിവേചനം കല്‍പിക്കാന്‍ ആഗ്രഹിക്കുകയും ഞങ്ങള്‍ ചിലരില്‍ വിശ്വസിക്കുകയും ചിലരെ നിഷേധിക്കുകയും ചെയ്യുന്നു എന്നു പറയുകയും അങ്ങനെ അതിനിടയില്‍ മറ്റൊരു മാര്‍ഗം സ്വീകരിക്കാനുദ്ദേശിക്കുകയും ചെയ്യുന്നവരാരോ അവര്‍ തന്നെയാകുന്നു യഥാര്‍ഥ കാഫിറുകള്‍ (സത്യനിഷേധികള്‍). കാഫിറുകള്‍ക്ക് അപമാനകരമായ ശിക്ഷ നാം ഒരുക്കിവെച്ചിട്ടുണ്ട്” (4:150,151).

”അല്ലാഹുവിന്റെ അനുഗ്രഹം അവര്‍ മനസ്സിലാക്കുകയും എന്നിട്ട് അതിനെ നിഷേധിക്കുകയുമാണ് ചെയ്യുന്നത്. അവരില്‍ അധികപേരും കാഫിറുകള്‍ (സത്യനിഷേധികള്‍/നന്ദികെട്ടവര്‍) ആകുന്നു” (16:83).

”തീര്‍ച്ചയായും നാം തന്നെയാകുന്നു തൗറാത്ത് അവതരിപ്പിച്ചിരിക്കുന്നത്. അതില്‍ മാര്‍ഗദര്‍ശനവും പ്രകാശവുമുണ്ട്. (അല്ലാഹുവിന്) കീഴ്‌പ്പെട്ട പ്രവാചകന്മാര്‍ യഹൂദന്മാര്‍ക്ക് അതിനനുസരിച്ച് വിധി കല്‍പിച്ചു വന്നു. പുണ്യവാന്മാരും മതപണ്ഡിതന്മാരും (അങ്ങനെതന്നെ ചെയ്തു). കാരണം അല്ലാഹുവിന്റെ ഗ്രന്ഥത്തിന്റെ സംരക്ഷണം അവര്‍ക്ക് ഏല്‍പിക്കപ്പെട്ടിരുന്നു. അവരതിന് സാക്ഷികളുമായിരുന്നു. അതിനാല്‍ നിങ്ങള്‍ ജനങ്ങളെ പേടിക്കാതെ എന്നെ മാത്രം ഭയപ്പെടുക. എന്റെ വചനങ്ങള്‍ നിങ്ങള്‍ തുച്ഛമായ വിലയ്ക്ക് വിറ്റുകളയാതിരിക്കുക. അല്ലാഹു അവതരിപ്പിച്ചതനുസരിച്ച് ആര്‍ വിധിക്കുന്നില്ലയോ അവര്‍തന്നെയാകുന്നു കാഫിറുകള്‍” (5:44).

”(നബിയേ) പറയുക: കാഫിറുകളേ, നിങ്ങള്‍ ആരാധിച്ചുവരുന്നതിനെ ഞാന്‍ ആരാധിക്കുന്നില്ല. ഞാന്‍ ആരാധിച്ചുവരുന്നതിനെ നിങ്ങളും ആരാധിക്കുന്നവരല്ല. നിങ്ങള്‍ ആരാധിച്ചു വന്നതിനെ ഞാന്‍ ആരാധിക്കുവാന്‍ പോകുന്നില്ല. ഞാന്‍ ആരാധിച്ചുവരുന്നതിനെ നിങ്ങളും ആരാധിക്കുന്നില്ല. നിങ്ങള്‍ക്ക് നിങ്ങളുടെ മതം, എനിക്ക് എന്റെ മതവും” (109:1-6).

ഈ വചനങ്ങളില്‍നിന്നെല്ലാം നമുക്ക് മനസ്സിലാവുന്നത് ദൈവിക മാര്‍ഗദര്‍ശനം അംഗീകരിക്കാതെ സത്യത്തെ നിഷേധിക്കുകയും അതുവഴി അനുഗ്രഹദാതാവായ അല്ലാഹുവിനോട് നന്ദികേട് കാണിക്കുകയും ചെയ്യുന്നവരെ കുറിക്കുവാന്‍ വേണ്ടിയാണ് ഖുര്‍ആന്‍ കാഫിര്‍ എന്നു പ്രയോഗിച്ചിരിക്കുന്നത് എന്നാണ്. പടച്ചവന്റെ അനുഗ്രഹങ്ങള്‍ അനുഭവിക്കുകയും അതോടൊപ്പം അനുഗ്രഹദാതാവായ അല്ലാഹുവെ അംഗീകരിക്കാതിരിക്കുകയും ചെയ്യുന്നവന്‍ കാഫിറാണ്. മനുഷ്യര്‍ക്കാവശ്യമായ സംവിധാനങ്ങളെല്ലാം ഭൂമിയില്‍ ചെയ്തുവെച്ച സ്രഷ്ടാവ് നമ്മില്‍നിന്ന് ആവശ്യപ്പെടുന്ന ഏകകാര്യമായ അവനെ മാത്രം ആരാധിക്കുന്നതില്‍നിന്ന് വ്യതിചലിച്ച് ആരാധന അര്‍ഹിക്കാത്ത സൃഷ്ടികളോട് പ്രാര്‍ഥിക്കുന്നവന്‍ കാഫിറാണ്. സന്മാര്‍ഗം കാണിച്ചുതരാന്‍ വേണ്ടി സ്രഷ്ടാവ് പറഞ്ഞയച്ച പ്രവാചകന്മാരെ അംഗീകരിക്കാതിരിക്കുന്നവന്‍ കാഫിറാണ്. സത്യാസത്യവിവേചകമായി പടച്ചതമ്പുരാന്‍ അവതരിപ്പിച്ച വേദഗ്രന്ഥങ്ങള്‍ അനുസരിച്ച് ജീവിതം ചിട്ടപ്പെടുത്താത്തവന്‍ കാഫിറാണ്. സത്യത്തിന്റെ ശത്രുക്കളായി മാറിയ കാഫിറുകളാണ് ദൈവികമായ പ്രകാശത്തെ ഊതിക്കെടുത്താന്‍ ശ്രമിക്കുന്നത്.

മുഹമ്മദ് നബി(സ) യുടെ നിയോഗത്തിന് മുമ്പോ ശേഷമോ ആരെയെങ്കിലും പുലഭ്യം പറയുന്നതിനുള്ള ഒരു പദപ്രയോഗമായി അറബികൾ കാഫിർ എന്ന് പ്രയോഗിക്കാറില്ല. ദൈവിക നിയമങ്ങളെ അനുസരിക്കുന്നവനെ മുസ്‌ലിം എന്ന് വിളിച്ചപ്പോൾ അത് അംഗീകരിക്കാത്തവരെ കാഫിർ എന്ന് വിളിച്ചുവെന്ന് മാത്രമേയുള്ളൂ. ആ വിളിയിൽ യാതൊരു വിധ വർഗീയതയുമില്ല.

ശിക്ഷാസമ്പ്രദായങ്ങളിലും അതല്ലാത്ത നിയമങ്ങളിലുമെല്ലാം ഇസ്‌ലാമിക രാഷ്ട്രത്തിലെ എല്ലാ പൗരന്മാരും തുല്യരായാണ് പരി ഗണിക്കപ്പെടുന്നത്. സമ്പന്നനും ദരിദ്രനുമിടയിലോ ഭരണാധികാരി ക്കും സാധാരണ പൗരന്നുമിടയിലോ യാതൊരുരീതിയിലുള്ള വി വേചനവും നീതിനിര്‍വഹണത്തിന്റെ കാര്യത്തില്‍ ഉണ്ടാവാന്‍ പാടില്ലെന്നാണ് ഇസ്‌ലാമിന്റെ അധ്യാപനം. 'സത്യവിശ്വാസികളേ, നിങ്ങള്‍ അല്ലാഹുവിനുവേണ്ടി സാക്ഷ്യം വഹിക്കുന്നവരെന്ന നില യയില്‍ കണിശമായി നീതി നിലനിര്‍ത്തുന്നവരായിരിക്കണം. അത് നിങ്ങള്‍ക്ക്തന്നെയോ, നിങ്ങളുടെ മാതാപിതാക്കള്‍, അടുത്ത ബ ന്ധുക്കള്‍ എന്നിവര്‍ക്കോ പ്രതികൂലമായിത്തീര്‍ന്നാലും ശരി' എ ന്നാണ് ക്വുര്‍ആന്‍ (4:135) പഠിപ്പിക്കുന്നത്.

ഒരിക്കല്‍ കുലീന കുടുംബത്തില്‍പ്പെട്ട ഒരു സ്ത്രീ മോഷണക്കു റ്റത്തിന് പിടിക്കപ്പെട്ടു. പ്രവാചക(സ)ന്റെ അടുത്ത അനുയായികളിലൊ രാളായ ഉസാമഃ (റ ) അവരുടെ കാര്യത്തില്‍ ശുപാര്‍ശ ചെയ്യുവാനായി പ്രവാചകനെ സമീപിച്ചു. അപ്പോള്‍ പ്രവാചകന്‍ (സ)പറഞ്ഞ വാക്കു കള്‍ ശ്രദ്ധേയമാണ്: 'നിങ്ങള്‍ക്ക് മുമ്പുള്ള സമുദായങ്ങള്‍ നശിപ്പിക്ക പ്പെട്ടത് ഇത്തരം പ്രവര്‍ത്തനങ്ങള്‍ മൂലമാണ്. സമ്പന്നര്‍ കുറ്റംചെയ് താല്‍ വെറുതെ വിടുകയും ദരിദ്രര്‍ കുറ്റംചെയ്താല്‍ ശിക്ഷിക്കുകയു മായിരുന്നു അവരുടെ സമ്പ്രദായം. അല്ലാഹുവാണെ സത്യം. ഈ പ്രവൃത്തി ചെയ്തത് മുഹമ്മദിന്റെ മകള്‍ ഫാത്തിമ തന്നെയായാ ലും അവളുടെ കരം ഞാന്‍ ഛേദിച്ചുകളയുന്നതാണ്'. നീതിയുടെ മുമ്പില്‍ ഭരണാധിപനും ഭരണീയനും സമമാണ് എന്നതാണ് ഇസ്‌ലാമിക രാഷ്ട്രസങ്കല്‍പത്തിന്റെ അടിസ്ഥാന സ്തംഭങ്ങളിലൊന്ന്. ഇക്കാര്യത്തിലേക്ക് വെളിച്ചംവീശുന്ന ഒട്ടന വധി സംഭവങ്ങള്‍ ഇസ്‌ലാമിക ചരിത്രത്തില്‍ കാണാനാവും. ഒരു സംഭവമിതാ:ഉമറി(റ )ന്റെ ഭരണകാലത്ത് ഈജിപ്തിലെ ഗവര്‍ണറായിരുന്ന അംറുബ്‌നുല്‍ ആസി(റ )ന്റെ പുത്രന്‍ മുഹമ്മദുബ്‌നു അംറ് കോപ്റ്റിക് വിഭാഗത്തില്‍പ്പെട്ട ഒരു സാധാരണക്കാരനെ തന്റെ കുതി രയുടെ മുമ്പില്‍ സ്വന്തം കുതിരയെ ഓടിച്ചതിന്റെ പേരില്‍ അടിച്ചു. 'ഞാന്‍ മാന്യന്മാരുടെ പുത്രനാണ്' എന്നു പറഞ്ഞുകൊണ്ടായിരുന്നു പ്രഹരം. ഈ പ്രശ്‌നം ഖലീഫയുടെ അടുക്കലെത്തി. ഗവര്‍ണറെയും പുത്രനെയും മദീനയിലേക്ക് വിളിപ്പിച്ചു കൊണ്ട് ഉമര്‍(റ) ചോദിച്ചു: 'എന്നുമുതല്‍ക്കാണ് നിങ്ങള്‍ ജനങ്ങളെ അടിമകളാക്കിത്തുടങ്ങിയ ത്? അവരുടെ മാതാക്കള്‍ അവരെ പ്രസവിച്ചത് സ്വതന്ത്രരായിട്ടല്ലേ'. തുടര്‍ന്ന് മര്‍ദിതനായ ആ കോപ്റ്റിക് വംശജന്റെ കയ്യില്‍ ചാട്ടവാറു നല്‍കിക്കൊണ്ട് പ്രതികാരം ചെയ്യുവാന്‍ ഉമര്‍(റ) കല്‍പിച്ചു. നിയമ ത്തിനു മുന്‍പില്‍ എല്ലാവരും സമന്മാരാണെന്ന തത്ത്വം ഇത്രത്തോ ളം പ്രായോഗികമാക്കിയ ഇസ്‌ലാമിക സമൂഹമല്ലാത്ത മറ്റൊരു സമൂ ഹത്തെ ലോകചരിത്രത്തില്‍ കാണുക സാധ്യമല്ല. ഇസ്‌ലാമികരാഷ്ട്രത്തില്‍ ജീവിക്കുന്ന അമുസ്‌ലിം പൗരന്മാരോ ടും നീതിനിര്‍വഹണത്തിന്റെ കാര്യത്തില്‍ വിവേചനമൊന്നും കാണിക്കാന്‍ പാടില്ലെന്നാണ് ഇസ്‌ലാം പഠിപ്പിക്കുന്നത്. വിശുദ്ധക്വുര്‍ആന്‍ നിഷ്‌കര്‍ഷിക്കുന്നത് കാണുക: 'സത്യവിശ്വാസികളേ, നിങ്ങ ള്‍ അല്ലാഹുവിനുവേണ്ടി നിലകൊള്ളുന്നവരും നീതിക്ക് സാക്ഷ്യം വഹിക്കുന്നവരുമായിരിക്കുക. ഒരു ജനതയോടുള്ള അമര്‍ഷം നീതി പാലിക്കാതിരിക്കാന്‍ നിങ്ങള്‍ക്ക് പ്രേരകമാകരുത്. നിങ്ങള്‍ നീതി പാലിക്കുക. അതാണ് ധര്‍മനിഷ്ഠയോട് ഏറ്റവും അടുത്തത.' (5:8) ഈ ക്വുര്‍ആനിക കല്‍പനയെ പൂര്‍ണമായി ഉള്‍ക്കൊണ്ടുകൊണ്ടായിരു ന്നു പ്രവാചകനും സച്ചരിതരായ ഖലീഫമാരും ഭരണം നടത്തിയത്.

ക്രൈസ്തവനായ കോപ്റ്റിക് വംശജനും മുസ്‌ലിമായ ഗവര്‍ണ റുടെ പുത്രനും തമ്മിലുണ്ടായ പ്രശ്‌നത്തില്‍ മുസ്‌ലിം അനീതിചെയ് തുവെന്ന് മനസ്സിലാക്കിയ ഖലീഫാ ഉമറിെേന്റ കാലത്തുതന്നെ നടന്ന മറ്റൊരു സംഭവമിതാ: ഗവര്‍ണറായിരുന്ന അംറുബ്‌നുല്‍ ആസ്വ്‌(റ ) ഒരു ക്രൈസ്തവസ്ത്രീയുടെ വീട് നിര്‍ബന്ധപൂര്‍വം പള്ളി യോട് ചേര്‍ക്കുകയുണ്ടായി. സ്ത്രീ ഇതേക്കുറിച്ച് ഖലീഫയോട് പ രാതി പറഞ്ഞു: ഖലീഫാ ഉമര്‍ ഗവര്‍ണറെ വിളിച്ച് വിശദീകരണ മാവശ്യപ്പെട്ടു. അദ്ദേഹം പറഞ്ഞു: 'മുസ്‌ലിംകളുടെ അംഗസംഖ്യ വര്‍ധിച്ചതു കാരണം പള്ളിയില്‍ സ്ഥലമില്ലാതെ വിഷമംനേരിട്ടു. പള്ളി വിശാലമാക്കേണ്ടതിന്നായി അതിന്റെ അടുത്തുള്ള ക്രൈസ് തവസ്ത്രീയുടെ ഭവനം ആവശ്യപ്പെടുകയും അതിന്ന് ന്യായമായ വില നല്‍കാമെന്ന് ഉറപ്പുനല്‍കുകയും ചെയ്തു. അവര്‍ വിസമ്മതി ച്ചതുകാരണം വീടുപൊളിച്ച് പള്ളിയോട് ചേര്‍ക്കാന്‍ നിര്‍ബന്ധിത മാവുകയാണുണ്ടായത്. അതിന്റെ വില ആ സ്ത്രീക്ക് ആവശ്യമുള്ള ഏതുസമയത്തും സ്വീകരിക്കുവാന്‍ സാധിക്കുമാറ് പൊതുഖജനാ വില്‍ നിക്ഷേപിക്കുകയും ചെയ്തിട്ടുണ്ട്.' സാധാരണഗതിയില്‍ ന്യാ യമെന്നു പറയാവുന്ന ഗവര്‍ണറുടെ നടപടിയെ ശരിവെക്കാന്‍ ഉമര്‍ തയാറായില്ല. പള്ളിയുടെ പ്രസ്തുത ഭാഗം പൊളിച്ചുമാറ്റി പകരം ആ ക്രൈസ്തവസ്ത്രീയുടെ വീട് യഥാസ്ഥാനത്ത് നിര്‍മിച്ചു കൊടു ക്കുവാന്‍ കല്‍പിക്കുകയാണ് നീതിമാനായ ആ ഭരണാധികാരി ചെ യ്തത്.

നാലാം ഖലീഫയായിരുന്ന അലി(റ )ടെ കാലത്ത് നടന്ന ഒരു സംഭവം നോക്കുക: ഖലീഫ തന്റെ പടയങ്കി ഒരു ജൂതന്‍ മോഷ്ടിച്ചു വെന്ന് പറഞ്ഞ് കോടതിയെ സമീപിച്ചു. ന്യായാധിപനായ ശുറൈഹ് ജൂതന്റെയും ഖലീഫയുടെയും അവകാശവാദങ്ങള്‍ പരിശോധിച്ചു. തന്റെ കൈവശമുള്ള പടയങ്കി തന്‍േറതു തന്നെയാണെന്നു ജൂതന്‍ വാദിച്ചപ്പോള്‍ അതു തന്‍േറതാണെന്നും ജൂതന്‍ മോഷ്ടിച്ചതാണെ ന്നും ഖലീഫ അലി മൊഴി നല്‍കി. തന്റെ വാദം തെളിയിക്കാനായി ഖലീഫ ഹാജരാക്കിയ സാക്ഷികള്‍ മകന്‍ ഹസനും ഭൃത്യന്‍ ഖമറുമായിരുന്നു. ഇവര്‍ രണ്ടുപേരും ഖലീഫയുടെ സ്വന്തക്കാരായ തിനാല്‍ സാക്ഷ്യം സ്വീകാര്യമല്ലെന്നും മറ്റു തെളിവുകള്‍ ഇല്ലാത്തതി നാല്‍ പടയങ്കി ജൂതനില്‍നിന്നു വാങ്ങുവാന്‍ നിയമം അനുവദിക്കു ന്നില്ലെന്നും ന്യായാധിപന്‍ വിധിച്ചു. ന്യായധിപന്റെയടുത്തുനിന്ന് പടയങ്കിവാങ്ങി നടന്നുനീങ്ങിയ ജൂതന്‍ അല്‍പദൂരം മുന്നോട്ടുപോ യി തിരിച്ചുവന്നുകൊണ്ട് പറഞ്ഞു: 'നിശ്ചയമായും ഇത് പ്രവാചക ന്മാരുടെ നിയമം തന്നെയെന്ന് ഞാന്‍ സാക്ഷ്യം വഹിക്കുന്നു. മതി യായ തെളിവില്ലാത്തതിനാല്‍ കോടതി അദ്ദേഹത്തിനെതിരെ വിധി പ്രസ്താവിക്കുന്നു. ഇതു വളരെ മഹത്തരം തന്നെ. അതിനാല്‍ അ ല്ലാഹുവല്ലാതെ ആരാധ്യനില്ലെന്നും മുഹമ്മദ്(സ) അല്ലാഹുവിന്റെ ദൂത നാണെന്നും ഞാന്‍ സാക്ഷ്യം വഹിക്കുന്നു'. തുടര്‍ന്ന് ഖലീഫയുടെ നേരെ തിരിഞ്ഞ് അദ്ദേഹം ഇങ്ങനെ പറഞ്ഞു: 'ഈ പടയങ്കി താങ്ക ളുടേത് തന്നെയാണ്. നിങ്ങള്‍ സ്വിഫ്ഫീനിലേക്ക് പോകുമ്പോള്‍ പിന്നിലുണ്ടായിരുന്ന ഞാന്‍ താങ്കളുടെ ചാരനിറത്തിലുള്ള ഒട്ടക ത്തിന്റെ പുറത്തുനിന്ന് അത് തട്ടിയെടുത്തതാണ്'. ഖലീഫ ചിരിച്ചു കൊണ്ട് പ്രതിവചിച്ചു. 'ഞാനത് താങ്കള്‍ക്ക് തന്നെ ദാനമായി തന്നി രിക്കുന്നു'.

ഇസ്‌ലാമിക രാഷ്ട്രത്തില്‍ ജീവിക്കുന്ന അമുസ്‌ലിം പൗരന്മാരുടെ വിശ്വാസസ്വാതന്ത്ര്യമോ ആരാധനാസ്വാതന്ത്ര്യമോ ഹനിക്കുവാന്‍ ഖലീഫമാര്‍ക്കവകാശമില്ല. അമുസ്‌ലിം പൗരന്മാരോട് അനീതി കാണിക്കുന്നത് അക്ഷന്തവ്യമായ അപരാധമാണെന്നാണ് പ്രവാചകന്‍പഠിപ്പിച്ചിരിക്കുന്നത്. അദ്ദേഹം പറഞ്ഞു: 'സൂക്ഷിച്ചുകൊ ള്ളുക. അമുസ്‌ലിം പൗരന്മാരെ ആരെങ്കിലും അടിച്ചമര്‍ത്തുകയോ അവരുടെമേല്‍ കഴിവിന്നതീതമായി നികുതിഭാരം കെട്ടിയേല്‍പി ക്കുകയോ അവരോട് ക്രൂരമായി പെരുമാറുകയോ അവരുടെ അവ കാശങ്ങള്‍ വെട്ടിക്കുറക്കുകയോ ചെയ്യുകയാണെങ്കില്‍ അന്ത്യനാളി ല്‍ അവന്നെതിരില്‍ ഞാന്‍ സ്വയംതന്നെ പരാതി ബോധിപ്പിക്കുന്ന താണ്'.

ചരിത്രപണ്ഡിതനായ സര്‍ തോമസ് ആര്‍നോള്‍ഡ് എഴുതുന്നത് കാണുക: 'മുഹമ്മദ് തന്നെ പല അറബ് ക്രൈസ്തവ ഗോത്രങ്ങളുമാ യും സന്ധിയിലേര്‍പെട്ടിരുന്നു. അവര്‍ക്ക് അദ്ദേഹം സംരക്ഷണവും സ്വന്തം മതമനുസരിച്ച് ജീവിക്കുവാനുള്ള സ്വാതന്ത്ര്യവും ഉറപ്പുവരു ത്തി. അവരുടെ പുരോഹിതര്‍ക്കുണ്ടായിരുന്ന സവിശേഷാധികാ രങ്ങള്‍ പഴയതുപോലെ നിലനിര്‍ത്തി'. പ്രവാചകന്റെ പാത പിന്‍തുടര്‍ന്ന അദ്ദേഹത്തിന്റെ ഖലീഫമാ രും അമുസ്‌ലിം പൗരന്മാരുടെ വിശ്വാസങ്ങളും ആരാധനാസമ്പ്രദായ ങ്ങളും തുടരാന്‍ അനുവദിച്ചിരുന്നതായും അവരുടെ ജീവനും സ്വ ത്തിനും പരിപൂര്‍ണ സംരക്ഷണം ഉറപ്പുവരുത്തിയിരുന്നതായും കാണാന്‍ കഴിയും. ഖലീഫ ഉമര്‍ േഈലിയാ നിവാസികള്‍ക്ക് നല്‍കിയ രക്ഷാകരാ റിലെ വ്യവസ്ഥകള്‍ കാണുക: 'ദൈവദാസനും വിശ്വാസികളുടെ നേതാവുമായ ഉമര്‍ ഈലിയാവാസികള്‍ക്ക് നല്‍കുന്ന സംരക്ഷണം. എല്ലാവരുടെയും ജീവന്നും സ്വത്തിനും ആരാധനാലയങ്ങള്‍ക്കും കുരിശുകള്‍ക്കും മതസംബന്ധിയായ എല്ലാറ്റിനും സംരക്ഷണം ഉറ പ്പുനല്‍കുന്നു. ആരുടെയും ചര്‍ച്ചുകള്‍ വാസസ്ഥലമാക്കുകയോ ന ശിപ്പിക്കുകയോ ചെയ്യരുത്. അവയോട് ചേര്‍ന്നു നില്‍ക്കുന്ന വസ്തു വഹകളോ കുരിശുകളോ പിടിച്ചെടുക്കരുത്. ആരെയും ഉപദ്രവിക്ക രുത്'."

ഇങ്ങനെ, അമുസ്‌ലിം പൗരന്മാരുടെ ജീവനും സ്വത്തും വിശ്വാസ സ്വാതന്ത്ര്യവും ആരാധനാസ്വാതന്ത്ര്യവുമെല്ലാം സംരക്ഷിക്കേ ണ്ടു ന്ന ചുമതലയേറ്റെടുക്കുന്ന ഇസ്‌ലാമിക രാഷ്ട്രത്തിന് അവന്‍ നല്‍കേണ്ടുന്ന നികുതിയാണ് ജിസ്‌യ. ഇസ്‌ലാമികരാഷ്ട്രങ്ങളിലും അല്ലാത്തിടത്തുമെല്ലാമുള്ള മുസ്‌ലിംകള്‍ നിര്‍ബന്ധമായും സകാത്ത് നല്‍കേണ്ടതുണ്ടെന്ന വസ്തുതയുടെ അടിസ്ഥാനത്തിലാണ് ഈ നികുതിയുടെ ന്യായാന്യായങ്ങള്‍ പരിശോധിക്കേണ്ടത്. തങ്ങളുടെ വാര്‍ഷികവരുമാനത്തിന്റെ രണ്ടരശതമാനവും പൊതുഖജനാവില്‍ അടക്കുവാന്‍ ഇസ്‌ലാമിക രാഷ്ട്രത്തില്‍ ജീവിക്കുന്ന മുസ്‌ലിം ബാധ്യസ്ഥനാണ്. സകാത്ത് മതപരമായ ഒരു ആരാധനാകര്‍മമായ തിനാല്‍ അമുസ്‌ലിംകളില്‍ നിന്ന് ഈടാക്കുവാന്‍ നിര്‍വാഹമില്ല. അതവരുടെ മതസ്വാതന്ത്ര്യത്തിന്റെ നിഷേധമായിരിക്കും. അതിനാ ല്‍ അമുസ്‌ലിം പൗരന്മാര്‍ക്ക് രാഷ്ട്രം നല്‍കുന്ന സംരക്ഷ ണത്തി ന്റെ ചെലവിലേക്കായി മറ്റൊരു നികുതി ഈടാക്കുവാനാണ് കല്‍പിക്കപ്പെട്ടിട്ടുള്ളത്. ജിസ്‌യ ഒരു മതനികുതിയല്ല; പ്രത്യുത സംരക്ഷണ നികുതിയത്രെ. ഇസ്‌ലാമിക രാഷ്ട്രത്തില്‍ ജീവിക്കുന്ന അമുസ്‌ലിം പൗരന്മാരുടെ ജീവന്റെ സംരക്ഷണം ഉറപ്പു നല്‍കുവാന്‍ തങ്ങള്‍ക്ക് കഴിയില്ലെന്ന അവസ്ഥ നിലനിന്നിരുന്ന യുദ്ധകാല സാഹചര്യങ്ങ ളില്‍ പിരിച്ചെടുത്ത 'ജിസ്‌യ' തിരിച്ചുവിതരണം ചെയ്യപ്പെട്ടിരുന്നതായി ചരിത്രത്തില്‍ കാണാനാവും. തങ്ങളുള്‍ക്കൊള്ളുന്ന രാജ്യ ത്തിന്റെ നിലനില്‍പിനുവേണ്ടി പൊരുതുകയും തങ്ങളുടെ സംര ക്ഷണം ഉറപ്പുവരുത്തുകയും ചെയ്യുന്ന ഇസ്‌ലാമിക ഗവണ്‍മെന്റിന് അമുസ്‌ലിം പൗരന്മാര്‍ നല്‍കുന്ന സംരക്ഷണ നികുതിയാണ് ജിസ് യയെന്ന് ഇതില്‍ നിന്ന് മനസ്സിലാക്കാം. രാഷ്ട്രത്തിന്റെ പ്രതി രോധ പ്രവര്‍ത്തനങ്ങളില്‍ പങ്കാളികളായിരുന്ന അമുസ്‌ലിം പൗരന്മാ രില്‍ നിന്ന് പ്രവാചകന്‍ ജിസ്‌യ ഈടാക്കിയിരുന്നില്ലെന്ന വസ്തുത യും ഇക്കാര്യം വ്യക്തമാക്കുന്നു.

ക്രൂരമാണ് ഇസ്‌ലാമിക ശിക്ഷാനിയമങ്ങളെന്ന് പറയുന്നത് ആ നിയമങ്ങളുടെ അടിത്തറയും അതുണ്ടാക്കാനുദ്ദേശിക്കുന്ന പരിവർത്തനത്തെയും കുറിച്ച് അറിയാത്തതു കൊണ്ടാണ്. വ്യക്തിക്കും സമൂഹത്തിനും സമാധാനം പ്രദാനം ചെയ്യുകയാണ് ഖുര്‍ആനിക നിയമങ്ങളുടെ ലക്ഷ്യം. വ്യക്തികള്‍ക്ക് ചില അവകാശങ്ങളുണ്ട്. ഇൗ അവകാശങ്ങള്‍ അന്യോന്യം അനുവദിച്ചുകൊടുക്കുക വഴിയാണ് സാമൂഹികമായ ഉദ്ഗ്രഥനം സാധ്യമാകുന്നത്. ഒരാളുടെയും അവകാശങ്ങള്‍ ഹനിക്കുവാന്‍ മറ്റൊരാളെയും അനുവദിച്ചുകൂടാ. ആരുടെയെങ്കിലും അവകാശങ്ങള്‍ ഹനിക്കപ്പെടുന്നുണ്ടോയെന്ന് ശ്രദ്ധിക്കേണ്ടതും ഉണ്ടെങ്കില്‍ അത് ഇല്ലാതെയാക്കേണ്ടതും രാഷ്ട്രത്തിന്റെ ബാധ്യതയാണ്. ഇതിനുവേണ്ടിയാണ് ശിക്ഷാനിയമങ്ങള്‍ നടപ്പിലാക്കുന്നത്. നേരായ മാര്‍ഗത്തിലൂടെ ചലിക്കുവാന്‍ വ്യക്തിയെ പ്രചോദിപ്പിക്കുകയാണ് ഖുര്‍ആനിലെ ശിക്ഷാനിയമങ്ങളുടെ ലക്ഷ്യം.

സംരക്ഷിക്കപ്പെടേണ്ട പ്രധാനപ്പെട്ട ചില മൂല്യങ്ങളുണ്ടെന്നാണ് ഇസ്‌ലാമിക വീക്ഷണം. വിശ്വാസം, യുക്തിയും ബുദ്ധിയും, അഭിമാനം,ജീവന്‍, സ്വത്ത്, കുടുംബത്തിന്റെ കെട്ടുറപ്പ്, സദാചാര മൂല്യങ്ങള്‍,സമൂഹത്തിന്റെ ഭദ്രത ഇവയെല്ലാം സംരക്ഷിക്കപ്പെടേണ്ടവയാണ്. ഇവ തകര്‍ക്കുവാന്‍ ആരെയും അനുവദിച്ചുകൂടാ. ആരെയും എന്നതുകൊണ്ട് അന്യനെ മാത്രമല്ല അര്‍ഥമാക്കുന്നത്; സ്വന്തത്തെകൂടിയാണ്. സ്വന്തം ജീവന്‍ വെടിയാനാഗ്രഹിച്ചുകൊണ്ട് ആത്മഹത്യക്കു ശ്രമിച്ചവനും സ്വന്തം മാനം തകര്‍ത്തുകൊണ്ട് വ്യഭിചാരവൃത്തിയിലേര്‍പ്പെട്ടവനും സ്വന്തം ബുദ്ധിയെ നശിപ്പിച്ചുകൊണ്ട് മദ്യപാനം ചെയ്യുന്നവനുമെല്ലാം കുറ്റവാളിയാകുന്നത് ഇതുകൊണ്ടാണ്.

സ്വന്തത്തെയോ അന്യനെയോ ഭയപ്പെടാതെ എല്ലാവര്‍ക്കും ജീവിക്കുവാന്‍ സാധിക്കുന്ന ഒരു സമൂഹമാണ് ഇസ്‌ലാമിക ശിക്ഷാനിയമങ്ങളുടെ ഉദ്ദേശ്യം. അത്തരമൊരു സമൂഹത്തില്‍ മാത്രമേ ശാന്തിയും സമാധാനവും നിലനില്‍ക്കൂ. എല്ലാവര്‍ക്കും വളരുവാനും വികസിക്കുവാനും സാധിക്കുന്ന, മാനവികതയില്‍ അധിഷ്ഠിതമായ ഒരു സമൂഹത്തിന്റെ സൃഷ്ടിയാണ് ഖുര്‍ആനിലെ ശിക്ഷാനിയമങ്ങള്‍ ലക്ഷ്യമാക്കുന്നത്.

വ്യക്തിയെയും സമൂഹത്തെയും പരിശുദ്ധമായി നിലനിര്‍ത്തുകയാണ് ഖുര്‍ആനിലെ ശിക്ഷാനിയമങ്ങളുടെ ലക്ഷ്യം. വ്യക്തിയെ സമൂഹത്തിനുവേണ്ടിയോ സമൂഹത്തെ വ്യക്തിക്കുവേണ്ടിയോ ബലികൊടുക്കണമെന്ന വീക്ഷണം ഇസ്‌ലാം ഉള്‍ക്കൊള്ളുന്നില്ല. വ്യക്തിസ്വാതന്ത്ര്യത്തിന്മേല്‍ സമൂഹത്തിന്റെ നേരിയ കൈകടത്തല്‍പോലും അക്ഷന്തവ്യമായിക്കരുതുന്ന മുതലാളിത്ത വീക്ഷണവും സമൂഹത്തിനുവേണ്ടി വ്യക്തിയുടെ സഹജവികാരങ്ങളെപ്പോലും ബലികൊടുക്കേണ്ടതുണ്ടെന്ന കമ്യൂണിസ്റ്റ് വീക്ഷണവും ഇസ്‌ലാമിന് അന്യമാണ്. വ്യക്തിയും സമൂഹവും തമ്മില്‍ നിലനില്‍ക്കേണ്ടത് സംഘട്ടനാത്മകമായ ബന്ധമല്ലെന്നതാണ് ഇസ്‌ലാമിന്റെ കാഴ്ചപ്പാട്. അവയെ ഉദ്ഗ്രഥിതമാക്കുന്നത് മൂല്യങ്ങളാണ്. ഇൗ മൂല്യങ്ങളെ സംരക്ഷിക്കുന്നതുവഴി വ്യക്തിയെയും സമൂഹത്തെയും വിമലീകരിക്കുകയാണ് ഖുര്‍ആനിലെ ശിക്ഷാനിയമങ്ങള്‍ ചെയ്യുന്നത്. അതുകൊണ്ടുതന്നെ അവ വ്യക്തികേന്ദ്രീകൃതമോ സമൂഹകേന്ദ്രീകൃതമോ അല്ല, പ്രത്യുത മൂല്യകേന്ദ്രീകൃതമാണ് എന്നു പറയുന്നതാവും ശരി.

ഇസ്‌ലാമിക രാഷ്ട്രത്തില്‍ നടപ്പാക്കേണ്ട ശിക്ഷാവിധികളെക്കു റിച്ച് പരാമര്‍ശിക്കുമ്പോള്‍ ഒരുകാര്യം വ്യക്തമാക്കേണ്ടതുണ്ട്. കുറ്റവാളികളെ ശിക്ഷിക്കുകയല്ല, പ്രത്യുത, കുറ്റകൃത്യ ങ്ങളില്ലാത്ത ഒരു സമൂഹത്തെ സൃഷ്ടിക്കുകയാണ് ഇസ്‌ലാമിന്റെ ലക്ഷ്യമെന്ന വസ്തുതയാണത്. കൊലപാതകിയെ കൊല്ലുകയോ മോഷ്ടാവിന്റെ കൈ വെട്ടുകയോ വ്യഭിചാരികളെ അടിക്കുകയോ ഒന്നുമല്ല, കൊലയും കൊള്ളയും അധാര്‍മികവൃത്തികളുമില്ലാത്ത ഒരു സമൂഹത്തെ സൃഷ്ടിക്കുകയാണ് ഇസ്‌ലാമിന്റെ ലക്ഷ്യം. ഇതിന് കര്‍ശനമായ ശിക്ഷാനിയമങ്ങള്‍ ആവശ്യമാണെന്നാണ് ഇസ്‌ലാമിന്റെ പക്ഷം. വ്യക്തിസ്വാതന്ത്ര്യത്തിന്റെയും പരിഷ്‌കാര ത്തിന്റെയും പേരില്‍ കുത്തഴിഞ്ഞ ജീവിതം അനുവദിക്കുന്ന 'പരി ഷ്‌കൃത'നാടുകളില്‍ കുറ്റകൃത്യങ്ങള്‍ വര്‍ധിച്ചുവരുന്നതിന്റെ കാര ണം അവിടത്തെ സമൂഹത്തിന്റെ ധാര്‍മികാടിത്തറയുടെ പിഴവും ശക്തമായ ശിക്ഷാനിയമങ്ങളുടെ അഭാവവുമാണെന്നാണ് സാമൂ ഹ്യ ശാസ്ത്രജ്ഞരുടെ പക്ഷം. കുറ്റകൃത്യങ്ങളില്‍ നിന്ന് മുക്തമായ ഒരു സമൂഹത്തിന്റെ സൃഷ്ടിക്ക് ശക്തമായ ശിക്ഷാനിയമങ്ങള്‍ ആവശ്യമാണെന്ന ഇസ്‌ലാമിന്റെ കാഴ്ചപ്പാട് നൂറ് ശതമാനവും മാനവികമാണെന്ന് സാരം.

ഒരു ശിക്ഷാനിയമം പ്രായോഗികമാണെന്ന് പറയാനാവുക അത് താഴെ പറയുന്ന ഗുണങ്ങള്‍ പ്രകടിപ്പിക്കുമ്പോഴാണ്.

ചെയ്ത തെറ്റിനുള്ള പ്രതികാരമാവുക.

തെറ്റുകളെ തടയാന്‍ കഴിയുക.

കുറ്റുവാളികളെ ഭയപ്പെടുത്താനാവുക.

കുറ്റം വഴി പ്രയാസമനുഭവിക്കേണ്ടിവന്നവര്‍ക്ക് സങ്കടനിവൃത്തി വരുത്തുന്നതാവുക.

കുറ്റവാളിയെ സംസ്‌കരിക്കുന്നതാവുക.

കുറ്റം വഴി നഷ്ടം നേരിട്ടവര്‍ക്ക് പരിഹാരം നല്‍കുന്നതാവുക.

കുറ്റവാളിയെ പാശ്ചാത്താപ വിവശനാക്കുന്നതാവുക.

സമൂഹത്തെ കുറ്റങ്ങളില്‍നിന്ന് സംരക്ഷിക്കുന്നതാവുക.

ഇവയിൽ ഒന്ന് പോലും ആധുനികാശിക്ഷാനിയമങ്ങൾ പൂര്തത്തീകരിക്കുന്നില്ല. അതുകൊണ്ട് തന്നെയാണ് ആധുനിക ജനാധിപത്യം നില നിൽക്കുന്ന നാടുകളിൽ കുറ്റകൃത്യങ്ങൾ വര്ധിച്ചുകൊണ്ടേയിരിക്കുന്നത്. ഇസ്‌ലാമിക നിയമങ്ങൾ നിലനിൽക്കുന്ന നാടുകളിലാണ് കുറ്റകൃത്യങ്ങൾ കുറവെന്ന വസ്തുത എല്ലാവരും അംഗീകരിക്കുന്നതാണ്. ഇസ്‌ലാമിലെ ഏതു ശിക്ഷാനിയമമെടുത്താലും ഈ ധര്‍മങ്ങള്‍ അവ നിര്‍വഹിക്കുന്നതായി കാണാന്‍ കഴിയും.  അത് തന്നെയാണ് ഇസ്‌ലാമിക ശിക്ഷാനിയമങ്ങളുടെ പ്രായോഗികതയും പ്രസക്തിയും.

സ്‌ലാമികരാഷ്ട്രം എന്ന് പറയുമ്പോള്‍ ഇസ്‌ലാം ഥിയോക്രസി അംഗീകരിക്കുന്നുവെന്ന് തെറ്റിദ്ധരിക്കുവാനിടയുണ്ട്. ഇസ്‌ലാം എല്ലാ അര്‍ഥത്തിലും ഥിയോക്രസിക്ക് എതിരാണ്. ദൈവത്തിന്റെ പേരില്‍ ഒരു പ്രത്യേക മതപുരോഹിത വര്‍ഗം അധികാരം കൈയാളുന്ന വ്യവസ്ഥിതിയാണ് ഥിയോക്രസി. പുരോഹിതന്മാരില്‍ പരിശുദ്ധാത്മാവിന്റെ സാന്നിധ്യമുണ്ടെന്നതിനാല്‍ അവര്‍ തെറ്റുപറ്റാത്തവരാണെന്ന ധാരണയാണ് ഈ വ്യവസ്ഥിതിയുടെ അടിത്തറ. അതു കൊണ്ടുതന്നെ ദൈവത്തിന്റെ പേരില്‍ പുരോഹിതസമൂഹം പറയുന്ന നിയമങ്ങള്‍ ഒരു ഥിയോക്രാറ്റിക് സ്‌റ്റെയിറ്റില്‍ ചോദ്യം ചെയ്യപ്പെടാതെ അംഗീകരിക്കപ്പെടുന്നു. പുരോഹിതന്‍ ദൈവത്തിന്റെപ്രതിനിധിയാണെന്നും ദൈവത്തിന്റെ ഭൂമിയില്‍ ഭരണംനടത്താനും നിയമനിര്‍മാണത്തിന്നും ദൈവപ്രതിനിധിക്കാണ് അധികാരമെന്നുമാണ് ഥിയോക്രസിയുടെ അടിസ്ഥാന തത്ത്വം.

ഇസ്‌ലാം ഭരണാധികാരിക്കോ പുരോഹിതനോ അപ്രമാദിത്വമുണ്ടെന്നോ അവർ ദൈവത്തിന്റെ പ്രതിനിധികളാണെന്നോ പഠിപ്പിക്കുന്നില്ല. ദൈവികനിയമങ്ങള്‍ക്ക നുസരിച്ച് വിധിനടത്തുവാന്‍ ജനങ്ങളാല്‍ തെരഞ്ഞെടുക്കപ്പെട്ട ഒരാള്‍ മാത്രമാണ് ഖലീഫ. അതുകൊണ്ടാണല്ലോ ഖലീഫാ ഉമര്‍(റ)ഒരു പൊതുപ്രസംഗത്തില്‍ താന്‍ തെറ്റായവഴിക്കു നീങ്ങുകയാണെങ്കില്‍ തന്നെ തിരുത്തണമെന്ന് ജനങ്ങളോട് അഭ്യര്‍ഥിച്ചപ്പോള്‍ വാള്‍ ഉയര്‍ത്തിപ്പിടിച്ചു കൊണ്ട് 'ഇതുകൊണ്ട് ഞങ്ങള്‍ താങ്കളെ തിരുത്തും' എന്ന് നിസ്സങ്കോചം പറയാന്‍ ഒരു സാധാരണ പൗരന്‍ ധൈര്യം കാണിച്ചത്.

ഒരു ഥിയോക്രാറ്റിക്‌സ്‌റ്റേറ്റില്‍ പൂരോഹിതന്‍ ദൈവപ്രതിനിധിയായിട്ടാണ് പരിഗണിക്കണപ്പെടുന്നതെന്നതിനാല്‍ തന്നെ അയാള്‍ പറയുന്നത് ചോദ്യംചെയ്യാതെ അംഗീകരിക്കപ്പെടുന്നു. ഇസ്‌ലാമിക രാഷ്ട്രത്തിലാവട്ടെ, ദൈവികവിധിവിലക്കുകള്‍ പ്രകാരം ഭരിക്കുവാനായി തെരഞ്ഞെടുക്കപ്പെട്ട ഒരു സാധാരണക്കാരനാണ് ഖലീഫ യെന്നതിനാല്‍ അയാളുടെ പ്രവര്‍ത്തനങ്ങളെയും വിജ്ഞാപനങ്ങളെയും മതത്തിന്റെ മൂലപ്രമാണങ്ങളുപയോഗിച്ച് വിമര്‍ശിക്കുവാന്‍ ഓരോ പൗരനും അവകാശമുണ്ട്. വരന്‍ വധുവിന് നല്‍കേണ്ട വിവാഹമൂല്യം (മഹ്ര്‍) വര്‍ധിച്ചുവരുന്നത് കണ്ട ഖലീഫ അതിന് പരിധി നിശ്ചയിക്കേണ്ടതിന്റെ ആവശ്യകതയെക്കുറിച്ച് ഒരു പൊതുപ്രസം ഗത്തില്‍ വ്യക്തമാക്കി. സദസ്സില്‍നിന്ന് ഒരു സ്ത്രീ എഴുന്നേറ്റുനിന്ന് പ്രമാണങ്ങളുടെ അടിസ്ഥാനത്തിലുള്ള ന്യായങ്ങളുദ്ധരിച്ച് അതിനെ എതിര്‍ത്തു. ന്യായങ്ങള്‍ ശരിയാണെന്ന് ബോധ്യപ്പെട്ട ഖലീഫാ ഉമര്‍(റ)മഹ്ര്‍ നിയന്ത്രിക്കുവാനുള്ള തന്റെ തീരുമാനം പിന്‍വ ലിച്ചു. ഖലീഫയുടെ തീരുമാനങ്ങളെയും വിജ്ഞാപനങ്ങളെയും ഏതൊരു സാധാരണ പൗരനും വിമര്‍ശിക്കാന്‍ കഴിയുന്ന ഇസ്ലാമിക ഭരണസംവിധാനം ഥിയോക്രസിക്ക് തികച്ചും വിപരീതമാണ്.

ഖലീഫയുടെ വ്യക്തിപരമായ കാര്യങ്ങളെക്കുറിച്ച് പോലും ചോദ്യംചെയ്യുവാനുള്ള അവകാശം ഇസ്‌ലാമികരാഷ്ട്രത്തിലെ പൗരന്മാര്‍ക്കുണ്ട്. ഒരിക്കല്‍ ഖലീഫാഉമര്‍(റ)ഒരു പൊതുപ്രസംഗം നടത്തിക്കൊണ്ടിരിക്കെ സദസ്സില്‍ നിന്നൊരാള്‍ എഴുന്നേറ്റ് നിന്ന് 'താങ്കള്‍ അണിഞ്ഞ മേലങ്കിയുടെ തുണി എവിടെ നിന്ന് ലഭിച്ചുവെന്ന് അറിയുന്നതുവരെ താങ്കള്‍ പറയുന്നത് കേള്‍ക്കാന്‍ ഞങ്ങള്‍ തയാറല്ല' എന്ന് പറഞ്ഞു. ജനങ്ങള്‍ക്ക് പൊതുഖജനാവില്‍ നിന്ന് വസ്ത്രം വിതരണംചെയ്ത സന്ദര്‍ഭമായിരുന്നു അത്. എല്ലാവരെയും പോലെ ഭരണാധികാരിക്കും ലഭിക്കേണ്ടത് തുല്യ ഓഹരിയാണെന്നും പ്രസ്തുത ഓഹരി ഉപയോഗിച്ച് ഉമറിനേ് ഉടുപ്പ് തയ്ക്കുവാനാവില്ലെന്നും മനസ്സിലാക്കിയ ആളായിരുന്നു ചോദ്യകര്‍ത്താവ്. അയാളുടെ വിമര്‍ശനത്തിന്റെ ഉന്നം മനസ്സിലാക്കിയ ഉമര്‍(റ)തന്റെപുത്രനായ അബ്ദുല്ല(റ)യെ വിളിച്ച് ആ ചോദ്യത്തിന് ഉത്തരം നല്‍കാനാവശ്യപ്പെട്ടു. തനിക്ക് ലഭിച്ച ഓഹരികൂടി താന്‍ പിതാവിന് നല്‍കിയെന്നും പ്രസ്തുത രണ്ട് ഓഹരികള്‍ കൂട്ടിച്ചേര്‍ത്താണ് തന്റെ പിതാവ് വസ്ത്രം തയ്ച്ചതെന്നും അബ്ദുല്ലയെവിശദീകരിച്ച ശേഷമാണ് ചോദ്യകര്‍ത്താവ് അടങ്ങിയിരുന്ന് ഖലീഫയുടെ പ്രസംഗം തുടരുവാനനുവദിച്ചത്.

വിയോജിക്കുവാനും വിമര്‍ശിക്കുവാനുമുള്ള സ്വാതന്ത്ര്യം ഇസ്‌ലാമിക രാഷ്ട്രത്തിലെ പൗരന്മാരുടെ മൗലി കാവകാശമാണെന്ന് മനസ്സിലാക്കാന്‍ ഇയൊരു സംഭവം തന്നെ ധാരാളമാണ്. ഇത്തരം സംഭവങ്ങള്‍ ഇസ്‌ലാമിക ചരിത്രത്തിലുടനീളം കാണാം. എന്നാല്‍ ദൈവികനിയമങ്ങളുടെയും നിര്‍ദേശങ്ങളുടെയും അടിസ്ഥാനത്തില്‍ ഭരണനിര്‍വഹണം നടത്തുന്ന ഖലീഫയെ അനുസരിക്കാന്‍ രാഷ്ട്രത്തിലെ പൗരന്മാരെല്ലാം ബാധ്യസ്ഥരാണ്. കൈകാര്യകര്‍ത്താക്കള്‍ (ഉലുല്‍അംറ്) എന്ന് ക്വുര്‍ആന്‍ വിശേഷിപ്പിച്ചിരിക്കുന്നത് ഇത്തരം ഭരണകര്‍ത്താക്കളെയാണ്. ക്വുര്‍ആന്‍ പറയുന്നത് കാണുക: 'സത്യവിശ്വാസികളേ, നിങ്ങള്‍ അല്ലാഹുവിനെ അനുസരിക്കുക. (അല്ലാഹുവിന്റെ) ദൂതനെയും നിങ്ങളില്‍ നിന്നുള്ള കൈകാര്യകര്‍ത്താക്കളെയും (ഉലുല്‍അംറ്) നിങ്ങള്‍ അനുസരിക്കുക'.

അല്ലാഹുവിന്റെയും ദൂതന്റെയും നിയമനിര്‍ദേശങ്ങള്‍ക്ക് വിധേ യമായി പ്രവര്‍ത്തിക്കുമ്പോള്‍ മാത്രമെ ഈ അനുസരണം ബാധ്യസ്ഥമാവുന്നുള്ളൂന്നെ് പ്രവാചകന്‍ പഠിപ്പിച്ചിട്ടുണ്ട്. 'വിരൂപനായ ഒരു അടിമയാണ് നിങ്ങളുടെ നേതാവെന്നിരിക്കിലും ദൈവത്തിന്റെ ഗ്രന്ഥമനുസരിച്ച് നിങ്ങളെ നയിക്കുന്നിടത്തോളം അയാളെ നിങ്ങള്‍ അനുസരിക്കണമെന്ന് നിഷ്‌കര്‍ഷിച്ച പ്രവാചകന്‍ തന്നെ 'നന്മയില്‍ പഠിപ്പിച്ചിട്ടുണ്ട്. ഇവയില്‍ നിന്ന് ഇസ്‌ലാമികരാഷ്ട്രത്തിന്റെ ഖലീഫ ദൈവത്തിന്റെ പ്രതിനിധിയായി അവരോധിതനായി തന്നിഷ്ടം പ്രവര്‍ത്തിക്കേണ്ടവനല്ലെന്നും അങ്ങനെ ചെയ്യുന്നുവെങ്കില്‍ അയാളെ അനുസരിക്കേണ്ട ബാധ്യത വിശ്വാസികള്‍ക്കില്ലെ ന്നും വ്യക്തമായി മനസ്സിലാക്കാന്‍ കഴിയുന്നു.

ഇസ്‌ലാമികരാഷ്ട്രം ഥിയോക്രാറ്റിക് അല്ലെങ്കില്‍ ഡമോക്രാറ്റിക് ആണോയെന്ന ചോദ്യം പ്രസക്തമാണ്. ജനങ്ങള്‍ക്ക് അഭിപ്രായം പറയുവാനും നീതിനേടിയെടുക്കുവാനുമുള്ള സംവിധാനങ്ങള്‍ നിലനില്‍ക്കുന്ന രാജ്യത്തിന്നാണ് ഡെമോക്രാറ്റിക് എന്നു പറയു ന്നതെങ്കില്‍ ഇസ്‌ലാമികരാഷ്ട്രം നൂറുശതമാനവും ഡമോക്രാറ്റിക് ആണെന്ന് പറയാവുന്നതാണ്. എന്നാല്‍ ജനഹിതത്തിന്റെ പേരില്‍ അധാര്‍മികതകളും അനാശാസ്യപ്രവര്‍ത്തനങ്ങളും അനുവദിക്കപ്പെടുകയെന്നാണ് ഡമോക്രസി അര്‍ഥമാക്കുന്നതെങ്കില്‍ അതിന്ന് ഇസ്‌ലാം നൂറുശതമാനം എതിരാണ്. ഇസ്‌ലാമിക മൂല്യങ്ങളുടെ അടിസ്ഥാനത്തില്‍ മാത്രമെ രാഷ്ട്രത്തില്‍ ജനഹിതം നടപ്പാക്കപ്പെടുകയുള്ളൂവെന്ന് സാരം. ഭൂരിപക്ഷം ജനങ്ങള്‍ വ്യഭിചാരം സാര്‍വത്രികമാക്കണമെന്ന് അഭിപ്രായം പറഞ്ഞാലും ഇസ്‌ലാമികരാഷ്ട്ര ത്തില്‍ വ്യഭിചാരം അനുവദിക്കപ്പെടുകയില്ല. കാരണം, അത് വിശുദ്ധ ക്വുര്‍ആന്‍ പ്രദാനംചെയ്യുന്ന മൂല്യസങ്കല്‍പത്തിന് വിരുദ്ധമാണ്. ഭൂരിപക്ഷത്തിന്റെ പിന്തുണ നേടിയെടുക്കാന്‍ വേണ്ടി അവര്‍ക്കാവശ്യമുള്ളതെല്ലാം ചെയ്തുകൊടുക്കുകയെന്ന ജനായത്തരാഷ്ട്രീയത്തിന്റെ കറുത്ത മുഖം ഇസ്‌ലാമിക രാഷ്ട്രത്തിനുണ്ടാവില്ല. അവിടെ ജനഹിതം പരിശോധിക്കപ്പെടുന്നത് വ്യക്തമായ ധാര്‍മിക നിയമങ്ങളുടെ വെളിച്ചത്തിലായിരിക്കും. അതുകൊണ്ടുതന്നെ ഭൂരിപക്ഷത്തിനു വേണ്ടി എന്തു തോന്നിവാസവും ചെയ്യുവാന്‍ ഇസ്‌ലാമികരാഷ്ട്രത്തിലെ ഭരണാധികാരികള്‍ക്ക് കഴിയില്ല.

ഈ അര്‍ഥത്തില്‍, ഇസ്ലാമിക രാഷ്ട്രം ഥിയോക്രസിക്കും ഡമോക്രസിക്കും മധ്യെയാണെന്നു പറയാം. ഈ രണ്ടു മീമാംസകളിലെയും നല്ല വശങ്ങള്‍ ഇസ്‌ലാമികരാഷ്ട്രം ഉള്‍ക്കൊള്ളുന്നുണ്ടെന്നു സാരം.

 ദൈവം ഒന്നേയുള്ളൂ എന്ന ഇസ്‌ലാമിന്റെ വാദം അനാവശ്യമായ ഒരു വാശിയല്ലേ? ദൈവത്തിന്റെ ഏകത്വത്തിന്‌ പ്രപഞ്ചത്തിൽ വല്ല തെളിവുമുണ്ടോ?

  • പ്രാപഞ്ചികപ്രതിഭാസങ്ങള്‍ ചൂണ്ടിക്കാണിച്ചുകൊണ്ട് സര്‍വശക്തനായ സ്രഷ്ടാവിന്റെ അസ്തിത്വം സ്ഥാപിക്കുന്ന വിശുദ്ധക്വുര്‍ആന്‍ ബഹുദൈവസങ്കല്‍പത്തിന്റെ നിരര്‍ഥകതയിലേക്കു വിരല്‍ചൂണ്ടുന്നതും പ്രകൃതിയിലെ താളക്രമങ്ങളെക്കുറിച്ച് സൂചിപ്പിച്ചുകൊണ്ടാണ്. മനുഷ്യശരീരത്തിലെ ഓരോ വ്യവസ്ഥയും പരസ്പരം ബന്ധപ്പെട്ടാണിരിക്കുന്നത്. വ്യത്യസ്ത ജന്തുജാലങ്ങള്‍ തമ്മിലുള്ള ബന്ധം സുവിദിതമാണല്ലോ. ചെറുതും വലുതുമായ ജന്തുക്കള്‍ പരസ്പരം ആശ്രയിച്ചാണ് ജീവിക്കുന്നത്. ജന്തുക്കളും സസ്യങ്ങളും സ്വന്തം നിലനില്‍പിനുവേണ്ടി പരസ്പരം ആശ്രയിക്കുന്നു. ജന്തുക്കളില്ലെങ്കില്‍ സസ്യങ്ങള്‍ക്കോ സസ്യങ്ങളില്ലെങ്കില്‍ ജന്തുക്കള്‍ക്കോ നിലനില്‍ക്കാന്‍ സാധിക്കാത്ത രീതിയിലാണ് ഭൂമി സംവിധാനിക്കപ്പെട്ടിരിക്കുന്നത്. ഓരോ ജീവിക്കും വ്യത്യസ്ത സ്രഷ്ടാക്കളായിരുന്നുവെങ്കില്‍ അവ തമ്മിലുള്ള പാരസ്പര്യം നിലനില്‍ക്കുമായിരുന്നില്ല. സസ്യങ്ങളുടെ സ്രഷ്ടാവും ജന്തുക്കളുടെ സ്രഷ്ടാവും തങ്ങളിച്ഛിക്കുന്ന രീതിയില്‍ അവയെ നിയന്ത്രിച്ചിരുന്നുവെങ്കില്‍ ആ രണ്ടു വിഭാഗവും നിലനില്‍ക്കുമായിരുന്നില്ല. ഈ വസ്തുതയിലേക്ക് ക്വുര്‍ആന്‍ വെളിച്ചം വീശുന്നത് നോക്കുക: 'അല്ലാഹു യാതൊരു സന്താനത്തെയും സ്വീകരിച്ചിട്ടില്ല. അവനോടൊപ്പം യാതൊരു ദൈവവുമുണ്ടായിട്ടുമില്ല. അങ്ങനെയായിരുന്നുവെങ്കില്‍ ഓരോ ദൈവവും താന്‍ സൃഷ്ടിച്ചതുമായി പൊയ്ക്കളയുകയും അവരില്‍ ചിലര്‍ ചിലരെ അടിച്ചമര്‍ത്തുകയും ചെയ്യുമായിരുന്നു. നിങ്ങള്‍പറഞ്ഞുണ്ടാക്കുന്നതില്‍ നിന്നെല്ലാം അല്ലാഹു എത്ര പരിശുദ്ധന്‍'! (23:91)

ഈ താളക്രമം പ്രാപഞ്ചികവ്യവസ്ഥയിലുടനീളം കാണപ്പെടുന്നുവെന്നതാണ് വസ്തുത. പദാര്‍ഥത്തിന്റെ ഏറ്റവും ചെറിയ കണമായ ആറ്റത്തിനകത്തെ ഇലക്‌ട്രോണുകളും പ്രോട്ടോണുകളും ന്യൂട്രോണുകളും മീസോണുകളും പോസിട്രോണുകളുമെല്ലാം തമ്മിലുള്ള പാരസ്പര്യം വിസ്മയാവഹമാണ്. സ്ഥൂലപ്രപഞ്ചത്തിന്റെ കാര്യത്തിലും സ്ഥിതി ഇതുതന്നെ. പ്രാപഞ്ചിക നിയമങ്ങളുടെ ഐക്യ രൂപ്യം അവയെയെല്ലാം സൃഷ്ടിച്ചത് ഒരേ സ്രഷ്ടാവ് തന്നെയാണെന്ന വസ്തുത വ്യക്തമാക്കുന്നു. വ്യത്യസ്ത ദൈവങ്ങളായിരുന്നു പ്രപഞ്ചത്തിലെ വ്യത്യസ്ത പ്രതിഭാസങ്ങള്‍ക്കു പിന്നിലെങ്കില്‍ അവ തമ്മില്‍ താളപ്പൊരുത്തം ഉണ്ടാവുന്നതെങ്ങനെയാണ്്? സ്ഥലകാലനൈരന്തര്യത്തില്‍ ദ്രവ്യമുണ്ടാക്കുന്ന വളവുകളാണ് പ്രപഞ്ചത്തില്‍ ഇന്നു കാണപ്പെടുന്ന പ്രതിഭാസങ്ങളുടെയെല്ലാം അടിസ്ഥാനകാരണമെന്ന ഐന്‍സ്റ്റയിന്റെ ആപേക്ഷികതാസിദ്ധാന്തം അംഗീകരിച്ചാല്‍ വ്യത്യസ്ത പ്രാപഞ്ചികവസ്തുക്കളെയെല്ലാം പ്രസ്തുത നിയമത്തിന് വിധേയനാക്കിയ ഏകനായ സ്രഷ്ടാവിന്റെ അസ്തിത്വം അംഗീകരിക്കേണ്ടിവരും. ഒരൊറ്റ പ്രാപഞ്ചികവസ്തു പോലും പ്രസ്തുത നിയമത്തിനതീതമല്ലെന്നാണല്ലോ ഐന്‍സ്റ്റയിന്‍ സിദ്ധാന്തിച്ചത്. വ്യത്യസ്ത ആകാശഗോളങ്ങള്‍ക്ക് വ്യത്യസ്ത സ്രഷ്ടാക്കളായിരുന്നു ഉണ്ടായിരുന്നതെങ്കില്‍ അവ തമ്മില്‍ കൂട്ടിയിടിച്ചു എന്നോ നശിച്ചുപോകുമായിരുന്നു. പ്രപഞ്ചത്തിന്റെ ആധിപത്യത്തില്‍ ഒന്നിലേറെ ശക്തികള്‍ക്ക് പങ്കുണ്ടായിരുന്നുവെങ്കില്‍ അവരുടെ താല്‍പര്യങ്ങള്‍ തമ്മിലുള്ള സംഘട്ടനംപ്രപഞ്ചത്തിന്റെയാകെ നാശത്തിന് കാരണമാകുമായിരുന്നു. ഈ വസ്തുതയിലേക്ക് വിശുദ്ധ ക്വുര്‍ആന്‍ സൂചന നല്‍കുന്നത് ശ്രദ്ധിക്കുക: 'ആകാശ ഭൂമികളില്‍ അല്ലാഹുവല്ലാത്ത ദൈവങ്ങളുണ്ടായിരുന്നുവെങ്കില്‍ അത് രണ്ടും തകരാറാകുമായിരുന്നു. അപ്പോള്‍ സിംഹാസനത്തിന്റെന്‍നാഥനായ അല്ലാഹു, അവര്‍ പറഞ്ഞുണ്ടാക്കുന്നതില്‍ നിന്നെല്ലാം എത്ര പരിശുദ്ധനാകുന്നു!' (21:22)

ഈ പ്രപഞ്ചത്തിന് ഒരു ആസൂത്രകനുണ്ടെന്നത് അന്ധമായ വിശ്വാസമല്ല; ചിന്തിക്കുന്ന ആർക്കും അനുഭവപ്പെടുന്ന ഒരു യാഥാർഥ്യമാണ്. സ്രഷ്ടാവുണ്ടെന്ന അറിവ് കേവല വിശ്വാസത്തിലുപരിയായി തന്നെയും ചുറ്റുപാടുകളെയും ഗൗരവതരമായി നോക്കിക്കാണുന്നവർക്കെല്ലാമുണ്ടാവുന്ന യഥാതഥമായ അനുഭവമാണ്. അതുകൊണ്ട് തന്നെ, തന്റെ ചുറ്റുപാടുകളിലേക്ക് കണ്ണോടിച്ചുകൊണ്ട് പ്രപഞ്ചസംവിധാനത്തിനുപിന്നിലെ മഹാചൈതന്യത്തെക്കുറിച്ച് മനസ്സിലാക്കുവാനാണ് ക്വുര്‍ആനിന്റെ കല്‍പന. കണ്ണുകൊണ്ട് കാണുകയും കാതുകൊണ്ട് കേള്‍ക്കുകയും ചെയ്ത് മനസ്സുപയോഗിച്ച് ചിന്തിക്കുവാനാണ് സ്രഷ് ടാവിന്റെ അസ്തിത്വത്തിലേക്ക് ശ്രദ്ധക്ഷണിക്കുന്ന ക്വുര്‍ആന്‍ സൂക്തങ്ങള്‍ മനുഷ്യരോടാവശ്യപ്പെടുന്നത്. അന്ധമായി ദൈവവിശ്വാസിയാകുവാന്‍ പഠിപ്പിക്കുന്ന ഒരൊറ്റ സൂക്തം പോലും ക്വുര്‍ആനിലില്ല. ഈ വിഷയത്തില്‍ ഇസ്‌ലാമിനെപ്പോലെ ബുദ്ധിയെ പ്രവര്‍ത്തനക്ഷമമാക്കിയ മറ്റൊരു മതവുമില്ലെന്നതാണ് വസ്തുത. തന്നിലേക്കും തന്റെ ചുറ്റുപാടുകളിലേക്കും ദൃഷ്ടിപായിച്ച് പടച്ചവന്റെഅജയ്യതയെക്കുറിച്ച് മനസ്സിലാക്കുവാന്‍ മനുഷ്യരോട് ആഹ്വാനംചെയ്യുന്ന ക്വുര്‍ആന്‍ സൂക്തങ്ങള്‍ സാധാരണക്കാര്‍ മുതല്‍ ശാസ് ത്രജ്ഞര്‍വരെയുള്ളവരെ ചിന്തിപ്പിക്കുവാന്‍ പര്യാപ്തമാണ്.

പ്രവാചകന് ആദ്യമായി അവതരിപ്പിക്കപ്പെട്ട ക്വുര്‍ആന്‍ സൂക്തങ്ങള്‍ തന്നെ നോക്കുക.: 'സൃഷ്ടിച്ചവനായ നിന്റെ നാഥന്റെ നാമ ത്തില്‍ വായിക്കുക; മനുഷ്യനെ അവന്‍ ഭ്രൂണത്തില്‍ നിന്ന് സൃഷ്ടിച്ചിരിക്കുന്നു'. (96:1, 2).

മനുഷ്യരോട് സ്വന്തത്തെക്കുറിച്ച് ചിന്തിക്കുവാനാണ് ഈ സൂക്തം ആഹ്വാനംചെയ്യുന്നത്. ഏതാനും വര്‍ഷങ്ങള്‍ക്കുമുമ്പ് നാം ഒന്നുമല്ലായിരുന്നു. പിന്നെ പിതാവിന്റെ ശരീരത്തിലെ കോടിക്കണക്കിന് ബീജങ്ങളിലൊന്നും മാതാവിന്റെ ശരീരത്തിലെ അണ്ഡങ്ങളിലൊന്നുമായി, തികച്ചും അപരിചിതമായ രണ്ട് ഭാഗങ്ങളായിരുന്നു നാം. മാതാവിന്റെ ഗര്‍ഭപാത്രത്തില്‍വെച്ച് ബീജവും അണ്ഡവു ചേര്‍ന്ന സിക്താണ്ഡമു ണ്ടായി. അത് വളര്‍ന്ന് മാംസപിണ്ഡമായി. പിന്നീട് ഈ പിണ്ഡത്തിനകത്ത് അസ്ഥികൂടം രൂപപ്പെട്ടു. പ്രസ്തുത അസ്ഥികള്‍ പേശികളാല്‍ മറയ്ക്കപ്പെട്ടു. കണ്ണുകളും കാതുകളും നാക്കും മൂക്കും കൈകളും കാലുകളുമെല്ലാമുണ്ടായി. അങ്ങനെ ഒരു ദിവസം നാം ഭൂമിയിലേക്ക് വന്നു. ഇവിടത്തെ കോടിക്കണക്കിന് മനുഷ്യരില്‍ ഒരാളായി നാം ജീവിച്ചുകൊണ്ടിരിക്കുന്നു. നമ്മുടെ ശരീരത്തില്‍ വ്യത്യസ്തങ്ങളായ വ്യവസ്ഥകളുണ്ട്. പ്രസ്തുത വ്യവസ്ഥകളെല്ലാം പരസ്പരം ബന് ധപ്പെട്ടാണിരിക്കുന്നത്. അസ്ഥിവ്യൂഹവും ചംക്രമണവ്യൂഹവും മൂത്ര വ്യൂഹവും പ്രത്യുല്‍പാദനവ്യൂഹവും അന്തഃസ്രാവവ്യൂഹവുമൊന്നും മനുഷ്യന്റെ നിയന്ത്രണത്തിലല്ല നടക്കുന്നത്. അപരിചിതങ്ങളായ ബീജത്തെയും അണ്ഡത്തെയും പരസ്പരം സംയോജിപ്പിച്ച് സിക്താണ്ഡമുണ്ടാക്കി, അതില്‍ നിന്നു നമ്മെ പടിപടിയായി വളര്‍ത്തിക്കൊണ്ടുവന്നവനാരോ അവന്‍ തന്നെ ഈ വ്യവസ്ഥകളെ നിയന്ത്രിച്ചുനടത്തുവാനാവശ്യമായ സംവിധാനങ്ങള്‍ ശരീരത്തില്‍ ചെയ്തുവെച്ചിരിക്കുന്നു. സ്വന്തത്തെക്കുറിച്ച് ചിന്തിക്കുന്ന സാധാരണക്കാരനും മനുഷ്യശരീരത്തെക്കുറിച്ച് ഗവേഷണം നടത്തുന്ന ശാസ്ത്രകാരനും ഒരേപോലെ സ്രഷ്ടാവിന്റെ വൈഭവം അനുഭവിച്ചറിയുന്നു.ക്വുര്‍ആന്‍ ചോദിക്കുന്നു: 'നിങ്ങള്‍ക്കെങ്ങനെയാണ് അല്ലാഹുവിനെ നിഷേധിക്കാന്‍ കഴിയുക? നിങ്ങള്‍ നിര്‍ജീവ വസ്തുക്കളായിരുന്ന അവസ്ഥക്ക് ശേഷം അവന്‍ നിങ്ങള്‍ക്ക് ജീവന്‍ നല്‍കി'. (2:28)

'തീര്‍ച്ചയായും മനുഷ്യനെ കളിമണ്ണിന്റെ സത്തില്‍ നിന്ന് നാം സൃഷ്ടിച്ചിരിക്കുന്നു. പിന്നീട് ഒരു ബീജമായിക്കൊണ്ട് അവനെ നാം ഭദ്രമായ ഒരു സ്ഥാനത്ത്‌വെച്ചു. പിന്നെ ആ ബീജത്തെ നാം ഒരു ഭ്രൂണമായി രൂപപ്പെടുത്തി. അനന്തരം ആ ഭ്രൂണത്തെ നാം ഒരു മാംസപിണ്ഡമായി രൂപപ്പെടുത്തി. തുടര്‍ന്ന് നാം ആ മാംസപിണ്ഡത്തെ അസ്ഥികൂടമായി രൂപപ്പെടുത്തി. എന്നിട്ട് നാം അസ്ഥികൂടത്തെ മാംസം കൊണ്ട് പൊതിഞ്ഞു. പിന്നീട് മറ്റൊരു സൃഷ്ടിയായി നാം അതിനെ വളര്‍ത്തിയെടുത്തു. അപ്പോള്‍ ഏറ്റവും നല്ല സൃഷ്ടി കര്‍ത്താവായ അല്ലാഹു അനുഗ്രഹപൂര്‍ണനായിരിക്കുന്നു'. (23:12-14)

'അപ്പോള്‍ നിങ്ങള്‍ സ്രവിക്കുന്ന ശുക്ലത്തെപ്പറ്റി നിങ്ങള്‍ ചിന്തിച്ചുനോക്കിയിട്ടുണ്ടോ? നിങ്ങളാണോ അവ സൃഷ്ടിച്ചുണ്ടാക്കുന്നത്? അതല്ല, നാമാണോ സൃഷ്ടികര്‍ത്താവ്?' (56:58,59)

പെറ്റുവീഴുന്ന കുഞ്ഞിന് ആവശ്യമായതെല്ലാം സ്രഷ്ടാവ് അവനുചുറ്റും ഒരുക്കിവെച്ചിരിക്കുന്നു. കരയുമ്പോള്‍ തൊണ്ട വരളാതിരിക്കാനുള്ള സംവിധാനം. ശരീരത്തിന്റെ ഊഷ്മാവ് നിലനിര്‍ത്തുവാനുള്ള സംവിധാനം. ശരീരത്തിന്റെ ജലാംശം നിലനിര്‍ത്തുവാനുള്ള സംവിധാനം. ഓക്‌സിജന്‍ വലിച്ചെടുക്കുവാനും കാര്‍ബണ്‍ഡയോക്‌സൈഡ് ഉഛ്വസിക്കാനുമാവശ്യമായ സംവിധാനങ്ങള്‍. ഇങ്ങനെ എന്തെന്ത് സംവിധാനങ്ങളാണ് സ്രഷ്ടാവ് നമുക്കായി ഒരുക്കിവെച്ചിരിക്കുന്നത്! കുഞ്ഞിന് ആവശ്യമായ സമീകൃതാഹാരം തയാര്‍ചെയ്യുന്നത് മാതാവ് തന്നെയാണ്. കുഞ്ഞിന് കൊടുക്കാന്‍ മുലപ്പാലിനോളം നല്ലൊരു ആഹാരവുമില്ലെന്നാണ് വാസ്തവം. കുഞ്ഞിന്റെ വളര്‍ച്ചക്കും പ്രതിരോധത്തിനുമാവശ്യമായ എല്ലാ പോഷകമൂല്യങ്ങളും മുലപ്പാലിലടങ്ങിയിരിക്കുന്നു. മുലപ്പാല്‍ കുടിച്ചുവളരുന്ന കുട്ടിക്ക് അല്‍പം പ്രായമാകുന്നതോടുകൂടി നല്‍കാനാവശ്യമായ പോഷകസമൃദ്ധമായ ഭക്ഷ്യവസ്തുക്കളും അവന്റെ ചുറ്റുമുണ്ട്. അതില്‍ പ്രധാനപ്പെട്ട ഒന്നാണ് ആടുകളുടെയും മാടുകളുടെയും പാല്. പശുവിന്റെയും എരുമയുടെയും പാല്‍ കുട്ടികള്‍ക്ക് മാത്രമല്ല, വലിയവര്‍ക്കും പോഷകം നല്‍കുന്ന ഗുണസമ്പുഷ്ടമായ ഒരു ആഹാരമാണ്. പശുവിന്‍പാലില്‍ 87.2% വെള്ളവും 3.7% കൊഴുപ്പും 4.9% പഞ്ചസാരയും 3.5% പോഷകങ്ങളും മറ്റ് ഒട്ടേറെ ധാതുക്കളും ജീവകങ്ങളും അടങ്ങിയിട്ടുണ്ട്. ആടുകളുടെയും മാടുകളുടെയും ശരീരത്തില്‍ മനുഷ്യനാവശ്യമായ പാല്‍ നിര്‍മിക്കാന്‍വേണ്ട സംവിധാനങ്ങള്‍ ചെയ്തുവെച്ചത് ആരാണ്? അവയല്ല; നമ്മളുമല്ല. അവയെ യും നമ്മെയും സൃഷ്ടിച്ച തമ്പുരാന്‍ തന്നെയാണെന്ന ഉത്തരമാണ് ഈ ചോദ്യത്തിന് പാല്‍ക്കാരനും ക്ഷീരഗവേഷകനും നല്‍കുവാനുള്ളത്. ക്വുര്‍ആന്‍ പറയുന്നത് നോക്കുക: 'തീര്‍ച്ചയായും നിങ്ങള്‍ക്ക് കന്നുകാലികളില്‍ ഒരു ഗുണപാഠമുണ്ട്. അവയുടെ ഉദരങ്ങ ളിലുള്ളതില്‍നിന്ന് നിങ്ങള്‍ക്ക് നാം കുടിക്കാന്‍ തരുന്നു. നിങ്ങള്‍ക്ക് അവയില്‍ ധാരാളം പ്രയോജനങ്ങളുണ്ട്. അവയില്‍നിന്ന് (മാംസം) നിങ്ങള്‍ ഭക്ഷിക്കുകയുംചെയ്യുന്നു'. 'കാലികളുടെ കാര്യത്തില്‍ തീര്‍ച്ചയായും നിങ്ങള്‍ക്കൊരു പാഠമു ണ്ട്. അവയുടെ ഉദരങ്ങളില്‍ ഉള്ളതില്‍നിന്ന്-കാഷ്ടത്തിനും രക്ത ത്തിനുമിടയില്‍നിന്ന് കുടിക്കുന്നവര്‍ക്ക് സുഖദായകമായ പാല്‍ നിങ്ങള്‍ക്ക് കുടിക്കാനായി നാം നല്‍കുന്നു' (23:21) നമ്മുടെ നിലനില്‍പിനുതന്നെ ആധാരമായിട്ടുള്ള സസ്യലതാതികളെക്കുറിച്ച് ചിന്തിക്കുമ്പോഴും സ്രഷ്ടാവിന്റെ വൈഭവം നമുക്ക് ബോധ്യമാകുന്നു. പച്ചിലകളാണ് ഭൂമിയുടെ ഭക്ഷ്യനിര്‍മാണശാല. വേരുകള്‍ വെള്ളവും വളവും വലിച്ചെടുക്കുന്നു. അവ കാണ്ഡത്തിലൂടെ ഇലകളിലെത്തുന്നു. അന്തരീക്ഷത്തില്‍ നിന്ന് അവ കാര്‍ബണ്‍ഡയോക്‌സൈഡ് വലിച്ചെടുക്കുന്നു. ഇവയെല്ലാംകൂടി സൂര്യപ്രകാശത്തിന്റെ സാന്നിധ്യത്തില്‍ ഇലകളിലെ ഹരിതകത്തില്‍വെച്ച് ഗ്ലൂക്കോസായി മാറുന്നു. ഈ പ്രവര്‍ത്തനത്തെയാണ് പ്രകാശസം ശ്ലേഷ ണം (photosynthesis) എന്നു പറയുന്നത്. പ്രകാശസംശ്ലേഷണത്തിലൂടെ നിര്‍മിക്കപ്പെടുന്ന ഗ്ലൂക്കോസാണ് സസ്യങ്ങളുടെയും ജന്തുക്കളുടെയും അടിസ്ഥാന ഭക്ഷണം. ജന്തുക്കള്‍ പുറത്തുവിടന്ന കാര്‍ബണ്‍ഡയോക്‌സൈഡ് വലിച്ചെടുത്ത് പകരം ജന്തുക്കള്‍ക്കാവശ്യമുള്ള ഓക്‌സിജന്‍ പുറത്തുവിടുകയെന്ന പ്രവര്‍ത്തനംകൂടി പ്രകാശസംശ്ലേഷണത്തോടൊപ്പം നടക്കുന്നുണ്ട്. ജന്തുക്കളുടെയും സസ്യങ്ങളുടെയും നിലനില്‍പ് പരസ്പരം പൂരകമായിട്ടാണെന്നര്‍ഥം. ജന്തുക്കളില്ലെങ്കില്‍ സസ്യങ്ങള്‍ക്കോ, തിരിച്ചോ നിലനില്‍ക്കാന്‍ കഴിയാത്ത അവസ്ഥ. സസ്യജാലങ്ങളുടെ നിലനില്‍പിന് പിന്നില്‍ കൃഷിക്കാരനും പക്ഷിശാസ്ത്രജ്ഞനും ഒരേപോലെ സ്രഷ്ടാവിന്റെ അസ്തിത്വം അനുഭവിച്ചറിയുന്നു. വിശുദ്ധക്വുര്‍ആന്‍ ചോദിക്കുന്നു: 'എന്നാല്‍ നിങ്ങള്‍ കൃഷിചെയ്യുന്നതിനെക്കുറിച്ച് നിങ്ങള്‍ ചിന്തിച്ചുനോക്കിയിട്ടുണ്ടോ? നിങ്ങളാണോ അത് മുളപ്പിച്ച്‌വളര്‍ത്തുന്നത്. അതല്ല, നാമാണോ അത് മുളപ്പിച്ച് വളര്‍ത്തുന്നവന്‍?' (56:63,64)

വിത്ത് കുഴിച്ചിടുന്ന കൃഷിക്കാരന് അത് മുളച്ച് വളര്‍ന്ന് ഉല്‍പാദനക്ഷമമാക്കുന്നതില്‍ മൗലികമായ യാതൊരു പങ്കുമില്ല. വിത്തിന്റെ ബീജകോശത്തിലെ ജീനുകളില്‍ രേഖപ്പെടുത്തപ്പെട്ട വ്യവസ്ഥയനുസരിച്ചാണ് സസ്യവളര്‍ച്ച നടക്കുന്നത്. ആ വ്യവസ്ഥ തീര്‍ത്തും ദൈവിക മത്രെ.'അത് (വെള്ളം) മൂലം ധാന്യവിളകളും ഒലീവും ഈത്തപ്പനയും മുന്തിരികളും അവന്‍ നിങ്ങള്‍ക്ക് മുളപ്പിച്ചുതരുന്നു. എല്ലാത്തരം ഫലവര്‍ഗങ്ങളും (അവന്‍ ഉല്‍പാദിപ്പിച്ചുതരുന്നു). ചിന്തിക്കുന്ന ആളുകള്‍ക്ക് തീര്‍ച്ചയായും അതില്‍ ദൃഷ്ടാന്തമുണ്ട്'. (16:11)

കാര്‍ഷികവിളകളുടെയും ജന്തുവര്‍ഗങ്ങളുടെയുമെല്ലാം നിലനില്‍പിനാവശ്യമായ സജ്ജീകരണങ്ങള്‍ പ്രകൃതിയില്‍ തന്നെയുണ്ട്. ഭൂമി സ്വയം ഭ്രമണംചെയ്യുന്നത് 23.50 ചെരിഞ്ഞിട്ടാണ്. ഈ ചെരിവാണ് ഋതുഭേദങ്ങള്‍ക്ക് കാരണമാകുന്നത്. ഋതുഭേദങ്ങളാണല്ലോ ഭൂമിയെ കൃഷിക്ക് പറ്റിയ ഗ്രഹമാക്കി മാറ്റുന്നത്. ഭൂമിയെ ജീവിതയോഗ്യമാക്കുന്നതില്‍ മഴക്കുള്ള പങ്ക് അതിപ്രധാനമാണ്. ഭൂമിയിലെ സമുദ്ര ങ്ങളില്‍നിന്നും ജലാശയങ്ങളില്‍നിന്നും സൂര്യതാപമേറ്റ് വെള്ളം നീരാവിയായി മാറി അന്തരീക്ഷത്തിലേക്ക് ഉയരുന്നു. അന്തരീക്ഷത്തിലെ പൊടിപടലങ്ങള്‍ക്ക് ചുറ്റും പറ്റിക്കൂടി നീരാവി ഘനീഭവിക്കുന്നു. കുറെ യേറെ നീണ്ടതും സങ്കീര്‍ണവുമായ പ്രക്രിയയിലൂടെയാണ് ഇങ്ങനെ ഘനീഭവിച്ച മേഘങ്ങളില്‍നിന്ന് മഴയുണ്ടാകുന്നത്. കണ്ടുപിടിക്കപ്പെട്ട ആകാശഗോളങ്ങളില്‍ ഭൂമിയൊഴിച്ച് മറ്റൊന്നിലും ഈ രൂപത്തില്‍ മഴയുണ്ടാകുന്നില്ല. ഭൂമിയില്‍ ജീവന്‍ നിലനിര്‍ത്താന്‍വേണ്ടി സ്രഷ്ടാവുണ്ടാക്കിയ ഒരു പ്രത്യേക സംവിധാനമാണ് മഴയെന്ന വസ്തുത കാലാവസ്ഥാനിരീക്ഷകനും കര്‍ഷകനും ഒരുപോലെ മനസ്സിലാക്കാന്‍ കഴിയും. മഴയോടനുബന്ധിച്ചുള്ള ഇടിമിന്നല്‍പോലും ഭൂമിയിലെ കാര്‍ഷികവിളകള്‍ക്ക് ഗുണദായകമാണ്. അന്തരീക്ഷത്തിലെ നൈ ട്രജനെ സസ്യങ്ങള്‍ക്ക് ഉപയോഗിക്കാന്‍ പറ്റുന്ന നൈട്രജന്‍ സംയുക്തങ്ങളാക്കി മാറ്റുന്നതില്‍ പ്രധാനപ്പെട്ട പങ്കുവഹിക്കുന്നത് ഇടിമിന്നലാണ്. ഇവ ഒരു വര്‍ഷത്തില്‍ പത്തുകോടി ടണ്‍ നൈട്രജന്‍ വള ങ്ങള്‍ (സസ്യങ്ങള്‍ക്ക് ഉപയോഗിക്കാന്‍ പറ്റുന്ന രീതിയിലുള്ള നൈട്ര ജന്‍ മിശ്രിതങ്ങള്‍) ഉല്‍പാദിപ്പിക്കുന്നുണ്ടത്രെ.മഴയും ഇടിമിന്നലുമെല്ലാം പടച്ചതമ്പുരാന്റെ അസ്തിത്വത്തിനുള്ള തെളിവാണെന്നാണ് വിശുദ്ധക്വുര്‍ആന്‍ പറയുന്നത്. 'ഭയവും ആശയും ഉളവാക്കികൊണ്ട് നിങ്ങള്‍ക്ക് മിന്നല്‍ കാണിച്ചുതരുന്നതും ആകാശത്തുനിന്ന് വെള്ളം ചൊരിയുകയും അതുമൂലം നിങ്ങള്‍ക്ക തിന്റെ നിര്‍ജീവാവസ്ഥയ്ക്ക് ശേഷം ജീവന്‍ നല്‍കുകയുംചെയ്യുന്നത് അവന്റെ ദൃഷ്ടാന്തങ്ങളില്‍പ്പെട്ടതത്രെ. തീര്‍ച്ചയായും അതി ല്‍ ചിന്തിച്ചുമനസ്സിലാക്കുന്ന ജനങ്ങള്‍ക്ക് ദൃഷ്ടാന്തമുണ്ട്'. (30:24)

ഭൂമിയുടെ അന്തരീക്ഷമാണ് നമുക്കെല്ലാം ഇവിടെ ജീവിക്കാന്‍വേണ്ട സുരക്ഷിതത്വം പ്രദാനംചെയ്യുന്ന മറ്റൊരു ഘടകം. സൂര്യനില്‍നിന്നു വരുന്ന മാരകമായ അള്‍ട്രാവയലറ്റ് രശ്മികളെയും മറ്റും തട ഞ്ഞുനിര്‍ത്തുകയും നിര്‍ദോഷികളും ഭൂമിയില്‍ ജീവന്‍ നിലനില്‍ക്കാന്‍ ആവശ്യമുള്ളതുമായ രശ്മികളെ കടത്തിവിടുകയും ചെയ്തുകൊണ്ട് ഭൂമിക്കു മുകളില്‍ അരിപ്പപോലെ പ്രവര്‍ത്തിക്കുന്ന ഓ സോണ്‍ പാളിയാണ് ഭൂമിയില്‍ ജീവന്‍ നിലനില്‍ക്കുവാനാവശ്യമായ സാഹചര്യം സൃഷ്ടിക്കുന്നത്. ഭൂമിയിലേക്ക് പതിച്ചുകൊണ്ടിരിക്കുന്ന ഉല്‍ക്കകളെ ഘര്‍ഷണം മൂലം കരിച്ചുകളഞ്ഞ് നശിപ്പിക്കുന്നതും അന്തരീക്ഷംതന്നെയാണ്. അന്തരീക്ഷം എല്ലാ അര്‍ഥത്തിലും നമ്മുടെയൊരു മേല്‍ക്കൂരതന്നെ. ഈ മേല്‍ക്കൂരക്ക് താഴെ ജീവിക്കുന്ന നമുക്ക് നമ്മുടെ സ്രഷ്ടാവിനെ അറിയാന്‍ അന്തരീക്ഷത്തില്‍ നമ്മു ടെ നിലനില്‍പിന് വേണ്ടി സംവിധാനക്കപ്പെട്ടിട്ടുള്ള കാര്യങ്ങളെക്കു റിച്ച് അല്‍പം ചിന്തിച്ചാല്‍മതി. ക്വുര്‍ആന്‍ പഠിപ്പിക്കുന്നു: 'നിങ്ങള്‍ക്കുവേണ്ടി ഭൂമിയെ മെത്തയും ഉപരിലോകത്തെ മേല്‍ക്കൂരയുമാക്കുകയും ഉപരിഭാഗത്തുനിന്നുള്ള വെള്ളം ചൊരിഞ്ഞുതന്നിട്ട് അതുമുഖേന നിങ്ങള്‍ക്ക് ഭക്ഷിക്കുവാനുള്ള കായ്കനികള്‍ ഉല്‍പാദിപ്പിച്ച് തരികയും ചെയ്ത (നാഥന്‍). അതിനാല്‍ അറിഞ്ഞുകൊണ്ട് നിങ്ങ ള്‍ അല്ലാഹുവിന് സമന്മാരെ ഉണ്ടാക്കരുത്'. (2:22)

നമുക്കുചുറ്റും ജീവിക്കുന്ന നമ്മുടെ സഹജീവികളിലോരോന്നും സ്രഷ്ടാവിന്റെ വൈഭവം വിളിച്ചോതുന്നവയാണ്. പ്രാണികളിലെ എഞ്ചിനീയര്‍ എന്നു വിശേഷിപ്പിക്കാവുന്ന തേനീച്ചയെക്കുറിച്ച് പഠിച്ചാല്‍ മാത്രംമതി, ചിന്തിക്കുന്നവര്‍ക്ക് സര്‍വശക്തന്റെ ആസ്തിക്യം ബോധ്യമാകും. കുറഞ്ഞ മെഴുകും കുറഞ്ഞ അധ്വാനവും ഉപയോഗിച്ചുകൊണ്ട് ഉറപ്പിന് അല്‍പംപോലും കോട്ടംതട്ടാതെ ഏറ്റവും കൂടുതല്‍ അറകളുണ്ടാക്കാന്‍ തേനീച്ചകള്‍ക്ക് സാധിക്കും. തേനീച്ചക്കൂടുകളുടെ ജ്യാമിതീയമായ കൃത്യത ശാസ്ത്രജ്ഞന്മാരെ അല്‍ഭുതപ്പെടുത്തുന്നു. തേനീച്ചസമൂഹത്തില്‍ കാണുന്ന തൊഴില്‍ വിഭ ജനവും അല്‍ഭുതാവഹമാണ്. തേന്‍ ശേഖരിക്കാന്‍ ഒരുകൂട്ടര്‍; പ്രത്യുല്‍പാദനം നടത്താന്‍ മറ്റൊരുകൂട്ടര്‍; കൂടുണ്ടാക്കാന്‍ ഒരുവിഭാഗം; കൂടുകാക്കാന്‍ മറ്റൊരു വിഭാഗം; ഇങ്ങനെയാണ് തേനീച്ചസമൂഹത്തിന്റെ തൊഴില്‍ വിഭജനം. തങ്ങളുടെ ബാധ്യതകള്‍ ജീവന്‍ ബലികഴിപ്പിച്ചും നിര്‍വഹിക്കുന്നവയാണ് തേനീച്ചകള്‍. അവ തമ്മിലുള്ള ആശയവിനിമയരീതി വിസ്മയാവഹമാണ്. തേന്‍ തേടി പുറ ത്തുപോകുന്ന തേനീച്ച തിരിച്ചെത്തിയശേഷം തേനിരിക്കുന്ന സ്ഥ ലത്തെക്കുറിച്ച് മറ്റ് തേനീച്ചകള്‍ക്ക് വിവരം നല്‍കാന്‍വേണ്ടി ഒരു പ്രത്യേകരീതിയിലുള്ള നൃത്തമാണ് ഉപയോഗിക്കുന്നത്. 'വിംഗ്ള്‍ ഡാന്‍സ്' എന്നറിയപ്പെടുന്ന നൃത്തത്തില്‍ നിര്‍മിക്കുന്ന വൃത്തങ്ങ ളുടെ ആകൃതിയും ചരിവുംകണ്ട് മറ്റ് തേനീച്ചകള്‍ തേനിരിക്കുന്ന സ്ഥലത്തേക്ക് എത്രദൂരമുണ്ടെന്നും ഏത് ദിശയിലാണതെന്നും മന സ്സിലാക്കുന്നു. അവ വിസര്‍ജിക്കുന്ന തേന്‍ സിദ്ധൗഷധമാണെന്ന കാര്യം സുവിദിതമാണല്ലോ. തേനീച്ച ഈ കഴിവുകളൊന്നും തന്നെ സ്വന്തമായി വളര്‍ത്തിയെടുത്തതല്ല. വളരെ ചെറിയ മസ്തിഷ്‌കമുപയോഗിച്ച് ചിന്തിച്ച് കാര്യങ്ങള്‍ കണ്ടുപിടിക്കാന്‍ തേനീച്ചകള്‍ക്കാവുകയുമില്ല. പിന്നെ ആരാണവയ്ക്ക് ഈ കഴിവുകള്‍ നല്‍കുന്നത്? ഉത്തരം 'തേനീച്ചയെ സൃഷ്ടിച്ചവന്‍' എന്നുമാത്രമാണ്. വിശുദ്ധക്വുര്‍ആന്‍ പറഞ്ഞതെത്ര ശരി! 'നിന്റെ നാഥന്‍ തേനീച്ചകള്‍ക്ക് ഇപ്രകാരം ബോധനം നല്‍കുകയും ചെയ്തിരിക്കുന്നു: മലകളിലും മരങ്ങളിലും മനുഷ്യന്‍ കെട്ടിയുയര്‍ത്തുന്നതിലും നീ പാര്‍പ്പിടങ്ങളുണ്ടാക്കിക്കൊള്ളുക. എല്ലാത്തരം ഫലങ്ങളില്‍നിന്നും ഭക്ഷിച്ചുകൊള്ളുക. എന്നിട്ട് നിന്റെ രക്ഷിതാവ് സൗകര്യപ്രദമായി ഒരുക്കിത്തന്നിട്ടുള്ള മാര്‍ഗങ്ങളില്‍ നീ പ്രവേശിച്ചുകൊള്ളുക. അവയുടെ ഉദരങ്ങളില്‍നിന്ന് വ്യത്യസ്ത വര്‍ണങ്ങളിലുള്ള പാനീയം പുറത്തുവരുന്നു. അതില്‍ മനുഷ്യര്‍ക്ക് രോഗശമനമുണ്ട്. ചിന്തിക്കുന്ന ആളുകള്‍ക്ക് അതില്‍ തീര്‍ച്ചയായും ദൃഷ്ടാന്തമുണ്ട്'. (16:68,69)

ഏതു ജീവികളെ എടുത്താലും സ്ഥിതി ഇതുതന്നെയാണ്. കര യില്‍ ജീവിക്കുന്ന ചെറുതും വലുതുമായ ജീവികള്‍; വെള്ളത്തില്‍ ജീവിക്കുന്നവ; പറക്കും പറവകള്‍. ഇവയെല്ലാം സ്രഷ്ടാവിന്റെ ആസ്തിക്യം നമുക്ക് വ്യക്തമാക്കിത്തരുന്നു.'എല്ലാ ജന്തുക്കളെയും അല്ലാഹു വെള്ളത്തില്‍നിന്ന് സൃഷ്ടിച്ചിരിക്കുന്നു. അവയുടെ കൂട്ടത്തില്‍ ഉദരത്തിന്മേല്‍ ഇഴഞ്ഞുനടക്കുന്നവയുണ്ട്. രണ്ടുകാലില്‍ നടക്കുന്നവയും അവയിലുണ്ട്. നാലുകാലില്‍ നടക്കുന്നവയും അവയിലുണ്ട്. അല്ലാഹു താന്‍ ഉദ്ദേശിക്കുന്നത് സൃഷ്ടിക്കുന്നു. തീര്‍ച്ചയായും അല്ലാഹു എല്ലാകാര്യത്തിനും കഴിവുള്ളവനാകുന്നു.' (24:45)

മനുഷ്യരുടെ ദൈനംദിനജീവിതവുമായി ബന്ധപ്പെടുന്ന ആകാശഗോളങ്ങളായ സൂര്യനും ചന്ദ്രനും എന്നെന്നും മനുഷ്യരെ അല്‍ഭുത പരതന്ത്രരാക്കിയിട്ടുണ്ട്. സൂര്യോദയവും അസ്തമയവുമായിരുന്നു പുരാതന മനുഷ്യരെ അത്ഭുതപ്പെടുത്തിയതെങ്കില്‍ സൂര്യനില്‍ നട ക്കുന്ന പ്രവര്‍ത്തനങ്ങളും അതിന്റെ സ്ഥാനവുമെല്ലാമാണ് ആധുനിക മനുഷ്യരെ അത്ഭുതപ്പെടുത്തുന്നത്. സൗരാന്തര്‍ഭാഗത്തെ ഊഷ്മാവ് ഏകദേശം 65 ലക്ഷം ഡിഗ്രി സെല്‍ഷ്യസാണ്. ഒരു ചതുര ശ്ര ഇഞ്ചില്‍ ഇവിടെ അനുഭവപ്പെടുന്ന മര്‍ദം ഒരു ലക്ഷം പൗണ്ട്! സൗരവസ്തുവിന്റെ ഭാരം, ഒരു ചതുരശ്രകിലോമീറ്ററിന് 703.1 ലക്ഷം കോടി കിലോഗ്രാമാണ്. സൗരയൂഥത്തിലെ ഊര്‍ജത്തിന്റെ സ്രോതസ്സായ സൂര്യനില്‍ ഊര്‍ജോല്‍പാദനം നടക്കുന്നത് ന്യൂക്ലിയര്‍ ഫ്യൂഷന്‍ (ഹൈഡ്രജന്‍ ബോംബില്‍ നടക്കുന്ന പ്രവര്‍ത്തനം) വഴിയാണ്. അഥവാ ഓരോ സെക്കന്റിലും ആയിരക്കണക്കിന് ഹൈ ഡ്രജന്‍ ബോംബ് സ്‌ഫോടനങ്ങള്‍ നടന്നുകൊണ്ടിരിക്കുകയാണ്. അവിടെ ഒരു സെക്കന്‍ഡില്‍ ഉല്‍പാദിപ്പിക്കപ്പെടുന്ന ഊര്‍ജം 1023 ജൂള്‍ ആണ്. ക്ഷീരപഥമെന്ന (milky way) താരാസമൂഹ (galaxy) ത്തിലെ അംഗമാണ് സൂര്യന്‍. താരാസമൂഹത്തിന്റെ കേന്ദ്രത്തില്‍നിന്ന് ഏകദേശം 32000 പ്രകാശവര്‍ഷം അകലെയാണ് സൂര്യന്‍ സ്ഥിതിചെയ്യുന്നത്. സൂര്യന്‍ അതിന്റെ താരാസമൂഹകേന്ദ്രത്തെ വൃത്താകാരാമായ പഥത്തിലൂടെ ഭ്രമണം ചെയ്തുകൊണ്ടിരിക്കുന്നു. ഈ ഭ്രമണത്തിന്റെ വേഗത സെക്കന്റില്‍ 250 കിലോമീറ്ററാണ്. താരാസമൂഹകേന്ദ്രത്തെ ഒരു പ്രാവശ്യം ചുറ്റാന്‍ സൂര്യനെടുക്കുന്ന സമയത്തിന്നാണ്-25 കോടിവര്‍ഷം-ഒരു കോസ്മിക് വര്‍ഷ(രീാെശര ്യലമൃ)മെന്ന് പറയുന്നത്.ഇങ്ങനെ സൂര്യനെക്കുറിച്ച് പഠിക്കുംതോറും സര്‍വശക്തന്റെ അജയ്യത നമുക്ക് കൂടുതല്‍ കൂടുതല്‍ മനസ്സിലാകുന്നു. അവന്‍ കല്‍പിച്ചുവെച്ച കണക്കുകള്‍ അനുസരിച്ചാണ് സൂര്യചന്ദ്രന്മാര്‍ ചലിച്ചുകൊണ്ടിരിക്കുന്നത്. പ്രസ്തുത കണക്കുകള്‍ ഒരിക്കലും തെറ്റുകയില്ല. ക്വുര്‍ആന്‍ പറയുന്നത് നോക്കുക: 'സൂര്യ ന്‍ അതിന്ന് സ്ഥിരമായുള്ള ഒരു സ്ഥാനത്തേക്ക് സഞ്ചരിക്കുന്നു. പ്രതാപിയും സര്‍വജ്ഞനുമായ അല്ലാഹു കണക്കാക്കിയതാണിത്'. (36:38) 'അവനത്രെ രാത്രി, പകല്‍, സൂര്യന്‍, ചന്ദ്രന്‍ എന്നിവയെ സൃഷ്ടിച്ചത്. ഓരോന്നും ഓരോ ഭ്രമണ പഥത്തിലൂടെ നീന്തി(സഞ്ചരിച്ചു) ക്കൊണ്ടിരിക്കുന്നു'. (21:33)

പ്രപഞ്ചത്തിന്റെ വിസ്തൃതിയും അതില്‍ നമ്മുടെ സ്ഥാനവുമറി യുന്ന മനുഷ്യന്‍ തീര്‍ച്ചയായും സ്രഷ്ടാവിനു മുമ്പില്‍ നമ്രശിരസ്‌കനായിപ്പോകും. പ്രപഞ്ചത്തില്‍ ആയിരക്കണക്കിന് ക്ലസ്റ്ററുകള്‍; ഓരോ ക്ലസ്റ്ററിലും ആയിരക്കണക്കിന് ഗാലക്‌സികള്‍; ഓരോ ഗാല ക്‌സിയിലും കോടിക്കണക്കിന് നക്ഷത്രങ്ങള്‍, പള്‍സാറുകള്‍, ക്വാസാറുകള്‍, നെബുലകള്‍, തമോഗര്‍ത്തങ്ങള്‍. ഗാലക്‌സികളിലൊന്നായ ക്ഷീരപഥത്തിലെ ഒരംഗമാണ് സൂര്യന്‍. വ്യാപ്തിയില്‍ ഭൂമിയുടെ പതിമൂന്നര ലക്ഷം ഇരട്ടിയാണ് സൂര്യന്‍. ഏകദേശം പതിമൂന്നര ലക്ഷം ഭൂമികളെ സൂര്യനുള്ളില്‍ അടുക്കിവെക്കാമെന്നര്‍ഥം. സൂര്യന്റെ ലക്ഷക്കണക്കിന് ഇരട്ടി വലുപ്പമുള്ള നക്ഷത്രങ്ങളും ക്ഷീരപ ഥത്തിലുണ്ട്. ഒരു ഇടത്തരം നക്ഷത്രമായ സൂര്യനുചുറ്റും ചുറ്റിത്തിരിയുന്ന ഒമ്പത് ഗ്രഹങ്ങള്‍. സൂര്യനെയും ഗ്രഹങ്ങളെയും കൂടാതെ ഉപഗ്രഹങ്ങള്‍, ധൂമകേതുക്കള്‍, ക്ഷുദ്രഗ്രഹങ്ങള്‍, ഉല്‍ക്കകള്‍ തുട ങ്ങിയവയും സൗരയൂഥത്തിലെ അംഗങ്ങളാണ്. ഒമ്പത് ഗ്രഹങ്ങളി ലൊന്നാണ് ഭൂമി. ഭൂമിയിലെ ആയിരക്കണക്കിന് ജീവജാലങ്ങളിലൊരു വര്‍ഗമാണ് നരവര്‍ഗം. മനുഷ്യവര്‍ഗത്തിലെ കോടിക്കണക്കിന്അംഗങ്ങളില്‍ ഓരോരുത്തരാണ് നമ്മളെല്ലാം. ക്ലസ്റ്ററുകളെയും ഗാലക്‌സികളെയും നക്ഷത്രങ്ങളെയും സൂര്യനെയും ഗ്രഹങ്ങളെ യും ഭൂമിയെയും ജീവജാലങ്ങളെയും മനുഷ്യനെയും പരിപാലിക്കുന്ന മഹാചൈതന്യത്തിനു മുന്നില്‍ നമ്മളോരോരുത്തരുടെയും കഴിവുകളും ശക്തിയും എത്രമാത്രം നിസ്സാരമാണെന്ന ബോധം അവയെക്കുറിച്ച് പഠിക്കുമ്പോള്‍ നമ്മളിലുണ്ടാകുന്നു. ഈ ബോധം മനുഷ്യരെ സ്രഷ്ടാവിനു മുന്നില്‍ നമ്രശിരസ്‌കരാക്കുന്നു. വിശുദ്ധക്വുര്‍ആന്‍ പറഞ്ഞതത്രെ ശരി: 'ആകാശ ഭൂമികളുടെ സൃഷ്ടിയിലും രാപ്പകലുകളുടെ മാറ്റത്തിലും മനുഷ്യര്‍ക്കുപകാരപ്രദമായ വസ്തുക്കളുമായി കടലിലൂടെ സഞ്ചരിക്കുന്ന കപ്പലിലും ആകാശത്തുനിന്ന് അല്ലാഹു മഴ ചൊരിഞ്ഞുതന്നിട്ട് നിര്‍ജീവാവസ്ഥയ്ക്ക് ശേഷം ഭൂമിക്ക് അതുമുഖേന ജീവന്‍ നല്‍കിയതിലും കാറ്റുകളുടെ ഗതിക്രമത്തിലും ആകാശഭൂമികള്‍ക്കിടയിലൂടെ നിയന്ത്രിച്ച് നയിക്കപ്പെടുന്ന മേഘത്തിലും ചിന്തിക്കുന്ന ജനങ്ങള്‍ക്ക് പല ദൃഷ്ടാന്തങ്ങളുമുണ്ട്; തീര്‍ച്ച'. (2:164)

'ആകാശഭൂമികളുടെ ആധിപത്യരഹസ്യത്തെപ്പറ്റിയും അല്ലാഹു സൃഷ്ടിച്ച ഏതൊരു വസ്തുവെപ്പറ്റിയും അവരുടെ അവധി അടു ത്തിട്ടുണ്ടായിരിക്കാം എന്നതിനെപ്പറ്റിയും അവര്‍ ചിന്തിച്ചു നോക്കിയില്ലേ?' 'ഉപരിലോകങ്ങളും ഭൂമിയും മുറപ്രകാരം അല്ലാഹു സൃഷ്ടിച്ചിരി ക്കുന്നു. തീര്‍ച്ചയായും സത്യവിശ്വാസികള്‍ക്ക് അതില്‍ ദൃഷ്ടാന്ത ങ്ങളുണ്ട്.' (7:185)

നമുക്കു ചുറ്റുമുള്ള വസ്തുക്കളിലേക്കും നമ്മിലേക്കുതന്നെയും നോക്കിക്കൊണ്ട് സ്രഷ്ടാവിനെക്കുറിച്ചു മനസ്സിലാക്കാനാണ് ക്വുര്‍ആന്‍ പഠിപ്പിക്കുന്നത്. അന്ധമായ വിശ്വാസത്തിന് അത് പ്രേരിപ്പിക്കു ന്നില്ല. ഇസ്‌ലാം മനുഷ്യബുദ്ധിയോടാണ് സംസാരിക്കുന്നത്. പ്രപ ഞ്ചത്തിലെ ചെറുതും വലുതുമായ ഏതു വസ്തുവിനെയും ചൂഴ്ന്നു നില്‍ക്കുന്ന വിസ്മയകരമായ വ്യവസ്ഥ ചൂണ്ടിക്കാണിച്ചു കൊണ്ടാ ണ് പ്രപഞ്ചനാഥന്റെ അസ്തിത്വക്കുറിച്ച അവബോധത്തിലേക്ക് വിശുദ്ധ ക്വുര്‍ആന്‍ മനുഷ്യരെ നയിക്കുന്നത്. ഈ അവബോധം അന്ധമായ കേവല വിശ്വാസമല്ല; തെളിവുകൾ നമ്മെ ബോധ്യപ്പെടുത്തുന്ന അനുഭവമാണ്.

 

ടിമത്തം ഇല്ലാതാക്കുവാനാവശ്യമായ പ്രായോഗികമായ നടപടിക്രമങ്ങള്‍ സ്വീകരിച്ച ഇസ്‌ലാം പക്ഷേ, മദ്യമോ വ്യഭിചാരമോ നിരോധിച്ചതുപോലെ അടിമത്തത്തെ പാടെ നിരോധിച്ചുകൊണ്ടുള്ള ഉത്തരവുകളൊന്നും പുറപ്പെടുവിച്ചിട്ടില്ല. എന്തുകൊണ്ടാണിത്?

ഒന്നിലധികം കാരണങ്ങളുണ്ട്. അടിമത്തത്തെ പാടെ നിരോധിക്കാത്ത ഖുര്‍ആനിന്റെ നടപടി അതിന്റെ സര്‍വകാലികതയാണ് വ്യക്തമാക്കുന്നത്. മനുഷ്യസമൂഹത്തിന്റെ ഗതിവിഗതികളെയും പരിണാമപ്രക്രിയയെയും കുറിച്ച് ശരിക്കറിയാവുന്ന ദൈവം തമ്പുരാനില്‍നിന്നുള്ളതാണ് ഖുര്‍ആന്‍ എന്ന വസ്തുതയാണ് ഈ വിഷയത്തിലെ അതിന്റെ നിലപാടില്‍നിന്നും നമുക്ക് മനസ്സിലാക്കാന്‍ കഴിയുന്നത്. ഇസ്‌ലാം കാലാതിവര്‍ത്തിയാണെന്നും അതിന്റെ നിര്‍ദേശങ്ങള്‍ എക്കാലത്തും പ്രായോഗികമാണെന്നുമുള്ള വസ്തുതയാണ് അടിമത്തം പാടെ നിരോധിക്കാത്ത അതിന്റെ നടപടിയെക്കുറിച്ച് അവഗാഹമായി പഠിച്ചാല്‍ ബോധ്യപ്പെടുക.

അടിമത്ത വ്യവസ്ഥിതിയുടെ ആരംഭം തന്നെ യുദ്ധത്തടവുകാരില്‍നിന്നായിരുന്നുവല്ലോ. അടിമത്തത്തെ പാടെ നിരോധിച്ചുകൊണ്ട് ആധുനിക രാഷ്ട്രങ്ങള്‍ നടത്തിയ പ്രഖ്യാപനങ്ങള്‍ക്കു മുമ്പ് യുദ്ധത്തടവുകാരെ അടിമകളാക്കി മാറ്റുന്ന സമ്പ്രദായമായിരുന്നു വ്യാപകമായി നിലനിന്നിരുന്നത്. യുദ്ധത്തില്‍ ബന്ദികളായി പിടിക്കപ്പെടുന്നവരെ ഒന്നുകില്‍ കൊന്നുകളയുക, അല്ലെങ്കില്‍ അടിമകളാക്കുക. ഇതാണ് നടന്നിരുന്നത്. ഇവ മാത്രമായിരുന്നു പ്രായോഗികമായ മാര്‍ഗങ്ങള്‍. അതല്ലാതെ അവരെ തടവുകാരായി പാര്‍പ്പിക്കുവാനാവശ്യമായ സംവിധാനങ്ങളൊന്നും അന്നുണ്ടായിരുന്നില്ലല്ലോ.

യുദ്ധത്തില്‍ തടവുകാരായി പിടിക്കപ്പെടുന്നവരെ എന്തു ചെയ്യണം?ഇക്കാര്യത്തില്‍ ഖുര്‍ആന്‍ നല്‍കുന്ന നിര്‍ദേശമിങ്ങനെയാണ്: ”ആകയാല്‍ സത്യനിഷേധികളുമായി നിങ്ങള്‍ ഏറ്റുമുട്ടിയാല്‍ (നിങ്ങള്‍) പിരടികളില്‍ വെട്ടുക. അങ്ങനെ അവരെ നിങ്ങള്‍ അമര്‍ച്ച ചെയ്തുകഴിഞ്ഞാല്‍ നിങ്ങള്‍ അവരെ ശക്തിയായി ബന്ധിക്കുക. എന്നിട്ട് അതിനുശേഷം (അവരോട്) ദാക്ഷിണ്യം കാണിക്കുകയോ, അല്ലെങ്കില്‍ മോചനമൂല്യം വാങ്ങി വിട്ടയക്കുകയോ ചെയ്യുക. യുദ്ധം അതിന്റെ ഭാരങ്ങള്‍ ഇറക്കിവെക്കുന്നതുവരെയാണിത്. അതാണ് (യുദ്ധത്തിന്റെ) മുറ” (47:4)ശത്രുക്കളെ യുദ്ധഭൂമിയില്‍ വെച്ച് വധിക്കുവാന്‍ അനുശാസിക്കുന്ന ഈ സൂക്തത്തില്‍ ബന്ധനസ്ഥരായവരെ പ്രതിഫലം വാങ്ങിയോ അല്ലാതെയോ വിട്ടയക്കുവാനാണ് കല്‍പിച്ചിരിക്കുന്നത്. ഈ സൂക്തത്തിന്റെ വെളിച്ചത്തില്‍ പ്രവാചകാനുചരന്മാരില്‍ പ്രമുഖരെല്ലാം യുദ്ധത്തടവുകാരെ വധിക്കാന്‍ പാടില്ലെന്ന് അഭിപ്രായപ്പെട്ടിട്ടുണ്ട്.

യുദ്ധത്തടവുകാരെ നാലു വിധത്തില്‍ കൈകാര്യം ചെയ്യുവാന്‍ പ്രവാചകന്‍ (ﷺ) മാതൃക കാണിച്ചിട്ടുണ്ട്.

  1. വെറുതെ വിട്ടയക്കുക. അവരെ വിട്ടയക്കുന്നത് മുസ്‌ലിം സമൂഹത്തിന് ഹാനികരമല്ലെന്ന് ബോധ്യപ്പെടുന്ന അവസ്ഥയില്‍ യുദ്ധത്തടവുകാരെ വെറുതെ വിട്ടയക്കാവുന്നതാണ്.
  1. ശത്രുക്കള്‍ പിടിച്ചുവെച്ച മുസ്‌ലിം തടവുകാര്‍ക്കു പകരമായി അവരെ കൈമാറുക.
  1. പ്രതിഫലം വാങ്ങി തടവുകാരെ വിട്ടയക്കുക.
  1. മുസ്‌ലിം യോദ്ധാക്കള്‍ക്ക് അടിമകളെ ഭാഗിച്ച് നല്‍കുക.

പ്രവാചകന്‍ (ﷺ) വിവിധ യുദ്ധങ്ങളില്‍ മുകളില്‍ പറഞ്ഞ വ്യത്യസ്ത മാര്‍ഗങ്ങള്‍ സ്വീകരിച്ചിരുന്നതായി കാണാം. ഇതില്‍ നാലാമത്തെ മാര്‍ഗമായ യുദ്ധത്തടവുകാരെ അടിമകളാക്കി മാറ്റുന്ന രീതി,മറ്റു മൂന്നു മാര്‍ഗങ്ങളും അപ്രായോഗികമായിത്തീരുന്ന അവസ്ഥകളിലാണ് സ്വീകരിച്ചിരുന്നത്. അടിമത്തം പൂര്‍ണമായി നിരോധിക്കപ്പെട്ടിരുന്നുവെങ്കില്‍ ഈ മാര്‍ഗം സ്വീകരിക്കുവാന്‍ മുസ്‌ലിം സമൂഹത്തിന് ഒരിക്കലും സാധ്യമാകാത്ത അവസ്ഥ സംജാതമാകുമായിരുന്നു. അത്തരമൊരു അവസ്ഥ അടിമത്തം ഒരു സ്ഥാപനമായി നിലനിന്നിരുന്ന സാമൂഹിക സംവിധാനത്തില്‍ മുസ്‌ലിംകള്‍ക്ക് ഏറെ പ്രയാസങ്ങള്‍ സൃഷ്ടിക്കുമായിരുന്നു എന്നതാണ് വസ്തുത.

മുസ്‌ലിം സമൂഹവുമായി യുദ്ധം ചെയ്യുന്നവര്‍ അടിമത്തത്തെ ഒരു മാര്‍ഗമായി അംഗീകരിക്കുന്നവരും അടിമകളെ ലഭിക്കുക എന്നതുകൂടി ലക്ഷ്യമായിക്കണ്ട് യുദ്ധത്തില്‍ ഏര്‍പ്പെടുന്നവരുമായിരുന്നു. അവരുമായി യുദ്ധം ചെയ്യുമ്പോള്‍ മുസ്‌ലിംകളില്‍നിന്ന് അവര്‍ തടവുകാരായി പിടിക്കുന്നവരെ അവര്‍ അടിമകളാക്കി മാറ്റുകയോ വധിച്ചുകളയുകയോ ചെയ്യുമായിരുന്നു. അടിമത്തം നിരോധിക്കപ്പെട്ടിരുന്നുവെങ്കില്‍ മുസ്‌ലിംകള്‍ക്ക് അവരില്‍നിന്നുള്ള ബന്ദികളെ അടിമകളാക്കുവാന്‍ പറ്റുകയില്ല. ഇത് ശത്രുക്കള്‍ക്ക് മുസ്‌ലിം ബന്ദികളുടെ മേല്‍ കൂടുതല്‍ ക്രൂരത കാണിക്കുവാനുള്ള അവസരമുണ്ടാക്കുകയാണ് ചെയ്യുക. മുസ്‌ലിംകള്‍ക്കാണെങ്കില്‍ അവരില്‍നിന്ന് പിടിക്കപ്പെട്ടവര്‍ക്കു വേണ്ടി വില പേശുവാനായി ശത്രുക്കളില്‍നിന്ന് പിടിക്കപ്പെട്ട ബന്ദികളെ ഉപയോഗിക്കുവാനും കഴിയില്ല.

ഇസ്‌ലാമില്‍ അടിമത്തം നിരോധിക്കപ്പെട്ടാല്‍ അവരില്‍നിന്നുള്ളവരെ അടിമകളാക്കുവാനോ വധിക്കുവാനോ മുസ്‌ലിംകള്‍ക്ക് കഴിയുകയില്ലെന്ന് ശത്രുക്കള്‍ക്കറിയാം. അതുകൊണ്ടുതന്നെ അവരില്‍നിന്ന് പിടിക്കപ്പെട്ട ബന്ദികള്‍ക്ക് പകരമായി മുസ്‌ലിംകളില്‍നിന്ന് പിടിക്കപ്പെട്ട ബന്ദികളെ സ്വതന്ത്രരാക്കുകയെന്ന പരസ്പര ധാരണക്ക് ശത്രുക്കള്‍ സന്നദ്ധരാവുകയില്ല.

മുസ്‌ലിംകള്‍ക്കാണെങ്കില്‍ ശത്രുക്കളില്‍നിന്നുള്ള ബന്ദികള്‍ ഒരു തലവേദന മാത്രമായിത്തീരുകയും ചെയ്യും. അവര്‍ക്കുള്ള താമസസ്ഥലം ഉണ്ടാക്കുക മുസ്‌ലിം സമൂഹത്തിന്റെ ബാധ്യതതായിത്തീരും. ആയിരക്കണക്കിനാളുകള്‍ ബന്ദികളായി പിടിക്കപ്പെടുന്ന അവസരങ്ങളില്‍ അവര്‍ക്കെല്ലാം താമസിക്കാനാവശ്യമായ സൗകര്യങ്ങളുണ്ടാക്കുക ഏറെ ദുഷ്‌കരമായിത്തീരുമെന്ന് പറയേണ്ടതില്ലല്ലോ. അവര്‍ക്കുള്ള ഭക്ഷണവും വസ്ത്രവുമെല്ലാം നല്‍കാന്‍ മുസ്‌ലിം സമൂഹം ബാധ്യസ്ഥമായിത്തീരും. അവര്‍ ഇവിടെ ഇസ്‌ലാമിക സമൂഹത്തിന്റെ സംരക്ഷണത്തില്‍ സുഖകരമായി ജീവിക്കുമ്പോള്‍ മുസ്‌ലിംകളില്‍നിന്ന് പിടിക്കപ്പെട്ട ബന്ദികള്‍ ഇസ്‌ലാമിന്റെ ശത്രുക്കളുടെ ക്രൂരതകള്‍ സഹിച്ച് അവര്‍ ഏല്‍പിക്കുന്ന കഠിനമായ ജോലികള്‍ ചെയ്തുകൊണ്ടിരിക്കുകയാവും. ഇത് ഒരിക്കലും നീതിയാവുകയില്ലല്ലോ. മുസ്‌ലിം സമൂഹത്തിന്റെ നാശത്തിനാണ് അതു നിമിത്തമാവുക. യുദ്ധം ഇസ്‌ലാമിക സമൂഹത്തെ നശിപ്പിക്കുകയല്ലാതെ മറ്റൊന്നും ചെയ്യാത്ത ഒരു അവസ്ഥയാണ് ഇതുവഴി സംജാതമാവുക. അതുകൊണ്ടുതന്നെ ലോകം മുഴുവനായി അടിമത്തം നിരോധിക്കാത്ത അവസ്ഥയില്‍ ഇസ്‌ലാം അടിമത്തം നിരോധിച്ചിരുന്നുവെങ്കില്‍ അത് ആത്മഹത്യാപരമാകുമായിരുന്നു. അതുകൊണ്ടുതന്നെയാണ് സര്‍വകാലജ്ഞാനിയായ അല്ലാഹു അടിമത്തം നിരോധിക്കാതിരുന്നത്.

ലോകത്ത് അടിമത്തം ഒരു സ്ഥാപനമായി നിലനില്‍ക്കെ ഇസ്‌ലാം അതു നിരോധിക്കുന്നതുകൊണ്ട് പ്രായോഗിക തലത്തില്‍ ഗുണത്തേക്കാളേറെ ദോഷമാണുണ്ടാവുകയെന്നുള്ളതാണ് വാസ്തവം. അടിമത്തം അനുവദിച്ചിരിക്കുന്ന ഇസ്‌ലാം അടിമയും ഉടമയുമെല്ലാം സഹോദരന്മാരാണെന്നും അടിമക്ക് അവകാശങ്ങളുണ്ടെന്നും പഠിപ്പിക്കുകയും അവനുമായി നല്ലനിലയില്‍ വര്‍ത്തിക്കണമെന്നും ക്രൂരമായി പെരുമാറരുതെന്നും അപമാനിക്കരുതെന്നുമെല്ലാം നിഷ്‌കര്‍ഷിക്കുകയും ചെയ്യുന്നു. അതുകൊണ്ടുതന്നെ ഒരു മുസ്‌ലിമിന്റെ കീഴില്‍ ജീവിക്കുന്ന അടിമയെ സംബന്ധിച്ചിടത്തോളം  അടിമത്തം അവന് ഒരു ഭാരമായിത്തീരുകയില്ല. അതോടൊപ്പംതന്നെ അവന്‍ സ്വതന്ത്രനാകുവാന്‍ ഏതു സമയത്തും സാധ്യതയുണ്ടുതാനും. സ്വാതന്ത്ര്യം വേണമെന്ന് സ്വയം തോന്നുമ്പോള്‍ അവന് സ്വാതന്ത്ര്യം നേടുവാന്‍ സാധിക്കുകയും ചെയ്യും.

എന്നാല്‍ ഇതേ അടിമ ഇത്തരം ധര്‍മങ്ങളിലൊന്നും വിശ്വാസമില്ലാത്ത ഒരു അമുസ്‌ലിമിന്റെ കീഴിലാണുള്ളതെങ്കിലോ?അയാള്‍ക്ക് അതിക്രൂരമായ പെരുമാറ്റവും അതിനീചമായ അപമാനവുമാണ് ലഭിക്കുക. അയാളെ സംബന്ധിച്ചിടത്തോളം അടിമത്തത്തില്‍നിന്നുള്ള മോചനം ഒരിക്കലും നടപ്പിലാകാത്ത സ്വപ്‌നം മാത്രമായിരിക്കും. ഒരു മുസ്‌ലിമിന്റെ കീഴിലുള്ള അടിമയായിരിക്കാനാണ് അതുകൊണ്ടുതന്നെ അടിമകള്‍ ഇഷ്ടപ്പെടുക. അവിടെ മാന്യമായ പെരുമാറ്റവും സഹാനുഭൂതിയോടുകൂടിയുള്ള സഹകരണവും കിട്ടുമല്ലോ. എന്നാല്‍, ഇസ്‌ലാം അടിമത്തം നിരോധിച്ചിരുന്നെങ്കില്‍ ഒരിക്കലും അടിമക്ക് അത്തരമൊരു ജീവിതം നല്‍കുവാന്‍ ആരും സന്നദ്ധരാവുകയില്ല. മുസ്‌ലിമിനാണെങ്കില്‍ അടിമകളെ വെച്ചുകൊണ്ടിരിക്കാന്‍ പറ്റുകയുമില്ലല്ലോ.

അടിമത്തം നിലനില്‍ക്കുന്ന ഒരു സാമൂഹിക സംവിധാനത്തില്‍ ഇസ്‌ലാം മാത്രം അടിമത്തം നിരോധിക്കുന്നതുകൊണ്ട് കാര്യമായ ഗുണങ്ങളൊന്നുമില്ലെന്ന് മാത്രമല്ല അടിമയെ സംബന്ധിച്ചിടത്തോളം അത് കൂടുതല്‍ പ്രയാസങ്ങളുണ്ടാക്കുക മാത്രമേ ചെയ്യൂ. മുസ്‌ലിം സമൂഹത്തിന്റെ നിലനില്‍പിനെത്തന്നെ ആ നിരോധം പ്രതികൂലമായി ബാധിക്കുകയും ചെയ്യും. അതുകൊണ്ടുതന്നെ അടിമകളുടെ മാനസികവും ശാരീരികവുമായ മോചനത്തിനുവേണ്ടി ശ്രമിക്കുകയും അതിനാവശ്യമായ പ്രായോഗിക നിയമങ്ങള്‍ ആവിഷ്‌കരിക്കുകയുമാണ് ഇസ്‌ലാം ചെയ്തത്. അതു മാത്രമാണ് അത്തരമൊരു സമൂഹത്തില്‍ കരണീയമായിട്ടുള്ളത്;പ്രായോഗികവും.

വിവാഹം നാലില്‍ പരിമിതപ്പെടുത്തണമെന്ന് നിഷ്‌കര്‍ഷിച്ച ഇസ്‌ലാം പക്ഷേ, കൈവശം വെക്കാവുന്ന അടിമസ്ത്രീകളുടെ എണ്ണത്തിന് യാതൊരു നിയന്ത്രണവുമേര്‍പ്പെടുത്തിയിടില്ല. ഒരാള്‍ക്ക് എത്ര അടിമസ്ത്രീകളെയും കൈവശം വെച്ചുകൊണ്ടിരിക്കാം എന്നര്‍ഥം. എന്തുകൊണ്ടാണ് ഇസ്‌ലാം ഇത് അനുവദിച്ചത്?

അടിമകള്‍ യജമാനന്റെ കൈവശം എത്തിച്ചേരുന്നത് മൂന്നു മാര്‍ഗങ്ങളിലൂടെയാണ്. അനന്തരാവകാശമായി, സ്വയം വാങ്ങുക, യുദ്ധത്തില്‍ തടവുകാരായി പിടിക്കപ്പെടുക എന്നീ വഴികളിലൂടെ. ഇതില്‍ ഒരാള്‍ക്ക് സ്വന്തം ഇച്ഛ പ്രകാരം അടിമകളുടെ എണ്ണം പരിമിതപ്പെടുത്തുവാന്‍ കഴിയുക സ്വയം വാങ്ങുന്ന കാര്യത്തില്‍ മാത്രമാണ്. അനന്തരാവകാശമായി കിട്ടുകയോ യുദ്ധത്തില്‍ തടവുകാരായി പിടിക്കപ്പെട്ട് അടിമകളായിത്തീര്‍ന്ന് ഒരാളുടെ കൈവശം എത്തിച്ചേരുകയോ ചെയ്യുന്ന അടിമകളുടെ എണ്ണം അയാള്‍ക്ക് നിയന്ത്രിക്കുവാനോ പരിമിതപ്പെടുത്തുവാനോ കഴിയില്ല. യുദ്ധത്തടവുകാരായി പിടിക്കപ്പെടുന്നവരെ അടിമകളാക്കുവാനാണ് ഭരണകൂടം തീരുമാനിക്കുന്നതെങ്കില്‍ യുദ്ധത്തില്‍ പങ്കെടുത്തവര്‍ക്കിടയില്‍ അവരെ വീതിച്ചു നല്‍കുകയാണ് ചെയ്യുക. കുറെയേറെപ്പേരെ തടവുകാരായി പിടിക്കുകയാണെങ്കില്‍ ഓരോരുത്തരുടെയും കൈവശം എത്തിപ്പെടുന്ന അടിമകളുടെ എണ്ണവും കൂടും. ഹുനൈന്‍ യുദ്ധത്തില്‍ ആറായിരത്തോളം പേരെ തടവുകാരായി പിടിച്ചിരുന്നുവെന്ന് ചരിത്രത്തില്‍ കാണാം.

യുദ്ധത്തില്‍ പിടിക്കപ്പെടുന്നവരെ അനിവാര്യമായ സാഹചര്യങ്ങളില്‍ മാത്രമേ അടിമകളാക്കി മാറ്റിയിരുന്നുള്ളൂ. ഖലീഫമാരുടെ കാലത്ത് നടന്ന യുദ്ധങ്ങളില്‍ സിറിയ, ഫലസ്തീന്‍, ഇറാഖ്, ഈജിപ്ത് എന്നീ രാജ്യങ്ങളില്‍ ആരെയും അടിമകളാക്കി മാറ്റിയിരുന്നില്ലെന്ന് കാണാനാവും. യുദ്ധത്തില്‍ പുരുഷന്മാര്‍ വധിക്കപ്പെടുകയോ ബന്ധനസ്ഥരായി പിടിക്കപ്പെടുകയോ ചെയ്താല്‍ സ്ത്രീകളും കുട്ടികളും അനാഥരായിത്തീരുകയായിരിക്കും ഫലം. അവരെ യുദ്ധത്തില്‍ വധിക്കുവാന്‍ പാടില്ലെന്ന് ഇസ്‌ലാം നിഷ്‌കര്‍ഷിച്ചിട്ടുണ്ട്. പുരുഷന്മാരോടൊപ്പം യുദ്ധത്തിന് പുറപ്പെടുന്ന സ്ത്രീകളും കുട്ടികളും തടവുകാരായി പിടിക്കപ്പെട്ടാല്‍തന്നെ മുസ്‌ലിം തടവുകാര്‍ക്ക് പകരമായി കൈമാറുകയായിരുന്നു പലപ്പോഴും ചെയ്തിരുന്നത്. ചില അവസരങ്ങളില്‍ അവരെ അടിമകളാക്കി മാറ്റുവാനും ഇസ്‌ലാം അനുവദിച്ചിരുന്നു. അടിമത്തം നിലനിന്നിരുന്ന ഒരു സമൂഹത്തിലായിരുന്നു ഈ അനുവാദമെന്നോര്‍ക്കണം.

ഏതായിരുന്നാലും ഈ വഴികളിലൂടെയെല്ലാം തങ്ങളുടെ കൈവശമെത്തിച്ചേരുന്ന അടിമകളെ പരിമിതപ്പെടുത്തുക അന്നത്തെ സാഹചര്യത്തില്‍ തികച്ചും പ്രയാസകരമായിരുന്നു. സ്ത്രീകളുടെ സ്ഥിതിയും അതുതന്നെ.  ഇങ്ങനെ കൈവശം എത്തിച്ചേരുന്ന സ്ത്രീകളെ എന്തുചെയ്യണമെന്നുള്ളതാണ് പ്രശ്‌നം. അവരെ മറ്റൊരാള്‍ക്ക് വിവാഹം ചെയ്തുകൊടുക്കാം. ഒരു സ്വതന്ത്രന്‍ അടിമയെ വിവാഹം ചെയ്യുവാനുള്ള സാധ്യത തുലോം വിരളമായിരുന്നുവെന്നോര്‍ക്കുക. അല്ലെങ്കില്‍ മറ്റൊരു അടിമക്കു വിവാഹം ചെയ്തുകൊടുക്കാം. രണ്ടാണെങ്കിലും അവള്‍ അയാളുടെ സ്വത്തായിരിക്കുന്നിടത്തോളം കാലം അവളിലുണ്ടാകുന്ന കുഞ്ഞുങ്ങളും ഇയാളുടെ സ്വത്തായിരിക്കും. അവരും അടിമകളായിത്തീരുമെന്നര്‍ഥം. അതല്ലെങ്കില്‍ നിരുപാധികം അവരെ സ്വതന്ത്രരാക്കി വിടാം. അത്തരത്തിലുള്ള സ്വാതന്ത്ര്യം അവരുടെ അനാഥത്വത്തിനാണ് വഴിവെക്കുക; ഗുരുതരമായ മൂല്യത്തകര്‍ച്ചക്കും. മറ്റൊരു മാര്‍ഗമാണ് അവളെ വിവാഹം കഴിക്കാതെതന്നെ, അവളുടെ മാനുഷികമായ എല്ലാ അവകാശങ്ങളും അനുവദിച്ചുകൊണ്ട് ഉടമയോടൊപ്പം താമസിപ്പിക്കുകയെന്നത്. (അവളെ സ്വതന്ത്രയാക്കി വിവാഹം ചെയ്യുക എല്ലാ ഉടമകളുടെ കാര്യത്തിലും പ്രായോഗികമായിരിക്കുകയില്ലല്ലോ).

അങ്ങനെ ജീവിക്കുന്ന അടിമകളുടെ എണ്ണം പരിമിതപ്പെടുത്തിയാല്‍ ആ പരിധിക്കു മുകളില്‍ വരുന്ന ഉടമയോടൊപ്പം കഴിയുന്ന അടിമസ്ത്രീകളെ എന്തു ചെയ്യണമെന്ന പ്രശ്‌നമുത്ഭവിക്കും. അവര്‍ക്ക് സ്വാതന്ത്ര്യം പ്രാപിക്കുവാന്‍ മാര്‍ഗങ്ങളൊന്നുമുണ്ടാവുകയില്ല. അവരുടെ ലൈംഗികത അവഗണിക്കപ്പെടുകയോ ചൂഷണം ചെയ്യപ്പെടുകയോ ചെയ്യും. ഇത് വമ്പിച്ച ധാര്‍മിക പ്രശ്‌നങ്ങള്‍ക്ക് നിമിത്തമാകും.

അടിമവ്യവസ്ഥിതി നിലനില്‍ക്കുന്ന സമൂഹത്തിന്റെ ഭൂമികയില്‍നിന്നുകൊണ്ട് ഈ പ്രശ്‌നത്തെയും നോക്കിക്കണ്ടാല്‍ ഇക്കാര്യത്തില്‍ ഇസ്‌ലാം  നിശ്ചയിച്ച നിയമങ്ങള്‍ പ്രായോഗികമാണെന്ന വസ്തുത വ്യക്തമാവും. പ്രസ്തുത സമൂഹത്തില്‍ ഒരാളുടെ കൈവശം എത്തിച്ചേരുന്ന പുരുഷ അടിമകളുടെ എണ്ണത്തിന് പരിധി കല്‍പിക്കാന്‍ കഴിയില്ല. ഇതുതന്നെയാണ് സ്ത്രീ അടിമകളുടെയും അവസ്ഥ. അത്തരം നിയന്ത്രണങ്ങള്‍ പ്രസ്തുത സമൂഹത്തില്‍ അപ്രായോഗികമാണ് അതുകൊണ്ടുതന്നെയാണ് ഇസ്‌ലാം അതിനു തുനിയാതിരുന്നത്.

ടിമസ്ത്രീയെ വിവാഹം ചെയ്യണമെന്നുള്ളവര്‍ക്ക് അങ്ങനെ ചെയ്യാനുള്ള അനുവാദം ഖുര്‍ആന്‍ നല്‍കുന്നുണ്ട് (4:27)

ഇങ്ങനെയുള്ള വിവാഹം ഇരട്ടി പ്രതിഫലം നല്‍കുന്നതാണെന്നാണ് പ്രവാചകന്‍ (ﷺ)പഠിപ്പിച്ചിരിക്കുന്നത്. ”തന്റെ കീഴിലുള്ള അടിമസ്ത്രീയെ സംസ്‌കാര സമ്പന്നയാക്കുക യും അവള്‍ക്ക് ഏറ്റവും നന്നായി വിദ്യാഭ്യാസം നല്‍കുകയും പിന്നീട് അവളെ മോചിപ്പിച്ച് സ്വയം വിവാഹം കഴിക്കുകയും ചെയ്തവനും ഇരട്ടി പ്രതിഫലമുണ്ട്” (ബുഖാരി, മുസ്‌ലിം).

അടിമയുടെ രക്ഷിതാവ് ഉടമയാണ്, പുരുഷനായിരുന്നാലും സ്ത്രീയായിരുന്നാലുമെല്ലാം. പുരുഷനായ ഉടമയുടെ കീഴില്‍ കഴിയുന്ന അടിമസ്ത്രീയുടെ കൈകാര്യകര്‍തൃത്വം ആ പുരുഷനിലാണ് നിക്ഷിപ്തമായിരിക്കുന്നത്.  അവളെ മറ്റൊരാള്‍ക്ക് വിവാഹം ചെയ്തുകൊടുക്കുകയാണെങ്കില്‍ അവനാണ് അത് നിര്‍വഹിക്കേണ്ടത്. അവളുടെ കാര്യങ്ങളെല്ലാം ശ്രദ്ധിക്കേണ്ടതും അവന്‍തന്നെ. അതുകൊണ്ടുതന്നെ അവളെ വിവാഹം ചെയ്യുകയെന്ന കര്‍മം നടക്കേണ്ടതില്ല. സ്ത്രീയുടെ രക്ഷിതാവും വരനും തമ്മില്‍ നടക്കുന്ന കരാറാണ് ഇസ്‌ലാമിലെ വിവാഹം. ഇവിടെ രണ്ടു പേരും ഉടമതന്നെയാണ്. അതുകൊണ്ടുതന്നെ വിവാഹച്ചടങ്ങ് അപ്രസക്തമാണ്.

പുരുഷന്റെ ഭാര്യമാരുടെ എണ്ണം പരമാവധി നാലായിരിക്കണമെന്നാണ് ഖുര്‍ആന്‍ നിഷ്‌കര്‍ഷിക്കുന്നത് (4:3). നാലു ഭാര്യമാരുള്ള ഒരാളുടെ കീഴില്‍ ജീവിക്കുന്ന ഒരു അടിമസ്ത്രീ ഉണ്ടെന്നിരിക്കട്ടെ, അയാള്‍ക്ക് അവളെ വിവാഹം ചെയ്യുവാന്‍ പറ്റുകയില്ല. അവളെ സ്വതന്ത്രയാക്കുവാന്‍ അയാള്‍ സന്നദ്ധനല്ലെങ്കില്‍ പാരതന്ത്ര്യത്തില്‍നിന്നുള്ള മോചനം അവള്‍ക്ക് ഒരു സ്വപ്‌നമായി അവശേഷിക്കും. യജമാനന്‍ വിവാഹത്തിന് സൗകര്യമൊരുക്കിയില്ലെങ്കില്‍ അവളുടെ ലൈംഗികദാഹം ശമിപ്പിക്കുവാന്‍ വ്യഭിചാരത്തെ സമീപിക്കുവാന്‍ അവള്‍ നിര്‍ബന്ധിതയാവും.

എന്നാല്‍, ഉടമക്ക് വിവാഹം കൂടാതെതന്നെ അവളുമായി ലൈംഗികബന്ധം പുലര്‍ത്താമെന്ന നിയമമുള്ളതിനാല്‍ ഈ പ്രശ്‌നങ്ങള്‍ക്കെല്ലാം പരിഹാരമാവും. ഉടമയുടെ കുഞ്ഞിനെ പ്രസവിക്കുകവഴി സ്വാതന്ത്ര്യത്തിലേക്ക് നടന്നുപോകാന്‍ അവള്‍ക്ക് സാധിക്കും. അവളുടെ ലൈംഗികതക്കുള്ള പരിഹാരമാകും. അടിമത്തം നിലനില്‍ക്കുന്ന ഒരു സമൂഹത്തില്‍ അവളുടെ ലൈംഗികദാഹം ശമിപ്പിക്കുവാനുള്ള സംവിധാനമുണ്ടാക്കിയില്ലെങ്കില്‍ വമ്പിച്ച മൂല്യത്തകര്‍ച്ചക്കാണ് അതു നിമിത്തമാവുക. അതോടൊപ്പംതന്നെ വ്യഭിചാരത്തിലൂടെയുണ്ടാകുന്ന കുഞ്ഞുങ്ങളുടെ പ്രശ്‌നവുമുണ്ട്.  അവരും സ്വാഭാവികമായും അടിമകളായി മാറുകയാണ് ചെയ്യുക. അടിമത്തം ഒരിക്കലും അവസാനിപ്പിക്കാനാവാത്ത ഒരു സ്ഥാപനമായിത്തീരുകയാണ് ഇതിന്റെ ഫലം.

ഇസ്‌ലാമാകട്ടെ, അടിമസ്ത്രീകളുമായി ബന്ധപ്പെടുവാന്‍ ഉടമകളെ അനുവദിക്കുക വഴി അടിമത്തത്തെ ഒരു തലമുറയോടെ ഇല്ലാതാക്കുവാനുള്ള സംവിധാനമാണുണ്ടാക്കുന്നത്. അതിന് ‘വിവാഹം‘ഒരു നിബന്ധനയായി വെക്കുകയാണെങ്കില്‍ ഇസ്‌ലാം ഉദ്ദേശിക്കുന്ന ഫലങ്ങളുണ്ടാക്കുവാന്‍ അതുമൂലം കഴിയുകയില്ല. അടിമസ്ത്രീയെ വിവാഹം ചെയ്യുവാന്‍ എല്ലാ ഉടമകളും തയാറാവുകയില്ലല്ലോ. നാല് ഭാര്യമാരുള്ളവര്‍ക്ക് അത് അസാധ്യവുമാണ്.

അടിമത്ത വ്യവസ്ഥിതിയുടെ സ്വാഭാവികമായ ഉൽപ്പന്നമാണ് അടിമ സ്ത്രീകള് .അടിമ സ്ത്രീകളെ യഥേഷ്ടം ഉപയോഗികുകയും അവളെ  വ്യഭിചാരത്തിന് പ്രേരിപ്പിച്ചുകൊണ്ട് അതിൽനിന്ന് സാമ്പത്തിക ലാഭമുണ്ടാക്കുകയുമായിരുന്നു  അടിമത്തം നിലനിന്ന സമൂഹങ്ങളിലെല്ലാം ഉണ്ടായിരുന്ന അവസ്ഥ. ഇസ്ലാമാകട്ടെ, അടിമസ്ത്രീകളിലൂടെ പ്രസ്തുത വ്യവസ്ഥ തന്നെ ഒരു തലമുറയോടുകൂടി അവസ്സാനിപ്പിക്കുവാനാവശ്യമായ സംവിധാനങ്ങളാണ് ചെയ്തത്.

അടിമസ്ത്രീ ഉടമയുടെ സ്വത്താണ്. എന്നാല്‍, അവളെ വ്യഭിചാരത്തിന് പ്രേരിപ്പിക്കാന്‍ ഉടമക്ക് അവകാശമില്ല. (24:33).പുരുഷന്മാരായ അടിമകളെപ്പോലെ സ്ത്രീ അടിമകള്‍ക്കും അവകാശങ്ങളുണ്ട്. അവര്‍ക്ക് ഭക്ഷണം, പാര്‍പ്പിടം, വസ്ത്രം എന്നിവയെല്ലാം നല്‍കേണ്ടത് യജമാനന്റെ കടമയാണ്. അവര്‍ക്ക് വിവാഹത്തിനുള്ള സൗകര്യം ചെയ്തുകൊടുക്കാനും ഖുര്‍ആ ന്‍ ഉടമയോട് ആവശ്യപ്പെടുന്നുണ്ട് (24:32).

അവരുടെ ലൈംഗികമായ ആവശ്യങ്ങളുടെ പൂര്‍ത്തീകരണത്തിനാവശ്യമായ സംവിധാനങ്ങള്‍ ചെയ്യാന്‍ ഖുര്‍ആന്‍ ഉടമകളെ പ്രേരിപ്പിക്കുന്നുണ്ടെന്നര്‍ഥം.

എന്നാല്‍, വിവാഹിതയല്ലാത്ത ഒരു അടിമ സ്ത്രീയുമായി ലൈംഗിക ബന്ധം സ്ഥാപിക്കുന്നതില്‍നിന്ന് ഉടമയെ ഇസ്‌ലാം വിലക്കുന്നില്ല. ഈ അനുവാദം ഉടമയില്‍ മാത്രം പരിമിതമാണ്. മറ്റൊരാള്‍ക്കും അവളെ ഉപയോഗിക്കുവാന്‍ അനുവാദമില്ല. യജമാനന്റെ അനുവാദത്തോടെയാണെങ്കിലും ശരി!

യജമാനന്റെ കുഞ്ഞിനെ പ്രസവിക്കുന്നതോടുകൂടി അടിമസ്ത്രീക്ക് പുതിയ അവകാശങ്ങളുണ്ടാവുകയാണ്. അവളെ പിന്നെ വില്‍ക്കുവാന്‍ യജമാനന് അവകാശമില്ല. അവള്‍ പിന്നെ യജമാനന്റെ കുട്ടികളുടെ മാതാവാണ്. ആ കുട്ടികള്‍ക്കോ യജമാനന്റെ മറ്റു കുട്ടികളെപ്പോലെയുള്ള എല്ലാ അവകാശങ്ങളുമുണ്ടുതാനും. അടിമസ്ത്രീയിലുണ്ടാവുന്ന കുഞ്ഞുങ്ങള്‍ക്ക് പിതൃസ്വത്തില്‍ അവകാശങ്ങളൊന്നുമില്ലെന്ന യഹൂദ നിലപാടുമായി ഇസ്‌ലാം വിയോജിക്കുന്നു. ആ കുട്ടികള്‍ എല്ലാ അര്‍ഥത്തിലും അയാളുടെ മ ക്കള്‍തന്നെയാണ്. യാതൊരു തരത്തിലുള്ള ഉച്ചനീചത്വങ്ങളും അവരും മറ്റു മക്കളും തമ്മില്‍ ഉണ്ടാകുവാന്‍ പാടില്ല.

യജമാനന്റെ മരണത്തോടെ അയാളുടെ മക്കളുടെ ഉമ്മയായ അടിമസ്ത്രീ സ്വതന്ത്രയായിത്തീരുന്നു. പിന്നെ അവളെ സംരക്ഷിക്കുന്നത് അവളുടെ മക്കളാണ്. അവര്‍ക്കാണെങ്കില്‍ പിതൃസ്വത്തില്‍നിന്ന് മറ്റു മക്കളെപ്പോലെതന്നെയുള്ള അവകാശം ലഭിക്കുകയും ചെയ്യും.

ഒരു തലമുറയോടെ അടിമത്തത്തിന്റെ വേരറുക്കുന്നതിനുവേണ്ടിയുള്ള ഒരു സംവിധാനമാണ് ദാസിമാരെ ജീവിത പങ്കാളികളായി സ്വീകരിക്കാനുള്ള അനുവാദം. അടിമത്തം പ്രായോഗികമായി ഇല്ലാതാക്കുന്നതിനുവേണ്ടിയുള്ള ഇസ്‌ലാമിന്റെ വ്യത്യസ്തമായ നടപടികളിലൊന്നായിരുന്നു അടിമസ്ത്രീകളുമായി ബന്ധപ്പെടുന്നതിനുവേണ്ടിയുള്ള അനുവാദമെന്നുള്ളതാണ് വാസ്തവം. ഇസ്‌ലാമിക ലോകത്ത് നടന്നതും അതുതന്നെയാണ്.

അടിമസ്ത്രീകളുമായി ബന്ധപ്പെടുന്നതിനുള്ള അനുവാദത്തെ വ്യഭിചാരവുമായി താരതമ്യം ചെയ്യാനേ കഴിയില്ല. രണ്ടും രണ്ട് വിരുദ്ധ ധ്രൂവങ്ങളില്‍ നില്‍ക്കുന്നു. ഒന്ന് ഉടമയുടെ കീഴില്‍ അയാളുടെ സംരക്ഷണത്തില്‍ കഴിയുന്നവളുമായുള്ള ബന്ധമാണ്. ആ ബന്ധത്തില്‍നിന്നുണ്ടാകുന്ന ബാധ്യതകള്‍ ഏറ്റെടുക്കാന്‍ സന്നദ്ധനായിക്കൊണ്ടുള്ള ബന്ധം. ലൈംഗികതയ്ക്കപ്പുറമുള്ള അവളുടെ വ്യക്തിത്വം അംഗീകരിച്ചുകൊണ്ടുള്ള ബന്ധം. അവള്‍ക്ക് സ്വാതന്ത്ര്യത്തിലേക്കുള്ള വാതായനം തുറന്നുകൊടുക്കുന്ന ബന്ധം. രണ്ടാമത്തേതോ ഒരു ഉത്തരവാദിത്വവുമില്ലാത്ത വേശ്യാബന്ധം. വേശ്യ യഥാര്‍ഥത്തില്‍ അടിമയേക്കാള്‍ പതിതയാണ്. അവള്‍ ആത്മാവില്ലാത്ത ഒരു മൃഗം മാത്രം. പുരുഷന്റെ മാംസദാഹം തീര്‍ക്കുകയാണ് അവളുടെ കര്‍ത്തവ്യം. ആ ബന്ധത്തില്‍ സ്‌നേഹത്തിന്റെയും കാരുണ്യത്തിന്റെയും കണികപോലുമില്ല. ആത്മസംതൃപ്തിയുടെ സ്പര്‍ശം ലേശം പോലുമില്ല. പണത്തിനുവേണ്ടി നടത്തുന്ന ഒരു കച്ചവടം മാത്രമാണത്. മാംസക്കച്ചവടം! അതില്‍നിന്നുള്ള ബാധ്യതയേറ്റെടുക്കുവാന്‍ മാംസദാഹം തീര്‍ക്കുവാന്‍ വേണ്ടി വന്ന പുരുഷന്‍ സന്നദ്ധനല്ല. അവള്‍ക്ക് എന്തെങ്കിലുമൊരു അവകാശം അവന്റെ മേല്‍ ഇല്ല. അവന്റെ മാംസദാഹം തീര്‍ക്കാന്‍ വിധിക്കപ്പെട്ട ഒരു മൃഗം മാത്രമാണവള്‍. അവളുടെ ഓരോ ബന്ധവും അവളെ വേശ്യാവൃത്തിയുടെ മൃഗീയതയിലേക്ക് കൂടുതല്‍ ആഴത്തില്‍ ആപതിപ്പിക്കുകയാണ് ചെയ്യുന്നത്. അവള്‍ക്ക് സ്വാതന്ത്ര്യത്തെക്കുറിച്ച സ്വപ്‌നം പോലും അന്യമാണ്. തൊലി ചുളിഞ്ഞ് ആര്‍ക്കും വേണ്ടാതായി മാറി രോഗിണിയാവുമ്പോള്‍ അനാഥത്വം പേറുവാന്‍ വിധിക്കപ്പെട്ടവള്‍!

അടിമക്കു സ്വാതന്ത്ര്യത്തിലേക്കുള്ള വാതായനം തുറന്നുകൊടുക്കുന്ന, ഇസ്‌ലാം അനുവദിച്ച സമ്പ്രദായമെവിടെ? സ്ത്രീയെ പാരതന്ത്ര്യത്തില്‍നിന്ന് പാരതന്ത്ര്യത്തിലേക്കു നയിക്കുന്ന ദുഷിച്ച വ്യഭിചാരവ്യവസ്ഥയെവിടെ? ഇവ രണ്ടും തമ്മില്‍ താരതമ്യം പോലും അസാധ്യമാണ്. രണ്ടും രണ്ടു വിരുദ്ധ ധ്രുവങ്ങളില്‍ നില്‍ക്കുന്ന സമ്പ്രദായങ്ങള്‍. ഒന്ന് മനുഷ്യത്വം അംഗീകരിക്കുന്നത്, മറ്റേത് മൃഗീയതയിലേക്ക് ആപതിക്കുന്നത്.

അടിമത്തം നിര്‍മാര്‍ജനം ചെയ്യുന്നതിനായി അഞ്ച് മാര്‍ഗങ്ങളിലൂടെ ഖുര്‍ആന്‍ ശ്രമിച്ചതായി കാണാന്‍ കഴിയും.

  1. സാഹോദര്യം വളര്‍ത്തി: സര്‍വ മനുഷ്യരും ദൈവസൃഷ്ടികളും ഒരേ മാതാപിതാക്കളുടെ മക്കളുമാണെന്ന ബോധം വളര്‍ത്തിക്കൊണ്ട് അടിമയും ഉടമയുമെല്ലാം സഹോദരങ്ങളാണെന്ന ധാരണയുണ്ടാക്കുകയാണ് ഖുര്‍ആന്‍ ആദ്യമായി ചെയ്തത്.”മനുഷ്യരേ,ഒരു പുരുഷനില്‍നിന്നും സ്ത്രീയില്‍നിന്നുമാണ് നിങ്ങളെ നാം പടച്ചിരിക്കുന്നത്, തീര്‍ച്ച. ഗോത്രങ്ങളും ജനപദങ്ങളുമായി നിങ്ങളെ തിരിച്ചിരിക്കുന്നത് പരസ്പരം തിരിച്ചറിയുന്നതിനായാണ്. അല്ലാഹുവിങ്കല്‍ നിങ്ങളിലെ ഭക്തനാണ് ഉത്തമന്‍” (ഖുര്‍ആന്‍ 49:13).

ജന്മത്തിന്റെ പേരിലുള്ള സകലമാന സങ്കുചിതത്തങ്ങളുടെയും അടിവേരറുക്കുകയാണ് ഇവിടെ ഖുര്‍ആന്‍ ചെയ്തിരിക്കുന്നത്. നിറത്തിന്റെയോ കുലത്തിന്റെയോ പണത്തിന്റെയോ അടിസ്ഥാനത്തിലല്ല പ്രത്യുത ഭക്തിയുടെ അടിസ്ഥാനത്തിലാണ് ശ്രേഷ്ഠത നിശ്ചയിക്കപ്പെടുന്നതെന്നാണ് പ്രവാചകന്‍ ( ﷺ) പഠിപ്പിച്ചത്. ”അറബിക്ക് അനറബിയേക്കാളുമോ അനറബിക്ക് അറബിയെക്കാളുമോ വെളുത്തവന് കറുത്തവനെക്കാളുമോ കറുത്തവന് വെളുത്തവനെക്കാളുമോ യാതൊരു ശ്രേഷ്ഠതയുമില്ല, ദൈവഭക്തിയുടെ അടിസ്ഥാനത്തിലല്ലാതെ” (ത്വബ്‌രി).

അടിമകളെക്കുറിച്ച് പരാമര്‍ശിക്കുന്നിടത്ത് ”നിങ്ങള്‍ ചിലര്‍ ചിലരില്‍ നിന്നുണ്ടായവരാണല്ലോ” (ഖുര്‍ആന്‍ 4:25) എന്ന് വിശുദ്ധ ഖുര്‍ആന്‍ പറയുന്നുണ്ട്. അടിമയും ഉടമയുമെല്ലാം സഹോദരന്മാരാണെന്നും സാഹചര്യങ്ങളാണ് ചിലരുടെ മേല്‍ അടിമത്തം അടിച്ചേല്‍പിച്ചതെന്നുമുള്ള വസ്തുതകള്‍ വ്യക്തമാക്കുകയാണ് ഇവിടെ ഖുര്‍ആന്‍ ചെയ്യുന്നത്.

  1. അടിമയുടെ അവകാശങ്ങളെക്കുറിച്ച് ബോധ്യം വരുത്തി: അടിമ കേവലം ഒരു ഉപഭോഗവസ്തു മാത്രമായിരുന്നു,പൗരാണിക സമൂഹങ്ങളിലെല്ലാം. അവന് ബാധ്യതകള്‍ മാത്രമേ ഉണ്ടായിരുന്നുള്ളൂ. ഉടമയുടെ സുഖസൗകര്യങ്ങള്‍ വര്‍ധിപ്പിക്കുന്നതിനു വേണ്ടി യത്‌നിക്കുകയായിരുന്നു അവന്റെ ബാധ്യത- അതില്‍ യാതൊരു വിട്ടുവീഴ്ചയുമുണ്ടായിരുന്നില്ല. ഉടമക്കുവേണ്ടി പണിയെടുക്കുന്നതിന് അടിമയുടെ ആരോഗ്യം നിലനിര്‍ത്തേണ്ടത് അനിവാര്യമായിരുന്നു. അതിനുവേണ്ടി മാത്രമായിരുന്നു അവന് ഭക്ഷണം നല്‍കിയിരുന്നത്. കാലികള്‍ക്കു നല്‍കുന്ന സൗകര്യംപോലും ഇല്ലാത്ത തൊഴുത്തുകളിലായിരുന്നു അവരെ താമസിപ്പിച്ചിരുന്നത്. അവര്‍ക്ക് നല്‍കിയിരുന്ന വസ്ത്രമാകട്ടെ, കേവലം നാണം മറക്കാന്‍പോലും അപര്യാപ്തമായ രീതിയിലുള്ളതായിരുന്നു. അതും വൃത്തികെട്ട തുണിക്കഷ്ണങ്ങള്‍!

ഇസ്‌ലാം ഈ അവസ്ഥക്ക് മാറ്റം വരുത്തി. അടിമ ഉടമയുടെ സഹോദരനാണെന്നും അവന് അവകാശങ്ങളുണ്ടെന്നും പഠിപ്പിച്ചു. പ്രവാചകന്‍ നിഷ്‌കര്‍ഷിച്ചു: ”നിങ്ങളുടെ സഹോദരങ്ങളും ബന്ധുക്കളുമാണവര്‍! തന്റെ കീഴിലുള്ള ഒരു സഹോദരന് താന്‍ കഴിക്കുന്നതുപോലെയുള്ള ഭക്ഷണവും താന്‍ ധരിക്കുന്നതുപോലെയുള്ള വസ്ത്രവും നല്‍കിക്കൊള്ളട്ടെ. അവര്‍ക്ക് കഴിയാത്ത ജോലികളൊന്നും അവരെ ഏല്‍പിക്കരുത്. അവര്‍ക്ക് പ്രയാസകരമായ വല്ല പണികളും ഏല്‍പിക്കുകയാണെങ്കില്‍ നിങ്ങള്‍ അവരെ സഹായിക്കണം” (ബുഖാരി,മുസ്‌ലിം).

അധ്വാനിക്കുകയെന്നതു മാത്രമായിരുന്നില്ല പൗരാണിക സമൂഹങ്ങളില്‍ അടിമയുടെ കര്‍ത്തവ്യം. യജമാനന്റെ ക്രൂരമായ വിനോദങ്ങള്‍ ഏറ്റുവാങ്ങുവാന്‍ കൂടി വിധിക്കപ്പെട്ടവനായിരുന്നു അവന്‍. അധ്വാനവേളകളില്‍ ക്രൂരമായ ചാട്ടവാറടികള്‍! യജമാനന്റെ ആസ്വാദനത്തിനുവേണ്ടി കൊല്ലുവാനും കൊല്ലപ്പെടുവാനും തയാറാവേണ്ട അവസ്ഥ! ഇത് മാറണമെന്ന് ഖുര്‍ആന്‍ കല്‍പിച്ചു. അടിമകളോട് നല്ല നിലയില്‍ പെരുമാറണമെന്ന് നിഷ്‌കര്‍ഷിച്ചു. ”ബന്ധുക്കളോടും അനാഥകളോടും പാവങ്ങളോടും കുടുംബബന്ധമുള്ള അയല്‍ക്കാരോടും അന്യരായ അയല്‍ക്കാരോടും സഹവാസിയോടും വഴിപോക്കനോടും നിങ്ങളുടെ വലതുകൈകള്‍ ഉടമപ്പെടുത്തിയ അടിമകളോടും നല്ല നിലയില്‍ വര്‍ത്തിക്കുക” (ഖുര്‍ആന്‍ 4:36).

പ്രവാചകന്‍ വ്യക്തമായി പറഞ്ഞു: ”വല്ലവനും തന്റെ അടിമയെ വധിച്ചാല്‍ നാം അവനെയും വധിക്കും. വല്ലവനും തന്റെ അടിമയെ അംഗവിഛേദം ചെയ്താല്‍  നാം അവനെയും അംഗവിഛേദം ചെയ്യും. വല്ലവനുംതന്റെ അടിമയെ ശണ്ഡീകരിച്ചാല്‍ നാം അവനെയും ശണ്ഡീകരിക്കും” (മുസ്‌ലിം, അബൂദാവൂദ്).

യജമാനന് ഇഷ്ടമുള്ളതെല്ലാം ചെയ്യാവുന്ന ‘ചരക്ക്‘ എന്ന അവസ്ഥ യില്‍ നിന്ന് അടിമ സ്വന്തമായ വ്യക്തിത്വവും സ്വന്തമായ അവകാശങ്ങളുമുള്ളവനായിത്തീരുകയായിരുന്നു. അടിമകളെ ഷണ്ഡീകരിക്കുകയെന്ന അതിനികൃഷ്ടമായ സമ്പ്രദായം നിലനിന്നിരുന്ന സമൂഹത്തിലാണ് അവരെ ഷണ്ഡീകരിച്ചാല്‍ അതു ചെയ്ത യജമാനനെ ഞാനും ഷണ്ഡീകരിക്കുമെന്ന് പ്രവാചകന്‍ ( ﷺ) അര്‍ഥശങ്കയില്ലാത്തവിധം വ്യക്തമാക്കിയത്. ലൈംഗിക വികാരം നശിപ്പിച്ചുകൊണ്ട് അടിമകളെക്കൊണ്ട് മൃഗതുല്യമായി അധ്വാനിപ്പിക്കുന്നതിനുവേണ്ടിയായിരുന്നു അവരെ ഷണ്ഡീകരിച്ചിരുന്നത്. ഇത് നിരോധിച്ച ഇസ്‌ലാം അടിമകള്‍ക്കും വികാരശമനത്തിനും മാര്‍ഗമുണ്ടാക്കണമെന്ന് പ്രത്യേകം നിഷ്‌കര്‍ഷിക്കുന്നുണ്ട്. ”നിങ്ങളിലുള്ള അവിവാഹിതരെയും നിങ്ങളുടെ അടിമകളില്‍ നിന്നും അടിമസ്ത്രീകളില്‍നിന്നും നല്ലവരായിട്ടുള്ളവരെയും നിങ്ങള്‍ വിവാഹബന്ധത്തിലേര്‍പ്പെടുത്തുക. അവര്‍ ദരിദ്രരാണെങ്കില്‍ അല്ലാഹു തന്റെ അനുഗ്രഹത്തില്‍നിന്ന് അവര്‍ക്ക് ഐശ്വര്യം നല്‍കുന്നതാണ്. അല്ലാഹു വിപുലമായ കഴിവുള്ളവനും സര്‍വജ്ഞനുമത്രെ” (24:32).

അടിമസ്ത്രീകളെ വേശ്യാവൃത്തിക്ക് നിര്‍ബന്ധിക്കുന്ന സമ്പ്രദായത്തെ ഖര്‍ആന്‍ വിലക്കി. ”ചാരിത്ര്യശുദ്ധിയോടെ ജീവിക്കാനാഗ്രഹിക്കുന്ന നിങ്ങളുടെ അടിമസ്ത്രീകളെ ഐഹിക ജീവിതത്തിന്റെ വിഭവം ആഗ്രഹിച്ചുകൊണ്ട് നിങ്ങള്‍ വേശ്യാവൃത്തിക്ക് നിര്‍ബന്ധിക്കരുത് (24:33).

അടിമത്തം നിലനിന്ന സമൂഹങ്ങളെല്ലാം സ്വതന്ത്രമായ സര്‍ഗശേഷിയോ അഭിമാനമോ ഇല്ലാത്തവരായി ആയിരുന്നു അവരെ പരിഗണിച്ചിരുന്നത്. എന്നാല്‍, അടിമക്കും അഭിമാനമുണ്ടെന്നും അത് ക്ഷതപ്പെടുത്താന്‍ ആര്‍ക്കും അവകാശമില്ലെന്നുമുള്ള വസ്തുതയാണ് പ്രവാചകന്‍ പഠിപ്പിച്ചത്. ഒരു അടിമയുമായി ശണ്ഠകൂടിയപ്പോള്‍ അയാളെ ‘കറുത്ത പെണ്ണിന്റെ മോനേ‘ എന്നുവിളിച്ച തന്റെ ശിക്ഷ്യനായ അബുദര്‍റിനെ പ്രവാചകന്‍( ﷺ) ഗുണദോഷിച്ചത് ഇങ്ങനെയായിരുന്നു.”അബുദര്‍റേ… അജ്ഞാനകാലത്തെ സംസ്‌കാരത്തില്‍ ചിലത് ഇനിയും താങ്കളില്‍ ബാക്കിയുണ്ട്”(അബൂദാവൂദ്).

”നിങ്ങളുടെ നേതാവായി വരുന്നത് ഉണങ്ങിയ മുന്തിരിപോലെ തലയുള്ള ഒരു നീഗ്രോ അടിമയാണെങ്കിലും നിങ്ങള്‍ അയാളെ കേള്‍ക്കുകയും അനുസരിക്കുകയും വേണം” എന്നാണ് നബി( ﷺ)പഠിപ്പിച്ചത്. അടിമയെ പിറകില്‍ നടത്തിക്കൊണ്ട് വാഹനത്തില്‍ സഞ്ചരിക്കുകയായിരുന്ന ഒരാളോട് പ്രവാചക ശിക്ഷ്യനായ അബൂഹുറയ്‌റ(റ) പറഞ്ഞു. ”നിന്റെ പിറകില്‍ അവനെയും കയറ്റുക. നിന്റെ സഹോദരനാണവന്‍, നിന്‍േറതുപോലുള്ള ആത്മാവാണ് അവനുമുള്ളത്”.

അടിമക്കും ഉടമക്കും ഒരേ ആത്മാവാണുള്ളതെന്നും അവര്‍ തമ്മില്‍ സഹോദരങ്ങളാണെന്നും പഠിപ്പിച്ചുകൊണ്ട് അടിമ-ഉടമ ബന്ധത്തിന് ഒരു പുതിയ മാനം നല്‍കുകയാണ് ഇസ്‌ലാം ചെയ്തത്. അടിമ, ഉടമയുടെ അധീനത്തിലാണെന്നത് ശരിതന്നെ. എന്നാല്‍, അടിമയുടെ അവകാശങ്ങള്‍ വകവെച്ചുകൊടുക്കാന്‍ ഉടമ ബാധ്യസ്ഥനാണ്. ഭക്ഷണം, വസ്ത്രം,ലൈംഗികത തുടങ്ങിയ അടിമയുടെ ആവശ്യങ്ങള്‍ നിര്‍വഹിച്ചുകൊടുക്കേണ്ടത് അയാളുടെ ചുമതലയാണ്. അടിമയെ ഉപദ്രവിക്കാന്‍ പാടില്ല. അയാളെ പ്രയാസകരമായ ജോലികള്‍ ഏല്‍പിച്ച് ക്ലേശിപ്പിക്കുവാനും പാടില്ല. ഇങ്ങനെ, ചരിത്രത്തിലാദ്യമായി അടിമയെ സ്വതന്ത്രന്റെ വിതാനത്തിലേക്കുയര്‍ത്തുകയെന്ന വിപ്ലവം സൃഷ്ടിക്കുകയാണ് ഇസ്‌ലാം ചെയ്തത്. ഇതുവഴി ഉടമയുടെയും അടിമയുടെയും മാനസികാവസ്ഥകള്‍ തമ്മിലുള്ള അന്തരം കുറക്കുവാന്‍ ഇസ്‌ലാമിന് സാധിച്ചു. തന്റെ ഇഷ്ടങ്ങളെല്ലാം പ്രയോഗിക്കാവുന്ന ഒരു ചരക്ക് മാത്രമാണ് അടിമയെന്ന വിചാരത്തില്‍നിന്ന് ഉടമയും,സഹിക്കുവാനും ക്ഷമിക്കുവാനും നിര്‍വഹിക്കുവാനും മാത്രം വിധിക്കപ്പെട്ടവനാണ് താനെന്ന വിചാരത്തില്‍നിന്ന് അടിമയും സ്വതന്ത്രരാവുകയായിരുന്നു ഈ വിപ്ലവത്തിന്റെ ഫലം.

  1. അടിമമോചനം ഒരു പുണ്യകര്‍മമായി പ്രഖ്യാപിച്ചു: അവകാശങ്ങളുള്ള ഒരു അസ്തിത്വമായി അടിമയെ പ്രഖ്യാപിക്കുക വഴി അടിമത്തത്തെ സാങ്കേതികമായി ഇല്ലാതാക്കുകയാണ് ഇസ്‌ലാം ചെയ്തത്. എന്നാല്‍,ഇതുകൊണ്ടും നിര്‍ത്താതെ ആ സമ്പ്രദായത്തെ പ്രായോഗികമായിത്തന്നെ നിര്‍ മൂലനം ചെയ്യുവാന്‍  ആവശ്യമായ നടപടിയിലേക്ക്  ഇസ്‌ലാം തിരിയുകയുണ്ടായി. അടിമമോചനം ഒരു പുണ്യകര്‍മമായി പ്രഖ്യാപിക്കുകയായിരുന്നു അടിമ സമ്പ്രദായത്തെ പ്രായോഗികമായി ഇല്ലാതാക്കുവാന്‍ ഇസ്‌ലാം സ്വീകരിച്ച നടപടി.”അവരുടെ ഭാരങ്ങളും അവരുടെ മേലുണ്ടായിരുന്ന വിലങ്ങുകളും അദ്ദേഹം ഇറക്കിവെക്കുകയും ചെയ്യുന്നു” (7:157) എന്ന ഖുര്‍ആനിക പരാമര്‍ശത്തെ അന്വര്‍ഥമാക്കുന്നതായിരുന്നു അടിമമോചനത്തിന്റെ വിഷയത്തില്‍ പ്രവാചക ( ﷺ)ന്റെ നിലപാട്.

അടിമമോചനം അതിവിശിഷ്ടമായ ഒരു പുണ്യകര്‍മമാണെന്ന് വ്യക്തമാക്കുന്ന ഖുര്‍ആന്‍ സൂക്തം ഇങ്ങനെയാണ്. ”എന്നിട്ട് അവന്‍ ആ മലമ്പാത താണ്ടിക്കടന്നില്ല. ആ മലമ്പാതയെന്താണെന്ന് നിനക്കറിയാമോ?അടിമ മോചനം. അല്ലെങ്കില്‍ പട്ടിണിയുടെ നാളില്‍ കുടുംബബന്ധമുള്ള ഒരുഅനാഥക്കോ കടുത്ത ദാരിദ്ര്യമുള്ള ഒരു സാധുവിനോ ഭക്ഷണം നല്‍കല്‍” (90:12-16)

അടിമമോചനത്തിന്റെ കാര്യത്തില്‍ പ്രവാചകന്‍ ( ﷺ)തന്നെ മാതൃക കാണിച്ചുകൊണ്ടാണ് അനുചരന്മാരെ അതിനുവേണ്ടി പ്രേരിപ്പിച്ചത്. തന്റെ കൈവശമുണ്ടായിരുന്ന അടിമയെ അദ്ദേഹം മോചിപ്പിച്ചു. അദ്ദേഹത്തിന്റെ അനുചരന്മാര്‍ പ്രസ്തുത പാത പിന്തുടര്‍ന്നു. സഖാക്കളില്‍ പ്രമുഖനായിരുന്ന അബൂബക്കര്‍( ﷺ)സത്യനിഷേധികളില്‍നിന്ന് അടിമകളെ വിലയ്ക്കുവാങ്ങി മോചിപ്പിക്കുന്നതിനായി അളവറ്റ സമ്പത്ത് ചെലവഴിച്ചിരുന്നതായി കാണാനാവും.

അടിമമോചനത്തെ പ്രോല്‍സാഹിപ്പിക്കുന്ന ഒട്ടേറെ നബിവചനങ്ങള്‍ കാണാന്‍ കഴിയും: ”സത്യവിശ്വാസിയായ ഒരു അടിമയെ ആരെങ്കിലും മോചിപ്പിച്ചാല്‍ ആ അടിമയുടെ ഓരോ അവയവത്തിനും പകരം അല്ലാഹു അവന്റെ അവയവത്തിന് നരകത്തില്‍നിന്ന് മോചനം നല്‍കുന്നതാണ്. അഥവാ കയ്യിന് കയ്യും കാലിന് കാലും ഗുഹ്യാവയവത്തിന് ഗുഹ്യാവയവവും വരെ” (ബുഖാരി, മുസ്‌ലിം).

സഹാബിയായിരുന്ന അബുദര്‍റ്(റ) ഒരിക്കല്‍ നബി( ﷺ)യോട് ചോദിച്ചു: ‘അടിമമോചനത്തില്‍ ഏറ്റവും ശ്രേഷ്ഠമായത് ഏതാണ്?തിരുമേനി പ്രതിവചിച്ചു: ‘യജമാനന് ഏറ്റവും വിലപ്പെട്ട  അടിമകളെ മോചിപ്പിക്കല്‍‘.

അല്ലാഹുവിന്റെ പ്രതിഫലത്തിന് രണ്ടു തവണ അര്‍ഹരാവുന്നവരെ എണ്ണിപ്പറയവെ നബി( ﷺ) പറഞ്ഞു: ”തന്റെ കീഴിലുള്ള അടിമസ്ത്രീയെ സംസ്‌കാര സമ്പന്നയാക്കുകയും അവള്‍ക്ക് ഏറ്റവും നന്നായി വിദ്യാഭ്യാസം നല്‍കുകയും പിന്നീട് അവളെ മോചിപ്പിച്ച് സ്വയം വിവാഹം കഴിക്കുകയും ചെയ്തവനും ഇരട്ടി പ്രതിഫലമുണ്ട്” (ബുഖാരി,മുസ്‌ലിം).

പടച്ചതമ്പുരാനില്‍നിന്നുള്ള പ്രതിഫലം കാംക്ഷിച്ചുകൊണ്ട് സത്യവിശ്വാസികള്‍ പ്രവാചകന്റെ കാലത്തും ശേഷവും അടിമകളെ മോചിപ്പിക്കുവാന്‍ തുടങ്ങി. ഇതുകൂടാതെ സകാത്തിന്റെ ധനം പോലും അടിമമോചനത്തിന് ചെലവഴിക്കുന്ന അവസ്ഥയുണ്ടായി. ഉമറുബ്‌നു അബ്ദില്‍ അസീസി(റ)ന്റെ ഭരണകാലത്ത് സകാത്ത് സ്വീകരിക്കുവാന്‍ ഒരു ദരിദ്രന്‍ പോലുമില്ലാത്ത അവസ്ഥ സംജാതമായെന്നും അപ്പോള്‍ അടിമകളെ വിലക്കെടുത്ത് മോചിപ്പിക്കാനാണ് സകാത്ത് ഇനത്തിലുള്ള ധനം ചെലവഴിക്കപ്പെട്ടതെന്നും ചരിത്രത്തില്‍ കാണാന്‍ കഴിയും.

  1. പലതരം കുറ്റങ്ങള്‍ക്കുമുള്ള പ്രായശ്ചിത്തമായി അടിമമോചനം നിശ്ചയിക്കപ്പെട്ടു: അടിമമോചനത്തെ ഒരു പുണ്യകര്‍മമായി അവതരിപ്പിച്ചുകൊണ്ട് സത്യവിശ്വാസികളെ അക്കാര്യത്തില്‍ പ്രോല്‍സാഹിപ്പിച്ചതോടൊപ്പം തന്നെ പലതരം കുറ്റകൃത്യങ്ങള്‍ക്കുള്ള പ്രായശ്ചിത്തമായി അടിമമോചനത്തെ ഇസ്‌ലാം നിശ്ചയിച്ചു. മനഃപൂര്‍വമല്ലാത്ത കൊലപാതകം,അല്ലാഹുവിന്റെ പേരില്‍ ശപഥം ചെയ്ത ശേഷം അത് ലംഘിക്കല്‍, ഭാര്യയെ സമീപിക്കുകയില്ലെന്ന ശപഥത്തിന്റെ ലംഘനം തുടങ്ങിയ പാപങ്ങള്‍ക്കുള്ള പ്രായശ്ചിത്തങ്ങളിൽ ഒന്ന് ഒരു അടിമയെ മോചിപ്പിക്കുകയാണ്. ദൈവിക പ്രതിഫലം കാംക്ഷിച്ചുകൊണ്ടുമാത്രം അടിമകളെ മോചിപ്പിക്കാന്‍ തയാറില്ലാത്തവരെ സംബന്ധിച്ചിടത്തോളം അത് നിര്‍ബന്ധമാക്കിത്തീര്‍ക്കുന്ന അവസ്ഥയാണ് തെറ്റുകള്‍ക്കുള്ള പ്രായശ്ചിത്തമായി അടിമകളെ മോചിപ്പിക്കണമെന്ന വിധി.
  1. മോചനമൂല്യത്തിനു പകരമായി സ്വാതന്ത്ര്യം നല്‍കുന്ന സംവിധാനമുണ്ടാക്കി: മുകളില്‍ പറഞ്ഞ മാര്‍ഗങ്ങളിലൂടെയൊന്നും സ്വതന്ത്രനാകാന്‍ ഒരു അടിമക്ക് സാധിച്ചില്ലെന്നിരിക്കട്ടെ. അപ്പോഴും അവന് മോചനം അസാധ്യമല്ല. സ്വയം മോചനമാഗ്രഹിക്കുന്ന ഏതൊരു അടിമക്കും മോചിതനാകുവാനുള്ള മാര്‍ഗം ഇസ്‌ലാം തുറന്നുകൊടുത്തിട്ടുണ്ട്.‘മുകാതബ‘യെന്ന് സാങ്കേതികമായി വിളിക്കുന്ന മോചനപത്രത്തിലൂടെയാണ് ഇത് സാധ്യമാവുക. സ്വാതന്ത്ര്യമെന്ന അഭിലാഷം ഹൃദയത്തിനകത്ത് മൊട്ടിട്ടു കഴിഞ്ഞാല്‍ ‘മുകാതബ‘യിലൂടെ ഏതൊരു അടിമക്കും സ്വതന്ത്രനാകാവുന്നതാണ്. അടിമയും ഉടമയും യോജിച്ച് ഒരു മോചനമൂല്യവും അത് അടച്ചുതീര്‍ക്കേണ്ട സമയവും തീരുമാനിക്കുന്നു. ഈ മോചനമൂല്യം സമാഹരിക്കുന്നതിനുവേണ്ടി അടിമയ്ക്ക് പുറത്തുപോയി ജോലി ചെയ്യാം. അങ്ങനെ ഗഡുക്കളായി അടിമ മോചനദ്രവ്യം അടച്ചുതീര്‍ക്കുന്നു. അത് അടച്ചുതീര്‍ക്കുന്നതോടെ അയാള്‍ സ്വതന്ത്രനാവുന്നു.

സ്വാതന്ത്ര്യമെന്ന സ്വപ്‌നം പൂവണിയുന്നതിനായി ആ ആഗ്രഹം മനസ്സില്‍ മൊട്ടിട്ടു കഴിഞ്ഞ ഏതൊരു അടിമക്കും അവസരമുണ്ടാക്കി കൊടുക്കുകയാണ് ഇൗ സംവിധാനത്തിലൂടെ ഇസ്‌ലാം ചെയ്തിരിക്കുന്നത്. മോചനപത്രമെഴുതിയ ഒരു അടിമക്ക് നിശ്ചിത സമയത്തിനകം മോചനമൂല്യം അടച്ചുതീര്‍ക്കാന്‍ കഴിഞ്ഞില്ലെങ്കിലോ? അതിനുള്ള സംവിധാനവും ഇസ്ലാം നിര്‍ദേശിക്കുന്നുണ്ട്. സകാത്ത് ധനം ചെലവഴിക്കപ്പെടേണ്ട എട്ടു വകുപ്പുകളിലൊന്ന് അടിമമോചനമാണ് (ഖുര്‍ആന്‍ 9:60). മുകാതബ പ്രകാരമുള്ള മോചനദ്രവ്യം കൊടുത്തുതീര്‍ക്കാന്‍ ഒരു അടിമക്ക് കഴിയാത്ത സാഹചര്യങ്ങളില്‍ അയാള്‍ക്ക് ബൈത്തുല്‍മാലിനെ (പൊതുഖജനാവ്) സമീപിക്കാം. അതില്‍നിന്ന്  നിശ്ചിത സംഖ്യയടച്ച് അയാളെ മോചിപ്പിക്കേണ്ടത് അതു കൈകാര്യം ചെയ്യുന്നവരുടെ കടമയാണ്. പണക്കാരന്‍ നല്‍കുന്ന സ്വത്തില്‍ നിന്നുതന്നെ അടിമയെ മോചിപ്പിക്കുവാനുള്ള വക കണ്ടെത്തുകയാണ് ഇസ്‌ലാം ഇവിടെ ചെയ്തിരിക്കുന്നത്.

അടിമകളെ സ്വാതന്ത്ര്യമെന്താണെന്ന് പഠിപ്പിക്കുകയും പാരതന്ത്ര്യത്തില്‍നിന്ന് മോചിതരാകുവാന്‍  അവരെ സ്വയം സന്നദ്ധരാക്കുകയും ചെയ്തുകൊണ്ട് ചങ്ങലക്കെട്ടുകളില്‍നിന്ന് മുക്തമാക്കുകയെന്ന പ്രായോഗികമായ നടപടിക്രമമാണ് ഇസ്‌ലാം അടിമത്തത്തിന്റെ കാര്യത്തില്‍ സ്വീകരിച്ചത്. അക്കാര്യത്തില്‍ ഇസ്‌ലാം സ്വീകരിച്ചതിനേക്കാള്‍ ഉത്തമമായ മാര്‍ഗമിതായി രുന്നുവെന്ന് ചൂണ്ടിക്കാണിക്കുവാന്‍ പറ്റിയ ഒരു മാര്‍ഗവും നിര്‍ദേശിക്കുവാന്‍ ആര്‍ക്കും കഴിയില്ലെന്നതാണ് വാസ്തവം. അത് യഥാര്‍ഥത്തില്‍ ഉള്‍ക്കൊള്ളണമെങ്കില്‍ അടിമത്തം ഒരു സ്ഥാപനമായി നിലനിന്നിരുന്ന സമൂഹത്തിന്റെ ഭൂമികയില്‍നിന്നുകൊണ്ട് പ്രശ്‌നത്തെ നോക്കിക്കാണണമെന്നുമാത്രം.

മ്മുടെ സമൂഹത്തില്‍ ഇന്ന് നിലവിലില്ലാത്ത ഒരു സമ്പ്രദായമാണ് അടിമത്തം. ഇന്നത്തെ ചുറ്റുപാടുകളിലിരുന്നുകൊണ്ട് പ്രസ്തുത ഭൂതകാലപ്രതിഭാസത്തെ അപഗ്രഥിക്കുമ്പോള്‍ അതിന്റെ വേരുകളെയും അതു നിലനിന്നിരുന്ന സമൂഹങ്ങളില്‍ അതിനുണ്ടായിരുന്ന സ്വാധീനത്തെയും കുറിച്ച് വ്യക്തമായി മനസ്സിലാക്കേണ്ടതുണ്ട്. വര്‍ത്തമാനത്തിന്റെ ഭൂമികയില്‍നിന്നുകൊണ്ട് ഭൂതകാലപ്രതിഭാസങ്ങളെ അപഗ്രഥിച്ച് കേവല നിഗമനത്തിലെത്തുവാന്‍ കഴിയില്ല. അടിമത്തമെന്നാല്‍ എന്താണെന്നും പുരാതന സമൂഹങ്ങളില്‍ അത് ചെലുത്തിയ സ്വാധീനമെന്തായിരുന്നുവെന്നും മനസ്സിലാക്കുമ്പോഴേ അതിനെ ഖുര്‍ആന്‍ സമീപിച്ച രീതിയുടെ മഹത്വം നമുക്ക് ബോധ്യമാകൂ.

ഒരു വ്യക്തി മറ്റൊരാളുടെ സമ്പൂര്‍ണമായ അധികാരത്തിന് വിധേയമായിത്തീരുന്ന സ്ഥിതിക്കാണ് അടിമത്തമെന്ന് പറയുന്നത്. ശരീരവും ജീവനും കുടുംബവും അങ്ങനെ തനിക്ക് എന്തൊക്കെ സ്വന്തമായുണ്ടോ അതെല്ലാം മറ്റൊരാള്‍ക്ക് അധീനമാക്കപ്പെട്ട രീതിയില്‍ ജീവിതം നയിക്കുന്നവനാണ് അടിമ. അവന്‍ ഉടമയുടെ ജംഗമസ്വത്താണ്. ഉടമ ഒരു വ്യക്തിയോ സമൂഹമോ രാഷ്ട്രമോ ആകാം. ആരായിരുന്നാലും അയാള്‍ക്ക് നല്‍കുന്ന അവകാശങ്ങള്‍ മാത്രം അനുഭവിച്ച് ഉടമക്കുകീഴില്‍ ജീവിക്കാന്‍ വിധിക്കപ്പെട്ടവനാണ് അടിമ.

അടിമസമ്പ്രദായത്തിന്റെ ഉല്‍പത്തി എങ്ങനെയാണെന്നോ, അത് എവിടെ, എന്നാണ് തുടങ്ങിയതെന്നോ ഉറപ്പിച്ച് പറയാന്‍ പറ്റിയ രേഖകളൊന്നും ഉപലബ്ധമല്ല. ക്രിസ്തുവിന് 20 നൂറ്റാണ്ടുകള്‍ക്കുമുമ്പുതന്നെ ഈ സമ്പ്രദായം ആരംഭിച്ചുകഴിഞ്ഞിരുന്നുവെന്ന് ഉറപ്പാണ്. ബി.സി 2050-നോടടുത്ത് നിലനിന്നിരുന്ന ഉര്‍നാമു (Ur Namu) നിയമസംഹിതയില്‍ അടിമകളെക്കുറിച്ച് പരാമര്‍ശിക്കപ്പെട്ടിട്ടുണ്ട്. യുദ്ധത്തില്‍ തടവുകാരായി പിടിക്കപ്പെടുന്നവര്‍ക്ക് സ്വാതന്ത്ര്യം നിഷേധിച്ചുകൊണ്ട് കൂലിയില്ലാതെ ജോലിചെയ്യിക്കുന്ന പതിവില്‍നിന്നാവണം അടിമത്തം നിലവില്‍ വന്നതെന്നാണ് അനുമാനം. പുരാതന സുമേറിയന്‍ ഭാഷയില്‍ അടിമകളെ വിളിക്കാന്‍ ഉപയോഗിച്ചിരുന്ന പദങ്ങളാണ് ഈ അനുമാനത്തിന് നിദാനം. പുരുഷഅടിമക്ക് നിദാ-കൂര്‍ എന്നും സ്ത്രീ അടിമക്ക് മുനുസ്-കൂര്‍ എന്നുമായിരുന്നു പേര്‍. വിദേശിയായ പുരുഷന്‍, വിദേശിയായ സ്ത്രീ എന്നിങ്ങനെയാണ് യഥാക്രമം ഈ പദങ്ങളുടെ അര്‍ഥം. യുദ്ധത്തടവുകാരെ കൊണ്ടുവന്നിരുന്നത് വിദേശത്തുനിന്നായിരുന്നതിനാല്‍ അവരെ അടിമകളാക്കിയപ്പോള്‍ ഈ പേരുകള്‍ വിളിക്കപ്പെട്ടുവെന്നാണ് ഊഹിക്കപ്പെടുന്നത്.

ലോകത്ത് ഏകദേശം എല്ലാ പ്രദേശങ്ങളിലും ഒരു രൂപത്തിലല്ലെങ്കില്‍ മറ്റൊരു രൂപത്തില്‍ അടിമത്തം നിലനിന്നിരുന്നു. പുരാതന ഇസ്രായേല്‍ സമുദായത്തിന്റെ കഥ പറയുന്ന പഴയ നിയമ ബൈബിളില്‍ അടിമത്തത്തെക്കുറിച്ച് ഒരുപാട് പരാമര്‍ശങ്ങളുണ്ട്. പൗരാണിക പ്രവാചകനായിരുന്ന അബ്രഹാമിന്റെ കാലത്തുതന്നെ മനുഷ്യരെ വിലയ്ക്കു വാങ്ങുന്ന സമ്പ്രദായം നിലനിന്നിരുന്നുവെന്ന് കാണാന്‍ കഴിയും (ഉല്‍പത്തി 17:13,14). യുദ്ധത്തടവുകാരെ അടിമകളാക്കണമെന്നാണ് ബൈബിളിന്റെ അനുശാസന (ആവര്‍ത്തനം20:10,11) അടിമയെ യഥേഷ്ടം മര്‍ദിക്കുവാന്‍ യജമാനന് സ്വാതന്ത്ര്യം നല്‍കുന്ന ബൈബിള്‍ പക്ഷേ, പ്രസ്തുത മര്‍ദനങ്ങള്‍ക്കിടയില്‍ അടിമ മരിക്കാനിടയാകരുതെന്ന് പ്രത്യേകം നിഷ്‌കര്‍ഷിക്കുന്നുണ്ട്. ‘ഒരുവന്‍ തന്റെ ആണ്‍അടിമയെയോ പെണ്‍അടിമയെയോ വടികൊണ്ടടിക്കുകയും അയാളുടെ കൈയാല്‍ അടിമ മരിക്കുകയും ചെയ്താല്‍ അയാളെ ശിക്ഷിക്കണം. പക്ഷേ, അടിമ ഒന്നോ രണ്ടോ ദിവസം ജീവിച്ചാല്‍ അയാളെ ശിക്ഷിക്കരുത്. കാരണം അടിമ അയാളുടെ സ്വത്താണ്‘ (പുറപ്പാട് 21:20,21) എന്നതായിരുന്നു ബൈബിള്‍ പറയുനനതനുസരിച്ച് ഇവ്വിഷയകമായി ഇസ്രായേല്‍ സമുദായത്തില്‍ നിലനിന്നിരുന്ന നിയമം. യേശുക്രിസ്തുവിന്റെ കാലത്തും ശേഷവുമെല്ലാം അടിമസമ്പ്രദായം നിലനിന്നിരുന്നു. അടിമകളോടു സ്വീകരിക്കേണ്ട നിലപാടുകളെ സംബന്ധിച്ച ഉപദേശങ്ങളൊന്നും യേശുവിന്റെ വചനങ്ങളിലില്ല.‘കര്‍ത്താവിന്റെ വിളി ലഭിച്ചുകഴിഞ്ഞ അടിമകള്‍ ആത്മാര്‍ഥമായി യജമാനന്മാരെ സേവിക്കണം‘ (എേഫ 6:5-9). ‘അടിമകളേ, നിങ്ങളുടെ ലൗകിക യജമാനന്മാരെ എല്ലാ കാര്യങ്ങളിലും അനുസരിക്കുക;യജനമാനന്മാര്‍ കാണ്‍കെ, അവരെ പ്രീണിപ്പിക്കാന്‍ വേണ്ടി മാത്രമല്ല,ആത്മാര്‍ഥതയോടുകൂടി കര്‍ത്താവിനെ ഭയപ്പെട്ട് യജമാനന്മരെ അനുസരിക്കുക‘ (കൊളോ 3:22) എന്നുമുള്ള പരാമര്‍ശങ്ങള്‍ ഒഴിച്ചാല്‍ അടിമത്തവുമായി ബന്ധപ്പെട്ട മറ്റു പ്രധാന നിര്‍ദേശങ്ങളൊന്നും പൗലോസിന്റെ ലേഖനങ്ങളില്‍ കാണാന്‍ കഴിയുന്നില്ല. ഗ്രീക്കോ-റോമന്‍ നാഗരികതയില്‍ നിലനിന്നിരുന്ന അതിക്രൂരമായ അടിമത്ത സമ്പ്രദായം അവയുടെ ക്രൈസ്തവവത്കരണത്തിനു ശേഷവും മാറ്റമൊന്നുമില്ലാതെ നിലനിന്നിരുന്നുവെന്ന് കാണാനാവും. അടിമവ്യവസ്ഥിതിയുടെ ക്രൂരവും നികൃഷ്ടവുമായ കഥകള്‍ ഏറെ പറയാനുള്ള റോമാ സംസ്‌കാരത്തിന്റെ ഔദ്യോഗിക മതം ക്രൈസ്തവതയായിരുന്നുവെന്ന വസ്തുത പ്രത്യേകം പരാമര്‍ശമര്‍ഹിക്കുന്നു.

ഇന്ത്യയില്‍ അടിമത്തം നിലനിന്നത് മതത്തിന്റെ ഭാഗമായിക്കൊണ്ടാണ്. ൈവദിക മതത്തിന്റെ അവിഭാജ്യ ഘടകമാണ് വര്‍ണാശ്രമ വ്യവസ്ഥ. ശ്രുതികളില്‍ പ്രഥമ ഗണനീയമായി പരിഗണിക്കപ്പെടുന്ന വേദസംഹിതകളില്‍ ഒന്നാമതായി വ്യവഹരിക്കപ്പെടുന്ന ഋഗ്വേദത്തിലെ പുരുഷസൂക്തത്തിലാണ് (10:90:12)ജാതി വ്യവസ്ഥയുടെ ബീജങ്ങള്‍ നമുക്ക് കാണാന്‍ കഴിയുന്നത്.‘പരമപുരുഷന്റെ ശിരസ്സില്‍നിന്ന് ബ്രാഹ്മണനും കൈകളില്‍നിന്ന് വൈശ്യനും പാദങ്ങളില്‍ ശൂദ്രനും സൃഷ്ടിക്കപ്പെട്ടുവെന്ന ഋഗ്വേദ പരാമര്‍ശത്തിന്റെ അടിസ്ഥാനത്തില്‍ നിലവില്‍ വന്ന വര്‍ണാശ്രമ വ്യവസ്ഥ ബ്രാഹ്മണനെ ഉത്തമനും ശൂദ്രനെ അധമനുമായി കണക്കാക്കിയത്‌സ്വാഭാവികമായിരുന്നു. എല്ലാ ഹൈന്ദവ ഗ്രന്ഥങ്ങളും ചാതുര്‍വര്‍ണ്യ വ്യവസ്ഥ ന്യായീകരിക്കുകയാണ് ചെയ്തിട്ടുള്ളത്. ആധുനിക ഹൈന്ദവതയുടെ ശ്രുതിഗ്രന്ഥമായി അറിയപ്പെടുന്ന ഭഗവത് ഗീത ‘ചാതുര്‍വര്‍ണ്യം മയാസൃഷ്ടം ഗുണ കര്‍മ വിഭാഗശഃ‘ (4:13) എന്നാണ് പറഞ്ഞിരിക്കുന്നത്. ‘ഗുണകര്‍മങ്ങളുടെ വിഭാഗത്തിനനുസരിച്ച് നാലു വര്‍ണങ്ങളെ സൃഷ്ടിച്ചിരിക്കുന്നത് ഞാന്‍ തന്നെയാണെ‘ന്നര്‍ഥം.

ദൈവത്തിന്റെ പാദങ്ങളില്‍നിന്ന് പടക്കപ്പെട്ടവര്‍ പാദസേവ ചെയ്യുവാനായി വിധിക്കപ്പെട്ടത് സ്വാഭാവികമായിരുന്നു. ദാസ്യവേലക്കു വേണ്ടി പ്രത്യേകമായി പടക്കപ്പെട്ടവരാണ് ശൂദ്രരെന്നായിരുന്നു വിശ്വാസം. മുജ്ജന്മപാപത്തിന്റെ ശിക്ഷയായാണ് അവര്‍ണനായി ജനിക്കേണ്ടിവന്നതെന്നും അടുത്ത ജന്മത്തിലെങ്കിലും പാപമോചനം ലഭിച്ച് സവര്‍ണനായി ജനിക്കണമെങ്കില്‍ ഈ ജീവിതം മുഴുവന്‍ സവര്‍ണരുടെ പാദസേവ ചെയ്ത് അവരെ സംതൃപ്തരാക്കുകയാണ് വേണ്ടതെന്നുമാണ് അവരെ മതഗ്രന്ഥങ്ങള്‍ പഠിപ്പിച്ചത്. അടിമകളായി ജനിക്കാന്‍ വിധിക്കപ്പെട്ട ചണ്ഡാളന്മാരെ പന്നികളോടും പട്ടികളോടുമൊപ്പമാണ് ഛന്ദോഗ്യോപനിഷത്ത് (5:10:7) പരിഗണിച്ചിരിക്കുന്നത്. അവരോടുള്ള പെരുമാറ്റ രീതിയും ഈ മൃഗങ്ങളോടുള്ളതിനേക്കാള്‍ നീചവും നികൃഷ്ടവുമായിരുന്നുവെന്ന് മനുസ്മൃതിയും പരാശരസ്മൃതിയുമെല്ലാം വായിച്ചാല്‍ മനസ്സിലാകും.

ജന്മത്തിന്റെ പേരില്‍ അടിമത്തം വിധിക്കപ്പെടുന്ന സമ്പ്രദായത്തോടൊപ്പംതന്നെ ഇന്ത്യയില്‍ അടിമ വ്യാപാരവും അതിന്റെ സകലവിധ ക്രൂരഭാവങ്ങളോടുംകൂടി നിലനിന്നിരുന്നതായി കാണാനാവും. തമിഴ്‌നാട്ടില്‍നിന്ന് ലഭിച്ച ശിലാലിഖിതങ്ങളില്‍നിന്ന് ചോള കാലത്തും ശേഷവും ക്ഷേത്രങ്ങളോട് ബന്ധപ്പെട്ടുകൊണ്ട് അടിമ വ്യാപാരം നിലനിന്നിരുന്നുവെന്ന് മനസ്സിലാകുന്നുണ്ട്. മൈസൂരിലും ബീഹാറിലും കേരളത്തിലുമെല്ലാം അടിമ വ്യാപാരം നിലനിന്നിരുന്നു. ഇംഗ്ലീഷ് ഈസ്റ്റ് ഇന്ത്യാ കമ്പനിയുടെ ആഗമനത്തോടെ ഇന്ത്യയില്‍നിന്ന് ആളുകളെ പിടിച്ച് അടിമകളാക്കി വിദേശങ്ങളിലെത്തിക്കുന്ന സമ്പ്രദായവും നിലവില്‍ വന്നു. ട്രാന്‍ക്യൂബാറിലെ ഒരു ഇറ്റാലിയന്‍ പുരോഹിതന്‍, മധുരക്കാരനായ ഒരു ക്രിസ്ത്യാനിയുടെ ഭാര്യയെയും നാലു മക്കളെയും മുപ്പത് ‘പഗോഡ‘ക്ക് മനിലയിലേക്ക് പോകുന്ന ഒരു കപ്പലിലെ കപ്പിത്താന് വിറ്റതായി ചില ചരിത്രരേഖകളിലുണ്ട് (സര്‍വവിജ്ഞാനകോശം. വാല്യം 1, പുറം 258). 1841-ലെ ഒരു സര്‍വേപ്രകാരം  അന്ന് ഇന്ത്യയില്‍ എണ്‍പത് ലക്ഷത്തിനും തൊണ്ണൂറു ലക്ഷത്തിനുമിടയില്‍ അടിമകളുണ്ടായിരുന്നു. മലബാറിലായിരുന്നു ഇന്ത്യയിലെ അടിമകളുടെ നല്ലൊരു ശതമാനമുണ്ടായിരുന്നത്. അവിടത്തെ ആകെ ജനസംഖ്യയില്‍ 15 ശതമാനം അടിമകളായിരുന്നുവത്രേ! (Encyclo paedia Britanica Vol 27, page 289).

റോമാ സാമ്രാജ്യത്തില്‍ ക്രിസ്തുവിന് മുമ്പ് രണ്ടാം നൂറ്റാണ്ടു മുതല്‍ നീണ്ട ആറു ശതാബ്ദക്കാലം നിലനിന്ന അടിമവ്യവസ്ഥയായിരുന്നു ചരിത്രത്തിലെ ഏറ്റവും ക്രൂരമായ അടിമ സമ്പ്രദായമെന്നാണ് മനസ്സിലാവുന്നത്.  ഏതെങ്കിലും രീതിയിലുള്ള യാതൊരു അവകാശവുമില്ലാത്ത വെറും കച്ചവടച്ചരക്കായിരുന്നു റോമാ സാമ്രാജ്യത്തിലെ അടിമ. ഉടമയെ രസിപ്പിക്കുന്നതിനുവേണ്ടി മറ്റൊരു അടിമയുമായി ദ്വന്ദയുദ്ധത്തിലേര്‍പ്പെട്ട് മരിച്ചു വീഴാന്‍ മാത്രം വിധിക്കപ്പെട്ടവനായിരുന്നു അവന്‍. അടിമകളുടെ ശരീരത്തില്‍നിന്ന് ദ്വന്ദയുദ്ധക്കളരിയില്‍ ഉറ്റിവീഴുന്ന രക്തത്തിന്റെ അളവ് വര്‍ധിക്കുമ്പോള്‍ യജമാനന്മാര്‍ ‘ഹുറേ‘ വിളികളുമായി അവരെ പ്രോല്‍സാഹിപ്പിക്കുകയും ചാട്ടവാര്‍ ചുഴറ്റിക്കൊണ്ട് അവരെ ഭീതിപ്പെടുത്തുകയും ചെയ്യുകയായിരുന്നു പതിവ്! അടിമത്തത്തിന്റെ അതിക്രൂരമായ രൂപം!

കൊളംബസിന്റെ അമേരിക്ക കണ്ടുപിടിത്തമാണ് ആധുനിക ലോകത്ത് അടിമത്തത്തെ ആഗോളവ്യാപകമാക്കിത്തീര്‍ത്തത്. നീഗ്രോകള്‍ അടിമകളാക്കപ്പെട്ടു. കമ്പോളങ്ങളില്‍ വെച്ച് കച്ചവടം ചെയ്യപ്പെട്ടു. ഒരു സ്പാനിഷ് ബിഷപ്പായിരുന്ന ബാര്‍തലോച ദെലാസ്‌കാസാസ് ആയിരുന്നു അമേരിക്കന്‍ അടിമത്തൊഴില്‍ വ്യവസ്ഥക്ക് തുടക്കം കുറിച്ചത്. അടിമവ്യാപാരത്തിനായി മാത്രം രജിസ്റ്റര്‍ ചെയ്യപ്പെട്ട കമ്പനികള്‍ യൂറോപ്പിലുണ്ടായിരുന്നു. ആഫ്രിക്കന്‍ തീരപ്രദേശങ്ങളില്‍നിന്ന് അടിമകളെക്കൊണ്ടുവന്ന് അമേരിക്കയില്‍ വില്‍ക്കുകയായിരുന്നു ഈ കമ്പനികളുടെ വ്യാപാരം. പതിനേഴ് മുതല്‍ പത്തൊമ്പത് വരെ നൂറ്റാണ്ടുകള്‍ക്കിടക്ക് അമേരിക്കയില്‍ ഇങ്ങനെ ഒന്നരക്കോടിയോളം അടിമകള്‍ ഇറക്കുമതി ചെയ്യപ്പെട്ടിട്ടുണ്ടെന്നാണ് കണക്ക്. മണിബന്ധത്തിലും കണങ്കാലിലും രണ്ട് അടിമകളെ വീതം കൂട്ടിക്കെട്ടിയായിരുന്നു കപ്പലില്‍ കൊണ്ടുപോയിരുന്നത്. അത്‌ലാന്റിക് സമുദ്രം തരണം ചെയ്യുന്നതിനിടക്ക് നല്ലൊരു ശതമാനം അടിമകള്‍ മരിച്ചുപോകുമായിരുന്നു. ഇങ്ങനെ മരണമടഞ്ഞവരുടെ എണ്ണമെത്രയെന്നതിന് യാതൊരു രേഖകളുമില്ല. അടിമയുടെ ജീവന് എന്തു വില?!

അടിമത്തത്തെക്കുറിച്ച ഖുര്‍ആനിക വീക്ഷണത്തെയും അതിനോടുള്ള സമീപനത്തെയുംകുറിച്ച് ചര്‍ച്ച ചെയ്യുന്നതിന്റെ മുന്നോടിയായാണ് പ്രസ്തുത സമ്പ്രദായത്തിന്റെ ഉല്‍പത്തിയെയും ചരിത്രത്തെയും കുറിച്ച് സംക്ഷിപ്തമായി പ്രതിപാദിച്ചത്. നൂറ്റാണ്ടുകളായി നിലനിന്നിരുന്ന ഒരു വ്യവസ്ഥയെ ഒരു പ്രഖ്യാപനത്തിലൂടെ തുടച്ചു നീക്കുകയെന്ന അപ്രായോഗികവും അശാസ്ത്രീയവുമായ നിലപാടിനുപകരം പ്രായോഗികമായി അടിമത്തം ഇല്ലാതാക്കുന്നതിനുവേണ്ടിയുള്ള ഒരു പദ്ധതിയാണ് ഇസ്‌ലാം ആവിഷ്‌കരിച്ചു നടപ്പിലാക്കിയതെന്നാണ് മനസ്സിലാവുന്നത്്.

ഈ രംഗത്ത് ഇസ്‌ലാം സ്വീകരിച്ച നടപടിക്രമത്തിന്റെ പ്രായോഗികത മനസ്സിലാകണമെങ്കില്‍ അടിമയുടെ മനഃശാസ്ത്രമെന്താണെന്ന് നാം പഠിക്കണം. അടിമയുടെ മാനസിക ഘടനയും സ്വതന്ത്രന്റെ മാനസിക ഘടനയും തമ്മില്‍ വളരെയേറെ വ്യത്യാസങ്ങളുണ്ട്. നിരന്തരമായ അടിമത്ത ജീവിതം അടിമയുടെ മനോനിലയെ രൂപപ്പെടുത്തുന്നതില്‍ കാര്യമായ സ്വാധീനം ചെലുത്തുന്നുണ്ട്. തലമുറകളായി കൈമാറ്റം ചെയ്യപ്പെടുന്ന അടിമത്ത നുകം ചുമലില്‍ വഹിക്കുന്നതുകൊണ്ട് അവന്റെ മനസ്സില്‍ അനുസരണത്തിന്റെയും കീഴ്‌വഴക്കത്തിന്റെയും ശീലങ്ങള്‍ ആഴത്തില്‍ വേരൂന്നിയിട്ടുണ്ടാവും. ചുമതലകള്‍ ഏറ്റെടുക്കുവാനോ ഉത്തരവാദിത്തങ്ങള്‍ വഹിക്കുവാനോ അവന് കഴിയില്ല. ഉടമയുടെ കല്‍പന ശിരസാവഹിക്കാന്‍ അവന്റെ മനസ്സ് സദാ സന്നദ്ധമാണ്. അയാളുടെ ഇച്ഛക്കനുസരിച്ച് കാര്യങ്ങളെല്ലാം നിര്‍വഹിക്കാന്‍ അടിമക്ക് നന്നായറിയാം. എന്നാല്‍, അനുസരിക്കാനും നടപ്പാക്കാനും മാത്രമാണ് അവനു സാധിക്കുക. ഉത്തരവാദിത്തമേറ്റെടുക്കുവാന്‍ അവന്റെ മനസ്സ് അശക്തമായിരിക്കും. ഭാരം താങ്ങുവാന്‍ അവന്റെ മനസ്സിന് കഴിയില്ല. ചുമതലകള്‍ ഏറ്റെടുക്കുന്നതില്‍നിന്ന് ഓടിയകലാനേ അവനു സാധിക്കൂ. എന്നാല്‍ യജമാനന്‍ എന്തു കല്‍പിച്ചാലും അതു ശിരസാവഹിക്കാന്‍ അവന്‍ സദാ സന്നദ്ധനുമായിരിക്കും.

അടിമയുടെയും ഉടമയുടെയും മാനസികാവസ്ഥകള്‍ രണ്ട് വിരുദ്ധധ്രുവങ്ങളില്‍ സ്ഥിതി ചെയ്യുന്നവയാണ്. ഒന്ന് അഹങ്കാരത്തിന്‍േറതാണെങ്കില്‍ മറ്റേത് അധമത്വത്തിന്‍േറതാണ്. വിരുദ്ധ ധ്രുവങ്ങളില്‍ സ്ഥിതി ചെയ്യുന്ന ഈ മാനസികാവസ്ഥകളെ ഒരേ വിതാനത്തിലേക്ക് കൊണ്ടുവരാതെ അടിമമോചനം യഥാര്‍ഥത്തിലുള്ള മോചനത്തിനുതകുകയില്ലെന്നതിന് ഏറ്റവും നല്ല ഉദാഹരണം അമേരിക്കയുടേതുതന്നെയാണ്. എബ്രഹാം ലിങ്കന്റെ പ്രവര്‍ത്തനങ്ങള്‍ വഴി, ഒരൊറ്റ പ്രഖ്യാപനത്തിലൂടെ അമേരിക്കയിലെ അടിമകള്‍ക്ക് മോചനം ലഭിക്കുകയായിരുന്നു, ഒരു പ്രഭാതത്തില്‍! പക്ഷേ, എന്താണവിടെ സംഭവിച്ചത്? നിയമം മൂലം സ്വാതന്ത്ര്യം ലഭിച്ച അടിമകള്‍ക്ക് പക്ഷേ,സ്വാതന്ത്ര്യത്തിന്റെ ‘ഭാരം‘ താങ്ങുവാന്‍ കഴിഞ്ഞില്ല. എന്തു ചെയ്യണമെന്നറിയാതെ അവര്‍ ചുറ്റുപാടും നോക്കി. ആരും കല്‍പിക്കാനില്ലാത്തതുകൊണ്ട് അവര്‍ക്ക് ഒന്നും ചെയ്യുവാനായില്ല. അവര്‍ തിരിച്ച് യജമാനന്മാരുടെ അടുത്തുചെന്ന് തങ്ങളെ അടിമകളായിത്തന്നെ സ്വീകരിക്കണമെന്നപേക്ഷിച്ചു. മാനസികമായി സ്വതന്ത്രരായി കഴിയാത്തവരെ ശാരീരികമായി സ്വതന്ത്രരാക്കുന്നത് വ്യര്‍ഥമാണെന്ന വസ്തുതയാണ് ഇവിടെ അനാവൃതമാവുന്നത്.

മനുഷ്യരുടെ ശരീരത്തെയും മനസ്സിനെയും പറ്റി ശരിക്കറിയാവുന്ന ദൈവത്തില്‍നിന്ന് അവതീര്‍ണമായ ഖുര്‍ആന്‍ ഇക്കാര്യത്തില്‍ തികച്ചും പ്രായോഗികമായ നടപടിക്രമത്തിനാണ് രൂപം നല്‍കിയിട്ടുള്ളത്. അറേബ്യന്‍ സമ്പദ് ഘടനയുടെ സ്തംഭങ്ങളിലൊന്നായിരുന്നു അടിമവ്യവസ്ഥിതി. ഒരു കേവല നിരോധത്തിലൂടെ പിഴുതെറിയുവാന്‍ സാധിക്കുന്നതിലും എത്രയോ ആഴത്തിലുള്ളവയായിരുന്നു അതിന്റെ വേരുകള്‍. ഇസ്‌ലാം പ്രചരിച്ച പ്രദേശങ്ങളിലും അല്ലാത്തയിടങ്ങളിലുമെല്ലാം നിലനിന്നിരുന്ന ഒരു വ്യവസ്ഥയെന്ന നിലയ്ക്ക് അതു നിരോധിക്കുന്നത് പ്രായോഗികമായി പ്രയാസകരമായിരിക്കുമെന്നു മാത്രമല്ല, അത്തരമൊരു നടപടി ഗുണത്തേക്കാളധികം ദോഷമാണ് ചെയ്യുകയെന്നുള്ളതാണ് സത്യം. അതുകൊണ്ടുതന്നെ മനുഷ്യസമൂഹത്തെക്കുറിച്ച് കൃത്യമായി അറിയാവുന്ന അല്ലാഹു അടിമത്തത്തെ പാടെ നിരോധിക്കുന്ന ഒരു നിയമം കൊണ്ടു വരികയല്ല. പ്രത്യുത, അത് ഇല്ലാതാക്കുവാനുള്ള പ്രായോഗികമായ നടപടികള്‍ സ്വീകരിക്കുകയാണ് ചെയ്തത്.

രണ്ടു വിരുദ്ധ തീവ്രമാനസിക നിലകളില്‍ സ്ഥിതി ചെയ്യുന്നവരെ ഒരേ വിതാനത്തിലേക്ക് കൊണ്ടുവരികയാണ് ഇസ്‌ലാം ആദ്യമായി ചെയ്തത്.  അടിമയെയും ഉടമയെയും സംസ്‌കരിക്കുകയെന്ന പദ്ധതിയാണ് ഖുര്‍ആന്‍ മുന്നോട്ടുവെച്ചത്. പിന്നെ, സ്വാതന്ത്ര്യം ദാനമായും അധ്വാനിച്ചും നേടിയെടുക്കുവാനാവശ്യമായ വഴികളെല്ലാം തുറന്നുവെക്കുകയും ചെയ്തു. അടിമയെയും ഉടമയെയും സമാനമായ മാനസിക നിലവാരത്തിലെത്തിച്ചുകൊണ്ട് സ്വാതന്ത്ര്യം നേടുവാനുള്ള വഴികള്‍ തുറക്കുകയും അതു ലഭിച്ചുകഴിഞ്ഞാല്‍ അതു സംരക്ഷിക്കുവാന്‍ അവനെ പ്രാപ്തനാക്കുകയുമാണ് ഖുര്‍ആന്‍ ചെയ്തത്. അതുമാത്രമായിരുന്നു അക്കാര്യത്തില്‍ പ്രായോഗികമായിരുന്നത്.

കൊലയാളിക്ക് കൊലശിക്ഷ നൽകിയാൽ അയാളുടെ കുടുംബം അനാഥമാവില്ലേയെന്നും അവരെക്കുറിച്ച് ഇസ്‌ലാമികനിയമത്തിന്റെ വക്താക്കൾ എന്താണ് ചിന്തിക്കാത്തതെന്നും ചോദിക്കുന്നവരുണ്ട്. കൊലക്കുറ്റത്തിന് ജീവപര്യന്തം ശിക്ഷ വിധിക്കുമ്പോഴും ഇതേ പ്രശ്നങ്ങളുണ്ടെന്ന വസ്തുത അവർ ശ്രദ്ധിക്കാറില്ല. ഇസ്‌ലാമാകട്ടെ ഈ രംഗത്ത് ഏറ്റവും മാനവികമായ സമീപനമാണ് സ്വീകരിച്ചിട്ടുള്ളത്. കൊലക്കുറ്റത്തിന് എല്ലാ സന്ദര്‍ഭത്തിലും ഒരു പോലെ വധശിക്ഷ നല്‍കണമെന്ന് ഖുര്‍ആന്‍ നിര്‍ബന്ധിക്കുന്നില്ല. വധശിക്ഷയെക്കുറിച്ച് പ്രതിപാദിക്കുന്ന ഖുര്‍ആന്‍ സൂക്തം കാണുക: ”വിശ്വസിച്ചവരേ,വധിക്കപ്പെട്ടവരുടെ കാര്യത്തില്‍ തുല്യമായ പ്രതിക്രിയ നിങ്ങള്‍ക്ക് നിയമമാക്കപ്പെട്ടിരിക്കുന്നു. സ്വതന്ത്രന് സ്വതന്ത്രന്‍, അടിമക്ക് അടിമ,സ്ത്രീക്കു സ്ത്രീ. എന്നാല്‍, ഘാതകന് തന്റെ സഹോദരനില്‍നിന്ന് വല്ല ഇളവും ചെയ്തുകിട്ടിയാല്‍ മര്യാദ പ്രകാരം അത് അംഗീകരിക്കപ്പെടുകയും നല്ല നിലയില്‍ പിഴ കൊടുത്തുവിടുകയും വേണ്ടതാകുന്നു. ഇത് നിങ്ങളുടെ രക്ഷിതാവില്‍നിന്നുള്ള ലഘൂകരണവും അനുഗ്രഹവുമത്രെ” (ഖുര്‍ആന്‍ 2:178). ഘാതകനെ മരണത്തില്‍നിന്നു രക്ഷിക്കണമോ വേണ്ടയോയെന്ന് തീരുമാനിക്കുന്നത് കൊല്ലപ്പെട്ടവന്റെ അടുത്ത ബന്ധുക്കളാണ്. അവര്‍ക്ക് വേണമെങ്കില്‍ പ്രതികാരമൂല്യം (ദിയഃ) വാങ്ങി അയാള വെറുതെ വിടാം. അയാളെ വെറുതെ വിടാനാണ് ബന്ധുക്കളുടെ തീരുമാനമെങ്കില്‍ അതിന് എതിര് നില്‍ക്കുവാന്‍ കോടതിക്ക് അവകാശമില്ല. നൂറ് ഒട്ടകമാണ് കൊലക്കുറ്റത്തിനുള്ള പ്രതികാരമൂല്യം. അതുവാങ്ങി ഘാതകനെ വെറുതെ വിട്ടു കഴിഞ്ഞാല്‍ പിന്നെ അയാള്‍ക്കെതിരെ യാതൊരുവിധ പ്രതികാര നടപടിയും പാടില്ല. ചുരുക്കത്തില്‍, കൊല്ലപ്പെട്ടവന്റെ ബന്ധുക്കള്‍ക്ക് സമ്മതമെങ്കില്‍ നഷ്ടപരിഹാരം വാങ്ങി ഘാതകനെ വെറുതെ വിടുകയും പ്രസ്തുത നഷ്ടപരിഹാരമുപയോഗിച്ച് അനാഥമായിത്തീര്‍ന്നവരെ പുനരധിവസിപ്പിക്കുകയും അവരുടെ ജീവസന്ധാരണത്തിനുള്ള മാര്‍ഗമുണ്ടാക്കുകയും ചെയ്യാം. ഘാതകനെ എന്തു ചെയ്യണമെന്ന് തീരുമാനിക്കുവാനുള്ള ആത്യന്തികമായ അധികാരം കൊല്ലപ്പെട്ടവന്റെ ബന്ധുക്കള്‍ക്കു നല്‍കിയ ഖുര്‍ആന്‍, അതിന്റെ ശിക്ഷാനിയമങ്ങളുടെ പ്രോജ്വലമായ മാനവിക മുഖമാണ് ഇവിടെ പ്രകടിപ്പിക്കുന്നത്.

അകാരണമായി കൊല്ലപ്പെടുന്നവന്റെ പ്രയാസങ്ങളോ പ്രസ്തുത കൊല മൂലം അനാഥമാകുന്ന കുടുംബത്തിന്റെ പ്രശ്‌നങ്ങളോ സമൂഹത്തിലുണ്ടാവുന്ന വിടവോ ഒന്നും പരിഗണിക്കാതെ കൊലയാളിയില്‍ കാരുണ്യവര്‍ഷം നടത്തുകയും അവനെ സംസ്‌കരിക്കുവാന്‍ സാധിക്കുമെന്ന മിഥ്യാബോധത്തിന്റെ അടിത്തറയില്‍ സിദ്ധാന്തങ്ങള്‍ മെനയുകയും ചെയ്യുന്നവര്‍ക്ക് ഖുര്‍ആനിലെ നിയമങ്ങള്‍ അപ്രായോഗികവും അപരിഷ്‌കൃതവുമായി തോന്നുക സ്വാഭാവികമാണ്. എന്നാല്‍, അനുഭവങ്ങള്‍ കാണിക്കുന്നത്, ഇവരുടെ ഗവേഷണഫലത്തിന് എതിരായ വസ്തുതകളാണെന്ന സത്യം നാം മനസ്സിലാക്കണം.

കൊലക്കുറ്റത്തിന് ആധുനിക കോടതികള്‍ വിധിക്കുന്നത് പരമാവധി ജീവപര്യന്തം തടവാണ്. ഏതാനും വര്‍ഷങ്ങള്‍ മാത്രം നീണ്ടുനില്‍ക്കുന്ന ജയില്‍വാസമായിട്ടാണ് ജീവപര്യന്തതടവ് മാറാറുള്ളത്. ഇതുതന്നെ ശിക്ഷിക്കപ്പെടാറുള്ളവര്‍ക്കു മാത്രം. പണവും സ്വാധീനവുമുള്ളവര്‍ എത്ര പേരെ കൊന്നാലും സുഖമായി രക്ഷപ്പെടുന്നുവെന്ന വസ്തുതയാണല്ലോ നാം ദിനേന അറിഞ്ഞുകൊണ്ടിരിക്കുന്നത്.

ആരെ കൊന്നാലും ഒന്നുമുണ്ടാകാന്‍ പോകുന്നില്ലെന്ന സ്ഥിതിയുടെ പരിണിത ഫലമെന്താണ്? കൊലപാതകക്കുറ്റങ്ങളുടെ അഭൂതപൂര്‍വമായ വളര്‍ച്ച! കൊലപാതകക്കുറ്റങ്ങള്‍ ചെയ്യാന്‍ യുവാക്കള്‍ കൂടുതല്‍ കൂടുതല്‍ തയാറാകുന്ന അവസ്ഥ!ഇന്ത്യയില്‍ നടക്കു ന്ന പൈശാചിക കൊലപാതകങ്ങളില്‍ തൊണ്ണൂറ്റിമൂന്ന് ശതമാനവും ഈ രംഗത്തെ പുതുമുഖങ്ങളായ യുവാക്കള്‍ ചെയ്യുന്നതാണെന്നാണ് കണക്കുകള്‍ കാണിക്കുന്നത്.   പണത്തിനും സുഖസൗകര്യങ്ങള്‍ക്കും വേണ്ടി ആരെയും കൊല്ലാന്‍ മടിയില്ലാത്ത ഒരു തലമുറ വളര്‍ന്നുകൊണ്ടിരിക്കുന്നു.

ഇരുപത്തിനാലുകാരായ ശ്യാമിന്റെയും രവിയുടെയും കഥ നോക്കുക: ഇരുപത്തിനാല് വയസ്സ് പ്രായമുള്ള രവിയും ശ്യാമും മോട്ടോര്‍ ബൈക്കില്‍ നഗരം (ബാംഗ്ലൂര്‍) ചുറ്റുന്നു. ഒഴിഞ്ഞ ഇരുണ്ട തെരുവുകളില്‍ ഒറ്റയ്ക്ക് ഇരുചക്രവാഹനം ഓടിച്ചുപോകുന്ന സ്ത്രീകളെയാണ് അവര്‍ ലക്ഷ്യം വെക്കുന്നത്. ഒമ്പതു മാസങ്ങള്‍ക്കുള്ളില്‍ ഇരുപത്തിമൂന്ന് പേരെ ഇങ്ങനെ കൊലപ്പെടുത്തിയിട്ടുള്ള അവര്‍ക്ക് പലപ്പോഴും അമ്പത് രൂപയൊക്കെയായിരിക്കും ലഭിക്കുക (Ibid). അമ്പതു രൂപക്കുവേണ്ടി ഒരു ജീവന്‍ നഷ്ടപ്പെടുത്തുവാന്‍ യാതൊരു വൈമനസ്യവുമില്ലാത്ത യുവാക്കള്‍!

പണത്തില്‍ മുങ്ങിക്കുളിച്ച സഞ്ജീവ് നന്ദിയുടെ കഥ മറ്റൊന്നാണ്: അമേരിക്കയിലെ മികച്ച ബിസിനസ് സ്‌കൂളിലൊന്നില്‍ അയച്ച് മാതാപിതാക്കള്‍ അവനെ പഠിപ്പിച്ചു. അറുപത് ലക്ഷം രൂപ വില വരുന്ന ബി.എം.ഡബ്ലിയു ഏഴാം പരമ്പരയില്‍പെട്ട കാര്‍ ഇന്ത്യയില്‍ അവധിക്കാലം ചെലവഴിക്കുമ്പോള്‍ ഉപയോഗിക്കാനായി അവര്‍ അവനു മാത്രമായി നല്‍കി. ഇങ്ങനെ എല്ലാവിധ സുഖസൗകര്യങ്ങളുണ്ടായിട്ടും ആ ജീവിതം തകര്‍ക്കാന്‍ നന്ദി എന്തേ വഴിയൊരുക്കി? എന്തുകൊണ്ടാണ് മദ്യലഹരിയില്‍ തന്റെ ബി.എം.ഡബ്ലിയു. ഇടിച്ച് അഞ്ചുപേരെ കൊന്ന ശേഷം നിര്‍ത്താതെ ഓടിച്ചുപോയത്? പരിക്കേറ്റവരെ സഹായിക്കാനായി ഒരു നിമിഷം നിര്‍ത്തുകപോലും ചെയ്യാതെ കാറുമായി തന്റെ സുഹൃത്തിന്റെ വസതിയിലെത്തി. കാറിലെ തെളിവുകളെല്ലാം എന്തിനാണ് അയാള്‍ കഴുകിക്കളഞ്ഞത്?

അഞ്ചു നിരപരാധികളെ കൊന്ന് കാറുമായി കടന്നുപോകുവാന്‍ യാതൊരു മടിയുമില്ലാത്ത തലമുറ!

സുഖത്തിലേക്കുള്ള തങ്ങളുടെ പ്രയാണത്തില്‍ കാറിന്റെ ചക്രങ്ങള്‍ക്കിടയില്‍ കിടന്ന് ചതഞ്ഞരഞ്ഞവരുടെ നേര്‍ക്ക് ഒന്ന് ദയയോടുകൂടി നോക്കുവാന്‍ പോലും തയാറാവാത്ത യുവാക്കള്‍!

കുറ്റവാളികളെ ജയിലിലടച്ച് സംസ്‌കരിച്ചു കളയാമെന്ന ക്രിമിനോളജിസ്റ്റ് വാദത്തിനെതിരെയുള്ള ജീവിക്കുന്ന തെളിവുകളാണിവ! കൂടുതല്‍ പേരെ കുറ്റവാളികളാക്കുവാന്‍ മാത്രമേ കുറ്റവാളികളോടുള്ള ദാക്ഷിണ്യത്തോടെയുള്ള പെരുമാറ്റം നിമിത്തമാവുകയുള്ളൂ. കുറ്റകൃത്യങ്ങള്‍ ഇല്ലാതാക്കുവാനാവശ്യമായ സാഹചര്യങ്ങള്‍ സൃഷ്ടിക്കുകയും എന്നിട്ട് കുറ്റം ചെയ്യാന്‍ വാസന പ്രകടിപ്പിക്കുന്നവരെ കഠിനമായി ശിക്ഷിക്കുകയും ചെയ്താല്‍ മാത്രമേ സമാധാനപൂര്‍ണമായ സാമൂഹിക ജീവിതം സാധ്യമാകൂ എന്ന വസ്തുത കണക്കിലെടുത്തുകൊണ്ടാണ് കൊലക്കുറ്റത്തിന് കൊലയെന്ന ശിക്ഷ ഖുര്‍ആന്‍ നിര്‍ദേശിക്കുന്നത്.

”സത്യവിശ്വാസികളേ, കൊല ചെയ്യപ്പെടുന്നവരുടെ കാര്യത്തില്‍ തുല്യശിക്ഷ നടപ്പാക്കുകയെന്നത് നിങ്ങള്‍ക്ക് നിയമമാക്കപ്പെട്ടിരിക്കുന്നു. സ്വതന്ത്രനു പകരം സ്വതന്ത്രനും അടിമക്കു പകരം അടിമയും സ്ത്രീക്കു പകരം സ്ത്രീയും (കൊല്ലപ്പെടേണ്ടതാണ്)” (ഖുര്‍ആന്‍ 2:178).

ഗോത്ര വഴക്കുകള്‍ കാരണം പരസ്പരം രക്തം ചിന്തിക്കൊണ്ടിരുന്ന അറേബ്യന്‍ സമൂഹത്തില്‍ നിലവിലുണ്ടായിരുന്ന അതിക്രൂരമായ പ്രതികാര നടപടികളുടെ കടയ്ക്ക് കത്തിവെച്ചുകൊണ്ടാണ് ഈ സൂക്തം അവതരിപ്പിക്കപ്പെട്ടത്. ഒരു ഗോത്രത്തില്‍നിന്ന് ആരെങ്കിലും വധിക്കപ്പെട്ടാല്‍ പകരം കൊന്നവനെ കൊല്ലുകയെന്ന സമ്പ്രദായമായിരുന്നില്ല അവിടെ നിലനിന്നിരുന്നത്. പ്രത്യുത, കൊല്ലപ്പെട്ട വ്യക്തിക്ക് എത്ര വിലമതിച്ചിരുന്നുവോ അതു കണക്കാക്കി അതിനു പകരമായി ഘാതകന്റെ ഗോത്രത്തില്‍നിന്ന് ആളുകളെ കൊന്നൊടുക്കുകയായിരുന്നു അവരുടെ രീതി. ഒരാള്‍ക്ക് പകരം പത്തും നൂറും ആളുകളെ കൊന്നൊടുക്കുവാന്‍ അവര്‍ക്ക് യാതൊരു മടിയുമുണ്ടായിരുന്നില്ല. തിരിച്ചും ഇതുതന്നെയായിരുന്നു അവസ്ഥ. ഒരു ഉന്നതന്‍ മറ്റൊരു ഗോത്രത്തിലെ നിസ്സാരനെ വധിച്ചാല്‍ കൊന്നവനെ കൊല്ലുകയെന്ന നിയമം നടപ്പാക്കാന്‍ അവര്‍ക്ക് വൈമനസ്യമായിരുന്നു.‘ഒരു പാവപ്പെട്ടവനു പകരം ഉന്നതനോ?’ എന്നായിരുന്നു അവരുടെ ചോദ്യം. ഈ സമ്പ്രദായങ്ങള്‍ക്ക് അറുതി വരുത്തിയ ഖുര്‍ആന്‍ പ്രതിക്രിയ നടപ്പാക്കേണ്ടത് പ്രതിയുടെ മേല്‍ മാത്രമാണെന്നു വ്യക്തമാക്കുകയാണ് ഉദ്ധരിക്കപ്പെട്ട സൂക്തത്തില്‍ ചെയ്യുന്നത്.

മനുഷ്യജീവന് ഉന്നതമായ വിലയാണ് ഇസ്‌ലാം കല്‍പിക്കുന്നത്. ഗോത്ര വഴക്കിന്റെയോ വിരോധത്തിന്റെയോ പ്രതികാരത്തിന്റെയോ പേരില്‍ നശിപ്പിക്കപ്പെടാനുള്ളതല്ല ഒരാളുടെ ജീവന്‍. ഖുര്‍ആന്‍ വ്യക്തമാക്കുന്നു: ”മറ്റൊരാളെ കൊന്നതിന് പകരമായോ ഭൂമിയില്‍ കുഴപ്പമുണ്ടാക്കിയതിന്റെ പേരിലോ അല്ലാതെ വല്ലവനും ഒരാളെ കൊലപ്പെടുത്തിയാല്‍, അത് മനുഷ്യരെ മുഴുവന്‍ കൊലപ്പെടുത്തിയതിന് തുല്യമാകുന്നു. ഒരാളുടെ ജീവന്‍ വല്ലവനും രക്ഷിച്ചാല്‍ അത് മനുഷ്യരുടെ മുഴുവന്‍ ജീവന്‍ രക്ഷിച്ചതിന് തുല്യമാകുന്നു” (5:32).

എന്നാല്‍, വധശിക്ഷ ശരിയല്ലെന്ന വാദം അടിസ്ഥാന രഹിതമാണ്. കൊലക്കുറ്റത്തിന് പ്രതികാരം ചെയ്യാന്‍ സന്നദ്ധമല്ലാത്ത ഒരു സമൂഹത്തില്‍ കൊലപാതകങ്ങളുടെ പരമ്പരകളുണ്ടാവും.ആര്‍ക്കും ഭയരഹിതമായി ജീവിക്കുവാന്‍ സാധ്യമല്ലാത്ത അവസ്ഥ സംജാതമാകും. അതുകൊണ്ടുതന്നെ ഖുര്‍ആന്‍ പറഞ്ഞു: ”ബുദ്ധിമാന്മാരെ, (കൊലക്കു കൊലയെന്ന) തുല്യ ശിക്ഷ നല്‍കുന്നതിലാണ് നിങ്ങളുടെ ജീവിതത്തിന്റെ നിലനില്‍പ് (2:179)..

കുറ്റവാളികള്‍ അല്ലാത്തവര്‍ ഒരിക്കലും ശിക്ഷിക്കപ്പെട്ടുകൂടാ എന്നതാണ് ഇസ്‌ലാമിക ശിക്ഷാനിയമങ്ങളുടെ ഒരു അടിസ്ഥാനതത്ത്വം. അതുകൊണ്ടുതന്നെ സംശുദ്ധമായി ജീവിതം നയിക്കുന്നവരെ ആരോപണങ്ങളുന്നയിച്ച് അപകീര്‍ത്തിപ്പെടുത്തുവാന്‍ ഇസ്‌ലാം അനുവദിക്കുന്നില്ല. അത്തരം ആരോപണങ്ങളുന്നയിക്കുന്നവര്‍ നാലു സാക്ഷികളെ ഹാജരാക്കുവാന്‍ സന്നദ്ധരാവണം. അല്ലാത്ത പക്ഷം ആരോപിക്കപ്പെടുന്നവരല്ല, പ്രത്യുത ആരോപിക്കുന്നവരാണ് ശിക്ഷിക്കപ്പെടുക. വ്യഭിചാരാരോപണമുന്നയിക്കുന്നവര്‍ക്കുള്ള ശിക്ഷയെപ്പറ്റി ഖുര്‍ആന്‍ വ്യക്തമാക്കുന്നതിങ്ങനെയാണ്:”ചാരിത്രവതികളുടെ മേല്‍ (വ്യഭിചാരം) ആരോപിക്കുകയും എന്നിട്ട് നാലു സാക്ഷികളെ കൊണ്ടുവരാതിരിക്കുകയും ചെയ്യുന്നവരെ നിങ്ങള്‍ എണ്‍പത് അടി അടിക്കുക. അവരുടെ സാക്ഷ്യം നിങ്ങള്‍ ഒരിക്കലും സ്വീകരിക്കുകയും ചെയ്യരുത്. അവര്‍ തന്നെയാണ് അധര്‍മകാരികള്‍” (ഖുര്‍ആന്‍ 24:4).

പതിവ്രതകളെപ്പറ്റി ആരോപണങ്ങള്‍ പറഞ്ഞുണ്ടാക്കുക ചിലരുടെ ഹോബിയാണ്. അത്തരമാളുകള്‍ സമൂഹത്തില്‍ ഉണ്ടാക്കുന്ന കുഴപ്പങ്ങള്‍ ചില്ലറയൊന്നുമല്ല. എണ്‍പതടി കിട്ടുമെന്ന് വന്നാല്‍ ആരും അത്തരം ദുരാരോപണങ്ങളുമായി നടക്കുകയില്ല. നാലു സാക്ഷികളില്ലാതെ വ്യഭിചാരാരോപണം ഉന്നയിക്കുവാന്‍ ആരും മുതിരുകയില്ല. ആരോപണങ്ങള്‍ പുകഞ്ഞ് നാലാളുടെ മുമ്പില്‍ നടക്കാന്‍ വയ്യാതെയായ എത്രയെത്ര പേര്‍ നമ്മുടെ സമൂഹ ത്തിലുണ്ട്. നമ്മുടെ മീഡിയകള്‍ സര്‍ക്കുലേഷന്‍ വര്‍ധിപ്പിക്കുന്നത് ഇത്തരം ഗോസിപ്പുകള്‍ ഉപയോഗിച്ചുകൊണ്ടാണല്ലോ. ഇത്തരം ദുഷ്പ്രവര്‍ത്തനങ്ങളെല്ലാം ഇസ്‌ലാമിക സമൂഹത്തിന് അന്യമായിരിക്കും. മാന്യന്‍മാരെ അകാരണമായി ആരോപണങ്ങളില്‍ മുക്കിക്കൊല്ലുന്ന അവസ്ഥ ആ സമൂഹത്തില്‍ നിലനില്‍ക്കുകയില്ല. ആരെങ്കിലും അതിന് മുതിര്‍ന്നാല്‍ അവരെ പരസ്യമായി എണ്‍പത് അടി അടിക്കണമെന്നാണ് ഖുര്‍ആനിന്റെ അനുശാസന.

വ്യഭിചാരത്തിന് ഇസ്‌ലാം നിശ്ചയിച്ച ശിക്ഷകള്‍ കഠിനമാണ്. വിവാഹിതരെങ്കില്‍ കല്ലെറിഞ്ഞുകൊല്ലുക! അവിവാഹിതരെങ്കില്‍ പരസ്യമായി നൂറടി! ഇത്തരം ശിക്ഷകള്‍ വിധിച്ച ഇസ്‌ലാം അതോടൊപ്പംതന്നെ നിരപരാധികള്‍ ശിക്ഷിക്കപ്പെടാതിരിക്കുവാന്‍ ആവശ്യമായ നിയമങ്ങള്‍ കൂടി ആവിഷ്‌കരിച്ചിട്ടുണ്ട്. നാലു ദൃക്‌സാക്ഷികള്‍ ഉണ്ടെങ്കില്‍ മാത്രമേ ഒരാള്‍ മറ്റൊരാളുടെ പേരില്‍ വ്യഭിചാരാരോപണമുന്നയിക്കുവാന്‍ പാടുള്ളൂ. അല്ലെങ്കില്‍ ആരോപണം ഉന്നയിച്ചവര്‍ കുടുങ്ങും. അവര്‍ക്ക് എണ്‍പത് അടി വീതം ലഭിക്കും. കള്ള സാക്ഷ്യത്തിനുള്ള സാധ്യത ഇവിടെ തീരെ വിരളമാണ്. ഒരു പാടുപേര്‍ കണ്ടുവെന്ന് ഉറപ്പുണ്ടായാല്‍ മാത്രമേ ഒരാള്‍ ഇത്തരം ആരോപണം ഉന്നയിക്കാന്‍ മുതിരുകയുള്ളൂ. അതുകൊണ്ടുതന്നെ നിരപരാധി ശിക്ഷിക്കപ്പെടുവാന്‍ ഉള്ള സാധ്യത തീരെയില്ലെന്നുതന്നെ പറയാം.

സ്‌ലാമിന്റെ പല നിയമങ്ങളെയും പോലെ ശിക്ഷാനിയമവും പടിപടിയായാണ് അവതരിപ്പിക്കപ്പെട്ടത്. വ്യഭിചാരത്തിന് ആദ്യം വിധിക്കപ്പെട്ടത് വീട്ടുതടങ്കലായിരുന്നു. ”നിങ്ങളുടെ സ്ത്രീകളില്‍നിന്ന് നീചവൃത്തിയിലേര്‍പ്പെടുന്നവരാരോ അവര്‍ക്കെതിരില്‍ സാക്ഷികളായി നിങ്ങളില്‍നിന്ന് നാലു പേരെ നിങ്ങള്‍ കൊണ്ടുവരുവിന്‍. അങ്ങനെ അവര്‍ സാക്ഷ്യം വഹിച്ചാല്‍ അവരെ നിങ്ങള്‍ വീടുകളില്‍ തടഞ്ഞുവെച്ചുകൊണ്ടിരിക്കുക. അവരെ മരണം ഏറ്റെടുക്കുകയോ അല്ലാഹു അവര്‍ക്കൊരു മാര്‍ഗം ഉണ്ടാക്കിത്തരികയോ ചെയ്യുന്നതുവരെ” (ഖുര്‍ആന്‍ 4:15).

ഈ സൂക്തത്തില്‍ ‘അല്ലാഹു അവര്‍ക്കൊരു മാര്‍ഗം നിശ്ചയിക്കുന്നതുവരെ‘യെന്ന് പറഞ്ഞതിനെ അന്വര്‍ഥമാക്കിക്കൊണ്ട് വ്യഭിചാരത്തിനുള്ള ഖണ്ഡിതമായ വിധി പിന്നീട് വന്നു. അതിങ്ങനെയാണ്:”വ്യഭിചരിക്കുന്ന സ്ത്രീ-പുരുഷന്മാരില്‍ ഓരോരുത്തരെയും നിങ്ങള്‍ നൂറ് അടി അടിക്കുക. നിങ്ങള്‍ അല്ലാഹുവിലും അന്ത്യദിനത്തിലും വിശ്വസിക്കുന്നവരാണെങ്കില്‍ അല്ലാഹുവിന്റെ മതനിയമത്തില്‍ (അതു നടപ്പാക്കുന്ന വിഷയത്തില്‍) അവരോടുള്ള ദയയൊന്നും നിങ്ങളെ ബാധിക്കാതിരിക്കട്ടെ. അവരുടെ ശിക്ഷ നടക്കുന്നേടത്ത് സത്യവിശ്വാസികളില്‍നിന്നുള്ള ഒരു സംഘം സന്നിഹിതരാവുകയും ചെയ്യട്ടെ” (ഖുര്‍ആന്‍ 24:2).

ഈ സൂക്തത്തില്‍ നൂറടി വിധിച്ചിരിക്കുന്നത് അവിവാഹിതരായ വ്യഭിചാരികള്‍ക്കാണ്. അവര്‍ വിവാഹിതരാണെങ്കില്‍ എറിഞ്ഞുകൊല്ലണമെന്നാണ് ഇസ്‌ലാമിന്റെ വിധി. പ്രവാചകന്‍ (ﷺ)തന്റെ ഭരണകാലത്ത് ഇത്തരം നാല് കേസുകളില്‍ എറിഞ്ഞുകൊല്ലാന്‍ വിധിച്ചിരുന്നുവെന്ന് കാണാന്‍ കഴിയും. അതില്‍ ഒരെണ്ണത്തിലെ പ്രതികള്‍ ജൂതന്മാരായിരുന്നു. ബാക്കി മൂന്നിലും മുസ്‌ലിംകളും. വിവാഹിതരായ വ്യഭിചാരികളെ എറിഞ്ഞുകൊല്ലുവാനുള കല്‍പന മിക്ക ഹദീസുഗ്രന്ഥങ്ങളും (മുസ്‌ലിം, അബൂദാവൂദ്, ഇബ്‌നുമാജ,ബൈഹഖി, അഹ്മദ്) റിപ്പോര്‍ട്ട് ചെയ്തിട്ടുണ്ട്. അതുകൊണ്ടുതന്നെ വിവാഹിതരായ വ്യഭിചാരികളെ എറിഞ്ഞുകൊല്ലുകയാണ് വേണ്ടതെന്ന കാര്യത്തില്‍ മുസ്‌ലിം പണ്ഡിതന്മാര്‍ക്കിടയില്‍ അഭിപ്രായ വ്യത്യാസങ്ങളൊന്നുമില്ല. പ്രസ്തുത നിയമം ഖുര്‍ആനില്‍ പരാമര്‍ശിക്കപ്പെട്ടിട്ടില്ലെങ്കിലും ഖണ്ഡിതവും സത്യസന്ധവുമായ ഹദീസുകളാല്‍ സ്ഥിരപ്പെട്ടിട്ടുള്ളതുകൊണ്ടാണിത്.

കുറ്റം ഒന്നുതന്നെയാണെങ്കിലും അതു ചെയ്യുന്ന വ്യക്തികളുടെ നിലവാരവും അതു സമൂഹത്തിലുണ്ടാക്കുന്ന പ്രത്യാഘാതവുംകൂടി പരിഗണിച്ചുകൊണ്ടാണ് ഇസ്‌ലാം ശിക്ഷാവിധികള്‍ നിശ്ചയിച്ചിരിക്കുന്നത്. അവിവാഹിതരുടെ വ്യഭിചാരം ഒരു കുറ്റമാണ്. പക്ഷേ അവര്‍ക്ക് തങ്ങളുടെ ലൈംഗിക വികാരം ശമിപ്പിക്കുവാന്‍ വിഹിതമായ മാര്‍ഗങ്ങള്‍ മുന്നിലില്ല;അവരുടെ പ്രവര്‍ത്തനം മൂലം കുടുംബബന്ധങ്ങളൊന്നും തകരുന്നില്ല. എന്നാല്‍ വിവാഹിതരുടെ വ്യഭിചാരമോ? അവര്‍ക്കു മുന്നില്‍ തങ്ങളുടെ വികാരശമനത്തിന് നിയമാനുസൃതം പരിണയിച്ച ഇണകളുണ്ട്. പ്രസ്തുത ലൈംഗികബന്ധത്തിന്റെ പരിണിത ഫലമോ? കുടുംബത്തകര്‍ച്ച! അങ്ങനെ സമൂഹത്തില്‍ മുഴുവന്‍ അരാചകത്വം!! അതുകൊണ്ടുതന്നെ ഇവയ്ക്കുള്ള ശിക്ഷകള്‍ വ്യത്യസ്തമായിരിക്കണം. വിവാഹിതരുടെ വ്യഭിചാരവുമായി താരതമ്യം ചെയ്യുമ്പോള്‍ അവിവാഹിതരുടേത് ചെറിയ കുറ്റമാണ്. വിഹിതമാര്‍ഗമുണ്ടായിട്ടും അവിഹിതമാര്‍ഗങ്ങള്‍ തേടിപ്പോകുന്നവരെ വെച്ചുകൊണ്ടിരുന്നുകൂടാ. അവര്‍ മാതൃകാപരമായി ശിക്ഷിക്കപ്പെടണം. അതു കാണുന്ന ഒരാളും ഇനി അതിന് മുതിരാത്ത രീതിയിലുള്ള ശിക്ഷ. അതുകൊണ്ടാണ് അത്തരം ആളുകളെ മരണംവരെ കല്ലെറിയുക എന്ന നിയമം ഇസ്‌ലാം നിശ്ചയിച്ചിരിക്കുന്നത്.

അവിവാഹിതര്‍ക്കാകട്ടെ അവരുടെ കുറ്റത്തിന്റെ തോത് പ്രകാരം പരസ്യമായി നൂറ് അടി അടിക്കുവാന്‍ ഇസ്‌ലാം കല്‍പിച്ചു. അവര്‍ സമൂഹത്തില്‍ അരാജകത്വം ഉണ്ടാക്കുന്നുവെങ്കിലും കുടുംബത്തിന്റെ തകര്‍ച്ചക്കോ അതു മൂലമുള്ള സാമൂഹിക പ്രശ്‌നങ്ങള്‍ക്കോ അത് നിമിത്തമാകുന്നില്ലല്ലോ?

ശിക്ഷാവിധികൾ കൊണ്ട് മാത്രമായി തിന്മകളെല്ലാം ഇല്ലാതാകുമെന്ന് ഇസ്‌ലാം കരുതുന്നില്ല. ഖുര്‍ആനില്‍ കേവലം ശിക്ഷാവിധികളെക്കുറിച്ചു മാത്രമല്ല പരാമര്‍ശിക്കുന്നത്. ശിക്ഷാവിധികള്‍ അവസാനത്തെ പടിയാണെന്നാണ് ഇസ്‌ലാമിന്റെ വീക്ഷണം. വിവാഹേതര ലൈംഗികബന്ധത്തിലേക്ക് നയിക്കുന്ന സാഹചര്യങ്ങളെയെല്ലാം ഇല്ലായ്മ ചെയ്യണമെന്നാണ് ഖുര്‍ആന്‍ പഠിപ്പിക്കുന്നത്. അതിന് ആവശ്യമായ നിയമങ്ങളും നിര്‍ദേശങ്ങളുമെല്ലാം ഇസ്‌ലാം പ്രദാനം ചെയ്യുന്നുണ്ട്. അവയെ ഇങ്ങനെ സംഗ്രഹിക്കാം.

ഒന്ന്: സ്ത്രീകളും പുരുഷന്മാരും മാന്യമായി വസ്ത്രം ധരിക്കണം. പുരുഷനിലെ ലൈംഗിക ഉത്തേജനത്തിന് കാഴ്ച ഒരു പ്രധാന കാരണമായതുകൊണ്ടുതന്നെ സ്ത്രീകള്‍ അവരുടെ സൗന്ദര്യം പ്രകടിപ്പിക്കുന്ന രീതിയില്‍ വസ്ത്രം ധരിക്കരുത്.

രണ്ട്: ലൈംഗികമായി പ്രലോഭിപ്പിക്കുന്ന യാതൊന്നും സമൂഹത്തില്‍ ഉണ്ടാകരുത്. കാബറെ, നൃത്തങ്ങള്‍, സൗന്ദര്യ മല്‍സരം, ബാലെ തുടങ്ങിയവ ഇസ്‌ലാമിക സമൂഹത്തില്‍ ഉണ്ടാവുകയില്ല.

മൂന്ന്: വ്യഭിചാരത്തിലേക്ക് നയിക്കുന്ന രീതിയുള്ള നിര്‍ബാധമായ സ്ത്രീ-പുരുഷ സമ്പര്‍ക്കം പാടില്ല.

നാല്: ലൈംഗികത ഒരു തൊഴിലായി സ്വീകരിക്കുന്നത് പാടെ വിപാടനം ചെയ്യണം. വേശ്യകളോ കാള്‍ഗേളുകളോ സെക്‌സ് ബോംബുകളോ നഗ്‌നമോഡലുകളോ ഒന്നും ഇസ്‌ലാമിക സമൂഹത്തില്‍ ഉണ്ടാവുകയില്ല.

അഞ്ച്: അന്യ സ്ത്രീ-പുരുഷന്മാര്‍ ഒന്നിച്ച് (ഭര്‍ത്താവോ വിവാഹം നിഷിദ്ധമായ ബന്ധുവോ കൂടെയില്ലാതെ) യാത്ര ചെയ്യരുത്.

ആറ്: അന്യസ്ത്രീ പുരുഷന്മാര്‍ മറ്റൊരാളുടെ സാന്നിധ്യത്തിലല്ലാതെ സ്വകാര്യ സംഭാഷണത്തിലേര്‍പ്പെടരുത്.

ഏഴ്: പുരുഷന്‍ സ്ത്രീയെയോ, സ്ത്രീ പുരുഷനെയോ, അവര്‍ വിവാഹത്തിലൂടെ ഇണകളായി മാറിയിട്ടില്ലെങ്കില്‍, കാമവികാരത്തോടെ നോക്കരുത്.

എട്ട്: കാമവികാരമുണ്ടാക്കുന്ന രീതിയില്‍ സംസാരിക്കുകയോ കൊഞ്ചിക്കുഴയുകയോ ചെയ്യരുത്.

ഒമ്പത്: പുരുഷന്‍ വിവാഹാന്വേഷണവുമായി വന്നാല്‍ അവന്‍ സംസ്‌കാര സമ്പന്നനാണെങ്കില്‍ പെണ്‍കുട്ടിയെ വിവാഹം ചെയ്തുകൊടുക്കാന്‍ രക്ഷിതാക്കള്‍ സന്നദ്ധരാകണം.

പത്ത്: ഒരു സ്ത്രീയെക്കൊണ്ട് വികാരശമനം സാധ്യമല്ലാത്തവര്‍ക്ക് ഒന്നിലധികം പേരെ ചില വ്യവസ്ഥകള്‍ക്ക് വിധേയമായി വിവാഹം ചെയ്യുവാന്‍ അനുവാദമുണ്ട്.

ഖുര്‍ആന്‍ ഒന്നാമതായി, ലൈംഗിക വികാരം ഉത്തേജിപ്പിക്കുകയും കുറ്റകൃത്യങ്ങള്‍ക്ക് പ്രേരിപ്പിക്കുകയും ചെയ്യുന്ന സാഹചര്യങ്ങള്‍ ഇല്ലായ്മ ചെയ്യുന്നു. രണ്ടാമതായി, വിഹിതമായ മാര്‍ഗത്തില്‍ വികാരശമനത്തിനാവശ്യമായ തുറന്ന അംഗീകാരം നല്‍കുന്നു. ഇതിനുശേഷവും വികാരശമനത്തിന് അസാന്മാര്‍ഗിക മാര്‍ഗങ്ങളെ അവലംബിക്കുന്നവര്‍ സമൂഹത്തിന്റെ ധാര്‍മിക നിലവാരത്തെ തകര്‍ക്കുകയും കുടുംബത്തെയും സമൂഹത്തെയുമെല്ലാം നശിപ്പിക്കുകയുമാണ് ചെയ്യുന്നത്. അത്തരം ആളുകളെ കഠിനമായി ശിക്ഷിക്കണമെന്നാണ് ഇസ്‌ലാമിന്റെ നിര്‍ദേശം.

മനുഷ്യരെ അസാന്മാര്‍ഗികളാക്കുന്നതില്‍ സാഹചര്യങ്ങള്‍ക്ക് അനല്‍പമായ പങ്കുണ്ട്. ലൈംഗിക വികാരത്തെ ഉത്തേജിപ്പിക്കുന്ന രീതിയില്‍ മീഡിയയും മാര്‍ക്കറ്റുകളും മാറുകയും വിവാഹേതര ലൈംഗികബന്ധം ഒരു പാപമല്ലെന്ന രീതിയില്‍ സമൂഹം കൈകാര്യം ചെയ്യുവാനാരംഭിക്കുകയും ചെയ്തതു കാരണം സ്ത്രീകള്‍ക്കു നേരെയുള്ള അക്രമങ്ങള്‍ വര്‍ധിച്ചു കൊണ്ടിരിക്കുന്നുവെന്നതാണ് വാസ്തവം. കേരളത്തിലെ അവസ്ഥതന്നെയെടുക്കുക. 1994-ല്‍ കേരളത്തില്‍ രജിസ്റ്റര്‍ചെയ്യപ്പെട്ടിരുന്ന ബലാല്‍സംഗക്കേസുകള്‍ 193-ഉം 1995-ല്‍ 266-ഉം1996-ല്‍ 339-ഉം ആയിരുന്നുവെങ്കില്‍ 1997-ല്‍ അത് 588 ആയി ഉയര്‍ന്നു. രണ്ടുവര്‍ഷത്തിനിടയില്‍ 121.05 ശതമാനം വര്‍ധന! 98 ഒക്‌ടോബര്‍ മാസമായപ്പോഴേക്ക് 461 ബലാല്‍സംഗക്കേസുകള്‍ റിപ്പോര്‍ട്ട് ചെയ്യപ്പെട്ടിട്ടുണ്ട്. (അവലംബം: മാതൃഭൂമി ആഴ്ചപ്പതിപ്പ് 24.1.1999)എന്താണിതിന് കാരണം? വിവാഹേതര ബന്ധത്തോടുള്ള സമൂഹത്തിന്റെ സമീപനത്തില്‍ വന്ന മാറ്റവും മീഡിയകളും മാര്‍ക്കറ്റുകളും സ്ത്രീസൗന്ദര്യത്തെ ഒരു വില്‍പനച്ചരക്കായി ഉപയോഗിക്കാനാരംഭിച്ചതും കുറ്റകൃത്യങ്ങളുടെ വര്‍ധനയില്‍ ചെറുതല്ലാത്ത പങ്കുവഹിച്ചിട്ടുണ്ട്. ഈ സ്ഥിതി തുടര്‍ന്നാല്‍ മാന്യമായി ജീവിക്കണമെന്നാഗ്രഹിക്കുന്ന വനിതകള്‍ക്ക് സ്വൈര്യമായി നടക്കാന്‍ പറ്റാത്ത സ്ഥിതിയാണ് കേരളത്തിലുണ്ടാവുക.

ഇത്തരമൊരവസ്ഥ ഇസ്‌ലാമിക സമൂഹത്തിലുണ്ടാവുകയില്ല. അവിടെ സ്ത്രീകള്‍ക്ക് തങ്ങളുടെ മാനം അപഹരിക്കപ്പെടുമെന്ന ഭീതിയോടെ ജീവിക്കേണ്ട ഗതികേടുണ്ടാവുകയില്ല. പ്രവാചകന്റെ കാലത്ത് വിരലിലെണ്ണാവുന്ന വ്യക്തികളെ മാത്രമേ വ്യഭിചാരത്തിന് ശിക്ഷിച്ചിട്ടുള്ളൂ. ഖലീഫമാരുടെ ഭരണകാലത്തും തഥൈവ. മാധ്യമങ്ങളുടെ കടന്നുകയറ്റവും പാശ്ചാത്യ സംസ്‌കാരത്തിന്റെന സ്വാധീനവുമെല്ലാം ഏറെ ജീര്‍ണതകള്‍ സൃഷ്ടിച്ചിട്ടുണ്ടെങ്കിലും ഇന്നും ഖുര്‍ആനിക ശിക്ഷാവിധികള്‍ സ്വീകരിച്ചിരിക്കുന്ന നാടുകളില്‍ ലൈംഗിക കുറ്റകൃത്യങ്ങള്‍ താരതമ്യേന കുറവാണെന്ന വസ്തുത ഇതിന്റെ പ്രായോഗികത വ്യക്തമാക്കുകയാണ് ചെയ്യുന്നത്.

ലൈംഗികത ഒരു ദൈവിക ദാനമാണ്. ജീവികളില്‍ അതിന്റെ പരമമായ ലക്ഷ്യം പ്രത്യുല്‍പാദനമാണ്. മനുഷ്യരിലാകട്ടെ,പ്രത്യുല്‍പാദനമെന്ന ലക്ഷ്യത്തോടൊപ്പംതന്നെ അവന്റെ മാനസികാരോഗ്യവും കുടുംബത്തിന്റെ കെട്ടുറപ്പും സാമൂഹിക ജീവിതത്തിലെ സമാധാനവുമെല്ലാം ലൈംഗികതയുമായി ബന്ധപ്പെട്ടിരിക്കുന്നു. ദൈവിക മാര്‍ഗദര്‍ശനപ്രകാരമല്ലാതെയുള്ള ലൈംഗികതയുടെ ഉപയോഗം വ്യക്തിയുടെ മാനസികനിലയെയും കുടുംബഭദ്രതയെയും സാമൂഹിക ഘടനയെത്തന്നെയും പ്രതികൂലമായി ബാധിക്കും. അതു മാത്രമല്ല, ലൈംഗിക രോഗങ്ങള്‍ക്കും അതുവഴി സമൂഹത്തിന്റെ നിത്യനാശത്തിനുമായിരിക്കും വിവാഹേതര ലൈംഗികബന്ധങ്ങള്‍ ഇടവരുത്തുക. ഈ വസ്തുത അനുഭവത്തില്‍നിന്ന് പഠിച്ചവരാണല്ലോ ആധുനിക സംസ്‌കാരത്തിന്റെ വക്താക്കളെന്നവകാശപ്പെടുന്നവര്‍.

രണ്ടു വ്യക്തികള്‍ ലൈംഗികമായി ബന്ധപ്പെടണമെങ്കില്‍ വിവാഹം എന്ന കരാറിലൂടെയാകണമെന്നാണ് ഇസ്‌ലാം പഠിപ്പിക്കുന്നത്. അതല്ലാതെയുള്ള ബന്ധങ്ങളെല്ലാം നാശം വിതക്കുന്നവയാണ്. അതു സമൂഹത്തില്‍ നിലനില്‍േക്കണ്ട മൂല്യങ്ങളെയെല്ലാം തകര്‍ക്കും. വൈവാഹിക ജീവിതത്തില്‍ സംശയത്തിന്റെ വിത്തുകള്‍ വിതയ്ക്കും. പ്രസ്തുത സംശയങ്ങള്‍ മനസ്സുകള്‍ തമ്മില്‍ വിടവുകളുണ്ടാക്കും. അതു കുടുംബബന്ധത്തെ ഉലയ്ക്കും. ഭാവി തലമുറയുടെ മാനസികാരോഗ്യത്തെ പോലും അതു ബാധിക്കും.

പാശ്ചാത്യ മൂല്യങ്ങള്‍ സ്വീകരിച്ചുകൊണ്ട് പുരോഗമിച്ചുകൊണ്ടിരിക്കുന്ന കേരളത്തിലെ ഇവ്വിഷയകമായ സ്ഥിതി വിവരക്കണക്കുകള്‍ ധാര്‍മികബോധമുള്ള ആരെയും ഞെട്ടിക്കുന്നതാണ്. തിരുവനന്തപുരത്തെ രാജീവ്ഗാന്ധി സെന്റര്‍ ഫോര്‍ ബയോടെക്‌നോളജിയില്‍ ഒാരോ മാസവും മുന്നൂറ് പേരെങ്കിലും ഡി.എന്‍.എ വിരലടയാള പരിശോധന നടത്തി തങ്ങളുടെ ഭാര്യക്ക് പിറന്ന കുഞ്ഞ് തങ്ങളുടേതുതന്നെയാണോയെന്ന് ഉറപ്പിക്കാന്‍ വേണ്ടി എത്തുന്നുണ്ടത്രേ! (മാതൃഭൂമി ആഴ്ചപ്പതിപ്പ് 31.1.99) ഇതു കാണിക്കുന്നതെന്താണ്? പരസ്പരം വിശ്വാസമില്ലാത്ത ഇണകളുടെ എണ്ണം വര്‍ധിച്ചുവരുന്നുവെന്ന്. എന്താണിതിന് കാരണം? ഉത്തരം‘മാതൃഭൂമി‘തന്നെ പറയുന്നുണ്ട്. ‘സര്‍വേയില്‍ പങ്കെടുത്ത 30 ശതമാനം പുരുഷന്മാരും 18 ശതമാനം സ്ത്രീകളും വിവാഹബാഹ്യബന്ധത്തില്‍ ഏര്‍പ്പെടുന്നവരാണ്‘.

സദാചാര മൂല്യങ്ങള്‍ മുറുകെ പിടിക്കുന്നുവെന്നവകാശപ്പെടുന്ന കേരളീയ സമൂഹത്തിന്റെ സ്ഥിതിയാണിത്. പാശ്ചാത്യ സമൂഹങ്ങളിലെ സ്ഥിതിയാകട്ടെ ഇതിലും കഷ്ടമാണ്. ഗര്‍ഭിണികളാകുന്ന കൊച്ചുകുഞ്ഞുങ്ങളാണ് അവിടത്തെ ഏറ്റവും വലിയ സാമൂഹികപ്രശ്‌നം. ജാര സന്തതികളാണ് ഗവണ്‍മെന്റിനെ അലട്ടുന്ന പ്രധാനപ്പെട്ട മറ്റൊരു പ്രശ്‌നം. ഇതൊന്നും ഒരു വാര്‍ത്തയേ അല്ലെന്ന സ്ഥിതിയാണവിടെ. പക്ഷേ, ഇത്തരം സദാചാരലംഘനങ്ങള്‍ വഴി കുടുംബമെന്ന സ്ഥാപനം അവിടെ തകര്‍ന്നു തരിപ്പണമായിട്ടുണ്ടെന്നും അതു വമ്പിച്ച സാമൂഹിക പ്രശ്‌നങ്ങള്‍ സൃഷ്ടിച്ചുകൊണ്ടിരിക്കുകയാണെന്നും ഇത് പാശ്ചാത്യലോകത്തിന്റെ സമ്പൂര്‍ണ നാശത്തിലാണ് കലാശിക്കുകയെന്നും മുന്നറിയിപ്പ് നല്‍കിക്കൊണ്ടിരിക്കുകയാണ്‌സാമൂഹിക ശാസ്ത്രജ്ഞന്മാര്‍.

ഇസ്‌ലാം വിഭാവനം ചെയ്യുന്ന സമൂഹം ഇത്തരത്തിലുള്ളതല്ല. ശാന്തമായ കുടുംബാന്തരീക്ഷവും സമാധാന പൂര്‍ണമായ ദാമ്പത്യവും നിലനില്‍ക്കുന്ന സമൂഹത്തെ സൃഷ്ടിക്കുവാനാണ് ഇസ്‌ലാം പരിശ്രമിക്കുന്നത്.

അതിന് വിവാഹത്തിന് പുറത്തുള്ള സകല ലൈംഗികബന്ധങ്ങളും നിരോധിക്കപ്പെടണമെന്നാണ് ഇസ്‌ലാം കരുതുന്നത്. അതുകൊണ്ട് അത്തരം ലൈംഗിക ബന്ധങ്ങള്‍ ഇല്ലാതെയാക്കുവാനാവശ്യമായ ശിക്ഷകളാണ് ഖുര്‍ആന്‍ അനുശാസിക്കുന്നത്. ലൈംഗികത അതിശക്തമായ ഒരു വികാരമാണെന്നിരിക്കെ അതില്‍നിന്ന് മനുഷ്യരെ തടഞ്ഞുനിര്‍ത്താന്‍ ശക്തമായ നടപടികള്‍ ആവശ്യമാണ്. ഖുര്‍ആനിലെ ശിക്ഷകള്‍ പ്രസക്തമാകുന്നത് ഇവിടെയാണ്.

ളവിനുള്ള ഖുര്‍ആനിലെ ശിക്ഷാനിയമത്തിന്റെ ലക്ഷ്യം കുറേ അംഗവൈകല്യമുള്ളവരെ സൃഷ്ടിക്കുകയല്ല, പ്രത്യുത കളവുചെയ്യപ്പെടാത്ത അവസ്ഥ സംജാതമാക്കുയാണ്. കവര്‍ച്ച ഇല്ലാതെയാകണമെങ്കില്‍ ആദ്യം പാവപ്പെട്ടവന്റെ പട്ടിണിക്ക് പരിഹാരം കാണണമെന്ന് അറിയാവുന്ന പടച്ചതമ്പുരാനാണ് ഖുര്‍ആന്‍ അവതരിപ്പിച്ചിരിക്കുന്നത്. അതുകൊണ്ടുതന്നെ പട്ടിണിക്കുള്ള പരിഹാര നിര്‍ദേശങ്ങള്‍ നല്‍കിയ ശേഷമാണ് ഖുര്‍ആന്‍ ശിക്ഷാനിയമങ്ങളെക്കുറിച്ച് പരാമര്‍ശിക്കുന്നതുതന്നെ.

ഇസ്‌ലാമിലെ സകാത്ത് വ്യവസ്ഥ പാവപ്പെട്ടവരുടെ പ്രയാസങ്ങള്‍ പരിഹരിക്കുവാന്‍ വേണ്ടി നിശ്ചയിക്കപ്പെട്ടിട്ടുള്ളതാണ്. സമ്പത്തിന്റെ ഒരു നിശ്ചിത വിഹിതം പണക്കാരനില്‍നിന്ന് പിടിച്ചെടുത്ത് അതിന്റെ അവകാശികള്‍ക്ക് വിതരണം ചെയ്യണമെന്നാണ് ഇസ്‌ലാമിന്റെ അനുശാസന. സകാത്ത് പണക്കാരന്റെ ഔദാര്യമല്ല, പ്രത്യുത പാവപ്പെട്ടവന്റെ അവകാശമാണ് എന്നാണ് പ്രവാചകന്‍ (ﷺ)പഠിപ്പിച്ചിരിക്കുന്നത്. ഇസ്‌ലാമിലെ സകാത്ത് സമ്പ്രദായം ഫലപ്രദമായി നടപ്പാക്കിയാല്‍തന്നെ സമൂഹത്തിലെ പാവപ്പെട്ടവരുടെ അടിസ്ഥാനാവശ്യങ്ങള്‍ നിര്‍വഹിക്കപ്പെടുമെന്ന കാര്യത്തില്‍ സംശയമില്ല. ചരിത്രം നല്‍കുന്ന പാഠമതാണ്. സകാത്ത് വ്യവസ്ഥ യഥാക്രമം പ്രയോഗവത്കരിച്ചിരുന്ന സമൂഹങ്ങളില്‍ ദാനധര്‍മങ്ങള്‍ വാങ്ങുവാന്‍ ആരുമില്ലാത്ത അവസ്ഥ സംജാതമായിരുന്നുവെന്നതിന് ഇസ്‌ലാമിക ചരിത്രം നിരവധി ഉദാഹരണങ്ങള്‍ നിരത്തുന്നുണ്ട്. സകാത്ത് വ്യവസ്ഥ നടപ്പാക്കിയിട്ടും പാവപ്പെട്ടവന്റെ പട്ടിണി പരിഹരിക്കുവാനായില്ലെങ്കില്‍ അതിനു മറ്റു മാര്‍ഗങ്ങള്‍ കണ്ടെത്തുവാന്‍ രാഷ്ട്രം ബാധ്യസ്ഥമാണെന്നതാണ് ഇസ്‌ലാമിന്റെ വീക്ഷണം. ‘അയല്‍വാസി പട്ടിണി കിടക്കുമ്പോള്‍ വയറുനിറച്ച് ഉണ്ണുന്നവര്‍ നമ്മില്‍പെട്ടവരല്ല‘ (ത്വബ്‌റാനി, ഹാക്കിം) എന്ന് പഠിപ്പിച്ച പ്രവാചകന്റെ ജീവിതക്രമത്തെ ആധാരമാക്കി നടക്കുന്ന ഇസ്‌ലാമിക രാഷ്ട്രത്തില്‍ പട്ടിണിക്കുള്ള പരിഹാരം കാണുവാന്‍ ഭരണാധികാരികള്‍ക്ക് ബാധ്യതയുണ്ട്.

ഇങ്ങനെ, പട്ടിണി നിര്‍മാര്‍ജനം ചെയ്യാനാവശ്യമായ നിയമങ്ങള്‍ ആവിഷ്‌കരിക്കുകയും അത് നടപ്പാക്കി ലോകത്തിന് മാതൃകയാവുകയും ചെയ്ത മതം ഇസ്‌ലാം മാത്രമാണ്. അങ്ങനെ കുറ്റം ചെയ്യല്‍ അനിവാര്യമാക്കിത്തീര്‍ക്കുന്ന സാഹചര്യങ്ങളെ ഇല്ലായ്മ ചെയ്തതിനുശേഷമാണ് ശിക്ഷാ നിയമങ്ങളെപ്പറ്റി ഖുര്‍ആന്‍ സംസാരിക്കുന്നത്. തൊഴിലില്ലായ്മയും ദാരിദ്ര്യവും നടമാടുന്ന ഒരു സമൂഹത്തിലല്ല ഇസ്‌ലാം ശിക്ഷാനിയമങ്ങള്‍ നടപ്പാക്കാന്‍ ആവശ്യപ്പെടുന്നത്. ആഹാരത്തിനോ അടിസ്ഥാനാവശ്യങ്ങള്‍ക്കോ വേണ്ടി മോഷണമോ കൊള്ളയോ നടത്തേണ്ടതില്ലാത്ത സാഹചര്യം സൃഷ്ടിച്ചതിനു ശേഷവും ജനങ്ങളുടെ സൈ്വര്യജീവിതത്തെ തടസ്സപ്പെടുത്തുന്ന മോഷ്ടാക്കളുണ്ടെങ്കില്‍ അവരുടെ കരം ഛേദിക്കണമെന്നുതന്നെയാണ് ഇസ്‌ലാമിന്റെ അനുശാസന.

ഇന്ന് ഇന്ത്യയില്‍ നടക്കുന്ന കൊള്ളകള്‍തന്നെ നോക്കുക. അവ പട്ടിണി മാറുന്നതിനു വേണ്ടിയുള്ളതാണോ? ഇന്ത്യയില്‍ നടക്കുന്ന കവര്‍ച്ചകളില്‍ തൊണ്ണൂറ്റിഒമ്പത് ശതമാനവും സുഖിക്കാന്‍ വേണ്ടിയുള്ള യുവാക്കളുടെ എളുപ്പവഴിയായിക്കൊണ്ടുള്ളതാണെന്നതത്രേ യാഥാര്‍ഥ്യം. പുതിയ കാറുകളും ആഢംബര ഹോട്ടലുകളിലെ താമസവും കാമുകിമാരുടെ നീണ്ട നിരയും നേടിയെടുക്കുന്നതിനുവേണ്ടി കൊള്ളയും കൊലയും നടത്തുന്നവര്‍. അവരില്‍ കുറ്റം തെളിയിക്കപ്പെടുന്ന കുറച്ചുപേരുടെ കരം ഛേദിക്കാന്‍ സന്നദ്ധമായാല്‍ നടക്കുന്ന കുറ്റകൃത്യങ്ങളുടെ തൊണ്ണൂറു ശതമാനവും ഇല്ലാതെയാകുമെന്നുറപ്പാണ്. അതിനു നാം തയാറാകുമോയെന്നതാണ് പ്രശ്‌നം.

ഇസ്‌ലാമിക രാഷ്ട്രത്തില്‍തന്നെ ചിലപ്പോള്‍ ക്ഷാമവും വറുതിയുമുണ്ടാകാം. അത്തരം അവസരങ്ങളില്‍ പട്ടിണി മാറ്റുന്നതിനുവേണ്ടി ഒരാള്‍ മോഷ്ടിച്ചാല്‍ അയാളുടെ കരം ഛേദിക്കുവാന്‍ ഇസ്‌ലാം കല്‍പിക്കുന്നില്ല. ഖലീഫ ഉമറി(റ)ന്റെ ഭരണകാലത്ത്, രാജ്യത്ത് ക്ഷാമം പടര്‍ന്നുപിടിച്ച സമയത്ത് ഒരു മോഷ്ടാവിനെ പിടികൂടിയപ്പോള്‍ പട്ടിണിമൂലം മോഷണത്തിന് അയാള്‍ നിര്‍ബന്ധിതനായതായിരിക്കാമെന്ന സംശയത്തിന്റെ ആനുകൂല്യം നല്‍കി അയാളെ വെറുതെ വിടുകയുണ്ടായി. കുറ്റവാളികളെ ഇല്ലാതെയാക്കുകയെന്ന ലക്ഷ്യത്തോടെ ശിക്ഷാവിധികള്‍ വിധിക്കുകയും അത് പ്രായോഗികമാണെന്ന് തെളിയിക്കുകയും ചെയ്ത ഇസ്‌ലാമിന്റെ മാനവിക മുഖമാണ് ഇവിടെ നമുക്ക് കാണാന്‍ കഴിയുന്നത്.

കുറ്റവാളികളോട് സഹതാപപൂര്‍ണമായ സമീപനമാണ് വേണ്ടതെന്ന് വാദിക്കുന്നവരൊക്കെ കുറ്റകൃത്യങ്ങള്‍ ഇല്ലാതെയാക്കി സമാധാനപൂര്‍ണമായ സാമൂഹിക ജീവിതം സാധിക്കുന്നതിന് പ്രായോഗികമായി ചെയ്യേണ്ടതെന്താണെന്ന് വിശദീകരിക്കുന്നതില്‍ പരാജയപ്പെടുകയാണ് പതിവ്. കുറ്റവാളികളോട് സഹതാപം കാണിക്കണമെന്ന് പറയുന്നവര്‍ പ്രസ്തുത കുറ്റങ്ങള്‍ വഴി നഷ്ടങ്ങള്‍ സഹിക്കേണ്ടിവരുന്നവരുടെ സങ്കടനിവൃത്തിയെക്കുറിച്ച് ഒന്നും ഉരിയാടാറില്ല. യാതൊരു കുറ്റവും ചെയ്യാതെ ഓര്‍ക്കാപ്പുറത്ത് ജീവന്‍ നഷ്ടപ്പെടുന്ന നിരപരാധികള്‍. കഷ്ടപ്പെട്ട് സമ്പാദിച്ച ധനം കൊള്ളയടിക്കപ്പെട്ട് വഴിയാധാരമാകുന്ന മനുഷ്യര്‍. ഇണയുടെ അപഥസഞ്ചാരത്തില്‍ തകര്‍ന്നു തരിപ്പണമാകുന്ന കുടുംബബന്ധങ്ങള്‍. ആരും നോക്കാനില്ലാതെ തെരുവ് തെണ്ടുന്ന ജാരസന്തതികള്‍. കുടുംബനാഥന്റെ മദ്യപാനം വഴി തകരുന്ന കുടുംബങ്ങള്‍. ഈ സങ്കടങ്ങളോടാണോ അതല്ല ഇവക്കു ഉത്തരവാദികളായ ക്രൂരരും നിഷ്ഠൂരരും ഭോഗാലസരുമായ കുറ്റവാളികളോടാണോ സഹതാപപൂര്‍ണമായ സമീപനമുണ്ടാകേണ്ടത്? രണ്ടും കൂടി ഒരേസമയത്ത് അസാധ്യമാണ്. കുറ്റവാളിയോടല്ല, പ്രയാസമനുഭവിച്ചവനോടാണ് സഹാനുഭൂതി വേണ്ടതെന്നാണ് ഇസ്‌ലാമിന്റെ വീക്ഷണം. പ്രസ്തുത വീക്ഷണമാണ് മാനവികമെന്നും അതിന്റെ അടിസ്ഥാനത്തിലുള്ള നിയമങ്ങള്‍ക്ക് മാത്രമേ മനുഷ്യരെ കുറ്റകൃത്യങ്ങളില്‍നിന്ന് മോചിപ്പിക്കാന്‍ കഴിയുകയുള്ളൂവെന്നും ഇസ്‌ലാം കരുതുന്നു. അതു തന്നെയാണ് ശരിയെന്ന വസ്തുതയാണല്ലോ ഇപ്പോള്‍ വ്യക്തമായിക്കൊണ്ടിരിക്കുന്നത്.
കാരാഗൃഹത്തില്‍ അടക്കുന്നതുകൊണ്ടുമാത്രം കുറ്റകൃത്യങ്ങളില്‍നിന്ന് സമൂഹം മുക്തമാവുകയില്ലെന്ന സത്യം ഇന്ന് വ്യത്യസ്ത രാജ്യങ്ങളില്‍ വര്‍ധിച്ചുകൊണ്ടിരിക്കുന്ന കുറ്റകൃത്യങ്ങള്‍ വ്യക്തമാക്കുന്നുണ്ട്. പണമുണ്ടാക്കുകയും സുഖിക്കുകയുമാണ് ജീവിതത്തിന്റെ ആത്യന്തിക ലക്ഷ്യമെന്ന് പഠിപ്പിക്കപ്പെടുന്ന യുവതലമുറയെ സംബന്ധിച്ചിടത്തോളം പണമുണ്ടാക്കുവാനുള്ള കുറുക്കുവഴികളാണ് കുറ്റ്യകൃത്യങ്ങള്‍. എല്ലാ ആധുനിക സമൂഹങ്ങളിലും കുറ്റകൃത്യങ്ങള്‍ വര്‍ധിച്ചുവരികയാണെന്നാണ് സ്ഥിതി വിവരക്കണക്കുകള്‍ കാണിക്കുന്നത്. ഇന്ത്യയിലെ സ്ഥിതിതന്നെയെടുക്കുക: കഴിഞ്ഞ ഒരു ദശാബ്ദത്തിനിടക്ക് കുറ്റകൃത്യങ്ങളുടെ നിരക്കില്‍ വമ്പിച്ച വര്‍ധനയാണുണ്ടായിട്ടുള്ളത്. സുഖം നേടാന്‍ വേണ്ടി മല്‍സരിച്ചുകൊണ്ടിരിക്കുന്ന യുവാക്കള്‍ക്കിടയിലെ കുറ്റവാസന ഭീതിദായകമായ രീതിയില്‍ വര്‍ധിച്ചിട്ടുണ്ടെന്ന് ‘ഇന്ത്യാ ടുഡേ‘ തയാറാക്കിയ ഒരു റിപ്പോര്‍ട്ടില്‍ വ്യക്തമാക്കുന്നത് കാണുക: ‘ടാറ്റാ ഇന്‍സ്റ്റിറ്റ്യൂട്ട് ഓഫ് സോഷ്യല്‍ സയന്‍സിലെ ക്രിമിനോളജി വിഭാഗം കഴിഞ്ഞ ഒരു ദശകത്തില്‍ നടന്ന കുറ്റകൃത്യങ്ങളുടെ രീതിയെക്കുറിച്ച് നടത്തിയ ആഴത്തിലുള്ള പഠനം പൂര്‍ത്തിയായിവരികയാണ്. യുവാക്കള്‍ക്കിടയിലുള്ള കുറ്റകൃത്യങ്ങള്‍ 40ശതമാനം കണ്ട് വര്‍ധിച്ചിരിക്കുന്നു എന്നാണ് ആ പഠനം നല്‍കുന്ന ഭയാനകമായ വിവരം. ഇത്തരം കുറ്റകൃത്യങ്ങള്‍ എല്ലാ വിഭാഗങ്ങളിലും പെട്ട യുവാക്കളും ചെയ്യുന്നുണ്ടെങ്കിലും മധ്യവര്‍ഗ, ഉപരി-മധ്യവര്‍ഗ കുടുംങ്ങളിലെ യുവാക്കള്‍ക്കിടയില്‍ ഇത്തരം ക്രൂരതകള്‍ വര്‍ധിച്ചുവരുന്നുവെന്ന വസ്തുതയാണ് ശ്രദ്ധേയമായിരിക്കുന്നത്. രാജ്യത്ത് നടക്കുന്ന കുറ്റകൃത്യങ്ങളില്‍ 56 ശതമാനത്തിലും ഉത്തരവാദികളായ യുവാക്കള്‍-16-നും 25-നും ഇടയില്‍ പ്രായമുള്ളവരാണെന്ന് നാഷനല്‍ ക്രൈം റിക്കാര്‍ഡ്‌സ് ബ്യൂറോ അതിന്റെ പുതിയ റിപ്പോര്‍ട്ടില്‍ പറഞ്ഞിരിക്കുന്നു. മുംബൈയില്‍ കഴിഞ്ഞ 11 മാസങ്ങളില്‍ 551 ഭീഷണിപ്പെടുത്തി പണം തട്ടിയെടുക്കല്‍ സംഭവങ്ങള്‍ ഉണ്ടായതില്‍ 80 ശതമാനവും ആദ്യമായി ഇത്തരം പ്രവര്‍ത്തിക്കിറങ്ങുന്ന യുവാക്കള്‍ ഉള്‍പ്പെട്ടതാണ്. അവരില്‍ 50ശതമാനവും 20 വയസ്സില്‍ താഴെയുള്ളവരാണ്. ബാംഗ്ലൂരില്‍ കവര്‍ച്ചയും കൊള്ളയും വര്‍ധിച്ചുവരികയാണെന്നും അതില്‍ 60 ശതമാനവും യുവാക്കള്‍ ഉള്‍പ്പെട്ടതാണെന്നും റിപ്പോര്‍ട്ടു ചെയ്യുന്നു. ദല്‍ഹിയില്‍ കഴിഞ്ഞ വര്‍ഷം നടന്ന ക്രൂരമായ കുറ്റകൃത്യങ്ങളില്‍ 93 ശതമാനവും ആദ്യമായി അതിന് ഇറങ്ങിയിരിക്കുന്ന യുവാക്കള്‍ ചെയ്തവയാണ്‘ (ഇന്ത്യാ ടുഡേ, 2-1-1999). താനിഷ്ടപ്പെടുന്ന ഒരു പെണ്‍കുട്ടിയുമായി കറങ്ങി നടന്ന സഹപാഠിയെയും കൂട്ടുകാരനെയും കൊന്ന കല്‍ക്കത്തയിലെ പതിനൊന്നാം ക്ലാസുകാരന്‍; പണമുണ്ടാക്കാനായി ചുരുങ്ങിയത് 23പേരെയെങ്കിലും തലയ്ക്കടിച്ച് കൊന്ന ശ്യാമും രവിയും (രണ്ടു പേര്‍ക്കും 24 വയസ്സ്); സ്‌നേഹിതന്റെ മാതാവിനെ കൊന്ന് പണം കൊള്ളയടിച്ച എഞ്ചിനിയറിംഗ് കോളേജ് വിദ്യാര്‍ഥി; കൂട്ടുകാരന്റെ വീടുകൊള്ളയടിക്കാനായി അവന്റെ അമ്മയെയും സഹോദരിയെയും കൊന്ന 21-കാരന്‍; നാലു പെണ്‍കുട്ടികളെ തട്ടിക്കൊണ്ടുപോയി ബലാല്‍സംഗം ചെയ്യുകയും ഒന്നിലധികം പേരെ കൊലപ്പെടുത്തുകയും ചെയ്ത കുബേര പുത്രന്‍ (26 വയസ്സ്); 98 കൊലപാതകക്കേസുകളില്‍ പ്രതിയായ 25-കാരന്‍. ഇങ്ങനെ ഇന്ത്യാ ടുഡേയില്‍ വിവരിക്കുന്ന കുറ്റവാളികളുടെ നിര വളരെ നീണ്ടതാണ്. എന്തുകൊണ്ട് ഈ കുറ്റകൃത്യങ്ങളുണ്ടാകുന്നു? ഒന്നാമതായി,പ്രപഞ്ചനാഥനിലും മരണാനന്തര ജീവിതത്തിലുമുള്ള വിശ്വാസം നഷ്ടപ്പെട്ടത്. രണ്ടാമതായി, പണമുണ്ടാക്കുകയും പരമാവധി സുഖിക്കുകയുമാണ് മനുഷ്യജീവിതത്തിന്റെ ലക്ഷ്യമെന്ന് പഠിപ്പിക്കപ്പെട്ടത്. മൂന്നാമതായി, എന്തു കുറ്റംചെയ്താലും തങ്ങള്‍ പിടിക്കപ്പെടുകയില്ലെന്നും അഥവാ പിടിക്കപ്പെട്ടാല്‍തന്നെ സ്വാധീനങ്ങളുപയോഗിച്ചുകൊണ്ട് ശിക്ഷയില്‍നിന്ന് രക്ഷപ്പെടാമെന്നും ഇനി ശിക്ഷിക്കപ്പെട്ടാല്‍തന്നെ ഏതാനും മാസത്തെ ജയില്‍വാസത്തിനുശേഷം സുഖമായി ജീവിക്കാമെന്നുള്ള വിചാരം. മുതലാളിത്തമൂല്യങ്ങള്‍ നിലനില്‍ക്കുന്ന ഒരു സമൂഹത്തില്‍ ഈ വിചാരം നിലനിലനില്‍ക്കുന്നത് കുറ്റകൃത്യങ്ങളുടെ ഭീതിദമായ വളര്‍ച്ചക്ക് നിമിത്തമാകും. ആധുനികമെന്ന് സ്വയം വിശേഷിപ്പിക്കുന്ന രാഷ്ട്രങ്ങളിലെല്ലാം ഈ പ്രശ്‌നം സാമൂഹിക ശാസ്ത്രജ്ഞന്മാരെ അലട്ടിക്കൊണ്ടിരിക്കുന്നുവെന്നതാണ് സത്യം. എന്താണൊരു പരിഹാരം? കുറ്റകൃത്യങ്ങള്‍ക്ക് കഠിനമായ ശിക്ഷ നല്‍കുക. പ്രസ്തുത ശിക്ഷ പരസ്യമായി നടപ്പാക്കുക. കൊള്ള നടത്തിയാല്‍ കരം ഛേദിക്കെപ്പടുമെന്നും കൊല ചെയ്താല്‍ തന്റെ ജീവന്‍ നഷ്ടപ്പെടുമെന്നുമെല്ലാമുള്ള സ്ഥിതിയുണ്ടായാല്‍ കുറ്റകൃത്യങ്ങള്‍ കുറയുമെന്നുറപ്പാണ്. ഇതിന് ജീവിക്കുന്ന ഉദാഹരണമാണല്ലോ ഇസ്‌ലാമിക ശിക്ഷാവിധികള്‍ നടപ്പാക്കുന്ന രാഷ്ട്രങ്ങള്‍. പരേതനായ അബ്ദുല്‍ അസീസ് രാജാവിന്റെ കാലത്ത് നീണ്ട 25വര്‍ഷക്കാലത്തിനുള്ളില്‍ 16 കരഛേദങ്ങള്‍ മാത്രമേ സഊദി അറേബ്യയില്‍ വേണ്ടിവന്നിട്ടുള്ളൂ. അഥവാ 16 മോഷണങ്ങളേ 25വര്‍ഷത്തിനിടക്ക് നടന്നുള്ളൂവെന്നര്‍ഥം. മോഷ്ടിച്ചവന് കൈ നഷ്ടപ്പെടുമെന്ന അവസ്ഥയുണ്ടാവുമ്പോള്‍, മോഷ്ടിച്ചതു വഴി കൈ നഷ്ടപ്പെട്ടവര്‍ ജീവിക്കുന്ന സമൂഹത്തില്‍ പിന്നെ ആ കുറ്റകൃത്യം ചെയ്യുവാന്‍ പെട്ടെന്നൊന്നും ആരും മുതിരുകയില്ലെന്നുറപ്പാണ്. എന്തെന്തു പ്രലോഭനങ്ങളുണ്ടായാലും കഠിനമായ ശിക്ഷ ഭയപ്പെട്ട് കുറ്റത്തില്‍നിന്ന് ഒഴിഞ്ഞുനില്‍ക്കുന്നവരായിരിക്കും ഭൂരിപക്ഷം. ഈ വസ്തുത ഭൗതികവാദികള്‍ പോലും സമ്മതിക്കുന്നതാണ്. ഇ.എസ്. ഗംഗാധരന്‍ എഴുതി: ‘കൊള്ള, കൊല, കളവ്, വഞ്ചന, വ്യഭിചാരം, അടിപിടികള്‍ എന്നിവക്കെതിരായ കഠിന ശിക്ഷ നല്‍കുന്ന ഇസ്‌ലാമിക നിയമങ്ങള്‍ കര്‍ശനമായി നടപ്പാക്കുന്നതില്‍ ദയാദാക്ഷിണ്യങ്ങളില്ല. അറബ് നാടുകളില്‍ അതിനാല്‍ കലഹകാരണങ്ങളും ദുര്‍നടപടികളും കുറവാണ്‘ (ദേശാഭിമാനി വാരിക 11. 3.1979). എന്നാല്‍ ജയില്‍വാസത്തിന്റെ സ്ഥിതിയോ? അത് മറ്റുള്ളവരില്‍ യാ തൊരുവിധ സ്വാധീനവുമുണ്ടാക്കുന്നില്ല. കുറ്റവാളിയില്‍ വല്ല മാറ്റവുമുണ്ടാക്കുന്നുവോ? അതും ഇല്ലെന്നതാണല്ലോ സത്യം. ജയില്‍ ശിക്ഷയനുഭവിച്ച് പുറത്തുവരുന്നവര്‍ പലപ്പോഴും പ്രൊഫഷനല്‍ കുറ്റവാളികളായി മാറുന്നതാണല്ലോ നാം കാണുന്നത്. കാരാഗൃഹവാസം കഴിഞ്ഞ് പുറത്തുവരുന്ന കുറ്റവാളികളില്‍ പലരും തങ്ങളുടെ പാപപങ്കിലമായ ജീവിതം പൂര്‍വാധികം വാശിയോടെയും നിര്‍ഭയമായും തുടര്‍ന്നത് കാണിക്കുന്നത് എന്താണ്? ശിക്ഷാ നിയമത്തിന്റെ ധര്‍മം കാരാഗൃഹവാസമെന്ന ശിക്ഷ നിര്‍വഹിക്കുന്നില്ലെന്നുതന്നെ.
പല മതഗ്രന്ഥങ്ങളും കുറ്റങ്ങള്‍ക്കുള്ള ശിക്ഷകളെക്കുറിച്ച് വിശദീകരിക്കുന്നുണ്ട്. അവയില്‍ പലതും മനുഷ്യരുടെ കൈകടത്തലുകള്‍ക്ക് വിധേയമായിരിക്കുന്നു എന്നതുകൊണ്ടുതന്നെ മനുഷ്യത്വ വിരുദ്ധമായ പലതും അവയില്‍ കാണാന്‍ കഴിയും. ഖുര്‍ആനിന്റെ സ്ഥിതി ഇതില്‍നിന്ന് വ്യത്യസ്തമാണ്. അതിലെ നിയമങ്ങള്‍ മുഴുവന്‍ ദൈവികമായതുകൊണ്ടുതന്നെ മാനവികമാണ്;സാര്‍വജനീനവും സര്‍വകാല പ്രസക്തവുമാണ്. ഉദാഹരണത്തിന് വ്യഭിചാരത്തിന് വ്യത്യസ്ത മതഗ്രന്ഥങ്ങള്‍ വിധിക്കുന്ന ശിക്ഷയെന്താണെന്ന് നോക്കുക. ‘ഒരുത്തന്റെ ഭാര്യയുമായി വ്യഭിചാരം ചെയ്യുന്നവന്‍,കൂട്ടുകാരന്റെ ഭാര്യയുമായി വ്യഭിചാരം ചെയ്യുന്ന വ്യഭിചാരിയും വ്യഭിചാരിണിയും തന്നെ മരണശിക്ഷയനുഭവിക്കണം (ലേവ്യ 20:10). ‘ഒരു പുരുഷന്റെ ഭാര്യയായ സ്ത്രീയോടുകൂടെ ഒരുത്തന്‍ ശയിക്കുന്നതുകണ്ടാല്‍ സ്ത്രീയോടുകൂടെ ശയിച്ച പുരുഷനും സ്ത്രീയും ഇരുവരും മരണശിക്ഷയനുഭവിക്കണം. ഇങ്ങനെ ഇസ്രായീലില്‍നിന്ന് ദോഷം നീക്കിക്കളയേണം‘ (ആവ.:22:22) ഇവിടെ ബൈബിള്‍ പഴയനിയമത്തില്‍ മരണശിക്ഷവിധിച്ചിരിക്കുന്നത് വിവാഹിതയായ സ്ത്രീയുമായി ലൈംഗികബന്ധം പുലര്‍ത്തുന്നതിനു മാത്രമാണ്. കന്യകയുമായി വ്യഭിചരിച്ചാല്‍ അതിന് ശിക്ഷയൊന്നും ബൈബിള്‍ വിധിക്കുന്നില്ല. അതു കണ്ടുപിടിക്കപ്പെട്ടാല്‍ അവളെ വിവാഹം ചെയ്യണമെന്നതു മാത്രമാണ് ശിക്ഷ. ‘വിവാഹനിശ്ചയം കഴിയാത്ത കന്യകയായ ഒരു യുവതിയെ ഒരുത്തന്‍ കണ്ടു അവളെ പിടിച്ച് അവളോടുകൂടി ശയിക്കുകയും അവരെ കണ്ടുപിടിക്കുകയും ചെയ്താല്‍ അവളോടുകൂടി ശയിച്ച പുരുഷന്‍ യുവതിയുടെ അപ്പന് അമ്പത് വെള്ളിക്കാശ് കൊടുക്കണം. അവള്‍ അവന്റെ ഭാര്യയാവുകയും വേണം‘ (ആവ: 22:28,29) വിവാഹിതയായ സ്ത്രീയുമായുള്ള ലൈംഗികബന്ധത്തിന് മരണശിക്ഷ വിധിക്കുവാനുള്ള കാരണമെന്താണ്? (അതേസമയം പുരുഷന്‍ വിവാഹിതനാണോ അല്ലയോ എന്നത് ഒരു പ്രശ്‌നമായിത്തന്നെ ബൈബിള്‍ കാണുന്നുമില്ല). സ്ത്രീ വിവാഹം ചെയ്യപ്പെടുന്നതുവരെ പിതാവിന്റെയും വിവാഹം ചെയ്യപ്പെട്ടുകഴിഞ്ഞാല്‍ ഭര്‍ത്താവിന്റെയും സ്വത്താണെന്നാണ് ബൈബിള്‍ പഠിപ്പിക്കുന്നത്. അതുകൊണ്ടാണ് സ്ത്രീകളെ വില്‍ക്കാന്‍ അത് പുരുഷന്മാരെ അനുവദിക്കുന്നത് (പുറ.21:7, നെഹമ്യ 5:5 നോക്കുക). ഒരു പുരുഷന്റെ സ്വത്തായ സ്ത്രീയെ അനധികൃതമായി ഉപയോഗിച്ചുവെന്നതാണ് അയാളുടെ ഭാര്യയെ വ്യഭിചരിക്കുന്ന വ്യക്തിചെയ്യുന്ന കുറ്റം. അത് ചെയ്യുന്ന ആള്‍ വിവാഹിതനായാലും അല്ലെങ്കിലും കുറ്റം ഒന്നുതന്നെയാണ്. പുരുഷന്‍ സ്ത്രീയുടെ സ്വത്തല്ലാത്തതിനാല്‍ അയാള്‍ വ്യഭിചരിക്കുന്നത് ഒരു തെറ്റായിത്തന്നെ ബൈബിള്‍ കാണുന്നുമില്ല. ഈ വസ്തുത ‘യഹൂദ വിജ്ഞാനകോശം‘ തന്നെ സമ്മതിക്കുന്നതാണ്. (Encyclopedia Judaica Vol II col 313-) ചുരുക്കത്തില്‍ ബൈബിള്‍ വ്യഭിചാരമെന്ന തിന്മയെ കാണുന്നത് മറ്റൊരാളുടെ സ്വത്തിലുള്ള അനധികൃതമായ കൈയ്യേറ്റമായിക്കൊണ്ടാണ്. പ്രസ്തുത കൈയ്യേറ്റത്തിന് മരണശിക്ഷതന്നെ വിധിക്കുന്നുണ്ടെന്നത് ശരിയാണ്. എന്നാല്‍ വ്യഭിചാരം സൃഷ്ടിക്കുന്ന സാമൂഹിക പ്രശ്‌നങ്ങളോ കുടുംബ ശൈഥില്യമോ ധാര്‍മിക പ്രതിസന്ധികളോ ഒന്നുംതന്നെ ബൈബിളിന്റെ പരിഗണനയില്‍ വരുന്നില്ല. കൊലപാതകത്തിനുള്ള ആപസ്തംബ ധര്‍മ സൂത്രത്തിലെ ശിക്ഷാ നിയമങ്ങള്‍ കാണുക: ‘ബ്രാഹ്മണനെക്കൊല്ലുന്ന ശൂദ്രനെ മൂന്നു പ്രാവശ്യമായി തീയിലിട്ട് കുറച്ചു കുറച്ചായി ചിത്രവധം ചെയ്ത് കൊല്ലണം. എന്നാല്‍ ശൂദ്രനെ മറ്റുള്ളവര്‍ കൊന്നാല്‍ ഒരു വര്‍ഷത്തെ തടവ് വിധിക്കുകയും പന്ത്രണ്ട് പശുക്കളെ പിഴയായി ഈടാക്കുകയും ചെയ്താല്‍ മതി‘ (കൃഷ്ണാനന്ദ സ്വാമി ഉദ്ധരിച്ചത്: ഇന്ത്യയിലെ വര്‍ണസമരം പുറം 94) ഹൈന്ദവസ്മൃതികളിലെ നിയമങ്ങളെല്ലാം വര്‍ണാശ്രമ വ്യവസ്ഥയുടെ അടിസ്ഥാനത്തിലുള്ളവയാണ്. ബ്രാഹ്മണനെ പൂജ്യനായും ശൂദ്രനെ അധമനായും കണ്ടുകൊണ്ടുള്ള നിയമങ്ങളില്‍ ഉടനീളം ഈ ഉച്ചനീചത്വം പ്രകടമാണ്. ഒരേ തെറ്റ് ബ്രാഹ്മണന്‍ ചെയ്താലുള്ള ശിക്ഷയും ശൂദ്രന്‍ ചെയ്താലുള്ള ശിക്ഷയും തമ്മില്‍ വലിയ അന്തരമുണ്ടായിരിക്കും. ഈ നിയമ ങ്ങള്‍ മനുഷ്യര്‍ക്കു വേണ്ടിയുള്ളവയല്ല;ജാതികള്‍ക്കുവേണ്ടിയുള്ളവയാണെന്ന് സാരം. ഖുര്‍ആനിലെ ശിക്ഷാവിധികളില്‍ ഇത്തരം പ്രശ്‌നങ്ങളൊന്നും കാണുക സാധ്യമല്ല. അതില്‍ യാതൊരുവിധ ഉച്ചനീചത്വങ്ങളുമില്ല. രാജാവിനും പ്രജക്കും ഒരേ കുറ്റത്തിന് ഒരേ ശിക്ഷ. തികച്ചും മാനവികമായ കാഴ്ചപ്പാട്. അതുപോലെതന്നെ, ഖുര്‍ആന്‍ ലൈംഗിക സദാചാരത്തിന്റെ ലംഘനത്തെ കാണുന്നത് കുടുംബഭദ്രതയെയും സമൂഹത്തിന്റെ കെട്ടുറപ്പിനെയും തകര്‍ക്കുന്ന പ്രവര്‍ത്തനമായിട്ടാണ്. അവിടെ പുരുഷനും സ്ത്രീയുമെല്ലാം തുല്യരാണ്. തെറ്റ് ആര് ചെയ്യുന്നുവെന്നും അത് സമൂഹത്തിലുണ്ടാക്കുന്ന പ്രശ്‌നങ്ങളുടെ ആഴം എത്രത്തോളമുണ്ടെന്നുമുള്ളതാണ് ശിക്ഷയുടെ അളവ് നിര്‍ണയിക്കുന്നത്. വിവാഹിതരുടെയും അവിവാഹിതരുടെയും വ്യഭിചാരം സൃഷ്ടിക്കുന്ന പ്രശ്‌നങ്ങള്‍ വ്യത്യസ്തമായതിനാല്‍ അവക്കുള്ള ശിക്ഷകള്‍ വ്യത്യാസപ്പെട്ടിരിക്കുന്നുവെന്ന് കാണാം. ഇവിടെയും ഖുര്‍ആനിക ശിക്ഷാവിധികളുടെ മാനവികതയാണ് നമുക്ക് കാണാന്‍ കഴിയുന്നത്.
ഒരു ശിക്ഷാനിയമം പ്രായോഗികമാണെന്ന് പറയാനാവുക അത് താഴെ പറയുന്ന ഗുണങ്ങള്‍ പ്രകടിപ്പിക്കുമ്പോഴാണ്. 1.ചെയ്ത തെറ്റിനുള്ള പ്രതികാരമാവുക. 2.തെറ്റുകളെ തടയാന്‍ കഴിയുക. 3.കുറ്റുവാളികളെ ഭയപ്പെടുത്താനാവുക. 4.കുറ്റം വഴി പ്രയാസമനുഭവിക്കേണ്ടിവന്നവര്‍ക്ക് സങ്കടനിവൃത്തി വരുത്തുന്നതാവുക. 5.കുറ്റവാളിയെ സംസ്‌കരിക്കുന്നതാവുക. 6.കുറ്റം വഴി നഷ്ടം നേരിട്ടവര്‍ക്ക് പരിഹാരം നല്‍കുന്നതാവുക. 7.കുറ്റവാളിയെ പാശ്ചാത്താപ വിവശനാക്കുന്നതാവുക. 8.സമൂഹത്തെ കുറ്റങ്ങളില്‍നിന്ന് സംരക്ഷിക്കുന്നതാവുക. ഇസ്‌ലാമിലെ ഏതു ശിക്ഷാനിയമമെടുത്താലും ഈ ധര്‍മങ്ങള്‍ അവ നിര്‍വഹിക്കുന്നതായി കാണാന്‍ കഴിയും. അതുകൊണ്ടുതന്നെ അവ പ്രായോഗികമാണെന്ന് സംശയലേശമന്യേ പറയാനാകും.
വ്യക്തിക്കും സമൂഹത്തിനും സമാധാനം പ്രദാനം ചെയ്യുകയാണ് ഖുര്‍ആനിക നിയമങ്ങളുടെ ലക്ഷ്യം. വ്യക്തികള്‍ക്ക് ചില അവകാശങ്ങളുണ്ട്. ഇൗ അവകാശങ്ങള്‍ അന്യോന്യം അനുവദിച്ചുകൊടുക്കുക വഴിയാണ് സാമൂഹികമായ ഉദ്ഗ്രഥനം സാധ്യമാകുന്നത്. ഒരാളുടെയും അവകാശങ്ങള്‍ ഹനിക്കുവാന്‍ മറ്റൊരാളെയും അനുവദിച്ചുകൂടാ. ആരുടെയെങ്കിലും അവകാശങ്ങള്‍ ഹനിക്കപ്പെടുന്നുണ്ടോയെന്ന് ശ്രദ്ധിക്കേണ്ടതും ഉണ്ടെങ്കില്‍ അത് ഇല്ലാതെയാക്കേണ്ടതും രാഷ്ട്രത്തിന്റെ ബാധ്യതയാണ്. ഇതിനുവേണ്ടിയാണ് ശിക്ഷാനിയമങ്ങള്‍ നടപ്പിലാക്കുന്നത്. നേരായ മാര്‍ഗത്തിലൂടെ ചലിക്കുവാന്‍ വ്യക്തിയെ പ്രചോദിപ്പിക്കുകയാണ് ഖുര്‍ആനിലെ ശിക്ഷാനിയമങ്ങളുടെ ലക്ഷ്യം. സംരക്ഷിക്കപ്പെടേണ്ട പ്രധാനപ്പെട്ട ചില മൂല്യങ്ങളുണ്ടെന്നാണ് ഇസ്‌ലാമിക വീക്ഷണം. വിശ്വാസം, യുക്തിയും ബുദ്ധിയും, അഭിമാനം,ജീവന്‍, സ്വത്ത്, കുടുംബത്തിന്റെ കെട്ടുറപ്പ്, സദാചാര മൂല്യങ്ങള്‍,സമൂഹത്തിന്റെ ഭദ്രത ഇവയെല്ലാം സംരക്ഷിക്കപ്പെടേണ്ടവയാണ്. ഇവ തകര്‍ക്കുവാന്‍ ആരെയും അനുവദിച്ചുകൂടാ. ആരെയും എന്നതുകൊണ്ട് അന്യനെ മാത്രമല്ല അര്‍ഥമാക്കുന്നത്; സ്വന്തത്തെകൂടിയാണ്. സ്വന്തം ജീവന്‍ വെടിയാനാഗ്രഹിച്ചുകൊണ്ട് ആത്മഹത്യക്കു ശ്രമിച്ചവനും സ്വന്തം മാനം തകര്‍ത്തുകൊണ്ട് വ്യഭിചാരവൃത്തിയിലേര്‍പ്പെട്ടവനും സ്വന്തം ബുദ്ധിയെ നശിപ്പിച്ചുകൊണ്ട് മദ്യപാനം ചെയ്യുന്നവനുമെല്ലാം കുറ്റവാളിയാകുന്നത് ഇതുകൊണ്ടാണ്. സ്വന്തത്തെയോ അന്യനെയോ ഭയപ്പെടാതെ എല്ലാവര്‍ക്കും ജീവിക്കുവാന്‍ സാധിക്കുന്ന ഒരു സമൂഹമാണ് ഇസ്‌ലാമിക ശിക്ഷാനിയമങ്ങളുടെ ഉദ്ദേശ്യം. അത്തരമൊരു സമൂഹത്തില്‍ മാത്രമേ ശാന്തിയും സമാധാനവും നിലനില്‍ക്കൂ. എല്ലാവര്‍ക്കും വളരുവാനും വികസിക്കുവാനും സാധിക്കുന്ന, മാനവികതയില്‍ അധിഷ്ഠിതമായ ഒരു സമൂഹത്തിന്റെ സൃഷ്ടിയാണ് ഖുര്‍ആനിലെ ശിക്ഷാനിയമങ്ങള്‍ ലക്ഷ്യമാക്കുന്നത്.

വ്യക്തി, സമൂഹം തുടങ്ങിയ അമൂര്‍ത്ത സങ്കല്‍പങ്ങളെ ഖുര്‍ആന്‍ നോക്കിക്കാണുന്നത് ഭൗതിക പ്രത്യയശാസ്ത്രങ്ങളുടെ വീക്ഷണത്തില്‍നിന്ന് തികച്ചും വ്യത്യസ്തമായ കാഴ്ചപ്പാടിലൂടെയാണ്. അത് അതിന്റെ ശിക്ഷാനിയമങ്ങളിലും പ്രകടമാണ്. ജനിച്ചുവളര്‍ന്ന പ്രത്യേക ചുറ്റുപാടുകളുടെയും സാഹചര്യങ്ങളുടെയും സ്വാധീനവലയത്തില്‍നിന്ന് ഒരിക്കലും മോചിതനാകുവാന്‍ കഴിയാത്ത ഒരു കളിപ്പാവ മാത്രമായി മനുഷ്യനെ കാണുന്ന ഫ്രോയിഡിയന്‍ ചിന്താരീതിയുമായി ഇസ്‌ലാം പൊരുത്തപ്പെടുന്നില്ല. സമൂഹത്തിലെ സാമ്പത്തിക മാറ്റങ്ങള്‍ മാത്രമാണ് വ്യക്തിയിലെ അഹംബോധത്തെയും മൂല്യങ്ങളെയുമെല്ലാം നിശ്ചയിക്കുന്നത് എന്ന മാര്‍ക്‌സിയന്‍ വീക്ഷണവും ഇസ്‌ലാമിന് അന്യമാണ്. സ്വതന്ത്രരായി ജനിച്ചവരെ സ്വതന്ത്രമായിത്തന്നെ ജീവിക്കുവാന്‍ അനുവദിക്കുന്നതിലൂടെയാണ് അവരുടെ വ്യക്തിത്വത്തിന്റെ പൂര്‍ണമായ പ്രകാശനം സാധ്യമാവുകയെന്ന മുതലാളിത്തത്തിന്റെ കാഴ്ചപ്പാടിനെയും ഇസ്‌ലാം നിരാകരിക്കുന്നു. സമ്പത്തും സാഹചര്യങ്ങളും ചുറ്റുപാടുകളുമെല്ലാം മനുഷ്യരുടെ വ്യക്തിത്വത്തെ സ്വാധീനിക്കുന്നുവെന്ന വസ്തുത ഇസ്‌ലാം അംഗീകരിക്കുന്നു. എന്നാല്‍ വ്യക്തിയെ സൃഷ്ടിക്കുന്നത് അതൊന്നുമല്ല. വ്യക്തിയിലെ അഹംബോധത്തെ സൃഷ്ടിക്കുന്നതും സാഹചര്യങ്ങള്‍ക്കൊത്ത് തന്റെ നിലപാട് എന്താണെന്ന് തീരുമാനിക്കുവാന്‍ അവനെ പര്യാപ്തനാക്കുന്നതും അവന്റെ മാത്രം സവിശേഷതയായ ആത്മാവാണ്. മനുഷ്യനു മാത്രം നല്‍കപ്പെട്ട ദൈവികദാനമാണത്. നന്മയെയും തിന്മയെയും തെരഞ്ഞെടുക്കുന്നതിനുള്ള കഴിവ് അവന് നല്‍കുന്നത് ഈ ആത്മാവത്രെ.

വ്യക്തികളാണ് സമൂഹത്തെ സൃഷ്ടിക്കുന്നത്. വ്യക്തിയെ വിമലീകരിക്കുന്നത് ദൈവിക നിയമങ്ങളാണ്. ധാര്‍മിക നിയമങ്ങള്‍ അനുസരിക്കുന്ന വ്യക്തികള്‍ ഉള്‍ക്കൊള്ളുന്ന സമൂഹം സമാധാനപൂര്‍ണവും നന്മ ഉള്‍ക്കൊള്ളുന്നതുമായിരിക്കുമെന്നുറപ്പാണ്. ഈ നിയമങ്ങള്‍ സ്വമേധയാ അനുസരിക്കുകയാണ് വ്യക്തി ചെയ്യേണ്ടത്. അതുവഴി മാത്രമേ ആത്മസംസ്‌കരണം സാധ്യമാകൂ. എന്നാല്‍, ഏതൊരു സമൂഹത്തിലും ധാര്‍മിക നിയമങ്ങളില്‍നിന്ന് വ്യതിചലിക്കുവാന്‍ ശ്രമിക്കുന്ന ചിലരെങ്കിലുമുണ്ടാകും. അവരെ തടഞ്ഞുനിര്‍ത്തിയിട്ടില്ലെങ്കില്‍ സമൂഹത്തില്‍ തിന്മകള്‍ വ്യാപിക്കുന്നതിനും അതുവഴി അരാജകത്വത്തിനും നിമിത്തമാവും. ഇങ്ങനെ തിന്മകള്‍ വ്യാപിക്കുന്നത് തടഞ്ഞുനിര്‍ത്തുന്നതിനായുള്ളതാണ് ശിക്ഷാനിയമങ്ങള്‍.

വ്യക്തിയെയും സമൂഹത്തെയും പരിശുദ്ധമായി നിലനിര്‍ത്തുകയാണ് ഖുര്‍ആനിലെ ശിക്ഷാനിയമങ്ങളുടെ ലക്ഷ്യം. വ്യക്തിയെ സമൂഹത്തിനുവേണ്ടിയോ സമൂഹത്തെ വ്യക്തിക്കുവേണ്ടിയോ ബലികൊടുക്കണമെന്ന വീക്ഷണം ഇസ്‌ലാം ഉള്‍ക്കൊള്ളുന്നില്ല. വ്യക്തിസ്വാതന്ത്ര്യത്തിന്മേല്‍ സമൂഹത്തിന്റെ നേരിയ കൈകടത്തല്‍പോലും അക്ഷന്തവ്യമായിക്കരുതുന്ന മുതലാളിത്ത വീക്ഷണവും സമൂഹത്തിനുവേണ്ടി വ്യക്തിയുടെ സഹജവികാരങ്ങളെപ്പോലും ബലികൊടുക്കേണ്ടതുണ്ടെന്ന കമ്യൂണിസ്റ്റ് വീക്ഷണവും ഇസ്‌ലാമിന് അന്യമാണ്. വ്യക്തിയും സമൂഹവും തമ്മില്‍ നിലനില്‍ക്കേണ്ടത് സംഘട്ടനാത്മകമായ ബന്ധമല്ലെന്നതാണ് ഇസ്‌ലാമിന്റെ കാഴ്ചപ്പാട്. അവയെ ഉദ്ഗ്രഥിതമാക്കുന്നത് മൂല്യങ്ങളാണ്. ഇൗ മൂല്യങ്ങളെ സംരക്ഷിക്കുന്നതുവഴി വ്യക്തിയെയും സമൂഹത്തെയും വിമലീകരിക്കുകയാണ് ഖുര്‍ആനിലെ ശിക്ഷാനിയമങ്ങള്‍ ചെയ്യുന്നത്. അതുകൊണ്ടുതന്നെ അവ വ്യക്തികേന്ദ്രീകൃതമോ സമൂഹകേന്ദ്രീകൃതമോ അല്ല, പ്രത്യുത മൂല്യകേന്ദ്രീകൃതമാണ് എന്നു പറയുന്നതാവും ശരി.

അല്ല. ശിക്ഷാനിയമങ്ങളുടെ ലക്ഷ്യം കുറ്റകൃത്യങ്ങള്‍ ഇല്ലാതെയാക്കുകയാണെങ്കില്‍ ഖുര്‍ആന്‍ നിര്‍ദേശിച്ച ശിക്ഷാനിയമങ്ങള്‍ പൗരാണിക കാലത്തേതു പോലെതന്നെ ഇന്നും പ്രസക്തമാണ്; എന്നും പ്രസക്തമായിരിക്കുകയും ചെയ്യും.

വ്യക്തികള്‍ക്ക് തങ്ങളുടെ ഇഷ്ടാനുസരണം പ്രവര്‍ത്തിക്കുവാനുള്ള സ്വാതന്ത്ര്യം നല്‍കുകയെന്നാണ് ജനാധിപത്യത്തിന്റെ അര്‍ഥമെങ്കില്‍ അത്തരം സമൂഹങ്ങളില്‍ ഖുര്‍ആനിക നിയമങ്ങള്‍ അപ്രായോഗികമായിരിക്കുമെന്ന കാര്യത്തില്‍ സംശയമില്ല. എന്നാല്‍,പൗരന്മാര്‍ക്ക് സൃഷ്ടിപരമായി പുരോഗമിക്കുവാനുള്ള സകല സ്വാതന്ത്ര്യവും നല്‍കുകയും പ്രസ്തുത സ്വാതന്ത്ര്യത്തെ സമൂഹത്തിന് ദോഷകരമായ രീതിയില്‍ വിനിയോഗിക്കുന്നത് തടയുകയും ചെയ്യുകയാണ് ജനാധിപത്യ സമൂഹത്തിലെ നിയമങ്ങളുടെ ലക്ഷ്യമെങ്കില്‍ അവിടെ ഖുര്‍ആന്‍ പ്രദാനം ചെയ്യുന്ന ശിക്ഷാനിയമങ്ങളെപ്പോലെ പ്രസക്തവും പ്രായോഗികവുമായ മറ്റൊന്നുമില്ലെന്നതാണ് വസ്തുത.

മനുഷ്യസമൂഹത്തിന്റെ ഘടനയില്‍ എന്തെന്തു മാറ്റങ്ങളുണ്ടായാലും വ്യക്തിയുടെ വികാരങ്ങളിലോ ചോദനകളിലോ അടിസ്ഥാനപരമായി യാതൊരു മാറ്റവുമുണ്ടായിട്ടില്ലെന്ന വാസ്തവം നാം മനസ്സിലാക്കേണ്ടതുണ്ട്. പൗരാണിക കാലത്ത് എന്തെല്ലാം മൂല്യങ്ങള്‍ സമൂഹത്തിന്റെ സ്വച്ഛമായ നിലനില്‍പിന് അനിവാര്യമായിരുന്നുവോ അതേ മൂല്യങ്ങള്‍തന്നെയാണ് ആധുനിക സമൂഹത്തിലും സംരക്ഷിക്കപ്പെടേണ്ടതായിട്ടുള്ളത്. പ്രസ്തുത മൂല്ല്യങ്ങളില്‍നിന്ന് വ്യതിചലിക്കുവാന്‍ വ്യക്തികള്‍ മുതിരുന്നത് അരാജകത്വത്തിനും അതുവഴി സാമൂഹിക ഘടനയെത്തന്നെ തകര്‍ക്കുന്നതിനും നിമിത്തമാകും.

സമൂഹത്തിന്റെ നേരെ വ്യക്തി നടത്തുന്ന ആക്രമണത്തെയാണ് കുറ്റം എന്നു പറയുന്നത്. കുറ്റങ്ങള്‍ ഇല്ലാതെയാകുന്നതിലൂടെ മാത്രമേ സമൂഹത്തിന്റെ സ്വച്ഛന്ദമായ ഒഴുക്ക് സാധ്യമാകൂ. കുറ്റം ചെയ്യുന്നവരെ ശിക്ഷിക്കുകയെന്നതിലുപരിയായി കുറ്റങ്ങള്‍ ഇല്ലാതെയാക്കുവാന്‍ പരിശ്രമിച്ചു കൊണ്ട് സമാധാനപരമായ സാമൂഹിക ജീവിതം സാധ്യമാക്കുകയെന്നതാണ് ശിക്ഷാനിയമങ്ങളുടെ ആത്യന്തിക ലക്ഷ്യം. ഈ ലക്ഷ്യം സാക്ഷാത്കരിക്കുവാന്‍ സാധിക്കുന്ന ശിക്ഷാനിയമങ്ങള്‍ നിര്‍ദേശിക്കുന്നുവെന്നുള്ളതാണ് ഖുര്‍ആനിന്റെ സവിശേഷത. ഈ രംഗത്ത് ഖുര്‍ആനിനെ പ്രായോഗികമാക്കുന്നത് ഈ സവിശേഷതയാണ്.

മരണപ്പെട്ട വ്യക്തിയുടെ മൂന്ന് പുത്രിമാരും മാതാപിതാക്കളും ഭാര്യയും ജീവിച്ചിരിപ്പുണ്ടെങ്കില്‍ ക്വുർആനിക വിധിപ്രകാരം മക്കള്ക്കൊല്ലാംകൂടി അനന്തരസ്വത്തിന്റെ 2/3 ഭാഗവും (ക്വുർആന്‍ 4:11) മാതാപിതാക്കള്ക്ക് 1/3 ഭാഗവും (4:11) നല്കിക്കഴിഞ്ഞാല്‍ പിന്നെ സ്വത്തൊന്നും ബാക്കിയുണ്ടാവുകയില്ലല്ലോ. പിന്നെ ഭാര്യക്ക് ലഭിക്കേണ്ട 1/8 ഭാഗം സ്വത്ത് (4:12) എവിടെനിന്നാണ് കൊടുക്കുക? ക്വുർആനിലെ അനന്തരാവകാശ നിയമങ്ങള്‍ അപ്രായോഗികമാണെന്നല്ലേ ഇത് കാണിക്കുന്നത്?

    • ഇസ്‌ലാമിന്റെ മൗലികപ്രമാണങ്ങള്‍ ഖുര്‍ആനും ഹദീഥുകളുമാണ്. നിയമനിര്‍ദേശങ്ങളെയോ കര്‍മാനുഷ്ഠാനങ്ങളെയോ കുറിച്ച് വിശദാംശങ്ങള്‍ ഖുര്‍ആനില്‍ വിവരിക്കപ്പെട്ടിട്ടില്ല. നമസ്‌കാരം, സക്കാത്ത്,നോമ്പ്, ഹജ്ജ് തുടങ്ങിയ അതിപ്രധാനങ്ങളായ അടിസ്ഥാനാരാധനകളുടെ പോലും വിശദാംശങ്ങള്‍ ഖുര്‍ആനിലില്ല. പ്രസ്തുത വിശദാംശങ്ങള്‍ വ്യക്തമാക്കപ്പെട്ടിട്ടുള്ളത് ഹദീഥുകളിലാണ്. അനന്തരാവകാശനിയമങ്ങളും തഥൈവ. ഖുര്‍ ആനിന്റെയും ഹദീഥുകളുടെയും അടിസ്ഥാനത്തില്‍ മാത്രമേ ഇസ്‌ലാമിലെ ഏതു നിയമവും പൂര്‍ണമായി നിര്‍ണയിക്കുവാന്‍ കഴിയൂവെന്ന് സാരം.

സൂറത്തുന്നിസാഇലെ 11, 12 സൂക്തങ്ങളില്‍ ദായക്രമത്തിന്റെ മൗലികതത്ത്വങ്ങള്‍ മാത്രമാണ് വിശദീകരിക്കപ്പെട്ടിരിക്കുന്നത്. ഈ തത്ത്വങ്ങളുടെ അടിസ്ഥാനത്തില്‍ അനന്തരസ്വത്ത് എങ്ങനെ കണിശവും വ്യവസ്ഥാപിതവുമായി ഓഹരിവെക്കാമെന്ന് ഹദീഥുകളില്‍ വിവരിക്കപ്പെട്ടിട്ടുണ്ട്. ഖുര്‍ആനിന്റെ മൗലികതത്ത്വങ്ങള്‍ ലംഘിക്കാതെ ഓരോ അവസ്ഥകളിലും എങ്ങനെ സ്വത്ത് ഓഹരിവെക്കാമെന്ന് ഹദീഥ് വ്യാഖ്യാന ഗ്രന്ഥങ്ങളിലും കര്‍മശാസ്ത്ര ഗ്രന്ഥങ്ങളിലും വിശദമായി വിവരിക്കപ്പെട്ടിട്ടുണ്ട്.

അനന്തരാവകാശങ്ങള്‍ നിര്‍ണയിക്കപ്പെട്ട ഓഹരികള്‍ തികയാതെ വരുന്ന പല സന്ദര്‍ഭങ്ങളുമുണ്ട്. അത്തരം സന്ദര്‍ഭങ്ങളില്‍ ഛേദം വര്‍ധിപ്പിച്ചുകൊണ്ട്, കമ്മി ഓരോ അവകാശിയും പങ്കിടുന്ന വിധത്തില്‍ ഓഹരികള്‍ അധികരിപ്പിക്കുകയാണ് വേണ്ടതെന്ന് കര്‍മശാസ്ത്രഗ്രന്ഥങ്ങളില്‍ രേഖപ്പെടുത്തിയിട്ടുണ്ട്. നിര്‍ണിത വിഹിതങ്ങള്‍ നല്‍കാന്‍ ഓഹരികള്‍ തികയാതെ വരുമ്പോള്‍ വീതാംശം പൂര്‍ത്തീകരിക്കാനായി ഛേദം വര്‍ധിപ്പിക്കുന്നതിനാണ് ‘ഔല്‍‘ എന്നു പറയുക. ‘ഔല്‍‘ എന്നാല്‍ ‘അധികരിക്കല്‍‘ എന്നര്‍ഥം. ഓഹരികള്‍ തികയാതെ വരുന്ന എല്ലാ സന്ദര്‍ഭങ്ങളിലും ഔല്‍ തത്ത്വമനുസരിച്ചാണ് സ്വത്ത് ഭാഗിക്കേണ്ടതെന്നാണ് ഇസ്‌ലാമിക വിധി.

കര്‍മശാസ്ത്ര പണ്ഡിതന്മാര്‍ ദായക്രമത്തില്‍ കടന്നുവരുന്ന അടിസ്ഥാന ഛേദങ്ങളെ ഏഴായി നിശ്ചയിച്ചിട്ടുണ്ട്. 2,3,4,6,8,12,24 എന്നിവയാണ് ഏഴ് അടിസ്ഥാന ഛേദങ്ങള്‍. ഇതില്‍ ഔലിന് വിധേയമാകുന്നവ 6,12,24 എന്നീ മൂന്നെണ്ണമാണ്. 2,3,4,8 തുടങ്ങിയ നാലു സംഖ്യകള്‍ ഛേദങ്ങളായി വരുന്ന അവസരങ്ങളില്‍, അവയുടെ അംശങ്ങള്‍ ഒരിക്കലും അവയേക്കാള്‍ അധികമാകാത്തതിനാല്‍, ഔല്‍ ആവശ്യമായി വരികയില്ല. അടിസ്ഥാനഛേദം 6 ആണെങ്കില്‍, ഔല്‍ ആവശ്യമായി വരുന്ന സന്ദര്‍ഭങ്ങളില്‍, അത് ഏഴോ, എട്ടോ, ഒമ്പതോ, പത്തോ ആക്കി വര്‍ധിപ്പിച്ചുകൊണ്ട് സ്വത്ത് വിഭജിക്കാവുന്നതാണ്. അടിസ്ഥാന ഛേദം പന്ത്രണ്ട് ആണെങ്കില്‍ പതിമൂന്നോ പതിനഞ്ചോ പതിനേഴോ ആക്കി വര്‍ധിപ്പിച്ചുകൊണ്ടും 24 ആണെങ്കില്‍ 27 ആക്കി വര്‍ധിപ്പിച്ചുകൊണ്ടുമാണ് ഔല്‍ ആവശ്യമായി വരുന്നുവെങ്കില്‍ സ്വത്ത് വിഭജിക്കേണ്ടത്. ഇങ്ങനെ, ദായക്രമത്തിലെ അംശവര്‍ധനവിനനുസരിച്ച് എങ്ങനെയെല്ലാമാണ് സ്വത്ത് വിഭജനം നടത്തേണ്ടതെന്ന് സൂക്ഷ്മവും വ്യക്തവുമായ രീതിയില്‍ കര്‍മശാസ്ത്രഗ്രന്ഥങ്ങളില്‍ പ്രതിപാദിക്കപ്പെട്ടിട്ടുണ്ട്.

ചോദ്യത്തില്‍ പ്രതിപാദിക്കപ്പെട്ട പ്രശ്‌നത്തില്‍ പെണ്‍മക്കളെല്ലാം കൂടി സ്വത്തിന്റെ 16/27 ഭാഗവും മാതാപിതാക്കള്‍ക്ക് 8/27 ഭാഗവും ഭാര്യക്ക് 3/27 ഭാഗവുമാണ് ലഭിക്കുക. അഥവാ 24 ആയിരുന്ന ഛേദത്തെ27 ആക്കി ഉയര്‍ത്തിക്കൊണ്ടാണ് ഇവിടെ സ്വത്ത് വിഭജനം നടത്തേണ്ടത്. ഖുര്‍ആനും അതിന്റെ വ്യാഖ്യാനമായ ഹദീഥുകളും ഒരുമിച്ച് പരിശോധിച്ചാല്‍ ഇത്തരം പ്രശ്‌നങ്ങള്‍ക്ക് ഉത്തരം കാണുക പ്രയാസകരമല്ല. ‘തീര്‍ച്ചയായും നിങ്ങള്‍ക്ക് അല്ലാഹുവിന്റെ ദൂതനില്‍ ഉത്തമമായ മാതൃകയുണ്ട് (33:21) എന്നും ‘സത്യവിശ്വാസികളേ, നിങ്ങള്‍ അല്ലാഹുവെ അനുസരിക്കുക, ദൂതനെയും നിങ്ങളില്‍നിന്നുള്ള കൈകാര്യ കര്‍ത്താക്കളെയും അനുസരിക്കുക‘ (4:59) എന്നും പഠിപ്പിക്കുന്ന വിശുദ്ധ ഖുര്‍ആനിന്റെ താല്‍പര്യംതന്നെയാണ് അത് പ്രതിപാദിപ്പിക്കുന്ന നിയമങ്ങളെ പ്രവാചക ചര്യയുടെയും അദ്ദേഹത്തിന്റെ സഖാക്കളുടെ വ്യാഖ്യാനങ്ങളുടെയും അടിസ്ഥാനത്തില്‍ മനസ്സിലാക്കുക എന്നത്. അങ്ങനെ മനസ്സിലാക്കുമ്പോള്‍ ഇത്തരം പ്രശ്‌നങ്ങള്‍ക്കൊന്നും പരിഹാരം കണ്ടെത്തുക പ്രയാസമുള്ള കാര്യമാവുകയില്ല.

ഒരു മകന്‍ മാത്രം അനന്തരാവകാശിയാവുകയാണെങ്കില്‍ അയാൾക്ക് പിതൃസ്വത്ത് മുഴുവനായി ലഭിക്കുമെന്നിരിക്കെ മകള്‍ മാത്രമാണ് അനന്തരാവകാശിയെങ്കില്‍ അവൾക്ക് പകുതി മാത്രവും ഒന്നിലധികം പെണ്മമക്കളുണ്ടെങ്കില്‍ അവർക്കെല്ലാം കൂടി പിതൃസ്വത്തിന്റെ മൂന്നില്‍ രണ്ടു ഭാഗം മാത്രവുമാണ് ലഭിക്കുകയെന്ന ഖുര്ആനിക നിയമം വ്യക്തമായ അനീതിയല്ലേ?

ആണ്‍മക്കളില്ലാത്തവരുടെ അനന്തരാവകാശവുമായി ബന്ധപ്പെട്ട പ്രശ്‌നമാണിത്. ഖുര്‍ആനിക ദായക്രമപ്രകാരം (4:11) മരണപ്പെട്ടയാള്‍ക്ക് ഒരേയൊരു പെണ്‍കുട്ടി മാത്രമാണുള്ളതെങ്കില്‍ അവള്‍ക്ക് സ്വത്തിന്റെ പകുതിയും ഒന്നിലധികമുണ്ടെങ്കില്‍ അവര്‍ക്കെല്ലാം കൂടി മൂന്നില്‍ രണ്ടു ഭാഗവുമാണ് ലഭിക്കുക. ബാക്കി അടുത്ത ബന്ധുക്കള്‍ക്കാണ് ലഭിക്കുക.

ഇത് ഖുര്‍ആനിക ദായക്രമത്തിലെ പുരുഷമേധാവിത്വമല്ല, പ്രത്യുത മനുഷ്യപ്രകൃതിയെക്കുറിച്ച് വ്യക്തമായി അറിയാവുന്നവനാണ് ഖുര്‍ആനിന്റെ കര്‍ത്താവെന്ന വസ്തുതയാണ് വ്യക്തമാക്കുന്നത്. മനുഷ്യര്‍ക്ക് ആത്യന്തികമായി ഗുണം ചെയ്യുന്ന കാര്യങ്ങളെന്തൊക്കെയാണെന്ന് വ്യക്തമായി അറിയാവുന്നവനാണ് ഖുര്‍ആനിന്റെ കര്‍ത്താവെന്ന യാഥാര്‍ഥ്യം വെളിപ്പെടുത്തുകയാണ് ഈ അനന്തരാവകാശ നിയമവുമെന്ന വസ്തുത അവധാനതയോടെ പ്രശ്‌നത്തെ സമീപിച്ചാല്‍ മനസ്സിലാകും

.മരണപ്പെട്ടയാളുടെ കുടുംബബന്ധങ്ങള്‍ മാത്രം പരിഗണിച്ചു കൊണ്ടല്ല ഖുര്‍ആന്‍ ദായക്രമം അനുശാസിക്കുന്നത്. മരണത്തിന് മുമ്പ് സ്വത്തിന്റെ അവകാശിക്ക് ലഭിക്കേണ്ട ശുശ്രൂഷയും സ്‌നേഹവാല്‍സല്യങ്ങളുമെല്ലാം ദായക്രമം നിശ്ചയിക്കുമ്പോള്‍ ഖുര്‍ആനിന്റെ പരിഗണനക്ക് വരുന്നുണ്ട്. അതോടൊപ്പംതന്നെ, പെണ്‍മക്കള്‍ മാത്രമുള്ള വ്യക്തിയുടെ മരണത്തിനുശേഷം പ്രസ്തുത സന്താനങ്ങളുടെ സംരക്ഷണ ചുമതലയും ഖുര്‍ആന്‍ സജീവമായി പരിഗണിക്കുന്നു. ഓഹരി നിശ്ചയങ്ങളെക്കുറിച്ച് പരാമര്‍ശിക്കവെ ഖുര്‍ആന്‍ പറയുന്ന ഒരു കാര്യം പ്രത്യേകം ശ്രദ്ധേയമാണ്. ‘നിങ്ങളുടെ പിതാക്കളിലും നിങ്ങളുടെ സന്താനങ്ങളിലും ഉപകാരം കൊണ്ട് നിങ്ങളോട് ഏറ്റവും അടുത്തവര്‍ ആരാണെന്ന് നിങ്ങള്‍ക്കറിയില്ല. അല്ലാഹുവിന്റെ പക്കല്‍നിന്നുള്ള ഓഹരി നിര്‍ണയമാണിത്. തീര്‍ച്ചയായും അല്ലാഹു എല്ലാം അറിയുന്നവനും യുക്തിമാനുമാകുന്നു‘ (4:11).മനുഷ്യനെക്കുറിച്ച് സമഗ്രമായി അറിയാവുന്ന നാഥന്റെ നിയമനിര്‍ദേശങ്ങള്‍ സൂക്ഷ്മവും പ്രായോഗികവും മാനവികവുമായിരിക്കും. ഇത് വ്യക്തമാക്കുന്നതാണ് മുകളില്‍ പരാമര്‍ശിക്കപ്പെട്ട പ്രശ്‌നത്തില്‍ ഖുര്‍ആനിക വിധിയുമെന്നുള്ളതാണ് വസ്തുത.

താഴെപ്പറയുന്ന കാര്യങ്ങള്‍ ശ്രദ്ധേയമാണ്.

ഒന്ന്: മാതാപിതാക്കളുടെ വാര്‍ധക്യകാലത്ത് അവരെ സംരക്ഷിക്കേണ്ടത് ആണ്‍മക്കളുടെ ബാധ്യതയാണ്: ആണ്‍മക്കളില്ലെങ്കില്‍ സഹോദരന്മാരോ അവരുടെ പുത്രന്മാരോ ആണ് വാര്‍ധക്യത്തിലെത്തിയവരെ സംരക്ഷിക്കേണ്ടത്.

രണ്ട്: വാര്‍ധക്യത്തിലിരിക്കുന്ന മാതാപിതാക്കള്‍ക്ക് ചെലവിനുകൊടുക്കുകയോ അവരെ സംരക്ഷിക്കുകയോ ചെയ്യേണ്ട ഉത്തരവാദിത്തം പെണ്‍മക്കളില്‍ ഇസ്‌ലാം ഭരമേല്‍പിക്കുന്നില്ല. അന്യകുടുംബങ്ങളില്‍ ഭര്‍ത്താക്കന്മാരോടും കുട്ടികളോടുമൊപ്പം കഴിയുന്ന പെണ്‍മക്കളില്‍ മാതാപിതാക്കളുടെ സംരക്ഷണമെന്ന ബാധ്യത ഏല്‍പിക്കുന്നത് പ്രായോഗികമല്ലെന്നാണ് ഇസ്‌ലാമിന്റെ കാഴ്ചപ്പാട്. വാര്‍ധക്യത്തിലെത്തിക്കഴിഞ്ഞവര്‍ക്ക് ആണ്‍മക്കളില്ലെങ്കില്‍ സഹോദരങ്ങളോ സഹോദരപുത്രന്മാരോ ആണ്, പെണ്‍മക്കളല്ല അവരുടെ സംരക്ഷണം ഏറ്റെടുക്കേണ്ടത്.

മൂന്ന്: പിതാവിന്റെ മരണശേഷം പെണ്‍കുട്ടികളുടെ രക്ഷിതാക്കള്‍ (വലിയ്യ്) സഹോദരങ്ങളാണ്. മരണപ്പെട്ടയാള്‍ക്ക് ആണ്‍കുട്ടികളില്ലെങ്കില്‍ അയാളുടെ പെണ്‍കുട്ടികളുടെ രക്ഷിതാക്കള്‍ പിതൃസഹോദരങ്ങളോ അവരുടെ മക്കളോ അതല്ലെങ്കില്‍ അടുത്ത ബന്ധുക്കളോ ആണ്. മരണപ്പെട്ടയാളുടെ പെണ്‍കുട്ടികളെ വിവാഹം കഴിച്ച് പറഞ്ഞയക്കേണ്ട ബാധ്യത ഈ രക്ഷിതാക്കളിലാണ് നിക്ഷിപ്തമായിരിക്കുന്നത്. അവര്‍ വിധവകളാകുകയും അവരുടെ സന്തതികള്‍ അനാഥരാകുകയും ചെയ്താല്‍ അവരുടെ സംരക്ഷണവും ഈ ബന്ധുക്കളുടെ ബാധ്യതയാണ്. അവര്‍ വിവാഹമോചനം ചെയ്യപ്പെട്ടാല്‍ പുനര്‍വിവാഹത്തിനാവശ്യമായ സംവിധാനങ്ങള്‍ ചെയ്യേണ്ടതും അടുത്ത ബന്ധുക്കളായ രക്ഷിതാക്കളുടെ ഉത്തരവാദിത്തംതന്നെ.

ഈ വസ്തുതകളുടെ വെളിച്ചത്തിലാണ് ആണ്‍മക്കളില്ലാതെ മരണപ്പെടുന്നയാളുടെ അനന്തരാവകാശികളെക്കുറിച്ച് ഖുര്‍ആനിക നിയമങ്ങള്‍ പരിശോധനാവിധേയമാക്കേണ്ടത്്. അപ്പോഴാണ് അവ എത്രമാത്രം കുറ്റമറ്റതാണെന്ന് ബോധ്യമാവുക. യാതൊരു അവകാശങ്ങളും ഇല്ലാതെ ബാധ്യതകള്‍ മാത്രം ഏറ്റെടുക്കുവാന്‍ ബന്ധുക്കള്‍ സന്നദ്ധരായിക്കൊള്ളണമെന്നില്ല. പെണ്‍മക്കള്‍ മാത്രമുള്ള ഒരാളുടെ ബന്ധുക്കള്‍ക്ക് നിരവധി ബാധ്യതകള്‍ ഇസ്‌ലാം നിഷ്‌കര്‍ഷിക്കുന്നുണ്ട്. അയാളുടെ വാര്‍ധക്യത്തിലെ സംരക്ഷണബാധ്യത ഈ ബന്ധുക്കളിലാണ് നിക്ഷിപ്തമായിരിക്കുന്നത്. അയാള്‍ക്ക് ജീവനാംശവും സ്‌നേഹപരിചരണങ്ങളും നല്‍കേണ്ടത് ഇവര്‍തന്നെയാണ്. അയാളുടെ മരണശേഷം പെണ്‍മക്കളുടെ രക്ഷിതാക്കളാവേണ്ടതും ഈ ബന്ധുക്കള്‍തന്നെ. പെണ്‍മക്കളോ അവരുടെ സന്താനങ്ങളോ നിരാലംബരാകുകയാണെങ്കില്‍ അവരെ സംരക്ഷിക്കേണ്ട ഉത്തരവാദിത്തവും ഈ ബന്ധുക്കള്‍ക്കാണ്. ഇങ്ങനെ, ആണ്‍മക്കളില്ലാത്ത ആളുടെ ബന്ധുക്കള്‍ക്ക് നിരവധി ബാധ്യതകളുണ്ട്. ഈ ബാധ്യതകള്‍ നല്‍കുന്നതോടൊപ്പം അയാളുടെ അനന്തര സ്വത്തില്‍ ചെറിയ അവകാശവും ബന്ധുക്കള്‍ക്ക് നല്‍കുകയാണ് ഇസ്‌ലാം ചെയ്യുന്നത്. ഉത്തരവാദിത്തങ്ങളും അവകാശങ്ങളും പരസ്പര പൂരകമായി കാണുന്ന ഇസ്‌ലാമിക നിയമങ്ങളുടെ സവിശേഷതയാണ് ഇവിടെയും പ്രകടമാകുന്നത്.

ആണ്‍മക്കളില്ലാത്ത ചിലര്‍ തന്റെ സ്വത്ത് മുഴുവന്‍ പെണ്‍കുട്ടികള്‍ക്ക് ലഭിക്കാനായി ജീവിതകാലത്തു തന്നെ അവര്‍ക്ക് ഇഷ്ദാടനം ചെയ്യാറുണ്ട്. എന്നാല്‍, ഇക്കാര്യത്തില്‍ ദൈവിക വിധിവിലക്കുകള്‍ പാലിച്ചുകൊണ്ട് സ്വത്തിന്റെ ചെറിയൊരംശം അടുത്ത ബന്ധുക്കള്‍ക്കുകൂടി നല്‍കുന്നതാണ് തന്റെ വാര്‍ധക്യകാല സംരക്ഷണത്തിനും പെണ്‍മക്കളുടെ ഭാവിക്കും നല്ലതെന്ന വസ്തുതയാണ് അനുഭവങ്ങള്‍ കാണിക്കുന്നത്. നമ്മുടെ കണക്കുകൂട്ടലുകള്‍ക്ക് അപ്പുറമുള്ള നന്മകളെയും തിന്മകളെയും കുറിച്ച് വ്യക്തവും കൃത്യവുമായി അറിയാവുന്ന സര്‍വശക്തന്റെ വിധിവിലക്കുകള്‍ അനുസരിക്കുന്നതിലൂടെ മാത്രമേ വ്യക്തിക്കും കുടുംബത്തിനുമെല്ലാം ശാന്തി കൈവരിക്കാന്‍ കഴിയുകയുള്ളൂവെന്നതല്ലോ വസ്തുത. ഖുര്‍ആന്‍ പറയുന്നത് എത്ര ശരി. ‘ഒരു കാര്യം നിങ്ങള്‍ വെറുക്കുകയും (യഥാര്‍ഥത്തില്‍) അത് നിങ്ങള്‍ക്ക് ഗുണകരമായിരിക്കുകയും ചെയ്യാം. നിങ്ങള്‍ക്ക് ഒരു കാര്യം ഇഷ്ടപ്പെടുകയും (യഥാര്‍ഥത്തില്‍) അത് നിങ്ങള്‍ക്ക് ദോഷകരമായിരിക്കുകയും ചെയ്‌തെന്നും വരാം. അല്ലാഹു അറിയുന്നു. നിങ്ങള്‍ അറിയുന്നില്ല‘ (2:216).

അനാഥ പൗത്രന് സ്വത്തവകാശം നല്കുവാന്‍ നിയമമില്ലാത്തതിനാല്‍ അവനെ വഴിയാധാരമാക്കണമെന്നാണോ ഇസ്‌ലാം വിവക്ഷിക്കുന്നത്? ഈ പ്രശ്‌നത്തില്‍ ഇസ്‌ലാമിന്റെ പരിഹാരമെന്താണ്?

  • അന്തരാവകാശ നിയമങ്ങള്‍ മാത്രമല്ല ഇസ്‌ലാമിലുള്ളത്; സംരക്ഷണ നിയമങ്ങള്‍ കൂടിയുണ്ട്. ഈ നിയമങ്ങള്‍ രണ്ടും പരസ്പരം ബന്ധപ്പെട്ടാണ് ഇരിക്കുന്നത്. അനന്തരാവകാശത്തിന്റെ അടിസ്ഥാനംതന്നെ അന്യോന്യമുള്ള സംരക്ഷണ ബാധ്യതയാണ്. ഒരാളെ അവശതയില്‍ സംരക്ഷിക്കുവാന്‍ ബാധ്യസ്ഥനാരോ അയാള്‍ക്കാണ് സാധാരണ ഗതിയില്‍ അനന്തരാവകാശം ഉണ്ടാകുന്നത്. പിതാവിനെ സംരക്ഷിക്കാന്‍ പുത്രന്‍ ബാധ്യസ്ഥനാണ്. പുത്രനെ സംരക്ഷിക്കാന്‍ പിതാവും. മകനുണ്ടെങ്കില്‍ പിതാമഹനെ സംരക്ഷിക്കാന്‍ പൗത്രന്‍ ബാധ്യസ്ഥനല്ല; മകനില്ലെങ്കില്‍ ബാധ്യസ്ഥനാണുതാനും. (ഇതുകൊണ്ടുകൂടിയാണ് അനാഥപൗത്രന് നിയമപരമായി സ്വത്തില്‍ അവകാശമില്ലാത്തത്) പിതാവില്ലാത്ത കുട്ടികളെ സംരക്ഷിക്കേണ്ട ബാധ്യത പിതാമഹനാണുള്ളത്. അനാഥകളെ സംരക്ഷിക്കാതിരിക്കുന്നത് മതനിഷേധമായിട്ടാണ് ഖുര്‍ആന്‍ ഗണിച്ചിരിക്കുന്നത്.”മതനിഷേധിയെ നീ കണ്ടുവോ? അനാഥകളെ അവഗണിക്കുന്നവനാണവന്‍” (107:1,2).”അനാഥയെ നീ അടിച്ചമര്‍ത്തരുത്” (93:9).

    ”മാതാപിതാക്കള്‍ക്കും ബന്ധുക്കള്‍ക്കും അനാഥകള്‍ക്കും അഗതികള്‍ക്കും നന്മ ചെയ്യണം” (2:83).

    അനാഥകളുടെ സമ്പത്ത് അന്യായമായി ഭക്ഷിക്കുന്നത് കൊടുംപാതകമാണെന്നാണ് ഖുര്‍ആന്‍ പഠിപ്പിക്കുന്നത്.

    ”അനാഥകള്‍ക്ക് അവരുടെ സ്വത്തുക്കള്‍ നിങ്ങള്‍ വിട്ടുകൊടുക്കുക. നല്ലതിനു പകരം ദുഷിച്ചത് നിങ്ങള്‍ മാറ്റിയെടുക്കരുത്. നിങ്ങളുടെ ധനത്തോട് കൂട്ടിച്ചേര്‍ത്ത് അവരുടെ ധനം നിങ്ങള്‍ തിന്നുകളയുകയുമരുത്. തീര്‍ച്ചയായും അത് ഒരു കൊടുംപാതകമാകുന്നു” (4:2).

    ”ഏറ്റവും ഉത്തമമായ രൂപത്തിലല്ലാതെ നിങ്ങള്‍ അനാഥരുടെ സ്വത്തിനെ സമീപിച്ചുപോകരുത്. അവന് കാര്യപ്രാപ്തിയെത്തുന്നതുവരെ” (6:152).

    അനാഥകളോട് അനാദരവ് കാണിക്കുന്ന യാതൊരുവിധ പ്രവര്‍ത്തനങ്ങളുമുണ്ടാവാന്‍ പാടില്ലെന്നാണ് നബി(ﷺ) പഠിപ്പിച്ചത്. അനാഥകളെ സംരക്ഷിക്കുന്നവന്‍ തന്നോടൊപ്പം സ്വര്‍ഗത്തില്‍ പ്രവേശിക്കുമെന്ന് സുവിശേഷമറിയിച്ച മുഹമ്മദ്(ﷺ) അനാഥയുടെ സ്വത്ത് അന്യായമായി ഭുജിക്കുന്നത് മഹാപാപങ്ങളിലൊന്നാണെന്ന മുന്നറിയിപ്പുകൂടി നല്‍കിയിട്ടുണ്ട്. കുടുംബബന്ധങ്ങളൊന്നുമില്ലാത്ത അനാഥകളെതന്നെ സംരക്ഷിക്കേണ്ടത് ഓരോ മുസ്‌ലിമിന്റെയും ബാധ്യതയാണ്. കുടുംബത്തില്‍പെട്ട അനാഥകളുടെ സ്ഥിതി പിന്നെ പറയേണ്ടതുണ്ടോ? അവരെ സംരക്ഷിക്കേണ്ട ഉത്തരവാദിത്തം പ്രധാനമായും പിതാമഹനിലാണ് നിക്ഷിപ്തമായിരിക്കുന്നത്. പിതാമഹന്‍ മരണപ്പെട്ടാല്‍ പിതൃവ്യനാണ് സംരക്ഷണമേറ്റെടുക്കേണ്ടത്.

    അനാഥ പൗത്രനെ സംരക്ഷിക്കുന്നത് പിതാമഹനാണ്. അയാള്‍ക്കാവശ്യമായ എല്ലാം നല്‍കുന്നത് പിതാമഹനാണ്. അദ്ദേഹത്തിനറിയാം, അവന്റെ പ്രശ്‌നങ്ങളും പ്രയാസങ്ങളുമെന്തെല്ലാമാണെന്ന്. തന്റെ മരണശേഷം നിയമപ്രകാരം പൗത്രന് അനന്തരാവകാശമില്ലെന്നും അദ്ദേഹത്തിനറിയാം. അവന്റെ സംരക്ഷണത്തിനുവേണ്ടി തന്റെ സ്വത്തിന്റെ നല്ലൊരു ഭാഗം നീക്കിവെക്കുവാന്‍ അദ്ദേഹത്തിന് അവകാശമുണ്ട്. ഇത്തരം ആവശ്യങ്ങള്‍ക്കുവേണ്ടിയാണ് ഖുര്‍ആന്‍ ‘വസ്വിയ്യത്ത്‘നിര്‍ബന്ധമാക്കിയിട്ടുള്ളത്.

    ചെയ്യുന്ന വ്യക്തിയുടെ മരണത്തോടെ ചെയ്യപ്പെടുന്ന വ്യക്തിക്ക് ഉടമസ്ഥാവകാശം ലഭിക്കുന്ന ദാനമാണ് വസ്വിയ്യത്ത്. വിശുദ്ധ ഖുര്‍ആന്‍ വസ്വിയ്യത്തിന് വളരെയേറെ പ്രാധാന്യം കല്‍പിച്ചിട്ടുണ്ട്.

    ”നിങ്ങളില്‍ ആര്‍ക്കെങ്കിലും മരണം ആസന്നമാകുമ്പോള്‍ അയാള്‍ ധനം വിട്ടുപോകുന്നുണ്ടെങ്കില്‍ മാതാപിതാക്കള്‍ക്കും അടുത്ത ബന്ധുക്കള്‍ക്കും വേണ്ടി ന്യായപ്രകാരം വസ്വിയ്യത്ത് ചെയ്യുവാന്‍ നിങ്ങള്‍ നിര്‍ബന്ധിതമായി കല്‍പിക്കപ്പെട്ടിരിക്കുന്നു. സൂക്ഷ്മത പുലര്‍ത്തുന്നവര്‍ക്ക് ഒരു കടമയത്രേ അത്” (2:180).

    ഏതു സമയത്തും മരണം കടന്നുവരാമെന്നും അത്് പ്രതീക്ഷിച്ചുകൊണ്ട് വസ്വിയ്യത്ത് എഴുതി വെക്കണമെന്നും പ്രവാചകന്‍(ﷺ) നിര്‍ദേശിച്ചതായും കാണാന്‍ കഴിയും (ബുഖാരി,മുസ്‌ലിം). വസ്വിയ്യത്തിന് ദൈവദൂതന്‍ അതിയായി പ്രോല്‍സാഹിപ്പിച്ചിരുന്നു. അത് ചെയ്യാതിരിക്കുന്നതിന് നിരുല്‍സാഹപ്പെടുത്തുകയും ചെയ്തിരുന്നു (അഹ്മദ്, തുര്‍മുദി,അബൂദാവൂദ്) ഇവയില്‍നിന്ന് വസ്വിയ്യത്തിന് ഇസ്‌ലാം വളരെയേറെ പ്രാധാന്യം കല്‍പിച്ചിരിക്കുന്നുവെന്ന് മനസ്സിലാക്കാനാവും.

    ആകെ സ്വത്തിന്റെ മൂന്നിലൊന്നില്‍ അധികമാകാത്തവിധം വസ്വിയ്യത്ത് ചെയ്യാന്‍ ഒരാള്‍ക്ക് അവകാശമുണ്ട്. ആര്‍ക്കുവേണ്ടിയാണ് വസ്വിയ്യത്ത്? അനന്തരാവകാശികള്‍ക്കുവേണ്ടി വസ്വിയ്യത്ത് ചെയ്യാവതല്ലെന്നാണ് പ്രവാചകന്‍ (ﷺ) പഠിപ്പിച്ചിരിക്കുന്നത്.”പിന്തുടര്‍ച്ചക്കാരന് വസ്വിയ്യത്തില്ല” (അഹ്മദ്, തിര്‍മുദി).

    പിന്നെയാര്‍ക്കുവേണ്ടിയാണ് വസ്വിയ്യത്ത് ചെയ്യേണ്ടത്? അത് തീരുമാനിക്കേണ്ടത് അത് ചെയ്യുന്ന വ്യക്തിയാണ്. പിന്തുടര്‍ച്ചക്കാരല്ലാത്ത അടുത്ത ബന്ധുക്കളെയാണ് ആദ്യമായി പരിഗണിക്കേണ്ടതെന്നാണല്ലോ നടേ ഉദ്ധരിച്ച ഖുര്‍ആന്‍ സൂക്തം (2.180) വ്യക്തമാക്കുന്നത്. അതില്‍ പ്രധാനമായും ഉള്‍പ്പെടുക അനാഥ പൗത്രന്‍തന്നെയായിരിക്കും. അനാഥരായ പൗത്രന്മാര്‍ക്ക് എത്ര സ്വത്ത് നല്‍കുവാനും പിതാമഹന് അവകാശമുണ്ട്. അദ്ദേഹം ജീവിച്ചിരിക്കുമ്പോള്‍തന്നെ ദാനമായി എത്രവേണമെങ്കിലും നല്‍കാം. ആകെ സ്വത്തിന്റെ മൂന്നിലൊന്നുവരെ വസ്വിയ്യത്തായും നല്‍കാം. അവകാശികള്‍ക്കും നിരാലംബരായി തീരുന്ന ആശ്രിതര്‍ക്കും നീതി നിഷേധിക്കുന്ന തരത്തിലാകരുത് ഇഷ്ടദാനവും വസ്വിയ്യത്തും എന്നു മാത്രമേയുള്ളൂ.

    അനാഥ പൗത്രന്റെ കാര്യത്തില്‍ ഇസ്‌ലാം ചെയ്തിട്ടുള്ളത് ഇതാണ്. അയാളുടെ സംരക്ഷണത്തിന്റെ ഉത്തരവാദിത്തം പിതാമഹനെ ഏല്‍പിച്ചു; പിതാമഹന്‍ മരിച്ചാല്‍ പിതൃവ്യനെയും. അയാളുടെ ആവശ്യങ്ങളും അവശതകളും മനസ്സിലാക്കി എത്ര സ്വത്ത് വേണമെങ്കിലും നല്‍കാനുള്ള സ്വാതന്ത്ര്യം പിതാമഹന് നല്‍കി. അയാളെക്കുറിച്ച് മറ്റാരെക്കാളും അറിയുക പിതാമഹനാണല്ലോ. അയാള്‍ക്കുവേണ്ടി – മറ്റു അവശര്‍ക്കും അശരണര്‍ക്കും വേണ്ടിയും – മൂന്നുലൊന്നുവരെ വസ്വിയ്യത്ത് ചെയ്യുവാനുള്ള അവകാശവും പിതാമഹന് നല്‍കി. പ്രസ്തുത വസ്വിയ്യത്ത് പ്രകാരമുള്ള സ്വത്ത് നീക്കിവെച്ച ശേഷം ബാക്കിയുള്ള സ്വത്തു മാത്രമേ അനന്തരാവകാശികള്‍ ഭാഗിച്ചെടുക്കാന്‍ പാടുള്ളൂവെന്ന് പ്രത്യേകം നിഷ്‌കര്‍ഷിച്ചു.(4:11)അനാഥകള്‍ സ്വത്തിന് അവകാശികളായിത്തീരുമ്പോള്‍ പ്രായപൂര്‍ത്തിയെത്താത്തവരാണെങ്കില്‍ പ്രസ്തുത സ്വത്ത് സംരക്ഷിക്കാന്‍ അടുത്ത ബന്ധുക്കളെ ചുമതലപ്പെടുത്തുകയും അവര്‍ക്ക് കാര്യബോധമെത്തുമ്പോള്‍ കൈമാറണമെന്ന് നിഷ്‌കര്‍ഷിക്കുകയും ചെയ്തു (4:6).

    പിതാമഹന്‍ വസ്വിയ്യത്ത് ചെയ്തിട്ടില്ലെങ്കിലും പിതൃവ്യന്‍ ഉള്‍പ്പെടെയുള്ള ബന്ധുക്കള്‍ അനാഥരായ പൗത്രനോ പൗത്രന്മാര്‍ക്കോ ന്യായമായ വിഹിതം നല്‍കുന്ന കാര്യം ദായധനം ഭാഗിച്ചെടുക്കുന്ന സമയത്ത് പരിഗണിക്കണമെന്ന് വിശുദ്ധ ഖുര്‍ആന്‍ ഗൗരവപൂര്‍വം അനുശാസിച്ചിട്ടുണ്ട് (4:8,9).

    നിയമത്തിന് അതിന്‍േറതായ ഒരു രീതിശാസ്ത്രമുണ്ട്. പ്രസ്തുത രീതിശാസ്ത്രമനുസരിക്കുകയാണ് ഇസ്‌ലാം ചെയ്യുന്നത്. എന്നാല്‍,അതോടൊപ്പംതന്നെ നിയമത്തിന്റെ ഊരാക്കുടുക്കുകളില്ലാതെതന്നെ അനാഥപൗത്രന്റെ പ്രശ്‌നം പോലുള്ളവ അത് കൈകാര്യം ചെയ്യുകയും ചെയ്യുന്നു. ധര്‍മബോധത്തെ ഉത്തേജിപ്പിച്ചുകൊണ്ടാണ് ഇസ്‌ലാം ഇത് സാധിക്കുന്നത്. ഏറ്റവും പ്രായോഗികമായ മാര്‍ഗവും, ഇത്തരം വിഷയങ്ങളില്‍ അതുതന്നെയാണ്.

പരേതന്ന് പുത്രനുള്ളപ്പോള്‍ അനാഥ പൗത്രന്‍ അനന്തരാവകാശിയാവുകയില്ലെന്നാണോ ക്വുർആനിക നിയമം. ഇത് അന്യായവും അനാഥരോടുള്ള അനീതിയുമല്ലേ?

  • ദായധനത്തെക്കുറിച്ച വിശുദ്ധ ഖുര്‍ആന്റെ സമീപനത്തിലേക്ക് ആഴത്തിലിറങ്ങിച്ചെല്ലാത്തതുകൊണ്ടുള്ള സംശയമാണ് ഇത്. ഇവ്വിഷയകമായ ഏറ്റവും ശാസ്ത്രീയവും നീതിയിലധിഷ്ഠിതവുമായ നിയമമാണ് ഖുര്‍ആന്‍ പ്രദാനം ചെയ്യുന്നതെന്നുള്ളതാണ് സത്യം.അനന്തരാവകാശത്തെക്കുറിച്ച് വിശദീകരിക്കുന്നത് സൂറത്തിന്നിസാഇലെ 11, 12 സൂക്തങ്ങളാണ്. പിതാവോ സന്താനങ്ങളോ ഇല്ലാത്ത വ്യക്തിയുടെ അനന്തരാവകാശത്തെക്കുറിച്ച് ഇതേ അധ്യായത്തിലെ അവസാനത്തിലെ സൂക്തത്തിലും (176) വിവരിക്കുന്നുണ്ട്. ഇവയില്‍നിന്നും പ്രവാചകചര്യയില്‍നിന്നുമാണ് ദായധനത്തെക്കുറിച്ച ഇസ്‌ലാമിക സമീപനം ഉരുത്തിരിഞ്ഞിരിക്കുന്നത്. പ്രസ്തുത സമീപനത്തിന്റെ അടിസ്ഥാന തത്ത്വങ്ങള്‍ താഴെ പറയുന്നവയാണ്.

1.ഒരാളുടെ സ്വത്തില്‍ അയാളുടെ ജീവിതകാലത്ത് അയാള്‍ക്കല്ലാതെ മറ്റൊരാള്‍ക്കും യാതൊരവകാശവുമില്ല.

2.അയാളുടെ ജീവിതകാലത്ത് മരിച്ചുപോയ പിന്തുടര്‍ച്ചക്കാര്‍ക്ക് അയാളുടെ അനന്തരസ്വത്തില്‍ അവകാശമൊന്നുമില്ല. (അനന്തര സ്വത്ത് രൂപപ്പെടുന്നതുതന്നെ അയാള്‍ മരിക്കുന്നതോടുകൂടിയാണല്ലോ. അതിനുമുമ്പ് അത് അയാളുടെ സ്വത്തു മാത്രമാണ്. അനന്തരസ്വത്തല്ല).

3.അയാളുടെ മരണസമയത്ത് ജീവിച്ചിരിക്കുന്ന പിന്തുടര്‍ച്ചക്കാര്‍ക്ക് മാത്രമേ അനന്തര സ്വത്തില്‍ അവകാശമുണ്ടാവുകയുള്ളൂ.

4.അനന്തരാവകാശത്തിന്റെ അടിസ്ഥാനമായി അംഗീകരിക്കപ്പെട്ടിട്ടുള്ളത് അടുത്ത ബന്ധമാണ്. വിവാഹബന്ധവും രക്തബന്ധവും ഇതില്‍ ഉള്‍പ്പെടുന്നു.

5.അയാളുടെ അടുത്ത ബന്ധുക്കള്‍ അതേ താവഴിയിലെ അകന്ന ബന്ധുക്കളുടെ അവകാശം തടയും. (മാതാപിതാക്കള്‍,ഭാര്യാഭര്‍ത്താക്കന്മാര്‍,പുത്രപുത്രിമാര്‍ എന്നിവരാണ് അടുത്ത ബന്ധുക്കള്‍. ഇവരുടെ സാന്നിധ്യത്തില്‍ അതേ താവഴിയിലുള്ള മറ്റാരും അവകാശികളായി തീരുകയില്ല.)

6.വ്യക്തിയുടെ സാമ്പത്തികാവസ്ഥയോ അവശതയോ അല്ല,മരിച്ചയാളുമായുള്ള ബന്ധത്തിന്റെ സ്വഭാവമാണ് ദായധനവിഭജനത്തിലെ അംഗീകൃത മാനദണ്ഡം.

7.മരിച്ചയാളുടെ ബന്ധുക്കളായി ഏറ്റവും അടുത്ത കണ്ണികളില്ലെങ്കില്‍ അവകാശം തൊട്ടടുത്ത കണ്ണികളിലേക്ക് നീങ്ങുന്നു. പിതാവില്ലെങ്കില്‍ പിതാമഹനും പുത്രനില്ലെങ്കില്‍ പൗത്രനും പിന്തുടര്‍ച്ചാവകാശം ലഭിക്കുന്നത് ഇതുകൊണ്ടാണ്.

ഈ അടിസ്ഥാനതത്ത്വങ്ങളുടെ വെളിച്ചത്തില്‍ പിതാവ് ജീവിച്ചിരിക്കെ മരണപ്പെട്ട മക്കളുടെ മക്കള്‍ക്ക് അദ്ദേഹത്തിന്റെ മരണസമയത്ത് ജീവിച്ചിരിക്കുന്ന മക്കളുണ്ടെങ്കില്‍ നിയമപ്രകാരം ദായധനം ലഭിക്കുകയില്ല. കുടുംബ ശൃംഖലയുടെ ആദ്യത്തെ കണ്ണിയായ മക്കള്‍ ജീവിച്ചിരിക്കുന്നതിനാല്‍ രണ്ടാമത്തെ കണ്ണിയായ പൗത്രന്മാരിലേക്ക് അനന്തരാവകാശം എത്തുന്നില്ലെന്നതാണ് ഇതിന് കാരണം. ഇത് അനീതിയല്ലേ എന്നാണ് ചോദ്യം.

ദായധനത്തിന്റെ വിതരണത്തില്‍ സമ്പൂര്‍ണ നീതി നടപ്പാക്കാന്‍ നിയമങ്ങളെക്കൊണ്ട് കഴിയുമോ എന്ന മറുചോദ്യമാണ് ഇതിനുള്ള ആദ്യത്തെ മറുപടി. ഒരു നിയമവ്യവസ്ഥക്കും ഇക്കാര്യത്തില്‍ നൂറു ശതമാനം നീതി നടപ്പാക്കാന്‍ കഴിയില്ലെന്നതാണ് വസ്തുത. ഈ വസ്തുതക്ക് ഉപോദ്ബലകമായി അവതരിപ്പിക്കാവുന്ന ഏതാനും മോഡലുകള്‍ കാണുക.

1.പരേതന് രണ്ടു മക്കള്‍. ഒരാള്‍ വികലാംഗന്‍. മറ്റെയാള്‍ അരോഗദൃഢഗാത്രന്‍. ഒന്നാമത്തെയാള്‍ക്ക് അധ്വാനിക്കാനാവില്ല. രണ്ടാമന് അധ്വാനിച്ച് പണം സമ്പാദിക്കാം. എങ്ങനെ സ്വത്ത് ഓഹരിവെക്കും?അധ്വാനിക്കാന്‍ കഴിയുന്നവന് കുറച്ചും കഴിയാത്തവന് കൂടുതലുമായി ഓഹരിവെക്കുന്നതാണ് നീതി. ഏതെങ്കിലും വ്യവസ്ഥകള്‍ക്ക് ഈ നീതിയെ നിയമമാക്കുവാന്‍ കഴിയുമോ?

2.പരേതന് മൂന്നു മക്കള്‍. മൂത്തയാള്‍ നാല്‍പതുകാരന്‍. കച്ചവടക്കാരന്‍. പിതാവിന്റെ കച്ചവടത്തില്‍ സഹകാരിയായി തുടങ്ങി സ്വന്തമായി കച്ചവടത്തിലെത്തിച്ചേര്‍ന്നയാള്‍.രണ്ടാമത്തെയാള്‍ ഭിഷഗ്വരന്‍. പിതാവിന്റെ പണം ചെലവഴിച്ചുകൊണ്ടാണയാള്‍ പഠിച്ചത്. ഇന്നയാള്‍ പണം വാരുന്നു. മൂന്നാമന്‍ പതിനെട്ടുകാരന്‍. വിദ്യാര്‍ഥി. എവിടെയെങ്കിലുമെല്ലാം എത്തുന്നതിനുമുമ്പ് പിതാവ് മരിച്ചുപോയി. എങ്ങനെ സ്വത്ത് ഓഹരിവെക്കണം? മൂത്തവര്‍ രണ്ടും സ്വയം സമ്പാദിക്കുന്നവരാണ്. പിതാവിന്റെ സ്വത്തില്‍നിന്നാണവര്‍ സമ്പാദ്യം തുടങ്ങിയത്. ഇളയവനാകട്ടെ പിതാവ് ജീവിച്ചിരിക്കുമ്പോള്‍ അദ്ദേഹത്തിന്റെ സ്വത്തില്‍നിന്ന് കാര്യമായി ഒന്നും ലഭിച്ചില്ല. അപ്പോള്‍ ദായധനമെങ്കിലും ഇളയപുത്രന് കൂടുതല്‍ ലഭിക്കണമെന്നതാണ് നീതി. പക്ഷേ, ഈ നീതി നടപ്പാക്കുന്ന രീതിയില്‍ ദായധനം ഓഹരി വെക്കുന്നതിനാവശ്യമായ നിയമം ഉണ്ടാക്കുവാന്‍ കഴിയുമോ?

3.പരേതന് മൂന്നു മക്കള്‍. ഒരാള്‍ സമര്‍ഥന്‍. പണം കൊണ്ട് പണം വാരാന്‍ കഴിവുള്ളവന്‍. രണ്ടാമന്‍ സാമൂഹിക സേവകന്‍. പണം ചെലവഴിച്ച് മറ്റുള്ളവരുടെ ക്ഷേമത്തിനു വേണ്ടി പ്രവര്‍ത്തിക്കുന്നവന്‍. അവസാനത്തെയാള്‍ മഠയന്‍. കിട്ടിയ പണം സൂക്ഷിച്ചുവെച്ച് അതില്‍നിന്ന് മാത്രമായി ചെലവ് കണ്ടെത്തുന്നവന്‍. മൂന്നു പേര്‍ക്കും പത്തു രൂപ വീതം നല്‍കിയാല്‍ ഒന്നാമന്‍ അത് ഇരുപത് രൂപയാക്കും. രണ്ടാമന്‍ തനിക്കും അയല്‍ക്കാരനായ ദരിദ്രനും കൂടി ഒരു നേരത്തെ ഭക്ഷണം ഒരുക്കും,മൂന്നാമന്‍ രണ്ടു നേരത്തെ ഭക്ഷണം കഴിക്കും. ഇവര്‍ക്ക് മൂന്നുപേര്‍ക്കും ദായധനം ഒരേ പോലെ വീതിക്കുകയാണോ വേണ്ടത്? നീതിയതല്ല. പക്ഷേ, പ്രസ്തുത നീതി ഒരു നിയമക്രമത്തിലൂടെ നടപ്പാക്കുക പ്രായോഗികമല്ലെന്നു മാത്രം.

ദായധനത്തിന്റെയും മറ്റു സാമ്പത്തിക പ്രശ്‌നങ്ങളുടെയും കാര്യത്തില്‍ കേവല നിയമങ്ങള്‍ക്ക് എല്ലാ അര്‍ഥത്തിലുമുള്ള സമ്പൂര്‍ണ നീതി നടപ്പിലാക്കുവാന്‍ കഴിയുകയില്ലെന്ന വസ്തുതയാണ് ഇവിടെ വ്യക്തമാകുന്നത്. ഇസ്‌ലാം ഇത്തരം പ്രശ്‌നങ്ങളില്‍ കേവല നിയമങ്ങളെ മാത്രം ആശ്രയിച്ചുകൊണ്ടല്ല പരിഹാരങ്ങള്‍ നിര്‍ദേശിക്കുന്നത്. മനുഷ്യരുടെ ധര്‍മബോധത്തെ ഉത്തേജിപ്പിക്കുകയും പ്രയാസങ്ങളനുഭവിക്കുന്നവരോട് കരുണ കാണിക്കുവാന്‍ പ്രേരിപ്പിക്കുകയും ചെയ്തുകൊണ്ടാണ് ഇത്തരം കാര്യങ്ങളില്‍ നീതി നടപ്പാക്കപ്പെടുന്ന സാഹചര്യം സൃഷ്ടിക്കുന്നത്.

എന്തുകൊണ്ട് അനാഥ പൗത്രന് സ്വത്തവകാശം നല്‍കിക്കൊണ്ട് ഒരു നിയമം ഉണ്ടാക്കിക്കൂടാ? അത്തരം ഒരു നിയമം ഉണ്ടാക്കുന്നത് ദായധനക്രമത്തില്‍ ഇസ്‌ലാം മുന്നോട്ടുവെക്കുന്ന അടിസ്ഥാനാശയങ്ങളെയെല്ലാം തകര്‍ത്തുകളയുമെന്നതാണ് യാഥാര്‍ഥ്യം.

അനാഥപൗത്രന് സ്വത്തവകാശം നല്‍കുന്നതിന് ഒരു മാര്‍ഗമേയുള്ളൂ. മരിച്ച മകന്‍ ജീവിച്ചിരിക്കുന്നതായി സങ്കല്‍പിക്കുക. അയാളുടെ സന്താനങ്ങളെ മരിച്ചയാളുടെ പ്രതിനിധികളാക്കി തല്‍സ്ഥാനത്ത് പ്രതിഷ്ഠിക്കുക. എന്നിട്ട് മരിച്ച മകനു നല്‍കേണ്ട സ്വത്ത് അയാളുടെ മക്കള്‍ക്ക് വിഭജിച്ച് നല്‍കുക. ഒരാള്‍ക്ക് രണ്ടു മക്കളുണ്ട്, മൂത്ത മകന് മൂന്നും ഇളയവന് രണ്ടും മക്കള്‍ വീതമുണ്ട്. പിതാവ് ജീവിച്ചിരിക്കെ മൂത്തയാള്‍ മരിച്ചു. പിതാവ് മരിക്കുമ്പോള്‍ ഇളയമകനേയുള്ളൂ. അനന്തര സ്വത്തായി ആയിരം രൂപയുണ്ട്. ഇളയമകന്‍ അഞ്ഞൂറ് രൂപയെടുക്കുക. ബാക്കി അഞ്ഞൂറു രൂപ മരിച്ച മൂത്തമകന്റെ മൂന്നു മക്കള്‍ക്കും വിഭജിച്ച് നല്‍കുക. ഇതാണ് പൊതുവായി നിര്‍ദേശിക്കപ്പെടുന്ന പ്രാതിനിധ്യതത്ത്വം.

ഇസ്‌ലാമികദായക്രമത്തില്‍ ഈ പ്രാതിനിധ്യതത്ത്വം എന്തുമാത്രം പ്രായോഗികമാണ്? അനന്തരാവകാശ വ്യവസ്ഥയിലെവിടെയെങ്കിലും പ്രാതിനിധ്യതത്ത്വം അംഗീകരിക്കുകയാണെങ്കില്‍ അതിന്റെ എല്ലാ വശങ്ങളിലും അതു നടപ്പാക്കേണ്ടിവരുമെന്ന വസ്തുത മറക്കരുത്. ഇത് എത്രമാത്രം ശരിയാവും? പരിശോധിക്കുക. ഏതാനും ചില കാര്യങ്ങള്‍ കാണുക:

1.ഭാര്യയുടെ അനന്തര സ്വത്തില്‍ ഭര്‍ത്താവിന് അവകാശമുണ്ട്. സന്തതിയുണ്ടെങ്കില്‍ നാലിലൊന്നും ഇല്ലെങ്കില്‍ പകുതിയുമാണ് അയാളുടെ അവകാശം. ഭാര്യ മരിക്കുന്നതിനു മുമ്പ് ഭര്‍ത്താവ് മരിച്ചുവെന്ന് കരുതുക. പ്രാതിനിധ്യതത്ത്വം അംഗീകരിക്കുകയാണെങ്കില്‍ ഭാര്യയുടെ അനന്തര സ്വത്തില്‍ ഭര്‍ത്താവിന്റെ പിതാവ്,മാതാവ്, മറ്റു ഭാര്യമാരിലുള്ള മക്കള്‍ എന്നിവര്‍ക്കെല്ലാം അവകാശമുണ്ടായിരിക്കും.

2.ഭര്‍ത്താവിന്റെ അനന്തരസ്വത്തില്‍ ഭാര്യക്കുള്ള അവകാശത്തിന്റെ സ്ഥിതിയും ഇതുതന്നെ. ഭാര്യയുടെ മരണശേഷമാണ് ഭര്‍ത്താവിന്റെ മരണമെങ്കില്‍ അവളുടെ വിഹിതം അവളുടെ മാതാവ്,പിതാവ് തുടങ്ങിയ ബന്ധുക്കള്‍ക്ക് നല്‍കേണ്ടിവരും.

3.മക്കളുടെ സ്വത്തില്‍ പിതാക്കന്മാര്‍ക്ക് അവകാശമുണ്ട്. മകന്റെ മുമ്പ് പിതാവ് മരണപ്പെട്ടു എന്ന് കരുതുക. പിതാവിന് വേറെയും മക്കളുണ്ട്താനും. മകന് മക്കളുണ്ടെങ്കിലും അയാളുടെ മരണശേഷം പിതാവ് ജീവിച്ചിരിക്കുന്നുവെങ്കില്‍ അയാള്‍ക്ക് ലഭിക്കുമായിരുന്ന അനന്തരാവകാശ സ്വത്ത് അയാളുടെ അടുത്ത ബന്ധുക്കള്‍ക്ക് നല്‍കേണ്ടിവരും.

4.മാതാക്കള്‍ക്ക് മക്കളുടെ സ്വത്തിലുള്ള അവകാശത്തിന്റെ അവസ്ഥയും ഇതുതന്നെ. മകനുമുമ്പ് മാതാവ് മരിച്ചിട്ടുണ്ടെങ്കിലും അവരുടെ അടുത്ത ബന്ധുക്കള്‍ക്ക് മകന്റെ സ്വത്തില്‍ അവകാശമുണ്ടെന്ന അവസ്ഥയാണ് പ്രാതിനിധ്യതത്ത്വം അംഗീകരിച്ചാല്‍ വന്നുചേരുക.

പ്രാതിനിധ്യതത്ത്വം അംഗീകരിക്കുന്നത് ദായക്രമത്തില്‍ അസന്തുലിതത്വം ഉണ്ടാകുന്നതിന് നിമിത്തമാകും. ഒരാള്‍ക്ക് രണ്ടു മക്കള്‍,രണ്ടുപേരും മരിച്ചു. ഒരു മകന് ഒരു പുത്രനും മറ്റേയാള്‍ക്ക് രണ്ടു പുത്രന്മാരുമാണുള്ളത്. പിതാമഹന്റെ സ്വത്തില്‍ ഇസ്‌ലാമികദായധന വിതരണക്രമപ്രകാരം മൂന്നു പൗത്രന്മാര്‍ക്കും ഒരേ അവകാശമാണുണ്ടാവുക. പ്രാതിനിധ്യ സിദ്ധാന്തം അംഗീകരിക്കുകയാണെങ്കില്‍ സഹോദരനില്ലാത്ത പൗത്രന് ലഭിക്കുന്നതിന്റെ പകുതി മാത്രമേ അയാളുടെ പിതൃവ്യന്റെ മക്കളില്‍ ഓരോരുത്തര്‍ക്കും ലഭിക്കുകയുള്ളൂ. ഒരാളുമായി ഒരേബന്ധം പുലര്‍ത്തുന്ന വ്യത്യസ്ത വ്യക്തികള്‍ക്ക് വ്യത്യസ്ത രൂപങ്ങളില്‍ അവകാശം നല്‍കുന്നത് ദായക്രമത്തില്‍ അസന്തുലിതത്വം സൃഷ്ടിക്കുമെന്ന് പറയേണ്ടതില്ല.

ഒരു നിയമമെന്ന നിലയ്ക്ക് പ്രാതിനിധ്യതത്ത്വം അംഗീകരിക്കുമ്പോഴുള്ള പ്രായോഗിക പ്രശ്‌നങ്ങളാണ് മുകളില്‍ വിവരിച്ചത്. ഇതുകൊണ്ടായിരിക്കാം മക്കള്‍ ജീവിച്ചിരിക്കുമ്പോള്‍ പേരക്കുട്ടികള്‍ക്ക് സ്വത്തില്‍ അവകാശം നല്‍കുന്ന രീതിയിലുള്ള ഒരു നിയമം ഖുര്‍ആന്‍ കൊണ്ടുവരാതിരുന്നത്. ഒരു നിയമം കൊണ്ടുവരുമ്പോള്‍ അതിന്റെ പ്രായോഗികമായ പ്രശ്‌നങ്ങളെല്ലാം പരിഹരിക്കേണ്ടതുണ്ടല്ലോ. പൗത്രന് സ്വത്തവകാശം ഒരു നിയമനിര്‍മാണം വഴി കൊണ്ടുവരികയാണെങ്കില്‍, ഇസ്‌ലാമിക ദായക്രമത്തിന്റെ അടിത്തറ തകരുകയും അത് അപ്രായോഗികമായി തീരുകയും ചെയ്യുമായിരുന്നു. നിയ മങ്ങളുടെ വരുംവരായ്കകളെക്കുറിച്ച് വ്യക്തമായി അറിയുന്നവനാണ് ഖുര്‍ആന്‍ അവതരിപ്പിച്ചത് എന്ന സത്യമാണ് ഇവിടെ നമുക്ക് വ്യക്തമാകുന്നത്.

ഭര്‍ത്താവ് മരിച്ച സ്ത്രീ നാലുമാസവും പത്തു ദിവസവും ദുഃഖമാചരിക്കണമെന്ന് ഖുര്‍ആന്‍ അനുശാസിക്കുന്നുണ്ട്.”നിങ്ങളാരെങ്കിലും തങ്ങളുടെ ഭാര്യമാരെ വിട്ടേച്ചുകൊണ്ട് മരണപ്പെടുകയാണെങ്കില്‍ അവര്‍ തങ്ങളുടെ കാര്യത്തില്‍ നാലു മാസവും പത്തുദിവസവും കാത്തിരിക്കേണ്ടതാണ്. എന്നിട്ട് അവരുടെ ആ അവധിയെത്തിയാല്‍ തങ്ങളുടെ കാര്യത്തില്‍ അവര്‍ മര്യാദയനുസരിച്ചു പ്രവര്‍ത്തിക്കുന്നതില്‍ നിങ്ങള്‍ക്ക് കുറ്റമൊന്നുമില്ല” (2:224).എന്തിനാണത്?

വിധവയുടെ ദുഃഖാചരണത്തിന് പിന്നിൽ രണ്ടു ഉദ്ദേശ്യങ്ങളാണുള്ളത്. തന്റെ ജീവിത പങ്കാളിയുടെ വേര്‍പാടില്‍ ദുഃഖാചരണം നടത്തുകയും മറ്റൊരു വിവാഹത്തിലേർപ്പെടുവാനുള്ള മാനസികപക്വത നേടിയെടുക്കുകയുമാണ് ഒന്ന്. അന്തരിച്ച ഭര്‍ത്താവില്‍നിന്ന് താന്‍ ഗര്‍ഭം ധരിച്ചിട്ടുണ്ടോ എന്ന സംശയം ദുരീകരിക്കുകയാണ് മറ്റൊന്ന്.

ദുഃഖാചരണകാലത്ത് അവള്‍ ചെയ്യേണ്ടതെന്താണ്? ദുഃഖാചരണകാലത്ത് അവള്‍ വിവാഹിതയാകാന്‍ പാടില്ല. വിവാഹാലോചനകളും ഇക്കാലത്ത് വിലക്കപ്പെട്ടിരിക്കുന്നു. അഴകും മോടിയും കൂട്ടി പുരുഷന്മാരെ ആകര്‍ഷിക്കുകയോ സ്വമനസ്സില്‍ ലൈംഗികതൃഷ്ണ വളര്‍ത്തുകയോ ചെയ്തുകൂടാ. വര്‍ണശബളമായ ആടയാഭരണങ്ങള്‍ ധരിക്കുകയും ചായവും സുറുമയും ഉപയോഗിക്കുകയും സുഗന്ധദ്രവ്യങ്ങള്‍ പുരട്ടുകയും ചെയ്യുന്നതില്‍നിന്ന് ഇക്കാലത്ത് അവള്‍ വിലക്കപ്പെട്ടിരിക്കുന്നു. അത്യാവശ്യകാര്യങ്ങള്‍ക്കായി പുറത്തുപോകുന്നതിനെയോ മാന്യവും വൃത്തിയുള്ളതുമായ വസ്ത്രം ധരിക്കുന്നതിനെയോ നിരോധിച്ചതായി കാണാന്‍ കഴിയില്ല. ചുരുക്കത്തില്‍, ലൈംഗിക ചിന്തയുണ്ടാക്കുന്ന എല്ലാ കാര്യങ്ങളില്‍നിന്നും അകന്നുനില്‍ക്കാന്‍ ദുഃഖാചരണകാലത്ത് സ്ത്രീ ബാധ്യസ്ഥയാണ്.

ഭര്‍ത്താവ് മരിച്ച് നാലു മാസവും പത്തു ദിവസവും കഴിഞ്ഞാല്‍- ഗര്‍ഭിണിയാണെങ്കില്‍ പ്രസവിച്ചാല്‍- അവള്‍ക്ക് ഇഷ്ടമുള്ളതുപോലെ ചെയ്യാവുന്നതാണ്. ഒന്നുകില്‍ പുനര്‍വിവാഹം ചെയ്യാം. അല്ലെങ്കില്‍ തല്‍ക്കാലം വിവാഹം വേണ്ടെന്നു വെക്കാം. എല്ലാം അവളുടെ ഇഷ്ടത്തിന് വിടേണ്ടതാണ്.

അജ്ഞാന കാലത്ത് അറേബ്യയില്‍ വിധവകള്‍ ഒരു വര്‍ഷം ദുഃഖാചരണം നടത്തുമായിരുന്നു. അങ്ങേയറ്റം മലിനമായി വസ്ത്രം ധരിച്ച്, കുളിക്കുകയോ വൃത്തിയാവുകയോ ചെയ്യാതെയുള്ള ദുഃഖാചരണം. ഇതില്‍നിന്ന് പരിവര്‍ത്തനം ഉണ്ടാക്കുകയാണ് ഇസ്‌ലാം ചെയ്തത്.

ഭര്‍ത്താവ് മരിച്ച ഹൈന്ദവ സ്ത്രീ എന്തു ചെയ്യണം?

മനുസ്മൃതിയുടെ വിധി കാണുക:

കാമം തുക്ഷ പയേ ദ്ദേഹം പുഷ്പ മൂല ഫലൈഃ ശുഭൈഃ

ന തു നാമാഭി ഗൃഹ്ണീ യാത്പത്യൗ പ്രേത പരസ്യതു

ആസീതാ മരണാല്‍ ക്ഷാന്താ നിയതാ ബ്രഹ്മചാരിണീ

യോ ധര്‍മ്മ ഏകപത്‌നീ നാം കാംക്ഷന്തീ തമനുത്തമം (5:157, 158).

(ഭര്‍ത്താവ് മരിച്ച ശേഷം പരിശുദ്ധമായ കിഴങ്ങ്, ഫലം, പുഷ്പം മുതലായ ആഹാരങ്ങള്‍കൊണ്ട് ദേഹത്തിന് ക്ഷതം വരുത്തി കാലം നയിക്കേണ്ടതാണ്. കാമവികാരോദ്ദേശ്യത്തിന്മേല്‍ മറ്റൊരു പുരുഷന്റെ പേരുപറയരുത്. ഭര്‍ത്താവ് മരിച്ച ശേഷം ജീവാവസാനം വരെ സഹനശീലയായി പരിശുദ്ധയായി ബ്രഹ്മധ്യാനമുള്ളവളായും മധുമാംസഭക്ഷണം ചെയ്യാത്തവളായും ഉത്കൃഷ്ടയായ പതിവ്രതയുടെ ധര്‍മത്തെ ആഗ്രഹിക്കുന്നവളായും ഇരിക്കേണ്ടതാകുന്നു).

ഇത് മനുസ്മൃതിയുടെ വിധി. ഇന്ത്യയില്‍ നിലവിലുണ്ടായിരുന്ന അവസ്ഥ ഇതിലും ഭീകരമായിരുന്നു. ഭര്‍ത്താക്കന്മാര്‍ മരിച്ചാല്‍ അവരുടെ ചിതയില്‍ ചാടി മരിക്കണമെന്ന് സ്ത്രീ നിര്‍ദേശിക്കപ്പെട്ടിരുന്നു. ക്രൂരമായ സതി സമ്പ്രദായം! അത് അനുഷ്ഠിക്കുവാന്‍ വിസമ്മതിക്കുന്ന വിധവകള്‍ തലമൊട്ടയടിച്ച് സമൂഹത്തില്‍ ഒറ്റപ്പെട്ടു കഴിയണമായിരുന്നു. ശൈശവ വിവാഹത്തിന് ശേഷം വിധവകളാകുന്ന ആറും ഏഴും വയസ്സ് പ്രായമുള്ള പെണ്‍കിടാങ്ങള്‍പോലും തലമൊട്ടയടിച്ച് ജീവിതകാലം മുഴുവന്‍ ഭിക്ഷുണികളായി കഴിഞ്ഞുകൂടണമെന്നായിരുന്നു നിയമം. ഇവര്‍ക്ക് അനുവദിക്കപ്പെട്ടിരുന്നതോ ഒരു നേരത്തെ ഭക്ഷണം മാത്രം!

വിധവകളെ പുനര്‍വിവാഹത്തില്‍നിന്ന് ഖുര്‍ആന്‍ വിലക്കുന്നില്ല. അവര്‍ നാലു മാസവും പത്തു ദിവസവും കാത്തിരിക്കണമെന്നു മാത്രമാണ് അനുശാസിക്കുന്നത്. ഈ കാത്തിരിപ്പാകട്ടെ തികച്ചും ശാസ്ത്രീയവും സ്ത്രീക്ക് ഗുണം ചെയ്യുന്നതുമാണുതാനും. പുരുഷനിൽ നിന്ന് വ്യത്യസ്തമായി ഇണയായി ഒരാളെ മാത്രമേ സ്ത്രീക്ക് തന്റെ മനസ്സിൽ ഉൾക്കൊള്ളാൻ കഴിയൂ. പുതിയൊരു ദാമ്പത്യം വിജയകരമായാവണമെങ്കിൽ മനസ്സിലുള്ള ഇണയെ താഴെയിറക്കാൻ കഴിയണം. ഏറെക്കാലം കൂടെ ജീവിച്ച ഇണയെ മനസ്സില്നിന്നിറക്കി വെക്കാനും മറ്റൊരു ഇണയെ സ്വീകരിക്കുവാൻ മാനസികമായി ഒരുങ്ങാനും ദുഃഖാചരണം വഴി അവൾക്ക് കഴിയും.

ഭർത്താവിന്റെ മരണം നടന്ന ഉടനെ സ്ത്രീ വിവാഹിതയാവുകയും  ഗര്‍ഭിണിയായി അവര്‍ക്ക് കുഞ്ഞുണ്ടാവുകയുമാണെങ്കില്‍ അതിന്റെ പിതൃത്വത്തെക്കുറിച്ച് സംശയം ഉണ്ടാകുവാന്‍ സാധ്യതയുണ്ട്. ഈ സംശയം തന്റെ കുടുംബഭദ്രതയും മനസ്സമാധാനവും തകര്‍ക്കുന്നതിലേക്ക് നയിച്ചേക്കും. ഖുര്‍ആന്‍ പറഞ്ഞ പ്രകാരം കാത്തിരുന്ന ശേഷം പുനര്‍വിവാഹം ചെയ്യുന്ന സ്ത്രീ ഗര്‍ഭിണിയാകുമ്പോള്‍ ഈ പ്രശ്‌നം ഉദിക്കുന്നില്ല. അത് രണ്ടാം ഭര്‍ത്താവിന്റെ കുഞ്ഞുതന്നെയാണെന്ന് ഉറപ്പിക്കാനാവും. വിധവയുടെ ദുഃഖാചരണം സംബന്ധിച്ച ഖുര്‍ആനിക ഇദ്ദയുടെ നിയമവും സ്ത്രീക്ക് അനുഗുണമാണെന്നും പ്രയാസപ്പെടുത്താതിരിക്കാനുള്ളതാണെന്നുമുള്ള വസ്തുതയാണ് ഇവിടെ വ്യക്തമാകുന്നത്.

മുന്നോട്ടു പോകാൻ കഴിയാത്ത വിധം പരാജയമാണ് ദാമ്പത്യജീവിതമെങ്കിൽ വിവാഹമോചനത്തിന് ആവശ്യപ്പെടാൻ സ്ത്രീക്കും അവകാശമുണ്ട്. സ്ത്രീയുടെ വിവാഹമോചനം രണ്ടു തരമാണ്. ഖുല്‍ഉം ഫസ്ഖും.

തന്റെ ഭര്‍ത്താവിനെ വെറുക്കുകയും അയാളോടൊപ്പം ജീവിക്കുവാന്‍ ഇഷ്ടപ്പെടാതിരിക്കുകയും ചെയ്യുന്ന സ്ത്രീക്ക് അയാളോട് വിവാഹമോചനത്തിന് ആവശ്യപ്പെടാവുന്നതാണ്. ഇതാണ് ‘ഖുല്‍അ്‘.ഭര്‍ത്താവില്‍നിന്ന് ലഭിച്ച വിവാഹമൂല്യം തിരിച്ചുകൊടുക്കണമെന്നുള്ളതാണ് ‘ഖുല്‍ഇ‘നുള്ള നിബന്ധന. വിവാഹം വഴി ഭാര്യക്ക് ലഭിച്ച സമ്പത്ത് തിരിച്ചുകൊടുക്കണമെന്നര്‍ഥം. ഇക്കാര്യം വിവരിക്കുന്ന ഖുര്‍ആന്‍ സൂക്തം നോക്കുക: ”അങ്ങനെ അവര്‍ക്ക് (ദമ്പതികള്‍ക്ക്) അല്ലാഹുവിന്റെ നിയമപരിധികള്‍ പാലിക്കുവാന്‍ കഴിയില്ലെന്ന് നിങ്ങള്‍ക്ക് ഉത്കണ്ഠ തോന്നുകയാണെങ്കില്‍ അവള്‍ വല്ലതും വിട്ടുകൊടുത്ത് സ്വയം മോചനം നേടുന്നതിന് അവര്‍ ഇരുവര്‍ക്കും കുറ്റമില്ല. (2:229).

‘ഖുല്‍ഇ‘നുള്ള നിബന്ധനകള്‍ താഴെ പറയുന്നവയാണ്.

ഒന്ന്: ത്വലാഖിനെപ്പോലെതന്നെ അനിവാര്യമായ സാഹചര്യങ്ങളില്ലാതെ ഖുല്‍അ് ചെയ്യാന്‍ പാടില്ലാത്തതാകുന്നു. പ്രവാചകന്‍ (ﷺ) പറഞ്ഞു: ”പ്രയാസമുണ്ടാവുമ്പോഴല്ലാതെ ഭര്‍ത്താവില്‍നിന്ന് വിവാഹമോചനം ആവശ്യപ്പെടുന്ന സ്ത്രീക്ക് സ്വര്‍ഗത്തിന്റെ സുഗന്ധം പോലും നിഷിദ്ധമാണ്” (അബൂദാവൂദ്, തിര്‍മുദി).

രണ്ട്: സ്ത്രീ ഖുല്‍അ് ആവശ്യപ്പെട്ടാല്‍ അവളെ മോചിപ്പിക്കേണ്ടത് പുരുഷന്റെ ബാധ്യതയാണ്.

മൂന്ന്: താന്‍ നല്‍കിയ വിവാഹമൂല്യം പൂര്‍ണമായോ ഭാഗികമായോ ആവശ്യപ്പെടാന്‍ പുരുഷന് അവകാശമുണ്ട്. വിവാഹമൂല്യത്തില്‍ കവിഞ്ഞ യാതൊന്നും ആവശ്യപ്പെടാവതല്ല.

നാല്: താന്‍ ആവശ്യപ്പെട്ട തുക നല്‍കുന്നതോടുകൂടി ഖുല്‍അ് സാധുവായിത്തീരുന്നു. അഥവാ ആ സ്ത്രീ പുരുഷന്റെ ഭാര്യയല്ലാതായിമാറുന്നു.

ഇത്തരം വിവാഹമോചനങ്ങള്‍ പ്രവാചകന്റെ(ﷺ) കാലത്തു നടന്നതായി കാണാനാവും. താന്‍ ഇഷ്ടപ്പെടാത്ത ഭാര്യയെക്കൊണ്ട് ഖുല്‍അ് ചെയ്യിക്കുന്നതിനുവേണ്ടി അവളെ പ്രയാസപ്പെടുത്തുന്ന സമ്പ്രദായം നിലവിലുണ്ടായിരുന്നു. താന്‍ നല്‍കിയ വിവാഹമൂല്യം തിരിച്ചുവാങ്ങുന്നതിനുവേണ്ടിയായിരുന്നു അത്. ഖുര്‍ആന്‍ ഇൗ സമ്പ്രദായത്തെ ശക്തിയായി വിലക്കുന്നുണ്ട്.

”നിങ്ങള്‍ കൊടുത്തിട്ടുള്ളതില്‍ ഒരു ഭാഗം തട്ടിയെടുക്കാനായി നിങ്ങള്‍ അവരെ മുടക്കിയിടുകയും ചെയ്യരുത്” (4:19).

സ്ത്രീയുടെ രണ്ടാമത്തെ വിവാഹമോചന രീതിയാണ് ‘ഫസ്ഖ്‘.ഭാര്യയുടെ അവകാശങ്ങള്‍ നിഷേധിക്കുകയും അതോടൊപ്പം വിവാഹമോചനം നല്‍കാതിരിക്കുകയും ചെയ്യുന്ന പുരുഷന്മാരില്‍നിന്ന് ന്യായാധിപന്റെ സഹായത്തോടെ നേടുന്ന വിവാഹമോചനമാണിത്. ഭര്‍ത്താവിന് സന്താനോല്‍പാദനശേഷി ഇല്ലെന്ന് തെളിയുക,ലൈംഗികബന്ധത്തിന് സാധിക്കാതിരിക്കുക, അവിഹിത വേഴ്ചകളില്‍ മുഴുകുക, ക്രൂരമായി പെരുമാറുക, തന്നെ അധാര്‍മിക വൃത്തിക്ക് നിര്‍ബന്ധിക്കുക, ജീവിതത്തിന്റെ അടിസ്ഥാനാവശ്യങ്ങള്‍ നിഷേധിക്കുക,തന്റെ സ്വത്തുക്കള്‍ അന്യായമായി ഉപയോഗിക്കുക, ഒന്നിലധികം ഭാര്യമാരുള്ളയാളാണെങ്കില്‍ തന്നോട് നീതിപൂര്‍വം വര്‍ത്തിക്കാതിരിക്കുക, തുടങ്ങിയ അവസരങ്ങളില്‍ ഭാര്യക്ക് ന്യായാധിപന്‍ മുഖേന വിവാഹബന്ധം വേര്‍പെടുത്താവുന്നതാണ്. ഇതാണ് ഫസ്ഖ്. തന്റെ അനുവാദമില്ലാതെ രക്ഷാധികാരികള്‍ വിവാഹം ചെയ്തുകൊടുത്താലും ഭര്‍ത്താവ് എവിടെയാണെന്നറിയാത്ത സ്ഥിതി ഉണ്ടെങ്കിലും ഭാര്യക്ക് ഫസ്ഖ് ചെയ്യാവുന്നതാണ്.

ഫസ്ഖ് ചെയ്യുന്നത് ന്യായാധിപനിലൂടെയായിരിക്കണമെന്നുള്ളതാണ് അതിനുള്ള നിബന്ധന. ഭാര്യ ഉന്നയിക്കുന്ന കാരണങ്ങള്‍ ഫസ്ഖിന് പ്രേരിപ്പിക്കാവുന്ന തരത്തിലുള്ളതാണോ എന്ന് പരിശോധിക്കുന്നത് ന്യായാധിപനാണ്. അങ്ങനെയാണെങ്കില്‍ വിവാഹമൂല്യം തിരിച്ചുനല്‍കാതെതന്നെ അവള്‍ക്ക് അവനുമായുള്ള ബന്ധത്തില്‍നിന്ന് പിരിയാനുള്ള സംവിധാനമുണ്ട്.

അനിവാര്യമായ സാഹചര്യത്തിൽ ഇസ്‌ലാം വിവാഹമോചനം അനുവദിക്കുന്നുണ്ട്. പുരുഷന്‍ തന്റെ അധികാരമുപയോഗിച്ച് വിവാഹബന്ധം വേര്‍പെടുത്തുന്നതിനാണ് സാങ്കേതികമായി ത്വലാഖ് എന്നു പറയുന്നത്. ത്വലാഖിലെത്തിച്ചേരാതെ സൂക്ഷിക്കുവാന്‍ കഴിയുന്നത്ര ശ്രമിക്കണമെന്നാണ് ഖുര്‍ആനിന്റെ താല്‍പര്യം. പുരുഷന്‍ തന്റെ ഇണയെ ഇഷ്ടപ്പെടുന്നില്ലെങ്കില്‍തന്നെ സാധ്യമാകുന്നത്ര അവളോടൊത്തു ജീവിക്കുവാന്‍ പരിശ്രമിക്കണമെന്നാണ് അത് അനുശാസിക്കുന്നത്. ”അവരോട് നിങ്ങള്‍ മര്യാദയോടെ സഹവര്‍ത്തിക്കുകയും ചെയ്യുക. നിങ്ങള്‍ക്ക് അവരോട് വെറുപ്പ് തോന്നിയേക്കാം. എന്നാല്‍, നിങ്ങള്‍ക്ക് വെറുപ്പ് തോന്നുന്ന ഒന്നില്‍ തന്നെ അല്ലാഹു ധാരാളം നന്മ നിശ്ചയിച്ചിരിക്കുകയും ചെയ്യാം” (4:19)

ദമ്പതിമാര്‍ക്കിടയില്‍ ഐക്യം നിലനിര്‍ത്താന്‍ ആവുന്നതൊക്കെ ചെയ്യേണ്ടതുണ്ടെന്നാണ് ഖുര്‍ആനിന്റെ നിലപാട്. എന്നാല്‍, സ്‌നേഹവും ഐക്യവും ഇല്ലാതായിത്തീരുകയും വൈവാഹിക ജീവിതത്തിന്റെ ലക്ഷ്യങ്ങള്‍ സാക്ഷാത്കരിക്കപ്പെടാതിരിക്കുകയും ചെയ്യുന്ന അവസ്ഥ സംജാതമായാല്‍ അവര്‍ തമ്മില്‍ വേര്‍പിരിയുന്നതിന് വിരോധമില്ല. ഈ വേര്‍പിരിയലിന് പുരുഷന്‍ മുന്‍കൈയെടുക്കുമ്പോള്‍ അതിന് ത്വലാഖ് എന്നു പറയുന്നു. സ്ത്രീയാണ് മുന്കയ്യെടുക്കുന്നതെങ്കിൽ അത് ഖുൽഅ എന്നും ന്യായാധിപനിലൂടെ നടക്കുന്നതാണെങ്കിൽ ഫസ്ഖ് എന്നുമാണ് അറിയപ്പെടുക.

ആര്‍ത്തവ സമയത്ത് സ്ത്രീയെ ത്വലാഖ് ചെയ്യുന്നത് ഇസ്‌ലാം വിലക്കിയിട്ടുണ്ട്. ഇക്കാലത്ത് സ്ത്രീയുടെ ശാരീരിക-മാനസിക നിലകളില്‍ സ്പഷ്ടമായ മാറ്റമുണ്ടാവുമെന്ന കാര്യം തെളിയിക്കപ്പെട്ടതാണ്. അവള്‍ക്ക് ശുണ്ഠിയും മറവിയും കൂടുതലായിരിക്കും. അക്കാരണത്താല്‍തന്നെ ആര്‍ത്തവകാലത്ത് തമ്മില്‍ പിണങ്ങാനും സാധ്യത കൂടുതലാണ്. ഈ പിണക്കം വിവാഹമോചനത്തിലേക്ക് നയിച്ചുകൂടാ. ദമ്പതികള്‍ തമ്മില്‍ താല്‍പര്യവും ആഭിമുഖ്യവുമുണ്ടാക്കുവാനുതകുന്ന ലൈംഗികബന്ധം ഇക്കാലത്ത് നിഷിദ്ധവുമാണ്. പിണക്കമെല്ലാം തീരുന്നത് കിടപ്പറയില്‍ വെച്ചാണല്ലോ. ആര്‍ത്തവകാലത്തുണ്ടാകുന്ന പിണക്കം തീരാന്‍ ശുദ്ധിയായതിന് ശേഷമുള്ള ലൈംഗികബന്ധം മതിയാവും. അതുകൊണ്ടുതന്നെ ആര്‍ത്തവകാലത്ത് ഭാര്യയെ മോചിപ്പിക്കുന്നത് ശരിയല്ലെന്നും അങ്ങനെ മോചിപ്പിച്ചവര്‍ അവളെ തിരിച്ചെടുക്കേണ്ടതുണ്ടെന്നും പ്രവാചകന്‍ (ﷺ) പഠിപ്പിച്ചിട്ടുണ്ട്.

ശുദ്ധികാലത്ത് തന്റെ ഭാര്യയെ ത്വലാഖ് ചെയ്യുന്ന പുരുഷന്‍ പക്ഷേ, അവളെ വീട്ടില്‍നിന്ന് പുറത്താക്കാന്‍ പാടില്ല. അവള്‍ പുറത്തുപോകാനും പാടില്ല. മൂന്നു തവണ ആര്‍ത്തവമുണ്ടാകുന്നതുവരെ അവള്‍ ഭര്‍തൃഗൃഹത്തില്‍തന്നെ താമസിക്കേണ്ടതാണ്. ആര്‍ത്തവം നിലച്ചവര്‍ക്ക് മൂന്നു മാസക്കാലവും ഗര്‍ഭിണികള്‍ക്ക് പ്രസവം വരെയുമാണ് ഈ കാലാവധി. ഇദ്ദാ കാലമെന്നാണ് ഈ കാലാവധിക്ക് സാങ്കേതികമായ പേര്. ഈ കാലത്ത് വിവാഹമോചിത ഭര്‍തൃഗൃഹത്തില്‍തന്നെ താമസിക്കണമെന്നാണ് ഖുര്‍ആനിന്റെ വിധി.

”വിവാഹമുക്തകള്‍ തങ്ങളുടെ സ്വന്തം കാര്യത്തില്‍, മൂന്ന് തവണ ആര്‍ത്തവമുണ്ടാവുന്നത് വരെ കാത്തിരിക്കേണ്ടതാണ്. അവര്‍ അല്ലാഹുവിലും അന്ത്യദിനത്തിലും വിശ്വസിക്കുന്നവരാണെങ്കില്‍ തങ്ങളുടെ ഗര്‍ഭാശയങ്ങളില്‍ അല്ലാഹു സൃഷ്ടിച്ചിട്ടുള്ളതിനെ അവര്‍ ഒളിച്ചുവെക്കുവാന്‍ പാടില്ല” (2:228).

”നിങ്ങള്‍ സ്ത്രീകളെ വിവാഹമോചനം ചെയ്യുകയാണെങ്കില്‍ അവരുടെ ഇദ്ദാ കാലത്തിന് (കണക്കാക്കി) വിവാഹമോചനം ചെയ്യുകയും ഇദ്ദാകാലം നിങ്ങള്‍ എണ്ണികണക്കാക്കുകയും ചെയ്യുക. നിങ്ങളുടെ രക്ഷിതാവായ അല്ലാഹുവെ നിങ്ങള്‍ സൂക്ഷിക്കുകയും ചെയ്യുക. അവരുടെ വീടുകളില്‍നിന്ന് അവരെ നിങ്ങള്‍ പുറത്താക്കരുത്. അവര്‍ പുറത്തുപോവുകയും ചെയ്യരുത്. പ്രത്യക്ഷമായ വല്ല നീചവൃത്തിയും അവള്‍ ചെയ്യുകയാണെങ്കിലല്ലാതെ… അങ്ങനെ അവര്‍ അവരുടെ അവധിയില്‍ എത്തുമ്പോള്‍ നിങ്ങള്‍ ന്യായമായ നിലയില്‍ അവരെ പിടിച്ചുനിര്‍ത്തുകയോ ന്യായമായ നിലയില്‍ അവരുമായി വേര്‍പിരിയുകയോ ചെയ്യുക” (65:1,2).

ഇദ്ദയുടെ കാലത്ത് സ്ത്രീയും പുരുഷനും ഭാര്യാഭര്‍ത്താക്കന്മാരല്ല. എന്നാല്‍, അന്യരുമല്ല. പുരുഷന്റെ വീട്ടിലാണ് അവള്‍ കഴിയുന്നത്. വിവാഹമോചനം ചെയ്ത ശേഷവും സ്ത്രീ ഭര്‍ത്താവിന്റെ വീട്ടില്‍തന്നെ താമസിക്കുന്നത് ഇരുവരുടെയും മനസ്സ് മാറ്റുവാന്‍ ഉപകരിക്കും. ഇന്നലെവരെ കൂടെക്കിടന്നവര്‍ ഇന്ന് രണ്ടായി കഴിയുകയാണ്. അവളെയാണെങ്കില്‍ അയാള്‍ കാണുകയും ചെയ്യുന്നു. അയാളുടെ ആസക്തിയെ ഇളക്കിവിടുവാനും കോപം ശമിപ്പിക്കുവാനും ഇതുമൂലം കഴിഞ്ഞേക്കും. ഇദ്ദാകാലത്ത് അവളെ മടക്കിയെടുക്കുവാന്‍ പുരുഷന് അവകാശമുണ്ട്. നിരുപാധികം അയാള്‍ക്ക് അതിന് സാധിക്കും. കുടുംബസ്ഥാപനം തകരാതിരിക്കുന്നതിന് എത്ര ശാസ്ത്രീയമായ മാര്‍ഗങ്ങളാണ് ഖുര്‍ആന്‍ സ്വീകരിക്കുന്നത്; കര്‍ക്കശമായ നിയമങ്ങള്‍ അടിച്ചേല്‍പിക്കാതെതന്നെ.

വിവാഹമോചനം നടത്തി. മൂന്ന് ആര്‍ത്തവകാലം കഴിയുന്നതുവരെ ഭര്‍തൃഗൃഹത്തില്‍ അവള്‍ താമസിക്കുകയും ചെയ്തു. എന്നിട്ടും അവര്‍ തമ്മില്‍ ഇണങ്ങാന്‍ മാര്‍ഗമില്ല. എങ്കില്‍ പിന്നെ മോചനംതന്നെയാണ് പരിഹാരം. ഈ മോചനംപോലും മാന്യമായിരിക്കണമെന്നാണ് ഖുര്‍ആനിന്റെ അനുശാസന. ”ഒന്നുകില്‍ മാന്യമായി അവളെ പിടിച്ചുനിര്‍ത്തുക, അല്ലെങ്കില്‍ മാന്യമായി അവളെ പിരിച്ചയക്കുക” (65:2).

വിവാഹസമയത്ത് വരന്‍ നല്‍കിയ വിവാഹമൂല്യം പൂര്‍ണമായി ഇങ്ങനെ മോചിപ്പിക്കുന്ന സ്ത്രീക്ക് അവകാശപ്പെട്ടതാണ്. കൂടുതലായാലും കുറച്ചായാലും അത് തിരിച്ചുവാങ്ങാന്‍ പാടില്ല. ഖുര്‍ആന്‍ പറയുന്നു:”നിങ്ങള്‍ ഒരു ഭാര്യയുടെ സ്ഥാനത്ത് മറ്റൊരു ഭാര്യയെ പകരം സ്വീകരിക്കുവാന്‍ ഉദ്ദേശിക്കുന്നപക്ഷം അവരില്‍ ഒരുവള്‍ക്ക് നിങ്ങള്‍ ഒരു കൂമ്പാരംതന്നെ കൊടുത്തിട്ടുണ്ടായിരുന്നുവെങ്കിലും അതില്‍നിന്ന് യാതൊന്നുംതന്നെ നിങ്ങള്‍ തിരിച്ചുവാങ്ങരുത്”(4:20).

”എന്നാല്‍, ഭാര്യയെ സ്പര്‍ശിക്കുന്നതിനു മുമ്പാണ് മോചനമെങ്കില്‍ നിശ്ചയിക്കപ്പെട്ട വിവാഹമൂല്യത്തിന്റെ പകുതി അവള്‍ക്ക് നല്‍കിയാല്‍ മതിയാകുന്നതാണ്”(2:237).

വിവാഹമോചന സമയത്ത് സ്ത്രീകള്‍ക്ക് മാന്യമായ പാരിതോഷികം നല്‍കണമെന്നും ഖുര്‍ആന്‍ അനുശാസിക്കുന്നുണ്ട്. ”വിവാഹമോചിതരായ സ്ത്രീകള്‍ക്ക് ന്യായപ്രകാരം എന്തെങ്കിലും ജീവിതവിഭവമായി നല്‍കേണ്ടതുണ്ട്. ഭയഭക്തിയുള്ളവര്‍ക്ക് അതൊരു ബാധ്യതയത്രേ” (2:241).

ഒരാള്‍ ഒരു സ്ത്രീയെ വിവാഹമോചനം നടത്തി. അല്‍പകാലത്തിനുശേഷം തന്റെ പ്രവൃത്തിയില്‍ അയാള്‍ക്ക് പാശ്ചാതാപം തോന്നി. മോചിതയായ സ്ത്രീയാണെങ്കില്‍ പുനര്‍വിവാഹം ചെയ്യപ്പെട്ടിട്ടുമില്ല. അയാള്‍ക്ക് അവളെ തന്റെ ഭാര്യയായി സ്വീകരിക്കണമെന്ന് ആഗ്രഹം ജനിച്ചു. എങ്കില്‍ അയാള്‍ക്ക് അവളെ തിരിച്ചെടുക്കാന്‍ ഖുര്‍ആന്‍ അനുവദിക്കുന്നു. ഇങ്ങനെ തിരിച്ചെടുത്തതിനുശേഷം ഒരിക്കല്‍കൂടി അതേസ്ത്രീയെതന്നെ വിവാഹമോചനം ചെയ്യുന്നുവെന്നു കരുതുക. ഒരു പ്രാവശ്യം കൂടി മാത്രമേ അയാള്‍ക്ക് അവളെ തിരിച്ചെടുക്കാന്‍ അവകാശമുള്ളൂ. മൂന്നാം തവണയും അയാള്‍ അവളെ ത്വലാഖ് ചെയ്യുകയാണെങ്കില്‍ പിന്നെ അയാള്‍ക്ക് അവളെ തിരിച്ചെടുക്കാന്‍ കഴിയില്ല. ഇതാണ് ഖുര്‍ആന്‍ പ്രതിപാദിക്കുന്ന മൂന്നു ത്വലാഖുകള്‍. ഖുര്‍ആന്‍തന്നെ പറയട്ടെ:”(മടക്കിയെടുക്കാന്‍ അനുമതിയുള്ള) വിവാഹമോചനം രണ്ടു പ്രാവശ്യം മാത്രമാകുന്നു. പിന്നെ ഒന്നുകില്‍ മര്യാദയനുസരിച്ച് കൂടെ നിര്‍ത്തുകയോ അല്ലെങ്കില്‍ നല്ല നിലയില്‍ പിരിച്ചയക്കുകയോ ആണ് വേണ്ടത്… ഇനിയും (മൂന്നാമതും) അവന്‍ അവളെ വിവാഹമോചനം ചെയ്യുകയാണെങ്കില്‍ അതിനുശേഷം അവളുമായി ബന്ധപ്പെടല്‍ അവന് അനുവദനീയമാവില്ല” (2:229-230).

ഇതാണ് ഖുര്‍ആനില്‍ പ്രതിപാദിക്കുന്ന മൂന്ന് ത്വലാഖുകള്‍. മൂന്നും മൂന്നു പ്രാവശ്യമായി നടക്കുന്ന വിവാഹമോചനങ്ങളാണവ. ഒരേസമയം മൂന്ന് ത്വലാഖ് ചൊല്ലുന്നത് നിഷിദ്ധമാണെന്ന കാര്യത്തില്‍ പ്രമുഖ മുസ്‌ലിം പണ്ഡിതന്മാര്‍ക്കിടയില്‍ പക്ഷാന്തരമില്ല. മൂന്നു ത്വലാഖും ഒന്നിച്ചു ചൊല്ലിയ ഒരാളെ ഉമര്‍(റ) ചമ്മട്ടികൊണ്ട് അടിക്കുവാന്‍ കല്‍പിക്കുകയുണ്ടായി. ഇതില്‍ നിന്ന് ഇത്തരമൊരു നടപടിയെ ഇസ്‌ലാം എന്തുമാത്രം വെറുക്കുന്നുവെന്ന് മനസ്സിലാക്കാന്‍ കഴിയും.

മൂന്ന് ത്വലാഖുകള്‍ എന്ന പദ്ധതി യഥാര്‍ഥത്തില്‍ സ്ത്രീക്ക് ഗുണകരമാണെന്നതാണ് വാസ്തവം. ഖുര്‍ആന്‍ പറഞ്ഞ രീതിയില്‍ ജീവിക്കുന്ന ഒരാള്‍ക്ക് അയാളുടെ ഹൃദയത്തിനകത്ത് സ്‌നേഹത്തിന്റെ ലാഞ്ഛനയെങ്കിലും ബാക്കിയുണ്ടെങ്കില്‍ മൂന്നാമത് ത്വലാഖ് ചെയ്യാന്‍ കഴിയില്ല. സ്വന്തം ഭാര്യയോടൊപ്പം ഒന്നിച്ചുകഴിയാന്‍ എന്തെങ്കിലും പഴുതുണ്ടോയെന്ന് അന്വേഷിക്കുകയും ഉണ്ടെങ്കില്‍ അതുപയോഗപ്പെടുത്തുകയും ചെയ്യുകയാണ് മൂന്നാമത്തെ ത്വലാഖിന് മുമ്പ് അയാള്‍ ചെയ്യുക. രണ്ടു പ്രാവശ്യം അയാള്‍ സഹിച്ച വിരഹദുഃഖം അയാളെ അലട്ടിക്കൊണ്ടിരിക്കും. ഇനിയൊരിക്കലും ഒന്നിച്ചുകഴിയാന്‍ സാധിക്കില്ലെന്ന് ഉറപ്പായതിന് ശേഷം മാത്രമേ മൂന്നാം പ്രാവശ്യം അയാള്‍ അവളെ വിവാഹമോചനം ചെയ്യുകയുള്ളൂ. ഏറ്റവും ശാസ്ത്രീയവും സ്ത്രീയെ പരിഗണിക്കുന്നതുമാണ് ഇസ്‌ലാമിലെ വിവാഹമോചനം എന്ന് പറയുന്നത് അതുകൊണ്ടാണ്.

വിഷയവുമായി ബന്ധപ്പെട്ട വീഡിയോ

”ഭാര്യമാരെ അടിക്കുന്നവര്‍ മാന്യന്മാരല്ല” എന്ന് പഠിപ്പിച്ച മുഹമ്മദ് നബി (സ) പൂര്ത്തീകരിച്ച ഇസ്‌ലാം നിരുപാധികം പെണ്ണിനെ അടിക്കാൻ പുരുഷൻ യാതൊരു അവകാശവും നൽകിയിട്ടില്ല. എന്നാൽ ഒരു ശിക്ഷണനടപടി എന്ന നിലയിൽ, വിവാഹമോചനം എന്ന ദുരന്തം ഇല്ലാതാക്കുന്നതിനുള്ള ഒരു അവസാന ശ്രമമെന്ന രീതിയിൽ ഇന്നലെ വരെ സ്നേഹിച്ച് തലോടിയ കൈ കൊണ്ടുള്ള ഒരു താഡനം ഇസ്‌ലാം അനുവദിച്ചിട്ടുണ്ട്. അത് എപ്പോൾ എങ്ങനെയെന്നും ഇസ്‌ലാം കൃത്യമായി പഠിപ്പിച്ചിട്ടുണ്ട്.

കുടുംബമെന്ന സ്ഥാപനത്തിലെ രണ്ട് പാതികളാണ് പുരുഷനും സ്ത്രീയും. എന്നാല്‍, സ്ഥാപനത്തിന്റെ നിയന്ത്രണാധികാരം പുരുഷനിലാണ് നിക്ഷിപ്തമായിരിക്കുന്നത്. സ്ഥാപനം തകരാതെ സൂക്ഷിേക്കണ്ടത് അവന്റെ ബാധ്യതയാണ്. ഇതിനുവേണ്ടി പരമാവധി പരിശ്രമിക്കണമെന്ന് ഖുര്‍ആന്‍ പുരുഷനോട് നിഷ്‌കര്‍ഷിക്കുന്നു. അതിനുവേണ്ടിയുള്ള നടപടിക്രമങ്ങളെക്കുറിച്ച് വിശദീകരിക്കുന്ന സൂക്തം ശ്രദ്ധിക്കുക:

”അതിനാല്‍ നല്ലവരായ സ്ത്രീകള്‍ അച്ചടക്കമുള്ളവരും അല്ലാഹു കാത്തത് മറവിലും കാത്തുസൂക്ഷിക്കുന്നവളുമാണ്. അച്ചടക്കരാഹിത്യം നിങ്ങള്‍ ഭയപ്പെടുന്ന സ്ത്രീകളെ നിങ്ങള്‍ ശാസിക്കുക; കിടപ്പറകളില്‍ അവരുമായി അകന്നുനില്‍ക്കുക; അവരെ അടിക്കുകയും ചെയ്യുക. എന്നിട്ട് അവര്‍ നിങ്ങളെ അനുസരിക്കുന്നപക്ഷം പിന്നെ നിങ്ങള്‍ അവര്‍ക്കെതിരില്‍ യാതൊരു മാര്‍ഗവും തേടരുത്” (4:34).

ഈ സൂക്തത്തില്‍ അച്ചടക്കരാഹിത്യത്തെക്കുറിച്ച് പറയുന്നത് നല്ല സ്ത്രീ ആരാണെന്ന് നിര്‍വചിച്ചതിനുശേഷമാണ്. ”അച്ചടക്കമുള്ളവളും അല്ലാഹു കാത്തത് മറവിലും കാത്തുസൂക്ഷിക്കുന്നവളും” ആണ് ഖുര്‍ആനിക വീക്ഷണത്തിലെ നല്ല സ്ത്രീ. കുടുംബത്തിന്റെ ഭദ്രതയ്ക്കും സമൂഹത്തിന്റെ ധാര്‍മികതക്കും സ്ത്രീകളില്‍ ഈ സ്വഭാവങ്ങള്‍ ആവശ്യമാണ്. അവള്‍ അച്ചടക്കമുള്ളവളായിരിക്കണം. അതോടൊപ്പംതന്നെ അല്ലാഹു കാത്തത് മറവിലും സംരക്ഷിക്കുന്നവളുമായിരിക്കണം.

ഭര്‍ത്താവിനോട് കയര്‍ക്കുകയും അയാള്‍ പറയുന്നതിനോടെല്ലാം എതിരു പ്രവര്‍ത്തിക്കുകയും ചെയ്യുന്ന ഭാര്യയുമൊത്തുള്ള ജീവിതം സഹിക്കാന്‍ എത്ര പേര്‍ക്ക് കഴിയും? പരസ്പരം പിണങ്ങിയും ശണ്ഠ കൂടിയും നിലനില്‍ക്കുന്ന കുടുംബാന്തരീക്ഷത്തില്‍ വളരുന്ന കുഞ്ഞുങ്ങളുടെ അവസ്ഥയെന്തായിരിക്കും? അത്തരമൊരു അവസ്ഥയുണ്ടാകുവാന്‍ പാടില്ലെന്ന് ഖുര്‍ആന്‍ നിഷ്‌കര്‍ഷിക്കുന്നു. ഭാര്യയില്‍നിന്ന് ഭര്‍ത്താവിന് മാത്രം അര്‍ഹതപ്പെടുന്ന പലതുമുണ്ട്. അവ അയാളുടെ സാന്നിധ്യത്തില്‍ അയാള്‍ക്ക് നല്‍കുകയും അസാന്നിധ്യത്തില്‍ മറ്റു പലര്‍ക്കും നല്‍കുകയും ചെയ്യുക നല്ല സ്ത്രീയുടെ സ്വഭാവമല്ല. ഭര്‍ത്താവിന് മാത്രം അവകാശപ്പെട്ട ഒരു നോട്ടമോ വാക്കോ പോലും അവളില്‍നിന്ന് അന്യര്‍ക്കായി ഉണ്ടായിക്കൂടാ. അതുണ്ടാവുന്നത് കുടുംബത്തിന്റെ തകര്‍ച്ചക്ക് കാരണമാവും. ഒരു കാരണവശാലും അത്തരമൊരു തകര്‍ച്ചയുണ്ടാവരുത്. ഖുര്‍ആനികമായ മാര്‍ഗനിര്‍ദേശങ്ങള്‍ ഈയൊരു ലക്ഷ്യത്തോടുകൂടിയുള്ളവയാണ്.

കുടുംബത്തിന്റെ തകര്‍ച്ചക്ക് നിമിത്തമായേക്കാവുന്ന അച്ചടക്കരാഹിത്യത്തെ മുളയിലേ നുള്ളിക്കളയണമെന്നാണ് ഖുര്‍ആനിന്റെ അനുശാസന. അത് പരമകാഷ്ഠ പ്രാപിച്ച് ധിക്കാരത്തിന്റെ പാരമ്യത്തിലെത്തുന്നതുവരെ കാത്തിരിക്കുന്നത് കുടുംബമെന്ന സ്ഥാപനം പൊട്ടിപ്പിളരുന്നതിന് കാരണമാവും. ആ തലത്തിലെത്തിയാല്‍ പിന്നെ ചികില്‍സകള്‍ ഫലിക്കുകയില്ല. ശാന്തിയും സമാധാനവും തകര്‍ന്ന് സര്‍വനാശത്തിലേക്ക് പോയിക്കൊണ്ടിരിക്കുന്ന കുടുംബത്തില്‍ ജീവിക്കുന്ന കുട്ടികളുടെ സ്ഥിതി പരിതാപകരമായിരിക്കും. അതിനാല്‍ അച്ചടക്കരാഹിത്യത്തിന്റെ ലക്ഷണങ്ങള്‍ വളരെ വിദൂരത്തുതന്നെ പ്രത്യക്ഷമായിത്തുടങ്ങിയാല്‍ കുടുംബത്തെ നാശത്തില്‍നിന്നു രക്ഷിക്കുവാന്‍ ക്രമപ്രവൃദ്ധമായ ചില നടപടികളാവശ്യമാണ്.അങ്ങനെയുള്ള സന്ദര്‍ഭത്തില്‍ അച്ചടക്കരാഹിത്യം ഇല്ലാതാക്കുന്നതിനുവേണ്ടി ചില നടപടികള്‍ കൈക്കൊള്ളുവാന്‍ പുരുഷനെ ഇസ്‌ലാം അനുവദിച്ചിട്ടുണ്ട്. ഈ നടപടികള്‍ നിന്ദിക്കുന്നതിനോ പ്രതികാരം ചെയ്യുന്നതിനോ വേണ്ടിയുള്ളതല്ല. പ്രത്യുത, സംസ്‌കരണത്തിനും അച്ചടക്കരാഹിത്യം ഇല്ലാതാക്കുന്നതിലൂടെ ഐക്യപ്പെടുത്തുന്നതിനും വേണ്ടിയുള്ളതാകുന്നു.

ഖുര്‍ആന്‍ നിര്‍ദേശിക്കുന്ന പ്രസ്തുത നടപടിക്രമം ഇങ്ങനെയാണ്:”ശാസിക്കുക, കിടപ്പറയില്‍ അവളെ ബഹിഷ്‌കരിക്കുക, പിന്നെ അവളെ അടിക്കുക”.

അച്ചടക്കമില്ലാത്ത സ്ത്രീയെ ആദ്യം ശാസിക്കുകയാണ് വേണ്ടത്. അവളുടെ പ്രവര്‍ത്തനങ്ങള്‍ കൊണ്ട് ഇഹത്തിലും പരത്തിലുമുണ്ടാകുവാന്‍ പോകുന്ന പ്രതിഫലനങ്ങളെക്കുറിച്ച് ബോധ്യപ്പെടുത്തുക. പെണ്ണിന്റെ പ്രത്യേകമായ സ്വഭാവങ്ങളാല്‍ സംഭവിച്ചുപോയ പാകപ്പിഴവുകളാണെങ്കില്‍ തിരുത്തുവാന്‍ ഉപദേശം ഫലം ചെയ്യും.

ശാസനയും ഉപദേശവും ഫലം ചെയ്യാത്ത സ്ഥിതിയുണ്ടാവാം. ഭര്‍ത്താവിന്റെ സ്‌നേഹവായ്‌പോടെയുള്ള ശാസനയും വികാരസാന്ദ്രമായ ഉപദേശവും ഫലം ചെയ്യാതിരിക്കുന്നതിന് കാരണം പലപ്പോഴും അഹങ്കാരമായിരിക്കും. സൗന്ദര്യത്തിന്റെയും ധനത്തിന്റെയും കുടുംബ മാഹാത്മ്യത്തിന്റെയും പേരിലുള്ള അഹന്ത. ഇവിടെയാണ് രണ്ടാമത്തെ നടപടിക്രമം വരുന്നത്. കിടപ്പറയില്‍ അവളെ ബഹിഷ്‌കരിക്കുക. ആകര്‍ഷണത്തിന്റെയും പ്രലോഭനത്തിന്റെയും കേന്ദ്രമാണ് കിടക്ക. അച്ചടക്കമില്ലാത്ത അഹങ്കാരിയായ സ്ത്രീയുടെ അധീശത്വത്തിന്റെ ഉച്ചകോടി അവിടെയാണല്ലോ. അവിടെ അവള്‍ ബഹിഷ്‌കരിക്കപ്പെടുകയെന്നു പറഞ്ഞാല്‍ അവളുടെ അഹന്തയെ പുല്ലുവില പോലും കല്‍പിക്കാതെ പുച്ഛിച്ചുതള്ളുന്നുവെന്നര്‍ഥം. അച്ചടക്കമില്ലാത്ത സ്ത്രീയുടെ ഏറ്റവും മൂര്‍ച്ചയുള്ള ആയുധത്തിനെതിരെയുള്ള ശക്തമായ നടപടി. ഈ നടപടിക്കു മുതിരുന്ന പുരുഷന് അപാ രമായ നിയന്ത്രണവും നിശ്ചയദാര്‍ഢ്യവുമാവശ്യമാണ്. ഏതൊരു അഹങ്കാരിയെയും ചിന്തിപ്പിക്കുന്ന രാത്രികളായിരിക്കും അത്. എന്തിന്റെ പേരിലാണോ താന്‍ അധീശത്വം നടിച്ചിരുന്നത് അത് തന്റെ ഇണയ്ക്ക് ആവശ്യമില്ലെന്ന രീതിയിലുള്ള ബഹിഷ്‌കരണം സ്ത്രീയുടെ മനസ്സ് മാറ്റുകതന്നെ ചെയ്യും.

ബഹിഷ്‌കരണവും പരാജയപ്പെടുന്ന സന്ദര്‍ഭങ്ങളിലും കുടുംബത്തെ തകരാന്‍ അനുവദിക്കരുതെന്നാണ് ഖുര്‍ആനിന്റെ അനുശാസന. ശാസനകള്‍ ഫലിക്കാതിരിക്കുകയും ശയ്യാബഹിഷ്‌കരണം വിജയിക്കാതിരിക്കുകയും ചെയ്യുന്ന സാഹചര്യം വളരെ വിരളമായിരിക്കും. അത്തരം സാഹചര്യങ്ങളുണ്ടാവുകയാണെങ്കില്‍ അവിടെ അച്ചടക്കരാഹിത്യം അതിന്റെ പരമകാഷ്ഠയിലെത്തിയിരിക്കും. ചെറിയ ശിക്ഷകളല്ലാതെ ഇനി മാര്‍ഗങ്ങളൊന്നുമില്ല.

അടുത്ത മാര്‍ഗമെന്ന നിലയ്ക്കാണ് ഖുര്‍ആന്‍ ‘പ്രഹരം‘നിര്‍ദേശിക്കുന്നത്. സമാധാനപരമായ മാര്‍ഗങ്ങളെല്ലാം പരാജയപ്പെടുമ്പോള്‍ ഒരു കരുതല്‍ നടപടിയെന്ന നിലയില്‍ നിര്‍ദേശിക്കപ്പെട്ടതാണ് അവളെ അടിയ്ക്കുകയെന്നത്. സാധാരണ ഗതിയില്‍ സ്ത്രീയെ അടിക്കുന്നതിനെതിരെ ശക്തമായി സംസാരിച്ചിട്ടുള്ള വ്യക്തിയാണ് മുഹമ്മദ്(ﷺ). ”ഭാര്യമാരെ അടിക്കുന്നവര്‍ മാന്യന്മാരല്ല” (അബൂദാവൂദ്, ഇബ്‌നുമാജ) എന്നാണ് അദ്ദേഹം അഭിപ്രായപ്പെട്ടത്. അദ്ദേഹം ചോദിച്ചു: ”നാണമില്ലേ നിങ്ങള്‍ക്ക്? അടിമയെ അടിക്കുന്നതുപോലെ സ്വന്തം ഭാര്യയെ അടിക്കാന്‍; പിന്നെ അവളോടൊത്ത് ശയിക്കാനും” (മുസ്‌ലിം, അഹ്മദ്). ”നിങ്ങളില്‍ ഭാര്യമാരോട് നന്നായി പെരുമാറുന്നവരാണ് ഏറ്റവും നല്ലവന്‍” (ബുഖാരി,തുര്‍മുദി) എന്നു പഠിപ്പിച്ച പ്രവാചകനിലൂടെ അവതീര്‍ണമായ ഖുര്‍ആന്‍ വെറുതെ സ്ത്രീയെ അടിക്കണമെന്ന് കല്‍പിക്കുകയില്ലെന്നുറപ്പാണ്.

വലിയ തിന്മയില്ലാതാക്കുവാനുള്ള ശിക്ഷണമായി, മറ്റു മാര്‍ഗങ്ങള്‍ പരാജയപ്പെടുമ്പോഴുള്ള അവസാന മാര്‍ഗമായിട്ടാണ് ഖുര്‍ആന്‍ അടി നിര്‍ദേശിക്കുന്നത്. അതുതന്നെ അവള്‍ക്ക് അഭിമാനക്ഷതമുണ്ടാകുന്ന രീതിയില്‍ മുഖത്തോ മറ്റോ ആകരുതെന്ന് പ്രവാചകന്‍ (ﷺ) പ്രത്യേകം നിര്‍ദേശിച്ചിട്ടുമുണ്ട്. സ്ത്രീയെ നിന്ദിക്കുവാനോ അപമാനിക്കുവാനോ വേണ്ടിയല്ല, പ്രത്യുത നന്നാക്കുന്നതിനും സംസ്‌കരിക്കുന്നതിനും വേണ്ടിയുള്ള അവസാനത്തെ മാര്‍ഗമെന്ന നിലയ്ക്കാണ് ഖുര്‍ആന്‍ അടി നിര്‍ദേശിക്കുന്നത്. പിതാവ് മക്കളെ അടിക്കുന്നതുപോലെ, അധ്യാപകന്‍ വിദ്യാര്‍ഥികളെ കൈകാര്യം ചെയ്യുന്നതുപോലെ, ഒരു പരിശീലകന്റെ വികാരത്തോടെയുള്ള ശിക്ഷണമാണത്. എപ്പോഴും സ്‌നേഹം നല്‍കുകയും തന്റെ സ്‌നേഹപ്രകടനങ്ങള്‍ക്ക് പാത്രമാവുകയും ചെയ്യുന്ന തന്റെ ഇണയുടെ പ്രഹരം അവളെ വീണ്ടു വിചാരത്തിനും ഖേദപ്രകടനത്തിനും അങ്ങനെ തെറ്റുതിരുത്തലിനും പ്രേരകമാക്കിയേക്കാം. അങ്ങനെ തകര്‍ച്ചയുടെ വക്കിലെത്തിനില്‍ക്കുന്ന കുടുംബം തകരാതെ രക്ഷപ്പെടാനിടയുണ്ട്. ഈ ശിക്ഷണത്തിന്റെ ആത്യന്തിക ലക്ഷ്യം കുടുംബമെന്ന സ്ഥാപനത്തെ തകര്‍ച്ചയില്‍നിന്ന് രക്ഷിക്കുകയാണെന്നര്‍ഥം.

ഇസ്‌ലാമിക വസ്ത്രധാരണം അടിമത്തത്തിന്റെ അടയാളമല്ല. പ്രത്യുത ആഭിജാത്യത്തിന്റെ ചിഹ്‌നമാണ് എന്ന് അല്‍പം ചിന്തിച്ചാല്‍ ബോധ്യമാകും. മുഖവും മുന്‍കൈയും ഒഴികെയുള്ള ശരീരഭാഗങ്ങളെല്ലാം മറക്കണമെന്ന് ഇസ്‌ലാം സ്ത്രീയോട് കല്‍പിക്കുന്നുവെന്നത് ശരിയാണ്. എന്തിനാണ് ഈ കല്‍പന? സ്ത്രീകളെ അടിമത്തത്തിന്റെ കാരാഗൃഹത്തിലടക്കുകയോ സുരക്ഷിതത്വത്തിന്റെ താഴ്‌വരയില്‍ വിഹരിക്കാനനുവദിക്കുകയോ എന്താണ് ഈ കല്‍പന ചെയ്യുന്നത്?ഇസ്‌ലാമിക വസ്ത്രധാരണം നിര്‍ബന്ധമാക്കിക്കൊണ്ടുള്ള ഖുര്‍ആന്‍ സൂക്തങ്ങള്‍ ഈ ചോദ്യങ്ങള്‍ക്ക് വ്യക്തമായ ഉത്തരം നല്‍കുന്നുണ്ട്. അത് ഇങ്ങനെയാണ്:

”നബിയേ, താങ്കളുടെ പത്‌നിമാരോടും പുത്രിമാരോടും സത്യവിശ്വാസികളുടെ സ്ത്രീകളോടും അവര്‍ തങ്ങളുടെ മൂടുപടങ്ങള്‍ തങ്ങളുടെ മേല്‍ താഴ്ത്തിയിടാന്‍ പറയുക. അവര്‍ തിരിച്ചറിയപ്പെടുവാനും അങ്ങനെ അവര്‍ ശല്യം ചെയ്യപ്പെടാതിരിക്കാനും അതാണ് ഏറ്റവും അനുയോജ്യമായത്. അല്ലാഹു ഏറെ പൊറുക്കുന്നവനും കരുണാനിധിയുമാകുന്നു” (33:59).

”സത്യവിശ്വാസിനികളോട് അവരുടെ ദൃഷ്ടികള്‍ താഴ്ത്തുവാനും അവരുടെ ഗുഹ്യാവയവങ്ങള്‍ കാത്തുരക്ഷിക്കാനും അവരുടെ ഭംഗിയില്‍നിന്ന് പ്രത്യക്ഷമായതൊഴിച്ച് മറ്റൊന്നും വെളിപ്പെടുത്താതിരിക്കുവാനും നീ പറയുക. അവരുടെ മക്കനകള്‍ കുപ്പായമാറുകള്‍ക്ക് മീതെ അവര്‍ താഴ്ത്തിയിട്ടുകൊള്ളട്ടെ” (24:31).

”പഴയ അജ്ഞാനകാലത്തെ സൗന്ദര്യപ്രകടനം പോലെയുള്ള സൗന്ദര്യ പ്രകടനം നിങ്ങള്‍ നടത്തരുത്” (33:33).

സ്ത്രീയോട് മാന്യമായ വസ്ത്രധാരണരീതി സ്വീകരിക്കാന്‍ കല്‍പിച്ചതിന് പിന്നിലുള്ള ലക്ഷ്യങ്ങള്‍ ഈ സൂക്തങ്ങളില്‍ല്‍നിന്ന് സുതരാം വ്യക്തമാണ്.

ഒന്ന്, തിരിച്ചറിയപ്പെടുക.

രണ്ട്, ശല്യം ചെയ്യപ്പെടാതിരിക്കുക.

സമൂഹത്തിന്റെ വ്യത്യസ്ത തുറകളില്‍ ജീവിക്കുന്നവര്‍ തിരിച്ചറിയപ്പെടുന്നതിനുവേണ്ടി വ്യത്യസ്ത വസ്ത്രധാരണരീതികള്‍ സ്വീകരിക്കാറുണ്ട്. സ്ത്രീകെള സംബന്ധിച്ചിടത്തോളം അവരുടെ വസ്ത്രധാരണരീതിയില്‍നിന്നുതന്നെ ഒരളവോളം അവരുടെ ജീവിതരീതിയെയും പെരുമാറ്റ രീതിയെയും നമുക്ക് അളക്കുവാന്‍ സാധിക്കും.

ആവശ്യക്കാര്‍ക്ക് തിരിച്ചറിയുവാന്‍ സാധിക്കുന്ന രീതിയിലുള്ള വസ്ത്രധാരണരീതിയാണ് വേശ്യകള്‍ സ്വീകരിക്കുക. ക്ഷേത്രങ്ങളോട് ബന്ധപ്പെട്ട് ജീവിച്ചിരുന്ന ദേവദാസികള്‍ക്ക് അവരുടേതായ വസ്ത്രധാരണ രീതിയുണ്ടായിരുന്നു. ഗ്രീസിലെ ഹെറ്റേയ്‌റേകള്‍ക്കും ചൈനയിലെ ചിന്‍കുവാന്‍ ജെന്നുകള്‍ക്കും ജപ്പാനിലെ ഗായിഷേകള്‍ക്കുമെല്ലാം അവരുടേതായ വസ്ത്രധാരണരീതികളുണ്ടായിരുന്നതായി കാണാന്‍ കഴിയും. ഈ വസ്ത്രധാരണത്തില്‍ നിന്ന് അവരെ മനസ്സിലാക്കാം. ആവശ്യക്കാര്‍ക്ക് ഉപയോഗിക്കുവാന്‍ ക്ഷണിക്കുകയും ചെയ്യാം.

ഇസ്‌ലാം വിഭാവനം ചെയ്യുന്ന സ്ത്രീ, മാന്യയും കുലീനയുമാണ്;ചാരിത്രവതിയും സദ്‌വൃത്തയുമാണ്. അവളുടെയടുത്തേക്ക് ലൈംഗികദാഹം പൂണ്ട ചെന്നായ്ക്കള്‍ ഓടിയടുക്കേണ്ടതില്ല. കാമാഭ്യര്‍ഥനയുമായി അവളെ ആരും സമീപിക്കേണ്ടതില്ല. ഇത് അവളുടെ വസ്ത്രത്തില്‍നിന്നുതന്നെ തിരിച്ചറിയണം. പതിനഞ്ചാം നൂറ്റാണ്ടിലെ വെനീസിലെ നിയമസംഹിതയില്‍ വേശ്യകള്‍ മാറുമറയ്ക്കാതെ ജനാലക്കല്‍ ഇരുന്നുകൊള്ളണമെന്ന കല്‍ പനയുണ്ടായിരുന്നു. മാംസദാഹം തീര്‍ക്കുവാന്‍ വരുന്നവര്‍ക്ക് മാംസഗുണമളക്കുവാന്‍ വേണ്ടിയുള്ള നടപടി! ഇന്നലെകളില്‍ ആവശ്യക്കാരെ ആകര്‍ഷിക്കുന്നതിനുവേണ്ടി അഭിസാരികകള്‍ സ്വീകരിച്ചിരുന്ന വസ്ത്രങ്ങള്‍ക്ക് സമാനമായ ഉടയാടകളാണ് ആധുനിക വനിതകളുടെ വേഷമെന്ന കാര്യം എന്തു മാത്രം വിചിത്രമല്ല! സത്യവിശ്വാസികളെയും മാംസവില്‍പനക്കാരികളെയും തിരിച്ചറിയണമെന്ന് ഖുര്‍ആന്‍ നിര്‍ദേശിക്കുന്നു; അവരുടെ വസ്ത്രധാരണത്തിലൂടെ.

എക്കാലത്തും ശല്യം ചെയ്യപ്പെട്ടുകൊണ്ടിരിക്കുന്ന ഒരു വിഭാഗമാണ് സ്ത്രീകള്‍. അവരുടെ മാംസത്തിനുവേണ്ടി-ചാരിത്ര്യത്തിനുവേണ്ടി-കടിപിടി കൂടുന്നവരാണ് എന്നത്തെയും സാഹിത്യ-സാംസ്‌കാരിക രംഗത്തെ നായകന്മാര്‍. നഗ്‌നനൃത്തങ്ങളും നഗ്‌നതാ വിവരങ്ങളുള്‍ക്കൊള്ളുന്ന കവിതകളും ഉപയോഗിച്ചുകൊണ്ടായിരുന്നു ഇന്നലെ സ്ത്രീയുടെ മാനത്തെ പിച്ചിച്ചീന്തിയിരുന്നതെങ്കില്‍ ഇന്നത്‘വിഡ്ഢിപ്പെട്ടി‘കളിലൂടെയും ഇന്റര്‍നെറ്റിലൂടെയും കുടുംബത്തിന്റെ ഇടനാഴികളിലേക്ക് കടന്നുവന്നുകൊണ്ടിരിക്കുകയാണ്. ആധുനിക ജനതയുടെ മുഴുജീവിതവും ലൈംഗികവത്കരിക്കപ്പെട്ടിരിക്കുകയാണ്. അതിരാവിലെ കുടിക്കേണ്ട കാപ്പിയേതാണെന്ന് തെരഞ്ഞെടുക്കുന്നതിനും രാത്രി ഉറങ്ങുമ്പോള്‍ വെയ്‌ക്കേണ്ട തലയിണ ഏതാണെന്ന് തീരുമാനിക്കുന്നതിനുംപോലും പെണ്ണിന്റെ നിമ്‌നോന്നതികളിലൂടെ കണ്ണ് പായിക്കണമെന്നുള്ള അവസ്ഥയാണിന്നുള്ളത്.

അതുകൊണ്ടുതന്നെ, പെണ്ണിനു നേരെയുള്ള കൈയേറ്റങ്ങളും കൂടി ക്കൊണ്ടിരിക്കുന്നു. സ്വന്തം മകളെ മാനഭംഗം ചെയ്യുന്ന അച്ഛനും പെറ്റമ്മയുമായി ലൈംഗികകേളികളിലേര്‍പ്പെടുന്ന മകനും നമ്മുടെ മസ്തിഷ്‌കങ്ങളില്‍ യാതൊരു ആന്ദോളനവും സൃഷ്ടിക്കാത്ത കഥാപാത്രങ്ങളായിക്കൊണ്ടിരിക്കുന്നു. വിദ്യാര്‍ഥിനികളെ മാനഭംഗപ്പെടുത്തുന്ന അധ്യാപകര്‍! അധ്യാപികമാരുമായി ഊരുചുറ്റുന്ന വിദ്യാര്‍ഥികള്‍! വനിതാ സെക്രട്ടറിയുമായി ബന്ധപ്പെട്ട ലൈംഗിക അപവാദങ്ങള്‍ മൂലം രാജിവെച്ചൊഴിയേണ്ടിവരുന്ന ഉദ്യോഗസ്ഥ പ്രമുഖര്‍! പലരുമായി ലൈംഗികബന്ധമുണ്ടെന്ന് പരസ്യമായി പ്രഖ്യാപിക്കുന്ന രാജകുമാരിമാര്‍! ഇങ്ങനെ പോകുന്നു ദിനപത്രങ്ങളില്‍ ദിനേന നാം വായിക്കുന്ന വര്‍ത്തമാനങ്ങള്‍. സ്ത്രീകള്‍ക്ക് സൈ്വരമായി യാത്ര ചെയ്യാന്‍ കഴിയാത്ത അവസ്ഥ! സ്വൈര്യമായി ജോലി ചെയ്യാനാവാത്ത സ്ഥിതി! എന്തിനധികം, സ്വൈര്യമായി വീട്ടില്‍ അടങ്ങിക്കൂടി നില്‍ക്കുവാന്‍ പോലും കഴിയാത്ത അവസ്ഥയിലേക്കാണ് നമ്മുടെ സമൂഹം പൊയ്‌ക്കൊണ്ടിരിക്കുന്നത്. ഇതിനെന്താണ് കാരണം?പക്വമതികളായ വിദഗ്ധര്‍ പറയുന്ന ഉത്തരം ശ്രദ്ധിക്കുക:

‘കുമാരി‘ വാരികയിലെ ‘പ്രതിവാര ചിന്തകള്‍‘ എന്ന പംക്തിയില്‍ എന്‍. വി. കൃഷ്ണവാരിയര്‍ എഴുതി: ”സ്ത്രീകളുടെ മാദകമായ വസ്ത്രധാരണവും ചേഷ്ടകളും നിമിത്തം മതിമറന്ന് താല്‍ക്കാലികമായ ഒരു ഉന്മാദാവസ്ഥയിലാണ് പുരുഷന്‍ ബലാല്‍സംഗം നടത്തുന്നതെന്ന് പൊതുവെ വിശ്വസിക്കപ്പെടുന്നു. പുരുഷനെ ഉത്തേജിപ്പിക്കുമാറ് വസ്ത്രം ധരിച്ച ഒാരോ സ്ത്രീയും ബലാല്‍സംഗം അര്‍ഹിക്കുന്നുവെന്ന് ഇന്ത്യയില്‍ ഒരു സുപ്രീംകോടതി ജഡ്ജി കുറെമുമ്പ് പരസ്യമായി പ്രസ്താവിക്കുകയുണ്ടായി” (കുമാരി വാരിക 11.3.83).

അപ്പോള്‍ വസ്ത്രധാരണത്തില്‍ മാന്യത പുലര്‍ത്തുന്നതുവഴി സ്ത്രീ സ്വന്തം ശരീരത്തെ സംരക്ഷിക്കുകയാണ് ചെയ്യുന്നത്. പടച്ചതമ്പുരാന്‍ പറഞ്ഞതെത്ര ശരി!

”അവര്‍ തിരിച്ചറിയപ്പെടാനും ശല്യം ചെയ്യപ്പെടാതിരിക്കാനും അതാണ് അനുയോജ്യം” (33:59).

വ്യഭിചാരവും ബലാല്‍സംഗങ്ങളും സ്ത്രീകള്‍ക്ക് നേരെയുള്ള കൈയേറ്റങ്ങളും അവസാനിപ്പിക്കുന്നതിന്റെ ആദ്യപടിയെന്ന നിലക്കാണ് മാന്യമായി വസ്ത്രധാരണം ചെയ്യണമെന്ന് ഖുര്‍ആന്‍ സ്ത്രീകളോട് ഉപദേശിക്കുന്നത്.

മുഖവും മുന്‍കൈയും ഒഴികെയുള്ള ശരീരഭാഗങ്ങളെല്ലാം മറയ്ക്കണമെന്നുതന്നെയായിരുന്നു സത്യവിശ്വാസിനികളായ സ്ത്രീകളോട് മുന്‍ പ്രവാചകന്മാരും പഠിപ്പിച്ചിരുന്നത് എന്നാണ് മനസ്സിലാക്കാന്‍ കഴിയുന്നത്. അന്യപുരുഷന്മാരെ കാണുമ്പോള്‍ മൂടുപടം അണിയുന്ന പതിവ് ഇസ്രായേല്‍ സമൂഹത്തില്‍ ആദ്യം മുതല്‍ക്കുതന്നെ നിലനിന്നിരുന്നുവെന്നാണ് പഴയനിയമ ചരിത്രം നല്‍കുന്ന സൂചന (ഉല്‍പത്തി 24:62-65). ഒരു സ്ത്രീയുടെ മൂടുപടം എടുത്തുകളയുന്നത് അവളെ മാനഭംഗം ചെയ്യുന്നതിന് തുല്യമായിക്കൊണ്ട് വിശേഷിപ്പിക്കപ്പെട്ടതില്‍നിന്ന് (ഉത്തമഗീതം 5:7) അതിനുണ്ടായിരുന്ന പ്രാധാന്യം എത്രത്തോളമായിരുന്നുവെന്ന് ഊഹിക്കുവാന്‍ കഴിയും.

യേശുക്രിസ്തുവിന് ശേഷവും മൂടുപടം ഉപയോഗിക്കുന്ന സമ്പ്രദായം നിലനിന്നിരുന്നതായി കാണാന്‍ കഴിയും. പൗലോസിന്റെ എഴുത്തുകളില്‍നിന്ന് നമുക്ക് ഇക്കാര്യം മനസ്സിലാക്കാനാവും. അദ്ദേഹം എഴുതി: ”സ്വന്തം ശിരസ്സ് മൂടാതെ പ്രാര്‍ഥിക്കയോ പ്രവചിക്കയോ ചെയ്യുന്ന സ്ത്രീ തന്റെ ശിരസ്സിനെ അപമാനിക്കുന്നു. അവളുടെ തല മുണ്ഡനം ചെയ്യുന്നതിന് സമമാണത്. തല മൂടാത്ത സ്ത്രീ തന്റെ മുടി മുറിക്കണം. മുടി മുറിക്കുന്നതും മുണ്ഡനം ചെയ്യുന്നതും അപമാനമാണെന്ന് കരുതുന്നവര്‍ ശിരോവസ്ത്രം ധരിക്കട്ടെ” (1കൊരിന്ത്യര്‍ 11:5-7).

”വ്യഭിചാരത്തെ സമീപിക്കുകപോലും ചെയ്യരുത്” (17:32) എന്ന സത്യവിശ്വാസികളോടുള്ള ഖുര്‍ആനിക കല്‍പനയുടെ പ്രയോഗവത്കരണത്തിന്റെ ഭാഗമായിട്ടാണ് മാന്യമായ വസ്ത്രധാരണം വേണമെന്ന് അത് സ്ത്രീകളോട് അനുശാസിക്കുന്നത്. കാമാര്‍ത്തമായ നോട്ടവും വാക്കും അംഗചലനങ്ങളുമെല്ലാം വ്യഭിചാരത്തിന്റെ അംശങ്ങളുള്‍ക്കൊള്ളുന്നവയാണെന്നാണ് മുഹമ്മദ് നബി(ﷺ)പഠിപ്പിച്ചത്. വ്യഭിചാരത്തിലേക്കും തദ്വാരാ സദാചാര തകര്‍ച്ചയിലേക്കും നയിക്കുന്ന ‘കൊച്ചു വ്യഭിചാരങ്ങ‘ളുടെ വാതിലടയ്ക്കണമെന്ന് ഇസ്‌ലാം നിഷ്‌കര്‍ഷിക്കുന്നു. മാദകമായ വസ്ത്രധാരണവും ലൈംഗികചേഷ്ടയിലെ അംഗചലനങ്ങളുള്‍ക്കൊള്ളുന്ന നൃത്തനര്‍ത്യങ്ങളും ഇസ്‌ലാം നിരോധിക്കുന്നത് അതുകൊണ്ടാണ്.

മാന്യമായാണ് സ്ത്രീ വസ്ത്രം ധരിക്കേണ്ടതെന്ന് കാര്യബോധമുള്ളവരെല്ലാം സമ്മതിക്കും. സ്ത്രീകൾക്ക് നേരെയുള്ള അക്രമങ്ങൾക്കുള്ള കാരണങ്ങളിലൊന്ന് പുരുഷനെ ത്രസിപ്പിക്കുന്ന അവളുടെ വസ്ത്രധാരണയാണെന്നും അവർ പറയും. എങ്ങനെയാണ് ഒരു സ്ത്രീ മാന്യമായി വസ്ത്രം ധരിക്കേണ്ടത്?കാര്‍ക്കൂന്തലുകളും മാറിന്റെ സിംഹഭാഗവും വയറുമെല്ലാം പുറത്തുകാണിച്ചുകൊണ്ടുള്ള പഴയ ദേവദാസികളുടേതിനു തുല്യമായ വസ്ത്രധാരണാ രീതിയോ? കാല്‍മുട്ടുവരെയും കഴുത്തും കാര്‍ക്കൂന്തലുകളും പുറത്ത് കാണിച്ചുകൊണ്ടുള്ള ഗ്രീസിലെ ഹെറ്റയ്‌റേകളുടെ വസ്ത്രധാരണ സമ്പ്രദായമോ? ഇറുകിയ വസ്ത്രങ്ങളിലൂടെ ശരീരത്തിന്റെ നിമ്‌നോന്നതികള്‍ പുരുഷന് മുന്നില്‍ പ്രദര്‍ശിപ്പിക്കുന്ന ചൈനയിലെ ചിന്‍കുവാന്‍ ജെന്നുകളുടെ ഉടയാടകള്‍ക്ക് തുല്യമായ പുടവകളോ? അതല്ല, മുഖവും മുന്‍കൈയും മാത്രം പുറത്തുകാണിക്കുകയും ശരീരഭാഗങ്ങള്‍ വെളിവാകാത്ത രൂപത്തില്‍ അയഞ്ഞ വസ്ത്രം ധരിക്കുകയും ചെയ്യുന്ന ഇസ്‌ലാമിക രീതിയോ?മുന്‍ധാരണയില്ലാത്ത ആര്‍ക്കും അവസാനത്തേതല്ലാത്ത മറ്റൊരു ഉത്തരം തെരഞ്ഞെടുക്കാന്‍ കഴിയില്ല.

ഇസ്‌ലാം സ്ത്രീയോട് മാന്യമായി വസ്ത്രം ധരിക്കാന്‍ പറയുക മാത്രമല്ല, എങ്ങനെയാണ് ആ വസ്ത്രധാരണരീതിയെന്ന് പഠിപ്പിക്കുക കൂടി ചെയ്തുവെന്നുള്ളതാണ് അതിന്റെ ഏറ്റവും വലിയ സവിശേഷത. പുരുഷന്മാരെ വഴിതെറ്റിക്കുന്ന രീതിയിലുള്ള വസ്ത്രധാരണം സ്വീകരിക്കരുതെന്ന് പറയുന്ന മറ്റുള്ളവര്‍ക്ക് പലപ്പോഴും പ്രസ്തുത വസ്ത്രധാരണ രീതിയെക്കുറിച്ച് വ്യക്തമായൊരു ചിത്രം നല്‍കാന്‍ കഴിയാറില്ല. ഇസ്‌ലാം വിജയിക്കുന്നത് ഇവിടെയാണ്. ഇസ്‌ലാമിക വസ്ത്രധാരണാരീതി സ്വീകരിച്ചിരിക്കുന്ന സമൂഹങ്ങളില്‍ സ്ത്രീകള്‍ക്കെതിരെയുള്ള കുറ്റകൃത്യങ്ങള്‍ തുലോം വിരളമാണെന്ന വസ്തുത ‘അവര്‍ ശല്യപ്പെടാതിരിക്കാന്‍ വേണ്ടി” (33:59)എന്ന ഖുര്‍ആനിക നിര്‍ദേശത്തിന്റെ സത്യതയും പ്രായോഗികതയും വ്യക്തമാക്കുന്നതാണ്.

ഇസ്‌ലാമിക വസ്ത്രധാരണം സ്ത്രീയെ അടുക്കളയില്‍ തളച്ചിടുന്നതിനുവേണ്ടി സൃഷ്ടിച്ചെടുത്തതാണെന്ന ആരോപണം അടിസ്ഥാനരഹിതമാണ്. ഈ വസ്ത്രധാരണാരീതി സ്വീകരിച്ചുകൊണ്ടുതന്നെ സമൂഹത്തിന്റെ വിവിധ തുറകളില്‍ പ്രശോഭിച്ച ഒട്ടനവധി മഹിളാരത്‌നങ്ങളെക്കുറിച്ച് ചരിത്രം നമുക്ക് പറഞ്ഞുതരുന്നുണ്ട്. പ്രവാചകനില്‍നിന്ന് കാര്യങ്ങള്‍ പഠിക്കുകയും അദ്ദേഹത്തിന്റെ ജീവിതകാലത്തും ശേഷവും പ്രവാചകാനുചരന്മാരെ പഠിപ്പിക്കുകയും ചെയ്യുന്നതിന് പ്രവാചകപത്‌നി ആഇശ(റ)ക്ക് ഇസ്‌ലാമിക വസ്ത്രധാരണം ഒരു തടസ്സമായി നിന്നിട്ടില്ല. പ്രസ്തുത വസ്ത്രം ധരിച്ചുകൊണ്ടുതന്നെയായിരുന്നു അവര്‍ ജമല്‍ യുദ്ധം നയിച്ചത്. പുരുഷന്മാരില്‍ ഭൂരിപക്ഷവും യുദ്ധരംഗം വിട്ടോടിയ സന്ദര്‍ഭത്തില്‍ -ഉഹ്ദ് യുദ്ധത്തില്‍ -ആയുധമെടുത്ത് അടരാടിയ ഉമ്മു അമ്മാറ(റ)ധരിച്ചത് ഹിജാബ് തന്നെയായിരുന്നു. ഏഴ് യുദ്ധങ്ങളില്‍ പ്രവാചകനോടൊപ്പം പങ്കെടുത്ത് പരിക്കേറ്റവരെ പരിചരിച്ചും ഭക്ഷണം പാകം ചെയ്തും പ്രശസ്തയായ ഉമ്മുഅത്വിയ്യ(റ)ക്ക് തന്റെ ദൗത്യനിര്‍വഹണത്തിനു മുമ്പില്‍ ഇസ്‌ലാമിക വസ്ത്രധാരണം ഒരു വിലങ്ങായിത്തീര്‍ന്നിട്ടില്ല. ഇങ്ങനെ പ്രവാചകാനുചരന്മാരില്‍തന്നെ എത്രയെത്ര മഹിളാരത്‌നങ്ങള്‍! മുഖവും മുന്‍കൈയും മാത്രം പുറത്തുകാണിച്ചുകൊണ്ടുതന്നെ സമൂഹത്തിന്റെ വ്യത്യസ്ത തുറകളില്‍ വ്യക്തിമുദ്ര പതിപ്പിച്ച മഹതികള്‍!

ഇന്നും ഇസ്‌ലാമിക സമൂഹത്തില്‍ ഇത്തരം സഹോദരിമാരുണ്ട്. ഇസ്‌ലാമിക വസ്ത്രധാരണരീതി സ്വീകരിച്ചുകൊണ്ട് സാമൂഹിക മേഖലകളിലേക്ക് സേവന സന്നദ്ധരായി സധൈര്യം കയറിച്ചെല്ലുന്ന സഹോദരികള്‍. ഇസ്‌ലാമിക വസ്ത്രധാരണം സ്ത്രീയെ ചങ്ങലകളില്‍ ബന്ധിക്കുന്നുവെന്ന ആരോപണം അര്‍ഥമില്ലാത്തതാണെന്ന വസ്തുത ഇവിടെ അനാവൃതമാകുന്നു.

സത്യത്തില്‍, മാന്യമായി വസ്ത്രം ധരിക്കണമെന്ന് നിര്‍ദേശിക്കുക വഴി ഖുര്‍ആന്‍ സ്ത്രീകളുടെ ആത്മാഭിമാനം ഉയര്‍ത്തുകയും അവര്‍ ആക്രമിക്കപ്പെടുന്ന അവസ്ഥ ഇല്ലാതാക്കുവാനുള്ള പ്രായോഗിക പദ്ധതിക്ക് രൂപം നല്‍കുകയുമാണ് ചെയ്യുന്നത്.

വിഷയവുമായി ബന്ധപ്പെട്ട വീഡിയോ

ഒരു പുരുഷനു പകരം രണ്ടു സ്ത്രീകള്‍ സാക്ഷികളായി ഉണ്ടാവണമെന്ന ഖുര്‍ആനിന്റെ അനുശാസന സ്ത്രീയെ അവഗണിക്കുകയും അവളോട് അനീതി ചെയ്യുന്നതുമല്ലേയെന്നാണ് വിമർശനം.

കടമിടപാടുകളെ സംബന്ധിച്ച് പ്രതിപാദിക്കുന്നിടത്ത് ഖുര്‍ആന്‍ പറയുന്നു:

”നിങ്ങളില്‍പെട്ട രണ്ടു പുരുഷന്മാരെ നിങ്ങള്‍ സാക്ഷിനിര്‍ത്തുക. ഇനി ഇരുവരും പുരുഷന്മാരായില്ലെങ്കില്‍ നിങ്ങളിഷ്ടപ്പെടുന്ന സാക്ഷികളില്‍നിന്ന് ഒരു പുരുഷനും രണ്ടു സ്ത്രീകളും ആയാലും മതി. അവരില്‍ ഒരുവള്‍ക്ക് തെറ്റുപറ്റിയാല്‍ മറ്റവള്‍ അവളെ ഓര്‍മിപ്പിക്കുവാന്‍ വേണ്ടി” (2:283).

പല മതഗ്രന്ഥങ്ങളും സ്ത്രീ, സാക്ഷ്യത്തിനുതന്നെ അയോഗ്യയാണെന്നാണ് വിധിച്ചിരിക്കുന്നത്. ഉദാഹരണത്തിന്,യാജ്ഞവല്‍ക്യസ്മൃതിയുടെ വിധി കാണുക:

സ്ത്രീ ബാലവൃദ്ധ കിവത മത്തോന്‍മത്താഭിശസ്തകാഃ

രംഗാവതാരി പാഖണ്ഡി കുടകൃദ്വിലേന്ദ്രിയഃ

പതിതാപതാര്‍ത്ഥ സംബന്ധി സഹായരി പുതസ്‌കരാഃ

സാഹസീ ദൃഷ്ട ദോഷശ്ച നിര്‍ദ്ധുതാദ്യാസ്ത്വ സാക്ഷിണഃ (2:70,71).

(സ്ത്രീ, ബാലന്‍, വൃദ്ധന്‍ ചൂതുകളിക്കാരന്‍, മത്തനായവന്‍,ഉന്മാദമുള്ളവന്‍, ബ്രഹ്മഹത്യ തുടങ്ങിയ പാപമുള്ളവന്‍, ചാരണന്‍ (ഗായകന്‍, നടന്‍ തുടങ്ങിയവര്‍), പാഖണ്ഡി (നാസ്തികന്‍), വ്യാജരേഖ ചമക്കുന്നവന്‍, വികലാംഗന്‍, പതിതന്‍, സുഹൃത്ത്, പണം കൊടുക്കുന്നവന്‍, സഹായി, ശത്രു, കള്ളന്‍, സാഹസി (പിടിച്ചുപറിക്കാരന്‍), പ്രത്യക്ഷമായ ദോഷമുള്ളവന്‍, ബന്ധുക്കള്‍ ഉപേക്ഷിച്ചവന്‍ തുടങ്ങിയവര്‍ സാക്ഷികളാവാന്‍ യോഗ്യരല്ല)

എന്തുകൊണ്ടാണ് സ്ത്രീകളെ സാക്ഷ്യത്തിനു പറ്റാത്തത്?മനുസ്മൃതിയുടെ വിശദീകരണം ഇങ്ങനെയാണ്:

ഏകോലുബ്ധസ്തു സാക്ഷീസ്യാല്‍ ബഹ്യശ്ശൂ ച്യോപിന സ്ത്രീയഃ

സ്ത്രീ ബുദ്ധേര സ്ഥിരത്വാത്തു ദോഷൈശ്ചാന്യോപിയേ വൃതാഃ

(8:77)

(നിഷ്‌കാമനായ ഒരുത്തനെ സാക്ഷിയായി സ്വീകരിക്കാം. സ്ത്രീകള്‍ വളരെപ്പേരായാലും അവരുടെ ബുദ്ധിക്കു സ്‌ഥൈര്യമില്ലാത്തതിനാലും അവരെയും മുന്‍പറഞ്ഞ ദോഷികളെയും കടം മുതലായ വിഷയത്തില്‍ സാക്ഷിത്വേന സ്വീകരിക്കരുത്).

സ്ത്രീയെ സാക്ഷ്യത്തിനേ കൊള്ളുകയില്ലെന്ന നിലപാടുമായി ഇസ്‌ലാം വിയോജിക്കുന്നു. അവളെ സാക്ഷിയാക്കാമെന്നുതന്നെയാണ് ഇസ്‌ലാമിന്റെ നിലപാട്. എന്നാല്‍, അവളുടെ സാക്ഷ്യത്തിന് വ്യത്യസ്ത തലങ്ങളുണ്ട്. വിവാഹമോചനത്തെയും മരണസമയത്തെ വസ്വിയത്തിനെയും കുറിച്ച് പ്രതിപാദിക്കുമ്പോള്‍ ഖുര്‍ആന്‍ അവക്ക് രണ്ടു സാക്ഷികള്‍ വേണമെന്ന് നിഷ്‌കര്‍ഷിക്കുന്നുണ്ട് (65:2, 5:106).ഇവിടെയെല്ലാം സ്ത്രീയായിരുന്നാലും പുരുഷനായിരുന്നാലും രണ്ടു സാക്ഷികളാണ് വേണ്ടതെന്ന അഭിപ്രായക്കാരാണ് പ്രമുഖരായ ഇസ്‌ലാമിക പണ്ഡിതന്മാര്‍. അതുപോലെതന്നെ ആര്‍ത്തവം, പ്രസവം തുടങ്ങിയ കാര്യങ്ങളുമായി ബന്ധപ്പെട്ട വിഷയങ്ങളില്‍ സ്ത്രീകളുടെ സാക്ഷ്യം മാത്രമേ സ്വീകാര്യമാകൂ എന്ന കാര്യത്തില്‍ ഇസ്‌ലാമിക പണ്ഡിതന്മാര്‍ക്കിടയില്‍ അഭിപ്രായാന്തരങ്ങളൊന്നുമില്ല. സാദാചാരലംഘനം ആരോപിക്കപ്പെടുന്ന ഘട്ടങ്ങളില്‍ സത്യം ചെയ്യുകയും സ്വയം സാക്ഷ്യം വഹിക്കുകയും ചെയ്യേണ്ടിവരുമ്പോഴും സ്ത്രീ-പുരുഷ വ്യത്യാസങ്ങളൊന്നുമില്ലെന്നതാണ് ഖുര്‍ആനിക നിലപാട്. (ഖുര്‍ആന്‍ 24:6-9). എന്നാല്‍, കടമിടപാടുകളുടെ സ്ഥിതി ഇതില്‍നിന്ന് വ്യത്യസ്തമാണ്. സാക്ഷ്യത്തിനുതന്നെ സ്ത്രീകളെ കൊള്ളുകയില്ലായെന്ന ‘മത‘ വീക്ഷണം പുലര്‍ത്തുന്ന കാലത്താണ് സ്ത്രീയെ സാക്ഷ്യത്തിന് കൊള്ളുമെന്നും കടമിടപാടുകളുടെ കാര്യത്തില്‍ രണ്ടു സ്ത്രീകള്‍ ഒരു പുരുഷനു പകരം സാക്ഷ്യം വഹിച്ചാല്‍ മതിയെന്നുമുള്ള നിയമം ഖുര്‍ആന്‍ പ്രഖ്യാപിക്കുന്നത്. എന്തുകൊണ്ട് ഒരു പുരുഷനുപകരം രണ്ട് സ്ത്രീകള്‍ വേണം? ഉത്തരവും ഖുര്‍ആന്‍ തന്നെ പറയുന്നുണ്ട്: ”അവരില്‍ ഒരുവള്‍ക്ക് തെറ്റു പറ്റിയാല്‍ മറ്റവള്‍ അവളെ ഓര്‍മിപ്പിക്കുവാന്‍ വേണ്ടി”.

സത്യത്തില്‍ ഈ ഖുര്‍ആനിക നിര്‍ദേശം അതിന്റെ ദൈവികത മനസ്സിലാക്കിത്തരികയാണ് ചെയ്യുന്നത്; സ്ത്രീയെയും പുരുഷനെയും വ്യക്തമായി അറിയാവുന്ന സ്രഷ്ടാവിന്റെ നിയമസംഹിതയാണ് ഖുര്‍ആന്‍ എന്ന വസ്തുത. സ്ത്രീയെ തരം താഴ്ത്തുകയല്ല പ്രത്യുത അവളുടെ അബലതകള്‍ മനസ്സലാക്കുകയാണ് ഇവിടെ ഖുര്‍ആന്‍ ചെയ്യുന്നത്. നീതി നിര്‍വഹണത്തിന് ഉപയുക്തമാകുംവിധമായിരിക്കണം സ്ത്രീയുടെയും പുരുഷന്റെയും ഓരോ രംഗത്തെയും പങ്കാളിത്തം നിര്‍ണയിക്കേണ്ടതെന്ന ഖുര്‍ആനിന്റെ പൊതുതത്ത്വംതന്നെയാണ് ഇവിടെയും തെളിഞ്ഞുകാണുന്നത്. താഴെ പറയുന്ന വസ്തുതകള്‍ ശ്രദ്ധിച്ചാല്‍ ഇക്കാര്യം ബോധ്യമാവും.

ഒന്ന്: ഈ സൂക്തത്തില്‍ കടമിടപാടുകളെക്കുറിച്ചാണ് പരാമര്‍ശിച്ചിരിക്കുന്നത്. സാമ്പത്തിക ബാധ്യത പുരുഷന്മാരിലാണ് ഇസ്‌ലാം നിക്ഷിപ്തമാക്കുന്നതെന്നതിനാല്‍തന്നെ സാമ്പത്തിക ക്രയവിക്രയങ്ങളില്‍ പൊതുവേ പുരുഷന്മാരായിരിക്കും പങ്കാളികളായുണ്ടാവുക. ഇസ്‌ലാമിക സമൂഹത്തില്‍ പരസ്ത്രീ-പുരുഷ സംഗമം പ്രോല്‍സാഹിപ്പിക്കപ്പെടാത്തതിനാല്‍ പുരുഷന്മാര്‍ പരസ്പരമുള്ള ഇടപാടുകളിലും അവര്‍ മാത്രം വിഹരിക്കുന്ന രംഗങ്ങളിലും സ്ത്രീകള്‍ സാക്ഷികളായുണ്ടാവുക സ്വാഭാവികമല്ല. ഇടപാടുകള്‍ക്ക് സ്ത്രീകള്‍ സാക്ഷികളാണെങ്കില്‍തന്നെ അവര്‍ ഇസ്‌ലാമികമായ അച്ചടക്കം പാലിക്കാന്‍ ബാധ്യസ്ഥരുമാണ്. അങ്ങനെ അച്ചടക്കം പാലിക്കപ്പെടുന്ന അവസ്ഥയില്‍ ഇടപാടുകളുമായി ബന്ധപ്പെട്ട വ്യക്തികളെ വേണ്ട വിധം തിരിച്ചറിയാന്‍ കഴിഞ്ഞുകൊള്ളണമെന്നില്ല.

രണ്ട്: സ്ത്രീകള്‍ പൊതുവേ വികാരജീവികളാണ്. ചടുലമായ വികാരത്താല്‍ സ്വാധീനിക്കപ്പെടുന്ന സ്ത്രീ സത്യത്തില്‍നിന്നും വ്യതിചലിച്ചേക്കാന്‍ ഇടയുണ്ട്. സാക്ഷ്യം വഹിക്കപ്പെടുന്നത് സാക്ഷിനില്‍ക്കുന്നവളുടെ അസൂയയെ ഇളക്കിവിടാന്‍ മാത്രം സൗന്ദര്യമുള്ളവളുടെ കാര്യത്തിലായിരിക്കാം. അല്ലെങ്കില്‍ അവളിലെ മൃദുല വികാരങ്ങളെ തൊട്ടുണര്‍ത്താന്‍ പോന്ന ഒരു യുവാവിന്റെ കാര്യത്തിലായിരിക്കാം. മാതൃത്വത്തെ തഴുകിയുണര്‍ത്തുന്ന സാഹചര്യങ്ങളുടെ സ്വാധീനമുണ്ടാകാനും സാധ്യതയുണ്ട്. ഈ അവസ്ഥകളിലെ വൈകാരിക സമ്മര്‍ദങ്ങള്‍ അവളുടെ സാക്ഷ്യത്തെ സ്വാധീനിക്കാനിടയുണ്ട്.

മൂന്ന്: സ്ത്രീയുടെ ശാരീരികമായ പ്രത്യേകതകള്‍ അവളില്‍ പല തരത്തിലുള്ള പ്രയാസങ്ങളുമുണ്ടാക്കാറുണ്ട്. ആര്‍ത്തവത്തിന് തൊട്ടുമുമ്പുള്ള ദിവസങ്ങളിലെ മനഃസംഘര്‍ഷം, ഗര്‍ഭധാരണത്തിന്റെ ആദ്യനാളുകളിലെ ശാരീരിക- മാനസിക പ്രശ്‌നങ്ങള്‍, ആര്‍ത്തവ വിരാമത്തോടനുബന്ധിച്ചുള്ള പ്രയാസങ്ങള്‍, പ്രസവകാലത്തെ പ്രശ്‌നങ്ങള്‍,ഗര്‍ഭഛിദ്രമുണ്ടാക്കുന്ന മാനസികാഘാതം ഇങ്ങനെ ഒട്ടനവധി പ്രശ്‌നങ്ങള്‍ സ്ത്രീകള്‍ മാത്രം നേരിടേണ്ടവയാണ്. ഈ സാഹചര്യങ്ങളില്‍ ശാരീരിക പ്രയാസങ്ങള്‍ക്കുപുറമെ ഒട്ടനവധി മാനസിക പ്രശ്‌നങ്ങള്‍ക്കും സ്ത്രീകള്‍ വിധേയരാവുന്നുവെന്നാണ് ശാസ്ത്രീയ ഗവേഷണങ്ങള്‍ വ്യക്തമാക്കുന്നത്. മനോമാന്ദ്യം (slow mindedness), ഏകാഗ്രതയില്ലായ്മ, ഓര്‍മക്കുറവ് തുടങ്ങിയവ ഈ സാഹചര്യങ്ങളിലെ മാനസിക പ്രശ്‌നങ്ങളാണെന്നാണ് കണ്ടെത്തിയിട്ടുള്ളത്. സ്ത്രീകളുടെ സാക്ഷ്യത്തെക്കുറിച്ച് പറയുമ്പോള്‍ ഇത്തരം സാഹചര്യങ്ങളെകൂടി കണക്കിലെടുക്കേണ്ടതുണ്ടല്ലോ. ഒരു പുരുഷനുപകരം രണ്ടു സ്ത്രീകള്‍ സാക്ഷികളാവണമെന്ന് പറഞ്ഞ സൂക്തത്തില്‍ ‘ഒരുവള്‍ക്ക് തെറ്റിയാല്‍ മറ്റെവള്‍ ഓര്‍മിപ്പിക്കാന്‍ വേണ്ടി‘യെന്നാണ് പറഞ്ഞിട്ടുള്ളതെന്ന കാര്യം പ്രത്യേകം ശ്രദ്ധേയമാണ്.

സത്യത്തില്‍, ഈ ഖുര്‍ആനിക നിയമം സ്ത്രീകളുടെ വിലയിടിക്കുകയല്ല, പ്രത്യുത അവളുടെ അബലതകളും പ്രയാസങ്ങളും മനസ്സിലാക്കി അതിനുള്ള പരിഹാരമാര്‍ഗം നിര്‍ദേശിക്കുകയും അവള്‍ക്കുകൂടി പുരുഷനെപ്പോലെ സാക്ഷിയാകുവാനുള്ള അവസരം നല്‍കുകയുമാണ് ചെയ്യുന്നത്. മനോമാന്ദ്യത്തിന്റെയും ഓര്‍മക്കുറവിന്റെയും അവസ്ഥകളില്‍ ഒരുവളെ തിരുത്താന്‍ മറ്റവള്‍ക്ക് സാധിക്കുകയെന്നതാണ് ഇതിന്റെ താല്‍പര്യം. അതേസമയം,സ്ത്രീകളുമായി ബന്ധപ്പെട്ട വിഷയങ്ങളിലും അവളുടേതായ ഇടപാടുകളിലും ഒറ്റ സ്ത്രീയുടെ സാക്ഷ്യംതന്നെ പൂര്‍ണമായി പരിഗണിക്കപ്പെടുകയും ചെയ്യും. പ്രകൃതിമതത്തിന്റെ നിയമ നിര്‍ദേശങ്ങളെല്ലാം പ്രകൃതിയുമായി പൂര്‍ണമായും പൊരുത്തപ്പെടുന്നതാണെന്ന യാഥാര്‍ഥ്യമാണ് ഇവിടെ നമുക്ക് കാണാന്‍ കഴിയുന്നത്.

വിഷയവുമായി ബന്ധപ്പെട്ട വീഡിയോ

അനന്തര സ്വത്തില്‍ പുരുഷന് സ്ത്രീയുടേതിന്റെ ഇരട്ടി അവകാശമുണ്ടെന്നാണല്ലോ ഖുര്ആന്‍ അനുശാസിക്കുന്നത്. ഇത് വ്യക്തമായ വിവേചനമല്ലേ?

ത്യത്തില്‍, സ്ത്രീകള്‍ക്ക് അനന്തരസ്വത്ത് നല്‍കുവാന്‍ ആഹ്വാനം ചെയ്യുന്ന ഏക മതഗ്രന്ഥമാണ് ഖുര്‍ആന്‍. പരിഷ്‌കൃതമെന്നു വിശേഷിപ്പിക്കപ്പെടുന്ന പല രാജ്യങ്ങളും ഇരുപതാം നൂറ്റാണ്ടില്‍ മാത്രമാണ് സ്ത്രീക്ക് അനന്തര സ്വത്തില്‍ അവകാശം നല്‍കിയത്. ഖുര്‍ആനാകട്ടെ ഏഴാം നൂറ്റാണ്ടില്‍തന്നെ അസന്നിഗ്ധമായി പ്രഖ്യാപിച്ചു.”മാതാപിതാക്കളും അടുത്ത ബന്ധുക്കളും വിട്ടേച്ചുപോയ സ്വത്തില്‍ പുരുഷന്മാര്‍ക്ക് ഓഹരിയുണ്ട്. മാതാപിതാക്കളും അടുത്ത ബന്ധുക്കളും വിട്ടേച്ചുപോയ സ്വത്തില്‍ സ്ത്രീകള്‍ക്കും അവകാശമുണ്ട്” (4:7).

ബൈബിള്‍ പഴയനിയമപ്രകാരം പുത്രന്മാരുണ്ടെങ്കില്‍ അവര്‍ക്കു മാത്രമാണ് അനന്തര സ്വത്തില്‍ അവകാശമുള്ളത്. മരിച്ചയാളുടെ സ്വത്തുക്കള്‍ മക്കള്‍ക്കാണ് ഭാഗിച്ചുകൊടുക്കപ്പെടുന്നതെന്ന് സൂചിപ്പിക്കുന്ന വചനങ്ങള്‍ കാണാനാവും (ആവര്‍ത്തനം 21:15-17). പുത്രന്മാരില്ലെങ്കില്‍ പുത്രിമാര്‍ക്ക് അവകാശം നല്‍കണമെന്ന് നിര്‍ദേശമുണ്ട്. ”പുത്രനില്ലാതെ മരിക്കുന്നയാളുടെ പിന്തുടര്‍ച്ചാവകാശം പുത്രിക്കു ലഭിക്കുമാറാകണം” (സംഖ്യ 27:8). വിധവയ്ക്കുപോലും ഭര്‍ത്താവിന്റെ സ്വത്തില്‍ അവകാശമുണ്ടായിരുന്നില്ല (റവ. എ.സി. ക്ലെയ്റ്റണ്‍: ബൈബിള്‍ നിഘണ്ടു, പുറം 113).

ബൈബിള്‍ പുതിയ നിയമത്തിലാകട്ടെ ദായക്രമത്തെക്കുറിച്ച് പുതിയ നിയമങ്ങളൊന്നുംതന്നെ കാണാന്‍ കഴിയുന്നില്ല. ക്രൈസ്തവസഭ പൊതുവെ ഇക്കാര്യത്തില്‍ പഴയ നിയമത്തിലെ കല്‍പനകള്‍ അനുസരിക്കുകയായിരുന്നു ചെയ്തിരുന്നത്. അതുകൊണ്ടുതന്നെ ക്രൈസ്തവ ഭൂരിപക്ഷ പ്രദേശങ്ങളില്‍ ഈ അടുത്ത കാലംവരെ അനന്തരാവകാശം മാത്രമല്ല, സ്വത്തു സമ്പാദിക്കുവാന്‍ വരെ സ്ത്രീകള്‍ക്ക് അവകാശം നല്‍കപ്പെട്ടിരു ന്നില്ല. സ്വന്തം പേരില്‍ സ്വത്ത് സമ്പാദിക്കാന്‍ ന്യൂയോര്‍ക്കിലെ സ്ത്രീകളെ അനുവദിക്കുന്നത് 1848-ലാണ്. 1850-ലാണ് അമേരിക്കയിലെ എല്ലാ സംസ്ഥാനങ്ങളിലും സ്ത്രീകള്‍ക്ക് അനന്തരാവകാശം നല്‍കുന്ന നിയമം പ്രാബല്യത്തിലായത്.

പുരുഷന്റെ സ്വകാര്യ സ്വത്തായി സ്ത്രീയെ കണക്കാക്കുകയും അതുപ്രകാരമുള്ള നിയമങ്ങളാവിഷ്‌കരിക്കുകയും ചെയ്യുന്ന ഹൈന്ദവഗ്രന്ഥങ്ങള്‍ അവളെ അനന്തര സ്വത്തില്‍ പങ്കാളിയാക്കുന്നതിനെക്കുറിച്ച് പരാമര്‍ശിക്കുന്നുപോലുമില്ല. ഭര്‍ത്താവിനു ദാനം ചെയ്യാനും വില്‍ക്കാനും ഉപയോഗിക്കുവാനുമെല്ലാം അവകാശമുള്ള സ്വകാര്യ സ്വത്താണ് ഭാര്യ എന്നാണ് ഇതിഹാസകഥകള്‍ വായിച്ചാല്‍ നമുക്ക് ബോധ്യപ്പെടുക. അതിഥി പൂജക്കുവേണ്ടി സ്വന്തം ഭാര്യയെ നല്‍കുന്ന സുദര്‍ശനനും (മഹാഭാരതം അനുശാസനപര്‍വം) ഭാര്യയെ വസിഷ്ഠന് നല്‍കുന്ന മിത്രസഹനും (ശാന്തിപര്‍വം) നല്‍കുന്ന സൂചനയിതാണ്. പിതാവിന്റെ സ്വത്തില്‍ പെണ്‍മക്കള്‍ക്ക് അവകാശമുള്ളതായി സൂചിപ്പിക്കുന്ന വചനങ്ങളൊന്നും ഹിന്ദുമതഗ്രന്ഥങ്ങളില്‍ കാണാന്‍ കഴിയില്ല. പുത്രന്മാരാണ് അനന്തര സ്വത്തില്‍ അവകാശികളായിട്ടുള്ളവരെന്നാണ് മനുസ്മൃതിയുടെ നിയമം.

ഉൗര്‍ദ്ധ്വം പിതുശ്ച മാതുശ്ച സമേത്യ ഭ്രാതരഃ സ്സമം

ഭജേരന്‍ പൈതൃകം രിക്ഥമനീശാസ്‌തേ ഹി ജീവതൊ (9:104)

(മാതാപിതാക്കള്‍ രണ്ടുപേരും മരിച്ചശേഷം പുത്രന്മാരെല്ലാം ഒന്നുചേര്‍ന്ന് അവരുടെ സമ്പാദ്യം വിഭജിച്ച് എടുക്കണം. എന്തുകൊണ്ടെന്നാല്‍ മാതാപിതാക്കന്മാര്‍ ഇരിക്കുമ്പോള്‍ അവരുടെ ധനം പുത്രന്മാര്‍ക്കു സ്വാധീനമല്ല).

മാതാപിതാക്കളുടെ സ്വത്തില്‍ പുത്രന്മാര്‍ക്കും പുത്രിമാര്‍ക്കുമുള്ള അവകാശം ഖുര്‍ആന്‍ അംഗീകരിക്കുന്നു. പുത്രന്മാര്‍ക്കും പുത്രിമാര്‍ക്കും മാത്രമല്ല, മാതാപിതാക്കള്‍ക്കും ഭാര്യാഭര്‍ത്താക്കന്മാര്‍ക്കും സഹോദരീസഹോദരന്മാര്‍ക്കുമെല്ലാം മരണപ്പെട്ടയാളുടെ സ്വത്തിലുള്ള അവകാശം എത്രയാണെന്നും എങ്ങനെയാണെന്നുമെല്ലാം ഖുര്‍ആന്‍ വ്യക്തമായി പ്രതിപാദിക്കുന്നുണ്ട്. ഇതിന്റെ ഭാഗം മാത്രമാണ് പുത്രന്റെയും പുത്രിയുടെയും അവകാശം. അനന്തരാവകാശത്തെക്കുറിച്ച് വിശദമായി വിവരിക്കുന്ന ഖുര്‍ആന്‍ സൂക്തങ്ങളുടെ (4:11,12) തുടക്കം ഇങ്ങനെയാണ്: ”നിങ്ങളുടെ സന്താനങ്ങളുടെ കാര്യത്തില്‍ അല്ലാഹു നിങ്ങള്‍ക്ക് നിര്‍ദേശം നല്‍കുന്നു. ആണിന് രണ്ട് പെണ്ണിന്‍േറതിന് തുല്യമായ ഓഹരിയാണുള്ളത്” (4:11) മരിച്ചയാളുടെ പുത്രന് പുത്രിക്കു ലഭിക്കുന്നതിന്റെ ഇരട്ടി സ്വത്ത് അനന്തരമായി ലഭിക്കുമെന്ന് സാരം.

ഇത് സ്ത്രീകളോടുള്ള അവഗണനയാണോ? പുരുഷപക്ഷപാതം പ്രകടിപ്പിക്കുന്ന നിയമമാണോ? വിധി പറയുന്നതിനുമുമ്പ് താഴെ പറയുന്ന വസ്തുതകള്‍ മനസ്സിലാക്കുക.

ഒന്ന്: സ്ത്രീക്ക് സ്വത്ത് സമ്പാദിക്കുവാനുള്ള അവകാശം ഖുര്‍ആന്‍ അംഗീകരിക്കുന്നു. എത്ര വേണമെങ്കിലും സമ്പാദിക്കാം. പ്രസ്തുത സമ്പാദ്യത്തില്‍ പുരുഷന് യാതൊരു അവകാശവുമില്ല. അവളുടെ സമ്പാദ്യം അവളുടേതു മാത്രമാണ്.

രണ്ട്: സ്ത്രീയുടെയോ കുട്ടികളുടെയോ മാതാപിതാക്കളുടെയോ സംരക്ഷണത്തിനുള്ള ഉത്തരവാദിത്തം ഒരു പരിതസ്ഥിതിയിലും സ്ത്രീയുടെ ബാധ്യതയായിത്തീരുന്നില്ല. എത്രതന്നെ സമ്പത്തുള്ളവളായിരുന്നാലും തന്റെയും മക്കളുടെയും മാതാപിതാക്കളുടെയും ഭര്‍ത്താവിന്റെയും ചെലവ് വഹിക്കാന്‍ സ്ത്രീക്ക് ബാധ്യതയില്ലെന്നര്‍ഥം.

മൂന്ന്: വിവാഹാവസരത്തില്‍ വരനില്‍നിന് വിവാഹമൂല്യം നേടിയെടുക്കുവാന്‍ സ്ത്രീക്ക് അവകാശമുണ്ട്. പ്രസ്തുത വിവാഹമൂല്യം (മഹ്ര്‍) അവളുടെ സമ്പത്തായാണ് ഗണിക്കപ്പെടുന്നത്.

നാല്: കുടുംബത്തിന്റെ സംരക്ഷണം പുരുഷന്റെ ബാധ്യതയാണ്. ഭാര്യയുടെയും കുട്ടികളുടെയും ചെലവുകള്‍ വഹിക്കാന്‍ പുരുഷന്‍ ബാധ്യസ്ഥനാണ്. മാതാപിതാക്കളെയും അടുത്ത ബന്ധുക്കളെയും സംരക്ഷിക്കേണ്ട ബാധ്യതയും പുരുഷന്റെതുതന്നെ. എല്ലാവിധ സാമ്പത്തിക ബാധ്യതയും പുരുഷനാണുള്ളതെന്നര്‍ഥം.

അഞ്ച്: ഭാര്യ എത്രതന്നെ വലിയ പണക്കാരിയാണെങ്കിലും അവളുടെ സ്വത്തില്‍നിന്ന് അവളുടെ അനുവാദമില്ലാതെ ഒന്നും എടുത്തുപയോഗിക്കുവാന്‍ ഭര്‍ത്താവിന് അവകാശമില്ല.

ഇനി പറയൂ, സ്ത്രീയോട് നീതി പുലര്‍ത്തുകയാണോ അതല്ല വിവേചനം കാണിക്കുകയാണോ എന്താണ് ഖുര്‍ആന്‍ ചെയ്തിട്ടുള്ളത്?

സ്ത്രീക്ക് ലഭിക്കുന്ന അനന്തരസ്വത്ത് അവളുടേത് മാത്രമാണ്. മറ്റാര്‍ക്കും അതില്‍ യാതൊരു പങ്കുമില്ല. പുരുഷന് ലഭിക്കുന്നതോ?അവന്‍ വിവാഹമൂല്യം നല്‍കണം, സ്ത്രീയുടെ സംരക്ഷണം ഏറ്റെടുക്കണം, അവള്‍ക്കും കുട്ടികള്‍ക്കുമുള്ള ചെലവുകള്‍ വഹിക്കണം. എല്ലാം പുരുഷന്റെ ഉത്തരവാദിത്തം. അപ്പോള്‍ സ്ത്രീയെയാണോ പുരുഷനെയാണോ ഖുര്‍ആന്‍ കൂടുതല്‍ പരിഗണിച്ചിരിക്കുന്നത്?

സാമ്പത്തിക ബാധ്യതകള്‍ പുരുഷനില്‍ നിക്ഷിപ്തമാക്കുന്ന മറ്റു മതഗ്രന്ഥങ്ങളെല്ലാം പ്രസ്തുത ബാധ്യതകള്‍ക്കു പകരമായി അനന്തരാവകാശം പുരുഷനില്‍ പരിമിതപ്പെടുത്തുകയാണ് ചെയ്തിരിക്കുന്നത്. ഖുര്‍ആനാകട്ടെ എല്ലാ സാമ്പത്തിക ബാധ്യതകളും പുരുഷനാണെന്ന് പഠിപ്പിക്കുന്നതോടൊപ്പംതന്നെ സ്ത്രീക്ക് അനന്തരാവകാശം നല്‍കുകയും ചെയ്യുന്നു. പുരുഷന്റെ പകുതി അനന്തരസ്വത്ത് നല്‍കിക്കൊണ്ട് അത് അവളെ ബഹുമാനിക്കുകയാണ് ചെയ്തിരിക്കുന്നത്.

ഖുര്‍ആനിന്റെതല്ലാത്ത ഏത് നിര്‍ദേശമാണ് ഈ രംഗത്ത് വിമര്‍ശകരുടെ കൈവശമുള്ളത്? രണ്ട് നിര്‍ദേശങ്ങള്‍ ഉന്നയിക്കപ്പെടാം.

1.സ്ത്രീക്ക് പുരുഷന്റെ ഇരട്ടി സ്വത്ത് നല്‍കുക. സാമ്പത്തിക ബാധ്യതകള്‍ സ്ത്രീയില്‍ നിക്ഷപ്തമാക്കുക.

2.സ്ത്രീക്കും പുരുഷനും സ്വത്തില്‍ തുല്യാവകാശം നല്‍കുക. സാമ്പത്തിക ബാധ്യതകള്‍ തുല്യമായി വീതിച്ചെടുക്കുക.

ഈ രണ്ട് നിര്‍ദേശങ്ങളിലും സാമ്പത്തിക ബാധ്യതകള്‍ സ്ത്രീയില്‍ കെട്ടിയേല്‍പിക്കുകയാണ് ചെയ്യുന്നത്. സ്‌ത്രൈണപ്രകൃതിക്ക് വിരുദ്ധമായ ഒരു ആശയമാണിത്. ഗര്‍ഭകാലത്തും പ്രസവകാലത്തുമെല്ലാം പുരുഷന്റെ പരിരക്ഷയും സഹായവുമാണ് അവള്‍ കാംക്ഷിക്കുന്നത്. സാമ്പത്തിക ബാ ധ്യതകള്‍ ഒരു നിയമമെന്ന നിലയില്‍ സ്ത്രീയുടെ ചുമലില്‍ വെക്കുന്നത് ദൂരവ്യാപകമായ പ്രത്യാഘാതങ്ങള്‍ക്കു കാരണമാകും. അതുകൊണ്ട് സ്ത്രീക്ക് ഏറ്റവും അനുഗുണമായ നിയമംതന്നെയാണ് സ്വത്തവകാശത്തിന്റെ വിഷയത്തില്‍ ഖുര്‍ആന്‍ മുന്നോട്ടുവെച്ചിരിക്കുന്നത്.

‘നിങ്ങളുടെ ഭാര്യമാര്‍ നിങ്ങളുടെ കൃഷിയിടമാകുന്നു. അതിനാല്‍ നിങ്ങള്‍ ഇച്ഛിക്കുംവിധം നിങ്ങള്‍ക്ക് നിങ്ങളുടെ കൃഷിയിടത്തില്‍ ചെല്ലാവുന്നതാണ്‘ (2:223) എന്ന ഖുര്‍ആന്‍ സൂക്തമാണ് ഇവിടെ വിമര്‍ശിക്കപ്പെട്ടിരിക്കുന്നത്. ഭാര്യയെ കൃഷിയിടത്തോടുപമിക്കുന്ന ഖുര്‍ആന്‍ അവളെ വെറുമൊരു ഉല്‍പാദനോപകരണം മാത്രമാക്കിയെന്നാണ് ആക്ഷേപം.

ഖുര്‍ആനില്‍ ഒരുപാട് ഉപമാലങ്കാരങ്ങളുണ്ട്. സ്ത്രീയെ കൃഷിയിടത്തോടും വസ്ത്രത്തോടും ഉപമിക്കുന്നത് അവയില്‍ ചിലതുമാത്രം. ഉപമകള്‍ക്ക് ഒരു പ്രത്യേകതയുണ്ട്. ഓരോരുത്തര്‍ക്കും അവരുടെ മനോഗതംപോലെ അവയെ വ്യാഖ്യാനിക്കാന്‍ കഴിയും. പ്രസ്തുത വ്യാഖ്യാനങ്ങള്‍ വ്യാഖ്യാതാവിന്റെ മനസ്സിന്റെ നിമ്‌നോന്നതികളെയാണ് പ്രതിഫലിപ്പിക്കുക. കൃഷിസ്ഥലത്തോട് ഭാര്യയെ ഉപമിച്ചതിനാല്‍ കൃഷിയിടം ചവിട്ടിമെതിക്കുന്നതുപോലെ അവളെ ചവിട്ടിമെതിക്കാമെന്നും അത് വില്‍ക്കുന്നതുപോലെ സ്ത്രീയെ ഏതു സമയത്തും വില്‍പന നടത്താമെന്നും അതിനെ ഉഴുതുമറിക്കുന്നതുപോലെ അവളെ ഉഴുതുമറിക്കാമെന്നുമാണ് ഖുര്‍ആന്‍ പറയുന്നതെന്ന് ഒരാള്‍ക്ക് വാദിക്കാം. ഭാര്യയെ വസ്ത്രത്തോടുപമിച്ചതില്‍നിന്ന് അവളെ വസ്ത്രം മാറുന്നതുപോലെ മാറ്റുവാനാണ് ഖുര്‍ആന്‍ കല്‍പിക്കുന്നതെന്ന് വ്യാഖ്യാനിക്കാനും സാധിക്കും. പക്ഷേ, ഈ വ്യാഖ്യാനങ്ങളെല്ലാം വ്യാഖ്യാതാക്കളുടെ മനോഗതിയെയും മുന്‍ധാരണകളെയുമല്ലാതെ മറ്റൊന്നിനെയും പ്രതിഫലിപ്പിക്കുന്നില്ല എന്നല്ലോ മനഃശാസ്ത്ര മതം.

ഏതൊരു ഗ്രന്ഥത്തിലെയും ഉപമാലങ്കാരങ്ങളെ വ്യാഖ്യാനിക്കുവാന്‍ ആ ഗ്രന്ഥത്തിന്റെ ഇതിവൃത്തത്തെയും അത് പ്രഖ്യാപിക്കുന്ന ആദര്‍ശത്തെയും അത് മുന്നോട്ടുവെക്കുന്ന സാമൂഹികസംവിധാനത്തെയും കുറിച്ച അടിസ്ഥാന വസ്തുതകള്‍ അറിയേണ്ടതുണ്ട്. ‘സ്ത്രീകള്‍ക്ക് ബാധ്യതയുള്ളപോലെ അവകാശങ്ങളുമുണ്ട്‘(2:228) എന്ന ഖുര്‍ആന്‍ സൂക്തം സ്ത്രീപുരുഷബന്ധത്തെക്കുറിച്ച അതിന്റെ വീക്ഷണത്തെ സംബന്ധിച്ച അടിസ്ഥാനപരമായ അറിവ് നല്‍കുന്നുണ്ട്. ‘ഭൂമിയിലെ വിഭവങ്ങളില്‍ ഉത്തമമാണ് സദ്‌വൃത്തയായ സ്ത്രീ‘(മുസ്‌ലിം) എന്ന പ്രവാചക വചനം ഇതിന് അനുബന്ധമായി സ്ഥിതി ചെയ്യുന്നു. ഈ അടിത്തറയില്‍നിന്നുകൊണ്ട് സ്ത്രീയെക്കുറിച്ച ഉപമകള്‍ മനസ്സിലാക്കിയാലേ പ്രസ്തുത ഉപമകളുടെ സൗന്ദര്യം ആസ്വദിക്കാനാവൂ.

സ്ത്രീയെ വസ്ത്രത്തോടുപമിച്ച ഖുര്‍ആന്‍ എന്താണ് അര്‍ഥമാക്കുന്നത്? ശരീരവുമായി ഒട്ടിച്ചേര്‍ന്നുനില്‍ക്കുന്ന ഭൗതികമായി ഏറ്റവും അടുത്ത വസ്തുവാണ് വസ്ത്രം. അത് അന്യന്‍ കാണാതിരിക്കേണ്ട ശരീരഭാഗങ്ങളെ മറച്ചുവെക്കുന്നു. കാലാവസ്ഥയുടെ അസുഖകരമായ അവസ്ഥകളില്‍നിന്ന് ശരീരത്തെ സംരക്ഷിക്കുന്നത് വസ്ത്രമാണ്. മനുഷ്യന്റെ അന്തസ്സിന്റെ പ്രകടനവും വസ്ത്രത്തില്‍ കുടികൊള്ളുന്നു. സൗന്ദര്യവും ആനന്ദവും വര്‍ധിപ്പിക്കുന്നതിനും വസ്ത്രം ഉപയോഗിക്കുന്നു. സര്‍വോപരി ഒരാളുടെ സംസ്‌കാരത്തിന്റെ പ്രകടനമാണ് വസ്ത്രം. ഖുര്‍ആന്‍ സ്ത്രീയെ പുരുഷന്റെ വസ്ത്രമായി മാത്രമല്ല പറഞ്ഞിട്ടുള്ളതെന്ന കാര്യം പ്രത്യേകം പ്രസ്താവ്യമാണ്. ‘അവര്‍ നിങ്ങള്‍ക്കൊരു വസ്ത്രമാണ്, നിങ്ങള്‍ അവര്‍ക്കും ഒരു വസ്ത്രമാണ്‘ (2:187) എന്നാണ് ഖുര്‍ആന്‍ പറയുന്നത്. ഖുര്‍ആനിന്റെ ഉപമ എത്ര സുന്ദരം! കൃത്യം. പരസ്പരം വസ്ത്രമാകാതിരിക്കുന്നതല്ലേ ഇന്നത്തെ കുടുംബപ്രശ്‌നങ്ങളുടെ പ്രധാനപ്പെട്ട കാരണം?

സ്ത്രീയെ കൃഷിയിടത്തോടും പുരുഷനെ കൃഷിക്കാരനോടും ഉപമിച്ച ഖുര്‍ആന്‍ എന്താണ് അര്‍ഥമാക്കിയിരിക്കുന്നത്? കൃഷിയിടവും കൃഷിക്കാരനും തമ്മിലുള്ള ബന്ധമറിയാന്‍ കൃഷിക്കാരനോടുതന്നെ ചോദിക്കണം. കൃഷിയിടത്തിനുവേണ്ടി മരിക്കാന്‍ സന്നദ്ധനാണവന്‍. മണ്ണെന്ന് കേള്‍ക്കുമ്പോള്‍ അയാള്‍ വികാരതരളിതനാവും. കൃഷിഭൂമിയുടെ നിയമത്തെക്കുറിച്ച് അറിയുന്നവനാണവന്‍. സ്വന്തം കൃഷിയിടത്തില്‍ അന്യനെ വിത്തിടാന്‍ അയാള്‍ അനുവദിക്കുകയില്ല. അപരന്റെ കൃഷി സ്ഥലത്ത് വിത്തിറക്കാന്‍ അയാളൊട്ട് മുതിരുകയുമില്ല. കൃഷിഭൂമി പാഴാക്കരുത്്. തരിശിടരുത്. വളമിടണം. ജലസേചനം ചെയ്യണം. മണ്ണിന്റെ ഗുണം കൂട്ടണം. മണ്ണൊലിപ്പ് തടയണം.‘നിങ്ങളുടെ ഭാര്യമാര്‍ നിങ്ങളുടെ കൃഷിയിടമാണ്‘ (2:223) എന്ന ഖുര്‍ആനികാധ്യാപനം ശ്രവിക്കുന്ന കര്‍ഷകന് പെണ്ണിനെ കേവലം ഒരു ഉല്‍പാദനയന്ത്രമായി കാണാന്‍ കഴിയില്ല. കൃഷിയിടവും കര്‍ഷകനും തമ്മിലുള്ള ബന്ധത്തിന്റെ ആന്തരികമായ ആഴമറിയാത്തവര്‍ക്ക് ഈ ഉപമ ആസ്വദിക്കാന്‍ കഴിയില്ല. എന്നാല്‍ കൃഷിക്കാരന്റെ സ്ഥിതി അതല്ല. അവന്‍ പ്രസ്തുത ഉപമയുടെ അര്‍ഥം മനസ്സിലാക്കുന്നു. സൗന്ദര്യമുള്‍ക്കൊള്ളുന്നു. ഖുര്‍ആന്‍ സംസാരിക്കുന്നത് പച്ചയായ മനുഷ്യരോടാണ്; സാങ്കല്‍പിക ലോകത്ത് ബുദ്ധി വ്യായാമം ചെയ്യുന്ന‘ജീവി‘കളോടല്ലെന്ന കാര്യം പ്രത്യേകം പ്രസ്താവ്യമാണ്.

സ്ത്രീയെ കൃഷിയിടത്തോടുപമിച്ച ഖുര്‍ആനിക സൂക്തത്തിന്റെ അവതരണ പശ്ചാത്തലംകൂടി മനസ്സിലാക്കുന്നത് നന്നായിരിക്കും. സ്ത്രീകളുമായി ലൈംഗികബന്ധം പുലര്‍ത്തുന്നത് ചില പ്രത്യേക രീതികളിലായിരുന്നാല്‍ അത് പാപമാണെന്നും ജനിക്കുന്ന കുഞ്ഞിന്റെ കണ്ണിനു തകരാറുണ്ടാവുമെന്നുമുള്ള അന്ധവിശ്വാസങ്ങള്‍ മദീനയിലെ യഹൂദര്‍ക്കിടയില്‍ പ്രചരിച്ചിരുന്നു. ഇതിനെക്കുറിച്ച് അനുചരന്മാര്‍ പ്രവാചകനോട് (ﷺ) ചോദിച്ചു: അപ്പോഴാണ് ഈ സൂക്തം അവതരിപ്പിക്കപ്പെട്ടതെന്നാണ് പല ഖുര്‍ആന്‍ വ്യാഖ്യാതാക്കളുടെയും അഭിപ്രായം. ‘നിങ്ങളുടെ ഭാര്യമാര്‍ നിങ്ങളുടെ കൃഷിസ്ഥലമാകുന്നു. അതിനാല്‍ നിങ്ങള്‍ ഇച്ഛിക്കുന്ന വിധം നിങ്ങളുടെ കൃഷിസ്ഥലത്തു ചെല്ലുക‘ എന്ന സൂക്തത്തിന്റെ വിവക്ഷ ഈ പശ്ചാത്തലം വെച്ചുകൊണ്ട് മനസ്സിലാക്കുന്നത് തെറ്റിദ്ധാരണ നീങ്ങാന്‍ സഹായകമാവും. കൃഷിയിടത്തിലേക്ക് പല മാര്‍ഗങ്ങളുപയോഗിച്ച് കടന്നുചെല്ലുന്ന കൃഷിക്കാരനെപ്പോലെ ലൈംഗികബന്ധത്തില്‍ വ്യത്യസ്ത മാര്‍ഗങ്ങളുപയോഗിക്കുന്നതുകൊണ്ട് കുഴപ്പമില്ലെന്നാണ് ഖുര്‍ആന്‍ ഇവിടെ പഠിപ്പിക്കുന്നത്. കൃഷി സ്ഥലത്തുതന്നെയാണ് വിത്തുവിതക്കുന്നതെന്ന് ഉറപ്പുവരുത്തണമെന്നുമാത്രം. ഗുദമൈഥുനമൊഴിച്ചുള്ള മാര്ഗങ്ങളിലൂടെയെല്ലാം ലൈംഗീകാസ്വാദനം നേടാമെന്ന് പഠിപ്പിക്കുന്ന ഈ വചനം പെണ് വിരുദ്ധമാണെന്ന് പറയുന്നവർക്ക് ലൈംഗികതയെക്കുറിച്ചും അതിന്റെ മഴവിൽ വർണങ്ങളിലുള്ള ആസ്വാദനരീതികളെക്കുറിച്ചും യാതൊന്നുമറിയില്ലേയെന്ന് ആരെങ്കിലും സംശയിച്ചാൽ അവരെ കുറ്റം പറയാനാവില്ല. .

  'പുരുഷന്മാര്‍ സ്ത്രീകളുടെമേല്‍ നിയന്ത്രണാധികാരമുള്ളവരാകുന്നു‘(4:34).  

‘പുരുഷന്മാര്‍ക്ക് അവരേക്കാളുപരി ഒരു പദവിയുണ്ട്‘ (2:228).

വിശുദ്ധ ഖുര്‍ആനില്‍ പുരുഷമേധാവിത്തമാരോപിക്കുന്നവര്‍ഉദ്ധരിക്കാറുള്ള സൂക്തങ്ങളാണിവ. ഈ സൂക്തങ്ങള്‍ അറബികളുടെ ആണ്‍കോയ്മാവ്യവസ്ഥിതിയുടെ ഉല്‍പന്നമാണ് ഖുര്‍ആന്‍ എന്ന് വ്യക്തമാക്കുന്നതായി വാദിക്കപ്പെടുന്നു. എന്നാല്‍, വസ്തുതയെന്താണ്?

ആദ്യം ഉദ്ധരിക്കപ്പെട്ട സൂക്തത്തില്‍ പുരുഷന്‍ സ്ത്രീയുടെ മേല്‍‘ഖവ്വാം‘ ആണ് എന്നാണ് ഖുര്‍ആന്‍ പറയുന്നത്. ഒരാളുടെയോ സ്ഥാപനത്തിന്റെയോ കാര്യങ്ങള്‍ യഥോചിതം കൊണ്ടുനടക്കുകയും മേല്‍നോട്ടം വഹിക്കുകയും അതിനാവശ്യമായത് സജ്ജീകരിക്കുകയും ചെയ്യുന്ന വ്യക്തിക്കാണ് അറബിയില്‍ ‘ഖവ്വാം‘ എന്നും ‘ഖയ്യിം‘എന്നുമെല്ലാം പറയുന്നത്. അത് ഒരു അവകാശത്തേക്കാളധികം ഉത്തരവാദിത്തത്തെയാണ് ദ്യോതിപ്പിക്കുന്നത്. സ്ത്രീയും കുട്ടികളും അടങ്ങുന്ന കുടുംബമെന്ന സ്ഥാപനത്തിന്റെ നിയന്ത്രണാധികാരം,അതല്ലെങ്കില്‍ നിയന്ത്രണത്തിനുള്ള ഉത്തരവാദിത്തം പുരുഷനിലാണ് നിക്ഷിപ്തമായിരിക്കുന്നത് എന്നാണ് പ്രസ്തുത സൂക്തത്തിന്റെ സാരം.

കുടുംബം ഒരു സ്ഥാപനമാണ്. ഇരുപാതികള്‍ക്കും ശാന്തിയും സമാധാനവും സായൂജ്യവും പ്രദാനം ചെയ്യുന്ന മഹത്തായ സ്ഥാപനം. സാമൂഹിക സ്ഥാപനങ്ങളില്‍ ഏറ്റവും പ്രധാനപ്പെട്ടതാണ് കുടുംബമെന്നുള്ളതാണ് വാസ്തവം. ഏതൊരു സ്ഥാപനത്തിനും അതിന്റെ പ്രശ്‌നങ്ങള്‍ പരിഹരിക്കുന്നതിനും ഉയര്‍ച്ചക്കും വളര്‍ച്ചക്കും വേണ്ടിയത്‌നിക്കുന്നതിനും ഒരു മേലധികാരി ഉണ്ടായിരിക്കണമെന്ന കാര്യം ആരും അംഗീകരിക്കുന്നതാണ്. എത്രതന്നെ ആത്മാര്‍ഥമായ സംരംഭമാണെന്നിരിക്കിലും ഒരു നിയന്ത്രണാധികാരിയുടെ അഭാവത്തില്‍ അത് മുരടിച്ചുപോവുമെന്നത് കാര്യനിര്‍വഹണശാസ്ത്രത്തിന്റെ (administrative science) ബാലപാഠമെങ്കിലും മനസ്സിലാക്കിയിട്ടുള്ളവര്‍ക്ക് അറിയാവുന്നതാണ്്. അപ്പോള്‍ പിന്നെ സമൂഹത്തിന്റെ ധാര്‍മികാടിത്തറയുടെ രൂപീകരണം നടക്കുന്ന കുടുംബത്തിന് ഒരു മേലധികാരി ആവശ്യമില്ലേ? സ്ത്രീയും പുരുഷനും ചേര്‍ന്നുണ്ടാവുന്ന കൂട്ടുസ്ഥാപനമായ കുടുംബത്തിന്റെയും അതില്‍ വളര്‍ന്നുവരുന്ന സന്താനങ്ങളുടെയും അവയോടനുബന്ധിച്ചുണ്ടാവുന്ന ഉത്തരവാദിത്തങ്ങളുടെയും നടത്തിപ്പിന് ഒരു മേല്‍നോട്ടക്കാരന്‍ അത്യാവശ്യമാണ്. അല്ലാത്തപക്ഷം അരാജകത്വവും സര്‍വത്ര വിനാശവുമായിരിക്കും ഫലം.

കുടുംബത്തിന് നായകത്വം വഹിക്കുവാന്‍ ഒരാള്‍ വേണമെന്ന് വ്യക്തം. ആര്‍ക്കാണിതിന് അര്‍ഹതയുള്ളത് എന്നു ചോദിക്കുന്നതിനേക്കാള്‍ ആര്‍ക്കാണതിന് സാധിക്കുകയെന്ന് പരതുന്നതാവും ശരി. ഒന്നുകില്‍ രണ്ടുപേരും കൂടി നായകത്വം വഹിക്കുക. അല്ലെങ്കില്‍ സ്ത്രീ കുടുംബത്തിന്റെ നായകത്വമേറ്റെടുക്കുക. ഇവ രണ്ടും പ്രായോഗികമല്ലെങ്കില്‍ മാത്രം പുരുഷനെ ആ ചുമതല ഏല്‍പിക്കുക എന്ന പൊതുധാരണയുടെ അടിസ്ഥാനത്തില്‍ നാം പ്രശ്‌നത്തെ സമീപിക്കുക; നിഷ്‌കളങ്കതയോടുകൂടി.

ഒരു സ്ഥാപനത്തിന് മേലധികാരിയില്ലാതിരിക്കുന്നതിനേക്കാള്‍ അപകടമാണ് അതിന് രണ്ടു നായകന്മാരുണ്ടാവുകയെന്നത്. സ്ഥാപനങ്ങള്‍ നോക്കിനടത്തിയിരുന്ന അച്ഛന്‍ മരിച്ചാല്‍ ഉടന്‍ അവ വിഭജിച്ചെടുക്കുകയോ അല്ലെങ്കില്‍ മക്കളില്‍ ആരെങ്കിലുമൊരാളെ നിയന്ത്രണാധികാരം ഏല്‍പിക്കുകയോ ചെയ്യാതിരുന്നാലുണ്ടാവാറുള്ള പ്രശ്‌നങ്ങളില്‍ പലപ്പോഴും സ്ഥാപനങ്ങള്‍ തന്നെ തകര്‍ന്ന് നാശമാകാറാണ് പതിവ്. ഒന്നിലധികം നായകന്മാരുള്ള സ്ഥാപനങ്ങളില്‍ നായകത്വത്തിന് വേണ്ടിയുള്ള കിടമല്‍സരങ്ങളും പ്രശ്‌നങ്ങളും കാരണം അതു തകരും. തകരാതെ നിലനില്‍ക്കുന്നുവെങ്കില്‍തന്നെ അതിന്റെ‘ഉല്‍പന്നങ്ങള്‍‘ക്ക് എന്തെങ്കിലും വൈകല്യങ്ങളുണ്ടാവും. കുടുംബത്തിന്റെ അവസ്ഥയും ഇതുതന്നെ. രണ്ടുപേരെയും നായകന്മാരാക്കിയാല്‍ പ്രശ്‌നങ്ങളിലുള്ള സമീപനത്തെക്കുറിച്ച സംഘട്ടനങ്ങളുണ്ടാവും. ഇതു നേതൃത്വത്തിനുവേണ്ടിയുള്ള മല്‍സരത്തില്‍ കലാശിക്കും. അശാന്തമായ കുടുംബാന്തരീക്ഷമായിരിക്കും ഇതിന്റെ ഫലം. അത്തരമൊരു കുടുംബാന്തരീക്ഷത്തില്‍ വളരുന്ന കുഞ്ഞുങ്ങളില്‍ മാനസിക സംഘര്‍ഷങ്ങളും വൈകാരിക താളപ്പിഴകളുമുണ്ടാവും. അത് അടുത്ത തലമുറയില്‍ ധാര്‍മികത്തകര്‍ച്ചക്ക് നിമിത്തമാകും.

കുടുംബത്തിന്റെ നിയന്ത്രണാധികാരം സ്ത്രീക്ക് ഏറ്റെടുക്കുവാന്‍ പറ്റുമോ? അതല്ല പുരുഷനിലാണോ ആ ഉത്തരവാദിത്തം ഏല്‍പിക്കേണ്ടത്? ഈ ചോദ്യത്തിനുള്ള ഉത്തരം ‘കുടുംബത്തിന്റെ നിയന്ത്രണത്തിനാവശ്യം വിചാരമോ അതല്ല വികാരമോ?, എന്ന മറുചോദ്യമാണ്.വിചാരമെന്നാണ് ഉത്തരമെങ്കില്‍ പുരുഷനെയാണ് കുടുംബത്തിന്റെ നിയന്ത്രണമേല്‍പിക്കേണ്ടത്, വികാരമെന്നാണെങ്കില്‍ സ്ത്രീയെയും.

സ്ത്രീയുടെയും പുരുഷന്റെയും ശരീരപ്രകൃതിയും മാനസികാവസ്ഥയും അവരേറ്റെടുക്കേണ്ട ധര്‍മത്തിനനുസൃതമായ രീതിയിലാണ് സംവിധാനിക്കപ്പെട്ടിരിക്കുന്നത്. ശാരീരിക പ്രകൃതിയെന്നു പറയുമ്പോള്‍ കേവലം ബാഹ്യമായ വ്യത്യാസങ്ങള്‍ മാത്രമല്ല വിവക്ഷിക്കുന്നത്. അസ്ഥി വ്യവസ്ഥ മുതല്‍ പേശീവ്യവസ്ഥ വരെയുള്ള ആന്തരിക വ്യവസ്ഥകള്‍ പോലും ഓരോരുത്തര്‍ക്കും പ്രകൃത്യാ നിശ്ചയിക്കപ്പെട്ട ധര്‍മത്തിനനുസൃതമായ രീതിയിലാണുള്ളത്. പ്രസിദ്ധ ലൈംഗികശാസ്ത്രജ്ഞനായ ഹാവ്‌ലോക്ക് എല്ലിസിന്റെ ‘ആണ് തന്റെ കൈവിരല്‍ തുമ്പുവരെ പുരുഷനും സ്ത്രീ തന്റെ കാല്‍വിരല്‍ തുമ്പുവരെ പെണ്ണുമാണ്‘ എന്ന പ്രസിദ്ധമായ അഭിപ്രായം നൂറുശതമാനം ശരിയാണെന്നുള്ളതാണ് വസ്തുത.

പുരുഷന്റെ എല്ലുകള്‍ അധ്വാനത്തിനു പറ്റിയ രീതിയിലുള്ളവയാണെങ്കില്‍ സ്ത്രീയുടേത് ഗര്‍ഭധാരണത്തിന് അനുയോജ്യമായതാണ്. കഠിനാധ്വാനത്തിനാവശ്യമായ പേശികളാണ് പുരുഷനുള്ളതെങ്കില്‍ മാംസളതയും മിനുസവും നല്‍കുന്ന കൊഴുപ്പാണ് സ്ത്രീ ശരീരത്തിലുള്ളത്. അധ്വാനത്തിന് പറ്റിയ രീതിയിലുള്ള ആണിന്റെ കൈകള്‍! ആലിംഗനത്തിന് പറ്റുന്ന പെണ്ണിന്റെ കൈകള്‍…! ഇങ്ങനെ പോകുന്നു ശാരീരിക വ്യത്യാസങ്ങള്‍.

മാതൃത്വത്തിന് പറ്റിയ രീതിയില്‍ സ്ത്രീ ശരീരവും അധ്വാനത്തിന് സാധിക്കുന്ന രൂപത്തില്‍ പുരുഷശരീരവും സംവിധാനിക്കപ്പെട്ടപ്പോള്‍ അവരവരുടെ ധര്‍മത്തിന് അനുഗുണമായ മാനസിക ഗുണങ്ങളും അതിനോടനുബന്ധിച്ച് നല്‍കപ്പെട്ടിരിക്കുമല്ലോ. ദയയും വാല്‍സല്യവും ക്ഷിപ്രവൈകാരികതയുമാണ് സ്ത്രീ മനസ്സിന്റെ പ്രത്യേകതകള്‍. അത് വികാരപ്രധാനമാണ്. ശൈശവത്തിലും ബാല്യത്തിലും പെണ്‍കുട്ടികള്‍ കാണിക്കുന്ന ബൗദ്ധിക കഴിവുകള്‍ പോലും കൗമാരത്തോടെ മന്ദീഭവിക്കുന്നുവെന്നാണ് പഠനങ്ങള്‍ കാണിക്കുന്നത്. മാതൃത്വത്തിന് തയാറാകുമ്പോള്‍ മനസ്സും അതിനൊത്ത് മാറുന്നുവെന്നര്‍ഥം. സ്ത്രീയുടെ മനസ്സിനെക്കുറിച്ച് വ്യവഹരിക്കുമ്പോള്‍ നമ്മുടെ മനസ്സിലോടിയെത്തുന്നതെന്താണ്? അലിവാര്‍ന്ന ഹൃദയം, അതി ലോലമായ മനസ്സ്, പെട്ടെന്ന് പ്രതികരിക്കുന്ന പ്രകൃതം, നിരന്തരം നിര്‍ഗളിക്കുന്ന സ്‌നേഹവായ്പ്, നുരഞ്ഞുപൊങ്ങുന്ന വൈകാരികത…ഇതെല്ലാംതന്നെ സ്ത്രീമനസ്സ് വികാരപ്രധാനമാണെന്ന് വ്യക്തമാക്കുന്നു. എന്നാല്‍,പുരുഷമനസ്സിന്റെ അവസ്ഥയോ? ചിന്തിച്ചുള്ള പ്രതികരണം,പാരുഷ്യത്തോടെയുള്ള പെരുമാറ്റം, അവധാനതയോടുകൂടിയുള്ളപ്രത്യുത്തരം, ആലോചനയോടെയുള്ള പ്രവര്‍ത്തനം. ഇവയാണ് പുരുഷമനസ്സിന്റെ പ്രതിബിംബം. ഇവ വിചാരപ്രധാനമാണ്. അധ്വാനത്തിന് പറ്റിയ രീതിയില്‍ പുരുഷമനസ്സ് സംവിധാനിക്കപ്പെട്ടിരിക്കുന്നവെന്ന് സാരം.

(ഇത് പൊതുവായ വിലയിരുത്തലാണ്. ഭരിക്കാനും നീതിന്യായം നടത്താനും യുദ്ധം നയിക്കാനും ഭാരം ചുമക്കാനും കഠിനാധ്വാനം ചെയ്യാനും കഴിയുന്ന സ്ത്രീകളില്ലേ? പാചകത്തിനും വാല്‍സല്യത്തോടെ ശിശുക്കളെ പോറ്റുവാനും കുടുംബഭരണത്തിനും പറ്റിയ പുരുഷന്മാരില്ലേ? ‘ഉണ്ട്‘ എന്നുതന്നെയാണുത്തരം. ഇത് ചില അപവാദങ്ങള്‍ മാത്രമാണ്. അവര്‍ പലപ്പോഴും ലൈംഗികമായി മാത്രമേ തങ്ങളുടെ ലിംഗത്തിലുള്ളവരില്‍ ഉള്‍പ്പെടുകയുള്ളൂ. പെരുമാറ്റത്തിലും രീതിയിലും ധര്‍മനിര്‍വഹണത്തിലും എതിര്‍ ലിംഗത്തിലുള്ളവരോടായിരിക്കും അവര്‍ക്ക് അടുപ്പം)

കുടുംബത്തിന്റെ രക്ഷാധികാരത്തിന് പുരുഷനെ പ്രാപ്തനാക്കുന്നത് വിചാരത്തോടുകൂടി കാര്യങ്ങള്‍ കൈകാര്യം ചെയ്യാനുള്ള അവന്റെ കഴിവാണ്. അവന്റെ ശാരീരിക ഘടന അവനില്‍ അടിച്ചേല്‍പിച്ച ധര്‍മത്തിന്റെ നിര്‍വഹണമാണത്. അവന്‍ അധ്വാനിക്കണം, കുടുംബത്തെ പോറ്റുവാനുള്ള സമ്പത്തുണ്ടാക്കണം -അവനിലാണ് കുടുംബത്തിന്റെ കൈകാര്യകര്‍തൃത്വം ഏല്‍പിക്കപ്പെട്ടിരിക്കുന്നത്. ആ സ്ഥാപനത്തിന്റെയും അതിലുള്‍പ്പെട്ടവരുടെയും ജീവിതച്ചെലവിനുവേണ്ടിയുള്ള ആസൂത്രണവും ആ മാര്‍ഗത്തിലുള്ള സാമ്പത്തിക മേല്‍നോട്ടവും അവന്റെ ബാധ്യതയാക്കിത്തീര്‍ക്കുകയാണ് ഈ കൈകാര്യകര്‍തൃത്വം ചെയ്യുന്നത്. അതുകൊണ്ടാണ് ‘പുരുഷന്മാര്‍ സ്ത്രീകളുടെമേല്‍ നിയന്ത്രണാധികാരമുള്ളവരാകുന്നു‘വെന്ന് പറഞ്ഞതോടൊപ്പംതന്നെ അതിന്റെ കാരണമായി ‘മനുഷ്യരില്‍ ഒരു വിഭാഗത്തിന് മറുവിഭാഗത്തേക്കാള്‍ അല്ലാഹു കൂടുതല്‍ കഴിവ് നല്‍കിയതിനാലും അവരുടെ ധനം ചെലവഴിച്ചതിനാലുമാണിത്‘(4:34)എന്ന് ഖുര്‍ആന്‍ എടുത്തു പറഞ്ഞത്്. കുടുംബത്തിന്റെ നിയന്ത്രണാധികാരം നല്‍കുക വഴി പുരുഷനുമേല്‍ ഒരു വലിയ ഉത്തരവാദിത്തമേല്‍പിക്കുകയാണ് ഖുര്‍ആന്‍ ചെയ്യുന്നതെന്ന് പറയാന്‍ ഇതാണ് കാരണം.

സ്ത്രീയുടെ മേലും ഗൃഹഭരണത്തിന്‍മേലും പുരുഷന്‍ ഏകാധിപതിയായിരിക്കണമെന്നല്ല അവന്ന് നിയന്ത്രണാധികാരം നല്‍കിയതുകൊണ്ട് വിവക്ഷിക്കുന്നത്. പരസ്പര സഹകരണവും കൂടിയാലോചനയുമുണ്ടാവുമ്പോഴേ നായകത്വം ജീവസ്സുറ്റതാവൂ.‘സ്ത്രീകളുമായി നന്മയില്‍ വര്‍ത്തിക്കണം‘(4:19) എന്ന ഖുര്‍ആനിക നിര്‍ദേശവും, ‘നിങ്ങളുടെ വീട്ടുകാരോട് നന്നായി പെരുമാറുന്നവനാണ് നിങ്ങളില്‍ ഉത്തമന്‍‘ (ബുഖാരി) എന്ന പ്രവാചകന്റെ ഉപദേശവും നായകത്വമേല്‍പിക്കപ്പെട്ട പുരുഷന്‍ സ്വീകരിക്കുമ്പോഴാണ് സംതൃപ്തമായ കുടുംബജീവിതം സംജാതമാവുക.

‘പുരുഷന്മാര്‍ക്ക് സ്ത്രീകളേക്കാള്‍ ഒരു പദവിയുണ്ട്.(2:228) എന്നു ഖുര്‍ആന്‍ പറഞ്ഞതും ഈ അടിസ്ഥാനത്തിലാണ്. കുടുംബത്തിന്റെ സാമ്പത്തിക ഭാരമേറ്റെടുക്കുന്നതിലൂടെ കൈവരുന്ന പദവിയാണിത്. കുടുംബത്തിന്റെ രക്ഷാകര്‍തൃത്വമാണ് ആ പദവി.

ഉയര്‍ന്ന ശമ്പളമുള്ള ഒരു വനിതക്ക് കുടുംബത്തിന്റെ നായകത്വം നല്‍കിയെന്നുവെക്കുക. വൈവാഹിക ജീവിതത്തിന്റെ ആദ്യനാളുകളില്‍ കുടുംബസംരക്ഷണമെന്ന ഉത്തരവാദിത്വം തലയിലെത്തുന്നതിന് മുമ്പ് അത് അവള്‍ക്കൊരു പ്രയാസമായി അനുഭവപ്പെടുകയില്ലായിരിക്കാം. എന്നാല്‍,അവള്‍ ഗര്‍ഭിണിയും അമ്മയുമാവുമ്പോള്‍ നായകത്വത്തിന്റെ ഭാരം ചുമക്കാന്‍ അവള്‍ക്ക് കഴിയില്ല. പുരുഷനില്‍ കുടുംബനായകത്വമേല്‍പിക്കുന്നതിലൂടെ ഖുര്‍ആന്‍ സ്ത്രീക്ക് തണലേകുകയാണ് ചെയ്തിട്ടുള്ളതെന്ന് സാരം. സ്‌ത്രൈണതയെക്കുറിച്ചറിയുന്നവരൊന്നും ഇക്കാര്യത്തില്‍ ഖുര്‍ആനിന് എതിര് നില്‍ക്കുകയില്ല.

വിഷയവുമായി ബന്ധപ്പെട്ട വീഡിയോ

ഇസ്‌ലാമിന് ആണ്കോയ്മയോ പെണ്കോയ്മയോ പരിചയമില്ല. ആണിനോടും പെണ്ണിനോടും തങ്ങളുടെ ബാധ്യത കളെയും അവകാശങ്ങളെയുംകുറിച്ച് സംസാരിക്കുന്ന ആദർശമാണ് ഇസ്‌ലാം. ”സ്ത്രീകള്‍ക്ക് ബാധ്യതകള്‍ ഉള്ളതുപോലെതന്നെ ന്യായമായ അവകാശങ്ങളുമുണ്ട്”(2:228) എന്നാണ് ഖുര്‍ആന്‍ അസന്നിഗ്ധമായി പ്രഖ്യാപിക്കുന്നത്. പെണ്ണവകാശങ്ങളെക്കുറിച്ച് ഈ രൂപത്തിലുള്ള ഒരു പ്രസ്താവന പതിനെട്ടാം നൂറ്റാണ്ടിനു മുൻപ് എഴുതപ്പെട്ട ഏതെങ്കിലും ധര്മശാസ്ത്രഗ്രന്തങ്ങളിൽ കാണാനാവുമോ എന്ന ചോദ്യത്തിന്റെ ഉത്തരം ഇല്ലായെന്നാണ്. ന്യായമായും നൽകേണ്ട പെണ്ണവകാശങ്ങളെക്കുറിച്ച് ഇത്രയും വ്യക്തമായി പ്രഖ്യാപിക്കുന്ന ഖുര്‍ആന്‍ ആണ്‍കോയ്മാ വ്യവസ്ഥിതിയുടെ സൃഷ്ടിയാണെന്ന് പറയുന്നതെങ്ങനെ?സ്ത്രീയുടെ അവകാശങ്ങളെക്കുറിച്ച് ഖുര്‍ആനിനെപ്പോലെ വ്യക്തവും വിശദവുമായി പ്രതിപാദിക്കുന്ന മറ്റൊരു ധര്മശാസ്ത്രഗ്രന്ഥവുമില്ലെന്നതാണ് വാസ്തവം.

സ്ത്രീക്ക് ഇസ്‌ലാം അനുവദിച്ച-അല്ല, നേടിക്കൊടുത്ത-അവകാശങ്ങളുടെ മഹത്വമറിയണമെങ്കില്‍ അതിന്റെ അവതരണകാലത്തുണ്ടായിരുന്ന പെണ്ണിന്റെ പദവിയെന്തായിരുന്നുവെന്ന് മനസ്സിലാക്കണം. യവനന്മാര്‍ പി ശാചിന്റെ പ്രതിരൂപമായിട്ടായിരുന്നു പെണ്ണിനെ കണ്ടിരുന്നത്. പത്‌നിയെ അറുകൊല നടത്താന്‍ പോലും പുരുഷന് സ്വാതന്ത്ര്യം നല്‍കുന്നതായിരുന്നു റോമന്‍ നിയമവ്യവസ്ഥ. ഭര്‍ത്താവിന്റെ ചിതയില്‍ ചാടി മരിക്കണമെന്നതായിരുന്നുവല്ലോ ഭാരതീയ സ്ത്രീയോടുള്ള മതോപദേശം. പാപം കടന്നുവരാന്‍ കാരണക്കാരിയായ (?) പെണ്ണിനു നേരെയുള്ള യഹൂദന്മാരുടെ പെരുമാറ്റം ക്രൂരമായിരുന്നു. യഹൂദമതത്തിന്റെ പിന്തുടര്‍ച്ചയായി വന്ന ക്രിസ്തുമതത്തിലെ സ്ഥിതിയും മെച്ചപ്പെട്ടതായിരുന്നില്ല. സ്ത്രീകള്‍ക്ക് ആത്മാവുണ്ടോ എന്നതായിരുന്നു പതിനാറാം നൂറ്റാണ്ടില്‍ പോലും പാതിരിമാരുടെ ചര്‍ച്ചാവിഷയം. മുഹമ്മദ് നബി(ﷺ)ക്ക് മുമ്പ് അറേബ്യയിലെ പെണ്ണിന്റെ സ്ഥിതിയും ഇതിനേക്കാളെല്ലാം കഷ്ടമായിരുന്നു. അവള്‍ക്ക് ജീവിക്കാനുള്ള അവകാശം പോലും നിഷേധിക്കപ്പെട്ടു. പ്രസവിക്കപ്പെട്ടത് പെണ്‍കുഞ്ഞാണെന്നറിഞ്ഞാല്‍ ജീവനോടെ കുഴിച്ചുമൂടാന്‍ സന്നദ്ധരായിരുന്ന ജനങ്ങളുള്‍ക്കെളളുന്ന സമൂഹം. ഇത്തരമൊരു സാമൂഹിക സാഹചര്യത്തിലാണ് സ്ത്രീകളുടെ അവകാശങ്ങളെക്കുറിച്ച് ഖുര്‍ആന്‍ സംസാരിക്കുവാനാരംഭിച്ചത്.

പെണ്ണിന് ഖുര്‍ആന്‍ നല്‍കിയ അവകാശങ്ങളെ ഇങ്ങനെ സംഗ്രഹിക്കാം:

1.ജീവിക്കാനുള്ള അവകാശം: ഭാര്യ പ്രസവിച്ചത് പെണ്‍കുഞ്ഞാണെന്ന് മനസ്സിലാക്കിയാല്‍ അതിനെ കൊന്നുകളയുന്നതിനെക്കുറിച്ച് ചിന്തിച്ചിരുന്നവരായിരുന്നു അറബികള്‍ (ഖുര്‍ആന്‍16:59). ആധുനിക സാങ്കേതിക വിദ്യകളുപയോഗിച്ച് ഭ്രൂണത്തിന്റെ ലിംഗം നിര്‍ണയിക്കുകയും പ്രസവിക്കാന്‍ പോകുന്നത് പെണ്‍കുഞ്ഞാണെന്നറിഞ്ഞാല്‍ അതിനെ ഭ്രൂണാവസ്ഥയില്‍തന്നെ നശിപ്പിക്കാനൊരുമ്പെടുകയും ചെയ്യുന്ന സമകാലീന സമൂഹത്തിന്റെ ധാര്‍മിക നിലവാരം അജ്ഞാനകാലത്തെ അറബികളില്‍ നിന്ന് അല്‍പം പോലും ഉയര്‍ന്നതല്ല. പെണ്ണിനെ ജീവിക്കുവാന്‍ അനുവദിക്കാത്ത കുടിലതയെ ഖുര്‍ആന്‍ വിമര്‍ശിക്കുന്നു (16:59,81:9) പുരുഷനെപ്പോലെ അവള്‍ക്കും ജനിക്കുവാനും ജീവിക്കാനും അവകാശമുണ്ടെന്ന് അത് പ്രഖ്യാപിക്കുന്നു.

2.സ്വത്തവകാശം: പുരുഷനെപ്പോലെ സമ്പാദിക്കാനുള്ള അവകാശം ഖുര്‍ആന്‍ സ്ത്രീക്ക് നല്‍കുന്നു. സ്വന്തമായി ഉണ്ടാക്കിയതോ അനന്തരമായി ലഭിച്ചതോ ആയ സമ്പാദ്യങ്ങളെല്ലാം അവളുടേത് മാത്രമാണ് എന്നാണ് ഖുര്‍ആനിന്റെ കാഴ്ചപ്പാട്.സ്ത്രീക്ക് സ്വന്തം പേരിൽ സ്വത്ത് സമ്പാദിക്കുവാൻ 1870 നു ശേഷം മാത്രമാണ് യൂറോപ്പിൽ നിയമപരമായ അനുമതി ലഭിച്ചതെന്ന ചരിത്രം ഇതോട് ചേർത്ത് വായിക്കേണ്ടതാണ്. സ്ത്രീയുടെ സമ്പാദ്യത്തില്‍നിന്ന് അവളുടെ സമ്മതമില്ലാതെ യാതൊന്നും എടുക്കുവാന്‍ ഭര്‍ത്താവിന് പോലും അവകാശമില്ലെന്നാണ് ഇസ്‌ലാമിക നിയമം. .”പുരുഷന്മാര്‍ക്ക് അവര്‍ സമ്പാദിച്ചതിന്റെ വിഹിതവും സ്ത്രീകള്‍ക്ക് അവര്‍ സമ്പാദിച്ചതിന്റെ വിഹിതവുമുണ്ട് (ഖുര്‍ആന്‍ 4:32).

3.അനന്തരാവകാശം: മാതാപിതാക്കളുടെ സ്വത്തില്‍ പുത്രിമാര്‍ക്കും ഓഹരിയുണ്ടെന്നാണ് ഖുര്‍ആനിന്റെ അധ്യാപനം. മറ്റൊരു മതഗ്രന്ഥവും സ്ത്രീക്ക് അനന്തരസ്വത്തില്‍ അവകാശമുണ്ടെന്ന് പ്രഖ്യാപിക്കുന്നില്ലെന്നതാണ് വാസ്തവം. പരിഷ്‌കൃതമെന്നറിയപ്പെടുന്ന യൂറോപ്പില്‍ പോലും വനിതകള്‍ക്ക് അനന്തരസ്വത്തില്‍ അവകാശമുണ്ടെന്ന നിയമം കൊണ്ടുവന്നത് ഏതാനും പതിറ്റാണ്ടുകള്‍ക്കു മുമ്പാണ്. ഖുര്‍ആനാകട്ടെ,പതിനാലു നൂറ്റാണ്ടുകള്‍ക്കുമുമ്പുതന്നെ സ്ത്രീകള്‍ക്ക് അനന്തരസ്വത്തില്‍ അവകാശമുണ്ടെന്ന് പ്രഖ്യാപിക്കുകയും അത് നടപ്പാക്കുകയും ചെയ്തിട്ടുണ്ട്. ”മാതാപിതാക്കളും അടുത്ത ബന്ധുക്കളും വിട്ടേച്ചുപോയ സ്വത്തില്‍ പുരുഷന്മാര്‍ക്ക് ഓഹരിയുണ്ട്. മാതാപിതാക്കളും അടുത്ത ബന്ധുക്കളും വിട്ടേച്ചുപോയ സ്വത്തില്‍ സ്ത്രീകള്‍ക്കും ഓഹരിയുണ്ട്” (ഖുര്‍ആന്‍ 4:7).

4.ഇണയെ തീരുമാനിക്കുവാനുള്ള അവകാശം: വിവാഹാലോചനാവേളയില്‍ സ്ത്രീയുടെ ഇഷ്ടാനിഷ്ടങ്ങള്‍ പരിഗണിക്കപ്പെടണമെന്നാണ് ഇസ്‌ലാമിന്റെ ശാസന. ഒരു സ്ത്രീയെ അവള്‍ക്കിഷ്ടമില്ലാത്ത ഒരാള്‍ക്ക് വിവാഹം ചെയ്തുകൊടുക്കുവാന്‍ ആര്‍ക്കും അവകാശമില്ല; സ്വന്തം പിതാവിനുപോലും. മുഹമ്മദ് നബി(ﷺ) പറഞ്ഞു: ”വിധവയോട് അനുവാദം ചോദിക്കാതെ അവളെ വിവാഹം ചെയ്തുകൊടുക്കരുത്. കന്യകയോട് സമ്മതമാവശ്യപ്പെടാതെ അവളെയും കല്യാണം കഴിച്ചുകൊടുക്കാന്‍ പാടില്ല. മൗനമാണ് കന്യകയുടെ സമ്മതം” (ബുഖാരി, മുസ്‌ലിം).

5.പഠിക്കുവാനും ചിന്തിക്കുവാനുമുള്ള അവകാശം: സ്ത്രീകള്‍ക്ക് പഠിക്കുവാനും ചിന്തിക്കുവാനും അവകാശമുണ്ടെന്നാണ് ഇസ്‌ലാമിന്റെ കാഴ്ചപ്പാട്. ഇത് കേവലം ഉപദേശങ്ങളിലൊതുക്കുകയല്ല, പ്രായോഗികമായി കാണിച്ചുതരികയാണ് പ്രവാചകന്‍ (ﷺ)ചെയ്തത്. പ്രവാചകാനുചരകളായ വനിതകള്‍ വിജ്ഞാന സമ്പാദനത്തില്‍ പ്രകടിപ്പിച്ചിരുന്ന ശുഷ്‌കാന്തി സുവിദിതമാണ്. പ്രവാചകന്റെയും പത്‌നിമാരുടെയും അടുക്കല്‍ വിജ്ഞാന സമ്പാദനത്തിനായി വനിതകള്‍ സദാ എത്താറുണ്ടായിരുന്നുവെന്ന് ചരിത്രത്തില്‍ കാണാനാവും. അവരുമായി വിജ്ഞാന വിനിമയം നടത്താനായി പ്രവാചകന്‍ (ﷺ) ഒരു ദിവസം നീക്കിവെച്ചിരുന്നുവെന്ന് ഇമാം ബുഖാരി റിപ്പോര്‍ട്ട് ചെയ്ത ഹദീസില്‍ കാണാം.

6.വിമര്‍ശിക്കുവാനുള്ള അവകാശം: വിമര്‍ശിക്കുവാനും ചോദ്യം ചെയ്യുവാനുമുള്ള അവകാശം ഇസ്‌ലാം സ്ത്രീകള്‍ക്കു നല്‍കുന്നുണ്ട്. ജാഹിലിയ്യാ കാലത്ത് നിലനിന്നിരുന്ന‘ളിഹാർ‘ എന്ന സമ്പ്രദായത്തെക്കുറിച്ച് പ്രവാചകനോട് തര്‍ക്കിച്ച സ്വഹാബിവനിതയുടെ ചോദ്യങ്ങള്‍ക്കുള്ള ഉത്തരമായാണ് മുജാദിലഃ (തര്‍ക്കിക്കുന്നവള്‍) എന്ന അദ്ധ്യായത്തിലെ ആദ്യവചനങ്ങള്‍ അവതരിപ്പിക്കപ്പെട്ടത്. പ്രവാചകന്റെ മുമ്പില്‍ പോലും സ്ത്രീകള്‍ക്ക് അവകാശങ്ങള്‍ക്ക് വേണ്ടി സംവദിക്കാനുളള അവകാശം അനുവദിക്കപ്പെട്ടിരുന്നുവെന്നാണല്ലോ ഇത് വ്യക്തമാക്കുന്നത്. ഈ സൂക്തത്തിലെവിടെയും സ്വഹാബി വനിതയുടെ തര്‍ക്കത്തെ വിമര്‍ശിച്ചിട്ടില്ലെന്ന കാര്യം പ്രത്യേകം ശ്രദ്ധേയമാണ്.

7.സാമൂഹ്യപ്രവര്‍ത്തനങ്ങളില്‍ പങ്കെടുക്കുന്നതിനുള്ള അവകാശം: സാമൂഹ്യപ്രവർത്തനങ്ങളിൽ സംബന്ധിക്കുന്നത് സ്വാഭാവികമായും പുരുഷന്മാരാണെങ്കിലും സ്ത്രീകള്‍ക്കും അത്തരം കാര്യങ്ങളില്‍ പങ്കുവഹിക്കാന്‍ ഇസ്‌ലാം സ്വാതന്ത്ര്യം നല്‍കിയിട്ടുണ്ട്. വിശ്വാസ സ്വാതന്ത്ര്യത്തിനുവേണ്ടിയുള്ള സമരത്തില്‍ നേരിട്ട് സംബന്ധിക്കുന്നതിന് ഇസ്‌ലാം സ്ത്രീകളെ നിര്‍ബന്ധിക്കുന്നില്ല. എന്നാല്‍,യുദ്ധരംഗത്തും മറ്റും പട പൊരുതുന്നവര്‍ക്ക് സഹായികളായി വര്‍ത്തിക്കുവാന്‍ മുസ്‌ലിം വനിതകള്‍ രംഗത്തുണ്ടായിരുന്നു. പുരുഷന്മാരൊടൊപ്പം യുദ്ധത്തിന് പുറപ്പെടുകയും അവര്‍ക്ക് ഭക്ഷണം പാകം ചെയ്യുകയും പാനീയങ്ങള്‍ വിതരണം നടത്തുകയും മുറിവേറ്റവരെ ശുശ്രൂഷിക്കുകയും ചെയ്തിരുന്ന സ്വഹാബി വനിതകളെക്കുറിച്ച് ചരിത്രം നമുക്ക് വിവരിച്ചുതരുന്നുണ്ട്. സന്നിഗ്ധ ഘട്ടങ്ങളിൽ സമരമുഖത്തിറങ്ങാന്‍ വരെ സന്നദ്ധത കാണിച്ചിരുന്ന മഹിളാ രത്‌നങ്ങളുണ്ടായിട്ടുണ്ട്, ഇസ്‌ലാമിക ചരിത്രത്തില്‍. പ്രവാചക പത്‌നിയായിരുന്ന ആയിശ(റ)യായിരുന്നു ഖലീഫ ഉസ്മാന്റെ ഘാതകരെ ശിക്ഷിക്കാതെ അലി(റ)യെ ഖലീഫയായി തെരഞ്ഞെടുക്കരുതെന്ന അഭിപ്രായത്തില്‍നിന്ന് ഉരുണ്ടുകൂടിയ ജമല്‍ യുദ്ധത്തിന് നേതൃത്വം വഹിച്ചത്.

8.വിവാഹമൂല്യത്തിനുള്ള അവകാശം: വിവാഹം ചെയ്യപ്പെടുന്ന സ്ത്രീയുടെ അവകാശമാണ്‘മഹര്‍‘ ലഭിക്കുകയെന്നത്. തനിക്ക് ആവശ്യമുള്ള ‘മഹര്‍‘ തന്റെ കൈകാര്യകര്‍ത്താവ് മുഖേന ആവശ്യപ്പെടുവാന്‍ സ്ത്രീക്ക് അവകാശമുണ്ട്. ഈ വിവാഹമൂല്യം നല്‍കേണ്ടത് പുരുഷന്റെ ബാധ്യതയാണ്. നല്‍കപ്പെടുന്ന വിവാഹമൂല്യം സ്ത്രീയുടെ സമ്പത്തായാണ് പരിഗ ണിക്കപ്പെടുന്നത്. അവളുടെ സമ്മതമില്ലാതെ ആര്‍ക്കും അതില്‍നിന്നും ഒന്നും എടുക്കാനാവില്ല.”സ്ത്രീകള്‍ക്ക് അവരുടെ വിവാഹമൂല്യങ്ങള്‍ മനഃസംതൃപ്തിയോടുകൂടി നിങ്ങള്‍ നല്‍കുക” (4:4) എന്നാണ് ഖുര്‍ആനിന്റെ കല്‍പന.

9.വിവാഹമോചനത്തിനുള്ള അവകാശം: ഭര്‍ത്താവിനോടൊപ്പം ജീവിക്കുവാന്‍ സാധ്യമല്ലാത്ത അവസ്ഥയില്‍ വിവാഹമോചനം നേടാന്‍ സ്ത്രീക്ക് അവകാശമുണ്ട്.‘ഖുല്‍അ്‘, ‘ഫസ്ഖ്‘ എന്നീ രണ്ട് സാങ്കേതിക ശബ്ദങ്ങളിലാണ് സ്ത്രീകളുടെ വിവാഹമോചനം വ്യവഹരിക്കപ്പെടുന്നത്. വിവാഹ മൂല്യം തിരിച്ചുനല്‍കിക്കൊണ്ടുള്ള മോചനമാണ് ഒന്നാമത്തേത്. തിരിച്ചു നല്‍കാതെയുള്ളതാണ് രണ്ടാമത്തേത്. ഏതായിരുന്നാലും താനിഷ്ടപ്പെടാത്ത ഒരു ഭര്‍ത്താവിനോടൊപ്പം പൊറുക്കാന്‍ ഇസ്‌ലാം സ്ത്രീയെ നിര്‍ബന്ധിക്കുന്നില്ല. അവള്‍ക്ക് അനിവാര്യമായ സാഹചര്യത്തില്‍ വിവാഹമോചനം നേടാവുന്നതാണ്.ഇങ്ങനെ എല്ലാ നിലയ്ക്കുമുള്ള പെണ്ണവകാശങ്ങളെ പരിഗണിക്കുകയും അവയെന്തൊക്കെയാണെന്ന് കൃത്യമായി പഠിപ്പിക്കുകയും അവ നൽകേണ്ടത് ബാധ്യതയാണെന്ന് നിഷ്കര്ഷിക്കുകയും ചെയ്യുന്ന ദർശനമാണ് ഇസ്‌ലാം; സ്ത്രീകളുടെ ഒരു അവകാശത്തോടും അത് പുറം തിരിഞ്ഞു നിൽക്കുന്നില്ല.

ഇങ്ങനെ എല്ലാ നിലയ്ക്കുമുള്ള പെണ്ണവകാശങ്ങളെ പരിഗണിക്കുകയും അവയെന്തൊക്കെയാണെന്ന് കൃത്യമായി പഠിപ്പിക്കുകയും അവ നൽകേണ്ടത് ബാധ്യതയാണെന്ന് നിഷ്കര്ഷിക്കുകയും ചെയ്യുന്ന ദർശനമാണ് ഇസ്‌ലാം; സ്ത്രീകളുടെ ഒരു അവകാശത്തോടും അത് പുറം തിരിഞ്ഞു നിൽക്കുന്നില്ല.

വിഷയവുമായി ബന്ധപ്പെട്ട വീഡിയോ

 ഏതൊരു പ്രസ്ഥാനമായിരുന്നാലും അതിന്റെ അടിസ്ഥാന സങ്കല്‍പത്തിന്റെ പ്രതിഫലനമായിരിക്കും നിയമങ്ങളിലും നിര്‍ദേശങ്ങളിലും നമുക്ക് കാണാനാവുക. സ്ത്രീയെ സംബന്ധിച്ച ഇസ്‌ലാമിക നിര്‍ദേശങ്ങളുടെ വേര് സ്ഥിതി ചെയ്യുന്നത് അവള്‍ ആരാണെന്ന പ്രശ്‌നത്തിന് ഖുര്‍ആന്‍ നല്‍കുന്ന ഉത്തരത്തിലാണ്.

പുരുഷനെപ്പോലെതന്നെ സ്ത്രീയും പടച്ചതമ്പുരാന്റെ സവിശേഷ സൃഷ്ടിയാണെന്നാണ് ഖുര്‍ആനികാധ്യാപനം.

”മനുഷ്യരേ, നിങ്ങളെ ഒരേ ആത്മാവില്‍നിന്ന് സൃഷ്ടിക്കുകയും അതില്‍ നിന്നുതന്നെ അതിന്റെ ഇണയെയും സൃഷ്ടിക്കുകയും അവര്‍ ഇരുവരില്‍നിന്നുമായി ധാരാളം പുരുഷന്മാരെയും സ്ത്രീകളെയും വ്യാപിപ്പിക്കുകയും ചെയ്തവനായ നിങ്ങളുടെ രക്ഷിതാവിനെ നിങ്ങള്‍ സൂക്ഷിക്കുവിന്‍” (4:1)

ഇവിടെ പുരുഷനും സ്ത്രീയും ഒരേ ആത്മാവില്‍നിന്നാണ് സൃഷ്ടിക്കപ്പെട്ടിട്ടുള്ളതെന്ന വസ്തുതയാണ് ഖുര്‍ആന്‍ സൂചിപ്പിക്കുന്നത്;പുരുഷനും സ്ത്രീയും ഒരേ ആത്മാവിന്റെ രണ്ട് അംശങ്ങളാണെന്ന വസ്തുത. ഈ രണ്ട് അംശങ്ങളും കൂടിച്ചേരുമ്പോഴാണ് അതിന് പൂര്‍ണത കൈവരുന്നത്. അഥവാ സ്ത്രീയുടെയും പുരുഷന്റെയും പാരസ്പര്യത്തിലാണ് ജീവിതം പൂര്‍ണമാവുന്നത്. സ്ത്രീ-പുരുഷബന്ധത്തിലെ സ്‌നേഹത്തിന്റെയും കാരുണ്യത്തിന്റെയുമെല്ലാം ഉറവിടം ഈ പാരസ്പര്യമാണ്. ദമ്പതികള്‍ തമ്മില്‍ നിലനില്‍ക്കുന്ന കരുണയും സ്‌നേഹവുമെല്ലാം ദൈവിക ദൃഷ്ടാന്തങ്ങളാണെന്നാണ് ഖുര്‍ആനിക കാഴ്ചപ്പാട്. ‘നിങ്ങള്‍ക്ക് സമാധാനപൂര്‍വം ഒത്തുചേരേണ്ടതിനായി നിങ്ങളില്‍നിന്നുതന്നെ നിങ്ങള്‍ക്ക് ഇണകളെ സൃഷ്ടിക്കുകയും, നിങ്ങള്‍ക്കിടയില്‍ സ്‌നേഹവും കാരുണ്യവും ഉണ്ടാക്കുകയും ചെയ്തതും അവന്റെ ദൃഷ്ടാന്തങ്ങളില്‍ പെട്ടതത്രേ. തീര്‍ച്ചയായും അതില്‍ ചിന്തിക്കുന്ന ജനങ്ങള്‍ക്ക് ദൃഷ്ടാന്തങ്ങളുണ്ട്”(30:21).

ലിംഗത്തിന്റെ അടിസ്ഥാനത്തിലുള്ള വിവേചനത്തെ ഖുര്‍ആന്‍ അംഗീകരിക്കുന്നില്ല. സ്ത്രീ-പുരുഷ സമത്വമെന്ന ആശയത്തെ അത് നിരാകരിക്കുകയും ചെയ്യുന്നു. സ്ത്രീ, പുരുഷന് സമമോ പുരുഷന്‍,സ്ത്രീക്ക് സമമോ ആവുക അസാധ്യമാണെന്നാണ് അതിന്റെ വീക്ഷണം. അങ്ങനെ ആക്കുവാന്‍ ശ്രമിക്കുന്നത് പ്രകൃതി വിരുദ്ധമാണ്. സ്ത്രീയെയും പുരുഷനെയും പ്രകൃതി അവര്‍ക്കനുവദിച്ച സ്ഥാനങ്ങളില്‍തന്നെ നിര്‍ത്തുകയാണ് ഖുര്‍ആ ന്‍ ചെയ്യുന്നത്. പ്രകൃതി സ്ത്രീക്കും പുരുഷനും നല്‍കിയ സ്ഥാനങ്ങള്‍ തന്നെയാണ് പ്രകൃതിമതമായ ഇസ്‌ലാമും അവര്‍ക്ക് നല്‍കുന്നത്.

ഖുര്‍ആനിനുമുമ്പ് അവതരിപ്പിക്കപ്പെട്ട വേദഗ്രന്ഥങ്ങളെയെല്ലാം അത് അംഗീകരിക്കുന്നു. ആകെ എത്ര വേദഗ്രന്ഥങ്ങള്‍ അവതരിപ്പിക്കപ്പെട്ടുവെന്ന് ഖണ്ഡിതമായി ഖുര്‍ആന്‍ പ്രസ്താവിക്കുന്നില്ല. നാല് വേദഗ്രന്ഥങ്ങളുടെ പേര് മാത്രമാണ് ഖുര്‍ആനില്‍ പരാമര്‍ശിക്കപ്പെട്ടിരിക്കുന്നത്. മൂസാ നബി(അ)ക്ക് അവതരിപ്പിക്കപ്പെട്ട തൗറാത്തും ദാവൂദ് നബി(അ)ക്ക് അവതരിപ്പിക്കപ്പെട്ട സബൂറും ഈസാനബി(അ)ക്ക് അവതരിപ്പിക്കപ്പെട്ട ഇന്‍ജീലും മുഹമ്മദ്( ﷺ)ക്ക് അവതരിപ്പിക്കപ്പെട്ട ഖുര്‍ആനുമാണവ. ഈ നാലു വേദഗ്രന്ഥങ്ങള്‍ക്കു പുറമെയും എഴുതപ്പെട്ട രേഖകള്‍ പടച്ചതമ്പുരാനില്‍ നിന്ന് അവതരിപ്പിക്കപ്പെട്ടിട്ടുണ്ടെന്നാണ് ഖുര്‍ആന്‍ നല്‍കുന്ന സൂചന.

‘നിങ്ങള്‍ പറയുക: അല്ലാഹുവിലും അവങ്കല്‍നിന്ന് ഞങ്ങള്‍ക്ക് അവതരിപ്പിച്ചു കിട്ടിയതിലും ഇബ്‌റാഹീമിനും ഇസ്മായിലിനും ഇസ്ഹാഖിനും യഅ്ഖൂബിനും യഅ്ഖൂബ് സന്തതികള്‍ക്കും അവതരിപ്പിച്ചുകൊടുത്തതിലും മൂസാ, ഈസാ എന്നിവര്‍ക്ക് നല്‍കപ്പെട്ടതിലും സര്‍വപ്രവാചകന്മാര്‍ക്കും അവരുടെ രക്ഷിതാവിങ്കല്‍നിന്ന് നല്‍കപ്പെട്ടതിലും ഞങ്ങള്‍ വിശ്വസിച്ചിരിക്കുന്നു‘ (2:136)

‘തീര്‍ച്ചയായും ഇത് ആദ്യത്തെ ഏടുകളില്‍തന്നെയുണ്ട്, അഥവാ ഇബ്റാഹീമിന്റെയും മൂസായുടെയും ഏടുകളില്‍ (87:18,19).

മുമ്പുള്ള വേദങ്ങളെ മുഴുവന്‍ ഖുര്‍ആന്‍ സത്യപ്പെടുത്തുന്നു: ‘അവന്‍ ഈ വേദഗ്രന്ഥത്തെ മുന്‍വേദങ്ങളെ ശരിവെക്കുന്നതായിക്കൊണ്ട് സത്യവുമായി നിനക്ക് അവതരിപ്പിച്ചുതന്നിരിക്കുന്നു. മനുഷ്യര്‍ക്ക് മാര്‍ഗദര്‍ശനത്തിനായി ഇതിനുമുമ്പ് അവന്‍ തൗറാത്തും ഇന്‍ജീലും അവതരിപ്പിച്ചു. ഫുര്‍ഖാനും അവന്‍ അവതരിപ്പിച്ചിരിക്കുന്നു‘ (3:3).

അല്ലാഹുവില്‍നിന്ന് അവതരിപ്പിക്കപ്പെട്ട വേദഗ്രന്ഥങ്ങളിലെല്ലാം വിശ്വസിക്കേണ്ടത് മുസ്‌ലിമിന്റെ നിര്‍ബന്ധബാധ്യതയാണ്. മുമ്പ് അവതരിപ്പിക്കപ്പെട്ട വേദഗ്രന്ഥങ്ങളിലേതെങ്കിലും ദൈവികമല്ലെന്ന് വിശ്വസിക്കുന്നത് വലിയൊരു അപരാധമായിട്ടാണ് ഖുര്‍ആന്‍ കാണുന്നത്.

‘സത്യവിശ്വാസികളേ, അല്ലാഹുവിലും അവന്റെ ദൂതനിലും അവന്റെ ദൂതന് അവന്‍ അവതരിപ്പിച്ച ഗ്രന്ഥത്തിലും അവന്‍ മുമ്പ് അവതരിപ്പിച്ച ഗ്രന്ഥത്തിലും നിങ്ങള്‍ വിശ്വസിക്കുവിന്‍. അല്ലാഹുവിലും അവന്റെ മലക്കുകളിലും അവന്റെ ഗ്രന്ഥങ്ങളിലും അവന്റെ ദൂതന്മാരിലും അന്ത്യദിനത്തിലും വല്ലവനും അവിശ്വസിക്കുന്ന പക്ഷം തീര്‍ച്ചയായും അവന്‍ ബഹുദൂരം പിഴച്ചുപോയിരിക്കുന്നു‘ (4:136).

എല്ലാ ജനസമൂഹങ്ങളിലേക്കും പ്രവാചകന്മാര്‍ നിയോഗിക്കപ്പെട്ടിട്ടുണ്ട്. ‘ഒരു താക്കീതുകാരന്‍ കഴിഞ്ഞുപോകാത്ത ഒരു സമുദായവുമില്ല‘ (35:24) എന്നാണ് ഖുര്‍ആന്‍ അര്‍ഥശങ്കക്കിടയില്ലാത്തവിധം വ്യക്തമാക്കുന്നത്. അപ്പോള്‍ ചിരപുരാതനമായ ഒരു സംസ്‌കാരം നിലനിന്നിരുന്ന പ്രദേശമെന്ന നിലയ്ക്ക് ഇന്ത്യയിലും പ്രവാചകന്മാര്‍ വന്നിട്ടുണ്ടാവണം. ആ പ്രവാചകന്മാരില്‍ ചിലര്‍ക്ക് വേദഗ്രന്ഥങ്ങളും നല്‍കപ്പെട്ടിരിക്കണം. ഈ പ്രവാചകന്മാരെയോ വേദഗ്രന്ഥങ്ങളെയോ ഇകഴ്ത്തുകയോ അവമതിക്കുകയോ ചെയ്യാ ന്‍ മുസ്‌ലിമിന് പാടില്ല. പ്രവാചകന്മാര്‍ക്കിടയില്‍ വിവേചനം കല്‍പിക്കുന്നതിനെതിരെ ഖുര്‍ആന്‍ ശക്തമായി താക്കീത് നല്‍കുന്നുണ്ട് (4:150). അപ്പോള്‍ ഇന്ത്യയിലേക്കു പ്രവാചകന്മാര്‍ നിയോഗിക്കപ്പെട്ടിട്ടുണ്ടെങ്കില്‍ അവരെയും അവര്‍ക്ക് അവതരിപ്പിക്കപ്പെട്ട വേദഗ്രന്ഥങ്ങളെയും ഖുര്‍ആന്‍ ആദരി ക്കുന്നു. അംഗീകരിക്കുന്നു. എന്നാല്‍, ഇന്നുനിലനില്‍ക്കുന്ന ശ്രുതി ഗ്രന്ഥങ്ങളിലേതെങ്കിലും (വേദ സംഹിതകള്‍, ബ്രാഹ്മണങ്ങള്‍, ആരണ്യകങ്ങള്‍, ഉപനിഷത്തുകള്‍) പടച്ച തമ്പുരാന്‍ പ്രവാചകന്മാര്‍ക്ക് അവതരിപ്പിച്ച ഗ്രന്ഥങ്ങളാണെന്ന് പറയാന്‍ കഴിയുമോ? ഇവ ദൈവത്തിങ്കല്‍നിന്ന് ശ്രവിക്കപ്പെട്ടതിനാലാണ് ശ്രുതിയെന്നു വിളിക്കുന്നതെന്നാണ് വിശ്വാസം. ദൈവത്തിങ്കല്‍നിന്ന് മനുഷ്യര്‍ക്ക് പ്രത്യേകമായ സന്ദേശങ്ങള്‍ അവതരിപ്പിക്കപ്പെടുന്നുവെന്ന വിശ്വാസം ഹിന്ദുക്കള്‍ക്കിടയില്‍ നിലനിന്നിരുന്നുവെന്ന് ശ്രുതി സങ്കല്‍പം വ്യക്തമാക്കുന്നു. നടേ പറഞ്ഞ ഗ്രന്ഥങ്ങളെല്ലാം ശ്രുതികളായി വ്യവഹരിക്കപ്പെടുന്നുണ്ടെങ്കിലും അവയില്‍ ഏതെല്ലാം പ്രാമാണികമാണെന്ന കാര്യത്തില്‍ അഭിപ്രായാന്തരങ്ങളുണ്ട്. ആര്യസമാജ സ്ഥാപകനായ സ്വാമി ദയാനന്ദസരസ്വതി നാല് വേദസംഹിതകള്‍ക്കു മാത്രമാണ് അപ്രമാദിത്വമുള്ളതെന്ന് വാദിക്കുമ്പോള്‍ സ്വാമി വിവേകാനന്ദനെ പോലുള്ളവര്‍ ഉപനിഷത്തുകള്‍ക്കാണ് പ്രഥമ പ്രാധാന്യം നല്‍കുന്നത്. അടിസ്ഥാന ശ്രുതിഗ്രന്ഥങ്ങള്‍ക്കുപോലും തെറ്റുകള്‍ പറ്റാമെന്ന് അഭിപ്രായപ്പെട്ട ഹിന്ദുമത പണ്ഡിതന്മാരുണ്ട്.‘വേദങ്ങള്‍ തെറ്റു പറ്റാത്തവയോ സര്‍വതും ഉള്‍ക്കൊള്ളുന്നവയോ അല്ല‘ (Indian Religions page 22) എന്ന ഡോ. രാധാകൃഷ്ണന്റെ വീക്ഷണവും ‘വേദങ്ങളില്‍ യുക്തിയുമായി പൊരുത്തപ്പെടുന്നിടത്തോളം ഭാഗങ്ങള്‍ ഞാന്‍ സ്വീകരിക്കുന്നു. വേദങ്ങളിലെ ചില ഭാഗങ്ങള്‍ പ്രഥമ ദൃഷ്ടിയില്‍ പരസ്പരവിരുദ്ധങ്ങളാണ്‘. (വിവേകാനന്ദ സാഹിത്യ സര്‍വസ്വം വാല്യം-4, പുറം-55) എന്ന സ്വാമി വിവേകാനന്ദന്റെ നിലപാടും വേദങ്ങള്‍ നൂറുശതമാനം ദൈവിക വചനങ്ങളാണുള്‍ക്കൊള്ളുന്നതെന്ന വാദഗതിയുടെ മുനയൊടിക്കാന്‍ പോന്നതാണ്. പൊതുവെ പറഞ്ഞാല്‍, ഇന്ത്യയില്‍ നിലനിന്നിരുന്ന വിശ്വാസങ്ങളെയും ആചാരങ്ങളെയും കുറിച്ച വ്യക്തമായ ചിത്രം നല്‍കുന്ന ഗ്രന്ഥങ്ങളാണ് ശ്രുതികള്‍. ഇന്ത്യയിലേക്ക് നിയുക്തരായ പ്രവാചകന്മാര്‍ പ്രബോധനം ചെയ്ത ആശയങ്ങളുടെ ശകലങ്ങള്‍ ഇവയില്‍ കാണാനാവുമായിരിക്കണം. എന്നാല്‍, ഇവ പൂര്‍ണമായും ദൈവികമാണെന്ന വാദം ഇസ്‌ലാം അംഗീകരിക്കുന്നില്ല. വേദപണ്ഡിതന്മാർക്കും അങ്ങനെയൊരു വാദമില്ലല്ലോ.
മൂസാനബി(عليهالسلام)യുടെ കാലത്ത് ക്രൂശീകരണം ഒരു ശിക്ഷയായി നിലനിന്നിരുന്നുവെന്ന് സൂചിപ്പിക്കുന്ന ഒന്നിലധികം ഖുര്‍ആന്‍ സൂക്തങ്ങളുണ്ട്. ചില സൂക്തങ്ങളുടെ സാരം കാണുക: ”നിങ്ങളുടെ കൈകളും കാലുകളും എതിര്‍വശങ്ങളില്‍നിന്നായി ഞാന്‍ മുറിച്ചുകളയുക തന്നെ ചെയ്യും. പിന്നെ നിങ്ങളെ മുഴുവന്‍ ഞാന്‍ ക്രൂശിക്കുകയും ചെയ്യും, തീര്‍ച്ച”(വി.ഖു.7:124). ”അവന്‍(ഫിര്‍ഔന്‍) പറഞ്ഞു: ഞാന്‍ നിങ്ങള്‍ക്ക് അനുവാദം തരുന്നതിന് മുമ്പായി നിങ്ങള്‍ അവനില്‍ വിശ്വസിച്ചുവെന്നോ. തീര്‍ച്ചയായും ഇവന്‍ നിങ്ങള്‍ക്ക് ജാലവിദ്യ പഠിപ്പിച്ച നിങ്ങളുടെ തലവന്‍ തന്നെയാണ്. വഴിയെ നിങ്ങള്‍ അറിഞ്ഞുകൊള്ളും. തീര്‍ച്ചയായും നിങ്ങളുടെ കൈകളും നിങ്ങളുടെ കാലുകളും എതിര്‍വശങ്ങളില്‍നിന്നായിക്കൊണ്ട് ഞാന്‍ മുറിച്ചുകളയുകയും നിങ്ങളെ മുഴുവന്‍ ഞാന്‍ ക്രൂശിക്കുകയും ചെയ്യുന്നതാണ്” (വി.ഖു. 26:49). ”അവന്‍(ഫിര്‍ഔന്‍) പറഞ്ഞു: ഞാന്‍ നിങ്ങള്‍ക്ക് സമ്മതം തരുന്നതിന് മുമ്പ് നിങ്ങള്‍ അവനെ വിശ്വസിച്ചുകഴിഞ്ഞുവെന്നോ? തീര്‍ച്ചയായും നിങ്ങള്‍ക്ക് ജാലവിദ്യ പഠിപ്പിച്ചുതന്ന നിങ്ങളുടെ നേതാവ് തന്നെയാണ് അവന്‍. ആകയാല്‍ തീര്‍ച്ചയായും ഞാന്‍ നിങ്ങളുടെ കൈകളും കാലുകളും എതിര്‍വശങ്ങളില്‍നിന്നായി മുറിച്ചുകളയുകയും ഈന്തപ്പന തടികളില്‍ നിങ്ങളെ ഞാന്‍ ക്രൂശിക്കുകയും ചെയ്യുന്നതാണ്. ഞങ്ങളില്‍ ആരാണ് ഏറ്റവും കഠിനമായതും നീണ്ടുനില്‍ക്കുന്നതുമായ ശിക്ഷ നല്‍കുന്നവന്‍ എന്ന് തീര്‍ച്ചയായും നിങ്ങള്‍ക്ക് മനസ്സിലാകുകയും ചെയ്യും” (വി.ഖു. 20:71). മൂസാനബി(عليهالسلام)യുടെ മുമ്പ് യൂസുഫ് നബി (عليهالسلام)യുടെകാലത്തുതന്നെ ക്രൂശീകരണം നിലനിന്നിരുന്നതായി സൂചിപ്പിക്കുന്ന ഖുര്‍ആന്‍ വചനങ്ങളുമുണ്ട്. യൂസുഫ് നബി (عليهالسلام)യോടൊപ്പം ജയിലിലടക്കപ്പെട്ടയാളുടെ സ്വപ്‌നത്തിന് അദ്ദേഹം നല്‍കിയ വ്യാഖ്യാനത്തെക്കുറിച്ച് പരാമര്‍ശിക്കവെ ഖുര്‍ആന്‍ ഇങ്ങനെ പറയുന്നത്കാണാം: ”ജയിലിലെ രണ്ട് സുഹൃത്തുക്കളേ എന്നാല്‍ നിങ്ങളിലൊരുവന്‍ തന്റെ യജമാനന് വീഞ്ഞ് കുടിപ്പിച്ചുകൊണ്ടിരിക്കും. എന്നാല്‍ മറ്റേയാള്‍ ക്രൂശിക്കപ്പെടും. എന്നിട്ട് അയാളുടെ തലയില്‍നിന്ന് പറവകള്‍ കൊത്തിത്തിന്നും. ഏതൊരു കാര്യത്തെപ്പറ്റി നിങ്ങളിരുവരും വിധി ആരായുന്നുവോ ആ കാര്യം തീരുമാനിക്കപ്പെട്ടുകഴിഞ്ഞിരിക്കുന്നു” (വി.ഖു. 12:41) മൂസാനബി (عليهالسلام)ക്ക് മുമ്പുതന്നെ ഈജിപ്തില്‍ നിലനിന്നിരുന്ന ഒരു ശിക്ഷാമുറയായാണ് ഖുര്‍ആന്‍ ക്രൂശീകരണത്തെ പരിചയപ്പെടുത്തുന്നത് എന്ന് ഈ സൂക്തങ്ങളില്‍നിന്ന് സുതരാം വ്യക്തമാണ്. പൗരാണിക ഈജിപ്തിനെക്കുറിച്ച പഠനഗ്രന്ഥങ്ങളിലൊന്നുംതന്നെ ഫറോവമാരുടെകാലത്ത് ക്രൂശീകരണം നിലനിന്നതായി വ്യക്തമാക്കുന്ന രേഖകള്‍ അവതരിപ്പിക്കുന്നില്ലെന്നത് ശരിയാണ്. എന്നാല്‍ അതുകൊണ്ടുമാത്രം അന്ന് അങ്ങനെയൊരു ശിക്ഷാസമ്പ്രദായം തന്നെ നിലനിന്നിരുന്നില്ലായെന്ന് പറയാനാവില്ല. പൗരാണിക ഈജിപ്തിനെക്കുറിച്ച പഠനത്തിന് അവിടെനിന്ന് ഉല്‍ഖനനം ചെയ്‌തെടുത്ത ശിലാരേഖകളെയും സീലുകളെയും പ്രതിമകളെയുമെല്ലാമാണ് പ്രധാനമായും ഉപയോഗിക്കുന്നത്. ഇവയിൽ ‍നിന്ന് മാത്രമായി ഈജിപ്തിന്റെ ഒരു സമ്പൂര്‍ണ്ണ ചരിത്രം നിര്‍മിക്കാനാവില്ല. ഇവയില്‍ രേഖപ്പെടുത്തപ്പെടാത്ത നിരവധി കാര്യങ്ങളുണ്ടായിരിക്കാം. രേഖപ്പെടുത്തപ്പെട്ട കാര്യങ്ങളില്‍ തന്നെ വായിക്കാന്‍ കഴിയാത്തവയുണ്ടായിരിക്കാം; വായിക്കാന്‍ കഴിഞ്ഞവയില്‍ തന്നെ സ്ഖലിതങ്ങളുണ്ടാകുവാനുള്ള സാധ്യതയും പൂര്‍ണമായി തള്ളിക്കളയാനാവില്ല. അതുകൊണ്ടുതന്നെ പൗരാണിക ഈജിപ്തിനെക്കുറിച്ച പഠനങ്ങള്‍ അവിടെ ക്രൂശീകരണം നിലനിന്നിട്ടില്ലായെന്ന് വ്യക്തമാക്കുന്ന രേഖകള്‍ വെളിപ്പെടുത്താത്തിടത്തോളം അവിടെ ക്രൂശീകരണമുണ്ടായിരുന്നുവെന്ന് വ്യക്തമാക്കുന്ന ഖുര്‍ആനിക പരാമര്‍ശങ്ങള്‍ അബദ്ധമാണെന്ന് പറയാനാകില്ല. ഖുര്‍ആനിക പരാമര്‍ശങ്ങള്‍ക്ക് ഉപോല്‍ബലകമായ രേഖകള്‍ ലഭിച്ചിട്ടില്ലാത്തതുപോലെതന്നെ അതിനെ നിഷേധിക്കുന്ന രേഖകളുമില്ലാത്ത സ്ഥിതിക്ക് ആ പരാമര്‍ശങ്ങളില്‍ അബദ്ധമാരോപിക്കുന്നതിന് യാതൊരു അടിസ്ഥാനവുമില്ല. എന്താണ് ക്രൂശീകരണം? ‘ഓക്‌സ്‌ഫോര്‍ഡ് കംപാനിയന്‍ ടു ദ ബൈബിള്‍’പറയുന്നത് കാണുക: ”വധശിക്ഷയായോ മൃതശരീരത്തെ പ്രദര്‍ശിപ്പിക്കുന്നതിന്നായോ കുരിശിലോ മരത്തിലോ ഒരാളെ ആണിയടിക്കുകയോ ബന്ധിക്കുകയോ ചെയ്യുക”. (Bruce M Metzger Michael D Coogan (Ed), Oxford Companion to the Bible, 1993, Oxford University Press, Oxford & Newyork page 141) ഇന്ന് ക്രൈസ്തവര്‍ മതചിഹ്‌നമായി ഉപയോഗിക്കുന്ന രീതിയിലുള്ളതായിരുന്നില്ല ആദ്യകാലത്തെ കുരിശ്. ഒരു മരത്തടിയില്‍ കൈകളും കാലുകളും അരക്കെട്ടുമെല്ലാം ആണിയില്‍ തറച്ച് ഇഞ്ചിഞ്ചായികൊല്ലുന്ന സമ്പ്രദായമാണ് ക്രൂശീകരണത്തിന്റെ ആദിമരൂപം. പിന്നീട്എക്‌സ് (X) ആകൃതിയില്‍ രണ്ട് തടികള് ‍വെച്ച് അതില്‍ ക്രൂശീകരിക്കുന്നരീതിയുണ്ടായി. അതിനും ശേഷമാണ് ഒരു തടിമരത്തിന്റെ മുകള്‍ഭാഗത്ത് മറ്റൊരു മരക്കഷണം കൂട്ടിവെച്ച് ടി (T) ആകൃതിയിലും ഇന്ന് ക്രൈസ്തവര്‍ മതചിഹ്‌നമായി ഉപയോഗിക്കുന്ന + ആകൃതിയിലുമെല്ലാമുള്ളകുരിശുകളുണ്ടായത്. അതുകൊണ്ടാണ് ‘മരത്തിലോ കുരിശിലോ ആണിയടിച്ചുകൊല്ലുന്നതാണ് ക്രൂശീകരണം‘ എന്ന് ഓക്‌സ്‌ഫോര്‍ഡ് കംപാനിയന്‍ ടു ദി ബൈബിളില്‍ പറഞ്ഞത്. ആണിയടിച്ച് കൊല്ലുകയെന്നകര്‍മ്മമാണ്, അതിനുപയോഗിക്കുന്ന വസ്തുവിന്റെ ആകൃതിയല്ല ക്രൂശീകരണത്തെ അന്വര്‍ത്ഥമാക്കുന്നത് എന്ന് സാരം. മരത്തില്‍ തറച്ചുകൊല്ലുന്ന ഏര്‍പ്പാട് മോശയുടെ കാലത്തും യോസഫിന്റെ കാലത്തുമെല്ലാം നിലനിന്നിരുന്നുവെന്നതിന് ബൈബിള്‍തന്നെ തെളിവുകള്‍ നല്‍കുന്നുണ്ട്. യോസഫിന്റെ സ്വപ്‌നവ്യാഖ്യാനത്തെക്കുറിച്ച് ഉല്‍പത്തി പുസ്തകം പറയുന്നത് ഇങ്ങനെയാണ്: ”യോസഫ് പറഞ്ഞു: ഇതാണ് സ്വപ്‌നത്തിന്റെ വ്യാഖ്യാനം. മൂന്ന് കുട്ട മൂന്ന് ദിവസമാണ്. മൂന്ന്ദിവസത്തിനകം ഫറോവാന്‍ നിന്റെ തലവെട്ടി നിന്നെ കഴുമരത്തില്‍ കെട്ടിത്തൂക്കും. പക്ഷികള്‍ നിന്റെ മാംസം ഭക്ഷിക്കും” (ഉല്‍പത്തി 40:18-19) ഇക്കാര്യം പരാമര്‍ശിക്കുമ്പോള്‍ ഖുര്‍ആന്‍ ‘ക്രൂശിക്കുക‘യെന്നാണ്പറഞ്ഞിട്ടുള്ളതെന്ന കാര്യം പ്രത്യേകം ശ്രദ്ധേയമാണ്. മോശ എഴുതിയതായി കരുതപ്പെടുന്ന ആവര്‍ത്തന പുസ്തകത്തിലുംമരത്തില്‍ തൂക്കിക്കൊല്ലുന്നതിനെക്കുറിച്ച പരാമര്‍ശങ്ങളുണ്ട്: ”വധശിക്ഷഅര്‍ഹിക്കുന്ന കുറ്റം ചെയ്തവനെ വധിച്ച് മരത്തില്‍ തൂക്കിക്കഴിഞ്ഞാല്‍അയാളുടെ ജഡം രാത്രി മുഴുവന്‍ ആ മരത്തില്‍ കിടക്കരുത്. ആ ദിവസംതന്നെഅയാളെ സംസ്‌ക്കരിക്കണം. തൂക്കിക്കൊല്ലപ്പെടുന്നവന്‍ ദൈവത്താല്‍ശപിക്കപ്പെട്ടവനാണ്. നിന്റെ ദൈവമായ കര്‍ത്താവ് നിനക്ക് അവകാശമായിതരുന്ന ദേശം നീ മലിനമാക്കരുത്” (ആവ 21:22-23) മോശക്ക് ശേഷം വന്ന യോശുവയുടെ കാലത്തും ഈ ശിക്ഷാ രീതിനിലനിന്നിരുന്നുവെന്ന് ബൈബിള്‍ വ്യക്തമാക്കുന്നുണ്ട്: ”ആയിയിലെരാജാവിനെ വൈകുന്നേരംവരെ ഒരു മരത്തില്‍ കെട്ടിത്തൂക്കിയിട്ടു.സൂര്യാസ്തമയം ആയപ്പോള്‍ യോശുവയുടെ കല്‍പനപ്രകാരം ശവംമരത്തില്‍നിന്ന് ഇറക്കി; നഗരവാതില്‍ക്കലിട്ടു. അവര്‍ അതിനുമുകളില്‍ ഒരുകല്‍ക്കൂന ഉണ്ടാക്കി. അത് ഇന്നോളം അവിടെയുണ്ട്” (യോശുവ 8:29) ആവര്‍ത്തന പുസ്തകത്തിലെ ‘മരത്തില്‍ തൂക്കപ്പെട്ടവനെ‘ക്കുറിച്ച പരാമര്‍ശങ്ങളെ യേശുവില്‍ ആരോപിക്കപ്പെട്ട ക്രൂശീകരണവുമായി പൗലോസ് ബന്ധിപ്പിക്കുന്നത് കാണുക: ”നമുക്കുവേണ്ടി ശാപവിധേയനായിത്തീര്‍ന്ന ക്രിസ്തു നിയമത്തിന്റെ ശാപത്തില്‍ നിന്ന് നമ്മെ മോചിപ്പിച്ചിരിക്കുന്നു. ‘മരത്തില്‍ തൂങ്ങി മരിക്കുന്നവരെല്ലാം ശപിക്കപ്പെട്ടവരാണ്‘ എന്ന് എഴുതപ്പെട്ടിരിക്കുന്നുവല്ലോ. അങ്ങനെ അബ്രാഹാമിന് ദൈവം നല്‍കിയ അനുഗ്രഹം ക്രിസ്തുയേശുവിലൂടെ വിജാതീയര്‍ക്കും ലഭിക്കാനും തത്ഫലമായി വാഗ്ദാനം ചെയ്യപ്പെട്ട പരിശുദ്ധാത്മാവ് വിശ്വാസം വഴി നമുക്കും ലഭിക്കാനും യേശുക്രിസ്തുഅരുള്‍ചെയ്തു” (ഗലാത്യര്‍ 3:13) അപ്പോസ്തല പ്രവൃത്തികളിലും യേശുവിനെക്കുറിച്ച് പറയുമ്പോള്‍ ‘അവര്‍ അവനെ മരത്തില്‍ തൂക്കിക്കൊന്നു‘ (അപ്പോ 10:39) വെന്നാണ്പറയുന്നത്. ഇതില്‍നിന്നെല്ലാം യോസഫിന്റെ കാലത്ത് നിലനിന്നതും മോശ ആവര്‍ത്തനപുസ്തകത്തില്‍ പറഞ്ഞതും യോശുവ നടപ്പിലാക്കിയതുമെല്ലാം യേശുവിന്റെ കാലത്ത് നിലനിന്നിരുന്ന ക്രൂശീകരണത്തിന്റെ തന്നെ വ്യത്യസ്ത രൂപങ്ങളായിരുന്നുവെന്ന് സുതരാം വ്യക്തമാണ്. ഈ തെളിവുകളുടെ അടിസ്ഥാനത്തില്‍ ഈജിപ്തുകാര്‍ക്കിടയില്‍ ക്രൂശീകരണമെന്ന ശിക്ഷാ സമ്പ്രദായം നിലനിന്നിരുന്നതായി ബൈബിള്‍പണ്ഡിതന്മാര്‍ തന്നെ വ്യക്തമാക്കിയിട്ടുണ്ട്. സ്മിത്തിന്റെ ബൈബിള്‍ ഡിക്ഷ്ണറി പറയുന്നത് കാണുക: ”ഈജിപ്തുകാരുടെയും (ഉല്‍പത്തി 40:19), കാര്‍ത്തേജിനിയന്മാരുടെയും പേര്‍ഷ്യക്കാരുടെയും (എസ്‌തേര്‍ 7:10) അസീറിയക്കാരുടെയും സ്‌കീത്യരുടെയും ഇന്ത്യക്കാരുടെയും ജര്‍മന്‍കാരുടെയും വളരെ ആദ്യകാലം തൊട്ടുതന്നെ ഗ്രീക്കുകാരുടെയും റോമക്കാരുടെയും ഇടയില്‍ ക്രൂശീകരണം ഉപയോഗിക്കപ്പെട്ടിരുന്നു. ആദിമയഹൂദന്മാര്‍ക്ക് ഈ ശിക്ഷാ സമ്പ്രദായം അറിയാമായിരുന്നോയെന്ന വിഷയത്തില്‍ തര്‍ക്കം നിലനില്‍ക്കുന്നുണ്ട്. യഹൂദന്മാര്‍ റോമക്കാരില്‍നിന്നായിരിക്കണം ഈ സമ്പ്രദായം സ്വീകരിച്ചത്. ഇത് ഏറ്റവുംഭീകരമായ മരണരീതിയായി എല്ലാവരും ഒരുപോലെ അംഗീകരിച്ചിരുന്നു” (“Crucifixion” Smith’s Bible Dictionary Online) പുരാതന ഈജിപ്തില്‍ ക്രൂശീകരണം നിലനിന്നതിന് തെളിവുകളില്ലെന്ന് പറഞ്ഞ് ഖുര്‍ആനില്‍ അബദ്ധം ആരോപിക്കുന്നതിന് മുമ്പ് മിഷനറിമാര്‍ സ്വന്തം വേദഗ്രന്ഥം ഒന്ന് മനസ്സിരുത്തി വായിച്ചുനോക്കേണ്ടതായിരുന്നു. യോസഫിന്റെയും മോശയുടെയും കാലത്ത് ക്രൂശീകരണം നിലനിന്നിരുന്നുവെന്ന് പറയുന്നത് ഖുര്‍ആന്‍ മാത്രമല്ല; ബൈബിളും കൂടിയാണ്. ഇരുവേദഗ്രന്ഥങ്ങളും ഒരുപോലെ പറയുന്ന ഇക്കാര്യത്തിന് വിരുദ്ധമായ തെളിവുകള്‍ പുരാതന ഈജിപ്തിനെക്കുറിച്ച പഠനങ്ങള്‍ നല്‍കാത്തിടത്തോളം ഇതില്‍ അബദ്ധമാരോപിക്കുന്നതില്‍ യാതൊരു കഴമ്പുമില്ല.